Sie sind auf Seite 1von 2840

OBMEP

At 2011
Matemtica Olmpica das Escolas Pblicas
Compilao Omegaleph do

Material de preparao para a


OBMEP at 2011
23 de Maio de 2014

Aritmetica
2009/6/29
i page 1
Estilo OBMEP

i
i

Iniciao Aritmtica
Abramo Hefez

i
i

Aritmetica
2009/6/29
i page 2
Estilo OBMEP

i
i

Texto j revisado pela nova ortografia.

i
i

Aritmetica
2009/6/29
i page 3
Estilo OBMEP

i
i

Sobre o Autor
Abramo Hefez nasceu no Egito, mas brasileiro por opo e carioca de corao. Cursou o ginasial e cientfico no Rio de Janeiro,
graduou-se na PUC-Rio em Matemtica e prosseguiu seus estudos
na Universidade de Pisa, Itlia e nos Estados Unidos, doutorando-se,
em Geometria Algbrica no Massachusetts Institute of Technology.
Professor Titular no Instituto de Matemtica da Universidade Federal
Fluminense, onde exerce docncia na graduao e na ps-graduao
e desenvolve atividade de pesquisa.

i
i

Aritmetica
2009/6/29
i page 4
Estilo OBMEP

i
i

Aritmetica
2009/6/29
i page i
Estilo OBMEP

i
i

Prefcio
O nosso objeto de estudo neste pequeno livro, aos alunos premiados que participam do Programa de Iniciao Cientfica da OBMEP,
o conjunto dos nmeros inteiros e algumas de suas propriedades.
Os nmeros inteiros so obtidos estendendo os nmeros naturais e
esses so os mais simples de todos os nmeros, mas ao mesmo tempo
muito ricos em problemas. Voc ver ao longo do texto alguns desses
problemas, muito fceis de enunciar, mas ainda no resolvidos e com
certeza se maravilhar de como, apesar do ser humano estar estudando os nmeros naturais h vrios milnios, eles ainda encerrem
grandes mistrios a serem desvendados.
Nessas notas, alm de possivelmente estar vendo pela primeira
vez a noo de congruncia, voc revisitar as noes de mltiplo,
de divisor, de nmero primo, de mnimo mltiplo comum e de mximo divisor comum, e estudar algumas de suas propriedades. Muito
provavelmente voc ainda no estudou esses conceitos com o grau de
formalizao que encontrar aqui, mas que ainda no representa o
maior rigor possvel, pois nos permitiremos fazer dedues por analogia e por induo emprica (isto , estabelecer regras gerais atravs
i

i
i

Aritmetica
2009/6/29
i page ii
Estilo OBMEP

i
i

ii
da anlise de um nmero finito de casos). Essas dedues podem se
transformar em verdadeiras demonstraes utilizando-se o Princpio
de Induo Matemtica, que assunto de um outro texto do autor,
publicado nesta coleo e destinado aos alunos do nvel III.
Este texto no existiria no fosse o desafio lanado por Suely
Druck, Diretora Acadmica da OBMEP, a quem agradeo calorosamente pela preciosa oportunidade de me dirigir aqui a vocs.
Agradeo tambm ao colega Dinamrico Pombo por sua leitura cuidadosa do manuscrito original.
Finalmente, espero que voc aprecie o material aqui apresentado
e que faa de seu estudo uma atividade prazerosa. Bom divertimento!

Niteri, maro de 2009.


O Autor

i
i

Aritmetica
2009/6/29
i page iii
Estilo OBMEP

i
i

Sumrio
1 Os Nmeros Naturais

1.1

Os Naturais . . . . . . . . . . . . . . . . . . . . . . . .

1.2

Ordem . . . . . . . . . . . . . . . . . . . . . . . . . . .

1.3

Adio . . . . . . . . . . . . . . . . . . . . . . . . . . .

1.4

Subtrao . . . . . . . . . . . . . . . . . . . . . . . . .

10

1.5

Mltiplos . . . . . . . . . . . . . . . . . . . . . . . . .

13

1.6

Multiplicao . . . . . . . . . . . . . . . . . . . . . . .

16

1.7

Mltiplos Comuns . . . . . . . . . . . . . . . . . . . .

19

1.8

Potenciao . . . . . . . . . . . . . . . . . . . . . . . .

21

2 Representao dos Naturais

23

2.1

O Sistema Decimal . . . . . . . . . . . . . . . . . . . .

23

2.2

Critrios de Multiplicidade de 2, 5 e 10 . . . . . . . . .

26

2.3

Critrios de Multiplicidade de 9 e de 3 . . . . . . . . .

29

2.4

Nmeros Primos . . . . . . . . . . . . . . . . . . . . .

31

iii

i
i

Aritmetica
2009/6/29
i page iv
Estilo OBMEP

i
i

iv
2.5

O Crivo de Eratstenes . . . . . . . . . . . . . . . . . .

33

2.6

Teorema Fundamental da Aritmtica . . . . . . . . . .

38

3 Os Inteiros e suas Propriedades

42

3.1

Os Inteiros

. . . . . . . . . . . . . . . . . . . . . . . .

42

3.2

Mltiplos Inteiros de um Nmero . . . . . . . . . . . .

45

3.3

Divisores . . . . . . . . . . . . . . . . . . . . . . . . . .

47

3.4

Algoritmo da Diviso . . . . . . . . . . . . . . . . . . .

53

3.5

Par ou mpar?

. . . . . . . . . . . . . . . . . . . . . .

58

3.6

Zero, Um ou Dois? . . . . . . . . . . . . . . . . . . . .

60

3.7

Mnimo Mltiplo Comum . . . . . . . . . . . . . . . .

62

3.8

Algoritmo do mdc de Euclides . . . . . . . . . . . . . .

66

3.9

Aplicaes da Relao de Bzout . . . . . . . . . . . .

70

3.10 Equaes Diofantinas Lineares . . . . . . . . . . . . . .

75

4 A Aritmtica dos Restos

81

4.1

Congruncias . . . . . . . . . . . . . . . . . . . . . . .

81

4.2

Critrios de Multiplicidade e Restos

. . . . . . . . . .

84

4.3

Congruncias e Somas . . . . . . . . . . . . . . . . . .

85

4.4

Congruncias e Produtos . . . . . . . . . . . . . . . . .

87

4.5

Algumas Aplicaes . . . . . . . . . . . . . . . . . . .

90

4.6

Aritmtica Modular . . . . . . . . . . . . . . . . . . .

96

5 Problemas Suplementares

99

i
i

Aritmetica
2009/6/29
i page 1
Estilo OBMEP

i
i

Captulo 1

Os Nmeros Naturais
1.1

Os Naturais

Os nmeros naturais formam um conjunto cujos elementos so


descritos de modo ordenado como segue:
1, 2, 3, 4, 5, 6, 7, 8, 9, 10, . . .
ou ainda, de modo mais sugestivo:
- 2n
- 3n
- 4n
- 5n
- 6n
- 7n
- 8n
- 9n
- 10
n
- ...
1n

Essa descrio no completa, pois s explicitamos alguns poucos


de seus elementos, guardando o restante na nossa imaginao.
No entanto, todos ns sabemos perfeitamente do que estamos falando. Tudo comea com o nmero um, simbolizado por 1, que repre1

i
i

Aritmetica
2009/6/29
i page 2
Estilo OBMEP

i
i

CAP. 1: OS NMEROS NATURAIS

senta a unidade, e com uma lei, simbolizada pelas flechas, que a cada
nmero, comeando pelo 1, fornece o seu sucessor, isto , o nmero
que lhe segue.
Sabemos tambm que esta sequncia nunca termina; ou seja, os
nmeros naturais so em quantidade infinita.
Cada elemento desse conjunto tem de ser obviamente representado por um smbolo distinto. Como fazer isto de modo a poder
memorizar todos esses smbolos? A resposta, muito engenhosa,
dada pela adoo de um sistema de numerao, que no nosso caso
o sistema decimal posicional, que ser descrito no prximo captulo.
Assim, por exemplo, sabemos que nesse sistema sucedendo o 10 vem
o 11 e sucedendo o 999 vem o 1 000 etc.
Os nmeros naturais permitem contar objetos, inclusive subconjuntos do prprio conjunto dos naturais. Por exemplo, de 1 a n,
inclusive, existem exatamente n nmeros naturais.

1.2

Ordem

Quando um nmero a aparece na sequncia, acima mencionada,


antes do nmero b, ou seja, esquerda de b, escrevemos a < b e
dizemos que a menor do que b, ou ainda, escrevemos b > a e dizemos
que b maior do que a.
. . . - an- . . .
Por exemplo,

1 < 2,

- bn- . . .

5 < 7,

9>6

etc.

Essa relao que ordena os nmeros naturais tem claramente a

i
i

Aritmetica
2009/6/29
i page 3
Estilo OBMEP

i
i

N SEC. 1.2: ORDEM

seguinte propriedade transitiva:


Se a aparece antes de b e b aparece antes de c, ento a aparece
antes de c.
. . . - an- . . .

- bn- . . .

- cn - . . .

Em smbolos:
Se a < b e b < c, ento a < c.

Escreveremos tambm a b para representar a situao:


a<b

ou

a = b.

Por exemplo, temos que 2 3 e tambm que 2 2.


A ordem nos naturais total, o que significa que dados dois
nmeros naturais a e b temos verificada uma e apenas uma das trs
seguintes possibilidades (tricotomia):
a < b,

a = b,

ou

a > b.

Sejam dados dois nmeros naturais a e b com a < b. Definimos os


seguintes conjuntos:
[a, b] o conjunto dos nmeros naturais x tais que a x b,
(a, b) o conjunto dos nmeros naturais x tais que a < x < b,

i
i

Aritmetica
2009/6/29
i page 4
Estilo OBMEP

i
i

CAP. 1: OS NMEROS NATURAIS

(a, b] o conjunto dos nmeros naturais x tais que a < x b,


[a, b) o conjunto dos nmeros naturais x tais que a x < b.
O primeiro e o segundo conjunto so chamados, respectivamente,
de intervalo fechado e intervalo aberto. Os dois outros conjuntos
so chamados indiferentemente de intervalos semiabertos, ou semifechados.
Exemplos:
O intervalo (2, 5) = {3, 4}:
- 2n
- 3n
l- 4l
m
m
n
- 5n
- 6n
- 7n
- 8n
- 9n
- 10
n
1n

O intervalo (2, 5] = {3, 4, 5}:


- 2n
- 3n
l- 4l
m
m
n
- 5n
l
m
- 6n
- 7n
- 8n
- 9n
- 10
n
1n

O intervalo [2, 5) = {2, 3, 4}:


- 2n
l- 3n
m
l- 4l
m
m
n
- 5n
- 6n
- 7n
- 8n
- 9n
- 10
n
1n

O intervalo [2, 5] = {2, 3, 4, 5}:


- 2n
l- 3n
m
l- 4l
m
m
n
- 5n
l
m
- 6n
- 7n
- 8n
- 9n
- 10
n
1n

Problema 1.1. Determine os elementos dos seguintes intervalos:


(2, 3), (2, 3], [2, 3), [2, 3], (3, 7), (3, 7], [3, 7) e [3, 7].
Uma propriedade caracterstica e fundamental do conjunto dos

i
i

Aritmetica
2009/6/29
i page 5
Estilo OBMEP

i
i

N SEC. 1.3: ADIO

nmeros naturais, que no procuraremos justificar por parecer to


bvia, a seguinte:
Princpio da Boa Ordem. Todo subconjunto no vazio do conjunto
dos nmeros naturais possui um menor elemento.
A afirmao acima significa que dado um subconjunto A de N, no
vazio, existe um elemento a de A tal que a b, para todo elemento b
de A.
Problema 1.2. Determine o menor elemento de cada um dos seguintes conjuntos: [2, 8], (2, 8], (3, 5), (3, 4), [3, 7] [2, 5], [3, 7] [2, 5].

1.3

Adio

Vamos a seguir introduzir a operao bsica nos naturais.


Seja dado um nmero natural a, o sucessor de a ser tambm
representado por a + 1:
...


- a
-a+1
- ...

Sejam dados dois nmeros naturais a e b, quaisquer. Podemos


deslocar a de b posies para a direita, obtendo um nmero que ser
denotado por a + b. Essa operao entre nmeros naturais chamada
de adio e o nmero a + b chamado soma de a e b.
... -

a
a+1
a+2 - ...

- a+b - ...

i
i

Aritmetica
2009/6/29
i page 6
Estilo OBMEP

i
i

CAP. 1: OS NMEROS NATURAIS

Por exemplo, dados a = 2 e b = 3, ao deslocarmos a de trs


posies para a direita, obtemos a sequncia
2,

2 + 1 = 3,

3 + 1 = 4,

4 + 1 = 5,

obtendo assim o nmero 2 + 3 = 5.


Agora, suponha que deslocamos b = 3 de a = 2 posies para a
direita, obtemos
3,

3 + 1 = 4,

3 + 2 = 5,

logo, tambm, 3 + 2 = 5.
Portanto,
2 + 3 = 3 + 2 = 5.
Este fato no uma mera coincidncia, ocorre sempre!
Propriedade comutativa da adio. Quaisquer que sejam os nmeros naturais a e b, temos que
a + b = b + a.

Esse fato, devido nossa experincia com os nmeros, nos parece


bvio, mas voc teria alguma ideia de como mostrar que ao deslocar a
para a direita de b posies alcana-se o mesmo nmero que deslocar
b para a direita de a posies?
Vamos agora introduzir um smbolo para representar o no deslocamento de um nmero. Diremos que deslocamos um nmero a de

i
i

Aritmetica
2009/6/29
i page 7
Estilo OBMEP

i
i

N SEC. 1.3: ADIO

zero posies para a direita quando no o movemos do seu lugar.


Escreveremos, neste caso,
a + 0 = a.
Vamos colocar o smbolo 0, chamado zero, esquerda de todos os
nmeros naturais, obtendo o conjunto ordenado:
- 1n
- 2n
- 3n
- 4n
- 5n
- 6n
- 7n
- 8n
- 9n
- ...
0n

Portanto, consideraremos 0 < a, para todo nmero natural a.


Denotaremos o conjunto acima por N, continuando a cham-lo de
conjunto (ampliado) dos nmeros naturais.
Se deslocarmos agora 0 de 1 posio para a direita, obtemos o
nmero 1, se o deslocarmos de 2 posies direita, obtemos 2, se o
deslocarmos de 3 posies direita obtemos 3. Portanto, intuitivo
aceitar que se deslocarmos 0 de a posies direita obtemos o nmero
a. Finalmente, claro que 0 + 0 = 0, pois ao no deslocarmos o zero
nos mantemos no zero. Portanto, para todo a no conjunto N, temos
que
0 + a = a = a + 0.

Assim, quaisquer que sejam a e b no conjunto N (incluindo agora


o elemento 0), temos que a + b = b + a.
Podemos estender a soma para uma quantidade de nmeros maior
do que dois. Por exemplo, para somar trs nmeros a, b e c, podemos

i
i

Aritmetica
2009/6/29
i page 8
Estilo OBMEP

i
i

CAP. 1: OS NMEROS NATURAIS

proceder da seguinte forma: somamos inicialmente a e b, formando o


nmero (a + b), depois somamos esse novo nmero com c, obtendo o
nmero (a + b) + c. Por exemplo dados 3, 5 e 6, formaramos 3 + 5 = 8
e o somaramos com 6 obtendo (3 + 5) + 6 = 8 + 6 = 14.
Por outro lado, poderamos somar a com (b+c), obtendo o nmero
a+(b+c). No exemplo acima, isso nos daria 3+(5+6) = 3+11 = 14.
Acontece que a adio tem tambm a seguinte propriedade:
Propriedade associativa da adio. Quaisquer que sejam os nmeros a, b e c de N, tem-se
(a + b) + c = a + (b + c).

Problema 1.3. Utilizando as propriedades comutativa e associativa


da adio, mostre que os 12 modos de somar trs nmeros a, b e c:
(a + b) + c, a + (b + c), (a + c) + b, a + (c + b), (b + a) + c, b + (a + c),
(b + c) + a, b + (c + a), c + (b + a), (c + a) + b, c + (a + b), (c + b) + a,
do o mesmo resultado.
Adio e Ordem. H uma relao de compatibilidade entre a ordem
e a adio de nmeros naturais, que a seguinte:
Dados trs nmeros naturais a, b e c quaisquer,
se a < b, ento a + c < b + c.

i
i

Aritmetica
2009/6/29
i page 9
Estilo OBMEP

i
i

N SEC. 1.3: ADIO

De fato, se a est esquerda de b, ento ao deslocarmos a e b


simultaneamente de c posies direita, no difcil aceitar que a + c
se mantm esquerda de b + c.
. . .-

.
.
.
a
a+1 a+2
b
b+1
b+2 - ...

A propriedade acima admite uma recproca, ou seja:


Dados trs nmeros naturais a, b e c, quaisquer,
se a + c < b + c, ento a < b.

Prova-se esta propriedade utilizando a tricotomia. De fato, suponhamos que a + c < b + c. Pela tricotomia, temos uma das trs
possibilidades:
b < a, b = a, ou a < b.
A primeira possibilidade no pode ser verificada, pois se b < a,
teramos b + c < a + c, pela propriedade j provada, o que est em
contradio com a nossa hiptese a + c < b + c.
A segunda possibilidade tambm no pode ser verificada, pois se
a = b, teramos a + c = b + c, o que tambm est em contradio com
a nossa hiptese.
S resta portanto a nica possibilidade: a < b.
Voc percebeu que utilizamos a tricotomia diversas vezes na prova
acima?

i
i

Aritmetica
2009/6/29
i page 10
Estilo OBMEP

i
i

10

CAP. 1: OS NMEROS NATURAIS

Problema 1.4. Mostre que dados trs nmeros naturais a, b e c,


quaisquer,
se a + c = b + c, ento a = b.

Problema 1.5. Usando a propriedade de compatibilidade da adio


com a ordem e a transitividade da ordem, mostre que:
Se a < b e c < d, ento a + c < b + d.
Vale a recproca dessa propriedade?
Sugesto: Usando a compatibilidade da adio com a ordem, some c
a ambos os lados da primeira desigualdade, some b a ambos os lados da
segunda desigualdade. Finalmente, compare as novas desigualdades
assim obtidas.

1.4

Subtrao

Dados dois nmeros naturais a e b tais que a b, o nmero


de deslocamentos para a direita partindo de a para atingir b ser
representado por b a e ser chamado de diferena entre b e a.
Por exemplo, dados a = 3 e b = 7, preciso deslocar 3 para a
direita de 4 posies para alcanar 7, logo 7 3 = 4.
Portanto, pela definio de b a, temos que
a + (b a) = b.

(1.1)

i
i

Aritmetica
2009/6/29
i page 11
Estilo OBMEP

i
i

11

N SEC. 1.4: SUBTRAO

O nmero b a tambm o quanto devemos deslocar b para a


esquerda para alcanar a.
Devido equao (1.1), o nmero ba pode ser interpretado como
o quanto falta a a para atingir b.
Portanto, da equao (1.1) e do Problema 1.4, seque que se tivermos uma igualdade entre nmeros naturais do tipo a + c = b, ento
c = b a.
Problema 1.6. Tenho 50 reais, mas uma bicicleta custa 200 reais,
quanto falta para eu poder comprar a bicicleta?
Problema 1.7. Mostre que se c a < b, ento a c < b c.
Note que a a = 0, pois devemos deslocar a de zero para atingir
a; ou seja no falta nada a a para atingir a.
Note tambm que a 0 = a, pois devemos deslocar 0 de a para a
direita para atingir a; ou seja, falta a a zero para atingir a.
Observe que, no contexto dos nmeros naturais, s faz sentido
formar a diferena b a quando b a: caso contrrio, isto , se
b < a,
. . . - bn- . . .

- an- . . .

no h como deslocar b para a esquerda para alcanar a, ou o que


o mesmo, no h como deslocar a para a direita para atingir b.
Quando a b, a diferena b a, entre b e a, define uma operao
sobre pares de nmeros naturais (a, b), que chamaremos de subtrao.

i
i

Aritmetica
2009/6/29
i page 12
Estilo OBMEP

i
i

12

CAP. 1: OS NMEROS NATURAIS

A subtrao a operao inversa da adio, pois ao deslocarmos a


para a direita de b posies encontramos a + b, depois ao deslocarmos
a + b para a esquerda de b posies voltamos para a. Em smbolos:
(a + b) b = a.
Reciprocamente, se deslocarmos b para a esquerda de a posies
encontramos b a, depois ao deslocarmos b a para a direita de a
posies encontramos b. Em smbolos:
(b a) + a = b.
Quando b > a, o nmero b a nos auxilia na contagem de quantos
nmeros inteiros maiores ou iguais a a e menores ou iguais a b existem.
Para contar esses nmeros considere a sequncia:
a + 0, a + 1, a + 2, a + 3, . . . , a + (b a) = b,
cujo nmero de elementos igual ao nmero de naturais entre 0 e
b a, inclusive, o que nos d exatamente b a + 1 nmeros.
Portanto,
se a < b, o intervalo [a, b] possui b a + 1 elementos.
Problema 1.8. Quantos nmeros naturais existem maiores ou iguais
a 37 e menores ou iguais a 72?
Problema 1.9. Quantos nmeros naturais existem em cada um dos
intervalos (32, 75], [32, 75) e (32, 75)?

i
i

Aritmetica
2009/6/29
i page 13
Estilo OBMEP

i
i

13

N SEC. 1.5: MLTIPLOS

Problema 1.10. Se a < b, quantos nmeros naturais existem nos


intervalos (a, b], [a, b) e (a, b)?

1.5

Mltiplos
Dado a N, podemos considerar os mltiplos de a:

0 vezes a (nenhuma vez a), uma vez a, duas vezes a, trs vezes a
etc., obtendo assim a sequncia:
0 a = 0, 1 a = a, 2 a = a + a, 3 a = a + a + a, . . .
Por exemplo, 0 dzias, uma dzia, duas dzias, trs dzias etc.,
so os mltiplos de 12.
Outro exemplo dado pelos mltiplos de 2:
0, 2, 4, 6, 8, 10,
que so chamados de nmeros pares. Um nmero que no par
chamado de mpar.
Problema 1.11. Os nmeros mpares so mltiplos de algum nmero
fixado maior do que 1? Voc seria capaz de justificar de modo convincente a sua resposta?
Problema 1.12. Liste os 10 primeiros mltiplos de 5.
Problema 1.13. Descubra quantos mltiplos de 7 existem entre 14
e 63, inclusive.

i
i

Aritmetica
2009/6/29
i page 14
Estilo OBMEP

i
i

14

CAP. 1: OS NMEROS NATURAIS

Soluo: O modo mais direto de proceder listar esses nmeros para


depois cont-los:
14, 21, 28, 35, 42, 49, 56, 63.
Assim, conclumos que esses so 8 em nmero.
Problema 1.14. Descubra quantos mltiplos de 7 existem entre 14
e 7 000, inclusive.
Soluo: Resolver o problema listando todos esses nmeros, como na
soluo do Problema 1.13, seria muito trabalhoso. Podemos abordar o problema fazendo-o recair num caso j considerado e de fcil
resoluo:
2 7 (= 14), 3 7, 4 7, . . . , 1 000 7 (= 7 000).
Agora s contar quantos so os nmeros de 2 a 1 000, que sabemos serem 1 000 2 + 1 = 999.
Note que o nico mltiplo de 0 apenas o 0. Todos os nmeros
so mltiplos de 1 e de si prprios. Note tambm que, pela definio
de mltiplo, um mltiplo no nulo, isto diferente de zero, de um
nmero a > 0 sempre maior ou igual do que a.
Assim, temos a seguinte propriedade importante:

Se a b = 0, ento a = 0 ou b = 0.

i
i

Aritmetica
2009/6/29
i page 15
Estilo OBMEP

i
i

N SEC. 1.5: MLTIPLOS

15

Problema 1.15.
(a) Quantos mltiplos de 8 existem entre 32 e 8 000, inclusive?
(b) Quantos nmeros pares existem entre 3 211 e 6 321?
(c) Quantas dzias podemos formar com 180 laranjas? E com 220
laranjas?
(d) Quantas semanas formam 280 dias? E 360 dias?

Problema 1.16. Seja c 6= 0.


(a) Mostre que
0 < c < 2 c < 3 c < 4 c < 5 c.
Fica assim bastante evidente, por analogia, ou por induo emprica,
que se a < b, ento a c < b c (uma prova rigorosa disto pode ser
dada usando Induo Matemtica).
(b) Mostre que vale a recproca da propriedade acima, isto que se
a c < b c, ento a < b.
Sugesto: Mostre que qualquer uma das opes, a = b ou b < a,
implica numa contradio, restando assim, por tricotomia (recorde
que a ordem total), a nica possibilidade: a < b.

i
i

Aritmetica
2009/6/29
i page 16
Estilo OBMEP

i
i

16

CAP. 1: OS NMEROS NATURAIS

1.6

Multiplicao

Tomar mltiplos define uma operao nos nmeros naturais, ab,


que se l a vezes b, representando o mltiplo a vezes b de b. Assim,

0,
se a = 0,

b,
se a = 1,
ab=
b + b + + b, se a > 1.

{z
}
|
a parcelas

O nmero a b ser chamado o produto de a por b e ser tambm


denotado por ab, quando no houver risco de confuso.
Exemplos: 2 3 = 3 + 3 = 6,
3 2 = 2 + 2 + 2 = 6,
5 2 = 2 + 2 + 2 + 2 + 2 = 10, 2 5 = 5 + 5 = 10 etc.
Dos exemplos acima temos que
2 3 = 6 = 3 2 e 5 2 = 10 = 2 5.
De novo, isto no mera coincidncia, pois ocorre sempre. Vamos
admitir que a multiplicao possua a seguinte propriedade:
Propriedade comutativa da multiplicao. Quaisquer que sejam
os nmeros naturais a e b, temos que
a b = b a.

De modo semelhante adio, a multiplicao tambm possui a


seguinte propriedade:

i
i

Aritmetica
2009/6/29
i page 17
Estilo OBMEP

i
i

17

N SEC. 1.6: MULTIPLICAO

Propriedade associativa da multiplicao. Quaisquer que sejam


os nmeros naturais a, b e c, temos que
a (b c) = (a b) c.

Problema 1.17. Mostre que ser mltiplo uma relao transitiva,


isto , se c mltiplo de b e b mltiplo de a, ento c mltiplo de
a.
Recorde que definimos a multiplicao nos nmeros naturais
atravs da noo de mltiplo, que em ltima anlise se reduz a ir
somando, sucessivamente, a cpias de um mesmo nmero b. portanto natural esperar que as operaes de adio e de multiplicao
tenham uma forte relao. Uma dessas relaes se d atravs da propriedade distributiva que passamos a discutir.
Propriedade distributiva da multiplicao com relao
adio. Considere dois mltiplos de um mesmo nmero natural, por
exemplo 6 12 e 3 12, somando esses nmeros obtemos
6 12 + 3 12 =
=
=
=
=
=

6 12 + (1 12 + 2 12)
(6 12 + 1 12) + 2 12
7 12 + (1 12 + 1 12)
(7 12 + 1 12) + 1 12
8 12 + 1 12
9 12 = (6 + 3) 12.

Um procedimento como o acima, mais um argumento de induo

i
i

Aritmetica
2009/6/29
i page 18
Estilo OBMEP

i
i

18

CAP. 1: OS NMEROS NATURAIS

que no queremos explicitar agora, permitiria mostrar que, em geral,


dados nmeros naturais a, b e c, tem-se que
(a + b) c = a c + b c.

Problema 1.18. Mostre que


c (a + b) = c a + c b.
Problema 1.19. Mostre que a soma de dois mltiplos de um mesmo
nmero mltiplo desse nmero.
Propriedade distributiva da multiplicao com relao subtrao. Podemos agora mostrar que se a < b, ento
c (b a) = c b c a.

De fato, temos que


c a + c (b a) = c [a + (b a)] = c b.
Assim, pela definio da subtrao, temos que
c (b a) = c b c a.
Problema 1.20. Mostre que a diferena de dois mltiplos de um
mesmo nmero, quando faz sentido, mltiplo desse nmero.

i
i

Aritmetica
2009/6/29
i page 19
Estilo OBMEP

i
i

N SEC. 1.7: MLTIPLOS COMUNS

19

Problema 1.21. Sejam dados nmeros naturais a, b e c tais que a


mltiplo de c. Mostre que a + b mltiplo de c se, e somente se, b
mltiplo de c.
Multiplicao e Ordem. A relao entre a adio e a ordem se
reflete numa relao entre a multiplicao e a ordem que j tivemos
oportunidade de abordar no Problema 1.16:

Se a < b e c > 0, ento c a < c b.

Problema 1.22. Mostre que o menor elemento do conjunto dos


mltiplos no nulos de um nmero natural a > 0 o prprio a.

1.7

Mltiplos Comuns

Um conceito importante o de mltiplo comum de dois nmeros.


Por exemplo, considere a sequncia dos mltiplos de 3:
0, 3, 6, 9, 12, 15, 18, 21, 24, 27, 30, 33, 36, 39, 42, 45, . . .
e a sequncia dos mltiplos de 5:
0, 5, 10, 15, 20, 25, 30, 35, 40, 45, . . .
Assim, a sequncia dos nmeros que so simultaneamente mltiplos de 3 e de 5 :
0, 15, 30, 45, . . .

i
i

Aritmetica
2009/6/29
i page 20
Estilo OBMEP

i
i

20

CAP. 1: OS NMEROS NATURAIS

Voc saberia continuar a sequncia acima? Aparentemente, tratase da sequncia dos mltiplos de 15, ou seja, os mltiplos do menor
mltiplo comum no nulo de 3 e de 5, que 15.
Isso absolutamente correto e um resultado geral que provaremos a seu tempo.
Problema 1.23. Determine os dois primeiros mltiplos comuns de
4 e 14. Como voc continuaria esta sequncia?
Se a e b so nmeros naturais no nulos, sabemos por definio
que o nmero a b um mltiplo no nulo de b. Por outro lado, pela
propriedade comutativa da multiplicao, tem-se que ele tambm
um mltiplo de a. Assim, o conjunto dos mltiplos comuns de a e b,
alm de conter o nmero 0, contm tambm o nmero a b 6= 0.
Definio. O menor mltiplo comum no nulo de dois nmeros naturais no nulos a e b denotado por mmc(a, b) e ser chamado de
mnimo mltiplo comum1 de a e b (ou abreviadamente mmc).
Problema 1.24. Ache o mmc dos seguintes pares de nmeros:
3 e 4;

6 e 11;

6 e 8;

3 e 9.

Voce percebeu que algumas vezes mmc(a, b) = a b e outras


vezes no? Qual ser a razo? Desvendaremos mais este mistrio no
Captulo 3.

Este nmero existe em funo da observao acima e do Princpio da Boa


Ordem.

i
i

Aritmetica
2009/6/29
i page 21
Estilo OBMEP

i
i

21

N SEC. 1.8: POTENCIAO

1.8

Potenciao

Dados dois nmeros naturais a 6= 0 e n qualquer, definimos a


operao de potenciao como segue:

1,
se n = 0,

a,
se n = 1,
an =

aa

{z a}, se n > 1.
|
n fatores

Define-se tambm 0n = 0, para todo n 6= 0.


Exemplo: 20 = 1, 21 = 2, 22 = 2 2 = 4, 23 = 8, 02 = 0 etc.
Observao. Fica de fora 00 , que no definido.
Problema 1.25. Convena-se de que a potenciao possui as seguintes propriedades:
(a) 1n = 1;

(b) an am = an+m ;

(c) (an )m = anm ;

(d) an bn = (ab)n .

Existem tambm frmulas para escrever a potncia de uma soma.


Por exemplo,
(a + b)2 = a2 + 2ab + b2 ,
(a + b)3 = a3 + 3a2 b + 3ab2 + b3 ,
(a + b)4 = a4 + 4a3 b + 6a2 b2 + 4ab3 + b4 .

i
i

Aritmetica
2009/6/29
i page 22
Estilo OBMEP

i
i

22

CAP. 1: OS NMEROS NATURAIS

Em geral, (a + b)n se escreve como a soma dos produtos de potncias ai bj , onde i + j = n, multiplicados por certos nmeros naturais.
Esta frmula geral que no apresentaremos aqui chamada de frmula do binmio de Newton. Para maiores informaes sobre esta
frmula, veja o texto sobre induo do autor, j citado anteriormente
e listado na bibliografia no final do livro.
Problema 1.26. Desenvolva (a + b)5 .

i
i

Aritmetica
2009/6/29
i page 23
Estilo OBMEP

i
i

Captulo 2

Representao dos Naturais


2.1

O Sistema Decimal

Os nmeros naturais foram representados ao longo da histria de


vrios modos distintos. O modo universalmente utilizado na atualidade a representao decimal posicional. Esse sistema, variante do
sistema sexagesimal utilizado pelos babilnios h cerca de 1 700 anos
antes de Cristo, foi desenvolvido na China e na ndia. Nesse sistema,
todo nmero natural representado por uma sequncia formada pelos
algarismos
0, 1, 2, 3, 4, 5, 6, 7, 8, 9.
Por serem 10 esses algarismos, o sistema chamado de decimal.
O sistema tambm dito posicional, pois cada algarismo, alm de seu
valor intrnseco, possui um peso que lhe atribudo em funo de sua
posio dentro da sequncia. Esse peso uma potncia de 10 e varia
23

i
i

Aritmetica
2009/6/29
i page 24
Estilo OBMEP

i
i

24

CAP. 2: REPRESENTAO DOS NATURAIS

do seguinte modo:
O algarismo da extrema direita tem peso 100 = 1; o seguinte,
sempre da direita para a esquerda, tem peso 101 = 10; o seguinte tem
peso 102 = 100; o seguinte tem peso 103 = 1 000 etc.
Assim, o nmero 1 458, no sistema decimal representa o nmero
1 103 + 4 102 + 5 10 + 8.
Os zeros esquerda em um nmero so irrelevantes, pois por exemplo,
0231 = 0 103 + 2 102 + 3 10 + 1 = 2 102 + 3 10 + 1 = 231.
Cada algarismo de um nmero possui uma ordem, contada da
direita para a esquerda. Assim, no exemplo acima, o 8 de primeira
ordem, o 5 de segunda ordem, o 4 de terceira ordem e o 1 de quarta
ordem.
Cada trs ordens, tambm contadas da direita para a esquerda,
constituem uma classe. As classes so usualmente separadas por um
ponto. A seguir, damos os nomes das primeiras classes e ordens:

unidades 1 ordem
Classe das Unidades
dezenas 2a ordem

centenas 3a ordem

unidades de milhar 4 ordem


Classe do Milhar
dezenas de milhar 5a ordem

centenas de milhar 6a ordem

i
i

Aritmetica
2009/6/29
i page 25
Estilo OBMEP

i
i

N SEC. 2.1: O SISTEMA DECIMAL

Classe do Milho

25

unidades de milho 7 ordem


dezenas de milho 8a ordem

centenas de milho 9a ordem

Problema 2.1. Determine a soma de todos os mltiplos de 6 que se


escrevem no sistema decimal com dois algarismos.
Problema 2.2. Fixe trs algarismos distintos e diferentes de zero.
Forme os seis nmeros com dois algarismos distintos tomados dentre
os algarismos fixados. Mostre que a soma desses nmeros igual a 22
vezes a soma dos trs algarismos fixados.
Problema 2.3. Nos tempos de seus avs no existiam as calculadoras
eletrnicas e por isso eram ensinadas vrias regras de clculo mental.
Uma delas era a seguinte:
Seja a um nmero natural cujo algarismo da unidade 5,
ou seja, a = 10q + 5, com q um nmero natural. Mostre que
a2 = 100q(q + 1) + 25. Com isto, ache uma regra para calcular
mentalmente o quadrado de a. Aplique a sua regra para calcular os
quadrados dos nmeros; 15, 25, 35, 45, 55, 65, 75, 85, 95, 105
e 205.
Problema 2.4. Qual o menor nmero de dois algarismos? E qual
o maior? Quantos so os nmeros de dois algarismos? Quantos
algarismos precisa-se para escrev-los?
Problema 2.5. Quantos algarismos so usados para numerar um
livro de 300 pginas? Quantas vezes usa-se cada algarismo?
Curiosidade. Existe uma frmula interessante para descrever o nmero Q(x) de algarismos necessrios para escrever todos os nmeros

i
i

Aritmetica
2009/6/29
i page 26
Estilo OBMEP

i
i

26

CAP. 2: REPRESENTAO DOS NATURAIS

naturais de 0 a x, no sistema decimal:


Q(x) = n(x + 1) (10n1 + + 10),
onde n o nmero de algarismos de x (cf. Revista do Professor de
Matemtica, n. 5, p. 32).
Utilize esta frmula para conferir a sua resposta ao Problema 2.5.

2.2

Critrios de Multiplicidade de 2, 5 e 10

Critrios de multiplicidade so alguma regras prticas para decidir


se um dado nmero mltiplo de algum outro prefixado.
A seguir, veremos alguns desses critrios.
Seja dado um nmero n escrito no sistema decimal como
n = nr n1 n0 = nr 10r + + n1 10 + n0 .
Podemos ento escrever
n = (nr 10r1 + + n1 )10 + n0 ,
onde n0 o algarismo das unidades de n.
Reciprocamente, se n da forma n = 10m + n0 , onde n0 um dos
algarismos de 0 a 9, ento n0 o algarismo das unidades de n.
Problema 2.6. Mostre que o algarismo das unidades de um quadrado
perfeito, isto , um nmero da forma a2 , onde a um nmero natural,

i
i

Aritmetica
2009/6/29
i page 27
Estilo OBMEP

i
i

N SEC. 2.2: CRITRIOS DE MULTIPLICIDADE DE 2, 5 E 10

27

s pode ser 0, 1, 4, 5, 6 ou 9.
Critrio de multiplicidade de 2.
Inicialmente, consideremos a tabela:
20=0
21=2
22=4
23=6
24=8

2 5 = 10 = 10 + 0
2 6 = 12 = 10 + 2
2 7 = 14 = 10 + 4
2 8 = 16 = 10 + 6
2 9 = 18 = 10 + 8

Note que todo nmero acima um mltiplo de 10 somado com


um dos nmeros: 0, 2, 4, 6, ou 8.
Suponha agora que um dado nmero natural n seja par, ou seja,
n = 2m, onde m um nmero natural. Escrevendo m da forma
m0 10 + m0 , onde m0 o algarismo das unidades de m, temos
n = 2(m0 10 + m0 ) = 2m0 10 + 2m0 .
Sendo 2m0 um dos nmeros da tabela, temos que ele um mltiplo de 10 somado com um dos nmeros: 0, 2, 4, 6, ou 8. Logo,
n = 2m0 10 + 2m0 um mltiplo de 10 somado com um dos nmeros:
0, 2, 4, 6, ou 8, e, portanto, o seu algarismo das unidades 0, 2, 4, 6,
ou 8.
Problema 2.7. Mostre a recproca do que provamos acima, ou seja,
mostre que par um nmero cujo algarismo das unidades um dos
algarismos 0, 2, 4, 6 ou 8.

i
i

Aritmetica
2009/6/29
i page 28
Estilo OBMEP

i
i

28

CAP. 2: REPRESENTAO DOS NATURAIS

Juntando essas informaes temos o seguinte resultado:


Teorema (Critrio de Multiplicidade de 2)
Um nmero mltiplo de 2 se, e somente se, o seu algarismo das
unidades par.
Critrio de multiplicidade de 5 e de 10.
Seja n um nmero natural escrito na forma n = 10m + n0 , onde
n0 o algarismo das unidades de n. Como 10m mltiplo de 5 e de
10, temos que n mltiplo de 5 ou de 10 se, e somente se, n0 mltiplo de 5 ou de 10, respectivamente (cf. Problema 1.21). Isto ocorre
se, e somente se, n0 = 0 ou n0 = 5, no primeiro caso; e n0 = 0, no
segundo. Assim, provamos o seguinte resultado:
Teorema (Critrio de Multiplicidade de 5 ou de 10)
Um nmero mltiplo de 5 se, e somente se, o seu algarismo das
unidades for 0 ou 5. Um nmero mltiplo de 10 se, e somente se,
o seu algarismo das unidades for 0.
Problema 2.8. Determine se mltiplo de 2, de 5 ou de 10 cada
nmero a seguir:
17, 22, 25, 28, 30, 35 420, 523 475.
Problema 2.9. Com a informao de que 100 mltiplo de 4 e de
25, voc seria capaz de achar um critrio de multiplicidade de 4 ou de
25?
Sugesto: Note que um nmero n = nr n2 n1 n0 pode ser escrito
na forma n = nr n2 100 + n1 n0 .

i
i

Aritmetica
2009/6/29
i page 29
Estilo OBMEP

i
i

N SEC. 2.3: CRITRIOS DE MULTIPLICIDADE DE 9 E DE 3

29

Problema 2.10. Com a informao de que 1 000 mltiplo de 8


(respectivamente de 125), voc seria capaz de achar um critrio de
multiplicidade de 8? (respectivamente de 125?)
Sugesto: Note que um nmero n = nr n3 n2 n1 n0 pode ser escrito
na forma n = nr n3 1 000 + n2 n1 n0 .

2.3

Critrios de Multiplicidade de 9 e de 3

Inicialmente note os seguintes fatos:


10 1 = 9 = 1 9,
102 1 = 100 1 = 99 = 11 9,
103 1 = 1.000 1 = 999 = 111 9,
104 1 = 10 000 1 = 9 999 = 1 111 9.
Em geral, para n um nmero natural no nulo, temos
10n 1 = |11 {z
1} 9.
n vezes
Portanto, todos os nmeros da forma 10n 1 so mltiplos de 9
e tambm de 3, j que 9 mltiplo de 3.
Seja dado agora um nmero n escrito no sistema decimal como
n = nr n1 n0 = nr 10r + + n1 10 + n0 .
Subtraiamos a soma nr + +n1 +n0 , dos algarismos que compem

i
i

Aritmetica
2009/6/29
i page 30
Estilo OBMEP

i
i

30

CAP. 2: REPRESENTAO DOS NATURAIS

o nmero n, de ambos os lados da igualdade acima:


n (nr + + n1 + n0 ) = nr 10r nr + + n1 10 n1 + n0 n0
= (10r 1)nr + + (10 1)n1 .
Note agora que a ltima expresso sempre mltiplo de 9 (logo, de
3). Portanto, pelo Problema 1.21, temos que n mltiplo de 9 ou de
3 se, e somente se, o nmero nr + + n1 + n0 mltiplo de 9 ou de
3. Assim, obtemos o seguinte resultado:
Teorema (Critrio de Multiplicidade de 9 ou de 3)
Um nmero n = nr n1 n0 mltiplo de 9 ou de 3 se, e somente se,
o nmero nr + +n1 +n0 for mltiplo de 9 ou de 3, respectivamente.
O teorema acima reduz o problema de saber se um dado nmero
mltiplo de 9 ou de 3 ao problema de saber se um outro nmero obtido
a partir desse mltiplo de 9 ou de 3. O que ganhamos com isto?
Bem, o nmero nr + + n1 + n0 consideravelmente menor do que
n e se ele ainda for grande podemos aplicar o teorema a ele obtendo
um nmero ainda menor e assim, sucessivamente, at encontrar um
nmero para o qual seja fcil decidir se mltiplo de 9 ou de 3.
Por exemplo, dado o nmero 257 985 921, somando os seus algarismos obtemos 2 + 5 + 7 + 9 + 8 + 5 + 9 + 2 + 1 = 48. Repetindo o
mesmo procedimento para o nmero 48, obtemos 4 + 8 = 12, o qual
mltiplo de 3 mas no de 9. Logo, o nmero dado inicialmente
mltiplo de 3, mas no mltiplo de 9.
Problema 2.11. Determine se mltiplo de 3 ou de 9 cada um dos

i
i

Aritmetica
2009/6/29
i page 31
Estilo OBMEP

i
i

N SEC. 2.4: NMEROS PRIMOS

31

nmeros a seguir:
108, 111, 225, 328, 930, 35 424, 523 476.

2.4

Nmeros Primos

Os nmeros primos so nmeros especiais que desempenham um


papel importante dentro da teoria e entre outras coisas os seus produtos representam todos os nmeros naturais, como veremos ainda
nesta seo.
Definio. Um nmero natural diferente de 0 e de 1 e que apenas
mltiplo de 1 e de si prprio chamado de nmero primo. Um nmero
diferente de 0 e de 1 que no primo chamado de nmero composto.
Por exemplo, 2, 3, 5 e 7 so nmeros primos, enquanto 4, 6 e 8
so nmeros compostos, por serem mltiplos de 2.
Mais geralmente, todo nmero par maior do que 2 no primo,
ou seja, composto (justifique).
Note que a definio acima no classifica os nmeros 0 e 1 nem
como primos nem como compostos. Exceto esses dois nmeros, todo
nmero natural ou primo ou composto.
Problema 2.12. Diga quais dos seguintes nmeros so primos e quais
so compostos:
9, 10, 11, 12, 13, 15, 17, 21, 23, 47, 49.
Certamente, os nmeros compostos so em nmero infinito, pois

i
i

Aritmetica
2009/6/29
i page 32
Estilo OBMEP

i
i

32

CAP. 2: REPRESENTAO DOS NATURAIS

j os nmeros pares diferentes de 2 so em nmero infinito (justifique).

Uma pergunta que surge espontaneamente a seguinte: Quantos


so os nmeros primos?
Euclides de Alexandria, em 300 a.C., ou seja, h mais de 2 300
anos, mostrou que existem infinitos nmeros primos.
Como ter Euclides feito isto? Ser que ele exibiu todos os nmeros primos? Seria isto possvel? Veremos na prxima seo como ele
realizou esta faanha.

Determinar se um dado nmero primo ou composto pode ser


uma tarefa muito rdua. Para se ter uma ideia da dificuldade, voc
saberia dizer se o nmero 241 primo?

Muito mais difcil decidir se o nmero 4 294 967 297 primo ou


composto. O matemtico francs Pierre de Fermat (1601-1655) afirmou que esse nmero primo, enquanto o matemtico suo Leonhard
Euler (1707-1783) afirmou que composto. Qual deles estava com a
razo? Daremos a resposta na Seo 4.5.

A tarefa de decidir se um nmero primo ou mltiplo de outro


pode ser ligeiramente auxiliada com critrios de multiplicidade, como
os que vimos nas Sees 2.2 e 2.3.

i
i

Aritmetica
2009/6/29
i page 33
Estilo OBMEP

i
i

N SEC. 2.5: O CRIVO DE ERATSTENES

2.5

33

O Crivo de Eratstenes

Um mtodo muito antigo para se obter de modo sistemtico


nmeros primos o chamado Crivo de Eratstenes,1 devido ao
matemtico grego Eratstenes.
A eficincia do mtodo baseada na observao bem simples a
seguir.
Se um nmero natural a > 1 composto, ento ele mltiplo de
algum nmero primo p tal que p2 a. Equivalentemente, primo
todo nmero a que no mltiplo de nenhum nmero primo p tal que
p2 < a.
De fato, se a composto e p o menor nmero primo do qual
a mltiplo, ento a = p b, onde p e b so menores do que a.
De todo modo, sendo b primo ou composto, ele ser mltiplo de um
nmero primo q. Como a mltiplo de b e b mltiplo de q, pela
transitividade da relao de ser mltiplo (Problema 1.17), temos que
a tambm mltiplo de q e sendo p o menor primo do qual a
mltiplo, temos p q. Logo, p2 p q a.
Por exemplo, para mostrar que o nmero 221(= 13 17), composto, bastaria testar se ele mltiplo de algum dos nmeros primos p = 2, 3, 5, 7, 11 ou 13, j que o prximo primo 17 tal que
172 = 289 > 221.
Para se obter os nmeros primos at uma certa ordem n, escreva
os nmeros de 2 at n em uma tabela.
1

A palavra crivo significa peneira. O mtodo consiste em peneirar os nmeros


naturais em um intervalo [2, n], jogando fora os nmeros que no so primos.

i
i

Aritmetica
2009/6/29
i page 34
Estilo OBMEP

i
i

34

CAP. 2: REPRESENTAO DOS NATURAIS

O primeiro desses nmeros, o 2, primo, pois no mltiplo de


nenhum nmero anterior. Risque todos os demais mltiplos de 2 na
tabela, pois esses no so primos.
O primeiro nmero no riscado nessa nova tabela o 3 que
primo, pois no mltiplo de nenhum nmero anterior diferente de 1.
Risque todos os demais mltiplos de 3 na tabela, pois esses no so
primos.
O primeiro nmero maior que 3 e no riscado na tabela o 5 que
um nmero primo, pois no mltiplo de nenhum nmero anterior
diferente de 1. Risque os demais mltiplos de 5 na tabela.
O primeiro nmero maior do que 5 e que no foi riscado o 7, que
primo. Risque os demais mltiplos de 7 na tabela.
Ao trmino desse procedimento, os nmeros no riscados so todos
os primos menores ou iguais a n.
Note que o procedimento termina assim que atingirmos um
nmero primo p tal que p2 n, pois, pela observao que fizemos
acima, j teramos riscado todos os nmeros compostos menores ou
iguais a n.
Exibimos a seguir o resultado do crivo para n = 250. Note que,
neste caso, o procedimento termina to logo cheguemos ao nmero
primo p = 17.

i
i

Aritmetica
2009/6/29
i page 35
Estilo OBMEP

i
i

35

N SEC. 2.5: O CRIVO DE ERATSTENES

2n 3n 6 4
13n 6 14

6 15

6 16

5n 6 6
17n 6 18

7n 6 8
19n 6 20

69

6 10

6 21

6 22

31n 6 32
43n 6 44

6 33

6 34

6 45

6 46

6 55
6 56
n
6 68
67
79n 6 80

6 57

6 58

6 69

6 70

6 81

6 82

6 91
6 92

103 16 04

6 93

6 94

16 05

16 06

16 17

16 18

16 29

16 30

16 19 16 20

131 16 32

16 41

16 42

16 43

6 25
6 26
37n 6 38

6 27

6 28

6 39

6 40

6 49
6 50
n
6 62
61
73n 6 74

6 51

6 52

29n 6 30
41n 6 42
53n 6 54

6 63

6 64

6 65

6 75

6 76

6 85
6 86
n
6 98
97

109 16 10

6 87

6 88

6 99

16 00

16 11

16 12

6 77
6 78
89n 6 90

101 16 02

113 16 14

16 21

16 22

16 23

16 24

16 33

16 34

16 35

16 36

16 45 16 46

157 16 58

16 47

16 48

16 59

16 60

16 71

16 72

6 66

16 25 16 26

137 16 38

149 16 50

16 61 16 62

173 16 74

71n 6 72
83n 6 84
6 95
6 96

107 16 08

16 53

16 54

16 65

16 66

16 75

16 76

16 77

16 78

16 87 16 88

199 26 00

211 26 12

223 26 24

16 89

16 90

16 55 16 56

167 16 68

179 16 80

191 16 92

26 01

26 02

26 03

26 13

26 14

26 25

26 26

26 15 26 16

227 26 28

239 26 40

16 83

16 84

16 95

16 96

16 85 16 86

197 16 98

26 05

26 06

26 07

26 08

26 09

26 17 26 18

229 26 30

241 26 42

26 19

26 20

26 31

26 32

26 21 26 22

233 26 34

26 35

26 36

26 37

26 38

26 43

26 44

26 45

26 47

26 48

26 49

26 50

26 46

6 35
6 36
47n 6 48
59n 6 60

16 15 16 16

127 16 28

139 16 40

151 16 52

163 16 64

16 69 16 70

181 16 82

193 16 94

26 10

n 6 12
11
23n 6 24

16 44

26 04

Consultando a tabela acima temos que o nmero 241 primo,


respondendo pergunta que formulamos anteriormente.
Da tabela acima, extramos todos os nmeros primos at 250:

i
i

Aritmetica
2009/6/29
i page 36
Estilo OBMEP

i
i

36

CAP. 2: REPRESENTAO DOS NATURAIS

11

13

17

19

23

29

31

37

41

43

47

53

59

61

67

71

73

79

83

89

97

101

103

107

109

113

127

131

137

139

149

151

157

163

167

173

179

181

191

193

197

199

211

223

227

229

233

239

241

Note que a diferena de dois nmeros primos consecutivos, excetuando 2 e 3 (que diferem de 1) de no mnimo 2 (justifique).
Dois primos consecutivos so chamados primos gmeos se eles
diferem de 2.
Assim, consultando a tabela dos primos acima, os seguintes so
pares de primos gmeos:
(3, 5), (5, 7), (11, 13), (17, 19), (29, 31), (41, 43), (59, 61), (71, 73),
(101, 103), (107, 109), (137, 139), (149, 151), (179, 181), (191, 193),
(197, 199), (227, 229), (239, 241).
O que surpreendente que at o presente momento os matemticos ainda no saibam dizer se os pares de primos gmeos formam um
conjunto finito ou infinito.
Trs primos consecutivos sero chamados primos trigmeos se a
diferena entre cada dois primos consecutivos da terna 2.
Por exemplo, (3, 5, 7) uma terna de primos trigmeos. Voc seria
capaz de exibir outra terna de primos trigmeos?
Ao contrrio dos pares de primos gmeos, vamos mais adiante ver
que ser muito fcil responder questo da finitude ou no dessas

i
i

Aritmetica
2009/6/29
i page 37
Estilo OBMEP

i
i

N SEC. 2.5: O CRIVO DE ERATSTENES

37

ternas.
Outro problema muito simples de ser enunciado, mas que ainda
no tem resposta, a chamada Conjectura de Goldbach.2
O matemtico prussiano3 Christian Goldbach, numa carta de 7 de
junho de 1742 endereada a Leonhard Euler, o maior matemtico da
poca e um dos maiores matemticos de todos os tempos, props que
se provasse que todo nmero maior do que 5 a soma de trs primos.
Por exemplo, 6 = 2 + 2 + 2, 7 = 3 + 2 + 2, 8 = 3 + 3 + 2,
9 = 5 + 2 + 2, 10 = 5 + 3 + 2, 11 = 5 + 3 + 3 = 7 + 2 + 2,
12 = 5 + 5 + 2 = 3 + 7 + 2 etc.
Euler respondeu que acreditava nessa conjectura, porm no sabia
demonstr-la, mas que ela era equivalente a mostrar que todo nmero
par maior ou igual do que 4 era soma de dois nmeros primos.
Por exemplo, 4 = 2 + 2, 6 = 3 + 3, 8 = 5 + 3, 10 = 3 + 7 = 5 + 5,
12 = 5 + 7 etc.
Pois bem, esta conjectura, at o presente momento, no foi
provada, nem desmentida.
Problema 2.13. Teste a Conjectura de Goldbach e a verso de Euler
para os nmeros de 14 a 40. Voc acredita que esta conjectura seja
verdadeira?
2

O termo conjectura numa linguagem mais coloquial significa palpite, chute.


A Prssia tem uma histria muito rica dentro do contexto europeu dos sculos
18, 19 e 20, marcado por guerras interminveis. No tempo de Goldbach a Prssia
era um reino muito pobre, mas que posteriormente tornou-se um potente imprio
chegando a ocupar grande parte da Europa do Norte. Para saber mais consulte o
seu professor de Histria.
3

i
i

Aritmetica
2009/6/29
i page 38
Estilo OBMEP

i
i

38

CAP. 2: REPRESENTAO DOS NATURAIS

Um outro problema proposto em 1845 pelo matemtico francs


Joseph Bertrand (1822-1900) foi que, dado um nmero natural n > 3,
sempre existe um nmero primo p no intervalo (n, 2n2). Cinco anos
depois, o matemtico russo Pafnuti Chebyshev (1821-1894) provou de
modo surpreendentemente elementar, mas no o suficiente para que
o faamos aqui, que a afirmao era verdadeira.
Problema 2.14. Usando a nossa tabela de primos, verifique o Postulado de Bertrand para n 125.
H uma conjectura semelhante ao Postulado de Bertrand, proposta anteriormente pelo matemtico francs Adrien-Marie Legendre
(1752-1833), mas que ainda no foi provada nem desmentida, que a
seguinte:
Dado um nmero natural n sempre existe um nmero primo no
intervalo (n2 , (n + 1)2 ).
Problema 2.15. Usando a nossa tabela de primos, verifique a Conjectura de Legendre para n 15.

2.6

Teorema Fundamental da Aritmtica

O mtodo do Crivo de Eratstenes nos mostra que dado um


nmero natural a, existe um nmero primo p0 tal que ou a = p0 , ou
a um mltiplo no trivial de p0 ; isto , a = p0 a1 , com 1 < a1 < a.
Se a segunda possibilidade verificada, segue que existe um
nmero primo p1 , tal que ou a1 = p1 , ou a1 = p1 a2 , onde

i
i

Aritmetica
2009/6/29
i page 39
Estilo OBMEP

i
i

N SEC. 2.6: TEOREMA FUNDAMENTAL DA ARITMTICA

39

1 < a2 < a1 < a. Assim,


a = p0 p1 , ou

a = p0 p1 a2 .

Continuando a argumentao para a2 , temos a = p0 p1 p2 , ou


a = p0 p1 p2 a3 , para algum primo p2 e 1 < a3 < a2 < a1 < a.
Note que desigualdades como a acima no podem continuar indefinidamente (justifique). Logo, para algum r, o nmero ar um
primo pr , obtendo desse modo uma decomposio de a em fatores
primos:
a = p1 p2 pr .
Obtemos, assim, o seguinte resultado que se encontra no livro Os
Elementos de Euclides de Alexandria.
Proposio (Euclides)
Todo nmero natural a > 1, ou primo, ou se escreve como produto
de nmeros primos.
Prova-se com um pouco mais de trabalho, que faremos na Seo
3.9, que esta escrita nica a menos da ordem dos fatores. Com esta
informao adicional, o resultado de Euclides pode ser reformulado
do seguinte modo:
Teorema Fundamental da Aritmtica
Dado um nmero natural a 2, existem um nmero r > 0, nmeros
primos p1 < < pr e nmeros naturais no nulos n1 , . . . , nr tais
que
a = pn1 1 pnr r ;

i
i

Aritmetica
2009/6/29
i page 40
Estilo OBMEP

i
i

40

CAP. 2: REPRESENTAO DOS NATURAIS

alm disso, esta escrita nica.4


Problema 2.16. Decomponha em produtos de primos os seguintes
nmeros: 4, 6, 8, 28, 36, 84, 320 e 2 597.
Sugesto: Para o nmero 2 597, note que se esse nmero composto h certamente um nmero primo p < 51 que o divide, pois
512 > 2 597 (veja a observao que fizemos ao descrevermos o Crivo
de Eratstenes).
Vamos aproveitar que j temos os ingredientes para dar a demonstrao de Euclides de que existem infinitos nmeros primos.
Suponha por absurdo que os nmeros primos sejam em nmero
finito e seja a o produto de todos eles. O nmero a+1 no seria primo
pois ele seria maior do que qualquer nmero primo. Logo, a + 1 sendo
composto, ele seria mltiplo de algum nmero primo q. Mas sendo
a tambm mltiplo de q, teramos, pelo Problema 1.21, que 1 seria
mltiplo do nmero primo q, o que um absurdo.
E foi assim que o astuto Euclides provou que existem infinitos
nmeros primos, sem ter o trabalho de exibi-los todos. O mtodo
utilizado na prova acima chamado de reduo ao absurdo e consiste
em negar a afirmao que se quer provar e mostrar que isto leva a uma
contradio. Assim, mostra-se que a negao da afirmao falsa e,
portanto, a prpria afirmao verdadeira.
4

Observe que ordenamos os primos que intervm na fatorao de a por ordem


crescente, da a unicidade da escrita. Esta parte do teorema no se encontra
nos Elementos de Euclides, apesar daquela obra conter todos os ingredientes para
prov-la. A prova completa foi dada por Gauss mais de dois sculos depois e
acredita-se que Euclides no a fez por falta de notaes adequadas.

i
i

Aritmetica
2009/6/29
i page 41
Estilo OBMEP

i
i

N SEC. 2.6: TEOREMA FUNDAMENTAL DA ARITMTICA

41

Os nmeros primos se distribuem dentro de N de modo bastante irregular. J vimos que existem primos consecutivos cuja diferena
2: so os primos gmeos. Por outro lado, dado um nmero n arbitrrio, existem dois primos consecutivos cuja diferena maior do
que n.
De fato, dado n, considere o nmero a = 123 n. Assim,
a + 2, a + 3, a + 4, . . . , a + n,
so inteiros consecutivos todos compostos, pois a + 2 mltiplo de 2,
a + 3 mltiplo de 3, . . ., a + n mltiplo de n. Sejam p o maior
primo menor do que a + 2 e q o menor primo maior do que a + n
(que existe pois os primos so infinitos); logo p e q so dois primos
consecutivos, com q p > n.
Alguns dos problemas mais profundos ainda por resolver esto
relacionados com a distribuio dos nmeros primos dentro da sequncia dos nmeros naturais.

i
i

Aritmetica
2009/6/29
i page 42
Estilo OBMEP

i
i

Captulo 3

Os Inteiros e suas
Propriedades
3.1

Os Inteiros

Dados dois nmeros naturais a e b, at o momento, o nmero


b a s foi definido quando b a. Como remediar esta situao? O
jeito que os matemticos encontraram para que seja sempre definido
o nmero b a foi o de ampliar o conjunto dos nmeros naturais
formando um novo conjunto Z chamado de conjunto dos nmeros
inteiros, cujos elementos so dados ordenadamente como segue:

- 1 - 2 - 3
- ...

. . . - 3 - 2 - 1 - 0

Os nmeros esquerda do zero so chamados de nmeros negativos e os direita so chamados de nmeros positivos. Os pares
42

i
i

Aritmetica
2009/6/29
i page 43
Estilo OBMEP

i
i

43

N SEC. 3.1: OS INTEIROS

de nmeros 1 e 1, 2 e 2, 3 e 3 etc., so chamados de nmeros


simtricos. O elemento 0, que no nem positivo, nem negativo, o
seu prprio simtrico.
Em Z temos uma relao de ordem que estende a relao de ordem
de N, onde declaramos a < b quando a se encontra esquerda de b.
Esta relao continua transitiva e total (i.e., satisfazendo tricotomia). Os intervalos em Z so definidos de modo anlogo aos intervalos
de N.
Representando por a o simtrico de a, seja ele positivo, negativo
ou nulo, temos sempre que
(a) = a.
No conjunto Z, temos definida a adio como segue:
Para todo nmero inteiro a, definimos a + b como sendo o nmero
obtido pelo deslocamento de a para a direita de b posies, se b 0 ou
de b posies para a esquerda se b < 0. A adio no conjunto Z continua tendo as propriedades comutativa e associativa e compatvel
com a relao de ordem.
Definimos a diferena b a como sendo o nmero obtido deslocando b para a esquerda a posies, se a > 0; e deslocando b para a
direita a posies, se a < 0. Isto define uma operao em Z, sem
restries, chamada de subtrao. Assim, temos que a subtrao a
operao inversa da adio e
b a = b + (a).

i
i

Aritmetica
2009/6/29
i page 44
Estilo OBMEP

i
i

44

CAP. 3: OS INTEIROS E SUAS PROPRIEDADES

Problema 3.1. Mostre que em Z continua valendo a propriedade do


Problema 1.4.
Problema 3.2. Mostre que em Z continua valendo que (ba)+a = b
e que (a + b) b = a.
Problema 3.3. Mostre com exemplos que a subtrao no uma
operao nem comutativa nem associativa.
Problema 3.4. Mostre que em Z um intervalo [a, b], onde a b,
tem b a + 1 elementos.
A multiplicao nos inteiros definida como segue: Se a, b 0,
sabemos o que a b. Definimos
(a) b = a (b) = (a b),
e
(a) (b) = a b.
Assim, a b est definido para quaisquer inteiros a e b. A multiplicao em Z continua sendo comutativa, associativa e distributiva
com relao adio e subtrao.
Tem-se tambm que se a b = 0, com a e b inteiros, ento a = 0
ou b = 0.
Problema 3.5. Mostre que se a c = b c, com c 6= 0, ento a = b.
A multiplicao tambm continua compatvel com a ordem, no
seguinte sentido:

i
i

Aritmetica
2009/6/29
i page 45
Estilo OBMEP

i
i

N SEC. 3.2: MLTIPLOS INTEIROS DE UM NMERO

45

Se a < b e c > 0, ento c a < c b.

Problema 3.6. Mostre com um exemplo que em Z no vale a propriedade:


Se a < b, ento a c < b c, qualquer que seja c.
Nem a sua recproca:
Se a c < b c, ento a < b, qualquer que seja c.

3.2

Mltiplos Inteiros de um Nmero

Dado um inteiro a, consideremos o conjunto dos mltiplos inteiros


de a:
aZ = {a d; d Z}.
Problema 3.7. Mostre que os mltiplos inteiros de um elemento a
possuem as seguintes propriedades:
(i) 0 mltiplo de a.
(ii) Se m um mltiplo de a, ento m mltiplo de a.
(iii) Um mltiplo de um mltiplo de a um mltiplo de a.
(iv) Se m e m0 so mltiplos de a, ento m + m0 e m m0 so tambm
mltiplos de a.

i
i

Aritmetica
2009/6/29
i page 46
Estilo OBMEP

i
i

46

CAP. 3: OS INTEIROS E SUAS PROPRIEDADES

(v) Se m e m0 so mltiplos de a, ento e m + f m0 mltiplo


de a, quaisquer que sejam os inteiros e e f (note que (iv) um caso
particular da presente propriedade).
(vi) Se m + m0 ou m m0 mltiplo de a e m mltiplo de a, ento
m0 mltiplo de a.
O mesmo resultado vale para os mltiplos comuns de dois inteiros
a e b. De fato, o seguinte problema lida com esta situao.
Problema 3.8. Mostre que os mltiplos inteiros comuns de dois elementos a e b possuem as seguintes propriedades:
(i) 0 mltiplo comum de a e b.
(ii) Se m um mltiplo comum de a e b, ento m mltiplo comum
de a e b.
(iii) Um mltiplo de um mltiplo comum de a e b um mltiplo
comum de a e b.
(iv) Se m e m0 so mltiplos comuns de a e b, ento m + m0 e m m0
so tambm mltiplos comuns de a e b.
(v) Se m e m0 so mltiplos comuns de a e b, ento e m + f m0
mltiplo comum de a e b, quaisquer que sejam os inteiros e e f (note
que (iv) um caso particular da presente propriedade).
(vi) Se m + m0 ou m m0 mltiplo comum de a e b e m mltiplo
comum de a e b, ento m0 mltiplo comum de a e b.
Vimos que dois nmeros naturais a e b possuem sempre um mmc
que um nmero natural. Se um dos nmeros a ou b nulo e o outro

i
i

Aritmetica
2009/6/29
i page 47
Estilo OBMEP

i
i

47

N SEC. 3.3: DIVISORES

um inteiro qualquer, ento esses nmeros s admitem o zero como


mltiplo comum (justifique), que ser chamado do mnimo mltiplo
comum (mmc) de a e b. Se a e b so ambos no nulos, mesmo que
no sejam ambos positivos, ento define-se o mnimo mltiplo comum
(mmc) de a e b como sendo o menor mltiplo comum positivo; ou seja,
o menor elemento positivo do conjunto
aZ bZ.
Problema 3.9. Suponha que os nmeros 216 e 144 sejam mltiplos comuns de um determinado par de nmeros a e b. Mostre que
mmc(a, b) 72.
Sugesto: Utilize a propriedade (iv) do Problema 3.8.

3.3

Divisores

Nesta seo olharemos a noo de mltiplo sob outro ponto de


vista.
Definio. Diremos que um nmero inteiro d um divisor de outro
inteiro a, se a mltiplo de d; ou seja, se a = dc, para algum inteiro
c.
Quando a mltiplo de d dizemos tambm que a divisvel por
d ou que d divide a.
Representaremos o fato de um nmero d ser divisor de um nmero
a, ou d dividir a, pelo smbolo d | a. Caso d no divida a, escrevemos
d - a.

i
i

Aritmetica
2009/6/29
i page 48
Estilo OBMEP

i
i

48

CAP. 3: OS INTEIROS E SUAS PROPRIEDADES

Assim, por exemplo, temos que


1 | 6, 2 | 6, 3 | 6, 6 | 6, 6 | 6, 3 | 6, 2 | 6, 1 | 6.
Alm disso, se d 6 {6, 3, 2, 1, 1, 2, 3, 6}, ento d - 6.
Temos tambm que 1 | a e d | 0, para todo d, inclusive quando
d = 0, pois 0 mltiplo de qualquer nmero1 .
Note tambm que se d | a, ento d | a, d | a e d | a
Note que se a e d so nmeros naturais, com a 6= 0, e se d | a,
ento d a. De fato, sendo a um mltiplo natural no nulo do
nmero natural d, sabemos que a d.
Problema 3.10. Mostre que das duas propriedades acima segue que,
se a um inteiro no nulo, os divisores de a so em nmero finito.
Problema 3.11. Mostre que se a e b so nmeros naturais no nulos,
ento a | b e b | a se, e somente se, a = b.
Os critrios de multiplicidade podem ser reenunciados como
critrios de divisibilidade.
Por exemplo, dado um nmero n = nr . . . n1 n0 na sua representao decimal, temos o resultado:
n divisvel por 2 (ou seja mltiplo de 2) se e somente se n0 um
nmero par.
1
Isto absolutamente no quer dizer que podemos dividir zero por zero, pois
como 0 = c 0 para todo c, o quociente de 0 por 0 poderia ser qualquer nmero,
logo no estaria bem definido.

i
i

Aritmetica
2009/6/29
i page 49
Estilo OBMEP

i
i

49

N SEC. 3.3: DIVISORES

Problema 3.12. Enuncie critrios de divisibilidade por 3, 4, 5, 8, 9


e 10.
Utilizando a noo de divisor, podemos tambm redefinir a noo
de nmero primo como sendo um nmero p > 1 que s possui 1 e o
prprio p como divisores positivos.
A divisibilidade possui vrias propriedades importantes decorrentes das propriedades dos mltiplos e cuja utilizao vai nos facilitar
a vida.
A relao de divisibilidade transitiva, ou seja, se a | b e b | c,
ento a | c.
De fato, isto o mesmo que a transitividade da relao de ser
mltiplo (veja Problema 1.17).
Problema 3.13. Mostre as seguintes propriedades importantes da
divisibilidade:
(a) Se d | a e d | b, ento d | (b + a) e d | (b a).
(b) Se d | (b + a) ou d | (b a) e d | a, ento d | b.
(c) Conclua que d um divisor comum de a e de b se e somente se d
um divisor comum de a e de b a.
Definio. Dados dois nmeros inteiros a e b no simultaneamente
nulos, o maior divisor comum de a e b ser chamado de mximo divisor
comum de a e b e denotado por mdc(a, b).
Note que
mdc(a, b) = mdc(b, a).

i
i

Aritmetica
2009/6/29
i page 50
Estilo OBMEP

i
i

50

CAP. 3: OS INTEIROS E SUAS PROPRIEDADES

Problema 3.14.
(a) Mostre que mdc(0, 0) no existe.
(b) Mostre que
(
mdc(0, b) =

b, se b > 0
b, se b < 0.

(c) Mostre que se a 6= 0 ou b 6= 0, ento


mdc(a, b) = mdc(a, b) = mdc(a, b) = mdc(a, b).
O problema de determinar o mdc de dois nmeros bem simples
quando os nmeros so pequenos, pois neste caso podemos listar todos
os divisores comuns desses nmeros e escolher o maior deles, que ser
o seu mdc.
Por exemplo, para calcular mdc(12, 18), determinamos os divisores
de 12, que so:
1, 2, 3, 4, 6, 12;
e os divisores de 18, que so:
1, 2, 3, 6, 9, 18.
Tomando o maior divisor comum, obtemos: mdc(12, 18) = 6.
No entanto, quando um dos dois nmeros for grande, esse mtodo
fica impraticvel, pois achar os divisores de um nmero grande muito
complicado. O que fazer ento? Euclides, trs sculos antes de Cristo,
nos d uma soluo para este problema descrevendo um algoritmo

i
i

Aritmetica
2009/6/29
i page 51
Estilo OBMEP

i
i

N SEC. 3.3: DIVISORES

51

muito eficiente para fazer este clculo. O Algoritmo de Euclides, como


conhecido o mtodo por ele desenvolvido, ser descrito no prximo
captulo e repousa numa generalizao da propriedade do Problema
3.13(c) que recordamos abaixo:
Um nmero d divisor comum de a e b, no ambos nulos, se, e
somente se, ele um divisor comum de a e b a.
Tomando o mximo divisor comum, obtemos a seguinte identidade:
mdc(a, b) = mdc(a, b a),
que permite ir reduzindo sucessivamente o clculo do mdc de dois
nmeros ao clculo do mdc de nmeros cada vez menores.
Como exemplo de aplicao, vejamos como isto vai permitir o
clculo de mdc(3 264, 1 234):
mdc(3 264, 1 234) = mdc(1 234, 3 264 1 234) =
mdc(1 234, 2 030) = mdc(1 234, 2 030 1 234) =
mdc(1 234, 796) = mdc(796, 1 234 796) =
mdc(796, 438) = mdc(796 438, 438) =
mdc(358, 438) = mdc(358, 438 358) =
mdc(358, 80) = mdc(358 80, 80) =
mdc(278, 80) = mdc(198, 80) =
mdc(118, 80) = mdc(38, 80) =
mdc(38, 42) = mdc(38, 4) =
mdc(34, 4) = mdc(30, 4) =
mdc(26, 4) = mdc(22, 4) =
mdc(18, 4) = mdc(14, 4) =
mdc(10, 4) = mdc(6, 4) = 2

i
i

Aritmetica
2009/6/29
i page 52
Estilo OBMEP

i
i

52

CAP. 3: OS INTEIROS E SUAS PROPRIEDADES

As contas anteriores sero abreviadas de modo drstico com o


algoritmo de Euclides para o clculo do mdc que iremos apresentar
na Seo 3.8.
Problema 3.15. Sejam a e b dois nmeros com um divisor comum
d. Mostre que d divide a n + b m, quaisquer que sejam os nmeros
inteiros n e m.
Dois nmeros inteiros, no ambos nulos, sero ditos primos entre
si se no forem mltiplos de um mesmo nmero diferente de 1 e de
1.
Portanto, dois inteiros a e b, no ambos nulos, so primos entre
si se os nicos divisores comuns de a e b so 1 e 1, o que equivale a
dizer que mdc(a, b) = 1.
Exemplos de pares de inteiros primos entre si so: 2 e 3; 4 e 15; 9
e 7. No so primos entre si os pares: 2 e 4; 3 e 6; 9 e 12.
Dois nmeros primos distintos so sempre primos entre si.
Dois nmeros consecutivos so sempre primos entre si. De fato,
podemos escrever os dois nmeros na forma n e n + 1, logo
mdc(n, n + 1) = mdc(n, n + 1 n) = mdc(n, 1) = 1.
Problema 3.16.
(a) Mostre que dois nmeros inteiros da forma n e 2n + 1 so sempre
primos entre si.
(b) Mostre que se n um nmero mpar, ento mdc(n, 2n + 2) = 1.

i
i

Aritmetica
2009/6/29
i page 53
Estilo OBMEP

i
i

N SEC. 3.4: ALGORITMO DA DIVISO

53

(c) Mostre que se n um nmero par, ento mdc(n, 2n + 2) = 2.


Problema 3.17. Sejam a e b dois nmeros naturais no ambos nulos
e seja d = mdc(a, b). Se a0 e b0 so os dois nmeros naturais tais que
a = a0 d e b = b0 d, mostre que mdc(a0 , b0 ) = 1.

3.4

Algoritmo da Diviso

Uma das propriedades mais importantes dos nmeros naturais


a possibilidade de dividir um nmero por outro com resto pequeno.
Essa a chamada diviso euclidiana.
Sejam dados dois nmeros naturais a e b, com a > 0 e b qualquer.
Queremos comparar o nmero natural b com os mltiplos do nmero
a. Para isto, considere todos os intervalos da forma [na, (n + 1)a),
para n um nmero natural qualquer. Isto nos d uma partio de N,
ou seja,
N = [0, a) [a, 2a) [2a, 3a) [na, (n + 1) a)
e os intervalos acima so dois a dois sem elementos em comum.
Portanto, o nmero b estar em um e apenas um dos intervalos
acima. Digamos que b pertena ao intervalo
[qa, (q + 1) a).
Logo, existem dois nmeros naturais q e r, unicamente determi-

i
i

Aritmetica
2009/6/29
i page 54
Estilo OBMEP

i
i

54

CAP. 3: OS INTEIROS E SUAS PROPRIEDADES

nados, tais que


b = aq + r,

com 0 r < a.

O nmero b chamado dividendo, o nmero a divisor, os nmeros


q e r so chamados, respectivamente, quociente e resto da diviso de
b por a.
Note que dados dois nmeros naturais a e b, nem sempre b
mltiplo de a, este ser o caso se, e somente se, r = 0.

Como determinar os nmeros q e r na diviso euclidiana?


Caso b < a Como b = 0 a + b, temos que q = 0 e r = b.
Caso b = a Neste caso, tomamos q = 1 e r = 0.
Caso b > a Podemos considerar a sequncia:
b a, b 2a, . . . , b na,
at encontrar um nmero natural q tal que b (q + 1)a < 0, com
b qa 0. Assim, obtemos b = qa + r, onde r = b qa e, portanto,
0 r < a.

Por exemplo, para dividir o nmero 54 por 13, determinamos os


resultados da subtrao de 54 pelos mltiplos de 13:

i
i

Aritmetica
2009/6/29
i page 55
Estilo OBMEP

i
i

55

N SEC. 3.4: ALGORITMO DA DIVISO

54 13 = 41,
54 2 13 = 28,
54 3 13 = 15,
54 4 13 = 2
54 5 13 = 11 < 0.
Assim, a diviso euclidiana de 54 por 13 se expressa como:
54 = 4 13 + 2.
Problema 3.18. Efetue a diviso euclidiana nos seguintes casos:
(a) de 43 por 3

(b) de 43 por 5

(c) de 233 por 4

(d) de 1 453 por 10, por 100, por 1 000 e por 10 000.
Problema 3.19. Mostre o chamado Algoritmo da Diviso Euclidiana
nos inteiros:
Dados inteiros a e b, com a > 0, existe um nico par de inteiros q
e r tal que
b = aq + r, com 0 r < a.
Sugesto: Considere os intervalos da forma [na, (n + 1) a), com n
em Z.
Problema 3.20. Efetue a diviso euclidiana nos seguintes casos:
(a) de 43 por 3

(b) de 43 por 5

(c) de 233 por 4

(d) de 1 453 por 10, por 100, por 1 000 e por 10 000.
Pelo Problema 3.19, se a > 0, os possveis restos da diviso de um

i
i

Aritmetica
2009/6/29
i page 56
Estilo OBMEP

i
i

56

CAP. 3: OS INTEIROS E SUAS PROPRIEDADES

nmero qualquer por a so os nmeros 0, 1, . . . , a 1.


Por exemplo, os possveis restos da diviso de um nmero inteiro
por 2 so r = 0 ou r = 1.
Se um dado nmero quando divido por 2 deixa resto r = 0, ele
divisvel por 2, ou seja, ele par.
Se, ao contrrio, esse nmero deixa resto 1 quando dividido por
2, ele mpar.
Assim, um nmero par se da forma 2q e mpar se da forma
2q + 1, para algum inteiro q.
Problema 3.21. Mostre que dentre dois inteiros consecutivos um
deles par e o outro mpar.
Problema 3.22. Mostre que um nmero n escrito no sistema decimal como nr . . . n1 n0 deixa resto n0 quando dividido por 10. Como
se relacionam os restos da diviso de n por 2 ou 5 com os restos da
diviso de n0 por 2 ou 5?
Um nmero quando dividido por 3 pode deixar restos r = 0, r = 1
ou r = 2.
Problema 3.23. Mostre que de trs inteiros consecutivos um e apenas um deles mltiplo de 3.
Soluo: Suponha que os trs inteiros consecutivos sejam a, a + 1
e a + 2. Temos as seguintes possibilidades: a deixa resto 0, 1 ou 2
quando dividido por 3.
1) Suponha que a deixe resto 0 quando dividido por 3, ou seja, a = 3q.
Logo, a + 1 = 3q + 1 e a + 2 = 3q + 2. Assim, um e apenas um dos

i
i

Aritmetica
2009/6/29
i page 57
Estilo OBMEP

i
i

N SEC. 3.4: ALGORITMO DA DIVISO

57

trs nmeros mltiplo de 3, a saber, a.


2) Suponha que a deixe resto 1 quando dividido por 3, ou seja,
a = 3q + 1. Logo, a + 1 = 3q + 2 e a + 2 = 3q + 3 = 3(q + 1).
Assim, um e apenas um dos trs nmeros mltiplo de 3, a saber,
a + 2.
3) Suponha que a deixe resto 2 quando dividido por 3, ou seja,
a = 3q+2. Logo, a+1 = 3q+3 = 3(q+1) e a+2 = 3q+4 = 3(q+1)+1.
Assim, um e apenas um dos trs nmeros mltiplo de 3, a saber,
a + 1.
Problema 3.24. Mostre que dados trs nmeros a, a + 2 e a + 4,
um e apenas um deles mltiplo de 3. Usando este fato, mostre que
a nica terna de primos trigmeos (3, 5, 7).
Problema 3.25. Mostre que dados trs nmeros 2a, 2(a + 1) e
2(a + 2), um e apenas um deles mltiplo de 3.
Problema 3.26.
(a) Mostre que a soma de trs inteiros consecutivos sempre mltiplo
de 3.
(b) Dados trs inteiros consecutivos, mostre que um deles mltiplo
de 3 e a soma dos outros dois tambm.
Dividir por a > 0 agrupar em conjuntos com a elementos. Por
exemplo, para saber quantas dzias de ovos temos no quintal, temos
que dividir o nmero de ovos por 12, a diviso podendo ser exata ou
no. Se tivermos 36 ovos, teremos 3 dzias exatas, mas se tivermos
38 ovos, teremos ainda 3 dzias de ovos e sobrariam 2 ovos.

i
i

Aritmetica
2009/6/29
i page 58
Estilo OBMEP

i
i

58

CAP. 3: OS INTEIROS E SUAS PROPRIEDADES

Problema 3.27. Uma fbrica produz chicletes que so embalados em


pacotes de cinco unidades cada. Quantos pacotes sero produzidos
com 3 257 unidades?

3.5

Par ou mpar?

Nesta seo veremos, em um caso bem simples, como lidar com os


restos da diviso de nmeros inteiros por um nmero natural dado,
introduzindo uma nova aritmtica chamada aritmtica residual ou
aritmtica modular.
A soma de dois nmeros pares par. De fato, os dois nmeros
podem ser escritos na forma 2a e 2b, cuja soma 2(a + b), logo par.
A soma de dois nmeros mpares par. De fato, os nmeros so
da forma 2a + 1 e 2b + 1, cuja soma 2(a + b + 1), logo par.
A soma de um nmero par com um nmero mpar mpar. De
fato, um dos nmeros da forma 2a e o outro 2b + 1, cuja soma
2(a + b) + 1, logo mpar.
A paridade, isto , a qualidade de ser par ou mpar, da soma de
dois nmeros s depende da paridade de cada um dos nmeros e no
dos nmeros em si.
O produto de dois nmeros pares par. De fato, os nmeros sendo
da forma 2a e 2b, temos que o seu produto 4ab e, portanto, mltiplo
de 4, logo par.
O produto de um nmero par por um nmero mpar par. De
fato, um nmero da forma 2a e um nmero da forma 2b + 1 tm um
produto igual a 2a(2b + 1), que par.

i
i

Aritmetica
2009/6/29
i page 59
Estilo OBMEP

i
i

59

N SEC. 3.5: PAR OU MPAR?

O produto de dois nmeros mpares mpar. De fato, sendo os


nmeros da forma 2a + 1 e 2b + 1, o seu produto 2(2ab + a + b) + 1,
logo mpar.
Novamente, como no caso da soma, temos que a paridade do produto de dois nmeros s depende da paridade desses nmeros e no
dos nmeros em si.
Assim, podemos decidir a paridade de uma expresso complexa
envolvendo produtos e somas de inteiros do modo a seguir.
Atribuindo o smbolo 0 aos nmeros pares e o smbolo 1 aos
nmeros mpares, as observaes acima nos fornecem as seguintes
tabelas que regem a paridade das somas e produtos dos nmeros inteiros.
+

0
1

0
1

1
0

0
1

0
0

0
1

Por exemplo, se quisermos saber a paridade do nmero


11200 + 2119 no ser necessrio desenvolver as contas indicadas para saber se o resultado final par ou mpar. O que fazemos
substituir na expresso acima o nmero 20 por 0, por ser par; e
os nmeros 11 e 21 por 1, por serem mpares. Obtemos, assim, a
expresso
10
200
19
0 1 +1 ,
2010

que operada segundo as tabelas acima nos d 1 como resultado. Portanto, o nmero dado mpar.2
2

Tente explicar por que no substitumos os expoentes 10, 200 e 19 pelos


smbolos 0 e 1, segundo a sua paridade.

i
i

Aritmetica
2009/6/29
i page 60
Estilo OBMEP

i
i

60

CAP. 3: OS INTEIROS E SUAS PROPRIEDADES

O mtodo acima pode ser generalizado para controlar os restos da


diviso dos nmeros inteiros por qualquer nmero natural fixado m.
Veremos na prxima seo mais um caso especial, o caso m = 3.
No prximo captulo analisaremos o caso geral. Esse mtodo foi idealizado pelo matemtico alemo Carl Friedrich Gauss (1777-1855), considerado o maior matemtico de todos os tempos, quando tinha perto
de 17 anos.
Problema 3.28. Mostre que o dobro de um nmero mpar par mas
nunca mltiplo de 4.
Problema 3.29. Determine a paridade do seguinte nmero:
(123 275 + 346 231)234 + (3 451 + 4 532)542 .
Problema 3.30. Mostre que para todos a inteiro e n natural no
nulos, os nmeros a e an tm mesma paridade.
Problema 3.31. Dado um nmero inteiro a e dados dois nmeros
naturais n e m, no nulos, mostre que so sempre pares os nmeros
an + am e an am .
Problema 3.32. Qual a paridade da soma dos nmeros naturais
de um a 10? E de seu produto?

3.6

Zero, Um ou Dois?

Nesta seo analisaremos a aritmtica dos restos da diviso por 3.


Vamos organizar os nmeros inteiros numa tabela como segue:

i
i

Aritmetica
2009/6/29
i page 61
Estilo OBMEP

i
i

61

N SEC. 3.6: ZERO, UM OU DOIS?

..
.
9
6
3
0
3
6
9
..
.

..
.
8
5
2
1
4
7
10
..
.

..
.
7
4
1
2
5
8
11
..
.

Note que os nmeros da primeira coluna so os mltiplos de 3,


ou seja, os nmeros que deixam resto nulo quando divididos por 3.
Os nmeros da segunda e da terceira coluna so, respectivamente,
aqueles que deixam resto 1 e 2 quando divididos por 3.
Fazendo uma anlise semelhante quela feita na seo anterior,
nota-se que o resto da diviso por 3 da soma ou do produto de dois
nmeros s depende da coluna ocupada por esses nmeros, ou seja s
depende dos restos da diviso desses nmeros por 3 e no dos nmeros
em si.
Assim, atribuindo o smbolo 0 aos nmeros da primeira coluna
(que so os mltiplos de 3) e os smbolos 1 e 2, respectivamente, aos
nmeros que ocupam a segunda e terceira coluna (que so os nmeros
que deixam restos 1 e 2, quando divididos por 3), obtemos as seguintes
tabelas que regem os restos da diviso por 3 das somas e produtos
dos nmeros naturais:

i
i

Aritmetica
2009/6/29
i page 62
Estilo OBMEP

i
i

62

CAP. 3: OS INTEIROS E SUAS PROPRIEDADES

0
1
2

0
1
2

1
2
0

2
0
1

0
1
2

0
0
0

0
1
2

0
2
1

Problema 3.33. Usando as tabelas acima, ache o resto da diviso


por 3 do nmero 4100 + 3230 .

3.7

Mnimo Mltiplo Comum

Sabemos que todo mltiplo do mmc de dois inteiros um


mltiplo comum desses inteiros (Problema 3.8(iii)). Mostraremos no
prximo resultado que vale a recproca desse fato.
Teorema 3.1. Todo mltiplo comum de dois inteiros a e b mltiplo
de mmc(a, b).
Demonstrao. Seja m = mmc(a, b). Suponha que m0 seja um mltiplo comum de a e b. Se m0 = 0, nada temos a provar, pois 0 mltiplo
de qualquer inteiro, inclusive de m. Suponha que m0 6= 0, logo a 6= 0
e b 6= 0, o que mostra que m = mmc(a, b) > 0. Pelo algoritmo da
diviso euclidiana, podemos escrever
m0 = mq + r,

com 0 r < m.

Logo, r = m0 mq e, sendo m0 e mq mltiplos comuns de a e b,


segue do Problema 3.8(iv) que r mltiplo de comum de a e b. Mas
ento r = 0, pois caso contrrio teramos um mltiplo comum r de a
e b, tal que 0 < r < m, contradizendo a definio de mmc.

i
i

Aritmetica
2009/6/29
i page 63
Estilo OBMEP

i
i

N SEC. 3.7: MNIMO MLTIPLO COMUM

63

O Teorema acima nos fornece a seguinte relao:


aZ bZ = mmc(a, b)Z.
Problema 3.34. Mostre que um nmero mltiplo de 6 se, e somente
se, ele simultaneamente mltiplo de 2 e de 3.
Problema 3.35. Baseado no problema anterior, d um critrio de
multiplicidade de 6, conhecendo os critrios de multiplicidade de 2 e
de 3.
Problema 3.36. Sendo n um nmero inteiro qualquer, mostre que
o nmero n(n + 1)(2n + 1) sempre mltiplo de 6.
Problema 3.37. Utilizando os critrios de multiplicidade de 3 e de
4, enuncie um critrio de multiplicidade de 12.
Problema 3.38. Enuncie critrios de multiplicidade de 15, de 20 e
de 45.

Dados trs nmeros inteiros a, b e c, no nulos, podemos nos


perguntar como calcular o seu mnimo mltiplo comum mmc(a, b, c),
ou seja, o menor elemento positivo do conjunto dos mltiplos comuns
de a, b e c.
Portanto, queremos determinar o menor elemento positivo do conjunto
aZ bZ cZ = (aZ bZ) cZ = mmc(a, b)Z cZ.

i
i

Aritmetica
2009/6/29
i page 64
Estilo OBMEP

i
i

64

CAP. 3: OS INTEIROS E SUAS PROPRIEDADES

Isto nos mostra que


mmc(a, b, c) = mmc (mmc(a, b), c) .
Assim, para calcular o mmc de trs nmeros recai-se no clculo
de dois mmc de dois nmeros.
Problema 3.39. Calcule mmc(4, 6, 9).
Voc deve ter notado que calcular o mmc de dois nmeros ainda
uma tarefa muito trabalhosa, pois o que aprendemos at o momento
foi escrever ordenadamente os mltiplos de cada um dos nmeros at
encontrarmos o menor mltiplo comum positivo. Com este mtodo,
praticamente impossvel calcular o mmc de dois nmeros quando
um deles for bastante grande. Na prxima seo finalizaremos um
mtodo muito mais eficiente para se determinar o mmc, baseado no
Algoritmo do mdc de Euclides e no teorema a seguir.
Problema 3.40. Sejam a, b, d e m quatro inteiros positivos tais que
a b = m d. Mostre que m um mltiplo comum de a e b se, e
somente se, d um divisor comum de a e b.
Teorema 3.2. Sejam a e b dois inteiros positivos. Tem-se a seguinte
identidade:
mmc(a, b) mdc(a, b) = a b.
Demonstrao. Como a um mltiplo de mdc(a, b), segue que a b
mltiplo de mdc(a, b). Logo, a b = m mdc(a, b), para algum
inteiro positivo m. Pelo Problema 3.40, temos que m um mltiplo

i
i

Aritmetica
2009/6/29
i page 65
Estilo OBMEP

i
i

N SEC. 3.7: MNIMO MLTIPLO COMUM

65

comum de a e b e, consequentemente, pelo Teorema 3.1 temos que


m = mmc(a, b) c, para algum c positivo. Assim,
a b = mmc(a, b) (c mdc(a, b)).

(3.1)

Novamente, pelo Problema 3.40, segue que c mdc(a, b) um divisor


comum de a e b, logo sendo o mdc o maior dentre esses divisores,
segue que
c mdc(a, b) mdc(a, b).
(3.2)
Como c 1, temos que
mdc(a, b) c mdc(a, b),
o que juntamente com a desigualdade (3.2) implica que c = 1. Agora,
o resultado segue da equao (3.1).

Podemos agora esclarecer o mistrio a que nos referimos na Seo 1.7:


O mmc de dois nmeros igual ao seu produto se, e somente se, os
dois nmeros so primos entre si.
Problema 3.41. Seja n um nmero natural no nulo. Calcule
mmc(n, 2n + 1).
Problema 3.42. Suponha que n seja um nmero natural divisvel
por a e por b. Sabendo que mdc(a, b) = 1, mostre que n divisvel
por a b.

i
i

Aritmetica
2009/6/29
i page 66
Estilo OBMEP

i
i

66

CAP. 3: OS INTEIROS E SUAS PROPRIEDADES

3.8

Algoritmo do mdc de Euclides

O Lema de Euclides: Dados inteiros a e b, os divisores comuns de


a e b so os mesmos que os divisores comuns de a e b c a, para
todo nmero inteiro c fixado.
Demonstrao. Se d um divisor comum de a e b, claro que d
divisor comum de a e de b c a.
Reciprocamente, suponha que d seja divisor comum de a e de
b c a. Logo, d divisor comum de b c a e de c a e, portanto,
pelo Problema 3.13(c), tem-se que d divisor de b. Assim, d divisor
comum de a e b.
Esta simples observao, que generaliza a relao do Problema
3.13(c), vai nos dar um modo prtico para calcular o mdc de dois
nmeros, mais eficiente do que o utilizado na Seo 3.3.
O Lema de Euclides nos diz que os divisores de comuns de a e
b so os mesmos divisores comuns de a e b a c, logo tomando o
maior divisor comum em ambos os casos, obtemos a frmula:
mdc(a, b) = mdc(a, b a c),
o que permite ir diminuindo passo a passo a complexidade do problema, at torn-lo trivial.
Algoritmo de Euclides para o clculo do mdc
Nada melhor do que um exemplo para entender o mtodo.
Vamos calcular mdc(a, b), onde a = 162 e b = 372.

i
i

Aritmetica
2009/6/29
i page 67
Estilo OBMEP

i
i

N SEC. 3.8: ALGORITMO DO MDC DE EUCLIDES

67

Pelo Lema de Euclides, sabemos que o mdc de a e b o mesmo


que o de a e de b menos um mltiplo qualquer de a. Otimizamos
os clculos ao tomarmos o menor dos nmeros da forma b menos um
mltiplo de a e isto realizado pelo algoritmo da diviso:
372 = 162 2 + 48.
Assim,

mdc(372, 162) = mdc(372 162 2, 162) = mdc(48, 162).


Apliquemos o mesmo argumento ao par a1 = 48 e b1 = 162:
162 = 48 3 + 18.
Assim,
mdc(372, 162) = mdc(162, 48)
= mdc(162 48 3, 48)
= mdc(18, 48).
Apliquemos novamente o mesmo argumento ao par a2 = 18 e
b2 = 48:
48 = 18 2 + 12.
Assim,
mdc(372, 162) = mdc(48, 18) = mdc(48 18 2, 18) = mdc(12, 18).

i
i

Aritmetica
2009/6/29
i page 68
Estilo OBMEP

i
i

68

CAP. 3: OS INTEIROS E SUAS PROPRIEDADES

Novamente, o mesmo argumento para o par a3 = 18 e b3 = 12 nos d:


18 = 12 1 + 6.
Assim,
mdc(372, 162) = mdc(18, 12) = mdc(18 12 1, 12) = mdc(6, 12).
Finalmente, obtemos
mdc(372, 162) = mdc(12, 6) = mdc(12 6 2, 6) = mdc(0, 6) = 6.
Logo,
mdc(372, 162) = 6.

O procedimento acima pode ser sistematizado como segue:


2

372

162

48

18

12

6=mdc

48

18

12

O Algoritmo de Euclides usado de trs para frente nos d uma


informao adicional fundamental.
Das igualdades acima podemos escrever:
6n= 18 12 1
12 = 48 18 2

i
i

Aritmetica
2009/6/29
i page 69
Estilo OBMEP

i
i

N SEC. 3.8: ALGORITMO DO MDC DE EUCLIDES

69

18 = 162 48 3
48 = 372 162 2
Donde,
6n= 18 12 1 = 18 (48 18 2)
= 18 3 48
= (162 48 3) 3 48
= 162 3 48 10
= 162 (372 162 2) 10
= 162 23 372 10.
Assim, podemos escrever:
6n= mdc(372, 162) = 162 23 + 372 (10).
Este mtodo sempre se aplica conduzindo ao seguinte importante
resultado:
Teorema 3.3 (Relao de Bzout). Dados inteiros a e b, quaisquer,
mas no ambos nulos, existem dois inteiros n e m tais que
mdc(a, b) = a n + b m.
Problema 3.43. Determine mdc(a, b), mmc(a, b) e inteiros n e m
tais que mdc(a, b) = a n + b m para os seguintes pares de nmeros
a e b.
(a) a = 728 e b = 1 496
(b) a = 108 e b = 294.

i
i

Aritmetica
2009/6/29
i page 70
Estilo OBMEP

i
i

70

CAP. 3: OS INTEIROS E SUAS PROPRIEDADES

3.9

Aplicaes da Relao de Bzout

Esta seo pode ser omitida sem prejuzo na primeira leitura, exceto a Proposio 3.3 que ser utilizada na Seo 3.10.
Uma propriedade notvel do mximo divisor comum que decorre
da Relao de Bzout a seguinte:
Se d um divisor comum de dois nmeros a e b, no simultaneamente nulos, ento d divide mdc(a, b).
De fato, sendo d um divisor de a e de b, temos que d um divisor
de todo nmero da forma a n + b m, logo, em particular, de
mdc(a, b).
Definindo
aZ + bZ = {a n + b m; n, m Z},
temos o seguinte resultado:
Proposio 3.1. Dados dois inteiros a e b, no ambos nulos, o menor
elemento positivo do conjunto aZ + bZ mdc(a, b).
Demonstrao. De fato, ponhamos d = mdc(a, b). Como d | a e d | b,
temos que d divide todo elemento de aZ + bZ, logo d menor ou
igual do que qualquer elemento positivo de aZ + bZ. Pela Relao de
Bzout, temos que d aZ + bZ, logo d o menor elemento positivo
do conjunto aZ + bZ.
Da decorre um importante critrio para que dois nmeros sejam
primos entre si.

i
i

Aritmetica
2009/6/29
i page 71
Estilo OBMEP

i
i

N SEC. 3.9: APLICAES DA RELAO DE BZOUT

71

Proposio 3.2. Dois nmeros inteiros a e b so primos entre si se,


e somente se, existem inteiros m e n tais que a n + b m = 1.
Demonstrao. Suponhamos que a e b sejam primos entre si, isto ,
mdc(a, b) = 1. Como, pela Relao de Bzout, existem inteiros n e m
tais que a n + b m = mdc(a, b), segue que a n + b m = 1.
Reciprocamente, se existem n e m tais que a n + b m = 1,
segue que 1 o menor elemento positivo do conjunto aZ + bZ, logo
ele o mdc de a e b. Portanto, a e b so primos entre si.
Problema 3.44. Sejam a e b dois nmeros naturais no ambos nulos
e c um terceiro nmero natural no nulo. Mostre que
mdc(c a, c b) = c mdc(a, b).
Problema 3.45. Sejam a, b e c trs nmeros naturais no nulos.
Mostre que
mmc(c a, c b) = c mmc(a, b).
Outra propriedade fundamental que decorre da Relao de Bzout
o resultado a seguir:
Proposio 3.3. Sejam a, b e c trs inteiros tais que a divide b c
e a e b so primos entre si, ento a divide c.
Demonstrao. Como a | b c, ento existe um inteiro e tal que
b c = a e. Como a e b so primos entre si, ento existem inteiros n e m tais que a n + b m = 1. Multiplicando esta ltima

i
i

Aritmetica
2009/6/29
i page 72
Estilo OBMEP

i
i

72

CAP. 3: OS INTEIROS E SUAS PROPRIEDADES

igualdade por c obtemos


a n c + b m c = c.
Substituindo a b c por a e, temos que
c = a n c + a e m = a (n c + e m),
mostrando que a | c.
A srie de problemas a seguir nos permitir deduzir a unicidade
referida no Teorema Fundamental da Aritmtica.
Problema 3.46. Sejam a um nmero inteiro qualquer e p um nmero
primo. Mostre que uma das seguintes possibilidades acontece: p | a
ou mdc(a, p) = 1.
Problema 3.47. Sejam a e b dois inteiros e p um nmero primo.
Mostre que se p | a b, ento p | a ou p | b.
Problema 3.48. Sejam p, p1 e p2 trs nmeros primos. Mostre que
se p | p1 p2 , ento p = p1 ou p = p2 .
A propriedade acima pode se generalizar como segue:
Se p, p1 , p2 , . . . , pr so nmeros primos e se p | p1 p2 pr ,
ento para algum ndice i tem-se que p = pi .
Problema 3.49. Mostre que se p1 , . . . , pr e q1 , . . . , qs so duas colees de nmeros primos e se
p1 pr = q1 qs ,

i
i

Aritmetica
2009/6/29
i page 73
Estilo OBMEP

i
i

73

N SEC. 3.9: APLICAES DA RELAO DE BZOUT

ento r = s e reordenando q1 , . . . qr , se necessrio, tem-se que


p1 = q1 , . . . , pr = qr .
Este ltimo problema a prova da unicidade da escrita como produto de primos de qualquer nmero natural maior do que 1, contida
no enunciado do Teorema Fundamental da Aritmtica.
Seja n um nmero natural escrito na sua decomposio em fatores
primos como
n = pa11 par r ,
e seja n0 um divisor positivo de n. Logo na decomposio de n0 em
fatores primos s podem aparecer os fatores primos p1 , . . . , pr , com
expoentes b1 , . . . , br , respectivamente, satisfazendo
0 b1 a1 , . . . , 0 br ar .

(3.3)

Note que permitimos que alguns dos bi sejam nulos, pois o correspondente primo pi pode no constar da fatorao de n0 .
Por exemplo, os divisores positivos de 60 = 22 3 5 so:
20 30 50
20 31 51
21 30 51
22 31 50

= 1,
= 15,
= 10,
= 12,

20 31 50
21 30 50
21 31 51
22 30 51

= 3,
= 2,
= 30,
= 20,

20 30 51
21 31 50
22 30 50
22 31 51

=5
= 6,
= 4,
= 60.

O nmero de divisores de n = pa11 par r exatamente o


nmero de nmeros inteiros b1 , . . . , br satisfazendo s desigualdades

i
i

Aritmetica
2009/6/29
i page 74
Estilo OBMEP

i
i

74

CAP. 3: OS INTEIROS E SUAS PROPRIEDADES

(3.3), logo esse nmero


(a1 + 1) (ar + 1).
Problema 3.50. Ache os divisores positivos de 40 e de 120. Quais
so todos os divisores?
Problema 3.51. Quantos divisores positivos tem o nmero 63 25?
fcil determinar o mdc e o mmc de dois nmeros decompostos
em fatores primos.Por exemplo, se
a = 23 35 73 17 e b = 34 75 19,
temos que mdc(a, b) = 20 34 73 , enquanto
mmc(a, b) = 23 35 75 17 19.
Os nmeros a e b acima podem ser representados como produtos de potncias dos mesmos primos, com o artifcio de introduzir
fatores extras da forma p0 (= 1) para certos nmeros primos p. Mais
precisamente, podemos escrever
a = 23 35 73 17 190 e b = 20 34 75 170 19.
Problema 3.52. Ache o mdc e mmc dos nmeros a = 1 080 e b = 210.
Problema 3.53. Dados a = pa11 par r e b = pb11 pbrr
dois nmeros decompostos em fatores primos, escritos ambos como
produtos de potncias dos mesmos primos, onde a1 0, . . . , ar 0 e

i
i

Aritmetica
2009/6/29
i page 75
Estilo OBMEP

i
i

N SEC. 3.10: EQUAES DIOFANTINAS LINEARES

75

b1 0, . . . , br 0, mostre que
mdc(a, b) = pc11 pcrr

mmc(a, b) = pd11 pdr r ,

onde
ci = min{ai , bi } e di = max{ai , bi }, i = 1, . . . , r.
Mostre como obter disto uma nova prova da igualdade
mdc(a, b)mmc(c, b) = ab.
O leitor no deve se iludir sobre a facilidade em calcular o mdc e o
mmc com o mtodo acima, pois para utiliz-lo necessrio que os dois
nmeros estejam decompostos em fatores primos. Se os dois nmeros
so grandes e no so dados na forma fatorada, muito trabalhoso
fator-los para calcular o mdc ou o mmc, sendo, nesse caso, muito
mais eficiente o Algoritmo de Euclides.

3.10

Equaes Diofantinas Lineares

A resoluo de muitos problemas de aritmtica depende da resoluo de equaes do tipo ax + by = c, onde a, b e c so nmeros
inteiros dados e x e y so incgnitas a serem determinadas em Z. Um
exemplo tpico de um problema modelado por este tipo de equao
o seguinte:
Problema 3.54. De quantos modos podemos comprar selos de cinco
e de trs reais, de modo a gastar cinquenta reais?

i
i

Aritmetica
2009/6/29
i page 76
Estilo OBMEP

i
i

76

CAP. 3: OS INTEIROS E SUAS PROPRIEDADES

Dada uma equao, as perguntas naturais que se colocam so as


seguintes:
1) Quais so as condies para que a equao possua soluo?
2) Quantas so as solues?
3) Como calcular as solues, caso existam?
Daremos a seguir respostas a essas perguntas no caso das equaes
em questo.
A primeira pergunta admite a resposta a seguir.
Teorema 3.4. A equao diofantina ax + by = c admite soluo se,
e somente se, mdc(a, b) divide c.
Demonstrao. Suponha que a equao admita uma soluo x0 , y0 .
Ento vale a igualdade ax0 + by0 = c. Como mdc(a, b) divide a e
divide b, segue que ele divide ax0 + by0 , logo divide c.
Reciprocamente, suponha que mdc(a, b) divida c, ou seja
c = mdc(a, b) d, para algum inteiro d. Por outro lado, sabemos
que existem inteiros n e m tais que
mdc(a, b) = a n + b m.
Multiplicando ambos os lados da igualdade acima por d, obtemos
c = mdc(a, b) d = a (n d) + b (m d).
Logo, a equao diofantina ax + by = c admite pelo menos a

i
i

Aritmetica
2009/6/29
i page 77
Estilo OBMEP

i
i

77

N SEC. 3.10: EQUAES DIOFANTINAS LINEARES

soluo
x=nd

y = m d.

Problema 3.55. Diga quais so as equaes diofantinas a seguir que


possuem pelo menos uma soluo:
(a) 3x + 5y = 223
(d) 3x + 12y = 312

(b) 5x + 15y = 33
(e) 23x + 150y = 12 354

(c) 2x + 16y = 2 354


f) 7x + 14y = 77

Problema 3.56. Mostre que se a e b so nmeros inteiros tais que


mdc(a, b) = 1, ento toda equao diofantina ax + by = c possui
soluo, independentemente do valor de c.
Problema 3.57. Para quais valores de c, onde c inteiro, a equao
30x + 42y = c admite solues inteiras?
Se a equao ax + by = c admite uma soluo, ento o nmero
d = mdc(a, b) divide c e, portanto, temos que a = a0 d, b = b0 d e
c = c0 d, onde mdc(a0 , b0 ) = 1 (Problema 3.17).
Assim, imediato verificar que x0 , y0 uma soluo da equao
ax + by = c se, e somente se, soluo da equao a0 x + b0 y = c0 , onde
agora mdc(a0 , b0 ) = 1.
Portanto, toda equao diofantina linear que possui soluo
equivalente a uma equao reduzida, ou seja, da forma
ax + by = c,

com mdc(a, b) = 1.

i
i

Aritmetica
2009/6/29
i page 78
Estilo OBMEP

i
i

78

CAP. 3: OS INTEIROS E SUAS PROPRIEDADES

O prximo resultado nos dar uma frmula para resolver a equao


diofantina linear ax + by = c, onde mdc(a, b) = 1, conhecida uma
soluo particular x0 e y0 da equao.
Teorema 3.5. Seja x0 e y0 uma soluo particular, arbitrariamente
dada, da equao ax + by = c, onde mdc(a, b) = 1. Ento as solues
da equao so da forma x = x0 + tb e y = y0 ta, para t variando
em Z.
Demonstrao. Se x, y uma soluo qualquer da equao, temos que
ax + by = ax0 + by0 = c,
donde
a(x x0 ) = b(y0 y).

(3.4)

Da segue que a | b(y0 y) e b | a(x x0 ). Como mdc(a, b) = 1,


da Proposio 3.3 segue que a | (y0 y) e b | (x x0 ). Assim,
y0 y = ta

x x0 = sb,

(3.5)

para alguns inteiros t e s. Substituindo esse valores em (3.4), obtemos


asb = bta,
o que implica que s = t. Logo, de (3.5), temos que a soluo dada
por x = x0 + tb e y = y0 ta.
Reciprocamente, se x = x0 + bt e y = y0 at, substituindo esses
valores na equao ax + by = c, obtemos

i
i

Aritmetica
2009/6/29
i page 79
Estilo OBMEP

i
i

N SEC. 3.10: EQUAES DIOFANTINAS LINEARES

79

a(x0 + bt) + b(y0 at) = ax0 + by0 + abt bat = ax0 + by0 = c.

Por exemplo, a equao 3x + 5y = 50 admite a soluo particular


x0 = 0 e y0 = 10. Assim, a soluo geral dessa equao dada
por x = 0 + 5t e y = 10 3t. Se estivermos procura de solues
no negativas ento deveramos ter 10 3t 0, o que implica que
t = 0, 1, 2 ou 3. Assim, o Problema 3.54 admite as seguintes solues:
(a) 10 selos de 5 reais.
(b) 5 selos de 3 reais e 7 selos de 5 reais.
(c) 10 selos de 3 reais e 4 selos de 5 reais.
(d) 15 selos de 3 reais e um selo de 5 reais.
O nico verdadeiro trabalho que se tem para resolver uma equao
diofantina linear ax + by = c calcular mdc(a, b) para verificar se
divide ou no c e descobrir uma soluo particular x0 , y0 . O primeiro
problema se resolve utilizando o algoritmo de Euclides para o clculo
do mdc. Quanto ao segundo, o de determinar uma soluo particular
da equao, procede-se por inspeo se a e b so nmeros pequenos.
Caso a ou b seja grande, podemos usar o algoritmo de Euclides de
trs para a frente para determinar inteiros n e m tais que
an + bm = mdc(a, b) = 1,
e depois multiplicar ambos os membros da equao acima por c, ob-

i
i

Aritmetica
2009/6/29
i page 80
Estilo OBMEP

i
i

80

CAP. 3: OS INTEIROS E SUAS PROPRIEDADES

tendo
a(nc) + b(mc) = c,
dando-nos a soluo particular x0 = nc e y0 = mc.
Problema 3.58. De que maneiras podemos comprar selos de cinco
e de sete reais, de modo a gastar cem reais?
Problema 3.59. Se um macaco sobe uma escada de dois em dois
degraus, sobra um degrau; se ele sobe de trs em trs degraus, sobram
dois degraus. Quantos degraus a escada possui, sabendo que o nmero
de degraus mltiplo de sete e est compreendido entre 40 e 100.
Problema 3.60. Mostre que nenhum nmero pode deixar resto 5
quando dividido por 12 e resto 4 quando dividido por 15.
Problema 3.61. Ache todos os nmeros naturais que quando divididos por 18 deixam resto 4 e quando divididos por 14 deixam resto 6.

i
i

Aritmetica
2009/6/29
i page 81
Estilo OBMEP

i
i

Captulo 4

A Aritmtica dos Restos


4.1

Congruncias

Vamos agora introduzir a grande ideia de Gauss de desenvolver


uma aritmtica dos restos da diviso por um certo nmero fixado, o
que j foi explorado nas Sees 2.2 e 2.3.
Definio. Seja dado um nmero inteiro m maior do que 1. Diremos que dois nmeros inteiros a e b so congruentes mdulo m se
a e b possurem mesmo resto quando divididos por m. Neste caso,
simbolizaremos esta situao como segue:
a b mod m.
Quando a e b no so congruentes mdulo m, escreve-se
a 6 b mod m.
81

i
i

Aritmetica
2009/6/29
i page 82
Estilo OBMEP

i
i

82

CAP. 4: A ARITMTICA DOS RESTOS

Exemplos:
1) 15 8 mod 7, pois o restos das divises de 15 e de 8 por 7 so os
mesmos (iguais a 1).
2) 27 32 mod 5, pois os restos das divises de 27 e 32 por 5 so os
mesmos (iguais a 2).
3) 31 6 29 mod 3, pois o resto da diviso de 31 por 3 1, enquanto
o resto da diviso de 29 por 3 2.
Para mostrar que a b mod m no necessrio efetuar a diviso
de a e de b por m, como mostrado a seguir.
Proposio 4.1. Tem-se que a b mod m se e somente se m divide
b a.
Demonstrao. De fato, pelo algoritmo da diviso, podemos escrever
a = mq1 + r1

e b = mq2 + r2 ,

onde 0 r1 < m e 0 r2 < m. Sem perda de generalidade, podemos


supor que r1 r2 (se o contrrio ocorrer, basta trocar os papis de
r1 e r2 ). Assim, podemos escrever
b a = m(q2 q1 ) + r2 r1 .
Logo, m divide b a se, e somente se, m divide r2 r1 . Por ser
0 r2 r1 < m, segue que m divide b a se e somente se r2 r1 = 0,
ou seja, se e somente se r2 = r1 .

i
i

Aritmetica
2009/6/29
i page 83
Estilo OBMEP

i
i

N SEC. 4.1: CONGRUNCIAS

83

Problema 4.1. Verifique se so verdadeiras ou falsas as seguintes


afirmaes:
35 27 mod 4; 72 32 mod 5; 83 72 mod 5; 78 33 mod 9.
Problema 4.2. Se a b mod 4, mostre que a b mod 2.
Problema 4.3. Mostre que 10n 1 mod 9, para todo nmero natural n.
Sugesto: Veja o incio da Seo 2.3.
Problema 4.4. Dados a, b e c inteiros quaisquer e m um inteiro maior
do que 1, mostre as seguintes afirmaes:
(a) a a mod m.
(b) Se a b mod m, ento b a mod m.
(c) Se a b mod m e b c mod m, ento a c mod m.
Pela definio, as congruncias mdulo m tem tudo a ver com
os restos da diviso por m. A seguir exploramos mais a fundo esta
relao.
Segue-se, da definio de congruncia mdulo m e das propriedades do problema acima, o seguinte fato:
Todo nmero inteiro a congruente mdulo m a um e somente um
dos nmeros 0, 1, . . . , m 1.
De fato, os possveis restos da diviso de a por m so precisamente
os nmeros 0, 1, . . . , m 1, cujos restos da diviso por m so eles
prprios, logo dois a dois no congruentes mdulo m.

i
i

Aritmetica
2009/6/29
i page 84
Estilo OBMEP

i
i

84

CAP. 4: A ARITMTICA DOS RESTOS

Problema 4.5. Sejam a um nmero inteiro qualquer e m um inteiro maior do que 1. Suponha que r seja um nmero inteiro tal que
0 r < m e a r mod m. Mostre que r o resto da diviso de a
por m.
Sugesto: Utilize a unicidade da escrita no Algoritmo da Diviso.

4.2

Critrios de Multiplicidade e Restos

fcil determinar o resto da diviso de um inteiro n por 2, pois


esse 0 ou 1, dependendo de n ser par ou mpar.
Para facilitar a determinao do resto da diviso de um inteiro
n por 3 ou por 9, podemos utilizar os conhecimentos j adquiridos,
evitando o trabalho de efetuar a diviso em questo.
De fato, sabemos da Seo 2.3 que se nr . . . n1 n0 a escrita de n
no sistema decimal, ento
n (nr + + n1 + n0 ) = (10r 1)nr + + (10 1)n1 .
Como o segundo membro da igualdade acima divisvel por 3 e
por 9, o mesmo ocorre com o primeiro membro, logo
n (nr + + n1 + n0 ) mod 3; e mod 9.
Assim, pela definio de congruncia, temos os seguintes fatos:
O resto da diviso por 3 (respectivamente por 9) de um nmero
n = nr . . . n1 n0 , escrito no sistema decimal, igual ao resto da diviso
por 3 (respectivamente por 9) do nmero nr + + n1 + n0 .

i
i

Aritmetica
2009/6/29
i page 85
Estilo OBMEP

i
i

N SEC. 4.3: CONGRUNCIAS E SOMAS

85

Problema 4.6. Determine os restos da diviso por 3 e por 9 dos


nmeros: 3 254, 12 736, 54 827, 33 875 435, 57 612 510.
Da Seo 2.2 tambm sabemos que todo nmero n da forma
n = n0 + 10m, onde n0 o algarismo das unidades de n. Assim,
n n0 mod 5 e n n0 mod 10. Isto acarreta que:
Os restos da diviso de n por 5 e por 10 so, respectivamente, os
restos da diviso de n0 por 5 e por 10.
Problema 4.7. Determine os restos da diviso por 5 e por 10 dos
nmeros: 3 254, 12 736, 54 827, 33 875 435, 57 612 510.
Problema 4.8. Descreva critrios semelhantes aos estabelecidos acima para determinar os restos da diviso de um nmero por 4, 8, 25 e
125.
Problema 4.9. Determine os restos da diviso por 4, 8, 25 e 125 dos
nmeros: 3 254, 12 736, 54 827, 33 875 435, 57 612 510.
As congruncias possuem propriedades operatrias notveis que
exploraremos a seguir.

4.3

Congruncias e Somas

Proposio 4.2. Sejam a1 , a2 , b1 , b2 inteiros quaisquer e seja m um


inteiro maior do que 1. Se a1 b1 mod m e a2 b2 mod m, ento
a1 a2 b1 b2 mod m.
Demonstrao. De fato, como a1 b1 mod m e a2 b2 mod m, ento

i
i

Aritmetica
2009/6/29
i page 86
Estilo OBMEP

i
i

86

CAP. 4: A ARITMTICA DOS RESTOS

m divide b1 a1 e divide b2 a2 . Logo


m divide (b1 a1 ) (b2 a2 ) = (b1 b2 ) (a1 a2 ),
mostrando que b1 b2 a1 a2 mod m.
Conclui-se que as congruncias de mesmo mdulo somam-se e
subtraem-se membro a membro tal qual as igualdades.
Problema 4.10. Suponha que a b mod m. Mostre que
a c b c mod m,
qualquer que seja o inteiro c.
Problema 4.11. Suponha que a c b c mod m, mostre que
a b mod m.
Considere agora dois inteiros a e b cujos restos na diviso por m
sejam respectivamente r1 e r2 .
Ento temos que
a r1 mod m e b r2 mod m.
Assim,
a + b r1 + r2 mod m.
Seja r o resto da diviso de r1 + r2 por m; logo
a + b r1 + r2 r mod m, com 0 r < m.

i
i

Aritmetica
2009/6/29
i page 87
Estilo OBMEP

i
i

N SEC. 4.4: CONGRUNCIAS E PRODUTOS

87

Logo, pelo Problema 4.5, o resto da diviso de a + b por m o


nmero r.
Assim, acabamos de verificar o seguinte fato:
O resto da diviso da soma a + b de dois nmeros a e b por um outro
nmero m > 1 depende apenas dos restos da diviso de a e de b por
m e no desses nmeros em si.
Esse fato generaliza o que foi dito nas Sees 3.5 e 3.6, onde os
casos m = 2 e m = 3 foram analisados.
Problema 4.12. Sejam a e b dois nmeros inteiros cujos restos da
diviso por 7 so respectivamente 6 e 2. Determine os restos da diviso
de a + b, a b e de b a por 7
Sugesto: Para o ltimo resto, observe que 4 3 mod 7.
Problema 4.13. Sem efetuar as somas e subtraes indicadas, determine os restos da diviso por 2, 3, 4, 5, 8, 9, 10, 25 e 125 do nmero
3 534 785 + 87 538 9 535 832.

4.4

Congruncias e Produtos

Proposio 4.3. Sejam a1 , a2 , b1 , b2 inteiros quaisquer e seja m um


inteiro maior do que 1. Se a1 b1 mod m e a2 b2 mod m, ento
a1 a2 b1 b2 mod m.
Demonstrao. De fato, como a1 b1 mod m e a2 b2 mod m, ento

i
i

Aritmetica
2009/6/29
i page 88
Estilo OBMEP

i
i

88

CAP. 4: A ARITMTICA DOS RESTOS

m divide a1 b1 e a2 b2 . Por outro lado, como


a1 a2 b1 b2 = a1 (a2 b2 ) + b2 (a1 b1 ),
segue que m divide a1 a2 b1 b2 , o que prova o resultado.

Conclui-se que as congruncias de mesmo mdulo multiplicam-se


membro a membro tal qual as igualdades.
Problema 4.14. Suponha que a b mod m. Mostre que
a c b c mod m,
qualquer que seja o inteiro c.
Repetidas aplicaes da Proposio 4.3 fornecem o seguinte resultado:
Se a b mod m, ento an bn mod m, para todo n natural.
Sejam a e b dois inteiros cujos restos da diviso por m sejam
respectivamente r1 e r2 .
Ento temos que
a r1 mod m e b r2 mod m.
Assim,
a b r1 r2 mod m.

i
i

Aritmetica
2009/6/29
i page 89
Estilo OBMEP

i
i

N SEC. 4.4: CONGRUNCIAS E PRODUTOS

89

Seja r o resto da diviso de r1 r2 por m; logo


a b r1 r2 r mod m, com 0 r < m.
Logo, pelo Problema 4.5, o resto da diviso de a b por m o
nmero r.
Assim, acabamos de verificar que, como no caso da adio, vale
tambm seguinte fato para a multiplicao:
O resto da diviso do produto a b de dois nmeros a e b por um
outro nmero m > 1 depende apenas dos restos da diviso de a e de
b por m e no desses nmeros em si.
Isso tambm generaliza para a multiplicao o que foi dito nas
Sees 3.5 e 3.6, onde os casos m = 2 e m = 3 foram analisados.
Problema 4.15. Sejam a e b dois nmeros inteiros cujos restos da
diviso por 7 so respectivamente 6 e 2. Determine o resto da diviso
de a b por 7.
Problema 4.16. Sem efetuar as multiplicaes indicadas, determine os restos da diviso por 2, 3, 4, 5, 8, 9, 10, 25 e 125 do nmero
5 327 8343 3 842 5362 9 369 270 00120 .
Note que 2 3 2 6 mod 6, mas no entanto 3 6 6 mod 6.
Portanto, no caso das congruncias no vale um cancelamento anlogo
ao caso da igualdade.
Problema 4.17.
Sejam a, b, c e m nmeros inteiros e com
m > 1. Mostre que se a c b c mod m e se mdc(c, m) = 1, ento
a b mod m.

i
i

Aritmetica
2009/6/29
i page 90
Estilo OBMEP

i
i

90

CAP. 4: A ARITMTICA DOS RESTOS

Sugesto: Utilize a Proposio 3.3.

4.5

Algumas Aplicaes

1. Um critrio de divisibilidade por 6


Observe inicialmente que
10 4 mod 6,
102 42 4 mod 6,
103 102 10 4 4 4 mod 6,
104 103 10 4 4 4 mod 6.
Voc tem ainda alguma dvida de que 10i 4 mod 6, para todo
nmero natural i > 0?
Assim, se um nmero natural n escrito no sistema decimal como
nr . . . n1 n0 , temos que
n = n0 +10n1 +102 n2 + +10r nr n0 +4n1 +4n2 + +4nr mod 6.
Com isto, temos que o resto da diviso de n por 6 igual ao resto da
diviso de n0 + 4n1 + 4n2 + + 4nr por 6.
Logo, provamos que:
Um nmero n = nr . . . n1 n0 divisvel por 6 se e somente se
n0 + 4n1 + 4n2 + + 4nr divisvel por 6.
Problema 4.18. Ache o resto da diviso por 6 do nmero 3 215 529.

i
i

Aritmetica
2009/6/29
i page 91
Estilo OBMEP

i
i

N SEC. 4.5: ALGUMAS APLICAES

91

2. Um critrio de divisibilidade por 7, 11 e 13


Note que 7 11 13 = 1 001. Logo,
1 000 1 mod 7,

1 000 1 mod 11 e 1 000 1 mod 13.

Assim, mdulo 7, 11 e 13, temos que


103 1,
106 (1)2 1,
109 (1)3 1,
1012 (1)4 1,
etc.
Escrevendo um nmero n na representao decimal como
nr . . . n2 n1 n0 , temos, mdulo 7, 11 ou 13, que
n = n0 n1 n2 + n3 n4 n5 103 + n6 n7 n8 106 +
n0 n1 n2 n3 n4 n5 + n6 n7 n8 .
Assim, o resto da diviso de n por 7,11 ou 13 igual ao resto da
diviso de n0 n1 n2 n3 n4 n5 + n6 n7 n8 por 7, 11 ou 13, respectivamente.
Desse modo, obtemos o seguinte critrio de divisibilidade por 7,
11 ou 13:
O nmero nr . . . n2 n1 n0 divisvel por 7, 11 ou 13 se, e somente se,
o nmero n0 n1 n2 n3 n4 n5 + n6 n7 n8 divisvel por 7, 11 ou 13,
respectivamente.

i
i

Aritmetica
2009/6/29
i page 92
Estilo OBMEP

i
i

92

CAP. 4: A ARITMTICA DOS RESTOS

Problema 4.19. Ache o resto da diviso por 7, 11 e 13 do nmero


3 215 529.
Problema 4.20. Mostre que 10i (1)i mod 11, para todo natural
i. Deduza este outro critrio de divisibilidade por 11:
Um nmero nr . . . n2 n1 n0 divisvel por 11 se, e somente se, o nmero
n0 n1 + n2 divisvel por 11.
3. Os restos da diviso das potncias de 2 por 7
Observe que
21 2 mod 7,
22 4 mod 7,
23 1 mod 7.
Dado um nmero inteiro n, pelo algoritmo da diviso, podemos
escrev-lo na forma n = 3q + r, onde r = 0, 1 ou 2.
Assim,
2n = 23q+r = (23 )q 2r 2r mod 7.
Por exemplo, se n = 132 = 3 44, ento 2132 1 mod 7, pois
r = 0.
Se n = 133 = 3 44 + 1, ento 2133 2 mod 7, pois r = 1.
Se n = 134 = 3 44 + 2, ento 2134 4 mod 7, pois r = 2.
Problema 4.21. Ache o resto da diviso por 7 dos seguintes nmeros:
10
25 345 , 23 765 839 , 210 .

i
i

Aritmetica
2009/6/29
i page 93
Estilo OBMEP

i
i

N SEC. 4.5: ALGUMAS APLICAES

93

Problema 4.22. Sabendo que 24 = 16 1 mod 17, ache o resto


da diviso de 230 por 17.
Problema 4.23. Determine o resto da diviso de 2325 por 17.
4. A equao diofantina x3 117y3 = 5
Esta equao possui uma histria curiosa que relatada no livro
de S. Collier citado na bibliografia.
Vamos mostrar que esta equao no possui solues inteiras. De
fato, suponhamos, por absurdo, que x0 , y0 seja uma soluo inteira
da equao. Ento
x30 5 mod 9,
(4.1)
j que 117 0 mod 9.
Mas, sendo x0 congruente a 0, 1, 2, 3, 4, 5, 6, 7 ou 8 mdulo 9, segue
por contas elementares que x30 congruente a 0, 1 ou 8, mdulo 9.
Logo, a congruncia (4.1) no possui soluo, o que fornece uma contradio.
Problema 4.24. Mostre que a equao diofantina
x2 + y 2 + z 2 = 8w + 7
no possui solues x, y, z, w inteiros.
Sugesto: Reduza a equao mdulo 8 e mostre que
x20 + y02 + z02 7 mod 8
nunca ocorre.

i
i

Aritmetica
2009/6/29
i page 94
Estilo OBMEP

i
i

94

CAP. 4: A ARITMTICA DOS RESTOS

5. Os nmeros da forma 36n 26n so divisveis por 35


Temos que
36 = 33 33 (1) (1) 1 mod 7,
26 = 23 23 1 1 1 mod 7.

Por outro lado,


36 = 33 33 2 2 1 mod 5,
26 = 23 23 3 3 1 mod 5.

Logo, 36n 26n 0 mod 7 e 36n 26n 0 mod 5.


Assim, 36n 26n divisvel por 5 e por 7 e como mdc(5, 7) = 1,
segue, do Problema 3.42, que 36n 26n divisvel por 35.
Problema 4.25. Mostre que todo nmero da forma 198n 1 divisvel por 17.
Problema 4.26. Mostre que todo nmero da forma 133n + 173n
divisvel por 45, quando n mpar.
6. Euler tinha razo, Fermat estava enganado!
Na Seo 2.4 nos perguntamos se o nmero 4 294 967 297 era primo
ou composto?

i
i

Aritmetica
2009/6/29
i page 95
Estilo OBMEP

i
i

95

N SEC. 4.5: ALGUMAS APLICAES

De fato, esse nmero corresponde a n = 5 dos chamados nmeros


de Fermat que so da forma:
n

Fn = 22 + 1.
Fermat afirmou que esses nmeros, para qualquer valor natural
de n, eram primos e dava como exemplos F0 = 3, F1 = 5, F2 = 17,
F3 = 257 e F4 = 65 537, que so efetivamente primos.
5

No entanto, o nmero F5 = 22 + 1 = 4 294 967 297 era muito


grande para se poder verificar se era primo ou no.
Euler, estudando a forma dos divisores de um nmero do tipo de
Fn , chegou concluso de que se F5 fosse composto, ele deveria ser
divisvel pelo primo 641.
Euler, um exmio calculista, mostrou que 641 divide F5 com uma
verificao semelhante a que segue:1
Observemos inicialmente que 641 = 5 27 + 1, logo
5 27 1 mod 641.
Elevando quarta potncia ambos os membros da congruncia acima,
obtemos
54 228 1 mod 641.
(4.2)
Por outro lado, da igualdade 641 = 54 + 24 (verifique!), obtemos
que
54 24 mod 641.

(4.3)

Fizemos uma adaptao do argumento de Euler, pois no seu tempo ainda no


existia a noo de congruncia.

i
i

Aritmetica
2009/6/29
i page 96
Estilo OBMEP

i
i

96

CAP. 4: A ARITMTICA DOS RESTOS

Juntando (4.2) e (4.3), obtemos que 232 1 mod 641, o que


implica F5 = 232 + 1 0 mod 641, donde 641 divide F5 . Portanto,
F5 no primo.

4.6

Aritmtica Modular

A Aritmtica Modular foi introduzida por Gauss no seu livro


Disquisitiones Aritmeticae publicado em 1801.
Fixado um nmero inteiro m > 1, vamos associar a um nmero
inteiro a qualquer o smbolo a representando o resto da sua diviso
por m, tal qual fizemos nas Sees 3.5 e 3.6, nos casos m = 2 e m = 3.
Portanto, dados dois nmeros a e b tem-se que a = b se, e somente
se, os restos da diviso de a e de b por m so iguais, ou seja,
a = b se, e somente se, a b mod m.
Sendo todos os possveis restos da diviso por m os nmeros
0, 1, 2, . . . , m 1, temos qualquer a igual a um dos seguintes:
0, 1, . . . , m 1.
Nas Sees 4.3 e 4.4 observamos que os restos da diviso da soma
e do produto de dois nmeros no dependem dos nmeros em si, mas
apenas dos restos da diviso desses nmeros. Sendo assim, para achar
(a + b) e (a b) s precisamos saber como operar aditivamente e multiplicativamente com os smbolos a e b, que so justamente elementos
da forma 0, 1, . . . , m 1, a exemplo do que fizemos nas sees 3.5 e
3.6, nos casos m = 2 e m = 3.

i
i

Aritmetica
2009/6/29
i page 97
Estilo OBMEP

i
i

97

N SEC. 4.6: ARITMTICA MODULAR

Aritmtica mdulo m = 4
Para efeito de ilustrao, tomemos o caso m = 4. Neste caso, temos apenas os smbolos 0, 1, 2 e 3 a considerar.
Pede-se ao leitor verificar as seguintes tabelas:

0
1
2
3

0
1
2
3

1
2
3
0

2
3
0
1

3
0
1
2

0
1
2
3

0
0
0
0

0
1
2
3

0
2
0
2

0
3
2
1

Note que diferentemente da aritmtica dos nmeros inteiros, surge


um novo fenmeno: 2 6= 0 e, no entanto, 2 2 = 0.
Problema 4.27. Mostre que se i = 0, 1, 2, 3, ento i = 4 i.
Problema 4.28. Determine o resto da diviso por 4 do nmero:
45 769 834532 63 8761 654 + 87 987 5451 345 874 95 973 434

Aritmtica mdulo m = 5
Analisaremos agora o caso m = 5. Neste caso, temos apenas os
smbolos 0, 1, 2, 3 e 4 a considerar.
Pede-se ao leitor verificar as seguintes tabelas:

i
i

Aritmetica
2009/6/29
i page 98
Estilo OBMEP

i
i

98

CAP. 4: A ARITMTICA DOS RESTOS

0
1
2
3
4

0
1
2
3
4

1
2
3
4
0

2
3
4
0
1

3
4
0
1
2

4
0
1
2
3

0
1
2
3
4

0
0
0
0
0

0
1
2
3
4

0
2
4
1
3

0
3
1
4
2

0
4
3
2
1

Note que aqui volta a valer a regra: se a 6= 0 e b 6= 0, ento


a b 6= 0.
Problema 4.29. Mostre que se i = 0, 1, 2, 3, 4, ento i = 5 i.
Problema 4.30. Determine o resto da diviso por 5 do nmero:
45 769 834532 63 8761 654 + 87 987 5451 345 874 95 973 434
Problema 4.31. Determine as tabelas da adio e da multiplicao
para m = 6 e para m = 7. Que diferenas voc nota entre os dois
casos?

i
i

Aritmetica
2009/6/29
i page 99
Estilo OBMEP

i
i

Captulo 5

Problemas Suplementares
Apresentaremos neste captulo uma lista de problemas mais desafiadores do que aqueles que se encontram no texto, cujo objetivo se
restringia a complement-lo, alm de testar a compreenso do leitor
nos conceitos apresentados.
Nos dois primeiros captulos apresentamos a linguagem bsica e
os resultados fundamentais, sem os quais no seria possvel enunciar,
muito menos resolver, problemas mais elaborados. Os problemas propostos a seguir dizem respeito ao material exposto nos Captulos 3 e
4. Os problemas marcados com asterisco tm um grau de dificuldade
maior que os demais.
Antes porm de iniciar os problemas propriamente ditos, relacionamos a seguir algumas identidades muito teis na resoluo de
alguns dos problemas.

99

i
i

Aritmetica
2009/6/29
i page 100
Estilo OBMEP

i
i

100

CAP. 5: PROBLEMAS SUPLEMENTARES

Expresses do tipo a n 1, com n qualquer


a2 1 = (a 1)(a + 1)
a3 1 = (a 1)(a2 + a + 1)
a4 1 = (a 1)(a3 + a2 + a + 1)
a5 1 = (a 1)(a4 + a3 + a2 + a + 1)
Em geral,
an 1 = (a 1)(an1 + an2 + + a + 1).
Expresses do tipo a m 1, com m par
a2 1 = (a + 1)(a 1)
a4 1 = (a + 1)(a3 a2 + a 1)
a6 1 = (a + 1)(a5 a4 + a3 a2 + a 1)
Em geral,
a2n 1 = (a + 1)(a2n1 a2n2 + + a 1).
Expresses do tipo a m + 1, com m mpar
a3 + 1 = (a + 1)(a2 a + 1)
a5 + 1 = (a + 1)(a4 a3 + a2 a + 1)
a7 + 1 = (a + 1)(a6 a5 + a4 a3 + a2 a + 1)
Em geral,
a2n+1 + 1 = (a + 1)(a2n a2n1 + a + 1).

i
i

Aritmetica
2009/6/29
i page 101
Estilo OBMEP

i
i

101
Problemas sobre o Captulo 3
S-3.1 Mostre que todo nmero inteiro no nulo a se escreve de modo
nico na forma a = 2r b, onde r N e b um nmero mpar. O nmero
2r a maior potncia de 2 que divide a. Generalize esta propriedade
para um primo p qualquer no lugar de 2.
S-3.2
(a) Quantos mltiplos de 5 existem no intervalo [1, 120]? e no intervalo [1, 174]?
(b) Quantos mltiplos de 7 existem em cada um dos intervalos
[70, 342] e [72, 342]?
S-3.3 Dados 0 < a n < m, mostre que no intervalo [1, n] existem
q mltiplos de a, onde q o quociente da diviso de n por q. Quantos
so os mltiplos de a no intervalo [n, m]? (Na ltima situao, divida
a anlise em dois casos: n mltiplo de a e o contrrio.)
S-3.4 Mostre que dados m inteiros consecutivos um, e apenas um,
deles mltiplo de m.
S-3.5 Com quantos zeros termina o nmero 2 3 4 120? E
o nmero 2 3 4 174?
S-3.6 Mostre que o produto de quatro nmeros inteiros consecutivos,
quaisquer, sempre mltiplo de 24.
S-3.7
(a) Mostre que se n mpar, ento n2 1 mltiplo de 8.

i
i

Aritmetica
2009/6/29
i page 102
Estilo OBMEP

i
i

102

CAP. 5: PROBLEMAS SUPLEMENTARES

(b) Mostre que se n mpar, ento n(n2 1) mltiplo de 24.


(c) Mostre que se n no mltiplo de 2 nem de 3, ento n2 1
mltiplo de 24. Mostre que o mesmo vale para n2 + 23.
S-3.8
(a) Mostre que se um nmero a no divisvel por 3, ento o resto
da diviso de a2 por 3 1.
(b) A partir desse dado, mostre que se um inteiro da forma a2 + b2
mltiplo de 3, ento a e b so ambos mltiplos de 3.
S-3.9 Mostre que se n > 1, ento o nmero n4 + 4 composto.
S-3.10
(a) Mostre que o nico nmero primo da forma n3 + 1 2.
(b) Mostre que o nico nmero primo da forma n3 1 7.
S-3.11* Mostre que, dado n > 2, entre n e 2 3 n existe
sempre um nmero primo. (Note que esta afirmao bem mais fraca
do que o Postulado de Bertrand.)
S-3.12
(a) Ache o menor inteiro positivo n tal que o nmero 4n2 + 1 seja
divisvel por 65.
(b) Mostre que existem infinitos mltiplos de 65 da forma 4n2 + 1.

i
i

Aritmetica
2009/6/29
i page 103
Estilo OBMEP

i
i

103
(c) Mostre que se um dado nmero divide um nmero da forma
4n2 + 1, ele dividir uma infinidade desses nmeros.
(d) Para este ltimo resultado, existe algo de especial nos nmeros
da forma 4n2 + 1?Teste o seu resultado para nmeros da forma
an2 + bn + c, onde a, b, c Z, com a e b no simultaneamente
nulos.
(e) Mostre que existem infinitos mltiplos de 7 da forma 8n2 +3n+4.
S-3.13
(a) Sejam dados os dois nmeros a = 10c + r e b = c 2r, com
c, r Z. Mostre que a mltiplo de 7 se, e somente se, b
mltiplo de 7.
(b) Deduza o seguinte critrio de multiplicidade de 7:
O nmero n = ar a1 a0 mltiplo de 7 se, e somente se, o
nmero ar a1 2a0 mltiplo de 7.
(c) Utilize repetidas vezes o critrio acima para verificar se 2 368
ou no mltiplo de 7.
Um nmero inteiro n dito um quadrado se existe a Z tal que
n = a2 . Dizemos que n uma potncia m-sima quando n = am .
S-3.14
(a) Mostre que o algarismo das unidades de um quadrado s pode
ser um dos seguintes: 0, 1, 4, 5, 6 e 9.

i
i

Aritmetica
2009/6/29
i page 104
Estilo OBMEP

i
i

104

CAP. 5: PROBLEMAS SUPLEMENTARES

(b) Mostre que nenhum dos nmeros 22, 222, 2 222, . . ., ou


33, 333, 3 333, . . ., ou 77, 777, 7 777, . . ., ou ainda
88, 888, 8 888, . . . pode ser um quadrado.
S-3.15
(a) Mostre que todo quadrado mpar da forma 4n + 1.
(b) Mostre que nenhum nmero na sequncia 11, 111, 1 111, 11 111
etc., um quadrado.
(c) Mostre que nenhum nmero na sequncia 44, 444, 4 444, 44 444
etc., um quadrado.
(d) Mostre que nenhum nmero na sequncia 99, 999, 9 999, 99 999
etc., um quadrado.
(e) Mostre que nenhum nmero na sequncia 55, 555, 5 555, 55 555
etc., um quadrado.
S-3.16
(a) Mostre que nenhum nmero da forma 4n + 2 um quadrado.
(b) Mostre que nenhum dos nmeros 66, 666, 6 666, . . . um
quadrado.
S-3.17
(a) Mostre que a soma de quatro inteiros consecutivos nunca um
quadrado.

i
i

Aritmetica
2009/6/29
i page 105
Estilo OBMEP

i
i

105
(b) Mostre que a soma dos quadrados de quatro inteiros consecutivos nunca um quadrado. Faa o mesmo para a soma dos
quadrados de trs inteiros consecutivos.
S-3.18
(a) Mostre que todo quadrado da forma 8n, 8n + 1 ou 8n + 4.
(b) Mostre que nenhum nmero na sequncia 3, 11, 19, 27 etc.,
um quadrado.
S-3.19 Mostre que numa sequncia de inteiros da forma
a, a + d, a + 2d, a + 3d, . . .
se existir algum nmero que quadrado, existiro infinitos nmeros
que so quadrados.
S-3.20*
(a) Mostre que todo nmero inteiro mpar pode ser representado
como diferena de dois quadrados.
(b) Mostre que se p = 1 ou se p > 2 um nmero primo, ento p
se escreve de modo nico como diferena de dois quadrados de
nmeros naturais.
(c) Mostre que todo nmero da forma 4k n, onde n mpar se escreve
como diferena de dois quadrados.

i
i

Aritmetica
2009/6/29
i page 106
Estilo OBMEP

i
i

106

CAP. 5: PROBLEMAS SUPLEMENTARES

(d) Mostre que se um nmero par diferena de dois quadrados,


ento ele mltiplo de 4.
S-3.21 Mostre que todo cubo diferena de dois quadrados, ou seja,
dado a Z, existem x, y Z tais que a3 = x2 y 2 .
S-3.22* Ache os nmeros n para os quais o nmero n(n + 14) seja
um quadrado.
Um nmero inteiro m 6= 0 dito livre de quadrados, quando no
houver nenhum nmero a 6= 1 tal que a2 divide m.
Diremos que m 6= 0 livre de cubos quando no houver nenhum
nmero a 6= 1 tal que a3 divide m.
S-3.23
(a) Mostre que m livre de quadrados se, e somente se, a decomposio de m em fatores primos da forma p1 pr , onde
p1 , . . . , pr so primos distintos.
(b) Mostre que m livre de cubos se, e somente se, a decomposio
de m em fatores primos da forma pn1 1 pnr r , onde p1 , . . . , pr
so primos distintos e ni 2, para todo i = 1, . . . , r.
S-3.24 Qual o maior nmero de inteiros positivos consecutivos
livres de quadrados? E livres de cubos? Generalize.
S-3.25 Mostre que 5 o nico nmero primo que pertence a dois
pares distintos de primos gmeos.

i
i

Aritmetica
2009/6/29
i page 107
Estilo OBMEP

i
i

107
S-3.26 Mostre que se n composto, ento n divide o produto
1 2 3 (n 1).

S-3.27 Dados dois inteiros a e b distintos, mostre que existem infinitos nmeros n para os quais mdc(a + n, b + n) = 1.
S-3.28 Calcule mdc(n + 1, n2 + 1), para n Z.
S-3.29 Mostre que se a e b so dois nmeros naturais tais que
mdc(a, b) = mmc(a, b), ento a = b.
S-3.30 Resolva o seguinte sistema de equaes:
(

mdc(x, y) = 6
mmc(x, y) = 60

S-3.31 Observe que mdc(x, y) divide mmc(x, y), quaisquer que sejam
x, y Z, no nulos.
Mostre que se no seguinte sistema:
(

mdc(x, y) = d
mmc(x, y) = m

d - m, ele no admite soluo. Mostre que se d | m, o sistema sempre


admite soluo.
S-3.32 Observe que [mdc(x, y)]2 divide xy, quaisquer que sejam
x, y Z, no nulos.

i
i

Aritmetica
2009/6/29
i page 108
Estilo OBMEP

i
i

108

CAP. 5: PROBLEMAS SUPLEMENTARES

Mostre que se o seguinte sistema:


(

mdc(x, y) = d
xy = m

tal que d2 - m, ele no admite soluo. Mostre que se d2 | m, o sistema sempre admite soluo.
S-3.33
(a) Ache os nmeros primos da forma a2 1.
(b) Existem primos da forma a3 1? E a4 1?
(c) Mostre que se a > 2 e n > 1, ento an 1 composto.
(d) Mostre que se n composto, ento 2n 1 composto.
Portanto, se 2n 1 primo, ento n primo. Nmeros primos da
forma 2p 1, onde p primo so chamados primos de Mersenne.
S-3.34
(a) Mostre que todo cubo que tambm um quadrado da forma
5n, 5n+1 ou 5n+4 (ou seja, nunca da forma 5n+2 ou 5n+3).
(b) Mostre que todo cubo que tambm um quadrado da forma
7n, 7n + 1.
S-3.35
(a) Mostre que todo primo maior do que 3 da forma 3n + 1 ou
3n + 2.

i
i

Aritmetica
2009/6/29
i page 109
Estilo OBMEP

i
i

109
(b) Mostre que qualquer nmero da forma 3n + 2 tem um fator
primo da mesma forma.
(c*) Mostre que existem infinitos primos da forma 3n + 2.
(d) Existem infinitos primos da forma 3n + 1, mas a prova disso
mais sutil.
S-3.36
(a) Mostre que todo primo maior do que 3 da forma 4n + 1 ou
4n + 3.
(b) Mostre que qualquer nmero da forma 4n + 3 tem um fator
primo da mesma forma.
(c*) Mostre que existem infinitos primos da forma 4n + 3.
(d) Existem infinitos primos da forma 4n + 1, mas a prova disso
um pouco mais sutil (veja Elementos de Aritmtica, Proposio
8.1.4).
S-3.37 Mostre que todo nmero primo da forma 3k + 1 da forma
6n + 1.
S-3.38
(a) Mostre que todo primo maior do que 3 da forma 6n + 1 ou
6n 1.
(b) Mostre que qualquer nmero da forma 6n 1 tem um fator
primo da mesma forma.

i
i

Aritmetica
2009/6/29
i page 110
Estilo OBMEP

i
i

110

CAP. 5: PROBLEMAS SUPLEMENTARES

(c*) Mostre que existem infinitos primos da forma 6n 1.


(d) Mostre que existem infinitos primos da forma 6n + 1 (Utilize os
Problemas S-3.37 e S-3.35 (d)).
As propriedades enunciadas nos Problemas S-3.35 (c) e (d),
S-3.36 (c) e (d) e S-3.38 (c) e (d) so casos particulares de um teorema
profundo e de difcil demonstrao do matemtico Alemo LejeuneDirichlet (1805-1859), que afirma que se a e b so dois nmeros primos
entre si, ento h infinitos nmeros primos da forma an + b.
S-3.39 Verifique caso a caso que p divide 2p 2 para p primo e p 7.
S-3.40
(a) Mostre que em geral p divide ap a, para todo a Z e para
todo p primo p 7.
(b) Verifique que se p no divide a, com p nas condies de (a),
ento p divide ap1 1, para todo a Z.
(c) Ache o resto da diviso por 7 do nmero 16 + 26 + 36 + + 156 .
(d) Mostre que se a e b so primos com 7, ento b6 a6 mltiplo
de 7. Em particular, 236 186 mltiplo de 7.
Os problemas S-3.39 e S-3.40 so casos particulares de um resultado geral chamado Pequeno Teorema de Fermat, cujo enunciado :
Para todo primo p e todo inteiro a tem-se que p divide ap a. Alm
disso, se p no divide a, ento p divide ap1 1.

i
i

Aritmetica
2009/6/29
i page 111
Estilo OBMEP

i
i

111
Para uma prova, consulte o livro Elementos de Aritmtica, Teorema 7.3.1 e o seu corolrio.
S-3.41
(a) Mostre que 30 divide n5 n.
(b) Mostre que n5 e n tm sempre o mesmo algarismo das unidades.
(c) Mostre que o nmero 15 n5 + 13 n3 +
inteiro n.

7
15 n

um inteiro para todo

S-3.42 Mostre que 42 divide n7 n.


S-3.43 Utilizando o Pequeno Teorema de Fermat, enunciado acima,
mostre que se p um nmero primo, com p 6= 2, 5, ento p divide
infinitos elementos da sequncia 9, 99, 999, 9999, . . . Mostre tambm
que p divide infinitos elementos da sequncia 1, 11, 111, 1111, . . .
S-3.44 Quantos divisores positivos tem um nmero primo p? E pn ?
E pn q m , com p e q primos distintos?
S-3.45 Ache o menor nmero natural que possui exatamente seis
divisores positivos. Faa o mesmo para 15 divisores e para 100 divisores.
S-3.46 Mostre que se mdc(a, c) = 1 e mdc(b, c) = 1, ento
mdc(ab, c) = 1.
S-3.47 Mostre que
(a) mdc(a2 , b2 ) = [mdc(a, b)]2 .

(b) mmc(a2 , b2 ) = [mmc(a, b)]2 .

i
i

Aritmetica
2009/6/29
i page 112
Estilo OBMEP

i
i

112

CAP. 5: PROBLEMAS SUPLEMENTARES

(c) Generalize.
S-3.48 Sejam a e b inteiros e n um nmero natural. Mostre que
se a b uma potncia n-sima e mdc(a, b) = 1, ento a e b so
potncias n-simas.
S-3.49 (Esse um problema proposto no sculo 16) Um total de
41 pessoas entre homens, mulheres e crianas foram a um banquete
e juntos gastaram 40 patacas. Cada homem pagou 4 patacas, cada
mulher 3 patacas e cada criana um tero de pataca. Quantos homens,
quantas mulheres e quantas crianas havia no banquete?
S-3.50 (Proposto por Euler) Um grupo de homens e mulheres gastaram numa taberna 1 000 patacas. Cada homem pagou 19 patacas
e cada mulher 13. Quantos eram os homens e quantas eram as mulheres?
S-3.51 (Proposto por Euler) Uma pessoa comprou cavalos e bois.
Foram pagos 31 escudos por cavalo e 20 por boi e sabe-se que todos
os bois custaram 7 escudos a mais do que todos os cavalos. Quantos
cavalos e quantos bois foram comprados?
S-3.52
(a) Dados a e b inteiros fixados, quando que os nmeros da forma
ax + by, com x, y Z representam todos os inteiros?
(b) Quais so os nmeros representados por 2x + 3y?
(c) Quais so os nmeros representados por 6x + 9y?

i
i

Aritmetica
2009/6/29
i page 113
Estilo OBMEP

i
i

113
S-3.53 Em um certo pas, as cdulas so de $4 e $7. Quais das afirmaes a seguir so verdadeiras? Com elas possvel pagar, sem troco, qualquer quantia inteira
(a) a partir de $11, inclusive.
(b) a partir de $18, inclusive.
(c) mpar, a partir de $7, inclusive.
(d) que seja $1 maior do que um mltiplo de $3.
(e) que seja $1 menor do que um mltiplo de $3.
S-3.54 Em um quintal onde so criados coelhos e galinhas contaramse 400 ps. Quantas so as galinhas e quantos so os coelhos, sabendo
que diferena entre esses dois nmeros a menor possvel.
S-3.55 Vimos no Problema S-3.16 que um quadrado nunca da forma 4n + 2. Usando este fato, mostre que a equao x2 + y 2 = z 2 no
admite nenhuma soluo em x, y e z, com x e y mpares.
S-3.56 Mostre que a equao x2 + y 2 = z 2 no admite nenhuma soluo em x, y e z, com x e y ambos primos com 3.
S-3.57 Mostre que se m e n so nmeros inteiros, ento x = 2mn,
y = m2 n2 e z = m2 + n2 so solues da equao pitagrica
x2 + y 2 = z 2 .

i
i

Aritmetica
2009/6/29
i page 114
Estilo OBMEP

i
i

114

CAP. 5: PROBLEMAS SUPLEMENTARES

Problemas sobre o Captulo 4


S-4.1
(a) Mostre que os restos da diviso de n inteiros consecutivos so
os nmeros 1, 2, . . . , n em alguma ordem.
(b) Utilizando a frmula:
1 + 2 + + n =

n(n + 1)
,
2

conclua que a soma de n inteiros consecutivos quando dividida


por n deixa resto zero se n mpar e metade de n, se n par.
(c) Ache os restos da diviso de 2 356 + 2 357 + 2 358 + 2 359 + 2 360
por 5 e de 32 141+32 142+ +32 149+32 150+32 151+32 152
por 12.
S-4.2 Mostre que, para todo n N,
(a) 7 divide 32n+1 + 2n+2 .
(b) 9 divide 10n + 3 4n+2 + 5.
(c) 24 divide 2 7n + 3 5n 5.
(d) 35 divide 36n 26n .
(e) 64 divide 72n + 16n 1.

i
i

Aritmetica
2009/6/29
i page 115
Estilo OBMEP

i
i

115
S-4.3 Sabendo que 74 = 2 401, ache os ltimos dois dgitos de 799 999 .
S-4.4 Ache o resto da diviso de 21 000 000 por 77.
S-4.5 Mostre que 1436 + 9110 + 7712 1 mltiplo de 1 001.
S-4.6 Mostre que 2 2225 555 + 5 5552 222 mltiplo de 7.
S-4.7 Mostre que 19 nunca divide um nmero da forma 4n2 + 4.
S-4.8 Quais so os dois ltimos algarismos na representao no sistema decimal do nmero 3400 ? E do nmero 2400 ?
S-4.9 Qual o algarismo da unidade na representao decimal do
9
7
nmero 99 ? E do nmero 77 ?
S-4.10 Ache os algarismos das centenas e das unidades na represen1 000
tao decimal dos nmeros 7999 999 e 77
.
S-4.11 Ache o resto da diviso
(a) de 560 por 26.

(b) de 3100 por 34

(c) de 21 000 000 por 77.

S-4.12 Determine os restos da diviso por 4 dos nmeros:


(a) 1 + 2 + 22 + 23 + + 2100

(b) 15 + 25 + 35 + + 205 .

S-4.13 Mostre que a congruncia x2 + 1 0 mod 7 no possui solues. Conclua que a equao x2 6x 77 = 7y no admite solues
inteiras.
S-4.14 Mostre que a equao x2 13y 2 = 275 no admite solues
inteiras.

i
i

Aritmetica
2009/6/29
i page 116
Estilo OBMEP

i
i

116

CAP. 5: PROBLEMAS SUPLEMENTARES

S-4.15 Mostre que se um nmero da forma 7n 5 mltiplo de 5,


ento o nmero 12n2 + 19n 19 mltiplo de 25.
S-4.16 Mostre que se um nmero da forma 2n + 1 mltiplo de 3,
ento o nmero 28n2 13n 5 mltiplo de 9.
S-4.17 Mostre que valem as seguintes congruncias:
(a) n13 n mod p, para p = 2, 3, 5, 7 e 13, e para todo n Z.
(b) Se mdc(n, p) = 1, mostre que n12 1 mod p, para p = 2, 3, 5, 7
e 13.
Partes do problema acima so casos particulares do Pequeno Teorema de Fermat, que pode ser reenunciado como segue:
Para todo primo p e todo inteiro a tem-se que ap a mod p. Alm
disso, se p no divide a, ento ap1 1 mod p.
S-4.18 Resolva a congruncia 3x 5 mod 11.
S-4.19 Determine os inteiros que deixam restos 1, 2 e 3, quando divididos respectivamente por 3, 4 e 5.
S-4.20 Mostre que nenhum nmero da forma 4n + 3 pode ser escrito
como soma de dois quadrados.

i
i

Aritmetica
2009/6/29
i page 117
Estilo OBMEP

i
i

Solues e Respostas
Problemas do Captulo 1
1.1 , {3}, {2}, {2, 3}, {4, 5, 6}, {4, 5, 6, 7}, {3, 4, 5, 6} e {3, 4, 5, 6, 7}.
1.2 2, 3, 4, no tem, 3 e 2.
1.3 Por causa da comutatividade da adio pode-se separar essas 12
expresses em trs grupos:
(a + b) + c = (b + a) + c = c + (a + b) = c + (b + a),
a + (b + c) = a + (c + b) = (b + c) + a = (c + b) + a,
(a + c) + b = b + (a + c) = b + (c + a) = (c + a) + b.
Portanto, novamente, pela comutatividade da adio, temos
(a + b) + c = a + (b + c) = a + (c + b) = (a + c) + b,
e, consequentemente, os 12 nmeros listados acima so iguais.
1.6 Faltam 200 50 = 150 reais.
117

i
i

Aritmetica
2009/6/29
i page 118
Estilo OBMEP

i
i

118
1.7 Pela tricotomia, temos trs possibilidades:
a c > b c, a c = b c ou a c < b c.
Somando c a ambos os lados da primeira e da segunda possibilidade
obtemos uma contradio, logo s resta a terceira possibilidade.
1.8 So 72 37 + 1 = 36 nmeros.
1.9 So 75 32 = 43 nmeros, tanto no intervalo (32, 75], quanto no
intervalo [32, 75) e 75 32 1 = 42 nmeros no intervalo (32, 35).
1.10 b a nos dois primeiros casos e b a 1 no ltimo.
1.11 No so. Se fossem, teramos 1 = la, com a > 1, o que no
possvel.
1.12 5, 10, 15, 20, 25, 30, 35, 40, 45, 50.
1.15
(a) Considerando a sequncia 32 = 8 4, 8 5, . . . , 8 1 000, segue
que o nmero de mltiplos de 8 1 000 4 + 1 = 997.
(b) Considerando a sequncia 1 606 2, . . . , 3 160 2, segue que o
nmero de nmeros pares 3 160 1 606 + 1 = 1 555.
(c) 15 e 18 dzias, respectivamente.
(d) 40 e 51 semanas, respectivamente.
1.23 28, 56, 84, . . .

i
i

Aritmetica
2009/6/29
i page 119
Estilo OBMEP

i
i

119
1.24 12, 66, 24 e 9.
1.26 (a + b)5 = a5 + 5a4 b + 10a3 b2 + 10a2 b3 + 5ab4 + b5 .
Problemas do Captulo 2
2.1 Os nmeros so
2 6, 3 6, . . . , 16 6,
cuja soma
(2 + 3 + + 16) 6 = 135 6 = 810.
2.2 Se os algarismos so a, b e c, os seis nmeros so ab = 10a + b,
ba = 10b + a, ac = 10a + c, ca = 10c + a, bc = 10b + c e cb = 10c + b,
logo a sua soma
10a + b + 10b + a + 10a + c + 10c + a + 10b + c + 10c + b = 22(a + b + c).
2.4 10; 99; 99 10 + 1 = 90; 2 90 = 180.
2.5 So necessrios 792 algarismos. Ao confrontar com a frmula
Q(x) no se esquea que no existe pgina 0.
2.6 Seja n0 , onde 0 n0 9, o algarismo das unidades de a. Escreva
a na forma 10m + n0 , e o eleve ao quadrado.
2.16 4 = 22 , 6 = 2 3, 8 = 23 , 36 = 22 32 , 84 = 22 3 7,
320 = 26 5 e 2.597 = 72 53.

i
i

Aritmetica
2009/6/29
i page 120
Estilo OBMEP

i
i

120
Problemas do Captulo 3
3.9 Pela propriedade sugerida, tem-se que 216 144 = 72 um
mltiplo comum, logo mmc(a, b) 72.
3.16
(a) mdc(n, 2n + 1) = mdc(n, 2n + 1 2n) = mdc(n, 1) = 1.
(b) e (c) mdc(n, 2n + 2) = mdc(n, 2n + 2 2n) = mdc(n, 2), que 1 ou
2 segundo se n mpar ou par.
3.17 Se mdc(a0 , b0 ) = d0 > 1, ento a = a00 d0 d e b = b00 d0 d, logo dd0
seria um divisor comum de a e b maior do que d, absurdo.
3.18 43 = 3 14 + 1, 43 = 5 8 + 3, 233 = 4 58 + 1,
1 453 = 10 145 + 3, 1 453 = 100 14 + 53, 1 453 = 1 000 1 + 453
e 1 453 = 10 000 0 + 1 453.
3.20 43 = 3(15) + 2, 43 = 5(9) + 2 233 = 4(59) + 3,
1 453 = 10 (146) + 7, 1 453 = 100(15) + 47, 1 453 =
1 000(2) + 547, 1 453 = 10 000(1) + 8 547.
3.24 Um nmero a da forma 3q + i, onde i = 0, 1, 2. Agora analise
cada caso separadamente. Se a, a + 2 e a + 4 so primos trigmeos,
um deles divisvel por 3 e sendo um nmero primo, ele igual a
3. Analisando as trs possibilidades conclui-se que a = 3 e, portanto,
3, 5 e 7 a nica terna de primos trigmeos.
3.25 e 3.26 Escreva a na forma 3q + i, i = 0, 1, 2.

i
i

Aritmetica
2009/6/29
i page 121
Estilo OBMEP

i
i

121
3.27 3 257 = 5 651 + 2. Logo so produzidos 651 pacotes de chicletes.
3.28 Escrevamos o nmero mpar na forma 2n + 1, logo
2(2n + 1) = 4n + 2,
que no mltiplo de 4.
3.29 A paridade determinada por
234

(1 + 1)234 + (1 + 0)542 = 0

542

+1

= 1,

logo mpar.
3.33 O resto da diviso por 3 se calcula como segue:
100

+ (2 )15 = 1 + 1

15

= 1 + 1 = 2.

Portanto, o resto 2.
3.34 Um mltiplo de 6 obviamente mltiplo de 2 e de 3. Reciprocamente, todo mltiplo de 2 e de 3 mltiplo do mmc desses nmeros
que 6.
3.35 Um nmero mltiplo de 6 se, e somente se, o seu algarismo
das unidades par e a soma de seus algarismos mltiplo de 3.
3.36 Podemos escrever
n(n + 1)(2n + 1) = n(n + 1)(n 1 + n + 2)
= (n 1)n(n + 1) + n(n + 1)(n + 2).

i
i

Aritmetica
2009/6/29
i page 122
Estilo OBMEP

i
i

122
Agora note que cada parcela na ltima linha mltiplo de 2 e de
3, donde o resultado segue levando em conta o Problema 3.34.
3.39 mmc(4, 6, 9) = mmc(mmc(4, 6), 9) = mmc(12, 9) = 36.
3.40 Se m mltiplo comum de a e b, temos m = r a e m = s b.
Logo, a b = r a d e a b = s b d. Assim, temos que b = r d
e a = s d, mostrando que d divisor comum de a e b.
Reciprocamente, se d divisor comum de a e b temos que b = r d
e a = s d. Logo de a b = m d, obtemos s b = m e r a = m.
Conclumos assim que m mltiplo comum de a e b.
3.41 Como mdc(n, 2n+1) = 1, segue que mmc(n, 2n+1) = n(2n+1).
3.42 Sendo n mltiplo de a e de b, ele mltiplo de seu mmc. Por
outro lado, sendo mdc(a, b) = 1, temos que mmc(a, b) = a b, logo n
mltiplo de a b, logo divisvel por ele.
3.43
(a) 8 = 728 37 + 1 496 (1).
(b) 6 = 108 (15) + 294 7.
3.44 Denotemos por min A o menor elemento de um conjunto de
nmeros naturais A. Sabemos da Proposio 3.1 que
mdc(a, b) = min{x aZ + bZ; x > 0}.

i
i

Aritmetica
2009/6/29
i page 123
Estilo OBMEP

i
i

123
Portanto,
mdc(ca, cb) = min{x acZ + bcZ; x > 0}
= c min{x aZ + bZ; x > 0}
= c mdc(a, b).
3.45 O resultado segue da frmula do Teorema 3.2:
mdc(a, b) mmc(a, b) = a b,
e do Problema 3.44.
3.46 Como p primo, os seus nicos divisores so 1 e p. Logo
mdc(a, p) = 1 ou mdc(a, p) = p. Na segunda possibilidade teremos
p | a.
3.47 Do exerccio anterior, temos que p | a ou mdc(a, p) = 1. No
primeiro caso, nada temos a provar. No segundo caso, como p | a b,
segue da Proposio 3.3 que p | b.
3.48 Sendo p primo, se p | p1 p2 , pelo problema anterior, p | p1 ou
p | p2 . Como p1 e p2 so primos, isto acarreta que p = p1 ou p = p2 .
3.49 Suponhamos que p1 pr = q1 qs . Portanto, p1
divide q1 qs , logo p1 igual a um dos qi , que aps reordenamento
podemos supor ser q1 . Assim, p1 pr = p1 qs , logo
p2 pr = q2 qs . Continuando desse modo, se r = s,
a demonstrao est completa. Suponhamos s > r (o outro caso
semelhante) temos que 1 = qr+1 qs , o que no possvel.

i
i

Aritmetica
2009/6/29
i page 124
Estilo OBMEP

i
i

124
3.50 1, 2, 4, 8, 5, 10, 20, 40 e 1, 2, 4, 8, 5, 10, 20, 40, 3, 6, 12, 24,
15, 30, 60, 120.
3.51 Tem 48 divisores.
3.52 Sendo 1 080 = 23 33 5 70 e 210 = 2 3 5 7, temos que
mdc(1 080, 210) = 2 3 5 e mmc(1 080, 210) = 23 33 5 7.
3.55 (a) tem soluo (b) no tem soluo (c) tem soluo
(d) tem soluo (e) tem soluo (f) tem soluo.
3.56 mdc(a, b) = 1 divide qualquer nmero c.
3.57 Quando c for mltiplo de mdc(30, 42) = 6.
3.58 A equao a ser resolvida 5x + 7y = 100, que possui soluo
pois 5 e 7 so primos entre si. Uma soluo particular dada por
x0 = 20 e y0 = 0. Logo a soluo geral da forma: x = 20 7t e
y = 5t, com t nmero natural e 20 7t 0 para que as solues
sejam no negativas. Assim obtemos as seguintes possibilidades:
x = 20, y = 0; x = 13, y = 5 e x = 6, y = 10.
3.59 Se D o nmero de degraus, temos D = 2x + 1 e D = 3y + 2.
Assim, temos que 2x 3y = 1, da qual uma soluo particular
x0 = 2 e y0 = 1. Portanto, x = 2 + 3t e y = 1 + 2t. Por outro lado,
40 D 100 e mltiplo de 7. Isto implica que 6 t 15, e para
que D seja mltiplo de 7, preciso que t = 12, ou seja, D = 77.
3.60 O problema conduz equao 15x 12y = 1, que no possui
solues, pois mdc(15, 12) = 3 no divide 1.
3.61 Temos n = 18x + 4 e n = 14y + 6, o que nos conduz equao

i
i

Aritmetica
2009/6/29
i page 125
Estilo OBMEP

i
i

125
9x 7y = 1. Uma soluo particular x0 = 3 e y0 = 4. Assim,
x = 3 + 7t, logo n = 18(3 + 7t) + 4, que natual quando t 1.
Logo os nmeros so da forma n = 126t 50, onde t 1.
Problemas do Captulo 4
4.3 J vimos que 10n 1 = 99 9, logo 9 divide 10n 1, donde
segue o resultado.
4.6 3 254 deixa resto 2 e 5, quando dividido por 3 e 9, respectivamente. 12 736 deixa resto 1, quando dividido por 3 e 9. 54 827 deixa
resto 2 e 8, quando dividido por 3 e 9, respectivamente. 33 875 435
deixa resto 2, quando dividido por 3 e 9. 57 612 510 deixa resto 0,
quando dividido por 3 e 9.
4.7 3 254 deixa resto 4 quando dividido por 5 e 10. 12 736 deixa resto
1 e 6, quando dividido por 5 e 10, respectivamente. 54 827 deixa resto
2 e 7, quando dividido por 5 e 10, respectivamente. 33 875 435 deixa
resto 0 e 5, quando dividido por 5 e 10, respectivamente. 57 612 510
deixa resto 0 quando dividido por 5 e 10.
4.12 1, 4 e 3.
4.15 O resto 5.
4.18 O resto 3.
4.19 O resto da diviso por 7 2. O resto da diviso por 11 9 e o
resto da diviso por 13 5.
4.21

Os restos da diviso por 3 de 5 345, 3 765 839 e 1010 so,

i
i

Aritmetica
2009/6/29
i page 126
Estilo OBMEP

i
i

126
respectivamente, 2, 2 e 1, logo 25 345 22 mod 7, 23 765 839 22 mod 7
10
e 210 2 mod 7.
4.22 Temos que 30 = 4 7 + 2, logo
230 = (24 )7 22 (1)7 4 3 mod 17.
Logo o resto da diviso 3.
4.23 Temos que 325 = 4 81 + 1, logo 2325 2 15 mod 17.
4.25 19 2 mod 17, logo 194n = (194 )2n (1)2n = 1 mod 17.
Assim, 194n 1 divisvel por 17.
4.26 Observe que se tem
133 = 2 197 37 8 mod 45,
e que
173 = 4 913 8 mod 45,
dos quais o resultado segue.
4.28 O resto 3.
4.30 O resto 2.

i
i

Aritmetica
2009/6/29
i page 127
Estilo OBMEP

i
i

Para Aprender Mais


COUTINHO, S. C. Nmeros Inteiros e Criptografia RSA. Rio de
Janeiro: IMPA, [s.d.]. (Srie de Computao e Matemtica.)
HEFEZ, A. Elementos de Aritmtica. [S.l.: s.n., s.d.] (Srie Textos
Universitrios, Sociedade Brasileira de Matemtica.)
HEFEZ, A. Induo Matemtica. Em http://www.obmep.org.br

127

i
i

contagemfran
2009/10/29
i page 1
Estilo OBMEP

i
i

Mtodos de Contagem
e Probabilidade
Paulo Cezar Pinto Carvalho

i
i

contagemfran
2009/10/29
i page 2
Estilo OBMEP

i
i

Texto j revisado pela nova ortografia.

i
i

contagemfran
2009/10/29
i page 3
Estilo OBMEP

i
i

Sobre o Autor
Paulo Cezar Pinto Carvalho formado em Engenharia pelo Instituto Militar de Engenharia, Mestre em Estatstica pelo IMPA e PhD
em Pesquisa Operacional pela Universidade Cornell. Atualmente
pesquisador do IMPA, na rea de Viso Computacional e Computao
Grfica. Divide o tempo devotado pesquisa com atividades ligadas
melhoria do ensino em todos os nveis. Desde 1991 professor do
Programa de Aperfeioamento de Professores, promovido pelo IMPA.
autor de diversos livros da Coleo do Professor de Matemtica,
publicada pela SBM. Tambm tem se dedicado s Olimpadas de
Matemtica, participando da organizao da Olimpada Brasileira de
Matemtica, desempenhando a funo de lder em vrias olimpadas
internacionais e, mais recentemente, servindo no Comit Executivo
da OBMEP.

i
i

contagemfran
2009/10/29
i page 4
Estilo OBMEP

i
i

contagemfran
2009/10/29
i page i
Estilo OBMEP

i
i

Antes de comear
Este livro dedicado a um tema que, paradoxalmente, extremamente simples, mas muitas vezes considerado difcil por alunos e
professores. Talvez isto se deva ao fato de que, diferentemente do
que ocorre com outros assuntos da matemtica secundria, cujo ensino muitas vezes fortemente baseado na aplicao de frmulas e
repetio de problemas-modelo, preciso pensar para resolver problemas, mesmo os mais simples, de contagem. Isto faz com que o
tema seja especialmente apropriado para este estgio, contribuindo
para desenvolver a imaginao dos alunos e a sua confiana para resolver problemas.
Durante muitos anos, o estudo de problemas de contagem (e mais
recentemente de probabilidade) fez parte, exclusivamente, do Ensino
Mdio. Entretanto, o tema perfeitamente acessvel aos alunos do
Ensino Fundamental, o que tem sido reconhecido, por exemplo, pelos
Parmetros Curriculares editados pelo MEC.
Esta apostila possui 5 captulos. O primeiro destinado a ambos
os grupos. desejvel que os alunos, principalmente os do grupo 1,
resolvam todos os problemas propostos e que os instrutores encorajem a exposio de solues diferentes e, principalmente, de solues
erradas. Como em outras reas da Matemtica, muitas vezes aprendemos mais com os erros do que com os acertos ao resolver problemas
de contagem. Os Captulos 2 e 3 contm essencialmente o mesmo
i
i

i
i

contagemfran
2009/10/29
i page ii
Estilo OBMEP

i
i

ii
material (uma introduo noo de probabilidade), escritos para
diferentes nveis de maturidade, sendo, em princpio, indicados para
os grupos 1 e 2, respectivamente. Os Captulos 4 e 5 foram escritos
com os alunos do grupo 2 em mente, mas tambm so acessveis aos
do grupo 1, caso haja tempo.
Solues para todos os problemas podem ser encontradas no final.
Mas claro que s devem ser consultadas aps uma tentativa sria
de resoluo dos problemas.
Gostaria de terminar com dois agradecimentos. O primeiro para
a Profa. Maria Lcia Villela, pela reviso extremamente benfeita do
material, tendo contribudo com diversas sugestes que foram incorporadas ao texto. Mas o agradecimento mais especial vai para o Prof.
Augusto Csar Morgado. Se os leitores acharem que estas notas so
parecidas com os seus escritos e suas aulas, isto no mera coincidncia. Tive a sorte de ter sido aluno do Prof. Morgado no 3o ano do
Ensino Mdio, quando tive ocasio de aprender sobre contagem do
modo exposto nesta apostila. At hoje continuo aprendendo com ele,
como colega e coautor. Espero que cada um de vocs, alunos, tenha
a oportunidade de ter um professor de Matemtica to inspirador
quanto ele.

O autor

i
i

contagemfran
2009/10/29
i page iii
Estilo OBMEP

i
i

Sumrio
1 Mtodos de Contagem

2 Probabilidade (grupo 1)

16

3 Probabilidade (grupo 2)

21

4 Mais Permutaes e Combinaes (grupo 2)

30

5 Probabilidade Condicional (grupo 2)

40

Exerccios Adicionais

47

Solues dos Exerccios

55

iii
i

i
i

contagemfran
2009/10/29
i page iv
Estilo OBMEP

i
i

contagemfran
2009/10/29
i page 1
Estilo OBMEP

i
i

Captulo 1

Mtodos de Contagem
Problemas de contagem so, muitas vezes, considerados difceis entre alunos e professores, apesar de as tcnicas matemticas necessrias
serem bastante elementares: essencialmente, o conhecimento das operaes aritmticas de soma, subtrao, multiplicao e diviso. O
objetivo deste material habituar o aluno a trabalhar com problemas de contagem e a ver que, afinal de contas, tais problemas podem
ser resolvidos com raciocnios simples na grande maioria dos casos,
sem exigir o uso de frmulas complicadas. isto o que procuramos
mostrar nos exemplos a seguir.
Exemplo 1. Uma bandeira com a forma abaixo vai ser pintada
utilizando duas das cores dadas.

1
i

i
i

contagemfran
2009/10/29
i page 2
Estilo OBMEP

i
i

CAP. 1: MTODOS DE CONTAGEM

(a) Liste todas as possveis bandeiras. Quantas so elas?


Soluo: importante ter um procedimento sistemtico para listar
todas as possveis bandeiras, sem repeti-las. Para tal, devemos identificar as diferentes decises a serem tomadas e examinar todas as
possibilidades para cada uma delas. No caso deste problema, uma
forma natural para planejar o preenchimento da bandeira :
escolher a cor a ser utilizada para a parte externa;
a seguir, escolher a cor para o crculo interno.
A primeira deciso pode ser feita de 3 modos diferentes, j que a cor
externa pode ser qualquer uma das disponveis. Uma vez tomada esta
deciso, a cor escolhida no pode mais ser usada para o crculo interno.
Por exemplo, se a cor preta for escolhida para a parte externa, a cor
interna dever ser cinza ou branca.
Podemos, ento, listar todas as possveis bandeiras, que so 6, de
acordo com a figura abaixo.

Com a cor externa preta:

Cor a cor externa cinza:

Cor a cor externa branca:

Um fato importante, que pode ser explorado na contagem eficiente do

i
i

contagemfran
2009/10/29
i page 3
Estilo OBMEP

i
i

3
nmero possvel de bandeiras, o seguinte: as cores disponveis para
pintar o crculo mudam de acordo com a escolha da parte externa,
mas a sua quantidade sempre a mesma, j que, qualquer que seja a
cor externa escolhida, h sempre duas cores restantes para o crculo.
Portanto, poderamos ter empregado o seguinte raciocnio para contar
o nmero de possveis bandeiras, sem list-las:
A cor externa pode ser escolhida de trs modos diferentes. Qualquer
que seja essa escolha, a cor do crculo pode ser escolhida de dois modos. Logo, o nmero total de possibilidades 2 + 2 + 2 = 3 2 = 6.
O procedimento acima ilustra o Princpio Multiplicativo ou Princpio
Fundamental da Contagem:
Se uma deciso D1 pode ser tomada de p modos e, qualquer que seja
essa escolha, a deciso D2 pode ser tomada de q modos, ento o
nmero de maneiras de se tomarem consecutivamente as decises D1
e D2 igual a pq.
O Princpio Multiplicativo pode ser ilustrado com o auxlio de uma
rvore de enumerao como a da figura a seguir.

Cor
externa

Cor do
crculo

i
i

contagemfran
2009/10/29
i page 4
Estilo OBMEP

i
i

CAP. 1: MTODOS DE CONTAGEM

(b) Quantas so as possveis bandeiras no caso em que 4 cores esto


disponveis?
Soluo: As decises a serem tomadas so exatamente as mesmas do
caso anterior, tendo mudado apenas o nmero de possibilidades de
escolha. Para a cor externa, temos agora 4 possibilidades. Uma vez
escolhida a cor externa, a cor do crculo pode ser qualquer uma das
outras 3. Logo, pelo Princpio Multiplicativo, o nmero de modos
diferentes para pintar a bandeira 4 3 = 12.

Exemplo 2. Quantas so as formas de pintar a bandeira a seguir


utilizando 3 cores diferentes dentre 4 dadas?

Soluo: Agora, temos 3 decises consecutivas a tomar: a cor externa,


a do retngulo e a do crculo. A cor externa pode ser qualquer uma
das 4 cores; uma vez escolhida a cor externa, o retngulo pode ser
pintado de trs modos distintos. Logo, a escolha combinada da cor
externa e do retngulo pode ser feita de 4 3 = 12 modos. Para
cada um destes 12 modos, o crculo pode ser pintado com uma das
duas cores que sobraram. Logo, o nmero total de possibilidades
4 3 2 = 24.

i
i

contagemfran
2009/10/29
i page 5
Estilo OBMEP

i
i

5
O raciocnio acima mostra que o Princpio Multiplicativo pode, na
realidade, ser aplicado quando temos diversas etapas de deciso: desde
que o nmero de possibilidades em cada etapa no dependa das decises anteriores, basta multiplic-los para achar o nmero total de
possibilidades.

Exemplo 3. Para pintar a bandeira abaixo, h 4 cores disponveis.


De quantos modos ela pode ser pintada de modo que faixas adjacentes
tenham cores distintas?

Soluo: O primeiro passo escolher em que ordem vamos pintar


a bandeira. Podemos, por exemplo, pintar as faixas de cima para
baixo (veja, no exerccio 16, o que ocorre quando escolhemos mal a
ordem de preenchimento). A cor da primeira faixa pode ser qualquer
uma das 4 cores. Qualquer que seja a cor escolhida, para a segunda
faixa temos 3 cores para escolher. Escolhida a cor da segunda faixa,
a terceira pode ser pintada de qualquer cor, exceto a usada para a
segunda faixa. Assim, temos novamente 3 possibilidades de escolha.

i
i

contagemfran
2009/10/29
i page 6
Estilo OBMEP

i
i

CAP. 1: MTODOS DE CONTAGEM

O nmero total de possibilidades , ento:


4

3
= 36

a
a
a
1 faixa
2 faixa
3 faixa

Exemplo 4. Quantos so os nmeros de trs algarismos distintos?


Soluo: Vamos escolher, sucessivamente, os trs algarismos,
comeando com o da esquerda (isto importante, como veremos
abaixo). O primeiro algarismo pode ser escolhido de 9 modos, pois
no pode ser igual a 0. O segundo algarismo pode ser escolhido de
9 modos, pois no pode ser igual ao primeiro algarismo. O terceiro
algarismo pode ser escolhido de 8 modos, pois no pode ser igual nem
ao primeiro nem ao segundo algarismo.
A resposta 9 9 8 = 648.
Exemplo 5. O cdigo Morse usa duas letras, ponto e trao, e as
palavras tm de 1 a 4 letras. Quantas so as palavras do cdigo
Morse?

Soluo: H palavras de 1, 2, 3 e 4 letras, em quantidades diferentes.


Assim, nossa estratgia a de usar o Princpio Multiplicativo para
contar separadamente estas palavras e, depois, somar estas quantidades. H 2 palavras de uma letra; h 2 2 = 4 palavras de duas
letras, pois h dois modos de escolher a primeira letra e dois modos
de escolher a segunda letra; analogamente, h 2 2 2 = 8 palavras
de trs letras e 2 2 2 2 = 16 palavras de 4 letras. O nmero

i
i

contagemfran
2009/10/29
i page 7
Estilo OBMEP

i
i

7
total de palavras 2 + 4 + 8 + 16 = 30.

Voc j deve ter percebido nesses exemplos qual a estratgia para


resolver problemas de contagem:
1. Postura: Devemos sempre nos colocar no papel da pessoa que
deve fazer a ao solicitada pelo problema e ver que decises devemos
tomar. Nas diversas situaes dos Exemplos 1 a 3, ns nos colocamos
no papel da pessoa que deveria colorir a bandeira; no Exemplo 4,
colocamo-nos no papel da pessoa que deveria escrever o nmero.
2. Diviso: Devemos, sempre que possvel, dividir as decises a
serem tomadas em decises mais simples, correspondentes s diversas
etapas do processo de deciso. Colorir a bandeira foi dividido em
colorir cada regio; formar um nmero de trs algarismos foi dividido
em escolher cada um dos trs algarismos. Formar a palavra no cdigo
Morse foi dividido em escolher o nmero de letras e, a seguir, em
escolher cada letra.
A ordem em que as decises so tomadas pode ser extremamente importante para a simplicidade do processo de resoluo. Vamos voltar
ao Exemplo 4 (Quantos so os nmeros de trs algarismos distintos?)
para ver como uma estratgia equivocada pode levar a uma soluo
desnecessariamente complicada.
Comeando a escolha dos algarismos pelo ltimo algarismo, h 10
modos de escolher o ltimo algarismo. Em seguida, h 9 modos de
escolher o algarismo central, pois no podemos repetir o algarismo j
usado. Agora temos um impasse: de quantos modos podemos escolher o primeiro algarismo? A resposta depende. Se no tivermos

i
i

contagemfran
2009/10/29
i page 8
Estilo OBMEP

i
i

CAP. 1: MTODOS DE CONTAGEM

usado o 0, haver 7 modos de escolher o primeiro algarismo, pois no


poderemos usar nem o 0 nem os dois algarismos j usados nas demais
casas; se j tivermos usado o 0, haver 8 modos de escolher o primeiro
algarismo.
Para evitar, na medida do possvel, impasses como o acima, uma outra
recomendao importante :

3. No adiar dificuldades. Pequenas dificuldades adiadas costumam se transformar em imensas dificuldades. Se uma das decises
a serem tomadas for mais restrita que as demais, essa a deciso
que deve ser tomada em primeiro lugar. No Exemplo 4, a escolha do
primeiro algarismo era uma deciso mais restrita do que as outras,
pois o primeiro algarismo no pode ser igual a 0. Essa , portanto, a
deciso que deve ser tomada em primeiro lugar, e, conforme acabamos
de ver, posterg-la s serve para causar problemas.

Exemplo 6. Quantos so os nmeros pares de trs algarismos distintos?


Soluo: H 5 modos de escolher o ltimo algarismo. Note que comeamos pelo ltimo algarismo, que o mais restrito; o ltimo algarismo
s pode ser 0, 2, 4, 6 ou 8.
Em seguida, vamos ao primeiro algarismo. De quantos modos se
pode escolher o primeiro algarismo? A resposta depende: se no
tivermos usado o 0, haver 8 modos de escolher o primeiro algarismo,
pois no poderemos usar nem o 0 nem o algarismo j usado na ltima
casa; se j tivermos usado o 0, haver 9 modos de escolher o primeiro
algarismo, pois apenas o 0 no poder ser usado na primeira casa.

i
i

contagemfran
2009/10/29
i page 9
Estilo OBMEP

i
i

9
Assim, apesar de termos procurado atacar inicialmente a escolha mais
restrita, chegamos a um impasse no uso do Princpio Multiplicativo.
Esse tipo de impasse comum na resoluo de problemas e h dois
mtodos para venc-lo.
O primeiro mtodo consiste em voltar atrs e contar separadamente.
Contaremos separadamente os nmeros que terminam em 0 e os que
no terminam em 0. Comecemos pelos que terminam em 0. H 1
modo de escolher o ltimo algarismo, 9 modos de escolher o primeiro
e 8 modos de escolher o algarismo central. H, portanto, 198 = 72
nmeros de trs algarismos distintos terminados em 0.
Para os que no terminam em 0, h 4 modos de escolher o ltimo
algarismo, 8 modos de escolher o primeiro e 8 modos de escolher o
algarismo central. H 488 = 256 nmeros pares de trs algarismos
distintos que no terminam em 0.
A resposta 72 + 256 = 328.
O segundo mtodo consiste em ignorar uma das restries do problema, o que nos far contar em demasia. Depois descontaremos o
que houver sido contado indevidamente.
Primeiramente fazemos de conta que o 0 pode ser usado na primeira
casa do nmero. Procedendo assim, h 5 modos de escolher o ltimo
algarismo (s pode ser 0, 2, 4, 6 ou 8), 9 modos de escolher o primeiro
algarismo (no podemos repetir o algarismo usado na ltima casa
note que estamos permitindo o uso do 0 na primeira casa) e 8 modos
de escolher o algarismo central. H 5 9 8 = 360 nmeros, a
inclusos os que comeam por 0.

i
i

contagemfran
2009/10/29
i page 10
Estilo OBMEP

i
i

10

CAP. 1: MTODOS DE CONTAGEM

Agora vamos determinar quantos desses nmeros comeam por zero;


so esses os nmeros que foram contados indevidamente. H 1 modo
de escolher o primeiro algarismo (tem que ser 0), 4 modos de escolher
o ltimo (s pode ser 2, 4, 6 ou 8 lembre-se de que os algarismos
so distintos) e 8 modos de escolher o algarismo central (no podemos
repetir os algarismos j usados). H 148 = 32 nmeros comeados
por 0.
A resposta 360 32 = 328.
Exemplo 7. De quantos modos diferentes 6 pessoas podem ser colocadas em fila?
Soluo: Este um problema clssico de contagem, chamado de problema das permutaes simples, que facilmente resolvido pelo Princpio Multiplicativo. De fato, basta escolher sucessivamente as pessoas
colocadas em cada posio da fila. Para escolher o primeiro da fila,
temos 6 possibilidades; o segundo pode ser qualquer uma das 5 pessoas
restantes, e assim por diante. Logo, o nmero total de possibilidades
6 5 4 3 2 1 = 720. De um modo geral, o nmero de modos
de ordenar n objetos igual a n (n 1) 1, que representado
por n! (l-se: n fatorial).
Exemplo 8. De quantos modos podem-se escolher trs dos jogadores
de um time de futebol para represent-lo em uma cerimnia de premiao?
Soluo: Este um outro problema clssico de contagem, chamado
de problema das combinaes simples. primeira vista, parece ser
simples resolv-lo pelo Princpio Multiplicativo: basta escolher um
representante de cada vez. O primeiro pode ser escolhido de 11 modos, o segundo, de 10 e o terceiro, de 9. Logo, o nmero total de

i
i

contagemfran
2009/10/29
i page 11
Estilo OBMEP

i
i

11
possibilidades parece ser 11 10 9 = 990. Esta soluo est incorreta, mas podemos consert-la para chegar resposta certa. Suponha
que tivssemos escolhido, sucessivamente, os jogadores A, B e C. A
comisso de representantes assim formada seria exatamente a mesma
se tivssemos selecionado, por exemplo, primeiro B, depois A, depois C. No entanto, as duas escolhas foram contadas por ns como
se fossem distintas. O que nos permite corrigir o resultado da contagem o fato de que todas as possveis comisses so repetidas o
mesmo nmero de vezes, correspondente a todas as suas possveis ordenaes. Por exemplo, A, B e C vo surgir, em nosso processo de
enumerao, 3 2 1 = 6 vezes, o mesmo ocorrendo com todas as
possveis comisses. Logo, o nmero correto de comisses igual a
990/6 = 165.
De modo geral, o nmero de modos de escolher p dentre n objetos
representado por Cnp (l-se: combinao de n tomados p a p) e igual
n(n 1) (n p + 1)
a
.
p(p 1) 1

Exerccios
1) Um grupo de 4 alunos (Alice, Bernardo, Carolina e Daniel) tem
que escolher um lder e um vice-lder para um debate.
(a) Faa uma lista de todas as possveis escolhas (use a inicial de cada nome, para facilitar). Organize a sua lista
do seguinte modo: primeiro, escreva todas as possibilidades em que Alice a presidente, depois, aquelas em que
Bernardo presidente, e assim por diante.
(b) Conte o nmero de possveis escolhas e verifique que o
Princpio Multiplicativo fornece a mesma resposta.

i
i

contagemfran
2009/10/29
i page 12
Estilo OBMEP

i
i

12

CAP. 1: MTODOS DE CONTAGEM

2) Um restaurante possui um cardpio que apresenta escolhas de


saladas (salada verde, salada russa ou salpico), sopas (caldo
verde, canja ou de legumes) e pratos principais (bife com fritas,
peixe com pur, frango com legumes ou lasanha).
(a) De quantos modos se pode escolher um prato deste cardpio?
(b) De quantos modos se pode escolher uma refeio completa,
formada por uma salada, uma sopa e um prato principal?
3) Quantos algarismos so escritos ao se escreverem os nmeros
inteiros de 1 a 100?
4 ) Joo e Isabel lanam, cada um, um dado.
(a) Quantas so as possveis combinaes de resultado?
(b) Quantas so as possveis somas que eles podem obter?
5) Cada dgito de uma calculadora mostrado no visor acendendo
filamentos dispostos como mostra a figura a seguir. Quantos
smbolos diferentes podem ser representados? (No inclua o
caso em que nenhum filamento aceso.)

6) Para pintar a bandeira abaixo esto disponveis as seis cores


dadas, sendo que regies adjacentes devem ser pintadas de cores
diferentes.
(a) Qual o nmero mnimo de cores a serem usadas?

i
i

contagemfran
2009/10/29
i page 13
Estilo OBMEP

i
i

13

(b) De quantos modos a bandeira pode ser pintada?


7) Dispomos de 5 cores distintas. De quantos modos podemos
colorir os quatro quadrantes de um crculo, cada quadrante com
uma s cor, se quadrantes cuja fronteira uma linha no podem
receber a mesma cor?
8) Quantos so os gabaritos possveis de um teste de 10 questes de
mltipla escolha, com 5 alternativas por questo? Em quantos
destes gabaritos a letra A aparece exatamente uma vez? Em
quantos a letra A no aparece?
9) Liste todos os subconjuntos de {1, 2, 3}. Quantos so eles? De
modo geral, quantos so os subconjuntos de um conjunto que
tem n elementos?
10) De quantos modos 3 pessoas podem se sentar em 5 cadeiras em
fila?
11) De quantos modos 5 homens e 5 mulheres podem se sentar em
5 bancos de 2 lugares, se em cada banco deve haver um homem
e uma mulher?
12) De quantos modos podemos colocar 2 reis diferentes em casas
no adjacentes de um tabuleiro 88? E se os reis fossem iguais?
13) De quantos modos podemos formar uma palavra de 5 letras de
um alfabeto de 26 letras, se a letra A deve figurar na palavra

i
i

contagemfran
2009/10/29
i page 14
Estilo OBMEP

i
i

14

CAP. 1: MTODOS DE CONTAGEM

mas no pode ser a primeira letra da palavra? E se a palavra


devesse ter letras distintas?
14) As placas dos veculos so formadas por trs letras (de um alfabeto de 26) seguidas por 4 algarismos. Quantas placas podero
ser formadas?
15) Um vago do metr tem 10 bancos individuais, sendo 5 de frente
e 5 de costas. De 10 passageiros, 4 preferem se sentar de frente,
3 preferem se sentar de costas, e os demais no tm preferncia.
De quantos modos eles podem se sentar, respeitadas as preferncias?
16) Escrevem-se os inteiros de 1 at 2 222.
(a) Quantas vezes o algarismo 0 escrito?
(b) Em quantos nmeros aparece o algarismo 0?
17) Quantos so os inteiros positivos de 4 algarismos nos quais o
algarismo 5 figura?
18) Em uma banca h 5 exemplares iguais da Veja, 6 exemplares
iguais da poca e 4 exemplares iguais da Isto . Quantas
colees no vazias de revistas dessa banca podem ser formadas?
19) Tendo 4 cores disponveis, de quantos modos se pode pintar uma
bandeira com 3 listras, tendo listras adjacentes de cores distintas? Um aluno deu a seguinte soluo: Primeiro, eu vou pintar
as listras extremas; para cada uma, eu tenho 4 possibilidades
de escolha. Depois, eu pinto a listra central; como ela tem que
ter cor diferente das duas vizinhas, eu posso escolher sua cor de
apenas 2 modos. Logo, o nmero total de modos de pintar a
bandeira 4 4 2 = 32. A soluo est certa ou errada? Se
estiver errada, onde est o erro?

i
i

contagemfran
2009/10/29
i page 15
Estilo OBMEP

i
i

15
20) Com 5 homens e 5 mulheres, de quantos modos se pode formar um casal? Este problema foi resolvido por um aluno do
modo a seguir: A primeira pessoa do casal pode ser escolhida
de 10 modos, pois ela pode ser homem ou mulher. Escolhida a
primeira pessoa, a segunda pessoa s poder ser escolhida de 5
modos, pois deve ser de sexo diferente do da primeira pessoa.
H, portanto, 10 5 = 50 modos de formar um casal.
A soluo est certa ou errada? Se estiver errada, onde est o
erro?
21) Cada pea de um domin apresenta um par de nmeros de 0 a
6, no necessariamente distintos. Quantas so essas peas? E
se os nmeros forem de 0 a 8?
22) Quantos retngulos h formados por casas adjacentes em um
tabuleiro de xadrez 8 8? Por exemplo, em um tabuleiro 2 2
h 9 retngulos, como mostra a figura abaixo.

7
5

9
1
8

i
i

contagemfran
2009/10/29
i page 16
Estilo OBMEP

i
i

Captulo 2

Probabilidade (grupo 1)
Uma das principais aplicaes das tcnicas de contagem a resoluo de problemas simples de Probabilidade. O interesse dos
matemticos no estudo sistemtico de probabilidades relativamente
recente e tem suas razes no estudo dos jogos de azar.
No estudo desses jogos, normalmente ocorre a seguinte situao:
todos os possveis resultados tm a mesma chance de ocorrer. Por
exemplo, ao lanar um dado honesto (quer dizer, construdo de
forma perfeitamente cbica e homognea), todas as faces tm a mesma
chance de sair. Como as faces so 6, esperamos que cada uma delas
ocorra em aproximadamente 1/6 dos lanamentos. Dizemos, ento,
que cada uma delas tem probabilidade 1/6 de sair.
Tambm atribumos probabilidades a conjuntos de resultados possveis, chamados de eventos. A probabilidade de um evento simplesmente a soma das probabilidades dos resultados que o compem.
Exemplo 1. Qual a probabilidade de se obter um resultado maior
que 4 ao se lanar um dado honesto?
Soluo: Dizer que sai resultado maior do que 4 equivalente a dizer
16
i

i
i

contagemfran
2009/10/29
i page 17
Estilo OBMEP

i
i

17
1
que sai 5 ou 6. Como cada uma destas faces tm probabilidade de
6
ocorrer, a probabilidade de sair um nmero maior do que 4 igual a
1 1
2
1
+ = = .
6 6
6
3
De um modo geral, quando todos os resultados tm a mesma
chance de ocorrer, a probabilidade de um evento a razo entre o
nmero de resultados relativos ao evento e o nmero total de resultados. Em outras palavras, a razo entre o nmero de casos favorveis
ocorrncia do evento e o nmero total de casos.
Exemplo 2. Ao lanar um dado duas vezes, qual a probabilidade
de se obter soma 5?
Soluo: Como em cada lanamento h 6 possibilidades, o nmero
de casos possveis 6 6 = 36, todos com a mesma probabilidade de
ocorrncia. Destes, aqueles em que a soma 5 so (1, 4), (2, 3), (3, 2)
e (4, 1). Logo, o nmero de casos favorveis ao evento 4, e sua
probabilidade 4/36 = 1/9.
Exemplo 3. Em uma urna h 5 bolas vermelhas e 4 pretas, todas de
mesmo tamanho e feitas do mesmo material. Retiramos duas bolas
sucessivamente da urna, sem rep-las. Qual a probabilidade de que
sejam retiradas duas bolas vermelhas?
Soluo: Precisamos, antes de mais nada, identificar quais so os possveis resultados. Como tudo o que observamos a cor de cada bola retirada (as bolas de mesma cor so indistinguveis entre si), poderamos
ser tentados a dizer que temos apenas 4 casos: vv, vp, pv, pp. O problema que estes casos no tm a mesma chance de ocorrer ( bvio,
por exemplo, que duas bolas vermelhas saem com mais frequncia que
duas bolas pretas, j que h mais bolas vermelhas). A soluo con-

i
i

contagemfran
2009/10/29
i page 18
Estilo OBMEP

i
i

18

CAP. 2: PROBABILIDADE (GRUPO 1)

siste em considerar individualmente as 9 bolas presentes na urna. Ou


seja, os resultados possveis so todos os pares de bolas distintas, cuja
quantidade 9 8 = 72. Como todas as bolas so iguais (a menos
da cor), todos estes pares tm a mesma probabilidade de sair. Para
calcular o nmero destes pares em que ambas as bolas so vermelhas,
devemos observar que a primeira bola vermelha pode ser escolhida de
5 modos, enquanto a segunda pode ser qualquer uma das 4 restantes.
Logo, o nmero de casos favorveis igual a 5 4 = 20. Portanto, a
probabilidade de que sejam retiradas duas bolas vermelhas igual a
20/72 = 5/18.
Exemplo 4. Pedro e Joo combinaram de lanar uma moeda 4 vezes.
Pedro apostou que, nesses 4 lanamentos, no apareceriam 2 caras seguidas; Joo aceitou a aposta. Quem tem maior chance de ganhar a
aposta?
Soluo: Vamos considerar todas as sequncias possveis de resultados. Como em cada lanamento sai cara (C) ou coroa (K), h
2 possibilidades; logo, o nmero total de possibilidades igual a
2 2 2 2 = 16. Todas essas sequncias tm a mesma probabilidade de ocorrncia, j que o resultado de um lanamento no afeta
os demais e h a mesma chance de sair cara ou coroa. Vamos agora
verificar quais dessas sequncias levam vitria de Pedro.
Se s sarem coroas (KKKK), claro que Pedro vence.
Se s sair uma cara (CKKK, KCKK, KKCK, KKKC), Pedro tambm vence.
Com duas caras, Pedro vence nos casos KCKC, CKCK e
CKKC.
Quando saem trs ou mais caras, Pedro perde.

i
i

contagemfran
2009/10/29
i page 19
Estilo OBMEP

i
i

19
Logo, o nmero de sequncias favorveis a Pedro igual a 8, e sua
probabilidade de vitria igual a 8/16 = 1/2. Portanto, Pedro e Joo
tm a mesma chance de vitria.

Exerccios
1) Dois dados so lanados e observa-se a soma de suas faces.
(a) Quais so os possveis resultados para esta soma?
(b) Esses resultados so equiprovveis? Caso contrrio, que
resultado mais provvel? Com que probabilidade? E o
menos provvel?
(c) Qual a probabilidade de cada resultado possvel?
2) Uma moeda lanada 3 vezes. Qual a probabilidade de que
saiam 2 caras?
3) Um casal decidiu que vai ter 4 filhos. O que mais provvel:
que tenham dois casais ou trs filhos de um sexo e um de outro?
4) Laura e Telma retiram um bilhete cada de uma urna em que h
100 bilhetes numerados de 1 a 100. Qual a probabilidade de
que o nmero retirado por Laura seja maior do que o de Telma?
E se elas, depois de consultarem o nmero, devolvem o bilhete
urna?
5) Duas peas de um domin comum so sorteadas. Qual a
probabilidade de que tenham um nmero em comum?
6) Ana, Joana e Carolina apostam em um jogo de cara-e-coroa.
Ana vence na primeira vez que sarem duas caras seguidas;
Joana vence na primeira vez que sarem duas coroas seguidas;
Carolina vence quando sair uma cara seguida de uma coroa.
Qual a probabilidade que cada uma tem de vencer?

i
i

contagemfran
2009/10/29
i page 20
Estilo OBMEP

i
i

20

CAP. 2: PROBABILIDADE (GRUPO 1)

7) O trecho a seguir foi obtido em um site de internet que se prope


a aumentar as chances de vitria no jogo da Sena (que consiste
em sortear 6 dentre 60 dezenas). Quando afirmamos, por exemplo, que as dezenas atrasadas so importantes, porque j observamos, em nossos estudos, que todas as dezenas so sorteadas
a cada quarenta testes, portanto, seria til voc acompanhar e
apostar em dezenas atrasadas; voc estaria assim aumentando
muito suas chances. Voc concorda que apostar em uma dezena
atrasada aumenta as chances de vitria na Sena?
8) Suponhamos que voc tenha duas escolhas para apostar na Sena.
Na primeira escolha aposta nas dezenas 1 3 5 7 9 11,
e na segunda escolha nas dezenas 8 17 31 45 49 55.
Qual voc acha que tem maiores chances de ser vitoriosa?
9) (O Problema do Bode) Este problema foi proposto em um programa de rdio nos Estados Unidos e causou um enorme debate
na internet.
Em um programa de prmios, o candidato tem diante de si trs
portas. Atrs de uma dessas portas, h um grande prmio; atrs
das demais h um bode. O candidato escolhe inicialmente uma
das portas. O apresentador (que sabe qual a porta que contm
o prmio) abre uma das portas no indicadas pelo candidato,
mostrando necessariamente um bode. A seguir, ele pergunta se
o candidato mantm sua escolha ou deseja trocar de porta. O
candidato deve trocar ou no? (Uma forma de voc guiar sua
intuio consiste em simular o problema.)

i
i

contagemfran
2009/10/29
i page 21
Estilo OBMEP

i
i

Captulo 3

Probabilidade (grupo 2)
Uma das principais aplicaes das tcnicas de contagem a resoluo de problemas simples de Probabilidade. O interesse dos
matemticos no estudo sistemtico de probabilidades relativamente
recente e tem suas razes no estudo dos jogos de azar. Um problema
clssico, que tem origem em autores do sculo XV e que despertou o
interesse de autores como Pascal e Fermat, o
Problema dos pontos: Dois jogadores apostaram R$ 10,00 cada um
em um jogo de cara-e-coroa, combinando que o primeiro a conseguir 6
vitrias ficaria com o dinheiro da aposta. O jogo, no entanto, precisa
ser interrompido quando um dos jogadores tem 5 vitrias e o outro
tem 3. Qual a diviso justa da quantia apostada?
(Para um clsssico moderno, veja o exerccio 9, que provocou grande
discusso na internet alguns anos atrs). Parece razovel que a quantia apostada seja dividida de forma proporcional chance (ou probabilidade) de vitria de cada jogador. O clculo dessas probabilidades
se baseia, como veremos mais adiante, na hiptese de que a moeda

21
i

i
i

contagemfran
2009/10/29
i page 22
Estilo OBMEP

i
i

22

CAP. 3: PROBABILIDADE (GRUPO 2)

seja honesta, ou seja, de que haja iguais chances, em um lanamento,


de sair cara ou coroa. Esta crena, por sua vez, corresponde seguinte
ideia intuitiva: em uma sequncia longa de lanamentos, esperamos
observar, aproximadamente, o mesmo nmero de caras e coroas.
De modo mais geral, suponhamos que um determinado experimento
tenha n resultados possveis 1 , 2 , . . . , n ; o conjunto desses possveis resultados chamado de espao amostral. Suponhamos, ainda,
que julguemos que, ao repetir o experimento um grande nmero de
vezes, esperemos que o resultado i ocorra em uma certa frao pi
das realizaes do experimento. Dizemos, ento, que a probabilidade
de se observar i igual a pi . Evidentemente, devemos ter pi 0
para cada i e, alm disso, p1 + + pn = 1. Uma vez estabelecidos os
valores para as probabilidades de cada resultado possvel, podemos
definir a probabilidade de qualquer evento A (ou seja, de qualquer
subconjunto de ) como a soma das probabilidades dos resultados
em A.
Mas como encontrar os valores das probabilidades pi ? No caso geral,
esses valores so obtidos de forma experimental. Mas h certos casos em que razovel supor que todos os resultados so igualmente
provveis e que, portanto, a probabilidade de cada um deles igual
a 1/n. Por exemplo, ao lanar um dado perfeitamente cbico no h
nenhuma razo para esperar que uma face aparea com mais frequncia que qualquer das outras. Logo, a probabilidade associada a cada
face igual a 1/6. Modelos probabilsticos que tm esta caracterstica so chamados de equiprovveis e esto frequentemente associados
a jogos de azar. Nos modelos probabilsticos equiprovveis, a proba1
p
bilidade associada a um evento A com p elementos igual a p = .
n
n
Muitas vezes se exprime este fato dizendo que a probabilidade de um
evento igual razo entre o nmero de casos favorveis ao evento

i
i

contagemfran
2009/10/29
i page 23
Estilo OBMEP

i
i

23
e o nmero de casos possveis.
Exemplo 1. Qual a probabilidade de se obter um resultado maior
que 4 ao se lanar um dado honesto?
Soluo: O espao amostral = {1, 2, 3, 4, 5, 6}, com todos os resultados tendo probabilidade 1/6. Desejamos calcular a probabilidade
1
1
do evento A = {5, 6}, que dada por P (A) = 2 = .
6
3
Exemplo 2. Ao lanar um dado duas vezes, qual a probabilidade
de se obter soma 5?
Soluo: O espao amostral formado por todos os pares de resultados possveis. Como em cada lanamento h 6 possibilidades, o
nmero de casos possveis 6 6 = 36, todos com a mesma probabilidade de ocorrncia. Destes, aqueles em que a soma 5 so
(1, 4), (2, 3), (3, 2) e (4, 1). Logo, o nmero de casos favorveis ao
evento 4, e sua probabilidade 4/36 = 1/9.
Exemplo 3. Em uma urna h 5 bolas vermelhas e 4 pretas, todas de
mesmo tamanho e feitas do mesmo material. Retiramos duas bolas
sucessivamente da urna, sem rep-las. Qual a probabilidade de que
sejam retiradas duas bolas vermelhas?
Soluo: Precisamos, antes de mais nada, encontrar um espao
amostral apropriado para descrever os resultados dos experimentos.
Como tudo o que observamos a cor de cada bola retirada (as bolas
de mesma cor so indistinguveis entre si), poderamos ser tentados
a escolher o espao amostral {vv, vp, pv, pp}, formado pelos pares de
cores observadas. Essa escolha no est errada, mas no conveniente
para a soluo do problema. O que ocorre que o modelo probabilstico baseado nesse espao amostral no equiprovvel ( bvio, por
exemplo, que duas bolas vermelhas saiam com mais frequncia que

i
i

contagemfran
2009/10/29
i page 24
Estilo OBMEP

i
i

24

CAP. 3: PROBABILIDADE (GRUPO 2)

duas bolas pretas, j que h mais bolas vermelhas). Para obter um


espao equiprovvel, devemos considerar individualmente as 9 bolas
presentes na urna. Ou seja, o espao amostral o conjunto de todos
os pares de bolas distintas, que tem 98 = 72 elementos. Como todas
as bolas so iguais (ao menos na cor), todos esses pares tm a mesma
probabilidade de sair. Para calcular o nmero desses pares em que
ambas as bolas so vermelhas, devemos observar que a primeira bola
vermelha pode ser escolhida de 5 modos, enquanto a segunda pode
ser qualquer uma das 4 restantes. Logo, o nmero de casos favorveis
igual a 5 4 = 20. Portanto, a probabilidade de que sejam retiradas
duas bolas vermelhas igual a 20/72 = 5/18.
Exemplo 4. Pedro e Joo combinaram de lanar uma moeda 4 vezes.
Pedro apostou que, nesses 4 lanamentos, no apareceriam 2 caras seguidas; Joo aceitou a aposta. Quem tem maior chance de ganhar a
aposta?
Soluo: O espao amostral apropriado formado por todas as sequncias possveis de resultados. Como em cada lanamento sai cara
(C) ou coroa (K), h 2 possibilidades; logo, o nmero total de possibilidades igual a 2 2 2 2 = 16. Todas essas sequncias tm a
mesma probabilidade de ocorrncia, j que o resultado de um lanamento no afeta os demais e h a mesma chance de sair cara ou coroa.
Vamos verificar quais dessas sequncias levam vitria de Pedro.
Se s sarem coroas (KKKK), claro que Pedro vence.
Se s sair uma cara (CKKK, KCKK, KKCK, KKKC), Pedro tambm vence.
Com duas caras, Pedro vence nos casos KCKC, CKCK e
CKKC.
Quando saem trs ou mais caras, Pedro perde.

i
i

contagemfran
2009/10/29
i page 25
Estilo OBMEP

i
i

25
Logo, o nmero de sequncias favorveis a Pedro igual a 8, e sua
probabilidade de vitria igual a 8/16 = 1/2. Portanto, Pedro e Joo
tm a mesma chance de vitria.
Exemplo 5. Qual a forma justa de dividir os R$ 20,00 apostados
no problema dos pontos?
Soluo: O jogador I tem 5 vitrias, faltando apenas uma para
vencer o jogo. O jogador II tem apenas 3 vitrias, necessitando
de mais 3 para vencer. Portanto, para que II vena, ele tem que
vencer trs partidas seguidas. H 2 2 2 = 8 possibilidades
para os resultados dessas partidas, e apenas um destes favorvel
vitria de II. Logo, II vence com probabilidade 1/8, enquanto
a probabilidade de vitria de I 7/8. Logo, I deve ficar com
R$ 17,50 e II com R$ 2,50.
Uma possvel objeo quanto soluo acima o fato de construirmos
nosso espao amostral com base nas trs partidas restantes, quando
o jogo pode, na verdade, terminar em uma, duas ou trs partidas.
Fizemos isto para obter um espao amostral para o qual o modelo
equiprovvel. Note que usar esse espao amostral equivalente a supor que, mesmo que I tenha vencido na primeira ou segunda partida,
eles continuam a disputar, como amistosos, as partidas seguintes.
claro que isso no modifica em nada as chances de vitria de cada
jogador.
Vimos acima que a ideia intuitiva de probabilidade de um evento est
ligada frequncia observada desse evento quando o experimento
realizado um grande nmero de vezes. Essa relao pode ser estabelecida de modo preciso, atravs de um teorema conhecido como a
Lei dos Grandes Nmeros. Embora, por vezes, ela no seja muito
bem entendida (veja, por exemplo, o exerccio 7), a Lei dos Grandes

i
i

contagemfran
2009/10/29
i page 26
Estilo OBMEP

i
i

26

CAP. 3: PROBABILIDADE (GRUPO 2)

Nmeros um instrumento fundamental para estabelecer uma via de


mo dupla entre modelos probabilsticos tericos e os experimentos
aleatrios.
Consideremos, novamente, o exemplo 5. Uma forma de se ter uma
ideia da resposta do problema seria utilizar uma simulao da situao
pretendida. Essa simulao repetida um grande nmero de vezes e,
atravs da frequncia de vitrias de cada jogador, estimaramos sua
probabilidade de vitria. A simulao pode ser feita manualmente,
usando uma moeda ( uma atividade apropriada para sala de aula:
cada aluno repete o experimento algumas poucas vezes e, reunindo
todos os resultados, temos uma quantidade razovel de repeties).
possvel, tambm, fazer a simulao com auxlio de um computador,
atravs da gerao de nmeros aleatrios. A tabela abaixo mostra o
resultado obtido simulando 100 realizaes do jogo.
I ganha na primeira partida

52 vezes

I ganha na segunda partida

20 vezes

I ganha na terceira partida

13 vezes

II ganha (na terceira partida)

15 vezes

Os resultados obtidos mostram, ao mesmo tempo, o poder e a limitao do mtodo de simulao. Por um lado, permite estimar que II
tem uma chance de vitria muito menor do que a de I. Na simulao que fizemos, II ganhou em apenas 15% das vezes (o que est
razoavelmente prximo da probabilidade exata, que 1/8 = 0,125).
Por outro lado, o valor obtido na simulao sempre uma aproximao, cujo erro diminui com o nmero de repeties.

i
i

contagemfran
2009/10/29
i page 27
Estilo OBMEP

i
i

27

Exerccios
1) Dois dados so lanados e observa-se a soma de suas faces.
(a) Quais so os possveis resultados para essa soma?
(b) Esses resultados so equiprovveis? Caso contrrio, que
resultado mais provvel? Com que probabilidade? E o
menos provvel?
(c) Qual a probabilidade de cada resultado possvel?
2) Uma moeda lanada 3 vezes. Qual a probabilidade de que
saiam 2 caras?
3) Um casal decidiu que vai ter 4 filhos. O que mais provvel:
que tenham dois casais ou trs filhos de um sexo e um de outro?
4) Laura e Telma retiram um bilhete cada de uma urna em que h
100 bilhetes numerados de 1 a 100. Qual a probabilidade de
que o nmero retirado por Laura seja maior do que o de Telma?
E se elas, depois de consultarem o nmero, devolvem o bilhete
urna?
5) Duas peas de um domin comum so sorteadas. Qual a
probabilidade de que tenham um nmero em comum?
6) Ana, Joana e Carolina apostam em um jogo de cara-e-coroa.
Ana vence na primeira vez que sarem duas caras seguidas;
Joana vence na primeira vez que sarem duas coroas seguidas;
Carolina vence quando sair uma cara seguida de uma coroa.
Qual a probabilidade que cada uma tem de vencer?
7) O trecho a seguir foi obtido em um site de internet que se prope
a aumentar as chances de vitria no jogo da Sena (que consiste

i
i

contagemfran
2009/10/29
i page 28
Estilo OBMEP

i
i

28

CAP. 3: PROBABILIDADE (GRUPO 2)

em sortear 6 dentre 60 dezenas). Quando afirmamos, por exemplo, que as dezenas atrasadas so importantes, porque j observamos, em nossos estudos, que todas as dezenas so sorteadas
a cada quarenta testes, portanto, seria til voc acompanhar e
apostar em dezenas atrasadas; voc estaria assim aumentando
muito suas chances. Voc concorda que apostar em uma dezena
atrasada aumenta as chances de vitria na Sena?
8) Suponhamos que voc tenha duas escolhas para apostar na Sena.
Na primeira escolha aposta nas dezenas 1 3 5 7 9 11,
e na segunda escolha nas dezenas 8 17 31 45 49 55.
Qual voc acha que tem maiores chances de ser vitoriosa?
9) (O Problema do Bode) Este problema foi proposto em um programa de rdio nos Estados Unidos e causou um enorme debate
na internet.
Em um programa de prmios, o candidato tem diante de si trs
portas. Atrs de uma dessas portas, h um grande prmio; atrs
das demais h um bode. O candidato escolhe inicialmente uma
das portas. O apresentador (que sabe qual a porta que contm
o prmio) abre uma das portas no indicadas pelo candidato,
mostrando necessariamente um bode. A seguir, ele pergunta se
o candidato mantm sua escolha ou deseja trocar de porta. O
candidato deve trocar ou no? (Uma forma de voc guiar sua
intuio consiste em simular o problema.)
10) Suponha que 16 selees, entre as quais Brasil e Argentina, vo
participar de um torneio. Sero formados quatro grupos de
quatro selees, atravs de sorteio. Qual a probabilidade de
que Brasil e Argentina fiquem no mesmo grupo?
11) A China tem um srio problema de controle de populao.
Vrias polticas foram propostas (e algumas colocadas em efeito)

i
i

contagemfran
2009/10/29
i page 29
Estilo OBMEP

i
i

29
visando proibir as famlias de terem mais de um filho. Algumas dessas polticas, no entanto, tiveram consequncias trgicas. Por exemplo, muitas famlias de camponeses abandonaram
suas filhas recm-nascidas, para terem uma outra chance de ter
um filho do sexo masculino. Por essa razo, leis menos restritivas foram consideradas. Uma das leis propostas foi a de que as
famlias teriam o direito a um segundo (e ltimo) filho, caso o
primeiro fosse do sexo feminino. Deseja-se saber que consequncias isso traria para a composio da populao, a longo prazo.
Haveria uma maior proporo de mulheres? De homens?
(a) Com auxlio de uma moeda, simule a prole de um conjunto de 10 famlias (jogue a moeda; se obtiver cara,
um menino, e a famlia para por a; se der coroa, uma
menina; jogue a moeda mais uma vez e veja se o segundo
filho menino ou menina).
(b) Rena os resultados obtidos pelos integrantes do grupo e
produza estatsticas mostrando o nmero mdio de crianas por famlia, a proporo de meninos e meninas na populao e a proporo de famlias que tm um filho homem.
O que esses resultados sugerem?
(c) Qual a probabilidade de que uma famlia tenha um filho
do sexo masculino? Qual o nmero mdio de filhos por
famlia? Dentre todas as crianas nascidas, qual a proporo de meninos e meninas?

i
i

contagemfran
2009/10/29
i page 30
Estilo OBMEP

i
i

Captulo 4

Mais Permutaes e
Combinaes (grupo 2)
Como vimos anteriormente, possvel resolver um grande nmero
de problemas interessantes de contagem sem utilizar frmulas, apenas
empregando apropriadamente as quatro operaes. H, no entanto,
certos problemas que ocorrem com frequncia e que no so imediatos,
como o problema das combinaes simples, para os quais interessante conhecer a frmula que expressa sua soluo, para empreg-la
em outros problemas. Neste material adicional, veremos alguns problemas que utilizam permutaes e combinaes em sua soluo e
travaremos contato com algumas outras frmulas combinatrias que
podem ser teis.
Exemplo 1. De quantos modos 4 crianas podem formar uma roda?
Soluo: primeira vista, pode parecer que para formar uma roda
com as 4 crianas basta escolher uma ordem para elas, o que pode
ser feito de 4! = 24 modos. Entretanto, as rodas ABCD, BCDA,
CDAB e DABC mostradas na figura abaixo so iguais, j que cada
30
i

i
i

contagemfran
2009/10/29
i page 31
Estilo OBMEP

i
i

31
uma resulta da anterior por uma virada de 1/4 de volta.
B
A

C
C

D
D

A
A

B
C

Para calcular o nmero de maneiras possveis de formar uma roda,


podemos raciocinar de dois modos diferentes. Um deles consiste em
partir do resultado anterior (4! = 24) e perceber que cada roda est
sendo contada 4 vezes. Logo, o nmero correto de rodas que podem
ser formadas 24
4 = 6. Alternativamente, podemos comear por fixar
a criana A na posio esquerda (j que em qualquer roda A pode
ficar nesta posio). Agora, temos 3 lugares para as 3 crianas que
restaram, para um total de 3! = 6 possibilidades.
De modo geral, o nmero de modos de colocar n objetos em
crculo, considerando-se iguais disposies que coincidam por rotao (ou seja, o nmero de permutaes circulares de n objetos)
P Cn = (n 1)!.
Exemplo 2. Considere um grupo formado por 7 homens (entre os
quais Jos) e 5 mulheres (entre as quais Maria), do qual se quer extrair
uma comisso constituda por 4 pessoas. Quantas so as comisses:
(a) Possveis?
Soluo: Devemos escolher 4 das 12 pessoas, o que pode ser feito de
4 modos, que igual a 1211109 = 495 comisses.
C12
1234
(b) Formadas por 2 homens e 2 mulheres?

i
i

contagemfran
2009/10/29
i page 32
Estilo OBMEP

i
i

32

CAP. 4: MAIS PERMUTAES E COMBINAES (GRUPO 2)

Soluo: Para formar uma comisso, devemos escolher os 2 homens,


o que pode ser feito de C72 modos, e, a seguir, as 2 mulheres, o que
pode ser feito de C52 maneiras. O nmero total de possibilidades de
escolha, pelo princpio multiplicativo, C72 C52 = 21 10 = 210
comisses.
(c) Em que haja pelo menos 2 mulheres?
Soluo: H 3 tipos de comisso possveis: com 2 homens e 2 mulheres, com 1 homem e 2 mulheres e com 4 mulheres. Para obter o
nmero total de comisses, contamos separadamente as comisses de
cada tipo e somamos os resultados, obtendo
C72 C52 + C71 C53 + C54 = 210 + 70 + 5 = 285 comisses.
Uma tentativa de contagem que leva a um erro muito comum a
seguinte: como a comisso deve ter pelo menos 2 mulheres, inicialmente escolhemos 2 mulheres, o que podemos fazer de C52 = 10 modos.
A seguir, basta escolher 2 pessoas quaisquer entre as 10 que sobraram,
2 = 45 modos. Logo, por este raciocnio,
o que pode ser feito de C10
teramos 10 45 = 450, que difere do resultado (correto) encontrado
acima. Essa soluo, portanto, est errada. Voc sabe explicar onde
est o erro no raciocnio?
(d) Em que Jos participe, mas Maria no?
Soluo: Como Jos deve participar da comisso, resta escolher apenas 3 outras pessoas, entre as 10 restantes (j que Jos j foi escolhido
e Maria no pode ser escolhida). Logo, o nmero de possibilidades
3 = 120.
igual a C10
(e) Formadas por 2 homens, entre os quais Jos, e 2 mulheres, mas
sem incluir Maria?

i
i

contagemfran
2009/10/29
i page 33
Estilo OBMEP

i
i

33
Soluo: Temos que escolher 1 homem entre 6 (Jos j est escolhido)
e 2 mulheres entre 4 (Maria no pode ser escolhida). O nmero de
comisses 6 C42 = 6 6 = 36.
Exemplo 3. Quantos anagramas podemos formar com a palavra
MATEMATICA?
Soluo: Um anagrama uma palavra (no necessariamente fazendo
sentido) formada com as mesmas letras, mas em uma ordem qualquer.
Quando as n letras de uma palavra so todas distintas, o nmero de
anagramas igual ao nmero de permutaes de n, que, como vimos,
igual a n!. Mas a palavra MATEMATICA tem letras repetidas: h
3 A, 2 M e 2 T, alm de E, I e C, que aparecem uma vez cada.
Uma soluo (consistente com o princpio de atacar o mais complicado antes) , antes de mais nada, decidir o que fazemos com as
letras repetidas. Para colocar os A, temos que escolher 3 dentre os 10
3 modos. Para colocar os
lugares possveis, o que pode ser feito de C10
M, restam agora 7 lugares, dos quais devemos escolher 2, o que pode
ser feito de C72 maneiras. Agora s restam 5 lugares, dos quais devemos escolher 2 para colocar os T; temos C52 possibilidades. Agora, s
restam 3 lugares, nos quais devem ser colocadas as 3 letras restantes,
o que pode ser feito de 3 2 1 modos. Logo, o nmero total de
3 C 2 C 2 6 = 151 200.
anagramas C10
7 5
Mas h um outro modo de pensar, partindo do nmero de
permutaes de 10 letras distintas (igual a 10!). Esta contagem
no est correta, porque consideramos letras iguais como se fossem distintas. Ou seja, como se considerssemos as permutaes
de A1 , A2 , A3 , M1 , M2 , T1 , T2 , E, I e C. Para corrigir a contagem,
basta contar quantas vezes cada anagrama foi contado. Por exemplo, o anagrama AAAM M T T EIC foi contado vrias vezes: um
como A1 A2 A3 M1 M2 T1 T2 EIC, outro como A2 A1 A3 M1 M2 T1 T2 EIC

i
i

contagemfran
2009/10/29
i page 34
Estilo OBMEP

i
i

34

CAP. 4: MAIS PERMUTAES E COMBINAES (GRUPO 2)

etc. Na verdade, ele foi contado tantas vezes como os modos de ordenar os 3 A, os 2 M e os 2 T, que igual a 3! 2! 2!. O nmero de
10!
anagramas , ento, 3!2!2!
= 151 200, como encontrado anteriormente.
O segundo raciocnio pode ser facilmente estendido para uma situao geral. O nmero de permutaes de n objetos nem todos
distintos, em que um deles aparece n1 vezes, outro n2 vezes, e assim
por diante, Pnn1 ,n2 ,... = n1 !nn!2 !... .
Exemplo 4. De quantos modos 6 pessoas (Joo, Maria, Pedro,
Janete, Paulo e Alice) podem ser divididas em 3 duplas?
Soluo: O problema mais sutil do que parece a princpio.
primeira vista, pode parecer que a situao a mesma do problema
anterior. Uma maneira de dividir as 6 pessoas em duplas colocar as
pessoas em fila e formar uma permutao de AABBCC. Como visto
6!
no exemplo anterior, isto pode ser feito de 2!2!2!
= 90 modos. Mas
isto no est correto, pois atribuiu nomes especficos (A, B e C) s
duplas formadas. Note que colocar Joo e Maria na dupla A e Pedro
e Janete na dupla B equivalente a colocar Joo e Maria na dupla B
e Pedro e Janete na dupla A. Portanto, uma mesma distribuio em
duplas est sendo contada vrias vezes. Mais precisamente, cada distribuio em duplas est sendo contada tantas vezes quanto o nmero
de modos de ordenar A, B e C, ou seja, 3! = 6 vezes. Logo, o nmero
de possveis distribuies em duplas 90
6 = 15.
Exemplo 5. Uma professora tem 3 bolas de gude para distribuir
para 5 meninos (digamos, Alfredo, Bernardo, Carlos, Diogo e Eduardo). De quantos modos ela pode fazer essa distribuio:
(a) Supondo que ela d as bolas para 3 alunos distintos?
Soluo: Neste caso, ela deve escolher 3 dentre os 5 meninos para

i
i

contagemfran
2009/10/29
i page 35
Estilo OBMEP

i
i

35
receber as bolas, o que pode ser feito de C53 = 10 modos.
(b) Supondo que os contemplados possam ganhar mais de uma bola?
(Por exemplo, Carlos pode receber todas as bolas.)
Soluo: Listamos abaixo algumas possveis escolhas dos contemplados:
Alfredo, Bernardo, Eduardo
Alfredo, Alfredo, Diogo
Alfredo, Diogo, Diogo
Carlos, Carlos, Carlos
Esses grupamentos so chamados de combinaes completas (ou
com repetio) dos 5 meninos tomados 3 a 3. Note que o que distingue as diferentes distribuies o nmero de bolas que cada aluno
recebe. Portanto, o nmero de possibilidades igual ao nmero
de listas (x1 , x2 , x3 , x4 , x5 ) de nmeros inteiros no negativos (representando o nmero de objetos dados a Alfredo, Bernardo, Carlos, Diogo e Eduardo, respectivamente) que satisfazem a equao
x1 + x2 + x3 + x4 + x5 = 3.
Neste caso simples, podemos resolver o problema separando a contagem em casos. A primeira possibilidade a de que haja trs premiados, cada um ganhando uma bola. Como vimos acima, isto pode ser
feito de C53 = 10 modos. A segunda possibilidade de que haja dois
premiados, um ganhando 1 bola e outro 2 bolas. O primeiro menino
pode ser escolhido de 5 modos, e o segundo, de 4; logo, h 4 5 = 20
maneiras de distribuir as bolas para dois dos meninos. Finalmente, as
bolas podem ir todas para um s menino, que pode ser escolhido de 5
modos. Portanto, o nmero total de possibilidades 10 + 20 + 5 = 35.
No entanto, dividir a contagem em casos, como fizemos acima, no

i
i

contagemfran
2009/10/29
i page 36
Estilo OBMEP

i
i

36

CAP. 4: MAIS PERMUTAES E COMBINAES (GRUPO 2)

vai ser prtico caso o nmero de bolas e meninos seja maior. Para
contar de modo eficiente o nmero de distribuies, vamos recorrer
a um truque, que nos permite transformar este problema em outro
mais simples. Para formar as diferentes distribuies, colocamos as
bolas em fila e as separamos em cinco lotes (correspondentes a cada
um dos meninos), atravs de traos verticais. claro que, neste caso,
alguns desses lotes estaro vazios.
Vejamos alguns exemplos:
0||0|0| corresponde a dar 1 bola para Alfredo, para Carlos e para
Diogo, enquanto Bernardo e Eduardo no ganham bolas.
||00||0 corresponde a dar 2 bolas para Carlos e 1 para Eduardo,
enquanto Alfredo, Bernardo e Carlos no ganham bolas.
Note que h uma correspondncia perfeita entre as possveis distribuies e as listas formadas por 3 bolas e 4 traos. Mas estas
ltimas ns j sabemos contar! Basta escolher 3 das 7 posies para
colocar as bolas, o que pode ser feito de C73 = 35 maneiras, como
encontramos acima.
Naturalmente, podemos aplicar esta soluo para o problema geral
de contar o nmero de maneiras de distribuir p objetos para n pessoas
(ou seja, de calcular o nmero de solues inteiras e no negativas
de x1 + x2 + . . . + xn = p, ou ainda, de calcular o nmero CRnp de
combinaes completas de n elementos tomados p a p). Temos p bolas,
que devem ser separadas por n 1 tracinhos. Ou seja, precisamos
escolher p das n + p 1 posies para as bolas. A resposta, portanto,
p
CRnp = Cn+p1
.

Exerccios
1) De quantos modos podemos formar uma roda com 5 meninos

i
i

contagemfran
2009/10/29
i page 37
Estilo OBMEP

i
i

37
e 5 meninas de modo que crianas de mesmo sexo no fiquem
juntas?
2) De quantos modos podemos formar uma roda de ciranda com
6 crianas, de modo que duas delas, Vera e Isadora, no fiquem
juntas?
3) De quantos modos possvel dividir 15 atletas em trs times de
5 atletas, denominados Esporte, Tupi e Minas?
4) De quantos modos possvel dividir 15 atletas em trs times de
5 atletas?
5) De quantos modos possvel dividir 20 objetos em 4 grupos de
3 e 2 grupos de 4?
6) Um campeonato disputado por 12 clubes em rodadas de 6
jogos cada. De quantos modos possvel selecionar os jogos da
primeira rodada?
7) Quantos so os anagramas da palavra ESTRELADA?
8) Quantos so os nmeros naturais de 7 algarismos nos quais o algarismo 4 figura exatamente 3 vezes e o algarismo 8 exatamente
2 vezes?
9) Quantos so os subconjuntos de {a1 , a2 , . . . , an }, com p elementos, nos quais:
(a) a1 figura?
(b) a1 no figura?
(c) a1 e a2 figuram?
(d) pelo menos um dos elementos a1 , a2 figura?
(e) exatamente um dos elementos a1 , a2 figura?

i
i

contagemfran
2009/10/29
i page 38
Estilo OBMEP

i
i

38

CAP. 4: MAIS PERMUTAES E COMBINAES (GRUPO 2)

10) Considere um conjunto C de 20 pontos do espao que tem um


subconjunto C1 formado por 8 pontos coplanares. Sabe-se que
toda vez que 4 pontos de C so coplanares, ento, eles so pontos
de C1 . Quantos so os planos que contm pelo menos trs pontos
de C?
11) Quantos so os anagramas da palavra PARAGUAIO que no
possuem consoantes juntas?
12) De quantos modos podemos selecionar p elementos do conjunto
{1, 2, . . . , n} sem selecionar dois nmeros consecutivos?
13) Depois de ter dado um curso, um professor resolve se despedir de
seus 7 alunos oferecendo, durante 7 dias consecutivos, 7 jantares
para 3 alunos cada, de modo que o mesmo par de alunos no
comparea a mais de um jantar.
(a) Prove que cada aluno deve comparecer a exatamente 3
jantares.
(b) De quantos modos o professor pode fazer os convites para
os jantares?
14) Em uma escola, um certo nmero de professores se distribuem
em 8 bancas examinadoras de modo que cada professor participa
de exatamente duas bancas e cada duas bancas tm exatamente
um professor em comum.
(a) Quantos so os professores?
(b) Quantos professores h em cada banca?
15) Quantas so as solues inteiras e positivas de x + y + z = 7?
16) Quantas so as solues inteiras e no negativas da desigualdade
x + y + z 6?

i
i

contagemfran
2009/10/29
i page 39
Estilo OBMEP

i
i

39
17) Uma indstria fabrica 5 tipos de balas, que so vendidas em
caixas de 20 balas, de um s tipo ou sortidas. Quantos tipos
diferentes de caixa podem ser fabricados?

i
i

contagemfran
2009/10/29
i page 40
Estilo OBMEP

i
i

Captulo 5

Probabilidade Condicional
(grupo 2)
Veremos a seguir exemplos de situaes em que a probabilidade
de um evento modificada pela informao de que um outro evento
ocorreu, levando-nos a definir probabilidades condicionais.
Exemplo 1. Em uma urna h duas moedas aparentemente iguais.
Uma delas uma moeda comum, com uma cara e uma coroa. A
outra, no entanto, uma moeda falsa, com duas caras. Suponhamos
que uma dessas moedas seja sorteada e lanada.
(a) Qual a probabilidade de que a moeda lanada seja a comum?
Soluo: A resposta 1/2, j que ambas as moedas tm a mesma
chance de serem sorteadas.
(b) Qual a probabilidade de que saia uma cara?
Soluo: H quatro possveis resultados para o sorteio da moeda e o
resultado do lanamento, todos com a mesma probabilidade:
40
i

i
i

contagemfran
2009/10/29
i page 41
Estilo OBMEP

i
i

41
a moeda sorteada a comum, e o resultado cara;
a moeda sorteada a comum, e o resultado coroa;
a moeda sorteada a falsa, e o resultado cara;
a moeda sorteada a falsa, e o resultado tambm cara, mas
saindo a outra face.
Como em 3 dos 4 casos acima o resultado cara, a probabilidade
3
de sair cara .
4
(c) Se o resultado do lanamento cara, qual a probabilidade de
que a moeda sorteada tenha sido a comum?
Soluo: No item (a) verificamos que a probabilidade de sair cara
1/2. Mas a situao diferente agora: temos uma informao adicional, a de que, aps o lanamento da moeda, o resultado foi cara.
Com esta informao, podemos rever o clculo da probabilidade da
moeda honesta ter sido sorteada. Dos quatro resultados possveis para
o experimento, listados acima, o segundo deve ser excludo. Restam,
assim, trs possibilidades igualmente provveis. Delas, apenas na
primeira a moeda sorteada a comum. Logo, com a informao de
que o lanamento resultou em cara, a probabilidade de que a moeda
sorteada tenha sido a comum se reduziu a 1/3.
A probabilidade que calculamos no exemplo anterior uma probabilidade condicional. De um modo geral, a probabilidade condicional de um evento A, na certeza da ocorrncia de um evento B (de
probabilidade no nula) denotada por P (A|B) e definida como
P (A|B) =

P (A B)
.
P (B)

No caso do exemplo anterior, chamemos de A o evento sortear a

i
i

contagemfran
2009/10/29
i page 42
Estilo OBMEP

i
i

42

CAP. 5: PROBABILIDADE CONDICIONAL (GRUPO 2)

moeda comum, e de B o evento obter resultado cara. O evento


A B sortear a moeda comum e tirar cara. Temos:
P (A B) = 1/4, P (B) = 3/4 e, assim, P (A|B) =

1/4
1
= ,
3/4
3

como encontramos anteriormente.


Exemplo 2. Uma carta sorteada de um baralho comum, que possui
13 cartas (A, 2, 3, 4, 5, 6, 7, 8, 9, 10, J, Q, K) de cada naipe (ouros,
copas, paus e espadas).
(a) Qual a probabilidade de que a carta sorteada seja um A?
Soluo: Como o baralho tem 13 4 = 52 cartas e 4 delas so ases,
4
1
a probabilidade de tirar um A
= .
52
13
(b) Sabendo que a carta sorteada de copas, qual a probabilidade
de que ela seja um A?
Soluo: O fato de que a carta sorteada de copas restringe os casos
possveis s 13 cartas de copas, das quais exatamente uma A. Logo,
a probabilidade de ser sorteado um A, dado que a carta sorteada de
1
copas, permanece igual a
. Mais formalmente, designando por A o
13
evento sortear A e, por B, sortear copas, o evento A B sortear
o A de copas, e a probabilidade pedida
P (A|B) =

P (A B)
1/52
1
=
= .
P (B)
13/52
13

O exemplo acima ilustra uma situao importante: aquela na qual


a probabilidade condicional de A na certeza de B igual probabilidade de A (ou seja a ocorrncia de B no influi na probabilidade de
P (A B)
= P (B), ou
ocorrncia de A). Esta condio implica em
P (B)

i
i

contagemfran
2009/10/29
i page 43
Estilo OBMEP

i
i

43
seja, P (A B) = P (A)P (B). Dizemos, ento, que dois eventos A e
B tais que P (A B) = P (A)P (B) so independentes.
Exemplo 3. Um sistema de segurana tem dois dispositivos que
funcionam de modo independente e que tem probabilidades iguais a
0,2 e 0,3 de falharem. Qual a probabilidade de que pelo menos um
dos dois componentes no falhe?
Soluo: Como os componentes funcionam independentemente, os
eventos A = o primeiro dispositivo falha e B = o segundo dispositivo falha so independentes. Logo, o evento A B = ambos
falham tem probabilidade P (A B) = P (A)P (B) = 0, 2 0,3 = 0,06
e, assim, a probabilidade de que pelo menos um no falhe igual a
1 0,06 = 0,94.
Exemplo 4. Uma questo de mltipla escolha tem 5 alternativas.
Dos alunos de uma turma, 50% sabem resolver a questo, enquanto
os demais chutam a resposta. Um aluno da turma escolhido ao
acaso.
(a) Qual a probabilidade de que ele tenha acertado a questo?
Soluo: Neste caso, vamos utilizar probabilidades condicionais conhecidas para calcular a probabilidade de dois eventos ocorrerem siP (A B)
multaneamente. Observe que, da expresso P (A|B) =
P (B)
decorre P (AB) = P (B)P (A|B). Se o aluno sabe resolver a questo,
ele tem probabilidade 1 de acert-la, enquanto, se ele no sabe, sua
probabilidade de acerto 1/5 = 0,2. Portanto, P (acerta|sabe) = 1,
enquanto P (acerta|no sabe) = 0,2. Podemos, ento, obter as
seguintes probabilidades:
P (sabe e acerta) = P (sabe) P (acerta|sabe) = (0,5) 1 = 0,5

i
i

contagemfran
2009/10/29
i page 44
Estilo OBMEP

i
i

44

CAP. 5: PROBABILIDADE CONDICIONAL (GRUPO 2)

P (no sabe e acerta) = P (no sabe) P (acerta|no sabe)


= 0,5 0,2 = 0,1.
Finalmente,
P (acerta) = P (sabe e acerta) + P (no sabe e acerta)
= 0,5 + 0,1 = 0,6.
(b) Dado que o aluno acertou a questo, qual a probabilidade de
que ele tenha chutado?
Soluo: O que desejamos calcular a probabilidade condicional de
que o aluno no saiba resolver a questo, dado que ele a acertou.
Temos:

P (no sabe|acerta) =

P (no sabe e acerta)


0,1
1
=
= .
P (acerta)
0,6
6

Exerccios
1) Joga-se um dado viciado duas vezes. Determine a probabilidade
condicional de obter 3 na primeira jogada sabendo que a soma
dos resultados foi 7.
2) Um juiz de futebol meio trapalho tem no bolso um carto
amarelo, um carto vermelho e um carto com uma face amarela
e uma face vermelha. Depois de uma jogada violenta, o juiz
mostra um carto, retirado do bolso ao acaso, para um atleta.
Se a face que o jogador v amarela, qual a probabilidade da
face voltada para o juiz ser vermelha?

i
i

contagemfran
2009/10/29
i page 45
Estilo OBMEP

i
i

45
3) Um exame de laboratrio tem eficincia de 95% para detectar uma doena quando ela de fato existe. Alm disso, o teste
aponta um resultado falso positivo para 1% das pessoas sadias
testadas. Se 0,5% da populao tem a doena, qual a probabilidade de que uma pessoa, escolhida ao acaso, tenha a doena,
sabendo que o seu exame foi positivo?
4) Quantas vezes, no mnimo, se deve lanar um dado para que a
probabilidade de obter algum 6 seja superior a 0,9?
5) Em uma cidade, as pessoas falam a verdade com probabilidade
1/3. Suponha que A faz uma afirmao e D diz que C diz que
B diz que A falou a verdade. Qual a probabilidade de que A
tenha falado a verdade?
6) 2n jogadores de igual habilidade disputam um torneio. Eles so
divididos em grupos de 2, ao acaso, e jogadores de um mesmo
grupo jogam entre si. Os perdedores so eliminados, e os vencedores so divididos novamente em grupos de 2 e assim por diante, at restar apenas um jogador, que proclamado campeo.
(a) Qual a probabilidade de os jogadores A e B se enfrentarem
durante o torneio?
(b) Qual a probabilidade de o jogador A jogar exatamente k
partidas?
7) Duas mquinas A e B produzem 3 000 peas em um dia. A
mquina A produz 1 000 peas, das quais 3% so defeituosas. A
mquina B produz as restantes 2 000, das quais 1% so defeituosas. Da produo total de um dia, uma pea escolhida ao
acaso e, examinando-a, constata-se que ela defeituosa. Qual
a probabilidade de que ela tenha sido produzida pela mquina
A?

i
i

contagemfran
2009/10/29
i page 46
Estilo OBMEP

i
i

46

CAP. 5: PROBABILIDADE CONDICIONAL (GRUPO 2)

8) Um prisioneiro recebe 50 bolas brancas e 50 bolas pretas. O


prisioneiro deve distribuir, do modo que preferir, as bolas em
duas urnas, mas de modo que nenhuma das duas urnas fique
vazia. As urnas sero embaralhadas e o prisioneiro dever, de
olhos fechados, escolher uma urna e, nesta urna, uma bola. Se
a bola for branca, ele ser libertado; caso contrrio, ele ser
condenado. De que modo o prisioneiro deve distribuir as bolas
nas urnas para que a probabilidade de ele ser libertado seja
mxima? Qual essa probabilidade?

i
i

contagemfran
2009/10/29
i page 47
Estilo OBMEP

i
i

Exerccios Adicionais
Para os alunos dos Grupos 1 e 2
1) Um grupo de 4 alunos (Alice, Bernardo, Carolina e Daniel) tem
que escolher um lder e um vice-lder para um debate.
(a) Faa uma lista de todas as possveis escolhas (use a inicial de cada nome, para facilitar). Organize a sua lista
do seguinte modo: primeiro, escreva todas as possibilidades em que Alice a presidente, depois aquelas em que
Bernardo presidente, e assim por diante.
(b) Usando agora o princpio multiplicativo, ache quantas so
as escolhas possveis de lder e vice-lder em que os alunos
tm sexos diferentes.
2) De quantos modos possvel colocar 8 pessoas em fila de modo
que duas dessas pessoas, Vera e Paulo, no fiquem juntas e duas
outras, Helena e Pedro, permaneam juntas?
3) Permutam-se de todas as formas possveis os algarismos 1, 2,
4, 6, 7 e escrevem-se os nmeros assim formados em ordem
crescente. Determine:
(a) Que lugar ocupa o nmero 62 417?
(b) Que nmero que ocupa o 66o lugar?
47
i

i
i

contagemfran
2009/10/29
i page 48
Estilo OBMEP

i
i

48

EXERCCIOS ADICIONAIS

(c) Qual o 166o algarismo escrito?


4) De um baralho comum de 52 cartas, sacam-se, sucessivamente
e sem reposio, duas cartas. De quantos modos isso pode ser
feito se a primeira carta deve ser de copas, e a segunda no deve
ser um rei?
5) Uma turma tem aulas s segundas, quartas e sextas, das 13h
s 14h e das 14h s 15h. As matrias so Matemtica, Fsica e
Qumica, cada uma com duas aulas semanais em dias diferentes.
De quantos modos pode ser feito o horrio dessa turma?
6) De quantos modos podemos colocar uma torre branca e outra
preta em um tabuleiro de xadrez, sem que uma ameace a outra?
(Ou seja, as duas torres no devem estar na mesma linha ou
coluna.)
7) Um anagrama de uma palavra uma nova palavra obtida reordenando suas letras (essa nova palavra pode no fazer sentido).
(a) Quantos so os anagramas da palavra SAVEIRO?
(b) Quantos deles comeam com S?
(c) Quantos deles terminam com vogal?
(d) Quantos apresentam o pedao VEIR?
8) Em uma festa h 5 homens e 5 mulheres, que vo formar 5
casais para uma dana de quadrilha. Quantas so as maneiras
de formar esses casais? E se houvesse 5 homens e 8 mulheres?
9) De quantos modos 5 homens e 5 mulheres podem se sentar em
5 bancos de 2 lugares, se em cada banco deve haver um homem
e uma mulher?

i
i

contagemfran
2009/10/29
i page 49
Estilo OBMEP

i
i

49
10) Para pintar a bandeira abaixo, esto disponveis as seis cores
dadas, sendo que regies adjacentes devem ser pintadas de cores
diferentes.

(a) Qual o nmero mnimo de cores a serem usadas?


(b) De quantos modos a bandeira pode ser pintada?
11) Supondo que as mesmas 6 cores estejam disponveis, de quantos
modos pode-se pintar o smbolo abaixo de modo que quadrantes
adjacentes no tenham a mesma cor (quadrantes opostos podem
ter a mesma cor)?

12) Quantos dados diferentes possvel formar gravando nmeros


de 1 a 6 sobre as faces de um cubo?
(a) Suponha uma face de cada cor.
(b) Suponha as faces iguais.
(c) Suponha que as faces so iguais e que a soma dos pontos
de faces opostas deva ser igual a 7.
13) Um estacionamento, inicialmente vazio, tem 10 vagas adjacentes. O primeiro carro pode parar em qualquer vaga. A partir

i
i

contagemfran
2009/10/29
i page 50
Estilo OBMEP

i
i

50

EXERCCIOS ADICIONAIS

do segundo carro, porm, cada carro deve parar em uma vaga


vizinha a uma vaga j ocupada. De quantos modos diferentes
as vagas podem ser preenchidas? [Sugesto: passe o filme ao
contrrio; de onde sai o ltimo carro? E o penltimo?]
14) Para sortear uma vaga em uma reunio de condomnio, da qual
participaram 12 pessoas, foram colocados 12 pedaos de papel
idnticos, todos em branco, exceto um, no qual foi escrita a
palavra vaga. Cada pessoa retira, na sua vez, um papel da
urna. O que melhor: ser o primeiro ou o ltimo a sortear seu
papel?
15) Considere uma turma de 20 alunos.
(a) Quantas so as maneiras de escolher um representante, um
secretrio e um tesoureiro?
(b) Considere agora que desejemos escolher trs dos alunos
para formar uma comisso. Por que a resposta no a
mesma do item anterior?
(c) O que necessrio fazer com a resposta do item a para
obter a resposta do item b?
16) Um casal decidiu que vai ter 4 filhos. Qual a probabilidade de
que:
(a) Tenham pelo menos um menino?
(b) Tenham filhos de ambos os sexos?
(c) Tenham dois filhos de cada sexo?
17) Os alunos de um certo curso fazem 4 matrias, entre as quais
Clculo e lgebra Linear. As provas finais sero realizadas em
uma nica semana (de segunda a sexta). Admitindo que cada

i
i

contagemfran
2009/10/29
i page 51
Estilo OBMEP

i
i

51
professor escolha o dia da sua prova ao acaso, qual a probabilidade de que:
(a) As provas de lgebra Linear e Probabilidade sejam marcadas para o mesmo dia?
(b) No haja mais do que uma prova em cada dia?
18) 24 times so divididos em dois grupos de 12 times cada. Qual
a probabilidade de dois desses times ficarem no mesmo grupo?
19) Em um armrio h 6 pares de sapatos. Escolhem-se 2 ps de
sapatos. Qual a probabilidade de se formar um par de sapatos?

Para os alunos do Grupo 2


20) Em uma turma h 12 rapazes e 15 moas. Quantos so os modos
de escolher uma comisso de 4 pessoas:
(a) Sem restries?
(b) Que incluam Jos (que um dos alunos)?
(c) Que no incluam Mrcia (que uma das alunas)?
(d) Com 2 rapazes e 2 moas?
(e) Que tenham pelo menos um rapaz e uma moa?
21) No jogo da Megassena so sorteados, a cada extrao, 6 dos
nmeros de 1 a 60.
(a) Quantos so os resultados possveis da Megassena?
(b) Um apostador aposta nos nmeros 2, 7, 21, 34, 41 e 52.
Qual a sua chance de ganhar? E se ele tivesse apostado
nos nmeros 1, 2, 3, 4, 5 e 6?

i
i

contagemfran
2009/10/29
i page 52
Estilo OBMEP

i
i

52

EXERCCIOS ADICIONAIS

(c) Quantas vezes maiores so as chances de ganhar de quem


aposta em 8 nmeros?
(d) Suponha que o nmero 17 no sorteado h muito tempo.
Isso modifica as chances de ele ser sorteado da prxima
vez?
22) Cinco dados so jogados simultaneamente. Determine a probabilidade de se obter:
(a) um par;
(b) dois pares;
(c) uma trinca;
(d) uma quadra;
(e) uma quina;
(f) uma sequncia;
(g) um full hand, isto , uma trinca e um par.
23) Em um grupo de 4 pessoas, qual a probabilidade de:
(a) Haver alguma coincidncia de signos zodiacais?
(b) Haver exatamente trs pessoas com um mesmo signo e uma
pessoa com outro signo?
(c) As quatro pessoas terem o mesmo signo?
(d) Haver duas pessoas com um mesmo signo e duas outras
pessoas com outro signo?
24) Em um torneio h 16 jogadores de habilidades diferentes. Eles
so sorteados em grupos de 2, que jogam entre si. Os perdedores
so eliminados, e os vencedores jogam entre si, novamente divididos em grupos de 2, at restar s um jogador, que declarado

i
i

contagemfran
2009/10/29
i page 53
Estilo OBMEP

i
i

53
campeo. Suponha que no haja zebras (ou seja, o jogador de
habilidade superior sempre vence).
(a) Qual a probabilidade de o segundo melhor jogador ser
vice-campeo do torneio?
(b) Qual a probabilidade de o quarto melhor jogador ser vicecampeo do torneio?
(c) Qual o nmero mximo de partidas que o dcimo melhor
jogador consegue disputar?
(d) Qual a probabilidade de ele disputar esse nmero mximo
de partidas?
25) Um dado honesto tem duas de suas faces pintadas de vermelho
e as demais, de azul. O dado lanado trs vezes, anotando-se
a cor da face obtida.
(a) Qual a probabilidade de que a cor obtida no 1o lanamento seja igual obtida no 3o ?
(b) Dado que a mesma cor foi obtida nos 1o e 2o lanamentos,
qual a probabilidade de que no 3o lanamento saia esta
mesma cor?
26) Sejam Im = {1, 2, ..., m} e In = {1, 2, ..., n}, com m n. Quantas so as funes f : Im In estritamente crescentes? E no
decrescentes?
27) Quantos so os nmeros naturais de 7 dgitos nos quais o dgito
4 figura exatamente 3 vezes, e o dgito 8, exatamente 2 vezes?
28) O conjunto A possui p elementos, e o conjunto B possui n elementos. Determine o nmero de funes f : A B sobrejetivas
para:
(a) p = n;

i
i

contagemfran
2009/10/29
i page 54
Estilo OBMEP

i
i

54

EXERCCIOS ADICIONAIS

(b) p = n + 1;
(c) p = n + 2.
29) Considere um conjunto C de 20 pontos do espao que tem um
subconjunto C1 formado por 8 pontos coplanares. Sabe-se que
toda vez que 4 pontos de C so coplanares, ento, eles so pontos
de C1 . Quantos so os planos que contm pelo menos trs pontos
de C?
30) Onze cientistas trabalham num projeto sigiloso. Por questes
de segurana, os planos so guardados em um cofre protegido
por muitos cadeados, de modo que s possvel abri-los todos
se houver pelo menos 5 cientistas presentes.
(a) Qual o nmero mnimo possvel de cadeados?
(b) Na situao do item a, quantas chaves cada cientista deve
ter?

i
i

contagemfran
2009/10/29
i page 55
Estilo OBMEP

i
i

Solues dos Exerccios

Captulo 1
1) (a) As possveis escolhas de lder e vice-lder so (usando somente as iniciais): A-B, A-C, A-D, B-A, B-C, B-D, C-A,
C-B, C-D, D-A, D-B, D-C. Portanto, no total h 12 escolhas possveis.
(b) H 4 maneiras de escolher o lder. Para cada uma dessas
escolhas, o vide-lder pode ser escolhido de 3 modos (j que
a mesma pessoa no pode, ao mesmo tempo, ser lder e
vice-lder). Logo, pelo Princpio Multiplicativo, o nmero
de possibilidades 4 3 = 12, que foi o que obtivemos
contando diretamente.

2) (a) Como h 3 opes de saladas, 3 de sopas e 4 de pratos


principais, h 3 + 3 + 4 = 20 modos de escolher um prato
do cardpio.
(b) O nmero de possveis refeies
3 (saladas) 3 (sopas) 4 (pratos principais) = 36.
55
i

i
i

contagemfran
2009/10/29
i page 56
Estilo OBMEP

i
i

56
3) So escritos 9 nmeros de 1 algarismo, 90 nmeros de 2 algarismos (de 10 a 99) e 1 nmero de 3 algarismos. Logo, o total de
algarismos escritos 9 + 2 90 + 3 = 192.
4) (a) Cada um dos dois jogadores pode obter qualquer dos
nmeros de 1 a 6. Logo, o nmero de possveis combinaes de resultados 6 6 = 36.
(b) A soma pode ser qualquer nmero inteiro de 1 + 1 = 2 at
6 + 6 = 12. H, portanto, 11 somas possveis.
5) So 7 filamentos. Para cada um, h duas possibilidades (aceso
ou apagado). Logo, o nmero total de configuraes possveis
2 2 2 2 2 2 2 = 27 = 128.
Excluindo aquela em que esto todos apagados, obtemos 127
smbolos diferentes.
6) (a) So necessrias pelo menos 3 cores.
(b) A faixa vertical pode ser pintada de 6 modos. Pintando
a faixa vertical de cima para baixo, temos que a primeira
pode ser pintada de 5 modos (no pode usar a cor da faixa
vertical), a segunda de 4 (no pode usar a cor da faixa
vertical e a da 1a faixa horizontal) e a terceira tambm de
4 (no pode usar a cor da faixa vertical e a da 2a faixa
horizontal). Logo, o nmero total de bandeiras
6 5 4 4 = 480.
7) Vamos contar separadamente os casos em que os quadrantes 1
e 3 tm cores iguais e cores diferentes. Pondo cores iguais nos

i
i

contagemfran
2009/10/29
i page 57
Estilo OBMEP

i
i

i
Solues dos Exerccios

57

quadrantes 1 e 3, temos 5 4 4 = 80 possibilidades, pois h


5 modos de escolher a cor nica para os quadrantes 1 e 3, h 4
modos de escolher a cor do quadrante 2 e h 4 modos de escolher
a cor do quadrante 4. Pondo cores diferentes nos quadrantes 1
e 3, h 5 4 3 3 = 180 possibilidades, pois h 5 modos de
escolher a cor para o quadrante 1, h 4 modos de escolher a cor
do quadrante 3, h 3 modos de escolher a cor do quadrante 2
e h 3 modos de escolher a cor do quadrante 4. A resposta
80 + 180 = 260.
8) H 510 gabaritos possveis. Para ter a letra A aparecendo exatamente uma vez, devemos escolher a questo em que ela aparece
(10 possibilidades) e, a seguir, escolher a alternativa das demais
(4 para cada, para um total de 49 ). Logo, o nmero total de
possibilidades 10 49 . Se a letra A no aparece, temos somente 4 possibilidades de escolha para cada questo, para um
total de 410 possibilidades.
9) Os subconjuntos de {1, 2, 3} so 8 :
, {1}, {2}, {3}, {1, 2}, {1, 3}, {2, 3}, {1, 2, 3}.
De um modo geral, um subconjunto de um conjunto de n elementos formado decidindo se cada elemento entra ou no no
subconjunto. Para cada elemento h 2 possibilidades; o nmero
total de possibilidades 2n .
10) A primeira pessoa pode escolher sua cadeira de 5 modos; a segunda, de 4; a terceira, de 3. A resposta 5 4 3 = 60.
11) A primeira mulher pode escolher sua posio de 10 modos. A

i
i

contagemfran
2009/10/29
i page 58
Estilo OBMEP

i
i

58
segunda, de 8 modos. As outras, de 6, de 4 e de 2 modos. O
primeiro homem, de 5 modos. Os demais, de 4, de 3, de 2 e de
1. A resposta 10 8 6 4 2 5 4 3 2 1 = 460 800.

12) O tabuleiro de 64 casas possui 4 casas de canto (vrtices), 24


casas laterais que no so vrtices e 36 casas centrais. Cada
casa de canto possui 3 casas adjacentes; cada lateral possui 5
casas adjacentes e cada central possui 8 casas adjacentes. Vamos
contar separadamente os casos que ocorrem conforme o rei negro
ocupe uma casa de canto, lateral ou central. Se o rei negro
ocupar uma casa de canto, haver 4 posies para o rei negro e
60 posies para o rei branco, pois das 64 casas do tabuleiro uma
estar ocupada, e as 3 a ela adjacentes no podero ser ocupadas
pelo rei branco. Haver, portanto, 460 = 240 modos de dispor
os reis.
Se o rei negro ocupar uma casa lateral que no seja de canto,
haver 24 posies para o rei negro e 58 posies para o rei
branco, pois das 64 casas do tabuleiro uma estar ocupada e
as 5 a ela adjacentes no podero ser ocupadas pelo rei branco.
Haver, portanto,
24 58 = 1 392
modos de dispor os reis.
Se o rei negro ocupar uma casa central, haver 36 posies para
o rei negro e 55 posies para o rei branco, pois das 64 casas
do tabuleiro uma estar ocupada, e as 8 a ela adjacentes no
podero ser ocupadas pelo rei branco. Haver, portanto,
36 55 = 1 980

i
i

contagemfran
2009/10/29
i page 59
Estilo OBMEP

i
i

59

Solues dos Exerccios

modos de dispor os reis. Portanto, a resposta


240 + 1 392 + 1 980 = 3 612.
Se os reis fossem iguais, a resposta seria a metade da resposta
anterior, 1 806.
13) Note que no caso em que so permitidas repeties, a condio
de a letra A figurar na palavra terrvel, pois ela pode figurar
uma s vez, ou duas etc... Por isso, melhor contar todas
as palavras do alfabeto e diminuir as que no tm A e as que
comeam por A. A resposta 265 255 264 = 1 658 775.
No caso sem repetio, pode-se contar diretamente: h 4 modos
de escolher a posio do A, 25 modos de escolher a letra da
primeira casa restante, 24 para a segunda casa restante etc. A
resposta 4 25 24 23 22 = 1 214 400. Pode-se tambm
repetir o raciocnio do caso com repetio:
26252423222524232221125242322 =
1 214 400.
14) H 26 modos de escolher cada letra e 10 modos de escolher cada
algarismo. A resposta 263 104 = 175 760 000.
15) Os passageiros que preferem se sentar de frente podem faz-lo
de 5432 = 120 modos; os que preferem se sentar de costas
podem faz-lo de 5 4 3 = 60 modos; os demais podem se
colocar nos lugares restantes de 321 = 6 modos. A resposta
120 60 6 = 43 200.
16) (a) O algarismo 0 aparece nas unidades 222 vezes, nos nmeros
10, 20, 30, . . . , 2 200. Aparece nas dezenas 220 vezes, nos

i
i

contagemfran
2009/10/29
i page 60
Estilo OBMEP

i
i

60
nmeros 10x, 20x, . . . , 220x. Aparece nas centenas 200
vezes, nos nmeros 10xy e 20xy. A resposta
222 + 220 + 200 = 642.
(b) Contamos os nmeros com algum algarismo igual a 0,
descontando do clculo anterior o que houver sido contado indevidamente. O 0 aparece nas unidades 222 vezes,
nos nmeros 10, 20, 30, . . . , 2 200. Das 220 vezes em que
aparece nas dezenas devemos descontar o total dos nmeros
do conjunto
{10x, 20x, . . . , 220x ; x = 0},
que 22. Das 200 vezes em que aparece nas centenas devemos descontar o total dos nmeros do conjunto
{10xy, 20xy ; x = 0 ou y = 0}, que 2 (9 + 9 + 1) = 38.
A resposta
222 + (220 22) + (200 38) = 222 + 198 + 162 = 582.
Outra soluo: O algarismo 0 aparece nas unidades
222 vezes, nos nmeros 10, 20, 30, . . . , 2 200. Faltam os
nmeros dos conjuntos {10x, 20x, . . . , 220x ; x 6= 0} e
{10xy, 20xy ; x 6= 0 e y 6= 0}. O primeiro tem 229 = 198
nmeros, e o segundo, 2 9 9 = 162 nmeros. A resposta
222 + 198 + 162 = 582.
17) O mais simples fazer todos os nmeros menos aqueles em que
o 5 no figura. A resposta 91010108999 = 3 168.
18) Para formar uma coleo, voc deve decidir quantas Veja faro

i
i

contagemfran
2009/10/29
i page 61
Estilo OBMEP

i
i

i
Solues dos Exerccios

61

parte da coleo etc. A quantidade de revistas Veja pode ser


escolhida de 6 modos (0, 1, 2, 3, 4, 5). A de poca, de 7 modos.
A de Isto , de 5 modos. O nmero de colees 675 = 210.
O nmero de colees no vazias 209.
19) A soluo est errada. possvel que a mesma cor tenha sido
escolhida para as faixas extremas. Neste caso, o nmero de
possibilidades de escolha para a cor da faixa central 3, e no
2. Logo, para esta ordem de pintura no possvel aplicar
diretamente o Princpio Multiplicativo.
20) O casal Joo e Maria foi considerado diferente do casal Maria e
Joo. Isso devido ao fato de termos trabalhado com o conceito
de primeira pessoa do casal. Por isso a resposta encontrada o
dobro da resposta real.
21) H dois tipos de peas: as formadas por nmeros iguais (que
so 7: de 0 0 at 6 6) e as formadas por um par de nmeros
distintos. Destas, h 7 6/2 = 21 peas. O total 28. Se os
nmeros forem at 8, o nmero de peas 9 + 9 8/2 = 45.
22) Cada retngulo corresponde a escolher 2 linhas e 2 colunas entre
as 9 linhas e colunas de separao das casas. As duas linhas
podem ser escolhidas de 9 8/2 = 36 modos. O nmero de
possibilidades para as colunas o mesmo. Logo, o nmero total
de retngulos 36 36 = 1 296.

i
i

contagemfran
2009/10/29
i page 62
Estilo OBMEP

i
i

62

Captulo 2
1) (a) Os resultados possveis so 2, 3, 4, 5, 6, 7, 8, 9, 10, 11 e 12.
(b) As probabilidades so diferentes, porque o nmero de casos favorveis varia. O resultado mais provvel o 7, que
pode ocorrer de 6 modos diferentes e que tem, portanto,
probabilidade 6/36 = 1/6. Os menos provveis so 2 e 12,
que s podem ocorrer de um modo e que tm, cada um,
probabilidade 1/36.
(c) A tabela abaixo tem todos os resultados possveis s + t,
onde s resultado do lanamento do primeiro dado e t, do
segundo. Colocamos os valores de s na primeira linha e os
de t na primeira coluna.

10

10 11

9 10 11 12

A probabilidade dos resultados 2, 12 e 7 j foi calculada. O resultado 3 tem probabilidade 2/36 = 1/18, assim
como o resultado 11. O resultado 4 tem probabilidade
3/36 = 1/12, assim como o resultado 10. O resultado 5
tem probabilidade 4/36 = 1/9, assim como o resultado
9. O resultado 6 tem probabilidade 5/36, assim como o
resultado 8.

i
i

contagemfran
2009/10/29
i page 63
Estilo OBMEP

i
i

i
Solues dos Exerccios

63

2) Vamos considerar todas as sequncias possveis de resultados.


Como em cada lanamento sai cara C ou coroa K, h 2 possibilidades; logo, o nmero de possibilidades igual a 222 = 8.
Todas as sequncias tm a mesma probabilidade de ocorrncia
igual a 1/8. Com duas caras temos CCK, CKC e KCC. Logo,
a probabilidade de que saiam duas caras 3/8.
3) So 2222=16 possveis sequncias para os sexos
das crianas, todas equiprovveis. Em 6 delas h 2 casais (HM HM , M HM H, HHM M , M M HH, M HHM ,
HM M H). Em 8 delas h 3 filhos de um sexo e um de outro
(M HHH, HM HH, HHM H, HHHM, M M M H, M M HM,
M HM M, HM M M ). Logo, mais provvel ter trs filhos de
um sexo e um de outro.
4) Em ambos os casos, Laura e Telma tm a mesma probabilidade
de tirar um nmero maior que o da outra. Se no h devoluo,
no pode haver empate, e a probabilidade de que Laura tenha
o maior nmero 50%. Se h devoluo, h possibilidade de
empate, e a probabilidade de que isso ocorra igual a 100 casos
de empate dividido por 100 100 casos possveis, que igual a
100
0, 01, ou seja,
= 0, 01. Logo, neste caso a probabili100 100
dade de que Laura tenha um nmero maior do que o de Telma
(1 0,01)/2 = 0,99/2 = 0,495.
5) Um domin tem 28 peas, como vimos no Captulo 1. Logo,
podemos selecionar duas peas, uma de cada vez, de 2827 modos. Se a primeira pea uma das 7 que so duplas, h 6 modos
de escolher a segunda de modo a conter o mesmo nmero (h,
no total, 7 peas em que esse nmero aparece). Se a primeira
pea uma das 21 que tm dois nmeros, a segunda pode ser

i
i

contagemfran
2009/10/29
i page 64
Estilo OBMEP

i
i

64
escolhida de 12 modos (6 para cada). Logo, a probabilidade
6 7 + 21 12
7
= .
28 27
18
6) A rvore de possibilidades abaixo mostra que o jogo pode terminar em 2 ou 3 lanamentos.

Ana

Carolina

Ana

Carolina

C
K

Joana

Ana s vence em dois casos (CC, com probabilidade 1/4 , e


KCC, com probabilidade 1/8); logo, tem probabilidade 3/8 de
vencer. Carolina vence se sai CK (probabilidade 1/4) ou KCK
(probabilidade 1/8); logo, tambm tem probabilidade 3/8 de
vencer. J Joana s vence se sair KK, que tem probabilidade
1/4.
7) Embora haja pessoas que ganhem a vida com esse tipo de afirmao, ela completamente sem sentido. As extraes so independentes, o que faz com que o fato de uma dezena estar
atrasada seja completamente irrelevante para o que vai acontecer no futuro. Na verdade, se estamos em dvida sobre a
equiprobabilidade das diversas dezenas, poderamos concluir
exatamente o contrrio: se uma dezena sai menos que outras,

i
i

contagemfran
2009/10/29
i page 65
Estilo OBMEP

i
i

i
Solues dos Exerccios

65

talvez seja porque seja menos provvel (por exemplo, a bolinha


correspondente pode ser maior ou mais leve que as outras).
8) Obviamente, os dois jogos tm a mesma probabilidade de serem
vitoriosos (mas voc acha que as pessoas, em geral, concordariam com isso? Por qu?).
9) O candidato deve trocar a porta. Se ele no o faz, sua chance
de vitria est em ter escolhido a porta certa da primeira vez,
o que ocorre com probabilidade 1/3. Trocando a porta, ele vai
ganhar o prmio exatamente nos casos em que a porta escolhida
a errada, o que tem probabilidade 2/3.

Captulo 3
As solues dos exerccios 1 a 9 esto no Captulo 2, pois so os
mesmos. A seguir, as solues dos exerccios 10 e 11 desse captulo.
10) Soluo rpida: Suponha que o sorteio feito com cada seleo retirando uma bola de uma urna, onde h quatro bolas de
mesma cor. Suponha, ainda, que o Brasil seja o primeiro, e a
Argentina, a segunda a retirar (isto no afeta a probabilidade
pedida; se voc no acredita nisso, veja a segunda soluo). Depois que o Brasil retirou sua bolinha, restam 15 bolas na urna,
3 das quais tm a mesma cor da retirada pelo Brasil. Logo, a
probabilidade de que a Argentina retire uma bola de mesma cor
3/15 = 1/5.
Soluo mais detalhada: O espao amostral das bolas retiradas

i
i

contagemfran
2009/10/29
i page 66
Estilo OBMEP

i
i

66
por Brasil e Argentina formado por todos os pares de bolas
distintas, que so 16 15 = 240. claro que todos os pares so
equiprovveis. Deles, os favorveis s duas selees ficarem no
mesmo grupo so aqueles em que as cores so iguais. Estes so
4 cores 4 bolas (Brasil) 3 bolas (Argentina) = 48. Logo, a
probabilidade de que eles estejam no mesmo grupo
48/240 = 1/5.

11) As sequncias possveis de filhos so H (prob. 1/2), M H (prob.


1/4) e M M (prob. 1/4). Logo, as famlias tm um filho do sexo
masculino com probabilidade 3/4. A probabilidade de que elas
tenham pelo menos um filho do sexo feminino 1/2. As famlias
tm um ou dois filhos com probabilidade 1/2 cada. Logo, em
mdia, elas tm 1, 5 filhos. O nmero de meninos 0 (com prob.
1/4) ou 1 (com prob. 3/4). Logo, em mdia h 0, 75 meninos por
famlia. Em consequncia, h tambm 0, 75 meninas em mdia
por famlia. Na verdade, isso era bvio: a poltica adotada no
modifica o fato de que os nascimentos so divididos igualmente
entre meninos e meninas.

Captulo 4
1) H P C5 = 4! modos de formar uma roda com as meninas. Depois disso, os 5 meninos devem ser postos nos 5 lugares entre
as meninas, o que pode ser feito de 5! modos. A resposta
4! 5! = 24 120 = 2 880.
2) mais simples calcular o nmero total de rodas e excluir aquelas

i
i

contagemfran
2009/10/29
i page 67
Estilo OBMEP

i
i

i
Solues dos Exerccios

67

em que Vera e Isadora ficam juntas. O nmero total de rodas


P C6 = 5! = 120. Para formar as rodas em que Vera e Isadora
ficam juntas, a primeira deciso a tomar a ordem em que
Vera e Isadora se colocaro na roda. H 2 possibilidades: VeraIsadora e Isadora-Vera. Agora tudo se passa como se Vera e
Isadora fossem uma nica criana. Assim, h
2(P C5 ) = 24! = 48 rodas em que Vera e Isadora ficam juntas.
A resposta 120 48 = 72 rodas.
5 modos de formar o Esporte; depois disso, C 5 modos de
3) H C15
10
formar o Tupi; finalmente, 1 nico modo de formar o Minas.
5 C 5 1 = 756 756.
A resposta C15
10

4) O nmero de possibilidades igual ao nmero obtido no problema anterior dividido por 3! = 6, j que permutando os
nomes dos times a subdiviso continua a mesma. A resposta
756 756/6 = 126 126.
5) Escolha, sucessivamente, 3 objetos para formar os 4 grupos de
3 , C3 , C3 e C3
3; isso pode ser feito, sucessivamente, de C20
17
14
11
modos. A seguir, com os 8 objetos restantes forme os 2 grupos
restantes, o que pode ser feito de C84 e C44 modos, respectivamente. Fazendo isso, contamos cada diviso 4!2! vezes, porque,
quando formamos os mesmos grupos de 3 e os mesmos grupos
4 em outra ordem, contamos como se fosse outra diviso em
grupos.
A resposta
3 C 3 C 3 C 3 C 4 .C 4
20!
C20
17
14
11
8
4
=
= 67 897 830 000.
4
4! 2!
(3!) (4!)2 4!2!

i
i

contagemfran
2009/10/29
i page 68
Estilo OBMEP

i
i

68
Outra soluo: Forme uma fila com as 20 pessoas. Isso automaticamente as divide em 4 grupos de 3 e 2 grupos de 4: as 3
primeiras formam um grupo, as 3 seguintes formam outro etc.
H 20! modos de formar a fila. Entretanto, uma mesma diviso
em grupos corresponde a vrias filas diferentes, o que faz com
que, no resultado 20!, cada diviso tenha sido contada vrias
vezes. Devemos corrigir nossa contagem dividindo o resultado
pelo nmero de vezes em que cada diviso foi contada. Trocando
a ordem dos elementos em cada grupo, o que pode ser feito de
3! 3! 3! 3! 4! 4! modos, ou a ordem dos grupos, o que pode
ser feito de 4! 2! modos, a diviso em grupos no se altera, mas
a fila sim. Cada diviso foi, assim, contada (3!)4 (4!)2 4! 2!
20!
.
vezes, e a resposta
4
(3!) (4!)2 4!2!
6) Os adversrios em cada jogo podem ser escolhidos, sucessiva2 , C 2 , C 2 , C 2 , C 2 e C 2 modos. No entanto, assim
mente, de C12
10
8
6
4
2
contamos cada possvel rodada 6! vezes, j que contamos diferentes ordens dos jogos como se fossem rodadas diferentes. A
resposta
2 C2 C2 C2 C2 C2
C12
12!
10
8
6
4
2
= 6
= 10 395.
6!
2 6!

Outra soluo: Colocando os 12 times em fila automaticamente


formamos os 6 jogos da rodada. No entanto, a mesma rodada
contada vrias vezes; os adversrios em cada jogo podem ser
ordenados de 2 modos, enquanto os jogos podem ser ordenados
12!
de 6! modos. A resposta , portanto, 6
.
2 6!
7) Em ESTRELADA as letras A e E aparecem 2 vezes cada, e
as letras S, T, R, L e D aparecem 1 vez cada uma, havendo,

i
i

contagemfran
2009/10/29
i page 69
Estilo OBMEP

i
i

69

Solues dos Exerccios

portanto, 9 letras na palavra.


Para formar um anagrama, devemos escolher 2 das 9 posies
para colocar as letras A, o que pode ser feito de C92 modos, 2
das 7 posies restantes para colocar as letras E, o que pode
ser feito de C72 modos, e arrumar as letras S, T, R, L e D nas 5
posies restantes, o que pode ser feito de 5! modos. A resposta
C92 C72 5! = 90 720.
Outra soluo: O nmero de anagramas
P92,2,1,1,1,1,1 =

9!
= 90 720.
2!2!1!1!1!1!1!

8) Vamos esquecer que a primeira casa do nmero no pode ser


igual a zero. Isso far com que contemos a mais e, depois,
descontaremos o que foi contado indevidamente.
H C73 modos de escolher as casas que sero ocupadas pelo algarismo 4; depois disso, h C42 modos de selecionar as casas
que sero ocupadas pelo algarismo 8; finalmente, as duas casas
restantes podem ser preenchidas de 8 8 modos (no podemos
usar nessas casas os algarismos 4 e 8).
A resposta seria C73 C42 8 8 = 35 6 64 = 13 440.
Devemos subtrair os nmeros comeados por 0. Se o nmero
comea por 0, h C63 modos de escolher as casas que sero ocupadas pelo algarismo 4; depois disso, h C32 modos de selecionar
as casas que sero ocupadas pelo algarismo 8; finalmente, a casa
restante pode ser preenchida de 8 modos (no podemos usar
nessa casa os algarismos 4 e 8). H C63 C32 8 = 2038 = 480
nmeros comeados por 0.
A resposta 13 440 480 = 12 960.

i
i

contagemfran
2009/10/29
i page 70
Estilo OBMEP

i
i

70
Outra soluo: Vamos contar separadamente:
i) nmeros que comeam em 4;
ii) nmeros que comeam em 8;
iii) nmeros que no comeam nem em 4 nem em 8.
i) H 1 modo de preencher a primeira casa; depois disso, h
C62 modos de escolher as outras duas casas do nmero que
tambm sero preenchidas com o algarismo 4; depois disso,
h C42 modos de escolher as duas casas que sero ocupadas
pelo algarismo 8; finalmente, as duas casas restantes podem ser preenchidas de 8 8 modos (no podemos usar
nessas casas os algarismos 4 e 8).
H 1 C62 C42 8 8 = 1 15 6 64 = 5 760 nmeros
do tipo i).
ii) H 1 modo de preencher a primeira casa; depois disso, h
6 modos de escolher a outra casa do nmero que tambm
ser preenchida com o algarismo 8; depois disso, h C53
modos de escolher as trs casas que sero ocupadas pelo
algarismo 4; finalmente, as duas casas restantes podem ser
preenchidas de 88 modos (no podemos usar nessas casas
os algarismos 4 e 8).
H 1 6 C53 8 8 = 6 10 64 = 3 840 nmeros do
tipo ii).
iii) H 7 modos de preencher a primeira casa (no podemos
usar nem 4, nem 8, nem 0); depois disso, h C63 modos
de escolher as trs casas do nmero que sero preenchidas
com o algarismo 4; depois disso, h C32 modos de escolher
as duas casas que sero ocupadas pelo algarismo 8; finalmente, a casa restante pode ser preenchida de 8 modos
(no podemos usar nessas casas os algarismos 4 e 8).

i
i

contagemfran
2009/10/29
i page 71
Estilo OBMEP

i
i

i
Solues dos Exerccios

71

H 7 C63 C32 8 = 7 20 3 8 = 3 360 nmeros do


tipo iii).
A resposta 5 760 + 3 840 + 3 360 = 12 960.

9) (a) Para formar o subconjunto devemos escolher os p1 outros


elementos do subconjunto dentre os n 1 outros elementos
do conjunto.
p1
A resposta Cn1
.

(b) Para formar o subconjunto devemos escolher os p elementos do subconjunto dentre os n 1 outros elementos do
conjunto.
p
A resposta Cn1
.

Outra soluo: H Cnp p-subconjuntos, ou seja, subconjunp1


tos com p elementos, e o elemento a1 figura em Cn1
deles.
p
p1
Logo, h Cn Cn1 subconjuntos nos quais o elemento a1
no figura.
p1
A resposta Cnp Cn1
.

Observao. As duas solues apresentadas mostram que


p1
p
Cnp Cn1
= Cn1
. Essa a famosa Relao de Stifel.
(c) Para formar o subconjunto devemos escolher os p2 outros
elementos do subconjunto dentre os n 2 outros elementos
do conjunto.
p2
A resposta Cn2
.

(d) O total de p-subconjuntos Cnp . Para formar um subconjunto em que nem a1 nem a2 figurem devemos escolher os p
elementos do subconjunto dentre os n 2 outros elementos
p
do conjunto. H, portanto, Cn2
subconjuntos nos quais
nem a1 nem a2 figuram. Logo, o nmero de subconjuntos

i
i

contagemfran
2009/10/29
i page 72
Estilo OBMEP

i
i

72
nos quais pelo menos um desses dois elementos figura
p
Cnp Cn2
.
p1
Outra soluo: H Cn1
p-subconjuntos nos quais o elep1
mento a1 figura e h Cn1 subconjuntos nos quais o elep2
mento a2 figura. H, tambm, Cn2
p-subconjuntos nos
quais os elementos a1 e a2 figuram ambos. Ao somarmos
p1
p1
p1
Cn1
+ Cn1
= 2Cn1
obtemos o nmero de subconjuntos nos quais pelo menos um dos elementos a1 e a2 figura,
mas contamos duas vezes aqueles em que a1 e a2 figuram
ambos.
p1
p2
A resposta , portanto, 2Cn1
Cn2
.
p2
Outra soluo: H, como mostrado em (c), Cn2
psubconjuntos em que os elementos a1 e a2 figuram ambos.
p1
H Cn2
p-subconjuntos em que o elemento a1 figura e
o elemento a2 no figura, pois, para formar um tal subconjunto, basta escolher os outros p 1 elementos do subconjunto dentre os n 2 elementos do conjunto que so
diferentes de a1 e de a2 .
p1
H,
analogamente,
Cn2
p-subconjuntos
em
que
o
elemento
a2
figura
e
o
elemento
a1 no figura.
Portanto,
o nmero de
p-subconjuntos em que figura pelo menos um desses
p1
p2
dois elementos 2Cn2
+ Cn2
.
p1
(e) Como visto na soluo anterior, a resposta 2Cn2
.
p1
p2
Outra soluo: H, como visto em (d), 2Cn1
Cn2
psubconjuntos nos quais pelo menos um dos elementos a1 e
p2
a2 figura. H, como visto em (c), Cn2
p-subconjuntos em
que os elementos a1 e a2 figuram ambos.

i
i

contagemfran
2009/10/29
i page 73
Estilo OBMEP

i
i

73

Solues dos Exerccios

A resposta , portanto,
p1
p2
p2
p1
p2
2Cn1
Cn2
Cn2
= 2Cn1
2Cn2
.
p1
p2
Outra soluo: H, como visto em (d), 2Cn2
+ Cn2
psubconjuntos nos quais pelo menos um dos elementos a1 e
p2
a2 figura. H, como visto em (c), Cn2
p-subconjuntos em
que os elementos a1 e a2 figuram ambos.

A resposta , portanto,
p1
p2
p2
p1
2Cn2
+ Cn2
Cn2
= 2Cn2
.

10) Chamemos de D o conjunto C C1 .


H quatro tipos de planos:
i) determinados por trs pontos de D;
ii) determinados por dois pontos de D e um de C1 ;
iii) determinados por um ponto de D e dois de C1 ;
iv) determinados por trs pontos de C1 .
3 + (C 2 ) 8 + 12 C 2 + 1 = 1 085.
A resposta C12
12
8

Outra soluo: Para determinar um plano, devemos selecionar 3


3 = 1 140 modos. Nessa
dos 20 pontos, o que pode ser feito de C20
contagem, o plano que contm os 8 pontos de C1 foi contado
C83 = 56 vezes.
A resposta 1 140 56 + 1 = 1 085.
11) Primeiro, colocamos as vogais. Como a letra A aparece 3 vezes
e as letras U, I e O aparecem 1 vez cada, o nmero de modos

i
i

contagemfran
2009/10/29
i page 74
Estilo OBMEP

i
i

74
6!
720
de disp-las P63,1,1,1 =
=
= 120. A seguir, colocamos
3!
6
as consoantes em trs dos 7 espaos antes, entre e depois das
vogais. O lugar do P pode ser qualquer um desses 7 espaos, o do
R, qualquer dos 6 restantes, e o do G, qualquer dos 5 restantes.
O nmero total de possibilidades 120 7 6 5 = 25 200.
12) Vamos formar uma fila com os nmeros 1, 2, . . . , n e assinalar
com E os p nmeros escolhidos e com N os n p no escolhidos.
A condio para que no sejam escolhidos nmeros consecutivos que entre dois E haja pelo menos um N. Comeamos
escrevendo os n p E. A seguir, devemos escolher, para colocar
os E, p dentre os n p + 1 espaos situados antes, entre e depois
p
dos N. Isso pode ser feito de Cnp+1
modos.
13) (a) Nenhum aluno pode comparecer a mais de trs jantares.
Com efeito, se A1 vai a um jantar com A2 e A3 , ele s pode
ir a outro jantar com outros dois estudantes, digamos A4
e A5 e s pode ir a um terceiro jantar em companhia de
outros dois, digamos A6 e A7 e no ter companhia para
ir a um quarto jantar. Como h 21 convites e so 7 estudantes, cada estudante ter que comparecer a exatamente
3 jantares.
(b) Se A1 comparece a trs jantares, podemos escolher os seus
companheiros dividindo os outros 6 estudantes em 3 grupos
C 2 C42 1
de 2, o que pode ser feito de 6
= 15 modos.
3!
Ento, os 3 jantares so, digamos, A1 A2 A3 , A1 A4 A5 e
A1 A6 A7 .
A2 dever comparecer a mais dois jantares, nenhum deles
em companhia de A3 , e A3 tambm dever comparecer a
mais dois jantares. Portanto, os 4 jantares que faltam so:

i
i

contagemfran
2009/10/29
i page 75
Estilo OBMEP

i
i

75

Solues dos Exerccios

A2 __, A2 __, A3 __, A3 __.


Como A4 deve comparecer a mais dois jantares (A4 no
pode comparecer a ambos em companhia de A2 nem a
ambos em companhia de A3 ), esses quatro jantares so:
A2 A4 _, A2 __, A3 A4 _, A3 __.
A5 tem que comparecer ainda a dois jantares, nenhum deles
em companhia de A4 .
A2 A4 _, A2 A5 _, A3 A4 _, A3 A5 _.
Agora h duas possibilidades:
A2 A4 A6 , A2 A5 A7 , A3 A4 A7 , A3 A5 A6 e
A2 A4 A7 , A2 A5 A6 , A3 A4 A6 , A3 A5 A7 .
H, portanto, 15 2 = 30 maneiras de escolher os grupos
de convidados.
Para distribuir os 7 grupos nos 7 dias, h 7! alternativas.
A resposta 7! 30 = 151 200.
14) (a) Cada professor fica caracterizado pelas duas bancas a que
pertence. O nmero de professores igual ao nmero de
modos de escolher duas das oito bancas.
A resposta C82 = 28.
(b) O nmero de professores pertencentes a uma banca igual
ao nmero de modos de escolher a outra banca a que ele
pertence.
A resposta 7.
15) Chamando x de 1 + a, y de 1 + b e z de 1 + c, o problema se
transforma em encontrar todas as solues inteiras e no negativas de (a + 1) + (b + 1) + (c + 1) = 7, ou seja, de a + b + c = 4.
A resposta CR34 = C64 = 15.

i
i

contagemfran1
2009/10/29
i page 76
Estilo OBMEP
i

i
i

76
16) Cada soluo inteira e no negativa de x+y +z 6 corresponde
a uma soluo inteira e no negativa da equao x+y+z+f = 6.
Logo, h CR46 = C96 = 84 solues.
17) Para formar uma caixa, devemos selecionar 20 dentre os 5 tipos,
valendo repetio na escolha. Ou seja, devemos formar solues
inteiras e no negativas de x1 + x2 + x3 + x4 + x5 = 20, onde xi
o nmero de bombons do tipo i. A resposta
20
CR520 = C24
= 10 626.

Captulo 5

1) Sejam X e Y os resultados do primeiro e segundo lanamentos,


respectivamente.
P (X = 3 | X + Y = 7) =

P (X = 3, X + Y = 7)
P (X + Y = 7)
1
1/6 1/6
= .
6/36
6

Outra soluo: Se a soma 7, h 6 casos possveis igualmente


provveis: X = 1, Y = 6; X = 2, Y = 5; X = 3, Y = 4; X = 4,
Y = 3; X = 5, Y = 2; X = 6, Y = 1. Dos seis casos, somente
1
X = 3, Y = 4 favorvel. A resposta .
6
2) P (v vermelha | mostra amarela) =
=

P (v vermelha e mostra amarela)


1/6
1
=
=
P (mostra amarela)
1/2
3

i
i

contagemfran
2009/10/29
i page 77
Estilo OBMEP

i
i

77

Solues dos Exerccios

3) P (doente | positivo) =

P (doente e positivo)
=
P (positivo)

P (doente) P (positivo | doente)


=
P (doente) P (positivo | doente) + P (sadio) P (positivo | sadio)

0,005 0,95
95
=
= 0,3231
0,005 0,95 + 0,995 0,01
294

4) A
de no obter nenhum seis em n lanamentos

probabilidade

5 n
5 n
, e a de obter pelo menos um seis 1
.
6
6
n
n
5
5
Devemos ter 1
> 0, 9, ou seja,
< 0, 1. Da,
6
6
n
5
ln
<
6
5
<
n ln
6
n >

ln 0,1
ln 0,1
ln 0,1
= 12,6.
5
ln
6

A resposta 13.
5) Considere os eventos:
A = {A falou a verdade};
B = {B disse que A falou a verdade};
C = {C disse que B disse que A falou a verdade};
D = {D disse que C disse que B disse que A falou a verdade}.
Vamos aliviar a notao escrevendo XY para representar X Y .
P (AD)
Queremos calcular P (A | D) =
.
P (D)

i
i

contagemfran
2009/10/29
i page 78
Estilo OBMEP

i
i

78

P (AD) = P (ABCD) + P (ABCD) + P (ABCD) + P (ABCD)


=

1 1 1 1 1 2 2 1 1 1 2 2 1 2 1 2
+ + +
3 3 3 3 3 3 3 3 3 3 3 3 3 3 3 3

13
.
81

P (AD) = P (ABCD) + P (ABCD) + P (ABCD) + P (ABCD)


=

2 2 1 1 2 1 2 1 2 2 2 2 2 1 1 2
+ + +
3 3 3 3 3 3 3 3 3 3 3 3 3 3 3 3

28
.
81

P (D) = P (AD) + P (AD) =


A resposta P (A | D) =

41
13 28
+
= .
81 81
81

P (AD)
13/81
13
=
= .
P (D)
41/81
41

6) (a) A probabilidade de eles se enfrentarem na primeira


1
rodada n
porque, posto A na tabela, h 2n 1
2 1
posies possveis para B, e em 1 delas ele enfrenta
B. A probabilidade de eles se enfrentarem na segunda
2
2
1
1
1
rodada n

= n
, porque, posto A
2 1
2
2 1 2
na tabela, h 2n 1 posies possveis para B, e em
2 delas ele pode vir a enfrentar B na segunda rodada,
desde que, naturalmente, A e B venam seus jogos da
1 1
primeira rodada, o que ocorre com probabilidade . A
2 2
probabilidade de eles se enfrentarem na terceira rodada
2
22
1
1
1

= n

etc.
2n 1
4
2 1 22
A resposta

i
i

contagemfran
2009/10/29
i page 79
Estilo OBMEP

i
i

79

Solues dos Exerccios


1
1
1
1
1
1
1
2n 1 + 2n 1 2 + 2n 1 22 + + 2n 1 2n1
1
.
2n1

1 n

1( 2 )
1
2n 1 1 1

(b) Se k < n, o jogador disputa exatamente k partidas se,


e somente se, perde a k-sima partida e ganha as k 1
partidas anteriores. A probabilidade de isso acontecer
k1
1
1
1
= k.
2
2
2
O jogador disputa n partidas ou seja, chega final
se, e somente se, ganha as n 1 partidas anteriores. A
n1
1
1
= n1 .
probabilidade de isso acontecer
2
2
A resposta

1
1
, se k < n; n1 , se k = n.
k
2
2

7)
P (A | defeituosa) =

P (A e defeituosa)
P (defeituosa)

P (A)P (defeituosa|A)
P (A)P (defeituosa|A)+P (B)P (defeituosa|B)

(1/3)0,03
(1/3)0,03+(2/3)0,01

3
5

8) Uma urna recebe uma bola branca e a outra urna recebe as


demais 99 bolas. Com efeito, se a 1a urna recebe k bolas das
quais a so brancas, a probabilidade de libertao

1 a
50 a
1 50k + a(100 2k)
f (a, k) =
+
=
.
2 k 100 k
2
k(100 k)
1
Observe que para k = 50 a expresso vale , independentemente
2
do valor de a.

i
i

contagemfran
2009/10/29
i page 80
Estilo OBMEP

i
i

80
Observe tambm que basta estudar agora o caso k < 50 (isto ,
podemos considerar a primeira urna como sendo a que recebeu
menos bolas). Nesse caso, claro que, fixado o valor de k,
quanto maior for a, maior ser f (a, k). Logo, para f (a, k) ser
mximo, devemos ter a = k, e a probabilidade ser
g(k) =

1 150 2k
75 k
25

=
=1
,
2 100 k
100 k
100 k

que mxima para k mnimo.


Devemos, pois, ter k = 1, o que d uma probabilidade de liber74
tao de
= 0, 75.
99

Exerccios Adicionais
1) (a) AB, AC, AD, BA, BC, BD, CA, CB, CD, DA, DB, DC.
(b) O lder pode ser escolhido de 4 modos; uma vez escolhido o
lder, o vice-lder pode ser escolhido de 3 modos. O nmero
total de possibilidades 4 3 = 12.
2) As filas em que Helena e Pedro esto juntos so 2! 7! = 10 080.
As filas em que Helena e Pedro esto juntos e Vera e Paulo
tambm esto juntos so em nmero de 2! 2! 6! = 2 880. A
resposta 10 080 2 880 = 7 200.
3) (a) Para descobrir o lugar do 62 417 voc tem que contar
quantos so os nmeros que o antecedem. Antecedem-no todos os nmeros comeados em 1 (4! = 24 nmeros),
em 2 (4! = 24 nmeros), em 4 (4! = 24 nmeros), em
61(3! = 6 nmeros) e em 621 (2! = 2 nmeros), num total

i
i

contagemfran
2009/10/29
i page 81
Estilo OBMEP

i
i

i
Solues dos Exerccios

81

de 24 + 24 + 24 + 6 + 2 = 80 nmeros. Ele ocupa o 81o


lugar.
(b) Ao escrever os nmeros comeados por 1, escrevemos
4! = 24 nmeros; incluindo agora os comeados por 2,
teremos mais 4! = 24 nmeros, acumulando um total de
48 nmeros; incluindo agora os comeados por 41, 42 e
46, teremos mais 3! + 3! + 3! = 18 nmeros, acumulando
um total de 66 nmeros. O 66o nmero o ltimo dos
comeados por 46, ou seja, 46 721.
(c) Como em cada nmero h 5 algarismos e 166 = 533+1, o
166o algarismo escrito o 1o algarismo do 34o nmero. Ao
escrever os nmeros comeados por 1, escrevemos 4! = 24
nmeros; incluindo agora os comeados por 2, teremos mais
4! = 24 nmeros, acumulando um total de 48 nmeros.
Logo, todos os nmeros do 25o ao 48o , inclusive, comeam
por 2. A resposta 2.
4) Contaremos separadamente os casos em que a carta de copas
um rei e em que a carta de copas no um rei. A resposta
1 48 + 12 47 = 612.
5) H 3 modos de escolher os dias de Matemtica. Escolhidos os
dias, digamos segundas e quartas, h 2 modos de escolher o
horrio da aula de Matemtica da segunda e 2 modos de escolher o horrio da aula de Matemtica da quarta. H 2 modos
de escolher os dias da Fsica (no podem ser os mesmos da
Matemtica, seno a Qumica ficaria com as aulas no mesmo
dia). Escolhidos os dias da Fsica, em um deles h 2 modos de
escolher o horrio da aula e, no outro, apenas 1. Finalmente,
h apenas 1 modo de pr as aulas de Qumica no horrio. A
resposta 3 2 2 2 2 1 1 = 48.
6) A torre branca pode ser colocada em qualquer uma das 64 casas.

i
i

contagemfran
2009/10/29
i page 82
Estilo OBMEP

i
i

82
H um total de 15 casas que esto na mesma linha ou coluna
em que ela foi colocada. A torre preta pode estar em qualquer
uma das 64 15 = 49 casas restantes. Logo, o nmero de
possibilidades 64 49 = 3 136.
7) (a) 7! = 7 6 5 4 3 2 1 = 5 040.
(b) 6! = 720.
(c) A vogal final pode ser A, E, I ou O (4 possibilidades). Para
as primeiras 6 letras h 6! possibilidades. Logo, o nmero
de anagramas terminados com vogal 4 6! = 2 880.
(d) Tudo se passa como se VEIR fosse uma nica letra (digamos ). Assim, o problema se reduz a encontrar o nmero
de anagramas de SAO, que igual a 4! = 24.
8) O par do primeiro homem pode ser escolhido de 5 modos, do
segundo, de 4 e assim por diante, para um total de
5 4 3 2 1 = 120 possibilidades.
No segundo caso, a resposta 8 7 6 5 4 = 6 720. (Se considerar que os casais devem ser dispostos na quadrilha, o nmero
de possibilidades, em ambos os casos, deve ser multiplicado por
5! ou por 5!25, conforme a interpretao.)
9) A primeira mulher pode escolher sua posio de 10 modos. A
segunda, de 8 modos. As outras, de 6, de 4 e de 2 modos. O
primeiro homem, de 5 modos. Os demais, de 4, de 3, de 2, de
1. A resposta 10 8 6 4 2 5 4 3 2 1 = 460 800.
10) (a) No mnimo devem ser usadas 3 cores (duas, no mnimo,
para a parte central e pelo menos mais uma para as laterais).

i
i

contagemfran
2009/10/29
i page 83
Estilo OBMEP

i
i

i
Solues dos Exerccios

83

(b) A faixa do topo pode ser pintada de 6 modos, a do meio,


de 5, a de baixo, outra vez de 5 modos. Mas o nmero
de possibilidades para as faixas laterais depende de termos
usado 2 ou 3 cores para as faixas horizontais. No possvel, assim, usar diretamente o princpio multiplicativo.
Vamos dividir a contagem em dois casos:
i) 3 cores so utilizadas para a parte central: neste
caso, a faixa de cima pode ser pintada de 6 modos, a do
meio, de 5, e a de baixo, de 4 modos. Para a faixa da
esquerda temos 3 possibilidades, o mesmo ocorrendo
com a da direita. So, portanto, 65433 = 1 080
modos.
ii) 2 cores so utilizadas para a parte central: neste
caso, as faixas de cima e de baixo tm a mesma cor, que
pode ser escolhida de 6 modos. A faixa central pode
ser escolhida de 5 modos, e a cor de cada faixa lateral,
de 4 modos. Logo, o nmero de possibilidades neste
caso 6544 = 480 modos. Logo, o nmero total
de modos de pintar a bandeira 1 080 + 480 = 1 560.
11) O primeiro quadrante pode ser pintado de 6 modos, o segundo,
de 5, e o terceiro, novamente de 5 modos. Mas o nmero de
modos de pintar o quarto quadrante vai depender de termos
usado ou no a mesma cor para o primeiro e terceiro quadrantes.
Portanto, outra vez temos que dividir em casos:
i) cores distintas so usadas para o primeiro e terceiro
quadrantes: neste caso, a cor do primeiro quadrante pode
ser escolhida de 6 modos, a do segundo, de 5, a do terceiro,
de 4 (tem que ser diferente das duas anteriores), e a do
quarto, tambm de 4. Logo, o nmero de possibilidades
6 5 4 4 = 480.

i
i

contagemfran
2009/10/29
i page 84
Estilo OBMEP

i
i

84
ii) a mesma cor usada para o primeiro e terceiro
quadrantes: neste caso, essa cor comum pode ser escolhida de 6 modos, e as cores do segundo e quarto quadrantes
podem ser escolhidas de 5 modos cada. Logo, o nmero de
possibilidades 6 5 5 = 150. Portanto, o nmero total
de possibilidades 480 + 150 = 730.
12) (a) Devemos colocar 6 nmeros em 6 lugares. A resposta
6! = 720.
(b) Agora, quando mudamos o cubo de posio, obtemos o
mesmo dado. Por exemplo, um dado que tem o 1 e o 6 em
faces opostas. Antes, colocar o 1 em cima, na face preta,
e o 6 em baixo, na face branca, era diferente de colocar o
6 em cima e o 1 embaixo. Agora no, o mesmo dado de
cabea para baixo. A resposta a anterior dividida pelo
nmero de posies de colocar um cubo. Como h 6 modos
de escolher a face que fica em baixo e 4 modos de escolher
nessa face a aresta que fica de frente, so 6 4 = 24 as
posies de colocar um cubo. A resposta 720/24 = 30.
13) A ltima vaga a ser ocupada necessariamente uma das duas
extremas (h possibilidades, portanto). A penltima uma das
vagas extremas, ou a vaga adjacente outra (2 possibilidades,
de novo). De modo geral, para cada carro, exceto o primeiro, h
2 possibilidades. O nmero total de modos de ocupar as vagas
, portanto, 29 = 512.
14) O espao amostral, neste caso, o conjunto de todas as possveis
ordenaes dos papis. O nmero de vezes em que o papel
premiado aparece em cada posio o mesmo. Logo, as chances
de premiao so iguais, qualquer que seja a ordem em que os
papis so sorteados.

i
i

contagemfran
2009/10/29
i page 85
Estilo OBMEP

i
i

i
Solues dos Exerccios

85

15) (a) 20 19 18 = 6 840.


(b) A resposta no a mesma porque cada comisso de 3 membros corresponde a 6 modos diferentes para escolher representante, secretrio e tesoureiro.
(c) Dividir o resultado em (a) por 6. Portanto, o nmero de
comisses 6 840/6 = 1 140.
16) H 2 2 2 2 = 16 possibilidades para os sexos dos filhos.
(a) Dos 16 casos possveis, em apenas 1 so todas meninas.
Logo, em 15 casos h pelo menos um menino, e a probabilidade correspondente 15/16.
(b) H 1 caso em que os filhos so todos do sexo masculino e
1 caso em que so todos do sexo feminino. Logo, h 14
casos em que h filhos de ambos os sexos. A probabilidade
correspondente 14/16 = 7/8.
(c) Os possveis casos so 6: HHMM, HMHM, HMMH,
MHHM, MHMH, MMHH (que correspondem a). Logo,
a probabilidade de que os filhos formem 2 casais
6/16 = 3/8.
17) (a) Os professores de Clculo e lgebra Linear podem escolher
seus dias de provas de 55 = 25 modos. Em 5 desses casos,
as provas caem no mesmo dia. Logo, a probabilidade de
que as provas sejam marcadas para o mesmo dia igual
a 5/25 = 1/5. Outro raciocnio: uma vez que o professor
de Clculo tenha marcado sua prova, a chance de que o de
lgebra Linear escolha o mesmo dia 1/5.
(b) O nmero total de escolhas para os dias de prova
5555 = 625. O nmero de modos de marcar as provas
sem que caiam duas no mesmo dia 5 4 3 2 = 120
(o primeiro professor pode escolher qualquer dos 5 dias, o

i
i

contagemfran
2009/10/29
i page 86
Estilo OBMEP

i
i

86
segundo, um dos 4 restantes e assim por diante). Logo, a
probabilidade de que as provas caiam em dias distintos
120/625 = 24/125.
18) Suponha o time A posicionado em seu grupo. B ter 23 posies
possveis, em 11 das quais fica no grupo de A. A resposta
11/23.
6 . Para
19) O nmero de modos de selecionar 2 ps de sapatos C12
selecionar 1 par de sapatos devemos selecionar um dos 6 pares.
A probabilidade de que se forme um par igual a 6/66 = 1/11.
4 = 17 750.
20) (a) C27

(b) Como Jos j est escolhido, devemos escolher 3 pessoas


3 = 2 600.
dentre as 26 que sobraram. A resposta C26
(c) Como Mrcia no pode ser escolhida, devemos escolher 4
4 = 14 950.
dentre 26 pessoas. A resposta C26
2 .C 2 = 6 930.
(d) C12
15
1 .C 3 + C 2 .C 2 + C 3 .C 1 = 15 690.
(e) C12
15
12 15
12 15
6 = 50 063 860.
21) (a) C60

(b) Em ambos os casos, a probabilidade de ganhar


1/50 063 860.
(c) Quem aposta em 8 nmeros, aposta em C86 = 56 resultados. Logo, as chances de ganhar so 56 vezes maiores.
(d) No.
22) O nmero de resultados possveis 65 .
(a) Para formar um par, deve-se primeiramente selecionar o
tipo do par (6 modos), depois, os dados em que o par se
formar (C52 = 10 modos) e, finalmente, os resultados dos

i
i

contagemfran
2009/10/29
i page 87
Estilo OBMEP

i
i

87

Solues dos Exerccios

outros trs dados (5 4 3 = 60 modos). A resposta


6 10 60
25
= .
5
6
54
(b) Para formar dois pares, deve-se primeiramente selecionar
os tipos dos pares (C62 = 15 modos), depois os dados em
que os pares se formaro (C52 .C32 = 30 modos) e, finalmente, o resultado do outro dado (4 modos). A resposta
15 30 4
25
=
.
5
6
108
(c) Para formar uma trinca, deve-se primeiramente selecionar
o tipo da trinca (6 modos), depois, os dados em que a trinca
se formar (C53 = 10 modos) e, finalmente, os resultados
dos outros dois dados (5 4 = 20 modos). A resposta
6 10 20
25
.
=
65
162
(d) Para formar uma quadra, deve-se primeiramente selecionar
o tipo da quadra (6 modos), depois, os dados em que a
quadra se formar ( = 5 modos) e, finalmente, os resultados
do outro dado (C54 = 5 modos). A resposta
655
25
.
=
5
6
1 296
6
1
.
=
65
1 296
(f) H dois tipos de sequncias (12345 e 23456). Para formar uma delas, basta escolher o resultado de cada dado
2 120
5
=
.
(54321 = 120 modos). A resposta
65
162
(g) Para formar um full hand, deve-se primeiramente selecionar o tipo da trinca (6 modos), depois, os dados em
que a trinca se formar (C53 = 10 modos) e, finalmente, o
6 10 5
25
tipo do par (5 modos). A resposta
=
.
5
6
648
23) O nmero de casos possveis para os signos
(e) H apenas 6 quinas. A resposta

12 12 12 12 = 124 .

i
i

contagemfran
2009/10/29
i page 88
Estilo OBMEP

i
i

88
(a) O nmero de casos em que os signos so diferentes
12 11 10 9. Logo, a probabilidade de haver alguma
41
12.11.10.9
= .
coincidncia de signos zodiacais 1
4
12
96
(b) As trs pessoas podem ser selecionadas de C43 = 4 modos;
o signo delas, de 12 modos; o signo da pessoa restante, de
11
4 12 11
=
11 modos. A resposta
.
124
432
(c) H 12 casos em que as quatro pessoas tm o mesmo signo.
12
1
.
A resposta 4 =
12
1 728
(d) Para que haja duas pessoas com um mesmo signo e duas
outras pessoas com outro signo, os signos podem ser se2 = 66 modos; depois, os pares de cada
lecionados de C12
signo podem ser selecionados de C42 = 6 modos. A respos66 6
11
ta
.
=
4
12
576
24) (a) O segundo melhor jogador ser vice-campeo se, e somente
se, no enfrentar o melhor jogador antes da final. Posto o
segundo melhor jogador na tabela, h 15 posies possveis
para o melhor, e em 8 delas ele enfrenta o segundo melhor
jogador apenas na final. A resposta 8/15.
(b) O quarto melhor jogador ser vice-campeo se, e somente
se, no enfrentar nenhum dos trs melhores jogadores antes
da final. Posto o quarto melhor jogador na tabela, h 15
posies possveis para os melhores que ele, em 8 das quais
eles s enfrentaro o quarto melhor jogador na final. A
C3
8
resposta 38 = .
65
C15
(c) O nmero mximo 3. Suponhamos os 16 jogadores numerados de 1 a 16 e os jogos sendo: primeira fase: 1 2, 3
4, ..., 15, 16; segunda fase: vencedor do jogo 1 vencedor do 2, ... vencedor do jogo 7 vencedor do 8. H 6
jogadores piores que ele. Se ele ocupa a posio 1, devem

i
i

contagemfran
2009/10/29
i page 89
Estilo OBMEP

i
i

89

Solues dos Exerccios

ser ocupadas por piores que ele as posies 2 (para que


ele passe para a segunda fase), 3 e 4 (para que ele passe
para a terceira fase); para que ele passe para a quarta fase,
as posies 5, 6, 7 e 8 tambm devem ser ocupadas por
jogadores piores que ele, o que impossvel.
(d) A probabilidade de ele disputar 3 partidas a probabilidade de as posies 2, 3 e 4 serem ocupadas por jogadores
C3
4
piores que ele, que igual a 36 = .
91
C15
25) (a) Cada lanamento possui 6 resultados possveis. Assim,
h 6 6 = 36 resultados possveis para os resultados do
1o e 3o dados (o 2o no importa aqui). Desses, h 22 = 4
casos em que em ambos os casos sai uma face azul e
4 4 = 16 casos em que as faces so ambas vermelhas.
Logo, a probabilidade de que as faces tenham a mesma cor
5
4 + 16
no 1o e 3o lanamentos
= .
36
9
(b) O nmero total de resultados em que o 1o e 2o dados
fornecem o mesmo resultado igual a
2 2 6 + 4 4 6 = 120
(a primeira parcela corresponde situao em que as
duas primeiras faces so vermelhas, e a segunda, situao em que as duas primeiras faces so azuis). O
nmero de resultados em que as 3 faces tm a mesma cor
2 2 2 + 4 4 4 = 72. Logo, a probabilidade pedida
72
3

= (note que esta probabilidade maior do que a


120
5
do item anterior).
26) A funo fica determinada quando se escolhem os m elementos de In que formaro a imagem, o que pode ser feito de Cnm
m
maneiras, no
maneiras, no primeiro caso, e de CRnm = Cn+m1

i
i

contagemfran
2009/10/29
i page 90
Estilo OBMEP

i
i

90
segundo caso.
27) Ignoremos o problema do 0 na primeira casa. H C73 = 35 modos
de escolher os lugares dos algarismos 4, C42 = 6 de escolher os
lugares dos 8, e 88 = 64 modos de preencher as casas restantes,
num total de 35 6 64 = 13 440 nmeros. Devemos descontar
os nmeros comeados em 0. H C63 = 20 modos de escolher os
lugares dos algarismos 4, C32 = 3 de escolher os lugares dos 8, e 8
modos de preencher a casa restante, num total de 2038 = 480
nmeros comeados em 0. A resposta 13 440 480 = 12 960.
28) (a) Essas funes so bijetoras. A resposta n!.
(b) Um elemento de B tem sua imagem inversa formada por
dois elementos, e os demais tm imagens inversas unitrias.
Esse elemento de B pode ser selecionado de n modos, e sua
2
imagem inversa, de Cn+1
modos. Depois disso, h (n 1)!
modos de determinar as imagens dos demais elementos de
A, pois a correspondncia entre esses elementos restantes
em A e B biunvoca. A resposta
2
n.Cn+1
.(n 1)! =

(n + 1)!n
.
2

(c) Neste caso, temos as alternativas:


i) Trs elementos de A tm a mesma imagem em B, e a
correspondncia entre os demais n1 elementos de A e
3
os demais n 1 elementos de B biunvoca. H Cn+2
modos de escolher os trs elementos de A, n modos
de escolher a imagem deles em B e (n 1)! modos
de construir uma correspondncia biunvoca entre os
3 .n.(n 1)! = n(n + 2)!
elementos restantes. H Cn+2
6
funes desse tipo.

i
i

contagemfran
2009/10/29
i page 91
Estilo OBMEP

i
i

i
Solues dos Exerccios

91

ii) H dois pares de elementos de A com imagens idnticas em B, e a correspondncia entre os demais
n 2 elementos de A e os demais n 2 elementos
de B biunvoca. H Cn2 modos de escolher os dois
2 .C 2 modos de escolher suas
elementos de B, Cn+2
n
imagens inversas em A e (n 2)! modos estabelecer a correspondncia entre os elementos restantes.
2 .C 2 .(n 2)! = n(3n + 1)(n + 2)! funes
H Cn2 .Cn+2
n
24
desse tipo.
A resposta
n(n + 2)! n(3n + 1)(n + 2)!
n(3n + 1)(n + 2)!
+
=
.
6
24
24
29) Chamemos de D o conjunto C C1 . H quatro tipos de planos,
determinados por:
i) trs pontos de D;
ii) dois pontos de D e um de C1 ;
iii) um ponto de D e dois de C1 ;
iv) trs pontos de C1 .
3 + 8.C 2 + 12.C 2 + 1 = 1 085.
A resposta C12
12
8

30) Chegam 4 cientistas A, B, C, D. Com as chaves que possuem,


abrem alguns cadeados, mas no todos. Existe pelo menos um
cadeado que eles no conseguem abrir. Na situao do nmero
mnimo de cadeados, existe exatamente um cadeado que eles
no conseguem abrir. Batize tal cadeado de ABCD. Portanto,
ABCD o cadeado cuja chave no est em poder de A, nem
de B, nem de C e nem de D. Qualquer outro cientista tem a
chave desse cadeado, pois esse cientista e A, B, C e D formam
um grupo de 5 cientistas e, portanto, nesse grupo algum possui

i
i

contagemfran
2009/10/29
i page 92
Estilo OBMEP

i
i

92
a chave. Como o algum no nem A, nem B, nem C e nem
D, deve ser o outro. Batize, analogamente, os demais cadeados. Verifique agora que a correspondncia entre cadeados e
seus nomes biunvoca, isto , cadeados diferentes tm nomes
diferentes (isso porque estamos na situao do nmero mnimo
de cadeados) e cadeados de nomes diferentes so diferentes (se
X est no nome de um cadeado e no est no nome do outro, X
tem a chave deste e no tem a chave daquele).
(a) O nmero mnimo de cadeados igual ao nmero de nomes
4 = 330.
de cadeados, C11
(b) Cada cientista X possui as chaves dos cadeados que no
4 = 210.
possuem X no nome. A resposta C10

i
i

lp_pitagoras
2009/8/17
i page 1
Estilo OBMEP

i
i

Teorema de Pitgoras e
reas
Eduardo Wagner

i
i

lp_pitagoras
2009/8/17
i page 2
Estilo OBMEP

i
i

Texto j revisado pela nova ortografia.

i
i

lp_pitagoras
2009/8/17
i page 3
Estilo OBMEP

i
i

Sobre o Autor
Eduardo Wagner formado em engenharia pela UFRJ e mestre em
matemtica pelo IMPA. Como professor de matemtica, atua tanto no
Ensino Mdio quanto no superior. Suas atividades com olimpadas de
Matemtica comearam em 1989, tendo sido coordenador da Olimpada de Matemtica do Estado do Rio de Janeiro por trs anos e da
Olimpada Brasileira de Matemtica por cinco anos, participando
at hoje como membro da Comisso da OBM, da sua organizao
e desenvolvimento. Tambm tem desempenhando a funo de lder
da delegao brasileira em diversas olimpadas internacionais. Desde
1991 professor do Programa de Aperfeioamento de Professores promovido pelo IMPA e tambm membro do Comit Editorial da Revista do Professor de Matemtica publicada pela SBM. autor de
diversos livros dentre os quais seis volumes da Coleo do professor
de Matemtica, publicada pela SBM e de uma extensa coleo de
artigos publicados na RPM e em outras revistas especializadas.

i
i

lp_pitagoras
2009/8/17
i page 4
Estilo OBMEP

i
i

lp_pitagoras
2009/8/17
i page i
Estilo OBMEP

i
i

Sumrio
1 O Teorema de Pitgoras

1.1

Leia um Pouco da Histria . . . . . . . . . . . . . . . .

1.2

O Enunciado do Teorema de Pitgoras . . . . . . . . .

1.3

A Recproca do Teorema de Pitgoras . . . . . . . . .

1.4

Ternos Pitagricos . . . . . . . . . . . . . . . . . . . .

10

1.5

Generalizando o Teorema de Pitgoras . . . . . . . . .

12

1.6

Construes Geomtricas e o Tringulo Retngulo

. .

13

1.7

Problemas . . . . . . . . . . . . . . . . . . . . . . . . .

20

2 reas

24

2.1

Propriedades Importantes . . . . . . . . . . . . . . . .

25

2.2

Nmero . . . . . . . . . . . . . . . . . . . . . . . . .

38

2.3

Problemas . . . . . . . . . . . . . . . . . . . . . . . . .

46

3 Solues dos Problemas

55

3.1

Captulo 1 . . . . . . . . . . . . . . . . . . . . . . . . .

55

3.2

Captulo 2 . . . . . . . . . . . . . . . . . . . . . . . . .

65

i
i

lp_pitagoras
2009/8/17
i page ii
Estilo OBMEP

i
i

ii

SUMRIO

i
i

lp_pitagoras
2009/8/17
i page iii
Estilo OBMEP

i
i

Antes de comear
Este pequeno livro dedicado a dois temas de geometria da maior
importncia: o Teorema de Pitgoras e as reas. Estes dois assuntos,
que possuem forte conexo, so abordados em geral na 9 o ano do Ensino Fundamental, de forma bastante breve e em nvel naturalmente
adequado aos alunos dessa faixa etria. Como estes temas no so
normalmente retomados no Ensino Mdio, grande parte dos alunos
no tem oportunidade de conhecer a enorme riqueza das aplicaes,
muitas por vezes, surpreendentes.
O Teorema de Pitgoras aparece com um pouco de seu contexto
histrico, algumas demonstraes e importantes generalizaes. Apresentamos tambm algumas construes geomtricas associadas ao
tringulo retngulo com a finalidade principal de despertar a curiosidade dos leitores para as construes com rgua e compasso que
fornecem situaes muito educativas, intrigantes e desafiadoras.
O captulo sobre reas, alm de conter todo o material necessrio
para a obteno das frmulas das figuras simples, inclui propriedades
que permitem realizar demonstraes de diversos teoremas usando o
conceito de rea como ferramenta. O captulo termina com a rea do
crculo e uma bastante precisa apresentao do nmero .

iii

i
i

lp_pitagoras
2009/8/17
i page iv
Estilo OBMEP

i
i

lp_pitagoras
2009/8/17
i page 1
Estilo OBMEP

i
i

Captulo 1

O Teorema de Pitgoras
1.1

Leia um Pouco da Histria

Pitgoras (c.569 c.480 a.C.) nasceu na ilha de Samos, perto de


Mileto onde 50 anos antes tinha nascido Tales. Foi a partir das ideias
desses dois grandes personagens que a Matemtica se inicia como
cincia e pode se desenvolver enormemente nos sculos seguintes.
Pitgoras viajou bastante. Esteve no Egito e na Babilnia (talvez
tenha ido at a ndia) onde absorveu os conhecimentos matemticos e
as ideias religiosas de cada regio. Voltando ao mundo grego, fundou
1

i
i

lp_pitagoras
2009/8/17
i page 2
Estilo OBMEP

i
i

 CAP. 1: O TEOREMA DE PITGORAS

em Crotona (sudeste da Itlia de hoje) uma escola, na verdade uma


sociedade secreta, dedicada ao estudo da Matemtica e Filosofia, principalmente. Como todos os documentos daquela poca se perderam,
tudo o que sabemos veio atravs de referncias de outros autores que
viveram sculos depois. Por isso, Pitgoras uma figura obscura na
histria da Matemtica e, para dificultar ainda mais as coisas, a sua
escola, alm de secreta, era comunitria, ou seja, todo o conhecimento
e todas as descobertas eram comuns, pertenciam a todos. Assim, no
sabemos sequer se foi o prprio Pitgoras que descobriu o teorema que
leva o seu nome, pois era comum naquela poca dar todo o crdito
de uma descoberta ao mestre. No conhecemos tambm qual foi a
demonstrao original, mas historiadores acreditam que deva ter sido
alguma usando reas.
O Teorema de Pitgoras um dos mais belos e importantes teoremas da Matemtica de todos os tempos e ocupa uma posio especial na histria do nosso conhecimento matemtico. Foi onde tudo
comeou. Desde o sculo 5 a.C. at o sculo 20 d.C. inmeras demonstraes do Teorema de Pitgoras apareceram. Em 1940, o matemtico
americano E. S. Loomis publicou 370 demonstraes, mas ainda h
mais.

Antes de Pitgoras (Na Babilnia)


Temos provas concretas que os babilnios antigos conheciam o
Teorema de Pitgoras. Muitos tabletes de barro datados do perodo
de 1800 a 1600 a.C. foram encontrados, decifrados e hoje se encontram
em diversos museus. Um deles, chamado Plimpton 322 est na Uni-

i
i

lp_pitagoras
2009/8/17
i page 3
Estilo OBMEP

i
i

N SEC. 1.1: LEIA UM POUCO DA HISTRIA

versidade de Columbia e o fragmento que foi preservado mostra uma


tabela de 15 linhas e 3 colunas de nmeros. Os pesquisadores descobriram que esta tabela continha ternos pitagricos, ou seja, lados de
um tringulo retngulo. Como o que restou apenas um pedao de
um tablete, que deveria fazer parte de um conjunto de tabletes, no
se sabe como esses nmeros foram encontrados. Mas uma pista, que
os babilnios conheciam alguma forma de encontrar esses nmeros,
est em um tablete guardado hoje no Museu Britnico. Nesse tablete
est escrito o seguinte:
4 o comprimento
5 a diagonal
Qual a altura?
4 vezes 4 d 16
5 vezes 5 d 25
Tirando 16 de 25 o resto 9
Quanto vezes quanto devo tomar para ter 9?
3 vezes 3 d 9
3 a altura
Isto mostra, sem dvida, que os babilnios tinham conhecimento
da relao entre os lados de um tringulo retngulo. No h nenhuma
demonstrao, naturalmente, pois isto ainda estava longe de ser uma
preocupao dos matemticos da poca. Eles conheciam receitas que
davam certo e, com elas, resolviam inmeros problemas.
Um outro tablete que merece ateno est no museu da Universidade de Yale. o nico que contm figuras: um quadrado e suas
diagonais. Neste fragmento de tablete que se pode ver a seguir, o lado

i
i

lp_pitagoras
2009/8/17
i page 4
Estilo OBMEP

i
i

 CAP. 1: O TEOREMA DE PITGORAS

do quadrado tomado como igual a 30 e o comprimento da diagonal


aparece como 42, 25, 35.

Como os babilnios escreviam os nmeros na base 60, o comprimento da diagonal , na nossa notao decimal,
24 +

25
35
+
= 42,4263889.
60 3 600

Isto, dividido por 30, d 1, 414213..., uma aproximao excepcional

para 2 com seis casas decimais corretas.

1.2

O Enunciado do Teorema de Pitgoras

Em qualquer tringulo retngulo, a rea do quadrado


cujo lado a hipotenusa igual soma das reas dos quadrados que tm como lados cada um dos catetos.
Se a a medida da hipotenusa e se b e c so as medidas dos catetos,

i
i

lp_pitagoras
2009/8/17
i page 5
Estilo OBMEP

i
i

N SEC. 1.2: O ENUNCIADO DO TEOREMA DE PITGORAS

o enunciado do Teorema de Pitgoras equivale a afirmar que


a2 = b 2 + c 2

Observando a figura acima, o Teorema de Pitgoras afirma que a


rea sombreada em tom mais claro igual rea mais escura.
Este fato no evidente! Muito pelo contrrio, misterioso e intrigante. Para que possamos nos convencer da verdade dessa afirmao,
precisamos de uma demonstrao. Vamos ver algumas.

A demonstrao clssica
Dado um tringulo retngulo de hipotenusa a e catetos b e c,
considere o quadrado cujo lado b + c.

i
i

lp_pitagoras
2009/8/17
i page 6
Estilo OBMEP

i
i

 CAP. 1: O TEOREMA DE PITGORAS

Na figura da esquerda, retiramos do quadrado de lado b + c quatro


tringulos iguais ao tringulo retngulo dado, restando um quadrado
de lado a. Na figura da direita, retiramos tambm do quadrado de
lado b + c os quatro tringulos iguais ao tringulo retngulo dado,
restando um quadrado de lado b e um quadrado de lado c. Logo, a
rea do quadrado de lado a igual soma das reas dos quadrados
cujos lados medem b e c.
Esta simples e engenhosa demonstrao pode ter sido a que os
pitagricos imaginaram.

A demonstrao que usa semelhana


Esta talvez seja a demonstrao mais frequente. A partir de um
tringulo ABC, retngulo em A, traamos a altura AH e verificamos
que os tringulos AHB e AHC so semelhantes ao tringulo ABC.

i
i

lp_pitagoras
2009/8/17
i page 7
Estilo OBMEP

i
i

N SEC. 1.2: O ENUNCIADO DO TEOREMA DE PITGORAS

Da semelhana dos tringulos AHC e ABC temos b 2 = am e, da


semelhana dos tringulos AHB e ABC, temos c 2 = an. Somando
essas duas relaes membro a membro, encontramos:
b2 + c2 = am + an = a(m + n) = a a = a2 .
Esta demonstrao a mais frequente hoje nas escolas porque permite, com um nico e pequeno esforo, no s demonstrar o Teorema
de Pitgoras de forma bastante simples, como tambm encontrar as
relaes importantes do tringulo retngulo. Alm das duas relaes,
que deram origem demonstrao do teorema, obtemos a relao
bc = ah, que tambm se interpreta com o conceito de rea, e h 2 = mn,
que revela o importante fato de que a altura mdia geomtrica entre
as projees dos catetos sobre a hipotenusa.

A demonstrao de Perigal
Henry Perigal, um livreiro em Londres, publicou em 1873 a demonstrao que se pode apreciar na figura a seguir. Trata-se da forma
mais evidente de mostrar que a soma das reas dos quadrados construdos sobre os catetos preenchem o quadrado construdo sobre a

i
i

lp_pitagoras
2009/8/17
i page 8
Estilo OBMEP

i
i

 CAP. 1: O TEOREMA DE PITGORAS

hipotenusa.

Perigal corta o quadrado construdo sobre o maior cateto por duas


retas passando pelo seu centro, uma paralela hipotenusa do tringulo e outra perpendicular, dividindo esse quadrado em quatro partes
congruentes. Essas quatro partes e mais o quadrado construdo sobre o menor cateto, preenchem completamente o quadrado construdo
sobre a hipotenusa.

1.3

A Recproca do Teorema de Pitgoras

A pergunta agora : se a, b e c so reais positivos com a 2 = b2 + c2


ser o tringulo de lados a, b, e c retngulo? Intuitivamente, pensamos
que sim. Mas, devemos demonstrar isto. Consideremos ento um
tringulo ABC com AB = c, BC = a e CA = b.

i
i

lp_pitagoras
2009/8/17
i page 9
Estilo OBMEP

i
i

N SEC. 1.3: A RECPROCA DO TEOREMA DE PITGORAS

1o caso: A < 90o


Imaginemos que b c. Assim, o ponto D, projeo de C sobre AB,
cai no interior do lado AB. Sejam AD = x e CD = h.

Como o tringulo ADC retngulo, temos b 2 = h2 + x2 . Como o


tringulo BDC retngulo, temos:
a2 = h2 + (c x)2
a2 = b2 x2 + c2 2cx + x2
a2 = b2 + c2 2cx
ou seja, a2 < b2 + c2 , que contradiz a condio inicial.
2o caso: A > 90o
Agora, o ponto D cai fora do lado AB.

i
i

lp_pitagoras
2009/8/17
i page 10
Estilo OBMEP

i
i

10

 CAP. 1: O TEOREMA DE PITGORAS

Os mesmos clculos que fizemos no caso anterior nos levam a


a2 = b2 + c2 + 2cx,
ou seja, a2 > b2 + c2 , novamente contradizendo a condio inicial.
Demonstramos ento que em um tringulo ABC, de lados a, b e c,
A < 90o a2 < b2 + c2
A > 90o a2 > b2 + c2
Assim, a condio a2 = b2 + c2 implica necessariamente que A = 90o .

1.4

Ternos Pitagricos

O tringulo de lados 1, 3 e

10 retngulo? Sim, pois

( 10)2 = 12 + 32 .
Durante toda a histria antiga e mesmo at hoje, temos curiosidade em encontrar tringulos retngulos cujos lados so medidos por
nmeros inteiros. Todos ns sabemos que o tringulo de lados 3, 4 e
5 retngulo, mas voc sabia que o tringulo de lados 372, 925 e 997
retngulo? Possivelmente no, e eu tambm no o conhecia antes
de redigir estas notas. Este inclusive o tringulo retngulo de maior
permetro que tem lados menores que 1 000. Nossa curiosidade nos
leva a seguinte pergunta:

i
i

lp_pitagoras
2009/8/17
i page 11
Estilo OBMEP

i
i

11

N SEC. 1.4: TERNOS PITAGRICOS

Como encontrar tringulos retngulos cujos lados tenham medidas inteiras?


Definio. Sendo a, b e c inteiros positivos com b < c < a dizemos
que (b, c, a) um terno pitagrico se a 2 = b2 + c2 . Assim, (3, 4, 5) e
(5, 12, 13) so exemplos de ternos pitagricos.
Um terno pitagrico (b, c, a) chamado primitivo, quando b e c
so primos entre si, ou seja, quando mdc(b, c) = 1. Assim, (3, 4, 5)
um terno pitagrico primitivo. Naturalmente, qualquer terno da
forma (3k, 4k, 5k) com k inteiro e maior que 1 tambm pitagrico,
mas no primitivo.

Uma frmula que gera ternos pitagricos


Sendo m e n inteiros positivos com m > n considere:
b = m 2 n2 ,

c = 2mn,

a = m 2 + n2 .

Veja que (b, c, a) um terno pitagrico pois:


b2 + c2 = (m2 n2 )2 + (2mn)2 = m4 + n4 + 2m2 n2 = (m2 + n2 )2 = a2 .
Assim, para qualquer escolha de nmeros inteiros m e n, o terno
(b, c, a) pitagrico. Por exemplo, para m = 7 e n = 4 encontramos
o terno pitagrico (33, 56, 65). Observe que, se nesta frmula voc
atribuir para m e n valores ambos pares ou ambos mpares, voc
encontrar um terno pitagrico no primitivo, pois todos os termos
do terno sero pares. Se a sua escolha de m e n conduzir a valores de

i
i

lp_pitagoras
2009/8/17
i page 12
Estilo OBMEP

i
i

12

 CAP. 1: O TEOREMA DE PITGORAS

b e c que sejam primos entre si, voc encontrar um terno pitagrico


primitivo. Esta frmula atribuda a Plato (sc.4 a.C.), mas existem
outras que voc ver nos exerccios.

1.5

Generalizando o Teorema de Pitgoras

O Teorema de Pitgoras afirma que a rea do quadrado construdo


sobre a hipotenusa de um tringulo retngulo igual soma das reas
dos quadrados construdos sobre os catetos. Agora, imaginemos figuras semelhantes quaisquer, construdas sobre os lados de um tringulo
retngulo.

Sejam ento A, B e C as reas de figuras semelhantes, construdas


sobre a hipotenusa a e sobre os catetos b e c de um tringulo retngulo,
como mostra a figura acima. Sabemos que a razo entre as reas
de figuras semelhantes igual ao quadrado da razo de semelhana.
Ento,
A  a 2
A
B
=
ou 2 = 2
B
b
a
b

i
i

lp_pitagoras
2009/8/17
i page 13
Estilo OBMEP

i
i

N SEC. 1.6: CONSTRUES GEOMTRICAS E O TRINGULO RETNGULO

Portanto,

A  a 2
=
C
c

ou

13

A
C
= 2.
2
a
c

A
B
C
= 2 = 2.
2
a
b
c

Pela propriedade das propores, como a 2 = b2 + c2 , conclumos que


A = B+C. Isto quer dizer que, se figuras semelhantes so construdas
sobre os lados de um tringulo retngulo, a rea da figura construda
sobre a hipotenusa igual soma das reas das figuras construdas
sobre os catetos. Esta uma generalizao do teorema de Pitgoras.

1.6

Construes Geomtricas e o
Tringulo Retngulo

Construes iniciais
Construir um tringulo retngulo conhecendo dois de seus lados
no difcil.
a) Se os dois catetos so conhecidos, traamos duas semirretas perpendiculares e, com o compasso, transportamos sobre elas as
medidas dos catetos.
b) Se conhecemos a hipotenusa e um dos catetos, traamos novamente as duas semirretas perpendiculares, assinalamos sobre
uma delas o cateto AC = b e, com centro em C, traamos uma
circunferncia de raio a, que determina na outra semirreta o
vrtice B.

i
i

lp_pitagoras
2009/8/17
i page 14
Estilo OBMEP

i
i

14

 CAP. 1: O TEOREMA DE PITGORAS

c) Suponha agora que se conhea a hipotenusa (BC = a) e a altura relativa a ela (AH = h). Como o tringulo retngulo
pode ser inscrito em uma semicircunferncia cujo dimetro
a hipotenusa, fazemos o seguinte. Traamos a circunferncia
de dimetro BC = a e, sobre uma perpendicular reta BC
traamos o segmento P Q = h. A paralela a BC traada por Q
determina o vrtice A sobre a semicircunferncia.

A mdia aritmtica e a mdia geomtrica


Dados dois nmeros positivos x e y definimos a mdia aritmtica
e a mdia geomtrica deles da seguinte forma:
mdia aritmtica: A =

x+y
;
2

i
i

lp_pitagoras
2009/8/17
i page 15
Estilo OBMEP

i
i

N SEC. 1.6: CONSTRUES GEOMTRICAS E O TRINGULO RETNGULO

mdia geomtrica: G =

15

xy.

Dados dois segmentos quaisquer, sejam x e y suas medidas. Podemos visualizar estas duas mdias no desenho abaixo. O dimetro da
semicircunferncia x + y, o segmento que representa a mdia aritmtica o raio, e o segmento que representa a mdia geomtrica a
altura do tringulo retngulo que possui x e y como as projees dos
catetos sobre a hipotenusa. Ento G A e G = A equivale a x = y.

Vamos mostrar agora a soluo grfica de uma equao do tipo


x2 2ax + b2 = 0.
Inicialmente, explicaremos por que a equao est escrita desta forma.
Nas construes geomtricas, cada letra representa um segmento. Por
sua vez, cada segmento representa um nmero real positivo que a
sua medida em uma certa unidade. Antigamente, h dois mil anos,
no existia o conceito de nmero real. A palavra nmero significava, na Grcia antiga, nmero natural. As fraes existiam, mas no
eram consideradas nmeros, eram apenas razes entre nmeros. De
qualquer forma, o que chamamos hoje de nmeros racionais, j existiam, mas os nmeros irracionais ainda estavam muito longe de serem

i
i

lp_pitagoras
2009/8/17
i page 16
Estilo OBMEP

i
i

16

 CAP. 1: O TEOREMA DE PITGORAS

descobertos. Para contornar esta dificuldade, os gregos imaginaram


uma soluo genial: representar todas as grandezas por segmentos
de reta. Eles, naturalmente, no conseguiam medir todos os segmentos, porque no tinham nmeros suficientes, mas isto no importava.
Toda grandeza podia ser representada por um segmento de algum
tamanho. As operaes de adio e subtrao podem ser feitas com
segmentos. Um segmento pode ser multiplicado por um nmero natural ou dividido em qualquer nmero de partes iguais.
As construes geomtricas nada mais so que operaes com segmentos. Alm de somar, subtrair, multiplicar ou dividir por nmero
natural, o que mais se pode fazer com recursos exclusivamente grficos, usando basicamente a rgua e o compasso? Muita coisa, desde
que se perceba que regras so naturalmente impostas.
Em primeiro lugar, se a e b so segmentos, no existe nada, por
exemplo, que se represente por a2 + b. Isto porque a2 a rea de
um quadrado de lado a que, naturalmente, no pode ser somado com
um segmento. Portanto, contas que hoje fazemos sem preocupao
com nmeros naturais, no tinham significado no passado. Assim, a
equao x2 2ax + b2 = 0, que vamos resolver, tinha antigamente o
seguinte significado.
Os segmentos a e b so dados. A soluo da equao o segmento x,
tal que a rea do quadrado de lado x somada com a rea do quadrado
de lado b igual rea do retngulo, cuja base o dobro de a e
cuja altura x. Para encontrar este segmento x vamos, inicialmente,

i
i

lp_pitagoras
2009/8/17
i page 17
Estilo OBMEP

i
i

N SEC. 1.6: CONSTRUES GEOMTRICAS E O TRINGULO RETNGULO

17

aplicar a conhecida frmula da equao do segundo grau:


p

p
(2a)2 4b2
= a a2 b2 .
2

Esta expresso fcil de construir, pois a2 b2 representa um dos


catetos de um tringulo retngulo que possui hipotenusa a e o outro
cateto igual a b. Portanto, dados dois segmentos a e b com
a > b construmos o tringulo ABC com cateto AC = b e hipotenusa
BC = a e as solues x1 e x2 da equao x2 2ax + b2 = 0 esto na
figura a seguir:
x=

2a

Nesta figura, AC = b, CB = a, BA = BD = BE = a2 b2 e,

portanto, CD = x1 = a a2 b2 e CE = a + a2 b2 .

Segmentos do tipo a n

Observe que, dado um segmento a, obter o segmento a 2 muito


fcil. Basta desenhar um tringulo retngulo com os dois catetos

iguais a a. A hipotenusa desse tringulo igual a a 2. Na figura a


seguir, mostramos que, traando segmentos de comprimento a, per-

i
i

lp_pitagoras
2009/8/17
i page 18
Estilo OBMEP

i
i

18

 CAP. 1: O TEOREMA DE PITGORAS

pendiculares hipotenusa de cada tringulo anterior, obtemos a se


quncia de segmentos a n, com n natural.

Construes com a unidade de medida


Dados os segmentos a e b, voc j sabe como construir, por exem

plo, os segmentos 2a, b 3 e a n. Perguntamos agora se, dado um

segmento a, possvel construir segmentos tipo a ou a2 . A resposta ao mesmo tempo no e sim. Observe que, nas construes
anteriores, os segmentos construdos eram independentes da unidade
de medida. Por exemplo, dados dois segmentos a e b, no h sequer
necessidade de estabelecer uma unidade de medida de comprimento

para conhecer a2 + b2 . Neste sentido, no se pode representar a por


um segmento. Se estabelecermos que a unidade de medida igual a
a, ento a2 = a, mas se estabelecermos que a unidade de medida
a metade de a, ento a2 o dobro de a. Fica claro ento que, para

representar a ou a2 por segmentos, devemos estabelecer antes uma


unidade de medida, e saber que os resultados sero diferentes para
cada unidade escolhida.

i
i

lp_pitagoras
2009/8/17
i page 19
Estilo OBMEP

i
i

N SEC. 1.6: CONSTRUES GEOMTRICAS E O TRINGULO RETNGULO

19

a) Dado o segmento a, para construir a fazemos o seguinte. Desenhamos na mesma reta os segmentos AB = 1 e BC = a.
Em seguida, desenhamos a semicircunferncia de dimetro AC
e o segmento BD = x, perpendicular a AC. fcil ver que

x = 1 a = a.

b) Dado o segmento a, para construir a 2 fazemos o seguinte. Sobre


uma reta r, desenhamos o segmento AB = 1 e na perpendicular
a r passando por B desenhamos BD = a. A perpendicular a
AD passando por D encontra a reta r em C, e fcil ver que
BC = x = a2 .

Observao. No possvel construir um segmento do tipo

a 3 2 nem com a unidade. Na verdade, s podemos construir


quando o ndice da raiz for potncia de 2.

i
i

lp_pitagoras
2009/8/17
i page 20
Estilo OBMEP

i
i

20

1.7

 CAP. 1: O TEOREMA DE PITGORAS

Problemas

1) Determine todos os tringulos retngulos cujos lados so inteiros


e esto em progresso aritmtica.
2) dado um quadrado ABCD de lado a. Determine o raio da
circunferncia que contm os vrtices A e B e tangente ao lado
CD.

3) O tringulo ABC tem lados AB = 12, BC = 4 e CA = 20.


Calcule a rea de ABC.
4) Os lados de um tringulo medem 3, 4 e x. Determine para que
valores de x esse tringulo obtusngulo.
5) Se b = 2k + 1, c = 2k 2 + 2k, a = 2k 2 + 2k + 1, onde k um inteiro
positivo, mostre que (b, c, a) um terno pitagrico.
6) Os trs lados de um tringulo retngulo so nmeros inteiros. Um
dos catetos mede 17. Qual o permetro desse tringulo?
7) Em um tringulo retngulo de permetro p, a altura relativa
hipotenusa h. Calcule o comprimento da hipotenusa em funo
dos elementos dados.
8) Sendo b, c e h os catetos e a altura de um tringulo retngulo,
1
1
1
mostre que 2 = 2 + 2 .
h
b
c
9) O antigo livro chins Jiuzhang suanshu contm 246 problemas.
Para a soluo de alguns, necessrio o uso do gou gu, ou seja,
do Teorema de Pitgoras. Veja um desses problemas traduzido
do Captulo 9 do Jiuzhang. No alto de um bambu vertical est

i
i

lp_pitagoras
2009/8/17
i page 21
Estilo OBMEP

i
i

N SEC. 1.7: PROBLEMAS

21

presa uma corda. A parte da corda em contato com o solo mede


3 chih. Quando a corda esticada, sua extremidade toca no solo a
uma distncia de 8 chih do p do bambu. Que comprimento tem
o bambu?

10) Em um tringulo ABC, retngulo em A, trace a altura AH.


Mostre que a soma das reas dos crculos inscritos nos tringulos AHB e AHC igual a rea do crculo inscrito em ABC.
11) O problema de Hipcrates.
A figura a seguir mostra um tringulo retngulo e trs semicircunferncias tendo os lados como dimetros. Mostre que a soma das
reas das duas lnulas sombreadas igual rea do tringulo.

12) Em um tringulo ABC, as medianas que partem de A e B so


perpendiculares. Se BC = 8 e AC = 6, calcule AB.
13) Se b e c so os catetos de um tringulo retngulo de hipotenusa a
e altura h, mostre que b + c < a + h.

i
i

lp_pitagoras
2009/8/17
i page 22
Estilo OBMEP

i
i

22

 CAP. 1: O TEOREMA DE PITGORAS

14) O ponto P interior ao retngulo ABCD e tal que P A = 3,


P B = 4 e P C = 5. Calcule P D.
15) Determine o raio da circunferncia circunscrita ao tringulo cujos
lados medem 6 cm, 6 cm e 4 cm.
16) Duas cordas perpendiculares AB e CD de uma circunferncia
cortam-se em P . Se P A = a, P B = b, e P C = c, calcule o
raio da circunferncia.
Construes Geomtricas
17) Dados os segmentos a, b e c, construa o segmento

x = a2 + b2 c2 .
18) Resolva graficamente: um retngulo tem 24 cm de permetro e
25 cm2 de rea. Construa este retngulo.
19) Dado um segmento de comprimento a, construa um segmento cujo

comprimento a 14.
20) Dados os segmentos a e b, construa x =

2a2 + 3b2 .

21) Dados os segmentos a e b, construa o segmento x =

4 4
a + b4 .

Mais difceis
22) Duas circunferncias de raios R e r so tangentes exteriormente e
so tangentes a uma reta t nos pontos A e B.
(a) Determine AB em funo dos dois raios.

i
i

lp_pitagoras
2009/8/17
i page 23
Estilo OBMEP

i
i

N SEC. 1.7: PROBLEMAS

23

(b) Determine a seguir o raio de uma circunferncia que tangente reta t e s duas circunferncias dadas
23) No tringulo ABC, retngulo em A, traam-se a altura AH e os
segmentos HE e HF , perpendiculares a AB e AC, respectivap
p

3
mente. Se BE = p e CF = q, mostre que 3 p2 + 3 q 2 = a2 ,
onde a a hipotenusa do tringulo ABC.
24) Um ponto P interior a um quadrado ABCD tal que P A = a,
P B = b e P C = b + c, onde os nmeros a, b e c satisfazem a
relao a2 = b2 + c2 . Mostre que o ngulo BP C reto.

i
i

lp_pitagoras
2009/8/17
i page 24
Estilo OBMEP

i
i

Captulo 2

reas
Os alunos, em geral, trabalham com reas desde muito cedo. Vamos ento, neste captulo, imaginar que as frmulas que calculam
reas das figuras simples como o quadrado, retngulo, paralelogramo,
tringulo e trapzio sejam conhecidas.
Inicialmente, daremos toda ateno ao tringulo. Conhecendo
bem o tringulo, no teremos dificuldade nos polgonos pois, afinal,
eles podem ser decompostos em tringulos. Com certeza, voc j sabe
calcular a rea de um tringulo, fazendo a metade do produto da base
pela altura.
1. A frmula tradicional.

S=

ah
2

2. Se voc j conhece um pouco de trigonometria, a frmula seguinte


muito boa.
24

i
i

lp_pitagoras
2009/8/17
i page 25
Estilo OBMEP

i
i

N SEC. 2.1: PROPRIEDADES IMPORTANTES

25

1
S = ab sen
2

3. Quando os trs lados so conhecidos, calcular a altura ou um


dos ngulos d algum trabalho. Nesse momento, a frmula de
Heron tima (no daremos aqui a demonstrao dela).

S=

p
p(p a)(p b)(p c)

onde p =

a+b+c
2

Vamos tratar agora do mais importante: as propriedades.

2.1

Propriedades Importantes

Propriedade 1
A rea de um tringulo no se altera quando sua base permanece
fixa e o terceiro vrtice percorre uma reta paralela base.

i
i

lp_pitagoras
2009/8/17
i page 26
Estilo OBMEP

i
i

26

 CAP. 2: REAS

Na figura acima, a reta r paralela a BC. Os tringulos ABC e


A0 BC tm mesma rea, pois possuem mesma base e mesma altura.

Propriedade 2
Em um tringulo, uma mediana divide sua rea em partes iguais.

S1 = S 2

De fato, os dois tringulos interiores possuem mesma base e mesma


altura. Logo, possuem mesma rea.
Quando duas figuras possuem mesma rea, dizemos que elas so
equivalentes. Portanto, o enunciado desta propriedade pode ser: Uma
mediana divide o tringulo em dois outros equivalentes.
Antes de prosseguir com as propriedades, vamos resolver dois exerccios cujos enunciados no so comuns nos livros didticos atuais.

i
i

lp_pitagoras
2009/8/17
i page 27
Estilo OBMEP

i
i

N SEC. 2.1: PROPRIEDADES IMPORTANTES

27

Exerccio 1
O tringulo ABC da figura abaixo tem rea igual a 30. O lado
BC est dividido em quatro partes iguais, pelos pontos D, E e F , e
o lado AC est dividido em trs partes iguais pelos pontos G e H.
Qual a rea do tringulo GDE?

Soluo: Observe o tringulo ABC com as cevianas BG e BH.

Pela propriedade 2 os tringulos BAG, BGH e BHC tm mesma


rea. Cada um tem, portanto, rea igual a 10 e o tringulo BGC tem
rea igual a 20.

i
i

lp_pitagoras
2009/8/17
i page 28
Estilo OBMEP

i
i

28

 CAP. 2: REAS

Observe agora o tringulo BGC com as cevianas GD, GE e GF .

Pela mesma propriedade, os tringulos GBD, GDE, GEF e GF C


tm mesma rea. Logo, cada um deles tem rea 5. A rea do tringulo
GDE igual a 5.
Repare que a soluo do problema no necessitou de frmulas.
Uma propriedade simples e convenientemente aplicada resolveu a questo. Vamos ver outro problema.

Exerccio 2
dado um tringulo ABC e um ponto P do lado AC mais prximo
de A que de C. Traar uma reta por P que divida o tringulo ABC
em duas partes de mesma rea.

i
i

lp_pitagoras
2009/8/17
i page 29
Estilo OBMEP

i
i

N SEC. 2.1: PROPRIEDADES IMPORTANTES

29

Soluo: Faamos o seguinte. Trace BP e uma paralela a BP por A


que encontra a reta BC em D.

Os tringulos ABP e DBP tm reas iguais pela propriedade 1.


Assim, o tringulo P DC tem mesma rea que o tringulo ABC. Mas,
tomando o ponto mdio M de DC, a reta P M divide P DC em duas
partes de mesma rea (propriedade 2). Logo, P M divide tambm
ABC em duas partes de mesma rea.
Vamos continuar com mais duas propriedades importantes.

i
i

lp_pitagoras
2009/8/17
i page 30
Estilo OBMEP

i
i

30

 CAP. 2: REAS

Propriedade 3
Se dois tringulos tm mesma altura, ento a razo entre suas
reas igual razo entre suas bases. A afirmao acima tem comprovao imediata a partir da frmula que calcula a rea do tringulo.

S
a
= 0
S0
a

Propriedade 4
A razo entre as reas de tringulos semelhantes igual ao quadrado
da razo de semelhana.
Observe, na figura a seguir, dois tringulos semelhantes com bases
a e a0 e alturas h e h0 .

Como so semelhantes, a razo entre as bases a mesma razo entre

i
i

lp_pitagoras
2009/8/17
i page 31
Estilo OBMEP

i
i

N SEC. 2.1: PROPRIEDADES IMPORTANTES

31

as alturas. Esse nmero a razo de semelhana das duas figuras:


k=

h
a
= 0.
a0
h

Porm, se S e S 0 so as reas dos dois tringulos temos:


ah/2
S
a h
= 0 0 = 0 0 = k k = k2.
0
S
a h /2
a h
Vejamos um exemplo simples.
Os dois tringulos da figura abaixo so semelhantes. Se a rea do
menor igual a 8, qual a rea do maior?

Para esta pergunta, alunos tm uma tendncia irresistvel de responder rapidamente que a rea do tringulo maior 24. Porm, isto no
verdade. A razo de semelhana dos dois tringulos k = 1/3 e,
portanto, a razo entre suas reas 1/9. Da, se a rea do menor
igual a 8, a rea do maior 72.
Voc pode ver esta relao na figura a seguir. Realmente, o tringulo pequeno cabe 9 vezes dentro do grande.

i
i

lp_pitagoras
2009/8/17
i page 32
Estilo OBMEP

i
i

32

 CAP. 2: REAS

A propriedade 4, que mostramos para tringulos, vale naturalmente para polgonos, pois estes podem ser divididos em tringulos.
Mas, importante saber que esta propriedade vale para quaisquer
figuras semelhantes.
A razo entre as reas de figuras semelhantes quaisquer igual ao
quadrado da razo de semelhana.
O exerccio a seguir, caiu em um vestibular da FGV-RJ.

Exerccio 3
Em algum momento, na primeira metade do sculo passado, uma
pessoa chamada Afrnio tinha um valioso terreno desocupado, perto
do centro da cidade do Rio de Janeiro. Com a urbanizao da cidade,
ruas novas foram abertas e o terreno de Afrnio ficou reduzido a
um tringulo ABC, retngulo em B, ainda de grande valor, pois o
lado AB media 156 metros. Pois bem, Afrnio morreu e em seu
testamento os advogados encontraram as instrues para dividir o
terreno igualmente entre seus dois filhos. Era assim: um muro
deve ser construdo perpendicularmente ao lado AB, de forma que
os dois terrenos resultantes da diviso tenham mesmo valor; o que

i
i

lp_pitagoras
2009/8/17
i page 33
Estilo OBMEP

i
i

N SEC. 2.1: PROPRIEDADES IMPORTANTES

33

tem a forma de um trapzio ser do meu filho mais velho e o outro


ser do mais novo.
Os advogados concluram que os terrenos deviam ter mesma rea,
pois o testamento dizia que deveriam ter mesmo valor. Mas no foram
capazes de decidir em que posio deveria ficar o muro. Conta meu
av que o episdio ganhou as pginas dos jornais por vrios dias, com
leitores opinando de diversas maneiras sobre a posio correta do
muro. Ele falava e se divertia muito com as opinies absurdas mas,
ao mesmo tempo, me instigava a resolver o problema. E o problema
retorna para vocs.
Em que posio, relativamente ao lado AB do terreno, o muro
deve ser construdo?
Soluo:

Na figura acima, M N o muro que deve ser construdo perpendicularmente ao lado AB. Seja AM = x, de forma que o tringulo AM N
e o trapzio M BCN tenham mesma rea S. Os tringulos AM N
e ABC so semelhantes e a razo de semelhana entre eles x/156.

i
i

lp_pitagoras
2009/8/17
i page 34
Estilo OBMEP

i
i

34

 CAP. 2: REAS

Como a razo entre suas reas o quadrado da razo de semelhana


devemos ter:
 x 2
S
=
.
2S
156
Extraindo a raiz quadrada de ambos os lados ficamos com
1
x
=
156
2

o que d x = 78 2
= 110. Temos a soluo. O muro deve ser
construdo a 110 metros de A. As reas dos dois terrenos sero iguais
e Afrnio ficar feliz em ver sua vontade atendida.

A construo geomtrica do exerccio 3

Acabamos de resolver o problema da diviso do terreno em duas


partes de mesma rea. Mas como poderemos fazer isto utilizando
apenas a rgua e o compasso? Imagine que o engenheiro tem a planta
do terreno e deseja desenhar o muro na posio exata, sem contas,
sem aproximaes. Vamos ver como se faz isto.
Resolva o problema novamente considerando AB= 2a. Voc vai

encontrar AM = a 2. Faa ento o seguinte. Pelo ponto P , mdio


de AB trace uma perpendicular P Q a AB de comprimento a como
na figura seguinte:

i
i

lp_pitagoras
2009/8/17
i page 35
Estilo OBMEP

i
i

N SEC. 2.1: PROPRIEDADES IMPORTANTES

35

Como AQ = a 2, transfira com o compasso essa medida para a reta


AB, encontrando a posio exata de M .

Exerccio 4
As medianas de um tringulo dividem esse tringulo em 6 outros
tringulos. Mostre que todos tm mesma rea.
Soluo: Representemos por (ABC) a rea de um tringulo ABC.

Seja (ABC) = S. O ponto de interseo das medianas G, o

i
i

lp_pitagoras
2009/8/17
i page 36
Estilo OBMEP

i
i

36

 CAP. 2: REAS

baricentro. Sabemos que BG = 2/3 BN . Logo,


2
2 S
S
(ABC) = (ABN ) = = .
3
3 2
3
Analogamente, (BCG) = (CAG) = S/3. Mas GP mediana no
tringulo ABG. Da, (AP G) = (BP G) = S/6. Assim, os seis tringulos tm rea S/6.

Usando reas
O estudante pensa, em geral, que um problema sobre reas significa sempre calcular a rea de alguma figura. Na verdade no s isso.
A ferramenta rea pode ser usada na soluo de diversos problemas
de geometria plana de aparncia algo complicada. Veja um exemplo.

Exerccio 5
A figura a seguir mostra um trapzio com bases medindo 20 cm e
14 cm e com os outros dois lados medindo 5 cm cada um. Duas
circunferncias com centros A e B so tangentes s bases, uma ao
lado esquerdo e outra ao lado direito. Pergunta-se qual o comprimento do segmento AB.

i
i

lp_pitagoras
2009/8/17
i page 37
Estilo OBMEP

i
i

N SEC. 2.1: PROPRIEDADES IMPORTANTES

37

Soluo: Vamos inicialmente calcular a altura do trapzio, que o


dimetro de cada circunferncia. Dividindo o trapzio em um retngulo e dois tringulos retngulos iguais, temos a evidente situao
seguinte:

A altura do trapzio mede 4 cm e o raio de cada circunferncia


mede 2 cm. Vamos agora ligar os dois vrtices da esquerda ao ponto
A e os dois vrtices da direita ao ponto B. Vemos agora o trapzio
original dividido em dois outros trapzios e dois tringulos iguais.

Lembrando que a rea do trapzio o produto da base mdia


pela altura e observando que os dois tringulos de vrtices A e B tm
base igual a 5 e altura igual a 2, vamos escrever a equao que diz
que a soma das reas dessas quatro figuras igual rea do trapzio
original. Fazendo AB = x, temos:
(20 + x) 2 (14 + x) 2
52
(20 + 14) 4
+
+2
=
.
2
2
2
2

i
i

lp_pitagoras
2009/8/17
i page 38
Estilo OBMEP

i
i

38

 CAP. 2: REAS

Isto d x = 12, resolvendo nosso problema.


claro que outra forma de resolver pode ser conseguida com outros meios. O que desejamos enfatizar que a ferramenta rea
muitas vezes til para resolver problemas diversos. Nesse caso, ela
propiciou uma soluo limpa e elegante.

2.2

Nmero

O nmero a razo entre o comprimento de uma circunferncia


e seu dimetro. Esta razo d sempre o mesmo valor, ou seja, independe da circunferncia, porque duas circunferncias quaisquer so
semelhantes. Todas as circunferncias so semelhantes entre si. Se C
o comprimento da circunferncia de raio R, ento por definio:
C
= .
2R
Mas, o que o comprimento de uma circunferncia? Ns sabemos
o que o comprimento de um segmento, mas temos apenas uma ideia
intuitiva do que seja o comprimento de uma circunferncia. Podemos
pensar em passar um barbante bem fino em volta da circunferncia,
estic-lo e medir seu comprimento com uma rgua. Isto d uma boa
ideia do que seja o comprimento da circunferncia, mas este mtodo
experimental permite apenas avaliar (com pouca preciso) essa medida. Vamos tornar mais preciso este conceito.
O comprimento da circunferncia , por definio, o nmero real
cujas aproximaes por falta so os permetros dos polgonos regu-

i
i

lp_pitagoras
2009/8/17
i page 39
Estilo OBMEP

i
i

39

N SEC. 2.2: NMERO

lares inscritos e cujas aproximaes por excesso so os permetros dos


polgonos regulares circunscritos.

Observe a figura anterior. Voc v uma circunferncia com um


dodecgono regular inscrito e outro circunscrito. Pense agora nesta
situao com polgonos regulares de n lados. Se C o comprimento da
circunferncia, pn o permetro do polgono inscrito e P n o permetro
do circunscrito temos, por definio,
pn < C < P n .
Quando n cresce, os valores de pn aumentam, os de Pn diminuem e
ambos se aproximam cada vez mais de C.

Sendo R o raio da circunferncia, as razes

pn
Pn
e
, quando n
2R 2R

i
i

lp_pitagoras
2009/8/17
i page 40
Estilo OBMEP

i
i

40

 CAP. 2: REAS

cresce, vo se aproximando, uma por um lado e outra pelo outro, de


C
, ou seja, de .
2R
Veja, a seguir, estas aproximaes para alguns valores de n.

n
6
12
24
48
96
192
384

pn
2R
3,00000
3,10582
3,13262
3,13935
3,14103
3,14145
3,14156

Pn
2R
3,46411
3,21540
3,15967
3,14609
3,14272
3,14188
3,14167

Repare no quadro acima, que os valores das duas colunas vo


se aproximando, mas para polgonos de 384 lados s conseguimos
corretas as trs primeiras decimais.
O nmero um nmero irracional, aproximadamente igual a
3,1416. O uso da letra grega para representar a razo entre o
comprimento da circunferncia e seu dimetro deve-se a Euler, que
a adotou em 1 737. Mas esta razo sempre fascinou matemticos e
curiosos em toda a histria. Hoje conhecemos mais de cinco bilhes
de casas decimais de e os fanticos em computao vo em breve
aumentar em muito esse nmero.

i
i

lp_pitagoras
2009/8/17
i page 41
Estilo OBMEP

i
i

N SEC. 2.2: NMERO

41

A rea do crculo

Continuando com a ideia dos polgonos, a rea do crculo o


nmero real cujas aproximaes por falta so as reas dos polgonos
regulares inscritos.
Imaginemos um polgono regular com n lados (n bem grande)
inscrito na circunferncia de raio R. Dividamos o polgono em tringulos issceles iguais, todos com vrtice no centro da circunferncia.
Cada tringulo tem dois lados iguais a R, um lado igual a a, lado do
polgono, e altura h relativa a essa base.
ah
(na)h
pn h
=
=
, onde pn o
A rea do polgono An = n
2
2
2
permetro do polgono. Quando n cresce indefinidamente, p n tende
ao comprimento da circunferncia e h tende ao raio. A rea do crculo
ento:
2RR
S=
= R2 .
2

i
i

lp_pitagoras
2009/8/17
i page 42
Estilo OBMEP

i
i

42

 CAP. 2: REAS

reas de setores circulares


Frequentemente precisaremos calcular reas de setores circulares.
Repare que a rea de um setor de um crculo proporcional ao ngulo
central, ou ainda, proporcional ao comprimento de seu arco. Para
justificar isto, basta observar que dobrando o ngulo central a rea
do setor dobra, triplicando o ngulo central a rea do setor triplica,
e assim por diante.
Assim, se o ngulo central tem medida em graus, a rea do
setor

R2 .
S=
360

Por outro lado, como a rea do setor tambm proporcional ao


comprimento L do seu arco, podemos exprimir essa rea assim:
S=

L
LR
R2 =
2R
2

uma frmula bastante interessante, pois d a idia de um tringulo


de base de comprimento L e altura R.

i
i

lp_pitagoras
2009/8/17
i page 43
Estilo OBMEP

i
i

43

N SEC. 2.2: NMERO

Apndice
O Clculo de pelo Mtodo dos Polgonos
Os matemticos antigos, at o sculo 16 (e, portanto, antes da
inveno do Clculo), tentaram obter valores de usando polgonos
regulares inscritos na circunferncia com nmero de lados cada vez
maior. Vamos mostrar como faziam isto. A ideia era tomar um
polgono pequeno e ir dobrando o nmero de lados.
Na figura a seguir, ln = AB o lado do polgono regular de n
lados inscrito em uma circunferncia de raio 1. Se C o ponto mdio
do arco AB, ento AC = l2n o lado do polgono regular de 2n lados
inscrito na mesma circunferncia.

Sendo CD o dimetro, O o centro da circunferncia e P o ponto


de interseo de AB com CD temos, no tringulo retngulo ACD, a
relao AC 2 = CD CP . Observe que AC = l2n e
OP =

(ln )2
1
=
4
2

q
4 (ln )2 .

i
i

lp_pitagoras
2009/8/17
i page 44
Estilo OBMEP

i
i

44

 CAP. 2: REAS

Da,


1
(l2n ) = 2 1
2
2

Assim,


q
q
2
4 (ln ) = 2 4 (ln )2 .

l2n =

q
2 4 (ln )2 .

Esta bela frmula permite calcular o lado de um polgono regular


de 2n lados inscrito em uma circunferncia de raio 1 em funo do
lado do polgono regular de n lados inscrito na mesma circunferncia. Como o lado do quadrado inscrito na circunferncia de raio 1

l4 = 2, podemos facilmente prosseguir e encontrar:


q

l8 = 2 2
l16 =
s

l32 =

l64

2+ 2

2+

2+ 2

v
s
u
r
u
q

t
= 2 2+ 2+ 2+ 2

e assim por diante. Repare que 64 = 26 e a expresso que calcula o


lado do polgono de 64 lados possui 5 radicais. L dentro o primeiro
sinal negativo e todos os outros so positivos. Como cada vez que
dobramos o nmero de lados acrescentamos mais um radical, o lado

i
i

lp_pitagoras
2009/8/17
i page 45
Estilo OBMEP

i
i

45

N SEC. 2.2: NMERO

de um polgono regular de 2n+1 lados :

l2n+1 =

2 + 2 + 2 +

com n radicais.

O permetro do polgono 2n+1 lados igual a 2n+1 l2n+1 , que tende


a 2 quando n cresce. Assim, aproximaes de podem ser obtidas
por:
v
s
u
r
u
q
t

=2 2 2+ 2+ 2+ 2

com n radicais na expresso acima.

O leitor que chegou at aqui deve estar pensando que pode calcular por este mtodo qualquer nmero de casas decimais de . Infelizmente, isto no verdade. Seria, se o leitor tivesse uma calculadora
que trabalhasse com infinitas casas decimais. Mas esta calculadora
ainda no foi inventada. Observe que, na expresso acima, temos um
produto, onde o primeiro fator muito grande e o segundo muito
pequeno. Se a sua calculadora trabalha com, por exemplo, 12 dgitos, no possvel aumentar muito o valor de n. O segundo fator
vai perdendo preciso e o resultado idem. Os matemticos antigos
calculavam essas razes manualmente, com um nmero absurdo de
casas decimais, para conseguirem obter umas poucas casas decimais
precisas de .
O recorde ainda est com L. van Ceulen que conseguiu 35 casas
decimais exatas. Como morreu logo em seguida, a viva mandou

i
i

lp_pitagoras
2009/8/17
i page 46
Estilo OBMEP

i
i

46

 CAP. 2: REAS

gravar esse valor de em sua lpide:

3,14159265358979323846264338327950288

2.3

Problemas

1) Na figura a seguir, cada quadrcula representa uma unidade de


rea. Qual a rea do polgono que aparece no interior do quadriculado?

2) Observe a figura a seguir. Por um ponto da diagonal do retngulo


foram traadas paralelas a seus lados. Mostre que as reas dos
retngulos sombreados so iguais.

i
i

lp_pitagoras
2009/8/17
i page 47
Estilo OBMEP

i
i

N SEC. 2.3: PROBLEMAS

47

3) Do pentgono ABCDE sabe-se o seguinte: A = E = 90 o ,


B = C = 120o , AB = CD = 4 e BC = 8. Calcule a rea
desse pentgono.
4) O tringulo ABC tem lados medindo 5 cm, 7 cm e 8 cm. Calcule
sua rea e o raio da circunferncia inscrita.
5) No paralelogramo ABCD de rea 1, os pontos P , Q e R, nesta
ordem, dividem a diagonal AC em quatro partes iguais. Qual a
rea do tringulo DP Q?
6) No tringulo ABC de rea 1, as medianas BM e CN cortam-se
em G. Qual a rea do tringulo GM N ?
7) Na figura a seguir, AD = 23 AB e AE = 23 AC. O segmento DE
divide o tringulo em duas partes: um tringulo de rea S 1 e um
trapzio de rea S2 . Qual destas duas reas maior?

8) Na figura a seguir, as retas r e s so paralelas e o segmento AB


perpendicular a ambas. Os segmentos AD e BC cortam-se em P .

i
i

lp_pitagoras
2009/8/17
i page 48
Estilo OBMEP

i
i

48

 CAP. 2: REAS

(a) Mostre que as reas dos tringulos P AB e P CD so iguais.


(b) Dados AB = 10, BD = 7 e AC = 18, calcule a rea do
tringulo P DC.

9) No mximo, quantos tringulos equilteros de lado 1 cabem (sem


superposio) dentro de um hexgono regular de lado 12?
10) No interior do quadrado ABCD de lado 1 da figura abaixo foram
traadas as semicircunferncias de dimetros AB e BC. Qual o
valor da rea sombreada?

11) Abaixo voc v dois retngulos iguais. Colocando um sobre o


outro, como mostra a figura, determine se o retngulo de cima cobriu mais da metade do retngulo de baixo, exatamente a metade
ou menos da metade.

i
i

lp_pitagoras
2009/8/17
i page 49
Estilo OBMEP

i
i

N SEC. 2.3: PROBLEMAS

49

12) Usando apenas seus instrumentos de desenho, trace por P uma


reta que divida o quadriltero da figura abaixo em duas partes de
mesma rea.

13) ABCDEF um hexgono regular. Os pontos M , N e P so


mdios dos lados AB, CD e EF . Qual a razo entre a rea do
tringulo M N P e a rea do hexgono?
14) Um hexgono regular e um tringulo equiltero esto inscritos na
mesma circunferncia. Qual a razo entre as reas dessas duas
figuras?
15) A figura abaixo mostra um tringulo de altura 1 dividido por duas
retas paralelas sua base em trs partes de mesma rea. Qual a
altura do trapzio central?

i
i

lp_pitagoras
2009/8/17
i page 50
Estilo OBMEP

i
i

50

 CAP. 2: REAS

16) Em qualquer tringulo ABC, mostre que sua rea


1
S = AB AC senA.
2
17) Com os dados da figura abaixo, calcule a razo entre as reas A e
B.

18) A letra N da figura abaixo foi construda a partir de um retngulo


de base 10 e altura 12. Calcule sua rea.

i
i

lp_pitagoras
2009/8/17
i page 51
Estilo OBMEP

i
i

N SEC. 2.3: PROBLEMAS

51

19) Seja ABCD um quadrado de lado 1 e sejam M e N os pontos


mdios dos lados BC e CD, respectivamente. Traando os segmentos AM , AN e N B, calcule as reas das cinco partes em que
o quadrado ficou dividido.

20) O tringulo ABC da figura a seguir tem rea igual a 1. Cada um


de seus lados foi dividido em trs partes iguais. Calcule a rea do
tringulo sombreado.

i
i

lp_pitagoras
2009/8/17
i page 52
Estilo OBMEP

i
i

52

 CAP. 2: REAS

21) No manuscrito de Aryabhatiya do sculo VI encontrou-se a seguinte


afirmao: Some 4 a 100, multiplique por 8 e some 62 000. O resultado aproximadamente a circunferncia do crculo de dimetro
20 000. Qual o valor de que est implcito nesta afirmao?
22) Na figura a seguir, os ngulos BAD e DAC so iguais a 60 o ,
AB = 6 e AC = 4. Quanto mede AD?

23) Em um crculo de raio 1 um arco tem comprimento x


(0 < x ). Determine a rea do segmento circular correspondente a esse arco.
24) Os pontos A, B e C, nesta ordem, sobre uma circunferncia de
raio 1 so tais que o arco AB mede 80o e o arco BC mede 50o .

i
i

lp_pitagoras
2009/8/17
i page 53
Estilo OBMEP

i
i

N SEC. 2.3: PROBLEMAS

53

Calcule a rea da regio do crculo limitada pelas cordas AC e BC


e pelo arco AB.
25) Em uma circunferncia de raio 1, as cordas AB e CD so paralelas
e o centro da circunferncia no est entre elas. A corda AB igual
ao lado do hexgono regular inscrito na circunferncia e CD igual
ao lado do tringulo equiltero inscrito na mesma circunferncia.
Calcule a rea da regio do crculo compreendida entre as duas
cordas.
26) A figura a seguir mostra trs circunferncias de raios iguais a 1,
tangentes entre si duas a duas, e uma circunferncia maior tangente
s trs primeiras. Calcule a rea da regio A.

27) A figura abaixo mostra duas semicircunferncias de dimetros


AB = 4 e AC = 6. Calcule o raio da circunferncia que

i
i

lp_pitagoras
2009/8/17
i page 54
Estilo OBMEP

i
i

54

 CAP. 2: REAS

tangente s duas semicircunferncias e ao segmento BC.

i
i

lp_pitagoras
2009/8/17
i page 55
Estilo OBMEP

i
i

Captulo 3

Solues dos Problemas


3.1

Captulo 1

1) Sejam: x r, x e x + r os lados de um tringulo retngulo. Considerando r > 0, x + r a hipotenusa e, portanto,


(x + r)2 = x2 + (x r)2 . Desenvolvendo e simplificando, obtemos
x = 4r. Portanto, os lados medem 3r, 4r e 5r.
2)

Trace pelo centro O da circunferncia o segmento M N perpendicular a AB, como na figura acima. Como M mdio de AB
55

i
i

lp_pitagoras
2009/8/17
i page 56
Estilo OBMEP

i
i

56

 CAP. 3: SOLUES DOS PROBLEMAS

temos, no tringulo retngulo OM B, OB = R, M B = a/2 e


OM = a R. Aplicando o teorema de Pitgoras no tringulo
5a
OM B, encontramos R =
.
8
3) Considere a figura:

O Teorema de Pitgoras em cada um dos tringulos retngulos


fornece:
x2 + h2 = 12 e (4 x)2 + h2 = 20.
Logo 16 8x + x2 + 12 x2 = 20 , o que d x =
1. Portanto,

4 11
altura do tringulo igual a 11 e a rea S =
= 2 11.
2
4) Para a existncia do tringulo devemos ter 1 < x < 7. Se o tringulo obtusngulo e x o maior lado, devemos ter
x2 > 32 + 42 , ou seja, x > 5. Se o lado que mede 4 o maior, de
vemos ter 42 > x2 + 32 , ou seja, x < 7. Portanto, esse tringulo

obtusngulo para 1 < x < 7 ou 5 < x < 7.


5) a2 = 4k 2 (k + 1)2 + 4k(k + 1) + 1 = b2 + c2 .
6) Usando as frmulas que geram ternos pitagricos, devemos ter
m2 n2 = 17, ou seja, (m + n)(m n) = 17. Como 17 primo,
ento, necessariamente, m + n = 17 e m n = 1, o que d m = 9
e n = 8. Portanto, o outro cateto 2mn = 2 9 8 = 144 e a
hipotenusa m2 + n2 = 81 + 64 = 145. O permetro igual a 306.

i
i

lp_pitagoras
2009/8/17
i page 57
Estilo OBMEP

i
i

N SEC. 3.1: CAPTULO 1

57

7) Se a + b + c = p, escrevemos b + c = p a. Elevando ao quadrado, temos b2 + c2 + 2bc = a2 + p2 2ap. Usando o Teorema de


Pitgoras e a relao bc = ah, ficamos com 2ah = p 2 2ap, o que
p2
d a =
.
2(h + p)
8)

1
1
b2 + c 2
a2
1
+
=
=
= 2.
2
2
2
2
b
c
(bc)
(ah)
h

9) Se x o comprimento do bambu temos: (x + 3) 2 = x2 + 82 , o que


55
d x =
= 9, 17 chih.
6
10) Sejam r1 , r2 e r os raios dos crculos inscritos nos tringulos AHB,
AHC e ABC. Esses trs tringulos so semelhantes e, portanto,
r1
r2
r
=
= . Elevando ao quadrado e multiplicando por
c
b
a
r1 r
r2 r
rr
temos:
=
= 2 . Como b2 + c2 = a2 , conclumos que
2
2
c
b
a
r12 + r22 = r 2 .
11) Sejam: T a rea do tringulo, P e Q as reas das lnulas e U e V
as reas das outras duas regies.

Como a rea do semicrculo construdo sobre a hipotenusa igual a


soma das reas dos semicrculos construdos sobre os catetos temos
T + U + V = P + U + Q + V , ou seja, T = P + Q, como queramos
demonstrar.

i
i

lp_pitagoras
2009/8/17
i page 58
Estilo OBMEP

i
i

58

 CAP. 3: SOLUES DOS PROBLEMAS

12) Sejam M e N os pontos mdios dos lados BC e AC, respectivamente. As medianas AM e BN cortam-se no baricentro, que divide cada mediana na razo 2/1. Observando a figura a seguir, vamos aplicar o Teorema de Pitgoras nos tringulos AGN e BGM .

4x2 + y 2 = 9

x + 4y = 16

Somando e simplificando obtemos x2 + y 2

M N = 5 e, portanto, AB = 2 5.

= 5, ou seja,

13) Como os dois lados da desigualdade so positivos, observe as equivalncias:


b+c <a+h

(b + c)2 < (a + h)2


b2 + c2 + 2bc < a2 + h2 + 2ah
2bc < h2 + 2ah
0 < h2 .

Como h > 0, a desigualdade verdadeira.


14) Traando por P paralelas aos lados do retngulo, temos a situao
da figura abaixo.

i
i

lp_pitagoras
2009/8/17
i page 59
Estilo OBMEP

i
i

59

N SEC. 3.1: CAPTULO 1

Usaremos o Teorema de Pitgoras quatro vezes.


m2 + n 2 = 9

p + q = 25

Somando,
m2 + q 2 + n2 + p2 = 34

x2 + 16 = 34

x = 3 2.

15) Traamos a altura AM que passa pelo centro O da circunferncia


circunscrita ao tringulo ABC. No tringulo retngulo AM B cal
culamos AM = 4 2. Sendo R o raio da circunferncia, o tringulo

OM B fornece: R2 = 22 + (4 2 R)2

9 2
o que d R =
.
4

i
i

lp_pitagoras
2009/8/17
i page 60
Estilo OBMEP

i
i

60

 CAP. 3: SOLUES DOS PROBLEMAS

16)

Traando o dimetro CE, temos que o tringulo CBE retngulo


em B e que os ngulos CAB e CEB so iguais, pois subtendem o
mesmo arco BC. Portanto, os tringulos CP A e CBE so semelhantes e
CA
CP
=
ou seja,
CE
BC
p
(a2 + c2 )(b2 + c2 )
Logo, R =
.
2c

2R =

CA
.
CP

17) Suponha, naturalmente, que c < a2 + b2 para que o problema


tenha soluo. Construmos, inicialmente, o tringulo retngulo

de catetos a e b. Sua hipotenusa mede y = a2 + b2 . Em seguida, construmos o tringulo retngulo de hipotenusa y e um cateto
igual a c. O outro cateto x.

i
i

lp_pitagoras
2009/8/17
i page 61
Estilo OBMEP

i
i

N SEC. 3.1: CAPTULO 1

61

18) Sejam m e n, base e altura do retngulo. Pelo enunciado temos

m+n = 12 e mn = 25. Portanto, o problema consiste em determinar graficamente dois nmeros, conhecendo sua soma
e seu produto. Observe que resolver este sistema equivale a
construir um tringulo retngulo conhecendo sua hipotenusa e
sua altura. Como a hipotenusa a = m + n = 12 e a altura

h = mn = 25 = 5, desenhamos uma semicircunferncia de


dimetro BC = a = 12 cm e uma paralela distando 5 cm desse
dimetro. A interseo dessa paralela com a circunferncia nos d
o vrtice A do ngulo reto do tringulo. O ponto H, projeo de

i
i

lp_pitagoras
2009/8/17
i page 62
Estilo OBMEP

i
i

62

 CAP. 3: SOLUES DOS PROBLEMAS

A sobre BC, fornece os segmentos BH = m e HC = n que so as


medidas do retngulo que procuramos.
19) A figura a seguir mostra dois tringulos retngulos justapostos. O

Teorema de Pitgoras, usado duas vezes, fornece x = a 14.

r
 2  2
20) Veja que x =
a 2 + a 3 . Portanto, x a hipotenusa

de um tringulo retngulo cujos catetos medem a 2 e a 3.


21)

Nestas figuras, u um segmento qualquer. Construindo a primeia2


b2
ra figura, encontramos m =
e construindo a segunda, n = .
u
u
Construindo agora um tringulo retngulo cujos catetos so m e
n, temos sua hipotenusa que
t=

a4
b4
1p 4
+
=
a + b4 .
u2 u2
u

i
i

lp_pitagoras
2009/8/17
i page 63
Estilo OBMEP

i
i

63

N SEC. 3.1: CAPTULO 1

Construmos, finalmente, a figura seguinte:

p
1p 4
4
Desta forma, x = ut =
u
a + b4 =
a4 + b4 . Repau
re que esta construo independente da unidade.
22) Observe a figura a seguir.

O Teorema de Pitgoras no tringulo retngulo construdo


na figura fornece: AB 2 = (R + r)2 (R r)2 , o que d

AB = 2 Rr. Para prosseguir, seja C o ponto de tangncia da


terceira circunferncia com a reta.

i
i

lp_pitagoras
2009/8/17
i page 64
Estilo OBMEP

i
i

64

 CAP. 3: SOLUES DOS PROBLEMAS

Se x o raio da terceira circunferncia, como AC + CB = AB,


temos:

2 Rx + 2 rx = 2Rr

o que d x =

Rr
.
R + r + 2 Rr

23)

Os tringulos BEH e BAC so semelhantes:

n
p
= . Como
a
c

c2
, temos c3 = a2 p, ou ainda, c6 = a4 p2 . Os tringua
m
q
b2
los CHF e CBA so semelhantes:
= . Como m =
,
a
b
a
3
2
6
4
2
2
2
2
temos b = a q, ou ainda, b = a q . Como b + c = a , temos
p
p
p
p

3
3
3
a4 p2 + 3 a4 q 2 = a6 , ou seja, 3 p2 + 3 q 2 = a2 .
n =

i
i

lp_pitagoras
2009/8/17
i page 65
Estilo OBMEP

i
i

N SEC. 3.2: CAPTULO 2

65

24) Na figura a seguir, sejam: BP C = , BCP = , P BA = e


P BC = . Assinale o ponto Q no segmento P C tal que CQ = b.
Consequentemente, QP = c.

Suponha > 90o . Ento, x2 > b2 + c2 , ou seja, x > a.


Observando-se os tringulos QCB e P BA, que tm dois lados em
comum, conclui-se que > . No tringulo BP C, como > 90 o ,
conclui-se que + < 90o . Mas como + = 90o , pois ABCD
um quadrado, ento < , uma contradio. Da mesma forma,
supor > 90o conduz a uma outra contradio equivalente. Logo,
< 90o .

3.2

Captulo 2

1) A figura permite a diviso por segmentos horizontais em partes


fceis de calcular.

i
i

lp_pitagoras
2009/8/17
i page 66
Estilo OBMEP

i
i

66

 CAP. 3: SOLUES DOS PROBLEMAS

As partes 1, 3 e 4 so trapzios e a parte 2 um paralelogramo.


Temos ento:
A1 =
A3 =

5+2
2 =7
2

6+5
4 = 22
2

A2 = 5 2 = 10
A4 =

6+5
3 = 16,5
2

A rea da figura 55,5.


2) A diagonal de um retngulo divide esse retngulo em dois tringulos congruentes, portanto de mesma rea.

Observando a figura acima e sendo S1 e S2 as reas dos dois retngulos sombreados, devemos ter S1 +A+B = S2 +A+B e, portanto,
S1 = S 2 .

i
i

lp_pitagoras
2009/8/17
i page 67
Estilo OBMEP

i
i

N SEC. 3.2: CAPTULO 2

67

3) A figura assim:

Se AB = CD e se os ngulos B e C so iguais, ento AD


paralela a BC. Portanto, ABCD um trapzio e ADE um
tringulo retngulo com ngulos de 30 o e 60o . As medidas so
fceis de calcular e a rea do pentgono :

66 3
12 + 8
2 3+
= 20 3 + 18 3 = 38 3
S=
= 65,74.
2
2

4) O semipermetro p =
fornece:
S=

5+7+8
= 10. A frmula de Heron
2

10(10 5)(10 7)(10 8) = 10 5 3 2 = 10 3 cm2 .

Observe agora a figura abaixo, onde I o centro da circunferncia


inscrita e r o seu raio.

i
i

lp_pitagoras
2009/8/17
i page 68
Estilo OBMEP

i
i

68

 CAP. 3: SOLUES DOS PROBLEMAS

A rea do tringulo ABC a soma das reas dos tringulos BIC,


CIA e AIB. Logo,

ar br cr
a+b+c
10 3 =
+
+
=
r = 10r
2
2
2
2

e ento, r = 3cm.
5) Os tringulos ABC e ADC so congruentes. Logo, ambos possuem
rea igual a 1/2. Os segmentos DP , DQ e DR dividem o tringulo
ADC em quatro tringulos de mesma rea. O tringulo DP Q tem
rea 1/8.
6) O ponto G, interseo das medianas, o baricentro e, como se sabe,
GM/BM = 1/3. Como M B mediana no tringulo ABC, ento
a rea do tringulo ABC (ABM ) = 1/2. Como M N mediana
no tringulo ABM , ento (BM N ) = 1/4. Como GM/BM = 1/3,
ento (GM N )/(BM N ) = 1/3. Assim,
(GM N ) = (1/3)(1/4) = 1/12.

i
i

lp_pitagoras
2009/8/17
i page 69
Estilo OBMEP

i
i

N SEC. 3.2: CAPTULO 2

69

7)

A razo de semelhana entre os tringulos ADE e ABC


AD/AB = 2/3. Ento, a razo entre suas reas (2/3) 2 . Se
S a rea do tringulo ABC, ento S1 /S = 4/9. Logo, S1
menor que a metade de S e, portanto, S 2 maior que a metade
de S. Da, S2 > S1 .
8)

i
i

lp_pitagoras
2009/8/17
i page 70
Estilo OBMEP

i
i

70

 CAP. 3: SOLUES DOS PROBLEMAS

(a) Os tringulos BAC e DAC tm mesma rea, pois possuem


mesma base e mesma altura. Como ambos tm em comum
a parte P AB, ento os tringulos P AB e P CD tm mesma
rea.
10 7
(b) A rea do tringulo ABD igual a
= 35. Obser2
vando a semelhana dos tringulos P BD e P AC, temos que
P D/AP = 7/18. Logo,
AD = AP + P D = AP + (7/18)AP = (25/18)AP e
P D/AD = 7/25. Ento,
(P BD)
PD
7
49
=
=
e (P DB) =
= 9,8.
(ABD)
AD
25
5
9)

O hexgono regular pode ser dividido em 6 tringulos equilteros,


como mostra a figura acima. A razo de semelhana entre um
desses tringulos e o tringulo equiltero de lado unitrio 12.
Logo, a razo entre suas reas 122 = 144. Cabem, portanto, 144
tringulos de lado unitrio dentro de cada tringulo de lado 12.
Assim, dentro do hexgono, cabero 144 6 = 864 tringulos de
lado unitrio.

i
i

lp_pitagoras
2009/8/17
i page 71
Estilo OBMEP

i
i

N SEC. 3.2: CAPTULO 2

71

10) As circunferncias de dimetros AB e BC cortam-se no centro do


quadrado, ponto de interseo das diagonais. Observe a figura a
seguir e conclua que a rea sombreada igual rea do tringulo
OBC, ou seja, 1/4 da rea do quadrado.

11)

A parte coberta o quadriltero DP QC. Mas o tringulo DP C


tem rea igual metade da rea do retngulo. Logo, a parte
coberta tem rea maior que metade da rea do retngulo.

i
i

lp_pitagoras
2009/8/17
i page 72
Estilo OBMEP

i
i

72

 CAP. 3: SOLUES DOS PROBLEMAS

12) Seja ABCD o nosso quadriltero com o ponto P sobre o lado AB.
Traamos a reta CD.

Observe a figura acima. Traamos AE paralela a P D e BF paralela a P C. Assim, os tringulos P AD e P ED possuem mesma
rea e tambm os tringulos P BC e P F C possuem mesma rea.
Logo, a rea do quadriltero ABCD igual rea do tringulo
P EF . Dividir a rea de ABCD em duas partes iguais por uma
reta passando por P ento o mesmo que dividir a rea de P EF
em duas partes iguais por uma reta passando por P . Mas isto
fcil. Basta traar a mediana do tringulo P EF . Assinale
ento o ponto M , mdio de EF , e a reta P M divide tanto o
tringulo P EF quanto o quadriltero ABCD em duas partes de
mesma rea. O problema est resolvido.
Nota: Na construo acima, o ponto M , mdio de EF ficou
no interior do lado CD do quadriltero e a soluo est perfeita.
E se isto no acontecesse? Fica o desafio para o leitor encontrar a
soluo neste caso.

i
i

lp_pitagoras
2009/8/17
i page 73
Estilo OBMEP

i
i

N SEC. 3.2: CAPTULO 2

73

13) Observe a figura a seguir.

O hexgono est dividido em 24 tringulos equilteros iguais e o


tringulo M N P contm 9 deles. A razo entre a rea do tringulo
e do hexgono 9/24, ou seja, 3/8.
14) Observe a figura a seguir. O tringulo ABC tem rea igual
metade da rea do hexgono.

i
i

lp_pitagoras
2009/8/17
i page 74
Estilo OBMEP

i
i

74

 CAP. 3: SOLUES DOS PROBLEMAS

15)

Seja h a altura do tringulo menor. A figura mostra trs tringulos


semelhantes: um de rea A e altura h, outro de rea 2A e altura
h+x e o terceiro de rea 3A e altura 1. Como a razo entre as reas
de tringulos semelhantes o quadrado da razo de semelhana
temos, entre o primeiro e o terceiro,
A
=
3A

 2
h
1

ou seja

h=

1
.
3

Agora, entre o segundo e o terceiro, temos:




h+x
1

A
=
3A
Portanto, x =

2

ou seja

h+x=

1
.
3

1
.
3

16) Se A agudo (1a figura) ento a altura relativa a AB


h = AC senA.
Ento, a rea do tringulo ABC
1
1
S = AB h = AB AC senA. Se A obtuso (2a figura),
2
2
ento a altura relativa a AB h = AC sen( A) = AC senA
e a frmula verdadeira. Se A reto, senA = 1 e a frmula vale.

i
i

lp_pitagoras
2009/8/17
i page 75
Estilo OBMEP

i
i

N SEC. 3.2: CAPTULO 2

75

17) Na figura dada nesse exerccio, seja o ngulo do vrtice superior


do tringulo. Sendo T = A + B a rea do tringulo maior temos:

1
10 9 sen
A
1
= 2
= .
1
T
2
12 15 sen
2
Se a rea A a metade da rea do tringulo, ento a rea B
tambm . Logo, a razo entre as reas A e B igual a 1.

i
i

lp_pitagoras
2009/8/17
i page 76
Estilo OBMEP

i
i

76

 CAP. 3: SOLUES DOS PROBLEMAS

18)

A ideia mais simples parece que calcular a rea do retngulo e


subtrair a rea dos dois tringulos retngulos vazios. O retngulo
tem rea 10 12 = 120 e cada tringulo retngulo tem um cateto
igual a 6 e outro igual a x. Uma simples semelhana nos d
x
6
=
12
8
ou seja, x = 9. Ento, a rea dos dois tringulos juntos
6 9 = 54 e a rea sombreada igual a 120 54 = 66.
19)

i
i

lp_pitagoras
2009/8/17
i page 77
Estilo OBMEP

i
i

77

N SEC. 3.2: CAPTULO 2

Veja a figura anterior. O ponto Q a interseo de AM com BN


e traamos por N a perpendicular N E a AB, que cortou AM em
P . Fazendo BM = M C = 2a temos P E = a e N P = 3a. Como
os tringulos QPN e QMB so semelhantes, se fizermos QM = 2b,
teremos P Q = 3b e AP = 5b. Vamos representar por (XY Z...) a
rea do polgono XY Z... Como o quadrado ABCD tem lado 1,
a rea do tringulo ABM igual a 1/4. Vamos agora calcular a
razo entre as reas dos tringulos ABQ e ABM .
(ABQ)
8b
4
=
= .
(ABM )
10b
5
4 1
1
= e assim calculamos a rea de uma das
5 4
5
1 1
1
partes. A rea de BQM a diferena: (BQM ) = = . Os
4 5
20
tringulos ABM e BCN so congruentes e, portanto, tm mesma
rea. Logo a rea do quadriltero M CN Q a mesma rea do
1
tringulo ABQ. Logo, (M CN Q) = .
5
1
Como a rea do tringulo ADN , ento a rea do tringulo
4
AQN


1 1
1
1
3
(AQN ) = 1
+ +
+
= .
5 5 20 4
10
Logo, (ABQ) =

O problema est resolvido. Considerando a rea do quadrado igual


a 100, as reas das partes podem ser vistas na figura a seguir.

i
i

lp_pitagoras
2009/8/17
i page 78
Estilo OBMEP

i
i

78

 CAP. 3: SOLUES DOS PROBLEMAS

20) Observe a figura do problema

Como a rea do tringulo ABC igual a 1, a rea de ABD


1/3. Vamos, inicialmente, calcular a razo AM/AD, o que vai nos
permitir encontrar a rea do tringulo ABM .
Traamos GE, paralela a BC, que corta AD em H. Fazendo
BD = 3a, como AG/AB = 2/3, temos GH = 2a. Como DC
o dobro de BD, ento HE o dobro de GH, ou seja, HE = 4a.
Mas, os tringulos M HE e M DB so semelhantes e, portanto,

i
i

lp_pitagoras
2009/8/17
i page 79
Estilo OBMEP

i
i

79

N SEC. 3.2: CAPTULO 2

a razo HE/BD igual a razo HM/M D. Faamos ento


HM = 4b e M D = 3b. Ainda, como AG o dobro de GB,
ento AH o dobro de HD e, consequentemente, AH = 14b.
Temos ento a razo que procurvamos:
18b
6
AM
=
= .
AD
21b
7
A razo entre as reas dos tringulos ABM e ABD igual a razo
entre AM e AD. Logo,
(ABM )
AM
6
=
=
ABD
AD
7
e como (ABD) =

1
6 1
2
, temos (ABM ) = = .
3
7 3
7

Este foi o passo importante do problema e, de forma inteiramente


anloga, conclumos que os tringulos BCN e CP A tm tambm
2
rea igual a . A rea do tringulo central igual rea de ABC
7
subtrada das reas dos tringulos ABM , BCN e CAP . Portanto,
(M N P ) = 1 3

2
1
= .
7
7

Um belo e inesperado resultado.


21) 104 8 + 62 000 = 62 832 = 210 000. Isto d = 3,1416.
22) A rea de ABC a soma das reas de ABD e ADC. Seja x = AD.
Assim:
1
1
1
6 4 sen(120o ) = x sen(60o ) + 4 x sen(60o ).
2
2
2

i
i

lp_pitagoras
2009/8/17
i page 80
Estilo OBMEP

i
i

80

 CAP. 3: SOLUES DOS PROBLEMAS

Como sen(120o ) = sen(60o ) ficamos com 24 = 10x, ou seja,


x = 2,4.
23) A rea do segmento circular correspondente ao arco de comprimento x igual rea do setor circular, cujo ngulo central x
(em radianos) menos a rea do tringulo que tem dois lados iguais
ao raio com ngulo x entre eles. Ento, a rea do segmento circular
:
x1 1
x senx
S=
1 1 senx =
.
2
2
2

24) Observe a figura a seguir, onde traamos o dimetro CD.

i
i

lp_pitagoras
2009/8/17
i page 81
Estilo OBMEP

i
i

81

N SEC. 3.2: CAPTULO 2

Como o arco AB mede 80o e o arco BC mede 50o , conclumos


que o arco DA mede 50o . Os arcos BC e DA so iguais e esta
feliz coincidncia nos permite concluir que a reta AB paralela
ao dimetro CD. Assim, podemos mover C at O, centro da
circunferncia, e a rea da regio que devemos calcular no se
altera. A rea que procuramos ento igual rea de um setor
cujo ngulo central mede 80o . Isto fcil,
s=

80 2 2
1 =
.
360
9

25) A rea que devemos calcular est na figura a seguir.

A rea do crculo . Sejam H a rea do hexgono regular


e T a rea do tringulo equiltero inscritos nessa circunferncia. Vamos utilizar aqui o resultado do Problema 14: a rea
do hexgono regular o dobro da rea do tringulo equiltero,
ambos inscritos na mesma circunferncia. Isto quer dizer que
H = 2T .
A rea que vamos calcular a diferena entre as reas dos segmen-

i
i

lp_pitagoras
2009/8/17
i page 82
Estilo OBMEP

i
i

82

 CAP. 3: SOLUES DOS PROBLEMAS

tos circulares cujas cordas so CD e AB. Temos ento:


S=

T
H
2 2T + H

=
= .
3
6
6
6

26) Sejam A, B e C os centros das trs circunferncias pequenas e seja


O, o centro da circunferncia grande. O ponto O o centro do
tringulo equiltero ABC (de lado igual a 2) e, como sabemos,
sua distncia a cada um dos vrtices igual a 2/3 de sua altura.
Temos ento:

2 2 3
2 3
OA =
=
.
3
2
3

Assim, o raio da circunferncia grande

2 3+3
2 3
+1=
.
R=
3
3

A rea S da regio central igual rea do tringulo ABC subtrada de trs setores de 60o . Mas, esses trs setores formam um

i
i

lp_pitagoras
2009/8/17
i page 83
Estilo OBMEP

i
i

83

N SEC. 3.2: CAPTULO 2

semicrculo. Ento,

22 3 12
S=

= 3 .
4
2
2
O triplo da rea A assinalada na figura igual rea do crculo
grande subtrada dos trs crculos pequenos e da regio central.
Portanto,



R2 3
3
2 .
A=
3
Substituindo o valor de R, isto d, aps alguns clculos,
A=



1 
8 31 6 3 .
18

27)

Sejam M e N os centros das semicircunferncias, seja P o centro


da circunferncia e seja P R = P S = P T = x o seu raio. fcil
ver que M A = M R = 2, M N = 1 e N T = 3x. O semipermetro
do tringulo M N P
p=

2+x+3x+1
= 3.
2

i
i

lp_pitagoras
2009/8/17
i page 84
Estilo OBMEP

i
i

84

 CAP. 3: SOLUES DOS PROBLEMAS

Podemos ento calcular a rea do tringulo M N P de duas formas:


pela frmula de Heron e pela metade do produto da base M N pela
altura P S. Assim,
p

3(3 2 x)(3 3 + x)(3 1) =

Da, 3(1 x)x 2 =

1x
.
2

x2
24
e, consequentemente, x =
= 0,96.
4
25

i
i

lp_pitagoras
2009/8/17
i page 85
Estilo OBMEP

i
i

Para saber mais


WAGNER, E. Construes Geomtricas SBM.
Aborda as construes elementares, mtodo dos lugares geomtricos,
mtodo algbrico, reas, construes aproximadas, transformaes
geomtricas e traz um apndice onde se mostra quais so as construes possveis com rgua e compasso.
LIMA, Elon Meu professor de Matemtica SBM.
Coleo de artigos interessantssimos sobre assuntos diversos da matemtica elementar incluindo o Teorema de Pitgoras e aplicaes.
WAGNER, E. Usando reas RPM 21.
Mostra diversas demonstraes de propriedades geomtricas usando
reas, incluindo o teorema da bissetriz e frmulas trigonomtricas.

Na internet
http://en.wikipedia.org/wiki/Pythagorean_theorem
Contm diversas demonstraes do Teorema de Pitgoras incluindo
a mais antiga, que aparece no livro de Euclides (Os Elementos, sc 3
a.C.)
http://www.frontiernet.net/ imaging/pythagorean.html
Contm a demonstrao de Perigal (1873) que a forma mais econmica de dividir o quadrado construdo sobre a hipotenusa em partes que
85

i
i

lp_pitagoras
2009/8/17
i page 86
Estilo OBMEP

i
i

preenchem os dois quadrados menores. Pode ser manipulado.


http://www.ies.co.jp/math/java/geo/pythathr/pythathr.html
Possui diversas aplicaes interessantes e interativas, permitindo a
manipulao do usurio em problemas como: a menor distncia entre
pontos da superfcie de um paraleleppedo, o problema de Hipcrates,
o teorema da mediana de um tringulo qualquer, fractais com Pitgoras e muito mais.
http://www.jimloy.com/geometry/construc.htm
Mostra as construes elementares com rgua e compasso. timo para o iniciante.
http://www.nvcc.edu/home/tstreilein/constructions/Circle/circle4.htm
Mostra diversas construes elementares (e outras nem tanto) com a
possibilidade de usar um programa de geometria dinmica. Muito
educativo e interessante.

86

i
i

inducaofinal
2009/6/30
i page 1
Estilo OBMEP

i
i

Induo Matemtica
Abramo Hefez

i
i

inducaofinal
2009/6/30
i page 2
Estilo OBMEP

i
i

Texto j revisado pela nova ortografia.

i
i

inducaofinal
2009/6/30
i page 3
Estilo OBMEP

i
i

Sobre o Autor
Abramo Hefez nasceu no Egito, mas brasileiro por opo e carioca de corao. Cursou o ginasial e cientfico no Rio de Janeiro,
graduou-se na PUC-Rio em Matemtica e prosseguiu seus estudos
na Universidade de Pisa, Itlia e nos Estados Unidos, doutorando-se,
em Geometria Algbrica no Massachusetts Institute of Technology.
Professor Titular no Instituto de Matemtica da Universidade Federal Fluminense, onde desenvolve atividades de pesquisa e leciona
na graduao e ps-graduao. Foi eleito recentemente membro da
Academia Brasileira de Cincias.

i
i

inducaofinal
2009/6/30
i page 4
Estilo OBMEP

i
i

inducaofinal
2009/6/30
i page i
Estilo OBMEP

i
i

Sumrio
1 Induo Matemtica

1.1

O Princpio de Induo Matemtica

. . . . . . . . . .

1.2

Definio por Recorrncia . . . . . . . . . . . . . . . .

14

1.3

Progresses . . . . . . . . . . . . . . . . . . . . . . . .

24

2 Induo e Mundo Material

31

2.1

A Torre de Hani . . . . . . . . . . . . . . . . . . . . .

31

2.2

O Enigma do Cavalo de Alexandre . . . . . . . . . . .

35

2.3

Descobrindo a Moeda Falsa . . . . . . . . . . . . . . .

37

2.4

A Pizza de Steiner . . . . . . . . . . . . . . . . . . . .

38

2.5

Os Coelhos de Fibonacci . . . . . . . . . . . . . . . . .

40

3 Induo e Matemtica

45

3.1

Somatrios . . . . . . . . . . . . . . . . . . . . . . . .

45

3.2

Binmio de Newton . . . . . . . . . . . . . . . . . . . .

54

i
i

inducaofinal
2009/6/30
i page ii
Estilo OBMEP

i
i

ii

SUMRIO

3.3

Princpio do Menor Inteiro . . . . . . . . . . . . . . . .

59

3.4

O Princpio das Gavetas . . . . . . . . . . . . . . . . .

68

3.5

Desigualdades . . . . . . . . . . . . . . . . . . . . . . .

73

Respostas

83

i
i

inducaofinal
2009/6/30
i page iii
Estilo OBMEP

i
i

Introduo
Se algum me perguntasse o que que todo estudante de Ensino
Mdio deveria saber de matemtica, sem sombra de dvida, o tema
Induo figuraria na minha lista.
com o conceito de Induo que se estabelece o primeiro contato com a noo de infinito em Matemtica, e por isso ele muito
importante; porm, , ao mesmo tempo, sutil e delicado.
O material aqui apresentado uma pequena seleo de assuntos
relacionados com esse tema, cujo desenvolvimento se espalha por cerca
de dois mil anos, originando-se nos magnficos trabalhos dos Gregos
Antigos, que tm em Os Elementos de Euclides, de aproximadamente
300 a.C., o seu ponto culminante.
Estas notas se destinam a voc, aluno do Ensino Mdio, que est
envolvido em atividades promovidas pela OBMEP. Elas cobrem assuntos que provavelmente no lhe foram ensinados, pelo menos com
este grau de detalhe nem de profundidade, na escola, mas que, na
minha opinio, como mencionado acima, deveriam fazer parte de sua
bagagem cultural.
No tenho a expectativa de que voc absorva todo o material
iii

i
i

inducaofinal
2009/6/30
i page iv
Estilo OBMEP

i
i

iv
aqui apresentado numa primeira leitura, pois ele possui um grau de
abstrao um pouco maior do que o costumeiro nessa fase de sua
formao. Estude estas notas, procure entender os exemplos e, sobretudo, tente seriamente resolver os problemas, pois nunca esquea que
a Matemtica s se aprende fazendo. Se necessrio, volte a elas depois
de algum tempo, pois, assim procedendo, voc estar plantando uma
semente que lhe trar valiosos frutos.
Finalmente, no poderia encerrar essa introduo antes de agradecer Coordenao da OBMEP pelo convite para escrever este texto e
ao meu colega Dinamrico Pereira Pombo Jr. pela leitura cuidadosa
do manuscrito.
Niteri, julho de 2007.
Abramo Hefez
Departamento de Matemtica Aplicada
Universidade Federal Fluminense

i
i

inducaofinal
2009/6/30
i page v
Estilo OBMEP

i
i

Para o Professor
O nosso ponto de vista, nessas notas, que o estudante do Ensino
Mdio tem, de modo intuitivo e bastante vago, uma certa familiaridade com os nmeros, sejam eles naturais, inteiros, racionais ou reais.
Apesar disso, ele no tem a menor dvida sobre a sua existncia (as
dvidas so em geral de outra natureza: racionais versus irracionais)
e conhece bem algumas de suas propriedades como, por exemplo, o
fato desses conjuntos possurem uma adio e uma multiplicao com
as propriedades usuais. Optamos por no ignorar esse conhecimento;
muito pelo contrrio, utiliz-lo-emos como ponto de partida (ou seja,
implicitamente, como axioma zero) do nosso estudo.
Enfatizamos, logo no incio do texto, que esse conhecimento insuficiente para provar qualquer fato significativo. Mostramos ento,
na melhor tradio das teorias axiomticas, como, isolando algumas
propriedades (no nosso caso, as propriedades (1), (2) e (3), no incio do Captulo 1) que caracterizam os nmeros naturais dentro do
conjunto dos nmeros reais, possvel demonstrar muitas das suas
demais propriedades. Assim, esperamos convencer o jovem leitor da
necessidade de fundamentar melhor os seus conceitos e das vantagens
do mtodo axiomtico.
Decidimos, deliberadamente, nessas notas no descrever a trajetria do desenvolvimento dos nmeros reais e de sua fundamentao rigorosa, pois, nesse caso, o caminho seria longo e certamente
prematuro para a grande maioria dos leitores aos quais se destinam
estas notas. Por outro lado, se tivssemos iniciado a exposio com
os axiomas de Peano, teramos que arcar com o nus da construo

i
i

inducaofinal
2009/6/30
i page vi
Estilo OBMEP

i
i

vi
das operaes de adio e de multiplicao e da prova de suas propriedades, trabalho esse que consumiria algum esforo e desinteressaria a maioria dos leitores. Por outro lado, para poder prosseguir
com as notas, a um certo momento, teramos de aceitar a existncia
dos nmeros reais, pois estes so livremente utilizados no texto, o que
recairia no mesmo impasse do incio.
A ttulo de conforto para os mais ortodoxos sobre os Fundamentos
da Matemtica, pedimos que imaginem que o que estamos fazendo
moralmente (i.e. de modo implcito) nestas notas axiomatizar a
existncia dos nmeros reais como corpo ordenado completo (veja
Elon Lages Lima, Anlise Real, Volume 1, Seo 3, Captulo 2) e
admitir que N subconjunto de R (ib. Teorema 3 (i), pgina 17),
que ser por ns caracterizado univocamente por trs propriedades
explicitadas logo no incio do texto.

i
i

inducaofinal
2009/6/30
i page 1
Estilo OBMEP

i
i

Captulo 1

Induo Matemtica
Dentre todos os nmeros que o ser humano j considerou, os
nmeros naturais foram os primeiros a serem criados, inicialmente
com o intuito de contar. Apesar desses nmeros serem os mais simples, isso, absolutamente, no quer dizer que eles sejam totalmente
entendidos, havendo ainda muitos mistrios que os cercam a serem
desvendados.

1.1

O Princpio de Induo Matemtica

Mas, afinal, o que o conjunto N dos nmeros naturais?


Bem, podemos intuitivamente descrev-lo dizendo quais so os
seus elementos; eles so os nmeros reais da forma:
1, 2 = 1 + 1, 3 = 2 + 1 = (1 + 1) + 1, 4 = 3 + 1 = (1 + 1 + 1) + 1,

i
i

inducaofinal
2009/6/30
i page 2
Estilo OBMEP

i
i

CAP. 1: INDUO MATEMTICA

Ocorre, porm, que dificilmente poderemos provar alguma propriedade desses nmeros utilizando apenas esta descrio, pois, apesar de sabermos intuitivamente quais so os nmeros que os pontinhos
acima representam, teramos dificuldade de descrev-los de modo suficientemente explcito.
Uma alternativa consiste em dar algumas propriedades que caracterizem de modo inequvoco o conjunto dos naturais dentro do conjunto dos nmeros reais.
Inicialmente, considere um subconjunto S dos nmeros reais que
possui as seguintes propriedades:
(1) S contm o nmero 1.
(2) Toda vez que S contm um nmero n, ele necessariamente contm
o nmero n + 1.
(3) No existe subconjunto prprio de S satisfazendo as condies (1)
e (2).
Em outras palavras, (3) nos diz que se S possui as propriedades
(1), (2) e (3), acima, e se S 0 um subconjunto de S que possui as
propriedades (1) e (2), ento S 0 = S.
Vamos provar que se existe um subconjunto S dos nmeros reais
satisfazendo s trs condies acima, ento esse conjunto nico. De
fato, se S1 e S2 so dois os subconjuntos, temos que S1 S2 possui
as propriedades (1) e (2), logo pela propriedade (3) segue que
S1 = S1 S2 = S2 .

i
i

inducaofinal
2009/6/30
i page 3
Estilo OBMEP

i
i

N SEC. 1.1: O PRINCPIO DE INDUO MATEMTICA

No estgio em que estamos no temos como provar que tal conjunto S existe. Portanto, admitiremos o seguinte axioma:
Axioma: Existe um subconjunto dos reais que possui as propriedades
(1), (2) e (3).
Esse nico subconjunto ser chamado de conjunto dos nmeros
naturais e denotado por N.
A propriedade (3) o que se chama de Princpio de Induo
Matemtica. Mais precisamente:
Princpio de Induo Matemtica: Dado um subconjunto S do
conjunto dos nmeros naturais N, tal que 1 pertence a S e sempre
que um nmero n pertence a S, o nmero n + 1 tambm pertence a
S, tem-se que S = N.
Esta simples propriedade fornece uma das mais poderosas tcnicas
de demonstrao em Matemtica: a demonstrao por induo.
Suponha que seja dada uma sentena matemtica P (n) que dependa de uma varivel natural n, a qual se torna verdadeira ou falsa
quando substitumos n por um nmero natural dado qualquer. Tais
sentenas sero ditas sentenas abertas definidas sobre o conjunto dos
naturais.
A seguir damos alguns exemplos de sentenas abertas definidas
sobre N:
(a)

P (n) : n par.
claro que a afirmao P (1) falsa, pois ela diz que 1 par;

i
i

inducaofinal
2009/6/30
i page 4
Estilo OBMEP

i
i

CAP. 1: INDUO MATEMTICA

P (3), P (5) e P (9) so falsas, pois afirmam, respectivamente, que 3, 5


e 9 so pares.
Por outro lado, tambm claro que P (2), P (4), P (8) e P (22) so
verdadeiras, pois 2, 4, 8 e 22 so pares.
(b)

P (n) : n mltiplo de 3.

Temos, por exemplo, que P (1), P (2), P (4) e P (5) so falsas, enquanto P (3) e P (6) so verdadeiras.
(c)

P (n) : 1 + 3 + 5 + 7 + + (2n 1) = n2 .
Temos que P (1), P (2), P (3), P (4), . . . , P (10) so verdadeiras.

Aqui sabemos precisamente o que significa a sentena aberta P (n),


apesar dos pontinhos na sua definio. Ela significa:
A soma dos n primeiros nmeros mpares igual a n2 .
Voc consegue visualizar algum nmero natural m tal que P (m)
seja falsa? Bem, aps mais algumas tentativas, voc se convencer
de que esta frmula tem grandes chances de ser verdadeira para todo
nmero natural n; ou seja, P (n) verdadeira para todo n N.
(d)

P (n) : n2 n + 41 um nmero primo, para todo n N.

fcil verificar que as sentenas P (1), P (2), P (3) so verdadeiras.


Com algum trabalho, possvel ir alm, verificando tambm que P (4),
P (5), . . ., P (35) so verdadeiras.
Portanto, plausvel que tenhamos encontrado um polinmio cujos valores nos nmeros inteiros sejam sempre nmeros primos.

i
i

inducaofinal
2009/6/30
i page 5
Estilo OBMEP

i
i

N SEC. 1.1: O PRINCPIO DE INDUO MATEMTICA

Mais alguns testes para confirmar a nossa suspeita? L vai, P (36),


P (37), P (38) e P (40) so verdadeiras.
Podemos parar por aqui e nos sentir felizes com a nossa descoberta? Bom, para satisfazer os mais cticos, faremos s mais um teste
com n = 41.
Note que 412 41 + 41 = 412 no primo. Logo, para a nossa
desiluso, P (41) falsa!
Para a sua informao, pode-se provar que no existe nenhum
polinmio em uma varivel com coeficientes inteiros cujos valores nos
naturais sejam sempre nmeros primos. Portanto, no havia a priori
nenhuma chance de P (n) ser verdadeira para todo nmero natural n.
Como provar ento que uma sentena aberta definida sobre os
naturais sempre verdadeira? Voc h de convir que no seria possvel
testar, um por um, todos os nmeros naturais, pois eles so em nmero
infinito. Portanto, ser preciso encontrar algum outro mtodo.
Vamos a seguir expor a tcnica da demonstrao por induo
matemtica que resolver esse nosso problema.
Seja P (n) uma sentena aberta sobre os naturais e denote por V o
seu conjunto verdade em N, isto , o subconjunto de N, definido como
V = {n N; P (n) verdadeira}.
Para provar que P (n) verdadeira para todo n N, basta mostrar
que V = N.
Isso pode ser feito usando o Princpio de Induo Matemtica.

i
i

inducaofinal
2009/6/30
i page 6
Estilo OBMEP

i
i

CAP. 1: INDUO MATEMTICA

Basta, para isto, mostrar que 1 pertence a V e que n + 1 pertence a


V , toda vez que n pertence a V .
Provamos, assim, o seguinte teorema:
Teorema 1.1.1 (Prova por Induo Matemtica). Seja P (n) uma
sentena aberta sobre N. Suponha que
(i) P (1) verdadeira; e
(ii) qualquer que seja n N, sempre que P (n) verdadeira, segue que
P (n + 1) verdadeira.
Ento, P (n) verdadeira para todo n N.
Vejamos como usar esse mtodo para mostrar a validade, para
todo natural n, da frmula
1 + 3 + + (2n 1) = n2 .
Observe que P (1) verdadeira, j que a frmula trivialmente
vlida para n = 1. Suponha agora que, para algum n natural, P (n)
seja verdadeira; ou seja, que
1 + 3 + + (2n 1) = n2 .
Queremos provar que P (n + 1) verdadeira. Somando 2n + 1, que
o prximo nmero mpar aps 2n1, a ambos os lados da igualdade
acima, obtemos a igualdade tambm verdadeira:
1 + 3 + + (2n 1) + (2n + 1) = n2 + (2n + 1) = (n + 1)2 .

i
i

inducaofinal
2009/6/30
i page 7
Estilo OBMEP

i
i

N SEC. 1.1: O PRINCPIO DE INDUO MATEMTICA

Isso mostra que P (n + 1) verdadeira, toda vez que P (n) verdadeira. Pelo teorema, a frmula vlida para todo nmero natural n.
Voc tem ideia de quando foi feita pela primeira vez a demonstrao acima? Bem, o primeiro registro que se tem de 1575 e foi
realizada por Francesco Maurolycos.
Note que, na demonstrao acima, poderia parecer que estamos
usando o fato de P (n) ser verdadeira para deduzir que P (n + 1)
verdadeira para em seguida concluir que P (n) verdadeira. O que
est ocorrendo? Estamos usando a tese para provar o teorema?
A resposta no! Preste bem ateno, pois essa a parte mais
delicada de toda a histria.
Dado um nmero natural n, temos duas possibilidades:
(a) P (n) verdadeira, ou (b) P (n) falsa.
A hiptese (ii) do Teorema no exige em absoluto que assumamos
P (n) verdadeira para todo n N, podendo eventualmente ser falsa
para algum valor de n, ou mesmo para todos os valores de n. O que
a hiptese (ii) exige que sempre que algum n pertena categoria
(a) acima, ento n + 1 tambm pertena a essa mesma categoria; no
exigindo nada quando n pertencer categoria (b).
Por exemplo, a sentena aberta P (n) : n = n + 1 satisfaz (por
vacuidade) hiptese (ii) do Teorema, j que nenhum n N pertence
categoria (a). O que falha para que o Teorema nos garanta que
P (n) verdadeira para todo n que a hiptese (i) no verificada,
pois P (1) : 1 = 2 falsa!
preciso ter clareza que a Induo Matemtica diferente da

i
i

inducaofinal
2009/6/30
i page 8
Estilo OBMEP

i
i

CAP. 1: INDUO MATEMTICA

induo emprica das cincias naturais, em que comum, aps um


certo nmero, necessariamente finito, de experimentos, enunciar leis
gerais que governam o fenmeno em estudo. Essas leis so tidas como
verdades, at prova em contrrio. Na matemtica, no h lugar para
afirmaes verdadeiras at prova em contrrio. A Prova por Induo
Matemtica trata de estabelecer que determinada sentena aberta
sobre os naturais sempre verdadeira.
A induo emprica foi batizada, de modo irnico, pelo
matemtico, filsofo e grande humanista ingls do sculo passado,
Bertrand Russel (1872-1970), de induo galincea, com base na
seguinte historinha:
Havia uma galinha nova no quintal de uma velha senhora. Diariamente, ao entardecer, a boa senhora levava milho s galinhas. No
primeiro dia, a galinha, desconfiada, esperou que a senhora se retirasse para se alimentar. No segundo dia, a galinha, prudentemente,
foi se alimentando enquanto a senhora se retirava. No nonagsimo
dia, a galinha, cheia de intimidade, j no fazia caso da velha senhora. No centsimo dia, ao se aproximar a senhora, a galinha, por
induo, foi ao encontro dela para reclamar o seu milho. Qual no foi
a sua surpresa quando a senhora pegou-a pelo pescoo com a inteno
de p-la na panela.
Exemplo 1.1.1. Queremos determinar uma frmula para a soma
dos n primeiros nmeros naturais.
Conta-se a seguinte histria sobre o matemtico alemo Carl
Friedrich Gauss (1777-1855)1 , quando ainda garoto. Na escola, o pro1

Gauss considerado um dos maiores gnios da matemtica de todos os tempos.

i
i

inducaofinal
2009/6/30
i page 9
Estilo OBMEP

i
i

N SEC. 1.1: O PRINCPIO DE INDUO MATEMTICA

fessor, para aquietar a turma de Gauss, mandou os alunos calcularem


a soma de todos os nmeros naturais de 1 a 100. Qual no foi a surpresa quando, pouco tempo depois, o menino deu a resposta: 5 050.
Indagado como tinha descoberto to rapidamente o resultado, Gauss,
ento com nove anos de idade, descreveu o mtodo a seguir.
Sendo
Sn = 1 + 2 + + n,
o objetivo encontrar uma frmula fechada2 para Sn .
Somando a igualdade acima, membro a membro, com ela mesma,
porm com as parcelas do segundo membro em ordem invertida, temos
que
Sn
Sn

=
=

1
n

+
2
+ +
+ (n 1) + +

n
1

2Sn = (n + 1) + (n + 1) + + (n + 1)

Da segue-se que 2Sn = n(n + 1) e, portanto,


Sn =

n(n + 1)
.
2

Vamos ser crticos com relao prova acima. Para a maioria das
pessoas, essa prova parece impecvel, mas se algum nos perguntasse
o que est escondido atrs dos pontinhos, talvez nos sentssemos embaraados. Tambm, como ter absoluta certeza de que nada acontece
2
Uma frmula fechada, a grosso modo, uma frmula que depende dos dados
iniciais do problema e que permite calcular diretamente os valores do objeto em
estudo fazendo um nmero pequeno de contas.

i
i

inducaofinal
2009/6/30
i page 10
Estilo OBMEP

i
i

10

CAP. 1: INDUO MATEMTICA

fora do nosso controle, exatamente na imensa regio coberta pelos


pontinhos?
Para no pairar nenhuma dvida sobre o nosso resultado, vamos
provar a frmula utilizando Induo Matemtica.
Considere a sentena aberta sobre os naturais
P (n) : 1 + 2 + + n =
Note que
P (1) :

1=

n(n + 1)
.
2

(1.1)

1(1 + 1)
2

verdadeira.
Observe tambm que
P (n + 1) : 1 + 2 + + n + (n + 1) =

(n + 1)(n + 2)
.
2

Agora, suponhamos que para algum n N, tenhamos P (n) verdadeira, isto , a frmula (1.1) vlida para tal valor de n. Somando
n + 1 a ambos os lados dessa igualdade, temos que verdadeira a
igualdade
1 + 2 + + n + (n + 1) =
=
=

n(n + 1)
+n+1
2
n(n + 1) + 2(n + 1)
2
(n + 1)(n + 2)
,
2

o que estabelece a veracidade de P (n + 1).

i
i

inducaofinal
2009/6/30
i page 11
Estilo OBMEP

i
i

N SEC. 1.1: O PRINCPIO DE INDUO MATEMTICA

11

Pelo teorema, tem-se que a frmula P (n) verdadeira para todo


n N.
Exemplo 1.1.2. Queremos validar a frmula
P (n) : 12 + 22 + + n2 =
Note que
P (1) : 12 =

n(n + 1)(2n + 1)
.
6

(1.2)

1(1 + 1)(2 + 1)
6

verdadeira.
Suponha que, para algum n N, se tenha que P (n) verdadeira,
isto , (1.2) vlida. Somando (n + 1)2 a ambos os lados da igualdade
(1.2), temos que
12 + 22 + + n2 + (n + 1)2 =
=
=
=

n(n + 1)(2n + 1)
+ (n + 1)2
6
n(n + 1)(2n + 1) + 6(n + 1)2
6
(n + 1)[n(2n + 1) + 6(n + 1)]
6
(n + 1)[(n + 1) + 1][2(n + 1) + 1]
,
6

estabelecendo assim a veracidade de P (n + 1).


Portanto, a frmula vlida para todo n N.
Exemplo 1.1.3. Vamos provar que verdadeira, para todo n N, a

i
i

inducaofinal
2009/6/30
i page 12
Estilo OBMEP

i
i

12

CAP. 1: INDUO MATEMTICA

frmula:
P (n) :

1
1
n
1
+
+ +
=
.
1.2 2.3
n(n + 1)
n+1

(1.3)

Observemos inicialmente que


P (1) :

1
1
=
1.2
1+1

verdadeira.
Suponhamos que, para algum n, tem-se que P (n) verdadeira,
ou seja, que a frmula (1.3) seja verdadeira para esse valor de n.
1
, temos
Somando a ambos os lados dessa igualdade
(n + 1)(n + 2)
que
1
1
1
1
+
+ +
+
1.2 2.3
n(n + 1) (n + 1)(n + 2)

=
=

n
1
+
n + 1 (n + 1)(n + 2)
n+1
,
n+2

mostrando, assim, que P (n + 1) verdadeira.


Portanto, pelo Teorema 1.1.1, temos que a frmula vale para todo
n N.

Problemas
1.1.1 Mostre, por induo, a validade das seguintes frmulas:
(a) 1 22 + 32 + (1)n1 n2 = (1)n1

n(n + 1)
.
2

i
i

inducaofinal
2009/6/30
i page 13
Estilo OBMEP

i
i

N SEC. 1.1: O PRINCPIO DE INDUO MATEMTICA

13

1
(b) 12 + 32 + + (2n 1)2 = n(2n 1)(2n + 1).
3

(c) 13 + 23 + + n3 =

n(n + 1)
2

2
.

1.1.2 Mostre, por induo, a validade das seguintes frmulas:


(a)

1
1
n
1
+
+ +
=
.
1.3 3.5
(2n 1)(2n + 1)
2n + 1

(b)

1
1
1
1
n
+
+
+ +
=
.
1.4 4.7 7.10
(3n 2)(3n + 1)
3n + 1

(c)

1
1
1
1
n
+
+
+ +
=
.
1.5 5.9 9.13
(4n 3)(4n + 1)
4n + 1

(d)

1
1
1
n(n + 3)
+
+ +
=
.
1.2.3 2.3.4
n(n + 1)(n + 2)
4(n + 1)(n + 2)

(e)

22
n2
n(n + 1)
12
+
+ +
=
.
1.3 3.5
(2n 1)(2n + 1)
2(2n + 1)

1.1.3 Mostre, para n, m N, que


1 2 m + 2 3 m(m + 1) + + n(n + 1) (n + m 1) =
1
n(n + 1) (n + m).
m+1
Sugesto: Fixe m arbitrrio e proceda por induo sobre n.

i
i

inducaofinal
2009/6/30
i page 14
Estilo OBMEP

i
i

14

CAP. 1: INDUO MATEMTICA

1.1.4 Mostre que a soma dos cubos de trs nmeros naturais consecutivos sempre divisvel por 9.
Sugesto: Considere a sentena aberta
P (n) : n3 + (n + 1)3 + (n + 2)3 divisvel por 9,
e mostre, por induo, que ela verdadeira para todo n N.
1.1.5 Dada a sentena aberta em N:
P (n) : 1 + 2 + + n =

n(n + 1)
+ 1,
2

mostre que:
(i) Qualquer que seja n N, se P (n) verdadeira, ento P (n + 1)
verdadeira.
(ii) P (n) no verdadeira para nenhum valor de n N.

1.2

Definio por Recorrncia

Recorde que fizemos objees na seo anterior ao uso dos pontinhos nas demonstraes de algumas frmulas; no que sejamos contra, eles ajudam muito a representar situaes em que h um nmero
grande (eventualmente infinito) de objetos a serem descritos e a visualizar propriedades desses objetos.
Nessas notas, estamos tentando mostrar como se pode estabelecer um maior padro de rigor no tratamento de certos problemas
matemticos, mas isso no deve ser tomado ao p da letra. Certos argumentos informais, quando acompanhados de um raciocnio correto,

i
i

inducaofinal
2009/6/30
i page 15
Estilo OBMEP

i
i

N SEC. 1.2: DEFINIO POR RECORRNCIA

15

so corriqueiramente aceitos. Por exemplo, o argumento utilizado por


Gauss para somar os n primeiros nmeros naturais perfeitamente
aceitvel. Portanto, um conselho: use o formalismo para ajudar e no
para atrapalhar; nunca deixe ele se sobrepor criatividade, pois, em
regra, primeiro vem a descoberta, e depois, a formalizao.
Voltemos agora ao problema que queremos abordar. O que realmente significa uma expresso da forma
1 + 3 + 5 + + (2n 1),
que consideramos no Exemplo 1.1.1?
Apesar de intuirmos o que se quer dizer, isso formalmente ainda
no faz sentido, pois a operao de adio de nmeros definida para
um par de nmeros, e aqui temos n nmeros sendo somados de uma
s vez, alm do inconveniente dos pontinhos, claro. Para dar um
sentido preciso a esse tipo de expresso, vamos ver como a Induo
Matemtica pode nos ajudar.
Para definir uma expresso En , para todo nmero natural n, basta
definirmos E1 e mostrar como obter En+1 a partir de En , para todo
n N.
De fato, para verificar que temos efetivamente uma definio para
todo nmero natural n, consideremos a sentena aberta
P (n) : En est definido
e provemos, por Induo Matemtica, que P (n) verdadeira para
todo n N.

i
i

inducaofinal
2009/6/30
i page 16
Estilo OBMEP

i
i

16

CAP. 1: INDUO MATEMTICA

Por construo dos En , temos que:


(i) P (1) verdadeira.
(ii) Qualquer que seja n N, se P (n) verdadeira, ento P (n + 1)
tambm verdadeira.
Portanto, pelo Teorema 1.1.1, temos que P (n) verdadeira para
todo nmero natural n.
Nesse caso, dizemos que En foi definido por recorrncia.
Para continuarmos a nossa discusso, precisaremos de um conceito
que no introduzimos ainda, mas do qual voc certamente j ouviu
falar.
Voc sabe o que uma sequncia? Certamente voc j foi apresentado seguinte definio:
Seja a1 , a2 , . . . , an , . . . uma sequncia de nmeros em que cada
elemento an , a partir do segundo, igual ao anterior an1 somado
com um nmero constante r.
Isso o que se chama de Progresso Aritmtica.
Mas, o que uma sequncia em geral? Uma sequncia, como
sugere o nome, uma coleo de elementos de natureza qualquer,
ordenados. Na verdade, trata-se apenas de elementos de um conjunto
etiquetados com os nmeros naturais.
Etiquetar com os nmeros naturais os elementos de um conjunto

i
i

inducaofinal
2009/6/30
i page 17
Estilo OBMEP

i
i

N SEC. 1.2: DEFINIO POR RECORRNCIA

17

A, significa dar uma funo


a : N A
n 7 a(n)
A definio formal de uma sequncia em um conjunto A apenas
uma funo a de N em A.
Como uma funo dada quando se conhece a imagem de todos
os elementos do seu domnio, uma sequncia a pode ser representada
como
a(1), a(2), . . . , a(n), . . . ;
ou ainda, denotando a(n) por an , podemos represent-la por
(an ) : a1 , a2 , . . . , an , . . .
Quando dissermos que um conjunto A possui uma adio ou uma
multiplicao satisfazendo s leis bsicas da aritmtica, estaremos
supondo que em A est definida uma operao com propriedades
semelhantes correspondente operao nos reais.

Exemplo 1.2.1. Seja (an ) uma sequncia de elementos de um conjunto munido de uma adio sujeita s leis bsicas da aritmtica. Para
dar sentido s somas
Sn = a1 + a2 + + an ,
basta definir recorrentemente Sn .

i
i

inducaofinal
2009/6/30
i page 18
Estilo OBMEP

i
i

18

CAP. 1: INDUO MATEMTICA

Pomos S1 = a1 e, supondo Sn definido, definimos


Sn+1 = Sn + an+1 .
Somas como Sn sero tambm denotadas com a notao de somatrios:
n
X
Sn =
ai ,
i=1

que se l como somatrio quando i varia de 1 at n de ai .


Um conceito que se define naturalmente por recorrncia o fatorial
de um nmero natural.
Exemplo 1.2.2. Define-se o fatorial n! de um nmero natural n
como:
1! = 1, e (n + 1)! = n! (n + 1).
Outro conceito que, naturalmente, definido por recorrncia o
de potncia.
Exemplo 1.2.3. Seja a um elemento de um conjunto A munido de
uma multiplicao sujeita s leis bsicas da aritmtica. Vamos definir
as potncias an , com n N, por recorrncia.
Ponhamos a1 = a. Supondo an definido, defina
an+1 = an a.
Vamos estabelecer, por meio de induo, as propriedades usuais
das potncias.

i
i

inducaofinal
2009/6/30
i page 19
Estilo OBMEP

i
i

N SEC. 1.2: DEFINIO POR RECORRNCIA

19

Proposio 1.2.1. Sejam a, b A e m, n N. Ento,


(i) am an = an+m ;
(ii) (am )n = amn ;
(iii) (a b)n = an bn .

Demonstrao. Provaremos (i), deixando o restante como exerccio.


Fixemos a A e m N, arbitrariamente. Demonstremos a propriedade por induo sobre n.
Para n = 1, a propriedade vlida, pois, pelas definies,
am a1 = am a = am+1 .
Por outro lado, supondo que am an = am+n , temos que:
am an+1 = am (an a) = (am an ) a = am+n a = am+n+1 .
Isso, pelo Teorema 1.1.1, prova a nossa propriedade.

Exemplo 1.2.4. Vamos provar que 3 divide 5n +2 11n , nos inteiros,


para todo n N.
De fato, para n = 1, temos que 3 divide 51 + 2 111 = 27.
5n

Suponha, agora, que, para algum n 1, saibamos que 3 divide


+ 2 11n . Logo, existe um nmero inteiro a tal que
5n + 2 11n = 3a.

i
i

inducaofinal
2009/6/30
i page 20
Estilo OBMEP

i
i

20

CAP. 1: INDUO MATEMTICA

Multiplicando por 5 ambos os lados da igualdade acima, temos


5 3a = 5n+1 + 5 2 11n = 5n+1 + 2 11 11n 12 11n .
Da segue a igualdade
5n+1 + 2 11n+1 = 5 3a + 12 11n ,
cujo segundo membro divisvel por 3 por ser igual a 3(5a + 4 11n ).
Assim, provamos que 3 divide 5n+1 +211n+1 , o que, pelo Teorema
1.1.1, acarreta que 3 divide 5n + 2 11n , para todo nmero natural n.

Pode ocorrer que uma determinada propriedade seja vlida para


todos os nmeros naturais a partir de um determinado valor a, mas
no necessariamente para valores menores. Como proceder nesses
casos?
Por exemplo, como provar que a desigualdade 2n > n2 verdadeira
para todo valor de n natural maior do que ou igual a 5?
Fazemos isso baseados na seguinte pequena generalizao do Teorema 1.1.1:

Teorema 1.2.1. Seja P (n) uma sentena aberta sobre N, e seja


a N. Suponha que
(i) P (a) verdadeira, e
(ii) qualquer que seja n N, com n a, sempre que P (n) verdadeira, segue-se que P (n + 1) verdadeira.

i
i

inducaofinal
2009/6/30
i page 21
Estilo OBMEP

i
i

N SEC. 1.2: DEFINIO POR RECORRNCIA

21

Ento, P (n) verdadeira para todo nmero natural n a.


Demonstrao. Defina o conjunto
S = {m N; P (m + a 1) verdadeira}.
Por (i) temos que 1 S. Por outro lado, se m S, temos que
P (m+a1) verdadeira. Logo, por (ii), P (m+1+a1) verdadeira.
Portanto, m + 1 S. Em vista do Teorema 1.1.1, temos que S = N.
Consequentemente, P (n) verdadeira para todo n a.
Exemplo 1.2.5. Vamos mostrar que a desigualdade na sentena
aberta P (n) : 2n > n2 verdadeira, para todo nmero natural n 5.
Note que P (1) : 21 > 12 verdadeira, P (2) : 22 > 22 falsa,
P (3) : 23 > 32 falsa e P (4) : 24 > 42 falsa. Tudo isso no importa,
pois queremos verificar a veracidade dessa desigualdade para n 5.
De fato, temos que P (5) : 25 > 52 verdadeira. Seja n 5 tal
que 2n > n2 . Multiplicando ambos os lados da desigualdade acima
por 2, obtemos 2n+1 > 2n2 . Note que 2n2 > (n + 1)2 , se n 3, pois
tal desigualdade equivalente a n(n 2) > 1. Da, deduzimos que
2n+1 > (n + 1)2 , o que significa que P (n + 1) verdadeira, estabelecendo o resultado em vista do Teorema 1.2.1.
Exemplo 1.2.6. Vamos mostrar que a sentena aberta:
a equao 3x + 5y = n tem soluo em (N {0})2 ,
verdadeira para todo n 8.
De fato, ela verdadeira para n = 8, pois a equao 3x + 5y = 8
admite a soluo (x, y) = (1, 1).

i
i

inducaofinal
2009/6/30
i page 22
Estilo OBMEP

i
i

22

CAP. 1: INDUO MATEMTICA

Suponha agora que a equao 3x + 5y = n tenha uma soluo


(a, b) para algum n 8; isto , 3a + 5b = n. Note que, para qualquer
soluo (a, b), devemos ter a 1 ou b 1.
Se b 1, observando que 3 2 5 1 = 1, segue que
3(a + 2) + 5(b 1) = 3a + 5b + 3 2 5 1 = 3a + 5b + 1 = n + 1,
o que mostra que a equao 3x + 5y = n + 1 admite a soluo
(a + 2, b 1) em (N {0})2 .
Se, por acaso, b = 0, ento, a 3; usando a igualdade
3 3 + 5 2 = 1,
temos
3(a 3) + 5 2 = 3a 3 3 + 5 2 = 3a + 5b + 1 = n + 1,
o que mostra que a equao 3x + 5y = n + 1 admite a soluo
(a 3, b + 2) em (N {0})2 .
Mostramos assim que, em qualquer caso, a equao 3x+5y = n+1
admite soluo, sempre que a equao 3x+5y = n, para algum n 8,
tenha soluo. Como o resultado vale para n = 8, segue a concluso
desejada pelo Teorema 1.2.1.
Note que n0 = 8 o menor valor de n para o qual a equao tem
soluo para todo n n0 .

i
i

inducaofinal
2009/6/30
i page 23
Estilo OBMEP

i
i

23

N SEC. 1.2: DEFINIO POR RECORRNCIA

Problemas
1.2.1 Mostre, por induo, a validade das seguintes frmulas:
(a) 1.20 + 2.21 + 3.22 + + n.2n1 = 1 + (n 1)2n .

n1
1
1 2
1
nn1
(b)
1+
1+
1 +
=
.
1
2
n1
(n 1)!
(c) 1.1! + 2.2! + 3.3! + + n.n! = (n + 1)! 1.
1.2.2 Sejam a e b nmeros reais distintos. Mostre que, para todo
n N, vale a igualdade:
bn + abn1 + a2 bn2 + + an1 b + an =

bn+1 an+1
.
ba

1.2.3 Se sen 6= 0, mostre que, para todo n N, vale a igualdade:


cos cos 2 cos 22 cos 2n =

sen 2n+1
2n+1 sen

Sugesto: Use a frmula sen 2 = 2 sen cos .


1.2.4 Para todo n N, mostre que, nos inteiros,
(a) 80 divide 34n 1;

(b) 9 divide 4n + 6n 1;

(c) 8 divide 32n + 7;

(d) 9 divide n4n+1 (n + 1)4n + 1.

i
i

inducaofinal
2009/6/30
i page 24
Estilo OBMEP

i
i

24

CAP. 1: INDUO MATEMTICA

1.2.5 Mostre que:


(a) n! > 2n , se n 4.
(b) n! > 3n , se n 7.
(c) n! > 4n , se n 9.
1.2.6 Prove que, para todo n natural, vale a desigualdade:
1 3 5
2n 1
1

.
2 4 6
2n
3n + 1
1.2.7 Mostre que o nmero de diagonais de um polgono convexo de
n lados dado por
n(n 3)
.
dn =
2

1.2.8 Mostre que n0 = 32 o menor valor para o qual a equao


5x + 9y = n possui soluo em (N {0})2 para todo n n0 .

1.3

Progresses

Iremos agora, usando recorrncia, definir progresses aritmticas


e progresses geomtricas.
Exemplo 1.3.1. Uma Progresso Aritmtica (P.A.) uma sequncia
de nmeros (an ) tal que, a partir do segundo termo, cada termo an
igual ao anterior an1 somado a um nmero fixo r chamado de razo.

i
i

inducaofinal
2009/6/30
i page 25
Estilo OBMEP

i
i

25

N SEC. 1.3: PROGRESSES

Portanto, dado o primeiro termo a1 e define-se recorrentemente


an = an1 + r,

se n 2.

Para achar uma frmula fechada para o termo de ordem n da


sequncia, observe que
a2 = a1 + r,

a3 = a2 + r = a1 + 2r,

a4 = a3 + r = a1 + 3r.

Pelo mtodo da galinha de Bertrand Russel, j podemos adivinhar os prximos termos:


a5 = a4 + r = a1 + 4r, a6 = a1 + 5r, . . . , an = a1 + (n 1)r, . . .
Portanto, parece plausvel que a frmula para o termo geral de
uma P.A. de primeiro termo a1 e razo r seja
an = a1 + (n 1)r, para todo n N.
Vamos agora demonstrar essa frmula por induo.
Inicialmente, observe que a frmula verdadeira para n = 1, pois
ela se reduz igualdade a1 = a1 .
Suponha agora que a frmula seja correta para algum n N;
isto , que an = a1 + (n 1)r. Somando r a ambos os lados dessa
igualdade, segue a igualdade:
an+1 = an + r = a1 + (n 1)r + r = a1 + nr,

i
i

inducaofinal
2009/6/30
i page 26
Estilo OBMEP

i
i

26

CAP. 1: INDUO MATEMTICA

o que mostra que a frmula verdadeira para n + 1. Portanto, ela


correta para todo n N.
Note que, numa P.A., tem-se que
ai + ani+1 = [a1 + (i 1)r] + [a1 + (n i)r]
= a1 + a1 + (n 1)r

(1.4)

= a1 + an .
Agora, nos propomos a achar uma frmula para a soma
Sn = a1 + a2 + + an
dos n primeiros termos de uma P.A. (an ).
Vamos usar, para isso, o mtodo de Gauss que exibimos no Exemplo 1.1.1.
Somando a igualdade acima, membro a membro, com ela mesma,
porm com as parcelas do segundo membro em ordem invertida,
temos, por (1.4) que
Sn
Sn

=
=

a1
an

+
+

a2
an1

+ +
+ +

an
a1

2Sn = (a1 + an ) + (a2 + an1 ) + + (an + a1 )

Da, segue-se que 2Sn = (a1 + an )n e, portanto,


Sn =

(a1 + an )n
.
2

i
i

inducaofinal
2009/6/30
i page 27
Estilo OBMEP

i
i

27

N SEC. 1.3: PROGRESSES

Deixamos a validao dessa frmula por induo como exerccio.


Exemplo 1.3.2. Uma Progresso Geomtrica (P.G.) uma sequncia de nmeros (an ) tal que, a partir do segundo termo, cada termo an
igual ao anterior an1 multiplicado por um nmero fixo q chamado
de razo.
Portanto, dado o primeiro termo a1 e define-se recorrentemente
an = an1 q,

se n 2.

Para achar uma frmula fechada para o termo de ordem n da


sequncia, observe que
a2 = a1 q,

a3 = a2 q = a1 q 2 ,

a4 = a3 q = a1 q 3 ,

a5 = a4 q = a1 q 4 .

Novamente, pelo mtodo da galinha de Bertrand Russel, podemos adivinhar os prximos termos:
a6 = a1 q 5 ,

a7 = a1 q 6 ,

...,

an = a1 q n1 ,

...

Portanto, plausvel que a frmula para o termo geral de uma


P.G. de primeiro termo a1 e razo q seja an = a1 q n1 , para todo
n N.
Vamos demonstrar essa frmula por induo.
Inicialmente, observe que a frmula verdadeira para n = 1, pois
ela se reduz igualdade a1 = a1 .
Suponha, agora, que a frmula seja correta para algum n N,
isto , que an = a1 q n1 . Multiplicando por q ambos os lados dessa

i
i

inducaofinal
2009/6/30
i page 28
Estilo OBMEP

i
i

28

CAP. 1: INDUO MATEMTICA

igualdade, segue que


an+1 = an q = a1 q n1 q = a1 q n ,
o que mostra que a frmula correta para n + 1. Portanto, ela
correta para todo n N.
Vamos, a seguir, achar uma frmula para a soma Sn dos n
primeiros termos de uma P.G.
Vejamos se, animados pelo truque de Gauss, achamos uma
soluo inteligente para esse problema.
Escreva
Sn = a1 + a1 q + a1 q 2 + + a1 q n1 .
Note que

qSn Sn = a1 q + a1 q 2 + + a1 q n1 + a1 q n
a1 a1 q a1 q 2 a1 q n1
= a1 q n a1 .
Portanto,
Sn =

a1 q n a1
an q a1
=
.
q1
q1

Problemas
1.3.1 Ache uma frmula fechada para cada uma das somas:

i
i

inducaofinal
2009/6/30
i page 29
Estilo OBMEP

i
i

N SEC. 1.3: PROGRESSES

29

(a) 2 + 4 + + 2n.
(b) 2 + 5 + 8 + + (3n 1).
1.3.2 Ache uma frmula fechada para cada uma das somas:
(a) 2 + 4 + + 2n .
(b)

1 1
1
+ + + n.
2 4
2

Para quanto tende a soma em (b) quando o nmero de parcelas aumenta indefinidamente?
1.3.3 Uma vitria rgia encontra-se em um tanque de gua. Sabendo
que ela dobra de rea a cada dia e que, no final de 20 dias, ocupa toda
a superfcie do tanque, em qual dia ela ocupar a metade da superfcie
do tanque?
Comentrio: Esse problema admite duas solues, uma usando frmulas, outra usando a cabea.
1.3.4 Em uma cidade de 5 000 habitantes, algum resolve espalhar
um boato. Considerando que, a cada 10 minutos, uma pessoa capaz
de contar o caso para 3 pessoas desinformadas, determine em quanto
tempo toda a cidade fica conhecendo o boato.
1.3.5 Uma progresso aritmtico-geomtrica uma sequncia (an )
tal que a1 , q e r so nmeros reais dados, com q 6= 1, e, para todo
n N, tem-se que
an+1 = qan + r.

i
i

inducaofinal
2009/6/30
i page 30
Estilo OBMEP

i
i

30

CAP. 1: INDUO MATEMTICA

(a) Mostre que an = a1 q n1 + r

q n1 1
.
q1

(b) Se Sn = a1 + + an , mostre que


Sn = a1

qn 1
n
qn 1
+r
+r
.
2
q1
(q 1)
1q

(c) Ache o termo geral e a soma dos n primeiros termos da progresso aritmtico-geomtrica onde a1 = 1, q = 2 e r = 1.

i
i

inducaofinal
2009/6/30
i page 31
Estilo OBMEP

i
i

Captulo 2

Induo e Mundo Material


Neste captulo, mostraremos algumas aplicaes da induo
matemtica ao mundo material.

2.1

A Torre de Hani

Voc provavelmente j conhece esse jogo, pois trata-se de um jogo


bastante popular que pode ser facilmente fabricado ou ainda encontrado em lojas de brinquedos de madeira.
O jogo formado por n discos de dimetros distintos com um furo
no seu centro e uma base onde esto fincadas trs hastes. Numa das
hastes, esto enfiados os discos, de modo que nenhum disco esteja
sobre um outro de dimetro menor (veja figura a seguir).

31

i
i

inducaofinal
2009/6/30
i page 32
Estilo OBMEP

i
i

32

CAP. 2: INDUO E MUNDO MATERIAL

O jogo consiste em transferir a pilha de discos para uma outra


haste, deslocando um disco de cada vez, de modo que, a cada passo,
a regra acima seja observada.
As perguntas naturais que surgem so as seguintes:
1. O jogo tem soluo para cada n N?
2. Em caso afirmativo, qual o nmero mnimo jn de movimentos
para resolver o problema com n discos?
Usando Induo Matemtica, vamos ver que a resposta primeira
pergunta afirmativa, qualquer que seja o valor de n. Em seguida,
deduziremos uma frmula que nos fornecer o nmero jn .
Considere a sentena aberta
P (n) : O jogo com n discos tem soluo.
Obviamente, P (1) verdade. Suponha que P (n) seja verdadeiro,
para algum n; ou seja, que o jogo com n discos tem soluo. Vamos
provar que o jogo com n + 1 discos tem soluo.
Para ver isso, resolva inicialmente o problema para os n discos

i
i

inducaofinal
2009/6/30
i page 33
Estilo OBMEP

i
i

33

N SEC. 2.1: A TORRE DE HANI

superiores da pilha, transferindo-os para uma das hastes livre (isso


possvel, pois estamos admitindo que o problema com n discos possua
soluo):

Em seguida, transfira o disco que restou na pilha original (o maior


dos discos) para a haste vazia:

Feito isto, resolva novamente o problema para os n discos que


esto juntos, transferindo-os para a haste que contm o maior dos
discos:
Isso mostra que o problema com n + 1 discos tambm possui
soluo, e, portanto, por Induo Matemtica, que P (n) verdadeira

i
i

inducaofinal
2009/6/30
i page 34
Estilo OBMEP

i
i

34

CAP. 2: INDUO E MUNDO MATERIAL

para todo n N.
Para determinar uma frmula para jn , veja que, para resolver o
problema para n + 1 discos com o menor nmero de passos, temos,
necessariamente, que passar duas vezes pela soluo mnima do problema com n discos. Temos, ento, que
jn+1 = 2jn + 1.
Obtemos, assim, uma progresso aritmtico-geomtrica (jn ) cujo
termo geral , pelo Problema 1.3.5 (c), dado por
jn = 2n 1.
Esse jogo foi idealizado e publicado pelo matemtico francs
Edouard Lucas, em 1882, que, para dar mais sabor sua criao,
inventou a seguinte lenda:
Na origem do tempo, num templo oriental, Deus colocou 64 discos
perfurados de ouro puro ao redor de uma de trs colunas de diamante
e ordenou a um grupo de sacerdotes que movessem os discos de uma

i
i

inducaofinal
2009/6/30
i page 35
Estilo OBMEP

i
i

N SEC. 2.2: O ENIGMA DO CAVALO DE ALEXANDRE

35

coluna para outra, respeitando as regras acima explicadas. Quando


todos os 64 discos fossem transferidos para uma outra coluna, o mundo
acabaria.
Voc no deve se preocupar com a iminncia do fim do mundo,
pois, se, a cada segundo, um sacerdote movesse um disco, o tempo
mnimo para que ocorresse a fatalidade seria de 264 1 segundos e
isto daria, aproximadamente, um bilho de sculos!

2.2

O Enigma do Cavalo de Alexandre

Num mosaico romano, Bucfalo, o cavalo de Alexandre, o Grande,


representado como um fogoso corcel cor de bronze. Nesse exemplo,
vamos provar que isso uma falcia (uma grande mentira).
Inicialmente, provaremos que todos os cavalos tm mesma cor.
De fato, considere a sentena aberta:
P (n) : Num conjunto com n cavalos, todos tm a mesma cor.
Note que P (1) obviamente verdadeira. Agora, suponha o resultado vlido para conjuntos contendo n cavalos. Considere um conjunto
C = {C1 , C2 , . . . , Cn , Cn+1 }
com n + 1 cavalos. Decompomos o conjunto C numa unio de dois
conjuntos:
C = C 0 C 00 = {C1 , . . . , Cn } {C2 , . . . , Cn+1 },

i
i

inducaofinal
2009/6/30
i page 36
Estilo OBMEP

i
i

36

CAP. 2: INDUO E MUNDO MATERIAL

cada um dos quais contm n cavalos.


Pela hiptese indutiva, segue-se que os cavalos em C 0 tm mesma
cor, ocorrendo o mesmo para os cavalos em C 00 . Como
C2 C 0 C 00 ,
segue-se que os cavalos de C 0 tm a mesma cor dos cavalos de C 00 ,
permitindo assim concluir que todos os cavalos em C tm a mesma
cor.
Assim, a nossa demonstrao por induo est terminada,
provando que P (n) verdadeira para todo n N.
Agora, todo mundo sabe (voc sabia?) que Marengo, o famoso
cavalo de Napoleo, era branco. Logo, Bucfalo deveria ser branco.
Onde est o erro nessa prova? Para ach-lo, sugerimos que voc
tente provar que, se P (1) verdadeira, ento P (2) verdadeira.
Esse problema foi inventado pelo matemtico hngaro George
Plya (1887-1985).

Problemas
2.2.1 Ache o erro na prova do seguinte Teorema:
Todos os numeros naturais so iguais.
Demonstrao. Vamos provar o resultado mostrando que, para todo
n N, verdadeira a sentena aberta

i
i

inducaofinal
2009/6/30
i page 37
Estilo OBMEP

i
i

N SEC. 2.3: DESCOBRINDO A MOEDA FALSA

37

P (n): Dado n N, todos os nmero naturais menores ou iguais


do que n so iguais.
(i) P (1) claramente verdadeira.
(ii) Suponha que P (n) seja verdadeira, logo n 1 = n. Somando 1
a ambos os lados dessa igualdade, obtemos n = n + 1. Como n era
igual a todos os naturais anteriores, segue que P (n + 1) verdadeira.
Portanto, P (n) verdadeira para todo n N .

2.3

Descobrindo a Moeda Falsa

Tm-se 2n moedas de ouro, sendo uma delas falsa, com peso menor
do que as demais. Dispe-se de uma balana de dois pratos, sem nenhum peso. Vamos mostrar, por induo sobre n, que possvel achar
a moeda falsa com n pesagens.
Para n = 1, isso fcil de ver, pois, dadas as duas moedas, basta
pr uma moeda em cada prato da balana e descobre-se imediatamente qual a moeda falsa.
Suponha, agora, que o resultado seja vlido para algum valor de
n e que se tenha que achar a moeda falsa dentre 2n+1 moedas dadas.
Separemos as 2n+1 moedas em 2 grupos de 2n moedas cada. Coloca-se
um grupo de 2n moedas em cada prato da balana. Assim, poderemos
descobrir em que grupo de 2n moedas encontra-se a moeda falsa.
Agora, pela hiptese de induo, descobre-se a moeda falsa com n
pesagens, que, junto com a pesagem j efetuada, perfazem o total de
n + 1 pesagens.
No Captulo 3, iremos generalizar esse problema, resolvendo-o

i
i

inducaofinal
2009/6/30
i page 38
Estilo OBMEP

i
i

38

CAP. 2: INDUO E MUNDO MATERIAL

para um nmero qualquer de moedas.

Problemas
2.3.1 Mostre que o problema da moeda falsa para 3n moedas tambm
se resolve com n pesagens.

2.4

A Pizza de Steiner

O grande gemetra alemo Jacob Steiner (1796-1863) props e


resolveu, em 1826, o seguinte problema:
Qual o maior nmero de partes em que se pode dividir o plano
com n cortes retos?
Pensando o plano como se fosse uma grande pizza, temos uma
explicao para o nome do problema.
Denotando o nmero mximo de pedaos com n cortes por pn ,
vamos provar por induo a frmula:
pn =

n(n + 1)
+ 1.
2

Para n = 1, ou seja, com apenas um corte, claro que s podemos


obter dois pedaos. Portanto, a frmula est correta, pois
p1 =

1(1 + 1)
+ 1 = 2.
2

Admitamos agora que, para algum valor de n, a frmula para pn

i
i

inducaofinal
2009/6/30
i page 39
Estilo OBMEP

i
i

39

N SEC. 2.4: A PIZZA DE STEINER

esteja correta. Vamos mostrar que a frmula para pn+1 tambm est
correta.
Suponhamos que, com n cortes, obtivemos o nmero mximo
n(n + 1)/2 + 1 de pedaos e queremos fazer mais um corte, de modo
a obter o maior nmero possvel de pedaos.
Vamos conseguir isso se o (n + 1)-simo corte encontrar cada um
dos n cortes anteriores em pontos que no so de interseo de dois
cortes (faa um desenho para se convencer disso).
Por outro lado, se o (n + 1)-simo corte encontra todos os n cortes
anteriores, ele produz n + 1 novos pedaos: o corte comea em um
determinado pedao e, ao encontrar o primeiro corte, ele separa em
dois o pedao em que est, entrando em outro pedao. Ao encontrar
o segundo corte, ele separa em dois o pedao em que est, entrando
em outro pedao, e assim sucessivamente, at encontrar o n-simo
corte separando o ltimo pedao em que entrar em dois. Assim, so
obtidos n + 1 pedaos a mais dos que j existiam; logo,
pn+1 = pn + n + 1 =

n(n + 1)
(n + 1)(n + 2)
+1+n+1=
+ 1,
2
2

mostrando que a frmula est correta para n + 1 cortes. O resultado


segue ento do Teorema 1.1.1.
Problemas
2.4.1 (O queijo de Steiner) Para fazer a sua pizza, Steiner teve que
cortar, primeiro, o queijo. Imaginando que o espao um enorme
queijo, voc seria capaz de achar uma frmula para o nmero mximo
de pedaos que poderamos obter ao cort-lo por n planos?

i
i

inducaofinal
2009/6/30
i page 40
Estilo OBMEP

i
i

40

CAP. 2: INDUO E MUNDO MATERIAL

2.5

Os Coelhos de Fibonacci

Trata-se do seguinte problema proposto e resolvido pelo


matemtico italiano Leonardo de Pisa em seu livro Liber Abacci, de
1202:
Quot paria coniculorum in uno anno ex uno pario germinentur.
Como no se ensina mais latim nas escolas, a vai uma explicao:
um casal de coelhos recm-nascidos foi posto num lugar cercado. Determinar quantos casais de coelhos ter-se-o aps um ano, supondo
que, a cada ms, um casal de coelhos produz outro casal e que um
casal comea a procriar dois meses aps o seu nascimento.
Leonardo apresenta a seguinte soluo:

ms

nmero de casais
do ms anterior

nmero de casais
recm-nascidos

total

1o

2o

3o

4o

5o

6o

7o

13

8o

13

21

9o

21

13

34

10o

34

21

55

11o

55

34

89

12o

89

55

144

i
i

inducaofinal
2009/6/30
i page 41
Estilo OBMEP

i
i

41

N SEC. 2.5: OS COELHOS DE FIBONACCI

Portanto, o nmero de casais de coelhos num determinado ms


igual ao nmero total de casais do ms anterior acrescido do nmero
de casais nascidos no ms em curso, que igual ao nmero total de
casais do ms anterior ao anterior.
Se denotarmos o nmero de coelhos existentes no n-simo ms por
un , temos, ento, que
un = un1 + un2 ,

u1 = u2 = 1.

Essas relaes definem, por recorrncia, uma sequncia de nmeros


naturais, chamada de sequncia de Fibonacci, cujos elementos, chamados de nmeros de Fibonacci, possuem propriedades aritmticas
notveis, que ainda hoje so objeto de investigao.
Uma recorrncia1 do tipo
xn = xn1 + xn2

(2.1)

s permite determinar o elemento xn se conhecermos os elementos


anteriores xn1 e xn2 , que, para serem calculados, necessitam do
conhecimento dos dois elementos anteriores, e assim por diante. Fica,
portanto, univocamente definida a sequncia quando so dados x1 e
x2 . A sequncia de Fibonacci corresponde recorrncia (2.1), onde
x1 = x2 = 1.
Quando dada uma recorrncia, um problema importante determinar uma frmula fechada para o termo geral da sequncia, isto
, uma frmula que no recorre aos termos anteriores. No caso da
1

Uma recorrncia uma frmula que define um elemento de uma sequncia a


partir de termos anteriores.

i
i

inducaofinal
2009/6/30
i page 42
Estilo OBMEP

i
i

42

CAP. 2: INDUO E MUNDO MATERIAL

sequncia de Fibonacci, existe uma tal frmula, chamada frmula de


Binet, que apresentamos a seguir.
Proposio 2.5.1. Para todo n N, tem-se que

un =

n
1+ 5
2

n
12 5

.
5

Demonstrao. Procuremos as progresses geomtricas vn = q n , com


q 6= 0, que satisfazem recorrncia (2.1). Temos que
q n = q n1 + q n2 ,
cujas solues so

1+ 5
q1 =
2

1 5
e q2 =
.
2

Defina vn = q1n e wn = q2n . Note que, como as duas sequncias vn


e wn satisfazem recorrncia (2.1), ento, para quaisquer e reais,
a sequncia un = vn + wn tambm satisfaz recorrncia. Agora,
impomos u1 = u2 = 1, o que nos d um sistema de duas equaes com
as duas incgnitas e , cujas solues so = 15 e = 15 .
notvel que seja necessrio recorrer a frmulas envolvendo
nmeros irracionais para representar os elementos da sequncia de
Fibonacci, que so nmeros naturais. Mais notvel, ainda, que o
1+ 5
nmero
seja a proporo urea que aparece nas artes, e que
2

1 5
seja o simtrico de seu inverso 1 . Intrigante essa inesperada
2
relao entre criar coelhos e a divina proporo, no?

i
i

inducaofinal
2009/6/30
i page 43
Estilo OBMEP

i
i

43

N SEC. 2.5: OS COELHOS DE FIBONACCI

Leonardo de Pisa (1170-1250), filho de Bonacci, e por isso apelidado Fibonacci, teve um papel fundamental no desenvolvimento da
Matemtica no Ocidente. Em 1202, publicou o livro Liber Abacci,
que continha grande parte do conhecimento sobre nmeros e lgebra
da poca. Esta obra foi responsvel pela introduo na Europa do
sistema de numerao indo-arbico e pelo posterior desenvolvimento
da lgebra e da aritmtica no mundo ocidental.
Problemas
2.5.1 Mostre que a sequncia de Fibonacci satisfaz s seguintes identidades:
(a) u1 + u2 + + un = un+2 1.
(b) u1 + u3 + + u2n1 = u2n .
(c) u2 + u4 + + u2n = u2n+1 1.
(d) u21 + u22 + + u2n = un un+1 .

1+ 5
2.5.2 Sabendo que q =
raiz da equao x2 = x + 1, mostre
2
que q n = un q + un1 .
2.5.3 Prove que
u3 + u6 + u9 + + u3n =

u3n+2 1
.
2

2.5.4 Dada a recorrncia an+2 = 2an+1 + an , com a1 = 1 e a2 = 3,


ache uma frmula para an .

i
i

inducaofinal
2009/6/30
i page 44
Estilo OBMEP

i
i

44

CAP. 2: INDUO E MUNDO MATERIAL

2.5.5 Mostre que a recorrncia vn = 3vn1 2vn2 , v0 = 2 e v1 = 3


tem por soluo vn = 2n + 1.

i
i

inducaofinal
2009/6/30
i page 45
Estilo OBMEP

i
i

Captulo 3

Induo e Matemtica
O Princpio de Induo Matemtica possui inmeras aplicaes
em Matemtica. Neste captulo, veremos algumas delas.

3.1

Somatrios

Vamos recordar a noo de somatrio que introduzimos na Seo


2 do Captulo 1.
Seja A um conjunto com duas operaes satisfazendo s leis bsicas da aritmtica.
Se (an ) uma sequncia em A, definimos o somatrio dos seus n
primeiros termos como sendo
n
X

ai = a1 + a2 + + an .

i=1

45

i
i

inducaofinal
2009/6/30
i page 46
Estilo OBMEP

i
i

46

CAP. 3: INDUO E MATEMTICA

Para apreciar o poder do que iremos apresentar, tente, neste exato


momento, calcular a soma:
1 2 + 2 3 + 3 4 + + n(n + 1).
Se conseguiu, parabns! Se no conseguiu, no desanime, pois,
com os instrumentos de que voc dispe at o momento, o problema
difcil. Veremos adiante como isso vai se transformar, como num
passe de mgica, em algo fcil de calcular.
Provaremos a seguir alguns resultados bem simples sobre somatrios que iro nos ajudar a resolver este e muitos outros problemas
do mesmo tipo.
Proposio 3.1.1. Sejam (ai ), (bi ) duas sequncias de elementos do
conjunto A e seja c A. Ento,
n
n
n
X
X
X
(i)
(ai + bi ) =
ai +
bi ;
i=1

(ii)

n
X

i=1

c ai = c

i=1

(iii)

n
X

i=1

ai ;

i=1

n
X
(ai+1 ai ) = an+1 a1 ;
i=1

(iv)

n
X

c = nc.

i=1

P
Demonstrao. (i) O que significa a soma ni=1 (ai + bi )? Significa
que estamos somando os n primeiros termos da nova sequncia (cn ),

i
i

inducaofinal
2009/6/30
i page 47
Estilo OBMEP

i
i

47

N SEC. 3.1: SOMATRIOS

onde, para cada n N, define-se cn = an + bn .


Provemos o resultado por induo sobre n. Para n = 1, temos que
1
1
1
X
X
X
(ai + bi ) = a1 + b1 =
ai +
bi ,
i=1

i=1

i=1

mostrando que a frmula vlida nesse caso.


Suponha a frmula vlida para algum nmero natural n. Temos
ento que
Pn+1
i=1

(ai + bi ) =

Pn

i=1 ai

Pn

i=1 ai

Pn

i=1 (ai

Pn

i=1 bi

+ an+1 +

+ bi ) + (an+1 + bn+1 ) =

+ (an+1 + bn+1 ) =
Pn

i=1 bi

+ bn+1 =

Pn+1
i=1

ai +

Pn+1
i=1

bi ,

mostrando assim que a frmula vlida para n + 1. Pelo Teorema


1.1.1, temos que a frmula vlida para todo n N.
(ii) A prova tambm se faz por induo e a deixamos como exerccio.
(iii) Vamos provar, tambm por induo sobre n, esta frmula. Para
n = 1, temos que
1
X
(ai+1 ai ) = a2 a1 ,
i=1

o que mostra a validade da frmula neste caso.


Suponhamos que a frmula seja vlida para um nmero natural

i
i

inducaofinal
2009/6/30
i page 48
Estilo OBMEP

i
i

48

CAP. 3: INDUO E MATEMTICA

n. Logo,
n+1
X

n
X

i=1

i=1

(ai+1 ai ) =

(ai+1 ai ) + (an+2 an+1 ) =

an+1 a1 + an+2 an+1 = an+2 a1 ,


mostrando que a frmula vale para n + 1 e, portanto, vale para todo
n N.
P
(iv) O somatrio ni=1 c representa a soma de n parcelas iguais a c,
e, portanto, igual a nc.

Vejamos agora, com alguns exemplos, como podemos tirar partido


deste resultado.
Exemplo 3.1.1. Vamos ao desafio, que lanamos acima, de calcular
a soma:
Sn = 1 2 + 2 3 + 3 4 + + n(n + 1).
Com a notao de somatrio, podemos escrever
Sn =

n
X

i(i + 1).

i=1

Ora, aplicando o item (i) da proposio acima, temos


Sn =

Pn

i=1 i(i

+ 1). =

Pn

2
i=1 i

Pn

i=1 i

(12 + 22 + + n2 ) + (1 + 2 + + n),

i
i

inducaofinal
2009/6/30
i page 49
Estilo OBMEP

i
i

49

N SEC. 3.1: SOMATRIOS

somas estas que j calculamos nos Exemplos 1.1.1 e 1.1.2. Portanto,


temos que
Sn =

n(n + 1)(2n + 1) n(n + 1)


n(n + 1)(n + 2)
+
=
.
6
2
3

A frmula do item (iii) da Proposio 3.1.1, chamada de soma


telescpica, nos fornece um mtodo para calcular termos gerais de
certas recorrncias e somas, como veremos nos dois exemplos a seguir.
Exemplo 3.1.2. Vamos deduzir a expresso do termo geral da recorrncia da Pizza de Steiner:
pn+1 = pn + n + 1,

p1 = 2.

A expresso acima pode ser escrita do seguinte modo:


pi+1 pi = i + 1.
Tomando somatrios de ambos os lados, obtemos
n1
X
i=1

(pi+1 pi ) =

n1
X

(i + 1).

i=1

O primeiro membro da igualdade acima uma soma telescpica e


vale pn p1 , enquanto o segundo membro por ns conhecido e vale

i
i

inducaofinal
2009/6/30
i page 50
Estilo OBMEP

i
i

50

CAP. 3: INDUO E MATEMTICA

(n 1)n
+ n 1. Portanto, temos que
2
(n 1)n
n(n + 1)
+n1+2=
+ 1.
2
2

pn =

Exemplo 3.1.3. Seja i N e considere a seguinte identidade:1


(i + 1)4 = i4 + 4i3 + 6i2 + 4i + 1.
Da, segue que
(i + 1)4 i4 = 4i3 + 6i2 + 4i + 1.
Tomando os somatrios de ambos os membros da igualdade acima
e notando que o lado esquerdo uma soma telescpica, obtemos
n
n
X
X
4
4
(n + 1) 1 =
[(i + 1) i ] =
(4i3 + 6i2 + 4i + 1).
4

i=1

i=1

Usando agora as propriedades (i) e (ii) dos somatrios enunciados


na Proposio 3.1.1 e as frmulas obtidas nos Exemplos 1.1.1 e 1.1.2,

Esta identidade, que pode ser verificada diretamente, um caso particular da


frmula do binmio de Newton, que estudaremos em geral na prxima seo.

i
i

inducaofinal
2009/6/30
i page 51
Estilo OBMEP

i
i

51

N SEC. 3.1: SOMATRIOS

obtemos
(n + 1)4 1 =
4
4

Pn

i=1 (4i

+ 6i2 + 4i + 1) =

Pn

+6

Pn

+ n(n + 1)(2n + 1) + 2n(n + 1) + n.

i=1 i
i=1 i

Pn

2
i=1 i

+4

Pn

i=1 i

+n=

Da, segue que


Pn

3
i=1 i =

(n + 1)4 1 n(n + 1)(2n + 1) 2n(n + 1) n


=
4

n4 + 2n3 + n2
n(n + 1) 2
=
.
4
2

Obtemos, assim, a frmula do Problema 1.1.1 (i):

n(n + 1)
1 + 2 + + n =
2
3

2
.

possvel generalizar este procedimento para obter frmulas


recorrentes para as somas
1p + 2p + + np ,
quando p varia nos naturais (veja Problema 3.1.2).

i
i

inducaofinal
2009/6/30
i page 52
Estilo OBMEP

i
i

52

CAP. 3: INDUO E MATEMTICA

Problemas

3.1.1 Calcule frmulas fechadas para as seguintes somas:

(a) 1 + (1 + 2) + (1 + 2 + 3) + + (1 + 2 + + n).
(b) 1 2 3 + 2 3 4 + 3 4 5 + + n(n + 1)(n + 2).
(c) 1 3 + 3 5 + 5 7 + + (2n 1)(2n + 1).
(d) 1 + (1 + 22 ) + (1 + 22 + 32 ) + + (1 + 22 + 32 + + n2 ).
(e) 12 + 32 + 52 + + (2n 1)2 .
(f) 13 + 33 + + (2n 1)3 .

3.1.2
(a) Considere, para i N, a seguinte identidade:
(i + 1)5 i5 = 5i4 + 10i3 + 10i2 + 5i + 1.
Efetue o somatrio de ambos os lados para i variando de 1 at n.
Utilizando os Problemas 1.1.1 e 1.1.2, determine uma frmula
P
para ni=1 i4 .

i
i

inducaofinal
2009/6/30
i page 53
Estilo OBMEP

i
i

53

N SEC. 3.1: SOMATRIOS

(b) Pense em um modo de calcular


ser generalizado.

Pn

i=1 i

5.

Mostre como isto pode

3.1.3 Demonstre a Propriedade (ii) na Proposio 3.1.1.


3.1.4 Prove as desigualdades:

1
1
1
2( n + 1 1) < 1 + + + + < 2 n.
n
2
3
Sugesto: Mostre inicialmente que

1
2 n+12 n< <2 n2 n1
n
e em seguida use somas telescpicas.
3.1.5 Seja a1 , a2 , . . . , an+1 uma P.A. com de razo r. Calcule a soma
Sn =

1
1
1
+
+ +
.
a1 a2 a2 a3
an an+1

Este problema generaliza os Problemas 1.1.2 (a), (b) e (c).


Sugesto: Mostre inicialmente que

1
1
1
1
=

.
ai ai+1
r ai+1 ai
Tome o somatrio, para i variando de 1 at n, em ambos o lados da
igualdade acima e note que o somatrio do lado direito um mltiplo

i
i

inducaofinal
2009/6/30
i page 54
Estilo OBMEP
i

i
i

54

CAP. 3: INDUO E MATEMTICA

de uma soma telescpica. Conclua que


Sn =

3.2

1
1
1
n

=
.
r an+1 a1
a1 an+1

Binmio de Newton

Considere a expresso (1 + X)n , onde X uma indeterminada e n


um nmero natural. claro que o desenvolvimento dessa potncia
um polinmio de grau n em X, cujos coeficientes so nmeros naturais
(voc pode provar esta afirmao por induo sobre n):
(1 + X)n = a0 + a1 X + a2 X 2 + + an1 X n1 + an X n .
Os coeficientes ai , i = 0, . . . , n, serochamados
de nmeros bi
nomiais e denotados pelos smbolos ai =
definir


n
= 0.
i

n
. Se i > n, cmodo
i

Observe que, tomando X = 1 no desenvolvimento de (1 + X)n ,


obtemos a seguinte identidade:


n
n
n
2 =
+
+ +
.
0
1
n
n

Queremos determinar frmulas explcitas para esses nmeros binomiais.


Como os coeficientes do termo independente de X, do termo em
X e do termo em X n no desenvolvimento de (1 + X)n so, respecti-

i
i

inducaofinal
2009/6/30
i page 55
Estilo OBMEP
i

i
i

55

N SEC. 3.2: BINMIO DE NEWTON

vamente, 1, n e 1, temos que



n
= 1,
0


n
=n
1


n
e
= 1.
n

Lema 3.2.1 (Relao de Stifel). Para todo n N e todo i N com


0 i n, tem-se que

n
n
n+1
+
=
.
i
i+1
i+1

Demonstrao. Para i = n, a relao acima trivialmente verificada.


Para 0 i < n, as relaes decorrem, imediatamente, das seguintes
igualdades:

n+1
n+1
n+1
n+1
+
X + +
Xn +
X n+1 =
0
1
n
n+1
(1 + X)n+1 = (1 + X)


n
n
n
n
+
X + +
X n1 +
Xn =
0
1
n1
n



n
n
n
n
n
n
n
+
+
X + +
+
X +
X n+1 .
0
0
1
n1
n
n

Lema 3.2.2. Para todos n, i N, com 1 i n, tem-se que


i!


n
= n(n 1) (n i + 1).
i

Demonstrao. Vamos provar isto por induo sobre n. A igualdade

i
i

inducaofinal
2009/6/30
i page 56
Estilo OBMEP

i
i

56

CAP. 3: INDUO E MATEMTICA

trivialmente verificada para n = 1. Suponha que as igualdades sejam


vlidas para algum n N e todo i com 1 i n. Pela relao de
Stifel, temos, para i n, que


n+1
n
n
i!
= i(i 1)!
+ i!
=
i
i1
i

in(n 1) (n i + 2) + n(n 1) (n i + 1) =
n(n 1) (n i + 2)(i + n i + 1) =
(n + 1)n(n 1) (n + 1 i + 1),
o que prova a igualdade para n + 1 e para todo i com 1 i n. Uma
verificao direta mostra que a frmula tambm vale para i = n + 1.
Portanto, a igualdade vale para todo n e todo i com 1 i n.

Segue-se do Lema 3.2.2 que, para n, i N, com 1 i n, vale a


seguinte frmula para os coeficientes binomiais:

n
n(n 1) (n i + 1)
n!
=
=
.
i
i!
i!(n i)!

Note que os termos extremos nas igualdades acima tm sentido e


so iguais quando i = 0.
Da frmula acima, decorre imediatamente, para todo n N e todo
i com 0 i n, a seguinte identidade fundamental:

n
n
=
.
i
ni

i
i

inducaofinal
2009/6/30
i page 57
Estilo OBMEP

i
i

N SEC. 3.2: BINMIO DE NEWTON

57

Seja A um conjunto com duas operaes, uma adio e uma multiplicao, sujeitas s leis bsicas da aritmtica.
Teorema 3.2.1 (Binmio de Newton). Sejam a e b elementos do
conjunto A e seja n N. Tem-se que
(a + b)n = an +

n n1
n n2 2
n
a
b+
a
b + +
abn1 + bn .
1
2
n1

Demonstrao. Se a = 0, o resultado bvio. Se a 6= 0, substitua X


b
na expanso de (1 + X)n e multiplique ambos os lados por
por
a
an .
Exemplo 3.2.1.
(a + b)2 = a2 + 2ab + b2 .
(a + b)3 = a3 + 3a2 b + 3ab2 + b3 .
(a + b)4 = a4 + 4a3 b + 6a2 b2 + 4ab3 + b4 .
(a + b)5 = a5 + 5a4 b + 10a3 b2 + 10a2 b3 + 5ab4 + b5 .

Problemas

3.2.1 Demonstre a identidade das colunas:


i
i+1
n
n+1
+
+ +
=
.
i
i
i
i+1

i
i

inducaofinal
2009/6/30
i page 58
Estilo OBMEP

i
i

58

CAP. 3: INDUO E MATEMTICA

3.2.2 Demonstre a identidade das diagonais:



n
n+1
n+2
n+m
n+m+1
+
+
+ +
=
.
0
1
2
m
m

3.2.3
(a) Demonstre, para todos n, m, k N, a identidade de Euler:

k
X
m
n
n+m
=
.
i
ki
k
i=0

(b) Em particular, deduza a identidade de Lagrange:


n 2
X
n
i=0


2n
.
n

3.2.4
(a) Mostre que


n
o nmero de subconjuntos distintos com i
i

elementos de um conjunto com n elementos.

(b) Mostre que o conjunto das partes de um conjunto com n elementos tem 2n elementos.
(c) Usando os itens acima, d uma outra prova para a igualdade:


n
n
n
+
+ +
= 2n .
0
1
n

i
i

inducaofinal
2009/6/30
i page 59
Estilo OBMEP

i
i

N SEC. 3.3: PRINCPIO DO MENOR INTEIRO

59

3.2.5 Seja n N. Mostre que


n
n
n1
;
<
, se 0 i <
i
i+1
2

e que

n1
n
n
>
, se i >
.
i
i+1
2

3.3

Princpio do Menor Inteiro

Seja S um subconjunto no vazio de N. Dizemos que um nmero


natural a um menor elemento de S se possui as seguintes propriedades:
i) a S,
ii) a n, para todo n S.
imediato verificar que, se S possui um menor elemento, este
nico. De fato, se a e a0 so menores elementos de S, ento a a0 ,
pois a um menor elemento de S, e a0 um elemento de S, e, analogamente, a0 a, o que implica que a = a0 .
O menor elemento de S, quando existe, denotado por min S.
Por que fizemos a ressalva acima sobre a existncia de min S? Se
lhe parece to bvio que todo subconjunto no vazio dos naturais
possua um menor elemento, tente prov-lo!
preciso ter muito cuidado com as afirmaes do tipo bvio que,
pois devem ser utilizadas apenas quando qualquer um possa verificlas sem grande esforo.

i
i

inducaofinal
2009/6/30
i page 60
Estilo OBMEP

i
i

60

CAP. 3: INDUO E MATEMTICA

Vamos, agora, efetivamente provar o que parece bvio.


Teorema 3.3.1 (Princpio do Menor Inteiro). Todo subconjunto no
vazio de N possui um menor elemento.
Demonstrao. A demonstrao ser feita por reduo ao absurdo.
Seja S um subconjunto no vazio de N. Suponha, por absurdo,
que S no possua um menor elemento. Mostraremos que S vazio,
conduzindo a uma contradio.
Considere o conjunto T , complementar de S em N, ou seja, o conjunto dos nmeros naturais que no esto em S. Queremos, portanto,
mostrar que T = N, ou seja, que S = .
Defina o conjunto
In = {k N; k n},
e considere a sentena aberta
P (n) : In T.
Como 1 n, para todo n N, segue-se que 1 T , pois, caso
contrrio, 1 seria um menor elemento de S. Logo, P (1) verdadeira.
Suponha agora que P (n) seja verdadeira, para algum n. Se
n + 1 S, como nenhum elemento de In est em S, teramos que
n + 1 um menor elemento de S, o que no permitido. Logo,
n + 1 T , seguindo da que
In+1 = In {n + 1} T,

i
i

inducaofinal
2009/6/30
i page 61
Estilo OBMEP

i
i

N SEC. 3.3: PRINCPIO DO MENOR INTEIRO

61

o que prova que, para todo n, temos que In T ; portanto, N T N


e, consequentemente, T = N.

Voc entendeu a demonstrao acima? Se no entendeu, no desanime, pois certamente ainda no est na hora de voc apreciar todas
estas sutilezas. Isto vir naturalmente com o tempo. Qual o remdio, ento? Bem, faa de conta que realmente a afirmao contida no
teorema bvia e siga em frente.
O Princpio do Menor Inteiro tem vrias aplicaes, conforme veremos ao longo deste captulo. Como primeira aplicao, provaremos
uma variante da Induo Matemtica que muito til.
Teorema 3.3.2 (Induo Completa). Sejam a N e P (n) uma sentena aberta. Suponha que:
i) P (a) verdadeira, e que;
ii) qualquer que seja n a, se P (i) verdadeira para todo a i n,
ento P (n + 1) verdadeira.
Ento, P (n) verdadeira para todo n a.
Demonstrao. Considere o conjunto
V = {n N; n a e P (n) verdadeira}.
Queremos provar que o conjunto W = {n N; n a} \ V vazio.
Suponha, por absurdo, que vale o contrrio. Logo, pelo Princpio do
Menor Inteiro, W teria um menor elemento k, e, como sabemos de (i)
que a 6 W , segue-se que existe n tal que k = a + n > a. Portanto,

i
i

inducaofinal
2009/6/30
i page 62
Estilo OBMEP

i
i

62

CAP. 3: INDUO E MATEMTICA

a, a+1, . . . , k 1 6 W ; logo a, a+1, . . . , k 1 V . Por (ii), conclui-se


que k = k 1 + 1 V , o que contradiz o fato de k W .

O fato que apresentaremos a seguir j era conhecido de Euclides,


cerca de trezentos anos antes de Cristo, enunciado, porm sem demonstrao, em Os Elementos.
Teorema 3.3.3. Sejam dados nmeros naturais n e m. Existem dois
nicos nmeros inteiros no negativos q e r, com r < m, tais que
n = mq + r.
Demonstrao. Existncia Se n < m, basta tomar q = 0 e r = n.
Se n = m, basta tomar q = 1 e r = 0. Portanto, resta apenas provar
a propriedade quando n > m.
A demonstrao ser por induo completa sobre n. Se n = 1, o
resultado vlido, pelas consideraes acima, pois 1 = n m.
Suponha agora que o resultado seja vlido para todo i, com
1 i n. Seja m < n, logo 1 n + 1 m n e, portanto,
pela hiptese de induo, existem q 0 e r, com r < m, tais que
n + 1 m = q 0 m + r; logo n + 1 = (q 0 + 1)m + r, e o resultado
vlido para n + 1, tomando q = q 0 + 1.

Unicidade Se n = m, s h um jeito de escrever n da forma mq + r,


com r < m, que : n = m 1 + 0. Se n < m, tambm s h um
jeito de escrever n nessa forma: n = 0q + n. O resultado portanto
verdadeiro quando n = 1, j que, neste caso, 1 = n m.

i
i

inducaofinal
2009/6/30
i page 63
Estilo OBMEP

i
i

N SEC. 3.3: PRINCPIO DO MENOR INTEIRO

63

A prova ser tambm por induo completa sobre n. Vimos acima


que a unicidade est garantida quando n = 1. Suponha o resultado
vlido para todos os nmeros naturais menores ou iguais a n.
Suponha agora que n + 1 = qm + r = q 0 m + r0 , com r, r0 < m.
Podemos supor que n + 1 > m, j que o resultado est garantido
quando n + 1 m.
Subtraindo na igualdade acima m, obtemos que
n + 1 m = (q 1)m + r = (q 0 1)m + r0 ,
e, pela hiptese de induo, temos que q 1 = q 0 1 e r = r0 , da
seguindo a unicidade da escrita de n + 1.
Pelo Teorema da Induo Completa, o resultado fica estabelecido
para todo nmero natural n.

O resultado a seguir a base sobre a qual se apoiam os sistemas


de numerao.
Teorema 3.3.4. Seja dado um nmero natural b > 1. Todo nmero
natural a se escreve de modo nico na forma
a = a0 + a1 b + a2 b2 + + an bn ,
onde n um inteiro no negativo, todos os ai satisfazem s desigualdades 0 ai < b e an 6= 0.
Demonstrao. Note que a = 1 se escreve na forma acima, pois, para
isto, basta tomar n = 0 e a0 = 1.

i
i

inducaofinal
2009/6/30
i page 64
Estilo OBMEP

i
i

64

CAP. 3: INDUO E MATEMTICA

Seja S o subconjunto dos naturais que no admitem uma representao como acima. Queremos mostrar que S = .
Note que N \ S 6= , pois vimos acima que 1 6 S.
Suponha agora, por absurdo, que S 6= , logo, S possui um menor
elemento, que certamente maior do que 1. Seja a0 este menor elemento. Pelo algoritmo euclidiano da diviso, temos que a0 = bq + r,
onde 0 r < b. Mas, ento q < a0 e, portanto, q 6 S. Logo, q se
escreve na forma do teorema e, portanto, a0 tambm, o que uma
contradio, provando assim que S = .
Deixaremos a prova da unicidade da escrita como um desafio para
voc.

Podemos agora generalizar o problema da moeda falsa que apresentamos na Seo 2.3.
Exemplo 3.4.1. Seja m o nmero total de moedas das quais sabe-se
que uma falsa, mais leve do que as demais. No Teorema acima,
tomando b = 2, temos que todo nmero natural m se escreve como
somas de potncias de 2 (note que, neste caso, cada ai 0 ou
1), chamada de expanso binria. Isto , existem nmeros inteiros
0 n1 < n2 < < nr tais que
m = 2n1 + 2n2 + + 2nr .
Vamos provar, usando Induo Completa sobre nr , que bastam nr
pesagens para descobrir a moeda falsa.
Suponha nr = 1, ou seja, temos, no mximo, trs moedas. Pondo

i
i

inducaofinal
2009/6/30
i page 65
Estilo OBMEP

i
i

65

N SEC. 3.3: PRINCPIO DO MENOR INTEIRO

uma moeda em cada prato da balana, descobre-se imediatamente


a moeda falsa e, portanto, o resultado trivialmente verificado.
Suponha o resultado verdadeiro para todo n0 < nr .
Sejam agora 2n1 + 2n2 + + 2nr moedas, das quais uma
falsa. Separemos as moedas em 2 lotes com, respectivamente, 2nr e
2n1 + + 2nr1 moedas cada um. Comeamos analisando o primeiro
lote com 2nr moedas. Colocamos metade dessas moedas em cada prato da balana. Se a moeda falsa est neste lote, com o mtodo discutido no Captulo 2, aplicado s 2nr 1 moedas que esto no prato mais
leve, sabemos que podemos descobrir a moeda falsa com, no mximo,
nr 1 pesagens, com a pesagem j efetuada, descobrimos a moeda
com no mximo nr pesagens. Se a moeda falsa no est nesse lote,
descartamos o lote todo. Sobram, ento, 2n1 + + 2nr1 moedas a
serem analisadas. Pela hiptese de induo, bastam nr1 pesagens
para descobrir a moeda falsa, que, juntamente com a pesagem j
realizada, perfazem um total de nr1 + 1 pesagens, que certamente
menor do que ou igual a nr .
A frmula do binmio de Newton se generaliza para m parcelas,
conforme veremos a seguir.
Teorema 3.3.5 (Frmula de Leibniz). Sejam a1 , a2 , . . . , am elementos de um conjunto A munido de uma adio e de uma multiplicao
sujeitas s leis bsicas da aritmtica, e seja n N. Tem-se que
(a1 + a2 + + am )n =

X
i1 +i2 ++im =n

n!
ai1 ai2 aimm .
i1 !i2 ! im ! 1 2

Demonstrao. A prova ser por induo completa sobre m.

Se

i
i

inducaofinal
2009/6/30
i page 66
Estilo OBMEP

i
i

66

CAP. 3: INDUO E MATEMTICA

m = 2, esta a frmula do binmio de Newton.


Suponha a frmula vlida para todos os naturais menores do que
ou iguais a m, e vamos mostrar que tambm vlida para m +1. Pela
frmula do binmio de Newton,
X n
(a1 + + am + am+1 ) =
(a1 + + am )i ajm+1 .
i
i+j=n
n

A hiptese de induo nos fornece


X

(a1 + + am )i =

i1 +i2 ++im

i!
ai11 ai22 aimm ;
i
!i
!

i
!
1
2
m
=i

logo,
X n
i

(a1 + + am + am+1 )n =
X

i+j=n

i1 +i2 ++im =i

i+j=n

i1 +i2 ++im =i

X
i1 +i2 ++im +im+1 =n

i!
ai11 ai22 aimm ajm+1 =
i1 !i2 ! im !


n
i!
ai1 ai2 aimm ajm+1 =
i i1 !i2 ! im ! 1 2
n!
im+1
ai11 ai22 aimm am+1
,
i1 !i2 ! im !im+1 !

pois

n!
i!
n!
n
i!
=
=
,
i!(n i)! i1 !i2 ! im !
i1 !i2 ! im !im+1 !
i i1 !i2 ! im !

onde pusemos im+1 = n i = j.

Se voc no entendeu a manipulao com os duplos somatrios,

i
i

inducaofinal
2009/6/30
i page 67
Estilo OBMEP

i
i

67

N SEC. 3.3: PRINCPIO DO MENOR INTEIRO

acima, pergunte a algum. Caso a dvida persista, no faz mal, fica


para uma prxima leitura.
A frmula do teorema acima tem o nome de frmula de Leibniz
em homenagem ao matemtico e filsofo alemo Gottfried Wilhelm
von Leibniz (1646-1716), que compartilha com Newton o crdito pela
inveno do Clculo Diferencial.

Problemas
3.3.1 Um nmero natural p > 1 primo quando os nicos divisores
dele so 1 e o prprio p. Mostre que todo nmero natural n 2
possui algum divisor primo.
3.3.2 Mostre que todo nmero natural n 2 se decompe como
produto de nmeros primos.
3.3.3 Usando a frmula do binmio de Newton e Induo Completa,
mostre que, para cada r N, a soma
Sr (n) =

n
X

ir

i=1

um polinmio de grau r + 1 em n com termo de maior grau igual a


1
nr+1 .
r+1
3.3.4 Para n, m N, demonstre a igualdade:
X
i1 +i2 ++im =n

n!
= mn .
i1 !i2 ! im !

i
i

inducaofinal
2009/6/30
i page 68
Estilo OBMEP

i
i

68

CAP. 3: INDUO E MATEMTICA

3.3.5 Seja (un ) a sequncia de Fibonacci. Dados n, m N, com


n 2, mostre que:
(a) un+m = un1 um + un um+1 ;
(b) u2n1 = u2n1 + u2n ;
(c) u2n = u2n+1 u2n1 ;
(d) u3n = u3n+1 + u3n u3n1 .

3.4

O Princpio das Gavetas

Em 1834, o destacado matemtico alemo Johann Peter Gustav


Lejeune Dirichlet (1805-1859), criador do conceito moderno de funo,
enunciou o seguinte princpio, apelidado de Princpio da Casa de
Pombos:
Seja dada uma casa de pombos com n buracos e suponha que haja
m pombos querendo ocup-los. Se m > n, ento algum buraco dever
ser ocupado por mais de um pombo.
Isto parece realmente bvio, pois tem todo o respaldo da nossa
experincia do dia a dia. Tente ento provar esta afirmao.
Conseguiu? Parabns. Se no conseguiu, mos obra!
Este princpio tambm leva o nome de Princpio das Gavetas, pois
pode ser reenunciado, de modo equivalente, como segue:
Teorema 3.4.1 (Princpio de Dirichlet). Queremos guardar m obje-

i
i

inducaofinal
2009/6/30
i page 69
Estilo OBMEP

i
i

N SEC. 3.4: O PRINCPIO DAS GAVETAS

69

tos em n gavetas. Se m > n, ento alguma gaveta dever conter mais


de um objeto.
Demonstrao. Vamos provar este resultado por Induo Matemtica
sobre o nmero n de gavetas.
Para n = 1, o resultado bvio pois, se temos mais de um objeto
e uma s gaveta, teremos que acomodar nesta gaveta mais de um
objeto.
Suponha ento o resultado vlido para um certo nmero n de
gavetas e consideremos a situao de termos n+1 gavetas e m > n+1
objetos. Queremos mostrar que o resultado vale tambm neste caso,
para aplicar a Induo Matemtica e concluir que vale para todo
nmero natural n.
Depois de acomodar todos os objetos nas n + 1 gavetas, escolha
uma gaveta ao acaso. Se nesta gaveta h mais de um objeto, a nossa
assero est provada. Se nesta gaveta no h nenhum objeto, nas n
gavetas restantes esto acomodados m > n + 1 > n objetos, o que,
pela hiptese de induo, acarreta que em uma das gavetas h mais de
um objeto. Se na gaveta escolhida h um objeto, logo, nas n gavetas
restantes, esto distribudos m 1 > n objetos, o que, novamente,
pela hiptese de induo, acarreta que em uma das gavetas h mais
de um objeto.

Este simples princpio tem inmeras aplicaes, matemticas ou


no, algumas das quais veremos a seguir.
Exemplo 3.4.1. Na regio metropolitana de So Paulo, existem pelo

i
i

inducaofinal
2009/6/30
i page 70
Estilo OBMEP

i
i

70

CAP. 3: INDUO E MATEMTICA

menos duas mulheres com a mesma quantidade de fios de cabelo, o


mesmo ocorrendo com, pelo menos, dois homens.
De fato, uma estimativa por cima nos diz que uma pessoa pode
ter, no mximo, 7.106 fios de cabelo. Na regio metropolitana de
So Paulo, existem mais de 10.106 mulheres e mais de 9.106 homens
(fonte: PNAD 2004). O Princpio das Gavetas agora permite concluir
o desejado.
Exemplo 3.4.2. Existem n pessoas em uma festa. Algumas se conhecem, outras no. Mostre que na festa existem duas pessoas que tm
mesmo nmero de conhecidos, supondo que a relao de conhecido
simtrica: se x conhecido de y, ento y conhecido de x; e no
reflexiva: ningum conhecido de si mesmo (ser essa relao transitiva?).
De fato, cada pessoa tem um nmero de conhecidos que varia de
0 a n 1 (uma pessoa no conhecida de si mesma!), as duas situaes no podendo ocorrer ao mesmo tempo, pois, se uma pessoa
conhece todo mundo, pela simetria, no pode haver uma pessoa que
no conhea ningum. Portanto, ao associarmos os n indivduos s
n 1 possibilidades de nmero de conhecidos, pelo princpio de
Dirichlet, duas pessoas devero ter o mesmo nmero de conhecidos.
Vejamos agora algumas aplicaes mais srias.
Exemplo 3.4.3. Dentre cinco pontos escolhidos no interior de um
tringulo equiltero de lado 1 cm, existem dois pontos que distam
entre si menos do que 0, 5 cm.
De fato, divida o tringulo em quatro tringulos menores,

i
i

inducaofinal
2009/6/30
i page 71
Estilo OBMEP

i
i

N SEC. 3.4: O PRINCPIO DAS GAVETAS

71

conectando os pontos mdios dos lados do tringulo original. A distncia entre dois pontos que esto em um dos tringulos pequenos e
no interior do tringulo maior menor do que o seu lado que mede
0, 5 cm. Ao escolhermos cinco pontos no interior do tringulo dado,
pelo Princpio das Gavetas, dois dos pontos pertencero a um dos
tringulos pequenos, o que prova a nossa afirmao.
Exemplo 3.4.4. Se cada ponto do plano pintado de vermelho ou
de azul, ento algum retngulo no plano tem seus vrtices de uma
mesma cor.
Trace trs retas horizontais. Acharemos um retngulo com vrtices
sobre duas destas retas. Os outros lados so verticais. Uma reta vertical, ao cortar as trs paralelas, tem trs candidatos a vrtice do retngulo procurado.
Trs pontos podem ser coloridos com 2 cores de 8 modos distintos. Portanto, se voc escolher 9 retas verticais, pelo Princpio das
Gavetas, duas dessas retas vo encontrar cada uma das trs retas horizontais em um par de pontos de mesma cor. Agora, dos trs pares de
pontos, certamente dois tero a mesma cor, o que fornece os vrtices
do retngulo.
Exemplo 3.4.5. Existem duas potncias de 3 cuja diferena divisvel por 2 007.
Existem 2 007 possveis restos pela diviso por 2 007. Considere a
sequncia das potncias de 3:
1, 3, 32 , . . . , 32 007 .

i
i

inducaofinal
2009/6/30
i page 72
Estilo OBMEP

i
i

72

CAP. 3: INDUO E MATEMTICA

Esta sequncia composta de 2 008 nmeros. Portanto, pelo Princpio das Gavetas, dois desses, digamos 3n e 3m , com n > m, tm
mesmo resto quando divididos por 2 007. Logo, a sua diferena 3n 3m
divisvel por 2 007 (voc saberia justificar isso?).
Exemplo 3.4.6. Existe uma potncia de 3 que termina em 001.
Argumentando como no exemplo anterior, conclui-se que existem
m e n com n > m tais que 3n e 3m tm mesmo resto quando divididos
por 1 000. Logo, 3n 3m = 3m (3nm 1) divisvel por 1 000. Como
1 000 e 3m no tm fatores comuns, 1 000 deve dividir o segundo fator
3nm 1. Isto significa que 3nm termina em 001.
Exemplo 3.4.7. Suponha que n + 1 inteiros so tomados ao acaso
dentre os inteiros 1, 2, . . . , 2n. Pelo menos um desses inteiros mltiplo de um outro.
De fato, sejam a1 , . . . , an+1 os inteiros escolhidos. Note que cada
nmero ai pode ser escrito como 2ni bi , onde bi um nmero mpar.
Como no intervalo 1, . . . , 2n existem n nmeros mpares, e os n + 1
nmeros bi necessariamente se encontram neste intervalo, pelo princpio de Dirichlet, devemos ter br = bs para algum par de inteiros r
e s variando no conjunto {1, . . . , n + 1}, com nr > ns . claro que
ar = 2nr br um mltiplo de as = 2ns br .
Alguns dos exemplos acima foram tomados emprestados de
www.cut the knot.org/do you know/pigeon.shtml,
onde voc poder encontrar muitos outros.

i
i

inducaofinal
2009/6/30
i page 73
Estilo OBMEP

i
i

73

N SEC. 3.5: DESIGUALDADES

Problemas

3.4.1 Pode-se afirmar, com toda certeza, que em So Paulo existem


um homem e uma mulher com a mesma quantidade de fios de cabelo?
3.4.2 Mostre que existem duas potncias de 3 cuja diferena divisvel pelo ano em que voc nasceu.
3.4.3 Dados quaisquer seis inteiros de 1 a 10, mostre que dois deles
possuem soma mpar.

3.5

Desigualdades

Nesta seo, estabeleceremos algumas desigualdades importantes


que tm inmeras aplicaes em vrios contextos.
Teorema 3.5.1 (Desigualdade de Bernoulli). Se c um nmero real
tal que c > 1 e c 6= 0, ento para todo nmero natural n 2 vale a
desigualdade:
(1 + c)n > 1 + nc.

Demonstrao. Seja P (n) a desigualdade acima; vamos prov-la por


induo sobre n. claro que P (2) verdadeira, pois
(1 + c)2 = 1 + 2c + c2 > 1 + 2c.
Suponha P (n) verdadeira para algum n 2. Multiplicando ambos

i
i

inducaofinal
2009/6/30
i page 74
Estilo OBMEP

i
i

74

CAP. 3: INDUO E MATEMTICA

os lados da desigualdade acima por 1 + c (que > 0), obtemos


(1 + c)n+1 > (1 + n c)(1 + c) = 1 + (n + 1)c + nc2 > 1 + (n + 1)c,
donde conclumos que P (n + 1) verdadeira.
Por Induo Matemtica, segue que P (n) verdadeira para todo
nmero natural n.

Mdias so objetos matemticos que tm muitas aplicaes na


vida real. H vrias mdias; definiremos aqui trs delas, que relacionaremos entre si.
Dados nmeros reais positivos a1 , a2 , . . . , an , os nmeros
An =
Hn =

a1 + a2 + + an
,
n
n
1
1
1
+
+ +
a1 a2
an

Gn =

n
a1 a2 an ,

so chamados, respectivamente, de Mdia Aritmtica, Mdia Geomtrica e Mdia Harmnica dos nmeros dados.
Existe uma relao entre essas trs mdias dada por
Hn Gn An ,

(3.1)

cuja demonstrao pode ser feita por induo, mas que no trivial
se tentarmos faz-la diretamente.
No caso em que n = 2, a propriedade (3.1) fcil de provar. o

i
i

inducaofinal
2009/6/30
i page 75
Estilo OBMEP

i
i

75

N SEC. 3.5: DESIGUALDADES

que faremos a seguir.


Note que
0

a2 2 a21 a22 a1 a2
=
+

,
2
4
4
2

o que implica
a1 a2

a21 a22 a1 a2 a1 a2 2
,
+
+
=
+
4
4
2
2
2

seguindo da que

(3.2)

a1 + a2
a1 a2
.
2

Na desigualdade acima, valer a igualdade se, e somente se,

a1 a2 =

a1 + a2
2

2
=

a21 a22 a1 a2
+
+
,
4
4
2

o que ocorre se, e somente se,


a

a2 2 a21 a22 a1 a2
+

= 0,
=
2
4
4
2

o que equivale a ter a1 = a2 .


Por outro lado, de (3.2) segue facilmente que
4a21 a22 (a1 + a2 )2 a1 a2 ;
logo,

2a1 a2 (a1 + a2 ) a1 a2

i
i

inducaofinal
2009/6/30
i page 76
Estilo OBMEP

i
i

76

CAP. 3: INDUO E MATEMTICA

e, portanto,

2
a1 a2 .
1
1
+
a1 a2

No difcil verificar (faa-o) que, tambm neste caso, vale a igualdade na desigualdade acima se, e somente se, a1 = a2 .
Os dois prximos exemplos nos daro aplicaes geomtricas da
desigualdade entre Mdia Geomtrica e Mdia Aritmtica.
Exemplo 3.5.1. De todos os retngulos de permetro p dado, o quadrado o que tem maior rea.
De fato, suponha que os lados do retngulo tenham medidas a e
b. Pela desigualdade G2 A2 , segue que

p
a+b
= .
ab
2
4
Da, segue que a rea do retngulo de permetro p dado limitada
superiormente pela constante p2 /16. Segundo o que provamos acima,
a igualdade e, portanto, o mximo, ocorre s quando a = b, ou seja,
s quando o retngulo um quadrado.
Voc saberia dizer se, nessa situao, existe um retngulo de rea
mnima?
Exemplo 3.5.2. De todos os retngulos de rea dada A, o de menor
permetro o quadrado.
Suponha que os lados do retngulo tenham medidas a e b. Nova-

i
i

inducaofinal
2009/6/30
i page 77
Estilo OBMEP
i

i
i

N SEC. 3.5: DESIGUALDADES

77

mente, pela desigualdade G2 A2 , segue que

a+b
p
A = ab
= .
2
4
Da, segue que o permetro mnimo de todos os retngulos de
rea dada A ocorre quando a = b, ou seja, quando o retngulo um
quadrado. Ser que existe um retngulo de permetro mximo?
A prova da desigualdade (3.1) ser enormemente facilitada com a
demonstrao do seguinte resultado intermedirio.
Teorema 3.5.2. Sejam a1 , . . . , an nmeros reais positivos dados, tais
que a1 an = 1, ento a1 + + an n, valendo a igualdade se, e
somente se, a1 = = an = 1.
Demonstrao. A demonstrao ser feita por induo sobre n. Para
n = 1, o resultado trivialmente verificado.
Suponha o resultado vlido para algum n, e sejam a1 , . . . , an+1
nmeros reais positivos tais que a1 an an+1 = 1. Dois casos podem
se apresentar.
1o Caso: Todos os nmeros so iguais, ou seja,
a1 = a2 = = an+1 .
Neste caso, eles tm que ser iguais a 1. Portanto,
a1 + + an+1 = n + 1,
e o resultado, neste caso, vale para n + 1.

i
i

inducaofinal
2009/6/30
i page 78
Estilo OBMEP

i
i

78

CAP. 3: INDUO E MATEMTICA

2o Caso: Nem todos os nmeros so iguais. Neste caso, certamente


um dos nmeros maior do que 1 e um outro menor do que 1
(justifique). Podemos ento supor que a1 < 1 e an+1 > 1. Denotando
a1 an+1 por b1 , temos que b1 a2 an = 1. Logo, pela hiptese de
induo, segue que b1 + a2 + + an n, logo,
a1 + a2 + + an+1 = b1 + a2 + + an + a1 b1 + an+1 n + a1 b1 + an+1 .
(3.3)

Mas,
a1 b1 + an+1 = a1 a1 an+1 + an+1 = 1 (1 a1 )(1 an+1 ) > 1, (3.4)

j que a1 < 1 e an+1 > 1. Juntando (3.3) e (3.4), obtemos que


a1 + a2 + + an+1 > n + 1,
como queramos provar.

Corolrio 1. Sejam a1 , a2 , . . . , an nmeros reais positivos, ento,


an1 an
a1 a2
+
+ +
+
n,
a2 a3
an
a1
valendo a igualdade se, e somente se, a1 = a2 = = an .
a1 a2
an1 an

= 1; logo, pelo teorema


a2 a3
an a1
anterior, segue a desigualdade desejada. A igualdade vale se, e somente se,
a1
a2
an1
an
=
= =
=
= 1,
a2
a3
an
a1

Demonstrao. claro que

o que equivale a dizer que a1 = a2 = = an .

i
i

inducaofinal
2009/6/30
i page 79
Estilo OBMEP

i
i

79

N SEC. 3.5: DESIGUALDADES

Exemplo 3.5.3. Para todo x real, vale a desigualdade:


x2 + 2

2.
x2 + 1
De fato, temos que
p
x2 + 2
x2 + 1
1
1

=
+
= x2 + 1 +
.
2
2
2
2
x +1
x +1
x +1
x +1
Visto que o produto das duas ltimas parcelas 1, a desigualdade
segue do teorema anterior.
Exemplo 3.5.4. Seja a > 1 um nmero real. Temos que
log10 a + loga 10 2.
Esta desigualdade tambm segue do teorema anterior, tendo em
vista que log10 a loga 10 = 1.
Teorema 3.5.3. Temos que Gn An , valendo a igualdade se, e
somente se, a1 = a2 = = an .
Demonstrao. Ponhamos g = Gn . Logo, da igualdade
g = Gn =
segue que

r
1=

isto ,

n
a1 an ,

a1
an
,
g
g

a1
an

= 1.
g
g

i
i

inducaofinal
2009/6/30
i page 80
Estilo OBMEP

i
i

80

CAP. 3: INDUO E MATEMTICA

Portanto, pelo Teorema 3.5.2, temos que


a1
an
+ +
n,
g
g
o que nos d a desigualdade requerida. A igualdade vale se, e somente se, a1 /g = a2 /g = = an /g, o que ocorre se, e somente se,
a1 = a2 = = an .
Exemplo 3.5.5. Para n 2, vale a desigualdade:

n! <

n+1
2

n
.

De fato, pelo Teorema 3.5.2, temos que

1 + 2 + + n
n+1
n
n
n! = 1 2 n <
=
.
n
2
O resultado segue elevando potncia n ambos os lados da desigualdade acima.
Teorema 3.5.4. Temos que Hn Gn , valendo a igualdade se, e
somente se, a1 = a2 = = an .
Demonstrao. Pelo Teorema 3.5.3, temos que
1
=
Gn

q
n

1
a1
1 an

1
1
a1
1 + + an
=
,
n
Hn

provando assim a desigualdade. A igualdade vale se, e somente se,


1
1
a1
1 = a2 = = an , o que equivale ao fato de que a1 = a2 = =
an .

i
i

inducaofinal
2009/6/30
i page 81
Estilo OBMEP

i
i

81

N SEC. 3.5: DESIGUALDADES

O mtodo da prova da desigualdade (3.1) que utilizamos aqui, bem


como alguns dos exemplos, foram tomados emprestado do livrinho
Desigualdades, de autoria de P. P. Korovkin, Editorial MIR Moscou,
1976.

Problemas
3.5.1 Se x um nmero real positivo, mostre que:
xn + xn2 + +

1
xn2

1
n + 1.
xn

3.5.2 Prove que, para todo x real, vale a desigualdade


x2
1
.
4
1+x
2
3.5.3 Sejam a, b R, com a + b > 0 e a 6= b. Mostre que, para todo
n N, com n 2,
2n1 (an + bn ) > (a + b)n .
3.5.4 Prove que se a1 , . . . , an e p so nmeros reais positivos, ento

n
a1 an

r
p

ap1 + + apn
,
n

valendo a igualdade se, e somente se, a1 = = an .


3.5.5 Prove, para c 0, a seguinte generalizao da desigualdade de

i
i

inducaofinal
2009/6/30
i page 82
Estilo OBMEP

i
i

82

CAP. 3: INDUO E MATEMTICA

Bernouilli
(1 + c)n 1 + nc +

n(n 1) 2
c .
2

3.5.6 Defina a sequncia (xn ), n N, pela regra xn =

n
n 1.

(a) Mostre que, para todo n 2, tem-se que xn > 0.


(b) Mostre que, para todo n 2, tem-se que
n = (1 + xn )n
(c) Conclua que

n(n 1)x2n
.
2

2
.
n1

(d) Voc saberia dizer para quanto tende n n quando n cresce indefinidamente?
0 xn

Sugesto: para (b): use o Problema 3.5.5.

i
i

inducaofinal
2009/6/30
i page 83
Estilo OBMEP

i
i

Respostas
Captulo 1
1.3.1 (a) n(n + 1)
1.3.1 (b)

n(3n + 1)
2

1.3.2 (a) 2n+1 2


1
1.3.2 (b) 1 n . A soma tende para 1 quando n cresce indefinida2
mente.
1.3.3) A vitria rgia ocupar toda a superfcie do tanque no penltimo dia; ou seja, no dcimo nono dia.
1.3.4) O boato leva 80 minutos para tomar conta de toda a cidade.
1.3.5 (c) an = 2n 1 e Sn = 2n+1 (n + 2)
83

i
i

inducaofinal
2009/6/30
i page 84
Estilo OBMEP

i
i

84

CAP. 3: INDUO E MATEMTICA

Captulo 2
2.4.1)

n(n + 1)
+1
2

( 2 1)(1 + 2)n 3(1 + 2)(1 2)n


2.5.4) an =
4

Captulo 3
3.1.1 (a)

n(n + 1)(2n + 7)
12

3.1.1 (b)

n(n + 1)(n + 2)(n + 3)


4

3.1.1 (c)

n(4n2 + 6n 1)
3

3.1.1 (d)

n(n + 1) 2
(n + 5n + 2)
24

3.1.1 (e)

n(4n2 1)
3

3.1.1 (f) n(2n3 n + 2)

i
i

inducaofinal
2009/6/30
i page 85
Estilo OBMEP

i
i

N SEC. 3.5: DESIGUALDADES

3.1.2 (a)

85

"

2
1
n(n + 1)
4
5
i =
(n + 1) 1 10

5
2
i=1

n
X

n(n + 1)
n(n + 1)(2n + 1)
10
5
n .
6
2

3.4.1) No.
3.5.6 (d)

n
n tende a 1 quando n cresce indefinidamente.

i
i

GrafosModfranci
2009/6/30
page 1
Estilo OBMEP

Grafos Uma Introduo


Samuel Jurkiewicz

GrafosModfranci
2009/6/30
page 2
Estilo OBMEP

Texto j revisado pela nova ortografia.

GrafosModfranci
2009/6/30
page 3
Estilo OBMEP

Sobre o Autor
Samuel Jurkiewicz carioca e Doutor em Matemtica pela Universidade Pierre et Marie, em Paris. Atualmente professor da Escola de
Engenharia da UFRJ. J atuou como docente em todos os nveis, inclusive no pr-escolar. Alm do ensino de graduao e ps-graduao,
tem desenvolvido atividades junto a professores e alunos do Ensino
Mdio atravs de oficinas de Matemtica Discreta.

GrafosModfranci
2009/6/30
page 4
Estilo OBMEP

GrafosModfranci
2009/6/30
page i
Estilo OBMEP

Sumrio
1 O que um Grafo?

1.1

Primeiras Noes . . . . . . . . . . . . . . . . . . . . .

1.2

Grau de um Vrtice . . . . . . . . . . . . . . . . . . .

1.3

Nosso Primeiro Resultado . . . . . . . . . . . . . . . .

10

1.4

Alguns Problemas com as Definies . . . . . . . . . .

11

1.5

Isomorfismo . . . . . . . . . . . . . . . . . . . . . . . .

13

1.6

Outras Definies . . . . . . . . . . . . . . . . . . . . .

16

1.7

Tipos Especiais de Grafos . . . . . . . . . . . . . . . .

17

1.8

Representao por Matrizes . . . . . . . . . . . . . . .

22

2 Ciclos e Caminhos

28

2.1

Conexidade Outra Vez . . . . . . . . . . . . . . . . . .

28

2.2

O Problema do Menor Caminho

. . . . . . . . . . . .

31

Algoritmos e Computadores . . . . . . . . . . . . . . .

31

Qual o Menor Caminho at a Escola? . . . . . . . . . .

32

3 Mais Ciclos e mais Caminhos


i

45

GrafosModfranci
2009/6/30
page ii
Estilo OBMEP

ii

SUMRIO

3.1

Euler e as Pontes de Kenisberg . . . . . . . . . . . . .

45

Esse Problema Importante? . . . . . . . . . . . . . .

47

3.2

Estrutura de Dados . . . . . . . . . . . . . . . . . . . .

48

3.3

Grafos Eulerianos . . . . . . . . . . . . . . . . . . . . .

51

3.4

O Problema Chins do Carteiro . . . . . . . . . . . . .

57

3.5

Grafos e Ciclos Hamiltonianos . . . . . . . . . . . . . .

58

3.6

O Problema do Caixeiro Viajante PCV . . . . . . . .

59

3.7

Uma Palavra sobre Complexidade . . . . . . . . . . . .

62

4 rvores

66

4.1

Definies e Caracterizaes . . . . . . . . . . . . . . .

66

4.2

rvores Geradoras . . . . . . . . . . . . . . . . . . . .

68

O Problema de Conexo de Peso Mnimo . . . . . . . .

68

5 Subconjuntos Especiais de um Grafo

73

5.1

Conjuntos Independentes

. . . . . . . . . . . . . . . .

73

5.2

Colorao . . . . . . . . . . . . . . . . . . . . . . . . .

76

5.3

Aplicaes de Colorao . . . . . . . . . . . . . . . . .

77

5.4

Cliques . . . . . . . . . . . . . . . . . . . . . . . . . . .

82

5.5

Acoplamentos . . . . . . . . . . . . . . . . . . . . . . .

82

5.6

Acoplamentos em Grafos Bipartidos . . . . . . . . . .

84

GrafosModfranci
2009/6/30
page iii
Estilo OBMEP

iii
5.7

Colorao de Arestas . . . . . . . . . . . . . . . . . . .

85

5.8

Outros Subconjuntos . . . . . . . . . . . . . . . . . . .

92

6 Grafos Planares

95

6.1

Definies e Resultados Simples . . . . . . . . . . . . .

95

6.2

Teorema de Kuratowski . . . . . . . . . . . . . . . . .

99

6.3

Dualidade . . . . . . . . . . . . . . . . . . . . . . . . . 100

6.4

O Problema das 4 Cores . . . . . . . . . . . . . . . . . 101

ndice

111

GrafosModfranci
2009/6/30
page iv
Estilo OBMEP

GrafosModfranci
2009/6/30
page 1
Estilo OBMEP

Introduo
O leitor seria capaz de desenhar a figura 1 abaixo sem tirar o
lpis do papel? Tem que ir de ponto a ponto e no pode passar pela
mesma linha duas vezes.
C

E
Figura 1

Foi fcil? Experimente agora comear pelo ponto B.


Bem, esse problema importante? Pensemos numa pequena
cidade com pequeno oramento. O servio de recolhimento de lixo
feito por um pequeno caminho. Queremos evitar o desperdcio;
uma boa ideia seria fazer o caminho passar uma nica vez por cada
1

GrafosModfranci
2009/6/30
page 2
Estilo OBMEP

2
rua e retornar ao ponto de partida. Na verdade, o mesmo problema.
Um outro problema que propomos s crianas para que se
aquietem o seguinte: temos que ligar Luz, Gs e Telefone a trs
casas sem que as linhas se cruzem. Voc j tentou? (veja a figura 2)

casa 3

casa 2

casa 1

Figura 2
Outra vez, cabe a pergunta: esse problema importante? Pensemos ento numa fbrica de placas de circuito integrado. Encontrar
esquemas de ligao que evitem cruzamento crucial para baratear os
custos de manufatura; quanto menos camadas, mais rpido e rentvel
se torna o servio.
Nos dois casos s nos interessou considerar um conjunto de pontos
e um conjunto de ligaes entre eles. a essa estrutura que chamamos
grafo.
Estas notas tratam da Teoria dos Grafos uma modesta introduo. Desde o sculo XVIII at nossos dias essa teoria tem conhecido
extraordinrio desenvolvimento terico e aplicado. Adotamos ento

GrafosModfranci
2009/6/30
page 3
Estilo OBMEP

3
a prtica de introduzir alguns temas gerais que dessem uma pequena
ideia da variedade de abordagens e problemas que ela pode oferecer.
Certamente, muito ficou para depois. O que esperamos que ao final o leitor tenha se convencido da utilidade dos conceitos e processos
apresentados, mas guardamos o secreto desejo de que o aspecto ldico
dos grafos o contaminem com o que costumamos chamar de graphical
desease, ou melhor, traduzindo, a febre dos grafos.
Uma observao: sendo essa uma primeira abordagem da teoria
dos grafos, tratamos aqui apenas de grafos sem orientao. A inteno
foi apresentar os conceitos da forma mais simplificada possvel. Para
o leitor interessado, a bibliografia contempla grafos com orientao.
Cada captulo acompanhado de exerccios sem a soluo, preferimos deixar o prazer desta tarefa ao leitor. A bibliografia ao fim
das notas mais do que suficiente para adquirir um conhecimento
razovel de teoria dos grafos, e inclui trabalhos de nvel diversificado.
Enfim, deve haver erros; as crticas (construtivas, por favor) so
bem vindas.
Esperamos que apreciem estas notas.

Samuel Jurkiewicz
Escola de Engenharia/UFRJ Departamento de Engenharia Industrial
COPPE/UFRJ Programa de Engenharia de Produo
jurki@pep.ufrj.br

GrafosModfranci
2009/6/30
page 4
Estilo OBMEP

GrafosModfranci
2009/6/30
page 5
Estilo OBMEP

Captulo 1

O que um Grafo?
1.1

Primeiras Noes

Numa escola algumas turmas resolveram realizar um torneio


de vlei. Participam do torneio as turmas 6A, 6B, 7A, 7B, 8A e 8B.
Alguns jogos foram realizados at agora:
6A
6B
7A
7B
8A
8B

jogou
jogou
jogou
jogou
jogou
jogou

com
com
com
com
com
com

7A, 7B, 8B
7A, 8A, 8B
6A, 6B
6A, 8A, 8B
6B, 7B, 8B
6A, 6B, 7B, 8A

Mas ser que isto est correto? Pode ter havido um erro na
listagem. Uma maneira de representar a situao atravs de uma
figura. As turmas sero representadas por pontos e os jogos sero
representados por linhas.
No difcil agora constatar a consistncia das informaes. A
estrutura que acabamos de conhecer um grafo. Apresentamos duas
formas de representar esta estrutura
5

GrafosModfranci
2009/6/30
page 6
Estilo OBMEP

 CAP. 1: O QUE UM GRAFO?


6A
8B

6B

7A

8A
7B

Figura 1.1: Grafo do Campeonato de Vlei


Por uma lista, dizendo quem se relaciona com quem.
Por um desenho, isto , uma representao grfica.
Qual a forma correta? As duas so corretas. A estrutura grafo
admite vrias maneiras de ser representada. Isso no novidade:
a palavra dois e o smbolo 2 representam o mesmo conceito
matemtico.
Para que um grafo fique bem definido temos que ter dois conjuntos:
O conjunto V , dos vrtices no nosso exemplo, o conjunto das
turmas.
O conjunto A, das arestas no nosso exemplo, so os jogos
realizados.
Em outra palavras, o que nos interessa num grafo :
Quem so os vrtices.

GrafosModfranci
2009/6/30
page 7
Estilo OBMEP

N SEC. 1.2: GRAU DE UM VRTICE

Que pares de vrtices esto ligados e quais no esto (isto ,


quem so as arestas).

Quando existe uma aresta ligando dois vrtices dizemos que os vrtices
so adjacentes e que a aresta incidente aos vrtices. No nosso
exemplo podemos representar o grafo de forma sucinta como:
V = {6A; 6B; 7A; 7B; 8A; 8B}

A = {(6A; 7A); (6A; 7B); (6A; 8B); (6B; 7A); (6B; 8A); (6B; 8B); (7B; 8A);
(7B; 8B); (8A; 8B)}
Observe que no precisamos colocar (8A; 7B) no conjunto de
arestas pois j tnhamos colocado (7B; 8A).
O nmero de vrtices ser simbolizado por |V | ou pela letra n.
O nmero de arestas ser simbolizado por |A| ou pela letra m.
No nosso exemplo n = 6 e m = 9.

1.2

Grau de um Vrtice

No nosso exemplo vimos que cada turma jogou um nmero diferente de jogos:

GrafosModfranci
2009/6/30
page 8
Estilo OBMEP

 CAP. 1: O QUE UM GRAFO?

6A
6B
7A
7B
8A
8B

jogou
jogou
jogou
jogou
jogou
jogou

3
3
2
3
3
4

jogos
jogos
jogos
jogos
jogos
jogos

Por isso, no nosso desenho, o vrtice 6A tem 3 arestas ligadas a


ele, o vrtice A7 tem 2 arestas ligadas a ele e assim por diante.
Dizemos que estas arestas so incidentes ao vrtice. O nmero
de vezes que as arestas incidem sobre o vrtice v chamado grau
do vrtice v, simbolizado por d(v). No nosso exemplo, d(6A) = 3;
d(7A) = 2.
Exerccios
1. Usando o grafo do campeonato:
(a) D o grau de cada um dos vrtices
(b) Qual a soma de todos os graus?
(c) Qual o nmero de arestas?
(d) O que voc observou? Ser coincidncia?
2. Faa o mesmo exerccio anterior usando os grafos da figura 1.2:

GrafosModfranci
2009/6/30
page 9
Estilo OBMEP

N SEC. 1.2: GRAU DE UM VRTICE

Figura 1.2:

GrafosModfranci
2009/6/30
page 10
Estilo OBMEP

10

1.3

 CAP. 1: O QUE UM GRAFO?

Nosso Primeiro Resultado

No exerccio anterior voc deve ter observado que a soma dos graus
de um grafo sempre o dobro do nmero de arestas (e isso no deve
ser coincidncia...). Isso pode ser escrito em linguagem matemtica.
Para isso, denotaremos um grafo pela letra G e representaremos
por V (G) e A(G) respectivamente, os conjuntos de vrtices e das
arestas de G.
Teorema. Para todo grafo G
X

d(v) = 2 m.

vV (G)

Isto : A soma dos graus dos vrtices de um grafo sempre o dobro


do nmero de arestas.
Demonstrao. Quando contamos os graus dos vrtices estamos contando as extremidades das arestas uma vez. Como cada aresta tem
duas extremidades, cada aresta foi contada duas vezes.

Corolrio. Todo grafo G possui um nmero par de vrtices de grau


mpar.
Demonstrao. Se tivssemos um nmero mpar de vrtices de grau
mpar a soma dos graus seria mpar. Mas a soma dos graus o dobro
do nmero de arestas e, portanto um nmero par.

GrafosModfranci
2009/6/30
page 11
Estilo OBMEP

N SEC. 1.4: ALGUNS PROBLEMAS COM AS DEFINIES

1.4

11

Alguns Problemas com as Definies

Algumas perguntas acerca das definies podem nos deixar


atrapa-lhados. Vamos examinar algumas.
Uma aresta pode ligar um vrtice a ele mesmo?
Pode. o que chamamos de lao (veja figura 1.3). Por
exemplo, vamos construir o grafo em que V = {2, 3, 4, 5, 6} e
dois vrtices sero ligados quando tiverem um divisor comum
(diferente de 1).

Figura 1.3: Grafo com laos


Pela definio do grafo vemos que o 5 no est ligado a nenhum outro vrtice mas tem um lao (como alis todos os outros
vrtices deste grafo). Para haver coerncia com os resultados da
seo anterior, temos que contar o lao duas vezes (uma para
cada extremidade) quando calcularmos o grau do vrtice. No
nosso exemplo:
d(2) = 4; d(3) = 3; d(4) = 4; d(5) = 2; d(6) = 5
e o teorema continua valendo.

GrafosModfranci
2009/6/30
page 12
Estilo OBMEP

12

 CAP. 1: O QUE UM GRAFO?

Dois vrtices podem estar ligados por mais de uma aresta?


Podem. Neste caso usamos o nome especial de multigrafo
(veja figura 1.4). Um exemplo que veremos adiante resulta no
seguinte grafo:

Figura 1.4: Multigrafo (com arestas mltiplas)


Grafos sem laos ou arestas mltiplas so chamados de grafos
simples. Neste texto estaremos trabalhando quase sempre com
grafos simples.
A figura 1.5 mostra um grafo ou dois grafos?

Figura 1.5: Um grafo ou dois?


Depende da situao. Em princpio parecem dois grafos distintos, e podemos consider-los assim. Mas podemos pensar que
esse grafo representa as ligaes entre casas de uma cidade onde
passa um rio (veja figura a seguir).
Se as pontes forem destrudas em um temporal a cidade ainda

GrafosModfranci
2009/6/30
page 13
Estilo OBMEP

13

N SEC. 1.5: ISOMORFISMO

Figura 1.6:
uma s, apenas foi desconectada. O grafo da figura 1.5
poderia ser o que chamamos de grafo desconexo. Essa uma
noo importante e voltaremos a ela algumas vezes. Cada parte
conexa do grafo (no nosso exemplo o quadrado e o tringulo)
chamada de componente conexa do grafo. Dizemos que
um grafo conexo se qualquer par de pontos ligado por ao
menos um caminho.

1.5

Isomorfismo
Observe o grafo mostrado na figura adiante.

Verifique que a situao representada exatamente a mesma do


grafo inicial do campeonato. Apenas nesse caso procuramos fazer o
desenho de forma a no haver pontos comuns entre as arestas (fora dos
vrtices, claro). Quando dois grafos representam a mesma situao
dizemos que eles so grafos isomorfos.
Esse conceito s vezes gera polmica. o mesmo grafo ou no?
Claramente as caractersticas de um e de outro so as mesmas (graus,
nmero de arestas e outras que veremos mais tarde). E na verdade

GrafosModfranci
2009/6/30
page 14
Estilo OBMEP

14

 CAP. 1: O QUE UM GRAFO?

6A
8B

6B

7A

8A
7B

Figura 1.7:
esta no uma questo realmente importante. O essencial saber
discernir quando dois grafos so isomorfos ou no. Para isso vamos
usar uma definio tcnica.
Dois grafos G1 e G2 so ditos isomorfos se existe uma
correspondncia 1-a-1 entre seus conjuntos de vrtices que preserve as
adjacncias.
Vejamos um exemplo:
b

d
a

Figura 1.8:
Vamos estabelecer uma correspondncia 1 1 entre os conjuntos

GrafosModfranci
2009/6/30
page 15
Estilo OBMEP

15

N SEC. 1.5: ISOMORFISMO

de vrtices:
f :aw
bx
cz
dy
Esta funo funciona perfeitamente. Se tomarmos uma aresta no
primeiro grafo (digamos (a; d)) a funo far a correspondncia com
(w; y) que uma aresta no segundo grafo. Se tomarmos dois vrtices
que no so ligados por uma aresta (digamos a e c) a funo far
corresponder dois vrtices (w e z) que tambm no so ligados.
Exerccios
1. Verifique que a correspondncia a seguir no serve para mostrar
o isomorfismo dos grafos da figura 1.8.
Sugesto: Tome dois vrtices que no sejam ligados, faa a
correspondncia e veja o que acontece.
f : ax
by
cz
dw

GrafosModfranci
2009/6/30
page 16
Estilo OBMEP

16

 CAP. 1: O QUE UM GRAFO?

2. Mostre que os pares de grafos da figura 1.9 e 1.10 so isomorfos:

Figura 1.9:

Figura 1.10:

3. Mostre que os grafos 1.11 e 1.12 no so isomorfos:

Figura 1.11:

1.6

Figura 1.12:

Outras Definies

O conjunto de vrtices adjacentes a v chamado vizinhana


aberta de v, denotado por N (v). A vizinhana fechada de v
denotada e definida por N [v] = N (v) {v}, isto , inclui a vizinhana
e o prprio vrtice. Podemos estender esta definio para conjuntos de
vrtices (N (S)eN [S]). Por exemplo, no grafo do campeonato temos
N (7B) = {6A; 8A; 8B} e N [7B] = {6A; 7B; 8A; 8B}.

GrafosModfranci
2009/6/30
page 17
Estilo OBMEP

17

N SEC. 1.7: TIPOS ESPECIAIS DE GRAFOS

Um vrtice de grau 0 dito isolado; um vrtice de grau 1 dito


pendente. A sequncia de graus de um grafo a sequncia no
crescente formada pelos graus dos vrtices dos grafos. Por exemplo,
a sequncia de graus do grafo do campeonato (4, 3, 3, 3, 3, 2).
O menor grau de um vrtice em G o grau mnimo, denotado
(G), e o maior o grau mximo, denotado (G). No caso do
campeonato temos (G) = 4 e (G) = 2.
G0 dito um subgrafo de G se V (G0 ) V (G) e A(G0 ) A(G).
Na figura a seguir, o grafo G0 um subgrafo de G. O grafo G
dito um subgrafo induzido pelo subconjunto {a, b, c, d} de V (G),
pois todas as arestas incidentes aos vrtices de a, b, c, d em G esto
presentes em G(veja a figura 1.13).

a
e

Figura 1.13:

1.7

Tipos Especiais de Grafos

Grafo completo Imagine o grafo do campeonato quando todos


os jogos tiverem sido jogados. Ele ficaria com o aspecto da 1.14:

GrafosModfranci
2009/6/30
page 18
Estilo OBMEP

18

 CAP. 1: O QUE UM GRAFO?


6A
8B

6B

7A

8A
7B

Figura 1.14: O grafo completo K6


Isto o que chamamos um grafo completo. Um grafo completo definido como um grafo onde todo par de vrtices
ligado por uma aresta. Um grafo completo com n vrtices
denotado por Kn (O nosso exemplo K6 ).
Exerccios
1. Quantas arestas tm K7 ? e K12 ? e Kn ?
2. Quantos vrtices um grafo simples precisa ter para poder
ter 200 arestas?
Grafo complementar (veja figura 1.15)
Imagine agora que temos o grafo do campeonato e queremos
fazer o grafo dos jogos que faltam. Faramos um grafo com o
mesmo conjunto de vrtices mas com as arestas que faltam no
grafo original. Veja a figura.

GrafosModfranci
2009/6/30
page 19
Estilo OBMEP

19

N SEC. 1.7: TIPOS ESPECIAIS DE GRAFOS

6A

6A

8B

8B

6B

7A

8A

6B

7A

8A

7
B

7
B

Figura 1.15: Dois grafos complementares


Chamamos este grafo de grafo complementar do grafo G,
denotado por G. fcil perceber que V (G) = V (G) e que
A(G) A(G) inclui todas as arestas de G.
Grafo nulo ou vazio (figura 1.16)
Um grafo G nulo ou vazio quando o conjunto de arestas
A(G) vazio.
Por exemplo, antes de comear o campeonato nenhum jogo
havia sido jogado. Nosso grafo ficaria como na figura 1.16:
6A

8B

6B

7A

8A
7B

Figura 1.16: Grafo nulo ou vazio

GrafosModfranci
2009/6/30
page 20
Estilo OBMEP

20

 CAP. 1: O QUE UM GRAFO?

Grafo regular (figura 1.17)


Um grafo regular (de grau k, ou ainda k-regular) quando
todos os seus vrtices tm o mesmo grau (k). A figura 1.17
mostra um grafo 3-regular, isto , todos os vrtices tem grau 3.

Figura 1.17: Um grafo k-regular de grau 3


Ciclo (figura 1.18)
Um ciclo um grafo conexo regular de grau 2. A notao C n

Figura 1.18: Exemplos de ciclo: C5 e C6


Caminho (figura 1.19)
Um caminho um ciclo do qual retiramos uma aresta. O comprimento do caminho dado pelo nmero de arestas (o que faz
sentido: o nmero de passos que gastamos para percorrer o
caminho). Assim, o caminho Pn obtido retirando uma aresta
do ciclo Cn+1 .

GrafosModfranci
2009/6/30
page 21
Estilo OBMEP

N SEC. 1.7: TIPOS ESPECIAIS DE GRAFOS

21

Figura 1.19: Exemplos de caminho: P 4 e P5


rvores (figura 1.20)
Uma rvore um grafo conexo sem ciclos como subgrafos. Note
que o fato de no ter ciclos faz com que a rvore seja a maneira
mais econmica de conectar os vrtices. As rvores formam
uma famlia importante de grafos e voltaremos a elas mais tarde.

Figura 1.20: Exemplos de rvores


Grafos bipartidos (figura 1.21)
um grafo em que o conjunto V de vrtices pode ser particionado em dois subconjuntos disjuntos V 1 e V2 tal que toda
aresta de G tem uma extremidade em V1 e outra em V2 . O subconjunto V1 dito um subconjunto independente de vrtices do grafo G pois no h arestas ligando dois vrtices de
V1 . Temos tambm que V2 um subconjunto independente de

GrafosModfranci
2009/6/30
page 22
Estilo OBMEP

22

 CAP. 1: O QUE UM GRAFO?

vrtices de G.

Figura 1.21: Grafo bipartido

Grafos bipartidos completos Notao K p,q (figura 1.22).


um grafo bipartido em que todos os vrtices de V 1 so ligados
a todos os vrtices de V2 .

Figura 1.22: Grafo bipartido completo K 2,4

1.8

Representao por Matrizes

Matrizes um assunto tpico do ensino mdio mas o que


mostraremos aqui pode ser entendido por todos. Uma das formas

GrafosModfranci
2009/6/30
page 23
Estilo OBMEP

23

N SEC. 1.8: REPRESENTAO POR MATRIZES

mais comuns de informar uma estrutura de grafo para um computador atravs de matrizes. Uma matriz nada mais do que uma
tabela com linhas e colunas. Um exemplo bastante conhecido a
tabuada:

0
1
2
3
4
5
6
7
8
9

0
0
0
0
0
0
0
0
0
0

0
1
2
3
4
5
6
7
8
9

0
2
4
6
8
10
12
14
16
18

0
3
6
9
12
15
18
21
24
27

0
4
8
12
16
20
24
28
32
36

0
5
10
15
20
25
30
35
40
45

0
6
12
18
24
30
36
42
48
54

0
7
14
21
28
35
42
49
56
63

0
8
16
24
32
40
48
56
64
72

0
9
18
27
36
45
54
63
72
81

Se quisermos saber o valor de 3 5 procuramos o valor na linha


do 3 e na coluna do 5, isto 15.
Mas as matrizes tm outras utilidades. No caso dos grafos elas
podem ser usadas na representao de vrias formas. Eis algumas
delas. Exemplificaremos com as representaes do grafo a seguir:

GrafosModfranci
2009/6/30
page 24
Estilo OBMEP

24

 CAP. 1: O QUE UM GRAFO?

Figura 1.23:
Matriz de adjacncia a matriz definida por

xij =

1 se ij A(G)
0 se ij
/ A(G)

No exemplo da figura 1.23, a matriz de adjacncia :


0
1
1
1

1
0
1
0

1
1
0
1

1
0
1
0

Matriz de incidncia a matriz n m definida por

xij =

1 se a aresta ej incidente em vi
0 caso contrrio

GrafosModfranci
2009/6/30
page 25
Estilo OBMEP

25

N SEC. 1.8: REPRESENTAO POR MATRIZES

No exemplo da figura 1.23 a matriz de incidncia :

a
b
c
d

ab

ac

ad

bc

cd

1
1
0
0

1
0
1
0

1
0
0
1

0
1
1
0

0
0
1
1

Exerccios
1. Qual o grafo complementar do grafo desconexo formado por
duas componentes conexas isomorfas a K 3 e K7 ?
2. Qual o grafo complementar do grafo desconexo formado por
duas componentes conexas isomorfas a K r e Ks ?
3. Mostre que um grafo G desconexo, ento seu complementar
G tem um subgrafo bipartido completo. Mostre que a recproca
no verdadeira.
4. Mostre que as sequncias (9,8,7,6,5,5,4,3,3) e (7,7, 7,6,5,4,3,2)
no correspondem a sequncias de graus de nenhum grafo.
5. Mostre que a sequncia (3, 3, 3, 3, 3, 3) corresponde a pelo menos
dois grafos no isomorfos.
6. Mostre que uma mesma sequncia pode corresponder a grafos
no isomorfos.
7. Prove que

2.m
n

GrafosModfranci
2009/6/30
page 26
Estilo OBMEP

26

 CAP. 1: O QUE UM GRAFO?

8. Mostre que em um grafo bipartido m

n2
4 .

9. (a) Mostre que se G conexo, ento m n 1.


(b) Mostre que a recproca no verdadeira.
(c) Qual o menor valor de m que garante que G conexo?
10. Desenhe uma representao do grafo cuja matriz de adjacncia
:
0
1
0
1
1

1
0
1
1
0

0
1
0
1
0

1
1
1
0
1

1
0
0
1
0

11. Um grafo autocomplementar se for isomorfo ao seu complemento. Mostre que se G autocomplementar, ento n = 4k
ou n = 4.k + 1 para algum k inteiro.
12. O grafo de linha ou grafo adjunto, notao L(G), o grafo
cujos vrtices esto em correspondncia 1 a 1 com as arestas
de G e cujas arestas ligam vrtices que correspondem a arestas
incidentes em G.
(a) Mostre que L(K3 ) = L(K1,3 ).
(b) Mostre que se G regular de grau k, L(G) regular de
grau 2.k 2.
(c) Encontre uma expresso para o nmero de arestas de L(G)
em funo dos graus de G.

GrafosModfranci
2009/6/30
page 27
Estilo OBMEP

N SEC. 1.8: REPRESENTAO POR MATRIZES

27

13. Suponha que as arestas de K6 sejam coloridas de azul ou de


vermelho. Mostre que, seja qual for a forma de colorir, o grafo
ter um subgrafo isomorfo a K3 colorido com uma s cor.
Roteiro: Suponha, por absurdo, que isso no verdade.
(a) Escolha um vrtice v qualquer; mostre que existem (pelo
menos) 3 arestas incidentes a v com a mesma cor (digamos, sem perda de generalidade, (v; a); (v; b); e (v; c) so
coloridas de azul).
(b) Mostre que (a; b); (a; c); e (b; c) no podem ser coloridas de
azul.
(c) Conclua que (a; b); (a; c); e (b; c) devem ser coloridas de
vermelho, mostrando o absurdo, e provando a afirmao.
14. Suponha que as arestas de K17 sejam coloridas de azul, verde
ou de vermelho. Mostre que, seja qual for a forma de colorir, o
grafo ter um subgrafo isomorfo a K3 colorido com uma s cor.
Sugesto: Use o exerccio anterior.
15. Mostre que num grafo simples pelo menos dois vrtices tm o
mesmo grau.

GrafosModfranci
2009/6/30
page 28
Estilo OBMEP

Captulo 2

Ciclos e Caminhos
2.1

Conexidade Outra Vez

Observao. Quando no houver risco de confuso a aresta (v, w)


ser denotada simplesmente por vw.
Um passeio uma sequncia de arestas do tipo
v0 v1 ,v1 v2 ,v2 v3 ,...vs1 vs ; s o comprimento do passeio.
Se
todas as arestas do passeio so distintas, o passeio chamado
trilha; se v0 = vs o passeio uma trilha fechada. Se, alm das
arestas, todos os vrtices so distintos ento temos um caminho
e se v0 = vs temos um ciclo (como visto anteriormente). Uma
outra forma de definir a conexidade observar que um grafo G
conexo se, e s se, existe um caminho entre quaisquer dois vrtices
de G. As componentes conexas podem ser vistas como as classes de
equivalncia da relao:
x ' y se, e somente se, existe um caminho ligando x a y.
(Para isso, estamos considerando que entre um vrtice e ele mesmo
existe um caminho de comprimento 0.) O menor comprimento pos28

GrafosModfranci
2009/6/30
page 29
Estilo OBMEP

N SEC. 2.1: CONEXIDADE OUTRA VEZ

29

svel para um caminho entre os vrtices u e v chamado de distncia


entre u e v. Podemos tambm sinalizar as sequncias de arestas descritas acima pela sucesso de vrtices v 0 ,v1 ,v2 ,...,vs1 ,vs .
Dizemos que um grafo conexo k -conexo se, ao retirarmos k 1
vrtices do grafo, ele continua conexo. Por exemplo, o grafo da figura
1.17 3 conexo, pois podemos escolher 2 vrtices quaisquer para
retirar, e mesmo assim o grafo continuar conexo.

Teorema. Um grafo G bipartido se, e somente se, no contm


ciclos de comprimento mpar.

Demonstrao.
() Seja G bipartido. Se no houver ciclo em G, no h o que
mostrar. Se h um ciclo em G este alterna vrtices de V 1 e V2 ,
dois subconjuntos independentes e disjuntos. Partindo de V 1 (por
exemplo), para retornar ao ponto de partida teremos que utilizar um
nmero par de arestas. O ciclo , portanto, de comprimento par.
() Podemos considerar apenas grafos conexos. Seja G um grafo sem
ciclos mpares. Vamos particionar seu conjunto de vrtices em dois
subconjuntos V1 e V2 , independentes e disjuntos. Tomamos primeiramente um vrtice qualquer v. O subconjunto V 1 ser formado por
todos os vrtices w tais que exista um caminho de comprimento par
entre v e w. O subconjunto V2 ser formado por todos os vrtices
w tais que exista um caminho de comprimento mpar entre v e w.
Os conjuntos V1 e V2 so disjuntos, pois se w estivesse em V 1 e V2
ao mesmo tempo, haveria um caminho de comprimento par e um

GrafosModfranci
2009/6/30
page 30
Estilo OBMEP

30

 CAP. 2: CICLOS E CAMINHOS

caminho de comprimento mpar ligando v a w. Esses dois caminhos podem se cruzar (ou no) antes de chegar em w, produzindo
alguns ciclos (veja a figura a seguir). Como o nmero de arestas
usado nestes ciclos mpar ( a soma do nmero de arestas dos dois
caminhos) isso produziria pelo menos um ciclo mpar em G, contrariando a hiptese.

Figura 2.1:

J sabemos que o conjunto de vrtices de um grafo bipartido


particionado em dois subconjuntos V 1 e V2 . O conjunto V1 (e tambm
o conjunto V2 ) chamado conjunto independente, isto , se w e t
forem ambos vrtices de V1 eles no so adjacentes.
Exerccio
Nos pares de grafos das figuras 2.2 e 2.3, mostre qual dos grafos
bipartido e qual no .

Figura 2.2:

Figura 2.3:

GrafosModfranci
2009/6/30
page 31
Estilo OBMEP

N SEC. 2.2: O PROBLEMA DO MENOR CAMINHO

2.2

31

O Problema do Menor Caminho

Algoritmos e Computadores
Nesta seo vamos tratar de um problema relativamente simples.
Por exemplo, algum precisa se deslocar de uma cidade para outra e
para isso dispe de vrias estradas que passam por diversas cidades.
Qual caminho oferece uma trajetria de menor comprimento?
O algoritmo que soluciona este problema (e at hoje no se encontrou forma melhor) foi criado por Edsger Wybe Dijkstra, em 1952.
Dijkstra nasceu em 1930, na cidade de Roterdan Holanda, e morreu
em 2002. Foi um cientista de computao e recebeu o Turing Award
de 1972 por suas contribuies fundamentais na rea de linguagens
de programao.
Notem um fato interessante: geralmente o que estudamos em
Matemtica foi criado h muito tempo. Mas a Matemtica, como
veremos no problema que estamos estudando, continua a oferecer
solues e com o desenvolvimento da Informtica a ideia de uma
soluo para um problema tem se modificado. Em vez de procurarmos um nmero, uma resposta (o que em muitos casos necessrio),
procuramos um algoritmo, isto , uma srie de procedimentos que nos
levem soluo. A vantagem que, se o problema for muito extenso,
poderemos programar um computador para realizar este algoritmo.
Este problema um excelente exemplo disso.
Veremos mais tarde que isso no quer dizer que no precisamos
de teoria, muito pelo contrrio. Um bom algoritmo depende de boa
matemtica. Mas, voltaremos a isso adiante. Por enquanto vamos ver

GrafosModfranci
2009/6/30
page 32
Estilo OBMEP

32

 CAP. 2: CICLOS E CAMINHOS

a soluo, simples e interessante, oferecida por Dijkstra, que viveu no


nosso tempo, ou dos nossos pais.
Observe que trabalharemos com grafos valorados, isto , estaremos atribuindo valores s arestas. Estes valores podem ser distncias, tempo gasto no trajeto, custo com a ligao etc. Usaremos as
expresses custo ou distncia para nos referirmos a estes valores.
Estes valores geralmente so estimados por engenheiros, economistas
e consideraremos nos prximos exemplos que eles so dados. Este algoritmo trabalha apenas com grafos valorados com valores positivos
e nossa tarefa minimizar custo ou distncia.

Qual o Menor Caminho at a Escola?


5

Armazm
Armazem

Casa do
Casa
do
Joo
Joo

13

10

Pracinha
Pracinha

11
Banca de
de
Banca
Jornal
Jornal

3
6

Quitanda
Quitanda

4
Cancela
Cancela

Escola
Escola

Figura 2.4:
Lembremos que este grafo valorado, isto , atribumos valores

GrafosModfranci
2009/6/30
page 33
Estilo OBMEP

33

N SEC. 2.2: O PROBLEMA DO MENOR CAMINHO

s arestas. A distncia diferente da que estamos acostumados.


Por exemplo, na figura 2.4, entre a Pracinha (P) e a Banca de Jornal
(B) colocamos a distncia 11 pois h um cachorro que nos assusta.
Entre a Quitanda (Q) e a Cancela (C) a distncia 4 pois h uma
moa (ou rapaz) interessante. Usaremos este grafo simples e pequeno
para vermos como o algoritmo de Dijkstra funciona. Comeamos calculando todas as distncias a partir da Casa de Joo (J). A distncia
de J at J 0 (zero).
Vamos comear com o mapa sem ligaes (2.5).

Armazm
Armazem

Casa do
Joo

Pracinha
Pracinha

Banca de
Banca
de
Jornal
Jornal

Cancela
Cancela

Quitanda
Quitanda

Escola
Escola

Figura 2.5:

At onde posso chegar a partir da casa de Joo (J) em uma nica


etapa? Qual o custo? Vamos preencher a tabela a seguir.

GrafosModfranci
2009/6/30
page 34
Estilo OBMEP

34
Determinado
(fechado)

 CAP. 2: CICLOS E CAMINHOS

Posso chegar
at...

...com custo
ou distncia...

J - Casa de Joo
A - Armazm

P - Pracinha

Q - Quitanda

B - Banca de Jornal

C - Cancela

E - Escola

...vindo de...
***

Ainda
Ainda
Ainda
Ainda
Ainda
Ainda

no
no
no
no
no
no

atingimos
atingimos
atingimos
atingimos
atingimos
atingimos

Ateno: colocamos a distncia para dizer que ainda no atingimos este vrtice.
Vamos entender a figura e a tabela; na figura escurecemos a Casa
de Joo, pois j sabemos a menor distncia: 0. Os outros vrtices
ainda podem ser melhorados, por isso no esto escurecidos, e a etiqueta mostra que ainda no foram atingidos.
A partir da casa de Joo, quem podemos atingir imediatamente?
O Armazm, que est a distncia 5 da Casa de Joo, a Pracinha que
est a distncia 6 e a Quitanda, que est a distncia 10. Vou assinalar
isto no meu grafo. Mais ainda, eu agora percebo que a distncia ao
armazm no ir diminuir. De fato, qualquer outro caminho que
eu tome, j comea com um valor maior que 5 (ou eventualmente
igual). Ento escureo o vrtice do armazm para mostrar que ele
est fechado.

GrafosModfranci
2009/6/30
page 35
Estilo OBMEP

35

N SEC. 2.2: O PROBLEMA DO MENOR CAMINHO

0
5

Armazm
Armazem

Casa do
Casa
do
Joo
Joo

Pracinha
Pracinha

Banca
Bancade
de
Jornal
Jornal

10

10

Quitanda
Quitanda

Escola
Escola

Cancela
Cancela

Figura 2.6:

Vamos preencher a tabela de acordo:


Determinado
(fechado)

Posso chegar
at...

...com custo
ou distncia...

...vindo de...

J - Casa de Joo

***

A - Armazm
P - Pracinha

5
6

J
J

Q - Quitanda

10

B - Banca de Jornal

C - Cancela

E - Escola

Ainda no atingimos
Ainda no atingimos
Ainda no atingimos

Como a distncia at o armazm no vai diminuir, a nossa vez


de investigar se indo pelo caminho do armazm poderemos melhorar
as distncias. A partir do Armazm s podemos chegar Banca de

GrafosModfranci
2009/6/30
page 36
Estilo OBMEP

36

 CAP. 2: CICLOS E CAMINHOS

Jornais (B) (Lembre-se que J j est fechado). Note que a etiqueta de


distncia da Banca de Jornal passa a ser 18 = 5+13 (5 da etiqueta do
Armazm mais 13 da distncia Armazm-Banca de Jornais). Como
18 < a melhor distncia at a Banca de 18.
Nosso grafo e tabela ficam assim e o prximo vrtice a ser fechado
a Pracinha (P).
5

0
5

Casa
do
Casa do
Joo
Joo
6

Armazm
Armazem

13

18

Pracinha
Pracinha

Banca
de
Banca de
Jornal
Jornal

10

10

Quitanda
Quitanda

Escola
Escola

Cancela
Cancela

Figura 2.7:
Determinado
(fechado)

Posso chegar
at...

...com custo
ou distncia...

...vindo de...

J - Casa de Joo

***

A - Armazm

P - Pracinha

Q - Quitanda

10

B - Banca de Jornal

18

C - Cancela

E - Escola

Ainda no atingimos
Ainda no atingimos

GrafosModfranci
2009/6/30
page 37
Estilo OBMEP

N SEC. 2.2: O PROBLEMA DO MENOR CAMINHO

37

Como a distncia Pracinha no pode ser melhorada a partir


dela que investigaremos. Podemos chegar, passando pela Pracinha
Quitanda, Banca de Jornal e Cancela. Vamos ver o que acontece
nos trs casos:

Quitanda: 6 (etiqueta da Pracinha) + 3 (distncia PracinhaQuitanda) = 9; como 9 < 10 (que a etiqueta atual da Quitanda), o caminho melhor passa a ser pela Pracinha.
Cancela: 6 + 6 = 12 < logo o caminho para a cancela passa a ser
pela Pracinha.
Banca de Jornal: 6 + 11 = 17 < 18 e o caminho para a Banca de
Jornal passa a ser pela Pracinha.
O vrtice a ser fechado a Quitanda pois o menor valor em aberto.
Nosso grafo e tabela ficam assim:

GrafosModfranci
2009/6/30
page 38
Estilo OBMEP

38

 CAP. 2: CICLOS E CAMINHOS

0
5

Armazem
Armazm

Casa
do
Casa do
Joo
Joo
6

17

Pracinha
Pracinha

11

Bancade
de
Banca
Jornal

3
9

Quitanda
Quitanda
8

12

Escola
Escola

Cancela
Cancela

Figura 2.8:

Determinado
(fechado)

Posso chegar
at...

...com custo
ou distncia...

...vindo de...

J - Casa de Joo

***

A - Armazm

P - Pracinha

Q - Quitanda

B - Banca de Jornal

17

C - Cancela

12

E - Escola

Ainda no atingimos

Agora vamos estudar se podemos melhorar a distncia a partir da


Quitanda (que fechamos por ser o menor valor em aberto).

GrafosModfranci
2009/6/30
page 39
Estilo OBMEP

39

N SEC. 2.2: O PROBLEMA DO MENOR CAMINHO

Da Quitanda posso alcanar a Banca de Jornais com distncia


total 9 + 6 = 15 < 17 logo meu caminho para a Banca de Jornais
passa a usar a Quitanda.
Tambm posso alcanar a Cancela mas com distncia
9 + 4 = 13 > 12.
Ento no vantagem, e continuo a ir para a Cancela passando pela
Pracinha.
Nosso grafo e tabela ficam assim:

0
5

Armazem

Casa do
Joo

15

Pracinha

3
9

11

Banca de
Jornal

6
6
12
Cancela

Figura 2.9:

Quitanda

Escola

GrafosModfranci
2009/6/30
page 40
Estilo OBMEP

40

 CAP. 2: CICLOS E CAMINHOS

Determinado
(fechado)

Posso chegar
at...

...com custo
ou distncia...

...vindo de...

*
*

J - Casa de Joo
A - Armazm

0
5

***
J

P - Pracinha

*
*

Q - Quitanda

B - Banca de Jornal

15

C - Cancela

12

E - Escola

Ainda no atingimos

Observe que escurecemos o vrtice da Cancela, que o que tem


menor distncia acumulada entre os abertos. Isso mostra que nem
sempre fechamos os vrtices na ordem da tabela. Neste caso
fechamos a Cancela antes da Banca de Jornal.
J estamos quase terminando. Da Cancela s consigo ir Escola
com distncia acumulada 12 + 8 = 20 < .
Minha tabela e grafo ficam assim (escurecemos o vrtice da Banca
de Jornais):

Determinado
(fechado)

Posso chegar
at...

...com custo
ou distncia...

...vindo de...

*
*

J - Casa de Joo
A - Armazm

0
5

***
J

P - Pracinha

Q - Quitanda

B - Banca de Jornal

17

C - Cancela

12

E - Escola

20

GrafosModfranci
2009/6/30
page 41
Estilo OBMEP

41

N SEC. 2.2: O PROBLEMA DO MENOR CAMINHO

0
5

Armazem

Casa do
Joo

15

Pracinha

11

Banca de
Jornal

Quitanda
20

12
8

Cancela

Escola

Figura 2.10:
E finalmente, vemos que pela Banca de Jornal conseguimos chegar
Escola com distncia acumulada de 15 + 3 < 20.
A tabela e o grafo finais ficam:

Determinado
(fechado)

Posso chegar
at...

...com custo
ou distncia...

...vindo de...

J - Casa de Joo

***

A - Armazm

P - Pracinha

Q - Quitanda

B - Banca de Jornal

17

C - Cancela

12

E - Escola

18

GrafosModfranci
2009/6/30
page 42
Estilo OBMEP

42

 CAP. 2: CICLOS E CAMINHOS

0
5

Armazem

Casa do
Joo

15

Pracinha

3
9

11

Banca de
Jornal

6
6

Quitanda
12
Cancela

18
Escola

Figura 2.11:
Observe que:
O grafo final uma rvore conexa e sem ciclos (sempre que
chegvamos num vrtice, eliminvamos uma aresta, impedindo
a formao de ciclos).
O algoritmo encontra o menor caminho da Casa de Joo a todos
os outros pontos. Ele no encontra o menor caminho entre dois
vrtices quaisquer. Por exemplo para ir da Cancela Banca de
Jornais a distncia 11 e no 15 como a rvore sugere.
A representao grfica foi til para entendermos o problema,
mas poderamos perfeitamente ter usado apenas uma matriz
de distncia:

GrafosModfranci
2009/6/30
page 43
Estilo OBMEP

43

N SEC. 2.2: O PROBLEMA DO MENOR CAMINHO

10

13

11

10

13

11

Exerccios
1. Nas figura abaixo, use o algoritmo de Dijkstra para descobrir
qual o menor caminho do vrtice A a todos os outros vrtices.
P

70

110

31

61

70

30
65

67

I
74

100

126

105
26

30

12

140

19
85

Figura 2.12:

39

GrafosModfranci
2009/6/30
page 44
Estilo OBMEP

44

 CAP. 2: CICLOS E CAMINHOS

2. Abaixo, temos uma tabela de distncias entre uma Mercearia


e as localidades onde ela faz entregas. Use o algoritmo de
Dijkstra para descobrir qual o menor caminho da Mercearia a
todas as outras localidades.

Mercearia

Mercearia

11

11

12

11

15

15

12

11

GrafosModfranci
2009/6/30
page 45
Estilo OBMEP

Captulo 3

Mais Ciclos e mais


Caminhos
3.1

Euler e as Pontes de Kenisberg

Na introduo, perguntamos se voc conseguiria desenhar a


casinha abaixo sem tirar o lpis do papel. A figura mostra uma
soluo e, na verdade, o problema bastante fcil.

Figura 3.1:

45

GrafosModfranci
2009/6/30
page 46
Estilo OBMEP

46

 CAP. 3: MAIS CICLOS E MAIS CAMINHOS

Mas se quisermos comear pelo vrtice B? (voc pode tentar o


tempo que quiser).
O fato que esse outro problema impossvel. Todas as solues
comeam/terminam pelo vrtice A/E. Se comeam em A terminam
em E, e vice-versa.
O problema tem origem no famoso problema das pontes de
Kenisberg, considerado o marco fundador da Teoria dos Grafos.
Os habitantes de Kenisberg (hoje Kaliningrado) se perguntavam se
seria possvel atravessar as sete pontes do Rio Prega, sem passar duas
vezes na mesma ponte, retornando ao ponto de partida. O problema
e sua modelagem por grafos est apresentada na figura a seguir.

Figura 3.2:
Observamos que o problema d origem a um grafo com arestas
mltiplas, o que no afetar a soluo. Leonard Euler mostrou que a
resposta era negativa, estabelecendo assim uma condio necessria;
embora se acredite que a suficincia no lhe fosse desconhecida. Esta
segunda parte foi publicada por Hierholzer em 1873, muito mais tarde.

GrafosModfranci
2009/6/30
page 47
Estilo OBMEP

N SEC. 3.1: EULER E AS PONTES DE KENISBERG

47

Antes de prosseguir com a soluo, vamos tecer algumas consideraes sobre grafos, computadores e problemas finitos.

Esse Problema Importante?


Sim! Para comeo de conversa, ele interessante, simples de propor e veremos que sua soluo atraente, interessante e tem consequncias importantes.
Mas no aspecto imediato, pense numa pequena cidade com um
nico caminho para recolher o lixo onde o prefeito deseja economizar,
o que significa que ele prefere que o caminho passe uma nica vez
por todas as ruas e retorne ao ponto de partida.
O problema idntico ao problema da casinha e, se a cidade tivesse
essa configurao, no teria soluo (pois o caminho no retornaria
ao ponto inicial (Voc experimentou?). Se o mapa da cidade fosse
como na figura a seguir, o prefeito ficaria contente (experimente desenhar esta figura sem tirar o lpis do papel mas voltando ao ponto
inicial).

Figura 3.3:

GrafosModfranci
2009/6/30
page 48
Estilo OBMEP

48

 CAP. 3: MAIS CICLOS E MAIS CAMINHOS

E em que um computador pode nos ajudar neste caso?

3.2

Estrutura de Dados

O desenho ajuda a ns, pessoas, mas os computadores preferem letras e nmeros. Lembre-se que a casinha representa o grafo G(V, A) em que V (G) = {A, B, C, D, E} e
A(G) = {(A; B); (A; D); (A; E); (B; C); (B; D); (B; E); (C; D); (D; E)}.
Observe que usamos uma ordem semelhante ordem do dicionrio;
isso facilita encontrar a aresta que procuramos e isso vale para o
computador tambm (essa ordem tem o nome de ordem lexicogrfica).
Bem, queremos saber se realmente todas as solues
comeam/terminam por A/E. No haver exceo? Como o nosso
problema tem um nmero de possibilidades finito e pequeno, podemos
examinar todas. Como um computador pode fazer isso?

Calma! No precisamos saber programao de computadores.


Basta lembrar que computadores tm facilidade para tratar informaes organizadas. Como isso funciona no nosso caso?
Digamos que achei a soluo codificada pela sequncia de letras
AEBDCBADE. Mesmo sem o desenho, podemos verificar que esta
de fato uma soluo. As arestas disponveis so:
AB AD AE BC BD BE CD DE
Comeamos pela aresta AE. Ela est disponvel? Sim. Retiramos ela

GrafosModfranci
2009/6/30
page 49
Estilo OBMEP

N SEC. 3.2: ESTRUTURA DE DADOS

49

da lista de disponveis:
AB AD AE
== BC BD BE CD DE
A prxima aresta a ser examinada EB. Est disponvel? Sim.
Retiramos ela da lista de disponveis:
AB AD AE
== BC BD BE
== CD DE
(Repare que no nosso problema EB e BE so a mesma coisa.)
E assim por diante. A sequncia da verificao est a abaixo:
AEBDCBADE

AB AD AE
== BC BD BE CD DE

AEBDCBADE

AB AD AE
== BC BD BE
== CD DE

AEBDCBADE

AB AD AE
== BC BD
== BE
== CD DE

AEBDCBADE

AB AD AE
== BC BD
== BE
== CD
== DE

AEBDCBADE

AB AD AE
== BC
== BD
== BE
== CD
== DE

AEBDCBADE

AB
== AD AE
== BC
== BD
== BE
== CD
== DE

AEBDCBADE

AB
== AD
== AE
== BC
== BD
== BE
== CD
== DE

AEBDCBADE

AB
== AD
== AE
== BC
== BD
== BE
== CD
== DE
==

E a verificao mostra que a soluo boa.


Observe que no usamos o desenho. E que foi fundamental a
maneira como apresentamos os dados. o que chamamos uma estrutura de dados. Lembre-se, computadores so mquinas e no

GrafosModfranci
2009/6/30
page 50
Estilo OBMEP

50

 CAP. 3: MAIS CICLOS E MAIS CAMINHOS

podemos passar informaes de qualquer jeito. A estrutura de dados


fundamental.
No temos a inteno aqui de explicitar o funcionamento de um
computador, mas intuitivamente percebemos que com a estrutura adequada e uma sequncia de procedimentos (um programa!), isto , um
algoritmo, podemos verificar se uma sequncia de 9 letras (por qu
9?) ou no uma soluo.
Vamos fazer algumas contas. Temos 8 arestas disponveis e podemos numer-las de 1 a 8. Podemos pensar num procedimento (diferente do que usamos antes) que verifique se uma determinada sequncia de 8 algarismos do tipo (1, 2, 3, 4, 5, 6, 7, 8) ou (3, 5, 6, 2, 8, 4, 7, 1)
ou no uma soluo para o problema da casinha. Melhor ainda,
podemos colocar estas sequncias em ordem de (1, 2, 3, 4, 5, 6, 7, 8) at
(8, 7, 6, 5, 4, 3, 2, 1).
Quantas sequncias temos? Na apostila [2] vimos que teremos
8! = 8 7 6 5 4 3 2 1 = 40 320 sequncias. So as permutaes de 8 elementos. Ora, um bom computador pode gerar e
verificar estas sequncias todas em segundos! Poderemos ter certeza
de que todas as solues realmente comeam (ou terminam) com a
letra A ou E.
Isto se chama uma soluo por fora bruta e no usamos
nenhuma sofisticao matemtica, nenhum teorema. Ser o fim da
Matemtica? No bem assim...
Lembre-se do prefeito. Digamos que a cidade dele no tenha 8
ruas, mas 20. No uma grande cidade e podemos tentar usar a
mesma fora bruta do computador para resolver o problema de per-

GrafosModfranci
2009/6/30
page 51
Estilo OBMEP

N SEC. 3.3: GRAFOS EULERIANOS

51

correr com o caminho sem repetio de ruas. Se temos 20 ruas,


teremos 20! sequncias. Quanto isso?
20! = 2 432 902 008 176 640 000 sequncias
So muitas sequncias. Mas ser que um bom computador no
resolveria este problema? Se o computador verificasse um milho
de sequncias por segundo (e poucos computadores o fazem hoje em
dia) ele demoraria (os clculos s incluem a parte inteira):
2 432 902 008 176 640 000 1 000 000 2 432 902 008 170 segundos
2 432 902 008 170 60 40 548 366 800 minutos
40 548 366 800 60 675 806 110 horas
675 806 110 24 28 158 580 dias
28 158 580 365 77 140 anos
77 140 1 000 77 milnios
O prefeito no pode esperar tanto tempo (nem ns, nem ningum).
Quem vir nos socorrer? Um teorema de Euler.

3.3

Grafos Eulerianos

Um grafo com m arestas dito euleriano se existe uma trilha


fechada de comprimento m em G; em outras palavras, se podemos
percorrer cada aresta uma e s uma vez partindo de um vrtice e a

GrafosModfranci
2009/6/30
page 52
Estilo OBMEP

52

 CAP. 3: MAIS CICLOS E MAIS CAMINHOS

ele retornando. Se o grafo no euleriano mas tem uma trilha aberta


de comprimento m, ele dito semieuleriano.
Em outras palavras, podemos desenhar um grafo euleriano (ou
melhor, uma representao grfica dele) sem retirar o lpis do papel
e retornando ao ponto inicial. Num grafo semieuleriano comeamos
num ponto e terminamos em outro.

Figura 3.4:
Na figura acima, G1 euleriano (a trilha pode ser a-b-c-d-e-f-a-db-e-a), G2 semieuleriano (a trilha pode ser a-e-b-d-c-b-a-d-e) e G 3
no euleriano, nem semieuleriano.
J vimos que o problema (e o nome euleriano) se originou com
o problema das pontes de Kenisberg. Euler mostrou que a resposta
era negativa, estabelecendo assim uma condio necessria.
Comeamos por um lema simples porm necessrio.
Lema. Se todo vrtice de um grafo (no necessariamente simples) G
tem grau maior ou igual a 2, ento G contm um ciclo.
Demonstrao. Se G contm laos ou arestas mltiplas, no h o que

GrafosModfranci
2009/6/30
page 53
Estilo OBMEP

N SEC. 3.3: GRAFOS EULERIANOS

53

provar, pois, automaticamente, G contm um ciclo. Consideramos,


portanto, apenas os grafos simples. partir de um vrtice v 0 , qualquer, iniciamos nossa trilha. Quando chegamos a um vrtice qualquer, ou o estamos visitando pela primeira vez e podemos continuar,
ou chegamos a um vrtice j visitado, produzindo um ciclo. Como o
nmero de vrtices finito, o lema est provado.
E agora, o teorema.

Teorema de Euler (Euler 1736). Um grafo conexo (no


necessariamente simples) G euleriano se, e somente se, todos os
seus vrtices tem grau par.
Demonstrao.
() Suponhamos que G tenha uma trilha fechada de comprimento m.
Cada vez que a trilha passa por um vrtice utiliza duas novas arestas,
uma para entrar e outra para sair. Logo, o grau de cada vrtice deve
ser obrigatoriamente par.
() Usaremos induo sobre o nmero de arestas m do grafo. Por
vacuidade, o teorema vlido quando m = 0. Suponhamos que o
teorema seja vlido para todos os grafos com menos do que m arestas.
Sendo G conexo, todos os vrtices tm grau maior do que 2, pois os
graus so pares. Pelo lema anterior, G contm um ciclo (que uma
trilha fechada). Dentre todos as trilhas fechadas em G escolhemos
uma trilha T com comprimento mximo. Se T tem comprimento
m, o teorema est provado. Caso contrrio, consideramos o grafo H
resultante da retirada das arestas de T . Como retiramos um nmero

GrafosModfranci
2009/6/30
page 54
Estilo OBMEP

54

 CAP. 3: MAIS CICLOS E MAIS CAMINHOS

par de arestas de cada vrtice de T , e todos os vrtices do grafo tem


grau par (pela hiptese), pelo menos uma das componentes de H
tem um vrtice em comum com T e tem todos os vrtices com grau
par. Pela hiptese de induo, H tem uma trilha fechada que passa
por todos os vrtices de H, e podemos formar uma trilha fechada
maior concatenando T com a trilha em H. Mas isto contraria a
maximalidade na escolha de T .

Corolrio. Um grafo conexo (no necessariamente simples) G


semieuleriano se, e somente se, no mximo, dois vrtices tm grau
mpar.
Demonstrao. Deixada ao leitor. (Sugesto: pense em acrescentar
uma aresta a dois vrtices de grau mpar.)
Um algoritmo decorrente da demonstrao do teorema acima assegura a construo de uma trilha fechada de comprimento m num grafo
euleriano. A demonstrao da correo do algoritmo pode ser encontrada em [6]. Podemos dar uma ideia do funcionamento do algoritmo e
do motivo pelo qual ele funciona. Veja a figura 3.5. Comeando nossa
trilha pelo vrtice a poderamos percorrer abf cedcbef a, chegando a
um beco sem sada. Repare que os graus eram todos pares e a retirada
de um ciclo subtrai sempre nmeros pares dos graus.
O grafo restante tambm tem vrtices com grau par (veja ainda a
figura 3.5).
Este resto pode ser percorrido pela trilha fechada dghijkcjhd.
Basta agora incluir essa trilha na trilha inicial onde est o vrtice d.

GrafosModfranci
2009/6/30
page 55
Estilo OBMEP

55

N SEC. 3.3: GRAFOS EULERIANOS

Figura 3.5:
Nossa trilha fica abf ced(dghijkcjhd)dcbef a(veja a figura 3.6).

g
h

a
b

c
k
Figura 3.6:

GrafosModfranci
2009/6/30
page 56
Estilo OBMEP

56

 CAP. 3: MAIS CICLOS E MAIS CAMINHOS

Exerccio

Na figura 3.7, quais grafos so eulerianos? Quais so semieulerianos? No caso dos semieulerianos, por onde devemos comear (terminar) nossa trilha?

Figura 3.7:

GrafosModfranci
2009/6/30
page 57
Estilo OBMEP

N SEC. 3.4: O PROBLEMA CHINS DO CARTEIRO

3.4

57

O Problema Chins do Carteiro

Esse problema uma aplicao bastante importante do conceito


de grafo euleriano. Usamos um grafo valorado onde s arestas
associado um peso, isto , uma funo f : A < + . Este peso pode
representar comprimento, custo, tempo, ou o que a modelagem do
problema exigir. J vimos este conceito no caso do algoritmo de
Dijkstra.
O problema chins do carteiro (que tem este nome por ter sido
apresentado pela primeira vez por um pesquisador chins e no pela
nacionalidade do carteiro...) consiste em minimizar o esforo de um
carteiro que percorre todas as ruas de uma cidade. Ora, se o grafo
em questo euleriano, no h problema. Mas se este no for o caso,
teremos que eulerizar o grafo. Lembramos que o nmero de vrtices
de grau mpar par (veja o corolrio na seo 2.3), logo poderemos
unir pares destes vrtices por novas arestas, tornando-os pares.
claro que no construiremos novas ruas! A ideia fazer o carteiro
percorrer ruas repetidas de forma econmica. O problema pode se
complicar bastante, mas hoje h algoritmos que produzem resultados
aproximados com bastante eficincia. um problema bastante estudado devido economia que uma boa soluo pode gerar. Vamos
ilustrar o caso mais simples possvel, quando o grafo semi-euleriano,
isto , quando tem apenas dois vrtices de grau mpar.
O menor caminho entre os vrtices a e b (calculado pelo algoritmo
de Dijkstra) indica que o melhor meio de eulerizar o grafo construir
uma aresta virtual entre a e b, o que significa simplesmente percorrer o caminho av2 , v2 v3 , v3 v4 , v4 b como se fosse uma aresta. Assim,

GrafosModfranci
2009/6/30
page 58
Estilo OBMEP

58

 CAP. 3: MAIS CICLOS E MAIS CAMINHOS

aresta virtual
v5

10

a
3

v1
5

8
3 v2
3

v3

8
2

v4

12

8
v2

v1

v5

10

12

3
5

4
v6

v3

8
7

b
2
v4

4
v6

Figura 3.8:
gastaremos menos a sola do carteiro.

3.5

Grafos e Ciclos Hamiltonianos

Um problema aparentemente similar ao dos grafos eulerianos o


de procurar em G uma trilha fechada que passe por todos os vrtices uma e s uma vez. Uma trilha assim teria de ser necessariamente um ciclo (salvo no caso do grafo nulo com um vrtice);
chamamos um tal ciclo de ciclo hamiltoniano. O nome homenageia Sir Willian R. Hamilton, que estudou e divulgou o problema
embora a primeira formulao tenha sido feita por Kirkman em
1885. As primeiras definies de grafo hamiltoniano e de grafo
semi-hamiltoniano seguem as mesmas diretrizes dos grafos eulerianos. Um grafo e seu ciclo hamiltoniano aparecem na figura 3.9(a);
um grafo semi-hamiltoniano aparece na figura 3.9(b).

GrafosModfranci
2009/6/30
page 59
Estilo OBMEP

N SEC. 3.6: O PROBLEMA DO CAIXEIRO VIAJANTE PCV

59

(b)

(a)

Figura 3.9:

As semelhanas, entretanto, param por aqui. O problema de saber


se um grafo ou no hamiltoniano um dos mais estudados da teoria dos grafos por sua aplicabilidade em comunicao, transporte e
planejamento. Entretanto, at hoje, nenhuma condio necessria e
suficiente elegante para que um grafo seja hamiltoniano foi encontrada. Na verdade, todos os teoremas se encontram muito longe de
oferecer uma previso razovel de soluo.

3.6

O Problema do Caixeiro Viajante PCV

O PCV um dos problemas mais estudados no campo da pesquisa


operacional, mas at hoje no foi encontrado um algoritmo computacionalmente eficiente para resolv-lo. Sua formulao simples:
dado um grafo completo valorado G, desejamos determinar o valor
do menor ciclo hamiltoniano de G. Tomemos o exemplo dado pela
seguinte matriz valorada de adjacncia

GrafosModfrancisc
2009/6/30
page 60
Estilo OBMEP

60

 CAP. 3: MAIS CICLOS E MAIS CAMINHOS

XXX

404

270

490

490

338

258

404

XXX

618

890

890

460

320

270

618

XXX

360

360

210

240

490

890

360

XXX

78

390

330

490

890

360

78

XXX

390

330

338

460

210

390

390

XXX

270

258

320

240

390

330

270

XXX

Como o grafo em questo K7 , uma soluo bvia seria examinar


todas as permutaes entre os vrtices, cada uma correspondendo a
um ciclo hamiltoniano.
Com 7 vrtices, teremos 7! = 5760 permutaes; na verdade so
6! = 820, pois so permutaes circulares. Seja como for, uma tarefa
at modesta para um computador. Mas o PCV frequentemente trata
de grafos com mais de 60 vrtices. Isso nos daria 60!, o que nos
tomaria milnios, mesmo usando todos os computadores do mundo!
Nossa atitude ser ento de procurar um algoritmo heurstico,
isto , que usa uma ideia razovel, mesmo que no assegure a melhor soluo, a soluo tima. A primeira tentativa um algoritmo
guloso que parte do ponto A e procura sempre a menor distncia
ao ponto da vez. No nosso caso, o ciclo produzido seria a-g-c-f-gb-d-e-a, com valor 2470. A contraindicao para o algoritmo guloso
que no final terminamos por aceitar arestas de valores muito altos. Observamos, entretanto, que estamos procura de um ciclo, e
no temos portanto necessidade de agir sequencialmente. Uma outra
tentativa heurstica seria procurar agregar sempre a aresta de menor
valor que no produza ciclo com menos de 7 vrtices nem produza

GrafosModfranci
2009/6/30
page 61
Estilo OBMEP

N SEC. 3.6: O PROBLEMA DO CAIXEIRO VIAJANTE PCV

61

vrtices de grau 3 (num ciclo, todos os vrtices so de grau 2). As


escolhas recaem sobre:
Aresta

Valor

DE
CF
CG
GA
AC
FG
AF
CD
CE
DF
BE
AB

78
210
240
258
Bifurcao
Bifurcao
Fecha ciclo
Bifurcao
Bifurcao
390
890
404

O ciclo a-c-d-e-f-g-b-a e o valor conseguido tambm 2 470. Isso


foi coincidncia, como veremos em outros exemplos. A ideia parecia
boa e o resultado foi um pouco melhor. Entretanto, o melhor valor
encontrado, examinando todas as possibilidades, corresponde ao ciclo
a-c-d-e-f-g-b-a com o valor, bem inferior, de 2 092.
claro, se tivermos que examinar o PCV para 20 cidades teramos
que examinar cerca de 20! permutaes e j vimos que este um
nmero muito grande. Pior ainda, no foi descoberto at o momento
um algoritmo eficiente para este problema (como no caso euleriano,
em que o teorema de Euler nos salvou). E, ainda pior, os cientistas
da computao acreditam que ele pertena a uma classe de problema
para os quais no h uma soluo elegante. Vamos falar um pouco
sobre isto adiante.

GrafosModfranci
2009/6/30
page 62
Estilo OBMEP

62

 CAP. 3: MAIS CICLOS E MAIS CAMINHOS

Exerccio
Na figura 3.10 temos um grafo completo, valorado nas arestas,
e desejamos encontrar o ciclo hamiltoniano com menor valor total
(Problema do Caixeiro Viajante). Para isto, use os algoritmos gulosos
descritos nesta seo e constate que o valor obtido sempre maior do
que o melhor valor (que pode ser encontrado por exame exaustivo).
A

20

10
50

D
80

150
C

70

Figura 3.10:

3.7

Uma Palavra sobre Complexidade

A anlise da complexidade de algoritmos um assunto bastante


tcnico e que foge inteno destas notas. Entretanto, as dificuldades
enfrentadas por quem trabalha com problemas combinatrios (entre
os quais os da teoria dos grafos) podem ser informalmente compreendidas. J viemos fazendo isto quando falamos de solues elegantes,
eficincia computacional, enfim, sugerindo qualitativamente que certos problemas tm sido mais resistentes a uma abordagem algortmica
e computacional do que outros.

GrafosModfranci
2009/6/30
page 63
Estilo OBMEP

N SEC. 3.7: UMA PALAVRA SOBRE COMPLEXIDADE

63

Um algoritmo composto de passos elementares; se a totalidade


dos passos exigidos por qualquer problema que este algoritmo resolva
dado por uma funo polinomial do tamanho da entrada do algoritmo,
um aumento de poder computacional pode reduzir significativamente
o tempo utilizado.
Entretanto, se a totalidade dos passos do algoritmo, no pior dos
casos, uma funo exponencial do tamanho da entrada, o aumento
do poder computacional tem pouco efeito sobre o tempo de execuo;
basta um pequeno incremento na entrada para inutilizar o aumento
computacional.
Dos algoritmos que j examinamos, o de pesquisa de menor distncia (Dijkstra) de complexidade polinomial assim como o da determinao se um grafo ou no euleriano (e de sua exibio, se este
for o caso). Para o PCV, entretanto, at hoje no foi descoberto um
algoritmo polinomial; mais ainda, a maior parte dos pesquisadores
acredita que isto no ser mesmo possvel.
Maior informao sobre complexidade computacional pode ser encontrada em Garey e Johnson [5].
Exerccios
1. Uma ponte uma aresta que, quando retirada, desconecta o
grafo.
Dado um grafo conexo G, um vrtice v ser chamado de vrtice
separador quando a sua retirada resultar num grafo desconexo
ou nulo. Prove que um grafo s tem uma ponte se tiver um

GrafosModfranci
2009/6/30
page 64
Estilo OBMEP

64

 CAP. 3: MAIS CICLOS E MAIS CAMINHOS


Ponte

Figura 3.11:
vrtice separador, mas a recproca no verdadeira.
2. Prove que dentre G e G, pelo menos um conexo.
3. Mostre que A2 , o quadrado da matriz de adjacncia de um grafo,
nos d o nmero de caminhos de comprimento 2 entre cada par
de vrtices do grafo. Que nmero aparece na diagonal principal de A2 ? Qual o significado da matriz Ak ? (Teorema de
Festinger).
4. Mostre que se um grafo tem 2.k vrtices de grau mpar seu conjunto de arestas pode ser particionado em k caminhos disjuntos.
5. Para que valores de n, p e q os grafos K n , Kp,q , Pn so eulerianos? semieulerianos? hamiltonianos? semi-hamiltonianos?
6. Mostre que Ki,j hamiltoniano se e s se i = j; e que neste
caso, existem b 2i c ciclos hamiltonianos disjuntos.
Observao: bxc o maior nmero inteiro menor ou igual a x.
6
Por exemplo: b 57 c = 0, b 41
3 c = 13 e b 2 c = 3.
7. Seja o grafo Qj = (Xj , Uj ) no qual Xj = {vetores de j coordenadas, cada uma igual a 0 ou 1 } e Uj = {(vj , wj )|vj difere de
wj por uma s coordenada}.

GrafosModfranci
2009/6/30
page 65
Estilo OBMEP

65

N SEC. 3.7: UMA PALAVRA SOBRE COMPLEXIDADE

A figura 3.12 mostra Q1 , Q2 e Q3 .


(1,1,1)

(1,1,0)
(1,0)

(1,1)

(1,0
,1)

(1,0,0)

(0,1,0)
(0)

(1)
(0,1)

(0,0)

Q1

(0,0,1)

(0,0,0)

Q2

(0,1,1)

Q3

Figura 3.12:
(a) Calcule nj = |Xj | e mj = |Uj |.
(b) Para que valores de j Qj euleriano? Justifique.
(c) Mostre que Qj bipartido.
(d) Para que valores de j Qj hamiltoniano? Justifique.
8. Mostre que o grafo de Petersen (ver figura 3.13) no hamiltoniano.

Figura 3.13:
9. Mostre que se G for euleriano, L(G) ser hamiltoniano, mas a
recproca no verdadeira.

GrafosModfranci
2009/6/30
page 66
Estilo OBMEP

Captulo 4

rvores
4.1

Definies e Caracterizaes

Um dos tipos mais frequentes de grafos so as rvores, j definidos


anteriormente como grafos conexos sem ciclos. Um grafo cujas componentes conexas so rvores chamado de floresta.

floresta

rvore

Figura 4.1:
Para um dado nmero de vrtices n, uma rvore o grafo conexo
com menor nmero de arestas. As vrias caracterizaes das rvores
podem ser reunidas no teorema a seguir.

66

GrafosModfranci
2009/6/30
page 67
Estilo OBMEP

N SEC. 4.1: DEFINIES E CARACTERIZAES

67

Teorema. Seja T um grafo com n vrtices. As seguintes afirmaes


so equivalentes:
(i) T uma rvore.
(ii) T no contm ciclos e tem n 1 arestas.
(iii) T conexo e tem n 1 arestas.
(iv) T conexo e toda aresta uma ponte.
(v) Todo par de vrtices de T ligado por um nico caminho.
(vi) T no contm ciclos, mas a adio de uma aresta produz um
nico ciclo.
Demonstrao.
(i) (ii): Pela definio de rvore, T no contm ciclos. Portanto, a
retirada de uma aresta uv separa u de v e o grafo separado em um
par de rvores T 0 e T 00 com n0 e n00 vrtices, respectivamente, tais que
n = n0 +n00 . Por induo, o nmero de arestas de T 0 n0 1 e o nmero
de arestas de T 00 n00 1. Acrescentando a aresta uv, conclumos que
o nmero de arestas de T , portanto, (n 0 1) + (n00 1) + 1 = n 1.
(ii) (iii): Se T fosse desconexo, cada componente seria uma rvore.
Por induo, o nmero de arestas em cada componente inferior em
uma unidade ao nmero de vrtices e o nmero total de arestas seria
inferior a n 1.
(iii) (iv): A retirada de qualquer aresta separa o grafo, pois n 2
arestas so insuficientes para conectar o grafo.

GrafosModfranci
2009/6/30
page 68
Estilo OBMEP

68

 CAP. 4: RVORES

(iv) (v): Se existisse mais de um caminho entre dois vrtices, o


grafo teria um ciclo e haveria uma aresta que no separaria o grafo.
(v) (vi): Se T contivesse um ciclo, haveria um par de vrtices ligado por mais de um caminho. A adio de uma aresta uv, concatenada
com o caminho (nico) entre u e v, produz um ciclo. Se este ciclo no
fosse nico, a retirada da aresta uv deixaria dois caminhos distintos
entre u e v.
(vi) (i): Basta mostrar que T conexo. Se T fosse desconexo, uma
aresta ligando duas componentes no produziria um ciclo.

4.2

rvores Geradoras

O Problema de Conexo de Peso Mnimo


Uma rvore geradora de uma componente conexa de um grafo
G, com n vrtices, um subgrafo que uma rvore com n 1 arestas;
isto , toca todos os vrtices.
Vimos que um algoritmo guloso pode ser fcil de implementar, mas dificilmente dar um bom resultado (da o nome...). Uma
exceo ocorre na soluo do seguinte problema: Dado um grafo G
valorado, qual a rvore geradora de menor valor?. Por exemplo, se
queremos realizar a ligao de computadores em rede a custo mnimo,
que ligaes deveremos fazer?
A resposta ser uma rvore geradora, claro. Mas qual?

GrafosModfranci
2009/6/30
page 69
Estilo OBMEP

69

N SEC. 4.2: RVORES GERADORAS

O grafo da figura 4.2 mostra o custo entre as ligaes de um grafo


K5 .
a
100

40

60

42

102
6

44

44

42

46

Figura 4.2:
Para resolver o problema, usaremos o algoritmo de Kruskal.
Este algoritmo consiste em tomar a aresta de menor valor; se ela no
forma ciclo, a acrescentamos nossa rvore. Caso contrrio, ns a
desprezamos. Quando tivermos conseguido n1 arestas, nossa rvore
estar pronta.
No nosso caso:
ce6
a e 40
Agora h um empate entre ac e bd. Podemos escolher qualquer
uma.
a c forma ciclo.

GrafosModfranci
2009/6/30
page 70
Estilo OBMEP

70

 CAP. 4: RVORES

b d 42
Temos outro empate, agora entre b c e d e. Podemos escolher
qualquer uma.
b c 44
Ja temos 4 arestas. Nossa rvore est completa.
Total: 132
Nossa rvore ficar assim:

a
40
b

e
6

42
44

Figura 4.3:
Teorema. O algoritmo de Kruskal fornece uma soluo tima para
o problema da conexo de peso mnimo.
Demonstrao. O algoritmo, evidentemente, fornece uma rvore geradora T . Suponhamos que T no tenha peso mnimo, isto , existe
uma rvore geradora T 0 tal que o peso de T 0 menor do que o peso
de T . Seja e a primeira aresta escolhida para T que no pertence a
T 0 . Se adicionarmos e a T 0 obtemos um ciclo que contm uma aresta
ek que no est em T . Retiramos a aresta e k e temos uma rvore

GrafosModfranci
2009/6/30
page 71
Estilo OBMEP

71

N SEC. 4.2: RVORES GERADORAS

T 00 com peso menor que T . Mas neste caso, esta aresta e k teria sido
escolhida pelo algoritmo no lugar de e, o que mostra que o algoritmo
constri efetivamente uma rvore de menor peso.
Um algoritmo guloso pode ser usado para obter um limite inferior
para o PCV. Como um ciclo um caminho adicionado de uma aresta,
um limite inferior para o PCV dado pelo valor da rvore geradora
mnima (obtido por um algoritmo guloso) mais o menor valor de uma
aresta no usada na rvore.
Exerccios
1. Desenhe todas as rvores com 6 vrtices e com 7 vrtices.
2. Mostre que um grafo conexo, com n vrtices e m arestas, tem,
no mnimo, m n + 1 ciclos distintos.
3. Determine todas as rvores geradoras do grafo da figura 4.4.

Figura 4.4:
4. (a) Mostre que toda rvore um grafo bipartido.
(b) Quais rvores so tambm grafos bipartidos completos?

GrafosModfranci
2009/6/30
page 72
Estilo OBMEP

72

 CAP. 4: RVORES

5. Como podemos adaptar o algoritmo de Kruskal para obter o


valor de uma rvore geradora de valor mximo?
6. Prove que um grafo conexo uma rvore se, e somente se, tem
uma nica rvore geradora.
7. Prove que uma rvore com > 1 tem, no mnimo, vrtices
pendentes.
8. Prove que uma rvore em que exatamente 2 vrtices no so
vrtices separadores um caminho.

GrafosModfranci
2009/6/30
page 73
Estilo OBMEP

Captulo 5

Subconjuntos Especiais de
um Grafo
5.1

Conjuntos Independentes

J vimos, pelo menos, um exemplo de subconjunto notvel de um


grafo: um subgrafo independente, no qual nenhum par de vrtices
est ligado. Um conjunto independente pode desempenhar papel importante em uma modelagem.
Suponhamos que um grafo represente a incompatibilidade de
horrios entre professores que devem dar prova final; os vrtices x
e y estaro ligados se representarem professores que tm alunos em
comum para ministrar a prova. Qual o maior nmero de professores
que podem dar prova ao mesmo tempo? A resposta dada pelo subconjunto independente mximo de vrtices do grafo.
O subconjunto assinalado com quadrados negros no grafo da figura 5.1 mostra um conjunto com estas caractersticas. O nmero
de independncia (G) a cardinalidade do subconjunto independente mximo de vrtices do grafo. No nosso exemplo (figura 5.1),
73

GrafosModfranci
2009/6/30
page 74
Estilo OBMEP

74

 CAP. 5: SUBCONJUNTOS ESPECIAIS DE UM GRAFO

(G) = 4.

Figura 5.1:
Aplicaes do conceito de conjunto independente surgem quando,
por exemplo, desejamos evitar duplicao de esforos. Suponhamos
que num parque, representado pelo grafo da figura 5.2, eu quisesse
instalar barracas para venda de sorvete. A operadora das barracas
faz as seguintes restries:
Uma barraca deve ser localizada em uma esquina (vrtice).
Esquinas prximas (vrtices adjacentes) s admitem uma barraca.
Estamos procurando ento um conjunto independente. Para instalar o mximo de barracas procuramos um conjunto independente
mximo. J vimos que esta pode ser uma tarefa complexa. Na figura 5.3 a configurao da esquerda mostra um conjunto independente
maximal, isto , no podemos acrescentar mais barracas de sorvete.
Mas a configurao da direita tambm independente e contm quase
o dobro de barracas.

GrafosModfranci
2009/6/30
page 75
Estilo OBMEP

N SEC. 5.1: CONJUNTOS INDEPENDENTES

Figura 5.2:

Figura 5.3:

75

GrafosModfranci
2009/6/30
page 76
Estilo OBMEP

76

5.2

 CAP. 5: SUBCONJUNTOS ESPECIAIS DE UM GRAFO

Colorao

Suponha, no exemplo anterior, que quisssemos saber qual o


menor nmero de horrios necessrios para ministrar as provas. Para
isto, devemos resolver o problema de particionar o conjunto de vrtices do grafo em subconjuntos independentes; cada conjunto corresponder a um horrio de prova. Uma forma de resolver o problema
atribuir cores aos vrtices de forma que vrtices adjacentes tenham
necessariamente cores diferentes. O menor nmero de cores que se
pode utilizar ser portanto a soluo do problema.
Observao. No precisamos efetivamente colorir os vrtices,
basta atribuir um nmero ou um smbolo aos vrtices.
Podemos colorir os vrtices com 12 cores (uma para cada vrtice),
mas o menor nmero possvel de cores 4 (veja a figura 5.1). O
menor nmero de cores para colorir os vrtices de um grafo G
chamado nmero cromtico de G e denotado por (G). No caso,
(G) = 4.

Teorema. Para todo grafo G, tem-se que (G) + 1.


Demonstrao. Colorimos vrtice por vrtice. Cada vrtice pode ser
adjacente a, no mximo, vrtices. Podemos sempre encontrar uma
cor com a qual colorir o vrtice da vez.
A demonstrao acima fornece um algoritmo para colorir um grafo
com + 1 cores.

GrafosModfranci
2009/6/30
page 77
Estilo OBMEP

N SEC. 5.3: APLICAES DE COLORAO

77

Apresentamos, sem demonstrar, um teorema clssico que reduz


um pouco o limite acima.
Teorema (Brooks 1941). Se G um grafo conexo que no seja
Kn e tal que (G) 3, ento (G) (G).
Teorema. Um grafo G bipartido se, e somente se, (G) = 2.
Demonstrao. Basta fazer corresponder cada uma das parties independentes de G a uma cor.

5.3

Aplicaes de Colorao

As aplicaes de colorao aparecem quando precisamos repartir o


conjunto de vrtices em conjuntos de vrtices independentes disjuntos.
Voltando ao problema do parque da Seo 6.1, suponha que quisssemos instalar barracas de sorvete, pipocas, cachorro-quente etc. As
restries agora sero:
Uma barraca deve ser localizada em uma esquina (vrtice).
Esquinas prximas (vrtices adjacentes) s admitem barracas
com servios diferentes.
Por motivos comerciais, queremos evitar a diversificao excessiva
de servios. Qual seria o menor nmero de servios que poderamos
usar? Vemos na figura 5.4 que podemos colorir os vrtices com apenas 3 cores. Este nmero mnimo pois o grafo inclui um subgrafo
isomorfo a K3 .

GrafosModfranci
2009/6/30
page 78
Estilo OBMEP

78

 CAP. 5: SUBCONJUNTOS ESPECIAIS DE UM GRAFO

Figura 5.4:
Uma outra aplicao clssica de colorao o problema dos exames. A tabela abaixo mostra a alocao de alunos nos exames finais
que eles devem prestar:
Alunos
Matemtica
Portugus
Ingls
Geografia
Histria
Fsica
Qumica
Biologia

1
X
X

X
X

8
X

10

X
X
X
X
X
X

11

12
X

13

14

15
X

X
X

X
X

X
X

16

Duas disciplinas s podem ter exames realizados simultaneamente


se no houver alunos comuns. Vamos construir um grafo com os vrtices {M, P, I, G, H, F, Q, B}; dois vrtices estaro ligados se tiverem
um aluno em comum.
A figura 5.5 mostra uma partio dos vrtices em dois conjuntos independentes disjuntos. Os exames podem ser realizados em 2
horrios, um para {B, G, H, M } e outro para {F, I, P, Q}.

GrafosModfranci
2009/6/30
page 79
Estilo OBMEP

79

N SEC. 5.3: APLICAES DE COLORAO

Figura 5.5:
Uma outra aplicao a determinao de perodos de um sinal de
trnsito. O desenho abaixo representa um cruzamento. As direes
permitidas esto assinaladas por setas. Veja a figura 5.6.
D

Figura 5.6:
Como organizar o trnsito? Vamos formar um grafo de incompatibilidade. Os vrtices sero as direes possveis:
V = AB, AC, AD, BA, BC, BD, DA, DB, DC, EA, EB, EC, ED

GrafosModfranci
2009/6/30
page 80
Estilo OBMEP

80

 CAP. 5: SUBCONJUNTOS ESPECIAIS DE UM GRAFO

Ligamos dois vrtices sempre que as direes forem incompatveis


(por exemplo AD e EB). Veja a figura 5.7.

Figura 5.7:
Observe que BA, DC e ED so compatveis com todas as direes,
sendo por isso vrtices isolados. Uma colorao dos vrtices corresponde a uma diviso em perodos. Poderamos usar 13 cores, uma
para cada direo, mas isso seria um desperdcio de tempo. Como os
vrtices AC, BD, DA e EB formam um K4 precisamos de pelo menos
4 cores. A partio em conjuntos independentes {AB, AC, AD},
{BC, BD, EA}, {BA, EB, EC, ED}, {DA, DB, DC} mostra que de
fato 4 cores (4 perodos) so suficientes, isto , (G) = 4.

Exerccios
1. O dono de uma loja de animais comprou uma certa quantidade
de peixes ornamentais de diversas espcies. Alguns destes peixes

GrafosModfranci
2009/6/30
page 81
Estilo OBMEP

81

N SEC. 5.3: APLICAES DE COLORAO

no podem ficar no mesmo aqurio. A compatibilidade entre as


espcies est retratada na tabela a seguir, onde X significa que
as espcies no devem ficar no mesmo aqurio.
(a) Qual o menor nmero de aqurios necessrio para abrigar
sem problemas todos os peixes?
(b) possvel distribuir os peixes de forma que cada aqurio
tenha (aproximadamente) o mesmo nmero de peixes?
A

A
B

X
X

X
X

X
X

X
X

X
X

X
X

2. Para os cruzamentos da figura 5.8, d uma sequncia econmica


de perodos para o sinal de trnsito.
A

Figura 5.8:

GrafosModfranci
2009/6/30
page 82
Estilo OBMEP

82

 CAP. 5: SUBCONJUNTOS ESPECIAIS DE UM GRAFO

3. Determine o nmero cromtico dos grafos da figura 5.9.

Figura 5.9:

5.4

Cliques

Uma clique de G um subgrafo completo de G. O nmero de


vrtices da clique mxima o nmero de clique de G, denotado
por (G). Note-se que uma clique de G corresponde a um conjunto
independente em G, isto (G) = (G).

5.5

Acoplamentos

Da mesma forma que selecionamos um conjunto independente de


vrtices, podemos considerar um conjunto independente de arestas,

GrafosModfranci
2009/6/30
page 83
Estilo OBMEP

83

N SEC. 5.5: ACOPLAMENTOS

isto , de arestas no incidentes duas a duas. Um conjunto deste tipo


chamado um acoplamento do grafo G.

G1

G2

G3

Figura 5.10:
Na figura 5.10 o acoplamento em G1 maximal (pois no pode
ser aumentado) mas no mximo. O acoplamento em G 2 mximo,
mas no toca todos os vrtices; os que so tocados so ditos vrtices
saturados e os outros vrtices no saturados. O acoplamento
em G3 mximo e satura todos os vrtices; dizemos ento que um
acoplamento perfeito. O nmero de acoplamento de um grafo G,
denotado por 0 (G), a cardinalidade do maior acoplamento de G.
Observao. Note a diferena entre os conceitos de mximo (o
conjunto de maior cardinal possvel dentro das condies exigidas)
e maximal (um conjunto que no pode ser aumentado sem violar as
condies exigidas). A mesma ideia se aplica a conjuntos mnimos
e minimais.
Dado um grafo G e um acoplamento M , um caminho M aumentante em G um caminho que liga dois vrtices no
saturados por M que alternam arestas de M e arestas de G M .

GrafosModfranci
2009/6/30
page 84
Estilo OBMEP

84

 CAP. 5: SUBCONJUNTOS ESPECIAIS DE UM GRAFO

Teorema (Berge). Um acoplamento M de um grafo G mximo


se, e somente se, no contm um caminho M -aumentante.

Demonstrao.
() Se h um caminho M -aumentante, podemos obter um acoplamento uma unidade maior adicionando as arestas do caminho fora de
M ao acoplamento e retirando as arestas em M do acoplamento. A
definio de caminho aumentante garante que o resultado ainda um
acoplamento.
() Se M no mximo, ento existe M 0 mximo. Considere
D = M M 0 , a diferena simtrica entre M e M 0 (isto , o conjunto de arestas de M e M 0 que no pertencem a M M 0 ); como
so acoplamentos, os vrtices em D tm grau no mximo 2. Logo, as
componentes de D so ciclos pares (alternam arestas de M e M 0 ) ou
caminhos. Como |M 0 | |M |, uma das componentes, ao menos, um
caminho alternando arestas de |M 0 | e |M | comeando e terminando
em M 0 . Este um caminho M -alternante.

5.6

Acoplamentos em Grafos Bipartidos

O acoplamento modela situaes em que formamos pares; se o


grafo G for bipartido, o acoplamento assume a forma de formao de
casais, e estudado de forma ligeiramente diferente. Seja G um grafo
bipartido com parties dos vrtices X e Y . Dizemos que temos um
acoplamento de X em Y quando um acoplamento de G satura Y
(mas no necessariamente X).

GrafosModfranci
2009/6/30
page 85
Estilo OBMEP

N SEC. 5.7: COLORAO DE ARESTAS

85

Apresentamos o seguinte teorema, sem demonstrao.

Teorema. Se G um grafo bipartido com parties de vrtices X


e Y , ento G tem um acoplamento de X em Y se, e somente se,
|N (S)| |S|, S X, sendo N (S) a vizinhana aberta de S.
Demonstrao. Ver em West [6].
A condio deste teorema tambm conhecida como Condio
de Hall.

Teorema. Se k > 0, qualquer grafo k-regular bipartido admite um


acoplamento perfeito.
Demonstrao. Comeamos contando as arestas pelas extremidades
em X e Y , as parties de vrtices. Cada aresta tem uma extremidade
em X e outra em Y , logo k.|X| = k.|Y | e, portanto, |X| = |Y |. S
precisamos ento provar a condio de Hall. Considere S X, tal
que haja r arestas entre S e N (S). Como G k-regular, temos que
r = k|S|. Do lado de Y temos r k.|N (S)|. Logo, k.|S| k.|N (S)|
e, finalmente, |S| |N (S)|.

5.7

Colorao de Arestas

Suponhamos que num grupo de pessoas vrias duplas devam ser


formadas para cumprir determinadas tarefas num laboratrio. O
grafo da figura 5.11 ilustra esta situao. Observe que uma mesma

GrafosModfranci
2009/6/30
page 86
Estilo OBMEP

86

 CAP. 5: SUBCONJUNTOS ESPECIAIS DE UM GRAFO

pessoa pode ter que cumprir uma tarefa em diversas duplas. Cada
tarefa destas necessita de 1 hora para ser executada. Qual o menor
nmero de horas necessrias para que todas as tarefas sejam realizadas?
a

f
c
e
d

Figura 5.11:
As arestas representam as duplas e, como cada indivduo s pode
trabalhar em uma tarefa de cada vez, tarefas executadas simultaneamente correspondem a um acoplamento. Podemos fazer corresponder
uma cor a cada horrio (j sabemos que esta cor pode ser um nmero
ou um smbolo) e nossa pergunta passa a ser:
Qual o mnimo de cores para colorir as arestas do grafo de modo
que arestas incidentes num mesmo vrtice recebam cores diferentes?
O menor nmero usado para colorir (propriamente) as arestas de
um grafo chamado ndice cromtico do grafo, notado por 0 (G). No
nosso exemplo conseguimos colorir as arestas com 4 cores (veja figura

GrafosModfrancis
2009/6/30
page 87
Estilo OBMEP

87

N SEC. 5.7: COLORAO DE ARESTAS

5.12) que evidentemente o menor nmero possvel pois o vrtice a


tem quatro arestas incidentes. Logo 0 (G) = 4. Os horrios ficariam
assim distribudos:
a

3
f

4
1

2
e
4

2
2

1
4

Figura 5.12:

Horrio(cor)

Duplas

ab, ce, df

ac, bd, ef

af, bc, de

ae, cd

Pelo que vimos acima, fica claro que 0 (G) . O teorema


a seguir (que apresentamos sem demonstrao) nos d um limite
superior bem estreito.

GrafosModfranci
2009/6/30
page 88
Estilo OBMEP

88

 CAP. 5: SUBCONJUNTOS ESPECIAIS DE UM GRAFO

Teorema (Vizing). Para qualquer grafo G, tem-se que


0 (G) + 1.
Para grafos bipartidos, entretanto, 0 (G) conhecido.
Teorema (Vizing). Para qualquer grafo G bipartido, 0 (G) = .
Demonstrao. Suponha que estamos colorindo as arestas uma por
uma, dispondo de cores. Ao colorir a aresta xy tentaremos encontrar uma cor que no esteja presente em arestas incidentes a x e nem
em arestas incidentes a y. Se for possvel, tudo bem. Se este no for
o caso, observemos que as arestas incidentes a x ocupam no mximo
1 cores (pois xy no est colorida), o mesmo acontecendo com
y. Isto nos garante que h uma aresta incidente a x que est colorida
com a cor cx , ausente nas arestas incidentes em y; por seu lado, existe
uma cor cy presente nas arestas incidentes em y e ausente nas arestas
incidentes a x. Formemos uma cadeia de arestas comeando em x
e alternando arestas de cor cx e cy (esta cadeia pode at, eventualmente, s possuir uma aresta). Com o grafo bipartido, as arestas c x
vo de uma partio para outra e as arestas c y retornam primeira
partio. Como cx est ausente em y, esta cadeia no passa pelo vrtice y. Podemos ento recolorir a cadeia intercambiando as cores c x e
cy , sem afetar a propriedade da colorao. Depois deste intercmbio
a cor cx estar ausente em x e y e podemos colorir a aresta xy. Isto
mostra que todas as arestas podem ser coloridas utilizando apenas
cores.

GrafosModfranci
2009/6/30
page 89
Estilo OBMEP

89

N SEC. 5.7: COLORAO DE ARESTAS

Observao. A tcnica desta demonstrao se baseia numa ideia de


Kempe e retornaremos a ela quando falarmos do Problema das 4 cores
no captulo sobre planaridade.
Uma aplicao conhecida da colorao de arestas o problema dos
exames orais. Trs professores devem examinar 6 estudantes, segundo
a seguinte lista:
Professor 11

A, C, D

Professor 22

A, C

Professor 33

A, B, D

A cada hora um professor chama um dos alunos para ser examinado. Dois professores no podem examinar um aluno e cada professor examina apenas um aluno. Qual o menor espao de tempo
que podemos utilizar? Usaremos um modelo de grafo bipartido (veja
figura 5.13) de um lado os professores, do outro os alunos. Uma
colorao das arestas representa uma diviso de horrios. A colorao {P 1A, P 2C, P 3D}, {P 1C, P 2A, P 3B}, {P 1D, P 2E, P 3F },
uma partio das arestas em acoplamentos disjuntos o que garantido pelo teorema demonstrado anteriormente.
Outro problema clssico da colorao de arestas a organizao de
passeios por duplas. Suponha que um batalho com 2.t soldados sai
para marchar todo dia. Quantos passeios podemos fazer de modo que
cada soldado tenha sempre um companheiro diferente? Este nmero
, no mximo 2.t 1 pois este o nmero de companheiros que cada
soldado tem. Veremos que este o nmero exato. Para melhor enxergar este fato vamos dar o exemplo com t = 3, isto , com 6 soldados.

GrafosModfranci
2009/6/30
page 90
Estilo OBMEP

90

 CAP. 5: SUBCONJUNTOS ESPECIAIS DE UM GRAFO

A
P1

P2

D
P3
E
F

Figura 5.13:
Se pensarmos em todas as duplas possveis estamos pensando no grafo
K6 , os soldados sendo os vrtices e as arestas as duplas. Um passeio
corresponder a um acoplamento perfeito e uma colorao das arestas
usando acoplamentos perfeitos nos dar o nmero possvel de passeios.
Desenhamos K6 da seguinte forma:

Figura 5.14:
Os acoplamentos so obtidos pelas arestas paralelas e perpendiculares (figura 5.15):

GrafosModfranci
2009/6/30
page 91
Estilo OBMEP

N SEC. 5.7: COLORAO DE ARESTAS

91

Figura 5.15:
A colorao produzida :
{12, 36, 45}, {13, 24, 56}, {14, 26, 35}, {15, 23, 46}, {16, 25, 34}

Exerccios
1. Exiba uma colorao mnima das arestas de K 10 .
2. (ndice cromtico de K2t1 )
(a) K5 tem 5 vrtices e 10 arestas. Um acoplamento de K 5
pode ter no mximo ......... arestas.
(b) Para uma colorao das 10 arestas de K 5 precisamos de
(no mnimo) ......... acoplamentos (cores).
(c) Mostre que para obter uma colorao de K 5 basta tomar

GrafosModfranci
2009/6/30
page 92
Estilo OBMEP

92

 CAP. 5: SUBCONJUNTOS ESPECIAIS DE UM GRAFO

uma colorao de K6 e desconsiderar as arestas que contenham o vrtice 6.


(d) Mostre que:
0 (Kt ) = t 1, se t par.
0 (Kt ) = t, se t mpar.

5.8

Outros Subconjuntos

Outros tipos de subconjuntos e de invariantes tm sido estudados.


Citaremos apenas trs.
Coberturas de vrtices um subconjunto de vrtices tal que
toda aresta incidente a um vrtice do conjunto. O nmero
de cobertura de vrtices de um grafo G, denotado por (G),
a cardinalidade da maior cobertura de vrtices de G.
Coberturas de arestas um subconjunto de arestas tal que
todo vrtice tocado por uma aresta do conjunto. O nmero
de cobertura de arestas de um grafo G, denotado por 0 (G),
a cardinalidade da maior cobertura de arestas de G.
Conjuntos dominantes um subconjunto de vrtices tal que
todo vrtice do grafo est no conjunto ou adjacente a um de
seus vrtices. O nmero de dominncia de um grafo G, denotado por (G), a cardinalidade do maior conjunto dominante
de G.

GrafosModfranci
2009/6/30
page 93
Estilo OBMEP

N SEC. 5.8: OUTROS SUBCONJUNTOS

93

Exerccios
1. Qual o nmero de independncia (P et) do grafo de Petersen?
2. Qual o nmero de colorao (P et) do grafo de Petersen?
3. Apresente um acoplamento maximal do grafo de Petersen com 3
arestas. Encontre caminhos aumentantes que forneam acoplamentos de 4 e 5 arestas.
4. Prove que

n
2

(G) n + 1

5. Mostre que se Kt subgrafo de G, ento (G) t. verdade


que se (G) = t, ento Kt subgrafo de G?
6. O ndice cromtico do grafo G, denotado por 0 (G), o menor
nmero de cores com que podemos colorir as arestas de maneira
que duas arestas incidentes tenham cores diferentes.
(a) Calcule 0 (Kn ).
(b) Calcule 0 (P et), o ndice cromtico do grafo de Petersen.
7. (a) Prove que um conjunto independente maximal um conjunto dominante.
(b) Prove que um conjunto dominante minimal pode no ser
um conjunto independente.
8. Mostre que:
(a) 0 (G) (G).
(b) (G) 0 (G).

GrafosModfranci
2009/6/30
page 94
Estilo OBMEP

94

 CAP. 5: SUBCONJUNTOS ESPECIAIS DE UM GRAFO

(c) (G).(G) n.
(d) (G) n2 .

GrafosModfranci
2009/6/30
page 95
Estilo OBMEP

Captulo 6

Grafos Planares
6.1

Definies e Resultados Simples

Um grafo planar um grafo que admite uma representao grfica em que as arestas s se encontrem (possivelmente) nos vrtices a
que so incidentes. Exemplos clssicos de grafos planares so dados
pelos grafos que representam os poliedros. Na figura 6.1, apresentamos os grafos dos 5 slidos platnicos: tetraedro, cubo, octaedro,
dodecaedro e icosaedro.

Figura 6.1:

95

GrafosModfranci
2009/6/30
page 96
Estilo OBMEP

96

 CAP. 6: GRAFOS PLANARES

Uma pergunta que pode ser feita se existe um grafo que no


seja planar. Mostraremos que o grafo K 5 no planar. De fato,
qualquer representao de K5 dever ter um ciclo de comprimento 5
que divida o plano em interior e exterior. S conseguimos colocar
duas arestas no interior sem que se cruzem; no exterior, a situao
a mesma. Nos sobra uma aresta.
Quantas arestas pode ter um grafo planar? Uma representao
grfica de um grafo com pelo menos um ciclo separa o plano em
regies (no caso das rvores, temos uma nica regio). Estas regies
so chamadas faces; no devemos esquecer que uma das faces tudo
que sobra do plano a face ilimitada. O nmero de faces de um
grafo ser designado por f . A figura 6.2 mostra duas representaes
do mesmo grafo, ilustrando que qualquer face pode ser colocada como
face ilimitada.

Figura 6.2:
Para grafos planares, vale a relao de Euler para poliedros
convexos.

Teorema de Euler. Num grafo planar conexo vale f m + n = 2.

GrafosModfranci
2009/6/30
page 97
Estilo OBMEP

N SEC. 6.1: DEFINIES E RESULTADOS SIMPLES

97

Demonstrao. Demonstraremos o teorema por induo sobre o


nmero de arestas. Tomemos um grafo conexo qualquer. Se for uma
rvore, temos f m + n = 1 (n 1) + n = 2. Se houver um ciclo,
retiramos uma aresta do ciclo, e o grafo fica com uma face a menos,
mas pela hiptese de induo a relao vale para o novo grafo. Temos
ento (f 1) (m 1) + n = 2 e, portanto, f m + n = 2.
Observamos que podemos acrescentar arestas a um grafo planar
sempre que uma poro do plano estiver limitada por um ciclo de
comprimento maior do que 3. Logo, um grafo maximal planar
(i.e., um grafo ao qual no poderemos acrescentar arestas sem
comprometer a planaridade) tem uma representao composta por
ciclos de comprimento 3. Isto nos d outra relao importante.

Teorema. Num grafo planar conexo G vale m 3.n 6; a igualdade


vale se G maximal planar.
Demonstrao. Se formos contar as arestas de cada face, contaremos
duas vezes cada aresta do grafo. Como cada face tem no mnimo 3
arestas (a igualdade valendo no caso maximal) temos:
3.f 2.m.
Substituindo na frmla de Euler:
f m + n = 2,
3.f 3.m + 3.n, = 6,

GrafosModfranci
2009/6/30
page 98
Estilo OBMEP

98

 CAP. 6: GRAFOS PLANARES

2.m 3.m + 3.n 6,


m 3.n 6.

Este teorema nos d outra demonstrao de que K 5 no planar.


De fato, K5 (e de resto todos os grafos completos com mais do que 4
vrtices) no obedece relao acima: 10 > 3.5 6.

Teorema. Num grafo planar bipartido conexo G vale


m 2.n 4.

Demonstrao. Observamos que um grafo bipartido s tem ciclos


pares. Cada face tem no mnimo 4 arestas.
4.f 2.m.
Substituindo na frmula de Euler:
f m + n = 2,
4.f 4.m + 4.n = 8,
2.m 4.m + 4.n 8,
m 2.n 4.

GrafosModfranci
2009/6/30
page 99
Estilo OBMEP

99

N SEC. 6.2: TEOREMA DE KURATOWSKI

Vemos agora que K3,3 no planar, pois 9 > 2.6 4. O problema


das casinhas, na introduo, acaba de ser resolvido.

6.2

Teorema de Kuratowski

A ideia de planaridade aparentemente topolgica, mas sempre


pairou a questo sobre se haveria uma caracterizao combinatria
dos grafos planares. A resposta foi dada atravs de um teorema, que
apresentaremos, sem demonstrao, depois de algumas definies.
Uma subdiviso do grafo G o grafo G0 que obtemos pela insero de P2 (caminho de comprimento 2) no lugar de uma aresta
de G. Um grafo G0 dito homeomorfo ao grafo G se G0 puder ser
obtido de G por sucessivas operaes de subdiviso (veja figura 6.3)

Figura 6.3:
Teorema (Kuratowski). Um grafo planar se no contiver subgrafo
homeomorfo a K5 ou a K3,3 .
Demonstrao: Ver em Fournier[7].
Como aplicao mostramos na figura 6.4 que o grafo de Petersen
no planar.

GrafosModfranci
2009/6/30
page 100
Estilo OBMEP

100

 CAP. 6: GRAFOS PLANARES

a b

K3,3

Figura 6.4:
Observamos que embora tenhamos tratado o exemplo graficamente, a verificao das condies do teorema pode ser feita de forma
computacional (embora possa ser complexa).

6.3

Dualidade

O Dual GD de um grafo simples planar G o grafo construdo


da seguinte maneira:
(i) A cada face de G associamos um vrtice em G D .
(ii) A cada aresta de G (que separa duas faces) associamos uma
aresta em GD ligando os vrtices correspondentes s faces.
Um bom exemplo so os slidos platnicos apresentados na figura
6.4. O cubo o dual do octaedro, o icosaedro o dual do dodecaedro e
o tetraedro o dual dele mesmo (autodual). Esses duais correspondem
aos duais da geometria clssica. A figura 6.5 mostra a correspondncia
entre as faces do cubo e os vrtices do octaedro.

GrafosModfrancis
2009/6/30
page 101
Estilo OBMEP

101

N SEC. 6.4: O PROBLEMA DAS 4 CORES

v5
f2
f4

f3

f1
f5

v4

f6

v2

v1
fi

v6
v3

vi

Figura 6.5:
Verifica-se com facilidade que o dual do dual de G o prprio
grafo G (desde que G tenha conexidade maior ou igual a 3).
A dualidade aparece num dos problemas mais famosos, no s da
teoria dos grafos, mas da matemtica.

6.4

O Problema das 4 Cores

Em 1852 Frederick Guthrie, aluno de Augustus de Morgan,


trouxe a este um problema proposto por seu irmo Francis Guthrie.
Na verdade, tratava-se de uma conjectura, hoje um teorema.

Teorema das 4 cores. Um mapa pode ser colorido com 4 cores.


Colorir um mapa colorir as regies de maneira que regies
fronteirias no sejam coloridas com a mesma cor. Usando a dualidade podemos formular o teorema em forma de colorao de vrtices.

GrafosModfranci
2009/6/30
page 102
Estilo OBMEP

102

 CAP. 6: GRAFOS PLANARES

Teorema das 4 cores formulao. Num grafo planar G tem-se que


(G) 4.
O grafo K4 mostra que 4 cores so necessrias, mas sero suficientes? O problema demorou um sculo para ser resolvido. Em
1976, Appel, Haken e Koch, com o auxlio de 1200 horas do computador mais rpido de sua poca, executando mais do que 10 10 operaes
computacionais, provaram o teorema. Embora a teoria envolvida seja
profunda muitos consideram esta a mais feia prova da matemtica.
As tentativas anteriores so, entretanto, dignas de nota. Kempe
utilizou uma tcnica (por isso chamada de cadeias de Kempe) e
apresentou uma demonstrao em 1879. Heawood, 11 anos depois,
percebeu uma falha sutil na demonstrao, que a invalidava. Entretanto, utilizou as cadeias de Kempe para demonstrar um resultado
um pouco mais fraco. Comearemos por um lema.
Lema. Num grafo planar h pelo menos um vrtice com grau menor
ou igual a 5.
Demonstrao. J sabemos que
Se d(v) > 5, v V , ento
6.n

vV (G)

d(v) = 2.m.

d(v) = 2.m.

vV (G)

Mas num grafo planar temos m 3.n 6; isto , 2m 6.n 12.


Ficamos com
6.n 6.n 12,

GrafosModfranci
2009/6/30
page 103
Estilo OBMEP

N SEC. 6.4: O PROBLEMA DAS 4 CORES

103

o que impossvel.

Teorema das 5 cores. Num grafo planar simples G, tem-se


(G) 5.
Demonstrao. Em todo grafo planar existe um vrtice com grau
menor ou igual a 5. Podemos decompor o grafo retirando sempre
um vrtice de grau menor que 5 e recomp-lo colorindo, vrtice a vrtice. Desta forma, podemos sempre supor que estamos colorindo um
vrtice v de grau menor ou igual a 5. Se os vrtices em N (v) esto
coloridas com menos do que 5 cores, basta colorir o vrtice v. Podemos ento supor que o vrtice est cercado por 5 vrtices coloridos
cada um com uma cor do conjunto {a, b, c, d, e}.
Consideremos o subgrafo induzido pelos vrtices coloridos com as
cores a e c. Se a componente que contm o vrtice de N (v) colorido
com a no contiver o vrtice colorido com c, podemos trocar as cores
desta componente: quem est colorido com a fica colorido com c e
vice-versa. Podemos ento colorir o vrtice v com a cor a.
Se a componente que contm o vrtice de N (v), colorido com a,
for o mesmo do vrtice colorido com c, existe um caminho de vrtices
que cerca o vrtice b (veja figura 6.6).
Ento, tomamos a componente do grafo induzido por vrtices coloridos com b e d, que contm o vrtice de N (v) colorido com b. Depois
de trocar as cores b e d nesta componente, podemos colorir o vrtice
v com a cor b.

GrafosModfranci
2009/6/30
page 104
Estilo OBMEP

104

 CAP. 6: GRAFOS PLANARES

Figura 6.6:

Exerccios
1. Construa o grafo com sequncia de graus (4, 4, 3, 3, 3, 3):
(a) Que seja planar.
(b) Que no seja planar.
2. Mostre que um grafo planar com = 5 tem no mnimo 12
vrtices. D um exemplo de grafo com = 5 e n = 12.
3. Um grafo autodual se GD isomorfo a G.
(a) Mostre que se G autodual ento 2.n = m + 2.
(b) Um grafo roda (notao Wn ) o grafo obtido pela adio
de um vrtice de grau n1 a Cn1 (ver figura 6.7). Mostre
que os grafos roda Wn so autoduais.
4. Mostre que um grafo planar G bipartido se e s se G D
euleriano.

GrafosModfranci
2009/6/30
page 105
Estilo OBMEP

105

N SEC. 6.4: O PROBLEMA DAS 4 CORES

W6

Figura 6.7:
5. Mostre que um grafo planar conexo pode ter suas faces coloridas
com 2 cores se, e somente se, G euleriano.
6. Mostre que os grafos abaixo (figura 6.8) so isomorfos mas seus
duais no so. Este fato contraria o texto do captulo?

Figura 6.8:
7. A cintura de um grafo, denotada por g(G) o comprimento do
seu menor ciclo. Mostre que num grafo planar temos:
m

(n 2).g
.
g2

Sugesto: adapte a demonstrao dos dois ltimos teoremas da


Seo 7.1.

GrafosModfranci
2009/6/30
page 106
Estilo OBMEP

106

 CAP. 6: GRAFOS PLANARES

8. Mostre que possvel obter um grafo planar a partir do grafo


de Petersen pela retirada de 2 arestas.
9. Mostre que um grafo no planar tem 5 vrtices de grau no mnimo 4 ou tem 6 vrtices de grau no mnimo 3.
10. (a) (Resolvido) Mostre que o grafo no planar K 3,3 pode ser
desenhado sem cruzamentos num toro. E numa esfera,
pode?
Soluo: A sequncia apresentada na figura 6.9 mostra
como podemos recortar o toro para transform-lo num
retngulo. As setas mostram como podemos passar as
arestas pelos cortes.
(b) Mostre como podemos desenhar K5 num toro. O teorema
das 4 cores vale para o toro?
(c) Mostre como podemos desenhar K7 num toro. Voc consegue dividir o toro em 7 regies de maneira que cada uma
faa fronteira com todas as outras 6?

GrafosModfranci
2009/6/30
page 107
Estilo OBMEP

107

N SEC. 6.4: O PROBLEMA DAS 4 CORES

Figura 6.9:

GrafosModfranci
2009/6/30
page 108
Estilo OBMEP

108

 CAP. 6: GRAFOS PLANARES

11. Um jogo, usando a figura 6.10 tem as seguintes regras: Dois jogadores escolhem alternadamente uma regio para colorir. Duas
regies no podem receber a mesma cor. Quem for obrigado a
usar uma quinta cor ser o perdedor.

2
1
5

Figura 6.10:
12. Exiba uma colorao desses mapas com o menor nmero de
cores possvel.
(a) Quem ser o vencedor o primeiro ou o segundo jogador?
(b) Como modificar o tabuleiro para que a vantagem seja invertida?

GrafosModfranci
2009/6/30
page 109
Estilo OBMEP

N SEC. 6.4: O PROBLEMA DAS 4 CORES

Figura 6.11:

109

GrafosModfranci
2009/6/30
page 110
Estilo OBMEP

110

GrafosModfranci
2009/6/30
page 111
Estilo OBMEP

Referncias Bibliogrficas
[1] BOAVENTURA NETTO, P. O. Grafos: Teoria, Modelos, Algoritmos. 2 ed, Edgard Blcher (1996).
[2] CARVALHO, P. C. P. Contagem. Apostila 2 do Estgio de treinamento dos alunos premiados da OBMEP, 2006.
[3] WILSON, R.
Wesley(1996).

Introduction

to

Graph

Theory.

Addison

[4] BALAKRISHNAN, J.; RANGANATHAN, K. A Textbook of


Graph Theory. Springer-Verlag (1999).
[5] GAREY, M. R.; JOHNSON, D. S. Computers and Intractability:
A Guide to the Theory of NP-Completeness. W.WH. Freeman
(1979).
[6] WEST, D. Introduction to Graph Theory. Prentice Hall (1996).
[7] FOURNIER, J-C. Demonstration simple du thoreme de
Kuratowski et de sa forme duale. Discrete Mathematics, 31
(1980) 329-332.

111

principal1
2009/8/19
page 1
Estilo OBMEP

A Geometria do Globo Terrestre


Srgio Alves

Os Trs Problemas Clssicos da


Matemtica Grega
Joo Pitombeira de Carvalho

A Matemtica dos Cdigos


de Barras
Francisco Csar Polcino Milies

principal1
2009/8/19
page 2
Estilo OBMEP

Texto j revisado pela nova ortografia.

principal1
2009/8/19
page i
Estilo OBMEP

Apresentao
Os textos apresentados a seguir foram produzidos para a II
Bienal da Sociedade Brasileira de Matemtica, realizada em 2004,
em Salvador.
A utilizao desses trabalhos pelos bolsistas de Iniciao Cientfica da OBMEP motivada pelo fato de tratarem de questes muito
interessantes e, a nosso ver, motivadoras.
O primeiro, de Srgio Alves, aborda o problema do estabelecimento de coordenadas no globo terrestre. O domnio de tais coordenadas e o uso de instrumentos para medi-las, propiciou grandes
avanos para a humanidade, desde os tempos das grandes navegaes
(e o consequente aumento do comrcio mundial) at o uso do GPS
para a construo de mapas e localizao.
O segundo texto, de autoria de Joo Bosco Pitombeira, tem forte
sabor histrico pois trata de trs problemas clssicos da geometria
euclidiana plana: a duplicao do cubo, a quadratura do crculo e
a trisseco do ngulo. So perguntas naturais cuja resposta custou
muito esforo e fazem parte da bagagem cultural da matemtica.
Quem viveu a poca das filas nos supermercados, onde o preo de
cada produto tinha que ser registrado manualmente no caixa, sabe
muito bem o avano que representa o uso do cdigo de barras. Uma
ideia simples e genial, de grande utilidade. Esse o tema do teri

principal1
2009/8/19
page ii
Estilo OBMEP
ii
ceiro texto, em que Francisco Csar Polcino Miles expe os aspectos
matemticos da codificao. Vemos aqui um belo exemplo de como
estruturas abstratas ajudam a resolver problemas concretos.
Como ser possvel notar, os textos no esgotam os respectivos
assuntos e possuem referncias bibliogrficas teis para aqueles mais
curiosos que desejarem aprofundar seus conhecimentos sobre os assuntos. A leitura de um trabalho matemtico deve ser pausada e
refletida, e aqui no ser diferente. O leitor deve se envolver nas
questes e ideias discutidas pelo autor e muitas vezes dialogar com
ele, quem sabe buscando outras solues.
Esperamos que os bolsistas apreciem tanto a beleza dos problemas e de suas solues quanto a elegncia com que os temas foram
tratados.
O Comit Acadmico da OBMEP agradece aos autores por terem
permitido o uso dos seus trabalhos no Programa de Iniciao Cientfica da OBMEP.

Comit Acadmico

Direo Acadmica

principal1
2009/8/19
page iii
Estilo OBMEP

Sumrio
A Geometria do Globo Terrestre
Os Trs Problemas Clssicos da Matemtica Grega
A Matemtica dos Cdigos de Barras

iii

1
81
133

principal1
2009/8/19
page iv
Estilo OBMEP

principal1
2009/8/19
page 1
Estilo OBMEP

A Geometria do Globo Terrestre


Srgio Alves

principal1
2009/8/19
page 2
Estilo OBMEP

principal1
2009/8/19
page 3
Estilo OBMEP

Prefcio
Num encontro de professores que atuam no ensino bsico fomos
questionados sobre a utilizao de aplicaes da Matemtica em sala
de aula. Nossas primeiras consideraes foram as esperadas: sempre
bom poder mostrar aos alunos que a Matemtica faz parte de nosso
cotidiano, ilustrando como ideias e conceitos abstratos levam ao desenvolvimento de conhecimentos e tecnologias que visam o bem-estar
da nossa sociedade. Com isso, certamente nossas aulas de Matemtica
ficam mais interessantes e motivadoras para o aluno.
Porm, existe um preo a se pagar para conseguirmos esse intento
e notamos certa surpresa em nossos interlocutores quando passamos
a explicar melhor esse lado da questo.
Primeiramente, a chamada contextualizao da Matemtica tem
seu lugar e hora para acontecer. Ela deve ser feita de modo que o
aluno entenda seu significado e sua necessidade e no se criando situaes, por vezes extremamente artificiais, somente pelo gosto de
poder dar alguma aplicao prtica de um determinado tema. Nem
todo tpico da Matemtica passvel de contextualizao!
Um segundo ponto por ns levantado que a apresentao de
aplicaes significativas da Matemtica invariavelmente exige do professor certo conhecimento de alguma outra rea como, por exemplo,
Fsica, Biologia ou Geografia. Esse professor deve estar ciente que tal

principal1
2009/8/19
page 4
Estilo OBMEP
4
exigncia vai requerer dele um maior empenho e dedicao.
Por fim destacamos o fato que, embora a Matemtica tenha a
marca da cincia exata por excelncia, nas suas aplicaes frequentemente temos que lidar com a questo das aproximaes fazendo uso,
ou no, de calculadoras ou programas de computador. Raramente na
soluo de um problema contextualizado comparecem nmeros como

16 ou ainda cos 60o !


Motivado por todas essas consideraes escrevemos o texto que
aqui se apresenta. Escolhemos como tema a geometria da esfera que
encontra na Geografia uma natural contextualizao por meio do estudo do globo terrestre e dos vrios assuntos a ele relacionados. Incorporamos no texto todos os conceitos geogrficos necessrios, o que
no exime o leitor destas notas de procurar aprofund-los com suas
prprias fontes.
Procuramos explicar, com a maior clareza possvel, as ideias
matemticas envolvidas no funcionamento do sistema de posicionamento global (GPS), um dos mais modernos sistemas de localizao
por satlites, e fazemos ainda uma discusso atualizada do clculo
da distncia entre dois pontos da superfcie terrestre. Esse clculo,
no passado, era justificado com o uso da trigonometria esfrica, tema
este que j no frequenta nossos livros didticos.
Enfim, produzimos este texto na esperana que ele fornea aos
professores uma possvel resposta quela pergunta que frequentemente
lhes feita: Para que serve isso que o senhor est ensinando?

Srgio Alves

principal1
2009/8/19
page 5
Estilo OBME

Sumrio
Introduo

1 A Superfcie Esfrica e seus Elementos

1.1

A Interseo entre um Plano e uma Superfcie Esfrica

2 Qual a Forma da Terra?

10
19

2.1

As Coordenadas Geogrficas . . . . . . . . . . . . . . .

25

2.2

Os Movimentos da Terra . . . . . . . . . . . . . . . . .

31

2.3

Os Fusos Horrios . . . . . . . . . . . . . . . . . . . .

38

3 O ngulo de Elevao do Sol e da Energia Solar

41

3.1

O ngulo de Elevao do Sol nos Equincios . . . . .

45

3.2

O ngulo de Elevao do Sol nos Solstcios . . . . . .

47

4 A Superfcie Esfrica em Coordenadas Cartesianas

54

5 A Matemtica do GPS

64

6 A Distncia numa Superfcie Esfrica

71

principal1
2009/8/19
page 6
Estilo OBMEP
6
Referncias Bibliogrficas

79

principal1
2009/8/19
page 7
Estilo OBMEP

Introduo
Neste curso pretendemos desenvolver o estudo da esfera e seus
elementos explorando sua associao com o globo terrestre. Conceitos geogrficos como paralelos, meridianos, latitudes, longitudes e fusos horrios esto baseados em importantes ideias geomtricas que,
quando trabalhadas neste contexto, conduzem o aluno a uma melhor
compreenso e aprendizagem do tema.
O estudo dos movimentos da Terra nos permite entender, alm
das quatro estaes do ano com seus equincios e solstcios, porque
o Trpico de Capricrnio ou o Crculo Polar rtico so paralelos
notveis. Veremos ainda que as relaes entre longitude e fusos horrios bem como entre latitude e o ngulo de elevao do Sol nos levam
a problemas geomtricos relevantes.
O estudo da posio relativa de duas ou mais esferas e as relaes
entre as coordenadas geogrficas e as coordenadas cartesianas constituem a fundamentao matemtica necessria para o entendimento
de alguns modernos sistemas de navegao por satlites, em especial
do GPS.

principal1
2009/8/19
page 8
Estilo OBMEP
8
A utilizao do globo terrestre, com suas consequentes questes
envolvendo, por exemplo, clculo de distncias e ngulos sobre a esfera, ou ainda, a confeco de mapas por meio de diversas projees,
abre caminho para um interessante trabalho interdisciplinar entre a
Matemtica e a Geografia.

principal1
2009/8/19
page 9
Estilo OBMEP

Captulo 1

A Superfcie Esfrica e seus


Elementos
Seja O um ponto e r um nmero real positivo. A superfcie
esfrica de centro O e raio r o conjunto de todos os pontos P
do espao cuja distncia a O igual a r.
Q
r

Os pontos do espao cuja distncia a O menor que r so interiores superfcie esfrica e aqueles cuja distncia a O maior que
r so exteriores a ela.
A reunio da superfcie esfrica de centro O e raio r com seus
pontos interiores chamada a esfera de centro O e raio r .
O segmento que une o centro a um ponto qualquer da super9

principal1
2009/8/19
page 10
Estilo OBMEP
10

CAP. 1: A SUPERFCIE ESFRICA E SEUS ELEMENTOS

fcie esfrica denominado um raio da superfcie esfrica enquanto


que o segmento que une dois pontos distintos da superfcie esfrica
chamado uma corda da superfcie esfrica.
Uma corda que contm o centro chamada um dimetro da
superfcie esfrica.
Evidentemente o comprimento de qualquer dimetro o nmero
2r que chamado o dimetro.
Observamos que a palavra raio usada com dois sentidos: pode
ser o nmero r ou um segmento OP . Isto, porm, no causa confuso uma vez que sempre claro qual o significado utilizado. Um
comentrio anlogo vale para a palavra dimetro.
Neste texto a notao P Q representa o segmento de extremos P
e Q enquanto que PQ indica o seu comprimento.
Na figura anterior r o raio da superfcie esfrica, OP , OQ e OT
so raios, P Q e P T so cordas, QT um dimetro e 2r = QT o
dimetro da superfcie esfrica.

1.1

A Interseo entre um Plano e uma Superfcie Esfrica

A relao entre superfcies esfricas e planos no espao similar


com a relao entre circunferncias e retas no plano.
Um plano E tangente a uma superfcie esfrica S se ES contm
exatamente um ponto. Esse ponto chamado ponto de tangncia.
Dizemos que o plano e a superfcie esfrica se tangenciam nesse ponto.
Se E S contm mais do que um ponto, ento o plano secante
superfcie esfrica. Podemos caracterizar o plano tangente conforme
o seguinte resultado.

principal1
2009/8/19
page 11
Estilo OBMEP

N SEC. 1.1: A INTERSEO ENTRE UM PLANO E UMA SUPERFCIE ESFRICA

11

Teorema 1.1. Um plano perpendicular a um raio na sua extremidade


comum com a superfcie esfrica tangente mesma. Reciprocamente,
todo plano tangente a uma superfcie esfrica perpendicular ao raio
que contm o ponto de tangncia.
Demonstrao. Sendo E um plano perpendicular ao raio OT em T,
vamos mostrar que nenhum outro ponto de E est na superfcie esfrica.
S

T
P

Seja P um ponto qualquer de E, P distinto de T. Como E


perpendicular a OT temos que 4OP T um tringulo retngulo com
hipotenusa OP e catetos OT e P T . Logo OP > OT = r e, portanto,
P no est na superfcie esfrica.
Reciprocamente seja E um plano tangente superfcie esfrica no
ponto T. Suponha, por absurdo, que E no seja perpendicular ao raio
OT . Vamos mostrar que essa hiptese nos leva a uma contradio.

OO
T

R
F
E

Sendo F o p da perpendicular ao plano E, traada a partir de O,


temos F 6= T , pois E no perpendicular a OT . Seja R o ponto da

principal1
2009/8/19
page 12
Estilo OBMEP
12

CAP. 1: A SUPERFCIE ESFRICA E SEUS ELEMENTOS

reta F T tal que T F R e F R = F T . Ento 4OF R


= 4OF T
(pelo critrio LAL de congruncia de tringulos) de modo que
OR = OT = r e, portanto, R est na superfcie esfrica. Logo E
intersecta a superfcie esfrica em um ponto R distinto de T. Isso
impossvel, pois E um plano tangente.

Vamos agora investigar a figura obtida quando um plano secante


corta uma superfcie esfrica. Se o plano passa pelo centro a resposta
imediata.
Teorema 1.2. A interseco de uma superfcie esfrica com um plano
passando pelo seu centro uma circunferncia de mesmo centro e
mesmo raio.
S
E

Demonstrao. Dada uma superfcie esfrica S com centro O e raio


r e um plano E que passa por O, a interseco E S o conjunto de
todos os pontos de E cuja distncia a O igual a r. Essa exatamente
a definio de uma circunferncia de centro O e raio r.

O prximo resultado nos d a situao mais geral.

principal1
2009/8/19
page 13
Estilo OBMEP

N SEC. 1.1: A INTERSEO ENTRE UM PLANO E UMA SUPERFCIE ESFRICA

13

Teorema 1.3. Se um plano contm um ponto do interior de uma


superfcie esfrica, ento a interseco do plano com a superfcie esfrica uma circunferncia. O centro dessa circunferncia o p da
perpendicular ao plano traada a partir do centro da superfcie esfrica.

O O
X

F Y
E

Demonstrao. Seja E um plano que no passa pelo centro O da


superfcie esfrica S e que contm um ponto Y do seu interior. Sendo
F o p da perpendicular a E traada a partir de O, vamos mostrar
que a interseco E S uma circunferncia de centro F.
Sabemos que OY < r, pois Y est no interior de S. No tringulo
retngulo 4OF Y temos que OY hipotenusa, enquanto que OF
cateto. Logo OF < OY < r, ou seja, F tambm est no interior de S.
Seja X um ponto qualquer na interseco E S. Ento 4OF X
tem um ngulo reto em F e, pelo teorema de Pitgoras,
OF 2 + F X 2 = OX 2 = r2

r2 OF 2 (note que r2 OF 2 > 0).

Logo X est na circunferncia de centro F e raio r2 OF 2 .


Provamos assim que a interseco E S est contida na circunferncia

de centro F e raio r2 OF 2 .
e, portanto, F X =

principal1
2009/8/19
page 14
Estilo OBMEP
14

CAP. 1: A SUPERFCIE ESFRICA E SEUS ELEMENTOS

Isso ainda no significa que a interseco a circunferncia. Para


completar a demonstrao, precisamos mostrar que todo ponto da
circunferncia pertence interseco.
Seja X um ponto qualquer da circunferncia, em E, com centro F

e raio r2 OF 2 . Novamente pelo teorema de Pitgoras,


OX 2

=
=
=

OF 2 + F X 2
OF 2 + r2 OF 2
r2 .

Portanto OX = r e X pertence superfcie esfrica S.

A interseco da superfcie esfrica com um plano passando pelo


seu centro chamada uma circunferncia mxima da superfcie
esfrica.
H uma forte razo para esse nome: as circunferncias mximas
so as circunferncias de maior raio contidas na superfcie esfrica.
Veremos adiante que o Equador uma circunferncia mxima, mas
os outros paralelos no globo terrestre no o so. Eles so menores que
o Equador, tornando-se muito pequenos perto dos Polos Norte e Sul.
POLO NORTE

30

30

POLO SUL

principal1
2009/8/19
page 15
Estilo OBMEP

N SEC. 1.1: A INTERSEO ENTRE UM PLANO E UMA SUPERFCIE ESFRICA

15

Atividades

1) A superfcie esfrica S tangente ao plano E em A. O ponto O


o centro de S e B, C, D esto em E. Que relao existe entre OA
e AB , AC e AD? Explique.
S

O
E
B
D

2) A perpendicular do centro de uma superfcie esfrica a uma corda


divide-a ao meio. Utilize este resultado na resoluo do seguinte
problema. Numa superfcie esfrica de raio 15, a distncia de uma
corda ao centro igual a 9. Qual o comprimento da corda? Apresente ainda uma prova do resultado acima.
3) Dados um plano E e uma superfcie esfrica S de centro O e raio
r, podemos caracterizar a posio relativa entre E e S por meio da
distncia d(O,E) de O ao plano E. Faa isso completando a tabela
abaixo.
d(O,E)

Posio relativa
E secante a S
E tangente a S
E disjunto a S

4) Sejam A e B dois pontos de uma superfcie esfrica S que no so


extremos de um dimetro de S. Mostre que existe uma e somente

principal1
2009/8/19
page 16
Estilo OBMEP
16

CAP. 1: A SUPERFCIE ESFRICA E SEUS ELEMENTOS

uma circunferncia mxima de S passando por A e B. (Sugesto:


Sendo O o centro de S, considere o plano E determinado pelos
pontos no-colineares A, B e O.)
5) Explique porque duas circunferncias mximas quaisquer de uma
superfcie esfrica, se cortam nas extremidades de um dimetro da
superfcie esfrica.

6) Na figura abaixo um plano E intersecta a superfcie esfrica S de


centro O. Os pontos A, B, C e M esto em E sendo que A e B
tambm esto em S. Sabe-se que OM E, AM M B, C
ponto mdio de AB, AM = OM e AB = 5.
B
C

A
O

Calcule o raio da superfcie esfrica S, a medida angular m(AOB)


e a medida OC.
7) Duas circunferncias mximas so ditas perpendiculares se estiverem em planos perpendiculares. Mostre que para cada duas
circunferncias mximas existe uma terceira circunferncia mxima perpendicular a ambas. Se duas circunferncias mximas

principal1
2009/8/19
page 17
Estilo OBMEP

N SEC. 1.1: A INTERSEO ENTRE UM PLANO E UMA SUPERFCIE ESFRICA

17

no globo terrestre passam pelos polos, que circunferncia mxima


perpendicular a ambas?
8) Na geometria esfrica as linhas retas so representadas por circunferncias mximas. Se puder, encontre cada uma das seguintes
figuras em tal geometria.
(a) Um tringulo equiltero.
(b) Um tringulo com dois ngulos retos.
(c) Um tringulo com trs ngulos retos.
(d) Um tringulo cujas medidas de seus ngulos somem 500.
(e) Um retngulo, isto , um quadriltero com quatro ngulos
retos.
9) Sejam S e S 0 duas superfcies esfricas de centros distintos O e
O0 , respectivamente. Sendo r e r0 , r r, seus respectivos raios,
verifique que
S S 0 vazia
S S 0 um ponto
S S 0 uma circunferncia

OO0 > r + r0 ou OO0 < r r0


OO0 = r + r0 ou OO0 = r r0
OO0 < r + r0 ou OO0 < r r0 .

Faa desenhos ilustrando todas estas possibilidades.


(Sugesto: Observe inicialmente que P S S P S e
P O2 P O02 = r2 r02 . A seguir, mostre que o conjunto dos
pontos P do espao que satisfazem P O2 P O02 = r2 r02

um plano E perpendicular reta OO0 num ponto H definido por


r2 r02
com O M H, onde M o ponto mdio do segMH =
2OO0
mento OO0 . Conclua a prova utilizando a atividade 3 para obter
as relaes desejadas.)
10) Quais as possibilidades para a quantidade de pontos da interseco
de trs superfcies esfricas? Faa desenhos ilustrando suas respostas.

principal1
2009/8/19
page 18
Estilo OBMEP
18

CAP. 1: A SUPERFCIE ESFRICA E SEUS ELEMENTOS

Respostas
2) 24.
6) 5,

60o

5 3
e
.
2

10) 0, 1, 2 ou infinitos pontos.

principal1
2009/8/19
page 19
Estilo OBMEP

Captulo 2

Qual a Forma da Terra?


A Terra redonda, mas no uma esfera perfeita, uma vez que
achatada nos polos. Na verdade, a Terra aproximadamente um
elipside.
A figura abaixo mostra uma seco da superfcie terrestre atravs
de um plano que contm a reta que liga os Polos Norte e Sul. Esta
seco aproxima-se de uma elipse cujo semieixo maior a a metade
do dimetro do Equador e o semieixo menor b a metade da distncia
entre os polos.

N
b
a

19

principal1
2009/8/19
page 20
Estilo OBMEP
20

CAP. 2: QUAL A FORMA DA TERRA?

A superfcie terrestre a superfcie de revoluo gerada por essa


elipse quando esta gira em torno da reta que passa pelos polos.
ab
A razo =
chama-se achatamento da Terra.
b
A tabela seguinte mostra valores dos semieixos a e b e do correspondente achatamento da Terra, obtidos por alguns pesquisadores.
a (metros)

b (metros)

Bessel (1841)

6 377 397

6 356 078

0,0033541

Clarke (1880)

6 378 249

6 356 515

0,0034191

Helmert (1906)

6 378 200

6 356 940

0,0033443

Hayford (1924)

6 378 388

6 356 911

0,0033785

Krassovski (1940)

6 378 245

6 356 863

0,0033636

O valor extremamente pequeno de nos permite, para efeitos


didticos, desprezar esse achatamento e considerar a Terra como se
fosse uma esfera.
A superfcie da Terra passa a ser considerada daqui por diante
como um globo: o globo terrestre. O globo terrestre ainda o nome
do instrumento didtico que representa a Terra como uma esfera.
Iniciamos estabelecendo a nomenclatura apropriada. A figura a
seguir ilustra um globo terrestre.
O ponto N representa o Polo Norte e o ponto S o Polo Sul. A
reta determinada por N e S chamada o eixo polar. Ela a reta em
torno da qual a Terra efetua seu movimento de rotao.
O plano que passa pelo centro da superfcie esfrica e perpendicular ao eixo polar chama-se o plano do Equador.
O Equador a interseco do plano do Equador com a superfcie
esfrica. O Equador , portanto, uma circunferncia mxima.
Um plano que passa pelo centro da superfcie esfrica divide-a em

principal1
2009/8/19
page 21
Estilo OBMEP
21
N
Meridiano

Paralelo

Equador
S

duas partes chamadas hemisfrios.


O plano do Equador divide a superfcie terrestre em dois hemisfrios: o Hemisfrio Norte (que contm o Polo Norte) e o Hemisfrio Sul (que contm o Polo Sul).
Os paralelos so as seces da superfcie terrestre atravs de
planos paralelos (ou coincidentes) ao plano do Equador. So ento
circunferncias. Os paralelos notveis so:
o Equador
o Trpico de Cncer
o Trpico de Capricrnio
o Crculo Polar rtico
o Crculo Polar Antrtico
Os meridianos so semicircunferncias que ligam os Polos Norte
e Sul por meio de arcos mximos, isto , arcos contidos em circunferncias mximas que passam pelos polos. Convm ressaltar que os
meridianos, ao contrrio dos paralelos, no so circunferncias. Alm
disso, eles esto contidos em planos perpendiculares ao plano do

principal1
2009/8/19
page 22
Estilo OBMEP
22

CAP. 2: QUAL A FORMA DA TERRA?

Equador. O meridiano mais notvel o de Greenwich, nome de uma


localidade prxima a Londres, onde est instalado um observatrio
astronmico.
Observao. Os livros de Geografia utilizam com muita frequncia a
palavra crculo para designar a circunferncia. Em particular, descrevem o Equador como um crculo mximo e utilizam nomes como
crculo polar. Isto , na verdade, um abuso de linguagem consagrado
pelo uso sistemtico ao longo do tempo. A nomenclatura utilizada por
profissionais no matemticos, no caso os gegrafos, no precisa coincidir necessariamente com a usada pelos matemticos.

Atividades

1) Qual o comprimento do Equador, supondo que o raio da Terra


mea 6 400 km? Considere = 3,14159.
2) Como se mede o raio da Terra? Uma das maneiras escalar o
topo de uma montanha cuja altitude acima do mar seja conhecida
e medir o ngulo entre a vertical e a linha do horizonte. A altura
do monte Shasta na Califrnia 4,3 km. Do seu topo, o horizonte
sobre o Oceano Pacfico faz um ngulo de 87 530 com a vertical.
Utilize estes dados para estimar o raio da Terra em quilmetros.
Considere sen 87 530 = 0,99932.

principal1
2009/8/19
page 23
Estilo OBMEP
23

Linha do Horizonte
O

3) O clculo do raio da Terra mais clebre da Antiguidade foi


realizado pelo grego Eratstenes (276-196 a.C.). Consultando as
observaes astronmicas acumuladas durante sculos na biblioteca de Alexandria, Eratstenes soube que em Siena, 5 000 estdios (medida grega de comprimento) ao sul de Alexandria e situada
aproximadamente no mesmo meridiano, o Sol se refletia no fundo
de um poo ao meio-dia de um determinado dia de cada ano. Ao
meio-dia deste tal dia, Eratstenes mediu o ngulo que o raio do
Sol fazia com a vertical de Alexandria, achando aproximadamente
7 120 .
Raios
Solares

712?
Alexandria

Siena

principal1
2009/8/19
page 24
Estilo OBMEP
24

CAP. 2: QUAL A FORMA DA TERRA?

Admitindo que os raios solares cheguem ao nosso planeta praticamente paralelos, mostre como obter para o raio da Terra o valor
aproximado de 250 000/2 estdios. Supondo que a medida de
um estdio utilizada por Eratstenes era de aproximadamente 185
metros, calcule o valor acima em quilmetros.
4) Na ponte de um navio em alto-mar, o capito pediu a um
jovem oficial que estava ao seu lado que determinasse a distncia ao horizonte. O oficial pegou papel e lpis e, em poucos
instantes, deu uma resposta. No papel ele havia escrito a fr8
mula d =
5h. Admitindo-se que o raio da Terra mea
5
6 400 km, mostre que essa frmula uma boa aproximao da
distncia d, em quilmetros, ao horizonte onde h a altura, em
metros, do observador acima da gua. Se a ponte do navio est a
30 metros acima da gua, qual a distncia ao horizonte?

5) Em um dia claro, at de que distncia se poderia enxergar um edifcio de 150 metros de altura, supondo-se que no haja obstrues
no meio? Utilize a frmula do exerccio anterior.

Respostas
1) Cerca de 40 212 km.
2) Cerca de 6 319 km.
3) Cerca de 7 360 km.

principal1
2009/8/19
page 25
Estilo OBMEP

N SEC. 2.1: AS COORDENADAS GEOGRFICAS

25

4) Cerca de 19 km.
5) Cerca de 43 km.

2.1

As Coordenadas Geogrficas

O globo terrestre serve para localizar um determinado ponto ou


regio da Terra. O aluno deve ter acesso a um para efetuar esta
localizao.
H razes prticas para se ultrapassar o estgio da localizao
ingnua (isto , apontar o dedo para o lugar e dizer: aqui!) sendo
necessrio criar um sistema de coordenadas para dar a localizao
precisa de um ponto no globo.
Para isto utilizamos as chamadas coordenadas geogrficas: latitude e longitude.
A latitude de um ponto P a medida do arco de meridiano que
passa por P situado entre o paralelo que contm P e o Equador. A
latitude expressa em graus, minutos e segundos e se mede de 0 a
90 N (norte) ou de 0 a 90 S (sul).
A longitude de um ponto P a medida do arco de paralelo que
passa por P situado entre o meridiano que contm P e o meridiano
de Greenwich. A longitude expressa em graus, minutos e segundos
e se mede de 0 a 180 E (leste) ou de 0 a 180 W (oeste).
Na figura a seguir temos que = m(EOP ) a latitude de P
enquanto que = m(GM P ) a longitude de P. Pontos sobre um
mesmo paralelo possuem latitudes iguais e pontos sobre um mesmo
meridiano possuem longitudes iguais.

principal1
2009/8/19
page 26
Estilo OBMEP
26

CAP. 2: QUAL A FORMA DA TERRA?

N
M
G

?: latitude de P
?: longitude de P

Equador
E

Greenwich

Atividades

1) Observando um globo terrestre ou um mapa apropriado estabelea


as coordenadas geogrficas de cada uma das cidades abaixo.
Cidade
So Paulo
Macei
Belo Horizonte
Nova Orleans
Chicago
Roma
Nova York
Buenos Aires
Londres
Moscou
Cairo
Tquio
Quito
Los Angeles

Latitude

Longitude

principal1
2009/8/19
page 27
Estilo OBMEP
27

N SEC. 2.1: AS COORDENADAS GEOGRFICAS

2) Qual a relao entre o raio da superfcie terrestre, o raio de um


paralelo e a sua respectiva latitude?
3) Verifique que a longitude de um ponto P a medida do arco do
Equador situado entre o meridiano que passa P e o meridiano de
Greenwich.
4) Um dos primeiros fatos que um estudante de astronomia aprende
que a latitude de um ponto da Terra localizado no Hemisfrio Norte
igual medida do ngulo que Polaris (a Estrela do Norte) forma
com a linha do horizonte, quando observada daquele ponto. Geometricamente, esta afirmao pode ser interpretada da seguinte

maneira. A reta N S o eixo polar da Terra, a circunferncia C


contm um meridiano, E est no Equador, P o observador, a reta

P H a linha do horizonte e m(XP H) a elevao de Polaris

onde P X paralela a N S.
X
N

H
P

C
S

O fato acima equivale a dizer que m(EOP ) = m(XP H).


Mostre porque isto verdadeiro.
5) A acelerao da gravidade, frequentemente denotada por g, considerada constante para movimentos prximos da superfcie ter-

principal1
2009/8/19
page 28
Estilo OBMEP
28

CAP. 2: QUAL A FORMA DA TERRA?

restre. Na verdade, porm, g no constante, variando ligeiramente com a latitude . Uma boa aproximao para g, ao nvel
do mar, dada pela expresso
g = 9,78049(1 + 0,005288 sen 2 0,000006 sen 2 2) m/s2
(a) Utilizando uma calculadora, determine g para a latitude do
lugar onde voc mora.
(b) Expresse g apenas em funo de sen , isto , elimine o ngulo
duplo.
(c) Em que lugar da Terra o valor de g mximo? E mnimo?
6) Supondo que o raio da Terra mea 6 400 km, qual o comprimento
de um grau de longitude em uma latitude arbitrria ? Em particular, qual o comprimento numa latitude de 30 N (aproximadamente a latitude de Nova Orleans), 10 S (aproximadamente a
latitude de Macei) e 20 S (aproximadamente a latitude de Belo
Horizonte)?
7) A que latitude um grau de longitude tem comprimento igual a 48
km? A que latitude um paralelo mede 16 000 km?
8) Qual a altura mnima para que um satlite consiga fotografar
o Brasil inteiro? Admita que Oiapoque e Chu estejam sobre
o mesmo meridiano, mas em paralelos diferentes: a latitude de
Oiapoque 6 N e a de Chu 34 S. Suponha que o raio da Terra
mea 6 400 km.
9) Um astronauta encontra-se numa nave espacial que gira numa rbita em torno da Terra.No momento em que a nave est 160 km
acima da superfcie da Terra, que frao da superfcie da Terra
visvel para o astronauta? (Esta frao chamada calota esfrica e sua rea dada por 2rh, onde r = 6 400 km o raio

principal1
2009/8/19
page 29
Estilo OBMEP

N SEC. 2.1: AS COORDENADAS GEOGRFICAS

29

da Terra e h a altura da calota). Qual seria a frao visvel se


a nave estivesse a 20 200 km acima da superfcie da Terra? Neste
caso, quanto mede o ngulo de visualizao sobre a Terra?
h

10) Zona a regio da superfcie terrestre compreendida entre dois


paralelos. Utilize a expresso da rea da calota esfrica para deduzir que a rea da zona delimitada por um paralelo de latitude e
o Equador dada por 2rd = 2r2 sen , onde r o raio da Terra
e d a distncia entre o plano que contm o paralelo e o plano
do Equador. Conclua da o surpreendente fato: cortando-se uma
superfcie esfrica em fatias de igual espessura, as reas das zonas
so iguais, sendo indiferente que o corte tenha sido feito perto do
Equador ou perto do polo.
11) Um ponto P do globo terrestre tem latitude e longitude . Quais
so as coordenadas geogrficas do ponto Q diametralmente oposto
a P? Discuta cada uma das possibilidades N/S para e E/W para
.

principal1
2009/8/19
page 30
Estilo OBMEP
30

CAP. 2: QUAL A FORMA DA TERRA?

Respostas
2) rparalelo = rterra cos .
5) (c) mximo nos polos e mnimo no Equador.
6) Aproximadamente 111,7 cos km; 96,7 km; 110 km; 104,9 km.
7) 65 (N ou S) e 67 (N ou S), ambas aproximadamente.
8) Aproximadamente 410 km.
9) Na figura abaixo temos x = rsen e, portanto, a rea A da
calota esfrica dada por
A = 2rh = 2r(r x) = 2r2 (1 sen ).

r ?
?
x

Sendo F a frao visvel temos


F =

2r2 (1 sen )
1 sen
.
=
2
4r
2

r
1 d
Por outro lado, como sen =
segue que F =
.
r+d
2r+d
Para r = 6 400 e d = 160 obtemos F = 0, 012 de modo que
a frao visvel cerca de 1,2%. Para d = 20 200 obtemos
F = 0, 379 e a frao visvel cerca de 37,9%. Neste caso, a
medida 2 do ngulo de visualizao aproximadamente 28 .

principal1
2009/8/19
page 31
Estilo OBMEP
31

N SEC. 2.2: OS MOVIMENTOS DA TERRA

2.2

Os Movimentos da Terra

A Terra no est parada. Ela tem dois movimentos principais:


o de rotao, em torno de seu eixo polar, e o de translao, em
torno do Sol. As consequncias destes dois movimentos so muito
importantes em nosso dia-a-dia.
Para dar uma volta completa em torno de seu eixo polar, a Terra
leva 24 horas ou um dia. Durante esta rotao, uma metade da
Terra est voltada para o Sol, do qual recebe luz e calor: nela ento
dia. A outra metade no recebe os raios solares: nela ento noite.
Assim, a sucesso dos dias e das noites uma consequncia da rotao
da Terra.
Sob a hiptese de que os raios solares so paralelos temos que,
em cada instante, a curva sobre o globo terrestre que separa o dia da
noite uma circunferncia mxima.

N
A

NOITE

DIA

Raios
Solares

S
A

Todos os dias vemos o Sol aparecer no nascente, tambm


chamado oriente ou leste, subir no cu e se deitar no poente, tambm chamado ocidente ou oeste. Temos a impresso de que o Sol
que caminha no cu. Na realidade, no isso que acontece. A Terra
que gira em torno de seu eixo polar da esquerda para direita, isto
, do oeste para leste, causando a impresso do movimento do Sol.
Chamamos a isto movimento aparente do Sol.

principal1
2009/8/19
page 32
Estilo OBMEP
32

CAP. 2: QUAL A FORMA DA TERRA?

O momento em que o Sol, em sua trajetria aparente, est mais


alto no cu o meio-dia solar. Neste momento a sombra projetada
de uma haste vertical tem o menor comprimento possvel. O meio-dia
solar para uma particular localidade ocorre quando o meridiano que
a contm corta a reta imaginria determinada pelos centros da Terra
e do Sol.
Alm de girar em torno de seu eixo, a Terra gira tambm em
torno do Sol. Nesta translao em torno do Sol, a Terra descreve uma
curva elptica denominada rbita. A trajetria da rbita da Terra
chamada eclptica e o plano que a contm referido como o plano
da eclptica. Para dar uma volta completa em torno do Sol, a Terra
leva 365 dias e 6 horas, isto , pouco mais que um ano. Para corrigir
esta diferena, foi necessrio aumentar um dia no ano, a cada perodo
de quatro anos. Nesses anos, chamados bissextos, o ms de fevereiro
tem 29 dias.
Um fato extremamente importante deve ser observado: em sua
translao em torno do Sol, o eixo da Terra no perpendicular ao
plano da eclptica, mas inclinado, sempre na mesma direo, formando um ngulo de 23 270 com esta perpendicular.
2327
N

Equador

Plano da rbita
terrestre
2327

principal1
2009/8/19
page 33
Estilo OBMEP

N SEC. 2.2: OS MOVIMENTOS DA TERRA

33

Se o eixo da Terra fosse perpendicular ao plano da rbita, durante


qualquer dia do ano, o Hemisfrio Norte receberia a mesma quantidade de luz e calor que o Hemisfrio Sul. Mas, como o eixo inclinado,
os dias e as noites no tm a mesma durao em todos os lugares da
Terra. Vejamos por que.
Observe a figura abaixo atentamente. Ela mostra as posies da
Terra em relao ao Sol ao longo do ano. Como voc pode ver, a
posio da Terra no a mesma em todos os meses do ano.

Apenas em duas ocasies a circunferncia mxima que separa a


zona iluminada daquela que est no escuro passa rigorosamente pelos
polos. Aproximadamente em 21 de maro e em 23 de setembro a
noite e o dia duram, em todos os lugares da Terra, cerca de 12 horas
cada um: so os equincios (noites iguais).
Nestas datas a reta imaginria determinada pelos centros da Terra
e do Sol corta o globo terrestre num ponto do paralelo de latitude 0 ,
isto , os raios solares incidem perpendicularmente sobre a
linha do Equador, que recebe ento mais calor. Para o norte ou
para o sul do Equador, o calor vai diminuindo em direo aos polos e

principal1
2009/8/19
page 34
Estilo OBMEP
34

CAP. 2: QUAL A FORMA DA TERRA?

os dois hemisfrios so igualmente iluminados e aquecidos.


No dia 21 de maro comea para ns (no Hemisfrio Sul) o outono, enquanto que para os habitantes do Hemisfrio Norte a primavera que tem incio.
O dia 23 de setembro marca o comeo da primavera no Hemisfrio
Sul e o do outono no Hemisfrio Norte.

N
Raios
Solares

Equador
S

21 de maro e 23 de setembro

Vejamos agora o que acontece de setembro a dezembro no Hemisfrio Sul, de acordo com o movimento que a Terra faz em torno do
Sol.
Durante esta poca, os dias vo ficando mais longos e as noites
mais curtas, at que, por volta de 21 de dezembro, temos o dia
mais longo e a noite mais curta: o que chamamos de solstcio de
vero. A Terra inclina o Polo Sul na direo mais prxima do Sol.
Nesse dia, comea no Hemisfrio Sul o vero.
O que se passa no Hemisfrio Norte nessa poca? A a situao
se inverte: os dias vo ficando mais curtos e as noites mais longas. A
21 de dezembro, os habitantes daquele hemisfrio tm a noite mais
longa do ano e o dia mais curto: o solstcio de inverno. Para eles,
comea o inverno.

principal1
2009/8/19
page 35
Estilo OBMEP
35

N SEC. 2.2: OS MOVIMENTOS DA TERRA

Voc j entendeu por que isso acontece? Observe a figura abaixo.


Qual hemisfrio est mais exposto aos raios solares em 21 de dezembro? Nessa data a reta imaginria determinada pelos centros da Terra
e do Sol corta o globo terrestre num ponto do paralelo de latitude
23 270 S, chamado Trpico de Capricrnio. Assim, no solstcio de
dezembro, os raios solares incidem perpendicularmente sobre
o Trpico de Capricrnio que recebe ento mais luz e calor fazendo
com que o Hemisfrio Sul seja mais iluminado e mais quente que o
Hemisfrio Norte.

Circ. Polar rtico

Trp. de Cncer

Raios
Solares

Equador

Trp. de Capricrnio
S

Circ. Polar

21 de dezembro

Voc j percebeu o que acontece nas regies polares em 21 de


dezembro? Nesse dia, os raios solares no ultrapassam o limite do
paralelo de latitude 66 330 N chamado Crculo Polar rtico. Os pontos situados entre o Crculo Polar rtico e o Polo Norte ficam inteiramente no escuro, independentemente da sua longitude. No h assim
dia, e a noite tem durao de 24 horas.
Enquanto isso, no Hemisfrio Sul, a zona situada entre o Crculo
Polar Antrtico, paralelo de latitude 66 330 S, e o Polo Sul fica iluminada durante 24 horas. No h assim noite, e o dia tem durao de
24 horas.
Vamos agora acompanhar o movimento da Terra de maro at

principal1
2009/8/19
page 36
Estilo OBMEP
36

CAP. 2: QUAL A FORMA DA TERRA?

junho. Nessa poca, o Hemisfrio Sul vai ficando cada vez menos exposto aos raios solares, ao contrrio do Hemisfrio Norte. Assim, no
Hemisfrio Sul onde moramos, os dias vo ficando mais curtos e as
noites mais longas at que, por volta de 21 de junho, temos a noite
mais longa do ano: o nosso solstcio de inverno. Nessa data tem
incio para ns o inverno.
No Hemisfrio Norte, a data 21 de junho marca o dia mais longo
do ano: o solstcio de vero, ou incio do vero. A Terra inclina
seu Polo Norte na direo mais prxima do Sol. A reta imaginria determinada pelos centros da Terra e do Sol corta o globo terrestre num
ponto do paralelo de latitude 23 270 N, chamado Trpico de Cncer.
Logo, no solstcio de junho, os raios solares incidem perpendicularmente sobre o Trpico de Cncer e o Hemisfrio Norte ,
portanto, mais iluminado e mais quente que o Hemisfrio Sul.
Circ. Polar rtico
Trp. de Cncer
Equador
Raios
Solares
Trp. de Capricrnio

Circ. Polar Antrtico

21 de junho

Vejamos o que ocorre nas regies polares nesta poca. A zona


situada entre o Crculo Polar Antrtico e o Polo Sul no atingida
pelos raios solares e a noite dura 24 horas, enquanto que a situada
entre o Crculo Polar rtico e o Polo Norte permanece iluminada
durante 24 horas. No h noite e o dia tem durao de 24 horas.

principal1
2009/8/19
page 37
Estilo OBMEP

N SEC. 2.2: OS MOVIMENTOS DA TERRA

37

Atividades

1) A Terra gira 360 em torno de seu eixo em 24 horas. Quantos


graus de longitude a reta determinada pelos centros da Terra e do
Sol cruza a cada hora? E a cada minuto?
2) Voc est numa localidade cujo meio-dia solar ocorreu 6 horas e
8 minutos depois do meio-dia solar em Greenwich. Qual a sua
longitude?
3) Quanto tempo depois do meio-dia solar em Greenwich ocorre o
meio-dia solar em So Paulo? Qual a diferena de tempo entre o
meio-dia solar em So Paulo e em Roma?
4) A variao sazonal no tempo de durao do dia pode ser modelada
por uma senide. O dia em Nova Orleans tem cerca de 14 horas
no solstcio de vero (21 de junho) e cerca de 9 horas e 20 minutos no solstcio de inverno (21 de dezembro). Nos itens abaixo
desconsidere a possibilidade de um ano bissexto.
(a) Esboce o grfico do nmero h de horas dos dias de Nova
Orleans como funo do nmero x de dias a partir de 21 de
maro.
(b) Encontre uma expresso para h como funo de x, determinando A, B e de modo que h(x) = A + Bsen (x).
(Sugesto: Calcule sabendo que h uma funo peridica de perodo 365. A seguir, obtenha B usando os valores mximos e mnimos da funo h.)
(c) Seja y o nmero de dias aps 1 de janeiro. Encontre uma
expresso para h como funo de y.
(d) Que dias do ano tem durao aproximada de 13 horas em
Nova Orleans?

principal1
2009/8/19
page 38
Estilo OBMEP
38

CAP. 2: QUAL A FORMA DA TERRA?

Respostas
1) 15 ; 0, 25 .
2) 92 W.
3) 3 horas e 8 minutos; 3 horas e 56 minutos.
7
2x
4) (b) h(x) = 12 + sen (
).
3
365

2.3

Os Fusos Horrios

No momento em que for meio-dia em So Paulo, ser tambm


meio-dia em todos os lugares situados ao longo do meridiano que
passa por So Paulo: o que chamamos hora verdadeira. Todas as
localidades situadas sobre um mesmo meridiano tm a mesma hora
verdadeira.
Assim, se meio-dia em Belo Horizonte, isto , se o Sol sobre Belo
Horizonte est em seu ponto mais alto no cu, o mesmo no acontecer
em Braslia. Isso ocorre porque Belo Horizonte est no meridiano de
longitude 44 W, enquanto Braslia encontra-se no meridiano de longitude 48 W. Portanto, meio-dia em Braslia ocorrer um pouco depois
do meio-dia em Belo Horizonte (lembre-se que a Terra gira em torno
de seu eixo polar da esquerda para direita). Afinal, se em Belo Horizonte exatamente meio-dia, qual ser a hora verdadeira em Braslia?
Para responder essa pergunta basta repetir o argumento utilizado na
atividade 3 da pgina anterior.
Assim, a hora verdadeira no a mesma em Belo Horizonte,
Braslia ou So Paulo. Voc j imaginou a confuso que seria se cada
lugar tivesse uma hora diferente, mesmo se tratando de lugares prximos uns dos outros? Cada vez que viajssemos para Leste ou para
Oeste teramos que mudar vrias vezes os ponteiros do relgio.

principal1
2009/8/19
page 39
Estilo OBMEP

N SEC. 2.3: OS FUSOS HORRIOS

39

Para tornar as coisas mais simples, foi necessrio dividir o globo


em 24 partes aproximadamente iguais, uma vez que a Terra leva
24 horas para efetuar seu movimento de rotao. Resultou assim o
que chamamos de fusos horrios. Cada fuso horrio corresponde a
uma faixa limitada por dois meridianos, distantes 15 um do outro
(observe que 360 : 24 = 15 ). Dessa forma, embora haja diferena
na hora verdadeira das localidades situadas dentro de um mesmo fuso
horrio, a hora marcada nos relgios ser a mesma para todas essas localidades. o que chamamos de hora oficial. Assim, Rio de Janeiro,
Braslia, Belo Horizonte, So Paulo, embora sejam cidades situadas
em meridianos diferentes, por encontrarem-se no mesmo fuso horrio
tm a mesma hora oficial.
Conforme mostra a figura abaixo, pases pouco extensos no sentido da longitude, como a Itlia e o Paraguai, tm um nico fuso
horrio. Mas pases muito extensos possuem vrios fusos horrios: os
Estados Unidos, por exemplo, possuem quatro fusos. Pesquise num
atlas geogrfico a situao da hora oficial no Brasil.

principal1
2009/8/19
page 40
Estilo OBMEP
40

CAP. 2: QUAL A FORMA DA TERRA?

Atividades

1) Fuso a regio da superfcie terrestre compreendida entre dois


meridianos. Calcule a rea do fuso limitado pelos meridianos de
longitude 20 E e 40 W. (Sugesto: A rea de um fuso proporcional sua abertura.)
2) Indo-se de Oeste para Leste, deve-se adiantar o relgio de uma hora
cada vez que se ultrapassa um limite de fuso horrio. Utilizando-se
um avio bastante rpido, pode-se partir de Greenwich a zero hora
do sbado, e chegar ao seu meridiano oposto s 24 horas (tempo
local). Nessa altura, sero 12 horas do sbado em Greenwich.
Continuando com a mesma velocidade, o viajante completar a
volta, podendo chegar a Greenwich s 24 horas do sbado; mas seu
relgio, modificado somente em funo dos fusos horrios marcar
24 horas de domingo! Como resolver este problema?
3) Um piloto sai de uma cidade A de latitude 40 N s 12 horas e
15 minutos (hora local verdadeira). Ele voa no rumo Leste a uma
velocidade mdia de 1 000 km/h permanecendo o tempo todo na
mesma latitude. Seu destino B alcanado aps percorrer 3 000
km. Supondo que o raio da Terra mea 6 400 km, qual a hora local
verdadeira em B?

Respostas
1)

2r2
onde r o raio da Terra.
3

2) Pesquise sobre a linha internacional de data.


3) 17 horas e 35 minutos.

principal1
2009/8/19
page 41
Estilo OBMEP

Captulo 3

O ngulo de Elevao do
Sol e da Energia Solar
Vimos anteriormente que as estaes do ano so consequncia da
inclinao do eixo da Terra em relao perpendicular ao plano da
eclptica. A relao entre esta inclinao e a quantidade relativa de
energia solar recebida por localidades com diferentes latitudes uma
questo do mundo real que ser agora analisada.
A grande maioria da energia solar nunca alcana a superfcie da
Terra. Porm, a intensidade de radiao que a alcana varia de acordo
com o chamado ngulo de elevao do Sol.
C

Raios
Solares

?: ngulo de elevao do
Sol em P

?
P
t

41

principal1
2009/8/19
page 42
Estilo OBMEP
42

CAP. 3: O NGULO DE ELEVAO DO SOL E DA ENERGIA SOLAR

Na figura da pgina anterior, a circunferncia C contm o meridiano que passa por uma localidade P do globo terrestre e t a reta
tangente a C em P. O menor ngulo formado pelos raios solares e pela
reta t chamado o ngulo de elevao do Sol ao meio-dia solar
em P.
Nesta parte do curso, iremos calcular a intensidade relativa solar
que alcana a superfcie da Terra em diferentes localidades do mundo
e em diferentes pocas do ano. Para nossos propsitos, intensidade
relativa definida como a razo da intensidade de radiao solar incidente na superfcie pela intensidade de radiao se os raios incidissem
perpendicularmente superfcie (ngulo de elevao com medida igual
a 90 ).
Voc j reparou que a luz solar parece mais quente quando o Sol
est a pino? Voc capaz de explicar por que ao meio-dia de um dia
claro de vero muito mais quente que a noite ou a manh do mesmo
dia? O que faz o inverno frio e o vero quente? As respostas a todas
estas perguntas esto relacionadas com o ngulo de elevao do Sol.
Veremos a seguir que quanto mais prximo de 90 a medida deste
ngulo estiver, maior a intensidade da radiao solar. Apesar disso,
muitas vezes mais quente s 15h que ao meio-dia, pois, embora a
superfcie da Terra receba mais energia solar ao meio-dia, a energia
acumulada na atmosfera, desde manh at este horrio, frequentemente torna estas horas as mais quentes do dia.
A atividade seguinte oferece um bom modelo para quantificar a
relao entre a medida do ngulo de elevao do Sol e a intensidade relativa da radiao solar recebida por uma particular cidade. Escurea
a sala e acenda uma lanterna sobre um pedao de papel branco A.
Toda a rea do papel ser iluminada pela posio perpendicular da
lanterna. Coloque outro pedao de papel idntico na frente de A, mas
incline-o de maneira como mostrada por B. Discuta qual posio do

principal1
2009/8/19
page 43
Estilo OBMEP
43
papel, A ou B,

A
A

recebe mais energia luminosa. Observe a sombra que B projeta em A,


quando B mais e mais inclinado. Note que a diminuio da sombra
indica que a quantidade de luz sobre B est diminuindo. A quantidade
de energia luminosa recebida pelo papel inclinado a mesma recebida
por C, onde C paralelo a A.
Na figura abaixo, a ilustrao da esquerda mostra a viso
lateral quando os raios solares atingem A verticalmente, ou perpendicularmente superfcie. As ilustraes do meio e da direita
mostram a quantidade relativamente menor de energia que a folha
de papel recebe quando o ngulo de elevao do Sol reduzido.

A
B

C
C
?

Quanto mais o papel inclinado, menos energia luminosa ele recebe.


Nas ilustraes anteriores, A e B so na realidade reas retangu-

principal1
2009/8/19
page 44
Estilo OBMEP
44

CAP. 3: O NGULO DE ELEVAO DO SOL E DA ENERGIA SOLAR

lares, mas como as larguras destas reas so as mesmas, podemos


desconsider-las e levar em conta apenas os comprimentos mostrados
na figura acima.
O comprimento de B a medida da hipotenusa do tringulo retngulo, a medida do ngulo de elevao do Sol e o comprimento de
C a medida do lado oposto a . A razo entre o comprimento de C
e o comprimento de B, que depende do ngulo de elevao e igual a
sen , nos d uma medida para a intensidade relativa da luz solar. Se
o comprimento de B tomado como 1, ento sen = C/1 = C. Por
exemplo, se o ngulo do papel B mede 90 , isto , B perpendicular
direo da luz solar, ento sen 90 = 1 e a intensidade relativa da
luz solar 1 ou, em porcentagem, 100%. Como sen 80 0.9848,
temos que quando o ngulo de elevao mede 80 , o papel recebe
aproximadamente 98% da radiao solar disponvel.
Neste modelo, o pedao de papel representa uma rea da Terra e o
ngulo do papel representa o ngulo de elevao do Sol. Quando o Sol
se torna visvel pela primeira vez de manh, certa rea recebe a luz
solar sob um ngulo extremamente pequeno e a intensidade relativa
da radiao solar pequena nesta rea. Quando meio-dia a luz solar
inclinada ao mnimo, ou est perto de se tornar vertical. Ao fim do
dia, os raios solares incidem na Terra novamente com o menor ngulo
possvel. Isto explica por que a luz solar pela manh ou pela tarde
no to quente quanto ao meio-dia.
Atividade
Seja a medida do ngulo de elevao do Sol ao meio-dia solar
em P e a medida do ngulo de elevao do Sol ao meio-dia solar no
Equador, ambas consideradas num mesmo dia. Quais as expresses
relacionando , e a latitude de P? Existem duas possveis respostas
conforme P seja um ponto do Hemisfrio Norte ou Sul.

principal1
2009/8/19
page 45
Estilo OBMEP
45

N SEC. 3.1: O NGULO DE ELEVAO DO SOL NOS EQUINCIOS

3.1

O ngulo de Elevao do Sol nos


Equincios

Vamos calcular a medida desse ngulo ao meio-dia solar nos dias de


equincios de primavera e outono. Como vimos anteriormente, nesses
dias os raios solares incidem perpendicularmente sobre o Equador.
Tomemos, por exemplo, a cidade de Porto Alegre localizada a 30 de
latitude sul.

Na figura abaixo, a reta P B tangente a Terra no ponto P, que


representa Porto Alegre, O o centro da Terra e C um ponto

da reta OP com P entre C e O. Como m(BOP ) = 30o temos


m(AP C) = 30o (so ngulos correspondentes no contexto de re
tas paralelas note que AP e BO so paralelas aos raios solares).
Logo m(AP B) = 60o uma vez que BP C reto.
O ngulo de elevao do Sol ao meio-dia solar mede ento 60 durante os equincios de primavera e outono em Porto Alegre e tambm
em todas as localidades que esto situadas a 30 de latitude sul. Note
que o ngulo de elevao do Sol nesses dias e a latitude so ngulos complementares. Para calcular a intensidade relativa da radiao
solar encontre sen 60o . Uma calculadora mostrar aproximadamente
0,8660,
N
Trp. de Cncer
B

Equador

E
30

A 60
A
Trp. de Capricrnio
S

Equador

principal1
2009/8/19
page 46
Estilo OBMEP
46

CAP. 3: O NGULO DE ELEVAO DO SOL E DA ENERGIA SOLAR

indicando que a intensidade de Sol em Porto Alegre nos dias de


equincios cerca de 86% da que seria se os raios solares incidissem perpendicularmente superfcie, ou ainda, 86% da intensidade
do Sol que atinge o ponto E localizado sobre o Equador e que tem a
mesma longitude de Porto Alegre.
Pelo mesmo mtodo, podemos calcular a intensidade relativa da
radiao solar recebida por qualquer cidade cuja latitude seja conhecida. Por exemplo, Santa Cruz, Argentina, est localizada a 50 de
latitude sul. Portanto, o ngulo de elevao do Sol ao meio-dia solar
nos dias de equincios mede 90 50 = 40 .
Para calcular a intensidade relativa da radiao solar que a cidade
recebe nesses dias, encontre sen 40 . A calculadora mostrar aproximadamente 0,6427. A intensidade relativa da radiao solar cerca
de 64% ao meio-dia solar em Santa Cruz, nos equincios de primavera
e outono.
Atividade
Complete a tabela abaixo, obtendo a intensidade relativa da radiao solar ao meio-dia solar nos dias de equincios.
Cidade
Recife, Brasil
Cuiab, Brasil
So Paulo, Brasil
Melbourne, Austrlia
Quito, Equador
Miami, EUA
Madri, Espanha
Oslo, Noruega
Barrow, Alaska

Latitude
8o S
16o S
23o S
38o S
0o S
26o N
41o N
60o N
71o N

ngulo
de
Elevao do Sol

Intensidade
Relativa

principal1
2009/8/19
page 47
Estilo OBMEP
47

N SEC. 3.2: O NGULO DE ELEVAO DO SOL NOS SOLSTCIOS

3.2

O ngulo de Elevao do Sol nos Solstcios

Vamos agora calcular a medida do ngulo de elevao do Sol ao


meio-dia solar no dia de solstcio de junho. Sabemos que nesse dia os
raios solares incidem perpendicularmente sobre o Trpico de Cncer,
que est aproximadamente a 23 300 de latitude norte.
N

Trp. de Cncer
Equador

2330?

7230?

Trp. de Cncer

Trp. de Cncer

C
Equador

Equador

A
D

B
B

D
E

1730?
3630?
A

5330?
P

A ilustrao da esquerda na figura acima mostra o ngulo de elevao do Sol em Cleveland (EUA) localizada a 41 de latitude norte.

A reta P B tangente a Terra no ponto P que representa Cleveland,

O o centro da Terra e C um ponto da reta OP , com P entre C e



O. Como AP e BO so paralelas aos raios solares temos
m(AP C) = m(BOP )
= m(EOP ) m(EOB)
= 41 23 300
= 17 300 .

principal1
2009/8/19
page 48
Estilo OBMEP
48

CAP. 3: O NGULO DE ELEVAO DO SOL E DA ENERGIA SOLAR

Uma vez que o ngulo BPC reto segue que APB, que o ngulo
de elevao do Sol ao meio-dia solar, tem medida
90 17 300 = 72 300 .
Cleveland e outras cidades localizadas a 41 de latitude norte recebem o maior brilho solar direto do ano durante o dia de solstcio de junho, quando o ngulo de elevao do Sol mede 72 300 ao
meio-dia solar. Para obter a intensidade relativa da radiao solar
que atinge Cleveland nesta hora encontre sen 72 300 . A calculadora
mostrar aproximadamente 0,9537 indicando que Cleveland recebe
cerca de 95% da radiao solar que incidiria se o ngulo fosse de 90
ao meio-dia no solstcio de junho, ou ainda, 95% da intensidade do
Sol que atinge o ponto D localizado sobre o Trpico de Cncer e que
tem a mesma longitude de Cleveland.
A ilustrao da direita na figura acima mostra o ngulo de elevao
do Sol em Porto Alegre. A reta tangente a Terra no ponto P que
representa Porto Alegre, P est C e O com e sendo paralelas aos raios
solares. Como P est no Hemisfrio Sul, a medida do ngulo BOP
igual a
m(EOP ) + m(EOB) = 30 + 23 300 = 53 300
e, pela congruncia de ngulos correspondentes no paralelismo,
m(AP C) = 53 300 . Portanto, AP B, que o ngulo de elevao
do Sol ao meio-dia solar, tem medida
90 53 300 = 36 300 .
Porto Alegre e outras cidades localizadas a 30 de latitude sul
recebem o menor brilho solar direto do ano durante o dia de solstcio
de junho, quando o ngulo de elevao do Sol mede 36 300 ao meio-dia
solar. Para obter a intensidade relativa da radiao solar que atinge

principal1
2009/8/19
page 49
Estilo OBMEP

N SEC. 3.2: O NGULO DE ELEVAO DO SOL NOS SOLSTCIOS

49

Porto Alegre nesta hora encontre sen 36 300 . A calculadora mostrar


aproximadamente 0,5948 indicando que Porto Alegre recebe cerca de
59% da radiao solar que incidiria se o ngulo fosse de 90 ao meiodia no solstcio de junho, isto , 59% da intensidade do Sol que atinge
o ponto D localizado sobre o Trpico de Cncer e que tem a mesma
longitude de Porto Alegre.
N
2330?

Equador

Trp. de Capricrnio
S

Analogamente podemos calcular a medida do ngulo de elevao


do Sol ao meio-dia solar no dia de solstcio de dezembro. Nesse dia
os raios solares incidem perpendicularmente sobre o Trpico de
Capricrnio, que est aproximadamente a 23 300 de latitude sul.

C
P

A
2530?
E
B

6430?
O

Equador

Trp. de Capricrnio

A figura acima mostra o ngulo de elevao do Sol novamente em

Cleveland. A reta P B tangente a Terra no ponto P que representa

principal1
2009/8/19
page 50
Estilo OBMEP
50

CAP. 3: O NGULO DE ELEVAO DO SOL E DA ENERGIA SOLAR


Cleveland, P est entre C e O com AP e BO sendo paralelas aos
raios solares. Como P est no Hemisfrio Norte, a medida do ngulo
BOP igual a
m(EOP ) + m(EOB) = 41 + 23 300 = 64 300

e, pela congruncia de ngulos correspondentes no paralelismo,


m(AP C) = 64 300 . Portanto, AP B, que o ngulo de elevao
do Sol ao meio-dia solar, tem medida
90 64 300 = 25 300 .
Cleveland e outras cidades localizadas a 41 de latitude norte recebem o menor brilho solar direto do ano durante o dia de solstcio
de dezembro, quando o ngulo de elevao do Sol mede 25 300 ao
meio-dia solar. Para obter a intensidade relativa da radiao solar
que atinge Cleveland nesta hora encontre sen 25 300 . A calculadora
mostrar aproximadamente 0,4305 indicando que Cleveland recebe
cerca de 43% da radiao solar que incidiria se o ngulo fosse de 90
ao meio-dia no solstcio de dezembro, ou ainda, 43% da intensidade
do Sol que atinge o ponto D localizado sobre o Trpico de Capricrnio
e que tem a mesma longitude de Cleveland.

8330?

630?
O

Equador

A
P
C
Trp. de
Capricrnio

principal1
2009/8/19
page 51
Estilo OBMEP

N SEC. 3.2: O NGULO DE ELEVAO DO SOL NOS SOLSTCIOS

51

A figura anterior mostra o ngulo de elevao do Sol em Porto

Alegre. A reta P B tangente a Terra no ponto P que representa



Porto Alegre, P est entre C e O com AP e BO sendo paralelas aos
raios solares. Como P est no Hemisfrio Sul, a medida do ngulo
BOP igual a
m(EOP ) m(EOB) = 30 23 300 = 6 300

e, pela congruncia de ngulos correspondentes no paralelismo,


m(AP C) = 6 300 . Portanto, AP B, que o ngulo de elevao
do Sol ao meio-dia solar, tem medida
90 6 300 = 83 300 .
Porto Alegre e outras cidades localizadas a 30 de latitude sul
recebem o maior brilho solar direto do ano durante o dia de solstcio
de dezembro, quando o ngulo de elevao do Sol mede 83 300 ao
meio-dia solar. Para obter a intensidade relativa da radiao solar
que atinge Porto Alegre nesta hora encontre sen 83 300 . A calculadora
mostrar aproximadamente 0,9935 indicando que Porto Alegre recebe
cerca de 99% da radiao solar que incidiria se o ngulo fosse de 90
ao meio-dia no solstcio de dezembro, isto , 99% da intensidade do
Sol que atinge o ponto localizado sobre Trpico de Capricrnio e que
tem a mesma longitude de Porto Alegre.
Vimos nessa seo de que maneira a latitude de um ponto pode
ser usada para calcular o ngulo de elevao do Sol ao meio-dia solar
naquele ponto em determinadas pocas do ano. Sob certas condies,
esse processo pode ser invertido obtendo-se a latitude a partir do
ngulo de elevao do Sol (veja atividade 2 a seguir).

principal1
2009/8/19
page 52
Estilo OBMEP
52

CAP. 3: O NGULO DE ELEVAO DO SOL E DA ENERGIA SOLAR

Atividades
1) Complete a tabela abaixo, obtendo a intensidade relativa da radiao solar ao meio-dia solar nos dias de solstcios.

Latitude
Cidade
Rio Branco, Brasil
So Paulo, Brasil
Melbourne, Austrlia
Santa Cruz, Argentina
Quito, Equador
So Jos, Costa Rica
Miami, EUA
Paris, Frana
Forel, Groelndia

Solstcio de
Junho
ngulo de
Intensidade
elevao
Relativa
do Sol

Solstcio de
Dezembro
ngulo de
Intensidade
elevao
Relativa
do Sol

10 S
23 S
38 S
50 S
0
10 N
26 N
49 N
66 N

2) No dia 26 de janeiro de 2004 as seguintes medidas foram tomadas


numa localidade desconhecida P da regio sul do Brasil.
Altura de uma haste vertical = 35 cm
Comprimento da menor sombra projetada = 5,4 cm
Horrio local da menor sombra = 12 horas 25 minutos 56 segundos
Consultando um almanaque sabe-se que nesse dia os raios solares
incidem perpendicularmente sobre o paralelo de latitude 18 45S e
que o meio-dia solar em Greenwich ocorre s 12 horas 12 minutos
36 segundos. Sabe-se ainda que o horrio na localidade P o de
Greenwich diminudo de 3 horas.
(a) Determine o tempo decorrido entre o meio-dia solar em
Greenwich e o meio-dia solar em P.
(b) Calcule a longitude de P.

principal1
2009/8/19
page 53
Estilo OBMEP

N SEC. 3.2: O NGULO DE ELEVAO DO SOL NOS SOLSTCIOS

53

(c) Determine a medida do ngulo de elevao do Sol ao meio-dia


solar naquele dia.
(d) Calcule a latitude de P.
(e) Consulte um globo terrestre ou um atlas geogrfico para identificar a localidade desconhecida P.

principal1
2009/8/19
page 54
Estilo OBMEP

Captulo 4

A Superfcie Esfrica em
Coordenadas Cartesianas
A partir desta seo trabalharemos num sistema ortogonal de
coordenadas cartesianas com origem O. Dado um ponto P = (x, y, z)
do espao, uma dupla aplicao do teorema de Pitgoras mostra que
a distncia de P a O expressa por
d(P, O) =

p
x2 + y 2 + z 2 .

x 2 ? y 2 ? z2

P = (x, y, z)

O
x2 ? y2

54

principal1
2009/8/19
page 55
Estilo OBMEP
55
Mais geralmente, a distncia entre os pontos P = (x, y, z) e
C = (u, v, w) dada pela frmula
d(P, C) =

(x u)2 + (y v)2 + (z w)2 .

Sendo r um nmero real positivo, a superfcie esfrica S de centro


C = (u, v, w) e raio r o conjunto dos pontos P = (x, y, z) tais que:
(x u)2 + (y v)2 + (z w)2 = r2 .

(4.1)

A equao (4.1) acima denominada equao reduzida de S. Assim, por exemplo, (x + 1)2 + (y 2)2 + z 2 = 4 a equao reduzida

da superfcie esfrica de centro C = (1, 2, 0) e raio r = 4 = 2.


Desenvolvendo os quadrados em (4.1), obtemos:

x2 + y 2 + z 2 2xu 2yv 2zw + u2 + v 2 + w2 r2 = 0

(4.2)

que uma equao da forma


x2 + y 2 + z 2 + ax + by + cz + d = 0

(4.3)

onde a, b, c, d so nmeros reais.


A equao (4.2) chamada equao geral de S. Assim, a superfcie esfrica de centro C = (1, 2, 0) e raio r = 2 tem como equao
geral x2 + y 2 + z 2 + 2x 4y + 1 = 0.
Dada uma equao da forma (4.3) como decidir se ela a equao
geral de alguma superfcie esfrica S? Em caso afirmativo, quais as
coordenadas do centro e qual o raio de S?

principal1
2009/8/19
page 56
Estilo OBMEP
56

CAP. 4: A SUPERFCIE ESFRICA EM COORDENADAS CARTESIANAS

Considerando, por exemplo, a equao


x2 + y 2 + z 2 + 4x 2y 6z + 8 = 0,
a ideia completar os quadrados e colocar (4.3) na forma (4.1).
Assim:
x2 + 4x = x2 + 2.2x = x2 + 2.2x + 22 22 = (x + 2)2 4
y 2 2y = y 2 2y + 12 12 = (y 1)2 1
z 2 6z = z 2 2.3z = z 2 2.3z + 32 32 = (z 3)2 9
Substituindo na equao dada obtemos
(x + 2)2 4 + (y 1)2 1 + (z 3)2 9 + 8 = 0,
ou seja, (x + 2)2 + (y 1)2 + (z 3)2 = 6.
Trata-se, portanto, de uma superfcie esfrica de centro

C = (-2,1,3) e raio r = 6.
Repetindo-se

mesmo argumento para a equao


= 0 obtemos (x 1)2 + (y 2)2 + z 2 = 5

x2 + y 2 + z 2 2x 4y + 10
(faa voc).

Como uma soma de quadrados nunca um nmero negativo conclumos que nenhum ponto do espao tem coordenadas (x,y,z) satisfazendo a equao acima. Trata-se, portanto, do conjunto vazio.
J a equao x2 + y 2 + z 2 2x 4y 6z + 14 = 0 pode ser escrita
como (x 1)2 + (y 2)2 + (z 3)2 = 0 cuja nica soluo o ponto
de coordenadas (x,y,z) = (1,2,3).
O resultado a seguir desempenhar um papel importante no prximo pargrafo onde discutiremos a fundamentao matemtica do
funcionamento do GPS.

principal1
2009/8/19
page 57
Estilo OBMEP
57
Teorema 4.1. Se quatro superfcies esfricas se intersectam e seus
centros so no coplanares ento essa interseco consiste de um
nico ponto.
Demonstrao. Sejam S1 , S2 , S3 e S4 superfcies esfricas de
centros C1 , C2 , C3 e C4 , respectivamente. Mostraremos que se
S1 S2 S3 S4 6= e C1 , C2 , C3 , C4 so no coplanares ento
S1 S2 S3 S4 = {P }.
Sendo x2 + y 2 + z 2 + aj x + bj y + cj z + dj = 0 as equaes gerais
de Sj , onde j = 1, 2, 3, 4, ao subtrairmos essas equaes duas a duas
obtemos equaes lineares em x, y e z uma vez que os termos x2 , y 2 e
z 2 so eliminados.
Tal equao linear determina o plano que contm a correspondente
interseco. Por exemplo, subtraindo as equaes de S1 e S2 obtm-se
uma equao do plano que contm S1 S2 .
Considerando-se os planos que contm S1 S2 , S1 S3 e S1 S4
temos que se P = (x, y, z) est em S1 S2 S3 S4 ento (x, y, z)
a soluo do sistema linear
(a1 a2 )x + (b1 b2 )y + (c1 c2 )z + (d1 d2 ) = 0
(*)

(a1 a3 )x + (b1 b3 )y + (c1 c3 )z + (d1 d3 ) = 0


(a1 a4 )x + (b1 b4 )y + (c1 c4 )z + (d1 d4 ) = 0

A prova do teorema estar terminada se mostrarmos que o sistema


(*) tem uma nica soluo, pois a existncia de dois pontos distintos
em S1 S2 S3 S4 acarretariam duas solues distintas do sistema
linear (*).
Sendo Cj = (uj , vj , wj ) o centro de Sj , j = 1, 2, 3, 4, comparando as equaes (4.2) e (4.3) acima temos aj = 2uj , bj = 2vj ,

principal1
2009/8/19
page 58
Estilo OBMEP
58

CAP. 4: A SUPERFCIE ESFRICA EM COORDENADAS CARTESIANAS

cj = 2wj de modo que

a1 a2 b1 b2 c1 c2

a1 a3 b1 b3 c1 c3

a1 a4 b1 b4 c1 c4

u2 u1 v2 v1 w2 w1

=
8

u3 u1 v3 v1 w3 w1

u4 u1 v4 v1 w4 w1

Como C1 , C2 , C3 , C4 so no-coplanares segue que o determinante


direita no nulo e, portanto, (*) um sistema linear com determinante no nulo tendo assim uma nica soluo.
Evidentemente o simples fato do sistema linear (*) ter uma nica
soluo, significando que os centros so no-coplanares, no acarreta necessariamente que a interseco das quatro superfcies esfricas consiste de um nico ponto P. Em outras palavras, a hiptese
S1 S2 S3 S4 6= essencial para a validade do teorema.
A eventual soluo de (*) nos dar o procurado ponto P desde que
pertena simultaneamente s quatro superfcies esfricas S1 , S2 , S3 e
S4 .
Considere, por exemplo, as superfcies esfricas abaixo.

S1 : centro (0, 0, 1) e raio 2

S2 : centro (0, 3, 0) e raio 10


S3 : centro (2, 0, 0) e raio 1
S4 : centro (0, 0, 0) e raio 1
Seus centros so no-coplanares e o sistema linear (*), neste caso
dado por
6y 2z = 0
4x 2z 4 = 0
2z = 0,
tem como nica soluo x = 1, y = 0 e z = 0. Uma verificao

principal1
2009/8/19
page 59
Estilo OBMEP
59
simples mostra que o ponto P = (1, 0, 0) pertence simultaneamente
S1 , S2 , S3 e S4 de modo que S1 S2 S3 S4 = {(1, 0, 0)}.
Vejamos agora, a relao entre as coordenadas geogrficas e as
coordenadas cartesianas.
Para tanto consideramos um sistema ortogonal de coordenadas
cartesianas com origem O no centro da Terra, o eixo Oz positivo
apontando na direo do Polo Norte N, o plano Oxy sendo o plano do
Equador com o eixo Ox positivo cortando o meridiano de Greenwich
e o eixo Oy positivo cortando o meridiano de longitude 90 E.
Dado um ponto P = (x,y,z) do espao, sejam e os ngulos
assinalados na figura abaixo.

(0,0,z) = B
q = m(?AOP)
j = m(?COA)

P = (x, y, z)
0
j

(x,0,0) = C
A = (x, y, 0)

Quando P est sobre a superfcie terrestre os ngulos e


acima indicados correspondem exatamente latitude e longitude do
ponto P como anteriormente definidos na seo 2.1. A diferena entre
p
OP = x2 + y 2 + z 2 e o raio da Terra chamada de elevao (ou
altitude) de P = (x,y,z).
No tringulo retngulo 4OP B da figura acima temos
cos(90o ) =

OB
z
=p
2
OP
x + y2 + z2

principal1
2009/8/19
page 60
Estilo OBMEP
60

CAP. 4: A SUPERFCIE ESFRICA EM COORDENADAS CARTESIANAS

e, como cos(90o ) = sen , segue que sen = p

.
+ y2 + z2
Esta expresso atribui a um nico valor entre 0 e 90 quando
z > 0 e um nico valor entre -90 e 0 quando z < 0. No primeiro
caso dizemos que a latitude de P N enquanto que no segundo a
latitude de P () S.
x2

Por outro lado, no tringulo retngulo 4OAC temos


sen =

y
OC
x
AC
=p
e cos =
=p
.
2
2
2
OA
OA
x +y
x + y2

Estas expresses definem um nico valor entre 0 e 180 quando y > 0


e dizemos que a longitude de P E. Quando y < 0, assume um
nico valor entre -180 e 0 e, neste caso, a longitude de P () W.
Como exemplo, vamos determinar as coordenadas geogrficas do ponto P cujas coordenadas cartesianas so dadas por

P = (3 3 106 , 3 106 , 6 3 106 ). Considere como unidade de


medida o metro.
Temos
x2 + y 2 + z 2 = 27 1012 + 9 1012 + 108 1012 = 144 1012
e
x2 + y 2 = 27 1012 + 9 1012 = 36 1012 .

6 3 106
3
Logo, sen =
e, portanto, = 60o .
=
12 106
2

3 106
1
3 3 106
3
Como sen =
=
e cos =
=
6
6
6 10
2
6 10
2
obtemos = 30o .

Assim as coordenadas geogrficas de P so = 60 N e = 30 W.


Supondo o raio da Terra igual a 6, 4106 metros temos que a elevao

principal1
2009/8/19
page 61
Estilo OBMEP
61
de P mede 12 106 6, 4 106 = 5, 6 106 metros.
O processo acima pode ser invertido: conhecendo-se a latitude ,
a longitude e a elevao de um ponto P, podemos determinar suas
coordenadas cartesianas x, y e z.
Como antes interpretamos as designaes N/S para e E/W para
como positivas/negativas, respectivamente. Por exemplo, um ponto
com latitude 40 N e longitude 70 W ter = 40 e = 70
enquanto que um ponto com latitude 40 S e longitude 70 E ter
= 40 e = 70 .
p
A partir da elevao de P obtemos o valor de x2 + y 2 + z 2 que
z
denotaremos por r. Logo sen = e, portanto, z = rsen .
r
p
x2 + y 2
segue
Por outro lado, como cos = sen (90 ) =
r
p
p
2
2
2
2
que x = x + y cos = r cos cos e y = x + y sen =
r cos sen .
Para referncia futura repetimos no quadro abaixo as relaes entre as coordenadas geogrficas e as coordenadas cartesianas.
x = r cos cos
y = r cos sen
x = rsen

Atividades

1) Determine a equao geral de cada uma das superfcies esfricas


abaixo.
(a) S1 : centro (5,5,15) e raio 13.
(b) S2 : centro (4,2,5) e raio 3.

principal1
2009/8/19
page 62
Estilo OBMEP
62

CAP. 4: A SUPERFCIE ESFRICA EM COORDENADAS CARTESIANAS

(c) S3 : centro (5,1,7) e raio 5.


(d) S4 : centro (0, -2, -3) e raio 7.
2) Verifique se as equaes dadas abaixo so equaes gerais de superfcies esfricas. Em caso afirmativo, determine o centro e o
raio.
(a) x2 + y 2 + z 2 4x + 6y + 2z 2 = 0.
(b) x2 + y 2 + z 2 2x + 2y = 0.
(c) x2 + y 2 + z 2 10x + 2y 6z + 35 = 0.
(d) x2 + y 2 + z 2 2x + 4y + 15 = 0.
3) Obtenha uma equao
C = (3,-2,5) que :

da

superfcie

esfrica

de

centro

(a) tangente ao plano Oxy;


(b) tangente ao plano Oyz;
(c) tangente ao plano Oxz.
4) Considere as quatro superfcies esfricas dadas na atividade 1. Determine as equaes dos planos que contm S1 S2 , S1 S3 e S1 S4 .
Resolva o sistema linear obtido e ache o ponto de interseco desses
trs planos. Verifique que esse ponto pertence simultaneamente s
quatro superfcies esfricas S1 , S2 , S3 e S4 .
5) Determine as coordenadas geogrficas e a elevao dos seguintes
pontos do espao (considere as coordenadas cartesianas dadas em
metros e o raio da Terra medindo 6,4 106 metros):

(a) P = (3 3 106 , 3 106 , 6 3 106 );

(b) P = (3 3 106 , 3 106 , 6 3 106 );

(c) P = (3 106 , 3 3 106 , 6 106 );

principal1
2009/8/19
page 63
Estilo OBMEP
63

(d) P = (3 105 , 3 3 105 , 6 105 ).

Respostas
1) (a) S1 : x2 + y 2 + z 2 10x 10y 30z + 106 = 0.
(b) S2 : x2 + y 2 + z 2 8x 4y 10z + 36 = 0.
(c) S3 : x2 + y 2 + z 2 10x 2y 14z + 50 = 0.
(d) S4 : x2 + y 2 + z 2 + 4y + 6z 36 = 0.
4) P = (2, 1, 3).

principal1
2009/8/19
page 64
Estilo OBMEP

Captulo 5

A Matemtica do GPS
A sigla GPS nada mais do que a abreviatura para Global
Positioning System. Trata-se de uma constelao de vinte e quatro
satlites, orbitando em torno da Terra a uma altura aproximada de
20 200 km acima do nvel do mar, permitindo a receptores determinar a sua posio em qualquer lugar sobre a Terra com uma notvel
preciso.

64

principal1
2009/8/19
page 65
Estilo OBMEP
65
O projeto foi iniciado em 1973 pelo Departamento de Defesa dos
Estados Unidos com o propsito de que aeronaves e navios militares pudessem determinar, em qualquer circunstncia de tempo, sua
posio exata. Ajuda no lanamento de msseis e a localizao de
tropas terrestres em movimento foram outras necessidades que motivaram tal projeto.
Os projetistas do GPS tambm o planejaram para uso civil,
porm, com preciso menor do que para as operaes militares.
O sistema NAVSTAR (abreviatura para Navigation Satellite
Timing and Ranging), nome oficial dado pelo Departamento de Defesa dos Estados Unidos ao GPS, consiste de um segmento espacial (os
satlites), um segmento de controle (as estaes terrestres de gerenciamento) e um segmento do usurio.

Os vinte e quatro satlites que formam o segmento espacial do


GPS trafegam em torno da Terra em seis rbitas estveis e predeterminadas com quatro satlites em cada rbita. Os satlites percorrem
uma rbita completa a cada 12 horas e cada satlite tem 28 de visualizao sobre a Terra (veja atividade 9 da seo 2.1). Isso assegura
com que todo ponto da superfcie terrestre, em qualquer instante,

principal1
2009/8/19
page 66
Estilo OBMEP
66

CAP. 5: A MATEMTICA DO GPS

esteja visualizado por pelo menos quatro satlites. Vrias reas da


Terra so, por alguns momentos, visualizadas por at dez satlites.
Todos os vinte e quatro satlites so controlados pelas estaes
terrestres de gerenciamento. Existe uma estao master, localizada
no Colorado (Estados Unidos), que com o auxlio de quatro estaes
de gerenciamento espalhadas pelo planeta, monitoram o desempenho
total do sistema, corrigindo as posies dos satlites e reprogramando
o sistema com o padro necessrio. Aps o processamento de todos
esses dados, as correes e sinais de controle so transferidas de volta
para os satlites.

Afinal, de que maneira o GPS determina a localizao de um ponto


sobre a superfcie terrestre?
Cada um dos satlites do GPS transmite por rdio um padro fixado que recebido por um receptor na Terra (segmento do usurio)
funcionando como um cronmetro extremamente acurado. O receptor mede a diferena entre o tempo que o padro recebido
e o tempo que foi emitido. Esta diferena, no mais do que um

principal1
2009/8/19
page 67
Estilo OBMEP
67
dcimo de segundo, permite que o receptor calcule a distncia ao
satlite emissor multiplicando-se a velocidade do sinal (aproximadamente 2,99792458 108 m/s a velocidade da luz) pelo tempo que
o sinal de rdio levou do satlite ao receptor.
Essa informao localiza uma pessoa sobre uma imaginria superfcie esfrica com centro no satlite e raio igual distncia acima
calculada.
Cada satlite programado para emitir o que se chama
efemride, que informa a sua posio exata, naquele instante, em
relao a um sistema ortogonal de coordenadas como o descrito na
seo anterior. Tal posio permanentemente rastreada e conferida
pelas estaes terrestres de gerenciamento. A unidade receptora processa esses sinais. Com a posio do satlite e a distncia acima
calculada obtm-se a equao geral da superfcie esfrica imaginria.
Coletando-se sinais emitidos por quatro satlites, o receptor determina a posio do usurio calculando-a como interseco das quatro
superfcies esfricas obtidas. A localizao dada, no em coordenadas cartesianas, mas por meio das coordenadas geogrficas (latitude,
longitude) e a elevao.
A preciso do tempo essencial na operao do GPS. Um erro
de um microssegundo (106 segundos) no registro do lapso de tempo
desde a transmisso at a sua recepo resulta num erro de 300 metros.
Unidades receptoras do GPS extremamente precisas (e caras!) podem
determinar sua posio a menos de um metro.
Com o fim da guerra fria, o sistema GPS passou a oferecer uma
preciso muito maior para o usurio civil, disponibilizando a ele a
mesma preciso que s os militares tinham a algum tempo atrs. Hoje
em dia, com auxlio do piloto automtico e do GPS, uma aeronave
civil capaz de percorrer distncias transatlnticas e pousar sem a
interferncia do piloto com erro de alguns centmetros com o eixo da

principal1
2009/8/19
page 68
Estilo OBMEP
68

CAP. 5: A MATEMTICA DO GPS

pista.
A navegao a funo primria do GPS sendo usado em aeronaves, navios, veculos e por indivduos que usam o receptor porttil
(de bolso). Atualmente o GPS tem se mostrado til em diversas
situaes das quais destacamos algumas.
1. Roteirista de viagens: determinam alm da sua posio dentro de uma cidade, quais as atraes e pontos tursticos mais
prximos, hotis, postos de emergncias, etc.
2. Monitoramento de abalos ssmicos: tais abalos so precedidos
por alteraes no campo gravitacional que distorcem as ondas
de rdio permitindo, atravs do GPS, tentar prever a ocorrncia
de um terremoto com algumas horas de antecedncia.
3. Meteorologia: o GPS gera informaes para a previso da meteorologia, estudo do clima e outros campos de pesquisa relacionados.
4. Localizao para resgate: o servio usa o GPS para guiar helicpteros de socorro at o lugar do acidente.
5. Aplicaes industriais: reas infectadas por pestes so identificadas por fotografias areas e, com uso do GPS, um trator pode
ser guiado para aplicaes de pesticidas.
6. Uso militar: coordenadas de ataque, orientao e controle para
msseis balsticos, marcao para artilharia, bombardeio de aeronaves, defesa area, rastreamento de submarinos, localizao
de minas e radares inimigos, atos terroristas, etc.
Na atividade a seguir pretendemos discutir, do ponto de vista
matemtico, o mtodo utilizado pelo GPS na determinao da posio
de um ponto sobre a superfcie terrestre. As informaes transmitidas

principal1
2009/8/19
page 69
Estilo OBMEP
69
no sistema GPS envolvem, por uma questo de preciso, dez ou mais
dgitos. Para um aproveitamento mais realista da atividade, sugerimos a utilizao de calculadoras ou softwares com capacidade de
resolver sistemas lineares com coeficientes dessa ordem. Uma alternativa, abrindo mo eventualmente da preciso, trabalhar com um
nmero menor de dgitos utilizando a notao cientfica. Suponha que
o raio da Terra mea 6,378164 106 metros e considere a velocidade
da luz sendo de 2,99792458 108 m/s.

Atividade
A tabela abaixo indica as efemrides (em metros) de cada satlite.

Satlite
Satlite
Satlite
Satlite

1
2
3
4

x
1,877191188 106
1,098145713 107
2,459587359 107
3,855818937 106

y
-1,064608026 107
-1,308719098 107
-4,336916128 106
7,251740720 106

z
2,428036099 107
2,036005484 107
9,090267461 106
2,527733606 107

Um receptor GPS registra os seguintes lapsos de tempo (em segundos) entre a transmisso e a recepo do sinal de cada satlite.
Satlite 1

Satlite 2

Satlite 3

Satlite 4

0,08251731391

0,07718558331

0,06890629029

0,07815826940

(a) Calcule a distncia entre o receptor e cada satlite.


(b) Escreva as equaes gerais das imaginrias superfcies esfricas
centradas em cada satlite e raios iguais s distncias calculadas
no item anterior.

principal1
2009/8/19
page 70
Estilo OBMEP
70

CAP. 5: A MATEMTICA DO GPS

(c) Determine as coordenadas cartesianas do ponto P que pertence


simultaneamente s quatro superfcies esfricas obtidas no item
anterior. (Sugesto: Veja atividade 4 do captulo 4)
(d) Determine a latitude, a longitude e a elevao do ponto P.
(e) Consulte um atlas geogrfico ou um globo terrestre para identificar a posio desse usurio do GPS.

principal1
2009/8/19
page 71
Estilo OBMEP

Captulo 6

A Distncia numa Superfcie


Esfrica
A distncia d(A, B) entre dois pontos A e B , essencialmente, o
menor dos comprimentos das trajetrias ligando A a B. No plano,
a trajetria de menor comprimento o segmento de linha reta AB e
seu comprimento AB a distncia entre A e B. Sobre uma superfcie
esfrica, no entanto, no existe um segmento de linha reta uma vez
que ela curvada em todas as direes e tneis atravs da Terra no
so permitidos. Como medir a distncia entre dois pontos A e B neste
caso?
A
B

71

principal1
2009/8/19
page 72
Estilo OBMEP
72

CAP. 6: A DISTNCIA NUMA SUPERFCIE ESFRICA

Quanto maior o raio de uma circunferncia, mais ela se aproxima


de ser uma reta. Como as circunferncias de maior raio contidas numa
superfcie esfrica S so as circunferncias mximas, razovel esperar
que a distncia (em S) entre dois pontos A e B seja o comprimento
_

do arco menor AB da circunferncia mxima que passa por A e B.


Uma prova formal desta afirmao ser dada adiante.
O clculo desse comprimento pode ser feito a partir do conhecimento da medida do ngulo AOB onde O o centro da superfcie
esfrica S. Como o comprimento do arco proporcional medida do
ngulo central correspondente, uma regra de trs simples nos d o
valor procurado.
Sendo r o raio da superfcie esfrica temos
360o
o

.....
.....

2r
d(A,B)

de modo que d(A,B) = (/360).(2).r.


Todos os meridianos esto contidos em circunferncias mximas
enquanto que, entre os paralelos, apenas o Equador uma circunferncia mxima. Logo quando A e B possuem a mesma longitude, a
diferena entre as latitudes pode ser usada para achar a medida .
Analogamente quando A e B esto sobre o Equador a diferena
entre as longitudes que nos permite calcular . Vejamos como.
As cidades de Curitiba e Goinia esto sobre o mesmo meridiano
W) e suas latitudes so 26o S e 17o S, respectivamente. Esto
assim separadas por 9o de latitude e, tomando o raio da Terra como
6 400 km, segue que a distncia entre elas dada por

(49o

(9/360).(2).6 400 1 005 km.

principal1
2009/8/19
page 73
Estilo OBMEP
73
As cidades de Quito, no Equador, e Entebe, em Uganda, esto
ambas sobre o Equador. A longitude de Quito 79o W enquanto que
a de Entebe 32o E. Logo a diferena entre suas longitudes de 111o
de modo que a distncia entre elas igual a
(111/360).(2).6 400 12 399 km.

Quando duas cidades A e B esto sobre um mesmo paralelo, que


no seja o Equador, o caminho mais curto possvel entre elas, ao
contrrio do que diz nossa intuio, no o comprimento do arco
_
_
menor AB daquele paralelo e sim o comprimento do arco menor AB
da circunferncia mxima que passa por A e B.
paralelo
A

B
?

circunferncia mxima

Por exemplo, as cidades de Nova York e Npoles esto praticamente sobre o mesmo paralelo (41 N) e suas longitudes so 74 W e
14 E, respectivamente. O comprimento do arco menor do paralelo
entre as duas cidades cerca de 7 419 km (verifique).
Se A e B representam as cidades de Nova York e Npoles, respectivamente, vejamos como calcular neste caso o comprimento do arco
_
menor AB da circunferncia mxima que passa por A e B, ou seja,
como calcular = m(AOB).
Considerando-se um sistema ortogonal de coordenadas cartesianas
como descrito no captulo 4 e supondo, como de costume, o raio da

principal1
2009/8/19
page 74
Estilo OBMEP
74

CAP. 6: A DISTNCIA NUMA SUPERFCIE ESFRICA

Terra medindo 6 400 km, podemos escrever


A = 6 400(cos 41 cos(74 ), cos 41 sen (74 ), sen 41 )
B = 6 400(cos 41 cos 14 , cos 41 sen 14 , sen 41 ),
ou seja,
A = 6 400(0,20802, 0,72547, 0,65606)
B = 6 400(0,73229, 0,18257, 0,65606).
A medida procurada ser obtida por meio da relao


< OA, OB >= kOAkkOBk cos , onde <, > indica o produto in

terno usual entre os vetores OA e OB enquanto que kOAk, kOBk so


os mdulos desses vetores, neste caso ambos iguais as 6 400.
Como

< OA, OB > = 6 4002 [0,152330,13244+0,43041] = 6 4002 0,4503,
segue que cos = 0,4503 e, portanto, = 63 .
_

O comprimento do arco menor AB da circunferncia mxima que


passa por A e B, que a distncia entre A e B, ento dada por
(63/360).2.6 400 7 037km. Note como esta distncia menor do
que aquela calculada ao longo do paralelo.
O argumento acima pode ser utilizado para calcular a distncia entre dois pontos quaisquer A e B do globo terrestre.
Dadas suas coordenadas geogrficas, obtemos suas coordenadas carte
sianas e, usando o produto interno < OA, OB >, determinamos
= m(AOB). A distncia procurada d(A, B) ento dada por
d(A, B) = (/360).(2).6 400.
Encerramos a seo apresentando uma prova formal do fato que
_
d(A, B) o comprimento do arco menor AB da circunferncia mxima que passa por A e B. Isso pode ser feito com umas poucas
simplificaes e a ajuda do clculo diferencial e integral.

principal1
2009/8/19
page 75
Estilo OBMEP
75
Primeiro, podemos supor sem perda de generalidade que a superfcie esfrica que estamos trabalhando tem raio igual a 1 unidade (a
que voc quiser) e que seu centro a origem do sistema ortogonal de
coordenadas cartesianas.
Segundo, pela simetria da superfcie esfrica podemos assumir que
o ponto A o Polo Norte. O outro ponto, B, ser dado pelas suas
coordenadas geogrficas que vamos supor medidas em radianos. Digamos que a latitude de B 1 enquanto que sua longitude 1 .
A circunferncia mxima que passa por A e B contm o meridiano
_

por B de modo que o arco menor AB tem por comprimento 1


2
(lembre-se que o raio da superfcie esfrica 1).
Devemos agora considerar uma trajetria arbitrria ligando A e

B e mostrar que seu comprimento maior ou igual a 1 .


2

A
r (t)

B
q1
j1

Se nos imaginarmos viajando de A at B, em cada instante t,


estaremos num ponto do globo terrestre com uma latitude (t) e uma
longitude (t). Iniciando nossa viagem no instante t = 0 e a finali
zando no instante t1 teremos (0) = , (t1 ) = 1 e (t1 ) = 1 .
2
Uma trajetria arbitrria ligando A e B ento descrita por um

principal1
2009/8/19
page 76
Estilo OBMEP
76

CAP. 6: A DISTNCIA NUMA SUPERFCIE ESFRICA

vetor posio
r (t) dado por

r (t) = (cos (t) cos (t), cos (t)sen (t), sen (t)), 0 t t1 .
Derivando-se essa funo vetorial em relao ao tempo t obtemos

o vetor velocidade r0 (t) da trajetria e consequentemente sua veloci

dade k r0 (t)k no instante t.


Clculos rotineiros nos do que
p
p

k r0 (t)k = 0 (t)2 + 0 (t)2 cos2 (t) 0 (t)2 = |0 (t)| 0 (t).


Lembrando que o comprimento da trajetria calculado pela exRt

presso L = 0 1 k r0 (t)kdt, conclumos que


Rt

L 0 1 [0 (t)]dt = [(t1 ) (0)] = 1


2
e temos assim provado o resultado desejado.
Atividades
Em todos os exerccios desta srie suponha que o raio da Terra
mea 6 400 km.
1) As cidades de Macap (Brasil) e Pontianak (Indonsia) esto
ambas situadas sobre o Equador. Consulte um atlas geogrfico
para achar a longitude de cada uma dessas localidades e determine a distncia entre elas.
2) Qual a distncia de Salvador ao Polo Sul? E ao Polo Norte? E
a Fortaleza? (Sugesto: Salvador e Fortaleza esto sobre um
mesmo meridiano.)
3) Chicago e Roma situam-se na mesma latitude (42 N), mas em
longitudes diferentes: a de Chicago aproximadamente 88 W e a
de Roma aproximadamente 12 E.

principal1
2009/8/19
page 77
Estilo OBMEP
77
(a) Suponha que um piloto tenha ido de Chicago a Roma em
voo no rumo leste, permanecendo o tempo todo na mesma
latitude. (Muita gente acha que esse o caminho mais curto possvel.) Que distncia o piloto teve de voar?
(b) Qual a distncia de Chicago a Roma por circunferncia mxima?
(c) Quanto tempo se economizaria voando em uma circunferncia mxima num avio a jato capaz de fazer uma velocidade
mdia de 900 km/h?
4) Qual a distncia entre Nova York (40 40N, 74 W) e Buenos
Aires (34 30S, 58 30W)?

Respostas
3) (a) Cerca de 8 300 km.
(b) Cerca de 7 752 km.
(c) Aproximadamente 36 minutos.
4) Cerca de 8 545 km.

principal1
2009/8/19
page 78
Estilo OBMEP

principal1
2009/8/19
page 79
Estilo OBMEP

Referncias Bibliogrficas
[1] AZEVEDO, G.G.; SANTOS, F.M. Panorama do Brasil I. Rio de
Janeiro: Editora Atual, 1990.
[2] FEEMAN, T.G. Portraits of the Earth. [S.L.]: American Mathematical Society, 2002.
[3] HY, Kim. Angled Sunshine, Seasons and Solar Energy. The
Mathematics Teacher, v. 90, n. 7, October 1997.
[4] JENNINGS, G.A.
[S.L.:s.n.], 1997.

Modern

Geometry

with

Applications.

[5] MOISE, E. Elementary Geometry from an Advanced Standpoint.


Third Edition. [S.L.]: Addison Wesley Publishing Co., 1990.
[6] NORD, G.D.; JABON, D.; NORD, J. The Mathematics of the
Global Positioning System. The Mathematics Teacher, v. 90,
n. 6, September 1997.
[7] WAIVERIS, C.; CRAINE, T.V. Where are we?. The Mathematics Teacher, v. 89, n. 6, September 1996.
79

principal1
2009/8/19
page 80
Estilo OBMEP
80

Sites Consultados

[1] www.garmin.com
[2] http://astro.if.ufrgs.br/indexhtml

REFERNCIAS BIBLIOGRFICAS

principal1
2009/8/19
page 81
Estilo OBMEP

Os Trs Problemas Clssicos da


Matemtica Grega

Joo Pitombeira de Carvalho

81

principal1
2009/8/19
page 82
Estilo OBMEP

principal1
2009/8/19
page 83
Estilo OBMEP

Prefcio
Este trabalho foi escrito originalmente para um minicurso apresentado na Bienal da Sociedade Brasileira de Matemtica, em Salvador,
em 2004. Posteriormente, foi feita uma reviso completa do texto,
com a correo de erros de impresso, e incorporadas sugestes dos
alunos. esta verso aqui impressa. Mais tarde, foi refeito e publicado como um dos captulos do livro de Joo Bosco Pitombeira de
Carvalho, Trs excurses pela Histria da Matemtica. 1
No minicurso, usamos o aplicativo de geometria dinmica
TABULAE, para que os alunos pudessem fazer, realmente, as construes geomtricas. A mais elaborada, que exige pacincia, a da
quadratriz. Acreditamos que a utilizao da geometria dinmica d
vida ao estudo dos problemas clssicos, pois, com ela, possvel realmente ver que as construes feitas pelos gregos funcionam. Isso
instrutivo principalmente no caso das mquinas de Plato e de
Eratstenes (ver, 7.1 e 7.2, respectivamente). Esta fuso da histria
com as tecnologias torna o estudo dos problemas clssicos bem mais
interessante. Outra ocasio em que a geometria dinmica d vida ao
estudo das construes geomtricas dos gregos no estudo de construes por neusis ou ajustamento. Com um aplicativo de geometria
dinmica, o leitor pode ver que a construo possvel.
1

Os dois outros captulos tratam, respectivamente, da equivalncia e aplicao


de reas na matemtica grega e da histria da equao do segundo grau.

83

principal1
2009/8/19
page 84
Estilo OBMEP
84
Os tpicos expostos neste texto tm encantado sucessivas geraes
de matemticos profissionais ou amadores. Eles so os problemas da
duplicao do cubo, quadratura do crculo e trisseco do ngulo,
os quais moldaram muito das atividades de pesquisa matemtica dos
gregos, e deram origem, inclusive, ao estudo das cnicas. vastssima
a produo matemtica dedicada a estes trs problemas, provenientes
de erros, por vezes sutis, sobre as exigncias feitas para a soluo
dos problemas. Somente no sculo XIX que foi demonstrado que
eles no podem ser resolvidos somente com rgua (no graduada) e
compasso, o que no impede o aparecimento, at hoje, de provas de
que possvel efetuar as trs construes. Essas tentativas no devem
ser olhadas com desprezo, mas sim como testemunho de interesse pela
Matemtica em vrios setores na sociedade.

Cabe um pedido de desculpas e, ao mesmo tempo, uma explicao: os leitores notaro por certo a predominncia de referncias
bibliogrficas em lnguas estrangeiras. Isso se deve ao pouco nmero
de fontes em portugus sobre o assunto. Embora progressos estejam
sendo feitos no sentido de tornar disponveis, em nossa lngua, bons
textos sobre os tpicos cobertos neste livro, at hoje a maior parte dos
trabalhos sobre o assunto esto escritos em outros idiomas. Em verdade, a motivao para a redao deste material foi tornar disponvel
exposies introdutrias sobre os tpicos aqui tratados.
Neste trabalho, o livro Science Awakening, de Van der Waerden,
simplesmente citado como van der Waerden. A edio padro, facilmente disponvel, dos Elementos de Euclides a de Heath (1956).
Para os leitores que no tm acesso a esta edio, recomendamos a
traduo da edio de Simson (1773) para o portugus, perfeitamente
suficiente para nossas finalidades. 2 Teoremas, figuras, equaes ou
2

Ela se encontra disponvel gratuitamente em www.dominiopublico.gov.br.

principal1
2009/8/19
page 85
Estilo OBMEP
85
igualdades so numeradas sequencialmente. O smbolo 2 indica o fim
de uma demonstrao ou construo.
Com exceo das Figuras 7.8, 8.1 e 9.4, todas as ilustraes foram
R
feitas com o aplicativo de Geometria dinmica TABULAE
, desenvolvido no Instituto de Matemtica da Universidade Federal do Rio
de Janeiro.
Embora o autor tenha procurado corrigir todos os erros tipogrficos ou matemticos do texto, certamente alguns lhe escaparam. Como
disse Monteiro Lobato, os erros tipogrficos so como sacis. Quando
os procuramos, eles se escondem. Uma vez publicado o texto, saltam
alegres chamando a ateno dos leitores. Fico, desde j, muito grato
pelas indicaes dos erros remanescentes (matemticos ou tipogrficos), as quais podem ser encaminhadas para jbpfcarvalho@gmail.com.

Joo Bosco Pitombeira de Carvalho

principal1
2009/8/19
page 86
Estilo OBMEP

principal1
2009/8/19
page 87
Estilo OBME

Sumrio
Introduo

89

7 A Duplicao do Cubo

97

7.1

A mquina de Plato . . . . . . . . . . . . . . . . . . .

7.2

A mquina de Eratstenes . . . . . . . . . . . . . . . . 100

7.3

A soluo de Nicomedes . . . . . . . . . . . . . . . . . 104

7.4

A Construo de rquitas . . . . . . . . . . . . . . . . 110

7.5

A soluo achada por Mencmo . . . . . . . . . . . . . 113

7.6

O mtodo de Diocles . . . . . . . . . . . . . . . . . . . 114

7.7

O mtodo de Hiero . . . . . . . . . . . . . . . . . . . 117

8 A Quadratura do Crculo
8.1

99

119

A Quadratriz . . . . . . . . . . . . . . . . . . . . . . . 119

9 A Trisseco do ngulo

122

9.1

A Trisseco do ngulo por Arquimedes . . . . . . . . 123

9.2

A Trisseco do ngulo por Nicomedes . . . . . . . . . 124

9.3

A Trisseco do ngulo Usando a Quadratriz . . . . . 126


87

principal1
2009/8/19
page 88
Estilo OBMEP
88
Referncias Bibliogrficas

129

principal1
2009/8/19
page 89
Estilo OBMEP

Introduo
indubitvel que, na histria da Matemtica, alguns problemas tm significao
especial: agindo como catalisadores eles
influenciam muito o desenvolvimento da cincia. Tais problemas atraem devido simplicidade e lucidez de seus enunciados, fascinando muitos matemticos. Como resultado,
so elaborados novos mtodos e, at mesmo
novas teorias e novas perguntas, profundas
e abrangentes, so formuladas (Raigorodski,
2004).

Os matemticos gregos estudaram trs problemas de Geometria que desempenharam papel importante no desenvolvimento da
Matemtica. Eles so problemas de construo e resistiram a todas
as tentativas dos gregos para resolv-los utilizando somente a rgua
sem graduao e o compasso, os nicos instrumentos empregados por
Euclides nos Elementos.

89

principal1
2009/8/19
page 90
Estilo OBMEP
90
Os problemas, que ficaram conhecidos como os trs problemas
clssicos, so:
1 A duplicao do cubo.
2 A quadratura do crculo.
3 A trisseco do ngulo.
Sabemos, desde o sculo XIX, que esses problemas no podem ser
resolvidos somente com a rgua e o compasso. Referncias acessveis
sobre isso so, por exemplo, Courant and Robbins (1996), Hadlock
(1978), Klein (1930), Bunt, Jones and Bedient (1988, p. 89-121).
Uma discusso de por que os gregos tentavam resolver problemas de
construo usando somente a rgua e o compasso, pode ser encontrada, por exemplo, em Bkouche et Jolle (1993).
Para os primeiros gemetras gregos, uma linha era o percurso de
um ponto, e a linha reta era um percurso sem asperezas e desvios
(Szab, 2000). No entanto, aos poucos, os matemticos gregos se
distanciaram da realidade palpvel, como se v, por exemplo, em
Plato:
[a Geometria] tem por objeto o conhecimento do que sempre e no do que nasce e perece.
Passaram tambm a dar ao crculo e reta papis destacados:
...Aristteles O que no tem nem comeo nem fim
portanto ilimitado.
Parmnides Ele ilimitado.
Aristteles Portanto ele no tem forma, pois no participa nem do redondo nem do reto.

principal1
2009/8/19
page 91
Estilo OBMEP
91

Alm da ideia de perfeio ideal atribuda ao crculo e linha


reta uma outra razo para a restrio rgua e ao compasso pode

ter sido a crise devida descoberta da irracionalidade de 2, nmero


que pode, no entanto ser construdo com rgua e compasso. Esses
instrumentos eram a garantia da existncia de tais nmeros.
No entanto, falsa a crena de que os gregos, na resoluo de
problemas de construes geomtricas, trabalhavam somente com a
rgua e o compasso. Exatamente como os matemticos de hoje, para
resolverem um problema eles usavam todas as ferramentas disponveis
ou criavam novas ferramentas apropriadas. De suas tentativas para
achar solues para os problemas clssicos, surgiram vrias curvas e
mtodos que enriqueceram a Matemtica. Encontram-se em Knorr
(1986) e Bos (2001) construes geomtricas, incluindo solues dos
trs problemas clssicos, utilizando vrias curvas e outros instrumentos.
O matemtico Van der Waerden (p. 263) resumiu a situao como
segue:

A ideia por vezes expressa de que os gregos permitiam somente construes com rgua e compasso inadmissvel. Ela negada pelas numerosas construes que
nos chegaram para a duplicao do cubo e a trisseco do
ngulo. No entanto, verdade que tais construes eram
consideradas mais elementares, e Pappus afirma que sempre que uma construo for possvel com rgua e compasso
mtodos mais avanados no deveriam ser usados.
Isso parece seguir a mesma linha que o princpio da navalha de
Ockham, muito posterior. Na linguagem popular, poderamos dizer

principal1
2009/8/19
page 92
Estilo OBMEP
92
que no se deve usar um canho para matar uma mosca.

impossvel superestimar a importncia destes problemas. Como


diz Yates (1971, p. 5).
Na histria da Matemtica h trs problemas que persistiram com vigor impressionante durante mais de dois
mil anos. Eles so a trisseco do ngulo, a duplicao do
cubo e a quadratura do crculo, e devido sua existncia robusta eles so atualmente chamados de problemas
famosos. (...) Estes trs problemas, solidamente inexpugnveis malgrado todas as tentativas usando geometria
plana, o mtodo matemtico dos antigos gregos, fizeram
com que os matemticos ficassem fascinados e construssem novas tcnicas e teoremas para sua soluo. Por meio
deste estmulo surgiu grande parte das estruturas atuais
da lgebra e da geometria.
A procura constante de solues para os trs problemas durante
tanto tempo forneceu descobertas frutferas e que lanaram luz sobre
tpicos bem distantes. Somente em 1837 que foi demonstrado, por
Wantzel, que um nmero real construtvel com rgua e compasso
se, e somente se, ele um nmero algbrico, de grau uma potncia de
dois, sobre os racionais.
Os trs problemas so muito naturais para quem tem curiosidade
matemtica. Scrates, no dilogo Meno de Plato, usando perguntas
apropriadas, faz com que um jovem escravo ache um quadrado cuja
rea duas vezes a rea de um quadrado dado. Isso pode ser feito
facilmente usando rgua e compasso.
Com efeito, o quadrado cujo lado a diagonal do quadrado
dado a soluo do problema. Se AB o lado do quadrado dado

principal1
2009/8/19
page 93
Estilo OBMEP
93
(Figura 6.1), ento
DB =

2 AB = DB 2 = 2 AB 2 .

Figura 6.1: Duplicao do quadrado

De DB 2 = 2 AB 2 , vemos imediatamente que


AB
DB
=
.
DB
2AB
Assim, achar o comprimento de DB equivalente a inserir uma
meia proporcional entre AB e 2AB.
De maneira mais geral, se desejarmos construir um quadrado cuja
rea seja b vezes a rea a do quadrado ABCD, devemos ter
AB
DB
=
.
DB
bAB
Como veremos, a ideia de inserir meias proporcionais entre duas
grandezas dadas est por traz da maioria das tentativas de duplicar
o cubo.
Quadrar uma regio plana consiste em traar, somente com

principal1
2009/8/19
page 94
Estilo OBMEP
94
rgua e compasso, um quadrado cuja rea seja igual rea da regio
dada. O problema de quadrar qualquer regio poligonal est completamente resolvido nos Elementos de Euclides, e ir ser estudado no
Captulo 7. O problema da quadratura do crculo tambm muito
natural. Uma vez resolvido o problema da quadratura de qualquer
regio poligonal, o prximo passo tentar quadrar regies limitadas
por linhas curvas. Entre estas regies, o crculo uma escolha bvia.
Isso levou investigao das lnulas por Hipcrates de Quios, em
torno de 430 a.C. (Van der Waerden, p. 131-132). Curiosamente, somente h pouco tempo, em 1947, usando tcnicas muito sofisticadas,
que o problema de achar todas as lnulas quadrveis foi completamente resolvido (Scriba, 1987)!

A primeira meno conhecida do problema da quadratura do crculo encontra-se no problema 50 do papiro Rhind, em torno de 1600
a.C.:
Um campo circular tem 9 khet de dimetro. Qual a sua rea?
Resoluo: Tira 1/9 do dimetro do seu dimetro, isto 1 Khet. O
resto 8 Khet. Multiplica 8 por 8; o que faz 64. Por isso, contm 64
setat de terra.
Nota: 1 setat khet ao quadrado
Em sua comdia Os pssaros, Aristfanes introduz o astrnomo
Meton e o ridiculariza por causa de suas tentativas de fazer a
quadratura do crculo:
Farei minhas medies com um esquadro reto [90o graus],
e assim voc observa que o crculo se torna quadrangular.
Para Szab (2000), o problema de quadratura que deu origem a todos os outros foi o de fazer a quadratura do retngulo. Este problema
facilmente resolvido com rgua e compasso usando o resultado que,

principal1
2009/8/19
page 95
Estilo OBMEP
95
em um tringulo retngulo, a altura relativa hipotenusa a meia
proporcional entre os segmentos que ela determina sobre a hipotenusa.
Aristteles,3 por sua vez, pensa que este problema surgiu da
procura da mdia geomtrica (meia proporcional), mas que isso foi
esquecido e s restou o prprio problema:
A definio no deve contentar-se em exprimir em que
consiste a coisa (...), mas ela deve tambm incluir e exibir a causa. Ora, as definies so geralmente concluses.
Por exemplo: O que a quadratura? a igualdade de um
quadrado e de um retngulo. Uma definio como essa
uma concluso. Mas dizer que a quadratura a descoberta
da meia proporcional exprimir a causa do que definido.

Semelhantemente, uma vez que se sabe como bissectar um ngulo


(Elementos, I-9), natural perguntar como dividir um ngulo em n
partes; em particular, em 3 partes.
Embora no tenham conseguido resolver estes problemas com os
instrumentos especificados, os matemticos gregos no se deixaram
intimidar e, com engenho notvel, foram capazes de achar solues
para os trs problemas, usando vrios outros tipos de instrumentos e
construes.
Em verdade, da mesma maneira que a Matemtica moderna cresce
com respostas aos desafios de novos problemas, muito da Matemtica
grega se desenvolveu devido a tentativas de resolver os trs problemas clssicos. Neste sentido, os matemticos gregos eram realmente
nossos colegas, pois tinham a mesma atitude mental dos matemticos
atuais e tentavam conscientemente atacar novos desafios. Quando os
3

Tratado da alma, II,2, 413, 13-20.

principal1
2009/8/19
page 96
Estilo OBMEP
96
conceitos e tcnicas existentes no conseguiam resolver estes problemas, eles inventavam novos conceitos e tcnicas apropriadas para a
tarefa.

principal1
2009/8/19
page 97
Estilo OBMEP

Captulo 7

A Duplicao do Cubo
O que sabemos sobre este problema encontra-se principalmente
em Eutcio, um comentador de Arquimedes.
H duas lendas sobre a origem da duplicao do cubo, com detalhes contraditrios. Uma delas se refere duplicao de um tmulo e
a outra duplicao de um altar (Van der Waerden, p. 160-161).
Segundo a primeira lenda, Minos mandou fazer um tmulo para
Glauco. Ao saber que o tmulo era um cubo cuja aresta media 100
ps, ele disse que a residncia real tinha sido construda demasiadamente pequena e que ela deveria ser duas vezes maior e ordenou imediatamente que duplicassem cada aresta do tmulo, sem estragar sua
bela forma.
De acordo com a segunda lenda, quando um orculo anunciou
aos habitantes de Delos que, para se verem livres da peste, deveriam
construir um altar duas vezes maior do que o existente, os arquitetos
ficaram muito confusos, pois no sabiam como construir um cubo
duas vezes maior do que outro.
Hipcrates de Quios (viveu em torno de 430 a.C.) reduziu este
problema ao de achar duas meias proporcionais x e y entre 1 e 2.
97

principal1
2009/8/19
page 98
Estilo OBMEP
98

CAP. 7: A DUPLICAO DO CUBO

Com efeito, se

1
x
y
= = ,
x
y
2

vemos que
x2 = y
e, portanto, multiplicando ambos os membros por x, obtemos
x3 = xy.
Mas como
xy = 2
temos que
x3 = 2.
No caso geral, se x e y so duas meias proporcionais entre a e b, temos
a
x
y
= =
x
y
b
e disso vemos que
x2 = ay
e que
xy = ab
e da segue-se que
x3 = axy = a2 b
e assim

x3
a2 b
b
= 3 = .
3
a
a
a

Apresentaremos agora sete solues do problema da duplicao do


cubo, quase todas baseadas em achar duas meias proporcionais entre
duas grandezas, usando construes que no podem ser efetuadas
somente com rgua e compasso e curvas que no podem ser traadas

principal1
2009/8/19
page 99
Estilo OBMEP

N SEC. 7.1: A MQUINA DE PLATO

99

usando somente estes dois instrumentos.

7.1

A Mquina de Plato

O filsofo grego Plato (viveu de 429 a 347 a.C.) tinha grande


interesse pela Matemtica e lhe atribua importncia particular. Gravitaram em torno dele excelentes matemticos, como, por exemplo,
rquitas, Eudoxo, Mencmo, Teeteto, entre outros.
bem conhecido que Plato desprezava construes mecnicas,
materiais (ver Van der Waerden, p. 162-163) em Matemtica. Assim, irnico que a soluo discutida a seguir seja conhecida como
mquina de Plato.

Figura 7.1: A mquina de Plato


A mquina de Plato (Figura 7.1) um dispositivo, ACDF , formado por partes rgidas, com AC e F D paralelas e CD perpendicular
a ambas. O segmento BE paralelo a CD e pode deslizar ao longo
de AC e de F D.
Para achar duas meias proporcionais entre ON = a e OM = b,
movimentamos ACDF de maneira que o segmento CD passe por M ,
C esteja sobre o eixo horizontal e fazemos BE deslizar at que passe
por N e B esteja sobre o eixo vertical, como mostrado na Figura 7.2.

principal1
2009/8/19
page 100
Estilo OBMEP
100

CAP. 7: A DUPLICAO DO CUBO

Figura 7.2: Meia proporcional entre ON = a e OM = b


Vemos que os tringulos N OB e M OC so semelhantes e, portanto,
OC
ON
=
.
OB
OM
Como os tringulos N OB e OCB tambm so semelhantes, temos
que
ON
OB
=
OB
OC
e assim chegamos a
ON
OB
OC
a
OB
OC
=
=
=
=
=
,
OB
OC
OM
OB
OC
b
o que mostra que OB e OC so de fato meias proporcionais entre a
e b.

7.2

A Mquina de Eratstenes

Considere trs placas retangulares AEZF , N M HJ e SGT L


(Figura 7.3), que podem deslizar sobre uma reta de maneira que a

principal1
2009/8/19
page 101
Estilo OBMEP

N SEC. 7.2: A MQUINA DE ERATSTENES

101

placa mdia (N M HJ), pode passar por traz da primeira (AEZF ),


e que a ltima (SGT L), deslize por traz da do meio. Suponha que
desejamos achar duas meias proporcionais entre a = AE e b = DT .

Figura 7.3: A mquina de Eratstenes


Traamos o segmento de reta AD e o prolongamos at a interseco, K, com a reta que passa por ET . Fazemos as placas deslizar,
como descrito acima, de maneira que o lado direito ZF da primeira placa (AEZF ), intercepte a diagonal N H da segunda placa
(N M HJ), exatamente sobre a reta AD, no ponto B, como mostra
na Figura 7.4.
De maneira semelhante, fazemos deslizar a terceira placa (HGT L)
de tal forma que o lado direito da segunda placa (JH) corte a diagonal
(ST ) exatamente sobre AD, no ponto C.
Afirmamos ento que BZ e HC so meias proporcionais entre AE
e DT , ou seja, que
AE
BZ
GH
=
=
.
BZ
GH
DT

principal1
2009/8/19
page 102
Estilo OBMEP
102

CAP. 7: A DUPLICAO DO CUBO

Figura 7.4: Meias proporcionais entre AE = a e DT = b


Com efeito, como os tringulos AEK e BZK so semelhantes,
temos
AE
KA
KE
=
=
.
BZ
KB
KZ

(7.1)

Temos tambm que os tringulos BZK e CHK so semelhantes,


e, portanto
BZ
KZ
KB
=
=
.
CH
KH
KC

(7.2)

Da semelhana dos tringulos CHK e DT K segue-se que


CH
KH
KC
=
=
.
DT
KT
KD

(7.3)

Da semelhana dos tringulos AZK e BHK decorre


AZ
KZ
KA
=
=
.
BH
KH
KB

(7.4)

principal1
2009/8/19
page 103
Estilo OBMEP
103

N SEC. 7.2: A MQUINA DE ERATSTENES

Alm disso, como os tringulos BHK e CT K so semelhantes,


vemos que
BH
KH
KB
=
=
.
CT
KT
KC

(7.5)

De 7.3 e 7.5 obtemos


CH
KH
KC
BH
KB
=
=
=
=
,
DT
KT
KD
CT
KC

(7.6)

KH
em comum.
KT
KB
em comum, segue-se que
Como 7.2 e 7.6 tm
KC

pois ambos tm

BZ
KZ
BK
BH
KH
CH
KC
=
=
=
=
=
=
.
CH
KH
KC
CT
KT
DT
DK
Como 7.1 e 7.4 tm

(7.7)

KA
em comum, segue-se que
KB

AE
KA
KE
AZ
KZ
=
=
=
=
.
BZ
KB
KZ
BH
KH

(7.8)

De 7.7 mantenhamos somente as razes que nos interessam:


BZ
KZ
CH
=
=
.
CH
KH
DT

(7.9)

De 7.8 mantenhamos somente as razes que nos interessam:


AE
KZ
=
.
BZ
KH
Como 7.9 e 7.10 tm

KZ
em comum, obtemos, enfim, que
KH

(7.10)

principal1
2009/8/19
page 104
Estilo OBMEP
104

CAP. 7: A DUPLICAO DO CUBO

AE
BZ
CH
=
=
,
BZ
CH
DT
o que queramos demonstrar.

(7.11)
2

O processo descoberto por Eratstenes pode ser usado para inserir


qualquer nmero de meias proporcionais entre a e b. Para inserir n
meias proporcionais, suficiente tomar n + 1 retngulos e proceder
como acima.

7.3

A Soluo de Nicomedes

O mtodo para duplicar o cubo encontrado por Nicomedes, que


viveu em torno de 240 a.C., verdadeiramente muito engenhoso.
Suponha que queremos inserir duas meias proporcionais entre
b = AB e a = BC (Figura 7.5).
Construa o retngulo ABCL. Seja D o ponto mdio de AB e
trace a reta que passa por L e D. Seja G seu ponto de interseco
com a reta que passa por C e B. Seja E o ponto mdio de BC e trace
a perpendicular a BC por E. Chame de F o ponto desta reta para o
qual BF = F C = AD.
Trace a reta s que passa por F e por G e a paralela a S que
passa por C. Por F trace a reta F HK construda de tal maneira
que HK = CF = AD. Trace a reta que passa pelos pontos K e L e
chame de M sua interseco com a reta definida por A e B.
Afirmamos que AM e CK so meias proporcionais entre a e b, ou
seja, que
BC
MA
CK
=
=
.
MA
CK
AB

(7.12)

Para compreender isso, devemos usar a Proposio II.6, dos

principal1
2009/8/19
page 105
Estilo OBMEP

N SEC. 7.3: A SOLUO DE NICOMEDES

105

Figura 7.5: Meias proporcionais entre a = BC e b = AB


Elementos de Euclides, cujo enunciado o seguinte:
Elementos, II-6: Se uma linha reta dividida em duas partes
iguais e se uma outra linha reta lhe adicionada, prolongando-a, o
retngulo determinado pela linha reta e pela reta adicionada igual,
se lhe for adicionado o quadrado sobre a metade da reta, ao quadrado
sobre a reta formada pela metade e pela reta adicionada.
Ou seja, na figura, a soma das reas do retngulo de base AD e
altura DB e do quadrado LEGH (que igual ao quadrado de lado
CB) igual rea do quadrado de lado CD (Ver Figura 7.6).
Voltemos justificao da construo de Nicomedes, aplicando
II-6 ao segmento BC cortado ao meio por E e prolongado at K:
BK KC + CE 2 = EK 2 .

(7.13)

principal1
2009/8/19
page 106
Estilo OBMEP
106

CAP. 7: A DUPLICAO DO CUBO

Figura 7.6: Elementos II.6


Adicionando EF 2 a ambos os lados desta igualdade e aplicando o
teorema de Pitgoras aos tringulos EF K e CEF , temos
BK KC + CF 2 = F K 2 .

(7.14)

Alm disso, devido semelhana dos tringulos AM L e M BK


temos que
AB
LK
CK
=
=
.
MA
ML
BC

(7.15)

1
Observe que BC = GC, devido congruncia dos tringulos
2
GBD e DAL.
Como AB = 2AD, temos
2 AD
2 CK
=
MA
GC

(7.16)

AD
CK
=
.
MA
GC

(7.17)

e assim,

principal1
2009/8/19
page 107
Estilo OBMEP

N SEC. 7.3: A SOLUO DE NICOMEDES

107

Como os tringulos GF K e CHK so semelhantes, temos


CK
HK
=
GC
FH

(7.18)

AD
CK
HK
=
=
.
MA
GC
FH

(7.19)

HK
AD
=
,
M A + AD
F H + HK

(7.20)

HK
AD
=
.
MD
FK

(7.21)

e da segue-se que

Temos ento que

e assim

Como HK = AD por construo, segue-se de 7.21 que M D = F K


e assim, de 7.14, temos
BK KC + CF 2 = F K 2 .

(7.22)

Usaremos mais uma vez Euclides II-6, agora aplicada ao segmento


BA, cujo ponto mdio D, e prolongado at M :
BM M A + AD2 = M D2 .

(7.23)

Como M D = F K, vemos que


BM M A + AD2 = F K 2 .
Comparando 7.22 e 7.24 vemos que

(7.24)

principal1
2009/8/19
page 108
Estilo OBMEP
108

CAP. 7: A DUPLICAO DO CUBO

BM M A = BK KC

(7.25)

BK
MA
=
.
BM
CK

(7.26)

de maneira que

Da semelhana dos tringulos M BK, M AL e LCK segue-se que


BK
AL
CK
=
=
.
BM
MA
LC

(7.27)

Finalmente, usando 7.26 e 7.27, temos que:


CK
MA
AL
=
=
.
LC
CK
MA

(7.28)

Como LC = AB, AL = BC, temos, enfim, que


BC
MA
CK
=
=
,
MA
CK
AB
o que queramos demonstrar.

(7.29)
2

Esta demonstrao encontra-se em Heath (1981). Daremos a seguir uma demonstrao analtica para a construo de Nicomedes
(veja Bos, 2001).
Na Figura 7.5, faa x = F H e y = CK. Ento, nos tringulos
retngulos EF K e EF C temos que
EF 2 + EK 2 = F K 2 ,

(7.30)

EF 2 + EC 2 = F C 2 .

(7.31)

principal1
2009/8/19
page 109
Estilo OBMEP
109

N SEC. 7.3: A SOLUO DE NICOMEDES

De 7.30 e de 7.31 vemos imediatamente que


F K 2 EK 2 = F C 2 EC 2 ,

(7.32)

e assim

1
b+x
2

1
a+y
2

2
1 2
1
b
a .
2
2

(7.33)

Disso, segue-se imediatamente que


ay + y 2 = bx + x2 .

(7.34)

a+y
x
= .
b+x
y

(7.35)

Logo

Os tringulos CHK e GF K so semelhantes e podemos portanto


escrever
HK
FH
=
,
CK
GC

(7.36)

ou seja
1
b
2 = x = b = x = a = y .
y
2a
y
a
x
b

(7.37)

Mas ento
a+y
x+b
a+y
y
=
=
= .
y
b
x+b
b
De 7.34 e 7.36 vemos que

(7.38)

principal1
2009/8/19
page 110
Estilo OBMEP
110

CAP. 7: A DUPLICAO DO CUBO

x
y
x
y
a
= = = =
y
b
y
b
x

(7.39)

a
x
y
= = .
x
y
b

(7.40)

e assim

Como, pela semelhana dos tringulos AM L e LCK, temos que


a
y
= ,
AM
b

(7.41)

AM = x,

(7.42)

vemos imediatamente que

o que conclui a demonstrao.


Na construo apresentada por Nicomedes (Figura 7.5), foi
necessrio construir a reta F HK tal que HK = CF = AD. Ou
seja, tivemos que ajustar o segmento de comprimento AD sobre o
segmento F K, a partir de K. Este tipo de construo, que no pode
ser efetuado com rgua e compasso, chamado de construo por
neusis ou por ajustamento. Ela utilizada em vrias solues dos
trs problemas clssicos, em particular por Arquimedes.

7.4

A Construo de rquitas

Vamos agora considerar uma das solues mais engenhosas do


problema de duplicar um cubo, devida a rquitas (viveu em torno
de 390 a.C.). Reproduzimos aqui a soluo como apresentada em
Teixeira (1995, p. 289-290). Embora o raciocnio de rquitas seja
puramente geomtrico (veja Van der Waerden, p. 151 ou Heath, 1981,

principal1
2009/8/19
page 111
Estilo OBMEP

N SEC. 7.4: A CONSTRUO DE RQUITAS

111

vol. 1, p. 246-249), usaremos seu equivalente analtico, mais familiar


para ns, habituados com a linguagem algbrica.
Sejam a e b os dois segmentos dados, com b < a, para os quais
desejamos construir duas meias proporcionais. Sejam OC 0 A uma circunferncia cujo dimetro OA0 igual a a e uma outra circunferncia
OBA, cujo dimetro igual a b e que est contida em um plano perpendicular ao plano da primeira circunferncia (veja a Figura 7.7).

Figura 7.7: Duplicao do cubo por rquitas


Considere o cilindro circular reto gerado pela circunferncia OCA0
e o toro gerado pela circunferncia OBA ao girar em torno da reta
OZ, perpendicular ao plano de OCA0 .
A interseco destas duas superfcies define uma curva, conhecida
como curva de rquitas, dada pelas equaes
x2 + y 2 = ax,
(x2 + y 2 + z 2 )2 = a2 (x2 + y 2 ).

principal1
2009/8/19
page 112
Estilo OBMEP
112

CAP. 7: A DUPLICAO DO CUBO

b
Seja =
e considere o cone cujo eixo a reta OA0 e cuja
a
geratriz forma com o eixo o ngulo . A equao deste cone
(x2 + y 2 + z 2 ) =

a2 2
x .
b2

(7.43)

Seja D a projeo de C sobre Ox. Ento a curva de rquitas


corta o cone no ponto B cuja projeo sobre o plano de OCA0 o
OC
b
ponto C da circunferncia tal que
= .
OB
a
Afirmamos que OC e OB so duas meias proporcionais entre a e
b. Com efeito,
OC 2 = a OD,
OB 2 = a OC,
b OB = a OD.
Disso, decorre que
OC 2 = b OD,
OB 2 = a OC,
e temos enfim que
a
OB
OC
=
=
.
OB
OC
b
2
Teixeira (1995, p. 290) comenta que,
esta soluo muito engenhosa e tem grande interesse
histrico, porque o mais antigo exemplo de soluo
de um problema de geometria plana usando geometria

principal1
2009/8/19
page 113
Estilo OBMEP

N SEC. 7.5: A SOLUO ACHADA POR MENCMO

113

espacial, e a curva usada a mais antiga curva reversa


conhecida.
Comentando a verso original, geomtrica, desta soluo, Van der
Waerden (p. 151) afirma que rquitas deve ter tido uma inspirao
divina para achar esta construo.

7.5

A Soluo Achada por Mencmo

Mencmo viveu em torno de 350 a.C. Em seu Comentrio sobre


o primeiro livro de Euclides, Proclus, no Sumrio de Eudemo, afirma
que Mencmo foi um aluno de Eudoxo e um membro dos filsofos e
matemticos em torno de Plato. Ele era irmo de Dinstrato o qual,
segundo Proclus, aperfeioou ainda mais a geometria.

Figura 7.8: Duplicao do cubo por Mencmo

principal1
2009/8/19
page 114
Estilo OBMEP
114

CAP. 7: A DUPLICAO DO CUBO

Se x e y so duas meias proporcionais entre a e b, temos que


x
y
a
= = .
x
y
b
Isso inteiramente equivalente s equaes simultneas,
y 2 = bx,

xy = ab.

Assim, para resolver o problema, suficiente achar a interseco das


parbolas e hiprboles definidas por estas equaes, respectivamente
(veja a Figura 7.8). As coordenadas deste ponto so as meias proporcionais que estamos procurando.
bvio que 7.5 tambm equivalente a
y 2 = bx,

x2 = ay.

(7.44)

Desta maneira, o problema pode tambm ser resolvido usando


duas parbolas cujos vrtices coincidem e cujos eixos so ortogonais.
Estas duas solues so descritas por Eutcio em seu comentrio do
Tratado sobre a esfera e o cilindro, de Arquimedes.
Nem todas as solues deste problema consistiam em inserir duas
meias proporcionais entre duas grandezas dadas. Isso pode ser verificado pela soluo devida a Diocles. claro que Mencmo no formulou sua soluo usando a representao analtica das parbolas ou
hiprboles, mas seu raciocnio equivalente a isso.

7.6

O Mtodo de Diocles

A soluo de Diocles se baseia na cisside, uma curva definida


como segue.
Seja uma circunferncia que passa pela origem O do sistema de

principal1
2009/8/19
page 115
Estilo OBMEP

N SEC. 7.6: O MTODO DE DIOCLES

115

coordenadas, tem seu centro sobre o eixo dos x e dimetro igual a


1. Sejam T = (1, 0) e r a reta vertical que passa por T . Seja P um
ponto qualquer sobre a circunferncia. A reta que passa por O e por
P intercepta r no ponto W . Tome o ponto M sobre OW tal que
OP = M W . A cisside o lugar geomtrico do ponto M quando P
percorre a circunferncia (veja a Figura 7.9, que no mostra toda a
cisside, mas somente a parte que nos interessa para a resoluo do
problema da duplicao do cubo).

Figura 7.9: Duplicao do cubo usando a cisside


A equao polar de uma circunferncia que passa pela origem, tem
raio R, e centro sobre o eixo dos x

= cos .
2R

principal1
2009/8/19
page 116
Estilo OBMEP
116

CAP. 7: A DUPLICAO DO CUBO

1
Como em nosso caso R = , a equao 7.6 se reduz a = cos .
2
1
Por outro lado, OW =
.
cos
Como OM = OW OP , a equao polar da cisside
=

1
sen2
cos =
.
cos
cos

Para achar a equao cartesiana da cisside temos que:


y2
y2
x2
x = cos = sen2 =
=
=
,
x2 + y 2
1 + tg 2
x2 + y 2
x2
tg 2

ou seja,
x(x2 + y 2 ) y 2 = 0.
Seja U (0, 2) e considere o ponto de interseco, Z = (a, b), da reta
T U com a cisside. A equao cartesiana da reta T U y = 2(1 x),
de maneira que b = 2(1 a), e assim, como Z pertence cisside,
a(a2 + b2 ) b2 = 0 = a3 + ab2 b2 = 0 =
b3
a3 = b2 (1 a) =
= 2a3 = b3 .
2

b
= 3 2. Assim, a equao cartesiana da
a

reta que passa por O e por Z y = 3 2x. Seja Q o ponto de interseco


dessa reta com a reta x = 1. Temos, ento, que as coordenadas de Q

so (1, 3 2). Se temos um cubo cuja aresta mede 1, para duplicar seu
volume, devemos achar a aresta y de um cubo que tem volume 2. Ou

seja, devemos ter y = 3 2. Vemos portanto que a ordenada do ponto


Q, achada acima, resolve o problema.
imediato ento que

principal1
2009/8/19
page 117
Estilo OBMEP
117

N SEC. 7.7: O MTODO DE HIERO

7.7

O Mtodo de Hiero

Descrevemos a seguir o mtodo proposto por Hiero


duas meias proporcionais entre os segmentos a e b.

para achar

Figura 7.10: Mtodo de Hiero para achar meias proporcionais


Construa o retngulo OACB, no qual AO = a e OB = b
(Figura 7.10). Seja D o centro do retngulo. Tome uma rgua que
passa por C e sejam E e F seus pontos de interseco com as retas
definidas por AO e OB respectivamente. Faa a rgua girar at que
DF = DE. Afirmamos ento que BF = x e AE = y so as duas
meias proporcionais entre a e b.

Matemtico e fsico grego, que viveu em torno de 60 d.C., em Alexandria.

principal1
2009/8/19
page 118
Estilo OBMEP
118

CAP. 7: A DUPLICAO DO CUBO

Com efeito, usando a semelhana dos tringulos F BC, CAE e


F OE, temos que
a
y
a+y
= =
.
x
b
b+x
Como DE 2 = DF 2 , segue-se que

1 2 1 2
1 2 1 2
y+ a + b = x+ b + a ,
2
4
2
4
de que obtemos y(a + y) = x(b + x). Desta ltima igualdade decorre
que
z
a+y
a
y
=
= = ,
y
b+x
x
b
e da vemos imediatamente que
x
y
a
= = .
x
y
b
Mais detalhes sobre a duplicao do cubo podem ser achados em
Heath (1981, vol. 1, p. 244-270). Uma explicao lcida da soluo de
rquitas encontra-se em Van der Waerden (p. 150-152), Heath (1981,
vol. 1, p. 246-249) e Teixeira (1995, p. 285-326), o qual apresenta um
total de 17 solues para a duplicao do cubo, incluindo os mtodos
propostos por Vite, Descartes, Fermat, Newton e Clairaut.

principal1
2009/8/19
page 119
Estilo OBMEP

Captulo 8

A Quadratura do Crculo
Como j dissemos, quadrar o crculo, ou seja, construir, com
rgua e compasso, um quadrado com rea igual rea do crculo
um problema bem natural, uma vez resolvido o problema de fazer a
quadratura de polgonos! Veremos agora como os matemticos gregos encontraram maneiras de resolver este problema usando curvas
e construes que no podem ser obtidas somente com rgua e compasso.
A origem do interesse grego nos problemas de quadratura pouco
conhecida. Segundo Zsab (2000), o problema primitivo do qual se
originaram todos os outros foi o da quadratura do retngulo. Aristteles afirma que a origem deste problema foi a procura da mdia geomtrica, mas que isso foi esquecido e que s foi preservado o problema.

8.1

A Quadratriz

Esta curva notvel resolve dois dos problemas clssicos: a


quadratura do crculo e a trisseco de um ngulo arbitrrio. Para
constru-la, suponhamos que no quadrado ABCD o lado AD gira com
119

principal1
2009/8/19
page 120
Estilo OBMEP
120

CAP. 8: A QUADRATURA DO CRCULO

movimento circular uniforme em torno de A at que coincide com o


lado AB. Ao mesmo tempo, o lado DC desce com velocidade constante at coincidir com AB. Os dois movimentos esto sincronizados
de maneira que ambos os lados, DC e AD coincidam com AB no
mesmo instante.

Figura 8.1: Duplicao do cubo com a quadratriz

A quadratriz o lugar geomtrico gerado pelas interseces destes


dois lados mveis. a curva DP Z da Figura 8.1. Ela foi inventada
por Hpias de Elis (viveu em torno de 420 a.C.), originariamente
em suas tentativas para trissectar o ngulo. Tudo indica que foi
Dinstrato (viveu em torno de 350 a.C.) quem pela primeira vez usou
esta curva para fazer a quadratura do crculo.
2a
Afirmamos que AZ =
, com a o comprimento do lado do

quadrado. Com efeito, sejam o ngulo P AZ, x = M P , y = AM e


AB = AD = DC = a. Ento, devido proporcionalidade dos dois
y
movimentos, temos que = k, com k a constante de proporcionali

principal1
2009/8/19
page 121
Estilo OBMEP
121

N SEC. 8.1: A QUADRATRIZ

dade. Quando =

, temos que
2
a
= k,
2

de maneira que
k=

2a

e podemos concluir que


=
Assim,

y
2a
= y =
.
2a

y
2a
y
= sen = =
=
.

sen
sen

Temos ento que

2a
.
0

AZ = lim = lim
0

Quando 0,
lim

= 1,
sen

e assim vemos que


AZ = =

2a
.

2a
imediato construir
Aps obter um segmento de comprimento

para fazer a quadratura do crculo. Com efeito, fcil dividir, usando


2a
somente rgua e compasso,
por 2a e, em seguida, tomar o inverso

1
de .

Um tratamento mais completo do problema da quadratura do crculo pode ser encontrado em Heath (1981, vol. I, p. 220-235). Uma
boa exposio encontra-se em Teixeira (1995, p. 362-384). A histria
detalhada do nmero pode ser lida em Beckmann (1977).

principal1
2009/8/19
page 122
Estilo OBMEP

Captulo 9

A Trisseco do ngulo
Voltamo-nos agora para o terceiro dos problemas clssicos, ou seja,
a trisseco de um ngulo qualquer. Este problema pode ser resolvido
de vrias maneiras. Acredita-se que Hpias de Elis, que viveu no sculo
V a.C. foi um dos primeiros a tentar resolver este problema, utilizando
curvas e construes que no podem ser efetuadas somente com rgua
e compasso.
Faremos, em primeiro lugar, uma digresso sobre as construes
por ajustamento ou por neusis.
Em uma construo por neusis deve-se ajustar um segmento dado
entre duas curvas dadas, com a exigncia de que o segmento passe
por um ponto dado. Nas palavras de Heath (1953, p. c):

Assim, uma linha reta tem que colocada entre duas


linhas ou curvas de maneira que passe por um ponto dado
e o segmento determinado sobre ela pelas interseces com
as linhas ou curvas seja igual a um comprimento dado.
Mostremos um exemplo de uma construo por neusis.
122

principal1
2009/8/19
page 123
Estilo OBMEP

N SEC. 9.1: A TRISSECO DO NGULO POR ARQUIMEDES

9.1

123

A Trisseco do ngulo por Arquimedes

A construo que mostraremos a seguir, que utiliza neusis, um


exemplo das vrias solues do problema da trisseco do ngulo
propostas por Arquimedes.

Figura 9.1: Trisseco do ngulo por Arquimedes

Suponha que desejamos trissectar o ngulo BOA. Tome uma reta


r que passa por B e, tendo o cuidado para que ela sempre passe por
B, movimente-a para que o segmento M N seja igual ao raio OM do
crculo. Isso exatamente o que se denomina uma construo por
neusis: Ajustamos um segmento (o raio OM ) entre o crculo e a linha
reta que passa por C e por A.
Observe que os tringulos N M O e M OB so issceles, de
b O = M ON
b . De maneira semelhante,
maneira que x = M N
cO = OBM
b .
BM
No tringulo N OM , o teorema do ngulo externo permite escrever
cO = M BO
b = 2 MN
b O. Aplicando o teorema do ngulo
que B M

principal1
2009/8/19
page 124
Estilo OBMEP
124

CAP. 9: A TRISSECO DO NGULO

externo ao tringulo BN O, vemos que


b = BN
b O + M BO
b = 3 BN
b O,
B OA
e vemos assim que com esta construo possvel dividir o ngulo
b em trs partes iguais.
B OA
2

9.2

A Trisseco do ngulo por Nicomedes

Examinemos agora outro exemplo de trisseco do ngulo, devido


a Nicomedes, no qual ele utiliza uma construo por ajustamento.
Suponha que desejamos trissectar o ngulo = AOB. Por B
levante uma reta perpendicular a OB e por A uma reta paralela a OB
(Figura 9.2). Trace uma reta por O e sejam P e C suas interseces
com AB e AC respectivamente. Mova-a de maneira que P C = 2OA.
b = AOB/3.
b
Afirmamos que P OB

Figura 9.2: Trisseco do ngulo por Nicomedes


Com efeito, seja D o ponto mdio de P C. Ento o tringulo
AP C est inscrito em um crculo de centro D e raio P D, e assim
b
b
P D = AD = DC = OA. Sejam = AOD
= ADO
e
b
b
= DAC = ACD. Aplicando o teorema do ngulo externo ao tringulo ADC vemos que = 2. Como AC e OB so paralelas, segue-se
b e portanto = 3.
que = P OB

principal1
2009/8/19
page 125
Estilo OBMEP

N SEC. 9.2: A TRISSECO DO NGULO POR NICOMEDES

125

A construo neusis usada neste problema pode ser efetuada


usando a conchide de Nicomedes, cuja definio dada a seguir.

Figura 9.3: Conchide de Nicomedes

Sejam K um ponto fixo, o polo da conchide, e uma linha reta SA,


denominada diretriz da conchide, e um comprimento fixo d, chamado
de distncia da conchide.
A conchide a curva gerada por P quando S se desloca sobre
SA e o comprimento do segmento P S permanece sempre igual a d
(Figura 9.3).
Para vermos como a conchide pode ser usada para a trisseco
do ngulo proposta por Nicomedes, trace a conchide com polo C,
diretriz AB e distncia igual a duas vezes OA. A interseco C desta
conchide com a paralela a OB que passa por A o ponto procurado,
como j foi mostrado.
A conchide pode tambm ser usada na duplicao do cubo feita
por Nicomedes feita na pgina 104. Com efeito, a interseco K da
conchide cujo polo F , diretriz CH e distncia AD com a linha reta
definida por G e C exatamente o ponto que achamos anteriormente
(compare com a Figura 7.5).

principal1
2009/8/19
page 126
Estilo OBMEP
126

9.3

CAP. 9: A TRISSECO DO NGULO

Trisseco

do

ngulo

Usando

Quadratriz

Figura 9.4: Trisseco do ngulo com a quadratriz


Como j dissemos, a quadratriz foi utilizada em primeiro lugar
para resolver o problema da trisseco do ngulo. Como acontece
frequentemente em Matemtica, s vezes uma ideia que permite atacar com sucesso um problema mostra, posteriormente, ser capaz de
resolver outros problemas. Isso aconteceu com a quadratriz, pois foi
logo depois observado que ela tambm permite resolver o problema
da quadratura do crculo.
Suponha que desejamos dividir o ngulo P AZ em trs partes
iguais. Por P , trace a paralela a AZ que intercepta AD em M (Figura
9.4). Divida o segmento AM em trs partes iguais e seja AU uma
dessas partes. Por U trace a paralela a AZ que corta a quadratriz no
ponto T . Por proporcionalidade, o tempo gasto por P para chegar a
T igual ao tempo gasto por M para chegar a U , e igual a 2/3 do
b = 3 T AZ.
b
tempo que P gasta para chegar a Z. Assim, P AZ

principal1
2009/8/19
page 127
Estilo OBMEP

N SEC. 9.3: A TRISSECO DO NGULO USANDO AQUADRATRIZ

127

Esta a soluo mais antiga que conhecemos para o problema


da trisseco do ngulo. Um tratamento mais amplo deste problema
pode ser encontrado em Heath (1981, vol. I, p. 235-244). Para uma
discusso geral de problemas resolvidos usando construes neusis, ver
Heath (1953, Captulo V, p. c-cxxii) e um tratamento mais didtico
em Yates (1971). Vrias outras construes podem ser achadas em
Teixeira (1995, p. 327-358). Consultar tambm Dudley (1987) em que
se encontram muitos processos aproximados ou exatos para trissectar
um ngulo.

principal1
2009/8/19
page 128
Estilo OBMEP

principal1
2009/8/19
page 129
Estilo OBMEP

Referncias Bibliogrficas
[1] BECKMANN, Peter. A history of pi. 4th ed. Boulder. Colorado:
Golem Press, 1977.
[2] BOS, Henk J. M. Redefining geometrical exactness: Descartes
transformation of the early modern concept of construction. New
York: Springer, 2001.
[3] BUNT, Lucas N. H.; Phillip S. Jones; Jack D. Bedient. The
historical roots of elementary mathematics. New York: Dover,
1988.
[4] CARVALHO, Joo Pitombeira de. Trs excurses pela histria
da Matemtica. Rio de Janeiro: Intermat, 2008.
[5] COURANT, Richard; ROBBINS, Herbert. What is mathematics? New York: Oxford University Press, 1996.
[6] BKOUCHE, Rudolf; DELATTRE, Jolle. Pourquoi la rgle et le
compas. Comission Inter-IREM. Histoire de problmes, Histoire
des Mathmatiques. Paris: Ellipses, 1993.
[7] DUDLEY, Underwood. A budget of trisections. New York:
Springer, 1987.
[8] EVES, Howard. Introduo Histria da Matemtica. Campinas: Editora da UNICAMP, 1995.
129

principal1
2009/8/19
page 130
Estilo OBMEP
130

REFERNCIAS BIBLIOGRFICAS

[9] HADLOCK, Charles Robert. Field theory and its classical


problems. Washington, DC: Mathematical Association of America, 1978. (Carus Mathematical Monographs 18.)
[10] HEATH, Thomas L. The works of Archimedes. New York: Dover,
1953.
[11] HEATH, Thomas L. The Thirteen books of Euclids Elements,
vols. 1, 2, 3. New York: Dover, 1956.
[12] HEATH, Thomas L. A history of Greek mathematics, 2 v. New
York: Dover, 1981.
[13] KATZ, Victor J. A History of Matematics an introduction. New
York: Harper Collins, 1993.
[14] KLEIN, Felix. Famous problems of elementary geometry, translated by Wooster Woodruff Beman e David Eugene Smith. New
York: G. E. Stechert & Co. 1930.
[15] KNORR, Wilbur Richard. The evolution of the euclidean elements: a study of the theory of incommensurable magnitudes
and its significance for early Greek geometry. Dordrecht [u.a.],
Holanda: Reidel, 1975.
[16] KNORR, Wilbur Richard. The ancient tradition of geometric
problems. Boston, Basel, Stuttgart: Birkhuser, 1986.
[17] RAIGORODSKI, Andrei M. The Borsuk partition problem: the
seventieth anniversary. The mathematical intelligencer, vol. 26,
n. 3, 2004, p. 4-12.
[18] SANTOS, Christovam dos. Trisseco de um ngulo. Revista
brasileira de Matemtica, ano 2, n. 5, jan./1931, p. 43-47.

principal1
2009/8/19
page 131
Estilo OBMEP

REFERNCIAS BIBLIOGRFICAS

131

[19] SCRIBA, Christoph J. On the so-called Classical Problems


in the History of Mathematics. Ivor Grattan-Guinness (Ed.)
Cahiers dHistoire & de Philosophie des Sciences, n. 21, 1987,
History in mathematics education. Proceedings of a Workshop
held at the University of Toronto, Canada, July August 1983.
Paris: Belin, 1987.
[20] SIMSON, Robert. Elementos de Euclides. Dos seis primeiros
livros, do undcimo e duodcimo, da verso latina de Frederico
Commandino, addicionados e ilustrados por Roberto Simson,
Professor de Mathematica na Academia de Glasgow. Coimbra:
Real Imprensa da Universidade, 1773. Com privilgio real.
[21] SZAB, Arpad. Laube des mathmatiques grecques. Paris:
Vrin, 2000.
[22] TEIXEIRA, Francisco Gomes. Trait des courbes spciales
planes et gauches, vol III. Paris: Jacques Gabay, 1995. 1. ed.
Coimbra: Universidade de Coimbra, 1909.
[23] VAN DER WAERDEN, B. L. Science Awakening I. Third edition. Grnigen: Wolters Noordhoff, s/d.
[24] VAN DER WAERDEN, B. L. Geometry and algebra in ancient
civilizations. New York: Springer Verlag, 1983.
[25] VAN DER WAERDEN, B. L. A History os algebra, from alKhwarizmi to Emmi Noether. New York: Springer Verlag, 1985.
[26] WANTZEL, Pierre. Recherches sur les moyens de reconnatre si
un problme de gomtrie peut se resoudre avec rgle et compas.
Journal de Mathmatiques, 2, 366-372, 1837.
[27] YATES, Robert C. The trisection problem. Reston, Virginia:
The National Council of Teachers of Mathematics, 1971. Classics
in mathematics education, vol. 4.

principal1
2009/8/19
page 132
Estilo OBMEP

principal1
2009/8/19
page 133
Estilo OBMEP

A Matemtica dos Cdigos


de Barras

Francisco Csar Polcino Milies

133

principal1
2009/8/19
page 134
Estilo OBMEP

principal1
2009/8/19
page 135
Estilo OBMEP

Prefcio
Em 1977 fiz meu primeiro estgio de pesquisa no exterior,
no Canad. Foi l, na biblioteca da universidade em que estagiava,
que vi pela primeira vez a utilizao de cdigos de barras, usados
para catalogar os livros do acervo. Logo depois, pude testemunhar o
seu emprego nos caixas de supermercados e grandes magazines.
Parecia coisa de mgica. Fiquei admirado de ver como agilizavam os
atendimentos e diminuiam o tamanho das filas.
Hoje em dia, os cdigos de barras so de uso rotineiro e nos deparamos com eles continuamente no nosso dia a dia. No somente
esto presentes nos diversos artigos que compramos, como aparecem
tambm, em verses bem mais longas, em recibos, contas a pagar e
inmeras outras aplicaes.
Mesmo assim, eles esto completamente ausentes das nossas salas
de aula. Sua estrutura muito simples e certamente eles poderiam
ser usados para motivar o estudo da alguns aspectos da teoria da
divisibilidade e para plantear questes instigantes.
A exposio que se segue tem por objetivo apresentar os conceitos e ideias bsicas ligadas a estes cdigos. Vamos descrever como
atribudo um nmero a cada produto, como feita a traduo de
nmeros para barras de espessura varivel e, finalmente, vamos discutir um dos aspectos mais interessantes da teoria: como se pode

135

principal1
2009/8/19
page 136
Estilo OBMEP
136
estruturar o cdigo de modo que um clculo simples (em geral efetuado automaticamente pela caixa que registra a compra) pode detectar se foi cometido algum erro de leitura. Partes deste trabalho
j foram publicadas na forma de artigos, na Revista do Professor de
Matemtica.
claro que este um tpico em contnuo desenvolvimento, que
est relacionado com outras reas da matemtica atual e suas aplicaes, tais como a moderna criptografia ou a teoria de cdigos corretores de erros. Esperamos que a leitura destas notas possa servir de
estmulo para que o leitor continue seus estudos, aprofundando-se nos
assuntos aqui tratados.

Csar Polcino Milies


Instituto de Matemtica e Estatstica
Universidade de So Paulo.

principal1
2009/8/19
page 137
Estilo OBMEP

Sumrio
Introduo

139

Um Pouco de Histria . . . . . . . . . . . . . . . . . . . . . 141


10 Cdigo de Barras

149

10.1 Escrevendo com Barras . . . . . . . . . . . . . . . . . . 149


10.2 A Deteco de Erros . . . . . . . . . . . . . . . . . . . 155
10.3 Outros Cdigos Numricos . . . . . . . . . . . . . . . . 160
11 Cdigos sobre Grupos

171

11.1 O Grupo Dihedral . . . . . . . . . . . . . . . . . . . . 171


Referncias Bibliogrficas

181

137

principal1
2009/8/19
page 138
Estilo OBMEP

principal1
2009/8/19
page 139
Estilo OBMEP

Introduo
Hoje em dia, muitos produtos so identificados com um cdigo
numrico. O progresso da tecnologia, que tornou relativamente baratos e acessveis aparelhos de leitura ptica e computadores, tornou
tambm o uso deste tipo de cdigos bastante frequente. Por exemplo,
os produtos que compramos num supermercado esto identificados
por um cdigo de barras, como o que mostramos na Figura 9.5. Ele
no mais do que um nmero, assignado ao produto para sua identificao, escrito de forma a permitir uma leitura rpida no caixa. Note
que imediatamente abaixo das barras, aparece o mesmo nmero escrito em algaritmos correntes, de forma que o leitor humano tambm
possa ler o nmero.

Figura 9.5: Cdigo de barras

139

principal1
2009/8/19
page 140
Estilo OBMEP
140

Porm, algumas vezes acontece que, ao passar um produto pela


leitora tica (por exemplo, quando a embalagem est mida ou enrugada), esta no consegue realizar a leitura. O que vemos ento a
pessoa que est no caixa tenta passar o produto em sentido contrrio,
ou inverte o produto, de modo que o cdigo de barras fique de cabea
para baixo, e tentar pass-lo mais uma vez. Se nem assim der certo,
ento ele prprio l o cdigo e o digita rapidamente.
Naturalmente, estas atitudes sugerem algumas perguntas. Em
primeiro lugar, uma vez que o desenho das barras totalmente
simtrico para a mquina, que o l usando um feixe de luz transversal, ao pass-lo de ponta cabea, ela no deveria ler o nmero na
ordem contrria? E, o que pior, o operador do caixa, ao digitar o
nmero rapidamente, no poderia cometer um erro e ns acabarmos
pagando por um produto muito mais caro do que aquele que estamos
comprando?
Na verdade, isso no ocorre. Tanto quando lido numa ordem,
como na ordem contrria, o cdigo sempre interpretado de forma
correta. Mais ainda, quando o operador comete algum erro de digitao e todos ns j vimos isso acontecer algumas vezes a mquina
simplesmente emite um som, para avisar que houve um erro!
O objetivo destas notas explicar como e porqu isso acontece.
Comearemos contando um pouco da histria destas ideias na prxima
seo. No captulo seguinte vamos explicar como feita a traduo
dos nmeros em termos de barras e como a leitora distingue entre
esquerda e direita; na seo 10.2 vamos explicar as ideias matemticas que fazem com que eventuais erros possam ser detectados. No
captulo final, mostraremos brevemente como estas ideias podem ser
extendidas para se obter cdigos mais sofisticados.

principal1
2009/8/19
page 141
Estilo OBMEP
141

Um Pouco de Histria

A ideia de utilizar mquinas para realizar clculos , na


verdade, bastante antiga. J em 1642, Blaise Pascal construiu a
primeira mquina de calcular de que se tem notcia. Ela funcionava
com engrenagens mecnicas e era capaz de realizar apenas somas. Em
1694, Wilhelm Leibniz aprimorou o invento de Pascal e criou uma
mquina capaz de realizar tambm multiplicaes. Do ponto de vista
destas notas, o que realmente interessante a forma de transmitir
dados mquina j que, praticamente desde o comeo da automao,
isto foi feito com cartes perfurados que so antepassados diretos dos
cdigos de barras.
De forma muito esquemtica, os estgios do processo que nos interessa so os seguintes:
Em 1728, B. Bouchon concebeu a ideia de cifrar informaes
em folhas de papel perfurado. A descoberta verdadeiramente
importante veio em 1801, quando Joseph-Marie Jacquard
(1752-1834) construiu um tear que era comandado por cartes
perfurados e que foi, talvez, a primeira mquina programvel.
O tear de Jacquard era capaz de realizar todos os movimentos
necessrios e ele foi o primeiro a produzir tecidos com padres
figurativos. Essa mquina deu mpeto revoluo tecnolgica
da era industrial e foi a base para o desenvolvimento do moderno
tear automtico.
Em 1857, Sir Charles Wheatstone utilizou fitas de papel
para armazenar dados, seguindo o mesmo princpio bsico dos
cartes perfurados, mas com a vantagem de poder alimentar
dados de forma contnua.

principal1
2009/8/19
page 142
Estilo OBMEP
142
Em 1822, Charles P. Babbage (1792-1871), um professor de
matemtica de Cambridge e um dos fundadores de Analytical
Society (grupo de professores que modificaria o ensino da matemtica na Inglaterra e que tiveram influncia fundamental na
criao da lgebra abstrata) inventou um instrumento de clculo
mais sofisticado, que denominou Mquina Diferencial. Nessa
poca, ele observou que as operaes matemticas repetitivas
poderiam ser desenvolvidas com mais agilidade e confiabilidade
pelas mquinas que pelos homens. Mais adiante, em 1833, projetou uma Mquina Analtica, isto , uma mquina capaz de
executar todas as operaes aritmticas, de fazer comparaes
e analisar seus prprios resultados, que era programada atravs
de cartes perfurados. Ele foi o primeiro a perceber que uma
mquina de computar deveria ter um dispositivo de entrada,
uma memria (que ele chamou de mill = moinho) e um dispositivo de sada. Sua mquina, em particular, seria alimentada
por duas sries de cartes perfurados: uma com os dados e
outra com as operaes a serem executadas. Por causa disto,
ele considerado o pai do computador digital.
Suas ideias despertaram o interesse de Ada August, condessa de
Lovelace e filha de Lord Byron, que foi a primeira programadora
da histria. No perodo de 1842-1843 ela traduziu do italiano um
artigo de Luigi Menebrea sobre a mquina da Babbage e, num
apndice, detalhou um mtodo completo para calcular nmeros
de Bernoulli com a mquina.
Babbage conseguiu convencer o governo britnico a financiar seu
projeto mas, apesar dos esforos de anos e de vrios investimentos governamentais, a mquina jamais chegou a ser construda.
De acordo com o plano original, ela seria movida a vapor e de
tamanho maior do que uma locomotiva.

principal1
2009/8/19
page 143
Estilo OBMEP
143
A prxima personagem importante nesta histria Hermann
Hollerith (1860-1951) que obteve um doutorado em estatstica na Columbia University em 1879 e, logo em seguida, foi
empregado pelo Bureau de Censos dos EUA para trabalhar
com seu professor, William P. Trowbridge, no censo de 1880.
Foram necessrios dez anos de trabalho para tabular completamente todos os dados recolhidos. Durante este perodo, Hollerith deu tambm aulas durante algum tempo no Massachussets
Institute of Technology e trabalhou no Escritrio de Patentes de
Washington. Sobretudo, ele empregou seu tempo projetando
uma mquina que pudesse tabular dados automaticamente.
Para isso, ele utilizou novamente a ideia dos cartes perfurados
de Jacquard, escrevendo dados em oito colunas que utilizavam
o sistema de numerao binria. Esses cartes eram ento lidos
por sua mquina que utilizava sensores eltricos. Quando foi
realizado um novo censo, em 1890, apesar dele ser mais sofisticado e coletar mais dados que o anterior, a inveno de Hollerith
pode tabular todos os dados em apenas seis semanas.
Sua criao teve sucesso imediato e ele deixou seu emprego
para fundar uma companhia dedicada ao desenvolvimento de
mquinas semelhantes, a Tabulating Machine Company que,
com o decorrer dos anos, se transformou na atual IBM.

O desenvolvimento de computadores eletrnicos ganhou mais


fora a partir da segunda guerra mundial, quando foi percebido
seu potencial estratgico.
Muitos autores consideram que o Atanasoff-Berry Computer
(ABC), desenvolvido nos EUA no perodo 1937-42 o primeiro
computador eletrnico digital. Porm, ele no tinha capacidade
de programao geral e servia apenas para resolver sistemas li-

principal1
2009/8/19
page 144
Estilo OBMEP
144
neares, alm de ter outras limitaes tcnicas.
Em 1941 os alemes desenvolveram o Z3, desenhado por
Konrad Zuse. Era eletromecnico, mas com objetivos mais gerais e totalmente programvel mediante fitas perfuradas.
Em 1944, Tommy Flowers desenhou, na Inglaterra, o computador Colossus, totalmente eletrnico, concebido para decodificar
mensagens interceptadas aos alemes.
O computador mais famoso deste perodo foi o ENIAC
(Electronic Numerical Integrator and Computer), desenhado
por John Mauchly e J. Presper Eckert, da Universidade de
Pennsylvania e construdo na Escola Penn Moore de Engenharia
Eletrnica daquela universidade. Embora ele fosse um computador de propsitos gerais, ele foi desenhado originalmente
para calcular tabelas de fogo de artilharia para o Laboratrio
de Pesquisas Balsticas, durante a guerra, mas sua construo
s foi completada trs meses aps o fim da guerra. Os primeiros
problemas computados pelo ENIAC foram relacionados construo da bomba de hidrognio. Tanto a entrada quanto a sada
de dados era feita atravs de cartes perfurados.

O aparelho pesava 27 toneladas, usava 17 468 vlvulas e precisou, para sua construo, de mais de cinco milhes de soldas
feitas a mo. Ocupava todo um galpo e consumia 150 kW
de energia eltrica. Costuma-se dizer que, quando estava em
uso, provocava apages na cidade de Pennsylvania mas isto no
deve ser verdade pois possua alimentao independente da rede
eltrica. Quando estava em operao, elevava a temperatura do
local a 50 graus. Ele foi desativado em 2 de outubro de 1955.

principal1
2009/8/19
page 145
Estilo OBMEP
145

Figura 9.6: O computador ENIAC

Outro computador de grande porte construdo nesse perodo foi


o Mark I. O projeto, concebido por Howard Aiken da Universidade de Harvard, iniciou-se em 1939 mas foi concludo apenas
em 1943, na IBM. Foi transladado a Harvard onde foi mostrado
publicamente, pela primeira vez, em 1944 e foi batizado oficialmente como Harvard-IBM Automatic Sequence Controlled
Calculator (ASCC). Media 15.5 m de comprimento, 2.40 m de
altura e aproximadamente 60 cm de largura.

A partir de ento, os progressos da tecnologia permitiram diminuir gradativamente o tamanho (e o custo!) dos computadores at
populariz-los definitivamente. Foi tambm a tecnologia que permitiu
usar feixes de luz e scanners para transmitir dados direta e rapidamente aos computadores, criando assim condies para a utilizao
da codificao que nos interessa estudar aqui.

principal1
2009/8/19
page 146
Estilo OBMEP
146

Cdigos de Barras

A primeira patente de um cdigo de barras foi atribuda em 1952 a


Joseph Woodland e Bernard Silver. Seu cdigo consistia num padro
de circunferncias concntricas de espessura varivel. Ao dar entrada
ao pedido de patentes, eles descreviam seu invento como uma classificao de artigos atravs de identificao de padres.
Em torno de 1970, uma firma de assessoria, a McKinsey & Co.,
junto com a Uniform Grocery Product Code Council1 definiu um
formato numrico para identificar produtos e pediu a diversas companhias que elaborassem um cdigo adequado. Dentre as firmas contactadas, a que acabou apresentando a proposta vencedora foi a IBM
e o cdigo foi criado por George J. Laurer.2

Figura 9.7: George J. Laurer

O cdigo proposto, formalmente aceito em maio de 1973, passou a


1

A palavra Grocery em ingls, equivale aproximadamente a nossa expresso


secos e molhados.
2
Estes dados, bem como a foto do autor, foram obtidos no site do prprio
Laurer, que diz ter tido a colaborao de outras duas pessoas, mas no cita os
nomes. O endereo do site :
http://www.bellsouthpwp.net/l/a/laurergj/upc_work.html

principal1
2009/8/19
page 147
Estilo OBMEP
147
ser conhecido como cdigo UPC (Universal Product Code) e foi adotado nos Estados Unidos e Canad. Ele consistia de uma sequncia
de 12 dgitos, traduzidos para barras da forma que analizaremos detalhadamente no prximo captulo. Existem vrias verses sucessivas
do UPC, com pequenas modificaes. Posteriormente foi solicitado a
Laurer que ampliasse o cdigo, para permitir uma maior difuso do
sistema, de modo a identificar tambm o pas de origem de cada produto classificado. Baseado no UPC-A, ele acabou criando um novo
cdigo, com 13 dgitos, que foi adotado em dezembro de 1976 com
o nome EAN (European Article Numbering system). Alguns pases
adotam este mesmo sistema, dando-lhe outro nome. Por exemplo, no
Japo o sistema conhecido como JAN (Japanese Article Numbering
system).

principal1
2009/8/19
page 148
Estilo OBMEP

principal1
2009/8/19
page 149
Estilo OBMEP

Captulo 10

Cdigos de Barras
10.1

Escrevendo com Barras

Vamos estudar inicialmente o cdigo UPC, que mais simples.


Se observamos o cdigo de barras da Figura 10.1, notamos imediatamente que ele formado por listras brancas e pretas alternadas, de
espessura varivel. H, na verdade, quatro espessuras possveis para
essas listras, que podem ser classificadas como finas, mdias, grossas
ou muito grossas.

Figura 10.1: Cdigo UPC

149

principal1
2009/8/19
page 150
Estilo OBMEP
150

CAP. 10: CDIGOS DE BARRAS

Vamos utilizar o smbolo 0 para indicar uma listra branca fina, o


smbolo 00 para uma listra branca mdia, 000 para uma listra branca
grossa e 0000 para uma listra muito grossa. Da mesma forma, vamos
representar por 1, 11, 111 e 1111, uma listra preta fina, mdia, grossa
ou muito grossa, respectivamente.
Assim, as primeiras quatro listras da figura (sem contar, claro as
listras que servem de limite e que aparecem mais compridas na figura),
que so uma listra branca grossa, uma preta mdia, uma branca fina
e uma preta fina respectivamente, podem ser representadas pela sequncia 0001101.
Como j dissemos, o cdigo de barras representa uma srie de
nmeros. A cada nmero lhe assignado um espao de espessura fixa,
que corresponde sempre a uma sequncia de sete dgitos iguais a 1 ou
0. Por exemplo, a sequncia 0001101 que achamos acima representa
o nmero 0, o primeiro do cdigo da figura. O seguinte nmero do
cdigo, o 7, representado pela sequncia 0111011.
Agora que comeamos a compreender a forma de escrever com
barras, j podemos responder primeira das nossas perguntas: como
a leitora distingue direita da esquerda, quando o artigo pode ser
passado em uma ou outra direo?
A resposta muito engenhosa e tambm bastante simples. Os
dgitos so codificados de maneira diferente quando esto do lado
direito ou do esquerdo do cdigo de barras. Isto feito conforme a
seguinte tabela:

principal1
2009/8/19
page 151
Estilo OBMEP
151

N SEC. 10.1: ESCREVENDO COM BARRAS

dgito

do lado esquerdo

do lado direito

0001101

1110010

0011001

1100110

0010011

1101100

0111101

1000010

0100011

1011100

0110001

1001110

0101111

1010000

0111011

1000100

0110111

1001000

0001011

1110100

Note que a codificao de um dado nmero, direita, se obtm da


sua codificao esquerda, trocando cada 0 por 1 e reciprocamente.
Agora, o mecanismo de reconhecimento fica claro se notamos que
cada sequncia do lado esquerdo tem um nmero mpar de dgitos
iguais a 1 e, consequentemente, cada uma das que esto direita tem
um nmero par . Assim, verificando a paridade de cada sequncia de
sete dgitos, a mquina sabe imediatamente de que lado est lendo
o cdigo.

A elaborao do cdigo EAN se deparou com um problema bastante delicado. Era necessrio adicionar um dgito cada cdigo, de
modo a permitir a identificao do pas de origem do produto, mas
se desejava fazer isto de uma forma tal que a mesma mquina leitora
pudesse ler indistintamente cdigos UPC e EAN.
Se observamos a figura a seguir, que representa o mesmo cdigo
numrico escrito em ambos sistemas, veremos que, a primeira vista,
parecem diferentes pois, no nmero escrito para o leitor humano, vemos que h um 0 a mais, escrito no incio da sequncia. Porm, se

principal1
2009/8/19
page 152
Estilo OBMEP
152

CAP. 10: CDIGOS DE BARRAS

observamos o cdigo de barras, vemos que exatamente o mesmo.

Figura 10.2: Os cdigos UPC-A e EAN-13


O que foi feito o seguinte. Os pases que utilizavam o cdigo
UPC antigo, EUA e Canad, so identificados com um 0, na frente,
e o resto da codificao feita utilizando-se o sistema anterior.
Para outros pases, os primeiros dois ou trs dgitos, identificam o
pas. Por exemplo, o cdigo de barras de todos os produtos produzidos
no Brasil comea com a sequncia 789, que a que identifica o pas.1
Como era necessrio adicionar um dgito e tambm manter o mesmo
padro de tamanho do cdigo de barras, para no ter que modificar
todas as leitoras, a ideia utilizada foi fazer com que o novo dgito
estivesse implcito na forma de escrita de todos os outros. Para isso,
no foi modificada a codificao do lado direito (permitindo assim que
as leitoras continuassem a identificar o lado correspondente) mas a
codificao do lado esquerdo varia, dependendo do dgito inicial.

Uma tabela completa, com os nmeros identificatrios de cada pas, pode ser
encontrada na pgina internet http://www.barcodeisland.com/ean13.phtml

principal1
2009/8/19
page 153
Estilo OBMEP
153

N SEC. 10.1: ESCREVENDO COM BARRAS

Um dgito do lado esquerdo pode ser agora codificado com um


nmero par ou mpar de dgitos iguais a 1, de acordo com a seguinte
tabela:
dgito

lado esquerdo mpar

lado esquerdo par

lado direito

0001101

0100111

1110010

0011001

0110011

1100110

0010011

0011011

1101100

0111101

0100001

1000010

0100011

0011101

1011100

0110001

0111001

1001110

0101111

0000101

1010000

0111011

0010001

1000100

0110111

0001001

1001000

0001011

0010111

1110100

Finalmente, para cada dgito inicial escolhe-se uma alternncia


diferente de pares e mpares de acordo com o seguinte critrio:
1o

2o

3o

4o

5o

6o

mpar

mpar

mpar

mpar

mpar

mpar

mpar

mpar

par

mpar

par

par

mpar

mpar

par

par

mpar

par

mpar

mpar

par

par

par

mpar

mpar

par

mpar

mpar

par

par

mpar

par

par

mpar

mpar

par

mpar

par

par

par

mpar

mpar

mpar

par

mpar

par

mpar

par

mpar

par

mpar

par

par

mpar

mpar

par

par

mpar

par

mpar

Dgito inicial

principal1
2009/8/19
page 154
Estilo OBMEP
154

CAP. 10: CDIGOS DE BARRAS

Vamos ver um exemplo. Uma barra de cereais produzida no Brasil


identificada pelo cdigo 7895000266241. Como corresponde, comea
com a sequncia 789, de modo que o primeiro dgito, que estar implcito na codificao dos demais, sete. Consequentemene, deve-se
usar, do lado esquerdo, a seguinte ordem de codificao (obtida na
tabela acima):
mpar, par, mpar, par, mpar, par.
Consultando ento a tabela de codificao do EAN-13 obtemos:
8 7 0110111 9 7 0010111 5 7 0110001
0 7 0100111 0 7 0001101 0 7 0100111
Para os dgitos do lado direito no temos que nos preocupar com
paridade, e obtemos, diretamente da tabela, a seguinte codificao:
2 7 1101100 6 7 1010000 6 7 1010000
2 7 1101100 4 7 1011100 1 7 1100110
Por tanto, o cdigo de barras correspondente :

Figura 10.3:

principal1
2009/8/19
page 155
Estilo OBMEP

N SEC. 10.2: A DETECO DE ERROS

155

Um ltimo comentrio respeito deste cdigo. Como j dissemos,


os primeiros dois ou trs dgitos do cdigo de barras (dependendo do
caso) servem para identificar o pas de origem do produto. Os cinco
ou quatro dgitos que restam, at as barras centrais, identificam o
fabricante. Os primeiros cinco dgitos do lado direito identificam o
produto especfico, desse fabricante. O ltimo dgito, chamado dgito
de verificao, adicionado no final do processo de elaborao do
cdigo, de acordo com um mtodo que veremos adiante.
Falta ainda responder a nossa segunda pergunta: como faz a
mquina para detectar quando um operador apressado comete um
erro de digitao? Isto ser o assunto da nossa prxima seo.

10.2

A Deteco de Erros

Para compreender como funciona o processo de deteco de erros


precisamos entender, inicialmente, como se atribui a cada produto, o
dgito de verificao.
Suponhamos que um determinado produto est identificado, no
sistema EAN-13, por uma dada sequncia de dgitos a1 a2 . . . a12 a13 .
Como os primeiros dgitos identificam o pas de origem, o fabricante
e o produto especfico, os primeiros doze dgitos da sequncia, esto determinados naturalmente, por um mtodo-padro, a cargo de
uma autoridade classificadora em cada pas. Denotaremos o dcimo
terceiro dgito, de verificao, por x.
Para facilitar nossa exposio, vamos escrever esta sequncia como
um vetor
= (a1 , a2 , . . . , a11 , a12 , x).

principal1
2009/8/19
page 156
Estilo OBMEP
156

CAP. 10: CDIGOS DE BARRAS

O sistema EAN-13, se utiliza de um vetor fixo, que chamaremos,


vetor de pesos que :
w = (1, 3, 1, 3, 1, 3, 1, 3, 1, 3, 1, 3, 1).
Calcula-se, ento o produto escalar de ambos vetores:
= (a1 , . . . , a12 , x) (1, 3, 1, 3, 1, 3, 1, 3, 1, 3, 1, 3, 1) =
= a1 + 3a2 + a3 + 3a4 + a5 + 3a6 + a7 + 3a8 + a9 + 3a10 + a11 + 3a12 + x.
Agora, o dgito de verificao x se escolhe de forma tal que a soma
acima seja mltiplo de 10, isto , tal que
= 0 (mod 10).
Por exemplo, no caso do cdigo da Figura 10.3, os nmeros que
indicam o pas de origem, o fabricante e o produto so 789500026624.
Vamos ver como foi determinado o dgito de verificao. Chamando
este dgito de x e fazendo o produto escalar com o vetor de pesos,
temos:
7+(38)+9+(35)+0+(30)+0+(32)+6+(36)+2+(34)+x =
= 99 + x.
Consequentemente, deve-se tomar x = 1.
Vejamos agora um exemplo de como funciona a deteco de erros.
Um livro do autor [14] recebeu o cdigo de barras 9781402002380. Suponhamos que, por um erro de digitao no quarto dgito, este nmero
transmitido como = 9782402002380. Ao fazer a verificao de
leitura, o computador que recebeu a informao faz a operao

principal1
2009/8/19
page 157
Estilo OBMEP

N SEC. 10.2: A DETECO DE ERROS

157

e obtm:
9+(37)+8+(32)+4+(30)+2+(30)+0+(32)+3+(38)+0 = 73.
Como o resultado no um mltiplo de 10, o computador avisa
que foi cometido algum erro.
O cdigo UPC muito semelhante. Como utiliza apenas 12 dgitos (pois usa apenas um para identificar o pas de origem do artigo,
enquanto o EAN utiliza-se de dois), e o vetor de pesos utilizado pelo
UPC tambm tem um dgito a menos; ele :
= (3, 1, 3, 1, 3, , 1, 3, 1, 3, 1, 3, 1).
O leitor notar que, se o digitador comete apenas UM erro de
digitao, trocando um dos dgitos ai por um outro valor, ento
necessariamente o produto no ser congruente a 0 em mdulo
10 e assim ser possvel detectar que o erro foi cometido. Se mais de
um erro for cometido na digitao, o fato provavelmente ainda ser
detectado, mas j no podemos ter certeza, pois eles poderiam se
compensar mutuamente e a soma poderia ainda continuar sendo
um mltiplo de 10.
O leitor pode-se perguntar qual a funo do vetor de pesos . De
fato, se a escolha do dgito de verificao x fosse feita simplesmente
de modo que
a1 + a2 + + a12 + x 0 (mod 10),
ainda assim UM erro de digitao seria detectado. Acontece que h
um outro tipo de erro de digitao muito comum, que consiste em
digitar todos os nmeros corretamente, mas trocar a ordem de dois

principal1
2009/8/19
page 158
Estilo OBMEP
158

CAP. 10: CDIGOS DE BARRAS

dgitos consecutivos.
Suponha que, ao digitar o nmero 9 788531 404580 do nosso
primeiro exemplo, tenha se cometido esse tipo de erro, e que o nmero
de fato digitado fosse 9 788351 404580. Ao efetuar a verificao
ter-se-ia:
(9, 7, 8, 8, 5, 3, 1, 4, 0, 4, 5, 8, 0)(1, 3, 1, 3, 1, 3, 1, 3, 1, 3, 1, 3, 1) =
= 9 + 21 + 8 + 24 + 3 + 15 + 1 + 12 + 12 + 5 + 24
= 134 6 0 (mod 10).
Desta forma, o erro seria detectado.
Suponha agora que, ao digitar o nmero 9 781402 002380 do
nosso segundo exemplo, tenha se cometido um erro desse mesmo
tipo, e que o nmero de fato digitado fosse 9 781402 002830. Ao
efetuar a verificao ter-se-ia:
(9, 7, 8, 1, 4, 0, 2, 0, 0, 2, 8, 3, 0) (1, 3, 1, 3, 1, 3, 1, 3, 1, 3, 1, 3, 1) =
= 9 + 21 + 8 + 3 + 4 + 2 + 6 + 3 + 24
= 80 0 (mod 10).
Este exemplo mostra que o sistema de deteco adotado acima
no tem a capacidade de detectar todo erro de transposio cometido.
fcil ver que a transposio de dois dgitos consecutivos ai e ai+1
no detectada, neste sistema de codificao, se, e somente se,
|ai ai+1 | = 5 (veja a atividade 4).

principal1
2009/8/19
page 159
Estilo OBMEP

N SEC. 10.2: A DETECO DE ERROS

159

Atividades

1)

(i) Um determinado produto deve ser identificado, no cdigo


UPC pelo nmero 7 897595 90071. Determine o dgito de
segurana correspondente.
(ii) Faa o mesmo para um produto cujo nmero 7 894900
01152.

2) Vamos definir um sistema de deteco de erros da seguinte


maneira. A cada nmero de 12 dgitos a0 . . . a11 vamos asignar
um dgito de verificao a12 de forma tal que
12
X

ai 0 (mod 10).

i=0

(i) Achar o dgito de verificao que deve se adicionar ao nmero


723443501297.
(ii) Provar que toda vez que apenas um nmero alterado na
digitao, este sistema capaz de detectar o erro.
(iii) Mostrar que este sistema no capaz de detectar qualquer
erro de transposio.
3) Ns afirmamos no texto que se apenas um erro de digitao for
cometido, alterando um dos dgitos ai para um outro valor bi ,
ento o sistema UPC sempre ser capaz de detectar o erro. D
uma demonstrao cuidadosa deste fato.

principal1
2009/8/19
page 160
Estilo OBMEP
160
4)

CAP. 10: CDIGOS DE BARRAS

(i) Mostrar que uma transposio adjacente; isto , um erro do


tipo
. . . ai ai+1 . . . 7 . . . ai+1 ai . . .
detectada pelo sistema EAN-13 se, e somente se,
|ai ai+1 | =
6 5. (Sugesto: note que |wi wi+1 | = 2).
(ii) Mostre que um erro de transposio no adjacente do tipo
. . . ai ai+1 ai+2 . . . 7 . . . ai+2 ai+1 ai . . .
no pode ser detectado pelo sistema EAN-13.
(iii) Mostrar que um erro de transposio em que dois dgitos no
adjacentes ai e aj so trocados no pode ser detectado pelo
sistema se a diferena i j par.
(iv) Mostrar que, num erro como o descrito no item anterior, se a
diferena i j mpar, ento o erro pode ser detectado pelo
sistema EAN-13 se, e somente se, |ai aj | 6= 5.

10.3

Outros Cdigos Numricos

Como observamos na seo anterior, existem diversos tipos de erros que podem ser cometidos ao digitar um vetor de identificao.
Os erros num nico dgito e as transposies so, de longe, os mais
frequentes. Autores como D.F. Beckley [1] e J. Verhoeff [16] investigaram sistematicamente os erros cometidos por operadores humanos. No quadro abaixo damos as frequncias relativas obtidas por
Verhoeff, que citamos abreviando quadro publicado por H.H. Schulz
[15] e tambm por G.B. Belyavskaya, V.I. Izbash e V.A. Shcherbacov
[2].

principal1
2009/8/19
page 161
Estilo OBMEP
161

N SEC. 10.3: OUTROS CDIGOS NUMRICOS

Tipo de erro
erro nico
. . . a . . . 7 . . . b . . .
transposio adjacente
. . . ab . . . 7 . . . ba. . .
transposio alterna
. . . abc. . . 7 . . . cba. . .
erro gmeo
. . . aa. . . 7 . . . bb. . .
erro gmeo alternado
. . . aba. . . 7 . . . cbc . . .
outros

Frequncia relativa %
79
10.2
0.8
0.6
0.3
9.1

Tabela 10.1: Tipos de erros e suas frequncias segundo Verhoeff


Para descrever vrios dos mtodos existentes, vamos introduzir
um pouco de linguagem geral. Denotaremos por A o conjunto de
valores que podem assumir os dgitos utilizados na codificao. Por
exemplo, no caso do cdigo UPC da seo anterior, esse conjunto
A = {x Z | 0 x m 1}.
O vetor com os dados 0 = (a1 , . . . an1 ) ser chamado de vetor
de informao e o vetor, j acrescido do dgito de verificao ser
chamado de nmero ou vetor de indentificao.
Definio 10.1. Sejam = (w1 , . . . wn ), com wi A, 1 i n
um vetor de pesos e c A um inteiro fixado. Dados dois inteiros
positivos m e n e um conjunto de nmeros a1 , . . . an1 tais que ai A,
1 i n 1, define-se o nmero de verificao an como o nico
elemento de A que verifica a equao:
n
X

ai wi c (mod m).

i=1

Um sistema de codificao assim definido ser denotado por


C = (A, m, n, c, ).
Note que frequentemente A = {0, 1, . . . , m 1}. Neste caso,
tomando classes mdulo m, temos que an o nico elemento de A

principal1
2009/8/19
page 162
Estilo OBMEP
162

CAP. 10: CDIGOS DE BARRAS

que verifica:

an = w1
n

n1
X

!
ai wi

i=1

Exemplo 10.3.1.
Um sistema usado em alguns bancos (mas no todos) o seguinte:
o nmero de conta de um cliente composto de 9 dgitos, sendo que
o ltimo o dgito de verificao. Na nossa notao, o sistema pode
ser descrito como C = (A, 10, 2, 0, ) onde A o conjunto dos dgitos
de 0 a 9 e = (7, 3, 9, 7, 3, 9, 7, 3, 9). Por exemplo, o nmero de uma
conta num certo banco 95-005541-9. Podemos verificar que
(9, 5, 0, 0, 5, 5, 4, 1, 9) (7, 3, 9, 7, 3, 9, 7, 3, 9) =
= 63 + 15 + 15 + 45 + 28 + 3 + 81
= 250 10 (mod 10).
Nosso prximo Teorema descreve a capacidade que tem um sistema definido desta forma, para detectar os diversos tipos de erros
mais frequentes.
Teorema 10.2. (Capacidade de deteco) Sejam m um inteiro positivo e = (w1 , . . . wn ) um vetor de pesos. Suponhamos que um vetor
de identificao = (a1 . . . , an ) (onde assumimos que 0 ai < m,
para todo ndice i, 1 i n) satisfaz a condio
= a1 w1 + + an wn c (mod m).
Ento:
1. Todo erro consistente numa nica alterao na posio i-sima
ser detectado se e somente se mdc(wi , m) = 1.

principal1
2009/8/19
page 163
Estilo OBMEP

N SEC. 10.3: OUTROS CDIGOS NUMRICOS

163

2. Todo erro de transposio da forma


. . . ai . . . aj . . . 7 . . . aj . . . ai . . .
ser detectado se e somente se mdc(wi wj , m) = 1.

Demonstrao. Suponhamos inicialmente que o dgito ai , na posio


i, foi trocado por um outro valor bi e vamos denotar por o vetor
resultante deste erro. claro que o erro no ser detectado se, e
somente se,
0 (mod m).
Mas = (ai bi )wi , de modo que o erro no ser
detectado se, e somente se, m | (ai bi )wi 0 (mod m) ou, se
denotamos por x a classe de um inteiro x em Zm , se, e somente se,
(ai bi )wi = 0 em Zm .
Se mdc(wi , m) = 1 tem-se que wi inversvel em Zm , donde a
condio acima implica que ai = bi , logo ai bi (mod m) e, como
ambos os nmeros so menores que m, isto s aconteceria se ai = bi .
Logo, o erro ser detectado.
Por outro lado, se mdc(wi , m) = d 6= 1, dado ai tem-se que
dentre os nmeros bi = ai + m/d e bi = ai m/d, um deles
verifica a condio 0 bi < m e o erro que substitui ai por esse
nmero no pode ser detectado. Isto completa a demonstrao de (i).
Suponhamos agora que foi cometido um erro do tipo
= . . . ai . . . aj . . . 7 0 = . . . aj . . . ai . . . .

principal1
2009/8/19
page 164
Estilo OBMEP
164

CAP. 10: CDIGOS DE BARRAS

Note que, neste caso podemos calcular a diferena


0 = (ai wi + aj wj ) (aj wi ai wj ) = (ai aj )(wi wj ).
Assim, este erro no ser detectado se, e somente se,
(ai aj )(wi wj ) 0 (mod m).
Daqui em diante, o argumento inteiramente anlogo ao anterior.

A luz deste teorema, resulta claro que a melhor forma de ter


certeza que o sistema de codificao ser capaz de detectar todos
os erros nicos e todos os erros de transposio (contigua ou no)
tomar, para o valor do mdulo m, um nmero primo. De fato, existem
vrios sistemas em uso que procedem desta forma.
Exemplo 10.3.2.
Um sistema universalmente adotado para a classificao de livros
o ISBN (International Standard Book Number). Ele trabalha
mdulo 11, mas para facilitar a notao, utiliza tambm como conjunto de valores A os dgitos de 0 a 9 e os vetores de identificao
tem 10 componentes. Ele pode ser descrito, na nossa notao, por
(A, 11, 10, 0, ) com = (10, 9, 8, 7, 6, 5, 4, 3, 2, 1).
Por exemplo, o livro do autor [14] mencionado anteriormente tem
o nmero ISBN 1-4020-0238-6. O dgito final, de verificao 6 porque
(1, 4, 0, 2, 0, 0, 2, 3, 8, 6) (10, 9, 8, 7, 6, 5, 4, 3, 2, 1) =
= 10 + 36 + 14 + 8 + 9 + 16 + 6
= 99 0 (mod 11).

principal1
2009/8/19
page 165
Estilo OBMEP

N SEC. 10.3: OUTROS CDIGOS NUMRICOS

165

Este mtodo, porm, tem um pequeno inconveniente, que se compreender melhor analizando mais um exemplo. Um conhecido livro
de lgebra [13] tem como vetor de informao de seu cdigo ISBN o
nmero 0-387-96035. Vamos tentar determinar o dgito de verificao.
Ele ser um nmero a A tal que
(0, 3, 8, 7, 9, 6, 0, 3, 5, a) (10, 9, 8, 7, 6, 5, 4, 3, 2, 1) 0 (mod 11).
Efetuando as operaes necessrias, obtemos que
243 + a 0 (mod 11)
ou
a 243 (mod 11).
Como 243 1 (mod 11) e 1 10 (mod 11) temos que a = 10.
Porm, no conjunto dos dgitos de 0 a 9, no temos nenhum que
represente o nmero 10. Devemos introduzir ento mais um smbolo para representar este nmero. A conveno usual utilizar o
smbolo X e assim, o cdigo ISBN que aparece neste livro ISBN
0-387-96035-X.
Finalmente observamos que, se tomamos o nmero m de modo
que seja primo e o conjunto A formado por inteiros menores do que
m como em todos os exemplos acima como cada componente wi
do vetor de pesos prima com m, resulta que multiplicar por wi , em
mdulo m, equivale a definir uma permutao do conjunto A (isto ,
uma bijeo de A em si mesmo). Isto sugere um mtodo mais geral
de definir o vetor de pesos.
Dado um vetor de informao 0 = (a1 , . . . , an1 ) podemos escolher n permutaes 1 , . . . n do conjunto A, definir um vetor de
pesos por = (1 , . . . , n ), fixar um nmero c A e escolher o dgito

principal1
2009/8/19
page 166
Estilo OBMEP
166

CAP. 10: CDIGOS DE BARRAS

de verificao an de modo que verifique a equao:


() = 1 (a1 ) + n (an ) c (mod m).
Neste caso, o dgito de verificao fica definido por:

an = n1

n1
X

!
i (ai ) .

i=1

Este tipo de codificao tambm usada na prtica, como mostrado


a seguir.

Exemplo 10.3.3.

Um cdigo usado pela IBM utiliza como conjunto A os dgitos de


0 a 9; o valores m = 10, um valor qualquer c A e a permutao

0 1 2 3 4 5 6 7 8 9
0 2 4 6 8 1 3 5 7 9

!
.

No caso de um vetor de identificao com um nmero mpar de


componentes n, se utiliza o vetor de pesos
= (I, , I, , . . . , , I).
Assim, a equao de verificao resulta:
an + (an1 ) + an2 + (an3 ) c (mod 10)
e o dgito de verificao dado pela frmula:
an c ((an1 ) an2 (an3 ) ) (mod 10).

principal1
2009/8/19
page 167
Estilo OBMEP
167

N SEC. 10.3: OUTROS CDIGOS NUMRICOS

No caso em que o nmero de componentes do vetor de identificao


par utiliza-se o vetor de pesos
= (, I, , I, . . . , , I),
e resulta uma equao de verificao anloga, com uma frmula similar para o clculo do dgito de verificao.
Este o sistema utilizado para determinar os nmeros nos cartes
de crdito. Por exemplo, um determinado carto tem o nmero 5745
5195 0431 5412. Vamos a aplicar a ele o sistema de verificao IBM:
5

7
I
7

5
I
7

1
I
1

5
I
1

4
I
4

1
I
1

4
I
9

Agora, calculamos:
1+7+8+7+1+1+9+1+0+4+6+1+1+9+2+2 = 60 0 (mod 10)
Note que esta codificao permite detectar todo erro nico de digitao e toda transposio adjacente exceto no caso em que ai e aj
assumem os valores 0 e 9 ou 9 e 0 respectivamente (veja a atividade
3). Observe, porm, que este sistema no detecta transposies do
tipo
. . . ai . . . aj . . . 7 . . . aj . . . ai . . .
quando a diferena i j par e tambm no permite detectar erros
gmeos.
O cdigo IBM generalizado utiliza a mesma permutao, com o
vetor de pesos:
= ( n1 , n2 , . . . , , 0 )

2
I
2

principal1
2009/8/19
page 168
Estilo OBMEP
168

CAP. 10: CDIGOS DE BARRAS

onde 0 = I. A equao ento:


n
X

i1 (an+1i ) c (mod 10)

i=1

e
an c

n
X

i1 (an+1i ) (mod 10).

i=2

Este sistema detecta todo erro nico de digitao, toda transposio, adjacente ou no, (exceto no caso j apontado, quando os
dgitos envolvidos so 0 e 9) e todo erro gmeo.
Vimos acima que o cdigo ISBN detecta todo erro nico de digitao e todo erro de transposio. natural se perguntar, ento, se
existe um cdigo com essa capacidade de deteco, trabalhando na
base 10. Infelizmente, a resposta negativa, como mostra o seguinte
teorema.
Teorema 10.3. (Gumm [10]) Se um sistema numrico de deteco
de erros, com um mdulo par, detecta todo erro nico de digitao,
ento, para todo par de ndices i, j existe um erro de transposio
entre as posies i e j que no detectada pelo sistema.
Demonstrao. Como vamos trabalhar com os nmeros de 0 a 2m 1
e tomar congruncias em mdulo 2m, vamos considerar nossos dgitos como elementos de Z2m , para simplificar nossos argumentos. Suponhamos que o sistema transforma o vetor (a1 , . . . , an ) num outro
vetor, que denotaremos por (1 (a1 ), . . . , n (an )). Claramente, se o
sistema capaz de detectar todo erro nico de digitao, ento a
aplicao na posio i-sima x 7 i (x) deve ser uma permutao de
Z2m .
Para que o sistema detecte todo erro de transposio entre as
posies i e j necessrio que i (a) + j (b) 6= j (a) + i (b), todo par

principal1
2009/8/19
page 169
Estilo OBMEP
169

N SEC. 10.3: OUTROS CDIGOS NUMRICOS

de elementos diferentes a, b Z2m . Isto equivalente a dizer que a


aplicao = i j uma permutao Z2m .
Mas m [0, 2m 1] e temos que
0+1+2+ +2m1 =

(2m 1) 2m
= 2mmm m (mod 2m),
2

ou seja,
0 + 1 + + 2m 1 = m em Z2m .
Logo:
m =

x=

xZ2m

(x)

xZ2m

(i (x) j (x)) =

xZ2m

= m m = 0,

xZ2m

i (x)

j (x)

xZ2m

em Z2m .

uma contradio.

Atividades

1) Calcular o dgito de verificao para um livro cujo nmero ISBN


tem, como vetor de informao o nmero 85-314-0458.
2) Use o sistema do exemplo 10.3.1 para determinar o dgito de verificao do nmero 13-010765.
3) Demonstre que o cdigo IBM e o cdigo IBM generalizado
tem a capacidade de deteco de erros mencionada no texto.
(Sugesto: Para discutir erros de transposio adjacente, note
que (x) = 2x, se x [0, 4] e que d(x) = 2x 9 se x [5, 9]. Considere separadamente trs casos: (i) a, b [0, 4], (ii) a, b [5, 9] e

principal1
2009/8/19
page 170
Estilo OBMEP
170

CAP. 10: CDIGOS DE BARRAS

(iii) a [0, 4], b [5, 9] ou vice-versa. Mostre que nos casos (i) e
(ii) o erro sempre detectado e que, no caso (iii) o erro s no
detectado se a = 0 e b = 9 ou a = 9 e b = 0.
4) Mostre que no cdigo UPC podem ocorrer 90 erros de transposio
adjacente e que o cdigo capaz de detectar todos eles, exceto
quando os pares de nmeros adjacentes so 05, 16, 27, 38, 49 ou
aqueles que se obtm invertindo estes.

principal1
2009/8/19
page 171
Estilo OBMEP

Captulo 11

Cdigos Sobre Grupos


11.1

O Grupo Dihedral

Na seo anterior apresentamos diversos mtodos de detectar erros


usando um dgito de verificao. Dentre estes, s o sistema ISBN para
livros era capaz de detectar todo erro nico de digitao e todo erro
de transposio. Ele tinha, porm, o inconveniente de precisar da
introduo de um dgito extra, para representar o nmero 10, que
denotamos por X.
Em 1969 Verhoeff, na sua tese de doutoramento [16], desenvolveu
um mtodo simples, baseado no em clculos com nmeros inteiros,
mas com os elementos de um certo grupo, que tambm detecta erros
nicos de digitao e todos os erros de transposo adjacentes, sem
necessidade de smbolos extras. A exposio elementar deste mtodo
que damos a seguir aparece em [6] e num texto bsico de lgebra, do
mesmo autor [5, Captulo V].
Consideremos o grupo dihedral D5 , que pode ser definido como
o grupo das isometrias do plano que deixam fixo um pentgono regular
dado. Este grupo contm dez elementos. Cinco deles so rotaes: a
171

principal1
2009/8/19
page 172
Estilo OBMEP
172

CAP. 11: CDIGOS SOBRE GRUPOS

identidade R0 , a rotao R1 de ngulo 2/5 em sentido anti-horrio,


e as rotaes R2 , R3 e R4 de ngulos 2(2/5), 3(2/5) e 4(2/5)
respectivamente. Contm ainda cinco reflexes, em relao aos seus
eixos de simetria, que passam por cada um dos vrtices e o ponto
mdio do lado oposto: S6 , S7 , S8 , S9 e S10 .

Figura 11.1: Simetrias do pentgono


Vamos usar simplesmente os subndices 0, 1, 2, 3, 4 para designar as
rotaes correspondentes e os subndices 5, 6, 7, 8, 9 para as respectivas
reflexes.
Assim, por exemplo, a composio das rotaes R3 e R4 seria a
rotao de ngulo 3(2/5) + 4(2/5) = 7(2/5) = 2(2/5); isto , a
rotao R2 . Em vez de escrever ento que R3 R4 = R2 escreveremos
simplesmente que 3 4 = 2.
Da mesma forma, como a composio S6 S5 = R1 (lembre que,
como se trata de composio de funes, aplicamos primeiro S1 e
depois S2 ) escrevemos 6 5 = 1. Por outro lado, fcil verificar que
S5 S6 = R0 donde escrevemos 6 5 = 0.

principal1
2009/8/19
page 173
Estilo OBMEP
173

N SEC. 11.1: O GRUPO DIHEDRAL

Procedendo desta forma, obtemos a seguinte tabela de multiplicao para D5 .

0
1
2
3
4
5
6
7
8
9

0
0
1
2
3
4
5
6
7
8
9

1
1
2
3
4
0
9
5
6
7
8

2
2
3
4
0
1
8
9
5
6
7

3
3
4
0
1
2
7
8
9
5
6

4
4
0
1
2
3
6
7
8
9
5

5
5
6
7
8
9
0
1
2
3
4

6
6
7
8
9
5
4
0
1
2
3

7
7
8
9
5
6
3
4
0
1
2

8
8
9
5
6
7
2
3
4
0
1

9
9
5
6
7
8
1
2
3
4
0

Tabela 11.1: A tabela de multiplicao do grupo D5


Consideremos a permutao:

0 1 2 3 4 5 6 7 8 9
1 5 7 6 2 8 3 0 9 4

!
.

A ideia de Verhoeff consiste em transformar um vetor de informao


(a1 , . . . , an1 ) num vetor de codificao, adicionando um dgito de
verificao an de forma tal que
(a1 ) 2 (a2 ) n1 (an1 ) an = 0 em D5 .
Note que uma permutao de D5 . Ela foi escolhida para desenvolver este sistema porque pode-se verificar diretamente que
a (b) 6= b (a) para todo a, b D5 .
Logo abaixo veremos a importncia deste fato.

(11.1)

principal1
2009/8/19
page 174
Estilo OBMEP
174

CAP. 11: CDIGOS SOBRE GRUPOS

Como i tambm uma permutao de D5 , para todo inteiro


positivo i, resulta claro que este sistema de codificao detecta todo
erro nico de digitao.
Note
que
um
erro
de
transposio
adjacente
. . . ai ai+1 . . . 7 . . . ai+1 ai . . . ser detectado se, e somente se,
i (ai ) i+1 (ai+1 ) 6= i (ai+1 si+1 (ai ). Mas, vimos na equao 11.1
que a (b) 6= b (a) para todo a, b D5 o que implica, aplicando a
permutao i a ambos os membros desta equao, que
i (a) i+1 (b) 6= i (b) i+1 (a) para todo a, b D5
como queramos demonstrar.
Uma variante do mtodo de Verhoeff1 foi usada pelo rgo
emissor de dinheiro da Alemanha, o Deutsche Bundesbank. As
notas de dinheiro so numeradas num cdigo alfanumrico; isto
, se utilizam tanto de letras quanto de nmeros. Na Figura 11.2
reproduzimos uma nota de 10 marcos (que j est fora de circulao
desde o advento da moeda unificada da Europa, o Euro).

Figura 11.2:

Citado por Gallian [7].

principal1
2009/8/19
page 175
Estilo OBMEP
175

N SEC. 11.1: O GRUPO DIHEDRAL

O cdigo utilizado pelo banco segue uma variante do mtodo anterior. Eles numeram as notas usando os dgitos de 0 a 9 e tambm
dez letras: A, D, G, K, L, N, U, V e Z. Tambm se utiliza da tabela
da operao do grupo D5 , mas ao invs de usar uma permutao e
suas potncias, utiliza dez permutaes diferentes. Para determinlas damos, na tabela abaixo, em cada fila i os valores da funo i .
Em outras palavras, na posio i, j est o valor de i (j).
0
1
5
8
9
4
2
7
0
1
5

1
2
3
4
5
6
7
8
9
10

1
5
8
9
4
2
7
0
1
5
8

2
7
0
1
5
8
9
4
2
7
0

3
6
3
6
3
6
3
6
3
6
3

4
2
7
0
1
5
8
9
4
2
7

5
8
9
4
2
7
0
1
5
8
9

6
3
6
3
6
3
6
3
6
3
6

7
0
1
5
8
9
4
2
7
0
1

8
9
4
2
7
0
1
5
8
9
4

9
4
2
7
0
1
5
8
9
4
2

Isto significa, por exemplo, que a permutao 5 :

5 =

0 1 2 3 4 5 6 7 8 9
4 2 8 6 5 7 3 9 0 1

!
.

O nmero de srie da nota da Figura 11.2 DU7124458G6. Vamos


verificar que esta uma numerao vlida. Para poder usar o mtodo
de Verhoeff devemos trabalhar apenas com os dgitos de 0 a 9; por
causa disso, a cada letra das usadas pelo Deutch Bundesbank deve-se
lhe assignar um valor numrico. Isto feito de acordo com a seguinte
tabela.
A
0

D
1

G
2

K
3

L
4

N
5

S
6

U
7

Y
8

Z
9

principal1
2009/8/19
page 176
Estilo OBMEP
176

CAP. 11: CDIGOS SOBRE GRUPOS

Usando esta tabela, o nmero da nota em questo se transforma


em 17712445826. Aplicamos ento ordenadamente as permutaes
dadas:
1
1
5

7
2
1

7
3
5

1
4
4

2
5
8

4
6
8

4
7
9

5
8
5

8
9
9

2
10
0

Agora precisamos multiplicar estes nmeros, com ajuda da


tabela 11.1. Temos:
5 1 = 9,

9 5 = 4,

4 4 = 3,

3 8 = 6,

6 8 = 3,

3 9 = 7,

7 5 = 2,

2 9 = 6,

6 0 = 6,

e finalmente
6 6 = 0.
Este mtodo, porm, tem um inconveniente. Nos clculos, ele no
distingue entre uma letra e o nmero que lhe assignado. Assim por
exemplo, se a letra K for trocada pelo nmero 3, o sistema ser incapaz
de detectar o erro. O mesmo acontece se ocorre uma transposio de
3 e K, ou vice-versa. Para evitar este problema, poder-se-ia usar
o grupo D18 , que tem 36 elementos (e portanto os vinte smbolos
usado no cdigo alfanumrico das notas corresponderiam a elementos
diferentes em D5 ), com uma permutao adequada. Sugestes nesse
sentido apareceram, por exemplo, em trabalhos de Winters, em 1990
[17] e de Gallian e Mullin, em 1995 [8].
Note que a equao 11.1 foi essencial para podermos mostrar que o
mtodo de Verhoeff permite detectar erros de transposio adjacentes.
Isto justifica a seguinte definio.

principal1
2009/8/19
page 177
Estilo OBMEP

N SEC. 11.1: O GRUPO DIHEDRAL

177

Definio 11.1. Uma permutao de um grupo G diz-se uma aplicao antissimtrica se verifica a seguinte condio:
x(y) 6= y(x),

para todo par de elementos x, y G.

O grupo D5 desempenha um papel importante na elaborao de


cdigos detectores de erros porque pode-se mostrar que ele o nico
grupo de ordem 10 que possui uma aplicao antissimtrica.
Como as transposies adjacentes se encontram entre os erros mais
comuns e como cdigos que detectam estes erros podem-se elaborar a
partir de grupos com aplicaes antissimtricas, houve vrios trabalhos que dedicaram especial ateno a este tipo de grupos.
No caso dos grupos abelianos, h um tipo de permutao que
tambm importante.
Definio 11.2. Uma permutao de um grupo G diz-se uma aplicao completa se a funo x 7 x(x), para todo x G, uma
permutao de G.
Pode-se demonstrar que se G um grupo abeliano, ento G possui
uma aplicao antissimtrica se, e somente se, G possui uma aplicao
completa.
Um grupo abeliano de ordem 2m, com m mpar no possui aplicaes completas [3]. Como consequncia imediata, temos que Um
grupo abeliano de ordem 2m, com m mpar no possui aplicaes antissimtricas.
Outros resultados relativos a este tipo de aplicaes so os
seguintes:
Um grupo cclico admite uma aplicao antissimtrica se, e somente se, de ordem mpar [15].

principal1
2009/8/19
page 178
Estilo OBMEP
178

CAP. 11: CDIGOS SOBRE GRUPOS

Todo grupo solvel no abeliano admite uma aplicao antissimtrica [11].


Todo grupo simples, exceto Z2 , admite uma aplicao antissimtrica [8].
Foi anunciado por Heiss que todo grupo finito no abeliano tambm admite uma aplicao antissimtrica [12].
Para os grupos dihedrais, diversas classes de aplicaes antissimtricas foram achadas em [4] e [9]. Como vimos, isto implica que
estes grupos podem ser usados para construir cdigos que detectam
erros nicos de digitao ou transposies adjacentes. Porm, eles
no podem detectar outros erros frequentes, como mostra o seguinte
teorema, devido a Damm [3, Teorema 5].
Teorema 11.3. Seja m > 2 um inteiro mpar. No existe um sistema de dgito de controle sobre Dm que seja capaz de detectar todas
as transposies alternadas, todas os erros gmeos ou todos os erros
gmeos alternados.

Atividades

1) Determine todos os subgrupos cclicos de D5 .


2) Ache o centro Z(D5 ) de D5 e determine o quociente D5 /Z(D5 ).
3) Determine a decomposio em produto de ciclos disjuntos e a paridade da permutao do mtodo de Verhoeff.
4) Determine o valor de x para que o nmero 3572498x seja um
nmero vlido no mtodo de Verhoeff.

principal1
2009/8/19
page 179
Estilo OBMEP

N SEC. 11.1: O GRUPO DIHEDRAL

179

5) Mostre que tomando a permutao = (1 4)(2 3)(5 8 6 9 7) no


mtodo de Verhoeff, tambm possvel detectar todo erro nico
de digitao e toda transposio adjacente.
6) Determine o valor do dgito x para que o nmero AD377345654Kx
seja um nmero vlido para uma nota emitida pelo Deustche Bank.
7) Idem, para o nmero N Z357x29477L2.
8) (Gallian [5]) Seja = (1 2 4 8 7 5)(3 6). A cada nmero da
forma a1 a2 . . . an (com n mpar) atribumos o dgito de controle
((a1 ) + a2 + (a3 ) + a4 + + (an )) (mod 10). Calcule o valor
do dgito de controle para o nmero 3125600196431. Prove que
este mtodo detecta todo erro nico de digitao. Determine
quais transposies adjacentes no podem ser detectadas por este
mtodo.

principal1
2009/8/19
page 180
Estilo OBMEP

principal1
2009/8/19
page 181
Estilo OBMEP

Referncias Bibliogrficas
[1] BECKLEY, D.F. An optimum system with modulo 11. The
Computer Bulletim, n. 11, p. 213-215, 1967.
[2] BELYAVSKAYA, G.B.; IZBASH V.I.; MULLEN, G.L. Check
character systems over quasegroups and loops. Quasigroups and
related systems, n. 10, p. 1-28, 2003.
[3] DAMN, M. Check digit over groups and anti-symmetric
mappings. Archiv der Math., n. 75, p. 413-421, 2000.
[4] ECKER, A.; POCH, G. Check character systems. Computing,
n. 37, p. 277-301, 1986.
[5] GALLIAN, J.A. Contemporary Abstract Algebra, D.C. Heath and
Co., Lexington, 1990.
[6] GALLIAN, J.A. The Mathematics of Identification Numbers,
The College Math. J., v. 22, n. 3, p. 194-202, 1991.
[7] GALLIAN, J.A. Error detecting methods,
Surveys, v. 28, n. 3, p. 504-517, 1996.

ACM Computing

[8] GALLIAN, J.A.; MULLIN, M. Groups with antisymmetric


mappings, Archiv der Math., n. 65, p. 273-280, 1995.
[9] GUMM, H.P. A new class of check-digit methods for arbitrary
number systems, IEEE Trans, Inf. Th., n. 31, p. 102-105, 1985.
181

principal1
2009/8/19
page 182
Estilo OBMEP
182

REFERNCIAS BIBLIOGRFICAS

[10] GUMM, H.P. Encoding of numbers to detect typing errors, Inter. J. Applied Eng. Ed., n. 2, p. 61-65, 1986.
[11] HEISS, S. Anti-symmetric mappings for finite solvable groups,
Archiv der Math., n. 69, p. 445-454, 1997.
[12] HEISS, S. Anti-symmetric mappings for finite groups, preprint,
1999.
[13] LIDL, R.; PILZ, G. Applied Abstract Algebra. Undergraduate
Texts in Math. New York: , Springer Verlag, 1984.
[14] POLCINO MILIES, C.; SEHGAL, S.K. An introduction to
Group Rings, Kluwer Acad. Publ., Dordrecht, 2002.
[15] SCHULZ, R.H. On check digit systems using anti-symmetric
mappings. In Numbers, Information and Complexity, p. 295-310,
Dordrecht: Kluwer Acad. Publ., 2000.
[16] VERHOEFF, J. Error detecting decimal codes. Math. Centre
Tracts. Amsterdam: Mathematische Centrum, 1969.
[17] WINTERS, S. Error detecting codes using dihedral groups.
UMAP J., n. 11, p. 299-308, 1990.

H tambm muitas pginas na internet com informaes sobre o


assunto. Veja, por exemplo:
http://en.wikipedia.org./wiki/ENIAC
http://en.wikipedia.org/wiki/Harvard_Mark-I

principal1
2009/8/19
page 183
Estilo OBMEP

REFERNCIAS BIBLIOGRFICAS

http://en.wikipedia.org/wiki/Ada_Lovelace
http://www-etsi2.ugr.es/alumnos/mili/Harvard20I.htm
http://www.bellsouthpwp.net/l/a/laurergj/upc_work.html
http://www.barcodeisland.com/ean13.phtml
http://www.adams1.com/pub/russadam/barcode1.html

183

cripto
2009/6/30
i page 1
Estilo OBMEP

i
i

Criptografia
S. C. Coutinho

i
i

cripto
2009/6/30
i page 2
Estilo OBMEP

i
i

Texto j revisado pela nova ortografia.

i
i

cripto
2009/6/30
i page i
Estilo OBMEP

i
i

Antes de Comear
Estas notas tratam de uma aplicao da matemtica criptografia.
Embora algumas pessoas ainda associem mensagens codificadas a 007
ou outros agentes igualmente secretos, h mais de uma dcada que
esta no a aplicao mais importante da criptografia. Isto porque,
hoje em dia, uma grande variedade de transaes que envolvem dinheiro so feitas de maneira eletrnica, desde compras por carto de
crdito via internet a saques em caixas eletrnicos. A informao referente a estas transaes segue por linha telefnica ou redes de alta-velocidade e, em ambos os casos, est facilmente sujeita a escutas.
Se a histria acabasse a, eu seria o primeiro a desejar que os
bancos regridissem era do papel! Felizmente, estas informaes
no trafegam em aberto pela rede telefnica, elas so codificadas, de
modo que s o banco, empresa de carto de crdito ou loja que voc
est utilizando consegue ler a informao. Assim, mesmo que algum
intercepte a informao com a inteno de esvaziar sua conta, ele no
conseguir interpretar suas informaes, que continuaro seguras.
Os processos pelos quais informaes enviadas eletronicamente so
codificadas depende, de maneira crucial, do uso da matemtica. O
i

i
i

cripto
2009/6/30
i page ii
Estilo OBMEP

i
i

ii
mais curioso que at os anos 1960, a teoria dos nmeros, que a
parte da matemtica mais utilizada nas aplicaes criptografia, era
considerada quase que destituda de utilidade prtica.
O que os matemticos entendem como teoria dos nmeros o
estudo das propriedades dos nmeros inteiros, e no de quaisquer
tipos de nmeros. Por exemplo, questes referentes fatorao de
inteiros, ao clculo do mximo divisor comum e ao estudo dos nmeros
primos, fazem parte desta teoria. Na verdade, juntamente com a geometria, essa uma das reas mais antigas da matemtica.
Nestas notas desenvolvemos os mtodos da teoria dos nmeros
necessrios s aplicaes em um sistema de criptografia especfico, o
chamado RSA. H duas razes para isto. A primeira que os resultados matemticos utilizados neste sistema so relativamente elementares; a segunda que se trata do mais utilizado dos mtodos
de criptografia atualmente em uso.
Estas notas se dirigem a um estudante com conhecimento bsico
sobre a fatorao de inteiros e primos, que tenha certa facilidade no
clculo com frmulas elementares e que tenha interesse matemtico
suficiente para apreciar argumentos de demonstraes bastante bsicas. Gostaria de agradecer a todas as pessoas que me ajudaram na
preparao das notas, especialmente Florncio Ferreira Guimares
Filho que primeiro sugeriu a ideia destas notas, Suely Druck e Mrio
Jorge Dias Carneiro que leram todo o texto e deram inmeras sugestes para melhor-lo e a Francisca Frana que leu todo o texto,
corrigindo-o, revisando-o e preparando-o para a publicao.
Rio de Janeiro, 13 de maio de 2008

S. C. Coutinho

i
i

cripto
2009/6/30
i page iii
Estilo OBMEP

i
i

Sumrio
Introduo

Criptografia . . . . . . . . . . . . . . . . . . . . . . . . . . .

Criptografia RSA . . . . . . . . . . . . . . . . . . . . . . . .

1 Nmeros Inteiros

15

1.1

Fatores e Nmeros Primos . . . . . . . . . . . . . . . .

15

1.2

Fatorando Inteiros . . . . . . . . . . . . . . . . . . . .

19

2 Aritmtica Modular

37

2.1

Fenmenos Peridicos e Aritmtica . . . . . . . . . . .

37

2.2

Definies e Primeiras Propriedades

. . . . . . . . . .

45

2.3

Critrios de Divisibilidade . . . . . . . . . . . . . . . .

61

3 Inversos Modulares

79

3.1

Motivao e Definies . . . . . . . . . . . . . . . . . .

79

3.2

Inexistncia de Inverso . . . . . . . . . . . . . . . . . .

84

iii

i
i

cripto
2009/6/30
i page iv
Estilo OBME

i
i

iv

SUMRIO

3.3

Existncia de Inverso . . . . . . . . . . . . . . . . . . .

92

3.4

O Teorema e um Exemplo . . . . . . . . . . . . . . . .

97

4 Algoritmo Chins do Resto

102

4.1

Exemplos . . . . . . . . . . . . . . . . . . . . . . . . . 102

4.2

O Teorema Chins do Resto . . . . . . . . . . . . . . . 113

5 Potncias

121

5.1

Restos de Potncias . . . . . . . . . . . . . . . . . . . . 121

5.2

O Teorema de Fermat . . . . . . . . . . . . . . . . . . 134

5.3

Potncias . . . . . . . . . . . . . . . . . . . . . . . . . 139

6 Criptografia RSA

146

6.1

Pr-codificao . . . . . . . . . . . . . . . . . . . . . . 147

6.2

Codificando e Decodificando uma Mensagem . . . . . . 149

6.3

Por que funciona? . . . . . . . . . . . . . . . . . . . . . 161

7 Encontrando Primos

168

7.1

Infinidade dos Primos . . . . . . . . . . . . . . . . . . 169

7.2

Encontrando os Primos . . . . . . . . . . . . . . . . . . 176

7.3

Um Teste de Composio . . . . . . . . . . . . . . . . 186

Solues

200

Exerccios . . . . . . . . . . . . . . . . . . . . . . . . . . . . 200
Desafios . . . . . . . . . . . . . . . . . . . . . . . . . . . . . 212

i
i

cripto
2009/6/30
i page v
Estilo OBMEP

i
i

SUMRIO

Referncias Bibliogrficas

v
217

i
i

cripto
2009/6/30
i page vi
Estilo OBMEP

i
i

cripto
2009/6/30
i page 1
Estilo OBMEP

i
i

Introduo
O foco deste livro o mtodo de criptografia de chave pblica
conhecido como RSA.1 Toda a matemtica que vamos estudar estar
ligada diretamente a este mtodo. Na introduo apresentaremos a
ideia central por trs do funcionamento do RSA.

Criptografia
Em grego, cryptos significa secreto, oculto. A criptografia estuda
os mtodos para codificar uma mensagem de modo que s seu destinatrio legtimo consiga interpret-la. a arte dos cdigos secretos.

O Cdigo de Csar
Um dos cdigos secretos mais simples consiste em substituir uma
letra do alfabeto pela seguinte. Por exemplo, a mensagem AMO A
1

Se sua curiosidade para saber o que as letras significam irresistvel, olhe na


pgina 9

i
i

cripto
2009/6/30
i page 2
Estilo OBMEP

i
i

2
OBMEP seria codificada como
BN P BP CN F Q.
Um cdigo semelhante a este foi usado, por exemplo, pelo ditador
romano Jlio Csar para comunicar-se com as legies romanas em
combate pela Europa. Este parece ser o primeiro exemplo de um cdigo secreto de que se tem notcia.

Figura 1: Jlio Csar (100-44 a.C.)


Vejamos como codificar uma mensagem simples. Cdigos como
o de Csar padecem de um grande problema: so muito fceis de
quebrar. Quebrar um cdigo significa ser capaz de ler a mensagem,
mesmo no sendo seu destinatrio legtimo. Na verdade, qualquer
cdigo que envolva substituir cada letra sistematicamente por outro
smbolo qualquer sofre do mesmo problema. Isto ocorre porque a
frequncia mdia com que cada letra aparece em um texto de uma
dada lngua mais ou menos constante. Por exemplo, a frequncia
mdia de cada letra na lngua portuguesa dada na tabela 1.

i
i

cripto
2009/6/30
i page 3
Estilo OBMEP

i
i

3
Letra
A
B
C
D
E
F

%
14,64
1,04
3,88
4,10
12,57
1,02

Letra
G
H
I
J
L
M

%
1,30
1,28
6,18
0,40
2,78
4,75

Letra
N
O
P
Q
R
S

%
5,05
10,73
2,52
1,20
6,53
7,81

Letra
T
U
V
X
Z

%
4,34
4,64
1,70
0,21
0,47

Tabela 1: Frequncia das letras no portugus


Assim, apenas contando a frequncia de cada smbolo no texto,
podemos descobrir a que letra correspondem os smbolos mais frequentes. Isto geralmente suficiente para quebrar o cdigo e ler toda
a mensagem. Observe, entretanto, que este mtodo para quebrar o
cdigo s funciona bem se a mensagem for longa. fcil escrever uma
mensagem curta cuja contagem de frequncia seja totalmente diferente da contagem de frequncia mdia do portugus. Por exemplo,
em Zuza zoou da Zez a letra mais frequente o Z que aparece 5
vezes em um texto de 14 letras. Como 5/14 = 0, 35... a porcentagem do Z no texto acima de cerca de 35%; muito acima dos usuais
0, 47%. J o A aparece uma s vez, o que d uma porcentagem de
cerca de 7%; portanto, abaixo dos 14% usuais.
SUMZFI GCSGC SVZFC LZLSJ EZQSL HIFUI JDZQS LTSRF
SGCSJ UOZSZ OJTZL ZOEEO LHMSE ESDSL IECLU ILHCD
ZTIFE SZMOJ QCZSU IJPSU OTZZL ZOIFH ZFDST IHFIU SEEIH
ITSES FZCDI LZDOA ZTIIG CSDIF JZOJB OZBSO EDITI EIEUI
TOQIE GCSSJ BIMBS LECVE DODCO UZITS MSDFZ EUILI
IGCSS EDZLIE CDOMO AZJTI HZFZU ITORO UZFSE DZLSJ
EZQSL JZBSF TZTSZ MQCJE TIEHF OLSOF IEUIL HCDZT

i
i

cripto
2009/6/30
i page 4
Estilo OBMEP

i
i

4
IFSER IFZLU FOZTIE HFSUO EZLSJ DSHZF ZZNCT ZFZGC
SVFZFI EUITO QIEES UFSDI ECEZTI EHSMIE ZMSLZSE
TCFZJDS ZESQC JTZQC SFFZL CJTOZM SJDFS SEDSE SEDZB
ZIUIM IEEICL UILHC DZTIF UIEJD FCOTI JZOJQ MZDSF
FZHIF CLZSG COHSM OTSFZ TZHIF ZMZJD CFOJQ CLTIE
RCJTZ TIFSE TZUILH CDZUZI UOSJDO ROUZ
Exerccio 1. Ser que voc notou que o pargrafo acima foi codificado? Use o mtodo de contagem de frequncia para quebrar o cdigo
e poder decodificar e ler o pargrafo. Para no simplificar as coisas,
foram eliminados espaos, acentos e pontuao.

Cdigos em Bloco
Por sorte, existe uma maneira simples de tornar invivel a aplicao de uma contagem de frequncia. Para isso, subdividimos a
mensagem em blocos de vrias letras e embaralhamos estes blocos.
Por isso este processo de criptografar uma mensagem conhecido
como cdigo de bloco. Por exemplo, considere a mensagem AMO A
OBMEP. Para codific-la seguiremos os seguintes passos:
eliminamos os espaos e completamos a mensagem com um A
no final, caso tenha uma quantidade mpar de letras;
subdividimos a mensagem em blocos de duas letras;
refletimos cada bloco;
permutamos os blocos trocando o primeiro com o ltimo, o terceiro com a antepenltimo, e assim por diante, mas deixando os
outros como esto.

i
i

cripto
2009/6/30
i page 5
Estilo OBMEP

i
i

5
Aplicando isto, passo a passo, mensagem acima, obtemos primeiro
AMOAOBMEPA
depois
AM-OA-OB-ME-PA
em seguida
MA-AO-BO-EM-AP
e, finalmente,
AP-AO-BO-EM-MA
que nos d como mensagem codificada
APAOBOEMMA.
Exerccio 2. Discuta as seguintes questes com seus colegas:
(a) Por que a contagem de frequncia no funciona quando usamos cdigos em bloco?
(b) Por que escolhemos acrescentar exatamente a letra A quando
a mensagem tem quantidade mpar de letras, em vez de usar, por
exemplo, X ou Y?
Apesar de cdigos como este serem melhores que o cdigo de
Csar, eles apresentam uma grande desvantagem quando se trata de

i
i

cripto
2009/6/30
i page 6
Estilo OBMEP

i
i

6
aplicaes comerciais da criptografia. Por exemplo, digamos que resolvo fazer uma compra via web usando o meu computador, em uma
loja em que nunca comprei antes. Para isso entro na pgina da loja,
escolho os produtos que desejo e, quando estou pronto para comprar,
escolho ir para o caixa. O pagamento ser feito usando o meu carto
de crdito. Para isso, preciso informar a loja sobre os dados do meu
carto: geralmente o nmero e a data de vencimento. Mas isto significa que qualquer outra pessoa que tenha estes dados pode fazer
compras em meu nome. Para evitar este problema, as informaes
sobre o meu carto so codificadas pelo meu computador antes de
serem enviadas.
Note, contudo, que meu computador no pode usar um cdigo
qualquer para codificar estas informaes, porque a loja precisa l-las
e, para isso, tem que saber como decodificar a mensagem. Na prtica
o que ocorre que o meu computador comunica-se com o da loja, que
lhe informa como deve ser feito o processo de codificao. Isto , meu
computador codifica as informaes do carto de crdito usando um
processo de codificao que enviado pela loja.
Infelizmente os cdigos de blocos no se prestam a este tipo de
aplicao porque o computador da loja usa a linha telefnica (ou de
banda larga) qual meu computador esta interligado para enviar o
processo de codificao a ser utilizado. Como fcil pr uma escuta na linha, uma outra pessoa pode facilmente descobrir como meu
computador vai codificar as informaes sigilosas que sero enviadas
loja. Usando a mesma escuta fcil interceptar tambm as mensagens que contm os dados do carto. Mas isto basta porque, se
sabemos como foi feito o embaralhamento dos blocos, podemos facil-

i
i

cripto
2009/6/30
i page 7
Estilo OBMEP

i
i

7
mente desfaz-lo e ler os dados do carto!
A nica maneira de contornar este problema ter acesso ao que
conhecido como um canal seguro: uma maneira secreta de fazer
a informao sobre o processo de codificao chegar at o computador do usurio da loja. Talvez a loja pudesse mandar, pelo correio
registrado, um carto especial com os dados a serem usados para a
codificao. O problema que isto tornaria a transao lenta, j
que seria necessrio esperar dias pela chegada do carto nesse meio
tempo eu talvez preferisse escolher uma loja real, mesmo que fosse
longe da minha casa. E ainda h outro problema, mais srio. Se o
meu computador for invadido por um hacker, o processo de codificao ser descoberto e qualquer mensagem enviada com ele poder
ser lida.

Cdigos de Chave Pblica


As dificuldades que relacionamos acima parecem condenar de maneira irremedivel a possibilidade de fazer transaes pela web. Afinal, seja qual for o cdigo utilizado, se sabemos como fazer a codificao, basta desfaz-la e decodificamos a mensagem. Ou no?
De fato, isto basicamente verdade; mas h um porm. Acontece
que podemos imaginar um processo que seja fcil de fazer mas muito
difcil de desfazer e, ao utiliz-lo para criptografar uma mensagem, estaramos garantindo que quem a interceptasse, mesmo sabendo como
foi codificada, teria um trabalho enorme em decodific-la. Abusando
um pouco da fantasia, podemos imaginar que o trabalho de desfazer
o processo levasse tanto tempo que ningum conseguisse p-lo em

i
i

cripto
2009/6/30
i page 8
Estilo OBMEP

i
i

8
prtica. claro que quo difcil ser desfazer o procedimento depende dos recursos disponveis a quem interceptou a mensagem.
Vejamos um exemplo. Voc j viu uma dessas armadilhas usadas
para pescar lagostas? Elas consistem de uma gaiola com uma porta
fechada atrs e uma entrada para a lagosta na frente. O segredo est
na entrada, que tem a forma de um funil: larga na parte externa e
cada vez menor medida que a lagosta vai entrando na gaiola. Para
uma ilustrao da entrada da armadilha veja a figura 2.

Figura 2: Entrada de armadilha de lagosta


A lagosta fica presa na gaiola porque, para poder sair, teria que
encontrar e passar pela parte estreita do funil, que um problema
complicado demais para uma lagosta, cujo crebro tem o tamanho
aproximado de uma ervilha. No preciso dizer que uma armadilha
desse tipo no funcionaria para pegar um macaco, nem mesmo um
passarinho.
Muito interessante, mas que problema matemtico satisfaz esta
condio de ser fcil de fazer e difcil de desfazer, para que possamos
utiliz-lo em criptografia? Isto o que veremos na prxima seo. Por

i
i

cripto
2009/6/30
i page 9
Estilo OBMEP

i
i

9
enquanto, vamos s observar que tais cdigos so conhecidos como de
chave pblica, j que o processo (ou chave) de codificao pode ser
conhecido de qualquer um sem comprometer a segurana do cdigo.

Criptografia RSA
O mais conhecido dos mtodos de criptografia de chave pblica o
RSA. Este cdigo foi inventado em 1977 por R. L. Rivest, A. Shamir
e L. Adleman, que na poca trabalhavam no Massachussets Institute
of Technology (M.I.T.), uma das melhores universidades americanas.
As letras RSA correspondem s iniciais dos inventores do cdigo. H
vrios outros cdigos de chave pblica, mas o RSA continua sendo o
mais usado em aplicaes comerciais.

O Mtodo RSA
A descrio completa do funcionamento do RSA justamente o
tema desta apostila. Para entender como funciona precisaremos estudar vrias ideias e tcnicas novas de matemtica. Nesta seo explicaremos apenas o suficiente sobre o RSA para que voc entenda como
possvel um problema ser fcil de fazer e difcil de desfazer. Isto
tambm nos ajudar a identificar os problemas matemticos que precisaremos abordar para poder discutir os detalhes do funcionamento
do RSA.
Digamos que voc vai criar uma implementao do RSA para
uma determinada loja, que vai us-lo na codificao de dados de
clientes em compras pela internet. Para comear, voc precisa escolher

i
i

cripto
2009/6/30
i page 10
Estilo OBMEP

i
i

10
dois nmeros primos distintos e multiplic-los, obtendo um nmero
inteiro n. A loja manter secreta a informao sobre quais so os
primos escolhidos, porque isto que necessrio para decodificar as
mensagens enviadas usando a verso do RSA que voc est construindo. J n vai ser enviado para o computador de qualquer pessoa
que compre nessa loja pela web, porque dele que o computador do
usurio necessita para codificar os dados sobre o do carto de crdito
e envi-los ao computador da loja. Portanto, no caso do RSA, o problema fcil de fazer e difcil de desfazer simplesmente multiplicar
dois primos.
J consigo imaginar voc pensando:
S isso? Mas para desfazer o problema basta fatorar o
nmero e achar os primos!
verdade, mas h um detalhe que esqueci de contar: estes nmeros
primos sero muito, muito grandes. Na prtica uma chave segura de
RSA gerada a partir de nmeros primos de cerca de 100 algarismos
cada, de forma que n, que o produto destes primos, ter cerca de
200 algarismos. Acontece que, como veremos na pgina 29, podem
ser necessrios zilhes de anos para fatorar um nmero deste tamanho
e achar seus fatores primos mesmo se usarmos os mais poderosos
computadores existentes atualmente.
Resumindo:
para implementar o RSA escolhemos dois primos distintos muito
grandes p e q e calculamos o produto n = p q;
para codificar uma mensagem usamos n;

i
i

cripto
2009/6/30
i page 11
Estilo OBMEP

i
i

11
para decodificar uma mensagem usamos p e q;
n pode ser tornado pblico;
p e q precisam ser mantidos em segredo;
quebrar o RSA consiste em fatorar n, que leva muito tempo se
n for grande.

Teoria de Nmeros
O que vimos acima sugere que os principais problemas matemticos relacionados ao RSA so: como achar nmeros primos e como
fatorar um nmero. A rea da matemtica a que estes problemas pertencem conhecida como teoria de nmeros e tem por objetivo geral
o estudo das propriedades dos nmeros inteiros. Entre os problemas
que teremos que estudar para podermos descrever completamente o
RSA tambm esto:
como calcular os restos da diviso de uma potncia por um
nmero dado;
como achar um nmero que deixa restos especificados quando
dividido por uma srie de nmeros dados;
como estabelecer critrios de divisibilidade por nmeros primos.
H muitos outros problemas que so parte da teoria dos nmeros,
mas dos quais no trataremos aqui, entre eles:
1. calcular o mximo divisor comum entre dois nmeros dados;

i
i

cripto
2009/6/30
i page 12
Estilo OBMEP

i
i

12
2. determinar todos os inteiros a, b e c que satisfazem a2 + b2 = c2 ;
3. mostrar que se trs inteiros a, b e c satisfazem an + bn = cn ,
onde n > 2 um inteiro positivo, ento a, b ou c tm que ser
iguais a zero;
n

4. provar que 22 + 1 composto se n > 4;


5. provar que todo nmero par soma de dois primos mpares;
6. determinar todos os inteiros consecutivos que so potncias de
nmeros inteiros.
Os problemas acima tm grau de dificuldade muito varivel. A
soluo de alguns deles conhecida desde a antiguidade, como o
caso de (1) e (2). Na verdade, bem provvel que voc saiba resolver
(1) usando o mtodo descrito por Euclides em seu livro Elementos
escrito por volta de 300 a.C.; j (2) est relacionado ao Teorema de
Pitgoras o que talvez baste para lembrar-lhe de algumas solues
possveis.
Todas as outras questes so muito mais difceis. Para comear
temos (3), que muito parecida com (2), exceto pelo fato do expoente n ter que ser pelo menos 3. Este problema tem uma histria
muito interessante. Em algum momento entre 1621 e 1636 o francs
Pierre de Fermat, magistrado da corte de Toulouse, adquiriu uma
cpia da recm-publicada traduo latina da Aritmtica escrita pelo
matemtico grego Diofanto mais de mil anos antes. Fermat, que era
um matemtico amador, leu o texto de Diofanto, fazendo vrias anotaes na margem do texto. Em uma dessas anotaes ele afirmou
ter uma demonstrao do fato enunciado em (3) mas, segundo ele, o

i
i

cripto
2009/6/30
i page 13
Estilo OBMEP

i
i

13
espao disponvel na margem do livro no seria suficiente para conter
seu argumento.
improvvel que a demonstrao de Fermat estivesse correta,
j que o resultado permaneceu sem demonstrao at 1995. Como
este foi o ltimo resultado enunciado por Fermat a ser demonstrado,
tornou-se conhecido como o ltimo Teorema de Fermat. Para complicar, os mtodos usados por A. Wiles em sua prova de (3) so extremamente sofisticados e sequer existiam h 50 anos atrs.
A questo (4) outra que est ligada ao nome de Fermat. Na
verdade, o nmero
n
F (n) = 22 + 1
conhecido como o n-simo nmero de Fermat porque, em uma de
suas cartas a um outro matemtico, Fermat props que F (n) seria
sempre primo, qualquer que fosse o valor de n. De fato, calculando
F (n) para n de 0 a 4 obtemos os nmeros listados na tabela 2.
n
0
1
2
3
4

F (n)
3
5
17
257
65537

Tabela 2: Nmeros de Fermat primos


que so todos primos. Aparentemente, foi nessa tabela que Fermat
baseou-se para fazer a sua afirmao. Infelizmente, generalizar a partir de alguns casos sempre uma prtica perigosa em matemtica e,
neste caso, Fermat deu-se realmente mal. Nenhum nmero primo da

i
i

cripto
2009/6/30
i page 14
Estilo OBMEP

i
i

14
forma F (n) conhecido quando n > 4, da o problema enunciado em
(4), que ningum at hoje sabe como resolver.
A questo (5) conhecida como Conjectura de Goldbach, em homenagem a Christian Goldbach, um outro matemtico amador, que
viveu na mesma poca que Euler, com quem trocava frequentes cartas sobre matemtica. Embora se saiba que todo nmero par com
menos de 18 algarismos seja mesmo a soma de dois primos mpares,
ningum at hoje conseguiu provar o enunciado de Goldbach. Apesar
disso, alguns resultados parciais so conhecidos. Um dos mais recentes
foi a demonstrao descoberta em 2002 por Roger Heath-Brown e
Jan-Christoph Schlage-Puchta de que todo nmero par muito grande
pode ser escrito como a soma de dois primos mpares e exatamente
13 potncias de 2.
Se voc tentar descobrir duas potncias de inteiros pequenos, que
sejam consecutivas, vai logo dar de cara com 8 e 9, que so iguais
a 23 e 32 , respectivamente. Por mais que procure, no encontrar
outros exemplos. Em vista disso, o matemtico belga Eugne Charles
Catalan props em 1844 que essas duas potncias seriam as nicas
solues do problema (5). Isto correto, como foi provado pelo matemtico romeno Preda Mihilescu em 2002.
Talvez voc tenha percebido que, embora os enunciados das cinco
questes acima sejam muito fceis de entender, resolv-las pode ser
muito difcil: o ltimo Teorema de Fermat levou mais de 300 anos
para ser provado e o problema proposto por Catalan levou 158 anos.
Sem falar da conjectura de Goldbach e do problema relativo aos
nmeros de Fermat, que at hoje ningum sabe resolver.

i
i

cripto
2009/6/30
i page 15
Estilo OBMEP

i
i

Captulo 1

Nmeros Inteiros
Neste captulo estudaremos algumas propriedades bsicas dos nmeros inteiros que sero necessrias em nossa descrio do RSA no
captulo 6. Comearemos relembrando algumas definies bastante
simples.

1.1

Fatores e Nmeros Primos

Comeamos revisando algumas noes bsicas relativas divisibilidade de inteiros.

1.1.1

Divisores e Mltiplos

Um inteiro b divide outro inteiro a se existe um terceiro nmero


inteiro c tal que a = bc. Neste caso, tambm dizemos que b um
divisor ou fator de a, ou ainda que a mltiplo de b. Todas estas
15

i
i

cripto
2009/6/30
i page 16
Estilo OBMEP

i
i

16

CAP. 1: NMEROS INTEIROS

expresses significam a mesma coisa. Quando 1 < b < a, dizemos que


b um fator ou divisor prprio de a. Naturalmente s h dois divisores
que no so prprios, 1 e o prprio a. O nmero c, na definio acima
chamado de cofator de b em a. Por exemplo, 5 divide 20 porque
20 = 5 4. Neste exemplo 4 o cofator de 5 em 20.
Na prtica, determinamos que b divide a efetuando a diviso e
verificando que o resto zero. O cofator o quociente da diviso.
Nosso primeiro resultado uma lista das propriedades dos mltiplos.
Dois inteiros quaisquer sempre tm pelo menos 1 como fator comum; afinal, um divide qualquer inteiro. Se 1 for o nico fator comum a dois nmeros, diremos no tm fator prprio comum ou que
so primos entre si. Note que um par de primos distintos no tm
fator prprio comum. Mas h muitos nmeros compostos sem fator
prprio comum, como o caso de 6 e 35, por exemplo.
Propriedades dos Mltiplos. Sejam a, b, c e d quatro nmeros
inteiros.
1. d divide 0;
2. se d divide a e b, ento tambm divide a + b;
3. se d divide a ento divide a c.
Demonstrao. Vamos provar que cada uma destas propriedades
verdadeira. A primeira mais ou menos bvia porque
0 = 0 d;
de modo que 0 mltiplo de qualquer nmero. Para provar a segunda

i
i

cripto
2009/6/30
i page 17
Estilo OBMEP

i
i

N SEC. 1.1: FATORES E NMEROS PRIMOS

17

propriedade, observemos que dizer que d divide a e b significa, pela


definio, que existem inteiros a0 e b0 tais que
a = d a0 e b = d b0 ;
isto , estamos chamando de a0 e de b0 os cofatores de d em a e b,
respectivamente. Mas, usando as expresses acima,
a + b = (d a0 ) + (d b0 )
e pondo d em evidncia
a + b = d(a0 + b0 )
mostrando que d divide a + b, tendo a0 + b0 como cofator. Finalmente,
para mostrar (3), apenas multiplicamos a = d a0 por c, o que nos d,
c a = c (d a0 ) = d (c a0 );
de forma que d divide c a com cofator igual a c a0 .
Estas no so as nicas propriedades dos mltiplos, embora sejam
as mais importantes. Algumas outras propriedades so listadas no
prximo exerccio.
Exerccio 3. Sejam a, b e d nmeros inteiros. Suponha que d divide
a. Mostre que:
(a) se d tambm divide b ento d divide a b;
(b) se d tambm divide a + b ento d divide b;

i
i

cripto
2009/6/30
i page 18
Estilo OBMEP

i
i

18

CAP. 1: NMEROS INTEIROS

(c) se d tambm divide a b ento d divide b;


(d) a e a + 1 no podem ter nenhum fator prprio comum.
O prximo exerccio do Banco de Questes da OBMEP-2006
uma consequncia fcil destas propriedades.
Exerccio 4. Da igualdade 9 174 532 13 = 119 268 916 pode-se concluir que um dos nmeros abaixo divisvel por 13. Qual este
nmero?
(a) 119 268 903
(d) 119 268 913

1.1.2

(b) 119 268 907


(e) 119 268 923

(c) 119 268 911

Primos e Compostos

Se vamos decompor inteiros em primos, conveniente comearmos


recordando a definio de nmero primo. Um nmero inteiro p primo
se p 6= 1 e os nicos divisores de p so 1 e p. Portanto 2, 3, 5
e 7 so primos, mas 45 = 5 9 no primo. Um nmero inteiro,
diferente de 1, que no primo chamado de composto. Logo 45
composto.
Observe que a definio de primo exclui os nmeros 1. Isto ,
os nmeros 1 no so primos; mas tambm no so compostos!
Voltaremos a esta questo ao final do captulo.
Exerccio 5. Seja n > 1 um inteiro. Lembre-se que n! definido
como o produto de todos os nmeros inteiros positivos menores ou
iguais a n; isto
n! = 1 2 (n 1) n.

i
i

cripto
2009/6/30
i page 19
Estilo OBMEP

i
i

N SEC. 1.2: FATORANDO INTEIROS

19

Mostre que os nmeros


n! + 2, n! + 3, . . . , n! + (n 1)
so todos compostos.
Finalmente, uma questo histrica (ou melhor dizendo, etimolgica), voc j se perguntou porque os nmeros primos tm este
nome? O nome uma herana grega e, naturalmente, no se refere a
nenhuma relao de parentesco. Os gregos classificavam os nmeros
em primeiros ou indecomponveis e secundrios ou compostos. Os
nmeros compostos so secundrios por serem formados a partir dos
primos. Os romanos apenas traduziram literalmente a palavra grega
para primeiro, que em latim primus. da que vm nossos nmeros
primos.

1.2

Fatorando Inteiros

Nesta seo tratamos de maneira sistemtica um problema que


voc j deve ter aprendido a resolver: como fatorar um inteiro; isto ,
como encontrar todos os seus fatores primos. Comearemos descrevendo um problema mais simples: como calcular um fator (ou divisor)
de um nmero.

1.2.1

Encontrando um Fator

O procedimento mais bsico consiste em uma busca sistemtica


por um fator, comeando de 2 e prosseguindo at chegar ao nmero

i
i

cripto
2009/6/30
i page 20
Estilo OBMEP

i
i

20

CAP. 1: NMEROS INTEIROS

que se quer fatorar. Se nenhum fator for encontrado, podemos concluir que o nmero dado primo. Por exemplo, se queremos fatorar
91, devemos verificar se divisvel
por 2? no, pois mpar;
por 3? como 9 + 1 = 10 no divisvel por 3 ento 91 tambm
no ;
por 4? podemos pular 4 j que 91 mpar;
por 5? no, j que no acaba em 5 nem em 0;
por 6? outro par que podemos pular;
por 7? dividindo 91 por 7 achamos resto zero e quociente 13;
logo, 7 e 13 so fatores de 91. Note que houve bastante redundncia
neste processo. De fato, se 2 no divide 91, nenhum nmero par
vai dividi-lo. Com isto poderamos ter restringido as tentativas aos
mpares.
Exerccio 6. Generalize a afirmao feita no pargrafo acima, mostrando que, se um inteiro k divide outro inteiro m, que por sua vez
divide ainda outro inteiro n, ento k divide n.
Vamos parar para pensar um minuto. Este exerccio nos diz que
o que fizemos para 2 se aplica tambm a outros nmeros; 3, por
exemplo. Ento, se 3 no divide 91, nenhum mltiplo de 3 pode

i
i

cripto
2009/6/30
i page 21
Estilo OBMEP

i
i

N SEC. 1.2: FATORANDO INTEIROS

21

dividi-lo. Isto significa que, tendo verificado que 3 no divide 91


poderamos pular todos os seus mltiplos se o procedimento acima
tivesse continuado. Apesar de parecer uma ideia esperta, essa maneira
de proceder acaba sendo pouco til porque introduz uma complicao
extra no nosso mtodo de achar um fator. Afinal, para aplic-la,
teramos que ser capazes de detectar que um dado nmero mltiplo
de 3 para poder pul-lo. Se isto j complicado de fazer com 3,
imagine se tentssemos com 7 ou 13. Apesar disto, veremos na seo
7.2 do captulo 7 que a mesma ideia pode ser reciclada como um
mtodo para achar primos.
Nosso algoritmo para achar fatores tem algumas propriedades importantes que ainda precisamos analisar.

1.2.2

Algoritmo?

Como assim, algoritmo? Os matemticos chamam de algoritmo


qualquer mtodo sistemtico utilizado para fazer alguma coisa. Meio
vago, no? Afinal, uma receita de bolo e um conjunto de instrues
sobre como ir de uma cidade outra so mtodos sistemticos para
fazer alguma coisa, ou no? Claro que so, e nada nos impede
de cham-los de algoritmos (embora talvez no seja uma boa ideia
cham-los assim em pblico...). Alis uma receita um bom lugar
para comear, se queremos falar de algoritmos. Vejamos um exemplo.

i
i

cripto
2009/6/30
i page 22
Estilo OBMEP

i
i

22

CAP. 1: NMEROS INTEIROS

Po-de-l
Ingredientes:
3 xcaras de farinha de trigo;
3 ovos;
3 colheres de sopa de acar.
Modo de fazer: Ponha o forno para esquentar, em temperatura mdia, por 10 minutos. Enquanto isto, separe a clara e a gema dos ovos.
Bata as claras em neve. Acrescente as gemas e continue batendo
at que a misture fique bem clara. Adicione o acar e continue
batendo. Acrescente a farinha, uma colher de cada vez, misturando-a
bem massa com uma colher. Asse por mais ou menos vinte minutos.

Uma olhada rpida nesta receita nos mostra que vem em trs
partes: o ttulo, os ingredientes e o procedimento a ser seguido. O
ttulo nos diz o que vai resultar se fizermos a receita; neste caso, um
bolo, e no um biscoito ou um mingau. Os ingredientes indicam o que
precisamos ter mo para fazer o bolo. J o procedimento descreve
passo a passo o que devemos fazer para obter um bolo de verdade.
Todos os algoritmos, mesmo os de natureza matemtica, tm uma
estrutura semelhante receita acima. Ao ttulo da receita corresponde a sada do algoritmo; isto , o que vai resultar se utilizarmos
o algoritmo. Os ingredientes por sua vez, correspondem entrada do
algoritmo. No caso do algoritmo descrito na seo 1.2.1, a entrada

i
i

cripto
2009/6/30
i page 23
Estilo OBMEP

i
i

N SEC. 1.2: FATORANDO INTEIROS

23

o nmero do qual desejamos achar um fator. Finalmente, o procedimento da receita ... Bem, o procedimento do algoritmo ( difcil
dizer isto de outro jeito).
Podemos organizar nosso algoritmo segundo estas etapas. Como
geralmente h muitos algoritmos com a mesma entrada e sada,
costume dar um nome ao algoritmo que se descreve. Isto comum
em receitas tambm, como quando escrevemos Po-de-L da Vov
para distinguir uma receita de outra. Na verdade, os algoritmos so
frequentemente nomeados em homenagem a quem os criou. Como
nosso algoritmo to antigo que ningum lembra quem o inventou,
vamos cham-lo de Algoritmo acha-fator.

Algoritmo acha-fator
Entrada: um inteiro positivo n;
Sada: um fator prprio de n ou a concluso de que n primo;
Procedimento: tente dividir n por 2. Se for divisvel pre, pois
descobrimos que 2 fator de n, se no for, tente dividi-lo por 3. Se
for divisvel pre, pois descobrimos que 3 fator de n, se no for, tente
dividi-lo por 3. Continue desta maneira at encontrar um nmero que
divida n ou at que o candidato a divisor seja n. Neste ltimo caso,
n primo.
A nica coisa que os matemticos exigem de um algoritmo que
a execuo do procedimento que ele descreve sempre chegue ao fim.

i
i

cripto
2009/6/30
i page 24
Estilo OBMEP

i
i

24

CAP. 1: NMEROS INTEIROS

fcil dar exemplos de procedimentos que no param nunca. Que


tal este:
comece com k = 3; verifique se k divisvel por 2: se
for, pre; se no for, incremente k de 2 (isto , passe para
k + 2) e tente dividir novamente por 3; continue repetindo
isto enquanto um mltiplo de 2 no for encontrado.
Como nenhum nmero , simultaneamente, par e mpar, este procedimento vai se repetir para sempre, de modo que no um algoritmo.
Observe que no resta a menor dvida de que acha-fator satisfaz
esta condio. Afinal de contas, estamos procurando por fatores positivos de um nmero n que, por serem fatores, tm que ser menores
n. Mas, por maior que seja n, a quantidade de inteiros positivos
menores que n tem que ser finita. Logo, na pior das hipteses, visitamos cada um dos inteiros entre 2 e n sem achar fator e paramos
porque encontramos n que, neste caso, ser primo.

1.2.3

Algoritmo e Al-Khowarazmi

A origem da palavra algoritmo muito curiosa. Originalmente a


palavra era escrita algorismo, que vem da palavra rabe
Al-Khowarazmi, o homem de Khowarazm. Esse era o nome pelo
qual o matemtico rabe Ibn Musa ficou conhecido. Ele, que viveu no
sculo IX, escreveu um livro chamado Al-jabr wal muqabalah atravs
do qual o sistema de numerao usado na ndia chegou Europa
Medieval. por isso que, ainda hoje, falamos em algarismos indoarbicos. Alis algorismo e algarismo so variantes da mesma palavra
e significavam, originalmente, os numerais indo-arbicos.

i
i

cripto
2009/6/30
i page 25
Estilo OBMEP

i
i

N SEC. 1.2: FATORANDO INTEIROS

25

Figura 1.1: Al Khowarazmi

Com o passar do tempo, a palavra algorismo deixou de significar


apenas os nmeros e passou a ser usada tambm para descrever a
aritmtica e o clculo com nmeros. A maneira como algorismo ganhou um t no menos curiosa. Outra palavra usada para nmero
na Idade Mdia era aritmos que simplesmente nmero em grego.
Algum, em algum momento, confundiu-se na ortografia e misturou
as duas, trocando o s de algorismo pelo t de aritmos. Como, naquela
poca, os livros eram copiados mo, uns dos outros, o erro acabou
se propagando.
O sentido atual da palavra algoritmo, contudo, bem mais recente. No claro como a palavra passou a significar mtodo sistemtico, mas ela j estava sendo usada mais ou menos neste sentido
em 1800. Assim, algoritmo uma palavra muito antiga, mas que
ganhou um significado novo.
Voc reparou no nome do livro de Ibn Musa? Al-jabr no lhe
lembra nada? da que vem a palavra lgebra. Hoje em dia dizemos
que um algebrista um matemtico que trabalha em lgebra, mas

i
i

cripto
2009/6/30
i page 26
Estilo OBMEP

i
i

26

CAP. 1: NMEROS INTEIROS

este no era, originalmente, o significado da palavra. No passado, um


algebrista era um mdico que consertava ossos.
Mas chega de conversa mole, voltemos matemtica.

1.2.4

O Algoritmo acha-fator

O algoritmo acha-fator trs um bnus grtis: o fator que ele encontra , necessariamente, um nmero primo. Para entender o porqu,
lembre-se que o algoritmo consiste em fazer uma busca pelo fator de
um nmero n, comeando sempre por 2, que o menor fator prprio
positivo possvel para qualquer nmero. Por isso, o fator encontrado
por este algoritmo sempre o menor fator possvel p do nmero n
dado. Contudo, se p no for primo, ento admite um fator q < p.
Acontece que, segundo o exerccio 6, como q divide p, que divide n,
devemos ter que q divide n. Mas isto no possvel, uma vez que
q < p e j tnhamos concordado que p era o menor fator positivo
possvel de n.
Outro detalhe importante deste algoritmo que podemos parar
nossa busca, e decretar que n primo, muito antes de chegar a n. A
chave para entender isto , mais uma vez, o fato do algoritmo achar
sempre o menor fator do nmero n que se quer fatorar.
Para poder discutir os detalhes, suponhamos que o nmero inteiro
positivo n, que se deseja fatorar, composto. Neste caso o algoritmo
acha-fator encontra o menor fator p de n. Portanto, podemos escrever
n=pc
onde c o cofator de p como divisor de n. Contudo, c tambm um

i
i

cripto
2009/6/30
i page 27
Estilo OBMEP

i
i

27

N SEC. 1.2: FATORANDO INTEIROS

divisor de n. Levando em conta que p o menor destes divisores,


podemos escrever
c p.
Combinando esta desigualdade com a equao anterior, obtemos
n = p c p p.
Em outras palavras,
n p2

que equivalente a

n.

Resumimos o resultado final em uma proposio para referncia futura.


Proposio 1. Se n for composto, o menor fator prprio de n
menor ou igual raiz quadrada de n.
Assim, se n for composto, algum fator dever ser encontrado antes

de nossa busca ultrapassar n. Isto nos permite reformular o algoritmo acha-fator de maneira bem mais eficiente, como segue.

Algoritmo acha-fator

Entrada: um inteiro positivo n;


Sada: um fator prprio de n ou a concluso de que n primo;

i
i

cripto
2009/6/30
i page 28
Estilo OBMEP

i
i

28

CAP. 1: NMEROS INTEIROS

Procedimento: tente dividir n por 2. Se for divisvel pre, pois


descobrimos que 2 fator de n, se no for, tente dividi-lo por 3. Se
for divisvel pre, pois descobrimos que 3 fator de n, se no for, tente
dividi-lo por 5. Continue desta maneira at encontrar um nmero que

divida n ou at que o candidato a divisor seja maior que n. Neste


ltimo caso, n primo.

Naturalmente, a nica diferena entre esta verso e a anterior


que paramos assim que o divisor a ser experimentado ultrapassa a raiz
quadrada de n. Com isto, buscamos o divisor entre uma quantidade
muito menor de inteiros do que vnhamos fazendo anteriormente.
Finalmente, convm resumir tudo o que aprendemos nesta subseo como uma proposio.
Proposio 2. O fator de um nmero inteiro n > 1 encontrado pelo
algoritmo acha-fator acima sempre um nmero primo menor ou
igual que a raiz quadrada de n.
Encerraremos este tpico com dois exerccios.
Exerccio 7. Seja n um nmero inteiro positivo composto e p seu
menor fator primo. Sabe-se que:
1. p

n;

2. p 4 divide 6n + 7 e 3n + 2.
Determine todos os possveis valores de n.
Desafio 1. Qual o maior nmero possvel de fatores primos de um
inteiro n que no tem nenhum fator n1/3 ?

i
i

cripto
2009/6/30
i page 29
Estilo OBMEP

i
i

N SEC. 1.2: FATORANDO INTEIROS

1.2.5

29

Custo da Fatorao

Apesar de ser fcil de entender e de utilizar, o algoritmo acha-fator


muito ineficiente, mesmo se usarmos um computador. Isto facilmente ilustrado se estimarmos o tempo que um computador levaria
para achar um fator de um nmero grande usando este algoritmo.
Lembre-se que, tendo n por entrada, acha-fator executa no m
ximo n tentativas de diviso antes de encontrar um fator para n.
Na verdade, o pior caso possvel ocorre quando precisamos efetuar

exatamente n tentativas de diviso, o que corresponde a dizer que


n primo. precisamente este o caso cujo tempo de execuo vamos
estimar.
Para fixar as ideias, consideremos um nmero primo p, de 100 ou

mais algarismos. Isto p 10100 e, portanto, p 1050 . Assim,


precisaremos executar pelo menos 1050 divises para garantir que p
primo pelo algoritmo acha-fator. Para transformar isto em tempo
de clculo, precisamos ter uma ideia de quantas divises um computador capaz de efetuar em um segundo. Vamos exagerar e supor
que usamos um supercomputador capaz de executar 1010 divises por
segundo. Para voc ter uma ideia de quo exagerado isto , o computador no qual estou escrevendo esta apostila no faz mais do que
50 divises por segundo!
Seja como for, usando nosso suposto supercomputador, precisaramos de, pelo menos,
1050
= 1040 segundos
1010

i
i

cripto
2009/6/30
i page 30
Estilo OBMEP

i
i

30

CAP. 1: NMEROS INTEIROS

para determinar que n primo usando acha-fator. Como um ano tem


60 60 24 365 = 31 536 000 segundos,
conclumos que 1040 segundos corresponde a
1040
31 536 000
que aproximadamente igual a
317 000 000 000 000 000 000 000 000 000 000

(so 30 zeros)

anos que muito mais tempo do que conseguimos imaginar. Afinal


de contas, as ltimas estimativas da idade do universo indicam que
no deve ultrapassar 20 bilhes de anos; ou seja
200 000 000 000

(meros 11 zeros)

anos. Podemos, portanto, concluir, sem qualquer receio, que impossvel confirmar que um nmero de 100 ou mais algarismos primo
usando este algoritmo.
Isto significa que o algoritmo intil? Certamente que no. Se
vamos fatorar um inteiro sobre o qual nada sabemos, h sempre a possibilidade que tenha um fator primo pequeno, digamos menor que um
milho. Neste caso, o acha-fator encontrar um tal fator rapidamente.

i
i

cripto
2009/6/30
i page 31
Estilo OBMEP

i
i

N SEC. 1.2: FATORANDO INTEIROS

1.2.6

31

Fatorando Nmeros Inteiros

At aqui vimos apenas como encontrar um fator prprio de um


nmero inteiro n, se existir tal fator, ou comprovar que o nmero
primo. Entretanto, nosso objetivo inicial era bem mais ousado:
queramos escrever n como produto de potncias de nmeros primos.
Mas, de posse do algoritmo acha-fator, isto fcil de fazer, basta
aplicar acha-fator vrias vezes. Vejamos um exemplo.
Considere o inteiro 12 103. Aplicando o algoritmo acha-fator a este
nmero (deixo as contas para voc fazer) achamos o fator 7. Como
12 103
= 1 729,
7
temos que
12 103 = 7 1 729.
Como os fatores encontrados por acha-fator so sempre primos, sabemos que 7 primo. Portanto, s necessrio aplicar acha-fator novamente ao cofator 1 729 de 7 em 12 103.
Aplicando acha-fator a 1 729, descobrimos que 7 tambm fator
deste nmero. Mas,
1 729
= 247,
7
de modo que
12 103 = 7 1 729 = 7 (7 247) = 72 247.
Novamente, resta-nos aplicar acha-fator ao cofator 247. Desta vez,

i
i

cripto
2009/6/30
i page 32
Estilo OBMEP

i
i

32

CAP. 1: NMEROS INTEIROS

o fator encontrado 13 e
247
= 19,
13
de modo que
12 103 = 72 247 = 72 (13 19).

Contudo, 19 = 4, 35... e fcil verificar que 19 no divisvel por 2,


nem 3. Isto nos permite concluir, pela proposio 2 que 19 primo.
Reunindo tudo isto conclumos que a fatorao de 12 103 em potncias de primos
12 103 = 72 13 19.
Uma maneira bastante ilustrativa de organizar os clculos que fizemos
acima disp-los ao longo de ramos, da seguinte forma:
12 103

ww
ww
w
ww
ww
7
1 719
w
ww
w
w
ww
ww
7
247
w
ww
w
w
ww
ww

13
19

w
ww
ww
w
w
ww

19
1

Quando este algoritmo efetuado no papel, costume organiz-lo da


seguinte maneira:

i
i

cripto
2009/6/30
i page 33
Estilo OBMEP

i
i

33

N SEC. 1.2: FATORANDO INTEIROS

12 103
12 103
12 103
12 103
1 729
247
247
247
19
19
19
19

2
3
5
7
7
9
11
13
13
15
17
19

...
...
...
...
...
...
...
...
...
...
...
...

no divisvel
no divisvel
no divisvel
divisvel
divisvel
no divisvel
no divisvel
divisvel
no divisvel
no divisvel
no divisvel
divisvel

A primeira coisa a observar que, desta maneira, executamos o


algoritmo acha-fator algumas vezes sucessivamente de maneira sistemtica; sempre sobre o cofator do primo achado na rodada anterior. A segunda coisa tem a ver com a passagem da quarta para a
quinta linha. Na quarta linha achamos 7 como fator de 12 103; o cofator encontrado foi 1 729. A partir da quinta linha deveramos aplicar
acha-fator a 1 729 mas, estranhamente, comeamos de 7 e no de 2:
por qu? A explicao est no prximo exerccio.
Exerccio 8. Seja n um inteiro positivo e p o fator encontrado pelo
algoritmo acha-fator. Se c o cofator de p como divisor de n, mostre
que o menor fator que c pode ter p.
Podemos formular tudo o que fizemos at agora da seguinte maneira:

i
i

cripto
2009/6/30
i page 34
Estilo OBMEP

i
i

34

CAP. 1: NMEROS INTEIROS

Teorema da Fatorao. Dado um inteiro positivo n 2 podemos


sempre escrev-lo na forma
n = pe11 . . . pekk ,
onde 1 < p1 < p2 < p3 < < pk so nmeros primos e e1 , . . . , ek
so inteiros positivos.
Os expoentes e1 , . . . , ek na fatorao acima so chamados de multiplicidades. Assim, a multiplicidade de p1 na fatorao de n e1 .
Observe que n tem k fatores primos distintos, mas que a quantidade
total de fatores primos (distintos ou no) a soma da multiplicidades
e1 + + ek . Por exemplo, na fatorao
12 103 = 72 13 19;
o primo 7 tem multiplicidade 2, ao passo que 13 e 19 tm multiplicidade 1 cada.

Figura 1.2: C. F. Gauss

i
i

cripto
2009/6/30
i page 35
Estilo OBMEP

i
i

N SEC. 1.2: FATORANDO INTEIROS

35

O primeiro a enunciar o resultado acima foi C.F. Gauss no 16 de


seu famoso livro Disquisitiones arithmetic. Isto no significa que este
fato no houvesse sido usado implicitamente por matemticos desde
a Grcia Antiga. Afinal Euclides j havia provado na Proposio 31
do Livro VII de seus Elementos que
todo nmero composto divisvel por algum primo.

1.2.7

O Teorema da Fatorao nica

Para ser honesto, h mais sobre a fatorao de inteiros do que


o enunciado acima leva a crer. De fato cada inteiro maior que 1
admite apenas uma fatorao, desde que, como no enunciado acima,
ordenemos os primos em ordem crescente e agrupemos primos iguais
em uma nica potncia. Isto pode parecer bvio afinal, quem j viu
acontecer de duas pessoas obterem fatoraes diferentes de um mesmo
nmero? mas no . Discutiremos esta questo com mais detalhes
na seo seguinte. O enunciado final do teorema da fatorao, incluindo sua unicidade, dado a seguir.
Teorema da Fatorao nica. Dado um inteiro positivo n 2
podemos escrev-lo, de modo nico, na forma
n = pe11 . . . pekk ,
onde 1 < p1 < p2 < p3 < < pk so nmeros primos ao passo que
e1 , . . . , ek so inteiros positivos.
Tendo o enunciado preciso deste teorema, podemos explicar porque
necessrio excluir 1 da definio de nmero primo. A verdade

i
i

cripto
2009/6/30
i page 36
Estilo OBMEP

i
i

36

CAP. 1: NMEROS INTEIROS

que, se no fizssemos isto no poderamos falar da unicidade da fatorao no teorema acima. Por exemplo, se 1 fosse primo, ento
2 e 12 2 seriam duas fatoraes distintas do nmero 2. Usando a
mesma ideia de multiplicar o nmero por uma potncia de 1
(ou de 1) teramos uma infinidade de fatoraes distintas para cada
inteiro. Para excluir este tipo de fatorao trivial, dizemos que 1
no so primos.
No provaremos a unicidade da fatorao nesta apostila, mas os
detalhes podem ser encontrados nas referncias [2, captulo 2], [1]
ou [3]. Para que voc possa apreciar a importncia da unicidade na
fatorao, aqui esto dois exerccios que seriam muito difceis de fazer,
no fosse por ela (especialmente o 10). Ao fazer os exerccios procure
identificar exatamente onde est utilizando a unicidade da fatorao.
Exerccio 9. Determine se existem inteiros positivos x, y e z que
satisfaam a equao 30x 35y = 21x 140 52x .
Exerccio 10. Determine se existem inteiros positivos x, y e z que
satisfaam a equao 2x 34 26y = 39z .
Exerccio 11. Seja n um inteiro positivo e p > 1 um nmero primo
que divide n. Mostre que a multiplicidade de p na fatorao de n o
maior expoente e tal que pe divide n.
O prximo exerccio apareceu originalmente no Banco de Questes
da OBMEP-2007 (p. 99).
Exerccio 12. Quais nmeros naturais m e n satisfazem a
2n + 1 = m2 ?

i
i

cripto
2009/6/30
i page 37
Estilo OBMEP

i
i

Captulo 2

Aritmtica Modular
Neste captulo estudaremos a aritmtica dos fenmenos peridicos;
isto , aqueles que se repetem a intervalos regulares. No dia-a-dia nos
deparamos constantemente com fenmenos deste tipo: o dia que tem
24 horas, a semana que tem 7 dias, o ano que tem 365 dias, a OBMEP
ocorre uma vez a cada ano e o Colquio Brasileiro de Matemtica uma
vez a cada dois anos, s para citar alguns.

2.1

Fenmenos Peridicos e Aritmtica

Naturalmente, o que caracteriza os fenmenos peridicos o fato


de se repetirem com regularidade. O tempo que decorre entre uma
ocorrncia e outra destes fenmenos chamado de perodo do fenmeno. Assim, a Terra leva 24 horas. para dar uma volta em torno
de si mesma, de forma que seu perodo de rotao de 24 horas. J
o perodo de revoluo da Terra de 365 dias e um quarto, e corres37

i
i

cripto
2009/6/30
i page 38
Estilo OBMEP

i
i

38

CAP. 2: ARITMTICA MODULAR

ponde ao menor tempo que leva para dar uma volta em torno do Sol.
A Lua, por sua vez, tem perodo de rotao de 27 dias e perodo de
revoluo (em torno da Terra) de 27 dias.
Antes que voc ache que encontrou um erro tipogrfico (Ele estava
distrado e repetiu o mesmo nmero do perodo de translao!) deixa
eu esclarecer que no se trata disto. Na verdade, os perodos de
revoluo da Lua em torno da Terra e de sua rotao em torno de
seu prprio eixo so exatamente os mesmos, e por isso que a Lua
sempre tem a mesma face voltada para a Terra. Se voc est pensando
mas que incrvel coincidncia!, ento prepare-se para um desapontamento. A verdade que esta coincidncia de perodos foi causada
por um efeito de frico relacionado s mars que a Lua provoca na
Terra. Fascinante, no?

2.1.1

Horrios Escolares

Quando um fenmeno quase que perfeitamente peridico, tudo


se passa como se a histria do fenmeno se repetisse cada vez que
o perodo se completa. Em outras palavras, se conhecemos quanto
vale o perodo de um tal fenmeno, tudo que precisamos saber a seu
respeito pode ser resumido em uma descrio do que ocorre ao longo
da passagem de um perodo.
Vivemos isto todo dia, por exemplo, nos horrios de aula de uma
escola. Embora seja necessrio descrever os horrios de aula de cada
matria ao longo de todo o ano, simplificamos esta tarefa utilizando o
fato destes horrios se repetirem a cada sete dias. Assim, descrevendo
a distribuio de aulas ao longo de uma semana, podemos estend-la

i
i

cripto
2009/6/30
i page 39
Estilo OBMEP

i
i

39

N SEC. 2.1: FENMENOS PERIDICOS E ARITMTICA

para todo o ano letivo, simplesmente repetindo o mesmo horrio a


cada semana.
Por exemplo, imagine que sua me lhe pergunta se voc ter aula
de matemtica no dia 23 de setembro. Para responder esta pergunta
basta voc descobrir em que dia da semana cai 23 de setembro e olhar
o seu horrio. Como hoje segunda-feira 10 de setembro e como
23 10 = 13, o dia 23 est a 13 dias desta segunda. Por outro
lado, 13 = 7 + 6. S que, passado sete dias, estaremos de volta a
uma segunda-feira e, a seis dias desta segunda temos um domingo;
portanto, a resposta que no h aula de matemtica neste dia
qualquer que seja o seu horrio escolar.
Antes de encerrar este exemplo, faamos uma anlise matemtica
mais detalhada do procedimento usado para resolver o problema do
pargrafo anterior. Em primeiro lugar, precisamos conhecer a periodicidade do horrio, que de 7 dias, e quanto tempo vai passar
entre hoje e o dia no qual queremos saber se vai ou no haver aula de
matemtica. Se d dias vo se passar, dividimos d por 7 e tomamos
nota do quociente q e do resto r desta diviso. Mas, a cada sete
dias camos no mesmo dia da semana que hoje. Portanto, daqui a
d r = 7 q dias tero passado exatamente q semanas e estaremos de
volta a uma segunda-feira, como o dia de hoje. O dia da semana
daqui a d dias pode ento ser determinado a partir do resto r conforme
mostra a tabela 2.1.

Resto
Dia

0
segunda

1
tera

2
quarta

3
quinta

4
sexta

5
sbado

6
domingo

Tabela 2.1: Dias da semana

i
i

cripto
2009/6/30
i page 40
Estilo OBMEP

i
i

40

CAP. 2: ARITMTICA MODULAR

A observao crucial que, do ponto de vista deste problema,


quaisquer dois dias que diferem por um intervalo de sete
dias, representam o mesmo dia da semana.
Uma vez que isto tenha sido observado, o problema se reduz a determinar o resto da diviso de um dado nmero pelo perodo do problema,
que neste caso 7.

2.1.2

Um Jogo de Tabuleiro

Embora seja natural comear pensando no perodo como o intervalo de tempo entre duas ocorrncias de um dado fenmeno, esta no
sua nica aplicao. Para um exemplo que no envolve tempo, considere um jogo de dados cujo tabuleiro formado por um caminho
quadrado na forma ilustrada na tabela 2.2.
I

Tabela 2.2: A tabela do jogo


No incio do jogo, todos os jogadores devem pr suas peas na
casa inicial marcada com o I. Para sair desta casa, cada jogador deve

i
i

cripto
2009/6/30
i page 41
Estilo OBMEP

i
i

N SEC. 2.1: FENMENOS PERIDICOS E ARITMTICA

41

atirar o dado duas vezes consecutivas. Se tirar q da primeira vez e


r na segunda, deve andar 6q + r casas no sentido dos ponteiros do
relgio. claro que tanto r como q s podem ser nmeros entre 1 e
6, j que foram tirados no dado. Por exemplo, se tirei 3 na primeira
jogada do dado e 2 na segunda, devo andar
6 3 + 2 = 20 casas no tabuleiro.
Os jogadores continuam atirando os dados desta maneira e avanando
ao longo do tabuleiro. Quem chegar primeiro casa final, marcada
com I, ganha o jogo.
Digamos que, depois de um certo nmero de jogadas, voc se
encontra na casa do tabuleiro marcada com na tabela 2.3.
I

Tabela 2.3: Quanto ganhar para encerrar o jogo?


A pergunta :
Quanto voc tem que tirar em cada uma das jogadas do
dado para ganhar o jogo nesta rodada?
Uma simples contagem mostra que h 21 casas entre a posio que

i
i

cripto
2009/6/30
i page 42
Estilo OBMEP

i
i

42

CAP. 2: ARITMTICA MODULAR

voc est neste momento e a casa inicial. Mas 21 pode ser escrito na
forma
21 = 6 3 + 3,
de modo que, para ganhar nesta rodada preciso tirar 3 nas duas jogadas do dado. Note que, mais uma vez, o clculo matemtico requerido para resolver o problema foi uma diviso.
Exerccio 13. Quanto voc deve tirar nas duas jogadas do dado para
ganhar em uma jogada a partir da posio marcada pelo no tabuleiro
2.4?
I

Tabela 2.4: Tabela do Exerccio 13


Uma pergunta interessante est formulada no prximo problema.
Exerccio 14. Ser que possvel ganhar o jogo j na primeira rodada? Quanto algum teria que tirar em cada uma dos lances de
dados para que isto acontecesse?
Uma coisa ruim deste jogo que ele pode nunca terminar.
Exerccio 15. D exemplo de uma sucesso infinita de jogadas que
faz com que o jogo nunca acabe para um determinado jogador.

i
i

cripto
2009/6/30
i page 43
Estilo OBMEP

i
i

N SEC. 2.1: FENMENOS PERIDICOS E ARITMTICA

2.1.3

43

Prova dos Nove

Outra situao em que o perodo no corresponde a uma variao


de tempo ocorre na prova dos nove que aprendemos a fazer no ensino
fundamental. Por exemplo, so dados dois nmeros que queremos
somar; digamos que so 175 e 234. Efetuamos o resultado e obtemos
+

175
234 .
409

Para conferir se fizemos a conta corretamente, somamos os algarismos das duas parcelas, subtraindo nove cada vez que a soma chegue,
ou passe, de nove ou, como costume dizer, fazendo noves fora.
Aplicando a prova dos nove ao exemplo acima somamos 1 + 7 + 5
que d 13, noves fora 4 (isto , 13 9 = 4). Continuando, somamos
os algarismos da segunda parcela: 4 + 2 + 3 = 9, noves fora zero, de
modo que as parcelas do como resultado 4+0 = 4. Se a conta estiver
correta, devemos obter 4 ao aplicar o mesmo processo ao resultado
que calculamos. Mas, 13 noves fora d 4, que era o valor esperado.
Isto indica (mas no garante!) que a conta esteja certa.
Observe que, ao fazer noves fora, estamos calculando o resto da
diviso de um nmero por 9. Na prtica, a prova dos nove consiste
em calcular o resto de diviso de uma soma por 9 de duas maneiras
diferentes, como veremos na pgina 66.
Exerccio 16. D exemplo onde a prova dos nove falha. Explique
o que precisa acontecer para que a prova dos nove no seja capaz de
detectar um erro cometido em uma adio.

i
i

cripto
2009/6/30
i page 44
Estilo OBMEP

i
i

44

CAP. 2: ARITMTICA MODULAR

Exerccio 17. A prova dos nove tambm funciona para a multiplicao. D exemplo de uma multiplicao errada que a prova dos nove
no detecta como tal. Explique o que precisa acontecer para que a
prova dos nove no seja capaz de detectar um erro cometido em uma
multiplicao.

2.1.4

Restos de Inteiros

Nos exemplos anteriores, resolvemos os problemas propostos usando diviso de inteiros com resto. Isto sugere que o prprio resto da
diviso se comporta de maneira peridica. Por exemplo, os mltiplos
de 2 se repetem de dois em dois e, portanto, com perodo igual a
2. J os mltiplos de 3 tm perodo 3 e os de 12, perodo 12. Mais
precisamente,
os restos dos inteiros sucessivos na diviso por um inteiro
positivo qualquer n repetem-se com perodo n.
Por exemplo, dividindo os nmeros de 0 em diante por 4, obtemos os
restos como na tabela 2.5.
Inteiros
Restos

0
0

1
1

2
2

3
3

4
0

5
1

6
2

7
3

8
0

9
1

10
2

11
3

12
0

13
1

14
2

Tabela 2.5: Alguns restos mdulo 4


Em geral, dividindo um inteiro positivo a por outro inteiro positivo
n, obtemos
a = nq + r e 0 r < n.

i
i

cripto
2009/6/30
i page 45
Estilo OBMEP

i
i

N SEC. 2.2: DEFINIES E PRIMEIRAS PROPRIEDADES

45

Por isto, o mesmo resto na diviso por n se repete a cada n inteiros


sucessivos, como vimos, experimentalmente, na tabela 2.5 para o caso
n = 4.
Em vista disto, podemos dizer que os restos da diviso por n se
repetem com perodo exatamente igual a n. Note que se trata aqui
de uma extenso da utilizao usual da palavra perodo que, neste
contexto, no se refere a um intervalo de tempo. Para evitar confuso
chamaremos estes perodos generalizados de mdulos.

2.2

Definies e Primeiras Propriedades

hora de sistematizar os clculos efetuados nos vrios exemplos


da seo anterior e de considerar algumas aplicaes elementares.

2.2.1

Sistematizando

Para comear, temos um inteiro positivo n que representa o perodo


ou mdulo do fenmeno que estamos estudando. Dias, anos, horas e
casas na tabela so todos dados por nmeros inteiros e este o nico
caso que vamos considerar. Isto , no vamos tratar de perodos como
365 41 que o nmero exato de dias que formam um ano. Alis, por
isso que a cada quatro anos temos um ano bissexto, que aquele no
qual fevereiro tem 29 dias.
Analisando cada um dos trs exemplos vistos na seo anterior,
verificamos que:
no calendrio a cada sete dias estamos no mesmo dia da semana.

i
i

cripto
2009/6/30
i page 46
Estilo OBMEP

i
i

46

CAP. 2: ARITMTICA MODULAR

no jogo a cada 32 movimentos de uma pea, chegamos mesma casa


do tabuleiro;
na prova dos nove cada vez que a soma d maior ou igual a 9,
retemos apenas sua diferena por 9;
nos restos a cada n inteiros obtemos um nmero que deixa o mesmo
resto na diviso por n.
Lembrando que o mdulo , no primeiro caso 7, no segundo 38, no
terceiro 9 e no quarto n, vamos fazer a seguinte definio:
se n o mdulo e a e b so nmeros inteiros, ento diremos
que a congruente a b mdulo n se a b um mltiplo
de n.
Assim:
o nmero de dias que se passaram, desde primeiro de janeiro,
entre dois sbados de um mesmo ano so congruentes
mdulo 7;
o nmero de movimentos de uma pea, desde o comeo do jogo,
ao final de duas jogadas diferentes que levam a pea a uma
mesma casa do tabuleiro so congruentes mdulo 32;
dois nmeros que so iguais noves fora, diferem por um mltiplo
de 9;
dois inteiros com o mesmo resto na diviso por n so congruentes
mdulo n.

i
i

cripto
2009/6/30
i page 47
Estilo OBMEP

i
i

47

N SEC. 2.2: DEFINIES E PRIMEIRAS PROPRIEDADES

Se dois inteiros a e b so congruentes mdulo n, escrevemos


a b (mod n);
se no so congruentes, escrevemos
a 6 b (mod n).
Assim,
3 8 (mod 5), ao passo que 3 6 8

(mod 7).

Por outro lado,


3 25 (mod 7), embora 3 6 25

2.2.2

(mod 5).

Propriedades da Congruncia Modular

A congruncia modular satisfaz algumas propriedades que a tornam muito semelhante igualdade usual. As propriedades mais elementares da igualdade so as seguintes:
reflexiva todo nmero igual a si prprio;
simtrica se a = b ento b = a;
transitiva se a = b e b = c, ento a = c.
Na verdade, costumamos usar estas propriedades da igualdade sem
ter sequer conscincia que o fazemos.

i
i

cripto
2009/6/30
i page 48
Estilo OBMEP

i
i

48

CAP. 2: ARITMTICA MODULAR

No caso da congruncia modular no assim to bvio que estas propriedades so satisfeitas, mas podemos verific-las sem muito
trabalho como faremos adiante. Antes porm, convm perguntarmos para que fazer o esforo de provar que estas propriedades valem
para a congruncia modular. Ser mera curiosidade? A resposta,
naturalmente, que no se trata apenas de curiosidade: precisamos
dessas propriedades para poder utilizar de forma correta a congruncia modular nas contas que faremos nas prximas sees, incluindo-se
a codificao de uma mensagem pelo RSA. para isto que vamos
provar que a congruncia modular satisfaz propriedades anlogas s
enunciadas acima para a igualdade; mais precisamente:
reflexiva todo nmero congruente mdulo n a si prprio;
simtrica se a b (mod n) ento b a (mod n);
transitiva se a b (mod n) e b c (mod n)
ento a c (mod n);
onde n um inteiro positivo.
Para mostrar que a congruncia mdulo n reflexiva, devemos
verificar que
a a (mod n).
Mas, pela definio, isto o mesmo que dizer que a a = 0 mltiplo
de n. Contudo, zero mltiplo de qualquer inteiro n, uma vez que
0 n = 0.
Passemos simtrica. Pela definio de congruncia mdulo n,
a b (mod n) o mesmo que dizer que a b mltiplo de n. Em

i
i

cripto
2009/6/30
i page 49
Estilo OBMEP

i
i

N SEC. 2.2: DEFINIES E PRIMEIRAS PROPRIEDADES

49

outras palavras, se a b (mod n) ento existe algum inteiro k tal


que
a b = k n.
Multiplicando esta equao por 1, obtemos
b a = (k) n;
isto , b a mltiplo de n, ou ainda, b a (mod n).
Para a transitiva, tomamos por hiptese que
a b (mod n) e que b c (mod n).
Mas estas duas congruncias se traduzem, por definio, nas igualdades
a b = k n e b c = ` n,
onde k e ` so inteiros escolhidos de maneira adequada. Somando
estas duas ltimas equaes,
(a b) + (b c) = k n + ` n.
Cancelando o b esquerda e usando a distributividade da direita,
obtemos
a c = (k + `) n,
que equivalente congruncia a c (mod n), como requerido pela
propriedade transitiva.

i
i

cripto
2009/6/30
i page 50
Estilo OBMEP

i
i

50

2.2.3

CAP. 2: ARITMTICA MODULAR

Resduos

Antes de prosseguir, precisamos estudar em mais detalhes a relao entre a congruncia mdulo n e a divisibilidade de inteiros, j
que isto que torna a congruncia to til. Para comear, observe
que a propriedade reflexiva da congruncia mdulo n equivalente
afirmao de que zero divisvel por n. Por sua vez, propriedade
simtrica equivale a dizer que se um dado nmero divisvel por n
ento, ao multiplic-lo por 1, obtemos outro mltiplo de n. Finalmente, a transitiva nos diz apenas que a soma de mltiplos de n
tambm um mltiplo de n. Em outras palavras, as trs propriedades
que provamos correspondem s propriedades dos mltiplos listadas na
proposio em 1.1.1.
Mas podemos ir bem mais longe que isto. Digamos que a um
inteiro positivo. Dividindo a por n temos
a = n q + r e 0 r < n.
Assim,
a r = n q;
que equivale a dizer que
ar

(mod n).

Verificamos com isto que todo inteiro positivo congruente mdulo n


ao resto de sua diviso por n, que um nmero entre 0 e n.
Em geral, se a r (mod n) e 0 r < n, dizemos que r o
resduo de a mdulo n. Note que usamos o artigo definido ao definir

i
i

cripto
2009/6/30
i page 51
Estilo OBMEP

i
i

N SEC. 2.2: DEFINIES E PRIMEIRAS PROPRIEDADES

51

resduo: o resduo e no um resduo. Isto porque cada nmero s


pode ter um resduo mdulo n. De fato, se
ar

(mod n) com 0 r n 1;

a r0

(mod n) com 0 r0 n 1;

ento, pelas propriedades simtrica e transitiva, temos que


r r0 (mod n). Digamos que r r0 . Pela definio da congruncia,
isto significa que r r0 um mltiplo de n. Mas tanto r, quanto r0
so menores que n, de modo que 0 r r0 < n. Isto significa que
r r0 s pode ser mltiplo de n se o cofator correspondente for zero;
o que nos d r = r0 , mostrando que os dois resduos, r e r0 tm que
ser iguais.
Aparentemente a nica coisa que fizemos ao introduzir os resduos
foi inventar um nome novo para o resto, mas no bem assim. Note
que o termo resduo se aplica a qualquer inteiro, positivo ou negativo,
ao passo que o resto geralmente usado quando dividimos um inteiro
positivo por n. O que ocorre, ento, se a for negativo?
Para tornar o argumento mais claro, convm comear com um
exemplo. Seja n = 6 e a = 55. Nosso objetivo calcular o resduo
de 55 mdulo 6; em outras palavras, queremos achar um inteiro
0 r < 6 tal que 55 r (mod 6). Poderamos proceder por
tentativa, mas vamos tratar o problema de maneira mais sistemtica
para podermos lidar mesmo com o caso em que o n for grande. Para
isto, dividimos 55 por 6, obtendo quociente 9 e resto 1:
55 = 9 6 + 1.

i
i

cripto
2009/6/30
i page 52
Estilo OBMEP

i
i

52

CAP. 2: ARITMTICA MODULAR

Multiplicando tudo por 1,


55 = (9) 6 1,
de forma que
55 1

(mod 6).

Observe que 1 no o resduo de 55 mdulo 6 porque 1 negativo. Contudo, como 6 = 5 (1), obtemos
1 5 (mod 6);
e a propriedade transitiva da congruncia nos permite concluir que
55 5 (mod 6).
Portanto, 55 tem resduo 5 mdulo 6.
Para tratar o caso geral, podemos seguir as etapas do exemplo
acima. Primeiramente, como estamos supondo que a negativo, ento
a deve ser positivo. Dividindo-o por n,
a = n q + r

e 0 r < n,

onde q e r so o quociente e o resto da diviso. Multiplicando esta


equao por 1, obtemos
a = n (q) r

e 0 r < n;

isto
a r

(mod n)

e 0 r < n.

i
i

cripto
2009/6/30
i page 53
Estilo OBMEP

i
i

N SEC. 2.2: DEFINIES E PRIMEIRAS PROPRIEDADES

53

Se r = 0, ento a 0 (mod n) e j achamos o resduo. Se r 6= 0,


ento (n r) (r) = n nos diz que
r n r

(mod n),

de modo que a transitividade da congruncia nos permite concluir que


anr

(mod n).

Ainda precisamos nos certificar que n r um resduo mas, para isto,


basta verificar que est entre 0 e n 1. Como r 0 e r 6= 0, temos
que r > 0. Logo n r < n. Entretanto, r < n, donde conclumos que
n r > 0.
Para poder descrever o que fizemos de maneira sucinta, definimos

a se a 0;
|a| =
a se a < 0;
que chamado de mdulo de a. Por exemplo,
|4| = 4 ao passo que | 5| = 5.
Proposio 3. Sejam a e n > 1 nmeros inteiros e r o resto da
diviso de |a| por n, ento o resduo de a mdulo n igual a:
0 se r = 0;
r se r 6= 0 e a 0;
n r se r 6= 0 e a < 0.

i
i

cripto
2009/6/30
i page 54
Estilo OBMEP

i
i

54

CAP. 2: ARITMTICA MODULAR

Vejamos um exemplo:
Quais so os resduos possveis mdulo 6 que um primo
p > 3 pode ter?
Para comear, os possveis resduos mdulo 6 so 0, 1, 2, 3, 4 ou 5.
Como p primo, ento 0 certamente no um resduo possvel. J 1
possvel, afinal 7 primo e tem resduo 1 mdulo 6. Quanto a 2,
p 2 (mod 6)
implica que p 2 par. Mas isto s possvel se p for par e todo
par maior que 2 composto. Um argumento semelhante mostra que
4 tambm no pode ser resduo de um tal primo. Por outro lado,
p3

(mod 6)

equivale a
p = 6 k + 3 para algum inteiro k 0.
Disto segue que
p = 3 (2 k + 1),
que tambm no admissvel, porque p primo e maior que 3.
Finalmente, 5 um resduo possvel; afinal, o prprio 5 primo.
Vamos resumir este resultado para referncia futura.
Proposio 4. Se p > 3 primo, ento p s pode ter resduos iguais
a 1 ou a 5 mdulo 6.

i
i

cripto
2009/6/30
i page 55
Estilo OBMEP

i
i

N SEC. 2.2: DEFINIES E PRIMEIRAS PROPRIEDADES

55

H uma outra maneira de dizer que a deixa resduo r mdulo n


que, apesar de s vezes produzir alguma confuso, usual e muito
conveniente. Como a r (mod n) significa que, para algum inteiro
k,
a = k n + r,
dizemos simplesmente que a da forma kn + r. Usando esta terminologia, o enunciado da proposio 4 passaria a ser
todo primo p > 3 da forma 6k + 1 ou da forma 6k + 5.
O prximo exerccio est enunciado usando esta terminologia.
Exerccio 18. Mostre que todo primo mpar da forma 4k + 1 ou da
forma 4k + 3. D exemplos de nmeros da forma 4k + 1 e da forma
4k + 3 que no so primos.
O desafio abaixo uma generalizao da proposio 4. Antes de
abord-lo talvez voc queira rever o exerccio 5, ao qual est relacionado.
Desafio 2. Seja n > 3 um inteiro e p > n um nmero primo. Quais
os resduos possveis para n! mdulo p?

2.2.4

Adio, Multiplicao e Congruncia

Antes de poder apreciar completamente o poder de fogo da congruncia mdulo n, precisamos estabelecer a relao entre a congruncia e as operaes usuais de adio e multiplicao de inteiros.
H dois fatos importantes que sabemos sobre a congruncia mdulo n. O primeiro, discutido no artigo anterior, nos diz que a con-

i
i

cripto
2009/6/30
i page 56
Estilo OBMEP

i
i

56

CAP. 2: ARITMTICA MODULAR

gruncia funciona de maneira muito semelhante igualdade de inteiros. O segundo consequncia da prpria definio e nos diz que
nmeros inteiros diferentes podem ser congruentes mdulo n. Para
usar uma imagem concreta, a congruncia mdulo n funciona como
uma espcie de filtro: quando olhamos os inteiros atravs dela, h
muitos inteiros que no conseguimos mais distinguir de outros. No
exatamente isto que acontece quando olhamos atravs de um filtro colorido? Por exemplo, se olhamos para vrias bolas coloridas
iguais, exceto pela cor, atravs de um filtro vermelho, veremos as bolas brancas e vermelhas como se fossem da mesma cor (neste caso,
vermelho); e as bolas azuis e pretas como se fossem da mesma cor
(neste caso, preto). Da mesma forma, 31, 1 e 51 so diferentes, mas se
olhamos para eles atravs da congruncia mdulo 5, no conseguimos
distingui-los entre si: eles so todos congruentes mdulo 5.
Este papo todo tem como nica meta introduzir a seguinte pergunta:
Se a a0 (mod n) e b b0 (mod n), o que podemos
afirmar sobre a relao entre a + b e a0 + b0 ?
Usando a imagem da congruncia como um filtro fcil descobrir
qual deveria ser a resposta a esta pergunta. De fato, se no temos
como distinguir entre duas bolas coloridas a e a0 , nem entre duas
bolas coloridas b e b0 porque estamos olhando para elas atravs de um
filtro vermelho, como poderemos ser capazes de distinguir entre os
conjuntos formados por a e b, de um lado, e a0 e b0 do outro? claro
que, neste caso, os conjuntos parecero iguais. Portanto, inspirados
na concluso fornecida por esta imagem, esperamos poder mostrar

i
i

cripto
2009/6/30
i page 57
Estilo OBMEP

i
i

57

N SEC. 2.2: DEFINIES E PRIMEIRAS PROPRIEDADES

que:

se a a0 (mod n) e b b0 (mod n), ento a + b a0 + b0 (mod n).

O problema que o filtro colorido apenas uma imagem para nos


guiar, no h como ter certeza de que esta imagem vai ser adequada
neste caso. Contudo, uma vez que tenhamos tomado uma deciso
sobre qual deveria ser a resposta, podemos test-la em alguns casos
simples. Por exemplo,
51 31 (mod 5) e 43 103

(mod 5),

ao passo que
51 + 43 = 94 e 31 + 103 = 134;
contudo,
94 134 = 40 = 5 8 donde 94 134

(mod 5).

Embora exemplos como este e quaisquer outros que decidamos inventar aumentem nossa confiana na concluso, no podemos d-la como
certa a no ser que consigamos prov-lo com rigor matemtico.
Como ainda estamos no incio de nosso estudo de congruncia,
pouco sabemos sobre as suas propriedades. Assim, para ter um ponto
de partida slido e j estabelecido no qual nos apoiar, mostraremos
que a propriedade desejada consequncia de fatos bem conhecidos
dos nmeros inteiros. Nisto vamos apenas repetir a mesma estratgia
que j usamos em 2.2.2.

i
i

cripto
2009/6/30
i page 58
Estilo OBMEP

i
i

58

CAP. 2: ARITMTICA MODULAR

Temos, ento, como hiptese que


a a0

(mod n) e b b0

(mod n).

Mas, usando a definio, podemos traduzir estas congruncias na


forma das seguintes igualdades de inteiros
a a0 = k n e b b0 = ` n,
onde, como em ocasies anteriores, k e ` so os respectivos cofatores.
Somando, agora, as duas equaes, resulta que
(a a0 ) + (b b0 ) = k n + ` n.
Rearrumando o lado esquerdo e usando a distributividade do lado
direito,
(a + b) (a0 + b0 ) = (k + `) n;
que, traduzido na linguagem de congruncias, nos d
(a + b) (a0 + b0 ) (mod n);
confirmando nossas suspeitas.
Exerccio 19. Mostre que, se a a0 (mod n) e b b0 (mod n),
ento a b a0 b0 (mod n).
Passando, agora, pergunta anloga para a multiplicao:
Se a a0 (mod n) e b b0 (mod n), o que podemos
afirmar sobre a relao entre a b e a0 b0 ?

i
i

cripto
2009/6/30
i page 59
Estilo OBMEP

i
i

59

N SEC. 2.2: DEFINIES E PRIMEIRAS PROPRIEDADES

Nossa experincia anterior com a adio nos permite afirmar, com


uma confiana razovel, que a resposta deve ser que
a b a0 b0

(mod n).

Testando nossa concluso no mesmo exemplo usado no caso da adio,


temos de
51 31 (mod 5) e 43 103

(mod 5),

que
51 43 = 2 193 e 31 103 = 3 193;
cuja diferena 1 000 e, portanto, um mltiplo de 5, assim
51 43 31 103 (mod 5),
como havamos previsto. Logo, esperamos poder provar que
se a a0 (mod n) e b b0 (mod n), ento a b a0 b0 (mod n).
Como no caso da adio, as congruncias
a a0

(mod n) e b b0

(mod n).

se traduzem como as igualdades de inteiros


a a0 = k n e b b0 = ` n
onde k e ` so inteiros. Copiando o que fizemos no caso da adio,
deveramos multiplicar estas equaes; s que, desta vez, mais con-

i
i

cripto
2009/6/30
i page 60
Estilo OBMEP

i
i

60

CAP. 2: ARITMTICA MODULAR

veniente reescrever estas equaes em outra forma, antes de fazer


clculos com elas. A forma desejada
a = a0 + k n
para a primeira, e
b = b0 + ` n,
para a segunda. Multiplicando estas duas equaes, membro a membro, temos
a b = (a0 + k n)(b0 + ` n).
(2.2.1)
Utilizando a distributividade da multiplicao sobre a soma, o lado
direito se expande na forma
(a0 + k n)(b0 + ` n) = a0 b0 + a0 ` n + k n b0 + k n ` n.
Pondo n em evidncia,
(a0 + k n)(b0 + ` n) = a0 b0 + n (a0 ` + k b0 + k ` n).
Comparando esta ltima equao com (2.2.1), vemos que
a b = a0 b0 + n (a0 ` + k b0 + k ` n);
donde,
a b a0 b0 = n (a0 ` + k b0 + k ` n),
de modo que a diferena a b a0 b0 um mltiplo de n. Mas isto
equivale a dizer que ab a0 b0 (mod n), como pretendamos mostrar.
Resumindo, provamos as seguintes frmulas.

i
i

cripto
2009/6/30
i page 61
Estilo OBMEP

i
i

N SEC. 2.3: CRITRIOS DE DIVISIBILIDADE

61

Proposio 5. Se a a0 (mod n) e b b0 (mod n), ento:


a + b a0 + b0 (mod n);
a b a0 b0 (mod n).
Em particular,
ak (a0 )k (mod n), para qualquer k 0.
Tendo feito o esforo de verificar estas propriedades, resta-nos ver
que realmente valeu a pena.

2.3

Critrios de Divisibilidade

Nesta seo utilizaremos o que aprendemos sobre a congruncia


modular para estabelecer alguns critrios simples de divisibilidade
por primos. Como bnus, provaremos tambm que a prova dos nove
funciona corretamente.
Se n for um inteiro positivo, ento um critrio de divisibilidade por
n uma regra que nos permite determinar se um dado inteiro , ou
no divisvel por n, a um custo menor que o de efetuar a diviso.
claro que a parte crtica desta definio a ltima: o custo de aplicar
a regra que corresponde ao critrio tem que ser menor que dividir o
nmero dado por n, seno o critrio simplesmente no vale a pena.
Por exemplo,
um nmero inteiro divisvel por 5 se, e somente se, seu
algarismo das unidades 0 ou 5

i
i

cripto
2009/6/30
i page 62
Estilo OBMEP

i
i

62

CAP. 2: ARITMTICA MODULAR

nos d o bem conhecido critrio de divisibilidade por 5. Nossa familiaridade com este critrio mera consequncia da observao do
comportamento dos mltiplos de 5. Outro critrio que tambm conhecemos bem por pura experincia o dos mltiplos de 2:
um nmero inteiro divisvel por 2 se, e somente se, seu
algarismo das unidades 0, 2, 4, 6 ou 8;
ou, dito em outras palavras,
um nmero inteiro par se, e somente se, seu algarismo
das unidades par.
Exerccio 20. Prove os critrios de divisibilidade por 2 e por 5 enunciados acima.

2.3.1

Divisibilidade por 3

Um outro critrio bem conhecido o de divisibilidade por 3:


um nmero inteiro divisvel por 3 se, e somente se, a
soma dos seus algarismos divisvel por 3.
S que, desta vez, o critrio no segue imediatamente as regularidades
relativas aos mltiplos de 3 que nos so familiares. Usaremos, a seguir,
a congruncia mdulo 3 para provar que este critrio verdadeiro.
Para comear, seja a o nmero inteiro que queremos saber se ou
no divisvel por 3. Para poder aplicar o critrio, precisamos conhecer
os algarismos de a. Digamos que
an an1 . . . a1 a0 ,

i
i

cripto
2009/6/30
i page 63
Estilo OBMEP

i
i

N SEC. 2.3: CRITRIOS DE DIVISIBILIDADE

63

so estes algarismos, onde a0 o algarismo das unidades, a1 o algarismo das dezenas, e assim por diante. Note que, como a0 , a1 , . . . , an
so algarismos, cada um deles maior ou igual a 0 e menor ou igual
a 9, exceto por an que no pode ser igual a zero. Dizer que a tem
como algarismos decimais an an1 . . . a1 a0 o mesmo que dizer que
a = an 10n + an1 10n1 + + a1 10 + a0 .

(2.3.1)

Contudo, 10 1 (mod 3). Por outro lado,


102 = 10 10,
satisfaz
102 10 10 1 1 1 (mod 3).
Observe que a congruncia da esquerda bvia pois dois nmeros
iguais so congruentes qualquer que seja o mdulo escolhido. J a
segunda congruncia bem mais interessante. Como vimos em 2.2.4,
se a a0 (mod n) e b b0 (mod n), ento a b a0 b0 (mod n).
Tomando, a = b = 10 e a0 = b0 = 1, podemos concluir de
10 1 (mod 3) que
102 12 1 (mod 3).
Nada nos impede de parar aqui. Assim,
103 = 102 10,

i
i

cripto
2009/6/30
i page 64
Estilo OBMEP

i
i

64

CAP. 2: ARITMTICA MODULAR

nos diz que


103 102 10 (mod 3).
Como j sabemos que 10 1 (mod 3) e acabamos de verificar que
102 1 (mod 3), podemos concluir da propriedade da multiplicao
e da transitividade da congruncia que
103 102 10 1 1 1 (mod 3).
Exerccio 21. Use um argumento semelhante para calcular 104 mdulo 3. Voc consegue imaginar duas maneiras diferentes de organizar
este clculo?
fcil imaginar que este procedimento pode ser continuado indefinidamente e que nos permite verificar que
10k 1 (mod 3) qualquer que seja o inteiro k 0 escolhido.
O que que 10k 1 (mod 3) nos diz sobre a divisibilidade por
3? Voltando um pouco atrs, lembre-se que havamos chegado concluso que
a = an 10n + an1 10n1 + + a1 10 + a0 .
Contudo, pela propriedade reflexiva da congruncia, isto implica que
a an 10n + an1 10n1 + + a1 10 + a0

(mod 3).

Mas, como acabamos de ver, cada uma das potncias de 10 mdulo 3


congruente a 1. Usando isto e as duas propriedades que relacionam

i
i

cripto
2009/6/30
i page 65
Estilo OBMEP

i
i

65

N SEC. 2.3: CRITRIOS DE DIVISIBILIDADE

a adio e a multiplicao congruncia, conclumos que


an 10n +an1 10n1 + +a1 10+a0 an +an1 + +a1 +a0

(mod 3).

Logo, pela transitividade,


a an + an1 + + a1 + a0

(mod 3),

que exatamente o que precisamos para podermos concluir o critrio


de divisibilidade por 3. De fato, dizer que a divisvel por 3 o
mesmo que dizer que a 0 (mod 3). Como
a an + an1 + + a1 + a0

(mod 3),

a transitividade nos garante que a 0 (mod 3) ocorre exatamente


quando
an + an1 + + a1 + a0 0 (mod 3);
que, por sua vez, equivale a dizer que
an + an1 + + a1 + a0
divisvel por 3. Mas,
an + an1 + + a1 + a0
a soma dos algarismos de a; portanto,
a divisvel por 3 se, e somente se, a soma
an + an1 + + a1 + a0 dos seus algarismos for divisvel
por 3,

i
i

cripto
2009/6/30
i page 66
Estilo OBMEP

i
i

66

CAP. 2: ARITMTICA MODULAR

o que prova que o critrio est correto.


Este argumento pode ser copiado para nos dar um critrio semelhante, s que desta vez para a divisibilidade por 11. Para obt-lo,
resolva o prximo exerccio.
Exerccio 22. Com este exerccio procuramos determinar um critrio
de divisibilidade por 11. Procederemos de maneira semelhante ao que
foi feito acima. Para isto:
(a) determine quanto vale 10k mdulo 11;
(b) escrevendo a na forma da equao (2.3.1), calcule a
mdulo 11 usando o resultado obtido em (a).

2.3.2

Prova dos Nove

Com o que fizemos para provar o critrio de divisibilidade por


3 estamos prontos para verificar porque a prova dos nove funciona
corretamente. Como vimos na pgina 43, a principal operao da
prova dos nove pode ser definida, sobre um inteiro positivo a, da
seguinte maneira

a
se a < 9;
a noves fora igual a
a 9 se a 9.
Sejam a e a0 os dois nmeros inteiros positivos que desejamos somar. Como a prova aplicada aos algarismos dos nmeros, precisamos
list-los. Mas um nmero a com n + 1 algarismos da forma
an an1 . . . a1 a0 ,

i
i

cripto
2009/6/30
i page 67
Estilo OBMEP

i
i

67

N SEC. 2.3: CRITRIOS DE DIVISIBILIDADE

onde a0 o algarismo das unidades, a1 o algarismo das dezenas, e


assim por diante. Isto equivale a dizer que, se a tem n algarismos,
ento
a = an 10n + an1 10n1 + + a1 10 + a0 ;
donde
a an 10n + an1 10n1 + + a1 10 + a0

(mod 9).

Como, 10 1 (mod 9), temos que qualquer potncia de 10 tambm


congruente a 1 mdulo 9, e assim,
a an + an1 + + a1 + a0

(mod 9);

de forma que o resduo de a mdulo 9 igual ao resduo mdulo


9 da soma dos seus algarismos. Alm disso, usando as frmulas da
proposio 5, podemos calcular o resduo de
an + an1 + + a1 + a0
passo a passo. Comeamos determinando o resduo rn de an mdulo
9. Como an um algarismo e, portanto, est entre 0 e 9, isto pode
ser feito usando a operao noves fora; o que nos d

a
se an < 9;
n
rn =
0
se a = 9.
n

Em seguida, calculamos o resduo rn1 de


rn + an1 an + an1

(mod 9).

i
i

cripto
2009/6/30
i page 68
Estilo OBMEP

i
i

68

CAP. 2: ARITMTICA MODULAR

Contudo, como 0 rn < 9 e 0 an1 9, temos que


0 rn + an1 < 18, de modo que, para achar seu resduo mdulo 9,
precisamos, no mximo, subtrair 9 de rn + an1 . Em outras palavras,
basta fazer noves fora em rn + an1 . Em seguida vem o resduo rn2
de
rn1 + an2 an + an1 + an2 (mod 9)
que, pelo mesmo argumento, igual a noves fora rn1 + an2 . O
processo continua desta maneira at obtermos r0 que, sendo o resduo
de
r1 + a0 an + an1 + an2 + + a0 (mod 9)
igual a noves fora r1 + a0 . Moral da histria, aplicando noves fora
soma progressiva dos algarismos de a, obtemos o resduo r0 de a
mdulo 9.
Evidentemente, ao aplicarmos o mesmo processo a a0 e a a + a0
obtemos seus resduos r00 e s0 mdulo 9. A prova dos nove segue,
ento, das frmulas da proposio 5, segundo as quais
r0 + r00 a + a0 s0

(mod 9).

Exerccio 23. Mostre que possvel obter, de maneira semelhante,


uma prova dos trs.
A prova dos trs ainda mais fcil de aplicar que a dos nove, ento
por que nunca ouvimos falar dela?
Exerccio 24. Mostre que h muitas contas incorretas que a prova
dos nove detecta que esto erradas, mas que parecem corretas quando
aplicamos a prova dos trs.

i
i

cripto
2009/6/30
i page 69
Estilo OBMEP

i
i

N SEC. 2.3: CRITRIOS DE DIVISIBILIDADE

69

Como vimos em 2.1.3, a prova dos nove no se aplica apenas


adio, um argumento anlogo ao que fizemos acima mostra que tambm pode ser usada para a multiplicao. E mais: apesar de raramente ouvirmos falar da prova dos nove para a diviso, ela tambm
pode ser utilizada para esta operao.
Exerccio 25. Formule uma verso da prova dos nove para a diviso
e mostre que funciona corretamente.

2.3.3

Divisibilidade por 7

Em 2.3.1 vimos como enunciar e provar os critrios de divisibilidade por 2, 3 e 5. Alm disso, propusemos no exerccio 22 um critrio
para a divisibilidade por 11. Considerando os primos de 2 a 11, o
nico para o qual ainda no temos um critrio 7. Voc conhece
algum critrio de divisibilidade por 7?
Muito provavelmente sua resposta pergunta foi no. Mas nada
nos impede de usar a mesma estratgia utilizada na obteno de um
critrio por 3 e por 11 para tentar achar um critrio para divisibilidade
por 7? A resposta, naturalmente, nada! Vejamos o que acontece
se fizermos isto. Para comear, sabemos que expressar um inteiro a a
partir dos seus algarismos
an an1 a1 a0
significa que
a = an 10n + an1 10n1 + a1 10 + a0 .

i
i

cripto
2009/6/30
i page 70
Estilo OBMEP

i
i

70

CAP. 2: ARITMTICA MODULAR

Como isto implica que


a an 10n + an1 10n1 a1 10 + a0

(mod 7),

o critrio segue por transitividade se pudermos determinar a que so


congruentes as vrias potncias de 10 mdulo 7.
Aplicando a mesma estratgia j usada em 2.3.1, temos as seguintes
congruncias mdulo 7:
101 3

(mod 7),

102 32 2 (mod 7),


103 10 102 3 2 6 1
4

10 10 10 (1) 3 4

(mod 7),

(mod 7),

105 10 104 3 4 5 (mod 7),


106 10 105 3 5 1 (mod 7).
Para efetuar estes clculos usamos livremente as vrias propriedades
da congruncia que j conhecemos. Paramos na sexta potncia simplesmente porque, da em diante os restos vo se repetir. De fato,
107 106 10 1 3 3 (mod 7),
ao passo que
108 106 102 1 102 2

(mod 7),

e assim por diante. Mais precisamente, se m um inteiro qualquer e

i
i

cripto
2009/6/30
i page 71
Estilo OBMEP

i
i

71

N SEC. 2.3: CRITRIOS DE DIVISIBILIDADE

q e r seu quociente e resto mdulo 6, ento


10m 106q+r (106 )q 10r

(mod 7).

Como 106 1 (mod 7), conclumos que


(106 )q 10r 10r

(mod 7);

e a transitividade nos d
10m 10r

(mod 7).

Mas isto timo porque, sendo um resto da diviso por 6, r satisfaz a


desigualdade 0 r 5, de modo que 10r pode ser determinado facilmente da lista das potncias mdulo 7 calculada acima. Por exemplo,
101 007 105 5 (mod 7),
porque 1 007 deixa resto 5 na diviso por 6.
Exerccio 26. Calcule o valor das seguintes potncias de 10 mdulo
7:
109 876 , 106 543 e 101 247 .
Voltando questo da divisibilidade por 7, devemos aplicar o valor
das potncias calculadas acima congruncia
a = an 10n + an1 10n1 + a1 10 + a0 .

i
i

cripto
2009/6/30
i page 72
Estilo OBMEP

i
i

72

CAP. 2: ARITMTICA MODULAR

Fazendo isto, e escrevendo as potncias da maior para a menor,


temos
a a0 + a1 3 + a2 2 + a3 6 + a4 4 + a5 5 + a6 +
+a7 3 + + an 10r (mod 7),
onde r o resto da diviso de n por 6. O que voc acha? mais fcil
aplicar isto, ou dividir a por 7 e ver se o resto zero? Como tenho
dificuldade em lembrar sequncias de nmeros (inclusive, infelizmente,
nmeros de telefone...) e como dificilmente vou precisar verificar, na
mo, se um nmero maior que 1 000 000 divisvel por 7, a minha
escolha recairia em efetuar a diviso diretamente.
Por sorte, h uma outra maneira de enunciar o critrio de divisibilidade por 7 que o torna mais fcil de lembrar. Antes, porm,
precisamos de alguma preparao. Digamos que, como antes,
a = an 10n + an1 10n1 + a1 10 + a0 .
Isolando o algarismo das unidades de a podemos escrever
a = (an 10n1 + an1 10n2 + a1 ) 10 + a0 .
Note que pusemos 10 em evidncia. Assim, se escrevermos
a
= (an 10n1 + an1 10n2 + a1 ),
teremos
a=a
10 + a0 .

i
i

cripto
2009/6/30
i page 73
Estilo OBMEP

i
i

73

N SEC. 2.3: CRITRIOS DE DIVISIBILIDADE

Um exemplo pode esclarecer o que fizemos. Digamos que


a = 12 346 998 654 343
que tem como algarismo das unidades a0 = 3. Assim,
a = 1 234 699 865 434 10 + 3,
de modo que, neste caso,
a
= 1 234 699 865 434.
Exerccio 27. Determine a0 e a
para cada um dos nmeros a indicados abaixo:
a = 87 645 564 348, a = 85 735 214 421 e a = 981 231 111.
Uma vez que tenhamos escrito a na forma
a = 10 a
+ a0 ,
aplicamos a congruncia mdulo 7. Como 10 3 (mod 7), temos que
a 10 a
+ a0 3 a
+ a0

(mod 7).

Portanto, pela transitividade,


a3a
+ a0

(mod 7);

(2.3.2)

isto ,

i
i

cripto
2009/6/30
i page 74
Estilo OBMEP

i
i

74

CAP. 2: ARITMTICA MODULAR

a divisvel por 7 se, e somente se 3 a


+ a0 tambm for.
Por exemplo, digamos que desejamos saber se 128 divisvel por 7.
Como, se a = 128, ento a0 = 8 e a
= 12, temos que
3a
+ a0 = 12 3 + 8 = 44.
Como 44 no divisvel por 7, o critrio acima nos garante que 128
tambm no pode ser.
Podemos reformular este critrio de uma maneira ainda mais
agradvel. Para isto multiplicamos ambos os membros de (2.3.2) por
2, o que nos d
2 a 2 (3 a
+ a0 ) 6 a
+ 2 a0

(mod 7),

pela distributividade. Contudo, 6 1 (mod 7), donde


2 a
a + 2 a0

(mod 7).

Claramente, se a 0 (mod 7), ento

a + 2 a0 0 (mod 7),

(2.3.3)

pela transitividade da congruncia. Por exemplo, sabendo que 875


mltiplo de 7, podemos concluir que
87 + 2 5 = 77
tambm tem que ser. Infelizmente estamos andando na direo errada. Afinal, difcil imaginar algum que soubesse que 875 mltiplo

i
i

cripto
2009/6/30
i page 75
Estilo OBMEP

i
i

N SEC. 2.3: CRITRIOS DE DIVISIBILIDADE

75

de 7, mas ignorasse que 77 tambm .


O que seria til se fssemos capazes de provar a recproca; isto
, que se

a + 2 a0 0 (mod 7),
(2.3.4)
ento a 0 (mod 7). Para isto precisamos desfazer o que fizemos ao
multiplicar a equao toda por 2. Mas isto fcil de fazer. Como
2 3 1 (mod 7),
temos ao multiplicar ambos os lados de (2.3.4) por 3, que
3 (
a + 2 a0 ) 3 0 (mod 7).
Usando a distributividade do lado esquerdo,
3 a
+ 6 a0 0 (mod 7);
de modo que 6 1 (mod 7) nos d
3 a
a0 0 (mod 7).
Pondo 1 em evidncia
(3 a
+ a0 ) 0 (mod 7).

(2.3.5)

Acontece que
3a
+ a0 a (mod 7)

i
i

cripto
2009/6/30
i page 76
Estilo OBMEP

i
i

76

CAP. 2: ARITMTICA MODULAR

de forma que (2.3.5) pode ser reescrita na forma


a 0 (mod 7),
que o mesmo que dizer que 7 divide a. Resumindo, mostramos que
se
a + 2 a0 0 (mod 7) ento a 0 (mod 7).
Logo,
para mostrar que 7 divide a, basta testar se 7 divide
3 a
a0 , de acordo com a decomposio de a explicada anteriormente.
Voltando ao exemplo anterior, como
87 + 2 5 = 77
obviamente divisvel por 7, podemos concluir que 875 tambm .
Partindo para um exemplo mais impressionante, digamos que queremos saber se a = 10 794 , ou no divisvel por 7. Neste caso,
a0 = 4 e a
= 1079,
de modo que, pelo critrio estabelecido acima, basta descobrir se 7
divide ou no

a + 2 a0 = 1 079 + 2 4 = 1 071.
Como isto ainda no fcil de determinar, vamos usar o critrio

i
i

cripto
2009/6/30
i page 77
Estilo OBMEP

i
i

N SEC. 2.3: CRITRIOS DE DIVISIBILIDADE

77

novamente, s que desta vez para


b = 1 071.
Temos que
b0 = 1 e a
= 107,
e assim
b + 2 b0 = 107 + 2 1 = 105.
Portanto, usando novamente o critrio, se 7 divide 105 ou, o que d
no mesmo, se 7 divide 105, ento 7 divide b. Como ainda no sei se
7 divide ou no 105, vou aplicar o critrio uma ltima vez, agora a
c = 105 que tem
c0 = 5 e c = 10.
Como

c + 2 c0 = 10 + 2 5 = 0
claramente divisvel por 7, ento 7 divide c = 105. Mas isto implica,
pelo critrio, que 7 divide b = 1 071 que, por sua vez, implica que 7
divide a = 10 794, que o que queramos saber.
Observe que aplicamos a regra dada pelo critrio a nmeros sucessivamente menores, at obter um caso em que sabamos a resposta
sem fazer sequer uma conta. Temos, assim, uma regra recursiva, que
o termo utilizado pelos matemticos para descrever uma regra que
reduz um dado problema a um problema anlogo mas com dados
menores.
Exerccio 28. Use o critrio de divisibilidade por 7 tantas vezes quantas forem necessrias para determinar se 35 994 e se 36 003 so di-

i
i

cripto
2009/6/30
i page 78
Estilo OBMEP

i
i

78

CAP. 2: ARITMTICA MODULAR

visveis por 7.
Para encerrar, aqui est um exerccio um pouco diferente que pode
ser resolvido facilmente usando congruncias.
Exerccio 29. Ache um fator primo mpar de 525 1.

i
i

cripto
2009/6/30
i page 79
Estilo OBMEP

i
i

Captulo 3

Inversos Modulares
Nesta seo discutiremos um tema que vai ter importncia decisiva tanto para o principal teorema desta apostila, quanto para o
funcionamento do prprio RSA. Comeamos analisando os clculos
que efetuamos para obter o critrio de divisibilidade por 7 na seo
anterior.

3.1

Motivao e Definies

Voc deve ter observado o importante papel que os nmeros 2 e 3


desempenharam no argumento em 2.3.3. Precisvamos mostrar que
duas congruncias eram equivalentes; mais precisamente, as congruncias dadas nas equaes (2.3.2) e (2.3.3). Verificamos que, multiplicando (2.3.2) por 2, obtnhamos (2.3.3), ao passo que, multiplicando
(2.3.3) por 3, obtnhamos (2.3.2). O segredo para o sucesso desta
79

i
i

cripto
2009/6/30
i page 80
Estilo OBMEP

i
i

80

CAP. 3: INVERSOS MODULARES

conta est na congruncia


2 3 1 (mod 7).
Utilizando a linguagem que usaramos se estivssemos calculando com
nmeros racionais, podemos dizer que 1 dividido por 2 igual a
3. Multiplicando toda a congruncia por 1 obtemos
2 (3) 1

(mod 7) e tambm (2) 3 1

(mod 7).

Como
3 4

(mod 7) e

25

(mod 7),

2 4 1 (mod 7) e 5 3 1

(mod 7).

podemos concluir que

Neste caso, tambm podemos dizer que 1 dividido por 2 mdulo 7 d


como resultado 5 e 1 dividido por 3 d 5. Quando isto ocorre, dizemos
que 2 e 4 so inversos mdulo 7, e o mesmo se d com 3 e 5.
e

a0

Sistematizando o contedo do pargrafo anterior, diremos que a


so inversos mdulo n se
a a0 1 (mod n).

Neste caso, tambm dizemos que a0 o inverso de a mdulo n, e


vice-versa. Na tabela abaixo listamos cada um dos resduos distintos
possveis de inteiros mdulo 11, indicando o resduo do seu respectivo
inverso. Note que 0 no pode ter inverso mdulo n no importa que

i
i

cripto
2009/6/30
i page 81
Estilo OBMEP

i
i

N SEC. 3.1: MOTIVAO E DEFINIES

81

valor n assuma, j que


0 b 0 (mod n) qualquer que seja b Z.
Por isso sequer listamos zero entre os resduos na tabela. Voc pode

Resduo
1
2
3
4
5
6
7
8
9
10

Inverso Mdulo 11
1
6
4
3
9
2
8
7
5
10

estar se perguntando como os inversos nesta tabela foram obtidos.


Embora exista uma maneira sistemtica de calcular inversos mdulo
n, ela trabalhosa demais para valer pena aplic-la quando o mdulo n um nmero pequeno. Por isso, os inversos na tabela foram
determinados por tentativa. Em outras palavras, para achar o inverso de 2 mdulo 11, multiplicamos 2 pelos inteiros de 2 em diante

i
i

cripto
2009/6/30
i page 82
Estilo OBMEP

i
i

82

CAP. 3: INVERSOS MODULARES

at obter a congruncia desejada; neste caso,


2 2 4 6 1 (mod 11)
2 3 6 6 1 (mod 11)
2 4 8 6 1 (mod 11)
2 5 10 6 1 (mod 11)
2 6 12 1 (mod 11)

e obtivemos o inverso procurado. Note que isto significa que 6 tem


como inverso 2, de modo que acabamos por preencher duas linhas da
tabela, a segunda e a sexta.
Na verdade, podemos utilizar um pouco mais que mera tentativa,
porque se nos restringimos aos inteiros entre 1 e n 1, ento cada um
destes inteiros tem exatamente um inverso neste intervalo. De fato,
se a0 e a00 so ambos inversos de a mdulo n, ambos entre 1 e n 1,
ento
a a0 1 (mod n) e a a00 1 (mod n);
donde conclumos que
a00 (a a0 ) a00 1 a00

(mod n)

(a00 a) a0 1 a0 a0

(mod n).

e tambm que

Mas tudo o que fizemos foi mudar a posio dos parnteses, e isto no

i
i

cripto
2009/6/30
i page 83
Estilo OBMEP

i
i

83

N SEC. 3.1: MOTIVAO E DEFINIES

altera o resultado da conta, logo


a0 a00

(mod n).

Mas isto significa que a diferena a0 a00 divisvel por n. Contudo,


a0 e a00 so positivos e menores que n, de modo que
n < a0 a00 < n.
Logo, a nica maneira de a0 a00 ser mltiplo de n se for igual a
zero; donde a0 = a00 .
Voc pode estar pensando:
Muito bom, muito bem; mas de que forma isto ajuda na
hora de calcular a tabela?
A resposta que, se j sabemos, por exemplo, que 2 e 6 so inversos
um do outro mdulo 11, ento nem 2 nem 6 podem ser inversos de 3
mdulo 11. Assim, procuraremos pelo resduo do inverso de 3 apenas
entre os inteiros 3, 4, 5, 7, 8, 9 e 10. Por isso, quanto mais inversos
determinamos, mais rpido fica determinar os que ainda faltam.
Exerccio 30. Usando as estratgias descritas acima, determine um
inverso para cada um dos resduos distintos mdulo 7 e para cada um
dos resduos distintos mdulo 13.
Note que existem alguns nmeros que so seus prprios inversos.
Na tabela mdulo 11, isto vale para 1 e 10. No caso de 1 isto no
nenhuma surpresa, afinal 1 1 = 1; j para 10 o resultado no parece
to bvio. Contudo, o fato de 10 ser seu prprio inverso mdulo 11

i
i

cripto
2009/6/30
i page 84
Estilo OBMEP

i
i

84

CAP. 3: INVERSOS MODULARES

no mera coincidncia, como talvez voc j tenha desconfiado ao


calcular as tabelas correspondentes a 7 e a 13 no exerccio 30. No
prximo exerccio voc encontrar a explicao para este fenmeno.

Exerccio 31. Mostre que n 1 sempre seu prprio inverso


mdulo n.

Uma questo mais sutil se pode haver algum nmero n para o


qual existe um inteiro a entre 2 e n 2, que seu prprio inverso
mdulo n. Em outras palavras, existem inteiros n > 1 e a de modo
que
2 a n 2 e a2 1 (mod n)?
A resposta sim, mas o desafio de construir tais nmeros fica para
voc.

Desafio 3. Construa infinitos nmeros n para os quais existe um


inteiro a, entre 2 e n 2, que seu prprio inverso mdulo n.

3.2

Inexistncia de Inverso

Vamos calcular uma nova tabela de inversos, desta vez a tabela


dos inversos dos resduos distintos mdulo 8. Como 1 seu prprio

i
i

cripto
2009/6/30
i page 85
Estilo OBMEP

i
i

N SEC. 3.2: INEXISTNCIA DE INVERSO

85

inverso, comearemos com 2; efetuando os clculos vemos que


2 2 4 6 1 (mod 8)
2 3 6 6 1 (mod 8)
2 4 0 6 1 (mod 8)
2 5 2 6 1 (mod 8)
2 6 4 6 1 (mod 8)
2 7 6 6 1 (mod 8)

e, surpreendentemente, descobrimos que 2 no tem inverso mdulo 8.


Talvez voc ache que teria sido mais preciso dizer descobrimos que 2
no tem inverso mdulo 8 entre os nmeros inteiros menores que 8.
Lembre-se, contudo, que todo inteiro congruente mdulo 8 ao seu
resduo. Como calcular com um nmero ou com seu resduo produzem
o mesmo resultado mdulo 8, no pode haver nenhum inteiro que
inverta 2, j que tal inteiro no existe entre os nmeros inteiros de 1 a
8. Como se isto no bastasse, apareceu um resultado muito estranho
nos clculos acima:
embora 2 e 4 no sejam congruentes a zero mdulo 8, o
produto deles dois 8, que congruente a zero mdulo 8.
como se estivssemos dizendo que o produto de dois nmeros no
nulos deu zero, o que muito esquisito. A lista completa dos inversos
mdulo 8 dada na tabela 3.1

i
i

cripto
2009/6/30
i page 86
Estilo OBMEP

i
i

86

CAP. 3: INVERSOS MODULARES

Resduos
1
2
3
4
5
6
7

Inverso Mdulo 8
1

Tabela 3.1: Inversos mdulo 8


O asterisco que aparece na coluna dos inversos indica que o elemento
correspondente no tem inverso. Neste caso, 2, 4 e 6 no admitem
inverso mdulo 8. A propsito, voc observou que cada um dos elementos que tem inverso mdulo 8 seu prprio inverso?
Ser que h uma regularidade clara que nos permita determinar
quais so os elementos que tm inverso, e quais os que no tm inverso
mdulo n, para um dado n? Pelo menos no caso do mdulo 8, a
regularidade clara: os mpares tm inverso, os pares no. Vejamos
o que acontece com outros mdulos; para isso, calcularemos mais
algumas tabelas.
Exerccio 32. Determine um inverso para cada um dos resduos distintos mdulo 6 e para cada um dos resduos distintos mdulo 15.
Tendo calculado as tabelas, voc ter verificado que s vezes um
mpar pode no ter inverso, e s vezes um par pode ter inverso, e com
isso l se foi nossa proposta de regularidade. Mas se voc est lendo
isto de maneira crtica (sem se deixar levar pela minha lbia...), deve
estar se perguntando:

i
i

cripto
2009/6/30
i page 87
Estilo OBMEP

i
i

N SEC. 3.2: INEXISTNCIA DE INVERSO

87

Tem uma coisa esquisita nisso tudo. Eu j tinha feito um


exerccio assim, s que para 7 e 13, e todos os resduos
tinham inverso; uma maravilha! Agora ele me fala para
fazer para 6 e 12 e aparecem vrios resduos sem inverso.
Por que s agora? O que que 7 e 13 tm de bom, que
falta a 6 e 15?
A resposta, evidentemente, que 7 e 13 so primos, ao passo que 6 e
15 so compostos. Ento eu pergunto: olhando para as tabelas que
voc calculou, qual a relao entre os nmeros que no tm inverso e
os mdulos correspondentes?
Se voc pensou com cuidado, ter visto que tanto para o mdulo
8, quanto para os mdulos 6 e 15, os resduos que no tm inverso
so aqueles que tm um fator primo comum com o mdulo. por
isso que os pares no tm inverso mdulo 8. Para falar a verdade,
fcil entender porque isto acontece. Mas antes, um exerccio. Nossa
tentativa frustrada de descobrir o inverso de 2 mdulo 8 revelou-nos
que
2 4 0 (mod 8),
muito embora, nem 2, nem 4 sejam congruentes a zero mdulo 8.
Exerccio 33. Para cada um dos resduos a que no tm inverso
mdulo 6, determine um resduo b 6 0 (mod 6) tal que
a b 0 (mod 6). Faa o mesmo para os resduos que no tm
inverso mdulo 15.
Este exerccio parece sugerir que h uma forte ligao entre no
ter inverso mdulo n e ser anulado mdulo n pelo produto com um

i
i

cripto
2009/6/30
i page 88
Estilo OBMEP

i
i

88

CAP. 3: INVERSOS MODULARES

resduo no nulo. Como veremos abaixo, exatamente isto que acontece.


Digamos que n e 1 < a < n so inteiros positivos que tm um
fator primo comum 1 < p < n. Podemos, ento, escrever
n=pc

e a = p e,

onde c e e so os cofatores correspondentes. Como 1 < p < n ento


c = n/p tambm satisfaz 1 < c < n. Por sua vez, como 1 < a < n
por hiptese, temos que nem c, nem a so congruentes a zero mdulo
n. Contudo,
c a c p e (mod n).
Ocorre que n = c p, e assim
cpn0

(mod n);

donde
c a c p e 0 (mod n).

(3.2.1)

Bacana, no? Mas, como usar isto para verificar que a no tem inverso mdulo n? Bem, de fato estes clculos mostram que a no pode
ter inverso mdulo n. Para entender porqu, procederemos por contradio. Suponhamos que a realmente tivesse inverso a0 mdulo n.
Neste caso, deveramos ter que
a a0 1 (mod n).

i
i

cripto
2009/6/30
i page 89
Estilo OBMEP

i
i

89

N SEC. 3.2: INEXISTNCIA DE INVERSO

Multiplicando ambos os membros da congruncia por c (o mesmo


cofator c determinado acima), obtemos
c (a a0 ) c (mod n).
Reagrupando os parntesis,
(c a) a0 c (mod n).

(3.2.2)

S que, pela equao (3.2.1),


c a 0 (mod n);
de modo que
(c a) a0 0 a0 0

(mod n).

Comparando isto com a equao (3.2.2), conclumos que


c 0 (mod n);
isto , n divide c. S que isto no pode ser verdade porque, como
vimos acima, 1 < c < n. Obtivemos, assim, uma concluso absurda.
Isto ocorreu porque fizemos uma hiptese falsa ao supor que a tem
inverso mdulo n. Portanto, a no pode ter inverso mdulo n, como
havamos afirmado antes. Resumindo, mostramos o seguinte resultado.

Teorema 1. Se existir um fator primo comum entre a e n, ento a


no tem inverso mdulo n.

i
i

cripto
2009/6/30
i page 90
Estilo OBMEP

i
i

90

CAP. 3: INVERSOS MODULARES

3.2.1

Cancelamento

H uma consequncia importante da inexistncia do inverso que


vai surgir em nossas aplicaes posteriores, por isso vamos discuti-la
agora.
Se estamos calculando com nmero inteiros e nos deparamos com
uma igualdade do tipo
a c = b c,
pensamos imediatamente em cancelar o c e concluir que a = b. Contudo, sabemos que isto s possvel se c 6= 0, porque multiplicar por
zero iguala o produto a zero. Infelizmente, quando trabalhamos com
congruncias a situao torna-se bem pior.
Comecemos por um exemplo. Sabemos que
2 6 0

(mod 6) e 3 6 0

(mod 6)

ao passo que
2360

(mod 6).

Assim, apesar da congruncia


2 3 2 0 (mod 6)
ser verdadeira, no podemos cancelar o 2 que multiplica os dois lados
e concluir que
3 0 (mod 6)
porque isto, como j vimos, falso. Logo, neste exemplo, o cancelamento no permitido.

i
i

cripto
2009/6/30
i page 91
Estilo OBMEP

i
i

91

N SEC. 3.2: INEXISTNCIA DE INVERSO

Passando ao caso geral, digamos que n > 0 e 1 a n 1 so


inteiros que tm um fator primo comum 1 < p < n. Escrevendo
n=pc e

a = p e,

onde c e e so os cofatores correspondentes, temos que


a c a 0 (mod n);
embora a e c no possam ser congruentes a zero mdulo n, j que
so ambos positivos e menores que n. Com isto chegamos seguinte
concluso:

se a, b e n > 1 so inteiros que tm algum fator primo em


comum, ento a no pode ser cancelado em congruncias
do tipo
a b a 0 (mod n).

Por outro lado, se a admite um inverso mdulo n e b e c so inteiros


tais que
a b a c (mod n),
(3.2.3)
ento o a pode ser cancelado e podemos concluir que b c (mod n).
Para provar isto, procedemos como no argumento usado para provar
o teorema 1. Seja a0 o inverso de a mdulo n. Multiplicando a congruncia (3.2.3) por a0 , obtemos
(a0 a) b (a0 a) c (mod n).

i
i

cripto
2009/6/30
i page 92
Estilo OBMEP

i
i

92

CAP. 3: INVERSOS MODULARES

Como
a a0 1 (mod n),
resta apenas
b c (mod n);
mostrando que o cancelamento pode mesmo ser feito neste caso. Resumimos isto em um teorema para referncia futura.
Teorema 2. Suponha que a tem inverso mdulo n. Se
a b a c (mod n),
para b, c Z, ento
b c (mod n).

3.3

Existncia de Inverso

Tudo isto pode ser muito interessante, mas no deixa de ser muito
negativo. Descobrimos como detectar que certos nmeros no tm
inverso mdulo n, e provamos que nosso palpite estava correto. Mas,
e quanto aqueles que tm inverso? O palpite mais bvio, claro,
que todos os nmeros que no tm fator prprio comum com n tero
inverso mdulo n. Sem esquecer, que este palpite confirmado por
todas as tabelas que calculamos anteriormente.
De fato, este resultado verdadeiro mas, para prov-lo, teremos
que trabalhar um pouco. Voltando s definies, sabemos que um

i
i

cripto
2009/6/30
i page 93
Estilo OBMEP

i
i

93

N SEC. 3.3: EXISTNCIA DE INVERSO

inteiro a tem inverso mdulo n se existir um inteiro a0 tal que


a a0 1 (mod n).

(3.3.1)

Traduzindo isto em termos de divisibilidade de inteiros, temos que


n divide a diferena a a0 1.
Em outras palavras, existe um inteiro k para o qual
a a0 1 = n k.

(3.3.2)

Como esta ltima equao equivalente a (3.3.1), podemos concluir


que o que precisamos mostrar que
se a e n no admitem nenhum fator prprio comum, ento
existe um inteiro k para o qual a a0 1 = n k.
Para provar este resultado, procedemos da seguinte forma. Considere, para comear, o conjunto V (a, n) formado pelos inteiros positivos que podem ser escritos na forma
xa+yn
para alguma escolha de inteiros x e y. Note que x ou y podem ser
nulos ou negativos, embora estejamos exigindo que x a + y n seja
positivo. Por exemplo, se a = 5 e n = 12, ento tomando x = 1 e
y = 1, temos que
x a + y n = (1) 5 + 1 12 = 7 > 0.
Logo, 7 V (5, 12).

i
i

cripto
2009/6/30
i page 94
Estilo OBMEP

i
i

94

CAP. 3: INVERSOS MODULARES

Exerccio 34. Calcule 5 elementos em cada um dos seguintes conjuntos V (5, 12), V (7, 15) e V (5, 10).
Uma pergunta razovel :
Por que introduzimos este estranho conjunto V (a, n)?
A resposta simples. Se fomos capazes de mostrar que 1 V (a, n)
ento tm que existir dois inteiros, digamos x0 e y0 , tais que
1 = x0 a + y0 b.
Mas, tomando a0 = x0 e k = y0 , obtemos
1 = a0 a + k b,
que equivalente equao desejada (3.3.2). Outro ponto importante
a ser notado que,
se 1 V (a, n), ento ele tem que ser o menor elemento
de V (a, n),
pois este conjunto s tem elementos positivos.
Voltando ao caso geral, observamos que V (a, n) no pode ser vazio
porque tomando x = 0 e y = 1, vemos que
x a + y n = 0 a + 1 n = n > 0;
logo n pertence a V (a, n). Na verdade, isto nos diz mais. Como a
quantidade de inteiros entre 1 e n finita, podemos escolher o menor

i
i

cripto
2009/6/30
i page 95
Estilo OBMEP

i
i

N SEC. 3.3: EXISTNCIA DE INVERSO

95

destes nmeros que pertence a V (a, n). Mas qualquer inteiro em


V (a, n) que esteja fora do intervalo que vai de 1 a n tem que ser
maior que n e, portanto, maior que m. Logo,
m o menor elemento do conjunto V (a, n).
Para podermos concluir nossa demonstrao precisamos verificar
que m = 1. Como a e n so primos entre si, bastaria que fssemos
capazes de mostrar que m divide tanto a como n para que pudssemos
concluir que igual a 1. Afinal, para um inteiro positivo dividir 1, ele
tem que ser igual a 1. Vejamos como mostrar que m divide a e divide
n.
Para comear, como m V (a, m), ento tm que existir inteiros
x1 e y1 tais que
m = x1 a + y1 n.
(3.3.3)
Dividindo n por m, temos que
n = m q + r e 0 r < m,
onde q o quociente e r o resto da diviso de n por m. Substituindo
nesta equao a expresso para m dada em (3.3.3), obtemos
n = m q + r = (x1 a + y1 n) q + r,
que pode ser rearrumada na forma
r = (x1 ) a + (1 y1 ) n.
Em particular, podemos concluir que r V (a, n) por causa da maneira

i
i

cripto
2009/6/30
i page 96
Estilo OBMEP

i
i

96

CAP. 3: INVERSOS MODULARES

como conseguimos express-lo. Contudo, r o resto da diviso de n


por m, de modo que r = 0 ou r 6= 0. S que, neste ltimo caso,
r < m (j que m o divisor), o que bate de frente com o fato de m
ter sido escolhido como o menor elemento de V (a, n). Portanto, s
resta a primeira possibilidade. Mas se r = 0, ento m divide n, como
queramos mostrar. Um argumento inteiramente anlogo mostra que
m divide a.
Exerccio 35. Mostre em detalhes que m divide a.
Vamos recapitular o que fizemos acima:
verificamos que a a0 1 (mod n) o mesmo que dizer que
existe um inteiro k tal que a a0 + k n = 1;
definimos o conjunto V (a, n) formado pelos inteiros positivos
que podem ser escritos na forma
x a + y n;
se 1 V (a, n) ento existem inteiros x0 e y0 para os quais
x0 a + y0 n = 1,
e tomando a0 = x0 e k = y0 provamos o resultado desejado.
Portanto, basta mostrar que 1 V (a, n).
Seja, ento, m o menor elemento de V (a, n). Mostraremos que,
como a e n so primos entre si, ento m = 1:

i
i

cripto
2009/6/30
i page 97
Estilo OBMEP

i
i

N SEC. 3.4: O TEOREMA E UM EXEMPLO

97

se r for o resto da diviso de n por m ento r = 0 ou r 6= 0;


se r 6= 0 ento mostramos que r pertenceria a V (a, n);
como o resto sempre menor que o divisor, teramos r < m;
mas isto no possvel pela escolha que fizemos para m;
portanto, r = 0 e m divide n.
Um argumento anlogo (ver exerccio 35) mostra que m divide n.
Assim,
m um divisor comum de a e de n;
mas o nico divisor comum positivo de a e n 1;
logo m = 1 e provamos o resultado desejado.

3.4

O Teorema e um Exemplo

hora de reunir todos os resultados que provamos neste captulo


em um nico teorema, que utilizaremos com frequncia nesta apostila.
Teorema 3. Sejam a < n inteiros positivos. O resduo a tem inverso
mdulo n se, e somente se, a e n no tm fatores primos em comum.
Mostramos este resultado em duas partes. No teorema 1 verificamos que se a e n tm fatores primos em comum ento a no pode
ter inverso mdulo n. J a recproca foi analisada na seo 3.3, onde
mostramos que, quando a e n no tm fatores primos em comum,

i
i

cripto
2009/6/30
i page 98
Estilo OBMEP

i
i

98

CAP. 3: INVERSOS MODULARES

ento possvel achar inteiros a0 e k tais que a a0 + k n = 1. Como


isto implica que a a0 1 (mod n), podemos concluir que a0 o
inverso de a mdulo n.
Como consequncia deste teorema, temos que se n for primo, ento todo resduo no nulo admite inverso mdulo n. Isto explica
porque as tabelas de inversos de 7, 11 e 13 podem ser completamente
preenchidas. Por outro lado, o teorema tambm nos diz que, se n for
composto, ento sua tabela ficar incompleta, pois haver resduos
sem inverso; o que explica o comportamento das tabelas de 6, 8 e 15.
Outro ponto importante que nossa demonstrao do teorema
o que os matemticos chamam de no construtiva: ela nos garante a
existncia de um inverso para a quando a e n no tm fator primo
comum, mas no nos diz como proceder para calcular este inverso.
importante entender que esta uma deficincia de nossa demonstrao
e no do teorema em si. Para uma demonstrao construtiva deste
mesmo teorema, consulte o captulo 1 da referncia [2].
Combinando o teorema acima com o teorema 2 da pgina 92,
obtemos o seguinte resultado.
Corolrio 1. Sejam a < n inteiros positivos sem fatores prprios
comuns. Se
a b a c (mod n),
para a, b Z, ento
b c (mod n).
Corolrio? O que isto? A palavra corolrio em portugus vem
do latim corollarium que tem uma histria (ou etimologia) muito interessante. Originalmente corolla em latim era apenas o diminutivo

i
i

cripto
2009/6/30
i page 99
Estilo OBMEP

i
i

99

N SEC. 3.4: O TEOREMA E UM EXEMPLO

de corona, que quer dizer coroa. Da a palavra passou a ser usada para
significar tambm uma pequena guirlanda de flores entrelaadas, por
causa de sua semelhana a uma coroa pequena. Corollarium comeou
significando o dinheiro pago para comprar uma corona, mas seu sentido acabou se generalizando para cobrir um presente ou qualquer
coisa dada de graa. Foi da que veio o significado moderno: uma
consequncia quase que imediata (portanto, gratuita) de uma afirmao ou teorema.

3.4.1

Um Exemplo

Encerramos o captulo considerando um exemplo mais geral de


clculo do inverso, que ser muito importante em nossas aplicaes
do RSA. Suponha que o inteiro positivo n possa ser escrito na forma
n = 6 k 2, onde k > 0 um inteiro. Os primeiros dez nmeros que
satisfazem esta propriedade esto listados na prxima tabela juntamente com os valores correspondentes para k:
k
6k2

1
4

2
10

3
16

4
22

5
28

6
34

7
40

8
46

9
52

10
58

Acontece que 3 e 6 k 2 no podem ter nenhum fator primo comum,


mas, ao invs de provar isto, vou deixar como exerccio.
Exerccio 36. Mostre que 3 e 6 k 2 no admitem nenhum fator
primo em comum.
Em vista disto, o teorema nos garante que 3 deve ter inverso mdulo n = 6 k 2. Mas ser que somos capazes de calcular este

i
i

cripto
2009/6/30
i page 100
Estilo OBMEP

i
i

100

CAP. 3: INVERSOS MODULARES

inverso? A resposta bem fcil. Como n = 6 k 2, ento


n 1 = 6 k 3.
Pondo 3 em evidncia
n 1 = 3(2 k 1);
isto ,
n = 3(2 k 1) + 1.
Assim,
3(2 k 1) + 1 0

(mod n);

donde
3(2 k 1) 1

(mod n);

que pode ser reescrito como


3(1 2 k) 1

(mod n).

Logo, 1 2 k o inverso de 3 mdulo n. Como 1 2 k negativo,


vamos determinar o seu resduo. Somando n = 6 k 2, obtemos
1 2 k + n = 1 2 k + 6 k 2 = 4 k 1;
que positivo para todo k 1. Alm disso, como 4k < 6k, tambm
temos que
4 k 1 < 6 k 2;
de forma que 4k1 mesmo o resduo de 12k mdulo n = 6k2.

i
i

cripto
2009/6/30
i page 101
Estilo OBMEP

i
i

N SEC. 3.4: O TEOREMA E UM EXEMPLO

101

Exerccio 37. Calcule os inversos de 2, 3 e 6 mdulo 6 k + 1.

i
i

cripto
2009/6/30
i page 102
Estilo OBMEP

i
i

Captulo 4

Algoritmo Chins do Resto


Neste captulo veremos como calcular um inteiro que satisfaz simultaneamente a vrias congruncias com mdulos distintos: o chamado algoritmo chins do resto.

4.1

Exemplos

Comeamos analisando um exemplo bastante simples.

4.1.1

Restos

Considere o seguinte problema:


determine o menor inteiro positivo que deixa resto 1 na
diviso por 3 e resto 2 na diviso por 5.
102

i
i

cripto
2009/6/30
i page 103
Estilo OBMEP

i
i

103

N SEC. 4.1: EXEMPLOS

Note que este exemplo simples o suficiente para que possamos


resolv-lo de cabea. Contudo, nas aplicaes ao RSA, encontraremos
sistemas muito maiores, que s conseguiremos resolver procedendo de
maneira sistemtica, que outra forma de dizer usando um algoritmo.
Comearemos descrevendo a aplicao do algoritmo geral ao exemplo
acima.
Chamando de n o inteiro que buscamos, podemos escrever as
equaes correspondentes diviso de n por 3 e 5 na forma
n = 3q1 + 1,
n = 5q2 + 2.
Observe que usamos smbolos diferentes (q1 e q2 ) para denotar os
quocientes destas divises. Afinal, no h nenhuma razo para que os
quocientes das duas divises sejam os mesmos, e usar a mesma letra
automaticamente implicaria esta igualdade incorreta.
Voltando ao sistema, temos duas equaes com trs variveis, a
saber n, q1 e q2 . Como se isto no bastasse, queremos determinar uma
soluo inteira, o que complica ainda mais o problema. Entretanto,
estas equaes podem ser reescritas de uma maneira mais simples se
usarmos congruncias. Fazendo isto, obtemos
n 1 (mod 3),
n 2 (mod 5).
primeira vista a reformulao foi tima; afinal, sobrou apenas
uma varivel: o que podia ser melhor? O problema como usar
as congruncias para determinar o inteiro desejado. Geralmente,

i
i

cripto
2009/6/30
i page 104
Estilo OBMEP

i
i

104

CAP. 4: ALGORITMO CHINS DO RESTO

quando temos mais de uma equao para resolver, tentamos combinlas para achar a resposta desejada. Entretanto, estas duas congruncias tm mdulos diferentes e, portanto, no podemos combin-las
diretamente. O que fazer?
A sada usar uma estratgia hbrida: substituiremos n = 5q2 + 2
no na equao n = 3 q1 + 1, mas sim na primeira congruncia, isto
, em n 1 (mod 3). Efetuando a substituio, obtemos
5q2 + 2 1

(mod 3).

Acontece que 5 2 (mod 3), de forma que a congruncia pode ser


reescrita na forma
2q2 + 2 1 (mod 3).
Subtraindo 2 dos dois lados da congruncia, chegamos a
2q2 1

(mod 3);

ou ainda a
2q2 2

(mod 3),

j que 1 2 (mod 3). Como 2 inversvel mdulo 3, podemos


cancel-lo na congruncia acima pelo teorema 2, o que nos d
q2 1 (mod 3).
Em outras palavras, q2 deixa resto 1 na diviso por 3, de modo que
podemos escrev-lo como
q2 = 3q3 + 1,

i
i

cripto
2009/6/30
i page 105
Estilo OBMEP

i
i

105

N SEC. 4.1: EXEMPLOS

onde q3 corresponde ao quociente desta diviso.


Voltando ao problema original, temos, alm das equaes
n = 3q1 + 1,
n = 5q2 + 2,
originalmente obtidas, uma nova equao
q2 = 3q3 + 1,
que explicita q2 , ainda que seja ao preo de introduzir uma nova varivel. Mas isto nos permite substituir o valor de q2 diretamente na
segunda das duas equaes originalmente obtidas, o que nos d
n = 5q2 + 2 = 5(3q3 + 1) + 2.
Fazendo as contas,
n = 15q3 + 7.
E da? Tnhamos duas equaes. Fizemos uma peripcia usando
congruncias. Chegamos a uma nova equao em tudo semelhante s
originais. Grande coisa!
Se estes pensamentos lhe passaram pela cabea, ento prepare-se
para uma surpresa. O que acontece se dividirmos 15q3 + 7 por 3?
Para comear, 15 = 3 5, de forma que
15q3 + 7 = 3 5q3 + 7.

i
i

cripto
2009/6/30
i page 106
Estilo OBMEP

i
i

106

CAP. 4: ALGORITMO CHINS DO RESTO

Se 7 fosse menor que 3, seria o resto desta diviso, como 7 3,


precisamos escrev-lo na forma
7 = 3 2 + 1.
Combinando as duas equaes e pondo 3 em evidncia, obtemos
15q3 + 7 = 3 (5q3 + 2) + 1;
logo 15q3 +7 deixa resto 1 na diviso por 3, exatamente o que queramos
que acontecesse com o n a ser determinado em nosso problema. Com
uma vantagem: isto acontece qualquer que seja o valor escolhido para
q3 !
Passando diviso por 5, temos que
15q3 + 7 = 5 3q3 + 5 + 2 = 5(3q3 + 1) + 2;
de forma que 15q3 + 7 deixa resto 2 na diviso por 5, satisfazendo,
mais uma vez, ao que foi pedido no problema. Isto sugere que devemos
considerar a soluo como sendo n = 15q3 + 7.
Observe, contudo, que o que obtivemos no foi uma soluo, mas
sim uma famlia de solues. De fato, obteremos uma soluo diferente para cada valor inteiro que escolhermos para q3 , como ilustrado
na tabela 4.1.

i
i

cripto
2009/6/30
i page 107
Estilo OBMEP

i
i

107

N SEC. 4.1: EXEMPLOS

q3
-3
-2
-1
0
1
2
3

15q3 + 7
-38
-23
-8
7
22
37
52

Tabela 4.1: Tabelando n = 15q3 + 7


Dito isto, fica difcil no perguntar se todas as possveis solues
deste problema podem ser obtidas da frmula n = 15q3 + 7 simplesmente escolhendo um valor adequado para q3 . A resposta sim, mas
para entender porque voc ter que esperar at a seo 4.2.
Relendo o problema, verificamos que ainda h uma condio a ser
satisfeita: queremos o menor n positivo que satisfaz as duas condies
sobre os restos. Entretanto, como mostra a tabela:
se q3 < 0, ento 15q3 + 7 < 0;
se q3 > 0, ento 15q3 + 7 > 7;
de modo que o valor desejado mesmo n = 7.
Antes de passar a um novo exemplo, vamos refazer a verificao
de que n = 15q3 + 7 nos d uma famlia de solues para a equao.
S que desta vez usaremos congruncias. Da igualdade n = 15q3 + 7
obtemos a congruncia
n 15q3 + 7 1 1 (mod 3);

i
i

cripto
2009/6/30
i page 108
Estilo OBMEP

i
i

108

CAP. 4: ALGORITMO CHINS DO RESTO

pois 15 0 (mod 3) e 7 1 (mod 3). Da forma semelhante,


n 15q3 + 7 2 2

(mod 5).

Estas verificaes so muito mais diretas e automticas e, daqui por


diante, sero usadas como nossa maneira-padro de testar a correo
de nossas solues.
Exerccio 38. Na encenao de uma batalha, duas tropas se enfrentam, posicionando-se, atirando com festim, e recarregando seus
mosquetes, cada uma a sua vez. Cada lado comea com o mesmo
nmero de cartuchos. Uma tropa tem 100 mosquetes e, depois de atirar tantos tiros de festim quanto possveis, lhe sobram 13 cartuchos.
A outra tropa tem 67 mosquetes, e ao fim da exibio, sobram-lhe 32
cartuchos. Supondo que a cada salva de tiros todos os soldados de
cada lado atiraram exatamente uma vez, determine o nmero mnimo
de cartuchos com que cada tropa iniciou a exibio.

4.1.2

Um Exemplo Astronmico

Desta vez o problema trata de tempos e no de restos:


Trs satlites passaro sobre o Rio esta noite. O primeiro
1 hora da madrugada, o segundo s 4 horas e o terceiro s 8
horas da manh. Cada satlite tem um perodo diferente.
O primeiro leva 13 horas para completar uma volta em
torno da Terra, o segundo 15 horas e o terceiro 19 horas.
Determine quantas horas decorrero, a partir da meianoite, at que os trs satlites passem ao mesmo tempo
sobre o Rio.

i
i

cripto
2009/6/30
i page 109
Estilo OBMEP

i
i

109

N SEC. 4.1: EXEMPLOS

Podemos formular o problema de maneira muito semelhante que


adotamos na seo anterior, basta lembrar nossa interpretao do
mdulo como o perodo de um movimento que se repete a intervalos
regulares. Neste caso, o movimento o dos satlites que giram em
torno da Terra.
Chamaremos de x o nmero de horas, contadas a partir da meianoite de hoje, quando os trs satlites passaro juntos sobre o Rio.
O primeiro satlite passa sobre o Rio a cada 13 horas, a contar da 1
da madrugada. Logo precisamos ter que x = 1 + 13n1 , para algum
inteiro positivo n1 , que representa o nmero de voltas que o satlite
1 tem que dar em torno da Terra antes que passe junto com os dois
outros satlites.
As equaes correspondentes aos outros dois satlites so
x = 4 + 15n2

x = 8 + 19n3 ;

onde n2 e n3 representam o nmero de voltas que os satlites 2 e 3


daro antes dos trs passarem juntos.
Como fizemos para o problema anterior, podemos reformular estas
equaes em termos de congruncias, o que nos d
x1

(mod 13),

(4.1.1)

x 4 (mod 15),
x 8 (mod 19).
Desta vez temos trs equaes, ao contrrio das duas do problema
anterior, mas no vamos nos deixar intimidar. J que o mtodo

i
i

cripto
2009/6/30
i page 110
Estilo OBMEP

i
i

110

CAP. 4: ALGORITMO CHINS DO RESTO

que desenvolvemos s permite resolver duas equaes de cada vez,


comearemos com as duas ltimas. Tomando a ltima equao e
substituindo-a na penltima congruncia, obtemos
8 + 19n3 4 (mod 15); que equivale a 19n3 4

(mod 15).

Como 19 4 (mod 15), isto nos d


4n3 4 (mod 15).
Como 4 inversvel mdulo 15 pelo teorema 3, podemos cancel-lo,
de modo que
n3 1 14 (mod 15).
Assim, n3 = 14 + 15n4 , para algum inteiro positivo n4 . Mas, segundo
a terceira equao, x = 8 + 19n3 . Combinando estas duas expresses
x = 8 + 19(14 + 15n4 ) = 274 + 285n4 .
O que isto representa? Certamente no a soluo do problema,
j que sequer usamos as condies impostas pelo primeiro satlite.
Entretanto, como fcil verificar usando congruncias,
x = 274 + 285n4
nos d uma soluo das duas ltimas equaes.Isto significa que esta
famlia de solues deve corresponder aos tempos nos quais os satlites
2 e 3 passam juntos sobre o Rio.

i
i

cripto
2009/6/30
i page 111
Estilo OBMEP

i
i

111

N SEC. 4.1: EXEMPLOS

E quanto ao satlite 1? Para incluir na soluo a informao


referente ao primeiro satlite, basta encontrar as solues da forma
x = 274 + 285n4 (isto , as solues comuns aos satlites 2 e 3) que,
alm disso, satisfazem a congruncia x 1 (mod 13), relativa ao
primeiro satlite. Efetuando a substituio,
274 + 285n4 1 (mod 13);
que depois da reduo mdulo 13 nos d
1 + 12n4 1 (mod 13).
Logo 12n4 0 (mod 13) e, como 12 inversvel mdulo 13, conclumos que n4 = 13n5 . Desta forma, a soluo final ser
x = 274 + 285n4 = 274 + 285(13n5 ) = 274 + 3705n5 ,
como fcil verificar substituindo esta frmula para x nas congruncias (4.1.1).
Resta-nos explicitar o que esta soluo nos diz sobre os satlites.
Em primeiro lugar, como fcil verificar, 274 o menor inteiro positivo que satisfaz as congruncias (4.1.1). Portanto, os satlites passam
juntos sobre o cu do Rio pela primeira vez 274 horas depois da meianoite de hoje. Isto equivale a 11 dias e 10 horas. Mas isto no tudo.
Afinal, no importa qual seja o valor de n5 , a frmula 274 + 3 705n5
nos d uma soluo do problema. Portanto, depois de passar juntos
uma vez sobre o Rio 274 horas depois da zero hora de hoje, os satlites
passaro juntos novamente a cada 3 705 horas; isto , a cada 154 dias
e 9 horas.

i
i

cripto
2009/6/30
i page 112
Estilo OBMEP

i
i

112

CAP. 4: ALGORITMO CHINS DO RESTO

Na prxima seo faremos uma anlise detalhada do mtodo acima.


Observe que nossa estratgia consistiu em dividir a soluo do sistema
(4.1.1) de 3 equaes em duas etapas. Primeiro achamos uma soluo
comum s duas ltimas congruncias, que foi x = 274 + 285n4 . Em
seguida, buscamos as solues comuns s duas ltimas congruncias
que tambm satisfazem primeira. Como x = 274 + 285n4 corresponde congruncia,
x 274 (mod 285),
substitu-la na primeira congruncia equivale a resolver o sistema
x 1 (mod 13),
x 274 (mod 285).
Uma outra maneira de expressar isto consiste em dizer que a
soluo de um sistema de muitas equaes obtida atravs da soluo
de vrios sistemas de duas equaes cada. Por isso, na seo 3 suficiente analisar o algoritmo correspondente soluo de um sistema
de duas equaes.
Nosso prximo exerccio vem do banco de questes da OBMEP2007 (p. 76).
Exerccio 39. O nmero 119 tem a seguinte propriedade:
a diviso por 2 deixa resto 1;
a diviso por 3 deixa resto 2;
a diviso por 4 deixa resto 3;

i
i

cripto
2009/6/30
i page 113
Estilo OBMEP

i
i

N SEC. 4.2: O TEOREMA CHINS DO RESTO

113

a diviso por 5 deixa resto 4;


a diviso por 6 deixa resto 5.
Quantos inteiros positivos menores que 2007 satisfazem essa propriedade?
Exerccio 40. Um velho problema chins:
Trs fazendeiros cultivavam juntos todo o seu arroz e o
dividiam igualmente entre si no tempo da colheita. Um
certo ano cada um deles foi a um mercado diferente vender
o seu arroz. Cada um destes mercados s comprava arroz
em mltiplos de um peso padro, que diferia em cada um
dos mercados. O primeiro fazendeiro vendeu o seu arroz
em um mercado onde o peso padro era 87 kg. Ele vendeu
tudo o que podia e voltou para casa com 18 kg de arroz.
O segundo fazendeiro vendeu todo o arroz que podia em
um mercado cujo peso padro era de 170 kg e voltou para
casa com 58 kg. O terceiro fazendeiro vendeu todo o arroz
que podia em um mercado cujo peso padro era de 143 kg
e voltou (ao mesmo tempo que os outros dois) com 40 kg.
Qual a quantidade mnima de arroz que eles podem ter
cultivado, no total?

4.2

O Teorema Chins do Resto

O procedimento de substituio que utilizamos nas sees anteriores para resolver sistemas de congruncias conhecido como algoritmo chins do resto, porque um dos primeiros lugares em que

i
i

cripto
2009/6/30
i page 114
Estilo OBMEP

i
i

114

CAP. 4: ALGORITMO CHINS DO RESTO

aparece o livro Manual de aritmtica do mestre Sun, escrito entre


287 d.C. e 473 d.C. Entretanto, o mesmo resultado mencionado na
Aritmtica de Nicmaco de Gerasa, escrita por volta de 100 d.C. O
teorema desta seo apenas sistematiza o resultado final do mtodo
utilizado nos problemas das sees anteriores.
Considere o sistema
xa

(mod m),

(4.2.1)

x b (mod n),
onde m e n so inteiros positivos distintos e digamos que o nmero
inteiro x0 uma soluo desta congruncia. Isto significa que x0
satisfaz a ambas as congruncias:
x0 a (mod m),
x0 b (mod n).
Como os mdulos so diferentes, s podemos combinar as duas congruncias se convertermos uma delas em uma igualdade de inteiros.
Fazendo isto com a primeira equao, verificamos que
x0 = a + m k, onde k um inteiro qualquer,

(4.2.2)

de forma que podemos concluir que


a + mk b (mod n),
ou ainda
mk (b a)

(mod n).

(4.2.3)

i
i

cripto
2009/6/30
i page 115
Estilo OBMEP

i
i

115

N SEC. 4.2: O TEOREMA CHINS DO RESTO

Supondo que m e n sejam primos entre si, conclumos pelo teorema


3 que m inversvel mdulo n como vimos no teorema 4.2 da pgina
113. Digamos que m0 o inverso de m mdulo n. Multiplicando
(4.2.3) por m0 , obtemos
k m0 (b a) (mod n).
Em outras palavras,
k = m0 (b a) + n t para algum inteiro `.
Substituindo esta expresso para k em (4.2.2), vemos que
x0 = a + m(m0 (b a) + n t).
Resumindo, provamos que se x0 uma soluo de (4.2.1), ento
x0 = a + m (m0 (b a) + n t).

(4.2.4)

Mas fcil ver que, qualquer que seja o inteiro t, uma expresso
da forma a + m(m0 (b a) + n t) tem que ser soluo do sistema
(4.2.1). Para comeo de conversa, a + m(m0 (b a) + n t) claramente
congruente a a mdulo m. Por outro lado,
a + m (m0 (b a) + n t) a + m m0 (b a) (mod n).
Como, mm0 1 (mod n) por construo, ento
a + m (m0 (b a) + n t) a + 1 (b a) b (mod n);

i
i

cripto
2009/6/30
i page 116
Estilo OBMEP

i
i

116

CAP. 4: ALGORITMO CHINS DO RESTO

comprovando que a + m (m0 (b a) + n t) mesmo uma soluo do


sistema (4.2.1). Podemos resumir o que fizemos no seguinte teorema.
Teorema Chins do Resto. Sejam m e n inteiros positivos primos
entre si. Se a e b so inteiros quaisquer, ento o sistema
xa

(mod m),

x b (mod n),
sempre tem soluo e qualquer uma de suas solues pode ser escrita
na forma
a + m (m0 (b a) + n t),
onde t um inteiro qualquer e m0 o inverso de m mdulo n.
Cuidado para no se confundir e achar que mm0 = 1, j que m e
m0 so inversos um do outro. De fato eles so inversos, mas somente
mdulo n, de modo que a relao correta mm0 1 (mod n); que
no simplifica a frmula de nenhuma maneira significativa.

4.2.1

Quando os Mdulos No so Primos Entre Si

Apesar de termos obtido uma frmula exata para a soluo de


sistemas de duas congruncias, isto foi feito ao preo de uma hiptese
bastante forte, a de que os mdulos so primos entre si. Ser que a
frmula continua verdadeira mesmo se esta hiptese no se verifica?
Se voc reler o argumento usado para provar a frmula ver que
precisamos que os mdulos fossem primos entre si em apenas um
ponto: para inverter m na congruncia mk (b a) (mod n) e assim

i
i

cripto
2009/6/30
i page 117
Estilo OBMEP

i
i

N SEC. 4.2: O TEOREMA CHINS DO RESTO

117

determinar o valor de k. Isto significa que a estratgia usada acima


no funcionaria se m e n no fossem primos entre si. Mas ser que
no h outra estratgia possvel neste caso? A resposta sim... e no.
Vejamos por qu?
Para isto analisaremos dois exemplos muito semelhantes. O primeiro deles
x3

(mod 4),

x 1 (mod 6),
e o segundo
x2

(mod 4),

x 1 (mod 6).
Note que a nica diferena entre eles est no coeficiente direita da
primeira congruncia que, no primeiro exemplo 3 e no segundo
2. Procederemos exatamente como antes. Portanto, comeamos por
tirar o valor de x da segunda congruncia, que nos d
x = 1 + 6y para algum inteiro y.

(4.2.5)

Substituindo isto na primeira, obtemos no primeiro exemplo


6y 2 (mod 4);

(4.2.6)

6y 1 (mod 4).

(4.2.7)

e no segundo

i
i

cripto
2009/6/30
i page 118
Estilo OBMEP

i
i

118

CAP. 4: ALGORITMO CHINS DO RESTO

Chegados a este ponto, no podemos prosseguir, porque 6 e 4 tm 2


como fator comum, de modo que 6 no inversvel mdulo 4. Contudo, convertendo (4.2.6) para uma igualdade de inteiros, vemos que
6y = 2 + 4z, para algum inteiro z.
Acontece que 2 divide cada uma das parcelas desta equao. Efetuando a diviso, obtemos
3y = 1 + 2z.
Convertendo esta igualdade em uma congruncia, ficamos com
3y 1 (mod 2);
que, como 3 1 (mod 2), nos d
y1

(mod 2);

isto
y = 1 + 2t para algum inteiro t.
Substituindo em (4.2.5),
x = 1 + 6(1 + 2t) = 7 + 12t,
que a soluo do sistema, como podemos facilmente verificar por
substituio.
Passando agora ao outro sistema, precisamos resolver a congrun-

i
i

cripto
2009/6/30
i page 119
Estilo OBMEP

i
i

119

N SEC. 4.2: O TEOREMA CHINS DO RESTO

cia (4.2.7). Convertendo-a em uma igualdade de inteiros, temos


6y = 1 + 4z, para algum inteiro z.
Contudo, desta vez o divisor comum dos trs coeficientes da equao
1. Rearrumando a equao anterior, obtemos
6y 4z = 1
que, como 2 divide 6 e 4, pode ser reescrita na forma
2(3y 2z) = 1.

(4.2.8)

Entretanto, se existissem nmeros inteiros y e z que satisfizessem


esta equao, teramos que 1 mltiplo de 2; o que evidentemente
falso. Mas (4.2.8) consequncia de (4.2.7), de modo que esta ltima
tambm no pode ter soluo!
Resumindo, estes exemplos nos mostram que, quando os mdulos no so primos entre si, o sistema pode ou no ter soluo, dependendo dos coeficientes constantes que aparecem nas congruncias.
Ser que podemos prever isto s de olhar para os coeficientes? A resposta sim e enunciada abaixo. Provar que est correta fica como
desafio para voc.
Desafio 4. Considere o sistema de congruncias
x a (mod m),
x b (mod n).
Suponha que o mximo divisor comum entre m e n d. Aplique o

i
i

cripto
2009/6/30
i page 120
Estilo OBMEP

i
i

120

CAP. 4: ALGORITMO CHINS DO RESTO

procedimento de substituio do algoritmo chins a este sistema para


mostrar que:
(a) se d divide b a ento o sistema tem soluo;
(b) se d no divide b a ento o sistema no tem soluo.

i
i

cripto
2009/6/30
i page 121
Estilo OBMEP

i
i

Captulo 5

Potncias
Neste captulo veremos como calcular os restos de potncias usando
aritmtica modular. Lembre-se que j fizemos um pouco disto quando
calculamos as potncias de 10 mdulo 3, mdulo 7 e mdulo 11 ao
tratar dos critrios de divisibilidade na seo 2.3 do captulo 2.

5.1

Restos de Potncias

Uma aplicao importante das congruncias ao clculo de restos


da diviso de uma potncia por um nmero qualquer. Comearemos
com alguns exemplos simples.

5.1.1

Minhas Primeiras Potncias Modulares

Suponhamos que queremos calcular o resto da diviso de 10135


por 7. Vimos na pgina 70 que 106 1 (mod 7). Dividindo 135
121

i
i

cripto
2009/6/30
i page 122
Estilo OBMEP

i
i

122

CAP. 5: POTNCIAS

por 6 temos 135 = 6 22 + 3. Temos ento as seguintes congruncias


mdulo 7:
10135 (106 )22 103 (1)22 103 6

(mod 7).

Logo, 10135 6 (mod 7). Como 0 6 < 7, podemos concluir que o


resto da diviso de 10135 por 7 6.
Exerccio 41. Calcule o resto da diviso por 7 das potncias 1065
e 378 .
Outro exemplo, mais exagerado. Qual o resto da diviso de 2124 512
por 31? Calculando as potncias de 2 mdulo 31, vemos que
22 4 (mod 31),
23 8 (mod 31),
24 16 (mod 31),
25 32 1

(mod 31).

De modo semelhante ao que ocorreu com as potncias de 10 mdulo


7, somos capazes de descobrir uma potncia de 2 que d 1 mdulo 31.
Procederemos como no exemplo anterior, s que desta vez usaremos a
congruncia 25 1 (mod 31) para fazer as simplificaes. Dividimos
124 512 por 5, obtemos quociente 4 016 e resto 2. Portanto,
2124 512 224 9025+2 (25 )24 902 22

(mod 31).

Como 25 1 (mod 31), temos


2124 512 (1)24 902 22 4

(mod 31).

i
i

cripto
2009/6/30
i page 123
Estilo OBMEP

i
i

123

N SEC. 5.1: RESTOS DE POTNCIAS

Como 0 4 < 31, podemos concluir que 2124 512 deixa resto 4 na
diviso por 31.

Exerccio 42. Calcule o resto da diviso por 31 das potncias 26 556 423
e 27 987 668 .

Para falar a verdade, podemos exagerar ainda mais. Por exemplo,


98 765
qual o resto da diviso de 211
por 31? Lembre-se que para calcular
98 765
11
2
determinamos primeiro 1198 765 e depois elevamos 2 a este
expoente. O resultado um nmero enorme, com mais de 25 mil
algarismos. O primeiro problema que esta questo pe o de como
calcular o quociente e o resto da diviso de 1198 765 por 5. A bem da
verdade, o problema como calcular o quociente porque, para o resto,
podemos usar congruncias. De fato, como 11 1 (mod 5), ento
1198 765 198 765 1 (mod 5).
Logo, ao dividir 1198 765 por 5 obtemos resto 1. Quanto ao quociente,
no precisamos sequer saber quanto vale. Para se convencer disso,
releia os exemplos que acabamos de fazer. Em ambos, apenas 1 que
elevamos ao quociente. Escrevendo, ento,
1198 765 = 5 q + 1,
onde q o tal quociente que no conhecemos, obtemos
211

98 765

25q+1 (25 )q 2

(mod 31).

i
i

cripto
2009/6/30
i page 124
Estilo OBMEP

i
i

124

CAP. 5: POTNCIAS

Como 25 1 (mod 31),


211

98 765

(1)q 2 2 (mod 31);

e o resto da diviso de 211

98 765

por 31 2.

Se voc prestou muita ateno s contas, talvez tenha pensado:


Ele est blefando! A conta s ficou fcil porque 11 deixa
resto 1 na diviso por 5 e 1 elevado a 98 765 d 1. Se em
vez de 11 fosse 13, seria muiiiiito mais difcil!
Tudo bem, vejamos o que acontece quando tentamos calcular o resto
98 765
da diviso de 213
por 31. Neste caso, o ponto crucial calcular o
resto da diviso de 1398 765 por 5. Usando congruncias,
1398 765 398 765

(mod 5),

o que parece sugerir que seu comentrio se justifica. Porm, calculando as potncias de 3 mdulo 5, vemos facilmente que
34 81 1

(mod 5).

Portanto, podemos aplicar a 398 765 o j conhecido argumento, e dividir o expoente da potncia por 4. Como o resto da diviso de 98 765
por 4 1 e o quociente 24 691, obtemos
398 765 3424 691+1 3

(mod 5).

Logo, 1398 765 deixa resto 3 na diviso por 5; isto , 1398 765 = 5 q 0 + 3

i
i

cripto
2009/6/30
i page 125
Estilo OBMEP

i
i

125

N SEC. 5.1: RESTOS DE POTNCIAS

e, mais uma vez, o quociente q 0 no precisa ser calculado. Assim,


98 765

213

(25 )q 23 23 8

e o resto da diviso de 213


muiiiiito mais difcil!

98 765

(mod 31);

por 31 8. Mais difcil foi, mas no

Exerccio 43. Calcule o resto da diviso por 31 das potncias 214


498 766 543 335 231
9 876
215
e 643
.

5.1.2

45 231

Ordem de um Inteiro Modular

Os clculos com potncias feitos acima s foram to fceis de executar porque, em cada caso, descobrimos um expoente positivo para o
qual uma potncia da base dava 1 quando tomada em mdulo. Assim,
106 36 1

(mod 7), ao passo que, 34 25 1 (mod 31).

Ser que isto sempre possvel? Isto ,


ser que, dados dois inteiros positivos b < n sempre existe
um inteiro positivo k tal que bk 1 (mod n)?
Observe que estamos exigindo que k seja positivo; sem esta hiptese
poderamos tomar k = 0, mas isto em nada nos ajuda em nossos
clculos.
Como dar nome aos conceitos facilita falar sobre eles, vamos introduzir a seguinte terminologia. Se 1 b n 1 so inteiros, diremos
que a ordem de b mdulo n o menor inteiro positivo k para o qual
bk 1 (mod n). Note que, embora anteriormente apenas falssemos

i
i

cripto
2009/6/30
i page 126
Estilo OBMEP

i
i

126

CAP. 5: POTNCIAS

de uma potncia congruente a 1 com expoente positivo, acabamos por


introduzir o adjetivo menor ao escrever a definio. A razo que,
do contrrio, o expoente k no estaria completamente determinado.
Por exemplo, j vimos que
25 1 (mod 31);
contudo,
210 (25 )2 12 1 (mod 31),
assim como
2105 (25 )21 121 1 (mod 31).
Na verdade,
25k (25 )k 1k 1 (mod 31),
no importa qual seja o inteiro positivo k. Este exemplo facilmente
generalizvel. De fato, se ak 1 (mod n), ento
akm (ak )m 1m 1

(mod n),

para qualquer inteiro m 1 que voc escolher. Interpretando os


clculos do incio da seo usando esta terminologia, podemos dizer
que 3 tem ordem 6 mdulo 7 e que 2 tem ordem 5 mdulo 31. Antes
de prosseguir seria bom voc fazer alguns exemplos para verificar que
entendeu mesmo o conceito de ordem.
Exerccio 44. Calcule a ordem de
(a) 3 mdulo 7;
(b) 2 mdulo 11;

i
i

cripto
2009/6/30
i page 127
Estilo OBMEP

i
i

127

N SEC. 5.1: RESTOS DE POTNCIAS

(c) 5 mdulo 31;


(d) 7 mdulo 43.
Voltando pergunta, podemos agora reformul-la da seguinte
maneira:
Ser que todo inteiro 1 b n 1 tem alguma ordem
mdulo n?
Precisamos experimentar um pouco mais, antes de ensaiar uma concluso. Revendo os exemplos do incio desta seo constatamos que 7
e 31 so primos, mas o que acontece se escolhermos um mdulo que
no seja primo? Por exemplo, ser que existe uma potncia de 2 que
d 1 mdulo 6? Tentando:
21 2 (mod 6),
22 4

(mod 6),

23 8 2 (mod 6),
24 16 4 (mod 6),
e j deu para ver que os valores das potncias de 2 mdulo 6 vo se
alternar entre 2 e 4. Assim, podemos concluir que nenhuma potncia
de 2 d congruente a 1 mdulo 6. Alis, isto fcil de generalizar,
como mostra o prximo exerccio.
Exerccio 45. Mostre que se a e n so inteiros positivos pares, ento
nenhuma potncia de a congruente a 1 mdulo n.
Voltando ao exemplo, o que mais voc acha que podemos concluir
dos clculos acima? Algum mais ousado talvez ache que isto indica

i
i

cripto
2009/6/30
i page 128
Estilo OBMEP

i
i

128

CAP. 5: POTNCIAS

que potncias de inteiros mdulo 6 nunca do 1. Ou, quem sabe, at


que potncias de inteiros mdulo um nmero composto nunca do 1.
A verdade, contudo, bem mais sutil.
Voltando aos nossos experimentos, porque parar em 2? Por que
no tentar tambm 3? Pois bem, aqui esto as potncias de 3 mdulo
6:
31 3

(mod 6),

32 9 3 (mod 6),
33 27 3 (mod 6),
34 81 3

(mod 6).

No que 3 foi ainda pior que 2! Todas as potncias positivas de 3


so congruentes a 3. Mas, no desanimemos, tentemos as potncias
de 4,
41 4

(mod 6),

4 16 4 (mod 6),
tudo bem, j podemos parar: toda potncia de 4 mdulo 6 d 4. Ao
que tudo indica, nenhuma potncia positiva de um inteiro mdulo 6
d igual a 1 a no ser que o inteiro seja 1, claro! Mas, s para
tirar a prova, testemos o nico inteiro menor que 6 cujas potncias
ainda no calculamos, o 5. Contudo,
52 25 1 (mod 6),
de modo que 5 tem ordem 2 mdulo 6.

i
i

cripto
2009/6/30
i page 129
Estilo OBMEP

i
i

N SEC. 5.1: RESTOS DE POTNCIAS

129

Surpreso? A bem da verdade, voc no devia estar porque este


resultado poderia ter sido previsto, desde que voc lembrasse do exerccio 31. Segundo aquele exerccio n 1 seu prprio inverso mdulo
n. Mas isto significa que
(n 1) (n 1) 1 (mod n),
que podemos reescrever como
(n 1)2 1 (mod n);
o que mostra que n 1 sempre tem ordem dois mdulo n. E isto vale,
no importa qual seja o valor do inteiro n > 1. O que vimos no caso
n = 6 que os nicos inteiros entre 1 e 6 que tm ordem mdulo 6
so 1 e n 1 = 5.
O caso do n 1 acena com a possibilidade de haver uma relao
entre invertibilidade mdulo n e a existncia de uma ordem mdulo
n. Para poder explorar melhor esta relao suponha que b, n e k so
inteiros positivos e que
bk 1 (mod n).
Se k = 1, ento b 1 (mod n) e no h nada a dizer. Por isso podemos supor que k 2. Neste caso,
b bk1 1 (mod n).
Mas isto significa que bk1 funciona como o inverso de b mdulo n.
Pelo teorema 3 da pgina 97 isto s possvel se b e n forem primos

i
i

cripto
2009/6/30
i page 130
Estilo OBMEP

i
i

130

CAP. 5: POTNCIAS

entre si. Portanto,

se 1 b n 1 tem ordem mdulo n ento b e n so


primos entre si.

Isto explica porque apenas 1 e 5 admitem potncias positivas congruentes a um mdulo 6 entre todos os inteiros positivos menores que
6, afinal, 2, 3 e 4 tm fatores prprios comuns com 6. Por outro lado,
se p > 1 primo ento nenhum inteiro 1 b p1 tem fator prprio
comum com p e, portanto, todos estes inteiros so inversveis mdulo
p:

Ser que todos estes nmeros admitem uma ordem


mdulo p?

A resposta sim, como veremos na seo 5.2. Por enquanto, vamos


determinar a ordem mdulo 7 de cada um dos inteiros positivos menores que 7.
22

Comeamos por 2, j que 1 tem obviamente ordem um. Como


= 4 < 7, a primeira potncia interessante o cubo, mas
23 8 1

(mod 7);

logo, 2 tem ordem 3 mdulo 7. J sabemos que 3 tem ordem 6, por


isso passamos ao 4. Porm,
42 16 2 (mod 7).

i
i

cripto
2009/6/30
i page 131
Estilo OBMEP

i
i

131

N SEC. 5.1: RESTOS DE POTNCIAS

Como 2 tem ordem 3, vamos precisar elevar 42 ao cubo para encontrar


1. Logo,
46 1 (mod 7).
Contudo, isto no impede, em princpio, que uma potncia menor de
4 no possa dar igual a 1. Testando as demais potncias, vemos que
43 42 4 2 4 8 1 (mod 7);
de modo que a ordem de 4 mdulo 7 3 e no 6 como o clculo
anterior nos teria feito esperar!
O ltimo nmero a considerar 5, porque j vimos que 6 = 7 1
tem que ter ordem 2 mdulo 7. Neste caso,
52 25 4 (mod 7),
53 5 4 20 6

(mod 7),

54 5 6 30 2

(mod 7),

55 5 2 10 3

(mod 7),

5 5 3 15 1

(mod 7).

Portanto, 5 tem ordem 6 mdulo 7. Podemos organizar o que descobrimos em uma tabela:

i
i

cripto
2009/6/30
i page 132
Estilo OBMEP

i
i

132

CAP. 5: POTNCIAS

Nmero
1
2
3
4
5
6

Ordem mdulo 7
1
3
6
3
6
2

Exerccio 46. Determine a ordem de cada um dos inteiros 1 b 10


mdulo 11.
Exerccio 47. Determine a ordem de cada um dos inteiros 1 b 11
mdulo 12. Lembre-se que alguns destes inteiros nem sequer admitem
uma ordem mdulo 12. Voc pode comear por descobrir quais so e
assim nem sequer precisar calcular com eles.

5.1.3

Mais Exemplos

J vimos que fica muito fcil calcular potncias de um nmero


mdulo n quando sua ordem (mdulo n) conhecida. O problema
que nem todo nmero tem ordem mdulo n quando n no primo.
Como proceder neste caso? Por exemplo, como determinar o resto de
635 por 16? Como 6 e 16 tm 2 como fator comum, podemos concluir
que 6 no tem ordem mdulo 16 e teremos que proceder de alguma
outra maneira. Contudo,
64 24 34 0 34 0

(mod 16);

de modo que
635 64 631 0 (mod 16),

i
i

cripto
2009/6/30
i page 133
Estilo OBMEP

i
i

133

N SEC. 5.1: RESTOS DE POTNCIAS

e, neste caso, as contas acabaram ficando bastante simples.


Por outro lado, mesmo quando um nmero tem ordem mdulo n,
esta pode ser to grande que fica difcil determin-la. Este o caso,
por exemplo, da ordem de 3 mdulo 31. J dissemos que, quando
p primo, todo nmero positivo menor que p tem ordem mdulo p:
logo 3 tem ordem mdulo 31. Mas a ordem grande e precisamos
de muito trabalho para determin-la. Em casos como este prefervel reduzir o expoente h algo mais fcil de calcular. Digamos, por
exemplo, que quisssemos determinar o resto da diviso de 364 por
31. Calculando os restos das potncias de 3 encontramos
33 27 4 (mod 31).
Mas 4 = 22 , de modo que
33 22

(mod 31).

claro que a vantagem de trabalhar com 2 est no fato de j conhecermos a ordem de 2. Usando esta ltima congruncia,
364 (33 )21 3 (22 )21 3 (2)42 3 (mod 31).
Como 25 1 (mod 31) e 42 = 8 5 + 2, temos que
242 (25 )8 22 4 (mod 31).
Assim,
364 (2)42 3 4 3 12 (mod 31).
Como 12 19 (mod 31), o resto da diviso de 364 por 31 19.

i
i

cripto
2009/6/30
i page 134
Estilo OBMEP

i
i

134

CAP. 5: POTNCIAS

Exerccio 48. Determine a ordem de 3 mdulo 31 e refaa o clculo do resto de 364 por 31 usando o resultado obtido. Mas, tenha pacincia, a ordem bem grande.
Exerccio 49. Calcule o resto da diviso de 398 745 por 43 procedendo
da seguinte maneira:
(a) calcule a ordem de 6 mdulo 43;
(b) determine uma potncia de 3 que d congruente a
6 mdulo 43;
(c) use (a) e (b) para calcular o resto desejado.

5.2

O Teorema de Fermat

Determinar a ordem exata de um dado inteiro mdulo n pode ser


uma tarefa bastante difcil se n for grande. Felizmente, no caso em
que n primo h um teorema que facilita muito nossa vida.
Teorema de Fermat. Se p um primo e a um inteiro que no
divisvel por p, ento
ap1 1 (mod p).
H quem chame este teorema de Pequeno Teorema de Fermat.
Contudo, levando em conta que este o resultado mais importante de
todo o nosso texto, cham-lo de Pequeno no parece muito apropriado.
A demonstrao do Teorema de Fermat que demos aqui foi descoberta

i
i

cripto
2009/6/30
i page 135
Estilo OBMEP

i
i

135

N SEC. 5.2: O TEOREMA DE FERMAT

pelo matemtico suo Leonard Euler no sculo XVIII e uma das


mais elementares.

Demonstrao. Comeamos a demonstrao do Teorema de Fermat


listando os possveis resduos mdulo p, que so
1, 2, 3, . . . , p 1.
Multiplicando cada um destes resduos por a, temos
a 1, a 2, a 3, . . . , a (p 1).
Digamos que r1 o resduo de a 1, que r2 o resduo de a 2 e assim
por diante at rp1 , que ser o resduo de a (p 1). Vamos calcular
o produto
r1 r2 rp1
mdulo p de duas maneiras diferentes.
Primeira maneira: levando em conta que
r1 a 1 (mod p),
r2 a 2 (mod p),
..
..
.
.
rp1 a (p 1)

(mod p);

podemos concluir que


r1 r2 r3 rp1 (a 1) (a 2) (a 3) (a (p 1)) (mod p).

i
i

cripto
2009/6/30
i page 136
Estilo OBMEP

i
i

136

CAP. 5: POTNCIAS

Contudo,
(a 1) (a 2) (a 3) (a (p 1)) = ap1 (1 2 3 (p 1));
de forma que
r1 r2 r3 rp1 ap1 (1 2 3 (p 1))

(mod p).

Segunda maneira: esta mais sutil. Comeamos observando que


no pode haver dois resduos iguais entre
r1 , r2 , r3 , . . . , rp1 .
Para provar isto, suponhamos que rk = r` para dois inteiros k e
`, ambos entre 1 e p 1. De acordo com a definio dos resduos,
teramos que
a k rk r` a ` (mod p);
isto ,
a k a ` (mod p).
Entretanto, como p no divide a e p primo, estes nmeros no tm
fator prprio comum. Mas isto implica que a inversvel mdulo p de
forma que, pelo teorema 2, podemos cancel-lo na congruncia acima,
obtendo
k ` (mod p).
Mas k e ` so inteiros positivos menores que p, e s podem ser congruentes se forem iguais. Logo,
se rk = r` , ento k = `.

i
i

cripto
2009/6/30
i page 137
Estilo OBMEP

i
i

N SEC. 5.2: O TEOREMA DE FERMAT

137

Isto nos mostra que


r1 , r2 , r3 , . . . , rp1
so p 1 resduos no nulos (pois p no divide a) e diferentes entre
si. Acontece que s h p 1 resduos no nulos diferentes mdulo p,
a saber
1, 2, 3, . . . , p 1;
o que nos permite deduzir que a sequncia de nmeros
r1 , r2 , r3 , . . . , rp1
apenas um embaralhamento de
1, 2, 3, . . . , p 1.
Em particular,
r1 r2 r3 rp1 = 1 2 3 (p 1).

Concluso geral: Da primeira maneira de calcular o produto dos


resduos temos que
r1 r2 r3 rp1 ap1 (1 2 3 (p 1)) (mod p)
e da segunda que
r1 r2 r3 rp1 = 1 2 3 (p 1).

i
i

cripto
2009/6/30
i page 138
Estilo OBMEP

i
i

138

CAP. 5: POTNCIAS

Portanto,
ap1 (1 2 3 (p 1)) 1 2 3 (p 1)

(mod p).

Contudo, 1 2 3 (p 1) produto de inversveis mdulo p logo


, ele prprio, inversvel mdulo p. Com isto podemos cancel-lo dos
dois lados da congruncia, o que nos d
ap1 1 (mod p);
que o que precisvamos mostrar.

Pelo Teorema de Fermat, se p primo, ento todo elemento de


resduo no nulo mdulo p tem uma potncia congruente a 1. Em
particular, qualquer um destes elementos admite uma ordem mdulo
p. Note, contudo, que no podemos afirmar que, como
bp1 1

(mod p) se b 6 0

(mod p),

ento b tem ordem p 1 mdulo p. Para comear, 1 tem ordem 1


mdulo p qualquer que seja o p que voc escolher. Alm disso, como
(p 1) 1 (mod p),
temos que
(p 1)2 (1)2 1 (mod p);
donde podemos concluir que p 1 tem ordem dois qualquer que seja o
p. Se estes exemplos ainda no lhe satisfazem, que tal este: de acordo

i
i

cripto
2009/6/30
i page 139
Estilo OBMEP

i
i

139

N SEC. 5.3: POTNCIAS

com o Teorema de Fermat,


230 1 (mod 31);
contudo, como vimos na seo 5.1, a ordem de 2 mdulo 31 5, e no
30. Para terminar de uma maneira mais positiva, aqui est um desafio
que mostra como a ordem de um inteiro mdulo p est relacionada
ao expoente p 1 do Teorema de Fermat.
Desafio 5. Seja p um primo positivo e b um inteiro que no divisvel
por p. Digamos que k a ordem de b mdulo p.
(a) Explique porque k p 1.
(b) Seja r o resto da diviso de p 1 por k. Mostre que, como
ap1 ak 1

(mod p),

ento ar 1 (mod p).


(c) Lembrando que 0 r k 1, mostre que r = 0.
(d) Conclua que a ordem de b um divisor de p 1.

5.3

Potncias

Agora que temos o Teorema de Fermat, podemos us-lo para simplificar o clculo de restos de potncias.

i
i

cripto
2009/6/30
i page 140
Estilo OBMEP

i
i

140

5.3.1

CAP. 5: POTNCIAS

Mdulos Primos

Comeamos reprisando o clculo do resto da diviso de 364 por 31,


que j fizemos na pgina 133. S que, desta vez, usaremos o Teorema
de Fermat. Como
330 1 (mod 31),
pelo Teorema de Fermat e 64 = 2 30 + 4, ento
364 (330 )2 34 1 81 19 (mod 31),
confirmando o resultado de nossos clculos anteriores de uma maneira
bem mais simples.
Exerccio 50. Calcule o resto da diviso de 398 745 por 43 usando o
Teorema de Fermat.
Vejamos outro exemplo, um pouco mais sutil. Digamos que quere2
mos calcular o resto da diviso de 31 034 por 1 033. A primeira coisa a
fazer verificar que 1 033 primo, pois s podemos aplicar o Teorema
de Fermat quando o mdulo primo. Como

1 033 = 32, 14 . . .
s precisamos mostrar que 1 033 no divisvel pelos primos menores
que 32 para ter certeza que primo. Estes primos so,
2, 3, 5, 7, 11, 13, 17, 19, 23, 29, 31,
e fcil verificar que nenhum deles divide 1 033. Agora que temos

i
i

cripto
2009/6/30
i page 141
Estilo OBMEP

i
i

141

N SEC. 5.3: POTNCIAS

certeza que 1 033 primo, podemos afirmar que


31 032 1 (mod 1 033)
pelo Teorema de Fermat. Em seguida precisamos dividir 1 0342 por
1 032. Dividir maneira de dizer, o que precisamos mesmo do resto
da diviso de 1 0342 por 1 032; o quociente no importa porque, por
Fermat, vai ser o expoente de 1. Com isso, podemos usar congruncias
para calcular o resto. Como 1 034 2 (mod 1 032), temos que
1 0342 22 4 (mod 1 032).
Logo o resto da diviso de 1 0342 por 1 032 4. Como no conhecemos
o quociente, vamos cham-lo de q. Mas, seja l qual for o valor de q,
temos que
1 0342 = 1 032 q + 4;
donde
2

31 034 31 032q+4 (31 032 )q 34

(mod 1 033)

aplicando o Teorema de Fermat, conclumos que


2

31 034 1 81 (mod 1 033);


2

de forma que 31 034 deixa resto 81 na diviso por 1 033.


2

Exerccio 51. Calcule o resto da diviso de 241 048 por 41 047.


Exerccio 52. Calcule o resto da diviso de 319! por 307.

i
i

cripto
2009/6/30
i page 142
Estilo OBMEP

i
i

142

CAP. 5: POTNCIAS

Exerccio 53. Calcule o resto da diviso de


1p1 + 2p1 + + (p 1)p1 por p,
sabendo-se apenas que p > 2 primo.
Desafio 6. Determine todos os primos positivos p para os quais a
equao
2x + xp + xp! 1 (mod p),
tem soluo x 6 0 (mod p).

5.3.2

Mdulos Compostos

Aparentemente a nica coisa que teramos a dizer sobre a aplicao do Teorema de Fermat ao clculo de potncias quando o mdulo composto seria isto no possvel ! O que faria desta a seo
mais curta da apostila. Contudo, podemos combinar o Teorema de
Fermat com o Algoritmo Chins do Resto e, com isso, simplificar
drasticamente as contas dos clculos com potncias em alguns casos
especiais, mesmo quando o mdulo composto.
Vejamos um exemplo numrico. Digamos que queremos calcular
o resto da diviso de 26 754 por 1 155. Fatorando 1 155 vemos que
igual a 3 5 7 11. Aplicando o Teorema de Fermat a cada um destes
primos, obtemos
22 1 (mod 3),
24 1 (mod 5),
26 1 (mod 7),
210 1 (mod 11).

i
i

cripto
2009/6/30
i page 143
Estilo OBMEP

i
i

143

N SEC. 5.3: POTNCIAS

A seguir dividimos 6 754 por p 1, para cada um dos fatores primos


p de 1 155,
6 754 = 2 3 377,
6 754 = 4 1 688 + 2,
6 754 = 6 1 125 + 4,
6 754 = 10 675 + 4.
Substituindo isto nas congruncias,
26 754 (22 )3 377

(mod 3),

26 754 (24 )1 688 22

(mod 5),

26 754 (26 )1 125 24

(mod 7),

26 754 (210 )675 24

(mod 11).

Mas aplicando o Teorema de Fermat, estas congruncias se reduzem a:

26 754 1
6 754

(mod 3),

2 4 (mod 5),

26 754 24 2 (mod 7),


26 754 24 5 (mod 11),

i
i

cripto
2009/6/30
i page 144
Estilo OBMEP

i
i

144

CAP. 5: POTNCIAS

logo,
26 754 1 (mod 3),
26 754 4 (mod 5),
26 754 2 (mod 7),
26 754 5 (mod 11).
Precisamos, portanto resolver o sistema
x 1 (mod 3),
x 4 (mod 5),
x 2 (mod 7),
x 5 (mod 11).
Usando o algoritmo chins, que foi descrito na seo 4.2 do Captulo 4, temos que x = 1 + 3y. Substituindo isto na segunda equao,
obtemos
1 + 3y 4 (mod 5),

isto ,

y 1 (mod 5),

j que 3 inversvel mdulo 5 e pode ser cancelado nos dois membros


da equao. Assim x = 4+15z. Substituindo isto na terceira equao
e resolvendo-a obtemos z 5 (mod 7); ou seja x = 79 + 105t. Finalmente substituindo isto na ltima equao, teremos t 6 (mod 11),
o que d x = 709 + 1 155u. Conclumos que 26 754 709 (mod 1 155).
Para realmente apreciar as vantagens deste mtodo, experimente refazer os clculos sem us-lo.

i
i

cripto
2009/6/30
i page 145
Estilo OBMEP

i
i

N SEC. 5.3: POTNCIAS

145

Exerccio 54. Calcule o resto da diviso de


(a) 2495 por 15 841;
(b) de 241 045 por 41 041;
(c) de 277 por 2 465.

i
i

cripto
2009/6/30
i page 146
Estilo OBMEP

i
i

Captulo 6

Criptografia RSA
chegada a hora de reunir tudo o que fizemos anteriormente, na
descrio do mtodo RSA. A descrio do RSA propriamente dita
consiste em explicitar as receitas usadas para codificao e decodificao de mensagens. Isto fcil de fazer, uma vez que depende apenas
do clculo dos resduos de potncias, assunto de que j tratamos com
detalhes anteriormente. Lembre-se, contudo, que decodificar significa
passar da mensagem codificada mensagem original. Por isso, nossa
misso neste captulo no se resume a descrever as receitas de codificao e decodificao; precisamos tambm verificar que se aplicadas
nesta ordem voltamos a obter mensagem original. Afinal, se isto
no fosse verdade, de que serviria este mtodo de criptografia?

146

i
i

cripto
2009/6/30
i page 147
Estilo OBMEP

i
i

147

N SEC. 6.1: PR-CODIFICAO

6.1

Pr-codificao

Como dissemos acima, o que fazemos para codificar uma mensagem no RSA calcular sua potncia mdulo n relativamente a
um expoente especialmente escolhido. Entretanto, para que isto seja
vivel, a mensagem deve ser um nmero inteiro. Mas no isto o
que ocorre em geral: a maior parte das mensagens um texto. Por
isso, a primeira coisa a fazer, se desejamos usar o mtodo RSA, inventar uma maneira de converter a mensagem em uma sequncia de
nmeros.
Suponhamos, para simplificar, que a mensagem original um texto
onde no h nmeros, apenas palavras, e no qual todas as letras so
maisculas. Portanto, em ltima anlise a mensagem constituda
pelas letras que formam as palavras e pelos espaos entre palavras.
Chamaremos esta primeira etapa de pr-codificao, para distingui-la
do processo de codificao propriamente dito.
Na pr-codificao convertemos as letras em nmeros usando a
seguinte tabela de converso:

10

11

12

13

14

15

16

17

18

19

20

21

22

23

24

25

26

27

28

29

30

31

32

33

34

35

O espao entre duas palavras ser substitudo pelo nmero 99, quando
for feita a converso. Por exemplo, a frase AMO A OBMEP con-

i
i

cripto
2009/6/30
i page 148
Estilo OBMEP

i
i

148

CAP. 6: CRIPTOGRAFIA RSA

vertida no nmero
1022249910992411221425.
Observe que precisamos fazer cada letra corresponder a um nmero
de, pelo menos, dois algarismos para evitar ambiguidades. Se fizssemos A corresponder ao nmero 1, B ao 2, e assim por diante, no
teramos como saber se 12 representa AB ou L, j que esta ltima
a dcima segunda letra do alfabeto.
Antes de continuar precisamos determinar os parmetros do sistema RSA que vamos usar. Estes parmetros so dois primos distintos, que vamos denotar por p e q, e cujo resto na diviso por 6 tem que
ser 5. A razo para esta estranha condio ser explicada na seo
6.3.
Em seguida, ponha n = pq. A ltima fase do processo de prcodificao consiste em quebrar em blocos o longo nmero produzido
anteriormente. Estes blocos devem ser nmeros menores que n. Por
exemplo, se escolhermos p = 17 e q = 23, ento n = 391. Neste caso, a mensagem, cuja converso numrica foi feita acima, pode ser
quebrada nos seguintes blocos:
102 224 99 109 92 41 122 142 5.
A maneira de escolher os blocos no nica e os blocos no precisam
sequer ter o mesmo tamanho. Contudo, certos cuidados devem ser
tomados. Por exemplo, no permitido escolher um bloco que comece
por 0 porque isto traria problemas na hora de decodificar, j que, por
exemplo, no temos como distinguir o bloco 071 do bloco 71.

i
i

cripto
2009/6/30
i page 149
Estilo OBMEP

i
i

N SEC. 6.2: CODIFICANDO E DECODIFICANDO UMA MENSAGEM

149

Observe que os blocos em que quebramos a mensagem no correspondem a nenhuma unidade lingustica, seja ela palavra, letra ou
qualquer outra. Isto muito bom, porque torna a decodificao por
contagem de frequncia essencialmente impossvel.

6.2

Codificando e Decodificando uma Mensagem

Encerramos assim a pr-codificao, e podemos passar etapa de


codificao propriamente dita. Para codificar a mensagem precisamos
apenas de n, que o produto dos primos. Diremos que n a chave de
codificao do sistema RSA que estamos usando. Esta chave pode ser
tornada pblica; isto , podemos envi-la a qualquer um que queira nos
mandar uma mensagem, sem preocupao de mant-la secreta. Por
isso a chave de codificao tambm conhecida como chave pblica
do sistema.
Supondo que j submetemos a mensagem pr-codificao, temos
uma sequncia de nmeros que, como na seo anterior, chamaremos
de blocos. Codificaremos cada bloco separadamente. A mensagem
codificada ser a sequncia dos blocos codificados. Isto muito importante porque depois de codificados os blocos no podem mais ser
reunidos de modo a formar um longo nmero. Se isto for feito, ser
impossvel decodificar a mensagem, como ficar claro na seo 6.3, na
qual discutimos o funcionamento do RSA.
Exerccio 55. Usando a tabela de primos da pgina 180, construa
uma chave pblica para voc utilizar na codificao de mensagens RSA
para seus colegas.

i
i

cripto
2009/6/30
i page 150
Estilo OBMEP

i
i

150

6.2.1

CAP. 6: CRIPTOGRAFIA RSA

Codificao

Digamos, ento, que a chave de codificao n. Como faremos


para codificar um bloco b? Lembre-se que b um inteiro positivo
menor que n. Vamos denotar o bloco codificado por C(b). A receita
para calcular C(b) a seguinte:
C(b) = resto da diviso de b3 por n.
Observe que, em termos de aritmtica modular, C(b) o resduo de
b3 mdulo n. Na verdade, como b > 0, o nmero C(b) mesmo o
resto da diviso de b3 por n.
Vejamos o que aconteceria no exemplo que estamos considerando.
Temos n = 391. Assim, o bloco 102 da mensagem anterior deve ser
codificado como o resto da diviso de 1023 por 391. Fazendo as contas,
obtemos C(102) = 34. claro que, para simplificar nosso trabalho,
executamos a conta calculando o resduo de 1023 mdulo 391:
1023 1022 102 238 102 24276 34 (mod 391).
Codificando toda a mensagem passo a passo, temos o seguinte:
2243 2242 224 128 224 129 (mod 391);
993 992 99 26 99 228
3

(mod 391);

109 109 109 151 109 37 (mod 391);


923 922 92 253 92 207 (mod 391);

i
i

cripto
2009/6/30
i page 151
Estilo OBMEP

i
i

N SEC. 6.2: CODIFICANDO E DECODIFICANDO UMA MENSAGEM

151

413 412 41 117 41 105 (mod 391);


1223 1222 122 26 122 44
1423 1422 142 223 142 386
3

5 5 5 25 5 125

(mod 391);
(mod 391);

(mod 391).

Reunindo todos os blocos, descobrimos que a mensagem codificada


34 129 228 37 207 105 44 386 125.
Exerccio 56. Use a chave pblica que voc construiu no exerccio
55 para codificar seu nome. Escreva a chave e a mensagem em um
papel. Os papis devero ser reunidos, embaralhados e sorteados entre
os alunos para o prximo exerccio.

6.2.2

Decodificao

Vejamos como fazer para decodificar um bloco da mensagem codificada. Em outras palavras, queremos saber qual a receita que
nos permite, de posse de um bloco codificado e da chave pblica,
reconstruir o bloco original, antes da codificao.
A informao que precisamos para poder decodificar consiste de
dois nmeros: n e o inverso d > 0 de 3 mdulo (p 1)(q 1). Pela
definio de inverso isto significa que devemos ter
3d 1 (mod (p 1)(q 1)).

i
i

cripto
2009/6/30
i page 152
Estilo OBMEP

i
i

152

CAP. 6: CRIPTOGRAFIA RSA

A explicao de onde saiu este nmero misterioso voc encontrar na


prxima seo. Chamaremos o par (n, d) de chave de decodificao.
Esta chave tem que ser mantida secreta. Quem a descobrir vai poder
decodificar qualquer mensagem endereada a voc.
De posse do par (n, d), como devemos proceder para decodificar
uma mensagem? Se a for um bloco codificado, denotaremos por D(a)
o resultado do processo de decodificao do bloco a. A receita para
calcular D(a) a seguinte:
D(a) = resto da diviso de ad por n.
Em termos de aritmtica modular, D(a) o resduo de ad mdulo
n. Como no caso da codificao, o bloco a positivo e este resduo
coincide com o resto da diviso de bd por n.
Note que, ao chamarmos o processo acima de decodificao, estamos assumindo um compromisso importante, que o de mostrar que
ao decodificar um bloco codificado, obtemos o bloco original. Dizendo
de outra maneira, se b um bloco da mensagem original, s ser legtimo chamar o processo acima de decodificao se
D(C(b)) = b.
No de forma alguma bvio que isto verdade: a demonstrao
de que esta igualdade realmente vlida dada em detalhes na
seo 6.3.
Alguns comentrios so necessrios antes de fazermos um exemplo.
Em primeiro lugar, muito fcil calcular d. Como estamos supondo

i
i

cripto
2009/6/30
i page 153
Estilo OBMEP

i
i

N SEC. 6.2: CODIFICANDO E DECODIFICANDO UMA MENSAGEM

153

que p e q deixam resto 5 na diviso por 6, temos que


p 5 (mod 6) e q 5 (mod 6).
Assim,
(p 1)(q 1) 4 4 16 4 2 (mod 6);
donde
(p 1)(q 1) = 6 k 2,
para algum inteiro positivo k. Contudo, como j vimos em (3.4.1), o
inverso de 3 mdulo 6 k 2 igual a 4 k 1. Logo, podemos tomar
d = 4 k 1.
No exemplo que vimos considerando p = 17 e q = 23, de forma
que
(p 1)(q 1) = 16 22 = 352 = 6 58 + 4
que igual a
(p 1)(q 1) = 6 59 2.
Portanto, neste caso, k = 59 e
d = 4 59 1 = 235.
Aplicando a receita dada anteriormente ao primeiro bloco da mensagem codificada, temos que D(34) igual ao resto da diviso de
34235 por n = 391.

i
i

cripto
2009/6/30
i page 154
Estilo OBMEP

i
i

154

CAP. 6: CRIPTOGRAFIA RSA

Efetuar esta conta sem um computador seria totalmente impossvel, se no tivssemos o algoritmo chins do resto e o Teorema de
Fermat. Aplicando o mtodo estudado na seo 5.3 do captulo 5, calculamos 34235 mdulo 17 e mdulo 23, que so os primos em que n
se fatora. Para comeo de conversa,
34 0 (mod 17),
34 11 (mod 23).
Assim,
34235 0235 0 (mod 17).
Aplicando o Teorema de Fermat outra congruncia,
11235 (1122 )10 1115 1115

(mod 23).

Mas,
11 12 4 3

(mod 23);

de forma que
11235 1115 415 315

(mod 23).

Contudo,
411 1 (mod 23),
311 1 (mod 23),

i
i

cripto
2009/6/30
i page 155
Estilo OBMEP

i
i

N SEC. 6.2: CODIFICANDO E DECODIFICANDO UMA MENSAGEM

155

de modo que
415 230 (211 )2 28 28 3

(mod 23),

315 311 34 34 12 (mod 23).


Donde podemos concluir que
11235 415 315 3 12 10 (mod 23).
Portanto,
34235 0 (mod 17),
34235 10 (mod 23).

Isto corresponde ao sistema


x 0 (mod 17),
x 10 (mod 23),

que podemos resolver utilizando o algoritmo chins do resto. Da segunda congruncia, obtemos
x = 10 + 23y
que, ao ser substitudo na primeira congruncia, nos d
10 + 23y 0 (mod 17).

i
i

cripto
2009/6/30
i page 156
Estilo OBMEP

i
i

156

CAP. 6: CRIPTOGRAFIA RSA

Assim,
6y 7 (mod 17).
Mas, 6 tem inverso 3 mdulo 17, de forma que
y 3 7 4 (mod 17).
Portanto,
x = 10 + 23y = 10 + 23 4 = 102;
como seria de esperar, afinal estamos decodificando 34, que corresponde codificao do bloco 102.
Exerccio 57. Decodifique os demais blocos da mensagem
34 129 228 37 105 44 386 125,
usando o procedimento acima.
Exerccio 58. Fatore a chave pblica que voc recebeu quando fez o
exerccio 56, calcule d e decodifique a mensagem para saber de quem
ela veio.

6.2.3

Segurana

Antes de prosseguir para a explicao de porque o RSA funciona,


conveniente discutir com um pouco mais de detalhes em que se
fundamenta a segurana do RSA. Neste contexto, o termo-chave
quebrar o cdigo. Digamos que algum, que vamos chamar de A, pe
uma escuta (tambm conhecida como um grampo) na linha que uma
empresa usa para transmitir mensagens codificadas a um banco. Se

i
i

cripto
2009/6/30
i page 157
Estilo OBMEP

i
i

N SEC. 6.2: CODIFICANDO E DECODIFICANDO UMA MENSAGEM

157

o cdigo utilizado for o RSA, ento A vai ter acesso no apenas s


mensagens codificadas que a empresa envia ao banco (obtidas pelo
grampo), mas tambm chave de codificao n usada pela empresa
que, afinal de contas, pblica.
Lembre-se que a chave n igual ao produto de dois nmeros primos p e q que foram escolhidos pela empresa no momento em que sua
implementao do RSA foi feita. Em princpio, A no deveria ter
nenhuma dificuldade em decodificar a mensagem. De posse de n,
precisaria apenas fator-lo, descobrir p e q e us-los para calcular
d. Uma vez obtido d, a receita de decodificao explicada em 6.2.2
pode ser aplicada para reconstituir a mensagem original.
Embora tudo isto parea muito simples em princpio, na prtica
totalmente invivel. A razo est em um problema de natureza tecnolgica: no existem computadores rpidos o suficiente, nem algoritmos bons o suficiente, que nos permitam fatorar um nmero inteiro
muito grande que no tenha fatores relativamente pequenos. Lembrese que, na seo 1.2.5 do captulo 1 mostramos que o tempo necessrio
para fatorar um nmero de uns cem algarismos pelo mtodo usual de
tentativa imenso, e excede, em muito, a idade estimada do universo.
Entretanto, a afirmao que acabamos de fazer muito mais forte:
no existe nenhum algoritmo conhecido capaz de fatorar
inteiros grandes de modo realmente eficiente.
Na verdade, no se sabe nem mesmo se possvel que um tal algoritmo
exista!
Mas, o que significa a palavra grande neste contexto? Mais precisamente, quo grande deve ser a chave n usada no RSA para que,

i
i

cripto
2009/6/30
i page 158
Estilo OBMEP

i
i

158

CAP. 6: CRIPTOGRAFIA RSA

mesmo tendo interceptado a mensagem codificada pela empresa e conhecendo n, o agente A no seja capaz de achar p e q e, assim, decodificar a mensagem? A resposta que, atualmente, as implementaes
comerciais do RSA usam chaves pblicas com cerca de 200 algarismos, mas algumas destas implementaes chegam a permitir chaves
pblicas com at 2 467 algarismos.
Durante algum tempo, o RSA Laboratory, que pertence empresa
que detm os direitos do sistema de codificao RSA, lanou desafios,
que consistiam de uma possvel chave pblica de RSA que deveria ser
fatorada. A ltima destas chaves a ser fatorada tem 193 algarismos e
corresponde ao produto dos primos
16347336458092538484431338838650908598417836700330
92312181110852389333100104508151212118167511579
e
1900871281664822113126851573935413975471896789968
515493666638539088027103802104498957191261465571.
A fatorao foi finalizada em novembro de 2005 por F. Bahr, M.
Boehm, J. Franke e T. Kleinjung no Escritrio Federal de Segurana
de Informao da Alemanha. Os clculos utilizaram 80 computadores
de 2.2 GHz cada um e, mesmo assim, foram necessrios 5 meses para
completar as contas! A maior das chaves proposta como desafio tem
617 algarismos e, evidentemente, est longe de ser fatorada. Mais
detalhes podem ser encontrados no verbete RSA numbers da verso
em ingls da Wikipedia.

i
i

cripto
2009/6/30
i page 159
Estilo OBMEP

i
i

N SEC. 6.2: CODIFICANDO E DECODIFICANDO UMA MENSAGEM

159

Na prtica, isto significa que se a empresa est usando uma implementao do RSA com chave pblica de uns 200 algarismos, ento
A no tem a menor chance de ler a mensagem. Outro detalhe prtico
importante que segue desta argumentao que a empresa precisa
calcular o valor de d a partir dos valores de p e q: se n for calculado e
p e q forem esquecidos, j no temos mais como determinar o valor de
n, porque ningum mais ser capaz de fatorar n. Portanto, primeiro
escolhem-se p e q, que so usados para calcular d; depois multiplicamse p e q para determinar n. Uma vez de posse do par (n, d) os valores
de p e q podem at ser apagados por medida de segurana.
Uma observao final. Quando usamos congruncias para efetuar
a codificao do bloco 102 na pgina 150, dissemos que estvamos
usando congruncias para facilitar as contas. Isto no estritamente
verdade, porque em uma aplicao comercial do RSA teramos que
calcular potncias de nmeros muito grandes, com mdulos maiores
ainda, e isto no vivel se no utilizarmos aritmtica modular. Em
outras palavras, no mera questo de facilitar nada, os clculos
seriam impossveis sem aritmtica modular. Para convenc-lo disto,
aqui vai um exemplo. Comecei escolhendo dois primos
p = 100000000000000000000000000000000000000000000000151
e
q = 100000000000000000000000000000000000000162735465691

i
i

cripto
2009/6/30
i page 160
Estilo OBMEP

i
i

160

CAP. 6: CRIPTOGRAFIA RSA

Calculei ento o quociente


(p 1)(q 1) + 2
= 166666666666666666666666666666666
6
66666693789244306666666666666666666666666666666666666
70735053308917
que me d o valor de k; donde
d = 4k1 = 6666666666666666666666666666666666666677515697722
666666666666666666666666666666666666682940213235667.
Com isto podemos codificar a mensagem AMO A OBMEP que, neste
caso, pode ser tomada como um nico bloco
m = 1022249910992411221425
j que este um nmero menor do que n = pq. O resduo de
10222499109924112214253 mdulo n

106824592360317689994495293731276889004322696993731601
3731140625,
que corresponde codificao C(m) da mensagem m. Se j difcil
imaginar o clculo do cubo de m mdulo n, o que dizer da decodificao de C(m), que consiste em elevar este nmero de 63 algarismos

i
i

cripto
2009/6/30
i page 161
Estilo OBMEP

i
i

N SEC. 6.3: POR QUE FUNCIONA?

161

a d, que um nmero de 99 algarismos. Na verdade, um computador


no consegue escrever todos os algarismos de C(m)d : h tantos deles que no cabem na memria de nenhum computador. No entanto,
usando congruncia mdulo n o meu computador consegue calcular o
resduo de C(m)d mdulo n em menos de um centsimo de segundo!
Custa-me crer que, tendo lido este ltimo exemplo, voc no esteja
perguntando:
Como ele fez para obter este nmeros primos enormes?
Esta uma tima pergunta, que fica melhor ainda se voc lembrar
que:
1. para saber se um nmero primo precisamos garantir que no
tem fatores prprios e que;
2. no existem meios rpidos para fatorar nmeros to grandes.
A concluso aparentemente inevitvel de (1) e (2) que deveria ser impossvel determinar com certeza se nmeros muito grandes so primos.
Curiosamente, a concluso falsa, muito embora tanto (1) quanto (2)
sejam verdadeiros. O fato, bastante surpreendente, que possvel
determinar que nmeros muito grandes so primos ou compostos sem
que haja necessidade de fator-los. Discutiremos isto com um pouco
mais de detalhes no prximo captulo.

6.3

Por que funciona?

Para que o procedimento exposto acima seja realmente til,


preciso que, ao decodificar uma mensagem, obtenhamos a mensagem

i
i

cripto
2009/6/30
i page 162
Estilo OBMEP

i
i

162

CAP. 6: CRIPTOGRAFIA RSA

original. Vimos nos exerccios 57 e 58 que, ao menos nestes exemplos, a decodificao reproduziu a mensagem original. Falta, apenas,
convencer-nos de que isto sempre ocorre.

6.3.1

Explicando o Funcionamento do RSA

Digamos que temos um sistema RSA de parmetros p e q, com


n = pq. Ento, para a codificao usamos a chave pblica n, e para
a decodificao o par (n, d), onde
(p 1) (q 1) = 6 k 2 e

d = 4 k 1.

Usando a notao das sees anteriores, precisamos verificar que, se


b um bloco da mensagem a ser codificada, isto um inteiro que
satisfaz 1 b n 1, ento DC(b) = b. Em outras palavras,
queremos mostrar que aplicando o processo de decodifio a um bloco
codificado, obtemos de volta o bloco correspondente da mensagem
original.
Na verdade, precisamos provar apenas que
DC(b) b (mod n).
Isto suficiente porque tanto DC(b) quanto b esto no intervalo que
vai de 1 a n 1, logo s podem ser congruentes mdulo n se forem
iguais.
Isto explica porque precisamos escolher b menor que n e porque
temos que manter os blocos separados, mesmo depois da codificao.
Se no tomssemos estes cuidados, continuaramos obtendo blocos

i
i

cripto
2009/6/30
i page 163
Estilo OBMEP

i
i

163

N SEC. 6.3: POR QUE FUNCIONA?

congruentes depois da decodificao, mas eles no seriam necessariamente iguais. Em outras palavras, no teramos de volta a mensagem
original o que, convenhamos, no seria muito satisfatrio.
Vamos ao argumento. Recapitulando, o que queremos mostrar
a congruncia
DC(b) b (mod n).
Mas, pela definio de D e de C temos que
C(b) b3

(mod n);

C(a) ad

(mod n).

e que

Combinando estas duas congruncias, obtemos


DC(b) D(b3 ) b3d

(mod n).

(6.3.1)

Queremos, portanto, mostrar que b3d b (mod n). Mas, por definio,
3d 1 (mod (p 1)(q 1)),
donde
3d = 1 + k(p 1)(q 1).

(6.3.2)

Lembrando que n = pq, onde p e q so primos distintos, calcularemos os resduos de b3d mdulo p e mdulo q e usaremos o teorema
chins do resto para construir, a partir deles, o resduo mdulo n.
Como os clculos dos resduos so anlogos para ambos os primos,
basta mostrar como executar um deles. Digamos que queremos achar

i
i

cripto
2009/6/30
i page 164
Estilo OBMEP

i
i

164

CAP. 6: CRIPTOGRAFIA RSA

o resduo de b3d mdulo p. Levando em conta a expresso para 3d


obtida em (6.3.2), temos que
b3d b (bp1 )k(q1)

(mod p).

Em seguida queremos usar o Teorema de Fermat, mas para isto precisamos saber que p no divide b. Se isto for verdade, ento
bp1 1 (mod p)
por Fermat, e obtemos
b3d b (1)k(q1) b (mod p)
mostrando o que queramos. Por outro lado, se p dividir b, ento tanto b quanto b3d so congruentes a zero mdulo n. Logo, tambm neste
caso, b3d b (mod p). Resumindo, no importa qual seja o inteiro b,
sempre temos que
b3d b (mod p).
Fazendo um argumento anlogo para o primo q, obtemos o par de
congruncias
b3d b (mod p),
3d

(6.3.3)

b (mod q).

i
i

cripto
2009/6/30
i page 165
Estilo OBMEP

i
i

N SEC. 6.3: POR QUE FUNCIONA?

165

Observe que b uma soluo de


x b (mod p),
x b (mod q);
de modo que, pelo teorema chins do resto, este sistema tem soluo
geral igual a
b + p q t,
onde t Z. Logo b3d que, por (6.3.3) tambm soluo do mesmo
sistema, tem que satisfazer
b3d = b + p q k,
para algum inteiro k. Mas isto equivalente a
b3d b (mod pq);
que a congruncia que desejvamos provar.
Exerccio 59. Discuta em grupo os seguintes problemas relativos
segurana do RSA:
(a) se as chaves pblicas de duas pessoas diferentes tm um primo
em comum, ento fcil quebrar o RSA destas duas pessoas;
(b) se usamos o RSA, mas codificamos a mensagem partindoa em blocos que consistem de uma nica letra, ento fcil
decodificar a mensagem, embora o cdigo no seja quebrado.
Um problema semelhante, porm mais difcil, proposto no seguinte
desafio.

i
i

cripto
2009/6/30
i page 166
Estilo OBMEP

i
i

166

CAP. 6: CRIPTOGRAFIA RSA

Desafio 7. Sabemos que se n a chave pblica de uma implementao


do RSA, ento n = pq, onde p e q so primos positivos distintos.
Imagine que algum lhe emprestou um computador (que voc no tem
a menor ideia de como funciona) que, ao receber a chave pblica n
calcula o nmero m = (p1)(q 1). Mostre que possvel determinar
p e q a partir de n e m.

6.3.2

Comentrio

Se voc leu o argumento usado para provar que o RSA funciona


corretamente em detalhes e com bastante senso crtico, pode estar
perguntando:
Onde usamos o fato dos primos terem que deixar resduo
5 mdulo 6?
A resposta que isto s necessrio para garantir que 3 inversvel
mdulo (p 1)(q 1). Como a demonstrao toda depende disto,
a hiptese parece realmente essencial. Mas no . O fato que o
RSA pode ser implementado usando quaisquer dois expoentes inteiros
positivos, e para codificao e d para decodificao, desde que
ed 1

(mod (p 1)(q 1)).

A demonstrao de que o sistema se comporta da maneira desejada


para tais expoentes essencialmente a mesma que foi dada acima.
Ento, por que estamos nos limitando ao caso em que o expoente
de codificao e igual a 3? A resposta que, com isto, fcil
determinar d. Para que pudssemos permitir expoentes mais gerais,

i
i

cripto
2009/6/30
i page 167
Estilo OBMEP

i
i

N SEC. 6.3: POR QUE FUNCIONA?

167

precisaramos de um outro algoritmo que nos permitisse determinar


o inverso de um dado nmero mdulo (p 1)(q 1), quando este
inverso existe. Este algoritmo existe e bem conhecido, trata-se de
uma extenso do algoritmo euclidiano que utilizado para calcular
o mximo divisor comum de dois nmeros. Mais detalhes sobre este
algoritmo podem ser encontrados no captulo 1 da referncia [2].

i
i

cripto
2009/6/30
i page 168
Estilo OBMEP

i
i

Captulo 7

Encontrando Primos
Neste captulo veremos como encontrar primos para utilizar no
RSA. Como o conhecimento dos fatores primos p e q de n permite
a qualquer um descobrir uma mensagem codificada usando n como
chave pblica, os primos p e q precisam ser muito grandes. Por isto
o problema que desejamos resolver neste captulo pode ser mais precisamente formulado pela pergunta:
Como achar primos grandes cujo resto na diviso
por 6 5?
Responderemos a esta pergunta:
1. provando que existem infinitos primos cujo resto na diviso por
6 5;
2. descrevendo um procedimento pelo qual podemos encontrar todos os primos deste tipo menores que um dado inteiro t.
168

i
i

cripto
2009/6/30
i page 169
Estilo OBMEP

i
i

N SEC. 7.1: INFINIDADE DOS PRIMOS

169

Na prtica a resposta a (2) no satisfatria, porque no vivel


encontrar todos os primos que tenham menos dos que 50 algarismos.
H muitos deles, e o tempo necessrio para determinar todos seria
longo demais. Para achar um primo de 50 algarismos precisamos de
um procedimento que nos leve diretamente a ele, sem ter que achar
todos os primos intermedirios. Isto possvel mas est alm das possibilidades desta apostila; para mais detalhes consulte o captulo 6 da
referncia [2], por exemplo. Entretanto, para no deixar o problema
inteiramente sem resposta analisaremos na seo 7.3 um teste que
permite identificar que certos nmeros so compostos sem precisar
fator-los.

7.1

Infinidade dos Primos

Comearemos discutindo o argumento, dado por Euclides em seus


Elementos, que mostra que existem infinitos nmeros primos.

7.1.1

Infinitos Primos

Na verdade o que mostraremos que, dado um conjunto finito


qualquer P de primos, tem que existir um primo fora de P.
Digamos que
P = {p1 , . . . , ps },
um conjunto finito formado apenas por nmeros primos e consideremos o nmero
N = p1 ps ,

i
i

cripto
2009/6/30
i page 170
Estilo OBMEP

i
i

170

CAP. 7: ENCONTRANDO PRIMOS

que igual ao produto de todos os primos em P. Como N e N + 1


no podem ter nenhum fator prprio comum (veja Exerccio 3), um
primo que divide N no pode dividir N + 1. Mas, todos os primos em
P dividem N ; logo nenhum primo em P pode dividir N + 1. Contudo,
pelo Teorema da Fatorao nica o nmero inteiro N + 1 tem que ter
algum fator primo. Como estes fatores no podem dividir N , ento
so primos que no pertencem a P, provando assim o que queramos.
O resultado que acabamos de mostrar importante o suficiente
para ser enunciado como um teorema.
Teorema 4. Se P um conjunto finito de nmeros primos, ento
existe um primo que no pertence a P.
Observe que isto basta para garantir que o conjunto de todos os
primos no pode ser finito. Afinal, dado um conjunto finito qualquer
de primos, mostramos que sempre h um primo fora deste conjunto.
Teorema de Euclides. Existem infinitos nmeros primos.
Curiosamente, apesar do resultado acima ser frequentemente
atribudo a Euclides, o enunciado que aparece nos Elementos mais
parecido com o Teorema 4. O que Euclides diz, em uma traduo quase literal do grego :
h mais nmeros primos do que qualquer quantidade proposta de primos.
Um erro que muita gente comete ao ler a demonstrao do teorema
4 consiste em achar que o argumento mostra que o nmero N + 1
primo. Em outras palavras, multiplicando uma quantidade finita de

i
i

cripto
2009/6/30
i page 171
Estilo OBMEP

i
i

N SEC. 7.1: INFINIDADE DOS PRIMOS

171

primos e somando um obtemos um primo. Isto no verdade, como


voc vai verificar no prximo exerccio.
Exerccio 60. Se p for um primo, denote por p# o produto de todos
os primos positivos menores ou iguais que p. Por exemplo,
11# = 2 3 5 7 11.
Chamamos p# de primorial de p, porque sua definio parece com
a do fatorial. Determine o menor valor de p para o qual p# + 1
composto.
Na verdade, h apenas 22 primos p para os quais p# + 1 tambm
primo. O maior deles 392 113; cujo primorial 392 113# o produto
de 33 237 primos e d lugar a um primo 392 113# + 1 de 169 966
algarismos.
Quase tudo o que fizemos acima relativamente a p# +1 tambm se
aplica a p# 1, como voc chamado a mostrar no prximo exerccio.
Exerccio 61. Seja p um primo.
(a) Mostre que p# e p# 1 no tm fatores comuns.
(b) Use (a) para mostrar que existem infinitos nmeros primos.
(c) Determine o menor primo mpar p para o qual p# 1 composto.
So conhecidos apenas 18 primos da forma p# 1, o maior deles
15 877# 1 que tem 6 845 algarismos.

i
i

cripto
2009/6/30
i page 172
Estilo OBMEP

i
i

172

CAP. 7: ENCONTRANDO PRIMOS

7.1.2

Primos da Forma 6k+5

Embora o que acabamos de fazer seja muito interessante, no


podemos esquecer que no era bem isto que queramos provar, mas
sim que existem infinitos primos cujo resto na diviso por 6 5. Vejamos se o que j mostramos basta para provar esta afirmao mais
restrita.
Em primeiro lugar, qualquer inteiro na diviso por 6 tem que
deixar como resto um nmero entre 0 e 5. Mas, se p for primo ento
as possibilidades de restos so mais restritas. De fato, se o resto for
0, o nmero divisvel por 6. Por outro lado, se o resto for 2 ou 4,
ento o nmero par, logo divisvel por 2; ao passo que se o resto for
3, o nmero divisvel por 3. Portanto,
se p for primo s pode deixar resto 1 ou resto 5 quando
dividido por 6.
Isto, infelizmente no bom para o nosso argumento. Embora seja
fcil produzir exemplos de primos que deixam resto 5 na diviso por
6 (como 5 e 17), talvez s haja uma quantidade finita destes primos,
ao passo que os que deixam resto 1 na diviso por 6 so infinitos.
Apesar de no ser verdade, isto compatvel com o fato de existirem
infinitos primos.
Isto no esgota nossa caixa de ferramentas, porque ainda podemos
pensar em usar o teorema 4 diretamente, em vez de apelar para o Teorema de Euclides, que apenas uma de suas possveis consequncias.
Mais precisamente, queremos mostrar que
se P for um conjunto finito de primos da forma 6k + 5

i
i

cripto
2009/6/30
i page 173
Estilo OBMEP

i
i

N SEC. 7.1: INFINIDADE DOS PRIMOS

173

ento existe um primo da mesma forma que no pertence


a P.
O problema que o teorema 4 nos diz apenas que existe algum primo
fora de P e, como j vimos, h primos que no so da forma 6k + 5.
Em princpio, poderamos tentar refinar nossa anlise repetindo a
demonstrao do teorema 4 neste caso especial para ver se continua
funcionando. Para isso, suponhamos que
P = {p1 , . . . , ps },
um conjunto finito formado apenas por nmeros primos da forma
6k + 5 e consideraremos o nmero
N = p1 ps ,
que igual ao produto de todos os primos em P. Como antes,
N e N + 1 no tm fator prprio comum,
s que, como N um nmero mpar, N +1 tem que ser par. Portanto,
N + 1 admite 2 como fator e no chegamos a nenhuma contradio
porque, por exemplo, isto compatvel com N + 1 ser uma potncia
de 2.
Para sair desta enrascada precisaremos de um argumento muito
mais delicado. Ao invs de considerar simplesmente N + 1, escolheremos trabalhar com 6N +5, porque este ltimo nmero deixa resto 5 na
diviso por 6, como era o caso dos primos com os quais comeamos.
Infelizmente, 5 pertence ao conjunto P, de modo que um divisor

i
i

cripto
2009/6/30
i page 174
Estilo OBMEP

i
i

174

CAP. 7: ENCONTRANDO PRIMOS

comum entre N e 6N + 5, de modo que


mdc(N, 6N + 5) 6= 1.
Mas isto no razo para desanimarmos. Depois de pensar um pouco,
vemos que esta dificuldade pode ser contornada se exclurmos 5 do
produto que define N . Para deixar o argumento mais claro, redefiniremos todos os dados iniciais.
Suponha, ento, que
P = {5, p1 , . . . , ps },
seja o conjunto finito formado por todos os nmeros primos que
deixam resto 5 na diviso por 6. Considere o nmero
N = p1 ps .
Observe que deixamos o 5 fora deste produto. Pelo teorema da fatorao nica, podemos escrever 6N + 5 na forma
em
6N + 5 = q1e1 qm
,

(7.1.1)

em que os qs so primos positivos e os es so inteiros positivos. Como


6N + 5 mpar, todos os seus fatores tm que ser mpares. Portanto,
os q so mpares e tm que deixar resto 1 ou 5 na diviso por 6.
Digamos, por um momento, que todos os qs deixassem resto um na
diviso por 6. Neste caso teramos que
em
q1e1 qm
1 (mod 6).

i
i

cripto
2009/6/30
i page 175
Estilo OBMEP

i
i

N SEC. 7.1: INFINIDADE DOS PRIMOS

175

Mas isto impossvel, porque pela igualdade (7.1.1),


em
q1e1 qm
6N + 5 5 (mod 6),

e 1 e 5 no so congruentes mdulo 6. Isto nos permite concluir que


6N + 5 tem que ter pelo menos um fator primo que deixa
resto 5 na diviso por 6.
Acontece que P , por hiptese, a lista completa dos primos da forma
6k + 5. Logo, 6N + 5 tem que ser divisvel por algum elemento de P.
Isto possvel?
Vejamos. Para comear, se 5 dividisse 6N + 5 ento teria que
dividir
(6N + 5) 5 = 6N = 2 3 p1 ps ,
o que no possvel, j que 5 no aparece entre os primos nesta
fatorao. Por outro lado, se 6N + 5 fosse divisvel por um dos primos
que dividem N , ento
(6N + 5) 6N = 5,
teria que ser divisvel pelo mesmo primo, o que tambm no possvel.
Isto mostra que 6N + 5 no pode ser divisvel por nenhum elemento
de P, e nos d a contradio desejada. Disto segue imediatamente
que h uma quantidade infinita de primos da forma 6k + 5, como
espervamos mostrar.
Desafio 8. O objetivo deste desafio dar uma demonstrao de que
existem infinitos nmeros primos da forma 4n + 3.

i
i

cripto
2009/6/30
i page 176
Estilo OBMEP

i
i

176

CAP. 7: ENCONTRANDO PRIMOS

(a) Mostre que todo nmero primo mpar tem resduo 1 ou 3


mdulo 4.
(b) D exemplos de cinco nmeros primos que tm resduo 1
mdulo 4 e cinco que tm resduo 3 mdulo 4.
(c) Mostre que o produto de dois nmeros inteiros da forma
4n + 1 da forma 4n + 1.
(d) O produto de dois nmeros da forma 4n + 3 da
forma 4n + 3?
(e) Suponha que 3 < p1 < < pk sejam primos da forma
4n + 3. Usando (c), verifique que 4(p1 . . . pk ) + 3 tem que ser
divisvel por um primo da forma 4n + 3 que no pertence ao
conjunto {3, p1 , . . . , pk }.
(f ) Use (e) para mostrar que existem infinitos nmeros primos
da forma 4n + 3.

7.2

Encontrando os Primos

Agora que sabemos que h uma infinidade de primos da forma


6k + 5, s nos resta explicar como se deve proceder para encontrar
primos cada vez maiores que so desta forma. Como na seo anterior,
comearemos tratando dos primos em geral; s depois veremos o que
acontece se nos restringimos apenas aos primos da forma 6k + 5.

i
i

cripto
2009/6/30
i page 177
Estilo OBMEP

i
i

N SEC. 7.2: ENCONTRANDO OS PRIMOS

7.2.1

177

O Crivo de Eratstenes

Descreveremos aqui o mais antigo dos mtodos para achar primos,


conhecido como crivo de Eratstenes. Como no podia deixar de ser,
Eratstenes foi um matemtico grego, e nasceu por volta de 284 a.C.
Apesar de sua proficincia em muitos dos ramos de conhecimento, os
contemporneos de Eratstenes julgavam que no havia chegado
perfeio em nenhum. Por isso era chamado de Beta (segunda letra
no alfabeto grego) e Pentatlos. Competitivos, esses gregos, no?
Antes de mais nada, voc precisa saber que um crivo uma
peneira. Nicmaco em sua Aritmtica, publicada por volta do ano
100 d.C., introduz o crivo de Eratstenes da seguinte maneira:
o mtodo para obt-los [os nmeros primos] chamado
por Eratstenes uma peneira, porque tomamos os nmeros
mpares misturados de maneira indiscriminada e, por este
mtodo, como se fosse pelo uso de um instrumento ou
peneira, separamos os primos ou indecomponveis dos secundrios ou compostos.
Portanto o crivo atua como uma peneira que s deixa passar os
nmeros primos. Vejamos como funciona.
Em primeiro lugar o crivo determina todos os primos at um certo
inteiro positivo n previamente escolhido. Para realizar o crivo com
lpis e papel podemos proceder da seguinte maneira. Listamos os mpares de 3 a n. claro que s listamos os mpares porque 2 o nico
primo par. Comeamos ento a operar com o crivo propriamente dito.
O primeiro nmero da nossa lista 3; riscamos os demais nmeros da
lista, de 3 em 3. Assim sero riscados todos os mltiplos de 3 maiores

i
i

cripto
2009/6/30
i page 178
Estilo OBMEP

i
i

178

CAP. 7: ENCONTRANDO PRIMOS

que ele prprio. Em seguida procuramos o menor elemento da lista,


maior que 3, que no tenha sido riscado, que 5. Riscamos os demais
nmeros da lista, de 5 em 5. Assim sero riscados todos os mltiplos
de 5 maiores que ele prprio. E assim por diante, at chegar a n.
Por exemplo, se n = 60, a lista de nmeros
21
41

3
23
43

5
25
45

7
27
47

9
29
49

11
31
51

13
33
53

15
35
55

17
37
57

19
39
59

Ao final da primeira passagem do crivo (de 3 em 3), ficamos com


6 21
41

63
23
43

5
25
6 45

7
6 27
47

69
29
49

11
31
6 51

13
6 33
53

6 15
35
55

17
37
6 57

19
6 39
59

Ao final da segunda passagem do crivo (de 5 em 5) a lista


6 21
41

63
23
43

65
6 25
6 45

7
6 27
47

69
29
49

11
31
6 51

13
6 33
53

6 15
6 35
6 55

17
37
6 57

19
6 39
59

Ao final da terceira passagem do crivo (de 7 em 7), a lista se torna

6 21
41

63
23
43

65
6 25
6 45

7
6 27
47

69
29
6 49

11
31
6 51

13
6 33
53

6 15
6 35
6 55

17
37
6 57

19
6 39
59

Ao final da quarta passagem do crivo (de 11 em 11), a lista continua a mesma acima. A quinta passagem seria de 13 em 13, mas
novamente nada vai mudar na lista. Na verdade nenhuma passagem

i
i

cripto
2009/6/30
i page 179
Estilo OBMEP

i
i

N SEC. 7.2: ENCONTRANDO OS PRIMOS

179

posterior do crivo vai eliminar nenhum nmero adicional. Logo os


primos mpares positivos menores que 35 so
3, 5, 7, 11, 13, 17, 19, 23, 29, 31, 37, 41, 43, 47, 53 e 59.
Este exemplo nos leva a observar algumas caractersticas do crivo.
Em primeiro lugar, alguns nmeros so riscados da lista mais de uma
vez. o caso de 15 que j havia sido riscado na primeira passagem
(3 em 3), e foi riscado tambm na segunda (5 em 5). Em segundo
lugar, j havamos riscado da lista todos os nmeros compostos na
terceira passagem do crivo. Todas as passagens posteriores foram
redundantes.
Consideremos a segunda observao. Ela indica que deve ser possvel parar de riscar os nmeros muito antes de chegar a n. De fato, se
m um inteiro da lista, ento m n. Se m for composto, ento ter

um fator menor ou igual a m pela proposio 1. Mas m n.


Isto , qualquer nmero composto da lista tem um fator menor ou

igual a n. Desta forma no precisamos riscar nmeros de r em r

quando r > [ n]. No exemplo acima [ 60] = 7; por isso suficiente


riscar de 3 em 3, de 5 em 5, de 7 em 7 e nada mais.
A outra observao mais delicada. Infelizmente no possvel
evitar completamente o fato de que alguns nmeros sero riscados
vrias vezes. Mas podemos melhorar um pouco o crivo acima. Digamos que queremos achar os primos at n, e que estamos prestes
a riscar os nmeros de p em p para algum primo p. claro que os
mltiplos de p que tambm so mltiplos de primos menores que p j
foram riscados da lista. Portanto, nesta etapa, podemos comear a

i
i

cripto
2009/6/30
i page 180
Estilo OBMEP

i
i

180

CAP. 7: ENCONTRANDO PRIMOS

riscar de p em p a partir do menor mltiplo de p, que no mltiplo


de um primo menor que p. Mas os mltiplos positivos de p so da
forma
k p para algum inteiro k 0,
e se k < p, o inteiro k p tambm mltiplo do nmero k que menor
que p. Logo, o menor mltiplo de p, que no mltiplo de um primo
menor que p p2 . Resumindo:
podemos riscar de p em p a comear de p2 .
Isto evita algumas duplicaes e torna o crivo um pouco mais econmico.

7.2.2

Primos da Forma 6k + 5

Uma maneira de determinar os primos menores que um inteiro


positivo n e que so da forma 6k + 5 listar todos os primos at n
usando o crivo e testar para ver quais deixam resto 5 quando dividimos por 6. Fazendo isto lista de primos menores que 60 obtida
anteriormente, sobram apenas
5, 11, 17, 23, 29, 41, 47, 53 e 59.
O problema desta estratgia que muito ineficiente. Digamos,
por exemplo, que queremos encontrar todos os primos que deixam
resto 5 na diviso por 6 e que so menores ou iguais a 1 000. Utilizando
o crivo de Eratstenes na forma apresentada anteriormente, teramos
que gerar uma lista de 1 000/2 = 500 nmeros mpares para riscar.
Contudo, somente um em cada seis elementos da lista deixa resto 5

i
i

cripto
2009/6/30
i page 181
Estilo OBMEP

i
i

N SEC. 7.2: ENCONTRANDO OS PRIMOS

181

na diviso por 6. Como


1 000 = 6 166 + 4;
isto significa que bastaria procurar pelos primos que realmente nos
interessam entre 166 nmeros: aqueles que deixam resto 5 quando
divididos por 6. Mas esta uma lista muito menor e mais fcil de manipular que a do crivo de Eratstenes. A questo :
Podemos continuar riscando de p em p para determinar se
um nmero mltiplo de p, mesmo com a lista reduzida
somente aos nmeros da forma 6k + 5?
Uma observao, antes de continuarmos, para o caso de voc ter
pensado:
Mas para que se preocupar com isto se posso verificar se
o nmero mltiplo de p simplesmente dividindo-o por p
e vendo se o resto zero?
De fato isto pode ser feito mas, para nmeros mais ou menos grandes,
muito mais trabalhoso do que contar de p em p. E isto continua
sendo verdadeiro mesmo se usarmos um computador para fazer o
risca-risca.
Para que o crivo possa restringir-se apenas aos nmeros da forma
de resduo 5 mdulo 6, precisamos mostrar duas coisas. A primeira
que
(1) todo nmero composto que tem resduo 5 mdulo 6 admite um
fator do mesmo tipo.

i
i

cripto
2009/6/30
i page 182
Estilo OBMEP

i
i

182

CAP. 7: ENCONTRANDO PRIMOS

Do contrrio alguns compostos no seriam riscados j que o riscarisca de p em p est agora limitado aos nmero que deixam resto 5
na diviso por 6. Que (1) na verdade consequncia de resultados
que j vimos antes. De fato, como vimos em 2.2.3, qualquer nmero
primo deixa resto 1 ou 5 quando dividido por 6. Porm, se todos os
fatores primos de um nmero n deixarem resto 1 na diviso por 6,
teremos n 1 (mod 6), de modo que n ter necessariamente resto 1
na diviso por 6. Isto nos permite concluir que,
se um inteiro positivo deixa resto 5 quando dividido por
6 ento pelo menos um dos seus fatores primos deixa o
mesmo resto quando dividido por 6.
A segunda coisa que precisamos mostrar que
(2) todos os mltiplos de p que aparecem na tabela continuam espaados de p em p.
Afinal de contas, removemos 5/6 dos nmeros da tabela, o que alterou
completamente a posio de cada um deles em relao aos outros,
pondo em risco nossa capacidade de detectar mltiplos apenas por
manterem um espaamento constante.
Talvez este ltimo ponto precise ser um pouco melhor elaborado.
Considere, por exemplo, a lista dos mpares at 30:
1

11

13

15

17

19

21

23

25

27

29

Nesta tabela os mltiplos de 3 aparecem a cada 3 casas e os mltiplos


de 5 a cada 5 casas. Removendo os mltiplos de 3, obtemos a seguinte
tabela mais curta:

i
i

cripto
2009/6/30
i page 183
Estilo OBMEP

i
i

183

N SEC. 7.2: ENCONTRANDO OS PRIMOS

11

13

17

19

23

25

29

Note que j no mais verdade que os mltiplos de 5 aparecem a


cada 5 casas. A remoo dos mltiplos de 3, alguns dos quais tambm
so mltiplos de 5, alterou a posio dos nmeros uns em relao aos
outros e destruiu o fato dos mltiplos de um mesmo nmero manterem
uma distncia fixa uns dos outros.
Para mostrar (2), comearemos por tentar identificar qual a
forma de um inteiro que deixa resto 5 na diviso por 6 e que divisvel
por p. Chamando este inteiro de x, estas condies se traduzem no
sistema de congruncias
x5

(mod 6),

x 0 (mod p).
Para resolver o sistema pelo algoritmo chins do resto, tomamos
x = yp da segunda congruncia e substitumos na primeira, obtendo
yp 5 (mod 6).

(7.2.1)

Como s vamos riscar usando primos cujo resduo mdulo 6 igual a


5, temos que
p 5 (mod 6);
de forma que (7.2.1) se torna
y5 5 (mod 6);

i
i

cripto
2009/6/30
i page 184
Estilo OBMEP

i
i

184

CAP. 7: ENCONTRANDO PRIMOS

donde,
y 1 (mod 6).
Assim, y = 1 + 6r e, portanto,
x = p + 6rp.
Em outras palavras, mostramos que todos os mltiplos de p em uma
tabela que s contm nmeros de resduo 5 mdulo 6 so da forma
p + 6rp, o que nos permite concluir que
para irmos de um mltiplo de p que deixa resto 5 na diviso por 6 ao seguinte basta somar 6p a este nmero.
Como os nmeros em nossa tabela j esto espaados de seis em seis
(pois so da forma 6k+5), se pularmos de um mltiplo de p ao seguinte
na tabela teremos dois nmeros cuja diferena 6p. Pela concluso
enunciada acima estes so precisamente dois mltiplos de p da forma
6k + 5 consecutivos, o que prova (2).
Antes de executarmos o crivo restrito a uma tabela que s contenha os nmeros que deixam resto 5 na diviso por 6, h um detalhe
que precisamos considerar. Na verso original do crivo, vimos que
qualquer nmero composto admite um fator primo menor
ou igual que sua raiz quadrada.
Por outro lado, de acordo com (1),
se um inteiro positivo que deixa resto 5 quando dividido
por 6 ento pelo menos dos seus fatores primos deixa o
mesmo resto quando dividido por 6.

i
i

cripto
2009/6/30
i page 185
Estilo OBMEP

i
i

185

N SEC. 7.2: ENCONTRANDO OS PRIMOS

Infelizmente, no podemos combinar estas duas afirmaes e deduzir


que um inteiro positivo deixa resto 5 quando dividido por 6 tem que
ter um fator primo menor ou igual sua raiz quadrada que satisfaa a
mesma propriedade. S poderamos chegar a esta concluso se todos
os fatores primos de um nmero cujo resduo mdulo 6 5 fossem
do mesmo tipo; mas isto falso. Por exemplo, 161 deixa resto 5 na
diviso por 6 e tem dois fatores, 7 e 23, dos quais somente 23 deixa
resto 5 na diviso por 6; entretanto,
23 >

161 = 12, 68 . . .

de modo que 161 no tem nenhum fator menor que sua raiz quadrada
que deixe resto 5 na diviso por 6. Do ponto de vista prtico isto
significa que, ao contrrio do que fizemos na verso usual do crivo de
Eratstenes,

no podemos parar de riscar quando p > n se restringirmos nossa tabela apenas aos nmeros da forma 6k + 5.
Passando ao exemplo, usaremos a estratgia desenvolvida acima
para determinar os primos da forma 6k + 5 que so menores que 60.
Os nmeros da forma 6k + 5 menores que 60 so
5

11

17

23

29

35

41

47

53

59

Como 5 o primeiro nmero, comeamos riscando de cinco em cinco:


5

11

17

23

29

6 35

41

47

53

59

Em seguida, riscaramos de 11 em 11, acontece que a tabela s tem


10 casas: o dcimo primeiro nmero a partir de 11 j est fora da

i
i

cripto
2009/6/30
i page 186
Estilo OBMEP

i
i

186

CAP. 7: ENCONTRANDO PRIMOS

tabela. claro que se isto aconteceu com 11, tambm acontecer


com qualquer outro nmero maior que 11. Portanto, tendo riscado de
5 em 5, j obtivemos os primos desejados, que so
5, 11, 17, 23, 29, 41, 47, 53 e 59.
Antes que voc fique animado demais talvez seja melhor previni-lo
de que as coisas no so assim to boas quando o limite superior da
tabela um nmero muito grande.
Exerccio 62. Use esta verso especial do crivo de Eratstenes para
determinar todos os primos da forma 6k + 5 menores que 500.

7.3

Um Teste de Composio

Como dissemos na introduo deste captulo, o crivo de Eratstenes


no uma maneira eficiente de achar primos realmente grandes, como
os que apareceram ao final da seo 6.2 do captulo 6. O problema
que o intervalo ao qual o crivo est sendo aplicado grande demais, o que o tornar muito lento e far com que ocupe um enorme
espao na memria do computador, mais at do que teramos nossa
disposio! Para contornar o problema, os matemticos criaram os
chamados testes de primalidade: critrios que permitem determinar
com segurana que um dado nmero primo. Estes testes procedem
de maneira indireta; ao invs de tentar fatorar o nmero, calculam
apenas certas potncias modulares, por isso tm execuo bastante
rpida. O problema que os testes mais eficientes podem, em alguns
casos, ter resultados inconclusivos; isto , o teste pode no conseguir

i
i

cripto
2009/6/30
i page 187
Estilo OBMEP

i
i

N SEC. 7.3: UM TESTE DE COMPOSIO

187

decidir se um dado nmero ou no primo.

Tudo isto pode estar parecendo um tanto confuso e misterioso,


por isso daremos um exemplo de um tal teste, o mais simples de
todos. Trata-se de um teste que, ao receber um inteiro n > 2, tem
uma de duas sadas possveis: o nmero n composto ou o teste foi
inconclusivo. Note que o teste tem a sada oposta ao que realmente
queremos. Desejamos garantir primalidade, mas este teste s garante
composio: se a sada for inconclusiva, o nmero dado pode ser
composto ou no: o teste no suficiente para garantir qual a resposta
correta. Mas lembre-se: vamos apresentar este teste em particular
porque muito simples e d uma ideia razovel de como testes deste
tipo funcionam; s isso.
O teste tem por base o Teorema de Fermat, que enunciamos aqui
para lembr-lo do seu contedo exato.
Teorema de Fermat. Se p um primo e a um inteiro que no
divisvel por p, ento
ap1 1 (mod p).
Portanto, o teorema nos diz que se p primo ento uma certa congruncia tem que ser verdadeira. Considere, ento, o seguinte nmero
de 101 algarismos,
R(101) = 1111111111111111111111111111111111111111111111111
1111111111111111111111111111111111111111111111111111.

i
i

cripto
2009/6/30
i page 188
Estilo OBMEP

i
i

188

CAP. 7: ENCONTRANDO PRIMOS

Este nmero primo? Calculando o resduo r de 2n1 mdulo n


com auxlio de um computador, verificamos que
r = 5292914187654273058571598885199202595940728758186639
710565760508670021985609520505802825349865669415.
Mas, espera a: se p fosse primo, o resduo no devia dar 1? Afinal
isso que diz o Teorema de Fermat, no ? Mas se r 6= 1 alguma
coisa tem que ter sado errado. Ou o clculo do resduo est errado
(no est, fiz o clculo no meu computador e testei o resultado),
ou 2 divide n (brincadeirinha...), ou o Teorema de Fermat falso
(no , vimos uma demonstrao no captulo 5) ou nossa impresso
de que n fosse primo no se justifica; isto , na verdade n composto.
O mais interessante que o algoritmo de fatorao do meu computador no consegue calcular nenhum fator para este nmero, embora o
Teorema de Fermat nos garanta que ele composto!
fcil generalizar este argumento. Seja n o nmero inteiro que
queremos saber se composto. Escolhemos um inteiro b qualquer,
que no seja divisvel por n e calculamos o resduo r de bn1 mdulo
n. Se acontecer de r 6= 1 ento o Teorema de Fermat nos garante que
n no pode ser primo. Temos, assim, uma maneira indireta de provar
que um dado nmero composto. Em outras palavras, mesmo no
tendo determinado nenhum fator de n podemos ter certeza de que n
composto.
Naturalmente, a pergunta que precisamos fazer :

i
i

cripto
2009/6/30
i page 189
Estilo OBMEP

i
i

189

N SEC. 7.3: UM TESTE DE COMPOSIO

O que podemos afirmar sobre


r bn1 (mod n) for igual a 1?

se

resduo

G. W. Leibniz, o famoso matemtico alemo do sculo XVII, acreditava que se 2n1 1 (mod n) ento n teria que ser primo. Se isto
fosse verdade, teramos um teste extremamente eficiente para determinar a primalidade de um nmero inteiro. Infelizmente, a habilidade
de Leibniz em matemtica no lhe impediu de cometer este erro; de
fato,
pode acontecer que um nmero mpar composto satisfaa
bn1 1 (mod n) mesmo quando b 6 1 (mod n).
Um exemplo simples n = 25 quando b = 7. Neste caso, precisamos
calcular o resduo de 724 mdulo 25. Mas 24 = 8 3 e
73 18

(mod 25);

donde
724 ((73 )2 )4 (182 )4 (24)4 (1)4 1 (mod 25).
Portanto, o nmero composto 25 satisfaz a congruncia
724 1 (mod 25).
Em outras palavras, 25 comporta-se como se fosse um nmero primo
relativamente congruncia do Teorema de Fermat, quando tomamos
a base da potncia como sendo 7. Tais nmeros so conhecidos como
pseudoprimos; isto , falsos primos (pseudo um prefixo grego que

i
i

cripto
2009/6/30
i page 190
Estilo OBMEP

i
i

190

CAP. 7: ENCONTRANDO PRIMOS

significa falso). Mais precisamente, um nmero inteiro positivo n


um pseudoprimo relativamente base b se n for mpar, composto e
satisfizer a congruncia bn1 1 (mod n). Embora a base b possa
ser qualquer inteiro, no h necessidade de considerar bases maiores
que n 1 ou menores que 1. A razo que estamos efetuando apenas
clculos mdulo n, de forma que qualquer inteiro pode ser substitudo
por seu resduo, sem alterar o clculo da potncia.
Por que escolhemos 7 como base no exemplo acima e no, digamos,
2? A razo que
224 (27 )8 38 11

(mod 25),

de modo que o teste detectaria corretamente que 25 composto se


escolhssemos 2 como base. Isto significa, em particular, que 25 no
contradiz a afirmao de Leibniz. O menor inteiro positivo que composto e satisfaz 2n1 1 (mod n) 341. Como este nmero j bastante grande, efetuaremos os clculos da potncia usando o mtodo
do captulo 5.
Em primeiro lugar, fatoramos 341, obtendo
341 = 11 31.
Portanto, se r o resduo de 2340 mdulo 341, teremos o sistema
r 2340

(mod 11),

r 2340

(mod 31).

Contudo, pelo Teorema de Fermat 210 1 (mod 11), ao passo que

i
i

cripto
2009/6/30
i page 191
Estilo OBMEP

i
i

191

N SEC. 7.3: UM TESTE DE COMPOSIO

um clculo direto mostra que 25 1 (mod 31). Logo,

2340 (210 )34 1

(mod 11),

2340 (25 )68 1 (mod 31).


Com isso, o sistema pode ser reescrito na forma
r 1 (mod 11),
r 1 (mod 31);
e sequer precisamos aplicar o algoritmo chins do resto porque, evidentemente, r = 1 satisfaz ambas as congruncias. Logo,
b340 1 (mod 341);
e podemos concluir que 341 um pseudoprimo para a base 2, contradizendo assim a afirmao do Leibniz.
Feitas estas consideraes, podemos formular o teste resultante do
Teorema de Fermat da seguinte maneira.
Teste de composio. Seja n > 1 um inteiro mpar e b um nmero
inteiro que no divisvel por n. Calcule o resduo r de bn1 mdulo
n. Se
r 6= 1 ento n composto;
r = 1 ento o teste inconclusivo.
Naturalmente inconclusivo aqui significa que no podemos ter

i
i

cripto
2009/6/30
i page 192
Estilo OBMEP

i
i

192

CAP. 7: ENCONTRANDO PRIMOS

certeza se n primo ou composto. Os viles so os pseudoprimos:


nmeros compostos que deixam resduo 1 na congruncia do teste
acima. Antes que voc ache que fizemos muito esforo por nada,
lembre-se do exemplo com o qual comeamos esta seo. Usando o
teste, descobrimos que R(101) composto, mas no consigo calcular
nenhum fator com o meu computador, porque so grandes demais.
H muitos outros exemplos como este, dos quais o nmero
14

F (14) = 22 + 1
um dos mais espetaculares. Este nmero de 4 933 algarismos pseudoprimo para a base 2. Contudo, o resduo de 3F (14)1 mdulo F (14)

9266364202294755118302156584814258901280315479290422530388
6097614299720435210171437432968640006392902224715504620168
0095001725604630114472589558837940517046729438437748180083
1645097710516064151644731353621343233869471436258644642614
3404140194390198961769077788060540423932971248642995994518
5380752855773923829816713629478959807118043023411329069843
5918386510735172924058647619992288213210540491141093275957
0335468822169985631103470075481624938903793797306018707624
7843687388416174331487323349153272420349291556136382834077
1679624096206834211574398349814328539564737533064291530596
5788572000985851480646485181676557975568586764218376704438
continua...

i
i

cripto
2009/6/30
i page 193
Estilo OBMEP

i
i

N SEC. 7.3: UM TESTE DE COMPOSIO

193

0609884881302631172126361083233964505570126176409342149337
8652139954789464193009285094711628063432954634626708587436
7594463444139625279802423469187735988169809740798016076355
8756660915732373393355687126433151023684356948795074681449
5366102470470308386446567739129683404278437245037495377700
7491128599236856457726437662611536680740287906911185798788
7842108444553841368100395987050604541817029567526486493583
1300066049180516581348572289935984373033218327619142378706
7066989761084120454048840413780649009776228255511924299511
0548464682066431343714167187770687195870049693812084690516
4150156692624094182349789590144877540962982320907818130345
4651634419058586295097381771400433332253182091087518191373
9964774561356018160131641245372602875035168510650472282456
0043757300004448144561923398161715371451535611350494602034
9421627541720123626726413278735673890406938540849234508550
9297197760087826382470805424211327315246255840301596485309
8216754717890271627922382773031557667932618099223155763262
2957058064387928193998735873865903801681009121549568217921
6272435643998723867142738576833301406169458707777337835643
1259035182961453497743067137867268169891960319091925185387
continua...

i
i

cripto
2009/6/30
i page 194
Estilo OBMEP

i
i

194

CAP. 7: ENCONTRANDO PRIMOS

6764699510693890723124314879886857934757435609511498303631
3184482333406227325114404370409493199218730847899005245713
7262589157129312078042493366031343888921155946218315119689
0824691664947272076796853149272031772292723384678001487673
7289485199611040141169941570563464614331433825101237485028
2551540449120046780973081044244014767741281300291825405153
6662450553353080033618137315962021316238275075900462137814
6887272133760235138339124403618026146022497784037955983873
8106235330733724665038646006196783875613152976381049791233
9519502118800801010921866059557028885049053649233386186846
2293457874567301266626810211810517093398552281226240984566
1532354438296954820814066000994097063264255070300898292412
5496882595564447114246751422155740073823527127475651798531
1028057096845788109976392735550166125004557244441709616600
4678524716450796935361023345710385526247352937331815633300
1700220182361523365263097713881060293294769367726122819869
0730538976503932288071552286660658027816772182941835429267
6105433021000072632686528671653305019350583545229716468801
7600704028734065754263297922208019302142304992036994114511
5395692083829829127759337589147382283144757893138698782367
continua...

i
i

cripto
2009/6/30
i page 195
Estilo OBMEP

i
i

N SEC. 7.3: UM TESTE DE COMPOSIO

195

4960132062939955165969811950826493537398880556427059940676
0151759614385138660551353702532710344209861260306992151220
6491196140182604994527812792098357721628098443058943757381
8993574605834732242634770665833451870497469411031398760678
4691748814808710597793230354951146962786688724393282734233
6766674755399503915926422345813552266000767650733554802250
3290586961513720156305182728786969285962256153101352016029
2789911403225602267534784409091403252749485669809224389864
2225228726999955715234451099649284822014988445646906870856
0687344984932764868189113207434302275831390136874473275610
2049318851588609633761376006845924402609516283154658749932
4411209386536068440370854512156044540340439706700963382206
1448672696638585769812464786783087209776034508314405974562
8538838044904201295785245824550483832529686053887853018932
6847879111220292345808615508661056210593212298139150716832
7376850039576125448497337508566882256055070174814260336989
9198249347826265874560095153249285038574130230205892752609
9314884508593056148018263020465818819981893510070049325812
6858764215906417840636307876012637703053991445764802732796
3986565069519909277264869794934951743339858296269946169751
continua...

i
i

cripto
2009/6/30
i page 196
Estilo OBMEP

i
i

196

CAP. 7: ENCONTRANDO PRIMOS

5767722567946729190402238410127417897149773901275188203885
4875445968557341857521192305032447353155620391519830057834
4876043229651234507101232599676867795300534399281154330366
9561543133291104876022846635712773603755805368778662398435
0641126547572149014680068579593366753172005548119021674441
4461318127515595223785701620105649922262886079532365232976
6625056061650120464853768291381463183738603219745698745725
8065949430682912755117725405126442032692160838612266387888
3517903902198194036900560975373472283211077363328017623773
4638563339609601320040017448859818675802421313089099680661
2395196333178133084564313983286325230943963680919341251290
5466765334951445060146002079878932880934085645861962318355
5766170926604159837144983067589856601662702757157733868440
784972302728473757238028342337093591349409533609378842206167
que, com certeza, no igual a 1, provando assim que F (14) um
nmero composto. Este caso ainda mais espetacular do que R(101)
porque, apesar de meu computador precisar de apenas 13 segundos
para determinar que F (14) composto usando o teste de composio,
ningum ainda foi capaz de determinar fatores para este nmero!
Voc talvez esteja imaginando porque os nmeros R(101) e F (14)
ganharam estes nomes, em vez de serem chamados simplesmente de

i
i

cripto
2009/6/30
i page 197
Estilo OBMEP

i
i

197

N SEC. 7.3: UM TESTE DE COMPOSIO

n ou m. A razo que estes nmeros pertencem a famlias especiais


e bem estudadas. A famlia R(n) corresponde aos nmeros da forma
11111
1111} .
|
{z
n nmeros uns

O R vem do fato dos nmeros serem obtidos pela Repetio da


unidade. J a famlia F (n) constituda pelos nmeros da forma
n

F (n) = 22 + 1;
chamados de nmeros de Fermat, e dos quais j falamos um pouco na
introduo.
Como vimos na pgina 13, Fermat acreditava que todos os nmeros
da forma F (n) fossem primos, qualquer que fosse o inteiro n 0 escolhido. Esta afirmao de Fermat foi trazida ao conhecimento de
Euler, e em 1730 ele provou que 641 fator de F (5), mostrando, assim, que a afirmao de Fermat falsa. Como estes nmeros vm
sendo estudados h bastante tempo, sabe-se muito sobre eles. Por
exemplo, conhecemos a fatorao completa dos nmeros de Fermat
correspondentes a 5 n 11. J para
n = 12, 13, 15, 16, 17, 18, 19, 21 e 23,
conhecemos alguns fatores, mas no todos. Tambm so conhecidos
fatores de nmeros de Fermat muito grandes. A mais recente descoberta deve-se a Payam Samidoost que, em agosto de 2008, mostrou
que 6 089 279 223 + 1 divide F (79 221). Por outro lado, apesar de
sabermos se n = 14, 20, 22 e 24, ento o os nmeros F (n) corres-

i
i

cripto
2009/6/30
i page 198
Estilo OBMEP

i
i

198

CAP. 7: ENCONTRANDO PRIMOS

pondentes so compostos, nenhum fator conhecido para nenhum


destes nmeros de Fermat. Isto inesperado. Como possvel descobrir um fator para um nmero gigantesco como F (79 221), mas no
para F (14) uma outra histria, muito interessante, mas que no d
para contar aqui. Os detalhes podem ser encontrados no captulo 9
do livro [2].
Exerccio 63. O objetivo deste exerccio comprovar o resultado de
Euler, segundo o qual 641 divide F (5).
(a) Mostre que 641 = 27 5 + 1 e que 641 = 24 + 54 .
(b) Use (a) para mostrar que F (5) 5 0 (mod 641).
(c) Explique porque (b) implica que F (5) 0 (mod 641).
Os nmeros da famlia R(n) tm uma histria mais recente: foram
estudados a partir do sculo XIX como parte de uma tentativa de
entender melhor os padres que aparecem em dzimas peridicas. Os
nicos valores conhecidos de n para os quais R(n) primo so
2, 19, 23, 317, 1 031, 49 081, 86 453 e 109 297.
Como estes nmeros so mais fceis de tratar e tm propriedades mais
simples que os nmeros de Fermat, reservaremos o estudo deles para
nosso ltimo desafio.
Desafio 9. Seja n um nmero inteiro positivo e R(n) o nmero que
consiste em n nmeros uns repetidos.
(a) Mostre que R(n) = (10n 1)/9.

i
i

cripto
2009/6/30
i page 199
Estilo OBMEP

i
i

N SEC. 7.3: UM TESTE DE COMPOSIO

199

(b) Mostre que se k um fator de n ento R(k) divide R(n).


(c) Determine todos os fatores primos de R(6).
(d) Mostre que se R(n) for primo ento n tem que ser primo.
(e) Mostre que R(7) composto usando o teste desta seo.
Voc vai precisar de uma calculadora para fazer os itens (c) e (d)
deste exerccio.

i
i

cripto
2009/6/30
i page 200
Estilo OBMEP

i
i

Solues
Exerccios
1. O texto o seguinte:
claro que quebrar uma mensagem por contagem de
frequncia ainda mais simples se temos um computador. Se a lngua conhecida a maior parte do
processo pode ser automatizado, o que torna inviveis
todos os cdigos que envolvem substituio de letras,
como o que estamos utilizando para codificar esta
mensagem. Na verdade alguns dos primeiros computadores foram criados precisamente para ajudar a
quebrar os cdigos secretos usados pelos alemes durante a Segunda Guerra Mundial. Entre estes estava
o Colosso, um computador construdo na Inglaterra
por uma equipe liderada por Alan Turing, um dos
fundadores da computao cientfica.

200

i
i

cripto
2009/6/30
i page 201
Estilo OBMEP

i
i

201
2. A contagem no funciona porque a estrutura da frase foi desfeita; acrescentamos o A porque a letra mais frequente.
3. (d) Se a e a + 1 tiverem um divisor comum ento por (a) este
divisor tambm divide (a + 1) a = 1; logo o divisor tem que
ser 1.
4. A.
5. k fator de n! + k para todo 2 k n 1. Alm disso, o
cofator no pode ser um.
6. Se k divide m, ento m = c k. Se m divide n, ento n = d m.
Substituindo a primeira equao na segunda, n = d c k; donde
k divide n.
7. p = 3 ou p = 5 e n = p2 .
8. Se c tivesse um fator menor que p, este que seria o menor fator
de n, e no p.
9. x = 2 e y = 3.
10. No h soluo.
11. Se n = pe11 pet t e p1 < < pt ento a multiplicidade de p1
na fatorao de n , por definio, e1 . Contudo,
n
= pe22 pet t
p1

i
i

cripto
2009/6/30
i page 202
Estilo OBMEP

i
i

202
de modo que pe11 divide n. Por outro lado, se pe11 +1 dividisse n,
ento poderamos escrever
pe21 pet t = pe11 +1 c
para algum inteiro positivo c. Cancelando, pe1 dos dois lados
da equao, obtemos
pe22 pet t = p1 c.
Como do lado esquerdo todos os primos so distintos e diferentes de p1 , a unicidade da fatorao nos d uma contradio.
Portanto, nenhuma potncia de p1 maior que e1 divide n e uma
afirmao anloga vale para todos os outros primos.
12. Escreva 2n = (m 1)(m + 1) e use a unicidade da fatorao. A
resposta n = 3.
13. 3 na primeira jogada e 6 na segunda.
14. 5 na primeira jogada e 2 na segunda.
15. Uma possibilidade tirar 5 na primeira jogada e 3 na segunda
e, da em diante, 5 na primeira jogada e 2 na segunda de cada
vez.
16. Basta haver uma diferena de nove unidades entre a soma correta e a soma que foi calculada erradamente.

i
i

cripto
2009/6/30
i page 203
Estilo OBMEP

i
i

203
17. Basta haver uma diferena de nove unidades entre o produto
correto e o produto que foi calculado erradamente.
18. Os resduos possveis de qualquer nmero por 4 so 0, 1, 2 e 3.
Mas se o resduo for 0 ou 2 o nmero par. Logo um primo
mpar tem resduo igual a 1 ou 3. Por exemplo, 5 tem resduo
1 mdulo 4, ao passo que 7 tem resduo 3 mdulo 4.
19. Se a a0 (mod n) e b b0 (mod n), ento a a0 = k n e
b b0 = ` n, onde k e ` so os respectivos cofatores. Subtraindo
estas duas igualdades, obtemos
(a b) (a0 b0 ) = k n ` n = (k `) n;
que equivale a dizer que a b a0 b0 (mod n).
20. Basta notar que o algarismo das unidades de um nmero igual
a seu resduo mdulo 10 e determinar quais os resduos possveis
mdulo 10 para um nmero par e para um mltiplo de 5.
21. As duas maneiras diferentes correspondem a escrever 104 como
103 10 ou 102 102 e usar os valores j encontrados para estas
potncias mdulo 7.
22. A soma alternada (isto , com os sinais alternando entre mais e
menos) dos algarismos do nmero divisvel por 11 se, e somente
se, o nmero divisvel por 11.
23. Faa trs fora, como fizemos no caso dos noves fora.

i
i

cripto
2009/6/30
i page 204
Estilo OBMEP

i
i

204
24. Basta haver uma diferena de trs unidades entre a soma correta
e a soma que foi calculada erradamente.
25. Suponha que, dividindo n por m obtemos quociente q e resto r.
Se a conta estiver correta devemos ter que n noves fora tem que
ser igual a m noves fora vezes q noves fora mais r noves fora.
26. Os restos so, respectivamente, 1, 6 e 5.
27. Os valores so dados na tabela 7.1.
a
87 645 564 348
85 735 214 421
981 231 111

a0
8
1
1

8 764 556 434


8 573 521 442
98 123 111

Tabela 7.1: Exerccio 27

28. Para 35 994 ser divisvel por 7 preciso que 3 599+8 = 3 591;
para 3 591 ser divisvel por 7 preciso que 359 + 2 = 357;
para 357 ser divisvel por 7 preciso que 35+14 = 21. Como
21 divisvel por 7, ento 35 994 tambm . O outro nmero
no divisvel por 7.
29. Calcule 525 mdulo cada um dos primos mpares at achar resduo 1. A resposta 11.

i
i

cripto
2009/6/30
i page 205
Estilo OBMEP

i
i

205
30. Os inversos so dados nas tabelas abaixo.
Resduo
1
2
3
4
5
6

Inverso
1
4
5
2
3
6

Tabela 7.2: Inversos mdulo 7


Resduo
1
2
3
4
5
6
7
8
9
10
11
12

Inverso
1
7
9
10
8
11
2
5
3
4
6
12

Tabela 7.3: Inversos mdulo 13


31. Basta lembrar que n 1 1 (mod n).

i
i

cripto
2009/6/30
i page 206
Estilo OBMEP

i
i

206
32. Os inversos so dados nas tabelas abaixo.
Resduo
1
2
3
4
5

Inverso
1
?
?
?
5

Tabela 7.4: Inversos mdulo 6


Resduo
1
2
3
4
5
6
7
8
9
10
11
12
13
14

Inverso
1
8
?
4
?
?
13
2
?
?
11
?
7
14

Tabela 7.5: Inversos mdulo 15

i
i

cripto
2009/6/30
i page 207
Estilo OBMEP

i
i

207
33. Os valores so os seguintes:
Resduo
2
3
4

Inverso
3
2
3

Tabela 7.6: b tal que a b 0 (mod 6)


Resduo
3
5
6
9
10
12

Inverso
5
3
5
5
3
5

Tabela 7.7: b tal que a b 0 (mod 13)

34. Alguns exemplos so


1, 7, 5, 12, 22 V (5, 12)
1, 7, 15, 29, 50 V (7, 15)
5, 10, 15, 30, 35 V (15, 10)
mas talvez voc escolha outros.

35. Dividindo a por m, temos que


a = m q0 + r0 .

i
i

cripto
2009/6/30
i page 208
Estilo OBMEP

i
i

208
Substituindo isto na equao (3.3.3),
a = (x1 a + y1 n) q 0 + r0 ,
que equivale a
r0 = (x1 ) a + (1 y1 q 0 ) n.
Assim, r0 V (a, n). Contudo, se r0 < M fosse positivo teramos
um elemento em V (a, n) ainda menor que m, o que no possvel. Logo r0 = 0, e m divide a.
36. Se houvesse um fator primo comum, teria que dividir 3. Como
3 primo o fator teria que ser igual a 3. Mas se 3 dividisse
6 k 2, ento tambm dividiria
3 (2 k) (6 k 2) = 2,
o que no possvel.
37. Como 6k 1 (mod 6k + 1) temos que os inversos so congruentes, respectivamente, a 3k, 2k e k.
38. 2 913.
39. 33.
40. A quantidade (mnima) total de arroz 3 105 288.

i
i

cripto
2009/6/30
i page 209
Estilo OBMEP

i
i

209
41. 1065 deixa resto 5 na diviso por 7 e 378 deixa resto 1 na diviso
por 7.
42. Ambos os restos so iguais a 3.
45 231

43. Os restos na diviso por 31 so 1 quando o dividendo 214


498 766 543 335 231
9 876
e 215
e 2 quando o dividendo 643
.

44. 3 tem ordem 6 mdulo 7, 2 tem ordem 10 mdulo 11, 5 tem


ordem 3 mdulo 31 e 7 tem ordem 6 mdulo 43.
45. Se ak 1 (mod n), ento ak cn = 1 para algum inteiro c.
Mas se a e n forem pares, o lado esquerdo ser um nmero par;
logo no pode ser igual a 1.
46. A tabela com as ordens encontra-se abaixo:
Resduo mdulo 11
1
2
3
4
5
6
7
8
9
10

Ordem
1
10
5
5
5
10
10
10
5
2

Tabela 7.8: Ordens dos resduos mdulo 11

i
i

cripto
2009/6/30
i page 210
Estilo OBMEP

i
i

210
47. Os nicos resduos que tm ordem mdulo 12 so 1, que tem
ordem 1 e 5, 7 e 11, que tm ordem 2 cada um.
48. A ordem de 3 mdulo 31 30.
49. 6 tem ordem 3 mdulo 43; 37 37 6 (mod 43) e
398 745 27 (mod 43).
50. Pelo Teorema de Fermat, 342 1 (mod 43). Mas,
98 745 = 2 351 42 + 3.
Logo,
398 745 (342 )2 351 33 27 (mod 43).

51. O resto 16.


52. O resto 1.
53. O resto p 1.
54. Os restos so dados pelas congruncias
(a) 2495 1 (mod 15 841);
(b) de 241 045 32 (mod 41 041);
(c) de 277 1902 (mod 2 465).

i
i

cripto
2009/6/30
i page 211
Estilo OBMEP

i
i

211
59. Se duas chaves pblicas tm um primo em comum ento podemos descobri-lo, e com isso fatorar as chaves, calculando o mximo divisor comum das duas chaves pblicas.
60. p = 13.
61. (a) Como p# e p# 1 so nmeros consecutivos, tm que ser
primos entre si por ???.
(b) Se houvesse infinitos nmeros primos e o maior deles fosse
p, ento p# 1 teria que ter um fator primo maior que p
pelo resultado em (a), que uma contradio.
(c) p = 7.
62. Os primos so:
5
11 17 23
83 89 101 107
179 191 197 227
281 293 311 317
419 431 443 449

29
113
233
347
461

41
131
239
353
467

47
137
251
359
479

53 59 71
149 167 173
257 263 269
383 389 401
491

63. (b) Como 32 = 4 7 + 4, temos que


F (5) 54 (27 5)4 24 + 54 0

(mod 641).

(c) Como 5 inversvel mdulo 641 podemos concluir de (b)


que F (5) 0 (mod 641).

i
i

cripto
2009/6/30
i page 212
Estilo OBMEP

i
i

212

Desafios
1. trs.
2. Os resduos possveis so iguais aos primos entre n + 1 e n!.
3. Por exemplo, se a > 2 um inteiro qualquer, ento a tem inverso
mdulo n = a2 1.
4. A primeira congruncia nos diz que x = a + my para algum
inteiro y. Substituindo na segunda congruncia, obtemos
my b a (mod n).
Se m e n no so primos entre si, no podemos simplesmente
inverter o m para deixar o y livre. Convertendo esta ltima congruncia em uma expresso de inteiros, temos que
my = b a + nk, para algum inteiro k.
Em outras palavras,
my nk = b a.
Se d o mximo divisor comum entre m e n ento existem
inteiros m0 e n0 tais que m = dm0 e n = dn0 . Substituindo na
ltima equao, obtemos
(m0 y n0 k)d = b a.

i
i

cripto
2009/6/30
i page 213
Estilo OBMEP

i
i

213
Portanto, para que haja soluo preciso que d divida b a. Se
b a = dc para algum inteiro c, ento
m0 y n0 k = c,
que nos d m0 y c (mod n0 ). Como cancelamos o mximo divisor comum entre m e n, os inteiros so m0 e n0 so primos
entre si e m0 pode ser invertido em m0 y c (mod n0 ), o que
nos d o valor de y. Portanto, quando d divide b a o sistema
tem soluo.

5. (a) Pelo Teorema de Fermat, ap1 1 (mod p).


(b) Dividindo p 1 por k obtemos p 1 = kq + r, donde
ap1 (ak )q ar 1 ar

(mod p),

pela definio de k. Como ap1 1 (mod p), pelo Teorema de Fermat, podemos concluir que ar 1 (mod p).
(c) Se r 6= 0 ento teramos que 0 < r < k satisfaz ar 1
(mod p), o que impossvel pela definio de k como sendo
o menor inteiro positivo tal que ak 1 (mod p). Logo,
r = 0.
(d) Substituindo r = 0 em p1 = kq+r, temos que p1 = kq,
de modo que k tem que dividir p 1.

6. p = 3.

i
i

cripto
2009/6/30
i page 214
Estilo OBMEP

i
i

214
7. Temos que
m = (p 1)(q 1) = pq (p + q) + 1 = n (p + q) + 1.
Logo,
(p + q) = n m + 1.
Para achar p e q quando conhecemos p + q e pq basta resolver
uma equao quadrtica. Outra soluo consiste em notar que
(p q)2 = (p + q)2 2pq = (n m + 1)2 2n.
Com isto temos p + q e p q em funo de n e m e podemos
resolver um sistema linear para achar p e q.
8. (a) Qualquer nmero dividido por 4 tem resto 0, 1, 2 ou 3.
Como um primo diferente de 2 mpar, os nicos resduos
possveis mdulo 4 neste caso so 1 e 3.
(b) Tm resduo 1 mdulo 4 os primos 5, 13, 17, 29 e 37 e tm
resduo 3 mdulo 4 os primos 3, 7, 11, 19 e 23.
(c) (4n + 1)(4k + 1) = 4(4nk + n + k) + 1 ou use congruncia
mdulo 4. Note que foi preciso escolher letras diferentes k
e n, porque os nmeros 4n+1 e 4k+1 podem ser diferentes.
(d) No. Por exemplo 3 7 = 21 = 4 5 + 1.
(e) Pelo Teorema de Fatorao nica, o nmero 4(p1 . . . pk )+3
pode ser escrito como um produto de primos. Estes primos
no podem pertencer ao conjunto {p1 , . . . , pk }. S nos resta mostrar que os primos na fatorao de 4(p1 . . . pk ) + 3
no podem ser todos da forma 4n + 1. Mas se fosse este

i
i

cripto
2009/6/30
i page 215
Estilo OBMEP

i
i

215
o caso, o produto destes primos seria da forma 4n + 1 por
(c), o que no verdade pois 4(p1 . . . pk ) + 3 deixa resto 3
na diviso por 4.
(f) Suponha, por absurdo, que {3, p1 , . . . , pk } o conjunto de
todos os primos da forma 4n + 3 e aplique (e).
9. (a) Mostre que
R(n) = (10n 1)/9 =

99
= 1 1.
9

(b) Use que


10kr 1
= 10k(r1) + 10k(r2) + + 10k + 1.
10k 1
(c) Como 6 = 2 3, temos que R(2) = 11 e R(3) = 111 = 3 37
dividem R(6) por (b). Mas,
R(6)
= 91 = 7 13.
3 11 37
Logo,
R(6) = 3 7 11 13 37.
(d) Consequncia imediata de (b).
(e) Como R(7) 1 = 10 R(6) podemos usar a fatorao de
R(6) obtida acima para facilitar o clculo do resduo de 2
mdulo R(7), que d 553 891.

i
i

cripto
2009/6/30
i page 216
Estilo OBMEP

i
i

216

i
i

cripto
2009/6/30
i page 217
Estilo OBMEP

i
i

Referncias Bibliogrficas
[1] COELHO, S. P.; POLCINO MILIES, C. Nmeros: uma Introduo Matemtica. So Paulo: Editora da Universidade de So
Paulo, 2 000.
[2] COUTINHO, S. C. Nmeros inteiros e criptografia RSA. Srie de
Computao e Matemtica n. 2, IMPA e SBM, segunda edio
(revisada e ampliada), 2 000.
[3] HEFEZ, A. Elementos de Aritmtica. Sociedade Brasileira de
Matemtica, 2 005.

217

i
i

construcciones*ge
2009/8/12
i page 1
Estilo OBMEP

i
i

Uma Introduo s
Construes Geomtricas
Eduardo Wagner

i
i

construcciones*ge
2009/8/12
i page 2
Estilo OBMEP

i
i

Texto j revisado pela nova ortografia.

i
i

construcciones*ge
2009/8/12
i page 3
Estilo OBMEP

i
i

Eduardo Wagner

i
i

construcciones*ge
2009/8/12
i page 4
Estilo OBMEP

i
i

construcciones*ge
2009/8/12
i page i
Estilo OBMEP

i
i

Apresentao
O

E

As construes geomtricas tiveram incio na Grcia antiga.


Esta a razo do ttulo desta apostila estar escrito em grego. O
desenvolvimento acelerado da Matemtica no mundo antigo deveuse a gregos geniais, pensadores, filsofos, cientistas que colocaram o
raciocnio, a lgica e a razo como ferramentas para descobrir coisas
novas e tentar explicar o mundo em que viviam. Tudo nmero disse
Pitgoras sintetizando o pensamento que tudo na natureza pode ser
explicado pelos nmeros, ou seja, pela Matemtica. As construes
geomtricas estavam no centro desse desenvolvimento da Matemtica.
As construes geomtricas continuam at hoje a ter grande importncia na compreenso da Matemtica elementar. Seus problemas
desafiam o raciocnio e exigem slido conhecimento dos teoremas de
geometria e das propriedades das figuras e no exagero dizer que
no h nada melhor para aprender geometria do que praticar as construes geomtricas.
Esta apostila dedicada aos alunos da OBMEP traz uma introi

i
i

construcciones*ge
2009/8/12
i page ii
Estilo OBMEP

i
i

ii
duo s construes geomtricas. Nela, estamos dando a base para
as construes abordando apenas dois temas: os lugares geomtricos
e as expresses algbricas. Com estes contedos bem estudados, o
aluno ter facilidade em estudar um mundo novo que vem a seguir
cujo foco principal o das transformaes geomtricas. Mas isto fica
para mais tarde. Por ora, desejo a todos um bom proveito nesta
leitura. Voc ter contato com problemas intrigantes, desafiadores,
mesmo que a maioria no seja difcil. Mas certamente gostoso resolver algo novo enquanto que ler problemas que j conhecemos
definitivamente aborrecido.

i
i

construcciones*ge
2009/8/12
i page iii
Estilo OBMEP

i
i

Sumrio
1 Construes Elementares

1.1

Introduo . . . . . . . . . . . . . . . . . . . . . . . . .

1.2

Paralelas e Perpendiculares . . . . . . . . . . . . . . .

1.3

Tornando as Construes mais Prticas

. . . . . . . .

1.4

Diviso de um Segmento em Partes Iguais . . . . . . .

13

2 Lugares Geomtricos

16

2.1

A Paralela . . . . . . . . . . . . . . . . . . . . . . . . .

17

2.2

A Mediatriz . . . . . . . . . . . . . . . . . . . . . . . .

18

2.3

A Bissetriz

. . . . . . . . . . . . . . . . . . . . . . . .

20

2.4

O Arco Capaz . . . . . . . . . . . . . . . . . . . . . . .

21

3 Expresses Algbricas
3.1
3.2

40

A 4a Proporcional . . . . . . . . . . . . . . . . . . . .
p
a2 b2 . . . . . . . . . . . . . . . . . . . . . . . . .

41
44

iii

i
i

construcciones*ge
2009/8/12
i page iv
Estilo OBMEP

i
i

iv

SUMRIO

3.3

a n, n natural . . . . . . . . . . . . . . . . . . . . . .

47

3.4

A Mdia Geomtrica . . . . . . . . . . . . . . . . . . .

48

3.5

A Equao do Segundo Grau . . . . . . . . . . . . . .

51

3.6

Expresses Homogneas . . . . . . . . . . . . . . . . .

56

3.7

Construes com Segmento Unitrio . . . . . . . . . .

58

4 Solues dos Exerccios Propostos

64

i
i

construcciones*ge
2009/8/12
i page 1
Estilo OBMEP

i
i

Captulo 1

Construes Elementares
1.1

Introduo

As construes geomtricas aparecem na antiguidade e tiveram


enorme importncia no desenvolvimento da Matemtica. H 2 000
anos a palavra nmero significava nmero natural. No havia nmeros
negativos e as fraes no eram consideradas nmeros, eram apenas
razes entre nmeros. Era de fato complicado. Se no havia ainda a
noo de nmero racional, os nmeros reais ento estavam a sculos
de distncia. Entretanto os gregos tiveram uma ideia engenhosa. A
de representar uma grandeza qualquer por um segmento de reta. Esta
ideia equivalente a dizer que todo nmero real positivo est associado
a um ponto de uma semirreta graduada. Hoje, visualizamos o nmero
real x assim:

i
i

construcciones*ge
2009/8/12
i page 2
Estilo OBMEP

i
i

 CAP. 1: CONSTRUES ELEMENTARES

Antigamente, a mesma ideia era vista assim:


B

As operaes de adio e subtrao de segmentos so inteiramente


intuitivas.
a

a+b
a
ab

A multiplicao de dois segmentos podia ser visualizada como a


rea de um retngulo e a razo entre dois segmentos era . . . Bem, era
simplesmente isso mesmo, a razo entre dois segmentos.
Um problema comum hoje , por exemplo, o de calcular a hipotenusa de um tringulo retngulo cujos catetos so 2 e 3. A soluo
simples e usa o teorema de Pitgoras.
Se x o comprimento da hipotenusa ento
x=

22 + 32 =

4 + 9 = 13.

O mesmo problema antigamente era enunciado assim: construir


o tringulo retngulo cujos catetos medem 2 unidades e 3 unidades.
A soluo era completamente geomtrica. Era dado um segmento
unitrio u e o tringulo era construdo com as medidas dadas.

i
i

construcciones*ge
2009/8/12
i page 3
Estilo OBMEP

i
i

N SEC. 1.2: PARALELAS E PERPENDICULARES

B
u
u
u
A

Observe a figura acima. Se associarmos o segmento u ao nmero

1, o segmento AB a visualizao do nmero real 13.


Desta forma, calcular de hoje sinnimo do construir de antigamente e as dificuldades so equivalentes. Se hoje achamos difcil calcular a hipotenusa de um tringulo retngulo conhecendo o permetro
e a altura relativa hipotenusa, igualmente difcil desenhar o tringulo retngulo onde o permetro e a altura so dados atravs de dois
segmentos.

1.2

Paralelas e Perpendiculares

Nas construes geomtricas so permitidos apenas a rgua (no


graduada) e o compasso. A rgua serve apenas para desenhar uma
reta passando por dois pontos dados e o compasso serve apenas para
desenhar uma circunferncia cujo raio dado por um segmento e cujo
centro um ponto dado. Estes instrumentos no podem ser utilizados
de nenhuma outra maneira.
A pureza das construes com rgua e compasso a mesma da
geometria analtica que tambm resolve, de forma equivalente, proble-

i
i

construcciones*ge
2009/8/12
i page 4
Estilo OBMEP

i
i

 CAP. 1: CONSTRUES ELEMENTARES

mas de geometria usando as coordenadas (pontos dados), a equao


da reta (rgua) e a equao da circunferncia (compasso).
Para comear a desenhar, h dois problemas bsicos que precisamos aprender.
1. Traar por um ponto dado uma reta perpendicular a uma reta
dada.
2. Traar por um ponto dado uma reta paralela a uma reta dada.
Para resolver o primeiro, seja P um ponto dado fora de uma reta
r dada. A construo a seguinte. Com centro em P trace uma
circunferncia qualquer cortando a reta r nos pontos A e B como
mostra a figura a seguir.

Figura 1

Em seguida, desenhamos dois arcos de circunferncia de mesmo


raio, com centros nos pontos A e B, determinando na interseo o

i
i

construcciones*ge
2009/8/12
i page 5
Estilo OBMEP

i
i

N SEC. 1.2: PARALELAS E PERPENDICULARES

ponto Q. A reta P Q perpendicular reta r e o primeiro problema


est resolvido.
O fato importante das construes geomtricas que no basta
encontrar a soluo. preciso justificar por que ela correta. Neste
primeiro problema, a primeira circunferncia desenhada garante que
P A = P B e as duas seguintes, garantem que QA = QB . Assim,
os pontos P e Q equidistam de A e B. Portanto, eles pertencem
mediatriz do segmento AB que a reta perpendicular a AB passando
pelo seu ponto mdio.
Para resolver o segundo problema, seja P um ponto dado fora de
uma reta r dada. A construo a seguinte. Traamos trs circunferncias com mesmo raio: a primeira com centro em P cortando a
reta r em A; a segunda com centro em A cortando a reta r em B e a
terceira com centro em B cortando a primeira circunferncia em Q.

Figura2
A reta P Q paralela reta r e o problema est resolvido.
Para justificar, observe que, pelas construes efetuadas, P ABQ
um losango e, portanto seus lados opostos so paralelos.

i
i

construcciones*ge
2009/8/12
i page 6
Estilo OBMEP

i
i

 CAP. 1: CONSTRUES ELEMENTARES

Com a rgua e o compasso, resolva o problema seguinte.


Problema 1.
Dado um segmento AB construa o tringulo equiltero ABC e sua
altura CM.
Soluo: Coloque a ponta seca do compasso em A e desenhe um
arco de circunferncia de raio AB e, em seguida faa o contrrio: um
arco de centro B e raio BA. Estes arcos cortam-se em C e D. Ento,
o tringulo ABC equiltero e a reta CD a mediatriz de AB.

Figura 3

1.3

Tornando as Construes mais Prticas

Para tornar as construes mais prticas vamos permitir a utilizao dos primeiros instrumentos impuros: os esquadros. Eles so
construdos para facilitar e agilizar o traado das construes de paralelas e perpendiculares. Eles so de dois tipos: um deles com ngulos
de 90 , 45 , 45 e outro com ngulos de 90 , 60 , 30 .

i
i

construcciones*ge
2009/8/12
i page 7
Estilo OBMEP

i
i

N SEC. 1.3: TORNANDO AS CONSTRUES MAIS PRTICAS

Veja, a seguir, como utilizamos a rgua e os esquadros para o


traado de retas paralelas e perpendiculares.
a) Traar pelo ponto P a reta paralela
reta r.
Soluo: Posicione a rgua e um dos
esquadros como na figura ao lado.

Fixe bem a rgua e deslize o esquadro at que seu bordo passe pelo
ponto P . Fixe o esquadro e trace
por P a reta paralela reta r.

b) Traar pelo ponto P a reta perpendicular reta r.


Soluo:
1o Passo.
Posicione a rgua e um dos esquadros como na figura ao lado.

i
i

construcciones*ge
2009/8/12
i page 8
Estilo OBMEP

i
i

 CAP. 1: CONSTRUES ELEMENTARES

2o Passo.
Fixe a rgua e afaste um pouco o
esquadro da reta r para permitir um
melhor traado da perpendicular.

3o Passo.
Posicione o segundo esquadro sobre
o primeiro e trace por P a perpendicular reta r.

Uma outra soluo a seguinte:

1o Passo.
Posicione a rgua e o esquadro
de 45 como na figura ao lado.

2o Passo.
Fixe a rgua e deslize o esquadro at
que o outro cateto passe por P .
Fixe o esquadro e trace por P a perpendicular reta r.

i
i

construcciones*ge
2009/8/12
i page 9
Estilo OBMEP

i
i

N SEC. 1.3: TORNANDO AS CONSTRUES MAIS PRTICAS

Problema 2.
Dado o segmento AB, construa o quadrado ABCD.

Soluo: (Figura por conta do aluno).


Trace por A e B retas perpendiculares ao segmento AB. Trace as
circunferncias de centro A, passando por B e de centro B passando
por A. As intersees dessas circunferncias com as perpendiculares
so os vrtices C e D.

Problema 3.
Construir o tringulo ABC sendo dados os trs lados:

Soluo: Desenhe uma reta r e sobre ela assinale um ponto que


chamaremos B. Para transportar o segmento a, pegue o compasso,
ponha a ponta seca em uma das extremidades e abra at que a ponta
do grafite coincida com a outra extremidade. Ponha agora a ponta
seca em B e trace um pequeno arco cortando a reta r. Este o ponto
C tal que BC = a.

i
i

construcciones*ge
2009/8/12
i page 10
Estilo OBMEP

i
i

10

 CAP. 1: CONSTRUES ELEMENTARES

Figura 4

Pegue agora o segmento b com o compasso. Com centro em C


desenhe, acima da reta r um arco de circunferncia de raio b. Pegue
o segmento c com o compasso e, com centro em B desenhe um arco
de raio c. A interseo desses dois arcos o vrtice A do tringulo.
O exemplo anterior mostrou como transportar segmentos de um
lugar para outro. Vamos mostrar agora como transportar ngulos.

Figura 5

i
i

construcciones*ge
2009/8/12
i page 11
Estilo OBMEP

i
i

N SEC. 1.3: TORNANDO AS CONSTRUES MAIS PRTICAS

11

Problema 4.
Dado o ngulo , e a semirreta OX construir o ngulo XOY = .

Soluo: Com centro no vrtice do ngulo dado trace um arco de


circunferncia cortando seus lados nos pontos A e B (veja figura 6).
Sem modificar a abertura do compasso trace um arco com centro O
cortando OX em C. Pegue com o compasso a distncia AB e trace,
com centro em C e com este raio, um arco determinando sobre o
primeiro o ponto D. A semirreta OY que passa por D tal que
XOY = .

Figura 6

i
i

construcciones*ge
2009/8/12
i page 12
Estilo OBMEP

i
i

12

 CAP. 1: CONSTRUES ELEMENTARES

Problema 5.
Construir o tringulo ABC dados o lado a e os ngulos B e C:

Soluo: (Figura por conta do aluno)


Desenhe na sua folha de papel o segmento BC = a e, em seguida
transporte os ngulos dados construindo as semirretas BX e CY de
forma que os ngulos CBX e BCY sejam iguais aos ngulos dados.
A interseo das duas semirretas o vrtice A.
A partir de agora, vamos permitir, por comodidade, utilizar a
rgua graduada para fornecer as medidas dos segmentos e o transferidor para as medidas dos ngulos.
Assim o problema anterior poderia ser enunciado assim: construir
o tringulo ABC sabendo que o lado BC mede 5 cm e que os ngulos
B e C medem 62 e 38 respectivamente.
Os esquadros, a rgua graduada e o transferidor so instrumentos
que permitem tornar mais rpida e prtica a execuo dos desenhos,
mas so apenas acessrios (podem ser dispensados). Os instrumentos
essenciais so apenas a rgua lisa e o compasso.

i
i

construcciones*ge
2009/8/12
i page 13
Estilo OBMEP

i
i

N SEC. 1.4: DIVISO DE UM SEGMENTO EM PARTES IGUAIS

13

Figura 7

1.4

Diviso de um Segmento em Partes Iguais

Dividir um segmento dado em um nmero qualquer de partes


iguais uma das construes bsicas e, frequentemente, precisaremos
us-la.
Dado o segmento AB, para dividi-lo, por exemplo, em 5 partes
iguais, traamos uma semirreta qualquer AX e sobre ela, com o compasso, determinamos 5 segmentos iguais: AA 1 , A1 A2 , A2 A3 , A3 A4 ,
A4 A5 (ver figura 8).

Figura 8

i
i

construcciones*ge
2009/8/12
i page 14
Estilo OBMEP

i
i

14

 CAP. 1: CONSTRUES ELEMENTARES

Trace agora a reta A5 B. As paralelas a esta reta traadas pelos


pontos A1 , A2 , A3 , A4 determinam sobre AB os pontos P1 , P2 , P3 , P4
que o dividiro em 5 partes iguais.
Problema 6.
Construir o tringulo ABC conhecendo o lado BC = 5,3 cm, e as
medianas mb = 4 cm e mc = 5 cm.
Soluo: Sabemos que a distncia do baricentro a um vrtice igual
a 2/3 da respectiva mediana. Assim, se G o baricentro do tringulo ABC, o tringulo GBC pode ser construdo porque o lado BC
2
conhecido e so tambm conhecidas as distncias GB = mb e
3
2
GC = mc .
3
Observe, na figura 9 que dividimos cada mediana em trs partes
iguais para obter 2/3 de cada uma.

O
Q'
Q
P'

P
Figura 9

i
i

construcciones*ge
2009/8/12
i page 15
Estilo OBMEP

i
i

N SEC. 1.4: DIVISO DE UM SEGMENTO EM PARTES IGUAIS

15

Uma vez construdo o tringulo GBC, determinamos (com rgua


e compasso) o ponto mdio de BC e, sobre a reta M G determinamos
o ponto A tal que M A = 3M G. O problema est resolvido.

Figura 9A

i
i

construcciones*ge
2009/8/12
i page 16
Estilo OBMEP

i
i

Captulo 2

Lugares Geomtricos
As primeiras ferramentas das construes geomtricas so os lugares geomtricos bsicos. Essas figuras, que mostraremos a seguir,
permitiro desenvolver um mtodo de construo que baseado nas
propriedades das figuras.
O que um lugar geomtrico?
A expresso (muito antiga) lugar geomtrico, nada mais que
um conjunto de pontos e, para definir tal conjunto, devemos enunciar
uma propriedade que esses pontos devem ter. Se essa propriedade
p, o conjunto dos pontos que possuem p o lugar geomtrico da
propriedade p.
Por exemplo, o lugar geomtrico dos pontos que distam 5 cm de
um ponto A a circunferncia de centro A e raio 5 cm.

16

i
i

construcciones*ge
2009/8/12
i page 17
Estilo OBMEP

i
i

N SEC. 2.1: A PARALELA

2.1

17

A Paralela

Imagine que a base BC de um tringulo ABC dada e que a


altura (h) relativa a esta base tambm dada. Ento, conhecemos a
distncia do vrtice A at a reta BC e o lugar geomtrico do vrtice
A , portanto, uma reta paralela reta BC distando h dela.

Problema 7.
Desenhe o tringulo ABC conhecendo os lados AB = 4,5 cm,
BC = 5,2 cm e a altura relativa ao lado BC = 3,8 cm.
Soluo: Trace uma reta r e sobre ela o segmento BC com o comprimento dado. Longe de BC desenhe uma reta perpendicular a r
e seja X o ponto de interseo (ver figura 10). Assinale sobre ela o
segmento XY = 3,8 cm e trace por Y uma paralela reta r. Este o
lugar geomtrico do vrtice A.
Longe do seu desenho, construa um segmento de 4,5 cm usando
a rgua. Agora, ponha o compasso com esta abertura e, com centro
em B, desenhe uma circunferncia com este raio. A circunferncia
cortar a reta paralela em dois pontos mostrando que h duas solues
(diferentes) para o problema.

i
i

construcciones*ge
2009/8/12
i page 18
Estilo OBMEP

i
i

18

 CAP. 2: LUGARES GEOMTRICOS

Figura 10

2.2

A Mediatriz

A mediatriz de um segmento AB a reta perpendicular a AB que


contm o seu ponto mdio. Veja que todo ponto da mediatriz tem
mesma distncia aos extremos do segmento.

B
Figura 11

i
i

construcciones*ge
2009/8/12
i page 19
Estilo OBMEP

i
i

19

N SEC. 2.2: A MEDIATRIZ

Observe tambm que se um ponto no est na mediatriz de AB


ento ele no equidista de A e B. Portanto, dizemos que a mediatriz
de um segmento AB o lugar geomtrico dos pontos que equidistam
de A e B.
Para construir, traamos dois arcos de circunferncia com centros
em A e B e com intersees P e Q como na figura 12.

Figura 12

A reta P Q a mediatriz de AB. Qual a justificativa?


Observe a figura anterior e pense um pouco.
Pela construo que fizemos, AP BQ um losango e, como sabemos, suas diagonais so perpendiculares.

i
i

construcciones*ge
2009/8/12
i page 20
Estilo OBMEP

i
i

20

 CAP. 2: LUGARES GEOMTRICOS

2.3

A Bissetriz

b a semirreta OC tal que


A bissetriz de um ngulo AOB
b = C OB.
b
AOC

Costumamos dizer que a bissetriz divide o ngulo em dois outros


congruentes. Todo ponto da bissetriz de um ngulo equidista dos
lados do ngulo. Na figura a seguir, P um ponto da bissetriz OC
b e P D e P E so perpendiculares aos lados OA e OB.
do ngulo AOB

Como os tringulos retngulos OP D e OP E so congruentes,


temos P D = P E.
Portanto, a bissetriz de um ngulo o lugar geomtrico dos pontos
que equidistam dos lados do ngulo.
b traamos com centro
Para construir a bissetriz do ngulo A OB
em O um arco de circunferncia cortando os lados do ngulo em X e
Y.
Em seguida, traamos dois arcos de mesmo raio com centros em
X e Y que se cortam em C.A semirreta OC bissetriz do ngulo
b
AOB.
Qual a justificativa?

i
i

construcciones*ge
2009/8/12
i page 21
Estilo OBMEP

i
i

21

N SEC. 2.4: O ARCO CAPAZ

X
C
O

Figura 13

Observe a figura 13 e pense um pouco.


Pela construo que fizemos, os tringulos OXC e OY C so conb = C OB.
b
gruentes (caso LLL) e, portanto, AOC

2.4

O Arco Capaz

Considere dois pontos A e B sobre uma circunferncia. Para todo


ponto M sobre um dos arcos, o ngulo AM B = constante.

i
i

construcciones*ge
2009/8/12
i page 22
Estilo OBMEP

i
i

22

 CAP. 2: LUGARES GEOMTRICOS

Um observador que percorra o maior arco AB da figura acima,


consegue ver o segmento AB sempre sob mesmo ngulo. Este arco
chama-se arco capaz do ngulo sobre o segmento AB.
Naturalmente que, se um ponto N pertence ao outro arco AB
ento o ngulo AN B tambm constante e igual a 180 .
Ainda interessante notar que se M qualquer ponto da circB reto. Por isso, cada
cunferncia de dimetro AB o ngulo A M
semicircunferncia de dimetro AB chama de arco capaz de 90
sobre AB.

Construo do arco capaz :


So dados o segmento AB e o ngulo . Para construir o lugar
geomtrico dos pontos que conseguem ver AB segundo ngulo faa
o seguinte:
1) Desenhe a mediatriz de AB.
b = .
2) Trace a semirreta AX tal que B AX

3) Trace por A a semirreta AY perpendicular a AX.


4) A interseo de AY com a mediatriz, o ponto O, centro do arco
capaz.

i
i

construcciones*ge
2009/8/12
i page 23
Estilo OBMEP

i
i

23

N SEC. 2.4: O ARCO CAPAZ

Com centro em O desenhe o arco AB.

Figura 14

O arco AB que voc desenhou o lugar geomtrico do ngulo


construdo sobre o segmento AB. Para justificar, observe que se
b = ento B AY
b = 90 e, sendo M o ponto mdio de AB,
B AX
b = . Assim AOB
b = 2 e, para qualquer ponto M
temos que AOM
cB = .
do arco AB tem-se AM
Problema 8.

Construir a circunferncia que passa por trs pontos A, B, e C dados


em posio.
Soluo: Seja O, o centro da circunferncia que passa por A, B e
C. Como OB = OC, ento O pertence mediatriz de AB. Como

i
i

construcciones*ge
2009/8/12
i page 24
Estilo OBMEP

i
i

24

 CAP. 2: LUGARES GEOMTRICOS

OB = OC ento O pertence mediatriz de BC. Assim, o ponto O


a interseo dessas duas mediatrizes. Veja figura 15.

Figura 15

Problema 9.
Construir a circunferncia inscrita em um tringulo dado.
Soluo: Seja ABC o tringulo dado. O centro da circunferncia
inscrita (incentro) o ponto de interseo das bissetrizes internas.
Precisamos ento traar as bissetrizes de dois ngulos do tringulo.
O ponto de interseo das duas bissetrizes (I) o centro da circunferncia inscrita, mas no podemos ainda desenh-la, pois no
conhecemos o raio.
Ateno: O compasso s pode ser usado para desenhar uma circunferncia com centro e raio conhecidos. No se pode ajeitar nada ou
traar nada no olho.

i
i

construcciones*ge
2009/8/12
i page 25
Estilo OBMEP

i
i

25

N SEC. 2.4: O ARCO CAPAZ

Figura 16

Continuando o problema, traamos por I uma reta perpendicular


a BC, cortando BC em D. Temos agora um ponto por onde passa a
circunferncia inscrita. Traamos ento a circunferncia de centro I
e raio ID e o problema est resolvido.
Nas construes geomtricas a soluo de um problema, em geral,
no nos ocorre imediatamente. preciso analisar a situao e pensar.
Para analisar a situao devemos imaginar o problema j resolvido
para buscar as propriedades que permitiro a soluo. Voc ver, a
partir de agora, os problemas sendo analisados desta maneira.
Problema 10.
Traar por um ponto exterior a uma circunferncia as duas retas tangentes.
Soluo: Imagine que o ponto P e a circunferncia de centro O estejam dados em posio. Imaginemos o problema resolvido.

i
i

construcciones*ge
2009/8/12
i page 26
Estilo OBMEP

i
i

26

 CAP. 2: LUGARES GEOMTRICOS

B
Figura 17

Se P A tangente em A circunferncia ento OA perpendicular


b reto ento o ponto A pertence a uma
a P A. Como o ngulo P AO
semicircunferncia de dimetro P O. Como o mesmo vale para o ponto
B a construo a seguinte.
Determinamos o ponto M mdio de P O traando a mediatriz
de P O. Traamos a circunferncia de centro M e raio M P = M O
que corta a circunferncia dada em A e B. As retas P A e P B so
tangentes circunferncia dada. O problema est resolvido.

Problema 11.
So dados: uma circunferncia de centro O, um ponto P e um segmento a. Pede-se traar por P uma reta que determine na circunferncia uma corda de comprimento a.

i
i

construcciones*ge
2009/8/12
i page 27
Estilo OBMEP

i
i

27

N SEC. 2.4: O ARCO CAPAZ

Figura 18

Soluo: Este um problema que, novamente, os dados esto em


posio. Para analisar o problema, imagine, na circunferncia, uma
corda AB de comprimento a. Imagine agora todas essas cordas.

Figura 19

Se M o ponto mdio da corda AB de comprimento a e em qualquer posio ento OM constante pois OA e AM so constantes.
Assim, o lugar geomtrico de M uma circunferncia de centro O.
Por outro lado, supondo o problema resolvido, a reta que passa por P

i
i

construcciones*ge
2009/8/12
i page 28
Estilo OBMEP

i
i

28

 CAP. 2: LUGARES GEOMTRICOS

e determina na circunferncia dada uma corda de comprimento a tal


cO = 90 e, portanto, M tambm pertence circunferncia
que P M
de dimetro BC.

Figura 20

A construo agora pode ser feita. Siga todos os passos.


1) Assinale um ponto X qualquer sobre a circunferncia dada.
2) Pegue com o compasso o segmento dado e determine, sobre a circunferncia um ponto Y tal que XY = a.
3) Trace por O uma perpendicular a XY determinando o ponto Z
mdio de XY .
4) Trace a circunferncia de centro O e raio OZ, que um lugar
geomtrico de M .
5) Trace a mediatriz de P O determinando o seu ponto mdio C.
6) Com centro em C trace a circunferncia de dimetro P O, que
outro lugar geomtrico de M .

i
i

construcciones*ge
2009/8/12
i page 29
Estilo OBMEP

i
i

29

N SEC. 2.4: O ARCO CAPAZ

7) As duas circunferncias cortam-se em M e M .


8) As retas P M e P M so a soluo do problema.

Figura 21
Construir figuras ou resolver situaes pelo mtodo dos lugares
geomtricos consiste essencialmente no que vimos no problema anterior. Existe um ponto-chave (no caso, M ) e conseguimos, atravs das
propriedades das figuras, encontrar dois lugares geomtricos para ele.
Assim, estando o ponto-chave determinado, o problema fica resolvido.
Frequentemente, o ponto-chave a prpria soluo do problema.
Veja a seguir.
Problema 12.
Construir o tringulo ABC sendo dados: o lado BC = 4,5 cm, o
ngulo A = 60 e a altura relativa ao lado BC, h = 3,2 cm.
b = 60 ento A est no arco capaz de 60 construdo
Soluo: Se B AC
sobre BC. Por outro lado, como o vrtice A dista 3,2 cm da reta BC,

i
i

construcciones*ge
2009/8/12
i page 30
Estilo OBMEP

i
i

30

 CAP. 2: LUGARES GEOMTRICOS

ele est em uma reta paralela a BC distando 3,2 cm da reta BC. A


construo est a seguir.

Figura 22

Sobre uma reta r assinale o ponto B e construa o segmento BC.


Construa o arco capaz de 60 sobre BC que o primeiro lugar geomtrico para o vrtice A. Para colocar a altura, assinale um ponto P
qualquer sobre a reta r (de preferncia longe do arco capaz), trace por
P uma perpendicular a r e, sobre ela, determine o ponto Q tal que
P Q = h. A paralela r traada por Q o segundo lugar geomtrico
de A e o problema est resolvido.
A reta paralela cortou o arco capaz em dois pontos, A e A. Como
os tringulos ABC e ABC so congruentes, dizemos que o problema
possui apenas uma soluo.

i
i

construcciones*ge
2009/8/12
i page 31
Estilo OBMEP

i
i

31

N SEC. 2.4: O ARCO CAPAZ

Problema 13.
Construir o tringulo ABC conhecendo os lados AB = 5,2 cm,
BC = 5,7 cm e a altura relativa ao lado AB, h = 4,5 cm.
Soluo: Faa um desenho imaginando o problema resolvido e seja
b reto, o
CD = h a altura relativa ao lado AB. Como o ngulo B DC

ponto D pertence ao arco capaz de 90 construdo sobre BC. Como


CD conhecido, determinamos o ponto D. Sobre a reta BD determinamos o ponto A e o problema est resolvido.

Figura 23

O prximo problema tem especial interesse pois o artifcio que


vamos utilizar ser til na soluo de vrios outros problemas.
Problema 14.
dado o tringulo ABC com AB = 4 cm, BC = 6,5 cm e CA = 7 cm.
Trace uma reta paralela a BC cortando AB em M e AC em N de

i
i

construcciones*ge
2009/8/12
i page 32
Estilo OBMEP

i
i

32

 CAP. 2: LUGARES GEOMTRICOS

forma que se tenha AN = BM .


Soluo: Imaginemos o problema resolvido.

Figura 24
Repare que no adianta nada termos dois segmentos de mesmo
comprimento sem conexo entre si. Uma ideia, portanto na nossa
figura de anlise traar por N o segmento N D paralelo a M B.
Como M N DB um paralelogramo temos N D = M B (dizemos que
foi feita uma translao no segmento M B). Logo, AN = N D e o
tringulo AN D issceles. Veja agora que:
ADN = DAN porque AN = N D.
ADN = DAB porque so alternos internos nas paralelas AB
e N D.
b do tringulo ABC e o problema
Assim, AD bissetriz do ngulo A
est resolvido.
Para construir:
Construa inicialmente o tringulo ABC com os trs lados dados.
b que corta BC em D.
Trace a bissetriz do ngulo B AC

i
i

construcciones*ge
2009/8/12
i page 33
Estilo OBMEP

i
i

33

N SEC. 2.4: O ARCO CAPAZ

Trace por D uma paralela a AB que corta AC em N .


Trace por N uma paralela a BC que corta AB em M .
(figura final por conta do leitor).
Problema 15.
Desenhe uma reta r e dois pontos A e B situados de um mesmo lado
de r. Determine o ponto P sobre a reta r de forma que a soma
AP + P B seja mnima.
Soluo: Para analisar o problema, desenhamos a reta r, e dois pontos
A e B quaisquer de um mesmo lado de r. Obtenha o ponto B,
simtrico de B em relao r. Para fazer isto, trace por B uma
perpendicular r e, com o compasso, passe B para o outro lado
obtendo o seu simtrico.

P
B'

Figura 25

Assinale um ponto Q qualquer, sobre a reta r. Trace QA, QB e


QB. Como r mediatriz de BB ento QB = QB. Assim a soma

i
i

construcciones*ge
2009/8/12
i page 34
Estilo OBMEP

i
i

34

 CAP. 2: LUGARES GEOMTRICOS

AQ + QB sempre igual a AQ + QB. Entretanto esta soma ser


mnima quando A, Q e B forem colineares. E nesta posio est o
ponto P procurado.
A construo do problema do caminho mnimo entre dois pontos
passando por uma reta ento imediata. Desenhe o simtrico de um
dos pontos em relao reta e ligue este simtrico ao outro ponto. A
interseo com a reta dada a soluo do problema.
A seguir daremos uma lista de problemas propostos sendo os
primeiros, claro, mais fceis. Cada problema um desafio novo,
desde a anlise at o momento de decidir o que se deve fazer primeiro.
Confira depois sua construo com a que est no gabarito e bom trabalho.

Problemas Propostos
1) Construa um quadrado cuja diagonal tenha 4,5 cm
2) Desenhe uma circunferncia de 3,2 cm de raio e construa o tringulo equiltero inscrito nela.
3) Desenhe um tringulo cujos lados medem 5 cm, 6 cm e 7 cm. Quanto
mede, aproximadamente o raio da circunferncia circunscrita?
4) Construa o tringulo ABC conhecendo os lados AB = 5,2 cm,
AC = 6,5 cm e a altura relativa ao vrtice A igual a 4,5 cm. Quanto
b
mede o ngulo B AC?

5) Construa o trapzio ABCD conhecendo a base maior


AB = 7 cm, a base menor CD = 2 cm, e os lados AD = 3,4 cm e

i
i

construcciones*ge
2009/8/12
i page 35
Estilo OBMEP

i
i

N SEC. 2.4: O ARCO CAPAZ

35

BC = 5,1 cm.
b = 50 e os
6) Construir o tringulo ABC conhecendo o ngulo B
lados AB = 6 cm e BC = 4,8 cm.
7) Construir o tringulo ABC conhecendo o lado BC = 4,7 cm e as
medianas BB= 5 cm e CC= 3,5 cm.
8) Construa o trapzio issceles sabendo que as bases medem 6,5 cm
e 2,5 cm e que as diagonais medem 5,5 cm.
9) Construa o hexgono regular cujo lado mede 2,4 cm.
b mede 60
10) No tringulo ABC o lado BC mede 5 cm, o ngulo A
e a mediana AA mede 4 cm. Se AC < AB quanto mede, aproxib
madamente o ngulo B?

11) Construir o tringulo ABC conhecendo o lado BC = 7 cm e as


alturas BD = 5,4 cm e CE = 6,7 cm.
12) No plano cartesiano com os eixos graduados em centmetros, uma
circunferncia C tem centro (0, 3) e raio 2 cm. Determine um ponto
P do eixo dos X tal que as tangentes traadas de P a C tenham
comprimento de 4,5 cm.
13) Construir o tringulo ABC conhecendo a mediana AA= 5 cm e
as alturas BD = 6 cm e CE = 4,7 cm.
14) Construir o tringulo ABC, retngulo em A conhecendo a hipotenusa BC = 6 cm e a soma dos catetos AB + AC = 8,1 cm.
15) Construir o tringulo ABC de permetro 11 cm sabendo que os
beC
b medem, respectivamente, 58 e 76 .
ngulos B

i
i

construcciones*ge
2009/8/12
i page 36
Estilo OBMEP

i
i

36

 CAP. 2: LUGARES GEOMTRICOS

16) Construir o trapzio ABCD conhecendo a soma das bases


AB + CD = 8,6 cm, as diagonais AC = 6 cm e BD = 5 cm e
o lado AD = 4 cm.
17) As paralelas r e s so as margens de um rio e os pontos A e B
esto em lados opostos desse rio. Determine a posio de uma
ponte P Q perpendicular s margens (P r e Q s) de forma que
o percurso AP + P Q + QB seja mnimo.

Figura 26

18) Construir o tringulo ABC sabendo que AB = 5,8 cm, cos A = 0,6
e que o lado BC o menor possvel.
19) Dado um segmento m e, em posio, os pontos P , A e B (figura
27), traar por P uma reta r de forma que A e B fiquem de um
mesmo lado de r e de tal forma que a soma das distncias de A e
B r seja igual a m.

i
i

construcciones*ge
2009/8/12
i page 37
Estilo OBMEP

i
i

37

N SEC. 2.4: O ARCO CAPAZ

Figura 27
20) So dadas duas circunferncias K e K e um segmento a (figura
28). Traar pelo ponto A a secante P AQ (P K e Q K) de
forma que se tenha P Q = a.

Figura 28

21) Usando uma figura igual do exerccio anterior, trace a secante

i
i

construcciones*ge
2009/8/12
i page 38
Estilo OBMEP

i
i

38

 CAP. 2: LUGARES GEOMTRICOS

P AQ de comprimento mximo.
22) Uma mesa de sinuca tem vrtices dados em coordenadas:
A = (0, 0), B = (8, 0), C = (8, 4) e D = (0, 4). Uma bola P
atirada, sem efeito, em um ponto Q da tabela BC. Aps as
reflexes nas tabelas BC e CD ela cai na caapa A. Determine a
posio exata do ponto Q e faa o desenho da trajetria.
23) De uma circunferncia C conhecemos apenas o arco abaixo (figura
29). Limitando-se ao espao disponvel (interior do retngulo),
determine o raio de C.

Figura 29

24) Na figura 30, cada um dos pontos M , N , P e Q pertence a um


lado de um quadrado. Construa esse quadrado.

i
i

construcciones*ge
2009/8/12
i page 39
Estilo OBMEP

i
i

39

N SEC. 2.4: O ARCO CAPAZ

Figura 30

25) So dados em posio (figura 31) os pontos A, B, C e D sobre a


reta r. Trace por A e B duas paralelas e trace por C e D outras
duas paralelas de forma que as intersees dessas retas formem um
quadrado.

Figura 31

i
i

construcciones*ge
2009/8/12
i page 40
Estilo OBMEP

i
i

Captulo 3

Expresses Algbricas
Neste captulo vamos aprender a construir as figuras e resolver os
problemas utilizando um ponto de vista muito diferente. No captulo
anterior, voc j reparou que, muitas vezes, necessitamos de altas
doses de criatividade para conseguir a chave para a resoluo de um
problema. O detalhe mnimo mas essencial, para conseguir encontrar
o caminho da soluo, os alunos chamam de mgica e, de fato, no
deixa de ser. Entretanto, nem sempre a mgica nos ocorre.
A outra abordagem de um problema de construo consiste em
escolher um segmento da figura a ser construda que ser tomado
como incgnita. Utilizando as propriedades e teoremas da geometria
podemos tentar resolver o problema algebricamente e encontrar uma
frmula que determina a incgnita em funo dos dados do problema.
Passaremos ento a construir, com rgua e compasso, a frmula encontrada e este caminho tambm bastante interessante.
Em todo o captulo cada segmento est identificado com sua me40

i
i

construcciones*ge
2009/8/12
i page 41
Estilo OBMEP

i
i

41

N SEC. 3.1: A 4A PROPORCIONAL

dida. Assim, quando se fala em um segmento a, voc tem toda a


liberdade de imaginar que a a medida desse segmento em uma dada
unidade. Mas para permitir esta dualidade, necessrio que nossas
frmulas sejam homogneas. Assim, se a e b so segmentos (ou os
nmeros que os representam), faz sentido escrever a + b ou a 2 + b2 .
No primeiro caso, estamos somando dois segmentos e no segundo, estamos somando as reas de dois quadrados de lados a e b. Por isso,
nas construes geomtricas nesta abordagem inicial, no tem sentido
escrever a+b2 , pois um segmento no pode ser somado com uma rea.
Vamos comear para que voc veja do que estamos falando.

3.1

A 4a Proporcional

Dados os segmentos a, b e c dizemos que o segmento x quarta


proporcional desses segmentos quando:
c
a
=
b
x
Esta relao de proporcionalidade j aparece no sculo 5 a.C. e
sua construo feita com o argumento do teorema de Tales.

Figura 1

i
i

construcciones*ge
2009/8/12
i page 42
Estilo OBMEP

i
i

42

 CAP. 3: EXPRESSES ALGBRICAS

Sobre um ngulo qualquer de vrtice O tomemos sobre um lado os


segmentos OA = a e AC = c e, sobre o outro lado, OB = b. Traando
por C uma paralela reta AB determinamos D na semirreta OB. O
segmento BD = x a soluo da equao.
Veja a seguir um problema cuja soluo pode ser feita com a 4 a
proporcional.
Problema 16.
Inscrever no tringulo ABC (figura 2) um quadrado tendo um lado
sobre BC.

Figura 2

Soluo: Suponha o problema resolvido. A figura 3 mostra o quadrado


M N P Q inscrito em ABC com o lado M N sobre BC.
Seja x o lado do quadrado. Vamos calcular este valor em funo
da base BC = a do tringulo e da altura relativa esta base (h). O
tringulo AP Q, que tem base P Q = x e altura h x semelhante ao

i
i

construcciones*ge
2009/8/12
i page 43
Estilo OBMEP

i
i

43

N SEC. 3.1: A 4A PROPORCIONAL

Figura 3

tringulo ABC. Logo,

x
hx
=
a
h

Da,
xh = ah ax
ax + xh = ah
e
x=

ah
.
a+h

Temos ento uma frmula que calcula x em funo de a e h. Vamos


tratar agora de construir esta frmula.
ah
a+h
h
Observe que x =
o mesmo que
= , ou seja, a
a+h
a
x
nossa incgnita x a 4a proporcional entre a + h, a e h. A figura 4
mostra como obter x usando o teorema de Tales.
Com a construo anterior, conhecemos o lado do quadrado e
agora, devemos pensar como constru-lo dentro do tringulo. No

i
i

construcciones*ge
2009/8/12
i page 44
Estilo OBMEP

i
i

44

 CAP. 3: EXPRESSES ALGBRICAS

difcil. Podemos traar a altura AD e, sobre ela (com o compasso)


construir o ponto E tal que DE = x. A paralela por E reta BC
determina os vrtices P e Q do quadrado.
Sendo a e b os segmentos dados, a terceira proporcional entre a e
a
b
b2
b o segmento x tal que = , ou seja, x = . A construo a
b
x
a
mesma que mostramos para a quarta proporcional.

Figura 4

3.2

p
a2 b 2

Sejam a e b segmentos dados. Se x = a2 b2 ento x a


hipotenusa de um tringulo retngulo cujos catetos so a e b. Basta
ento construir duas semirretas perpendiculares (voc pode usar os
esquadros) e assinalar os segmentos OA = a e OB = b. A hipotenusa
AB = x a soluo da equao.

i
i

construcciones*ge
2009/8/12
i page 45
Estilo OBMEP

i
i

N SEC. 3.2:

p
A2 B2

45

Figura 5

No outro caso, se a e b so os segmentos dados e x = a2 b2


ento x um cateto de um tringulo retngulo cuja hipotenusa a,
sendo b o outro cateto. Para construir devemos desenhar duas semirretas perpendiculares assinalar o segmento OB = b sobre uma
delas e, com centro em B, desenhar um arco de raio a cortando a
outra perpendicular em A. O cateto OA = x a soluo da equao.

Figura 6

i
i

construcciones*ge
2009/8/12
i page 46
Estilo OBMEP

i
i

46

 CAP. 3: EXPRESSES ALGBRICAS

Estas construes me lembram um aluno que me contou que,


quando estava na 7a srie, uma questo de uma prova de mltipla
p
escolha pedia para assinalar o valor aproximado de 6,72 + 8,62 .

Enquanto todos os colegas se esforavam nas contas ele construiu


com sua rgua e esquadro, com todo o cuidado, o tringulo retngulo
de catetos 6,7 cm e 8,6 cm. Depois, mediu a hipotenusa encontrando
10,9 cm. Ele tinha encontrado a resposta em menos de um minuto.

Expresses do tipo a2 b2 c2 . . . podem ser construdas


sem dificuldade bastando aplicar vrias vezes os procedimentos descritos acima.

Problema 17.
Construir a diagonal de um paraleleppedo retngulo conhecendo as
arestas a, b e c.

Figura 7

Soluo: Sabemos que a diagonal de um paraleleppedo retngulo de

dimenses a, b e c dado por x = a2 + b2+ c2 . Fazendo y = a2 + b2


p
e, em seguida, x = y 2 + c2 , determinamos x como mostra a figura
8.

i
i

construcciones*ge
2009/8/12
i page 47
Estilo OBMEP

i
i

N SEC. 3.3: A N, N NATURAL

47

Figura 8

3.3

a n, n natural

Dado um segmento a, podemos construir todos os elementos da

sequncia a 2, a 3, a 4, . . . pela construo abaixo que fcil de

entender. Observe que, na figura 9, AA n = a n.

Figura 9

i
i

construcciones*ge
2009/8/12
i page 48
Estilo OBMEP

i
i

48

 CAP. 3: EXPRESSES ALGBRICAS

Entretanto, quando n grande, podemos buscar um caminho mais


curto. Veja o problema seguinte.
Problema 18.

Dado o segmento a construir o segmento x = a 21.


Soluo: Pesquisando um pouco, podemos perceber que a hipotenusa
de um tringulo retngulo de catetos 4a e 2a :
y=

(4a)2 + (2a)2 = 16a2 + 4a2 = 20a2 = a 20.

Assim, com mais um passo, chegamos a x = a 21.

Figura 10

3.4

A Mdia Geomtrica

Dados dois segmentos a e b, definimos a sua mdia aritmtica por

a+b
m=
e sua mdia geomtrica por g = ab.
2

i
i

construcciones*ge
2009/8/12
i page 49
Estilo OBMEP

i
i

N SEC. 3.4: A MDIA GEOMTRICA

49

Para construir a mdia geomtrica precisamos recordar duas das


relaes mtricas no tringulo retngulo.

Figura 11

As relaes que utilizaremos so h2 = mn e b2 = am. A primeira

(h = mn ) significa que a altura relativa hipotenusa mdia


geomtrica entre as projees dos catetos sobre a hipotenusa e, a

segunda (b = am ), que um cateto mdia geomtrica entre a


hipotenusa e sua projeo sobre ela. Assim, podemos construir a
mdia geomtrica de duas formas.
Construmos sobre uma reta os segmentos AH = a e HB = b.
Traando a mediatriz de AB encontramos seu ponto mdio (O) e
traamos uma semicircunferncia de centro O e dimetro AB. A
perpendicular a AB traada por H determina o ponto C na semicircunferncia. Desta forma, CH a mdia geomtrica entre a e b, ou

seja, CH = g = ab.

Figura 12

i
i

construcciones*ge
2009/8/12
i page 50
Estilo OBMEP

i
i

50

 CAP. 3: EXPRESSES ALGBRICAS

A outra forma de construir consiste em desenhar o segmento


AB = a e, sobre ele, assinalar o ponto H tal que AH = b. Traamos
ento a semicircunferncia de dimetro AB e, por H, a perpendicular
a AB que determina o ponto C sobre a semicircunferncia. Desta

forma, AC a mdia geomtrica entre a e b, ou seja, AC = g = ab.

Figura 13

Problema 19.
Dados os segmentos a e b encontre os segmentos x e y tais que:
(
x+y =a
xy = b2

Figura 14
Soluo: Sobre uma reta r assinale um segmento AB = a, encontre seu ponto mdio e trace a semicircunferncia de dimetro AB.

i
i

construcciones*ge
2009/8/12
i page 51
Estilo OBMEP

i
i

N SEC. 3.5: A EQUAO DO SEGUNDO GRAU

51

Assinale um ponto P sobre r, trace por P uma perpendicular a r e


sobre ela construa o segmento P Q = b. A paralela a r traada por Q
determina o ponto C sobre a semicircunferncia. A perpendicular
r traada por C determina o ponto H interior a AB. Os segmentos
AH = x e HB = y so tais que x + y = a e xy = b2 .

Figura 15

3.5

A Equao do Segundo Grau

A equao do segundo grau que era construda ainda na antiguidade tinha a forma x2 + b2 = ax onde a e b so segmentos dados. O
significado era encontrar (com rgua e compasso) um segmento x tal
que a rea do quadrado de lado x somada com a rea do quadrado de
lado b seja igual rea de um retngulo de base a e altura x.

i
i

construcciones*ge
2009/8/12
i page 52
Estilo OBMEP

i
i

52

 CAP. 3: EXPRESSES ALGBRICAS

Depois disso, problemas de natureza variada, conduziam a equaes do segundo grau onde os coeficientes j eram representados por
nmeros, mas estava ainda muito longe a notao que usamos hoje.
Por exemplo, a equao x2 6x + 8 = 0 era, ainda no sculo XV,
escrita como census et 8 demptis 6 rebus (isto latim). Devemos
lembrar que, na antiguidade no existiam nmeros negativos e, cada
soluo de uma equao era certo segmento de reta (cujo equivalente
hoje sua medida que um nmero positivo).
A equao bsica

x2 + b2 = ax.

Primeira soluo: Com os nossos modernos conhecimentos sabemos


que a equao x2 + b2 = ax a mesma que x2 ax + b2 = 0 e suas
razes so dadas por
x=

a2 4b2
.
2

p
O radical r = a2 (2b)2 um cateto de um tringulo retngulo
cuja hipotenusa a e o outro cateto 2b. Naturalmente que, para que
o nosso problema tenha soluo devemos ter a 2 (2b)2 0, ou seja,
a 2b. Supondo esta hiptese e estando construdo o radical r, as

i
i

construcciones*ge
2009/8/12
i page 53
Estilo OBMEP

i
i

53

N SEC. 3.5: A EQUAO DO SEGUNDO GRAU

razes da equao so:


x1 =

a r

2 2

x2 =

a r
+ .
2 2

Figura 16
Na figura 16, o tringulo ABC, retngulo em A foi construdo com
AB = 2b e BC = a obtendo-se AC = r. Pelo ponto P , mdio de BC
r
traamos P Q paralela a AB para obter CQ = . A circunferncia de
2
centro C e raio CQ determina na reta BC os pontos M e N . Veja
a r
a r
que P M = = x1 e que P N = + = x2 .
a 2
2 2
O problema est resolvido.
Segunda soluo: Podemos imaginar uma soluo diferente para a
soluo da equao bsica x2 + b2 = ax. Inicialmente, vamos escrevla na forma x2 ax + b2 = 0 e lembremos que a e b so segmentos
dados. O que sabemos sobre as razes desta equao? Se x 1 e x2
so as razes, conhecemos as propriedades da soma e do produto:

i
i

construcciones*ge
2009/8/12
i page 54
Estilo OBMEP

i
i

54

 CAP. 3: EXPRESSES ALGBRICAS

x1 + x2 = a e x1 x2 = b2 . O problema passa a ser ento o de determinar dois segmentos, conhecendo sua soma e sua mdia geomtrica.
Podemos ento desenhar uma circunferncia de dimetro AB = a e
a
uma paralela a AB distando b de AB (figura 17). Se B , essa
2
paralela determinar um ponto C sobre a semicircunferncia e a projeo de C sobre AB o ponto P tal que AP = x 1 e P B = x2 .

Figura 17

Problema 20.
A figura 18 mostra uma circunferncia tangente no ponto T reta r e
um ponto P sobre r. Dado o segmento a, construa por P uma secante
P AB tal que AB = a.

Figura 18

i
i

construcciones*ge
2009/8/12
i page 55
Estilo OBMEP

i
i

N SEC. 3.5: A EQUAO DO SEGUNDO GRAU

55

Soluo: Inicialmente, observe que um problema muito parecido com


este j foi proposto e resolvido no captulo anterior. Vamos agora
resolv-lo algebricamente. Suponhamos o problema resolvido e seja
P A = x.

Utilizando o conceito de potncia de um ponto em relao a uma


circunferncia temos P A P B = P T 2 , ou seja, x(x + a) = t2 . Para
encontrar o valor de x devemos resolver a equao x 2 + ax t2 = 0.
Usando a frmula de resoluo da equao do segundo grau temos
que:

a + a2 + 4t2
x=
.
2
p
a2 + (2t)2 a
Arrumando ligeiramente esta frmula, temos x =
.
2
p
2
2
Ora o radical r = a + (2t) a hipotenusa de um tringulo retngulo cujos catetos so a e 2t. O resto fcil e a construo est a
seguir. Uma vez determinado o segmento x, basta traar uma circunferncia centro P e raio x para determinar o ponto A na circunferncia.

i
i

construcciones*ge
2009/8/12
i page 56
Estilo OBMEP

i
i

56

 CAP. 3: EXPRESSES ALGBRICAS

Figura 19

3.6

Expresses Homogneas

Todas as expresses algbricas que apareceram at agora so homogneas, ou seja, o resultado no depende da unidade de medida utilizada nos segmentos. Por exemplo, se a um segmento de 3,6 cm e b

um segmento de 4,2 cm, podemos construir o segmento x = a2 + b2


como hipotenusa do tringulo retngulo de catetos a e b, e este segmento x independente da unidade em que a e b foram medidos. Por

outro lado, podemos perfeitamente olhar para a frmula x = a2 + b2


de forma numrica, ou seja, podemos pensar que x o resultado do
p
clculo x = 3,62 + 4,22 ' 5,53. No exerccio a seguir, dados os
1
1 1
segmentos a e b, vamos determinar o segmento x tal que = + .
x
a b
claro que, se pensarmos nos segmentos a e b, esta expresso no faz

i
i

construcciones*ge
2009/8/12
i page 57
Estilo OBMEP

i
i

57

N SEC. 3.6: EXPRESSES HOMOGNEAS

o menor sentido, mas se pensarmos que a e b so os nmeros que medem esses segmentos em alguma unidade, faz total sentido perguntar
que segmento tem a medida igual a x. O curioso e, muito importante,
que esse segmento sempre o mesmo, independente da unidade em
que a e b foram medidos.
Como reconhecer expresses homogneas?
Uma expresso envolvendo segmentos a, b, c, . . . homognea
se, quando multiplicamos cada um deles por um nmero k > 0, o
resultado fica multiplicado por k.
Isto significa que o resultado independente da escala, ou seja,
com qualquer tipo de rgua utilizada para medir os segmentos dados,
o resultado sempre o mesmo.
Problema 21.
Dados os segmentos a e b, determine o segmento x tal que
Soluo: Fazendo as contas encontramos x =

1
1 1
= + .
x
a b

ab
. Observe que
a+b

b
a+b
= , o que mostra que x
esta relao pode ser escrita na forma
a
x
a quarta proporcional entre os segmentos a + b, a e b. A construo
natural est na figura 20.

Figura 20

i
i

construcciones*ge
2009/8/12
i page 58
Estilo OBMEP

i
i

58

3.7

 CAP. 3: EXPRESSES ALGBRICAS

Construes com Segmento Unitrio

Se a um segmento ento o smbolo a no tem significado geomtrico. Mesmo se pensarmos que a representa a medida de um
segmento em certa unidade, no podemos entender, a princpio, o

que significa o smbolo a. Se em certa unidade (u) o segmento a

mede 4, ento a deve ser igual a 2. Entretanto, se outra rgua


estiver graduada na unidade v = 4u ento o segmento a mede 1 e,

consequentemente, a deve ser tambm igual a 1.

Estas reflexes mostram que, na expresso x = a (que no


homognea), para representar x como um segmento precisamos saber
em que unidade o segmento a foi medido. Entretanto, se estabelecermos um segmento unitrio (u = 1) que ser usado para medir todos
os outros segmentos do problema, podemos interpretar a expresso

x = a, como x = a 1, ou seja, x a mdia geomtrica entre a e


o segmento unitrio.

Figura 21

Utilizando um segmento unitrio (u = 1), dado um segmento a


1
a
podemos construir x = a2 . Esta relao pode ser escrita como = ,
a
x
ou seja, x a quarta proporcional entre u, a e a.

i
i

construcciones*ge
2009/8/12
i page 59
Estilo OBMEP

i
i

N SEC. 3.7: CONSTRUES COM SEGMENTO UNITRIO

59

Figura 22

Outra construo de x = a2 utiliza a relao do tringulo retngulo que diz que o quadrado de um cateto igual ao produto da
hipotenusa pela sua projeo sobre ela. Veja a figura 23.

Figura 23

A mesma relao utilizada nesta ltima construo pode ser uti1


lizada para construir x =
onde a um segmento dado. Aqui, a
a
unidade a mdia geomtrica entre x e a.

i
i

construcciones*ge
2009/8/12
i page 60
Estilo OBMEP

i
i

60

 CAP. 3: EXPRESSES ALGBRICAS

Em cada um dos exerccios propostos, procure observar se a expresso dada homognea. Se for, imagine a construo independente
de unidade. Em caso contrrio, estabelea um segmento unitrio a
sua escolha.

Problemas Propostos
1) Dados os segmentos a, b, c, d, e (a sua escolha) construa
abc
.
x=
de

2) Dados os segmentos a e b (a sua escolha) construa x = a2 + 3b2 .


a
3) Dado o segmento a construa x = .
5
4) Construa um segmento de comprimento

5,8 centmetros.

5) Dados os segmentos a e b (a sua escolha) construa x =

a2
.
b

6) Dados os segmentos a e b do exerccio anterior construa x =

a
.
b

i
i

construcciones*ge
2009/8/12
i page 61
Estilo OBMEP

i
i

N SEC. 3.7: CONSTRUES COM SEGMENTO UNITRIO

61

7) Dados os segmentos a, b, c, d, (a sua escolha) construa


x=

a2 + bc
.
d

8) Dados os segmentos a e b (a sua escolha) resolva o sistema


(

xy =a
xy = b2

Determine que relao deve existir entre a e b para que o problema


tenha soluo.
9) Dados os segmentos a e b (a sua escolha) resolva o sistema
(

x2 y 2 = a 2
x+y = b

Determine que relao deve existir entre a e b para que o problema


tenha soluo.
10) Dados a = 3 e b = 2,6 resolva a equao x 2 ax b2 = 0.
11) Dados os segmentos a e b (a sua escolha) construa x tal que
1
1
1
= 2 + 2.
x2
a
b
12) Construir o tringulo retngulo conhecendo a soma dos catetos
s = 8 cm e a altura relativa hipotenusa h = 2,6 cm.
13) Desenhe uma circunferncia e assinale um ponto P exterior. Trace
por P uma secante P AB de forma que se tenha P A = AB.

i
i

construcciones*ge
2009/8/12
i page 62
Estilo OBMEP

i
i

62

 CAP. 3: EXPRESSES ALGBRICAS

14) A mdia harmnica entre dois segmentos a e b o segmento h tal


2ab
que h =
. Desenhe os segmentos a = 4,8 cm e b = 2,6 cm, e
a+b
construa a mdia harmnica deles.
15) Considere um segmento AB e um ponto C interior (mais prximo
de B do que de A). Dizemos que AC o segmento ureo de AB
CB
AC
=
.
quando
AC
AB
(a) Desenhe um segmento AB qualquer e construa o seu segmento ureo.
AC
(b) Qual o valor da razo
?
AB
16) Desenhe uma circunferncia de 3 cm de raio e inscreva nela um
retngulo de 16 cm de permetro.
17) Desenhe uma circunferncia C e uma reta tangente t. Construa
um quadrado que tenha dois vrtices sobre t e dois vrtices sobre
C.
18) Construa o trapzio issceles circunscritvel sabendo que suas bases
medem 2,2 cm e 5,4 cm.
19) Desenhe um quadrado de qualquer tamanho. Construa um octgono regular cortando os cantos desse quadrado.
20) So dados dois pontos A e B de um mesmo lado de uma reta r.
Determine o ponto P da reta r de forma que o ngulo AP B seja
mximo.
21) So dados os pontos A e B e um segmento k. Construa o lugar
geomtrico dos pontos P tais que P A2 + P B 2 = k 2 .

i
i

construcciones*ge
2009/8/12
i page 63
Estilo OBMEP

i
i

N SEC. 3.7: CONSTRUES COM SEGMENTO UNITRIO

63

22) Dados os segmentos a e b, e o segmento unitrio u = 1 construa


x = ab.
23) Dados os segmentos a,b e c e o segmento unitrio u = 1 construa

x = abc.
24) Dado o segmento a, e o segmento unitrio u = 1, construa x =

25) (OBM) Dados os segmentos a e b construa x = 4 a4 + b4 .

4
a.

i
i

construcciones*ge
2009/8/12
i page 64
Estilo OBMEP

i
i

Captulo 4

Solues dos Exerccios


Propostos
Captulo 2 - Lugares Geomtricos

1)

4,50 cm

64

i
i

construcciones*ge
2009/8/12
i page 65
Estilo OBMEP

i
i

65
2)

O
3,20 cm

3)

i
i

construcciones*ge
2009/8/12
i page 66
Estilo OBMEP

i
i

66

 CAP. 4: SOLUES DOS EXERCCIOS PROPOSTOS

4)
76,3

5,20 cm

6,50 cm
4,50 cm

5)

i
i

construcciones*ge
2009/8/12
i page 67
Estilo OBMEP

i
i

67

6)

A'

4,80 cm

50,0

6,00 cm

7)

i
i

construcciones*ge
2009/8/12
i page 68
Estilo OBMEP

i
i

68

 CAP. 4: SOLUES DOS EXERCCIOS PROPOSTOS

8)

9)

i
i

construcciones*ge
2009/8/12
i page 69
Estilo OBMEP

i
i

69

10)

4,00 cm

5,00 cm

A'

60,0

11)

5,40 cm

6,70 cm

7,00 cm

i
i

construcciones*ge
2009/8/12
i page 70
Estilo OBMEP

i
i

70

 CAP. 4: SOLUES DOS EXERCCIOS PROPOSTOS

12)

13)

i
i

construcciones*ge
2009/8/12
i page 71
Estilo OBMEP

i
i

71
14)

6,00 cm

45,0

A
8,10 cm

15)
A

29,0

38,0
11,00 cm

i
i

construcciones*ge
2009/8/12
i page 72
Estilo OBMEP

i
i

72

 CAP. 4: SOLUES DOS EXERCCIOS PROPOSTOS

16)

6,00 cm

5,00 cm

4,00 cm

B
8,60 cm

17)
A

Q
B'
B

i
i

construcciones*ge
2009/8/12
i page 73
Estilo OBMEP

i
i

73
18)

C
5,00 cm

3,00 cm

5,80 cm

19)
m/2

m/2
B
P
M
A

i
i

construcciones*ge
2009/8/12
i page 74
Estilo OBMEP

i
i

74

 CAP. 4: SOLUES DOS EXERCCIOS PROPOSTOS

20)

21)

i
i

construcciones*ge
2009/8/12
i page 75
Estilo OBMEP

i
i

75
22)

23)

K
A

R
B

i
i

construcciones*ge
2009/8/12
i page 76
Estilo OBMEP

i
i

76

 CAP. 4: SOLUES DOS EXERCCIOS PROPOSTOS

24)

A
N

P
D
Q
C

25)

AB
A

i
i

construcciones*ge
2009/8/12
i page 77
Estilo OBMEP

i
i

77

Captulo 3 - Expresses Algbricas

1)

de
a
=
bc
x
de
c
e
=y =
c
d
y
y
a
=
b
x

d
y
y

b
x

2)

b
x
b
a
b

i
i

construcciones*ge
2009/8/12
i page 78
Estilo OBMEP

i
i

78

 CAP. 4: SOLUES DOS EXERCCIOS PROPOSTOS

3)

x
a

4)

1
5,8

5)

b
a
=
a
x

i
i

construcciones*ge
2009/8/12
i page 79
Estilo OBMEP

i
i

79

6)

b
1
=
a
x

a
x

7) Seja y tal que y 2 = bc. Seja r tal que r =


r2
d
r
Ento x =
ou = .
d
r
x

p
a2 + y 2 .

a
d
y

x=y+a
(y + a)y = b2
y 2 + ay b2 = 0.
A raiz positiva
y=

b
8)

a +

p
a2 + (2b)2
.
2

a
x

i
i

construcciones*ge
2009/8/12
i page 80
Estilo OBMEP

i
i

80

 CAP. 4: SOLUES DOS EXERCCIOS PROPOSTOS

9) (x + y)(x y) = a2
xy =

a2
=c
b

a
c

(construo auxiliar)

b
x
(

10)

x+y =b
b+c
bc
e y=
.
= x =
2
2
xy =c

x=

a+

p
a2 + (2b)2
.
2

b
r
b
a

x = 4,5

i
i

construcciones*ge
2009/8/12
i page 81
Estilo OBMEP

i
i

81

11)
1
a2 + b 2
=
2
x
a2 b2

1
a2 + b2
=
.
x
ab

Construindo r = a2 + b2
temos:

b
r
= .
a
x

12) b + c = s
b2 + c2 + 2bc = s2
a2 + 2ah = s2
a2 + 2ah s2 = 0
a
=

2h + 4h2 + 4s2
2
a
=
h +

2
2
h +s

b
x

s =8
h = 2,6

4,94 cm
3,06 cm

a
s

13) Seja r o raio e P O = d. Usando o conceito de potncia de um


ponto em relao a uma circunferncia temos:
x 2x = d2 r 2

x 2 = d2 r 2

i
i

construcciones*ge
2009/8/12
i page 82
Estilo OBMEP

i
i

82

 CAP. 4: SOLUES DOS EXERCCIOS PROPOSTOS

p
d2 r 2 = t

t 2
x=
.
2

B
x

A
x
P

x
T

14)

2ab
a+b
2b
a+b
=
a
h

a = 4,8
b = 2,6

h=

a
h

i
i

construcciones*ge
2009/8/12
i page 83
Estilo OBMEP

i
i

83
15) AB = a , AC = x
ax
x
=
x
a

a/2

x2 + ax a2 = 0

a( 5 1)
x=
2

a/2
16) Seja 2r = 6 = d (dimetro).
Seja a + b = 8 = p (semipermetro).
(
a2 + b 2 = d 2
a+b=p

a/2

d=6
p=8

Temos que b = pa a2 +(pa)2 = d2 2a2 2pa+p2 d2 = 0.


Seja p2 d2 = c2 .
Assim,
p
p
2p 4p2 8c2
p p2 2c2
2
2
2a 2pa+c = 0 a =
a=
.
4
2

c. 2

a
c. 2

a
r =3

2,59 cm
5,41 cm

i
i

construcciones*ge
2009/8/12
i page 84
Estilo OBMEP

i
i

84

 CAP. 4: SOLUES DOS EXERCCIOS PROPOSTOS

17) Seja 2x o lado do quadrado.


Trace pelo centro o
dimetro que passa pelo
ponto de tangncia e seja
r o raio da circunferncia
dada.
r 2 = x2 + (2x r)2
4r
3r
x=
2x r = .
5
5

18) Se as bases de um
trapzio issceles circunscritvel medem a e

b, sua altura h = ab.

2,2
5,4

h
2,2

5,4

19) Seja a o lado do quadrado.


Seja x o tamanho do cateto de
cada tringulo.

a 2
x+x 2+x = a x = a
.
2

i
i

construcciones*ge
2009/8/12
i page 85
Estilo OBMEP

i
i

85
20) Existe uma circunferncia que
passa por A e B e tangente
a r. O ponto de tangncia
o ponto P . Para construir,
seja C o ponto onde a reta AB
encontra r. Usando potncia
temos P T = P A P B. Uma
construo est a seguir.

21)

22)

1
a
=
b
x

i
i

construcciones*ge
2009/8/12
i page 86
Estilo OBMEP

i
i

86

 CAP. 4: SOLUES DOS EXERCCIOS PROPOSTOS

23) x2 = abc
y = bc

1
b
=
c
y

x2 = ay.

24) Seja y = a.

Ento x = y.

x
y

25) As figuras mostram as construes dos segmentos: a 2 = m,

b2 = n, t = m2 + n2 = a4 + b4 e x = t = 4 a4 + b4 .

i
i

construcciones*ge
2009/8/12
i page 87
Estilo OBMEP

i
i

87
Foi utilizado um segmento unitrio, mas como a expresso homognea, o segmento x no depende do segmento unitrio.

m
n
a

n
m

i
i

principal
2010/4/20
i page 1
Estilo OBMEP

i
i

Oficina de Dobraduras
Mario Jorge Dias Carneiro
Michel Spira

i
i

principal
2010/4/20
i page 2
Estilo OBMEP

i
i

Texto j revisado pela nova ortografia.

i
i

principal
2010/4/20
i page i
Estilo OBMEP

i
i

Sumrio
Apresentao

iii

Parte I

Parte II

15

Referncias Bibliogrficas

37

i
i

principal
2010/4/20
i page ii
Estilo OBMEP

i
i

ii

SUMRIO

i
i

principal
2010/4/20
i page iii
Estilo OBMEP

i
i

Apresentao
O uso de dobraduras no ensino de geometria est tornando-se cada
vez mais reconhecido como um instrumento pedaggico interessante
e muitas vezes eficaz, tanto pelo seu carter ldico quanto pela sensao de descoberta que muitas vezes provoca. possvel encontrar
vrios locais na Internet, roteiros para oficinas e comentrios sobre as
justificativas e demonstraes.
O objetivo dessas notas apresentar alguns conceitos e fatos geomtricos para os bolsistas da OBMEP, especialmente os dos nveis I
e II, motivando-os a aprofund-los posteriormente.
Esse roteiro destinado aos professores orientadores e apresentado em duas partes: Na primeira, so sugeridos e ilustrados alguns
procedimentos, sem haver a preocupao de justificativa. Na segunda
parte, fazemos uma discusso sobre a geometria das dobraduras e
apresentamos algumas justificativas e problemas.
A inteno no apenas que o aluno siga as instrues e executeas, mas que experimente e reflita e, sempre que possvel, chegue s
suas prprias concluses verbalizando-as para os seus colegas.
O professor orientador tem um papel importante no s em aproiii

i
i

principal
2010/4/20
i page iv
Estilo OBMEP

i
i

iv
fundar as discusses, trazendo novas situaes e problemas, mas tambm apresentando fatos geomtricos e conceitos que possam ser explorados nas justificativas das construes.
As construes aqui desenvolvidas so baseadas numa oficina do
Projeto Olimpada Mineira de Matemtica 2007, apresentada pela
equipe de bolsistas do projeto de extenso do Departamento de Matemtica da UFMG, orientada pelos professores Michel Spira e Mrio
Jorge Dias Carneiro.

i
i

principal
2010/4/20
i page 1
Estilo OBMEP

i
i

Parte I

i
i

principal
2010/4/20
i page 2
Estilo OBMEP

i
i

principal
2010/4/20
i page 3
Estilo OBMEP

i
i

Perpendicular que passa por um ponto fora da reta


1. Usando uma dobra que passa em A, faa uma dobradura que
leve a reta sobre si mesma.
2. Desdobre.
3. Como obter a perpendicular no caso em que A pertence reta?
A

A Mediatriz
1. Faa uma dobradura de modo que o ponto A se sobreponha ao
ponto B.
2. Desdobre.
x

x
A
A B

i
i

principal
2010/4/20
i page 4
Estilo OBMEP

i
i

A Bissetriz
1. Dobre uma das semirretas do ngulo de modo que se sobreponha
sobre o outro.
2. Desdobre.

Alturas de Tringulos e Ortocentro


1. No caso de tringulo obtusngulo, use dobraduras para prolongar cada um dos lados.
2. Utilize a construo da perpendicular passando por um ponto
para obter as alturas relativas aos lados AB, AC e BC, respectivamente.
3. Desdobre.
4. Faa a construo para outros tringulos.

i
i

principal
2010/4/20
i page 5
Estilo OBMEP

i
i

5
A

Tringulo Equiltero
1. O lado do tringulo igual ao lado menor da folha de papel,
denote por A e B os extremos do segmento.
2. Dobre a folha ao meio de modo a encontrar a mediatriz do
segmento AB.
3. Dobre a folha de modo que o ponto refletido de B encontre a
mediatriz (construda em 2). Marque esse ponto C.
4. Dobre os segmentos AC e AB para completar o tringulo.
5. Desdobre.

i
i

principal
2010/4/20
i page 6
Estilo OBMEP

i
i

B C

i
i

principal
2010/4/20
i page 7
Estilo OBMEP

i
i

A Razo urea ou o Nmero de Ouro


1. Divida uma folha de papel quadrada ao meio (como obter um
quadrado?).
2. Faa uma dobradura ao longo de um segmento AF que liga um
vrtice A da folha ao ponto F , extremidade direita do segmento
mdio que encontra-se sobre a reta vertical oposta.
3. Use uma dobradura com dobra contendo F , para levar o vrtice
B at o segmento AF . Marque esse ponto C.
4. Use uma dobradura com dobra contendo A, para levar o ponto
C at o segmento AB. Marque o ponto P .
5. A razo ente AP e AB igual razo entre P B e AP que
igual ao nmero de ouro.

i
i

principal
2010/4/20
i page 8
Estilo OBMEP

i
i

C
B
A

C
P

i
i

principal
2010/4/20
i page 9
Estilo OBMEP

i
i

Pentgono Regular
1. Numa folha quadrada construa o ponto P , tal que
P B/AP = razo urea.
2. Divida o segmento P B ao meio e marque o ponto mdio R.
3. Dobre a folha ao meio e marque A refletido de R igual a S.
4. SR o lado do pentgono e os prximos passos servem para
obter os outros vrtices.
5. Usando uma dobra que passa em S, reflita o ponto R sobre o
lado esquerdo da folha determinando o ponto T .
6. Proceda analogamente com o lado direito da folha refletindo
o vrtice T sobre um ponto U . Este ponto pode ser obtido
tambm usando a mediatriz do segmento AB como dobra e
refletindo T sobre o lado direito da folha.
7. Finalmente, usando uma dobra que contm o ponto T reflita o
ponto U sobre um ponto V na mediatriz de AB. Os vrtices do
pentgono so SRU V T .

i
i

principal
2010/4/20
i page 10
Estilo OBMEP

i
i

10

PB R

T R

T U

SR

i
i

principal
2010/4/20
i page 11
Estilo OBMEP

i
i

11

S V
T

i
i

principal
2010/4/20
i page 12
Estilo OBMEP

i
i

12

Trisseco de um ngulo Agudo


1. Marque um ponto qualquer C sobre a perpendicular semirreta
AS que passa pelo vrtice A.
2. Marque a mediatriz n de AC, e o ponto mdio B.
3. Usando uma reta conveniente, dobre o ponto C sobre o ponto
C que est na semirreta AR e simultaneamente leve o ponto A
ao ponto A sobre a mediatriz n (traada no item 2).
4. Usando como dobra a reta que passa em A e A , faa uma dobra
e denote a imagem da semirreta AS por AS .
5. Desdobre. As semirretas AS e AA dividem o ngulo RAS em
trs partes iguais.

C
B
A

i
i

principal
2010/4/20
i page 13
Estilo OBMEP

i
i

Parte II

i
i

principal
2010/4/20
i page 14
Estilo OBMEP

i
i

principal
2010/4/20
i page 15
Estilo OBMEP

i
i

Introduo
Este um roteiro de atividades que envolvem o uso de dobraduras
para estudar problemas geomtricos.
Usaremos a expresso fazer uma dobradura como o ato de dobrar,
uma transformao do plano; o termo dobra ou vinco usado para a
marca no papel resultante da dobradura.
Na geometria das dobraduras, dobrar significa ao mesmo tempo
sobrepor pontos e obter a reta de dobra, que o lugar dos pontos que
permanecem fixos nesta transformao.
Pode-se observar experimentalmente que pontos sobrepostos (ou
seja que coincidem no processo de dobra) equidistam da dobra. Tal
fato essencial na justificativa das construes geomtricas que utilizam dobraduras.
Desse modo, associada a uma dobradura, temos a ideia de simetria
em relao dobra. Podemos imaginar que a dobra um espelho e
que pontos equidistantes correspondem a imagens refletidas (ou virtuais).

15

i
i

principal
2010/4/20
i page 16
Estilo OBMEP

i
i

16
Definio. Seja r uma reta. Chama-se uma reflexo com respeito
reta r transformao do plano que leva um ponto P ao ponto Q,
simtrico de P em relao reta, isto , tal que distncia de P
reta r igual distncia de Q a r.
Observe que os pontos sobre a reta r permanecem fixos pela reflexo. Sendo r chamado eixo da reflexo.
Deste modo, aps uma dobradura, obtemos uma reta r tal que,
pontos superpostos so exatamente aqueles que se correspondem por
uma reflexo.

Propriedade. Uma reflexo preserva comprimento de segmentos e a


medida de ngulos.

A prova desta propriedade um exerccio de congruncia de tringulos e pode ser feita comeando com o caso particular em que um
dos pontos est situado sobre a reta de reflexo.
Esta uma propriedade essencial na qual esto baseadas as justificativas das construes geomtricas que utilizam dobraduras.
Uma figura plana possui uma simetria por reflexo se existe uma
reta r tal que a reflexo da figura com respeito a r resulta na prpria
figura. Em outras palavras, uma figura possui uma simetria por reflexo se possvel encontrar um eixo de simetria com respeito ao
qual a figura pode ser refletida.
Vrias figuras planas apresentam este tipo de simetria. Por exemplo, num quadrado possvel encontrar 4 retas (ou eixos) em relao

i
i

principal
2010/4/20
i page 17
Estilo OBMEP

i
i

17
as quais, se fizermos uma reflexo os pontos do quadrado so associados a pontos do quadrado.

Outros exemplos desse tipo de simetria, podem ser encontrados


na natureza, na arquitetura e at na msica.
Os alunos podem verificar simetrias por reflexo em vrias situaes, usando fotografias, por exemplo.
Na discusso sobre simetria podemos propor aos alunos que encontrem todos os eixos de simetria das figuras abaixo:

possvel desenvolver uma abordagem rigorosa da geometria das


dobraduras e dar um tratamento abstrato s operaes e relacion-las
com os axiomas da geometria euclidiana (veja por exemplo [1] ou [2]).
No pretendemos fazer isso aqui, preferimos dar um tratamento
intuitivo para que se possa experimentar, perceber e aprender, por
meio da manipulao direta, alguns fatos geomtricos.

i
i

principal
2010/4/20
i page 18
Estilo OBMEP

i
i

18
O objetivo aprender algumas tcnicas, justificar algumas construes geomtricas e divertir-se!
Evidentemente as justificativas devem ser usadas de acordo com
o nvel de conhecimento da turma.
A ideia da oficina desenvolver um dilogo com os alunos de
modo a introduzir novos conceitos, primeiramente de maneira informal, e conforme a situao e interesse, aprofund-los. O professor
pode tambm sugerir outras construes e problemas a partir das
aqui apresentadas.
Algumas das provas usam os casos de congruncias e conceitos
que para muitos alunos sero novos; outras, utilizam o Teorema de
Pitgoras e semelhana de tringulos. Este um bom momento para
rever ou motivar os alunos para o estudo desses tpicos.

Preliminares
Iniciamos com algumas construes simples.
Pedimos que os alunos verifiquem experimentalmente que:
1) Dados dois pontos distintos numa folha de papel, existe
uma dobra (ou vinco) que os contm.
2) Dados dois pontos distintos, P1 e P2 numa folha de papel,
existe uma nica dobradura que sobrepe P1 sobre P2 .
3) Dadas duas dobras r1 e r2 que se intersectam em um ponto
P existe uma dobradura que sobrepe r1 sobre r2 .

i
i

principal
2010/4/20
i page 19
Estilo OBMEP

i
i

19
4) Dados dois pontos distintos P1 e P2 e duas dobras r1 e r2 ,
que se intersectam num ponto P , existe uma dobradura que
leva o ponto P1 sobre r1 e P2 sobre r2 .
5) Dados dois pontos distintos P1 e P2 e uma dobra r1 , existe
uma dobradura cuja dobra passa em P2 e que leva o ponto
P1 sobre a dobra r1 .

Outras construes
Seguindo o roteiro de atividades, apresentamos algumas justificativas para as construes. Algumas dessas justificativas utilizam fatos,
tais como os casos de congruncia de tringulos ou o Teorema de Pitgoras, que provavelmente no so familiares aos bolsistas do nvel I.
Portanto, o uso ou no desses argumentos fica a critrio do professor
orientador .
6) Reta perpendicular a uma reta r passando por um ponto
P.
Ao dobrarmos uma folha de papel duas vezes, superpondo os lados,
obtemos no centro quatro ngulos retos. A construo segue esta
ideia.
Sugerimos que se faa inicialmente o caso P r.
Justificativa: A reflexo obtida envia r sobre r. Como uma reflexo
preserva ngulos os dois ngulos obtidos possuem a mesma medida.
Como a soma desses ngulos igual a 180 graus, obtm-se que os
ngulos so retos.

i
i

principal
2010/4/20
i page 20
Estilo OBMEP

i
i

20

Observao. A partir dessa construo, propor a construo da reta


paralela a uma reta r dada passando por um ponto P fora de r.
7) Mediatriz de um segmento AB
Justificativa: Sejam m a reta obtida na construo e O a sua interseco com o segmento AB. De acordo com o item 6) a reta m
perpendicular ao segmento AB. Pela propriedade da reflexo, temos
de |AO| = |OB|.
Se Q m um ponto sobre a reta obtida, ento os tringulos
QOA e QOB so congruentes (caso LAL).
Segue-se que |QA| = |QB|. Ou seja Q equidistante de A e de
B, e portanto, est na mediatriz.

i
i

principal
2010/4/20
i page 21
Estilo OBMEP

i
i

21
Q

8) Bissetriz de um ngulo
Justificativa: Usa-se novamente o caso (LAL) de congruncia de
tringulos.
Como refletimos uma semirreta sobre a outra, fixado um ponto Q
sobre um dos raios e a sua imagem Q pela reflexo na reta obtida,
ento pela propriedade da reflexo obtemos dois tringulos retngulos
congruentes. De modo que os ngulos correspondentes possuem a
mesma medida.
Isto significa que a reta obtida divide o ngulo dado em dois ngulos iguais (bissetriz).

i
i

principal
2010/4/20
i page 22
Estilo OBMEP

i
i

22
9) Tringulo equiltero
Justificativa: Observe que iniciamos a construo obtendo a mediatriz da base AB.
Em seguida obtivemos uma reta em relao qual refletimos o
ponto B sobre a mediatriz. Esse ponto refletido denotamos por C.
Pela propriedade de reflexo, |AB| = |AC|. Como C pertence
mediatriz do segmento AB, temos:
|CB| = |AC|.
Veja a figura:

Logo, os pontos A, B e C so vrtices de um tringulo equiltero,


que obtido simplesmente dobrando-se o papel para traar os segmentos AC e CB.
Aps construdo o tringulo equiltero, podemos propor que os
alunos sobreponham os lados, de modo a traar as bissetrizes. Se
a construo for feita com cuidado, ser possvel notar a interseco

i
i

principal
2010/4/20
i page 23
Estilo OBMEP

i
i

23
dessas retas num ponto O. Podemos ento solicitar que os alunos
redijam o que observaram e provem que este ponto equidistante dos
lados.
Pergunta, ser que isso vlido para outros tringulos?

10) Hexgono regular


Justificativa: Na folha dobrada duas
vezes, primeiro no sentido vertical e depois no sentido horizontal, construmos um
trigulo equiltero. A base AB do tringulo
encontra-se sobre a segunda dobra.

A reta paralela base AB, passando por


C tem comprimento igual a |AB|. O ponto
C o ponto mdio desse segmento. Logo
ao desdobrarmos uma vez a folha, obtemos
trs tringulos equilteros congruentes. Finalmente, ao desdobrarmos mais uma vez
a folha, obtemos seis tringulos equilteros
que formam um hexgono de lado |AB|.

i
i

principal
2010/4/20
i page 24
Estilo OBMEP

i
i

24
11) Razo urea
Dizemos que um ponto H divide um segmento AB numa razo
urea se
|AH|
|AB|
=
.
|AH|
|HB|
Se |AH| = m e |HB| = n ento
Se denotamos por =

m+n
m
= .
m
n

m
, ento temos a seguinte equao:
n
1+

1
= .

2
Ou seja
a raiz positiva da equao x x 1 = 0, isto ,
1+ 5
1
=
(a outra raiz ).
2

O valor de irrelevante para o que feito nesta oficina e no h


necessidade de resolver a equao, entretanto, os alunos podem ficar
curiosos sobre o nome. Nesse caso vale a pena explorar algumas propriedades tanto aritmticas quanto geomtricas, como por exemplo,
a seguinte:

A reflexo de H com respeito ao ponto mdio de AB define um


ponto S tal que |AS| = |HB|.
O ponto S divide o segmento AH na razo urea pois
|AH|
|AH|
=
= .
|AS|
|HB|

i
i

principal
2010/4/20
i page 25
Estilo OBMEP

i
i

25
Vejamos como justificar a construo da razo urea:
Podemos usar o Teorema de Pitgoras.
Iniciamos com uma folha quadrada e a dividimos ao meio. Denotamos por AB o segmento correspondente ao lado inferior da folha.
Seja F o ponto mdio do lado direito.
Note que pela propriedade da reflexo, se B a imagem da reflexo de B sobre AF , ento
|B F | = |BF | =

|AB|
.
2

Pelo Teorema de Pitgoras: |AF |2 = |AB|2 + |F B|2 .

|AB|2
5
= |AB|2 .
4
4

5
51
|AB|

Portanto |AB | = |AF ||B F | =


|AB|
=
|AB|.
2
2
2

1+ 5
Ou seja, |AB| =
|AB |.
2
Na etapa final a reflexo foi usada simplesmente para trazer comprimento |AB | para o segmento AB, determinando assim o ponto
H.

1+ 5
|AB|
=
= seja o nmero de ouro ou razo
De modo que
|AH|
2
urea.
Ou |AF |2 = |AB|2 +

i
i

principal
2010/4/20
i page 26
Estilo OBMEP

i
i

26
Veja a figura:

Veremos a seguir como aparece a razo urea no pentgono regular.


12) Pentgono regular
Antes da construo do pentgono regular, importante fazer
alguns desenhos para que sejam percebidas (ou provadas) algumas
propriedades. Por exemplo, traando-se as diagonais do pentgono,
obtemos um polgono estrelado que permite estabelecer a relao entre
o comprimento da diagonal e o lado do pentgono:
Proposio. Se d a diagonal do pentgono regular e L o seu lado,
d
ento
= (a razo urea).
L
Usando este fato, que ser provado mais abaixo, podemos justificar
a construo do pentgono por meio de dobraduras. Nessa construo,
a diagonal do pentgono igual largura da folha de papel. Para
acompanhar a justificativa, volte para a primeira parte e siga os passos
indicados.
Se |AB| a largura da folha de papel e P um ponto tal que
|AB|
= ento segue da construo que |AP | = |SR|.
|AP |

i
i

principal
2010/4/20
i page 27
Estilo OBMEP

i
i

27
|AB|
= ou seja, |SR| o lado do pentgono cuja
|SR|
diagonal igual a |AB|.
Portanto,

Os demais passos so justificados pela propriedade da reflexo.


A primeira reflexo leva o vrtice R em um vrtice T do pentgono situado na borda lateral esquerda da folha de papel e a segunda
reflexo ou dobradura em relao a mediatriz de AB, leva o vrtice
T sobre um vrtice U do pentgono situado na borda lateral direita
da folha.
O segmento T U uma diagonal do pentgono.
O quinto vrtice, V , est na mediatriz de AB e foi construdo de
modo que kV U k = kSRk. Concluindo a justificativa da construo.
Passemos prova da Proposio:
Um pentgono regular de vrtices OS RSR .
O

i
i

principal
2010/4/20
i page 28
Estilo OBMEP

i
i

28
Tracemos a circunferncia circunscrita ao pentgono, e as diagonais R S , OS, OR. Denotemos F = OS R S e E = OR R S .
Sabe-se que numa circunferncia , ngulos inscritos que subtendem arcos iguais so congruentes.
Logo, OR S
= SOR
= R OR pois os arcos R O,SR e RR
so congruentes.
Portanto, o tringulo R EO issceles, com |ER | = |EO|. De
modo anlogo conclui-se que S F O issceles|F S | = |F O|.
Pela simetria da figura, temos |OE| = |OF |.
De fato,

OF R = OES (por qu?), o tringulo OEF issceles e


|OE| = |OF |.
Alm disso, segue da propriedade dos ngulos externos de um
tringulo, que OEF = 2R OE e como OEF = OF E, o tringulo OR F issceles e semelhante ao tringulo OEF (caso AAA).
Portanto |R E| + |EF | = |R O| = L (lado do pentgono).
Agora, usando a semelhana dos tringulos OEF e OR F
obtemos a razo urea do seguinte modo:
Se m = |R E| e n = |EF | ento
m+n
m
=
= .
m
n
Finalmente, observe que m + n = L e que a diagonal d mede
2m + n.

i
i

principal
2010/4/20
i page 29
Estilo OBMEP

i
i

29
De modo que
d
2m + n
2 + 1

1
=
=
=1+
= 1 + = .
L
m+n
+1
1+

Como queramos demonstrar.


13) Trisseco de um ngulo agudo (H. Abe)
Essa construo aparece como problema sugerido no nmero 65 da
Revista do Professor de Matemtica, 2007. A soluo est no nmero
66 da RPM, segundo quadrimestre de 2008, pgina 47.
Talvez seja interessante iniciar fazendo a construo para o caso
do ngulo reto.
Evidentemente h modos muito mais simples de obter um ngulo de 30 graus, e podemos propor como exerccio de aplicao de
construes anteriores (Bissetriz e Tringulo Equiltero) mas o nosso
objetivo entender a construo neste caso particular. Alis, este
o princpio geral a ser adotado nessas atividade, sempre que possvel
partir de um caso mais simples e tentar generalizar a construo.
Seja CAR um ngulo reto.
Obtenha n a mediatriz do segmento AC. Seja B = AC n o
ponto mdio de AC.
A reflexo do ponto A sobre a reta n por uma dobra que passa
por C, define um ponto A que satisfaz:
|AC| = |A C|.

i
i

principal
2010/4/20
i page 30
Estilo OBMEP

i
i

30
E pela propriedade dos ngulos alternos internos em retas paralelas, temos: AA B = A AR.

A
m

Como |CB| = |AB|, os tringulos CBA e ABA so congruentes (caso LAL).


Portanto CA B = AA B.
Alm disso, por construo, o tringulo ACA issceles, seguese que CAA = AA C que mede o dobro do ngulo A AR. Por
conseguinte, o segmento AA trissecciona o ngulo reto CAR.
Passemos agora justificativa da trisseco de um ngulo agudo
RAS:

Iniciamos com a obteno da perpendicular AC semirreta AS e


de sua mediatriz que denotamos por n.
Escolhendo uma dobra conveniente m, refletimos simultaneamente

A sobre n e C sobre a semirreta AR.


Denotemos por A , B e C as imagens dos pontos A, B e C por
essa primeira dobradura.

i
i

principal
2010/4/20
i page 31
Estilo OBMEP

i
i

31
Sejam O e O os pontos de interseo da dobra m com as semirretas

AS e n respectivamente.
Veja a figura:

R
C
C
B

A
n

O
m

Pela propriedade da reflexo


|A C | = |AC|, |A B | = |AB| = |BC| = |B C |.
Seja n1 a imagem de n. claro que n1 passa por O e por B , o
problema que trataremos agora, verificar que os pontos A, O e B
so colineares.
De fato, como BB k AA e |A B | = |AB|, temos um trapzio
issceles de vrtices ABB A .
A dobra m mediatriz das bases AA e BB , portanto, as diagonais do trapzio se intersectam em m. Ou seja, AB A B = O , o
que implica que A, O e B so colineares.

i
i

principal
2010/4/20
i page 32
Estilo OBMEP

i
i

32
Conforme a figura:
R
C
C
B
A

No prximo passo, observe que BA paralelo a AO.


Portanto, pela propriedade dos ngulos alternos internos, temos:
AA B = A AO.
Vamos provar agora que C AB = B AA = A AS.
Como o tringulo AO A issceles, segue-se que O AA =
AA O .
Por construo, AB BO .
Como a dobradura preserva a medida de ngulos, temos que
AB A B .
Finalmente, temos: |A B | = |AB| = |BC| = |B C |. Isto ,
AA C issceles.
Conclumos assim que C AB = B AA (= BA A = A AO)
a justificativa da trisseco.

i
i

principal
2010/4/20
i page 33
Estilo OBMEP

i
i

33
Para finalizar:
Alguns teoremas podem ser explorados com dobraduras. Sugerimos que aps vrios experimentos os alunos sejam convidados a
enunciar o respectivo resultado.
Vale observar, entretanto, que por causa da impreciso das dobras
alguns resultados esperados no ocorram.
14) A soma dos ngulos internos de um tringulo, dobre o
tringulo de modo que um vrtice se sobreponha ao lado oposto.
15) Ponto de encontro das medianas de um tringulo.
16) Ponto de encontro das alturas de um tringulo.
17) A base mdia de um tringulo, ou seja obtenha o segmento
que une os pontos mdios de dois lados de um tringulo e verifique
que este segmento paralelo ao terceiro lado.
18) Problema: Dada uma reta r e dois pontos A e B do mesmo
semiplano em relao a r encontrar o ponto X na reta tal que a soma
comprimento dos segmentos |AX| + |XB| seja a menor possvel.

i
i

principal
2010/4/20
i page 34
Estilo OBMEP

i
i

34

i
i

principal
2010/4/20
i page 35
Estilo OBMEP

i
i

Referncias Bibliogrficas
[1] MARTIN, George E. Geometric Constructions. Undergraduate
Texts of Mathematics. Nova York: Springer Verlag, 1998.
[2] www.cut-the-knot.org/pythagoras/ Paperfolding pgina de autoria de Alexander Bogomolny.

35

i
i

principal
2009/10/28
i page 1
Estilo OBMEP

i
i

Atividades de Contagem a
partir da Criptografia
Pedro Luiz Malagutti

i
i

principal
2009/10/28
i page 2
Estilo OBMEP

i
i

principal
2009/10/28
i page i
Estilo OBMEP

i
i

Sumrio
1 Mensagens Secretas Obtidas por Substituio
Introduo ao Estudo de Permutaes

1.1

Criptografia de Jlio Csar . . . . . . . . . . . . . . .

1.2

Construo de Aparatos que Ajudam a Criptografar .

1.3

Princpios de Contagem em Criptografia . . . . . . . .

15

1.4

Quantas Maneiras Diferentes de Criptografar Podemos


Construir? . . . . . . . . . . . . . . . . . . . . . . . . .

17

Como Quebrar o Cdigo de Jlio Csar . . . . . . . .

21

1.5

2 A Escrita Braille e o Cdigo Binrio


Introduo ao Estudo de Combinaes
2.1

30

Explorando Conceitos Matemticos com a Linguagem


Braille . . . . . . . . . . . . . . . . . . . . . . . . . . .

44

2.2

Combinaes Matemticas . . . . . . . . . . . . . . . .

48

2.3

As Combinaes e a Linguagem Braille . . . . . . . . .

51

2.4

Matemticos com Problemas Visuais . . . . . . . . . .

54

i
i

principal
2009/10/28
i page ii
Estilo OBMEP

i
i

ii

SUMRIO

2.5

O Sistema Binrio . . . . . . . . . . . . . . . . . . . .

56

i
i

principal
2009/10/28
i page 1
Estilo OBMEP

i
i

Captulo 1

Mensagens Secretas Obtidas


por Substituio
Introduo ao Estudo de
Permutaes
Enviar mensagens secretas uma tarefa muito antiga; ela nasceu
com a diplomacia e com as transaes militares. Hoje em dia, entretanto, com o advento da comunicao eletrnica, muitas atividades
essenciais dependem do sigilo na troca de mensagens, principalmente
aquelas que envolvem transaes financeiras e uso seguro da Internet.
A cincia que estuda sistemas de envio e recepo de mensagens
secretas chama-se CRIPTOLOGIA. Simplificadamente, temos o seguinte diagrama:

i
i

principal
2009/10/28
i page 2
Estilo OBMEP

i
i

CAP.1: MENSAGENS SECRETAS OBTIDAS POR SUBSTITUIO

Codificao
Mensagem
original

Decifrao

Mensagem
secreta

Mensagem
secreta
enviada

Mensagem
secreta
recuperada

Espionagem

Nosso objetivo apresentar atividades com criptografia atravs de


aparatos que possam efetivamente ser construdos com materiais simples (papel, palito de dente, clipe, furador de papel, cola e tesoura)
para explorar alguns aspectos matemticos destas construes, principalmente os ligados contagem.

1.1

Criptografia de Jlio Csar

Um dos primeiros sistemas de criptografia conhecido foi elaborado


pelo general Jlio Csar, no Imprio Romano. Jlio Csar substituiu
cada letra, pela terceira letra que a segue no alfabeto.

A
D

B
E

C
F

D
G

E
H

F
I

Q
T

R
U

S
V

T
W

U
X

V
Y

G
J
W
Z

H
K
X
A

I
L

J
M
Y
B

K
N

L
O

M
P

N
Q

O
R

P
S

Z
C

Segundo este sistema, a palavra MATEMTICA passa a ser


PDWHPDWLFD.

i
i

principal
2009/10/28
i page 3
Estilo OBMEP

i
i

SEC.1.2: CONSTRUO DE APARATOS QUE AJUDAM A CRIPTOGRAFAR

Atividade 1: Decifre a mensagem:


OHJDO FRQVHJXL
Ao invs de caminhar 3 letras para frente, podemos andar um
outro nmero de letras e teremos um novo mtodo de cifrar mensagens. Este nmero chamado de chave ou senha do sistema criptogrfico; ele deve ser conhecido apenas por que envia a mensagem e
por quem a recebe.
Podemos tambm transformar letras em nmeros, segundo uma
ordem pr-estabelecida. Por exemplo:
A=0

B=1

C=2

D=3

E=4

F=5

G=6

H=7

I=8

J=9

K=10

L=11

M=12

N=13

O=14

P=15

Q=16

R=17

S=18

T=19

U=20

V=21

W=22

X=23

Y=24

Z=25

Deste modo, a letra codificada obtida da letra original, somandose 3 ao nmero correspondente. E se o resultado ultrapassar 25? Caso
isto ocorra, a letra codificada estar associada ao resto da diviso por
26 do nmero associado letra original somado com 3. Por exemplo,
a letra Y corresponde originalmente ao nmero 24, somando-se 3,
obteremos 24 + 3 = 27 e, dividindo 27 por 26, obteremos resto 1 que
corresponde letra B. Assim Y deve ser codificado por B.
Se um espio conhecer a chave (o nmero de letras que andamos
- no nosso exemplo igual a 3), poder facilmente decifrar uma mensagem interceptada, trocando cada letra pela terceira anterior. Mas,
no se conhecendo a chave, como decifrar mensagens criptografadas?
Pense um pouco a respeito disso.

i
i

principal
2009/10/28
i page 4
Estilo OBMEP

i
i

1.2

CAP.1: MENSAGENS SECRETAS OBTIDAS POR SUBSTITUIO

Construo de Aparatos que Ajudam a Criptografar

Vamos apresentar cinco aparatos simples para agilizar a criptografia no estilo de Jlio Csar:
as rguas deslizantes,
o quadrado de Vigenre1 ,
os crculos giratrios e
dois projetos: a lata de criptografar e o CD para criptografar.
Como veremos, so todos variaes simples de um mesmo tema.
Atividade 2: Recorte e monte as rguas deslizantes, conforme as instrues na pgina seguinte.

Blaise Vigenre foi um diplomata francs, estudioso de Criptografia, que viveu


no sculo XVI.

i
i

principal
2009/10/28
i page 5
Estilo OBMEP

i
i

SEC.1.2: CONSTRUO DE APARATOS QUE AJUDAM A CRIPTOGRAFAR

A B C D E F G H I J K L M N O P Q R S T U V W X Y Z

25
26
26
25

23
23

24
24

21
20
19
18
17
22
22
21
20
19 18 17

16
16

15
15

14
14

13
13

12
12

11
11

10
10

9
9

8
8

7
7

6
6

5
5

B
C
D
E

Instrues da Rgua

F
G
H

Recorte os dois retngulos com as letras. Corte na linha


pontilhada e encaixe a Segunda fita no corte pontilhado
da primeira. Veja como ficaro encaixadas:

I J
K
L
M
N
O
P
Q
R

Deslize a rgua interna para codificar, letra a letra, as


mensagens.

S
T
U
V
W
X
Y
Z

A
B
C
D
E
F
G

Recorte este
pequeno retngulo

H
I J
K
L
M
N
O
P
Q
R

Como se deve proceder para decodificar uma mensagem


criptografada?

S
T
U
V
W
X
Y
Z

Aviso: Esta pgina est no encarte que veio junto com a apostila.

i
i

principal
2009/10/28
i page 6
Estilo OBMEP

i
i

CAP.1: MENSAGENS SECRETAS OBTIDAS POR SUBSTITUIO

i
i

principal
2009/10/28
i page 7
Estilo OBMEP

i
i

SEC.1.2: CONSTRUO DE APARATOS QUE AJUDAM A CRIPTOGRAFAR

As diferentes posies que a rgua deslizante ocupa quando movimentada podem ser simultaneamente visualizadas no quadrado de
Vigenre:
A B C D E F G H I J K L M N O P Q R S T U V W X Y Z
B C D E F G H I J K L M N O P Q R S T U V W X Y Z A
C D E F G H I J K L M N O P Q R S T U V W X Y Z A B
D E F G H I J K L M N O P Q R S T U V W X Y Z A B C
E F G H I J K L M N O P Q R S T U V W X Y Z A B C D
F G H I J K L M N O P Q R S T U V W X Y Z A B C D E
G H I J K L M N O P Q R S T U V W X Y Z A B C D E F
H I J K L M N O P Q R S T U V W X Y Z A B C D E F G
I J K L M N O P Q R S T U V W X Y Z A B C D E F G H
J K L M N O P Q R S T U V W X Y Z A B C D E F G H I
K L M N O P Q R S T U V W X Y Z A B C D E F G H I J
L M N O P Q R S T U V W X Y Z A B C D E F G H I J K
M N O P Q R S T U V W X Y Z A B C D E F G H I J K L
N O P Q R S T U V W X Y Z A B C D E F G H I J K L M
O P Q R S T U V W X Y Z A B C D E F G H I J K L M N
P Q R S T U V W X Y Z A B C D E F G H I J K L M N O
Q R S T U V W X Y Z A B C D E F G H I J K L M N O P
R S T U V W X Y Z A B C D E F G H I J K L M N O P Q
S T U V W X Y Z A B C D E F G H I J K L M N O P Q R
T U V W X Y Z A B C D E F G H I J K L M N O P Q R S
U V W X Y Z A B C D E F G H I J K L M N O P Q R S T
V W X Y Z A B C D E F G H I J K L M N O P Q R S T U
W X Y Z A B C D E F G H I J K L M N O P Q R S T U V
X Y Z A B C D E F G H I J K L M N O P Q R S T U V W
Y Z A B C D E F G H I J K L M N O P Q R S T U V W X
Z A B C D E F G H I J K L M N O P Q R S T U V W X Y

Atividade 3: Utilizando a rgua deslizante ou o


quadrado de Vigenre, decifre a mensagem:
UHV JVUZLNBP KLJPMYHY UHKH
Leia em voz alta o que voc decifrou.

i
i

principal
2009/10/28
i page 8
Estilo OBMEP

i
i

CAP.1: MENSAGENS SECRETAS OBTIDAS POR SUBSTITUIO

Atividade 4: Recorte e monte os discos giratrios, conforme as instrues da prxima pgina. Nos espaos
vazios complete cada um deles com uma letra diferente
sua escolha para que voc tenha sua maneira de criptografar.

i
i

principal
2009/10/28
i page 9
Estilo OBMEP

i
i

17
12
13 14 15 16

18
3
19 20 21 22 2

M N O P Q R
S T U V W X

J
I
F G H
b

J K L

G H I

S T U V W

A B C D E F

Recorte as letras do alfabeto abaixo, embaralhe-as


e cole-as uma a uma nos
espaos vazios do crculo
maior.

M N O P

25
26

24

10

SEC.1.2: CONSTRUO DE APARATOS QUE AJUDAM A CRIPTOGRAFAR

11

Z A B C

Y Z

i
i

principal
2009/10/28
i page 10
Estilo OBMEP

i
i

10

CAP.1: MENSAGENS SECRETAS OBTIDAS POR SUBSTITUIO

i
i

principal
2009/10/28
i page 11
Estilo OBMEP

i
i

SEC.1.2: CONSTRUO DE APARATOS QUE AJUDAM A CRIPTOGRAFAR

11

Atividade 5:
Projetos prticos de criptografia

Lata de criptografar

CD para criptografar

Para confeccionar este aparato voc vai precisar de um CD que


no tenha mais uso e tambm de sua caixinha.
Reproduza e recorte o crculo e cole-o no CD. O CD deve ser
encaixado dentro da caixinha.
O quadrado com o furo no meio deve ser colocado na capa do CD.
Para fazer a mquina funcionar voc deve recortar na parte detrs
da caixinha dois pequenos retngulos, suficientes para introduzir os
dedos e girar o CD.

i
i

principal
2009/10/28
i page 12
Estilo OBMEP

i
i

12

CAP.1: MENSAGENS SECRETAS OBTIDAS POR SUBSTITUIO

Faa entalhes nestes


locais que permitam
colocar os dedos e girar
o CD.

i
i

principal
2009/10/28
i page 13
Estilo OBMEP

i
i

SEC.1.2: CONSTRUO DE APARATOS QUE AJUDAM A CRIPTOGRAFAR

13

Crculo para Criptografia

M N O P

F G H

Z A B C

J
H

T
U

Aviso: Use um CD
usado como molde
em uma folha de papel e copie a figura
ajustando o tamanho
para ser igual ao do
CD.

Recorte este
crculo central e
obtenha um quadrado
com furo no meio. Este
deve ser colocado na capa
do CD. Atravs deste
furo pode-se ver o
CD
girar.

S T U V W

Recorte
este crculo e cole
em um CD que j foi
descartado. Encaixe
o CD na posio
usual dentro da
caixinha.

X
Y

i
i

principal
2009/10/28
i page 14
Estilo OBMEP

i
i

14

CAP.1: MENSAGENS SECRETAS OBTIDAS POR SUBSTITUIO

i
i

principal
2009/10/28
i page 15
Estilo OBMEP

i
i

SEC.1.3: PRINCPIOS DE CONTAGEM EM CRIPTOGRAFIA

15

Atividade 6: Resolva a seguinte questo (OBMEP 2007)

1.3

Princpios de Contagem em Criptografia

Nos sistemas que seguem o princpio do de Jlio Csar, podemos


usar 25 chaves diferentes para obter codificaes diferentes, j que
o sistema com chave 0 (ou 26), no codifica nada. Nestes sistemas
o alfabeto codificado seguindo a ordem usual, apenas iniciando em
um lugar diferente. Se, entretanto, pudermos alterar a ordem, obteremos um enorme nmero de maneiras de criptografar. Vejamos alguns
exemplos:

i
i

principal
2009/10/28
i page 16
Estilo OBMEP

i
i

16

CAP.1: MENSAGENS SECRETAS OBTIDAS POR SUBSTITUIO

a) Alfabeto quebrado ao meio:

b) Troca de dois vizinhos:

Observe que nenhuma letra ficou no seu lugar original. Neste


caso, dizemos que houve um desordenamento.
c) Usando a sequncia que aparece no teclado do computador:

Aqui tambm houve desordenamento.

i
i

principal
2009/10/28
i page 17
Estilo OBMEP

i
i

SEC.1.4: QUANTAS MANEIRAS DIFERENTES DE CRIPTOGRAFAR PODEMOS


CONSTRUIR?

17

Atividade 7: Usando o cdigo:


A

Decifre a mensagem:
Z TIZNZ V ZNZITZ
Leia de trs para frente a mensagem decifrada.

1.4

Quantas Maneiras Diferentes de Criptografar Podemos Construir?

Digamos que no planeta Plunct os alfabetos fossem formados por


apenas trs smbolos:
, , e . Poderamos criptografar mensagens
de seis maneiras diferentes:

A primeira dessas maneiras a trivial e no serve para codificar


nada. Sem listar as mensagens, poderamos concluir que existem seis
maneiras diferentes de permutar as letras deste alfabeto? claro que

i
i

principal
2009/10/28
i page 18
Estilo OBMEP

i
i

18

CAP.1: MENSAGENS SECRETAS OBTIDAS POR SUBSTITUIO

sim: para a primeira letra existem 3 possibilidades de codificao,


para a segunda apenas duas e para a terceira resta somente uma
possibilidade. Pelo Princpio Multiplicativo da Contagem, so
O Princpio Multiplicativo da Contagem:

321=6

Se uma deciso puder ser tomada de m maneiras


diferentes e se, uma vez tomada esta primeira deciso,
outra deciso puder ser tomada de n maneiras diferentes,
ento,

no

total

sero

tomadas

mn

decises.

as possibilidades. H uma notao muito til para se trabalhar como


produtos do tipo acima, camada fatorial. Por exemplo, o fatorial de
3 3! = 3 2 1. No caso geral, para um inteiro positivo n, define-se
n! = n (n 1) (n 2) . . . 3 2 1 e, por conveno, 0! = 1.
Observe que dentre estas, so trs as possibilidades que mantm
a ordem usual

(1 2 3 1) inalterada:

Quantos desordenamentos h neste caso? Apenas dois:

i
i

principal
2009/10/28
i page 19
Estilo OBMEP

i
i

SEC.1.4: QUANTAS MANEIRAS DIFERENTES DE CRIPTOGRAFAR PODEMOS


CONSTRUIR?

19

No planeta Plact (talvez mais evoludo que Plunct) so quatro as


letras empregadas: , , e . H, neste caso, 4 3 2 1 = 4! = 24
maneiras diferentes de permutar as letras. Dentre estas, apenas 4
respeitam a ordem usual
 . Quais so elas?
H 9 desordenamentos:

O que ocorre se usarmos as 26 letras de nosso alfabeto? Podemos


inferir algo?
Existem 26! maneiras diferentes de criptografar, isto d
403 291 461 126 605 635 584 000 000

i
i

principal
2009/10/28
i page 20
Estilo OBMEP

i
i

20

CAP.1: MENSAGENS SECRETAS OBTIDAS POR SUBSTITUIO

possibilidades! Se, entretanto, quisermos preservar a ordem usual das


letras, temos somente 26 maneiras, incluindo a trivial em que cada
letra trocada por ela mesma.
Em geral, se tivermos n letras, teremos
n! = n (n 1) (n 2) . . . 3 2 1
permutaes diferentes e somente n delas respeitam a ordem usual.
possvel tambm calcular os desordenamentos (em que nenhuma letra
fica no seu lugar natural). O nmero total de desordenamentos com
n letras :


(1)n
1
1
1
1
n!
+ + ... +
0!

1!

2!

3!

n!

(para ver a deduo desta frmula veja o livro Anlise Combinatria


e Probabilidade de Morgado, Pitombeira de Carvalho, Pinto Carvalho
e Fernandez IMPA, 1991).
Como curiosidade, com 26 letras o nmero de desordenamentos
: 148 362 637 348 470 135 821 287 825.
Com tantas possibilidades de codificao, parece extremamente
difcil se descobrir a chave para se quebrar um cdigo no estilo de Jlio
Csar, caso desconheamos qual foi a maneira com que as letras foram
inicialmente codificadas, no mesmo? No h esperana alguma de
se testar todas as possibilidades. Apesar disto o cdigo de Jlio Csar
e suas variaes so muito fceis de serem quebradas, como veremos
a seguir.

i
i

principal
2009/10/28
i page 21
Estilo OBMEP

i
i

SEC.1.5: COMO QUEBRAR O CDIGO DE JLIO CSAR

1.5

21

Como Quebrar o Cdigo de Jlio Csar

Mapas de tesouros
Vamos ilustrar a teoria da decifrao com
um trecho de um conto do escritor Edgar
Allan Poe, intitulado O Escaravelho de
ouro 2 .
O personagem principal deste conto, em
uma certa altura da obra, encontra um
velho pergaminho que acredita ser um
mapa de um tesouro, com os seguintes dizeres:
(53+305) )6*; 4826) 4.)4);806*;48+8
60))85;1(;:*8+83(88)5*+;46(;88*96*?;8)*(;485);5*+2:*(;4956* 2
(5*-4)88* ; 4069285);)6+8)4;1(9;48081 ; 8:81;48+85;4) 485 +
528806* 81(9;48; (88;4 ( ? 34;48) 4 ; 161 ; : 188; ?;

No conto, aps uma anlise baseada na freqncia das letras do alfabeto ingls feita pelo protagonista, a mensagem toma a seguinte
forma:
A good glass in the bishops hostel in the devils seat fortyone degrees and thirteen minutes north-east and by north main
branch seventh limb east side shoot from the left eye of the
deaths-head a bee line form the tree through the shot fifty feet
out.
2

Este conto faz parte do livro Histria de Mistrio e Imaginao de Edgar


Allan Poe, Editorial Verbo, no . 15, Lisboa.

i
i

principal
2009/10/28
i page 22
Estilo OBMEP

i
i

22

CAP.1: MENSAGENS SECRETAS OBTIDAS POR SUBSTITUIO

Como isto foi obtido? A letra que aparece com mais frequncia
na lngua inglesa a letra e; muitas vezes ela aparece dobrada ee. Na
mensagem secreta acima o smbolo 8 aparece 33 vezes, muito mais do
que as outras letras e portanto plausvel que 8 deva significar a letra
e. Substituindo 8 por e e tentando o mesmo esquema com outras
letras possvel decifrar a mensagem. Sua traduo para o portugus
:
Um bom vidro na hospedaria do bispo na cadeira do diabo
quarenta e um graus e treze minutos nordeste e quarta de norte
ramo principal stimo galho do lado leste a bala atravs do olho
esquerdo da cabea do morto uma linha de abelha da rvore
atravs da bala cinquenta ps para fora.

O conto ento revela, de uma maneira fantstica, como, a partir destas


informaes, o personagem principal encontra um tesouro h muito
tempo enterrado por um pirata que passou pelo lugar descrito na
mensagem.
O estudo da frequncia das letras do alfabeto constitui um mtodo
eficaz para se quebrar mensagens no estilo de Jlio Csar.

Frequncia aproximada das letras em portugus

i
i

principal
2009/10/28
i page 23
Estilo OBMEP

i
i

23

SEC.1.5: COMO QUEBRAR O CDIGO DE JLIO CSAR

14,6

1,0

3,8

4,9

12,5

1,0

1,3

1,2

6,1

0,4

0,02

2,7

4,7

5,0

10,7

2,5

1,2

6,5

7,8

4,3

4,6

1,6

0,01

0,2

0,01

0,4

Curiosidade: apesar da letra a aparecer em quase todos os textos


escritos em portugus, possvel encontrar textos em que esta letra
no aparece nunca ou quase nunca. Voc conseguiria escrever um
texto com 5 linhas sem usar nenhuma vez a letra a?
Como curiosidade, veja como aproximadamente se distribuem as
letras no espanhol e no ingls:

i
i

principal
2009/10/28
i page 24
Estilo OBMEP

i
i

24

CAP.1: MENSAGENS SECRETAS OBTIDAS POR SUBSTITUIO

Atividade 8: Usando as frequncias das letras em portugus, decifre a mensagem abaixo e complete a tabela para
registrar as substituies encontradas.
Urtklm tr dqapuakcftr ltr iasqtr aj nmqsuouar
lacfdqa t jakrtoaj tetfxm a cmjniasa t steait ntqt
qaofrsqtq tr ruersfsufcmar akcmksqtltr.
Observe na tabela as ocorrncias:

Letra

Nmero de vezes
que aparece
na frase

Letra

Nmero de vezes
que aparece
na frase

18

16

13

Como a frequncia de letras em portugus segue a ordem; A E O R S


I N , muito provavelmente a letra t deve ser a codificao da letra
A, pois a mais frequente, assim como a deve ser a codificao da
letra E, que a segunda mais freqente, mas isto, por enquanto, s

i
i

principal
2009/10/28
i page 25
Estilo OBMEP

i
i

SEC.1.5: COMO QUEBRAR O CDIGO DE JLIO CSAR

25

uma especulao.
Se substituirmos t por A e a por E na mensagem criptografada,
chegaremos a
UrAklm Ar dqEpuEkcfAr lAr iEsqAr Ej nmqsuouEr
lEcfdqE A jEkrAoEj AeAfxm E cmjniEsE A sAeEiA nAqA
qEofrsqAq Ar ruersfsufcmEr EkcmksqAlAr.
Ser que j conseguimos descobrir o que est escrito? Vamos analisar
a terceira letra mais frequente; a letra r que aparece 13 vezes na frase.
Ela pode estar codificando as seguintes letras ou O ou R ou S.
Se r codificar O a mensagem fica:
UOAklm AO dqEpuEkcfAO lAO iEsqAO Ej nmqsuouEO lEcfdqE A jEkOAoEj AeAfxm E cmjniEsE A sAeEiA
nAqA qEofOsqAq AO OueOsfsufcmEO EkcmksqAlAO.
Se r codificar R a mensagem fica:
URAklm AR dqEpuEkcfAR lAR iEsqAR Ej nmqsuouER lEcfdqE A jEkRAoEj AeAfxm E cmjniEsE A sAeEiA
nAqA qEofRsqAq AR RueRsfsufcmER EkcmksqAlAR.
Se r codificar S a mensagem fica:
USAklm AS dqEpuEkcfAS lAS iEsqAS Ej nmqsuouES
lEcfdqE A jEkSAoEj AeAfxm E cmjniEsE A sAeEiA nAqA
qEofSsqAq AS SueSsfsufcmES EkcmksqAlAS.

i
i

principal
2009/10/28
i page 26
Estilo OBMEP

i
i

26

CAP.1: MENSAGENS SECRETAS OBTIDAS POR SUBSTITUIO

Das trs opes acima a mais provvel a terceira, pois muitas palavras
terminam em S.
A quarta letra a ser analisada a letra q. Provavelmente ela a
codificao da letra O ou da letra R.
Se for da letra O, a mensagem fica:
USAklm AS dOEpuEkcfAS lAS iEsOAS Ej nmOsuouES lEcfdOE A jEkSAoEj AeAfxm E cmjniEsE A sAeEiA
nAOA OEofSsOAO AS SueSsfsufcmES EkcmksOAlAS.
(observe a palavra nAOA em negrito, ela corresponde a alguma palavra
em portugus?). melhor ficar com a segunda opo em que a letra
q corresponde letra R:
USAklm AS dREpuEkcfAS lAS iEsRAS Ej nmRsuouES lEcfdRE A jEkSAoEj AeAfxm E cmjniEsE A sAeEiA
nARA REofSsRAR AS SueSsfsufcmES EkcmksRAlAS.
A palavra Ej em negrito uma pista de que a letra j deve ser a
codificao da letra m. Se for, a mensagem se transforma em:
USAklm AS dREpuEkcfAS lAS iEsRAS EM nmRsuouES lEcfdRE A MEkSAoEM AeAfxm E cmMniEsE A
sAeEiA nARA REofSsRAR AS SueSsfsufcmES EkcmksRAlAS.
A palavra MEkSAoEM deve ser a codificao de MENSAGEM. Logo
k corresponde letra N e o corresponde letra G. Fazendo essas
substituies:

i
i

principal
2009/10/28
i page 27
Estilo OBMEP

i
i

SEC.1.5: COMO QUEBRAR O CDIGO DE JLIO CSAR

27

USANlm AS dREpuENcfAS lAS iEsRAS EM nmRsuGuES lEcfdRE A MENSAGEM AeAfxm E cmMniEsE


A sAeEiA nARA REGfSsRAR AS SueSsfsufcmES ENcmNsRAlAS.
Podemos reconhecer as palavras em negrito: lEcfdRE deve ser DECIFRE e REGfSsRAR deve ser REGISTRAR. Se assim for, l D, c
C mesmo, f I, d F e s T. Fazendo estas substituies:
USANDm AS FREpuENCIAS DAS iETRAS EM nmRTuGuES DECIFRE A MENSAGEM AeAIxm E CmMniETE A TAeEiA nARA REGISTRAR AS SueSTITuICmES ENCmNTRADAS.
possvel decifrar agora? Se voc no conseguir, volte e releia o
enunciado da atividade 8.
Se voc achou trabalhoso decifrar a mensagem da Atividade acima,
saiba que existem vrios softwares que fazem esta tarefa brincando.
Veja por exemplo os sites:
http://demonstrations.wolfram.com/CipherEncoder/
http://demonstrations.wolfram.com/LetterHighlightingInText/
Veja tambm a beleza do recurso criptogrfico utilizado na poesia
abaixo:

i
i

principal
2009/10/28
i page 28
Estilo OBMEP

i
i

28

CAP.1: MENSAGENS SECRETAS OBTIDAS POR SUBSTITUIO

PULSAR

Poesia Concreta de Augusto de Campos

Variaes
1) Uma maneira de encriptar mensagens consiste em escolher uma
palavra chave que deve ser mantida em segredo por quem as
envia e por quem as recebe. Esta palavra no deve ter letras
repetidas. Por exemplo, consideramos a palavra TECLADO.
Para fazer a troca de letras, podemos usar a seguinte correspondncia:
A

i
i

principal
2009/10/28
i page 29
Estilo OBMEP

i
i

29

SEC.1.5: COMO QUEBRAR O CDIGO DE JLIO CSAR

2) Outra maneira tambm inventada pelos gregos para codificar


mensagens feita trocando-se letras por nmeros, de acordo
com a seguinte tabela:
1

V/W

Por exemplo, a palavra LEGAL pode ser codificada como


31-15-22-11-31. Este cdigo pode ser transmitido com as mos:
os dedos da mo direita indicam as linhas e os da esquerda as
colunas. O que dizem as mozinhas abaixo?

i
i

principal
2009/10/28
i page 30
Estilo OBMEP

i
i

Captulo 2

A Escrita Braille e o Cdigo


Binrio
Introduo ao Estudo de
Combinaes
Voc consegue ler a seguinte mensagem?

30

i
i

principal
2009/10/28
i page 31
Estilo OBMEP

i
i

31
Trata-se de um texto escrito em Braille, um mtodo de escrita
desenvolvido para que pessoas com deficincias visuais possam ler
pelo tato. Seu criador, Louis Braille, ficou cego aos trs anos de idade
devido a um ferimento no olho feito com um objeto pontiagudo que
seu pai usava para fabricar selas de animais; o ferimento infeccionou
e isto provocou tambm a perda da viso no outro olho, provocando
sua deficincia visual total.
O cdigo Braille baseado em um arranjo 3 2 de pontos, dispostos como nas pedras de um domin:
32
(Trs linhas e duas colunas)

Para registrar uma dada letra do alfabeto, alguns desses 6 pontos


so marcados ou perfurados, de modo a se tornarem sobressalentes,
para que possam ser sentidos com as pontas dos dedos das mos.
Neste texto, quando um ponto estiver marcado, usaremos um crculo negro e, quando no estiver, um crculo branco. Veja os exemplos:
Letra

Letra

Somente a primeira casa foi marcada: o


ponto que est na primeira linha e na
primeira coluna aparece em negro.

A letra k tem marcas pretas e m dois pontos: o


ponto da primeira linha e da primeira coluna e
o ponto da terceira linha e primeira coluna.

i
i

principal
2009/10/28
i page 32
Estilo OBMEP

i
i

32

CAP.2: A ESCRITA BRAILLE E O CDIGO BINRIO

Atividade 1:
a) Quantos diferentes padres (disposies de pontos)
podemos formar usando o sistema 32 descrito acima?
b) Se quisermos codificar
todas as letras minsculas,
todas as letras maisculas,
os algarismos: 0, 1, 2, 3, 4, 5, 6, 7, 8 e 9,
os sinais de pontuao: . (ponto final), : (dois
pontos), ? (ponto de interrogao), ! (ponto de
exclamao) e , (vrgula),
os sinais de operaes matemticas: +, , e ,
os padres obtidos nas dimenses 32 sero suficientes?
c) Quantos padres podemos formar se dispusermos pontos arranjados em um quadrado 22? E em um retngulo 14? Porque ser que eles no so usados?
Eis aqui a maneira usual de codificar em Braille as letras minsculas e os algarismos:

i
i

principal
2009/10/28
i page 33
Estilo OBMEP

i
i

33

a
1

b
2

c
3

d
4

e
5

f
6

g
7

h
8

i
9

j
0

Para codificar letras maisculas, usamos o smbolo:


da letra que desejamos que seja a maiscula.

antes

Por exemplo, a letra A (maiscula) se escreve como:

A
Smbolo Letra a
para letra minscula
maiscula

i
i

principal
2009/10/28
i page 34
Estilo OBMEP

i
i

34

CAP.2: A ESCRITA BRAILLE E O CDIGO BINRIO

Observe tambm que a mesma configurao de


pontos usada ora para denotar algumas letras,
ora para denotar os algarismos 1, 2, 3, 4, 5, 6,
7, 8, 9 e 0. A fim de evitar confuso, para representar os nmeros, usa-se um smbolo inicial,
antecedendo as 10 primeiras letras do alfabeto.
Quando houver risco de confuso, para voltar novamente a usar o smbolo para significar letras,
devemos anteced-las com outro smbolo inicial,
indicando a restaurao, ou seja, indicando que
os smbolos que viro sero novamente letras.

Smbolo para
nmero

Smbolo
restituidor
para letra

Veja como so as representaes dos dez algarismos:

Smbolo Letra a
para
minscula

nmero

Este conjunto
representa
o nmero 1

Smbolo Letra f
para
minscula

nmero

Este conjunto
representa
o nmero 6

Letra b
Smbolo
para
minscula

nmero

Este conjunto
representa
o nmero 2

Smbolo
para

nmero

Letra g
minscula

Este conjunto
representa
o nmero 7

Smbolo Letra c
para
minscula

nmero

Este conjunto
representa
o nmero 3

Smbolo Letra h
para
minscula

nmero

Este conjunto
representa
o nmero 8

Smbolo Letra d
para minscula

nmero

Este conjunto
representa
o nmero 4

Smbolo Letra i
para
minscula

nmero

Este conjunto
representa
o nmero 9

Smbolo Letra e
para
minscula

nmero

Este conjunto
representa
o nmero 5

Smbolo Letra j
para minscula

nmero

Este conjunto
representa
o nmero 0

i
i

principal
2009/10/28
i page 35
Estilo OBMEP

i
i

35

Atividade 2:
a) Traduza para a linguagem usual

O que voc acha que

significa?

b) Escreva em Braille a frase: Louis Braille (1809-1852)


nasceu na Frana. No se esquea de usar o smbolo
para maisculas e o smbolo que transforma letras em
nmeros. Voc j leu esta frase antes?
Voc deve ter notado na atividade anterior que apenas letras e
nmeros no so suficientes para escrever todas as frases que desejamos. Na verdade existem muitos outros smbolos que so usados
em Braille; eles tambm variam de pas para pas. Veja alguns exemplos tpicos usados em portugus:

Todos os sinais grficos tm representao em Braille. Veja alguns:

Veja tambm alguns smbolos usuais da Matemtica:

i
i

principal
2009/10/28
i page 36
Estilo OBMEP

i
i

36

CAP.2: A ESCRITA BRAILLE E O CDIGO BINRIO

>

<

ndice
inferior

expoente

Para mais informaes consulte o site do Instituto Benjamin


Constant: http://www.ibc.gov.br/?catid=110&blogid=1&itemid=479
Atividade 3: Esta atividade tem o objetivo de simular a
leitura em Braille de frases e frmulas matemticas feitas pelos deficientes visuais. Voc deve recortar as fichas e perfurar
os crculos marcados em preto com um furador de papel (um
clipe ou um palito tambm podem ser usados).
Depois de recortadas monte com as fichas algumas expresses
matemticas para que um amigo consiga ler o que voc escreveu, mas sem que ele as olhe!
Com as mos, seu amigo deve sentir as perfuraes de cada
ficha e, a partir deste ponto, pode consultar a tabela da
pgina 43 para descobrir o valor da letra ou nmero correspondente que est lendo com suas mos. Mas observe: ele
no pode de modo algum ver diretamente a ficha que est
manuseando.
Veja alguns exemplos de expresses matemticas em Braille:

i
i

principal
2009/10/28
i page 37
Estilo OBMEP

i
i

37

a1

a1

a1

a1

a1

b2

b2

b2

b2

b2

c 3

c 3

c 3

c 3

c 3

d4

d4

d4

d4

d4

e 5

e 5

e 5

e 5

e 5

f 6

f 6

f 6

f 6

f 6

i
i

principal
2009/10/28
i page 38
Estilo OBMEP

i
i

38

CAP.2: A ESCRITA BRAILLE E O CDIGO BINRIO

i
i

principal
2009/10/28
i page 39
Estilo OBMEP

i
i

39

g7

g7

g7

g7

g7

h8

h8

h8

h8

h8

i 9

i 9

i 9

i 9

i 9

j 0

j 0

j 0

j 0

j 0

i
i

principal
2009/10/28
i page 40
Estilo OBMEP

i
i

40

CAP.2: A ESCRITA BRAILLE E O CDIGO BINRIO

i
i

principal
2009/10/28
i page 41
Estilo OBMEP

i
i

41

nmero

nmero

nmero

nmero

>

nmero

>

<

<

Sinal restituidor de
letra

nmero

Sinal restituidor de
letra

maiscula

maiscula

nmero

i
i

principal
2009/10/28
i page 42
Estilo OBMEP

i
i

42

CAP.2: A ESCRITA BRAILLE E O CDIGO BINRIO

i
i

principal
2009/10/28
i page 43
Estilo OBMEP

i
i

43

Tabela para consulta:


Letras e nmeros em Braille

i
i

principal
2009/10/28
i page 44
Estilo OBMEP

i
i

44

2.1

CAP.2: A ESCRITA BRAILLE E O CDIGO BINRIO

Explorando Conceitos Matemticos com a


Linguagem Braille

Existem 26 = 64 configuraes que podem ser obtidas no cdigo


de Braille usual 32. fcil descobrir que isto verdade, quando
aplicamos o Princpio Multiplicativo da Contagem: h duas possibilidades para a primeira casa ou ela marcada ou no (ou pintamos
de preto ou de branco) - do mesmo modo h duas possibilidades para
cada uma das outras casas, o que resulta em
2 2 2 2 2 2 = 26 possibilidades.
O Princpio Multiplicativo da Contagem:
Se uma deciso puder ser tomada de m maneiras
diferentes e se, uma vez tomada esta primeira deciso,
outra deciso puder ser tomada de n maneiras diferentes,
ento,

no

total

sero

tomadas

mn

decises.

Vale a pena explorar este exemplo com estratgias diferentes.

Mtodo 1: Focando na quantidade de pontos, independentes de estarem pintados ou no:

Comeando com um s ponto, teremos s 2 possibilidades:


Com dois pontos h 4 possibilidades, pois h duas escolhas para
cada uma das configuraes j vistas acima (com um ponto apenas):

i
i

principal
2009/10/28
i page 45
Estilo OBMEP

i
i

SEC.2.1: EXPLORANDO CONCEITOS MATEMTICOS COM A LINGUAGEM


BRAILLE

45

J com trs pontos h 8 possibilidades (duas para cada uma das


configuraes com dois pontos vistas acima):

Continuando assim, com quatro pontos teremos 16 configuraes


distintas, com cinco pontos 32 configuraes e, claro, com 6 pontos
chegaremos a 64 padres diferentes de pontos. Isto fcil de entender:
cada configurao em um estgio anterior produz duas novas configuraes no estgio seguinte. Dentre as 64 possibilidades, temos dois
casos extremos: um em que nenhum dos pontos marcado e outro
em que todos os seis pontos so marcados:
Em Braille, por
motivos bvios,
esta

Na linguagem Braille, esta


configurao tem a funo de
referencial de posio, para
auxiliar a indicar sinais grficos

configurao

tais como a crase ou o trema.

no usada.

usada para indicar a letra.

Mtodo 2: Focando na quantidade pintada de pontos:


Com nenhum ponto marcado temos apenas uma configurao:

i
i

principal
2009/10/28
i page 46
Estilo OBMEP

i
i

46

CAP.2: A ESCRITA BRAILLE E O CDIGO BINRIO

Com apenas um ponto marcado, temos 6 possibilidades:

As configuraes com dois pontos marcados totalizam 15. Veja:


Todas estas tm a primeira
casa marcada em preto (veja
a seta

To das

estas

configura es

tm

p onto

preto na casa com a seta, s est faltando uma que j foi contada, pois ela a
primeira configurao do grupo anterior.

Todas estas tm ponto preto na


casa com a seta, mas esto faltando
duas que j foram contadas anteriormente.
Todas estas tm ponto preto
na casa com a seta, esto faltando trs que j foram contadas anteriormente.
Resta somente esta ltima
configurao que no apareceu em nenhum dos grupos anteriores,

Deste modo, com dois pontos pretos h 1 + 2 + 3 + 4 + 5 = 15


possibilidades.
Com trs pontos marcados em negro, h 20 possibilidades. Veja:

i
i

principal
2009/10/28
i page 47
Estilo OBMEP

i
i

SEC.2.1: EXPLORANDO CONCEITOS MATEMTICOS COM A LINGUAGEM


BRAILLE

47

Poderamos continuar agora exibindo todas as configuraes com


4 pontos negros (so 15 ao todo), todas com 5 pontos pretos (so 6
no total) e todas com 6 pontos negros (apenas 1), mas no faremos
isto porque h um belo argumento de simetria aqui:
Escolher 4 pontos para marcar com preto entre 6 pontos brancos
o mesmo que escolher 2 pontos para marcar de branco entre
6 negros!
De modo anlogo, o nmero de escolhas de 5 pontos para pintlos de preto dentre 6 pontos brancos o mesmo nmero de
escolhas de 1 nico ponto para pintar de branco dentre 6 pontos
negros.
Simetricamente s h uma possibilidade em que todos os pontos esto marcados e s h uma possibilidade em que todos os
pontos no esto marcados.

i
i

principal
2009/10/28
i page 48
Estilo OBMEP

i
i

48

CAP.2: A ESCRITA BRAILLE E O CDIGO BINRIO

Resumidamente, temos:

Nmero de pontos

Nmero de possveis

negros

configuraes

15

20

15

T
R

Total

64

S
I
M
E

Estes padres so encontrados nas combinaes, que passaremos


a estudar.

2.2

Combinaes Matemticas

Existem situaes envolvendo contagens em que a ordem dos elementos importante e outras em que no. Para entender melhor este
fato, vamos comparar os dois exemplos abaixo:
Exemplo 2.1. De quantas maneiras diferentes podemos estacionar 3
carros em 2 garagens?

i
i

principal
2009/10/28
i page 49
Estilo OBMEP

i
i

49

SEC.2.2: COMBINAES MATEMTICAS

Soluo:
A resposta muito simples, se pensarmos
da seguinte maneira: existem 3 possibilidades para preencher a primeira garagem,
mas apenas duas para a estacionarmos na
segunda; pelo Princpio Multiplicativo, o
nmero total de maneira 3 2 = 6 possibilidades.Se A, B e C so os carros, essas 6
maneiras so as seguintes: AB, BA, AC,
CA, BC e CB.
Exemplo 2.2. Quantas saladas de frutas diferentes podemos fazer
usando duas das seguintes frutas: abacaxi, banana ou caqui?
Soluo:
Procedemos como antes: primeiro escolhemos
umas das trs frutas (3 possibilidades), depois
a segunda e ltima fruta (2 possibilidades).
Com isto teremos 3 2 = 6 possibilidades.
Entretanto o nmero de saladas de frutas no
6 e sim 3. Porqu?
Se a, b e c so as frutas, essas 6 escolhas so as seguintes: ab, ba,
ac, ca, bc e cb; mas uma salada de frutas feita com abacaxi e banana
a mesma que uma feita com banana e abacaxi, ou seja ab = ba e
de modo semelhante, ac = ca e bc = cb. O que importante observar aqui que quando duas frutas so permutadas, elas produzem
a mesma salada. Neste caso a ordem de escolha das frutas no
importante e o nmero correto de saladas
32
2

3.

i
i

principal
2009/10/28
i page 50
Estilo OBMEP

i
i

50

CAP.2: A ESCRITA BRAILLE E O CDIGO BINRIO

O nmero 2 no denominador corresponde permutao de duas frutas. Ou seja, contamos tudo como se a ordem fosse importante e
dividimos o resultado pelo nmero de permutaes de 2 elementos.
Este ltimo exemplo o prottipo do que se chama em Matemtica de uma combinao simples. Observe bem o que fizemos:
Aplicamos o Princpio Multiplicativo para se obter todas as possibilidades, respeitando a ordem (3 2 = 6).
Dividimos o resultado obtido acima pelo nmero de permutaes da quantidade previamente combinada que d o tamanho
de cada escolha (como combinamos fazer saladas com apenas 2
frutas, dividimos por 2! = 2, obtendo (3 2)/2 = 3).
No caso geral, se tivermos n objetos distintos nossa disposio
e tivermos que escolher p objetos distintos dentre esses, obteremos
as combinaes simples de n elementos tomados p a p. claro que
p n.
O nmero total dessa combinaes denotado por Cnp e calculado
da seguinte maneira:
Cnp =

n (n 1) . . . (n (p 1))
p (p 1) (p 2) . . . 3 2 1

ou, em notao fatorial:


Cnp =

n!
p!(n p)!

(voc sabe justificar porque vale esta ltima frmula?).

i
i

principal
2009/10/28
i page 51
Estilo OBMEP

i
i

SEC.2.3: AS COMBINAES E A LINGUAGEM BRAILLE

51

No exemplo das saladas de frutas, n = 3, p = 2 e o nmero de


3!
3 2!
saladas de frutas C32 =
=
= 3. Vejamos mais um
2!(3 2)!
2!1!
exemplo:

Exemplo 2.3. Quantos so os subconjuntos de {a, b, c, d, e} que possuem exatamente trs elementos?

5!

5 4 3!

Soluo: A resposta C53 =


=
= 10 pois a ordem
3!(5 3)!
3!2!
dos elementos listados em um conjunto no relevante. De fato os
subconjuntos so os seguintes: {a, b, c}, {a, b, d}, {a, b, e}, {a, c, d},
{a, c, e}, {a, d, e}, {b, c, d}, {b, c, e}, {b, d, e}, {c, d, e}.

2.3

As Combinaes e a Linguagem Braille

Podemos analisar agora todas as possibilidades da escrita Braille


em uma clula 3 2:

Neste caso, o nmero de pontos que podemos combinar entre si


n = 6.

i
i

principal
2009/10/28
i page 52
Estilo OBMEP

i
i

52

CAP.2: A ESCRITA BRAILLE E O CDIGO BINRIO

Nmero de pontos em preto

Quantidade de combinaes distintas


6!
=1
0!(6 0)!
6!
=6
C61 =
1!(6 1)!
6!
C62 =
= 15
2!(6 2)!
6!
C63 =
= 20
3!(6 3)!
6!
C64 =
= 15
4!(6 4)!
6!
C65 =
=6
5!(6 1)!
6!
C66 =
=1
6!(6 6)!

C60 =

p=0
p=1
p=2
p=3
p=4
p=5
p=6

Podemos, a partir deste exemplo, inferir algumas concluses:


A simetria dos resultados acima sugere que Cnp = Cnnp . De
fato,
Cnp =

n!
n!
=
= Cnnp .
p!(n p)!
(n p)!(n (n p))!

Cn2 igual soma dos n-1 primeiros nmeros naturais. De fato,


Cn2 =

n.(n 1)
n!
=
= 1 + 2 + . . . + (n 1).
2!(n 2)!
2

Cn0 + Cn1 + Cn2 + . . . + Cnn = 2n .


Para ver porque isto vlido, observe que Cnp o nmero de subconjuntos com exatamente p elementos do conjunto {1, 2, . . . , n} e portanto Cn0 + Cn1 + Cn2 + . . . + Cnn o nmero total de subconjuntos de

i
i

principal
2009/10/28
i page 53
Estilo OBMEP

i
i

SEC.2.4: MATEMTICAS COM PROBLEMAS VISUAIS

53

{1, 2, . . . , n}. Devemos responder ento a seguinte pergunta: quantos


so os subconjuntos de {1, 2, . . . , n}?
Para determinar um desses subconjuntos, olhamos para o nmero
1 e perguntamos: ele est ou no no subconjunto? Existem apenas
duas respostas: sim ou no. Olhamos para o nmero 2 e repetimos a
pergunta: 2 est ou no no subconjunto em considerao? Mais uma
vez temos duas respostas e continuamos assim at o nmero n. No
total teremos que tomar n decises e, cada uma delas, admite apenas
duas possibilidades. Pelo Princpio Multiplicativo, existiro ento
2 2 . . . 2 = 2n decises,
e como cada deciso determina um e um s subconjunto, teremos que
o nmero total de subconjuntos de {1, 2, . . . , n} 2n .
Atividade 4:
a) Procedendo como na linguagem Braille, se ao invs de
uma clula 32, tivermos uma 34, como a da figura,
quantas configuraes diferentes teremos no total?
b) Em uma clula 3 4, quantas so as configuraes que
possuem exatamente 5 pontos marcados?
c) Em uma clula nm, quantas configuraes diferentes
podemos formar?
d) Em uma clula n m, quantas configuraes tm exatamente p pontos marcados?

i
i

principal
2009/10/28
i page 54
Estilo OBMEP

i
i

54

2.4

CAP.2: A ESCRITA BRAILLE E O CDIGO BINRIO

Matemticos com Problemas Visuais


Eratstenes (nasceu em 276 a.C. e faleceu em
194 a.C.) foi o diretor da Biblioteca de Alexandria e
um homem de grande distino em muitos ramos do
conhecimento.

Em Matemtica trabalhou com o problema da duplicao do volume do cubo, com nmeros primos o
crivo de Eratstenes at hoje empregado na Teoria dos Nmeros e, surpreendentemente, calculou com preciso o raio da Terra, comparando sombras em duas diferentes cidades do Egito. Ele tambm
calculou a distncia entre o Sol e a Lua, usando medies durante
eclipses e fez muitas descobertas em Geografia. Mesmo atuando em
tantos ramos do conhecimento seu apelido era Beta, pois em cada
rea especfica sempre havia um outro pensador cujo trabalho especializado tinha mais volume ou profundidade que o de Eratstenes. Na
velhice se tornou deficiente visual e conta-se que morreu de inanio,
deixando voluntariamente de comer.
Leonhard Euler (1707-1783, l-se iler), foi
um grande matemtico do sculo XVIII, trabalhou
em quase todos os ramos da Matemtica: teoria dos nmeros, equaes diferenciais, clculo das
variaes, fundamentos do Clculo e Topologia,
ramo da Geometria que foi fundador. Para Euler estes campos de pesquisa estavam intimamente
conectados; seus trabalhos, to criativos eram, que chegam a beirar
a genialidade. Perdeu totalmente a viso de um olho em 1738, poca

i
i

principal
2009/10/28
i page 55
Estilo OBMEP

i
i

SEC.2.4: MATEMTICAS COM PROBLEMAS VISUAIS

55

em que uma catarata comeou a se desenvolver no outro olho. Em


1771 uma operao mal sucedida da catarata deixou-o com deficincia
visual total. Apesar da cegueira, devido a sua notvel memria, mais
metade de seu volumoso trabalho foi realizado depois que se tornou
completamente deficiente visual.
Joseph Plateau (1801-1883, l-se plat)
era professor de uma escola secundria em Lige
(Frana); durante o perodo que lecionava elaborou
sua tese de doutorado sobre as impresses que a luz
exerce no olho. Em 1829, realizou um experimento
imprudente que consistia em olhar diretamente para
a luz do sol, por isto seus olhos ficaram irritados durante anos e sua
viso ficou restrita. Em 1841 sofreu uma infeco nos olhos e, em dois
anos, perdeu completamente sua viso. Antes disto, porm, realizou
experimentos para estudar as formas das bolhas de sabo, originando
assim o estudo das superfcies mnimas em Geometria Diferencial.
Sua velhice foi atpica: bem humorado, apesar dos problemas com a
viso, seu estado fsico e mental eram excelentes, sempre envolvido
com ensino, em contato direto com estudantes.
Lev Pontryagin (1908-1988), nasceu quando
sua me Tatyana Pontryaguin tinha 29 anos de
idade. Ela foi uma costureira e uma mulher notvel
que teve um importante papel na carreira matemtica de seu filho. Com 14 anos de idade Pontryagin
sofreu um acidente com uma exploso que o tornou
deficiente visual total. Desta poca em diante sua
me assumiu completamente a responsabilidade de cuidar de todos

i
i

principal
2009/10/28
i page 56
Estilo OBMEP

i
i

56

CAP.2: A ESCRITA BRAILLE E O CDIGO BINRIO

os aspectos de sua vida. Apesar de grandes dificuldades, ela trabalhou por muitos anos como sua secretria, lendo artigos cientficos em diversas lnguas, escrevendo frmulas em manuscritos, corrigindo trabalhos, etc. Pontryagin foi um dos grandes matemticos do
sculo XX, foi chefe de departamento de Matemtica na Rssia e vicepresidente da Unio Internacional de Matemtica. Atuou em quase
todas as reas de Matemtica, com destaque em Topologia, lgebra,
Equaes Diferenciais, Teoria do Controle e Sistemas Dinmicos.
Para maiores informaes consulte o site
http://turnbull.mcs.st-and.ac.uk/history/

2.5

O Sistema Binrio

Vamos estudar agora o sistema em que as clulas possuem uma


linha somente e 6 colunas. Ele ser muito semelhante ao sistema usual
da linguagem Braille 3 2,pois existe uma correspondncia um-a-um
entre as configuraes com clulas 3 2 e configuraes em clulas
1 6. Veja:

3 2

1 6

Sistemas do tipo 1 n servem para escrever nmeros na base 2 e,


assim sendo, tm muitas aplicaes na Matemtica e na Informtica.
Podemos escrever qualquer nmero natural na base 2, utilizandose para isto apenas os dgitos 0 e 1. Para compreender como isto pode

i
i

principal
2009/10/28
i page 57
Estilo OBMEP

i
i

57

SEC.2.5: O SISTEMA BINRIO

ser realizado, trabalharemos, por simplicidade, com os nmeros que


vo de 0 a 63.
Dado um nmero qualquer (tomaremos como exemplo o nmero
41), veja como agrupar para escrever o nmero na base 2:

41 = 20 2 + 1

41 = 5 8 + 1

41 = 10 4 + 1

Formamos 20 pares,

Usamos os 20 pares

Usamos os 10 grupos de

observe que sobra

anteriores para formar

quatro elementos obtidos

uma unidade

10 grupos de quatro

anteriormente para agrup-

elementos cada um.

los em cinco grupos maio-

res

41 = 2 16 + 1 8 + 1

com

oito

elementos

cada.

41 = 1 32 + 1 8 + 1

Usamos os 5 grupos de oito

Finalmente usamos os dois grupos de

elementos obtidos anteriormente

dezesseis elementos que surgiram no

para agrup-los em dois grupos

estgio anterior para agrup-los em

maiores com dezesseis elementos

um nico grupo maior com trinta e

cada. Note que sobra um grupo de

dois elementos. Alm desses restam

oito elementos e tambm uma

um grupo de oito e uma unidade

unidade.

simples.

i
i

principal
2009/10/28
i page 58
Estilo OBMEP

i
i

58

CAP.2: A ESCRITA BRAILLE E O CDIGO BINRIO

Concluso:
41 = 1 25 + 0 24 + 1 23 + 0 22 + 0 21 + 1 20 .
Esta expresso escrita abreviadamente na seguinte forma:
41 = (1 0 1 0 0 1)2
(l-se: 41 um, zero, um, zero, zero, um, na base 2).

Veja como represent-lo no estilo Braille:

25

24

23

22

21

20

Os nmeros de 0 a 63 podem ser representados na base 2, de


acordo com o seguinte diagrama de rvore:

i
i

principal
2009/10/28
i page 59
Estilo OBMEP

i
i

SEC.2.5: O SISTEMA BINRIO

59

i
i

principal
2009/10/28
i page 60
Estilo OBMEP

i
i

60

CAP.2: A ESCRITA BRAILLE E O CDIGO BINRIO

Atividade 5: Resolva o seguinte problema (OBMEP 2007)

Atividade 6: Um pequeno computador de papel os


cartes binrios
Vamos utilizar cartes de cartolina perfurados para trabalhar com
nmeros de 0 a 31, utilizando-se a base 2. Esses nmeros podem ser
escritos com apenas 5 dgitos, observe:

i
i

principal
2009/10/28
i page 61
Estilo OBMEP

i
i

61

SEC.2.5: O SISTEMA BINRIO

Base 10

Base 2

Base 10

Base 2

Base 10

Base 2

Base 10

Base 2

00000

01000

16

10000

24

11000

00001

01001

17

10001

25

11001

00010

10

01010

18

10010

26

11010

00011

11

01011

19

10011

27

11011

00100

12

01100

20

10100

28

11100

00101

13

01101

21

10101

29

11101

00110

14

01110

22

10110

30

11110

00111

15

01111

23

10111

31

11111

a) Recorte, perfure e corte as fendas dos cartes das pginas seguintes.


b) Cada buraco representar o nmero 1 e cada fenda o nmero
0. Faa um mao com as cartas. Se voc colocar um palito (ou um canudo ou um clipe) por alguns dos buracos do
mao, e levant-lo, algumas cartas cairo e outras ficaro presas no palito. Repetindo organizadamente este procedimento
voc poder realizar vrias operaes com os nmeros binrios
de 0 a 31. Veja algumas delas:
Separar as cartas pares da mpares. Basta colocar o palito no
primeiro furo direita e levantar. Todas os cartes com nmeros
pares cairo.
Com somente 5 colocaes de palitos e levantamentos possvel
colocar as cartas de 0 a 31 em ordem crescente. Embaralhe as
cartas. Comece colocando o palito no primeiro buraco da direita
(casa das unidades). Com cuidado levante o mao, deixando que
as cartas caiam, mas mantendo a ordem. Coloque as cartas que
caram na frente das demais e repita o mesmo procedimento

i
i

principal
2009/10/28
i page 62
Estilo OBMEP

i
i

62

CAP.2: A ESCRITA BRAILLE E O CDIGO BINRIO

para todos os demais 4 buracos, sempre mantendo a ordem.


Quanto terminar as cartas estaro em ordem.
Pode-se localizar qualquer nmero de 0 a 31 com a colocao do
palito e levantamento do mao 5 vezes. Isto se deve ao fato de
que qualquer nmero natural tem representao nica na base
2. Veja como voc pode fazer para localizar a carta 23:

4
2 23 2

0 0

0 0

Coloque o palito na casa das unidades e levante o mao, descarte


as que caram. A seguir coloque no buraco 21 , levante e descarte as
que caram. Prossiga, colocando o palito na casa 22 , descarte as que
caram. Coloque na casa 23 , levante e descarte as que ficaram presas.
Agrupe as cartas que caram e mais uma vez use o palite na casa 24 .
A carta que ficou presa a 23.
Responda:
1. Se fizermos cartas com 6 buracos ao invs de 5, quantos nmeros
diferentes obteremos?
2. Qual o nmero mnimo de buracos que teremos fazer nos
cartes para representar os nmeros de 0 at 127?
3. Leia a prxima atividade O dia do aniversrio na pgina 71 e
descreva uma maneira de adivinhar o aniversrio de uma pessoa
usando os cartes perfurados que voc fabricou.

i
i

principal
2009/10/28
i page 63
Estilo OBMEP

i
i

SEC.2.5: O SISTEMA BINRIO

63

i
i

principal
2009/10/28
i page 64
Estilo OBMEP

i
i

64

CAP.2: A ESCRITA BRAILLE E O CDIGO BINRIO

i
i

principal
2009/10/28
i page 65
Estilo OBMEP

i
i

65

SEC.2.5: O SISTEMA BINRIO

21
=

20
(

1
16

16

+ 8

+ 4

1
1

1
16

+ 8

+ 4

+ 8

0
+ 4

)
2

+ 1

1
16

16

+ 8

0
+ 4

)
2

16
(

1
16

16

+ 8

1
+ 4

15

)
2

16

16

+ 8

+ 4

14
(

1
16

+ 8

+ 4

16

+ 8

+ 2

1
+ 4

1
16

13
=

1
16

17

18
16

1
16

19
=

+ 8

1
+ 4 +

+ 4 +

16

0
+ 8

+ 4 +

)
2

12
(

1
16

+ 8

0
+ 4

+ 4 +

1
16

+ 8

+ 4

1
16

+ 8

+ 4

0
+ 4

)
2

i
i

principal
2009/10/28
i page 66
Estilo OBMEP

i
i

66

CAP.2: A ESCRITA BRAILLE E O CDIGO BINRIO

i
i

principal
2009/10/28
i page 67
Estilo OBMEP

i
i

SEC.2.5: O SISTEMA BINRIO

67

i
i

principal
2009/10/28
i page 68
Estilo OBMEP

i
i

68

CAP.2: A ESCRITA BRAILLE E O CDIGO BINRIO

i
i

principal
2009/10/28
i page 69
Estilo OBMEP

i
i

SEC.2.5: O SISTEMA BINRIO

69

i
i

principal
2009/10/28
i page 70
Estilo OBMEP

i
i

70

CAP.2: A ESCRITA BRAILLE E O CDIGO BINRIO

i
i

principal
2009/10/28
i page 71
Estilo OBMEP

i
i

SEC.2.5: O SISTEMA BINRIO

71

Atividade 6: O dia de aniversrio

i
i

principal
2009/10/28
i page 72
Estilo OBMEP
i

i
i

72

CAP.2: A ESCRITA BRAILLE E O CDIGO BINRIO

Como funciona o truque:


Como adivinhar o dia em que uma pessoa nasceu:
1. Pea pessoa que indique em quais dos calendrios a data de
seu nascimento aparece sublinhada.
2. Some os primeiros nmeros sublinhados que aparecem nos calendrios que a pessoa escolheu e voc descobrir a data de seu
aniversrio, sem que ela lhe conte.
Por exemplo: Se a pessoa nasceu no dia 7, os calendrios em que este
nmero aparece sublinhado so: o primeiro, o segundo e o terceiro.
Somando os primeiros nmeros sublinhados destes trs calendrios teremos 1 + 2 + 4 = 7. No legal? Se voc seguir o roteiro
abaixo, poder descobrir o signo e tambm o ms que a pessoa nasceu
.
Como adivinhar o ms em que uma pessoa nasceu
consulte seu horscopo para encontrar o ms de
nascimento:
ries 21 de maro a 20 de abril
Hoje um dia muito favorvel para lidar com clculos matemticos. Abra
sua mente para a beleza da Matemtica e no deixe de acreditar no seu
potencial criativo. Voc nasceu no dia ...

i
i

principal
2009/10/28
i page 73
Estilo OBMEP
i

i
i

SEC.2.5: O SISTEMA BINRIO

73

Touro 21 de abril a 20 de maio


Um ciclo de novas idias se abre para voc. Tudo se alterna tal qual uma
funo trigonomtrica. tempo de estudar e desenvolver os seus potenciais
criativos. Voc nasceu no dia...

Gmeos 21 de maio a 20 de junho


Hoje um dia em que muitas coisas no caminham muito de acordo com seus
planos, mas lembre-se que problemas devem estar nos livros de Matemtica
e mesmo assim eles podem ser solucionados! Voc nasceu no dia ...

Cncer 21 de junho a 21 de julho


Alegre-se! Hoje um dia harmonioso para voc interagir, fazer novas
amizades e dar incio a projetos pessoais como o estudo da Matemtica.
Voc nasceu no dia ...

Leo 22 de julho a 22 de agosto


Problemas existem, mas com disposio, talento e criatividade voc vence
qualquer obstculo. Quando estudar Matemtica, no desista, a soluo
sempre estar a seu alcance. Voc nasceu no dia ...

Virgem 23 de agosto a 22 de setembro


Hoje um dia de muita sensibilidade e pensamento positivo. Realize hoje
mesmo seus sonhos, estudando Matemtica com dedicao. Acredite no seu
potencial. Voc nasceu no dia ...

i
i

principal
2009/10/28
i page 74
Estilo OBMEP
i

i
i

74

CAP.2: A ESCRITA BRAILLE E O CDIGO BINRIO

Libra 23 de setembro a 22 de outubro


Hoje um dia favorvel para lidar com os assuntos da Geometria. Comece
investindo no seu visual e surpreenda a todos, principalmente seu professor,
fazendo todos os exerccios de Matemtica. Voc nasceu no dia ...

Escorpio 23 de outubro a 21 de novembro


bem provvel que o que voc esteja procurando externamente esteja dentro
de voc, por isto, resolva voc mesmo seus problemas de Matemtica, sem
procurar ajuda. Voc consegue! Voc nasceu no dia ...

Sagitrio 22 de novembro a 21 de dezembro


Descubra seu potencial criativo, resolvendo problemas de Matemtica. Com
isso estar afastando a rotina e admirando a beleza desta cincia. Observe
o mundo com os olhos da razo! Voc nasceu no dia ...

Capricrnio 22 de dezembro a 20 de janeiro


No se aborrea com pequenos atritos do dia-a-dia. No de deixe abalar
quando no encontrar imediatamente a soluo de um problema de Matemtica. No desista e entenda que h propsitos maiores cujas portas sero
abertas pela dedicao e estudo. Voc nasceu no dia ...

Aqurio 21 de janeiro a 19 de fevereiro


Hoje a vida lhe dar tudo para ser feliz. H tesouros que temos e muitas
vezes no os percebemos; por exemplo, h uma satisfao enorme quando
resolvemos um belo problema de Matemtica. Voc nasceu no dia ...

i
i

principal
2009/10/28
i page 75
Estilo OBMEP

i
i

SEC.2.5: O SISTEMA BINRIO

75

Peixes 20 de fevereiro a 20 de maro


Hoje um dia de oportunidades e novidades, principalmente nos estudos.
Voc ir resolver com facilidade todos os problemas de Matemtica que lhe
forem apresentados. tempo fazer planos, trabalhar as idias criativas e
execut-las. Voc nasceu no dia...

Podemos implementar todas as operaes que so realizadas na


base 10 tambm na base 2. Existem mquinas simples que ajudam
a entender como estas operaes so feitas. Uma delas est indicada
abaixo.

Bons estudos! Bom divertimento!

i
i

arquivo_origa
2010/2/26
page 1
Estilo OBMEP

O
4/3

A
E

B
2/ 3

O
Q

2/3

B
F

Explorando Geometria com


Origami
Eduardo Cavacami
Yolanda Kioko Saito Furuya

arquivo_origa
2010/2/26
page 2
Estilo OBMEP

Texto j revisado pela nova ortografia.

arquivo_or
2010/2/26
page i
Estilo OBME

Sumrio
Introduo

As construes e os Axiomas de Huzita-Hatori . . . . . . . .

1 Seces de Segmentos
1 1
1.1 Construo de e a partir do Quadrado . . . . . .
3 5
1
1.2 Construo de a partir do Retngulo . . . . . . . .
n

9
9
13

2 Trisseco do ngulo

16

3 Quadrados e reas
1
3.1 Proporo da rea . . . . . . . . . . . . . . . . . . .
2
1
3.2 Proporo da rea . . . . . . . . . . . . . . . . . .
n

20

4 Quadratura do Retngulo

25

5 Duplicao do Cubo

29
i

20
22

arquivo_origa
2010/2/26
page ii
Estilo OBMEP

ii

SUMRIO

6 Pentgono e Retngulo ureo

32

7 Poliedros de Plato de Faces Triangulares


1
2
7.1 Retngulos e . . . . . . . . . . . . . . . . . . .
3
3
7.2 Construo das Unidades . . . . . . . . . . . . . . . .

41

7.3

7.4

41
46

7.2.1

Unidade A

. . . . . . . . . . . . . . . . . . . .

46

7.2.2

Unidade B . . . . . . . . . . . . . . . . . . . . .

49

Montagem dos Poliedros . . . . . . . . . . . . . . . . .

52

7.3.1

Tetraedro . . . . . . . . . . . . . . . . . . . . .

52

7.3.2

Octaedro . . . . . . . . . . . . . . . . . . . . .

53

7.3.3

Icosaedro . . . . . . . . . . . . . . . . . . . . .

55

Esqueleto do Icosaedro . . . . . . . . . . . . . . . . . .

58

8 Poliedro de Plato de Faces Quadradas

60

9 Poliedro de Plato de Faces Pentagonais

63

9.1

Do copo ao Pentgono Regular . . . . . . . . . . . . .

63

9.2

Dodecaedro . . . . . . . . . . . . . . . . . . . . . . . .

69

9.2.1

Construo do Retngulo para o Dodecaedro de


Aresta Dada . . . . . . . . . . . . . . . . . . .

71

10 Construes que se Encaixam

75

Referncias Bibliogrficas

78

arquivo_origa
2010/2/26
page 1
Estilo OBMEP

Introduo
As primeiras aplicaes da Geometria de que se tem notcia apareceram em problemas relacionados com diviso de suas terras e na Astronomia. Desde ento o uso da Geometria uma constante na vida
do homem e hoje o seu estudo inserido no ensino da Matemtica
desde os primeiros anos escolares. No entanto, notrio a dificuldade
no aprendizado e a falta de motivao no estudo da Geometria.
A aplicao de Origami no ensino da Geometria pode auxiliar no
desenvolvimento cognitivo, trazendo assim uma melhor aprendizagem
e compreenso da Matemtica atravs da manipulao de um simples
pedao de papel.
O Origami, de origem desconhecida, tem etmologia japonesa e
significa dobrar (ori ) papel (kami ). No Brasil, utiliza-se tambm a
palavra dobradura, mas o termo Origami mundialmente reconhecido
e utilizado.
Este trabalho trata do relacionamento entre a Geometria e o Origami, atravs da implementao de dobraduras para apresentao de
resultados da Geometria e o uso da Geometria para justificar as construes. Com isto estamos lidando com mais uma metodologia de
1

arquivo_origa
2010/2/26
page 2
Estilo OBMEP

INTRODUO

ensino e estudo da Geometria Elementar, com o uso de uma tcnica


milenar, concreta e divertida, alm de acessvel a qualquer pessoa.
Inicialmente apresentamos alguns problemas clssicos da Geometria Euclidiana, incluindo a resoluo com Origami de dois problemas
no solveis com rgua e compasso: a trisseco do ngulo e a duplicao do cubo. Depois passamos aos poliedros de Plato, construdos
atravs de mdulos de Origami que se encaixam uns aos outros, formando as faces.
Este texto foi baseado principalmente no trabalho de Hisashi Abe,
do seu livro Sugoiz1 Origami, de 2003 (em japons, [1]). Hisashi Abe,
da Universidade de Hokaido, o autor da construo da trisseco do
ngulo apresentada neste texto.
Para um melhor aproveitamento das informaes obtidas neste
trabalho, espera-se um conhecimento bsico em Geometria Elementar.

As construes e os Axiomas de Huzita-Hatori


Apesar de existirem tcnicas de origami dobrando linhas curvas2
alm das retas, este trabalho se restringir s dobras em linha reta.
Cada dobra efetuada gera uma linha reta e os pontos de um dos
semiplanos so refletidos no outro semiplano, ou seja, se r a linha
de dobra, e P um ponto da folha a ser dobrada, P levado no seu
simtrico P em relao a r. Ou seja, a linha de dobra r a mediatriz
de cada par P, P , onde P o refletido de P (onde P levado).
1
2

Sugoiz = Impressionante
Como, por exemplo, na construo da Rosa de Kawasaki.

arquivo_origa
2010/2/26
page 3
Estilo OBMEP

INTRODUO

Alm disso, a linha de dobra r tambm a bissetriz de cada ngulo


b
P V P formado por um raio V P com origem V em r e seu raio refletido
V P , onde o raio levado.
Com isso, v-se que mediatrizes e bissetrizes so construes elementares com Origami, assim como perpendiculares e paralelas.

Construes elementares no sero detalhadas todas as vezes que


forem utilizadas, para no desviar a ateno da construo central.
Todas as construes utilizadas no texto so consequncias dos
Axiomas da Geometria do Origami, conhecidos como os Axiomas de
Huzita (ou Huzita-Hatori, ou Huzita-Justin), que podem ser obtidos no seguinte endereo eletrnico, de Robert Lang: http://www.
langorigami.com/science, e dadas a seguir:

arquivo_origa
2010/2/26
page 4
Estilo OBMEP

INTRODUO

1. Dados dois pontos distintos P1


e P2 , existe apenas uma dobra
que passa por eles.

2. Dados dois pontos distintos P1


e P2 , existe apenas uma dobra
que coloca P1 sobre P2 .

3. Dadas as retas r1 e r2 , existe uma dobra que coloca r1 sobre r2 .

arquivo_origa
2010/2/26
page 5
Estilo OBMEP

INTRODUO

4. Dados um ponto P e uma reta r, existe uma dobra nica que


perpendicular a r e que passa por P .

5. Dados dois pontos P1 e P2 e uma reta r1 , existe uma dobra que


coloca P1 sobre r1 e que passa por P2 .

arquivo_origa
2010/2/26
page 6
Estilo OBMEP

INTRODUO

6. Dados dois pontos P1 e P2 e duas retas r1 e r2 , existe uma dobra


que leva simultaneamente P1 sobre r1 e P2 sobre r2 .

7. Dados um ponto P e duas retas r1 e r2 , existe uma dobra que


coloca P1 sobre r1 e que perpendicular a r2 .

Nesta axiomtica, o papel tem o tamanho suficientemente grande


para conter todas as construes necessrias (suponha-o ilimitado).
Alm disso, por existe uma dobra entende-se que se a soluo geo-

arquivo_origa
2010/2/26
page 7
Estilo OBMEP

INTRODUO

mtrica existir, ento pode ser realizada atravs de uma dobra.


Por exemplo, a quantidade de solues do Axioma 5 pode ser 0, 1
ou 2, dependendo da posio dos pontos e da reta, pois o problema
equivalente a encontrar a interseco da reta r1 com a circunferncia
de centro P2 passando por P1 .
No h garantia de independncia entre os axiomas. Mas podese garantir que o sexto axioma (de Humiaki Huzita) no consequncia dos cinco primeiros, pois os cinco primeiros geram somente
construes possveis com rgua e compasso e, com o sexto axioma,
podemos obter resultados no construtveis com rgua e compasso
como veremos adiante.
O stimo axioma, acrescentado por Koshiro Hatori, em 2001, e
supostamente independente dos cinco primeiros, deixa uma dvida:
Observe na construo geomtrica do Axioma 7, que o ponto P
levado em P r1 . Ora P P deve ser paralelo a r2 para que a dobra
seja perpendicular a r2 . Assim, efetuando os seguintes passos:
Dobre perpendicularmente a r2 por P obtendo como vinco a
reta s1 (Axioma 4).
Dobre perpendicularmente a s1 por P obtendo s2 (Axioma 4).
Chame o ponto em r1 s2 de P .
Dobre levando o ponto P a P (Axioma 2), obtendo a reta .
Temos que a reta a mediatriz de P P e, portanto, perpendicular a
r2 k P P . Assim, a construo do Axioma 7 consequncia dos axiomas 4 e 2. Ou seja, o Axioma 7 decorre dos cinco primeiros axiomas
de Huzita. Pergunta-se: existe algum furo nesta argumentao? O

arquivo_origa
2010/2/26
page 8
Estilo OBMEP

INTRODUO

fato que a dobra do Axioma 7 pode ser construda com um nico


movimento, o de deslizar um ponto Q de r2 , mais distante que P de
r1 , sobre r2 at que P encontre r1 .
Um estudo mais avanado, de Robert J. Lang, sobre estes axiomas e construes geomtricas com Origami pode ser obtido gratuitamente em: http://www.langorigami.com/science/hha/origami_
constructions.pdf (em ingls). Nele demonstrado inclusive a completude do conjunto de axiomas, isto , que no h mais axiomas a se
acrescentar. E tal estudo feito com o envolvimento de outra grande
rea da Matemtica: a lgebra.

arquivo_origa
2010/2/26
page 9
Estilo OBMEP

Captulo 1

Seces de Segmentos
Como podemos facilmente determinar o ponto mdio de um segmento atravs do Origami, podemos tambm dividir um segmento em
2n partes, com n = 0, 1, 2, 3, . . . . Com rgua e compasso, os gregos
dividiam segmentos em n partes. Veremos agora que com o Origami
tambm possvel essa seco.
A seco urea do segmento ser trabalhada em momento oportuno, na construo de pentgonos.

1.1

Construo de

1 1
e a partir do Quadrado
3 5

Para se obter uma trisseco de segmento a partir de um quadrado,


procedemos da seguinte maneira:
9

arquivo_origa
2010/2/26
page 10
Estilo OBMEP

10

 CAP. 1: SECES DE SEGMENTOS

Seja dado um quadrado ABCD.


Suponha-o de lado igual a 1.
Encontre os pontos mdios E e F
dos lados AB e CD, respectivamente.

Leve o vrtice D ao ponto E.


O novo segmento CD determina sobre o antigo BC um ponto I.
Temos que IC, depois de aberto,
1
equivale a
do lado do quadrado
3
ABCD.
A demonstrao segue por semelhana de tringulos:
b e B EI
b so complementares, pois GEI
b reto por
Os ngulos AEG
b que reto.
ser o refletido de GDC
Os tringulos GAE e EBI so
b compleretngulos. Como AEG
b ento AEG
b conmentar de B EI,
b
gruente B IE.
Como os tringulos so retngulos e possuem um dos
ngulos congruentes, eles so semelhantes.

arquivo_origa
2010/2/26
page 11
Estilo OBMEP

N SEC. 1.1: CONSTRUO DE

1
1
E
A PARTIR DO QUADRADO
3
5

11

Denominaremos agora por x e y os segmentos AG e BI, respectivamente. Como G um ponto entre A e D, GD = GE = 1 x.


1
Temos tambm que AE = EB = .
2
Por Pitgoras, temos no tringulo GAE:
 1 2
2

+ x2 = (1 x)2 =

Pela semelhana
temos:
1
2
3
8

y
1
2

de

1
3
+ x2 = 1 2x + x2 = x = .
4
8

tringulos,

G
x

A
E

3
1
2
y = = y = .
8
4
3

Logo,

1x

E
1/2

1/2

1
IC = 1 y = .
3

No Origami tambm possvel dividir um segmento em cinco


partes. Vejamos:
Comece com um quadrado ABCD
de lado 1.
Encontre o ponto mdio E de AB.
Encontre, ento, o ponto mdio Q
entre A e E.
1
Temos que AQ = , donde
4

4 AQ = 2 AE = 2 EB = AB.

arquivo_origa
2010/2/26
page 12
Estilo OBMEP

12

 CAP. 1: SECES DE SEGMENTOS

Leve o vrtice D ao ponto Q, determinando J em BC.


Verifica-se, assim como no caso
da trisseco do segmento, dois
tringulos semelhantes, AGQ e
BQJ, pelos mesmos motivos do
outro caso.
1
1
Temos que = BJ.
5
2

De fato, nos tringulos semelhantes,


G

AG
AQ
GQ
=
=
.
BQ
BJ
DJ

3/4
x

Sejam x = AQ e y = BJ.

1x

1/4

Temos por Pitgoras que:


 2
1
15
1
+ x2 = 1 2x + x2 = x = .
+ x2 = (1 x)2 =
4
16
32
Por semelhana de tringulos temos:
15
32
1
4

Obtido

3
4

15
3
2
y=
y= .
32
16
5

2
1
do segmento, basta dividir por 2 para conseguir .
5
5

arquivo_origa
2010/2/26
page 13
Estilo OBMEP

N SEC. 1.2: CONSTRUO DE

1.2

1
A PARTIR DO RETNGULO
N

Construo de

13

1
a partir do Retngulo
n

Existe uma forma mais generalizada de se dividir por n partes,


no sendo necessrio o papel ser quadrado. Podemos comear com
qualquer papel retangular.
Vamos refazer a diviso do segmento no caso n = 3 e n = 5.
Seja dado um papel retangular qualquer ABCD. Dobre
uma das diagonais e depois ao
meio pelo lado maior, determinando os pontos E e F , pontos mdios dos respectivos segmentos AB e DC.

No retngulo EBCF que representa a metade do retngulo ABCD, dobre sua diagonal BF , encontrando o ponto
I, interseco da diagonal
maior com a menor.

Os tringulos ABI e CF I so semelhantes, pois os ngulos do


vrtice em comum so congruentes, opostos pelo vrtice e os outros
ngulos so alternos internos.

arquivo_origa
2010/2/26
page 14
Estilo OBMEP

14

 CAP. 1: SECES DE SEGMENTOS

AB
AI
2
2
=
= e, portanto, a diagonal AC
Temos ento que
1
1
FC
IC
est divida em trs partes iguais.
A

Dobre uma perpendicular a


AB, passando pelo ponto I e
obtenha o ponto G AB.

1
GB = AB
3
D

A ltima afirmao segue do Teorema de Tales, j que IG e CB


so paralelos.
1
O mtodo anterior pode ser aplicado para se obter do segmento.
5
A

Pegue um papel retangular


qualquer ABCD. Determine
E e F , pontos mdios de AB
e DC. Determine tambm G
e H, pontos mdios de EB e
F C.

arquivo_origa
2010/2/26
page 15
Estilo OBMEP

N SEC. 1.2: CONSTRUO DE

1
A PARTIR DO RETNGULO
N

15
E

Encontre BH, diagonal do


retngulo GBCH. Chame de
I a interseco de AC e BH.
I
D

G J

Dobre JL, perpendicular a


AB, passando por I. Verifica1
se que JB de AB, pois
5
AB
AI
4
=
= .
1
HC
IC

I
D

JB igual a

HL

1
de AB.
5

Com esse ltimo mtodo, podemos dividir qualquer segmento em


n partes, com n N, por induo: tendo o segmento JL da diviso
em n 1 partes, dobrando a diagonal LB do retngulo JBCL, encontrando o novo ponto I na interseco das diagonais, e dobrando
um novo segmento J L perpendicular a AB passando por I. Ento
1
J B = AB.
n

arquivo_origa
2010/2/26
page 16
Estilo OBMEP

Captulo 2

Trisseco do ngulo
Um dos famosos problemas da antiga Grcia era a trisseco de um
ngulo qualquer com rgua e compasso. Esse problema impossvel
com rgua e compasso, mas solvel com Origami. A construo
dada a seguir creditado a Hisashi Abe, conforme publicado em 1980
no Japo.
A

Seja um ngulo EBC menor que


90o conforme figura.
Para casos de ngulos obtusos, basta
aplicar apenas no ngulo excedente
a 90o e som-lo trisseco do
restante.
B

16

arquivo_origa
2010/2/26
page 17
Estilo OBMEP

17
Determine uma paralela F G a AD.
Se a construo seguinte no couber no papel, escolha outra paralela
mais convenientemente posicionada,
ou use papel maior (ainda com vrtice B na quina do papel).

Determine uma paralela HI, onde


H e I so os respectivos pontos mdios de F B e GC.
Dobre de modo a levar o ponto F ao
segmento EB e o ponto B ao segmento HI.
Esta ltima dobra dada pelo
Axioma 6 de Huzita.

Para uma melhor visualizao, marque os pontos H , F e B (onde


foram H, F e B), sobre o papel, e
trace o segmento F B .

arquivo_origa
2010/2/26
page 18
Estilo OBMEP

18

 CAP. 2: TRISSECO DO NGULO

Abra novamente e trace os segmentos B B e H B. Trace por B


uma paralela a HB, com extremidade N .
Temos que os tringulos BB N ,
BB H e BF H so congruentes, com os ngulos em B congruentes.

E
F

D
G

F
H
H

N C

De fato:
Os tringulos BB N e BB H so congruentes, pois possuem
a hipotenusa BB em comum e os catetos opostos aos ngulos no
vrtice B so congruentes, j que N B = BH = B H .
Os tringulos BB H e BF H so congruentes, pois possuem
um cateto BH em comum e os catetos opostos aos ngulos no vrtice
B so congruentes, pois H B = HB = HF = H F .

arquivo_origa
2010/2/26
page 19
Estilo OBMEP

19

Com isso, o vrtice dos tringulos que esto em B tm os mesmos


ngulos, assim, EBC est divido em trs partes congruentes.

O passo que no pode ser realizado com rgua e compasso o


passo do Axioma 6 de Huzita.

arquivo_origa
2010/2/26
page 20
Estilo OBMEP

Captulo 3

Quadrados e reas
3.1

Proporo

1
da rea
2

Um problema que simplesmente podemos obter com rgua e compasso e de fcil aplicao em Origami, com ajuda de uma tesoura,
como obter um quadrado com a metade da rea de um quadrado
inicial.
B

Seja um quadrado ABCD. Junte os


vrtices A e C para obter o segmento
BD. Analogamente, obtenha o segmento AC.
fcil ver que os ngulos juntos ao
centro so retos.

b
a

d
D

20

c
O

arquivo_origa
2010/2/26
page 21
Estilo OBMEP

N SEC. 3.1: PROPORO

1
DA REA
2

21

Recortando os tringulos obtidos e juntando-os dois a dois como


na figura abaixo, teremos dois quadrados, cada um com a metade da
rea do quadrado ABCD inicial.

b
a

c
d

possvel obter esse mesmo resultado de outras formas.


Seja dado um quadrado ABCD.
Leve todos os vrtices ao centro do
quadrado, ou seja, no encontro das
duas diagonais.
O quadrado resultante tem a metade
da rea do quadrado original.

D
B

A B
D C

arquivo_origa
2010/2/26
page 22
Estilo OBMEP

22

3.2

 CAP. 3: QUADRADOS E REAS

Proporo

1
da rea
n

Vimos no captulo anterior uma forma de se dividir um segmento


1
em n lados, obtendo assim, a proporo de . Na primeira parte
n
deste captulo, vimos como obter um quadrado com a metade da rea
de um quadrado dado. Veremos agora como obter um quadrado de
1
rea da rea original.
n
Iniciando com um quadrado ABCD,
1
dobre o lado na proporo que den
sejar. Marque os pontos E e F , conforme figura.
Fixando B, leve o vrtice A ao segmento EF , rotacionando por um
eixo BG.

Fixe C e leve B sobre BG, obtendo


o eixo HC (HCBG).
Repita o procedimento nos outros
vrtices.
Note que os eixos de rotao so ortogonais, pois o vrtice levado ao
eixo que o contm.

1
1
B

F
G

1/n

C
D

1/n

arquivo_origa
2010/2/26
page 23
Estilo OBMEP

N SEC. 3.2:

PROPORO

1
DA REA
N

23

Os eixos formaro um quadrado.


Esse quadrado A B C D est para
1
ABCD, assim como
est para
n
1. Em outras palavras, a rea de
ABCD dividido por n igual a rea
de A B C D .

1/n

Para demonstrar a proporo entre as reas de A B C D e ABCD,


vamos analisar as relaes entre os segmentos construdos, nomeando
os elementos conforme a figura:

a
d

a
D

b
c

1/n

1
, basta mostrar que
n
2

a = x, onde a o lado do quadrado A B C D .


Supondo que ABCD tem lado 1 e que x =

arquivo_origa
2010/2/26
page 24
Estilo OBMEP

24

 CAP. 3: QUADRADOS E REAS

Temos que AB = B O = c por construo e que os tringulos


AA D e AEO so semelhantes, por serem retngulos e possurem
o mesmo ngulo em A.
Da semelhana,
1x
a+c
=
,
2c
1
donde a + c =

1x
.
2c

Alm disso, no AA D, temos que


(a + c)2 + c2 = 1.
Mas (a+c)2 +c2 = a2 +2ac+2c2 = a2 +2c(a+c) = a2 +2c
a2 + 1 x.
Logo a2 + 1 x = 1, donde a2 = x =

1x
=
2c

1
.
n

1
Conclui-se ento que a rea do quadrado A B C D
da rea
n
do quadrado ABCD.

Observao: n no precisa ser nmero inteiro.

arquivo_origa
2010/2/26
page 25
Estilo OBMEP

Captulo 4

Quadratura do Retngulo
Dado um retngulo ABCD, o problema consiste em transform-lo
num quadrado de mesma rea.
A

Construa o quadrado AF ED
como na figura.

Dobre o segmento HG, onde


H e G so os pontos mdios
de AB e DC.

25

arquivo_origa
2010/2/26
page 26
Estilo OBMEP

26

 CAP. 4: QUADRATURA DO RETNGULO

Com o ponto H fixo, leve


o vrtice B ao segmento EF ,
encontrando o ponto O no segmento EF , e K no segmento
BC.

K
O

Os tringulos HKB e HKO so congruentes.


H

Prolongue BO at encontrar
DC no ponto I.
Recorte pelos segmentos OA e
BI.

K
O

GI

Nomeie as peas como a, b e


c, conforme a figura.
c
a

arquivo_origa
2010/2/26
page 27
Estilo OBMEP

27

Reorganize as peas de
modo a obter um quadrado
de mesma rea do retngulo
ABCD.

Mas ser que realmente obteremos um quadrado perfeito?


Provavelmente, devido algumas imprecises nas dobras o resultado pode ser duvidoso. Verificaremos ento, os segmentos e ngulos
obtidos:
Vimos que HOK
= HBK, j que so refletidos em relao a
HK.
Temos que BO perpendicular a HK pela construo. Se
P = BO HK, o ngulo HBP congruente ao ngulo ABO.
Como H ponto mdio de AB e P ponto mdio de OB, temos
ento que o tringulo ABO semelhante ao tringulo HBP , na
2
razo de .
1

arquivo_origa
2010/2/26
page 28
Estilo OBMEP

28

 CAP. 4: QUADRATURA DO RETNGULO

Logo, podemos concluir que AO


perpendicular a OB. Provado
isto, podemos concluir que os outros ngulos que formaro os vrtices do quadrado sero complementares, assim, conclumos que
os ngulos satisfazem os ngulos
de um quadrado (ou de um retngulo).

GI

Resta provar se BI = AO = lado do quadrado. Vamos chamar de


j o lado menor e l o lado maior do retngulo. Por serem retngulos
e um ngulo em comum, os tringulos AOB, ICB e AF O so
semelhantes.
A

AF
AO
=
AO
AB

j
AO
=
l
AO
p
AO = j l

Precisamos averiguar o valor de BI, outro lado do quadrado:


p
jl
BI
BC
BI
j
= BI =
= BI = j l .
=
=
=
l
jl
jl
AB
AO
A rea do retngulo de lados j e l dado por j l; a rea do

quadrado, de lado j l, tambm j l. Portanto, est satisfeita a


quadratura do retngulo.

arquivo_origa
2010/2/26
page 29
Estilo OBMEP

Captulo 5

Duplicao do Cubo
Problema: Dado um cubo
de aresta a, obter a aresta b
de um cubo com o dobro do volume.

Este mais um dos trs problemas clssicos de Euclides. Mais


uma vez, possvel com Origami e impossvel com rgua e compasso.

Primeiro, podemos obter o volume a3 do cubo de aresta a na


seguinte construo:

29

arquivo_origa
2010/2/26
page 30
Estilo OBMEP
b

30

 CAP. 5: DUPLICAO DO CUBO

Considere OU = 1 e OA = a.
Usando a propriedade

A
b

1 a a2
O

a3

temos que
OB = a2 e OC = a3 .
Exerccio: construa a4 .

Tendo a3 , vamos construir b =

2a3 .

Para isso, tendo obtido u = 2a3 e considere um retngulo ABCD


suficientemente grande para a construo seguinte:
b

Em AB marque E e F com
AE = 1 e AF = 2. Marque
EG e F H.
Em AD obtenha I e A com
AI = u = IA .
Marque IJ e A B .
Seja O = IJ EG.
(*) Dobre levando I sobre
r = F H e E sobre s =
A B , obtendo .
(**) determina em EG o

ponto P e OP = 3 u.

D
b

G
b

H
b

A
b

b
b

3
b
b

Q
b

u = 2a3

E B

P
b

u
I

C
b

1
E
b

arquivo_origa
2010/2/26
page 31
Estilo OBMEP

31
(*) Esta dobra do mesmo tipo utilizado na trisseco do ngulo,
dado pelo Axioma 6 de Hizuta.
(**) Como a mediatriz de II e passa por P de EG, temos a
b so
mesma situao da figura anterior, onde os ngulos I PbQ e P QE
3
retos e, portanto, u = OE = OP , donde segue o resultado.

arquivo_origa
2010/2/26
page 32
Estilo OBMEP

Captulo 6

Pentgono e Retngulo
ureo
Neste captulo ser feito uma das construes de um pentgono regular. Existem outras formas de se obt-lo, como veremos mais tarde.
A propriedade explorada nesta construo que o lado do pentgono
o segmento ureo da diagonal. Como subproduto, podemos construir
o retngulo ureo.
Lembramos que retngulo ureo de lados a e b segue a
proporo

b
51
ba
a
2
2
=
(a < b), donde a = b ab e, portanto, a =
.
b
a
2

Para b = 2 e a = 5 1.

32

arquivo_origa
2010/2/26
page 33
Estilo OBMEP

33
Considere um quadrado ABCD de
lado 2.
Observao: Esta medida para
simplificar a demonstrao.
Junte os vrtices A com B e D com
C, obtendo assim um retngulo de
2 1 e o lado EF .
No retngulo BEF D escolha uma
diagonal, digamos, BF .
Pelo Teorema de Pitgoras,
2

BF = 22 + 12 = BF =

5.

Usando a diagonal como eixo de rotao, dobre o vrtice C para fora.

Fixando F , leve o ponto C ao segmento BF e marque o ponto C .

arquivo_origa
2010/2/26
page 34
Estilo OBMEP

34

 CAP. 6: PENTGONO E RETNGULO UREO

Temos que CF igual a 1, pois assumimos que o lado do quadrado

2. Assim, BF CF igual a 5 1, ou seja, BC = 5 1.


Observe que BC o segmento ureo do lado (2).

Voltando para o quadrado inicial,


fixe B e leve o vrtice A at BF .

Subtraia BC = 5 1 do lado
AB = 2; o resto, ou seja, CA , divida ao meio no ponto O.

Com a mesma distncia de AO, a


partir de B, marque O .

arquivo_origa
2010/2/26
page 35
Estilo OBMEP

35
D

A
O

Temos ento que o segmento OO


tem comprimento igual a BC =

5 1, podendo ser um dos lados


do pentgono.

O
B

P
O

Fixando O, leve o ponto O at AD.


Marque como P o ponto onde O
toca AD.

G
B

O
F

Analogamente, marque como Q o


ponto onde O encontra BC.
Dobre por OP e O Q.

A
E

F
B

O
Q

arquivo_origa
2010/2/26
page 36
Estilo OBMEP

36

 CAP. 6: PENTGONO E RETNGULO UREO

Fixando P , leve O ao segmento EF .


Note que a dobra faz-se em torno do
eixo dos pontos P e o ponto mdio
de O Q.

A
E

F
B

O
Q

Neste momento, vamos analisar alguns resultados.


Temos que os pontos O e Q so simtricos a O e P em relao

a EF, por construo. Alm disso, OO = 5 1 e AO = BO =


3 5
.
2
Por Pitgoras,
2

AO + AP = OP ,
donde
!2
3 5
+ AP
2

e portanto, AP

2
51

5 5
=
.
2
2

Temos ainda que AP + AO = O P , lembrando que AO =


AO + OO .

5+1 2
5 5
2
+(
) = O P
2
2

arquivo_origa
2010/2/26
page 37
Estilo OBMEP

37

5 5
5 1
5
2

+ +
+ = O P
2
2
4
2
4
2

O P = 4 O P = 2
O P = P Q = 2 (lado do quadrado)
Com isso, temos que O P Q um tringulo issceles e por isso,
se R o ponto mdio de O Q, ento P R O Q. A reflexo do
quadriltero OP RO em torno do eixo P R, determina V , exatamente
sobre EF .
P

Continuando, leve os vrtices D e


C sobre o quadriltero OP RO .

O F

O
Q

C
P

Volte apenas a dobra efetuada sobre


o eixo P R, obtendo assim um pentgono regular.

A
V

E
B
O
Q

arquivo_origa
2010/2/26
page 38
Estilo OBMEP

38

 CAP. 6: PENTGONO E RETNGULO UREO

O pentgono mesmo regular?


Inicialmente supomos o lado do
quadrado igual a 2. Encontramos o

segmento 51 e o utilizamos como


lado do pentgono. Vimos que o vrtice V foi obtido atravs da reflexo
do quadriltero OP RO .
Vimos tambm, em passos da construo do pentgono, que os pontos
P e Q so os vrtices do pentgono,
e, pela construo,
OP = O P = OO =

51 e

O
M

P Q = O P = 2.

O tringulo P V Q issceles e congruente ao P OO , assim, traando


uma perpendicular a P Q e passando
por F , temos que M F , conforme
figura, bissetriz do P V Q = 2
e divide P Q ao meio.
Observao: V 6= F .

VF

arquivo_origa
2010/2/26
page 39
Estilo OBMEP

39
Temos que P M = 1, P V =
Ento
sen =

5 1.

1
= 54o .
51

Logo, P F Q = 108o , que corresponde


ao ngulo interno de um pentgono.

A demonstrao dos outros ngulos fica por conta do leitor.


Se quando obtivemos o segmento ureo BC do lado (2), transferssemos a medida a um dos lados do quadrado a partir de um dos
vrtices, teramos ento o retngulo ureo. Voltemos ento construo:
A

Fixe B e leve o vrtice A at BF .


O ponto (chamemos de A ) do lado BA
que levado em C tal que BA o
segmento ureo do lado.

A
E

F
C

Por A trace uma perpendicular ao


lado AB.

A
E

C
D

D
F
C

arquivo_origa
2010/2/26
page 40
Estilo OBMEP

40

 CAP. 6: PENTGONO E RETNGULO UREO

Est pronto o retngulo ureo A BCD .


A

arquivo_origa
2010/2/26
page 41
Estilo OBMEP

Captulo 7

Poliedros de Plato de Faces


Triangulares
Entre os cinco poliedros convexos regulares, conhecidos como Poliedros de Plato, trs deles so constitudos de faces triangulares:
tetraedro, octaedro e icosaedro.
Neste captulo construmos as unidades bsicas (ou mdulos) que
se encaixam de formas distintas, formando os poliedros de faces triangulares acima.
Para isso, vamos abordar algumas construes preliminares.

7.1

2
1
Retngulos e
3
3

A construo das unidades bsicas passa por preparao de retngulos de propores especiais.
41

arquivo_origa
2010/2/26
page 42
Estilo OBMEP

42

 CAP. 7: POLIEDROS DE PLATO DE FACES TRIANGULARES

1
2
Vamos trabalhar inicialmente com a relao e sobre os
3
3
lados do retngulo.
A

Essas medidas aparecem naturalmente no tringulo equiltero.

2/ 3
1

Seja um papel quadrado ABCD de


lado 1.
Encontre EF , onde E e F so pontos mdios de AD e BC, respectivamente.
Fixando B, leve C a EF .

Pelo ponto J obtido em EF , dobre


a perpendicular HI a EF .
Teremos, para o segmento HB que:

1 2
3
2
2
HB + ( ) = 1 HB =
.
2
2

Teremos agora, dois casos.

2/ 3

1/ 3

1/ 3

arquivo_origa
2010/2/26
page 43
Estilo OBMEP

1
2
N SEC. 7.1: RETNGULOS E
3
3

1
O primeiro caso, a razo de :
3
Corte por HI e EF .
Obteremos duas peas, cujas pro1
pores dos lados so de em
3
cada pea.
1
BF
FC
=
= .
HB
IC
3
2
No segundo caso a razo :
3
Corte somente por HI.
Sem cortar por EF , teremos um
retngulo com a seguinte proporo:
BC
2
= .
HB
3

43

I
G

I
G

Lembramos que o papel A4 de lados a e b (a > b) tal que


b
a
=
, ou seja, dobrando pelo lado maior, temos dois retngulos
b
a/2

a2
com a mesma proporo do original. Logo
= b2 , donde a = b 2,
2
isto , o lado maior a diagonal do quadrado de aresta igual ao lado
menor. interessante, mas no a proporo que queremos para os
nossos mdulos.
Mas podemos obter do papel A4 doze unidades com a proporo
1
, com uma perda muito pequena. Isto facilitar na construo dos
3
poliedros.

arquivo_origa
2010/2/26
page 44
Estilo OBMEP

44

 CAP. 7: POLIEDROS DE PLATO DE FACES TRIANGULARES

Seja um papel A4 com os vrtices ABCD. Dobre pelo lado


maior ao meio e depois ao
meio novamente, obtendo assim, trs vincos, dividindo o
papel em 4 partes iguais.

F
Fixando A, leve o vrtice B at o
segmento EF feito pelo vinco central, rotacionando em torno do eixo
AG.

arquivo_origa
2010/2/26
page 45
Estilo OBMEP

1
2
N SEC. 7.1: RETNGULOS E
3
3

Pelo ponto obtido em EF marque o


segmento HI perpendicular a AD.
Pelo que vimos anteriormente,
a al
3
se considetura AH equivale a
2
rarmos AB = 1, isto ,

AH
3
=
.
2
AB
Dividindo AH por 2 e AB por 4,
temos
HA

2 = 3.
1
AB
4

Para aproveitar o papel, dobre a proporo obtida usando como eixo HI,
conseguindo mais quatro peas com
1
razo .
3

45

D
M

C
N

arquivo_origa
2010/2/26
page 46
Estilo OBMEP

46

 CAP. 7: POLIEDROS DE PLATO DE FACES TRIANGULARES

O resultado a obteno de 12 peas


1
com a razo de .
3

7.2

Construo das Unidades

Construiremos agora os mdulos, que chamaremos de unidades


A e B dos poliedros de faces triangulares. Para isto, ser necessrio a
1
utilizao de retngulos de proporo , como as 12 peas obtidas
3
do papel A4, visto anteriormente. Estas unidades formam tringulos
equilteros, que ao se encaixarem, produziro os poliedros.

7.2.1

Unidade A
A

Com uma pea retangular ABCD, respeitando as propores, leve o vrtice B ao


D.

arquivo_origa
2010/2/26
page 47
Estilo OBMEP

47

N SEC. 7.2: CONSTRUO DAS UNIDADES

Ao levar B a D, surge um eixo de rotao


EF .
EF a mediatriz de BD.
Os EF D e EF B so equilteros de
2
lado .
3

Leve o vrtice B ao ponto F .


A nova dobra paralela a AD.

A
4/3

Leve o vrtice C sobre DF .

C
2/ 3

2/3

B
F

Vire a pea, de modo que a parte de trs


fique para frente.

A
2/ 3

1/ 3

E
2/3

2/ 3

arquivo_origa
2010/2/26
page 48
Estilo OBMEP

48

 CAP. 7: POLIEDROS DE PLATO DE FACES TRIANGULARES

A
D E

Leve o vrtice D ao ponto E.

Mova o vrtice A dobrando segundo o eixo


do ponto E.

2/3

2/ 3

2/3

F
E

Desfaa a dobra pelo eixo EF , de modo


que aparea um paralelogramo.

2/3

2/3

F
F

Vire a pea, de modo que a parte oculta


volte-se para frente.

Leve as duas extremidades cujos ngulos


so agudos sobre o lado oposto, fixando os
vrtices com ngulos obtusos.
Obtm-se um losango cujos lados e a dia2
gonal menor medem .
3

4/3

F
2/3

2/3
1/ 3

1/3
4/3

arquivo_origa
2010/2/26
page 49
Estilo OBMEP

49

N SEC. 7.2: CONSTRUO DAS UNIDADES

4
Nas figuras anteriores, temos que a base do paralelogramo , ou
3
2
seja, cabem duas vezes o lado .
3
O segmento pertencente base doparalelogramo e que forma um
1
3
tringulo retngulo e a altura
.
3
3
Esses valores satisfazem as medidas do tringulo equiltero citado
no incio deste captulo.
Abra o losango para obter a unidade A,
que composta por quatro tringulos equi2
lteros de lado .
3

7.2.2

Unidade B

A construo segue os mesmos procedimentos da unidade A, com


a diferena do lado pelo qual inicia-se a dobra.
Com uma pea retangular
ABCD, respeitando as propores, leve o vrtice C ao
A.
O eixo de rotao ser
chamado de EF .

arquivo_origa
2010/2/26
page 50
Estilo OBMEP

50

 CAP. 7: POLIEDROS DE PLATO DE FACES TRIANGULARES

Leve o vrtice C ao ponto F .

E
B
C
F

Leve o vrtice B sobre AF .

F
D

Vire a pea, de modo que a parte de trs


fique para frente.

F
D

Leve o vrtice A ao ponto E.

arquivo_origa
2010/2/26
page 51
Estilo OBMEP

51

N SEC. 7.2: CONSTRUO DAS UNIDADES

Pegue o vrtice D e dobre pelo eixo do


ponto E.

F
E

Desfaa a dobra pelo eixo EF , de modo


que aparea um paralelogramo.

F
F

Vire a pea, de modo que a parte oculta


volte-se para frente.
E
F

Leve as duas extremidades cujos ngulos


so agudos sobre o lado oposto, fixando os
vrtices com ngulos obtusos.

Obtm-se um losango.
E

Abrindo, tem-se a unidade B.

arquivo_origa
2010/2/26
page 52
Estilo OBMEP

52

7.3

 CAP. 7: POLIEDROS DE PLATO DE FACES TRIANGULARES

Montagem dos Poliedros

Foram produzidas, nas unidades A e B, faces na forma de tringulos equilteros. Com os tringulos equilteros podemos construir
apenas trs poliedros regulares: o tetraedro, o octaedro e o icosaedro,
que so os Poliedros de Plato de faces triangulares.

7.3.1

Tetraedro

Para a construo do tetraedro so necessrios dois mdulos, uma


unidade A e uma unidade B.

Note que em cada unidade temos quatro tringulos equilteros e os


tringulos das pontas no possuem corte. Os cortes formam aberturas
para encaixar os tringulos das pontas e ficaro no lado externo do
poliedro.

arquivo_origa
2010/2/26
page 53
Estilo OBMEP

N SEC. 7.3: MONTAGEM DOS POLIEDROS

53

Encaixe a unidade A em um dos


cortes da unidade B (ou B em A).

Dobre dando forma de um tetraedro


e encaixando todas as pontas.
Conclumos o tetraedro.

7.3.2

Octaedro

Para a construo do octaedro sero necessrios quatro mdulos,


AAAA ou BBBB ou AABB e, para que fique com faces bicolores, so
necessrias duas peas de cada cor.

arquivo_origa
2010/2/26
page 54
Estilo OBMEP

54

 CAP. 7: POLIEDROS DE PLATO DE FACES TRIANGULARES

Tome duas unidades A (ou B), de


cores distintas.

Encaixe em uma pea A na outra


pea conforme a foto.
Repita com outras duas peas
restantes.

Forma-se ento, duas pirmides de


base quadrada com abas triangulares em lados opostos da base.

arquivo_origa
2010/2/26
page 55
Estilo OBMEP

N SEC. 7.3: MONTAGEM DOS POLIEDROS

55

Encaixe as duas pirmides para finalizar o octaedro.

7.3.3

Icosaedro

Para a construo do nosso icosaedro bicolor so necessrios cinco


mdulos de cada tipo e cor, ou seja, cinco unidades A com cor 1 e
cinco unidades B com cor 2. Teremos uma faixa cilndrica com dez
faces bicolores e fechados com cinco faces de cor 1 de um lado e cinco
faces de cor 2 do outro lado. No possvel obter todas as faces
bicolores.

arquivo_origa
2010/2/26
page 56
Estilo OBMEP

56

 CAP. 7: POLIEDROS DE PLATO DE FACES TRIANGULARES

Inicia-se com duas unidades distintas, A e B.

Encaixe a unidade B na unidade A.

Repita o procedimento anterior,


encaixando a pea A na B, depois a
B na A, sucessivamente.
Para
facilitar
a
montagem,
recomenda-se que cole com alguma fita adesiva todos os encaixes
na parte interna.

arquivo_origa
2010/2/26
page 57
Estilo OBMEP

N SEC. 7.3: MONTAGEM DOS POLIEDROS

Encaixadas todas as peas, encaixe


a ltima pea, no caso a pea B, na
primeira pea A, dando um formato
cilndrico.

Com a faixa cilndrica pronta,


concentre-se nas pontas triangulares
de um dos lados. Encaixe um tringulo em outro adjacente sucessivamente, at fechar o lado com as cinco
faces.

57

arquivo_origa
2010/2/26
page 58
Estilo OBMEP

58

 CAP. 7: POLIEDROS DE PLATO DE FACES TRIANGULARES

Repita o passo anterior no outro


lado do cilindro, finalizando o
icosaedro.

7.4

Esqueleto do Icosaedro

No podemos deixar de apresentar o esqueleto do icosaedro, constitudo de trs retngulos ureos encaixantes. Isto porque no icosaedro, cada cinco faces triangulares com um vrtice em comum determina um pentgono regular, cuja diagonal o lado maior do retngulo
ureo. O lado menor uma aresta do icosaedro (do conjunto de cinco
faces correspondente a outro vrtice).
J vimos, junto com a construo do primeiro pentgono, a construo do retngulo ureo de lado maior 2, que pode ser usada aqui.

arquivo_origa
2010/2/26
page 59
Estilo OBMEP

59

N SEC. 7.4: ESQUELETO DO ICOSAEDRO

Recortados os retngulos com as devidas propores, passamos


para o seguinte:
D

Seja E = ponto mdio de AB.


Sejam EF e F G como na figura,
AD
com EF = F G =
2
Recorte por EF e F G.

Encaixe duas peas.


Encaixe a terceira pea.
Est pronto o esqueleto
do icosaedro.

Vemos que ligando os


vrtices da estrutura
com segmentos obtemos
um icosaedro.

Se estiver familiarizado com coordenadas cartesianas {O, x, y, z} no


espao, encontre os vrtices de um icosaedro com centro na origem do
espao, como exerccio. Escolha a posio e o tamanho, de forma a
facilitar as contas.

arquivo_origa
2010/2/26
page 60
Estilo OBMEP

Captulo 8

Poliedro de Plato de Faces


Quadradas
Certamente o nico poliedro de face quadrada o cubo, ou hexaedro regular.
Existem diversas formas de se montar um cubo com Origami. Esta
foi uma forma utilizada para visualizar bem o seu esqueleto, numa
montagem de poliedros encaixantes.
A

Comece com um papel quadrado


ABCD.
Leve B e C aos vrtices A e D respectivamente.

60

arquivo_origa
2010/2/26
page 61
Estilo OBMEP

61
AB

DC

Obtm-se o segmento EF .
E

FC

EB

Dobre de modo que leve B a E e C


a F , mantendo A e D no lugar. Gire
180 .
A

Vire o papel de modo que a parte


opaca fique na frente.
Dobre AD sobre EF , obtendo o
vinco GH e volte.
Leve todos os vrtices sobre GH,
mantendo G e H fixos, obtendo os
segmentos IL em EF , I L em BC
e JK em AD. A pea resultante
forma um feixe de trs trapzios em
GH.
O trapzio I L HG est atrs do
trapzio ILHG e o quadriltero
IJKL um quadrado.
Dobre as diagonais e os lados IJ e
KL do quadrado IJKL.

EB

FC

EB

I I

L L

FC
H

arquivo_origa
2010/2/26
page 62
Estilo OBMEP

62

 CAP. 8: POLIEDRO DE PLATO DE FACES QUADRADAS

Leve JK sobre IL e . . .

. . . e obtenha o trapzio triplo.

Abra pelo centro da base maior,


como na dobradura de um barco.

Abra adequadamente, e faa outra


pea igual. Juntando, forme o cubo.

Duas peas so suficientes se o cubo estiver com um esqueleto,


que pode ser um octaedro estrelado. Neste caso, deve-se tomar o
cuidado de verificar antes qual deve ser a medida da aresta do cubo
para envolver o esqueleto. E observe que na construo acima, se u
a aresta do papel quadrado, o cubo tem aresta igual diagonal do
u
quadrado de aresta .
4

arquivo_origa
2010/2/26
page 63
Estilo OBMEP

Captulo 9

Poliedro de Plato de Faces


Pentagonais
Veremos agora, a construo de um poliedro regular cujas faces so
pentgonos. Como o ngulo interno do pentgono de 108 em cada
vrtice, no existe a possibilidade de unio de mais de trs pentgonos,
restando assim o dodecaedro como nica soluo.

9.1

Do copo ao Pentgono Regular

Para a construo do dodecaedro temos que comear com um


papel de um tamanho especial, mas antes faremos uma anlise em
outra construo, a de um copo.
63

arquivo_origa
2010/2/26
page 64
Estilo OBMEP

64

 CAP. 9: POLIEDRO DE PLATO DE FACES PENTAGONAIS

Comeando com um retngulo


ABCD, dobre a diagonal BC.

Encontrando o ponto E,
interseco de AD com BC,
dobre por EB e ED, de
modo que no prenda a parte
oposta.
Observe que o tringulo
BED issceles.

11111111111111111111111111111
00000000000000000000000000000
00000000000000000000000000000
11111111111111111111111111111
00000000000000000000000000000
11111111111111111111111111111
00000000000000000000000000000
11111111111111111111111111111
00000000000000000000000000000
11111111111111111111111111111
00000000000000000000000000000
11111111111111111111111111111
00000000000000000000000000000
11111111111111111111111111111
00000000000000000000000000000
11111111111111111111111111111
E
A 1111111111
0000000000
00000000000000000000000000000D
000000000011111111111111111111111111111
1111111111
0000000000
1111111111
0000000000
1111111111
0000000000
1111111111
0000000000
1111111111
0000000000
1111111111
0000000000
1111111111
0000000000
1111111111
0000000000
1111111111
0000000000
1111111111
0000000000
1111111111
0000000000
1111111111
0000000000
1111111111
0000000000
1111111111
0000000000
1111111111
0000000000
1111111111
0000000000
1111111111
0000000000
1111111111
0000000000
1111111111
0000000000
1111111111
0000000000
1111111111
0000000000
1111111111
0000000000
1111111111
0000000000
1111111111
0000000000
1111111111
0000000000
1111111111
0000000000
1111111111

b
Dobre agora a bissetriz de B.
B

arquivo_origa
2010/2/26
page 65
Estilo OBMEP

65

N SEC. 9.1: DO COPO AO PENTGONO REGULAR

Encontrado o ponto F , interb com


seco da bissetriz de B
ED, dobre a mediatriz de BF ,
donde surge o segmento GH, com
G EB e H BD.

G
H

Como GH mediatriz de BF ,
dobre levando B a F .

G
H

Seguindo os mesmos procedimentos do vrtice B, dobre D sobre


G.
Um pentgono possivelmente irregular, mas simtrico, est
pronto.
Para terminar o copo, dobre o
vrtice E pelo eixo de rotao
GF .
Note que existem duas folhas,
sendo uma dobrada para frente e
outra para trs.

F
B

G
D

I
H

F
B

G
D

E
H

arquivo_origa
2010/2/26
page 66
Estilo OBMEP

66

 CAP. 9: POLIEDRO DE PLATO DE FACES PENTAGONAIS

E est pronto o copo.

Faremos agora uma anlise sobre a construo do copo, para deduzirmos o que necessrio para obtermos um pentgono regular.
E

Ao desdobrarmos a construo do copo teremos as linhas


de dobras.
Pela construo, sabemos que
GH mediatriz de BF e este
b
por sua vez bissetriz de E BD.

G
H

Sabemos que para que o pentgono seja regular, cada ngulo interno deve medir 108o . Como o tringulo HBG issceles, temos
b tambm mede 108o .
que o E GH

Seja E BD obtido onde


E levado na dobra por BF .
Por construo temos que
EG = HE .
b
Como BF bissetriz de E BE
e E e E so equidistantes de
B, tem-se que F equidistante
de E e E .

G
M

108

72

18
18

arquivo_origa
2010/2/26
page 67
Estilo OBMEP

67

N SEC. 9.1: DO COPO AO PENTGONO REGULAR

Por Tales, temos que prob corlongando BE, o B ED


b medindo
respondente ao E BC,
72o . Pelo tringulo EBF ,
temos que EF B = 54o , assim
como BF E . E finalmente o
F E B = 108 , por ser ngulo
interno do pentgono.

B
72

54

108
108

54

108
108

72

72

Temos ento um pentgono com todos os ngulos de 108o . Resta


provar que os lados so congruentes. Por construo, E e H sendo
b
os refletidos de E e G pela bissetriz BF de E BD,
os segmentos GE

e HE so congruentes, assim como EF e F E . Na construo, GD


b e os segmentos EF e HE , assim como
tambm bissetriz de ADB
EG e GH so congruentes.

Ou seja, para obtermos um pentgono regular devemos encontrar


um ngulo exato. E esse ngulo exato deve ser o ngulo entre a base
(o lado maior) e sua diagonal, devendo ter exatamente 36o , isto ,
se o lado maior for equivalente a 1, o lado menor dever ser igual a
tan 36 = 0, 726.... Ou ento, se o lado menor for equivalente a 1, o
lado maior dever ser igual a tan 54 = 1, 376....

arquivo_origa
2010/2/26
page 68
Estilo OBMEP

68

 CAP. 9: POLIEDRO DE PLATO DE FACES PENTAGONAIS


tg 54 = 1,376...

tg 36 = 0,726...

Vimos, na primeira construo pentgono regular, que iniciamos


com um quadrado de suposto lado 2. Verificamos tambm que a diagonal desse pentgono mantinha a mesma medida do lado do quadrado

inicial e o lado do pentgono media 5 1. Observe que aplicando


essas medidas na construo do copo, adotando j os devidos ngulos,
temos:
GF tem, por construo,
mesma medida de BG. Como
BG diagonal do pentgono,
vamos supor que mede 2. Assim, o lado do pentgono mede

5 1 e tambm pela construo, E D = 2 = F D.

5 1

5 1

G
5 1

2
2

5 1

b
Sabemos portanto a medida do ngulo C BD,
quanto deve medir
a diagonal. Precisamos saber agora, quanto mede um dos lados do
retngulo. Para isso, basta saber quanto mede AE.

arquivo_origa
2010/2/26
page 69
Estilo OBMEP

69

N SEC. 9.2: DODECAEDRO

Sabemos que ABE = 18

e que BG = 2 e GE = 5 1.
Ento:

AE = 1

5 1

5 1

AE
= sen18
BE
AE

= 0, 309...
5+1

E
5 1

18 2

H
2

Como construir um retngulo com essas propores, fixando o


tamanho da diagonal 2 do pentgono? Este um outro problema,
que vamos apresentar depois da montagem do dodecaedro.

9.2

Dodecaedro

Podemos finalmente montar o dodecaedro. Atravs da dobradura


do copo, iniciado com papel com as medidas especiais, na qual obtmse os pentgonos regulares, tome a seguinte pea:

Como o dodecaedro composto de 12 pentgonos, precisaremos


de 12 peas para a construo.

arquivo_origa
2010/2/26
page 70
Estilo OBMEP

70

 CAP. 9: POLIEDRO DE PLATO DE FACES PENTAGONAIS

Os tringulos das peas se encaixam em um dos lados do pentgono central da outra pea. preciso encaixar em uma certa sequncia para que fiquem firmes, sem necessidade de utilizao de cola
ou qualquer outro material adesivo. Na figura abaixo, a linha com a
seta indica por onde a ponta da pea deve entrar e at onde ela deve
chegar.

Notem que a ponta da pea, que composta pelos lados congruentes de um tringulo issceles fazem o papel de duas diagonais
do pentgono. E finalmente...

arquivo_origa
2010/2/26
page 71
Estilo OBMEP

N SEC. 9.2: DODECAEDRO

9.2.1

71

Construo do Retngulo para o Dodecaedro de


Aresta Dada

Na construo de poliedros encaixantes aparece o problema de


construir os poliedros com medidas predeterminadas. O dodecaedro
pode ser construdo envolvendo um esqueleto do tipo icosaedro estrelado (icosaedro mais tetraedros em cada face) e isto determina as
dimenses do dodecaedro.
Vimos na construo do dodecaedro que so necessrios retngulos
especiais, cuja diagonal divida o ngulo reto em ngulos de 54o e
36o . Veremos que esses retngulos so construtveis sem ajuda de um
transferidor, e com a medida desejada na diagonal do pentgono, que
continuar sendo chamada de 2.

arquivo_origa
2010/2/26
page 72
Estilo OBMEP

72

 CAP. 9: POLIEDRO DE PLATO DE FACES PENTAGONAIS

Nosso objetivo ser


encontrar um retngulo

com lado igual a 5 + 2

e diagonal 5 + 3.

5 1
2
2

Comece
com
retngulo com
medidas 6 4.

um
as

Dobre um quadrado de 2 2.

Depois dobre um retngulo 2 1 e a sua diagonal.


Essa diagonal corresponde a

5.

5 1

arquivo_origa
2010/2/26
page 73
Estilo OBMEP

N SEC. 9.2: DODECAEDRO

Abra o papel.
Podemos verificar que
cada segmento na horizontal equivale a 2, na
vertical 1 e na diagonal

5.

Dobre o segmento 5
sobre o lado e anote,
conforme a figura.

Dobre o segmento 1 sobre o lado que estamos construindo um dos


lados do retngulo e
transfira-o somando a

2 + 5.

73

arquivo_origa
2010/2/26
page 74
Estilo OBMEP

74

 CAP. 9: POLIEDRO DE PLATO DE FACES PENTAGONAIS

Encontramos os segmentos 2 + 5 e 3 + 5, ou seja, o lado e a


diagonal do retngulo, dados suficientes para a construo o retngulo.

Dobre agora uma perpen


dicular ao lado 2 + 5. Encontre o ponto que dista

3+ 5 do vrtice e pertence
perpendicular.

Temos assim um retngulo

cujo lado mede 2 + 5 e

a diagonal 3 + 5. Basta
agora, seguirmos a construo do copo e obtermos as
faces do dodecaedro.

arquivo_origa
2010/2/26
page 75
Estilo OBMEP

Captulo 10

Construes que se
Encaixam
Na foto ao lado so apresentadas, de
baixo para cima:
o esqueleto do icosaedro;
o icosaedro;
o icosaedro estrelado (icosaedro
com uma pirmide em cada face,
que pode ser construda peas
especiais de Origami, mas no o
faremos aqui), que o esqueleto
do dodecaedro;
o dodecaedro.
Estas peas se encaixam perfeitamente, na ordem acima.
75

arquivo_origa
2010/2/26
page 76
Estilo OBMEP

76

 CAP. 10: CONSTRUES QUE SE ENCAIXAM

Pode-se construir tambm com Origami outro conjunto de peas


encaixantes: esqueleto do octaedro, octaedro, octaedro estrelado e
envolvendo todos eles, o cubo.
Observe que esses encaixes so possveis devido dualidade entre
dodecaedro e icosaedro, e entre cubo e octaedro. A cada face do
dodecaedro corresponde um vrtice do icosaedro e vice-versa. A cada
face do cubo corresponde um vrtice do octaedro e vice-versa.

arquivo_origa
2010/2/26
page 77
Estilo OBMEP

Palavras Finais
Este trabalho foi baseado em um trabalho de concluso de curso
de Eduardo Cavacami no Curso de Licenciatura em Matemtica da
Universidade Federal de So Carlos e depois apresentado como Oficina
na IV Bienal da SBM, em 2008.
No decorrer do desenvolvimento do trabalho foi observado que
muitas pessoas associam Origami com dobraduras de animais, flores
e outras formas, mas nunca Geometria. Talvez este seja um dos
motivos do pouco uso no ensino. Mas o fato deste tipo de atividade
atrair a ateno tanto de crianas quanto de jovens e adultos, faz
pensar no mtodo como uma importante opo para o ensino.
Outra contribuio dessa metodologia pode ser na educao de
pessoas com problemas de viso, pois com a manipulao envolvida no
Origami, os elementos geomtricos podem ser melhor compreendidos.
Este trabalho uma pequena parte de um universo que h para
ser estudado. Mas o intuito deste trabalho foi mostrar a Matemtica
escondida em uma simples dobra, mostrando assim, mais um material
para o escasso campo do ensino da Matemtica.

77

arquivo_origa
2010/2/26
page 78
Estilo OBMEP

Referncias Bibliogrficas
[1] ABE, Hisashi. Sugoiz Origami. Tkio: Nippon Hyoronsha Co.
Ltd., 2003.
[2] CAVACAMI, Eduardo. Aplicaes do Origami com recortes
como formas de ensino. Trabalho de Graduao, UFSCar, 2007.
[3] CAVACAMI, Eduardo; FURUYA, Yolanda K. S. Explorando
Geometria com Origami. Oficina apresentada na IV Bienal da
SBM, em Maring, 2008. Disponvel em http:
//www.dm.ufscar.br/~yolanda/origami/origami2008.pdf
[4] HULL, Thomas. Origami Mathematics.
http://mars.wnec.edu/~th297133/origamimath.html
[5] KNOTT, Ron. Some Solid (Three-dimensional) Geometrical
Factos about the Golden Section. Publicado 1996 e atualizado em
2007. http://www.mcs.surrey.ac.uk/Personal/R.Knott/
Fibonacci/phi3DGeom.html.
78

arquivo_origa
2010/2/26
page 79
Estilo OBMEP

REFERNCIAS BIBLIOGRFICAS

79

[6] LANG, Robert J. Origami and Geometric Constructions.


http://www.langorigami.com/science/hha/origami_
constructions.pdf.
[7] PEDONE, Nelma M.D. Poliedros de Plato. Revista do
Professor de Matemtica, Rio Grande, n. 15. (CD-Rom com 60
edies da Revista do Professor de Matemtica.)

OBMEP

NVEL 1

1a Lista

NVEL 1 - 1a Lista

1) Quando Joana entrou em sua sala de aula, a professora estava apagando o quadro

negro, mas ela ainda pde ver algo escrito, conforme mostra a figura. Qual o nmero
que foi apagado?
A) 8
B) 9
C) 11
D) 12
E) 13
2) Numa papelaria, pacotes com 500 folhas de papel, cada um, so armazenados em

pilhas de 60 pacotes. Cada folha de papel tem espessura de 0,1mm . Ignorando a


espessura do papel utilizado para embrulhar os pacotes, o que podemos afirmar sobre
a altura de uma pilha?
A) aproximadamente a sua altura.
B) aproximadamente a altura de um beb de um ano.
C) aproximadamente a altura de uma mesa comum.
D) aproximadamente a altura de um prdio de dez andares.
E) aproximadamente a altura de uma sala de aula.
3) Considere dois nmeros naturais, cada um deles com trs algarismos diferentes. O

maior deles s tem algarismos pares e o menor s tem algarismos mpares. Se a


diferena entre eles a maior possvel, qual essa diferena?
A) 997

B) 777

C) 507

D) 531

E) 729

4) Uma farmcia d desconto de 30%, sobre o preo de tabela, em todos os medicamentos

que vende. Ao adquirir um remdio cujo preo de tabela 120 reais, quanto uma
pessoa ir pagar com esse desconto?
A) 36 reais

B) 84 reais

C) 64 reais

D) mais de 116 reais E) 94 reais

5) Quatro cidades A, B, C e D, foram construdas beira de uma rodovia reta, conforme a

ilustrao abaixo:
A

A distncia entre A e C de 50 km e a distncia entre B e D de 45 km . Alm disso, sabese que a distncia entre a primeira e a ltima de 80 km . Qual a distncia entre as
cidades B e C?
A) 15 km
B) 20 km
C) 25 km
D) 5 km
E) 10 km
Olimpada Brasileira de Matemtica das Escolas Pblicas

OBMEP
6) Na tabela a seguir vemos o consumo mensal de gua de uma famlia, durante os 5

primeiros meses de 2004.


Meses

Consumo
(m3)

Janeiro
Fevereiro
Maro
Abril
Maio

Qual o consumo mdio mensal dessa famlia de janeiro a maio?


A) 11,3m3

12,5
13,8
13,7
11,4
12,1

B) 11, 7 m3
C) 12, 7 m3
D) 63,5m3
E) 317,5 m3

7) Escreva os nmeros de 0 a 9 nos crculos ao lado, de

forma que eles cresam


seguida, subtraia 1 dos
aos nmeros pares.
consecutivos, qual a
obter?
A) 19

B) 21

no sentido anti-horrio. Em
nmeros mpares e some 1
Escolhendo trs crculos
maior soma que se pode

C) 23

D) 24

E) 25

8) Na malha quadriculada a seguir, todas as circunferncias tm o mesmo centro. Ento,

pode-se concluir que a rea cinza :


.
A) Dois quintos da rea do crculo maior.
B) Trs stimos da rea do crculo maior.
C) Metade da rea do crculo maior.
D) Quatro stimos da rea do crculo maior.
E) Trs quintos da rea do crculo maior

9) A prefeitura de uma certa cidade fez uma campanha que permite trocar 4 garrafas de
1 litro vazias por uma garrafa de 1 litro cheia de leite. Quantos litros de leite pode obter

uma pessoa que possua 43 dessas garrafas vazias fazendo vrias trocas?
A) 11

B) 12

C) 13

D) 14

E) 15

10) Ester vai a uma papelaria para comprar cadernos e canetas. Nesta papelaria os

cadernos custam R$ 6, 00 cada um. Se ela comprar 3 cadernos, sobram R$ 4, 00 . Se o seu


irmo lhe emprestar R$ 4, 00 , com o total ela conseguir comprar 2 cadernos e outras 7
canetas iguais.
a) Quanto custa cada caneta?
b) Se ela comprar 2 cadernos e no pedir dinheiro emprestado,
quantas das canetas acima Ester poder comprar?

Olimpada Brasileira de Matemtica das Escolas Pblicas

OBMEP

SOLUES

1a Lista

Solues-1a Lista

1. (D) Soluo 1 - Como 96 8 = 12 , temos 8 12 = 96 .


Observe que a soluo equivalente a resolver a equao 8 x = 96 , cuja raiz x =

96
= 12 .
8

Soluo 2 - Devemos encontrar na lista de cinco opes qual o nmero que multiplicado
por 8 d 96 . O algarismo das unidades deste nmero s pode ser 2 ou 7 .
Logo, s pode ser o nmero 12 .

2.

(E) Como a espessura de cada folha 0,1mm , a altura de um pacote com 500 folhas
500 0,1 mm = 50 mm . Logo, a altura de cada pilha ser 60 50 mm = 3000mm = 3m .

3. (E) Para que a diferena seja a maior possvel devemos escolher o maior nmero de 3
algarismos pares diferentes e o menor nmero de 3 algarismos mpares diferentes. O maior
nmero de 3 algarismos pares diferentes 864 e o menor nmero de 3 algarismos mpares
diferentes 135 . A diferena entre eles 864 - 135 = 729 .
4. (B) Soluo 1 - A pessoa ir pagar 120 reais menos o desconto que de 30 % sobre 120 . Ou seja:
120 - 0, 3 120 = 120 - 36 = 84 reais.
Soluo 2- Podemos tambm resolver este problema notando que se o desconto de 30 %
ento o preo que a pessoa pagar 70 % de 120 , ou seja: 0,7 120 = 84 reais.

5. (A) Soluo 1 - Temos CD = 80 - 50 = 30 e AB = 80 - 45 = 35 . Logo BC = 80 - 35 - 30 = 15 .


Soluo 2 Do enunciado temos: AC = 50 , BD = 45 e AD = 80 . Da figura
segue que BC = AC AB , logo BC = 50 AB .
Logo, basta calcular AB . Para isso, note na figura que
AB = AD BD , e portanto, AB = 80 - 45 = 35 .
Finalmente, BC = 50 - 35 = 15 km .
Soluo 3 Da figura temos que 45 - BC = 80 50 . Logo, BC= 15 km .
6. (C) Lembre que a mdia aritmtica de n nmeros a soma desses nmeros dividido por n. Por
exemplo: a mdia aritmtica dos nmeros 3, 6, 8 e 26

3 + 6 + 8 + 26 43
=
= 10,75 .
4
4

Analogamente, define-se o consumo mensal mdio como a razo entre a soma dos consumos mensais e
12, 5 + 13,8 + 13,7 + 11, 4 + 12,1
= 12,7 m3 .
o nmero de meses. Logo, o consumo mensal mdio igual a
5
Olimpada Brasileira de Matemtica das Escolas Pblicas

OBMEP

7. (C) A partir de qualquer crculo, obtemos inicialmente a seqncia 0, 1, 2, 3, 4, 5, 6, 7, 8, 9;

Subtraindo 1 dos mpares e somando 1 aos pares,


a seqncia torna-se 1, 0, 3, 2, 5, 4, 7, 6, 9, 8. Agora
fcil verificar que a maior soma possvel com 3
nmeros consecutivos 6 + 9 + 8 = 23 .

8. (C) Observe que a figura simtrica em relao reta r que passa pelo centro comum das
circunferncias. Para cada regio cinza de um lado de r existe uma regio branca equivalente do
outro lado de r , e vice-versa. Logo, a rea cinza igual rea branca. Alm disso, a soma dessas
duas reas igual rea do crculo maior. Portanto, a rea cinza metade da rea do crculo
maior.

9. (D) Como 43 = 10 4 + 3 , numa primeira vez, as 43 garrafas vazias


podem ser trocadas por 10 garrafas cheias, sobrando ainda 3 vazias.
Agora, consumindo o leite dessas 10 garrafas, ficamos com 13 vazias,
13 = 4 3 + 1 , que podem ser trocadas desta vez por 3 cheias, sobrando
1 vazia. Finalmente, consumindo o leite das 3 garrafas cheias, sobram
4 vazias, que podem ser trocadas por 1 cheia. Portanto, o total de
garrafas cheias de leite que podem ser obtidas 10 + 3 + 1 = 14 .

10. Comprando 3 cadernos por 6 reais cada um ainda sobram 4 reais para Ester, logo, a quantia
que ela possui :
3 6 + 4 = 22 reais.

(a) Se o irmo lhe empresta 4 reais, ela fica ento com 22 + 4 = 26 reais. Conforme dados do
problema, com 26 reais, Ester pode comprar 2 cadernos a 6 reais cada um e 7 canetas. Portanto,
o preo das 7 canetas 26 - 2 6 = 26 - 12 = 14 reais. Conclumos que o preo de cada caneta
14 7 = 2 reais.
(b) Como Ester tinha 22 reais, se ela comprar 2 cadernos, sobram-lhe ainda 22 - 2 6 = 22 - 12 = 10
reais. Como cada caneta custa 2 reais, ela poder comprar 10 2 = 5 canetas.

10

Olimpada Brasileira de Matemtica das Escolas Pblicas

OBMEP

NVEL 1

2a Lista

1) Um pedreiro capaz de assentar 8 metros de muro por dia. Quantos metros de muro
esse pedreiro consegue assentar em 15 dias?
A) 104

B) 110

C) 120

D) 128

E) 112

2) A balana da figura est em equilbrio com bolas e


saquinhos de areia em cada um de seus pratos. As
bolas so todas iguais e os saquinhos tambm. O peso
de um saquinho de areia igual ao peso de quantas
bolas?
A) 1

B) 2

A)

3 4 2
; ;
7 9 5

C) 3

D) 5

E)

3) Trs frascos, todos com capacidade igual a um litro, contm


quantidades diferentes de um mesmo lquido, conforme
ilustrao ao lado. Qual das alternativas abaixo melhor expressa,
aproximadamente, o volume de lquido contido nos frascos A, B e
C, nesta ordem?
B)

2 1 1
; ;
3 2 4

C)

2 4 2
; ;
3 6 4

D)

2 4 3
; ;
3 7 4

3 4 2
3 5 3

E) ; ;

4) Um litro de lcool custa R$ 0, 75 . O carro de Maria percorre 25 km com 3 litros de


lcool.Quantos reais Maria gastar com lcool para percorrer 600 km ?
A) 54

B) 72

C) 50

D) 52

E) 45

5) Num armazm foram empilhadas algumas caixas que formaram


o monte mostrado na figura. Se cada caixa pesa 25 kg quanto pesa o
monte com todas as caixas?
A) 300 kg

B) 325 kg

C) 350 kg

D) 375 kg

Olimpada Brasileira de Matemtica das Escolas Pblicas

E) 400 kg

11

OBMEP

6) Um livro de 100 pginas tem suas pginas numeradas de 1 a 100 . Quantas folhas desse
livro possuem o algarismo 5 em sua numerao?
A) 13

B) 14

C) 15

D) 16

E) 17

7) A figura abaixo foi desenhada em cartolina e dobrada de modo a formar um cubo.


Qual das alternativas mostra o cubo assim formado?

A)

B)

C)

E)

D)

8) Jos colou uma bandeirinha em cada um dos dois discos


dentados que formam uma engrenagem, como mostra a figura
ao lado:
Os dois discos so exatamente iguais. Jos girou a engrenagem, e claro que as
bandeirinhas mudaram de posio. Qual a nova posio das duas bandeirinhas?

A)

B)

C)

D)

E)
9) O desenho ao lado a planta de uma casa, cujo piso
retangular, e no qual esto desenhados 7 quadrados
numerados de 1 a 7 na figura. Se a rea do menor desses
quadrados 1m2, a rea total do piso, em metros
quadrados, igual a:
A) 42

B) 44

C) 45

D) 48

E) 49

10) O nmero da casa de Jlia tem exatamente trs algarismos, cuja soma 24 . Encontre
todos os possveis nmeros da casa de Jlia, em cada uma das situaes a seguir:
a) Os trs algarismos so iguais.
b) Os algarismos so todos diferentes.
c) Apenas dois algarismos so iguais.
12

Olimpada Brasileira de Matemtica das Escolas Pblicas

OBMEP

SOLUES

Solues - 2a Lista

1.(C)

Se

pedreiro

assenta

metros

por

2a Lista

dia,

em

15 dias

assentar

15 8 = 120 .

2. (B) Retirando-se dois saquinhos e quatro bolas de cada prato, a balana continua equilibrada, e
restam 3 saquinhos no prato esquerda e 6 bolas no prato da direita. Logo:
peso de 3 saquinhos = peso de 6 bolas.
Da, conclumos que o peso de 1 saquinho igual ao peso de 2 bolas.
Esta soluo corresponde a explicitar x em funo de y na equao 5x + 4 y = 2 x + 10 y , onde x
representa o peso de um saquinho e y o de uma bola. Desta equao segue que:
5x - 2 x = 10 y - 4 y 3 x = 6 y x = 2 y .

3. (B) Soluo 1 - As figuras mostram que os volumes ocupados pelos lquidos correspondem,
aproximadamente a mais da metade no frasco A, a metade no frasco B e menos da metade no
frasco C.
O nico grupo de fraes que corresponde a essas estimativas :
2
1
1
(mais que a metade); (metade); (menos que a metade) .
3
2
4

Soluo 2 As figuras mostram que os volumes ocupados pelos lquidos so nmeros


decrescentes. As nicas opes possveis so B e E. Como

3
3

= 1 e nenhum frasco est cheio, a

resposta B.
4. (A) Soluo 1 - Se num percurso de 25 km ela gasta 3 litros, ento para percorrer 100 km , Maria
gastar 4 3 = 12 litros. Portanto, para percorrer 600 km o carro gastar 6 12 = 72 litros. Como cada
litro custa 0, 75 reais, ento 72 litros custaro 0,75 72 = 54 reais.
Soluo 2 - Observe que podemos usar a Regra de Trs para calcular quantos litros so gastos em
600 km :
3 litros

x litros

______

25 km

______

600 km

Como esta regra de trs direta temos: 25 x = 3 600 x = 3

600
= 72 litros.
5

Soluo 3 - Como 600 = 25 24 , temos que o carro gastar 24 3 = 72 litros.

Olimpada Brasileira de Matemtica das Escolas Pblicas

13

OBMEP

5. (C) Na figura, vemos: 1 coluna com 3 caixas, 4 colunas com 2 caixas e 3 colunas com uma
caixa. Logo, o total de caixas 1 3 + 4 2 + 3 1 = 14 . Como cada caixa pesa 25 kg , o peso do monte
de caixas 14 25 = 350 kg.

6. (C) O algarismo 5 aparece nos nmeros 5, 15, 25, 35, 45, 50, 51, 52, 53, 54, 55, 56, 57, 58, 59, 65, 75, 85 e 95 .
Agora, como o livro numerado de 1 a 100 , a 1a folha contm as pginas 1 e 2 , a 2a folha as pginas 3
e 4 , a 3a folha as pginas 5 e 6 , e assim sucessivamente. Ou seja, as duas pginas que compem cada
folha tm a seguinte numerao: um nmero mpar e o nmero par consecutivo.
1,2; 3, 4 ; ; 47, 48 ; 49, 50 ; 51, 52 ; ; 59,60 ; ; 95,96 ; 97,98 ; 99,100 .

Assim, esto numa mesma folha as seguintes duplas de nmeros: 49, 50 ; 51, 52 ; 53, 54 ; 55, 56 ;
57, 58 ; 59,60 . Logo, neste grupo temos 6 folhas. Por outro lado, de 1 a 48 temos 5 folhas com o

algarismo 5 , e de 61 a 100 , 4 folhas. Portanto, o total de folhas contendo o algarismo 5 em sua


numerao : 6 + 5 + 4 = 15 .

7. (B) As opes A e E; C e D so iguais entre si e distintas de (B).

8. (A) A engrenagem desta questo formada por dois discos dentados.


Quando um deles gira no sentido horrio, o outro gira no sentido antihorrio.
As 5 opes de resposta mostram a bandeira do disco esquerda numa posio, que corresponde a
uma rotao deste disco no sentido horrio de um certo ngulo. Nesse caso, a engrenagem direita
girou desse mesmo ngulo no sentido anti-horrio, levando a bandeirinha para a posio indicada
na primeira alternativa.
9. (C) Soluo 1 - Como os quadrados menores tm 1m 2 de rea, cada um deles tem lado igual a
1m . Da figura conclumos que BC = 2m e BH = 3m .
Como

um quadrado segue que


BC = CD = AD = AB = 2m . Sendo CDEF tambm um
quadrado, temos CD = DE = 2m . Novamente da figura
temos: AH = AB + BH = 2 + 3 = 5 , JE = JA + AD + DE e
JA = AH .
Segue
que
JE = 5 + 2 + 2 = 9 .
Como
EG = AH = 5 , as dimenses do terreno so 9m de
comprimento por 5m de largura. Portanto, a sua rea
9 m 5m = 45m2 .

14

ABCD

Olimpada Brasileira de Matemtica das Escolas Pblicas

OBMEP

Soluo 2 - Quadriculando o retngulo maior com


quadrados de 1m 2 de rea, obtemos um retngulo
( BFGH ) formado por 12 quadrados de 1m 2 de rea,
dois quadrados ( ABCD e DCFE ) formados por 4
quadrados cada um de 1m 2 de rea, e um quadrado
( AHIJ ) formado por 25 quadrados de 1m 2 de rea.
Portanto, a rea pedida 12 + 4 + 4 + 25 = 45 m2.

10. Nesta questo, o nmero 24 deve ser escrito como uma soma de 3 algarismos. Inicialmente,
note que os algarismos 0, 1, 2, 3, 4 e 5 no podem ser usados. Se um deles fosse usado, por
exemplo o algarismo 5 , ento teramos que encontrar dois algarismos cuja soma 19 , pois
24 - 5 = 19 . Sabemos que isso no possvel. O mesmo ocorre com os algarismos 0, 1, 2, 3 e 4 . Logo,
o nmero da casa de Jlia s pode ser composto pelos algarismos 6, 7, 8 e 9.
a) Se os trs algarismos so iguais ento o nmero da casa 888 .
b) Se os trs algarismos so diferentes, temos apenas as seguintes alternativas:
Iniciando com o algarismo 9:
987 e 978
897 e 879
Iniciando com o algarismo 8:
Iniciando com o algarismo 7:
798 e 789
Note que neste item o nmero da casa no pode iniciar com o algarismo 6 , pois 24 - 6 = 18 , e a
nica maneira de escrever 18 como soma de dois algarismos 9 + 9 , o que daria um nmero com
dois algarismos iguais.
c) Com apenas dois algarismos iguais temos 3 nmeros: 996, 699 e 969 .

Olimpada Brasileira de Matemtica das Escolas Pblicas

15

OBMEP

NVEL 1

3a Lista

Nvel 1- 3Lista

1) O famoso matemtico grego Pitgoras chamou de nmeros triangulares os nmeros


obtidos pela soma dos primeiros nmeros inteiros maiores que 0. Por exemplo, 1, 3 , 6 e 10
so nmeros triangulares:
1 = 1
3 = 1 + 2
6 = 1 + 2 + 3
10 = 1 + 2 + 3 + 4
A figura ilustra a motivao para o nome nmeros triangulares.

1+ 2 = 3

1+ 2 + 3 = 6

1 + 2 + 3 + 4 = 10

A seqncia de nmeros triangulares continua com 1 + 2 + 3 + 4 + 5 = 15 ,


etc. Quantos so os nmeros triangulares menores do que 100?

1 + 2 + 3 + 4 + 5 + 6 = 21 ,

2) Uma bibliotecria recebe 130 livros de Matemtica e 195 livros de Portugus. Ela quer
arrum-los em estantes, colocando igual quantidade de livros em cada estante, sem
misturar livros de Matemtica e de Portugus na mesma estante. Quantos livros ela deve
colocar em cada estante para que o nmero de estantes utilizadas seja o menor possvel?
3) A sexta parte dos alunos de uma classe usam culos. Dentre os que usam culos,

1
3

so

meninas; alm disso, 4 meninos usam culos. Quantos so os alunos dessa classe?

35

12

0,4 0,5

4) Complete as casas em branco da tabela ao lado com fraes de modo


que a soma dos trs nmeros de qualquer linha, qualquer coluna e das
duas diagonais seja sempre a mesma.

5) Sejam A, B e C algarismos diferentes de zero tais que ( AB) 2 = CAB , isto , o nmero de
dois algarismos AB elevado ao quadrado d o nmero de 3 algarismos CAB. Determine o
valor de A + B + C .

16

Olimpada Brasileira de Matemtica das Escolas Pblicas

OBMEP

6) Uma faixa quadriculada tem 5 quadradinhos na largura e 250 quadradinhos no


comprimento.

Alguns quadradinhos sero pintados de cinza, comeando da esquerda, conforme o


modelo ilustrado na figura, e continuando com este padro at chegar ao final da faixa
direita. Quantos quadradinhos no sero pintados?

7) Joo tem, em seu jardim, uma cisterna na qual ele armazena gua de chuva e tira gua
para regar suas flores. meia-noite do dia 31 de dezembro de 2005 a cisterna continha
156 litros de gua. Joo tem o hbito de anotar em um quadro, todo dia, o nmero de litros
de gua gasta para regar as flores e de gua recolhida da chuva. Abaixo vemos parte do
quadro referente aos primeiros dias de 2006 :
Data

1o de janeiro
2 de janeiro
3 de janeiro
4 de janeiro
5 de janeiro
6 de janeiro
7 de janeiro
8 de janeiro

litros de gua
gastos para
regar as flores
6
9
0
4
9
0
11
0

litros de gua
recolhidos da chuva
2,5
0
5
0
3
0
4,5
0

Quantos litros de gua havia na cisterna


do Joo meia noite do dia 8 de janeiro de
2006 ?

Olimpada Brasileira de Matemtica das Escolas Pblicas

17

OBMEP

SOLUES

Solues- Lista 3

3a Lista

1. Notamos que o segundo nmero triangular obtido a partir do primeiro acrescentando-se 2 , o


terceiro obtido do segundo acrescentando-se 3 e assim por diante. Essa observao nos mostra
como calcular os prximos nmeros triangulares sem fazer muitas contas; por exemplo, j sabemos
que o quarto nmero triangular 10 , donde o quinto ser 10 + 5 = 15 , o sexto sendo ento
15 + 6 = 21 . Podemos assim escrever os nmeros triangulares at passar de 100:

+2
+3
+4
+5
+6
+7
+8
+9
+10
1
3
6
10
15
21
28
36
45
55
+11
+12
+13
+14
66 78 91 105

Logo, os nmeros triangulares menores que 100 , so: 1, 3, 6, 10, 15, 21, 28, 36, 45, 55, 66, 78 e 91 .
Assim, temos 13 nmeros triangulares menores do que 100 .

2. Chamemos de n o nmero de livros que a bibliotecria vai colocar em cada estante. Ento temos:
130 n = nmero de estantes para os livros de Matemtica e 195 n = nmero de estantes para os
livros de Portugus.

Isso mostra que n deve ser um divisor comum de 130 e 195 , pois o nmero de estantes utilizadas
inteiro. Sabemos que quando aumentamos o denominador de uma frao, esta frao diminui (por
27

exemplo:
fraes

10
130

menor do que
e

195

27
8

). Logo, quanto maior for o denominador n, menores sero as

, o que significa que menor ser o nmero de estantes utilizadas. Vemos assim

que n deve ser o maior divisor comum (MDC) de 130 e 195 . Como 130 = 2 5 13 e 195 = 3 5 13
segue que o MDC de 130 e 195 5 13 = 65 .

Logo, a bibliotecria vai colocar 65 livros em cada estante. Portanto, o nmero de estantes para os
livros de Matemtica 130 65 = 2 e o nmero de estantes para os de Portugus 195 65 = 3 , o que
d um total de 2 + 3 = 5 estantes.

18

Olimpada Brasileira de Matemtica das Escolas Pblicas

OBMEP

3. Nosso problema aqui achar o nmero de alunos da classe. O enunciado diz que
1

usam culos, e destes

so meninas, isto

1
3

de

1
6

1
6

dos alunos

so meninas. Como

corresponde a
meninas que usam culos

alunos que usam culos

meninos que usam culos

1 1

3 6

1 1 1 1
3
1
2 1
1
=
=

=
= , conclumos que
da classe consiste de meninos que
6 3 6 6 18 18 18 18 9
9

Como

usam culos, que so em nmero de 4. Temos


1
9
9
9

da classe

corresponde a

4 alunos

da classe

corresponde a

4 9 = 36 alunos

Logo, o nmero de alunos na classe 36.


4. Para facilitar nossas contas, conveniente reduzir todas as fraes que
aparecem a um mesmo denominador. Como 0, 4 =

4
10

e 0, 5 =

5
10

, podemos

reescrever a tabela como ao lado, onde indicamos com letras, os nmeros


que devemos calcular.

O problema pede que a soma dos nmeros em qualquer linha ou coluna e


nas duas diagonais seja sempre a mesma. Olhando para a diagonal
destacada no quadrado ao lado
4

vemos que esta soma


4
10
c=

5
10
5

10

+e =

15
10

, donde e =

10

10

6
10

5
10

6
10

15
10

Da 3a linha temos ento

; e da 2 a coluna temos

5
10

5
10

+c =

15
10

, donde

6 10

5 10

4 10

5 10

6 10

5 10

4 10

5 10

5 10

6 10

5 10

4 10

5 10

6 10

4 10

5 10

6 10

5 10

3 10

4 10

5 10

6 10

4 10

5 10

6 10

7 10

5 10

3 10

4 10

5 10

6 10

. Colocando estes valores de c e e no quadrado, obtemos

A 1a linha nos d ento a +


6

+d +

6
10

15
10

, donde d =

5
10
3

10

Do mesmo modo achamos b =

6
10

15
10

, donde a =

4
10

; e da 3a coluna temos

. O quadrado ento fica:

7
10

e o quadrado est completo:

Olimpada Brasileira de Matemtica das Escolas Pblicas

19

OBMEP

5. De acordo com a igualdade ( AB ) 2 = CAB , B o ltimo algarismo (o algarismo das unidades)


de ( AB ) 2 e tambm o ltimo algarismo de B 2 . Logo B um nmero entre 1 e 9 cujo quadrado
tambm tem B como seu ltimo algarismo. Logo, os valores possveis para B so 1, 5 e 6, pois esses
so os nicos algarismos (diferentes de zero) tais que cada um deles e seu respectivo quadrado
tm o mesmo algarismo das unidades:

1 = 1 , 5 = 25 e 6 = 36

CAB um nmero de 3 algarismos, logo menor que 1000 . Por isso, A no pode ser maior
que 3 , porque qualquer nmero da forma (4B)2 j maior do que 1000 . De fato, se A fosse maior
que 3 ento A seria no mnimo 4 ; ento AB seria no mnimo 41 , o que no pode acontecer pois
2
41 = 1681 j maior que 1000 . Logo, os valores possveis para A so 1, 2 e 3.

Vamos analisar cada caso separadamente.

1o caso: B = 1 .
se A = 1 temos 112 = 121 , o que implica CA1 = 121 , donde A = 2 ;
se A = 2 temos 212 = 441 , o que implica CA1 = 441 , donde A = 4 ,
se A = 3 temos 312 = 961 , o que implica CA1 = 961 , donde A = 6 .
Em qualquer caso chegamos a uma contradio, logo o caso B = 1 est excludo.
2o caso: B = 5
Nesse caso, (AB)2 termina em 25; isto ( AB ) 2 = ( A5) 2 = C 25 . Temos ento CA5 = C 25 , donde A = 2 .
Como 252 = 625 , conclumos que C = 6 .

3o caso: B = 6
Aqui, ( AB ) 2 acaba em 36 , isto ( AB ) 2 = ( A6) 2 = C 36 . Logo , CA6 = C 36 donde A = 3 e segue que
2

36 = 1296 um nmero com quatro algarismos, chegamos a uma outra


contradio. Logo exclumos a possibilidade B = 6 .
AB = 36 . Como

Desse modo, a nica possibilidade A = 2, B = 5 e C = 6 , onde temos A + B + C = 13 .

20

Olimpada Brasileira de Matemtica das Escolas Pblicas

OBMEP

6. Para pintar a faixa conforme o modelo, o retngulo padro (aquele que se


repete por toda a faixa) o retngulo de 5 linhas e 4 colunas mostrado na
figura ao lado; nele temos 7 quadradinhos pintados e 13 no pintados.
Precisamos saber quantos retngulos padro cabem na faixa. A faixa tem
250 colunas e cada retngulo padro tem 4 colunas. Da diviso de 250 por
4 temos que 250 = 4 62 + 2 , e conclumos que na faixa cabem 62 retngulos
padro, sobrando ainda duas colunas.

Nos 62 retngulos padro temos 62 13 = 806 quadradinhos no pintados. Falta agora verificar
quais os quadradinhos no pintados nas duas colunas finais. A figura mostra como so as duas
colunas de acordo com o modelo. Nessas colunas temos 6 quadradinhos no pintados. Finalmente,
o nmero de quadradinhos no pintados em toda a faixa 806 + 6 = 812.

7. O dia 1o de janeiro comea com 156 litros de gua na cisterna, e a partir da a cisterna recebe
gua da chuva e perde gua para a rega das flores. Como no dia 8 no houve alterao na
quantidade de gua na cisterna, ento o nmero de litros de gua na cisterna no dia 8
156 + litros de gua de chuva do dia 1 ao dia 7-litros de gua para regar do dia 1 ao dia 7

O enunciado diz que a segunda parcela da expresso acima a soma dos nmeros da 3a coluna,
que 2, 5 + 0 + 5 + 0 + 3 + 0 + 4, 5 = 15 , e a terceira parcela a soma dos nmeros da 2a coluna da
tabela, que 6 + 9 + 0 + 4 + 9 + 0 + 11 = 39 . Logo, o nmero de litros na cisterna meia noite do dia 8
156 + 15 39 = 132 .

Olimpada Brasileira de Matemtica das Escolas Pblicas

21

OBMEP

NVEL 1

4a Lista

1) Da igualdade 9 174 532 13 = 119 268 916 pode-se concluir que um dos nmeros abaixo
divisvel por 13 . Qual este nmero?
A) 119 268 903

B) 119 268 907

C) 119 268 911

D) 119 268 913

E) 119 268 923

2) Arnaldo disse que um bilho o mesmo que um milho de milhes. O Professor Piraldo
o corrigiu e disse, corretamente, que um bilho o mesmo que mil milhes. Qual a
diferena entre o valor correto de um bilho e a resposta de Arnaldo?
A) 1 000

B) 999 000

C) 1 000 000

D) 999 000 000

E) 999 000 000 000

3) Com a energia fornecida por um litro de mel, uma abelha consegue voar 7.000
quilmetros. Quantas abelhas conseguiriam voar 1 quilmetro, cada uma, com a energia
fornecida por 10 litros de mel?
A) 7 000

B) 70 000

C) 700 000

D) 7 000 000

E) 70 000 000

4) Um agricultor esperava receber cerca de R$ 100.000, 00 pela venda de sua safra. Entretanto,
a falta de chuva provocou uma perda da safra avaliada entre

1
5

1
4

do total previsto. Qual

dos valores a seguir pode representar a perda do agricultor?


A) R$ 21.987, 53

1m
1m
1m
1m

B) R$ 34.900, 00

C) R$ 44.999,99

D) R$ 51.987,53

5) Uma placa decorativa consiste num quadrado branco de 4


metros de lado, pintado de forma simtrica com, partes em cinza,
conforme desenho ao lado. Qual a frao da rea da placa que
foi pintada?

1m
1m

22

E) R$ 60.000,00

A)

1
2

B)

1
3

C)

3
8

D)

6
13

E)

7
11

Olimpada Brasileira de Matemtica das Escolas Pblicas

OBMEP

6) Diamantino colocou em um recipiente trs litros de gua e um litro de refresco. O


refresco composto de 20 % de suco de laranja e 80 % de gua. Depois de misturar tudo,
que porcentagem do volume final representa o suco de laranja?
A) 5 %

B) 7 %

C) 8 %

D) 20 %

E) 60 %

7) Nove amigos compraram 3 bolos, cada um deles cortado


em oito fatias. Todos comeram bolo e no sobrou nenhum
pedao. Sabendo que cada um s comeu fatias inteiras do
bolo, podemos ter certeza de que:
A)
B)
C)
D)
E)

Algum comeu quatro fatias.


Um deles comeu somente uma fatia.
Todos comeram duas fatias pelo menos.
Uns comeram duas fatias e os demais comeram trs fatias.
Um deles comeu, no mnimo, trs fatias.

8) Uma seqncia de mosaicos quadrados construda com azulejos quadrados pretos e


brancos, todos do mesmo tamanho, como se segue: o primeiro formado por um azulejo
branco cercado por azulejos pretos, o segundo de quatro azulejos brancos cercados por
azulejos pretos; e assim sucessivamente, como indica a figura. Se numa seqncia de
mosaicos formada de acordo com esta regra forem usados 80 azulejos pretos, quantos
sero os azulejos brancos utilizados?

A) 55

B) 65

C) 75

D) 85

Olimpada Brasileira de Matemtica das Escolas Pblicas

E) 100

23

OBMEP

9) No ltimo campeonato de futebol da escola do Marcelo participaram 6 equipes. Cada


equipe disputou com cada uma das outras exatamente uma partida. Abaixo, a tabela de
classificao do campeonato, onde:
Equipe
A
B
C
D
E
F

V E D GP GC
4 1 0 6
2
2 1 2 6
6
0 3 2 2
6
1 1 y 3
6
0 1 4 1
5
x 1 0 z
3

V o nmero de vitrias de uma equipe


E o nmero de empates
D o nmero de derrotas
GP o nmero de gols feitos por um time
GC o nmero de gols sofridos

a) Quantas partidas foram disputadas?


b) Determine a quantidade de vitrias da equipe F, a quantidade de derrotas da equipe D e
a quantidade de gols feitos pela equipe F, representados por x, y e z na tabela.

10) Um bloco retangular de madeira tem 320 cm de comprimento, 60 cm de largura e 75 cm de


altura. O bloco cortado vrias vezes, com cortes paralelos s suas faces, de modo a
subdividi-lo em blocos tambm retangulares de 80 cm de comprimento por 30 cm de largura
por 15 cm de altura.

a) Quantas peas foram obtidas?


b) Um metro cbico dessa madeira pesa aproximadamente 900 quilogramas.
Qual o peso de cada uma dessas peas?

24

Olimpada Brasileira de Matemtica das Escolas Pblicas

SOLUES

OBMEP

4a Lista

1. (A) Como 119 268 916 divisvel por 13 , j que 9174 532 13 = 119 268 916 ,podemos concluir que os
nmeros divisveis por 13 so aqueles obtidos somando-se ou subtraindo-se mltiplos de 13 ao
nmero 119 268 916 . Dentre os nmeros apresentados, o nmero 119 268 916-13=119 268 903 o
nico divisvel por 13 .
2. (E) Arnaldo disse que 1 bilho = 1 000 000 1 000 000=1 000 000 000 000=1012 . O Professor Piraldo
corrigiu-o,
12

dizendo
9

que

bilho

1 000 1 000 000 = 1 000 000 000=109 .

diferena

10 10 = 10 (10 1) = 999 10 = 999 000 000 000

3. (B) A energia gasta por uma abelha para voar 7.000 quilmetros
gastam para voar 1 quilmetro cada . Como o nmero de litros de
temos energia suficiente para que 10 vezes este nmero de abelhas
seja, 70.000 abelhas. Note que poderamos ter tambm 7.000 abelhas
entre outras alternativas.
4. (A) Como

1
5

de 100 000 =

100 000
5

= 20000 e

1
4

de 100 000 =

a mesma que 7.000 abelhas


mel foi multiplicado por 10 ,
voem 1 quilmetro cada, ou
voando 10 quilmetros cada,

100 000
4

= 25000 , conclumos que a

perda da safra est avaliada entre R$ 20.000, 00 e R$ 25.000, 00 .Logo, um possvel valor para a perda
R$ 21.987,53.
5. (C) Traando paralelas aos lados, podemos dividir a placa em
quadrados de 1 metro de lado, conforme indicado na figura.
Ento, a rea pintada igual a 12 metades desses quadrados, ou,
equivalentemente, 6 desses quadrados. Como a placa total tem
16 desses quadrados, conclumos que a frao da rea pintada
em relao rea da placa

6
16

3
8

6. (A) O refresco composto de 20 % de um litro = 0, 2 litros de


suco e 80 % de um litro = 0,8 litros de gua. Logo, a mistura final
tem 0, 2 litros de suco e 3 + 0,8 = 3,8 litros de gua. A porcentagem de suco em relao ao volume da
mistura ento

volume de suco 0, 2 2
=
=
= 0, 05 = 5% .
volume total
4
40

7. (E) Temos um total de 8 3 = 24 fatias de bolo que foram comidas. Como todos comeram bolo,
inicialmente cada um dos 9 amigos comeu uma fatia. Sobraram ainda 24 9 = 15 fatias para serem
comidas por 9 pessoas. Nesta situao, obrigatoriamente uma certa pessoa X deve ter comido pelo
menos 2 dessas 15 fatias. Caso contrrio, isto , se todas as 9 pessoas tivessem comido menos do
que 2 fatias significaria que poderamos escrever o nmero 15 como uma soma de 9 parcelas cada
uma delas sendo 0 (os que no comeram das 15 fatias) ou 1 (os que comeram 1 fatia das 15 ), o que
claramente no possvel. Como esta pessoa X j havia comido inicialmente 1 fatia, conclumos
que ela comeu no mnimo 3 fatias.

Olimpada Brasileira de Matemtica das Escolas Pblicas

25

OBMEP

8. (A) No primeiro mosaico, temos 3 + 3 + 1 + 1 = 8 azulejos pretos; no segundo, temos 4 + 4 + 2 + 2 = 12 ;


no terceiro, temos 5 + 5 + 3 + 3 = 16 ; no difcil perceber (e verificar) que os prximos mosaicos tm
20 e 24 azulejos pretos. Como 8 + 12 + 16 + 20 + 24 = 80 , possvel construir exatamente 5 mosaicos.
Finalmente, o nmero total de azulejos brancos nesta seqncia de cinco mosaicos :
12 + 2 2 + 32 + 4 2 + 52 = 1 + 4 + 9 + 16 + 25 = 55 .

9. a) Podemos contar, por listagem, quantas foram as partidas: AB, AC, AD, AE, AF, BC, BD,
BE, AF, CD, CE, CF, DE, DF e EF, num total de 15 partidas.
Por outro lado, podemos contar de um modo mais geral, como se segue. Como cada equipe jogou
com todas as outras, segue que cada equipe jogou 5 partidas. Parece ento que o nmero total de
partidas foi de 5 partidas por equipe 6 equipes = 30 partidas . No entanto, nesta contagem cada
partida foi contada duas vezes; por exemplo, o jogo entre os times A e B foi contado como AB e
como BA. Logo, o nmero correto de partidas jogadas

30
= 15 .
2

b) Como vimos no item (a), cada time jogou 5 partidas. Desse modo, a soma do nmero de
vitrias, empates e derrotas de um mesmo time deve ser igual a 5; por exemplo, para o time A,
observamos na tabela 4 vitrias + 1 empate + 0 derrotas = 5 partidas . Aplicando este raciocnio ao time
F, temos x + 1 + 0 = 5 , donde x = 4 . Do mesmo modo, para a equipe D temos 1 + 1 + y = 5 , donde y = 3 .
Notamos agora que em um campeonato de futebol o nmero de gols feitos igual ao nmero de
gols sofridos. Logo 6 + 6 + 2 + 3 + 1 + z = 2 + 6 + 6 + 6 + 5 + 3 , donde 18 + z = 28 , ou seja, z = 10 .
Gols feitos

Gols sofridos

10. a) As dimenses do bloco maior so 320 60 75 e as dos blocos menores 80 30 15 . Logo, no


bloco maior o comprimento foi dividido por 320 80 = 4 , a largura foi dividida 60 30 = 2 e a altura
foi dividida por 75 15 = 5 . Portanto teremos um total de 4 2 5 = 40 peas distribudas em cinco
camadas de oito peas cada, conforme ilustrado no desenho ao lado.
b) O volume de um bloco retangular dado por
comprimento largura altura. Logo, o volume de cada
um dos blocos menores 80 30 15 = 36000 cm3 . O
enunciado do problema nos d o peso de um metro cbico
de madeira; para saber o peso de um dos blocos pequenos
devemos primeiro saber seu volume em metros cbicos,
ou seja, fazer a converso de 36000 cm3 para m3 .Como
1cm3 = 106 m3 = 0, 000001 m3 , para fazer esta converso basta

deslocar a vrgula 6 casas para a esquerda; obtemos ento


36000 cm3 = 0, 036 m3 . Logo o peso de um bloco pequeno
0, 036 900 = 32, 4 quilogramas.
26

Olimpada Brasileira de Matemtica das Escolas Pblicas

NVEL 1

OBMEP

5a Lista

NVEL 1 -

1) Uma turma da escola fez uma eleio para eleger seu representante. Trs candidatos
concorreram eleio: Joo, Rosa e Marcos. Joo teve

2
7

dos votos, Rosa teve

3
5

dos votos.

Quem ganhou a eleio?


2) Qual o valor de 26 + 26 + 26 + 26 44 ?
A) 0

B) 2

C) 4

D) 42

3) Com seis retngulos idnticos


formamos um retngulo maior com
um dos lados medindo 21 cm, como
na figura. Qual a rea do retngulo
maior?

A) 210 cm2

B) 280 cm2

E) 44

21 cm

C) 430 cm2

D) 504 cm2

E) 588 cm 2

4) Trs anos atrs, a populao de Pirajussara era igual populao que Tucupira tem
hoje. De l para c, a populao de Pirajussara no mudou, mas a populao de Tucupira
cresceu 50%. Hoje a soma das populaes das duas cidades de 9000 habitantes. H trs
anos, qual era a soma destas duas populaes?
A) 3 600

B) 4 500

C) 5 000

D) 7 200

E) 7 500

5) As balanas (1) e (2) da figura abaixo esto em equilbrio. Sabe-se que todos os
tringulos tm o mesmo peso; todos os quadrados tambm tm o mesmo peso, assim
como os crculos. Quantos quadrados devem ser colocados no prato direito da balana (3)
para que ela tambm fique em equilbrio?

(1)
A) 7

(2)
B) 8

C) 9

(3)
D) 10

Olimpada Brasileira de Matemtica das Escolas Pblicas

E) 12

27

OBMEP

6) Em um ano, no mximo quantos meses tm cinco domingos?


A) 3

7)

B) 4

C) 5

D) 6

E) 7

Uma calculadora possui duas teclas especiais:

a tecla A que duplica o nmero que aparece no visor


a tecla B que acrescenta uma unidade ao nmero que aparece no visor.

Por exemplo, se o nmero 45 estiver inicialmente no visor e a tecla B for apertada, o


visor mostrar o nmero 46 . Se, em seguida, apertarmos a tecla A, o visor mostrar o
nmero 92 . Nesse exemplo, apertamos ao todo 2 vezes as teclas A e B: uma vez a tecla
B, e depois uma vez a tecla A, para, a partir de 45 , chegarmos ao 92 .
Suponha que o nmero no visor seja 1 .
a) Indique uma maneira de obter o nmero 10 apertando um total de
4 vezes as teclas A e B.
b) Indique uma maneira de obter o nmero 15 apertando um total de
6 vezes as teclas A e B.
c) Indique uma maneira de obter o nmero 100 apertando um total de
8 vezes as teclas A e B.

28

Olimpada Brasileira de Matemtica das Escolas Pblicas

SOLUES

OBMEP

5a Lista

Solues -5a Lista

1.

Joo recebeu:

2
7

do total de votos; Rosa recebeu:

3
5

do total de votos e Marcos recebeu:

31 4
2 3
do total de votos. O vencedor foi aquele que obteve a maior frao dos
+ = 1=
35 35
7 5

1-

votos. Para comparar essas fraes igualamos seus denominadores:


4

que

35
Marcos

<

2
7
Joo

<

10
35

21
35

. Da segue

3
5
Rosa

, e portanto, Rosa venceu a eleio.

Comentrio: muito interessante notar que a resposta no depende do nmero de alunos

da turma.
2. (A) Soluo 1: Temos: 2 6 + 2 6 + 2 6 + 2 6 - 4 4 = 4 2 6 - 4 4 = 4 (2 2 )3 - 4 4 = 4 4 3 - 4 4 = 4 4 - 4 4 = 0
) = 4 2 2

43

Soluo 2: Temos: 26 + 26 + 26 + 26 4 4 = 4 26 4 4 = 4(26

( )
2

=2

( )

Soluo 3: Temos: 26 + 26 + 26 + 26 4 4 = 4 26 4 4 = 2 2 26 2 2

= 0

= 2 2 =0

3. (E) A partir da figura, vemos que o comprimento a dos


retngulos menores o dobro da sua largura b, isto , a = 2 b . Temos
ento a + b = 2b + b = 3b = 21 , ou seja, b = 7 cm e a = 14cm . Portanto, o
comprimento do retngulo maior
4b = 28 e sua rea
2

21 28 = 588cm .

4. (D) Soluo 1: Seja p a populao de Tucupira h trs anos. Como esta populao cresceu de
50 %, atualmente Tucupira tem p + 50% de p habitantes, ou seja p +

50

p = p + 0, 5 p = 1, 5 p habitantes.
100
Como a populao de Pirajussara no cresceu nesses 3 anos e h 3 anos era igual de Tucupira,

podemos concluir que a populao atual de Pirajussara p. Hoje, a soma das populaes das duas
cidades 9 000 ; logo, p + 1, 5 p = 9000 , donde p =

9000
2, 5

= 3600 . Portanto, a soma das duas

populaes, h 3 anos, era de 3600 2 = 7 200 habitantes.


Soluo 2: De 2003 a 2006 a populao de Tucupira cresceu 50%, logo em 2006 esta populao
corresponde a 150% da populao em 2003. J a populao de Pirajussara no cresceu nesses 3
anos e em 2003 era igual de Tucupira. Temos ento:
Populao de Tucupira em 2006 + Populao de Pirajussara em 2006 = 9 000
corresponde a 150% da
populao de Tucupira em 2003

corresponde a 100% da
populao de Tucupira em 2003

Olimpada Brasileira de Matemtica das Escolas Pblicas

29

OBMEP

Logo, podemos concluir que em 2006 a soma das populaes das duas cidades corresponde a
250 % da populao de Tucupira em 2003 , como essa soma 9000 temos:
250%da populaodeTucupira em 2003
9000
50%da populaodeTucupira em 2003
9000 5 = 1 800
100%da populaodeTucupira em 2003
1800 2 = 3600

Portanto, a soma das duas populaes h 3 anos era de 3600 2 = 7 200 habitantes.

5. (D) Na primeira balana temos 3+1 = 6; na segunda temos 2+4


= 8, o que
equivalente a 1+2 = 4. Logo (3+1 ) + (1+2 ) = 6 + 4, ou seja, 4+3 = 10. Logo
ser necessrio colocar 10 quadrados no prato direito da balana (3) para que ela fique em
equilbrio.

(C) Um ano normal tem 365 dias e o ano bissexto 366 . Da diviso de 365 por 7 , obtemos
365 = 52 7 + 1 e da diviso de 366 por 7 obtemos 366 = 52 7 + 2 . Logo:

6.

ano normal = 52 semanas + 1 dia


ano bissexto = 52 semanas + 2 dias
Portanto, um ano normal ou bissexto tem no mnimo 52 domingos e no mximo 53 domingos ( 1
domingo para cada uma das 52 semanas e talvez outro domingo para os 1 ou 2 dias que
completam o ano).
Cada um dos doze meses do ano tem no mnimo 4 domingos. Logo, cada ano tem no mnimo
12 4 = 48 domingos.
(i)

(ii)

Num ano de 52 domingos, como cada ms tem no mnimo 4 domingos, sobram ainda
52 48 = 4 domingos . Cada um desses ficar num ms diferente, porque nenhum ms tem
6 domingos. Temos ento 4 meses com 5 domingos.
Analogamente, num ano com 53 domingos restaram 5 domingos, que ficaro um em cada
ms diferente. Portanto teremos 5 meses com 5 domingos

7. Com o nmero 1 no visor devemos aplicar sucessivamente as operaes das teclas A e B para
obter o nmero desejado.
A

B A B A
1 2 4 5 10

(a) 1 2 4 5 10 ou

(b) 1 2 3 6 7 14 15

B B A B A
B
1 2 3 6 7 14 15

ou
A

(c) 1 2 3 6 12 24 25 50 100
30

ou

B B A A
A
B
A
A
1 2 3 6 12 24 25 50 100

Olimpada Brasileira de Matemtica das Escolas Pblicas

OBMEP

NVEL 1

6a Lista

1) A metade do nmero 212 + 3 210 :


A) 2 6 + 3 2 5

B) 2 6 + 3 210

C) 211 + 3 2 5

D) 211 7

E) 2 9 7

2) Neste momento so 6 horas e 27 minutos da tarde. Qual era o horrio 2880717 minutos
mais cedo?
A) 6h e 22min

B) 6h e 24min

C) 6h e 27 min

D) 6h e 30min

E) 6h e 32min

3) Os alunos de uma escola participaram de uma excurso, para a qual dois nibus foram
contratados. Quando os nibus chegaram, 57 alunos entraram no primeiro nibus e apenas
31 , no segundo. Quantos alunos devem passar do primeiro para o segundo nibus para
que a mesma quantidade de alunos seja transportada nos dois nibus?
A) 8

B) 13

C) 16

D) 26

E) 31

4) Em qual das alternativas abaixo aparecem dois pedaos de papelo com os quais podese construir um cubo, dobrando pelas linhas tracejadas e colando pelas linhas contnuas?

A)

B)

C)

D)

E)

5) O algarismo das unidades do nmero 1 3 5 97 99 :


A) 1

B) 3

C) 5

D) 7

Olimpada Brasileira de Matemtica das Escolas Pblicas

E) 9

31

OBMEP

6) A figura mostra um retngulo


formado por 18 quadrados iguais
com algumas partes sombreadas.
Qual frao da rea do retngulo
sombreada?

A)

7
18

B)

4
9

C)

D)

5
9

E)

1
2

7) O desenho ao lado mostra o mapa de um pas


(imaginrio) constitudo por cinco estados. Deseja-se colorir
esse mapa com as cores verde, azul e amarela, de modo que
dois estados vizinhos no possuam a mesma cor. De
quantas maneiras diferentes o mapa pode ser pintado?

A) 12

B) 6

C) 10

D) 24

E) 120

8) As nove casas de um tabuleiro 3 3 devem ser pintadas de forma que em cada coluna,
cada linha e cada uma das duas diagonais no hajam duas casas de mesma cor. Qual o
menor nmero de cores necessrias para isso?
A) 3

B) 4

C) 5

D) 6

E) 7

9) Considere um nmero escrito na forma X,Y, onde X e Y so algarismos diferentes de 0.


Determine esse nmero sabendo que ele igual a

3
10

(X + Y) .

10) Em um mesmo lado de uma rua sero construdas 6 casas vizinhas. As casas podem
ser de tijolo ou de madeira, mas como medida de segurana contra incndio, duas casas de
madeira no podem ser vizinhas. De quantas maneiras se pode planejar a construo
dessas casas?

32

Olimpada Brasileira de Matemtica das Escolas Pblicas

SOLUES

OBMEP

6a Lista

1. (E)
2

12

Antes de dividir a expresso por 2, colocamos


+ 32

10

10

= 2 (2 + 3 1) = 2

10

7 . Logo:

12

+ 32
2

10

10

10

2 em

evidncia:

= 2 7 .

2. (D)

Dividindo 2880717 por 60, obtemos 2880717 = 48011 60 + 57 . Isso significa que
2880717 min = 48011 h + 57 min . Dividindo 48011 por 24 , obtemos 48011 = 2000 24 + 11 .
Podemos ento escrever:
2880717 min = 48000 h + 11 h + 57 min
2000dias

Os 2000 dias no interferem no horrio que estamos procurando. Como 6 horas e


27 minutos da tarde so exatamente 18 horas e 27 minutos, a resposta
18 h 27 min - 11h 57 min = 17 h 87 min - 11h 57 min = 6 h 30min.

3. (B)

O nmero total de alunos nos dois nibus 57 + 31 = 88 e

88

= 44 .

Para que

cada nibus tenha o mesmo nmero de alunos, devem ento passar


alunos do primeiro para o segundo nibus.

57 - 44 = 13

4. (E)

Com as peas abaixo.

5. (C) O ltimo algarismo de um mltiplo de 5 0 ou 5 ; os que terminam em 0 so


pares e os que terminam em 5 so mpares. Como 1 3 5 97 99 mltiplo de 5 e,
sendo um produto de nmeros mpares, tambm mpar; segue que o seu
algarismo das unidades 5 .
6. (B)

A parte sombreada consiste de 10 metades de quadrados mais 3 quadrados


10
+ 3 = 5 + 3 = 8 quadrados inteiros. Logo, a frao
2
rea sombreada rea de 8 quadrados
8 4
=
=
sombreada
=
rea total
rea de 18 quadrados 18 9

inteiros, o que equivale a


representa a parte

Olimpada Brasileira de Matemtica das Escolas Pblicas

que

33

OBMEP

7. (B) O estado A pode ser pintado de 3 formas: verde, azul ou amarelo. Para
qualquer estado vizinho, por exemplo, o estado B, temos duas possibilidades e os
demais estados tm suas cores determinadas (1 possibilidade). Logo, podemos
colorir o mapa de 3 2 = 6 formas. Abaixo ilustramos duas destas maneiras de pintar
o mapa; em ambas o estado A tem a mesma cor.

8. (C) Para satisfazer as condies do problema, as cinco casas marcadas com * devem
ter cores diferentes.

*
*

Por isso, precisaremos, no mnimo, de 5 cores distintas. Chamemos de


1, 2, 3, 4 e 5 as cinco cores distintas que usaremos para colorir essas 5 casas, e
vamos determinar como podemos escolher as cores para as 4 casas
restantes para satisfazer as condies pedidas.

3
2

1 2 4
4 3 1
2 4 5

Logo, podemos colorir as 4 casas restantes sem utilizar mais cores. Assim, bastam 5
cores. Outros exemplos de coloraes so:
2

3
1

2 4 3
4 1 2
5 2 4

9. Temos X, Y = X +

Y 10X + Y
=
10
10

4
3

1 3 2
2 4 1
3 2 5

2
4

, o enunciado nos diz que

1 3 5
3 2 4
4 1 2

10X + Y 3
= (X + Y) .
10
10

10X + Y = 3X + 3Y ,

Logo

ou seja, 7X = 2Y . Conclumos que 2Y mltiplo de 7, e como Y um


nmero inteiro entre 1 e 9, s temos a possibilidade Y = 7 , donde X = 2 . Assim, o
nmero 2,7.

34

Olimpada Brasileira de Matemtica das Escolas Pblicas

OBMEP

10. Como as casas so vizinhas, podemos pensar nelas como uma fila de casas com 6
posies.Vamos dividir a contagem em casos, de acordo com o nmero de casas de
madeira que podem ser construdas.
a) Nenhuma casa de madeira: aqui h apenas uma maneira de construir
as casas, ou seja, todas de tijolo.
b) Uma casa de madeira: aqui temos 6 maneiras de construir as casas,
pois a casa de madeira pode ser qualquer uma delas, sendo as outras de
tijolo.
c) Duas casas de madeira: as casas de madeira podem ocupar as
seguintes posies: 1 e 3, 1 e 4, 1 e 5, 1 e 6, 2 e 4, 2 e 5, 2 e 6, 3 e
5, 3 e 6 ou 4 e 6. Temos aqui 10 maneiras.
d) 3 casas de madeira: as casas de madeira podem ocupar as seguintes
posies: 1, 3 e 5; 1, 3 e 6; 1,4 e 6; 2, 4 e 6. Temos aqui 4maneiras
nototal.
e) 4 ou mais casas de madeira: impossvel, pois fcil ver neste caso que
sempre teremos duas casas de madeira juntas.
Dessa forma, h 1 + 6 + 10 + 4 = 21 maneiras de se planejar a construo.

Olimpada Brasileira de Matemtica das Escolas Pblicas

35

NVEL 1

OBMEP

7a Lista

1) Qual o maior dos nmeros?


(A) 1000 + 0, 01

(B) 1000 0, 01

(C) 1000/0, 01

(D) 0, 01/1000

(E) 1000 0, 01

2) Qual o maior nmero de 6 algarismos que se pode encontrar suprimindo-se


9 algarismos do nmero 778157260669103 sem mudar a ordem dos algarismos?
(A) 778152

(B) 781569

3) Se n um nmero natural e

(A) 5

(C) 879103

(D) 986103

(E) 987776

n
1 1
um nmero entre e , ento n igual a:
24
6 4

(B) 6

(C) 7

(D) 8

(E) 9

4) Correndo com velocidade de 10 km / h , Joo completa uma certa distncia em 6 minutos.


A qual velocidade ele pode completar a mesma distncia em 8 minutos?
(A) 7,5 km / h

(B) 7, 75 km / h

(C) 8 km / h

(D) 8, 25 km / h

(E) 8,5 km / h

5) As vizinhas Elza, Sueli, Patrcia, Helosa e Cludia chegam juntas do trabalho e


comeam a subir as escadas do prdio de 5 andares onde moram. Cada uma mora num
andar diferente. Helosa chega a seu andar depois de Elza, mas antes de Cludia. Quando
Sueli chega ao seu andar, Helosa ainda tem 2 andares para subir, e o mesmo ocorre a
Patrcia quando Elza chega ao seu andar. Sueli no mora no 1 andar. Em qual andar mora
cada uma delas?

36

Olimpada Brasileira de Matemtica das Escolas Pblicas

OBMEP

SOLUES

7a Lista

Solues -7 Lista

1.(C)Temos: 1000 + 0, 01 = 1000, 01 ; 1000 0, 01 = 1000

1000 1000
=
= 1000 100 = 100000 ;
1
0, 01
100

1
= 10 ;
100

0, 01
1000
o inverso de
,
1000
0, 01

logo,

de

(C)

temos

que

1000
1
=
= 0, 00001 . Agora, 1000 0, 01 menor do que 1000 (no preciso efetuar o
0, 01 100000
clculo para obter esta concluso). Portanto, o maior nmero

1000
.
0, 01

2. (C) Soluo 1. Para que seja o maior possvel, o nmero deve comear com o maior algarismo.
Para termos 6 algarismos sem mudar a ordem, o maior 8 depois 7 , faltam agora 4 algarismos
para completar o nmero, escolhemos 9103 . Logo, o nmero 879103 (77 8 15726066 9103)
Soluo 2. As opes D e E no servem, pois a ordem foi alterada, j nas opes A, B e C,
no. O maior nmero entre as opes A, B e C C.
3. (A) Como

1 4
1 6
, ento n s pode ser igual a 5 .
=
e =
6 24 4 24

4. (A) Soluo 1 :

6 minutos 1/10 da hora, logo a distncia corrida em 6 minutos 10 :10 = 1 km . Como,


espao = velocidade x tempo , temos 1 km = v 8 min v = 1 km / 8 min (onde v a velocidade).
Logo, Joo corre 1 km em 8 minutos, precisamos determinar essa velocidade em horas.

8 min

corresponde a

1km

4 min

corresponde a

0,5km

60 min

corresponde a

0,5 15km = 7,5km

Logo, a velocidade 7,5 km / h .

Olimpada Brasileira de Matemtica das Escolas Pblicas

37

OBMEP

Soluo

2 : Podemos usar diretamente a seguinte Regra de Trs:


Velocidade
em km / h
10
x

Tempo
em horas
6

60
8

60

Velocidade e tempo so grandezas inversamente proporcionais (aumentando a velocidade diminui


o tempo), logo:

6
x 60 6
60 15
=
= x=
= = 7,5km / h.
10 8 8
8
2
60
5. Vejamos as informaes dadas no enunciado:
Helosa chega a seu andar depois de Elza, mas antes de Cludia.

Helosa mora acima de Elza e abaixo de Cludia.


Quando Sueli chega ao seu andar, Helosa ainda tem 2 andares para subir, e o mesmo ocorre a Patrcia
quando Elza chega ao seu andar.
Helosa mora dois andares acima de Sueli e Patrcia dois andares acima de Elza.

Sueli no mora no 1o andar e Helosa mora 2


andares acima de Sueli, logo temos as seguintes
possibilidades ao lado.

Como Cludia mora acima de Helosa, ento


Helosa no pode morar no ltimo andar que o
5o andar. Logo, Sueli mora no 2o andar, Helosa
no 4o e Cludia s pode morar no 5o.

Finalmente, Patrcia mora dois andares acima de


Elza, logo Elza mora no 1o andar e Patrcia no 4o
andar.

38

1o andar
2o andar
3o andar
4o andar
5o andar
1o andar
2o andar
3o andar
4o andar
5o andar

Sueli
Sueli
Helosa

1o andar
2o andar
3o andar
4o andar
5o andar

Olimpada Brasileira de Matemtica das Escolas Pblicas

Helosa
Sueli
Helosa
Cludia
Elza
Sueli
Patrcia
Helosa
Cludia

OBMEP

NVEL 1

NVEL 1 - 8a Lista

8a Lista

1) 920 + 920 + 920 igual a:


(A) 920

(B) 366

(C) 923

(D) 341

(E) 323

2) Miguel escolheu um nmero de 3 algarismos e outro de 2 algarismos. Qual a soma


desses nmeros se a sua diferena 989 ?
(A) 1000

(B) 1001

(C) 1009

(D) 1010

(E) 2005

3) Qual o menor nmero natural n para o qual 10n 1 mltiplo de 37 ?


(A) 6

(B) 5

(C) 4

(D) 3

(E) 2

4) Num certo pas com 14 milhes de habitantes, 0,15 % da populao contraiu uma certa
gripe. Quantos habitantes no contraram a gripe?
(A) 13 979 000

(B) 1 397 900

(C) 139 790

(D) 13 979

(E) 139 790 000

5) O Cdigo Secreto. O cdigo secreto de um grupo de alunos um nmero de 3


algarismos distintos diferentes de 0 . Descubra o cdigo com as seguintes informaes:
1
4
6
5
8

2
5
1
4
4

3
6
2
7
3

(A) 137

Nenhum algarismo correto


Um s algarismo correto na posio certa
Um s algarismo correto, mas na posio errada
Um s algarismo correto, mas na posio errada
Um s algarismo correto na posio certa
(B) 876

(C) 768

(D) 678

Olimpada Brasileira de Matemtica das Escolas Pblicas

(E) 576

39

SOLUES

OBMEP

8a Lista

Solues - 8 Lista

1. (D) 920 + 920 + 920 = 3 920 = 3 (32 ) 20 = 3 340 = 341.

2. (C) Como a diferena 989 e o menor nmero tem


_ _ _ _ _ = 989
maior que maior que 9
2 algarismos (logo, maior do que 9 ), o nmero de
989+9
3 algarismos tem que ser maior do que 989 +9=998 , logo a
nica opo 999 .
Logo, o nmero de 2 algarismos 10 , e a soma dos dois 999+10=1009 .
3. (D) Observe que 10n 1 um nmero que tem todos os seus algarismos iguais a 9 . Os menores
mltiplos de 37 terminados em 9 so:
37 7 = 259 , 37 17 = 629 , 37 27 = 999 . Como 999 = 103 1 , segue que n = 3 .

4. (A) Os que no contraram a gripe representam 100% 0,15% = 99,85% da populao.


Temos: 99,85% de 14 milhes =

99,85
9985
14 000 000 =
14 000 000 = 9985 1400 = 13979 000
100
10 000

5. (B) O cdigo pode ser formado com os algarismos 1, 2, 3, 4, 5, 6, 7, 8 e 9 .

40

Da 1 a informao temos que 1, 2 e 3 no fazem parte do 1


cdigo. Da 3 a informao, conclumos que 6 faz parte do 4
6
cdigo, e sua posio _6_ ou _ _ 6.
5
8

2
5
1
4
4

3
6
2
7
3

Da 2 a informao segue que 4 e 5 no fazem parte do


cdigo e a posio do 6 no cdigo _ _ 6 . Da ltima
informao tem s que o cdigo da forma 8_ 6 . Com a 4
informao completamos o cdigo: 876 .

3
6
2
7
3

Olimpada Brasileira de Matemtica das Escolas Pblicas

1
4
6
5
8

1
4
4

OBMEP

NVEL 1

9a Lista

1) 2 2 22 22( 42 ) igual a:
(C) 2

(B) 2

(A) 0

2) Escrevendo as fraes em ordem crescente

(A)

2 3 4 4 6 4
< < < < <
5 5 6 5 5 3
(D)

(B)

3) Quantos nmeros maiores que


os algarismos 1, 3 e 5 ?
(A) 10

(B) 12

4 4 4 3 6 2
, , , , , , encontramos:
3 5 6 5 5 5

4 4 2 3 4 6
< < < < <
3 6 5 5 5 5

2 3 4 4 6 4
< < < < <
5 5 5 6 5 3

200

(E) 10

(D) 4

(E)

(C)

2 3 4 4 4 6
< < < < <
5 5 5 6 3 5

2 3 4 4 4 6
< < < < <
5 5 5 3 6 5

podem ser escritos, usando-se apenas

(C) 14

(D) 15

(E) 18

4) Uma maratona de 52 km comeou s 11: 30 horas e o vencedor terminou s 12 : 45 horas


do mesmo dia. Qual foi a velocidade mdia do vencedor em km/hora?
(A) 35

(B) 38

(C) 39,5

(D) 41, 6

(E) 52

5) Cinco alunas escreveram cada uma um nmero num papel, os nmeros s podiam ser 1
ou 2 ou 4 . Qual pode ser o produto dos cinco nmeros escritos?
(A) 100

(B) 120

(C) 256

(D) 768

Olimpada Brasileira de Matemtica das Escolas Pblicas

(E) 2048

41

SOLUES

OBMEP

9a Lista

Solues - 9Lista

1. (E) As ordens de prioridade para resolver uma expresso so:


parnteses colchete chaves e multiplicaes e divises somas e subtraes
2o

1o

3o

2o

1o

Temos:

2 2 2 2 2 2(4 2) = 2 2 2 2 2 2 2 = 2 2 2 2 2 4 =

2
2
4

= 2 2 2 2 (2) = 2 2 {2 (4)} = 2 2 2 + 4 = 2 2 6 = 2 12 = 10
6
12

4 4 4
< < (fraes de mesmo numerador, a menor a que tem o maior
6 5 3
2 3 4 6
< < < (fraes de mesmo denominador, a menor a que tem o menor
denominador) e
5 5 5 5
4 6
numerador). As duas maiores so
e por serem as nicas maiores do que 1 (numerador maior
3 5
4 20
6 18
6 4
2 3 4 6 4
e
= < e logo: < < < < . Falta apenas
do que denominador). Temos =
5 5 5 5 3
3 15
5 15
5 3
2 2 4
4 4
3 9
2 10
3 4
< =
e = , segue que <
encaixar 4/6=2/3. Note que
e
< . Como, =
5 3 6
6 5
5 15 3 15
5 6
2 3 4 4 6 4
Finalmente, temos: < < < < < .
5 5 6 5 5 3
2. (A) Soluo 1: Temos:

Soluo 2: Escrevendo as fraes na forma decimal, obtemos:

4
4
4
3
6
2
= 1,333 ; = 0,8 ; = 0, 666 ; = 0, 6 ; = 1, 2 ; = 0, 4.
3
5
6
5
5
5
Logo: 0, 4 < 0, 6 < 0, 666 < 0,8 < 1, 2 < 1,333
2
5

3
5

4
6

4
5

6
5

4
3

Soluo 3: Escrevendo as fraes com o mesmo denominador comum, temos:


4
3

40 4 24 4 20 3 18 6 36 2 12
; =
; =
; =
; =
; =
.
30 5 30 6 30 5 30 5 30 5 30

denominador,
42

menor

Assim,
a

que

2
5

<

3
5

<

4
6

tem

<

4
5

<

6
5

<

4
3

(fraes de mesmo

menor

Olimpada Brasileira de Matemtica das Escolas Pblicas

numerador).

OBMEP

3. (E) Por serem maiores que 200 , o algarimo das centenas s pode ser 3 ou 5 . Os nmeros so:

sem repetir algarismos: 315 e 351


repetindo algarismos: 311, 313, 331, 335, 353, 333, 355.

Comeando com 3 :

Nesse caso, temos 9 nmeros.

Assim,

Comeando com 5 : basta trocar o 3 com o 5 nos nmeros acima. Logo, teremos 9
nmeros.
temos

18

nmeros

que

satisfazem

as

condies

do

problema.

4. (D) O tempo que o vencedor gastou foi: 12h 45min -11h 30mim = 1h 15 min = 1 1/4h = 5/4h .
Logo, a velocidade mdia em km/hora :

espao percorrido em km 52
4
=
= 52 = 41, 6km / h.
5
tempo gasto em horas
5
4

5. (C) Se todos as alunas escreveram o nmero 1 , o produto seria 1 que no est entre as opes.
Logo, 2 ou 4 so fatores do produto, por isso o produto tem que ser uma potncia de 2 . O maior
produto possvel obtido no caso em que todas as 5 alunas escreveram o nmero 4 , e o produto
seria 4 4 4 4 4 = 45 = 210 = 1024 . Logo, podemos eliminar 2048 . Agora temos:

100 e 120 so divisveis por 5 , logo no so potncias de 2 ;


768 divisvel por 3 (7+6+8=21) , logo no potncia de 2 .

A nica resposta possvel 256 = 28 . Seria, por exemplo o caso em que duas alunas escreveram o
nmero 2 e trs escreveram o nmero 4 : 256 = 2 2 4 4 4 .

Olimpada Brasileira de Matemtica das Escolas Pblicas

43

NVEL 1

OBMEP

10a Lista

NVEL

1 1 1 1
1) O produto 1 1 1 1 :
2 3 4 5
(A)

119
120

(B)

5
7

(C) 2

43
60

(D)

1
5

(E)

1
120

2) A soma de dois nmeros naturais 11 . Qual o maior produto possvel que se pode
obter com esses nmeros?
(B) 22

(A) 30

(D) 24

(C) 66

(E) 28

3) Se m um nmero natural, tal que 3m = 81 , ento m3 igual a:


(A) 813

(B) 381

(D) 24

(C) 64

(E) 48

4) Se a 1 = b + 2 = c 3 = d + 4 , ento qual o maior dentre os nmeros a, b, c e d?


(A) a

(B) b

(C) c

(D) d

(E)todos so iguais.

5) Quatro formigas atravessam uma sala coberta de lajotas retangulares todas iguais. O
trajeto de cada formiga mostrado na figura em negrito. Qual o comprimento do trajeto
percorrido por Biloca?
Trajeto de Pipoca=25dm

(A) 30 dm
(B) 35 dm
(C) 43 dm
(D) 55dm
(E) 48 dm

Trajeto de Tonica=37dm
Trajeto de Cotinha=32dm
Trajeto de Biloca=

6) Clia quer trocar com Guilherme figurinhas de um lbum sobre animais brasileiros.
Celina quer trocar 4 figurinhas de borboleta, 5 de tubaro, 3 de cobra, 6 de periquito e 6
de macaco. Todas as figurinhas de Guilherme so de aranha. Eles sabem que:
(a) 1 figurinha de borboleta vale 3 figurinhas de tubaro
(b) 1 figurinha de cobra vale 3 figurinhas de periquito
(c) 1 figurinha de macaco vale 4 figurinhas de aranha
(d) 1 figurinha de periquito vale 3 figurinhas de aranha
(e) 1 figurinha de tubaro vale 2 figurinhas de periquito

Quantas figurinhas Clia receber se ela trocar todas que quiser?


44

Olimpada Brasileira de Matemtica das Escolas Pblicas

SOLUES

OBMEP

10a Lista

Solues - 10 Lista

1
2

1
3

1
4

1
5

1. (D) 1 1 1 1 =

1 2 3 4 1
= .
2 3 4 5 5

2. (A) Examinemos os produtos dos nmeros naturais cuja soma 11 :

11 = 1 + 10
11 = 2 + 9
11 = 3 + 8
11 = 4 + 7

1 10 = 10
2 9 = 18
3 8 = 24
4 7 = 28

e
e
e
e

11 = 5 + 6 e 5 6 = 30
3. (C) Temos 3m = 81 = 34 ; donde m = 4 . Logo, m3 = 43 = 4 4 4 = 64.
4. (C) Somando 3 a todos os membros obtemos:

a 1 + 3 = b + 2 + 3 = c 3 + 3 = d + 4 + 3 a + 2 = b + 5 = c = d + 7 , o que mostra que c o maior.

5. (B) O trajeto de Biloca : 3 diagonais + 4 larguras + 2 comprimentos.


Pipoca percorre 5 diagonais, logo o comprimento de 1 diagonal 25 5 = 5 dm .
Tonica percorre 5 diagonais mais 4 larguras da lajota,ou seja: 5 diagonais + 4 larguras = 37 , donde
25

4 larguras = 37 25 = 12 dm.
Cotinha percorre 5 comprimentos + 4 larguras =32 1comprimento = 20 5=4 dm.
12

Finalmente, Biloca percorre:

3diagonais + 4 larguras + 2 comprimentos = 15 + 12 + 8 = 35 dm. Observe que o comprimento de 1


35

2 4

12

largura 12 4 = 3 dm .
6. A moeda de troca de Guilherme so figurinhas de aranha, logo vamos calcular o valor-aranha
de cada tipo de figurinha usando as informaes (a), (b), (c), (d) e (e).

4 borboleta = 12 tubaro = 24 periquito = 72 aranha


(a)

(e)

43

122

(d )

243

5 tubaro = 10 periquito = 30 aranha


(e)

3 cobra =

(b)

(d)

52

103

9 periquito = 27 aranha
(d)

33

93

6 periquito = 18 aranha
(d)

63

6 macaco = 24 aranha
(c)

64

Logo, ela receber 72 + 30 + 27 + 18 + 24 = 171 figurinhas de aranha.

Olimpada Brasileira de Matemtica das Escolas Pblicas

45

OBMEP

NVEL 1

11a Lista

10 + 20 + 30 + 40
10
:
+
10
10 + 20 + 30 + 40

1) O valor de
(A) 1

(B) 20

(D) 10,1

(C) 30

(E) 1, 01

2) A figura ao lado formada por um tringulo e um retngulo


usando-se 60 palitos iguais. Para cada lado do tringulo so
necessrios 6 palitos. Se cada palito tem 5 cm de comprimento, qual
a rea do retngulo da figura?

(A)120cm 2

(B) 540cm 2

(C)1350cm 2

(D) 2700cm 2

(E) 5400cm 2

3) O incndio e o bombeiro Uma casa pega fogo. Um bombeiro se mantm no degrau do


meio de uma escada jogando gua sobre o incndio. As chamas diminuem e ele sobe 5
degraus. O vento sopra e o bombeiro desce 7 degraus. Um pouco depois ele sobe 8
degraus e fica l at que o incndio acabe. Em seguida, ele sobe os ltimos 7 degraus e
entra na casa. Quantos degraus tem a escada do bombeiro?
(A) 25

(B) 26

(C) 27

(D) 28

(E) 29

4) A figura mostra a rvore geneolgica de uma famlia. Cada flexa vai do pai em direo
ao seu filho. Quem o irmo do pai do irmo do pai de Evaristo?

Ado
Lus

Andr
Felipe

Jos

Cristvo

Evaristo

Jean
Francisco

(A) Francisco
(B) Jos
(C) Andr
(D)Felipe
(E) Simo

5) Uma colcha quadrada em branco e cinza feita com quadrados e


tringulos retngulos issceles. A parte em cinza representa que
percentagem da colcha?

(A) 36%

46

(B) 40%

(C) 45%

(D) 50%

Olimpada Brasileira de Matemtica das Escolas Pblicas

(E) 60%

OBMEP

SOLUES

11a Lista

1. (D) Soluo:

10 + 20 + 30 + 40
10
100 10
+
=
+
= 10 + 0,1 = 10,1.
10
10 + 20 + 30 + 40 10 100

2. (D) Para o tringulo foram usados 6 3 = 18 palitos, sobrando ento 60 18 = 42 palitos para
formar os 3 lados do retngulo. Da figura, temos que a largura do retngulo formada por

42 6
= 18 palitos. Como cada palito tem 5 cm de
2
comprimento, a rea do retngulo dada por 6 5 18 5 = 30 90 = 2700 cm 2
6 palitos, logo o comprimento formado por

largura comprimento

3. (C) O sobe-desce do bombeiro a partir do degrau do meio at chegar ao ltimo degrau o


seguinte:
sobe

sobe sobe

+5 7 +8 +7 , logo o bombeiro sobe 8 + 5 = 13 degraus acima do degrau do meio, chegando


desce

assim, ao ltimo degrau da escada. Logo, a escada tem 13 degraus acima do degrau do meio, e
portanto, 13 degraus abaixo do degrau do meio. Portanto, a escada tem 13 + 1 + 13 = 27 degraus.
Veja um esquema da movimentao do bombeiro.

Olimpada Brasileira de Matemtica das Escolas Pblicas

47

OBMEP

4. (C) Na figura vemos que o pai de Evaristo Jos. O irmo de Jos Jean. O pai de Jean Lus. O
irmo de Lus Andr.

irmo do pai de Evaristo = irmo de Jos = Jean


Jos

pai do irmo do pai de Evaristo = pai de Jean = Lus


Jos
Jean

irmo do pai do irmo do pai de Evaristo = irmo de Lus=Andr


Jos
Jean
Lus

5. (B) A colcha formada de 5 5 = 25 quadradinhos. Os quadradinhos so todos iguais. J os


tringulos, temos de dois tipos: tipo I que corresponde a meio quadrado e tipo II que corresponde
a 1 4 de um quadradinho. A parte em cinza composta de 8 tringulos do tipo I, 8 tringulos do
tipo II e 4 quadrados, ou seja:

8 tringulos tipo I + 8 tringulos tipo II + 4 quadrados = 10 quadrados .


4 quadrados

2 quadrados

Logo, a frao correspondente a parte cinza

48

10 40
=
= 40% .
25 100

Olimpada Brasileira de Matemtica das Escolas Pblicas

NVEL 1

OBMEP

12a Lista

1) Qual das igualdades est correta?


(i) 3 106 + 5 102 = 8 108
(ii) 23 + 23 = 20
(iii) 5 8 + 7 = 75
(iv) 5 + 5 5 = 2
(A) (i)

(B)(ii)

(C) (iii)

(D)(iv)

(E) nenhuma

2) Se a, b e c so nmeros naturais tais que 3a = 4b = 7c , ento o menor valor de a + b + c :

(A) 84

(B) 36

(C) 61

(E) 42

(D) 56

3) Um nmero um quadrado perfeito se igual a um nmero inteiro elevado ao quadrado.


Por exemplo, so quadrados perfeitos:. 25 = 52 , 49 = 7 2 e 125 = 252 . Qual o menor nmero
que devemos multiplicar 120 para obter um quadrado perfeito?
(A) 10

(B) 15

(C) 20

(D) 30

(E) 35

4) A mquina que registra o nmero de visitantes de um Museu marca 1879564 . Note que
esse nmero tem todos os algarismos distintos. Qual o menor nmero de visitantes que
so necessrios para que a mquina registre um nmero que tambm tenha todos os seus
algarismos distintos?
(A) 35

(B) 36

(C) 38

(D) 47

(E) 52

5) Os nmeros de 0 a 2000 foram ligados por flexas como mostra a figura:

12

13

10

11

14

15

e assim por diante.

Qual a sucesso de flexas que liga o nmero1997 ao nmero 2000?

(A)

(B)

(C)

(D)

(E)

Olimpada Brasileira de Matemtica das Escolas Pblicas

49

SOLUES

OBMEP

12a Lista

1. (E) Nenhuma igualdade est correta.


(i) Errada: 3 106 + 5 102 = 3000000 + 500 = 30000500 8 108

1
1
= 8 + 1 = 20
3
2
8
(iii) Errada: 5 8 + 7 = 40 + 7 = 47 75
(ii) Errada: 23 + 23 = 23 +
multiplicao
antes da
soma

(iv) Errada: 5 + 5 5 = 5 + 1 = 6 2
diviso
antes da
soma

2. (C) Como a, b e c so nmeros naturais, segue que 3a mltiplo de 3 , 4b mltiplo de 4 e 7c


mltiplo de 7 . Como 3, 4 e 7 so primos entre si (pois possuem 1 como divisor comum), o menor
mltiplo comum de 3, 4 e 7 3 4 7 = 84 . Portanto:

3a = 84 a = 28 ; 4b = 84 b = 21 ; 7c = 84 c = 12 . Logo, o menor valor para a + b + c


28 + 21 + 12 = 61 .
3. (D) Fatorando 120 , obtemos: 120 = 23 3 5 . Para obter um quadrado perfeito todos os
expoentes dessa decomposio devem ser pares, logo basta multiplicar 120 por 2 3 5 = 30 . De
fato, temos:

120 30 = 23 3 5 2 3 5 = 24 32 52 = (22 3 5) 2 = 602


4. (C) Observe que os nicos algarismos que no aparecem no nmero 1879564 so 0, 2 e 3 . O
prximo nmero com todos os algarismos distintos ocorrer quando mudar o algarismo das
centenas, e tivermos 18796 _ _ . Logo, o menor nmero ser 1879602 , e faltam ainda

1879602-1879564=38 visitantes.
5. (E) O caminho-padro aquele que se repete, no caso :

Esse caminho formado de 6 flexas e comea sempre nos mltiplos de 6 : 0, 6, 12 , etc. Vamos
averiguar qual a posio de 1997 em relao ao mltiplo de 6 mais prximo. Dividindo 1997 por
6, obtemos 1997 =
6 332 + 5 . Portanto, 1998 o mltiplo de 6 mais prximo de 1997 .
corresponde a 332
caminhos-padro

resto

1998

1999

Logo, 1998 ocupa a 1 posio no caminho-padro, ento,


a situao a seguinte:
1997

50

Olimpada Brasileira de Matemtica das Escolas Pblicas

2000

NVEL 2

OBMEP

1a Lista

NVEL 2 - 1a Lista

1) Em 1998, a populao do Canad era de 30,3 milhes. Qual das opes abaixo
representa a populao do Canad em 1998?
A) 30 300 000

B) 303 000 000

C) 30 300

D) 303 000

E) 30 300 000 000

2) Uma certa mquina capaz de produzir 8 rguas em cada minuto. Quantas rguas esta
mquina consegue produzir em 15 minutos?
A) 104

B) 110

C) 112

D) 128

E) 120

3) Luza, Maria, Antnio e Jlio so irmos. Dois deles tm a mesma altura. Sabe-se que:

Luza maior que Antnio


Maria menor que Luza
Antnio maior do que Jlio
Jlio menor do que Maria.

Quais deles tm a mesma altura?


A) Maria e Jlio
D) Antnio e Jlio

B) Jlio e Luza
E) Antnio e Maria

C) Antnio e Luza

4) O algarismo das unidades do nmero 1 3 5 79 97 113 :


A) 1

Seleo

B) 3

C) 5

D) 7

Jogos

GM

GS

Dinamarca

Senegal

Uruguai

Frana

E) 9

Utilize as informaes abaixo para


resolver as duas prximas
questes:
A tabela ao lado mostra o
desempenho das selees do
grupo A da Copa do Mundo de
2002:

Legenda:
V - vitrias, E - empates, D - derrotas, GM - Gols Marcados, GS - Gols Sofridos, P - Pontos.
Numa partida de futebol, a equipe vencedora ganha 3 pontos, em caso de empate as duas
ganham 1 ponto e a perdedora no ganha nem perde pontos.

Olimpada Brasileira de Matemtica das Escolas Pblicas

51

OBMEP

5) Quantos pontos obteve a seleo do Senegal?


A) 3

B) 4

C) 5

D) 6

E) 7

C) 4

D) 5

E) 6

6) Quantos gols sofreu a seleo do Uruguai?


A) 2

B) 3

7) Na figura abaixo temos dois quadrados. O maior tem lado a + b e o menor lado a.

Qual a rea da regio em cinza?


A) b

B) a + b

C) a2 + 2ab

D) b2

E) 2ab + b2

8) Passa-se um barbante atravs dos seis furos de uma cartolina. A frente da cartolina, com
o barbante, mostrada na figura.

Qual das figuras abaixo no pode ser o verso da cartolina?

52

Olimpada Brasileira de Matemtica das Escolas Pblicas

OBMEP

9) Adriano, Bruno, Csar e Daniel so quatro bons amigos. Daniel no tinha dinheiro, mas
os outros tinham. Adriano deu a Daniel um quinto do seu dinheiro, Bruno deu um quarto
do seu dinheiro e Csar deu um tero do seu dinheiro. Cada um deu a Daniel a mesma
quantia. A quantia que Daniel possui agora representa que frao da quantia total que
seus trs amigos juntos possuam inicialmente?
A)

1
10

B)

1
4

C)

1
3

D)

2
5

E)

1
2

10) O quadrado abaixo chamado quadrado mgico, porque a soma dos nmeros de cada
linha, de cada coluna e de cada diagonal sempre a mesma. Neste caso essa soma 15 .
4

Complete os cinco nmeros que faltam no quadrado abaixo para que ele seja um
quadrado mgico.
-12

-4
0

Olimpada Brasileira de Matemtica das Escolas Pblicas

53

SOLUES

OBMEP

1a Lista

1. (A) Temos que 1 milho = 1000 000 . Logo, 30, 3 milhes = 30, 3 1000 000 = 30 300 000
2. (E) Se a mquina produz 8 rguas em 1 minuto, em 8 minutos ela produzir 8 15 = 120 rguas.
3. (E) Soluo 1: Usaremos a notao a < b que significa que a menor do que b, ou
equivalentemente, b maior do que a. Assim, a<b<c significa que a menor do que b e b menor
do que c.
Para simplificar, vamos denotar a altura de cada um dos irmos pela letra inicial de seu nome.
Do enunciado temos:
(i) L maior que A ou, equivalentemente, A menor que L (A< L)
(ii) M menor que L (M<L)
(iii) A maior que J ou, equivalentemente, J menor que A (J<A)
(iv) J menor que M (J<M)
De (i) e (iii) segue que: J < A < L. Portanto, os irmos de mesma altura no esto entre Jlio,
Antnio e Luza.
De (ii) e (iv) segue que: J < M < L. Portanto, os irmos de mesma altura no esto entre Jlio, Maria
e Luza.
Logo, a nica opo que Antnio e Maria tenham a mesma altura.
Soluo 2: Pelo enunciado, as opes A, C e D no ocorrem. Como Luza maior do que
Antnio e Antnio maior do que Jlio, temos que Luza maior do que Jlia. Logo, a opo
correta (E).
4. (C) Como um dos fatores 5 , o produto um mltiplo de 5. Os mltiplos de 5 so aqueles cujo
algarismo das unidades 0 ou 5 . Alm disso, todos os fatores so nmeros mpares, ento o
produto um nmero mpar. Logo, o seu algarismo das unidades tem que ser 5 .
5. (C) Segundo as condies da copa, uma vitria vale 3 pontos, um empate vale 1 ponto e quem
sofre uma derrota no pontua. Como Senegal teve uma vitria e dois empates, ele somou:
1 3 + 2 1 = 5 pontos.
6. (D) Observe que num campeonato, o nmero total de gols marcados o mesmo que o total de
gols sofridos. Denotando por x o nmero de gols que sofreu a seleo do Uruguai temos:
5 + 5 + 4 + 0 = 2 + 4 + x + 3 14 = 9 + x

Da obtemos x = 5 .
7. (E) Soluo 1 Usaremos que ( a + b )2 = a2 + 2ab + b2 . Lembre que a rea de um quadrado de lado
l l 2 . Note que a rea da regio cinza a diferena entre as reas do maior e do menor
2
quadrado. O lado do maior a + b , portanto sua rea ( a + b ) . J o lado do menor a , logo sua
rea a 2 . Conclumos que a rea da regio cinza : ( a + b ) - a 2 = a 2 + 2ab + b 2 - a 2 = 2ab + b2 .
2

54

Olimpada Brasileira de Matemtica das Escolas Pblicas

OBMEP

Soluo 2 - Lembre que a rea de um retngulo o produto da largura pelo comprimento.


Podemos dividir a regio cinza em dois retngulos, um da
largura b e comprimento a , e o outro de largura b e
comprimento a + b , como mostra a figura. A rea em cinza a
soma das reas desses dois retngulos, ou seja:
a b + b ( a + b ) = ab + ab + b 2 = 2ab + b 2 .

Portanto, a rea solicitada 2ab + b 2 .

Soluo 3 A regio cinza formada por 2 retngulos de dimenses a b e um quadrado de lado


b. Logo a sua rea : 2 ab + b 2 .
8. (E) Observando a frente da cartolina, verificamos que o barbante entra e sai pelos furos da
primeira linha. Na opo (e) o verso mostra estes dois furos como consecutivos ao percorrer o
barbante, o que no possvel.
9. (B) Suponha que Daniel tenha recebido x reais de cada um de seus amigos. Ento, Adriano
tinha, inicialmente, 5 x reais, Bruno tinha 4 x reais e Csar tinha 3x reais. Segue que o total de
dinheiro dos trs no incio era de 5 x + 4 x + 3 x = 12 x reais. Como cada um de seus trs amigos lhe deu
x reais, Daniel tem agora 3x reais, o que representa a quarta parte de 12 x . Logo, ele possui agora
1
da quantia que seus trs amigos juntos possuam inicialmente.
4

10. Como a soma dos nmeros de uma diagonal 4 + 0 + ( 4 ) = 0 , este deve ser o valor da soma dos
nmeros de cada linha coluna e diagonal.
Assim, obtemos de imediato os nmeros que faltam nas casas em cinza no primeiro tabuleiro: 16, 8
e 12, pois ( 12 ) + 16 + ( 4 ) = 0 (na primeira linha), ( 12 ) + 8 + 4 = 0 (na primeira coluna) e

( 12 ) + 0 + (12 ) = 0 (na diagonal).


-12

-4
0

-12

16

-4

12

-12

16

-4

-8

-16

12

Agora, o nmero que falta na segunda linha do segundo tabuleiro ( 8 ) , porque 8 + 0 + ( 8 ) = 0 .


Para a terceira linha, obtemos ( 16 ) , pois 4 + ( 16 ) + 12 = 0 .

Olimpada Brasileira de Matemtica das Escolas Pblicas

55

NVEL 2

OBMEP

2a Lista

NVEL 2- 2 Lista

1) A, B, C, D, E, F, G e H so os fios de apoio que uma aranha usa para construir sua teia,
conforme mostra a figura. A aranha continua seu trabalho. Sobre qual fio de apoio estar o
nmero 118 ?

A) B

B) D

C) E

D) G

E) H

2) Na figura temos B = 50o , AD e CD so as bissetrizes dos


ngulos A e C respectivamente.
Qual a medida do ngulo AD C ?
A) 90

B) 100

C) 115

D) 122.5

E) 125

3) O grfico mostra o nmero de pontos que cada


jogador da seleo de basquete da escola marcou no
ltimo jogo.
O nmero total de pontos marcados pela equipe foi:
A) 54

56

B) 8

C) 12

D) 58

Olimpada Brasileira de Matemtica das Escolas Pblicas

E) 46

OBMEP

4) Geni cliente de um companhia telefnica que oferece o seguinte plano:

tarifa mensal fixa de R$ 18,00


gratuidade em 10 horas de ligaes por ms
R$ 0,03 por cada minuto que exceder s 10 horas.

Em janeiro, Geni usou seu telefone por 15 horas e 17 minutos, e em fevereiro por 9 horas
e 55 minutos. Qual a despesa de Geni com telefone nesses dois meses?
A) R$ 45, 51

B) R$ 131,10

C) R$ 455,10

D) R$ 13,11

E)R$ 4, 55

5) Veja as promoes de dois supermercados:


Supermercado A
6 latas de 3 litros do
sorvete QUENTE
R$ 24,00

Supermercado B
Sorvete QUENTE lata de 3
litros
4 latas - s R$ 14,00

Joana quer comprar 12 latas de sorvete para a festa de seu aniversrio. Em qual
supermercado ela deve comprar?
A) No A, pois economizar R$ 7,00 em relao ao B.
B) No A, pois economizar R$ 6,00 em relao ao B.
C) No B, pois economizar R$ 8,00 em relao ao A.
D) No B, pois economizar R$ 6,00 em relao ao A.
E) Tanto faz, porque o preo o mesmo nos dois supermercados.

6) Paulo quer comprar um sorvete com 4 bolas em uma sorveteria que dispe de trs
sabores: aa, baunilha e caj. De quantos modos diferentes ele pode fazer a compra?
A) 6

B) 9

C) 12

D) 15

E) 18

7) A prefeitura de uma certa cidade fez uma campanha que permite trocar 4 garrafas de
1litro vazias por uma garrafa de 1 litro cheia de leite. At quantos litros de leite pode obter
uma pessoa que possua 43 dessas garrafas vazias fazendo trocas sucessivas?
A) 11

B) 12

C) 13

D) 14

Olimpada Brasileira de Matemtica das Escolas Pblicas

E) 15

57

OBMEP

8) Pedro montou um quadrado com quatro das cinco peas abaixo. Qual a pea que ele
no usou?

9) Uma linha de nibus possui 12 paradas numa rua em linha reta. A distncia entre duas
paradas consecutivas sempre a mesma. Sabe-se que a distncia entre a terceira e a sexta
paradas 3300 metros. Qual a distncia entre a primeira e a ltima parada?
A) 8, 4 km

B) 12,1 km

C) 9,9 km

D) 13, 2 km

E) 9, 075 km

10) Sete equipes, divididas em dois grupos, participaram do torneio de futebol do meu
bairro.
O grupo 1 foi formado pelas equipes Avaqui, Botgua e Corinense.
O grupo 2 foi formado pelas equipes Dinossauros, Esquisitos, Flurinthians e Guaran.
Na primeira rodada do torneio, cada equipe enfrentou cada uma das equipes do seu
grupo exatamente uma vez.
Na segunda rodada do torneio, cada equipe enfrentou cada uma das equipes do outro
grupo exatamente uma vez.
(a) Quantas partidas foram disputadas na primeira rodada no grupo 1?
(b) Quantas partidas foram disputadas na primeira rodada no grupo 2?
(c) Quantas partidas foram disputadas na segunda rodada?

58

Olimpada Brasileira de Matemtica das Escolas Pblicas

SOLUES

OBMEP

2a Lista

1. (D) Observe que so 8 fios de apoio que a aranha utiliza, numerados a partir do fio A iniciando
com 0. Logo:

sobre o fio A aparecem os mltiplos

de

sobre o fio B aparecem os ( mltiplos

de

8) + 1

sobre o fio C aparecem os ( mltiplos

de

8) + 2

sobre o fio D aparecem os ( mltiplos

de

8) + 3

sobre o fio E aparecem os ( mltiplos

de

8) + 4

sobre o fio F aparecem os ( mltiplos

de

8) + 5

sobre o fio G aparecem os ( mltiplos

de

8) + 6

sobre o fio H aparecem os ( mltiplos

de

8) + 7

Na diviso de 118 por 8 encontramos resto 6 , o que significa que 118 = ( mltiplo

de

8) + 6 .

Portanto, 118 est sobre o fio G.

2. (C) Nesta questo, usaremos o seguinte importante teorema da Geometria Plana:


Teorema: A soma dos ngulos internos de um tringulo 180o .
Do teorema acima temos A + B + C = 180o , e como B = 50 o , segue que
A + 50o + C = 180 o A + C = 130 o.

Aplicando agora o teorema ao tringulo ADC, obtemos:


^

^
A C
+ + A D C = 180 o
2 2

A C A+ C 130
=
= 65 , conclumos da igualdade acima que
Como + =
2 2
2
2
= 180 o - 65o = 115o.
ADC

3. (A) Analisando o grfico, verificamos que os jogadores marcaram as seguintes quantidades de


pontos: Daniel 7 , Ramon 8 , Ian 2 , Bernardo 11 , Tiago 6 , Pedro 12 , Ed 1 e Andr 7 .
Total: 54 pontos.

Olimpada Brasileira de Matemtica das Escolas Pblicas

59

OBMEP

4. (A) Vejamos a despesa em janeiro. Como 10 horas so gratuitas e Geni usou seu telefone por
15 horas e 17 minutos, ela deve pagar o custo de apenas 5 horas e 17 minutos mais a tarifa fixa
mensal de 18 reais. Como o preo dado em minutos, vamos reduzir a minutos o tempo a pagar.
Sabemos que 1 hora = 60 minutos, portanto 5 horas = 5 60 = 300 minutos . Logo,
5 h 17 m = 300 + 17 = 317 m . Portanto, a conta telefnica de Geni em janeiro foi:
18 + 317 0,03 = 18 + 9, 51 = 27, 51 reais.

Em fevereiro, Geni usou seu telefone menos do que 10 horas, portanto neste ms ela s precisa
pagar a tarifa fixa mensal de 18 reais. Logo, a despesa de Geni com telefone nesses dois meses foi:
27, 51 + 18 = 45, 51 reais.

5. (D) Se comprar no supermercado A, Joana gastar 2 R$ 24,00 = R$ 48,00.


Se comprar no supermercado B, ela gastar 3 R$14,00 = R$ 42,00.

6. (D) Vamos denotar cada sabor de sorvete pela sua letra inicial:
a aa, b baunilha, c caj
Para enumerar todas as possibilidades de compra do aaaa
sorvete com quatro bolas, devemos considerar os seguintes bbbb
cccc
casos:
4 bolas do mesmo sabor (1a coluna ao lado);
3 bolas do mesmo sabor e 1 de sabor diferente
(2a coluna ao lado);
2 bolas de um mesmo sabor e 2 de outro sabor
(3a coluna ao lado);
2 bolas de um mesmo sabor e as outras 2 dos
outros dois sabores (4a coluna ao lado).

aaab aabb aabc


aaac aacc bbac
bbcc ccab
bbba
bbbc
ccca
cccb

Obtemos assim 15 modos de fazer a compra do sorvete.

7. (D) Ele separa 40 garrafas vazias e as troca por 10 garrafas de 1 litro cheias de leite. Esvaziadas as
10 garrafas, ele pode junt-las com as 3 vazias que restaram e troc-las por 3 garrafas cheias,
sobrando ainda 1 garrafa vazia. Esvaziando as 3 cheias e juntando com a garrafa vazia, ele ainda
pode obter em troca mais uma garrafa cheia. Ao todo, ele pode obter, por sucessivas trocas,
10 + 3 + 1 = 14 garrafas cheias de leite, todas elas a partir das 43 vazias que ele possua.

60

Olimpada Brasileira de Matemtica das Escolas Pblicas

OBMEP

8. (B)
Soluo 1 - Contando o total de quadrados nas peas.
Para que seja possvel montar o quadrado, o nmero total de quadradinhos deve ser um quadrado
perfeito (Um nmero um quadrado perfeito se ele igual ao quadrado de um nmero inteiro. Por
exemplo, 1, 9 e 16 so quadrados perfeitos pois 1 = 12 , 9 = 32 , 16 = 4 2 .).
Contando o total de quadradinhos apresentados nas cinco opes de resposta, obtemos:
4 + 5 + 6 + 7 + 8 = 30 .
Portanto, devemos eliminar uma pea de modo que o total de quadradinhos resultante seja um
quadrado perfeito. A nica possibilidade a (b). De fato, eliminando (b), a soma fica sendo 25 que
um quadrado perfeito, pois 25 = 52 .
Soluo 2 - Tentando montar o quadrado com 4 das
cinco peas.
Neste caso, conseguimos montar um quadrado com
as peas a, c, d e e, como na figura:

9. (B)

Como a distncia entre a 3a e a 6a paradas 3300 m , ento a distncia entre duas paradas
consecutivas 3300 3 = 1100 m.
Portanto, a distncia entre a primeira e a ltima paradas 1100 m 11 = 12100 m . Como as opes da
resposta so dadas em quilmetro, devemos reduzir 12100 m a quilmetro. Como 1 km = 1000 m ,
temos 12100 m = 12,1 km .

10.

(a) Foram disputadas 3 partidas que so: A B , B C , C A .


(b) Foram disputadas 6 partidas que so: D E , D F , D G , E F , E G , F G
(c) Na segunda rodada, cada equipe do grupo 1 joga 4 partidas; uma com cada equipe do
grupo 2. Como o grupo 1 tem 3 equipes, o total de partidas ser 3 4 = 12 .

Olimpada Brasileira de Matemtica das Escolas Pblicas

61

NVEL 2

OBMEP

3a Lista

1) Os quadrados brancos sem nmeros da figura ao lado


devem ser preenchidos com nmeros de modo que cada
nmero, a partir da segunda linha, seja igual soma dos dois
nmeros vizinhos da linha imediatamente superior. Por
exemplo, o nmero da primeira casa da segunda linha 11 ,
porque 11 = 5 + 6 . Qual o nmero que vai aparecer no quadrado indicado com x ?
A) 4

B) 6

C) 9

D) 15

E) 10

2) Uma bola de futebol feita com 32 peas de couro. Dessas peas 12 so pentgonos
regulares idnticos e as outras 20 so hexgonos, tambm regulares e idnticos. Os lados
dos pentgonos so iguais aos lados dos hexgonos. Para unir dois lados de duas dessas
peas necessria uma costura. Quantas so as costuras necessrias para fazer uma bola?
A) 60

B) 64

C) 90

D) 120

E) 180

3) A figura ao lado mostra uma grade formada por quadrados de


lado 1cm . Qual a razo entre a rea sombreada e a rea no
sombreada?
A) 1 4
B) 1 5
C) 1 6
D) 2 5
E) 2 7
4)Em um quente dia de vero, 64 crianas comeram, cada uma, um sorvete pela manh e
outro tarde. Os sorvetes eram de 4 sabores: abacaxi, banana, chocolate e doce de leite. A
tabela abaixo mostra quantas crianas consumiram um destes sabores pela manh e outro
tarde; por exemplo, o nmero 7 na tabela indica que 7 crianas tomaram sorvete de
banana pela manh e de chocolate tarde.
TARDE

M
A
N
H

Abacaxi
Banana
Chocolate
Doce de
Leite

Abacaxi Banana Chocolate Doce de


leite
1
8
0
3
6
2
7
5
3
3
0
5
2
9
9
1

Quantas crianas tomaram sorvetes de sabores diferentes neste dia?


A) 58

62

B) 59

C) 60

D) 61

Olimpada Brasileira de Matemtica das Escolas Pblicas

E) 62

OBMEP

5) Camila e Lara tm, cada uma, um tabuleiro 4 4 , inicialmente ambos em branco. Com
estes tabuleiros elas fazem uma brincadeira do seguinte modo:
Camila, escondida de Lara, pinta algumas casas de seu tabuleiro, de preto;
Ainda em seu tabuleiro, Camila escreve em cada casa o nmero de casas vizinhas
que esto pintadas de preto (duas casas distintas so vizinhas se possuem um lado
ou um vrtice em comum);
Camila copia os nmeros escritos em seu tabuleiro no tabuleiro de Lara;
Lara deve adivinhar, a partir dos nmeros escritos em seu tabuleiro, quantas so as
casas pretas do tabuleiro de Camila.
Por exemplo, se Camila pintou seu tabuleiro assim

ento ela vai colocar os nmeros no tabuleiro de Lara do seguinte modo:


1
2
1
1

1
3
3
2

3
2
1
0

1
2
1
0

1
0
2
1

2
2
3
0

1
1
3
2

1
2
1
1

Se o tabuleiro de Lara tem os nmeros

quantas foram as casas que Camila pintou?


A) 3

B) 4

C) 5

D) 6

E) 7

6) Larissa e Jorge esto jogando com cartes numerados de 1 a 6 que devem ser colocados
nas casas do tabuleiro abaixo de modo a formar um nmero de seis algarismos.
XX XX XX XX XX XX
XX XX XX XX XX XX
Jorge coloca o primeiro carto e a seguir as jogadas so alternadas entre os dois. O objetivo
de Larissa obter o maior nmero possvel e o de Jorge obter o menor nmero possvel.
Larissa tem os cartes com os algarismos 1, 3 e 5 e Jorge tem os cartes com os algarismos
2, 4 e 6 . Se os dois jogadores forem espertos, qual o nmero que aparecer ao final do jogo?
A) 254361

B) 253416

C) 251634

D) 256134

Olimpada Brasileira de Matemtica das Escolas Pblicas

E) 251346
63

SOLUES

OBMEP

3a Lista

1. (E) Preenchendo o tabuleiro de acordo com as regras do problema:

segue que 60 = ( x + 17) + (2 x + 13) = 3 x + 30 , donde x = 10 .

2. (C) Se somarmos os nmeros de lados de todos os polgonos ( 20 hexgonos e 12 pentgonos)


que compem a superfcie da bola, obteremos um valor que duas vezes o nmero de costuras,
pois cada costura lado comum de exatamente dois polgonos. Assim, temos que 2 (nmero de
costuras) = 12 5 + 20 6 = 180 , donde o nmero de costuras 90 .
3. (A) A grade um quadrado de lado igual a 5 cm , logo sua rea igual a 25cm 2 . A parte
sombreada da grade formada por quatro tringulos, sendo que dois deles tm base 1cm e altura
2cm e os outros dois tm base 1cm e altura 3cm . Logo a rea sombreada igual a
2
5
20

1 2

2
1
4

+ 2

1 3
2

= 5 cm e a rea no sombreada igual a 25 5 = 20cm . Assim, a razo pedida

4. (C) Vamos primeiro analisar a informao contida na diagonal da tabela indicada pelos nmeros
dentro dos quadradinhos.

Abacaxi
M
A
N
H

TARDE
Chocolate
0

Doce de leite
3

Abacaxi

Banana
8

Banana

Chocolate

Doce de
Leite

Esses nmeros indicam quantos foram as crianas que tomaram sorvetes com o mesmo sabor pela
manh e pela tarde: 1 tomou sorvetes de abacaxi, 2 de banana, 0 de chocolate e 1 de doce de leite.
Todos os outros estudantes comeram sorvetes de sabores diferentes pela manh e tarde; estes so
em nmero de 64 (1 + 2 + 0 + 1) = 60 .

64

Olimpada Brasileira de Matemtica das Escolas Pblicas

OBMEP

0
0

X X
X
X X X
X
X

5. (B) Notamos primeiro que se uma casa tem o algarismo


0 , ento nenhuma das casas vizinhas pode estar pintada.
Logo as casas marcadas com um X na figura ao lado no
foram pintadas:

Consideremos agora a casa do canto superior direito, na


qual aparece o nmero 1 . Ela tem 3 vizinhas, e j sabemos
que duas delas no foram pintadas; logo, a vizinha que
sobra (a casa imediatamente abaixo) foi pintada.

X X
X
X X X
X
X

Podemos aplicar o mesmo argumento s casas do canto inferior


esquerdo e do canto inferior direito.

Olhamos agora para o 2 na ltima linha. Como esta casa j tem


duas vizinhas pintadas, todas suas outras vizinhas no foram
pintadas:

X X
X
X X X
X
X

X X
X
X X X
X
X X

X X X X
X
X
X X X
X
X X

Argumento idntico se aplica casa da segunda linha e terceira


coluna, pois nela aparece um 1 e j temos uma de suas vizinhas
pintadas. Logo, as suas outras 3 vizinhas no foram pintadas

X X X X
X
X
X X X
X
X X

Finalmente, usamos o 3 que aparece na casa da terceira linha e


terceira coluna; esta casa j tem 2 vizinhas pintadas, logo deve
haver mais uma de suas vizinhas pintada. Esta vizinha s pode ser
a casa em branco na figura acima, e podemos completar a tabela:

Conclumos que o nmero de casas pintadas 4 .


6. (B) A formao de um nmero de 6 algarismos ilustrada a seguir.
centena
de milhar

dezena de
milhar

unidade
de
milhar

centena dezena unidade

Para se obter o menor nmero possvel, os menores algarismos devem estar o mais esquerda
possvel (na casa do milhar); e para se obter o maior nmero possvel os maiores algarismos
devem tambm estar o mais esquerda possvel (na casa do milhar).

Olimpada Brasileira de Matemtica das Escolas Pblicas

65

OBMEP

Jorge joga primeiro: Para obter o menor nmero possvel, ele coloca o menor algarismo que ele
possui, que o 2 , na casa das centenas de milhar. Se ele no fizesse isso, Larissa colocaria seu
5 nesta casa na prxima jogada, obtendo assim um nmero maior.
2

dezena de
milhar

unidade
de
milhar

centena dezena unidade

Agora a vez de Larissa: Para obter o maior nmero possvel, ela coloca o maior algarismo que ela
possui, que o 5 , na casa das dezenas de milhar, pois a casa das centenas de milhar j est
ocupada.
2

unidade
de
milhar

centena dezena unidade

Jorge tem agora os algarismos 4 e 6 , e Larissa 1 e 3 . Logo, os algarismos de Larissa so menores


dos que os de Jorge, o que determina a estratgia de Jorge : ele deve tentar colocar seus algarismos
o mais direita possvel, com o 6 direita do 4 . Por sua vez, Larissa deve tentar colocar seus
algarismos o mais esquerda possvel, com o 3 esquerda do 1 . Jorge ento coloca o 6 na casa das
unidades.
Jorge joga: Ele coloca o algarismo 6 na casadas unidades.
2

unidade
de
milhar

centena dezena

Larissa joga: Ela coloca seu 1 na casa das dezenas.


2

unidade
de
milhar

centena

Agora Jorge tem apenas o algarismo 4 e Larissa o 3 . Ele ento coloca o 4 na casa das centenas, e
Larissa coloca o 3 na casa das unidades de milhar, acabando assim o jogo.
2

Logo, o nmero final obtido se os dois jogadores forem espertos 253416 .

66

Olimpada Brasileira de Matemtica das Escolas Pblicas

NVEL 2

OBMEP

4a Lista

1) Uma professora tem 237 balas para dar a seus 31 alunos. Qual o nmero mnimo de
balas a mais que ela precisa conseguir para que todos os alunos recebam a mesma
quantidade de balas, sem sobrar nenhuma?
A) 11

B) 20

C) 21

D) 31

E) 41

2) Um arteso comea a trabalhar s 8 h e produz 6 braceletes a cada vinte minutos; j seu


auxiliar comea a trabalhar uma hora depois e produz 8 braceletes do mesmo tipo a cada
meia hora. O arteso pra de trabalhar s 12 h, mas avisa ao seu auxiliar que este dever
continuar trabalhando at produzir o mesmo que ele. A que horas o auxiliar ir parar?
A) 12 h

B) 12 h 30 min

C) 13h

D) 13h 30 min

E) 14h 30 min

3) Se girarmos o pentgono regular, ao lado, de um ngulo de 252,


em torno do seu centro, no sentido horrio, qual figura ser obtida?
Observao: Sentido horrio o sentido em que giram os ponteiros
do relgio; no caso ele est indicado pela seta no desenho.

A)

B)

E)

D)

C)

4) O permetro de um retngulo 100 cm e a diagonal mede x cm. Qual a rea do


retngulo em funo de x?
A) 625-x 2

B) 625-

x2
2

C) 1250-

x2
2

D) 250-

x2
2

E) 2500-

x2
2

5) Se x + y = 8 e xy = 15 , qual o valor de x 2 + 6 xy + y 2 ?
A) 64

B) 109

C) 120

D) 124

Olimpada Brasileira de Matemtica das Escolas Pblicas

E) 154

67

OBMEP

6) Na figura esto indicadas em graus as


medidas de alguns ngulos em funo de
x. Quanto vale x?
A) 6
D) 20

B) 12
E) 24

5x

3x

C) 18

2x
6x
4x

7) Qual dos seguintes desenhos no pode ser feito sem tirar o lpis do papel e passando
apenas uma vez por cada linha?

A)

C)

B)

D)

E)

8)Cortamos um canto de um cubo, como mostrado na seguinte figura.

Qual das representaes abaixo corresponde ao que restou do cubo?

68

Olimpada Brasileira de Matemtica das Escolas Pblicas

OBMEP

9)Voc j viu um truque numrico? Aqui vo os passos de um truque numrico:


(I) Escolha um nmero qualquer.
(II) Multiplique-o por 6 .
(III) Do resultado subtraia 21 .
(IV) Divida agora este novo resultado por 3 .
(V) Deste ltimo resultado subtraia o dobro do nmero que voc escolheu.

(a) Experimente fazer esses cinco passos trs vezes, iniciando cada vez com um nmero
diferente. Qual foi o resultado de seu experimento?
(b) A seguir, usando a letra x para representar o nmero que voc pensou, mostre por que
os resultados do item (a) no so apenas uma coincidncia, mas sim um fato matemtico.

10)Na figura abaixo vemos uma mesa de sinuca quadriculada e parte da trajetria de uma
bola, tacada a partir de um canto da mesa, de modo que, sempre, ao bater em uma das
bordas da mesa, segue seu movimento formando ngulos de 45 com a borda.

(a) Em qual das quatro caapas a bola


cair?
(b) Quantas vezes a bola bater nas
bordas da mesa antes de cair na caapa?
(c) A bola atravessar a diagonal de
quantos desse quadrados durante sua
trajetria?

Olimpada Brasileira de Matemtica das Escolas Pblicas

69

SOLUES

OBMEP

4a Lista

3a Lista
1. (A) O algoritmo de diviso de Euclides nos d 237 = 7 31 + 20 ; logo 237 no divisvel
por 31 . Isso quer dizer que a professora realmente vai ter que comprar mais balas para que
todos os alunos recebam o mesmo nmero de balas. De acordo com o enunciado, devemos
ento adicionar expresso 7 31 + 20 o menor inteiro positivo x tal que 7 31 + 20 + x seja
mltiplo de 31 . Como x = 31 20 = 11 , basta que a professora compre 11 balas.
produz
6
braceletes
a
cada
20
minutos.
Como
1 hora = 60 minutos = 3 20 minutos , o arteso produz 6 3 = 18 braceletes em 1 hora. Como ele
trabalhou 12 horas 8 horas = 4 horas , o nmero de braceletes feitos pelo arteso 18 4 = 72 .
O auxiliar produz 8 braceletes a cada meia-hora, portanto em 1 hora ele produz
2.(D)

arteso

16 braceletes. Para produzir 72 braceletes ele precisar de

72
= 4,5 horas = 4 horas e
16

30 minutos. Como ele inicia seu trabalho s 9 horas, ele terminar seu trabalho s
9 horas + 4 horas + 30 minutos = 13 horas e 30 minutos .

3. (B) O pentgono tem 5 lados, logo seu ngulo central

360
= 72
5

. Como 252 = 72 + 180 ,

podemos pensar na rotao de 252 como uma rotao de 72 seguida de outra de 180 ,
conforme ilustrado na figura abaixo, onde O o centro do polgono.

72o

180o

rotao de 72o

rotao de 180o

4. (C) Soluo 1: Como o permetro do retngulo 100 , seu semipermetro 50 . Como o semi-permetro de um retngulo a soma
do comprimento com a largura, conclumos que esses so da forma
a e 50 a. A rea de um retngulo o produto do comprimento pela
largura. No nosso caso, esta rea ( 50 a ) . a = 50a a 2 .Pelo teorema de

a
.

50 a

Pitgoras, temos x 2 = ( 50 a )2 + a 2 , ou seja, x 2 = 2500 100a + 2a 2 = 2500 2 ( 50a a 2 ) . Logo


1
( 2500 x 2 ) e obtemos a expresso da rea do retngulo em funo de x.
2
Soluo 2: rea do retngulo de medidas a e b A= ab. Como a + b = 50 , temos

50a a 2 =

( a + b)

= a 2 + b 2 + 2ab = 502 . Pelo Teorema de Pitgoras, x 2 = a 2 + b 2 , assim,

x 2 + 2 A = 2500
70

Olimpada Brasileira de Matemtica das Escolas Pblicas

OBMEP

5. (D) Usando a identidade ( x + y )2 = x 2 + 2 xy + y 2 , temos


x 2 + 6 xy + y 2 = ( x 2 + 2 xy + y 2 ) + 4 xy = ( x + y ) + 4 xy = 82 + 4 15 = 124
2

6. (C) Completamos a figura marcando os


ngulos e , lembrando que ngulos opostos
pelo vrtice so iguais. Como a soma dos
ngulos internos de um tringulo 180o,
podemos escrever as trs igualdades abaixo,
uma para cada um dos tringulos da figura:
+ 7 x = 180 o
+ 8 x = 180 o

+ + 5 x = 180 o

Logo,
( + 7 x ) + ( + 8 x ) ( + + 5 x ) = 180 o + 180 o 180 o = 180 o

e como

( + 7 x ) + ( + 8 x ) ( + + 5 x ) = + 7 x + + 8 x 5 x = 10 x

segue que 10 x = 180 o , donde x = 18o

7. (E)

Observe nas ilustraes (a), (b), (c) e (d) que iniciando o desenho no ponto P e seguindo as
setas de acordo com a ordem numrica, possvel completar cada desenho sem tirar o
lpis do papel.
J o desenho da opo (e) no pode ser construdo sem tirar o lpis do papel. De fato,
excetuando-se o vrtice de incio do traado e o vrtice de finalizao, os demais vrtices
do desenho devem possuir obrigatoriamente um nmero par de linhas chegando at eles,
pois a cada vez que se chega a um desses vrtices por uma linha, deixa-se esse mesmo
vrtice por outra linha. No caso da letra (e), os quatro vrtices externos possuem trs
linhas chegando a cada um deles, logo impossvel fazer tal traado.
Olimpada Brasileira de Matemtica das Escolas Pblicas

71

OBMEP

8. (E) Cortando um canto do cubo, eliminamos um de seus vrtices. Como cada vrtice se
liga a trs arestas do cubo, uma representao do cubo cortado deve mostrar trs cortes ao
redor de um mesmo vrtice.

9. (a) Vamos fazer o experimento com os nmeros 0, 5 e 4.

O resultado final sempre 7 .


(b) razovel conjeturar ento que para qualquer nmero escolhido o resultado final
deste procedimento ser sempre 7. Seja x o nmero inicial. Temos ento as operaes:

Portanto, o resultado ser 7 qualquer que seja o nmero inicialmente escolhido.

10. A bola muda a direo de sua trajetria cada vez


que bate na borda da mesa. Como a trajetria faz
sempre um ngulo de 45 com a borda, a bola seguir
sempre as diagonais dos quadrados que ela cruza.
a) Traando esta trajetria, conclumos que a bola cair
na caapa D ;
b) A bola bater 5 vezes na borda da mesa;
c)Contando
quantos
so
os
quadradinhos
atravessados, descobrimos que ela atravessar
23 quadradinhos.

72

Olimpada Brasileira de Matemtica das Escolas Pblicas

NVEL 2

OBMEP

5a Lista

1) Se m e n so inteiros maiores do que zero com m < n, definimos m n como a soma dos
inteiros entre m e n, incluindo m e n. Por exemplo,
5 8 = 5 + 6 + 7 + 8 = 26.
Ento o valor de

22 26
4 6

A) 4

:
B) 6

C) 8

E) 12

D) 10

2) O preo de uma corrida de txi R$ 2,50 fixos (bandeirada), mais R$ 0,10 por cada
100 metros rodados. Tenho apenas R$ 10, 00 no bolso. Logo, tenho dinheiro para uma
corrida de at:
A) 2,5 km

B) 5, 0 km

C) 7,5 km

D) 10, 0 km

E) 12,5 km

3) Quantos nmeros entre 1 e 601 so mltiplos de 3 ou mltiplos de 4 ?


A) 100

B) 150

C) 250

D) 300

E) 430

4) Se x, y e z so nmeros inteiros positivos tais que xyz = 240 , xy + z = 46 e x + yz = 64 , qual o


valor de x + y + z ?
A) 19

B) 20

C) 21

D) 24

E) 36

5) Na reta abaixo esto representados os cinco nmeros a, b, m, n, p e q

Ento os nmeros que melhor representam a + b, a b e ab so, respectivamente,


(A) m, p e q

(B) m, q e p

(C) n, q e p

(D) n, p e q

(E) q, m e p

6) Numa corrida de carros, um piloto percorreu trs trechos: um de 240 km , um de 300 km e


um de 400 km . O piloto sabe que as velocidades mdias nesses trechos foram 40 km / h ,
75 km / h e 80 km / h , mas no se lembra qual dessas velocidades corresponde a cada um
desses trechos.. Podemos garantir que o tempo total em horas gasto pelo piloto para
percorrer os trs trechos foi:
A)
B)
C)
D)
E)

menor ou igual a 13 horas


maior ou igual a 13 horas e menor ou igual a 16 horas
maior ou igual a 16 horas e menor ou igual a 17 horas
maior ou igual a 15 horas e menor ou igual a 18 horas
maior ou igual a 18 horas
Olimpada Brasileira de Matemtica das Escolas Pblicas

73

OBMEP

7) Do quadrado ABCD foram cortados os tringulos issceles


sombreados, como na figura, restando o retngulo PQRS. A rea
total do que foi cortada de 200m 2 . Qual o comprimento de
PR?

(A) 200m

(B) 20 m

(C) 800m

(D) 25 m

(E) 88 m

= 20o e BC = BD = BE .
8) Na figura o tringulo ABC issceles, BAC

.
Determine a medida do ngulo BDE

9) So dadas 4 moedas aparentemente iguais, das quais 3 so verdadeiras e por isso tm o


mesmo peso; uma falsa e por isso tem peso diferente. No se sabe se a moeda falsa
mais leve ou mais pesada que as demais. Mostre que possvel determinar a moeda
diferente empregando somente duas pesagens em uma balana de pratos. Observao:
Neste tipo de balana podemos comparar os pesos colocados nos dois pratos, ou seja, a
balana pode ficar equilibrada ou pender para o lado mais pesado.

74

Olimpada Brasileira de Matemtica das Escolas Pblicas

SOLUES

OBMEP

5a Lista

1. (C) De acordo com a definio de , temos

22 26
4 6

22 + 23 + 24 + 25 + 26
4+5+6

120
15

=8.

2. (C) Como a bandeirada fixa, temos 10,00 - 2, 50 = 7, 50 reais a serem gastos apenas com os
metros rodados. Cada trecho de 100 metros rodado custa R$ 0,10, ento com R$ 7,50 posso
fazer uma corrida de

7, 50
0,10

750
10

= 75 trechos de 100 metros cada um, ou seja 75 100 = 7500

metros. Como 1 quilmetro tem 1000 metros, segue que com R$ 10, 00 posso pagar uma
corrida de at 7500 metros =

7500
quilmetros = 7, 5 quilmetros .
1000

3. (D) Para achar o nmero de mltiplos de 3 compreendidos de 1 a 601 , basta usar o


algoritmo da diviso e escrever 601 = 200 3 + 1 . Isso mostra que 3 1 , 3 2 , , 3 200 so os

mltiplos de 3 de 1 a 601, ou seja, temos 200 destes mltiplos. Do mesmo modo vemos que
existem 150 mltiplos de 4 de 1 a 601.
Nesse total
200
+
150
= 350 , alguns nmeros aparecem contados duas vezes, pois
mltiplos mltiplos
de 3
de 4

so mltiplos de 3 e de 4 ao mesmo tempo. Por exemplo: 12,36 e 60 foram includos nos


200 mltiplos de 3 e tambm nos 150 mltiplos de 4 . Lembre que os mltiplos de 3 e de
4 so tambm mltiplos de 12 . O mesmo argumento usado acima mostra que temos
50 mltiplos de 12 de 1 a 601 . Logo o nmero de mltiplos de 3 ou 4 de 1 a 601
350 50 = 300 .
4. (B) Soluo 1: De xyz = 240 segue que xy =
240
+ z = 46 ,
z

240
;
z

substituindo em xy + z = 46 obtemos

ou seja, z 2 46 z + 240 = 0 . As razes desta equao so nmeros cuja soma 46 e

cujo produto 240 , ou seja, as razes so 6 e 40 . Logo, z = 6 ou z = 40 (I) . Do mesmo modo, a


substituio de yz =
y=

240
xz

240
x

em x + yz = 64 nos leva a x = 4 ou x = 60 (II) . De xyz = 240 , segue que

. Como y um nmero inteiro, ento xz um divisor de 240 . Segue de (I) e (II) que

as possibilidades para xz so:


4 6 = 24 , 4 40 = 160 , 60 6 = 360 , 60 40 = 2400
x z
x z
x z
x z

Vemos que s podemos ter x = 4 e z = 6 , pois em qualquer outro caso o produto xz no


um divisor de 240 . Segue que y =

240 240
=
= 10 , donde x + y + z = 4 + 10 + 6 = 20 .
xz
46

Olimpada Brasileira de Matemtica das Escolas Pblicas

75

OBMEP

Soluo 2: Somando xy + z = 46 e x + yz = 64 , obtemos:


xy + z + x + yz = (x + z) + y(x + z) = (x + z)(y + 1) = 110 (I)

e vemos que y + 1 um divisor de 110 . Logo, temos as possibilidades y + 1 = 1, 2, 5,10,11, 22, 55 e


110 , ou seja, y = 0,1, 4,9,10, 21, 54 e 109 . Por outro lado, y um divisor de 240 porque
xyz = 240 , alm disso y positivo, que nos deixa com as possibilidades e y = 1, 4 e 10 .

( x + z)(y + 1) = 110 x + z = 55
Se y = 1 ento
; o que no possvel. Logo y 1 .
xy + z = 46 x + z = 46
( x + z)(y + 1) = 110 x + z = 22
Se y = 4 ento
, e podemos verificar (por exemplo, com uma
xyz = 240 xz = 60
lista de divisores de 60 ou ento resolvendo a equao w 2 - 22w + 60 = 0 ) que no h valores
inteiros positivos de x e z que verifiquem estas duas condies. Logo y 4 .

( x + z)(y + 1) = 110 x + z = 10
Se y = 10 ento
, donde conclumos que x = 4 e z = 6 . Finalmente,
xyz = 240 xz = 24
temos x + y + z = 4 + 10 + 6 = 20 .
5. (B) Notamos que a e b so nmeros maiores que 1/2 e menores que 1. Logo a + b um
nmero maior que 1 e menor que 2 ; assim, a + b s pode ser representado por m. Como
a < b , segue que a - b negativo e portanto s pode ser representado por q . Quanto ao
produto ab , notamos primeiro que como a e b so positivos, seu produto positivo. Por
outro lado, temos b < 1 e a > 0 , donde ab < a .Logo o nico nmero que pode representar ab
p.
6. (D) O menor tempo de percurso obtido quando se percorre o maior trecho com a
maior velocidade e o menor trecho com a menor velocidade. J o maior tempo obtido
quando se percorre o maior trecho com a menor velocidade e o menor trecho com a maior
velocidade. Assim, o tempo total gasto pelo piloto nos trs trechos no mnimo
240
40

300
75

400
80

= 15

horas e no mximo

240
80

300
75

400
40

= 17

horas.

7. (B) Primeiro notamos que os tringulos APS e CQR so


congruentes, pois tm os trs ngulos iguais (um deles reto)
e tambm um de seus lados (PS = QR). Do mesmo modo os
tringulos BPQ e DRS tambm so congruentes. Sejam AP = x
1 2
x e a do tringulo
2
1
1
2 x2 + y 2 = x2 + y 2 , e
2
2

e BP = y ; ento a rea do tringulo APS


BPQ

1 2
y .
2

Logo a rea cortada foi de

conclumos que x 2 + y 2 = 200 .


Agora notamos que PR a hipotenusa do tringulo retngulo PSR; para calcular PR
basta saber o comprimento dos catetos PS e RS. Mas PS a hipotenusa do tringulo
retngulo APS; do teorema de Pitgoras segue que PS 2 = AS 2 + AP 2 = x 2 + x 2 = 2 x 2 ; do mesmo
modo obtemos RS 2 = 2y 2 . Logo PR 2 = PS 2 + RS 2 = 2 x 2 + 2 y 2 = 2( x 2 + y 2 ) = 2 200 = 400 , ou seja,
PR = 400 = 20 m .

76

Olimpada Brasileira de Matemtica das Escolas Pblicas

OBMEP

8. Lembramos que um tringulo issceles caracterizado tanto por ter dois lados iguais
como por ter dois ngulos iguais.
A soma dos ngulos internos de um tringulo 180. Como A = 20o e B = C , segue que
180o = 20o + B + C = 20o + 2 B . Logo, B = C = 80o .
Pelo enunciado temos BC = BD , donde o tringulo BDC tambm issceles de base CD.
= C e portanto CDB
= 80o . Considerando a soma dos ngulos internos do
Ento, CDB
+ CDB
+ C = 180o . Substituindo os valores acima, temos
tringulo BCD, temos CBD
+ 80o + 80o = 180o .Conclumos que CBD
= 20o , e segue ento que DBE
= B 20o = 80o 20 = 60o .
CBD
= BED

O tringulo BDE tambm issceles, pois BD = BE . Logo BDE


. Como
o
o
o
+ BED
+ 60 = 180 , conclumos que BDE
= 60 .
BDE

9. Sejam A, B, C e D as quatro moedas. Comparamos as moedas A e B na balana,


colocando uma em cada prato. Dois casos podem ocorrer: a balana fica em equilbrio ou a
balana no fica em equilbrio. Vamos analisar separadamente cada caso.
1o Caso: A balana fica equilibrada. Podemos concluir que A e B tm o mesmo peso, e logo
so verdadeiras. Vamos ento comparar A com C. Para isso, mantemos A na balana e
colocamos C no lugar de B. Se houver equilbrio novamente, porque A e C tm o mesmo
peso e logo so verdadeiras. Portanto, A, B e C so verdadeiras, e a nica opo que D
seja falsa. Se no houver equilbrio, C ser a moeda falsa.
2o Caso: A balana no fica equilibrada. Logo uma das duas moedas, A ou B ser falsa.
Substitumos A por C na balana. Se houver equilbrio, A ser a moeda falsa. Se no
houver equilbrio, a moeda falsa ser B.
Observe que nos dois casos s utilizamos a balana duas vezes.

Olimpada Brasileira de Matemtica das Escolas Pblicas

77

NVEL 2

OBMEP

6a Lista

1)Determine o valor de 123456123456 1000001 = .


2)Toda vez que Joozinho vai ao cinema, ele toma 2 refrigerantes. Ele gastou toda a sua
mesada de R$ 50, 00 indo ao cinema 6 vezes e tomando um total de 20 refrigerantes,
incluindo os que ele tomou quando foi ao cinema. Se Joozinho tivesse tomado s um
refrigerante cada vez que foi ao cinema, com essa economia ele poderia ter ido ao cinema
mais uma vez, tomando um refrigerante tambm nessa ocasio. A respeito do preo do
ingresso no cinema e preo do refrigerante, podemos afirmar que:
A) o preo do ingresso o triplo do preo do refrigerante.
B) o preo do ingresso o qudruplo do preo do refrigerante.
C) o preo do ingresso o quntuplo do preo do refrigerante.
D) o ingresso R$ 6, 00 mais caro que o refrigerante.
E) o ingresso R$ 5, 00 mais caro que o refrigerante
3)O quociente de 50 50 por 25 25 igual a :
A) 2525

B) 1025

C) 10025

D) 2 25

E) 2 2525

4)Voc possui apenas palitos com 6 cm e 7 cm de comprimento. O nmero mnimo de palitos


que voc precisa para cobrir com esses palitos um segmento de reta com 2 metros :
A) 29

B) 30

C) 31

D) 32

E) 33

C) 47

D) 50

E) 53

5)A maior raiz da equao ( x - 37 ) 2 - 169 = 0 :


A) 39

B) 43

6)Uma certa mquina tem um visor, onde aparece um nmero inteiro x, e duas teclas A e
B. Quando se aperta a tecla A o nmero do visor substitudo por 2x + 1. Quando se
aperta a tecla B o nmero do visor substitudo por 3x 1.
Se no visor est o nmero 5, o maior nmero de dois algarismos que se pode obter
apertando alguma seqncia das teclas A e B :
A) 85

78

B) 87

C) 92

D) 95

Olimpada Brasileira de Matemtica das Escolas Pblicas

E) 96

OBMEP

7)Em um quadrado mgico, a soma dos 3 nmeros de cada linha, coluna ou diagonal
sempre a mesma. A seguir temos um quadrado mgico, parcialmente preenchido.

14

26

13

Qual o valor de x?
A) 20

B) 22

C) 23

D) 25

E) 27

8)Um retngulo ABCD est dividido em quatro retngulos menores. As reas de trs deles
esto indicadas na figura abaixo. Qual a rea do retngulo ABCD?
A

D
16
27

12
B

A) 80

B) 84

C) 86

D) 88

E) 91

9) Quatro peas iguais, em forma de tringulo retngulo, foram dispostas de dois modos
diferentes, como mostram as figuras abaixo.
H
J

I
D
A
L

N
G

C
B
K

P
F

Os quadrados ABCD e EFGH tm lados respectivamente iguais a 3 cm e 9 cm. Determine a


medida do lado do quadrado IJKL.

Olimpada Brasileira de Matemtica das Escolas Pblicas

79

SOLUES

OBMEP

6a Lista

1. claro que com nmeros to grandes, a questo no pretende que se efetue a diviso. Para
resolv-la vamos usar alguns truques aritmticos:
123456123456 = 123456000000 + 123456 = 123456 1000000 + 123456 =
= 123456 (1000000 + 1) = 123456 1000001

Logo, 123456123456 1000001 = 123456.

2. (C) A economia teria sido equivalente a 6 refrigerantes, permitindo a Joozinho mais um


cinema e mais um refrigerante. Logo o ingresso do cinema 5 vezes o valor do refrigerante.

3.

(C) Soluo 1:

Soluo 2:

50

50

25

25

50

50

25

25

2 50

(2 5 )

2 25

(5 )

(2 25)
25

25

50

50

5
5

50

100

=2

50

25

25

25

50

50

2 25

= (2 5 )

= 100

25

50

=2

25

25

25

25

= 100

25

4. (A) A quantidade utilizada de palitos mnima quando o nmero de palitos de 7 cm utilizado


o maior possvel. Dividindo 200 por 7 obtemos 200 = 28 7 + 4 . Como 200 = 26 7 + 18 = 26 7 + 3 6 ,
usando 26 palitos de 7 cm e 3 palitos de 6cm obtemos o que queramos. Logo, o nmero mnimo de
palitos 26+3=29.
Comentrio: Observe que a soluo equivale a encontrar nmeros inteiros x e y tais que.
e y seja o maior possvel, onde y = nmero de palitos de 7cm e x = nmero
200 = 7 y + 6 x
mltiplo de 7

mltiplo de 6

de palitos de 6cm.

5. (D)
Soluo 1: Usando a fatorao a 2 b 2 = (a b)(a + b) :
( x 37) 2 132 = 0 ( x 37 13)( x 37 + 13) = 0 ( x 50)( x 24) = 0 .

Logo, as razes so 24 e 50.


Soluo 2: Extraindo a raiz quadrada em ambos os lados:
(x 37)2 =132 x 37 =13 ou x 37 = 13. Assim, x = 50 ou x = 24.

80

Olimpada Brasileira de Matemtica das Escolas Pblicas

OBMEP

6. (D) O diagrama a seguir mostra os resultados que podem ser obtidos a partir do nmero 5
apertando-se cada uma das duas teclas.

95

A
A

23
B

11
A

68
65

A
B

32
95

A
A

29
B

14
B
41

7.

A
83

(E) Seja y um dos nmeros do quadrado mgico, conforme a figura. De


acordo
com
a
regra
de
quadrado
mgico
temos
26 + 14 + y = y + x + 13 . Segue que 26 + 14 = x + 13 , donde x = 27.
soma dos nmeros
da diagonal quecontm y

y
1

14

26

soma dos nmeros


da coluna que contm y

x
13

8. (E) Soluo 1: Sejam x e y lados dos retngulos de reas 12 e 27 respectivamente como


indicado na figura. Logo, os outros lados desses retngulos so 12/x (retngulo de rea 12),
16/x (retngulo de rea 16) e 27/y (retngulo de rea 27), como indicado na figura. Assim, o
comprimento do retngulo ABCD x+y e sua largura

Temos:

12 27
y 27
y 9
=
=
=
x
y
x 12
x 4

16

16
x

16

12
x

12

12

28

. Claramente

27

12

27

27
y

A rea de um retngulo o produto do comprimento pela largura. Logo, a rea de ABCD


= ( x + y)

28
28 y
y
9
= 28 +
= 28 + 28 . Logo, = 28 + 28 = 28 + 7 9 = 91 .
x
x
x
4

Olimpada Brasileira de Matemtica das Escolas Pblicas

81

OBMEP

Soluo 2: xz = 12
yz = 27
xw = 16
xyzw = 27 16
yw =

27 16
xz

27 16
12

= 91

9. Soluo 1: Sejam x e y o maior e o menor catetos, respectivamente, do tringulo retngulo.


Como o lado do quadrado ABCD mede 3cm , temos x y = 3 . Por outro lado, como o lado de
EFGH mede 9cm , temos x + y = 9 . Resolvendo o sistema, encontramos x = 6 e y = 3 . Logo, o
lado

do

quadrado

IJKL,

que

hipotenusa

do

tringulo

retngulo,

mede

6 + 3 = 45 = 3 5 cm .
2

Soluo 2: Os quadrados IJKL e MNOP tm como lados as hipotenusas dos tringulos


retngulos dados, logo tm a mesma rea. Superpondo-se as duas figuras e fazendo esses
dois quadrados coincidirem, encontramos 8 tringulos e conclumos que
2
2
8 rea do tringulo = rea de EFGH rea de ABCD = 9 3 = 72 cm2. Logo a rea de cada
tringulo 9 cm2. Da figura temos
rea de IJKL = 4 rea do tringulo + rea de ABCD = 4 9 + 9 = 45.

Logo, o lado do quadrado IJKL

82

45 = 3 5 cm.

Olimpada Brasileira de Matemtica das Escolas Pblicas

NVEL 2

OBMEP

7a Lista

NV

1) Qual o maior dos nmeros?


(A) 2 0 2006

(B) 2 0 + 6

(C) 2 + 0 2006

(D) 2 (0 + 6)

(E) 2006 0 + 0 6

2) O smbolo
representa uma operao especial com nmeros. Veja alguns exemplos
2 4 = 10 , 3 8 = 27 , 4 27 = 112 , 5 1 = 10 . Quanto vale 4 (8 7) ?
(A) 19

(B) 39

(C) 120

(D) 240

(E) 260

3) Se dois lados de um tringulo medem 5 cm e 7 cm , ento o terceiro lado no pode


medir:
(A) 11 cm

4) Se

(B) 10 cm

(C) 6 cm

(D) 3 cm

(E) 1 cm

(C) 23

(D) 25

(E) 29

3 2 1
= , ento igual a:
24 8 3 6
(A) 20

(B) 21

5) O que representam as expresses (a), (b) e (c) na figura ao lado?


a

(a) a + 1, 5a
(b) 4 a + 3
(c) a(1, 5 + a)

1,5

6)A figura composta de tringulos retngulos issceles todos iguais. Qual a rea em
cm 2 da parte sombreada?

30 cm

(A) 20

(B) 25

(C) 35

(D) 45

Olimpada Brasileira de Matemtica das Escolas Pblicas

(E) 50

83

SOLUES

OBMEP

7a Lista

Solu

1. (D) Lembre que se num produto um dos fatores zero, ento o produto tambm zero. Temos:

2 0 2006 = 0; 2 0 + 6 = 0 + 6 = 6; 2 + 0 2006 = 2 + 0 = 2;
2006 0 + 0 6 = 0 + 0 = 0 . Logo, o maior 2 (0 + 6).
2.
(E)
que

Temos

que

descobrir

qual

regra

dessa

operao.

Note

27 = 112 = 4 27 + 4 , 5 1 = 10 = 5 1 + 5
a b = a b + a . Assim, temos:
Podemos concluir que a regra que define a operao
4 (8 7) = 4 (8 7 + 8) = 4 64 = 4 64 + 4 = 260.
2

4 = 10 = 2 4 + 2 , 3 8 = 27 = 3 8 + 3 , 4

2 (0 + 6) = 2 6 = 12 e

3. (E) Lembre que num tringulo a soma de dois lados quaisquer tem que ser maior que o terceiro
lado. Como 1 + 5 no maior do que 7 , o terceiro lado no pode ser 1.
4. (E)

3 2 3
=
+
24 8 3 24 8

3
2
+
=
24 8
3

25 25
2

=
.
=
24 24
3
24

Logo,

25 1 4
= =
, donde 25 = 4 = 29.
24
6 24
5. Note que a figura um retngulo formado por quadrado de lado a e um retngulo de lados 1,5
e a.
(a) a 2 = rea do quadrado e 1,5 a = rea do retngulo. Logo a 2 + 1, 5a representa a somas dessas
duas reas, e portanto a rea total da figura.
(b) 4 a + 3 = 3a + 1, 5 + a + 1, 5 o permetro da figura.
(c) A figura um retngulo de largura a e comprimento a + 1,5 , logo a(1, 5 + a) a rea total da
figura.

6. (D) Soluo 1: O comprimento da hipotenusa de cada um dos 5 tringulos


30 5 = 6cm . O quadrado formado por 4 desses tringulos tem lado igual a 6 cm ,
logo sua rea 36 cm 2 . Logo, cada um dos tringulos tem 36:4 = 9 cm 2 de rea.
6 cm

Portanto, a rea da parte sombreada 9 5 = 45 cm 2

Soluo 2: Pelo Teorema de Pitgoras, temos 36 = 2 x 2 x 2 = 18 . A

rea da parte sombreada 5

84

= 5

18
2

= 45cm

Olimpada Brasileira de Matemtica das Escolas Pblicas

NVEL 2

OBMEP

8a Lista

1) Se eu der duas barras de chocolate para Tio, ele me empresta sua bicicleta por 3 horas.
Se eu lhe der 12 bombons, ele me empresta a bicicleta por 2 horas. Amanh, eu lhe darei
uma barra de chocolate e 3 bombons. Por quantas horas ele me emprestar a bicicleta?
(A) 1 2

(B) 1

(C) 2

(D) 3

(C) 2

(D) 4

(E) 4

2) 2 2 2 2 2 2(4 2) igual a:
(A) 0

(B) 2

3) Na figura, as retas FD e EC so paralelas?

(E) 10

A
62o

E
F
42o
48o

C
28o

B
D

4) Se x > 5 , ento qual dos nmeros abaixo o menor?


(A) 5 / x

(B) 5 / ( x + 1)

(C) 5 / ( x 1)

(E) ( x + 1) / 5

(D) x / 5

5)O quadrado STUV formado de um quadrado limitado por 4 retngulos iguais. O


permetro de cada retngulo 40 cm. Qual a rea, em cm2, do quadrado STUV?
(A) 400
(B) 200
(C) 160
(D) 100
(E) 80

6) a) Calcule as diferenas: 1

1
;
2

b) Deduza de (a) o valor da soma:

c) Calcule a soma:

1 1
1 1

;
2 3
3 4

1 1

;
4 5

1 1

5 6

1
1
1 1 1
+ +
+
+
2 6 12 20 30

1 1 1 1
1
1
+ + + + + +
2 6 12 20 30 42

1
999 000

Olimpada Brasileira de Matemtica das Escolas Pblicas

85

SOLUES

OBMEP

7a Lista

8a Lista

1. (C)

2 barras corresponde
1 barra corresponde
3 horas
1,5 horas
logo

corresponde
corresponde
3 bombons 0, 5 horas
12 bombons 2 horas
Logo, Tio me emprestar a bicicleta por 1,5 + 0,5 = 2 horas

2. (E) As ordens de prioridade para resolver uma expresso so:

parnteses colchete chaves e multiplicaes e divises somas e subtraes


2o

1o

3o

2o

1o


2 2 2 2 2 2(4 2) = 2 2 2 2 2 2 2 = 2 2 2 2 2 4 =

4
2
Temos:

= 2 2 2 2 (2) = 2 2 {2 (4)} = 2 2 2 + 4 = 2 2 6 = 2 12 = 10

6
12
4

3.

No tringulo BCE, temos BC=180-(42+48)=90. No tringulo AFD, temos:


=180-(28+62)=90. Logo, as retas FD e EC so perpendiculares a AB, portanto, so paralelas.

4. (B) Soluo 1: Como a questo tem uma nica resposta, ela vlida para qualquer valor de x.
x =10
.
Podemos
ento
escolher
um
valor
para
x,
por
exemplo

x + 1 11
= . Vemos que x / 5 e ( x + 1) / 5
5
5
so maiores que 1 , logo esto excludos porque as outras trs opes so menores que 1 . Como
5 /10,5 /11 e 5 / 9 tm o mesmo numerador, o menor o que tiver maior denominador, que 5 /11 ,
5
.
ou seja,
x +1
Temos:

5 5
=
,
x 10

5
5
=
,
x + 1 11

Soluo 2 : Se x>5, ento

5
5
=
,
x 1 9

x 10
=
,
5 5

so menores do 1 e

x +1

so maiores do que 1. Logo, as

5
5
x x +1 x 1
5 5
5
opes D e E esto excludas. Como ,
, tm o mesmo numerador, o menor o que tem maior
e
x x +1 x 1
5

denominador, que

x +1

5. (A)Denotemos por C e L, o comprimento e a largura respectivamente de cada um dos quatro


retngulos. O permetro de cada retngulo 2(C + L ) . Ento, 2 (C + L ) = 40 C + L = 20 .
Observe na figura que o lado do quadrado STUV C+L, e portanto sua rea
A=(C + L) 2 =202 =400 cm 2 .

86

Olimpada Brasileira de Matemtica das Escolas Pblicas

OBMEP

6. Soluo:

1 1
1 1 1
=
;
=
;
2 2
2 3 6
1 1
1
1
1
+ + +
+
= 1
b)
2 6 12 20 30
a) 1

1
2

1 1

2 3

1 1

3 4

1 1

4 5

1 1 1
1 1
1
1 1 1
=
;
==
;
=
3 4 12
4 5
20
5 6 30
1 1 1 1 1 1 1 1 1
1 5
+ + + + = 1 =
2 2 3 3 4 4 5 5 6
6 6

1 1

5 6

c) Note que os denominadores so produtos de nmeros consecutivos , iniciando no 1:

1
1
1
1
1
1
+
+
+
+
= 1
2
6
12 20 30
6

12

2 3

3 4

45

56

Mas, geralmente, usando a decomposio de cada parcela como no item (a) podemos provar que:

1
1
1
1
1
1
+
+
+
+
+
+
1 2 2 3 3 4 4 5 5 6 6 7

1
1
= 1
n (n + 1)
n +1

Logo:

1
1
1
1
1
1
1
999
1
+
+
+
+
+
+L +
= 1
=
= 0, 999
6
12 20 30 42
999 000
1000 1000
2

12

2 3

3 4

45

56

6 7

9991000

Olimpada Brasileira de Matemtica das Escolas Pblicas

87

NVEL 2

OBMEP

9a Lista

1) Calcule os ngulos que no esto indicados e o permetro da figura sabendo que


BD=BC e
.

2) Quais os valores de x que satisfazem

(A) x <

9
4

(B) x > 2

3)Quantas solues inteiras


2000 < n(n + 1) < 2005 ?
(A) 1

4)

1
< 4?
x2

(C) 2 < x <

(B) 2

9
4

(D) x < 2

positivas

satisfazem

(D) 4

(C) 3

Na
figura,
O

o
centro
AB= 5cm . Qual o dimetro desse crculo?

(E) x < 2 ou x >

do

crculo

dupla

9
4

inequao

(E) 5

5) Se a, b e c so nmeros naturais tais que 3a = 4b = 7c , ento o menor valor de a + b + c :


(A) 84

(B) 36

(C) 61

6) Na figura temos TU=SV. Quanto vale o ngulo


(A) 30
(D) 65

88

(B) 50
(E) 70

(D) 56

(C) 55

Olimpada Brasileira de Matemtica das Escolas Pblicas

(E) 42

OBMEP

7) O caf, o bolo e o gato Dez minutos antes de colocar o bolo no forno, eu coloquei
meu gato do lado de fora da casa. O bolo deve cozinhar por 35 minutos, ento eu
coloquei o despertador para tocar 35 minutos, aps colocar o bolo no forno.
Imediatamente fiz um caf para mim, o que me tomou 6 minutos. Trs minutos antes
de acabar de beber o caf o gato entrou em casa. Isso foi 5 minutos antes do
despertador tocar. O telefone tocou no meio do tempo entre eu acabar de fazer o caf e
o gato entrar em casa. Falei ao telefone por 5 minutos e desliguei. Eram 3h59min da
tarde.
(a) A que horas coloquei o gato fora de casa?
(b) Quantos minutos depois de colocar o gato fora de casa, o despertador tocou?
(c) Quanto tempo o gato estava fora de casa at o momento em que o telefone tocou?

Olimpada Brasileira de Matemtica das Escolas Pblicas

89

OBMEP

SOLUES

9a Lista

Solu

1. O tringulo ABE issceles porque tem dois ngulos iguais.


Logo os lados AE e AB so iguais, portanto AB=120m. O
tringulo BCD tambm issceles porque tem dois lados iguais,
BC=BD, logo
. Como,
ento os trs
ngulos do tringulo BCD so iguais, logo cada um vale
180 o 3 = 60 o . Assim, ele equiltero e temos
BD=BC=CD=115m.
Assim, o permetro da figura : 120 2 + 115 2 + 226 = 696m.

2. (E)

1
1
1 4( x 2) 9 4 x
<4
4<0
=
<0
x2
x2
x2
x2

1o caso : 9 4 x > 0 e x 2 < 0 :


9
9 4x > 0 x <
e x 2 < 0 x < 2.
4
Como 2 <

9
a soluo so todos os nmeros x menores que 2, isto x < 2.
4

2o caso : 9 4 x < 0 e x 2 > 0 :


9
9 4x < 0 x >
e x2>0 x > 2
4

9
9
a soluo so todos os nmeros x maiores que 9/4, isto x > .
4
4
9
Logo, a soluo da inequao x < 2 ou x > .
4

Como 2 <

3. (E) Como os nmeros que aparecem so todos positivos, podemos elev-los ao quadrado
mantendo os sinais, isto : 20002 < n(n + 1) < 20052 . Observe que n e n + 1 so inteiros
consecutivos. Logo, temos as seguintes opes:

20002 < 2000 2001 < 20052


20002 < 2001 2002 < 20052
20002 < 2002 2003 < 20052
20002 < 2003 2004 < 20052
20002 < 2004 2005 < 20052
Logo, temos 5 possibilidades para n: 2000, 2001, 2002, 2003 e 2004 .

90

Olimpada Brasileira de Matemtica das Escolas Pblicas

OBMEP

4. Observe que OC um raio do crculo. Temos que OC=AB=5cm por serem as diagonais do
retngulo OABC. Logo, o dimetro 10 cm .
5. (C)Como a, b e c so nmeros naturais, segue que 3a mltiplo de 3 , 4b mltiplo de 4 e
7c mltiplo de 7 . Como 3, 4 e 7 so primos entre si (s possuem 1 como divisor comum), o
menor mltiplo comum de 3, 4 e 7 3 4 7 = 84 . Portanto:

3a = 84 a = 28 ; 4b = 84 b = 21 ; 7c = 84 c = 12 . Logo, o menor valor para a + b + c


28 + 21 + 12 = 61
.
6. (D) Lembre que a soma dos ngulos internos de um triangulo 180 .
Do tringulo STU temos que
=180- (75 + 30)=75. Logo, esse tringulo issceles (por ter
dois ngulos iguais) e portanto TU=SU. Como TU=SV, segue que SU=SV. Portanto, o tringulo
SUV tambm issceles, e portanto
=180-50=65.

2
7. Vamos listar os eventos ocorridos e contar o tempo gasto em cada um. A primeira atividade foi
colocar o gato fora da casa, logo nossa lista comea com essa atividade e o tempo contado a partir
dela.

Atividade
Gato fora de casa
Bolo no forno
Fazer o caf
Despertador toca
Gato entra em casa
Acabar de tomar o caf
Telefone toca
Desligar o telefone

Tempo depois que o gato


foi posto fora de casa
0 minutos
10 minutos
10+6=16 minutos
35+10=45 minutos
45-5=40 minutos
40+3=43 minutos
16+(40-16):2=28 minutos
28+5 =33 minutos

Podemos agora dar as respostas.


(a) s 3:59horas desliguei o telefone, o que ocorreu 33 minutos depois de colocar o gato fora
de casa. Logo a resposta 3:59-0:33=3:26.
(b) O despertador toca 45 minutos aps colocar o gato for a de casa.
(c) 28 minutos
Podemos saber exatamente a hora de cada atividade; veja na tabela a seguir.
Atividade
Gato fora de casa
Bolo no forno
Fazer o caf
Despertador toca
Gato entra em casa
Acabar de tomar o caf
Telefone toca
Desligar o telefone

Tempo depois que o gato


foi posto fora de casa
0 minutos
10 minutos
10+6=16 minutos
35+10=45 minutos
45-5=40 minutos
40+3=43 minutos
16+(40-16):2=28 minutos
28+5 =33 minutos

Hora atual
3:59-0:33=3:26
3:26+0:10=3:36
3:26+0:16=3:42
3:26+0:45=4:11
3:26+0:40=4:06
3:26+0:43=4:09
3:26+0:28=3:54
3:59

Olimpada Brasileira de Matemtica das Escolas Pblicas

91

NVEL 2

OBMEP

10a Lista

1) Se m um nmero natural tal que 3m = 81 , ento m3 igual a:


(A) 36

(B) 40

(C) 64

(D) 99

(E) 100

2. Quais figuras esto corretas?

FIGURA II
FIGURA I

B
FIGURA III

3) Sinal de um produto e sinal de um quociente: a, b, c e d so quatro nmeros no nulos


a
b
11
18
so positivos. Determine os sinais de a,
,
,
,
tais que os quocientes
5
7a
abc
abcd
b, c e d.
4) Quais dos nmeros abaixo so negativos?
10 3 11 ;

3 11 10 ;

18 5 13 ;

51 10 26 ;

10 26 51 .

5) As retas r e s so paralelas, encontre x e y:

Dia

2a-feira
3a-feira
4a-feira
5a-feira
6a-feira

92

Temperatura Temperatura
mxima
mnima em
o
em C
em oC
7
-12
0
-11
-2
-15
9
-8
13

-7

6) A tabela mostra as temperaturas mximas e

mnimas durante 5 dias seguidos em certa cidade.


Em qual dia ocorreu o maior variao de
temperatura?

Olimpada Brasileira de Matemtica das Escolas Pblicas

SOLUES

OBMEP

10a Lista

Solu
1. (C)Temos 3m = 81 = 34 ; donde m = 4 . Logo, m3 = 43 = 4 4 4 = 64.
2. Na figura I, temos 63o + 18o + 95o = 176o que menor do que 180o ; logo a figura est errada.
Na figura II, temos 112o + 72o = 184o que maior do que 180o ; logo a figura est errada.
Na figura III, temos 44o + 45o + 62o + 29o = 180o , e a figura est correta.
3. Soluo.

a
>0a >0
5
+

Temos a > 0 7 a > 0 , logo:

b
> 0 b > 0 b < 0
7a
+

11
> 0 abc > 0 . Como a > 0 e b < 0 segue que c < 0 (a b c > 0)
abc
+

18
> 0 abcd < 0 , como abc > 0 segue que d < 0 .
abcd

4. Como 100 > 99 ento 100 > 99 . Logo, 10 3 11 > 0 e 3 11 10 < 0 . Analogamente:
10

3 11

2601 > 2600 2601 > 2600 .


51

10 26

Assim, 51 10 26 > 0 e 10 26 51 < 0 .


Finalmente, 324 < 325 324 < 325 18 5 13 < 0. Os nmeros negativos so 3 11 10 ,
18

5 13

10 26 51 e 18 5 13.
5. Temos 80 + y = 180 y = 100

.Como as retas r e s so paralelas, segue que,

60 + x + 80 = 180 , donde x = 40 .
o

Dia

Temperatura Temperatura
mxima
mnima em
o
em C
em oC

Variao

6. A variao de temperatura a
diferena entre a mxima e a
mnima. Temos :
Logo, a maior variao ocorreu na
6a feira.

2a-feira
3a-feira
4a-feira

7
0
-2

-12
-11
-15

7-(-12)=7+12=19
0-(-11)=0+11=11
-2-(-15)=-2+15=13

5a-feira
6a-feira

9
13

-8
-7

9-(-8)=9+8=17
13-(-7)=13+7=20

Olimpada Brasileira de Matemtica das Escolas Pblicas

93

NVEL 2

OBMEP

11a Lista

Nvel

1) O nmero que fica entre 2/5 e 3/4


(A) 1 6

(B) 4 3

(C) 5 2

(D) 4 7

(E) 1 4

2) A figura mostra o retngulo maior dividido em


18 retngulos menores, todos com a mesma largura.
Que frao do retngulo maior representa a parte
em cinza?

3) Na lista de fraes, no quadro ao lado, temos:

5
4

5
2

2 fraes cuja soma

2 fraes cuja diferena

2 fraes cujo produto

5
2

17 -5 10 2
6
4 7 3
14 -1 5 -3
8
3 3 2

5
2

5
2
Encontre a frao que est sobrando.

2 fraes cujo quociente

4) No tringulo KLM temos KL=KM, KT=KS e LKS= 30 .


O ngulo x :
(A) 10
(B) 15
(C) 20
(D) 25
(E) 30
L

5) Escreva dentro dos crculos os nmeros inteiros que tornam correta a sucesso de
operaes.
x4

+1

+2

6
3

6) Iara possui R$ 50, 00 para comprar copos que custam R$ 2,50 e pratos que custam
R$ 7, 00 . Ela quer comprar no mnimo 4 pratos e 6 copos. O que ela pode comprar ?
94

Olimpada Brasileira de Matemtica das Escolas Pblicas

SOLUES

OBMEP

11a Lista

1. (D) 2/5 e 3/4 so menores que 1 (numerador menor que denominador)


; por sua vez, 4/3 e 5/2 so maiores que 1 (numerador maior que
denominador), logo (B) e (C) esto excludas. Temos 1/6 menor do que
1/4 . Como 1/4=0,25 e 2/5=0,4 segue que:

Um nmero x que fica


entre 2/5 e 3/4 um
nmero maior do que
2/5 e menor do que 3/4

1 1 2
. Logo o nico nmero entre 2/5 e 3/4 4/7.
<
<
6 4 5
0,25

ou seja

0,4

2
5

<x<

2. Observe na figura , a regio em cinza tem a mesma


rea que a do enunciado. Como todos os retngulos tm
a mesma largura, o retngulo maior est dividido em 4
partes iguais pelos segmentos paralelos ao seu
comprimento . Logo, a regio em cinza representa do
retngulo maior.

3. (a) 2 fraes cuja diferena

(b) 2 fraes cujo produto

(c) 2 fraes cuja soma

5 5 5 5 5 10 5
: = + =
=
2 4 4 4 4 4 2

5 10 14 10 7 10 5
:
= = =
2 7 8
7 4 4 2

5
4

17
6

14
8

-1
3

5
4
14
8

5 5 2 5 3 5
: = = .
2 3 3 3 2 2

Logo, o frao que est sobrando 3/2.

5
3

14
8
5
4

-1
3

Olimpada Brasileira de Matemtica das Escolas Pblicas

5
3
-5
4

-1
3

2
3

10
7

2
3

-3
2
10
7

5
3

2
3

-3
2

-5
4

17
6
14
8

10
7

5
3

17
6

10
7
-3
2

-5
4
-1
3

5
4

5 17 1 17 1 17 2 15 5
= = =
: + =
2 6 3 6 3 6 6 6 2

(d) 2 fraes cujo quociente

17
6

-5
4

2
3

-3
2

95

OBMEP

4. (B)

Sejam
. O tringulo KLM issceles porque tem dois lados
iguais; consequentemente seus ngulos da base so iguais, isto :
. Analogamente, o
. Usaremos agora que a soma dos
tringulo KST tambm issceles e portanto
ngulos internos de um tringulo 180o. Acompanhe na figura:
No tringulo STM temos: x + + 180o = 180o x =

No tringulo KLM temos: + + 30o + y = 180o y = 150o 2 . Logo,

+ + 150o 2 = 180o = 15o . Portanto, x = 15o .


5. Colocando x num dos crculos e aplicando a sucesso de operao obtemos x =

x+2
+1 ,
2

donde x=4.

4
x+2

+1=

+2

x+2

+1

x+2

x+2
2

6. Sejam c e p o nmero de copos e pratos que Iara pode comprar. Logo seu gasto 2, 5 c + 7 p . Ela
s tem R$ 50, 00 , logo 2, 5 c + 7 p 50 (I) Alm disso, ela quer comprar no mnimo 4 pratos e

6 copos, logo p 4 e c 6 (II). Devemos encontrar dois nmeros inteiros c e p (nmero de copos e
pratos so nmeros inteiros) que satisfaam (I) e (II).
Se ela comprar 4 pratos sobram 50 4 7 = 22 reais para os copos. Como 22 = 8 2, 50 + 2 , ela
podem comprar 8 copos (sobrando-lhe R$ 2, 00 ).
Se ela comprar 5 pratos sobram 50 5 7 = 15 reais para os copos. Como 15 = 6 2,50 , ela pode
comprar 6 copos.
Se ela comprar 6 pratos sobram 50 6 7 = 8 reais para os copos, o que lhe permite comprar
apenas 1 copo que no o que ela quer.
Logo, Iara pode comprar 4 pratos e 8 copos, ou 5 pratos e 6 copos.

96

Olimpada Brasileira de Matemtica das Escolas Pblicas

NVEL 2

OBMEP

12a Lista

1) Quantos so os nmeros inteiros x tais que 5 < x 1 5 ?


(A)

(B) 9

(C) 10

(D) 11

(E) 12

2) Na figura mostra nove quadrados. A rea do quadrado A 1cm2 e do quadrado B


81cm2. Qual a rea do quadrado I em centmetros quadrados?
(A) 196
(B) 256
(C) 289
(D) 324
(E) 361

3) Andr, Bruno, Celina e Dalva ganharam juntos 21 medalhas num concurso. Andr foi o
que mais ganhou medalhas, Bruno ganhou o dobro de Celina e Dalva 3 a mais que Bruno.
Quantas medalhas cada um pode ter ganhado?
4) Clia quer trocar com Guilherme figurinhas de um lbum sobre animais brasileiros.
Celina quer trocar figurinhas de 4 borboleta, 5 tubaro, 3 cobra, 3 periquito e 6 macaco.
Todas as figurinhas de Guilherme so de aranha. Eles sabem que:
(i) 1 figurinha de borboleta vale 3 figurinhas de tubaro
(ii) 1 figurinha de cobra vale 3 figurinhas de periquito
(iii)1 figurinha de macaco vale 4 figurinhas de aranha
(iv) 1 figurinha de periquito vale 3 figurinhas de aranha
(v) 1 figurinha de tubaro vale 2 figurinhas de periquito

Quantas figurinhas Clia receber se ela trocar todas que quiser?


5)Escreva numa linha os nmeros de 1 a 15 de modo que a soma de dois nmeros
adjacentes nessa linha seja um quadrado perfeito.

6) Um retngulo est dividido em 3 regies, duas


delas com reas 24 cm 2 e 13 cm 2 conforme
indicado na figura. Qual a rea da outra regio?

Olimpada Brasileira de Matemtica das Escolas Pblicas

97

SOLUES

OBMEP

12a Lista

Soluo

1. (C) Somando 1 a todos os membros das duas desigualdades temos


5 + 1 < x 1 + 1 5 + 1 4 < x 6 .
Os valores inteiros de x que satisfazem as duas desigualdades so: -3, -2, -1, 0, 1, 2, 3, 4, 5, 6.

2. (D) O lado de A

1 = 1cm e o de B

81 = 9cm . Agora temos:

Lado de G = lado de de B-lado de A=9-1=8cm


Lado de C = lado de B+lado de A=1+9=10cm
Lado de F = lado de G-Lado de A=8-1=7cm
Lado de H=lado de G+lado de F=8+7=15cm
Lado de B+lado de C=lado de G+lado
Logo, lado de E=4cm
Lado de D=ladoC+lado de E=10+4=14cm
Lado de I=lado de E+lado de D=18cm.

de

F+lado

de

9+10=8+7+lado de E.

Finalmente, a rea de I 182 = 324cm 2

3. Denotemos por A, B, C e D o nmero de medalhas ganhas por Andr,


respectivamente, ento A + B + C + D = 21 . Agora, temos:
Bruno ganhou o dobro de Celina B = 2C
Dalva 3 a mais que Bruno: D = B + 3
A
3
B
Da obtemos A + B + + B + 3 = 21 2 A + 5 B = 36. Como A e B 8
2
13
so nmeros inteiros, temos as seguintes possibilidades para A e B:
18

Como Andr foi o que mais recebeu medalhas,


a soluo A=3 e B=6 no serve. Agora usando
as condies C=B/2 e D=B+3, obtemos as
seguintes possibilidades de medalhas para
cada um deles, mostradas no quadro ao lado.

98

Andr
8
13
18

Bruno
4
2
0

Bruno, Celina e Dalva

Celina
4:2=2
2:2=1
0:2=0

Olimpada Brasileira de Matemtica das Escolas Pblicas

B
6
4
2
0

2A 5B
2356=36
2854=36
21352=36
21850=36

Dalva
4+3=7
2+3=5
0+3=3

Total
21
21
21

OBMEP

4. A moeda de troca de Guilherme so figurinhas de aranha, logo vamos calcular o valor-aranha


de cada tipo de figurinha usando as informaes (a), (b), (c), (d) e (e).

4 borboleta = 12 tubaro = 24 periquito = 72 aranha


(a)

(e)

43

(d )

1224

243

5 tubaro = 10 periquito = 30 aranha


(e)

(d)

5 2

103

3 cobra = 9 periquito = 27 aranha


(b)

(d)

33

93

6 periquito = 18 aranha
(d)

63

6 macaco = 24 aranha
(c)

6 4

Logo, ela receber 72 + 30 + 27 + 18 + 24 = 171 figurinhas de aranha.

5. Primeiro verificamos quais os nmeros que podem ser adjacentes.


Nmeros
Possveis
vizinhos

1
3
8
15

2
7
14

3
1
6
13

4
5
12

5
4
11

6 7
3 2
10 9

8
1

9
7

10
6
15

11
5
14

12
4
13

13
3
12

14
2
11

15
1
10

Os algarismos 8 e o 9 s tm cada um apenas um possvel vizinho, logo eles devem ser colocados
no incio e no fim da fila, seguidos de seus nicos vizinhos:
.
8 1 ? ?
? ?
?
?
?
?
?
?
?
7
9

Sobram os nmeros 2, 3, 4, 5, 6, 10,11 12, 13, 14 e 15. Na tabela de vizinhos vemos que ao lado do
7 s podemos colocar o 2 e ao lado do 2 o 14. Temos ento:
8 1

14

Olimpada Brasileira de Matemtica das Escolas Pblicas

99

OBMEP

Consultando a tabela de vizinhos e os nmeros que sobram, chegamos resposta. Veja a seguir
a soluo passo a passo.
Formao da linha em cada etapa

Sobram

8 1 ?

8 1 ?

2 7 9

3, 4, 5, 6, 10, 11, 12, 13,


14, 15

8 1 ?

14 2 7 9

3, 4, 5, 6, 10, 11, 12, 13,


15

8 1 ?

5 11 14 2 7 9

8 1 ?

4 5 11 14 2 7 9

8 1 ?

12 4 5 11 14 2 7 9

8 1 ?

13 12 4 5 11 14 2 7 9

8 1 ?

3 13 12 4 5 11 14 2 7 9

6, 10, 15

8 1 15 10 6 3 13 12 4 5 11 14 2 7 9

Resposta

6. Lembre que a rea de um tringulo

7 9

2, 3, 4, 5, 6, 10, 11, 12,


13, 14, 15

3, 4, 6, 10, 12, 13, 15


3, 6, 10, 12, 13, 15
3, 6, 10, 13, 15
3, 6, 10, 15

base altura
, onde a altura relativa base escolhida. No
2

tringulo AEB temos base = AB=comprimento do retngulo e a altura relativa a essa base BC=

AB BC
= 24 AB BC = 48 . Logo a rea do retngulo 48cm2.
2
Portanto, a rea pedida 48 (24 + 13) = 48 37 = 11cm 2 .
largura do retngulo. Logo,

58
100

Olimpada Brasileira de Matemtica das Escolas Pblicas

NVEL 3

OBMEP

1a Lista

NVEL 3

1) Juliano encaixou duas rodas dentadas iguais, cada uma com


uma bandeirinha igual desenhada, como mostra a figura ao
lado.
Ento ele girou a roda da esquerda um pouco. Qual das alternativas abaixo pode
representar a posio final das rodas?

A)

B)

C)

E)

D)

2) Quantas fraes da forma

n
n+1

so menores do que

7
, sabendo que
9

n um

nmero inteiro positivo?


A) 1

B) 2

C) 3

D) 4

E) 5

3) Numa certa povoao africana vivem 800 mulheres. Delas, 3 % usam apenas um
brinco; das restantes, metade usa dois brincos e a outra metade, nenhum. Qual o
nmero total de brincos usados por todas as mulheres?
A) 776

B) 788

C) 800

D) 812

E) 824

4) Ana, Bento e Lucas participam de um concurso que consta de 20 perguntas com a


seguinte regra:
Nmero
Nmero
Nmero
de
respostas
certas

cada resposta certa ganha 5 pontos,


cada resposta errada perde 3 pontos,
cada resposta em branco perde 2 pontos.

de
respostas
erradas

de
respostas
em branco

Ana
12
4
4
Bento
13
0
7
5
Lucas
3
12
Veja os resultados na tabela a seguir:
Escrevendo os nomes dos trs em ordem decrescente de classificao no concurso,
encontramos:
A) Ana, Bento, Lucas
B) Lucas, Bento, Ana
C) Ana, Lucas, Bento
D) Lucas, Ana , Bento E) Bento, Lucas, Ana
5) Uma cerca de arame reta tem 12 postes igualmente espaados. A distncia entre o
terceiro e o sexto poste de 3,3 m. Qual a distncia entre o primeiro e o ltimo poste?
A) 8, 4 m

B) 12,1 m

C) 9,9 m

D) 13, 2 m

Olimpada Brasileira de Matemtica das Escolas Pblicas

E) 9,075 m
101

OBMEP

6) Uma folha quadrada foi dobrada


duas vezes ao longo de suas diagonais
conforme ilustrao ao lado, obtendo-se
um tringulo.
Foi feito um corte reto na folha dobrada, paralelo ao lado maior desse tringulo,
passando pelos pontos mdios dos outros lados, e desdobrou-se a folha. A rea do
buraco na folha corresponde a qual frao da rea da folha original ?
A)

1
2

B)

1
6

C)

3
8

D)

7) Qual o menor nmero inteiro positivo N tal que

3
4

N
N
N
N
,
,
,
3
5
6
4

E)

1
4

N
7

nmeros inteiros?
A) 420

B) 350

C) 210

D) 300

E) 280

8) Uma formiguinha vai caminhar de A at C


passando por B, podendo passar apenas uma vez por
esses pontos e pelos caminhos indicados na figura.
Qual o nmero de maneiras diferentes que ela pode escolher para ir de A at C ?
A) 3

B) 5

C) 7

D) 8

E) 9

9) Dados a e b nmeros reais seja a b = a 2 ab + b2 . Quanto vale 1 0 ?


A) 1

B) 0

C) 2

D) -2

E) -1

10) O diagrama de barras mostra a distribuio dos


alunos de uma escola de acordo com o tempo que
gastam no trajeto de casa para a escola. As fraes de
minuto no foram consideradas; assim, se um aluno
gasta 40 minutos e 15 segundos neste trajeto, considerase que o tempo gasto de 40 minutos.
Responda as perguntas seguintes justificando sua
resposta.
(a) Quantos alunos gastam menos de 20 minutos para chegar escola?
(b) Quantos alunos tm esta escola?
(c) Quantos alunos gastam mais do que 40 minutos para chegar escola?
(d) verdade que a maioria dos alunos gasta mais de 20 minutos no trajeto escola?

102

Olimpada Brasileira de Matemtica das Escolas Pblicas

so

SOLUES

OBMEP

1a Lista

Solues

1. (A) Os dois discos giram em sentidos opostos; quando um gira no sentido horrio, o outro
gira no sentido anti-horrio. Considerando que a engrenagem da esquerda girou um ngulo x
em um sentido, a engrenagem da direita girou o mesmo ngulo x no sentido oposto, e portanto
a bandeirinha ficou na posio mostrada na alternativa (A).

2. (C) Soluo 1 1
2

As fraes da forma

n=1

2
3

n
, com n inteiro positivo so:
n+1
3
4

n=3

n=2

4
5

Observe que esta seqncia de fraes crescente, isto :

6
n=5

n=4

1 2 3 4
< < < <
2 3 4 5

Para comparar cada uma dessas fraes com 7/9 precisamos igualar os denominadores. Temos:
1 9 14 7
=
<
=
2 18 18 9

2 6 7
= <
3 9 9

3 27 28 7
=
<
=
4 36 36 9

4 36 35 7
=
>
= .
5 45 45 9

Logo, 4/5 maior do que 7/9, e como a seqncia crescente, a partir de 4/5 todas as fraes
desta seqncia so maiores do que 7/9. Assim, as fraes da forma

n
7
menores do que
so
9
n+1

1 2 3
, , . Portanto, a resposta 3.
2 3 4

Soluo 2 Transformando em nmeros decimais temos: 7/9= 0, 777 e 1/2=0,5 ;


2/3= 0, 666 ; 3/4=0,75 ; 4/5=0,8 ; 5/6= 0,8333
Logo, a seqncia crescente e apenas 1/2=0,5 ; 2/3= 0, 666 ; 3/4=0,75 so menores do que
7/9= 0, 777

9n 7 ( n + 1) 2n 7
n
7
7
n
<0
=
< 0 . Como
< , ento
n +1 9
9 ( n + 1)
9 ( n + 1)
n +1 9
7
9 ( n + 1) > 0 , devemos ter 2n 7 < 0 , isto n < = 3,5 . Logo, n = 1, 2,3 e portanto, as fraoes
2
1 2 3
so , e .
2 3 4
Soluo 3 - Se

3. (C) Soluo 1 - Do enunciado temos que o nmero de mulheres que usam apenas um
brinco 0,03 800=24 . Restam 800-24=776 , das quais 388 usam dois brincos e 388 no usam
brincos. Logo, o nmero total de brincos usados por todas as mulheres : 24+388 2=800 .
Soluo 2 - Se cada mulher com dois brincos der um dos seus a uma das que no tm
brincos, todas as 800 mulheres ficaro com um nico brinco. Logo, o nmero de brincos igual
ao de mulheres, ou seja, 800 .
Olimpada Brasileira de Matemtica das Escolas Pblicas

103

OBMEP

4. (E) O nmero de pontos de cada um deles :


Ana:

5 12 + (-3) 4 + (-2) 4 = 60 - 12 - 8 = 40

Bento: 5 13 + (-3) 7 + (-2) 0 = 65 - 21 = 44


Lucas: 5 12 + (-3) 3 + (-2) 5 = 60 - 9 - 10 = 41
Logo, Bento foi o mais bem classificado, seguido de Lucas e depois de Ana.

5. (B) A distncia entre dois postes consecutivos 3,3 m = 1,1 m , donde a distncia entre o
3

primeiro e o ltimo poste 11 1,1 m = 12,1 m

6. (E) Denotemos por a o lado do quadrado que dobrado.


Soluo 1 - Na figura abaixo mostra o tringulo obtido aps dobrar o quadrado ao longo das
duas diagonais. Temos BC = a , o lado da folha quadrada original. Como o corte feito pela
MN do tringulo, temos:
base mdia
MN =

BC a
=
. Desdobrando-se a folha, vemos que
2
2

o buraco um quadrado de lado MN . A rea do


2
quadrado inicial a e a do quadrado retirado
2

a2
a
. Logo, o buraco tem um quarto da rea
=

4
2

do quadrado original.

Soluo 2 - O corte realizado pela base mdia do tringulo,


retirando um tringulo pequeno semelhante ao original com razo
de semelhana 1 ; deste modo, o tringulo retirado tem um quarto
2

da rea do tringulo original. Abrindo a folha, vemos essa situao


reproduzida quatro vezes, donde o buraco tem um quarto da rea
do quadrado original.

104

Olimpada Brasileira de Matemtica das Escolas Pblicas

OBMEP

7. (A) Para que

N
N
N
N
N
,
,
,
e
sejam nmeros inteiros, N deve ser mltiplo comum de
3
5
6
4
7

3 , 4 , 5 , 6 e 7. Como queremos o menor N possvel, ele deve ser o menor mltiplo comum de 3,
4 , 5, 6 e 7. Sendo o MMC entre 3 , 4, 5, 6 e 7 igual a 420, temos N =420.

8. (E) Para cada um dos 3 caminhos para ir de A at B, existem 3 opes para ir de B a C.


Logo, h um total de 3 3 = 9 possibilidades. Mas geralmente, se fossem m os caminhos de A
at B e n os de B at C, ento o nmero de caminhos que nossa formiguinha poderia tomar de
A at C seria m n ; esta afirmativa um caso particular do Princpio multiplicativo.

9. (A) Fazendo a = 1 e b = 0 em ab= a2 ab+b2 obtemos: 10 = 12 - 1 0 + 0 2 = 1 .

10. Conforme o enunciado, os alunos foram divididos em 4 grupos distintos. Cada uma das
quatro barras do diagrama representa apenas um desses grupos.

(a) Os alunos que gastam menos de 20 minutos em seu trajeto de casa para a escola
esto representados pela barra mais alta, que atinge a marca 90 . Logo, 90 alunos
gastam menos de 20 minutos para chegar escola.
(b) Como j dito acima, cada barra representa um grupo diferente de alunos. Logo,
o total de alunos na escola a soma dos nmeros representados pelas quatro
barras; isto : 90 + 60 + 10 + 20 = 180 alunos.
(c) Os alunos que gastam mais de 40 minutos so aqueles que esto em dois
grupos: os que gastam de 41 a 60 minutos e os que gastam mais do que
60 minutos. No diagrama, esses grupos esto representados por duas barras; uma
atinge a marca 10 e a outra, a marca 20 , respectivamente. Logo, o total de alunos
que gastam mais do que 40 minutos para chegar escola de 10 + 20 = 30 .
(d) Do item anterior, sabe-se que 30 alunos gastam mais do que 40 minutos para
chegar escola. Do diagrama, observa-se que 60 alunos gastam de 20 a
40 minutos. Portanto, temos no mximo 30 + 60 = 90 alunos que gastam mais do que
20 minutos para chegar escola. Como a escola tem 180 alunos, conclumos que a
resposta para esta pergunta no.

Olimpada Brasileira de Matemtica das Escolas Pblicas

105

NVEL 3

OBMEP

2a Lista

NVE

1) Uma cidade ainda no tem iluminao eltrica e todos usam velas noite. Na casa de
Joo, usa-se uma vela por noite, sem queim-la totalmente. Com os tocos de quatro
destas velas, possvel fazer uma nova vela. Durante quantas noites Joo poder
iluminar sua casa com 43 velas?
A) 43

B) 53

C) 56

D) 57

E) 60

2) Uma fbrica embala 8 latas de palmito em caixas de papelo cbicas de 20 cm de lado.


Estas caixas so colocadas, sem deixar espaos vazios, em caixotes de madeira de 80 cm
de largura por 120 cm de comprimento por 60 cm de altura. Qual o nmero mximo de
latas de palmito em cada caixote?
A) 576

B) 4608

C) 2304

D) 720

E) 144

3) Um atleta corre 5000 m por semana em uma quadra de esportes, que tem uma pista
curta e outra longa. Em uma semana ele treinou seis dias, sendo que a cada dia correu
uma vez na pista longa e duas na pista curta. Na semana seguinte ele treinou sete dias,
sendo que a cada dia correu uma vez em cada pista. Podemos ento afirmar que:
A) a pista longa trs vezes maior que a curta.
B) a pista longa quatro vezes maior que a curta.
C) a pista longa cinco vezes maior que a curta.
D) a pista longa 600 m mais longa que a curta.
E) a pista longa 500 m mais longa que a curta.
4) O limite de peso que um caminho pode transportar corresponde a 50 sacos de areia
ou 400 tijolos. Se este caminho j contm 32 sacos de areia, quantos tijolos, no mximo,
ele ainda pode carregar?
A) 132

B) 144
1
3
1
B)
15

C) 146

D) 148

E) 152

5) Sabendo-se que 0, 333... = , qual a frao irredutvel equivalente a 0,1333 ?


A)

1
13

C)

1
30

D)

2
15

E)

1333
10000

6) Andr treina para a maratona dando voltas em torno de uma pista circular de raio
100 m . Para percorrer aproximadamente 42 km , o nmero de voltas que Andr precisa
dar est entre:
A) 1 e 10

106

B) 10 e 50

C) 50 e 100

D) 100 e 500

Olimpada Brasileira de Matemtica das Escolas Pblicas

E) 500 e 1000

OBMEP
1
so as razes da equao ax 2 - 6 x + c = 0 , qual o valor de a + c ?
3
9
18
A) 1
B) 0
C) D)
5
5

7) Se 3 e

E) -5

8) Os vrtices de um cubo so numerados com os nmeros de 1 a 8 , de tal modo que


uma das faces tem os vrtices {1, 2, 6, 7} e as outras cinco tm vrtices {1, 4, 6, 8}, {1, 2, 5,
8}, {2, 3, 5, 7}, {3, 4, 6, 7} e {3, 4, 5, 8}. Qual o nmero do vrtice que est mais distante
daquele de nmero 6?
A) 1

B) 3

C) 4

D) 5

E) 7

9) O grfico ao lado mostra o nmero de pontos que


os oito jogadores de basquete do time da escola
marcaram no ltimo jogo.
Qual o nmero total de pontos marcados pelo time?
A) 54

C) 12

B) 48
D) 58

E) 46

10) No ltimo campeonato de futebol do bairro em que moro participaram 6 equipes.


Cada equipe disputou com cada uma das outras exatamente uma partida. Abaixo, a
tabela de classificao do campeonato, onde
Equipe V E D GP GC
V o nmero de vitrias de uma equipe
A
4 1 0 6
2
E o nmero de empates
B
2 1 2 6
6
D o nmero de derrotas
C
0 3 2 2
6
GP o nmero de gols feitos por um time
D
1 1 y 3
6
GC o nmero de gols sofridos
E
0 1 4 1
5
F
3
x 1 0 z
a) Quantas partidas foram disputadas?
b) A tabela est incompleta. Determine a quantidade de vitrias da equipe F, a
quantidade de derrotas da equipe D e a quantidade de gols feitos pela equipe F,
representados por x , y e z na tabela.

Olimpada Brasileira de Matemtica das Escolas Pblicas

107

SOLUES

OBMEP

2a Lista

1. (D) De 43 velas obtm-se 43 tocos. Como 43= 4 10+3 , com esses 43 tocos se pode fazer
10 velas e guardar 3 tocos. Dessas 10 velas, obtemos 10 tocos que, com os 3 que sobraram, do
13 . Sendo 13= 4 3 + 1 , fazemos ento 3 velas com 12 tocos, sobrando 1 toco. Depois de usar estas
3 velas, teremos um total de 4 tocos, que nos d 1 vela extra. No total, obtemos 43 + 10 + 3 + 1 = 57 .

2. (A) Em cada caixote de madeira de dimenses a b c cabem, empilhados regularmente,


a b c
cubos de lado l . No nosso caso, a = 60 , b = 80 , c = 120 e l = 20 . Como 60 , 80 e 120 so
l l l
60 80 120

= 72 caixas de
mltiplos de 20 , podemos encher o caixote sem deixar espaos com
20 20 20
papelo cbicas de 20 cm de cada lado. Logo, em cada caixote cabem 72 8 = 576 latas de

palmito.
3. (C) Soluo 1 - Denotemos por x e y os comprimentos das pistas longa e curta,
respectivamente.
Numa semana, ele corre 6 ( x + 2 y ) e na outra 7( x + y ) . Como, em cada semana, ele corre os
mesmos 5000 metros, temos: 6 ( x + 2 y ) = 7( x + y ) .
Segue que 6 x + 12 y = 7 x + 7 y , e portanto, 5 y = x .
Assim, o comprimento da pista longa cinco vezes o da pista curta.
Soluo 2 - Na semana em que Joozinho treinou sete dias, ele correu uma pista longa a
mais e cinco pistas curtas a menos do que a semana em que ele treinou apenas seis dias. Como
a distncia corrida foi a mesma nas duas semanas, conclumos que o comprimento da pista
longa igual ao comprimento de cinco pistas curtas.
4. (B) O enunciado mostra que o peso de 1 saco de areia o mesmo que o de 8 tijolos. Se no
caminho j h 32 sacos de areia, ele pode carregar ainda 18 sacos, o que equivale
18 8 = 144 tijolos.
5. (D) Soluo 1 Usando o dado da questo temos:
0,1333 =

1, 333 1 + 0, 333 1 1 1 4 2
.
=
= 1+ = =
10
10
10 3 10 3 15

Soluo 2 Usando a regra que fornece a geratriz de uma dzima peridica, temos:
0,1333 =

13 - 1 12 2
=
=
.
90
90 15

6. (C) O comprimento de uma circunferncia de raio r 2r. Assim, em cada volta, Andr
percorre 2 100m =200m. Logo, o nmero de voltas que Andr precisa dar

42000 210
.
=
200

Podemos agora finalizar o problema de duas maneiras:


1a) A aproximao de t a segunda casa decimal 3,14. Da,

210 210

66,878 66,88 . Como

3,14

66 , 88 est entre 50 e 100 , a opo correta C.

108

Olimpada Brasileira de Matemtica das Escolas Pblicas

OBMEP
2a) Como 3 < < 4 segue que

1 1
<
3

1 1
< . Multiplicando ambos os lados dessas
4

desigualdades por 210 obtemos:


210 210
210 210
<
= 70 e
<
.

3
4

Como

210
= 52, 5 , conclumos que Andr deve dar entre 53 e 70 voltas na pista para percorrer
4

42000 m .

7. (D) Soluo 1 Como 3 e 1/3 so razes da equao ax 2 6x + c = 0 temos:


9 a 18 + c = 0

9a + c = 18 e

a
2+c = 0
9

a
+c = 2 .
9

Resolvendo o sistema
9 a + c = 18

a
9 + c = 2

obtemos a = c =

9
18
. Logo, a + c =
.
5
5

b
e o
a
1 10 6
c
9
c
1
9 18
produto . Logo: 3 + = =
a = e 3 = 1 = a = c . Assim, a + c = 2 = .
3 3 a
3
5 5
a
a
5

Soluo 2 Numa equao do 2o grau

ax 2 + bx + c = 0 , a soma das razes

8. (D) Soluo 1 Desenhando o cubo e numerando seus vrtices de


acordo com o enunciado da questo, obtemos a figura abaixo, onde
podemos ver que o vrtice 5 o mais distante do vrtice 6.
Soluo 2 O vrtice 6 est nas faces {1, 2, 6, 7}, {1, 4, 6, 8} e {3, 4,
6, 7}. Como nestas faces no aparece o 5 , segue que este o vrtice
diagonalmente oposto ao 6 , ou seja, o 5 o vrtice mais distante do 6 .
9. (B) Basta ler no grfico o nmero de pontos de cada aluno e somar para obter o total:
7 + 8 + 2 + 11 + 12 + 1 + 7 = 48

10. a) Cada uma das 6 equipes joga 5 partidas. Portanto, o nmero de partidas foi de

65
= 15 .
2

Outra maneira de contar: Podemos formar grupos de duas letras e contar o nmero de
grupos: AB, AC, AD, AE, AF, BC, BD, BE, BF, CD, CE, CF, DE, DF, EF o nmero de
partidas 15 .
b) Para cada time, a soma do nmero de vitrias, empates e derrotas igual a 5 . Assim,
temos 1 + 1 + y = 5 , ou seja, y = 3 . Temos, tambm, x + 1 + 0 = 5 , isto , x = 4 . O nmero total de
gols feitos igual ao nmero total de gols sofridos. Assim, z + 18 = 28 , ou seja, z = 10 .
Resumindo: O nmero de derrotas do time D 3, o nmero de vitrias da equipe F 4 e o
nmero de gols sofridos pela equipe F 10.

Olimpada Brasileira de Matemtica das Escolas Pblicas

109

OBMEP

NVEL 3

3a Lista

Nvel

1) Na figura ao lado ABCD um retngulo e ABE e CDF so


tringulos retngulos. A rea do tringulo ABE 150 cm 2 e os
segmentos AE e DF medem, respectivamente, 15 cm e 24 cm . Qual
o comprimento do segmento CF?

2) Usando apenas os dgitos 1, 2, 3, 4 e 5 , Peri construiu uma seqncia da seguinte


forma: um 1 , dois 2 , trs 3 , quatro 4 , cinco 5 , seis 1 , sete 2 e assim por diante; abaixo
vemos os primeiros termos desta seqncia:
1, 2, 2, 3, 3 ,3, 4, 4, 4, 4, 5, 5, 5, 5, 5, 1, 1, 1, 1, 1, 1, 2, 2, 2, ....

Qual o 100 termo dessa seqncia?


3) A figura ao lado foi montada com 12 azulejos quadrados
de lados iguais a 10 cm . Qual a rea da regio hachurada?
4) Capitu tem cem cartes numerados de 1 a 100 . Todos os
cartes tm uma face amarela e a outra vermelha, e o nmero de cada carto est
escrito em ambas as faces. Os cartes foram colocados sobre uma mesa, todos com a
face vermelha voltada para cima. Capitu virou todos os cartes de nmero par e depois
todos os cartes de nmero mltiplo de 3, colocando-os com a face amarela voltada
para cima. Quantos cartes ficaram com a face vermelha para cima ?
5) Para encher de gua um tanque em forma de
um bloco retangular de 300cm de comprimento,
50 cm de largura e 36 cm de altura, um homem
utiliza
um
balde
cilndrico, de 30 cm de
dimetro em sua base e
48cm de
altura,
para
pegar gua numa fonte. Cada vez que ele vai fonte, ele enche
4
5

do balde e no caminho derrama 10 % do seu contedo.

Estando o tanque inicialmente vazio, quantas viagens fonte o


homem ter que fazer para que a gua no tanque chegue a
sua altura?

110

Olimpada Brasileira de Matemtica das Escolas Pblicas

3
4

de

SOLUES

OBMEP

3a Lista

1. O segmento CF, que queremos calcular, um cateto do tringulo retngulo CDF. O teorema
de Pitgoras, aplicado a este tringulo, diz que CD 2 = CF 2 + FD 2 = CF 2 + 24 2 e da tiramos
2
2
2
CF = CD 24 . Ou seja, para achar CF basta conhecer CD. Como os lados opostos de um
retngulo (e, mais geralmente, de um paralelogramo) so iguais, temos CD = AB , e nosso
objetivo passa a ser o clculo de AB.
Para isso, olhemos para o tringulo ABE; sua rea
BE = 20 .

tiramos

15 BE
2

= 150 , donde

de Pitgoras aplicado a este tringulo nos d


AB = AE + BE = 15 + 20 = 625 = 25 , donde AB = 25 .
Logo CD = AB = 25 e, de acordo com nossa observao anterior, temos
2
2
2
2
2
CF = CD 24 = 25 24 = (25 + 24)(25 24) = 49 . Obtemos ento CF = 7 .
2

AE BE

teorema

Notamos que a soluo independe da medida dos lados AD e BE.


2. Agrupamos a seqncia em blocos numerados consecutivamente, cada bloco formado pelos
termos iguais consecutivos, como mostrado a seguir.

, 2, 2 , 3, 3, 3 , 4, 4, 4, 4 , 5, 5, 5, 5, 5 , 1,1,1,1,1,1 , 2, 2, 2, 2, 2, 2, 2 , 3, 3, 3, 3, 3, 3, 3, 3 , , k , k , k , k , k , , k

1
bloco1

bloco 2

bloco 3

bloco 4

bloco 5

bloco 6

bloco 7

bloco n
k{1,2,3,4,5}

bloco 8

Observe que a numerao de cada bloco coincide com o nmero de termos que ele contm: o
bloco 1 tem 1 termo, o bloco 2 tem 2 termos, o bloco 3 tem 3 termos,, o bloco n tem n termos.
A posio na seqncia do ltimo termo de cada bloco obtida somando todos os nmeros de
1 at o nmero atribudo ao bloco. Por exemplo:
o ltimo 3 do bloco 8 o 36 termo da seqncia porque 1 + 2 + 3 + 4 + 5 + 6 + 7 + 8 = 36 .
o

1
do
bloco
11

1 + 2 + 3 + 4 + 5 + 6 + 7 + 8 + 9 + 10 + 11 = 66 .

ltimo

66

termo

da

seqncia

porque

Em geral, o ltimo elemento do bloco n est na posio 1 + 2 + 3 + + n . Para calcular o valor


desta soma, lembramos que 1, 2, 3, , n uma progresso aritmtica de razo 1, termo inicial
a1 = 1 e n-simo termo an = n ; a soma de seus n primeiros termos
1+ 2 +

+n=

n( a1 + an )
2

n ( n + 1)
2
36o

, 2, 2 , 3, 3, 3 , 4, 4, 4, 4 , 5, 5, 5, 5, 5 , 1,1,1,1,1,1 , 2, 2, 2, 2, 2, 2, 2 , 3, 3, 3, 3, 3, 3, 3, 3 , 4, 4, 4, 4, 4, 4, 4, 4, 4 ,

bloco1 bloco 2

bloco 3

bloco 4

bloco 5

bloco 6

66o

5, 5, 5, 5, 5, 5, 5, 5, 5, 5 , 1,1,1,1,1,1,1,1,1,1, 1
bloco10

bloco11

bloco 7

bloco 9

bloco 8

n ( n +1) o
2

, 2, 2, 2, 2, 2, 2, 2, 2, 2, 2, 2, 2 , , k , k , k , k , , k
bloco12

Olimpada Brasileira de Matemtica das Escolas Pblicas

bloco n

111

OBMEP
Precisamos agora descobrir em qual bloco se encontra o 100 termo da seqncia. Suponhamos
que ele esteja no bloco n; ento sua posio no mximo a do ltimo termo deste bloco. Como
ele no est no bloco n + 1 , conclumos que n o menor inteiro tal que
100

n ( n + 1)
2

ou seja
200 n ( n + 1)

Para determinar o valor de n devemos resolver essa inequao e escolher entre suas solues o
menor nmero inteiro. Como a expresso bastante simples, mais fcil resolv-la por
tentativas. Fazendo isso, vemos que n = 14 ; de fato, 13 (13 + 1) = 182 < 200 e 14 (14 + 1) = 210 > 200 .
Logo o 100 termo da seqncia est no bloco 14 . Os nmeros que aparecem nos blocos se
repetem de 5 em 5 blocos na ordem 1, 2, 3, 4, 5 . Como 14 = 5 2 + 4 , o bloco 14 formado pelo
nmero 4 . Assim, o 100 termo da seqncia 4 .

Comentrio: A resoluo acima da inequao 200 n( n + 1) apesar de correta, no serviria se o


problema pedisse, por exemplo, para determinar o 10.000o termo da seqncia. Neste caso,
teramos que lidar com a inequao 20.000 n ( n + 1) , e claro que achar sua menor soluo
inteira por tentativas no funciona (a no ser com muita, muita sorte!). Por isso vamos resolvla como seria feito para um caso qualquer.
Primeiro escrevemos 200 n ( n + 1) como n 2 + n 200 0 , o que nos leva ao estudo de sinal da
funo quadrtica f ( x ) = x 2 + x 200 . As razes de f ( x ) so x1 =

x2 =

1 + 1 + 800
2

1 1 + 800
2

, que negativa, e

, que aproximadamente 13, 6 . O grfico

de f ( x ) est ilustrado na figura ao lado. Como f ( x ) 0


x x2 , segue que o n que estamos
procurando o menor inteiro que maior ou igual a x2 ,

para

x x1

ou seja, n = 14 .

Agora, se quisssemos determinar o 10.000 termo da seqncia repetindo procedimento acima,


encontraramos x2 =

1 + 1 + 80000
2

, que aproximadamente 140, 9 . Logo n = 141 e o 10.000 termo

da seqncia est no 141 bloco. Como 141 = 28 5 + 1 segue que o 10.000 termo 1 .

112

Olimpada Brasileira de Matemtica das Escolas Pblicas

OBMEP

3.A figura dada pode ser decomposta em quatro figuras iguais


figura ao lado. Para calcular a rea do tringulo escolhemos
como base o lado BC; a altura correspondente ento AE.
Como os azulejos so quadrados de lado 10 cm, segue que
AE = BC = 10 cm ,
e
a
rea
do
tringulo
BCE

base altura
2

10 10
2

= 50 cm2 . Logo, a rea da regio hachurada 4 50 = 200cm .

4. Soluo 1: Capitu virou, em primeiro lugar, os 50 cartes pares; aps isto, ficaram ento na
mesa os 50 cartes pares com a face amarela para cima e os 50 cartes mpares com a face
vermelha para cima. Ao virar agora os mltiplos de 3, ela virou apenas os mltiplos de 3
mpares, que so 3, 9, 15, 21, 27, 33, 39, 45, 51, 57, 63, 69, 75, 81, 87, 93 e 99 . Assim, temos 17
mltiplos de 3 que so mpares. Logo, Capitu virou para cima a face amarela de
50 + 17 = 67 cartes. Sobraram com a face vermelha para cima 100 67 = 33 cartes.
Comentrio: Nesta soluo, para determinar quantos so os mltiplos de 3 mpares menores do
que 100 suficiente escrever esses mltiplos e cont-los. No entanto, se Capitu tivesse 1000
cartes (ou mais) esse procedimento seria bastante trabalhoso. Mas, nesse caso podemos
proceder de modo mais geral. Notamos que os mltiplos mpares de 3 de 1 a 1000 formam uma
progresso aritmtica com primeiro termo a1 = 3 , razo r = 6 e o ltimo termo an = 999 . Para
determinar n usamos a frmula an = a1 + ( n 1) r , que no nosso caso 999 = 3 + ( n 1) 6 .
Conclumos que n = 167 , ou seja, temos 167 mltiplos mpares de 3 menores do que 1000.
Soluo 2: Capitu virou, em primeiro lugar, os 50 cartes pares; aps isto, ficaram ento na
mesa os 50 cartes pares com a face amarela para cima e os 50 cartes mpares com a face
vermelha para cima. Ao virar ento os cartes mltiplos de 3, Capitu fez o seguinte:
Entre os cartes pares: Ela virou os que eram tambm mltiplos de 3. Um nmero que
mltiplo de 2 e de 3 tambm mltiplo de 6. Como 100 = 16 6 + 4 , conclumos que
Capitu virou 16 cartes entre os cartes pares. Estes cartes voltaram a ficar com a face
vermelha para cima, ficando os outros 34 com a face amarela para cima.
Entre os cartes mpares: Como 100 = 33 3 + 1 , segue que o nmero total de cartes (pares
e mpares) mltiplos de 3 33. Como vimos acima, entre estes cartes 16 so pares, logo
17 so mpares. Assim, Capitu virou 17 cartes mpares, e estes cartes passaram a ter a
face amarela para cima, enquanto que os outros 33 continuaram com a face vermelha
para cima.

5. Nesta soluo todas as medidas de volume so dadas em cm3.


O volume V do balde dado pela frmula habitual do volume de um cilindro, ou seja,
V = rea da base altura . A base do balde um crculo de dimetro 30cm; seu raio ento
2
2
r = 15 cm cm e sua rea r = 225 cm . Logo V = 48 225 = 10.800 . A cada viagem, o volume

de gua que o homem coloca no balde

4
5

de V, e deste volume ele perde 10%.

Olimpada Brasileira de Matemtica das Escolas Pblicas

113

OBMEP
Logo, resta no balde 90% de

4
5

de V, isto ,

4
18
V =
V = 0, 72V = 0, 72 10800 = 7776 ; esta
10 5
25

quantidade de gua que ele coloca no tanque em cada viagem, que denotaremos por B.
3

temos

625
12

do tanque T =

300 36 50 = 405.000 . Logo, o nmero de baldes necessrios


4
405000 405000 625
para atingir esse volume
=
=
. Usando a aproximao 3,14 para o nmero
B
7776
12

O volume de

625
12 3,14

16, 587. Logo o homem necessitar 16 baldes mais 0,587 de um balde.

Conclumos que o homem dever fazer 17 viagens.

Comentrio: Usamos acima uma aproximao para o valor de ; importante entender o que
isto quer dizer. Como sabemos, um nmero irracional, e sua expanso decimal infinita e
no peridica. O valor aproximado de
com 31 casas decimais
3,1415926535897932384626433832795 (o smbolo quer dizer aproximadamente). Por qu
ento no usar 3,142 ou 3,1416 para resolver nosso problema, em vez de 3,14 ? Para
discutir isto, vamos a um exemplo.
Suponhamos que voc tem um balde cilndrico com raio da base 1m e altura 1m, e uma
caixa de gua de volume de exatamente 3,141 m3. O balde deve ser enchido em uma fonte.
Quantas viagens fonte sero necessrias para encher a caixa, supondo que o volume de gua
de cada balde integralmente transferido para a caixa?
Usando a aproximao 3,14 obtemos 3,14 m3 para o volume do balde. Como
volume do tanque
volume do balde

3,141
3,14

maior que 1 (e, claro, menor que 2), conclumos que sero

necessrias duas viagens fonte para encher a caixa de gua.


Vamos agora usar a aproximao = 3,1416 . Aqui calculamos o volume do balde e
obtemos 3,1415 m3. Ento

volume do tanque
volume do balde

3,141
3,1416

menor que 1, e conclumos agora que basta

uma viagem fonte para encher o balde, resultado diferente do anterior!

114

Olimpada Brasileira de Matemtica das Escolas Pblicas

OBMEP

Deve ficar claro com este exemplo que a escolha inicial de uma aproximao pode influenciar
fortemente o resultado final. Nesse caso dizemos que as condies do problema so sensveis
aproximao. No nosso problema original (problema 5) , os dados iniciais no eram sensveis
aproximao usada para . Pode-se verificar isto imediatamente repetindo a soluo com
3,142 ou 3,1416 ; em qualquer caso, obtem-se o resultado de 17 viagens.
Em geral, os problemas deste tipo propostos em livros nos ensinos fundamental e mdio
so enunciados de modo pouco sensvel aproximao. Isto justifica parcialmente o uso de
= 3,14 e de, por exemplo, 2 = 1, 41 (curiosidade: 2 1,4142135623730950488016887242097 ).
Notamos tambm que poucas casas decimais facilitam as contas, em particular quando
no se usam mquinas de calcular. Seria impossvel, na prtica, trabalhar manualmente com
aproximao de 31 casas que demos para no incio desta conversa.
O tratamento de problemas de aproximao feito atravs de desigualdades;
infelizmente, tempo e espao no permitem que abordemos este tpico com mais detalhes no
momento, mas esperamos ter despertado sua curiosidade para o assunto.

Olimpada Brasileira de Matemtica das Escolas Pblicas

115

NVEL 3

OBMEP

4a Lista

1)Qual o maior fator primo de 2006 ?


2)Entre 1986 e 1989 , a moeda do nosso pas era o cruzado (Cz$). De l para c, tivemos o
cruzado novo, o cruzeiro, o cruzeiro novo e, hoje, temos o real. Para comparar valores
do tempo do cruzado e de hoje, os economistas calcularam que 1 real equivale a
2.750.000.000 cruzados.
Imagine que a moeda no tivesse mudado e que Joo, que ganha hoje 640 reais por ms,
tivesse que receber seu salrio em notas de 1 cruzado cada uma. Se uma pilha de
100 notas de 1 cruzado tem 1,5cm de altura, qual seria a altura do salrio do Joo?
A) 26, 4 km

B) 264 km

C) 26 400 km

D) 264 000 km

E) 2 640 000 km

3)H 1002 balas de banana e 1002 balas de ma numa caixa. Lara tira, sem olhar o sabor,
duas balas da caixa. Se q a probabilidade das duas balas serem de sabores diferentes e
p a probabilidade das duas balas serem do mesmo sabor, qual o valor de q p ?
A) 0

B) 1/2004

C) 1/2003

D) 2/2003

E) 1/1001

4)Um ponto P est no centro de um quadrado com 10 cm de lado. Quantos pontos da


borda do quadrado esto a uma distncia de 6 cm de P ?
A) 1

B) 2

C) 4

D) 6

E) 8

C) 1

D) 2

E) 3

5) Se 2 ( 22x ) =4 x +64 , ento x igual a:


A) 2

B) 1

6)Dois espelhos formam um ngulo de 30o no ponto V. Um raio de luz parte de um


ponto S paralelamente a um dos espelhos e refletido pelo outro espelho no ponto A,
como mostra a figura. Depois de uma certa quantidade de reflexes, o raio retorna a S.
Se AS e AV tm ambos 1 metro, qual o
comprimento em metros do trajeto percorrido
pelo raio de luz?

A) 2

116

B) 2 + 3

C) 1 + 2 + 3

D)

2 1+ 3

Olimpada Brasileira de Matemtica das Escolas Pblicas

E) 5 3

SOLUES

OBMEP

4a Lista

1. A decomposio de 2006 em fatores primos 2006 = 2 17 59 . Logo, o maior fator primo de


2006 59 .
2. (D) O enunciado diz que 1 real = 275 107 cruzados. O salrio de Joo 640 reais, o que
equivalente a 640 275 107 = 176.000 107 = 176 1010 cruzados. O nmero de pilhas de 100 notas
176 1010
= 176 108 . Como cada
102
uma destas pilhas tem altura 1,5 cm, a altura de todas elas 1,5 176 108 = 264 108 cm.

que se podem fazer com este nmero de notas de 1 cruzado

Lembramos
3

agora

1km = 1000 m = 10 m

que

1m = 100 cm = 10 cm ,

1km = 10 10 = 10 cm . Logo uma pilha de 264 108 cm tem

264 10
10

donde

= 264 10 = 264.000 km de

altura.
3. (C) A primeira bala pode ser de qualquer sabor; para fixar idias suponhamos que seja de
banana. Depois que esta bala retirada sobram 1002 + 1001 balas na caixa no nosso caso 1002 de
ma e 1001 de banana.
A probabilidade q de que a segunda bala seja diferente (no nosso exemplo, de ma) q =
A probabilidade p de que a segunda bala seja igual (no nosso exemplo, de banana) p =
A diferena q-p , portanto, q p =

1002
2003

1001
2003

1002 1001
1

=
.
2003 2003 2003

4. (E) Os pontos que esto a 6 cm de distncia do ponto P formam uma


circunferncia de centro P e raio R= 6 cm .Se D denota a diagonal do
quadrado,
2

do
2

D = 10 + 10 =

teorema

de

Pitgoras

temos

2 10 = 10 2

A circunferncia de raio L/2=5 tangencia o quadrado em 4 pontos.


A circunferncia de raio D/2 toca o quadrado em 4 pontos (os vrtices do quadrado).
Temos: L = 10 ; R = 6 e D = 10 2 ,

logo

5
L/2

< 6<5 2.
R

(Observe

que

1, 2 < 2

logo,

D/2

5 1, 2 < 5 2 e portanto, 6 < 5 2 )

Assim, a circunferncia de raio R=6 est entreas duas circunferncias de raios 5 e 5 2 .


Logo, ela corta o quadrado em 8 pontos.
5. (E) Soluo 1: Notamos que os termos do lado direito da equao dada podem ser escritos
como potncias de 2 ; de fato, 4 x = (22 ) x = 22 x e 64 = 26 . Desse modo, a equao se torna
2x

2(22 x ) = 22 x + 23 . Temos ento 2(22 x ) 22 x = 26 , donde 2 (2 1) = 2 , ou seja 2

2x

=2

. Logo

2 x = 6 e segue que x = 3 .

( )

Soluo 2: 2 4 x = 4 x + 43 4 x = 43 x = 3
Olimpada Brasileira de Matemtica das Escolas Pblicas

117

OBMEP

6. (B) Vamos acompanhar o trajeto do raio de luz a partir do ponto S. Para isso, lembramos a
propriedade bsica da reflexo de um raio de luz em um espelho: o ngulo de reflexo igual
ao ngulo de incidncia. Por exemplo, na figura ao lado, os ngulos a e b so iguais, bem como
d e e. Notamos que temos na figura as paralelas AS e BV cortadas pela transversal AB, da segue
que:

a= 30o = b,

a + b + c = 180o , donde c=120o.


c + d = 180o , donde d=60o=e.

Como a soma dos ngulos internos do tringulo BCV 180o, segue que f=90o. Isso quer dizer
que o nosso raio de luz, ao atingir C, ser refletido sobre si mesmo e far ento o caminho
inverso.
C B A S. Desse modo, o trajeto completo do raio ser S A B C B A S.

Desse modo, o comprimento do trajeto do raio at retornar a S duas vezes a soma dos
comprimentos dos segmentos AS, AB e BC. O enunciado nos diz que AS = 1 m . Falta calcular AB
e BC. Para isso, olhamos para o tringulo ABC. Ele um tringulo retngulo com ngulos de 30o
e 60o. Sabemos que em um tal tringulo o cateto oposto ao ngulo de 30o tem comprimento
igual metade do comprimento da hipotenusa (exerccio) no nosso caso, temos BC =

1
AB .
2

Notamos agora que os tringulos ABC e VBC so congruentes, pois so tringulos


retngulos (f=90o) com ngulos iguais (b=30o) e um cateto comum (BC), o que nos mostra que
1
1
m . Pondo AB = x temos BC = x , e o teorema de Pitgoras aplicado ao tringulo ABC
2
2
2
2
1
1
3
1
1
3

nos d x 2 = + x ; simplificando, obtemos x 2 = , donde x =


=
. Desse modo,
3
4
4
3
2 2
AC =

temos o comprimento do trajeto do raio de luz:

3 1 3
2( SA + AB + BC ) = 2 1 +
+
= 2 + 3 cm
3 2 3

118

Olimpada Brasileira de Matemtica das Escolas Pblicas

NVEL 3

OBMEP

5a Lista

NVEL
1) Determine o valor de ( 666 666 666 )2 - ( 333 333 333)2 .
2) Na figura, o nmero 8 foi obtido somando-se os dois nmeros diretamente abaixo
de sua casa. Fazendo-se o mesmo para preencher as casas em branco, obtm-se o 42
na casa indicada. Qual o valor de x?
42

8
3

A) 7

B) 3

x
C) 5

D) 4

E) 6

3) Seja n = 9867 . Se voc calculasse n3 n 2 , encontraria um nmero cujo algarismo das


unidades :
A) 0

B) 2

C) 4

D) 6

E) 8

4) O grfico da parbola y = x 2 - 5x + 9 rodado de 180 em torno da origem. Qual a


equao da nova parbola?
A) y = x 2 + 5 x + 9

B) y = x 2 - 5 x - 9

C) y = -x 2 + 5 x - 9 D) y = -x 2 - 5 x + 9

E) y = -x 2 - 5 x - 9

5) A figura mostra a marca de uma empresa, formada por dois


crculos concntricos e outros quatro crculos de mesmo raio, cada
um deles tangente a dois dos outros e aos dois crculos
concntricos. O raio do crculo menor mede 1 cm. Qual , em
centmetros, o raio do crculo maior?

6) Um padeiro quer gastar toda sua farinha para fazer pes. Trabalhando sozinho, ele
conseguiria acabar com a farinha em 6 horas; com um ajudante, o mesmo poderia ser
feito em 2 horas. O padeiro comeou a trabalhar sozinho; depois de algum tempo,
cansado, ele chamou seu ajudante e assim, aps 150 minutos a farinha acabou. Quanto
tempo o padeiro trabalhou sozinho?

Olimpada Brasileira de Matemtica das Escolas Pblicas

119

OBMEP

7) Manoel testou sua pontaria lanando cinco flechas no alvo


reticulado de quadrados de comprimento 1 cm , ilustrado na
figura. Uma flecha que acerta dentro do crculo menor conta
300 pontos; na regio sombreada conta 100 pontos, entre a
regio sombreada e o crculo maior conta 50 pontos e fora do
crculo maior no conta nada. As flechas de Manoel acertaram
5
os pontos
A = (1, -1), B = ,1 , C = (1, - 4), D = (-4 , - 4) e
2
E = (6 , 5) .

(a) Marque na figura os pontos onde Manoel acertou suas flechas.


(b) Quantas flechas ele acertou no interior do menor crculo?
(c) Quantos pontos Manoel fez no total?

8) A festa de aniversrio de Andr tem menos do que 120 convidados. Para o jantar, ele
pode dividir os convidados em mesas completas de 6 pessoas ou em mesas completas
de 7 pessoas. Nos dois casos so necessrias mais do que 10 mesas e todos os
convidados ficam em alguma mesa. Quantos so os convidados?

9) (a) Calcule o nmero de diagonais do prisma hexagonal reto representado na figura


1.
(b) Calcule o nmero de diagonais do prisma representado na figura 2 .
Este poliedro muito utilizado na fabricao de dados, e obtido realizando-se oito
cortes em um cubo, cada corte prximo a um dos seus 8 vrtices (isso arredonda o
dado e facilita a sua rolagem).

Figura 1

120

Figura 2

Olimpada Brasileira de Matemtica das Escolas Pblicas

SOLUES

OBMEP

5a Lista

1. Usando a fatorao x 2 - y 2 = ( x - y)(x + y ) , obtemos :


= (666.666.666 333.333.333)(666.666.666 + 333.333.333)
= 333.333.333 999.999.999
= 333.333.333 (1.000.000000 1)
= 333.333.333.000.000.000 333.333.333
= 333.333.332.666.666.667

666.666.666 2 333.333.3332

2. (E) Usando a regra dada no enunciado, preenchemos as casas vazias a partir da segunda
linha a contar de baixo, obtemos:
42

13 + x

11+ 2 x

5+ x

8
3

x+6

Logo, (13 + x ) + (11 + 2 x ) = 42 . Assim, 24 + 3 x = 42 . Donde x = 6 .


3. (C) Soluo 1: O algarismo final de 98673 o mesmo que o de 73 = 343 , isto , 3 ; o algarismo
final de 9867 2 o mesmo que o de 7 2 = 49 , isto , 9 . Se de um nmero terminado em 3
subtramos outro terminado em 9 , o algarismo final do resultado 4 .
Comentrio: Observe que:
3

algarismo das unidades de (9867 - 9867 ) = algarismo das unidades de (7 - 7 )

Soluo 2: n3 n 2 = n 2 ( n 1) . Assim, n 2 = ( 9867 ) termina em 9 e n 1 = 9866 em 6.


2

Como, 9 6 = 54 , o algarismo final do resultado 4.


4. (E) Uma rotao de 180o tambm conhecida como
meia-volta. Neste problema, temos uma meia-volta em
torno da origem. O desenho ao lado ilustra o que esta
meia-volta faz com as coordenadas dos pontos do plano.
Por exemplo, o ponto A o resultado da meia-volta
aplicada ao ponto A; em outras palavras, A onde o
ponto A vai parar aps a meia-volta. Do mesmo modo,
B onde B vai parar aps a meia volta. fcil ver que na
passagem de A para A as coordenadas trocam de sinal.
Deste modo, vemos que uma meia-volta em torno da
origem leva um ponto qualquer ( x, y ) no ponto ( x, y ) .
Temos:

( a , b)

pertence

nova

parbola

y = x - 5x + 9 -b = a + 5a + 9 b = -a - 5a - 9 .
2

Logo

(-a , - b)

pertence

da

equao

nova

parbola

parbola

y = -x - 5x - 9 .

Olimpada Brasileira de Matemtica das Escolas Pblicas

121

OBMEP
5. Seja r o raio das quatro circunferncias iguais. Ligando os
centros A e B de duas destas circunferncias ao centro O das
circunferncias concntricas, obtemos o tringulo OAB como na
figura ao lado. Lembrando que a reta que une os centros de duas
circunferncias tangentes passa pelo ponto de tangncia, vemos
que OA = OB = 1 + r e AB = 2r . Lembrando tambm, que o tringulo
OAB retngulo em O, o teorema de Pitgoras nos diz ento que
(2r ) 2 = (1 + r ) 2 + (1 + r ) 2 , ou seja, 4r 2 = 2r 2 + 4r + 2 . Logo r 2 2r 1 = 0 ;
daqui tiramos

r=

22 2
= 1 2 .
2

Como

1 2

negativo,

descartamos esta raiz e obtemos r = 1 + 2 . Segue que o raio da circunferncia maior


1 + 2r = 3 + 2 2 .
6. (D) Seja x a quantidade de farinha, em quilos, de que o padeiro dispe. Trabalhando sozinho,
ele usaria

quilos de farinha em 1 hora; trabalhando com seu ajudante, eles usariam

quilos de farinha em 1 hora. Seja t o tempo, em horas, que o padeiro trabalhou sozinho.
Como a farinha acaba em 150 minutos ( 2 horas e 30 minutos = 2, 5 horas), o tempo que ele
trabalhou com seu ajudante foi 2, 5 - t horas. Logo, a quantidade gasta de farinha durante o
tempo que o padeiro trabalhou sozinho
padeiro trabalhou com seu ajudante
quantidade total
defarinha
x

temos x =

x
6

t+

x
2

x
6

t , e a quantidade gasta durante o tempo que o

(2, 5 - t ) . Como

quantidade farinha gasta pelo


padeiro trabalhando sozinho
x t
6

quantidade farinha gasta


+ pelo padeiro trabalhando
com o ajudante
x (2,5- t)
2

(2, 5 - t ) . A quantidade de farinha que o padeiro tinha inicialmente era no

nula, isto x 0 . Logo, podemos dividir ambos os membros por x e encontramos 1 =

2, 5 - t
2

portanto, t = 0, 75 horas = 0, 75 60 minutos = 45 minutos .

7. (a)Marcamos os pontos, conforme mostra a figura:


(b) No crculo menor temos apenas o ponto A. Portanto,
Manoel acertou apenas uma vez neste crculo, o que lhe d
300 pontos.
(c) Para calcular o total de pontos, observe que no ponto B
ele ganha 100 pontos, no C ganha 50 pontos e no D ganha 50
pontos. J no ponto no E, ele no ganha pontos, porque este
est fora do alvo. Logo, o nmero total de pontos foi
de 300 + 100 + 50 + 50 = 500 pontos.

122

Olimpada Brasileira de Matemtica das Escolas Pblicas

OBMEP
8. Como podemos repartir o total de convidados em mesas de 6 ou 7 , o nmero de convidados
um mltiplo de 6 e de 7 . Como o menor mltiplo comum de 6 e 7 42 , podemos ter
42, 84, 126, convidados. Como so menos do que 120 convidados, s podemos ter 42 ou
84 convidados. Por outro lado, como so necessrias mais do que 10 mesas, temos mais do
que 60 convidados. Logo, descartamos o 42 , e o nmero de convidados s pode ser 84 .

9. Em um poliedro qualquer, dois vrtices distintos determinam uma diagonal se eles estiverem
em faces distintas.

(a) No caso do prisma hexagonal, vemos na figura que o vrtice


v no forma uma diagonal com os vrtices marcados com *;
levando o prprio v em conta, vemos que v no forma uma
diagonal com exatamente 9 vrtices. Como o prisma tem
12 vrtices, segue que v forma uma diagonal com exatamente
12 9 = 3 vrtices. O mesmo raciocnio vale para qualquer
vrtice, e conclumos que de cada vrtice do prisma partem
exatamente 3 diagonais. Como a diagonal que parte de um
vrtice v para o vrtice w a mesma que parte de w para v,
segue que o nmero de diagonais

12 3
2

= 18

(b) Seja V um vrtice do poliedro. Observando a figura vemos que


V no forma uma diagonal com exatamente 14 vrtices:
13 marcados com X e mais o prprio V. Como o poliedro tem
24 vrtices no total, sobram 24 14 = 10 vrtices com os quais V
forma uma diagonal. Logo, o nmero de diagonais deste poliedro

2410
= 120 .
2

Olimpada Brasileira de Matemtica das Escolas Pblicas

123

NVEL 3

OBMEP

6a Lista

1) Uma loja de sabonetes realiza uma promoo com o anncio Compre um e leve outro
pela metade do preo. Outra promoo que a loja poderia fazer oferecendo o mesmo
desconto percentual :
A) Leve dois e pague um
C) Leve trs e pague dois
E) Leve cinco e pague quatro

B) Leve trs e pague um


D) Leve quatro e pague trs

2) Na figura, os dois tringulos ABC e FDE so eqilteros. Qual o valor do ngulo x?

A) 30

B) 40

C) 50

D) 60

E) 70

3) O desenho mostra um pedao de papelo que ser dobrado e colado ao longo das
bordas para formar uma caixa retangular. Os ngulos nos cantos do papelo so todos
retos. Qual ser o volume da caixa em cm3 ?

40 cm

20 cm

15 cm

A) 1 500

B) 3 000

C) 4 500

D) 6 000

E) 12 000

4) Numa seqncia, cada termo, a partir do terceiro, a soma dos dois termos
imediatamente anteriores, o segundo termo 1 e o quinto termo 2005 . Qual o sexto
termo?
A) 3 002

124

B) 3 008

C) 3 010

D) 4 002

Olimpada Brasileira de Matemtica das Escolas Pblicas

E) 5 004

OBMEP

5) Quantos nmeros entre 10 e 13 000 , quando lidos da esquerda para a direita, so


formados por algarismos consecutivos e em ordem crescente? Por exemplo, 456
um desses nmeros, mas 7 890 no .
A) 10

B) 13

C) 18

D) 22

E) 25

6) Num bloco de 1cm 2 cm 3 m ,marcamos trs faces com as letras X, Y e Z como na


figura. O bloco colocado sobre um tabuleiro de 8 cm 8 cm com a face X virada para
baixo (em contato com o tabuleiro) conforme mostra a figura. Giramos o bloco de
90o em torno de uma de suas arestas de modo que a face Y fique virada para baixo
(isto , totalmente em contato com o tabuleiro). Em seguida, giramos novamente o
bloco de 90 em torno de uma de suas arestas, mas desta
vez de modo que a face Z fique virada para baixo.
Z
Giramos o bloco mais trs vezes de 90 em torno de uma de
Y
suas arestas, fazendo com que as faces X, Y e Z fiquem
viradas para baixo, nessa ordem. Quantos quadradinhos
diferentes do tabuleiro estiveram em contato com o bloco?

A) 18

B) 19

C) 20

D) 21

E) 22

7) A funo f dada pela tabela a seguir.


x 1 2 3 4 5
f(x) 4 1 3 5 2

Por exemplo, f(2) = 1 e f (4) = 5 . Quanto vale f ( f ( f ( f ( f (4) )))) ?


2004 vezes

A) 1

B) 2

C) 3

D) 4

E) 5

8) Esmeralda escreveu em ordem crescente todos os nmeros de 1 a 999 , sem separlos, formando o nmero mostrado a seguir: 12345678910111213 997998999. Nesse
nmero, quantas vezes aparece o agrupamento 21, nesta ordem?

Olimpada Brasileira de Matemtica das Escolas Pblicas

125

SOLUES

OBMEP

6a Lista

1.
(D) Pela promoo, quem levar 2 unidades paga pelo preo de 1, 5 unidade, logo
quem levar 4 unidades paga pelo preo de 3 unidades, ou seja, leva quatro e paga trs.
2.
(B) Como ABC e DEF so tringulos eqilteros, cada um de seus ngulos
internos mede 60 o. No tringulo AGD temos
GD = 180 - 75 - 60 = 45 e
= 180 - 65 - 60 = 55
GDA

= 180 - 45 - 55 = 80 .Logo no tringulo CGH temos x + 80 + 60 = 180 ,


Portanto, AGD
donde x = 40o .

3.
(B) A figura mostra as dobras que sero feitas para montar a caixa. A caixa ter
dimenses 20 cm de largura, 15cm de comprimento e 10cm de altura. Logo, seu volume
ser igual a 20 15 10 = 3000 cm3 .

20 cm

40 cm

10 cm

15 cm

4.
(B) Seja x o primeiro termo. Como o segundo termo 1 e, a partir do terceiro,
cada termo a soma dos dois anteriores, temos:
terceiro termo: 1 + x ;
quarto termo: 1 + (1 + x) = 2 + x ;
quinto termo: (1 + x) + (2 + x) = 3 + 2x ;
sexto termo: (2 + x) + (3 + 2x) = 5 + 3x .
Como o quinto termo 2005, temos 3 + 2x = 2005 , donde x = 1001 ; logo o sexto termo
5 + 3 1001 = 3008 .

126

Olimpada Brasileira de Matemtica das Escolas Pblicas

OBMEP

5.

(D) Os nmeros em questo so:


com 2 algarismos: 12, 23, 34, 45, , 89 (8 nmeros),
com 3 algarismos: 123, 234, 345, , 789 (7 nmeros) ,
com 4 algarismos: 1234, 2345, , 6789 (6 nmeros)
e, por fim,
com 5 algarismos: 12345 , um total de 8 + 7 + 6 + 1 = 22 nmeros .

6.
(B) Note que giramos o bloco 5 vezes. Indicaremos os movimentos feitos pelo
bloco e as faces que entram em contato com os quadradinhos em cada etapa.
De acordo com a figura dada, podemos concluir que as dimenses das faces X, Y e Z
so:

X=12

Y=13

Z=23

As figuras a seguir mostram os quadradinhos do tabuleiro que ficam em contato com


cada um das 3 faces do bloco desde a posio inicial at a final, aps a ltima rotao.

X
X

Y
Y
Y
a
1 Rotao

Posio inicial

X X

Z Z
Z Z
Z Z
a
2 Rotao

Y Y Y
Z Z Z
Z Z Z

3a Rotao

4a Rotao
Olimpada Brasileira de Matemtica das Escolas Pblicas

5a Rotao
127

OBMEP

Alguns quadradinhos entram em contato com as faces mais de uma vez, como mostra a
figura a seguir, que mostra todos os quadradinhos que tiveram contato com as faces do
bloco desde a posio inicial at a ltima rotao:

Y
Z
Z

Y
Z
Z

X
X/Y
Y/Z
Y/Z
Y

X
Z
Z
Z

X
Z
Z
Z

Contando nesta ltima figura, vemos que o bloco esteve em contato com 19
quadradinhos do tabuleiro.
7.

(D) Da tabela temos: Da segue:

f (4) = 5

f ( f (4)) = f (5) = 2

f ( f ( f (4))) = f ( f (5)) = f (2) = 1 e

f ( f ( f ( f (4)))) = f ( f ( f (5))) = f ( f (2)) = f (1) = 4 Logo, f ( f ( f ( f (4) = 4 .


5

4 vezes

Como 2004 mltiplo de 4, segue que f ( f ( f ( f ( f (4) )))) = 4 . O diagrama a seguir


2004 vezes

ilustra esta afirmao.


A seqncia a seguir ilustra esta composio.
f

4 5 2 1 4 5 2 1 4 5 2 1 4 5 2 1 4
4 vezes

8 vezes

12 vezes

2004 vezes

8.
Vamos primeiro listar os nmeros que tm o agrupamento 21 no meio de sua
representao decimal:
10 nmeros
210, 211, ,219 10 nmeros
21, 121, 221, ,921

Temos tambm que contar os agrupamentos 21 obtidos a partir de um par de nmeros


consecutivos tal que o primeiro termina com 2 e o segundo comea com 1 , que so os
seguintes 11 casos:
12-13, 102-103, 112-113, 122-123, 132-133, 142-143, 152-153, 162-163, 172-173, 182-183, 192-193

Temos ento um total de 11 + 20 = 31 nmeros.

128

Olimpada Brasileira de Matemtica das Escolas Pblicas

NVEL 3

OBMEP

7a Lista

1
1
1 1
1) Encontre o produto: 1 1 1 1
.
4 9 16 225
(A)

10
125

(B)

5
9

(C)

3
5

(D)

8
15

(E)

1
120

2) Se dois lados de um tringulo medem 5 cm e 7 cm , ento o terceiro lado no pode


medir:
(A) 11 cm
(B) 10 cm
(C) 6 cm
(D) 3 cm
(E) 1 cm
3) Quais os valores de x que satisfazem

(A) x >

3
4

(B) x > 2

(C)

1
<4?
x2

3
<x<2
4

(D) x < 2

(E)todos os valores de x.

4) Quanto mede o ngulo da figura?


(A) 20

(B) 25

(C) 30

30o

(D) 35 (E) 40
50o

40o

5) Da figura, conclumos que | z x | + | w x | igual a :

(A) 11

(B) 12

(C) 13

(D) 14

(E) 15

6) Artur quer desenhar uma espiral de 4 metros de


comprimento formada de segmentos de reta. Ele j traou 7
segmentos, como mostra a figura. Quantos segmentos ainda
faltam traar?
(A) 28

(B) 30

(C) 24

(D) 32

(E) 36

7) A figura mostra um retngulo e suas duas diagonais. Qual


a afirmativa correta a respeito dos ngulos x e y da figura?
(A) x < y

(B) x = y

(C) 2 x = 3 y

(D) x = 2 y

(E) x = 3 y

Olimpada Brasileira de Matemtica das Escolas Pblicas

129

SOLUES

OBMEP

7a Lista

1. (D) Cada um dos fatores da forma diferena de quadrados isto a 2 b 2 , onde


a =1

1
1
1
1
1
1
1 1
1 1 1 1
= 1 2 1 2 1 2 1 2
4 9 16 225 2 3 4 15
Usando

a 2 b 2 = (a b)(a + b) ,

fatorao

obtemos:

1
1 1 1 1 1 1 1
1
1
1 1
1 1 1 1
= 1 1 + 1 1 + 1 1 + 1 1 + =
4 9 16 225 2 2 3 3 4 4 15 15
1 3 2 4 3 5 4
15 14 16 1 16 8
= = = .
2 2 3 3 4 4 5
14 15 15 2 15 15
1

2. (E)Lembre que num tringulo a soma de dois lados quaisquer tem que ser maior que
o terceiro lado. Como 1 + 5 no maior do que 7 , o terceiro lado no pode ser 1 .

1
1 4( x 2)
3 4x
1
<4
4<0
<0
< 0. Para que uma frao
x2
x2
x2
x2
seja negativa, o numerador e o denominador tm que ter sinais trocados.

3.(C)Temos:

1o caso: 3 4 x > 0 e x 2 < 0 .


3
3 4x > 0 x <
e x 2 > 0 x > 2 , o que impossvel.
4
2o caso: 3 4 x < 0 e x 2 > 0 .
3
3
3 4x < 0 x >
e x 2 < 0 x < 2 . Logo, a resposta < x < 2 .
4
4

3/4

4. (A) Os ngulos internos do quadriltero na figura so: 50 , ,180 40 e 180 30 .


140
150
o
Como, a soma dos ngulos internos de um quadriltero 360 , temos:
30o

50o + + 140o + 150o = 360o = 20o


50o

150o
140o

5. (E) Temos: | z x | = 3, 7 (1) = 4, 7 e | w x | = 9,3 (1) = 10,3 .


distncia
de x a z

distncia
de x a w

Logo, | z x | + | w x |= 4, 7 + 10,3 = 15 .
130

Olimpada Brasileira de Matemtica das Escolas Pblicas

40o

OBMEP

6. (D)A figura mostra que a espiral formada de segmentos cujos comprimentos


formam uma sequncia finita da forma 1, 1, 2, 2, 3, 3, 4, 4, , n, n (se os dois ltimos
segmentos da espiral tm o mesmo comprimento) ou da forma
1, 1, 2, 2, 3, 3, 4, 4, , n, n, n+1 (se os dois ltimos segmentos da espiral tm
comprimentos diferentes). Como o comprimento total 4 m = 400 cm , devemos ter:

1 + 1 + 2 + 2 + 3 + 3 + + n + n = 400 2 (1 + 2 + 3 + + n) = 400

ou
1 + 1 + 2 + 2 + 3 + 3 + + n + n + n + 1 = 400 2 (1 + 2 + 3 + + n) + n + 1 = 400

A soma dos n primeiros nmeros naturais 1 + 2 + 3 + + n =

n(n + 1)
, logo temos:
2

n(n + 1)
= 400 n(n + 1) = 400
2 2

ou
n(n + 1)
2
+ n + 1 = 400 n(n + 1) + n + 1 = 400 (n + 1) 2 = 400
2

No existem dois nmeros naturais consecutivos cujo produto seja 400 , logo, a equao
n(n + 1) = 400 no tem soluo.De (n + 1)2 = 400 segue que n + 1 = 20 . Portanto, o ltimo
segmento da espiral tem 20 cm e o penltimo 19 cm . Os comprimentos dos segmentos
da espiral formam a seqncia de nmeros 1,1, 2, 2,3,3, 4, 4, ,19,19, 20.
Portanto, so 19 2 + 1 = 39 segmentos. Como 7 j foram traados, faltam 32 .
7. (D) Seja O o ponto de interseo das duas diagonais do retngulo. Ento AO=BO,
portanto o tringulo AOB issceles e logo

Como a soma dos ngulos internos de um


tringulo 180o, no tringulo AOB temos:

2 y + 180 o x = 180 o x = 2 y .

Olimpada Brasileira de Matemtica das Escolas Pblicas

131

NVEL 3

OBMEP

8a Lista

1) Qual a menor das razes da equao 2 (x 3 5 )(x 5 3) = 0 ?

2)Quantas solues inteiras


2000 < n(n + 1) < 2005 ?
(A) 1

positivas

(B) 2

satisfazem

(C) 3

dupla

(D) 4

inequao

(E) 5

3) Seja v a soma das reas das regies pertencentes


unicamente aos trs discos pequenos (em cinza claro), e
seja w a rea da regio interior unicamente ao maior disco
(em cinza escuro). Os dimetros dos crculos so 6, 4, 4 e 2.
Qual das igualdades abaixo verdadeira?

4) A menor raiz da equao

(A)

1
3

(A) 3v = w

(B) 3v = 2 w

(D) v = 3w

(E) v = w

(C) v = w

| x 1|
= 6 :
x2

(B)

1
2

(C)

1
3

(D)

1
4

(E)

3
2

5) Uma mesa quadrada tem 1 metro de lado. Qual o menor dimetro de uma toalha
redonda que cubra completamente o tambo da mesa?
(A) 1

132

(B) 1,5

(C) 2

(D) 2

Olimpada Brasileira de Matemtica das Escolas Pblicas

(E) 3

SOLUES

OBMEP

8a Lista

1. Soluo 1: A equao j dada na forma fatorada a(x m)(x n) = 0 , logo as razes


so m = 3 5

e n = 5 3 . Devemos decidir qual delas a maior. Sabemos que

5 < 2, 3 e 1, 7 < 3 , logo 3 5 < 3 2, 3 e 5 1, 7 < 5 3 3 5 < 6, 9 e 8, 5 < 5 3 . Como


6,9 menor do que 8,5 , conclumos que 3 5 menor do que 5 3 .

6,9

3 5

8,5

5 3

) (
2

Soluo 2: Comparar 3 5 e 5 3 o mesmo que comparar 3 5 e 5 3 . Assim,

(3 5 )

= 9 5 = 45 < 75 = 25 3 = 5 3 . Logo, 3 5 menor do que 5 3 .

2. (E )Como os nmeros que aparecem so todos positivos, podemos elev-los ao


quadrado mantendo os sinais, isto : 2000 2 < n(n + 1) < 2005 2 . Observe que n e n + 1 so
inteiros consecutivos. Logo, temos as seguintes opes:

2000 2 < 2000 2001 < 2005 2


2000 2 < 2001 2002 < 2005 2
2000 2 < 2002 2003 < 2005 2
2000 2 < 2003 2004 < 2005 2
2000 2 < 2004 2005 < 2005 2
Logo, temos 5 possibilidades para n: 2000, 2001, 2002, 2003 e 2004 .

3. (C) Os raios dos trs discos menores so 1,2 e 2; e do disco maior 3.


Denotemos por b a rea em branco, temos:

v=

rea do disco

de raio 3

w = 9 b . Logo, v=w.

A rea do
disco de
raio
r r2

Olimpada Brasileira de Matemtica das Escolas Pblicas

133

OBMEP

4. (B) 1o caso: x 1
Nesse caso, x 1 0, donde | x 1 | = x 1 . A equao toma a forma

x 1
= 6 ou
x2

6x 2 x + 1 = 0 . Essa equao no tem razes reais porque


= (1)2 4 6 1 = 1 24 negativo.
2o caso: x < 1
Nesse caso, x 1 < 0, donde | x 1 | = ( x 1) = 1 x . A equao toma a forma
1 x
= 6 ou 6x 2 + x 1 = 0 . Resolvendo essa equao temos:
x2
1 1 4 6 (1) 1 25 1 5
1
1
x=
=
=
x = e x = . Como essas duas
26
12
12
2
3
razes so menores que 1 , elas so as razes da equao do enunciado. A menor
1
delas x = .
2

5. (D) Para que a toalha cubra inteiramente a mesa e que tenha o menor dimetro
possvel, o quadrado deve estar inscrito no crculo. A diagonal do quadrado o
dimetro do crculo, logo pelo Teorema de Pitgoras, temos:
d 2 = 12 + 12 d = 2

134

Olimpada Brasileira de Matemtica das Escolas Pblicas

NVEL 3

OBMEP

9a Lista

1) Os valores positivos de x para os quais ( x 1)( x 2)( x 3) < 0 formam o conjunto:


(1 , 3) (2 , 3)
(0 , 3)

(0,1) (2 , 3)

(1 , 2)

2) Num tringulo retngulo, definimos o cosseno de seus


cateto adjacente c
ngulos agudos por: cos =
= .
hipotenusa
a

B
N

O tringulo retngulo da figura tem cateto OA=1. Escreva


em ordem crescente os cossenos dos ngulos de 25o, 41o e 58o

17o
16o

25o

3) Os ramais de uma central telefnica tm apenas 2 algarismos, de 00 a 99 . Nem todos


os ramais esto em uso. Trocando a ordem de dois algarismos de um ramal em uso, ou
se obtm o mesmo nmero ou um nmero de um ramal que no est em uso. O maior
nmero possvel de ramais em uso :
(A)Menos que 45

(B)45

(C)entre 45 e 55

(D) mais que 55

(E)55

4) Um nibus, um trem e um avio partem no mesmo horrio da cidade A para a


cidade B. Se eu tomar o nibus cuja velocidade mdia 100 km / h , chegarei cidade B
s 20 horas. Se eu tomar o trem, cuja velocidade mdia 300 km / h , chegarei cidade B
s 14 horas. Qual ser o horrio de chegada do avio se sua velocidade mdia de
900 km / h ?

5) Na figura O o centro
Qual o dimetro desse crculo?

do

crculo

AB=

5 cm

6) Iara possui R$ 50, 00 para comprar copos que custam R$ 2,50 e pratos que custam
R$ 7, 00 . Ela quer comprar no mnimo 4 pratos e 6 copos. O que ela pode comprar ?
Olimpada Brasileira de Matemtica das Escolas Pblicas

135

OBMEP

SOLUES

9a Lista

1. Para que um produto de trs fatores seja negativo, devemos ter


dois fatores positivos e um fator negativo, ou os trs negativos.

As possibilidades so:
1) ( x 1) ( x 2) ( x 3) x > 1 , x > 2 e x < 3.
+

Nesse caso, a soluo 2 < x < 3 .

2) ( x 1) ( x 2) ( x 3) x > 1 , x < 2 e x > 3.


+

Nesse caso temos 1 < x < 2 e x>3, o que


no possvel. Logo, esse caso no pode
ocorrer.

3) ( x 1) ( x 2) ( x 3) x < 1 , x > 2 e x > 3.

Nesse caso temos x < 1 , x > 2 e x > 3 , o que


no possvel.

4) ( x 1) ( x 2) ( x 3) x < 1 , x < 2 e x < 3.

Nesse caso, a soluo x<1. Logo, a soluo so todos os nmeros reais x tais que x<1 ou 2<x<3;
ou seja, a unio de dois intervalos: (0,1) (2 , 3)

1
1
1
, cos 41o =
e cos 58 o =
.
OM
ON
OB
Na figura, vemos que OM < ON < OB , logo cos 58 o < cos 41o < cos 25 o .
2. De acordo com a definio de cosseno, temos: cos 25 o =

136

Olimpada Brasileira de Matemtica das Escolas Pblicas

OBMEP

3. (E) Existem dois tipos de ramais:


(i)
os dois algarismos so iguais ( 00, 11, 22, 33, 44, 55, 66, 77, 88, e 99) , esses so em
(ii)

nmero de 10
os dois algarismos so distintos, nesse caso temos 10 9 = 90 nmeros, e metade deles
podem ser usados.

Logo, temos no mximo 10 + 45 = 55 .

4. Seja d a distncia entre as duas cidades e h o horrio de partida comum do nibus, do trem e
do avio. Como, distncia = velocidade tempo , temos:

d = 100 (20 h) e d = 300 (14 h ) . Logo, 100 (20 h ) = 300 (14 h ) donde h = 11.
Portanto, a distncia entre as duas cidades d = 100 (20 11) = 900km . Logo, o avio gasta 1
hora da cidade A cidade B, e portanto ele chega s 12 horas.
5. Observe que OC um raio do crculo. Temos que OC=AB= 5 cm por serem as diagonais do
retngulo OACB. Logo, o dimetro 10 cm .

6. Sejam c e p o nmero de copos e pratos que Iara pode comprar. Logo seu gasto 2, 5 c + 7 p .
Ela s tem R$ 50, 00 , logo 2, 5 c + 7 p 50 (I) Alm disso, ela quer comprar no mnimo 4 pratos
e 6 copos, logo p 4 e c 6 (II). Devemos encontrar dois nmeros inteiros c e p (nmero de
copos e pratos so nmeros inteiros) que satisfaam (I) e (II).
Se ela comprar 4 pratos sobram 50 4 7 = 22 reais para os copos. Como 22 = 8 2, 50 + 2 ,
ela pode comprar 8 copos (sobrando-lhe R$ 2, 00 ).
Se ela comprar 5 pratos sobram 50 5 7 = 15 reais para os copos. Como 15 = 6 2,50 , ela
pode comprar 6 copos.
Se ela comprar 6 pratos sobram 50 6 7 = 8 reais para os copos, o que lhe permite compar
apenas 1 copo que no o que ela quer.

Logo, Iara pode comprar 4 pratos e 8 copos, ou 5 pratos e 6 copos.

Olimpada Brasileira de Matemtica das Escolas Pblicas

137

NVEL 3

OBMEP

10a Lista

1) Para fabricar 9 discos de papelo circulares para o Carnaval usam-se folhas


quadradas de 10 cm de lado como indicado na figura. Qual a rea do papel no
aproveitado?
(A) 25 cm 2
(B) 22,5cm 2
(C) 21,5 cm 2
(D) 21 cm 2
(E) 22 cm 2

2) Determine quais afirmaes so verdadeiras:


(A) | 108 |> 100
(D) | a 2 + 5 |= a 2 + 5
3) Se

(B) | 5 13 |=| 5 | | 13 |
(E) | 6 a |= 6 | a |

(C) | 2 9 |= 9 2

x+y
x
igual a:
= 5 ento
2y
y
(B) 3 2

(A) 5 2

(C) 13 y

(D) 25 y 2

4) A figura mostra um retngulo KGST e um tringulo KGR.


Os ngulos KRT e RGS so iguais. Se TR=6 e RS=2 qual a
rea de KGR?
(A) 12

(C) 8 2

(B)16

(D) 8 3

(E)14

(E) 13
R

5) Sinal de um produto e sinal de um quociente: a, b, c e d so quatro nmeros no nulos


a
18
b
11
tais que os quocientes
,
,
,
so positivos. Determine os sinais de
5
7a
abc
abcd
a, b, c e d.
K

6) No tringulo KLM temos KL=KM, KT=KS e


LKS= 30 . Qual a medida do ngulo TSM?
T

x
L

138

(A)10
(B) 15
(C) 20
(D) 25
(E) 30

Olimpada Brasileira de Matemtica das Escolas Pblicas

SOLUES

OBMEP

10a Lista

1. Lembre que a rea de um crculo r2, onde r o raio do crculo.


Se r o raio dos crculos da figura, ento a rea pedida :

A rea do
crculo de
raio
r r2

5
rea do quadrado - rea dos 9 crculos = 100 9 = 100 25
3
1010=100

9 r 2

Usando a aproximao 3,14 , obtemos 100 25 100 25 3,14 = 21, 5 cm 2 .

2. (A) | 108 |= 108 > 100 , verdadeira


(B) | 5 13 |=| 8 |= 8 e | 5 | | 13 |= 1 13 = 8 , falsa.
(C) | 2 9 |= (2 9) = 9 2 porque 2 9 < 0 , verdadeira.
(D) | a 2 + 5 |= a 2 + 5 porque a 2 + 5 > 0 para qualquer valor de a, verdadeira.
(E) | 6 a |=| 6 | | a |= 6 | a | , verdadeira.

3.(E) Elevando ao quadrado ambos os membros de

x
= 5 , obtemos
y

x
= 25 . Agora,
y

x + y 1 x + y 1 x y 1 x 1
=
= + = + 1 = (25 + 1) = 13 .
2y
y
2
2 y y 2 y 2

4.(D)Os tringulos TKR e GRS so proporcionais por serem tringulos retngulos com um
ngulo agudo igual. Logo, temos:

RS GS
. Como GS=TK segue que
=
TK TR

TK 2 = RS TR = 2 6 = 12 TK = 2 3 . A rea do tringulo KGR vale


base

altura

KG TK (TR + RS ) 2 3 8 2 3
=
=
=8 3
2
2
2

5. Soluo:

a
>0a >0
5
+

Temos a > 0 7a > 0 , logo:

b
> 0 b > 0 b < 0
7a
+

11
> 0 abc > 0 . Como a > 0 e b < 0 segue que c < 0 (a b c > 0)
abc
+

18
> 0 abcd < 0 , como abc > 0 segue que d < 0 .
abcd
Olimpada Brasileira de Matemtica das Escolas Pblicas

139

OBMEP

6. (B) Sejam
. O tringulo KLM issceles porque tem dois lados
iguais; consequentemente seus ngulos da base so iguais, isto :
. Analogamente,
o tringulo KST tambm issceles e portanto
. Usaremos agora que a soma dos
ngulos internos de um tringulo 180o. Acompanhe na figura:

No tringulo STM temos: x + + 180o = 180o

x =
No tringulo KLM temos: + + 30 + y = 180 y = 150o 2 .
o

Logo,

+ + 150o 2 = 180o = 15o . Portanto, x = 15o .

140

Olimpada Brasileira de Matemtica das Escolas Pblicas

NVEL 3

OBMEP

11a Lista

1) Quantos so os pares diferentes de inteiros positivos (a, b) tais que a + b 100 e


1
a+
b = 13 ?
1
+b
a
(A) 1
(B) 5
(C) 7
(D) 9
(E) 13
2) Se x+ | x | +y = 5 e x+ | y | y = 6 ento x + y :
(A) 1

(B) 11

(D) 1

(C) 9 5

(E) 11

3) Na figura, os trs crculos so concntricos, e as reas do menor crculo e do maior


anel (em cinza) so iguais. O raio do menor crculo 5 cm e do maior 13 cm . Qual o raio
do crculo intermedirio?
(A) 12
(B) 11
(C) 10 65
(D) 5 3
(E) 12 2

4) Encontre os algarismos que esto faltando sobre cada um dos traos:

(a)

126 21
=
8_ __

; (b)

_ _8 4
=
33_ 5

5) Uma a mais! Na lista de fraes, no quadro ao lado, temos:

2 fraes cuja soma

5
2

5
2 fraes cuja diferena
2
5
2 fraes cujo produto
2
5
2 fraes cujo quociente
2

5
4

17 -5 10 2
6
4 7 3
14 -1 5 -3
8
3 3 2

Encontre a frao que est sobrando.

Olimpada Brasileira de Matemtica das Escolas Pblicas

141

OBMEP

6) O caf, o bolo e o gato Dez minutos antes de colocar o bolo no forno, eu coloquei
meu gato do lado de fora da casa. O bolo deve cozinhar por 35 minutos, ento eu
coloquei o despertador para tocar 35 minutos, aps colocar o bolo no forno.
Imediatamente fiz um caf para mim, o que me tomou 6 minutos. Trs minutos antes
de acabar de beber o caf o gato entrou em casa. Isso foi 5 minutos antes do
despertador tocar.O telefone tocou no meio do tempo entre eu acabar de fazer o caf e o
gato entrar em casa. Falei ao telefone por 5 minutos e desliguei. Eram 3h59min da
tarde.
(a) A que horas coloquei o gato fora de casa?
(b) Quantos minutos depois de colocar o gato fora de casa, o despertador tocou?
Quanto tempo o gato estava fora de casa at o momento em que o telefone tocou?

7) Quais figuras esto corretas?

42O

142

18

12

74O

42O

15

Olimpada Brasileira de Matemtica das Escolas Pblicas

15

SOLUES

OBMEP

11a Lista

1 ab + 1
b = b = a . Logo, a =13b e como a + b 100 segue que
1. (C) Temos: 13 =
1
1 + ab b
+b
a
a
14b 100 b 7,14 . Como b inteiro devemos ter b 7. Logo os pares so em nmero de 7,
a+

a saber:

(13 , 1) , (26 , 2) , (39 , 3) , (52 , 4) , (65 , 5) , (78 , 6) , (91 , 7)

2. (C)( Se x < 0 , ento | x |= x e da 1a equao temos x + (x) + y = 5 y = 5 . Substituindo


esse valor na 2a equao obtemos x=6 o que no possvel pois estamos supondo x<0. Logo, no
h soluo para x < 0 .
$ Se y 0 , ento |y|=y e da 2a equao segue que x=6. Substituindo esse valor na 1a equao
encontramos y=-7, o que no possvel porque estamos supondo que y positivo.
$Conclumos que no h soluo para y 0 e x < 0 . Logo, y < 0 e x 0 , e as equaes so:

2x + y = 5 e x 2y = 6 . Resolvendo obtemos x =

16
7
9
e y = . Portanto, x + y = .
5
5
5

3. (A)A rea do menor crculo 5 2 = 25 cm 2 e do maior 132 = 169 cm 2 . Seja r o raio do


crculo intermedirio, ento a rea do maior anel 169 r 2 . Logo,
169 r 2 = 25 r 2 = 169 25 = 144 , donde r = 12cm

4.(a) Observe que 126 6 = 21 , logo, o numerador 126 foi dividido por 6 para obter o
numerador 21 da outra frao. Logo, o denominador 8_ tambm divisvel por 6 . O nico
nmero da forma 8_ que divisvel por 6 84 , e 84 6 = 18 . Podemos ento completar as
fraes:

126
84

21
18

(b) Note que 33_ deve ser mltiplo de 5 , logo s pode ser 330 ou 335 . Temos

__8
4
__8
= 0, 8 ,
segue
que
= 0,8 ou
= 0,8 .
Temos
335
5
330
330 0,8 = 264 e 335 0,8 = 268 , segue que 8=268 e 33_=335. Podemos completar as
268 4
268 268 67 4
= . Note que
=
= .
fraes:
335 5
335 335 67 5

Como

Olimpada Brasileira de Matemtica das Escolas Pblicas

143

OBMEP

5 10 14 10 7 10 5
= = =
5. (a) 2 fraes cujo produto :
2 7 8
7 4 4 2

(b) 2 fraes cuja diferena

5 5 5 5 5 10 5
=
: = + =
2 4 4 4 4 4 2

5
4

17
6

14
8

-1
3

5
4

17
6

5 17 1 17 1 17 2 15 5
= = =
: + =
2 6 3 6 3 6 6 6 2

5
4

-1
3
17
6

14
8

5 5 2 5 3 5
(d) 2 fraes cujo quociente : = = .
2 3 3 3 2 2

5
4

Logo, a frao que est sobrando 3/2.

10
7

2
3

10
7

2
3

-3
2

5
3

-3
2

-5
4
-1
3

17
6
14
8

5
3
-5
4

14
8

(c) 2 fraes cuja soma

-5
4

5
3
-5
4

-1
3

10
7
-3
2
10
7

5
3

-3
2

6. Vamos listar os eventos ocorridos e contar o tempo gasto em cada um. A primeira atividade
foi colocar o gato for a da casa, logo nossa lista comea com essa atividade e o tempo contado
a partir dela.
Atividade
Gato fora de casa
Bolo no forno
Fazer o caf
Despertador toca
Gato entra em casa
Acabar de tomar o caf
Telefone toca
Desligar o telefone

Tempo depois que o gato


foi posto fora de casa
0 minutos
10 minutos
10+6=16 minutos
35+10=45 minutos
45-5=40 minutos
40+3=43 minutos
16+(40-16):2=28 minutos
28+5 =33 minutos

Podemos agora dar as respostas.


(a) s 3:59horas desliguei o telefone, o que ocorreu 33 minutos depois de colocar o gato
fora de casa. Logo a resposta 3:59-0:33=3:26.
(b) O despertador toca 45 minutos aps colocar o gato for a de casa.
(c) 28 minutos
144

Olimpada Brasileira de Matemtica das Escolas Pblicas

2
3

2
3

OBMEP

Podemos saber exatamente a hora de cada atividade; veja na tabela a seguir.


Atividade
Gato fora de casa
Bolo no forno
Fazer o caf
Despertador toca
Gato entra em casa
Acabar de tomar o caf
Telefone toca
Desligar o telefone

7.

Tempo depois que o gato


foi posto fora de casa
0 minutos
10 minutos
10+6=16 minutos
35+10=45 minutos
45-5=40 minutos
40+3=43 minutos
16+(40-16):2=28 minutos
28+5 =33 minutos

Hora atual
3:59-0:33=3:26
3:26+0:10=3:36
3:26+0:16=3:42
3:26+0:45=4:11
3:26+0:40=4:06
3:26+0:43=4:09
3:26+0:28=3:54
3:59

Figura 1: No est correta porque a soma dos ngulos internos no d 180


Figura 2: No est correta porque o comprimento dos lados no satisfaz o Teorema
de Pitgoras, logo o tringulo no pode ser retngulo
Figura 3: No est correta porque um dos lados no menor que a soma dos outro
dois: 15 > 6 + 8

Olimpada Brasileira de Matemtica das Escolas Pblicas

145

NVEL 3

OBMEP

12a Lista

1) Resolva a equao

| x 1|
= 6.
x2

2) Se um arco de 60o num crculo I tem o mesmo comprimento que um arco de 45o num
crculo II, ento a razo entre a rea do crculo I com a do crculo II :
(A)16/9

(B) 9/16

(D) 3 4

(C) 4/3

(E) 6 9

3) Se x > 0 , y > 0, x > y e z 0 , ento a nica opo errada :


(A) x + z > y + z
(D)

(B) x z > y z

x
y
> 2
2
z
z

(C) xz > yz

(E) xz 2 > yz 2

4) Resolva geometricamente as equaes:


(a) | x 5 |= 2
(c) | 3x 7 |= 9

(b) | x + 3 |= 1
(d) | x + 2 |=| x 5 |

5) A pista de um autdromo tem 20 km de comprimento e forma circular. Os pontos


marcados na pista so: A, que o ponto de partida, B que dista 5 km de A no sentido do
percurso, C que dista 3 km de B no sentido do percurso, D que dista 4 km de C no
sentido do percurso e E que dista 5 km de D no sentido do percurso. Um carro que
parte de A e pra aps percorrer 367 km estar mais prxima de qual dos 5 pontos?
(A) A

(B) B

(C) C

(D) D

(E) E

6) No diagrama ao lado, todos os quadradinhos tm


1 cm de lado. Qual o maior comprimento?
(A)
(B)
(C)
(D)
(E)

7)

AE
CD+CF
AC+CF
FD
AC+CE

Quantos

dentre

os

nmeros

5 , 4 , 3, 2 , 1, 0 ,1 , 2 , 3

satisfazem

desigualdade 3x < 14?


(A)1
146

(B) 2

(C) 3

(D) 4

Olimpada Brasileira de Matemtica das Escolas Pblicas

(E) 5

OBMEP

SOLUES

12a Lista

1. 1o caso: x 1

x 1
= 6 ou
x2
6x 2 x + 1 = 0 . Esta equao no tem razes reais porque = (1)2 4 6 1 = 1 24

Nesse

x 1 0, donde | x 1 | = x 1 . A equao toma a forma

caso

negativo. Logo, no temos solues maiores ou iguais a 1.


2o caso: x < 1
Nesse caso x 1 < 0, donde | x 1 | = (x 1) = 1 x . A equao toma a forma

1 x
= 6 ou 6x 2 + x 1 = 0 . Resolvendo esta equao temos:
2
x
1 1 4 6 (1) 1 25 1 5
1
1
x=
=
=
x = e x = . Como essas duas raazes
26
12
12
2
3
so menores que 1, elas so as razes da equao do enunciado.

2. (B) Como o arco de 60o do crculo I tem o mesmo comprimento que o arco de 45o no crculo II,
conclumos que o raio do crculo I menor que o do crculo II. Denotemos por r e R os raios
dos crculos I e II respectivamente.

No crculo I o comprimento do arco de 60 , igual a 1/6


o

de seu comprimento, ou seja

60o

45o

2 r r
=
.
6
3

Analogamente, no crculo II o comprimento do arco de


45o , igual a 1/8 de seu comprimento, ou seja
Logo,

r
3

R
4

2 R R
=
.
8
4

r 3
= . Finalmente temos:
R 4

rea do crculo I r 2 r
9
3
=
= = =
2
R
4
rea do crculo II R
16

Olimpada Brasileira de Matemtica das Escolas Pblicas

147

OBMEP
3. Nessa questo usaremos as propriedades das desigualdades.
Podemos somar o mesmo nmero a ambos os membros de uma desigualdade sem alterar o

x + z > y + z (somando z a ambos os membros)


(A) e (B) corretas
x z > y z (somando -z a ambos os membros)

sinal, temos: x > y

A opo (C) falsa porque z pode ser negativo, por exemplo: x=5, y=3 e z=-2 temos:
5>3, no entanto 5 ( 2 ) = 10 < 6 = 3 ( 2 ) .
xz

yz

1
> 0 , logo as opes (D) e (E) esto corretas porque foram
z2
obtidas multiplicando-se ambos os membros de x > y por um nmero positivo; em (E) por z 2 e
1
em (D) por 2 .
z
Como z 0 ento z 2 > 0 e

Interpretao
geomtrica
de mdulo:

4. Soluo:

| a b |= distncia entre a e b
(a) | x 5 |= 2 nmeros cuja distncia ao 5 2.
Logo as razes so 3 e 7.

(b) | x + 3 |= 1 nmeros cuja distncia ao -3 1.


Logo as razes so 4 e 2 .

(c) Fazendo a mudana de varivel y = 3x , a equao toma a forma | y 7 |= 9 nmeros


cuja distncia ao 7 9.
Logo as razes so y = 2 e y = 16 .
Destrocando a varivel temos
3x = 2 e 3x = 16 , e obtemos
razes da equao: x =

as

2
16
e x=
.
3
3

(d) As razes da equao | x + 2 |=| x 5 | so os nmeros equidistantes de -2 e de 5. Esses


nmeros s podem estar entre -2 e 5.
Logo, a soluo x = 1,5

148

Olimpada Brasileira de Matemtica das Escolas Pblicas

OBMEP
5. (C)Vamos marcar os 4 pontos a partir de A.
Como o comprimento de 20 km , o comprimento de cada um dos 4 quadrantes 5 km .
Podemos ento marcar os pontos. Como 367 = 18 20 + 7 , o carro deu 18 voltas completas e
percorreu mais 7 km a partir de A. Logo, ele passa 2 km aps B, o que significa que ele pra

1 km de C. Portanto, C o ponto mais prximo.

6. Note que :
AE a hipotenusa de um tringulo de catetos 5cm e 9 cm
CF a hipotenusa de um tringulo de catetos 2cm e 3 cm
AC a hipotenusa de um tringulo de catetos 3cm e 3 cm
FD a hipotenusa de um tringulo de catetos 2cm e 9 cm
CE a hipotenusa de um tringulo de catetos 2cm e 6 cm

Usando o Teorema de Pitgoras calculamos essas hipotenusas:

AE = 52 + 92 = 106 10,3
CF = 22 + 32 = 13 3, 6 CD + CF 5 + 3, 6 = 8, 6
AC = 32 + 32 = 18 4, 2 AC + CF 4, 2 + 3, 6 = 7,8
FD = 22 + 92 = 85 9, 22
CE = 22 + 62 = 40 6,3 AC + CE 4, 2 + 6,3 = 10,5
Logo, o maior AC+CE

14
2
= 4 . Como estamos olhando apenas para
3
3
2
2
valores inteiros de x , ento x 2 tambm inteiro. Sendo x > 4 , conclumos que x2 no
3
mnimo 5. Os nmeros acima que satisfazem essa condio so 5 , 4 , 3 e 3 . Logo a
2

7. (D) Se 3x < 14 ento 3x > 14 ou x >

resposta 4.

Olimpada Brasileira de Matemtica das Escolas Pblicas

149

OBMEP

DESAFIOS

1. (N2/N3) Partindo do nmero 265863 e utilizando uma nica vez cada uma das
operaes + ; ; ; , e tambm uma nica vez os nmeros 51, 221, 6817, 13259 ,
podemos obter vrios nmeros, por exemplo 54911 :
221

51

13259

265863 1203
61353
48094 +6817
54911

Encontre a cadeia que permite obter o menor nmero inteiro positivo.


2. (N2/N3)Voc sabe repartir a figura ao lado em
duas partes idnticas (que possam
C
ser superpostas)?AB=AE=ED=CD=CA

3.(N1/N2/N3) Cada um em seu Estado - Amlia, Bruno, Constncia e Denise so


4 amigos que moram em Estados diferentes e se encontram sentados numa mesa
quadrada, cada um ocupa um lado da mesa.
direita de Amlia est quem mora no Amazonas;
Em frente Constncia est a pessoa que mora em So Paulo;
Bruno e Denise esto um ao lado do outro;
Uma mulher est esquerda da pessoa que mora no Cear.
Um dos quatro mora na Bahia. Quem?
4. (N1/N2) Diviso- Numa diviso, aumentando o dividendo de 1989 e o divisor de 13 ,
o quociente e o resto no se alteram. Qual o quociente?

???????
!!!!!!
5. (N1/N2) Extra-terrestre No planeta Staurus, os anos tm 228 dias ( 12 meses de
19 dias). Cada semana tem 8 dias: Zerum, Uni, Duodi, Trio, Quati, Quio, Seise e Sadi.
Sybock nasceu num duodi que foi o primeiro dia do quarto ms. Que dia da semana ele
festejar seu primeiro aniversrio?
150

Olimpada Brasileira de Matemtica das Escolas Pblicas

OBMEP

6. (N1/N2) Que famlia! Numa famlia cada menino tem o mesmo nmero de irmos
que de irms, e cada menina tem o dobro de irmos que de irms. Qual a composio
dessa famlia?

7. (N1) Siga a pista - Na pista de corrida ao lado, os 7 pontos de referncia so


marcados a cada 50 m . Os atletas devem fazer 2 km no sentido indicado pela flexa.
Eles partem do ponto P. Marque o ponto de chegada C.
P

8. Cara ou Coroa Jernimo joga no


tabuleiro ao lado da seguinte maneira: Ele
coloca uma pea na casa PARTIDA e ele
move a pea da seguinte maneira: ele lana
uma moeda, se der CARA ele avana duas
casas, e se der COROA ele recua uma casa.
Jernimo lanou a moeda 20 vezes e
conseguiu chegar na casa CHEGADA.
Quantas vezes a moeda deu CARA?

9. (N1) Os relgios Um s dos quatro relgios indica a hora correta. Um est


20 minutos adiantado, outro est 20 minutos atrasado, e o quarto est parado. Qual a
hora certa?

Olimpada Brasileira de Matemtica das Escolas Pblicas

151

OBMEP

10. (N1) Contas do papagaio - Rosa tem um papagaio que faz contas de um modo
estranho. Cada vez que Rosa diz dois nmeros ele faz a mesma conta, veja:

Se Rosa diz 4 e 2 o papagaio responde 9


Se Rosa diz 5 e 3 o papagaio responde 12
Se Rosa diz 3 e 5 o papagaio responde 14
Se Rosa diz 9 e 7 o papagaio responde 24
Se Rosa diz 0 e 0 o papagaio responde 1

Se Rosa diz 1 e 8 o que responde o papagaio?


11. (N1/N2) As frias de Toms - Durante suas frias, Toms teve 11 dias com chuva.
Durante esses 11 dias, se chovia pela manh havia sol sem chuva tarde, e se chovia
tarde, havia sol sem chuva pela manh. No total, Toms teve 9 manhs e 12 tardes sem
chuva. Quantos dias duraram as frias de Toms?
12. (N3) Maratona de Matemtica Numa Maratona de Matemtica, o nmero de
questes muito grande. O valor de cada questo igual sua posio na prova: 1
ponto para a questo 1, 2 pontos para a questo 2, 3 pontos para a questo 3, 4 pontos
para a questo 4, , 10 pontos para a questo 10, e assim por diante. Joana totalizou
1991 pontos na prova, errando apenas uma questo e acertando todas as outras. Qual
questo ela errou?Quantas questes tinha a prova?
13. (N1) Escolhi quatro fraes entre 1 2,1 4,1 6,1 10 e 1 12 cuja soma 1 . Quais foram
as fraes que eu no escolhi?
14. Um jogo- Regras;
(i) Partindo da casa em cinza com o nmero 3 deve-se chegar casa TOTAL
deslocando-se somente por linhas ou colunas e calculando os pontos.
(ii) Quando nos deslocamos por uma linha s podemos adicionar, por exemplo
passando da 3 para a 6 ao lado, obtemos 3+(-6)=-3 pontos
(iii) Quando nos deslocamos por uma coluna s podemos subtrair, por exemplo
passando da 3 para a 5 abaixo, obtemos 3-5=-2 pontos.
(iv) S permitido passar uma vez por cada casa.
Qual o caminho que d o maior total?
3
5
-8
-4
0

152

-6
7
-3
1
-2

9
2
-5
6
-7

-9
-1
4
8
TOTAL

Olimpada Brasileira de Matemtica das Escolas Pblicas

OBMEP

15. (N1/N2/N3) Produtos em linha - Em cada uma das casas em branco do quadro
abaixo escrevemos um algarismo dentre oito algarismos 1, 2, 3, 4, 5, 6, 7, 8 e 9 de modo
que os produtos efetuados em linha reta seguindo as flexas forneam os valores
indicados dentro dos casas em cinza. Em qual casa se encontra o nmero 2?

16

27

72

108

28

16. (N2/N3) Cdigo Postal Para fazer a separao em regies da correspondncia que
deve ser entregue, um servio postal indica sobre os envelopes um cdigo postal com
uma srie de 5 grupos de bastes, que podem ser lidos por um leitor tico. Os
algarismos so codificados como a seguir:

A leitura se faz da direita para a esquerda, por exemplo o cdigo postal 91720 se
escreve como

. Em detalhe:

Note que a codificao de 94,


, tem um eixo vertical de simetria. Encontre os
cdigos de 47000 a 47999 , aqueles que apresentam um eixo vertical de simetria.

Olimpada Brasileira de Matemtica das Escolas Pblicas

153

OBMEP

17. (N1/N2/N3) Anis olmpicos Os nmeros de 1 a 9 foram colocados dentro de cinco


anis olmpicos de tal modo que dentro de cada anel a soma 11.
6
8

3
7

9
4

2
5

Disponha os 9 nmeros de outra maneira para que a soma dentro de cada anel seja a
maior possvel.

18. (N2/N3)Denise e Antnio jogam uma srie de 8 jogos no qual o vencedor da


primeira partida ganha 1 ponto, o da segunda 2 pontos, o da terceira 4 pontos, o da
quarta 8 pontos e assim por diante, multiplicando por 2 o nmero de pontos de uma
partida para a outra. No final, Denise ganhou 31 pontos a mais que Antnio e no
houve empate em nenhuma das partidas. Quais partidas Denise ganhou?

19. (N1/N2)Voc sabe repartir um quadrado em 7 quadrados menores?

20.(N1/N2/N3) Ilha misteriosa -Numa misteriosa ilha havia 13 camalees cinza, 15


camalees marrons e 17 camalees vermelhos. Quando dois camalees de cores
diferentes se encontram, os dois tomam a terceira cor. Por exemplo, se um cinza se
encontra com um vermelho, ento os dois ficam marrons. Por causa de uma
tempestade, ocorreram 2 encontros cinza-vermelho, 3 encontros marrom-vermelho e 1
encontro cinza-vermelho, quantos camalees de cada cor ficaram na ilha?

21. (N3)Universo hostil - Num deserto h cobras, ratos e escorpies. Cada manh, cada
cobra mata um rato. Cada meio-dia, cada escorpio mata uma cobra. Cada noite, cada
rato mata um escorpio. Ao final de uma semana, noite, s restava um rato. Quantos
ratos havia na manh no incio da semana?

154

Olimpada Brasileira de Matemtica das Escolas Pblicas

RESPOSTAS

OBMEP

1.

6817

+ 221

51

265863
39 260
13260 13259

2.

45o
E

3. Bruno ou Amlia (O desafio tem duas solues).


4. 153
5. Seise
6. 3 meninas e 4 meninos

7.

C
8. 12
9. 17 h 05 min
10. 1
11. 16 dias
12. 25 e 63, respectivamente.
13.
1
4

1
2

1
1

12

Olimpada Brasileira de Matemtica das Escolas Pblicas

155

OBMEP
15.casa B
16. 47679 e 47779

17.
8

6
1

5
3

9
2

18. 1a, 2a, 3a, 4a e 8a

19.

20. 16 cinzas, 18 marrons e 11 vermelhos


21. 1873

156

Olimpada Brasileira de Matemtica das Escolas Pblicas

OBMEP

Olimpada Brasileira de Matemtica das Escolas Pblicas

157

Apresentacao

Apresentac
ao
A ideia de organizar e divulgar um Banco de Questoes com problemas propostos
em provas de olimpadas surgiu em 2005, por solicitacao de alunos e professores
que participavam da OBMEP e sentiram falta desse tipo de material. A excelente
acolhida que teve o Banco de Questoes-2006, por esses participantes, e tambem por
estudantes de cursos de licenciatura em Matematica, nos motivou a continuar esse
trabalho.
Boa parte dos problemas aqui apresentados foram extrados de olimpadas de
Matematica nacionais e internacionais. Nessa edicao, introduzimos alguns problemas com solucao mais complexa, que nao constaram do Banco Questoes-2006. O
mais importante e tentar resolve-los, nao importando o tempo gasto para isso e a
diculdade encontrada. Igualmente importante, e entender a solucao que apresentamos. Nao conseguir resolve-los nao deve ser motivo de desanimo. Entendemos que
resolver, ou tentar resolver, problemas desaadores, e uma das mais interessantes e
ecientes formas de aprender Matematica.
Os problemas estao separados em tres nveis, de acordo com a classicacao feita
pela OBMEP, mas muitos deles podem (e devem) ser resolvidos por todos os alunos.
Assim sendo, recomendamos que os alunos passeiem por todos os problemas e
selecionem alguns, de outros nveis, para resolver.
Desejamos que esse Banco de Questoes propricie a todos um bom trabalho e um
divertimento interessante.
Direcao Academica da OBMEP

OBMEP 2007

Lista 1

Nvel 1

Nvel 1
Lista 1
1. M
ultiplos de 9
(a) Qual e o menor m
ultiplo (positivo) de 9 que e escrito apenas com os
algarismos 0 e 1?
(b) Qual e o menor m
ultiplo (positivo) de 9 que e escrito apenas com os
algarismos 1 e 2?

2. A florista - Uma orista colheu 49kg de ores do campo que podem ser
vendidas imediatamente por R$1, 25 o quilo. A orista pode tambem vendelas desidratadas por 2 reais a mais no quilo. O processo de desidratacao faz as
ores perderem 5/7 de seu peso. Qual e o tipo de venda mais lucrativo para
a orista?

3. Divisores - Seja N o menor n


umero que tem 378 divisores e e da forma
2a 3b 5c 7d . Quanto vale cada um desses expoentes?

4. O produto dos algarismos - Denotemos por P (n) o produto dos algarismos


do n
umero n. Por exemplo: P (58) = 5 8 = 40 e P (319) = 3 1 9 = 27.
(a) Quais os n
umeros naturais menores que 1000 cujo produto de seus algarismos e 12, ou seja: os n
umeros naturais n < 1 000 tais que P (n) = 12?
(b) Quantos n
umeros naturais menores que 199 satisfazem P (n) = 0? Ou
seja: tem o produto de seus algarismos igual a 0?
OBMEP 2007

Nvel 1

Lista 1

(c) Quais n
umeros naturais menores que 200 satisfazem a desigualdade 37 <
P (n) < 45?
(d) Dentre os n
umeros de 1 a 250, qual o n
umero cujo produto de seus algarismos e o maior?

5. Suco de laranja - Davi vai a um armazem que vende uma garrafa de suco
de laranja por R$2, 80 e uma caixa com seis dessas garrafas por R$15, 00.
Ele precisa comprar 22 garrafas para seu aniversario. Quanto ele gastara no
mnimo?

6. A casa da Rosa - A gura mostra a planta da casa da Rosa. O quarto e


o quintal sao quadrados. Qual e a area da cozinha?

.......................................................................................................................................................................................................................................................................................
...
...
...
..
...
..
...
...
....
...
..
...
.....
.....
....
...
...
...
....
....
....
...
.....
.....
...
...
...
Quarto
Sala
...
....
....
...
...
...
...
....
....
...
...
...
....
....
....
2
2
...
16m
24m
...
...
.
.
.
.
.....
.....
....
...
...
...
....
...
....
.........................................................................................................................................................................................................................................................................................
...
...
...
.....
.....
.....
...
...
...
.... Quintal
....
....
...
...
...
Cozinha
....
....
....
...
...
...
2
4m
....
....
....
...
.
...
................................................................................................................................................................................................................................................................................

7. O passeio do Matias - Matias passeia em volta de 4 quarteiroes perto de


sua casa. O seu passeio consiste em fazer o maior percurso possvel de bicicleta, respeitando as seguintes condicoes:
2

OBMEP 2007

Lista 1

Nvel 1

ele pode passar varias vezes pelos cruzamentos das


ruas, mas ele nao pode passar mais do que uma vez
pela mesma quadra. Quando ele nao pode mais
respeitar essas condicoes, ele tem que saltar da bicicleta e voltar a pe. Ele parte de P e deve voltar
a P . Os quatro quarteiroes sao quadrados com
100 metros de lado em cada quadra. Qual o maior
percurso que ele pode fazer? A largura das ruas e
desprezvel.

..................................................................................................................................................................................
...
...
...
...
...
....
...............................................................
...............................................................
...
...
....
....
....
....
....
....
.....
.....
.....
.....
.....
.....
...
...
...
...
...
...
....
....
....
....
....
....
...
...
...
...
...
...
....
....
....
....
....
....
...
...
...
...
...
...
.
.
.
.
.
.
.....
.....
.....
.....
.....
.....
...............................................................
...............................................................
...
...
....
....
...
...
....
....
.............................................................
.............................................................
.....
.....
....
....
....
....
...
...
.....
.....
.....
.....
....
....
...
...
...
...
...
...
....
....
....
....
....
....
...
...
...
...
...
...
....
....
....
....
....
....
...
...
...
...
...
...
.
.
.
.
.
....
.....
.....
.....
.....
.....
...
...............................................................
...............................................................
...
...
....
...
...
...
....
..
................................................................................................................................................................................

8. O adesivo dos carros oficiais - O prefeito de uma cidade decidiu colocar


um adesivo em todos os carros ociais. O adesivo tera a forma retangular com
6 quadrados dispostos em 2x3 e com 3 cores: 1 quadrado azul, 2 quadrados
amarelos e 3 quadrados verdes. Dentre quantos tipos diferentes de adesivo o
prefeito tera que escolher?

OBMEP 2007

Nvel 1

Solucoes da Lista 1

Solu
co
es da Lista 1
1. M
ultiplos de 9
(a) Um n
umero e divisvel por 9 se a soma dos seus algarismos e um m
ultiplo
de 9. Logo, o n
umero deve ter 9 algarismos iguais a 1. Assim, o menor
n
umero e: 111 111 111.
(b) Devemos usar o maior n
umero possvel de algarismos iguais a 2, que
devem car nas casas mais a` direita. Assim, o menor n
umero e: 12 222.

2. A florista - Se a orista vender as ores sem desidrata-las, ela vai apurar


49 1, 25 = 61, 25 reais.
O peso das ores apos a desidratacao e

2
7

49 = 14 kg. Logo, vendendo as

ores desidratadas, ela apura 14 3, 25 = 45, 50. Portanto, a orista ganha


mais no processo sem a desidratacao.

3. Divisores - Como 2, 3, 5 e 7 sao primos, para que 2a 3b 5c 7d tenha


378 divisores, devemos ter:
(a + 1) (b + 1) (c + 1) (d + 1) = 378.
Decompondo 378 em fatores primos, encontramos 378 = 2 33 7. Logo,
(a + 1) (b + 1) (c + 1) (d + 1) = 2 33 7.
Por outro lado, como N e mnimo entao os expoentes estao ordenados do maior
para o menor, isto e, a b c d.
Armamos que d > 0, pois se d = 0 entao a + 1, b + 1 ou c + 1 tem dois fatores
maiores do que 1. Se a + 1 = mn com m n > 1 temos que
2a = 2mn1 = 2m1 2mnm = 2m1 (2m )n1 2m1 8n1 > 2m1 7n1 ,
4

OBMEP 2007

Solucoes da Lista 1

Nvel 1

onde na pen
ultima desigualdade usamos o fato que m 3. Assim, temos
que 2a 3b 5c 7d > 2m1 3b 5c 7n1 , logo encontramos um n
umero com a mesma
quantidade de divisores, mas menor. A prova e igual no caso em que b + 1 tem
dois fatores ou c + 1 tem dois fatores. Assim, d 1 e temos unicamente as
seguintes possibilidades
a

(a + 1)(b + 1)(c + 1)(d + 1) = 378

20

21 3 3 2

13

14 3 3 3

9732

7633

Por u
ltimo, como
27
220 32 52 71
> 1,
=
213 32 52 72
7
213 32 52 72
25 7
=
>1
28 36 51 71
34
e
28 36 52 71
22 3
> 1,
=
26 35 52 72
7
temos que o valor de N e 26 35 52 72 . Portanto, a = 6, b = 5, c = 2 e d = 2.

4. O produto dos algarismos


(a) Como 12 = 2 6 = 4 3 = 2 2 3, devemos utilizar os algarismos
1 , 2 , 3 , 4 e 6 cujos produtos sejam 12. Assim temos:
n
umeros com 2 algarismos: 26, 62, 34, 43
n
umeros com 3 algarismos:
com os algarismos 1, 2 e 6: 126, 162, 216, 261, 612, 621
OBMEP 2007

Nvel 1

Solucoes da Lista 1

com os algarismos 1, 3 e 4: 134, 143, 314, 341, 413, 431


com os algarismos 2, 2 e 3: 223, 232, 322.
(b) Se P (n) = 0, entao o produto de seus algarismos e igual a zero, logo pelo
menos um dos algarismos do n
umero n e zero. Temos 19 n
umeros com zero
so nas unidades, 9 n
umeros com zero so nas dezenas e ainda o n
umero 100,
totalizando 29 n
umeros:

0 , 10 , 20 , . . . , 90 , 110, . . . , 190 , 101, 102, . . . , 109 .



 



0 s
o nas unidades
0 s
o nas dezenas
(c) Queremos encontrar os n
umeros menores do que 200, cujo produto de seus
algarismos seja maior do que 37 e menor do que 45. Por exemplo, 58 e um
desses n
umeros porque 5 8 = 40.
Em primeiro lugar, note que nao existem n
umeros cujo produto de seus algarismos sejam 38, 39, 41, 43 e 44 porque esses n
umeros nao podem ser escritos
como produto de dois ou tres algarismos. Restam, entao: 40 e 42. Vejamos as
possibilidades:
n
umeros menores do que 200 cujo produto dos algarismos e 40: 58, 85,
158 e 185
n
umeros menores do que 200 cujo produto dos algarismos e 42: 67, 76,
167 e 176
(d) O n
umero e 249 = 2 4 9 = 72.

5. Suco de laranja - Se Davi comprar 6 garrafas individualmente, ele vai


gastar 6 2, 80 = 16, 80 reais, que e mais caro do que comprar uma caixa com
seis. Portanto, ele deve comprar a maior quantidade possvel de caixas. Para
ter pelo menos 22 garrafas, ele pode comprar 4 caixas e gastara 60 reais, ou
6

OBMEP 2007

Solucoes da Lista 1

Nvel 1

comprar 3 caixas e 4 garrafas individualmente, caso em que gastara 3 15 +


4 2, 80 = 56, 20 reais, que e o mnimo possvel.

6. A casa da Rosa - Como o quarto e quadrado e tem 16m2 de area, entao


suas dimensoes sao 4 m por 4 m. Da mesma forma, as dimensoes do quintal
sao 2 m por 2 m.
Agora, a sala tem 24 m2 e uma das
dimensoes e a mesma que a dimensao
do quarto, isto e 4 m, logo a outra dimensao da sala e 6 m. Assim, as dimensoes totais da casa sao 10 m por
6 m e a area total da casa e 60 metros

.......................................................................................................................................................................................................................................................................................
...
...
...
...
...
....
...
...
4
6
...
..
..
...
...
.....
.....
...
...
...
....
....
....
...
.....
.....
...
...
...
Quarto
Sala
...
....
....
...
...
...
... 4
....
....
...
...
...
....
....
....
...
...
...
.
.
...
....
.....
...
...
...
...
...
....
.
...
.........................................................................................................................................................................................................................................................................................
....
....
....
.....
.....
.....
...
...
...
.... Quintal
....
....
...
...
...
Cozinha
....
....
.... 2
...
...
...
....
....
....
...
.
...
................................................................................................................................................................................................................................................................................

quadrados. Logo, a area da cozinha e


60 16 24 4 = 16 m2 .

7. Passeio do Matias - Primeiro observamos que temos 12 quadras de 100


metros entre os 4 quarteiroes. Alem disso, entre os quatro quarteiroes temos 4
esquinas nas quais chegam 3 quadras e que estao marcadas com  no desenho.
Assim, no momento em que chegamos a uma das ditas esquinas temos que sair,
logo usamos 2 das quadras em cada passada e, no momento que chegamos de
novo, temos que parar.
Portanto, dentre as ditas 4 esquinas, em todo caminho que tracemos tem pelo menos duas esquinas 
em que nao usamos todas as quadras que chegam
`a esquina mencionada. Assim, o caminho de comprimento maximo usa no maximo 10 quadras. Na
gura desenhamos um dos trajetos maximos.

OBMEP 2007

-

............................................................................................................................................................
...
...
...
...
...
....
.......................................................
.......................................................
...
...
....
...
...
.....
...
...
...
...
...
...
....
....
....
....
....
....
...
...
...
...
...
...
....
....
....
....
....
....
...
...
...
...
...
...
....
....
....
....
....
....
.
...
.
...
...
...
.........................................................
...
.........................................................
...
...
...
...
...
....
....
.......................................................
.......................................................
...
...
.....
....
.....
.....
.....
....
.....
.....
.....
.....
.....
.....
...
...
...
...
...
...
....
....
....
....
....
....
...
...
...
...
...
...
....
....
....
....
....
....
...
...
...
...
...
...
......................................................
......................................................
....
....
.
.
...
...
...
...
..
.
.........................................................................................................................................................

?

r

Nvel 1

Solucoes da Lista 1

8. O adesivo oficial - Como o quadrado pintado da cor azul pode estar em


qualquer lugar, entao temos 6 possveis formas de escolher a posicao desse
quadrado. Entre os 5 quadrados restantes precisamos pintar dois de amarelo,
o que podemos fazer de 10 formas, assim os tres quadrados restantes sao
pintados de verde. Portanto, o prefeito tem 6 10 = 60 formas diferentes de
escolher o adesivo.

OBMEP 2007

Lista 2

Nvel 1

Lista 2
1. Adi
c
ao de n
umeros - Qual e o algarismo a em
a000 + a998 + a999 = 22997 ?

2. Cubo perfeito e divisibilidade - Quais os cubos perfeitos que dividem 94 ?

3. Localizando pontos - Qual e o ponto indicado no diagrama?

18

19

20

4. Calculando porcentagem - Num teste com 84 questoes se voce acerta


58/84 das questoes, entao qual e o seu percentual de acertos?

5. Comparando algarismos - Um n
umero se chama ascendente se cada um
de seus algarismos e maior do que o algarismo que esta a` sua esquerda. Por
exemplo, 2568 e ascendente e 175 nao. Quantos n
umeros ascendentes existem
entre 400 e 600?

6. Muro colorido - O muro da gura e construdo com 14 tijolos nas cores


amarelo, azul e vermelho e tal que dois tijolos que se tocam sao de cores
diferentes. Os precos dos tijolos sao dados na tabela. Qual o menor preco que
se gastara na compra dos tijolos para construir esse muro?
OBMEP 2007

Nvel 1
tijolo

R$

amarelo

azul

vermelho

Lista 2

................................................................................................................................................................
..
...
...
...
...
...
..
..
..
..
..
.
...........................................................................................................................................................................................................................
...
...
...
...
...
..
..
..
..
...
..
..
..
..
...
....................................................................................................................................................................................................................
...
...
...
...
.....
.....
.....
.....
.
.
.
.
.........................................................................................................................................................................................................................
....
...
...
.....
....
...
...
...
...
.
............................................................................................................................................................................................................

7. Divisores e fatora
c
ao - Decompor 96 em dois fatores cuja soma dos quadrados seja 208.

8. Brincando com
areas - Lus desenhou um retangulo de 6cm por 10cm e
quer dividi-lo em quatro partes. Cada parte deve ter de area, respectivamente,
8cm2 , 12cm2 , 16cm2 , 24cm2 . Desenhe como ele pode fazer essa divisao.

10

OBMEP 2007

Solucoes da Lista 2

Nvel 1

Solu
co
es da Lista 2
1. Adi
c
ao de n
umeros - Efetuando a adicao
a1 000
a 998
+
a 999
................................................................
 997
encontramos 997 = 22997, onde  = a + a + a + 1.
Logo, 22 = a + a + a + 1. Assim, a = 7.

2. Cubo perfeito e divisibilidade - Um cubo perfeito e um n


umero da forma
a3 , onde a e um natural. Como 94 = (32 )4 = 38 , os cubos perfeitos que dividem
38 sao: 33 e (32 )3 = 36 .

3. Localizando pontos - O ponto indicado esta 4 marcas a` direita de 19. Entre


18 e 19 e entre 19 e 20 sao feitas subdivisoes em 10 partes iguais, logo cada
marca equivale a 0, 1 nessa escala. Assim, o ponto indicado e 19, 4.

18

19

20

4. Calculando porcentagem - Temos 58 acertos em 84 questoes, logo a razao


de acertos e

58
.
84

Dividindo 58 por 84 encontramos 0, 69047 com aproximacao.

Logo, o percentual e aproximadamente 69, 047%.

OBMEP 2007

11

Nvel 1

Solucoes da Lista 2

5. Comparando algarismos - Os n
umeros que estamos procurando sao maiores do que 400 e menores do que 600, logo o algarismo das centenas e 4 ou 5.
Como sao n
umeros ascendentes, o algarismo das dezenas tem que ser menor
do que o algarismo das unidades. Vejamos como escolher os algarismos das
dezenas e das centenas.

56

57
4

58

59

67

4
68

69

67

68

69

78
4
79

78
5
79

89

89

Logo, temos 10 n
umeros ascendentes com 4 como algarismo das centenas e 6
n
umeros ascendentes com 5 como algarismo das centenas; no total temos 16
n
umeros ascendentes.

6. Muro colorido - Observamos que no momento que xamos a cor de dois


tijolos vizinhos, entao a cor de todos os tijolos ca xa. Assim, os tijolos
marcados por A, B ou C na gura tem que ter a mesma cor.
...............................................................................................................................................................
...
...
...
...
...
...
...
...
...
..
..
..
........................................................................................................................................................................................................................
...
...
...
...
...
..
..
..
..
...
..
..
..
..
...
...................................................................................................................................................................................................................
...
...
...
...
....
....
.....
.....
..
..
..
..
........................................................................................................................................................................................................................
....
....
....
....
.....
..
...
...
...
...
...........................................................................................................................................................................................................

Como a maior quantidade de tijolos esta marcada com A, 6 no total, entao


tais tijolos sao amarelos. Por outro lado, temos a mesma quantidade de tijolos
12

OBMEP 2007

Solucoes da Lista 2

Nvel 1

B e C, 4 de cada tipo, logo temos que pintar 4 tijolos de azul e 4 de vermelho.


Assim, o menor preco na compra dos tijolos e
6 6 + 4 7 + 4 8 = 96 reais.

7. Divisores e fatora
c
ao - Como o produto dos dois n
umeros e 96, eles sao
divisores de 96. Decompondo 96 em fatores primos, encontramos 96 = 25 3,
logo seus divisores sao:
1 , 2 , 3 , 4 , 6 , 8 , 12 , 16 , 24 , 32 , 48 , 96.
Os divisores 96, 48, 32, 24 e 16 nao servem pois seus quadrados ja sao maiores
do que 208. Sobram

1 , 2 , 3 , 4 , 6 , 8 , 12

cujos quadrados sao:


1 , 4 , 9 , 16 , 36 , 64 , 144.
Agora vemos que a u
nica possibilidade e 64 + 144 = 208. Como 8 12 = 96,
os n
umeros sao 8 e 12.

8. Brincando com
areas - Faremos a divisao com retangulos. Observamos
que 24 = 6 4 e 12 = 6 2, logo ele pode fazer um primeiro corte a 4 cm
no lado de 10cm e outro corte a 2 cm do corte anterior. Depois de tais cortes,
camos uma cartolina de tamanho 6 4. Por u
ltimo, como 16 = 4 4, basta
fazer um u
ltimo corte a 4 cm no lado de 6 cm. Os cortes estao ilustrados na
seguinte gura.
4

24

4
16

12

OBMEP 2007

13

Nvel 1

Lista 3

Lista 3
1. Compara
c
ao de n
umeros - Escreva em ordem crescente os n
umeros:

3
729 e 4 38416.

121,

2. As moedas - Uma brincadeira comeca com 7 moedas alinhadas em cima de


uma mesa, todas com a face coroa virada para cima. Para ganhar a brincadeira
e preciso virar algumas moedas de modo que no nal duas moedas vizinhas
estejam sempre com faces diferentes viradas para cima. A regra da brincadeira
e: em cada jogada tem-se que virar duas moedas vizinhas. Quantas jogadas,
no mnimo, sao necessarias para ganhar a brincadeira?
...
.............
.............
.............
.............
.............
.............
.............. ...................
.......... ..............
.......... ..............
.......... ..............
.......... ..............
.......... ..............
.......... ..............
.....
... .......
... .......
... .......
... .......
... .......
... .......
...
...
... ..
... ..
... ..
... ..
... ..
... ..
...
.
.
.
.
....
.
.
... .
... .
... ..
... ..
... ..
... ..
...
...
..
..
..
..
..
..
.
...
.. .....
.. .....
.. .....
.. .....
.. .....
.. .....
..
.
.
.
.
.
.
. ..
. ..
. ..
. ..
. ..
. ..
..
...
.
.
.
.
.
.
.
.
.
.
.
.
.
.
....
....
....
....
....
....
.
.....
..
..
..
..
..
..
.
.
.
.
.
.
.
.
.
.
.
.
........
.
.
.
.
.
.
.
.
.
.
.
.
.
.
.
.
.
.
.
.
.
.
........... ...............
.
.
.
.
.
.
.
.
.
.
.
..............................
..............................
..............................
..............................
..............................
.........................
..........

coroa coroa coroa coroa coroa coroa coroa

3. O pre
co do frango - O preco do quilo de frango era R$1, 00 em janeiro de
2000 e comecou a triplicar a cada 6 meses. Quando ele atingira R$81, 00?
(a) 1 ano

(b) 2 anos

(c) 2 1/2 anos

(d) 13 anos

(e) 13 1/2 anos

4. Excurs
oes a Foz do Igua
cu - Em 2005, uma agencia de turismo programou
uma excursao para a Foz do Iguacu, distribuindo as pessoas em onibus de 27
lugares, tendo sido necessario formar um onibus incompleto com 19 lugares.
Em 2006, aumentou em 53 o n
umero de participantes e continuou a utilizar
onibus de 27 de lugares. Quantos onibus a mais foram necessarios e quantas
pessoas caram no onibus incompleto em 2006?

14

OBMEP 2007

Lista 3

Nvel 1

5. As fra
c
oes de Laura -

Laura desenhou 5

crculos dentro dos quais ela quer colocar n


umeros.

m
+

Ela coloca os crculos am de formar uma fracao e

m
+

..............................................................................................................

= m

seu valor inteiro.


De quantas maneiras Laura colocou os n
umeros 2, 3 , 5 , 6 e 11 dentro dos
crculos para que a igualdade seja verdadeira?

6. C
alculo da unidade - Qual e o algarismo da unidade do produto

(5 + 1)(53 + 1)(56 + 1)(512 + 1) ?


(a) 0

(b) 1

(c) 2

(d) 5

(e) 6

7. N
umeros cruzados - Francisco escreveu 28 algarismos numa tabela 6 6 e
pintou de preto algumas casas, como nas palavras cruzadas. Ele fez uma lista
de todos os n
umeros que podem ser lidos horizontalmente ou verticalmente,
excluindo os n
umeros de um so algarismo. Veja a lista:
28

45

51

57

72

88

175

289

632

746

752

805

885

5647

5873 7592

8764

Preencha a tabela escrevendo os n


umeros dados. Um algarismo ja foi colocado.

2
OBMEP 2007

15

Nvel 1

Lista 3

8. Ovos e ma
c
as - Num armazem, uma d
uzia de ovos e 10 macas tinham o
mesmo preco. Depois de uma semana, o preco dos ovos caiu 10% e o da maca
subiu 2%. Quanto se gastara a mais na compra de uma d
uzia de ovos e 10
macas?
(a) 2%

16

(b) 4%

(c) 10%

(d) 12%

OBMEP 2007

(e) 12, 2%

Solucoes da Lista 3

Nvel 1

Solu
co
es da Lista 3
1. Compara
c
ao de n
umeros - Fatorando os n
umeros e extraindo as razes
temos:

121
=
112
= 11

3
3
729
=
93
= 9

4
38416 = 4 24 74 = 2 7 = 14.

Logo, em ordem crescente temos:

729, 121, 4 38416.

2. As moedas - Se damos o valor de 1 a`s coroas e 1 a`s caras e somamos


os resultados depois de cada jogada, inicialmente a brincadeira comeca com 7
como soma e temos que chegar a cara e coroa alternadas, logo a brincadeira
termina em 1 ou -1. Observamos que em cada passo da brincadeira temos
as seguintes possibilidades: trocamos duas coroas por duas caras e o valor da
soma diminui em 4; trocamos uma cara e uma coroa por uma coroa e uma
cara e o valor da soma ca inalterado ou trocamos duas caras por duas coroas
e o valor da soma aumenta em 4. Portanto, e impossvel de 7 como soma
inicial chegar a 1, mas e possvel chegar a 1, isto e, 4 caras e 3 coroas. Como
precisamos obter 4 caras nao consecutivas, entao precisamos de pelo menos 4
jogadas. As 4 jogadas se ilustram no seguinte desenho:
.....................
......
...
..
...
.....coroa ....
.
...
..
.
....
.
.
.......................

.....................
......
...
..
...
.....coroa ....
.
...
..
.
....
.
.
.......................

.....................
......
...
..
...
.....coroa ....
.
...
..
.
....
.
.
.......................

.....................
......
...
..
...
.....coroa ....
.
...
..
.
....
.
.
.......................

.....................
......
...
..
...
.....coroa ....
.
...
..
.
....
.
.
.......................

.....................
......
...
..
...
.....coroa ....
.
...
..
.
....
.
.
.......................

...........................
....
...
..
.... cara ....
...
.
....
...
........................

...........................
....
...
..
....coroa ....
...
.
....
...
........................

...........................
....
...
..
....coroa ....
...
.
....
...
........................

...........................
....
...
..
....coroa ....
...
.
....
...
........................

...........................
....
...
..
....coroa ....
...
.
....
...
........................

...........................
....
...
..
....coroa ....
...
.
....
...
........................

...........................
....
...
...
.... cara ...
.
...
...
....
.........................

...........................
....
...
...
.... cara ...
.
...
...
....
.........................

...........................
....
...
...
....coroa ...
.
...
...
....
.........................

...........................
....
...
...
....coroa ...
.
...
...
....
.........................

...........................
....
...
...
....coroa ...
.
...
...
....
.........................

...........................
....
...
...
.... cara ...
.
...
...
....
.........................

........................
.....
...
...
.
..... cara ....
.
...
...
....
.
.
.
......................

........................
.....
...
...
.
.....coroa ....
.
...
...
....
.
.
.
......................

........................
.....
...
...
.
..... cara ....
.
...
...
....
.
.
.
......................

........................
.....
...
...
.
.....coroa ....
.
...
...
....
.
.
.
......................

........................
.....
...
...
.
.....coroa ....
.
...
...
....
.
.
.
......................

........................
.....
...
...
.
..... cara ....
.
...
...
....
.
.
.
......................

........................
.....
...
...
.
..... cara ....
.
...
...
....
.
.
.
......................

..........................
...
....
..
.
..... cara ....
.
...
..
.
.....
.
......................

..........................
...
....
..
.
.....coroa ....
.
...
..
.
.....
.
......................

..........................
...
....
..
.
..... cara ....
.
...
..
.
.....
.
......................

..........................
...
....
..
.
.....coroa ....
.
...
..
.
.....
.
......................

..........................
...
....
..
.
..... cara ....
.
...
..
.
.....
.
......................

..........................
...
....
..
.
.....coroa ....
.
...
..
.
.....
.
......................

..........................
...
....
..
.
..... cara ....
.
...
..
.
.....
.
......................

.....................
......
...
..
...
.....coroa ....
.
...
..
.
....
.
.
.......................
...........................
....
...
..
.... cara ....
...
.
....
...
........................

OBMEP 2007

...........................
....
...
...
.... cara ...
.
...
...
....
.........................

17

Nvel 1

Solucoes da Lista 3

3. O pre
co do frango - Como 81 = 34 , entao o valor do franco triplicou 4
vezes, o n
umero de meses transcorridos foi 4 6 = 24 meses, isto e, 2 anos, ou
seja, em janeiro de 2002 o frango atingira o preco proposto. A opcao correta
e (b).

4. Excurs
oes a Foz do Igua
cu - Temos um onibus com 27 19 = 8 lugares
livres e ainda precisamos acomodar os 53 8 = 45 participantes em onibus
claro que um onibus nao e suciente, logo precisamos de 2
de 27 lugares. E
onibus e vamos ter 2 27 45 = 9 lugares livres no u
ltimo onibus. Ficaram
18 pessoas no onibus incompleto.

5. As fra
c
oes de Laura - Como a fracao e igual a um n
umero inteiro, o seu
numerador tem que ser um m
ultiplo do seu denominador. Vamos testar todas
as possibilidades e escolher as que satisfazem as condicoes do problema:
3 + 11 + 6
5 + 11 + 6
3+5+6
= 7;
= 10 ;
= 11 nao satisfazem
2
2
2
2 + 5 + 11
= 6
3

satisfaz

3 + 6 + 11
= 4
5

nao satisfaz

2 + 5 + 11
= 3
6

satisfaz

2+3+6
= 1
11

nao satisfaz.

Assim temos duas respostas:


+ 11m
2m+ 5m
= 6m
3m

..............................................................................................................

18

OBMEP 2007

+ 11m
2m+ 5m
= 3m
6m

..............................................................................................................

Solucoes da Lista 3

Nvel 1

6. C
alculo da unidade - O algarismo da unidade de qualquer potencia de 5 e
5, segue que o algarismo da unidade de cada fator do produto e 5+1 = 6. Mas,
6 6 = 36, ou seja, o produto de dois n
umeros terminados em 6 e tambem
um n
umero que termina em 6. Logo, o algarismo da unidade desse produto e
6. A opcao correta e (e).

7. N
umeros cruzados 7 5
8
5 7
6 3
4
7 5

2
8 5
1
2
8 7
9 2

8 8
0
7 5
4
6 4
5

8. Ovos e ma
c
as - Suponhamos, inicialmente, que uma d
uzia de ovos custava
R$ 1, 00. Assim, 10 macas tambem custavam R$ 1, 00. Como o preco dos ovos
subiu 10%, o novo valor dos ovos e R$ 1, 10. O preco das macas diminuiu 2%,
logo o novo preco das macas e R$ 0, 98.
Assim, antes gastava-se 2 reais na compra de 1 d
uzia de ovos e 10 macas, agora
gasta-se 1, 10 + 0, 98 = 2, 08. Da temos que o aumento foi de R$ 0, 08, que
corresponde ao percentual:
0, 08
4
= 0, 04 =
= 4%.
2
100
A opcao correta e (b).

OBMEP 2007

19

Nvel 1

Lista 4

Lista 4
1. Divis
ao de n
umeros decimais - Sabendo que 144177 = 25488 podemos
concluir que 254, 88 0, 177 e igual a
(a) 1440

(b) 14, 4

(c) 1, 44

(d) 0, 144

(e) 144

2. C
alculo de porcentagem - Num teste com 84 questoes se voce acerta 58/84
das questoes, entao qual e o seu percentual de acertos?

3. Almo
co dos amigos - J
ulio e
Denise almocaram num restaurante
que oferece tres tipos de prato e
tres tipos de vitamina, cujos precos

R$
prato simples

prato com carne

11

prato com peixe

14

vitamina de leite

vitamina de frutas

vitamina especial

estao na tabela ao lado. Cada um


escolheu um prato e uma vitamina.
J
ulio gastou 6 reais a mais do que
Denise. Quanto Denise gastou?

4. Adi
c
ao de inteiros positivos - Encontre quatro n
umeros inteiros distintos
e maiores do que 0 tais que somados de tres em tres dao 6, 7, 8 e 9.

5. O passeio do Jorge - Jorge passeia por um


caminho em forma de retangulo, onde estao dispostas doze arvores com 5 m de distancia entre
duas consecutivas. Jorge brinca de tocar cada
arvore durante seu passeio.

.......................................................................................................................................................................................
...
...
....
....
...
...
....
....
....
....
....
....
....
....
....
....
....
....
....
...
....
...
....
....
....
...
.......................................................................................................................................................................................

u
u

Primeiro ele toca a arvore do canto, assinalada com P na gura, e percorre 32


metros num mesmo sentido; a ele volta 18 metros e depois torna a andar para
frente mais 22 metros. Em quantas arvores ele tocou?
20

OBMEP 2007

Lista 4

Nvel 1

6. A descoberta do algarismo - Os quadrados dos n


umeros naturais de 1 a 99
foram escritos um apos o outro, formando o n
umero 14916253649 . . .. Qual e
o algarismo que ocupa a 100a posicao? (As posicoes sao contadas da esquerda
para a direita: a 1a posicao e o 1, a 2a e o 4, etc.)

7. OBMEP - Cada um dos 7 discos X, Z, O, B, M, E, P tem um peso diferente,


de 1 g a 7 g. Nas intersecoes dos discos indicamos a soma dos pesos desses dois
discos. Qual e a soma dos pesos dos cinco discos O, B, M, E, P ?
..........
..........
.............. .....................
.............. .....................
.......
.......
......
......
......
......
....
....
...
...
...
...
.
.
...
...
...
...
...
...
..
.
....
..
...
...
.
...
...............................
.............................
..............................
.
.
.
.
.
.
.
.
.
.
.
.
...
.
.
.
.
.
.
.
.
.
.
.
.
.
.
.
.
.
.
.
.
.
........
........
........ ........
...
.
......
...
.
.......
.
.
..
.
.
.
.
.....
.
.
.
.
.
.
.
.
.
.
.
.
.
....
......
...
.
..
.
.
...
....
.
...
.
.
.
.
.
.
.
.
.
.
.
.
.
...
... ..
...
...
..
.. ...
...
... ..
....
...
....
...
...
.. ....
.
...
.
.
.
.
.
..
.
.
......
.
.
.
.
.
.
.
.
...
.
.......
...
.....
.
.
........
....
....
.
.
.
.
.
.
.
.
.
.
.
.
.
.
.
...
.
.
.
.
.
.
.
.
.
.....................................
.....................................
..
.
.
.
.
.
.
.
.
.
.
.
.
.
.
.
.
.
.
.
.
.
.
.
.
.
.
.
...
.
....... ...... ..............
....... ... ..............
.
.
.
.
.
.
..
.
.
.
.
.
.
.
.
......
......
... ..
...
..
....
....
.
.....
.
.
.
.
.
.
.
.
.
.
.
... ..
...
...
...
.
.
. ....
.
.
.
.
...
.
.
.
.
.
.
.
.
.
...
...
......
....
.
. ....
...
...
...
...
......
......
....
....
..
..
.....
.....
..... ......
........
........
...
.................... .......................
.................... ....................... ....................... ...........................
....
........
.......
.......
...
.
.
.
.
.
...
...
...
...
...
...
..
..
...
...
...
...
....
....
....
....
.
.
.
.
......
.
.
.
.
.
.
.
.
.
.
.
........
.
.........
...
................ .......................
........................................
......

13

8. Pr
edio misterioso - As guras mostram as plantas do 1o e 2o andares de
um predio que guarda segredos muito perigosos. Os 9 elevadores estao representados por letras e em cada letra podemos pegar o elevador ou continuar.
Qual o caminho mais curto da entrada ate a sada?
qqqqqqqqqqqqqqqqqqqqqqqqqqqqqqqqqqqqqqqqqqqqqqqqqqqqqqqqqqqqqqq
q
q
A B qqqq C qqqq
q
qqqqqqqqqqqqqqqqqqqqqqqqqqqqqqqqqqqqqqqqqqqqqqqqqqqqqqqqqqqqqqqqqqqq
qq
qq
qq
qq
qq E qqqq
qq D
qq
qq
qq
qq
qq
qq
qq
qq
qq
qq
qq
qqqqqqqqqqqqqqqqqqqqqqqqqqqqqqqq
qq
qq
q
qq
qq F qqq G qqqq H qqqq
qq
q
q
qqqqqqqqqqqqqqqqqqqqqqqqqqqqqqqqqqqqq
qqqqqqqqqqqqqqqq
qq
qq
qq
qq
qq
qq J
qq
q
qqqqqqqqqqqqqqqqqqqqqqqqqqqqqqqqqqqqqqqqqqqqqqqqqqqqqqqqqqqqqqqq

entrada

qqqqqqqqqqqqqqqqqqqqqqqqqqqqqqqqqqqqqqqqqqqqqqqqqqqqqqqqqqqqqqqqq
q
qq
A qqq B C qqqq
qq
q
qqqqqqqqqqqqqqqqqqqqqqqqqqqqqqqqqqq
qqqqqqqqqqqqqqqq
qq
q
qq D
E qqqq
qq
qq
qq
qq
q
q
q
q
q
q
q
q
q
q
q
q
q
q
q
q
q
q
q
q
q
q
q
q
q
q
q
q
q
q
q
q
q
q
q
q
q
q
q
q
q
q
qq
qq
qq
qq
q
q
qq F qq G q H qqq
qq
q
q
q
qqqqqqqqqqqqqqqqq
qq
qq
qq
qq
qq
qq
qq
qq J
sada
qqqqqqqqqqqqqqqqqqqqqqqqqqqqqqqqqqqqqqqqqqqqqqqqqqqqqqqqqqqqqq

OBMEP 2007

21

Nvel 1

Solucoes da Lista 4

Solu
co
es da Lista 4
1. Divis
ao de n
umeros decimais - Efetuando a divisao temos:
254880
144 177 10
254, 88
=
=
= 1440.
0, 177
177
177

2. C
alculo de porcentagem - A divisao de 58 por 84 e: 58 84 = 0, 69047...
Multiplicando por 100 temos que o percentual de acertos e 0, 69047 100 =
69, 047%, que e aproximadamente 69%.

3. Almo
co dos amigos - Os precos de um prato mais uma vitamina sao:

14 , 
16 , 
17 , 
18 , 
20 , 
20 , 
21 , 
23
13 , 

7+6

7+7

7+9

11+6

11+7

11+9

14+6

14+7

14+9

Dentre esses, os que diferem de 6 sao: 14 e 20 ou 17 e 23. Logo, temos duas


solucoes: Denise pode gastar 7 + 7 = 14 e J
ulio 14 + 6 = 11 + 9 = 20 ou Denise
gasta 11 + 6 = 17 e J
ulio 14 + 9 = 23.

4. Adi
c
ao de inteiros positivos Solu
c
ao 1 - Inicialmente observe que se a maior soma de tres desses n
umeros
e 9, entao todos os n
umeros tem que ser menores do que 7, ou seja:
1 , 2 , 3 , 4 , 5 , 6.
Por outro lado, se a menor soma e 6, entao eles tem que ser menores do que
5, logo restam:

1 , 2 , 3 , 4.
22

OBMEP 2007

Solucoes da Lista 4

Nvel 1

Vericamos que esses sao os n


umeros:
1 + 2 + 3 = 6 , 1 + 2 + 4 = 7 , 1 + 3 + 4 = 8 , 2 + 3 + 4 = 9.
Solu
c
ao 2 - Somando de tres em tres quatro n
umeros a, b, c e d temos os
n
umeros a + b + c, a + b + d, a + c + d e b + c + d. Logo,
6 + 7 + 8 + 9 = (a + b + c) + (a + b + d) + (a + c + d) + (b + c + d) = 3(a + b + c + d).
Donde, a + b + c + d =
10 6 = 4 ;

30
3

= 10. Portanto, os n
umeros procurados sao

10 7 = 3 ;

10 8 = 2 ;

10 9 = 1.

5. O passeio do Jorge - As guras ilustram o percurso que Jorge fez:


caminhando 32 m no incio, ele toca em 7 arvores e para a 2 m da u
ltima
que tocou;
voltando 18 m, ele toca em 4 arvores e para a 1 m da u
ltima que tocou;
ao retornar os 22 m ele toca em 5 arvores e para a 1 m da u
ltima que
tocou.
- ........- .......P
....s
..............................s
......................s
...............................s
.........................s
..
.
.
.....
....
....
....
...
....
...
....
....
....
....
....
....
...
....
...
................................................................................................................................

- ...........P
....s
..............................s
..............................s
...............................s
.....................s
..

.
.
.....
....
....
....
...
....
...
....
....
....
....
....
....
...
....
...
................................................................................................................................

s?

 .......
P
....s
..............................s
..............................s
...............................s
.........................s
..

s s?


6
s
6
s
s
-

.
.
.....
....
....
....
...
....
...
....
....
....
....
....
....
...
....
...
................................................................................................................................

s?

s s
s?
 

Assim, ele tocou em 7 + 4 + 5 = 16 arvores.

6. A descoberta do algarismo - Separando os n


umeros cujos quadrados tem
1, 2 e 3 algarismos temos:
1 algarismo:

1, 2, 3

2 algarismos: 4, 5, 6, 7, 8, 9
3 algarismos: 10, 11, 12, . . . , 31
OBMEP 2007

23

Nvel 1

Solucoes da Lista 4

Ate 312 a seq


uencia tem 3 + 12 + 66 = 81 algarismos.
12 , 22 , 32 , 42 , . . . , 92 , 102 , . . . , 312
      


13 algs

26=12 algs

322=66 algs

Assim, faltam 100 81 = 19 algarismos para o 100o . Como 19 = 4 4 + 3,


teremos mais 4 n
umeros de 4 algarismos cada um, que sao 322 , 332 , 342 e 352 ,
e mais os 3 algarismos (milhar, centena, dezena) do n
umero: 362 = 1296.

9 6
12 , 22 , 32 , 42 , . . . , 92 , 102 , . . . , 312 , 322 , 332 , 342 , 352 , 12 
      

 


o
13 algs

26=12 algs

322=66 algs

44=16 algs

100 alg

Logo, o n
umero e 9.

7. OBMEP - Como
peso de X + peso de O = 13

peso de Z + peso de O = 9,

segue que
peso de X = peso de Z + 4.
Logo, as opcoes para os pesos de Z e de X sao:
1e5 ,

2e6 ,

3 e 7.

Por outro lado, temos:


peso de M + peso de P = 6

peso de B + peso de E = 6 .

Logo, os pesos de M, P, B e E sao todos menores do que 6, ou seja:


1 , 2 , 3 , 4 , 5.
Alem disso, nenhum deles pode ter peso 3 g.
Conclumos que os pesos de Z e de X sao 3 e 7, o que nos da o peso de O
igual a 6. Assim, temos:
peso de O + peso de B + peso de E + peso de M + peso de P = 6 + 6 + 6 = 18.
24

OBMEP 2007

Solucoes da Lista 4

Nvel 1

8. Pr
edio misterioso - Primeiro observamos que os elevadores A, C, D, E,
F e H conduzem a quartos fechados em algum dos dois andares e, portanto,
nao levam a` sada. Assim, desconsiderando os elevadores mencionados, nosso
desenho de elevadores u
teis e o seguinte
qqqqqqqqqqqqqqqqqqqqqqqqqqqqqqqqqqqqqqqqqqqqqqqqqqqqqqqqqqqqqqq
q
qq
qq
B qqqq
q
q
qqqqqqqqqqqqqqqqqqqqqqqqqqqqqqqqqqqqqqqqqqqqqqqqqqqqqqqqqqqqqqqqqqqq
qq
qq
qq
qq
qq
qq
qq
qq
qq
qq
qq
qq
qq
qq
qq
qq
qq
qq
qq
qqqqqqqqqqqqqqqqqqqqqqqqqqqqqqqq
qq
qq
qq
qq
qq
qq G qqq
qq
qq
q
q
q
q
qqqqqqqqqqqqqqqqqqqqqqqqqqqqqqqqqqqqq
qqqqqqqqqqqqqqqqq
qq
qq
qq
qq
qq
qq J
q
qq
qqqqqqqqqqqqqqqqqqqqqqqqqqqqqqqqqqqqqqqqqqqqqqqqqqqqqqqqqqqqqqqq

entrada

qqqqqqqqqqqqqqqqqqqqqqqqqqqqqqqqqqqqqqqqqqqqqqqqqqqqqqqqqqqqqqqqq
qq
qq B
qq
qq
q
qq
qq
qqqqqqqqqqqqqqqqqqqqqqqqqqqqqqqqqqq
qqqqqqqqqqqqqqqq
qq
qq
qq
qq
qq
qq
qq
qq
qqqqqqqqqqqqqqqqqqqqqqqqqqqqqqqqqqqqqqqqqqqqqq
qq
qq
qq
qq
qq
qq G qq
qq
qq
qq
qq
qq
q
qq
qq
q
q
q
q
q
q
q
q
q
q
q
q
q
q
q
qq
qq
q
qq
qq J
sada
qqqqqqqqqqqqqqqqqqqqqqqqqqqqqqqqqqqqqqqqqqqqqqqqqqqqqqqqqqqqqq

Assim, o caminho adequado ca evidente: primeiro pegar o elevador B, depois


o J e por u
ltimo o G.

OBMEP 2007

25

Nvel 1

Lista 5

Lista 5
1. Soma de fra
c
oes - Qual e o valor de

1
1
1
1

?
10 100 1000 10000

2. Biblioteca - A biblioteca de uma escola comprou 140 novos livros, cando


com

27
25

de livros. O n
umero de livros antes da compra, e:

(a) 1750

(b) 2500

(c) 2780

(d) 2140

(e) 1140

3. Compara
c
ao de fra
c
oes - Quantas fracoes menores do que 1 existem, tais
que o numerador e denominador sao n
umeros naturais de um algarismo?

4. Divis
ao com resto - Quantos sao os n
umeros que ao dividir 2007 deixam
resto 5?

5. Panelas - Uma panela pesa 645 g e outra 237 g. Jose divide 1 kg de carne
entre as duas panelas, de modo que as duas com seus conte
udos cam com o
mesmo peso. Quanto ele colocou de carne em cada panela?

6. Domin
os - Juliana representou uma multiplicacao com 5 dominos. Seu irmao
Bruno trocou dois dominos de posicao e agora a multiplicacao cou errada.
Troque a posicao de dois dominos para que a multiplicacao que correta novamente.
26

OBMEP 2007

Lista 5

Nvel 1
........................t..............................
.. t ...
...
.
... t ....
t ...
...
.. t
.
................................................

.................................t........t........t.....
..
...
...
.
... t ....
... t t t.....
..
................................................

......................................................
...
..
t ..
... t .... t ....
..
.
.
...................................................
.......t....................
.
..
.. t ...
.
t.
.................................t................t..... ........t................t.....
.
.
... t t ..
.. t .... ... t t...
...
.. t t .. t t.. .. t t..
..................................................... ...........................

3212
3
.............................................................
16456

7. C
odigo secreto - Antonio tem que descobrir um codigo de 3 algarismos
diferentes A B C. Ele sabe que B e maior que A, que A e menor do que C e
ainda:
.....................................................
.
.
.

.....................................................
...
...
...
....
..
..
......................................................

.....................................................
.
.
.

.............................................................................
.
.
.
.

...
...
...
...
...
...
...
...
...
.
...
.
.
.
.
.
.
.
B B + ..............A
. A ... + .... C ..... C .... = ..... 2 .... 4 .... 2 ....
...........................................
..............................................
....................................................................

..............................
....
....
..
...
.........................

............................

.............................................................................

............................

.
.
.
.
.
.
.....
....
....
...
...
...
B .................A
0.......
.. .... C .... = .... 3 .... 6 ....
.......................................................................
........................
.................

Qual e o codigo que Antonio procura?

8. Os doze pontos - Doze pontos estao marcados numa folha de papel quadriculada, conforme mostra a gura.

s
s

s
s
s
s

s
s
s
s

s
s

Qual o n
umero maximo de quadrados que podem ser formados unindo quatro
desses pontos?

OBMEP 2007

27

Nvel 1

Solucoes da Lista 5

Solu
co
es da Lista 5
1. Soma de fra
c
oes Solu
c
ao 1: Transformando as fracoes em n
umeros decimais temos:

1
1
1
909
1

= 0, 1 0, 01 + 0, 001 0, 00001 = 0, 0909 =


.
10 100 1000 10000
10000
Solu
c
ao 2: Efetuando temos:
1
1
1
1000 100 + 10 1
909
1

=
=
.
10 100 1000 10000
10000
10000
2. Biblioteca - Ao comprar 140 livros a biblioteca cou com
livros, logo 140 corresponde a

2
25

27
25

do n
umero de

dos livros da biblioteca. Logo, temos:

2
140
25
1
140 2 = 70
25
25
70 25 = 1750.
25
A opcao correta e (a).

3. Compara
c
ao de fra
c
oes - Para que uma fracao seja menor do que 1, o
numerador tem que ser menor do que o denominador. As fracoes sao:
1
2
1
2
com denominador 3:
e
3
3
2
3
1
,
e
com denominador 4:
4
4
4


com denominador 2:

1/2

28

OBMEP 2007

Solucoes da Lista 5

Nvel 1

2
3
4
1
,
,
,
5
5
5
5
1
3
4
5
2
com denominador 6:
,
,
,
,
6
6
6
6
6



com denominador 5:

1/3

1/2

2/3

2
3
4
5
6
1
,
,
,
,
,
7
7
7
7
7
7
1
3
5
7
2
4
6
com denominador 8:
,
,
,
,
,
,
8
8
8
8
8
8
8



com denominador 7:

1/4

com denominador 9:

1/2

3/4

1
4 5
7
2
3
6
8
,
,
,
,
,
,
,
9
9
9
9 9
9
9
9


1/3

2/3

Temos entao 27 fracoes.

4. Divis
ao com resto - Se um n
umero ao dividir 2007 deixa resto 5, entao
esse n
umero e um divisor de 2007 5 = 2002. Logo, temos que calcular os
divisores de 2002:
...
...
...
...
...
...
...
...
...
...
...
...
...
...
...
...
...
...
...
...
...
...
...
...
...
...
...
...
..

1
2002 2 2
1001 7 7 , 14
143 11 11 , 22 , 77 , 154
13 13 13 , 26 , 91 , 182 , 143 , 286 , 1001 , 2002
1
...
...
...
...
...
...
...
...
...
...
...
...
...
...
...
...
...
...
...
...
...
...
...
...
...
..

Logo, os n
umeros que ao dividirem 2007 deixam resto 5 sao:
1, 2, 7, 11, 13, 14, 22, 26, 77, 91, 143, 154, 182, 286, 1001, 2002

5. Panelas - Convertendo quilo para gramas temos que 1 kg = 1 000 g. As duas


panelas mais a carne pesam juntas
645 + 237 + 1 000 = 1 882 g.
OBMEP 2007

29

Nvel 1

Solucoes da Lista 5

Logo, cada panela mais o seu conte


udo de carne deve pesar 1882 2 = 941 g.
Logo, Jose colocou em cada panela, respectivamente,
941 645 = 296 g e 941 237 = 704 g
.

6. Domin
os - Dado que 2 3 = 6, suporemos por enquanto que os dominos
q qq

.................................................
...
...
...
...
...
....
...
...
.
.................................................

q qqq
qq qqq

.................................................
...
...
...
...
...
....
...
...
.
.................................................

estao na posicao certa. Caso isso seja verdade, dado que

1 3 = 3 temos que o algarismo na dezena do resultado e tres, logo temos que


trocar o domino

q q

.................................................
...
...
...
...
...
....
...
...
.
.................................................

qq

pelo domino

q q qqq
q q q q,

.................................................
...
...
...
...
...
....
...
...
.
.................................................

de tal forma que o 3 que na

dezena. Dado que temos um 2 na centena do resultado, entao na centena do


primeiro n
umero tem que ter um 4. Assim, o produto certo ca da forma
............................................................
..
...
t ....
... t ....
...
... t
...
..
........................................................
.......t.......................
.
..
.. t ...
.
..
t...
..
.......................................................t...... .......t..................t.....
. .
... t
...
..
...
... t .... ... t
t..
...
. .
.
.
........................t..............t........................ ........t..................t......

........t..................t..................................
t ..
...
.. t
...
... t ....
.
.
... t
.. t
t ....
t
........................................................

.....................................t.........t.........t.....
...
...
.
..
.. t ....
...
.
...
..
.. t t t....
......................................................

5412
3
.....................................................................
16236

7. C
odigo secreto - A u
nica maneira de obter 360 como produto de tres
n
umeros de um algarismos cada um e
360 = 9 8 5 .
Logo, a soma AA + BB + CC e igual a 55 + 88 + 99. Como A e menor do
que B e do que C , temos que A = 5. Logo, temos duas possibilidades para o
codigo: 589 ou 598.
30

OBMEP 2007

Solucoes da Lista 5

Nvel 1

8. Os doze pontos - No total, temos 11 possveis quadrados como mostrado a


seguir.
5 quadrados
....s
......................s
...
...
...
...
...
...
.
.
s
s
....................................................s
......................s
...
...
...
...
...
...
...
...
...
...
...
...
....s
.........................s
.........................s
......................s
....
....
...
...
...
.
..s
......................s

4 quadrados
s
s
s
s

s
s
s
s

....
.....
..... ..... ..... .....
..... ................. .........
.....
.....
.. ..
.....
.....
..... ..........
.
.
.
.
.
........
..... ................. ................. .........
.........
.........
.........
.
.
.
.
.
.
.
.
.... ........ ........ ........ ........ .........
.
.
.
.
..
........
....
........
..... ...........
.....
....
.... ....
.....
.....
..... ................. .........
..... ..... ..... .....
.......
.......

s
s

OBMEP 2007

s
s

2 quadrados
s
s
s
s

s
s
s
s

............................
... ..............................
...........
..........
........
.............
...............
... ...
......
....
....
.
..
. ..
.
.....
.
.. ........
..............
.
.
..........
.........
.
...........
.
.
.
.
... ..............................
............ .............

s
s

s
s

31

Nvel 1

Lista 6

Lista 6
1. Rel
ogio - O grande relogio de parede da escola marca a data (dia, mes e ano)
e as horas (horas e minutos) como na gura. Que dia, mes e ano esses mesmos
10 algarismos da gura voltarao a aparecer juntos no relogio pela primeira
vez?
............................................................
...........
..............
..........
.........
.........
.......
.......
.......
.
.
.
.
.
.
......
....
.
.
.
......
.
.
....
....
.
.
....
...
.
.
....
..
.
....
.
...
...
.
...
..
.
...
..
.
...
..
.
...
....
...
...
...
..
...
..
...
..
..
...
..
..
..
..
..
..
...
.
.
.
.
.
.
.
.
.
.
.
.
.
.
.
.
.
.
.
.
.
.
.
.
.
.
.
.
.
.
.
.
.
.
.
.
.
.
.
.
.
.
.
.
.
.
.............................................
.
.
.
.
...
...
.....
.....
....
....
...
...
...
....
....
...
...
..
....
....
...
...
...
..
.
.
.
.
.
.
.
.
.
.
.
.
.
.
...
.....
.....
.....
.....
...
...
....
..
....
..
....
...
................................................
.........................................
....
...
....
....
.
.
....
....
.....
...
......
......
.......
......
.......
.......
.
.
.
.
.........
.
.
.......
..........
.............
..........
.................................................................

28 05 94
14 h 00

2. L
apis - Em 13 caixas foram embalados 74 lapis. Se a capacidade maxima de
cada caixa e de 6 lapis, qual e o n
umero mnimo de lapis que pode haver em
uma caixa?
(a) 1

(b) 2

(c) 3

(d) 4

(e) 6

3. Contagem - Se o algarismo 1 aparece 171 vezes na numeracao das paginas


de um livro, quantas paginas tem o livro?

4. Viagem a Recife - Em meu voo para Recife, quando fui receber a medalha
de ouro que conquistei na OBMEP, as seguintes informacoes apareceram na
tela da cabine de passageiros:
32

OBMEP 2007

Lista 6

Nvel 1
Velocidade media:
Distancia do local de partida:
Tempo de chegada a Recife:

864 km/h
1 222 km
1 h 20 min

Se o aviao manteve a mesma velocidade, entao qual e, aproximadamente, a


distancia de Recife a` cidade onde tomei esse voo?
(a) 2 300 km (b) 2 400 km (c) 2 500 km (d) 2 600 km (e) 2 700 km

5. Pra
ca - Maria e Joao dao uma volta completa na praca juntos, contando
as casas que cam em volta da praca. Eles comecaram a contar as casas em
pontos diferentes. A quinta casa da Maria e a decima segunda do Joao e a
quinta casa do Joao e a trigesima da Maria. Quantas casas tem em volta da
praca?

6. Seq
u
encia de figuras - As guras , , , , , 2 sao repetidas na seq
uencia

, , , , , 2, , , , , , 2, . . .
(a) Que gura aparecera na 1000a posicao da seq
uencia?
(b) Em qual posicao aparece o milesimo ?

7. A brincadeira do quadrado - Um quadrado de 1 m de lado foi cortado, com


cortes paralelos aos seus lados, em quadradinhos de 1 mm de lado. Colocandose lado a lado os quadradinhos, sem superposicao, formou-se um retangulo de
1 mm de largura. Qual o comprimento desse retangulo?

OBMEP 2007

33

Nvel 1

Lista 6

8. O c
odigo da Arca do Tesouro - Simao precisa descobrir um n
umero que
e o codigo da Arca do Tesouro que esta escondido na tabela.
5

Para descobrir o codigo ele tem que formar grupos de 3 algarismos que estao
em casas sucessivas, na horizontal ou na vertical, cuja soma e 14. Retirados
esses grupos, o codigo e a soma dos n
umeros que nao aparecem nesses grupos.
Qual e esse codigo?

34

OBMEP 2007

Solucoes da Lista 6

Nvel 1

Solu
co
es da Lista 6
1. Rel
ogio - Vamos tentar uma data e um horario no mesmo ano de 94. Ja que
com os n
umeros dados nao podemos alterar o dia nem para 29 nem para 30
sem alterar o ano, entao a data procurada nao esta no mes 05. O seguinte mes
possvel e o 08. Como precisamos da data mais proxima possvel, observemos
que podemos formar o dia 01 sobrando os n
umeros 0, 2, 4 e 5 para formar
a hora. A menor hora possvel que podemos formar com esses algarismos e
02 : 45, logo a data procurada e 1 de agosto de 1994 a`s 2 horas e 45 minutos.
................................................................
.............
..........
..........
.........
........
.......
.
.
.
.
.
.
.......
...
......
......
.....
.....
.
.
.
.
....
..
.
.
....
.
..
.
....
.
.
..
....
.
.
.
...
...
.
...
..
.
...
..
...
.
..
...
.
...
....
...
...
..
...
..
..
...
..
...
.
..
..
..
..
..
.
.
..........................................
..............................................
...
.
.
.
.
.
...
.
..
..
..
...
.
.
.
.
.
.
.
.
.
.
.
.
.
...
.....
.....
.....
.....
...
...
...
....
....
....
....
...
..
....
....
....
....
...
..
.
...
....
...
....
....
.
.
....
..................................................
.............................................
....
....
....
....
....
....
...
.
.
.....
.
......
......
.......
......
.......
.......
.........
........
..........
.........
.
.
.
.
.
.
.
..............
.
.
.
...........................................................

28 05 94

................................................................
..........
.............
.........
..........
........
.......
.
.
.
.
.
.
.......
...
......
......
.....
.....
.
.
.
.
....
..
.
.
....
.
..
.
....
.
.
..
....
.
.
.
...
...
.
...
..
...
.
..
...
.
..
...
.
...
....
...
...
..
...
..
..
...
...
...
.
..
..
..
..
..
.
.
..........................................
..............................................
...
.
.
.
.
.
...
.
..
..
..
...
.
.
.
.
.
.
.
.
.
.
.
.
.
...
.....
.....
.....
.....
...
...
...
....
....
....
....
...
..
....
....
....
....
...
..
.
...
...
....
....
.
....
.
....
..................................................
.............................................
....
....
....
....
....
....
...
.
.
.....
.
......
......
.......
......
.......
.......
.........
.......
..........
.........
.
.
.
.
.
.
.
..............
.
.
.
...........................................................

01 08 94

14 h 00

14 h 45

2. L
apis - Vamos ver em quantas caixas podemos colocar o n
umero maximo de
lapis, que e 6 por caixa. Nas 13 caixas nao e possvel, pois 13 6 = 78, que
e maior do que o n
umero de lapis 74. Em 12 caixas teramos: 12 6 = 72.
Assim, sobraria uma caixa com 74 72 = 2 lapis. Logo, a opcao correta e (b).

3. Contagem - A cada 10 paginas aparece 1 nas unidades e a cada 100 paginas


aparece 10 vezes o n
umero 1 nas dezenas.
Contando o n
umero de paginas que contem o algarismo 1 em cada faixa abaixo
temos:
OBMEP 2007

35

Nvel 1

Solucoes da Lista 6

20 paginas entre 1-99:


1, 11, 21, 31, 41, 51, 61, 71, 81, 91: 10 (1 na unidade)
10, 11, 12, 13, 14, 15, 16, 17, 18, 19: 10 (1 na dezena)
120 paginas entre 100 - 199:
101, 111, 121, 131, 141, 151, 161, 171, 181, 191: 10 (1 na unidade)
110, 111, 112, 113, 114, 115, 116, 117, 118, 119: 10 (1 na dezena)
100, 101, 102, . . . , 199: 100 (1 na centena)
20 paginas entre 200-299:
201, 211, 221, 231, 241, 251, 261, 271, 281, 291: 10 (1 na unidade)
210, 211, 212, 213, 214, 215, 216, 217, 218, 219: 10 (1 na dezena)
Ate a pagina 299 temos 20 + 120 + 20 vezes que aparece o n
umero 1, faltando assim apenas 171 160 = 11 uns, que seriam os 2 primeiros que aparecem na unidade de 301, 311 e os 9 primeiros que aparecem nas dezenas de
310, 311 , 312 , 313 , 314 , 315 , 316 , 317 , 318. Logo, o livro tem 318 paginas.

4. Viagem a Recife - No momento em que a informacao foi dada, o tempo que


faltava de voo era de 1h 20 min, ou 4/3 h. Logo, nesse momento, a distancia
a Recife era de 864 43 = 1 152 km. Desde que estavamos a 1 222 km da cidade
de partida, a distancia entre essa cidade e Recife e de 1 152 + 1 222 = 2 374 km.
Dentre as opcoes, a mais proxima e 2 400 km, ou seja, a opcao (b).

5. Pra
ca - Como a 5a casa da Maria e a 12a casa do Joao, a diferenca entre as
contagens e de 7 casas. Assim, a 1a casa da Maria e a 8a casa do Joao e a 5a
casa do Joao corresponde a duas casas antes da casa que a Maria comecou a
contar. Mas, como a 5a casa do Joao e a 30a da Maria, entao a praca tem 32
36

OBMEP 2007

Solucoes da Lista 6

Nvel 1

casas: as 30 casas que Maria ja contou mais as 2 casas que faltam para Maria
chegar ao ponto onde comecou a contar.

6. Seq
u
encia de figuras - As guras se repetem de 6 em 6. Dividindo 1000
por 6 temos: 1000 = 6 166 + 4.
1000o
996o


, , , , , 2, . . . , , , , , , 2 , , , , , , 2, . . .






1o grupo de 6
166o grupo de 6

(a) A gura que ca em 1000o lugar e .


(b) O primeiro esta na 3a posicao, o segundo na posicao de n
umero 3 + 6,
o terceiro em 3 + 6 + 6, e assim por adiante, como indicado a seguir:
1o

3+06

2o

3+16

3o

3+26

4o
..
.

3+36

1000o 3 + 999 6
Logo, o 1000o aparece na posicao: 3 + 999 6 = 5997.

7. A brincadeira com o quadrado Solu


c
ao 1 - Convertendo metros em milmetros temos: 1 m = 1000 mm.
Assim, o quadrado cou dividido em 1000 1000 = 106 quadradinhos de lado
1 mm cada um. Colocando-se lado a lado os 106 quadradinhos, teremos um
retangulo de comprimento

1 + 1 + + 1 = 106 1 = 106 mm


106 parcelas

OBMEP 2007

37

Nvel 1

Solucoes da Lista 6

.
Solu
c
ao 2 - O quadrado tem area igual 1 m2 = 106 mm2 . A area do
retangulo e a mesma do quadrado. Como a largura do retangulo e  = 1 mm
temos que o comprimento c em milmitros e

c=

106
=
= 106 mm.

1

8. O c
odigo da Arca do Tesouro - Nas seguintes duas tabelas mostramos
unicamente os n
umeros cuja soma e 14, horizontalmente e verticalmente, respectivamente.

7
8

4
7

1
6

Assim, quando eliminamos esses n


umeros da tabela inicial, os n
umeros que
sobrevivem sao:
5

4
8
2
Portanto, a soma dos n
umeros que cam e 5 + 4 + 6 + 4 + 8 + 2 = 29.

38

OBMEP 2007

Lista 7

Nvel 1

Lista 7
(0, 2)3 + 1
1. Opera
c
oes com decimais - Efetue
0, 2 + 1

2. Fatores inteiros - Decompor 96 em dois fatores inteiros cuja soma dos


quadrados seja 208.

3. Divisibilidade - No n
umero 6a78b, a e o algarismo da unidade de milhar e
b e o algarismo da unidade. Se 6a78b e divisvel por 45, entao o valor de a + b
e:
(a) 5

(b) 6

(c) 7

(d) 8

(e) 9

4. N
umero simples - Um n
umero inteiro positivo e denominado simples se
ele tem apenas os algarismos 1 ou 2 (ou ambos). Quantos n
umeros simples
existem inferiores a um milhao?

5. O ret
angulo do Lus - Lus desenhou um retangulo de 6 cm por 10 cm, e
quer dividi-lo em quatro partes. Cada parte tem area, respectivamente, 8 cm2 ,
12 cm2 , 16 cm2 , 24 cm2 . Desenhe como ele pode fazer essa divisao.

6. Venda de TV - O gerente de uma loja foi vericar qual tinha sido o preco
de venda em 2006 de uma televisao da marca VejoTudo. Encontrou uma
fatura meio apagada, onde se lia: lote de 72 TVs da VejoTudo vendido
por R$ . . . 679 . . . reais, onde os algarismos da unidade e da dezena de milhar
estavam ilegveis. Qual foi o preco de venda em 2006 de cada uma dessas
televisoes?
OBMEP 2007

39

Nvel 1

Lista 7

7. Chocolate - Henrique comprou barras de chocolate por R$1, 35 cada uma.


Ele pagou com uma nota de R $10, 00 reais e recebeu de troco menos do que
R $1, 00. Quantas barras ele comprou?

40

OBMEP 2007

Solucoes da Lista 7

Nvel 1

Solu
co
es da Lista 7
1. Opera
c
oes com decimais - Temos:
(0, 2)3 + 1
0, 008 + 1
1, 008
=
=
= 0, 84
0, 2 + 1
1, 2
1, 2

2. Fatores inteiros - No Exerccio 7 da Lista 2, encontramos os fatores positivos 8 e 12. As duas possibilidades sao: 8 e 12 ou 8 e 12.

3. Divisibilidade - O n
umero e divisvel por 5 e 9.
Todo n
umero divisvel por 5 termina em 0 ou 5. Assim, b = 0 ou b = 5.
Todo n
umero divisvel por 9 tem como a soma dos seus algarismos um n
umero
m
ultiplo de 9.
Logo, temos que 6 + a + 7 + 8 + 0 = 21 + a ou 6 + a + 7 + 8 + 5 = 26 + a
sao m
ultiplos de 9. Donde, a = 6 ou a = 1, respectivamente. Da temos:
a + b = 6 + 0 = 6 ou a + b = 1 + 5 = 6.

4. N
umero simples - Se o n
umero e menor do que um milhao, entao ele tem 6
algarismos. Para cada posicao deste n
umero temos duas possibilidades: 1 ou
2. Como sao 6 posicoes temos 26 = 64 n
umeros simples.

5. O ret
angulo do Lus - Como 24 = 4 6, entao ele construiu o primeiro
retangulo, tirando 4 cm do lado de 10 cm, sobrando um quadrado de lado
6 cm. Sendo 16 = 4 4, ele construiu um quadrado de lado 4 cm sobrando
dois retangulos de areas (6 4) 4 = 8 cm2 e (6 4) 6 = 12 cm2 , como, por
exemplo, a divisao mostrada na gura ao lado.
OBMEP 2007

41

Nvel 1
4

4
2

Solucoes da Lista 7

................................................................................................................................................................
....
....
....
....
....
....
....
....
....
....
....
...
....
....
...
...
....
....
....
...
....
....
....
....
....
....
....
....
....
....
....
....
....
....
...
...
....
.................................................................................................
....
....
....
....
....
....
....
....
....
....
...
....
...........................................................................................................................................................

6. Venda de TV - Sejam a o algarismo da dezena de milhar e b o da unidade.


Como o n
umero e divisvel por 72 = 8 9 temos que 79b e um n
umero par
divisvel por 8. Testando os valores de b = 0, 2, 4, 6 e 8, vemos que b = 2.
Um n
umero e divisvel por 9 se a soma dos seus algarismos e um m
ultiplo de
9. Entao, a + 6 + 7 + 9 + 2 = a + 24 e um m
ultiplo de 9. Logo, a = 3. Assim,
cada TV custou: 36792 72 = 511 reais.

7. Chocolate - Como 8 1, 35 = 10, 8 e maior do que 10, entao ele comprou


7 barras de chocolate e recebeu de troco: 10 7 1, 35 = 0, 55 reais ou 55
centavos.

42

OBMEP 2007

Lista 8

Nvel 1

Lista 8
1. O quadradinho - Qual o valor de  em

6 400 000
= 1, 6  ?
400

2. Dois n
umeros - O produto de dois n
umeros de dois algarismos cada um e
1728. Se e o maximo divisor comum (mdc) deles e 12, quais sao esses n
umeros?

3. As idades dos irm


aos - No dia de seu aniversario de 7 anos, 13 de marco
de 2007, uma 3a -feira, Carlos disse a seu irmao: A contar de hoje, faltam
2000 dias para voce completar 15 anos. Em que dia da semana vai cair o
aniversario do irmao de Carlos?. Quantos anos tera Carlos nesse dia?

4. A mistura de concreto - Uma certa mistura de concreto e feita de cimento,


areia e terra na razao 1 : 3 : 5 por quilo. Quantos quilos dessa mistura pode
ser feita com 5 quilos de cimento?
(a) 13 13

(b) 15

(c) 25

(d) 40

(e) 45

5. Ponto na escala - A que n


umero corresponde o ponto P na escala abaixo?

12, 44

12, 62

6. O pomar do Francisco - O pomar do Francisco tem macieiras, pereiras,


laranjeiras, limoeiros e tangerineiras, dispostas em cinco las paralelas, cada
uma com uma u
nica variedade de arvores, da seguinte maneira:
OBMEP 2007

43

Nvel 1

Lista 8

as laranjeiras estao do lado dos limoeiros;


as pereiras nao estao do lado das laranjeiras nem dos limoeiros;
as macieiras estao do lado das pereiras, mas nao dos limoeiros, nem das
laranjeiras.
Em que la estao as tangerineiras?
(a) 1a

(b) 2a

(c) 3a

(d) 4a

(e) 5a

7. Quatro quadrados - Quatro quadrados iguais, com 3 cm2 de area cada um,
estao superpostos formando a gura abaixo. Qual e a area dessa gura?

.........................................................................................................
...
...
..
...
....
....
...
..
.........................................................................................................
............................................................................................................
....
...
...
..
..
.
.....
.
.....
...
...
....
....
...
...
...
...
...
...
.....
....
.....
....
...
...
.
.
.
...
.
.
.
.
.
...
...
....
....
.....
....
...
...
...
...
...
...
...
...
.
...
...
...
...
...
.
.
.
.....
...
...
...
...
...
...
...
.
.
.
.
...........................................................................................................
..
..
...
.
.
.
.
.
....
.
.
...
...
...
...
...
...
...
...
.
.
.
.
.
.
...
...
...
.....
.....
.....
....
.....
...
...
...
..
..
..
.
...
...
...
.
.
.
.
.....
.........................................................................................................
...
...
...
...
...
...
.
...
...
...
.
...
....
...
...
....
...
...
...
...
...
..
.
...
.
...
...
.
.
.
.
........................................................................................................
...
........................................................................................................
.
.
...
.
.
...
...
...
...
...
...
...
...
...
...
...
...
...
...
...
...
...
.
......................................................................................................

8. O fio de arame - Com um o de arame Ernesto formou a gura abaixo.


..............
..................
... .....................
...................
....................
..... ......................
.........

Qual das guras abaixo ele pode formar com o mesmo o de arame, cortando
ou nao o o?
............
(a) ..q..................................................................................................q

44

(b)

............................
...
.
......
....
.
..........................

............
............
............
(c) ..q.................... .......................... .......................... ..........................q (d)

OBMEP 2007

................
..........................
...
...
..........................
.....................
.................

(e)

................
............................
.......... ..........
......................
.....................
.................

Lista 8

Nvel 1

9. Quantos fosforos sao necessarios para formar o oitavo termo da seq


uencia,
cujos tres primeiros termos sao mostrados abaixo?
rr r r
r
r

................................
... .........
...
..
.... ...........
. ....
....
....................................

(a) 21

(b) 24

(c) 27

rr rr r r
r
r r

.......................................................
... .........
...
...
..
...
.... ...........
. ....
....
..
............................................................

rr rr rr r r p p p
r
r r r

...............................................................................
...
..
..
.. ......
...
....
....
.... ..............
..
..
..
.. .......
...............................................................................

(d) 30

OBMEP 2007

(e) 34

45

Nvel 1

Solucoes da Lista 8

Solu
co
es da Lista 8
1. O quadradinho - Por simplicacao
6 400 000
= 1, 6 
400

6 400 000
= 16 000, logo:
400
=

16 000 = 1, 6 .

Segue que  = 10000.

2. Dois n
umeros - Como 12 e o maior divisor comum dos dois n
umeros, ambos
sao m
ultiplos de 12, logo estao dentre os n
umeros
12 , 24 , 36 , 48 , 60 , 72 , 84 , 96 , 108 , 120 , 132 , 144 , . . .
Da lista acima, temos tres u
nicas possibili-

12 = 22 3, 144 = 24 32

dades:

mdc(12 , 144) = 22 3

12 144 = 1728 e mdc (12 , 144) = 12


24 72 = 1728 e

mdc (24 , 72) = 24

36 48 = 1728 e

mdc (36 , 48) = 12

72 e m
ultiplo de 24,
mdc(24 , 72) = 24
36 = 22 32 e 48 = 24 3,

Logo, temos duas solucoes: 12 e 144, ou 36

mdc(36 , 48) = 22 3

e 48.
3. As idades dos irm
aos - Dividindo 2000 por 7 obtemos 2000 = 7 285 + 5.
Logo, 2000 dias equivalem a 285 semanas mais 5 dias. Como o dia 13 de marco
de 2007 caiu em uma terca-feira, contando os 5 dias restantes, temos que o
aniversario do seu irmao caira em um domingo.
Agora, dividindo 2000 por 365 obtemos 2000 = 365 5 + 175. Logo, 2000 e,
aproximadamente, igual a cinco anos e meio, portanto Carlos tera 12 anos de
idade.

46

OBMEP 2007

Solucoes da Lista 8

Nvel 1

4. A mistura de concreto - De acordo com os dados do problema temos :


cimento
1 kg

areia

3 kg

terra

5 kg

Logo, com 5 kg de cimento temos:


cimento

areia

terra

1 kg 5 3 kg 5 5 kg 5
Assim, com 5 quilos de cimento essa mistura tem 5 + 15 + 25 = 45 kg.

5. Ponto na escala - A distancia entre os pontos inicial e nal e de: 12, 62


12, 44 = 0, 18. Como estao marcados 18 intervalos, o comprimento de cada
um deles e de 0, 18 18 = 0, 01.

12, 44

12, 62

O n
umero P esta na 6a posicao a` direita de 12, 44. Assim, o ponto P vale:
12, 44 + 0, 01 6 = 12, 50.

6. O pomar de Francisco - Podemos observar que temos os dois pares de


arvores:
laranjeiras e limoeiros,
macieiras e pereiras,
que nao sao vizinhos. Como 5 = 2 + 1 + 2, temos que as tangerineiras estao
na 3a la.

OBMEP 2007

47

Nvel 1

Solucoes da Lista 8

7. Quatro quadrados - Se a area de cada quadrado e 3 cm2 e cada um deles


esta dividido em 16 quadradinhos, entao a area de cada quadradinho e

3
16

cm2 .

Como os 4 quadrados se superpoem em 6 quadradinhos, temos que a area da


gura e:
436

3
9
= 12 = 10, 875 cm2 .
16
8

.........................................................................................................
...
...
...
...
....
....
...
..
.
.........................................................................................................
............................................................................................................
.
.
.
...
...
.....
.....
.....
.....
...
...
...
...
...
...
...
...
...
...
....
....
....
....
...
...
.
.
.
...
.
.
.
...
...
.....
.....
.....
....
...
...
...
...
...
...
...
...
...
...
...
...
....
....
.
.
...
..
...
...
.
...
...
...
...
...............................................................................................................
.
..
...
...
.
.
.
.
...
.
...
...
...
...
...
...
...
...
.
.
.
.
.
...
..
...
..
..
..
...
.
.
.
....
.
.
...
.
.
.
...
...
...
...
...
...
...
...
...
.
.
.
.
.
...
.......................................................................................................
...
...
....
.
.
.
...
...
..
...
...
.
.
.
.....
...
...
...
....
....
...
...
...
.
...
..
.
...
...
.
.
..
.
.
.
...
.........................................................................................................
.......................................................................................................
.
...
.
.
...
...
...
...
...
...
...
...
...
...
...
...
...
...
...
...
.
.........................................................................................................

8. O fio de arame - A gura e composta de 3 semicrculos, o que exclui as


opcoes (b), (c) e (e), e 4 segmentos de reta. A opcao (a) so tem 3 segmentos,
logo a opcao correta e (d).
Observacao: Esse exerccio usa uma certa informalidade, pois para decidirmos entre as opcoes (a) e (d), estamos admitindo que cada segmento de reta
na gura tem o comprimento do diametro dos crculos.

9. Observe que o n
umero de fosforos da seq
uencia e formado da seguinte maneira:
1o termo =

3+3=23

2o termo =

3+3+3=33

3o termo =

3+3+3+3=43

uencia e: (8 + 1) 3 = 27.
Logo, o 8o termo da seq
rr r r
r
r

................................
. ...
...
..... ..............
....
... .........
...
................................

48

rr rr r r
r
r r

.......................................................
. ...
...
...
..... ..............
...
....
...
... .........
...
.......................................................

OBMEP 2007

rr rr rr r r p p p
r
r r r

...............................................................................
...
...
...
... ..........
...
...
...
...
....
...
...
... .............
...
..............................................................................

Lista 1

Nvel 2

Nvel 2
Lista 1
1. Pot
encias de 10 - O valor de
(a) 101

(b) 102

0, 00001 (0, 01)2 1000


e:
0, 001

(c) 103

(d) 104

(e) 1

2. Diferen
ca de quadrados - Se (x + y)2 (x y)2 = 20, entao xy e igual a:
(a) 0

(b) 1

(c) 2

(d) 5

(e) 10

3. Um quadril
atero - O quadrilatero ABCD da gura e um paralelogramo?
.
..
..
..
..
... ........
. ..
........
..........
........ ... ..
........ .......
.
.
.
.
.
.
...
.
.
o ...
.
........
..
........
..
..
........
..
..
........
..
..
........
.
.
.
..
.
.
.
...
.
..
.....
.
.
.
.
.
.
.
.
.
...
..
..
.
........
..
..
o.................
..
....................
......
.....
........................
.
.. ... ........
........................
..
... ..............................................................
.
. . ....
..
..
....................
..
.. ...
o
..
.. ...
..
.......
...
......
.
.
..
..
..
..
..
..
..
..
.
...
.
........
o
...
.......................
.......
........................
.. ..
........................
.. ... ...............................
........................

45

45

115

65

4. Sexta-feira 13 -

Qual o n
umero maximo de sexta-feiras 13 que podem

ocorrer num ano nao bissexto? Neste caso, qual e o 10o dia do ano?

5. Tri
angulos com lados inteiros - Quantos triangulos existem cujos lados
sao n
umeros inteiros e o permetro e 12?
(a) 1

(b) 3

(c) 5

(d) 7
OBMEP 2007

(e) 9
49

Nvel 2

Lista 1

6. Festa de anivers
ario - Para comemorar seu aniversario, Ana vai preparar
tortas de pera e tortas de maca. No mercado, uma maca pesa 300g e uma
pera 200g. A sacola de Ana aguenta um peso maximo de 7k. Qual e o numero
maximo de frutas que ela pode comprar para poder fazer tortas das duas
frutas?

7. Os

dois

quadrados

As medidas em

centmetros dos lados de cada um dos dois quadrados sao n


umeros inteiros. Se o menor quadrado
tivesse 2001 cm2 a mais de area, os dois quadrados
seriam iguais. Quanto pode medir o lado do maior
quadrado?

............................................................................................................................................................
...
...
...
...
...
....
....
.....
...
...
....
....
...
...
....
....
...
...
....
....
...
...
...
...
...
...
...
...
....
....
.....
.....
...
...
......................................................
....
...
...
...
.
....
....
...
.
.
...
...
.
.
....
....
...
.
...
...
.
.
.
....
....
..
.
.
...
...
.
...
...
.....
..
.
...
.........................................................................................................................................................

8. A multiplica
c
ao - J
ulio faz multiplicacoes usando apenas os quadrados dos
n
umeros. Ele tem que calcular o produto 85 135. Para isso, ele desenha
um retangulo de 85 mm por 135 mm e traca nesse retangulo o maior quadrado
possvel; faz o mesmo no quadrado restante e assim sucessivamente. Dessa
maneira ele obtem oito quadrados. Desenhe a gura feita por J
ulio e escreva
85 135 como a soma de oito quadrados: 85 135 = 852 + . . .

50

OBMEP 2007

Solucoes da Lista 1

Nvel 2

Solu
co
es da Lista 1
1. Pot
encias de 10 - Temos:
105 (102 )2 103
0, 00001 (0, 01)2 1000
105 104 103
=
=
=
0, 001
103
103

106
105+(4)+3
=
= 106(3) = 103 .
3
3
10
10

A opcao correta e (c).

2. Diferen
ca de quadrados - Como (x + y)2 = x2 + 2xy + y 2 e (x y)2 =
x2 2xy + y 2 , temos:

(x + y)2 (x y)2 = x2 + 2xy + y 2 x2 + 2xy y 2 = 4xy = 20,


segue-se que xy = 5. A opcao correta e (d).

3. Um quadril
atero - Para que ABCD seja um paralelogramo, seus lados
devem ser dois a dois paralelos, isto e: AB//CD e AD//BC.
Como
 + ABC
 = 180 ,
DAB
entao as retas AD e BC sao paralelas. Alem disso, temos dois angulos alternos
internos de 45 entre as retas AB e DC, segue-se que elas sao paralelas. Logo
ABCD e um paralelogramo.

4. Sexta-feira 13 - Dado que os dias da semana se repetem a cada 7 dias,


entao a diferenca entre os dias da semana e dada pelo resto ao dividir por 7 o
n
umero de dias transcorridos.
Na tabela seguinte temos:
OBMEP 2007

51

Nvel 2

Solucoes da Lista 1

na primeira linha o n
umero de dias entre o dia 13 de um mes e o dia 13
do mes seguinte;
na segunda linha o resto quando dividimos esse numero por 7;
na terceira linha o resto quando dividimos por 7 o n
umero de dias entre
o 13 de janeiro e o 13 do mes correspondente, ou seja, e obtida somando
os resultados obtidos na linha anterior desde janeiro ate o mes correspondente e depois calculando o resto ao dividir por 7.
J-F

F-M

M-A

A-M

M-J

J-J

J-A

A-S

S-O

O-N

N-D

31

28

31

30

31

30

31

31

30

31

30

Os valores iguais na u
ltima linha, signicam que nestes meses o dia 13 caiu
no mesmo dia da semana. Em particular esta u
ltima linha nos diz que 13
de fevereiro, 13 de marco e 13 de novembro correspondem ao mesmo dia da
semana. Logo, temos no maximo tres sexta-feiras treze.
Nesse caso temos que 13 de janeiro ocorreu 3 dias antes de sexta-feira, isto e
terca-feira e o dia 10 de janeiro aconteceu 3 dias antes, isto e, no sabado.
Observacao: Note que a 6a -feira 13 ocorre apenas quando o 1o dia do mes e
um domingo. Assim, uma outra maneira, talvez mais simples, de resolver o
problema e determinar o n
umero maximo de vezes em que o 1o dia do mes e
um domingo num ano nao bissexto.

5. Tri
angulos com lados inteiros - Para que tres n
umeros a, b, c sejam os
comprimentos dos lados do triangulo, cada um deles deve ser maior que a
diferenca e menor que a soma dos outros dois.
Sejam a b c os comprimentos dos lados do triangulo. Assim, c < a + b.
52

OBMEP 2007

Solucoes da Lista 1

Nvel 2

Agora, somando c a ambos os membros temos que: 2c < a + b + c = 12, ou


seja, 2c < 12, logo c < 6.
Alem disso, como 3c a + b + c = 12 temos que: c 4. Logo, 4 c < 6.
No caso de c = 5, temos que a + b = 7. Os possveis valores de a e b sao:
a = 2 e b = 5 ou a = 3 e b = 4
No caso de c = 4, temos que a + b = 8, e portanto temos somente a solucao
a = b = 4.
assim temos 3 possveis triangulos. A opcao correta e (b).

6. Festa de anivers
ario - Denotemos por m o n
umero de macas e p o n
umero
de peras que Ana comprou, assim o peso que ela leva na sacola e 300m + 200p
gramas. Como a sacola aguenta no maximo 7000 gramas, temos que
300m + 200p 7000, que e equivalente a 3m + 2p 70.
Como as peras pesam menos, Ana tem que levar a maxima quantidade de
peras, e portanto, a mnima quantidade de macas. Assim, se ela levar 1 maca,
temos:
2p 70 3 = 67 = p 33, 5.
Logo, levando 1 maca, ela pode levar 33 peras. Entao, o numero maximo de
frutas e 34.
Na tabela abaixo vemos que Ana pode tambem levar 2 macas e 32 peras.
p

300m + 200p

p+m

34

6800

34

33

6900

34

32

7000

34

31

6800

33

OBMEP 2007

53

Nvel 2

Solucoes da Lista 1

7. Os dois quadrados - Se a e a medida do lado do quadrado maior e b a


medida do lado do quadrado menor, entao pelo enunciado temos
a2 = b2 + 2001 .
Logo:
2001 = a2 b2 = (a + b)(a b).
Como a e b sao n
umeros inteiros, temos que a + b e a b sao divisores de 2001.
Mas, 2001 = 3 23 29, segue que
(a + b)(a b) = 2001 1 = 667 3 = 87 23 = 69 29.
Consequentemente, temos 4 possveis formas de fatorar 2001 em dois fatores:
se a + b = 2001 e a b = 1 = a =

(a+b)+(ab)
2

= 1001;

se a + b = 667 e a b = 3 = a =

667+3
2

= 335;

se a + b = 87 e a b = 23 = a =

87+23
2

= 55;

se a + b = 69 e a b = 29 = a =

69+29
2

= 49.

Assim as possibilidades para o lado maior sao: 1001 , 335 , 55 e 49.

8. A multiplica
c
ao - O maior quadrado no retangulo de 85 135 e aquele de
85 85. Sobra entao um retangulo de 50 85, onde o maior quadrado e de
50 50. Continuando assim, obtemos:
85 135 = 852 + 502 + 352 + 152 + 152 + 52 + 52 + 52 .
54

OBMEP 2007

Solucoes da Lista 1

Nvel 2

......................................................................................................................................................................................................................................................................................................................................................................................................................
... 2.. 2.. 2..
...
...
...
...
.
.
.
....
....................................................
....
...
...
..
....
...
...
....
...
...
...
...
.
.
.
.
.....
.....
.....
.....
...
...
...
...
....
....
....
....
...
...
..
.
.
....
....
....................................................
...
.....
.....
.....
...
...
...
...
...
....
....
....
...
...
...
...
...
....
....
....
...
...
...
...
.
.
...
....
..
.
.
.
........................................................................................................................................................
...
...
.
.
.
....
.....
....
...
...
...
....
....
....
...
...
...
....
....
....
...
.....
.....
...
...
...
..
...
....
...
...
...
...
...
....
...
...
...
....
....
....
...
...
...
...
...
.
.....
...
...
...
...
..
....
....
....
...
...
...
....
....
....
...
...
.....
...
...
...
...
...
....
...
...
...
.
...
...
....................................................................................................................................................................................................................................................................................................................................................................................................................

5 5 5

352

152
152

852

502

OBMEP 2007

55

Nvel 2

Lista 2

Lista 2
xy
x
= 2, entao
e igual a:
y
x
1
(c)
(d) 1
(e) 2
2

1. Express
ao fracion
aria - Se
(a)

(b)

1
2

2. Pot
encias de 2 - Calcule:
a) 16782 16772

b) 10012 + 10002

c) 199992

d) 20012 + 20022 + 20032

3. Um queijo triangular - Osvaldo comprou um queijo em forma de um


triangulo equilatero. Ele quer dividir o queijo igualmente entre ele e seus
quatro primos. Faca um desenho indicando como ele deve fazer essa divisao.

4. Notas de Matem
atica - Joao e Claudia receberam suas notas numa prova
de matematica. A nota de Joao foi   e a de Claudia  . Juntos eles
obtiveram   . Alem disso, Claudia obteve 13 pontos a mais que Joao.
Qual a nota de cada um?

5. Opera
c
ao com raiz quadrada - O n
umero


A = ( 6 + 2)( 3 2)
3+2
e igual a:

(a) 3

(b)

(c)

(d) 1

(d) 2

6. O caminho da escola - Catia sai da escola todos os dias no mesmo horario


e volta para casa de bicicleta. Quando ela pedala a 20km/h, ela chega em
56

OBMEP 2007

Lista 2

Nvel 2

casa a`s 4 : 30 horas da tarde. Se ela pedalar a 10km/h, ela chega em casa a`s
5 : 15 horas da tarde. A qual velocidade ela deve pedalar para chegar em casa
`as 5 : 00 horas da tarde?

7. Dist
ancia na reta - Cinco pontos estao sobre uma mesma reta. Quando
listamos as dez distancias entre dois desses pontos, da menor para a maior,
encontramos 2, 4, 5, 7, 8, k, 13, 15, 17, 19. Qual o valor de k?

8. N
umero mpar - Se n e um n
umero inteiro qualquer, qual das seguintes
opcoes e um n
umero mpar?
(a) n2 n + 2 (b) n2 + n + 2 (c) n2 + n + 5
(d) n2 + 5

(e) n3 + 5

OBMEP 2007

57

Nvel 2

Solucoes da Lista 2

Solu
co
es da Lista 2
1. Express
ao fracion
aria Solu
c
ao 1: Temos:

Como

x y
y
xy
= =1 .
x
x x
x

x
y
1
= 2 temos que = , assim
y
x
2
x y
1
1
xy
= =1 = .
x
x x
2
2

A opcao correta e (c).


Solu
c
ao 2: Se

x
= 2, entao x = 2y. Logo
y
xy
2y y
y
1
=
=
= .
x
2y
2y
2

2. Pot
encias de 2 - Fatorando temos:
16782 16772 = (1678 + 1677)(1678 1677) = 3355.
(b) Como (a + b)2 = a2 + 2ab + b2 , temos:
10012 + 10002 = (1000 + 1)2 + 10002 = 10002 + 2000 + 1 + 10002 =
= 2 10002 + 2001 = 2002001.
(c) Como (a b)2 = a2 2ab b2 , temos:
199992 = (20000 1)2 = (2 104 )2 4 104 + 1 =
= 4 108 4 104 + 1 = 399 960 001.
(d) Colocando em funcao de 2000 temos:
20012 + 20022 + 20032 = (2000 + 1)2 + (2000 + 2)2 + (2000 + 3)2 =
= 3 20002 + 12 2000 + 14
= 12 024 014.

58

OBMEP 2007

Solucoes da Lista 2

Nvel 2

3. Um queijo triangular - Para dividir o queijo em 5 partes iguais, e suciente


dividi-lo em 5k partes iguais e dar k partes a cada um. Uma forma de fazer
essa particao, e mostrada na gura, onde o queijo foi partido em 25 = 5 5
triangulos.
T
 T

T
T
T
 T  T

T
T
T
T
T

 T
T

T


T
T
T

TT
T
T

T

 T

 T  T

T
T
T
T

T
T
T
T
T
TT
T

 T  T  T  T  T
T
T

T
T
T

4. Notas de Matem
atica - Temos que encontrar os valores dos smbolos na

soma +   .

As duas notas sao n
umeros de dois algarismos e a soma tem tres algarismos,
logo a soma tem que ser maior do que 100 e menor do que 200, assim temos
que  = 1.

Mas, Claudia obteve 13 pontos mais do que Joao, assim

+13 .
1

Agora como a soma de  e 3 tem que terminar em 1, temos que  = 8, e


portanto  = 6. Assim as notas de Claudia e Joao sao, respectivamente, 81 e
68.

OBMEP 2007

59

Nvel 2

Solucoes da Lista 2

5. Opera
c
ao com raiz quadrada - (c) Como


2
3+2
A = ( 6 + 2)( 3 2)
2

2
2
= ( 6 + 2) ( 3 2)
3+2

= ( 6 + 2)2 ( 3 2)2 ( 3 + 2)

= ( 6 + 2)2 ( 3 2)[( 3 2)( 3 + 2)]

= (6 + 2 12 + 2)( 3 2)(( 3)2 22 )

= (6 + 2 12 + 2)( 3 2)(1)

= (8 + 4 3)(2 3)

= 4(2 + 3)(2 3)

= 4(22 ( 3)2 ) = 4 1 = 4
Assim A2 = 4 e logo, A pode ser 2 ou 2. Como

3 2 e negativo, temos

que A tem que ser negativo, e portanto A = 2.

6. O caminho da escola - Seja t o tempo que ela gasta pedalando a 20km/h.


Pedalando a 10 km/h, ela faz o percurso no dobro do tempo que pedalando a
20km/h, isto e 2t. No entanto, como ela demora 45 minutos a mais temos:
2t t = 45 = t = 45min.
Logo, diariamente ela sai da escola a`s 4 : 30 h 45 min = 3 : 15 h, e o percurso
ate em casa e de 45min 20km/h =

3
4

20 = 15km. Para percorrer 15km

em 5 : 00 h 3 : 15 h = 1 : 45 h = 54 h, ela deve manter uma velocidade de


15km
= 12km/h.
5
h
4

7. Dist
ancia na reta Solu
c
ao 1: - Essa solucao e um pouco difcil de escrever porque e feita na base
60

OBMEP 2007

Solucoes da Lista 2

Nvel 2

detentativa e erro. Comecamos desenhando uma reta numerica e colocando


os pontos 0 e 19. Como a primeira distancia e 2, marcamos nossos primeiros
tres pontos:
0..........................................2.........................................................................................................................................................................................................................................................................................................................................19
.......
p1
p2
p3

Como temos que ter uma distancia 7, colocamos o ponto 7 na reta. Isso nos
da distancias que nao sao incompatveis com o problema:
0..........................................2.......................................................................................................7.......................................................................................................................................................................................................................................................19
.......
p1
p2
p4
p3

As distancias entre esses 4 pontos sao: 2, 7, 19, 5, 17 e 12. Finalmente,


colocando o ponto 15 na reta obtemos o seguinte:
0..........................................2.......................................................................................................7................................................................................................................................................15
19
..............................................................................................................
p1
p2
p4
p5
p3

Com esses pontos as distancias sao: 2, 7, 15, 19, 5, 13 , 17, 8, 12, 4, que sao
compatveis com os dados do problema. Logo, k = 12.
Note que temos tambem uma outra distribuicao dos n
umeros, a saber:
0..................................................................................4...................................................................................................................................................................12
14
19
..........................................................................................................................................................
p1
p3

Nessa distribuicao tambem obtemos k = 12.


Solu
c
ao 2: Como a maior distancia e 19 podemos, supor que um ponto e o 0
e outro e 19.
Se a e um outro ponto, entao na lista das distancias temos os n
umeros: a0 =
a e 19 a. De fato, na lista aparecem os pares 2 e 17, assim podemos supor
que o n
umero 2 e outro ponto sobre a reta.
OBMEP 2007

61

Nvel 2

Solucoes da Lista 2

Da mesma forma, como 4 e 15 estao na lista das distancias, temos que 4 ou 15


e outro ponto na reta. Mas, 4 nao pode ser um dos pontos porque a distancia
2 nao apareceu duas vezes. Logo, 15 e outro ponto na reta.
Por u
ltimo o quinto ponto tem que estar a uma distancia 5 de um dos pontos
e a 7 de outro, logo o ponto que falta e o ponto 7 e a distancia desconhecida
e k = 19 7 = 12.

8. N
umero mpar - Lembremos que:
n e n2 tem a mesma paridade: (par)2 =par e (mpar)2 =mpar;
a soma ou diferenca de n
umeros de mesma paridade e um n
umero par:
(par par=par e mpar mpar=par).
Solu
c
ao 1: Observemos que n2 + n e n2 n sao soma e diferenca de dois
n
umeros que sempre tem a mesma paridade, logo estes n
umeros sempre serao
pares. Portanto n2 + n + 5 = (n2 + n) + 5 e soma de um par e um mpar, logo
sempre sera mpar para todo valor inteiro de n. A opcao correta e (c).
Solu
c
ao 2: Observemos que n2 +n = n(n+1) e n2 n = n(n1) sao produtos
de dois n
umeros consecutivos, logo estes n
umeros sao pares. Portanto, n2 +
n + 5 = (n2 + n) + 5 e a soma de um par com um mpar, assim este numero e
mpar para todo valor inteiro de n.
Observemos que (a) e (b) sao sempre n
umeros pares para qualquer valor de n,
enquanto que (d) e (e) podem ser pares ou mpares, dependendo se n e mpar
ou par.

62

OBMEP 2007

Lista 3

Nvel 2

Lista 3
1. Quatro n
umeros inteiros - Se quatro inteiros positivos distintos m, n, p e
q satisfazem a equacao
(7 m)(7 n)(7 p)(7 q) = 4 ,
entao a soma m + n + p + q e igual a:
(a) 10

(b) 21

(c) 24

(d) 26

(e) 28

2. As p
aginas do dicion
ario - Para numerar as paginas de um dicionario,
imprimiu-se 1988 vezes o algarismo 1. Quantas paginas tem esse dicionario?

3. Soma de pot
encias de 2 - Determine um valor de n para o qual o numero
28 + 211 + 2n seja um quadrado perfeito.

4. Reverso de um n
umero - O reverso de um n
umero inteiro de dois algarismos e o n
umero que se obtem invertendo a ordem de seus algarismos. Por
exemplo, 34 e o reverso de 43. Quantos n
umeros existem que somados ao seu
reverso dao um quadrado perfeito?
....
.....
...
......
.. ...
.. ...
.. ...
.. ....
.. ...
.. ...
..
....
..
....
..
....
..
..
....
..
....
..
....
..
....
..
..
........
..
..
... ......
..
..
....
..
.
..
....
...
..
....
.. ...
....
.. ...
....
.. .
.. ..
....
....
....
....
....
...
.
....
.. ....
.
....
.. ...
....
.
....
.. ...
.
..
....
..
.
..
....
..
..
.
.
..
..
...........
.
.
..
............ ......
.
.
.
.
.
..
.
.
.
.
.
.
.
.
....
..
........
.
.
.
.
..
.
.
.
.
.
.
.
.
.
....
..
........
.
.
.
.
.
..
.
.
.
....
.
.
.
.
.
. ......
....
...
.............
....
.. ....................... ....
.
....
..
...............
.
.
.
.
....
..
.........
.
.
.
.
.
.
.
.
....
.
.
.
.
..
....... ...
.
.
.
.
.
....
.
.
.
.
.
.
.
..
.
........
.
.
....
.
.
.
.
.
.
.
.
.
.
.
.
..
.
.......
.
.
.
.
.
.
.
.
.
.
.
.
.
.
.
.
........ ................................................................................................................................................................................................................................................................

160

5. Angulos

externos

de

um

tri
angulo - Dados os angulos de
150 e 160 , indicados na gura,
calcule
x, y e z.

os

valores

dos

angulos

150

150

160

feita uma brincadeira com quatro n


6. Uma brincadeira - E
umeros inteiros
da seguinte maneira: some tres desses n
umeros, divida essa soma por 3 e o
OBMEP 2007

63

Nvel 2

Lista 3

resultado some com o quarto n


umero. Existem quatro formas de fazer esta
brincadeira, obtendo os seguintes resultados: 17, 21, 23 e 29. Qual e o maior
dos quatro n
umeros?

7. Ovos e ma
c
as - Num armazem, uma d
uzia de ovos e 10 macas tinham o
mesmo preco. Depois de uma semana, o preco dos ovos caiu 2% e o da maca
subiu 10%. Quanto se gastara a mais na compra de uma d
uzia de ovos e 10
macas?
(a) 2%

(b) 4%

(c) 10%

(d) 12%

(e) 12, 2%

8. Dividir um cubo - Se dividirmos um cubo de 1 m de aresta em cubinhos de


1 mm de aresta, que altura tera uma coluna formada por todos os cubinhos,
dispostos sucessivamente um em cima do outro?
(a) 1m

64

(b) 1km

(c) 10km

(d) 100km

OBMEP 2007

(e) 1000km

Solucoes da Lista 3

Nvel 2

Solu
co
es da Lista 3
1. Quatro n
umeros inteiros - Como m, n, p e q sao inteiros, entao 7 m,
7 n, 7 p e 7 q tambem sao inteiros. Agora,
4 = (1) (2) 1 2
e a u
nica decomposicao de 4 em um produto de n
umeros inteiros distintos.
Segue que
(7 m) + (7 n) + (7 p) + (7 q) = (1) + (2) + 1 + 2 ,
e da obtemos m + n + p + q = 28. A opcao correta e (e).

2. As p
aginas do dicion
ario - Observemos que:
a cada 10 n
umeros imprime-se 1 vez o 1 nas unidades,
a cada 100 n
umeros imprime-se 10 vezes o 1 nas dezenas,
a cada 1000 n
umeros imprime-se 100 vezes o 1 nas centenas.
Assim, de 1 ate 999 imprime-se o n
umero 1:
100 vezes nas unidades + 100 nas dezenas + 100 nas centenas = 300.
De 1000 ate 1999, imprime-se o numero 1 outras 300 vezes entre as unidades,
dezenas e centenas, e 1000 vezes na posicao dos milhares, portanto entre 1 e
1999 o numero de vezes que imprime-se o 1 e: 300 + 300 + 1000 = 1600.
Agora entre 2000 e 2999 imprime-se o 1 mais 300 vezes, completando 1600 +
300 = 1900.
De 3000 a 3099 temos 20 algarismos 1, de 3100 a 3119, temos 40 algarismos 1 e
de 3120 a 3139 temos 22 algarismos, portanto ate 3139 o numero de vezes que
imprime-se o 1 e: 1900 + 20 + 40 + 22 = 1982 vezes. Como faltam 6 algarismos
1, o n
umero de paginas do livro e 3144.

OBMEP 2007

65

Nvel 2

Solucoes da Lista 3

3. Soma de pot
encias de 2 - Observe que

28 + 211 + 2n = (24 )2 + 2 24 26 + (2 2 )2 .

Logo, se n = 12, temos


28 + 211 + 212 = (24 + 26 )2

Logo n = 12 e uma solucao.


Solucao Geral: Se 28 + 211 + 2n = k 2 , entao:
28 + 23 28 + 2n = k 2
9 28 + 2n = k 2
2n = k 2 (3 24 )2
2n = (k 3 24 )(k + 3 24 )
Logo, (k 3 24 ) e (k + 3 24 ) sao potencias de 2, ou seja:

k + 3 24 = 2a , k 3 24 = 2b e a + b = n
Temos:
2a 2b = (k + 3 24 ) (k 3 24 ) = 3 25 = 96.

Examinemos a lista das potencias de 2:

1 , 4 , 8 , 16 , 32 , 64 , 128 , 256 ,
Constatamos que 128 32 = 96. Logo, a = 7, b = 5 e n = 12.
66

OBMEP 2007

Solucoes da Lista 3

Nvel 2

4. Reverso de um n
umero - Denotemos por ab e
ba o n
umero e seu reverso. Temos que

Lembrete:

n
umeros de

dois algarismos onde a e

ab + ba = 10a + b + 10b + a = 11(a + b).

o algarismos das dezenas

Por outro lado, a 9 e b 9, logo, a + b 18.

e b o das unidades sao da

Como 11 e um n
umero primo e a + b 18, para que

forma

11(a + b) seja um quadrado perfeito, so podemos ter

10 a + b.

a + b = 11.
Assim, temos 8 n
umeros satisfazendo a condicao do

Ex: 47 = 4 10 + 7

problema: 29, 38, 47, 56, 65, 74, 83 e 92.

5. Angulos
externos de um tri
angulo - Observemos que os angulos y, 150
e 160 sao angulos externos de um triangulo, logo
y + 150 + 160 = 360 .
Assim y = 50 . Pela mesma razao conclumos que z = 50 . Como x, y e z sao
angulos internos de um triangulo entao x + y + z = 180 , portanto x = 80 .

6. Uma brincadeira - Sejam a, b, c e d os n


umeros procurados. Sao dados os
n
umeros
a+b+c
a+b+d
a+c+d
b+c+d
+d ,
+c ,
+b e
+a
3
3
3
3
mas nao sabemos a ordem deles. Como

a+b+c
3

+d+

a+b+d
3

+c+

a+c+d
3

+b+

b+c+d
3

+ a = 2(a + b + c + d)
= 17 + 21 + 23 + 29

Logo,
2(a + b + c + d) = 90 = a + b + c + d = 45.
OBMEP 2007

67

Nvel 2

Solucoes da Lista 3

Agora, seja d o maior dentre os n


umeros a. b, c e d. Assim,

d = 29

a+b+c
45 d
= 29
= d = 21 .
3
3

7. Ovos e ma
c
as - Podemos supor que o preco inicial de uma d
uzia de ovos
e R$ 1, 00, assim 10 macas tambem custa R$ 1, 00. Como o preco do ovo caiu
2%, entao o novo valor de uma d
uzia de ovos e R$ 0, 98. O preco das macas
subiu 10%, logo o novo preco das 10 macas e R$ 1, 10. Assim antes gastava-se
R$ 2, 00 na compra dos ovos e das macas e agora gasta-se 0, 98 + 1, 10 = 2, 08
0, 08
100% = 4%.
reais. Logo, o aumento foi de R$ 0, 08, que corresponde a
2
A opcao correta e (b).

8. Dividir um cubo -

Convertendo metros em milmetros temos: 1 m =

1000 mm. Assim, o cubo cou dividido em 1000 1000 = 106 cubinhos de
lado 1 mm cada um. Colocando-se lado a lado os 106 cubinhos, teremos uma
coluna de comprimento
1000 1000 = 106 mm = 106 103 m = 103 m = 1 km .

68

OBMEP 2007

Lista 4

Nvel 2

Lista 4
1. Uma express
ao - A expressao
a3
(a)
2

2
(b) 3
a

2. Uma igualdade -

a2
4a
1 3 onde a = 0, e igual a:
5
a
(2 a)

a5
(d)
2

1
(c) 3
2a

(e)

2
a5

Os n
umeros a e b sao inteiros positivos e satisfazem

96a2 = b3 . Qual e o menor valor de a?

3. O ret
angulo do Lus - Lus desenhou um retangulo de 6cm por 10cm, e
quer divid-lo em quatro partes. Cada parte deve ter de area, respectivamente,
8cm2 , 12cm2 , 16cm2 , 24cm2 . Desenhe como ele pode fazer essa divisao.

4. Somas de 3 em 3 - Encontre quatro n


umeros inteiros que somados de tres
em tres dao 6, 7, 8 e 9.

5. Uma f
abrica de blusas - Uma fabrica produz blusas a um custo de R$ 2, 00
por unidade alem de uma parte xa de R$ 500, 00. Se cada unidade produzida
e comercializada a R$ 2, 50 , a partir de quantas unidades produzidas a fabrica
obtem lucro?
(a) 250

(b) 500

(c) 1000

(d) 1200

(e) 1500

6. Exist
encia de tri
angulos - Qual dos seguintes triangulos nao pode existir?
(a) triangulo agudo isosceles
(b) triangulo retangulo isosceles
OBMEP 2007

69

Nvel 2

Lista 4

(c) triangulo retangulo obtusangulo


(d) triangulo retangulo escaleno
(e) triangulo escaleno obtusangulo

7. Os doze pontos - Doze pontos estao marcados numa


folha de papel quadriculada, conforme mostra a gura.
Qual o n
umero maximo de quadrados que podem ser
formados unindo quatro desses pontos?

8. O colar - Um colar e composto de perolas grandes e perolas pequenas, num


total de menos do que 500 perolas.
i - Se substituirmos 70% das perolas grandes por pequenas, o peso do colar
diminui de 60%.
ii - Se substituirmos 60% das perolas pequenas por grandes, o peso do colar
aumenta de 70%.
Quantas perolas tem o colar?

70

OBMEP 2007

Solucoes da Lista 4

Nvel 2

Solu
co
es da Lista 4
1. Uma express
ao - Temos:
4a
22 a
a2
25

=
a

a5
(21 a)3
23 a3
= a7

a1(3)
2

= a7

a4
2

a3
1
a7+4
=
= 3
2
2
2a

A opcao correta e (c)

2. Uma igualdade - Fatorando 96 temos: 25 3a2 = b3 . Para que 25 3a2


seja um cubo, o numero a deve ser da forma: a = 2n 3m . Assim,
25 3 a2 = 25 3 (2n 3m )2 = 25+2n 31+2m = b3
Logo, 5 + 2n e 1 + 2m sao m
ultiplos de 3. Os menores valores de n e m sao:
n = 2 e m = 1. Portanto, a = 22 3 = 12.

3. O ret
angulo do Lus - Como 24 = 4 6,
entao ele construiu o primeiro retangulo, tirando 4 cm do lado de 10 cm, sobrando um
quadrado de lado 6 cm. Sendo 16 = 4 4, ele
construiu um quadrado de lado 4 cm sobrando
dois retangulos de areas (6 4) 4 = 8 cm2
e (6 4) 6 = 12 cm2 , como, por exemplo, a
divisao mostrada na gura ao lado.

4
2

............................................................................................................................................................
...
...
..
..
....
....
....
...
....
....
....
...
....
....
....
....
....
....
....
....
....
....
....
...
....
....
...
...
....
....
....
....
....
....
....
....
....
....................................................................................................
...
....
...
....
...
...
....
....
...
....
....
....
...
..
...
.........................................................................................................................................................

A seguinte conguracao tambem e uma solucao para o problema.


OBMEP 2007

71

Nvel 2

24

12

Solucoes da Lista 4
4
16

4. Somas de 3 em 3 - Sejam a, b, c e d os n
umeros procurados. Sao dados os
n
umeros
a+b+c , a+b+d , a+c+d e b+c+d
logo
6 + 7 + 8 + 9 = (a +b +c)+(a +b +d)+(a +c +d)+(b +c +d) = 3(a+b +c +d) .
Assim,
a+b+c+d=

30
= 10 .
3

Note que cada numero e igual a` diferenca entre a soma dos quatro n
umeros e
a soma dos outros tres. Por exemplo: c = (a + b + c + d) (a + b + d). Logo,
os n
umeros procurados sao
10 6 = 4 ,

10 7 = 3 ,

10 8 = 2 e 10 9 = 1 .

5. Uma f
abrica de blusas - Denotemos por x o n
umero de unidades produzidas. Assim o custo de producao e 500 + 2x reais. Pela venda o fabricante
esta recebendo 2, 5x. Assim, ele tera lucro quando
2, 5x > 500 + 2x .
isto e, 0, 5x > 500. Portanto x > 1000. Logo, a opcao correta e (c).

72

OBMEP 2007

Solucoes da Lista 4

Nvel 2

6. Exist
encia de tri
angulos -

A soma dos tres angulos internos de um

triangulo e 180o . Logo, se um deles mede 90o , a soma dos outros dois e 90o , e
por isso nao podem ser maiores do que 90o . Portanto, nao existem triangulos
retangulos obtusangulos. Os seguintes exemplos de comprimentos de lados
mostram que os outros casos podem ocorrer:

(a) 2, 3, 3 ;

(b) 1, 1,

2 ;

(d) 3, 4, 5,

(e) 3, 4, 6.

7. Os doze pontos - No total, temos 11 possveis quadrados como mostrado


a seguir.
5 quadrados
s
s
s
s

s
s
s
s

4 quadrados

..........................
...
...
...
...
...
...
............................................................................
...
...
...
...
...
...
...
....
.
...
...
..
.
..........................................................................
...
...
...
...
....
..
..........................

s
s

s
s
s
s

s
s
s
s

......
.....
..... ..... .... .....
..... ................... .........
.....
.....
.... ....
.....
.....
...... ...........
.
.........
.
.
.
.
.
..... ....... ....... ....... ....... .........
.........
.........
.........
.
.
.
.
.
.
.
.
.
.
.
.
.
.
.
.
.
.
.
.
.
.
.... ................ ................ .........
.
.
.
.
..
......
......
..... .........
.....
.....
.. .
.....
.....
..... ................. ..........
..... .... ..... .....
.......
......

s
s

s
s

s
s

2 quadrados
s
s
s
s

s
s
s
s

..
...
....................... ..
.. ....... ....................
............
.........
.........
..............
..............
... ..
......
.....
.....
.
.....
. ..
.
.
... ...
......... ...
................
..........
.
............... ................
... ..................... ...
......... .........

s
s

s
s

8. O colar - Sejam n o n
umero de perolas grandes, p o n
umero de perolas
pequenas, a o peso de uma perola grande e b o de uma perola pequena. Com
essa notacao temos:
n
umero total de perolas no colar = p + n. Logo: n + p < 500
peso das perolas grandes = n a
peso das perolas pequenas = p b
peso total do colar = pb + na
Para equacionar o problema, vamos equacionar antes as duas hipoteses:
i - Ao substituirmos 70% das perolas grandes pelas pequenas, o colar ca
composto como
OBMEP 2007

73

Nvel 2

Solucoes da Lista 4

30%
n = 0, 3n + (p + 0, 7n)
  n + p + 70%


pequenas

grandes

e seu peso ca sendo


0, 3n a
  

peso das grandes

(p + 0, 7n) b = 0, 4(na + pb)








peso das pequenas

40%

do peso inicial

ii - Analogamente, temos que ao substituirmos 60% das perolas pequenas


pelas grandes, o colar ca composto como

n + 60% p + 40% p = (n + 0, 6p) + 0, 4p



  


pequenas

grandes

e seu peso ca sendo


(n + 0, 6p) a +



peso das grandes

0, 4p b
  
peso das pequenas

1, 7(na + pb)



170%

do peso inicial

Temos, entao, o sistema:

0, 3 n a + 0, 7n b + p b = 0, 4(n a + p b)

n a + 0, 6p a + 0, 4p b = 1, 7(n a + p b) .
Para resolve-lo, comecamos eliminando as incognitas a e b, escrevendo o sistema na seguinte forma:

a
7n + 6p

=
n
b

13p
a

= .
7n 6p
b
Segue que
13p
7n + 6p
=
= 36p2 13pn 49n2 = 0 .
n
7n 6p
74

OBMEP 2007

Solucoes da Lista 4

Nvel 2

Para fatorar essa expressao, escrevemos


13p n = 36p n 49p n ,
e temos:
36p2 13p n 49n2 = 36p2 + 36p n 49p n 49n2
= p (36p 49n) + n (36p 49n)
= (36p 49n)(p + n) .
Finalmente:
(36p 49n)(p + n) = 0 .
Obtemos 36p = 49n, e como p e n sao inteiros positivos, segue que n e m
ultiplo
de 36 e p de 49. Assim temos: n = 36k e p = 49k  , onde k e k  sao inteiros
maiores do que 1. Logo,
36 49k  = 49 36k = k = k  .
Portanto, n = 36k e p = 49k. Deduzimos que n + p = 85k. Como n + p < 500,
o colar so pode ter: 85 , 170 , 255 , 340 ou 425 perolas.

OBMEP 2007

75

Nvel 2

Lista 5

Lista 5
1. Suco de laranja - Davi vai a um armazem que vende uma garrafa de suco
de laranja por R$2, 80 e uma caixa com seis dessas garrafas por R$15, 00.
Ele precisa comprar 22 garrafas para seu aniversario. Quanto ele gastara no
mnimo?

2. Mulheres votantes - Numa cidade, 40% de todas as mulheres sao votantes


e 52% da populacao e de mulheres. Qual o percentual da populacao formado
de mulheres votantes?
(a) 18, 1%

(b) 20, 8%

(c) 26, 4%

(d) 40%

(d) 52%

3. Amigos do s
eculo XX - Dois amigos nasceram no seculo XX, com uma
semana de intervalo e no mesmo mes e ano. Escrevendo da esquerda para a
direita a data na forma o (ou os) algarismo(s) do dia, (ou os) algarismo(s) do
mes, e os dois u
ltimos algarismos do ano, obtemos dois n
umeros sendo um o
sextuplo do outro. Nao colocamos 0 na frente dos 9 primeiros meses. Qual e
a data de nascimento do amigo mais velho?

4. Opera
c
ao em uma fra
c
ao - Que n
umero se deve somar aos dois termos
de uma fracao para se obter o inverso dessa mesma fracao?

5. O n
umero 119 - O n
umero 119 tem a seguinte propriedade:
a divisao por 2 deixa resto 1;
a divisao por 3 deixa resto 2;
a divisao por 4 deixa resto 3;
76

OBMEP 2007

Lista 5

Nvel 2
a divisao por 5 deixa resto 4;
a divisao por 6 deixa resto 5.

Quantos inteiros positivos menores que 2007 satisfazem essa propriedade?

6. Fonte com 3 torneiras - Slvia vai a uma fonte que tem tres torneiras,
encher os seus dez garrafoes. Um dos garrafoes demora um minuto para encher, outro dois minutos, outro tres minutos e assim por diante. Como Slvia
devera distribuir os garrafoes pelas torneiras de modo a gastar o menor tempo
possvel? Qual e esse tempo?

7. A seq
u
encia xyz - Na seq
uencia

1 5 3 7
, , , , x , y , z , . . . . os valores de
2 8 4 8

x , y e z sao...

8. A mesa circular - Uma mesa circular tem 60 cadeiras em sua volta. Existem N pessoas sentadas nessas cadeiras de tal modo que a proxima pessoa a se
sentar vai ter que se sentar ao lado de alguem. Qual e o menor valor possvel
para N ?

OBMEP 2007

77

Nvel 2

Solucoes da Lista 5

Solu
co
es da Lista 5
1. Suco de laranja - Se Davi comprar 6 garrafas individualmente, ele gastara
6 2, 80 = 16, 80 reais
que e mais caro do que comprar uma caixa com seis. Portanto ele deve comprar
a maior quantidade possvel de caixas. Nesse caso, ele deve comprar 3 caixas
e 4 garrafas individualmente, caso em que gastara
3 15 + 4 2, 80 = 56, 20 reais ,
que e o mnimo possvel.

2. Mulheres votantes - A fracao de mulheres na populacao e


fracao que e votante e

40
.
100

52
,
100

e delas, a

Logo, a fracao de mulheres volantes e:

52
40
104
104

=
=
100% = 20, 8%.
100 100
5 100
5 100
A opcao correta e (b).

3. Amigos do s
eculo XX - Os dois amigos nasceram no mesmo mes e no
mesmo ano, com uma diferenca de 7 dias, assim um nasceu no dia d/m/a e o
outro no dia (d + 7)/m/a. Com esta datas formamos os n
umeros (d)(m)(a) e
(d + 7)(m)(a). Sabemos que:

(d + 7)(m)(a) = (d)(m)(a) + 7 10k .


Assim,
(d + 7)(m)(a) = 6 (d)(m)(a),
78

OBMEP 2007

7 10k = 5(d)(m)(a).

Solucoes da Lista 5

Nvel 2

Logo, k = 3 se o mes tem 1 algarismo e k = 4 se o mes tem 2 algarismos.


No primeiro caso, quando k = 3, temos que

7000
5

= 1400, isto e, 1 de abril de

1900. Logo, seu amigo nasceu em 8 de abril de 1900. No segundo caso, quando
k = 1,

70000
5

= 14000 nao e uma data valida.

4. Opera
c
ao em uma fra
c
ao - Seja
tal

a+c
b+c

a
b

a fracao procurada e seja c um n


umero

= ab . Esta igualdade e equivalente a (a + c)a = (b + c)b. Assim temos:

(a + c)a = (b + c)b = a2 + ac b2 bc = 0 = (a2 b2 ) + c(a b) = 0.


Donde
0 = (a2 b2 ) + c(a b) = (a b)(a + b) + c(a b) = (a b)(a + b + c).
Portanto (a b)(a + b + c) = 0. Temos dois casos:
1o ) a b = 0 = a = b. Nesse caso a fracao e igual a 1 =

a
a

e podemos somar

qualquer n
umero.
2o ) a + b + c = 0 = c = (a + b). Nesse caso temos que somar a b.

5. O n
umero 119 - Inicialmente note que se N dividido por d deixa resto r,
entao somando a N um m
ultiplo de d, o resto nao se altera, isto e:

(N + m
ultiplo de d)
tambem deixa resto r.
d
Por exemplo: 38 dividido por 3 deixa resto 2, logo o resto da divisao de
(38 + 5 3) tambem e 2.
Assim, se somamos a 119 um n
umero que seja m
ultiplo simultaneamente de 2,
3, 4, 5 e 6, esse n
umero deixa os mesmos restos que 119 quando dividido por
OBMEP 2007

79

Nvel 2

Solucoes da Lista 5

2, 3, 4, 5 e 6. O menor m
ultiplo comum de 2, 3, 4, 5 e 6 e 60, logo todo n
umero
da forma
119 + (m
ultiplo de 60)
satisfaz as cinco condicoes do enunciado.
Da divisao de 2007 por 60 temos:
2007 = 33 60 + 27 = 32 60 + 87 = 31 60 + 147.
Como 119 esta entre 87 e 147, temos que os n
umeros
59, 119, 179, . . . , 31 60 + 119
cumprem a mesma propriedade que 119. Logo, temos 33 possveis n
umeros.

6. Fonte com 3 torneiras Solu


c
ao 1: Para simplicar, numeramos os garrafoes de acordo com os respectivos tempos que gastam para car cheios. A ideia , e utilizar o tempo que
sobrade um garrafao para encher outro garrafao, enchendo simultaneamente
outros. As guras ilustram a solucao.
FIGURA I 10 min

...
...
...
....
...
...
...
...
....
...
....
...
....
...
..
...
.....................................

10

FIGURA II 9 min

...
...
...
....
...
...
...
...
....
...
....
...
....
...
..
...
.....................................

...
...
...
....
...
...
...
...
....
...
....
...
....
...
..
...
.....................................

...
...
...
....
...
...
...
...
....
...
....
...
....
...
...
..
....................................

...
...
...
....
...
...
...
...
....
...
....
...
....
...
...
..
....................................

...
...
...
....
...
...
...
...
....
...
....
...
....
...
...
..
....................................

...
....
...
...
...
...
....
...
....
...
....
...
...
....
..
...
....................................

...
....
...
...
...
...
....
...
....
...
....
...
...
....
..
...
....................................

...
....
...
...
...
...
....
...
....
...
....
...
...
....
..
...
....................................

...
....
...
...
...
...
....
...
....
...
....
...
...
....
..
...
....................................

...
....
...
...
...
...
....
...
....
...
....
...
...
....
..
...
....................................

Na gura I as 3 torneiras gastam 10 minutos para encher os garrafoes 10, 9, 8,


1 e 2. Na gura II as 3 torneiras gastam 9 minutos para encher os garraf
oes
7, 6, 5, 2, 3 e 4. Logo, o tempo total gasto e de 19 minutos.
Solu
c
ao 2: Se tivessemos uma torneira so, o tempo gasto para encher os 10
garrafoes e 1 + 2 + + 9 + 10 = 55 minutos. Como 55 = 18 3 + 1, se temos
80

OBMEP 2007

Solucoes da Lista 5

Nvel 2

3 torneira devemos gastar pelo menos 19 minutos. A seguinte tabela mostra


a forma de fazer o trabalho em 19 minutos.
Torneira 1

10

Torneira 2

Torneira 3

7. A seq
u
encia xyz - Igualando os denominadores, vericamos que a seq
uencia
dada e a mesma que a seq
uencia
4 5 6 7
,
,
,
, x , y , z , ...
8 8 8 8
Assim, o denominador e 8 e os numeradores sao n
umeros consecutivos. Logo
8
9
10
5
x = = 1, y = e z =
= .
8
8
8
4
8. A mesa circular - Se a proxima pessoa a se
sentar vai ter que se sentar ao lado de uma cadeira
ocupada, isso signica que existem no maximo 2
cadeiras desocupadas consecutivas. Veja na gura:
as cadeiras ocupadas estao representadas por quadradinhos brancos e as desocupadas por quadradi-

.............................................................
.........
............
........
.........
.......
.......
.......
......
.
.
.
.
.
.....
....
.
....
.
..
.
....
.
.
....
...
.
.
...
..
.
...
..
.
...
..
...
.
..
....
..
...
..
..
...
..
..
...
..
..
..
..
..
..
..
.
.
..
.
...
..
...
...
...
...
...
..
.
....
.
....
....
....
....
....
....
...
......
.
.
.
.
.
......
.
........
.......
.........
.......
...........
........
..................
...........
......................................

nhos pretos. Podemos entao pensar nas cadeiras


em grupos de 3 e a terceira esta ocupada. Logo, o
menor valor de N e 60 3 = 20.

OBMEP 2007

81

Nvel 2

Lista 6

Lista 6
1. N
umeros proporcionais - Se
2

(a) x9

(b) x3 z

(c) 3x2

3
x
= , entao 9y 2 e igual a:
y
z
(e) 19 x2 z 2

(d) x2 z 2

2. Esportistas de uma escola - Em um grupo de 40 estudantes, 20 jogam futebol, 19 jogam volei e 15 jogam exatamente uns destes dois esportes. Quantos
estudantes nao praticam futebol e volei?
(a) 7

(b) 5

(c) 13

(d) 9

(e) 10

3. Vamos ao teatro - Na campanha Vamos ao teatro, 5 ingressos podem ser


adquiridos pelo preco usual de 3 ingressos. Mario comprou 5 ingressos nessa
campanha. A economia que Mario fez representa que percentual sobre o preco
usual dos ingressos?
(a) 20%

(b) 33 13 %

(c) 40%

(d) 60%

4. Uma desigualdade - Os valores de x que satisfazem


(a) x < 2

(b) x > 1

(c) 1 < x < 2

(d) x < 1

(e) 66 23 %

1
> 1 sao:
x1
(e) x > 2

5. A sala do Newton- Professor Newton dividiu seus alunos em grupos de 4 e


sobraram 2. Ele dividiu seus alunos em grupos de 5 e um aluno cou de fora.
Se 15 alunos sao mulheres e tem mais mulheres do que homens, o n
umero de
alunos homens e:
(a) 7

82

(b) 8

(c) 9

(d) 10

OBMEP 2007

(e) 11

Lista 6

Nvel 2

6. Um jardim retangular -

O retangulo

ABCD representa um terreno retangular cuja


largura e 3/5 do comprimento. O retangulo
ABEF representa um jardim retangular cuja
largura e tambem 3/5 do comprimento. Qual

D...............................................................................................................................F
...................................................................... A
..
..
...
...
...
...
...
....
...
...
...
...
...
...
...
...
...
...
....
...
...
....
...
...
....
...
...
...
...
....
...
...
...
...
...
....
...
...
...
...
....
...
...
....
...
...
....
...
...
...
...
....
...
...
...
...
...
....
................................................................................................................................................................................................

a razao entre a area do jardim e a area total


do terreno?
(a) 30%

(b) 36%

(c) 40%

(d) 45%

(e) 50%

7. Os bombons misturados - Marta e Carmem ganharam, cada uma, muitos


bombons. Elas misturarm os bombons e agora nao sabem mais qual o n
umero
de bombons que cada uma ganhou. Vamos ajuda-las a descobrir os n
umeros
sabendo que:
juntas ganharam 200 bombons;
cada n
umero e m
ultiplo de 8;
Marta se lembra que ganhou menos de 100 bombons, mas mais do que
4/5 do que ganhou Carmem.

OBMEP 2007

83

Nvel 2

Solucoes da Lista 6

Solu
co
es da Lista 6
1. N
umeros proporcionais - Como

x
y

3
,
z

entao xz = 3y. Elevando ao

quadrado ambos os membros dessa igualdade obtemos x2 z 2 = 9y 2 . A opcao


correta e (d).

2. Esportistas de uma escola - Denotemos por x o n


umero de estudantes
que praticam simultaneamente os dois esportes. Logo, temos que o n
umero de
estudantes que pratica somente futebol e 20 x e o que pratica somente volei
e 19 x. Portanto os estudantes que praticam exatamente um esporte sao
(20 x) + (19 x) = 15.
Segue-se que x = 12 e teremos que os estudantes que praticam algum esporte
sao
20 + (19 x) = 27 .
Portanto, os que nao praticam esporte sao 13. A opcao correta e (c).

3. Vamos ao teatro - Mario pagou 3 e levou 5, logo ele pagou apenas


preco usual e portanto, economizou 25 . Como

2
5

40
,
100

3
5

do

a economia foi de 40%.

A opcao correta e (c).

4. Uma desigualdade - Note que o inverso de um n


umero b so e maior do que
1 quando b for positivo e menor do que 1. Portanto,
1
> 1 0 < x 1 < 1 1 < x < 2 .
x1
A opcao correta e (c).

84

OBMEP 2007

Solucoes da Lista 6

Nvel 2

5. A sala do Newton Solu


c
ao 1:

Como o n
umero de alunos homens e menor do que 15 e das

mulheres e 15, temos


15 < alunos homens + alunas mulheres < 15 + 15 = 30
ou seja: o n
umero de alunos esta entre 15 e 30.
Por outro lado quando dividimos por 4 sobram 2 alunos, entao o n
umero de
alunos e par. Quando dividimos por 5 sobra um, entao o u
ltimo algarismo do
n
umero e 1 ou 6, mas sendo par so pode ser 6. Assim so temos dois possveis
valores: 16 e 26. Descartamos 16 porque e divisvel por 4. Logo, a resposta e
26.
Solu
c
ao 2: Como acima, o n
umero de alunos esta entre 15 e 30. Observemos
que o n
umero 6 dividido por 4 deixa resto 2 e dividido por 5 deixa resto 1. Logo
se somamos a 6 um m
ultiplo comum de 4 e 5, o n
umero obtido tambem tera
esta propriedade. O menor m
ultiplo comum de 4 e 5 e 20, assim os possveis
valores para o n
umero de alunos e 6, 26, 46, 66, . . . . Dado que o n
umero de
alunos esta entre 15 e 30 entao a solucao e 26.

6. Um jardim retangular - Pelos dados do problema sabemos que


5
AD = AB
3

5
e AB = AF .
3

Logo,
5
25
AD = ( )2 AF = AF .
3
9
A area do terreno e AB AD e a area do jardim e AB AF , portanto a razao
entre as areas e
AB AF
AF
AF
9
=
= 5 2
= 36%.
=
AB AD
AD
25
( 3 ) AF
A opcao correta e (b).
OBMEP 2007

85

Nvel 2

Solucoes da Lista 6

7. N
umeros decrescentes - Escreva os n
umeros abaixo em ordem decrescente

2/3

3,

 3
1
,
3

Solu
c
ao: Sabemos que
32/3 =
2

1
32/3

1
32

 1
1
.
3

Se a, b e c sao nao nulos e

< 1,
a>b>c

< 1,
entao

( 13 )3 = 313 < 1,

1 < 5 3 < 3.

1
1
1
< < .
a
b
c

Como 33 > 32 > 32/3 temos entao :

1
1
1
5
<
<
<
1
<
3 < 3.
3
2
2/3
3
3
3
Portanto,

 3
 1

1
1
5
2
2/3
<3 <3
< 3<
.
3
3

8. Os bombons misturados -

Sejam x o n
umero de bombons que Marta

ganhou e y o que Carmem ganhou. Temos x + y = 200. Como x < 100 entao
y 100. Por outro lado, x > 45 y e y 100, conclumos que x >

4
5

100 = 80.

Logo, x e um inteiro compreendido entre 80 e 100 e m


ultiplo de 8, logo, so
pode ser 88 ou 96. Vamos decidir:
Se x = 88, entao y = 200 88 = 112. Logo: x >

4
5

112 = 89, 5 , o que

nao e possvel.
Se x = 96, entao y = 200 96 = 104 e x >
possvel.
Logo Marta ganhou 96 bombons e Carmem 104.

86

OBMEP 2007

4
5

104 = 83, 2, o que e

Lista 7

Nvel 2

Lista 7
1. Jantar aos s
abado - Tres casais jantam todo sabado no mesmo restaurante,
numa mesma mesa redonda. A poltica do restaurante e :

(a) jamais colocar juntos a` mesa como vizinhos marido e mulher;


(b) a disposicao dos seis a` mesa e diferente a cada sabado.

Durante quantos sabados os casais poderao ir ao restaurante sem repetir as


disposicoes a` mesa?


4
1 + 1 + 1 e:
2. Express
ao com radicais - O valor de


1
(a) 2 + 3 (b) 7 + 3 5 (c) 1 + 2 3 (d) 3 (e) 3 + 2 2
2

3
3. Uma diferen
ca - O valor de
(a)

3, 3

(b)

4, 7

(c)

0, 001 400
0, 036 0, 4

e:
0, 25
0, 4
4, 9

(d)

3, 8

(e)

7, 5

4. A Terra - A superfcie do globo terrestre consiste de agua (70%) e de terra


(30%). Dois quintos da terra sao desertos ou cobertos por gelo e, um terco
e pastagem, oresta ou montanha; o resto e cultivado. Que percentual da
superfcie total do globo terrestre e cultivada?

OBMEP 2007

87

Nvel 2

5. Uma fra
c
ao - Determine

Lista 7

A
......
... ....
.. .....
... ...
....
...
.
.
M...............................N
....
..
....
...
....
...
................................................
B
C

AN
.
AC

6. C
aculo de a
ngulo - Na gura P Q e paralelo a RS e T U = T V . Se o

QU V mede:
angulo T
W S = 110 , o angulo 
(a) 135

(b) 130

(c) 125

(d) 115

(e) 110

..
...
...
.
..
.
...
...
...
..
..
.
.
...
...
...
...
...
...
..
..
.
.
...
...
...
...
...
...
..
..
.
.
.
...
.......
....
...
... ...
...
.. ......
..
.
.
....
...
...
....
...
...
....
....
...
...
....
..
..
.
.
....
...
...
....
..
..
...
..
..
...................................................................................................................................................................................................
.
.
...
...
...
...
...
...
..
..
.
.
.
...
...
..

110o

7. Uma loja de brinquedos - Uma loja estava vendendo um brinquedo por


R$ 13, 00 a unidade. Para conseguir vender todo o seu estoque que nao era
superior a 100 unidades, resolveu abaixar o preco de um n
umero inteiro de
reais. Com isso, conseguiu vender todo o estoque por R$ 781, 00. Qual foi a
reducao do preco, por unidade?

88

OBMEP 2007

Solucoes da Lista 7

Nvel 2

Solu
co
es da Lista 7
1. Jantar aos s
abado - Para simplicar, vamos denotar cada casal por um
par de n
umeros: (1, 2), (3, 4), (5, 6), onde em cada par, um n
umero representa
o marido e o outro a mulher. Tres pares nao podem ser vizinhos (1, 2), (3, 4),
(5, 6)
Veja duas disposicoes possveis; no sentido horario comecando em 1: 1-3-2-54-6 e 1-6-4-5-2-3.
1

....................................
.......
....
....
...
...
..
..
...
.
..
..
...
..
.
...
..
...
.
..
....
.
.
.......
..
....................................

....................................
.......
....
....
...
...
..
..
...
.
..
..
...
..
.
...
..
...
.
..
....
.
.
.......
..
....................................

6............

3............

5
132546

5
164523

Fixando a posicao do 1 na mesa e lendo os n


umeros formados no sentido
horario, o problema se resume em encontrar todos os n
umeros de 6 algarismos
distintos que podem ser escritos com os algarismos 1, 2, 3, 4, 5 e 6, onde:
os n
umeros todos comecam com o algarismo 1;
nao podem aparecer juntos 1 e 2, 3 e 4, 5 e 6.
Encontramos os 16 n
umeros que estao na tabela.
132546

132645

135246

135264

135426

136245

136254

136425

142536

142635

145236

145326

146235

146325

153246

154236

Logo, a resposta e 16 sabados.

2. Express
ao com radicais 4


2

1 + 1 + 1 = 1 + 2 = 1 + 2 2 + 2 = 3 + 2 2.
OBMEP 2007

89

Nvel 2

Solucoes da Lista 7

A opcao correta e (e).


3. Possveis tri
angulos - Os lados de um triangulo tem comprimentos: a, a+2
e a + 5, onde a > 0. Determine os possveis valores de a.
Solu
c
ao: Como a soma dos comprimentos dos lados menores deve ser maior
que o comprimento do lado maior, entao temos que a + (a + 2) > a + 5, assim
a > 3.

4. Uma diferen
ca - (a) Temos:

0, 4 ( 0, 09 1)
0, 1 20
20

= (0, 3 1) = 4 0, 3 + 1 = 3, 3.
0, 5
5
0, 4

5. A Terra - A fracao da terra que e cultivada e


1
Como a terra e
isto e o

2
25

3
10

2 1
15 6 5
4
=
=
.
5 3
15
15

do globo, temos que area cultivada e

4
15

3
10
=

2
25

do globo,

100% = 8% do globo terrestre.

6. Uma fra
c
ao - A gura mostra que M N e paralelo a BC, logo os triangulos
ABC e AM N sao semelhantes, e por isso seus lados sao proporcionais. Usando
o lado dos quadradinhos da grade da gura, temos:

AM
AB

= 47 . Logo,

AN
AM
4
=
= .
AC
AB
7

7. C
aculo de a
ngulo - Como as retas P Q e RS sao paralelas, entao os angulos


T
W S e QT
W sao complementares. Assim temos que

QT
W = 180 110 = 70 .
90

OBMEP 2007

Solucoes da Lista 7

Nvel 2

Por outro lado, sabemos que o triangulo U T V e isosceles, logo os angulos


em U e em V sao iguais. Usando que a soma dos angulos internos de um
triangulo e 180 temos que
2T
U V = 180 70 = 110 .
Portanto
T
U V = 55

e 
QU V = 180 55 = 125 .

A opcao correta e (c).

8. Uma loja de brinquedos - Se x e o desconto em reais e y e o n


umero de
pecas, entao
(13 x) y = 781 e y < 100.
Assim, (13 x) e y sao divisores de 781. Como 781 = 11 71, a u
nica solucao
e y = 71 e 13 x = 11. Logo, a reducao foi de R$2, 00.
Observacao: x = 12 e y = 781 e solucao da equacao (13 x) y = 781, mas
nao do problema porque devemos ter y < 100.

OBMEP 2007

91

Nvel 2

Lista 8

Lista 8
1

1. Fra
c
ao de fra
c
ao - Qual o valor de 1 +

1+

1+

1
2

2. Pot
encias de 3 - Se 3n = 2 entao quanto vale 272n ?

3. Aumento de pre
co - Se o preco de um produto subiu de R$ 5, 00 para
R$ 5, 55, qual foi a taxa percentual de aumento?

4. Roseiras em fila - Jorge ganhou 15 roseiras para seu jardim, com a condicao
de planta-las em 6 las de 5 roseiras cada uma. Isso e possvel? Em caso
armativo faca um desenho indicando para Jorge como plantar as roseiras.

5. Calculadora diferente - Uma fabrica produziu uma calculadora original


que efetua duas operacoes:
a adicao usual +
a operacao 
Sabemos que para todo n
umero natural a tem-se:
(i) a  a = a e (ii) a  0 = 2a
e, para quaisquer quatro naturais a, b , c e d
(iii) (a  b) + (c  d) = (a + c)  (b + d) .
Quais sao os resultados das operacoes (2 + 3)  (0 + 3) e 1024  48 ?
92

OBMEP 2007

Lista 8

Nvel 2

..................................................................................................................................................................
... ......
.... ..
..... ..
.... .............
..... ....
... ... ........
.....
...
.....
.
.... .... .... .......
...
.
.
.
.
... ... ... ..........
...
.....
.....
.... .... .... ... ..........
.
....
.
.
.
... ... ... ... ... ..............................................................................
...
.... .... .... .... .... ............
....
......
.
.
.
.
... ... ... ... ... ... ......
...
... ....
.
.
.
... ... ... ... ... ... ......
...
.
.... ...
.
.....
.
.
... ... ... ... ... ...
...
.
...
.....
...
.
... ... ... ... ... ...
...
.
.
.
.
.
..... .....
..
..... .....
.... .... .... .... .... ....
....
.
...
.........
... .. ... ... ... ...
...
.
.
.... .........
.... .... .... .... .... ....
....
.
.
.
.
.
.
..
.....
...
.
.
.
.
.
..... .... ..... ..... .... ....
.....
.
.
.
.....
..... ....
... ... ... ... ... ... ..........
...
..... ..
.... ... .... .... ... ... ........
....
.
..... ..
... ... ... ... ... .........
...
.
.
.
.
.... ... .... .... ... ................................................................................
....
.....
... ... ... ... ........
...
.
.....
.... ... .... ............
....
.
.
.
.....
... ... ... .....
...
.....
... ... ........
..
.....
... ... .......
..... ....
..... ...
... ......
..... ..
...........
.
.
.........................................................................................................................................................

6. Dois quadrados - Na gura ao lado, a area do


quadrado maior e 10 cm2 e do menor e 4 cm2 . As
diagonais do quadrado maior contem as diagonais
do quadrado menor. Quanto mede a area da regiao
tracejada?

7. ParalelismoFN FU
.
FR FL

Sendo IL paralela a` EU e RE paralela a` N I, determine

............
............
............
............
.. . ........
.... ... ..............
.... .... ..................................
............
...
... ... ...
.
.
.
............
.
...
............
....
......
...
............
....
.... ..
...
....
... ....
................
.
.
.
.
.
.
.
...
................................................
..
...
.
.
.
.
.
.
.
.
.
.
.
.
.
.
.
.
.
.
.
.
.
.......................................
.
...
.
.
.
.
.
.
.........
............................
..........................

U L

8. Um subconjunto - O conjunto {1, 2, 3, . . . , 3000} contem um subconjunto


de 2000 elementos tal que nenhum elemento e o dobro do outro?

9. Tri
angulos ret
angulos - Dada a gura
com as marcas, determine v, w, x, y e z.

......
... ...
... ....
... .....
....w
.....
................ ......
.... ... .....
9
z
..
... ..
...............................................................................................................................................................................
... ..
....
......
. .
....
......
v
..................
....
......
...
....
......
.
.
.
.
.
.
.
....
.
......
..... x
....
......
...
......
y ........
......
....
....
...... 20
.
.
.
.
.
.
.
.... ...
...
.... ........... ...........
....... .... ..............
............
.

sr

sr

10. Uma desigualdade especial- Quais valores de x satisfazem x2 < |x| + 2?


(a) x < 1 ou x > 1 (b) x > 1 (c) 2 < x < 2 (d) x < 2 (e) x < 0
OBMEP 2007

93

Nvel 2

Solucoes da Lista 8

Solu
co
es da Lista 8
1. Fra
c
ao de fra
c
ao - Temos:
1

1+
1+

1
1+

=1+

1
1+

1
2

=1+

1+

3
2

2
3

=1+

1
5
3

=1+

3
8
= .
5
5

2. Pot
encias de 3 - Temos: 272n = (33 )2n = 36n = (3n )6 = 26 = 64.

3. Aumento de pre
co - O aumento em reais foi 5, 55 5 = 0, 55; entao o
percentual de aumento foi
0, 55 20
11
0, 55
=
=
= 11%.
5
5 20
100

possvel plantar as roseiras em 6 las de 5 roseiras


4. Roseiras em fila - E
cada uma, conforme mostra o desenho a seguir .
s

......
.. .....
.. .....
.. .....
.. .....
.....
..
..
........
..
.. .... ..........
.. ..
.....
.. ..
.....
.. ..
.....
.. ..
.....
.....
.. ..
.....
....
.....
...
.....
....
.
.....
....
.. ...
.
.. ...
.............
.
....... ..........
.
.. ...
.
.
.
.
.
.....
...
.
.
......... ...
.
.
.
.....
.
.
...
....
.....
... .......................... ............
.....
.. .
..
.....
...
.. ................................
.....
..
.
.
.
.
.............
.....
.. .........
..
.
.....
.
.
.
.
.
.
.
.
.
.............
.
......
.
..
.
.
.
.
.
.
.
.
.
.
.
.
............. ..........
............. .....
.. ............ ....
.
..................
..
.. ..........
.
.............
..
...........
..................
.
.
..
..... ............
....
..
.
.....
...........
..
.
.
..... ........................
.
.
.
.
..
.....
..... ...
.
.
.
...................
.
.
.
..
... ..
...............................................
.
.
.
.
.
.
.
.
.
.
.
.
.
.
.
.
.
.
.
.
.
.
.
.
.
.
.
.
.
.
.
.
.
.
.
.
.
.
.
.
.
.
.
.
.
.
.
.
.
.
.
.
.
.
.. ......................................................
.......
..
.....................................................................................................
.
.
.
.
.
.
.
..

5. Calculadora diferente - Para calcular (2 + 3)  (0 + 3) utilizaremos a


propriedade (iii), e temos:
94

OBMEP 2007

Solucoes da Lista 8

Nvel 2

(2 + 3)  (0 + 3) = (2  0) + (3  3)
Agora, por (i) temos 2  0 = 2 2 = 4, e por (ii) temos 3  3 = 3. Portanto,

(2 + 3)  (0 + 3) = 4 + 3 = 7
Agora, para calcular 1024  48 vamos usar a mesma estrategia que acima.
Para isso, note que 1024 = 976 + 48 e 48 = 0 + 48.

1024  48 = (976 + 48)  (0 + 48)


= (976  0) + (48  48)
= 2 976 + 48
= 1952 + 48 = 2000.

6. Dois quadrados - Observemos que a area do quadrado maior menos a area


do quadrado menor e igual a 4 vezes a area procurada. Logo a area tracejada
e
102 42
100 16
=
= 25 4 = 21.
4
4

7. Paralelismo- Dado que IL e EU sao paralelas entao


mente, como RE e paralela a N I temos que

FN
FR

FI
.
FE

FU
FL

FE
.
FI

Analoga-

Logo,

FE
FI
FN FU
=

= 1.
FR FL
FI
FE

8. Um subconjunto -

Vamos construir o subconjunto pedido da seguinte

forma:
OBMEP 2007

95

Nvel 2

Solucoes da Lista 8

ele contem todos os n


umeros mpares: 1, 3, 5, . . . , 2999. Aqui ja temos
uma lista com 1500 n
umeros.
o conjunto nao pode conter os n
umeros que sao da forma 2(n
umero mpar),
o conjunto pode conter os n
umeros que sao da forma 4 (n
umero mpar),
isto e,

3 , 4 
5 , . . . , 4 749
4 
1 , 4 
4

12

20

2996

essa lista tem 749 n


umeros e nenhum e o dobro do outro. Alem disso,
nenhum deles e o dobro de um n
umero mpar.
Logo, o nosso conjunto ja possui 1500 + 749 = 2249 elemento; assim qualquer
subconjunto dele com 2000 elementos satisfaz as condicoes pedidas.

9. Tri
angulos ret
angulos - Observemos que os quatro triangulos que aparecem na gura sao triangulos retangulos semelhantes, e portanto seus lados sao
proporcionais. Em particular
v
9
y
= = .
8
x
20
Alem disso, pelo teorema de Pitagoras temos que y 2 = x2 + 92 e portanto
x2 + 81
y2
81
=
,
=
x2
400
400
assim x4 + 81x2 81 400 = 0 e




2
81 + 81 + 4 81 400
9 + 81 + 1600
9 + 41
=3
=3
= 15,
x=
2
2
2

9
donde y = 152 + 92 = 3 34, z = 202 x2 = 5 7, v = 8 15
= 24
e
5

nalmente w = 82 + v 2 = 85 34.

96

OBMEP 2007

Solucoes da Lista 8

Nvel 2

10. Uma desigualdade especial- Observemos que se um n


umero a satisfaz
a desigualdade, entao a tambem satisfaz a desigualdade, logo os valores
que satisfazem a desigualdade formam um conjunto simetrico, portanto basta
considerar o caso em que x e positivo. Mas, (2 x)(1 + x) = x + 3 x2 > 0 e
positivo se 2 x e positivo, portanto x < 2. Como a solucao e simetrica temos
que 2 < x < 2 e a solucao da equacao inicial. A opcao correta e (c).

OBMEP 2007

97

Lista 1

Nvel 3

Nvel 3
Lista 1
1. Equa
c
ao c
ubica - Sobre a equacao 2007x3 + 2006x2 + 2005x = 0 e certo
armar:
(a) Nao possui razes

(b) Tem 3 razes reais distintas

(c) Tem 2 razes iguais

(d) Tem apenas uma raiz real

(e) Tem 3 razes positivas

2. O perfume de Rosa - Rosa ganhou um vidro de perfume no formato de um


cilindro de 7 cm de raio da base e 10 cm de altura. Depois de duas semanas
usando o perfume restou 0, 45 l no vidro. Qual a fracao que representa o
volume que Rosa ja usou?

3. Igualdade com inteiros - Quais n


umeros naturais m e n satisfazem a
2n + 1 = m2 ?

4. O caminho da pulga - Para percorrer um caminho reto de 10 metros de


comprimento, uma pulga usa a seguinte estrategia: a cada dia ela percorre a
metade do caminho que faltava no dia anterior. Portanto, no primeiro dia ela
percorre 5 metros, no segundo 2,5 metros e assim por diante (o tamanho da
pulga e desprezvel).
(a) Quantos metros ela tera percorrido ao nal do setimo dia? E do decimo?
(b) A partir de qual dia a pulga estara a menos de 0, 001m do nal do caminho?
OBMEP 2007

99

Nvel 3

Lista 1

5. Uma soma alternada - Se Sn = 1 2 + 3 4 + 5 6 + . . . + (1)n+1 n,


onde n e um inteiro positivo, entao S1992 + S1993 e:
(a)

(b)

(c) 0

(d) 1

(e) 2

6. O raio da circunfer
encia - Um arco de circunferencia mede 300 e o seu
comprimento e 2 km. Qual o n
umero inteiro mais proximo da medida do raio
em metros?
(a) 157

(b) 284

(c) 382

(d) 628

(e) 764

7. Quatro passageiros - Em um taxi podem se sentar um passageiro na frente


e tres atras. De quantas maneiras podem se sentar os quatro passageiros se
um deles quer car na janela?

8. Os cinco crculos - Cinco discos


de mesmo raio estao dispostos como
mostra a gura. Quatro centros sao
os vertices de um quadrado e tres
estao alinhados. Trace uma reta que
divida a gura formada pelos 5 discos
em duas partes de mesma area.

100

.............................
..............................................
..............
.........
.........
.......
.........
......
.......
......
......
.....
.....
....
....
.
.
.
.
.
.
.
...
...
...
... ....
.
...
... ...
..
...
.
......
...
....
....
..
...
...
..
.
..
.
...
..
..
.
.
..
...
.
...
.........
....
. ..
.
...
.
.
...
... .....
...
....
....
....
....
......
......
....
.....
.......
.......
......
......
.
.
.
.
.
..........
.
.
.
.
.
.
.
.
.
.
.
.
.
.
.
.
.
................ ...............
.......................................
.....................
..............
...
.................................
..........
..............
.........
.........
............
............
.......
.........
........
........
......
......
......
......
......
......
.....
....
....
....
.
....
....
....
.
.
.
.
.
.
.
.
...
...
... ....
..
.
...
.
.
.
...
.
... ..
... ..
..
...
.
.
.
... .
... ..
...
.....
.
....
.
....
....
..
.
...
.
..
..
..
..
.
.
..
...
...
..
.
.
..
.
.
.
.
...
..
......
......
...
.
.
.
..
.. ...
.. ...
...
.
.
.
.
. .
. .
...
...
... ......
... ......
....
.....
.....
....
....
....
......
......
......
......
......
......
.......
.........
.........
......
......
......
.
.
.
..........
.
.
.
.
.
.
.
.
.
.
.
.
.
.
.
.
.
.
.
.
.
.
.
................... ..................
................... ..................
.....................................
...
...

OBMEP 2007

Solucoes da Lista 1

Nvel 3

Solu
co
es da Lista 1
1. Equa
c
ao c
ubica - Observemos que x = 0 e uma solucao, logo as possibilidades (a) e (e) cam descartadas. Agora so precisamos estudar as solucoes de
2007x2 + 2006x + 2005 = 0, que e uma equacao do 2o grau com discriminante
= 20062 4 2007 2005 = 20062 4(2006 + 1)(2006 1)
= 20062 4(20062 1) = 3 20062 + 4 < 0,
logo nao possui razes reais. Portanto, a equacao inicial tem uma u
nica raiz
real, e a opcao correta e (d).
Observacao: Uma outra maneira (e mais simples) de mostrar que < 0 e:
como 2006 < 2007 e 2006 < 4 2005, entao
2006 2006 < 4 2005 2007 = 20062 4 2005 2007 < 0.

2. O perfume de Rosa - O volume de um cilindro e o produto da area da


base pela altura. Como o raio da base e 7 cm, a area da base e: 72 , e entao
o volume do vidro e
72 10 cm3 = 490 cm3 =

490
dm3 = 0, 49 litros,
1000

lembrando que 1000 cm3 = 1 dm3 = 1 litro.


Depois de duas semanas, restaram 0, 45 litros de perfume, entao ela gastou
(0, 49 0, 45) litros. Portanto, a fracao que representa o volume gasto e:
0, 49 0, 45
49 45
volume gasto
=
=
.
volume total
0, 49
49

3. Igualdade com inteiros - Como


2n = m2 1 = (m + 1)(m 1),
OBMEP 2007

101

Nvel 3

Solucoes da Lista 1

temos que m 1 e m + 1 sao potencias de 2 cuja diferenca e 2. Logo, a


u
nica solucao possvel e m 1 = 2 e m + 1 = 22 , donde m = 3. Segue que
2n + 1 = 32 , e obtemos n = 3.

4. O caminho da pulga - No 1o pulo a pulga percorre 10 12 , no 2o , 10 212 ,


e assim por diante.

Os pulos da pulga
............................................
...........
........
.......
........
......
.......
......
......
.
.
.
.
.
.
.
.
.
.
.
.
.
.
.
....
.
.
.
..
........ ..............
.
.
....
.
.
.
.
.
.
.....
..
.
....
.
.
.
.
.
.
.
.
.
....
....
..
..
....................
.
.
.
.
.
.
.
.
.
.
....
....
.... ....
..
.
...
....
.
.
.
.
.
.
... .
.... ...
..
..
.
.....
.
.
.
...........................................................................................................................................................................................................................................................................................................

1, 25

10

2, 5

10

10

10

Depois de 7 dias a pulga tera percorrido

10

1
2


10

1
2

10

+ 10

 1 2
2

 1 2
2

+ 10

 1 3
2

 1 3

+ 10

 1 4
2

 1 4
2

 1 5
2

+ 10

 1 5

 1 6

+ 10

 1 7
2

 1 6
2

+ 10

 1 7
2

26 + 25 + 24 + 23 + 22 + 2 + 1
127
9, 9.
= 10
7
2
128

Logo em 7 dias ela percorreu, aproximadamente 9, 9 m. Em geral, depois de n


dias a pulga tera percorrido



1
1
10
+ + n
2
2
1
Para calcular a soma acima, note que 2 + +

1
2n

e a soma dos n termos

de uma progressao geometrica cujo primeiro termo e a1 = 1/2 e a razao e


q = 1/2. A formula para essa soma e:
Sn =
102

1/2(1 1/2n )
1
a1 (1 q n )
=
= 1 n.
1q
1 1/2
2
OBMEP 2007

Solucoes da Lista 1

Logo, 10

Nvel 3

1
1
+ + n
2
2

1
= 10 1 n
2


.

Portanto, ao nal do decimo dia a pulga tera percorrido 10(1

1
)m.
210

A pulga estara a menos de 0, 001m do nal do caminho, quando ela ja tiver


percorrido pelo menos 10 0, 001 = 9, 999, ou seja quando


1
10 1 n 9, 999.
2
Vamos determinar o menor valor de n que satisfaz a desigualdade acima.


1
10 1 n
2

= 0, 001

9, 999 = 10

10
10

9,
999
=
10

9,
999

2n
2n

10
2n

10
0, 001

= 2n

= 2n 10000.

Agora,
210 = 25 25 = 32 32 = 1024 ,
segue que
213 = 210 23 = 1024 8 = 8192
Logo, devemos ter n = 14.

5. Uma soma alternada Solu


c
ao 1: Lembre que

1
se n e mpar
n+1
=
(1)
1 se n e par
Observemos que associando duas a duas parcelas consecutivas,
(1 2) + (3 4) + (5 6) +
obtemos uma soma de n parcelas todas iguais a 1. Logo,
OBMEP 2007

103

Nvel 3

Solucoes da Lista 1

S1992 = (1 2) + (3 4) + (5 6) + + (1991 1992) = (1)996 = 996.





19922=996 parcelas

S1993 = (12)+(34)+(56)+ +(19911992)+1993 = 996+1993 = 997.


Assim, S1992 + S1193 = 996 + 997 = 1. A opcao correta e (d).
Solu
c
ao 2: Note que
S2n = (1 2) + (3 4) + (5 6) + + [2n (2n + 1)]



n parcelas iguais a 1
Obtemos que S2n = n e S2n+1 = S2n + (2n + 1) = n + 2n + 1 = n + 1.
Logo, S2n + S2n+1 = 1.

6. O raio da circunfer
encia
r.
Solu
c
ao 1: Se o raio e r entao o comprimento de um arco de graus e 2 360

Assim, no problema dado, temos que


2

300
3
r = 2000 m = r = 2000
 382, 17 m.
360
5

Logo, a opcao correta e (c).


Solu
c
ao 2: Como a circunferencia tem 360 , um arco de 300 representa
circunferencia, logo, seu comprimento e

5
6

da

do comprimento da circunferencia,

isto e:
5
2 000 6
1 200
2 r = 2 000 m = r =
=
382, 17 m.
6
10

104

5
6

OBMEP 2007

Solucoes da Lista 1

Nvel 3

7. Quatro passageiros - O passageiro que quer car na janela tem 3 possveis


lugares para se sentar, o seguinte pode-se sentar em qualquer lugar livre, logo
tem 3 possveis lugares, o seguinte dois possveis lugares, e o u
ltimo nao tem
escolha. Conclumos que o n
umero de formas de se sentar e 3 3 2 = 18.

8. Os cinco crculos - Observemos que qualquer linha que passe por O, o


centro do quadrado ABCD, divide a area formada pelos crculos C1 , C2 , C3 e
C4 na metade. Por outro lado, qualquer linha reta que passe por F divide a
area do circulo C5 na metade. Assim, a reta procurada e a reta F O.
C2

C1 ......................................................................

.................................
............
.........
........
......
.....
......
....
....
....
....
....
.
...
.
.
.
.
... ..
...
..
.
.
.
...
... ..
..
.
.
...
.
.....
..
.
....
....
..
...
...
..
..
.
.
..
..
..
...
..
.......
....................
...
.
... ...
...
.
.
.
.
. ...
.
...
.
.
.
.
.
.
.
.
. ..
...
........... .
....
............. ......
...
....
.....
.............
.
....
......
...... ........................
.....
.....
.......
...........
.......
.......
..........
.
.
.
.
.
.
.
.
.
.
.
.
.
.
.
.
.
.
.
.
.
.
.
.
.
.
.
.
.
.
.....................................
....................................
........
.
.
.
.
.
.
.
.
.
.
.
.
.
.
.
.
.
.
.
.
.
.
.
.
.
.
.
.
.
.
.
.
.
.
.
.
.
.
.
.
.
.
.
.
.
.
.
.
.
.
.
.
.
.
.
.
.
.
.
.
.
.
.
.
.
.
.
.
.
.
.
.
.
.
.
..........
.......... ............
..........
......
......
..........
.......
.......
.......
.........
.........
........
......
......
............. ..........
......
......
....
....
......
............
....
....
....
....
....
....
.............
....
... .....
... .... ........................
...
..
.
.
.
.
... .
...
....................
..
.
.
.
.
.
.....
...
............
.
.
.
.
.
.
.
.
.
.
.
.
....
.
.
....
..
....
.........
.
.
.
.
.
.
.
.
..
.
.
.
.
...
.
.
.
..
..
........
.
.
.
.
.
.
.
.
..
.
.
.
.
..
.
...
...
.......
.
.
.
..
.
.
.
..
.
.
.
.
.
.
.
.
.
.
.
.
... ................
....
....
..
.
.
.
.
.
.
.
.
.
.
. ...
. ...
.
...........
.
.
.
.
.
.
.
.
.
.
.
.
.
.
.
.
.
..
..
...
. ..
.
... .....
....
... ......
...
....
....
.....
.....
....
....
......
....
......
......
.....
.....
........
.........
.........
......
.......
.......
.
.
.
.
...........
.
.
.
.
.
.
................ ........................
.
.
.
.
.
.
.
.
.
.
.
.
...................................
...............................
...........

Ds

Cs

Os

C5

C4

OBMEP 2007

C3

105

Nvel 3

Lista 2

Lista 2
1. O tri
angulo e o quadrado - Na gura ABCD

B ............................................................................................................................... C
... ......
............ .....
.. ....
.............

e um quadrado cujo lado mede 1 cm, E e o ponto


medio da diagonal BD e F o ponto medio do segmento BE. Qual e a area do triangulo BCF ?

... ......
...
..
............
.....
...
...
.............
.....
...
...
............................
...
...
.....
...
...
.....
...
...
.
.
.
F .......
...
...
.
.....
...
...
..... E
...
...
.....
...
...
.
.....
...
...
.
.
.....
...
...
.....
...
...
.....
...
...
.
.....
...
...
.
.
.
.....
...
...
.....
...
...
.....
...
...
.
.....
...
.
.
.
..... .....
...
.
.
.
...
. ..
.........................................................................................................................

2. Uma refei
c
ao - Um sanduche e um prato de refeicao custam em media
R$ 5, 00 e R$ 7, 00, respectivamente. De quantas maneiras pode-se comprar
sanduches e pratos de refei cao com R$ 90, 00, sem deixar troco?

3. Plano Cartesiano - O ponto


P = (a, b) esta marcado na gura
ao lado. Marque os pontos:

1
0

(a) A = (a/2, b + 1)

(b) B = (a 1, b/2)
(c) C = (a, b)
(d) D = (1 a, b 1)

4. Soma dos terminados em 9 - A soma Sn = 9 + 19 + 29 + 39 + + an


denota a soma dos primeiros n n
umeros naturais terminados em 9. Qual e o
menor valor de n para que Sn seja maior do que 105 ?

5. Tr
es cilindros - Tres cilindros tem alturas e raios das bases iguais a 10cm
10cm , 10cm 5cm e 20cm 5 cm, e volumes V1 , V2 e V3 , respectivamente.
106

OBMEP 2007

Lista 2

Nvel 3

...................................................................
.
.......................................
.......................................... ....
...
...
...
..
..
..
..
..
..
..
...
...
...
...
..
..
..
..
..
..
..
...
...
.
............................................................................ ...
...............
.
.
..........................................................

.........................
......................... ........................
...
.......
...
...
..
..
..
..
..
..
..
...
...
...
...
..
..
..
..
..
..
..
..
...
...
..........................................
.
........
.
.............................

............................
......................
..
... ................................
...
...
...
...
..
..
...
...
...
...
...
...
...
...
...
...
...
...
...
...
...
...
...
...
...
...
...
...
...
...
...
...
...
...
...
...
...
...
...
...
...
...
...
...
...
...
.... .............. ....
.
...... .
.............
.............. ....................
........

(a) Escreva em ordem crescente os volumes V1 , V2 e V3 dos tres cilindros.


(b) De as dimensoes de um cilindro V4 cujo volume esteja entre V2 e V3 .
(c) De as dimensoes de um cilindro V5 cujo volume esteja entre V1 e V3 .

6. Percentagem de mortalidade - Se 15% dos membros de uma populacao


afetados por uma doenca 8% morreram, a percentagem da mortalidade em
relacao a` populacao inteira e:
(a) 1, 2%

(b) 1, 8%

(c) 8%

(d) 12%

(e) 23%

7. Agenda de aulas - Eliane quer escolher o seu horario para a natacao. Ela
quer ir a duas aulas por semana, uma de manha e a outra de tarde, nao sendo
no mesmo dia nem em dias seguidos. De manha, ha aulas de natacao de
segunda-feira a sabado, a`s 9 h, `as 10 h e `as 11 h e de tarde, de segunda-feira
a sexta-feira, `as 17 h e `as 18 h. De quantas maneiras distintas pode Eliane
escolher o seu horario?

8. Jogo de cartas - Um grupo de amigos disputa um jogo que consiste em


mover sucessivamente a carta superior de uma pilha e coloca-la sobre uma
OBMEP 2007

107

Nvel 3

Lista 2

outra pilha, ate obter 4 novas pilhas que sao da forma: na Pilha 1 so tem

Ases,
na Pilha 2 so tem Valetes, na Pilha 3 so tem Damas e na Pilha 4 so
tem Reis. Ganha o jogo quem zer o menor n
umero de movimentos. Qual e o
n
umero de movimentos que sera sempre o vencedor?

108

Pilha 1

Pilha 2

Pilha 3

Pilha 4

Rei de

Dama de

Rei de 

Valete de

Dama de 

de 
As

Valete de

Rei de

Valete de 

de
As

Dama de

de
As

de
As

Valete de

Dama de

Rei de

OBMEP 2007

Solucoes da Lista 2

Nvel 3

Solu
co
es da Lista 2
B ................................................................................................................................ C
.. ...
... ......
.............

1. O tri
angulo e o quadrado -

As diagonais

do quadrado ABCD dividem o quadrado em 4


triangulos iguais, logo a area do triangulo BCE
e

... ......
...
.....
... ......
...
............
.............
.....
...
...
..... ........................
...
...
............
...
...
.....
...
...
.
.....
...
...
.
F
.
.
.....
...
...
.....
...
...
..... E
...
...
.
.....
...
...
.
.
.
.....
...
...
.....
...
...
.....
...
...
.
.....
...
...
.
.
.
.....
...
...
.
.
.
...
...
.....
.....
...
...
.....
...
..... .....
...
.
.
.
..... ....
...
..... ..
...
.................................................................................................................

1 4 = 0, 25 cm2 .
Como o comprimento de BF e a metade de BE e a altura relativa aos lados
BF e BE e CE, entao a area do triangulo CBF e a metade da area do
triangulo CBE. Assim, a area de dito triangulo e 0, 25 2 = 0, 125 cm2 .

2. Uma refei
c
ao - Se S corresponde ao n u
mero de sanduches e P o n
umero
de pratos de refeicao, entao 5S + 7P = 90. Precisamos encontrar solucoes
inteiras P e Q para essa equacao. Temos:
5S + 7P = 90 = P =

18 S
90 5S
=5
.
7
7

Como P e um n
umero natural temos que 7 tem que dividir 18S, assim S = 4,
11 ou 18, e em cada um destes casos P e igual a 10, 5 e 0, respectivamente.
Portanto, temos somente tres formas de fazer a compra.

3. Plano Cartesiano - As coordenadas do ponto


2

P satisfazem:

rP

0<a<1

e
1 < b < 2.
Lembremos tres propriedades de desigualdades:
OBMEP 2007

109

Nvel 3

Solucoes da Lista 2

(i) Uma desigualdade nao se altera se somarmos (ou subtrairmos) um mesmo


n
umero a ambos os seus membros: x < y = x + z < y + z.
(ii) Uma desigualdade nao se altera se multiplicarmos por um n
umero positivo ambos os seus membros: x < y = xz < yz,

z > 0.

(iii) Uma desigualdade inverte o seu sentido se multiplicarmos por um n


umero
negativo ambos os seus membros: x < y = xz > yz,

z < 0.

Assim temos:
(a) 0 < a/2 < 1/2 e 1 + 1 < b + 1 < 2 + 1;
(b) 1 < a 1 < 0 e 1/2 < b/2 < 1;
(c) 1 < a < 0 e

2 < b < 1;

(d) 0 < 1 a < 1 e 0 < b 1 < 1.


A gura mostra os pontos no Plano Cartesiano.

6
rA
2

r
r

P
1

D
0

4. Soma dos terminados em 9 - Note que Sn e a soma dos n primeiros


termos de uma progressao aritmetica cujo primeiro termo e a1 = 9 e a razao
110

OBMEP 2007

Solucoes da Lista 2

Nvel 3

e r = 10. Substituindo esses dados na formula an = a1 + (n 1)r obtemos


an = 9 + 10(n 1). Por outro lado, note que:

= 9 + 0 10

19

= 9 + 1 10

29

= 9 + 2 10

39

= 9 + 3 10

an

= 9 + (n 1) 10

Entao temos:
Sn = 9 + 19 + 29 + + an
= (9 + 0) + (9 + 10) + (9 + 2 10) + + [9 + (n 1) 10]
= 9n + [1 10 + 2 10 + 3 10 + + (n 1) 10]
= 9n + [1 + 2 + 3 + + (n 1)] 10
n(n 1)
10
= 9n +
2
= 9n + 5 n(n 1)
= 5n2 + 4n
Como queremos que Sn 105 entao precisamos encontrar o menor valor inteiro
positivo n tal que 5n2 + 4n 105 , ou equivalentemente, 5n2 + 4n 105 0.
Resolvendo a equacao 5x2 + 4x 105 = 0 temos que

4 16 + 20 105
.
x=
10

4 + 16 + 20 105
A raiz positiva e x1 =
 141, 02.
10
Mas, 5x2 + 4x 105 e positiva fora das razes. Como estamos procurando o
menor n
umero positivo e inteiro tal que 5x2 + 4x 105 0, entao n = 142.

OBMEP 2007

111

Nvel 3

Solucoes da Lista 2

5. Tr
es cilindros (a) Dado que o volume de um cilindro de raio R e altura h e R2 h temos que
os volumes V1 , V2 e V3 sao:
V1 = 103 = 1000 , V2 = 52 10 = 250 , V3 = 52 20 = 500
Assim, temos entao que V1 > V3 > V2 .
(b) Como os dois cilindros tem o mesmo raio, basta manter o raio do cilindro
com 5 cm e a altura entre 10 cm e 20 cm, por exemplo: h = 15 cm. Neste caso,
o volume V4 e: 52 15 = 375 cm3 .
(c) Para construir um cilindro de volume V5 entre V1 e V3 , podemos diminuir
o raio do cilindro de volume V5 para 8 cm e tomar como altura 10 cm, a menor
das duas alturas, obtendo um cilindro de volume 82 10 = 640 cm3 .

6. Percentagem de mortalidade - A proporcao de populacao que ca doente


pela enfermidade e
8
.
100

15
100

e dos que cam doentes, a proporcao que morre e

Logo, a proporcao de populacao que morre pela doenca e

15
100

8
,
100

que

corresponde a
12
1, 2
120
15 8
=
=
= 1, 2%.
=
1002
10000
1000
100
A opcao correta e (a).

7. Agenda de aulas - Se a aula da manha e segunda ou sexta (em qualquer


dos tres horarios), entao o dia da aula de tarde pode ser escolhida de 3 formas
diferentes (em qualquer dos dois horarios), assim temos 2 3 3 2 = 36
formas diferentes de escolher o horario. No caso em que a aula de manha seja
sabado entao o dia da aula da tarde pode ser qualquer dia de segunda a quinta,
assim temos 3 4 2 = 24 possveis formas. Por u
ltimo, se a aula da manha
112

OBMEP 2007

Solucoes da Lista 2

Nvel 3

e terca, quarta ou quinta, entao a aula da tarde so pode ser escolhida de duas
formas, assim temos 3 3 2 2 = 36 formas. Logo a Eliana pode escolher
seu horario de 36 + 24 + 36 = 96 formas distintas.

8. Jogo de Cartas - A estrategia abaixo permite realizar o jogo com 17 movimentos.


O primeiro n
umero indica a pilha sobre a qual a carta e tomada e o segundo
a pilha onde a carta e colocada, por exemplo: Movimento 1= pegar a carta
superior na Pilha 4 e colocar na Pilha 2.
(1) 4 sobre 2

(2) 4 sobre 3

(3)4 sobre 2

(4) 3 sobre 4

(5) 3 sobre 4

(6) 1 sobre 4

(7) 3 sobre 4

(8)1 sobre3

(9) 1 sobre 4

(10) 2 sobre 1

(11) 2 sobre 4

(12) 2 sobre 3

(13)2 sobre1

(14) 2 sobre 1

(15) 4 sobre2

(16) 4 sobre 2

(17) 4 sobre 2

O movimento 2 poderia ser tambem 4 sobre 1, o movimento 4 poderia ser


1 sobre 4, o movimento 5 poderia ser 1 sobre 4, o movimento 6 poderia ser
3 sobre 4. Os movimentos 4, 5 e 6 poderiam ser permutados em qualquer
ordem. Teramos assim, pelo menos, seis maneiras de se realizar o jogo com
17 movimentos.
Esse jogo pode ser realizado com um n
umero menor de movimentos?

OBMEP 2007

113

Nvel 3

Lista 3

Lista 3
1. Fra
c
oes inteiras - Quantos n
umeros inteiros positivos n existem tais que
2n2 + 4n + 18
e um inteiro?
3n + 3

2. Quatro prefeitos e um crculo - Quatro prefeitos decidem construir uma


rodovia circular que passe em suas cidades, entretanto, as quatro cidades nao
estao sobre um mesmo crculo. Eles contratam uma empresa para elaborar um
projeto para a construcao da rodovia circular eq
uidistante das quatro cidades.
Qual o maior n
umero de projetos geogracamente distintos que a empresa
elaborou?

3. Fatoriais - Se n e um n
umero natural, denotamos por n! o produto de
todos os inteiros de 1 a n. Por exemplo: 5! = 1 2 3 4 5 e 13! =
1 2 3 4 5 . . . 12 13. Por convencao, 0! = 1. Encontre tres n
umeros
inteiros diferentes a, b e c, entre 0 e 9 tais que o n
umero de tr es algarismos
a b c e igual a a! + b! + c!.

4. Para a escola de bicicleta - Catia sai da escola todos os dias no mesmo


horario e volta para casa de bicicleta. Quando ela pedala a 20km/h, ela chega
em casa a`s 4 : 30 horas da tarde. Se ela pedalar a 10km/h, ela chega em casa
`as 5 : 15 horas da tarde. A qual velocidade ela deve pedalar para chegar em
casa a`s 17 : 00 horas?

5. O Riquinho - Riquinho distribuiu R$ 1000, 00 reais entre os seus amigos:


Antonio, Bernardo e Carlos da seguinte maneira: deu, sucessivamente, 1 real
114

OBMEP 2007

Lista 3

Nvel 3

ao Antonio, 2 reais ao Bernardo, 3 reais ao Carlos, 4 reais ao Antonio, 5 reais


ao Bernardo, etc. Quanto que o Bernardo recebeu?

6. Ret
angulo com dimens
oes inteiras - As diagonais de um retangulo me
dem 1993 cm. Quais sao suas dimensoes, sabendo que elas sao n
umeros
inteiros?

7. M
ultiplos de 3 e quadrados perfeitos - Escreve-se em ordem crescente
cada m
ultiplo de 3 cuja soma com o n
umero 1 e um quadrado perfeito:

3 ;

15 ;

24 ;

48 ;

...

Qual e o m
ultiplo na posicao 2006o ?

8. Cinco cartas - As cinco cartas abaixo estao sobre uma mesa, e cada uma
tem um n
umero numa face e uma letra na outra. Simone deve decidir se a
seguinte frase e verdadeira: Se uma carta tem uma vogal numa face, ent
ao
ela tem um n
umero par na outra. Qual o menor n
umero de cartas que ela
precisa virar para decidir corretamente?
.........................
...
...
..
..
...
...
..
.
......................

.........................
...
...
..
..
...
...
.
..
.......................

.........................
...
...
..
...
...
...
........................

OBMEP 2007

.........................
...
...
..
...
...
...
........................

.........................
...
...
..
...
...
...
........................

115

Nvel 3

Solucoes da Lista 3

Solu
co
es da Lista 3
1. Fra
c
oes inteiras - Como
2n2 + 4n + 18
2 (n2 + 2n + 1) + 8
1
=
=
3n + 3
3
n+1
3

16
2n + 2 +
n+1


,

segue que n + 1 tem que dividir 16. Assim, n tem que pertencer ao conjunto
{1, 3, 7, 15}. Em cada um destes casos temos

2n2 +4n+18
3n+3

15

11

Portanto para os quatro valores de n, 1, 3, 7 e 11, tem- se que

2n2 + 4n + 18
3n + 3

e um inteiro.

2. Os prefeitos e o crculo - O n
umero de rodovias e igual ao n
umero de
pontos que podem ser centros da circunferencia formada pelas rodovias.
Observemos por outra parte que podemos ter dois tipos de conguracao. Na
primeira conguracao a circunferencia divide o conjunto das 4 cidades em dois
conjuntos: um conjunto com 3 cidades e outro com una cidade. Na segunda
conguracao a circunferencia divide o conjunto das cidades, em dois conjuntos,
cada um deles com 2 cidades.
Nas guras abaixo esta ilustrado um exemplo de cada um destas conguracoes
onde a circunferencia contnua e a rodovia planejada.
116

OBMEP 2007

Solucoes da Lista 3

Nvel 3
.... .... .... .... .....
..... .
..... ....
...
....
....
....
...................................................
.
.
...
..
.
.
.
.
.
.
.
.
.
........
......
..
.
.
.
.......
...
.
.
.
..
...... . ...
.... .... ......... .... ..
.
.
.
... .........
.
....
. ... .. ....
...
.
.
.
... ......
.
.... ...
..
. ....
...
.
.
.
.
.
.
.
...
.
.
... .... ..
...
... ...
...
... ...
.. .... ...
...
...
.
..
.
.. .... ..
.. .... ..
.
..........
.
..
..
... ....
.. ..
..
......... ................. ....
.
.
..
.
.
.
.
...
. ...........
...
.. .
.
.
.
.
.. .
... .. ...
..
..
..
..
..
.
.
... .. ...
..
.
..
.
.
.
.
.. .... ..
.
...
..
.
.
.
..
.
..
..
... .... ..
..
.. ............................... .... ...
. .......
... .. ...
.
..
.
..
...
... ... ....................................
... ...... ..
.........
.
... ......
.
.
.
.
... ...
.... ..
.....
....
...
.. ..
.
......
.....
.... ..
... ....
.... ..
........
......
...
...
.......
..........
...
................... ............................
...
.
.
.
.
.
.
.
.
.
.
.
.
....
..
.
.
....
.
.
..... ..
....
..
.... ..
.... ..
... .....
... ..
... ..

qs

sq

.... .... .... .... .....


..... .
..... .....
...
....
....
....
........................................
.
.
.
.
.
.
.
.
.
...
.
.
.
.
.........
......
.
.
.
.
.
.
.......
.
...
.
..
.....
......
.
.
.
.
.
.
.
.
.
.
.
.
.
.
.
.
.
...
.
.
.... ...
.....
.
....
.... .
.
.
.
.
.
.
.
.
.
....
..
...
..
..
.
.
.
.
.
.
.
.
... ...
..
..
...
...
.
.
.
.
.
.
........................................................................................ ....
... ....
... ...
...
.
.
..
.
.
..
.
.
..
.....
..
..
..
...
... ....
..
..
.
..
..
....
...
...
..
..
...
..
.
.
.
.
..
..
..
...
.
.
.
.
.
.
.
.
...
.
.
.. ...
.
... ......
...
.
..
.. .......... .
... .... ...
...
...
..
... .......... ..
...
..
....
..
.
.. ...................
.
.
.
...
....
... ... ........ ...
...
...
.... ..
....
..... ..... ..
.. ..
...
......
.... ..
...
... .... .............. ..........
.... ..
.......
...
...
..... .....
.........
............................
.
.............
.
...
.
.
.
.
.
.
.
.................................
....
....
....
...
.......
..... ..
.... .
..
.......
............
..... ..
... .....
... ..
... ..

Na primeira conguracao temos que o centro da circunferencia esta na mesma


distancia das tres cidades que cam do mesmo lado da rodovia e assim o centro
desta rodovia e o centro da circunferencia circunscrita ao triangulo formado
pelas tres cidades. Logo, o n
umero de rodovias e igual ao n
umero de triangulos
que podemos formar com as 4 cidades, isto e, 4 possveis rodovias.
Na segunda conguracao, temos que o centro da circunferencia formada pela
rodovia esta sobre a mediatriz das duas cidades que cam na parte interna
da rodovia e tambem sobre a mediatriz das duas cidades que cam na parte
externa da rodovia. Assim, o n
umero de rodovias e igual ao n
umero de formas
de dividir o conjunto de 4 elementos em dois conjuntos com 2 elementos cada
um, isto e 3 formas.
Logo o n
umero possvel de projetos e 4 + 3 = 7.

3. Fatoriais - Primeiramente observe que como o n u


mero tem 3 algarismos,
entao o maior dos algarismos tem que ser menor que ou igual a 6, ja que
7! > 1000. Como o n
umero tem que ter 3 algarismos e 4! = 1 2 3 4 = 12
entao um dos algarismos tem que ser 5 ou 6, mas 6! = 720 implicaria que a
soma teria um algarismo maior ou igual a 7, logo o maior dos algarismos e 5.
Por outra parte, 5! = 120 e 5! + 4! + 3! = 120 + 24 + 6 = 150, assim a soma
dos fatoriais esta entre 100 e 150, portanto o algarismo das centenas e 1. Por
u
ltimo como 5! + 1! + 4! = 145, entao 145 e solucao.

OBMEP 2007

117

Nvel 3

Solucoes da Lista 3

4. Para a escola de bicicleta - Seja t o tempo que ela gasta pedalando a


20km/h. Pedalando a 10 km/h, ela faz o percurso no dobro do tempo que
pedalando a 20km/h, isto e 2t. No entanto, como ela demora 45 minutos a
mais temos:
2t t = 45 = t = 45min.
Logo, diariamente ela sai da escola a`s
4 : 30 h 45 min = 3 : 45 h
e o percurso ate em casa e de
45min 20km/h =

3
20 = 15km .
4

Para percorrer 15km em 5 : 00 h 3 : 45 h = 1 : 15 h = 54 h, ela deve manter


uma velocidade de
15km
= 12km/h.
5
h
4

5. O Riquinho - O dinheiro foi repartido em parcelas na forma


1 + 2 + 3 + + n 1000.
umeros naturais a
Como 1 + 2 + 3 + + n e a soma Sn dos n primeiros n
partir de a1 = 1 temos:

Sn =

(1 + n)n
(a1 + an )n
=
1000 = n2 + n 2000 0.
2
2

Temos que
n2 + n 2000 < 0 para valores de n entre as razes .
Como a solucao positiva de n2 + n 2000 = 0 e

1 + 1 + 8000
 44, 22 ,
n=
2
118

OBMEP 2007

Solucoes da Lista 3

Nvel 3

entao n 44. Assim Bernardo recebeu


2 + 5 + 8 + 11 + + 44 =

(44 + 2) 15
= 23 15 = 345.
2

6. Ret
angulo com dimens
oes inteiras - Se a b sao os comprimentos dos
lados do retangulo, entao pelo teorema de Pitagoras temos
a2 + b2 = 1993 .
Como a2 b2 , segue que
2a2 a2 + b2 = 1993 > a2 .
Logo,

1993 > a

996, 5 .

Assim, 44 a 32. Usando o fato que a2 (a 1)2 = 2a 1 podemos


completar a seguinte tabela, somando aos elementos da segunda coluna na
linha a 1 o n
umero 2a 1 para obter o elemento da segunda coluna na
linha a.
a

b2 = 1993 a2

44

57

43

144

42
..
.

229
..
.

Assim, temos que a = 43 e b = 12 e solucao.

7. M
ultiplos de 3 e quadrados perfeitos - Chamemos a um n
umero qualquer da lista, entao sabemos que:
OBMEP 2007

119

Nvel 3

Solucoes da Lista 3

a e m
ultiplo de 3
umero natural.
a + 1 e um quadrado: a + 1 = k 2 , sendo k um n
Assim a = k 2 1, e logo
a = (k 1)(k + 1)
Como a e divisvel por 3, entao ou k + 1 ou k 1 e divisvel por 3. Logo, k
nao e divisvel por 3, portanto, k tem que ser da forma 3n + 1 ou 3n + 2, ou
seja para cada valor de n temos dois n
umeros que nao sao m
ultiplos de 3.
O n
umero desta lista que esta na posicao 2006 e 2006 32 1 = 3008, e neste
caso a = 30082 1.

8. Cinco cartas .........................


...
...
..
..
...
...
..
.
......................

.........................
...
...
..
..
...
...
.
..
......................

.........................
...
...
..
..
...
...
.
..
......................

.........................
...
...
..
..
...
...
.
..
.......................

.........................
...
...
..
...
...
...
.........................

Ela nao precisa virar a carta que tem o n


umero 2, porque sendo vogal ou
consoante, ela cumpre a condicao, de igual forma. Ela tambem nao precisa
virar a carta com a letra M. A carta que tem o n
umero 3 tem que ser virada,
para comprovar que na outra face tem uma consoante, e tambem as cartas
com a letra A e a letra E tem que ser viradas para vericar que os n
umeros
na outra face sao pares. Assim, ela precisa virar somente 3 cartas.

120

OBMEP 2007

Lista 4

Nvel 3

Lista 4
1. Lucro de uma companhia - Uma companhia tem um lucro de 6% nos
primeiros R$ 1000, 00 reais de venda diaria, e 5% em todas as vendas que
excedem R$ 1000, 00 reais, nesse mesmo dia. Qual e o lucro dessa companhia
num dia que as vendas alcancam R$ 6000, 00 reais?
(a) R$250

(b) R$300

(c) $310

(d) R$320

(e) R$360

2. Seq
u
encia triangular - Qual e o 21o termo da seq
uencia
1 ; 2 + 3 ; 4 + 5 + 6 ; 7 + 8 + 9 + 10 ; 11 + 12 + 13 + 14 + 15 ; . . . ?

3. O jardim octogonal - A gura mostra a planta


de um jardim de uma cidade, feita num papel quadriculado. O jardim tem a forma de um polgono
de oito lados com uma roseira quadrada no centro,
cercada de grama. A area total do jardim e de
700 m2 . Para colocar uma cerca em volta do jar-

................................
.....
....
.
.
.....
..
.
.
.
.....
..
.
.
....
.............
.
.
.
.
..
....
.... ........
.
.
.
.
.....
.
.....
.....
..
..
.....
.....
.
.
.
.
...... roseira ......
.....
.
..
..
.....
.....
.
.
.
.
.....
..... ..........
.
..
..... .....
.......
.....
..
.
.
.
.....
..
.....
...
.
.
.
.....
..................................

dim e da roseira, o prefeito dispoe de no maximo


R$650, 00. Qual o maior preco que ele podera pagar pelo metro de cerca?

4. N
umero de caracteres - Numa folha de papel cabem 100 caracteres na
largura e 100 na altura. Nessa folha sao escritos sucessivamente os n
umeros
1, 2, 3, . . . com um espaco entre cada um. Quando no nal de uma linha nao
ha espaco para escrever um n
umero este e escrito na linha seguinte. Qual e o
ultimo n
umero escrito na folha?

OBMEP 2007

121

Nvel 3

Lista 4

5. A a
rvore de Emlia - A arvore de Emlia cresce
r

..
...
.
..
..
..
..
...
..
..
...
...
...
..
..
..
..
...
..
..
..
...
...
.
.
.
..
.
.
.
...
..
..
...
...
..
...
....
..
..
... ..............
..
....
..
.. ...
.............
..
................
...
..
.
.
..
...
..
..
...
..
...
...
..
...
...
..
...
...
..
.
.
...
...
..
..
...
..
....
..
......
....
..
..... ...
.............................. ....
..
.
.
...
...
...
...
...
....
...
..
......
.
...
... .....
...
... ......
.......................
.....
....
...
..
...
...
...
.....
...
.
...

de acordo com a seguinte regra: apos 2 semanas

do aparecimento de um galho, esse mesmo galho

produz um novo galho a cada semana, e o galho


original continua a crescer. A arvore tem 5 galhos

depois de 5 semanas, como mostra a gura. Quan-

tos galhos, incluindo o galho principal a arvore tera


no nal de 8 semanas?

6. Um teste vocacional - Foi feito um teste vocacional em 1000 estudantes


de uma escola. A tabela a seguir apresenta os resultados por area de estudo e
sexo.
Exatas

Humanas

Biologicas

Masculino

232

116

207

Feminino

112

153

180

Se um aluno e escolhido ao acaso, determine a probabilidade de:


(a) Ser da area de exatas.
(b) Ser da area de humanas, sendo do sexo masculino.
(c) Ser do sexo feminino, dado que e da area biologica.

7. Dois setores circular - A area do crculo


 = 60
da gura ao lado mede 20 cm2 . Se AOB
 = 30 , quanto mede a area da regiao do
e COD

tC

tD

crculo que esta tracejada?

122

B
t

At

................................
..................
..........
..........
.........
.........
....
....... .....
... ...........
......
.
.
..
.
....
.
.
...
..
..
.
....
.
.
.
.
.
...
....
.
.
....
.
.
...
...
.
..
.
.
.
.
...
...
.
.
..
.
.
...
...
..
..
...
....
.
..
... ...
...
..
... ...
.
...
............................................................................
..
........
..
..
........
..
..
........
..
.
.
........
...
..
........
...
........
..
........ ...
...
........ ..
...
....
....
.
....
....
....
...
......
....
.......
......
........
.......
.
.
.
.
.
.
..........
.
.................... ............................
..........

OBMEP 2007

Lista 4

Nvel 3

8. Compra de televisores - Maria encomendou certo n


umero de televisores a
R$ 1 994, 00 cada um. Ela reparou que no total a pagar, nao tem nem 0, nem 7,
nem 8 e nem 9. Qual foi o menor n
umero de televisores que ela encomendou?

OBMEP 2007

123

Nvel 3

Solucoes da Lista 4

Solu
co
es da Lista 4
1. Lucro de uma companhia - (c) Nos primeiros R$ 1000 reais a companhia
tem lucro de R$ 60 reais, e para os R$ 5000 reais restantes tem lucro de 5000
5% = 250 reais. Logo o lucro da empresa nesse dia e R$ 310 reais.

umeros
2. Seq
u
encia triangular - Observe que o 21o termo e a soma de 21 n
consecutivos. Tomando a primeira parcela de cada termo, isto e, 1,2,4,7,11,
16, . . . , temos que
2 = 1+1
4 = 2+1+1
7 = 3+2+1+1
11 = 4 + 3 + 2 + 1 + 1
16 = 5 + 4 + 3 + 2 + 1 + 1
..
.
Assim, a primeira parcela do 21o termo e
20 + 19 + + 3 + 2 + 1 + 1 =

20 21
+ 1 = 211
2

e o 21o termo e
211 + 212 + + 230 + 231 =

(211 + 231) 21
= 221 21 = 4641.
2

3. O jardim octogonal - Observemos que a area


do jardim pode ser medida contando o n
umero de
quadradinhos na folha. De fato, se contarmos o
n
umero de quadrados obtemos em total

124

OBMEP 2007

.........................
.....
....
.
.
.
.....
..
.
.
.....
..
.
.
......
....
..... .....
..... .........
.
.
.
..
..
.
.....
...
.
.
.
.
.
.
.....
........ roseira
..
..
..
.....
.....
.
.....
.
.
.
..... .......
.
..... .....
.......
......
.
.
.
.
.....
.
...
.....
....
.....
.
.
......................

Solucoes da Lista 4

Nvel 3

24 quadradinhos + 8 meios quadradinhos = 28 quadradinhos

Como a area total e 700 m2 , a area de cada quadradinho corresponde a


700 28 = 25 m2 .
Assim, o lado de cada quadradinho corresponde a 5 m. Pelo Teorema de

Pitagoras, a diagonal d de cada quadradinho corresponde a d = 52 + 52 =

5 2 m.
O contorno da roseira e formado por 4 diagonais e do jardim por 8 diagonais
e 8 lados, logo temos:

permetro da roseira = 4 d = 4 5 2 = 20 2 m

permetro do jardim = 8 5 + 8 d = 40 + 40 2
Logo, o comprimento total de cerca que sera necessario e

20 2 + 40 + 40 2 = 40 + 60 2 m
Agora temos:
65
65
650
65
=

=
5, 21.
4 + 6 1, 414
12, 484
40 + 60 2
4+6 2
Assim, o preco maximo que o prefeito podera pagar e R$5, 21.

umeros de 1 a 9,
4. N
umero de caracteres - Na 1a linha escrevemos os n
cada um seguido de um espaco, ocupando 18 espacos, e sobram 82 espacos.
Cada n
umero de 2 algarismos mais um espaco ocupa 3 lugares na linha. Como
OBMEP 2007

125

Nvel 3

Solucoes da Lista 4

82 = 27 3 + 1, completamos a 1a linha com 27 n


umeros de dois algarismos
a partir do 10. Logo o u
ltimo n
umero da primeira linha e 36. Representando
cada espaco por um traco, a 1a linha ca como

36
1 2 3 4 5 6 7 8 9 10
 
18

82

Como 100 = 33 3 + 1, em cada linha podemos colocar 33 n


umeros de 2
algarismos, cada um seguido de um espaco, e no nal da linha ainda sobra um
espaco:
2a linha : 37
 38  69
99

Na 3a linha, colocamos de 70 a 99, ocupando 30 3 = 90 espacos. Os 10


espacos restantes ocupamos com dois n
umeros de 3 algarismos:
3a linha : 70
99 100
 
 101
90

Agora, em cada linha podemos colocar 100 4 = 25 n


umeros de 3 algarismos
com seus respectivos espacos. De 102 a 999 inclusive temos 999102+1 = 198
n
umeros. Como 198 = 257+23, ocupamos da 4a a 10a linha com os n
umeros
de 3 algarismos e ainda sobram 23 espacos na 10a linha, que podemos ocupar
com 23 5 = 4 n
umeros de 4 algarismos:
999 1000
1001
10a linha :
 1002
 

67

23

Em cada linha podemos colocar 100 5 = 20 n


umeros de 4 algarismos e
seus respectivos espacos. Portanto, nas 90 linhas restantes podemos colocar
90 20 = 1800 n
umeros de 4 algarismos. Comecando com 1003 chegaremos
ate o n
umero 2802.

126

OBMEP 2007

Solucoes da Lista 4

Nvel 3

5. A a
rvore de Emlia - Denotemos por fn o n
umero
de galhos da arvore depois de n semanas. Como depois de duas semanas aparece um galho entao f2 = 1,
Na seguinte semana este galho produz um novo galho, logo f3 = 2. Pela regra, o n
umero de galhos na
n + 1-esima semana e igual ao n
umero de galhos que
existiam na n-esima semana, mais os galhos novos.
Mas, os galhos novos nascem dos galhos que tem pelo
menos duas semanas, isto e, nasce um galho novo por

..
...
..
..
..
..
...
...
...
..
..
...
..
..
..
..
...
..
..
..
..
.
.
.
...
..
.
.
.
.
..
...
..
....
..
..
...
.....
...
..
..
... ..............
...
..
..
............
.. ....
..
...
..............
..
.
..
...
..
..
...
..
...
...
..
...
..
...
..
...
...
.
.
...
...
..
..
...
..
....
.....
..
..
....
..... ..
..
.............................. .....
.
.
.....
...
.....
...
..
...
..
.
.......
.
.
... ....
...
...
.... ........
......................
...
....
...
...
...
...
....
...
....
.....
...
.

r
r

r
r

cada galho que existia na semana n 1.


Assim, temos que fn+1 = fn + fn1 . Logo:
f4 = 2 + 1 = 3
f5 = 3 + 2 = 5
f6 = 5 + 3 = 8
f7 = 8 + 5 = 13
f8 = 13 + 8 = 21

6. Um teste vocacional (a) De exatas temos 232 + 112 = 344 estudantes, logo a probabilidade de
escolher ao acaso um aluno de exatas e

344
1000

= 0, 344.

(b) Como o n
umero de estudantes do sexo masculino e 555, temos que a
probabilidade de ser da area de humanas e

116
555

= 0, 209.

(c) O n
umero de estudantes da area biologica e 387. Assim, a probabilidade
de escolher um do sexo feminino e

180
387

OBMEP 2007

= 0, 465.

127

Nvel 3
7. Dois setores circular - Como

Solucoes da Lista 4
B
t

At

60 + 30 = 90 ,
segue que area tracejada representa um quarto da
area total del crculo. Como a area do circulo e
20 cm2 entao a area tracejada e 5 cm2 .

..........................................................
..........
........
.........
..
....... .....
... ............
......
....
..
.
.
.
...
..
..
....
.
.
.
.
.
.
...
....
.
.
....
.
...
.
.
...
.
..
.
...
.
.
...
.
..
.
...
.
.
.
...
.
...
.
....
.
...
..
...
... ....
..
... ...
.
...
............................................................................
..
........
.
..
.
..
........
..
..
........
..
.
........
...
..
........
...
..
.
.
.
.
.
.
.
........
...
........ ...
...
.......
....
.
....
....
....
....
.
......
...
.
.
.
.......
.
.
....
.
........
.
.
.
.
.
.
..
..........
.................... .............................
..........

tC

tD

8. Compra de televisores - Se Maria comprou n televisores, entao ela gastou


1994n, que e um m
ultiplo de 1994 onde nao aparecem os algarismos 0, 7, 8
e 9. Vamos tentar limitar o valor de n. Primeiro observe que
1994n = 2000n 6n
e tambem que se
6n < 300
entao o n
umero 2000n 6n tem 7 ou 8 ou 9 no algarismos das centenas (faca
alguns exemplos, lembre-se que 2000n termina com 3 zeros e depois convencase). Assim devemos ter
6n 300,

isto e n 50 .

Observemos que 50 nao pode ser porque o valor terminaria em 0, logo n 51.
Dado que 1994 51 = 101694 temos que n nao pode ser 51 e portanto n =
51 + m com m positivo. Agora como precisamos que o n
umero nao tenha 0,
assim 1994m tem que eliminar o 0 de 101694, portanto m 4, mas m = 4
nao e solucao porque 1994 55 termina em 0. Se testamos n = 56 temos que
1994 56 = 111664
e o n
umero procurado.

128

OBMEP 2007

Lista 5

Nvel 3

Lista 5
1. Dist
ancia entre n
umeros - Considere os n
umeros reais a, b, c e d representados em uma reta, conforme mostra a gura. Determine quais das armacoes
sao verdadeiras ou falsas.
a
4

(a)

|a| < 4

(b) |b| < 2

(e)

|c| < |d|

(f)

(i)

|c d| < 1 (j)

|a| < |d|

d
0

(c) |c| < 2

(d) |a| > |b|

(g) |a b| < 4 (h) |a b| 3

|b c| < 2 (l)

|b c| > 3

(m) |c a| > 1

2. Cart
oes premiados - Uma loja distribui 9999 cartoes entre os seus clientes.
Cada um dos cartoes possui um n
umero de 4 algarismos, entre 0001 e 9999.
Se a soma dos primeiros 2 algarismos for igual a` soma dos 2 u
ltimos, o cartao
e premiado. Por exemplo, o cartao 0743 e premiado. Prove que a soma dos
n
umeros de todos os cartoes premiados e divisvel por 101.

3. O pre
co da gasolina - Em 1972 encher o tanque de gasolina de um carro
pequeno custava R$29, 90, e em 1992, custava $149, 70 para encher o mesmo
tanque. Qual dos valores abaixo melhor aproxima o percentual de aumento
no preco da gasolina nesse perodo de 20 anos?
(a) 20%

(b) 125%

(d) 300%

(d) 400%

(e) 500%

4. O tri
angulo de latas - Um menino tentou alinhar 480 latas em forma de
um triangulo com uma lata na 1a linha, 2 latas na 2a e assim por diante. No
m sobraram 15 latas. Quantas linhas tem esse triangulo?
OBMEP 2007

129

Nvel 3

Lista 5

5. Circunfer
encia e tri
angulo ret
angulo - Inscreve-se uma circunferencia
num triangulo retangulo. O ponto de tangencia divide a hipotenusa em dois
segmentos de comprimentos 6cm e 7cm. Calcule a area do triangulo.

1
1
1
+ 2 + + n , qual e o menor n
umero
2
2
2
inteiro positivo n tal que Sn > 0, 99?

6. Soma de raz
ao

1
2

- Se Sn =

7. Soma de razes quadradas


r2 5
.
3, mostre que 6 =
2

(b) Se s = 215 + 300, mostre que s2 > 1015.


(a) Se r =

2+

8. Duas rodas - A roda A gira com 1200 voltas por minuto, e a roda B com
1500 voltas por minuto. Calcule os raios das duas rodas.
..
...................
........... ........................
............
..
........
................
........
...... ...................
.
.
.
.
.......
..... ..........
.. ...
.....
................. .....................................
.... .....
...... ...
........
....
... ......
.
.... ...
......
...
.
.. ...
.
.
.
... ...
... ...
.
... ..
... ..
... ..
... ...
... ...
.... ....
... ...
.....
... ...
.. ...
....
.. ....
..
......
.......
..
.
....
........
.
........
. ...
.......
... ...
.
.
.. ..
.... ...
... .....
... ..
.....
.... ......
....
.... .....
.......
...
......
...
.... ......
..........
....
.........
.
.
.
.......
.
.
.
.
.
.
.
.
.
.
.
.
.
.
.
.
.
.
.
.
.
.
.
.
.
.
.
.
.
.
.
....
....
.
.........
...
................ ........................
....
....
............
...
...
....
....
....
....

130

9 cm

OBMEP 2007

Solucoes da Lista 5

Nvel 3

Solu
co
es da Lista 5
1. Dist
ancia entre n
umeros a
4

d
0

Como os n
umeros a, b e c sao negativos e c e positivo, temos que
|a| = a , |b| = b , |c| = c , |d| = d
Assim, |a|, |b| e |c| sao simetricos de a , b e c em relacao ao zero. No seguinte
graco se mostram os pontos |a|, |b|, |c| e |d|.
|c|
4

|b||d|
1

ss

|a|
2

Note que nao podemos armar qual entre os dois,|b| e |d|, e o maior, as unicas
comparacoes que podemos fazer sao:

0 < |c| < 1 < |b| < 2 < |a| < 4 e 0 < |c| < 1 < |d| < 2 < |a| < 4
Portanto, (a), (b), (c), (d) e (e) sao verdadeiros e (f) e falso.
Lembre que |x y| = distancia de x a y.
Como a e b estao entre 4 e 1, a distancia entre eles e menor do que 3, isto
e: |a b| < 3, logo (g) e verdadeira e (h) e falso. Analogamente, temos:
1 < |c d| < 3 = (i) e falso
0 < |b c| < 2 = (j) e verdadeiro e (l) e falso
2 < |a c| = (m) e verdadeiro.

OBMEP 2007

131

Nvel 3

Solucoes da Lista 5

2. Cart
oes premiados - Observe que se o cartao abcd e premiado entao o
cartao cdab tambem e premiado, por exemplo: 2341 e 4123 sao ambos premiados. Assim sempre que ab = cd temos dois cartoes premiados cuja soma e
abcd + cdab = (ab 100 + cd) + (cd 100 + ab) = 101(ab + cd),
assim a soma desse dois cartoes e divisvel por 101.
No caso que o cartao ser da forma
abab = ab 100 + ab = 101 ab
o n
umero do cartao e divisvel por 101. Assim a soma de todos os cartoes e
divisvel por 101 ja que a soma pode ser feita agrupando cartoes do tipo abcd
com cdab.

3. O pre
co da gasolina - O aumento do valor foi
149, 70 29, 90 = 119, 80 reais ,
que corresponde a:
119, 80
100% = 400, 66%.
29, 90
A opcao correta e (d).

4. O tri
angulo de latas - Suponhamos que o triangulo esta composto por n
linhas, logo foram usadas 1 + 2 + 3 + + n latas, assim
480 15 = 1 + 2 + + n =

n(n + 1)
= n2 + n 930 = 0.
2

Resolvendo a equacao n2 + n 930 = 0, obtemos:

1 61
1 1 + 4 930
=
.
n=
2
2
132

OBMEP 2007

Solucoes da Lista 5

Nvel 3

Assim, n = 30 que e u
nica solucao positiva desta equacao. Logo o triangulo
tem 30 linhas.

5. Circunfer
encia e tri
angulo ret
angulo - Seja r o raio da circunferencia
inscrita. Usando o teorema de Pitagoras temos que
(6 + 7)2 = (r + 6)2 + (r + 7)2 = r2 + 12r + 36 + r2 + 14r + 49 = 2(r2 + 13r) + 85,
assim temos que r2 + 13r =

16985
2

= 42.
Q
Q6
Q........
..........
......................Q
.......
........
....
......
Q
....
...
.
.
...
..
Q
.
...
....
.. Q
..
...
.
Q7
..
..
...
..
Q
...
..
.
.
...
.
.
Q
.
r ...................... .....................
Q
..............................
Q

6
Por outro lado, a area do triangulo e

(r + 6)(r + 7)
r2 + 13r + 42
42 + 42
=
=
= 42.
2
2
2

6. Soma de raz
ao

1
2

- Como

Sn =

1
1 1 1
+ + + + n
2 4 8
2

segue que
1
1
Sn =
2
2

1 1 1
1
+ + + + n
2 4 8
2


=

1 1
1
+ + + n+1 .
4 8
2

Logo,
1
1
1
1
Sn = Sn Sn = n+1 .
2
2
2 2
Assim
Sn = 1

1
.
2n

Como queremos Sn > 0, 99, isto e equivalente a encontrar o menor n tal que
1

1
> 0, 99
2n

OBMEP 2007

133

Nvel 3

Solucoes da Lista 5

e assim
2n > 100 .
Logo, devemos ter n 7 porque 128 = 27 > 100 > 26 = 64.
Observacao: Outro modo de calcular Sn , e notar que e a soma de uma progressao geometrica com a1 = 1/2 e razao q = 1/2. Aplicando a formula,
temos:

1
2

21n 12
1
Sn =
= 1 n.
1
2
1 2

7. Soma de razes quadradas (a) Como

r2 = ( 2 + 3)2 = ( 2)2 + 2( 2)( 3) + ( 3)2 = 2 + 2 6 + 3 = 5 + 2 6,

r2 5
.
portanto r2 5 = 2 6 = 6 =
2
(b) Pelo mesmo argumento temos que

s2 = ( 215 + 300)2 = 215 + 2 215 300 + 300

= 515 + 10 43 60 = 515 + 10 2580 >

> 515 + 10 2500 = 515 + 500 = 1015

8. Duas rodas - Dos dados do problema podemos


dizer que quando a roda A da 12 voltas a roda B
da 15 voltas, ou equivalentemente, quando a roda
A da 4 voltas a roda B da 5 voltas. Denotemos
por R o raio da roda A e por r o raio da roda B.

........
...............
..........
........ ...............................................
..................
........
...... ................
.
.
.
......
.............................................
. ..
............
........ ......
....
.... .....
........
...... ...
....
.
.
... ......
.
.
.
.
.... ...
... .....
... ...
... ...
... ...
... ..
.... ....
... ..
... ...
......
... ...
.. ...
....
... ...
.. ..
....
. .....
..
......
..
........
.
.......
......
.
........
.
....
.. ...
.
.
.
... ...
... ..
.... ....
... ......
... ...
......
.... .....
....
...... .....
........
.....
...
............... ......................
...
....
.......
......
.
.
.
.
.
.
.
.
.
.
.
.
.
.
.
.........
....
.
....
................ .......................
...
...
............
....
....
....
....
..
..

9 cm
O comprimento da roda A e 2R e o da roda B e 2r. Logo, o comprimento

de 4 voltas da roda A e 4 (2R) e o comprimento de 5 voltas da roda B e


134

OBMEP 2007

Solucoes da Lista 5

Nvel 3

5 (2r). Como esses dois comprimentos sao iguais entao temos que 4R = 5r.
Por outro lado, da gura temos que 2(r + R) = 9, assim
  

5
5
9
2r + 2
r = 2+
r = r = 9,
4
2
2
portanto r = 2 e R = 52 .

OBMEP 2007

135

Nvel 3

Lista 6

Lista 6
1. Dois divisores - O n
umero 248 1 e divisvel por dois n
umeros compreendidos entre 60 e 70. Quais sao esses n
umeros?
(a) 61 e 63 (b) 61 e 65 (c) 63 e 65 (d) 63 e 67 (e) 67 e 69

2. Rede de esta
c
oes - Um servico de vigilancia vai ser instalado num parque
na forma de uma rede de estacoes. As estacoes devem ser conectadas por linhas
de telefone, de modo que qualquer uma das estacoes possa se comunicar com
todas as outras, seja por uma conexao direta seja atraves de no maximo uma
outra estacao. Cada estacao pode ser conectada diretamente por um cabo a
no maximo 3 outras estacoes.
O diagrama mostra um exemplo de uma rede desse
tipo conectando 7 estacoes. Qual e o maior n
umero
de estacoes que podem ser conectadas dessa maneira?

........................
...
...
.....
...
...
...
.
...........................
.
.
.
.
.
.
.
.
..
......
.
.
.
.
.
.
.
.
.
..
...
..
.........
..
.........
..
.........
.. ......................
............
............ ..........
. .
...... ........
...... ........
...
.
.
.
.
.............................
..
..................................
.....
.. ..
.
...
...
..
...
.. ...
...
.
.
.
.
.
...... .....
.. ........ .......
......................
.
.
.
.
.
.
.
.
.
.
.
.
.
.
.
.
.
.
.
.
.
.
.
.
.
.
.
.
.
.
.
...... .
.....
..... ........................ .... .....
...
.. ...
........
.. ...
..............
....
.. ..
............. ....
..
.. .............
.
.............
... .....
....................
..............................
.
.
.
.
.
.
.
.
...
.
.....
.
...
...
..
.....
...............................
...............................
.
.
.
...
.
.
.
.
...
...
.
...
.
...... .......
.
.
.
.
.
.
.
.
.
.
.................
................. .
............

3. Bolas brancas e pretas - Uma caixa tem exatamente 100 bolas pretas e 100
bolas brancas. Repetidamente, 3 bolas sao retiradas da caixa e substitudas
por outras bolas que estao em um saco da seguinte maneira:
BOLINHAS REMOVIDAS
3 pretas

SUBSTITUIDAS POR
= 1 preta

2 pretas e 1 branca

= 1 preta e 1 branca

1 preta e 2 brancas

= 2 brancas

3 brancas = 1 preta e 1 branca


Qual pode ser o conte
udo da caixa depois de seguidas aplicacoes desse procedimento?
(a) 2 pretas (b) 2 brancas (c) 1 preta (d) 1preta e 1 branca (e) 1 branca.
136

OBMEP 2007

Lista 6

Nvel 3

4. O cubo - Alice tem uma folha de cartolina de 60 cm por 25 cm. Ela quer
cortar a folha para montar um cubo. Qual o cubo de maior volume que ela
pode construir?

5. Um quadrado e um tri
angulo
X

- Na gura, ABCD e um quadrado cuja area e 7/32 da area do

........
.... ....
.... ......
....
...
.
.
...
...
...
...
....
...
....
.
.
...
.
..
...
.
.
.
...
...
...
.
.
...
..
.
.
.......................................................................................
.
.
.
...
......
.
.
..... ......
.
... ....
.
.... ....
.
.. ..
.... .....
.... .....
.
...
.
.
...
.
...
..
.
.
.
.
....
.
...
.
..
..
.
...
.
.
....
.
.
...
...
..
....
.
...
.
.
.
.
.
.
...
.
.
.
.
.
.
.
...
..
..
..
.
.
.
.
...
.
.
.
..
...
..
..
.
.
.
.
.
.
.
...
..
.
.
.
.
...
.
.
.
.
.
...
...
....
....
.
.
.
...
.
.
.
..
.
.
.
.
.
.
.
................................................................................................................................................................................................

triangulo XY Z. Qual e a razao


entre XA e XY ?.
Y

6. A urna - Uma urna tem 6 bolas numeradas de 1 a 6. Se duas bolas sao


extradas, qual e a probabilidade da diferenca entre os n
umeros dessas 2 bolas
ser 1?

7. Soma das razes de um equa


c
ao - Determine a soma das razes distintas
da equacao x2 + 3x + 2 = |x + 1|.

8. Produto de tr
es n
umeros - No diagrama abaixo cada crculo representa
um algarismo. Preencha o diagrama colocando em cada crculo um dos algarismos de 0 a 9, utilizando cada algarismo uma u
nica vez.
....................
...
...
....
.
..
....
...................

................... ......................
....
.

................... ...................... ......................


....
.
.

................... ...................... ...................... ......................


....
.
.
.

.................................................................... ...................................................................................................... =........................................................................................................................................

OBMEP 2007

137

Nvel 3

Solucoes da Lista 6

Solu
co
es da Lista 6
1. Dois divisores - Lembre que
a4 1 = (a 1)(a3 + a2 + a + 1) .
Logo, se a = 212 , temos:
248 1 = (212 )4 1 = (212 1)(236 + 224 + 212 + 1)
e 212 1 = (26 + 1)(26 1) = 65 63. A opcao correta e (c).

2. Rede de esta
c
oes - O exemplo mostra que podemos conectar pelo menos
7 estacoes dentro das condicoes propostas. Comecamos com uma estacao
particular, e vamos pensar nela como se fosse a base da rede. Ela pode ser
conectada a 1, 2 ou 3 estacoes conforme mostra o diagrama.
..............
.... ......
.
.....
.... ......
...............
...
...
...
..
...............
.... ......
.....
..
....
...
.............................
......
.......
.
.
.
.
.
......
......
......
.......
......
.......
.. ...........
.....................
.
.
...... .......
...
..
.....
.....
.... .....
.... .....
..............
..............

Base

Agora, as estacoes A, B e C tem ainda duas linhas nao utilizadas, logo podem
ser conectadas a duas outras estacoes como a seguir:
..............
..............
.... ......
.... ......
.
.
.....
...
......
.... ............
...
.
.
.
.
.............. ......
.. .. ....
......
...... ...........
...... .............. .........
......... ..........
...
....
.... ......
...............
..
....
...
.
..............
.... ......
.
.....
..
....
...
.............................
......
......
.
.
.
.
.
.
......
......
......
......
......
.......
..............
...........
.....................
....................
.
.
.
.
..... .......
..... .......
...
...
..
..
.
.
.....
.....
...
..
...
.... ............................................
.... ............................................
.
.
.................
..................
..............
..............
.
.
.....
.....
....
....
....
....
....
....
.
.
.
...............
................
.
.
.
.
.
.
.
.
...
...
.
.
...
..
.....
.....
.
....
.
.
..... .......
.................
...........

Base

138

OBMEP 2007

Solucoes da Lista 6

Nvel 3

Agora, e impossvel acrescentar mais estacoes porque qualquer outra a mais


nao poderia ser conectada a` base satisfazendo as condicoes do problema. Isso
mostra que nao podemos ter mais do que 10 estacoes. Vamos agora vericar se
podemos montar a rede com essas 10 estacoes. Observe no diagrama acima que
apenas a Base e conectada a todas as outras estacoes (atraves de um cabo ou
de uma conexao via uma estacao). As estacoes que estao nos extremos ainda
possuem duas linhas nao utilizadas, e agora vamos usa-las para fechara rede;
veja o diagrama a seguir.
......
...
............ .....
... ......................
............
...........
...
...
...........
...........
...
............
...........
...
...........
.
...........
.
...
.
.
.
.
.
.
.
.
....
.
..
...........
...
.............................
...
...
. ...........
.
.
.
.
.
.
.
.
.
.
...
.
...........
...
........
.
.
.
.
.
.
.
.
.
.
.
.
.
.
.
...
.
........... ...........
...
...................
.
.
.
.
.
.
.
.
.
.
.
...
.
.
...
...
.
...
..
.
.
.
.....
...
...
..
..
...
.
.
...
.
...
.
...
.
.
......................
................... ........
.
.
.
...
.
.
...
.
.
...
......
...
.
.
..
.
...
.
.
.
.
.
...
.
...... ............ .....
...
...
..
.......... ...........
.
...
...
...
...
..
...
.
...
.
.
...
.
...
...
..
.
...
.
.
...
....... .......
...
..
.
.
.
.
.
.
.
...
.
...
...
..
....
.
...
...
...
...
..
.
...
.
...
...
...
..
.
...
...
...
...............
.
..
.
.
...
...
.
.
...
...
..
....
...
.
...
.
.
...
...
...
..
...
.
...
.
...
....... .......
...
..
.
.
.
...
.
.
.
.
...
..... ..... ..........
.
...
.
..
.
.
...
.
.
...
......
....
.
...
.
..
.
.
.
.
.
.
.
...
.
......
...
...
.....
.
..
.
.
.
.
.
.
.
...
.
...... .....
........
...
.....
...
.
.
.
.
.
.
.
.
.
.
.
.
.
.
.
.
.
.
.
.
.
.
.
.
.
.
.
.
.
.
.
.
.
.
.
.
.
.
.
.
.
.
.
.
.
.
.
...
.
.
.
.
....
...
...
...
..
.
.
...
.
.
.
.
...
.
..
.
.
...
.
....
.
.
..
.
.....................................
...
...
...
.......................................
..
.
.
...
.
.
.
.
....
...
.... ...
.
.
....... ......
.
.
.
.
.
.
.
.
.
.
.
.
.
.
.
.
.
.
.
...
...
.
.
.
.
.
..........
..........
......... ..
........
...
...
...
....
....
....
...
...
...
...
...
...
...
.
....
....
........
...
...
.......
.
...............
.
.
.
.
.
.
.
.
.
.
.
.
.
.
.
.
.
.
.................
................
.
.
.
.
.
.
.
.
.
.
.
.
.
.
.
.
.
........
.....
.....
.......... ....
.
.
.
.
.
.
.
.
.
.
.
.
.
.
.
.
.
.
.
.
.
.
.
.
.
.
.... .............................
.
.
.
.
.
.
...
.... ...... ..........................
............... ...... ......
..
...
.
..........
...
..............
...
...
....................................................................
...
...... .......
....... ........
...
...
........
......
...
....
...
...
..
.
.
...........................................................................................................................................................................................................................................................................

Base

3. Bolas brancas e pretas - Inicialmente observe que depois de cada substituicao o n


umero de bolas brancas ou permance o mesmo ou decresce de 2.
Logo o n
umero de bolas brancas permanece par. Por outro lado, cada grupo de
bolas removidas que contem pelo menos 1 bola branca e substitudo por outro
que tambem contem 1 bola branca, o n
umero de bolas brancas nunca e zero.
Agora observe que a opcao (b) e a u
nica incluindo pelo menos 2 bolas brancas,
logo ela e a opcao correta. Um modo de obter esse resultado e remover 3 bolas
brancas 49 vezes ate obter 149 pretas e 2 brancas, e depois, remover 1 preta e
2 brancas 149 vezes.
OBMEP 2007

139

Nvel 3

Solucoes da Lista 6

4. O cubo - Seja a a aresta do cubo que queremos construir. Como a area


lateral do cubo e 6a2 , devemos ter 6a2 25 60, isto e a2 250 e assim
a < 16. Com a = 15 temos 4 = 60 15 quadrados de lado 15 cm e sobra um
retangulo de 60 cm por 10 cm.
Podemos cortar um retangulo de 60 cm por 2, 5 cm e os pedacos marcados
com  de dimensoes 15cm por 7, 5cm. Assim na gura a linha pontilhada
indica dobradura e a linha continua indica corte e com os pedacos de cartolina
marcados com  formamos a tampa.


5. Um quadrado e um tri
angulo -

7, 5

Sejam l o comprimento do lado do

quadrado, h a altura do triangulo XAB, H a altura do triangulo XY Z e


b o comprimento do lado Y Z.
A area do quadrado e l2 e a area do
tri angulo XY Z e

bH
.
2

Como os

triangulos XY Z e ABC sao semelhan-

....
.... ....
.... ......
...
....
.
.
...
...
...
...
....
...
....
.
...
.
.
..
...
.
.
.
...
...
...
.
.
...
.
.
.
...
.
..
.
.
.
.
.............................................................................................
.
.
.
..
.. ....
.
.
..... ......
.
... ....
.
.... .....
.
.
..
.... .....
.....
....
.
...
.
.
.
....
.
...
..
.
.
.
.
.
.
...
...
..
..
.
...
.
.
....
.
.
...
...
..
....
.
...
.
.
.
.
.
.
...
.
.
.
.
.
.
.
...
..
..
..
.
.
.
.
...
.
.
.
..
...
..
..
.
.
.
.
.
.
.
...
..
...
...
...
.
.
.
.
.
...
.
.
..
.
.
.
.
.
.
.
.....................................................................................................................................................................................................

tes, temos
b
H
XY
=
=
.
l
h
XA
Portanto b =

Hl
h

(h+l)l
.
h

(h + l)2 l
bH
=
e a razao
Assim a area do triangulo XY Z e:
2
2h
h
h
1
XA
=
=
=
.
XY
H
h+l
1 + hl
140

OBMEP 2007

XA
XY

Solucoes da Lista 6

Nvel 3

Logo, basta calcular hl .


32
,
7

Como a razao entre as area do triangulo XY Z e a area do quadrado e


entao

(h+l)2 l
2h
l2

32
64
50
= (h + l)2 = hl = l2 hl + h2 = 0.
7
7
7

Dividindo por h2 obtemos a equacao quadratica ( hl )2 50


( l ) + 1 = 0, que tem
7 h
como solucoes
l
=
h
Assim
1
,
8

l
h

50
7

( 50
)2 4
7
2

tem dois possveis valores

=
1
7

25

25 24
252 72
=
.
7
7

e 7, e em cada um destes casos

XA
XY

7
8

respectivamente.

6. A urna - Observemos que se extramos a primeira bola com um n


umero
entre 2 e 5, entao dentre as 5 bolas que cam na urna temos duas possveis
bolas que cumprem a condicao do problema, logo neste caso a probabilidade
que a segunda bola cumpra a condicao e

2
5

e a probabilidade que a primeira

bola tenha um n
umero entre 2 e 5 e 46 . Por outro lado, se a primeira bola
extrada e 1 ou 6, so temos uma bola na urna que cumpre a condicao, logo
neste caso a probabilidade para a escolha da segunda bola e

1
5

e a probabilidade

da primeira bola ser 1 ou 6 e 26 . Portanto, a probabilidade das bolas serem


consecutivas e
4 2 2 1
1
+ = .
6 5 6 5
3

7. Soma das razes de um equa


c
ao - Temos que considerar dois casos.
Caso 1: x 1.
Nesse caso, x2 + 3x + 2 = x + 1, e logo x2 + 2x + 1 = 0 que so possui a solucao
x = 1.
OBMEP 2007

141

Nvel 3

Solucoes da Lista 6

Caso 2: x < 1.
Nesse caso, x2 + 3x + 2 = x 1, logo x2 + 4x + 3 = 0 que tem, no intervalo,
apenas a solucao x = 3.
Assim as u
nicas solucoes distintas da equacao sao 1 e 3, cuja soma e 4.

8. Produto de tr
es n
umeros - Sejam a, b, c, .... os n
umeros em cada crculo
como indicado abaixo.
...................
....
...
.....
...
...
.....................

................... ......................
....
.

................... ...................... ......................


....
.
.

................... ...................... ...................... ......................


....
.
.
.

.
.
. .
.
... ...
... ...
...
... ...
... ...
... ...
...
...
.. ...
.. ...
.. =.... g .... .... h .... .... i .... .... j ....
a ...............b..................................c................... ...............d
..... .... ..... .... ..... .... ..... ....
... ... e ... ... f ...
.............
.............
.............
.............
............ ................... ...................

Temos que a, c e f nao podem ser zero, pois 0 x = 0.


Mas, o produto dos tres n
umeros e um n
umero de 4 algarismos, assim, abd < 10
e portanto os n
umeros que aparecem em dito produto sao 1, 2, 3 ou 1, 2, 4.
Observemos que a segunda e impossvel porque o mnimo produto que podemos
obter neste caso e
1 23 456 = 10488 ,
assim abd = 6 e o produto e maior do que 6000. Por outra parte a nao pode
ser 2 ou 3 porque nesse caso o mnimo valor que tem o produto e
2 14 356 = 9968
e os outro produtos cam maiores do que 10000. Portanto a = 1.
Continuando essa analise, obtemos a solucao:
.....................
...
..
....
.
....
..
...................

..............
..............
.... .... .... ....

..............
..............
..............
.... .... .... .... .... ....

..............
..............
..............
..............
.... .... .... .... .... .... .... ....

. .
. .
.
. .
.
. .
. .
. .
.
.
.
.
1 ..............2................................6.................. ..............3................................4................................5.................. =..............8................................9................................7................................0..................

142

OBMEP 2007

Lista 7

Nvel 3

Lista 7

1. Area
do tri
angulo - Determine a area do

A..... = (1, 2)

... ..........
... ..........
.........
..
.........
....
.........
...
.........
.........
....
.........
.........
...
...
......
......
....
.......
.
.
.
.
.
....
.
....
.
.
.
.
.
....
.
.....
....
.......
.......
....
......
....
.......
.... .............
... .........
......

triangulo ABC mostrado na gura.

B = (8, 0)
x

C = (1, 6)

2. Duas tabelas - As duas tabelas abaixo foram formadas de acordo com uma
mesma regra, mas na segunda indicamos apenas tres n
umeros. Qual o n
umero
que deve ser colocado na casa com ?
5

11

14

17

12

15

18

21

24

19

22

25

28

31

26

29

32

35

38

33

36

39

42

45

39

87
56


3. A seq
u
encia abc - A lei de formacao da seq
uencia 10, a, 30, b, c, . . . e: cada
termo, comecando com o 30, e o dobro da soma dos dois termos imediatamente
anteriores. Qual o valor de c?

4. Permetro e diagonal - O permetro de um retangulo ABCD a 20 m. O


menor comprimento, em metros, que a diagonal AC pode ter e:

(a) 0
(b)
50
(c) 10
(d)
200
(e) 20 5

5. As idades numa classe - Numa classe na escola, todos os alunos tem a


mesma idade, exceto sete que tem 1 ano a menos e dois que tem 2 anos a mais.
OBMEP 2007

143

Nvel 3

Lista 7

A soma das idades de todos os alunos dessa classe e 330. Quantos alunos tem
essa classe?

6. A mesa redonda - Uma mesa


redonda tem 1, 40 m de diametro.
Para uma festa, a mesa e aumentada colocando-se tres tabuas de
40 cm de largura cada uma, como
mostra a gura. Se cada pessoa

................................................
...........
.........
........
.......
.......
......
.....
.
....
.
.
....
...
.
.
....
...
.
...
..
.
...
..
...
.
...
....
..
...
..
...
..
..
..
..
...
....
.
...
.
...
...
...
..
...
...
....
....
.
.
....
.
..
.....
.....
.......
.......
........
........
...........
.....................................................

mesa fechada

`a mesa deve dispor de um espaco

.......
.................... ..............................
..........
........
........
......
......
.....
.
.
.
.
....
...
....
....
.
.
...
.
...
...
.
...
..
.
...
....
...
...
..
...................................................................................................................................
...
...
...
...
....
....
...
.
...................................................................................................................................
...
...
...
...
....
....
.....................................................................................................................................
...
...
....
....
...
.
...................................................................................................................................
..
.
...
..
.
.
...
.
.
...
...
...
..
...
...
....
....
.
....
.
.
.
.....
.....
......
......
.......
.......
..........
..........
................
...................................

mesa aumentada

de 60 cm, quantos convidados poderao se sentar na mesa?

7. Brincadeira com 7 n
umeros - Sete n
umeros inteiros positivos estao escritos em ordem crescente numa mesma linha. Coloque entre esses n
umeros
cinco sinais de + e um so de = para obter uma igualdade.

8. Um terreno compartilhado - Tres amigas compraram um terreno quadrado e querem reparti-lo como indicado na gura, por que em A se encontra uma
fonte de agua. Elas querem tambem que as areas das tres partes sejam iguais.
Onde devem estar os pontos M (sobre BC) e N (sobre CD)?
B ..............................................................................................................................A

M
C

144

.
. .
....... ... ..
....
....... ... ..
...
....... ... ..
....... ..... .....
....
.......
.
.
.
.
.
.
.
....
.
.....
.
..
......
....
....
...
.......
...
.......
....
....
...
.......
.
.
.
.
.
.
.
....
.
.
...
...
..
.
....
... ..............
.
...
.
.... ..........
....
.
..
.
....
........
.
..
.
.
....
....
...
.
.
...
...
..
.
....
....
.
...
.
....
....
.
..
.
..
...
.
..........................................................................................................

OBMEP 2007

Solucoes da Lista 7

Nvel 3

Solu
co
es da Lista 7

1. Area
do tri
angulo - Para determinar a a rea

A..... = (1, 2)

basta conhecer o comprimento de uma base e sua

respectiva altura. Se AC e uma base, entao a altura corta AC no ponto H = (1, 0). Assim, a base
78
2

B = (8, 0)
x

C = (1, 6)

AC = 8 e a altura BH relativa a essa base e 7.


Logo, a area do triangulo e

... ..........
... ..........
.........
....
.........
...
.........
.........
...
.........
...
.........
...
.........
....
......
.......
....
.......
.
.
.
.
....
.
...
.
.
.
.
.
.
....
.....
....
.......
.......
...
.......
....
......
.... ............
.... ..........
.......

= 28.

2. Duas tabelas 5

11

14

17

12

15

18

21

24

19

22

25

28

31

26

29

32

35

38

33

36

39

42

45

39

87
56


Observemos que na primeira tabela cada linha e uma progressao aritmetica de


razao 3 e cada coluna e uma progressao aritmetica de razao 7. Suponhamos
que na segunda tabela cada linha e uma progressao aritmetica de razao a e
cada coluna e uma progressao aritmetica de razao b. Assim temos que:
392a

39a

39

39+a

39+2a

392a + b

39+2a + b

392a + 2b

87

56


39 + 2a + 2b = 87
Logo:
Somando essas duas equacoes obtemos 78 +
39 2a + 3b = 56.
= 11. Portanto, o n
umero na posicao da
5b = 143, donde b = 13 e a = 482b
2
 e: 39 + a + 4b = 39 + 11 + 4 13 = 102.
OBMEP 2007

145

Nvel 3

Solucoes da Lista 7

3. A seq
u
encia abc - Sabemos que 30 = 2(10 + a), logo a = 5. Assim
b = 2(30 + a) = 2(30 + 5) = 70
e
c = 2(b + 30) = 2(70 + 30) = 200 .

4. Permetro e diagonal - Denotemos por a e b os comprimentos dos lados


do retangulo, assim 2a + 2b = 20, logo a + b = 10. Por outro lado quadrado do
comprimento da diagonal pode ser calculado usando o teorema de Pitagoras,
assim d2 = a2 + b2 . Como
2d2 = 2a2 + 2b2 = (a2 + 2ab + b2 ) + (a2 2ab + b2 )
= (a + b)2 + (a b)2
= 100 + (a b)2
temos que o comprimento da diagonal e mnimo quando a = b, e neste caso

2d2 = 100 e d = 50. A opcao correta e (b).

5. As idades numa classe - Denotemos por a a idade comum dos alunos e


n o n
umero de alunos, assim temos 7 alunos com a 1 anos, 2 com a + 2 anos
e o resto, isto e, n 9 com a anos. Assim a soma das idades e
7(a 1) + 2(a + 2) + (n 9)a = na 3 = 330,
logo
na = 333 = 9 37 .
Como a classe tem mais do que 9 alunos, entao a = 9 e n = 37, portanto a
classe tem 37 alunos.

146

OBMEP 2007

Solucoes da Lista 7

Nvel 3

6. A mesa redonda -

........................................................
..........
........
........
......
......
.....
....
.....
.
.
.
....
...
.
...
.
...
...
.
...
..
.
...
....
...
...
..
..
...
.
..
..
..
...
..
...
....
.
...
.
...
...
...
...
....
....
....
...
.
.
.....
.
....
......
......
.......
.......
..........
..........
................
...................................

mesa fechada

..............................................
............
.........
.........
.......
.......
......
......
....
.
.
.
....
...
.
.
....
...
.
...
..
...
.
..
...
.
...
....
..
...
.
.....................................................................................................................................
...
...
.....
.....
...
..
....................................................................................................................................
...
...
....
....
...
.
..................................................................................................................................
....
...
...
...
....
...
...................................................................................................................................
.
..
...
...
...
...
...
..
..
...
..
...
....
...
.
.
.
....
....
.....
....
......
......
.......
.......
..........
.........
.
.
................
.
.
.
.
.
.
.
.
.
.
.........................

mesa aumentada

O permetro de mesa aumentada e


140 + 40 6  140 3, 14 + 240 = 679, 60 cm.
Se cada convidado precisa de 60 cm para colocar-se ao redor da mesa e
679, 60
 11, 3.
60
Entao, podem se acomodar 11 convidados.

7. Brincadeira com 7 n
umeros Solu
c
ao 1 - Os 7 n
umeros podem ser escritos como
n 3,n 2,n 1,n,n + 1,n + 2,n + 3 .






3n6

3n+6

Observando que 3n 6 + 12 = 3n + 6, conclumos que n = 12. Logo, os


n
umeros sao
9 + 10 + 11 + 12 = 13 + 14 + 15
Solu
c
ao 2 - Seja n + 1, n + 2, . . . , n + 7 os sete n
umeros consecutivos e
suponhamos que
(n + 1) + + (n + k) = (n + k + 1) + + (n + 7).
OBMEP 2007

147

Nvel 3

Solucoes da Lista 7

Como os n
umeros a` esquerda sao menores, entao tem mais somandos a` esquerda, assim k 4. Supondo k = 4, a igualdade anterior e
4n + 1 + 2 + 3 + 4 = 3n + 5 + 6 + 7,
logo n = 8. No caso k = 5 temos que
5n + 1 + 2 + 3 + 4 + 5 = 2n + 6 + 7,
que nao gera solucao inteira. De igual forma k = 6 nao gera solucao inteira
positiva. Portanto a u
nica solucao e
9 + 10 + 11 + 12 = 13 + 14 + 15.

8. Um terreno compartilhado - Como as areas

B ..............................................................................................................................A

de ABM e ADN sao iguais e AB = AD temos


M

entao
BM AB
N D AD
=
= BM = DN .
2
2

....... ... ..
.....
....... ... ...
....
....... ... ..
...
....... ..... .....
.......
.
.
.
.
.
.
.
....
.....
.
.
.
......
....
...
....
.......
...
.......
....
...
...
.......
.
.
.
.
.
.
.
....
...
.
.
..
.....
.
.
.
.
.
.
....
.
....
.
.
...
..
.
.... .............
....
.
..
.
.
....
.........
...
.
.
...
...
..
.
....
....
.
...
.
....
....
.
..
.
....
.
....
...
.
...
....
.
.
..........................................................................................................

Assim, a gura AM CN e simetrica com respeito a` diagonal AC. Portanto, a


area do ACN e a metade da area do ADN . Agora, como esses triangulos
tem a mesma altura entao DN = 2N C e pela simetria temos que BM = 2M C.
Conclumos que BM e 2/3 do lado do quadrado, o mesmo ocorrendo com DN .

148

OBMEP 2007

Lista 8

Nvel 3

Lista 8
1. As duas partculas - Duas partculas, A e B, percorrem uma circunferencia
de 120 m de comprimento. A partcula A gasta 3 segundos menos que B, por
estar animada com uma velocidade maior de 2 metros por segundo. Qual e a
velocidade de cada partcula?

2. Queda livre - Um corpo em queda livre demora 11 segundos para tocar o


solo. No primeiro segundo ele percorre 4, 5 m e, em cada segundo que segue,
a distancia percorrida aumenta de 9, 8 m. Qual a altura da queda e quantos
metros ele percorreu no u
ltimo segundo?

3. Um caminho retangular - Janete passeia por um caminho de forma retangular ABCD com largura AB = 1992 m. Ela gasta 24 minutos para percorrer
a largura AB. Depois, com a mesma velocidade, ela percorre o comprimento
BC e a diagonal CA em 2 horas e 46 minutos. Qual e o comprimento BC?

4. O pre
co do feij
ao - A tabela e o graco, dados
a seguir, mostram a evolucao do preco medio de

R$

tres tipos de feijao, A, B e C, na bolsa de alimentos

110

durante os primeiros quatro meses de certo ano:

100

Desses 3 tipos, os que apresentam, respectiva-

90

mente, o maior e o menor crescimento percentual

80

no preco nesse perodo sao:

70

(a) A e B

(b) A e C

60

(c) B e C

(d) C e A

c
s

s
c

s

jan

c: A

(e) C e B
OBMEP 2007

c
s

...
.....
.....
.....
..... ...
.
.
.
.
.....
......
..... ......
..... ........... ...
... ........ ......
.
..........
.
.
.
.
.......
....
..... ......
.....
..
... .....
.
.
.
.
.
.
.
.......
...
.....
...
.....
...... ...
....... ...
.
...
...
.......
........
...
.........
...
....... ..
..
....... ....
.
.
..
......
... ..........
... .........
................

fev

mar abr

s: B

: C
149

Nvel 3

Lista 8

jan

fev

mar

abr

65, 67

83, 33

96, 67

103, 33

73, 30

80, 50

99, 55

109, 50

64, 50

71, 57

89, 55

100, 00

5. Intersec
c
ao de tri
angulos - Os 3 triangulos da gura se cortam em 12
pontos diferentes. Qual e o n
umero maximo de pontos de interseccao de 3
triangulos?
......
.... ...
.... ....
...
....
.
.
.
.
...
....
...
....
...
...
.
.
...
.
.
.
.
...
.
.
.
.
.
.
.
.
.
.
.
.
.
.
.
.
............... ........
.
.
.
.
.
.
... ....................................................
.
.
.
...
.
.
.
.
.
.
.
.
.
.
.
.
.
.
.....
...........................................
..
...
..
.
.
.
.
.
.
.
.
.
.
.
.
.
.
.
.
.
.
.
.
.
.
.
.
.
.............................
.....
..............
...
.
.
.
.
..
.
.
.
.
.
.
.
.
.
.
.
.
.
.
.
.
.
.
.
.
.
.
.
.
.
.
.
.
.
.
.
.
.
.
.
.
.
.
.............................
............................
..
.
.
.
.
.
.
.
.
.
.
.
.
.
.
.
.
.
.
.
.
.
...............................
.
.
.
.
.
.....
......
..
.
.
.
..
................................... .....
.
.
.
.....
...................
..
...................
.
.
.
..........................
.
.
.
.
.
.
.
.
.
...
...
........................
...
... .................................
................................. ....................................... ..........
.................
...
.
.
.
.
.
.
.
.
.
.
.
..
.................
.......... .................................... ....................................................................... ....
...
.........................
...
................. ... .............................
.....
...
......................
.................
.....
...
.................
.......................
.....
..............................................
.................
.
.....
.
.................
....
...
.................
................. ..........
.....................

s
s
s

6. Comparar tri
angulos - Na gura, estao
indicados os comprimentos dos segmentos.
 ao
Demonstre que AC divide o angulo DAB

.........
.... ...........................................
...................C
....
.. ..
....
.......... .............
....
.
.
.......
..........
.
12 ....
.......
.........
.
.
.
.
.
.
.
.
.
.......
..
......
.
.
.
.
.
.
.......
.
.
.
.
.
.....
..
.
....... 12
.
.
.
.
.
.
.
.
.
.
.
.......
..
....... 18
.
.
.
.
.
.
.
.......
.
.
.
.
..
......
.
.......
.
.
.
.
.
.
.
.
.
.
.......
... ................
.
.......
.
.......
..................
.
.
.
........................................................................................................................................................................................................................

meio.

27

7. Queima de velas - Dois tipos de vela tem o mesmo comprimento mas sao
feitas de material diferente; uma queima completamente em 3 horas e a outra
em 4 horas, ambas queimam com velocidade uniforme. A que horas as velas
devem ser acesas de modo que `as 16 horas o comprimento de uma seja o dobro
do da outra?
(a) 1 : 24 (b) 1 : 28 (c) 1 : 36 (d) 1 : 40 (e) 1 : 48

150

OBMEP 2007

Lista 8

Nvel 3

8. Uma distra
c
ao - Em vez de multiplicar certo n
umero por 6, Julia se distraiu
e dividiu o n
umero por 6. O erro cometido por Julia foi de aproximadamente
(a) 100%

(b) 97%

(c) 83%

(d) 17%

OBMEP 2007

(e) 3%

151

Nvel 3

Solucoes da Lista 8

Solu
co
es da Lista 8
1. As duas partculas - Seja v a velocidade da partcula B e v +2 a velocidade
de A. Assim, o tempo que demora B em dar uma volta e
demora A e

120
v+2

120
v

e o tempo que

que e tres segundos a menos do que B, portanto,


120
120
3=
= v 2 + 2v 80 = 0.
v
v+2

A raiz positiva dessa equacao e

2 + 4 + 320
= 1 + 81 = 8.
v=
2
Portanto, a velocidade de B e 8 m/s e a velocidade de A e 10 m/s.

2. Queda livre - Como a distancia percorrida aumenta em 9, 8 m com respeito


ao segundo anterior, no n + 1-esimo segundo ele percorre 4, 5 + 9, 8n metros.
Assim no 11o segundo o corpo percorre 4, 5 + 9, 8 10 = 102, 5 metros.
A distancia total percorrida pelo corpo ate o 11o segundo e
4, 5 + (4, 5 + 9, 8) + (4, 5 + 9, 8 2) + + (4, 5 + 9, 8 10) =
= 4, 5 11 + 9, 8(1 + 2 + + 10) = 49, 5 + 9, 8 55 = 588, 5 m.

3. Um caminho retangular - Se v representa a velocidade com que Janete


caminha, entao v =

1992
24

= 83 m/min.

Janete percorre BC + AC com a mesma velocidade v = 83 m/min e gasta


2 h e 46 min = 166 min, entao BC + AC = 83 166 = 13 778.
Pelo teorema de Pitagoras temos que a diagonal do quadrado satisfaz:
(AC)2 = (1922)2 + (BC)2 .
152

OBMEP 2007

Solucoes da Lista 8

Nvel 3

Da temos:
(AC)2 (BC)2 = (AC BC)(AC + BC) = (1992)2 .
2

(1992)
Substituindo o valor da soma BC + AC temos: AC BC = 83166
= 288.

AC + BC = 13 778
Logo:
= 2BC = 13 778 288 = 13 490.
AC BC = 288
13 490
2

Portanto BC =

= 6 745.

4. O pre
co do feij
ao - Se b e o preco nal e a o preco inicial, temos
que a variacao e b a, e o aumento percentual sera
ba
.
a
Assim os aumentos foram:
R$

110
100
90
80
70
60

b
r

....
.....
.....
.....
..... ............
.
.
.
.
.. ...
.............. .....
........ ....
...... ....
...... ......
.
.
.. ...
...... ..
...... ..
....... ...
....... ...
.
....... ..
....... ....
.........
....... .... ....
. .......
.
.
... .........
...............

rb

rb

A :

103, 33 65, 67
37, 66
=
= 0, 57 = 57%;
65, 67
65, 67

B :

36, 20
109, 50 73, 30
=
= 0, 49 = 49%;
73, 30
73, 30

b

r

35, 50
100, 00 64, 50
=
= 0, 55 = 55%.
64, 50
64, 50
Portanto, o maior aumento foi de A e o menor foi de B.
jan fev mar abr

b : A r: B

: C

C :

Observe que os valores intermediarios (meses de fevereiro e marco) nao alteram


a variacao do preco de janeiro a abril. A opcao correta e (a).

5. Intersec
c
ao de tri
angulos - Observemos que cada reta pode cortar no
maximo dois lados de um triangulo, assim cada lado de um triangulo cortara no
maximo dois lados do outro triangulo e, portanto, o n
umero maximo de cortes
OBMEP 2007

153

Nvel 3

Solucoes da Lista 8

entre dois triangulos e 6. Assim, se temos 3 triangulos, o n


umero maximo
de cortes e dado pelo n
umero de formas de pegar dois de ditos triangulos e
multiplicar por 6. Assim, a resposta e 18, como mostra a gura seguinte:
.....
........ ...
........ .....
........
.
.
.
.
.
.
...
....
.......
.........................
....... ......................... ..... .....
...
........................
... ......
... .....
.......................................................................
.
.
.
.
. .
...............
.........................
... ...
.....
... ....
.......
........ .............
....
.......
.......
........
.......
.......
........
.
.
....
.
.
.
.
.
.
.
.
.
.
.......
.
... ...
... .. .......
.......
... ........
....... ...... .....
... ..........
......... ...
.......... ....
.............. ....
.......
.
.
.
.
.
.
.
.
.
.
.
.
...
. ...........
...... ...
...
...
...
............. .....
...
... ........
...
...
... .............
.................
.
.
.
.
.
.
...
.......
........
...
.
.
.
...
.
.
.
....
.
.
.
.
...............................................................................................
.............................................................................................................
........ ..
.
........ ..
...
...
.........
...
....
.
..........
.
.
.
... ...........
...
...
....
... .......... .....
.
...........
.
...
...
......
...
...
... .......
...
...
... ..............
........
... .....
...
.
.
........
... ....
........ .....
.....
........ ...
......

s
s

s s
s

s s s

6. Comparar tri
angulos - De acordo com
os dados do problema temos:

.
...............................................
..................... C
....
....
....
.......... ............
....
.
.......
.
..........
.
12 ....
.......
.........
.
.
.
.
.
.
.
.
.
.......
.....
..
.
.
.
.
.
.
.
.......
.
.
.
.
.
..
.......
.......12
.
.
.
.
.
.
.
.
.
.
.
.......
..... 18
..
.
.
.
.
.
.
.
.
.......
.
.
.
.
..
.......
.......
.
.
.
.
.
.
.
.
.
.
.
.......
... ................
.
.......
.
...
.................
.
.
......................................................................................................................................................................................................

AB
BC
AC
2
=
=
= .
AC
CD
AD
3

27

Segue que os triangulo ABC e ACD tem seus lados proporcionais, por = CAD.

tanto sao semelhantes. Em particular temos que BAC

7. Queima de velas - Seja l o comprimento das velas. Assim, uma queima a


velocidade constante

l
3

e a outra a velocidade 4l . Depois de um tempo t o que

sobra da primeira vela e


l
l t
3
e da segunda
l
l t.
4
Queremos saber quanto tempo transcorre ate o momento em que o comprimento de uma vela e o dobro do comprimento da outra, o que equivale a
resolver a equacao

154



l
l
l t=2 l t .
4
3
OBMEP 2007

Solucoes da Lista 8

Nvel 3

Segue que


2 1
12
2

t = 1 donde t =
horas = 2
horas = 2 horas e 24 minutos.
3 4
5
5
Portanto, depois de 2 horas e 24 minutos o comprimento de uma vela e o dobro
do comprimento da outra. Como queremos que isso aconteca a`s 16 : 00, entao
as velas devem ser acesas a`s 13 horas e 36 minutos. A opcao correta e (c).

8. Uma distra
c
ao Solu
c
ao 1: Seja x o n
umero. Julia tinha que obter 6x e com sua distracao,
obteve x6 . Logo, seu erro foi de 6x x6 =
o erro foi de

35 x
6

6x

35 x
.
6

Portanto, em termos percentuais

35
0, 9722 = 97, 22%.
36

A opcao correta e (b).


Solu
c
ao 2: Se N e o valor que a Julia tinha que obter, entao ela com seu erro
encontrou

N
,
36

assim o erro absoluto cometido foi de N

N
36

35
N.
36

Portanto,

o erro relativo foi de


35
100% = 97, 22%.
36

OBMEP 2007

155

Solucoes da Lista 8

Desaos

Desafios
1. O jogo das fichas - Para iniciar um jogo com seus amigos, Manoel coloca
8 chas em cada uma das nove casas do tabuleiro mostrado na gura. Para
ganhar o jogo, ele precisa mover as chas de modo que em cada linha, cada
coluna e cada diagonal haja o mesmo n u
mero de chas. Na 1a jogada ele
coloca 11 chas na casa 3 e nenhuma na casa 2. Agora, quantas chas ele deve
colocar em cada uma das outras casas para ganhar o jogo, mantendo as chas
da 1a jogada?
1

2
3

0
fichas

11
fichas




2. Nas igualdades abaixo, cada letra representa um algarismo:


AB + BC = CD

e AB BC = BA.

quanto vale A + B + C + D?

3. Rosa, Margarida e Dalia sao tres constelacoes em forma de buques de ores.


Sabemos que:
(a) O n
umero de estrelas de Dalia, que e a menor das tres, e o quadrado de
um quadrado;
(b) O n
umero de estrelas de Rosa e tambem o quadrado de um quadrado;
OBMEP 2007

157

Desaos

Solucoes da Lista 8

(c) Margarida tem 28 561 estrelas;


(d) Dalia e Rosa tem juntas o mesmo n
umero de estrelas do que Margarida.
Quantas estrelas possuem Dalia e Rosa cada uma?

4. Veja a seguir a pagina do calendario de abril de 2005:


D

10

11

12

13

14

15

16

17

18

19

20

21

22

23

24

25

26

27

28

29

30

Qual mes de 2005 ou de 2006 tera uma pagina igual?

5. A faixa e o quadrado - Uma faixa retangular de cartolina tem 5cm por 1cm.
Corte a faixa com 4 cortes retilneos de modo a poder montar um quadrado
com as pecas obtidas (n vale superposicao das pecas).

6. Um n
umero e o s
extuplo - Um n
umero de 3 algarismos e seu sextuplo
sao formados pelos mesmos algarismos. A soma dos algarismos desse n
umero
e 17 e a de seu sextuplo e 21. Qual e esse n
umero? Existe mais do que um?

7. Oito dentro de um ret


angulo - Coloque dentro dos crculos do retangulo
abaixo os n
umeros de 1 a 8 de modo que a diferenca entre dois n
umeros ligados
por um segmento seja sempre maior do que 1.
158

OBMEP 2007

Solucoes da Lista 8

Desaos




........................................................



.......................................................

......

. ......
......
......
...... ..
..
..
..... ........... 
...... 
...
......
......
......
......
......
......
....
...
....
....
......
......
.
.
...
....
...
....
....
...
.

...
....
...
....
....
...
.

.....................................................

.......
.....


....
....
.....
.......
.....
.....
.....
.......
.....
.....
..... 
....
.....
.....
.......

.....
.....
.....
....
.....
.....
........................................................



.......................................................





8. Uma estrat
egia com um n
umero muito grande - Carlos escreveu em
seguida todos os n
umeros de 1 a 60:
1234567891011121314 57585960.
Depois ele riscou 100 algarismos de modo que o n
umero formado com os algarismos que nao foram riscados fosse o maior possvel, sem mudar a ordem
inicial de como os algarismos foram escritos. Qual e esse n
umero?

9. Um n
umero surpreendente - Um n
umero surpreendente e um n
umero
divisvel por 9, de nove algarismos diferentes, nenhum deles igual a 0 tal que:
(a) o n
umero formado pelos 2 primeiros algarismos e divisvel por 2;
(b) o n
umero formado pelos 3 primeiros algarismos e divisvel por 3;
(c) o n
umero formado pelos 4 primeiros algarismos e divisvel por 4;
(d) o n
umero formado pelos 5 primeiros algarismos e divisvel por 5;
(e) o n
umero formado pelos 6 primeiros algarismos e divisvel por 6;
(f) o n
umero formado pelos 7 primeiros algarismos e divisvel por 7;
(g) o n
umero formado pelos 8 primeiros algarismos e divisvel por 8;
Qual e esse n
umero?

OBMEP 2007

159

Desaos

Solucoes da Lista 8

10. Qual
e o erro? - Uma das armacoes abaixo e falsa:
(a) Andre e mais velho do que Bruno;
(b) Claudia e mais nova do que Bruno
(c) A soma das idades de Bruno e Claudia e o dobro da idade de Andre;
(d) Cla
udia e mais velha do que Andre.
Quem e o mais velho? E o mais novo?

11. Soma - Nessa exerccio, as letras representam algarismos. Determine cada


uma das parcelas da soma abaixo.

abcdef
abcdef
+
ghij
.................................................................................................

def hjf

12. Bolinhas - Rogerio coloca seis bolinhas sobre a mesa de modo a formar dois
quadrados, como na gura. Ele percebe que havia esquecido de colocar mais
uma bolinha. Complete a gura formada pelas bolinhas com essa bolinha a
mais, de modo a formar 3 quadrados.
u

..................................................................................................
.....
....
....
....
....
...
....
....
....
....
....
....
....
....
....
....
....
....
..
...
...
................................................................................................

160

OBMEP 2007

Solucoes da Lista 8

Desaos

13. Um n
umero n
ao divisvel por 5 - Determine quais n
umeros naturais n
entre 2001 e 2007, tornam o n
umero 1n + 2n + 3n + 4n nao divisvel por 5.

14. Quatro fra


c
oes e um inteiro - Quantos n
umeros naturais a, b, c e d,
1 1 1 1
todos distintos, existem tais que + + + seja um inteiro?
a b c d

15. O Rei Arthur e o Drag


ao das Tr
es Cabe
cas e Tr
es Caudas - O
Rei Arthur teve que lutar com o Dragao das Tres Cabecas e Tres Caudas.
Sua tarefa cou facilitada quando conseguiu arranjar uma espada magica que
podia, de um so golpe, fazer uma e somente uma das seguintes coisas:
cortar uma cabeca;
cortar duas cabecas;
cortar uma cauda;
cortar duas caudas.
Alem disso, a Fada Morgana lhe revelou o segredo do dragao:
se uma cabeca e cortada uma nova cresce;
se duas cabecas sao cortadas nada acontece;
no lugar de uma cauda nascem duas caudas novas;
se duas caudas sao cortadas uma nova cabeca crece e
o dragao morre se perder as tres cabecas e as tres caudas.
Quantos golpes o Rei Artur vai precisar para matar o dragao?

16. Num tabuleiro 5x5, um cavaleiro do jogo de xadrez esta na casa marcada
com A. Depois ele se move marcando as casa por onde passa:
OBMEP 2007

161

Desaos

Solucoes da Lista 8

A B C D EF G H.
A

G
H
B

F
D

Partindo da casa H, o cavaleiro se move pelo tabulaeiro ate ter passado por
todas as 25 casas. Descreva o trajeto que ele fez.

17. Oito dados sao agrupados formando um cubo. Quantas faces cam visveis?

162

OBMEP 2007

Respostas dos desafios

Desafios

Respostas dos desafios


1.
1

13
4

11

6
7

10

16

2. 23
3. D=4225=25x169 e R=144x169=24336
4. Setembro de 2006
5.
..............................................................................................................................................................................................................................................................................
...
... ...........
...
.. ............
.........
.........
....
....
....
....
.........
.........
.........
....
....
....
....
.........
.........
.
.
.
.
.
.
.
.
....
.
....
.
.........
.........
....
.........
....
.........
.
....
.
....
.........
.
.
.........
.
.
.
....
.
....
.........
.........
..
.........
....
.
.........
.
....
.
....
......... ...
......... ...
.
..
.
...
.
..........................................................................................................................................................................................................................................................................

6. 746 (solucao u
nica?)
7.





........................................................



.......................................................

......

... ..........
......
......
...... ...
..
..
......
...... 
...
....
......
......
...... 
......
......
......
.....
....
....
....
......
......
....
....
...
....
...
....

.....................................................

....
....
...
....
....
...







.
.....
.....
.....
.....
.
.
.
.
...

.......
.......
.......
.......
....

........................................................



.
.....
.....
.....
.....
.
.
.
.
...

.....
.....
.....
.....
.....
....

.......................................................





8. 9 999 785 960.


9. 381654729
10. Claudia e Bruno.
OBMEP 2007

163

Desaos

Respostas dos desaos

11. 3 solucoes:
231468
231468
+
5972

264538
264538
+
9102

.................................................................................................

.................................................................................................

.................................................................................................

468908

538178

548698

12.
t

...
............
..... ... .....
..... .. .....
..... ..... .........
.
.
.
.
.....
...
.....
.....
.....
.....
.....
.....
.....
.....
....
.
.....
........................................................................................................
.
..... ..
.....
..... ..........
..... .....
.
.
.
.
....
.... ........
.
.....
.....
.....
..
...
.....
.....
..... ..... ..........
....
...
..... .. .....
...
....
.. . .
..........................................................................................................

13. 2004
14. 1
15. 5
16.
A

17. 20

164

273548
273548
+
1602

OBMEP 2007

Uma palavra aos alunos e professores

Uma palavra aos alunos e professores


O Banco de Questoes foi concebido por solicitacao de alunos e professores que tem
participado da Olimpada Brasileira de Matematica das Escolas P
ublicas (OBMEP).
Com o objetivo de facilitar e motivar a preparacao dos alunos para as provas, o Banco
de Questoes inspirou a criacao de diversos clubes de matematica nas escolas para
trabalhar com esse material.
Nesses 3 anos temos recebido, com muita alegria, mensagens de alunos e professores informando-nos sobre incorrecoes no Banco de Questoes, tais como erros
de digitacao, trocas de resposta, e alguns tambem nos oferecem outras solucoes
de alguns problemas. Essa troca tem propiciado um dialogo interessante e um
maior conhecimento recproco entre a equipe da OBMEP e a rede p
ublica escolar.
Aproveitamos para agradecer essa colaboracao.
Os alunos e professores que tem usado o Banco de Questoes nesses 3 anos de
existencia da OBMEP vao reparar que ele nao segue um modelo rgido, a cada ano
mudamos o seu formato, a quantidade e a dificuldade dos problemas. Esperamos
dessa forma contribuir para dar aos alunos e professores uma visao bem abrangente
do mundo fascinante que e o dos problemas de matematica.
Parte dos problemas aqui apresentados fazem parte de provas de olimpadas
nacionais e internacionais. Dessa forma pretendemos colocar os alunos da rede
p
ublica em contato com o mesmo tipo de preparacao que tem seus colegas em
diversos pases.
Os problemas estao agrupados nos 3 nveis por questao de organizacao; no entanto aconselhamos todos os alunos a passearem tambem em outros nveis diferentes do seu, e lembrem-se que e absolutamente natural encontrar dificuldades
em alguns problemas - elas devem ser vistas como desafios e nao como motivo de
desanimo.
Desejamos que esse Banco de Questoes torne o estudo da Matematica em sua
escola mais motivante e instigador.
Direc
ao Academica
da OBMEP
OBMEP 2008

Uma palavra aos alunos e professores

Organizado por:
Suely Druck (UFF)
Maria Elasir Seabra Gomes (UFMG)
Com a colaboracao de:
Ana L
ucia da Silva (UEL)
Edson Roberto Abe (Colegio Objetivo)
Fabio Brochero (UFMG)
Francisco Dutenhefner (UFMG)

ii

OBMEP 2008

Conte
udo
Uma palavra aos alunos e professores

Nvel 1

Lista 1 . . . . . . . . . . . . . . . . . . . . . . . . . . . . . . . . . . . . . .

Lista 2 . . . . . . . . . . . . . . . . . . . . . . . . . . . . . . . . . . . . . .

Lista 3 . . . . . . . . . . . . . . . . . . . . . . . . . . . . . . . . . . . . . .

Lista 4 . . . . . . . . . . . . . . . . . . . . . . . . . . . . . . . . . . . . . .

Lista 5 . . . . . . . . . . . . . . . . . . . . . . . . . . . . . . . . . . . . . .

Lista 6 . . . . . . . . . . . . . . . . . . . . . . . . . . . . . . . . . . . . . . 10
Nvel 2

11

Lista 1 . . . . . . . . . . . . . . . . . . . . . . . . . . . . . . . . . . . . . . 11
Lista 2

. . . . . . . . . . . . . . . . . . . . . . . . . . . . . . . . . . . . . 13

Lista 3 . . . . . . . . . . . . . . . . . . . . . . . . . . . . . . . . . . . . . . 14
Lista 4 . . . . . . . . . . . . . . . . . . . . . . . . . . . . . . . . . . . . . . 15
Lista 5 . . . . . . . . . . . . . . . . . . . . . . . . . . . . . . . . . . . . . . 17
Lista 6 . . . . . . . . . . . . . . . . . . . . . . . . . . . . . . . . . . . . . . 18
Nvel 3

19

Lista 1 . . . . . . . . . . . . . . . . . . . . . . . . . . . . . . . . . . . . . . 19
Lista 2 . . . . . . . . . . . . . . . . . . . . . . . . . . . . . . . . . . . . . . 21
Lista 3 . . . . . . . . . . . . . . . . . . . . . . . . . . . . . . . . . . . . . . 23
Lista 4 . . . . . . . . . . . . . . . . . . . . . . . . . . . . . . . . . . . . . . 25
Lista 5 . . . . . . . . . . . . . . . . . . . . . . . . . . . . . . . . . . . . . . 27
Lista 6 . . . . . . . . . . . . . . . . . . . . . . . . . . . . . . . . . . . . . . 29
iii

Uma palavra aos alunos e professores


Solu
c
oes do Nvel 1

31

Lista 1 . . . . . . . . . . . . . . . . . . . . . . . . . . . . . . . . . . . . . 31
Lista 2 . . . . . . . . . . . . . . . . . . . . . . . . . . . . . . . . . . . . . 35
Lista 3 . . . . . . . . . . . . . . . . . . . . . . . . . . . . . . . . . . . . . 39
Lista 4 . . . . . . . . . . . . . . . . . . . . . . . . . . . . . . . . . . . . . 43
Lista 5 . . . . . . . . . . . . . . . . . . . . . . . . . . . . . . . . . . . . . 45
Lista 6 . . . . . . . . . . . . . . . . . . . . . . . . . . . . . . . . . . . . . 48
Solu
c
oes do Nvel 2

51

Lista 1 . . . . . . . . . . . . . . . . . . . . . . . . . . . . . . . . . . . . . . 51
Lista 2 . . . . . . . . . . . . . . . . . . . . . . . . . . . . . . . . . . . . . . 55
Lista 3 . . . . . . . . . . . . . . . . . . . . . . . . . . . . . . . . . . . . . . 59
Lista 4 . . . . . . . . . . . . . . . . . . . . . . . . . . . . . . . . . . . . . . 63
Lista 5 . . . . . . . . . . . . . . . . . . . . . . . . . . . . . . . . . . . . . . 67
Lista 6 . . . . . . . . . . . . . . . . . . . . . . . . . . . . . . . . . . . . . . 70
Solu
c
oes do Nvel 3

73

Lista 1 . . . . . . . . . . . . . . . . . . . . . . . . . . . . . . . . . . . . . . 73
Lista 2 . . . . . . . . . . . . . . . . . . . . . . . . . . . . . . . . . . . . . . 77
Lista 3 . . . . . . . . . . . . . . . . . . . . . . . . . . . . . . . . . . . . . . 82
Lista 4 . . . . . . . . . . . . . . . . . . . . . . . . . . . . . . . . . . . . . . 88
Lista 5 . . . . . . . . . . . . . . . . . . . . . . . . . . . . . . . . . . . . . . 92
Lista 6 . . . . . . . . . . . . . . . . . . . . . . . . . . . . . . . . . . . . . . 97

iv

OBMEP 2008

Lista 1

Nvel 1

Nvel 1
Lista 1
1. O trajeto das formiguinhas -

As formiguinhas Maricota e Nandinha

passeiam numa varanda cujo chao e formado por lajotas retangulares de 4 cm


de largura por 6 cm de comprimento. Maricota parte do ponto M e Nandinha
do N , andando ambas apenas pelos lados dos retangulos, percorrendo o trajeto
no sentido indicado na figura.
-

M ..r........................................................

..
.....................................................................................
....
..
.....
.
...........................................................
.
....
..............................
....
....
....
....
...................................................................................
....
...
....
....
...
....
....
....
.........................................................
....
....
..
...............................

(a) As duas se encontram depois de andarem a mesma distancia. Qual foi


essa distancia?
(b) Aonde elas se encontraram?

2. A soma
e 100 - A soma de 3 n
umeros e 100, dois sao primos e um e a
soma dos outros dois.
(a) Qual e o maior dos 3 n
umeros?
(b) De um exemplo desses 3 n
umeros.
(c) Quantas solucoes existem para esse problema?

OBMEP 2008

Nvel 1

Lista 1

3. C
odigo de barras - Um servico postal usa barras curtas e barras longas
para representar o Codigo de Enderecamento Postal - CEP. A barra curta
corresponde ao zero e a longa ao 1. A primeira e a u
ltima barra nao fazem
parte do codigo. A tabela de conversao do codigo e mostrada abaixo.
11000 = 0

01100 = 5

00011 = 1

10100 = 6

01010 = 2

00001 = 7

00101 = 3

10001 = 8

00110 = 4

10010 = 9

(a) Escreva os CEP 36470130 na forma de codigo de barras.


(b) Identifique o CEP que representa o codigo de barras abaixo:

||||||||||||||||||||||||||||||||||||||||||
4. Atletas da escola - Numa escola, um quarto dos alunos joga somente volei,
um terco joga somente futebol, 300 praticam os dois esportes e 1/12 nenhum
deles.
(a) Quantos alunos tem a escola?
(b) Quantos alunos jogam somente futebol?
(c) Quantos alunos jogam futebol?
(d) Quantos alunos praticam um dos 2 esportes?

5. Dzima peri
odica - Qual e o algarismo da 1997a casa decimal de:
(a)

1
22

OBMEP 2008

(b)

1
27

Lista 2

Nvel 1

Lista 2
1. Ana na corrida - Para ganhar uma corrida, Ana deve completar os u
ltimos
5 km em menos de 20 minutos. Qual deve ser sua velocidade em km/h?

2. Quadradinhos e o buraco - Quantos quadradinhos foram retirados do


tabuleiro 10x20? Se o lado de cada quadradinho mede 1 cm, qual e a area e o
permetro do buraco?

3. Quadrados perfeitos no ret


angulo - Complete as seis casas da tabela,
colocando um algarismo em cada uma, de modo que os dois n
umeros de tres
algarismos formados na horizontal e os tres n
umeros de dois algarismos formados na vertical sejam quadrados perfeitos.
(a) Quais sao os n
umeros?
(b) Quantas solucoes existem?

4. Aula de divis
ao - Na aula sobre divisao a professora pediu que seus alunos
colocassem n
umeros no lugar das estrelas. Quais sao esses n
umeros?
.

..
38 ................F
........................
4 ?
^

..
75 .....................12
...................
F ?
^

..
? .....................3...................
F 7
^

OBMEP 2008

.
42 .......................?...................

F 5
^

Nvel 1

Lista 2

5. A festa de Rosa - Os convidados para festa de aniversario de Rosa comecaram


a chegar a partir das 18 horas. Maria chegou na meia hora depois de Ceclia,
mas meia hora antes de Alice. Rosa soprou as velinhas `as 21 horas e apenas
Ceclia nao estava, ela tinha outra festa e ja tinha ido embora. Alice foi a
u
ltima convidada a ir embora, `as 23h15min. Quais das afirmacoes abaixo sao
verdadeiras?
(a) Ceclia ficou menos do que 3 horas na festa.
(b) Ceclia ficou menos tempo na festa do que Maria.
(c) Alice ficou mais tempo na festa do que Maria.

OBMEP 2008

Lista 3

Nvel 1

Lista 3
1. Linhas de
onibus - No ponto de onibus perto da casa de Quinzinho, existem
duas linhas de onibus que ele pode usar para ir a escola: uma passa de 15 em
15 minutos e a outra de 25 em 25 minutos.
(a) Se os dois onibus passaram juntos `as 7 h 30 min, a que horas passarao
juntos novamente?
(b) De 7 h 30 min ate meia noite, quais os horarios em que os onibus passarao
juntos no ponto perto da casa de Quinzinho?
2. Quadrados dentro de um ret
angulo -

retangulo da figura esta dividido em 8 quadrados.


O menor quadrado tem lado 1cm e o maior 14cm.
(a) Determine o lado dos outros quadrados.
(b) Qual e o permetro do retangulo?

..........................................................
..
..
..
..
..
..
..
..
..
..
..
..
..
..
.
..
.............................................................
..
..
..
..
...
...
.
..
..
....................
..
........................................................

3. Festa na escola - A professora Ana foi comprar pao de queijo para homenagear os alunos premiados na OBMEP e deparou-se com a seguinte questao:
cada 100 gramas de pao de queijo custam R$ 3, 20 e correspondem a 10
paes de queijo;
cada pessoa come, em media, 5 paes de queijo.
A professora tem 16 alunos, um monitor e 5 pais de alunos. A precisao da
balanca da padaria e de 100 gramas.
(a) Quantos gramas de pao de queijo ela deve comprar para que cada pessoa
coma pelo menos 5 paes?
(b) Quanto a professora gastara?
(c) Se cada pessoa comer 5 paes de queijo, sobrara algum pao de queijo?
OBMEP 2008

Nvel 1

Lista 3

4. Ai que fome - Observe a tabela abaixo:


Salgados

Bebidas

Doces

Empada: R$ 3, 90

Refrigerante: R$ 1, 90

Sorvete: R$ 1, 00

Sanduche: R$ 2, 20

Refresco: R$ 1, 20

Cocada: R$ 0, 40

Pastel: R$ 2, 00

Agua:
R$ 1, 00

Bombom: R$ 0, 50

Maria deseja fazer um lanche contendo um salgado, uma bebida e um doce.


Ela possui 5 moedas de R$ 0, 50 centavos, 7 moedas de R$ 0, 25 centavos, 4
moedas de R$ 0, 10 centavos e 5 moedas de R$ 0, 05 centavos.
(a) Quantos reais Maria possui?
(b) Se o valor da passagem de onibus e R$ 0, 90 centavos, com essa quantia
quais as possveis combinacoes que ela pode fazer?

5. Advinhe - Tenho n
umeros naturais primos entre si. Se eu somar 50 a cada
um deles encontro n
umeros de dois algarismos. Se eu subtrair 32 de cada
um deles tambem encontro n
umeros naturais de 2 algarismos. Quais sao os
n
umeros?

OBMEP 2008

Lista 4

Nvel 1

Lista 4
1. Produto de consecutivos - Dentre os n
umeros 712, 548, e 1680 qual e
o u
nico que pode ser escrito como um produto de quatro n
umeros naturais
consecutivos?
2. Palndromos -

O ano 2002 e palndromo

porque e o mesmo quando lido da direita para

373 e 1221
foram anos palndromos.

a esquerda.
(a) Qual sera o proximo ano palndromo depois de 2002?
(b) O u
ltimo ano palndromo, 1991, era mpar. Quando sera o proximo ano
palndromo mpar?
(c) O u
ltimo ano palndromo primo ocorreu ha mais de 1000 anos, em 929.
Quando ocorrera o proximo ano palndromo primo?

3. O maior mdc - Quais sao os seis n


umeros de dois algarismos cujo maximo
divisor comum e o maior possvel?

4. Quantidade de
agua na terra - A Terra tem aproximadamente o volume de 1 360 000 000 km3 de agua que se distribuem nos oceanos, mares,
geleiras, regioes subterraneas (aq
uferos), lagos, rios e atmosfera. Somente a
agua encontrada nos tres u
ltimos itens tem facil acesso ao consumo humano.
Com estes dados complete a tabela a seguir:
Especificac
oes

Volume de
agua em km3

Agua
salgada

Agua
doce

Percentual
97%

40 000 000

Gelo

1, 8%

Agua
subterr
anea
Lagos e rios

Forma decimal do percentual

0, 0096
250 000

Vapor de
agua

0, 00001

OBMEP 2008

Nvel 1

Lista 4

5. Salas - Maria e Joao querem dividir uma area retangular de 10 m por 20 m.


Eles querem ter uma sala de jantar quadrada, ao lado de uma sala de visitas,
como mostra a planta ao lado. Eles precisam que a sala de visitas tenha mais
de 20 m2 e menos de 25 m2 , e que a de visitas tenha 30 m2 .
Quais as dimensoes que cada sala pode ter para que a sala de jantar tenha a
menor area possvel? De a resposta com aproximacao de uma casa decimal.

jantar

visitas

OBMEP 2008

Lista 5

Nvel 1

Lista 5
1. Bolas - De quantas formas podemos repartir 14 bolas entre 3 criancas de
modo que cada crianca receba no mnimo 3 bolas?

2. Minutos - Uma prova de Matematica comeca `as 12h 35min e tem duracao
5
de 4 horas. A que horas termina a prova?
6

3. Menor n
umero - Qual e o menor n
umero de 5 algarismos que se pode
formar com os algarismos 1, 2, 3, 4 e 9, que seja divisvel por 4?

4. Contas do papagaio - Antonio tem um papagaio que faz contas fantasticas


com n
umeros inteiros, mas nao sabe nada sobre decimais. Quando Antonio
sopra um n
umero em seu ouvido, o papagaio multiplica esse n
umero por 5,
depois soma 14, divide o resultado por 6, finalmente subtrai 1 e grita o resultado.
(a) Se Antonio soprar o n
umero 8, qual n
umero o papagaio grita?
(b) Se o papagaio gritou 3, qual o n
umero que Antonio soprou em seu ouvido?
(c) Porque o papagaio nunca grita o n
umero 7?

5. Soma maior que 34 - Quantos n


umeros de 4 algarismos existem cuja soma
de seus algarismos e maior do que 34?

OBMEP 2008

Nvel 1

Lista 6

Lista 6
1. Sem 1s - Roberto quer escrever o n
umero 111 111 como um produto de
dois n
umeros, nenhum deles terminado em 1. Isso e possvel? Por que?

2. N
umeros equilibrados - Um n
umero e dito equilibrado se um dos seus
algarismos e a media aritmetica dos outros. Por exemplo, 132, 246 e 777 sao
equilibrados. Quantos n
umeros equilibrados de 3 algarismos existem?

3. N
umeros primos - Quais os n
umeros entre 70 e 110, cujos triplos somados
mais um dao um n
umero primo?

4. Quadro moderno - Para fazer um quadro bem moderno para sua escola,
Roberto divide uma tela quadrada em 8 partes com 4 faixas de mesma largura
e a diagonal, como na figura. Ele pinta o quadro de azul e verde, de modo que
duas partes vizinhas tenham cores diferentes. No final, ele repara que usou
mais verde do que azul. Que fracao do quadro foi pintada de azul?

10

OBMEP 2008

Lista 1

Nvel 2

Nvel 2
Lista 1
1. Sapo Cururu - Cururu e um sapo estranho, ele se desloca apenas com dois
tipos de saltos, veja a seguir :
Salto tipo I:

10 cm para Leste e 30 cm para Norte;

Salto tipo II: 20 cm para Oeste e 40 cm para Sul.


20cm

30cm

10cm

Tipo I

40cm

Tipo II

(a) Como Cururu pode chegar a um ponto situado a 190 cm para Leste e
950 cm para Norte de sua casa?
possvel Cururu chegar a um ponto situado a 180 cm a Leste e 950 cm
(b) E
ao Norte de sua casa?

OBMEP 2008

11

Nvel 2

Lista 1

2. Distribuindo algarismos em linhas - Joana escreveu uma seq


uencia em
10 linhas usando os algarismos de 0 a 9, seguindo o padrao:
0
1 1 0
2 2 2 1 1 0
3 3 3 3 2 2 2 1 1 0
..
.
Qual o algarismo mais usado? Quantas vezes esse algarismo foi utilizado?

3. Ser
a que existe? - Existe um n
umero inteiro N tal que
2008 N = 222 . . . 2 ?

verdade que 1 + 1 + 1 < 1 ?


4. Limite de uma soma - E
43 53 63
12

5. Parte inteira - A parte inteira de um n


umero inteiro x e o maior inteiro
que e menor ou igual a x. Vamos denota-lo por [x]. Por exemplo:
[2, 9] = 2, [0, 88] = 0 e [1, 7] = 1. Calcule:

(a) [ 12]

12

(b)

28756
12777

(c)

OBMEP 2008

2007

2008

(d) [ 3 111]

Lista 2

Nvel 2

Lista 2
1. Soma nove - Quantos n
umeros inteiros entre 10 e 999 tem a soma de seus
algarismos igual a 9?

2. Ret
angulos - As medidas dos lados de um retangulo sao n
umeros pares.
Quantos desses retangulos existem com area igual a 96?

3. N
umero de retas - Sabemos que dois pontos distintos em um plano determinam uma e somente uma reta.
Quantas retas sao determinadas pelos pontos marcados
no quadriculado ao lado?

4. Cubo - Pedro quer pintar uma caixa na forma de um cubo de tal maneira
que as faces que tem uma aresta em comum sao pintadas em cores diferentes.
Calcule o n
umero mnimo de cores necessarias para pintar o cubo.

- Um terreno retangular foi divido em 4 terrenos, tambem retangulares.


5. Area
As areas de 3 deles estao dadas na figura em km2 . Qual e a area do terreno
que foi dividido?

OBMEP 2008

13

Nvel 2

Lista 3

Lista 3
1. Inteiro mais pr
oximo - Determine o n
umero inteiro mais proximo de:
(a)

19 19
+
15
3

(b)

85 43 29 15
+
+
+
42 21 14
7

2. Brincando com n
umeros mpares -

(c)

11 1 7 2
+
10 2 5 3

Beatriz adora n
umeros mpares.

Quantos n
umeros entre 0 e 1000 ela pode escreve usando apenas algarismos
mpares?

3. Agua
no jarro - Joao e Maria tem um jarro grande, cada, com um litro de
agua em cada um. No primeiro dia, Joao coloca 1 ml da agua do seu jarro no
jarro da Maria. No segundo dia, Maria coloca 2 ml da agua do seu jarro no
jarro do Joao. No terceiro dia, Joao coloca 3 ml da agua do seu jarro no jarro
da Maria, e assim por diante. Depois de 200 dias, quantos mililitros de agua
tem no jarro de Maria?

4. Formiga no cubo - Uma formiga parte de um vertice de um cubo andando


somente sobre as arestas ate voltar ao vertice inicial. Ela nao passa duas vezes
por nenhum vertice. Qual e o passeio de maior comprimento que a formiga
pode fazer?

5. Promo
c
ao - Em uma promocao, Joana comprou blusas de R$15, 00 cada e
calcas de R$17, 00 cada, gastando ao todo R$143, 00. Quantas blusas e calcas
Joana comprou?

14

OBMEP 2008

Lista 4

Nvel 2

Lista 4
1. Soma de cubos - Se x + y = 1 e x2 + y 2 = 2, calcule x3 + y 3 .

2. O revezamento em uma corrida - Numa competicao de revezamento,


cada equipe tem dois atletas que tem que correr 21 km cada um. O segundo
atleta so inicia a corrida quando o primeiro atleta termina a sua parte e lhe
passa o bastao. O recorde dessa competicao e de 2 horas e 48 minutos. Na
equipe de Joao e Carlos, Joao inicia a corrida e corre a sua parte com uma
velocidade de 12 km/h. Para bater o recorde, qual deve ser a velocidade de
Carlos?

3. Produtos consecutivos - Divida os n


umeros 2, 3, 5, 7, 11, 13 e 17 em dois
grupos de tal forma que multiplicando todos os n
umeros de um grupo e todos
do outro encontramos n
umeros consecutivos.

4. Distraindo na fila - Vivi, Tania e Rosa estao em fila, nao necessariamente


nessa ordem e gritam, cada uma sucessivamente, um m
ultiplo de 3:
3

, 6

, 9,

12
..
.

, 15
.
, ..

, 18 ,
.
, .. ,

Vivi foi a primeira a gritar um n


umero maior que 2003 e Rosa a primeira a
gritar um n
umero de 4 algarismos. Quem gritou o n
umero 666? E o 888?

OBMEP 2008

15

Nvel 2

Lista 4

5. N
umero e o dobro - Um n
umero menor do que 200 e formado por 3 algarismos diferentes, e o dobro desse n
umero tambem tem todos os algarismos
diferentes. Ainda, o n
umero e seu dobro nao tem algarismos em comum. Qual
e esse n
umero? Quantas solucoes tem esse problema?

16

OBMEP 2008

Lista 5

Nvel 2

Lista 5
1. Invertendo os algarismos - Quantos n
umeros entre 10 e 99 existem tais
que invertendo a ordem de seus algarismos, obtemos um n
umero maior que o
n
umero original?

2. Raz
ao entre segmentos - Na figura, O e

o centro do semi-crculo de diametro P Q, e


_

RM e perpendicular a PQ. Se o arco P R e o


_

dobro do arco RQ, qual e a razao entre P M


e M Q?

3. Tri
angulos -

Quais os triangulos cujas medidas dos lados sao n


umeros

inteiros e com permetro 15 cm?

4. N
umero interessante - O n
umero 119 e muito interessante porque dividido
por 2 deixa resto 1, dividido por 3 deixa resto 2, dividido por 4 deixa resto 3,
dividido por 5 deixa resto 4 e finalmente dividido por 6 deixa resto 5. Existem
outros n
umeros de tres algarismos com esta mesma propriedade?

5. Time vencedor - Um time de futebol ganhou 60% das 45 partidas realizadas. Qual e o n
umero mnimo de partidas que ele precisa jogar para atingir
a porcentagem de 75% de vitorias?

OBMEP 2008

17

Nvel 2

Lista 6

Lista 6
1. Brincando com dados - Dois dados sao lancados. Qual e o percentual do
produto dos n
umeros obtidos nos 2 dados ser divisvel por 6?

2. Contando solu
c
oes - Quantos sao os pares de n
umeros inteiros positivos
xy
(x, y) tais que
= 144?
x+y

3. Crculos tangentes - Os vertices de um triangulo de lados 3 cm, 4 cm e


5 cm sao centros de tres crculos dois a dois tangentes . Qual e a soma das
areas destes tres crculos?

4. Grupo de amigos - Joao, Jorge, Jose e Jan sao bons amigos. Joao nao tem
dinheiro, mas seus amigos tem. Jorge deu a Joao um quinto de seu dinheiro,
Jose deu um quarto de seu dinheiro e Jan deu um terco de seu dinheiro. Se
todos eles deram para Joao a mesma quantidade de dinheiro, que fracao do
dinheiro do grupo ficou com Joao?

5. Um trap
ezio is
osceles - Na figura,
o trapezio ABCD e isosceles, AB e paralelo a CD e as diagonais AC e BD
cortam-se no ponto P . Se as areas dos
triangulos 4ABP e 4P CD sao 4 cm2
e 9 cm2 , respectivamente, qual e a area

do triangulo 4P BC?

18

H
"
HH
" T
"

T
HHP "

T
"
HH
"

T
HH
"
"

HH T
"

"
HH T
""
HTH

T
"

OBMEP 2008

Lista 1

Nvel 3

Nvel 3
Lista 1
1. Problema de nota - Um professor propoe 80 problemas a um aluno, informando que lhe atribuira cinco pontos por problema resolvido corretamente
e lhe retirara tres pontos por problema nao resolvido ou resolvido incorretamente. No final o aluno tinha oito pontos. Quantos problemas ele resolveu
corretamente?

2. Quadrados e tri
angulos - Na figura tem-se 16 pontos formando um reticulado quadrado e duas retas, r e s, perpendiculares entre si.

(a) Quantos quadrados podemos construir, de tal maneira que seus vertices
pertencam ao reticulado, porem nenhum de seus lados sejam paralelos `as
retas r e s?
(b) Quantos triangulos isosceles podemos construir, de tal maneira que seus
vertices pertencam ao reticulado, porem nenhum de seus lados sejam
paralelos `as retas r e s?
OBMEP 2008

19

Nvel 3

Lista 1

3. C
alculo de
areas - Em cada uma das figuras a seguir tem-se um quadrado
de lado r. As regioes hachuradas em cada uma destas figuras sao limitadas por
lados desse quadrado ou por arcos de crculo de raio r de centros nos vertices
do quadrado.
Calcule cada uma dessas areas em funcao de r.

(a)

(b)

4. Seq
u
encia de algarismos - Todos os n
umeros naturais de 1 em diante sao
escritos consecutivamente formando a seguinte seq
uencia de algarismos:

1234567891011121314151617181920212223...
Qual algarismo aparece na posicao de n
umero 206 788?

5. Soma constante - Coloque os n


umeros 663, 664, 665, 666, 667, 668, 669,
670 e 671, sem repetir, em uma tabela 3 3, de tal maneira que a soma em
cada linha, em cada coluna e cada diagonal seja 2001.
Caso nao seja possvel, justifique sua resposta.

20

OBMEP 2008

Lista 2

Nvel 3

Lista 2
1. Contando os zeros -

Quantos zeros existem no final do n


umero

92007 + 1?

possvel colocar um certo n


2. Crculos dentro do quadrado - E
umero de
crculos dentro de um quadrado de 1 centmetro de lado, tal que a soma dos
raios destes crculos seja maior que 2008 centmetros? Os crculos podem ser
apenas tangentes, nao vale intersecao de crculos em 2 pontos.

3. Construindo um n
umero - Encontre um n
umero de oito algarismos usando somente os algarismos 1, 2, 3, 4, cada um deles duas vezes, tal que:
(i) exista um u
nico algarismo entre os dois algarismos 1;
(ii) existam dois algarismos entre os dois algarismos 2;
(iii) existam tres algarismos entre os dois algarismos 3;
(iv) existam quatro algarismos entre os dois algarismos 4.

4. N
umero na circunfer
encia - Os n
umeros 1, 2, 3, 4, 5, 6, 7, 8 e 9 foram
escritos (em uma ordem desconhecida) ao redor de uma circunferencia. Lendo
esses n
umeros de 3 em 3 no sentido horario, formam-se 9 n
umeros de tres
algarismos. Determine a soma desses 9 n
umeros.

5. Cada pe
ca em seu lugar - Cinco pecas de metal, confeccionadas, respectivamente, de ouro, prata, bronze, platina e nquel, foram colocadas em 5 cofres
numerados de 1 a 5. Cada cofre contem uma peca, e o problema consiste em
descobrir qual peca esta em qual cofre.
OBMEP 2008

21

Nvel 3

Lista 2

Na porta de cada cofre esta escrita uma informacao. Das 5 informacoes, 4 sao
falsas e a u
nica que e verdadeira e aquela na porta do cofre que contem a peca
de ouro. Veja as informacoes:
Cofre 1: O ouro esta no cofre 2 ou 3.
Cofre 2: A prata esta no cofre 1.
Cofre 3: O bronze nao esta aqui.
Cofre 4: O nquel esta no cofre cujo n
umero e inferior de 1 ao que contem o
ouro.
Cofre 5: A platina esta no cofre cujo n
umero e superior de 1 ao que contem
o bronze.

22

OBMEP 2008

Lista 3

Nvel 3

Lista 3
1. Soma de quadrados - Encontre tres n
umeros em uma progressao aritmetica
de razao 2, tal que a soma de seus quadrados seja um n
umero formado de
quatro algarismos iguais.

2. Adivinhe o n
umero - Um n
umero quando dividido por 3, tem resto 1; por
4 tem resto 2; por 5 tem resto 3; por 6, tem resto 4. Qual o menor n
umero
inteiro positivo que satisfaz tais propriedades?

3. Um c
odigo - Na expressao abaixo, cada letra corresponde a um algarismo,
e letras diferentes correspondem a algarismos diferentes. Determine esses algarismos.
6 AOBM EP = 7 M EP AOB

4. Calculando dist
ancias - Na figura 4ABC e um triangulo equilatero de
3 cm de lado; e o triangulo retangulo 4BCD tem lados 3 cm, 4 cm e 5 cm.
Calcule a distancia entre os pontos A e D.

OBMEP 2008

23

Nvel 3

Lista 3

5. Calculando lados de um tri


angulo - Na figura, 4ABC e um triangulo
equilatero, e o ponto P e tal que P A = 3 cm, P B = 4 cm e P C = 5 cm.
Calcule o comprimento dos lados do triangulo 4ABC.

24

OBMEP 2008

Lista 4

Nvel 3

Lista 4
1. Amigo Oculto - Um grupo de 5 amigos decide brincar de amigo oculto.
Para isso, cada um dos 5 amigos compra um presente para seu amigo oculto.
Pelas regras do jogo cada um troca exatamente um presente com um u
nico
amigo. De quantas maneiras os presentes podem ser trocados?

2. Contando solu
co
es - Quantos sao os pares de n
umeros inteiros positivos
xy
(x, y) tais que
= 144?
x+y

3. Determinando uma seq


u
encia - Em uma seq
uencia de 80 n
umeros, qualquer termo, salvo os extremos, e igual ao produto de seus termos vizinhos. O
produto dos 40 primeiros termos da seq
uencia e 8. O produto de todos os
termos tambem e 8. Determine os dois primeiros termos desta seq
uencia.

4. Construindo uma cerca Carina esta desenhando a planta de um jardim


retangular que tera um de seus lados num muro

................................................................................................................................................
...

...
..
...
...
...
...
...
...
...
...
...
...
...
...
...
...
...
...
...
...
...
...
...
...
..
................................................................................................................................................

reto de pedras. Ela comprou 140 m de cerca, em

pedacos de 1m cada um para cercar os 3 outros

jardim

lados. Ela nao pode cortar esses pedacos e deve


gastar todos eles.

(a) Se os dois lados vizinhos ao muro de pedra tem 40 m cada um, qual sera
o comprimento do terceiro lado?
possvel que o maior dos lados a ser cercado tenha 85 m? E 65 m?
(b) E
Justifique.

OBMEP 2008

25

Nvel 3

Lista 4

5. Um quadril
atero especial - Na figura abaixo, os lados do quadrilatero
[ e ADC
\ sao retos,
da figura tem medidas inteiras e distintas, os angulos ABC
AD = 7 cm e BC = 11 cm . Quanto medem os lados AB e DC?
B

x
A

11

y
C

26

OBMEP 2008

Lista 5

Nvel 3

Lista 5
1. Tr
es quadrados -

No desenho abaixo, o quadrado ABCD tem area de

30 cm2 e o quadrado F HIJ tem area de 20 cm2 . Os vertices A, D, E, H e I


dos tres quadrados pertencem a uma mesma reta. Calcule a area do quadrado
BEF G.
G

B
F

2. Bolinha de gude - Tres amigos jogam uma partida de bolinha de gude com
a seguinte regra: o perdedor de cada rodada dobra as bolinhas dos outros jogadores; (ele da aos outros dois o n
umero de bolinhas de modo que fiquem com
o dobro do que tinham no incio da jogada). O 1 jogador perdeu a primeira
rodada, o 2 jogador a segunda, o 3 a terceira rodada e todos terminaram com
64 bolinhas cada um. Com quantas bolinhas cada amigo comecou a partida?

3. Uma soma - Calcule o valor da soma


S=

1
1
1
1
1
+
+
+ ... +
+
12 23 34
2006 2007 2007 2008

OBMEP 2008

27

Nvel 3

Lista 5

4. Dobrando papel - Uma folha retangular ABCD de area 1000 cm2 foi dobrada ao meio e em seguida desdobrada (segmento M N ); foi dobrada e desdobrada novamente (segmento M C) e finalmente, dobrada e desdobrada segundo
a diagonal BD. Calcule a area do pedaco de papel limitado pelos tres vincos
(regiao escura no desenho).
A

B
F

C
N

5. Uma
area - No triangulo ABC, M e o ponto medio do lado AC, D e um
b e P e o ponto de
ponto sobre o lado BC tal que AD e bissetriz do angulo B AC
intersecao de AD e BM . Sabendo que a area de ABC e 100 cm2 , AB = 10 cm
e AC = 30 cm, calcule a area do triangulo AP B.

28

OBMEP 2008

Lista 6

Nvel 3

Lista 6

1. Ultimos
algarismos - Quais sao os dois u
ltimos algarismos do n
umero
2008
z }| {
8 + 88 + 888 + + 88 88 ?

2. Idades m
ultiplas - Quando Isabel nasceu sua mae estava fazendo aniversario
de 20 anos. Se Isabel e sua mae viverem mais de 50 anos, quantas vezes a idade
das duas foram n
umeros m
ultiplos?

3. Blocos diferentes - Ana tem um cubo de 10 cm de lado. Ela cortou o cubo


em cubinhos de 1 cm de lado, e com esses cubinhos ela brinca de formar outros
blocos retangulares, mas sem que sobrem cubinhos. Por exemplo ela formou
um bloco de 10 20 5.

Quantos blocos diferentes ela pode construir com os cubinhos sem sobrar nenhum?
4. Quadro negro - A Ana escreveu os n
umeros de 1 ate 10 000 no quadro
negro e depois apagou todos os m
ultiplos de 7 e 11. Qual e o n
umero que ficou
na posicao 2008?

OBMEP 2008

29

Nvel 3

Lista 6

5. Conjunto sem m
ultiplos - Qual e o subconjunto de {1, 2, . . . , 100} com o
maior n
umero possvel de elementos e sem elementos que sejam m
ultiplos um
do outro?

30

OBMEP 2008

Lista 1

Solucoes do Nvel 1

Soluco
es do Nvel 1
Lista 1
1. O trajeto das formiguinhas (a) O trajeto de M a N e composto de 14 comprimentos e 12 larguras das
lajotas, logo seu comprimento e:
14 6 + 12 4 = 84 + 48 = 132 cm.
Como as formiguinhas percorrem a mesma distancia, cada uma deve andar 132 2 = 66 cm .
(b) Vamos acompanhar o percurso feito por Maricota desde o incio, ate completar 66 cm:
2 comprimentos
{z
}
|

26=12

2 comprimentos
|
{z
}

1 largura
| {z }

3 comprimentos
{z
}
|

4+12=16

12+42=54

1 largura
| {z }

18+16=34

8+34=42

1 comprimento
{z
}
|

4+54=58

2 larguras +
| {z }

6+58=64

1/2 largura
|
{z
}
2+64=66

O caminho de Maricota ate o ponto de encontro esta indicado na figura :


12
M ..r.................................................................................
.

.
....
...................................................................................................................34
...........
....
...
16
....
....
....
.
...................................................................................54
..
42
...
.
64 .............................................58
....
66 ....
...
....
...
....
...........................................................................................................................
....
.
.....
...
....
....
....
....
....
....
....
....
....................................................................................
....
...
...
....
...
..
..
...........................................

r
?

ponto de encontro

OBMEP 2008

31

Solucoes do Nvel 1

Lista 1

2. A soma
e 100 (a) Inicialmente observe que:
o maior n
umero e a soma dos outros dois;
o maior n
umero nao pode exceder 50, senao a soma dos tres seria
maior do que 100;
o maior n
umero nao pode ser menor que 50, senao a soma dos tres
seria menor do que 100.
Logo, o maior n
umero so pode ser 50.
(b) Os n
umeros 3, 47 e 50 formam uma solucao do problema.
(c) Existem tantas solucoes quantos sao os pares de primos que somam 50.
A tabela mostra todas as solucoes. Logo, esse problema tem 4 solucoes.

47

50

43

50

13

37

50

19

31

50

3. C
odigo de barras (a) Primeiramente, escrevemos o CEP na forma de 0s e 1s:
00101
| {z } 00110
| {z } 00001
| {z } 11000
| {z } 00011
| {z } 00101
| {z } 11000
| {z }
| {z } 10100
3

Podemos, agora, escrever o codigo de barras desse CEP:

||||||||||||||||||||||||||||||||||||||||||
Lembre que a primeira e a u
ltima barra nao fazem parte do codigo.
32

OBMEP 2008

Lista 1

Solucoes do Nvel 1

(b) Primeiramente, escrevemos o codigo de barras na forma de 0s e 1s:

| ||{z}
|||| ||
||| ||||| ||||| ||||| ||||| ||||| ||||| |
|{z} |{z} |{z} |{z} |{z} |{z} |{z}
01010 11000 01010 00110 11000 11000 01010 11000

Podemos, agora, escrever o CEP: 20240020.

4. Atletas da escola (a)


O n
umero total de alunos na escola e dado
pela fracao 12/12, que graficamente podemos
representar por um retangulo dividido em 12
partes iguais.
Denotaremos por V, F e NE o n
umero de alunos que jogam somente
volei, somente futebol e nenhum desses esportes, respectivamente. Agora
temos:
os 1/4 dos alunos que jogam somente volei correspondem a 3 quadrados;
os 1/3 dos alunos que jogam somente futebol correspondem a 4
quadrados;
os 1/12 dos alunos que nao jogam nenhum desses esportes correspondem a 1 quadrado.
V

NE

Sobram, entao, 4 retangulos para os alunos que nao jogam volei futebol,
ou seja esses 300 alunos correspondem a 4/12 = 1/3 do total dos alunos
OBMEP 2008

33

Solucoes do Nvel 1

Lista 1

da escola. Logo, o total de alunos na escola e


300 3 = 900 .
1
900 = 300 e o total de alunos que jogam somente futebol.
3
(c) Neste caso, os alunos que jogam futebol sao os que jogam so futebol mais

(b) Temos que

os que jogam futebol e volei, ou seja, 300 + 300 = 600.


11
(d) O total de alunos que praticam um dos esportes e
900 = 825, pois
12
1/12 dos alunos nao praticam nenhum dos esportes.

5. Dzima peri
odica 1
= 0, 0454545 . . .
22
Observemos que o algarismo 4 esta nas posicoes pares: 2, 4, 6, . . . e o

(a) Dividindo 1 por 22 temos:

algarismo 5 nas posicoes mpares: 3, 5, 7 . . .


Como 1997 e um n
umero mpar temos que o algarismo da 1997a casa
decimal e o 5.
1
= 0, 037037037 . . .
27
Observemos que os algarismos 0, 3 e 7 se repetem, sucessivamente, a cada

(b) Dividindo 1 por 27 temos:

tres casas decimais, sendo que o algarismo:


0 esta nas posicoes: 1, 4, 7, . . ., ou seja, se divididas por tres deixam
resto 1;
3 esta nas posicoes: 2, 5, 8, . . ., ou seja, se divididas por tres deixam
resto 2;
7 esta nas posicoes: 3, 6, 9, . . ., ou seja, sao m
ultiplos de 3.
Como a divisao 1997 3 deixa resto 2, o algarismo da
1997a casa decimal e o 3.

34

OBMEP 2008

..
1997 ................3........................
2 665
^

Lista 2

Solucoes do Nvel 1

Lista 2
1. Ana na corrida - Transformando minutos em horas temos que 20 minutos
20
1
corresponde a
= horas. Assim, a velocidade da Ana deve ser superior
60
3
5
av=
= 15 km/h. Nesse caso, a solucao e qualquer n
umero maior que 15,
1
3
logo temos varias solucoes.

2. Quadradinhos e o buraco - Comecando a contar os quadradinhos retirados


da linha de cima temos que o n
umero desses quadradinhos e
1 + 3 + 5 + 15 + 10 + 2 = 36.
Desde que cada quadradinho tem area 1 cm2 , a area do buraco e 36 cm2 . Contando quantos lados de quadradinhos tem o buraco obtemos 42 lados. Assim,
o permetro e 42 cm.

3. Quadrados perfeitos no ret


angulo (a) Os quadrados perfeitos sao n
umeros que terminam em
X

0 , 1 , 4 , 5 , 6 , 9. Os quadrados perfeitos de 2

algarismos sao: 16, 25, 36, 49, 64 e 81. Logo, 25, 36


e 81 nao podem aparecer na coluna assinalada com X.

Observe tambem que 0 nao pode aparecer nessa coluna.


Restam, entao, para essa coluna apenas os n
umeros 16, 49 e 64. Logo,
temos tres opcoes:
(I)

1
6

(II)

OBMEP 2008

4
9

(III)

6
4

35

Solucoes do Nvel 1

Lista 2

Vamos examinar cada uma das tres opcoes.


Opcao (I): Os quadrados perfeitos de 3 algarismos terminados em 6 sao
142 = 196 , 242 = 576 , 162 = 256 , 262 = 676 .
Como nenhum quadrado perfeito de 2 algarismos ter-

mina em 7 ou 2, os n
umeros 576, 256 e 676 nao podem

1 9 6
aparecer na segunda linha, so resta entao 196.
Agora, os quadrados perfeitos de 3 algarismos terminados em 1 sao:
112 = 121 , 212 = 441 , 312 = 961 , 192 = 361 , 292 = 841 .
Vemos que para ter os quadrados nas 3 colunas, so e

possvel completar a tabela com o n


umero 841.

Opcao (II): Os quadrados perfeitos de 3 algarismos terminados em 9 sao:


132 = 169 , 232 = 529 , 172 = 289 , 272 = 729 .
Analogamente, podemos preencher a segunda linha apenas com o n
umero 169. Na primeira coluna so pode
aparecer o n
umero 81, por ser o u
nico quadrado de 2

4
1

algarismos terminado em 1.
8
1

4
6

Temos agora duas opcoes para preencher a u


ltima casa em branco: 1 ou
3. No entanto, nem 814 nem 834 sao quadrados. Logo a opcao (II) e
impossvel.

36

OBMEP 2008

Lista 2

Solucoes do Nvel 1
Opcao (III): Os quadrados de 3 algarismos terminados em 4 sao:
122 = 144 , 222 = 484 , 182 = 324 , 282 = 784 .
Verificamos que so podemos preencher a segunda linha
com o n
umero 144 e na primeira coluna so pode aparecer

o n
umero 81. A u
nica escolha agora para a casa em

6
4

branco e o n
umero 6.
8

No entanto, 866 nao e quadrado perfeito. Logo a opcao (III) tambem e


impossvel.
(b) Pelo que vimos acima, existe apenas uma solucao:

4. Aula de divis
ao .

1a divisao:

..
38 ................F
........................
?
4
^

Temos: 38 4 = 34 = 2 17. Entao: F = 2 e ? = 17 ou F = 17 e ? = 2.


.

2a divisao:

..
75 .....................12
...................
F ?
^

Basta efetuar a divisao para obter: F = 3 e ? = 6.


.

3a divisao:

..
? .....................3...................
F 7
^

Temos: 3 7 = 21. Os possveis restos da divisao sao: 0, 1 e 2. Logo temos


as solucoes: ? = 21 e F = 0 ou ? = 22 e F = 1 ou ? = 23 e F = 2.

OBMEP 2008

37

Solucoes do Nvel 1

4a divisao:

Lista 2

.
42 .......................?...................

F 5
^

Trocando o divisor pelo quociente, observamos que basta efetuar a divisao,


para obter: ? = 8 e F = 2 .

5. A festa de Rosa (a) Verdadeira. Como todos chegaram a partir das 18 horas e Ceclia saiu
antes das 21 horas, ela ficou menos do que 3 horas na festa.
(b) Falsa. Pode ter acontecido a seguinte situacao:
chegada

partida

tempo na festa

Ceclia

18h

20h 55min

2h 55min

Maria

18h 30min

21h 05min

2h 35min

(c) Falsa. Maria chegou 30 minutos antes da Alice, mas pode ter sado 5
minutos antes, por exemplo:
chegada

38

partida

tempo na festa

Alice

19h

23h 15min

4h 15min

Maria

18h 30min

23h 10min

4h 40min

OBMEP 2008

Lista 3

Solucoes do Nvel 1

Lista 3
1. Linhas de
onibus (a) O menor m
ultiplo comum de 15 = 3 5 e 25 = 52 e 3 52 = 75. Assim,
se uma hora tem 60 minutos, entao 75 min correspondem a 1h 15 min.
Apos 1h 15 min, os dois onibus passarao novamente no ponto. Logo,
os onibus passarao novamente no ponto perto da casa de Quinzinho, `as
7 h 30 min + 1 h 15 min = 8h 45 min.

(b) Soluc
ao 1:
Para obtermos os horarios que os onibus passarao juntos no ponto de
onibus perto da casa de Quinzinho, devemos somar 1h 15min, obtendo:
8 h 45 min; 10 h; 11 h 15 min; 12 h 30 min; 13 h 45 min; 15 h ; 16 h 15 min ;
17 h 30 min; 18 h 45 min; 20 h; 21 h 15 min; 22 h 30 min; 23 h 45 min.
O proximo onibus ultrapassa o horario de meia noite.

Soluc
ao 2:
De 7h 30 min ate 24 h (meia noite) temos 24 7h 30 min = 16h 30 min,
que corresponde a 16 60 + 30 = 990 min.
Devemos, portanto encontrar os m
ultiplos de 75, que sao menores que
990. Eles sao:
75; 150; 225; 300; 375; 450; 525; 600; 675; 750; 825; 900; 975.
Note que 990 nao e m
ultiplo de 75.
Como 7h 30 min corresponde a 450 min, vamos somar 450 a cada um
dos m
ultiplos de 75h = 1h 15min, para obtermos os horarios em que os
onibus passarao juntos no ponto perto da casa de Quinzinho:

OBMEP 2008

39

Solucoes do Nvel 1

Lista 3

450 + 75 = 525 min = 8 h 45 min;


450 + 150 = 600 min = 10 h;
450 + 225 = 675 min = 11 h 15 min;
450 + 300 = 750 min = 12 h 30 min;
450 + 375 = 825 min = 13 h 45 min;
450 + 450 = 900 min = 15 h;
450 + 525 = 975 min = 16 h 15 min;
450 + 600 = 1050 min = 17 h 30 min;
450 + 675 = 1125 min = 18 h 45 min;
450 + 750 = 1200 min = 20 h;
450 + 825 = 1275 min = 21 h 15 min;
450 + 900 = 1350 min = 22 h 30 min;
450 + 975 = 1425 min = 23 h 45 min.

2. Quadrados dentro de um ret


angulo (a) Se o menor quadrado tem 1 cm de lado, entao
o lado do quadrado A mede 1 4 = 4 cm e
do quadrado B mede 4 + 1 = 5 cm. Como o
lado do maior quadrado mede 14 cm, entao o
quadrado C tem de lado 14 4 5 = 5 cm.
(b) Os

lados

do

retangulo

medem

14 cm e 14 + 5 = 19 cm, logo o permetro e


14 2 + 19 2 = 66 cm.

40

OBMEP 2008

..............................
............................14
..
..
..
..
..
..
..
..
..
..
..
..
..
..
.
..
............5....................5..................4...........
.
.
.
..
.. C .... B .... A ...
..........................................................................

Lista 3

Solucoes do Nvel 1

3. Festa na escola (a) O n


umero de pessoas que comerao os paes de queijo e:
a professora + 16 alunos + 1monitor + 5 pais = 23 pessoas.
Se cada pessoa come pelo menos 5 paes de queijo, sera necessario comprar
pelo menos
5 23 = 115 paes de queijo.
Cada pao de queijo pesa em media:

100
g. Logo, sera necessario comprar
10

10 115 = 1150 g de paes de queijo.


Mas, a precisao da balanca e de 100 g. Assim, arrendondando 1150 g
para 1200 g, temos a quantidade de pao de queijo que a professora deve
comprar .
(b) Como

1200
= 12, temos que a professora gastara:
100
12 3, 20 = R$ 38, 40 reais.

(c) A quantidade de paes de queijo comprado foi de

1200
= 120. Logo,
10

sobrara 120 115 = 5 paes de queijo.

4. Ai que fome (a) Maria possui:


50,50+70,25+40,10+50,05 = 2,50+1,75+0,40+0,25 = 4,90 reais.
(b) Tirando a passagem, resta para Maria fazer o lanche R$ 4, 00. Observe
que Maria nao pode escolher empada nem refrigerante. Temos entao as
seguintes opcoes de lanches que Maria pode escolher:
OBMEP 2008

41

Solucoes do Nvel 1

Lista 3

Opc
ao 1

Opc
ao 2

Opc
ao 3

Opc
ao 4

Sanduche: R$2, 20

Sanduche: R$2, 20

Sanduche: R$2, 20

Sanduche: R$2, 20

Refresco: R$1, 20

Refresco: R$1, 20

Agua:
R$1, 00

Agua:
R$1, 00

Cocada: R$ 0, 40

Bombom: R$0, 50

Cocada: R$0, 40

Bombom: R$0, 50

Total: R$3, 80

Total : R$3, 90

Total: R$3, 60

Total: R$3, 70

Opca
o 5

Opc
ao 6

Opc
ao 7

Opc
ao 8

Opc
ao 9

Pastel R$2, 00

Pastel R$2, 00

Pastel R$2, 00

Pastel R$2, 00

Pastel R$2, 00

Refresco: R$1, 20

Refresco: R$1, 20

Agua:
R$1, 00

Agua:
R$1, 00

Agua:
R$1, 00

Cocada: R$0, 40

Bombom: R$ 0, 50

Cocada: R$ 0, 40

Sorvete: R$ 1, 00

Bombom: R$0, 50

Total: R$3, 60

Total: R$3, 70

Total : R$3, 40

Total: R$3, 50

Total: R$4, 00

5. Advinhe - Como somando 50 ou subtraindo 32 ainda encontramos n


umeros
de 2 algarismos, os n
umeros procurados sao maiores do que que 41 e menores
do que 50.
Assim, os primos entre si, que estao entre 41 e 50 sao:

42

(a) 42 ;

43 ;

45 ;

47 ;

49.

(b) 43 ;

44 ;

45 ;

47 ;

49.

(c) 43 ;

45 ;

46 ;

47 ;

49.

(d) 43 ;

45 ;

47 ;

49.

OBMEP 2008

Lista 4

Solucoes do Nvel 1

Lista 4
1. Produto de consecutivos - Em primeiro lugar, note que se 3 n
umeros sao
consecutivos, entao um deles e divisvel por 3. Dentre os n
umeros dados apenas
1680 e divisvel por 3. Assim, temos: 1680 = 24 3 5 7 = 4 5 6 7.

2. Palndromos (a) O proximo e 2112.


(b) Como o n
umero deve ser mpar, entao e o n
umero 3003.
(c) O n
umero nao pode ter 4 algarismos, pois todo n
umero palndromo de 4
algarismos e do tipo abba e e divisvel por 11, pois a + b = b + a.
O primeiro n
umero palndromo de 5 algarismos e 10001 = 73 137 e nao
e primo. O proximo possvel candidato e 10201. Mas 10201 = 101 101.
Pode-se verificar que 10301 e n
umero palndromo primo.

3. O maior mdc - Para que o m.d.c. seja o maior possvel, o menor dos
n
umeros deve ser igual ao proprio m.d.c., e o maior dos n
umeros deve ser o
sextuplo do m.d.c. O maior m
ultiplo de 6 de 2 algarismos e 96. Logo, 96
e o maior dos n
umeros e o menor e 96 6 = 16. Portanto os n
umeros sao:
16, 32, 48, 64, 80 e 96.

4. Quantidade de
agua na terra - Denotemos V = 1 360 000 000. Lembre
1
. Assim,
que: 1% =
100
1 360 000 000
1% de V =
= 13 600 000.
100
97% =

97
= 0, 97 e 97% de V vale: 97 13 600 000 = 1 319 200 000.
100
OBMEP 2008

43

Solucoes do Nvel 1

Lista 4

40 000 000
= 0, 0294 = 0, 0294 100 = 2, 94%.
1 360 000 000
1, 8
= 0, 018 e 1, 8% de V vale:
100
1, 8 13 600 000 = 24 480 000.

1, 8% =

0, 0096 = 0, 0096 100 = 0, 96% e 0, 96% de V vale:


0, 96 13 600 000 = 13 056 000.

250 000
= 0, 00018 = 0, 00018 100 = 0, 018%.
1 360 000 000

0, 00001 = 0, 00001 100 = 0, 001% e 0, 001% de V vale:


0, 001 13 600 000 = 13 600.
Volume de
agua em km3

Especificac
oes

Percentual

Forma decimal do percentual

Agua
salgada

1 319 200 000

97%

0, 97

Agua
doce

40 000 000

2, 94%

0,0294

Gelo

24 480 000

1, 8%

0, 018

Agua
subterr
anea

13 056 000

0, 96%

0, 0096

Lagos e rios

250 000

0, 018%

0, 00018

Vapor de
agua

13 600

0, 001%

0, 00001

5. Salas - Designemos por ` o lado da sala de jantar. Logo, a sua area e `2 e,


de acordo com os dados, temos:

20 < `2 < 25
Sabemos que 2, 23 <

20 < ` <

25 2 5 < ` < 5 .

5 < 2, 24, segue que 4, 46 < 2 5 < 4, 48. Logo

4, 46 < 2 5 < ` < 5 .

Escolhemos l = 4, 46, logo l2 = 19, 9809.

44

OBMEP 2008

Lista 5

Solucoes do Nvel 1

Lista 5
1. Bolas - Primeiramente temos que saber de quantas maneiras podemos obter
14 como soma de 3 parcelas inteiras, cada uma delas maior ou igual a 3, isto
e:
14 = |{z}
. . . + |{z}
. . . + |{z}
...
3

14 =

14 =

As parcelas possveis sao:


14 =

14 =

14 =

3 + 3 + 8
3 + 4 + 7
3 + 5 + 6
4 + 4 + 6
4 + 5 + 5

Agora, para cada uma dessas possibilidades podemos fazer diferentes distribuicoes entre as 3 criancas, como podemos observar na tabela a seguir:

14 = 3 + 3 + 8

14 = 3 + 4 + 7

14 = 3 + 5 + 6

14 = 4 + 4 + 6

14 = 4 + 5 + 5

1a crianca

2a crianca

3a crianca

OBMEP 2008

45

Solucoes do Nvel 1

Lista 5

Temos, portanto, 3 + 6 + 6 + 3 + 3 = 21 maneiras diferentes para fazer a


distribuicao das balas entre as 3 criancas.
Observe que quando as 3 parcelas sao diferentes temos 6 possibilidades e
quando 2 sao iguais temos apenas 3 possibilidades.

2. Minutos - Observemos primeiramente que


5
5
h = 60 min = 50 min.
6
6
Logo a prova durou 4h 50min. Somando as horas e os minutos, temos:
12 h 35 min + 4 h 50 min = 16 h 85 min.
Mas, 85 min = 1 h 25 min. Logo, a prova termina `as 16 h 85 min = 17 h 25 min.
3. Menor n
umero - O n
umero tem que ser par, logo tem que terminar em
2 ou 4. Um n
umero e divisvel por 4 se o n
umero formado pelos 2 u
ltimos
algarismos for divisvel por 4. Assim, temos as possibilidades: 12, 24, 32, 92.
Como 9 e o maior algarismo, devemos coloca-lo o mais `a direita possvel.
Logo 9 e o algarismo da casa das dezenas. Os outros n
umeros devem ser
colocados em ordem decrescente `a esquerda de 92, ou seja, o n
umero deve
iniciar com o menor algarismo que e o 1. Portanto, o n
umero procurado e
13 492.

4. Contas do papagaio 5

+14

(a) Temos: 8 40 54 9 8. Logo o papagaio grita 8.

46

OBMEP 2008

Lista 5

Solucoes do Nvel 1

(b) Devemos fazer a operacao inversa do papagaio, comecando da u


ltima
operacao, ou seja, somar 1 ao n
umero, multiplicar o n
umero por 6, depois
subtrair 14 e o resultado dividir por 5:
+1

14

3 4 24 10 2.
Logo, Antonio soprou 2 no ouvido do papagaio.
+1

14

(c) Observe que 7 8 48 34 6, 8. Como 6,8 nao e um n


umero
inteiro, o papagaio nao sabe fazer divisao 34 5, por isso ele nunca grita
7.

5. Soma maior que 34 - O maior n


umero de 4 algarismos e 9999, cuja soma
dos seus algarismos e: 4 9 = 36.
Os n
umeros de 4 algarismos cuja soma dos seus algarismos e 35 sao:
8999 ;

9899 ;

9989 ;

9998.

Logo, temos 5 n
umeros de 4 algarismos com soma dos seus algarismos maior
do que 34.

OBMEP 2008

47

Solucoes do Nvel 1

Lista 6

Lista 6
1. Sem 1s - Fatorando 111 111 obtemos: 111 111 = 3 7 11 13 37. Como
3 7 = 21 e 1 1 = 1 temos que evitar 1 e 21 como fatores. Assim, temos os
produtos:
3 37037 ;

7 15873 ;

13 8547 ;

37 3003 ;

33 3367 ;

39 2849 ;

77 1443 ;

259 429 ;

143 777 ;

407 273.

Logo, Roberto tem 10 opcoes para escrever 111 111 como ele deseja.
2. N
umeros equilibrados - Note que se o n
umero equilibrado tem os tres
algarismos distintos, diferentes de zero, entao com os mesmos algarismos obtemos 6 n
umeros equilibrados. Para isso basta trocar os algarismos de posicao.
Por exemplo: 123 ;

132 ;

213 ;

231 ;

312 ;

321.

Se um dos 3 algarismos do n
umero equilibrado e 0, entao com esses algarismos
obtemos apenas 4 n
umeros equilibrados, pois o 0 nao pode estar na casa da
centena. Por exemplo: 102 ;

120 ;

201 ;

210.

Assim, vamos variar apenas os algarismos da centena e da dezena. O algarismo


da unidade sera a media dos 2 algarismos. Observe que os 2 algarismos sao
ambos pares ou mpares. Os possveis n
umeros equilibrados iniciando com:
total de n
umeros equilibrados

1 : ; 111 ;

132 ;

153 ;

174 ;

195 ; 1 + (4 6) = 25

2 : ; 201 ;

222 ;

243 ;

264 ;

285 ; (4 + 1 + 3 6) = 23

3:;

333 ;

354 ;

375 ;

396 ; (1 + 3 6) = 19

4:;

402 ;

444 ;

465 ;

486 ; (4 + 1 + 2 6) = 17

5:;

555 ;

576 ;

597 ; (1 + 2 6) = 13

6:;

603 ;

666 ;

687 ; (4 + 1 + 6) = 11

7:;

777 ;

798 ; (1 + 6) = 7

8:;

804 ;

888 ; (4 + 1) = 5

9:;
48

999 ; 1
OBMEP 2008

Lista 6

Solucoes do Nvel 1

Somando temos 121 n


umeros equilibrados de 3 algarismos.

3. N
umeros primos - Os n
umeros primos entre 70 e 110 sao:
71 ;

73 ;

79 ;

83 ;

89 ;

97 ;

101 ;

103 ;

107 ;

109.

102 ;

106 ;

108.

Subtraindo 1 de todos os n
umeros temos a lista:
70 ;

72 ;

78 ;

82 ;

88 ;

96 ;

100 ;

Desta lista os m
ultiplos de 3 sao:
72 ;

78 ;

96 ;

102 ;

108.

Logo, os n
umeros sao:
723 = 24 ;

783 = 26 ;

963 = 32 ;

1023 = 34 ;

1083 = 36.

De fato temos:
243+1 = 73, 263+1 = 79, 323+1 = 97, 343+1 = 103, 363+1 = 109.

4. Quadro moderno -

OBMEP 2008

49

Solucoes do Nvel 1

Lista 6

A figura (a) mostra como foi pintado o quadrado nas duas cores - ainda nao
sabemos qual dessas partes e azul ou verde. Agora, dividimos o quadrado em
4 faixas verticais como na figura (b). Note que dessa maneira, o quadrado
ficou dividido em 16 quadradinhos iguais.
A parte nao-hachurada compreende:
4 meios quadrados +8 quadrados = 10 quadrados.
|
{z
}
2 quadrados

10
Logo, a parte nao-hachurada corresponde a
do quadro, e portanto, a parte
16
16 10
6
hachurada corresponde a

= . Logo, a parte hachurada da figura e


16 16
16
6
de todo o quadrado.
a que foi pintada de azul e corresponde a
16

50

OBMEP 2008

Lista 1

Solucoes do Nvel 2

Soluco
es do Nvel 2
Lista 1
1. Sapo Cururu (a) Sejam x e y o n
umero de saltos do Tipo I e Tipo II, respectivamente.
Logo, devemos ter:

10x 20y = 190


30x 40y = 950

Resolvendo o sistema, encontramos x = 57 e y = 19. Logo, o sapo devera


dar 57 saltos do Tipo I e 19 do Tipo II.

(b) Uma vez que o n


umero de saltos, x e y, de cada tipo e um n
umero inteiro,
o sapo so alcancara o ponto desejado se o sistema

10x 20y = 180


30x 40y = 950
OBMEP 2008

51

Solucoes do Nvel 2

Lista 1

41
tiver solucao inteira. A solucao desse sistema e x = 59 e y = . Como
2
41
nao e inteiro, o nosso sapo nao conseguira alcancar o referido ponto.
2

2. Distribuindo algarismos em linhas -

De acordo com o padrao da

sequencia temos:
1a linha

2a linha

1 1 0

3a linha
..
.

2 2 2 1 1 0

10a linha 9 9 9 9 9 9 . . . 9 8 . . . . . . 1 1 0
Logo:
1 algarismo 0 em cada linha 1 10 = 10 algarismos 0 no total
2 algarismos 1 em 9 linhas

2 9 = 18 algarismos 1 no total

3 algarismos 2 em 8 linhas

3 8 = 24 algarismos 2 no total

4 algarismos 3 em 7 linhas

4 7 = 28 algarismos 3 no total

E assim por diante.


Portanto, trata-se de descobrir qual e o maior dos produtos abaixo, onde cada
um representa quantos algarismos de 0 a 1 aparecem na seq
uencia.
1|
9} , |3 {z
8} , 4| {z
7} , 5
6} , 6
5} , 7
4} , 8
3} , 9| {z
2} , 10
1}
{z10} , 2| {z
| {z
| {z
| {z
| {z
| {z
0

Como o maior produto e 30, os algarismos mais usados foram 4 e 5, trinta


vezes cada um.

3. Ser
a que existe? - Se esse n
umero N existir, entao
N=
52

2 111 . . . 1
111 . . . 1
222 . . . 2
=
=
.
2008
2 1004
1004
OBMEP 2008

Lista 1

Solucoes do Nvel 2

Logo, N nao e inteiro por ser o quociente de um n


umero mpar 111 . . . 1 por
um n
umero par 1004. Portanto nao existe tal N .

4. Limite de uma soma -

Uma maneira de verificar esta desigualdade e

efetuar a soma
1
1
1
+ 3 + 3,
3
4
5
6
para isso igualando os denominadores. Uma outra maneira e comparando cada
parcela desta soma, como fazemos a seguir.
Comparando as fracoes

1 1 1
1
,
e com temos:
5 6 3
4

1
1
1
<
= 3 =
5
4
5

3 3
1
1
1
<
= 3;
5
4
4

1
1
1
<
= 3 =
6
4
6

3 3
1
1
1
<
= 3;
6
4
4

1
1
1
<
= 3 =
4
3
4

3 3
1
1
1
<
= 3.
4
3
3

Entao:

1
1
1
1
1
1
3
3 1 1
3 1 1
1
+
+
<
+
+
=
=

<

=
.
43 53 63
43 43 43
43
4 4 4
4 3 3
12

5. Parte inteira -

1, 7

0, 88

2, 9

(a) Os n
umeros 9 e 16 sao quadrados perfeitos
e 9 < 12 < 16. Entao

9 < 12 < 16 = 3 < 12 < 4 = [ 12] = 3.


OBMEP 2008

12

53

Solucoes do Nvel 2

Lista 1

(b) Como 127772 < 28756 < 127773 temos:

28756
28756
2<
< 3 =
= 2.
12777
12777

(c) Como 2007 < 2008 temos:

2007
2007
1 <
< 0 =
= 1.
2008
2008

(d) Inicialmente, observamos que

a = 3 a,

para qualquer valor de a.

2 28756
12777

2007 0
2008

111 4

Como 43 = 64 < 111 < 53 = 125 temos:

53 < 111 < 43 = 5 < 3 111 < 4 = [ 3 111] = 5.

54

OBMEP 2008

Lista 2

Solucoes do Nvel 2

Lista 2
1. Soma nove -

Vamos dividir em dois casos: n


umeros de 2 algarismos e

n
umeros de 3 algarismos. No caso de n
umeros de 2 algarismos, basta listalos: 18, 27, 36, 45, 54, 63, 72, 81, 90, obtendo um total de 9 n
umeros. Os
n
umeros de tres algarismos podem ser obtidos da mesma maneira, ou seja,
listando os n
umeros:
108 ; 117 ; 126 ; 135 ; 144 ; 153 ; 162 ; 171 ; 180 ; 9 n
umeros
207 ; 216 ; 225 ; 234 ; 243 ; 252 ; 261 ; 270

; 8 n
umeros

306 ; 315 ; 324 ; 333 ; 342 ; 351 ; 360

; 7 n
umeros

405 ; 414 ; 423 ; 432 ; 441 ; 450

; 6 n
umeros

504 ; 513 ; 522 ; 531 ; 540

; 5 n
umeros

603 ; 612 ; 621 ; 630

; 4 n
umeros

702 ; 711 ; 720

; 3 n
umeros

801 ; 810

; 2 n
umeros

900

1 n
umero

Portanto, temos 9+8+7+6+5+4+3+2+1 = 45 n


umeros de tres algarismos.
Vamos fazer de uma maneira mais geral. Denotemos por n o algarismo da
centena. Entao a soma dos algarismos da dezenas e da unidade e 9 n, onde
n pode ser 1, 2, . . . , 9. Como o algarismo da dezena pode ser o algarismo 0,
temos 9 n + 1 = 10 n possibilidades de escolha, entre os algarismos 9 n
e 0.
Portanto, fixando o algarismo da centena em n, temos 10 n possibilidades
de escolha para o algarismo da dezena e alem disso, fica automaticamente
definido o algarismo da unidade.

OBMEP 2008

55

Solucoes do Nvel 2

Lista 2

Desde que o algarismo da centena pode ser: 1, 2, 3, 4, 5, 6, 7, 8 ou 9, temos:


(101)+(102)+(103)+(104)+(105)+(106)+(107)+(108)+(109) = 45,

n
umeros de tres algarismos cuja soma dos seus algarismos e 9.
Portanto, existem 9+45 = 54 n
umeros entre 10 e 999 cuja soma dos algarismos
e 9.

2. Ret
angulos - Se a e b denotam comprimento e largura do retangulo, temos
que a b = 96. Logo a e b sao divisores pares de 96. Assim, temos 4 retangulos
satisfazendo as condicoes dadas:
a

lados

48

2, 48

24

4, 24

16

6, 16

12

8, 12

3. N
umero de retas - Para contar o n
umero de retas dividiremos as retas de
acordo com suas posicoes:
retas parlelas aos lados dos quadrados: 3 horizontais e 3 verticais;

56

OBMEP 2008

Lista 2

Solucoes do Nvel 2
retas paralelas `as diagonais dos quadrados: 3 + 3 = 6;

outras retas: temos 42 = 8 retas, formando uma estrela, como mostrado


na figura.

Ao todo temos: 3 + 3 + 6 + 8 = 20 retas.

4. Cubo - Um cubo tem 6 faces distintas, duas a duas opostas. As faces opostas
nao tem aresta em comum. Temos 3 pares de faces opostas, logo tres cores
sao suficientes, basta pintar as faces opostas da mesma cor. Por outro lado, e
claro que duas cores nao bastam.

5. Area
- Sejam x, y, z e w as medidas dos
retangulos menores, como mostrado na figura. A
area procurada e:
(x + w)(y + z) = xy + xz + wy + wz.

Precisamos determinar xw, pois sabemos que: xy = 27, xz = 18 e wz = 72.


OBMEP 2008

57

Solucoes do Nvel 2
Mas,
x
xz
18
1
=
=
= .
w
zw
72
4
Como x =

27
segue que
y
27
1
y
=
yw = 4 27 = 108 .
w
4

Logo, a area e 27 + 18 + 72 + 108 = 225 km2 .

58

OBMEP 2008

Lista 2

Lista 3

Solucoes do Nvel 2

Lista 3
1. Inteiro mais pr
oximo (a) Temos:
19 19
4
1
9
3
+
=1+
+6+ =7+
=7+ .
15
3
15
3
15
5
19 19
3
1
+
esta entre 7 e 8. Como > , o n
umero inteiro
15
3
5
2
mais proximo e 8.
Logo, a soma

7+

7+

3
5

1
2

(b) Temos:
85 43 29 15
1
1
1
1
+
+
+
=2+
+2+
+2+
+2+ =
42 21 14
7
42
21
14
7
1
1
1
1
1
8+
+
+
+ =8+
42 21 14 7
7

1 1 1
+ + +1
6 3 2

2
=8+ .
7

85
43
29
15
2
1
+
+
+
esta entre 8 e 9. Sendo < , o
42
21
14
7
7
2
n
umero inteiro mais proximo e 8.
Logo, a soma

8+

2
7

8+

1
2

11 1 7 2
30 2
2
+ = + = 3 + . Logo, a expressao esta
10 2 5 3
10 3
3
2
1
entre 3 e 2. Como > , o n
umero inteiro mais proximo e 2.
3
2

(c) Temos:

3 +

3 +

OBMEP 2008

1
2

2
3

2
59

Solucoes do Nvel 2

Lista 3

2. Brincando com n
umeros mpares - Como cada algarismo e mpar temos
que:
n
umeros com 1 algarismo temos 5 possibilidades: 1, 3, 5, 7, 9;
n
umeros com 2 algarismos temos 5 possibilidades na casa das unidades e
5 na casa das dezenas, totalizando 5 5 = 25 n
umeros;
n
umeros com 3 algarismos temos 5 possibilidades na casa das unidades, 5
na casa das dezenas e 5 na casa das centenas, totalizando 5 5 5 = 125
n
umeros.
Logo, Beatriz pode escrever 5+25+125 = 155 n
umeros com todos os algarismos
mpares.

3. Agua
no jarro -

Inicialmente, o volume de agua no jarro da Maria e

1 l = 1000 ml. Depois de 200 dias e 1000 ml mais o que e colocado por Joao
menos o que ela tirou, para dar para o Joao, isto e:
1000 + 1 2 + 3 4+ +199 200
=

1000 + (1 2) + (3 4) + + (199 200)

z }| {
1000 (1 + + 1) = 900.

100

Logo, Maria tera 900 ml.

60

OBMEP 2008

Lista 3

Solucoes do Nvel 2

4. Formiga no cubo - Veja na figura um caminho percorrendo 8 arestas que


a formiga pode fazer partindo do vertice 1.

Sera que e possvel ela fazer um caminho passando por 9 arestas? Para fazer
esse caminho, ela teria que passar por 9 vertices, veja no desenho, lembrando
que o vertice de chegada e o mesmo que o de partida porque a formiguinha
volta ao vertice inicial:
vertice
de
partida

vertice
de
chegada


1
2 3
4
5
6
7 8
9

Como o cubo so tem 8 vertices, esse passeio nao e possvel. Logo, o passeio de
maior comprimento e o que tem 8 arestas.

5. Promo
c
ao - Sejam b e c o n
umero de blusas e calcas compradas, respectivamente. Logo temos:
15b + 17c = 143 ; sendo b e c n
umeros inteiros positivos .
Note que ambos, b e c, tem que ser menores do que 9, porque
15 9 + 17 9 > 143. Agora temos duas solucoes.

OBMEP 2008

61

Solucoes do Nvel 2

Lista 3

Solu
c
ao 1: Temos que
15b = 143 17c 143 17c e m
ultiplo de 15 .

Portanto, 143 17c termina em 0 ou 5. Isso significa que 17c termina em 3


ou 8. Logo,
c = 9 ou c = 4 .
Como c < 9, a u
nica solucao e c = 4. Segue que
b=

143 17 4
= 5.
15

Soluc
ao 2: Temos que
b=

143 17c
8 2c
=9c+
.
15
15

Note que 8 2c tem que ser m


ultiplo de 15 e c e um n
umero inteiro positivo.
Logo, 8 2c = 0, ou seja, c = 4. Da obtemos: b = 5.
Portanto, ele comprou 5 blusas e 4 calcas.

62

OBMEP 2008

Lista 4

Solucoes do Nvel 2

Lista 4
1. Soma de cubos - Temos: (x + y)2 = x2 + 2xy + y 2 . Substituindo os valores
1
de x + y e x2 + y 2 obtemos: 1 = 2 + 2xy = xy = .
2
Mas (x + y)3 = x3 + 3x2 y + 3xy 2 + y 3 . Logo,
x3 + y 3 = (x + y)3 3xy(x + y) = 1 + 3

5
1
1= .
2
2

2. O revezamento em uma corrida - Como tempo =

distancia
, o tempo
velocidade

gasto por Joao foi de:

21
t=
=
12

9
9
1+
h = 1h +
60 min = 1 h e 45 min.
12
12

Logo, Carlos tem que completar a prova num tempo inferior a


(2 h e 48 min) (1 h e 45 min) = 1 h e 3 min = 63 min.
Para isso sua velocidade v, em km/min deve satisfazer
21
1
60
21
< 63 ou seja, v >
= km/min =
km/h = 20 km/h.
v
63
3
3
Logo, Carlos deve correr com velocidade superior 20 km/h.
3. Produtos consecutivos Solu
c
ao 1: Como os produtos sao n
umeros consecutivos, denotemos esses
produtos por p e p + 1. Logo, temos
p(p + 1) = 2 3 5 7 11 13 17 = 510 510.
OBMEP 2008

63

Solucoes do Nvel 2

Lista 4

Resolvendo a equacao p2 +p510510 = 0, obtemos p = 714, e logo p+1 = 715.


Agora, fatorando esses n
umeros obtemos
714 = 2 3 7 17 e 715 = 5 11 13.

Soluc
ao 2: Se 2 e 5 estao no mesmo grupo, entao um dos produtos termina
em 0 e o outro, por ser consecutivo, tem que terminar em 1 ou 9. Os produtos
terminados em 1 sao 3 7 11 = 231, 3 17 11 = 561 e 7 11 13 = 1001.
Verifica-se que esses grupos nao sao solucao. Analogamente para os terminados
em 9. Conclumos que 2 e 5 estao em grupos diferentes. Logo um produto
termina em 5 e o outro em 4 ou 6, mas nao e possvel formar com os n
umeros
dados um produto terminado em 6. Logo, um dos produtos termina em 5 e o
outro em 4. Por tentativas obtemos a solucao
714 = 2 3 7 17 e 715 = 5 11 13.

4. Distraindo na fila - Observe que a que grita os n


umeros 9, 18, etc, vai
sempre gritar m
ultiplos de 9. O primeiro m
ultiplo de 3 com 4 algarismos e
1002 e o primeiro m
ultiplo de 3 maior que 2003 e 2004.
Logo Vivi gritou 2004 e Rosa 1002. Nenhum desses n
umeros e m
ultiplo de 9,
assim e Tania que grita os m
ultiplos de 9.

Rosa

64

Vivi

Tania

12

15

18

21
..
.

24
..
.

27
..
.

1002
..
.

2001

1005
..
.

,
,

1008
..
.

2004

2007

OBMEP 2008

Lista 4

Solucoes do Nvel 2

Desta forma, e Tania quem grita 666, por que 666 e m


ultiplo de 9. Ela tambem
grita o n
umero 891 = 888 + 3 por ser um m
ultiplo de 9. Logo, e Vivi quem
grita 888.

5. N
umero e o dobro - Inicialmente note que o dobro de um n
umero inteiro
e par, logo ele termina em 0, 2, 4, 6, ou 8. No entanto, o n
umero nao pode
terminar em 0, pois nesse caso o seu dobro tambem terminaria em 0, e logo
teriam 0 como algarismo comum. Portanto, temos os seguintes casos:
I
1 ... 5
2
............................................................................
(
2 ... 0
ou
3

II
1 ... 6
2
............................................................................
3 ... 2

III
1 ... 2
2
............................................................................
3 ... 4

IV
1 ... 7
2
............................................................................
(
2 ... 4
ou
3

V
1 ... 3
2
............................................................................
2 ... 6

VI
1 ... 8
2
............................................................................
(
2 ... 6
ou
3

Vamos, agora, determinar todas as possibilidades para cada caso, lembrando


sempre que o n
umero e seu dobro nao podem ter algarismos comuns.
Caso I temos 3 possibilidades:
135 2 = 270 ; 145 2 = 290 ; 185 2 = 370 .
Caso II temos 3 possibilidades:
176 2 = 352 ; 186 2 = 372 ; 196 2 = 392 .
OBMEP 2008

65

Solucoes do Nvel 2

Lista 4

Caso III temos 3 possibilidades:


152 2 = 304 ; 182 2 = 364 ; 192 2 = 384 .
Caso IV nao ha nenhuma possibilidade.
Caso V temos 2 possibilidades:
143 2 = 286 ; 153 2 = 206 .
Caso VI temos 5 possibilidades:
138 2 = 276 ; 148 2 = 296 ; 158 2 = 306 ;
178 2 = 356 ; 198 2 = 396 .
Finalmente, temos 3 + 3 + 3 + 2 + 5 = 16 solucoes para esse problema, a saber:
135 ; 145 ; 185 ; 176 ; 186 ; 196 ; 152 ; 182 ;
192 ; 143 ; 153 ; 138 ; 148 ; 158 ; 178 ; 198.

66

OBMEP 2008

Lista 5

Solucoes do Nvel 2

Lista 5
1. Invertendo os algarismos - Um n
umero de 2 algarismos e da forma a b.
Temos que contar os n
umeros que tem o algarismo da unidade maior do que
o algarismo da dezena, ou seja, b > a. Claramente, a nao pode ser 9.
Temos os seguintes casos:
1 b: O algarismo da unidade pode ser 2, 3, 4, 5, 6, 7, 8 ou 9, assim temos
8 possibilidades.
2 b: O algarismo da unidade pode ser 3, 4, 5, 6, 7, 8 ou 9, assim temos
7 possibilidades.
..
.
8 b: O algarismo da unidade so pode ser 9; ou seja, 1 possibilidade.
Logo, temos 8 + 7 + 6 + 5 + 4 + 3 + 2 + 1 = 36 n
umeros.

2. Raz
ao entre segmentos - Se o arco P R e o dobro do arco RQ, vale a
[ Como
mesma relacao entre os angulos centrais, logo: P[
OR = 2ROQ.
[ = 180 ,
P[
OR + ROQ
segue-se que
[ + ROQ
[ = 3ROQ
[ = 180 ,
2ROQ
[ = 60 . Mas, OR = OQ = r raio do crculo. Da conclumos que
donde ROQ
o triangulo 4ORQ e eq
uilatero. Portanto, a altura RM tambem e mediana,
ou seja: OM = M Q. Logo, se r e o raio do crculo temos:
r
r+
PM
P O + OM
=
= r 2 = 3.
OQ
MQ
2
2
OBMEP 2008

67

Solucoes do Nvel 2

Lista 5

3. Tri
angulos - Se a, b e c sao os comprimentos dos lados, podemos supor que
a b c. Desde que um lado de um triangulo e sempre menor que a soma
dos outros dois, temos que c < a + b. Segue-se que
2c < a + b + c 3c = 2c < 15 3c.
Como c e um n
umero inteiro, entao c = 5, 6, 7.
Se c = 7, entao a + b = 8 e temos 4 solucoes (a, b, c): (1, 7, 7), (2, 6, 7),
(3, 5, 7) e (4, 4, 7).
Se c = 6, entao a + b = 9 e temos 2 solucoes (a, b, c): (3, 6, 6) e (4, 5, 6).
Se c = 5, entao a + b = 10 e temos 1 solucao (a, b, c): (5, 5, 5).
Assim, temos 7 triangulos.

4. N
umero interessante - Suponhamos que N seja um dos n
umeros procurados. Como N e 119 deixam os mesmos restos quando divididos por 2, 3, 4, 5
e 6 temos que a diferenca entre eles N 119 deixa resto zero quando dividido
por esses n
umeros. Portanto N 119 e m
ultiplo de 2, 3, 4, 5 e 6. Como 60
e o mnimo m
ultiplo comum desses n
umeros, N 119 tambem e m
ultiplo de
60. Logo, N 119 = 60k, k N, ou seja, N = 119 + 60k. Atribuindo valores
para k temos:
119 + 0 ;

119 + 60 = 179 ;

119 + 2 60 = 239 ; . . . ;

119 + 14 60 = 959.

Logo, existem mais 14 n


umeros com esta propriedade.

60
= 27 partidas.
100
Se ele ganhar mais n partidas, a porcentagem de partidas ganha sera:

5. Time vencedor - O time ja ganhou 60% de 45 = 45

27 + n
75
no de partidas ganhas
=
= 75% =
.
o
n de partidas jogadas
45 + n
100
68

OBMEP 2008

Lista 5

Solucoes do Nvel 2

Logo 2700 + 100n = (45 + n) 75 e portanto 25n = 675. Da temos n = 27.

OBMEP 2008

69

Solucoes do Nvel 2

Lista 6

Lista 6
1. Brincando com dados - Na seguinte tabela marcamos com os produtos
que sao divisveis por 6
1

5
6

Assim temos 15 casos favoraveis de 36 possibilidades. Logo, o percentual do


15
= 41, 7%.
produto ser divisvel por 6 e
36

2. Contando solu
c
oes - Isolando x na equacao

xy
= 144 obtemos
x+y

144y
. Como x deve ser positivo, devemos ter y = 144 + n, onde n
y 144
e um n
umero inteiro positivo. Substituindo essa expressao de y no valor de
1442
+ 144. Como x deve ser um n
umero inteiro, n deve
x, obtemos x =
n
ser um divisor de 1442 . Sendo 1442 = 124 = 28 34 , segue que 1442 tem
x =

(8 + 1) (4 + 1) = 45 divisores. Assim, para cada divisor n de 1442 , obtemos


uma solucao

(x, y) =

1442
+ 144 , 144 + n
n

xy
= 144. Assim essa equacao possui 45 pares ordenados de
x+y
n
umeros inteiros positivos (x, y) que a satisfazem.

da equacao

70

OBMEP 2008

Lista 6

Solucoes do Nvel 2

3. Crculos tangentes -

Denotemos por r1 , r2 e r3 os raios dos tres crculos. Como os crculos sao


tangentes dois a dois temos que :

r + r2 = 3;

1
r1 + r3 = 4;

r + r = 5.
2
3
Substituindo os valores r2 = 3 r1 , r3 = 4 r1 na terceira equacao temos:
3 r1 + 4 r1 = 5. Da, obtemos que r1 = 1, r2 = 2 e r3 = 3. Logo, a soma
das areas dos tres crculos e (12 + 22 + 32 ) = 14 cm2 .

4. Grupo de amigos - Se A e a quantidade de dinheiro que Joao recebeu


de cada um de seus amigos, entao ele recebeu um total de 3A. Como ele
recebeu de Jorge um quinto do seu dinheiro, entao Jorge tinha 5A. Da mesma
maneira Jose tinha 4A e Jan tinha 3A. Assim, os tres amigos tinham
5A + 4A + 3A = 12A e a fracao do dinheiro do grupo que ficou com Joao foi
3A
1
de
= .
12A
4

OBMEP 2008

71

Solucoes do Nvel 2

Lista 6

5. Um trap
ezio is
osceles -

Seja H a altura dos triangulos 4DP C e 4CP B relativa `as bases DP e P B,


respectivamente. Logo, area(4DP C) = 21 H DP e area(4CP B) = 12 H P B,
e portanto

1
H DP
area(4DP C)
DP
= 21
.
=
area(4CP B)
P
B
H

P
B
2

Da mesma maneira, se h e a altura dos triangulos 4AP B e 4CP B relativa


`as bases AP e P C, respectivamente, temos que
1
h PC
area(4CP B)
PC
= 21
=
.
area(4AP B)
AP
h AP
2

\ = BCD.
\
Como o trapezioABCD e isosceles, temos que AD = BC e ADC
Da temos que os triangulos 4ADC

e 4DBC sao congruentes, pois tem

\
\
dois lados e o angulo entre eles iguais. Conseq
uentemente, P
DC = P
CD e
P[
AB = P[
BA. Portanto, DP = P C e P B = P A. Logo,
area(4DP C)
DP
PC
area(4CP B)
=
=
=
.
area(4CP B)
PB
PA
area(4AP B)
Logo,
[area(4CP B)]2 = area(4AP B) area(4DP C) = 4 9 = 36,
portanto area(4BP C) = 6 cm2 .

72

OBMEP 2008

Lista 1

Solucoes do Nvel 3

Soluco
es do Nvel 3
Lista 1
1. Problema de nota - Seja c o n
umero de problemas resolvidos corretamente
e seja e a soma do n
umero de problemas resolvidos incorretamente e do n
umero
de problemas nao resolvidos. Logo c + e = 80 e o n
umero de pontos do aluno
na avaliacao e 5c 3e. No caso,

c + e = 80
5c 3e = 8
Portanto, c = 31 e e = 49. Logo, o aluno resolveu 31 problemas corretamente.

2. Quadrados e tri
angulos (a) Os u
nicos quadrados que nao tem nenhum de seus lados paralelos `a reta
r, nem `a reta s sao os do tipo 1 e do tipo 2 (ver figuras).

Tipo 1

Tipo 2

OBMEP 2008

73

Solucoes do Nvel 3

Lista 1

Desta forma, ha um total de seis quadrados: quatro do primeiro tipo e


dois do segundo tipo.
(b) O total de triangulos e dezesseis, todos eles tem catetos iguais a

unidades, e hipotenusa de 10 unidades.

Cada um dos quadrados do segundo tipo, como feito em (a), nos da quatro
triangulos, obtendo assim oito triangulos. Os oito triangulos, restantes,
sao obtidos atraves de uma u
nica translacao horizontal ou vertical de cada
um dos anteriores. Na figura a seguir, esta a u
nica translacao possvel de
um dos quatro triangulos de um quadrado feito no item (a).

3. C
alculo de
areas (a) A area hachurada corresponde a um quarto da area
A area do crculo

de um crculo de raio r. Portanto a area hachurada


de raio r e r2 .
1
e igual r2 .
4
hachurada e
(b) Observe no item anterior que a area da regiao NAO

area do quadrado area da regiao hachurada = r2

74

OBMEP 2008

(4 )r2
r2
=
.
4
4

Lista 1

Solucoes do Nvel 3
Voltando ao nosso item, a area da regiao hachurada e

area do quadrado2 (area da regiao X) = r2 2

(4 )r2
r2 2
=
r .
4
2

4. Seq
u
encia de algarismos - N
umeros com 1 algarismo formam os 9 primeiros
termos da seq
uencia. Os 90 n
umeros de 2 algarismos formam os 180 termos
seguintes. Depois vem os 2 700 termos correspondentes aos n
umeros de tres
algarismos; depois mais 36 000 termos correspondentes aos n
umeros de 4 algarismos e finalmente, temos 450 000 termos que sao os correspondentes aos
n
umeros de 5 algarismos. Logo, enumerando os termos da seq
uencia temos:

a1 , . . . , a9 , a10 , . . . , a189 , a190 , . . . , a2889 , a2890 , . . . , a38889 , a38890 , . . . , a488889


{z
} |
{z
} |
{z
} |
{z
}
| {z } |
1 alg

2 algs

3 algs

4 algs

5 algs

Para escrever todos os termos de 1, 2, 3 e 4 algarismos, chegamos ao 38889a


casa da seq
uencia. Logo, o algarismo na 206788a casa faz parte de um n
umero
de 5 algarismos, ou seja esta no bloco
a38890 , . . . , a488889 .
|
{z
}
5 algs

Esse bloco e da forma


10 000 , 10 001 , . . . , 99 999 .
OBMEP 2008

75

Solucoes do Nvel 3

Lista 1

Para ver quantos n


umeros de 5 algarismos existem da posicao 38 889 ate a
posicao 206 788, divide-se esta diferenca por 5. Assim, temos 206 78838 889 =
167 899 e 167 899 = 5 33 579 + 4.
Portanto, precisamos de 33 579 n
umeros de 5 algarismos mais os quatro primeiros
algarismos do 33 580o n
umero de 5 algarismos (que e 43 579), para chegar ao
algarismo de n
umero 206 788. Como o quarto algarismo do n
umero 43 579 e
7, temos que o algarismo procurado e o 7.

5. Soma constante - Uma solucao e

76

670

665

666

663

667

671

668

669

664

OBMEP 2008

Lista 2

Solucoes do Nvel 3

Lista 2
1. Contando os zeros - A tabela ao lado mostra como
aparecem em ordem, dezena e unidade, os dois u
ltimos

algarismos de algumas potencias de 9. Observe que esses


dois u
ltimos algarismos de 90 e 910 sao os mesmos; logo
a partir 910 a segunda coluna da tabela comecara a se
repetir, formando uma seq
uencia periodica, de perodo
10. Como 2007 = 10 200 + 7 e os dois u
ltimos algarismos de 910200 sao 01, segue que os dois u
timos
algarismos de 92007 sao os dois u
ltimos algarismos de 97 ,
ou seja 69. Da os dois u
ltimos algarismos de 92007 + 1

0
1
2
3
4
5
6
7
8
9
10

dois u
ltimos
algarismos
de 9n
01
09
81
29
61
49
41
69
21
89
01

sao iguais a 69 + 1 = 70. Portanto, existe um u


nico zero
no final do n
umero 92007 + 1.

2. Crculos dentro do quadrado - A resposta desse problema e afirmativa:


sim, e possvel colocar um certo n
umero de crculos dentro de um quadrado de
1 centmetro de lado, tal que a soma dos raios desses crculos seja maior que
2008 centmetros.
Para exibir uma tal configuracao , desenhe linhas paralelas aos lados do quadrado, dividindo-o em n2 quadradinhos menores; cada um desses quadradinhos
1
tem lado igual a . Agora, dentro de cada um desses quadradinhos, desenhe
n
1
uma crculo de raio igual a
. Veja essa construcao, no caso particular n = 4,
2n
na figura a seguir.

n2

42 = 16 crculos

1
lados dos quadradinhos =
4

raio
dos
c
rculos
=

soma dos raios:16 1


8

OBMEP 2008

77

Solucoes do Nvel 3

Lista 2

1
n
=
.
2n
2
Como estamos interessados no caso desta soma ser maior que 2008, devemos
n
ter
> 2008, ou seja n > 4016. Logo dividindo o quadrado em mais de 40172
2
quadradinhos, a soma dos raios dos crculos sera maior que 2008.
Desse modo a soma dos raios desses n2 crculos e igual a n2

3. Construindo um n
umero - As condicoes dadas implicam que o n
umero
deve satisfazer:
(i) . . . 1

1 ...

(ii) . . . 2

2 ...

(iii) . . . 3

3 ...

(iv) . . . 4

4 ...

Vamos estudar as possveis posicoes dos dois algarismos 4 num n


umero de oito
dgitos. De acordo com (iv) existem apenas tres possibilidades:
caso A:

caso B:

caso C:

4
4

Em cada um desses casos, existem duas possibilidades de colocar os dois algarismos 3:


caso A:

3 4

caso B:

caso C:

3
3
4

4
4

ou
3

ou

ou

4
4

3
3

3 4

4 3
4 3
3

Na tentativa de colocar os algarismos 1 e 2 percebemos que as duas possibilidade do caso A sao impossveis tanto quanto as duas primeiras possibilidades
dos casos B e C. Os u
nicos casos que levam a solucoes do problema sao as
segundas possiblidades dos casos B e C.

78

OBMEP 2008

Lista 2

Solucoes do Nvel 3

Essas solucoes sao:


41312432
23421314

4. N
umero na circunfer
encia - A figura a seguir representa os 9 n
umeros
escritos ao redor da circunferencia.

Lendo de 3 em 3 no sentido horario, os algarismos escritos ao redor da circunferencia, obtemos os seguintes n


umeros de tres algarismos cada:
a1 a2 a3 , a2 a3 a4 , a3 a4 a5 , a4 a5 a6 , a5 a6 a7 , a6 a7 a8 , a7 a8 a9 , a8 a9 a1 e a9 a1 a2 .

Para somar esses n


umeros usamos o algoritmo da adicao como indicado a
seguir.
OBMEP 2008

79

Solucoes do Nvel 3

Lista 2

a1 a2 a3
a2 a3 a4
a3 a4 a5
a4 a5 a6
+

a5 a6 a7
a6 a7 a8
a7 a8 a9
a8 a9 a1
a9 a1 a2
???????

Analisando estes nove n


umeros notamos que todos tem os algarismos da unidade
diferentes, logo;
a3 + a4 + a5 + a6 + a7 + a8 + a9 + a1 + a2 = 1 + 2 + 3 + 4 + 5 + 6 + 7 + 8 + 9 = 45 .

Do mesmo modo, eles tambem tem todos os algarismos das dezenas e todos
os algarismos das centenas diferentes. Logo, a soma dos algarismos da dezena
e tambem 45, e o mesmo ocorre com os algarismos das centenas. Da a soma
desses n
umeros e igual a: 45 + 45 10 + 45 100 = 4995.

5. Cada pe
ca em seu lugar - A primeira informacao e certamente falsa, pois
se fosse verdadeira, o ouro estaria no Cofre 2 ou 3, mas deveria estar no Cofre
1. Absurdo. Logo o ouro nao esta nem no Cofre 2 nem no Cofre 3. A segunda
informacao nao pode estar correta, pois, caso contrario, o ouro estaria no Cofre
2, o que e incorreto. Logo, 1 e 2 sao falsas. Portanto, o ouro nao esta no Cofre
1, nem no 2 nem no 3, e a prata nao esta no Cofre 1.
80

OBMEP 2008

Lista 2

Solucoes do Nvel 3

Portanto, temos as seguintes possibilidades:


a)
, |{z}
, |{z}
, ouro
.
|{z}
|{z} , |{z}
1

Nessa possibilidade, a informacao 4 seria correta e o nquel estaria na Cofre 3.


Sendo a informacao em 3 falsa, deveramos ter o bronze tambem no cofre 3,
absurdo. Logo essa possibilidade fica descartada.

b)
, |{z}
, |{z}
, |{z}
, ouro
|{z}
|{z} .
1

Nessa possibilidade, a informacao 5 seria correta e a platina estara no Cofre


cujo n
umero e superior de 1 ao que contem o bronze. Pela afirmacao do Cofre
3, que e falsa, teramos o bronze no cofre 3, logo a platina estra no cofre 4.
Sendo a afirmacao 2 falsa, a prata nao esta no Cofre 1, so podendo estar no
Cofre 2. Portanto temos a seguinte solucao:
nquel , prata , bronze , platina , ouro .
| {z } | {z } | {z } | {z } |{z}
1

OBMEP 2008

81

Solucoes do Nvel 3

Lista 3

Lista 3
1. Soma de quadrados Solu
c
ao 1: Como a razao e 2 os n
umeros sao n 2, n e n + 2. Logo a soma
de seus quadrados e
(n 2)2 + n2 + (n + 2)2 = 3n2 + 8 = kkkk ,
onde kkkk representa o n
umero de 4 algarismos iguais.
Como kkkk = k 1111, segue que
3n2 + 8 = kkkk 3n2 = k 1111 8 k 1111 8 e m
ultiplo de 3 .

Verificamos que os valores possveis para k sao 2, 5 e 8 (e facil descartar os


valores 3, 6 e 9).
No caso k = 2, temos que
n2 =

2222 8
= 738 = 2 369,
3

e portanto, nao e um quadrado perfeito.


Se k = 5, entao
n2 =

5555 8
= 1849 = 432 .
3

Logo, os tres n
umeros procurados sao: 41, 43 e 45, e esses sao u
nicos.
De fato, no u
ltimo caso possvel, k = 8, temos que
n2 =

8888 8
= 2960 = 24 5 37,
3

e portanto, nao e um quadrado perfeito.

82

OBMEP 2008

Lista 3

Solucoes do Nvel 3

Solu
c
ao 2: Denotemos os n
umeros por n 2, n e n + 2, entao a soma de seus
quadrados e
(n 2)2 + n2 + (n + 2)2 = 3n2 + 8 = kkkk,
onde k e um n
umero menor do que ou igual a 9. Alem disso, como
3n2 = kkkk 8 = (kkk 10 + k) + (9 + 1) = (kkk 10 9) + (k + 1)
e kkk 10 9 e m
ultiplo de 3 , entao k + 1 tambem tem que ser m
ultiplo de
3. Logo, os possveis valores de k sao 2, 5 e 8.
No caso k = 2, temos que
n2 =

2222 8
= 738 = 2 369
3

e portanto, nao e um quadrado perfeito.


Se k = 5, entao
n2 =

5555 8
= 1849 = 432 .
3

Logo, os tres n
umeros procurados sao: 41, 43 e 45, e esses sao u
nicos.
De fato, no u
ltimo caso possvel, k = 8, temos que
n2 =

8888 8
= 2960 = 24 5 37,
3

e portanto, nao e um quadrado perfeito.

2. Adivinhe o n
umero - Seja x o n
umero procurado. Observe que x + 2 e
divisvel por 3, 4, 5 e 6. O menor m
ultiplo comum desses n
umeros e 60. Logo,
x + 2 = 60, e entao, x = 58.

OBMEP 2008

83

Solucoes do Nvel 3

Lista 3

3. Um c
odigo - Observe que:
AOBM EP = AOB 1000 + M EP e M EP AOB = M EP 1000 + AOB.
Denotemos AOB = m e M EP = n. Logo,
6 AOBM EP = 7 M EP AOB 6 (1000m + n) = 7 (1000n + m)
6000m 7m = 7000n 6n
5993 m = 6994 n
461 m = 538 n
Logo, 461 divide n e 538 divide m. Como AOB e M EP sao n
umeros de tres
algarismos, so podemos ter as solucoes n = 461 ou n = 822 e m = 538. A
solucao n = 822 nao serve, portanto, AOB = 538 e M EP = 461.
Logo, os algarismos sao: A = 5 , B = 8 , O = 3 , M = 4 , E = 6 e P = 1 .

4. Calculando dist
ancias - Seja E o ponto sobre a reta BD tal que o triangulo
4AEB seja retangulo no vertice E (veja figura a seguir).

84

OBMEP 2008

Lista 3

Solucoes do Nvel 3

No triangulo retangulo 4AEB temos:

EB
3
EB
3 3
cos 30 =
=
=
= EB =
AB
2
3
2
o

sin 30o =

AE
1
AE
3
= =
= AE = .
AB
2
3
2

Agora, aplicando o Teorema de Pitagoras no triangulo 4AED obtemos


!2
2

3
3
3
+
AD2 = AE 2 + ED2 = AD2 =
+ 4 = AD2 = 25 + 12 3.
2
2
q
Da, conclumos que AD =

25 + 12 3 cm.

5. Calculando lados de um tri


angulo -

Sobre o lado CB do triangulo

4ABC, construa um novo triangulo 4CBP 0 congruente ao triangulo 4ABP


\0 e ABP
[ = CBP
\0 .
tal que P[
AB = BCP

OBMEP 2008

85

Solucoes do Nvel 3

Lista 3

\
[ ou seja, mede 60o .
Note que o angulo P
BP 0 e congruente ao angulo ABC,
Assim, se tracarmos o segmento P P 0 temos que o triangulo 4P BP 0 , que e
isosceles ja que P B = BP 0 = 4cm, e equilatero e, por conseguinte, temos que
P P 0 = 4cm.

Aplicando a lei dos cossenos no triangulo 4CP P 0 ,


\
onde o angulo P
P 0 C = a, temos:
52 = 32 + 42 2.3.4.cos a 25 = 25 12.cos a
cos a = 0
a = 90o .

52 = 32 + 42 2.3.4.cos a 25 = 25 12.cos a cos a = 0 a = 90o .


0 B = a + 60o = 90o + 60o = 150o .
\
Desta forma, o angulo CP

86

OBMEP 2008

Lista 3

Solucoes do Nvel 3

Agora, aplicando a lei dos cossenos ao 4CBP 0 , onde o lado do triangulo


4ABC e l, temos:

l = 3 + 4 2.3.4.cos 150 l = 25 2.3.4.(

3
)
2

l = 25 + 12 3 l = 25 + 12 3.

Logo, o comprimento dos lados do triangulo equilatero 4ABC e


q
l=

25 + 12 3 cm.

OBMEP 2008

87

Solucoes do Nvel 3

Lista 4

Lista 4
1. Amigo Oculto - Primeiramente observemos que o n
umero de formas de
distribuir os presentes sem nenhuma restricao e 5! = 5 4 3 2 1 = 120.
Da temos que tirar os casos ruins, isto e, os casos em que exatamente uma
pessoa tirou o seu proprio presente, exatamente duas pessoas tiraram os seus
proprios presentes, etc. Assim temos os seguintes casos:
os 5 amigos ficarem com seus presentes. Nesse caso, temos somente uma
possibilidade.
exatamente 4 amigos ficarem com seus presentes. Isso nao e possvel.
exatamente 3 amigos ficarem com seu proprio presente. Nesta situacao,
os outros dois amigos trocam os presentes. Assim, temos que escolher 3
543
pessoas entre as 5, isto e,
= 10 possibilidades.
32
exatamente 2 amigos ficarem com seu proprio presente. Neste caso, temos
54
= 10. Os outros 3 amigos
que escolher 2 pessoas entre as 5, isto e,
2
trocam os presentes entre si, obtendo 10 2 = 20 possibilidades.
Por u
ltimo para que exatamente uma pessoa fique com seu presente e a
maneira de escolher essa pessoa, em um total de 5 possibilidades multiplicado pelo n
umero de formas que os outros amigos nao fiquem com seu
presente, que sao 9 maneiras, ou seja, nesta situacao temos um total de
5 9 = 45 possibilidades.
Portanto o n
umero de possibilidades para que ninguem fique com seu proprio
presente e:
120 45 20 10 1 = 44.

88

OBMEP 2008

Lista 4

Solucoes do Nvel 3

2. Contando solu
c
oes - Isolando x na equacao

xy
= 144 obtemos x =
x+y

144y
. Como x deve ser positivo, devemos ter y = 144 + n, onde n e
y 144
um n
umero inteiro positivo. Substituindo essa expressao de y no valor de
1442
x, obtemos x =
+ 144. Como x deve ser um n
umero inteiro, n deve
n
ser um divisor de 1442 . Sendo 1442 = 124 = 28 34 , segue que 1442 tem
(8 + 1) (4 + 1) = 45 divisores. Assim, para cada divisor n de 1442 , obtemos
uma solucao

(x, y) =

1442
+ 144 , 144 + n
n

xy
= 144. Assim essa equacao possui 45 pares ordenados de
x+y
n
umeros inteiros positivos (x, y) que a satisfazem.

da equacao

3. Determinando uma seq


u
encia - Sejam a1 , a2 , . . . , a80 os n
umeros desta
seq
uencia. Para cada i 1 temos

ai+1 = ai ai+2
ai+2 = ai+1 ai+3
Conseq
uentemente, ai+1 = ai ai+1 ai+3 , e como ai+1 6= 0, pois o produto dos
termos da seq
uencia e 8 6= 0, segue ai ai+3 = 1.
Logo, quaisquer dois n
umeros desta seq
uencia, cujos ndices distam tres um
do outro, sao tais que o seu produto e igual a 1. Portanto o produto de seis
n
umeros consecutivos nesta seq
uencia e sempre igual a 1.
Sendo o produto dos 40 primeiros termos da seq
uencia igual a 8, conclui-se
que o produto dos 4 primeiros termos tambem e 8, pois os 36 termos restantes
formam seis grupos de 6 termos consecutivos da seq
uencia, e em cada grupo
desse, o produto desses n
umeros e igual a 1. Isto e, a1 a2 a3 a4 = 8. Como
ai ai+3 = 1, segue a1 a4 = 1 e da a2 a3 = 8.
OBMEP 2008

89

Solucoes do Nvel 3

Lista 4

Temos tambem a hipotese de que os 80 termos da seq


uencia tem produto igual
a 8, donde podemos concluir que a1 a2 = 8 ja que os 78 u
ltimos termos podem
ser agrupados em 13 grupos de 6 termos consecutivos, cada um com produto
igual a 1, como ja vimos.
Entao, de a2 a3 = 8, a1 a2 = 8 e a2 = a1 a3 , obtemos a resposta:
a1 = 2, a2 = 4 e a3 = 2 .
Observe, mais ainda, que toda a seq
uencia esta agora determinada:
2, 4, 2,

1 1 1
1 1 1
, , , 2, 4, 2, , , , . . .
2 4 2
2 4 2

Nesta seq
uencia, os seis primeiros termos ficam se repetindo sempre na mesma
ordem.

4. Construindo uma cerca - A soma dos comprimentos dos 3 lados (os que
nao sao de pedra) e 140 m.

(a) Se os dois lados vizinhos ao muro de pedra tem 40 m cada um, os dois
juntos tem 80 m, e logo o terceiro lado tera
140 80 = 60 m .

(b) Se o maior dos lados a ser cercado tiver 85 m, ele nao pode estar encostado
no muro de pedras porque nesse caso esses dois muros mediriam
85 2 = 170 m que e maior do que 140 m. Logo ele teria que ser paralelo
ao muro de pedra, e nesse caso cada um dos outros lados mediria 27, 5 m, o
que tambem nao e possvel porque a cerca e composta de pedacos inteiros
de 1 m cada um.
90

OBMEP 2008

Lista 4

Solucoes do Nvel 3
Os dois lados que encostam no muro de pedra podem ter 65 m cada uma
pois nesse caso, o outro teria 140 2 65 = 10 m , o que nao contraria
as condicoes dadas.

5. Um quadril
atero especial B

x
A

11

y
C

Denotemos AB = x e DC = y.
Como os triangulos 4ABC e 4ACD sao retangulos e tem a mesma hipotenusa
AC, pelo teorema de Pitagoras temos:
x2 + 112 = y 2 + 72 = y 2 x2 = 72 = (y x)(y + x) = 72 = 23 32 .
Logo, y x e y + x sao divisores de 72. Para cada fatoracao temos que resolver
um sistema de duas equacoes com duas incognitas, como feito na tabela a
seguir.
Fator de 72

Medidas de

Observacoes

y+x

y-x

72

Nao ha solucao inteira

36

17

19

Possui solucao inteira

24

Nao ha solucao inteira

28

12

16

Possui solucao inteira

12

Possui solucao inteira

Nao ha solucao inteira

OBMEP 2008

91

Solucoes do Nvel 3

Lista 5

Lista 5
1. Tr
es quadrados -

B
x

y
D

x
E

\
[ Temos que
Sejam x = F
EH e y = AEB.
\
x+F
EB} +y = 180o x + y = 900 .
| {z
90o

[ = x e
Como os triangulos ABE e EF H sao retangulos, segue que ABE
\
EF
H = y. Logo, esses dois triangulos sao congruentes, pois tem os 3 angulos
iguais e um lado igual (BE = EF ). Em particular, AE = F H.
Podemos agora calcular a area do quadrado BEF G usando o Teorema de
Pitagoras:
area de BEF G = BE 2 = AB 2 + AE 2 = 302 + F H 2 = 302 + 202 = 1300.

92

OBMEP 2008

Lista 5

Solucoes do Nvel 3

2. Bolinha de gude Solu


c
ao 1: Denotemos por x, y e z o n
umero de bolinhas que cada um tinha
no incio da partida. De acordo com o enunciado temos:
1o

2o

3o

Incio

Apos a 1a rodada

xyz

2y

2z

Apos a 2a rodada

2(x y z)

2y 2z (x y z)

4z

4(x y z)

2(3y x z)

4z 2(x y z) (3y x z)

Apos a 3 rodada

Como cada um terminou a partida com 64 bolinhas, segue que:

x y z = 16
4(x

z)
=
64

x + 3y z = 32
2(3y x z) = 64

x y + 7z = 64
4z 2(x y z) (3y x z) = 64
Para resolver o sistema adicionamos a 1a e 2a equacoes, e a 1a e 3a , obtendo

y z = 24
y + 3z = 40
Da, obtemos: z = 32 e y = 56. Logo, x = 16 + 56 + 32 = 104 .

Solu
c
ao 2: Vamos preencher a tabela de de baixo para cima, isto e: do
final para o incio do jogo. Comecamos com 64 nas tres casas.
1o

2o

3o

64

64

64

Incio
Ap
os a 1a rodada
Ap
os a 2a rodada
Ap
os a 3a rodada

Como o 1o e o 2o jogadores dobraram a quantidade de bolinhas na 3a jogada,


cada um tinha 32 bolinhas, e o 3o jogador deu 32 a da um deles, logo possua
64 + 32 + 32 + 128 bolinhas.
OBMEP 2008

93

Solucoes do Nvel 3

Lista 5

1o

2o

3o

Apos a 2a rodada

32

32

128

Apos a 3a rodada

64

64

64

Incio
Apos a 1a rodada

Agora, quem perdeu a 2a jogada foi o 2o jogador, logo a tabela fica:


1o

2o

3o

Apos a 1a rodada

16

32 + 16 + 64 = 112

64

Apos a 2a rodada

32

32

128

Apos a 3a rodada

64

64

64

Incio

Finalmente,
1o

2o

3o

Incio

16 + 56 + 32 = 104

56

32

Apos a 1a rodada

16

32 + 16 + 64 = 112

64

Apos a 2a rodada

32

32

128

Apos a 3a rodada

64

64

64

3. Uma soma - Inicialmente, observe que

1
1
1
=

.
K (K + 1)
K K +1

Logo,
1
1
= 1
12
2

1
1 1
=
23
2 3

...

1
1
1
=

.
2007 2008
2007 2008

Assim, temos:
S =1
94

1
1
1
1
1 1 1 1 1
+ + + ... +

.
2 2 3 3 4
2006 2007 2007 2008
OBMEP 2008

Lista 5

Solucoes do Nvel 3

Logo, S = 1

1
2007
=
.
2008
2008

4. Dobrando papel - Sejam E e F os pontos de intersecao como mostramos


na figura. Sejam AB = 2a e BC = 2b. Entao AM = M B = DN = N C = a e
M E = EN = b. Trace AN e seja P o ponto de intersecao dos segmentos AN
e BD. Os segmentos AN e M C sao paralelos (pois AM = N C e AM k N C).
Como M e o ponto medio de AB e M F k AP , temos que F e o ponto medio
do segmento P B. Analogamente P e o ponto medio do segmento DF . Segue
entao que DP = P F = F B.
Por simetria verificamos que P E = EF e entao EF/F B = 1/2.
1
Por outro lado, a a
rea 4 M BE = a
rea 4 ABD = 125, donde a
4
125 2
1
cm , ja que 4M EF e 4M BE tem mesma altura
a
rea4M EF = 125 =
3
3
relativo ao vertice M e a base do primeiro e 1/3 da base do segundo.

5. Uma
area A

OBMEP 2008

95

Solucoes do Nvel 3

Lista 5

As alturas que passam por B dos triangulos ABC e ABM sao iguais a distancia
d de B `a reta AC, logo

AM.d
a
rea 4 ABM
AM
1
= 2 =
= segue que
AC.d
a
rea 4 ABC
AC
2
2
a
rea 4 ABM =

Analogamente,

1
1
a
rea 4 ABC = 100 = 50.
2
2

a
rea 4 ABP
BP
=
.
a
rea 4 ABM
BM

Pelo Teorema das bissetrizes,


BP
AB
10
2
3
=
=
= P M = BP.
PM
AM
15
3
2

Logo,
a
rea 4 ABP
BP
BP
BP
BP
2
=
=
=
= 5
= .
2
a
rea 4 ABM
BM
BP + BM
5
BP + 3 BP
BP
2
Assim: a
rea 4 ABP =

96

2
2
a
rea ABM = 50 = 20.
5
5

OBMEP 2008

Lista 6

Solucoes do Nvel 3

Lista 6
.............

1. Ultimos
algarismos Solu
c
ao 1:

Como so queremos

saber os dois u
ltimos algarismos,
basta conhecer as duas u
ltimas colunas dessa soma (das dezenas e das

8 ...........
8 8 ...........
8.8 8. ..........
.. .. ....... 2008
.. .. ......... parcelas
. . ......
8 . . . . . . 8 8 8 ............
8 . . . . . . 8 8 8 ..........
88 . . . . . . 8 8 8..................

................................................................................................

8
8.
..
..
.
8
8
8

8..............
8 ..........
8. ..........
.. ........ 2007
.. ........ parcelas
. .......
8 ...........
8 ..........
8................

............................................

........................................................................................................

unidades), ou seja:
8 + 88 2007 = 8 + . . . 16 .
Os u
ltimos algarismos sao 16 + 8 = 24.

Solu
c
ao 2: Observemos que os dois u
ltimos algarismos de
2008

z }| {
8 + 88 + 888 + + 88 88
sao iguais aos dois u
ltimos algarismos do n
umero
2007

z
}|
{
8 + 88 + + 88 = 8 + 2007 88,
que tambem coincide com os dois u
ltimos algarismos de 8 + 7 88 = 624, logo
o n
umero procurado e 24.

2. Idades m
ultiplas Quando Isabel tem a anos sua mae tem 20 + a. Se a e divisor de 20 + a, entao
20
20 + a
=
+ 1 e um n
umero inteiro.
a
a
OBMEP 2008

97

Solucoes do Nvel 3

Lista 6

Logo, a e divisor de 20. Portanto,


a {1, 2, 4, 5, 10, 20}.
Assim, temos um total de 6 vezes. De fato, temos:

a=1

a=2

a=4

a=5

a = 10

a = 20

Isabel

10

20

Mae

21

22

24

25

30

40

3. Blocos diferentes - O volume do cubo e 10 10 10 = 103 = 1000 cm3 .


O volume V de um bloco, e o produto de sua tres medidas:
V = largura comprimento altura.
Como para construir cada bloco Ana tem que usar todos os bloquinhos, o
volume de cada bloco sera
V = largura comprimento altura = 1000 cm3 .
Logo, precisamos saber de quantas maneiras podemos escrever 1000 como produto de 3 n
umeros naturais. Para isso, fatoramos 1000 e obtemos
1000 = 23 53 .

Soluc
ao 1: Uma maneira de encontrar esses n
umeros e listando as potencias
de 2 e 3, sem esquecer que uma das medidas pode ser 1. A tabela abaixo
mostra as 19 possibilidades para esses blocos.
98

OBMEP 2008

Lista 6

Solucoes do Nvel 3
potencia de 2

potencia de 5

23 53

23

53

22 53

22

2 53

22

53

1 , 2

1 , 1 , 1

2 53

1 , 2

23 5

52

23 52

23

52

1 , 1 , 1

23 5

1 , 2

1 , 2

25

22 52

2 52

22 5

22 52

22

25

52

22

2 52

22 5

52

1 , 2

1 , 1 ,1

25

22 5

1 , 1 , 1

1 , 2

25

2 52

1 , 1 , 1

1 , 1 ,1

25

25

25

Solu
c
ao 2: Se 1000 = l c a, com l c a, entao l3 lca 1000, isto e,
l 10. Logo, l = 1, 2, 4, 5, 8 ou 10.
Se l = 1, entao ca = 1000 = 23 53 , com 1 c a. Assim, temos 8 variacao
de c e a:
c = 1 e a = 1000 ; c = 2 e a = 500 ; c = 4 e a = 250 ; c = 5 e a = 200 ;
c = 8 e a = 125 ;
c = 10 e a = 100 ; c = 20 e a = 50 ; c = 25 e a = 40.
OBMEP 2008

99

Solucoes do Nvel 3

Lista 6

Se l = 2, entao ca = 500 = 22 53 , com 2 c a, e neste caso temos 5


blocos:
c = 2 e a = 250 ;

c = 4 e a = 125 ;

c = 10 e a = 50 ;

c = 5 e a = 100 ;

c = 20 e a = 25.

Se l = 4, entao ca = 250 = 2 53 , com 4 c a. Temos os 2 possveis casos:


c = 5 e a = 50 ;

c = 10 e a = 25.

Se l = 5, entao ca = 200 = 23 52 , com 5 c a. Temos os 3 possveis casos:


c = 5 e a = 40 ;

c = 8 e a = 25 ;

c = 10 e a = 20.

Se l = 8, entao ca = 125 = 53 , com 8 c a. Neste caso nao temos nehuma


possibilidade.
Por u
ltimo, se l = 10, entao c = a = 10, da temos apenas um bloco.
Logo, o n
umero de blocos diferentes e 8 + 5 + 2 + 3 + 1 = 19.

4. Quadro negro Inicialmente observe que de 1 a 77 Ana apagou 11 m


ultiplos de 7 e 7 m
ultiplos
de 11. Como 77 e m
ultiplo de 7 e de 11, ela entao apagou 11 + 7 1 = 17,
sobrando 77 17 = 60 n
umeros.
Agora, agrupando os 10 000 primeiros n
umeros em grupos de 77 n
umeros consecutivos, esse raciocnio se aplica em cada uma das linhas abaixo, isto e: em
cada linha sobraram 60 n
umeros.
100

OBMEP 2008

Lista 6

Solucoes do Nvel 3

1a linha

, ... ,

2a linha

78

79

, . . . , 154

77

3a linha 155 , 158 , . . . , 231


..
..
.
.
.
. , .. , . . . , ..
Como, 2008 = 33 60 + 28, sabemos que entre os primeiros 33 77 = 2541
n
umeros ficaram sem apagar 33 60 = 1980 n
umeros.

1a linha

, ... ,

77

2a linha

78

79

, ... ,

154

3a linha
..
.

157 , 158 , . . . ,
..
.
. , .. , . . . ,

231
..
.

33a linha . . . , . . . , . . . , 2541


Ainda faltam contar 28 n
umeros. Vamos, entao, examinar a 34a linha:
1a . . .

7a

...

11a

...

14a

...

21a

...

22a

...

28a

...

33a

35a

...

2542

2576

Lembre que os n
umeros apagados estao nas seguintes colunas: 7a , 11a , 14a , 21a ,
22a , 28a , 33a , 35a , etc. Ate a 35a coluna foram apagados 8 n
umeros, restando ent
ao
35 8 = 27 n
umeros na 34a linha. Logo, depois de apagados os m
ultiplos de 7 e de
11 nessa linha, o 28o n
umero e 2577. Assim, o n
umero na 2008a posic
ao e 2577.

5. Conjunto sem m
ultiplos - Inicialmente, observemos que o conjunto com 50 elementos
{51, 52, 53, . . . , 100}
satisfaz a condicao requerida. Assim o subconjunto, com mais elementos, tem no
mnimo 50 elementos.
OBMEP 2008

101

...

Solucoes do Nvel 3

Lista 6

Vamos mostrar que todo subconjunto A com um n


umero de elementos maior do que
50 possui dois n
umeros m
ultiplos. Para isto vamos dividir os n
umeros de 1 a 100
em 50 subconjuntos distintos da seguinte forma:
(n
umero mpar) 2n ; n natural .
1o subconjunto: 1 2n , A1 = {1 2n ; n N};
1 = 120 ;

2 = 12 ;

4 = 122 ;

8 = 123 ;

16 = 124 ;

32 = 125 ;

64 =

1 26 ;
2o subconjunto: 3 2n , A2 = {3 2n ; n N};
3 = 3 20 ;

6=32

12 = 3 22 ;

24 = 3 23 ;

48 = 3 24 ;

96 = 3 25 ;

3o subconjunto: 5 2n , A3 = {5 2n ; n N};
5 = 5 20 ;

10 = 5 2 ;

20 = 5 22 ;
..
.

40 = 5 23 ;

80 = 5 24 ;

50o subconjunto: 99 2n , A50 = {99 2n ; n N} = {99}.


Com isso podemos garantir que se dois elementos estao no mesmo subconjunto,
entao um e m
ultiplo do outro. Como existem apenas 50 n
umeros mpares entre 1 e
100, temos 50 subconjuntos disjuntos 2 a 2 construdos desta maneira.
Note que o conjunto {1, 2, . . . , 100} e a uniao dos 50 subconjuntos, isto e,
{1, 2, . . . , 100} = A1 A2 . . . A50 .

Com certeza, podemos afirmar que existem pelo menos dois elementos de A num
mesmo subconjunto Ai , e assim um e m
ultiplo do outro. O que nao e possvel. Logo,
o subconjunto com maior n
umero de elementos, sem m
ultiplos tem 50 elementos.

102

OBMEP 2008

Uma palavra aos alunos e professores

Uma palavra aos alunos e professores


O Banco de Questoes (BQ) foi concebido para divulgar nas escolas da rede p
ublica material
de competicoes de Matematica, nacionais ou internacionais. Por isso grande parte do conte
udo
n
ao e original, s
ao quest
oes dessas competicoes ou de preparacao para elas encontradas em
diversos sites e apostilas. Aproveitamos para agradecer a todos que mantem esses sites com
livre acesso pela grande contribuicao que d
ao a tantos alunos e professores.
Como temos feito desde 2 005, n
ao nos preocupamos com uniformidade. A cada ano o
BQ apresenta formato, quantidade e nvel de dificuldade diferentes dos anos anteriores. A
linguagem usada nas solucoes e bastante informal mas sem comprometer o rigor matem
atico.
O BQ n
ao e um livro didatico e por isso continuamos a produzi-lo de forma bastante artesanal.
Incentivamos alunos e professores a procurar solucoes diferentes das aqui apresentadas,
com certeza elas existem e podem ser mais interessantes.
Por solicitacao de muitos alunos, retomamos esse ano a sessao Desafios aonde os problemas
requerem mais paciencia, mais tempo e mais atencao. Aproveitamos para informar que temos
agora no site da OBMEP (www.obmep.org.br ) a sessao Problemas da 15na com material
muito instigante e desafiador para aqueles que gostam de quebrar a cabeca com problemas
de Matematica.
Os problemas est
ao agrupados em tres nveis conforme e feito nas provas da OBMEP, mas
muitos sao interessantes para todos os alunos.
Sugest
oes quaisquer (por exemplo, de solucoes diferentes) ou crticas serao bem recebidas
no email: contato@obmep.org.br
Desejamos que esse Banco de Questoes proporcione a todos bons momentos de reflex
ao e
descobertas.
Direcao Academica da OBMEP

OBMEP 2009

5
9

Uma palavra aos alunos e professores

Organizado por:
Suely Druck (UFF)
Maria Elasir Seabra Gomes (UFMG)
Com a colaboracao de:
Ana Catarina P. Hellmeister (USP/SP)
Fabio Brochero (UFMG)
Francisco Dutenhefner (UFMG)

Texto j
a revisado pela nova ortografia.

ii

OBMEP 2009

Conte
udo
Uma palavra aos alunos e professores

Nvel 1
Lista
Lista
Lista
Lista
Lista
Lista
Lista
Lista
Lista
Lista

1 .
2 .
3 .
4 .
5 .
6 .
7 .
8 .
9 .
10

.
.
.
.
.
.
.
.
.
.

.
.
.
.
.
.
.
.
.
.

.
.
.
.
.
.
.
.
.
.

.
.
.
.
.
.
.
.
.
.

.
.
.
.
.
.
.
.
.
.

.
.
.
.
.
.
.
.
.
.

.
.
.
.
.
.
.
.
.
.

.
.
.
.
.
.
.
.
.
.

.
.
.
.
.
.
.
.
.
.

.
.
.
.
.
.
.
.
.
.

.
.
.
.
.
.
.
.
.
.

.
.
.
.
.
.
.
.
.
.

.
.
.
.
.
.
.
.
.
.

.
.
.
.
.
.
.
.
.
.

.
.
.
.
.
.
.
.
.
.

.
.
.
.
.
.
.
.
.
.

.
.
.
.
.
.
.
.
.
.

.
.
.
.
.
.
.
.
.
.

.
.
.
.
.
.
.
.
.
.

.
.
.
.
.
.
.
.
.
.

.
.
.
.
.
.
.
.
.
.

.
.
.
.
.
.
.
.
.
.

.
.
.
.
.
.
.
.
.
.

.
.
.
.
.
.
.
.
.
.

.
.
.
.
.
.
.
.
.
.

.
.
.
.
.
.
.
.
.
.

.
.
.
.
.
.
.
.
.
.

.
.
.
.
.
.
.
.
.
.

.
.
.
.
.
.
.
.
.
.

.
.
.
.
.
.
.
.
.
.

.
.
.
.
.
.
.
.
.
.

.
.
.
.
.
.
.
.
.
.

.
.
.
.
.
.
.
.
.
.

.
.
.
.
.
.
.
.
.
.

.
.
.
.
.
.
.
.
.
.

.
.
.
.
.
.
.
.
.
.

.
.
.
.
.
.
.
.
.
.

.
.
.
.
.
.
.
.
.
.

.
.
.
.
.
.
.
.
.
.

.
.
.
.
.
.
.
.
.
.

1
. 1
. 2
. 3
. 4
. 5
. 6
. 7
. 8
. 9
. 10

Nvel 2
Lista
Lista
Lista
Lista
Lista
Lista
Lista
Lista
Lista
Lista

1 .
2 .
3 .
4 .
5 .
6 .
7 .
8 .
9 .
10

.
.
.
.
.
.
.
.
.
.

.
.
.
.
.
.
.
.
.
.

.
.
.
.
.
.
.
.
.
.

.
.
.
.
.
.
.
.
.
.

.
.
.
.
.
.
.
.
.
.

.
.
.
.
.
.
.
.
.
.

.
.
.
.
.
.
.
.
.
.

.
.
.
.
.
.
.
.
.
.

.
.
.
.
.
.
.
.
.
.

.
.
.
.
.
.
.
.
.
.

.
.
.
.
.
.
.
.
.
.

.
.
.
.
.
.
.
.
.
.

.
.
.
.
.
.
.
.
.
.

.
.
.
.
.
.
.
.
.
.

.
.
.
.
.
.
.
.
.
.

.
.
.
.
.
.
.
.
.
.

.
.
.
.
.
.
.
.
.
.

.
.
.
.
.
.
.
.
.
.

.
.
.
.
.
.
.
.
.
.

.
.
.
.
.
.
.
.
.
.

.
.
.
.
.
.
.
.
.
.

.
.
.
.
.
.
.
.
.
.

.
.
.
.
.
.
.
.
.
.

.
.
.
.
.
.
.
.
.
.

.
.
.
.
.
.
.
.
.
.

.
.
.
.
.
.
.
.
.
.

.
.
.
.
.
.
.
.
.
.

.
.
.
.
.
.
.
.
.
.

.
.
.
.
.
.
.
.
.
.

.
.
.
.
.
.
.
.
.
.

.
.
.
.
.
.
.
.
.
.

.
.
.
.
.
.
.
.
.
.

.
.
.
.
.
.
.
.
.
.

.
.
.
.
.
.
.
.
.
.

.
.
.
.
.
.
.
.
.
.

.
.
.
.
.
.
.
.
.
.

.
.
.
.
.
.
.
.
.
.

.
.
.
.
.
.
.
.
.
.

.
.
.
.
.
.
.
.
.
.

.
.
.
.
.
.
.
.
.
.

.
.
.
.
.
.
.
.
.
.

11
11
12
13
14
15
16
17
18
19
20

Nvel 3
Lista
Lista
Lista
Lista
Lista
Lista
Lista
Lista
Lista

1
2
3
4
5
6
7
8
9

.
.
.
.
.
.
.
.
.

.
.
.
.
.
.
.
.
.

.
.
.
.
.
.
.
.
.

.
.
.
.
.
.
.
.
.

.
.
.
.
.
.
.
.
.

.
.
.
.
.
.
.
.
.

.
.
.
.
.
.
.
.
.

.
.
.
.
.
.
.
.
.

.
.
.
.
.
.
.
.
.

.
.
.
.
.
.
.
.
.

.
.
.
.
.
.
.
.
.

.
.
.
.
.
.
.
.
.

.
.
.
.
.
.
.
.
.

.
.
.
.
.
.
.
.
.

.
.
.
.
.
.
.
.
.

.
.
.
.
.
.
.
.
.

.
.
.
.
.
.
.
.
.

.
.
.
.
.
.
.
.
.

.
.
.
.
.
.
.
.
.

.
.
.
.
.
.
.
.
.

.
.
.
.
.
.
.
.
.

.
.
.
.
.
.
.
.
.

.
.
.
.
.
.
.
.
.

.
.
.
.
.
.
.
.
.

.
.
.
.
.
.
.
.
.

.
.
.
.
.
.
.
.
.

.
.
.
.
.
.
.
.
.

.
.
.
.
.
.
.
.
.

.
.
.
.
.
.
.
.
.

.
.
.
.
.
.
.
.
.

.
.
.
.
.
.
.
.
.

.
.
.
.
.
.
.
.
.

.
.
.
.
.
.
.
.
.

.
.
.
.
.
.
.
.
.

.
.
.
.
.
.
.
.
.

.
.
.
.
.
.
.
.
.

.
.
.
.
.
.
.
.
.

.
.
.
.
.
.
.
.
.

.
.
.
.
.
.
.
.
.

.
.
.
.
.
.
.
.
.

.
.
.
.
.
.
.
.
.

21
21
22
23
25
26
27
28
29
30

.
.
.
.
.
.
.
.
.

iii

5
9

Uma palavra aos alunos e professores

Lista 10 . . . . . . . . . . . . . . . . . . . . . . . . . . . . . . . . . . . . . . . . . 31
Desafios

32

Soluco
es do Nvel 1
Lista 1 . . . . . .
Lista 2 . . . . . .
Lista 3 . . . . . .
Lista 4 . . . . . .
Lista 5 . . . . . .
Lista 6 . . . . . .
Lista 7 . . . . . .
Lista 8 . . . . . .
Lista 9 . . . . . .
Lista 10 . . . . .

.
.
.
.
.
.
.
.
.
.

.
.
.
.
.
.
.
.
.
.

.
.
.
.
.
.
.
.
.
.

.
.
.
.
.
.
.
.
.
.

.
.
.
.
.
.
.
.
.
.

.
.
.
.
.
.
.
.
.
.

.
.
.
.
.
.
.
.
.
.

.
.
.
.
.
.
.
.
.
.

.
.
.
.
.
.
.
.
.
.

.
.
.
.
.
.
.
.
.
.

.
.
.
.
.
.
.
.
.
.

.
.
.
.
.
.
.
.
.
.

.
.
.
.
.
.
.
.
.
.

.
.
.
.
.
.
.
.
.
.

.
.
.
.
.
.
.
.
.
.

.
.
.
.
.
.
.
.
.
.

.
.
.
.
.
.
.
.
.
.

.
.
.
.
.
.
.
.
.
.

.
.
.
.
.
.
.
.
.
.

.
.
.
.
.
.
.
.
.
.

.
.
.
.
.
.
.
.
.
.

.
.
.
.
.
.
.
.
.
.

.
.
.
.
.
.
.
.
.
.

.
.
.
.
.
.
.
.
.
.

.
.
.
.
.
.
.
.
.
.

.
.
.
.
.
.
.
.
.
.

.
.
.
.
.
.
.
.
.
.

.
.
.
.
.
.
.
.
.
.

.
.
.
.
.
.
.
.
.
.

.
.
.
.
.
.
.
.
.
.

.
.
.
.
.
.
.
.
.
.

.
.
.
.
.
.
.
.
.
.

.
.
.
.
.
.
.
.
.
.

.
.
.
.
.
.
.
.
.
.

.
.
.
.
.
.
.
.
.
.

.
.
.
.
.
.
.
.
.
.

35
35
38
40
43
46
49
53
56
58
60

Soluco
es do Nvel 2
Lista 1 . . . . . .
Lista 2 . . . . . .
Lista 3 . . . . . .
Lista 4 . . . . . .
Lista 5 . . . . . .
Lista 6 . . . . . .
Lista 7 . . . . . .
Lista 8 . . . . . .
Lista 9 . . . . . .
Lista 10 . . . . .

.
.
.
.
.
.
.
.
.
.

.
.
.
.
.
.
.
.
.
.

.
.
.
.
.
.
.
.
.
.

.
.
.
.
.
.
.
.
.
.

.
.
.
.
.
.
.
.
.
.

.
.
.
.
.
.
.
.
.
.

.
.
.
.
.
.
.
.
.
.

.
.
.
.
.
.
.
.
.
.

.
.
.
.
.
.
.
.
.
.

.
.
.
.
.
.
.
.
.
.

.
.
.
.
.
.
.
.
.
.

.
.
.
.
.
.
.
.
.
.

.
.
.
.
.
.
.
.
.
.

.
.
.
.
.
.
.
.
.
.

.
.
.
.
.
.
.
.
.
.

.
.
.
.
.
.
.
.
.
.

.
.
.
.
.
.
.
.
.
.

.
.
.
.
.
.
.
.
.
.

.
.
.
.
.
.
.
.
.
.

.
.
.
.
.
.
.
.
.
.

.
.
.
.
.
.
.
.
.
.

.
.
.
.
.
.
.
.
.
.

.
.
.
.
.
.
.
.
.
.

.
.
.
.
.
.
.
.
.
.

.
.
.
.
.
.
.
.
.
.

.
.
.
.
.
.
.
.
.
.

.
.
.
.
.
.
.
.
.
.

.
.
.
.
.
.
.
.
.
.

.
.
.
.
.
.
.
.
.
.

.
.
.
.
.
.
.
.
.
.

.
.
.
.
.
.
.
.
.
.

.
.
.
.
.
.
.
.
.
.

.
.
.
.
.
.
.
.
.
.

.
.
.
.
.
.
.
.
.
.

.
.
.
.
.
.
.
.
.
.

.
.
.
.
.
.
.
.
.
.

62
62
64
67
70
73
75
79
82
85
87

Soluco
es do Nvel 3
Lista 1 . . . . . .
Lista 2 . . . . . .
Lista 3 . . . . . .
Lista 4 . . . . . .
Lista 5 . . . . . .
Lista 6 . . . . . .
Lista 7 . . . . . .
Lista 8 . . . . . .
Lista 9 . . . . . .
Lista 10 . . . . .

.
.
.
.
.
.
.
.
.
.

.
.
.
.
.
.
.
.
.
.

.
.
.
.
.
.
.
.
.
.

.
.
.
.
.
.
.
.
.
.

.
.
.
.
.
.
.
.
.
.

.
.
.
.
.
.
.
.
.
.

.
.
.
.
.
.
.
.
.
.

.
.
.
.
.
.
.
.
.
.

.
.
.
.
.
.
.
.
.
.

.
.
.
.
.
.
.
.
.
.

.
.
.
.
.
.
.
.
.
.

.
.
.
.
.
.
.
.
.
.

.
.
.
.
.
.
.
.
.
.

.
.
.
.
.
.
.
.
.
.

.
.
.
.
.
.
.
.
.
.

.
.
.
.
.
.
.
.
.
.

.
.
.
.
.
.
.
.
.
.

.
.
.
.
.
.
.
.
.
.

.
.
.
.
.
.
.
.
.
.

.
.
.
.
.
.
.
.
.
.

.
.
.
.
.
.
.
.
.
.

.
.
.
.
.
.
.
.
.
.

.
.
.
.
.
.
.
.
.
.

.
.
.
.
.
.
.
.
.
.

.
.
.
.
.
.
.
.
.
.

.
.
.
.
.
.
.
.
.
.

.
.
.
.
.
.
.
.
.
.

.
.
.
.
.
.
.
.
.
.

.
.
.
.
.
.
.
.
.
.

.
.
.
.
.
.
.
.
.
.

.
.
.
.
.
.
.
.
.
.

.
.
.
.
.
.
.
.
.
.

.
.
.
.
.
.
.
.
.
.

.
.
.
.
.
.
.
.
.
.

.
.
.
.
.
.
.
.
.
.

.
.
.
.
.
.
.
.
.
.

89
89
92
94
97
101
104
107
111
115
119

Soluco
es dos Desafios

iv

121

OBMEP 2009

Lista 1

Nvel 1

Nvel 1
Lista 1
1. Encontro de amigos Embora eu esteja certo de que meu rel
ogio esta adiantado 5
minutos, ele est
a, na realidade, com 10 minutos de atraso. Por outro lado, o rel
ogio do
meu amigo esta realmente 5 minutos adiantado, embora ele pense que esta correto. Nos
marcamos um encontro a`s 10 horas e planejamos chegar pontualmente. Quem chegar
a
em primeiro lugar? Depois de quanto tempo chegar
a o outro?
2. Trabalho comunit
ario Uma classe tem 22 alunos e 18 alunas. Durante as ferias, 60%
dos alunos dessa classe foram prestar trabalho comunit
ario. No mnimo, quantas alunas
participaram desse trabalho?
(A) 1

(B) 2

(C) 4

(D) 6

(E) 8

3. Area
de trap
ezios Unindo quatro trapezios
iguais de bases 30 cm e 50 cm e lados nao paralelos iguais, como o da figura, podemos formar
um quadrado de area 2 500 cm2 , com um buraco
quadrado no meio. Qual e a area de cada trapezio,
em cm2 ?
(A) 200

(B) 250

(C) 300

(D) 350

(E) 400

4. Adivinhac
ao Pensei em 2 n
umeros de dois algarismos, que n
ao possuem algarismos
em comum, sendo um o dobro do outro. Alem disso, os algarismos do menor n
umero
sao a soma e a diferenca dos algarismos do maior n
umero. Quais s
ao os n
umeros?
5. 18 n
umeros consecutivos Escreva 18 n
umeros consecutivos de 3 algarismos e verifique que um deles e divisvel pela soma de seus algarismos.
Isso e sempre verdade. Ou seja: se voce escrever 18 n
umeros consecutivos de 3 algarismos, ent
ao um deles e divisvel pela soma de seus algarismos. Mostre este fato.

OBMEP 2009

5
9

Nvel 1

Lista 2

Lista 2
1. Completar uma tabela Descubra a regra utilizada para as casas j
a preenchidas e
complete a tabela. Qual e o valor de A?
0
1
2
3
4

1
2

2
5

3
10

2. Procurando m
ultiplos de 9 Consideremos um conjunto formado por 10 n
umeros
naturais diferentes. Se calculamos todas as diferencas entre esses n
umeros, pelo menos
uma dessas diferencas e um m
ultiplo de 9?
3. Correndo numa praca Um atleta costuma correr
15, 5 km ao redor de uma praca retangular de dimensoes
900 m 600 m. Ele inicia a corrida sempre do ponto P
situado a 550 m de um dos vertices correndo no sentido
hor
ario, como mostra a figura. Em que ponto da praca
ele para?
4. Ovos para um bolo Uma doceira foi ao mercado comprar ovos para fazer 43 bolos,
todos com a mesma receita, que gasta menos de 9 ovos. O vendedor repara que se
tentar embrulhar os ovos que a doceira comprou em grupos de 2 ou de 3 ou de 4 ou de
5 ou de 6 ovos, sempre sobra 1 ovo. Quantos ovos ela usa em cada bolo? Qual o menor
n
umero de ovos que a doceira vai gastar para fazer os 43 bolos?
5. C
alculos H e V Voce consegue colocar os n
umeros de 1 a 8
dentro dos crculos, sem repeti-los, de modo que os calculos na
horizontal e na vertical sejam corretos?
Dica: Quais as possibilidades para a multiplicacao? Quais os
possveis lugares para o n
umero 1?

OBMEP 2009

m m= m
m

....................................

....................................

m+ m= m

Lista 3

Nvel 1

Lista 3
1. Cortando uma cartolina Uma folha retangular de cartolina foi cortada ao longo de
sua diagonal. Num dos pedacos obtidos, foram feitos 2 cortes paralelos aos 2 lados
menores e pelos pontos medios desses lados. Ao final sobrou um ret
angulo de permetro
129 cm. O desenho abaixo indica a sequencia de cortes.
..........................................................................................................................
..
..
...
....
...
..
.
.
.....
.....
..
..
...
...
...
...
...
...
...
...
..
...
.......................................................................................................................

...
...... ...
...... ...
...
......
......
.
.
...
.
.
.
..
...
......
......
.
.
.
.
....
.
...
.
.
.
.
...
.
.....
.
.
.
...
.
.
....
.
.
...
.
.
.
....
.
.
...
.
.
.
....
.
...
.
.
.
.
..
............................................................................................................................

................................................................
...
....
....
....
...
..
...
..
...............................................................

Qual era o permetro da folha antes do corte?


2. A soma errada A soma ao lado est
a incorreta. Para corrigi-la basta
substituir um certo algarismo em todos os lugares que ele aparece na conta
por um outro algarismo. Quais sao esses dois algarismos?

742586
+829430
1212016

3. N
umero de 5 algarismos Os algarismos 1, 2, 3, 4 e 5 foram usados, cada um uma
u
nica vez, para escrever um n
umero de 5 algarismos a b c d e, tal que: a b c e divisvel
por 4, b c d por 5, e c d e por 3. Encontre esse n
umero.
4. Tabela misteriosa Complete a tabela 6 6 de modo
que em cada linha e cada coluna aparecam apenas
m
ultiplos de um dos n
umeros:
2, 3, 4, 5, 6, 7, 8, 9, 10, 11, 12 .

32

40
49
22
15
24
42

Voce pode repetir apenas um n


umero na tabela.

5. Habitantes e esporte Numa cidade com quase 30 mil habitantes, dois nonos dos
homens e dois quinze avos das mulheres pratica esporte somente nos finais de semana, e
o n
umero de habitantes que n
ao pratica esporte e o quntuplo dos que praticam esporte
regularmente. Com esses dados, complete a tabela.
Nao praticam esporte
fem.
8 563

masc.
8 322

Praticam esporte somente


nos finais de semana
fem.
masc.

OBMEP 2009

Praticam esporte
regularmente
fem.
masc.
1 252

Populacao
total

5
9

Nvel 1

Lista 4

Lista 4
1. Bot
oes luminosos No mecanismo luminoso da figura,
cada um dos oito bot
oes pode acender as cores verde ou
azul. O mecanismo funciona do seguinte modo: ao ser
ligado, todos os bot
oes acendem a luz azul, e se apertamos um botao, esse botao e seus vizinhos trocam de cor.
Se ligarmos o mecanismo e apertarmos sucessivamente os
bot
oes 1, 3 e 5, qual sera o n
umero de luzes verdes que
estarao acesas no final?
(A) 3

(B) 4

(C) 5

(D) 6

1
2

7
4

6
5

(E) 7

2. Qual
e o n
umero? Um n
umero de 6 algarismos comeca por 1. Se deslocamos esse
algarismo 1 da primeira posicao para a u
ltima `a direita, obtemos um novo n
umero de 6
algarismos que e o triplo do n
umero de partida. Qual e esse n
umero?
3. Jardim variado Um jardim retangular de 120 m por 80 m foi dividido em 6 regioes
como na figura, onde N, M e P sao pontos medios dos lados, e R divide o comprimento
na raz
ao 1/3. Em cada regi
ao sera plantado um dos seguintes tipos de flor: rosa,
margarida, cravo, bem-me-quer, violeta e bromelia, cujos precos, por m2 estao indicados
na tabela. Quais as possveis escolhas das flores em cada regiao, de modo a gastar o
mnimo possvel?
Tipo
rosa
margarida
cravo
bem-me-quer
violeta
bromelia

Preco por m2
3,50
1,20
2,20
0,80
1,70
3,00

4. O algarismo 3 Luis escreveu a sequencia de n


umeros naturais a partir de 1:
1, 2, 3, 4, 5, 6, 7, 8, 9, 10, 11, 12, .
Quando ele escreveu o algarismo 3 pela 25a vez?
5. Soma de pot
encias O n
umero 3444 + 4333 e divisvel por 5?

OBMEP 2009

Lista 5

Nvel 1

Lista 5
1. Telefonemas Joao mora em Salvador e seus pais em Recife. Para matar a saudade,
ele telefona para seus pais a cada tres dias. O primeiro telefonema foi feito no domingo,
o segundo telefonema na 4a feira, o terceiro telefonema no sabado, e assim por diante.
Em qual dia da semana Jo
ao telefonou para seus pais pela centesima vez?
2. O maior produto Com os algarismos de 1 a 5 e um sinal
de multiplicacao Clara forma o produto de 2 n
umeros, com
o sinal entre eles. Como Clara deve colocar os cartoes para
obter o maior produto possvel?
3. O caminho da Joaninha Dona Joaninha quer atravessar um p
atio com azulejos quadrados numerados como
mostra a figura. Ela vai partir do ponto P e quer chegar
ao ponto C andando somente sobre os lados dos azulejos. Dona Joaninha n
ao quer ter n
umeros primos `a sua
direita ao longo de todo o percurso. Qual e o menor
percurso que ela pode fazer?

Eu
Eu
Eu
Eu
Eu
Eu
Eu

4. O lugar dos amigos Sete amigos tracaram um tri


angulo, um
quadrado e um crculo. Cada um marcou seu lugar com um n
umero:
Ana:
Bento:
Celina:
Diana:
Elisa:
Fabio:
Guilherme:

nao falarei nada.


estou dentro de uma u
nica figura.
estou dentro das tres figuras.
estou dentro do triangulo mas nao do quadrado.
estou dentro do triangulo e do crculo.
nao estou dentro de um polgono.
estou dentro do crculo.

C
............................
........
.....
....
....
....
.
.........................................................
....
..
...
...
...
..
.
...
... ........
...
..
..... ....
..
...
.
.... ....
...
.
.
.
...
.
.... ..
.. ...
...
.
.
...
.
.
......
.... .... ..
.
.
...
..... ... ..
.
.......... ... ........... ......
...
. .....................
....
.
.
.
......................................................
...
.
.
.
....
...
.
.
.
............................................................................

4
5

Encontre o lugar de cada um.


5. Quadrado perfeito? Cada um dos cinco n
umeros abaixo tem 100 algarismos, e e
formado pela repeticao de um ou dois algarismos:
N1 = 333333 . . . 3
N2 = 666666 . . . 6
N3 = 151515 . . . 15
N4 = 212121 . . . 21
N5 = 272727 . . . 27
Algum destes n
umeros e um quadrado perfeito?

OBMEP 2009

5
9

Nvel 1

Lista 6

Lista 6
1. Preenchendo quadradinhos Complete os quadradinhos com os n
umeros 1, 2, 3, 5, 6.


= 4

2. Os 3 n
umeros Sofia brinca de escrever todos os n
umeros de 4 algarismos diferentes
que se pode escrever com os algarismos 1, 2, 4 e 7. Ela soma 3 desses n
umeros todos
diferentes e obtem 13 983. Quais s
ao esses 3 n
umeros?
3. Preencher uma tabela Jandira deve preencher uma tabela
4 4 que j
a vem com duas casas preenchidas com os n
umeros
1 e 2 - veja ao lado. Duas casas s
ao consideradas vizinhas se
tem um vertice ou um lado em comum.
As regras que ela tem que obedecer sao:

uma casa s
o pode ser preenchida se alguma de suas casas vizinhas j
a contem um
n
umero;
ao preencher uma casa, deve-se colocar a soma de todos os n
umeros que ja constam
em suas casas vizinhas.
Qual e o maior n
umero que e possvel escrever na tabela?
4. Olimpada de Pequim Na Olimpada de Pequim sentaram-se, em uma mesa quadrada,
as mulheres, Maria e Tania, e os homens, Juan e David, todos atletas. Cada um deles
pratica um esporte diferente: natacao, v
olei, gin
astica e atletismo. Eles estavam sentados
da seguinte maneira:
(a) Quem pratica a natacao estava a` esquerda de Maria.
(b) Quem pratica ginastica estava em frente a Juan.
(c) T
ania e David sentaram-se lado a lado.
(d) Uma mulher sentou-se ao lado de quem pratica volei.
Qual dos atletas pratica atletismo?
5. Culturas diferentes Jorge, que mora em Recife, se corresponde com seu amigo ingles
Ralph que mora na Inglaterra. Os dois se compreendem muito bem nas duas lnguas,
mas tem um problema com as datas: a data 08/10 no Brasil significa 8 de outubro, e na
Inglaterra 10 de agosto. Por causa disso, os dois combinaram n
ao se escrever nos dias
em que a data for ambgua. Eles preferem datas como 25/03 que so pode significar 25
de marco.
(a) Em quais das datas a seguir Jorge e Ralph n
ao podem se escrever?
(i) 3 de dezembro

(ii) 18 de agosto

(iii) 5 de maio

(b) Quando ocorre o maior perodo em que os dois amigos nao podem se escrever?

OBMEP 2009

Lista 7

Nvel 1

Lista 7
1. Uma liquidac
ao Na liquidacao da loja SUPER-SUPER todos os produtos est
ao 50%
mais baratos, e aos s
abados existe ainda um desconto adicional de 20%. Carla comprou uma calca antes da liquidacao, e agora ela se lamenta: Nesse sabado eu teria
economizado R$ 50,40 na calca. Qual era o preco da calca antes da liquidacao?
oes
2. N
umero com muitos zeros Se a e o n
umero 0, 000 .
. . 000 1, entao qual das express
2009 zeros

a seguir representa o maior n


umero?
(A)

3+a

(B)

3a

(C)

3a

(D)

3/a

(E)

a/3

3. Corrida das tartarugas Cinco tartarugas apostaram uma corrida em linha reta e na
chegada a situacao foi a seguinte: Sininha est
a 10 m atr
as de Olguinha e 25 m a` frente
de Rosinha que est
a 5 m atr
as de Elzinha que est
a 25 m atr
as de Pulinha. Qual foi a
ordem de chegada?
4. Que mem
oria... Esquecinaldo tem pessima memoria para guardar n
umeros, mas
otima para lembrar sequencias de operacoes. Por isso, para lembrar do seu c
odigo
bancario de 5 algarismos, ele consegue se lembrar que nenhum dos algarismos e zero,
os dois primeiros algarismos formam uma potencia de 5, os dois u
ltimos formam uma
potencia de 2, o do meio e um m
ultiplo de 3 e a soma de todos os algarismos e um
n
umero mpar. Agora ele n
ao precisa mais decorar o n
umero porque ele sabe que e o
maior n
umero que satisfaz essas condicoes e que nao tem algarismos repetidos. Qual e
esse codigo?
5. Uma frac
ao irredutvel Encontre uma fracao irredutvel tal que o produto de seu
numerador pelo denominador seja 2 3 4 5 . . . 10. Quantas dessas fracoes
irredutveis existem?

OBMEP 2009

5
9

Nvel 1

Lista 8

Lista 8
1. Transformar em decimal Escreva o resultado das seguintes expressoes na forma
decimal:


2
5
5
2
(b) 5 2
(c) 1 +
(a) 7 + 16
3
3

12

1+

1+4

2. Uma sequ
encia especial Escrevendo sucessivamente os n
umeros naturais, obtemos
a sequencia:
1 2 3 4 5 6 7 8 9 10 11 12 13 14 15 16 17 18 19 20 21 22 . . .
Qual algarismo est
a na 2 009a posicao dessa sequencia?
3. Cortar um ret
angulo Como cortar um retangulo de 13 cm por 7 cm em 13 retangulos
diferentes?

e B OY

sao angulos retos e a medida de DOY

4. Medida de
angulo Na figura, AOD

esta entre 40 e 50 . Alem disso, os pontos C e Y estao sobre a reta r, enquanto

variam de:
D e E estao sobre a reta s. Os possveis valores para a medida de AOC
(A)
(B)
(C)
(D)
(E)

30 a 40
40 a 50
50 a 60
40 a 60
n
ao podem ser determinados

+ 3 cm de lado, e as dimensoes de um
5. Permetros e
areas Um quadrado
tem
3

retangulo, em centmetros, sao 72 + 3 6 e 2. Qual dos dois tem maior area? E


maior permetro?

6. C
alculo de
angulo Encontre B AD,
sabendo

= 39 , AB = AC e AD = BD.
que DAC

OBMEP 2009

Lista 9

Nvel 1

Lista 9
1. O caminho da formiga Uma formiga sai de um ponto A, anda 7 cm para a esquerda,
5 cm para cima, 3 cm para a direita, 2 cm para baixo, 9 cm para a direita, 2 cm para
baixo, 1 cm para a esquerda e 1 cm para baixo, chegando no ponto B. Qual e a distancia
d entre A e B?
(A) 0 cm

(B) 1 cm

(C)

4 cm

(D) 5 cm

(E) 7 cm

2. Menino mentiroso Joaozinho mente nas tercas-feiras, quintas-feiras e s


abados e o
resto dos dias fala a verdade. Um dia Pedrinho encontra com Joaozinho e tem o seguinte
di
alogo:
Pedrinho pergunta: Que dia e hoje?
Joaozinho responde: S
abado.
Pedrinho pergunta: E que dia ser
a amanh
a?
Joaozinho responde: Quarta-feira.
Que dia da semana o Pedrinho encontrou com o Jo
aozinho?
3. Encontre os 4 n
umeros Encontre quatro n
umeros distintos de 3 algarismos, tais que
a soma de tres quaisquer deles e divisvel pelo quarto n
umero.
4. Colando 6 tri
angulos Construa 6 tri
angulos
equil
ateros, o primeiro com lado de comprimento 1 cm e
os triangulos seguintes com lado igual a metade do lado
do tri
angulo anterior, como indicado na figura ao lado.
Qual e o permetro desta figura?
5. Os livros da Elisa Elisa tem 24 livros de ciencias e outros de matematica e litera1
de seus livros seria de
tura. Se Elisa tivesse um livro a mais de matematica, ent
ao
9
matematica e um quarto de literatura. Se Elisa tem menos que 100 livros, quantos livros
de matematica ela possui?

OBMEP 2009

5
9

Nvel 1

Lista 10

Lista 10
1. Divis
ao por 9
(a) Listemos os primeiros 20 092 009 n
umeros naturais. Em seguida, substitumos,
sucessivamente, cada n
umero pela soma dos seus algarismos, ate obtermos uma
lista de n
umeros com apenas um algarismo. A lista tem mais algarismos 4 ou 5?
Quantos 9 tem a lista?
umero pela
(b) Aplicando o mesmo processo ao n
umero 32 009 , isto e, substituindo o n
soma dos seus algarismos, qual e o n
umero de apenas um algarismo obtido?
(c) E para o n
umero 172 009 ?
2. Uma brincadeira na sala de aula A professora Raquel inventou a seguinte brincadeira:
escreva um n
umero no quadro, se ele for mpar acrescente 3 unidades ao n
umero, e se
ele for par divida o n
umero por 2.
Esta operacao pode ser feita diversas vezes. A professora est
a interessada em obter no
final o n
umero 1 e perguntou para a classe: Como obter o n
umero 1 ap
os 3 operacoes?
E ap
os 4 operacoes? E apos 5 operacoes?
3. Calcule a idade Laura e sua av
o Ana acabaram de descobrir que, no ano passado,
suas idades eram divisveis por 8 e, no pr
oximo ano, ser
ao divisveis por 7. Vov
o Ana
ainda n
ao e centenaria. Qual e a idade de Laura?
4. Divis
oes e restos O dobro de um n
umero dividido por 5 deixa resto 1. Qual o resto
da divis
ao desse n
umero por 5?
5. Preenchendo o crculo Cada um dos sinais , , ,  e  representa um n
umero
de 1 algarismo. Descubra quem s
ao eles e complete o n
umero que falta no crculo em
branco.
 


/

 


47 423 282

 

10

+ 

1448

 

OBMEP 2009




Lista 1

Nvel 2

Nvel 2
Lista 1
1. Vista ruim Numa classe, 40% dos alunos n
ao enxergam bem. Desses, 70% usam
oculos e os 30% restantes usam lentes de contato. Sabendo que 21 alunos usam oculos,
quantos alunos tem essa classe?
2. Idade m
edia da populac
ao de Campo Verde A raz
ao entre o n
umero de homens
2
e o de mulheres na cidade de Campo Verde e . A idade media dos homens e 37 anos
3
e a das mulheres e 42 anos. Qual e a idade media dos habitantes de Campo Verde?
B

3. Area
de tri
angulo Se AC = 1,5 cm e AD = 4 cm,
qual e a relacao entre as areas dos tri
angulos ABC
e DBC?

..............
............... ....
.......... .....
...
........... .........
.
...
.
.
.
.
.
.
.
.
..
...
.....
...........
...
.....
...........
.
.
.
.
.
.
.
.
...
.
.
.
.
.
...
.......
.
.
.
...
.
.
.
.
.
.
.
.
.
.
.
...
......
...
.
.
.
.
.
.
.
.
.
.
.
.
.
.
...
..
.......
.
.
.
.
.
.
.
.
.
...
.
.
.
.
.
.
..
.......
.
.
.
...
.
.
.
.
.
.
.
.
.
.
.
...
..
.......
.
.
.
.
.
.
.
.
.
.
.
.
.
.
.
.
..
.....................................................................................................................................................................................................................

4. Construindo quadrados perfeitos Observe as seguintes igualdades:

1234+1 =
25 = 52

2345+1 =
121 = 112

..
.

10

11

12

13
+
1
= 17 161 = 1312

..

.
Ser
a que isso e sempre verdadeiro? Isto e: o produto de quatro n
umeros inteiros
consecutivos, mais 1, e sempre um quadrado perfeito?
5. Feira de Ci
encias Na Feira de Ciencias
de uma escola, observou-se que metade dos
alunos do ensino fundamental e um quarto
dos alunos do ensino medio presentes nesse
evento compraram um adesivo cada.

FEIRA DE CIENCIAS
Preco dos Adesivos (unidade)
R$ 0,30 alunos do ensino fundamental
R$ 0,50
alunos do ensino medio

Notou-se tambem que o n


umero de alunos do ensino medio presentes que nao compraram
adesivos foi o dobro do n
umero de alunos do ensino fundamental que n
ao compraram
adesivos. Sabendo que arrecadou-se R$ 38, 00 na venda de adesivos para os alunos desse
dois nveis quantos alunos de cada nvel participaram da feira?

OBMEP 2009

11

5
9

Nvel 2

Lista 2

Lista 2
1. Par perfeito Dizemos que 2 n
umeros naturais formam um par perfeito quando a soma
e o produto desses dois n
umeros sao quadrados perfeitos. Por exemplo, 5 e 20 formam
a que 122 forma um
um par perfeito, pois 5 + 20 = 25 = 52 e 5 20 = 100 = 102 . Ser
par perfeito com outro natural?
2. Um trap
ezio No trapezio da figura abaixo AB e paralelo a CD, AD = AB =

BC = 1 cm e DC = 2 cm. Quanto mede o angulo C AD?


(A) 30
(B)

45

(C) 60
(D) 90
(E) 120

.................................................................................
...
...
...
...
...
...
...
...
.
...
.
.
...
.
...
...
.
...
.
.
.
...
..
.
...
.
...
...
.
.
...
.
.
.
.....................................................................................................................................................................

3. Mist
erio das bolas Henrique tem duas urnas. A primeira urna contem somente bolas
pretas e a segunda somente bolas brancas. Henrique retirou um n
umero de bolas da
primeira urna e as colocou na segunda. Em seguida, retirou o mesmo n
umero de bolas
da segunda urna e as colocou na primeira. Depois disso o n
umero de bolas brancas na
primeira urna e maior, menor ou igual ao n
umero de bolas pretas na segunda urna?
4. Contando a palavra BRASIL Quantas vezes aparece
a palavra BRASIL na figura ao lado? S
o vale ler a palavra
emendando letras que est
ao escritas em quadradinhos
adjacentes.

B
R

B
R
A

B
R
A
S

B
R
A
S
I

B
R
A
S
I
L

5. Quais s
ao os n
umeros? Descubra quais n
umeros inteiros positivos x e y satisfazem
a equacao x4 = y 2 + 71.

12

OBMEP 2009

Lista 3

Nvel 2

Lista 3
1. No jogo Aldo, Bernardo e Carlos jogam baralho. No incio, a quantia em dinheiro que
eles tinham estava na proporcao 7 : 6 : 5. No final do jogo, a proporcao era 6 : 5 : 4.
Um dos jogadores ganhou 1 200 reais. Qual a quantidade de dinheiro com que ficou
cada jogador, no final da partida?



3
3
5+2
5 2 e um n
umero inteiro.
2. Um n
umero inteiro Mostre que M =

3. Area
de tri
angulos A area do quadrado ABCD e 300 cm2 . Na
figura, M e o ponto medio de CD e o ponto F pertence `a reta que
passa por B e C.
(a) Qual e a area do tri
angulo ABF ?
(b) Qual e area do tri
angulo ADF ?

4. Um quadriculado O retangulo quadriculado na figura e feito de


31 segmentos de 0,5 cm, e compreende 12 quadrados. Rosa desenhou numa folha retangular de 21 cm por 29,7 cm quadriculada com
quadrados de lado 0,5 cm, um grande ret
angulo quadriculado feito
com 1 997 segmentos. Quantos quadrados tem esse retangulo?
4

5. Inteiros de 4 algarismos Sabendo que a e um n


umero natural, e que 4a2 e a3
3
sao n
umeros naturais de 4 algarismos, determine a.

OBMEP 2009

13

5
9

Nvel 2

Lista 4

Lista 4
1. Pares positivos Quantos pares de inteiros positivos (x, y) sao solucoes da equacao
3x + 5y = 501?
2. Diferenca de quadrados Se a diferenca dos quadrados de dois n
umeros inteiros
consecutivos e 2 000, entao os dois n
umeros sao:
(A) menores que 100.
(B) menores que 1 000, porem maiores que 99.
(C) menores que 10 000, porem maiores que 999.
(D) menores que 100 000, porem maiores que 9 999.
(E) n
ao existem estes dois n
umeros.
3. C
alculo de
angulos Em cada uma das figuras a seguir, calcule o valor do angulo x,
sabendo que os segmentos AB e DE sao paralelos.

A........................................................................................................................................................B
.
....
....

25o ........
.
...

...
...
....
....
....
.
.
.
.......
....
....
....
....
....
....
....
....
....
...
55o
..........................................................................................................................................................

A........................................................................................................................................................B

....
....
....
....
....
....
....
....
....
....
....
...
....
...
.
.
...
.
.
...
...
....
....
...
o ......
150 ..
...........................................................................................................................................................

160o

4. Tabela Na tabela ao lado, com 6 colunas e diversas


linhas, est
ao escritos os n
umeros 1, 2, 3, 4, . . . Qual e
a posicao do n
umero 1 000?

1
7
13

2
8
14

3
9

4
10

5
11

6
12

..
.

..
.

..
.

..
.

..
.

..
.

5. Entre 1 e 2 Complete os numeradores com inteiros positivos para satisfazer as


b
a b
a
e
sao menores do que 1, e 1 < + < 2.
condicoes:
5
7
5 7

14

OBMEP 2009

Lista 5

Nvel 2

Lista 5
1. Triatlon Maria est
a planejando participar do Triatlon-Brasil que comeca a`s 24 horas
de domingo e consta de 800 m a nado, seguido de 20 km de bicicleta e finalmente
4 km de corrida. Maria corre a uma velocidade constante e que e o triplo da velocidade
que nada, e pedala 2,5 vezes mais rapido do que corre. Para terminar a prova em no
maximo 1 hora e 20 minutos, quanto tempo ela deve gastar em cada uma das 3 etapas?
2. Foto de formatura O diretor da escola decidiu tirar uma foto dos formandos de 2 008.
Ele colocou os alunos em filas paralelas, todas com o mesmo n
umero de alunos, mas
essa disposicao era muito larga para o campo de vis
ao de sua m
aquina fotogr
afica. Para
resolver esse problema, o diretor reparou que bastava tirar um aluno por fila e coloca-los
numa nova fila. Essa disposicao n
ao agradou o diretor porque a nova fila tinha 4 alunos
a menos que as outras. Ele decide entao tirar mais 1 aluno por fila colocando-os na nova
fila que ele criou, e constata que assim todas as filas ficam com o mesmo n
umero de
alunos, e finalmente tira a foto. Quantos alunos apareceram na foto?
3. Circunfer
encias tangentes Desenhe duas circunferencias de mesmo centro, uma de
raio 1 cm e a outra de raio 3 cm. Na regiao exterior a circunferencia de raio 1 cm e
interior a de raio 3 cm, desenhe circunferencias que sejam simultaneamente tangentes
`as duas circunferencias, como mostrado na figura a seguir.
........................................
...............
(a) Qual deve ser o raio dessas circunferencias?
(b) Qual o n
umero maximo dessas circunferencias, caso
elas n
ao se sobreponham?

... ......
.. .
... ....
...... .....
....
.....
.
...
....
...............................
...
...
...
....
....
.
....
.
......
..........................
...
..... ...........
.. ..
..
....
.. ...
...
.
...
.
.
....
......... ....... ...
.. .........
.
...
...
..
...
.
..
.
....
.
..
...
........................
...
..
.
.
...
.
...
...
...
...
....
....
....
....
.
......
.
.
.
...
.........
........................................

4. Festa na escola Para a festa de aniversario da escola, Ana, Pedro, Miriam e F


abio
levaram juntos 90 docinhos. A professora deles observou que:
se Ana tivesse levado 2 docinhos a mais;
se Pedro tivesse levado 2 docinhos a menos;
se Miriam tivesse levado o dobro;
se Fabio tivesse levado a metade;
os 4 amigos teriam levado todos o mesmo n
umero de docinhos. Quantos docinhos levou
cada um dos amigos?
5. Inflac
ao Marcia est
a numa loja comprando um gravador que ela queria h
a muito
tempo. Quando o caixa registra o preco ela exclama: Nao e possvel, voce registrou o
n
umero ao contr
ario, trocou a ordem de dois algarismos, lembro que na semana passada
custava menos que 50 reais! Responde o caixa: Sinto muito, mas ontem todos os
nossos artigos tiveram um aumento de 20%. Qual e o novo preco do gravador?

OBMEP 2009

15

5
9

Nvel 2

Lista 6

Lista 6
1. Gatos no condomnio Em um condomnio moram 29 famlias, cada uma delas possui
ou 1 gato ou 3 gatos ou 5 gatos. O n
umero de famlias que possuem apenas 1 gato e o
mesmo que o de famlias que possuem 5 gatos. Quantos gatos tem esse condomnio?
2. Soma constante Preencha as 5 casas em branco da tabela 3 3
com os n
umeros de 3 a 8, sem repeti-los, de modo que as somas dos
4 n
umeros escritos nas subtabelas formadas por quadrados 2 2 seja
a mesma nas 4 subtabelas.

2
9

AB C DE
BCDE
3. Qual
e o n
umero? Na adicao ao lado, letras iguais representam o
CDE
mesmo algarismo e letras diferentes algarismos diferentes. Encontre
DE
o n
umero ABCDE.
E
............................................................................
AAAAA
4. Proporc
ao triangular Num triangulo ABC, o ponto F esta sobre o lado AC e
F C = 2AF . Se G e o ponto medio do segmento BF e E o ponto de intersecao da reta
EC
.
passando por A e G com o segmento BC, calcule a raz
ao
EB

F
G

5. N
umeros primos entre si Encontre todos
de inteiros positivos x , y tais que

 os pares
x y
+
e um inteiro mpar.
x e y sao primos entre si, x < y e 2 000
y
x

16

OBMEP 2009

Lista 7

Nvel 2

Lista 7
1. Fique atento Determine todas as solucoes da equacao

x = x 2.

2. Soluco
es inteiras Determine todos os n
umeros inteiros x e y tais que:
1
1 1
+ =
.
x y
19
3. No ponto de
onibus Um grupo de meninos e meninas aguarda em um ponto pelo
onibus. No primeiro onibus que passa embarcam somente 15 meninas, e ficam 2 meninos
para cada menina no ponto de onibus. No segundo onibus que passa, embarcam somente
45 meninos, e ficam 5 meninas para cada menino no ponto de onibus. Determine
o n
umero de meninos e meninas que estavam no ponto antes da parada do primeiro
onibus.
4. Contorno circular A figura a seguir e formada por quatro crculos tangentes de raio
a. Determine o comprimento do contorno externo que esta com o tracado destacado.

5. Um quadril
atero especial Dois lados consecutivos de um quadril
atero medem 10 cm
e 15 cm. Se cada diagonal divide o quadril
atero em duas regi
oes de mesma area, calcule
o seu permetro.

OBMEP 2009

17

5
9

Nvel 2

Lista 8

Lista 8
1. N
umero curioso O n
umero 81 tem a seguinte propriedade: ele e divisvel pela soma
de seus algarismos 8 + 1 = 9. Quantos n
umeros de dois algarismos cumprem esta
propriedade?
2. N
umero premiado Um n
umero de 6 algarismos e premiado se a soma de seus
primeiros 3 algarismos e igual a` soma de seus 3 u
ltimos algarismos. Por exemplo 342 531
e premiado pois 3 + 4 + 2 = 5 + 3 + 1.
(a) Qual e o maior e o menor n
umero premiado, com 6 algarismos diferentes?
(b) Mostre que a soma de todos os n
umeros premiados, com 6 algarismos diferentes,
e divisvel por 13.
3. Altura versus lado Seja ABC um tri
angulo tal que a altura relativa ao lado BC
n
ao e menor do que o lado BC e a altura relativa ao lado AB n
ao e menor do que o
lado AB. Determine as medidas dos angulos deste tri
angulo.
4. Fraco
es egpcias Encontre n
umeros inteiros positivos a e b, com a > b, tais que:
1 1
2
=
+ .
7
a b
5. Tabuleiro de xadrez De quantas maneiras podemos colocar dois bispos num tabuleiro
de xadrez em filas, colunas e casas de cores distintas?

18

OBMEP 2009

Lista 9

Nvel 2

Lista 9
1. Quem
e menor? Sem usar calculadora, decida qual dos n
umeros 3312 , 6310 e 1278
e o menor.
2. Brincando com n
umeros A soma 1 + 1 + 4 dos algarismos do n
umero 114, divide o proprio n
umero. Qual e o maior n
umero, menor do que 900, que satisfaz esta
propriedade?
3. Cortando pap
eis No incio de uma brincadeira, Andre tinha 7 pedacos de papel. Na
primeira rodada, ele pegou alguns destes pedacos e cortou cada um deles em 7 pedacos,
que sao misturados aos pedacos de papel que n
ao foram cortados nesta rodada. Na
segunda rodada, ele novamente pegou alguns pedacos e cortou cada um deles em 7
pedacos que foram misturados aos demais papeis. Continuando desta maneira, ao final
de alguma rodada, Andre poder
a ter exatamente 2 009 pedacos de papel?
4. Um trap
ezio especial A base AD de um trapezio ABCD mede 30 cm. Suponhamos
que existe um ponto E sobre AD tal que os triangulos ABE, BCE e CDE
tenham permetros iguais. Determine o comprimento de BC.
5. Uma estrela Na estrela ABCDE na figura que se segue, sabemos que GBF = 20o
e GHI = 130o . Qual e o valor do angulo JEI?
A

E
I

H
C

OBMEP 2009

19

5
9

Nvel 2

Lista 10

Lista 10
1. N
umero palindrome Um n
umero e dito palindrome se a leitura da direita para a
esquerda e igual a da esquerda para a direita. Por exemplo, os n
umeros 23 432 e 18 781
sao palindromes. Quantos n
umeros palindromes de 4 algarismos sao divisveis por 9?
2. Multiplicac
ao com letras Na operacao abaixo, as letras a, b e c sao algarismos
distintos e diferentes de 1.
abb
c
bcb1
Determine os valores de a, b e c.
3. N
umeros sortudos Um n
umero sortudo e aquele cuja soma de seus algarismos e
divisvel por 7. Por exemplo, 7, 25 e 849 sao n
umeros sortudos. O menor par de
n
umeros sortudos e 7 e 16.
(a) Encontre oito n
umeros consecutivos, dos quais dois s
ao n
umeros sortudos.
(b) Encontre 12 n
umeros consecutivos, tal que nenhum seja sortudo.
(c) Mostre que qualquer sequencia de 13 n
umeros consecutivos contem pelo menos
um n
umero sortudo.
4. Uma sequ
encia especial Na sequencia 1, 3, 2, . . . cada termo depois dos dois primeiros
e igual ao termo precedente subtrado do termo que o precede, ou seja: se n > 2 entao
an = an1 an2 . Qual e a soma dos 100 primeiros termos dessa sequencia?
5. Tri
angulos e
angulos... Determine os angulos e .

20

OBMEP 2009

Lista 1

Nvel 3

Nvel 3
Lista 1
1. Brincando com a calculadora Digite numa calculadora um n
umero qualquer de
3 algarismos. Em seguida, digite o mesmo n
umero, obtendo assim um n
umero de 6
algarismos da forma a b c a b c. Divida esse n
umero por 7, divida o resultado por 11 e,
finalmente, divida o n
umero obtido por 13. O que aconteceu? Por que voce obteve este
resultado?
2. No galinheiro Um galinheiro com area igual a 240 m2 deve abrigar galinhas e
pintinhos, sendo desej
avel que haja um espaco livre de 4 m2 para cada galinha e
2
2 m para cada pintinho. Alem disso, cada pintinho come 40 g de racao por dia e
cada galinha come 160 g por dia, sendo permitido um gasto diario m
aximo de 8 kg de
racao.
(a) Represente algebricamente as condicoes do problema.
(b) Represente graficamente as condicoes acima no plano cartesiano xOy.
(c) Esse galinheiro comporta 20 galinhas e 80 pintinhos? E 30 galinhas e 100 pintinhos?
(d) Qual o n
umero maximo de galinhas que podem ser colocadas no galinheiro, respeitando os espacos desejaveis e o gasto maximo de racao? E de pintinhos?
3. Um n
umero perfeito Um n
umero natural n e dito perfeito se a soma de todos os
seus divisores pr
oprios, isto e, diferentes de n, e igual a n. Por exemplo, 6 e 28 sao
perfeitos, pois: 6 = 1 + 2 + 3 e 28 = 1 + 2 + 4 + 7 + 14. Sabendo que 231 1 e um
umero perfeito.
n
umero primo, mostre que 230 (231 1) e um n
4. Quinze minutos a mais Dois carros partem ao mesmo tempo de uma cidade A em
direcao a uma cidade B. Um deles viaja com velocidade constante de 60 km/h e o outro
com velocidade constante de 70 km/h. Se o carro mais rapido faz esta viagem em
15 minutos a menos que o outro carro, qual a dist
ancia entre as duas cidades?
5. Outros caminhos Partindo de sua casa para chegar na escola, J
ulia deve caminhar 8
quarteir
oes para a direita e 5 quarteir
oes para cima, como indicado na figura abaixo.

Ela sabe que existem muitas maneiras diferentes de fazer o percurso casa-escola, sempre
seguindo o caminho mais curto. Como ela e uma menina muito curiosa, ela gostaria de
sempre fazer caminhos diferentes.
Quantos desses caminhos existem da casa de J
ulia ate a escola?
OBMEP 2009

21

5
9

Nvel 3

Lista 2

Lista 2
1. Escrevendo em um tabuleiro Um tabuleiro quadrado de 3 linhas por 3 colunas
contem nove casas. De quantos modos diferentes podemos escrever as tres letras A, B
e C em tres casas diferentes, de modo que em cada linha esteja escrita exatamente uma
letra?
x
diminuirmos o numerador de 40% e o de2. Frac
ao e porcentagem Se na fracao
y
x
nominador y de 60%, entao a fracao :
y
(A) diminui 20%

(B) aumenta 20%

(C) diminui 50%

(D) aumenta 50%

3. Tri
angulos sobrepostos Dois tri
angulos ret
angulos congruentes possuem catetos de
medidas 4 cm e 7 cm. Na figura abaixo, `a esquerda, os tri
angulos foram desenhados de
modo a coincidirem os catetos de 7 cm. Assim, AB = 7 cm e AD = BC = 4 cm. Ja
na figura a` direita, eles foram desenhados de modo a coincidirem as hipotenusas donde,
AD = BC = 4 cm e AC = BD = 7 cm.

Calcule as areas sombreadas nas duas figuras.


4. Dois motoristas Dois motoristas viajam da cidade A ate a cidade B e, imediatamente,
regressam `a cidade A. O primeiro motorista viaja com velocidade constante de 80 km/h,
tanto na ida quanto na volta. O segundo motorista viaja ate a cidade B com velocidade
constante de 90 km/h e retorna com velocidade constante de 70 km/h. Qual desses
motoristas gasta menos tempo no percurso de ida e volta?
5. Soma e inverte Podemos formar sequencias a partir de um n
umero inicial, usando
duas operacoes +1 = somar 1 e i = menos o inverso. Por exemplo, iniciando
com o n
umero 3, podemos formar v
arias sequencias, veja uma delas:
+1

+1

3 4 5

1 +1 4 i
5 +1 1 +1 3 i
4
.
5
5
4
4
4
3

Iniciando com 0, com qual sequencia obteremos novamente o 0, usando apenas as duas
operacoes +1 e i?

22

OBMEP 2009

Lista 3

Nvel 3

Lista 3
1. Carro flex Um carro e denominado flex se ele pode ser abastecido com gasolina ou
com alcool. Considere que os precos do alcool e da gasolina sejam, respectivamente,
R$ 1,59 e R$ 2,49 por litro.
(a) Suponha que um carro flex rode 12, 3 km por litro de gasolina, que indicamos
12, 3 km/l. Qual deve ser a relacao km/l desse carro, para o alcool, para que a
utilizacao do alcool seja financeiramente mais vantajosa que a de gasolina?

x
+ 1 km/l
(b) Se o desempenho de um carro flex e de x km/l com gasolina e de
2
com alcool, escreva a expressao da funcao g(x) que fornece o custo desse carro
rodar 100 km utilizando gasolina e a expressao da funcao a(x) que fornece o custo
desse carro rodar 100 km utilizando alcool.
(c) Para que o custo seja o mesmo, tanto com alcool como com gasolina, qual deve
ser a relacao km/l para a gasolina e para o alcool?
(d) Em que condicao o uso do alcool e mais vantajoso, financeiramente, que o da
gasolina? De um exemplo numerico que satisfaca a condicao.
2. Contando tri
angulos Na figura a seguir estao marcados 11 pontos sobre dois segmentos. Quantos tri
angulos podem ser formados com estes 11 pontos?

3. Quadrado perfeito Existe um n


umero de 8 algarismos da forma
9999
que e um quadrado perfeito?
4. Diferenca quase nula Qual o menor n
umero inteiro positivo n tal que

n n 1 < 0,01?
5. Conjunto de Cantor Desenhe um segmento de reta de comprimento 1, e denote-o
por C1 . Remova o terco central (sem remover os extremos). Denote por C2 o que
sobrou. Agora, remova o terco central (sem os extremos) de cada segmento de reta de
C2 . Denote por C3 o que sobrou. Podemos continuar esse processo, em cada estagio
removendo o terco central de cada segmento em Cn para formar Cn+1 .
OBMEP 2009

23

5
9

Nvel 3

Lista 3

C1
C2
C3
(a) Desenhe C1 , C2 e C3 , indicando os n
umeros nos extremos dos segmentos.
4
1 4 3
, ,
,
.
(b) Quais dos seguintes pontos pertencem ao conjunto de Cantor?
3 9 81 81
ao para
(c) Quais s
ao os comprimentos de C3 , C4 e C5 ? Voce pode achar uma express
o comprimento de Cn ?

24

OBMEP 2009

Lista 4

Nvel 3

Lista 4
1. Enchendo uma piscina Uma piscina vazia foi abastecida de agua por duas torneiras
A e B, ambas com vaz
ao constante. Durante 4 horas, as duas torneiras ficaram abertas
e encheram 50% da piscina. Em seguida, a torneira B foi fechada e durante 2 horas a
torneira A encheu 15% do volume da piscina. Ap
os este perodo a torneira A foi fechada
e a torneira B aberta. Durante quanto tempo esta torneira teve de ficar aberta para que
ela sozinha terminasse de encher a piscina?
2. Probabilidade de ser um n
umero par Uma urna tem 9 bolas, numeradas com os
n
umeros de 1 a 9. Jose e Maria retiram simultaneamente uma bola da urna. Com as
bolas retiradas eles formam um n
umero de 2 algarismos, sendo que o n
umero que esta
escrito na bola de Jose e o algarismo das dezenas e o n
umero que esta escrito na bola
de Maria e o algarismo das unidades. Qual a probabilidade deste n
umero ser par?
3. M
ultiplo de 7 Mostre que se o produto N = (n + 6m)(2n + 5m)(3n + 4m) e m
ultiplo
3
de 7, com m e n n
umeros naturais, ent
ao N e m
ultiplo de 7 = 343.
4. Os
angulos 15 e 75 Na figura, ABCD e um quadrado de lado 1 cm e BCE e um
tri
angulo equil
atero. O ponto M e o ponto medio do segmento CE, DN e perpendicular
a BM e BM e perpendicular a CE.
C

(a) Calcule os comprimentos dos lados do


tri
angulo DBN .

F
N

(b) Use o item (a) para calcular o cosseno,


o seno e a tangente dos angulos de 15
e 75 .

5. Circunf
encias tangentes Na figura, estao desenhadas duas
circunferencias concentricas de raios r e R, com r < R, e
12 circunferencias, de raio x, compreendidas entre essas duas.
Alem disso, as 14 circunferencias sao disjuntas ou tangentes.
Rr
.
2

R
4+ 6 2

=
r
4 6+ 2

(a) Mostre que x =


(b) Mostre que

OBMEP 2009

25

5
9

Nvel 3

Lista 5

Lista 5
1. Mudando a base Um tri
angulo is
osceles tem base medindo
possvel mudar a base do
10 cm e dois lados iguais a 13 cm. E
tri
angulo e obter outro tri
angulo is
osceles com mesma area?

2. Clube de Matem
atica Eu faco parte de um clube de matematica onde tenho o
mesmo n
umero de colegas homens do que colegas mulheres. Quando um garoto falta,
os tres quartos da equipe sao de meninas. Eu sou homem ou mulher? Quantas mulheres
e quantos homens tem o clube?
3. Uma calculadora diferente Davi tem uma calculadora muito original; ela efetua
apenas duas operacoes: a adicao usual (+) e uma outra operacao, denotada por , que
satisfaz:
(i) a a = a
(ii) a 0 = 2a
(iii) (a b) + (c d) = (a c) + (b d)
Quais s
ao os resultados das operacoes (2 + 3) (0 + 3) e 1 024 48?
4. Ret
angulo m n O retangulo quadriculado na figura e feito de 31
segmentos de 0,5 cm e compreende 12 quadrados. Rosa desenhou
numa folha retangular de 21 cm por 29,7 cm, quadriculada com
quadrados de lado 0,5 cm, um grande ret
angulo quadriculado feito
com 1 997 segmentos. Quantos quadrados tem esse retangulo?
5. Cercando o Globo Terrestre O raio do Globo Terrestre e aproximadamente 6 670 km. Suponhamos que um fio esteja ajustado exatamente sobre o Equador, que e um crculo de raio aproximadamente
igual a 6 670 km.
Em seguida, suponhamos que o comprimento do fio seja aumentado em 1 m, de modo
que o fio e o Equador fiquem como crculos concentricos ao redor da Terra. Um homem
em pe, uma formiga ou um elefante sao capazes de passar por baixo desse fio?

26

OBMEP 2009

Lista 6

Nvel 3

Lista 6
1. Comprimento de uma corda Em uma circunferencia de raio 10 cm, o segmento AB
e um diametro e o segmento AC e uma corda de 12 cm. Determine a distancia entre
os pontos B e C.
2. Dois irm
aos A diferenca de idade entre dois irmaos e de 3 anos. Um ano atr
as, a
idade de seu pai era o d
obro da soma das idades dos irm
aos e, dentro de 20 anos, a
idade do pai ser
a a soma das idades desses dois filhos. Qual a idade de cada um?

3. Canelonis de ricota Todo domingo, Pedro


prepara canelonis para o almoco. Primeiro ele
corta ret
angulos de massa de 16 cm por 12 cm e
depois cola os dois lados mais longos, superpondo
uma faixa de 2 cm.
Dessa forma ele obtem cilindros que ele recheia com ricota, ele ja sabe que sempre gasta
500 g de ricota. Num belo domingo, com o mesmo n
umero de retangulos de massa
de 16 cm por 12 cm, ele decide produzir os cilindros colando os lados menores, sempre
superpondo uma faixa de 2 cm. Nessa situacao, ele vai gastar mais ou menos ricota que
antes? Quanto?
4. C
alculo de segmentos As medidas do retangulo
ABCD sao 1 200 m por 150 m. Alem disso, P
esta no prolongamento do lado BC e dista 350 m
de C. Determine AP, P Q, P D, CQ e DP .

5. Pr
a chegar junto! Ana e Luza treinam todos os dias para a Grande Corrida que vai
acontecer no final do ano na escola, cada uma delas sempre com a mesma velocidade.
O treino comeca num ponto A e termina no ponto B, distantes 3 000 m. Elas partem
no mesmo instante, mas quando Luza termina a corrida, ainda faltam 120 m para Ana
chegar ao ponto B. Ontem Luza deu uma chance para Ana: Partimos ao mesmo
tempo, mas eu parto alguns metros antes do ponto A para chegarmos juntas. Quantos
metros antes do ponto A Luza deve partir?

OBMEP 2009

27

5
9

Nvel 3

Lista 7

Lista 7
1. Um professor enfurecido Para castigar os alunos de sua turma por indisciplina, o
professor Zerus decidiu descontar da nota mensal de cada aluno uma percentagem igual
`a nota da prova, isto e: quem tirou 60, ter
a um desconto de 60% na nota, quem tirou
20, um desconto de 20% da nota, e assim por diante. A nota mensal m
axima e 100.
(a) Quem vai ficar com a maior nota?
(b) E a menor?
(c) Alunos que tiraram boas notas reclamaram que v
ao ficar com a mesma nota dos
que tiraram m
as notas. Eles estao certos?
2. O percurso de um atleta Um atleta resolveu fazer uma corrida de 15 km. Comecou
correndo 5 km na direcao Sul, depois virou para direcao Leste, correndo mais 5 km e,
novamente, virou para a direcao Norte, correndo os 5 km restantes. Apos esse percurso,
constatou, para seu espanto, que estava no ponto de onde havia partido.
Descubra dois possveis pontos sobre o Globo Terrestre de onde esse atleta possa ter
iniciado sua corrida.
A

3. Areas
iguais Na figura ao lado, o tri
angulo ABC
e retangulo e os semicrculos desenhados tem diametros
AB, BC e AC.
B

Mostre que a area sombreada e igual a` area do


tri
angulo ABC.
4. Func
ao definida por
area A funcao f esta definida para cada y, 0 y < 2, de modo
que f (y) =
area do quadril
atero sombreado, como indicado na figura abaixo.

(a) Escreva as equacoes das retas r e s.


(b) Determine f (0).
(c) Escreva a expressao de f (y), 0 y < 2.
(d) Esboce o grafico de f (y).

5. PA e PG Determine 4 n
umeros distintos a1 , a2 , a3 e a4 que sejam termos consecutivos
de uma progress
ao aritmetica e que os n
umeros a1 , a3 e a4 formem uma progressao
geometrica.

28

OBMEP 2009

Lista 8

Nvel 3

Lista 8
1. Plano cartesiano No plano cartesiano, chama-se ponto inteiro a um ponto de
coordenadas inteiras. Se n e inteiro positivo, seja f (n) o n
umero de pontos inteiros
que estao sobre o segmento que liga a origem ao ponto inteiro (n , n + 3), sem contar
os extremos. Mostre que:

2 se n e m
ultiplo de 3
f (n) =
0 se n n
ao e m
ultiplo de 3 .
2. Trabalhando com quadril
atero No quadrilatero ABCD,
tem-se: AB = 5, BC = 17, CD = 5, DA = 9, e a medida
do segmento DB e um inteiro. Determine DB.

..........
................... ......
...................
....
..............
....
...
.
....
.
.
.
....
....
....
....
.
.
....
...
.
....
.
..
.
.
.
..................................................................................................................................

3. O tri
angulo de Reuleaux O tri
angulo de Reuleaux e um disco
formado a partir de um tri
angulo equil
atero, agregando arcos de circunferencia com centros nos vertices do triangulo e raios iguais ao
lado do triangulo.
Qual e a area de um triangulo de Reuleaux, se o tri
angulo equil
atero tem lado de medida
1 cm?
4. Intersec
ao entre circunfer
encias Com centros nos
vertices do triangulo equil
atero ABC de lado a, foram
desenhadas tres circunferencias de raio r.
Se r < a e 2r > a, estas tres circunferencias sao duas
a duas concorrentes nos pontos X, Y e Z, exteriores ao
tri
angulo ABC. Mostre que XY Z e um triangulo
equil
atero e calcule o comprimento do seu lado em termos de a e r.
5. Valor m
aximo Para qual n
umero natural k a expressao

OBMEP 2009

k2
atinge seu maior valor?
1,001k

29

5
9

Nvel 3

Lista 9

Lista 9
1. Moedas falsas Aladim tem 10 sacos de moedas, onde cada
saco tem somente moedas verdadeiras ou moedas falsas. Cada
moeda verdadeira pesa 10 g e cada moeda falsa pesa 9 g.

(a) Suponhamos que em cada saco existam exatamente 10 moedas e somente um


dos sacos e de moedas falsas. Utilizando uma balanca e efetuando apenas uma
pesagem, como Aladim deve proceder para descobrir qual e o saco das moedas
falsas?
(b) Suponhamos que os sacos estejam cheios de moedas e que Aladim n
ao saiba quantos destes sacos sao de moedas falsas. Como pode ele identificar os sacos que tem
moedas falsas com apenas uma pesagem?
2. Menor inteiro Sejam p e q inteiros positivos tais que
valor de p para que p + q = 2 005?

p
7
5
< < . Qual e o menor
8
q
8

3. Mais
areas... Um tri
angulo tem vertice A = (3, 0), B = (0, 3) e C, onde C esta
sobre a reta x + y = 7. Qual e a area do tri
angulo?
4. Circunfer
encias tangentes Tres circunferencias de raios
1 cm, 2 cm e 3 cm sao duas a duas tangentes exteriormente,
como na figura ao lado.
Determine o raio da circunferencia tangente exteriormente
`as tres circunferencias.

5.
Soma finita Cada um dos n
umeros x1 , x2 , . . . , x2 004 pode ser igual a
2 + 1. Quantos valores inteiros distintos a soma
2
004

x2k1 x2k = x1 x2 + x3 x4 + x5 x6 + + x2 003 x2 004

k=1

pode assumir?

30

OBMEP 2009

2 1 ou a

Lista 10

Nvel 3

Lista 10
1. M
ultiplos Seja a um n
umero inteiro positivo tal que a e m
ultiplo de 5, a+1 e m
ultiplo
de 7, a + 2 e m
ultiplo de 9 e a + 3 e m
ultiplo de 11. Determine o menor valor que a
pode assumir.
2. Equac
ao de duas vari
aveis Determine todos os pares de inteiros (x, y) tais que
9xy x2 8y 2 = 2 005.
3. Trap
ezio ret
angulo Seja ABCD um trapezio retangulo de bases AB e CD, com
angulos retos em A e D. Dado que a diagonal menor BD e perpendicular ao lado BC,
CD
.
determine o menor valor possvel para a raz
ao
AD
4. Jogos de futebol Os doze alunos de uma turma de olimpada saam para jogar futebol
todos os dias apos a aula de matem
atica, formando dois times de 6 jogadores cada e
jogando entre si. A cada dia eles formavam dois times diferentes dos times formados
em dias anteriores. Ao final do ano, eles verificaram que cada 5 alunos haviam jogado
juntos num mesmo time exatamente uma vez. Quantos times diferentes foram formados
ao longo do ano?
5. A soma dos algarismos de um n
umero Denotemos por s(n) a soma dos algarismos
do n
umero n. Por exemplo s(2 345) = 2 + 3 + 4 + 5 = 14. Observemos que:
40s(40) = 36 = 94; 500s(500) = 495 = 955; 2 345s(2 345) = 2 331 = 9259 .
(a) O que podemos afirmar sobre o n
umero n s(n)?
(b) Usando o item anterior calcule s(s(s(22 009 ))).
o: Mostre que o n
Sugesta
umero procurado e menor do que 9.

OBMEP 2009

31

5
9

Desafios

Desafios
1. Data fatdica Em 1 950 um profeta anunciou que o fim do mundo ocorreria em
11.08.1 999 (11 de agosto de 1 999). Como nada aconteceu nesse dia, ele refez seus
calculos e fez a seguinte previs
ao: O fim do mundo ocorrer
a na pr
oxima data que se
escreve com 8 algarismos diferentes. Voce pode descobrir essa data?
2. Todos com o 2 Qual operacao devemos fazer com todos os 5 n
umeros
418 , 244 , 816 , 426 , 24
para obter 5 n
umeros que tenham todos o algarismo 2?
(a) dividir 2;
(b) somar 4;
(c) dividir por 6;
(d) subtrair 5;
(e) multiplicar por 3.
3. Tortas da vov
o Sofia foi levar uns docinhos para sua av
o; sao 7 docinhos de amora,
6 de coco e 3 de chocolate. Durante o caminho, a gulosa Sofia come 2 docinhos. Qual
das situacoes abaixo e possvel?
(A) Vovo n
ao recebeu docinhos de chocolate.
(B) Vovo recebeu menos docinhos de coco do que de chocolate.
(C) Vov
o recebeu o mesmo n
umero de docinhos de cada uma das 3 variedades.
(D) Existem 2 variedades de docinhos das quais vov
o recebeu o mesmo n
umero.
(E) O n
umero de docinhos de amora que vov
o recebeu e maior que o dos outros 2
somados.
4. Famlia S
etimo O Sr. e Sra. Setimo tem 7 filhos, todos nascidos em 1o de abril, na
arios, a Sra. Setimo
verdade em seis 1o de abril consecutivos. Este ano, para seus anivers
fez um bolo com velinhas para cada um o n
umero de velas igual ao n
umero de anos de
cada um. Jo
ao Setimo, o filho que mais gosta de Matematica, reparou que nesse ano o
n
umero total de velinhas e o dobro do que havia 2 anos atr
as e que h
a 2 bolos a mais.
Quantas velinhas ser
ao acesas esse ano?
32

OBMEP 2009

5
9

Desafios

5. O Salta-Ficha Temos 10 fichas numeradas colocadas em linha reta como na figura.


................
...
..
...
.... .....
...........

................
...
..
....
.
..... .....
........

...............
...
..
...
...
...
................

...............
..
...
.
....
.... ......
.........

................
...
..
....
.
..... ......
.......

................
...
.
.....
...
..................

...............
..
...
....
.
..... ......
........

................
...
..
...
.... .....
...........

................
...
..
....
.
..... ......
.......

.........
..... ....

...
9 ........10
...
... .....
.......

Queremos arruma-las em 5 pilhas com 2 fichas cada uma. A regra para isso e que so
podemos movimentar uma ficha fazendo-a saltar sobre uma ou mais fichas, ou sobre
uma pilha. Veja um exemplo de 3 movimentos:
a ficha 5 pode saltar sobre as fichas 6 e 7 e formar uma pilha com a 8.
a ficha 7 pode saltar sobre a ficha 8 e formar uma pilha com a 9.
a ficha 6 pode saltar sobre as fichas 5, 4 e 3 formar uma pilha com a 2.

..
....... .........
.
....
...
...
................

...........
..
.....
...........
.......
...........
.....
.......
...........
.....
.......
...........
.....
.......
.
.
.
.
.
.
.
...........
.
.
.
.....
...........
.....
.......
...........
.....
.....................
.....
.....
..... ..........
......... .............. .............. ................. ................. ................ .............. ...............
.
.
.
.
.
.
... ...
... ...
... ...
... ...
... ...
... ...
.. ...
..
..
.. ..
.. ..
.. ..
.. ..
.. ..
.. ..
.....
..
.. ..
..... ...... ...... ....... ....... ...... ....... ...... ....... ...... ...... ....... ....... ...... ....... ......
........
........
........
........
........
........
........
.......

?
2 3

? ? ............
...
..
...
8 9 ...........10
..
. .....

......

.......
..... .......
....
..
...
................
........
.
.
.
.
.
.
...
...
..
...
.... .....
...........

..........
.... .....
.....
..
..... ......
........

........
..... ......
.
....
..
...
................
........
.
.
.
.
.
.
...
..
.
....
..
...
................

................
...
...
....
...... .......
......

..........
.... .....
....
..
...
...............

.......
..... .......
.
....
..
...
................
........
.
.
.
.
.
.
...
..
..
.....
.... ......
.........

7
9

................
...
...
....
...... ........
.....

10

Como formar 5 pilhas de 2 fichas com apenas 5 movimentos?


6. O menor Qual e o menor: 52002 ou 32002 + 42002 ?
7. O maior resultado Qual o maior resultado que podemos encontrar quando dividimos
um n
umero de 2 algarismos pela soma de seus algarismos?
8. Dois mil O peso de um n
umero e a soma de seus algarismos. Qual e o menor n
umero
que pesa 2 000?
9. No cabeleireiro Tres clientes estao no cabeleireiro pagando cada um a sua conta no
caixa.
o primeiro cliente paga o mesmo montante que h
a no caixa e retirar 10 reais de
troco;
o segundo cliente efetua a mesma operacao que o primeiro;
o terceiro cliente efetua a mesma operacao que os dois primeiros.
Encontre o montante que estava inicialmente no caixa, sabendo que ao fim das 3
operacoes o caixa ficou zerado.
10. O macaco e a raposa O macaco diz para a raposa:
Voce ve as 3 pessoas que estao correndo l
a longe? Eu sei que o produto de suas idades
e 2 450; e que a soma das idades e o dobro da sua idade. Voce pode me dizer suas
idades?
Nao, responde a raposa.
E se eu te disser que o mais jovem dos tres e o u
nico louro, voce pode agora descobrir
as idades?
E a raposa d
a as idades das 3 pessoas.
Porque a raposa n
ao pode responder inicialmente? E porque pode responder depois?
OBMEP 2009

33

5
9

Desafios

11. Nova sequ


encia Encontre a lei que forma a sequencia e de seus proximos 2 termos:
425, 470, 535, 594, 716, 802, . . .
12. Ret
angulo quase quadrado Um terreno retangular e quase quadrado: sua largura e
seu comprimento sao n
umeros inteiros de metros que diferem exatamente de 1 metro.
A area do terreno, em metros quadrados, e um n
umero de 4 algarismos, sendo o das
unidades de milhar e o das centenas iguais, e o mesmo ocorre com o das dezenas e das
unidades. Quais s
ao as possveis dimensoes do terreno?
ao x = 0 e por isso
13. Aonde est
a o erro? Seja x solucao de x2 + x + 1 = 0. Ent
1
podemos dividir ambos os membros da equacao por x, obtendo x + 1 + = 0. Da
x
1
equacao temos que x + 1 = x2 , logo x2 + = 0, isto e: x2 = 1/x ou ainda x3 = 1
x
e x = 1. Substituindo x = 1 na equacao x2 + x + 1 = 0 encontramos 3 = 0 !!!! Aonde
erramos?

34

OBMEP 2009

Lista 1

Solucoes do Nvel 1

Soluco
es do Nvel 1
Lista 1
1. Encontro de amigos Eu chegarei quando meu relogio marcar 10 h 5 min, uma vez
que penso que o rel
ogio esta adiantado 5 min. Como ele esta atrasado 10 min, chegarei,
na verdade, as 10 h 15 min.
Meu amigo chegar
a quando seu rel
ogio marcar 10 horas, pois ele pensa que o rel
ogio
esta correto, mas na realidade ser
ao 9 h 55 min. Logo meu amigo chegar
a 20 min antes
de mim.
2. Trabalho comunit
ario A resposta correta e (B).
Do n
umero total de alunos dessa classe, 60% foram prestar trabalho comunit
ario, isto
e, 0,6 40 = 24. O n
umero mnimo de alunas que participaram desse trabalho e obtido
quando o n
umero de alunos que participaram e maximo, ou seja, quando 22 alunos se
envolverem no trabalho, restando o mnimo de 2 vagas para as alunas.

3. Area
de trap
ezios A resposta correta e (E).
Unindo os quatro trapezios, formamos um quadrado de lado
50 cm, e portanto de area 2 500 cm2 . Como o buraco
quadrado tem lado 30 cm, sua area e 30 30 = 900 cm2 .
Logo, a area de cada um dos 4 trapezios, em cm2 , e
(2 500 900) 4 = 1 600 4 = 400 .
4. Adivinhac
ao Ja de incio sabemos sobre o maior n
umero:
e par por ser o dobro do menor mas n
ao termina em zero porque o maior e o menor
n
umero n
ao possuem algarismos em comum;
seu algarismo das dezenas e no mnimo 2 porque sua metade e um n
umero com 2
algarismos;
a soma de seus algarismos e no maximo 9, porque essa soma e um dos algarismos
do menor n
umero;
Logo, os candidatos ao maior e menor n
umero sao:
maior
menor

22
11

32
16

62
31

72
36

34
17

44
22

54
27

26
13

36
18

Por verificacao, temos que 17 e 34 sao os n


umeros que satisfazem as condicoes do
problema.
OBMEP 2009

35

5
9

Solucoes do Nvel 1

Lista 1

5. 18 n
umeros consecutivos Uma sequencia de 18 n
umeros consecutivos possui sempre
2 termos que sao m
ultiplos de 9. Logo, a soma dos algarismos de cada um desses 2
n
umeros e um m
ultiplo de 9. Observe que como os n
umeros tem 3 algarismos, a maior
das somas que pode ocorrer e 27. Logo as possibilidades para as somas dos algarismos
desses 2 n
umeros sao:
(i) 9 e 9
(ii) 9 e 18
(iii) 18 e 18
(iv) 18 e 27
Vamos examinar alguns exemplos de cada um dos 4 casos.
(i) 9 e 9
Exemplo: um dos n
umeros e 144, e o outro 135 ou 153. Veja algumas possveis
sequencias:
144 ,  ,  ,  ,  ,  ,
141 ,  ,  , 

1o

2o

3o

4o

5o

6o

7o

8o

9o

153 ,  ,  ,  ,  , 


158 ,
 ,  ,  , 
10o

11o

12o

13o

14o

15o

16o

17o

18o

135 ,  ,  ,  ,


130 ,  ,  ,  ,  , 

1o

2o

3o

4o

5o

6o

7o

8o

9o

144 ,  ,  , 


147 .
 ,  ,  ,  ,  , 
10o

11o

12o

13o

14o

15o

16o

17o

18o

(ii) 9 e 18
Exemplo: um dos n
umeros e 900 e o outro 891 ou 909.
sequencias:

Veja algumas possveis

891 ,  ,  ,  ,  ,


887 ,  ,  ,  , 

1o

2o

3o

4o

5o

6o

7o

8o

9o

900 ,  ,  ,  , 


904 ,
 ,  ,  ,  , 
10o

11o

12o

13o

14o

15o

16o

17o

18o

900 ,  ,  ,  ,  ,  ,  ,


898 ,  , 

1o

2o

3o

4o

5o

6o

7o

8o

9o

909 ,  ,  ,  ,  ,  , 


915 .
 ,  , 
10o

11o

12o

13o

14o

(iii) 18 e 18

36

OBMEP 2009

15o

16o

17o

18o

Lista 1

Solucoes do Nvel 1

Exemplo: um dos n
umeros e 828 e o outro 819 ou 837.
sequencias:

Veja algumas possveis

819 ,
811 ,  ,  ,  ,  ,  ,  ,  , 

1o

2o

3o

4o

5o

6o

7o

8o

9o

828 ,
 ,  ,  ,  ,  ,  ,  ,  , 
10o

11o

12o

13o

14o

15o

16o

17o

18o

828 ,  ,  ,  ,


823 ,  ,  ,  ,  , 

1o

2o

3o

4o

5o

6o

7o

8o

9o

837 ,  ,  , 


840 .
 ,  ,  ,  ,  , 
10o

11o

12o

13o

14o

15o

16o

17o

18o

(iv) 18 e 27.
Nesse caso um dos n
umeros e 999 e temos uma u
nica opcao para a sequencia:
982 ,  ,  ,  ,  ,  ,  ,  , 
990 ,

1o

2o

3o

4o

5o

6o

7o

8o

9o

999 .
 ,  ,  ,  ,  ,  ,  ,  , 
10o

11o

12o

13o

14o

15o

16o

17o

18o

Vamos agora analisar cada caso. Nos casos (i) e (ii) um dos n
umeros e divisvel por 9
que e a soma de seus algarismos. No caso (iv) um dos n
umeros e 999 que e divisvel
por 27. Finalmente no caso (iii) um dos n
umeros tem de ser par, pois sao 2 m
ultiplos
consecutivos de 9. Logo, esse n
umero e m
ultiplo de 2 e 9, portanto m
ultiplo de 18.

OBMEP 2009

37

5
9

Solucoes do Nvel 1

Lista 2

Lista 2
1. Completar uma tabela Observe que em cada quadrado formado por 4 quadradinhos,
o n
umero que est
a na parte inferior direita e a soma dos outros 3 n
umeros. Assim,
temos:
0
1
2
3
4

1
2
1+2+2 =5
10
17

2
5
2 + 5 + 5 = 12
27
54

3
10
5 + 10 + 12 = 27
66
147

4
3 + 4 + 10 = 17
10 + 17 + 27 = 54
147
A

Logo:
A = 66 + 147 + 147 = 360 .
2. Procurando m
ultiplos de 9 Sempre existe uma diferenca que e um m
ultiplo de 9.
De fato, quando dividimos um n
umero por 9, podemos encontrar nove restos diferentes:
0, 1, 2, 3, 4, 5, 6, 7 ou 8. Logo, entre os 10 n
umeros do conjunto, pelo menos dois deles
tem mesmo resto quando divididos por 9, j
a que temos no maximo 9 restos diferentes.
Quando fazemos a diferenca desses dois n
umeros que tem o mesmo resto, obtemos um
n
umero com resto zero, ou seja, divisvel por 9.
3. Correndo numa praca A dist
ancia que ele percorre a cada volta completa e igual ao
permetro da praca:
2 900 + 2 600 = 3 000 m .
Como 15,5 km= 15 500 m e 15 500 = 5 3 000 + 500,
o atleta d
a 5 voltas completas (partindo de P e retornando
a P ), e corre ainda mais 500 m. Portanto, ele para no
ponto Q, a 150 m do vertice B, como na figura.

4. Ovos para um bolo Como os 43 bolos tem a mesma receita, o n


umero de ovos que a
doceira precisa e um m
ultiplo de 43. Por outro lado, esse n
umero tambem e um m
ultiplo
de 2, 3, 4, 5 e 6 acrescido de 1. O mmc de 2, 3, 4, 5 e 6 e 60, mas 60 + 1 = 61 n
ao e
m
ultiplo de 43! Precisamos, entao, encontrar um n
umero com essas duas propriedades:
e m
ultiplo de 43;
acrescido de 1 e m
ultiplo de 2, 3, 4, 5 e 6.
Lembre tambem que como a receita gasta menos de 9 ovos, o n
umero que estamos
procurando e menor do que 43 9 = 387. Temos:
38

OBMEP 2009

Lista 2

Solucoes do Nvel 1
60 2 + 1 = 121
60 3 + 1 = 181
60 4 + 1 = 241
60 5 + 1 = 301
60 6 + 1 = 361

n
ao e m
ultiplo de
n
ao e m
ultiplo de
n
ao e m
ultiplo de
e m
ultiplo de 43
n
ao e m
ultiplo de

43
43
43
43

Podemos parar por aqui porque os pr


oximos n
umeros serao maiores do que 387. Logo,
a doceira comprou 301 ovos.
5. C
alculos H e V Inicialmente, veja que os possveis lugares para
o n
umero 1 estao mostrados ao lado. J
a as multiplicacoes so
podem ser 2 3 = 6 e 2 4 = 8. Agora, repare que o 2 so pode
ser o multiplicando e n
ao o multiplicador (tente coloc
a-lo como
multiplicador e veja que isso nao e possvel).

m m= 2m
1m

....................................

....................................

1m+ 1m= m

Temos agora duas opcoes para preencher.


ao: 2 3 = 6
1a opc

m m= 2m
m

....................................

3m

....................................

8m 4m= 2m

....................................

m+ m= 6m

3m

....................................

m+ m= 6m

8m 4m= 2m
7m

....................................

3m

....................................

1m+ 5m= 6m

2a opc
ao: 2 4 = 8

m m= 2m
m

....................................

4m

....................................

m+ m= 8m

6m 3m= 2m

....................................

4m

....................................

m+ m= 8m

OBMEP 2009

6m 3m= 2m
5m

....................................

4m

....................................

1m+ 7m= 8m

39

5
9

Solucoes do Nvel 1

Lista 3

Lista 3
1. Cortando uma cartolina Os lados do retangulo final obtido ap
os os cortes sao, cada
um, a metade dos lados da cartolina original. Assim, o permetro do ret
angulo original e
o dobro do permetro do ret
angulo final. Logo, o permetro da cartolina antes do corte
e 2129 = 258 cm.
Observac
ao. Ao fazer um corte paralelo a um dos lados do tri
angulo e pelo ponto medio
desse lado, o outro corte que formar
a o ret
angulo, s
o pode ocorrer no ponto medio do
outro lado, em vista da semelhanca que ocorre desses triangulos. Assim, o enunciado
contem um dado a mais, desnecessario para os que conhecem semelhanca de tri
angulos.
` primeira inspecao, podemos admitir que os tres algarismos `a direita
2. A soma errada A
dos n
umeros estao corretos, isto e, estao corretos os algarismos 0, 1, 3, 4, 5, 6 e 8.
Portanto, dentre os algarismos 2, 7 e 9, um deles esta errado. O algarismo 9 esta certo,
pois se o mudarmos, a soma com 2 n
ao estar
a certa. Sendo assim, sobraram 2 e 7. Se
o 7 estiver errado, ent
ao 2 estar
a correto, mas isso n
ao e possvel pois 1 + 4 + 2 = 7.
Logo, o 2 e que deve ser substitudo. Olhando novamente para a soma 1 + 4 + 2, vemos
que o resultado e um n
umero com o algarismo da unidade igual a 1. Logo, o algarismo 2
deve ser substitudo por 6. Fazendo a substituicao, verificamos que a soma fica correta.
3. N
umero de 5 algarismos Para que a b c seja divisvel por 4, seus dois u
ltimos algarismos devem formar um n
umero divisvel por 4. Como os algarismos sao 1, 2, 3, 4 e 5,
as u
nicas possibilidades s
ao: b c = 12, b c = 24, b c = 32, b c = 52. Por outro lado,
os n
umeros divisveis por 5 terminam em 0 ou 5. Como 0 n
ao esta includo, segue que
d = 5 pois bcd e divisvel por 5. Isso exclui a possibilidade bc = 52 porque n
ao podemos
repetir o 5. Ate agora temos 3 possibilidades:
a 1 2 5 e, a 2 4 5 e, a 3 2 5 e .
Vamos agora examinar esses 3 casos, para escolher os algarismos a e e, lembrando que
n
ao pode haver repeticao.
a325e

a245e

a125e

e=3

e=4

e=1

e=3

e=1

e=4

41 253

31 254

32 451

12 453

43 251

13 254

mltiplo de 3

mltiplo de 3

mltiplo de 3

mltiplo de 3

Logo, o n
umero e 12 453.
40

OBMEP 2009

mltiplo de 3

mltiplo de 3

Lista 3

Solucoes do Nvel 1

4. Tabela misteriosa Observemos que:


na u
ltima coluna estarao os m
ultiplos de 9 porque essa coluna esta em branco e
nenhum dos n
umeros que aparecem na tabela e m
ultiplo de 9;
a
ultiplos de 12, pois e nessa linha que aparece o u
nico
na 5 linha estarao os m
m
ultiplo de 12 da tabela (24);
ultiplos de 10, pois 40 e o u
nico m
ultiplo de 10 na
na 4a coluna estarao os m
tabela;
na 5a coluna teremos m
ultiplos de 7, pois 42 e 49 sao os u
nicos m
ultiplos de 7 na
tabela;
ultiplos de 7, porque 1 e 7 sao os u
nicos divisores de 49
na 2a linha estarao os m
menores do que 12;
ao os m
ultiplos de 2, pois 2 e o u
nico divisor comum de 22
na 3a coluna aparecer
e 24 diferente de 1;
ao os m
ultiplos de 11, pois 22 = 2 11 e os m
ultiplos de 2 j
a
na 3a linha aparecer
a
estao na 3 coluna;
ao os m
ultiplos de 6, pois os divisores de 42 = 2 3 7
na 6a linha aparecer
menores do que 12 e diferentes de 1 sao 2, 3, 6 e 7. Os m
ultiplos de 2 e 7 j
a estao
em seus respectivos lugares. Faltam os m
ultiplos de 3 e 6. Os u
nicos m
ultiplos de
6 na tabela s
ao 24 e 42, e 24 j
a aparece na 5a linha.
Como 15 = 3 5 e os divisores comuns de 32 e 40, menores do que 12 e diferentes
de 1, sao 2 (j
a colocado na tabela), 4 e 8, ate o momento temos a seguinte situacao:
2

10
40

14

70

49

63

11

22

110

77

99

12

24

120

84

108

12

60

42

54

4 ou 8

4 ou 8
32

3 ou 5

3 ou 5

15

Examinemos agora as possibilidades:


I - Repeticao de 2 n
umeros: 30 e 60

II - Repeticao de 3 n
umeros: 24, 30 e 60

8
32

5
20

2
8

10
40

7
28

9
36

4
32

5
40

2
16

10
80

7
56

9
72

56

35

14

70

49

63

28

35

14

70

49

63

11

88

55

22

110

77

99

11

44

55

22

110

77

99

24

15

30

21

27

12

15

30

21

27

12

96

60

24

120

84

108

12

48

60

24

120

84

108

48

30

12

60

42

54

24

30

12

60

42

54

OBMEP 2009

41

5
9

Solucoes do Nvel 1

III - Repeticao de 2 n
umeros: 12 e 40

Lista 3

IV - Repeticao de apenas um n
umero: 24

8
32

3
12

2
8

10
40

7
28

9
36

4
32

3
24

2
16

10
80

7
56

9
72

56

21

14

70

49

63

28

21

14

70

49

63

11

88

33

22

110

77

99

11

44

33

22

110

77

99

40

15

10

50

35

45

20

15

10

50

35

45

12

96

36

24

120

84

108

12

48

36

24

120

84

108

48

18

12

60

42

54

24

18

12

60

42

54

Logo, a u
nica solucao e a tabela IV.
5. Habitantes e esporte Dos dados na tabela temos 8 563 + 8 322 = 16 885 pessoas
que n
ao praticam esporte. Logo, a cidade tem 16 885 5 = 3 377 pessoas que praticam
esporte regularmente, e portanto 3 3771 252 = 2 125 pessoas do sexo feminino praticam
esporte regularmente.
Note que o n
umero de pessoas que praticam esporte somente no fim de semana e
divisvel por 15 e por 9. Logo, precisamos encontrar o maior n
umero, n
ao superior a
30 000, m
ultiplo de 15 e 9. Este n
umero deve terminar em 0 ou 5 e a soma de seus
algarismos deve ser um m
ultiplo de 9. Como 29 970 e o n
umero mais proximo de 30 000,
menor do que 30 000 e m
ultiplo de 5 e 9, podemos assumir que ele e a populacao total
da cidade.
2

29 970 = 3 996 e 29 970 = 6 660 sao as mulheres e os homens, respectiLogo,


15
9
vamente, que praticam esporte somente nos finais de semana.

42

OBMEP 2009

Lista 4

Solucoes do Nvel 1

Lista 4
1. Bot
oes luminosos A resposta correta e (C).
A tabela mostra a cor de cada bot
ao em cada etapa.

incio
apertando botao 1
apertando botao 3
apertando botao 5

1
azul
verde
verde
verde

2
azul
verde
azul
azul

3
azul
azul
verde
verde

4
azul
azul
verde
azul

5
azul
azul
azul
verde

6
azul
azul
azul
verde

7
azul
azul
azul
azul

8
azul
verde
verde
verde

Logo, os botoes que ficaram com luzes verdes acesas no final sao 1, 3, 5, 6 e 8, o que
nos d
a um total de 5 bot
oes.
2. Qual
e o n
umero? O problema e determinar os algarismos
1abcde
3
a, b, c, d e e tais que o n
umero a b c d e 1 seja o triplo de
.....................................................................
abcde1
1abcde :
De incio vemos que e = 7, e a partir da podemos ir descobrindo cada um dos algarismos:
1abcd7
3
.....................................................................
abcd71

1abc57
3
.....................................................................

abc571

1ab857
3
.....................................................................

ab8571

1a2857
3
.....................................................................

a28571

Portanto, a = 4 e o n
umero de partida e 142 857.
3. Jardim variado Os tri
angulos 1, 2, 5 e 6
sao ret
angulos, logo para calcular suas areas vamos enxergar cada um deles como metade de
um ret
angulo.
Para isso precisamos saber dividir o terreno retangular em retangulos menores,
de modo que nossa estrategia funcione: subdividimos o terreno em 16 retangulos de 15 m por
40 m, como mostra a figura. Cada um desses
retangulos tem 1540=600 m2 de area.
Temos entao:

area do tri
angulo 1 = area do tri
angulo 5 =

1
4 600 = 1 200 m2
2

1
6 600 = 1 800 m2
2
1
area do tri
angulo 6 = 2 600 = 600 m2 .
2
area do tri
angulo 2 =

Observe que a area do tri


angulo 4 e a metade da area do terreno todo subtrada das
areas de 3 tri
angulos: tri
angulo 5, tri
angulo 6 e um tri
angulo formado por metade de 4
desses retangulos menores, temos entao:
area do tri
angulo 4 =

4 600
120 80 
1 200 + 600 +
= 4 800 3 000 = 1 800 m2 .
2
2
OBMEP 2009

43

5
9

Solucoes do Nvel 1

Lista 4

Finalmente, a area do tri


angulo 3 e a area total do terreno subtrada da soma das areas
j
a calculadas dos outros 5 tri
angulos
120 80 (2 1 200 + 2 1 800 + 600) = 9 600 6 600 = 3 000 m2 .
Para que o gasto seja o menor possvel, as flores mais caras devem ser plantadas nas
menores regi
oes. Assim, a menor regiao e a 6, onde deve ser plantada a flor mais cara,
rosa, gastando 3,50 600 = 2 100,00. A maior regi
ao e a 3 onde deve ser plantada a
flor mais barata, bem-me-quer, gastando 0,80 3 000 = 2 400,00.
As regi
oes 1 e 5 com areas iguais a 1 200 m2 devem ser plantadas bromelias e cravos,
tendo os gastos: (3,00 + 2,20) 1 200 = 6 240.
As regi
oes 2 e 4 com areas 1 800 m2 devem ser plantadas margarida e violeta com gasto
de (1,20 + 1,70) 1 800 = 5 220. Temos entao 4 diferentes maneiras de formar o jardim
mantendo o mesmo preco mnimo.
O gasto mnimo e 2 100 + 2 400 + 6 240 + 5 220 =R$ 15 960, 00. Veja a seguir uma das
4 possibilidades de escolhas das flores com o menor orcamento.

Regiao
1
2
3
4
5
6

Area
m2
1 200
1 800
3 000
1 800
1 200
600

Flor
bromelia
margarida
bem-me quer
violeta
cravo
rosa

Preco m2
3,00
1,20
0,80
1,70
2,20
3,50

Total por flor


3,00 1 200 = 3 600
1,20 1 800 = 2 160
0,80 3 000 = 2 400
1,70 1 800 = 3 060
2,20 1 200 = 2 640
3,50 6 = 2 100
TOTAL: 15 960

4. O algarismo 3 Vejamos cada vez que Luis escreveu o algarismo 3:


3 1;
13 , 23 , 30 , 31 , 32 , 33 , . . . , 39 , 43 , . . . , 93 2 + 6 + 11 = 19;

 


   
2

11

Ate aqui ele escreveu 20 vezes o algarismo 3. Da temos:


113 , 
123 , 
130 , 
131 .
103 , 

21a

22a

23a

24a

25a

Logo, ao escrever o n
umero 131, ele escreveu o algarismo 3 pela 25a vez.
44

OBMEP 2009

Lista 4

Solucoes do Nvel 1

5. Soma de pot
encias Existe um padrao para o algarismo das unidades de uma potencia
de 3: ele tem perodo 4, pois se repete de 4 em 4 vezes.
3
32 = 9
33 = 27
34 = 81
35 = 243
36 = . . . 9
37 = . . . 7
38 = . . . 1
Como 444 e m
ultiplo de 4, o algarismo das unidades de 3444 e 1.
Analogamente, o algarismo das unidades de potencias de 4 tem perodo 2. De fato
temos:
41 = 4 ; 43 = 64
42 = 16 ; 44 = 256
Como 333 e mpar, o algarismo das unidades de
4333 e 4. Portanto, o algarismo das unidades de
3444 + 4333 e 1 + 4 = 5, e logo ele e divisvel
por 5.

OBMEP 2009

Lembre: Os n
umeros divisveis por 5 terminam em 0 ou
em 5.

45

5
9

Solucoes do Nvel 1

Lista 5

Lista 5
1. Telefonemas Uma vez que Joao liga para seus pais a cada 3 dias, podemos montar
uma tabela que indica os dias da semana em que ocorreram os 14 primeiros telefonemas
do Jo
ao:
Domingo

Segunda

Terca

Quarta

Quinta

Sexta

S
abado

1o

6o

4o

2o

7o

5o

3o

8o

13o

11o

9o

14o

12o

10o

Analisando a primeira linha dessa tabela percebemos que s


ao 7 telefonemas, 1 em cada
o
dia da semana e que, a partir do 7 telefonema, os dias comecam a se repetir. Isto implica
que, os n
umeros que aparecem na segunda linha da tabela s
ao obtidos dos n
umeros que
aparecem na primeira linha somados de 7.
Por exemplo, Joao telefonar
a para seus pais aos domingos nos telefonemas de n
umeros:
1
1+7=8
8 + 7 = 15
15 + 7 = 22
22 + 7 = 29
29 + 7 = 36
..
.
ou seja, nos n
umeros que deixam resto 1 quando divididos por 7.
Com este raciocnio podemos determinar o dia da semana que cai uma ligacao, analisando
o resto da divis
ao do n
umero do telefonema por 7.
Domingo
1
8
..
.

Segunda
6
13
..
.

Terca
4
11
..
.

Quarta
2
9
..
.

Quinta
7
14
..
.

Sexta
5
12
..
.

S
abado
3
10
..
.

resto 1

resto 6

resto 4

resto 2

resto 0

resto 5

resto 3

Dividindo 100 por 7, obtemos 100 = 7 14 + 2. Logo, o resto da divis


ao de 100 por 7
e 2, e segue que o 100o telefonema sera numa quarta-feira.
2. O maior produto Observe que obtemos o maior resultado possvel se um dos n
umeros
comecar com o algarismo 5 e o outro com 4. Vejamos as possibilidades que d
ao o maior
produto:
46

OBMEP 2009

Lista 5

Solucoes do Nvel 1
um dos fatores tem 1 algarismo:
5 3214 = 21 284 ; 4 3215 = 21 605
um dos fatores tem 2 algarismos:
53241 = 21 812 ; 53142 = 22 302 ; 52143 = 22 403

bom usar
E
uma calculadora.

43251 = 22 032 ; 43152 = 22 412 ; 42153 = 22 313.


Logo, o melhor resultado e 431 52 = 22 412.
3. O caminho da Joaninha Os n
umeros primos que aparecem na tabela sao: 23, 73, 37, 17,
79, 19, 37, 53 e 251. Logo, o caminho a ser percorrido pela Joaninha e apresentado na
figura a seguir:
P

4. O lugar dos amigos


Observe que 3 e o u
nico n
umero dentro das tres figuras,
e 1 e o u
nico que nao esta dentro de um polgono, logo:
Celina 3; Fabio 1.

Agora, 4 e o u
nico n
umero dentro do tri
angulo e do
crculo, logo: Elisa 4. Nessa situacao, 5 e o u
nico dentro do tri
angulo mas n
ao do quadrado, assim Diana 5.

Finalmente, 7 e o u
nico n
umero dentro de uma u
nica
figura, logo: Bento 7. Resta entao, 2 dentro do
crculo, assim Guilherme 2, e 6 para Ana.

...............................
.....
........
....
....
...
..
....
................................................................
..
..
...
...
..
...
...
...
..
...
...
... .........
.... ....
..
..
...
.
.
...
.
...
...
.
.
...
.
.
.... .
..
......
.
.
...
.
...
.
.
... .
. ...
.
.
....
.
.
.
...
.
.
..... ..... ...
........
.
.
...
.
.........
.
... ....
.
.
.
.
.
.
.
.
.
.
.
.
.
.
... .............................................................................................
.
.
..
....
.
.
.
....
..
.
.
.
............................................................................................
...................................
.....
.......
....
.....
.
....
................................................................
....
...
....
....
....
..
...
... .....
..
..
..
...
... .... .....
..
..
.
.
.
.
.
.
...
.
...
...
.
.....
.
.
.
.
...
.
...
.
.... ..
... ...
.
...
.
.
...
.
.....
.... .... ...
.
.
.
.
...
.... ...
... ......
.
..
.......
.
.
.
....
.
.
....
. ................................
.
.
.
..................................................................
...
.
.
....
..
.
.
.
..
.
.............................................................................................
.
......................................
.
.
.
.
.
.
....
....
....
...
...
.................................................................
...
...
....
....
..
..
..
....
..
...
... ...........
...
..
..... ...
..
.
...
.
....
...
..
.
.
.
...
.
.
.
... ..
...
.. ...
.
.
.
...
.
.
.
......
.
...
.... ..... ....
...
.
.. .....
.... ...
.
.
.
.
...
.
.
....
.........
. ............
.
.
.
.
.
.
.
.
.
...
.
................
...
..
.
.
.
.
.
.
.
.
.
.
.
.
.
.
.
.
.
.
.
.
.
.
.
.
.
.
.
.
.
.
.
.
.
.
.
.
.
.
.
.
.
.
.
.
.
.
.
.
.
.
.
.
.
.
.
.
.
.
.
.
.
.
.
.
.... .
...
.
.
....
..
.
.
.
.
..........................................................................................

4
5

E
D

E
D

5. Quadrado perfeito? Lembre que um n


umero e um quadrado perfeito se na sua
decomposicao em fatores primos os expoentes sao todos pares. Por exemplo:
54 76 132 e quadrado perfeito, pois e igual a (52 73 13)2 .
OBMEP 2009

47

5
9

Solucoes do Nvel 1

Lista 5

Como nenhum n
umero elevado ao quadrado termina em 3, segue que N1 = 333 . . . 3
n
ao e um quadrado.
ao na decomTemos que N2 = 666 . . . 6 = 2 333 . . . 3. Como 333 . . . 3 e mpar, ent
posicao de N2 em fatores primos nao aparece 2 com expoente par. Logo, N2 n
ao e
quadrado.
Vejamos a divisibilidade por 3. A soma dos algarismos de cada um dos n
umeros e:
N3 ; 50 15 = 750
N4 ; 50 21 = 1 050
N5 ; 50 27 = 1 350
Como todas essas somas sao divisveis por 3, todos os n
umeros tambem sao divisveis
por 3. Logo, se algum deles fosse um quadrado perfeito teria que ser divisvel por 9.
A soma dos algarismos de N3 e N4 n
ao e divisvel por 9, logo esses n
umeros nao s
ao
divisveis por 9 e, consequentemente, n
ao s
ao quadrados perfeitos.
Como 1 350 e divisvel por 9, entao N5 e divisvel por 9. Temos:
2727272727 . . . 27 9 = 303030 . . . 03
e
303030 . . . 03 3 = 101010 . . . 01,
logo:

2727272727 . . . 27 = 32 303030 . . . 03 = 33 101010 . . . 01.

Note que 101010 . . . 01 tem 49 algarismos, dos quais 25 s


ao iguais a 1 e os outros iguais
a 0. Logo a soma de seus algarismos e 25 e portanto n
ao e divisvel por 3. Assim,
ao por 34 , e por isso conclumos que n
ao e um
2727272727 . . . 27 e divisvel por 32 mas n
quadrado perfeito.

48

OBMEP 2009

Lista 6

Solucoes do Nvel 1

Lista 6
1. Preenchendo quadradinhos A operacao e equivalente a


= 4

Logo, o lado esquerdo da igualdade e um m


ultiplo de 4, portanto as u
nicas possibilidades
sao:


2 = 4 1

ou

1 = 4 2

Da, podemos concluir que:




3 + 5 6

2 = 4 1

ou

6 + 5 3

1 = 4 2

2. Os 3 n
umeros Como 13 983 termina em 3, a
soma dos algarismos das unidades dos 3 n
umeros
deve ser 13, e para isso so temos uma opcao: 2 +
4 + 7 = 13 .

2
4
7
.................................................................................................................................................

Agora, a soma dos algarismos das dezenas deve ser 8 1 = 7, e logo tem de ser
1 + 2 + 4 = 7. Completamos os algarismos das dezenas, tendo o cuidado de n
ao repetir
o mesmo algarismo num mesmo n
umero. Temos tres opcoes:
1

.................................................................................................................................................

.................................................................................................................................................

.................................................................................................................................................

Os algarismos das centenas devem somar 9, a temos duas opcoes: 4 + 4 + 1 e 1 + 1 + 7.


Como nas tres possibilidades anteriores o algarismo 4 ocorre em dois dos tres n
umeros,
escolhemos a segunda opcao, para que n
ao apareca o algarismo 4 repetido. Temos de
tomar cuidado para que 1 e 7 tambem nao aparecam repetidos.
OBMEP 2009

49

5
9

Solucoes do Nvel 1
1

.................................................................................................................................................

Lista 6

.................................................................................................................................................

Finalmente, os algarismos das unidades de milhar devem somar 13, agora e f


acil escolhelos:
1

.................................................................................................................................................

.................................................................................................................................................

3. Preencher uma tabela Existem varias maneiras de preencher a tabela, dependendo


de como selecionamos a casa a ser preenchida. A cada vez temos varias casas que podem
ser preenchidas.
Veja um exemplo de como preencher a tabela: inicialmente
temos 4 casas que podem ser preenchidas marcadas com X.
Escolhemos uma delas e preenchemos de acordo com a segunda
X X X
regra, e repetimos esse processo ate a tabela estar completa1 2 X
mente preenchida.

3
1

3
1

4
2

3
1

4
2

Mas para colocar em cada casa o maior n


umero possvel, a ideia e a cada vez examinar
todas as casas que podem ser preenchidas, e s
o preencher a casa onde podemos colocar o maior n
umero. Se em duas dessas casas o n
umero a ser colocado e o mesmo,
preencheremos a que tem o menor n
umero de vizinhos preenchidos. Vamos la!

50

3
1

3
1

6
2

9
3
1

18
6
2

27
9
3
1

54
18
6
2

27
9
3
1

54
18
6
2

27
9
3
1

54
18
6
2

72
144

9
3
1

6
2
54
18
6
2

72

27
9
3
1

27
9
3
1

54
18
6
2

72
144

72
144

OBMEP 2009

216

216
432

Lista 6

Solucoes do Nvel 1
27
9
3
1

54
18
6
2

72
144

216
432
576

27
9
3
1

54
18
6
2

72
144
1178
3516

216
432
576
1754

27
9
3
1

54
18
6
2

72
144
1178

216
432
576

27
9
3
1

54
18
6
2

72
144
1178

216
432
576
1754

Logo, o maior n
umero e 3 516.
4. Olimpada de Pequim Para iniciar, escolhemos um lugar para Maria.

......................................................................
...
...
..
..
....
.....
.....
...
...
..
..
...
...
..
..
.
...
.
...
...
...
...
..................................................................

Maria

(a) Quem pratica natacao esta a` esquerda de Maria. Logo, s


o podemos ter a configuracao abaixo.

......................................................................
..
...
....
...
...
...
...
...
...
...
...
...
...
...
...
....
....
...
...
...
..................................................................

Maria

Natac
ao

(b) Quem pratica ginastica esta na frente de Juan. Existem duas u


nicas possibilidades:
Maria pratica ginastica ou Maria n
ao pratica gin
astica.
Maria pratica gin
astica

Juan

.................................................................................
...
...
...
...
..
..
...
...
..
..
...
...
...
...
.....
.....
...
...
...
...
..
..
...
..
................................................................................

Maria n
ao pratica gin
astica
Gin
astica

.................................................................................
...
...
...
...
..
..
...
...
..
..
...
...
...
...
.....
.....
...
...
...
...
..
..
...
..
................................................................................

Maria
Gin
astica

Natac
ao

Maria

Natac
ao
Juan

OBMEP 2009

51

5
9

Solucoes do Nvel 1

Lista 6

(c) Como Tania e David sentaram-se juntos, ent


ao somente a segunda opcao do item
anterior Maria n
ao pratica gin
astica - pode satisfazer essa condicao. Ela gera as
seguintes duas possibilidades:
Maria n
ao pratica gin
astica

Maria n
ao pratica gin
astica
T
ania
Gin
astica

David
Gin
astica

T
ania

.................................................................................
...
..
...
....
...
...
..
..
...
...
..
..
..
..
....
....
..
..
.
.
.....
.....
...
...
..
.
................................................................................

David

Maria

Natac
ao
Juan

.................................................................................
...
..
...
....
...
...
..
..
...
...
..
..
..
..
....
....
..
..
.
.
.....
.....
...
...
..
.
................................................................................

Maria

Natac
ao
Juan

(d) Como uma mulher sentou-se ao lado de quem pratica v


olei, a segunda opcao do
item anterior e a correta, e temos:
Maria n
ao pratica gin
astica

Maria n
ao pratica gin
astica

T
ania
Gin
astica

David
Atletismo

.................................................................................
...
...
...
...
...
...
..
..
...
...
.
.
....
....
...
...
...
...
...
...
...
...
....
....
...
.
.............................................................................

T
ania
Gin
astica

Maria
V
olei

David
V
olei

Natac
ao
Juan

.................................................................................
...
...
...
...
...
...
..
..
...
...
.
.
....
....
...
...
...
...
...
...
...
...
....
....
...
.
.............................................................................

Maria
Atletismo

Natac
ao
Juan

Logo, David ou Maria pratica atletismo.


5. Culturas diferentes
(a) (i) 03/12 significa para Ralph 12 de marco e para Jorge 3 de dezembro; logo e
ambgua.
(ii) 18/08 so pode ser mesmo 18 de agosto.
(iii) 05/05 so pode ser 5 de maio.
Logo, (i) e uma data que eles nao podem se escrever.
(b) A data s
o e ambgua quando o n
umero do dia pode representar tambem um n
umero
do mes, logo quando e um n
umero de 1 a 12. Por outro lado, nesses n
umeros nao
h
a ambiguidade quando o n
umero do mes e igual ao n
umero do dia, por exemplo
05/05, que s
o pode ser 5 de maio. Por isso, em cada mes eles tem de evitar 11
dias. Logo, os perodos mais longos que eles nao podem se escrever ocorrem em
11 dias consecutivos em janeiro de 02 a 12 de janeiro, e em dezembro, de 02 a 12
de dezembro. Observe que nos outros meses os perodos que eles nao podem se
escrever sao menores, veja os exemplos:
em abril eles nao podem se escrever de 01/04 a 03/04 e depois de 05/04 a
12/04.
em setembro eles nao podem se escrever de 01/09 a 08/09 e depois de 10/09
a 12/09.

52

OBMEP 2009

Lista 7

Solucoes do Nvel 1

Lista 7
1. Uma liquidac
ao Na liquidacao, exceto nos sabados, os precos estao valendo 50% dos
precos originais. Nos sabados, com o desconto adicional de 20%, os precos valem 80%
dos precos fora dos sabados, ou seja
80% de 50% =

50
40
80

=
= 40% do preco fora de liquidacao .
100 100
100

Logo, Roberta deixou de economizar 60% que corresponde a R$ 50,40. Temos:


50, 40
50, 40 6 = 8, 4

60%
10%

100% 8, 4 10 = 84 .
O preco da calca antes da liquidacao era de R$ 84,00.
2. N
umero com muitos zeros A resposta correta e (D).
Vamos tentar comparar os 5 n
umeros sem efetuar calculos. Temos:
3 + a = 3,000 . . . 0001 e menor do que 4
3a
3a

e menor do que 3
= 0, 000 . . . 0003 e menor do que 1

3
a

a
3

3
0, 000 . . . 0001

0, 000 . . . 0001
3

e menor do que 0, 000 . . . 0001 .

Logo, o maior n
umero e

3
1

= 3 102010 e maior do que 10

102010

3
.
a

3. Corrida das tartarugas Vamos representar cada tartaruga numa reta, utilizando a
sua letra inicial. Temos ent
ao a seguinte situacao:
25



Rs Es

5 


Ss Ps Os
  

10

25

Logo, Sininha esta 20 m `


a frente de Elzinha. Portanto, Pulinha est
a 5 m `a frente de
Sininha. A ordem de chegada forma a palavra: OPSER.
nica potencia de 5 com
4. Que mem
oria... O n
umero comeca com 25 porque 52 e a u
2 algarismos.
2

OBMEP 2009

53

5
9

Solucoes do Nvel 1

Lista 7

Os candidatos aos 2 u
ltimos algarismos s
ao as potencias de 2 com 2 algarismos: 16, 32
e 64:
25 16, 25 32, 25 64.
Ja o algarismo do meio pode ser 3, 6 ou 9. Para escolher entre esse n
umeros, lembremos
que a soma dos 5 algarismos e mpar, e como 2 + 5 e mpar, a soma dos 3 u
ltimos tem
de ser par. Nessa situacao temos os n
umeros
25 316, 25 916, 25 332, 25 932, 25 664.
Dentre esses n
umeros os que n
ao tem algarismos repetidos sao
25 316, 25 916 .
Logo, o codigo e 25 916.
5. Uma frac
ao irredutvel Para que a fracao seja irredutvel, o numerador e o denominador n
ao podem ter fator comum.
Inicialmente, vamos ver quais sao os fatores primos de N = 2 3 4 5 . . . 10:
6 7 
8 
9 
10 .
2 3 
4 5 
23

22

23

32

25

Logo, a decomposicao de N em fatores primos e:


N = 28 34 52 7 ()
Podemos escolher diversas fracoes que satisfazem o problema. Por exemplo:
(i) O numerador tem apenas 1 fator de ():
34
52
7
28
;
;
; 8
.
4
2
8
2
8
4
3 5 7 2 5 7 2 3 7 2 34 52
Nesse caso temos 4 fracoes mais as 4 fracoes inversas, com denominadores com
apenas 1 fator de ().
Nao podemos esquecer do n
umero 1, obtendo as 2 fracoes:
28 34 52 7
1
;
.
28 34 52 7
1
(ii) O numerador tem 2 fatores de ():
28 7
34 7
34 52
52 7
28 34 28 52
;
;
;
;
;
.
52 7
34 7
34 52
28 7
28 52 28 34
Nesse caso temos 6 fracoes.
(iii) O numerador tem 3 fatores de ():
28 34 52 28 34 7 28 52 7 34 52 7
;
;
;
.
7
52
34
28
54

OBMEP 2009

Lista 7

Solucoes do Nvel 1

Ao todo temos 16 fracoes irredutveis.

OBMEP 2009

55

5
9

Solucoes do Nvel 1

Lista 8

Lista 8
1. Transformar em decimal Temos:
2
5
14 20
34
(a) 7 + 16
=
+
=
= 11,3333 . . .
3
12
3
3
3




3
6
5
=5 2
= 5 = 5 1,2 = 3, 8
(b) 5 2
3
5
5
2
2
2
5
10
(c) 1 +
=1+
=1+ =1+2 =1+
= 1 + 1,25 = 2,25 .
3
3
8
8
8
1+
1+
1+4
5
5
2. Uma sequ
encia especial
os n
umeros 1 a 9 ocupam 9 posicoes;
os n
umeros 10 a 99 ocupam 2 90 = 180 posicoes;
os n
umeros 100 a 199 ocupam 3 100 = 300 posicoes; os de 200 a 299 ocupam
3 100 = 300 posicoes; os n
umeros 300 a 399 ocupam 3 100 = 300 posicoes;
etc.
100 , . . . 199 , 200 , . . . , 299 , 300 , . . . , 399 , 400 , . . . , 499 , 500 , . . . , 599 , 600 , . . . , 699

 

 

 

 

 



3100=300

3100=300

3100=300

3100=300

3100=300

3100=300

Assim, os algarismos usados para escrever de 1 a 699 ocupam 9 + 180 + 6 300 = 1 989
posicoes, logo faltam 2 0091 989 = 20 posicoes. Como 20 = 36+2 precisamos ainda
escrever de 700 a 706, obtendo 21 posicoes, com o algarismo 6 ocupando a posicao 21.
Logo o algarismo 0 e que ocupa a 2009a posicao.
3. Cortar um ret
angulo Dividimos o retangulo em 13 7 quadradinhos de 1 cm de lado
cada um. Agora, usamos que
13 = 1 + 3 + 4 + 5 = 6 + 7 = 0 + 13
para obter a divis
ao em 13 retangulos diferentes. Voce pode encontrar outras formas de
fazer essa divis
ao?
6

...............................................................................................................................................................................................................................
..
..
...
...
..
.
...................................................................................................................................................................................................................................
..
..
....
..
...
...
....
...
...
...
...
..
...
...
..
...............................................................................................................................................................................................................................
...
..
....
...
....................................................................................................................................................................................................................................
..
..
..
.
.
...
....
....
....
....
...
...
...
...
...
...
..
...
..
..
..
......................................................................................................................................................................................................................................
...
...
...
...
...
..
...
....
....
....
..............................................................................................................................................................................................................................

1
2
1
2
1

4. Medida de
angulo A resposta correta e (B).

= DOY

. Logo, AOC

= 90 DOY

. Como

+ C OE

= 90 e C OE
Temos que AOC

D OY esta entre 40 e 50 , segue que AOC esta entre 90 50 = 40 e 90 40 = 50 .


56

OBMEP 2009

Lista 8

Solucoes do Nvel 1

5. Permetros e
areas A area do quadrado e
2

( 3 + 3)2 = 3 + 2 3 3 + 32 = 12 + 6 3
e a do ret
angulo:

( 72 + 3 6) 2 = 144 + 3 12 = 12 + 6 3 .
Logo eles tem a mesma area. Vamos agora comparar os permetros. O do quadrado e

4 ( 3 + 3) = 4 3 + 12
e o do retangulo e

2 ( 72 + 3 6 + 2) = 2 (6 2 + 3 6 + 2) = 14 2 + 6 6 .

angulo
Como 4 3 < 6 6 e 12 < 14 2, segue que 4 3 + 12 < 6 6 + 14 2. Logo, o ret
tem o maior permetro.
6. C
alculo de
angulo Como AB = AC, o tri
angulo

ABC e isosceles, logo ABC = ACB. Sendo


AD = BD o tri
angulo ABD tambem e isosceles, logo

= B AD.

ABD
Temos entao

= ACB

= B AD

.
ABC

..........
..... .. .....
..... .. .........
..... ....
...........
....
.
.
.
.
.
.
.
.....
.
.....
.....
..
.....
.....
.
.
.
.
.
.....
..
.....
.....
..
....
.
.....
.
.
.
.
.
.....
.
....
.
.
.
.
.
.....
.
...
.
.
.
.....
.
.
.
.
....
...
.
.
.
.
.
.
....................................................................................................................................................................................

39

Na figura, esses 3 angulos iguais est


ao representados pela letra . Os angulos internos
de ABC sao + 39 , e ; logo:
+ 39 + + = 180 3 = 180 39 = 141 .

= 47 .
Assim, B AD
Lembrete 1: Em um
tri
angulo a soma dos
angulos internos e 180 :

+B

+C

= 180 .
A

Lembrete 2: Em um
tri
angulo is
osceles os
angulos da base s

ao
iguais:

=C

OBMEP 2009

AB = AC .

57

5
9

Solucoes do Nvel 1

Lista 9

Lista 9
1. O caminho da formiga
A resposta correta e (C).

2. Menino mentiroso Claramente Pedrinho encontrou Joaozinho em um dia que ele


mente. O sabado est
a descartado pois, caso contr
ario, ele estaria falando a verdade.
Assim, o encontro entre eles foi numa terca-feira ou quinta-feira. Como o dia seguinte
n
ao pode ser quarta-feira, a u
nica possibilidade e quinta-feira.
3. Encontre os 4 n
umeros Observemos que os n
umeros 1, 2, 3 e 6 satisfazem a propriedade. Portanto, os m
ultiplos a, 2a, 3a e 6a, para qualquer valor de a, tambem satisfazem a propriedade.
Como estamos procurando n
umeros de 3 algarismos e
999 6 = 166,5 entao basta considerar qualquer valor de a entre 100 e 166 para
obter os 4 n
umeros de 3 algarismos.
4. Colando 6 tri
angulos

1/8

1/16 V
IV
1/32

VI

III

1/4

II
1/2

A figura e formada por 12 segmentos, na sequencia de formacao dos tri


angulos.
1
cm no triangulo I.
2
1
1
1 segmento de cm e 1 segmento de cm no triangulo II.
2
4
1
1
1 segmento de cm e 1 segmento de cm no triangulo III.
4
8

2 segmentos de 1 cm e 1 segmento de

58

OBMEP 2009

Lista 9

Solucoes do Nvel 1
1
1
cm e 1 segmento de
cm no triangulo IV.
8
16
1
1
cm e 1 segmento de
cm no triangulo V.
1 segmento de
16
32
1
cm no triangulo VI.
2 segmentos de
32
1 segmento de

Logo o permetro e:
21+2

1
1
1
1
1
+2 +2 +2
+3
2
4
8
16
32

1 1 1
3
+ + +
2 4 8 32
16 + 8 + 4 + 3
= 3+
32
31
= 3+ .
32
= 2+1+

5. Os livros da Elisa Seja N o n


umero total de livros da Elisa. Como N + 1 e m
ultiplo
de 9 e 4, temos que ele e m
ultiplo de 36. Logo N + 1 e 36 ou 72, pois Elisa tem menos
que 100 livros. Se N = 35 entao, o n
umero de livros de matematica e 36 9 1 = 3 e
o n
umero de livros de literatura e 36 4 = 9. Logo, Elisa teria: 24 + 3 + 9 = 36 livros,
o que e impossvel porque 36 e maior que 35.
Portanto, N = 71 e o n
umero de livros de matematica e 72 9 1 = 7.

OBMEP 2009

59

5
9

Solucoes do Nvel 1

Lista 10

Lista 10
1. Divis
ao por 9
(a) Sabemos que um n
umero e a soma de seus algarismos sempre deixam o mesmo
resto quando divididos por 9. Assim, o n
umero inicial menos o n
umero final e
sempre divisvel por 9.
Efetuando, sucessivamente os passos, obtemos os algarismos de 1 a 9. Da, a lista
final e:
1, 2, 3, 4, 5, 6, 7, 8, 9, 1, 2, 3, . . .
Como o resto da divisao do n
umero 20 092 009 por 9 e 4, ent
ao os 6 u
ltimos
algarismos da lista s
ao: . . . , 8, 9, 1, 2, 3, 4. Portanto, a lista tem mais 4 do que 5.
O n
umero de vezes que aparece o 9 na lista, e o n
umero de m
ultiplos de 9, que
sao menores ou iguais a 20 092 009. Como 20 092 005 e o maior m
ultiplo de 9 que
e menor do que 20 092 009, temos que 20 092 005 9 = 2 232 445 vezes aparece o
algarismo 9 na lista.
ao por
(b) Como 32 009 = 32 008 3 = (32 )1 004 3 = 91 004 3, entao, o resto da divis
9 e 0. Logo, o n
umero final de apenas um algarismo e o 9.
ultiplo de 9 + 1. Logo,
(c) Observemos que 172 = m
ultiplo de 9 + 1,
172 008 = (172 )1 004 = m
ultiplo de 9 + 17 = m
ultiplo de 9 + 8.
assim 172 009 = m
Da, podemos concluir que, se fazemos o mesmo processo com o n
umero 172 009
obtemos no final o algarismo 8.
2. Uma brincadeira na sala de aula
(a) O n
umero 1 so pode ser
4 ; 2 = 4 2, mas o
8 ; 4 = 8 2.
Logo, temos 2 maneiras

1 ; 4 ; 2 ;
8 ; 4 ; 2 ;

obtido a partir do 2 ; 1 = 2 2, e o 2 a partir do


4 pode ser obtido a partir do 1 ; 1 + 3 = 4 ou do
de obter 1, a partir de 1 e 8 depois de 3 operacoes:
1
.
1

(b) Para uma operacao a mais vemos que o n


umero 8 pode ser obtido a partir do
5 ; 8 = 5 + 3ou do 16 ; 8 = 16 2. Logo, temos 3 maneiras de obter 1 a partir

2 ; 1 ; 4 ; 2 ; 1
de 2, 5 e 16:
5 ; 8 ; 4 ; 2 ; 1 .

16 ; 8 ; 4 ; 2 ; 1
(c) De maneira
similar vemos
4 ; 2 ;

10 ; 5 ;
ou 32:
13 ; 16 ;

32 ; 16 ;
60

que
1
8
8

para 5
; 4
; 4
; 4

operacoes temos os n
umeros: 4, 10, 13
; 2 ; 1
; 2 ; 1
.
; 2 ; 1

8 ; 4 ; 2 ; 1
OBMEP 2009

Lista 10

Solucoes do Nvel 1

3. Calcule a idade No pr
oximo ano Laura sera 2 anos mais velha do que no ano passado.
Logo sua idade no ano passado e um m
ultiplo de 8 que somado a 2 d
a um m
ultiplo de
7. Vamos procurar esse n
umero:
m
ultiplos de 7 : 7 14 21 28 35

42

49 56 63 70 . . .

98

...

(m
ultiplos de 7) 2 : 5 12 19 26 33

40

47 54 61 68 . . .

96

...

Note que 40 e 96 sao os u


nicos m
ultiplos de 8 menores que 100 que aparecem na segunda
linha. Como Vov
o Ana tem menos do que 100 anos, podemos concluir que ano passado
ela tinha 96 anos e Laura 40. Logo, a idade atual de Laura e 41 anos.
4. Divis
oes e restos De acordo com os dados do problema, o dobro do n
umero e um
m
ultiplo de 5 acrescido de 1. Como os m
ultiplos de 5 terminam em 0 ou 5, o dobro
termina em 1 ou 6. Mas o dobro e um n
umero par, logo termina em 6. Assim, o n
umero
termina em 3 ou 8, portanto dividido por 5 deixa resto 3.
5. Preenchendo o crculo Sabemos que  = 423 47 = 9. Por outro lado, temos que
1448 =

282 

m
ultiplo de 282




no com 2 algarismos

Como 282 tem 3 algarismos, conclumos que   so pode ser o resto da divis
ao de 1 448
por 282. Efetuando essa divis
ao, obtemos 1 448 = 282 5 + 38. Logo,  = 3 e  = 8.
Obtemos tambem que  = 5. Finalmente, temos:
423


= 282 141  = 282  = 2 .
3

A sequencia completa:
 
9

47

2/3

423

 

 
5

+ 38



282

1410 1448

 

OBMEP 2009



61

5
9

Solucoes do Nvel 2

Lista 1

Soluco
es do Nvel 2
Lista 1
1. Vista ruim Seja A o n
umero total de alunos da sala. Logo,
bem. Portanto,

70
100

40
100

40
100

A n
ao enchergam

A usam oculos. Consequentemente, temos que:

40
70

A = 21
100 100

A=

21 100
= 3 25 = 75 .
74

2. Idade m
edia da populac
ao de Campo Verde Se H indica o n
umero de homens e
M o de mulheres, ent
ao:
2
3H
H
=
M=
.
M
3
2
A idade media da populacao e:
37H + 42 3H
100H
100 2
37H + 42M
2
=
= 40 anos .
= 5H =
3H
H +M
5
H+ 2
2

3. Area
de tri
angulo Os tri
angulos ABC e DBC tem bases AC e CD respectivamente, e a mesma altura h em relacao a essas bases.
B

.
..............
............... .. ..
.......... ..... .... .....
........... .........
... ...
.
.
.
.
.
.
.
.
.
.
.
........
... .....
.....
..........
...
.....
...........
....
...
.....
..........
.
.
.
.
.
.
.
.
.
.
...
...
.
.
.
.
.
..
.......
.
.
.
.
.
...
.
.
.
.
...
.
.
.
.
.
.
...
.......
...
.
.
.
.
.
.
.
.
.
.
.
.
.
.
..
...
....
.......
.
.
.
.
.
.
.
.
.
.
.
...
.
.
.
.
.
..
...
........
.
.
.
...
.
.
.
.
.
.
.
.
.
.
.
.
...
...
......
.
.
.
.
.
.
.
.
.
.
.
.
.
.
.
.
.
...
..
...
.......
.
.
.
.
.
.
.
.
...
.
.
.
.
.
.
..
..
........
.
.
...
.
.
.
.
.
.
.
.
.
.
.
.
.
...
..
..
......
.
.
.
.
.
.
.
.
.
.
.
.
.
.
.
...............................................................................................................................................................................................................................................................................

Assim temos:
area ABC =

AC h
2

area DBC =

CD h
.
2

Logo, a relacao entre as areas e dada por:


area ABC
=
area DBC

ACh
2
CDh
2

1, 5
15
3
AC
=
=
= .
CD
4 1, 5
25
5

LEMBRE-SE: A area de um triangulo e a metade do produto de um dos seus lados


pela altura h relativa a este lado, como exemplificado nas duas figuras a seguir.
62

OBMEP 2009

Lista 1

Solucoes do Nvel 2
B

.
........
..... .....
.....
...
.....
.
...
.
.
.
.
...
.....
...
.....
.
.
...
.
.
...
...
.
.
.
.
...
...
.
.
.
.
...
...
.
.
...
.
.
...
...
.
.
.
.
...
...
.
.
.
.
.
...
...
.
.
.
...
.
....
...
.
.
.
.
...
...
.
.
.
...
.
....
.
.
.
.
. ......................................................................................................................................................................

..........
................
........... ....
.......... ..........
.
.
.
.
.
.
.
.
.
.
...
....
..........
.....
..........
.....
...........
.....
...........
....
..........
.
.
.
.
.
.
.
.
.
.
.
.
.
.
.
....
....
..........
.....
...........
......
...........
.....
...........
.....
..........
.
.
.
.
.
.
.
.
.
.
.
.
.
...
.....
...........
.....
..........
.....
...........
.....
...........
................................................................................................................

CD h

Area
doBCD =
2

C
AC h

Area
doABC =
2

4. Construindo quadrados perfeitos Sim, sera sempre um quadrado perfeito. De fato,


se n 1, n, n + 1 e n + 2, sao quatro inteiros consecutivos, entao seu produto mais 1, e
dado por:
(n 1)n(n + 1)(n + 2) + 1 = n(n2 1)(n + 2) + 1
= n(n3 + 2n2 n 2) + 1
= n4 + 2n3 n2 2n + 1
= n4 + 2n3 + (n2 2n2 ) 2n + 1
= (n4 + 2n3 + n2 ) 2n2 2n + 1
= (n2 + n)2 2(n2 + n) + 1
= [(n2 + n) 1]2 .
5. Feira de Ci
encias Sejam x e y o n
umero de alunos do ensino fundamental e do
medio respectivamente, presentes na feira. Logo, o n
umero daqueles que compraram
um adesivo e:
x
do ensino fundamental
2

y
do ensino medio;
4

3y
do ensino medio.
4

e os que nao compraram foram


x
do ensino fundamental
2

Dentre os alunos que nao compraram adesivos, os do ensino medio representam o dobro
dos do ensino fundamental. Logo,
x
x
3y
3y
=2 =
.
4
2
2
8
Sabendo que o total arrecadado foi de R$ 38,00, conclumos que:
38 = 0, 30

y
3y
y
x
+ 0, 50 = 0, 30
+ 0, 50
2
4
8
4

1, 90 y = 8 38
y = 160 .

Agora, de x =

3y
,
4

segue que x = 120.


OBMEP 2009

63

5
9

Solucoes do Nvel 2

Lista 2

Lista 2
1. Par perfeito Chamemos de n o natural candidato a formar um par perfeito com
umeros
122. Entao, devemos ter: 122 + n = A2 e 122 n = B 2 onde A e B sao n
naturais.
Como B 2 = 2 61 n, conclumos que n tem tambem os fatores primos 2 e 61. Logo,
podemos escrever n como n = 2 61 m2 = 122m2 .
Obtemos entao A2 = 122 + 122m2 = 122(1 + m2 ). O menor valor de (1 + m2 ) que
satisfaz esta igualdade e 1 + m2 = 122, ou seja, m2 = 121. Da segue que m = 11.
Consequentemente, n = 122 121 e temos:
A2 = 122 + 122 121 = 1222

e B 2 = 122 122 121 = (122 11)2 .

Logo, 122 e 122 121 formam um par perfeito.


Observac
ao. Na verdade, 122 121 e o menor natural que forma um par perfeito com
122. Ser
a que existem outros?
2. Um trap
ezio A resposta correta e (D).
Seja P o ponto medio do segmento CD e tracemos os segmentos AP e BP . Os tres triangulos formados ADP ,
ABP e BCP sao equil
ateros (porque?). Entao, os

Como o segmento AC
angulos DAP = 60 = P AB.

e a bissetriz do angulo P AB (porque?), conclumos que

= 30 . Portanto:
P AC

.................................................................................
. ..
......
... ....
... ..
.. .....
...
..
..
..
...
...
.
.
.
.
.
.
...
...
...
...
...
...
.
..
...
.
.
.
...
...
..
...
.
.. ..
...
..
...
.
.. .
...
.
...
.
.
...
.
.
... ....
...
...
.
... ..
...
.
....
................................................................................................................................................................

= DAP

+ P AC

= 60 + 30 = 90 .
C AD
3. Mist
erio das bolas Seja m o n
umero de bolas pretas na primeira urna e n o de bolas
brancas na segunda urna. Inicialmente, Henrique retirou k bolas pretas da primeira urna
e as colocou na segunda urna. Nesse ponto a situacao e a seguinte:
k
na 1a urna temos: m
pretas

na

2a

urna temos: 


n + 
k
brancas

pretas

Depois, ele retirou k bolas da segunda urna e as colocou na primeira urna. Agora esse
grupo de k bolas pode ter bolas brancas e pretas. Assim chamemos de p o n
umero de
bolas pretas e de b o de bolas brancas retiradas da 2a urna, e logo k = b + p. Temos
entao:
k + p + 
b = m k + p + 
b
na 1a urna temos: m

  
pretas

na

2a

brancas

pretas

brancas

urna temos: 


n + 
k 
b p = n 
b + k p

  
brancas

64

pretas

pretas

brancas

OBMEP 2009

pretas

brancas

pretas

Lista 2

Solucoes do Nvel 2

Assim, ele ficou com b bolas brancas na primeira urna e k p bolas pretas na segunda
urna. Mas, k = p + b, ou seja, b = k p. Logo, o n
umero de bolas brancas na primeira
urna e igual ao n
umero de bolas pretas na segunda urna.
4. Contando a palavra BRASIL Para ler a palavra BRASIL, devemos percorrer um
caminho que comeca em uma letra B e termina em uma letra L. Observemos que o
caminho a ser percorrido e composto sucessivamente de deslocamentos horizontais para a
direita e verticais para baixo. Assim, vamos representar estes caminhos por sequencias de
letras H (significando deslocamento para a direita) e letras V (significando deslocamento
para baixo).
Vamos ver dois exemplos:
(i) Comecamos em B na segunda linha (de cima para baixo) e seguimos o caminho
VHVVV.
(ii) Comecamos em B na terceira linha e seguimos o caminho HVVHH.
Para resolver o problema devemos contar quantos caminhos comecam com B e terminam
umero de tais caminhos
com L. Para isto, temos que listar esses caminhos. Seja Cj o n
comecando na linha j, onde j varia de 1 a 6:
Linha 1: VVVVV ; C1 = 1;
Linha 2: HVVVV, VHVVV, VVHVV, VVVHV, VVVVH ; C2 = 5;
Linha 3: HHVVV, HVHVV, HVVHV,
VVHVH, VVVHH ; C3 = 10;

HVVVH,

VHHVV,

VHVHV,

VHVVH,

VVHHV,

Linha 4: HHHVV, HHVHV, HHVVH,


VHVHH, VVHHH ; C4 = 10;

HVHHV,

HVHVH,

HVVHH,

VHHHV,

VHHVH,

Linha 5: HHHHV, HHHVH, HHVHH, HVHHH, VHHHH ; C5 = 5;


Linha 6: HHHHH ; C6 = 1.

Portanto, a palavra BRASIL aparece


C1 + C2 + C3 + C4 + C5 + C6 = 1 + 5 + 10 + 10 + 5 + 1 = 32
vezes na figura (Procure entender a simetria: C1 = C6 ; C2 = C5 e C3 = C4 ).
5. Quais s
ao os n
umeros? A equacao pode ser escrita na forma x4 y 2 = 71. Agora,
4
fatorando x y 2 temos:
(x2 y)(x2 + y) = 71 () .
Como x e y sao inteiros, ent
ao cada um dos fatores (x2 y) e (x2 + y) tambem e
um n
umero inteiro. Logo em (*) escrevemos 71 como o produto de 2 n
umeros inteiros.
Como 71 e um n
umero primo, ele s
o pode ser escrito como produto de inteiros na forma:
71 = 1 71. Temos entao dois casos a considerar: (x2 y) = 1 e (x2 + y) = 71 , ou
(x2 y) = 71 e (x2 + y) = 1.
Vamos estudar cada caso.
OBMEP 2009

65

5
9

Solucoes do Nvel 2


1o

caso:

Lista 2

x2 y = 1
.
x2 + y = 71

Somando as duas equacoes obtemos: 2x2 = 72, o que implica x = 6. Portanto,


y = (6)2 1 = 35. Como x, y sao inteiros positivos, conclumos que a solucao nesse
primeiro caso e: x = 6 e y = 35.

2o

caso:

x2 y = 71
.
x2 + y = 1

a que x, y sao inteiros positivos.


Se x2 + y = 1, entao x = 0 e y = 1 ou x = 1 e y = 0 j
Por outro lado, e f
acil verificar que tais valores n
ao satisfazem a equacao x4 = y 2 + 71.
Logo, a solucao para o problema e: x = 6 e y = 35.

66

OBMEP 2009

Lista 3

Solucoes do Nvel 2

Lista 3
1. No jogo Seja T a quantidade total de dinheiro no jogo. Assim, no incio, os jogadores
possuam:
7
T
Aldo:
18
6
T
Bernardo:
18
5
T.
Carlos:
18
No final eles possuam:
Aldo:
Bernardo:
Carlos:

6
T
15
5
T
15
4
T.
15

Para melhor comparar essas fracoes, coloquemo-las com um denominador comum:


No incio:
Aldo:
Bernardo:
Carlos:
No final:
Aldo:
Bernardo:
Carlos:

35
7
T =
T
18
90
30
6
T =
T
18
90
25
5
T =
T.
18
90
36
6
T =
T
15
90
30
5
T =
T
15
90
24
4
T =
T.
15
90

Logo, Carlos perdeu 1/90 do total e Aldo ganhou 1/90. Portanto, 1 220 corresponde a
1/90 do total de dinheiro. Portanto, o total T de dinheiro no incio o jogo e:
1
T = 1 200
90

T = 90 1 200 = 108 000

Assim, no final da partida os jogadores possuiam:


35
de 108 000 = 42 000
Aldo:
90
30
de 108 000 = 36 000
Bernardo:
90
25
de 108 000 = 30 000 .
Carlos:
90
OBMEP 2009

67

5
9

Solucoes do Nvel 2
Lista 3



3
3
2. Um n
umero inteiro Sejam a =
5+2 e b =
5 2. Assim, M = a b
e temos:
M 3 = (a b)3 = a3 b3 3ab(a b) .
Sabemos que a3 b3 = 4 e ab = 1. Assim, M 3 + 3M 4 = 0, ou seja, o n
umero M e
3
raiz do polin
omio x + 3x 4.
omio,
Por sua vez, o n
umero 1 e uma raiz do polin
omio x3 + 3x 4. Fatorando tal polin
2
2
obtemos (x 1)(x + x + 4). Mas o trin
omio x + x + 4 tem discriminante negativo.
Consequentemente, 1 e a u
nica raiz real de x3 + 3x 4. Portanto, M = 1.

3. Area
de tri
angulos
C e AM
D sao angulos opostos pelo vertice.
(a) Note que F M
C = AM
D. Como M C = M D e os triangulos
Logo, F M
AM D e F M C sao ret
angulos, conclumos que eles sao
congruentes. Logo, possuem a mesma area, donde conclumos
que a area do tri
angulo ABF e igual a area do quadrado
ABCD, ou seja 300 cm2 .
(b) Como AD = F C (do item anterior) e DM = M C, segue que os triangulos ADM ,
DM F e M CF tem a mesma area. Por outro lado, a area dos dois u
ltimos e a
metade da area do quadrado. Portanto, a area do tri
angulo ADF e a metade da area
do quadrado, ou seja 150 cm2 .
4. Um quadriculado Sejam m e n respectivamente, o n
umero de segmentos de 0, 5 cm
sobre dois lados consecutivos do ret
angulo. Sabemos que o n
umero total de segmentos de
0, 5 cm na divisao do ret
angulo em mn quadrados de lado 0, 5 cm e: m(n+1)+n(m+1)
(prove isso). Assim,
m(n + 1) + n(m + 1) = 1 997

n=

1 997 m
.
2m + 1

Alem disso, um dos lados considerados e menor ou igual ao outro, digamos: m n .


Nesse caso podemos concluir que m 31, pois
n m n(m + 1) + m(n + 1) 2m(m + 1).
Logo 1 997 2m(m + 1) e como 1 998 > 1 997 segue que
1 998 > 2m(m + 1) 999 > m(m + 1).
Da conclumos que m < 32.
Por outro lado temos que:
n=

3 994 2m
3 995 (2m + 1)
3 995
1 997 m
2n =
=
2n =
1.
2m + 1
2m + 1
2m + 1
2m + 1

Assim, a questao se resume agora em pesquisar os divisores de 3 995 = 5 17 47.


Os u
nicos valores de m que atendem a condicao 1 m 31 sao m = 2, m = 8 e
m = 23, que correspondem, respectivamente, aos divisores 5, 17 e 47. Para esses valores
68

OBMEP 2009

Lista 3

Solucoes do Nvel 2

de m temos n = 399, n = 117 e n = 42 respectivamente. Os outros divisores dar


ao
configuracoes equivalentes (trocando m por n).
Portanto, Rosa pode ter construdo 3 configuracoes diferentes com os 1 997 segmentos.
A primeira com 2 399 quadrados, a segunda com 8 117 quadrados e a terceira com
23 42 quadrados.
5. Inteiros de 4 algarismos Temos que 1 000 4a2 < 10 000 e tambem
4
1 000 a3 < 10 000.
3
umero inteiro e
De 1 000 4a2 < 10 000 segue que 250 a2 < 2 500. Sendo a um n
152 = 225, 162 = 256 e 502 = 2 500, temos que 15 < a < 50.
4
De 1 000 a3 < 10 000 temos 750 a3 < 7 500. Mas, 103 = 1 000, 93 = 729,
3
203 = 8 000 e 193 = 6 859. Assim, 9 < a < 20.
Portanto, temos a = 16, 17, 18 ou 19.
4
umero inteiro, conclumos que a3 e m
ultiplo de 3 e
Por outro lado, como a3 e um n
3
consequentemente, a e m
ultiplo de 3. Logo, a = 18.
Outra maneira de finalizar a solucao e substituir os 4 possveis valores para a e verificar
que o u
nico valor e a = 18.

OBMEP 2009

69

5
9

Solucoes do Nvel 2

Lista 4

Lista 4
3(167 x)

1. Pares positivos A equacao dada e equivalente a y =


5
positivo, 167 x deve ser um m
ultiplo positivo de 5, ou seja:
167x = 5k

x = 1675k

. Como y e um inteiro

x = 533+25k

x = 5(33k)+2

onde k e um inteiro positivo. Como x e positivo, devemos ter k 33. Consequentemente, k = 1, 2, . . . , 33 o que nos garante 33 solucoes para o problema proposto.
2. Diferenca de quadrados A resposta correta e (E).
Inicialmente, observe que o quadrado de um n
umero par e par, e o quadrado de um
n
umero mpar e mpar. Se os dois n
umeros sao consecutivos, entao um n
umero e par
e o outro e mpar. Portanto, elevando ao quadrado, um deles e par e o outro e mpar.
Mas, a diferenca entre um n
umero par e um n
umero mpar e sempre um n
umero mpar.
umeros consecutivos
Como 2 000 e um n
umero par, conclumos que nao existem dois n
cuja diferenca dos seus quadrados seja 2 000.
Outra solucao para o problema e a seguinte. Primeiramente, suponhamos que tais
inteiros a e a + 1 sao maiores ou iguais a zero. Nesse caso, temos:
(a + 1)2 a2 = 2 000 .
Fatorando a diferenca de quadrados (a + 1)2 a2 obtemos:
(a + 1 + a)(a + 1 a) = 2 000

2a + 1 = 2 000

que n
ao tem solucao pois 2a + 1 e mpar e 2 000 e par.
Se a e a + 1 fossem menores ou iguais a zero entao a e a 1 seriam inteiros maiores
ou iguais a zero e sucessivos, satisfazendo a condicao (a)2 (a 1)2 = 2 000 o que
n
ao pode ocorrer como provado acima.
3. C
alculo de
angulos Na primeira figura, prolongue o segmento BC ate que ele intercepte o segmento ED em um ponto F . Uma vez que os segmentos AB e ED sao

e B F
D sao alternos internos. Isto implica que esses angulos
paralelos, os angulos ABF
possuem a mesma medida, ou seja, C F
D = 25o . Agora vemos que o angulo x e externo
ao tri
angulo CDF . Logo, x e igual a soma dos dois angulos internos n
ao adjacentes,
o
o
o
ou seja, x = 25 + 55 = 80 .

A...............................................................................................................................B
...
....
....
.
.
.
....
...
....
...
.........
.
.
.
. ...
....
....
....
....
....
....
....
....
...
....
.
.
..
..
.
.
.
55o .......
..
.
.
.
..............................................................................................................................

25o ........

70

A...............................................................................................................................B
.
...

....
160o ........

....
....
....
....
....
....
.
... ....
.... .......
.
.
.
....
..
.
.
.
....
...
.
.
....
.
.
....
....
....
o .......
....
150 ....
.......................................................................................................................................................................................................................................

OBMEP 2009

Lista 4

Solucoes do Nvel 2

Na segunda figura, tambem prolongue o segmento BC ate que ele intercepte o prolongamento do segmento ED em um ponto F . Como os segmentos AB e EF sao paralelos,

e DF
B sao colaterais internos. Isto implica que esses angulos s
os angulos ABF
ao
suplementares, ou seja,
DF
C = 180o 160o = 20o .

e igual a 30o , pois ele e o suplemento do angulo


Por outro lado, o angulo C DF
o

= 150 . Finalmente, como x e angulo externo ao tri


E DC
angulo CDF temos que:
o
o
o
x = 20 + 30 = 50 .
4. Tabela Como a tabela tem 6
colunas, em cada linha escrevemos
6 n
umeros consecutivos. Dividindo
1 000 por 6 obtemos
1 000 = 6 166 + 4 .

1a linha
2a linha
3a linha
..
.
167a linha
..
.

1
7
13
..
.
997
..
.

2
8
14
..
.
998
..
.

3
9
15
..
.
999
..
.

4
10
16
..
.
1 000
..
.

5
11
17
..
.

6
12
18
..
.

..
.

..
.

Deste modo, para escrever o n


umero 1 000 na tabela ser
ao necessarias 166 linhas completas (terminando no n
umero 6 166 = 996) e mais uma linha com os 4 n
umeros:
997, 998, 999 e 1 000. Logo, 1 000 esta escrito na 167a linha e na 4a coluna.
5. Entre 1 e 2 Como as duas fracoes sao positivas e menores do que 1, seus numeradores
devem ser respectivamente menores que seus denominadores, logo devemos ter:
0 < a < 5 e 0 < b < 7 (1)
Temos

a
5

b
7

7a + 5b
35

, portanto:

1<

7a + 5b
< 2 35 < 7a + 5b < 70 (2)
35

Vejamos as opcoes para que a e b sejam inteiros positivos e satisfacam (1) e (2):
a = 1 7a + 5b = 7 + 5b. Logo,
35 < 7 + 5b < 70 28 < 5b < 63

63
28
<b<
5, 6 < b < 12, 6 .
5
5

Como b e um n
umero inteiro, conclumos que b = 6, 7, 8, . . . , 12. No entanto, s
o
podemos escolher b = 6, pois b < 7.
a = 2 7a + 5b = 14 + 5b. Logo,
35 < 14 + 5b < 70 21 < 5b < 56

56
21
<b<
b = 5, 6, 7, 8, . . . , 11 .
5
5

Nesse caso, so podemos escolher b = 5 e b = 6, pois b < 7.


a = 3 7a + 5b = 21 + 5b. Logo,
35 < 21 + 5b < 70 14 < 5b < 49

49
14
<b<
b = 3, 4, 5, 6, . . . , 9 .
5
5

Aqui, podemos escolher b = 3, 4, 5, 6.


OBMEP 2009

71

5
9

Solucoes do Nvel 2

Lista 4

a = 4 7a + 5b = 28 + 5b. Logo,
42
7
<b<
b = 2, 3, 4, 5, 6, . . . , 8 .
5
5

35 < 28 + 5b < 70 7 < 5b < 42


Podemos escolher b = 2, 3, 4, 5, 6.

Para finalizar, exibimos as solucoes na tabela abaixo:


a

5
6

3
4
5
6

2
3
4
5
6

72

1
5
2
5
2
5
3
5
3
5
3
5
3
5
4
5
4
5
4
5
4
5
4
5

a
5

+
+
+
+
+
+
+
+
+
+
+
+

+
6
7
5
7
6
7
3
7
4
7
5
7
6
7
2
7
3
7
4
7
5
7
6
7

OBMEP 2009

b
7

=
=
=
=
=
=
=
=
=
=
=
=

37
35
39
35
44
35
36
35
41
35
46
35
51
35
38
35
43
35
48
35
53
35
58
35

Lista 5

Solucoes do Nvel 2

Lista 5
1. Triatlon Seja x a velocidade em metros por minuto com que Maria nada. Logo, a
sua velocidade na corrida e 3x e na bicicleta 2,5 3x = 7,5x. Logo, o tempo total que
ela gastar
a nas 3 etapas e:
800
60 000
20 000
4 000
800 7,5 + 20 000 2, 5 + 4 000
=
.
+
+
=
x
3x
7,5x
7,5
x
7,5 x

  
  

nadando

correndo

pedalando

Logo, para que ela venca as 3 etapas em 1 hora e 20 minutos (=80min), devemos ter:
60 000
60 000
= 80 x =
= 100 m/min .
7,5 x
7,5 80
Segue que
3x = 300 m/min e 7,5x = 750 m/seg .
Assim, para que Maria termine a prova em no maximo 1 hora e 10 minutos, ela deve
desenvolver as seguintes velocidades:
nadar: a uma velocidade mnima de 100 m/min;
correr: a uma velocidade mnima de 300 m/min;
pedalar: a uma velocidade mnima de 750 m/min.
2. Foto de formatura As figuras a seguir representam a situacao do problema, onde
em preto estao representados os alunos que foram inicialmente retirados e em cinza os
alunos retirados na segunda vez.
d d d. . . d. . . d d d d t
d d d. . . d. . . d d d d t

.. .. ..
. . .

..
.

.. ..
. .

.. .. ..
. . .

d d d. . . d. . . d d d d t

d d d. . . d d d d d
d d d. . . d d d d d

d d d. . . d d d d td
d d d. . . d d d d td

d d d. . . d d d d d
t t t. . . t

d d d. . . d d d d td
t t t ...t

.. .. ..
. . .

.. .. .. .. ..
. . . . .

.. .. ..
. . .

.. .. .. .. ..
. . . . .

Sejam n e m o n
umero de filas (linhas horizontais) e de colunas da formacao inicial,
respectivamente. Ao retirar 1 aluno de cada fila, o diretor obtem uma formacao com
uma coluna a menos e uma fila incompleta faltam 4 alunos. Logo, podemos concluir
que:
n + 4 = m 1 m = n + 5.
Retirando agora mais um aluno de cada fila obtem-se
uma formacao retangular com 2 colunas a menos que a
formacao inicial. Logo, podemos concluir que o n
umero
de filas (iniciais) e n = 3. Assim, m = 8 e o n
umero de
alunos e dado por n m = 3 8 = 24.
OBMEP 2009

d d d. . . d d d d
d d d. . . d d d d

.. .. ..
. . .

.. .. .. ..
. . . .

d d d. . . d d d d
t t t ...t t t t

73

5
9

Solucoes do Nvel 2

Lista 5

3. Circunfer
encias tangentes
......................................................
.......... .....
... .......
...... .....
...
.....
.....
.
.....
...
....
..
.
.
.
........
...
.. ................................
...
.
.
.
.
.
....
.. ..............
...
....
....
.
..
....
.....
....... .... .......... .........
...
.
...
....................... ..
.. ...
..
..... ... ........... ..........
.
.
... ..
....
.
.
.. ....
.... ..............
..
.
.
....
.......... ................................ ...
...
...
.
.
..
...
..
..
...
...
..
..
....
..
...
..
......... ............
..
..
.........
...
...
...
...
...
...
...
...
....
...
.
....
.
.
......
...
.......
......
.......
.............
.......................................

(a) Como as circunferencias de raios 1 cm e 3 cm


sao concentricas, as outras circunferencia mostradas na
figura devem ter raio igual a 1 cm.
(b) Os centros das 3 circunferencias de raio 1 cm
mostradas na figura formam um tri
angulo equil
atero de
lado 2 cm. Logo seus angulos internos medem 60o .
360
= 6 conclumos que ate 6 circunferencias
Como
60
podem ser dispostas nas condicoes exigidas.

4. Festa na escola Representando o n


umero de docinhos que cada um dos 4 amigos
levou pela inicial de seu nome temos:

A + P + M + F = 90
.
A + 2 = P 2 = 2M = F
2

Segue da segunda equacao que:


P =A+4

M=

A+2
2

F = 2(A + 2) .

Substituindo esses valores na primeira equacao obtemos:


A+A+4+

A+2
+ 2(A + 2) = 90 9A = 180 18 A = 18 .
2

Logo:
P = 18 + 4 = 22 ; M =

18 + 2
= 10
2

e F = 2(18 + 2) = 40 .

5. Inflac
ao O preco antigo era menor que 50 reais e sofreu um acrescimo de 20%. Logo,
o novo preco ainda e um n
umero de 2 algarismos. Vamos representa-lo por ab, onde a
e o algarismo das dezenas e b e o algarismo das unidades. Logo, o novo preco e ba, e
temos:
10b + a = 1, 2(10a + b) 10b 1, 2b = 12a a .
Portanto:

4
8
b = b.
10
5
Como a e b sao algarismos, s
o podemos ter a = 4 e b = 5. Logo, o novo preco
e R$ 54,00.
8, 8 b = 11a

74

a=

OBMEP 2009

Lista 6

Solucoes do Nvel 2

Lista 6
1. Gatos no condomnio Sejam:
x
y
z

=
=
=

n
umero de famlias que possuem apenas 1 gato;
n
umero de famlias que possuem exatamente 3 gatos;
n
umero de famlias que possuem 5 gatos.

Segue entao que x + y + z = 29 e x = z. Portanto, 2x + y = 29. Por outro lado, o


n
umero de gatos e x + 3y + 5z. Da temos:
n
umero de gatos = x + 3y + 5z = 6x + 3y = 3(2x + y) = 3 29 = 87 .
2. Soma constante
Sejam a, b, c, d, e e f os n
umeros que colocaremos na tabela.
De acordo com a regra para as 4 subtabelas 2 2

1
b

a
9

temos:

a
9

2
c

b
d

9
e

1
b
d

a
9
e

2
c
f

9
e

c
f

1+a+b+9 =a+2+9+c b=c+1


1+a+b+9 =b+9+d+e a+1=d+e
a+2+9+c = 9+c+e+f a+2= e+f.

ao ficar
a assim:
Subtraindo a 2a igualdade da 3a , obtemos f = 1 + d. A nossa tabela ent
1
b
d

a
9
e

2
b-1
d+1

Como a+1 = d+e e {a, b, d, e} {3, 4, 5, 6, 7, 8}, temos os seguintes casos a considerar:
a
3
4
5

a+1 = d+e
4
5
6

OBMEP 2009

Soluc
ao
n
ao possui
n
ao possui
n
ao possui
1 6 2
8 9 7
4 3 5
n
ao possui
1 8 2
5 9 4
6 3 7

75

5
9

Solucoes do Nvel 2

Lista 6

3. Qual
e o n
umero?
Note que 5 E e um m
ultiplo de 5, e no caso, terminado em
A. Como A n
ao pode ser 0, segue que A = 5. Por outro lado,
E e mpar pois se fosse par teramos A = 0. Observe que E
n
ao pode ser 1, pois senao 4D = 5, o que e impossvel. Logo,
E = 3, 5, 7, 9.
Analizemos cada possibilidade:
E=3

4D + 1 termina em 5

D = 1 ou D = 6;

E=5
E=7

4D + 2 termina em 5
4D + 3 termina em 5

impossvel porque 4D + 2 e par;


D = 3 ou D = 5. Logo D = 3;

E=9

4D + 4 termina em 5

impossvel porque 4D + 4 e par.

5B C DE
BCDE
CDE
DE
E
............................................................................
AAAAA

Restaram ent
ao os seguintes 3 casos:
5B C 13
B C 13
C 13
13
3
............................................................................
55555

5B C 63
B C 63
C 63
63
3
............................................................................
55555

5B C 37
B C 37
C 37
37
7
............................................................................
55555

Antes de analisar cada caso, observe que B tem de ser menor do que 5, ou seja
B = 1, 2, 3, 4. Lembre que letras distintas representam algarismos distintos.
1o caso: 3C termina em 5.
Como 1, 3 e 5 j
a foram usados conclumos que esse caso nao ocorre pois C teria que
valer 5.
2o caso: 3C + 2 termina em 5.
Logo, C = 1 e portanto 2B = 5, o que n
ao e possvel.
3o caso: 3C + 1 termina em 5.
Logo, C = 8 e portanto 2B + 2 = 5, o que implica B = 2. Finalmente, temos
que ABCDE = 52 837.
FC
= 2. Agora, trace o segmento F H, paralelo
4. Proporc
ao triangular Temos que
AF
ao segmento AE onde H esta sobre o segmento BC, como na figura a seguir.
Os tri
angulos AEC e F HC sao semelhantes pois tem lados paralelos. Isto implica
que CH = 2EH.
Por outro lado, os triangulos BF H e BGE tambem sao semelhantes, pois tem
lados paralelos. Dessa semelhanca e do fato que G e ponto medio do segmento BF
conclumos que E e ponto medio do segmento BH.
Assim, BE = EH e, portanto, EC = EH + CH = EH + 2EH = 3EH = 3EB.
EC
= 3.
Consequentemente,
EB
76

OBMEP 2009

Lista 6

Solucoes do Nvel 2
A

F
G

5. N
umeros primos entre si
Soluc
ao 1: Temos:

2000

x y
+
y
x


= 16 125

x2 + y 2
xy


.

ao tem fatores em comum


Como x e y sao primos entre si, conclumos que xy e x2 + y 2 n
(prove isso). Logo, x e y sao divisores de 2 000 = 16 125 = 24 53 . Alem disso, 16
tem que dividir xy porque a express
ao tem que ser mpar. Portanto, 16 = 24 divide x
ou y.
1o caso: 16 divide x.
Se x > 16 entao, x e no mnimo 16 5 = 80, j
a que x divide 2000. Nesse caso, x > y
pois xy divide 2000. Logo, x = 16. Assim, como x e y sao primos entre si, x < y e y
divide 2000 conclumos que as u
nicas possibilidades s
ao y = 25 ou 125.
2o caso: 16 divide y.
Como no item anterior fazemos:
Se y > 16 entao as possibilidades seriam: y = 16 5 e x = 1; y = 16 25 e x = 1;
y = 16 125 e x = 1.
Se y = 16 entao as possibilidades para x seria: x = 1 ; x = 5.
Logo, os pares (x, y) satisfazendo as condicoes do problema sao:
(16, 25) ; (16, 125) ; (5, 16) ; (1, 16) ; (1, 80) ; (1, 400) ; (1, 2 000).
Porem, como podemos trocar x e y em vista da simetria da expressao, temos ainda as
solucoes:
(25, 16) ; (125, 16) ; (16, 5) ; (16, 1) ; (80, 1) ; (400, 1) ; (2 000, 1).
Soluc
ao 2:
Temos N = 2 000(

16 125 (x2 + y 2 )
24 53
x y
+ )=
=
(x2 + y 2 ).
y
x
xy
xy

Como N e mpar segue que

24 53
e x2 + y 2 sao mpares. As opcoes para isso sao:
xy
OBMEP 2009

77

5
9

Solucoes do Nvel 2

Lista 6

xy = 24 , 24 5, 24 52 , 24 53 e, x e y tem paridades distintas. Vamos determinar x


e y para cada uma dessas opcoes:
xy
24
24 .5
24 .52
24 .53

x
1
1
24
1
24
1
24

y
24
24 .5
5
24 .52
52
24 .53
53

Logo, os pares (x, y) satisfazendo as condicoes do problema sao:


(1, 16) ; (1, 80) ; (16, 5) ; (1, 400) ; (16, 25) ; (1, 2 000) ; (16, 125).
Porem, como podemos trocar x e y em vista da simetria da expressao, temos ainda as
solucoes:
(16, 1) ; (80, 1) ; (5, 16) ; (400, 1) ; (25, 16) ; (2 000, 1) ; (125, 16).

78

OBMEP 2009

Lista 7

Solucoes do Nvel 2

Lista 7
1. Fique atento Elevando ambos os membros da equacao ao quadrado, obtemos
x = x2 4x + 4, que e equivalente a x2 5x + 4 = 0. As razes dessa equacao
do segundo grau s
ao x = 1 e x= 4. Entretanto, quando substitumos x = 1 na equacao

original x = x 2 obtemos 1 = 1, que e falso. No entanto, quando substitumos


x = 4 obtemos 4 = 2, que e verdadeiro. Portanto, a equacao dada possui x = 4 como
u
nica solucao.
Atencao: O aparecimento da solucao estranha x = 1 deve-se ao fato que a implicacao
a2 = b2

a=b

n
ao e verdadeira em geral. O correto e
a2 = b2

a = b .

Deste modo, quando elevamos os dois membros de uma equacao ao quadrado, obtemos uma nova equacao que pode, eventualmente, conter mais solucoes que a equacao
original. Voce tambem pode ver isso com clareza, por exemplo, nas equacoes: x = 1 e
x2 = 12 .
2. Soluco
es inteiras A equacao e equivalente a xy = 19(x + y). Uma vez que estamos
procurando solucoes inteiras e 19 e um n
umero primo, esta igualdade implica que x ou
y devem ser divisveis por 19. Como a equacao e simetrica em relacao as vari
aveis x e
y, podemos supor que x e divisvel por 19. Isto e, x = 19k para algum valor inteiro de
k. Nessa condicao, temos:
xy = 19(x + y)

ky = 19k + y .

Desta igualdade conclumos que 19k + y e divisvel por k. Uma vez que 19k j
a e divisvel
por k conclumos que y e divisvel por k (prove isso). Isto e, y = km para algum valor
inteiro de m. Assim,
ky = 19k + y k2 m = 19k + km km m = 19 m(k 1) = 19 .
Visto que m e k sao n
umeros inteiros e 19 e um n
umero primo, existem somente 4
possibilidades para a igualdade m(k 1) = 19:
m = 19 e k 1 = 1. Isto implica que x = 38 e y = 38;
m = 19 e k 1 = 1. Isto implica que x = 0 e y = 0, que n
ao e possvel, pois
na equacao original, x = 0 e y = 0;
m = 1 e k 1 = 19. Isto implica que x = 380 e y = 20;
m = 1 e k 1 = 19. Isto implica que x = 342 e y = 18.
Deste modo, obtemos as seguintes possibilidades para o par de n
umeros inteiros (x, y)
que sao solucoes da equacao dada:
(38, 38) ; (380, 20) ; (20, 380) ; (342, 18) ; (18, 342) .
OBMEP 2009

79

5
9

Solucoes do Nvel 2

Lista 7

3. No ponto de
onibus Vamos representar por M o n
umero de meninas e por H o
n
umero de meninos que estavam no ponto antes da parada do primeiro onibus. Depois
do embarque das 15 meninas no primeiro onibus , ficaram no ponto M 15 meninas e H
meninos. Uma vez que, neste momento, ficam no ponto 2 meninos para cada menina,
temos: H = 2(M 15). No segundo onibus embarcam 45 meninos, e ficaram no ponto
M 15 meninas e H 45 meninos. Como, neste momento, ficaram no ponto 5 meninas
para cada menino, temos: M 15 = 5(H 45).
Deste modo, obtemos o sistema linear

H = 2(M 15)
.
M 15 = 5(H 45)
Substituindo a primeira equacao na segunda obtemos: M 15 = 5(2M 30 45).
Logo:
375 15 = 10M M M = 40
e H = 2(40 15) = 50.
4. Contorno circular Sejam A, B, C e D os centros dos crculos, e M , N , P e Q os
pontos de tangencia.

Observe que AD = DC = BC = AB = AC = 2a. Logo, os tri


angulos ABC e ACD
sao equil
ateros e por isso seus angulos internos s
ao iguais a 60o . Assim, temos:


= 60o M N =
ABC



5

= 120o M Q = 2 2a.
2a e B AD
6
3


Como M N = P Q e M Q = N P segue que o contorno externo da figura dada tem


comprimento igual a:


2
5
2a = 6a .
2 +2
6
3
5. Um quadril
atero especial Se cada diagonal divide o quadril
atero em duas regi
oes de
mesma area temos:

Area(ABD)
= Area(BCD)
e Area(ABC)
= Area(ACD)
.
80

OBMEP 2009

Lista 7

Solucoes do Nvel 2
qB

......
.....................
............... ......... ....
...............
.... ...
...............
.
.
.
.
.
.
.
.
.
.
.
.
.
.
.
.
.
.
.
.
......
....
..
................
....
..
..............
.....
..
...............
.....
..
...............
.....
..
...............
.
.
.
.
.
.
.
.
.
.
.
.
.
.
.
.
.
.
.
.
.
.
.
.
...............
.....
..
...............
....
..........
...
.....
..
.. ...............
.....
........
..
..
.....
.
.
.
........
...
.
.
....
........
..
.
....
..
........
..
.....
..
........
..
.....
........
..
..
........ .........
..
..
.
.
...
.
.
..
..
..... ...............
..
..
........
....
..
..
........
.....
...
..
........
.....
..
..
........
.....
.
.
.
.
...
.
........
.
..
........
.....
..
........ ....
.....
...
.
.....
..
..........
.....
..
...............
.
.
.
.
.
.....
.
.
.
.
.
.
.
.
.
.
...
.
.
......
.
....
...............
..
....
...............
..
.....
...............
..
.....
...............
..
.....
...............
.
.
.
.
.
.
.
.
.
.
.
.
.
.
.
.
.
.
...
.
.
.....
...............
.. .... .............................
..
.. .....
.. ..... ................
..................

Mas,

Aq

Area(ABD)
= X +W

Area(BCD) = Y + Z

Area(ABC)
= Z +W

Area(CDA)
= X +Y

Assim,

Z X = Area(ABC)
Area(ABD)
= Area(CDA)
Area(BCD)
=X Z
e portanto, Z = X. Consequentemente, tambem temos Y = W .
Como as areas opostas s
ao iguais, resulta da semelhanca de tri
angulos que:
AE ED = BE EC

e AE EB = CE ED .

Dividindo esta duas equacoes obtemos:


EB
ED
=
EB
ED

ED = EB .

Analogamente podemos mostrar que EA = EC. Logo, as diagonais se cortam no ponto


medio, e consequentemente o quadril
atero e um paralelogramo donde, o permetro e
igual a 2 10 + 2 15 = 50 cm.

OBMEP 2009

81

5
9

Solucoes do Nvel 2

Lista 8

Lista 8
1. N
umero curioso Seja ab um tal n
umero. Por hip
otese ab = 10a + b e divisvel por
a + b. Logo, a diferenca (10a + b) (a + b) = 9a, tambem e divisvel por a + b. Alem
disso, sabemos que 10a + b e divisvel por a + b se, e somente se, (10a + b) (a + b) = 9a
e divisvel por a + b (prove isso).
Antes de prosseguirmos na solucao, note que como ab e um n
umero de dois algarismos
entao a = 0. Agora, basta atribuir valores para a e calcular os valores de b para os quais
a + b divide 9a. O resultado e mostrado na tabela a seguir.
a
1
2
3
4
5
6
7
8
9

9a
9
18
27
36
45
54
63
72
81

b
0, 2, 8
0, 1, 4, 7
0, 6
0, 2, 5, 8
0, 4
0, 3
0, 2
0, 1, 4
0

Logo os n
umeros que satisfazem a propriedade sao:
10, 12, 18, 20, 21, 24, 27, 30, 36, 40, 42, 45, 48, 50, 54, 60, 63, 70, 72, 80, 81, 84, 90
ou seja, existem 23 n
umeros nas condicoes exigidas.
2. N
umero premiado (a) O maior n
umero premiado tem de comecar com 98. Assim o n
umero procurado e
da forma: 98abcd. Por hip
otese temos: 9 + 8 + a = b + c + d. Para que a seja maximo
precisamos que b + c + d seja maximo, e isto acontece quando b = 7, c = 6 e d = 5.
umero premiado e 981 765.
Neste caso, a = 1 e consequentemente, o maior n
Vamos agora determinar o menor n
umero premiado. Tentemos um n
umero da forma
10abcd. Agora, n
ao e difcil verificar que 108 234 e o menor n
umero premiado.
(b) Se o n
umero ABCDEF e premiado, ent
ao o n
umero DEF ABC tambem e
premiado. A soma desses n
umeros e:
ABCDEF + DEF ABC = (1000ABC + DEF ) + (1000DEF + ABC)
= 1001(ABC + DEF ) = 13 11 7 (ABC + DEF ) .
Somando todos os n
umeros premiados com 6 algarismos diferentes aos pares, resulta
que cada par e divisvel por 13. Logo, a soma de todos eles e divisvel por 13.
Nota: De fato tambem e divisvel por 11 e 7.
82

OBMEP 2009

Lista 8

Solucoes do Nvel 2

3. Altura versus lado Sejam ha e hc as alturas relativas aos lados BC = a e AB = c,


respectivamente. Por hip
otese temos que ha a e hc c. Como ha e hc sao os
comprimentos das alturas, entao ha c e hc a.
Um dos lados considerados e maior ou igual ao outro: digamos a c. Das desigualdades
acima temos:
ha a c ha a = c = ha .
Da, segue que AB e perpendicular a BC. Logo, o tri
angulo e retangulo is
osceles e

portanto, os angulos medem 45 , 45 e 90 .

4. Fraco
es egpcias A equacao e equivalente a 2ab = 7(a + b). Como 2 e 7 sao n
umeros
primos entre si, segue que ab e m
ultiplo de 7 e que a + b e m
ultiplo de 2. Mas, para
ab ser m
ultiplo de 7, a u
nica possibilidade e a ser m
ultiplo de 7 ou b ser m
ultiplo de 7.
Suponhamos primeiramente que a e m
ultiplo positivo de 7, ou seja, a = 7k para algum
inteiro positivo k. Da obtemos:
2ab = 7(a + b) 2kb = 7k + b (2k 1)b = 7k .
utiplos de 7. Se b e m
ultiplo de 7,
Esta u
ltima equacao implica que b ou 2k 1 sao m
b = 7m, e assim
1 1
2
1
1
1
1
2
= +

=
+

+
= 2.
7
a b
7
7k 7m
k m
1
1
1
1
1
1
1e
1. Assim +
2. Portanto, a equacao +
= 2 possui
Mas,
k
m
k
m
k
m
u
nica solucao inteira positiva, a saber, k = 1 = m. Entretanto, esta solucao n
ao nos
interessa, pois neste caso a = b.
Passemos entao ao caso em que 2k 1 e m
ultiplo de 7.
Uma possibilidade para 2k 1 ser m
ultiplo de 7 ocorre quando k = 4. Neste caso, temos
que
k = 4 a = 7k = 28 ;
(2k 1)b = 7k 7b = 28 b = 4 .
Obtemos entao

1
1
2
=
+ .
7
28 4

5. Tabuleiro de xadrez Um tabuleiro de xadrez e um quadrado reticulado de 64 quadradinhos, sendo 32 claros e 32 escuros, posicionados alternadamente. Cada quadradinho recebe o nome de casa. As pecas sao denominadas: rei, dama, torre, bispo,
cavalo e pe
ao. S
ao 16 pecas claras e 16 escuras, sendo 2 pecas de cada categoria.
OBMEP 2009

83

5
9

Solucoes do Nvel 2

Inicialmente, e possvel colocar um bispo em


8 8 = 64 casas. Suponhamos que o bispo
esta numa casa branca, ent
ao na fila e na coluna
onde ele esta temos 8 casas pretas. Assim, o segundo bispo pode ser colocado em qualquer uma
das 32 8 = 24 casas pretas restantes. Conclumos entao que se um dos bispos ocupa uma
das 32 casas brancas entao o outro ter
a 24 casas
pretas para se localizar. Portanto, o n
umero de
configuracoes distintas que podem ser obtidas e:
32 24.

84

OBMEP 2009

Lista 8
Nota: Aqui estamos entendendo que alternando a posicao
desses dois bispos nao mudamos a configuracao no tabuleiro de xadrez. Mais precisamente, os bispos tem a mesma
cor, isto e, pertencem a um
mesmo competidor.

Lista 9

Solucoes do Nvel 2

Lista 9
1. Quem
e menor? Observemos que:
3312 > 3212 = (25 )12 = 260 ;
6310 < 6410 = (26 )10 = 260 ;
1278 < 1288 = (27 )8 = 256 .
Logo, o maior dos n
umeros e 3312 .
Por outro lado,


127
63

2

127
63

=2+

1
63

< 2, 1. Logo:


< 2,1 < 7 e

127
63

4

< 49 < 63 1274 < 635 1278 < 6310 .

Logo, o menor dos tres n


umeros dados e 1278 .
2. Brincando com n
umeros Como queremos encontrar o maior n
umero possvel, menor
do que 900, iniciaremos com o algarismo 8 na casa da centena. Observemos que o
n
umero 800 satisfaz a propriedade. Logo, o n
umero procurado e maior que ou igual a
800.
Devemos entao encontrar a e b tais que 8 + a + b divida 8ab = 800 + 10a + b. Lembramos que 8 + a + b divide 8ab = 800 + 10a + b se, e somente se, 8 + a + b divide
800 + 10a + b (8 + a + b) = 792 + 9a. Agora, atribuindo valores para a na ordem
decrescente obtemos:
a = 9 792 + 9 9 = 873 = 9 97 e este n
umero nao possui nenhum divisor
entre 17 (b = 0) e 26 (b = 9).
umero entre 16 e
a = 8 792 + 9 8 = 864 = 25 33 . O maior divisor deste n
25 e 24, isto e, b = 8. Logo, o n
umero procurado e 888.
3. Cortando pap
eis Se na primeira rodada Andre pega n1 pedacos de papel para cortar
cada um deles em sete pedacos, ao final desta rodada ele ficar
a com 7 n1 pedacos sem
cortar mais 7n1 pedacos cortados, totalizando (7n1 )+7n1 = 7+6n1 pedacos de papel.
Analogamente, se na segunda rodada Andre pega n2 pedacos de papel para cortar, ao
ao foram cortados nesta
final desta rodada ele ficar
a com 7 + 6n1 n2 pedacos que n
rodada, mais 7n2 pedacos de papel provenientes dos cortes que ele fez nesta rodada.
Assim, ao final da segunda rodada Andre ficara com
(7 + 6n1 n2 ) + 7n2 = 7 + 6(n1 + n2 ).
Continuando deste modo, conclui-se que ao final de k rodadas Andre fica com
7 + 6(n1 + n2 + + nk ) pedacos de papel. Para ele ficar entao com 2 009 pedacos de
papel ao final de alguma rodada, deve-se ter
7 + 6(n1 + n2 + + nk ) = 2 009;
ou equivalentemente
6(n1 + n2 + + nk ) = 2 002.
OBMEP 2009

85

5
9

Solucoes do Nvel 2

Lista 9

Uma vez que 2 002 n


ao e m
utiplo de 6, esta equacao n
ao admite solucao e, portanto,
Andre nunca podera ficar com 2 009 pedacos ao final de alguma rodada do jogo.
C

4. Um trap
ezio especial Suponhamos que AE
seja maior do que BC, e seja A um ponto sobre
AE tal que A E = BC.
A

Como A E e BC sao paralelos temos que A BCE e um paralelogramo, em particular


BA = CE. Mas, AA + AB > BA pela desigualdade triangular. Assim:
AB + AE + BE = AB + AA + A E + BE > BA + A E + BE = BC + CE + EB .
Portanto, o permetro do tri
angulo ABE e maior que o permetro do tri
angulo BCE.
Desta forma, AE n
ao pode ser maior que BC.
Por um processo similar podemos tambem concluir que BC n
ao pode ser maior que AE
e portanto BC = AE.
Analogamente, temos que ED = BC. Consequentemente,
1
2

BC = (AE + ED) = 15 cm .
5. Uma estrela
A

No triangulo BHE temos:


B

20

J
130

= 180o B EH

= 30o .
20o + 130o + B EH

= B EH

= 30o .
Note que J EI

H
C

86

OBMEP 2009

Lista 10

Solucoes do Nvel 2

Lista 10
1. N
umero palindrome Um n
umero palindrome de 4 algarismos e da forma: abba, onde
a e um algarismo entre 1 e 9 e b e um algarismo entre 0 e 9. Como o n
umero e divisvel
por 9, entao a soma de seus algarismos: 2a + 2b = 2(a + b) e divisvel por 9, ou seja
a + b e divisvel por 9. Se a + b = 9, temos as 9 solucoes:
a=1 e b=8 ;

a=2 e b=7

a=3 e b=6

a=4 e b=5

a=5 e b=4 ;

a=6 e b=3

a=7 e b=2

a=8 e b=1

a = 9 e b = 0.
Se a + b = 18 entao a u
nica solucao e: a = b = 9.
Logo, o n
umero de palindromes de 4 algarismos divisveis por 9 e 10, sao eles: 1 881,
2 772, 3 663, 4 554, 8 118, 7 227, 6 336, 5 445, 9 009 e 9 999.
2. Multiplicac
ao com letras Se o produto de b por c termina em 1, entao b c pode ser
21 ou 81 segue que b c = 3 7 ou 9 9. A u
nica possibilidade de escrever o produto
de dois n
umeros distintos menores que 10 e 21 = 3 7. Assim temos dois possveis
casos:
1o caso: b = 3 e c = 7:
a33
7
3731
3731
= 533 e, consequentemente, a = 5.
7
2o caso: b = 7 e c = 3:
7371
= 2457. Logo, este caso nao ocorre.
Nesta caso
3
Neste caso

3. N
umeros sortudos
(a) A sequencia de oito n
umeros consecutivos de 52 a 59 tem exatamente, dois n
umeros
sortudos: 52 e 59. Outro exemplo e qualquer sequencia de 8 n
umeros que contenha
59 e 61, por exemplo: 55, 56, 57, 58, 59, 60, 61, 62.
(b) Dois exemplos: 994, . . . , 1 005 e 7 994, . . . , 8 005. Existem mais exemplos, encontre
alguns.
(c) Chamemos de decada qualquer sequencia de 10 n
umeros consecutivos cujo primeiro
termo e m
ultiplo de 10:
10 , 11 , 12 , 13 , 14 , 15 , 16 , 17 , 18 , 19
140 , 141 , 142 , 143 , 144 , 145 , 146 , 147 , 148 , 149.
Note que qualquer sequencia de 7 n
umeros consecutivos numa decada contem pelo
menos um n
umero sortudo porque a soma de seus algarismos e uma sequencia
de 7 n
umeros consecutivos, um dos quais tem de ser divisvel por 7. Finalmente,
qualquer sequencia de 13 n
umeros consecutivos contem pelo menos 7 n
umeros
OBMEP 2009

87

5
9

Solucoes do Nvel 2

Lista 10

consecutivos de uma decada, que sempre contem um n


umero sortudo. Examine
alguns exemplos para melhor entender essa justificativa.
4. Uma sequ
encia especial Vamos inicialmente escrever alguns termos:
1 , 3 , 2 , 1 , 3 , 2 , 1 , 3 , 2 , . . .
ao, respectivamente iguais ao 1o e 2o . Isso significa que a sequencia
O 7o e 8o termos s
se repete de 6 em 6 termos. A soma dos 6 primeiros termos e 1 + 3 + 2 1 3 2 = 0, e
portanto, a soma dos 96 primeiros termos tambem e 0. Logo, a soma dos 100 primeiros
termos dessa sequencia e igual a soma dos 4 u
ltimos termos, ou seja, 1 + 3 + 2 1 = 5 .
5. Tri
angulos e
angulos...
No triangulo
menor, os angulos medem 70o , 180o 130o = 50o
enquanto que o terceiro medir
a
180o (50o + 70o ) = 60o .
Assim,
= 180o 60o = 120o .
Agora, no tri
angulo maior temos:
45o + + 50o = 180o = 180o 95o = 85o .

88

OBMEP 2009

Lista 1

Solucoes do Nvel 3

Soluco
es do Nvel 3
Lista 1
1. Brincando com a calculadora O resultado e o mesmo n
umero inicial de 3 algarismos
a b c. De fato, se a b c e um n
umero de 3 algarismos ent
ao o n
umero a b c a b c de 6
algarismos e da forma:
a b c a b c = 1 000 a b c + a b c = 1 001 a b c .
Como 1 001 = 7 11 13, dividindo a b c a b c, sucessivamente, por 7, 11 e por 13,
obtemos:
abcabc
1 001 a b c
=
= abc.
7 11 13
7 11 13
2. No galinheiro Sejam x e y, respectivamente, o n
umero de galinhas e pintinhos no
galinheiro.
(a) Temos 4x + 2y = 240, ou seja, 2x + y = 120.
Como, 8 kg = 8 000 g temos: 160x + 40y 8 000. Assim, 4x + y 200. Em resumo,
o n
umero x de galinhas e y de pintinhos satisfazem:

2x + y = 120
()
4x + y 200.
(b) A reta 2x + y = 120 corta o eixo Ox em x = 60 e o eixo Oy em y = 120.
A reta 4x + y = 200 corta o eixo Ox em x = 50 e o eixo Oy em y = 200. Os graficos
dessas retas estao abaixo, onde a desigualdade 4x + y 200 e representada pela regi
ao
sombreada.
Observe que a condicao (*) e representada na figura pelo segmento que
liga os pontos P e (0 , 120). As coordenadas do ponto P sao a solucao do
sistema:

2x + y = 120
4x + y = 200;
ou seja, x = 40 e y = 40, e
P = (40, 40) .
(c) Temos que 2 20+ 80 = 120 e 4 20+ 80 200. Logo, x = 20 e y = 80 satisfazem
a condicao (*) e, por isso, a resposta e sim.
Agora 2 30 + 100 = 120, logo, x = 30 e y = 100 n
ao satisfazem a condicao (*) e, por
isso, a resposta e nao.
(d) O n
umero maximo de galinhas e 40, e nesse caso teremos tambem 40 pintinhos. O
n
umero maximo de pintinhos e 120, e nesse caso teremos 0 galinhas.
OBMEP 2009

89

5
9

Solucoes do Nvel 3

Lista 1

3. Um n
umero perfeito - Se 231 1 e um n
umero primo, seu u
nico divisor pr
oprio e o
30
31
n
umero 1. Ent
ao os divisores proprios de 2 (2 1) sao:
1 , 2 , 22 , 23 , . . . , 229 , 230 , (231 1) , 2(231 1) , 22 (231 1) , . . . , 229 (231 1) .
A soma S desses divisores e:
S = [1 + 2 + 22 + 23 + + 229 + 230 ] + (231 1)[1 + 2 + 22 + 23 + + 229 ] .
ao geometrica de
Em cada um dos dois colchetes aparece a soma Sn de uma progress
primeiro termo igual a 1 e raz
ao 2.
O primeiro colchete, S31 , contem 31 termos e o segundo, S30 , contem 30 termos. Usando
a f
ormula da soma dos termos de uma progress
ao geometrica, temos:
S31 =

231 1
230 1
= 231 1 e S30 =
= 230 1 .
21
21

Entao a soma dos divisores pr


oprios de 230 (231 1) e :
S = (231 1) + (231 1)[230 1] = (231 1)(1 + 230 1) = 230 (231 1) .
Logo, essa soma e igual a 230 (231 1), como queramos provar.
4. Quinze minutos a mais
Soluc
ao 1: Sabemos que espaco = velocidade tempo. Denotemos por t o tempo
gasto pelo carro menos r
apido (aquele que faz a viagem com velocidade de 60 km/h).
Logo, o tempo gasto pelo outro carro foi t 15. Como ambos percorrem a mesma
distancia, convertendo horas em minutos, segue que:
70
3
60
t=
(t 15) t = 105 min = 1 h .
60
60
4
Logo, a distancia entre as duas cidades e:
60 1

7
3
= 60 = 105 km .
4
4

Soluc
ao 2: Vamos representar por d a dist
ancia entre as cidades A e B, e por T o
tempo gasto, em horas, pelo carro mais veloz. Como o outro carro gasta 15 minutos a
mais para fazer o mesmo percurso, temos que o tempo gasto por ele e igual a T + 0, 25
horas, pois 15 min = 0,25 h.
Como a velocidade e a raz
ao da dist
ancia percorrida pelo tempo gasto, conclumos que
d
d
. Da segue que d = 70 T = 60(T +0, 25), ou seja, T = 1, 5 h.
70 = e 60 =
T
T + 0, 25
Logo, d = 70 1,5 = 105 km.
5. Outros caminhos Qualquer que seja a maneira que J
ulia caminhe da sua casa ate
a escola, ela deve percorrer 8 quarteir
oes para a direita e 5 quarteir
oes para cima.
Um caminho ligando a sua casa ate a escola e entao uma sequencia de travessias de
90

OBMEP 2009

Lista 1

Solucoes do Nvel 3

quarteir
oes, sendo 8 no sentido horizontal (para a direita) e 5 no sentido vertical (para
cima). Assim, para definir um caminho ela precisa apenas decidir em que ordem far
a
essas travessias.
Desse modo, imaginemos 8 cartelas impressas com a letra D e 5 cartelas impressas com a letra C. Uma permutacao qualquer destas cartelas pode ser interpretada
como um caminho a ser percorrido por J
ulia. Por exemplo, a sequencia de cartelas
DDCDCCDDDDCDC define o seguinte caminho:

Para determinar o n
umero de maneiras que se pode ordenar essas cartelas, devemos
contar de quantas maneiras diferentes se pode colocar 5 cartelas impressas com a letra
C em uma fila com 13 lugares vagos e os demais 8 lugares na fila ocupados com as
cartelas impressas com a letra D.
Inicialmente, devemos escolher um dos 13 lugares vagos para colocar uma letra C. Colocada esta letra, sobram 12 lugares vagos para a segunda letra C. Colocada esta letra,
sobram 11 lugares vagos para a terceira letra, 10 lugares para a quarta letra e, finalmente,
9 lugares para a quinta letra C. Agora, uma vez colocadas as cinco letras C, qualquer
permutacao dessas letras entre si nao altera a distribuicao das letras na fila. Como a
quantidade de permutacoes de cinco objetos e 5! = 120, pelo princpio multiplicativo
temos que o n
umero de maneiras de ordenar as 13 cartelas e
13 12 11 10 9
= 1 287 .
120

OBMEP 2009

91

5
9

Solucoes do Nvel 3

Lista 2

Lista 2
1. Escrevendo em um tabuleiro Comecando com a letra A, ela pode ser escrita em
qualquer uma das 9 casas do tabuleiro. Uma vez escrita a letra A, sobram 6 casas onde
a letra B pode ser escrita. Uma vez escritas as letras A e B no tabuleiro, sobram 3 casas
para a letra C ser escrita.
Assim, pelo princpio multiplicativo, existem 9 6 3 = 162 maneiras
diferentes das letras A, B e C serem escritas no tabuleiro.

2. Frac
ao e porcentagem A opcao correta e (D).
Se um n
umero x e diminudo de 40%, ele passa a valer 60% de x, ou seja: 0, 6x.
Do mesmo modo, quando um n
umero y e diminudo de 60%, ele passa a valer 0, 4y.
0, 6x
6 x
x
x
passa a ter o valor
=
= 1, 5 . Isto significa que a
Portanto, a fracao
y
0, 4y
4 y
y
x
fracao aumentou 50% do seu valor.
y
3. Tri
angulos sobrepostos Os pontos A, B, C e D formam o ret
angulo ABCD.

Como as diagonais de um retangulo o dividem em quatro tri


angulos de mesma area, a
area sombreada e igual a tres quartos da area do retangulo ABCD. Portanto, a area
3
sombreada e igual a (7 4) = 21 cm2 .
4
Vejamos agora o caso da outra figura. Sejam E o ponto de intersecao dos segmentos
AC e BD, x = DE = CE e y = AE = BE.

A area sombreada e a soma das areas dos tri


angulos ADE e ABC, ou seja:
4x 47
+
= 2x + 14 .
2
2
Logo, basta calcularmos x. Temos que x + y = 7 e, pelo Teorema de Pitagoras aplicado
92

OBMEP 2009

Lista 2

Solucoes do Nvel 3

ao tri
angulo AED, y 2 = x2 +42 . Substituindo y = 7x nessa u
ltima equacao obtemos:
(7 x)2 = x2 + 16 49 14x + x2 = x2 + 16 x =

49 16
33
=
.
14
14

Finalmente, a area sombreada e:


2

33
5
5
33
+ 14 =
+ 14 = 4 + 14 = 18 .
14
7
7
7

4. Dois motoristas Seja d a dist


ancia entre as cidades A e B, e lembre que
tempo = distancia/velocidade.
O primeiro motorista viaja a dist
ancia de 2d com velocidade constante igual a
80 km/h. Logo, o tempo total gasto por ele e:
t=

d
2d
=
.
80
40

O segundo motorista percorre a distancia d, na ida, com velocidade igual a


90 km/h e, na volta, a mesma dist
ancia com velocidade de 70 km/h. Logo o
tempo gasto na ida e volta e:
t =

d
16 d
8d
d
+
=
=
.
70 90
630
315

Como

8d
8d
d
=
<
,
40
320
315
conclui-se que o motorista que viaja com velocidade constante de 80 km/h e o que gasta
menos tempo no percurso de ida e volta.

5. Soma e inverte Para obter 0, a sequencia tem de terminar como:


+1

+1

2 1 0 .
Uma sequencia pedida e a seguinte:
1 +1 2 +1 5 +1 8 i
3 +1 5 +1 13 +1 21
+1
+1
+1
i

0 1 2 3
3
3
3
3
8
8
8
8
8 +1 13 i
21 +1
8 +1 5 i
13 +1 8 +1 3 +1 2



21
21
13
13
13
5
5
5
5
5 +1 3 +1 1 +1 1 i
i
+1
+1
2 1 0.
2
2
2
2
Temos outra solucao bem mais r
apida e simples:
i

+1

+1

0 1 1 0.

OBMEP 2009

93

5
9

Solucoes do Nvel 3

Lista 3

Lista 3
1. Carro flex
12,3
= 4,94 km por R$1, 00. Para que o alcool seja mais
2,49
vantajoso precisamos que o carro rode, com alcool, mais que 4, 94 km com R$1, 00.
y
Logo, se o desempenho com alcool e y km/l, precisamos que
> 4,94, o que implica
1,59
y > 7,85. Ou seja, o desempenho com alcool deve ser maior que 7,85 km/l.
(a) Com gasolina o carro faz

318
100
=
e a(x) = 1,59 x
.
x+2
+1
2
318
249
=
, o que leva a x = 7,22 km/l,
(c) Precisamos ter a(x) = g(x), ou seja,
x
x+2
que deve ser o desempenho com gasolina. Com alcool, o carro deve fazer

(b) Observe que g(x) = 2,49

249
100
=
x
x

7,22
+ 1 = 3,61 km/l .
2
(d) Supondo que o desempenho do carro seja x km/l com gasolina e y km/l com alcool
e pensando em um percurso de L km, devemos ter o custo com gasolina maior que o
custo com alcool:
2,49

L
L
> 1,59 2,49y > 1,59x y > 0,64x,
x
y

pois x e y sao valores positivos.


Um exemplo e um carro que faz 10 km/l com gasolina, teria que fazer mais que
6,4 km/l com alcool para que o uso do alcool seja mais vantajoso.
Observac
ao. Os valores determinados na solucao foram aproximados na segunda casa
decimal.
2. Contando tri
angulos Sejam A, B, . . . , K os 11 pontos nomeados como na seguinte
figura:
K
J
I
H
A

Dividiremos a contagem em tres casos:


(i) Um vertice e A. Neste caso, um vertice do triangulo deve estar no conjunto
{H, I, J, K} e o outro vertice no conjunto {B, C, D, E, F, G}. Como existem
4 escolhas para um vertice e 6 escolhas para o outro vertice, a quantidade de
tri
angulos com um vertice no ponto A e: 6 4 = 24.
94

OBMEP 2009

Lista 3

Solucoes do Nvel 3

(ii) Dois vertices em {B, C, D, E, F, G}. O outro vertice esta no conjunto {H, I, J, K},
pois j
a contamos os triangulos com vertice em A. Devemos escolher dois entre os
6 pontos {B, C, D, E, F, G}. Assim, temos a quantidade de escolhas:
C62 =

65
6!
=
= 15.
4!2!
2

O outro vertice do triangulo e qualquer um dos 4 pontos {H, I, J, K}. Da a


quantidade de tri
angulos e 4 15 = 60.
(iii) Dois vertices em {H, I, J, K}. O outro vertice esta no conjunto {B, C, D, E, F, G}.
O n
umero de maneira de escolher 2 entre os 4 pontos {H, I, J, K} e
C42 =

4!
43
=
= 6.
2!2!
2

Como o outro vertice pode ser escolhido de 6 maneira diferentes, temos que a
quantidade de tri
agulos e 6 6 = 36.
Logo, a quantidade de tri
angulos cujos vertices sao tomados dentre os 11 pontos da
figura e 24 + 60 + 36 = 120.
3. Quadrado perfeito Seja x um n
umero de oito algarismos da forma
x = 9999 .
Como o menor desses n
umeros e 99 990 000 e o maior e 99 999 999, temos que:
99 990 000 x 99 999 999 .
ao 99 990 000 x < 108 .
Observemos que 108 = 100 000 000 = 99 999 999 + 1. Ent


2
Como 108 = 104 = 10 0002 , temos que 99 990 000 x < 10 0002 . Agora, o maior
quadrado perfeito menor que 10 0002 e igual a
9 9992 = (10 000 1)2 = 10 0002 20 000 + 1 = 100 000 000 20 000 + 1 = 99 980 001.
Como 99 980 001 < 99 990 000 conclumos que 9 9992 < x < 10 0002 . Isto mostra que
x esta compreendido entre dois quadrados perfeitos consecutivos. Portanto, x n
ao pode
ser um quadrado perfeito.

n n 1 < 0,01 e equivalente a


4. Diferenca quase
nula A inequacao

n < 0,01 + n 1 . Como os dois lados desta inequacao s


ao n
umeros positivos,
podemos elevar esses dois membros ao quadrado para obter a inequacao equivalente:


2
 2
n
< 0, 01 + n 1
Da obtemos

n < 0,012 + 0,02 n 1 + n 1 .

1
1
2
1 0,012
1002 = 100 1 .
=
n1 >
2
0,02
200
100
OBMEP 2009

95

5
9

Solucoes do Nvel 3

Lista 3

Elevando novamente ao quadrado os dois lados (n


ao negativos) desta inequacao, obtemos:
n1 >

1
1
1004 2 1002 + 1
1002
(1002 1)2
+
=
=
,
2
2
200
4 100
4
2 4 1002

ou seja,
1
1
1
1
1
1
n > 2 500 +
+1 n > 2 500+ +
.
n1 > 2 500 +
2 40 000
2 40 000
2 40 000
1
1
+
< 1, temos que o menor n
umero inteiro maior que
Uma vez que
2
40 000
1
1
e 2 501.
2 500 + +
2 40 000
Da conclumos que o menor n
umero inteiro positivo que satisfaz a desigualdade dada e
o n
umero 2 501.
5. Conjunto de Cantor
(a) De acordo com a definicao do Conjunto de Cantor temos os seguintes desenhos:
C1
C2
C3

0
0

1
1/3

0 1/9 2/9 1/3

2/3

2/3 7/9 8/9 1

(b) 1/3 e um extremo de C2 , logo pertence ao conjunto de Cantor.


3/81 = 1/27 e 1/27 e um extremo de C4 , logo 3/81 pertence ao conjunto de Cantor.
4/9 esta entre 1/3 e 2/3, logo esta no terco central de C1 e e removido de C2 , logo 4/9
n
ao pertence ao conjunto de Cantor.
4/81 esta entre 1/27 e 2/27, e portanto est
a no terco central de C3 e e removido de
ao pertence ao conjunto de Cantor.
C4 . Assim, 4/81 n
(c) Vamos tentar achar um padr
ao para os comprimentos dos segmentos. Por exemplo,
a que isso ja fornece um padr
ao,
C1 tem comprimento 1 e C2 tem comprimento 2/3. Ser
ou seja o numerador e obtido multiplicando por 2 e o denominador por 3, ou seja por
2/3?
Agora C3 tem comprimento 4/9, C4 comprimento 8/27 e C5 comprimento 16/81. Logo,
2
o comprimento de Cn e ( )n1
3
Note que os comprimentos de C1 , C2 , C3 , . . . , Cn , . . . , formam uma progress
ao
geometrica de raz
ao q = 2/3 e primeiro termo a1 = 1.
1,

96

 2 n
2  2 2  2 3  2 4
,
,
,
,... ,
,...
3
3
3
3
3

OBMEP 2009

Lista 4

Solucoes do Nvel 3

Lista 4
1. Enchendo uma piscina Como as torneiras A e B despejam agua na piscina com vaz
ao
constante, o volume de agua despejado na piscina por cada torneira e proporcional ao
tempo em que ela fica aberta. Assim, se durante 2 horas a torneira A enche 15% do
volume da piscina, ent
ao em 4 horas ela encher
a 30% do volume da piscina.
Mas, quando as torneiras A e B ficam simultaneamente abertas durante 4 horas, elas
conseguem encher 50% do volume da piscina. Da temos que a torneira B enche
50% 30% = 20% do volume da piscina em 4 horas.
Para saber quanto tempo a torneira B deve ficar aberta para encher os 35% restantes
do volume da piscina, basta utilizar a proporcao:
horas percentual
4

20%
x

35%
Logo, a torneira B gastar
ax=

35 4
= 7 horas para encher os 35% restantes.
20

2. Probabilidade de ser um n
umero par Sejam a e b os n
umeros escritos nas bolas
retiradas por Jose e Maria, respectivamente. Existem entao 9 possibilidades para a e 8
possibilidades para b. Deste modo, existem 9 8 = 72 possibilidades para o n
umero ab.
Por outro lado, para contar quantos destes n
umeros sao pares, precisamos analisar separadamente dois casos:
os n
umeros a e b sao pares;
o n
umero a e mpar e o n
umero b e par.
No primeiro caso, em que a e b sao pares, existem 4 possibilidades para a e 3 possibilidades para b. Deste modo, existem 4 3 = 12 possibilidades.
No segundo caso, em que a e mpar e b e par, existem 5 possibilidades para a e 4
possibilidades para b. Assim, existem 5 4 = 20 possibilidades.
32
4
12 + 20
=
= .
Portanto, a probabilidade do n
umero ab ser par e
72
72
9
3. M
ultiplo de 7 Inicialmente, observemos que:
N

= (n + 6m)(2n + 5m)(3n + 4m)


= (n + 7m m)(2n + 7m 2m)(3n + 7m 3m)
= (n m + 7m)[2(n m) + 7m][3(n m) + 7m]
= (k + 7m)(2k + 7m)(3k + 7m),

onde k = n m.
Afirmamos que se N e m
ultiplo de 7, entao k e m
ultiplo de 7. De fato, como 7 e primo
ultiplo de 7. Temos:
e divide N , entao um dos fatores k + 7m, 2k + 7m ou 3k + 7m e m
OBMEP 2009

97

5
9

Solucoes do Nvel 3

Lista 4

k + 7m
k
(i) Se k + 7m e m
ultiplo de 7, entao
= + m e inteiro, logo k e m
ultiplo
7
7
de 7. Segue que 2k e 3k tambem sao m
ultiplos de 7 e portanto os tres fatores
k + 7m, 2k + 7m e 3k + 7m sao m
ultiplos de 7. Conclumos que N e m
ultiplo de
73 .
2k
2k + 7m
=
+ m e inteiro, logo 2k e m
ultiplo
(ii) Se 2k + 7m e m
ultiplo de 7, entao
7
7
de 7. Como 2 e 7 sao primos entre si, segue que k e m
ultiplo de 7, o que leva ao
caso anterior.
(iii) Se 3k + 7m e m
ultiplo de 7, analogamente conclumos que k e m
ultiplo de 7.
4. Os
angulos 15 e 75 Uma vez que DB e diagonal do quadrado
de lado 1 cm, pelo
Teorema de Pitagoras, temos que DB 2 = 11 + 12 implica DB = 2 .
Recordemos que:
cos 60

1
= ;
2

sen 60
= 3;

cos 60

3
;
2

sen 30
3
.
tan 30 =
=
cos 30
3

sen 30

tan 60 =

sen 60

cos 30

(a) O tri
angulo BCE e equil
atero, logo seus angulos internos valem 60 . A partir dessa
informacao obtemos os angulos assinalados na figura.
1

30

60

F
60

30

No CDF temos:

1
CD
3

=
. Como sen 60 =
, temos:
=
DF
DF
2

2 3
2
3
1
=
DF = =
.
DF
2
3
3

sen 60

CF
CF
1
. Como cos 60 = , temos:
=
DF
2
2 3/3

CF
3
1
CF =
=
.
2
3
2 3/3

Ainda no CDF temos: cos 60 =

98

OBMEP 2009

Lista 4

Solucoes do Nvel 3

Segue que BF = 1

3
. Temos agora:
3

3 3
1
FN
FN

FN =

=
=

BF
2
6
1 3/3

3
31
BN
BN

cos 30 =

=
.
BN =
3
BF
2
2
1
sen 30

Assim, calculamos os tres lados do triangulo DBN :


DB =

2;

1+ 3
2 3 3 3
+
=
;
DN = DF + F N =
3
6
2

31
.
BN =
2

= 15 .

= 45 + 30 = 75 , donde conclumos que B DN


(b) No DBN temos: DBN
Assim temos:

15

3 +1
2

3 -1

cos 15 =

DN
DB

75

cos 75 =

BN
DB

31

6 2
2

=
4
2

1+ 3

6+ 2
2

=
4
2

Como sen 15 = cos 75 e cos 15 = sen 75 , o


exerccio esta completo.

5. Circunf
encias tangentes -

(a) Na figura est


ao desenhadas as duas circunferencias
concentricas, de raios r e R, e uma circunferencia de raio
x simultaneamente tangente a essas duas. Logo, temos:
Rr
.
r + 2x = R donde, x =
2

OBMEP 2009

99

5
9

Solucoes do Nvel 3

Lista 4

(b) Na figura ao lado temos 2 circunferencias tangentes


de raio x, e tambem tangentes `as 2 circunferencias
concentricas de raio r e R. Os pontos A, B e C sao
os centros destas circunferencias.

x
r

Para tracar 12 circunferencias de raio x na regi


ao entre as 2 circunferencias concentricas,
360

= 30 .
deve-se ter ACB =
12
T

Se T e o ponto de tangencia das circunferencias de raio

= 15 .
x, T e ponto medio do segmento AB e ACT

x
15

r+x

x
Rr
AT
=
. Mas x =
e, do
Nesse triangulo retangulo temos sen 15 =
AC
r
+
x
2

6 2
. Da conclumos que
problema anterior, sen 15 =
4

6 2
Rr
=
.
R+r
4
Dividindo por r o numerador e o denominador do membro esquerdo dessa igualdade
encontramos
R

1
q

1
R
6

2
r
=
=
, onde q = .
R
q+1
4
r
+1
r
Segue que

4+ 6 2
R

.
=
q=
r
4 6+ 2

100

OBMEP 2009

Lista 5

Solucoes do Nvel 3

Lista 5
1. Mudando a base Em um triangulo is
osceles, a altura relativa a` base coincide com a
mediana. Tracando esta altura, obtemos dois tri
angulos ret
angulos com catetos medindo
h e 5, e hipotenusa 13. Pelo Teorema de Pitagoras temos:

h2 + 52 = 132 h2 = 132 52 = 144 h = 144 = 12.


Logo a area do tri
angulo e A =

bh
10 12
=
= 60 cm2 .
2
2

Vamos agora colar os 2 tri


angulos ret
angulos ao longo do lado
medindo 5, obtendo um tri
angulo is
osceles com base 12 + 12 = 24 m,
os lados com 13 cm e a altura relativa a base igual a 5 cm. Logo, este
24 5
= 60 cm2 .
novo tri
angulo is
osceles tem tambem area igual a
2

2. Clube de Matem
atica Sejam H e M os n
umeros de homens e mulheres, respectivamente, no clube. Temos duas possibilidades: Se eu sou menino, temos M = H 1.
Quando falta um menino, o n
umero total de pessoas no clube e
M + H 1 = H 1 + H 1 = 2H 2.
Logo:
3
3
M = (2H 2) H 1 = (2H 2) H = 1 .
4
4
Logo, M = 11 = 0, o que n
ao e possvel. Logo eu sou uma menina, entao M = H +1
e temos
3
H + 1 = (2H + 1 1) H = 2 e M = 3.
4
3. Uma calculadora diferente Para calcular (2 3) + (0 3) utilizamos as propriedades
(i), (ii) e (iii). Ent
ao
(2 3) + (0 3)

(iii)

(i) (ii)

(2 0) + (3 3)
2 2 + 3 = 7.

Para calcular 1 024 48, observe que 1 024 = 976 + 48. Temos:
1 024 48 = (976 + 48) (0 + 48)
= (976 0) + (48 48)
= 976 2 + 48
= 1 952 + 48 = 2 000.
OBMEP 2009

101

5
9

Solucoes do Nvel 3

Lista 5

4. Ret
angulo m n Sejam m e n respectivamente, o n
umero de segmentos de 0, 5 cm
sobre dois lados consecutivos do ret
angulo. Sabemos que o n
umero total de segmentos de
0, 5 cm na divisao do ret
angulo em mn quadrados de lado 0, 5 cm e: m(n+1)+n(m+1)
(prove isso). Assim,
m(n + 1) + n(m + 1) = 1 997

n=

1 997 m
.
2m + 1

Alem disso, um dos lados considerados e menor ou igual ao outro, digamos: m n .


Nesse caso podemos concluir que m 31, pois
n m n(m + 1) + m(n + 1) 2m(m + 1).
Logo 1 997 2m(m + 1) e como 1 998 > 1 997 segue que
1 998 > 2m(m + 1) 999 > m(m + 1).
Da conclumos que m < 32.
Por outro lado temos que
n=

3 994 2m
3 995 (2m + 1)
3 995
1997 m
2n =
=
2n =
1.
2m + 1
2m + 1
2m + 1
2m + 1

Assim, a questao se resume agora em pesquisar os divisores de 3 995 = 5 17 47.


Os u
nicos valores de m que atendem a condicao 1 m 31 sao m = 2, m = 8 e
m = 23, que correspondem, respectivamente, aos divisores 5, 17 e 47. Para esses valores
de m temos n = 399, n = 117 e n = 42 respectivamente. Os outros divisores dar
ao
configuracoes equivalentes (trocando m por n).
Portanto, Rosa pode ter construdo 3 configuracoes diferentes com os 1 997 segmentos.
A primeira com 2 399 quadrados, a segunda com 8 117 quadrados e a terceira com
23 42 quadrados.
5. Cercando o Globo Terrestre Como o raio da Terra e muito grande, e foi dado apenas
um acrescimo de 1 m no comprimento do fio, parece que a folga entre o fio e o Equador
e muito pequena. Mais ainda, se trocarmos o Globo Terrestre por J
upiter ou por uma
bolinha de gude e realizarmos esta mesma experiencia, parece que a altura da folga entre
o fio aumentado e o equador da esfera tambem muda, sendo que quanto maior a esfera
considerada, menor e a folga entre o fio e o equador da esfera.
Vejamos que esta ideia intuitiva e falsa e que a altura da folga, entre o fio e o Equador, e
de aproximadamente 16 cm, independentemente do raio da esfera em que a experiencia
e realizada.
Consideremos um crculo de raio R. Seu comprimento
e igual a 2R. Vamos considerar tambem um crculo
de mesmo centro, mas que tenha comprimento igual a
2R + 1.

102

OBMEP 2009

Lista 5

Solucoes do Nvel 3

Este crculo tem raio igual a R + h, sendo h a altura da folga entre os dois crculos.
Como um crculo de raio R + h tem comprimento 2(R + h) obtemos a igualdade
1
1

0, 16.
2(R + h) = 2R + 1. Simplificando esta expressao obtemos h =
2
6, 28
Portanto, para qualquer valor de R, a altura da folga e de aproximadamente 16 cm.
Assim, somente a formiga e capaz de passar por debaixo do fio.

OBMEP 2009

103

5
9

Solucoes do Nvel 3

Lista 6

Lista 6
1. Comprimento de uma corda Sendo AB um
di
ametro, o tri
angulo ABC esta inscrito numa semicircunferencia. Isto implica que este triangulo e retangulo
no vertice C. Pelo Teorema de Pitagoras,
BC 2 = AB 2 AC 2 ,

C
12 cm

20 cm

ou seja,
BC 2 = 202 122 = 256 = 162 .
Assim, obtemos que BC = 16.
2. Dois irm
aos Sejam

xy =
z1 =

z + 20 =

x, y as idades atuais dos dois

2[(x 1) + (y 1)]

(x + 20) + (y + 20)

irm
aos, e z a idade do pai. Temos:
xy
z1

= 3
= 2x + 2y 4 .

z + 20 = x + y + 40

Uma maneira simples de obter z e multiplicar a 3a equacao por 2 e do resultado subtrair


a 2a : 2z + 40 (z 1) = 80 (4), o que implica z = 43.
Vamos calcular agora a idade dos filhos usando as duas primeiras equacoes:

xy = 3
x =y+3
43 1 = 2x + 2y 4 x = 23 y
Obtemos 2x = 26, donde x = 13 e y = 10.
3. Canelonis de ricota Colando os ret
angulos de massa ao longo do maior lado, Pedro
obtem um cilindro de base circular com 10 cm de comprimento e 16 cm de altura. O
volume entao que ele recheia com ricota e o volume desse cilindro:
V = area da base altura .
ao calcular o raio
A area da base e dada por r 2 , onde r e o raio da base. Vamos ent
sabendo que o comprimento da base e 10 cm; temos:
2r = 10 r =

5
.

Logo, o volume de ricota para cada caneloni e


V =
104

400
16 25
52
=
cm3 .
16 =
2

OBMEP 2009

Lista 6

Solucoes do Nvel 3

Agora, colando os ret


angulos de massa ao longo
do menor lado, Pedro obtem um cilindro de base
circular com 14 cm de comprimento e 12 cm de
14
7
= , logo o
altura. O raio da base e r  =
2

volume de ricota para cada caneloni ser


a:
V =

72
588
cm3 .
12 =
2

Finalmente, para calcular o novo gasto com ricota, usamos a seguinte Regra de Tres
direta:
Ricota(g)
Volume (cm3 )
400

500

588

Segue que
x=

500 588
= 735 g .
400

4. C
alculo de segmentos O tri
angulo ABP e retangulo com catetos AB = 1 200
e BP = 150 + 350 = 500. Pelo Teorema de Pitagoras, temos:
AP 2 = 1 2002 + 5002 = (144 + 25) 104 = 169 104 = (13 102 )2 .
Logo, AP = 13 102 = 1 300 m .
Analogamente, considerando o tri
angulo ret
angulo P CD, temos:
DP 2 = 3502 + 1 2002 = (72 + 122 22 )(52 102 ) = 252 502 = DP = 1 250 m .
Os tri
angulos P CQ e P BA sao ret
angulos com um angulo em comum, logo sao
semelhantes; segue que:
PC
CQ
PQ
=
=
.
PA
PB
AB
Substituindo os valores conhecidos temos:
350
CQ
PQ
=
=
.
1 300
500
1 200
Logo,
PQ =

350 1 300
= 910 m
500

CQ =

350 1 200
= 840 m .
500

5. Pr
a chegar junto! Sabemos que espaco = velocidade tempo .
Sejam v e v  as velocidades de Ana e de Luza, respectivamente, e t o tempo que
Luza gasta para percorrer os 3 000 m. Logo, nesse mesmo tempo t, Ana percorre
OBMEP 2009

105

5
9

Solucoes do Nvel 3

3 000 120 = 2 880 m. Temos:

3 000 = v t

e
3 000 120 = v  t t =

3 000
2 880
.
=
v
v

v
24
=
.
v
25
Se denotarmos por x a dist
ancia que Luza percorrera a mais temos:

Portanto,

3 000 + x = v t
e
3 000 = v  t

3 000 + x
3 000
3 000
v

=
=
.
v
v
3 000 + x
v

Segue que

v
24
3 000
=
=
x = 125 .
3 000 + x
v
25
Logo, a resposta e 125 m.

106

OBMEP 2009

Lista 6

Lista 7

Solucoes do Nvel 3

Lista 7
1. Um professor enfurecido Quem teve x como nota mensal vai ter um desconto de
x% sobre essa nota, ou seja vai perder
x% de x =

x
x2
x =
.
100
100

x2
, onde x era a nota inicial.
100
x2
. Como as
Consideremos a funcao nota depois do castigo dada por f (x) = x
100
notas maximas e mnimas s
ao 0 e 100, vamos considerar essa funcao no domnio [0, 100],
ou seja, para 0 x 100. O gr
afico de f e uma par
abola com concavidade para baixo,
1
b
= 50. Sendo assim, a maior
=
e seu valor maximo ocorre no vertice: x =
2
2a
100
nota depois do castigo e para os alunos que antes do castigo tiraram 50. Essa nota e
Logo, depois do castigo, a nota fica sendo x

f (50) = 50

502
= 25 .
100

O valor mnimo dessa funcao e 0 ocorre em x = 0 e x = 100. Logo a menor nota ocorre
para os alunos que tiraram 0 ou 100(!!!!!) antes do castigo. De fato, f (0) = f (100) = 0.

2. O percurso de um atleta
Polo
Norte

O Polo Norte da Terra e o ponto mais f


acil de ser identificado
como solucao: Saindo o atleta do Polo Norte, correndo 5 km
para o sul, depois 5 km para o leste e finalmente 5 km para o
norte, ele volta novamente para o Polo Norte.
Vamos determinar um outro ponto sobre a Terra que satisfaz as hip
oteses do problema.
Consideremos um paralelo (linha paralela ao Equador) de comprimento 5 km. Existem
dois deles: um pr
oximo ao Polo Norte e outro pr
oximo ao Polo Sul. Vamos denotar
OBMEP 2009

107

5
9

Solucoes do Nvel 3

Lista 7

por C1 o que esta mais pr


oximo do Polo Sul. Denotemos por C2 o paralelo que est
a
5 km de distancia de C1 , medida ao longo de um meridiano. Afirmamos que qualquer
ponto A sobre o paralelo C2 satisfaz as hip
oteses do problema. De fato, saindo de A
e caminhando 5 km para o sul, chega-se a um ponto B do paralelo C1 . Como C1 tem
comprimento 5 km, saindo de B e caminhando 5 km para leste retorna-se novamente
para B.

C2

Finalmente, saindo de B e caminhando


5 km para o norte, retorna-se novamente
para o ponto de partida A.

C1

Polo
Sul

Paralelo com 5km


de comprimento

3. Areas
iguais Sejam T a area do tri
angulo ABC, a e c as areas sombreadas na
figura dada e b e d as areas compreendidas entre os catetos do triangulo e o semicrculo
de di
ametro AB.
A area a + b e a area do semicrculo de di
ametro
AB:


AB 2 AB 2
1
.
=
a+b =
2
2
8
A area c + d e a area do semicrculo de di
ametro
BC:


BC 2 BC 2
1
.
=
c+d=
2
2
8

b
T

A area b+d+T e a area do semicrculo de di


ametro
AC:


AC 2 AC 2
1
.
=
b+d+T =
2
2
8
Portanto,
(a + b) + (c + d) =
Como b + d =

108

AB 2 BC 2
+
.
8
8

AC 2
T temos
8


AB 2 BC 2
AC 2
T =
+
,
(a + c) +
8
8
8
OBMEP 2009

B
d
c

Lista 7

Solucoes do Nvel 3

ou equivalentemente,
(a + c) +




AC 2 =
AB 2 + BC 2 + T .
8
8

Uma vez que AC 2 = AB 2 + BC 2 , pelo Teorema de Pitagoras, podemos simplificar


a igualdade acima e obter a + c = T . Esta igualdade implica que a soma das areas
sombreadas e igual a area do tri
angulo ret
angulo ABC.
4. Func
ao definida por
area
(a) A reta r passa pelo ponto (0, 2), logo tem equacao y = mx + 2. Como ela passa
pelo ponto (2, 0), verifica-se que 0 = 2m + 2, que implica m = 1. Assim, r tem
equacao y = x + 2.
A reta s passa pelo ponto (0, 6) logo, y = mx+6 e como passa tambem pelo ponto (3, 0),
verifica-se que 0 = 3m + 6, que implica m = 2. Logo, s tem equacao y = 2x + 6.
angulo ABC mais a area do trapezio BOCD, sendo A o ponto
(b) f (0) e a area do tri
de encontro de r e s.

Para determinar A fazemos: x + 2 = 2x + 6 de onde x = 4/3. Substituindo esse valor


na equacao de r ou s obtemos y = 10/3. Logo, A = (4/3, 10/3). A altura do tri
angulo
ABC, em relacao a` base BC, e h = 10/3 2 = 4/3. O ponto C pertence `a reta s e
tem y = 2, logo tem-se 2 = 2x + 6 ou seja x = 2. Ent
ao C = (2, 2). Logo, a area do
4
3+2
4 1
= 5.
tri
angulo ABC e igual a 2 = e a area do trapezio BOCD e 2
3 2
3
2
Logo,
19
4
.
f (0) = 5 + =
3
3
(c) f (y) e igual a f (0) menos a area do trapezio de altura y e bases 3 e x, sendo x a
abscissa do ponto da reta s que tem ordenada y, logo
x=

6y
.
2

OBMEP 2009

109

5
9

Solucoes do Nvel 3

Lista 7

Da temos
f (y) =

6y
19 12y y 2
y2
19
19 3 + 2

y=

=
3y +
.
3
2
3
4
4
3

19
y2
3y +
e uma par
abola c
oncava
4
3
3
para cima. As coordenadas do vertice V sao: x = = 6 e
2
4
62
19
19
8
y = f (6) =
3.6 +
= 9 +
= .
4
3
3
3
8
Logo V = (6, ).
3
O gr
afico de f (y) =

(d)

Como f (2) =

4
o gr
afico de f , com 0 y 2 e a parte em linha grossa.
3

5. PA e PG Os 4 termos de uma progress


ao aritmetica de raz
ao r podem ser escritos
como:
x 2r , x r , x , x + r .
Logo, os 3 termos da progressao geometrica de raz
ao q serao
x 2r , x , x + r ,
onde
x = (x 2r)q e x + r = xq .
Da segue que:
x = xq 2rq x = x + r 2rq q =
Obtemos que x + r =

1
.
2

x
x
r = . Logo a progressao aritmetica e da forma:
2
2

x
3x
, x, .
2
2
Escolhendo um valor para x, por exemplo x = 1, obtemos 4 n
umeros formando uma
progressao aritmetica 2 , 3/2 , 1 , 1/2 de raz
ao 1/2 tais que 2 , 1 , 1/2 formam
uma progressao geometrica de raz
ao 1/2. Note que esse problema tem uma solucao
para cada escolha de x, portanto tem um infinidade de solucoes.
2x ,

110

OBMEP 2009

Lista 8

Solucoes do Nvel 3

Lista 8
1. Plano cartesiano Comecemos examinando alguns casos.
f (1) e o n
umero de pontos inteiros sobre o segmento que liga (0 , 0) ao ponto
(1 , 4). Logo, f (1) = 0 .
f (2) e o n
umero de pontos inteiros sobre o segmento que liga (0 , 0) ao ponto
(2 , 3). Logo, f (2) = 0 .
f (3) e o n
umero de pontos inteiros sobre o segmento que liga (0 , 0) ao ponto
(3 , 6). Como nesse segmento estao 2 pontos inteiros (1 , 2) e (2 , 4), segue que
f (3) = 2 .
6

6
5

.
..
..
..
..
...
.
..
..
..
...
..
..
..
..
...
.
..

.
...
...
....
..
...
...
....
..
...
...
....
..
...
...
....
..
..

4
2

...
...
...
....
..
...
...
....
..
...
...
....
..
...
...
....
..
...
...
....
.

Vejamos, agora o caso geral. Note que se um ponto inteiro (x , y) esta sobre o segmento
que une (0 , 0) a (n , n+3), sem ser um dos extremos, entao 0 < x < n e 0 < y < n+3 .
Vamos precisar do seguinte resultado:
Lema: Se n n
ao e m
ultiplo de 3, entao n e n + 3 sao primos entre si.
Demonstracao: Suponhamos que o mdc entre n e n + 3 seja d > 1. Ent
ao d divide n
e n + 3, portanto d divide (n + 3) n = 3. Logo, como d > 1, teremos d = 3, o que
n
ao e possvel porque partimos da hipotese que 3 n
ao divide n.
Se 3 n
ao divide n entao f (n) = 0 .
Isso equivale a dizer que nao h
a pontos inteiros sobre o segmento que une (0 , 0) a
(n , n + 3), excludos os extremos.
De fato, suponhamos que esse segmento contenha um ponto inteiro (x , y), entao
n
x
=
.
y
n+3
n
esta na forma irredutvel, logo, x seria m
ultiplo de n, o
n+3
que n
ao pode acontecer porque x < n.

Pelo lema, a fracao

Se 3 divide n entao f (n) = 2 .


Se n = 3k, com k inteiro, devemos achar o n
umero de pontos inteiros no segmento
que une (0 , 0) ao ponto (3k , 3k + 3). Seja (x , y) um desses pontos, entao
3k
k
x
=
=
.
y
3k + 3
k+1
OBMEP 2009

111

5
9

Solucoes do Nvel 3

Lista 8

Sendo a u
ltima fracao irredutvel, deduzimos que x e m
ultiplo de k, e como
0 < x < 3k, segue que x = k ou x = 2k. Os pontos inteiros sao (k , k + 1)
e (2k , 2k + 2). Assim, temos f (n) = 2.
2. Trabalhando com quadril
atero Lembre que, num
tri
angulo, qualquer lado e maior que a diferenca e menor
que a soma dos outros dois. Do triangulo ADB temos
AD AB < BD < AD + AB, e do tri
angulo CBD segue
que BC CD < BD < BC + CD. Sustituindo os valores
conhecidos obtemos:

...
....................... ......
....
....
.
....
.......
.
....
.
.
.
.
....
.
.
.
.....
..
.
....
.
.
.
.
.
.
.
.
.....
....
.
...
.
.
.
.
.
.
.
....
.. ............
.
.
....
.
.. ...........
.
....
.
.
..............
...
.
.
.
......................................................................................................................................................................

A.............................................................................

9 5 < BD < 5 + 9 e 17 5 < BD < 17 + 5,


ou seja,
4 < BD < 14

e 12 < BD < 22 .

Das duas desigualdades conclumos que:


12 < BD < 14 .
Como BD e inteiro, so podemos ter BD = 13.
3. O tri
angulo de Reuleaux O tri
angulo de Reuleaux e formado por 4 regi
oes: um
tri
angulo equil
atero e tres calotas. Cada calota e um sexto de um crculo de raio 1 do
qual foi retirado um tri
angulo equil
atero de lado 1.
Pelo Teorema de Pitagoras, a altura do tri
angulo equil
atero e :

 2
1
3
;
=
h= 1
2
2
logo a area do tri
angulo vale:
1
2

3
2

3
cm2 .
4

A area de um setor circular e um sexto da area do crculo, ou seja, igual a . Logo, a


6
area da calota e a diferenca:

.
6
4
Portanto, a area do tri
angulo de Reuleaux e



3
3
3

+
=
cm2 .
3
6
4
4
2
2
4. Intersec
ao entre circunfer
encias Seja G o baricentro (encontro das medianas) do
tri
angulo ABC. Como a figura e invariante por rotacoes de 60 ao redor do ponto G,
temos que o triangulo XY Z e equilatero, e que G tambem e o seu baricentro.
112

OBMEP 2009

Lista 8

Solucoes do Nvel 3

Vamos calcular o comprimento L do seu lado.


Seja CM a altura do tri
angulo ABC em relaca
o a` base AB. Uma vez que a altura de
a 3
, e que o baricentro divide a altura
um tri
angulo equil
atero de lado a tem medida
2
em dois segmentos, um com o dobro do comprimento do outro, temos que:

1
a 3
2
a 3
a 3
,
GM = CM =
e CG = CM =
.
CM =
2
3
6
3
3

Como AZ = BZ = r vemos que o ponto Z esta na reta mediatriz do segmento AB.


Entretanto, esta mediatriz e a reta suporte da altura CM do tri
angulo ABC. Isto implica
que os pontos C, G, M e Z estao alinhados, e que o triangulo M ZB e retangulo.

Pelo Teorema de Pitagoras, obtemos:


MZ =


ZB 2 M B 2 =

r2

a2
4

a2
a 3
+ r2
.
GZ = GM + M Z =
6
4
Agora vamos considerar a altura N Z do
angulo XY Z em relacao a` sua base XY .
tri
2
L 3
L 3
, e como GZ = N Z =
, conclumos
Como esta altura tem comprimento
2
3
3
que


a 3
a2
L 3
=
+ r2
.
GZ =
3
6
4
e

OBMEP 2009

113

5
9

Solucoes do Nvel 3

Lista 8

Esta u
ltima igualdade implica que

a2
a
.
L = + 3 r2
2
4
5. Valor m
aximo - Estamos procurando o valor de k para o qual e maximo o termo da
sequencia:
12
22
32
k2
,
,
,
.
.
.
,
, ...
1, 001 1, 0012 1, 0013
1, 001k
Considere as seguintes inequacoes equivalentes:
k2
(k + 1)2
1, 001k2
(k + 1)2
<

< 0.
1, 001k+1
1, 001k
1, 001k+1
1, 001k+1
A segunda inequacao tem denominadores iguais e positivos, logo ela e equivalente a
(k + 1)2 1, 001k2 < 0 k(k 2 000) > 1 000 k > 2 000 .
Assim, a sequencia decresce estritamente para k 2 001 e cresce estritamente para
k 2 000. Logo, o maior termo da sequencia corresponde a k = 2 001.

114

OBMEP 2009

Lista 9

Solucoes do Nvel 3

Lista 9
1. Moedas falsas
(a) Aladim deve retirar de cada saco um n
umero diferente de moedas, do seguinte
modo: retira uma moeda do primeiro saco, duas do segundo, tres do terceiro, e assim
sucessivamente, ate o u
ltimo saco de onde retira as dez moedas.
Ao todo foram retiradas 1 + 2 + 3 + 4 + 5 + 6 + 7 + 8 + 9 + 10 = 55 moedas que sao
colocadas na balanca.
Se todas essas moedas fossem verdadeiras, pesariam 55 10 = 550 g. Mas, como
algumas s
ao falsas, o peso ser
a menor. Se faltar um grama e porque h
a somente uma
moeda falsa e, portanto, o primeiro saco e o procurado. Se faltarem dois gramas, significa
que as duas moedas falsa s
ao do segundo saco, e assim sucessivamente.
(b) Vejamos que uma tentativa de solucao como a anterior n
ao permite a identificacao
dos sacos com moedas falsas. Suponhamos que Aladim retirou uma moeda do primeiro
saco, duas moedas do segundo, e assim sucessivamente, ate o u
ltimo saco, de onde
ele retirou dez moedas. Se existissem dois ou mais sacos com moedas falsas, esse
procedimento de pesar estas 55 moedas pode ser inconclusivo. Por exemplo, suponhamos
que na pesagem das 55 moedas faltassem 7 g, ou seja, foram pesadas 7 moedas falsas.
Neste caso poderiam existir moedas falsas nos sacos 1 e 6; moedas falsas nos sacos 2
e 5; moedas falsas nos sacos 1, 2 e 4 etc. Ou seja, procedendo dessa maneira n
ao e
possvel identificar quais sacos s
ao de moedas falsas.
Para resolver esse problema, ele pode proceder do seguinte modo: retira 1 moeda do
primeiro saco, 2 moedas do segundo saco, 4 moedas do terceiro saco, 8 moedas do
quarto saco, 16 moedas do quinto saco etc. Sempre dobrando o n
umero de moedas
retiradas do saco anterior. Ao todo s
ao retiradas
1 + 2 + 4 + 8 + 16 + 32 + 64 + 128 + 256 + 512 = 1 023 moedas,
que pesariam juntas 10 230 g, se todas as moedas fossem verdadeiras. A diferenca
entre o peso real obtido na pesagem dessas moedas e o peso ideal (10 230 gramas)
indica a quantidade de moedas falsas pesadas e em quais os sacos elas estao. Vejamos
isso atraves de um exemplo: imaginemos que na pesagem foram obtidos 10 125 g, ou
seja, faltaram 10 230 10 125 = 105 g, que corresponde ao n
umero de moedas falsas.
Retirando sucessivamente os n
umeros correspondentes `as moedas retiradas de cada saco,
comecando sempre do maior n
umero temos: 105 64 = 41; 41 32 = 9; 9 8 = 1, ou
seja, 105 = 1 + 8 + 32 + 64. Desse resultado Aladim pode concluir que foram retiradas
1, 8, 32 e 64 moedas falsas do 1o , 4o , 6o e 7o saco.
Vamos agora justificar, de um modo mais formal, o raciocnio desenvolvido no exemplo
numerico.
Seja p o peso obtido com a pesagem das 1 023 moedas. A diferenca 10 230 p e o
n
umero de moedas falsas retiradas dos sacos.
OBMEP 2009

115

5
9

Solucoes do Nvel 3

Lista 9

Efetuando divis
oes sucessivas por 2 pode-se provar que qualquer n
umero inteiro positivo
se escreve, de maneira u
nica, como uma soma de potencias de 2. Isso implica que
10 230 p = 1 a0 + 2 a1 + 22 a2 + 23 a3 + 24 a4 + + 29 a9
em que cada um dos n
umeros a0 , a1 , a2 , . . . , a9 e zero ou um.
De cada saco foram retiradas quantidades de moedas que sao potencias de 2 e cada
saco ou contem moedas falsas ou contem moedas verdadeiras, isto e, em um saco nao
existem os dois tipos de moedas. Da temos que se algum desses n
umeros, digamos aj
j
e 1, entao do saco j + 1 foram retiradas 2 moedas falsas. Por outro lado, se o n
umero
aj e 0, entao do saco j + 1 foram retiradas 2j moedas verdadeiras.
2. Menor inteiro Como q = 2 005 p, temos
p
7
5
<
< ,
8
2 005 p
8
do qual segue que
5(2 005 p) < 8p

e 8p < 7(2 005 p) .

Logo,

7 2 005
5 2 005
<p<
771, 15 < p < 935, 66.
13
15
Logo 772 e o menor valor de p que satisfaz as condicoes do problema.

3. Mais
areas... Observe que a altura h, relativa ao lado AB, de todos os triangulos
ABC que tem o vertice C sobre a reta x + y = 7, e a mesma, pois esta u
ltima reta
e paralela a` reta que passa por A e por B. Logo, esses triangulos tem todos a mesma
area, a saber:
AB h
.
2
Precisamos, entao determinar AB e h. Como AB e a hipotenusa de um tri
angulo
retangulo que tem os dois catetos iguais a 7 4 = 3, segue do Teorema de Pit
agoras
que:


AB = 33 + 32 = 18 = 3 2 .
Falta calcular h, que e a dist
ancia entre as retas paralelas. A reta x+y = 7 e determinada
pelos pontos C = (7, 0) e D = (0, 7). A reta x = y e perpendicular a`s retas paralelas
acima e forma um angulo de 45 com o eixo OY . Seja M o pe da perpendicular a` reta
x + y = 7 tracada a partir de B. Portanto, o tri
angulo BM C e retangulo is
osceles com
catetos iguais a h e hipotenusa 7 3 = 4 cm.
Do Teorema de Pitagoras segue que:

42 = h2 + h2 h = 2 2 .
116

OBMEP 2009

Lista 9

Solucoes do Nvel 3

Finalmente, a area procurada e:

3 22 2
= 6.
2
4. Circunfer
encias tangentes Ligando os centros das tres circunferencias obtemos o
tri
angulo ABC de lados AB = 3 cm, AC = 4 cm e BC = 5 cm. Como 32 + 42 = 52 ,
esse triangulo e retangulo, com hipotenusa BC.

Construa o retangulo ABDC, fazendo uma c


opia BCD, congruente ao triangulo
ABC e com lado comum BC.

Uma vez que DC = AB = 3 e que a circunferencia de centro C tambem tem raio 3 cm,
vemos que o ponto D esta sobre essa circunferencia.
Ligando o ponto D a cada um dos vertices do triangulo ABC e prolongando esses
segmentos ate interceptarem as circunferencias, obtemos os pontos P1 , P2 e P3 .

OBMEP 2009

117

5
9

Solucoes do Nvel 3

Lista 9

Temos que:
DP2 = DB + BP2 = CA + BP2 = 4 + 2 = 6.
DP1 = DA + AP1 = 5 + 1 = 6.
DP3 = DC + CP3 = 3 + 3 = 6.
Deste modo DP1 = DP2 = DP3 = 6. Assim se considerarmos a circunferencia de
centro D e raio 6 cm vemos que esta circunferencia passa por P1 , P2 e P3 . Alem
disso, como os pontos {D, A, P1 }, {D, B, P2 } e {D, C, P3 } estao alinhados, segue que
a circunferencia de centro D e raio 6 cm e tangente `as tres circunferencias dadas de
centros A, B e C.

5. Soma
finita
os poss
x2k1 x2k
onde k {1, 2, ...,
Temos que
veis produtos

2 004} sao

( 2 1)( 2 1) = 3 2 2 , ( 2 + 1)( 2 + 1) = 3 + 2 2 e ( 2 1)( 2 + 1) = 1.

ao iguais a 3 + 2 2 e
Suponha que a produtos s
ao iguais a 3 2 2 , b produtos s
1 002 a b produtos s
ao iguais a 1.
A soma e igual a

a(3 2 2) + b(3 + 2 2) + 1 002 a b = 1 002 + 2a + 2b + 2(b a) 2 .


Assim, para que a soma seja inteira, devemos ter a = b. Logo a soma e igual a 1 002+4a.
Como a varia de 0 a 501 (pois a+ b n
ao pode ser maior que 1 002), a soma pode assumir
502 valores inteiros distintos.

118

OBMEP 2009

Lista 10

Solucoes do Nvel 3

Lista 10
1. M
ultiplos As condicoes do problema equivalem a dizer que:
2a 5 = 2(a + 1) 7 = 2(a + 2) 9 = 2(a + 3) 11,
e m
ultiplo de 5, 7, 9 e 11, donde e m
ultiplo de 5 7 9 11 = 3 465. Assim, o menor valor
de a e tal que 2a 5 = 3 465, ou seja, a = 1 735.
2. Equac
ao de duas vari
aveis Temos:
9xy x2 8y 2 = 2 005 xy x2 + 8xy 8y 2 = 2 005
x(y x) + 8y(x y) = 2 005
(x y)(8y x) = 2 005().
Observemos que a fatoracao em primos de 2005 e 5 401.
Alem disso, a soma dos fatores x y e 8y x e 7y, que e m
ultiplo de 7. Devemos entao
escrever 2005 como produto de dois fatores, cuja soma e um m
ultiplo de 7. Para isso,
os fatores devem ser 5 e 401. A soma dos fatores e 406.
Assim, por () temos:

xy =5

x y = 401

e 8y x = 401
ou
e 8y x = 5

ou

x y = 5
e 8y x = 401

ou

x y = 401 e 8y x = 5

x = 63

x = 459

e y = 58
ou
e y = 58

ou

x = 63
e y = 58

ou

x = 459 e y = 58

As solucoes sao, portanto (63 , 58), (459 , 58), (63 , 58) e (459 , 58).
3. Trap
ezio ret
angulo

= B DC

= . Ent
Seja ABD
ao temos que
BD
e AD = BDsen, donde
DC =
cos
BD
1
2
DC
cos
=
=
=
2.
AD
BDsen
sencos
sen2
A igualdade ocorre quando sen2 = 1, ou
seja, quando = 450 .

OBMEP 2009

119

5
9

Solucoes do Nvel 3

Lista 10

4. Jogos de futebol Para cada grupo de 5 alunos, existe um u


nico time formado que
12.11.10.9.8
5 =
= 792 times para cada 5 alunos
os contem. Logo, contamos C12
5!
escolhidos. Por outro lado, em cada time de 6 jogadores, temos C65 = 6 modos de
escolhermos cinco jogadores, ou seja, existem 6 grupos de 5 jogadores que eram mesmo
792
= 132 .
time na nossa primeira contagem. Logo, o total de times formados e igual a
6
5. A soma dos algarismos de um n
umero
(a) Observe esses dois exemplos:
000 s(60 000) = 59 994 .
2
000 s(2 000) = 1 998 , 60
 
 
  
  
2103

6104

A partir deles e f
acil entender que se a e um algarismos entre 1 e 9, entao s(a 10k ) = a.
Da temos:
a 10k s(a 10k ) = a 10k a = a(10k 1) = a 9 
9 = a 9 1 
1 .
k noves

k uns

Como todo n
umero pode ser decomposto em unidades, dezenas, centenas etc, isto e,
todo n
umero pode ser escrito na forma:
n = a0 + a1 10 + a2 102 + + ak 10k ,
temos que
9 .
n s(n) = a1 9 + a2 99 + ak 9 
k noves

Logo, a diferenca n s(n) e sempre divisvel por 9.


(b) Seguindo o mesmo raciocnio temos que: s(n) s(s(n)) e s(s(n)) s(s(s(n))) sao
divisveis por 9, logo n s(s(s(n) e divisvel por 9. Em particular 22 009 s(s(s(22 009 )))
e divisvel por 9, ou equivalentemente, 22 009 e s(s(s(22 009 ))) deixam o mesmo resto
quando s
ao divididos por 9.
Como 26 1 = 63 e divisvel por 9 entao, (26 )334 1 = 22 004 1 e divisvel por 9 e,
portanto, 22 009 25 e divisvel por 9. Como 25 = 32 deixa resto 5 quando dividido por
9, temos que 22 009 deixa resto 5 quando dividido por 9.
Por outro lado

22 009 < (29 )224 < (103 )224 = 10672 .

Assim, 22 009 tem menos que 672 algarismos e, portanto,


s(22 009 ) < 9 672 = 6 048;
s(s(22 009 )) 5 + 9 + 9 + 9 = 32;
s(s(s(22 009 ))) 2 + 9 = 13 .
Como o u
nico n
umero menor ou igual a 13 que deixa resto 5 quando dividido por 9 e o
5 temos que s(s(s(22 009 ))) = 5.
120

OBMEP 2009

5
9

Solucoes dos Desafios

Soluco
es dos Desafios
1. Data fatdica Resposta: 17.06.2345
2. Todos com o 2 Resposta: multiplicar por 3.
3. Tortas da vov
o Vamos examinar cada uma das situacoes propostas. Lembre que no
final vov
o recebeu 7 + 6 + 3 2 = 14 docinhos.
Impossvel porque ela recebeu no mnimo 3 2 = 1 docinho de chocolate.
Impossvel porque ela recebeu no mnimo 6 2 = 4 docinhos de coco.
Impossvel porque 7 2 = 5 > 3.
Possvel porque Sofia pode ter comido 1 docinho de amora e 1 de chocolate,
restando para vov
o: 6 de amora, 6 de coco e 2 de chocolate.
(E) Impossvel porque 7 n
ao e maior do que 6 + 2 3.

(A)
(B)
(C)
(D)

Logo, a u
nica situacao possvel e (D).
4. Famlia S
etimo Os nascimentos ocorreram em seis 1o de abril, logo existem irmaos
gemeos. Como nesse ano temos 2 bolos a mais que ha 2 anos atr
as, entao h
a 2 anos
atr
as o mais jovem ainda n
ao tinha nascido, o pen
ultimo filho tinha acabado de nascer,
e os gemeos ja tinham nascido. Atualmente o mais jovem tem 1 ano e os gemeos tem
x anos com x 3. Temos:
1
4 + 5 + 6 + x = 2 (1 + 2 + 3 + 4 + x 2) x = 5 .
 + 2 + 3 + 



n
umero de velas nesse ano

n
umero de velas 2 anos atras

Logo serao acesas 1 + 2 + 3 + 4 + 2 5 + 6 = 26 velinhas.


5. O Salta-Ficha
(a)
(b)
(c)
(d)
(e)

ficha
ficha
ficha
ficha
ficha

7
4
6
5
1

salta
salta
salta
salta
salta

sobre
sobre
sobre
sobre
sobre

as fichas 8 e 9
as fichas 5 e 6
as fichas 3 e 5
a pilha (4 , 8)
a pilha (6 , 2)

formando
formando
formando
formando
formando

uma
uma
uma
uma
uma

pilha com
pilha com
pilha com
pilha com
pilha com

a
a
a
a
a

ficha
ficha
ficha
ficha
ficha

10;
8;
2;
9;
3.

Veja o resultado:
...........
.... .....
..
....
..... .....
.........
...............
.
.
.
...
...
..
...
................

6
2

...........
.... .....
..
....
..... ......
........
................
.
.
.
..
...
..
...
................

1
3

...........
.... .....
.
.....
..... .......
.......
................
.
.
.
..
...
..
...
................

4
8

...........
.... .....
....
..
..... .....
.........
..............
.
.
.
...
.
....
.
.... .....
...........

...........
.... .....
....
.
.... .....
.........
...............
.
.
.
..
....
.
.... ....
..........

9 10

6. O menor Como 52 = 32 + 42 , temos 52 002 = (32 + 42 )1 001 . Sabemos que para a > 0
e b > 0,
(a + b)1 001 > a1 001 + b1 001 .
Logo, 52 002 > 32 002 + 42 002 .
OBMEP 2009

121

5
9

Solucoes dos Desafios


10a + b

7. O maior resultado Estamos procurando o maior valor de


, onde a e b reprea+b
sentam algarismos, pelo menos um diferente de 0. Temos
10a + b
10a + 10b 9b
10a + 10b
9b
9b
=
=

= 10
10 .
a+b
a+b
a+b
a+b
a+b
10a + b

= 10, teremos o maior resultado.


Logo, se conseguirmos encontrar a e b tais que
a+b
Note que isso ocorre quando b = 0, ou seja:
20
30
40
50
60
70
80
90
10
=
=
=
=
=
=
=
=
= 10 .
1
2
3
4
5
6
7
8
9
Logo, a resposta e 10.
8. Dois mil Observe que os n
umeros 189, 8 307 e 99 tem todos peso 18, e que 99
e o menor n
umero que pesa 18. Note que: para aumentar o peso de um n
umero e
minimizar o n
umero e preciso que o n
umero seja composto do maior n
umero possvel de
algarismos 9. Por outro lado, podemos dizer que o 0 esta eliminado dos algarismos a ser
considerados porque ele aumenta o n
umero sem aumentar o peso.
Temos que 2 000 = 9 222 + 2. Logo, o n
umero procurado tem ent
ao 222 algarismos
9, e um algarismo 2 ou dois algarismos 1. Eliminamos o caso dos n
umeros com dois
algarismos 1 porque eles tem 224 algarismos, e logo sao maiores do que os n
umeros que
possuem o algarismo 2 e tem 223 algarismos. Finalmente, o n
umero procurado tem 222
algarismos 9 e um 2. Logo esse n
umero e 299 . . . 999, com 222 algarismos 9.
9. No cabeleireiro Seja x o montante inicial no caixa. Esse montante mais o que os 3
clientes pagaram nos dara o caixa zerado.
a x + x 10 = 2x 10
O 1o cliente paga x 10. Depois do primeiro cliente, h
reais no caixa.
O 2o cliente paga (2x 10) 10 = 2x 20.
(2x 10) + (2x 20) = 4x 30 no caixa.

Depois do 2o cliente, h
a

a
O 3o cliente paga (4x 30) 10 = 4x 40. Depois do 3o cliente, h
(4x 30) + (4x 40) = 8x 70 no caixa, que sabemos ser igual a 0.
Logo, 8x = 70 e obtemos x = 8, 75 reais.
10. O macaco e a raposa 2 450 e o produto dos n
umeros primos 1, 2, 5, 5, 7, 7. As 3
idades correspondem a uma combinacao particular desses n
umeros ou de seus produtos.
A raposa n
ao pode descobrir as idades no incio porque pelo menos duas dessas combinacoes tem por soma o dobro de sua idade. De todas as combinacoes possveis,
somente {5, 10 49} e {7, 7, 50} tem a mesma soma 64.
Primeira conclus
ao: a raposa tem 32 anos.
Depois da nova dica do macaco, a raposa descobriu as idades porque pode eliminar uma
combinacao: aquela que contem dois n
umeros iguais, uma vez que um deles e o mais
jovem de todos.
Segunda conclus
ao: as pessoas tem 5, 10 e 49 anos.
122

OBMEP 2009

5
9

Solucoes dos Desafios

11. Nova sequ


encia Cada termo e a soma do termo precedente com os quadrados de
cada um de seus algarismos:
470 = 425 + 42 + 22 + 52 , 535 = 470 + 42 + 72 + 02 , . . .
Assim, os proximos termos sao: 870 e 983.
12. Ret
angulo quase quadrado A area e um n
umero da forma aabb, onde a e b representam algarismos; agora lembre que
aabb = 1 100a + 11b = 11(100a + b) .
Seja x a largura do terreno, logo
x(x + 1) = 11(100a + b) (I),
e deduzimos que x ou x+1 e um m
ultiplo de 11. Procurar m
ultiplos de 11 que satisfacam
a condicao (I) e bastante trabalhoso, por isso, para simplificar, vamos estabelecer quais
os valores que x pode ter. Vamos procurar os valores mnimo e maximo para x:
Mnimo: a menor area possvel e 1111, logo x(x + 1) = 1 111 x > 32 (II).
M
aximo: a maior area possvel e 9999, logo x(x + 1) = 9 999 x < 100 (III).
Agora procuramos x e x + 1 satisfazendo (I), (II) e (III).
33 34 = 1 122 ; 43 44 = 1 892 ; 44 45 = 1 980 ; 54 55 = 2 970 ; 55 56 = 2 970 ;
65 66 = 4 290 ; 66 67 = 4 422 ; 76 77 = 5 852 ; 77 78 = 6 006 ;
87 88 = 7 656 ; 88 89 = 7 832 ; 99 100 = 9 900 .
Encontramos 3 possibilidades para x: 33, 66 e 99.
13. Aonde est
a o erro? Esse deixamos para os alunos!

OBMEP 2009

123

Anotaco
es

Caros alunos e professores


Desde a sua primeira edio em 2005 o Banco de Questes (BQ)
mostrou ser um material motivante para alunos e professores. O seu
objetivo divulgar nas escolas pblicas problemas de olimpadas nacionais e internacionais, por isso grande parte do BQ no de questes
originais. A nossa idia que o BQ seja um material informal, no modelo apostila, que entusiasme alunos e professores. No h qualquer
preocupao com uniformidade de contedos ou de nveis de dificuldade dos problemas. A nossa principal meta : problemas interessantes que despertem o prazer de raciocinar.
A publicao do BQ completou 5 anos em 2009, perfazendo 678 problemas e suas solues, e 50 desafios. Esse material preparado nesses
anos serve agora para compor o BQ-2010, que apresentamos dividido
em 3 nveis. Os problemas foram agrupados em nveis apenas por uma
questo de organizao, encorajo todos os alunos a passearem pelos
3 nveis.

Banco
de
Questes

2005

2006

Voc pode ou no conseguir resolver os problemas, mas vale a pena


tentar... no desista na primeira vez, nem na segunda, nem na terceira...! Persistncia essencial e bom humor fundamental! assim
que se aprende Matemtica.

2007

Nesses 5 anos o BQ tem servido para timos bate-papos com alunos e


professores, que gentilmente me escrevem apontando erros de digitao
ou nas respostas. Espero continuar recebendo essas valiosas crticas e
sugestes.

2008

Um convite: se voc tem solues diferentes das apresentadas no BQ


envie para famf@impa.br que ns as publicaremos em nosso site.

2009

Agradeo a Eduardo Brietzke (UFRGS) e Claus Doering (UFRGS)


o trabalho dedicado e competente de reviso e, principalmente, de
enriquecimento de todo o material do BQ-2010.

2010

Suely Druck
Diretora Acadmica da OBMEP

OBMEP 2010

Texto j revisado pela nova ortografia.

ii

OBMEP 2010

Contedo
Nvel 1

Nvel 2

35

Nvel 3

69

Desafios

105

Solues do Nvel 1

114

Solues do Nvel 2

182

Solues do Nvel 3

259

Solues dos Desafios

347

Nvel 1

Nvel 1
1. Qual o nmero? Quando Joana entrou em sua
sala de aula, a professora estava apagando o quadro negro, mas ela ainda pde ver algo escrito, conforme mostra a figura. Qual o nmero que foi apagado?
(a) 8

(b) 9

(c) 11

(d) 12

(e) 13

2. Muro em 15 dias Um pedreiro capaz de assentar 8 metros de muro por dia.


Quantos metros de muro esse pedreiro consegue assentar em 15 dias?
(a) 104

(b) 110

(c) 120

(d) 128

(e) 112

3. Medindo pilhas de papel Numa papelaria, so armazenados pacotes de papel em


pilhas de 60 pacotes. Cada pacote tem 500 folhas de papel e cada folha de papel tem
uma espessura de 0,1 mm. Ignorando a espessura do papel utilizado para embrulhar os
pacotes, podemos afirmar que a altura de uma pilha de 60 pacotes aproximadamente
igual altura de:
(a) uma pessoa adulta;

(d) um prdio de 10 andares;

(b) um beb de um ano;

(e) uma sala de aula.

(c) uma mesa comum;


4. Quanto pesa? A balana da figura est em equilbrio com bolas e saquinhos de
areia em cada um de seus pratos. As bolas so todas iguais e os saquinhos tambm.
O peso de um saquinho de areia igual ao peso de quantas bolas?
(a) 1
(b) 2
(c) 3
(d) 5
(e) 6
5. Calcule a diferena Considere dois nmeros naturais, cada um deles com trs
algarismos diferentes. O maior s tem algarismos pares e o menor s tem algarismos
mpares. Se a diferena entre eles a maior possvel, qual essa diferena?
(a) 997

(b) 777

(c) 507

(d) 531

(e) 729

6. Qual o volume? Trs frascos, todos com capacidade


igual a um litro, contm quantidades diferentes de um mesmo
lquido, conforme ilustrao. Qual das alternativas abaixo
melhor expressa, aproximadamente, o volume do lquido contido nos frascos A, B e C, nessa ordem?
(a)

3 4 2
; ;
7 9 5

(b)

2 1 1
; ;
3 2 4

(c)

2 4 2
; ;
3 6 4

OBMEP 2010

(d)

2 4 3
; ;
3 7 4

(e)

3 4 2
; ;
3 5 3
1

Nvel 1
7. Descontos e descontos Uma farmcia d desconto de 30% sobre o preo de tabela
de todos os medicamentos que vende. Ao adquirir um remdio cujo preo de tabela
R$ 120,00, quanto reais uma pessoa ir pagar?
(a) 36

(b) 84

(c) 64

(d) Mais do que 116

(e) 94

8. O carro de Maria Um litro de lcool custa R$ 0,75. O carro de Maria percorre


25 km com 3 litros de lcool. Quantos reais Maria gastar com o lcool necessrio para
percorrer 600 km?
(a) 54

(b) 72

(c) 50

(d) 52

(e) 45

9. Calculando distncias As quatro cidades A, B, C e D


foram construdas beira de uma rodovia reta, conforme a
ilustrao.

C D

A distncia entre A e C de 50 km e a distncia entre B e D de 45 km. Alm


disso, sabe-se que a distncia entre a primeira e a ltima cidade de 80 km. Qual a
distncia, em quilmetros, entre as cidades B e C?
(a) 15

(c) 20

(c) 25

(d) 5

(e) 10

10. Pesando caixas Num armazm foram empilhadas algumas caixas que
formaram o monte mostrado na figura. Se cada caixa pesa 25 kg, quantos
quilogramas pesa o monte com todas as caixas?
(a) 300

(b) 325

(c) 350

(d) 375

(e) 400

11. Consumo de gua Na tabela a seguir vemos o consumo mensal de gua de uma
famlia, durante os cinco primeiros meses de 2004.
Qual o consumo mensal mdio de janeiro a maio
dessa famlia, em m3 ?
(a) 11,3

(c) 12,7

(b) 11,7

(d) 63,5

(e) 317,5

Meses
Janeiro
Fevereiro
Maro
Abril
Maio

Consumo (m3 )
12,5
13,8
13,7
11,4
12,1

12. Folheando um livro Um livro de cem pginas tem suas pginas numeradas de 1 a
100. Quantas folhas desse livro possuem o algarismo 5 em sua numerao?
(Ateno: uma folha tem duas pginas.)
(a) 13

(b) 14

(c) 15

(d) 16

(e) 17

13. Calculando a soma Escreva os nmeros de 0 a 9 nos crculos


ao lado, de forma que eles cresam no sentido anti-horrio. Em
seguida, subtraia uma unidade dos nmeros mpares e some uma
unidade aos nmeros pares. Escolhendo trs crculos consecutivos,
qual a maior soma que se pode obter?
(a) 19
2

(b) 21

(c) 23

(d) 24
OBMEP 2010

(e) 25

Nvel 1
14. Desenhando o cubo A figura ao lado foi desenhada em
cartolina e dobrada de modo a formar um cubo.
Qual das alternativas mostra o cubo assim formado?

(a)

(b)

(c)

(d)

(e)

15. Crculos concntricos Na malha quadriculada a seguir, todas as circunferncias


tm o mesmo centro. Pode-se concluir que a rea da regio cinza destacada igual a
(a) dois quintos da rea do crculo maior;
(b) trs stimos da rea do crculo maior;
(c) metade da rea do crculo maior;
(d) quatro stimos da rea do crculo maior;
(e) trs quintos da rea do crculo maior.

16. Brincando com engrenagens Jos colou uma bandeirinha em cada um dos dois
discos dentados que formam uma engrenagem, como mostra a figura.
Os dois discos so exatamente iguais, inclusive os dentes em
cada um deles. Jos girou a engrenagem e claro que as
bandeirinhas mudaram de posio. Qual a nova posio
das duas bandeirinhas?

(a)

(b)

(d)

(e)

(c)

17. Troca de garrafas A prefeitura de uma certa cidade fez uma campanha que permite
trocar quatro garrafas de 1 litro vazias por uma garrafa de 1 litro cheia de leite. Quantos
litros de leite pode obter uma pessoa que possua 43 garrafas vazias de 1 litro fazendo
vrias dessas trocas?
(a) 11

(b) 12

(c) 13

(d) 14
OBMEP 2010

(e) 15
3

Nvel 1
18. Retngulo e quadrados A figura dada representa
um gramado retangular em que foram marcados sete
quadrados numerados de 1 a 7. Se a rea do menor
desses quadrados 1 m2 , a rea total do gramado, em
m2 , igual a
(a) 42

(b) 44

(c) 45

(d) 48

2
3
1

5
6

(e) 49

19. Quantas fatias de bolo? Nove amigos compraram trs bolos, cada um deles
cortado em oito fatias. Todos comeram bolo e no sobrou nenhum pedao. Sabendo
que cada um s comeu fatias inteiras do bolo, podemos ter certeza de que:
(a) algum comeu quatro fatias;
(b) um deles comeu somente uma fatia;
(c) todos comeram duas fatias, pelo menos;
(d) uns comeram duas fatias e os demais comeram
trs fatias;
(e) um deles comeu, no mnimo, trs fatias.
20. Mosaicos quadrados Uma sequncia de mosaicos quadrados construda com azulejos quadrados pretos e brancos, todos do mesmo tamanho, sendo o primeiro formado
por um azulejo branco cercado por azulejos pretos, o segundo por quatro azulejos
brancos cercados por azulejos pretos e assim, sucessivamente, como indica a figura.
Se numa sequncia de mosaicos formada de acordo com esta regra forem usados 80
azulejos pretos, quantos sero os azulejos brancos utilizados?
(a) 55

(d) 85

(b) 65

(e) 100

(c) 75
21. Quanto custa? Ester vai a uma papelaria para comprar cadernos e canetas. Nessa
papelaria, todos os cadernos custam R$ 6,00. Se ela comprar trs cadernos, sobram
R$ 4,00. Se, em vez disso, seu irmo lhe emprestar R$ 4,00 adicionais, ela conseguir
comprar dois cadernos e sete canetas, todas iguais.
(a) Quanto custa cada caneta?
(b) Se ela comprar dois cadernos e no pedir dinheiro emprestado, quantas canetas
Ester poder comprar?
22. Encontre o nmero O nmero da casa de Jlia tem exatamente trs algarismos,
cuja soma 24. Encontre todos os possveis nmeros da casa de Jlia, em cada uma
das situaes seguintes.
(a) Os trs algarismos so iguais.
(b) Apenas dois algarismos so iguais.
(c) Os algarismos so todos diferentes.
4

OBMEP 2010

Nvel 1
23. Campeonato de futebol No ltimo campeonato de futebol do bairro em que moro
participaram seis equipes, denominadas A, B, C, D, E e F. Cada equipe disputou, com
cada uma das outras, exatamente uma partida.
V E D GP GC
Na tabela de classificao do campeonato, ao lado,
A
4
1
0
6
2
V indica o nmero de vitrias, E o nmero de empaB
2
1
2
6
6
tes, D o nmero de derrotas, GP o nmero de gols
C
0
3
2
2
6
marcados e GC o nmero de gols sofridos de cada
D 1
1
y
3
6
equipe.
E
0
1
4
1
5
F
x
1
0
z
3
(a) Quantas partidas foram disputadas?
(b) A tabela est incompleta. Determine a quantidade de vitrias da equipe F, a
quantidade de derrotas da equipe D e a quantidade de gols marcados pela equipe F,
representados por x, y e z na tabela.

24. Dividindo o paraleleppedo Um bloco de madeira na


forma de um paraleleppedo retngulo tem 320 cm de comprimento, 60 cm de largura e 75 cm de altura. O bloco
cortado vrias vezes, com cortes paralelos s suas faces,
de modo a subdividi-lo em blocos menores, todos na forma
de paraleleppedos retngulo de 80 cm de comprimento por
30 cm de largura por 15 cm de altura.

60
75

320

(a) Quantas peas foram obtidas?


(b) Um metro cbico dessa madeira pesa aproximadamente 900 kg. Qual o peso de
cada uma dessas peas?
25. Uma calculadora Uma calculadora possui duas teclas especiais:
a tecla A, que duplica o nmero que aparece no visor; e

a tecla B, que acrescenta uma unidade ao nmero que aparece no


visor.

Por exemplo, se o nmero 45 estiver no visor e for apertada a tecla B, o visor mostrar
o nmero 46. Se, em seguida, apertarmos a tecla A, o visor mostrar o nmero 92.
Nesse exemplo, apertamos ao todo duas vezes as teclas A e B: uma vez a tecla B e
depois uma vez a tecla A, para, a partir de 45, chegar ao 92. Suponha que no visor
esteja o nmero 1. Indique uma maneira de obter o nmero:
(a) 10 apertando um total de quatro vezes as teclas A e B;
(b) 15 apertando um total de seis vezes as teclas A e B;
(c) 100 apertando um total de oito vezes as teclas A e B.
26. Ano bissexto Um ano comum tem 365 dias e um ano bissexto, 366 dias. O ano
bissexto, quando o ms de fevereiro tem 29 dias, ocorre a cada quatro anos.
(a) Com frequncia dizemos Um ano comum tem 52 semanas. Ser correta essa
afirmao? E para um ano bissexto? Justifique suas respostas.
(b) Se um ano comum inicia numa tera-feira, ento o ano seguinte iniciar em qual
dia da semana?
OBMEP 2010

Nvel 1
(c) Responda a pergunta anterior para um ano bissexto.
27. Nmeros triangulares O famoso matemtico grego Pitgoras denominou os nmeros obtidos pela soma dos primeiros nmeros inteiros positivos de nmeros triangulares.
Por exemplo, 1, 3, 6 e 10 so nmeros triangulares.
1
3
6
10

=
=
=
=

1
1+2
1+2+3
1+2+3+4
1

1+2=3

1+2+3=6

1 + 2 + 3 + 4 = 10

A figura ilustra a motivao para o nome dos nmeros triangulares. A sequncia de


nmeros triangulares continua com 1 + 2 + 3 + 4 + 5 = 15, 1 + 2 + 3 + 4 + 5 + 6 = 21
etc. Quantos so os nmeros triangulares menores do que 100?
28. Livros separados Uma bibliotecria recebe 130 livros de Matemtica e 195 livros
de Portugus. Ela quer arrum-los em estantes, colocando igual quantidade de livros
em cada estante, sem misturar livros de Matemtica e de Portugus na mesma estante. Quantos livros ela deve colocar em cada estante para que o nmero de estantes
utilizadas seja o menor possvel?
29. Alunos com culos A sexta parte dos alunos de uma classe usam culos. Dentre
os que usam culos, uma tera parte so meninas; alm disso, quatro meninos usam
culos. Quantos so os alunos dessa classe?
30. Quadrado mgico Complete as casas em branco da tabela ao
lado com fraes, de tal modo que a soma dos trs nmeros de
qualquer linha, qualquer coluna e das duas diagonais seja sempre a
mesma.

3/5
0,4

1/2
0,5

31. Trs algarismos Sejam A, B e C algarismos diferentes de zero tais que (AB)2 =
CAB, isto , o nmero de dois algarismos AB elevado ao quadrado d o nmero de
trs algarismos CAB. Determine o valor de A + B + C.
32. Pintando quadradinhos Uma faixa quadriculada tem 5 quadradinhos na largura
e 250 quadradinhos no comprimento. Alguns quadradinhos sero pintados de cinza,
comeando da esquerda, conforme o modelo ilustrado na figura, e continuando com
este padro at chegar ao final da faixa, direita.
Quantos quadradinhos no sero pintados?

250 quadradinhos

OBMEP 2010

Nvel 1
33. A cisterna do Joo Joo tem, em seu jardim, uma cisterna na qual ele armazena
gua de chuva e tira gua para regar suas flores. meia-noite do dia 31 de dezembro
de 2005, a cisterna continha 156 litros de gua. Joo tem o hbito de anotar em um
quadro, todo dia, o nmero de litros de gua que ele gasta para regar as flores e o de
gua recolhida da chuva.
Ao lado vemos parte do quadro referente aos primeiros 8
dias de janeiro de 2006. Quantos litros de gua havia na
cisterna do Joo meia noite do dia 8 de janeiro de 2006?

Jan
1
2
3
4
5
6
7
8

flores(l)
6
9
0
4
9
0
11
0

chuva(l)
2,5
0
5
0
3
0
4,5
0

34. O mltiplo de 13 Da igualdade 9 174 532 13 = 119 268 916 pode-se concluir que
um dos nmeros a seguir divisvel por 13. Qual esse nmero?
(a) 119 268 903

(c) 119 268 911

(e) 119 268 923

(b) 119 268 907

(d) 119 268 913

35. Um bilho Arnaldo afirmou que um bilho o mesmo que um milho de milhes.
O professor Piraldo o corrigiu e disse, corretamente, que um bilho o mesmo que
mil milhes. Qual a diferena entre o valor correto de um bilho e a afirmao de
Arnaldo?
(a) 1 000

(b) 999 000

(c) 1 000 000

(d) 999 000 000

(e) 999 000 000 000

36. Energia de abelha Com a energia fornecida por um litro de mel, uma abelha
consegue voar 7 000 quilmetros. Quantas abelhas conseguiriam voar um quilmetro,
cada uma, com a energia fornecida por 10 litros de mel?
(a) 7 000

(b) 70 000

(c) 700 000

(d) 7 000 000

(e) 70 000 000

37. Perda de safra Um agricultor esperava receber cerca de R$ 100 000,00 pela venda
de sua safra. Entretanto, a falta de chuva provocou uma perda da safra avaliada entre
uma quinta parte e uma quarta parte do total previsto. Qual dos valores a seguir pode
representar a perda do agricultor, em reais?
(a) 21 987,53

(b) 34 900,00

(c) 44 999,99

(d) 51 987,53

(e) 60 000,00

38. Placa decorativa Uma placa decorativa consiste num quadrado branco de quatro metros de lado, pintado de forma simtrica com partes em cinza, conforme a figura. Qual a
frao da rea da placa que foi pintada?
(a)

1
2

(b)

1
3

(c)

3
8

(d)

6
13

(e)

7
11

39. O suco do Diamantino Diamantino colocou trs litros de gua e um litro de


refresco num recipiente. O refresco composto de 20% de suco de laranja e 80% de
OBMEP 2010

Nvel 1
gua. Depois de misturar tudo, que porcentagem do volume final representa o suco de
laranja?
(a) 5%

(b) 7%

(c) 8%

(d) 20%

(e) 60%

40. Uma eleio Trs candidatos concorreram eleio de representante de uma turma
de escola: Joo, Rosa e Marcos. Joo obteve 2/7 dos votos e Rosa 2/5 dos votos. Quem
ganhou a eleio?
41. Soma de potncias Qual o valor de 26 + 26 + 26 + 26 44 ?
(a) 0

(b) 2

(c) 4

(d) 42

(e) 44

42. Seis retngulos Com seis retngulos idnticos formamos um retngulo maior, com um dos lados medindo
21 cm, como na figura. Qual a rea do retngulo maior,
em cm2 ?
(a) 210

(b) 280

(c) 430

(d) 504

21 cm

(e) 588

43. Duas populaes H trs anos, a populao de Pirajussara era igual populao
que Tucupira tem hoje. De l para c, a populao de Pirajussara no mudou, mas a
populao de Tucupira cresceu 50%. Hoje, a soma das populaes das duas cidades
de 9 000 habitantes. Qual era a soma dessas duas populaes h trs anos?
(a) 3 600

(b) 4 500

(c) 5 000

(d) 7 200

(e) 7 500

44. Trs balanas As balanas (1) e (2) da figura dada esto em equilbrio. Sabe-se
que todos os tringulos tm o mesmo peso, bem como todos os quadrados e tambm
todos os crculos. Quantos quadrados devem ser colocados no prato direito da balana
(3) para que ela tambm fique equilibrada?

(2)

(1)
(a) 7

(b) 8

(c) 9

(d) 10

(3)
(e) 12

45. Poucos domingos Em um ano, no mximo quantos meses tm cinco domingos?


(a) 3

(b) 4

(c) 5

(d) 6

(e) 7

46. Metade de potncia Qual a metade do nmero 212 + 3 210 ?


(a) 26 + 3 25

(b) 26 + 3 210

(c) 21 1 + 3 25

(d) 211 7

(e) 29 7

47. Minutos demais Neste momento, so 18 horas e 27 minutos. Qual era o horrio
2 880 717 minutos mais cedo?
(a) 6h22min
8

(b) 6h24min

(c) 6h27min
OBMEP 2010

(d) 6h30min

(e) 6h32min

Nvel 1
48. Dois nibus Os alunos de uma escola participaram de uma excurso, para a qual
foram contratados dois nibus. Quando os nibus chegaram, 57 alunos entraram no
primeiro nibus e apenas 31 no segundo. Quantos alunos devem passar do primeiro
para o segundo nibus para que seja transportada a mesma quantidade de alunos nos
dois nibus?
(a) 8

(b) 13

(c) 16

(d) 26

(e) 31

49. Cubo de papelo Em qual das alternativas abaixo aparecem dois pedaos de papelo
com os quais pode-se construir um cubo, dobrando pelas linhas tracejadas e colando
pelas linhas contnuas?

(a)

(b)

(d)

(e)

(c)

50. Algarismo das unidades Qual o algarismo das unidades do nmero


1 3 5 97 99?
(a) 1

(b) 3

(c) 5

(d) 7

(e) 9

51. Regio sombreada A figura mostra um retngulo


formado por 18 quadrados iguais com algumas partes
sombreadas.
Qual a frao da rea do retngulo que est sombreada?
(a)

7
18

(b)

4
9

(c)

1
3

(d)

5
9

(e)

1
2

52. Colorindo um mapa A figura mostra o mapa de um pas


(imaginrio) constitudo por cinco estados. Deseja-se colorir esse
mapa com as cores verde, azul e amarelo, de modo que dois
estados vizinhos no possuam a mesma cor. De quantas maneiras
diferentes o mapa pode ser pintado?
(a) 12

(b) 6

(c) 10

(d) 24

(e) 120

53. Pintando um tabuleiro As nove casas de um tabuleiro 3 3 devem ser pintadas


de forma que em cada coluna, cada linha e cada uma das duas diagonais no haja duas
casas de mesma cor. Qual o menor nmero de cores necessrias para isso?
(a) 3

(b) 4

(c) 5

(d) 6

(e) 7

OBMEP 2010

Nvel 1
54. Nmero X,Y Considere um nmero escrito na forma decimal X, Y, onde X e Y
so algarismos diferentes de 0. Determine esse nmero, sabendo que X, Y igual a
3
(X + Y ).
10
55. Construo de casas Em um mesmo lado de uma rua sero construdas seis
casas vizinhas. As casas podem ser de alvenaria ou de madeira, mas como medida de
segurana contra incndio, duas casas de madeira no podem ser vizinhas. De quantas
maneiras se pode planejar a construo dessas casas?
56. Comparao de grandezas Qual o maior dos nmeros dados?
(a) 1 000 + 0,01

(c) 1 000/0,01

(b) 1 000 0,01

(d) 0,01/1 000

(e) 1 000 0,01

57. Maior nmero de seis algarismos Qual o maior nmero de seis algarismos
que se pode encontrar suprimindo-se nove algarismos do nmero 778 157 260 669 103,
sem mudar a ordem de seus algarismos?
(a) 778 152

(b) 781 569

(c) 879 103

58. Qual o numerador? Se


(a) 5

(b) 6

(c) 7

(d) 986 103

(e) 987 776

n
1 1
um nmero entre e , quem n?
24
6 4
(d) 8
(e) 9

59. Correndo menos Correndo a uma velocidade de 10 km/h, Joo completa um certo
percurso em seis minutos. Com qual velocidade, em km/h, ele pode completar o mesmo
percurso em oito minutos?
(a) 7,5

(b) 7,75

(c) 8

(d) 8,25

(e) 8,5

60. Cinco vizinhas As vizinhas Elza, Sueli, Patrcia, Helosa e Cludia chegam juntas
do trabalho e comeam a subir as escadas do prdio de cinco andares onde moram.
Cada uma mora num andar diferente. Helosa chega a seu andar depois de Elza, mas
antes de Cludia. Quando Sueli chega ao seu andar, Helosa ainda tem dois andares
para subir e o mesmo ocorre com Patrcia quando Elza chega ao seu andar. Sueli no
mora no primeiro andar. Em qual andar mora cada uma delas?
61. Potncias de 9 Qual o valor da soma 920 + 920 + 920 ?
(a) 920

(b) 366

(c) 923

(d) 341

(e) 323

62. Dois nmeros Miguel escolheu um nmero de trs algarismos e outro de dois. Qual
a soma desses nmeros se sua diferena 989?
(a) 1 000

(b) 1 001

(c) 1 009

(d) 1 010

(e) 2 005

63. Menor natural Qual o menor nmero natural n para o qual 10n 1 um mltiplo
de 37?
(a) 6
10

(b) 5

(c) 4

(d) 3

(e) 2

OBMEP 2010

Nvel 1
64. Imunes gripes Num certo pas com 14 milhes de habitantes, 0,15% da populao
contraiu uma certa gripe. Quantos habitantes no contraram essa gripe?
(a) 13 979 000

(b) 1 397 900

(c) 139 790

(d) 13 979

(e) 139 790 000

65. O cdigo secreto O cdigo secreto de um grupo de alunos um nmero de trs


algarismos distintos diferentes de 0. Descubra o cdigo utilizando as informaes a
seguir.
1
4
6
5
8
(a) 137

2
5
1
4
4

Nenhum algarismo correto.


S um algarismo correto na posio certa.
S um algarismo correto, mas na posio errada.
S um algarismo correto, mas na posio errada.
S um algarismo correto na posio certa.

3
6
2
7
3

(b) 876

(c) 768

(d) 678

(e) 576

n

o
66. Parnteses, colchetes e chaves Qual o valor de 2 2 2 2 2 2(4 2) ?
(a) 0

(b) 2

(c) 2

(d) 4

(e) 10

4 4 4 3 6
67. Ordenando fraes Qual a ordem crescente correta das fraes , , , , e
3 5 6 5 5
2
?
5
2
3
4
4
6
4
2
3
4
4
6
4
(a)
< < < < <
(d)
< < < < <
5
5
6
5
5
3
5
5
5
6
5
3
4
4
2
3
4
6
3
4
4
4
6
2
(b)
< < < < <
(e)
< < < < <
3
6
5
5
5
5
5
5
5
3
6
5
2
3
4
4
4
6
< < < < <
(c)
5
5
5
6
3
5
68. Nmeros de trs algarismos Quantos nmeros de trs algarismos maiores do que
200 podem ser escritos, usando-se apenas os algarismos 1, 3 e 5?
(a) 10

(b) 12

(c) 14

(d) 15

(e) 18

69. Velocidade de maratona Uma maratona de 42 km comeou s 11h30min e o vencedor terminou s 13h45min do mesmo dia. Qual foi a velocidade mdia do vencedor,
em km/h?
(a) 18,6

(b) 25,3

(c) 29

(d) 32,5

(e) 38

70. Bilhetinhos com nmeros Cinco alunas escreveram cada uma um nmero num
papel. Os nmeros s podiam ser 1 ou 2 ou 4. Qual pode ser o produto dos cinco
nmeros escritos?
(a) 100

(b) 120

(c) 256

(d) 768

OBMEP 2010

(e) 2 048
11

Nvel 1

1 
1 
1
1 
71. Produto de fraes Qual o valor do produto 1
1
1
1 ?
2
3
4
5
119
5
43
1
1
(a)
(b)
(c) 2
(d)
(e)
120
7
60
5
120
72. Produto mximo A soma de dois nmeros naturais 11. Qual o maior produto
possvel que se pode obter com esses nmeros?
(a) 30

(b) 22

(c) 66

(d) 24

(e) 28

73. Quem o cubo? Se m um nmero natural tal que 3m = 81, quanto vale m3 ?
(a) 813

(b) 381

(c) 64

(d) 24

(e) 48

74. Qual o maior? Se a 1 = b + 2 = c 3 = d + 4, qual o maior dentre os


nmeros a, b, c e d?
(a) a

(b) b

(c) c

(d) d

(e) So todos iguais

75. Quatro formiguinhas Quatro formigas atravessam o piso de uma sala coberto de
lajotas retangulares, segundo os trajetos indicados na figura. Qual o comprimento
do trajeto percorrido por Biloca?
(a) 30 dm
Trajeto de Pipoca = 25 dm

(b) 43 dm
(c) 55 dm

Trajeto de Tonica = 37 dm

(d) 24 dm

Trajeto de Cotinha = 32 dm

(e) 48 dm

Trajeto de Biloca =

76. Trocando figurinhas Clia quer trocar com Guilherme figurinhas de um lbum
sobre animais brasileiros. Clia quer trocar quatro figurinhas de borboleta, cinco de
tubaro, trs de cobra, seis de periquito e seis de macaco. Todas as figurinhas de
Guilherme so de aranha. Eles sabem que
(a) uma figurinha de borboleta vale trs figurinhas de tubaro;
(b) uma figurinha de cobra vale trs figurinhas de periquito;
(c) uma figurinha de macaco vale quatro figurinhas de aranha;
(d) uma figurinha de periquito vale trs figurinhas de aranha;
(e) uma figurinha de tubaro vale duas figurinhas de periquito.
Quantas figurinhas Clia poder receber se ela trocar todas que quiser?
77. Soma de fraes Qual o valor da soma
(a) 1
12

(b) 20

(c) 30

10 + 20 + 30 + 40
10
+
?
10
10 + 20 + 30 + 40

(d) 10,1
OBMEP 2010

(e) 1,01

Nvel 1
78. Geometria com palitos A figura dada formada por um
tringulo e um retngulo, usando-se 60 palitos iguais. Para cada
lado do tringulo so necessrios seis palitos. Se cada palito mede
5 cm de comprimento, qual a rea (em cm2 ) do retngulo da
figura?
(a) 1 200

(c) 2 700

(b) 1 800

(d) 3 600

(e) 4 500

79. Um incndio e o bombeiro Uma casa pega fogo. Um bombeiro se mantm no


degrau do meio de uma escada, jogando gua sobre o incndio. As chamas diminuem
e ele sobe cinco degraus. O vento sopra e o bombeiro desce sete degraus. Um pouco
depois, ele sobe oito degraus e fica l at acabar o incndio. Ento, ele sobe os ltimos
sete degraus e entra na casa. Quantos degraus tem a escada do bombeiro?
(a) 25

(b) 26

(c) 27

(d) 28

(e) 29

80. rvore genealgica A figura mostra a rvore genealgica de uma famlia. Cada
flecha vai do pai em direo ao seu filho. Quem o irmo do pai do irmo do pai de
Evaristo?
Ado

(a) Francisco

Lus

Andr

(b) Jos
(c) Andr

Felipe

(d) Felipe

Cristvo

(e) Simo

Jean

Jos

Evaristo

Francisco

81. Colcha quadrada Uma colcha quadrada em branco e cinza


feita com quadrados e tringulos retngulos issceles. A parte em
cinza representa qual percentagem da colcha?
(a) 36%

(c) 45%

(b) 40%

(d) 50%

(e) 60%

82. Falsas igualdades Considere as igualdades a seguir.


(i) 3 106 + 5 102 = 8 108

(iii) 5 8 + 7 = 75

(ii) 23 + 23 = 20

(iv) 5 + 5 5 = 2

Qual delas est correta?


(a) (i)

(b) (ii)

(c) (iii)

(d) (iv)

(e) Nenhuma

83. Menor valor da soma Se a, b e c so nmeros inteiros positivos tais que


3a = 4b = 7c, qual o menor valor possvel de a + b + c?
(a) 84

(b) 36

(c) 61

(d) 56
OBMEP 2010

(e) 42
13

Nvel 1
84. Procurando um quadrado perfeito Um nmero um quadrado perfeito se igual
a um nmero inteiro elevado ao quadrado. Por exemplo, 25 = 52 , 49 = 72 e 125 = 252
so quadrados perfeitos. Qual o menor nmero pelo qual devemos multiplicar 120
para obter um quadrado perfeito?
(a) 10

(b) 15

(c) 20

(d) 30

(e) 35

85. Visitas num museu A mquina que registra o nmero de visitantes de um museu
marca 1 879 564. Note que esse nmero tem todos os algarismos distintos. Qual o
menor nmero de visitantes que so necessrios para que a mquina registre um outro
nmero que tambm tenha todos os seus algarismos distintos?
(a) 35

(b) 36

(c) 38

(d) 47

(e) 52

86. Ligando nmeros por flechas Os nmeros de 0 a 2 000 foram ligados por flechas;
a figura dada mostra o comeo do processo.

Qual a sucesso de flechas que liga o nmero 1 997 ao nmero 2 000?


(a)

(b)

(c)

(d)

(e)

87. Mltiplos de 9 Encontre o menor mltiplo positivo de 9 que pode ser escrito apenas
com os algarismos: (a) 0 e 1;
(b) 1 e 2.
88. A florista Uma florista colheu 49 kg de flores do campo. O quilograma das flores
pode ser vendido imediatamente a R$ 1,25 ou, mais tarde, com as flores desidratadas,
a R$ 3,25. O processo de desidratao faz as flores perderem 5/7 de seu peso. Qual
o tipo de venda mais lucrativo para a florista?
89. Divisores Seja N o menor nmero que tem 378 divisores e da forma 2a 3b 5c 7d .
Quanto vale cada um desses expoentes?
90. O produto dos algarismos Denotemos por P (n) o produto dos algarismos do
nmero n. Por exemplo, P (58) = 5 8 = 40 e P (319) = 3 1 9 = 27.
(a) Dentre os nmeros de 1 a 999, quais so os que tm produto dos algarismos igual
a 12, isto , quais so os inteiros n tais que 1 n < 1 000 e P (n) = 12?

(b) Quantos nmeros inteiros existem entre 0 e 200 cujo produto dos algarismos seja
igual a 0, isto , quantos inteiros n existem tais que 1 n < 200 e P (n) = 0?
(c) Quais so os nmeros inteiros 1 n < 200 tais que 37 < P (n) < 45?

(d) Dentre todos os inteiros de 1 a 250, qual o nmero cujo produto dos algarismos
o maior possvel?
14

OBMEP 2010

Nvel 1
91. Suco de laranja Davi vai a um armazm que vende uma garrafa de suco de laranja
por R$ 2,80 e uma caixa com seis dessas garrafas por R$ 15,00. Ele precisa comprar
22 garrafas para seu aniversrio. Quanto ele gastar, no mnimo?
92. A casa da Rosa A figura mostra a planta da casa da Rosa. O quarto e o quintal
so quadrados. Qual a rea da cozinha?
.....................................................................................................................................................................................................................
...
...
...
...
...
....
...
...
...
...
....
....
...
...
...
...
...
...
...
...
...
....
....
....
..
..
..
...
...
...
...
...
...
...
...
...
....
...
....
......................................................................................................................................................................................................................
...
...
...
...
...
...
... Quintal ...
...
...
...
...
...
...
...
2
...
...
...
........................................................................................................................................................................................................................

Quarto
16 m2

Sala
24 m2

Cozinha

4m

93. O passeio do Matias Matias passeia de bicicleta nas ruas em volta de quatro quarteires perto de sua casa, dispostos como na figura. O seu passeio consiste em fazer o maior percurso possvel, mas ele inventou uma regra para se divertir
mais, a saber: ele pode passar vrias vezes pelos cruzamentos das ..............................................................................................................................................................................................................
... ...
... ...
.. ..
.. ..
.. ...
... ..
... ...
ruas, mas ele no pode passar mais do que uma vez pela mesma rua, .......... .........
... ...
... ...
.. ...
.. ..
... ...
.
.
... ...
... ...
.
.
.
devendo terminar seu passeio de bicicleta quando no puder mais ..... ......................................... .............................................. .........
..
..
respeitar essa condio. Os quatro quarteires so quadrados, cada ......... ..................................................... ..................................................... .........
... ..
... ...
.. ...
... ...
... ...
.. ...
.. ..
um com 100 metros de lado e a largura das ruas no considerada ......... ........
.. ..
.. ..
.. ........................................ ......................................... ...
...
.
.
relevante. Partindo do ponto indicado por P na figura, qual o ...........................................................r............................................................
P
maior desses percursos de bicicleta que Matias pode fazer?
94. O adesivo dos carros oficiais O prefeito de uma cidade decidiu colocar adesivo em
cada carro oficial. O adesivo ter uma forma retangular, com seis quadrados dispostos
em 2 3 e com trs cores: um quadrado azul, dois quadrados amarelos e trs quadrados
verdes. Dentre quantos tipos diferentes de adesivo o prefeito poder escolher?
95. Adio de nmeros Qual o algarismo a em a 000 + a 998 + a 999 = 22 997?
96. Cubo perfeito e divisibilidade Quais so os cubos perfeitos que dividem 94 ?
97. Localizao de um ponto Qual o ponto indicado na figura?

18

19

20

98. Clculo de porcentagem Se voc acerta 58/84 das 84 questes de um teste, qual
o seu percentual de acertos?
99. Comparao de algarismos Dizemos que um nmero ascendente se cada um
de seus algarismos for maior do que o algarismo que est sua esquerda. Por exemplo,
2 568 ascendente e 175 no . Quantos nmeros ascendentes existem entre 400 e 600?
OBMEP 2010

15

Nvel 1
100. Muro colorido Um muro deve ser construdo conforme a figura com 14 tijolos
coloridos, disponveis em amarelo, azul e vermelho, cujos preos esto dados na tabela.
Se dois tijolos quaisquer que se toquem devem ser de cores diferentes, qual o menor
valor que se gastar na compra desses 14 tijolos?
tijolo
amarelo
azul
vermelho

..............................................................................................................................................................
..
..
..
..
...
...
..
..
..
..
..
..
.
.
.........................................................................................................................................................................................................................
..
..
..
..
..
...
...
...
...
...
...
..
..
...
.
.............................................................................................................................................................................................................
...
...
...
...
....
....
....
....
..
..
..
..
.....................................................................................................................................................................................................................
....
....
....
....
....
..
..
..
..
.
................................................................................................................................................................................................................

R$
6
7
8

101. Divisores e fatorao Decomponha 96 em dois fatores inteiros positivos cuja soma
dos quadrados seja 208.
102. O retngulo do Lus Lus desenhou um retngulo de 6 10 cm e quer dividi-lo
em quatro partes. As reas das 4 partes devem medir 8, 12, 16 e 24 cm2 . Desenhe
como ele pode fazer essa diviso.
103. Comparao de nmeros Escreva em ordem crescente os nmeros

3
4
121,
729
e
38 416 .
104. As moedas Uma brincadeira comea com sete moedas alinhadas em cima de uma
mesa, todas com a face coroa virada para cima. Para ganhar a brincadeira, preciso
virar algumas moedas, de tal modo que, no final, duas moedas vizinhas estejam sempre
com faces diferentes viradas para cima. A regra da brincadeira virar duas moedas
vizinhas em cada jogada. Quantas jogadas so necessrias, no mnimo, para ganhar a
brincadeira?
......
.........
.........
..........
............................
.............................
............................
.......... .............
......... .............
......... ............
......... ............
....
... ......
... ......
... ......
... ......
... ......
... ......
...
...
... ..
... ..
... ...
... ...
... ...
... ...
...
...
.. ...
... ..
... ..
... ..
... ..
... ...
...
.
.
.
.
.
.
.....
.. ...
.. ...
... ....
.... ....
.... ....
..
.
.
.... ....
...
..
.. ....
.. ....
. ...
.. ...
.. ...
.. ....
...
.
.
.
.
.
.
.
.
. ...
. ....
. ...
. ....
. ....
. ...
....
.
.
.
.
.
.
.
.
.
.
.
.
.
.
.
.
.
.
.
.
.
.....
......
......
......
.
.
......
.
......
.
.
......
.........................
.........................
........................
........................
........................
........................
.......................

coroa coroa coroa coroa coroa coroa coroa

105. O preo do frango O preo do quilograma de frango era R$ 1,00 em janeiro de


2000, quando comeou a triplicar a cada 6 meses. Em quanto tempo o preo atingir
R$ 81,00?
(a) 1 ano

(b) 2 anos

(c) 2

1
anos
2

(d) 13 anos

(e) 13

1
anos
2

106. Excurses a Foz do Iguau Em 2005, uma agncia de turismo programou uma
excurso para Foz do Iguau, distribuindo as pessoas em nibus de 27 lugares, tendo
sido necessrio formar um nibus incompleto, com 19 lugares ocupados. Em 2006,
aumentou em 53 o nmero de participantes e a agncia continuou a utilizar nibus
de 27 lugares. Quantos nibus a mais foram necessrios e quantas pessoas ficaram no
nibus incompleto em 2006?
107. As fraes de Laura Laura desenhou cinco crculos, dentro dos
quais ela quer colocar nmeros inteiros positivos, de tal modo que
formem uma igualdade entre uma frao e seu valor inteiro.
16

OBMEP 2010

+
+

........................................................................................

Nvel 1
De quantas maneiras pode Laura colocar os nmeros 2, 3, 5, 6 e 11 dentro dos cinco
crculos para que a igualdade seja verdadeira?
108. Clculo da unidade Qual o algarismo da unidade do produto
(5 + 1)(53 + 1)(56 + 1)(512 + 1)?
(a) 0

(b) 1

(c) 2

(d) 5

(e) 6

109. Nmeros cruzados Francisco escreveu 28 algarismos numa tabela 66 e pintou de


preto as demais casas, como nas palavras cruzadas. Ele fez a lista de todos os nmeros,
28
45
51
57
72
88
175 289
632
746
752 805
885 5 647 5 873 7 592 8 764
em ordem crescente, que podem ser lidos horizontal ou verticalmente, excluindo os nmeros de um s algarismo. Preencha a
tabela escrevendo de volta os nmeros de Francisco. Um algarismo j foi colocado.
2
110. Ovos e mas Num certo armazm, uma dzia de ovos e 10 mas tinham o
mesmo preo. Depois de uma semana, o preo dos ovos caiu 10% e o da ma subiu
2%. Quanto se gastar a mais na compra de uma dzia de ovos e 10 mas?
(a) 2%

(b) 4%

(c) 10%

(d) 12%

(e) 12,2%

111. Dividindo nmeros decimais Sabendo que 144 177 = 25 488, podemos concluir
que 254,88 0,177 igual a:
(a) 1 440;

(b) 14,4;

(c) 1,44;

(d) 0,144;

112. Almoo dos amigos Jlio e Denise almoaram num


restaurante que oferece trs tipos de prato e trs tipos
de vitamina, cujos preos esto na tabela ao lado. Cada
um escolheu um prato e uma vitamina. Jlio gastou 6
reais a mais do que Denise. Quanto Denise gastou?

(e) 144.

prato simples
prato com carne
prato com peixe
vitamina de leite
vitamina de frutas
vitamina especial

R$
7
11
14
6
7
9

113. Somas de trs em trs Encontre quatro nmeros inteiros positivos que, somados
de trs em trs, do somas 6, 7, 8 e 9.
...s
..............................s
..............................s
...............................s
...............................s
..
114. O passeio do Jorge Jorge passeia por um caminho em forma P
..
..
...
...
...
...
.
.
.
.
de retngulo, onde esto dispostas 12 rvores, brincando de tocar .....s
.....s
.
..
...
cada rvore durante seu passeio. Primeiro ele toca a rvore do .........
...
..
...s
..............................s
..............................s
...............................s
...............................s
..
canto, assinalada com P na figura, e percorre 32 metros num
mesmo sentido do percurso; a ele volta 18 metros e depois retorna ao sentido inicial

OBMEP 2010

17

Nvel 1
por mais 22 metros. Entre duas rvores consecutivas, a distncia de 5 m. Em quantas
rvores ele tocou?
115. A descoberta do algarismo Os quadrados dos nmeros naturais de 1 a 99 foram
escritos um aps o outro, formando o nmero 14916253649. . . . Qual o algarismo que
ocupa a 100a posio? (As posies so contadas da esquerda para a direita, portanto,
a 1a posio a do 1, a 2a a do 4, e assim por diante.)
116. OBMEP Cada um dos sete discos X, Z, O, B, M, E e P da figura tem um peso
diferente, que varia de 1 a 7 g. Em algumas intersees de dois discos, indicamos a
soma dos pesos desses dois discos. Qual a soma dos pesos dos cinco discos O, B, M, E
e P?
..................................
.....................................
........
.....
.......
.....
.....
....
....
....
....
...
....
...
...
...
...
.
.
.
.
...
...
....
...
....
...
.
..
...
.
..
.
.
.
.
.
.
.
.
.
.
.
.
.
.
.
.
.
.
...
.
.
.
.
.
.
.
.
.
.
.
.
.
.
.
.
.
.
.
.
.
.
.
....... ... ..............
......... ...............................
........ ... ..............
.
.
.
.
.
.
.
...
.
.
.
.
.
.
.
.
.
.
.....
.....
..... .....
...
..
..
.
...
....
.
..
.
.
.
.
....
.
.
.
.
.
.
....
...
.
.
.
...
.
...
...
........
.
.
.
.
.
.
.
.
.
...
... .
....
...
....
.. .
...
...
... ...
...
....
...
.. .....
...
.....
...
.
.
.
..
.
.
.
......
.
.
.
.
.
.
.
.
........ .... .........
..
..
.
........... ... ...............
.
.
.
.
.
.
.
.
.
.
.
.
..
.
......................................
.................................
.
.
.
.
.
.
.
.
.
.
...
.
.
.
.
.
.
.
.
.
.....
.....
...
..
...
..
...
.....
.
.
.
.
...
.
.
.
.
.
.
.
.
.
.
.
.
....
....
...
. ...
... ..
..
..
..
.
.
.
.
.
.
.
.
.
.
.
.
.
.
...
...
...
.
.
... ...
. .
.
....
...
...
.
... ......
...
...
...
.......
.....
...
..
..
...
....
.....
...
..... ..........
..
..
....
......
....
......
..
...........................................
..........................................
..........................................
....
..
..
..
...
...
..
.
...
...
...
...
...
...
..
..
....
....
..
..
.
.
.
.
.
.
....
.....
....
......
.....
......
................ ......................
........................................
....

13

117. Prdio misterioso As figuras mostram as plantas de dois andares de um prdio que
guarda segredos muito valiosos. H nove elevadores que atendem esses dois andares,
representados por letras. Qual o caminho mais curto entre a entrada indicada de um
andar e a sada indicada do outro?
qqqqqqqqqqqqqqqqqqqqqqqqqqqqqqqqqqqqqqqqqqqqqqqqqqqqqqqqqqqqqqqq
q
q
A B qqqq C qqqq
q
qqqqqqqqqqqqqqqqqqqqqqqqqqqqqqqqqqqqqqqqqqqqqqqqqqqqqqqqqqqqqqqqqqqqqq
qq
q
qq
qq D
qq
qq E qqq
qq
q
q
qqq
q
q
qq
qq
qq
qq
qq
qq
qq
qq
qqqqqqqqqqqqqqqqqqqqqqqqqqqqqq
qq
qq
qq
qq
qq
q
q
q
qq F qq G qqq H qqqq
qq
q
q
qqqqqqqqqqqqqqqqqqqqqqqqqqqqqqqqqqqq
qqqqqqqqqqqqqqqq
qqq
qqq
qq
qq
qq
qq J
q
qqqqqqqqqqqqqqqqqqqqqqqqqqqqqqqqqqqqqqqqqqqqqqqqqqqqqqqqqqqqqqq

qqqqqqqqqqqqqqqqqqqqqqqqqqqqqqqqqqqqqqqqqqqqqqqqqqqqqqqqqqqqqqqqqqqq
A q B C qqqq
qq
qqqqqqqqqqqqqqqqqqqqqqqqqqqqqqqqqqqqqq
qqqqqqqqqqqqqqqq
qq
q
qq D
E qqqqq
qq
qq
qq
q
qqqqqqqqqqqqqqqqqqqqqqqqqqqqqqqqqqqqqqqqqqqqqqq
qq
qq
qq
q
q
qq
qq F qq G qq H qqq
qq
qq
q
qq
q
q
qq
qqqqqqqqqqqqqqq
qq
qq
qq
qq J
sada
qqqqqqqqqqqqqqqqqqqqqqqqqqqqqqqqqqqqqqqqqqqqqqqqqqqqqqqqqqqqqqqqq

entrada

118. Soma de fraes Qual o valor de

1
1
1
1

?
10 100 1 000 10 000

27
119. Biblioteca A biblioteca de uma escola comprou 140 novos livros, ficando com
do
25
nmero de livros que tinha antes da compra. O nmero de livros antes dessa compra
era:
(a) 1 750;

(b) 2 500;

(c) 2 780;

(d) 2 140;

(e) 1 140.

120. Comparao de fraes Existem quantas fraes menores do que 1, nas quais o
numerador e o denominador so nmeros inteiros positivos de um s algarismo?
18

OBMEP 2010

Nvel 1
121. Diviso com resto Quais so os nmeros que deixam resto 5 ao dividir 2 007?
122. Panelas Uma panela pesa 645 g e uma outra 237 g. Jos divide 1 kg de carne entre
as duas panelas, de modo que as duas, com seus contedos, ficam com o mesmo peso.
Quanto de carne ele colocou em cada panela?
123. Domins Juliana representou uma multiplicao com 5 domins. Seu irmo Bruno
trocou dois domins de posio e agora a multiplicao ficou errada. Troque de volta
a posio de dois domins para que a multiplicao fique novamente correta.
.....................s...........................
...
.
.
... s ..... s s .....
.......s.........................................
..............................s.......s.......s....
...
...
.
... s .....
..
..............................s.......s.......s.....

................................................
...
.
.
... s ..... s s .....
................................................
......s...................
.. s ..
..
..
..............................s..............s..... .......s..............ss.....
. .
... s s ...
.
...
... s .... ... s s...
... s s .. s s.. .. s s..
.......................................... ....................

3212
3
16456

......................................................

124. Cdigo secreto Antnio precisa descobrir um cdigo de 3 algarismos diferentes


A, B e C. Ele sabe que B maior que A, que A menor do que C e tambm que valem
as igualdades seguintes.
.................................................
...
...
...
..
..
....
..
..
.................................................

.................................................
.
.
.

.................................................
.
.
.

.........................................................................
.
.
.
.

...
..
..
..
..
..
..
..
..
... A .... + .... C .... C .... = .... 2 .... 4 .... 2 ....
B B + .............A
..........................................
.................................................
.........................................................................

..........................
...
...
..
...
...........................

..........................

..........................

...........................................................................

.
...
...
...
...
...
...
...
... ... C ... = ... 3 ... 6 ... 0 ...
B ..............A
........................................................................
..........................
.................

Qual o cdigo que Antnio procura?


125. Os doze pontos Doze pontos esto marcados numa folha de papel
quadriculado, conforme mostra a figura. Qual o nmero mximo de
quadrados que podem ser formados unindo quatro desses pontos?

126. Relgio O grande relgio de parede da escola marca a data (dia,


ms e ano) e as horas (horas e minutos), como na figura. Em que
dia, ms e ano voltaro a aparecer juntos no relgio esses mesmos
10 algarismos pela primeira vez?

r r
r r r r
r r r r
r r

.........................................
..........
.......
......
......
......
....
...
....
.
.
.
...
...
.
...
..
.
...
...
....
..
...
..
...
..
..
...
..
..
..
..........................
.............................
..
.
.
.
...
.
.
.
.
.
.
.
.
.
..
.
.
.
.
...
.
.
.
.
.
.
.
.
.
... ...
...
....
... ...
... ..
............................ ....
... .........................
....
....
.....
....
.
.
......
......
........
........
..............
.
.
.
.
.
.
.
.
.
.
.
.
......................

28 05 94
14 h 00

127. Lpis Setenta e quatro lpis foram embalados em 13 caixas. Se a capacidade mxima
de cada caixa de seis lpis, qual o nmero mnimo de lpis que pode haver em uma
caixa?
(a) 1

(b) 2

(c) 3

(d) 4

(e) 6

128. Contagem Se o algarismo 1 aparece 171 vezes na numerao das pginas de um


livro, quantas pginas tem o livro?
OBMEP 2010

19

Nvel 1
129. Viagem a Recife Quando fui receber a medalha de ouro que conquistei na OBMEP,
apareceram as seguintes informaes nas telas da cabine de passageiros do meu voo
para Recife:
Velocidade mdia: 864 km/h
Distncia do local de partida: 1 222 km
Tempo de chegada a Recife: 1h20min
Se o avio manteve a mesma velocidade, ento qual a distncia, aproximadamente,
em quilmetros, entre Recife e a cidade em que comeou meu voo?
(a) 2 300

(b) 2 400

(c) 2 500

(d) 2 600

(e) 2 700

130. Praa Maria e Joo do uma volta completa na praa, juntos, contando as casas que
ficam em volta da praa. Eles comearam a contar as casas em pontos diferentes. A
quinta casa da Maria a dcima segunda do Joo e a quinta casa do Joo a trigsima
da Maria. Quantas casas h em volta da praa?
131. Sequncia de figuras As figuras , , , , e so repetidas indefinidamente
na sequncia
, , , , , , , , , , , , . . .
(a) Que figura aparecer na 1 000a posio da sequncia?
(b) Em qual posio aparece o milsimo ?
132. A brincadeira com o quadrado Um quadrado de 1 m de lado foi cortado, com
cortes paralelos aos seus lados, em quadradinhos de 1 mm de lado. Colocando-se lado a
lado os quadradinhos, sem superposio, formou-se um retngulo de 1 mm de largura.
Qual o comprimento desse retngulo?
133. O cdigo da Arca do Tesouro Simo precisa descobrir
um nmero escondido na tabela fornecida, que o cdigo da
Arca do Tesouro.
Para descobrir o cdigo, ele precisa formar todos os grupos de
trs algarismos que estejam em casas sucessivas, na horizontal
ou na vertical, e cuja soma seja 14. Retirados da tabela todos
os possveis nmeros desses grupos, o cdigo a soma dos
nmeros que restam na tabela. Qual esse cdigo?
134. Operaes com decimais Efetue a diviso

5
6
8
7
2
5

9
3
2
4
7
2

4
7
4
5
6
3

9
3
2
7
1
6

4
4
5
5
2
7

1
8
5
2
8
1

(0,2)3 + 1
.
0,2 + 1

135. Fatores inteiros Decompor 96 em dois fatores inteiros cuja soma dos quadrados
seja 208.
136. Divisibilidade No nmero 6a78b, a denota o algarismo da unidade de milhar e b
denota o algarismo da unidade. Se 6a78b for divisvel por 45, ento o valor de a + b :
(a) 5;
20

(b) 6;

(c) 7;

(d) 8;
OBMEP 2010

(e) 9.

Nvel 1
137. Nmero simples Digamos que um nmero inteiro positivo seja simples se ele tiver
apenas os algarismos 1 ou 2 (ou ambos). Quantos nmeros menores do que 1 milho
so simples?
138. Venda de TV O gerente de uma loja foi verificar qual tinha sido o preo de venda de
uma televiso da marca VejoTudo em 2006. Ele encontrou uma fatura meio apagada,
em que se podia ler lote de 72 TVs da VejoTudo vendido por R$ _ 6.79_ , 00, mas o
algarismo da dezena de milhar e o da unidade do preo pago pelo lote estavam ilegveis.
Qual foi o preo de venda de cada uma dessas televises em 2006?
139. Chocolate Henrique comprou barras de chocolate por R$ 1,35 cada uma. Ele pagou
com uma nota de R$ 10,00 e recebeu um troco inferior a R$ 1,00. Quantas barras ele
comprou?
140. O quadradinho Qual o valor de  em

6 400 000
= 1,6 ?
400

141. Dois nmeros O produto de dois nmeros de dois algarismos cada 1 728. Se o
mximo divisor comum (MDC) deles 12, quais so esses nmeros?
142. As idades dos irmos No dia de seu aniversrio de 7 anos, em 13 de maro de
2007, uma tera-feira, Carlos disse a seu irmo: A contar de hoje, faltam 2 000 dias
para voc completar 15 anos. Em que dia da semana vai cair o aniversrio do irmo
de Carlos? Quantos anos ter Carlos nesse dia?
143. A mistura de concreto Uma certa mistura de concreto feita de cimento, areia e
terra, na razo de 1 : 3 : 5 por quilo. Quantos quilos dessa mistura podem ser feitos
com 5 quilos de cimento?
(a) 13

1
3

(b) 15

(c) 25

(d) 40

(e) 45

144. Ponto na escala A que nmero corresponde o ponto P indicado na escala dada?
12,62

12,44
P

145. O pomar do Francisco O pomar do Francisco tem macieiras, pereiras, laranjeiras,


limoeiros e tangerineiras, dispostas em cinco filas paralelas, cada uma com uma nica
variedade de rvores, da seguinte maneira:
(a) as laranjeiras esto do lado dos limoeiros;
(b) as pereiras no esto do lado das laranjeiras nem dos limoeiros;
(c) as macieiras esto do lado das pereiras, mas no das laranjeiras, nem dos limoeiros.
Em que fila esto as tangerineiras?
(a) 1a

(b) 2a

(c) 3a

(d) 4a
OBMEP 2010

(e) 5a
21

Nvel 1
146. Quatro quadrados Quatro quadrados iguais esto superpostos formando a figura
dada. Se cada um dos quatro quadrados tem uma rea de 3 cm2 , qual a rea dessa
figura?
.........................................................................................
...
...
...
...
..
.
...
............................................................................................
.........................................................................................
...
...
...
...
...
...
...
...
...
...
..
..
...
...
...
...
...
...
...
...
...
...
...
...
...
...
...
...
..
..
.
...
.
.
...
...
..
.
...
....
...
...
.
..
..........................................................................................
...
.
...
..
..
...
...
...
....
....
....
....
...
...
...
...
...
...
...
...
...
...
...
...
...
...
...
...
...
...................................................................................
...
....
...
....
....
....
...
...
...
...
...
...
..
.
..
..
..
..
........................................................................................
...
.........................................................................................
...
...
....
....
...
...
...
...
...
...
.
...
.........................................................................................

147. O fio de arame Ernesto formou a figura abaixo com um fio de arame, em que cada
segmento de reta tem o comprimento do dimetro dos semicrculos.
q

............
.................
.... .....
...
.......................
.......................
........................
......................
.... .....
..........

Qual das figuras abaixo ele pode formar com esse mesmo fio de arame, cortando-o ou
no, mas sem dobr-lo ou desdobr-lo?
(a)

..................
...
........................................................................
...
....
...
................
...............

(b)

................................
....
..
....
..
........ .......
.........................

.
.................
..................
..................
...
............................
..........................
...
.......................
..
...
....... .........
..

(c)

(d)

.................
...
.
............................
..
...
........ ..
.......... .........
..........................
....
.
.
..............

(e)

...........
.... ....
..............................
......... .......
...........
.......... ...........
........................
.
...
................

148. Sequncia de fsforos Quantos fsforos so necessrios para formar o oitavo termo
da sequncia cujos trs primeiros termos esto dados?
rr

r
r

rr

..................................
..
..
.... ...........
.
..
...
.....
..
.. .......
..................................

(a) 21

(b) 24

rr

rr
r

r
r

rr

............................................................
.
..
..
..
..
.... ..........
.
...
..
...
.....
..
..
.. ........
............................................................

(c) 27

rr

rr
r

rr
r

r
r

rr

......................................................................................
..
..
..
.. ......
..
..
..
..
.....
...
...
...
..
..
..
..
..
.. ..........
.......................................................................................

(d) 30

qqq

(e) 34

149. O trajeto das formiguinhas As formiguinhas Maricota e Nandinha passeiam numa varanda cujo cho
formado por lajotas retangulares de 4 cm de largura por
6 cm de comprimento, conforme indicado na figura. Maricota parte do ponto M, Nandinha parte do N e, ambas, andam apenas pelos lados das lajotas, percorrendo
o trajeto no sentido indicado na figura.

........................................................
...
...................................................................................
..
...
...
........................................................
...
...
..............................
..
....
..
...
..................................................................................
...
..
...
....
...
...
..
...
.......................................................
...
.
....
................................

(a) As duas formiguinhas se encontram depois de andarem uma mesma distncia.


Qual foi essa distncia?
(b) Em que ponto elas se encontraram?
150. A soma 100 A soma de trs nmeros 100, dois so primos e um a soma dos
outros dois.
(a) Qual o maior dos trs nmeros?
22

OBMEP 2010

Nvel 1
(b) D um exemplo de tais trs nmeros.
(c) Quantas solues existem para esse problema?
151. Cdigo de barras Um servio postal usa barras curtas e barras longas para representar seu Cdigo de Endereamento Postal (CEP) composto por oito algarismos, em
que a barra curta corresponde ao 0 (zero) e a longa ao 1. A primeira e a ltima barras
no fazem parte do cdigo e a converso do cdigo dada como segue.
11000
00011
01010
00101
00110

=
=
=
=
=

0
1
2
3
4

01100
10100
00001
10001
10010

=
=
=
=
=

5
6
7
8
9

(a) Escreva o CEP 36470130 na forma de cdigo de barras.


(b) Identifique o CEP que representa o cdigo de barras seguinte.

||||||||||||||||||||||||||||||||||||||||||
152. Atletas da escola Numa escola, um quarto dos alunos joga somente vlei, um
tero joga somente futebol, 300 praticam os dois esportes e 1/12 nenhum desses dois
esportes.
(a) Quantos alunos tem a escola?
(b) Quantos alunos jogam somente futebol?
(c) Quantos alunos jogam futebol?
(d) Quantos alunos praticam pelo menos um dos dois esportes?
153. Dzima peridica Obtenha o algarismo da 1 997a casa decimal de cada uma das
1
1
fraes seguintes. (a)
(b)
22
27
154. Ana na corrida Para ganhar uma corrida, Ana precisa completar os ltimos cinco
quilmetros em menos de 20 minutos. Qual deve ser sua velocidade mnima, em km/h?
155. Quadradinhos e o buraco Quantos quadradinhos foram retirados do tabuleiro de
10 20 quadradinhos da figura? Se o lado de cada quadradinho mede 1 cm, qual a
rea e qual o permetro do buraco?

OBMEP 2010

23

Nvel 1
156. Quadrados perfeitos no retngulo Complete as seis casas da tabela dada, colocando um algarismo em cada uma, de modo que os dois nmeros de trs algarismos
formados na horizontal e os trs nmeros de dois algarismos formados na vertical sejam
quadrados perfeitos.
(a) Quais so os nmeros?
(b) Quantas solues existem?
157. Aula de diviso Na aula sobre diviso, a professora pediu que seus alunos colocassem nmeros no lugar das estrelas. Quais so esses nmeros?
.

..
38 .................
........................
(a)

..
75 ......................12
...................
(b)

..
......................3...................
(c)
7

(d)

.
42 ...........................................

158. Linhas de nibus No ponto de nibus perto de sua casa, Quinzinho pode pegar
os nibus de duas linhas para ir escola. Os nibus de uma linha passam de 15 em 15
minutos e os da outra de 25 em 25 minutos, sendo que s 7h30m da manh os nibus
das duas linhas passam juntos.
(a) A que horas passaro juntos novamente?
(b) Entre as 7h30min da manh e a meia noite, quais so os horrios em que os nibus
passam juntos nesse ponto perto da casa de Quinzinho?
159. Quadrados dentro de um retngulo O retngulo da figura est
dividido em oito quadrados. O lado do menor quadrado mede 1 cm.
(a) Quanto mede os lados dos outros quadrados?
(b) Qual o permetro desse retngulo?

...........................................
..
..
..
..
..
..
..
..
.
..
.............................................
.. ... ... ..
.. ... ..............
.......................................

160. Festa na escola A professora Ana foi comprar po de queijo para homenagear os
alunos premiados na OBMEP, sendo que
cada 100 gramas de po de queijo custam R$ 3,20 e correspondem a 10 pes de
queijo; e
cada pessoa come, em mdia, 5 pes de queijo.
Alm da professora, estaro presentes festa 16 alunos, um monitor e 5 pais de alunos.
A preciso da balana da padaria de 100 gramas.
(a) Quantos gramas de po de queijo ela deve comprar para que cada pessoa possa
comer, pelo menos, cinco pes?
(b) Nesse caso, quanto a professora gastar?
(c) Se cada pessoa comer cinco pes de queijo, sobrar algum po de queijo?

24

OBMEP 2010

Nvel 1
161. Ai que fome Maria est olhando a tabela seguinte.
Salgados
Empada: R$ 3,90
Sanduche: R$ 2,20
Pastel: R$ 2,00

Bebidas
Refrigerante: R$ 1,90
Suco: R$ 1,20
Refresco: R$ 1,00

Doces
Sorvete: R$ 1,00
Bombom: R$ 0,50
Cocada: R$ 0,40

Maria possui 5 moedas de 50 centavos, 7 moedas de 25 centavos, 4 moedas de 10


centavos e 5 moedas de 5 centavos.
(a) Quantos reais Maria possui?
(b) Se Maria precisa guardar 90 centavos para a passagem de nibus, quais os possveis
lanches que incluam um salgado, uma bebida e um doce ela poder pedir?
162. Advinhe Tenho alguns nmeros naturais cujo nico divisor comum 1. Se eu somar
50 a cada um deles, encontro nmeros de dois algarismos. Se eu subtrair 32 de cada um
deles, tambm encontro nmeros naturais de dois algarismos. Quais so os nmeros
que eu tenho?
163. Produto de consecutivos Dentre os nmeros 712, 1 262 e 1 680, qual o nico que
pode ser escrito como um produto de quatro nmeros naturais consecutivos?
164. Palndromos O ano de 2002 um palndromo porque
no se altera quando for lido da direita para a esquerda.

373 e 1 221
foram anos palndromos.

(a) Qual ser o prximo ano palndromo depois de 2002?


(b) O ltimo ano palndromo, 1991, foi mpar. Quando ser o prximo ano palndromo
mpar?
(c) O ltimo ano palndromo primo ocorreu h mais de 1 000 anos, em 929. Quando
ocorrer o prximo ano palndromo primo?
165. O maior MDC Quais so os seis nmeros de dois algarismos cujo mximo divisor
comum o maior possvel?
166. Quantidade de gua na Terra A Terra tem, aproximadamente, um volume de
1 360 000 000 km3 de gua, que se distribui entre os oceanos, os mares, as geleiras, as
regies subterrneas (os aquferos), os lagos, os rios e a atmosfera. Somente a gua
encontrada nesses trs ltimos itens oferece um acesso fcil ao consumo humano. Com
estes dados, complete a tabela a seguir.
Especificaes
gua salgada
gua doce
Gelo
gua subterrnea
Lagos e rios
Vapor de gua

Volume de gua (km3 )

Percentual
97%

Forma decimal do percentual

40 000 000
1,8%
0,00960
250 000
0,00001
OBMEP 2010

25

Nvel 1
167. Balas De quantas formas podemos repartir 14 balas idnticas entre trs crianas de
modo que cada criana receba, no mnimo, trs balas?
168. Minutos Uma prova de Matemtica comea s 12h35min e tem uma durao de
5
4 horas. A que horas termina a prova?
6
169. Menor nmero Qual o menor nmero de cinco algarismos divisvel por 4 que se
pode formar com os algarismos 1, 2, 3, 4 e 9?
170. Contas do papagaio Antnio tem um papagaio que faz contas fantsticas com
nmeros inteiros. Quando Antnio sopra certos nmeros em seu ouvido, o papagaio
multiplica esse nmero por 5, depois soma 14, da divide o resultado por 6 e, finalmente,
subtrai 1, gritando o resultado em seguida. Entretanto, o papagaio no sabe nada sobre
decimais, de modo que, s vezes, fica sem poder gritar resposta alguma.
(a) Se Antnio soprar o nmero 8, qual nmero o papagaio gritar?
(b) Se o papagaio gritou 3, qual foi o nmero que Antnio soprou em seu ouvido?
(c) Porque o papagaio nunca grita o nmero 7?
(d) Quais so os nmeros que, soprados por Antnio, provocam uma resposta do
papagaio?
171. Soma maior do que 34 Quantos nmeros de quatro algarismos existem cuja soma
dos algarismos seja maior do que 34?
172. Nenhum 1 Roberto quer escrever o nmero 111 111 como um produto de dois
nmeros, nenhum dos quais terminado em 1. Isso possvel? Por qu?
173. Nmeros equilibrados Um nmero dito equilibrado se um de seus algarismos
a mdia aritmtica dos outros. Por exemplo, 132, 246 e 777 so equilibrados. Quantos
nmeros equilibrados de trs algarismos existem?
174. Nmeros primos Quais so os nmeros cujos triplos somados com 1 do um
nmero primo entre 70 e 110?
175. Quadro moderno Para fazer um quadro bem moderno para
sua escola, Roberto divide uma tela quadrada em oito partes com
quatro faixas de mesma largura e uma diagonal, como na figura. Ele
pinta o quadro de azul e verde, de modo que duas partes vizinhas
sempre tenham cores diferentes. No final, ele repara que usou mais
verde do que azul. Que frao do quadro foi pintada de azul?
176. Encontro de amigos Embora eu tenha certeza de que meu relgio est 5 minutos
adiantado, na realidade ele est 10 minutos atrasado. Por outro lado, o relgio do meu
amigo est realmente 5 minutos adiantado, embora ele pense que seu relgio esteja
certo. Ns marcamos um encontro para as 10 horas e planejamos chegar pontualmente.
Quem chegar em primeiro lugar? Depois de quanto tempo chegar o outro?
26

OBMEP 2010

Nvel 1
177. Trabalho comunitrio Uma classe tem 22 alunos e 18 alunas. Durante as frias,
60% dos alunos dessa classe foram prestar trabalho comunitrio. No mnimo, quantas
alunas participaram desse trabalho?
(a) 1

(b) 2

(c) 4

(d) 6

(e) 8

178. rea de trapzios Unindo quatro trapzios


idnticos, que tm lados no paralelos iguais e bases medindo 50 e 30 cm, como o da figura dada, podemos formar um quadrado de 2 500 cm2 de rea,
que tem um buraco quadrado no meio. Qual a
rea, em cm2 , de cada um dos quatro trapzios?
(a) 200

(b) 250

(c) 300

(d) 350

(e) 400

179. Adivinhao Pensei em dois nmeros de dois algarismos, que no possuem algarismos em comum, sendo um o dobro do outro. Alm disso, os algarismos do menor
nmero so a soma e a diferena dos algarismos do maior nmero. Quais so os dois
nmeros?
180. Dezoito nmeros consecutivos Escreva dezoito nmeros consecutivos de trs
algarismos e verifique que pelo menos um deles divisvel pela soma de seus algarismos.
Isso sempre verdade. Ou seja, se voc escrever dezoito nmeros consecutivos de trs
algarismos, ento pelo menos um deles ser divisvel pela soma de seus algarismos.
Mostre esse fato.
181. Completar uma tabela Descubra a regra utilizada para as casas j preenchidas e
complete a tabela. Qual o valor de A?
0
1
2
3
4

1
2

2
5

3
10

182. Procurando mltiplos de 9 Consideremos um conjunto formado por dez nmeros


naturais diferentes. Se calcularmos todas as diferenas entre esses nmeros, pelo menos
uma dessas diferenas ser um mltiplo de 9?
183. Correndo numa praa Um atleta costuma correr
15,5 km ao redor de uma grande praa retangular de
900 600 m. Ele inicia a corrida sempre do ponto P, situado
a 550 m de um dos vrtices, correndo no sentido horrio, como
mostra a figura. Em que ponto da praa ele para?
184. Ovos para um bolo Uma doceira foi ao mercado comprar ovos para fazer 43 bolos,
todos com a mesma receita, que requer menos do que nove ovos. O vendedor repara
que se tentar embrulhar os ovos que a doceira comprou em grupos de dois, ou de trs,
OBMEP 2010

27

Nvel 1
quatro, cinco, ou seis ovos, sempre sobra um ovo. Quantos ovos ela usa em cada bolo?
Qual o menor nmero de ovos que a doceira vai gastar para fazer os 43 bolos?
185. Cortando uma cartolina Uma folha retangular de cartolina foi cortada ao longo
de sua diagonal. Num dos pedaos obtidos, foram feitos dois cortes paralelos aos dois
lados menores, pelos pontos mdios desses lados. Ao final, sobrou um retngulo de
129 cm de permetro. O desenho dado indica a sequncia de cortes.
..........................................................................................................................
...
...
..
...
..
...
...
.....
...
...
...
...
...
...
.....
.....
...
...
...
.
..........................................................................................................................

........
...... ....
......
..
......
.
...
.
.
.
.
..
...
......
......
.
.
.
.
....
.
....
.
.
..
.
.
.
....
.
.
....
.
.
.
....
.
..
.
.
.
.
....
.
....
.
.
.
.
....
..
.
.
.
.
.
....
..
.
.
.
.
.
......................................................................................................................

.............................................................
...
....
....
...
..
...
.....
...
................................................................

Qual era o permetro da folha antes do corte?


186. A soma errada A soma ao lado est incorreta. Para corrigi-la, basta
substituir um certo algarismo em todos os lugares que ele aparece na conta
por um outro algarismo. Qual o algarismo errado e qual seu substituto
correto?

742586
+829430
1212016

187. Nmero de cinco algarismos Os algarismos 1, 2, 3, 4 e 5 foram usados, cada um


uma nica vez, para escrever um certo nmero a b c d e de cinco algarismos tal que a b c
divisvel por 4, b c d divisvel por 5 e c d e divisvel por 3. Encontre esse nmero.
188. Tabela misteriosa Complete a tabela 6 6 de tal
modo que, em cada linha e cada coluna, apaream
apenas mltiplos de um dos nmeros

32

40
49
22
15

2, 3, 4, 5, 6, 7, 8, 9, 10, 11 e 12.

24

Alm disso, permitido repetir apenas um nmero


na tabela.

42

189. Habitantes e esporte Numa certa cidade com quase trinta mil habitantes, exatamente dois nonos dos habitantes so homens que praticam esporte somente nos finais
de semana e dois quinze avos so mulheres que praticam esporte somente nos finais
de semana. O nmero de habitantes que no pratica esporte o quntuplo dos que
praticam esporte regularmente. Com esses dados, complete a tabela dada.

No praticam esporte
fem.
8 563

28

masc.
7 582

Praticam esporte
somente nos fins
de semana
fem.
masc.

OBMEP 2010

Praticam
esporte
regularmente
fem. masc.
1 252

Populao
total

Nvel 1
190. Botes luminosos No mecanismo luminoso da figura,
cada um dos oito botes pode acender nas cores verde ou
azul. O mecanismo funciona do seguinte modo: ao ser ligado,
todos os botes acendem a luz azul, e se apertamos um boto,
esse boto e seus dois vizinhos trocam de cor. Se ligarmos o
mecanismo e apertarmos sucessivamente os botes 1, 3 e 5,
qual ser o nmero de luzes verdes que estaro acesas no final?
(a) 3

(b) 4

(c) 5

(d) 6

1
2

7
4

6
5

(e) 7

191. Qual o nmero? Um nmero de seis algarismos comea por 1. Se deslocamos


esse algarismo 1 da primeira para a ltima posio direita, obtemos um novo nmero
de seis algarismos, que o triplo do nmero de partida. Qual esse nmero?
192. Jardim variado Um jardim retangular de 120 por 80 m foi dividido em seis regies,
conforme indicado na figura, em que N, M e P so pontos mdios dos lados e R divide
o comprimento do lado na razo 1/3. Em cada regio ser plantado um dos seguintes
tipos de flor: rosa, margarida, cravo, bem-me-quer, violeta e bromlia, cujos preos,
por m2 , esto indicados na tabela. Quais so as possveis escolhas das flores em cada
regio, de modo a se gastar o mnimo possvel?
Tipo

rosa
margarida
cravo
bem-me-quer
violeta
bromlia

1
4

P
6

Preo por m2
3,50
1,20
2,20
0,80
1,70
3,00

193. O algarismo 3 Luis escreveu a sequncia dos nmeros naturais, ou seja,


1, 2, 3, 4, 5, 6, 7, 8, 9, 10, 11, 12, . . . .
Quando ele escreveu o algarismo 3 pela 25a vez?
194. Soma de potncias Ser o nmero 3444 + 4333 divisvel por 5?
195. Telefonemas Joo mora em Salvador e seus pais em Recife. Para matar a saudade, ele telefona para seus pais a cada trs dias. O primeiro telefonema foi feito num
domingo, o segundo telefonema na quarta feira seguinte, o terceiro telefonema no sbado, e assim por diante. Em qual dia da semana Joo telefonou para seus pais pela
centsima vez?
196. O maior produto Com os algarismos de 1 a 5 e um sinal de
multiplicao, Clara forma o produto de dois nmeros, com o sinal
entre eles. Como Clara deve colocar os cartes para obter o maior
produto possvel?
OBMEP 2010

5
29

Nvel 1
197. O caminho da Joaninha Dona Joaninha quer atravessar um ptio ladrilhado com azulejos quadrados numerados, como mostra a figura dada. Ela vai partir do
ponto P e quer chegar ao ponto C, andando somente
ao longo dos lados dos azulejos. Dona Joaninha no
quer ter nmeros primos imediatamente sua direita ao
longo de todo o percurso. Qual o menor percurso que
ela pode fazer?

198. O lugar dos amigos Sete amigos traaram um tringulo, um quadrado e um crculo. Cada um marcou seu lugar com um nmero e pronunciou uma frase.
Ana:
Bento:
Celina:
Diana:
Elisa:
Fbio:
Guilherme:

Eu
Eu
Eu
Eu
Eu
Eu
Eu

...........................
.....
.........
....
...
....
........................................................
....
...
....
...
....
..
..
...
... .........
...
..
..
...
..... ...... ...
..
.
.
.
.
...
.
.
...
... ..
.. ...
.
.
.
...
...
.
.
.
......
.... ..... ...
.
.
.
....
....... ..
... .....
.
.
.
.
.
.
.
.
.
.
.
.
.. ..................................................................................
.
.
....
...
.
.
...
..
..............................................................................

2
7
3
4 6
5

no falo coisa alguma.


estou dentro de uma nica figura.
estou dentro das trs figuras.
estou dentro do tringulo mas no do quadrado.
estou dentro do tringulo e do crculo.
no estou dentro de um polgono.
estou dentro do crculo.

Encontre o lugar de cada um.


199. Quadrado perfeito? Cada um dos cinco nmeros abaixo tem cem algarismos e
formado pela repetio de um ou dois algarismos, como segue.
N1 = 333333 . . . 3
N2 = 666666 . . . 6
N3 = 151515 . . . 15
N4 = 212121 . . . 21
N5 = 272727 . . . 27
Algum desses nmeros um quadrado perfeito?
200. Preenchendo quadradinhos Complete os quadradinhos com os nmeros 1, 2, 3,
5 e 6.
+

= 4

201. Os trs nmeros Sofia brinca de escrever todos os nmeros de quatro algarismos
diferentes que se pode escrever com os algarismos 1, 2, 4 e 7. Ela soma trs desses
nmeros todos diferentes e obtm 13 983. Quais so esses trs nmeros?
202. Preencher uma tabela Jandira deve terminar de preencher uma
tabela 44 que j tem duas casas preenchidas com os nmeros 1 e 2,
conforme indicado na figura. Duas casas so consideradas vizinhas
se tm um vrtice ou um lado em comum. As regras que ela precisa
respeitar so:
30

OBMEP 2010

Nvel 1
uma casa s pode ser preenchida se alguma de suas casas vizinhas j contiver
algum nmero;
ao preencher uma casa, deve-se colocar a soma de todos os nmeros que j constam
em suas casas vizinhas.
Qual o maior nmero que possvel escrever na tabela?
203. Olimpada de Pequim Na Olimpada de Pequim sentaram-se a uma mesa quadrada, conforme indicado a seguir, as mulheres Maria e Tnia e os homens Juan e
David. Todos so atletas e cada um deles pratica um esporte diferente: natao, vlei,
ginstica e atletismo.
(a) Quem pratica a natao estava esquerda de Maria.
(b) Quem pratica ginstica estava em frente a Juan.
(c) Tnia e David sentaram-se lado a lado.
(d) Uma mulher sentou-se ao lado de quem pratica volei.
Qual dos atletas pratica atletismo?
204. Culturas diferentes Jorge, que mora em Recife, se corresponde com seu amigo
ingls Ralph, que mora na Inglaterra. Os dois se compreendem muito bem nas duas
lnguas, mas tm um problema com as datas pois, no Brasil, a data 08/10 significa 08
de outubro e, na Inglaterra, 10 de agosto. Por causa disso, os dois combinaram no se
escrever nos dias em que a data for ambgua. Eles preferem datas como 25/03, que s
pode significar 25 de maro.
(a) Em quais das datas a seguir Jorge e Ralph no podem se escrever?
(i) 3 de dezembro

(ii) 18 de agosto

(iii) 5 de maio

(b) Quando ocorrem os maiores perodos em que os dois amigos no podem se escrever?
205. Uma liquidao Na liquidao da loja SUPER-SUPER todos os produtos esto
50% mais baratos e, aos sbados, existe ainda um desconto adicional de 20%. Carla
comprou uma cala antes da liquidao, e agora ela se lamenta: No sbado eu teria
economizado R$ 50,40 na cala. Qual era o preo da cala antes da liquidao?
206. Nmero com muitos zeros Se a representa o nmero 0, 000
. . 000} 1, ento qual
| .{z
das expresses a seguir representa o maior nmero?
(a) 3 + a

(b) 3 a

(c) 3a

(d) 3/a

2009 zeros

(e) a/3

207. Corrida das tartarugas Cinco tartarugas apostaram uma corrida em linha reta e
na chegada a situao foi a seguinte: Sininha estava 10 m atrs de Olguinha e 25 m
frente de Rosinha, que estava 5 m atrs de Elzinha, que estava 25 m atrs de Pulinha.
Qual foi a ordem de chegada?
OBMEP 2010

31

Nvel 1
208. Que memria... Esquecinaldo tem pssima memria para guardar nmeros, mas
tima para lembrar sequncias de operaes. Por isso, para lembrar do seu cdigo
bancrio de cinco algarismos, ele consegue se lembrar que o cdigo no tem algarismos
repetidos, nenhum dos algarismos zero, os dois primeiros algarismos formam uma
potncia de 5, os dois ltimos formam uma potncia de 2, o do meio um mltiplo
de 3 e a soma de todos os algarismos um nmero mpar. Agora ele no precisa mais
decorar o nmero, porque ele sabe que seu cdigo o maior nmero que satisfaz essas
condies. Qual esse cdigo?
209. Uma frao irredutvel Encontre uma frao irredutvel tal que o produto de seu
numerador pelo denominador seja 2 3 4 5 10. Quantas dessas fraes
irredutveis existem?
210. Transformar em decimal Escreva o resultado das seguintes expresses na forma
decimal.


2
5
5
2
(a) 7 + 16
(b) 5 2
(c) 1 +
3
3

12

1+

1+4

211. Uma sequncia especial Escrevendo sucessivamente os nmeros naturais, obtemos


a sequncia
1 2 3 4 5 6 7 8 9 10 11 12 13 14 15 16 17 18 19 20 21 22 . . .
Qual o algarismo que est na 2 009a posio dessa sequncia?
212. Cortar um retngulo Como podemos cortar um retngulo de 13 por 7 cm em
treze retngulos diferentes sem deixar sobras?
b e B OY
b so ngulos retos e a medida de
213. Medida de ngulo Na figura dada, AOD
b est entre 40 e 50 . Alm disso, os pontos C e Y esto sobre a reta r, enquanto
D OY
b est entre
D e E esto sobre a reta s. O possvel valor para a medida de AOC
(a)
(b)
(c)
(d)
(e)

30 e 40 ;
40 e 50 ;
50 e 60 ;
40 e 60 ou
no pode ser determinado.

D s

Y
r

214. Permetros e reas Um quadrado

tem 3+ 3 cm de lado e as dimenses de um


retngulo, em centmetros, so 2 e 72 + 3 6. Qual dos dois tem maior rea? E
maior permetro?
A

215. Clculo de ngulo Encontre a medida do nb sabendo que D AC


b = 39 , AB = AC e
gulo B AD,
AD = BD.
32

OBMEP 2010

..
..........
.... ... .........
..... .
.....
..... ....
.....
.
.
.
.....
..
.....
.....
..
.....
.
.....
.
.
.
.
.
.....
.
...
.
.
.
.
.
.
.....
.
...
.
.
.....
.
.
.
.
.
.....
....
.
.
.
.
.
.....
.
...
.
.
.
.....
.
.
.
.
.....
...
.
.
.
.
.
.
....
.
...
.
.
.
.
.
.
...............................................................................................................................................................................

Nvel 1
216. O caminho da formiga Uma formiga sai de um ponto A, anda 7 cm para a
esquerda, 5 cm para cima, 3 cm para a direita, 2 cm para baixo, 9 cm para a direita,
2 cm para baixo, 1 cm para a esquerda e 1 cm para baixo, chegando no ponto B. Qual
a distncia, em cm, entre A e B?
(a) 0

(b) 1

(c) 4

(d) 5

(e) 7

217. Menino mentiroso Joozinho sempre mente nas teras-feiras, quintas-feiras e sbados e, no restante dos dias da semana, sempre fala a verdade. Um dia, Pedrinho
encontra Joozinho e ocorre o dilogo seguinte.
Pedrinho pergunta: Que dia hoje?
Joozinho responde: Sbado.

Pedrinho pergunta: E que dia ser amanh?


Joozinho responde: Quarta-feira.

Em que dia da semana Pedrinho encontrou Joozinho?


218. Encontre quatro nmeros Observe que os nmeros 1, 2, 3 e 6 tm uma propriedade
notvel: a soma de trs quaisquer deles divisvel pelo quarto nmero. Encontre quatro
nmeros distintos de trs algarismos com essa mesma propriedade notvel.
219. Colando seis tringulos Construa uma figura com seis
tringulos equilteros adjacentes, o primeiro com lado de comprimento 1 cm e os tringulos seguintes com lado igual metade do lado do tringulo anterior, como indicado na figura
dada. Qual o permetro dessa figura?
220. Os livros da Elisa Elisa tem 24 livros de cincias e outros de matemtica e literatura. Se Elisa tivesse um livro a mais de matemtica, ento um nono de seus livros
seria de matemtica e um quarto de literatura. Se Elisa tem menos do que 100 livros,
quantos livros de matemtica ela possui?
221. Substituindo pela soma Comeando com nmero natural, Mrcio substitui esse
nmero pela soma de seus algarismos, obtendo um novo nmero, com o qual ele repete
o processo, at chegar, finalmente, num nmero de apenas um algarismo. Por exemplo,
Mrcio substitui 1 784 102 por 23 e, em seguida, por 8. Ele tambm aplica esse processo
a listas de N nmeros naturais, substituindo cada nmero da lista pela soma de seus
algarismos, obtendo, assim, uma nova lista de N nmeros, com a qual ele repete o
processo, at chegar numa lista final de N nmeros, cada um de apenas um algarismo.
(a) Comeando com 32 009 , qual o nmero final de apenas um algarismo?
(b) Comeando com 172 009 , qual o nmero final de apenas um algarismo?
(c) Comeando com a lista dos primeiros 20 092 009 nmeros naturais, a lista final
tem mais algarismos 4 ou 5? Quantos 9 tem a lista final?

OBMEP 2010

33

Nvel 1
222. Uma brincadeira em sala de aula A professora Raquel inventou a seguinte
brincadeira: escrevendo um nmero inteiro positivo no quadro, acrescente trs unidades
ao nmero se ele for mpar e divida o nmero por dois se ele for par. Essa operao pode
ser feita diversas vezes. A professora est interessada em obter, ao final, o nmero 1 e
perguntou para a classe: Como obter o nmero 1 aps trs operaes? E aps quatro
operaes? E aps cinco operaes?
223. Calcule a idade Laura e sua av Ana acabaram de descobrir que, no ano passado,
suas idades eram divisveis por 8 e que, no prximo ano, sero divisveis por 7. Vov
Ana ainda no centenria. Qual a idade de Laura?
224. Divises e restos O dobro de um nmero dividido por 5 deixa resto 1. Qual o
resto da diviso desse nmero por 5?
225. Preenchendo o crculo Cada um dos sinais , , , e representa um nmero
de um algarismo. Descubra quais so esses nmeros e complete o nmero que falta no
crculo em branco.



47 423 282

34

1 448

OBMEP 2010

Nvel 2

Nvel 2
1. Populao Em 1998, a populao do Canad era de 30,3 milhes. Qual das opes
abaixo representa a populao do Canad em 1998?
(a) 30 300 000

(b) 303 000 000

(c) 30 300

(d) 303 300

(e) 30 300 000 000

2. Rguas em 15 minutos Uma certa mquina capaz de produzir oito rguas por
minuto. Quantas rguas essa mquina consegue produzir em 15 minutos?
(a) 104

(b) 110

(c) 112

(d) 128

(e) 120

3. Alturas iguais Luza, Maria, Antnio e Jlio so irmos. Dois deles tm a mesma
altura. Sabe-se que
Luza maior que Antnio;
Antnio maior do que Jlio;

Maria menor que Luza;


Jlio menor do que Maria.

Quais deles tm a mesma altura?


(a) Maria e Jlio

(c) Antnio e Luza

(b) Jlio e Luza

(d) Antnio e Jlio

(e) Antnio e Maria

4. Unidade O algarismo da unidade do nmero 1 3 5 79 97 113


(a) 1

(b) 3

(c) 5

(d) 7

(e) 9

5. Em que fio? A, B, C, D, E, F, G e H so os fios de apoio que uma aranha usa para


construir sua teia, conforme mostra a figura. A aranha continua seu trabalho. Sobre
qual fio de apoio estar o nmero 118?
(a) B
(b) D
(c) E
(d) G
(e) H

Para resolver as duas prximas questes, utilize as informaes da tabela dada, que
mostra o desempenho das selees do grupo A da Copa do Mundo de 2002. Nessas
partidas de futebol, a equipe vencedora ganha trs pontos e a perdedora no ganha
nem perde pontos; em caso de empate, as duas ganham um ponto.
Seleo
Dinamarca
Senegal
Uruguai
Frana

J
3
3
3
3

V
2
1
0
0

E
1
2
2
1

D
0
0
1
2

OBMEP 2010

GP
5
5
4
0

GC
2
4
?
3

P
7
?
2
1
35

Nvel 2
Legenda: J jogos, V vitrias, E empates, D derrotas, GP gols marcados,
GC gols sofridos, P pontos.
6. Pontos ganhos Quantos pontos obteve a seleo do Senegal?
(a) 3

(b) 4

(c) 5

(d) 6

(e) 7

7. Gols sofridos Quantos gols sofreu a seleo do Uruguai?


(a) 2

(b) 3

(c) 4

(d) 5

(e) 6

b = 50 , sendo AD e
8. Qual o ngulo? Na figura, temos B
b e C,
b respectivamente.
CD as bissetrizes dos ngulos A
b
Qual a medida do ngulo ADC?

B
50

(a) 90

(b) 100
(c) 115

(d) 122,5

(e) 125

9. Basquete O grfico mostra o nmero de pontos


que cada jogador da seleo de basquete da escola
marcou no ltimo jogo. O nmero total de pontos
marcados pela equipe foi
(a) 54
(b) 8
(c) 12
(d) 58
(e) 46
10. Telefone Geni cliente de uma companhia telefnica que oferece o seguinte plano:
tarifa mensal fixa de R$ 18,00;

gratuidade em 10 horas de ligaes por ms;

R$ 0,03 por minuto que exceder as 10 horas gratuitas.


Em janeiro, Geni usou seu telefone por 15 horas e 17 minutos e, em fevereiro, por 9
horas e 55 minutos. Qual foi a despesa de Geni com telefone nesses dois meses, em
reais?
(a) 45,51

(b) 131,10

(c) 455,10

(d) 13,11

(e) 4,55

11. rea Na figura dada, temos dois quadrados. O lado do maior mede a + b e o do
menor a. Qual a rea da regio cinza destacada?
36

OBMEP 2010

Nvel 2
(a) b
(b) a + b
(c) a2 + 2ab

a+b

(d) b2
(e) 2ab + b2

12. Comprando sorvete Veja as promoes de dois supermercados:


Supermercado A
6 latas de 3 litros do sorvete QUENTE
R$ 24,00

Supermercado B
Sorvete QUENTE lata de 3 litros
4 latas s R$ 14,00

Joana quer comprar 12 latas de sorvete para a festa de seu aniversrio. Em qual
supermercado ela deve comprar e por qu?
(a) No A, pois economizar R$ 7,00 em relao ao B.
(b) No A, pois economizar R$ 6,00 em relao ao B.
(c) No B, pois economizar R$ 8,00 em relao ao A.
(d) No B, pois economizar R$ 6,00 em relao ao A.
(e) Tanto faz, porque o preo o mesmo nos dois supermercados.
13. Cartolina e barbante Passa-se um barbante atravs dos seis furos de
uma cartolina. A frente da cartolina, com o barbante, mostrada na figura.
Qual das figuras a seguir no pode ser o verso dessa cartolina?

(a)

(b)

(c)

(d)

(e)

14. Amigos e fraes Adriano, Bruno, Csar e Daniel so quatro bons amigos. Daniel
no tinha dinheiro, mas os outros tinham. Adriano deu a Daniel um quinto do seu
dinheiro, Bruno deu um quarto do seu dinheiro e Csar deu um tero do seu dinheiro.
Cada um deu a Daniel a mesma quantia. A quantia que Daniel possui agora representa
que frao da quantia total que seus trs amigos juntos possuam inicialmente?
(a)

1
10

(b)

1
4

(c)

1
3

(d)

2
5

(e)

1
2

15. Escolhendo sorvetes Paulo quer comprar um sorvete com quatro bolas em uma
sorveteria que dispe de trs sabores: aa, baunilha e caj. De quantos modos diferentes ele pode fazer essa compra?
(a) 6

(b) 9

(c) 12

(d) 15

OBMEP 2010

(e) 18

37

Nvel 2
16. Peas de um quadrado Pedro montou um quadrado com quatro das cinco peas
abaixo. Qual a pea que ele no usou?

(a)

(b)

(d)

(e)

(c)

17. Paradas de nibus Uma linha de nibus possui 12 paradas numa rua em linha
reta. A distncia entre duas paradas consecutivas sempre a mesma. Sabe-se que a
distncia entre a terceira e a sexta paradas de 3 300 metros. Qual a distncia, em
quilmetros, entre a primeira e a ltima parada?
(a) 8,4

(b) 12,1

(c) 9,9

(d) 13,2

(e) 9,075

18. Desenho Qual dos seguintes desenhos no pode ser feito sem tirar o lpis do
papel e passando apenas uma vez ao longo de cada linha?

(a)

(b)

(c)

(d)

(e)

19. Qual o cubo? Cortamos um canto de um cubo, conforme mostra


a figura.
Qual das representaes a seguir corresponde ao que restou do cubo?

(a)

(b)

(d)

(e)

(c)

20. Quadrado mgico Dizemos que o quadrado abaixo um quadrado


mgico porque a soma dos nmeros de cada linha, de cada coluna e
de cada diagonal sempre a mesma. No caso do quadrado mgico da
figura, essa soma 15.

38

OBMEP 2010

Nvel 2
Complete os cinco nmeros que faltam no quadrado abaixo para que ele seja um quadrado mgico.

21. Torneio Sete equipes, divididas em dois grupos, participaram do torneio de futebol
do meu bairro. O Grupo 1 foi formado pelas equipes Avaqui, Botgua e Corinense. O
Grupo 2 foi formado pelas equipes Dinossauros, Esquisitos, Flurinthians e Guaran.
Na primeira rodada do torneio, cada equipe enfrentou cada uma das equipes do seu
grupo exatamente uma vez. Na segunda rodada do torneio, cada equipe enfrentou
cada uma das equipes do outro grupo exatamente uma vez.
(a) Quantas partidas foram disputadas na primeira rodada no Grupo 1?
(b) Quantas partidas foram disputadas na primeira rodada no Grupo 2?
(c) Quantas partidas foram disputadas na segunda rodada?
22. Truque numrico Voc j viu um truque numrico? Aqui vo os passos de um
truque numrico:
(i) Escolha um nmero qualquer.
(ii) Multiplique-o por 6.
(iii) Do resultado subtraia 21.
(iv) Divida esse novo resultado por 3.
(v) Desse ltimo resultado subtraia o dobro do nmero que voc escolheu.
(a) Experimente essa sequncia de cinco passos trs vezes, iniciando cada vez com
um nmero diferente. Qual foi o resultado de seu experimento?
(b) A seguir, usando a letra x para representar o nmero que voc escolheu no primeiro
passo, mostre por que os resultados do item (a) no so apenas uma coincidncia,
mas sim um fato matemtico.
23. Jogando sinuca Na figura abaixo vemos uma mesa de sinuca quadriculada e parte
da trajetria de uma bola, tacada a partir de um canto da mesa, de modo que, sempre
que a bola bater em uma das beiradas da mesa, ela segue seu movimento formando
ngulos de 45 com a beirada.
(a) Em qual das quatro caapas a bola cair?
(b) Quantas vezes a bola bater nas beiradas da mesa
antes de cair na caapa?
(c) A bola seguir pela diagonal de quantos desses quadrados durante sua trajetria?
OBMEP 2010

39

Nvel 2
A

24. Tringulo issceles Na figura, o tringulo ABC issceles,


b = 20 .
com B AC

Sabendo que BC = BD = BE, determine a medida do ngulo


b
B DE.

20

E
D
B

25. Pesando moedas So dadas quatro moedas aparentemente iguais, das quais trs
so verdadeiras e uma falsa. As trs verdadeiras tm o mesmo peso e a falsa tem um
peso diferente das verdadeiras, mas no se sabe se a moeda falsa mais leve ou mais
pesada do que as verdadeiras.
Mostre que possvel determinar a moeda falsa empregando somente duas pesagens
em uma balana de dois pratos.
Observao: Numa balana de dois pratos s podemos comparar os pesos colocados
nos dois pratos: a balana s pode ficar equilibrada ou, ento, pender para o lado mais
pesado.
26. Nmeros binomiais Os quadrados em branco da figura devem ser preenchidos com
nmeros de tal modo que cada nmero, a partir da segunda linha, seja igual soma
dos dois nmeros vizinhos da linha imediatamente superior. Por exemplo, o nmero
da primeira casa da segunda linha 11, porque 11 = 5 + 6. Qual o nmero que vai
aparecer no quadrado indicado com ?
(a) 4

(b) 6

11

(c) 9
(d) 15

60

(e) 10

27. Costuras da bola Uma bola de futebol feita com 32 peas de couro. Dessas peas
12 so pentgonos regulares idnticos e as outras 20 so hexgonos, tambm regulares
e idnticos. Os lados dos pentgonos so iguais aos lados dos hexgonos. Para unir
dois lados de duas dessas peas necessria uma costura. Quantas so as costuras
necessrias para fazer uma bola?
(a) 60

(b) 64

(c) 90

(d) 120

(e) 180

28. Razo de reas A figura ao lado mostra uma grade formada por
quadrados de 1 cm de lado. Qual a razo entre a rea sombreada
e a no sombreada?
1
1
1
2
2
(a)
(b)
(c)
(d)
(e)
4
5
6
5
7
40

OBMEP 2010

Nvel 2
29. S sorvete Em um quente dia de vero, 64 crianas comeram, cada uma, um
sorvete pela manh e outro tarde. Os sorvetes eram de quatro sabores, abacaxi,
banana, chocolate e doce de leite. A tabela dada mostra quantas crianas consumiram
um desses sabores pela manh e outro tarde. Por exemplo, o nico nmero 7 que
aparece na tabela indica que sete crianas tomaram sorvete de banana pela manh e
de chocolate tarde.

Quantas crianas tomaram sorvetes de sabores diferentes nesse dia?


(a) 58

(b) 59

(c) 60

(d) 61

(e) 62

30. Brincando com tabuleiro Camila e Lara tm, cada uma, um tabuleiro 4 4.
Comeando com ambos tabuleiros em branco, elas fazem uma brincadeira com o desdobramento seguinte.
Camila, escondida de Lara, pinta de preto algumas casas de seu tabuleiro.

Ainda em seu tabuleiro, Camila escreve em cada casa o nmero de casas vizinhas
que esto pintadas de preto (duas casas distintas so vizinhas se possuem um lado
ou um vrtice em comum).
Camila copia os nmeros escritos em seu tabuleiro no tabuleiro de Lara.

Lara deve adivinhar, a partir dos nmeros escritos em seu tabuleiro, quantas so
as casas pretas do tabuleiro de Camila.
Por exemplo, se Camila pintar seu tabuleiro como o da figura esquerda, ento ela
coloca os nmeros no tabuleiro de Lara como na figura direita.

Quantas foram as casas que Camila pintou se o tabuleiro de Lara tiver os nmeros do
tabuleiro a seguir?
(a) 3

(d) 6

(b) 4

(e) 7

(c) 5

OBMEP 2010

41

Nvel 2
31. Cartes numerados Larissa e Jorge esto jogando com cartes numerados de 1 a
6 que devem ser colocados nas casas do tabuleiro a seguir de tal modo que formem um
nmero de seis algarismos.

Jorge coloca o primeiro carto e, a seguir, as jogadas so alternadas entre os dois.


O objetivo de Larissa obter o maior nmero possvel e o de Jorge obter o menor
nmero possvel. Larissa tem os cartes com os algarismos 1, 3 e 5 e Jorge tem os
cartes com os algarismos 2, 4 e 6. Se os dois jogadores forem espertos, qual o
nmero que aparecer ao final do jogo?
(a) 254 361

(b) 253 416

(c) 251 634

(d) 256 134

(e) 251 346

32. Faltam balas Uma professora tem 237 balas para dar a seus 31 alunos. Qual o
nmero mnimo de balas a mais que ela precisa conseguir para que todos seus alunos
recebam a mesma quantidade de balas, sem sobrar nenhuma?
(a) 11

(b) 20

(c) 21

(d) 31

(e) 41

33. Artesos de braceletes Um arteso comea a trabalhar s 08h e produz seis


braceletes a cada 20 minutos; j seu auxiliar comea a trabalhar uma hora depois e
produz oito braceletes do mesmo tipo a cada meia hora. O arteso para de trabalhar s
12h, mas avisa ao seu auxiliar que dever continuar trabalhando at produzir o mesmo
nmero de braceletes que ele. A que horas o auxiliar ir parar de trabalhar?
(a) 12h

(b) 12h30min

(c) 13h

(d) 13h30min

(e) 14h30min

34. Girando um pentgono Qual figura ser obtida se girarmos no sentido


horrio o pentgono regular por um ngulo de 252 em torno do seu centro?
Observao: o sentido horrio o sentido em que giram os ponteiros de um relgio; no
caso do pentgono, isso est indicado pela seta no desenho.

(a)

(b)

(c)

(d)

(e)

35. rea em funo da diagonal O permetro de um retngulo mede 100 cm e a


diagonal mede x cm. Qual a rea desse retngulo em funo de x?
(a) 625 x2

(b) 625

x2
2

(c) 1 250

x2
2

(d) 225

x2
2

(e) 2 500

x2
2

36. Valor de uma quadrtica Se x + y = 8 e xy = 15, qual o valor de x2 + 6xy + y 2?


(a) 64

(b) 109

(c) 120

(d) 124

(e) 154

37. ngulos em funo de x Na figura esto indicadas, em graus, as medidas de


alguns ngulos em funo de x. Quanto vale x?
42

OBMEP 2010

Nvel 2
(a) 6

5x

(b) 12
(c) 18
(d) 20

(e) 24

2x

3x
6x
4x

38. Operao diferente Se m e n so inteiros maiores do que zero e se m < n,


definimos mn como a soma dos inteiros entre m e n, incluindo m e n. Por exemplo,
2226
58 = 5 + 6 + 7 + 8 = 26. Qual o valor de
?
46
(a) 4
(b) 6
(c) 8
(d) 10
(e) 12
39. Taxi caro O preo de uma corrida de txi de R$ 2,50 fixos (a bandeirada)
mais R$ 0,10 por 100 metros rodados. Tenho apenas R$ 10,00 no bolso. Logo, tenho
dinheiro para uma corrida de, no mximo, quantos quilmetros?
(a) 2,5

(b) 5,0

(c) 7,5

(d) 10,0

(e) 12,5

40. Mltiplos de 3 ou 4 Quantos nmeros entre 1 e 601 so mltiplos de 3 ou mltiplos


de 4?
(a) 100

(b) 150

(c) 250

(d) 300

(e) 430

41. Lados de um paraleleppedo Se x e y so nmeros inteiros positivos tais que


xyz = 240, xy + z = 46 e x + yz = 64, qual o valor de x + y + z?
(a) 19

(b) 20

(c) 21

(d) 24

(e) 36

42. Pontos da reta Na reta dada esto representados os seis nmeros a, b, m, n, p e q,


1
alm dos nmeros 0, , 1 e 2.
2

1
2a b

m
1

n
2

Ento os nmeros que melhor representam a + b, a b e ab so, respectivamente:


(a) m, p e q;

(b) m, q e p;

(c) n, q e p;

(d) n, p e q;

(e) q, m e p.

43. Velocidades Numa corrida de carros, um piloto percorreu trs trechos: um de


240 km, um de 300 km e um de 400 km. O piloto sabe que as velocidades mdias
nesses trechos foram 40 km/h, 75 km/h e 80 km/h, mas no se lembra qual dessas
velocidades corresponde a cada um desses trechos. Podemos garantir que o tempo
total em horas gasto pelo piloto para percorrer os trs trechos foi:
(a) menor do que ou igual a 13 horas;
OBMEP 2010

43

Nvel 2
(b) maior do que ou igual a 13 horas e menor do que ou igual a 16 horas;
(c) maior do que ou igual a 16 horas e menor do que ou igual a 17 horas;
(d) maior do que ou igual a 15 horas e menor do que ou igual a 18 horas;
(e) maior do que ou igual a 18 horas.
44. Comprimento de diagonal Do quadrado ABCD foram
cortados os tringulos issceles sombreados, como na figura,
restando o retngulo P QRS. Sabendo que a rea total do que
foi cortado mede 200 cm2 , qual o comprimento de P R, em
cm?

(a) 200 (b) 200 (c) 800 (d) 25


(e) 88

B
Q

S
D

45. Diviso de nmeros grandes Determine o valor de 123 456 123 456 10 000 001.
46. Refrigerante no cinema Toda vez que Joozinho vai ao cinema, ele toma dois
refrigerantes. Ele gastou toda sua mesada de R$ 50,00 indo ao cinema seis vezes e
tomando um total de 20 refrigerantes, incluindo os que ele tomou quando foi ao cinema.
Se Joozinho tivesse tomado s um refrigerante cada vez que foi ao cinema, com essa
economia ele poderia ter ido ao cinema mais uma vez, tomando um refrigerante tambm
nessa ocasio. Em relao ao preo do ingresso do cinema e o preo do refrigerante,
podemos afirmar que
(a) o preo do ingresso o triplo do preo do refrigerante;
(b) o preo do ingresso o qudruplo do preo do refrigerante;
(c) o preo do ingresso o quntuplo do preo do refrigerante;
(d) o ingresso R$ 6,00 mais caro do que o refrigerante;
(e) o ingresso R$ 5,00 mais caro do que o refrigerante.
47. Diviso de potncias Qual o quociente de 5050 por 2525 ?
(a) 2525

(b) 1025

(c) 10025

(d) 225

(e) 2 2525

48. Palitos de dois tamanhos Voc possui apenas palitos com 6 e 7 cm de comprimento. Qual o nmero mnimo desses palitos que so necessrios para cobrir um
segmento de reta com 2 metros?
(a) 29

(b) 30

(c) 31

(d) 32

(e) 33

49. Maior raiz Qual a maior raiz da equao (x 37)2 169 = 0?


(a) 39

(b) 43

(c) 47

(d) 50

(e) 53

50. Mquina com visor Uma certa mquina tem um visor, onde aparece um nmero
inteiro x, e duas teclas, A e B. Quando se aperta a tecla A, o nmero x do visor
substitudo por 2x + 1. Quando se aperta a tecla B, o nmero x do visor substitudo
por 3x 1. Qual o maior nmero de dois algarismos que pode ser obtido apertando
44

OBMEP 2010

Nvel 2
alguma sequncia das teclas A e B a partir do nmero 5 no visor?
(a) 85

(b) 87

(c) 92

(d) 95

(e) 96

51. Quadrado mgico parcial Num quadrado mgico, a soma dos


trs nmeros de cada linha, coluna ou diagonal sempre a mesma.
Dado o quadrado mgico ao lado, parcialmente preenchido, qual
deve ser o valor de x?
(a) 20
(b) 22
(c) 23
(d) 25
(e) 27
52. rea do retngulo Um retngulo ABCD est dividido
em quatro retngulos menores. As reas de trs deles esto
indicadas na figura dada. Qual a rea do retngulo ABCD?
(a) 80

(b) 84

(c) 86

(d) 88

1
26

14

x
13

A
16

(e) 91

12

27
C

53. Lado do quadrado Quatro peas iguais, em forma de tringulo retngulo, foram
dispostas de dois modos diferentes, como mostram as figuras dadas. Os quadrados
ABCD e EF GH tm lados respectivamente iguais a 3 cm e 9 cm. Determine a
medida do lado do quadrado IJKL.
H
I
D

A
B

54. Maior nmero Qual o maior dentre os nmeros dados?


(a) 2 0 2 006
(b) 2 0 + 6

(c) 2 + 0 2 006
(d) 2 (0 + 6)

(e) 2 006 0 + 0 6

55. Operao O smbolo representa uma operao especial com nmeros; alguns
exemplos so 2 4 = 10, 3 8 = 27, 4 27 = 112 e 5 1 = 10. Quanto vale 4 (8 7)?
(a) 19

(b) 39

(c) 120

(d) 240

(e) 260

56. Terceiro lado Se dois lados de um tringulo medem 5 e 7 cm, ento o terceiro lado
no pode medir quantos centmetros?
(a) 11

(b) 10

(c) 6

(d) 3

(e) 1

3 2
1
= , quanto vale ?
24 8 3
6
(b) 21
(c) 23
(d) 25
(e) 29

57. Asterisco Se
(a) 20

OBMEP 2010

45

Nvel 2
58. Expresses algbricas O que representam, geometricamente, na figura dada, as expresses

a2 + 1,5 a

1,5

e
4a+3.
59. Faixa decorativa A figura dada composta de tringulos retngulos issceles, todos
congruentes. Qual a rea, em cm2 , da parte sombreada?
(a) 20

(d) 45

(b) 25

(e) 50
30 cm

(c) 35

60. Bicicleta e chocolate Se eu der duas barras de chocolate para Tio, ele me empresta
sua bicicleta por 3 horas. Se eu lhe der 12 bombons, ele me empresta a bicicleta por 2
horas. Amanh, eu lhe darei uma barra de chocolate e 3 bombons. Por quantas horas
ele me emprestar a bicicleta?
(a) 1/2

(b) 1

(c) 2

(d) 3

(e) 4

61. Retas paralelas? Na figura dada, as retas EC e F D sero paralelas?


A
62

E
F
42
48

C
28

62. Menor nmero Se x > 5, ento qual dos nmeros dados o menor?
(a) 5/x

(b) 5/(x + 1)

(c) 5/(x 1)

(d) x/5

(e) (x + 1)/5

63. rea de quadrado O quadrado ST UV formado de um quadrado limitado por 4 retngulos iguais. O permetro de cada retngulo 40 cm. Qual a rea, em cm2 , do quadrado ST UV ?

46

(a) 400

(c) 160

(b) 200

(d) 100

(e) 80

OBMEP 2010

Nvel 2
64. Operando fraes
1 1 1 1 1 1 1 1 1
,
,
,
e .
2 2 3 3 4 4 5 5 6
1 1
1
1
1
(b) Deduza de (a) o valor da soma + +
+
+
.
2 6 12 20 30
1
1 1
1
1
1
1
+
+
+
++
.
(c) Calcule a soma + +
2 6 12 20 30 42
999 000
(a) Calcule as diferenas 1

65. ngulos e permetro Calcule os ngulos que no esto indicados e o permetro da figura, sabendo que BD = BC e
b = B CD.
b
D BC

B
26

120 m
26
30
E

C
115 m

70

226 m

66. Desigualdade racional Quais so os valores de x que satisfazem a desigualdade


1
< 4?
x2
9
9
(a) x >
(c) x < 2 ou x >
(e) x < 2
4
4
9
(d) x < 2
(b) 2 < x e x <
4
67. Desigualdade p
dupla Quantos nmeros inteiros e positivos satisfazem a dupla inequao 2 000 < n(n 1) < 2 005?
(a) 1

(b) 2

(c) 3

(d) 4

(e) 5

68. Dimetro do crculo Na figura, O o centro do crculo


e AB = 5 cm. Qual o dimetro desse crculo?

B
O
A

69. Falta um ngulo Na figura dada, T U = SV.


Quanto vale o ngulo S Vb U, em graus?
(a) 30

(d) 65

(b) 50

(e) 70

5
4

U
30
T

75

(c) 55

50
S

70. Caf, bolo e gato Dez minutos antes de colocar o bolo no forno, coloquei meu
gato para fora da casa. O bolo deve cozinhar por 35 minutos, portanto coloquei o
despertador para tocar 35 minutos aps colocar o bolo no forno. De imediato fiz um
caf para mim, o que me tomou 6 minutos. Trs minutos antes de acabar de beber
o caf, meu gato entrou em casa. Isso foi 5 minutos antes do despertador tocar. O
telefone tocou no meio do tempo entre eu acabar de fazer o caf e o gato entrar em
casa. Falei ao telefone por 5 minutos e desliguei. Eram, ento, 3h59min da tarde.
OBMEP 2010

47

Nvel 2
(a) A que horas coloquei meu gato para fora?
(b) O despertador tocou quantos minutos depois de colocar o gato para fora?
(c) Por quanto tempo o gato j estava fora de casa quando o telefone tocou?
71. Muitos ngulos Quais figuras esto corretas?

Figura I

72
112
18

95

63

Figura II

45

62

Figura III

44

29

72. Sinal de produto e de quociente a, b, c e d so quatro nmeros no nulos tais que


os quocientes
b
11
18
a
,
,
e
5
7a
abc
abcd
so todos positivos. Determine os sinais de a, b, c e d.
73. Sinais e radicais Quais dos nmeros dados so negativos?

(a) 10 3 11
(c) 18 5 13
(e) 10 26 51

(b) 3 11 10
(d) 51 10 26
74. ngulos entre retas Sabe-se que as retas r e s so paralelas. Encontre os ngulos
x e y.
60

x
y
80

48

OBMEP 2010

Nvel 2
75. Variao de temperatura A tabela dada mostra as temperaturas mximas e mnimas em centgrados durante cinco dias seguidos em certa cidade. Em qual dia ocorreu a maior variao de
temperatura?

Dia
a
2 feira
3a feira
4a feira
5a feira
6a feira

Mx.(o C)
7
0
2
9
13

Mn.(o C)
12
11
15
8
7

76. Ordenando fraes Qual dos nmeros fica entre 2/5 e 3/4?
(a) 1/6

(b) 4/3

(c) 5/2

(d) 4/7

(e) 1/4

77. Frao de rea A figura mostra um retngulo maior


dividido em 18 retngulos menores, todos com a mesma
largura. Qual a frao do retngulo maior que representa a parte em cinza?
78. Uma a mais! Dentre as nove fraes
5
,
4

17
,
6

5
,
4

10
,
7

2
,
3

14
,
8

1
,
3

5
3

3
2

temos oito com as propriedades seguintes.


2 fraes cuja soma

2
5

2 fraes cujo produto

2
5

2 fraes cuja diferena

2
5

2 fraes cujo quociente

2
5

Encontre a frao que est sobrando.


79. Qual o ngulo? No tringulo KLM temos
b = 30 . Qual a
KL = KM, KT = KS e LKS
b ?
medida do ngulo x = T SM
(a) 10

(b) 15

(c) 20

(d) 25 (e) 30

K
T
L

x
S

80. Operao circular Dentro dos crculos, escreva os


nmeros inteiros que tornam correta a sucesso de operaes.
81. Pratos e copos Iara possui R$ 50,00 para comprar copos e pratos. Cada copo
custa R$ 2,50 e cada prato, R$ 7,00. Ela quer comprar, no mnimo, 4 pratos e 6 copos.
O que ela pode comprar?
82. Desigualdades de inteiros Quantos so os nmeros inteiros x tais que
5 < x 1 5?
(a) 8

(b) 9

(c) 10

(d) 11
OBMEP 2010

(e) 12
49

Nvel 2
83. Nove quadrados A figura dada mostra nove quadrados. A rea do quadrado A mede 1 cm2 e a do quadrado
B 81 cm2 . Qual a rea, em cm2 , do quadrado I?
(a) 196

(d) 324

(b) 256

(e) 361

(c) 289

84. Muitas medalhas Andr, Bruno, Celina e Dalva ganharam, juntos, 21 medalhas
num concurso. Andr foi o que mais ganhou medalhas, Bruno ganhou o dobro de
Celina e Dalva ganhou trs a mais do que Bruno. Quantas medalhas cada um pode
ter ganhado?
85. As somas so quadrados Escreva numa linha os nmeros de 1 a 15 de tal modo
que a soma de quaisquer dois nmeros adjacentes nessa linha seja um quadrado perfeito.
86. rea de uma regio Um retngulo est dividido em
trs regies, conforme indicado na figura. Se as reas de
duas delas medem 24 e 13 cm2 , qual a rea da terceira
regio?
87. Potncias de 10 O valor de
(a) 101 ;

(b) 102 ;

0,00001 (0,01)2 1 000


:
0,001

(c) 103 ;

(d) 104 ;

(e) 1.

88. Diferena de quadrados Se (x + y)2 (x y)2 = 20, ento xy igual a:


(a) 0;

(b) 1;

(c) 2;

(d) 5;

(e) 10.

89. Um quadriltero O quadriltero ABCD da figura um paralelogramo?


.
..
..
..
..........
..
.......
...........
....... ...
..
.
.....
.
.
.
.
...
.
.
.
.....
.
..
......
..
..
......
..
..
......
..
..
......
.
.
.
.
.
.
...
.
.
.......
...
..
.
.........................
...... .....
.......................
.
.. ......... ..........................
..
..................................
..
.
... .....
.

.
.........
...
.
..
..
..
..
..
..
..
.
.
.
.
.
.
.
.....
.
..

........................
.. ...
.......................
.. ... ..........................
.........................

45

45

115

65

90. Sexta-feira treze Qual o nmero mximo de sexta-feiras treze que podem ocorrer
num ano que no bissexto? Nesse caso, em que dia da semana cai o dcimo dia do
ano?
91. Tringulos com lados inteiros Quantos tringulos existem cujos lados so nmeros inteiros e o permetro mede 12 unidades?
(a) 1
50

(b) 3

(c) 5

(d) 7

(e) 9

OBMEP 2010

Nvel 2
92. Festa de aniversrio Para comemorar seu aniversrio, Ana vai preparar tortas de
pera e de ma. No mercado, uma ma pesa 300 g e uma pera, 200 g. A sacola de
Ana aguenta um peso mximo de 7 kg. Qual o nmero mximo de frutas que ela
pode comprar para poder fazer tortas das duas frutas?
93. Os dois quadrados As medidas em centmetros dos lados de cada um
dos dois quadrados da figura so nmeros inteiros. Se o menor quadrado
tivesse 2 001 cm2 a mais de rea, as reas dos dois quadrados seriam
iguais. Quanto pode medir o lado do maior quadrado?

..............................................................................
...
...
...
....
...
...
...
...
.....
...
...
...
...
....
....
...
.............................
...
...
...
...
...
...
..
...
...
..................................................................................

94. A multiplicao Jlio faz multiplicaes usando apenas os quadrados dos nmeros.
Ele tem que calcular o produto 85 135. Para isso, ele desenha um retngulo de
85 135 mm e, nesse retngulo, traa o maior quadrado possvel; faz o mesmo no
retngulo restante e assim sucessivamente. Dessa maneira, ele obtm oito quadrados
que ele, ento, soma. Desenhe a figura feita por Jlio e escreva 85 135 como a soma
de oito quadrados:
85 135 = 852 + . . . .
95. Expresso fracionria Se
(a) 1;

1
(b) ;
2

(c)

x
xy
= 2, ento
igual a:
y
x
1
;
2

(d) 1;

(e) 2;

96. Diferena e soma de quadrados Calcule:


(a) 1 6782 1 6772

(b) 1 0012 + 1 0002

(c) 19 9992

(d) 2 0012 + 2 0022 + 2 0032

97. Um queijo triangular Osvaldo comprou um queijo em forma de um tringulo


equiltero. Ele quer dividir o queijo igualmente entre ele e seus quatro primos. Faa
um desenho indicando como ele deve fazer essa diviso.
98. Notas de Matemtica Joo e Cludia receberam de volta suas provas de matemtica em que os algarismos das notas foram substitudos por smbolos. A nota de
Joo foi  e a de Cludia > . Juntos, eles obtiveram >  . Alm disso, Cludia
obteve 13 pontos a mais do que Joo. Qual foi a nota de cada um?
p


99. Operao com raiz quadrada O nmero ( 6 + 2)( 3 2)
3 + 2 igual a:

(a) 3;
(b) 2;
(c) 2;
(d) 1;
(e) 2.
100. Para a escola de bicicleta Ctia sai da escola todos os dias no mesmo horrio
e volta para casa de bicicleta. Quando ela pedala a 20 km/h, ela chega em casa s
16h30m. Se ela pedalar a 10 km/h, ela chega em casa s 17h15m. A que velocidade
ela deve pedalar para chegar em casa s 17h?
101. Distncia na reta Cinco pontos esto sobre uma mesma reta. Quando listamos
as 10 distncias entre dois desses pontos, da menor para a maior, encontramos 2, 4, 5,
7, 8, k, 13, 15, 17 e 19. Qual o valor de k?
OBMEP 2010

51

Nvel 2
102. Nmero mpar Se n um nmero inteiro qualquer, qual dos seguintes um nmero
mpar?
(a) n2 n + 2

(b) n2 + n + 2

(c) n2 + n + 5

(d) n2 + 5

(e) n3 + 5

103. Quatro nmeros inteiros Se quatro inteiros positivos distintos m, n, p e q satisfazem a equao (7 m)(7 n)(7 p)(7 q) = 4, ento a soma m + n + p + q igual
a:
(a) 10;

(b) 21;

(c) 24;

(d) 26;

(e) 28.

104. As pginas do dicionrio Para numerar as pginas de um dicionrio, imprimiu-se


1 988 vezes o algarismo 1. Quantas pginas tem esse dicionrio?
105. Soma de potncias de 2 Determine um valor de n para o qual o nmero 28 +211 +2n
seja um quadrado perfeito.
106. Reverso de um nmero O reverso de um nmero inteiro de dois algarismos
o nmero que se obtm invertendo a ordem de seus algarismos. Por exemplo, 34 o
reverso de 43. Quantos nmeros existem que, somados ao seu reverso, do um quadrado
perfeito?
107. ngulos externos de um tringulo Dados os ngulos de 150 e 160 indicados na figura, calcule os valores dos ngulos x, y e z.

160

150

150
160

108. Uma brincadeira feita uma brincadeira com quatro nmeros inteiros da seguinte
maneira: some trs desses nmeros, divida essa soma por 3 e o resultado some com o
quarto nmero. Existem quatro formas de fazer esta brincadeira, obtendo os seguintes
resultados: 17, 21, 23 e 29. Qual o maior dos quatro nmeros?
109. Ovos e mas Num armazm, uma dzia de ovos e 10 mas tinham o mesmo
preo. Depois de uma semana, o preo dos ovos caiu 2% e o da ma subiu 10%.
Quanto se gastar a mais na compra de uma dzia de ovos e de 10 maas?
(a) 2%

(b) 4%

(c) 10%

(d) 12%

(e) 12,2%

110. Dividir um cubo Se dividirmos um cubo de 1 m de aresta em cubinhos de


1 mm de aresta, que altura ter uma coluna formada por todos os cubinhos, dispostos
sucessivamente um em cima do outro?
(a) 1 m
52

(b) 1 km

(c) 10 km

(d) 100 km

OBMEP 2010

(e) 1 000 km

Nvel 2
111. Uma expresso A expresso
(a)

a3
;
2

(b)

2
;
a3

(c)

a2
4a

, em que a 6= 0, igual a:
a5
(21 a)3

1
;
2a3

(d)

a5
;
2

(e)

2
.
a5

112. Uma igualdade Os nmeros a e b so inteiros positivos que satisfazem 96a2 = b3 .


Qual o menor valor possvel de a?
113. Somas de trs em trs Encontre quatro nmeros inteiros positivos que, somados
de trs em trs, do somas 6, 7, 8 e 9.
114. O retngulo do Lus Lus desenhou um retngulo de 6 10 cm e quer dividi-lo
em quatro partes. As reas das 4 partes devem medir 8, 12, 16 e 24 cm2 . Desenhe
como ele pode fazer essa diviso.
115. Uma fbrica de blusas Uma fbrica produz blusas a um custo de R$ 2,00 por
unidade, alm de uma parte fixa de R$ 500,00. Se cada unidade produzida comercializada a R$ 2,50, a partir de quantas unidades produzidas a fbrica obtm lucro?
(a) 250

(b) 500

(c) 1 000

(d) 1 200

(e) 1 500

116. Existncia de tringulos Qual dos seguintes tringulos no pode existir?


(a) tringulo agudo issceles
(b) tringulo retngulo issceles
(c) tringulo retngulo obtusngulo
(d) tringulo retngulo escaleno
(e) tringulo escaleno obtusngulo
117. Os doze pontos Doze pontos esto marcados numa folha de papel
quadriculado, conforme mostra a figura. Qual o nmero mximo de
quadrados que podem ser formados unindo quatro desses pontos?

r r
r r r r
r r r r
r r

118. O colar Um colar composto de prolas grandes e prolas pequenas, num total de
menos do que 500 prolas.
(a) Se substituirmos 70% das prolas grandes por pequenas, o peso do colar diminui
60%.
(b) Se substituirmos 60% das prolas pequenas por grandes, o peso do colar aumenta
70%.
Quantas prolas tem o colar?
OBMEP 2010

53

Nvel 2
119. Mulheres votantes Numa certa cidade, 40% de todas as mulheres so votantes
e 52% da populao de mulheres. Qual o percentual da populao formado por
mulheres votantes?
(a) 18,1%

(b) 20,8%

(c) 26,4%

(d) 40%

(e) 52%

120. Amigos do sculo XX Dois amigos nasceram no mesmo ms e ano do sculo


XX, com uma semana de intervalo. Escrevendo as datas dos dois aniversrios da
esquerda para a direita, comeando com o (ou os) algarismo(s) do dia, depois o (ou
os) algarismo(s) do ms e, por ltimo, os dois ltimos algarismos do ano, obtemos dois
nmeros. No colocando o algarismo 0 na frente dos nove primeiros dias do ms nem
dos nove primeiros meses do ano e sabendo que um desses nmeros o sxtuplo do
outro, qual a data de nascimento do amigo mais velho?
121. Operao em uma frao Que nmero se deve somar aos dois termos de uma
frao para se obter o inverso dessa mesma frao?
122. O nmero 119 O nmero 119 tem as propriedades seguintes:
(a) a diviso por 2 deixa resto 1;
(b) a diviso por 3 deixa resto 2;
(c) a diviso por 4 deixa resto 3;
(d) a diviso por 5 deixa resto 4;
(e) a diviso por 6 deixa resto 5.
Quantos inteiros positivos menores que 2 007 satisfazem essas propriedades?
123. Fonte com trs torneiras Slvia vai encher seus 10 garrafes numa fonte que tem
trs torneiras. Um dos garrafes demora um minuto para encher, outro dois minutos,
outro trs minutos e assim por diante. Como Slvia dever distribuir os 10 garrafes
pelas trs torneiras de modo a gastar o menor tempo possvel? Qual esse tempo?
124. A sequncia xyz Quais so os valores provveis de x, y e z na sequncia
1 5 3 7
, , , , x, y, z?
2 8 4 8
125. A mesa circular J existem N pessoas sentadas em volta de uma mesa circular
com 60 cadeiras. Qual o menor valor possvel para N se a prxima pessoa a se sentar
vai ter que se sentar ao lado de algum?
126. Nmeros proporcionais Se
(a)

54

x2
;
9

(b) x3 z;

x
3
= , ento 9y 2 igual a:
y
z

(c) 3x2 ;

(d) x2 z 2 ;

OBMEP 2010

(e)

1 2 2
xz .
9

Nvel 2
127. Esportistas de uma escola Em um grupo de 40 estudantes, 20 jogam futebol, 19
jogam vlei e 15 jogam exatamente um desses dois esportes. Quantos estudantes no
praticam nem futebol nem vlei?
(a) 7

(b) 5

(c) 13

(d) 9

(e) 10

128. Vamos ao teatro Na campanha Vamos ao teatro, 5 ingressos podem ser adquiridos pelo preo usual de 3 ingressos. Mrio comprou 5 ingressos nessa campanha. A
economia que Mrio fez representa que percentual sobre o preo usual dos ingressos?
(a) 20%

(b) 33

1
%
3

(c) 40%

(d) 60%

129. Uma desigualdade Os valores de x que satisfazem


(a) x < 2;

(b) x > 1;

(c) 1 < x < 2;

2
%
3

(e) 66

1
> 1 so dados por:
x1

(d) x < 1;

(e) x > 2.

130. A sala do Professor Newton O professor Newton dividiu seus alunos em grupos
de 4 e sobraram 2. Ele dividiu seus alunos em grupos de 5 e um aluno ficou de fora.
Se 15 alunos so mulheres e tem mais mulheres do que homens, o nmero de alunos
homens :
(a) 7;

(b) 8;

(c) 9;

(d) 10;

(e) 11.

131. Um jardim retangular Na figura, o retngulo ABCD


representa um terreno retangular cuja largura mede 3/5 do
comprimento. O retngulo ABEF representa um jardim retangular cuja largura tambm mede 3/5 do comprimento.

D.....................................................................................F
.............................................. A
..
..
...
...
...
...
...
...
...
...
..
...
...
...
..
..
...
...
.
..
...
...
...
...
...
...
...
...
...
...
...
...
..
...
...
...................................................................................................................................

Qual a razo entre a rea do jardim e a rea total do terreno?


(a) 30%

(b) 36%

(c) 40%

(d) 45%

(e) 50%

132. Nmeros decrescentes Escreva em ordem decrescente os nmeros


 1 3
 1 1

5
3 , 32/3 , 32 ,
e
.
3
3

133. Os bombons misturados Marta e Carmem ganharam, cada uma, muitos bombons.
Elas misturaram todos os bombons e, agora, no sabem mais qual foi o nmero de
bombons que cada uma ganhou. Vamos ajud-las a descobrir esses nmeros? Sabe-se
que:
(a) juntas, elas ganharam 200 bombons;
(b) Marta se lembra que ganhou menos do que 100 bombons, mas mais do que 4/5
do que ganhou Carmem; e
(c) o nmero de bombons que cada uma ganhou um mltiplo de 8.
134. Jantar aos sbado Trs casais jantam todo sbado num mesmo restaurante, sempre
mesma mesa. A mesa redonda e os casais combinaram que
OBMEP 2010

55

Nvel 2
(a) jamais marido e mulher sentam mesa como vizinhos; e
(b) a disposio dos seis mesa diferente a cada sbado.
Desconsiderando rotaes nas disposies mesa, durante quantos sbados esses trs
casais podero ir a esse restaurante sem repetir sua disposio mesa?
135. Expresso com radicais O valor de
(a)

2+

3;

(b)

1
2


7+3 5 ;

q

4
p

1+ 1+ 1
:

(c) 1 + 2 3;

(d) 3;

(e) 3 + 2 2.

136. Possveis tringulos Os lados de um tringulo tm comprimentos a, a + 2 e a + 5,


sendo a > 0. Determine todos os possveis valores de a.
137. Uma diferena O valor de
(a) 3,3 ;

(b) 4,7 ;

0,001 400
0,036 0,4

:
0,25
0,4

(c) 4,9 ;

(d) 3,8 ;

(e) 7,5 .

138. A Terra A superfcie do globo terrestre consiste em 70% de gua e 30% de terra.
Dois quintos da terra so desertos ou cobertos por gelo e um tero da terra pastagem,
floresta ou montanha; o resto da terra cultivado. Qual o percentual da superfcie
total do globo terrestre que cultivada?
139. Uma frao Na figura dada, determine o valor da frao
AN
.
AC

...A...
... ....
.. ....
. .
M....................N
....
....
..................................
B
C

140. Cculo de ngulo Na figura dada, a reta P Q paralela reta RS e T U = T V. Se


c S mede 110, o ngulo QU
b V mede:
o ngulo T W
(a) 135 ;

(b) 130 ;

(c) 125 ;

(d) 115 ;

U
110

T
V
P

56

W
R

OBMEP 2010

(e) 110 .

Nvel 2
141. Uma loja de brinquedos Uma loja estava vendendo cada unidade de um brinquedo
a R$ 13,00. Para conseguir vender todo o seu estoque, que no era superior a 100
unidades, a gerncia da loja resolveu baixar o preo por um nmero inteiro de reais,
obtendo R$ 781,00 por todo o estoque. Qual foi a reduo do preo, por unidade?
1

142. Frao de frao Qual o valor de 1 +

1+

1+

1
2

143. Potncias de 3 Se 3a = 2, quanto vale 27 2 a ?


144. Aumento de preo Se o preo de um produto passou de 5,00 para 5,55 reais, qual
foi o percentual do aumento?
145. Roseiras em fila Jorge ganhou 15 roseiras para seu jardim e quer plant-las em
6 filas de 5 roseiras cada uma. Isso possvel? Em caso afirmativo, faa um desenho
indicando como Jorge pode plantar suas roseiras.
146. Calculadora diferente Uma fbrica produziu uma calculadora original que efetua
duas operaes,
(a) a adio usual, denotada por + e
(b) uma operao denotada por .
Sabemos que, para todo nmero natural a, valem
(i) a a = a

(ii) a 0 = 2a

e, para quaisquer quatro nmeros naturais a, b, c e d, vale


(iii) (a b) + (c d) = (a + c) (b + d) .
Quais so os resultados das operaes (2 + 3) (0 + 3) e 1 024 48?
147. Dois quadrados Na figura dada, o lado do quadrado maior mede
10 cm e o lado do menor mede 4 cm. As diagonais do quadrado
maior contm as diagonais do quadrado menor. Quanto mede a
rea da regio tracejada?
148. Paralelismo Sendo o segmento IL paralelo ao segmento EU e o segmento RE paralelo ao segmento NI,
determine o valor da frao

...................................................................................................
.. ........
.... ..
... .. .....
.... ..
... ... ... ......
.... ....
...
... .. .. ... ......
...
.
.
... ... ... ... .. ......................................................
...
.
... ... ... .... .... .... .......
...
.... ..
.. .. .. .. ... .. ..... ....... ....
..
......
.
... ... ... ... .. ...
...
.
.. .. .. .. .. .. .............. ...
..
.... ...
... .... .... .... .... .... .....
...
.... ..
.. .. .. .. .. .......
..
.
.
.
... .... .... .... ......................................................
...
... .. .. .....
..
....
.
.
.
. ..
.... ...
.... ..
..... ..............
.......
......
............................................................................................

............
............
.............
.............
. ...
.... ... ................
.... .... ...............................
....
... .... .. ...............
.
.
............
...
..
.......
.
.
............
.
...
............
...
.......
...
..
....
.... ....
.
.
.
................
...
.
.
...
..
..
..........................
.
.
.
.
.
.
.
.
.
.
.
.
.
.
.
.
.
.
.
.
.
.
.
.
.
.
.
.
.
.
.
..
.. .........................................
.
.
.
.
.
.
.......................................
.
.
.
.
.
..
..........................

FN FU
.
FR FL

OBMEP 2010

U L

57

Nvel 2
149. Um subconjunto O conjunto {1, 2, 3, . . . , 3 000} contm um subconjunto de 2 000
elementos em que nenhum elemento o dobro do outro?
s

150. Tringulos retngulos Determine os valores de ........................


.. ... w
v, w, x, y e z na figura dada, em que j esto marcados 8 ................r.......... ............... 9
z
...s
..................................s
........................................................s
......................................................................................s
..
... r ....
.....
trs ngulos retos e os comprimentos de trs segmentos.
.................
v ..............
.....
...
....
.....
.
.

....

.
...
.....
...
.....
..
.....
...
.....
....
...
.....
.
.
.
.... ..
..
.... ........... .........
....... .. ..........
....... .....
......

y ..............

20

r
s

151. Uma desigualdade especial Que valores de x satisfazem x2 < |x| + 2?


(a) x < 1 ou x > 1

(b) x > 1

(c) 2 < x < 2

(d) x < 2

(e) x < 0

152. Sapo Cururu Cururu um sapo estranho, que se desloca apenas com dois tipos de
saltos, o de
Tipo I:
Tipo II:

10 cm para o Leste e 30 cm para o Norte e o de;


20 cm para Oeste e 40 cm para o Sul.
20 cm

30 cm

10 cm

Tipo I

40 cm

Tipo II

(a) Como Cururu faz para chegar a um ponto situado a 190 cm para o Leste e
950 cm para o Norte de sua casa?
(b) possvel Cururu chegar a um ponto situado a 180 cm para o Leste e 950 cm
para o Norte de sua casa?
153. Distribuindo algarismos em linhas Joana escreveu uma sequncia em 10 linhas
usando os algarismos de 0 a 9, seguindo o padro seguinte.
0
1 1 0
2 2 2 1 1 0
3 3 3 3 2 2 2 1 1 0
..
..
.
.
Qual foi o algarismo mais usado? Quantas vezes esse algarismo foi utilizado?
154. Ser que existe? Existe algum nmero inteiro N tal que valha
2 008 N = 222 . . . 2?
58

OBMEP 2010

Nvel 2
155. Conferindo uma desigualdade Ser verdade que

1
1
1
1
+ 3+ 3 <
?
3
4
5
6
12

156. Parte inteira A parte inteira de um nmero real x o maior inteiro que menor
do que ou igual a x. Denotamos a parte inteira de x por [x]. Por exemplo, [2,9] = 2,
[0,88] = 0 e [1,7] = 2.
1,7

2,9

0,88

Calcule as partes inteiras seguintes.




 
28 756
(a)
12
(b)
12 777

(c)

2 007

2 008

(d)


3

111

157. Soma nove Quantos nmeros inteiros entre 10 e 999 tm a soma de seus algarismos
igual a 9?
158. Retngulos As medidas dos lados de um retngulo so nmeros pares. Quantos
retngulos desses existem com rea igual a 96?
159. Nmero de retas Sabemos que dois pontos distintos determinam uma
nica reta. Quantas retas so determinadas por dois quaisquer dos nove
pontos marcados no quadriculado dado?
160. Cubo Pedro quer pintar uma caixa de formato cbico de tal maneira que as faces que
tenham uma aresta em comum sejam pintadas em cores diferentes. Calcule o nmero
mnimo de cores que sero necessrias para pintar a caixa dessa maneira.
161. rea Um lote retangular foi divido em quatro terrenos,
todos retangulares. As reas de trs deles esto dadas na
figura, em km2 . Qual a rea do lote?

27

18

72

162. Inteiro mais prximo Determine o nmero inteiro mais prximo de


(a)

19 19
+
15
3

(b)

85 43 29 15
+
+
+
42 21 14
7

(c)

11 1 7 2
+
10 2 5 3

163. Brincando com nmeros mpares Beatriz adora nmeros mpares. Quantos
nmeros entre 0 e 1 000 ela pode escrever usando apenas algarismos mpares?
164. gua no jarro Joo e Maria tm, cada um, um jarro grande com um litro de gua.
No primeiro dia, Joo coloca 1 ml da gua do seu jarro no jarro da Maria. No segundo
dia, Maria coloca 2 ml da gua do seu jarro no jarro do Joo. No terceiro dia, Joo
coloca 3 ml da gua do seu jarro no jarro da Maria, e assim por diante. Depois de 200
dias, quantos mililitros de gua tem no jarro de Maria?
OBMEP 2010

59

Nvel 2
165. Formiga no cubo Uma formiga parte de um vrtice de um cubo, andando somente
ao longo das arestas, at voltar ao vrtice inicial, no passando duas vezes por nenhum
vrtice. Qual o passeio de maior comprimento que essa formiga pode fazer?
166. Promoo Em uma promoo, Joana comprou blusas por R$ 15,00 cada uma e
calas por R$ 17,00 cada uma, gastando, ao todo, R$ 143,00. Quantas blusas e calas
Joana comprou?
167. Soma de cubos Se x + y = 1 e x2 + y 2 = 2, calcule x3 + y 3.
168. O revezamento em uma corrida Numa competio de revezamento, em que
cada equipe tem dois atletas, cada atleta corre 21 km e o segundo atleta s inicia a
corrida quando o primeiro atleta termina a sua parte e lhe passa o basto. O recorde
dessa competio de 2 horas e 48 minutos. Na equipe de Joo e Carlos, Joo inicia
a corrida e corre a sua parte com uma velocidade de 12 km/h. Para bater o recorde,
qual deve ser a velocidade de Carlos?
169. Produtos consecutivos Divida os nmeros 2, 3, 5, 7, 11, 13 e 17 em dois grupos
de tal forma que, multiplicando todos os nmeros de um grupo e todos do outro,
encontremos nmeros consecutivos.
170. Distraindo na fila Vivi, Tnia e Rosa esto em fila, no necessariamente nessa
ordem, e gritam sucessivamente, cada uma, um mltiplo de 3.
3
12
..
.

6
15
..
.

9
18
..
.

Vivi foi a primeira a gritar um nmero maior que 2 003 e Rosa a primeira a gritar um
nmero de quatro algarismos. Quem gritou o nmero 666? E o 888?
171. Nmero e o dobro Um nmero menor do que 200 formado por trs algarismos
diferentes e o dobro desse nmero tambm tem todos os algarismos diferentes. Ainda,
o nmero e seu dobro no tm algarismos em comum. Qual esse nmero? Quantas
solues tm esse problema?
172. Invertendo os algarismos Entre 10 e 99, quantos nmeros existem tais que,
invertendo a ordem de seus algarismos, obtemos um nmero maior do que o nmero
original?
173. Razo entre segmentos Na figura, O o centro do semicrculo de dimetro P Q, R um ponto sobre o semicrculo e RM

perpendicular a P Q. Se a medida do arco P R o dobro da

medida do arco RQ, qual a razo entre P M e MQ?

174. Tringulos Quantos tringulos existem que tenham um permetro de 15 unidades


e lados medindo nmeros inteiros?
60

OBMEP 2010

Nvel 2
175. Nmero interessante O nmero 119 muito interessante porque deixa resto 1 ao
ser dividido por 2, deixa resto 2 ao ser dividido por 3, deixa resto 3 ao ser dividido por
4, deixa resto 4 ao ser dividido por 5 e, finalmente, deixa resto 5 ao ser dividido por 6.
Existem outros nmeros de trs algarismos com essas propriedades?
176. Time vencedor Um time de futebol ganhou 60% das 45 partidas j disputadas.
Qual o nmero mnimo de partidas que esse time ainda precisa vencer para atingir
uma porcentagem de 75% de vitrias?
177. Brincando com dados Dois dados so lanados. Qual a probabilidade de o
produto dos nmeros obtidos nos dois dados ser divisvel por 6?
178. Contando solues Quantos so os pares de nmeros inteiros positivos (x, y) tais
que
xy
= 144 ?
x+y
179. Crculos tangentes Os vrtices de um tringulo cujos lados medem 3, 4 e 5 cm,
so centros de trs crculos que so dois a dois tangentes exteriormente. Qual a soma
das reas desses trs crculos?
180. Grupo de amigos Joo, Jorge, Jos e Jnio so bons amigos. Certa vez, Joo
estava sem dinheiro, mas seus amigos tinham algum. Ento Jorge deu a Joo um
quinto de seu dinheiro, Jos deu um quarto de seu dinheiro e Jnio deu um tero de
seu dinheiro. Se todos eles deram a mesma quantidade de dinheiro para Joo, que
frao do dinheiro do grupo ficou com Joo?
181. Um trapzio A figura dada representa um trapzio ABCD
em que AB paralelo a CD e as diagonais AC e BD cortam-se
no ponto P. Se as reas dos tringulos AP B e CP D medem
4 e 9 cm2 , respectivamente, qual a rea do tringulo P CB?

B
P

182. Vista ruim Numa classe, 40% dos alunos no enxergam bem. Desses, 70% usam
culos e os 30% restantes usam lentes de contato. Sabendo que 21 alunos usam culos,
quantos alunos tem nessa classe?
183. Idade mdia da populao de Campo Verde A razo entre o nmero de homens
e o de mulheres na cidade de Campo Verde de 2/3. A idade mdia dos homens
37 anos e a das mulheres 42 anos. Qual a idade mdia dos habitantes de Campo
Verde?
184. rea de tringulo Se AC = 1 cm e AD = 4
cm, qual a relao entre as reas dos tringulos
ABC e CBD?

OBMEP 2010

.........
............... .....
........... ....
...
........... .....
...
.......... .........
.
.
.
.
.
...
.
.
.
.
.
..
...
....
...........
...
.....
..........
.
.
.
.
.
.
.
.
.
.
.
...
.
.
.
.
..
......
.
.
.
.
.
.
...
.
.
.
.
.
.
.
.
.
...
........
...
.
.
.
.
.
.
.
.
.
.
.
.
.
...
...
.......
.
.
.
.
.
.
.
.
.
.
.
...
.
.
...
........
.
.
.
.
...
.
.
.
.
.
.
.
.
.
......
...
.
.
.
.
.
.
.
.
.
.
.
.
.
.
.
...........................................................................................................................................................................................................

61

Nvel 2
185. Construindo quadrados perfeitos Observe as igualdades a seguir.

1234+1 =
2345+1 =
..
.

25 = 52
121 = 112

10 11 12 13 + 1 = 17.161 = 1312

..
.

Ser que isso sempre verdadeiro? Isto , ser sempre um quadrado perfeito o produto
de quatro nmeros inteiros consecutivos, mais 1?
186. Feira de Cincias Na Feira de Cincias
de uma escola, observou-se que metade dos
alunos do ensino fundamental e um quarto
dos alunos do ensino mdio presentes nesse
evento compraram um adesivo cada.

FEIRA DE CINCIAS
Preo dos Adesivos (unidade)
R$ 0,30 alunos do ensino fundamental
R$ 0,50
alunos do ensino mdio

Notou-se tambm que o nmero de alunos do ensino mdio presentes que no compraram adesivos foi o dobro do nmero de alunos do ensino fundamental que no
compraram adesivos. Sabendo-se que foram arrecadados R$ 38,00 na venda de adesivos para os alunos desses dois nveis, quantos alunos de cada nvel participaram da
feira?
187. Par perfeito Dizemos que dois nmeros naturais formam um par perfeito quando
a soma e o produto desses dois nmeros so quadrados perfeitos. Por exemplo, 5 e 20
formam um par perfeito, pois 5 + 20 = 25 = 52 e 5 20 = 100 = 102 . Ser que 122
forma um par perfeito com algum outro nmero natural?
188. Um trapzio No trapzio da figura dada, AB paralelo a DC, AD = AB = BC =
b
1 cm e DC = 2 cm. Quanto mede o ngulo D AC?
(a) 30

(b) 45

(c) 60
(d) 90

...................................................................................
...
...
...
...
...
...
.
...
..
...
.
.
...
.
.
...
..
.
.
...
...
...
.
..
...
.
...
..
.
...
.
.
.
.
.................................................................................................................................................................

(e) 120

189. Mistrio das bolas Henrique tm duas urnas. A primeira urna contm somente
bolas pretas e a segunda somente bolas brancas. Henrique retirou um certo nmero de
bolas da primeira urna e as colocou na segunda. Em seguida, retirou o mesmo nmero
de bolas da segunda urna e as colocou na primeira. Depois disso, o nmero de bolas
brancas na primeira urna maior do que, menor do que ou igual ao nmero de bolas
pretas na segunda urna?
62

OBMEP 2010

Nvel 2
190. Contando a palavra BRASIL Quantas vezes aparece a palavra BRASIL na figura dada? S vale ler a palavra emendando letras que esto escritas em quadradinhos
adjacentes.
B

B
R

B
R
A

B
R
A
S

B
R
A
S
I

B
R
A
S
I
L

191. Quais so os nmeros? Descubra quais so os nmeros inteiros positivos x e y


que satisfazem a equao x4 = y 2 + 71.
192. No jogo Aldo, Bernardo e Carlos jogam baralho. No incio, a quantia em dinheiro
que eles tinham, na ordem Aldo : Bernardo : Carlos, estava na proporo 7 : 6 : 5.
No final do jogo, na mesma ordem, a proporo era de 6 : 5 : 4. Se um dos jogadores
ganhou 12 reais, qual foi a quantidade de dinheiro com que ficou cada jogador, no final
da partida?
193. Um nmero inteiro Mostre que M =

5+2

5 2 um nmero inteiro.

194. rea de tringulos A rea do quadrado ABCD mede 300 cm2 .


Na figura, M o ponto mdio de DC e o ponto F pertence reta
que passa por B e C.

(a) Qual a rea do tringulo ABF ?

(b) Qual rea do tringulo AF D?

195. Um quadriculado Observe que o retngulo quadriculado na figura ao lado constitudo de 31 segmentos e compreende doze quadrados.
Numa folha retangular de 21 por 29,7 cm, quadriculada com quadrados de lados medindo 0,5 cm, Rosa desenhou um grande retngulo quadriculado, constitudo de 1 997
segmentos. Quantos quadrados tem esse retngulo?
196. Inteiros de quatro algarismos Determine o valor do nmero natural a, sabendo
4
que 4a2 e a3 so nmeros inteiros de quatro algarismos.
3
197. Pares positivos Quantos pares (x, y) de inteiros positivos so solues da equao
3x + 5y = 501?
198. Diferena de quadrados A diferena dos quadrados de dois nmeros inteiros
consecutivos 2 000.
(a) Os dois inteiros so menores do que 100.
(b) Os dois inteiros so menores do que 1 000, porm maiores do que 99.
(c) Os dois inteiros so menores do que 10 000, porm maiores do que 999.
(d) Os dois inteiros so menores do que 100 000, porm maiores do que 9 999.
OBMEP 2010

63

Nvel 2
(e) No existem esses dois nmeros.
199. Clculo de ngulos Calcule o valor do ngulo x em cada uma das figuras a seguir,
sabendo que os segmentos AB e ED so paralelos.

A.........................................................................................................................................................B
.

A.........................................................................................................................................................B

....
....
....
....
....
....
....
....
....
....
....
.
....
....
.
.
.
...
.
.
...
.
.
...
....
....
...
150o .......
.........................................................................................................................................................

....
....

25o ........
.

160o

....
....
....
....
.
.
.
...
...
.......
....
....
....
....
....
....
....
....
....
....
55o
.........................................................................................................................................................

200. Tabela Na tabela ao lado, com seis colunas e diversas


linhas, esto escritos, ordenadamente, os nmeros 1, 2,
3, 4, . . . Qual a posio do nmero 1 000 nessa tabela?

1
7
13

2
8
14

3
9

4
10

5
11

6
12

..
.

..
.

..
.

..
.

..
.

..
.

201. Entre 1 e 2 Encontre todos os inteiros positivos a e b tais que a/5 e b/7 sejam
menores do que 1 e valha a condio
1<

a b
+ < 2.
5 7

202. Triatlon Maria est planejando participar do Triatlon-Brasil que consta de 800 m
de nado, seguido de 20 km de bicicleta e, finalmente, 4 km de corrida. Maria corre a
uma velocidade constante que o triplo da velocidade com que nada e pedala 2,5 vezes
mais rpido do que corre. Para terminar a prova em, no mximo, 1 hora e 20 minutos,
qual deve ser sua velocidade mnima em cada uma das trs modalidades?
203. Foto de formatura O diretor de certa escola decidiu tirar uma foto dos formandos
de 2008. Ele colocou os alunos em filas paralelas, todas com o mesmo nmero de alunos,
mas essa disposio era muito larga para o campo de viso de sua mquina fotogrfica.
Para resolver esse problema, o diretor resolveu tirar um aluno por fila, colocando-os
numa nova fila. Essa disposio no agradou o diretor porque a nova fila tinha quatro
alunos a menos do que as outras. Ele decide, ento, tirar mais um aluno de cada fila
original, colocando-os na nova fila recm criada, e constata que, agora, todas as filas
ficaram com o mesmo nmero de alunos e finalmente tira sua foto. Quantos alunos
apareceram na foto?
204. Circunferncias tangentes Desenhe duas circunferncias de mesmo centro, uma
de raio medindo 1 cm e a outra de raio medindo 3 cm. Na regio exterior circunferncia de 1 cm de raio e interior de 3 cm de raio, desenhe circunferncias que sejam,
simultaneamente, tangentes s duas circunferncias, como mostrado na figura dada.
64

OBMEP 2010

Nvel 2
....................................
................
... .......
....... ..
... .....
..... ....
....
..
..
....
.
.
................................
...
..
.
.
....
...
....
.....
......................
....
..
.
....... ..............
..
... ...
....
.
.
...
.
......
.. ...
.
...
...
.. ...................... ...
..
.
.
..
...
...
..
...
....
...
......................
..
...
..
.
.
...
..
...
...
....
....
....
....
......
.....
.
.........
.
.
.
.
.
......................................

(a) Qual deve ser o raio dessas circunferncias?


(b) Qual o nmero mximo dessas circunferncias que podem
ser desenhadas, sem que elas se sobreponham?

205. Festa na escola Para a festa de aniversrio da escola, Ana, Pedro, Miriam e Fbio
levaram um total de 90 docinhos. A professora deles observou que
se Ana tivesse levado dois docinhos a mais;

se Pedro tivesse levado dois docinhos a menos;


se Miriam tivesse levado o dobro e
se Fbio tivesse levado a metade,

os quatro amigos teriam levado todos o mesmo nmero de docinhos. Quantos docinhos
levou cada um dos amigos?
206. Inflao Mrcia est numa loja comprando um gravador que ela queria h muito
tempo. Quando o caixa registra o preo ela exclama: No possvel, voc deve ter
registrado o nmero ao contrrio e trocou a ordem de dois algarismos, pois lembro que,
na semana passada, custava menos do que 50 reais! Responde o caixa: Sinto muito,
mas ontem todos os nossos artigos foram aumentados em 20%. Qual o novo preo
desse gravador?
207. Gatos no condomnio Num certo condomnio moram 29 famlias, cada uma das
quais possui ou um, ou trs, ou cinco gatos. O nmero de famlias que possuem apenas
um gato o mesmo que o de famlias que possuem cinco gatos. Quantos gatos tem
nesse condomnio?
208. Soma constante Preencha as cinco casas em branco da tabela 3 3
dada com os nmeros de 3 a 8, sem repeti-los, de modo que as somas dos
quatro nmeros escritos nas quatro subtabelas formadas por quadrados
2 2 seja sempre a mesma.

2
9

209. Qual o nmero? Na adio ao lado, letras iguais representam o


mesmo algarismo e letras diferentes, algarismos diferentes. Encontre o
nmero A B C D E.

A B C D
B C D
C D
D

E
E
E
E
E
A A A A A

210. Proporo triangular Num tringulo ABC, o ponto F


est sobre o lado AC e F C = 2AF. Se G o ponto mdio do
segmento BF e E o ponto de interseo da reta passando por
A e G com o segmento BC, calcule a razo EC/BE.

F
G
B

211. Nmeros primos entre si Encontre todos


 x os ypares
 de inteiros positivos (x, y) tais
que x < y, x e y so primos entre si e 2 000
+
seja um nmero inteiro mpar.
y x
OBMEP 2010

65

Nvel 2
212. Fique atento Determine todas as solues da equao

x = x 2.

213. Solues inteiras Determine todos os nmeros inteiros x e y tais que


1 1
1
+ =
.
x y
19
214. No ponto de nibus Um certo nmero de meninos e meninas aguardam pelo
nibus. No primeiro nibus que passa no ponto em que se encontram, embarcam
somente quinze meninas e ficam dois meninos para cada menina no ponto de nibus.
No segundo nibus que passa, embarcam somente 45 meninos e ficam cinco meninas
para cada menino no ponto de nibus. Determine o nmero de meninos e meninas que
estavam no ponto antes de passar o primeiro nibus.
215. Contorno circular A figura a seguir formada por quatro crculos tangentes de raio
a. Determine o comprimento do contorno externo, que est com o traado destacado.

216. Um quadriltero especial Dois lados consecutivos de um quadriltero medem 10 e


15 cm. Se cada diagonal divide o quadriltero em duas regies de mesma rea, calcule
seu permetro.
217. Nmero curioso O nmero 81 tem a seguinte propriedade: ele divisvel pela
soma de seus algarismos, 8+1=9. Quantos nmeros de dois algarismos cumprem essa
propriedade?
218. Nmero premiado Um nmero de seis algarismos premiado se a soma de seus
primeiros trs algarismos for igual soma de seus trs ltimos algarismos. Por exemplo,
342 531 premiado, pois 3 + 4 + 2 = 5 + 3 + 1.
(a) Quais so o maior e o menor nmero premiado com seis algarismos distintos?
(b) Mostre que a soma de todos os nmeros premiados com seis algarismos distintos
divisvel por 13.
219. Altura versus lado Seja ABC um tringulo tal que a altura relativa ao lado BC
no menor do que o lado BC e a altura relativa ao lado AB no menor do que o
lado AB. Determine as medidas dos ngulos deste tringulo.
66

OBMEP 2010

Nvel 2
220. Fraes egpcias Determine todos os nmeros inteiros positivos distintos x e y tais
que
1 1
2
+ = .
x y
7
221. Tabuleiro de xadrez De quantas maneiras podemos colocar dois bispos de mesma
cor num tabuleiro de xadrez em filas, colunas e casas de cores distintas?
222. Quem menor? Sem usar calculadora, decida qual dentre os nmeros 3312 , 6310
e 1278 o menor.
223. Brincando com nmeros A soma 1 + 1 + 4 dos algarismos do nmero 114 divide o prprio nmero. Qual o maior nmero menor do que 900 que satisfaz essa
propriedade?
224. Cortando papis No incio de uma brincadeira, Andr tinha sete pedaos de papel.
Na primeira rodada da brincadeira, ele pegou alguns destes pedaos e cortou cada um
deles em sete pedaos, que foram misturados aos pedaos de papel que no foram cortados nessa rodada. Na segunda rodada, ele novamente pegou alguns pedaos e cortou
cada um deles em sete pedaos, que foram misturados aos demais papis. Continuando
dessa maneira, ao final de alguma rodada, Andr poder ter exatamente 2 009 pedaos
de papel?
225. Um trapzio especial A base AD de um trapzio ABCD mede 30 cm. Suponhamos que exista um ponto E na base AD tal que os tringulos ABE, BCE e
CDE tenham permetros iguais. Determine o comprimento de BC.
b = 20o e GHI
b =
226. Uma estrela Na estrela ABCDE da figura dada, sabe-se que GBF
b
130o . Qual o valor do ngulo J EI?

E
I

H
C

227. Nmero palndromo Um nmero dito palndromo se sua leitura da direita para
a esquerda for igual da esquerda para a direita. Por exemplo, os nmeros 23 432
e 18 781 so palndromos. Quantos nmeros palndromos de quatro algarismos so
divisveis por 9?

OBMEP 2010

67

Nvel 2
228. Multiplicao com letras Na operao dada, as letras a, b e c representam algarismos distintos e diferentes de 1. Determine os valores de a, b e c.
abb
c
bcb1
229. Nmeros sortudos Digamos que um nmero sortudo se a soma de seus algarismos
for divisvel por sete. Por exemplo, 7, 25 e 849 so nmeros sortudos. Os dois menores
nmeros sortudos so 7 e 16.
(a) Encontre oito nmeros consecutivos, dos quais dois so nmeros sortudos.
(b) Encontre doze nmeros consecutivos, tais que nenhum seja sortudo.
(c) Mostre que qualquer sequncia de treze nmeros consecutivos contm, pelo menos,
um nmero sortudo.
230. Uma sequncia especial Na sequncia 1, 3, 2, . . . cada termo depois dos dois
primeiros igual ao termo precedente, subtrado do termo que o precede, ou seja,
se n > 2, ento an = an1 an2 . Qual a soma dos cem primeiros termos dessa
sequncia?
231. Tringulos e ngulos. . . Determine os ngulos e dados na figura.
130

70

45

68

OBMEP 2010

Nvel 3

Nvel 3
1. Usando velas Uma cidade ainda no tem iluminao eltrica, portanto, nas casas
usam-se velas noite. Na casa de Joo, usa-se uma vela por noite, sem queim-la
totalmente, e com quatro desses tocos de velas, Joo fabrica uma nova vela. Durante
quantas noites Joo poder iluminar sua casa dispondo de 43 velas?
(a) 43

(b) 53

(c) 56

(d) 57

(e) 60

2. Rodas e bandeiras Juliano encaixou duas rodas dentadas


iguais, cada uma com uma bandeirinha igual desenhada, como
mostra a figura.
Ento ele girou a roda da esquerda um pouco. Qual das alternativas abaixo pode
representar a posio final das rodas?

(a)

(b)

(d)

(e)

(c)

3. Nmero de latas Uma fbrica embala latas de palmito em caixas de papelo de


formato cbico de 20 cm de lado. Em cada caixa so colocadas 8 latas e as caixas
so colocadas, sem deixar espaos vazios, em caixotes de madeira de 80 cm de largura
por 120 cm de comprimento e 60 cm de altura. Qual o nmero mximo de latas de
palmito em cada caixote?
(a) 576

(b) 4 608

(c) 2 304

(d) 720

(e) 144

4. Qual a menor frao? Quantas fraes da forma


7/9, sabendo que n um nmero inteiro positivo?
(a) 1

(b) 2

(c) 3

(d) 4

n
so menores do que
n+1

(e) 5

5. Pistas de corrida Um atleta corre 5 000 m por semana em uma quadra de esportes,
que tem uma pista curta e outra longa. Em uma certa semana, ele treinou seis dias,
sendo que a cada dia correu uma vez na pista longa e duas na pista curta. Na semana
seguinte, ele treinou sete dias, sendo que a cada dia correu uma vez em cada pista.
Podemos, ento, afirmar que:
(a) a pista longa 500 m mais longa do que a curta;
(b) a pista longa quatro vezes maior do que a curta;
(c) a pista longa cinco vezes maior do que a curta;
OBMEP 2010

69

Nvel 3
(d) a pista longa 600 m mais longa do que a curta;
(e) a pista longa trs vezes maior do que a curta.
6. Brincos e brincos Numa certa povoao africana vivem 800 mulheres, 3% das
quais usam apenas um brinco. Das demais, a metade usa dois brincos e a outra
metade, nenhum. Qual o nmero total de brincos usados por todas as mulheres
dessa povoao?
(a) 776

(b) 788

(c) 800

(d) 812

(e) 824

7. Perguntas e respostas Ana, Bento e Lucas participam de um concurso que consta


de 20 perguntas, com as regras seguintes.
Cada resposta certa vale 5 pontos.

Cada resposta errada acarreta a perda de 3 pontos.

Cada resposta em branco acarreta a perda de 2 pontos.


certas
12
13
12

Ana
Bento
Lucas

erradas
3
7
4

em branco
5
0
4

Usando os resultados do concurso da tabela e escrevendo os nomes dos trs em ordem


decrescente de classificao no concurso, obtemos:
(a) Ana, Bento, Lucas;

(c) Ana, Lucas, Bento;

(b) Lucas, Bento, Ana;

(d) Lucas, Ana, Bento;

(e) Bento, Ana, Lucas.

8. Qual a carga? O limite de peso que um caminho pode transportar corresponde


a 50 sacos de areia ou a 400 tijolos. Se esse caminho j carrega 32 sacos de areia,
quantos tijolos, no mximo, ele ainda pode carregar?
(a) 132

(b) 144

(c) 146

(d) 148

(e) 152

9. Quanto mede a cerca? Uma cerca reta de arame tem 12 postes igualmente
espaados. A distncia entre o terceiro e o sexto poste de 3,3 m. Qual o comprimento
da cerca, em metros?
(a) 8,4

(b) 12,1

(c) 9,9

(d) 13,2

(e) 9,075

1
10. Dzima peridica Sabendo que 0,333. . . = , qual a frao irredutvel equivalente
3
a 0, 1333 . . .?
(a)

70

1
13

(b)

1
15

(c)

1
30

(d)

2
15

OBMEP 2010

(e)

1 333
10 000

Nvel 3
11. Valor absoluto O valor absoluto |a| de um nmero a qualquer definido por

a
|a| = 0

se a > 0,
se a = 0,
se a < 0.

Por exemplo, |6| = 6, | 4| = 4 e |0| = 0. Quanto vale N = |5| + |3 8| | 4|?


(a) 4

(b) 4

(c) 14

(d) 14

(e) 6

12. O peso das frutas Marcos quer pesar, numa balana de dois pratos, uma banana,
uma ma e um mamo. Em cada uma das figuras dadas, a balana est em equilbrio,
isto , os contedos que esto no prato da direita tm o mesmo peso que os que esto
no prato da esquerda. Em duas das trs pesagens foi utilizado um peso de 200 gramas.
Podemos afirmar que as trs frutas tm um peso total, em gramas, de

(a) 250;

(b) 300;

(c) 350;

(d) 400;

(e) 450.

13. Maratona Andr treina para a maratona dando voltas em torno de uma pista
circular com 100 m de raio. Para percorrer 42 km, o nmero de voltas que Andr
precisa dar est entre:
(a) 1 e 10;

(b) 10 e 50;

(c) 50 e 100;

(d) 100 e 500;

(e) 500 e 1 000.

14. Dobrando papel Uma folha quadrada foi dobrada duas vezes ao longo de suas
diagonais, obtendo-se um tringulo. Em seguida, foi feito um corte reto na folha
dobrada, paralelo ao lado maior desse tringulo, passando pelos pontos mdios dos
outros lados, conforme a ilustrao dada.

Desdobrando a folha, obteve-se um buraco quadrado no meio da folha. A rea do


buraco corresponde a qual frao da rea de toda a folha quadrada original?
(a)

1
2

(b)

1
6

(c)

3
8

(d)

3
4

(e)

1
4

15. Encontre o nmero Qual o menor nmero inteiro positivo N tal que N/3, N/4,
N/5, N/6 e N/7 sejam todos nmeros inteiros?
(a) 420

(b) 350

(c) 210

(d) 300

OBMEP 2010

(e) 280
71

Nvel 3
16. Equao quadrtica Se 3 e 1/3 so as razes da equao ax2 6x + c = 0, qual
o valor de a + c?
9
18
(a) 1
(b) 0
(c)
(d)
(e) 5
5
5
17. Cubo Os vrtices de um cubo so numerados de 1 a 8, de tal modo que uma das
faces tem os vrtices {1, 2, 6, 7} e as outras cinco tm os vrtices {1, 4, 6, 8}, {1, 2, 5, 8},
{2, 3, 5, 7}, {3, 4, 6, 7} e {3, 4, 5, 8}. Qual o nmero do vrtice que est mais distante
do vrtice de nmero 6?
(a) 1

(b) 3

(c) 4

(d) 5

(e) 7

18. Time de basquete O grfico dado mostra o nmero de pontos que os oito jogadores de basquete
do time da escola marcaram no ltimo jogo.
Qual o nmero total de pontos marcados pelo
time?
(a) 54 (b) 8 (c) 12 (d) 58 (e) 46
19. O caminho da formiguinha Uma formiguinha vai caminhar de A at C, podendo
passar apenas uma vez pelo ponto B e usando somente os caminhos indicados na figura.

B
A

Qual o nmero de maneiras diferentes que ela pode escolher para caminhar de A at
C?
(a) 3

(b) 5

(c) 7

(d) 8

(e) 9

20. Operao z Dados dois nmeros reais a e b, considere a z b = a2 ab + b2 . Quanto


vale 1 z 0?
(a) 1

(b) 0

(c) 2

(d) 2

(e) 1

21. Indo para a escola O diagrama de barras mostra


a distribuio dos alunos de uma escola de acordo com
o tempo que gastam no trajeto de casa para a escola.
As fraes de minuto foram desconsideradas; por exemplo, se um aluno gasta 40 minutos e 15 segundos neste
trajeto, considera-se que o tempo gasto de 40 minutos.

nmeros de alunos
100
menos de 20 min
de 20 a 40 min
50

de 41 a 60 min
mais de 60 min

10

Responda s perguntas seguintes justificando sua resposta.


(a) Quantos alunos gastam menos do que 20 minutos para chegar escola?
(b) Quantos alunos tem esta escola?
72

OBMEP 2010

Nvel 3
(c) Quantos alunos gastam mais do que 40 minutos para chegar escola?
(d) verdade que a maioria dos alunos gasta mais do que 20 minutos para chegar
escola?
22. Campeonato de futebol No ltimo campeonato de futebol do bairro em que moro
participaram seis equipes, denominadas A, B, C, D, E e F. Cada equipe disputou, com
cada uma das outras, exatamente uma partida. A tabela de classificao do campeonato fornecida a seguir, sendo V o nmero de vitrias, E o nmero de empates, D
o nmero de derrotas, GP o nmero de gols marcados e GC o nmero de gols sofridos
para cada equipe.

(b) A tabela est incompleta. Determine a


quantidade de vitrias da equipe F, a
quantidade de derrotas da equipe D e a
quantidade de gols marcados pela equipe
F, representados na tabela por x, y e z.

Equipe
A
B
C
D
E
F

V
4
2
0
1
0
x

E
1
1
3
1
1
1

23. Poste eltrico Uma companhia de eletricidade instalou um poste num terreno plano. Para fixar bem o
poste, foram presos cabos no poste, a uma altura de 1,4
metros do solo e a 2 metros de distncia do poste, sendo
que um dos cabos mede 2,5 metros, conforme a figura.

D
0
2
2
y
4
0

GP
6
6
2
3
1
z

1,4 m

(a) Quantas partidas foram disputadas?

GC
2
6
6
6
5
3

2,5 m
2m

Um professor de Matemtica, aps analisar estas medidas, afirmou que o poste no est
perpendicular ao solo. Voc acha que o professor est certo? Justifique sua resposta.
24. Equaes recprocas Briot (matemtico ingls, que viveu de 1817 a 1882) e Ruffini
(matemtico italiano, que viveu de 1765 a 1822) desenvolveram mtodos para encontrar
solues para as equaes chamadas recprocas. Nesta questo, voc vai desenvolver,
passo a passo, a essncia desses mtodos. O item (a) uma preparao para os demais
itens.
1
1
1
(a) Se y = x + , calcule as expresses x2 + 2 e x3 + 3 em termos de y.
x
x
x
1
5
(b) Determine todas as razes reais da equao x2 5x + 8 + 2 = 0.
x x
(c) Determine todas as razes reais de x4 5x3 + 8x2 5x + 1 = 0.
(d) Determine todas as razes reais de x6 2x5 5x4 + 12x3 5x2 2x + 1 = 0.
25. Atirando flechas Manoel testa sua pontaria lanando cinco flechas que atingiram o
alvo nos pontos A, B, C, D e E, de coordenadas A = (1, 1), B = (2,5; 1), C = (1, 4),
D = (4, 4) e E = (6, 5).
OBMEP 2010

73

Nvel 3
A tabela mostra quantos pontos se ganha quando
a flecha acerta um ponto dentro de cada uma das
trs regies, conforme mostra a figura.
(a) Marque os pontos A, B, C, D e E.
(b) Quantas flechas ele acertou no interior do menor crculo?
(c) Ao todo, quantos pontos Manoel fez?

26. Festa de aniversrio A festa de aniversrio de Andr tem menos do que 120
convidados. Para o jantar, ele pode dividir os convidados em mesas completas de seis
pessoas ou em mesas completas de sete pessoas. Em ambos os casos, so necessrias
mais do que 10 mesas e todos os convidados ficam em alguma mesa. Quantos so os
convidados?
27. Medida do cateto Na figura dada, ABCD um retngulo e ABE e CDF so tringulos retngulos. A rea
do tringulo ABE 150 cm2 e os segmentos AE e DF medem, respectivamente, 15 e 24 cm. Qual o comprimento do
segmento CF ?

15

F
E

24
C

28. Sequncia de Peri Usando apenas os dgitos 1, 2, 3, 4 e 5, Peri construiu a


sequncia
1, 2, 2, 3, 3, 3, 4, 4, 4, 4, 5, 5, 5, 5, 5, 1, 1, 1, 1, 1, 1, 2, 2, 2, 2, 2, 2, 2, . . .,
comeando com um 1, seguido de dois 2, trs 3, quatro 4, cinco 5, seis 1, sete 2, e assim
por diante. Qual o centsimo termo dessa sequncia?
29. rea em azulejo A figura dada foi montada com 12 azulejos quadrados de lados iguais a 10 cm. Qual a rea da
regio destacada?
30. Os cartes de Capitu Capitu tem cem cartes numerados de 1 a 100. Todos
cartes tm uma face amarela e a outra vermelha e o nmero de cada carto est
escrito em ambas as faces. Os cartes foram colocados sobre uma mesa, todos com a
face vermelha voltada para cima. Capitu virou todos os cartes de nmero par e depois
todos os cartes de nmero mltiplo de 3, colocando-os com a face amarela voltada
para cima. Quantos cartes ficaram com a face vermelha para cima?
31. Enchendo o tanque Para encher de gua um
tanque em forma de um bloco retangular de 3 m de
comprimento, 50 cm de largura e 0,36 m de altura,
um homem utiliza um balde cilndrico, de 30 cm de

36 cm
3m
50 cm

74

OBMEP 2010

Nvel 3

dimetro em sua base e 48 cm de altura, para pegar gua numa


fonte. Cada vez que ele vai fonte, ele enche 4/5 do balde e
no caminho derrama 10% do seu contedo. Estando o tanque
inicialmente vazio, quantas viagens fonte o homem ter de
fazer para que a gua no tanque chegue a 3/4 de sua altura?

48 cm

30 cm

32. Fator primo Qual o maior fator primo de 2 006?


33. Altura de salrio Entre 1986 e 1989, a moeda do nosso pas era o cruzado (Cz$).
De l para c, tivemos o cruzado novo, o cruzeiro, o cruzeiro novo e, hoje, temos o real.
Para comparar valores do tempo do cruzado e de hoje, os economistas calcularam que
1 real equivale a 2 750 000 000 cruzados. Imagine que a moeda no tivesse mudado e
que Joo, que ganha hoje 640 reais por ms, tivesse que receber seu salrio em notas
de 1 cruzado, somente. Se uma pilha de cem notas de 1 cruzado mede 1,5 cm de altura,
qual seria a altura (em quilmetros) do salrio do Joo?
(a) 26,4

(b) 264

(c) 26 400

(d) 264 000

(e) 2 640 000

34. S bala H 1 002 balas de banana e 1 002 balas de ma numa caixa. Lara tira, sem
olhar o sabor, duas balas da caixa. Se q a probabilidade de as duas balas serem de
sabores diferentes e p a probabilidade de as duas balas serem do mesmo sabor, qual
o valor de q p?
(a) 0

(b)

1
2 004

(c)

1
2 003

(d)

2
2 003

(e)

1
1 001

35. Distncia ao centro Um ponto P est no centro de um quadrado de 10 cm de


lado. Quantos pontos da borda do quadrado esto a uma distncia de 6 cm de P ?
(a) 1

(b) 2

(c) 4

(d) 6

(e) 8


36. Potncias e potncias Se 2 22x = 4x + 64, qual o valor de x?
(a) 2

(b) 1

(c) 1

(d) 2

(e) 3

37. Um raio de luz Dois espelhos formam um ngulo de 30 no ponto V. Um raio


de luz parte de um ponto S paralelamente a um dos espelhos e refletido pelo outro
espelho no ponto A, como mostra a figura.
S
Depois de uma certa quantidade de reflexes, o raio retorna
a S. Se AS e AV medem, ambos, 1 metro, qual o comprimento (em metros) do trajeto percorrido pelo raio de luz?

(a) 2
(b) 2 + 3
(c) 1 + 2 + 3
(d) 2(1 + 3)

OBMEP 2010

A
30

(e) 5 3
75

Nvel 3
38. Diferena de quadrados Determine o valor de (666 666 666)2 (333 333 333)2.
39. Escada de nmero Na figura, o nmero 8 foi obtido somandose os dois nmeros diretamente abaixo de sua casa. Fazendo-se
o mesmo para preencher as casas em branco, obtm-se o 42 na
casa indicada. Qual o valor de x?
(a) 7

(b) 3

(c) 5

(d) 4

(e) 6

42

8
3

40. Diferena de potncias Seja n = 9 867. Se voc calculasse n3 n2 , qual seria o


algarismo das unidades encontrado?
(a) 0

(b) 2

(c) 4

(d) 6

(e) 8

41. Parbola girada O grfico da parbola y = x2 5x + 9 rodado de 180 em torno


da origem. Qual a equao da nova parbola?
(a) y = x2 + 5x + 9

(c) y = x2 + 5x 9

(b) y = x2 5x 9

(d) y = x2 5x + 9

(e) y = x2 5x 9

42. Logotipo A figura mostra a marca de uma empresa, formada por


dois crculos concntricos e outros quatro crculos de mesmo raio, cada
um deles tangente a dois dos outros e aos dois crculos concntricos.
O raio do crculo menor mede 1 cm. Qual , em centmetros, o raio
do crculo maior?
43. Padeiro cansado Um padeiro quer gastar toda sua farinha para fazer pes. Trabalhando sozinho, ele conseguiria acabar com a farinha em 6 horas. Com um ajudante, o
mesmo poderia ser feito em 2 horas. O padeiro comeou a trabalhar sozinho e, depois
de algum tempo, cansado, ele chamou seu ajudante e assim, aps 150 minutos a farinha
acabou. Durante quantos minutos o padeiro trabalhou sozinho?
(a) 30

(b) 35

(c) 40

(d) 45

(e) 50

44. Muitas diagonais Calcule o nmero de diagonais de um prisma hexagonal reto,


como o da figura esquerda. Calcule o nmero de diagonais do poliedro obtido a
partir de um cubo pelo corte de seus oito vrtices, como o da figura direita. (Esse
poliedro muito utilizado na fabricao de dados, pois o corte prximo a cada um de
seus vrtices arredonda o dado e facilita a sua rolagem.)
*

*
*

*
*

*
*

76

OBMEP 2010

Nvel 3
45. Promoo de sabonete Uma loja de sabonetes realiza uma promoo com o anncio Compre um e leve outro pela metade do preo. Qual seria uma outra promoo
que a loja poderia fazer oferecendo o mesmo desconto percentual?
(a) Leve dois e pague um

(d) Leve trs e pague um

(b) Leve trs e pague dois

(e) Leve quatro e pague trs

(c) Leve cinco e pague quatro


46. Qual o ngulo? Na figura, os dois tringulos ABC e DEF so equilteros.
Qual o valor do ngulo x?
B

(a) 30

(b) 40

(c) 50
G

(d) 60

75

(e) 70

65
D

47. Caixa de papelo A figura mostra um pedao de papelo que ser dobrado e
colado ao longo das bordas para formar uma caixa retangular. Os ngulos nos cantos
do papelo so todos retos. Qual ser o volume, em cm3 , da caixa?
(a)

1 500

(b)

3 000

(c)

4 500

(d)

6 000

(e) 12 000
48. Soma de vizinhos Numa sequncia, cada termo, a partir do terceiro, a soma
dos dois termos imediatamente anteriores. Se o segundo termo 1 e o quinto termo
2005, qual o sexto termo?
(a) 3 002

(b) 3 008

(c) 3 010

(d) 4 002

(e) 5 004

49. Algarismos crescentes Quantos nmeros entre 10 e 13 000, quando lidos da esquerda para a direita, so formados por algarismos consecutivos e em ordem crescente?
Por exemplo, 456 um desses nmeros, mas 7 890 no .
(a) 10

(b) 13

(c) 18

(d) 22

(e) 25

50. Bloco girante Num bloco de 1 2 3 centmetros, marcamos trs faces com as
letras X, Y e Z, como na figura. O bloco colocado sobre um tabuleiro de 8 8
cm com a face X virada para baixo, em contato com o tabuleiro, conforme mostra a
figura. Giramos o bloco de 90 em torno de uma de suas arestas de modo que a face Y
fique virada para baixo (isto , totalmente em contato com o tabuleiro). Em seguida,
giramos novamente o bloco de 90 em torno de uma de suas arestas, mas desta vez de
modo que a face Z fique virada para baixo.
OBMEP 2010

77

Nvel 3
Giramos o bloco mais trs vezes de 90 em torno de
uma de suas arestas, fazendo com que as faces X, Y e Z
fiquem viradas para baixo, nessa ordem. Quantos quadradinhos diferentes do tabuleiro estiveram em contato
com o bloco?
(a) 18

(b) 19

(c) 20

(d) 21

(e) 22
x
f (x)

51. Iterando um ponto A funo f dada pela tabela

1
4

2
1

3
3

4
5

5
2

Por exemplo, f (2) = 1 e f (4) = 5. Quanto vale f (f (f (f (. . . f (4) . . .))))?


|
{z
}
2004 vezes
(a) 1

(b) 2

(c) 3

(d) 4

(e) 5

52. Esmeralda e o 21 Esmeralda escreveu em ordem crescente todos os nmeros de 1


a 999, sem separ-los, formando o nmero
12345678910111213 . . . 997998999.
Quantas vezes aparece o agrupamento 21, nessa ordem?

1 
1
1 
1 
1
1
1
?
53. Muitos fatores Qual o valor do produto 1
4
9
16
225
10
5
3
8
1
(a)
(b)
(c)
(d)
(e)
125
9
5
15
120


54. Falta um ngulo Quanto mede, em graus, o ngulo da


figura?
(a) 20

(b) 25

(c) 30

(d) 35

30

(e) 40
50

40

55. Soma de distncias Da figura, conclumos que |z x| + |w x| igual a

(a) 11

(b) 12

x y

1 0

3,7

9,3

(c) 13

(d) 14

(e) 15

56. Espiral do Artur Artur quer desenhar uma espiral de 4


metros de comprimento, formada de segmentos de reta. Ele
j traou sete segmentos, como mostra a figura. Quantos
segmentos ainda faltam traar?
(a) 28
78

(b) 30

(c) 24

(d) 32

(e) 36

OBMEP 2010

Nvel 3
57. Quais so os ngulos? A figura mostra um retngulo
e suas duas diagonais. Qual a afirmativa correta a respeito
dos ngulos x e y indicados na figura?
(a) x < y

(c) 2x = 3y

(b) x = y

(d) x = 2y

(e) x = 3y

.........................................................................................................................................................................
. .
.. ........ ..
....... ....
.......
...
.......
.......
....
.......
...
.......
......
.
.......
.
.
.
....
.
...
.
.......
.......
...
...
.......
.......
.
.
.......
.
...
.
.
.
.
...
.
....... .......... ...
...
...
................ ...
...
.
.
.
.
...
.
.........
...
...
.
.
.
.
.
...
.
.
.......
.....
..
.
.
.
.
.
.
.
.
.
...
.
.......
...
....
.
.
.
.
.
.
.
.
...
.
.
.
.......
....
.
.
.
.
.
.
.
..
.
...
.......
....
...... ....
... ............
.
..............................................................................................................................................................................

58. Raiz menor Qual a menor das razes da equao 2(x 3 5)(x 5 3) = 0?
59. Comparando reas Seja v a soma das reas das regies
pertencentes unicamente aos trs discos pequenos na figura
(em cinza claro) e seja w a rea da regio interior pertencente
unicamente ao maior disco (em cinza escuro). Os dimetros
dos crculos so 6, 4, 4 e 2. Qual das igualdades dadas
verdadeira?
(a) 3v = w
(c) v = w
(e) v = w
(b) 3v = 2w

(d) v = 3w

60. Menor raiz Qual a menor raiz da equao


(a)

1
3

(b)

1
2

(c)

1
3

(d)

1
4

|x 1|
= 6?
x2
3
(e)
2

61. Toalha redonda Uma mesa quadrada tem 1 metro de lado. Qual o menor dimetro
(em metros) de uma toalha redonda que cubra completamente o tampo da mesa?

(a) 1
(b) 1,5
(c) 2
(d) 2
(e) 3
62. Solues reais Qual o conjunto formado por todos os valores reais positivos de
x tais que (x 1)(x 2)(x 3) < 0?
(a) (1, 2)

(b) (1, 3)

(c) (0, 1) (2, 3)

(d) (0, 3)

(e) (0, 2)

63. Cossenos crescentes Num tringulo retngulo, definimos o cosseno de um ngulo


cateto adjacente
agudo por cos =
.
hipotenusa
B

O tringulo retngulo da figura tem cateto OA = 1.


Escreva, em ordem crescente, os cossenos dos ngulos
de 25 , 41 e 58 .

N
17
0

16
25
1

M
A

64. Central telefnica Os ramais de uma central telefnica tm apenas 2 algarismos,


de 00 a 99. Nem todos os ramais esto em uso. Trocando a ordem de dois algarismos
de um ramal em uso, ou se obtm o mesmo nmero ou um nmero de um ramal que
no est em uso. Qual o maior nmero possvel de ramais em uso?
(a) Menos do que 45

(c) Entre 45 e 55

(b) 45

(d) Mais do que 55


OBMEP 2010

(e) 55

79

Nvel 3
65. Horrio de avio Um nibus, um trem e um avio partem no mesmo horrio
da cidade A para a cidade B. Se eu tomar o nibus, cuja velocidade mdia de 100
km/h, chegarei cidade B s 20 horas. Se eu tomar o trem, cuja velocidade mdia de
300 km/h, chegarei cidade B s 14 horas. Qual ser o horrio de chegada do avio
se sua velocidade mdia for de 900 km/h?
66. Discos de papelo Para fabricar nove discos de papelo circulares para o Carnaval
usam-se folhas quadradas de 10 cm de lado, como indicado na figura. Qual a rea
(em cm2 ) do papel no aproveitado?
(a) 25
(b) 22,5
(c) 21,5
(d) 21
(e) 22
67. Afirmaes absolutas Determine quais so as afirmaes verdadeiras.
(a) | 108| > 100

(b) |2 9| = 9 2

(c) | 6a| = 6|a|

(d) |5 13| = |5| |13|

(e) |a2 + 5| = a2 + 5

x
x+y
?
68. Frao radical Se = 5, quanto
y
2y
(a)

5
2

(b) 3 2

(c) 13y

(d)

25y
2

(e) 13

69. rea de tringulo A figura mostra um retngulo


b e RGS
b
KGST e um tringulo KGR. Os ngulos K RT
so iguais. Se T R = 6 e RS = 2, qual a rea do tringulo
KGR?

(a) 12 (b) 16 (c) 8 2 (d) 8 3 (e) 14

70. Pares de inteiros Quantos so os pares diferentes de inteiros positivos (a, b) tais
1
a+
b = 13 ?
que a + b 100 e
1
+b
a
(a) 1

(b) 5

(c) 7

(d) 9

(e) 13

71. Qual a soma? Se x + |x| + y = 5 e x + |y| y = 6, qual o valor da soma x + y ?


(a) 1
80

(b) 11

(c)

9
5

(d) 1
OBMEP 2010

(e) 11

Nvel 3
72. Crculo intermedirio Na figura, os trs crculos so
concntricos, e a rea do menor crculo coincide com a rea
do maior anel, destacado em cinza. O raio do menor crculo
5 cm e do maior 13 cm. Qual o raio (em cm) do crculo
intermedirio?

(a) 12
(c) 10 65
(e) 12 2

(b) 11
(d) 5 3
73. Fraes incompletas Encontre os algarismos que esto faltando em cada um dos
espaos marcados com traos.
(a)

126
21
=
8

(b)

8
33

4
5

74. Tringulos impossveis Quais dessas figuras esto erradas?

75. Razo de reas Se um arco de 60 num crculo I tem o mesmo comprimento que
um arco de 45 num crculo II, encontre a razo entre a rea do crculo I e a rea do
crculo II.
(a)

16
9

(b)

9
16

(c)

4
3

3
4

(d)

(e)

6
9

76. Inequao errada Sendo x > 0, y > 0, x > y e z 6= 0, encontre a nica desigualdade
falsa.
(a) x + z > y + z
(b) x z > y z

(c) xz > yz
x
y
(d) 2 > 2
z
z

(e) xz 2 > yz 2

77. Equaes geomtricas Resolva as equaes dadas geometricamente, ou seja, interpretando o valor absoluto |a b| como a distncia entre a e b.
(a) |x 5| = 2

(b) |x + 3| = 1

(c) |3x 7| = 9

(d) |x + 2| = |x 5|

78. Pista circular A pista de um autdromo tem 20 km de comprimento e forma


circular, conforme figura.
OBMEP 2010

81

Nvel 3
Os pontos marcados na pista so A, que o ponto de partida; B, que dista 5 km de A no sentido do percurso; C,
que dista 3 km de B no sentido do percurso; D, que dista
4 km de C no sentido do percurso; e E, que dista 5 km
de D no sentido do percurso. Um carro que parte de A e
para aps percorrer 367 km estar mais prximo de qual
dos cinco pontos?
(a) A

(b) B

(c) C

(d) D

(e) E

79. Maior comprimento No diagrama dado, todos os quadradinhos tm 1 cm de lado. Qual dos segmentos dados o de maior
comprimento?
(a) AE
(b) CD + CF
(c) AC + CF
(d) F D
(e) AC + CE
80. Desigualdade entre inteiros Quantos dentre os nmeros 5, 4, 3, 2, 1, 0,
1, 2, 3 satisfazem a desigualdade 3x2 < 14?
(a) 1

(b) 2

(c) 3

(d) 4

(e) 5

81. Equao cbica Sobre a equao 2 007x3 + 2 006x2 + 2 005x = 0, o certo afirmar
que:
(a) no possui razes;

(d) tem apenas uma raiz real;

(b) tem trs razes reais distintas;

(e) tem trs razes positivas.

(c) tem duas razes iguais;


82. O perfume de Rosa Rosa ganhou um vidro de perfume com o formato de um
cilindro com 7 cm de raio da base e 10 cm de altura. Depois de duas semanas usando
o perfume, restaram 0,45 litros no vidro. Qual a frao que representa o volume que
Rosa j usou?
83. Igualdade com inteiros Quais nmeros naturais m e n satisfazem a equao
2 n + 1 = m2 ?
84. O caminho da pulga Para percorrer um caminho reto de 10 metros de comprimento, uma pulga usa a seguinte estratgia: a cada dia, ela percorre a metade do
caminho que falta. Assim, ela percorre 5 metros no primeiro dia, 2,5 metros no segundo, e assim por diante (o tamanho da pulga pode ser desconsiderado).
(a) Quantos metros ela ter percorrido ao final do stimo dia? E do dcimo?
(b) A partir de qual dia a pulga estar a menos de 0,001 m do final do caminho?
82

OBMEP 2010

Nvel 3
85. Uma soma alternada Se Sn = 1 2 + 3 4 + 5 6 + + (1)n+1 n para cada
inteiro positivo n, ento S1992 + S1993 igual a
(a) 2;

(b) 1;

(c) 0;

(d) 1;

(e) 2.

86. O raio da circunferncia Um arco de circunferncia mede 300 e o seu comprimento de 2 km. Qual o nmero inteiro mais prximo da medida do raio do crculo,
em metros?
(a) 157

(b) 284

(c) 382

(d) 628

(e) 764

87. Quatro passageiros Em um txi, um passageiro pode se sentar na frente e trs


passageiros atrs. De quantas maneiras podem se sentar quatro passageiros de um taxi
se um desses passageiros quiser ficar na janela?
88. Os cinco crculos Cinco discos de mesmo raio esto dispostos
como mostra a figura. Quatro centros so os vrtices de um quadrado e trs esto alinhados. Trace uma reta que divida a figura
formada pelos cinco discos em duas partes de mesma rea.

..........
..........
....... ........... ........... ...........
... ..
....
...
....
..
...
..
...
..
.
...
....
..
....
.. ......
..
.
.
.
.
.
.
...... ......
....... .......
.
.
.
.
.
.
.
.
.
.
.
.
.
.
.
.
.
.
.
.
.
.
.
.
.
.
.
.
.
.
.
.
.
.
.
.
.
.
.
.
.... ......... ........ ........ ........ ........
... ...
...
.....
... ...
.....
...
......
...
...
..
.
...
.
....
..
...
....
.
.
..
. ...
. ....
...
.
.
.
.
.
.
.
.
.
.....
........................ ........................... ............................

B.......................................................................................................C

89. O tringulo e o quadrado Na figura dada, ABCD um quadrado cujo lado mede 1 cm, E o ponto mdio da diagonal BD e
F o ponto mdio do segmento BE. Qual a rea do tringulo
CBF ?

...
... ....
......
..
.. ............................
...
...
....
....
...
...
....
...
...
....
...
...
....
...
...
....
....
...
...
....
.
...
.
.
.... ....
..
...
.... ...
........................................................................

E
r

90. Uma refeio Um sanduche e um prato de refeio custam R$ 5,00 e


R$ 7,00, respectivamente. De quantas maneiras pode-se comprar s sanduiches, s
pratos de refeio ou alguma combinao de sanduiches e pratos de refeio com
R$ 90,00, sem deixar troco?
y

91. Plano cartesiano O ponto P = (a, b) est marcado


na figura ao lado. Marque os pontos:

Pr

(a) A = (a + 1, b/2);

(b) B = (a/2, b 1);

(c) C = (a, b);

(d) D = (1 a, b 2).

2
3

92. Soma dos terminados em 9 A soma Sn = 9 + 19 + 29 + 39 + + an denota a


soma dos primeiros n nmeros naturais terminados em 9. Qual o menor valor de n
para que Sn seja maior do que 10 5 ?
93. Trs cilindros Trs cilindros de volumes V1 , V2 e V3 tm alturas e raios das bases
iguais a 10 e 10 cm, 10 e 5 cm e 20 e 5 cm.
OBMEP 2010

83

Nvel 3

...........
................................................................................
..
....
...
...
...
...
...
...
...
...
...
......................................................................
......................

..............................................
..
..
...
...
...
...
...
...
...
...
...
...
.....................................
...........

................................................
...
...
..
....
...
...
..
...
...
...
...
...
...
...
...
...
...
...
...
...
...
...
...
...
...
...
.
.............................
....................

(a) Escreva em ordem crescente os volumes V1 , V2 e V3 dos trs cilindros.


(b) D as dimenses de um cilindro cujo volume V4 esteja entre V2 e V3 .
(c) D as dimenses de um cilindro cujo volume V5 esteja entre V1 e V3 .
94. Porcentagem de mortalidade Se 15% dos membros de uma populao foram
afetados por uma doena e 8% dos afetados morreram, a porcentagem da mortalidade
em relao populao inteira foi de:
(a) 1,2% ;

(b) 1,8% ;

(c) 8% ;

(d) 12% ;

(e) 23% .

95. Agenda de aulas Eliane quer escolher o seu horrio para a natao. Ela quer ir
a duas aulas por semana, uma de manh e outra de tarde, no sendo no mesmo dia,
nem em dias seguidos. De manh, h aulas de natao de segunda-feira a sbado, s
9h, s 10h e s 11h e de tarde, de segunda a sexta-feira, s 17h e s 18h. De quantas
maneiras distintas pode Eliane escolher o seu horrio?
96. Jogo de cartas Um grupo de amigos disputa um jogo no qual 16 cartas (sendo
quatro ases, quatro reis, quatro damas e quatro valetes) esto inicialmente dispostas
em quatro pilhas de quatro cartas. O jogo consiste em mover sucessivamente a carta
superior de uma pilha e coloc-la sobre uma outra pilha, at obter quatro novas pilhas,
em que na primeira pilha s tenha ases, na segunda, s valetes, na terceira s damas
e na quarta pilha s reis. Ganha o jogo quem fizer o menor nmero de movimentos.
Com quantos movimentos sempre possvel terminar o jogo? Na figura dada, aparece
a disposio inicial das cartas nas pilhas.
pilha 1

pilha 2

pilha 3

pilha 4

rei de

dama de

rei de 

valete de

valete de 

s de

dama de 
s de

s de 

valete de

valete de
dama de
dama de

rei de
s de

rei de

97. Fraes inteiras Quantos nmeros inteiros positivos n existem tais que o quociente
2n2 + 4n + 18
seja um inteiro?
3n + 3
98. Quatro prefeitos e um crculo Quatro prefeitos decidem construir uma rodovia
circular que passe dentro dos limites de suas cidades. Como as quatro cidades no esto
sobre um mesmo crculo, os prefeitos contratam uma empresa para elaborar um projeto
84

OBMEP 2010

Nvel 3
para a construo de uma rodovia circular equidistante das quatro cidades. Qual o
maior nmero de projetos geograficamente distintos que a empresa pode elaborar?
99. Fatoriais Se n um nmero inteiro positivo, denotamos por n! o produto de
todos os inteiros de 1 a n. Por exemplo, 5! = 1 2 3 4 5 = 120 e 13! =
1 2 3 4 5 12 13. Por conveno, escrevemos 0! = 1! = 1. Encontre trs
nmeros inteiros a, b e c entre 0 e 9, que sejam distintos e tais que o nmero de trs
algarismos a b c seja igual a a! + b! + c!.
100. O Riquinho Riquinho distribuiu 1 000,00 reais entre os seus amigos Antnio, Bernardo e Carlos da seguinte maneira: deu, sucessivamente, 1 real ao Antnio, 2 reais
ao Bernardo, 3 reais ao Carlos, 4 reais ao Antnio, 5 reais ao Bernardo etc. Qual foi a
quantia recebida por Bernardo?
101. Retngulo
com dimenses inteiras As diagonais de um retngulo medem

1 993 cm. Quais so as dimenses do retngulo, sabendo que elas so nmeros inteiros?
102. Mltiplos de 3 e quadrados perfeitos Escreve-se em ordem crescente os mltiplos
de 3 que, somados com 1, sejam quadrados perfeitos, ou seja, 3, 15, 24, 48, . . . . Qual
o mltiplo de 3 na 2 006a posio?
103. Cinco cartas Cinco cartas esto sobre uma mesa, e cada uma tem um nmero numa
face e uma letra na outra. Simone deve decidir se a seguinte frase verdadeira: Se
uma carta tem uma vogal numa face, ento ela tem um nmero par na outra. Qual
o menor nmero de cartas que ela precisa virar para tomar uma deciso correta?
...................
... ...
... 2 ...
...................

...................
... ...
... 3 ...
...................

...................
... ...
...M...
...................

...................
... ...
... A...
...................

...................
... ...
... E ...
...................

104. O lucro de uma companhia Uma companhia tem um lucro de 6% nos primeiros
R$ 1 000,00 reais de venda diria e de 5% em todas as vendas que excedam
R$ 1 000,00 reais, nesse mesmo dia. Qual o lucro dessa companhia, em reais, num
dia em que as vendas alcanam R$ 6 000,00 reais?
(a) 250

(b) 300

(c) 310

(d) 320

(e) 360

105. Sequncia triangular Encontre o 21o termo da sequncia que comea assim:
1; 2 + 3; 4 + 5 + 6; 7 + 8 + 9 + 10; 11 + 12 + 13 + 14 + 15; . . .
OBMEP 2010

85

Nvel 3
106. O jardim octogonal A figura mostra a planta de um
jardim de uma cidade, feita num papel quadriculado. O
jardim tem a forma de um polgono de oito lados com
uma roseira quadrada no centro, cercada de grama. A
rea total do jardim de 700 m2 . Para colocar uma cerca
em volta do jardim e da roseira, o prefeito dispe de, no
mximo, R$ 650,00.
Qual o maior preo que o prefeito poder pagar pelo
metro dessa cerca?

..............................
...
.....
.
.
.
.....
...
.
.
.....
......
..
.....
.... .......
.
.
..
..
.
....
.
....
....
....
....
.
..
..
.
.... roseira....
....
.
....
....
..
..
....
...
.
.
.
.... ...
....
.....
.
.
.
.
.....
..
.
.
.....
..
.....
...
.
.
.
.
..........................

107. Nmero de caracteres Numa folha de papel cabem 100 caracteres na largura e
100 na altura. Nessa folha so escritos sucessivamente os nmeros 1, 2, 3, e assim por
diante, com um espao entre cada um e o seguinte. Se no final de uma linha no houver
espao para escrever o nmero seguinte, ele escrito no comeo da linha seguinte. Qual
o ltimo nmero escrito na folha?
108. A rvore de Emlia A rvore de Emlia cresce de acordo com
a seguinte regra: aps duas semanas do aparecimento de um galho,
esse galho produz um novo galho a cada semana e o galho original
continua crescendo. Depois de cinco semanas, a rvore tem cinco
galhos, como mostra a figura. Quantos galhos, incluindo o galho
principal, a rvore ter no final de oito semanas?

q q q

.
..
..
.
..
..
...
...
..
..
..
..
...
...
..
..
..
.. ...
... .......
..
.
.
.
..
.
.
.
.
..
............
..... ....
..
...
.. ...
...
...
..
...
...
..
.
...
.
..
.
...
..
...
...
..
..... .............
..
........ ..
..
.
..
....
.
.
...
...
.......
...
... .................
...
....
...
...
...
...
.

q
q

109. Um teste vocacional Foi aplicado um teste vocacional em 1 000 alunos de uma
escola. A tabela a seguir apresenta os resultados, por rea de estudo e sexo.
Exatas

Humanas

Biolgicas

Masculino

232

116

207

Feminino

112

153

180

Se um aluno for escolhido ao acaso, determine a probabilidade desse aluno ser:


(a) da rea de exatas;
(b) da rea de humanas, sendo do sexo masculino;
(c) do sexo feminino, sendo da rea de biolgicas.

110. Dois setores circulares A rea do crculo da figura mede


b = 60 e C OD
b = 30 , quanto mede a rea da
20 cm2 . Se AOB
regio do crculo que est destacada?

86

OBMEP 2010

As

sB

.........................................
.........
........
........
.
... .........
...... ...
.... ....
..
....
.
.
...
..
...
.
...
.
.
...
...
.
.
..
.
.
.
...
...
.
..
.
.
.
...
.
...
.
.
..
.
....
...
..
... ....
...
..
... ..
.
...
..................................................................
..
......
.
......
..
..
......
..
.
.
......
...
..
......
...
...... ....
...
.....
...
...
....
...
.....
....
.....
....
.
.
......
.
.
.
..........
.....
...........................................

sC

sD

Nvel 3
111. Compra de televisores Maria encomendou um certo nmero de televisores para
o estoque de uma grande loja, pagando R$ 1 994,00 por televisor. Ela reparou que,
no total a pagar, no aparece o algarismo 0, nem o 7, nem o 8 e nem o 9. Qual foi o
menor nmero de televisores que ela pode ter encomendado?
112. Distncia entre nmeros Considere os nmeros reais a, b, c e d representados em
uma reta, conforme mostra a figura. Determine quais das afirmaes so verdadeiras
e quais so falsas.
a
4

d
0

(a) |a| < 4

(d) |a| > |b|

(g) |a b| < 4

(j)

(b) |b| < 2

(e) |c| < |d|

(h) |a b| 3

(k) |b c| > 3

(c) |c| < 2

(f)

(i)

(l)

|a| < |d|

|c d| < 1

|b c| < 2
|c a| > 1

113. Cartes premiados Uma loja distribui 9 999 cartes entre os seus clientes. Cada
um dos cartes possui um nmero de quatro algarismos, entre 0001 e 9999. Um carto
premiado se a soma dos primeiros dois algarismos for igual soma dos dois ltimos;
por exemplo, o carto 0743 premiado. Prove que a soma dos nmeros de todos os
cartes premiados divisvel por 101.
114. O preo da gasolina Encher o tanque de gasolina de um carro pequeno custava,
em valores atualizados, R$ 29,90 em 1972 e R$ 149,70 em 1992. Qual dos valores
abaixo melhor aproxima o percentual de aumento do preo da gasolina nesse perodo
de 20 anos?
(a) 20%

(b) 125%

(c) 300%

(d) 400%

(e) 500%

115. O tringulo de moedas Um menino tentou alinhar 480 moedas em forma de um


tringulo, com uma moeda na primeira linha, duas moedas na segunda linha, e assim
por diante. Ao final da tentativa, sobraram 15 moedas. Quantas linhas tem esse
tringulo?
116. Circunferncia e tringulo retngulo Inscreve-se uma circunferncia num tringulo retngulo. O ponto de tangncia divide a hipotenusa em dois segmentos que
medem 6 e 7 cm. Calcule a rea desse tringulo.
1 1
1
117. Soma de razo 12 Se Sn = + 2 + + n , qual o menor nmero inteiro positivo
2 2
2
n tal que Sn > 0,99?
118. Soma de razes quadradas

r2 5
3, mostre que 6 =
.
2

(b) Se s = 215 + 300, mostre que s2 > 1 015.


(a) Se r =

2+

OBMEP 2010

87

Nvel 3
119. Duas rodas Na figura dada, a roda A gira a 1 200 voltas por
minuto e a roda B a 1 500 voltas por minuto. Calcule os raios dessas
duas rodas.

..............
.........
..... .............................
.... .....
.... ......................................
........
......
... ....
.....
........
......
......
...
... ...
.
..
....
........
....
.
.......
. ....
.......
.
.
.
.
.
.... .....
......... ....... ..
..
.
.
.
.
.
.
.
.
.
.
.
.
.. ..........
..
.
.....................
...
...
..
..

9 cm

120. Dois divisores O nmero 248 1 divisvel por dois nmeros compreendidos entre
60 e 70. Quais so esses nmeros?
(a) 61 e 63

(b) 61 e 65

(c) 63 e 65

(d) 63 e 67

(e) 67 e 69

121. Rede de estaes Um servio de vigilncia vai ser instalado num parque na forma
de uma rede de estaes. As estaes devem ser conectadas por linhas de telefone, de
modo que qualquer uma das estaes possa se comunicar com todas as outras, seja por
uma conexo direta, seja por meio de, no mximo, uma outra estao.
Cada estao pode ser conectada diretamente por um cabo a, no mximo, trs outras estaes. O diagrama mostra um exemplo de uma
rede desse tipo, conectando sete estaes. Qual o maior nmero de
estaes que podem ser conectadas dessa maneira?

................
....
.
..
..
................
.
.
.
.
.
.
.
..
.....
.
.
.
.
.
.
.
.
.
..
........ ..
.. ....
... ........
..
...... .........
...... ........ .... ....... ........
....................
.....................
....
.
.................
.................
.......... .......
...
.............................. ......
.
.
.
.
.
.
.
.
.
.
.
.
.
.
.
....
.
.
.
.. ..
....... .
. ...
... ............. ...............
...... .........
.
...
....... ........
....
....................
....................
.................
.................
.................

122. Bolas brancas e pretas Uma caixa tem exatamente cem bolas pretas e cem bolas
brancas. Repetidamente, trs bolas so retiradas da caixa e substitudas por outras
bolas, que esto em um saco, da maneira seguinte.
BOLINHAS REMOVIDAS
3 pretas

SUBSTITUDAS POR
= 1 preta

2 pretas e 1 branca

= 1 preta e 1 branca

1 preta e 2 brancas

= 2 brancas

3 brancas

= 1 preta e 1 branca

Qual pode ser o contedo da caixa depois de seguidas aplicaes desse procedimento?
(a) 2 pretas

(b) 2 brancas

(c) 1 preta

(d) 1 preta e 1 branca

(e) 1 branca.

123. O cubo Alice tem uma folha de cartolina de 60 por 25 cm. Ela quer cortar a
folha para montar um cubo, com arestas medindo um nmero inteiro de centmetros.
Permitindo cortes mas no permitindo superposio, qual o cubo de maior volume
que ela pode construir?
124. Um quadrado e um tringulo Na figura, ABCD
um quadrado, cuja rea mede 7/32 da rea do tringulo
XY Z. Qual a razo entre XA e XY ?

X
.
...........
........ .....
.........
A
........................................................B
.
.
.
.
.
.
.
.
..
...... .

....
... .....
...
.........
... ...
...
........
.........
... ....
...
........
.
.
.
.
.
...
.
.
.
...
.
.
......
.
.
...
.
.
.
.
.
..
.
.
....
.................................................................................................................................................................

125. A urna Uma urna tem seis bolas numeradas de 1 a 6. Se duas bolas so extradas,
qual a probabilidade de a diferena entre os nmeros dessas duas bolas ser igual a 1?
88

OBMEP 2010

Nvel 3
126. Soma das razes de uma equao Determine a soma das razes distintas da
equao x2 + 3x + 2 = |x + 1|.
127. Produto de trs nmeros No diagrama dado, cada um dos 10 crculos representa
um algarismo. Preencha o diagrama com uma igualdade vlida, colocando, em cada
crculo, um dos algarismos de 0 a 9 e utilizando cada algarismo uma nica vez.

....
....... .........
...
...
....
.
.... .....
..............

.............
.............
.... .... .... ....

.............
..............
.............
.... .... .... .... .... ....

..............
.............
............
.................
.... .... .... .... .... .... ....

................................ ................................ .................................. ................................ ................................. = .................................. ................................. ............................... ..................................


.

128. rea do tringulo Determine a rea do tringulo


ABC mostrado na figura.

A..........= (1, 2)

.. ........
........
...
........
...
........
..
.......
...
.......
........
..
........
...
......
...
.....
...
....
.
.
...
.
.
....
..
.
.
.
...
....
...
.....
.....
...
.....
...
.....
.
.
.
.
...
..
... .........
.........

B = (8,
0)

C = (1, 6)

129. Duas tabelas As linhas da primeira tabela dada so todas progresses aritmticas
de uma mesma razo e as colunas dessa tabela so todas progresses aritmticas de
uma mesma razo. Na segunda tabela dada foi utilizada a mesma lei de formao, mas
algum apagou alguns nmeros deixando apenas trs. Qual o nmero que estava na
posio indicada com ?
5

11

14

17

12

15

18

21

24

19

22

25

28

31

26

29

32

35

38

33

36

39

42

45

39
87
56

130. A sequncia abc A lei de formao da sequncia 10, a, 30, b, c, . . . , a partir de seu
terceiro termo, consiste em tomar o dobro da soma dos dois termos imediatamente
anteriores. Qual o valor de c?
131. Permetro e diagonal O permetro de um retngulo ABCD mede 20 m. O menor
comprimento que pode ter a diagonal AC, em metros, :

(a) 0;
(b) 50;
(c) 10;
(d) 200;
(e) 20 5.
132. As idades numa classe Numa classe na escola, todos os alunos tm a mesma
idade, exceto sete deles que tm 1 ano a menos e dois deles que tm 2 anos a mais.
A soma das idades de todos os alunos dessa classe 330. Quantos alunos tem nessa
classe?
OBMEP 2010

89

Nvel 3
133. A mesa redonda Uma mesa redonda tem 1,40 metros de dimetro.
Para uma festa, a mesa ampliada colocando-se trs
tbuas de 40 cm de largura cada uma, como mostra
a figura. Se cada pessoa mesa deve dispor de um
espao de 60 cm, quantos convidados podero se sentar
mesa?

.........................
........
.....
.....
....
....
...
..
...
.
..
....
.
....................................................................
..
..
..
.
.
...
.
.
.
...
..
....
....
......
..................................

mesa fechada

...............................
......
....
....
....
....
...
..
..
.
..
...
.
....................................................................
...
...
....................................................................
..
..
......................................................................
..
.
...................................................................
..
..
..
.
...
..
.
.
....
...
....
......
....
................................

mesa ampliada

134. Brincadeira com sete nmeros Sete nmeros inteiros positivos e consecutivos
esto escritos em ordem crescente numa mesma linha. Determine se possvel colocar
entre esses nmeros cinco sinais de + e s um de = de tal modo que resulte uma
igualdade.

135. Um terreno compartilhado Trs amigas compraram um


terreno quadrado e querem reparti-lo em trs terrenos de mesma
rea, conforme indicado na figura, pois no canto do terreno indicado por A se encontra uma boa fonte de gua. A que distncia
do vrtice C do terreno devem ficar os pontos de divisa M e N
indicados na figura?

B ......................................................................................................... A
M
C

.... .. ..
....
...... .. ..
...
...... .... ....
.
..
......
..
..
.....
.
.
.
.
...
....
.
.
......
..
..
...
..
......
.
.
.
.
...
.
.
...
.
..
.
.
... .........
.
...
...
...
.........
.
..
.
....
....
.
.
.
..
..
...
...
...
.
..
.
...
...
.........................................................................................

136. As duas partculas Duas partculas percorrem um caminho circular de 120 m


de comprimento. A velocidade de uma delas 2 m/s maior do que a da outra e ela
completa cada volta num tempo que 3 segundos inferior ao da outra. Qual a
velocidade de cada partcula?

137. Queda livre Um corpo em queda livre demora onze segundos para tocar o solo. No
primeiro segundo ele percorre 4,9 m e, em cada segundo seguinte, a distncia percorrida
aumenta em 9,8 m. Qual a altura da queda e quantos metros ele percorreu no ltimo
segundo?

138. Um caminho triangular Janete passeia por um caminho de forma triangular


ABC, com o lado AB medindo 1 992 m. Ela gasta 24 minutos para percorrer esse
lado AB e, depois, com a mesma velocidade, ela percorre o outro lado BC seguido da
hipotenusa CA em 2 horas e 46 minutos. Qual o comprimento do lado BC?

139. O preo do feijo A tabela e o grfico dados mostram a evoluo do preo mdio
de trs tipos A, B e C de feijo na bolsa de alimentos durante os primeiros quatro
meses de um certo ano. Desses trs tipos, os que apresentaram, respectivamente, o
maior e o menor aumento percentual do preo nesse perodo so:
(a) A e B;
90

(b) A e C;

(c) B e C;
OBMEP 2010

(d) C e A;

(e) C e B.

Nvel 3
R$

110

100

90

80

70

60

...
...
...
....
...
.
.
.... ..........
.
...
.... ....... ..
... ...... ......
..........
..
.
....
.......
.
.
.
....
.......
...
.
.......
.
.
.
.
....
....
...... ....
......
.
..
........
.
.
.
.
.
.
......
...
... ... ....
.
........
..
......
.
.
.
.
.
.
.
.
..... ..
...
..... ..
...
..... ....
.
...
.. .........
.
... ......
.. .......
.. ....
......

jan.

fev.

jan.

fev.

mar.

abr.

65,67

83,33

96,67

103,33

73,30

80,50

99,55

109,50

64,50

71,57

89,55

100,00

t: A

mar. abr.

: B

: C

140. Interseo de tringulos Os trs tringulos da figura se cortam em 12 pontos


diferentes. Qual o nmero mximo de pontos de interseo de trs tringulos quaisquer?
....
.... ...
.... ....
....
...
.
.
...
....
...
...
...
....
.
.
.
...
..
.
.
.
...
...........
.
.
.
.
.
.
.
.
.
.
..................................................
.
.
.
.
.
.
.
.
...
.
.
.
.
....
.................................................... .....
..
.
...
.
.
....
.
.............. ...............................................
.
.
.
.
.
.
.
.
.
.
.
.
...
.
.
.
.
.
.
.
.
.
.
.
............................
..
.........................................
.
.
.
.
.
.
.
.
.
.
.
.
.
.
.
.
.
.
.
.
.
.
.
.
.... ......................................
..
..........
.
.
.
.
.
.
.
.
.
.
.
.
.
.
.
.
.
.
.
.
.
.
.
.
.
.
.. .......................
....
..................... ..
..
.
.
..................
.
.
... .............................................
..
....
...
.. ..............................
..............................
.
.
.
......................
.
.
.
.................
.
.
.
..
.
.
.
...
.
.
.
.
.
.
.
.
.
.
.
.
.... ........
.................
...
............... ......................................... .................................................... .......
...
.
.
.
.
.
.
.
.
.
.
.
.
.
.
.
.
.
.
.
.
.
.
.
.
...
.
.
.
.
.
.
.
.
.
.
.
.
.
.
.
.
.
.
.
.
.
.
.
.
.
.
.
.
.
.
.
.
.
.
.
..................
.... ..........
...............
.
.
.
.
.
.
.
.
.
.
.
.
.
.
..
.
.
.
.
.
.
.
.
.
.
.
.
.
.
.
.
.
.................
..
....
.................
.. ...........................................
.
.
....
.................
..............
.
.................
.................. .......
................. ...
.........

141. Comparar tringulos Na figura esto indicados


os comprimentos de todos os segmentos. Demonstre
b
que AC divide ao meio o ngulo D AB.

s
s
s

Br

rC

.............................................
......................
....
.....
....
........ .........
....
.
........
.....
.
.
.........
.....
..
.
.
.
.
.
.
.
.
.
.
.
.....
.....
..
.
.
.
.
.
.....
.
.
.
.
.
.
.....
.....
.
.
...
.
.
.
.
.
.
.
.
.....
..
......
.
.
.
.
.....
.
.
.
.
.
.
.....
..
......
.
.
.
.
.
.
.
.
.
.
.....
.. .............
.
.....
.
.
.....
.. ............
.
.
.
.
....
...........
.
.
.
.
.
............................................................................................................................................................................................................

12

18

27

12

142. Queima de velas Dois tipos de vela tm o mesmo comprimento mas so feitas
de material diferente. Uma delas queima completamente em trs horas e a outra em
quatro horas, ambas queimando com velocidade uniforme. Quantos minutos depois
das 13 horas devem ser acesas simultaneamente as duas velas para que, s 16 horas, o
comprimento de uma seja o dobro do da outra?
(a) 24

(b) 28

(c) 36

(d) 40

(e) 48

143. Uma distrao Em vez de multiplicar certo nmero por 6, Jlia se distraiu e
dividiu o nmero por 6. O erro cometido por Jlia foi de aproximadamente:
(a) 100% ;

(b) 97% ;

(c) 83% ;
OBMEP 2010

(d) 17% ;

(e) 3% .
91

Nvel 3
144. Problema de nota Um professor prope 80 problemas a um aluno, informando que
lhe atribuir cinco pontos por problema resolvido corretamente e lhe descontar trs
pontos por problema no resolvido ou resolvido incorretamente. No final, o aluno fica
com oito pontos. Quantos problemas ele resolveu corretamente?
145. Quadrados e tringulos Na figura dada, temos 16 pontos formando um reticulado
quadrado e duas retas, r e s, perpendiculares entre si.

(a) Quantos quadrados podemos construir, de tal maneira que seus vrtices pertenam
ao reticulado, porm nenhum de seus lados seja paralelo, nem reta r, nem reta
s?
(b) Quantos tringulos retngulos issceles podemos construir de tal maneira que seus
vrtices pertenam ao reticulado, porm nenhum de seus lados seja paralelo, nem
reta r, nem reta s?
146. Clculo de reas Em cada uma das figuras a seguir tem-se um quadrado de lado
r. As regies hachuradas em cada uma destas figuras so limitadas por lados desse
quadrado ou por arcos de crculos de raio r de centros nos vrtices do quadrado.
Calcule cada uma dessas reas em funo de r.

(b)

(a)

147. Sequncia de algarismos Todos os nmeros naturais de 1 em diante foram escritos


consecutivamente, formando uma sequncia de algarismos, como segue.
1234567891011121314151617181920212223 . . ..
Qual o algarismo que aparece na posio de nmero 206 788?
148. Soma constante Coloque os nmeros 663, 664, 665, 666, 667, 668, 669, 670 e
671, sem repetir, numa tabela 3 3, de tal maneira que a soma em cada linha, em
cada coluna e em cada diagonal seja 2 001. Caso isso no seja possvel, justifique sua
resposta.
92

OBMEP 2010

Nvel 3
149. Contando os zeros Quantos zeros existem no final do nmero 92007 + 1?
150. Crculos dentro do quadrado Dentro de um quadrado so colocados crculos, dois
a dois disjuntos ou, ento, tangentes externamente. Se o lado do quadrado mede 1 cm,
ser possvel colocar tantos desses crculos de tal modo que a soma de seus raios, em
centmetros, seja maior do que 2 008?
151. Construindo um nmero Encontre todos os nmeros de oito algarismos formados
somente com os algarismos 1, 2, 3 e 4, cada um deles duas vezes, tais que:
(a) exista um nico algarismo entre os dois algarismos 1;
(b) existam dois algarismos entre os dois algarismos 2;
(c) existam trs algarismos entre os dois algarismos 3 e
(d) existam quatro algarismos entre os dois algarismos 4.
152. Nmero na circunferncia Os algarismos 1, 2, 3, 4, 5, 6, 7, 8 e 9 foram escritos
(numa ordem desconhecida) ao redor de uma circunferncia. Lendo esses algarismos de
trs em trs no sentido horrio, formam-se nove nmeros de trs algarismos. Determine
a soma desses nove nmeros.
153. Cada pea em seu lugar Cinco peas de metal, confeccionadas, respectivamente,
de ouro, prata, bronze, platina e nquel, foram colocadas em cinco cofres, numerados
de 1 a 5. Cada cofre contm uma pea e o problema consiste em descobrir qual pea
est em qual cofre. Na porta de cada cofre est escrita uma informao. Das cinco
informaes, quatro so falsas e a nica que verdadeira a que aparece na porta do
cofre que contm a pea de ouro. As informaes nas portas dos cofres so as seguintes.
Cofre 1: O ouro est no cofre 2 ou 3.
Cofre 2: A prata est no cofre 1.
Cofre 3: O bronze no est aqui.
Cofre 4: O nquel est no cofre cujo nmero inferior, em uma unidade, ao que
contm o ouro.
Cofre 5: A platina est no cofre cujo nmero superior, em uma unidade, ao que
contm o bronze.
154. Soma de quadrados Encontre trs nmeros, numa progresso aritmtica de razo
2, tais que a soma de seus quadrados seja um nmero formado de quatro algarismos
iguais.
155. Adivinhe o nmero Certo nmero deixa resto 1 quando dividido por 3, deixa resto
2 quando dividido por 4, deixa resto 3 quando dividido por 5 e deixa resto 4 quando
dividido por 6. Qual o menor nmero inteiro positivo que satisfaz essas propriedades?
156. Um cdigo Na expresso abaixo, cada letra corresponde a um algarismo, sendo que
letras diferentes correspondem a algarismos diferentes. Determine esses algarismos.
6 AOBMEP = 7 MEP AOB
OBMEP 2010

93

Nvel 3
157. Calculando distncias O tringulo ABC
equiltero, com lados medindo 3 cm, e o tringulo
CBD retngulo, com lados medindo 3, 4 e 5
cm, conforme a figura dada. Calcule a distncia
entre os pontos A e D.

3
A

3
3

158. Calculando lados de um tringulo O tringulo


ABC equiltero e o ponto P tal que P A = 3 cm,
P B = 4 cm e P C = 5 cm. Calcule o comprimento dos
lados do tringulo ABC.

5 cm

3 cm

4 cm

159. Amigo oculto Um grupo de cinco amigos decide brincar de amigo oculto, cada um
compra um presente para seu amigo oculto. Pelas regras do jogo, cada um d exatamente um presente e recebe exatamente um presente. De quantas maneiras podem os
presentes ser distribudos, de modo que ningum d presente para si mesmo?
160. Contando solues Quantos so os pares de nmeros inteiros positivos (x, y) tais
que
xy
= 144 ?
x+y
161. Determinando uma sequncia Numa certa sequncia de 80 nmeros, qualquer
termo, salvo as duas extremidades, igual ao produto de seus termos vizinhos. O
produto dos 40 primeiros termos da sequncia 8 e o produto de todos os termos
tambm 8. Determine os termos da sequncia.

............................................................................................................
..
...
...

....
...

...
...
...
jardim
...

.
...
...
...

..
...
..

...........................................................................................................

162. Construindo uma cerca Carina est desenhando a planta


de um jardim retangular que ter um de seus lados num muro
reto de pedras. Ela comprou 140 m de cerca, em pedaos de 1 m
cada um, para cercar os outros trs lados. Ela no pode cortar
esses pedaos e deve gastar todos eles.

(a) Se os dois lados vizinhos ao muro de pedra tm 40 m cada um, qual ser o
comprimento do terceiro lado?
(b) possvel que o maior dos lados a ser cercado tenha 85 m? E 65 m? Justifique.
b e C DA
b do
163. Um quadriltero especial Os ngulos ABC
quadriltero ABCD da figura so retos e os quatro lados do
quadriltero medem nmeros inteiros que so todos distintos.
Se AD = 7 e BC = 11, quanto medem os lados AB e CD?

OBMEP 2010

11

7
y
D

94

Nvel 3
164. Trs quadrados Dois quadrados, ABCD com uma rea de 30 cm2 e F HIJ com
uma rea de 20 cm2 , tm seus lados AD e HI sobre uma reta, conforme a figura. Se
o ponto E do segmento AH for tal que BEF G um quadrado, calcule a rea desse
quadrado.
G
C

165. Bolinha de gude Trs amigos jogam uma partida de bolinha de gude, convencionando que o perdedor de cada rodada dobra as bolinhas dos outros jogadores, ou seja,
ele d aos outros dois um nmero tal de bolinhas que eles fiquem com o dobro do que
tinham no incio da rodada. O primeiro jogador perdeu a primeira rodada, o segundo
jogador a segunda, o terceiro a terceira e todos terminaram com 64 bolinhas cada um.
Com quantas bolinhas cada um dos trs amigos comeou essa partida?
166. Uma soma Calcule o valor da soma
1
1
1
1
1
S=
+
+
++
+
.
12 23 34
2 006 2 007 2 007 2 008
167. Dobrando papel Uma folha ABCD retangular com 1 000 cm2 de rea foi dobrada
ao meio e, em seguida, desdobrada segundo MN, conforme a figura. Em seguida, foi
dobrada e desdobrada novamente, segundo MC e, finalmente, dobrada e desdobrada
segundo a diagonal BD. Calcule a rea do pedao de papel indicado na figura, que
limitado pelos trs vincos.
A

B
F

168. Uma rea No tringulo ABC, M o ponto mdio do lado AC, D um ponto do
b e P o ponto de interseo de AD
lado BC, tal que AD a bissetriz do ngulo B AC,
e BM. Sabendo que AB = 10 cm, AC = 30 cm e a rea do tringulo ABC mede
100 cm2 , calcule a rea do tringulo ABP.
169. ltimos algarismos Quais so os dois ltimos algarismos do nmero
2 008

z }| {
8 + 88 + 888 + + 88 88 ?
OBMEP 2010

95

Nvel 3
170. Idades mltiplas Quando Isabel nasceu, sua me estava fazendo aniversrio de 20
anos. Se Isabel e sua me viverem mais 100 anos, quantas vezes tero sido mltiplas
as idades das duas?
171. Blocos diferentes Ana tem um cubo de 10 cm de lado. Ela cortou o cubo em
cubinhos de 1 cm de lado e, com esses cubinhos, ela brinca de formar outros blocos
retangulares, mas sem que sobrem cubinhos. Por exemplo, ela formou um bloco de
10 20 5. No total, quantos blocos diferentes ela pode construir com esses cubinhos,
sem que sobre nenhum?
5 cm
10 cm
20 cm

172. Quadro negro Joana escreveu os nmeros de 1 a 10 000 no quadro negro e, depois,
apagou todos os mltiplos de 7 e 11. Qual foi o nmero que ficou na posio 2 008?
173. Conjunto sem mltiplos Qual o maior nmero possvel de elementos de um
subconjunto de {1, 2, . . . , 100} tal que nenhum de seus elementos seja um mltiplo de
algum outro?
174. Brincando com a calculadora Digite numa calculadora um nmero qualquer de
trs algarismos. Em seguida, digite o mesmo nmero obtendo, assim, um nmero de
seis algarismos, da forma a b c a b c. Divida esse nmero por 7, divida o resultado por 11
e, finalmente, divida o nmero obtido por 13. O que aconteceu? Por que voc obteve
esse resultado?
175. No galinheiro Um galinheiro com 240 m2 de rea deve abrigar galinhas e pintinhos,
sendo desejvel que haja um espao livre de 4 m2 para cada galinha e 2 m2 para cada
pintinho. Alm disso, cada pintinho come 40 g de rao por dia e cada galinha come
160 g por dia, sendo permitido um gasto dirio mximo de 8 kg de rao.
(a) Represente algebricamente as condies do problema.
(b) Represente graficamente, no plano cartesiano xOy, as condies do problema.
(c) Esse galinheiro comporta 20 galinhas e 80 pintinhos? E 30 galinhas e 100 pintinhos?
(d) Qual o nmero mximo de galinhas que podem ser colocadas no galinheiro,
respeitando os espaos desejveis e o gasto mximo de rao? E de pintinhos?
176. Um nmero perfeito Um nmero natural n dito perfeito se a soma de todos os
seus divisores prprios, isto , divisores diferentes de n, igual a n. Por exemplo, 6 e
28 so perfeitos, pois 6 = 1 + 2 + 3 e 28 = 1 + 2 + 4 + 7 + 14. Sabendo que 231 1
um nmero primo, mostre que 230 (231 1) um nmero perfeito.
96

OBMEP 2010

Nvel 3
177. Quinze minutos a mais Dois carros partem, ao mesmo tempo, de uma cidade A
em direo a uma cidade B. Um deles viaja velocidade constante de 60 km/h e o
outro velocidade constante de 70 km/h. Se o carro mais rpido faz a viagem de A
a B em 15 minutos a menos do que o outro carro, qual a distncia entre essas duas
cidades?
178. Outros caminhos Partindo de sua casa para chegar na escola, Jlia deve caminhar
oito quarteires para a direita e cinco quarteires para cima, conforme indicado na
figura dada.

Ela sabe que existem muitas maneiras diferentes de fazer o percurso casa-escola, sempre
seguindo o caminho mais curto. Como ela uma menina muito curiosa, ela gostaria de
sempre fazer caminhos diferentes. Quantos desses caminhos existem da casa de Jlia
at a escola?
179. Escrevendo no tabuleiro Um tabuleiro quadrado de trs linhas por trs colunas
contm nove casas. De quantos modos diferentes podemos escrever as trs letras A, B
e C em trs casas diferentes, de tal modo que, em cada linha, esteja escrita exatamente
uma dessas trs letras?
180. Frao e percentagem Se na frao x/y diminuirmos o numerador x de 40% e o
denominador y de 60%, ento a frao x/y
(a)diminui 20%;

(b)aumenta 20%;

(c)diminui 50%;

(d)aumenta 50%.

181. Tringulos sobrepostos Dois tringulos retngulos congruentes possuem catetos


que medem 4 cm e 7 cm. Na figura dada, esquerda, os tringulos foram desenhados
de modo a coincidirem os catetos de 7 cm. Assim, AB = 7 cm e AD = BC = 4 cm.
J na figura direita, eles foram desenhados de modo a coincidirem as hipotenusas,
donde AD = BC = 4 cm e AC = BD = 7 cm.
D

Calcule as reas sombreadas nas duas figuras.


OBMEP 2010

97

Nvel 3
182. Dois motoristas Dois motoristas viajam da cidade A at a cidade B e, imediatamente, regressam cidade A. O primeiro motorista viaja a uma velocidade constante
de 80 km/h, tanto na ida quanto na volta. O segundo motorista viaja at a cidade B
a uma velocidade constante de 90 km/h e retorna velocidade constante de 70 km/h.
Qual desses motoristas gasta menos tempo no percurso total de ida e volta?
183. Soma e inverte Usando somente as duas operaes +1 = somar 1 e
i = menos o inverso, podemos formar vrias sequncias a partir de um nmero
inicial. Por exemplo, iniciando com o nmero 3, podemos formar a sequncia
1 +1 4 i
5 +1
1 +1 3 i
4
+1
+1
i
3 4 5 .
5
5
4
4
4
3
Iniciando com 0, com qual sequncia obteremos novamente o 0, usando apenas essas
duas operaes +1 e i?
184. Carro flex Um carro denominado flex se ele pode ser abastecido com gasolina
ou com lcool. O consumo de um carro costuma ser dado (no Brasil) em quilmetros
por litro, que indicamos por km/l. J o custo desse consumo dado pelo preo do
quilmetro rodado, em reais por km. Suponha que os preos do litro de lcool e de
gasolina sejam, respectivamente, R$ 1,59 e R$ 2,49.
(a) Digamos que um certo carro flex rode 12,3 km por litro de gasolina. Qual deve
ser o consumo de lcool desse carro para que a utilizao do lcool seja financeiramente mais vantajosa que a de gasolina?
(b) Com os preos dados, em que condies mais vantajoso, financeiramente, o
uso do lcool em vez do de gasolina? D um exemplo numrico que satisfaa as
condies.
Daqui em diante,
que o consumo de um certo carro flex seja de x km/l com

 x suponha
gasolina e de
+ 1 km/l com lcool.
2

(c) Escreva a expresso da funo g(x) que fornece o custo de rodar 100 quilmetros
com esse carro utilizando gasolina e a expresso da funo a(x) que fornece o
custo de rodar 100 quilmetros utilizando lcool.
(c) Para que o custo seja o mesmo, tanto com lcool como com gasolina, qual deve
ser o consumo em km/l para a gasolina e para o lcool?

(d) Com o consumo dado, em que condies mais vantajoso, financeiramente, o


uso do lcool em vez do de gasolina? D um exemplo numrico que satisfaa as
condies.
185. Contando tringulos Na figura dada esto marcados onze pontos sobre dois segmentos. Quantos tringulos podem ser formados com esses onze pontos?

98

OBMEP 2010

Nvel 3
186. Quadrado perfeito Existe um nmero de oito algarismos da forma
9999
que seja um quadrado perfeito?
187. Diferena quase nula Qual o menor nmero inteiro positivo n tal que

n n 1 < 0,01?
188. Conjunto de Cantor Desenhe um segmento de reta com uma unidade de comprimento, denotando-o por C1 . Remova a tera parte central (sem remover as extremidades) e denote por C2 o que sobrou. Agora, remova a tera parte central (sem as
extremidades) de cada um dos dois segmento de reta que constituem C2 , denotando
por C3 o que sobrou. Esse processo pode ser continuado, sempre removendo, em cada
estgio, a tera parte central de cada segmento em Cn para formar Cn+1 . O conjunto de
Cantor formado pelos elementos de C1 que nunca so removidos, em etapa alguma.
C1
b

C2
b

C3

(a) Na figura dada, indique os nmeros nas extremidades dos segmentos C1 , C2 e C3 .


4
1 4 3
e
pertencem ao conjunto de Cantor?
(b) Quais dentre os pontos , ,
3 9 81 81
(c) Quais so os comprimentos de C3 , C4 e C5 ? Voc consegue encontrar alguma
expresso para o comprimento de Cn ?
189. Enchendo uma piscina Uma piscina vazia foi abastecida de gua por duas torneiras A e B, ambas de vazo constante. Durante quatro horas, as duas torneiras ficaram
abertas e encheram 50% da piscina. Em seguida, a torneira B foi fechada e, durante
duas horas, a torneira A encheu 15% do volume da piscina. Aps este perodo, a torneira A foi fechada e a torneira B aberta. Durante quanto tempo essa torneira teve de
ficar aberta para que ela, sozinha, terminasse de encher a piscina?
190. Probabilidade de ser um nmero par Uma urna tem nove bolas, numeradas de
1 a 9. Jos e Maria retiram, cada um, simultaneamente, uma bola da urna. Com as
bolas retiradas eles formam um nmero de dois algarismos, sendo que o nmero que
est escrito na bola de Jos o algarismo das dezenas e o nmero que est escrito na
bola de Maria o algarismo das unidades. Qual a probabilidade desse nmero ser
par?
191. Mltiplo de 7 Mostre que se o produto N = (n + 6m)(2n + 5m)(3n + 4m), com m
e n nmeros inteiros positivos, for um mltiplo de 7 ento esse produto N tambm
mltiplo de 73 = 343.
OBMEP 2010

99

Nvel 3
192. Os ngulos 15 e 75 Na figura dada, ABCD um quadrado com uma unidade
de lado e o tringulo BCE equiltero. O ponto M o ponto mdio do segmento
CE, o segmento DN perpendicular a BM e o segmento BM perpendicular a CE.
(a) Calcule os comprimentos dos lados do tringulo DBN.

(b) Use o item (a) para calcular o cosseno, o seno


e a tangente dos ngulos de 15 e 75 .

193. Crculos tangentes Na figura dada esto desenhados dois


crculos concntricos de raios r e R, sendo r < R, e doze
crculos compreendidos entre os dois primeiras, todos de raio
x. Alm disso, os quatorze crculos so disjuntos ou tangentes.
Rr
.
2

4+ 6 2
R

.
=
(b) Mostre que
r
4 6+ 2
(a) Mostre que x =

194. Mudando a base Um tringulo issceles tem uma base de 10 cm


e dois lados iguais medindo 13 cm. possvel cortar esse tringulo em
dois outros tringulos de tal modo que, juntando esses tringulos de outra
maneira obtenhamos um outro tringulo issceles (evidentemente com a
mesma rea)?

....
... ...
... ....
.... ....
...
..
...
...
...
...
...
..
.
...
..
.
...
....
...
.
.
...
..
.
...
..
...
.
..
..
.
..................................................................

13

13

10

195. Clube de Matemtica Eu fao parte de um Clube de Matemtica, onde tenho


o mesmo nmero de colegas homens do que de colegas mulheres. Quando um garoto
falta, trs quartos da equipe so de meninas. Eu sou homem ou mulher? Quantas
mulheres e quantos homens tem o clube?
196. Uma calculadora diferente Davi tem uma calculadora muito original, que efetua
apenas duas operaes, a adio usual (+) e uma outra operao, denotada por , que
satisfaz
(i) a a = a,

(ii) a 0 = 2a e

(iii) (a b) + (c d) = (a + c) (b + d),
para quaisquer nmeros inteiros a e b. Quais so os resultados das operaes
(2 3) + (0 3) e 1 024 48?
197. Cercando o globo terrestre O raio do globo terrestre mede, aproximadamente, 6 378 km no Equador. Suponhamos que um fio esteja
ajustado exatamente sobre o Equador.

100

OBMEP 2010

Nvel 3
Em seguida, suponhamos que o comprimento do fio seja aumentado em 1 m, de modo
que o fio e o Equador fiquem como crculos concntricos ao redor da Terra. Um homem
em p, uma formiga, ou um elefante so capazes de passar por baixo desse fio?
198. Comprimento de uma corda Numa circunferncia de 10 cm de raio, o segmento
AB um dimetro e o segmento AC uma corda de 12 cm. Determine a distncia
entre os pontos B e C.
199. Dois irmos A diferena de idade entre dois irmos de trs anos. Um ano atrs,
a idade do pai desses irmos era o dbro da soma das idades dos irmos e, dentro de
vinte anos, a idade do pai ser a soma das idades desses dois filhos. Qual a idade de
cada um dos irmos?
200. Canelonis de ricota Todo domingo, Pedro prepara canelonis para o almoo. Primeiro, ele corta retngulos de massa
de 16 por 12 cm e, depois, cola os dois lados mais longos,
superpondo uma faixa de 2 cm.

16 cm
10 cm
2 cm

Dessa forma, ele obtm cilindros que recheia com ricota, gastando 500 g de ricota por
cilindro. Num belo domingo, com o mesmo nmero de retngulos de massa de 16 por
12 cm, ele decide produzir os cilindros colando os lados menores, sempre superpondo
uma faixa de 2 cm. Nessa situao, ele vai gastar mais ou menos ricota que antes?
Quanto?
201. Clculo de segmentos As medidas do retngulo ABCD A
so de 1 200 por 150 m. Alm disso, P est no prolongamento
do lado BC e dista 350 m de C. Determine as medidas de
D
AP, P Q, P D, CQ e DP.

1200
Q

B
150
C
350

202. Pr chegar junto! Ada e Luisa treinam todos os dias, cada uma delas sempre
com a mesma velocidade, para a grande corrida que vai acontecer no final do ano na
escola. O treino comea num ponto A e termina no ponto B, distantes 3 000 m. Elas
partem no mesmo instante, mas quando Luisa termina a corrida, ainda faltam 120 m
para Ada chegar ao ponto B. Ontem, Luisa deu uma chance para Ada: Partimos ao
mesmo tempo, mas eu parto alguns metros antes do ponto A para chegarmos juntas.
Quantos metros antes do ponto A deve partir Luisa para chegar junto com Ada?
203. Um professor enfurecido Para castigar os alunos de sua turma por indisciplina,
o professor Zerus decidiu descontar da nota mensal de cada aluno uma percentagem
igual nota da prova, isto , quem tirou 60, ter um desconto de 60% na nota, quem
tirou 20, um desconto de 20% na nota e assim por diante. A nota mensal mxima
100.
(a) Quem vai ficar com a maior nota?
(b) E a menor?
OBMEP 2010

101

Nvel 3
(c) Alunos que tiraram notas boas reclamaram que vo ficar com a mesma nota de
alunos que tiraram notas ms. Eles tem razo?

204. O percurso de um atleta Um atleta resolveu fazer uma corrida de 15 km. Comeou correndo 5 km na direo Sul, depois virou para o Leste, correndo mais 5 km
e, novamente, virou para a direo Norte, correndo os 5 km restantes. Aps esse
percurso, constatou, para seu espanto, que estava no ponto de onde havia partido.
Descubra dois possveis pontos sobre o globo terrestre de onde esse atleta possa ter
iniciado sua corrida.
A

205. reas iguais Na figura dada, o tringulo ABC retngulo


e os semicrculos dados tm dimetros AB, BC ou AC. Mostre
que a rea sombreada igual rea do tringulo ABC.

206. Funo definida por rea A funo f, definida para cada y satisfazendo 0 y < 2, dada
por f (y) = rea do quadriltero sombreado, conforme indicado na figura dada.

r
6

(a) Escreva as equaes das retas r e s.

(b) Determine f (0).


(c) Escreva a expresso de f (y), para 0 y < 2.

(d) Esboce o grfico de f (y).

y
-2 -1

1 2 3

207. PA e PG Determine quatro nmeros distintos a1 , a2 , a3 e a4 que sejam termos


consecutivos de uma progresso aritmtica e tais que os nmeros a1 , a3 e a4 formem
uma progresso geomtrica.
208. Plano cartesiano Dizemos que um ponto do plano cartesiano um ponto inteiro se
suas coordenadas so inteiras. Dado um inteiro positivo n, denote por f (n) o nmero de
pontos inteiros que esto sobre o segmento que liga a origem ao ponto inteiro (n, n+ 3),
sem contar as extremidades. Mostre que

2, se n mltiplo de 3,
f (n) =

0, se n no mltiplo de 3 .
D

209. Trabalhando com um quadriltero No quadriltero ABCD da figura dada, tem-se AB = 5, BC = 17,
CD = 5 e DA = 9. Determine DB, sabendo que sua
medida um nmero inteiro.
102

OBMEP 2010

...
.................. ......
....
....
.
....
.
.
.
....
....
.
.
....
...
.
....
.
...
....
.
.
....
...
.
.
....
..
.
.
.
...
...
.
.
.................................................................................................................................................................

A.......................................................

Nvel 3
210. O tringulo de Reuleaux O tringulo de Reuleaux a figura
formada a partir de um tringulo equiltero, substituindo os lados
por arcos de circunferncia centrados nos vrtices do tringulo e de
raios iguais ao lado do tringulo. Qual a rea de um tringulo de
Reuleaux, se os lados do tringulo equiltero inicial medem 1 cm?
211. Interseo de crculos Na figura dada foram desenhados trs crculos de raio r centrados nos vrtices do tringulo
equiltero ABC de lado a. Se 12 a < r < a, esses trs crculos
so, dois a dois, concorrentes em trs pontos X, Y e Z exteriores ao tringulo ABC. Mostre que o tringulo XY Z
equiltero e calcule o comprimento do seu lado, em termos de
a e r.

b
b

C
Y

A
Z

212. Valor mximo A expresso


k?

k2
atinge seu maior valor com qual nmero natural
1,001k

213. Moedas falsas Aladim tem dez sacos de moedas, sendo que cada
saco tem somente moedas verdadeiras ou somente moedas falsas.
Cada moeda verdadeira pesa 10 g e cada moeda falsa pesa 1 g.

(a) Suponhamos que em cada saco existam exatamente dez moedas e que somente um
dos sacos seja de moedas falsas. Utilizando uma balana e efetuando apenas uma
pesagem, como deve Aladim proceder para descobrir qual o saco das moedas
falsas?
(b) Suponhamos que os sacos estejam cheios de moedas e que Aladim no saiba
quantos desses sacos sejam de moedas falsas. Como pode ele identificar, com
apenas uma pesagem, os sacos que tm moedas falsas?

214. Menor inteiro Sejam p e q inteiros positivos tais que


valor de p para que p + q = 2 005?

p
7
5
< < . Qual o menor
8
q
8

215. Mais reas... Um tringulo tem vrtices A = (3, 0), B = (0, 3) e C, onde C est
na reta de equao x + y = 7. Qual a rea desse tringulo?
216. Crculos tangentes Trs crculos, com raios medindo 1, 2
e 3 cm, so dois a dois tangentes exteriormente, como na figura
dada. Determine o raio do crculo tangente exteriormente aos
trs crculos.

OBMEP 2010

103

Nvel 3
217. Soma
finita Cada um dos nmeros x1 , x2 , . . . , x2 004 pode ser igual a

2 + 1. Quantos valores inteiros distintos pode valer a soma


1X
002
k=1

2 1 ou a

x2k1 x2k = x1 x2 + x3 x4 + x5 x6 + + x2 003 x2 004 ?

218. Mltiplos Seja a um nmero inteiro positivo que mltiplo de 5 e tal que a + 1
mltiplo de 7, a + 2 mltiplo de 9 e a + 3 mltiplo de 11. Determine o menor valor
possvel de a.
219. Equao de duas variveis Determine todos os pares de inteiros (x, y) tais que
9xy x2 8y 2 = 2 005.
220. Trapzio retngulo Seja ABCD um trapzio retngulo de bases AB e CD, com
ngulos retos em A e D. Dado que a diagonal menor BD perpendicular ao lado BC,
determine o menor valor possvel para a razo CD/AD.
221. Jogos de futebol Os doze alunos de uma turma de olimpada saam para jogar
futebol todos os dias aps a aula de matemtica, formando dois times de seis jogadores
cada e jogando entre si. A cada dia eles formavam dois times diferentes dos times
formados em dias anteriores. Ao final do ano, eles verificaram que cada cinco alunos
haviam jogado juntos num mesmo time exatamente uma vez. Quantos times diferentes
foram formados ao longo do ano?
222. A soma dos algarismos de um nmero Denotemos por s(n) a soma dos algarismos do nmero n. Por exemplo s(2 345) = 2 + 3 + 4 + 5 = 14. Observemos que:
40 s(40) = 36 = 9 4;

500 s(500) = 495 = 9 55;

2345 s(2345) = 2331 = 9 259 .


(a) O que podemos afirmar sobre o nmero n s(n)?

(b) Usando o item anterior, calcule s(s(s(22 009 ))).

Sugesto: Mostre que o nmero procurado menor do que 9.

104

OBMEP 2010

Desafios

Desafios
1. Cadeia do menor nmero (N2/N3) Partindo do nmero 265 863 e utilizando
uma nica vez cada uma das operaes +, , e e tambm uma nica vez os
nmeros 51, 221, 6 817, 13 259, podemos obter vrios nmeros. Por exemplo, 54 911,
como segue.
221

51

13 259

+6 817

z}|{
z}|{
z}|{
z}|{
265 863 1 203 61 353 48 094 54 911

Encontre a cadeia que permite obter o menor nmero inteiro positivo.


2. Qual a metade? (N2/N3) Considere a figura ao lado,
em que AB = AE = ED = CD = CA e o arco CB um arco
de crculo centrado no ponto E. Voc sabe repartir essa figura
em duas partes idnticas, que possam ser superpostas?

3. Cada um em seu estado (N1/N2/N3) Amlia, Bruno, Constncia e Denise so


quatro amigos que se encontram sentados numa mesa quadrada, cada um ocupando
um lado da mesa. Um dos quatro mora no Amazonas, outro em So Paulo, outro no
Cear e o quarto na Bahia. Sabendo que valem as condies a seguir, quem mora na
Bahia?
direita de Amlia est quem mora no Amazonas.

Em frente Constncia est a pessoa que mora em So Paulo.


Bruno e Denise esto um ao lado do outro.

Uma mulher est esquerda da pessoa que mora no Cear.


4. Diviso (N1/N2) Numa diviso, aumentando o dividendo de 1 989 e o divisor de
13, o quociente e o resto no se alteram. Qual o quociente?
5. Extraterrestre (N1/N2) No planeta Staurus, os anos tm 228 dias, divididos em
12 meses de 19 dias. Cada semana tem 8 dias: Zerum, Uni, Duodi, Trio, Quati, Quio,
Seise e Sadi. Sybock nasceu num duodi, que foi o primeiro dia do quarto ms. Que
dia da semana ele festejar seu primeiro aniversrio?
6. Que famlia! (N1/N2) Numa famlia, cada menino tem o mesmo nmero de
irmos que de irms, e cada menina tem o dobro de irmos que de irms. Qual a
composio dessa famlia?
7. Siga a pista (N1) Na pista de corrida dada, os sete pontos
de referncia so marcados a cada 50 m. Os atletas devem
fazer 2 km no sentido indicado pela flecha, partindo do ponto
P. Marque o ponto C de chegada.
OBMEP 2010

105

Desafios
8. Cara ou Coroa (N2) Jernimo joga no tabuleiro dado com uma pea e um dado
da maneira descrita a seguir.
Colocando a pea na casa P (de partida), ele
lana uma moeda. Se der cara, avana duas
casas e se der coroa recua uma casa. Jernimo
lanou a moeda 20 vezes e conseguiu chegar na
casa C (de chegada). Quantas vezes a moeda
deu cara?
9. Contas do papagaio (N1) Rosa tem um papagaio que faz contas de um modo
estranho. Cada vez que Rosa diz dois nmeros ele faz a mesma conta. Por exemplo:
se Rosa diz 4 e 2 o papagaio responde 12;
se Rosa diz 5 e 3 o papagaio responde 12;
se Rosa diz 3 e 5 o papagaio responde 14;
se Rosa diz 9 e 7 o papagaio responde 24;
se Rosa diz 0 e 0 o papagaio responde 1.
Se Rosa diz 1 e 8 o que responde o papagaio?
10. As frias de Toms (N1/N2) Durante as frias de Toms, houve 11 dias chuvosos.
Durante esses 11 dias, se chovia pela manh havia sol sem chuva tarde, e se chovia
tarde, havia sol sem chuva pela manh. No total, Toms teve 9 manhs e 12 tardes
sem chuva. Quantos dias duraram as frias de Toms?
11. Maratona de Matemtica (N3) Numa maratona de Matemtica, o nmero de
questes muito grande. O valor de cada questo igual sua posio na prova:
um ponto para a questo 1, dois pontos para a questo 2, trs pontos para a questo
3, quatro pontos para a questo 4, e assim por diante. Joana totalizou 1 991 pontos
na prova, errando apenas uma questo e acertando todas as outras. Qual questo ela
errou? Quantas questes tinha a prova?
1 1 1 1 1
1
12. Fraes ignoradas (N1) Escolhi quatro fraes dentre , , , ,
e
, cuja
2 4 6 8 10 12
soma 1. Quais foram as fraes que eu no escolhi?
13. Caminho de maior total (N2) As regras do jogo so as seguintes.
(a) Partindo da casa com o nmero 3 destacado, deve-se chegar casa TOTAL
deslocando-se somente por linhas ou colunas e calculando-se os pontos.
(b) Quando nos deslocamos por uma linha, s podemos adicionar, por exemplo passando da 3 para a 6 ao lado, obtemos 3 + (6) = 3 pontos.

(c) Quando nos deslocamos por uma coluna, s podemos subtrair, por exemplo passando da 3 para a 5 abaixo, obtemos 3 5 = 2 pontos.

(d) S permitido passar uma vez por cada casa.


Qual o caminho que d o maior total?
106

OBMEP 2010

Desafios
3
5
8
4
0

6
7
3
1
2

9
2
5
6
7

9
1
4
8
TOTAL

14. Produtos em linha (N1/N2/N3) Em cada uma das oito casas brancas do quadro
dado, escrevemos um algarismo dentre os algarismos 1, 2, 3, 4, 5, 6, 7, 8 e 9 de modo que
os produtos efetuados em linha reta, seguindo as flechas, forneam os valores indicados
dentro dos casas em cinza. Em qual casa se encontra o nmero 2?

15. Cdigo Postal (N2/N3) Para fazer a separao em regies da correspondncia


que deve ser entregue, um servio postal indica sobre os envelopes um cdigo postal
com uma srie de cinco blocos de pontos e bastes, que podem ser lidos por um leitor
tico. Os algarismos so codificados como segue.

A leitura se faz da direita para a esquerda. Por exemplo: o cdigo postal 91720 se
escreve como
, ou seja,

.
Note que a codificao de 94, que
, tem um eixo vertical de simetria.
Encontre os cdigos entre 47000 e 47999 que apresentam um eixo vertical de simetria.

OBMEP 2010

107

Desafios
16. Anis olmpicos (N1/N2/N3) Os nmeros de 1 a 9 foram colocados dentro de
cinco anis olmpicos, de tal modo que dentro de cada anel a soma 11.

Disponha os nove nmeros de outra maneira, para que a soma dentro de cada anel seja
sempre a mesma e a maior possvel.
17. Partidas de Denise (N2/N3) Denise e Antnio jogam uma srie de oito jogos,
em que o vencedor da primeira partida ganha um ponto, o da segunda dois pontos, o
da terceira quatro pontos, o da quarta oito pontos, e assim por diante, multiplicando
por dois o nmero de pontos de uma partida para a outra. No final, Denise ganhou
31 pontos a mais que Antnio e no houve empate em nenhuma das partidas. Quais
partidas Denise ganhou?
18. Sete quadrados (N1/N2) Voc sabe repartir um quadrado em sete quadrados
menores?
19. Ilha misteriosa (N1/N2/N3) Numa misteriosa ilha havia 13 camalees cinza,
15 camalees marrons e 17 camalees vermelhos. Quando dois camalees de cores
diferentes se encontram, os dois tomam a terceira cor. Por exemplo, se um cinza se
encontra com um vermelho, ento os dois ficam marrons. Por causa de uma tempestade, ocorreram dois encontros cinza-vermelho, trs encontros marrom-vermelho e um
encontro cinza-vermelho. Quantos camalees de cada cor ficaram na ilha?
20. Universo hostil (N3) Num deserto h cobras, ratos e escorpies. Cada manh,
cada cobra mata um rato. Cada meio-dia, cada escorpio mata uma cobra. Cada
noite, cada rato mata um escorpio. Ao final de uma semana, noite, s restava um
rato. Quantos ratos havia na manh do incio da semana?
21. O jogo das fichas Para iniciar um jogo com seus amigos, Manoel coloca oito fichas
em cada uma das nove casas do tabuleiro mostrado na figura. Para ganhar o jogo, ele
precisa mover as fichas pelo tabuleiro de tal modo que, ao final, todas as fichas estejam
colocadas e tenha sido alcanada uma outra configurao de fichas na qual, em cada
linha, cada coluna e cada diagonal, reste o mesmo nmero de fichas (mas no a inicial, com oito fichas em cada casa). Na primeira jogada, ele coloca mais trs fichas na
casa 3 e tira todas da casa 2, ficando com 5 na mo, para prosseguir. Quantas fichas ele

108

OBMEP 2010

Desafios
deve colocar em cada uma das outras sete casas para ganhar o jogo, mantendo as fichas
das casa 2 e 3 inalteradas depois dessa primeira jogada?

2
3

0
fichas

11
fichas

22. Um sistema Nas igualdades AB + BC = CD e AB BC = BA, cada letra


representa um algarismo. Quanto vale A + B + C + D?
23. Constelaes floridas Rosa, Margarida e Dlia so trs constelaes em forma de
buqus de flores. Sabemos que:
(a) o nmero de estrelas de Dlia, que a menor das trs constelaes, o produto
de dois quadrados;
(b) o nmero de estrelas de Rosa tambm um produto de dois quadrados;
(c) Dlia e Rosa, juntas, tm o mesmo nmero de estrelas de Margarida;
(d) Margarida tem 28 561 estrelas.
Quantas estrelas possuem, cada uma, Dlia e Rosa?
24. Dois meses iguais A seguir mostramos o calendrio de abril de 2 005.
D

3
10
17
24

4
11
18
25

5
12
19
26

6
13
20
27

7
14
21
28

S
1
8
15
22
29

S
2
9
16
23
30

Qual o primeiro ms depois desse, de 2005 ou de 2006, que teve uma pgina igual?
25. A faixa e o quadrado Uma faixa retangular de cartolina mede 5 por 1 cm. Corte
a faixa com quatro cortes retilneos de modo a poder montar um quadrado com as
peas obtidas, mas sem superposio das peas.
26. Um nmero e seu sxtuplo Um nmero de trs algarismos e seu sxtuplo so
formados pelos mesmos algarismos. A soma dos algarismos desse nmero 17 e a de
seu sxtuplo 21. Qual esse nmero? Existe mais do que um nmero com essas
propriedades?
OBMEP 2010

109

Desafios
27. Oito dentro de um retngulo Coloque os nmeros de 1 a 8
dentro dos crculos do retngulo dado, de tal modo que a diferena
entre dois nmeros ligados por um segmento seja sempre maior do
que 1.

..................................
..................................
..
... .......
....
.... ........
.....
.....
.... ..
...
.
.....
..... ....
...
................................
...
...
....
..
. .
.. ........
.... ....... ....
... ......
......
....
.... .
.
.
.
.
.
.
.
.
.
..................................
..................................

28. Uma estratgia com um nmero muito grande Carlos escreveu consecutivamente todos os nmeros de 1 a 60, ou seja,
1234567891011121314 57585960.
Depois ele riscou 100 algarismos de tal modo que o nmero formado com os algarismos
que no foram riscados fsse o maior possvel, sem mudar a ordem inicial em que os
algarismos foram escritos. Qual esse nmero?
29. Um nmero surpreendente Um certo nmero surpreendente divisvel por 9,
tem nove algarismos diferentes, nenhum dos quais igual a 0 e tal que:
(a) o nmero formado pelos 2 primeiros algarismos divisvel por 2;
(b) o nmero formado pelos 3 primeiros algarismos divisvel por 3;
(c) o nmero formado pelos 4 primeiros algarismos divisvel por 4;
(d) o nmero formado pelos 5 primeiros algarismos divisvel por 5;
(e) o nmero formado pelos 6 primeiros algarismos divisvel por 6;
(f) o nmero formado pelos 7 primeiros algarismos divisvel por 7;
(g) o nmero formado pelos 8 primeiros algarismos divisvel por 8.
Qual esse nmero?
30. Qual o erro? Uma das afirmaes dadas falsa:
(a) Andr mais velho do que Bruno;
(b) Cludia mais nova do que Bruno;
(c) A soma das idades de Bruno e Cludia o dobro da idade de Andr;
(d) Cladia mais velha do que Andr.
Quem o mais velho? E o mais novo?
31. Soma Neste exerccio, as letras representam algarismos. Determine cada uma das parcelas da soma dada.

abcdef
abcdef
+
ghij
.................................................................................................

def hjf
32. Bolinhas Rogrio colocou seis bolinhas sobre a mesa, de modo a
formar dois quadrados, como na figura. Ele percebe que tem mais
uma bolinha. Complete a figura formada pelas bolinhas com essa
bolinha a mais, de tal modo que forme trs quadrados.
110

OBMEP 2010

..................................................................................................
...
...
...
...
...
...
...
...
...
...
...
...
...
...
...
...
...
...
..
....
....
.................................................................................................

Desafios
33. Um nmero que no divisvel por 5 Determine quais nmeros naturais n
entre 2 001 e 2 007 tornam o nmero 1n + 2n + 3n + 4n no divisvel por 5.
34. Quatro fraes e um inteiro Quantos nmeros naturais a, b, c e d, todos distintos,
1 1 1 1
existem tais que
+ + +
seja um inteiro?
a b c d
35. O Rei Arthur e o Drago das Trs Cabeas e Trs Caudas O Rei Arthur
teve que lutar com o Drago das Trs Cabeas e Trs Caudas. Sua tarefa foi facilitada
quando conseguiu arranjar uma espada mgica que podia, a cada golpe, fazer uma e
somente uma das seguintes coisas:
(a) cortar uma cabea;
(b) cortar duas cabeas;
(c) cortar uma cauda;
(d) cortar duas caudas.
Alm disso, a Fada Morgana lhe revelou o segredo do drago:
(a) se uma cabea cortada, cresce uma nova;
(b) se duas cabeas so cortadas, nada mais acontece;
(c) no lugar de uma cauda cortada nascem duas caudas novas;
(d) se duas caudas so cortadas, cresce uma nova cabea;
(e) o drago morre se perder as trs cabeas e as trs caudas.
Para matar o drago, de quantos golpes o Rei Artur vai precisar, no mnimo?
36. O passeio do cavalo Num tabuleiro de 5 5 casas, um cavaleiro do jogo de xadrez
est na casa marcada com A. Depois ele segue movendo, marcando as casas por onde
passa, como na figura.
A B C D E F G H
A

G
H
B

F
D

Partindo da casa H, o cavaleiro se move pelo tabuleiro at ter passado por todas as 25
casas. Descreva o trajeto que ele fez.
37. As faces do cubo Oito dados so agrupados formando um cubo. Quantas faces
dos dados permanecem visveis?
OBMEP 2010

111

Desafios
38. Data fatdica Em 1950, um profeta anunciou que o fim do mundo ocorreria em
11 de agosto de 1999, que denotamos por 11081999. Como nada aconteceu nesse dia,
ele refez seus clculos e fez a seguinte previso: O fim do mundo ocorrer na prxima
data que se escrever com oito algarismos diferentes. Voc consegue descobrir essa
data?
39. Todos com o 2 Qual operao que devemos fazer com todos os cinco nmeros
418, 244, 816, 426 e 24 para obter cinco nmeros que tenham todos o algarismo 2?
(a) Dividir por 2.
(b) Somar 4.
(c) Dividir por 6.
(d) Subtrair 5.
(e) Multiplicar por 3.
40. Tortas da vov Sofia foi levar uns docinhos para sua av: foram sete docinhos de
amora, seis de cco e trs de chocolate. Durante o caminho, a gulosa Sofia comeu dois
docinhos. Qual das situaes abaixo possvel?
(a) Sua av no recebeu docinhos de chocolate.
(b) Sua av recebeu menos docinhos de cco do que de chocolate.
(c) Sua av recebeu o mesmo nmero de docinhos de cada uma das trs variedades.
(d) Existem duas variedades de docinhos das quais sua vov recebeu o mesmo nmero.
(e) O nmero de docinhos de amora que sua vov recebeu maior que o das outras
duas variedades somadas.
41. Famlia Stimo O Sr. e Sra. Stimo tm sete filhos, todos nascidos em 1o de abril;
na verdade, em seis 1o de abril consecutivos. Neste ano, para seus aniversrios, a Sra.
Stimo fez um bolo com velinhas para cada um de seus filhos, sendo o nmero de velas
em cada bolo igual ao nmero de anos do aniversariante. Joo Stimo, o filho que mais
gosta de Matemtica, reparou que, nesse ano, o nmero total de velinhas o dobro do
que havia dois anos atrs e que h dois bolos a mais. Quantas velinhas sero acesas
desta vez?
42. O salta-ficha Temos dez fichas numeradas colocadas em linha reta, como na figura
dada.
................ .................. ................. .................. .................. .................. .................. .................. .................. ..................
...
.. ..
... ..
.. ...
.. ...
.. ...
.. ...
.. ...
.. ...
... ...
...
....
..
.
.
.
..
.
..
.
..... ...... ...... ...... ...... ....... ....... ...... ...... ...... ...... ..... ....... ...... ....... ...... ....... ....... ...... ......
........
........
........
........
.......
.......
........
........
........
.......

9 10

Queremos arrum-las em cinco pilhas, com duas fichas em cada pilha. A regra para
isso que s podemos movimentar uma ficha fazendo-a saltar sobre uma ou mais fichas,
ou sobre uma nica pilha j formada. Um exemplo de trs movimentos o seguinte.
A ficha 7 pode saltar sobre a ficha 8 e formar uma pilha com a 9;

a ficha 5 pode saltar sobre as fichas 6 e 7 e formar uma pilha com a 8 e


a ficha 6 pode saltar sobre as fichas 5, 4 e 3 formar uma pilha com a 2.
112

OBMEP 2010

Desafios
...........
...
...........
.....
.......
...........
.....
.......
...........
.....
.......
...........
.......
.....
.
.
.
.
.
.
...........
.
.
.
.
.
.....
...........
.....
.......
...........
....
.....................
.....
...... .........
.....
.......... ............... ............... ............... .................. .................. ................ ............... ............... ...............
.
.
.
.
.
... ...
... ...
... ...
... ...
... ...
... ...
... ..
... ...
...
... ...
..
.. .
. ..
.. .
.. .
..
..
. ..
. ..
....
. ..
.
.... .... ...... ..... ..... .... ..... ..... ...... .... ...... .... ..... .... ..... .... ...... ..... ...... ....
............
...........
...........
...........
...........
...........
...........
...........
...........
...........

9 10

........
..... .......
..
.....
.... ......
.........
.......... ............... ............... ...............
.
.
.
.
.
.
... ...
... ...
...
... ...
.....
.
.
..
.. ..
..
.. ..
.. ..
...
............... .................. .................. ..................

........
.......
..... ....... ...... ......
.. ..
.....
..
..... ..... ...... .......
.........
........
.......... .............. ...............
.
.
.
.
.
.
... ...
...
... ..
....
.
..
.
.. ...
.. ..
..
...
............... .................. ..................

9 10

Como formar cinco pilhas de duas fichas com apenas cinco movimentos?
43. O menor Qual o menor nmero, 52 002 ou 32 002 + 42 002 ?
44. O maior resultado Qual o maior resultado que podemos encontrar quando
dividimos um nmero de dois algarismos pela soma de seus algarismos?
45. Dois mil Digamos que o peso de um nmero seja a soma de seus algarismos. Qual
o menor nmero que pesa 2 000?
46. No cabeleireiro Trs clientes esto no cabeleireiro, pagando cada um a sua conta
no caixa.
O primeiro cliente paga uma quantia igual ao montante que h no caixa e recebe
10 reais de troco.
O segundo cliente efetua a mesma operao que o primeiro.
O terceiro cliente efetua a mesma operao que o primeiro.

Encontre o montante que estava inicialmente no caixa, sabendo que, ao fim das trs
operaes, o caixa ficou zerado.
47. O macaco e a raposa O macaco diz para a raposa: Voc v as trs pessoas que
esto correndo l longe? Eu sei que o produto de suas idades 2 450 e que a soma de
suas idades o dobro da sua idade. Voc pode me dizer suas idades? No, responde
a raposa. E se eu te disser que o mais jovem dos trs o nico louro, voc pode agora
descobrir as idades? Ento a raposa d as idades das trs pessoas.
Porque a raposa no pode responder inicialmente? E porque pode responder depois?
48. Nova sequncia Encontre a lei que forma a sequncia 425, 470, 535, 594, 716,
802, . . . e d seus prximos dois termos.
49. Retngulo quase quadrado Um certo terreno retangular quase quadrado, pois
sua largura e seu comprimento medem nmeros inteiros que diferem exatamente por
uma unidade de medida. A rea desse terreno, em unidades quadradas, um nmero
de quatro algarismos, sendo iguais o das unidades de milhar e o das centenas, bem
como o das dezenas e o das unidades. Quais so as possveis dimenses desse terreno?
50. Onde est o erro? Seja x soluo de x2 + x + 1 = 0. Ento x 6= 0 e, por isso,
podemos dividir ambos os membros da equao por x, obtendo x + 1 + 1/x = 0. Da
equao original temos que x + 1 = x2 , portanto, x2 + 1/x = 0, isto , x2 = 1/x
ou, ainda, x3 = 1, de modo que x = 1. No entanto, substituindo x = 1 na equao
x2 + x + 1 = 0 original, encontramos 3 = 0, o que no est exatamente correto. Onde
erramos?

OBMEP 2010

113

Solues do Nvel 1

Solues do Nvel 1
1. Qual o nmero? A opo correta (d).
Como 96 8 = 12, temos 8 12 = 96. Observe que a soluo equivalente a resolver
96
a equao 8x = 96, cuja raiz x =
= 12.
8
2. Muro em 15 dias A opo correta (c).
Se o pedreiro assenta 8 metros por dia, em 15 dias assentar 15 8 = 120 metros.
3. Medindo pilhas de papel A opo correta (e).
Como a espessura de cada folha 0,1 mm, a altura de um pacote com 500 folhas
500 0,1 mm = 50 mm. Logo, a altura de cada pilha ser de 60 50 mm = 3 000 mm
= 3 m, aproximadamente, a altura de uma sala de aula.
4. Quanto pesa? A opo correta (b).
Soluo 1: Retirando-se dois saquinhos e quatro bolas de cada prato, a balana
continua equilibrada e restam trs saquinhos no prato esquerda e seis bolas no prato
da direita. Logo, o peso de trs saquinhos igual ao peso de seis bolas. Da conclumos
que o peso de um saquinho igual ao peso de duas bolas.
Soluo 2: Denotando o peso de um saquinho por x e o peso de uma bola por y,
o equilbrio da balana fornece a equao 5x + 4y = 2x + 10y, da qual decorre que
3x = 5x 2x = 10y 4y = 6y, ou seja, x = 2y.
5. Calcule a diferena A opo correta (e).
Para que a diferena seja a maior possvel devemos escolher o maior nmero de trs
algarismos pares diferentes e o menor nmero de trs algarismos mpares diferentes.
O maior nmero de trs algarismos pares diferentes 864 e o menor nmero de trs
algarismos mpares diferentes 135. A diferena entre eles 864 135 = 729.
6. Qual o volume? A opo correta (b).
As figuras mostram que os volumes ocupados pelos lquidos correspondem, aproximadamente, a mais do que da metade no frasco A, o que elimina as opes (a) e (e),
metade no frasco B e a menos da metade no frasco C, o que elimina (c) e (d). O nico
grupo de fraes que corresponde a essas estimativas 2/3 (mais do que a metade),
1/2 (metade) e 1/4 (menos do que a metade).
7. Descontos e descontos A opo correta (b).
Soluo 1: A pessoa ir pagar 120 reais menos o desconto, que de 30% sobre 120,
ou seja, de 0,3 120 = 36 reais. Assim, a pessoa paga 120 36 = 84 reais.
Soluo 2: Como o desconto de 30%, a pessoa pagar 70% de 120, ou seja,
0,7 120 = 84 reais.
114

OBMEP 2010

Solues do Nvel 1
8. O carro de Maria A opo correta (a).
Soluo 1: Se num percurso de 25 km ela gasta 3 litros, ento para percorrer 100
km Maria gastar 4 3 = 12 litros. Logo, para percorrer 600 km, o carro gastar
6 12 = 72 litros. Como cada litro custa 0,75 reais, 72 litros custaro 0,75 72 = 54
reais.
Soluo 2: Podemos usar a regra de trs para calcular quantos litros sero gastos em
600 km. Temos:
3 litros 25 km
x litros 600 km.
Como essa regra de trs direta, resulta 25x = 3 600 e, portanto,
x=3

600
= 72 litros.
25

Como cada litro custa 0,75 reais, 72 litros custaro 0,75 72 = 54 reais.
9. Calculando distncias A opo correta (a).
Soluo 1: Pelo enunciado, temos AC = 50, BD = 45
e AD = 80. Da figura segue que
CD = AD AC = 80 50 = 30.
Logo,
BC = BD CD = 45 30 = 15 km.
Soluo 2: Pela figura, temos que 45 BC = 80 50. Logo, BC = 15 km.
10. Pesando caixas A opo correta (e).
Na figura podemos ver uma coluna com trs caixas, quatro colunas com duas caixas e
trs colunas com uma caixa. Logo, o total de caixas
1 3 + 4 2 + 3 1 = 14.
Como cada caixa pesa 25 kg, o peso do monte de caixas 14 25 = 350 kg.
11. Consumo de gua A opo correta (c).
Lembre que a mdia aritmtica de n nmeros um ensimo da soma desses nmeros.
Por exemplo, a mdia aritmtica dos nmeros 3, 6, 8 e 26
3 + 6 + 8 + 26
43
=
= 10,75.
4
4
Analogamente, o consumo mensal mdio a soma dos consumos mensais dividida pelo
nmero de meses. Logo, o consumo mensal mdio igual a
12,5 + 13,8 + 13,7 + 11,4 + 12,1
= 12,7 m3 .
5
OBMEP 2010

115

Solues do Nvel 1
12. Folheando um livro A opo correta (c).
Entre 1 e 100, o algarismo 5 aparece nos nmeros 5, 15, 25, 35, 45, 50, 51, 52, 53,
54, 55, 56, 57, 58, 59, 65, 75, 85 e 95. A primeira folha contm as pginas 1 e 2, a
segunda folha as pginas 3 e 4, a terceira folha as pginas 5 e 6, e assim sucessivamente.
Logo, as duas pginas que compem cada folha tm a seguinte numerao: um nmero
mpar e o nmero par seguinte. Assim, esto numa mesma folha as duplas de nmeros
49, 50; 51, 52; 53, 54; 55, 56; 57, 58; 59, 60 e nesse grupo temos seis folhas. Por outro lado,
| {z } | {z } | {z } | {z } | {z } | {z }
de 1 a 48, temos cinco folhas com as pginas 5, 15, 25, 35 e 45 e, de 61 a 100, temos
quatro folhas com as pginas 65, 75, 85 e 95. Conclumos que o total de folhas com o
algarismo 5 em sua numerao 6 + 5 + 4 = 15.
13. Calculando a soma A opo correta (c).
A partir de qualquer crculo, obtemos inicialmente a sequncia 0, 1, 2, 3, 4, 5, 6, 7, 8, 9.
Subtraindo uma unidade dos mpares e somando uma unidade aos pares, a sequncia
torna-se 1, 0, 3, 2, 5, 4, 7, 6, 9, 8. Agora fcil verificar que a maior soma possvel
com trs nmeros consecutivos 8 + 9 + 6 = 23.

14. Desenhando o cubo A opo correta (b).


Vemos que o cubo (a) igual ao cubo (e) e o cubo (c) igual ao cubo (d). Como no
podemos trocar a estrela com o crculo cheio mantendo o crculo oco no topo, vemos
que a alternativa correta (b).
15. Crculos concntricos A opo correta (c).
r

Observe que a figura simtrica em relao reta r que passa


pelo centro comum das circunferncias. Para cada regio cinza
de um lado de r existe uma regio branca equivalente do outro
lado de r, e vice-versa. Logo, a rea da regio cinza igual
rea branca. Portanto, a rea da regio cinza igual metade
da rea do crculo maior.
16. Brincando com engrenagens A opo correta (a).
A engrenagem desta questo formada por dois
discos dentados. Quando um deles gira no sentido
horrio, o outro gira no sentido anti-horrio.
116

OBMEP 2010

Solues do Nvel 1
As cinco opes da resposta mostram a bandeira do disco esquerda, numa posio que
corresponde a uma rotao desse disco no sentido anti-horrio por um certo ngulo.
Nesse caso, a engrenagem direita girou por esse mesmo ngulo no sentido horrio,
levando a bandeirinha para a posio indicada na primeira alternativa.
17. Troca de garrafas A opo correta (d).
Como 43 = 10 4 + 3, numa primeira vez as 43 garrafas vazias podem ser trocadas
por 10 garrafas cheias, sobrando ainda 3 vazias. Agora, consumindo o leite dessas
10 garrafas, ficamos com 13 vazias, que desta vez podem ser trocadas por 3 cheias,
sobrando 1 vazia. Finalmente, consumindo o leite das 3 garrafas cheias, sobram 4
vazias, que podem ser trocadas por 1 cheia. Portanto, o total de garrafas cheias de
leite que podem ser obtidas nesse processo 10 + 3 + 1 = 14.
18. Retngulo e quadrados A opo correta (c).
Soluo 1: Como os quadrados menores tm 1 m2
de rea, cada um deles tem lado igual a 1 m. Pela
figura, conclumos que BC = 2 m. Como ABCD
um quadrado, segue que BC = CD = AD = 2 m.
Sendo CDEF tambm um quadrado, temos

H
1

DE = 2 m. Pela figura, temos


AH = AB + BH = 2 + 3 + 5,
EJ = AD+DE+AJ e AJ = AH. Segue que EJ = 2+2+5 = 9. Como EG = AH = 5,
as dimenses do terreno so 9 m de comprimento por 5 m de largura. Portanto, sua
rea de 9 5 = 45 m2 .
Soluo 2: Quadriculando o retngulo maior
com quadrados de 1 m2 de rea, obtemos
o retngulo BF GH formado por 12 quadrados
de 1 m2 de rea, os dois quadrados ABCD
e DCF E formados por quatro quadrados de
1 m2 de rea e o quadrado AHIJ formado por 25
quadrados de 1 m2 de rea. Portanto, a rea pedida
de 12 + 4 + 4 + 25 = 45 m2 .
19. Quantas fatias de bolo? A opo correta (e).
Temos um total de 8 3 = 24 fatias de bolo que foram comidas. Como todos comeram
bolo, inicialmente cada um dos nove amigos comeu uma fatia, sobrando, ainda, 249 =
15 fatias para serem comidas por nove pessoas. Se todos os nove amigos tivessem
comido menos do que duas dessas 15 fatias, poderamos escrever o nmero 15 como
uma soma de nove parcelas, cada uma delas sendo 0 (os que no comeram alguma das
15 fatias) ou 1 (os que comeram uma das 15 fatias), o que claramente no possvel.
Logo, obrigatoriamente algum comeu pelo menos duas dessas 15 fatias. Como todos
j haviam comido, inicialmente, uma fatia, conclumos que algum comeu 3 fatias, no
mnimo.
OBMEP 2010

117

Solues do Nvel 1
20. Mosaicos quadrados A opo correta (a).
No primeiro mosaico, temos 3 + 3 + 1 + 1 = 8 azulejos pretos, no segundo, temos
4+4+2+2 = 12, no terceiro, temos 5+5+3+3 = 16 e no difcil perceber (e verificar)
que os prximos mosaicos tm 20 e 24 azulejos pretos, pois a cada novo mosaico so
usados mais quatro azulejos pretos, um em cada lado. Como 8 + 12 + 16 + 20 + 24 = 80,
possvel construir exatamente cinco mosaicos. Finalmente, o nmero total de azulejos
brancos nesta sequncia de cinco mosaicos
12 + 22 + 32 + 42 + 52 = 1 + 4 + 9 + 16 + 25 = 55.

21. Quanto custa? Comprando trs cadernos por 6 reais cada um, ainda sobram 4
reais para Ester, de modo que a quantia que ela possui 3 6 + 4 = 22 reais.
(a) Se o irmo lhe empresta 4 reais, ela fica ento com 22 + 4 = 26 reais e pode
comprar 2 cadernos a 6 reais cada um, sobrando 26 2 6 = 26 12 = 14 reais
para 7 canetas. Conclumos que o preo de cada caneta 14 7 = 2 reais.

(b) Como Ester possui 22 reais, se ela comprar 2 cadernos, sobram-lhe


22 2 6 = 22 12 = 10 reais. Como cada caneta custa 2 reais, ela poder
comprar 10 2 = 5 canetas.

22. Encontre o nmero O nmero 24 deve ser escrito como uma soma de trs algarismos. Inicialmente, note que os algarismos 0, 1, 2, 3, 4 e 5 no podem ser usados.
Realmente, se um deles fosse usado, por exemplo o algarismo 5, ento teramos que
encontrar dois algarismos cuja soma fosse 19, pois 24 5 = 19; mas, sabemos que isso
no possvel. O mesmo ocorre com os algarismos 0, 1, 2, 3 e 4. Logo, o nmero da
casa de Jlia s pode ser composto pelos algarismos 6, 7, 8 e 9.
(a) Se os trs algarismos so iguais, ento o nmero da casa de Jlia 888.
(b) Se apenas dois desses algarismos so iguais, esses dois algarismos devem ser iguais
a 9 e o terceiro deve ser 6, obtendo os nmeros 699, 969 e 996. De fato, com dois
algarismos 8, recamos no caso anterior e, com dois 6 ou dois 7, a soma d, no
mximo 14, restando, no mnimo, 10 para o terceiro, o que no possvel.
(c) Se os trs algarismos so distintos, ento esses algarismos so 7, 8 e 9. De fato, se
ocorrer um 6, a soma dos outros dois deve ser 24 6 = 18, portanto precisamos de
dois 9 e recamos no caso anterior. Assim, restam apenas as alternativas seguintes:
789, 798, 879, 897, 978 e 987.
23. Campeonato de futebol
(a) A equipe A disputa com as cinco equipes B, C, D, E e F ; a equipe B, alm da
partida contra A, j computada, ainda disputa quatro partidas com as equipes
118

OBMEP 2010

Solues do Nvel 1
C, D, E e F ; a equipe C, ainda disputa com as equipes D, E e F ; a equipe D
ainda disputa com as equipes E e F e a equipe E ainda disputa com a equipe F.
No total, temos 5 + 4 + 3 + 2 + 1 = 15 partidas disputadas.
Outra maneira de contar : Podemos formar grupos de duas letras e contar, lembrando que AB e BA, por exemplo, so a mesma partida: AB, AC, AD, AE,
AF, BC, BD, BE, BF, CD, CE, CF, DE, DF e EF d 15 partidas.
Outra maneira de contar : Cada uma das seis equipes disputou, com cada uma
das outras cinco, exatamente uma partida. Portanto, foram disputadas um total
1
de (6 5) = 15 partidas.
2
(b) Cada equipe disputou cinco partidas, portanto, a soma do nmero de vitrias,
empates e derrotas de cada equipe, igual a cinco. Assim, temos x + 1 + 0 = 5,
portanto, x = 4, e temos 1 + 1 + y = 5, portanto, y = 3.
Outra maneira de calcular x e y: Sabemos o nmero total de empates (que sempre
envolvem duas equipes), dado por 1 + 1 + 3 + 1 + 1 + 1 = 8. Portanto, o nmero de
vitrias (ou derrotas), igual nmero de partidas, 15, menos a metade do nmero
de empates, 4, ou seja, o nmero total de vitrias (ou de derrotas) 15 4 = 11.
Assim, 4 + 2 + 1 + x = 11, portanto, x = 4, e 2 + 2 + y + 4 = 11, portanto, y = 3.
Finalmente, o nmero total de gols marcados no campeonato igual ao nmero
total de gols sofridos, que 2 + 6 + 6 + 6 + 5 + 3 = 28. Assim,
28 = 6 + 6 + 2 + 3 + 1 + z,
ou seja, z = 10.
Resumindo, o nmero x de vitrias da equipe F 4, o nmero y de derrotas da
equipe D 3 e o nmero z de gols marcados pela equipe F 10.
24. Dividindo o paraleleppedo
(a) Em centmetros, as dimenses do bloco maior so 320 60 75 e as dos blocos
menores so 80 30 15. Logo, o comprimento foi dividido por 4 = 320 80,
a largura foi dividida por 2 = 60 30 e a altura foi dividida por 5 = 75 15.
Portanto, teremos um total de 4 2 5 peas, conforme a figura.

15
80

15
15

80

15
30

15
30

80
80

(b) O volume de um bloco dado por comprimentolarguraaltura. Logo, o volume


de cada um dos blocos menores 80 30 15 = 36 000 cm3 . Como o peso
dado em metros cbicos, devemos reduzir o volume de cada bloco para metros
cbicos. Para isso, deslocamos a vrgula seis casas para a esquerda, obtendo
36 000 cm3 = 0,036 m3 . Como 1 m3 pesa 900 kg, ento cada bloco menor de 0,036
m3 pesar 0,036 900 = 32,4 kg.
OBMEP 2010

119

Solues do Nvel 1
25. Uma calculadora A partir do nmero 1 no visor devemos aplicar sucessivamente
as operaes das teclas A e B para obter o nmero desejado. Observe que, para obter o
nmero 2 a partir do nmero 1, podemos apertar tanto a tecla A quanto a B, portanto,
A
B
em cada uma das respostas dadas, podemos trocar cada 1 2 por 1 2.
A

(a) 1 2 4 5 10.

(b) 1 2 3 6 7 14 15.
A

(c) 1 2 3 6 12 24 25 50 100.

26. Ano bissexto


(a) Uma semana tem sete dias. Na diviso de 365 por 7 encontramos quociente 52 e
resto 1. Logo, o ano comum tem 52 semanas e 1 dia. Portanto, a frase correta
O ano comum tem sete semanas e um dia. Como o ano bissexto tem 366 dias,
ele possui 52 semanas e 2 dias. Portanto, o correto dizer O ano bissexto tem
sete semanas e dois dias.
(b) Se um ano comum inicia numa tera-feira, ento a sua 52a semana inicia numa
tera e termina numa segunda, ou seja, a 52a semana dada por tera quarta
quinta sexta sbado domingo segunda. Como esse ano tem 52 semanas e
mais 1 dia, o ltimo dia deste ano ser uma tera. Logo, o ano seguinte iniciar
numa quarta.
(c) No caso do ano bissexto, devemos considerar um dia a mais do que no item
anterior. Logo, o seu ltimo dia ser uma quarta e, portanto, o ano seguinte
iniciar numa quinta-feira.
27. Nmeros triangulares Notamos que o segundo nmero triangular obtido a partir
do primeiro acrescentando-se 2, o terceiro obtido do segundo acrescentando-se 3 e
assim por diante. Essa observao nos mostra como calcular os prximos nmeros
triangulares sem fazer muitas contas. Por exemplo, j sabemos que o quarto nmero
triangular 10, donde o quinto ser 10 + 5 = 15 e o sexto sendo, ento, 15 + 6 = 21.
Assim, podemos escrever os nmeros triangulares at passar de 100.
1 |{z}
3 |{z}
6 |{z}
10 |{z}
15 |{z}
21 |{z}
28 |{z}
36 |{z}
45
+2

+3

+4

+5

+6

+7

+8

+9

45 |{z}
55 |{z}
66 |{z}
78 |{z}
91 |{z}
105
+10

+11

+12

+13

+14

Logo, os nmeros triangulares menores do que 100, so 1, 3, 6, 10, 15, 21, 28, 36, 45,
55, 66, 78 e 91. Assim, temos 13 nmeros triangulares menores do que 100.
28. Livros separados Denotando por n o nmero de livros que a bibliotecria vai colocar em cada estante, temos 130 n = nmero de estantes para os livros de Matemtica
e 195 n = nmero de estantes para os livros de Portugus. Isso mostra que n deve
ser um divisor comum de 130 e de 195, pois o nmero de estantes utilizadas inteiro.
Sabemos que, quando aumentamos o denominador de uma frao, esta frao diminui;
por exemplo, 27/10 menor do que 27/8. Logo, quanto maior for o denominador n,
menores sero as fraes 130/n e 195/n, o que significa que menor ser o nmero de
120

OBMEP 2010

Solues do Nvel 1
estantes utilizadas. Vemos, assim, que n deve ser o maior divisor comum (MDC) de 130
e 195. Como as decomposies desses nmeros em fatores primos so 130 = 2 5 13
e 195 = 3 5 13, segue que o MDC de 130 e 195 5 13 = 65.

Logo, a bibliotecria vai colocar 65 livros em cada estante, o nmero de estantes para
os livros de Matemtica 130 65 = 2 e o nmero de estantes para os de Portugus
195 65 = 3, o que d um total de 2 + 3 = 5 estantes.
29. Alunos com culos Nosso problema aqui encontrar o nmero de alunos da classe.
1 1
1
Como 1/6 dos alunos usam culos e, desses, 1/3 so meninas, temos que =
3 6
18
dos alunos so meninas que usam culos. Como
alunos que usam culos meninas que usam culos corresponde a meninos que usam culos
|
{z
} |
{z
}
{z
}
|
1
6

1
18

1
1
3
1
2
1
1
=

=
= , conclumos que da classe consiste de meninos que
e
6 18
18 18
18
9
9
usam culos, que so em nmero de 4. Portanto,
1
da classe corresponde a 4 alunos
9
9
da classe corresponde a 4 9 = 36 alunos
9
Assim, o nmero de alunos na classe 36.
30. Quadrado mgico Para facilitar nossas contas, conveniente reduzir todas as
fraes que aparecem na tabela a um mesmo denominador. Como 0,4 = 4/10 e
0,5 = 5/10, podemos reescrever a tabela como segue, em que indicamos com as letras a, b, c, d e e os nmeros que devem ser calculados.
a
b
4/10

c
5/10
5/10

6/10
d
e

Olhando para a diagonal ascendente, vemos que a soma dos elementos dessa diagonal
4/10 + 5/10 + 6/10 = 15/10. Como a soma dos elementos da terceira linha deve
ser igual a essa soma dos elementos da diagonal, obtemos 4/10 + 5/10 + e = 15/10,
donde e = 6/10. Tambm obtemos, na segunda coluna, 5/10 + 5/10 + c = 15/10, donde
c = 5/10. Colocando esses valores de c e e na tabela, obtemos
a
b
4/10

5/10
5/10
5/10

6/10
d
6/10

Agora, a primeira linha fornece a + 5/10 + 6/10 = 15/10, donde a = 4/10. Da terceira
coluna, obtemos 6/10 + d + 6/10 = 15/10, donde d = 3/10; do mesmo modo, obtemos
b = 7/10 e a tabela est completa.
OBMEP 2010

121

Solues do Nvel 1
4/10
7/10
4/10

5/10
5/10
5/10

6/10
3/10
6/10

31. Trs algarismos A partir da igualdade (AB)2 = CAB e denotando o nmero


de dois algarismos AB por x, temos x2 = (AB)2 = CAB = C 100 + x, ou seja,
x2 x = C 100. Portanto, o produto x(x 1) = x2 x divisvel por 100. Levando
em conta a fatorao 100 = 22 52 , dividimos a resoluo em trs casos, conforme a
maior potncia de 5 que divide x.
1o Caso: 52 divide x.
Como x um nmero de dois algarismos, os possveis valores de x so 25, 50 e 75.
Construmos uma tabela.
x
25
50
75

x1
24
49
74

x (x 1)
600
2 450
5 550

Portanto, o nmero x(x1) um mltiplo de 100 somente se x = 25. Como 252 = 625,
nesse caso temos C = 6.
2o Caso: 5 divide x, mas 52 no divide x.
Ento, necessariamente, 5 divide x 1, pois 5 divide x (x 1). Mas isso uma
impossibilidade, porque 5 no pode dividir os dois nmeros consecutivos x 1 e x
(lembre que os nmeros divisveis por 5 terminam em 0 ou 5). Logo, esse caso est
excludo.
3o Caso: 5 no divide x.
Ento 52 divide x1. Como no Caso 1, temos x1 = 25,50 ou 75 e os possveis valores
de x so 26, 51 e 76. Construmos uma tabela.
x
26
51
76

x1
25
50
75

x (x 1)
650
2 550
5 700

O produto x (x 1) um mltiplo de 100 somente se x = 76, mas esse caso tambm


est excludo, pois 762 = 5 776 tem mais do que trs algarismos.
Assim, a nica possibilidade A = 2, B = 5 e C = 6, com soma A + B + C = 13.
32. Pintando quadradinhos Para pintar a faixa conforme o modelo, o
retngulo padro (aquele que se repete por toda a faixa) o retngulo de 5 linhas e 4 colunas mostrado na figura. Nele, temos 7 quadradinhos pintados e 13 no pintados. Precisamos saber quantos retngulos padro cabem na faixa. A faixa tem 250 colunas e cada retngulo padro tem 4 colunas. Da diviso de 250 por 4 temos que
250 = 4 62 + 2, e conclumos que na faixa cabem 62 retngulos padro, sobrando ainda duas colunas.
122

OBMEP 2010

Solues do Nvel 1

62 retngulo
padro

1 retngulo
padro

2 colunas
a mais

Nos 62 retngulos padro temos 62 13 = 806 quadradinhos no pintados. Agora


falta verificar quais so os quadradinhos no pintados nas duas colunas finais. A figura
mostra como so as duas colunas, de acordo com o modelo. Nessas colunas temos 6
quadradinhos no pintados. Assim, o nmero de quadradinhos no pintados em toda
a faixa 806 + 6 = 812.
33. A cisterna do Joo O dia 1o de janeiro comea com 156 litros de gua na cisterna
e, a partir da, a cisterna recebe gua da chuva e perde gua para regar as flores. Como
no dia 8 no houve alterao na quantidade de gua na cisterna, o nmero de litros de
gua na cisterna no dia 8
156 + gua de chuva do dia 1 ao dia 7 gua para regar do dia 1 ao dia 7.

O enunciado diz que a segunda parcela da expresso acima a soma dos nmeros da
terceira coluna, que 2,5 + 0 + 5 + 0 + 3 + 0 + 4,5 = 15 e a terceira parcela a soma
dos nmeros da segunda coluna da tabela, que 6 + 9 + 0 + 4 + 9 + 0 + 11 = 39. Assim,
o nmero de litros na cisterna, meia noite do dia 8, 156 + 15 39 = 132.
34. O mltiplo de 13 A opo correta (a).
Como 119 268 916 divisvel por 13, j que 9 174 532 13 = 119 268 916, podemos
concluir que os nmeros divisveis por 13 so aqueles obtidos somando ou subtraindo
mltiplos de 13 ao nmero 119 268 916. Dentre os nmeros apresentados, o nmero
119 268 916 13 = 119 268 903
o nico divisvel por 13.
35. Um bilho A opo correta (e).
Arnaldo disse que 1 bilho = 1 000 000 1 000 000 = 1 000 000 000 000 = 1012 . O
Professor Piraldo corrigiu-o, dizendo que 1 bilho = 1 0001 000 000 = 1 000 000 000 =
109 . A diferena
1 000 000 000 000 1 000 000 000 = 999 000 000 000.
36. Energia de abelha A opo correta (b).
A energia gasta por uma abelha para voar 7 000 quilmetros a mesma que 7 000
abelhas gastam para voar 1 quilmetro cada. Como o nmero de litros de mel foi
multiplicado por 10, temos energia suficiente para que 10 vezes esse nmero de abelhas
voem 1 quilmetro cada, ou seja, 70 000 abelhas.
OBMEP 2010

123

Solues do Nvel 1
37. Perda de safra A opo correta (a).
Como um quinto de 100 000 15 100 000 = 20 000 e um quarto de 100 000 14 100 000 =
25 000, conclumos que a perda da safra est avaliada entre 20 000 e 25 000 reais. Logo,
um possvel valor para a perda de R$ 21 987,53.
38. Placa decorativa A opo correta (c).
Traando paralelas aos lados, podemos dividir a placa
em quadrados de 1 metro de lado, conforme indicado
na figura. Ento, a rea pintada igual a 12 metades
desses quadrados, ou, equivalentemente, 6 desses quadrados. Como a placa total tem 16 desses quadrados,
conclumos que a frao da rea pintada em relao
3
6
= .
rea da placa
16
8

1m
1m
1m
1m
1m
1m

39. O suco do Diamantino A opo correta (a).


O refresco composto por 20% de um litro, ou seja, 0,2 litros de suco e por 80% de
um litro, ou seja, 0,8 litros de gua. Logo, a mistura final tem 0,2 litros de suco e
3 + 0,8 = 3,8 litros de gua. A porcentagem de suco em relao ao volume da mistura
, ento,
volume de suco
0,2
2
5
=
=
=
= 5%.
volume total
4
40
100
40. Uma eleio Joo recebeu 2/7 do total de votos, Rosa recebeu 2/5 do total de
votos e Marcos recebeu 1 2/7 + 2/5 = 1 24/35 = 11/35 do total de votos. O
vencedor foi aquele que obteve a maior frao dos votos. Para comparar essas fraes,
igualamos seus denominadores, obtendo 2/7 = 10/35 e 2/5 = 14/35. Assim, temos
2
11
2
<
<
,
7
35
5
|{z}
|{z}
|{z}
Joo Marcos Rosa

e, portanto, Rosa venceu a eleio. ( interessante notar que a resposta no depende


do nmero de alunos da turma.)
41. Soma de potncias A opo correta (a).
Temos 26 + 26 + 26 + 26 44 = 4 26 44 . H vrias maneiras de calcular isso.
Soluo 1: 4 26 44 = 4 (22 )3 44 = 4 43 44 = 44 44 = 0.





Soluo 2: 4 26 44 = 4 26 43 = 4 26 (22 )3 = 4 26 26 = 0.

Soluo 3: 4 26 44 = 22 26 (22 )4 = 28 28 = 0.
42. Seis retngulos A opo correta (e).
124

OBMEP 2010

Solues do Nvel 1
A partir da figura, vemos que o comprimento a dos
retngulos menores o dobro da sua largura b, isto
, a = 2b. Temos, ento,

a + b = 2b + b = 3b = 21,
ou seja, b = 7 cm e a = 14 cm. Portanto, o comprimento do retngulo maior 4b = 28 e sua rea
21 28 = 588 cm2 .

b
a

43. Duas populaes A opo correta (e).


Seja p a populao de Tucupira h trs anos. Como essa populao cresceu 50%,
atualmente Tucupira tem p + 50% de p habitantes, ou seja,
p+

50
p = p + 0,5 p = 1,5 p habitantes.
100

Como a populao de Pirajussara no cresceu nesses 3 anos e h 3 anos era igual


de Tucupira, podemos concluir que a populao atual de Pirajussara p. Como a
soma das populaes das duas cidades, hoje, de 9 000, obtemos p + 1,5 p = 9 000,
donde p = 9 000/2,5 = 3 600. Assim, a soma das duas populaes, h trs anos, era de
3 600 2 = 7 200 habitantes.
44. Trs balanas A opo correta (d).
Na primeira balana temos 3N + 1 = 6. Na segunda, temos 2N + 4 = 8, o
que equivalente a 1N + 2 = 4. Logo, (3N + 1) + (1N + 2) = 6 + 4, ou seja,
4N+3 = 10. Assim, ser necessrio colocar 10 quadrados no prato direito da balana
(3) para que ela fique equilibrada.
45. Poucos domingos A opo correta (c).
Um ano normal tem 365 dias e o ano bissexto 366. Da diviso de 365 por 7, obtemos
365 = 52 7 + 1 e da diviso de 366 por 7 obtemos 366 = 52 7 + 2. Logo,
ano normal = 52 semanas + 1 dia
ano bissexto = 52 semanas + 2 dias
Portanto, um ano normal ou bissexto tem, no mnimo, 52 e, no mximo, 53 domingos
(um domingo para cada uma das 52 semanas e, talvez, um outro domingo para o dia
ou os dois dias que completam o ano).
Cada um dos 12 meses do ano tem, no mnimo, 28 dias e, no mximo, 31 dias, portanto,
tem, no mnimo, 4 domingos e, no mximo, 5 domingos. Levando em conta que
12 4 = 48, conclumos que
i) Num ano de 52 domingos sobram ainda 52 48 = 4 domingos. Cada um desses
ficar num ms diferente, porque nenhum ms pode ter seis domingos; logo, temos
quatro meses com 5 domingos.
ii) Analogamente, num ano com 53 domingos restam 5 domingos, que ficaro um em
cada ms diferente. Portanto, nesse caso, teremos cinco meses com 5 domingos.
OBMEP 2010

125

Solues do Nvel 1
46. Metade de potncia A opo correta (e).
Antes de dividir a expresso
por 2, colocamos 210 em evidncia, obtendo

12
10
10 2
2 + 3 2 = 2 2 + 3 1 = 210 7. Assim,
212 + 3 210
210 7
=
= 29 7.
2
2

47. Minutos demais A opo correta (d).


Dividindo 2 880 717 por 60, obtemos 2 880 717 = 48 011 60 + 57. Isso significa que
2 880 717 min = 48 011 h + 57 min. Podemos, ento, escrever:
2 880 717 min = 48
{z h} + 11 h + 57 min.
| 000
2 000 dias

Os 2 000 dias no interferem no horrio que estamos procurando, e como 18 horas e 27


minutos so exatamente 17 horas e 87 minutos, a resposta 18h87min 11h57min =
6h30min.
48. Dois nibus A opo correta (b).
O nmero total de alunos nos dois nibus 57 + 31 = 88 e 12 88 = 44. Para que cada
nibus tenha o mesmo nmero de alunos, 5744 = 13 alunos devem passar do primeiro
para o segundo nibus.
49. Cubo de papelo A opo correta (e).
Com as peas ilustradas ao lado podemos construir um cubo.
50. Algarismo das unidades A opo correta (c).
O ltimo algarismo de um mltiplo de 5 0 ou 5; os que terminam em 0 so pares e
os que terminam em 5 so mpares. Como 1 3 5 97 99 mpar, por ser
um produto de nmeros mpares, e um mltiplo de 5, segue que seu algarismo das
unidades 5.
51. Regio sombreada A opo correta (b).
A parte sombreada consiste em 10 metades de quadrados mais 3 quadrados inteiros, o
que equivale a 21 10 + 3 = 5 + 3 = 8 quadrados inteiros. Logo, a frao que representa
a parte sombreada
rea sombreada
rea de 8 quadrados
8
4
=
=
= .
rea total
rea de 18 quadrados
18
9
52. Colorindo um mapa A opo correta (b).
O estado A pode ser pintado de trs formas: verde, azul ou amarelo. Para qualquer
estado vizinho, por exemplo, o estado B, temos duas possibilidades, e os demais estados tm suas cores determinadas. Logo, podemos colorir o mapa de 32 = 6 formas.

126

OBMEP 2010

Solues do Nvel 1
Abaixo ilustramos duas dessas maneiras de pintar o mapa; em ambas, o estado A tem
a mesma cor.

53. Pintando um tabuleiro A opo correta (c).


Para satisfazer as condies do problema, as cinco casas das diagonais, marcadas com *,
devem ter cores diferentes. Por isso, precisaremos de, no mnimo, cinco cores distintas.
Denotemos essas cinco cores distintas por 1, 2, 3, 4 e 5 e vamos determinar como
podemos escolher as cores para as quatro casas restantes de modo a satisfazer as
condies pedidas. Uma maneira dada direita, a seguir.
*

*
*

3
*

1
4
2

2
3
4

4
1
5

Logo, possvel pintar as quatro casas restantes sem utilizar mais cores. Assim, bastam
cinco cores. A seguir, mostramos outras trs maneiras de colorir as casas.
2

3
1

2
4
5

4
1
2

3
2
4

1
4
3
5

2
4

54. Nmero X,Y Temos X,Y = X +

1
3
4

3
2
1

1
2
3

3
4
2

2
1
5

5
4
2

Y
10X + Y
=
e sabemos que
10
10

10X + Y
3
= X, Y = (X + Y ).
10
10
Logo, 10X + Y = 3X + 3Y, ou seja, 7X = 2Y. Conclumos que 2Y mltiplo de 7 e,
como Y um nmero inteiro entre 1 e 9, s temos a possibilidade Y = 7, donde X = 2.
Assim, o nmero 2,7.
55. Construo de casas Como as casas so vizinhas, podemos pensar nelas como
uma fila de casas com seis posies. Vamos dividir a contagem em casos, de acordo
com o nmero de casas de madeira que podem ser construdas.
(a) Nenhuma casa de madeira: aqui h apenas uma maneira de construir as casas,
ou seja, todas de alvenaria.
OBMEP 2010

127

Solues do Nvel 1
(b) Uma casa de madeira: aqui temos seis maneiras de construir as casas, pois a casa
de madeira pode ser qualquer uma delas, sendo as outras de alvenaria.
(c) Duas casas de madeira: as casas de madeira podem ocupar as seguintes posies:
1 e 3, 1 e 4, 1 e 5, 1 e 6, 2 e 4, 2 e 5, 2 e 6, 3 e 5, 3 e 6 ou 4 e 6. Aqui temos 10
maneiras.
(d) Trs casas de madeira: as casas de madeira podem ocupar as seguintes posies:
1, 3 e 5; 1, 3 e 6; 1, 4 e 6; 2, 4 e 6. Aqui temos quatro maneiras.
(e) Quatro ou mais casas de madeira: impossvel, pois fcil ver que, nesse caso,
sempre teremos duas casas de madeira contguas.
Dessa forma, h 1 + 6 + 10 + 4 = 21 maneiras de planejar a construo.
56. Comparao de grandezas A opo correta (c).
Temos 1 000 + 0,01 = 1 000,01 e 1 000 0,01 = 1 000

1
= 10, bem como
100

1 000 1 000
= 1 = 1 000 100 = 100 000
0,01
100
e 0,01/1 000 = 0,00001. Finalmente, 1 000 0,01 menor do que 1 000 (no sendo preciso efetuar o clculo para obter esta concluso), de modo que o maior desses nmeros
1 000/0,01.
57. Maior nmero de seis algarismos A opo correta (c).
Soluo 1: Para que seja o maior possvel, o nmero deve comear com o maior
algarismo. Para termos seis algarismos sem mudar a ordem, o maior 8 e, depois, 7.
Agora faltam quatro algarismos para completar o nmero, portanto, escolhemos 9 103.
Logo, o nmero 879 103.
Soluo 2: As opes D e E no servem, pois a ordem foi alterada. Como nas opes
A, B e C no foi alterada, basta escolher o maior nmero dentre essas opes, que C.
58. Qual o numerador? A opo correta (a).
Como

1
4
1
6
=
e = , ento n s pode ser igual a 5.
6
24 4
24

59. Correndo menos A opo correta (a).


km
km
1
6 min = 10
6
h = 1 km.
h
h
60
Percorrendo essa mesma distncia de 1 km em 8 minutos, a velocidade ser
Soluo 1: A distncia percorrida d = 10

v=

1 km
1 km
60
15
=
km/h =
km/h = 7,5 km/h .
=
1
8 min
8
2
8 60 h

Soluo 2: Podemos usar diretamente a regra de trs, como segue.


128

OBMEP 2010

Solues do Nvel 1
Velocidade em km/h
10

Tempo em horas
6
60
8
60

Velocidade e tempo so grandezas inversamente


 8  proporcionais (aumentando a veloci6
dade, diminui o tempo), logo x/10 = 60
/ 60 = 6/8, portanto, x = 60/8, ou seja, a
velocidade ser
15
60
km/h =
km/h = 7,5 km/h .
x=
8
2
60. Cinco vizinhas Helosa chega a seu andar depois de Elza, mas antes de Cludia
significa que Helosa mora acima de Elza e abaixo de Cludia e Quando Sueli chega
ao seu andar, Helosa ainda tem 2 andares para subir, e o mesmo ocorre a Patrcia
quando Elza chega ao seu andar significa que Helosa mora dois andares acima de Sueli
e Patrcia dois andares acima de Elza. Entretanto, como Sueli no mora no primeiro
andar e Helosa mora dois andares acima de Sueli, ou Sueli mora no segundo e Helosa
no quarto ou Sueli mora no terceiro e Helosa no quinto. Mas
5o andar Cludia
Cludia mora acima de Helosa, portanto Helosa no pode
4o andar Helosa
morar no ltimo andar, o quinto. Assim, Sueli mora no se3o andar Patrcia
gundo andar, Helosa no quarto e Cludia no quinto. Final2o andar
Sueli
mente, Patrcia mora dois andares acima de Elza, logo Elza
o
1 andar
Elza
mora no primeiro andar e Patrcia no quarto andar.
61. Potncias de 9 A opo correta (d).
920 + 920 + 920 = 3 920 = 3 32
62. Dois nmeros A opo correta (c).

20

= 3 340 = 341 .

Como a diferena 989 e o menor nmero tem dois algarismos (sendo, portanto, maior
do que 9), o nmero de trs algarismos deve ser maior do que 989 + 9 = 998, de modo
que a nica opo 999. Assim, o nmero de dois algarismos 10 e a soma dos dois
999 + 10 = 1 009.
63. Menor natural A opo correta (d).
Observe que 10n 1 um nmero que tem todos os seus algarismos iguais a 9. Note,
tambm, que um mltiplo de 37, da forma 37 n, s termina em 9 se n terminar em
7. Ento, os menores mltiplos de 37 terminados em 9 so 37 7 = 259, 37 17 = 629
e 37 27 = 999. Como 999 = 103 1, segue que n = 3.
64. Imunes gripes A opo correta (a).
Contraram a gripe 0,15% de 14 000 000, ou seja,
0,15
140 000 000 = 0,0015 14 000 000 = 21 000
100
pessoas. Portanto, no contraram a gripe 14 000 000 21 000 = 13 979 000 pessoas.
OBMEP 2010

129

Solues do Nvel 1
65. O cdigo secreto A opo correta (b).
O cdigo s pode ser formado com os algarismos 1, 2, 3, 4, 5, 6, 7, 8, e 9.
Da primeira informao temos que 1, 2 e 3 no fazem parte do cdigo
(nmeros que no fazem parte esto sublinhados nas tabelas). Da terceira informao, conclumos que 6 faz parte do cdigo, e sua posio
___6___ ou ___ ___6.

1
4
6
5
8

2
5
1
4
4

3
6
2
7
3

Da segunda informao segue que 4 e 5 no fazem parte do cdigo e a


posio do 6 no cdigo ___ ___6. Da ltima informao s temos
que o cdigo da forma 8 ___6. Com a quarta informao completamos
o cdigo: 876.

1
4
6
5
8

2
5
1
4
4

3
6
2
7
3

66. Parnteses, colchetes e chaves A opo correta (e).


As ordens de prioridade para resolver uma expresso so
parnteses colchete
| {z } chaves
| {z }
| {z }
o
o
2

1o

multiplicaes e divises somas e subtraes .


|
{z
}
{z
}
|
2o

1o

Assim,

(
)

)

= 2 2 2 2 2 2| {z
2 2 2 2 2 2(4| {z
2})
2}
(

= 2 2 2 2 2| {z
4}

)

= 2 2 2 2 (2)
| {z }
2
4




= 2 2 2 (4) = 2 2 2| {z
+ 4} = 2 2| {z
6} = 2 12 = 10.
6

12

67. Ordenando fraes A opo correta (a).


Soluo 1: O mnimo mltiplo comum (MMC) dos denominadores 30. Reduzindo
todas as fraes a esse denominador comum, temos
4
40
= ,
3
30
Ordenando,

Conclumos que

130

4
24
= ,
5
30

4
20
= ,
6
30

3
18
= ,
5
30

6
36
=
5
30

12
18
20
24
36
40
<
<
<
<
<
.
30
30
30
30
30
30
2
3
4
4
6
4
< < < < < .
5
5
6
5
5
3
OBMEP 2010

2
12
=
.
5
30

Solues do Nvel 1
Soluo 2: Escrevendo as fraes na forma decimal, temos
4
= 1,33 . . . ,
3
Logo,

4
= 0,8,
5

4
= 0,66 . . . ,
6

3
= 0,6,
5

6
= 1,2 e
5

2
= 0,4.
5

2
3
4
4
6
4
<
<
<
<
<
.
5
5
6
5
5
3
|{z}
|{z}
|{z}
|{z}
|{z}
|{z}
0,4

0,6

0,66...

0,8

1,2

1,33...

68. Nmeros de trs algarismos A opo correta (e).

Por serem maiores do que 200, seus algarismos das centenas s podem ser 3 ou 5.
Comeando com 3, temos 315 e 351 (que no repetem algarismos) e 311, 313, 331, 335,
353, 333 e 355 (repetindo algarismos), ou seja, nove nmeros.
Comeando com 5, basta trocar o 3 com o 5 nos nmeros acima. Logo, temos 9 desses
nmeros. Assim, temos um total de 18 nmeros que podem ser escritos usando apenas
os algarismos 1, 3 e 5.
69. Velocidade de maratona A opo correta (d).
O tempo que o vencedor gastou foi de
13h45min 11h330min = 2h15min =2 +

9
1
h = h.
4
4

Logo, a velocidade mdia, em km/h,


espao percorrido em km
42
168
= 9 =
= 18,6 km/h.
tempo gasto em horas
9
4
70. Bilhetinhos com nmeros A opo correta (c).
Se todas as alunas escrevessem o nmero 1, o produto seria 1, que no est entre
as opes. Logo, 2 ou 4 so fatores do produto e, por isso, o produto deve ser uma
potncia de 2. O maior produto possvel seria obtido no caso em que todas as 5 alunas
escrevessem o nmero 4, e o produto seria
4 4 4 4 4 = 45 = 210 = 1 024.
Logo, podemos eliminar 2 048. Agora temos que:
100 e 120 so divisveis por 5, logo no so potncias de 2;

768 divisvel por 3 (7 + 6 + 8 = 21), logo no potncia de 2.


A nica resposta possvel 256 = 28 . Seria, por exemplo, o caso em que duas alunas
escrevessem o nmero 2 e trs escrevessem o nmero 4, com 256 = 2 2 4 4 4.
71. Produto de fraes A opo correta (d).

1 
1
1 
1 
1 1 2 3 4
1
= =
1
1
1
2
3
4
5
2 3 4 5
5
OBMEP 2010

131

Solues do Nvel 1
72. Produto mximo A opo correta (a).
Basta examinar os produtos dos nmeros naturais cuja soma 11.
e
e
e

11 = 1 + 10
11 = 3 + 8
11 = 5 + 6

1 10 = 10
3 8 = 24
5 6 = 30

11 = 2 + 9
11 = 4 + 7

e
e

2 9 = 18
4 7 = 28

73. Quem o cubo? A opo correta (c).


Temos 3m = 81 = 34 , donde m = 4. Logo, m3 = 43 = 4 4 4 = 64.
74. Qual o maior? A opo correta (c).
Somando 3 a todos os membros, obtemos a 1 + 3 = b + 2 + 3 = c 3 + 3 = d + 4 + 3,
de modo que a + 2 = b + 5 = c = d + 7, mostrando que c o maior dos nmeros.
75. Quatro formiguinhas A opo correta (b).
O trajeto de Biloca 3 diagonais + 4 larguras + 2 comprimentos. O trajeto de Pipoca
de 25 dm compreende 5 diagonais, logo o comprimento de uma diagonal 25 5 =
5 dm. O trajeto de Tonica de 37 dm compreende 5 diagonais mais 4 larguras da lajota,
ou seja, 25 + 4 larguras = 37, donde 4 larguras = 37 25 = 12 dm e a largura de
uma lajota 3 dm. O trajeto de Cotinha de 32 dm compreende 5 comprimentos + 4
larguras, ou seja, 5 comprimentos + 12 = 32, donde 5 comprimentos = 32 12 = 20
e o comprimento de uma lajota 4 dm. Assim, Biloca percorre
3 diagonais + 4 larguras + 2 comprimentos = 15 + 12 + 8 = 35 dm.
|
{z
} | {z } |
{z
}
35

43

24

76. Trocando figurinhas A moeda de troca de Guilherme so as figurinhas de aranha,


portanto calculamos o valor-aranha das figurinhas que Clia quer trocar, usando as
informaes dadas.
(a)

(e)

(d)

aranha
4 borboleta = 12
periquito = 72
| tubaro
{z } = 24
{z
} | {z }
|
43

122

(e)

(d)

5 tubaro = 10 periquito = 30
aranha
{z
} | {z }
|
103

52

(b)

(d)

3 cobra = 9 periquito = 27
aranha
| {z } | {z }
33

93

243

(c)

6 macaco = 24
| aranha
{z }
64

(d)

6 periquito = 18
| aranha
{z }
63

Logo, Clia receber 72 + 30 + 24 + 27 + 18 = 171 figurinhas de aranha.


77. Soma de fraes A opo correta (d).

10 + 20 + 30 + 40
10
100
10
+
=
+
= 10 + 0,1 = 10,1
10
10 + 20 + 30 + 40
10
100
132

OBMEP 2010

Solues do Nvel 1
78. Geometria com palitos A opo correta (c).
Para o tringulo foram usados 6 3 = 18 palitos, sobrando, ento, 60 18 = 42
palitos para formar os trs lados do retngulo. Da figura, temos que a largura do
retngulo formada por seis palitos, logo o comprimento formado por 21 42 6) = 18
palitos. Como cada palito tem 5 cm de comprimento, a rea do retngulo dada por
5} = 30 90 = 2 700 cm3 .
6| {z
5} 18
| {z
largura

comprimento

79. Um incndio e o bombeiro A opo correta (c).

O sobe-desce do bombeiro a partir do degrau do meio at chegar ao ltimo degrau


dado por
sobe
sobe sobe
z}|{
z}|{ z}|{
+5 |{z}
7 +8 +7 ,
desce

de modo que o bombeiro sobe 8 + 5 = 13 degraus acima do degrau do meio, chegando


ao ltimo degrau da escada. Portanto, a escada tem 13 degraus acima do degrau do
meio, e igualmente 13 degraus abaixo do degrau do meio. Portanto, a escada tem
13 + 1 + 13 = 27 degraus. Veja um esquema da movimentao do bombeiro.
(3)sobe 8
(1)sobe 5

ltimo
degrau

Degrau
do meio
(2)sobe 7

(4)sobe 7

80. rvore genealgica A opo correta (c).


Na figura vemos que o pai de Evaristo Jos. O irmo de Jos Jean. O pai de Jean
Lus. O irmo de Lus Andr.
irmo do pai de Evaristo = irmo de Jos = Jean
|
{z
}
Jos
pai do irmo do pai de Evaristo = pai de Jean = Lus
|
{z
}
|
{z Jos
}

Jean
irmo do pai do irmo do pai de Evaristo = irmo de Lus = Andr
|
{z
}
|
{z Jos
}
|

{z Jean
Lus

OBMEP 2010

133

Solues do Nvel 1
81. Colcha quadrada A opo correta (b).
A colcha formada de 5 5 = 25 quadradinhos, todos iguais. J os tringulos so
de dois tipos, o tipo I, que corresponde a meio quadrado e o tipo II, que corresponde
a 1/4 de um quadradinho. A parte em cinza composta de 8 tringulos do tipo I, 8
tringulos do tipo II e 4 quadrados, ou seja,
8 tringulos tipo I + 8 tringulos tipo II +4 quadrados = 10 quadrados.
|
{z
} |
{z
}
4 quadrados
2 quadrados

Logo, a frao correspondente parte cinza

10
40
=
= 40%.
25
100

82. Falsas igualdades A opo correta (e).


Nenhuma igualdade est correta.
(i) Errada: 3 106 + 5 102 = 3 000 000 + 500 = 3 000 500 6= 8 108 .
1
1
(ii) Errada: 23 + 23 = 23 + 3 = 8 + 6= 1 = 20 .
2
8
(iii) Errada, a multiplicao precede a soma: 5 8 + 7 = 40 + 7 = 47 6= 75.
(iv) Errada, a diviso precede a soma: 5 + 5 5 = 5 + 1 = 6 6= 2.

83. Menor valor da soma A opo correta (c).


Seja N o nmero dado por N = 3a = 4b = 7c. Ento, o nmero N um mltiplo de
3, 4 e 7. Portanto, quando fatoramos o nmero N em fatores primos, aparecem, pelo
menos, os fatores 2, 3 e 7, o primeiro dos quais com expoente, no mnimo, igual a 2.
Segue que N um mltiplo de 22 37 = 84. Por outro lado, os nmeros a = 47 = 28,
b = 3 7 = 21 e c = 4 3 = 12 satisfazem as igualdades 3a = 4b = 7c. Logo, a = 28,
b = 21 e c = 12 so os menores valores possveis para a, b e c e 28 + 21 + 12 = 61 o
menor valor possvel para a + b + c.
84. Procurando um quadrado perfeito A opo correta (d).
Fatorando 120 obtemos 120 = 23 3 5. Para obter um quadrado perfeito, todos os
expoentes dessa decomposio devem ser pares, logo basta multiplicar 120 por
2 3 5 = 30.
De fato, temos, 120 30 = 23 3 5 2 3 5 = 24 32 52 = (22 3 5)2 = 602 .
85. Visitas num museu A opo correta (c).
Observe que os nicos algarismos que no aparecem no nmero 1 879 564 so 0, 2 e
3. O prximo nmero com todos os algarismos distintos ocorrer quando mudar o
algarismo das centenas e tivermos 1 879 6 . Logo, o menor nmero ser 1 879 602 e
ainda faltam 1 879 602 1 879 564 = 38 visitantes.
86. Ligando nmeros por flechas A opo correta (e).
O caminho-padro o que se repete, a saber,
, formado por seis flechas,
sempre comeando nos mltiplos de 6, ou seja, em 0, 6, 12, etc. Vamos averiguar qual
134

OBMEP 2010

Solues do Nvel 1
a posio de 1 997 em relao ao mltiplo de 6 mais prximo. Dividindo 1 997 por 6,
obtemos 1 997 = 6 332 + 5, correspondendo a 336 caminhos-padro mais o resto de
5 flechas. Portanto, 1 998 mltiplo de 6 mais prximo de 1 997, ocupando a primeira
posio no caminho-padro. Assim,
1998

1999

1997

2000

o caminho que ocorre entre 1 997 e 2 000.


87. Mltiplos de 9 Um nmero s um mltiplo de 9 se a soma dos seus algarismos
for um mltiplo de 9.
(a) O nmero deve ter 9 algarismos iguais a 1, ou seja, 111 111 111.
(b) Devemos usar o maior nmero possvel de algarismos iguais a 2, todos ficando nas
casas mais direita. Assim, o menor nmero 12 222.
88. A florista Se a florista vender as flores sem desidrat-las, ela vai apurar um total
de 49 1,25 = 61,25 reais. O peso das flores depois da desidratao
1

2
5
49 = 49 = 14 kg.
7
7

Logo, vendendo as flores desidratadas, ela apura um total de 14 3,25 = 45,50 reais.
Assim, a florista ganha mais no processo sem a desidratao.
89. Divisores Como 2, 3, 5 e 7 so primos, os divisores do nmero N = 2a 3b 5c 7d
so os nmeros da forma 2m 3n 5p 7q , com 0 m a, 0 n b, 0 p c e
0 q d. Portanto, N tem (a + 1) (b + 1) (c + 1) (d + 1) divisores. Decompondo
378 em fatores primos, encontramos 378 = 2 33 7, portanto queremos a, b, c e d tais
que
(a + 1) (b + 1) (c + 1) (d + 1) = 2 33 7.
Por outro lado, para N ser mnimo, os expoentes devem ser ordenados do maior para
o menor, isto , a b c d.

Afirmamos que d > 0, pois se d = 0 ento a + 1, b + 1 ou c + 1 tem dois fatores maiores


do que 1. Se a + 1 = mn, com m n > 1, temos que
2a = 2mn1 = 2m1 2mnm = 2m1 (2m )n1 2m1 8n1 > 2m1 7n1 ,
onde na penltima desigualdade usamos o fato que m 3. Assim, temos que
2a 3b 5c 7d > 2m1 3b 5c 7n1

e, portanto, encontramos um nmero com a mesma quantidade de divisores, mas menor. O argumento igual no caso em que b + 1 ou c + 1 tem dois fatores. Assim, d 1
e restam somente as possibilidades dadas na tabela seguinte.
OBMEP 2010

135

Solues do Nvel 1
a
20
13
8
6

b
2
2
6
5

c
2
2
2
2

d
1
2
1
2

(a + 1)(b + 1)(c + 1)(d + 1) = 378


21 3 3 2
14 3 3 3
9732
7633

Por ltimo, como

27
220 32 52 71
=
> 1,
213 32 52 72
7

213 32 52 72
25 7
=
>1
28 36 52 71
34

28 36 52 71
22 3
=
> 1,
26 35 52 72
7

temos que o valor de N 26 35 52 72 . Portanto, a = 6, b = 5, c = 2 e d = 2.


90. O produto dos algarismos
(a) Como 12 = 2 6 = 4 3 = 2 2 3, devemos utilizar os algarismos 1, 2, 3, 4 e
6 cujos produtos sejam 12. Assim, temos:
nmeros com 2 algarismos: 26,
nmeros com 3 algarismos:
com os algarismos 1, 2 e 6:
com os algarismos 1, 3 e 4:
com os algarismos 2, 2 e 3:

62, 34 e 43;
126, 162, 216, 261, 612 e 621;
134, 143, 314, 341, 413 e 431;
223, 232 e 322.

(b) Se P (n) = 0, ento o produto de seus algarismos igual a zero e, portanto, pelo
menos um dos algarismos do nmero n zero. De 1 a 199 temos 18 nmeros com
zero s nas unidades, 9 nmeros com zero s nas dezenas e ainda o nmero 100,
totalizando 28 nmeros:
10, 20, . . . , 90, 110, . . . , 190, 101, 102, . . . , 109 .
{z
} |
{z
}
|
0 s nas unidades

0 s nas dezenas

(c) Queremos encontrar os inteiros positivos menores do que 200, cujo produto dos
algarismos seja maior do que 37 e menor do que 45. Por exemplo, 58 um desses
nmeros, porque 58 = 40. Em primeiro lugar, note que no existem nmeros cujo
produto dos algarismos seja 38, 39, 41, 43 e 44, porque esses nmeros possuem um
fator primo maior do que 10 e, portanto, no podem ser escritos como produto de
dois ou trs algarismos. Logo, restam apenas 40 e 42. Assim, os nmeros menores
do que 200 cujo produto dos algarismos
40 so 58, 85, 158 e 185;
42 so 67, 76, 167 e 176.

(d) O valor de P (n) o maior possvel quando n = 99 ou n = 199, quando


P (99) = P (199) = 81.
136

OBMEP 2010

Solues do Nvel 1
91. Suco de laranja Se Davi comprar seis garrafas individualmente, ele gastar
6 2,80 = 16,80 reais, que um valor maior do que o preo de uma caixa com
seis. Portanto, ele deve comprar a maior quantidade possvel de caixas. Nesse caso,
como 22 = 3 6 + 4, ele deve comprar trs caixas e quatro garrafas individualmente,
caso em que gastar 3 15 + 4 2,80 = 56,20 reais, que o mnimo possvel.
92. A casa da Rosa Como o quarto quadrado, com uma rea de 16 m2 , suas
dimenses so 4 4 m. Da mesma forma, as dimenses do quintal quadrado so
2 2 m. A sala tem uma rea de 24 m2 e uma dimen- .........................................................................................................................................................................................
...
...
...
...
...
4
6
...
..
so igual do quarto; portanto, as dimenses da sala so .........
...
...
...
...
...
.
...
...
...
...
6 4 m. Assim, as dimenses totais da casa so 10 6 ...... Quarto ......4 Sala
..
...
..
...
...
...
2
..
...
...
m e a rea total da casa de 60 m . Logo, a cozinha tem ......
..
..
..........................................................................................................................................................................................
..
..
..
...
...
...
uma rea de
.
... Quintal..
...
.
2

Cozinha

...
...
...
...
...
..
..
...
.
....................................................................................................................................................................................

60 16 24 4 = 16 m .

93. O passeio do Matias Observe que h 12 ruas, ou seja, lados de 100 metros, entre
os quatro quarteires. Tambm h quatro esquinas, marcadas com na figura, em
que se encontram trs ruas. Sempre que Matias passar por uma dessas quatro esquinas,
......................................................................................

...
usar duas dessas trs ruas. Assim, pela regra que ele mesmo ...........................................................................................
...
...............................................
............
..
..
..
...
...
...
...
...
...
.
.
.
...
.
.
.
.
.
..
..
..
se imps, quando voltar a passar numa dessas quatro esquinas, ..... .....
...
...
...
...
...
...
...
...
..
...
...
...
...
...
...
termina o passeio. Portanto, em todo caminho que percorrer, ..... ....................................................... ....................................................... .........
..
..
h, pelo menos, duas dentre essas quatro esquinas em que .......... ..................................................... ..................................................... ..........
...
...
..
...
...
..
...
...
...
...
...
...
...
...
no usou todas as ruas que chegam a essas esquinas. Assim, o ......... .........
..
..
..
.
..
..
..
..
...
..
...
...
...
...
...
...
...
...
...
...
...
caminho de comprimento mximo usa no mximo 10 ruas, ou ......
...........................................
...........................................
...

...
...
................................................................
.....r
..................................................................
seja, tem um total de 1 000 m. Na figura desenhamos um dos
P
trajetos mximos.
94. O adesivo oficial Como o quadrado pintado da cor azul pode estar em qualquer
lugar, temos seis possveis formas de escolher a posio desse quadrado. Entre os cinco
quadrados restantes, precisamos pintar dois de amarelo, o que podemos fazer de 10
maneiras. Os trs quadrados restantes so pintados de verde. Portanto, o prefeito tem
6 10 = 60 formas diferentes de escolher o adesivo.
95. Adio de nmeros Efetuando a adio
1 1 1

a 000
a 998
+ a 999
................................................................

 997
encontramos  997 = 22 997, onde  = a + a + a + 1. Logo, 22 = a + a + a + 1 e,
portanto, a = 7.
96. Cubo perfeito e divisibilidade Um cubo perfeito um nmero da forma a3 , onde
a um natural. Como 94 = (32 )4 = 38 , os cubos perfeitos que dividem 38 so 1, 33 e
(32 )3 = 36 .
OBMEP 2010

137

Solues do Nvel 1
97. Localizao de um ponto O ponto indicado est quatro marcas direita de 19.
Entre 19 e 20 aparecem subdivises em 10 partes iguais, portanto, cada marca equivale
a 0,1 nessa escala. Assim, o ponto indicado 19,4.

18

19

20

98. Clculo de porcentagem Temos 58 acertos em 84 questes, portanto, a razo de


58
acertos
. Dividindo 58 por 84, encontramos, aproximadamente, 0,69047 em 1, ou
84
69,047 em 100. Logo, o percentual , aproximadamente, 69,047%.
99. Comparao de algarismos Os nmeros que estamos procurando so maiores do
que 400 e menores do que 600, portanto, o algarismo das centenas s pode ser 4 ou 5.
Como so nmeros ascendentes, o algarismo das dezenas menor do que o algarismo
das unidades. Vejamos como escolher os algarismos das dezenas e das centenas.

56

57
4
58

59

67
68
4

69

67
68
5

69

78
79

78
79

89

89

Logo, temos 10 nmeros ascendentes com algarismo das centenas igual a 4 e seis nmeros ascendentes com algarismo das centenas igual a 5. Assim, temos 16 nmeros
ascendentes entre 400 e 600.
100. Muro colorido Observamos que no momento em que escolhermos a cor de dois
tijolos vizinhos, a cor de todos os demais tijolos estar decidida.
...............................................................................................................................................................
...
..
..
..
..
...
...
...
.
.
.
.
......................................................................................................................................................................................................................
..
.
.
.
...
..
..
..
..
.
.
.
..
..
..
...
...
...
..
...................................................................................................................................................................................................................
..
..
..
..
..
..
..
..
..
..
..
..
..
..
..
..
.
.
.
.
.....................................................................................................................................................................................................................
..
..
..
..
..
...
...
...
...
...
...............................................................................................................................................................................................................

Assim, denotando os tijolos de acordo com uma de suas trs cores A, B ou C, e seguindo
a exigncia de no ter tijolos de mesma cor se tocando, obtemos uma distribuio como
a da figura. Como a maior quantidade de tijolos est marcada com A, num total de
seis, e os tijolos amarelos so os mais baratos, devemos escolher tais tijolos amarelos.
Por outro lado, temos a mesma quantidade de tijolos B e C, quatro de cada tipo,
portanto, podemos escolher quatro tijolos azuis e quatro vermelhos. Assim, o menor
valor a ser pago na compra dos tijolos desse muro 6 6 + 4 7 + 4 8 = 96 reais.
138

OBMEP 2010

Solues do Nvel 1
101. Divisores e fatorao Como o produto dos dois fatores 96, eles so divisores
de 96. Decompondo 96 em fatores primos, encontramos 96 = 25 3, portanto, seus
divisores so 1, 2, 3, 4, 6, 8, 12, 16, 24, 32, 48 e 96.
Os divisores 96, 48, 32, 24 e 16 no servem, pois seus quadrados j so maiores do que
208, sobrando 1, 2, 3, 4, 6, 8 e 12, cujos quadrados so 1, 4, 9, 16, 36, 64 e 144.
Agora fcil ver que a nica possibilidade 64 + 144 = 208. Como 8 12 = 96, os
nmeros so 8 e 12.
102. O retngulo do Lus Faremos a diviso com retngulos. Observamos que 24 = 64
e 12 = 6 2, portanto, Lus pode fazer um primeiro corte a 4 cm no lado de 10 cm
e outro corte a 2 cm do corte anterior. Depois desses cortes, resta um retngulo de
tamanho 6 4. Por ltimo, como 16 = 4 4, basta fazer mais um corte a 4 cm no
lado que mede 6 cm. Os cortes esto ilustrados na figura seguinte, com indicao das
dimenses dos lados e das reas.

24

12

4
16

103. Comparao de nmeros Fatorando os nmeros e extraindo as razes, obtemos

121
=
112
= 11,

3
3
3
729
= 9
= 9e

4
4
4
4
38 416 =
2 7 = 2 7 = 14 .
Logo, em ordem crescente, temos

729,

121 e

38 416.

104. As moedas Atribuindo o valor 1 s coroas e 1 s caras e somando os resultados


depois de cada jogada, inicialmente a brincadeira comea com soma 7 e queremos
chegar a cara e coroa alternadas, de modo que a brincadeira termina em 1 ou em
1. Observamos que, em cada passo da brincadeira, temos as seguintes possibilidades:
trocamos duas coroas por duas caras e o valor da soma diminui em 4; trocamos uma
cara e uma coroa por uma coroa e uma cara e o valor da soma fica inalterado; ou
trocamos duas caras por duas coroas e o valor da soma aumenta em 4. Portanto,
impossvel partir de 7 como soma inicial e chegar a 1, mas vejamos que, efetivamente,
possvel chegar a 1, isto , a quatro caras e trs coroas. Como queremos obter quatro
caras no consecutivas, precisamos de, pelo menos, quatro jogadas.

OBMEP 2010

139

Solues do Nvel 1
As quatro jogadas, que fazem a soma passar de 7 para 3, de 3 para 1 e ento permanecer em 1, esto ilustradas na figura.
.....................
....
...
...
..
....coroa ...
...
..
....
..
.
.
....................

.....................
....
...
...
..
...coroa ...
.
...
..
....
.
.
.
....................

....................
.....
...
...
..
...coroa ...
.
...
..
....
.
.
.
....................

....................
.....
...
...
..
....coroa ...
.
...
..
....
.
.
.
....................

...................
...
.....
..
...
....coroa ....
.
...
..
....
.
.
.
....................

...................
...
.....
..
...
....coroa ...
.
...
..
....
.
.
.
.....................

.....................
....
...
...
.
....coroa ...
.
...
..
....
.
.
.
.....................

...
......... .............
....
...
..
.
.... cara ...
...
..
.
.
.....
....................

.........
....... ..........
...
...
..
.
....coroa ...
...
..
.
.
.....
....................

..........
....... .........
...
...
..
....coroa ....
...
..
.
.
.....
....................

..........
....... .........
...
...
..
....coroa ....
...
...
.....
....................

....
........ ............
...
....
..
.
....coroa ....
...
..
.....
.
....................

....
......... ............
....
...
..
....coroa ....
...
..
.
.....
.
....................

........................
....
...
.
...
.... cara ....
.
...
...
.....
...................

........................
....
...
..
...
.... cara ...
.
...
...
.....
....................

.......................
...
....
..
...
....coroa ...
.
...
...
.....
.
.
.
.
.
...............

.......................
....
...
.
...
....coroa ...
.
...
...
.....
.
.
.
.
.
...............

.......................
....
...
...
.
....coroa ...
.
...
...
.....
....................

........................
....
...
...
.
.... cara ...
...
...
....
......................

..............
..... ........
...
...
.... cara ....
...
....
...
.
.
.
...................

....................
.....
...
...
..
..
...coroa .....
...
....
...
.
.
.
...................

..............
...... ........
...
..
..
... cara ....
...
....
...
.
.
.
...................

.....................
...
.....
..
...
.....coroa ...
.
...
...
.....
.
.
.
.................

.....................
.....
...
...
..
.....coroa ...
.
...
...
.....
.
.
.
.................

.....................
.....
...
...
..
..... cara ...
.
...
...
.....
.
.
.
..................

......................
....
...
...
.
..... cara ....
.
...
..
.....
.
.
.
..................

........................
...
...
..
... cara ....
...
..
....
.
.
......................
.

....................
.....
...
...
.
....coroa ....
...
..
....
.
.......................
.

.........................
....
..
..
... cara ....
...
.
....
..
.
.
.....................
.

..............
...... ........
...
.
.....coroa ....
...
...
....
.......................

..................
......
...
...
..
..... cara ....
...
..
.
....
.
.
.....................

........................
....
...
..
...coroa ....
...
...
....
.
.
.....................

.............
...... ........
...
.
.... cara ....
...
..
..
....
.......................

...
......... .............
....
...
..
.... cara ....
.
...
...
.....
....................

.......................
....
...
.
...
.... cara ....
...
...
....
......................

105. O preo do frango A opo correta (b).


Como 81 = 34 , o valor do frango triplicou quatro vezes. O nmero de meses transcorridos foi 4 6 = 24 meses, isto , dois anos, ou seja, em janeiro de 2002 o frango
atingir o preo de R$ 81,00.
106. Excurses a Foz do Iguau Temos um nibus com 27 19 = 8 lugares livres e
ainda precisamos acomodar os 53 8 = 45 participantes em nibus de 27 lugares.
claro que um nibus s no suficiente, portanto, precisamos de dois nibus e teremos
2 27 45 = 9 lugares livres no ltimo nibus. Ficaram 27 9 = 18 pessoas no nibus
incompleto.
107. As fraes de Laura Como a frao igual a um nmero inteiro, o seu numerador
deve ser um mltiplo do seu denominador. Vamos testar todas essas possibilidades e
escolher as que satisfazem as condies do problema.
3+5+6
3 + 6 + 11
= 7,
= 10 e
2
2

5 + 6 + 11
= 11 no satisfazem;
2

2 + 5 + 11
= 6 satisfaz;
3

3 + 6 + 11
= 4 no satisfaz;
5

2 + 5 + 11
= 3 satisfaz e
6

2+3+6
= 1 no satisfaz.
11

Assim, temos somente as duas respostas seguintes.

2+ 5+ 11
= 6

..............................................................................................................

108. Clculo da unidade A opo correta (e).


140

OBMEP 2010

2+ 5+ 11
= 3

..............................................................................................................

Solues do Nvel 1
Como o algarismo da unidade de qualquer potncia de 5 5, segue que o algarismo da
unidade de cada fator do produto 5 + 1 = 6. Mas, 6 6 = 36, ou seja, o produto
de dois nmeros terminados em 6 tambm um nmero terminado em 6. Logo, o
algarismo da unidade desse produto 6.
109. Nmeros cruzados
7 5
8
5 7
6 3
4
7 5

2
8 5
1
2
8 7
9 2

8 8
0
7 5
4
6 4
5

110. Ovos e mas A opo correta (b).


Como o enunciado e a resposta so percentuais podemos, nesse caso, estipular qualquer
preo e qualquer unidade monetria, que a resposta ser, sempre, a mesma. O mais
simples, portanto, supor que, inicialmente, uma dzia de ovos custava 100 e, portanto,
que dez mas tambm custavam 100. Como o preo dos ovos subiu 10%, o novo valor
dos ovos 110. O preo das mas diminuiu 2%, portanto, o novo preo de dez mas
98. Assim, enquanto antes gastava-se 200 na compra de uma dzia de ovos e dez
mas, agora gasta-se 110 + 98 = 208. Da, temos que o aumento foi de 8 em 200, o
que corresponde ao percentual de
4
8
=
= 4%.
200
100
111. Diviso de nmeros decimais A opo correta (a).
Efetuando a diviso, temos
254 880
144 177 10
254,88
=
=
= 1 440 .
0,177
177
177
112. Almoo dos amigos Os preos de um prato mais uma vitamina so
13 , |{z}
14 , |{z}
16 , |{z}
17 , |{z}
18 , |{z}
20 , |{z}
20 , |{z}
21 , |{z}
23 .
|{z}
7+6

7+7

7+9

11+6

11+7

11+9

14+6

14+7

14+9

Dentre esses, os que diferem por 6 so 14 e 20, ou 17 e 23. Logo, temos duas solues:
ou Denise gasta 7 + 7 = 14 e Jlio 14 + 6 = 11 + 9 = 20, ou Denise gasta 11 + 6 = 17
e Jlio 14 + 9 = 23.
113. Somas de trs em trs Inicialmente, observe que se a maior soma de trs desses
nmeros for 9, ento todos os nmeros devem ser menores do que 7, ou seja, 1, 2, 3,
4, 5 ou 6. Por outro lado, se a menor soma de trs desses nmeros distintos for 6,
ento eles no podem incluir 5 ou 6, restando 1, 2, 3 e 4. Verificamos que esses so os
nmeros, pois
1 +2 +3 = 6, 1 +2 +4 = 7, 1 +3 +4 = 8
OBMEP 2010

e 2 +3 +4 = 9.
141

Solues do Nvel 1
114. O passeio do Jorge Lembrando que a distncia entre as rvores ao longo do
caminho de 5 m, ilustramos o sentido do percurso de Jorge nas figuras.
P
........
.......
..............................s
......................s
...............................s
.......................s
..
..s

P
.......
..............................s
..............................s
...............................s
.......................s
..
..s

P
...........
..s
..............................s
...............................s
..............................s
...................s
..

....
....
...
...
..
...
...
...
...
....
...
..
....
...
..
...
...
..
..............................................................................................................................

....
....
...
...
..
...
...
...
...
....
...
..
...
....
..
...
...
..
..............................................................................................................................

s s

s
s

....
....
...
...
..
...
...
...
...
....
...
..
...
....
..
...
...
..
..............................................................................................................................

s s
s

(a) Caminhando inicialmente 32 m, ele toca em sete rvores, parando 2 m depois da


ltima rvore que tocou.
(b) Voltando 18 m, ele toca em quatro rvores, parando 1 m depois da ltima que
tocou.
(c) Ao retornar 22 m, ele toca em cinco rvores, parando 1 m depois da ltima rvore
que tocou.
Assim, Jorge tocou em 7 + 4 + 5 = 16 rvores.
115. A descoberta do algarismo
algarismos, temos,
com 1
com 2
com 3

Separando os nmeros cujos quadrados tm 1, 2 e 3


algarismo: 1, 2, 3
algarismos: 4, 5, 6, 7, 8, 9
algarismos: 10, 11, 12,. . . , 31.

At 312 , o nmero j tem 3 + 12 + 66 = 81 algarismos. Abreviando algarismo por


algs, temos
12 , 22 , 32 , 42 , . . . , 92 , 102 , . . . , 312 .
| {z } | {z } |
{z
}
13 algs

26=12 algs

322=66 algs

Assim, faltam 100 81 = 19 algarismos para o 100o . Como s 1002 tem 5 algarismos,
e como 19 = 4 4 + 3, teremos mais 4 nmeros de 4 algarismos cada um, que so 322 ,
332 , 342 e 352 , e mais os 3 algarismos (milhar, centena, dezena) do nmero 362 = 1 296,
como segue.
12 , 22 , 32 , 42 , . . . , 92 , 102 , . . . , 312 , 322 , 332 , 342, 352 , 12 |{z}
9 6
| {z } | {z } |
{z
} |
{z
}
o
13 algs

26=12 algs

322=66 algs

44=16 algs

100 alg

Assim, vemos que o algarismo 9 ocupa a 100a posio.

116. OBMEP Como peso de B+ peso de E = 6 e peso de M+ peso de P = 6, segue


que os pesos de M, P, B e E so todos menores do que 6. Como no h dois discos
de mesmo peso, M, P, B e E no podem pesar 3 e, portanto, os pesos desses quatro
discos s podem ser 1, 2, 4 e 5. Agora, peso de X+ peso de O = 13 e peso de
Z+ peso de O = 9, portanto, peso de X = peso de Z + 4. Assim, a nica opo para
os pesos de Z e de X 3 e 7. Por excluso, o peso de O 6. Assim, obtemos
peso de O + peso de B + peso de M + peso de E + peso de P = 6 + 6 + 6 = 18.
117. Prdio misterioso Primeiro observamos que os elevadores denotados por A, C, D,
E, F e H conduzem a recintos fechados em algum dos dois andares e, portanto, no
levam sada. Desconsiderando esses elevadores, nosso desenho de elevadores teis
o seguinte.
142

OBMEP 2010

Solues do Nvel 1
qqqqqqqqqqqqqqqqqqqqqqqqqqqqqqqqqqqqqqqqqqqqqqqqqqqqqqqqqqqqqqqq
qq
q
qq
B qqqq
q
q
qqqqqqqqqqqqqqqqqqqqqqqqqqqqqqqqqqqqqqqqqqqqqqqqqqqqqqqqqqqqqqqqqqqqqq
qq
qq
qq
qq
qq
qq
qq
qq
q
qqq
q
q
qq
qq
qq
qq
q
q
qq
q
q
qq
qqqqqqqqqqqqqqqqqqqqqqqqqqqqqq
qq
qq
qq
qq
qq
qq
q
q
q
qq G qqq
qq
qq
q
q
q
qqqqqqqqqqqqqqqqqqqqqqqqqqqqqqqqqqqq
qqqqqqqqqqqqqqqqq
qqq
qqq
qq
qq
qq
qq J
q
qqqqqqqqqqqqqqqqqqqqqqqqqqqqqqqqqqqqqqqqqqqqqqqqqqqqqqqqqqqqqqq

qqqqqqqqqqqqqqqqqqqqqqqqqqqqqqqqqqqqqqqqqqqqqqqqqqqqqqqqqqqqqqqqqqqq
qq
qq
q B
qq
qqqqqqqqqqqqqqqqqqqqqqqqqqqqqqqqqqqqqq
qqqqqqqqqqqqqqqq
qq
qq
qq
qq
qq
qq
qq
qq
q
qqqqqqqqqqqqqqqqqqqqqqqqqqqqqqqqqqqqqqqqqqqqqqqq
qq
qq
qq
q
qq
qq
qq G qq
qq
qq
qq
qqqqqqqqqqqqqqqqq
qq
q
qq
q
qq
qq
qq
qq J
sada
qqqqqqqqqqqqqqqqqqqqqqqqqqqqqqqqqqqqqqqqqqqqqqqqqqqqqqqqqqqqqqqqq

entrada

Assim, o caminho mais curto entre a entrada de um andar at a sada do outro consiste
em primeiro pegar o elevador B, depois o J e, por ltimo, o G.
118. Soma de fraes
Soluo 1: Transformando as fraes em nmeros decimais, obtemos
909
1
1
1
1

= 0,1 0,01 + 0,001 0,00001 = 0,0909 =


.
10 100 1 000 10 000
10 000
Soluo 2: Efetuando a soma das fraes, obtemos
1
1
1
1 000 100 + 10 1
909
1

=
=
.
10 100 1000 10 000
10 000
10 000
27
119. Biblioteca Ao comprar 140 livros, a biblioteca ficou com
do nmero de livros,
25
2
2
dos livros da biblioteca. Se
corresponde a 140
portanto, 140 corresponde a
25
25
1
25
corresponde a 140 2 = 70 livros e
a 70 25 = 1 750 livros. A opo
livros,
25
25
correta (a).
120. Comparao de fraes Para que uma frao seja menor do que 1, o numerador
deve ser menor do que o denominador. Eliminando as repeties, obtemos a lista
seguinte.
1
2
1
2
(b) 2 fraes com denominador 3:
e
3
3
1
2
3
(c) 2 fraes com denominador 4: ,
e
4 |{z}
4
4
(a) 1 frao com denominador 2:

1/2

1 2 3
4
, ,
e
5 5 5
5
1
2
3
4
5
(e) 2 fraes com denominador 6: ,
,
,
e
6 |{z}
6 |{z}
6 |{z}
6
6

(d) 4 fraes com denominador 5:

1/3

(f) 6 fraes com denominador 7:

1/2

2/3

1 2 3 4 5
6
, , , ,
e
7 7 7 7 7
7

OBMEP 2010

143

Solues do Nvel 1
(g) 4 fraes com denominador 8:

3
5
7
1
2
4
6
,
,
,
,
,
e
8 |{z}
8
8 |{z}
8
8 |{z}
8
8
1/4

(h) 6 fraes com denominador 9:

1/2

1 2
4 5
7
3
6
8
, ,
,
, ,
,
e
.
9 9 |{z}
9
9 9 |{z}
9
9
9
1/3

Assim, temos 27 dessas fraes.

3/4

2/3

121. Diviso com resto Se a diviso de 2 007 por algum nmero deixar resto 5, ento
esse nmero divide 2 007 5 = 2 002. Assim, calculamos todos os divisores de 2 002 =
2 7 11 13, listados na coluna da direita da tabela seguinte.
...
... 1
...
..
..
...
...
.
...
2 002 .... 2 ......... 2
...
...
1 001 .......... 7 .......... 7, 14
...
...
143 ......... 11 .......... 11, 22, 77, 154
..
...
.
13 .........13 ........ 13, 26, 91, 182, 143, 286, 1 001, 2 002
...
...
.
1 ........ .......
Como o resto 5 deve ser menor do que o divisor, dividindo 2 007 por qualquer um dos
14 nmeros seguintes deixa resto 5:
7, 11, 13, 14, 22, 26, 77, 91, 143, 154, 182, 286, 1 001 e 2 002.
122. Panelas Convertendo 1 kg em 1 000 g, temos que as duas panelas juntas, mais a
carne, pesam 645 + 237 + 1 000 = 1 882 g. Logo, cada panela, mais o seu contedo de
carne, deve pesar 1 882 2 = 941 g. Assim, Jos colocou:
941 645 = 296 g

941 237 = 704 g

nessas duas panelas.

123. Domins Como 2 3 = 6, podemos comear supondo que os dois domins .........................................
.............................
e ...................................... estejam na posio certa. Se isso for verdade, e como 1 3 = 3, resulta que
o algarismo na dezena do resultado deve ser 3, portanto precisamos trocar o domin
.............................
.............................
... ... ...
................................
pelo domin ......................................... , de tal forma que o 3 fique na dezena. Como temos um 2 na
centena do resultado, a centena do primeiro nmero precisa ser um 4. Com essa troca,
a posio dos domins fica correta, como pode ser visto na figura.
.............................

.......s..............s..........s............s.....
..
..
...
.... s ..... s s.....
......s..............s.........................
..............................s.......s.......s....
...
...
.
... s .....
..
..............................s.......s.......s.....
144

................................................
...
.
.
... s ..... s s .....
..
.
...
..........................................
.......s...................
.. s ..
..
..
............................................s..... .......s..............ss.....
... s ...
.
...
... s .... ... s s...
.
.
s
...
..........................s.................. ......s..............s....

OBMEP 2010

5412
3
16236

......................................................

Solues do Nvel 1
124. Cdigo secreto A nica maneira de obter 360 = 23 32 5 como produto de trs
nmeros de um algarismo cada um 360 = 9 8 5 . Como A o menor dos trs,
A = 5. Logo B = 8 e C = 9, ou B = 9 e C = 8, ambas opes com AA + BB + CC =
55 + 88 + 99 = 242. Logo, temos duas possibilidades para o cdigo ABC, a saber, 589
ou 598.
125. Os doze pontos No total, temos 11 possveis quadrados, mostrados nas figuras
seguintes.
5 quadrados
s
s
s
s

s
s
s
s

4 quadrados

........................
...
...
...
...
..
..
........................................................................
...
...
...
...
...
..
..
....
..
..
..
.
......................................................................
...
...
..
..
...
...
......................

s
s

s
s
s
s

s
s
s
s

..
..
.... ... .... ....
... ........... ......
..
....
....
....
.... ......
....
......
.......
..
.
.
.
.... ..... .... ..... ...... .......
......
.....
......
.
.
.
.
.
.
.
.
.
.. ............ ........... ......
.
.
.
.
..
....
....
.... ......
....
....
....
....
....
.... ........... ......
........
.......

s
s

s
s

s
s

2 quadrados
s
s
s
s

s
s
s
s

..
...
... ..................... ....
.............. .................
.............
.........
.............
... ....
......
......
.....
.
.....
. ..
.
.. ..
.
........ ...
..............
.
.........
.
.
............. .................
... .................. ..
...... .......

s
s

s
s

126. Relgio Vamos tentar uma data e um horrio no mesmo ano de 1994. J que com
os nmeros dados no podemos alterar o dia nem para 29 nem para 30, sem alterar
o ano, ento a data procurada no est no ms 05. O seguinte ms possvel o 08.
Como precisamos da data mais prxima possvel, observemos que podemos formar o
dia 01, sobrando os algarismos 0, 2, 4 e 5 para formar a hora. A menor hora possvel
que podemos formar com esses algarismos 02h45m, de modo que a data procurada
1o de agosto de 1994, s 02 horas e 45 minutos.
...........................................
.............
.......
.......
.....
......
.....
.....
....
.
.
.
....
...
.
...
.
...
...
.
...
..
.
..
....
...
..
...
...
...
..
...
..
.
...
...
..
...
................................
..
...............................
.
.
.
..
.
.
.
..
.
.
.
.
.
.
.
.
.
...
.
.
.
.
....
...
...
...
...
...
.
...
..
...
...
...
... ................................
................................. .....
...
.
.
.
....
.
.....
....
......
....
.....
.......
.......
..........
................................................

28 05 94

.............................................
............
......
.......
.....
....
......
....
....
.
.
.
....
...
...
.
.
...
...
.
...
..
.
...
....
...
..
...
...
...
..
...
..
..
.
..
..
...
..............................
................................
...
.
.
.
.
.
.
...
..
.
.
.
.
.
.
.
.
.
.
.
...
.
....
...
...
...
..
...
.
..
...
...
...
... ....
... ..............................
................................. ....
....
.
.
.
.
....
....
....
....
.....
.......
......
......
..........
................................................

01 08 94

14 h 00

02 h 45

127. Lpis A opo correta (b).


Vamos ver em quantas caixas podemos colocar o nmero mximo de lpis, que 6 por
caixa. Nas 13 caixas no possvel, pois 13 6 = 78 maior do que o nmero 74
do total de lpis. Em 12 caixas podemos ter 12 6 = 72, sobrando uma caixa, com
74 72 = 2 lpis.
128. Contagem A cada 10 pginas, o algarismo 1 aparece uma vez nas unidades e, a
cada 100 pginas, aparece 10 vezes nas dezenas. Contando o nmero de pginas que
contm o algarismo 1 em cada faixa abaixo, temos
(a) 20 vezes entre 1 e 99:
1, 11, 21, 31, 41, 51, 61, 71, 81, 91, num total de 10 vezes na unidade;
10, 11, 12, 13, 14, 15, 16, 17, 18, 19, num total de 10 vezes na dezena.
OBMEP 2010

145

Solues do Nvel 1
(b) 120 vezes entre 100 e 199:
101, 111, 121, 131, 141, 151, 161, 171, 181, 191: 10 vezes na unidade;
110, 111, 112, 113, 114, 115, 116, 117, 118, 119: 10 vezes na dezena;
100, 101, 102, . . . , 199, num total de 100 vezes na centena.
(c) 20 vezes entre 200 e 299:
201, 211, 221, 231, 241, 251, 261, 271, 281, 291: 10 vezes na unidade;
210, 211, 212, 213, 214, 215, 216, 217, 218, 219: 10 vezes na dezena.
At a pgina 299, o nmero 1 aparece 20 + 120 + 20 vezes, faltando, portanto, apenas
171 160 = 11 vezes. Os dois primeiros que aparecem depois de 299 so dois na
unidade, em 301 e 311, e os nove primeiros das dezenas, em 310, 311, 312, 313, 314,
315, 316, 317 e 318. Assim, o livro tem 318 pginas.
129. Viagem a Recife A opo correta (b).
No momento em que a informao foi dada, o tempo de vo que faltava era de 1h20min,
ou 4/3 de hora. Logo, nesse momento, a distncia at Recife era de 864 43 = 1 152
km. Como estvamos a 1 222 km da cidade de partida, a distncia entre essa cidade e
Recife deve ser 1 152 + 1 222 = 2 374 km. Dentre as opes dadas, a mais prxima
2 400 km.
130. Praa Como a 5a casa da Maria a 12a casa do Joo, a diferena entre as contagens
de 7 casas e, portanto, a 1a casa da Maria a 8a casa do Joo. Como a 30a casa da
Maria a 5a casa do Joo, a 32a da Maria a 7a do Joo. A casa seguinte j a 8a do
Joo, ou seja, a 1a da Maria. Assim, a praa tem 32 casas.
131. Sequncia de figuras As figuras se repetem num grupo de seis, sempre terminando
com , tanto o 1o quanto o 166o grupo. Como 996 = 6 166, a ltima figura do 166o
grupo, ou seja, a 966a figura, .
o

1 000o

996
z}|{
z}|{
, , , , , , . . . , , , , , , , , , , , , , . . .
{z
}
{z
}
|
|
1o grupo de 6

166o grupo de 6

(a) A 1000a figura, portanto, .

(b) O primeiro est na 3a posio, o segundo na 1 6 + 3 = 9a posio, o terceiro


na 2 6 + 3 = 15a , o quarto na 3 6 + 3 = 21a posio, e assim por diante, at
o milsimo , que aparece na 999 6 + 3 = 5 997a posio.
132. A brincadeira com o quadrado
Soluo 1: Convertendo metros em milmetros, temos 1 m = 1 000 mm. Assim, o
quadrado ficou dividido em 1 000 1 000 = 106 quadradinhos, cada um com 1 mm
de lado. Colocando lado a lado todos os 106 quadradinhos, teremos um retngulo de
comprimento
1| + 1 +{z + 1} = 106 1 = 106 mm = 1 km.
106 parcelas

146

OBMEP 2010

Solues do Nvel 1
Soluo 2: O quadrado tem rea igual a 1 m2 = 106 mm2 . A rea do retngulo
a mesma do quadrado. Como a largura do retngulo mede = 1 mm, resulta que o
comprimento c do retngulo, em milmetros, mede

106
c=
=
= 106 mm.

1
133. O cdigo da Arca do Tesouro Nas duas tabelas seguintes mostramos unicamente
os grupos de trs nmeros em casas sucessivas, horizontais ou verticais, cuja soma seja
14.

2
7

7
4
6

9
3

4
4

7
1
6

9
3
2
7
2
5

5
6
3

9
3
2
7
1
6

4
5
5
2
7

1
8
5

Assim, quando eliminamos esses nmeros da tabela inicial, os nmeros que sobrevivem
so somente os indicados na tabela seguinte.
5
6

4
8
2
Portanto, a soma dos nmeros que restam 5 + 4 + 6 + 4 + 8 + 2 = 29, que o cdigo
da Arca do Tesouro.
134. Operaes com decimais Temos

(0,2)3 + 1
0,008 + 1
1,008
=
=
= 0,84 .
0,2 + 1
1,2
1,2

135. Fatores inteiros Como o produto dos dois fatores 96, eles so divisores de 96 =
25 3, ou seja, os possveis fatores positivos so 1, 2, 3, 4, 6, 8, 12, 16, 24, 32, 48 e 96.
Os nicos com quadrado menor do que 208 so 1, 2, 3, 4, 6, 8 e 12, cujos quadrados
so 1, 4, 9, 16, 36, 64 e 144.

A nica maneira de obter 208 como soma de dois dos nmeros listados acima,
64 + 144 = 208. Assim, os nicos fatores positivos so 8 e 12. Logo os nicos fatores inteiros cuja soma dos quadrados 208 so 8 e 12 ou, ento, 8 e 12.
136. Divisibilidade A opo correta (b).
O nmero divisvel por 45 = 5 9, portanto divisvel por 5 e 9. Todo nmero
divisvel por 5 termina em 0 ou 5. Assim, b = 0 ou b = 5. Todo nmero divisvel
por 9 tem como a soma dos seus algarismos um nmero que mltiplo de 9. Logo,
6 + a + 7 + 8 + b = 21 + a + b mltiplo de 9. Como a 9, e b = 0 ou b = 5, temos
21 21 + a + b 21 + 9 + 5 = 35. Mas, o nico mltiplo de 9 entre 21 e 35 27. Logo,
21 + a + b = 27. Conclumos que a + b = 6 e o nmero procurado 61 785 ou 66 780.
OBMEP 2010

147

Solues do Nvel 1
137. Nmero simples Com 1 algarismo, temos os nmeros simples 1 e 2; com 2 algarismos, temos os 22 = 4 nmeros simples 11, 12, 21 e 22; com 3 algarismos, temos os 23 = 8
nmeros simples 111, 112, 121, 122, 211, 212, 221 e 222. Com 4 algarismos, temos
24 = 16 nmeros simples, com 5 algarismos, temos 25 = 32 nmeros simples e com 6
algarismos, temos 26 = 64 nmeros simples. Como um nmero inferior a 1 milho tem,
no mximo, 6 algarismos, resulta que existem exatamente 2 + 4 + 8 + 16 + 32 + 64 = 126
nmeros simples menores do que 1 milho.
138. Venda de TV Sejam a o algarismo da dezena de milhar e b o da unidade. Como
o nmero divisvel por 72 = 8 9, temos que 79b um nmero par divisvel por
8. Testando os valores de b = 0, 2, 4, 6 e 8, vemos que, necessariamente, b = 2. Um
nmero divisvel por 9 se a soma dos seus algarismos for um mltiplo de 9. Ento,
a + 6 + 7 + 9 + 2 = a + 24 um mltiplo de 9 e, portanto, a = 3. Assim, na fatura
constava R$ 36 792,00 e, portanto, cada TV custou 36 792 72 = 511 reais.
139. Chocolate Como 8 1,35 = 10,8 maior do que 10, Henrique comprou 7 barras de
chocolate e recebeu 10 71,35 = 0,55 reais, ou 55 centavos, de troco.
140. O quadradinho Simplificando, obtemos
1,6  =

6 400 000
= 16 000 = 1,6 10 000 .
400

Assim,  = 10 000.
141. Dois nmeros Como 12 o MDC dos dois nmeros e cada um tem dois algarismos,
os nicos candidatos so os mltiplos de 12 menores do que 100, ou seja,
12, 24, 36, 48, 60, 72, 84 e 96.
Como 1 728 = 12 12 12 = 26 33 , os mltiplos 60 (com fator 5) e 84 (com fator 7)
no so divisores de 1 728. Tambm 1 728 12 = 144 e 1 728 96 = 18, de modo que
a lista reduz a 24, 36, 48 e 72, com 24 72 = 36 48 = 1 728. Como o MDC de 24 e
72 24, temos uma nica soluo, a saber, 36 e 48, cujo produto 1 728 e o MDC
12.
142. As idades dos irmos Dividindo 2 000 por 7, obtemos 2 000 = 7 285 + 5.
Logo, 2 000 dias equivalem a 285 semanas, mais 5 dias. Como o dia 13 de maro de
2 007 caiu em uma tera-feira, contando os 5 dias restantes, temos que o aniversrio
do irmo de Carlos cair em um domingo. Agora, dividindo 2 000 por 365, obtemos
2 000 = 365 5 + 175. Assim, 2 000 dias equivalem a, aproximadamente, cinco anos e
meio, portanto Carlos estar com 12 anos de idade.
143. A mistura de concreto A opo correta (e).
De acordo com os dados do problema, misturamos 1 kg de cimento com 3 kg de areia
e 5 kg de terra. Isso equivale a misturar 5 kg de cimento com 15 kg de areia e 25 kg
de terra, e essa mistura pesa 5 + 15 + 25 = 45 kg.
148

OBMEP 2010

Solues do Nvel 1
144. Ponto na escala A distncia entre os pontos inicial e final de 12,62 12,44 =
0,18 unidades. Como esto marcados 18 intervalos, o comprimento de cada um deles
de 0,18 18 = 0,01 unidades. O ponto P est na 6a posio direita de 12,44,
portanto corresponde a 12,44 + 0,01 6 = 12,50.
145. O pomar do Francisco A opo correta (c).
De acordo com os dados do problema, podemos observar que temos dois pares de
rvores vizinhas: as laranjeiras so vizinhas dos limoeiros e as macieiras so vizinhas
das pereiras. Como so cinco fileiras e as macieiras e pereiras no esto do lado das
laranjeiras e limoeiros, resulta que as tangerineiras esto na terceira fila, a do meio.
146. Quatro quadrados Se a rea de cada quadrado 3 cm2 e cada um deles est
3
dividido em 16 quadradinhos, ento a rea de cada quadradinho
cm2 . Como h
16
um total de 6 quadradinhos superpostos nos 4 quadrados, temos que a rea da figura

9
87
3
= 12 =
= 10,875 cm2 .
436
16
8
8
147. O fio de arame A opo correta (d).
A figura composta de 3 semicrculos, o que exclui as opes (b), (c) e (e), e 4 segmentos
de reta, o que exclui a opo (a), que s tem 3 segmentos.
148. Sequncia de fsforos A opo correta (c).
Observe que o nmero de fsforos da sequncia formado da seguinte maneira:
primeiro termo
segundo termo
terceiro termo

= 3 + 3 = 2 3 = (1 + 1) 3;
= 3 + 3 + 3 = 3 3 = (2 + 1) 3;
= 3 + 3 + 3 + 3 = 4 3 = (3 + 1) 3.

Logo, o oitavo termo da sequncia (8 + 1) 3 = 9 3 = 27.


149. O trajeto das formiguinhas
(a) O trajeto de M a N compreende 14 comprimentos e 12 larguras das lajotas. Logo,
seu comprimento 14 6 + 12 4 = 84 + 48 = 132 cm.
Como as duas formiguinhas percorrem a mesma distncia, cada uma deve andar
132 2 = 66 cm.
(b) Vamos acompanhar, desde o incio, o percurso feito por Maricota at completar
os 66 cm:
2 comprimentos
|
{z
}

26=12

2 comprimentos
|
{z
}
12+42=54

1 largura
| {z }

3 comprimentos
|
{z
}

4+12=16

1 largura
| {z }

18+16=34

1 comprimento
|
{z
}

4+54=58

6+58=64

2 larguras +
| {z }
8+34=42

1/2 largura .
|
{z
}
2+64=66

O caminho de Maricota at o ponto de encontro est indicado na figura.


OBMEP 2010

149

Solues do Nvel 1
12
M ..r..................................................
..............................
.

.
...
34
...
..........................................................................................................................
.
...
..
16
...
....
..
...................................................................................
..54
...
42
..
64 ............................................58
...
66 ....
...
..
...
...
............................................................................................................................
...
...
....
...
...
..
....
...
..
..
...
.
...................................................................................
...
..
...
...
...
..
...............................................

ponto de encontro

150. A soma 100


(a) Inicialmente observe que, como a soma dos trs nmeros 100 e o maior deles
igual soma dos outros dois, ento duas vezes o maior nmero 100, ou seja, o
maior nmero 50.
(b) Como 50 no primo, os outros dois nmeros so primos e tm soma igual a 50.
Por exemplo, 3 e 47 so primos e 3 + 47 = 50. Portanto, os nmeros 3, 47 e 50
formam uma soluo do problema.
(c) Existem outras solues para o problema. Para encontr-las, escrevemos a lista de
todos os primos entre 1 e 50, ou seja, 2, 3, 5, 7, 11, 13, 17, 19, 23, 29, 31, 37, 41, 43 e 47
e, para cada um desses nmeros, verificamos se a diferena para 50 tambm
primo. Encontramos um total de quatro solues
Soluo
Soluo
Soluo
Soluo

1
2
3
4

3
7
13
19

47
43
37
31

50
50
50
50

151. Cdigo de barras Lembre que a primeira e a ltima barra no fazem parte do
cdigo.
(a) Primeiramente, escrevemos o CEP dado com os algarismos 0 e 1:
00101
| {z } |10100
{z } 00110
| {z } 00001
| {z } 11000
| {z } 00011
| {z } |00101
{z } 11000
| {z } .
3

Em seguida, escrevemos o cdigo de barras desse CEP:

||||||||||||||||||||||||||||||||||||||||||

(b) Primeiramente, escrevemos o cdigo de barras dado com os algarismos 0 e 1 em


grupos de 5 algarismos:

| |{z}
||||| ||||| ||||| ||||| ||||| ||||| ||||| ||||| |
|{z} |{z} |{z} |{z} |{z} |{z} |{z}
01010 11000 01010 00110 11000 11000 01010 11000

Em seguida, escrevemos o CEP, que 20240020.


150

OBMEP 2010

Solues do Nvel 1
152. Atletas da escola O nmero total de alunos na escola dado
pela frao 12/12, que podemos representar graficamente por um
retngulo dividido em 12 partes iguais.
Denotemos por V, F e NE o nmero de alunos que jogam somente vlei, somente futebol
e nenhum desses dois esportes, respectivamente. A informao dada, em termos das
partes desse retngulo, a seguinte:
o 1/4 dos alunos que jogam somente vlei corresponde a trs partes;

o 1/3 dos alunos que jogam somente futebol corresponde a quatro partes;

o 1/12 dos alunos que no jogam nem vlei nem futebol corresponde a uma parte.
V
F

V
F

V
F

F
NE

(a) Sobram 4 partes do retngulo para os alunos que jogam vlei e futebol, ou seja,
esses 300 alunos correspondem a 4/12 = 1/3 do total dos alunos da escola. Logo,
o total de alunos na escola 300 3 = 900.
1
(b) O total de alunos que jogam somente futebol 900 = 300.
3
(c) Os alunos que jogam futebol so os que jogam s futebol mais os que jogam
futebol e vlei, ou seja, 300 + 300 = 600.
11
(d) O total de alunos que praticam um dos esportes
900 = 825, pois 1/12 dos
12
alunos no jogam nem futebol, nem vlei.
153. Dzima peridica
1
(a) Dividindo 1 por 22, obtemos 22
= 0,0454545 . . . . Observe que o algarismo 4 est
nas posies pares, ou seja, segunda, quarta, sexta, e assim por diante, enquanto
que o algarismo 5 est nas posies mpares, ou seja, a terceira, a quinta, a stima,
e assim por diante. Como 1 997 um nmero mpar, temos que o algarismo da
1 997a casa decimal 5.
1
(b) Dividindo 1 por 27, obtemos 27
= 0,037037037 . . . . Observe que os algarismos 0,
3 e 7 se repetem, sucessivamente, a cada trs casas decimais, sendo que

o algarismo 0 est nas posies 1a , 4a , 7a , . . . , ou seja, aquelas que, divididas


por 3, deixam resto 1;
o algarismo 3 est nas posies 2a , 5a , 8a , . . . , ou seja, aquelas que, divididas
por 3, deixam resto 2 e
o algarismo 7 est nas posies 3a , 6a , 9a , . . . , ou seja, aquelas que so mltiplas de 3.
.

Como a diviso 1 997 3 deixa resto 2, o algarismo da 1 997a


casa decimal 3.

OBMEP 2010

..
1 997 .................3........................
2 665

151

Solues do Nvel 1
154. Ana na corrida Transformando minutos em horas, temos que 20 minutos correspondem a 20/60 = 1/3 de hora. Assim, a velocidade de Ana deve ser maior do que
1
v = 5/ = 15 km/h.
3
155. Quadradinhos e o buraco Contando os quadradinhos retirados de cada linha,
temos que o nmero desses quadradinhos 1 + 3 + 5 + 15 + 10 + 2 = 36. Como cada
quadradinho tem 1 cm2 de rea, a rea do buraco 36 cm2 .
Para obter o permetro do buraco, podemos simplesmente contar quantos lados de
quadradinhos tm o buraco, obtendo 42 lados, de modo que o permetro mede 42 cm.
Entretanto, uma maneira alternativa de descobrir o permetro do buraco observar
que ele se estende por 6 linhas e 15 colunas, sendo que cada linha e cada coluna
ocupada pelo buraco contm exatamente dois lados de quadradinho que fazem parte
do permetro. Logo, o permetro do buraco mede 2 (6 + 15) = 42 cm.
156. Quadrados perfeitos no retngulo Para resolver esse problema, convm listar os
quadrados perfeitos de dois algarismos, que so
42 = 16, 52 = 25, 62 = 36, 72 = 49, 82 = 64 e 92 = 81,
bem como os quadrados perfeitos de trs algarismos, que so
102 = 100, 112 = 121, 122 = 144, 132 = 169, 142 = 196, 152 = 225,
162 = 256, 172 = 289, 182 = 324, 192 = 361, 202 = 400, 212 = 441,
222 = 484, 232 = 529, 242 = 576, 252 = 625, 262 = 676, 272 = 729,
282 = 784, 292 = 841, 302 = 900 e 312 = 961.
Em particular, vemos que todo quadrado perfeito de trs algarismos um nmero
terminado em 0, 1, 4, 5, 6 ou 9.
Assim, estabelecemos que, dos quadrados perfeitos de dois algarismos,
25, 36 e 81 no podem aparecer na terceira coluna, assinalada com X.
Para essa coluna, restam apenas os quadrados perfeitos 16, 49 e 64,
portanto, temos trs opes, como segue.
(I)

1
6

4
9

(II)

(III)

X
X

6
4

(a) Vamos examinar cada uma das trs opes.


Opo (I): Os quadrados perfeitos de trs algarismos terminados em 6 so 196,
256, 576 e 676. Como nenhum quadrado perfeito de dois algarismos
1
termina em 2 ou 7, os quadrados perfeitos 256, 576 e 676 no podem
1 9 6
aparecer na segunda linha, restando, apenas, 196.
Agora, os nicos quadrados perfeitos de dois algarismos terminados em 1 e 9 so,
respectivamente, 81 e 49. Obtemos a soluo seguinte, que a nica dentro da
Opo (I).
8
1
152

4
9

OBMEP 2010

1
6

Solues do Nvel 1

Opo (II): Os quadrados perfeitos de trs algarismos terminados em 9 so 169,


289, 529 e 729, de modo que a segunda linha pode ser preenchida apenas com o
quadrado perfeito 169. Na primeira coluna s pode aparecer o nmero 81, por ser
o nico quadrado perfeito de dois algarismos terminado em 1.
1

4
9

8
1

4
9

Temos, agora, duas opes para preencher a ltima casa em branco: 1 ou 3.


No entanto, nem 814 nem 834 so quadrados perfeitos. Assim, a opo (II)
impossvel.
Opo (III): Os quadrados perfeitos de trs algarismos terminados em 4 so 144,
324, 484 e 784, de modo que a segunda linha pode ser preenchida apenas com
o quadrado perfeito 144 e, na primeira coluna s pode aparecer o nmero 81.
Agora, a nica escolha para a casa em branco o nmero 6.
8
1

6
4

8
1

6
4

6
4

No entanto, 866 no quadrado perfeito. Logo a opo (III) tambm impossvel.


(b) Pelo que vimos acima, existe apenas uma soluo, encontrada no item precedente,
8 4 1
a saber,
.
1 9 6
157. Aula de diviso
(a) Temos 38 4 = 34 = 2 17 = 1 34, portanto, = 17 e = 2, ou = 34 e
= 1.
(b) Temos 75 = 6 12 + 3, portanto, = 3 e = 6.

(c) Temos 3 7 = 21. Os possveis restos da diviso por 3 so 0, 1 e 2, portanto,


= 21 e = 0, ou = 22 e = 1 ou, ainda, = 23 e = 2.

(d) Temos 42 = 5 8 + 2, portanto, podemos trocar o divisor pelo quociente para


obter = 8 e = 2.
158. Linhas de nibus
(a) Fatorando, temos 15 = 3 5 e 25 = 52 , portanto o menor mltiplo comum de 15 e
25 75 = 3 52 . Assim, os dois nibus passaro juntos novamente no ponto a cada
75 minutos, ou seja, a cada 1h15min. Logo, os nibus passaro juntos novamente
no ponto perto da casa de Quinzinho, s 7h30min + 1h15min = 8h45min.
(b) Para obter os horrios em que os nibus passaro juntos no ponto de nibus
perto da casa de Quinzinho, devemos ir somando 1h15min, obtendo 8h45min,
10h, 11h15min, 12h30min, 13h45min, 15h, 16h15min, 17h30min, 18h45min, 20h,
21h15min, 22h30min e 23h45min. O prximo nibus s passa depois da meia
noite.
OBMEP 2010

153

Solues do Nvel 1
159. Quadrados dentro de um retngulo
(a) Como o menor quadrado tem 1 cm de lado, o lado do quadrado
A mede 1 4 = 4 cm e o lado do quadrado B mede 4 + 1 = 5
cm. O quadrado C tem um lado em comum com o quadrado
B, portanto, o quadrado C tambm tem 5 cm de lado. Assim,
o lado do quadrado maior mede 5 + 5 + 4 = 14 cm.

........................
.......................14
..
..
..
..
..
..
..
..
..
..
..
..
...........5.................5...............4.........
.. C .. B .. A ..
..
. .
..
...........................................................

(b) Os lados do retngulo medem 14 cm e 14 + 5 = 19 cm, portanto, o permetro do


retngulo 14 2 + 19 2 = 66 cm.
160. Festa na escola
(a) A professora + 16 alunos + 1 monitor + 5 pais = 23 pessoas comero os pes
de queijo. Para que cada pessoa possa comer pelo menos 5 pes de queijo, ser
necessrio comprar, no mnimo, 5 23 = 115 pes de queijo. Cada po de queijo
100
= 10 gramas, de modo que ser necessrio comprar
pesa, em mdia,
10
10 115 = 1 150 gramas de pes de queijo.
Como a preciso da balana de 100 g, arredondamos 1 150 g para 1 200 g e
obtemos a quantidade de po de queijo que a professora deve comprar, em gramas.
1 200
(b) Como
= 12, temos que a professora gastar 12 3,20 = 38,40 reais.
100
1 200
(c) A quantidade de pes de queijo comprada foi de
= 120 pes. Logo, sobraro
10
120 115 = 5 pes de queijo.
161. Ai que fome
(a) Maria possui 50,50+70,25+40,10+50,05 = 2,50+1,75+0,40+0,25 = 4,90
reais.
(b) Tirando a passagem, restam R$ 4,00 para Maria fazer seu lanche. Observe que
Maria no pode escolher uma empada e, se escolher um sanduche, no pode mais
comprar um refrigerante. Assim, Maria s tem as nove seguintes opes de lanche.
Opo 1
Sanduche: R$ 2,20
Suco: R$ 1,20
Cocada: R$ 0,40
Total: R$ 3,80
Opo 5
Pastel: R$ 2,00
Suco: R$ 1,20
Cocada: R$ 0,40
Total: R$ 3,60

154

Opo 2
Sanduche: R$ 2,20
Suco: R$ 1,20
Bombom: R$ 0,50
Total: R$ 3,90

Opo 6
Pastel: R$ 2,00
Suco: R$ 1,20
Bombom: R$ 0,50
Total: R$ 3,70

Opo 3
Sanduche: R$ 2,20
Refresco: R$ 1,00
Cocada: R$ 0,40
Total: R$ 3,60

Opo 7
Pastel: R$ 2,00
Refresco: R$ 1,00
Sorvete: R$ 1,00
Total : R$ 4,00

OBMEP 2010

Opo 4
Sanduche: R$ 2,20
Refresco: R$ 1,00
Bombom: R$ 0,50
Total: R$ 3,70

Opo 8
Pastel: R$ 2,00
Refresco: R$ 1,00
Bombom: R$ 0,50
Total: R$ 3,50

Opo 9
Pastel: R$ 2,00
Refresco: R$ 1,00
Cocada: R$ 0,40
Total: R$ 3,40

Solues do Nvel 1
162. Advinhe Somando 50 + 50 = 100 obtemos trs dgitos e 41 32 = 9 tem um,
portanto, os nmeros procurados no podem ser maiores do que 49, nem menores do
que 42. Como 43 primo (bem como 47), meus nmeros so quaisquer dentre 42, 43,
44, 45, 46, 47, 48 e 49, que no tm divisor comum diferente de 1.
163. Produto de consecutivos Em primeiro lugar, note que se trs nmeros so consecutivos, ento um deles divisvel por 3. Portanto, qualquer nmero que seja o produto
de trs ou mais nmeros consecutivos, deve ser divisvel por 3. Mas, dentre os nmeros
dados, apenas 1 680 divisvel por 3 e, alm disso,
1 680 = 24 3 5 7 = 5 6 7 8.
Logo, 1 680 o nico dentre os trs nmeros dados que pode ser escrito como um
produto de quatro nmeros consecutivos.
164. Palndromos
(a) O prximo 2 112.
(b) O prximo palndromo mpar 3 003.
(c) Para ser primo, o palndromo no pode ter quatro algarismos, pois todo nmero
palndromo de quatro algarismos do tipo abba, que divisvel por 11, j que
abba = a00a + bb0 = a 1001 + b 110 = a 11 91 + b 11 10
= (91a + 10b) 11.
O primeiro nmero palndromo de cinco algarismos 10 001 = 73 137, que no
primo. Os prximos possveis candidatos so
10 101 = 3 367 3 e 10 201 = 101 101.
Assim, o primeiro nmero palndromo primo depois de 929 10 301.
165. O maior MDC Designemos por d o mximo divisor comum dos seis nmeros.
Ento, esses seis nmeros de dois algarismos so mltiplos distintos de d e podemos reformular a pergunta: queremos saber qual o maior nmero d que possui seis mltiplos
distintos menores do que 100.
Note que d, 2d, 3d, 4d, 5d e 6d so todos mltiplos de d. Logo, a melhor situao possvel
quando esses seis nmeros so os mltiplos considerados. Para isso, preciso que
6d 99. Dividindo 99 por 6, obtemos o quociente 16 e o resto 3, ou seja, 99 = 616+3.
Logo, d = 16. Portanto, os seis nmeros de dois algarismos cujo MDC o maior possvel
so 16, 32, 48, 64, 80 e 96. O MDC desses seis nmeros 16.
166. Quantidade de gua na Terra Denotemos V = 1 360 000 000. Lembre que 1% =
1/100, portanto, 1% de V igual a 1 360 000 000/100 = 13 600 000. Segue que:
97% =

97
= 0,97 e 97% de V vale 97 13 600 000 = 1 319 200 000;
100
OBMEP 2010

155

Solues do Nvel 1

1
40 000 000
= 0,0294 = 2,94
= 2,94%;
1 360 000 000
100

1,8% =

1,8
= 0,018 e 1,8% de V vale
100
1,8 13 600 000 = 24 480 000;

0,0096 = 0,96

1
= 0,96% e 0,96% de V vale
100
0,96 13 600 000 = 13 056 000;

250 000
1
= 0,00018 = 0,018
= 0,018%;
1 360 000 000
100
1
0,00001 = 0,001
= 0,001% e 0,001% de V vale
100

0,001 13 600 000 = 13 600.


Assim, a tabela completa a seguinte.
Especificaes
gua salgada
gua doce
Gelo
gua subterrnea
Lagos e rios
Vapor de gua

Volume de gua em km3


1 319 200 000
40 000 000
24 480 000
13 056 000
250 000
13 600

Percentual
97%
2,94%
1,8%
0,96%
0,018%
0,001%

Forma decimal do percentual


0,97
0,0294
0,018
0,0096
0,00018
0,00001

167. Balas Primeiramente, precisamos saber de quantas maneiras podemos obter 14


como soma de trs parcelas inteiras, cada uma delas maior do que ou igual a 3, isto ,
14 = |{z}
. . . + |{z}
. . . + |{z}
... .
3

14 =

14 =
14 =
As parcelas possveis so

14 =

14 =

3
3
3
4
4

+
+
+
+
+

3
4
5
4
5

+
+
+
+
+

8
7
6
6
5

Agora, para cada uma dessas possibilidades, podemos fazer diferentes distribuies
entre as trs crianas, conforme a tabela seguinte. Observe que, quando as trs parcelas
so diferentes, temos seis possibilidades e, quando duas so iguais, temos apenas trs
possibilidades.
156

OBMEP 2010

Solues do Nvel 1
14 = 3 + 3 + 8

14 = 3 + 4 + 7

14 = 3 + 5 + 6

14 = 4 + 4 + 6

14 = 4 + 5 + 5

1a criana
3
3
8
3
3
4
4
7
7
3
3
5
5
6
6
4
4
6
4
5
5

2a criana
3
8
3
4
7
3
7
3
4
5
6
3
6
3
5
4
6
4
5
4
5

3a criana
8
3
3
7
4
7
3
4
3
6
5
6
3
5
3
6
4
4
5
5
4

Assim, temos 3 + 6 + 6 + 3 + 3 = 21 maneiras diferentes de distribuir as balas entre as


trs crianas.
168. Minutos Observemos primeiramente que
5
5
h = 60 min = 50 min,
6
6
de modo que a prova durou 4h50min. Somando as horas e os minutos, obtemos
12h35min + 4h50min = 16h85min.
Mas, 85 min = 1h25min. Logo, a prova termina s 16h85min = 17h25min.
169. Menor nmero Um nmero s divisvel por 4 se o nmero formado pelos seus
dois ltimos algarismos for divisvel por 4. Assim, usando apenas os algarismos 1, 2,
3, 4 e 9, as nicas possibilidades so 12, 24, 32 ou 92. Como 9 o maior algarismo,
devemos coloc-lo o mais direita possvel, de modo que 9 deve ser o algarismo da
casa das dezenas, ou seja, nosso nmero termina com 92. Os outros algarismos 1, 3 e
4, devem aparecer em ordem decrescente esquerda de 92, ou seja, os trs primeiros
algarismos do nmero devem ser 134. Portanto, o nmero procurado 13 492.
170. Contas do papagaio
5

+14

(a) Temos 8 40 54 9 8. Logo, o papagaio grita 8.

(b) Devemos fazer a operao inversa daquela que o papagaio fez, comeando da
ltima operao, ou seja, somar 1 ao nmero, multiplicar o nmero por 6, depois
subtrair 14 e dividir por 5 o resultado:
+1

14

3 4 24 10 2.
Logo, Antnio soprou 2 no ouvido do papagaio.
OBMEP 2010

157

Solues do Nvel 1
+1

14

(c) Observe que 7 8 48 34 6,8. Como 6,8 no um nmero inteiro,


Antnio no vai sopr-lo ao ouvido do papagaio e, mesmo que soprasse, o papagaio
no saberia realizar a primeira operao,
que seria multiplicar
6,8 5.

(d) Quando Antnio sopra um nmero n, o papagaio faz as operaes


5

+14

n 5n 5n + 14

5n + 14 1 5n + 14

1.
6
6

O papagaio s saber calcular a resposta se 5n + 14 for divisvel por 6, ou


seja, se for da forma 6k, com k inteiro no-negativo. Se 5n + 14 = 6k, ento
5n + 2 = 6(k 2) e, multiplicando ambos os lados por 5, resulta
25n + 10 = 6(5k 10), donde n + 24n = 25n = 6(5k 10) 12 + 2, ou seja,
n = 6(5k 12) + 2 24n = 6(5k 12 4n) + 2. Assim, se Antnio sopra um
nmero n da forma 6m + 2, o papagaio faz as operaes
5

+14

6m + 2 30m + 10 30m + 24 5m + 4 5m + 3
e grita o nmero 5m + 3. Se n no for dessa forma, o papagaio permanece mudo.
Logo, Antnio s pode soprar os nmeros
2, 8, 14, 20, 26, 32, 38, . . .
e o papagaio s pode responder, respectivamente,
3, 8, 13, 18, 23, 28, 33, . . . .
171. Soma maior que 34 O maior nmero de quatro algarismos 9 999, cuja soma dos
algarismos 49 = 36. Os nmeros de quatro algarismos cuja soma dos algarismos 35
so 8 999, 9 899, 9 989 e 9 998. Logo, temos cinco nmeros de quatro algarismos
com soma dos seus algarismos maior do que 34, que so os nmeros 8 999, 9 899, 9 989,
9 998 e 9 999.
172. Nenhum 1 Fatorando 111 111, obtemos 111 111 = 3 7 11 13 37. Segue
da que possvel, sim, escrever o nmero 111 111 como um produto de dois fatores,
nenhum deles terminando em 1. Por exemplo, 111 111 = 3 37 037. Mas existem
outras possibilidades, como, por exemplo, 111 111 = 7 15 873.
Na verdade, possvel listar todas as possibilidades. So elas
3 37 037,

7 15 873,

13 8 547,

39 2 849,

77 1 443,

143 777,

33 3 367,

37 3 003,

259 429 e 273 407.

Logo, Roberto tem 10 opes para escrever 111 111 na forma desejada.

173. Nmeros equilibrados Note que se um nmero equilibrado tem os trs algarismos
distintos, diferentes de zero, ento, com os mesmos algarismos, obtemos seis nmeros
equilibrados. Para isso, basta trocar os algarismos de posio. Por exemplo, 123, 132,
213, 231, 312 e 321.
158

OBMEP 2010

Solues do Nvel 1
Se um dos trs algarismos do nmero equilibrado for 0, ento com esses algarismos
obtemos apenas quatro nmeros equilibrados, pois o 0 no pode estar na casa da
centena. Por exemplo, 102, 120, 201 e 210.
Assim, vamos variar apenas os algarismos da centena e da dezena. Como o algarismo da
unidade a mdia desses dois algarismos, esses dois algarismos devem ser ambos pares
ou ambos mpares. Listamos os possveis nmeros equilibrados a partir do algarismo
das centenas.
total de nmeros equilibrados

1 : 111 ;
2 : 201 ;
3:
4:
5:
6:
7:
8:
9:

132
222
333
402

;
;
;
;

153
243
354
444
555
603

;
;
;
;
;
;

174
264
375
465
576
666
777
804

;
;
;
;
;
;
;
;

195
285
396
486
597
687
798
888
999

1 + 4 6 = 25
4 + 1 + 3 6 = 23
1 + 3 6 = 19
4 + 1 + 2 6 = 17
1 + 2 6 = 13
4 + 1 + 6 = 11
1+6 =7
4+1 =5
1

Somando, temos 121 nmeros equilibrados de trs algarismos.


174. Nmeros primos Os nmeros primos entre 70 e 110 so
71,

73,

79,

83,

89,

97,

101,

103,

107 e 109.

Subtraindo 1 de todos esses nmeros, obtemos a lista


70,

72,

78,

82,

88,

96,

100,

102,

106 e 108.

Dessa lista, os mltiplos de 3 so 72, 78, 96, 102 e 108. Logo, os nmeros procurados
so
24 = 72 3, 26 = 78 3, 32 = 96 3, 34 = 102 3 e 36 = 108 3.

De fato, temos 24 3 + 1 = 73, 26 3 + 1 = 79, 32 3 + 1 = 97, 34 3 + 1 = 103 e


36 3 + 1 = 109.
175. Quadro moderno

(a)

(b)

A figura (a) mostra como foi pintado o quadrado nas duas cores, mas ainda no sabemos
qual dessas partes azul ou verde. Para isso, dividimos o quadrado em quatro faixas
verticais, como na figura (b), com o que o quadrado ficou dividido em 16 quadradinhos
iguais. A parte no hachurada compreende
4 meios quadrados + 8 quadrados = 10 quadrados.
|
{z
}
2 quadrados

OBMEP 2010

159

Solues do Nvel 1
Logo, a parte no hachurada corresponde a 10/16 do quadro, ou 5/8 e, portanto, a
parte hachurada corresponde a
16 10
6
3

=
= .
16 16
16
8
Assim, a parte hachurada da figura a que foi pintada de azul e corresponde a 3/8 do
quadro.
176. Encontro de amigos Eu chegarei quando meu relgio marcar 10h05min, uma vez
que penso que meu relgio est adiantado cinco minutos. Como ele est atrasado dez
minutos, chegarei, na verdade, s 10h15min. Meu amigo chegar quando seu relgio
marcar 10h, pois ele pensa que o relgio dele est correto, mas, na realidade, sero
09h55min. Logo, meu amigo chegar vinte minutos antes de mim.
177. Trabalho comunitrio A resposta correta (b).
O nmero total de alunos dessa classe 22 + 18 = 40, dos quais 60% foram prestar
trabalho comunitrio, isto , 0,6 40 = 24. O nmero mnimo de alunas que participaram desse trabalho obtido quando o nmero de alunos que participaram mximo,
ou seja, quando todos os 22 alunos se envolverem no trabalho, restando o mnimo de
duas vagas para as alunas.
178. rea de trapzios A resposta correta (e).
Unindo os quatro trapzios, formamos um quadrado de 50 cm
de lado e, portanto, de 2 500 cm2 de rea. Como o buraco
quadrado tem 30 cm de lado, sua rea de 30 30 = 900 cm2 .
Assim, a rea de cada um dos quatro trapzios, em cm2 , dada
por (2 500 900) 4 = 1 600 4 = 400 .
179. Adivinhao J de incio sabemos que o maior dos dois nmeros
par, por ser o dobro do menor, mas no termina em zero, porque o maior e o
menor nmero no possuem algarismos em comum;
seu algarismo das dezenas 2, no mnimo, porque sua metade um nmero com
dois algarismos e
a soma de seus algarismos 9, no mximo, porque essa soma um dos algarismos
do menor nmero.
Logo, o menor candidato a maior dos dois nmeros 22 e o maior 72. Depois de
22, o nmero par seguinte 24, que desconsideramos porque sua metade 12, que
repete o algarismo 2. J 26 candidato nesse critrio, mas 28 no , por ter soma de
algarismos igual a 10. Continuando at 72, obtemos todos os candidatos, indicados na
tabela seguinte.
maior
menor
160

22
11

26
13

32
16

34
17

36
18

OBMEP 2010

44
22

54
27

62
31

72
36

Solues do Nvel 1
Por verificao, temos que 34 e 17 a nica soluo, tendo sido os dois nmeros em
que pensei.

180. Dezoito nmeros consecutivos Uma sequncia de dezoito nmeros consecutivos


sempre possui dois termos que so mltiplos de 9. A soma dos algarismos de um
mltiplo de 9 sempre um mltiplo de 9. Logo, toda sequncia de dezoito nmeros
consecutivos sempre possui dois termos que so divisveis por 9 e cuja soma de seus
algarismos tambm divisvel por 9. Agora, cada um desses dois nmeros tm trs
algarismos, portanto, os nicos mltiplos de 9 que podem ser a soma dos algarismos
so 9, 18 e 27. No entanto, 999 o nico nmero de trs algarismos cuja soma dos
algarismos 27 e a nica sequncia de dezoito nmeros consecutivos de trs dgitos
que o inclua a sequncia de 982 a 999, que no inclui nmero de trs algarismos com
soma de algarismos igual a 9 e um nico com essa soma igual a 27. Assim, as nicas
possibilidades para as somas dos algarismos dos dois mltiplos de 9 da sequncia so
(i) 9 e 9;

(ii) 9 e 18;

(iii) 18 e 18;

(iv) 18 e 27.

Vejamos alguns exemplos de cada um desses quatro casos.

(i) 9 e 9: um dos nmeros 144 e o outro 135 = 144 9 ou 153 = 144 + 9. Duas
possveis sequncias so
130 , |{z} , |{z} , |{z} , |{z} , |{z}
135 , |{z} , |{z} , |{z} ,
|{z}
1o

2o

3o

4o

5o

6o

7o

8o

9o

144 , |{z} , |{z} , |{z}


147 ; e
|{z} , |{z} , |{z} , |{z} , |{z} , |{z}
10o

11o

12o

13o

14o

15o

16o

17o

18o

141 , |{z} , |{z} , |{z}


144 , |{z} , |{z} , |{z} , |{z} , |{z} ,
|{z}
1o

2o

3o

4o

5o

6o

7o

8o

9o

153 , |{z} , |{z} , |{z} , |{z} , |{z}


158 .
|{z} , |{z} , |{z} , |{z}
10o

11o

12o

13o

14o

15o

16o

17o

18o

(ii) 9 e 18: um dos nmeros 900 e o outro 891 ou 909. Duas possveis sequncias
so
887 , |{z} , |{z} , |{z} , |{z}
891 , |{z} , |{z} , |{z} , |{z} ,
|{z}
1o

2o

3o

4o

5o

6o

7o

8o

9o

900 , |{z} , |{z} , |{z} , |{z}


904 ; e
|{z} , |{z} , |{z} , |{z} , |{z}
10o

11o

12o

13o

14o

15o

16o

17o

18o

898 , |{z} , |{z}


900 , |{z} , |{z} , |{z} , |{z} , |{z} , |{z} ,
|{z}
1o

2o

3o

4o

5o

6o

7o

8o

9o

909 , |{z} , |{z} , |{z} , |{z} , |{z} , |{z}


915 .
|{z} , |{z} , |{z}
10o

11o

12o

13o

14o

15o

16o

17o

18o

(iii) 18 e 18: um dos nmeros 828 e o outro 819 ou 837. Duas possveis sequncias
OBMEP 2010

161

Solues do Nvel 1
so
811 , |{z} , |{z} , |{z} , |{z} , |{z} , |{z} , |{z} , |{z}
819 ,
|{z}
1o

2o

3o

4o

5o

6o

7o

8o

9o

828 ; e
|{z} , |{z} , |{z} , |{z} , |{z} , |{z} , |{z} , |{z} , |{z}
10o

11o

12o

13o

14o

15o

16o

17o

18o

828 , |{z} , |{z} , |{z} ,


823 , |{z} , |{z} , |{z} , |{z} , |{z}
|{z}
1o

2o

3o

4o

5o

6o

7o

8o

9o

837 , |{z} , |{z} , |{z}


840 .
|{z} , |{z} , |{z} , |{z} , |{z} , |{z}
10o

11o

12o

13o

14o

15o

16o

17o

18o

(iv) 18 e 27: um dos nmeros 999 e temos uma nica opo para a sequncia, a
saber,
982 , |{z} , |{z} , |{z} , |{z} , |{z} , |{z} , |{z} , |{z}
990 ,
|{z}
1o

2o

3o

4o

5o

6o

7o

8o

9o

999 .
|{z} , |{z} , |{z} , |{z} , |{z} , |{z} , |{z} , |{z} , |{z}
10o

11o

12o

13o

14o

15o

16o

17o

18o

Analisemos, agora, cada caso. Nos casos (i) e (ii), um dos nmeros divisvel por 9,
que a soma de seus algarismos. No caso (iv), um dos nmeros 999, que divisvel
por 27. Finalmente, no caso (iii), um dos dois mltiplos de 9 necessariamente par,
pois so dois mltiplos consecutivos de 9. Logo, esse nmero um mltiplo de 2 e de
9, portanto um mltiplo da soma de seus algarismos, que 18.
181. Completar uma tabela Observe que em cada quadrado formado por quatro quadradinhos, o nmero que est na parte inferior, direita, a soma dos outros trs
nmeros. Assim, preenchemos a tabela.
0
1
2
3
4

1
2
1+2+2=5
10
17

2
5
2 + 5 + 5 = 12
27
54

3
10
5 + 10 + 12 = 27
66
147

4
3 + 4 + 10 = 17
10 + 17 + 27 = 54
147
A

Logo:
A = 66 + 147 + 147 = 360.

182. Procurando mltiplos de 9 Sempre existe uma diferena que um mltiplo de 9.


De fato, quando dividimos um nmero por 9, podemos encontrar nove restos diferentes,
a saber, 0, 1, 2, 3, 4, 5, 6, 7 ou 8. Logo, entre os dez nmeros do conjunto, pelo menos
dois deles tm mesmo resto quando divididos por 9, j que temos, no mximo, nove
restos diferentes. Quando tomamos a diferena desses dois nmeros que tm o mesmo
resto, obtemos um nmero com resto zero, ou seja, divisvel por 9.
162

OBMEP 2010

Solues do Nvel 1
183. Correndo numa praa A distncia que o atleta
percorre a cada volta completa igual ao permetro da
praa, de 2 900 + 2 600 = 3 000 m.
Como 15,5 km = 15 500 m e 53 000 + 500 = 15 500
m, o atleta d cinco voltas completas (partindo de P e
retornando a P) e ainda corre mais 500 m. Portanto, ele
para no ponto Q, 150 m alm do vrtice B, indicado na
figura.
184. Ovos para um bolo Como os 43 bolos tm a mesma receita, o nmero de ovos
que a doceira precisa um mltiplo de 43. Por outro lado, esse nmero tambm
um mltiplo de 2, 3, 4, 5 e 6, acrescido de 1. O MMC de 2, 3, 4, 5 e 6 60, mas
60 + 1 = 61 no mltiplo de 43. Precisamos, ento, encontrar um nmero com essas
duas propriedades:
um mltiplo de 43;

acrescido de 1 mltiplo de 2, 3, 4, 5 e 6.
Lembre, tambm, que como a receita gasta menos do que nove ovos, o nmero que
estamos procurando menor do que 43 9 = 387. Temos:
60 2 + 1 = 121
60 3 + 1 = 181
60 4 + 1 = 241
60 5 + 1 = 301
60 6 + 1 = 361

no mltiplo de
no mltiplo de
no mltiplo de
mltiplo de 43;
no mltiplo de

43;
43;
43;
43.

Podemos parar por aqui, porque os prximos nmeros sero maiores do que 387. Logo,
a doceira comprou exatamente 301 ovos.
185. Cortando uma cartolina Os lados do retngulo final obtido aps os cortes so,
cada um, a metade dos lados da cartolina original. Assim, o permetro do retngulo
original o dobro do permetro do retngulo final. Logo, o permetro da cartolina
antes do corte media 2129 = 258 cm.
Observao: Ao fazer um corte paralelo a um dos lados do tringulo e pelo ponto
mdio desse lado, o outro corte que formar o retngulo s pode ocorrer no ponto
mdio do outro lado, em vista da semelhana desses tringulos. Assim, o enunciado
contm um dado a mais, desnecessrio para quem reconhece semelhana de tringulos
e suas propriedades.
186. A soma errada primeira inspeo, podemos admitir que os trs algarismos
direita dos nmeros estejam corretos, isto , esto corretos os algarismos 0, 1, 3, 4, 5,
6 e 8. Portanto, dentre os algarismos 2, 7 e 9, um deles est errado. O algarismo 9
est correto, pois se o mudarmos, a soma com 2 no estar certa. Assim, sobram 2
e 7. Se o 7 estivesse errado, ento o 2 estaria correto, mas isso no possvel, pois
1 + 4 + 2 = 7. Logo, o 2 que est errado e deve ser substitudo. Olhando novamente
para a soma 1 + 4 + 2, vemos que o resultado um nmero com o algarismo da unidade
igual a 1. Logo, o algarismo 2 deve ser substitudo quatro vezes pelo 6. Fazendo essa
substituio, verificamos que a soma fica correta.
OBMEP 2010

163

Solues do Nvel 1
187. Nmero de cinco algarismos Para que a b c seja divisvel por 4, seus dois ltimos
algarismos devem formar um nmero divisvel por 4. Como os algarismos so 1, 2, 3,
4 e 5, as nicas possibilidades so b c = 12, b c = 24, b c = 32 e b c = 52. Por outro
lado, os nmeros divisveis por 5 terminam em 0 ou 5. Como 0 no est includo, segue
que d = 5, pois b c d divisvel por 5. Isso exclui a possibilidade bc = 52, porque no
podemos repetir o 5. At agora temos trs possibilidades, a saber,
a245e e a325e.

a 1 2 5 e,

Examinemos esses trs casos para escolher os algarismos a e e, lembrando que no


pode haver repetio.
a325e

a245e

a125e

e=3

e=4

e=1

e=3

e=1

e=4

41 253

31 254

32 451

12 453

43 251

13 254

mltiplo de 3

mltiplo de 3

mltiplo de 3

mltiplo de 3

mltiplo de 3

mltiplo de 3

Logo, o nmero 12 453.


188. Tabela misteriosa Observemos que:
na ltima coluna estaro os mltiplos de 9, porque essa coluna est em branco e
nenhum dos nmeros que aparecem na tabela mltiplo de 9;
na 5a linha estaro os mltiplos de 12, pois nessa linha que aparece o nico
mltiplo de 12 da tabela (a saber, 24);
na 4a coluna estaro os mltiplos de 10, pois 40 o nico mltiplo de 10 na tabela;

na 5a coluna teremos mltiplos de 7, pois 42 e 49 so os nicos mltiplos de 7 na


tabela;
na 2a linha estaro os mltiplos de 7, porque 1 e 7 so os nicos divisores de 49
menores do que 12;
na 3a coluna aparecero os mltiplos de 2, pois 2 o nico divisor comum de 22
e 24 diferente de 1;
na 3a linha aparecero os mltiplos de 11, pois 22 = 2 11 e os mltiplos de 2 j
esto na 3a coluna;

na 6a linha aparecero os mltiplos de 6, pois os divisores de 42 = 2 3 7


menores do que 12 e diferentes de 1 so 2, 3, 6 e 7. Os mltiplos de 2 e 7 j esto
em seus respectivos lugares. Faltam os mltiplos de 3 e 6. Os nicos mltiplos de
6 na tabela so 24 e 42, e 24 j aparece na 5a linha;
na 2a coluna e na 4a linha aparecero os mltiplos de 3 ou 5, pois 15 = 3 5;

na 1a coluna e na 1a linha aparecero os mltiplos de 4 ou 8, pois os divisores


comuns de 32 e 40, menores do que 12 e diferentes de 1, so 2, 4 e 8, mas os
mltiplos de 2 j esto na 3a coluna.

164

OBMEP 2010

Solues do Nvel 1
At aqui, a situao a seguinte.
4 ou 8
32

10
40

14

70

49

63

11

22

10

77

99

12

24

120

84

108

12

60

42

54

4 ou 8

3 ou 5

3 ou 5

15

Examinemos agora as possibilidades que se apresentam.


I - Repetio de ambos 30 e 60

II - Trs nmeros repetidos

8
32

5
20

2
8

10
40

7
28

9
36

4
32

5
40

2
16

10
80

7
56

9
72

56

35

14

70

49

63

28

35

14

70

49

63

11

88

55

22

110

77

99

11

44

55

22

110

77

99

24

15

30

21

27

12

15

30

21

27

12

96

60

24

120

84

108

12

48

60

24

120

84

108

48

30

12

60

42

54

24

30

12

60

42

54

III - Repetio de ambos 12 e 40

IV - Apenas um nmero repetido

8
32

3
12

2
8

10
40

7
28

9
36

4
32

3
24

2
16

10
80

7
56

9
72

56

21

14

70

49

63

28

21

14

70

49

63

11

88

33

22

110

77

99

11

44

33

22

110

77

99

40

15

10

50

35

45

20

15

10

50

35

45

12

96

36

24

120

84

108

12

48

36

24

120

84

108

48

18

12

60

42

54

24

18

12

60

42

54

Logo, a nica soluo a da tabela IV.


189. Habitantes e esporte O total de habitantes desta cidade praticamente 30 000 e
divisvel por 9 e 15. Logo, deve terminar em 0 ou 5 e a soma de seus algarismos deve
ser um mltiplo de 9. Como 29 970 o maior nmero que menor do que 30 000 e tem
fatores 9 e 15, podemos supor que essa seja a populao total da cidade. Logo,
2
29 970 = 3 996 e
15

2
29 970 = 6 660
9

OBMEP 2010

165

Solues do Nvel 1
o nmero de mulheres e de homens, respectivamente, que praticam esporte somente
nos fins de semana. A tabela dada indica que 8 563 + 7 582 = 16 145 pessoas no
praticam esporte. Logo, a cidade tem 16 145 5 = 3 229 pessoas que praticam esporte
regularmente e, portanto, 3 229 1 252 = 1 977 pessoas do sexo feminino praticam
esporte regularmente. A tabela completa a seguinte.
No praticam esporte
fem.
8 563

masc.
7 582

Praticam esporte
somente nos fins
de semana
fem.
masc.
3 996
6 600

Praticam
esporte
regularmente
fem. masc.
1 977 1 252

Populao
total
29 970

190. Botes luminosos A resposta correta (c).


A tabela mostra a cor de cada boto em cada etapa.
incio
apertando boto 1
apertando boto 3
apertando boto 5

1
azul
verde
verde
verde

2
azul
verde
azul
azul

3
azul
azul
verde
verde

4
azul
azul
verde
azul

5
azul
azul
azul
verde

6
azul
azul
azul
verde

7
azul
azul
azul
azul

8
azul
verde
verde
verde

Logo, os botes que ficaram com luzes verdes acesas no final so 1, 3, 5, 6 e 8, o que
nos d um total de cinco botes.
191. Qual o nmero? O problema determinar os algarismos b, c, d, e
e f tais que o nmero b c d e f 1 seja o triplo de 1 b c d e f.

1bcdef
3
.....................................................................
bcdef 1

De incio vemos que f = 7 e, a partir da, podemos ir descobrindo cada um dos


algarismos, como segue.
1bcde7
3
.....................................................................
bcde71

1bcd57
3
.....................................................................
bcd571

1bc857
3
.....................................................................
bc8571

1b2857
3
.....................................................................
b28571

Portanto, b = 4 e o nmero de partida 142 857.


192. Jardim variado Os tringulos 1, 2, 5 e 6 so retngulos, de modo que, para calcular suas reas, vamos enxergar
cada um deles como metade de um retngulo. Para que a
nossa estratgia funcione, precisamos saber dividir o terreno
N
retangular em retngulos menores.
Subdividimos o terreno em dezesseis retngulos de 15 por
40 m, como mostra a figura, cada um com uma rea de 1540
= 600 m2 . Ento temos que
a rea do tringulo 1 = rea do tringulo 5 =
a rea do tringulo 2 =
166

1
3

1
4 600 = 1 200 m2 ;
2

1
6 600 = 1 800 m2 e
2
OBMEP 2010

P
6

Solues do Nvel 1
a rea do tringulo 6 =

1
2 600 = 600 m2 .
2

Observe que a rea do tringulo 4 igual rea do terreno todo, subtrada das reas
dos tringulo 5 e 6 e da rea da regio esquerda de MR. Contando retngulos, vemos
que essa rea mede 10 600 = 6 000 m2 . Logo, a rea do tringulo 4 dada por


120 80 1 200 + 600 + 6 000 = 9 600 7 800 = 1 800 m2 .
Finalmente, a rea do tringulo 3 a rea total do terreno subtrada da soma das reas
j calculadas dos outros cinco tringulos, ou seja,
120 80 (2 1 200 + 2 1 800 + 600) = 9 600 6 600 = 3 000 m2 .
Para que o gasto seja o menor possvel, as flores mais caras devem ser plantadas nas
regies menores. Como a menor regio a 6, nela deve ser plantada a flor mais cara, a
rosa, gastando 3,50 600 = 2 100 reais. A maior regio a 3, onde deve ser plantada
a flor mais barata, o bem-me-quer, gastando 0,80 3 000 = 2 400 reais.

Nas regies 1 e 5, com reas iguais a 1 200 m2 , devem ser plantadas bromlias e cravos, contribuindo com (3,00 + 2,20) 1 200 = 6 240 reais. Nas regies 2 e 4, com
reas iguais a 1 800 m2 , devem ser plantadas margarida e violeta, contribuindo com
(1,20 + 1,70) 1 800 = 5 220 reais.
Temos, ento, quatro diferentes maneiras de formar o jardim, mantendo o mesmo gasto
mnimo de 2 100 + 2 400 + 6 240 + 5 220 = 15 960 reais. Apresentamos a seguir uma das
quatro possibilidades de escolhas das flores com esse oramento mnimo.
Regio
1
2
3
4
5
6

rea m2
1 200
1 800
3 000
1 800
1 200
600

Flor
bromlia
margarida
bem-me quer
violeta
cravo
rosa

Preo m2
3,00
1,20
0,80
1,70
2,20
3,50

Total por flor


3,00 1 200 = 3 600
1,20 1 800 = 2 160
0,80 3 000 = 2 400
1,70 1 800 = 3 060
2,20 1 200 = 2 640
3,50 600 = 2 100
TOTAL: 15 960

193. O algarismo 3 Vejamos todas as vezes que Luis escreveu o algarismo 3:


3 1;

13, 23, 30, 31, 32, 33, . . . , 39, 43, . . . , 93 2 + 11 + 6 = 19.


| {z } |
{z
} |
{z
}
2

11

At aqui, ele escreveu vinte vezes o algarismo 3. Da temos

103 , |{z}
113 , |{z}
123 , |{z}
130 , |{z}
131 .
|{z}
21a

22a

23a

24a

25a

Logo, ao escrever o nmero 131, ele escreveu o algarismo 3 pela 25a vez.
OBMEP 2010

167

Solues do Nvel 1
194. Soma de potncias Existe um padro para o algarismo das unidades de uma
potncia de 3: ele tem perodo 4, pois se repete de quatro em quatro vezes. De fato,
temos
3
35 = 243
32 = 9
36 = . . . 9
33 = 27
37 = . . . 7
34 = 81
38 = . . . 1
Como 444 mltiplo de 4, o algarismo das unidades de 3444 1.
Analogamente, o algarismo das unidades de potncias de 4 tem perodo 2. De fato,
temos
41 = 4
43 = 64
42 = 16
44 = 256
Lembrete: Todo nmero
terminado em 0 ou 5 divisvel por 5.

Como 333 mpar, o algarismo das unidades de 4333


4. Portanto, o algarismo das unidades de 3444 +4333
1 + 4 = 5, de modo que ele divisvel por 5.

195. Telefonemas Como Joo telefona para seus pais a cada trs dias, podemos montar
uma tabela indicando os dias da semana em que ocorreram os quatorze primeiros
telefonemas de Joo.
Domingo Segunda Tera Quarta Quinta Sexta Sbado
1o

6o

4o

2o

7o

5o

3o

8o
13o
11o
9o
14o
12o
10o
Analisando a primeira linha dessa tabela, percebemos que so sete telefonemas, um em
cada dia da semana e que, a partir do stimo telefonema, os dias comeam a se repetir.
Isso implica que os nmeros que aparecem na segunda linha da tabela so obtidos dos
nmeros que aparecem na primeira linha somando 7. Por exemplo, Joo telefonar
para seus pais aos domingos nos telefonemas de nmeros
1
1+7 =8
8 + 7 = 15
15 + 7 = 22
22 + 7 = 29
29 + 7 = 36
..
.
ou seja, nos nmeros que deixam resto 1 quando divididos por 7. Com esse raciocnio,
podemos determinar o dia da semana em que cai uma ligao, analisando o resto da
diviso do nmero do telefonema por 7.
Domingo
1
8
..
.

Segunda
6
13
..
.

Tera
4
11
..
.

resto 1

168

resto 6

resto 4

Quarta Quinta
2
7
9
14
..
..
.
.

resto 2

OBMEP 2010

resto 0

Sexta
5
12
..
.

Sbado
3
10
..
.

resto 5

resto 3

Solues do Nvel 1
Dividindo 100 por 7, obtemos 100 = 7 14 + 2. Logo, o resto da diviso de 100 por 7
2 e segue que o centsimo telefonema ocorre numa quarta-feira.
196. O maior produto Observe que obtemos o maior resultado possvel se um dos
nmeros comear com o algarismo 5 e o outro com 4. Alm disso, como s temos cinco
algarismos, um dos dois nmeros deve ter somente um ou dois algarismos. Vejamos as
possibilidades que do o maior produto.
um dos fatores tem um algarismo:

bom usar
uma
calculadora.

5 3214 = 21 284 ; 4 3215 = 21 605.


um dos fatores tem dois algarismos:

532 41 = 21 812 ; 531 42 = 22 302 ; 521 43 = 22 403 ;


432 51 = 22 032 ; 431 52 = 22 412 ; 421 53 = 22 313.
Logo, o melhor resultado 431 52 = 22 412.
197. O caminho da Joaninha Os nmeros primos que aparecem na tabela so 23,
73, 37, 17, 79, 19, 37, 53 e 251. Logo, s h dois caminhos que Dona Joaninha pode
percorrer. Um o apresentado na figura. O outro idntico, exceto que o azulejo 87
fica esquerda, passando entre 87 e 231 e, depois, seguindo horizontalmente.

C
198. O lugar dos amigos Observe que 3 o nico nmero dentro das trs figuras e 1
o nico que no est dentro de um polgono, logo Celina 3 e Fbio 1. Agora, 4
o nico nmero dentro do tringulo e do crculo, logo Elisa 4. Nessa situao, 5
o nico dentro do tringulo, mas no do quadrado, assim Diana 5. Finalmente, 7
o nico nmero dentro de uma nica figura, logo Bento 7. Resta, ento, 2 dentro
do crculo, portanto, Guilherme 2 e Ana 6.
.................................
........
.....
.....
....
....
.
...
.................................................................
.
.
.
...
.
.
...
.
.
.
..
...
...
... ...
..
...
...
........ .....
..
... ...
...
.
...
.
..
.... ..
.
...
.
.
.
...
.
.... ..
...
... ...
.
.
.
.
...
.....
....
.. ...
.
.
.
...
.
.
.... ... ..
... .....
.
........
.
.
...
.
.
....
........................................
.
.
.
.
.
.................................................................
.
.
.
.
...
..
.
.
.
.
.
....
..
.
.
.
.
......................................................................................

4
5

.................................
.......
.....
.....
....
....
.
...
..................................................................
.
.
.
...
.
.
...
.
.
.
..
...
...
... ...
..
. ... ...
..
...
.
..
..
..... ....
.
.
...
.
...
....
..
..
.
.
.
...
.
.
.
...
.
.... ..
... ...
...
.
.
.
.
...
......
.. ...
....
.
.
.
...
.
.
.... .... ..
... .....
.
.
........
.
...
.
.
....
. ....................................
.
.
.
.
.
.................................................................
.
.
.
.
...
..
.
.
.
.
.
....
..
.
.
.
.
.......................................................................................

E
D

OBMEP 2010

...............................
.........
.....
....
.....
..
....
...
.................................................................
.
...
....
...
...
..
...
...
... ...
..
...
...
... ..... .....
..
...
... ...
.
...
.
..
.... ..
...
.
.
.
.
...
.
.... ..
...
... ...
.
.
.
.
...
. .
.....
....
.
...
.... ...... ...
.
... ......
.........
.
.
.
.
...
.
.
...
........................................
.
.
.
.
.
.................................................................
.
.
.
.
...
..
.
.
.
.
.
....
..
.
.
.
.
......................................................................................

E A
D

169

Solues do Nvel 1
199. Quadrado perfeito? Lembre que um nmero um quadrado perfeito se na sua
decomposio em fatores primos os expoentes so todos pares. Por exemplo,
54 76 132 um quadrado perfeito, pois igual a (52 73 13)2 .
Como nenhum nmero elevado ao quadrado termina em 3, segue que N1 = 333 . . . 3
no um quadrado.
Temos que N2 = 666 . . . 6 = 2 333 . . . 3. Como 333 . . . 3 mpar, ento na decomposio de N2 em fatores primos aparece s um fator 2. Logo, N2 no um quadrado.
Vejamos a divisibilidade por 3. A soma dos algarismos desses nmeros
N3 50 15 = 750

N4 50 21 = 1 050
N5 50 27 = 1 350

Como todas essas somas so divisveis por 3, essas trs somas tambm so divisveis
por 3. Logo, se algum deles fosse um quadrado perfeito, teria que ser divisvel por 9.
A soma dos algarismos de N3 e N4 no divisvel por 9, logo esses dois nmeros no
so divisveis por 9 e, consequentemente, no so quadrados perfeitos.
Como 1 350 divisvel por 9, ento N5 divisvel por 9. Temos
2727272727 . . . 27 9 = 303030 . . . 03
e
303030 . . . 03 3 = 101010 . . . 01,

portanto,

2727272727 . . . 27 = 32 303030 . . . 03 = 33 101010 . . . 01.


Note que 101010 . . . 01 tem 49 algarismos, dos quais 25 so iguais a 1 e os outros iguais
a 0. Logo, a soma de seus algarismos 25 e, portanto, no divisvel por 3. Assim,
2727272727 . . . 27 divisvel por 33 , mas no por 34 . Assim, conclumos que tampouco
N5 um quadrado perfeito.
200. Preenchendo quadradinhos A operao equivalente a

+

= 4

portanto, o lado esquerdo da igualdade um mltiplo de 4. Usando apenas os nmeros


1, 2, 3, 5 e 6, possvel verificar que as nicas possibilidades so
+

2 = 4 1

ou

1 = 4 2

1 = 4 2

Da, podemos concluir que


3 + 5 6

170

2 = 4 1

ou

OBMEP 2010

6 + 5 3

Solues do Nvel 1
so as nicas possibilidades de preenchimento.
1

201. Os trs nmeros Como 13 983 termina em 3, a soma dos


algarismos das unidades dos trs nmeros diferentes deve ser
13 ou 23. Como 23 no pode ser obtido na soma de 1, 2, 4 e
7, s temos uma opo, a saber, 2 + 4 + 7 = 13 .

2
4
7

........................................................................................................................................

8 3

Agora, a soma dos algarismos das dezenas deve ser 8 1 = 7 e, portanto, s pode ser
1 + 2 + 4 = 7. Completamos os algarismos das dezenas, tendo o cuidado de no repetir
o mesmo algarismo num mesmo nmero. Temos somente as trs opes seguintes.
1

1
2
4

2
4
7

4
1
2

........................................................................................................................................

1 3

2
4
7

4
2
1

........................................................................................................................................

8 3

2
4
7

........................................................................................................................................

8 3

9 8

Os algarismos das centenas devem somar 9, o que nos deixa duas possibilidades, 4+4+1
ou 1 + 1 + 7. Como nas trs opes o algarismo 4 ocorre em dois dos trs nmeros,
escolhemos a possibilidade 1 + 1 + 7 para a centena, para que no aparea repetido o
algarismo 4. Tambm precisamos cuidar para que no apaream repetidos o 1 e o 7,
o que elimina a terceira opo acima e nos leva a duas opes para as centenas, como
segue.
1

.................................................................................................................................................

1 3

.................................................................................................................................................

9 8

Finalmente, os algarismos das unidades de milhar devem somar 13 e fcil escolh-los.


Assim, Sofia pode chegar a 13 983 de duas maneiras, como segue.
1

.................................................................................................................................................

1 3

OBMEP 2010

.................................................................................................................................................

9 8

171

Solues do Nvel 1
202. Preencher uma tabela Existem vrias maneiras de preencher a tabela, dependendo
da casa que escolhemos para ser preenchida, o que pode ser feito de vrias maneiras.
Vejamos um exemplo de como preencher a tabela. Inicialmente,
temos quatro casas que podem ser preenchidas, todas marcadas com
X. Escolhemos uma delas e preenchemos de acordo com a segunda
X X X
regra. Repetimos esse processo at a tabela estar completamente
1 2 X
preenchida.

3
1

3
1

4
2

3
1

4
2

Mas, para colocar em cada casa o maior nmero possvel, a idia , a cada vez, examinar
todas as casas que podem ser preenchidas e s preencher a casa em que podemos colocar
o maior nmero. Se em duas dessas casas o nmero a ser colocado for o mesmo,
preencheremos a que tem o menor nmero de casas vizinhas j preenchidos. Vamos l!

3
1

3
1

6
2

9
3
1

18
6
2

27
9
3
1

54
18
6
2

54
18
6
2

72
144

27
9
3
1

54
18
6
2

72
144

216

27
9
3
1

27
9
3
1

54
18
6
2

72
144

216
432
576

27
9
3
1

54
18
6
2

72
144
1178

216
432
576

27
9
3
1

54
18
6
2

72
144
1 178
3 516

216
432
576
1 754

9
3
1

6
2
54
18
6
2

72

27
9
3
1

27
9
3
1

54
18
6
2

72
144

216
432

27
9
3
1

54
18
6
2

72
144
1 178

216
432
576
1 754

Logo, o maior nmero que pode ser escrito na tabela 3 516.


203. Olimpada de Pequim Para iniciar, escolhemos um lugar para um dos atletas,
digamos, para Maria.
......................................................................
...
...
..
..
..
..
....
....
..
..
....
....
...
...
.....
.....
...
...
...
.
....................................................................

172

Maria

OBMEP 2010

Solues do Nvel 1
(a) Quem pratica natao est esquerda de Maria. Logo, s podemos ter a configurao abaixo.
....................................................................
...
...
.
.
.....
.....
...
...
...
...
...
...
...
...
....
....
...
...
...
...
...
.
..................................................................

Maria

Natao

(b) Quem pratica ginstica est frente de Juan. Existem duas nicas possibilidades:
Maria pratica ginstica ou Maria no pratica ginstica.
Maria pratica ginstica

Juan

.................................................................................
..
...
..
...
....
...
...
...
...
....
....
..
..
...
...
..
..
....
....
...
...
...
...
.
...
.............................................................................

Maria no pratica ginstica


Ginstica

.................................................................................
..
...
..
...
....
...
...
...
...
....
....
..
..
...
...
..
..
....
....
...
...
...
...
.
...
.............................................................................

Maria
Ginstica

Natao

Maria

Natao
Juan

(c) Como Tnia e David sentaram-se juntos, ento somente a segunda opo do item
anterior Maria no pratica ginstica pode satisfazer essa condio. Ela gera
as seguintes duas possibilidades.
Maria no pratica ginstica

Maria no pratica ginstica

David
Ginstica
Tnia

................................................................................
...
...
..
....
...
...
...
...
...
....
....
...
...
...
...
...
...
...
...
..
..
...
...
..
...
..............................................................................

Tnia
Ginstica
Maria

David

Natao
Juan

................................................................................
...
...
..
....
...
...
...
...
...
....
....
...
...
...
...
...
...
...
...
..
..
...
...
..
...
..............................................................................

Maria

Natao
Juan

(d) Como uma mulher sentou-se ao lado de quem pratica vlei, a segunda opo
acima que a correta, e temos duas possibilidades para o atleta que pratica
atletismo: David ou Maria.
Tnia
Ginstica

.................................................................................
...
...
..
..
...
...
...
...
...
...
.....
.....
...
...
...
...
...
...
..
..
.....
.....
.
..
..............................................................................

David
Atletismo

Tnia
Ginstica

Maria
Vlei

David
Vlei

Natao
Juan

.................................................................................
...
...
..
..
...
...
...
...
...
...
.....
.....
...
...
...
...
...
...
..
..
.....
.....
.
..
..............................................................................

Maria
Atletismo

Natao
Juan

204. Culturas diferentes


(a) (i) 03/12 significa 12 de maro para Ralph e 03 de dezembro para Jorge, portanto,
uma data ambgua.
OBMEP 2010

173

Solues do Nvel 1
(ii) 18/08 s pode ser mesmo 18 de agosto.
(iii) 05/05 s pode ser 05 de maio.
Logo, (i) uma data em que eles no podem se escrever.
(b) A data s ambgua quando o nmero do dia tambm puder representar o nmero
do ms, logo quando um nmero de 1 a 12. Por outro lado, nesses nmeros no
h ambiguidade quando o nmero do ms for igual ao nmero do dia. Por exemplo,
05/05 s pode ser 05 de maio. Por isso, em cada ms, eles devem evitar 11 dias.
Logo, os perodos mais longos em que eles no podem se escrever ocorrem em 11
dias consecutivos de janeiro de 02 a 12 de janeiro e em dezembro de 02 a 12
de dezembro. Observe que nos outros meses os perodos em que eles no podem
se escrever so menores. Por exemplo,
em abril eles no podem se escrever de 01/04 a 12/04, exceto em 04/04;
em setembro eles no podem se escrever de 01/09 a 12/09, exceto em 09/09.
205. Uma liquidao Na liquidao, exceto aos sbados, os produtos esto 50% mais
baratos. Nos sbados, com o desconto adicional de 20%, os produtos esto custando
80% dos preos fora dos sbados, ou seja
80% de 50% =

80
50
40

=
= 40% do preo original. .
100 100
100

Logo, Roberta deixou de economizar 60%, que corresponde aos R$ 50,40. Como
60% 50,40,
10% 50,40 6 = 8,4 e
100% 8,4 10 = 84,00,
o preo da cala antes da liquidao era de R$ 84,00.
206. Nmero com muitos zeros A resposta correta (d).
Vamos comparar os cinco nmeros sem efetuar clculos. Temos
3 + a = 3,000 . . . 0001 menor do que 4;
3a
3a
3
a
a
3

menor do que 3;
= 0,000 . . . 0003 menor do que 1;
3
3
=
=
= 3 102010 maior do que 10 e
1
0,000 . . . 0001
102010
0,000 . . . 0001
=
menor do que 0,000 . . . 0001 .
3

Assim, 3/a representa o maior nmero.


207. Corrida das tartarugas Vamos representar cada tartaruga numa reta, utilizando
sua letra inicial. Os dados finais da corrida esto representados na figura dada.
174

OBMEP 2010

Solues do Nvel 1
25

z
}|
{
Ss Ps Os
Rs Es
| {z }
|{z}
5 |
{z
}10

25

Logo, Sininha est 20 m frente de Elzinha e, portanto, Pulinha est 5 m frente de


Sininha. A ordem de chegada O, P, S, E e R.
208. Que memria... O nmero comea com 25 porque 52 a nica potncia de 5 com
dois algarismos.
2

Os candidatos aos dois ltimos algarismos so as potncias de 2 com dois algarismos,


a saber, 16, 32 e 64. Como 32 no serve, por apresentar o 2 repetido, temos as opes
2

ou

4 .

O algarismo do meio um mltiplo de 3, portanto, s pode ser 3, 6 ou 9, mas o 6


no pode ser repetido. Para escolher entre as duas opes acima, basta lembrar que
a soma dos cinco algarismos deve ser mpar e, como 2 + 5 mpar, a soma dos trs
ltimos deve ser par. Assim, a segunda opo acima fica descartada, pois no podemos
complet-la com um mltiplo de 3, restando, apenas os nmeros
2
O maior dos dois, 2

ou

6 .

6 , o cdigo bancrio de Esquecinaldo.

209. Uma frao irredutvel Para que a frao seja irredutvel, o numerador e o denominador no podem ter fator comum. Comeamos calculando os fatores primos de
N = 2 3 4 5 10, que so
2 3 |{z}
4 5 |{z}
6 7 |{z}
8 |{z}
9 |{z}
10 .
22

23

23

25

32

Logo, a decomposio de N em fatores primos dada por


N = 28 34 52 7.

Podemos escolher diversas fraes que satisfazem o problema, como segue.


1
.
52 7
(ii) Se o numerador tem apenas um fator de N, temos as quatro fraes
(i) Se o numerador 1, temos a frao

28
;
34 52 7

28

34
;
28 52 7

34

52
28 34 7

OBMEP 2010

28

7
.
34 52
175

Solues do Nvel 1
(iii) Se o numerador tem dois fatores de N, temos as seis fraes
28 34 28 52 28 7 34 52 34 7
; 4
;
;
;
52 7
3 7 34 52 28 7 28 52

52 7
.
28 34

(iv) Se o numerador tem trs fatores de N, temos as quatro fraes


28 34 52
;
7

28 34 7
;
52

(v) Se o numerador N, temos a frao

28 52 7
34

34 52 7
.
28

28 34 52 7
.
1

Assim, ao todo, temos dezesseis dessas fraes irredutveis.


210. Transformar em decimal Temos:
5
1
2
14 20
34
+ 16
=
+
=
= 11 + = 11,3333 . . .
3
12
3
3
3
3




5
3
6
1
=5 2
= 5 = 4 = 3, 8
(b) 5 2
3
5
5
5
2
2
2
5
10
1
(c) 1 +
=1+
= 2 + = 2,25
3 = 1+
3 = 1+ 8 = 1+2
8
8
4
1 + 1+4
1+ 5
5
(a) 7

211. Uma sequncia especial Observe que:


os nmeros de 1 a 9 ocupam nove posies;

os nmeros de 10 a 99 ocupam 2 90 = 180 posies;

os nmeros de 100 a 199 ocupam 3 100 = 300 posies;

os de 200 a 299 ocupam 3 100 = 300 posies;

os de 300 a 399 ocupam 3 100 = 300 posies; etc.


100 , . . . 199 , 200 , . . . , 299 , 300 , . . . , 399 , 400 , . . . , 499 , 500 , . . . , 599 , 600 , . . . , 699
|
{z
} |
{z
} |
{z
} |
{z
} |
{z
} |
{z
}
3100=300

3100=300

3100=300

3100=300

3100=300

3100=300

Assim, os algarismos usados para escrever de 1 a 699 ocupam 9 + 180 + 6 300 = 1 989
posies, logo faltam 2 009 1 989 = 20 posies. Como 20 = 3 6 + 2, precisamos
ainda escrever de 700 a 706, obtendo 21 posies, com o algarismo 6 ocupando a posio
21. Logo, o algarismo 0 que que ocupa a 2009a posio.
6

212. Cortar um retngulo Dividimos o retngulo


em 13 7 quadradinhos de 1 cm de lado cada
um. Agora, usamos que 13 = 1 + 3 + 4 + 5 =
6 + 7 = 0 + 13 para obter a diviso em 13 retngulos diferentes. Voc consegue encontrar outras
formas de fazer essa diviso?
176

OBMEP 2010

...............................................................................................................................................................................................................................
...
....
....
.
....................................................................................................................................................................................................................................
..
...
..
...
...
...
...
...
...
...
...
...
...
...
..
................................................................................................................................................................................................................................
...
...
...
..
......................................................................................................................................................................................................................................
...
...
...
...
...
...
...
...
...
...
..
..
..
....
....
..
.
...
...
...
.........................................................................................................................................................................................................................................
..
...
...
...
..
...
..
..
..
..
......................................................................................................................................................................................................................................

1
2
1
2
1

Solues do Nvel 1
213. Medida de ngulo A resposta correta (b).
b + C OE
b = 90 e C OE
b = D OY.
b Logo, AOC
b = 90 D OY.
b Como
Temos que AOC

b
b
D OY est entre 40 e 50 , segue que AOC est entre 90 50 = 40 e 90 40 = 50 .

214. Permetros e reas A rea do quadrado ( 3+3)2 = 3 +23 3+32 = 12+6 3


e a do retngulo

( 72 + 3 6) 2 = 144 + 3 12 = 12 + 6 3 .

Logo, eles tm a mesma rea. Vamos agora comparar os permetros. O do quadrado

4 ( 3 + 3) = 4 3 + 12

e o do retngulo

(
72
+
3
6
+
2)
=
2

(6
2
+
3
6
+
2)
=
6
6
+
14
2.

Como 4 3 < 6 6 e, tambm, 12 < 14 2, segue que 4 3 + 12 < 6 6 + 14 2. Assim,


o retngulo tem o maior permetro.
215. Clculo de ngulo Como AB = AC, o trinb = ACB.
b
gulo ABC issceles, logo ABC
Sendo
AD = BD, o tringulo ABD tambm issceles, logo
b = B AD.
b Temos, ento,
ABD
b = ABC
b = ABD
b = B AD
b .
ACB

.....
....... .....
..... .. ..........
..... ..
.
..... ....
.
..........
.
.
.
..
.....
..
.....
.....
..
....
.
.
.....
.
.
.
.
.....
.....
.....
...
.....
.
.
.
.....
.
.
.
.
.....
....
.
.
.
.
.
.....
.
...
.
.
.
.....
.
.
.
.
...
.....
.
.
.
.
.
.
......................................................................................................................................................................................

39

Na figura, esses trs ngulos iguais esto representados pela letra . Os ngulos internos
de ABC so + 39 , e . Logo, + 39 + + = 180 , ou seja, 3 = 180 39 =
b = = 47 .
141 . Assim, B AD
Lembrete 1: Os
ngulos da base de
um tringulo issceles so iguais:

Lembrete 2: A
soma dos ngulos
internos de um tringulo 180:

b=C
b
B

b+ B
b+C
b = 180 .
A

e AB = AC.

216. O caminho da formiga A resposta correta (c).

OBMEP 2010

177

Solues do Nvel 1
217. Menino mentiroso Claramente, Pedrinho encontrou Joozinho num dia em que
ele mente. O sbado est descartado pois, caso contrrio, ele estaria falando a verdade.
Assim, o encontro entre eles foi numa tera ou quinta-feira. No pode ter sido numa
tera-feira, porque ento o dia seguinte no poderia ser uma quarta. Logo, a nica
possibilidade para o dia do encontro dos dois quinta-feira.
218. Encontre os quatro nmeros Como os nmeros 1, 2, 3 e 6 satisfazem a propriedade, fcil verificar que, dado qualquer nmero inteiro n, os mltiplos n, 2n, 3n e 6n
de n tambm satisfazem a propriedade. Como estamos procurando nmeros de trs
algarismos e 999 6 = 166,5, basta considerar qualquer valor de n entre 100 e 166 para
obter quatro nmeros de trs algarismos com a propriedade notvel.
219. Colando seis tringulos

1/8
1/16 V
IV
1/32

VI

III

1/4

1
I

II
1/2

O permetro da figura formada por treze segmentos, na sequncia de formao dos


tringulos, que podem ser descritos como segue.
1
cm no tringulo I,
2
1
1
1 segmento de cm e 1 segmento de cm no tringulo II,
2
4
1
1
1 segmento de cm e 1 segmento de cm no tringulo III,
4
8
1
1
1 segmento de cm e 1 segmento de
cm no tringulo IV,
8
16
1
1
1 segmento de
cm e 1 segmento de
cm no tringulo V e
16
32
1
2 segmentos de
cm no tringulo VI.
32
Soluo 1: Contando os comprimentos de segmentos, podemos ver que o permetro
mede
1
1
1
1
1
21+2 +2 +2 +2
+3
2
4
8
16
32
1 1 1
3
16 + 8 + 4 + 3
=2+1+ + + +
=3+
2 4 8 32
32
127
31
=3+
=
cm.
32
32
2 segmentos de 1 cm e 1 segmento de

178

OBMEP 2010

Solues do Nvel 1
Soluo 2: O contorno da figura, comeando no canto esquerdo e seguindo no sentido
anti-horrio, mede
1+

1 1 1
1
1
1
1
1
1 1 1
+ + +
+
+
+
+
+ + + +1
2 4 8 16 32 32 32 16 8 4 2

1
1
centmetros. A soma da PG de primeiro termo 1, razo e ltimo termo
dada
2
32
por

1
1
1
1 =2
1
.
64
2
32
Logo, o permetro da figura mede




1
1
1
1
127
2
+
= 4
+ 2
=
cm.
32
32
32
32
32

Soluo 3: Observe que cada vez que agregamos um tringulo de lado a, trocamos
um segmento de comprimento a do permetro por dois segmentos de comprimento a,
de modo que o permetro aumenta em a.
Como o primeiro tringulo tem permetro de 3 cm, agregando um tringulo de lado
1
1
cm, a nova figura tem um permetro de 3 + cm; se agregamos mais um tringulo
2
2
1
1
1
de lado
cm, a nova figura tem permetro 3 + + cm. Seguindo esse processo,
4
2
4
depois do sexto tringulo, a figura tem permetro de
3+

1
1
1
127
1 1 1
+ + +
+
=3+1
=
cm,
2 4 8 16 32
32
32

1
1
1
onde usamos a soma da PG de primeiro termo , razo e ltimo termo , dada por
2
2
32

1
1
1
1

1 =1
.
2 64
2
32
220. Os livros da Elisa Seja N o nmero total de livros da Elisa. Como N + 1 um
mltiplo de 9 e 4, temos que N + 1 um mltiplo de 36. Logo, N + 1 36 ou 72, pois
Elisa tem menos do que 100 livros. Se N = 35, ento o nmero de livros de matemtica
36 9 1 = 3 e o nmero de livros de literatura 36 4 = 9. Mas, ento, Elisa
teria 24 + 3 + 9 = 36 livros, o que impossvel, porque 36 maior do que 35. Assim,
N = 71 e Elisa tem 72 9 1 = 7 livros de matemtica.
221. Substituindo pela soma Sabemos que qualquer nmero e a soma de seus algarismos
sempre deixam o mesmo resto quando divididos por 9. Assim, Mrcio substitui o
nmero inicial por outro, muito menor, com o mesmo resto na diviso por 9, e continua
assim, at chegar num nmero de um nico algarismo que, evidentemente, igual ao
resto da diviso de todos os nmeros obtidos anteriormente inclusive do primeiro
por 9. Assim, o que Mrcio faz , to somente, um processo de um passo apenas, que
consiste na substituio de nmeros naturais por seus restos na diviso por 9.
OBMEP 2010

179

Solues do Nvel 1
(a) Como 32 009 = 32 008 3 = (32 )1 004 3 = 91 004 3, o resto da diviso de 32 009 por
9 0. Logo, o nmero final do processo de Mrcio 9.
(b) Observe que 172 = (18 1)2 = 182 2 9 + 1 = mltiplo de 9 + 1. Logo,
172 008 = (172 )1 004 = mltiplo de 9 + 1
e, portanto, 172 009 = mltiplo de 9 + 17 = mltiplo de 9 + 8. Logo, o nmero
final do processo de Mrcio 8.
(c) Aplicando o processo aos nmeros da lista dos nmeros naturais 1, 2, 3, 4, 5, 6,
7, 8, 9, 10, 11, 12, . . . , a lista final sempre 1, 2, 3, 4, 5, 6, 7, 8, 9, 1, 2, 3, . . . .
Como o resto da diviso do nmero 20 092 009 por 9 4, ento o ltimo nmero
da lista final 4 e os seis ltimos algarismos da lista final so . . . , 8, 9, 1, 2, 3, 4.
Portanto, essa lista tem os quatro algarismos 1, 2, 3 e 4 uma vez a mais do que os
algarismos 5, 6, 7, 8 e 9. Em particular, h mais 4 do que 5 na lista. O nmero de
vezes que aparece o 9 na lista o nmero de mltiplos de 9 que so menores do
que ou iguais a 20 092 009. Como 20 092 005 o maior mltiplo de 9 que menor
do que 20 092 009, temos que o algarismo 9 aparece 20 092 005 9 = 2 232 445
vezes na lista.
222. Uma brincadeira na sala de aula
(a) O nmero 1 s pode ser obtido por diviso a partir do 2, com 1 = 2 2 e o 2 s
pode ser obtido por diviso a partir do 4, com 2 = 4 2, mas o 4 pode ser obtido
por soma a partir do 1, com 4 = 1 + 3 ou por diviso a partir do 4, com 4 = 8 2.
Logo, 
temos duas maneiras de obter o 1 depois de trs operaes, a partir de 1 e
1 4 2 1
de 8:
.
8 4 2 1

(b) Com uma operao a mais, vemos que o nmero 8 pode ser obtido a partir do
5 por soma, com 8 = 5 + 3, ou do 16 por diviso, com 8 = 16 2. Logo, temos
trs
maneiras de obter o 1 depois de quatro operaes, a partir de 2, 5 e 16:

2 1 4 2 1
5 8 4 2 1 .

16 8 4 2 1
(c)
De maneira anloga, vemos que podemos obter o 1 depois de cinco operaes, com
4 2 1 4 2 1

10 5 8 4 2 1
, bastando comear com os nmeros
13 16 8 4 2 1

32 16 8 4 2 1
4, 10, 13 e 32.

223. Calcule a idade No prximo ano, Laura e sua av estaro dois anos mais velhas do
que no ano passado. Logo, suas idades no ano passado so mltiplos de 8 que, somados
com 2, do mltiplos de 7. Procuremos esses nmeros.
mltiplos de 7 : 7 14 21 28 35 42 49 56 63 . . . 98 . . .
(mltiplos de 7) 2 : 5 12 19 26 33 40 47 54 61 . . . 96 . . .
180

OBMEP 2010

Solues do Nvel 1
Note que 40 e 96 so os nicos mltiplos de 8 menores do que 100 que aparecem na
segunda linha. Como Vov Ana tem menos do que 100 anos, podemos concluir que
ano passado ela tinha 96 anos e Laura 40. Logo, a idade atual de Laura 41 anos.
224. Divises e restos
Soluo 1: O dobro do nmero procurado um mltiplo de 5 acrescido de 1. Como
os mltiplos de 5 terminam em 0 ou 5, o dobro termina em 1 ou 6. Mas o dobro
um nmero par, logo termina em 6. Assim, o nmero termina em 3 ou 8 e, portanto,
dividido por 5, deixa resto 3.
Soluo 2: Sabemos que o nmero inteiro n procurado satisfaz 2n = 5m + 1, para
algum inteiro m. Ento o produto 5m = 2n 1 de 5 por m mpar, o que implica que
m mpar. Assim, m = 2k + 1, para algum inteiro k e, portanto,
2n = 5m + 1 = 5(2k + 1) + 1 = 10k + 6 = 2(5k + 3),
ou seja, n = 5k + 3 deixa resto 3 na diviso por 5.
225. Preenchendo o crculo Sabemos que  = 423 47 = 9. Por outro lado, temos
que
1 448 = 282
} +
|{z}
| {z
mltiplo de 282

nmero de 2 algarismos

Como 282 tem trs algarismos, conclumos que s pode ser o resto da diviso de
1 448 por 282. Efetuando essa diviso, obtemos 1 448 = 282 5 + 38. Logo, = 3 e
= 8. Obtemos, tambm, que = 5. Finalmente, obtemos
423

= 282, ou seja, 141 = 282, portanto, = 2 .


3

A sequncia completa a seguinte.



9

2/3

47 423

+ 38

282 1410 1448

OBMEP 2010

181

Solues do Nvel 2

Solues do Nvel 2
1. Populao A opo correta (a).
Como 1 milho = 1 000 000, temos 30,3 milhes = 30,3 1 000 000 = 30 300 000.
2. Rguas em 15 minutos A opo correta (e).
Se a mquina produz oito rguas em um minuto, em 15 minutos ela produzir 8 15 =
120 rguas.
3. Alturas iguais A opo correta (e).
Usaremos a notao a < b, que significa que a menor do que b ou, equivalentemente,
que b maior do que a. Assim, a < b < c significa que a menor do que b e b menor
do que c. Para simplificar, vamos denotar a altura de cada um dos irmos pela letra
inicial de seu nome.
Do enunciado temos:
(i) L maior do que A (L > A ou, equivalentemente, A < L);
(ii) M menor do que L (M < L);
(iii) A maior do que J ( A > J ou, equivalentemente, J < A);
(iv) J menor do que M (J < M).
De (i) e (iii) segue que J < A < L. Portanto, os irmos de mesma altura no esto entre
Jlio, Antnio e Luza. De (ii) e (iv) segue que J < M < L. Portanto, os irmos de
mesma altura no esto entre Jlio, Maria e Luza. Logo, a nica opo que Antnio
e Maria tenham a mesma altura.
4. Unidade A opo correta (c).
O produto dado tem um de seus fatores igual a 5, portanto, um mltiplo de 5, que
sempre tem o algarismo da unidade igual a 0 ou 5. Alm disso, como todos os fatores
so nmeros mpares, o produto um nmero mpar. Assim, seu algarismo da unidade
5.
5. Em que fio? A opo correta (d).
Observe que a aranha utiliza oito fios de apoio, numerados a partir do fio A, iniciando
em 0. Logo,
sobre o fio A aparecem os mltiplos de 8;

sobre o fio B aparecem os (mltiplos de 8)+1;

sobre o fio C aparecem os (mltiplos de 8)+2;

sobre o fio D aparecem os (mltiplos de 8)+3;


sobre o fio E aparecem os (mltiplos de 8)+4;
sobre o fio F aparecem os (mltiplos de 8)+5;

sobre o fio G aparecem os (mltiplos de 8)+6;


182

OBMEP 2010

Solues do Nvel 2
sobre o fio H aparecem os (mltiplos de 8)+7.
Na diviso de 118 por 8 encontramos resto 6, o que significa que 118 dado por
(mltiplos de 8) + 6. Assim, 118 est sobre o fio G.
6. Pontos ganhos A opo correta (c).
Segundo as regras da Copa do Mundo, uma vitria vale trs pontos e um empate vale
s um ponto. Como a seleo do Senegal tem uma vitria e dois empates, ela obteve
1 3 + 2 1 = 5 pontos.
7. Gols sofridos A opo correta (d).
Numa tabela de jogos, o nmero total de gols marcados sempre igual ao nmero total
de gols sofridos. Denotando por x o nmero de gols que sofreu a seleo do Uruguai,
vemos que 5 + 5 + 4 + 0 = 2 + 4 + x + 3, portanto, 14 = 9 + x, e temos que x = 5, ou
seja, a seleo do Uruguai sofreu 5 gols.
8. Qual o ngulo? A opo correta (c).
Nesta questo usaremos um importante teorema da Geometria Plana, como segue.
Teorema: A soma dos ngulos internos de um tringulo
B
sempre 180 .
b+B
b+C
b = 180 e, como B
b = 50 ,
50
Pelo teorema, temos A

b
b
b
b
segue que A + 50 + C = 180 , ou seja, A + C = 130 .
b e C,
b
Como AD e CD so as bissetrizes dos ngulos A
respectivamente, o teorema aplicado ao tringulo ADC
d a relao

1b 1b
b = 180 .
A + C + ADC
2
2


A
2

C
2

b + 1C
b= 1 A
b+C
b = 1 130 = 65 , portanto, da igualdade acima decorre que
Mas 12 A
2
2
2
b = 180 65 = 115.
ADC

9. Basquete A opo correta (a).

Analisando o grfico, verificamos que os jogadores marcaram as seguintes quantidades


de pontos: Daniel 7, Ramon 8, Ian 2, Bernardo 11, Tiago 6, Pedro 12,
Ed 1 e Andr 7. O total 54 pontos.
10. Telefone A opo correta (a).
Vejamos a despesa em janeiro. Como 10 horas so gratuitas e Geni utilizou o telefone
por 15 horas e 17 minutos, ela deve pagar a tarifa fixa mensal de 18 reais mais o
custo de apenas 5 horas e 17 minutos. Como o preo dado em minutos, passamos
o tempo a pagar para minutos. Sabemos que 1 hora = 60 minutos, portanto, 5 horas
= 5 60 = 300 minutos. Logo, 5h17min = 300 + 17 = 317. Assim, a conta telefnica
de Geni em janeiro foi de 18 + 317 0,03 = 18 + 9,51 = 27,51 reais.
OBMEP 2010

183

Solues do Nvel 2
Em fevereiro, Geni usou seu telefone por menos do que 10 horas, portanto nesse ms
ela s precisa pagar a tarifa fixa mensal de 18 reais. Logo, a despesa de Geni com
telefone nesses dois meses foi de 27,51+18 = 45,51 reais.
11. rea A opo correta (e).
Soluo 1: A rea de um quadrado de lado l l2 e a rea da regio cinza a diferena
entre as reas dos quadrados maior e menor. O lado do quadrado maior a+b, portanto
sua rea (a + b)2 = a2 + 2ab + b2 . J o lado do quadrado menor a, portanto sua
rea a2 . Assim, a rea da regio cinza (a + b)2 a2 = a2 + 2ab + b2 a2 = 2ab + b2 .
Soluo 2: A rea de um retngulo o produto da largura pelo comprimento. Pelos
dados do problema, a largura da regio cinza (a + b) a = b.

Dividindo a regio cinza em dois retngulos, um


de largura b e comprimento a e o outro de largura
b e comprimento a + b (ver figura), vemos que a
rea da regio cinza a soma das reas desses dois
retngulos, ou seja,

rea

rea

a b + b (a + b) = ab + ab + b2
= 2ab + b2 .

Portanto, a rea da regio cinza 2ab + b2 .


Soluo 3: A regio cinza formada por dois retngulos de dimenses a b e um
quadrado de lado b. Logo, sua rea 2ab + b2 .
12. Comprando sorvete A opo correta (d).
Se comprar no supermercado A, Joana gastar 2 24 = 48 reais. Se comprar no
supermercado B, ela gastar 3 14 = 42 reais. Portanto, no supermercado B ela
economizar 6 reais em relao ao A.
13. Cartolina e barbante A opo correta (e).
Observando a frente da cartolina, verificamos que o barbante entra e sai pelos furos
da primeira linha. A opo (e) no possvel, pois no verso esses dois furos aparecem
como consecutivos ao percorrer o barbante, o que impede o barbante de continuar pelos
demais furos.
14. Amigos e fraes A opo correta (b).
Como cada amigo deu a Daniel a mesma quantia, digamos que Daniel tenha recebido
x reais de cada um de seus trs amigos. Inicialmente, ento, Adriano tinha 5x reais,
Bruno tinha 4x reais e Csar tinha 3x reais. Segue que o total de dinheiro inicial dos
trs amigos era de 5x + 4x + 3x = 12x reais. Como cada um de seus trs amigos
lhe deu x reais, Daniel tem agora 3x reais, o que representa a quarta parte do total
de 12x. Logo, ele agora possui 1/4 da quantia que seus trs amigos juntos possuam
inicialmente.
184

OBMEP 2010

Solues do Nvel 2
15. Escolhendo sorvetes A opo correta (d).
Vamos denotar cada sabor de sorvete pela sua letra inicial, ou seja, a aa,
b baunilha, c caj. Para enumerar todas as possibilidades de compra do sorvete
com quatro bolas, devemos considerar os seguintes casos:
quatro bolas do mesmo sabor (1a coluna ao
lado);
trs bolas do mesmo sabor e uma de sabor diferente (2a coluna ao lado);

aaaa
bbbb
cccc

duas bolas de um mesmo sabor e duas de outro


sabor (3a coluna ao lado);
duas bolas de um mesmo sabor e as outras duas
dos outros dois sabores (4a coluna ao lado).

aaab
aaac

aabb
aacc
bbcc

aabc
bbac
ccab

bbba
bbbc
ccca
cccb

Assim, obtemos 15 modos de fazer essa compra de sorvete.


16. Peas de um quadrado A opo correta (b).
Para que seja possvel montar o quadrado, o nmero total de quadradinhos deve ser
um quadrado perfeito. Um nmero inteiro um quadrado perfeito se ele igual ao
quadrado de algum nmero inteiro. Por exemplo, 1, 4, 9, 16 e 25 so quadrados
perfeitos, pois 1 = 12 , 4 = 22 , 9 = 32 , 16 = 43 e 25 = 52 . Observe que esses cinco
inteiros so os nicos quadrados perfeitos menores do que 30.
Contando o total de quadradinhos apresentados nas cinco opes de resposta obtemos
4 + 5 + 6 + 7 + 8 = 30. Portanto, devemos eliminar uma pea com 5 quadradinhos, para
restar 25, um quadrado perfeito, ou eliminar uma pea com 14 quadradinhos, para
restar 16, outro quadrado perfeito, ou eliminar uma com 21, para restar 9, ou eliminar
uma com 26, para restar 4, ou eliminar uma com 29 quadradinhos, para restar um
nico. Ocorre que no h peas com 14, 21, 26 ou 29 quadradinhos, restando a nica
opo de eliminar a pea (b), com 5 quadradinhos.
O nico quadrado que Pedro poderia ter montado com quatro
peas no usando a pea (b). Isto no significa que seja possvel montar um quadrado com as quatro peas restantes. Mas,
sabendo que devemos montar um quadrado de lado 5 com as
cinco peas (a), (c), (d) e (e), o problema j fica bem mais fcil.
A figura mostra como isso pode ser feito.

(c )

(d)

(a)

(e)

17. Paradas de nibus A opo correta (b).


Como a distncia entre a terceira e a sexta paradas 3 300 m, a distncia entre duas
paradas consecutivas 3 300 3 = 1 100 m. Portanto, a distncia entre a primeira e a
ltima paradas de 1 100 11 = 12 100 metros, ou seja, 12,1 quilmetros.

OBMEP 2010

185

Solues do Nvel 2
18. Desenho A opo correta (e).

(a)

(b)

(c)

(d)

Nas ilustraes (a), (b), (c) e (d) dadas, vemos que, iniciando o desenho no ponto P
e seguindo as setas de acordo com a ordem numrica, possvel completar cada um
desses desenhos sem tirar o lpis do papel.
Observe que, excetuando-se o vrtice de incio do traado e o vrtice de finalizao, os
demais vrtices do desenho devem possuir obrigatoriamente um nmero par de linhas
chegando at eles, pois a cada vez que se chega a um desses vrtices por uma linha,
deixa-se esse mesmo vrtice por outra linha.
Assim, impossvel fazer o traado da opo (e) do enunciado, que no pode ser
construdo sem tirar o lpis do papel, j que seus quatro vrtices externos possuem
trs linhas chegando a cada um deles.
19. Qual o cubo? A opo correta (e).
Ao cortar um canto do cubo, eliminamos um de seus vrtices. Como cada vrtice se
liga a trs arestas do cubo, uma representao do cubo cortado deve mostrar trs cortes
ao redor de um mesmo vrtice.

20. Quadrado mgico A soma dos nmeros de uma diagonal 4 + 0 + (4) = 0,


portanto, o valor da soma dos nmeros de cada linha, de cada coluna e da outra
diagonal tambm deve ser 0. Assim, obtemos de imediato os nmeros que faltam nas
casas cinza no primeiro tabuleiro, a saber, 16, 8 e 12, porque (12) + 16 + (4) = 0 na
primeira linha, (12) + 8 + 4 = 0 na primeira coluna e (12) + 0 + 12 = 0 na diagonal.

Agora, o nmero que falta na segunda linha do segundo tabuleiro 8, porque


8 + 0 + (8) = 0. Para a terceira linha, obtemos 16, pois 4 + (16) 12 = 0.
21. Torneio Denotemos as sete equipes pela sua letra inicial.
186

OBMEP 2010

Solues do Nvel 2
(a) Na primeira rodada do Grupo 1 foram disputadas trs partidas, AB, BC e
CA.
(b) Na primeira rodada do Grupo 2 foram disputadas seis partidas, DE, DF,
DG, EF, EG e FG.
(c) Na segunda rodada, cada equipe do Grupo 1 jogou quatro partidas, uma com
cada uma das equipes do Grupo 2. Como o Grupo 1 tem trs equipes, na segunda
rodada foram disputadas 3 4 = 12 partidas.
22. Truque numrico
(a) Vamos fazer o experimento com os nmeros 0, 5 e 4.
0
5
-4

6
6
6

-21

-21

30

-21

-24

-21

-45

-7

3
-15

-(02)=0
-(52)= -10

-(-42)= +8

-7

-7
-7

O resultado final sempre 7.

(b) razovel, ento, conjecturar que, para qualquer nmero inicial escolhido, o resultado final desse procedimento ser sempre 7. Seja x o nmero inicial. Temos,
ento, as operaes seguintes.
x

6x

-21

6x-21

6x-21
8
3

-2x

2x-7-2x = -7

Portanto, o resultado dessa mgica sempre ser igual a 7, qualquer que seja
o nmero inicialmente escolhido.
23. Jogando sinuca A bola muda a direo de sua trajetria cada vez que bate numa das beiradas da mesa.
Como a trajetria faz sempre um ngulo de 45 com
a beirada, a trajetria dessa bola, tacada a partir de
um canto, seguir sempre as diagonais dos quadrados
que ela cruzar. Traando essa trajetria, conclumos
que (b) a bola bater cinco vezes nas beiradas da mesa
antes de (a) cair na caapa superior esquerda.
Contando quadrados atravessados, vemos que (c) ela atravessar 23 quadrados pela
diagonal.
24. Tringulo issceles Por definio, um tringulo issceles se tiver dois lados
iguais. O terceiro lado chamado base do tringulo issceles, e os ngulos formados
entre a base e os dois lados iguais so os ngulos da base.
OBMEP 2010

187

Solues do Nvel 2
A

C
C

A figura mostra um tringulo issceles ABC, cujos lados iguais so AB e AC e a


b e ACB
b da base por B
b e C,
b respectivamente.
base BC. Denotamos os ngulos ABC
Demonstra-se que num tringulo issceles os ngulos da base so sempre iguais. No
b = C.
b
tringulo da figura temos, portanto, B
b+ B
b+C
b = 180 , j que a soma dos
Passando resoluo desta questo, observe que A

b = 20o e o
ngulos internos de qualquer tringulo 180 . Pelos dados do problema, A
b = C.
b Logo, 180 = 20 + B
b+C
b = 20 + 2B
b e,
tringulo issceles, de modo que B

b=C
b = 80 .
portanto, B

O tringulo CBD tambm issceles, pois dado que CB = DB. Como a base desse
b = C,
b portanto, C DB
b = 80 . Considetringulo CD, seus ngulos de base so C DB
rando a soma dos ngulos internos desse tringulo CBD, obtemos
b + C DB
b +C
b = 180. Substituindo os valores j obtidos, vemos que C BD
b + 80 +
C BD
b = 20 . Assim, D BE
b =B
b 20 = 80 20 = 60 .
80 = 180 , de modo que C BD

O tringulo DBE tambm issceles, porque tambm DB = BE. A base desse


b = D EB.
b Como
tringulo DE e os ngulos iguais da base BE so E DB
b + D EB
b + D BE
b = 2 B DE
b + 60 ,
180 = B DE

b = 60 .
conclumos que B DE

25. Pesando moedas Sejam A, B, C e D as quatro moedas aparentemente iguais.


Comparamos as moedas A e B na balana, colocando uma em cada prato. Dois
casos podem ocorrer: a balana fica em equilbrio ou a balana no fica em equilbrio.
Vamos analisar separadamente cada caso. Observe que, em ambos casos, s utilizamos
a balana duas vezes.
1o Caso: A balana fica equilibrada. Podemos concluir que A e B tm o mesmo peso,
portanto, so verdadeiras. Vamos ento comparar A com C. Para isso, mantemos A
na balana e colocamos C no lugar de B. Se houver equilbrio novamente, porque A
e C tm o mesmo peso e so, portanto, verdadeiras. Assim, A, B e C so verdadeiras
e a nica opo que D seja a moeda falsa. Se no houver equilbrio, C a moeda
falsa.
2o Caso: A balana no fica equilibrada. Logo uma das duas moedas, A ou B a
falsa. Substitumos A por C na balana. Se houver equilbrio, A a moeda falsa. Se
no houver equilbrio, a moeda falsa B.

188

OBMEP 2010

Solues do Nvel 2
26. Nmeros binomiais A opo correta (e).
Preenchendo o tabuleiro de acordo com as regras do problema, segue que
60 = ( + 17) + (2 +13) = 3 +30, donde = 10.
5

+6

11

+17

+7
2+13

60

27. Costuras da bola A opo correta (c).


Se somarmos os nmeros de lados de todos os polgonos (20 hexgonos e 12 pentgonos)
que compem a superfcie da bola, obteremos um valor que duas vezes o nmero de
costuras, pois cada costura lado comum de exatamente dois polgonos. Assim, temos
que 2 nmero de costuras = 12 5 + 20 6 = 180, donde o nmero de costuras 90.
28. Razo de reas A opo correta (a).
A grade um quadrado de lado igual a 5 cm, logo sua rea igual a 25 cm2 . A parte
sombreada da grade formada por quatro tringulos, sendo que dois deles tm base
1 cm e altura 2 cm e os outros dois tm base 1 cm e altura 3 cm. Logo a rea sombreada
igual a 2 12 (1 2) + 2 12 (1 3) = 5 cm2 e a rea no sombreada igual a 25 5 =
20 cm2 . Assim, a razo pedida 5/20 = 1/4.
29. S sorvete A opo correta (c).
Vamos primeiro analisar a informao contida na diagonal da tabela indicada pelos
nmeros dentro dos quadradinhos.

Esses nmeros indicam quantas foram as crianas que tomaram sorvetes com o mesmo
sabor pela manh e pela tarde: um tomou sorvetes de abacaxi, dois de banana, nenhum
de chocolate e um de doce de leite. Todos os outros estudantes comeram sorvetes de
sabores diferentes pela manh e tarde, num total de 64 (1 + 2 + 0 + 1) = 60.

OBMEP 2010

189

Solues do Nvel 2
30. Brincando com tabuleiro A opo correta (b).
Notamos primeiro que se uma casa tem o algarismo
0, ento nenhuma das casas vizinhas pode estar
pintada. Logo, as casas marcadas com um na
figura direita no foram pintadas.
Consideremos, agora, a casa do canto superior direito, na qual aparece o nmero 1. Ela tem trs
vizinhas, e j sabemos que duas delas no foram
pintadas.
Logo, a vizinha que sobra (a casa imediatamente
abaixo) foi pintada. Podemos aplicar o mesmo argumento s casas do canto inferior esquerdo e do
canto inferior direito.
Olhamos agora para o 2 na ltima linha. Como
esta casa j tem duas vizinhas pintadas, todas suas
outras vizinhas no foram pintadas.
Argumento idntico se aplica casa da segunda
linha e terceira coluna, pois nela aparece um 1 e
j temos uma de suas vizinhas pintadas. Logo, as
suas outras trs vizinhas no foram pintadas.
Finalmente, usamos o 3 que aparece na casa da
terceira linha e terceira coluna. Esta casa j tem
duas vizinhas pintadas, logo deve haver mais uma
de suas vizinhas pintada.
Esta vizinha s pode ser a casa em branco na figura acima, e podemos completar a
tabela. Conclumos que o nmero de casas pintadas 4.
31. Cartes numerados A opo correta (b).
A formao de um nmero de 6 algarismos ilustrada a seguir.
centena
de milhar

dezena
de milhar

unidade
de milhar

centena

dezena

unidade

Para se obter o menor nmero possvel, os menores algarismos devem estar o mais
esquerda possvel (na casa do milhar) e para se obter o maior nmero possvel os
maiores algarismos devem tambm estar o mais esquerda possvel (na casa do milhar).
Jorge joga primeiro: Para obter o menor nmero possvel, ele coloca o menor algarismo
que ele possui, que o 2, na casa da centena de milhar. Se ele no fizesse isso, Larissa
colocaria seu 5 nesta casa na prxima jogada e obteria, assim, um nmero maior.
2

dezena
de milhar

unidade
de milhar

centena

dezena

unidade

Agora a vez de Larissa: Para obter o maior nmero possvel, ela coloca o maior algarismo que ela possui, que o 5, na casa das dezenas de milhar, pois a casa das centenas
de milhar j est ocupada.
190

OBMEP 2010

Solues do Nvel 2
2

unidade
de milhar

centena

dezena

unidade

Agora, Jorge tem os algarismos 4 e 6, e Larissa 1 e 3. Logo, os algarismos de Larissa


so menores do que os de Jorge, o que determina a estratgia de Jorge: ele deve tentar
colocar seus algarismos o mais direita possvel, com o 6 direita do 4. Por sua
vez, Larissa deve tentar colocar seus algarismos o mais esquerda possvel, com o 3
esquerda do 1.
Jorge joga: Ele coloca o algarismo 6 na casa das unidades.
2

unidade
de milhar

centena

dezena

Larissa joga: Ela coloca seu 1 na casa das dezenas.


2

unidade
de milhar

centena

Agora, Jorge tem apenas o algarismo 4 e Larissa o 3. Ele ento coloca o 4 na casa das
centenas e Larissa coloca o 3 na casa das unidades de milhar, acabando o jogo.
2

Assim, o nmero final, obtido se os dois jogadores forem espertos, 253 416.
32. Faltam balas A opo correta (a).
Dividindo 237 por 37, obtemos 237 = 7 31 + 20. Logo, 237 no divisvel por 31.
Isso quer dizer que a professora realmente vai ter que comprar mais balas para que
todos os alunos recebam o mesmo nmero de balas. Devemos adicionar expresso
7 31 + 20 o menor inteiro positivo x tal que 7 31 + 20 + x seja mltiplo de 31. Como
20 + 11 = 31, basta que a professora compre 11 balas adicionais.
33. Artesos de braceletes A opo correta (d).
O arteso produz 6 braceletes a cada 20 minutos. Como 1 hora = 60 minutos =
3 20 minutos, o arteso produz 6 3 = 18 braceletes em uma hora. Como ele
trabalhou 12 horas 8 horas = 4 horas, o nmero de braceletes feitos pelo arteso
18 4 = 72. O auxiliar produz 8 braceletes a cada meia hora, portanto em 1 hora ele
produz 16 braceletes. Para produzir 72 braceletes ele precisar de 72/16 = 4,5 horas
= 4 horas e 30 minutos. Como ele inicia seu trabalho s 9 horas, ele terminar seu
trabalho s 9 + 4,5 = 13h30min.
34. Girando um pentgono A opo correta (b).
1
360 = 72 .
5
Soluo 1: Dividindo 252 por 72, obtemos 252 = 3 72 + 36. Como 36 = 72 2,
conclumos que uma rotao do pentgono de 252 em torno do seu centro corresponde
a uma rotao de um ngulo igual a trs vezes e meia o ngulo central.

O pentgono tem 5 lados. Logo, seu ngulo central mede

OBMEP 2010

191

Solues do Nvel 2
Soluo 2: Como 252 = 72 + 180 , podemos pensar na rotao de 252 como uma
rotao de 72 seguida de outra de 180, conforme ilustrado na figura dada, em que O
o centro do polgono.
A
A

O
rotao de 72

rotao de 180

35. rea em funo da diagonal A opo correta (c).


A rea A de um retngulo o produto do comprimento
pela largura. Sejam a e b o comprimento e a largura do
retngulo. Assim, A = ab. O permetro desse retngulo
dado por 2a + 2b. Como o permetro 100, temos que
2a + 2b = 100, portanto, a + b = 50. Elevando ao
quadrado ambos os lados dessa ltima igualdade, obte-

...................................................................................................................................................................................................
. .
..
....... ....
...
.......
.
.......
....
...
.......
.
.
.
..
.
.
...
.
.......
...
...
.......
.
.
.
..
.
.
.
....
.....
.
.
...
.
.
.
..
.
.....
...
.
.
.
.
...
.
.....
..
.
.
.
.
.
...
.
...
....
.
.
.
.
.
...
.
.
....
.
.
.
.
.
.
...
....
.....
.
.
.
..
.
.
....
.....
.
.
.
.
...
.
..
.
....
.
.
..
.
.
.
...
.
....
.
.
.
...
.
.
.
...
...
................
... .............
...................................................................................................................................................................................................

mos a2 + b2 + 2ab = (a + b)2 = 502 = 2 500. Se x denota o comprimento da diagonal, o


Teorema de Pitgoras afirma que x2 = a2 + b2 , portanto, x2 + 2A = x2 + 2ab = 2 500.
1
Conclumos que 2A = 2 500 x2 , ou seja, A = 1 250 x2 a expresso da rea do
2
retngulo em funo da diagonal x.
36. Valor de uma quadrtica A opo correta (d).
Elevando ao quadrado ambos os lados da igualdade x + y =
x2 + 2xy + y 2 = (x + y)2 = 82 = 64. Como xy = 15, conclumos que

8, obtemos

x2 + 6xy + y 2 = (x2 + 2xy + y 2 ) + 4xy = 64 + 4 15 = 124.


37. ngulos em funo de x A opo correta (c).
Completamos a figura marcando os ngulos e , lembrando que ngulos
opostos pelo vrtice so iguais. Lembremos que a soma dos ngulos internos de
um tringulo 180. Olhando para o tria
3x
ngulo mais esquerda, vemos que

5x

b
2x

b
6x

3x + 4x + = 180 .

4x

Segue que = 180 7x. Considerando o tringulo do meio, temos


(180 7x) + 5x + = 180 .

Conclumos que = 2x. Finalmente, do tringulo da direita, temos que


+ 2x + 6x = 180 , ou seja, 2x + 2x + 6x = 180 . Assim, x = 18 .
192

OBMEP 2010

Solues do Nvel 2
38. Operao diferente A opo correta (c).
Pela definio, obtemos

2226
22 + 23 + 24 + 25 + 26
120
=
=
= 8.
46
4+5+6
15

39. Taxi caro A opo correta (c).


Como a bandeirada fixa, temos 10,00 2,50 = 7,50 reais a serem gastos apenas
com os metros rodados. Cada trecho de 100 metros rodado custa R$ 0,10, ento com
R$ 7,50 posso fazer uma corrida de (7,50)/(0,10) = 750/10 = 75 trechos de 100 metros
cada um, ou seja, 75 100 = 7 500 metros. Como 1 quilmetro tem 1 000 metros, segue
que, com R$ 10,00, posso pagar uma corrida de at 7 500 metros, ou 7,5 quilmetros.
40. Mltiplos de 3 ou 4 A opo correta (d).
Para encontrar o nmero de mltiplos de 3 compreendidos entre 1 e 601, basta usar o
algoritmo da diviso e observar que 601 = 200 3 + 1. Isso mostra que 3 1, 3 2,
. . . , 3 200 so os mltiplos de 3 entre 1 e 601, ou seja, temos 200 desses mltiplos.
Do mesmo modo, vemos que existem 150 mltiplos de 4 entre 1 e 601. Nesse total de
200 + 150 = 350, alguns nmeros aparecem contados duas vezes, pois so mltiplos de
3 e de 4 ao mesmo tempo; por exemplo, foram includos 12, 36 e 60 nos 200 mltiplos de
3 e tambm nos 150 mltiplos de 4. Lembre que os mltiplos de 3 e de 4 so, tambm,
mltiplos de 12. O mesmo argumento usado acima mostra que temos 50 mltiplos de
12 entre 1 e 601. Logo, o nmero de mltiplos de 3 ou 4 entre 1 e 601 350 50 = 300.
41. Lados de um paraleleppedo A opo correta (b).
240
Soluo 1: De xyz = 240, segue que xy =
. Substituindo em xy +z = 46, obtemos
z
240
+ z = 46, ou seja, z 2 46z + 240 = 0. As razes dessa equao so nmeros cuja
z
soma 46 e cujo produto 240, e fcil verificar que essas razes so e 6 e 40. Logo,
240
z = 6 ou z = 40. De maneira completamente anloga, a substituio de yz =
em
x
x + yz = 64 nos leva a x = 4 ou x = 60.
240
Agora, de xyz = 240, segue que y =
. Como y um nmero inteiro, ento xz
xz
um divisor de 240. De x = 4 ou x = 60 e z = 6 ou z = 40 segue que as possibilidades
para xz so
4 |{z}
6 = 24, |{z}
4 |{z}
40 = 160, |{z}
60 |{z}
6 = 360, |{z}
60 |{z}
40 = 2 400.
|{z}
x

Vemos que s podemos ter x = 4 e z = 6, pois em qualquer outro caso o produto xz


240
240
=
= 10, donde
no um divisor de 240. Segue que y =
xz
46
x + y + z = 4 + 10 + 6 = 20.
Soluo 2: Somando xy + z = 46 e x + yz = 64, obtemos
(x + z)(y + 1) = (x + z)y + (x + z) = xy + z + x + yz = 46 + 64 = 110
OBMEP 2010

193

Solues do Nvel 2
e vemos que y + 1 um divisor de 110. Logo, temos as possibilidades
y + 1 = 1, 2, 5, 10, 11, 22, 55 e 110,
ou seja, y = 0, 1, 4, 9, 10, 21, 54 e 109. Por outro lado, y um divisor de 240, porque
xyz = 240 e, alm disso, y positivo, que nos deixa com as nicas possibilidades
y = 1, 4 e 10. Examinemos cada caso de y.
Se y = 1, ento 110 = (x + z)(y + 1) = (x + z) 2, portanto, x + z = 55. Como
tambm 46 = xy + z = x + z, esse caso y = 1 no possvel.
Se y = 4, ento 110 = (x + z)(y + 1) = (x + z) 5, portanto,
x + z = 22. Mas 240 = xyz = 4xz, portanto, xz = 60. Podemos verificar
(por exemplo, com uma lista de divisores de 60 ou, ento, resolvendo a equao
w 2 22w + 60 = 0) que no h valores inteiros positivos de x e z que verifiquem
essas duas condies x + z = 22 e xz = 60. Logo, esse caso y = 4 tambm no
possvel.
Se y = 10, ento 110 = (x + z)(y + 1) = (x + z) 10, portanto, x + z = 11. Mas
240 = xyz = 10xz, portanto, xz = 24. Podemos verificar (por exemplo, com uma
lista de divisores de 24 ou, ento, resolvendo a equao w 2 11w + 24 = 0) que
os nicos valores inteiros positivos de x e z que verifiquem essas duas condies
x + z = 11 e xz = 24 so x = 4 e z = 6.
Assim, a nica possibilidade x = 4, y = 10 e z = 6, com o que x + y + z = 20.
42. Pontos da reta A opo correta (b).
Notamos que a e b so nmeros maiores do que 1/2 e menores do que 1. Portanto, a+ b
um nmero maior do que 1 e menor do que 2. Logo, a + b s pode ser representado
por m. Como a < b, segue que a b negativo e, portanto, s pode ser representado
por q. Quanto ao produto ab, notamos primeiro que, como a e b so positivos, seu
produto positivo. Por outro lado, temos b < 1 e a > 0, donde ab < a. Assim, o nico
nmero que pode representar ab p.
43. Velocidades A opo correta (d).
O menor tempo de percurso obtido quando se percorre o maior trecho com a maior
velocidade e o menor trecho com a menor velocidade. J o maior tempo obtido
quando se percorre o maior trecho com a menor velocidade e o menor trecho com a
maior velocidade. Assim, o tempo total gasto pelo piloto nos trs trechos de, no
240 300 400
240 300 400
mnimo,
+
+
= 15 horas e de, no mximo,
+
+
= 17 horas.
40
75
80
800
75
400
44. Comprimento de diagonal A opo correta (b).
Primeiro notamos que os tringulos AP S e CQR so congruentes, pois tm os trs ngulos iguais (um deles sendo reto)
e tambm um de seus lados (P S = QR). Do mesmo modo,
os tringulos BP Q e DRS tambm so congruentes. Sejam AP = x e BP = y. Ento a rea do tringulo AP S
21 x2 e a dotringulo BP Q 21 y 2 e a rea cortada foi de
2 12 x2 + 12 y 2 = x2 + y 2 .
194

OBMEP 2010

P y B
y
Q

S
D

Solues do Nvel 2
Assim, estabelecemos que x2 + y 2 = 200. Agora notamos que P R a hipotenusa do
tringulo retngulo P SR. Para calcular P R, basta saber o comprimento dos catetos
P S e RS. Mas P S a hipotenusa do tringulo retngulo AP S e do Teorema de
Pitgoras segue que (P S)2 = (AS)2 + (AP )2 = x2 + x2 = 2x2 . Do mesmo modo,
obtemos (RS)2 = 2y 2 . Logo,
(P R)2 = (P S)2 + (RS)2 = 2x2 + 2y 2 = 2(x2 + y 2) = 2 200 = 400,

ou seja, P R = 400 = 20 m.
45. Diviso de nmeros grandes claro que com nmeros to grandes, o objetivo
da questo no efetuar a diviso. Em vez disso, decompomos o nmero em partes
convenientes.
123 456 123 456 = 123 456 000 000 + 123 456 = 123 456 1 000 000 + 123 456
= 123 456 (1 000 000 + 1) = 123 456 1 000 001
Logo, 123 456 123 456 1 000 001 = 123 456.
46. Refrigerante no cinema A opo correta (c).
A economia teria sido equivalente a seis refrigerantes, permitindo a Joozinho mais um
cinema e mais um refrigerante. Logo, o ingresso do cinema cinco vezes o valor do
refrigerante.
47. Diviso de potncias A opo correta (c).
Soluo 1:

5050
(2 52 )50
250 5100
=
=
= 250 550 = (22 52 )25 = 10025 .
2525
(52 )25
550

Soluo 2:

5050
(2 25)50
250 2550
=
=
= 225 225 2525 = 10025 .
2525
2525
2525

48. Palitos de dois tamanhos A opo correta (a).


A quantidade de palitos mnima quando o nmero de palitos de 7 cm utilizado
o maior possvel. O segmento mede 200 cm. Dividindo 200 por 7, obtemos 200 =
28 7 + 4. Portanto, se tentssemos utilizar apenas palitos de 7 cm, deveramos utilizar 29 palitos, mas ainda sobrariam 3 cm. Para que no sobrem esses 3 cm, basta
substituir 3 dos 29 palitos de 7 cm por palitos de 6 cm. Temos 26 7 + 3 6 = 200.
Logo, o nmero mnimo de palitos 26 + 3 = 29. Devemos utilizar 26 palitos de
7 cm e 3 palitos de 6 cm.
Observao: Observe que a soluo equivale a encontrar nmeros inteiros x e y tais
que 200 =
7y
+ |{z}
6x
e y seja o maior possvel, onde y denota o nmero de
|{z}
mltiplo de 7

mltiplo de 6

palitos de 7 cm e x o de palitos de 6 cm.

49. Maior raiz A opo correta (d).


Soluo 1: Usando a fatorao a2 b2 = (a b)(a + b), temos
0 = (x 37)2 169 = (x 37)2 132 = (x 37 13)(x 37 + 13) = (x 50)(x 24).
OBMEP 2010

195

Solues do Nvel 2
Logo, as razes so 24 e 50.
Soluo 2: Extraindo a raiz quadrada em ambos os lados de (x 37)2 = 132 , temos
x 37 = 13, ou x 37 = 13. Assim, x = 50 ou x = 24.
50. Mquina com visor A opo correta (d).
O diagrama a seguir mostra os resultados de dois algarismos que podem ser obtidos a
partir do nmero 5, apertando cada uma das duas teclas.

11
B

B
A

68
65

95

59

86

95

32

5
B

47

23

A
A

29

14
B

41
A

83

51. Quadrado mgico parcial A opo correta (e).


De acordo com a regra de quadrado mgico, temos que a soma dos
nmeros da diagonal que contm y igual soma dos nmeros da
coluna que contm y, ou seja, 26 + 14 + y = y + x + 13. Segue que
26 + 14 = x + 13, donde x = 13.

1
26

14

y
x
13

52. rea do retngulo A opo correta (e).


Soluo 1: Observemos, primeiro, que a razo entre as reas de dois retngulos que
tm a mesma base igual razo entre suas alturas. De fato, na figura esquerda,
esto representados dois retngulos que tm a mesma base b e alturas h1 e h2 .

S1

h1

16

h1

S2

h2

12

27

h2

Suas reas S1 e S2 so dadas por S1 = b h1 e S2 = b h2 , portanto,


S1
bh1
h1
=
=
.
S2
bh2
h2
196

OBMEP 2010

Solues do Nvel 2
Aplicando essa observao aos dois pares de retngulos dados (ver figura acima,
direita) e denotando por S a rea do quarto retngulo, temos
S
16
h1
4
=
=
= ,
27
h2
12
3
de modo que S = 31 (27 4) = 36. Assim, a rea do retngulo ABCD
12 + 16 + 27 + 36 = 91.
Soluo 2: Sejam x e y lados dos retngulos de
reas 12 e 27, respectivamente, como indicado na
figura. Logo os outros lados desses retngulos so
12/x (retngulo de rea 12), 16/x (retngulo de
rea 16) e 27/y (retngulo de rea 27), como indicado na figura.

16
8
x

16

12
8
x

12

27

h1
h2

16
12
28
+
=
.
Assim, o comprimento do retngulo ABCD x + y e sua largura
x
x
x
12
27
y
27
9
Claramente,
=
, de modo que =
= . A rea de um retngulo o produto
x
y
x
12
4
28
28y
do comprimento pela largura. Logo, a rea de ABCD A = (x+ y)
= 28 +
=
x
x
y
9
28 + 28 . Assim, A = 28 + 28 = 28 + 7 9 = 91.
x
4
53. Lado do quadrado
Soluo 1: Sejam x e y o maior e o menor catetos, respectivamente, do tringulo
retngulo. Como o lado do quadrado ABCD mede 3 cm, temos x y = 3. Por outro
lado, como o lado de EF GH mede 9 cm, temos x + y = 9. Resolvendo o sistema,
encontramos x = 6 e y = 3.
IJKL, que a hipotenusa do
Logo, o lado
do quadrado

tringulo retngulo, mede 62 + 32 = 45 = 3 5 cm, pelo Teorema de Pitgoras.


H

D
C

x x-y

x+y

B
L

P
E

Soluo 2: Os quadrados IJKL e MNOP tm como lados as hipotenusas dos tringulos retngulos dados, logo tm a mesma rea.
H

Superpondo-se as duas figuras e fazendo esses dois quadrados coincidirem, encontramos oito tringulos e conclumos que
8 a rea do tringulo igual rea de EF GH menos a rea
de ABCD, ou seja, igual a 92 32 = 72. Logo, a rea de
cada tringulo 9 cm2 . Da figura, temos que a rea de IJKL
igual a 4 a rea do tringulo mais a rea de ABCD, ou
seja, igual a 4 9 + 9 = 45.
OBMEP 2010

M
I

G
D

C
A
P L
E

K
O

197

Solues do Nvel 2
Logo, o lado do quadrado IJKL mede

45 = 3 5 cm.

54. Maior nmero A opo correta (d).


Lembre que, se num produto, um dos fatores zero, ento o produto tambm zero.
Temos 2 0 2 006 = 0, 2 0 + 6 = 0 + 6 = 6, 2 + 0 2 006 = 2 + 0 = 2,
2 (0 + 6) = 2 6 = 12 e 2 006 0 + 0 6 = 0 + 0 = 0. Logo, o maior nmero
2 (0 + 6) = 12.
55. Operao A opo correta (e).

Temos que descobrir qual a regra dessa operao. Note que


2 4 = 10 = 2 4 + 2, 3 8 = 27 = 3 8 + 3, 4 27 = 112 = 4 27 + 4

e 5 1 = 10 = 5 1 + 5.

Uma hiptese plausvel que a regra


a b = a b + a. Segundo essa regra, temos

que

define

operao

seja

4 (8 7) = 4 (8 7 + 8) = 4 64 = 4 64 + 4 = 260.
56. Terceiro lado A opo correta (e).
Lembre que, num tringulo, a soma de dois lados quaisquer deve ser maior que o
terceiro lado. Como 1 + 5 no maior do que 7, o terceiro lado no pode medir 1 cm.
57. Asterisco A opo correta (e).
3 2

3 2

25
25
1
=
=
=

=
.
6
24 8 3
24
8 3
24 24
24

Logo,

25
1
4
= =
, donde 25 = 4, ou seja, = 29.
24
6
24

58. Expresses algbricas Note que a figura um retngulo formado por um quadrado
de lado a e um retngulo de lados 1,5 e a. Logo, a2 a rea do quadrado e 1,5 a a
rea do retngulo. Assim, a2 + 1,5 a representa a soma dessas duas reas, ou seja, a
rea total da figura. J 4 a + 3 = 3 a + 1,5 + a + 1,5 o permetro da figura.
59. Faixa decorativa A opo correta (d).
Soluo 1: O comprimento da hipotenusa de cada um dos cinco
tringulos retngulos issceles da faixa mede 30 5 = 6 cm. O
quadrado formado por quatro desses tringulos tem lado igual a
6 cm, portanto, sua rea 36 cm2 . Logo, cada um dos tringulos
tem 36 4 = 9 cm2 de rea. Portanto, a rea da parte sombreada
mede 9 5 = 45 cm2 .

6 cm

Soluo 2: O comprimento da hipotenusa de cada um dos cinco tringulos retngulos


issceles da faixa mede 30 5 = 6 cm. Denotando os catetos desses tringulos por x,
198

OBMEP 2010

Solues do Nvel 2
o Teorema de Pitgoras fornece 36 = x2 + x2 = 2x2 , ou seja, x2 = 18, de modo que
a rea de cada um dos cinco tringulos da faixa mede 9 cm2 . Assim, a rea da parte
sombreada mede 5 9 = 45 cm2 .
60. Bicicleta e chocolate A opo correta (c).


2 barras d 3 h
1 barra d 1,5 h = 1 h 30 min
Como
, segue que
.
12 bombons d 2 h
3 bombons d 0,5 h = 30 min
Assim, Tio me emprestar a bicicleta por 1 h 30 min + 30 min = 2 horas.

61. Retas paralelas?


b = 180 (42 + 48 ) = 90 . No
Soluo 1: No tringulo BCE, temos B EC

tringulo AF D, temos AFbD = 180 (28 + 62 ) = 90 . Logo, as retas EC e F D


so perpendiculares reta AB, de modo que so paralelas.

b = 180 (48 + 62 ) = 70 . Portanto,


Soluo 2: No tringulo ABC, temos B CA
b = 70 42 = 28 = F DA.
b Logo, as retas EC e F D so paralelas, pois cortam
E CA
a reta AD segundo o mesmo ngulo.
62. Menor nmero A opo correta (b).
5
5
5
Como x > 5, temos 0 < x 1 < x < x + 1. Portanto,
< <
. Tambm
x+1
x
x1
x
x+1
5
temos
< 1 <
<
, pois 5 < x < x + 1. Assim, dentre os nmeros 5/x,
x
5
5
5/(x + 1), 5/(x 1), x/5 e (x + 1)/5, o menor 5/(x + 1).
63. rea de quadrado A opo correta (a).
Denotemos por C e L o comprimento e a largura, respectivamente, de cada um dos
quatro retngulos. O permetro de cada retngulo dado por 2(C + L). Como esse
permetro mede 40 cm, obtemos C + L = 20 cm. Observe, na figura dada, que o lado
do quadrado ST UV dado por C + L. Assim, sua rea de (C + L)2 = 202 = 400 cm2 .
64. Operando fraes
1
1 1 1
1 1 1
1
= ,
= ,
=
,
2
2 2 3
6 3 4
12
1
1
1
1
1
(b)
+
+
+
+
= 1
2
6
12
20
30
|{z}
|{z}
|{z}
|{z}
|{z}
(a) 1

1 12

1 1

2 3

1
1
3 4

1 1

4 5

1 1
1
1 1
1
=
,
=
.
4 5
20 5 6
30
1
1
1
1
1
1
1
1
1
+ + + + ;
2
2
3
3
4
4
5
5
6

1 1

5 6

cancelando as parcelas iguais de sinais opostos, resulta que


1 1
1
1
1
1
5
+ +
+
+
=1 = .
2 6 12 20 30
6
6

1
1 1
1
1
1
1
(c) Para calcular a soma + +
+
+
+
+ +
, comeamos
2 6 12 20 30 42
999 000
observando que todos os denominadores so produtos de nmeros consecutivos,
OBMEP 2010

199

Solues do Nvel 2
iniciando em 1; usando a decomposio de cada parcela dada no item (a), obtivemos, no item (b), que
1
1
1
1
1
1
+
+
+
+
=1 .
12 23 34 45 56
6

Mais geralmente, podemos provar que

1
1
1
1
1
1
1
+
+
+
+
++
=1
.
12 23 34 45 56
n (n + 1)
n+1

Assim,

1
1
1 1
1
1
1
999
+ +
+
+
++
=1
=
= 0,999.
2 6 12 20 30
999 000
1 000
1 000
65. ngulos e permetro O tringulo BCD issceles, porque tem dois lados iguais,
b = B CD.
b
BD = BC, logo B DC
A
120 m
b = B CD,
b portanto os trs ngulos desse
B
Mas D BC
115 m
26

C
60
80
tringulo so iguais, cada um valendo 180 3 =
120 m
60
60 , e o tringulo BCD equiltero. Assim,
26
60 115 m
30
BD = BC = CD = 115 m.
70
E

226 m

O tringulo ABE tambm issceles, porque tem dois ngulos iguais, logo os lados
AE e AB so iguais, portanto AB = AE = 120 m. Assim, o permetro da figura mede
120 2 + 115 2 + 226 = 696 m.
66. Desigualdade racional A opo correta (c).
1
1
1 4(x 2)
9 4x
Temos
< 4
4 < 0
< 0
< 0.
x2
x2
x2
x2
Para que uma frao seja negativa, o numerador e o denominador devem ter sinais
contrrios.
1o Caso: 9 4x > 0 e x 2 < 0. Devemos ter x < (9/4) e x < 2. Portanto, x < 2,
pois sendo menor do que 2, automaticamente x ser menor do que 9/4. Conclumos
que todo x < 2 satisfaz a desigualdade.
2o Caso: 9 4x < 0 e x 2 > 0. Devemos ter x > (9/4) e x > 2. Portanto, x > (9/4),
pois sendo maior do que 9/4, automaticamente x ser maior do que 2. Conclumos que
todo x > (9/4) satisfaz a desigualdade.
Juntando os dois casos, conclumos que x satisfaz a desigualdade se, e s se, x < 2 ou
x > (9/4).
67. Desigualdade dupla A opo correta (e).
Como os nmeros que aparecem so todos positivos, podemos elev-los ao quadrado
mantendo o sentido das desigualdades, obtendo
2 000 2 000 = 2 0002 < n(n + 1) < 2 0052 = 2 005 2 005.
Observe que n e n + 1 so inteiros consecutivos, portanto, as nicas opes so as
seguintes.
200

OBMEP 2010

Solues do Nvel 2
2 0002 < 2 000 2 001 < 2 0052

2 0002 < 2 001 2 002 < 2 0052

2 0002 < 2 002 2 003 < 2 0052

2 0002 < 2 003 2 004 < 2 0052

2 0002 < 2 004 2 005 < 2 0052


Logo, temos cinco possibilidades para n, a saber, 2 000, 2 001, 2 002, 2 003 e 2 004.
68. Dimetro do crculo Observe que OC um raio do crculo. Temos que OC =
AB = 5 cm, por serem as diagonais do retngulo OACB. Logo, o dimetro mede
10 cm.
69. Falta um ngulo A opo correta (d).
Lembre que a soma dos ngulos internos de um tringulo 180 . Do tringulo ST U,
b = 180 (75 + 30 ) = 75 . Logo, esse tringulo issceles (por ter
temos que T SU
dois ngulos iguais) e, portanto, T U = SU. Como T U = SV, segue que SU = SV.
Assim, o tringulo SUV tambm issceles e, portanto,
1
S Vb U = (180 50 ) = 65 .
2

70. Caf, bolo e gato Vamos listar os eventos ocorridos e contar o tempo gasto em
cada um. A primeira atividade foi colocar o gato fora da casa, logo nossa lista comea
com essa atividade e o tempo contado a partir dela.
Atividade
Gato fora de casa
Bolo no forno
Fazer o caf
Despertador toca
Gato entra em casa
Acabar de tomar o caf
Telefone toca
Desligar o telefone

Tempo depois que o gato


foi posto fora de casa
0 minutos
10 minutos
10 + 6 = 16 minutos
35 + 10 = 45 minutos
45 5 = 40 minutos
40 + 3 = 43 minutos
16 + (40 16) 2 = 28 minutos
28 + 5 = 33 minutos

Podemos, agora, dar as respostas.


(a) s 3h59min desliguei o telefone, o que ocorreu 33 minutos depois de colocar o
gato fora de casa. Como 59 33 = 26, coloquei o gato para fora s 3h26min.
(b) O despertador toca 45 minutos aps colocar o gato fora de casa.
(c) O gato j estava fora de casa por 28 minutos quando o telefone tocou.
Podemos saber exatamente a hora em que ocorreu cada atividade, conforme a tabela
seguinte.
OBMEP 2010

201

Solues do Nvel 2
Atividade
Gato fora de casa
Bolo no forno
Fazer o caf
Despertador toca
Gato entra em casa
Acabar de tomar o caf
Telefone toca
Desligar o telefone

Tempo depois que o gato


foi posto fora de casa
0 min
10 min
10 + 6 = 16 min
35 + 10 = 45 min
45 5 = 40 min
40 + 3 = 43 min
16 + (40 16) 2 = 28 min
28 + 5 = 33 min

Horrio
59 33 =
26 + 10 =
26 + 16 =
26 + 45 =
26 + 40 =
26 + 43 =
26 + 28 =
26 + 33 =

3h26min
3h36min
3h42min
4h11min
4h06min
4h09min
3h54min
3h59min

71. Muitos ngulos Na figura I, temos 63 + 18 + 95 = 176 , que menor do que


180 . Logo, esta figura est errada.
Na figura II, temos 112 + 72 = 184 , que maior do que 180 . Logo, esta figura est
errada.
Na figura III, temos 44 + 45 + 62 + 29 = 180 . Esta figura est correta.
72. Sinal de produto e de quociente
a
Como > 0 e 5 > 0, obtemos a > 0.
5
b
Como a > 0, temos 7a > 0. Como
> 0, segue que b > 0, portanto, b < 0.
7a
11
Como
> 0 e 11 > 0, obtemos abc > 0. Como b < 0 < a, segue que c < 0.
abc
18
Como
> 0 e 18 < 0, obtemos abcd < 0. Como abc > 0, segue que d < 0.
abcd

73. Sinais
e
radicais

Temos
3
11
=
9

11
=
99. Como 100 > 99, obtemos

10 = 100
10 3 11 > 0 e 3 11 10 < 0. Analogamente,
> 99

= 3 11, portanto,
temos 10 26 = 100 26 = 2 600 . Como 2 601 > 2 600, obtemos

51 = 2 601 > 2 600 = 10 26 ,

portanto, 51 10 26
>
0
e
10
26 51 < 0.
Finalmente, 182 = 324 < 325 = 25 13

garante que 18 < 5 13 , de modo que 18 5 13 < 0.

Os nmeros negativos so (b) 3 11 10, (c) 10 26 51 e (e) 18 5 13 .


74. ngulos entre retas Temos 80 + y = 180 , portanto, y = 100 . Como as retas r
e s so paralelas, segue que 60 + x + 80 = 180, donde x = 40 .
75. Variao de temperatura A variao de temperatura a diferena entre a mxima
e a mnima. Completamos a tabela dada com as variaes, como segue.
Dia
2a -feira
3a -feira
4a -feira
5a -feira
6a -feira
202

Temperatura
mxima,
em C
7
0
2
9
13

Temperatura
mnima,
em C
12
11
15
8
7
OBMEP 2010

Variao
da temperatura,
em C
7 (12) = 7 + 12 = 19
0 (11) = 0 + 11 = 11
2 (15) = 15 2 = 13
9 (8) = 9 + 8 = 17
13 (7) = 13 + 7 = 20

Solues do Nvel 2
Logo, a maior variao da temperatura ocorreu na sexta-feira.
76. Ordenando fraes A opo correta (d).
Lembre que a ordem entre fraes constitudas de inteiros positivos determinada pelo produto cruzado dos
a
c
inteiros, ou seja, < equivale afirmao ad < bc.
b
d
Desse modo, temos
1
1
2
4
3
4
5
< < < < <1< < ,
6
4
5
7
4
3
2
j que 4 < 6, 5 < 8, 14 < 20, 16 < 21, 3 < 4 (duas vezes) e 8 < 15, respectivamente.
Assim, 1/6 e 1/4 ficam esquerda de 2/5, 4/3 e 5/2 ficam direita de 3/4 e s 4/7
fica entre 2/5 e 3/4.
77. Frao de rea Observe que a regio em cinza na figura dada tem a mesma rea que a do enunciado. Como
todos os retngulos tm a mesma largura, o retngulo
maior est dividido em quatro partes iguais por segmentos paralelos ao seu comprimento.
Assim, a regio cinza representa uma quarta parte do retngulo maior.
78. Uma a mais!
5
:
2

(a) duas fraes cuja diferena

 
5
5
5 5
10
5

= + =
= .
4
4
4 4
4
2
(b) duas fraes cujo produto

5
:
2

10 7
10
5
10 14

=
=
= .
7
8
7
4
4
2

(c) duas fraes cuja soma

5
:
2

 
17
1
17 1
17 2
15
5
+
=
=
=
= .
6
3
6
3
6
6
6
2
(d) duas fraes cujo quociente

5
:
2

5 2
5 3
5
= = .
3 3
3 2
2

OBMEP 2010

203

Solues do Nvel 2
3
Logo, a frao que est sobrando .
2
79. Qual o ngulo? A opo correta (b).

K
b T
L

180-b

b = x, S KT
b = y, K LS
b = , K TbS = . O tringulo KLM issceles
Sejam T SM
porque tem dois lados iguais; consequentemente, seus ngulos da base so iguais, isto ,
cS = K LS
b = . Analogamente, o tringulo KST tambm issceles e, portanto,
KM
b
b
K ST = K T S = . Usaremos, agora, que a soma dos ngulos internos de um tringulo
180. Acompanhe na figura:
No tringulo SMT temos x + + 180 = 180 , portanto, x = .

No tringulo KLM temos + + 30 + y = 180 , portanto, y = 150 2.

No tringulo KST temos + + 150 2 = 180, portanto, = 15 .


Assim, x = 15 .

80. Operao circular Colocando x num dos crculos e aplicando a sucesso de opex+2
+ 1, donde x = 4.
raes obtemos x =
2
4
x+2+1
2

x
4

+2

x+2

+1

x+2
2

x+2
6

81. Pratos e copos Sejam c e p o nmero de copos e pratos que Iara pode comprar.
Observe que certamente c e p so nmeros inteiros e alm disso, como ela quer comprar,
no mnimo, quatro pratos e seis copos, temos p 4 e c 6. Como cada copo custa
R$ 2,50 e cada prato custa R$ 7,00, o custo de c copos e p pratos 2,5 c + 7 p. Mas
Iara s dispe de R$ 50,00, portanto, 2,5 c + 7 p 50. Assim, devemos encontrar dois
nmeros inteiros c e p que satisfaam p 4, c 6 e 2,5 c + 7 p 50.
Se Iara comprar quatro pratos, sobram 50 4 7 = 22 reais para os copos. Como
22 = 8 2,50 + 2, ela pode comprar mais oito copos (sobrando R$ 2,00).

Se Iara comprar cinco pratos, sobram 50 5 7 = 15 reais para os copos. Como


15 = 6 2,50, ela pode comprar mais seis copos.

204

OBMEP 2010

Solues do Nvel 2
Se Iara comprar seis pratos, sobram 50 6 7 = 8 reais para os copos, o que lhe
permite comprar apenas trs copos, que no o que ela quer.
Logo, Iara pode comprar quatro pratos e oito copos, ou cinco pratos e seis copos.
82. Desigualdades de inteiros A opo correta (c).
Somando 1 aos trs membros das duas desigualdades 5 < x 1 5 obtemos
5 + 1 < x 1 + 1 5 + 1, ou seja, 4 < x 6. Os valores inteiros de x que
satisfazem essas duas desigualdades so 3, 2, 1, 0, 1, 2, 3, 4, 5 e 6.
83. Nove quadrados A opo correta (d).

O lado de A mede 1 = 1 cm e o de B mede 81 = 9 cm. Agora temos:


Lado de G = lado de de B lado de A = 9 1 = 8 cm.
Lado de C = lado de B + lado de A = 1 + 9 = 10 cm.
Lado de F = lado de G lado de A = 8 1 = 7 cm.

Lado de H = lado de G + lado de F = 8 + 7 = 15 cm.

Lado de B + lado de C = lado de G + lado de F + lado de E, portanto,


9 + 10 = 8 + 7 + lado de E, ou seja, lado de E = 4 cm.
Lado de D = lado de C + lado de E = 10 + 4 = 14 cm.
Lado de I = lado de E + lado de D = 18 cm.
Finalmente, a rea de I mede 182 = 324 cm2 .
84. Muitas medalhas Denotemos por A, B, C e D o nmero de medalhas ganhas por
Andr, Bruno, Celina e Dalva, respectivamente. Ento A, B, C e D so inteiros no
negativos e A + B + C + D = 21. Temos que
Bruno ganhou o dobro de Celina, ou seja, B = 2 C.

Dalva ganhou trs a mais do que Bruno, ou seja, D = B + 3 = 2 C + 3.


Assim, atribuindo qualquer valor a C, automaticamente sabemos os valores de B e D.
Mas A + 2 C + C + 2 C + 3 = A + B + C + D = 21. Logo, A + 5 C = 18 e, portanto,
podemos expressar tambm A em termos de C, com A = 18 5 C. Observe que C 3,
pois se C = 4, ento A = 18 20 = 2, o que impossvel.
Como C um inteiro maior do que ou igual a 0 e menor do que 4, temos apenas as
possibilidades seguintes.
C
0
1
2
3

A
18
13
8
3

B
0
2
4
6

D
3
5
7
9

Como Andr foi o que recebeu mais medalhas, C = 3 no serve. O problema tem,
ento, trs possveis solues, listadas a seguir.
OBMEP 2010

205

Solues do Nvel 2
A
18
13
8

B
0
2
4

C
0
1
2

D
3
5
7

85. As somas so quadrados Com nmeros de 1 a 15, a soma de dois adjacentes , no


mnimo, 3 e, no mximo, 29. Os quadrados de nmeros inteiros de 3 a 29 so, apenas,
4, 9, 16 e 25. Verifiquemos quais so os nmeros de 1 a 15 que podem ser adjacentes,
ou vizinhos.
Nmeros
Vizinhos
possveis

1
3
8
15

2
7
14

3
1
6
13

4
5
12

5
4
11

6
3
10

7
2
9

8
1

9
7

10
6
15

11
5
14

12
4
13

13
3
12

14
2
11

15
1
10

Os nmeros 8 e 9 s tm, cada um, apenas um possvel vizinho, logo eles devem ser
colocados no incio e no fim da fila, seguidos de seus nicos vizinhos.
8

Sobram os nmeros 2, 3, 4, 5, 6, 10,11 12, 13, 14 e 15. Na tabela de vizinhos, vemos


que, ao lado do 7, s podemos colocar o 2 e, ao lado do 2, s o 14. Temos, ento,
8

14

Consultando a tabela de vizinhos e os nmeros que sobram, chegamos resposta.


8

15

10

13

12

11

14

Veja, a seguir, a soluo passo a passo.


Formao da linha em cada etapa
?
?
?
?
?
?
?

14

8
8
8
8
8
8

1
1
1
1
1
1

?
?
?
?
?
15

?
?
?
?
?
10

?
?
?
?
?
6

?
?
?
?
3
3

?
?
?
13
13
13

?
?
12
12
12
12

?
4
4
4
4
4

5
5
5
5
5
5

11
11
11
11
11
11

14
14
14
14
14
14

2
2
2
2
2
2

7
7
7
7
7
7

9
9
9
9
9
9

Sobram
2, 3, 4, 5, 6, 10, 11, 12,
13, 14, 15
3, 4, 5, 6, 10, 11, 12, 13,
14, 15
3, 4, 5, 6, 10, 11, 12, 13,
15
3, 4, 6, 10, 12, 13, 15
3, 6, 10, 12, 13, 15
3, 6, 10, 13, 15
3, 6, 10, 15
6, 10, 15
Resposta

86. rea de uma regio Lembre que a rea de um tringulo


1
base altura,
2
onde a altura relativa base escolhida. No tringulo AEB, temos base = AB =
comprimento do retngulo e a altura relativa a essa base BC = largura do retngulo.
1
Logo, AB BC = 24 e AB BC = 48. Logo, a rea do retngulo 48 cm2 . Assim,
2
a rea pedida 48 (24 + 13) = 48 37 = 11 cm2 .
206

OBMEP 2010

Solues do Nvel 2
87. Potncias de 10 A opo correta (c).
105 (102 )2 103
0,00001 (0,01)2 1 000
105 104 103
=
=
0,001
103
103
105+(4)+3
106
=
=
= 106(3) = 103
3
3
10
10
88. Diferena de quadrados A opo correta (d).
Como (x + y)2 = x2 + 2xy + y 2 e (x y)2 = x2 2xy + y 2 , temos

20 = (x + y)2 (x y)2 = x2 + 2xy + y 2 x2 + 2xy y 2 = 4xy,

portanto xy = 5.
89. Um quadriltero Para que ABCD seja um paralelogramo, seus lados devem ser
b + ABC
b = 180, as retas
dois a dois paralelos, isto , AB||CD e AD||BC. Como D AB
AD e BC so paralelas. Tambm as retas AB e DC so paralelas, pois temos dois
ngulos alternos internos de 45 entre essas retas. Assim, ABCD um paralelogramo.
90. Sexta-feira treze Como os dias da semana se repetem a cada 7 dias, a diferena
entre os dias da semana dada pelo resto ao dividir o nmero de dias transcorridos
por 7. Na tabela abaixo, temos
(a) na primeira linha, o nmero de dias entre o dia 13 de um ms e o dia 13 do ms
seguinte;
(b) na segunda linha, o resto obtido quando dividimos esse nmero por 7;
(c) na terceira linha, o resto obtido quando dividimos por 7 o nmero de dias entre o
13 de janeiro e o 13 do ms correspondente; assim, esse nmero obtido somando
os resultados obtidos na primeira linha, desde janeiro at o ms correspondente,
calculando, depois, o resto da diviso por 7.
J-F
31
3
3

F-M
28
0
3

M-A
31
3
6

A-M
30
2
1

M-J
31
3
4

J-J
30
2
6

J-A
31
3
2

A-S
31
3
5

S-O
30
2
0

O-N
31
3
3

N-D
30
2
5

Os valores iguais na ltima linha, significam que, nesses meses, o dia 13 caiu no mesmo
dia da semana. Em particular, a ltima linha nos diz que 13 de fevereiro, 13 de maro
e 13 de novembro correspondem ao mesmo dia da semana. Assim, no mximo, temos
trs sextas-feiras treze.
No caso de trs sextas-feiras treze num mesmo ano, o 13 de janeiro ocorreu 3 dias antes
de sexta-feira, isto , numa tera-feira, e o dia 10 de janeiro aconteceu 3 dias antes,
isto , num sbado.
Observao: Note que uma sexta-feira 13 ocorre apenas quando o primeiro dia do
ms cair num domingo. Assim, uma outra maneira, talvez mais simples, de resolver o
problema determinar o nmero mximo de vezes em que o primeiro dia do ms caia
num domingo num ano que no seja bissexto.
OBMEP 2010

207

Solues do Nvel 2
91. Tringulos com lados inteiros A opo correta (b).
Para que trs nmeros a, b e c sejam os comprimentos dos lados de um tringulo, cada
um deles deve ser maior do que a diferena e menor do que a soma dos outros dois.
Sejam a b c os comprimentos dos lados do tringulo, de modo que c < a + b.
Agora, somando c a ambos os membros, temos 2c < a + b + c = 12, ou seja, 2c < 12,
de modo que c < 6. Alm disso, como 3c a + b + c = 12, temos que c 4, de modo
que 4 c < 6.

No caso c = 5, temos que a + b = 7 e os possveis valores de a e b so a = 2 e b = 5,


ou a = 3 e b = 4. No caso c = 4, temos que a + b = 8 e, portanto, a nica soluo
a = b = 4. Concluso: temos 3 possveis tringulos.
92. Festa de aniversrio Denotemos por m o nmero de mas e por p o nmero de
peras que Ana compra, de modo que o peso que ela leva na sacola 300m+200p gramas.
Como a sacola aguenta, no mximo, 7 000 gramas, temos 300m + 200p 7 000, que
equivale a 3m + 2p 70. Como as peras pesam menos, Ana deve levar uma quantidade
maior de peras e, portanto, uma menor de mas. Como Ana quer fazer tortas de
ambas frutas, precisa levar pelo menos 1 ma.
Se ela levar uma ma, temos 2p 70 3 = 67, portanto p 33,5, o que significa que
Ana pode levar mais 33 peras, num total de 34 frutas. Se ela levar duas mas, temos
2p 70 6 = 64, portanto p 32, o que significa que Ana pode levar mais 32 peras,
novamente num total de 34 frutas. Se ela levar trs mas, temos 2p 70 9 = 61,
portanto p 30,5, o que significa que Ana s pode levar mais 30 peras, num total de
33 frutas.
Nas contas feitas, vemos que, a cada ma que Ana levar a mais, ela precisa comprar
1,5 peras a menos. Assim, se levar mais do que duas mas, nunca poder levar mais
do que 30 peras, num total sempre inferior a 34 frutas.
Concluso: o nmero mximo de frutas que Ana pode levar 34 frutas (ou uma ma
e 33 peras, ou duas mas e 32 peras).
93. Os dois quadrados Se a a medida do lado do quadrado maior e b a medida
do lado do quadrado menor, ento sabemos do enunciado que a2 = b2 + 2 001 . Logo
2 001 = a2 b2 = (a + b)(a b) e, como a e b so nmeros inteiros, temos que a + b
e a b so divisores de 2 001. Mas, 2 001 = 3 23 29, portanto, temos 4 possveis
formas de fatorar 2 001 em dois fatores, a saber,
(a + b)(a b) = 2 001 1 = 667 3 = 87 23 = 69 29.
Como (a + b) + (a b) = 2a, resulta
2 001 + 1
= 1 001;
2
667 + 3
(b) a + b = 667 e a b = 3, caso em que a =
= 335;
2
87 + 23
(c) a + b = 87 e a b = 23, caso em que a =
= 55;
2
69 + 29
(d) a + b = 69 e a b = 29, caso em que a =
= 49.
2
(a) a + b = 2 001 e a b = 1, caso em que a =

208

OBMEP 2010

Solues do Nvel 2
Assim, as possibilidades para o lado maior so 1 001, 335, 55 e 49 cm.
94. A multiplicao O maior quadrado no retngulo de 85 135 aquele de 85 85.
Sobra, ento, um retngulo de 50 85, em que o maior quadrado mede 50 50.
Continuando assim, obtemos
85 135 = 852 + 502 + 352 + 152 + 152 + 52 + 52 + 52 .
......................................................................................................................................................................................................................................................................................................................................................................................................................
...
...
.. 2.. 2.. 2..
.
.
.
..
..
...
......................................................
..
...
..
...
...
...
.....
...
..
..
..
...
...
.....
.....
.....
...
...
...
...
..
...
...
...
...
..
...
...
...
...
...................................................
...
...
....
....
...
...
..
..
...
...
...
.....
...
...
...
...
..
..
....
...
...
...
...
.
..
...........................................................................................................................................................
...
...
.
.
....
...
....
...
...
...
....
.....
....
...
...
...
...
...
...
...
...
...
....
...
....
....
....
....
..
...
...
...
..
..
..
...
..
...
..
...
...
...
...
.
.
.
....
....
....
..
..
..
...
...
...
...
..
..
...
...
...
.....
.....
.....
...
...
...
...
...
...
...
...
...
...
..
...
................................................................................................................................................................................................................................................................................................................................................................................................................

5 5 5

352

152

152

852

502

95. Expresso fracionria A opo


x
y
xy
=
Soluo 1: Temos
x
x
x
portanto,
xy
=
x
Soluo 2: Se

correta (c).
y
x
y
1
= 1 e, como
= 2, resulta que
= e,
x
y
x
2
x y
1
1
=1 = .
x x
2
2

x
xy
2y y
y
1
= 2, ento x = 2y e, portanto,
=
=
= .
y
x
2y
2y
2

96. Diferena e soma de quadrados


(a) Como a2 b2 = (a + b)(a b), temos
1 6782 1 6772 = (1 678 + 1 677)(1 678 1 677) = 3 355.
(b) Como (a + b)2 = a2 + 2ab + b2 , temos
1 0012 + 1 0002 = (1 000 + 1)2 + 1 0002 = 1 0002 + 2 000 + 1 + 1 0002 =
= 2 1 0002 + 2 001 = 2 002 001.
(c) Como (a b)2 = a2 2ab + b2 , temos
19 9992 = (20 000 1)2 = (2 104 )2 4 104 + 1 =
= 4 108 4 104 + 1 = 399 960 001.
OBMEP 2010

209

Solues do Nvel 2
(d) Colocando em funo de 2 000, temos
2 0012 + 2 0022 + 2 0032 = (2 000 + 1)2 + (2 000 + 2)2 + (2 000 + 3)2 =
= 3 2 0002 + 12 2 000 + 14 = 12 024 014.
97. Um queijo triangular Para dividir o queijo em 5 partes iguais, suficiente dividilo em 5k partes iguais e dar k partes a cada um. Observe que se dividirmos cada lado
para servir de base de dois tringulos equilteros menores, obtemos 4 = 22 tringulos
menores no total; dividindo cada lado para servir de base de trs ou quatro tringulos
equilteros menores, obtemos 9 = 32 ou 16 = 42 tringulos no total. Dessa maneira,
a menor diviso em 5k tringulos menores alcanada quando 5k um quadrado, ou
seja, quando k = 5. Essa partio mostrada na figura, em que o queijo foi partido
em 25 = 52 = 5k tringulos.

98. Notas de Matemtica Devemos encontrar os valores dos smbolos na soma



+ >
> 
As duas notas so nmeros de dois algarismos e a soma deles tm trs algarismos, de
modo que a soma precisa ser maior do que 100 e menor do que 200. Assim, temos que
> = 1. Mas, Cludia obteve 13 pontos a mais do que Joo, portanto,

+ 1 3
1
Agora, como a soma de e 3 termina em 1, temos que = 8 e, portanto,  = 6.
Assim, as notas de Cludia e Joo so, respectivamente, 81 e 68.
99. Operao com raiz quadrada A opo correta (c).
210

OBMEP 2010

Solues do Nvel 2
Observe que, denotando por A a expresso dada, temos
q
i2
h

2
A = ( 6 + 2)( 3 2)
3+2
q

2

= ( 6 + 2)2 ( 3 2)2
3+2

= ( 6 + 2)2 ( 3 2)2 ( 3 + 2)



= ( 6 + 2)2 ( 3 2) ( 3 2)( 3 + 2)


= (6 + 2 12 + 2)( 3 2) ( 3)2 22

= (6 + 2 12 + 2)( 3 2)(1)

= (8 + 4 3)(2 3) = 4(2 + 3)(2 3)

= 4(22 ( 3)2 ) = 4 1 = 4.

Assim, A2 = 4 e, portanto, A pode ser 2 ou 2. Como 3 2 negativo e os outros


dois fatores de A so positivos, temos que A deve ser negativo, ou seja, A = 2.
100. Para a escola de bicicleta Seja t o tempo que Ctia gasta pedalando a 20 km/h.
Pedalando a 10 km/h, ela faz o percurso no dobro do tempo que pedalando a 20 km/h,
isto 2t. No entanto, como ela demora 45 minutos a mais, temos 2t t = 45, de modo
que t = 45 min. Logo, diariamente, ela sai da escola 45 minutos antes das 16h30m,
isto , s 15h45m, e o percurso at sua casa, que feito em 45 min a 20 km/h, tem
3
20 = 15 km. Para sair da escola s 15h45m e chegar em casa s 17h, ela deve
4
percorrer esses 15 km entre a escola e sua casa em 1h15m, o que corresponde a 5/4 de
hora. Portanto, Ctia deve manter uma velocidade de
60
15 km
=
km/h = 12 km/h.
5/4 h
5

101. Distncia na reta


Soluo 1: Como a maior distncia entre dois pontos 19 e a menor 2, desenhamos
uma reta numrica com os dois pontos 0 e 19 nas extremidades e o ponto 2 a duas
unidades de 0, obtendo os primeiros trs pontos na figura.
0.........................................2.........................................................................................................................................................................................................................................................................................................................................19
.......
p1
p3
p2

Em seguida, colocamos dois outros pontos, para tentar fechar as distncias exigidas.
Como precisamos ter distncias 5 e 7, colocamos o ponto 7 na reta, o que nos d
distncias que no so incompatveis com os dados do problema, j que as distncias
entre esses 4 pontos so 2, 5, 7, 12, 17 e 19, conforme figura.
0.........................................2.......................................................................................................7.......................................................................................................................................................................................................................................................19
.......
p1
p3
p4
p2

OBMEP 2010

211

Solues do Nvel 2
Desse modo, se nossa tentativa de colocar todos os pontos tiver xito, necessariamente
k = 12. Temos sorte pois, para obter as distncias 4, 8, 13 e 15, basta colocar o ponto
15 na reta, obtendo todas as distncias 2, 4, 5, 7, 8, 12, 13, 15, 17 e 19, conforme
figura.
0..........................................2.......................................................................................................7...................................................................................................................................................................15
19
...........................................................................................
p1
p3
p4
p5
p2

Escolhendo 4 como terceiro ponto, obtemos uma outra distribuio de pontos com as
mesmas distncias entre eles, como na figura seguinte, em que, novamente, k = 12.
0..................................................................................4...................................................................................................................................................................12
14
19
..........................................................................................................................................................
p1
p3
p4
p5
p2

Soluo 2: Como a maior distncia 19 podemos, supor que um ponto 0 e outro


19. Se um terceiro ponto for igual a a, teremos as distncias a 0 = a e 19 a
na lista de distncias dada. Como nessa lista de distncias aparecem os pares 2 e 17,
bem como 4 e 15, podemos escolher o nmero a = 2 ou a = 4 como terceiro ponto.
Escolhamos o ponto 2. Como 4 e 15 esto na lista das distncias, temos que 4 ou 15
outro ponto na reta; mas, 4 no pode ser um dos pontos porque a distncia 2, entre 2
e 4, no aparece duas vezes. Logo, 15 o quarto ponto na reta. Por ltimo, o quinto
ponto tem que estar a uma distncia 5 de um dos pontos e 7 de outro. Assim, o ponto
que falta o ponto 7 e a distncia desconhecida k = 19 7 = 12.

Tomando 4 como terceiro ponto, obteramos os pontos 12 e 14 como quarto e quinto


pontos e, novamente, a distncia desconhecida k = 12.
102. Nmero mpar A opo correta (c).

Lembremos que a soma ou diferena de nmeros de mesma paridade um nmero par:


par par = par

mpar mpar = par.

Observemos que n2 e n3 podem ser pares ou mpares, portanto n2 + 5 e n3 + 5 podem


ser mpares ou pares, dependendo de n ser par ou mpar. Restam as opes (a), (b) e
(c).
Soluo 1: Ambos n2 n e n2 +n so soma e diferena de dois nmeros que sempre tm
a mesma paridade, portanto, esses nmeros sempre sero pares, do mesmo modo que
n2 n+2 e n2 +n+2. Finalmente, a opo correta (c), porque n2 +n+5 = (n2 +n)+5,
que soma de um par e um mpar, sempre ser um nmero mpar, para todo valor
inteiro de n.
Soluo 2: Observemos que n2 n = n(n 1) e n2 + n = n(n + 1) so o produto de
dois nmeros consecutivos, portanto, so sempre pares, do mesmo modo que n2 n + 2
e n2 + n + 2. Finalmente, a opo correta (c), porque n2 + n + 5 = (n2 + n) + 5 a
soma de um par com um mpar, que sempre mpar, para todo valor inteiro de n.
103. Quatro nmeros inteiros A opo correta (e).
212

OBMEP 2010

Solues do Nvel 2
Como m, n, p e q so inteiros, tambm 7 m, 7 n, 7 p e 7 q so inteiros. Agora,
4= 122 e
4 = (1) (2) 1 2
a nica decomposio de 4 em um produto de nmeros inteiros distintos. Segue que
(7 m) + (7 n) + (7 p) + (7 q) = (1) + (2) + 1 + 2 ,
portanto, m + n + p + q = 28.
104. As pginas do dicionrio Observemos que:
(a) a cada dez nmeros imprime-se uma vez o 1 nas unidades;
(b) a cada cem nmeros imprime-se dez vezes o 1 nas dezenas;
(c) a cada mil nmeros imprime-se cem vezes o 1 nas centenas.
Assim, de 1 at 999, imprime-se o algarismo 1 um total de 300 vezes, das quais 100
vezes nas unidades, 100 vezes nas dezenas e 100 vezes nas centenas.
De 1 000 at 1 999, imprime-se o algarismo 1 outras 300 vezes dentre unidades, dezenas
e centenas, mais 1 000 vezes na posio do milhar, portanto, entre 1 e 1 999, o nmero
de vezes que se imprime o 1 300 + 300 + 1 000 = 1 600.
Agora, entre 2 000 e 2 999, imprime-se o 1 mais 300 vezes,
1 600 + 300 = 1 900.

completando

De 3 000 a 3 099, temos 20 algarismos 1, de 3 100 a 3 119, temos 32 algarismos 1 e, de


3 120 a 3 149, temos 32 algarismos 1, portanto, at 3 149, o nmero de vezes que se
imprime o 1 1 900 + 20 + 32 + 32 = 1 984. Como faltam 4 algarismos 1, o nmero de
pginas do dicionrio 3 152.
105. Soma de potncias de 2
n
Soluo 1: Observe que 2 + 2 + 2 = (2 ) + 2 2 2 + (2 2 )2 . Logo, para n = 12,
temos 28 + 211 + 212 = (24 + 26 )2 . Assim, n = 12 uma soluo.
8

11

4 2

Soluo 2: Se 28 + 211 + 2n = k 2 , ento


28 + 23 28 + 2n
9 28 + 2n
2n
2n

= k2
= k2
= k 2 (3 24 )2
= (k 3 24 )(k + 3 24 ).

Logo, (k 3 24 ) e (k + 3 24 ) so potncias de 2, ou seja, k + 3 24 = 2a e


k 3 24 = 2b , com a + b = n e
2a 2b = (k + 3 24 ) (k 3 24 ) = 3 25 = 96.
Examinemos a lista das potncias de 2:
2, 4, 8, 16, 32, 64, 128, 256, . . . .
Constatamos que a diferena dessas potncias s 96 entre 128 = 27 e 32 = 25 . Logo,
a = 7 e b = 5. Assim, n = 7 + 5 = 12 a nica soluo.
OBMEP 2010

213

Solues do Nvel 2
106. Reverso de um nmero Lembremos que nmeros ab de dois algarismos, em que
a o algarismos das dezenas e b o das unidades, so dados por ab = a 10 + b. Por
exemplo, 47 = 4 10 + 7.
Seja ab um nmero de dois algarismos; seu reverso , ento, ba. Temos que
ab + ba = a 10 + b + b 10 + a = (a + b) 11.
Por outro lado, a, b 9, de modo que a + b 18. Como 11 um nmero primo e
a + b 18, 11 no divide a + b e, portanto, o produto (a + b) 11 s um quadrado
perfeito se a + b = 11. Assim, temos 8 nmeros satisfazendo a condio do problema:
29, 38, 47, 56, 65, 74, 83 e 92.
107. ngulos externos de um tringulo Observemos que os ngulos y, 150 e 160 so
ngulos externos de um tringulo, de modo que y +150 +160 = 360 . Assim, y = 50 .
Pela mesma razo, conclumos que z = 50 . Como x, y e z so ngulos internos de um
tringulo, temos x + y + z = 180 e, portanto, x = 80 .
108. Uma brincadeira Sejam a, b, c e d os nmeros procurados. So dados os nmeros
a+b+d
a+c+d
a+b+c
+ d,
+ c,
+b e
3
3
3

b+c+d
+ a,
3

mas no sabemos sua ordenao. No entanto,


90 = 17 + 21 + 23 + 29
a+b+c
a+b+d
a+c+d
b+c+d
=
+d+
+c+
+b+
+a
3
3
3
3
= 2(a + b + c + d)
e, portanto, 2(a + b + c + d) = 90, ou seja, a + b + c + d = 45. Seja, agora, d o maior
dentre os nmeros a, b, c e d. Ento
d = 29

a+b+c
45 d
= 29
,
3
3

e conclumos que d = 21.


109. Ovos e mas A opo correta (b).
Como o enunciado e a resposta so percentuais podemos, nesse caso, estipular qualquer
preo e qualquer unidade monetria, que a resposta ser, sempre, a mesma. O mais
simples, portanto, supor que, inicialmente, uma dzia de ovos custava 100 e, portanto,
que 10 mas tambm custavam 100. Como o preo dos ovos subiu 10%, o novo valor
dos ovos 110. O preo das mas diminuiu 2%, portanto, o novo preo de 10 mas
98. Assim, enquanto antes gastava-se 200 na compra de uma dzia de ovos e 10
mas, agora gasta-se 110 + 98 = 208. Da, temos que o aumento foi de 8 em 200, o
que corresponde ao percentual de
8
4
=
= 4%.
200
100

214

OBMEP 2010

Solues do Nvel 2
110. Dividir um cubo A opo correta (e).
Convertendo metros em milmetros, temos 1 m = 1 000 mm. Assim, o cubo ficou
dividido em 1000 1000 1000 = 109 cubinhos, cada um com uma aresta de 1 mm.
Empilhando-se os 109 cubinhos sucessivamente um em cima do outro, obtemos uma
coluna de 109 mm = 1000 km de altura.
111. Uma expresso A opo correta (b).
a2
4a
22 a
a1(3)
a4
25
7
7

=
a

=
a

=
a

a5
(21 a)3
23 a3
2
2
7+4
3
a
a
1
=
=
= 3
2
2
2a
112. Uma igualdade Fatorando 96, temos 25 3 a2 = b3 . Para que 25 3 a2 seja um
cubo, o nmero a deve ter, pelo menos, a fatorao 2n 3n . Como queremos o menor
valor de a, tomamos a = 2n 3n e, assim,
25 3 a2 = 25 3 (2n 3m )2 = 25+2n 31+2m .
Logo, 5 + 2n e 1 + 2m so mltiplos de 3. Os menores valores de n e m so n = 2 e
m = 1. Portanto, a = 22 3 = 12.
113. Somas de trs em trs Somando de trs em trs quatro nmeros a, b, c e d, obtemos
os nmeros a + b + c, a + b + d, a + c + d e b + c + d, sendo
(a + b + c) + (a + b + d) + (a + c + d) + (b + c + d) = 3(a + b + c + d).
Como 30 = 6 + 7 + 8 + 9 = (a + b + c) + (a + b + d) + (a + c + d) + (b + c + d), resulta
30
= 10. Como cada nmero igual
30 = 3(a + b + c + d), donde a + b + c + d =
3
diferena entre a soma dos quatro nmeros e a soma dos outros trs, os nmeros
procurados so
10 6 = 4 , 10 7 = 3 ,

10 8 = 2 e 10 9 = 1.

114. O retngulo do Lus Faremos a diviso com retngulos. Observamos que 24 = 64


e 12 = 6 2, portanto, Lus pode fazer um primeiro corte a 4 cm no lado de 10 cm
e outro corte a 2 cm do corte anterior. Depois desses cortes, resta um retngulo de
tamanho 6 4. Por ltimo, como 16 = 4 4, basta fazer mais um corte a 4 cm no
lado que mede 6 cm. Duas opes de cortes esto ilustrados nas figuras seguintes, com
indicao das dimenses dos lados e das reas.
4

24

12

4
16

4
4
6

OBMEP 2010

16

24
12

2
215

Solues do Nvel 2
115. Uma fbrica de blusas A opo correta (c).
Denotemos por x o nmero de unidades produzidas. Ento o custo de produo
500 + 2x reais. Na venda, o fabricante est recebendo 2,5x. Assim, ele ter lucro
quando 2,5x > 500 + 2x, isto , 0,5x > 500, ou x > 1 000.
116. Existncia de tringulos A opo correta (c).
A soma dos trs ngulos internos de um tringulo 180. Logo, se um deles mede 90 , a
soma dos outros dois 90 e, por isso, no podem ser maiores do que 90 . Portanto, no
existem tringulos retngulos obtusngulos. Os seguintes exemplos de comprimentos
de lados mostram que cada um dos outros casos pode ocorrer:

(a) 2, 3, 3;
(b) 1, 1, 2;
(d) 3, 4, 5;
(e) 3, 4, 6.
117. Os doze pontos No total, temos 11 possveis quadrados, mostrados nas figuras
seguintes.
5 quadrados
s
s
s
s

s
s
s
s

4 quadrados

..........................
...
...
..
..
..
.
.......................................................................
..
..
..
...
...
...
.
...
....
...
...
...
.
....................................................................
..
..
...
...
...
...
........................

s
s

s
s
s
s

s
s
s
s

2 quadrados

...
....
.... ....... ...... ......
....
....
..
.........
....
....
.
.
.
.
.
.
....
..
......
...
.... ............. ............. .......
.....
.......
.
.
. ... ......... ...........
.
.
. . .
.
...... .......... .......... .....
....
.... ....
..
....
. .
...
.... ........... ......
......... .... ....
...
..

s
s

s
s

s
s
s
s

s
s
s
s

....
....
... ................ ..
.. ......... ..................
...........
...........
.
.
.
.
.
.
... ......
........ ...
... ..
......
..
.....
.
.....
.. ...
........ .....
.............
........
.
.
.........
..
... ........ ..................
... ............... ..
....
....

s
s

s
s

s
s

118. O colar Sejam g o nmero de prolas grandes, p o de pequenas, a o peso (em


gramas) de uma prola grande e b o de uma pequena. Com essa notao, os dados so
os seguintes.
(a) O nmero total de prolas no colar g + p e temos g + p < 500.
(b) O peso das prolas grandes g a e o das pequenas p b.
(c) O peso total do colar g a + p b.
Antes de equacionar o problema, equacionamos as duas hipteses do problema.
(a) Ao substituirmos 70% das prolas grandes pelas pequenas, o nmero total de
prolas no colar fica composto por
30% g + p + 70% g = 0,3 g + (p + 0,7 g)
| {z }
|
{z
}
pequenas

grandes

prolas e seu peso fica sendo


0,3 g a
| {z }

peso das grandes

(p + 0,7 g) b = 0, 4(g a + p b) .
|
{z
}
|
{z
}

peso das pequenas

40%

do peso inicial

(b) Analogamente, ao substituirmos 60% das prolas pequenas pelas grandes, o nmero total de prolas no colar fica composto por
g + 60% p + 40% p = (g + 0,6 p) + 0,4 p
| {z }
|
{z
}
grandes

216

pequenas

OBMEP 2010

Solues do Nvel 2
prolas e seu peso fica sendo
(g + 0,6 p) a +
|
{z
}

peso das grandes

0,4 p b
| {z }

peso das pequenas

1,7(g a + p b)
|
{z
}

170%

do peso inicial

Assim, as duas hipteses podem ser resumidas no sistema



(0,3) g a + (0,7) g b + p b = 0,4(g a + p b) ,
g a + (0,6) p a + (0,4) p b = 1,7(g a + p b).

Para resolv-lo, comeamos multiplicando ambas equaes por 10 e simplificamos, obtendo


(
(7 g + 6 p) b = g a ,
(7 g 6 p) a = 13 p b.
7g +6p
a
13 p
= =
, resultando
g
b
7g 6p
49 g 2 36 p2 + 13 p g = 0 . Para fatorar essa expresso, desdobramos
Eliminando as incgnitas a e b, podemos escrever

13 p g = 49 p g 36 p g
e obtemos
0 = 49 g 2 + 13 p g 36 p2 = 49 g 2 + 49 p g 36 p2 36 p g
= g (49 g 36 p) + p (49 g 36 p)
= (g + p)(49 g 36 p) ,
de modo que (g + p)(49 g 36 p) = 0 , ou seja,
49 g = 36 p .
Como 36 e 49 so primos entre si e p e g so inteiros positivos, segue que g um
mltiplo de 36 e p um de 49, isto , g = 36 k e p = 49 k , para certos inteiros k e k
maiores do que 1. Decorre que 36 49 k = 36 p = 49 g = 49 36k, ou seja, k = k , de
modo que g = 36 k, p = 49 k e g + p = 85 k. Como g + p < 500, o colar s pode ter 85,
170, 255, 340 ou 425 prolas.
119. Mulheres votantes A opo correta (b).
52
40
e, delas, a frao que votante
. Logo,
A frao de mulheres na populao
100
100
a frao de mulheres votantes
52
40
52

100% =
100% = 0,208 100% = 20,8%.
100 100
250
120. Amigos do sculo XX Os dois amigos nasceram no mesmo ms e no mesmo ano,
com uma diferena de 7 dias, de modo que um nasceu no dia d/m/a e o outro no
dia (d + 7)/m/a. Com essas datas formamos os nmeros (d)(m)(a) e (d + 7)(m)(a).
Sabemos que
(d + 7)(m)(a) = (d)(m)(a) + 7 10k ,
OBMEP 2010

217

Solues do Nvel 2
onde k o nmero de algarismos de (m)(a). Observe que s podemos ter k = 3, se o
ms m tem um algarismo, ou k = 4, se o ms m tem dois algarismos. Como tambm
(d + 7)(m)(a) = 6 (d)(m)(a), resulta
7 10k = 5(d)(m)(a).

No caso k = 3, decorre que o amigo mais velho nasceu em


7 000
= 1 400,
5
70 000
isto , 1o de abril de 1900. No caso k = 4, decorre
= 14 000, que no uma
5
data vlida.
(d)(m)(a) =

121. Operao em uma frao Seja a/b uma frao qualquer e seja c um nmero
qualquer tal que
a+c
b
= .
b+c
a
Esta igualdade equivalente a (a + c)a = (b + c)b, ou seja, a2 + ac = b2 + bc ou, ainda
a c(a b) = b2 a2 = (b a)(a + b). Evidenciando a b, vemos que o que se quer
um nmero c tal que
(a b)(a + b + c) = 0.

Essa igualdade pode ser satisfeita de duas maneiras.


a
(a) Se a = b, temos simplesmente 1 =
e podemos somar qualquer nmero c aos
b
dois termos da frao para obter 1 novamente.
(b) Se a 6= b, precisamos ter a+b+c = 0 e, nesse caso, s podemos somar c = (a+b)
aos dois termos da frao a/b para obter b/a.

122. O nmero 119 Dados inteiros positivos d e r, dizemos que N dividido por d deixa
resto r se existir um inteiro n tal que N = nd + r. Se M dividido por d deixar o mesmo
resto r, ento existe um inteiro m tal que M = md + r. Nesse caso, se M > N, resulta
que m = n + p para algum inteiro n e, portanto,
M = md + r = (n + p)d + r = nd + r + pq = N + pd,
de modo que M N = pd um mltiplo de d. O mesmo ocorre se M < N.

Como 119 tem todas as propriedades arroladas, decorre que se N for algum nmero
com essas mesmas propriedades ento, necessariamente 119 N um mltiplo de 2,
3, 4, 5 e 6. Como o menor mltiplo comum de 2, 3, 4, 5 e 6 60, N tem as mesmas
propriedades de 119 se, e s se, 119 N for um mltiplo de 60. Assim, os nicos
nmeros N com as mesmas propriedades de 119 so da forma N = 119 + 60 p,
para algum inteiro p. Para obter N positivo, precisamos tomar p 1 e, para obter
N 2 007, precisamos tomar p 31, pois 119 + 60 32 = 2 039 > 2 007.
Assim, os nicos nmeros inteiros positivos e menores do que 2 007 com as mesmas
propriedades de diviso de 119 so
59, 119, 179, . . . , 1979 (= 119 + 60 31),
num total de 33 nmeros.
218

OBMEP 2010

Solues do Nvel 2
123. Fonte com trs torneiras Para simplificar, numeramos os 10 garrafes de acordo
com os respectivos tempos que levam para ficar cheios, de 1 a 10.
Soluo 1: Uma ideia utilizar o tempo que sobra de um garrafo para encher
outro garrafo, enchendo simultaneamente outros dois. As figuras seguintes ilustram
a soluo. Na Figura I, as 3 torneiras gastam 10 minutos para encher os garrafes 10,
Figura I: 10 min

..
..
...
....
...
..
...
...
...
...
...
...
...
...
.
....................................

10

..
..
...
....
...
..
...
...
...
...
...
...
...
...
.
....................................

..
..
...
....
...
..
...
...
...
...
...
...
...
...
.
....................................

...
...
...
...
...
..
...
...
..
..
...
...
...
.
...
...................................

...
...
...
...
...
..
...
...
..
..
...
...
...
.
...
...................................

Figura II: 9 min

..
..
...
....
...
..
...
...
...
...
...
...
...
...
.
....................................

..
..
...
....
...
..
...
...
...
...
...
...
...
...
.
....................................

..
..
...
....
...
..
...
...
...
...
...
...
...
...
.
....................................

...
...
...
...
...
..
...
...
..
..
...
...
...
.
...
...................................

...
...
...
...
...
..
...
...
..
..
...
...
...
.
...
...................................

9, 8, 2 e 1 e, na Figura II, as 3 torneiras gastam mais 9 minutos para encher os garrafes


7, 6, 5, 4 e 3. Logo, o tempo total gasto de 19 minutos.
Soluo 2: Se tivssemos uma torneira s, o tempo gasto para encher os 10 garrafes
seria de 1 + 2 + + 9 + 10 = 55 minutos. Como temos trs torneiras e 55 = 3 18 + 1,
uma torneira, pelo menos, vai levar 19 minutos e as outras duas, 18 minutos cada. A
tabela seguinte mostra uma forma de fazer o trabalho em 19 minutos.
Torneira 1
Torneira 2
Torneira 3

10
8
7

9
5
6

3
4

2
1

124. A sequncia xyz Igualando os denominadores, verificamos que a sequncia dada


igual a
4 5 6 7
, , , , x, y, z.
8 8 8 8
Assim, o denominador sempre 8 e os numeradores so consecutivos. Logo,
10
5
8
9
x = = 1, y = e z =
= .
8
8
8
4

.............................
...............
.......
.......
..... 
125. A mesa circular Se a prxima pessoa a se sentar vai ter que se
.....
.
....
.
.
.
.
....
....
...
...
... 
.
...
sentar ao lado de uma cadeira ocupada, isso significa que existem
..
.
...
....
...
...
..
.
..
.
no mximo 2 cadeiras consecutivas desocupadas. (Na figura, as
...
..
...
..
...
..
.
.
...
.
.
cadeiras ocupadas esto representadas por quadradinhos pretos e
.
...
..
...
...
....
....
....
....
.....
as desocupadas por quadradinhos brancos.)
....
.
.
.......
.
.
.
.
..........
...........
..........................

Podemos, ento, pensar nas cadeiras em grupos ordenados de 3 cadeiras, em que a


terceira j est ocupada. Logo, o menor valor de N 60 3 = 20.
126. Nmeros proporcionais A opo correta (d).
x
3
Como = , ento xz = 3y. Elevando ao quadrado ambos os membros dessa igualy
z
dade, obtemos x2 z 2 = 9y 2 .
OBMEP 2010

219

Solues do Nvel 2
127. Esportistas de uma escola A opo correta (c).
Denotemos por x o nmero de estudantes que praticam simultaneamente os dois esportes. Logo, o nmero de estudantes que pratica somente futebol 20 x e o que
pratica somente vlei 19 x. Portanto, os 15 estudantes que praticam exatamente
um esporte esto divididos em 15 = (20 x) + (19 x). Segue que x = 12 e resulta
que 20 + (19 x) = 27 estudantes praticam pelo menos um dos dois esportes. Assim,
13 = 47 27 estudantes no praticam nem futebol nem vlei.
128. Vamos ao teatro A opo correta (c).
3
Mrio pagou 3 e levou 5, portanto, pagou apenas do preo usual, tendo economizado
5
2
2
40
. Como =
, a economia foi de 40%.
5
5
100
129. Uma desigualdade A opo correta (c).
Note que o inverso de um nmero s maior do que 1 quando o nmero for positivo e
menor do que 1. Portanto,
1
> 1 0 < x 1 < 1 1 < x < 2 .
x1
130. A sala do Professor Newton A opo correta (c).
Como o nmero de alunos homens menor do que 15 e o das mulheres 15, temos que
15 < alunos homens + alunas mulheres < 15 + 15 = 30, ou seja, o nmero do total de
alunos est entre 15 e 30.
Soluo 1: Quando dividimos o nmero de alunos por 4 sobram 2 alunos, ento o
nmero de alunos par. Quando dividimos por 5 sobra um, ento o ltimo algarismo
do nmero de alunos 1 ou 6; sendo par, s pode ser 6. Assim, s temos dois possveis
valores, 16 ou 26. Descartamos 16 por ser divisvel por 4, de modo que o nmero de
alunos 26. Consequentemente, temos 26 15 = 9 alunos homens.

Soluo 2: Observemos que o nmero 6, dividido por 4, deixa resto 2 e, dividido por
5, deixa resto 1. Logo, se somamos a 6 um mltiplo comum de 4 e 5, o nmero obtido
tambm ter essa propriedade. O menor mltiplo comum de 4 e 5 20, portanto, os
possveis valores para o nmero de alunos 6, 26, 46, 66, . . . . Como o nmero de alunos
est entre 15 e 30, esse nmero 26 e resulta que temos 26 15 = 9 alunos homens.
131. Um jardim retangular A opo correta (b).
Pelos dados do problema sabemos que
3
3
AD = AB
e
AB = AF.
5
5
3
3 2
9
Logo, AF = AB =
AD =
AD. A rea do terreno AB AD e a rea do
5
5
25
jardim AB AF, portanto a razo entre essas reas
AB AF
AF
9
36
=
=
=
= 36%.
AB AD
AD
25
100

220

OBMEP 2010

Solues do Nvel 2
132. Nmeros decrescentes Observe que 0 <
1 < 32/3 < 32 < 33 , resulta

1
1
1
< <
0 < a < b < c. Como
c
b
a

1
1
1
< 2 < 2/3 < 1.
3
3
3
3

1
1
1
Tambm 1 < 3 < 35 , portanto 1 < 5 3 < 3. Resta observar que 3 = ( )3 , 2 = 32 ,
3
3
3
1
1 1
2/3
=3
e3=
, para estabelecer que
32/3
3
 1 1
 1 3

5
< 32 < 32/3 < 3 <
.
3
3
133. Os bombons misturados Sejam x o nmero de bombons que Marta ganhou e y o
4
que Carmem ganhou. Sabemos que x + y = 200, x < 100 e x > y. Ento y 100
5
4
e, portanto, x > 100 = 80. Assim, x um inteiro compreendido entre 80 e 100.
5
Como deve ser um mltiplo de 8, s pode ser 88 ou 96. Vamos decidir qual .
4
(a) Se x = 88, ento y = 200 88 = 112. Logo, 88 = x > 112 = 89,6, o que no
5
possvel.
4
(b) Se x = 96, ento y = 200 96 = 104 e 96 = x > 104 = 83,2, o que possvel.
5
Assim, Marta ganhou 96 bombons e Carmem 104.
134. Jantar aos sbado Para simplificar, vamos denotar cada casal por um par de
nmeros, um nmero representando o marido e o outro a mulher. Temos, ento, os
trs pares (1, 2), (3, 4), (5, 6), que no podem ser vizinhos. Podemos considerar o lugar
do marido 1 mesa como sendo fixo, j que desconsideramos rotaes na disposio
mesa. Duas disposies possveis so dadas na figura seguinte.
1

................................
.......
....
....
....
...
...
..
...
.
..
...
..
...
..
..
..
...
..
.
...
.
.
.
....
.
.....
....
............ ...............
........

................................
.......
....
....
....
....
...
....
..
..
...
..
...
..
..
..
..
.
.
...
.
.
....
.
..
....
.....
........
.............................

5
132 546

5
164 523

Essas disposies descrevemos por 132546 e 164523, sempre comeando em 1, fixado


numa posio mesa, e prosseguindo no sentido horrio. Assim, o problema se resume
em encontrar todos os nmeros de 6 algarismos distintos que podem ser escritos com
os algarismos 1, 2, 3, 4, 5 e 6, onde:
(a) todos os nmeros comeam com o algarismo 1;
(b) nenhum nmero pode terminar com o algarismo 2;
(c) no podem aparecer juntos 1 e 2, 3 e 4, 5 e 6.
Encontramos os 16 nmeros da tabela seguinte.
OBMEP 2010

221

Solues do Nvel 2
132 546
142 536

132 645
142 635

135 246
145 236

135 264
145 326

135 426
146 235

136 245
146 325

136 254
153 246

136 425
154 236

Logo, a resposta 16 sbados.


135. Expresso com radicais A opo correta (e).
r
4
q


2

1+ 1+ 1
= 1 + 2 = 1 + 2 2 + 2 = 3 + 2 2.
136. Possveis tringulos Como a soma dos comprimentos dos lados menores de um tringulo deve ser maior que o comprimento do lado maior, devemos ter
a + (a + 2) > a + 5, ou seja, a > 3.
137. Uma diferena A opo correta (a).
Efetuando as operaes indicadas, obtemos

0,1 20
0,4 ( 0,09 1)
20

= (0, 3 1) = 4 + 0,7 = 3,3.

0,5
5
0,4
138. A Terra A frao de terra que cultivada
1

15 6 5
4
2 1
=
=
.
5 3
15
15

Como a terra ocupa 3/10 da superfcie total do globo terrestre, resulta que a rea
3
2
2
4
8
2
4

=
, isto ,
=
=
= 8% da superfcie do globo
cultivada
15 10
25
25
25 4
100
terrestre.
139. Uma frao A figura mostra que MN paralelo a BC, portanto, os tringulos
ABC e AMN so semelhantes e, por isso, seus lados so proporcionais. Usando o lado
AM
4
dos quadradinhos da grade da figura, obtemos
= . Assim,
AB
7
AN
AM
4
=
= .
AC
AB
7
140. Clculo de ngulo A opo correta (c).
cS e QTbW so complementares.
Como as retas P Q e RS so paralelas, os ngulos T W

b
Assim, QT W = 180 110 = 70 . Por outro lado, sabemos que o tringulo UT V
issceles, portanto, os ngulos em U e em V so iguais. Usando que a soma dos ngulos
b = 180 70 = 110 e, portanto,
internos de um tringulo 180 , vemos que 2T UV
b = 55 . Como os ngulos T UV
b e QU
b V so complementares, resulta que
T UV
222

b V = 180 55 = 125 .
QU
OBMEP 2010

Solues do Nvel 2
141. Uma loja de brinquedos Se x denota o desconto em reais e y o nmero total de
peas, ento (13 x) y = 781. Assim, (13 x) e y so divisores de 781 e, como
781 = 11 71, os nicos divisores de 781 so 1, 11, 71 e 781. O divisor 13 x de 781
no pode ser igual a 1, pois sabemos que y 100. A nica opo, ento, 13 x = 11
e y = 71, de modo que a reduo foi de x = R$ 2,00 por unidade.
142. Frao de frao Temos
1

1+
1+

=1+

1+

1
1+
2

=1+

1
3
2

1+

2
3

=1+

1
3
8
=1+ = .
5
5
5
3

143. Potncias de 3 Temos 272 a = (33 )2 a = 36 a = (3a )6 = 26 = 64.


144. Aumento de preo Em reais, o aumento foi de 5,55 5 = 0,55 e, portanto, o
percentual de aumento foi de
0,55 20
0,55
11
=
=
= 11%.
5
5 20
100
145. Roseiras em fila possvel plantar as roseiras em 6 filas de 5 roseiras cada uma,
conforme mostra a figura.
s

.....
.. ....
.. ...
.. ....
.. ....
...
.
...
.......
.. .... ......
....
.. ..
... ..
....
....
... ...
....
.. ..
....
....
....
....
....
.
....
......
....
... ..
.......
... ....
.... .......
.
. ..
.
.
.
.
....
...
..... ..
.
.
....
.
.
.
.. .
..
....
... ........................... .........
....
.... ..
..
...
....
.. ...............................
..
....
.
.............
.. ....
....
..
........
.............
.
....
.............
...
.
.
....
.
.
.............
.. .
............. .......
... ......... ....
............. ...
. ....
.
.
..............
..
.........
....................
.
.
..
....
.... ...............
.
.
.
.
...
............
....
......
.
.
.
.............
.
....
..
... ...
.
.
.......
.
.
.
.
..
.
.............................................
... ...
.
.
.
.
.
.
.
.
.
.
.
.
.
.
.
.
.
.
.
.
.
.
.
.
.
.
.
.
.
.
.
.
.
.
.
.
.
.
.
.
.
.
.
.
.
.
.
...
.
............................
.
.
.
.
...
.
.
.
.
.
.
.
.
.
.
.
.
.
.
.
.
.
.
.
.
.
.
.
.
.
.
.
.
.
.
.
.
.
.
.
.
.
.
.
.
.
.
.
.
.
.
.
... ...........................................................................
.
.
.
.
..................

s s
s
s

146. Calculadora diferente Para calcular (2 + 3) (0 + 3) utilizaremos a propriedade


(iii), obtendo (2+3)(0+3) = (20)+(33). Agora, por (ii), temos 20 = 22 = 4
e, por (i), temos 3 3 = 3. Portanto, (2 + 3) (0 + 3) = 4 + 3 = 7 .
Para calcular 1 024 48 vamos usar a mesma estratgia, observando que
1 024 = 976 + 48 e 48 = 0 + 48. Assim,
1 024 48 = (976 + 48) (0 + 48)
= (976 0) + (48 48)
= 2 976 + 48 = 1 952 + 48 = 2 000.
OBMEP 2010

223

Solues do Nvel 2
147. Dois quadrados
Soluo 1: A regio tracejada um trapzio de bases medindo 10 e 4 cm. A altura do
trapzio, que a metade da diferena dos lados dos dois quadrados, mede 21 (104) = 3
cm. Assim, a rea procurada mede
1
3 (10 + 4) = 3 7 = 21 cm2 .
2
Soluo 2: A rea do quadrado maior menos a rea do quadrado menor igual a 4
vezes a rea procurada. Assim, a rea procurada mede
102 42
100 16
=
= 25 4 = 21 cm2 .
4
4
FE
FU
=
. Analogamente, sendo
148. Paralelismo Sendo IL e EU paralelos, temos
FL
FI
FN
FI
RE e NI paralelos, temos
=
. Assim,
FR
FE
FE
FN FU
FI
=

= 1.
FR FL
FE
FI
149. Um subconjunto Vamos construir um subconjunto de {1, 2, 3, . . . , 3 000} em que
nenhum elemento seja o dobro do outro. Comeamos incluindo todos os nmeros
mpares, 1, 3, 5, . . . , 2 999. Assim, j temos 1 500 nmeros e nenhum o dobro de algum
outro. Agora podemos acrescentar os nmeros que so o qudruplo de algum nmero
mpar, isto , acrescentar
3} , 4| {z
4| {z
1} , 4| {z
5} , . . . , 4| {z749} .
4

12

20

2 996

Essa lista tem 749 nmeros e nenhum deles o dobro do outro; alm disso, nenhum
deles o dobro de um nmero mpar.
Logo, nosso conjunto j possui 1 500 + 749 = 2 249 elementos. Basta tomar qualquer
subconjunto com 2 000 elementos para obter um subconjunto de {1, 2, 3, . . . , 3 000} em
que nenhum elemento o dobro do outro.
150. Tringulos retngulos Observemos que os quatro tringulos que aparecem na
figura so tringulos retngulos, dois a dois semelhantes, portanto, seus lados so
proporcionais. Em particular, temos 9/x = y/20, ou seja, 180 = xy. Alm disso, pelo
Teorema de Pitgoras, temos y 2 = x2 + 92 , de modo que
1802 = x2 y 2 = x2 (x2 + 92 ) = x4 + 92 x2 ,
isto , x4 + 92 x2 1802 = 0. Pela frmula de Bhaskara, obtemos

81 94 + 4 1802
9 92 + 4 202
9 41
2
x =
=9
=9
.
2
2
2
Mas x2 > 0, portanto necessariamente x2 = 9 16 e, portanto, como x > 0, a nica
opo x = 12.
224

OBMEP 2010

Solues do Nvel 2
A partir
de x = 12, obtemos todas as outras medidas.

2
y = x + 92 = 15 e, pelo Teorema de Pitgoras, obtemos

z = 202 x2 = 16.

Pelo visto, temos

Usando a proporcionalidade v/8 = 9/x, resulta


v = 72/x = 6 e, finalmente, pelo

Teorema de Pitgoras, conclumos que w = 82 + v 2 = 10.


151. Uma desigualdade especial A opo correta (c).
Observemos que se x satisfaz a desigualdade, ento x tambm satisfaz a desigualdade.
Assim, os valores que satisfazem a desigualdade formam um conjunto simtrico em
relao origem e, portanto, basta verificar quais x 0 satisfazem x2 < x + 2, ou seja,
(x 2)(x + 1) = x2 x 2 < 0.

Como x + 1 > 0 para x 0, devemos ter x 2 < 0, ou seja, x < 2. Pela simetria
observada, conclumos que 2 < x < 2 a soluo da desigualdade original.
152. Sapo Cururu A cada x saltos do tipo I, o sapo se desloca 10x cm para o Leste e
30x cm para o Norte e, a cada y saltos do tipo II, o sapo se desloca 20y cm para o
Oeste e 40y cm para o Sul. Assim, ao final de x saltos do tipo I e y do tipo II, o sapo
se deslocou 10x 20y cm para o Leste e 30x 40y cm para o Norte.
(a) Resolvendo

10x 20y = 190


30x 40y = 950

encontramos x = 57 e y = 19. Logo, o sapo dever dar 57 saltos do tipo I e 19


do tipo II, em qualquer ordem, para alcanar um ponto situado a 190 cm para o
Leste e 950 cm para o Norte de sua casa.
(b) Uma vez que o nmero de saltos de cada tipo um nmero inteiro, Cururu s
alcanar o ponto situado a 180 cm para o Leste e 950 cm para o Norte de sua
casa se o sistema

10x 20y = 180
30x 40y = 950

tiver soluo inteira. Mas a soluo desse sistema x = 59 e y = 41/2, que no


inteiro. Portanto, Cururu no conseguir alcanar aquele ponto.

153. Distribuindo algarismos em linhas De acordo com o padro da sequncia, temos


1a linha
2a linha
3a linha
..
.
Logo,

0
1 1 0
2 2 2 1 1 0

10a linha 9 9 9 9 9 9 . . . 9 8 . . . 1 1 0
um algarismo 0 em cada linha d 1 10 = 10 algarismos 0 no total;
dois algarismos 1 em nove linhas d 2 9 = 18 algarismos 1 no total;
trs algarismos 2 em oito linhas d 3 8 = 24 algarismos 2 no total;
quatro algarismos 3 em sete linhas d 4 7 = 28 algarismos 3 no total,
OBMEP 2010

225

Solues do Nvel 2
e assim por diante. Portanto, trata-se de descobrir qual o maior dos produtos a
seguir, onde cada um representa quantos algarismos, de 0 a 9, aparecem na sequncia.
1|
9} , 3| {z
8} , 4| {z
7} , 5| {z
6} , 6| {z
5} , 7| {z
4} , 8| {z
3} , 9| {z
2} , 10
1}
{z10} , |2 {z
| {z
0

Como o maior produto 30, os algarismos mais usados foram 4 e 5, 30 vezes cada um.
154. Ser que existe?
Soluo 1: Se existir esse nmero N, ento
222 . . . 2
111 . . . 1
2 111 . . . 1
=
=
.
2 008
2 1 004
1 004
Logo, N no inteiro, por ser o quociente do nmero mpar 111 . . . 1 pelo nmero par
1 004. Portanto, no existe tal N.
N=

Soluo 2: Fatorando 2 008, obtemos 2 008 = 23 251, portanto, 2 008 divisvel


por 8. Se existisse um inteiro N tal que 2 008 N = 222 . . . 2, teramos, ento, que
8 dividiria 222 . . . 2. Por outro lado, sabemos que um nmero divisvel por 8 se, e
somente se, o nmero formado pelos ltimos trs algarismos for divisvel por 8. Mas
222 = 27 8 + 6 no divisvel por 8. Logo, no existe um nmero N tal que
2 008 N = 222 . . . 2.
155. Conferindo uma desigualdade
Soluo 1: Uma maneira de verificar essa desigualdade comparando cada parcela
1
1 1 1
desta soma, como segue. Comparando as fraes , e com , obtemos
5 6 3
4




3
3
1
1
1
1
1
1
< , portanto, 3 =
<
= 3;
5
4
5
5
4
4




1
1
1 3
1 3
1
1
< , portanto, 3 =
<
= 3;
6
4
6
6
4
4




1
1
1
1
1 3
1 3
< , portanto, 3 =
<
= 3.
4
3
4
4
3
3
Assim,
1
1
1
1
1
1
3
3 1 1
3 1 1
1
+ 3+ 3 < 3+ 3+ 3 = 3 = < = .
3
4
5
6
4
4
4
4
4 4 4
4 3 3
12
Soluo 2: Uma outra maneira de verificar essa desigualdade testar se
1
1
1
1
+ 3+ 3
> 0.
3
4
5
6
12
Reduzindo ao mesmo denominador, temos
1
1
1
1
24 32 53 33 53 26 33 23 53
+
+

=
26 53 23 33 22 3
26 33 53
18 000 3 375 1 728 1 000
=
26 33 53
11 897
= 6
.
2 33 53
226

OBMEP 2010

Solues do Nvel 2
Nem preciso efetuar o produto indicado no denominador. Como o numerador
positivo, conclumos que a desigualdade se verifica.
156. Parte inteira
(a) Os nmeros 9 e 16 so quadrados perfeitos
e 9 < 12 < 16. Ento,

3 = 9 < 12 < 16 = 4
 
e, portanto,
12 = 3.

(b) Como 12 777 2 < 28 756 < 12 777 3,


temos


28 756
28 756
= 2.
2<
< 3, portanto,
12 777
12 777

007
(c) Como 2 007 < 2 008, temos 0 < 22 008
< 1,
ou


2 007
2 007
1 <
< 0, portanto,
= 1.
2 008
2 008

12

28 756
12 777

2 007
2 008

(d) Como 43 = 64 < 111 < 125 = 53 , temos


3

(5) = 5 = 125 < 111 < 4 = (4) ,


de modo que 5 <

111 < 4 e, portanto,


3

111


111 = 5.

157. Soma nove


Soluo 1: Vamos dividir em dois casos: nmeros de dois algarismos e nmeros de
trs algarismos. No caso de nmeros de dois algarismos, temos 18, 27, 36, 45, 54, 63,
72, 81 e 90, num total de 9 nmeros. Da mesma maneira, listamos os nmeros de trs
algarismos, como segue:
108, 117, 126, 135, 144, 153, 162, 171, 180
207, 216, 225, 234, 243, 252, 261, 270
306, 315, 324, 333, 342, 351, 360
405, 414, 423, 432, 441, 450
504, 513, 522, 531, 540
603, 612, 621, 630
702, 711, 720
801, 810
900

9 nmeros;
8 nmeros;
7 nmeros;
6 nmeros;
5 nmeros;
4 nmeros;
3 nmeros;
2 nmeros;
1 nmero.

Portanto, temos 9 + 8 + 7 + 6 + 5 + 4 + 3 + 2 + 1 = 45 nmeros de trs algarismos.


Assim, existem 9 + 45 = 54 nmeros entre 10 e 999 cuja soma dos algarismos igual
a 9.
OBMEP 2010

227

Solues do Nvel 2
Soluo 2: No caso de nmeros de dois algarismos, uma vez escolhido o algarismo da
dezena, o algarismo da unidade fica automaticamente definido. Como o algarismo da
dezena pode ser 1, 2, 3, 4, 5, 6, 7, 8 ou 9, temos nove nmeros de dois algarismos tais
que a soma de seus algarismos seja 9.
No caso de nmeros de trs algarismos, denotando por n o algarismo da centena, a soma
dos algarismos da dezena e da unidade 9 n, portanto, temos 9 n + 1 = 10 n
possibilidades de escolha para o algarismo da dezena, que pode ser inclusive 0, e o
algarismo da unidade fica automaticamente definido. Como o algarismo da centena
pode ser 1, 2, 3, 4, 5, 6, 7, 8 ou 9, temos
(101)+(102)+(103)+(104)+(105)+(106)+(107)+(108)+(109) = 45
nmeros de trs algarismos tais que a soma dos seus algarismos 9. Assim, existem
9 + 45 = 54 nmeros entre 10 e 999 cuja soma dos algarismos igual a 9.
158. Retngulos Se a e b denotam o comprimento e a largura do retngulo, temos
a b = 96. Logo, a e b so divisores pares de 96 e, portanto, temos quatro retngulos
satisfazendo as condies dadas, a saber, os retngulos de lados medindo 2 e 48; 4 e
24; 6 e 16 e 8 e 12.
159. Nmero de retas
Soluo 1: Para contar o nmero de retas, dividiremos as retas de acordo com suas
posies.
retas paralelas aos lados dos quadrados: trs horizontais e trs verticais;

retas paralelas s diagonais dos quadrados: 3 + 3 = 6;

outras retas: temos 4 2 = 8 retas, formando uma estrela, como na figura.

228

OBMEP 2010

Solues do Nvel 2
Assim, ao todo, temos 3 + 3 + 6 + 8 = 20 retas.
Soluo 2: Note que o ponto central suprfluo, pois toda reta que passa por ele
e um outro ponto do reticulado, passa tambm por um terceiro ponto do reticulado.
Logo, o ponto central pode ser eliminado de nossas consideraes. Assim, o problema
fica reduzido a calcular quantas retas so determinadas por dois pontos quaisquer dos
oito pontos do reticulado dado.

Com esses oito pontos, podem ser formados 8 7 possveis pares de pontos distintos
(8 possibilidades para o primeiro elemento do par e 7 possibilidades para o segundo).
Como a ordem em que o par formado no influi na reta determinada por ele, esse
nmero deve ser dividido por 2, ou seja,
87
= 28.
2
Finalmente, notamos que algumas retas esto sendo contadas trs vezes e, portanto,
devem ser subtradas duas vezes desse nmero 28. So elas as quatro retas que contm os lados do quadrado que delimita o reticulado. Logo, o nmero total de retas
determinadas pelo reticulado
87
2 4 = 20.
2
160. Cubo Um cubo tem seis faces distintas, duas a duas opostas, sendo que as faces
opostas no tm aresta em comum. Temos trs pares de faces opostas, logo trs cores
so suficientes, bastando pintar as faces opostas de uma mesma cor. Por outro lado,
claro que duas cores somente no bastam.
161. rea
Soluo 1: Observemos, primeiro, que a razo entre as reas de dois retngulos que
tm a mesma base igual razo entre suas alturas. De fato, na figura esquerda,
S1

h1

S2

h2

27

18

h1

72

h2

esto representados dois retngulos que tm a mesma base b e alturas h1 e h2 . Suas


reas S1 e S2 so dadas por S1 = b h1 e S2 = b h2 , portanto,
S1
bh1
h1
=
=
.
S2
bh2
h2
OBMEP 2010

229

Solues do Nvel 2
Aplicando essa observao aos dois pares de retngulos dados (ver figura anterior,
direita) e denotando por S a rea do quarto retngulo, temos
27
18
h1
1
=
=
= ,
S
h2
72
4
de modo que S = 27 4 = 108. Assim, a rea do lote que foi dividido mede 27 + 18 +
72 + 108 = 225 km2 .
Soluo 2: Sejam x, y, z e w as medidas dos retngulos menores, conforme a figura. A rea procurada

(x + w)(y + z) = xy + xz + wy + wz.

Basta determinar wy, pois


xy = 27, xz = 18 e wz = 72. Mas,

sabido

27

18

72

que

wz
72
w
=
=
=4
x
xz
18
e, portanto, w = 4x. Como xy = 27, segue que wy = 4x 27/x = 4 27 = 108. Assim,
a rea do lote que foi dividido mede
27 + 18 + 72 + 108 = 225 km2 .
162. Inteiro mais prximo
(a) Temos:

19 19
4
1
9
3
+
=1+
+6+ =7+
=7+ .
15
3
15
3
15
5
3
1
Logo, a soma dada est entre 7 e 8. Como > , o nmero inteiro mais prximo
5
2
8.
3
7+ 5

7+

1
2

(b) Temos:
85 43 29 15
+
+
+
=2+
42 21 14
7
1
1
=8+
+
+
42 21

1
1
1
1
+2+
+2+
+2+
42
21
14
7

1
1
11 1 1
2
+ =8+
+ + +1 =8+ .
14 7
7 6 3 2
7

Logo, a soma dada est entre 8 e 9. Como


8.
8 + 72

8
230

8+

OBMEP 2010

2
7

< 12 , o nmero inteiro mais prximo

9
1
2

Solues do Nvel 2
(c) Temos:

2
11 1 7 2
30 2
+ = + = 3 + .
10 2 5 3
10 3
3
2
1
Logo, a soma dada est entre 3 e 2. Como 3 > 2 , o nmero inteiro mais
prximo 2.
3 + 23

3 +

1
2

163. Brincando com nmeros mpares Como cada algarismo mpar, temos:
cinco possibilidades de nmeros com um algarismo: 1, 3, 5, 7 e 9;

para nmeros com dois algarismos, temos cinco possibilidades na casa das unidades e cinco na casa das dezenas, totalizando 5 5 = 25 possibilidades;

para nmeros com trs algarismos, temos cinco possibilidades na casa das unidades, cinco na casa das dezenas e cinco na casa das centenas, totalizando 555 =
125 possibilidades.
Assim, Beatriz pode escrever 5 + 25 + 125 = 155 nmeros com todos algarismos sendo
mpares.
164. gua no jarro Inicialmente, o volume de gua no jarro da Maria 1 l =
1 000 ml. Depois de 200 dias, o volume o mesmo, acrescido do que colocado
por Joo e diminudo do que ela tirou para colocar no do Joo, ou seja,
1 000 + 1 2 + 3 4 + + 199 200
= 1 000 + (1 2) + (3 4) + + (199 200)
= 1 000 (1| + {z
+ 1}) = 900.
100

Logo, depois de 200 dias, Maria ter 900 ml em seu jarro.

165. Formiga no cubo Na figura temos um caminho percorrendo oito arestas que a
formiga pode fazer partindo do vrtice identificado como 1.

Ser possvel ela fazer um caminho passando por nove arestas? Para fazer esse caminho,
ela teria que passar por nove vrtices, pois o vrtice de chegada o mesmo que o de
partida, j que a formiguinha volta ao vrtice inicial.
vrtice
de
chegada

vrtice
de
partida


1 2 3
4 5 6 7 8
9
OBMEP 2010

231

Solues do Nvel 2
Como o cubo s tem oito vrtices, esse passeio no possvel. Logo, o passeio de maior
comprimento percorre oito arestas.
166. Promoo Sejam b e c o nmero de blusas e calas compradas, respectivamente.
Logo, temos 15b + 17c = 143, sendo b e c nmeros inteiros positivos. Observe que
b < 10 e c < 9, pois tanto 15 10 quanto 17 9 so maiores do que 143. A partir
deste ponto, apresentamos duas possibilidades de soluo.
Soluo 1: Temos que 15b = 143 17c, portanto 143 17c um mltiplo de 15, de
modo que 143 17c termina em 0 ou 5. Isso significa que 17c termina em 3 ou 8. Logo,
c = 9 ou c = 4. Como c < 9, a nica soluo c = 4 e, portanto,
b=

143 17 4
= 5.
15

Assim, Joana comprou cinco blusas e quatro calas.


Soluo 2: Temos que
b=

143 17c
8 2c
= 9c+
.
15
15

Note que c um nmero inteiro positivo, portanto, 8 2c precisa ser um mltiplo de


15. Se 8 2c 15, c resulta negativo, portanto, 8 2c = 0, ou seja, c = 4. Da obtemos
que b = 5. Assim, Joana comprou cinco blusas e quatro calas.
167. Soma de cubos Temos a identidade do binmio, (x + y)2 = x2 + 2xy + y 2 , e a do
trinmio, (x + y)3 = x3 + 3x2 y + 3xy 2 + y 3 . Substituindo os valores de x + y e x2 + y 2
na identidade do binmio, obtemos 1 = 2 + 2xy e, portanto, xy = 21 . Assim, pela
identidade do trinmio,
x3 + y 3 = (x + y)3 3xy(x + y) = 1 3

168. O revezamento em uma corrida Como velocidade =

1
2

1 =

5
.
2

distncia
, ou seja, tempo =
tempo

distncia
, o tempo gasto por Joo foi de
velocidade

21
9
9
t=
h= 1+
h = 1h+
60 min = 1h45min.
12
12
12

Logo, Carlos precisa completar a prova em um tempo inferior a

(2h48min) (1h45min) = 1h3min = 63min.


Para isso, sua velocidade v, em km/min, deve satisfazer
v>

21
= t < 63, ou seja,
v

21
1
60
= km/min =
km/h = 20 km/h.
63
3
3

Logo, Carlos deve correr com uma velocidade superior a 20 km/h.


232

OBMEP 2010

Solues do Nvel 2
169. Produtos consecutivos
Soluo 1: Como os produtos so nmeros consecutivos, podemos denot-los por p e
p + 1. Temos, ento,
p2 + p = p(p + 1) = 2 3 5 7 11 13 17 = 510 510.
Resolvendo a equao p2 + p 510 510 = 0, encontramos uma nica raiz positiva,
p = 714. Assim, p + 1 = 715 e, fatorando, obtemos
714 = 2 3 7 17 e 715 = 5 11 13.
Soluo 2: Os nmeros dados so 2, 3, 5, 7, 11, 13 e 17. Se 2 e 5 estiverem no mesmo
grupo, ento um dos produtos termina em 0 e o outro, por ser consecutivo, termina
em 1 ou 9. Os possveis produtos terminados em 1 so 3 7 = 21, 3 17 = 51,
7 13 = 91, 13 17 = 221, 3 7 11 = 231, 3 11 17 = 561 e 7 11 13 = 1 001.
Verifica-se que esses grupos no constituem soluo e, analogamente, os terminados em
9. Conclumos que 2 e 5 esto em grupos diferentes. Logo, um produto termina em 5
e o outro em 4 ou 6. Como no possvel formar com os nmeros dados um produto
terminado em 6, necessariamente um dos produtos termina em 4 e o outro em 5. Por
tentativas, obtemos a soluo
714 = 2 3 7 17 e 715 = 5 11 13.
170. Distraindo na fila Observe que aquela que gritou os nmeros 9, 18, etc, sempre
gritou mltiplos de 9. O primeiro mltiplo de 3 com quatro algarismos 1 002 e o
primeiro mltiplo de 3 maior do que 2 003 2 004. Logo, Vivi gritou 2 004 e Rosa
1 002. Nenhum desses nmeros mltiplo de 9, portanto, foi Tnia quem gritou 9 e
seus mltiplos.
Quem gritou 3, tambm gritou 12 = 3 + 9, 21 = 3 + 2 9, 30 = 3 + 3 9 e assim por
diante, at 3 + 9k. Da mesma forma, quem grita 6, grita todos os nmeros da forma
6 + 9k. Dividindo por 9, obtemos 2 004 = 6 + 9 222 e 1 002 = 3 + 9 111, portanto,
quem gritou 3 foi Rosa e Vivi gritou 6.
Rosa
3
12
21
..
.

Vivi
6
15
24
..
.

Tnia
9
18
27
..
.

1 002
..
.

1 005
..
.

1 008
..
.

2 001

2 004

2 007

Da mesma forma, dividindo por 9, encontramos que 666 mltiplo de 9 e


888 = 6 + 98 9, portanto Tnia gritou 666 e Vivi gritou 888.
171. Nmero e o dobro Inicialmente note que o dobro de um nmero inteiro par, logo
ele termina em 0, 2, 4, 6 ou 8. No entanto, o nmero procurado no pode terminar em
0, pois nesse caso o seu dobro tambm terminaria em 0, e ambos teriam o algarismo 0
em comum. Portanto, temos os casos a seguir.
OBMEP 2010

233

Solues do Nvel 2
I
1 ... 2
2
............................................................................
3 ... 4

II
1 ... 3
2
............................................................................
2 ... 6

III
1 ... 4
2
............................................................................
(
2 ... 8
ou
3

IV
1 ... 5
2
............................................................................
(
2 ... 0
ou
3

V
1 ... 6
2
............................................................................
3 ... 2

VI
1 ... 7
2
............................................................................
(
2 ... 4
ou
3

VII
1 ... 8
2
............................................................................
(
2 ... 6
ou
3

VIII
1 ... 9
2
............................................................................
2 ... 8
ou

Vamos, agora, determinar todas as possibilidades para cada caso, lembrando sempre
que o nmero e seu dobro no podem ter algarismos comuns.
Caso I temos trs possibilidades:
152 2 = 304 ,

182 2 = 364 ,

192 2 = 384 .

Caso II temos duas possibilidades:


143 2 = 286 ,

153 2 = 206 .

Caso III temos trs possibilidades:


134 2 = 268 ,

154 2 = 308 ,

164 2 = 328 .

Caso IV temos trs possibilidades:


135 2 = 270 ,

145 2 = 290 ,

185 2 = 370 .

Caso V temos duas possibilidades:


176 2 = 352 ,

186 2 = 372 .

Caso VI no h nenhuma possibilidade.


Caso VII temos trs possibilidades:
138 2 = 276 ,

148 2 = 296 ,

178 2 = 356 .

Caso VIII temos duas possibilidades:


139 2 = 278 ,

179 2 = 358 .

Assim, temos 3 + 2 + 3 + 2 + 3 + 3 + 2 = 18 solues para esse problema, a saber:


134, 135, 138, 139, 143, 145, 148, 152, 153,
154, 164, 176, 178, 179, 182, 185, 186, 192.

234

OBMEP 2010

Solues do Nvel 2
172. Invertendo os algarismos Devemos contar os nmeros a b de dois algarismos que
tm o algarismo b da unidade maior do que o algarismo a da dezena, ou seja, tais que
b > a. Se a = 1, o algarismo b da unidade pode ser 2, 3, 4, 5, 6, 7, 8 ou 9, portanto, temos
oito possibilidades. Se a = 2, o algarismo b da unidade pode ser 3, 4, 5, 6, 7, 8 ou 9,
portanto, temos sete possibilidades. Continuando dessa maneira, chegamos at a = 8,
quando o algarismo b da unidade s pode ser 9, portanto, temos s uma possibilidade.
Claramente, a no pode ser 0 nem 9. Logo, existem 8 + 7 + 6 + 5 + 4 + 3 + 2 + 1 = 36
nmeros entre 10 e 99 tais que, invertendo a ordem de seus algarismos, obtemos um
nmero maior do que o nmero original.

173. Razo entre segmentos Se o arco P R o dobro do arco RQ, vale a mesma relao
b = 2 ROQ.
b Como P OR
b + ROQ
b = 180 , segue
entre os ngulos centrais, ou seja, P OR
que
b + ROQ
b = 3 ROQ,
b
180 = 2 ROQ
b = 60 . Mas, OR = OQ o raio do crculo, de modo que o trindonde ROQ
gulo ORQ equiltero. Assim, sua altura RM tambm a mediana, ou seja,
1
OM = MQ. Se r o raio do crculo, ento OM = MQ = r e
2
r + 21 r
PM
P O + OM
=
= 1
= 3,
MQ
MQ
r
2
ou seja, a razo entre P M e MQ 3.
174. Tringulos Vamos supor que a, b e c sejam os comprimentos dos lados do tringulo.
No h perda de generalidade em supor que a b c, de modo que a + b + c 3c.
Como cada lado de um tringulo menor do que a soma dos outros dois, temos que
c < a + b e, portanto, obtemos 2c < a + b + c 3c. Mas, a + b + c = 15, de modo que
2c < 15 3c e, como c um nmero inteiro, 5, 6 ou 7 so as nicas opes para c.
Se c = 5, ento a + b = 10 e temos uma nica possibilidade, a = b = c = 5. Se
c = 6, ento a + b = 9 e temos duas possibilidades para a, 3 ou 4, caso em que,
necessariamente, b 6 ou 5, respectivamente. Se c = 7, ento a + b = 8 e temos quatro
possibilidades para a, 1, 2, 3 ou 4. Nesses casos, necessariamente, b 7, 6, 5 ou 4,
respectivamente. Assim, no total, temos sete desses tringulos.

175. Nmero interessante Suponhamos que N seja um dos nmeros procurados. Como
N e 119 deixam os mesmos restos quando divididos por 2, 3, 4, 5 e 6, temos que
a diferena N 119 entre eles deixa resto zero quando dividido por esses nmeros.
Portanto, N 119 um mltiplo de 2, 3, 4, 5 e 6. Como 60 o mnimo mltiplo
comum desses nmeros, N 119 um mltiplo de 60, ou seja, N 119 = 60k, para
algum inteiro k. Assim, N = 119+60k um nmero interessante para qualquer nmero
inteiro k. Como queremos nmeros distintos de 119 e de trs algarismos, devemos tomar
k positivo e menor do que 15, j que 119+6015 = 1 019 tem quatro algarismos. Assim,
tomamos k de 1 a 14 e obtemos outros 14 nmeros interessantes de trs algarismos, a
saber,
179, 239, 299, 359, 419, 479, 539, 599, 659, 719, 779, 839, 899, 959.

OBMEP 2010

235

Solues do Nvel 2
60
176. Time vencedor O time ganhou 60% das 45 j disputadas, ou seja, 45
= 27
100
partidas. Se ele ganhar mais n partidas, a porcentagem de partidas ganhas ser
75
no de partidas ganhas
27 + n
3
=
= 75% =
= .
o
n de partidas disputadas
45 + n
100
4
Logo, 4 (27 + n) = 3 (45 + n), do que resulta n = 27 como o nmero mnimo de
partidas que o time ainda precisa vencer para atingir 75% de vitrias.
177. Brincando com dados Na tabela seguinte, marcamos com os produtos que so
divisveis por 6.

1
2
3
4
5
6

Logo, temos 15 casos favorveis dentre 36 possibilidades. Assim, a probabilidade de


que o produto seja divisvel por 6 15/36 = 5/12 = 41,7%.
178. Contando solues A equao dada equivalente a xy = 144(x+y) = 144x+144y,
144y
portanto, isolando x, obtemos x =
. Como x e y devem ser inteiros positivos,
y 144
o denominador y 144 deve ser um nmero inteiro positivo, digamos, y 144 = n.
Substituindo essa expresso no valor de x, obtemos
x=

144(n + 144)
1442
= 144 +
.
n
n

Como x deve ser um nmero inteiro, n deve ser um divisor de 1442 . Sendo
1442 = 124 = 28 34 , seus divisores so os nmeros d da forma d = 2a 3b , com
0 a 8 e 0 b 4. Como h 9 valores possveis para a e 5 valores possveis para
b, conclumos que 1442 tem 9 5 = 45 divisores.
Assim, para cada divisor n de 1442 , obtemos uma soluo


1442
(x, y) = 144 +
, n + 144
n
xy
da equao
= 144 dada. Portanto, essa equao possui 45 pares de nmeros
x+y
inteiros positivos (x, y) que a satisfazem.

179. Crculos tangentes Denotemos por r1 , r2 e r3 os raios dos trs crculos. Como os
crculos so tangentes dois a dois, temos

r1 + r2 = 3,
r1 + r3 = 4,

r2 + r3 = 5.

236

OBMEP 2010

Solues do Nvel 2
Substituindo os valores r2 = 3 r1 e r3 = 4 r1 na terceira equao, obtemos
3 r1 + 4 r1 = 5. Da, r1 = 1, r2 = 2 e r3 = 3. Logo, a soma das reas dos
trs crculos (12 + 22 + 32 ) = 14 cm2 .

r3
5 cm

r2

3 cm
4 cm
r1

180. Grupo de amigos Se A a quantidade de dinheiro que Joo recebeu de cada um


de seus amigos, ento ele recebeu um total de 3A. Como ele recebeu, de Jorge, um
quinto do seu dinheiro, ento Jorge tinha 5A. Da mesma maneira, Jos tinha 4A e
Jnio tinha 3A. Assim, os trs amigos tinham, juntos, 5A + 4A + 3A = 12A e a frao
do dinheiro do grupo que ficou com Joo foi de (3A)/(12A)=1/4, ou seja, uma quarta
parte.
181. Um trapzio Os tringulos AP B e CP D so semelhantes, pois o ngulo APbB
b igual ao ngulo B DC
b
igual ao ngulo C PbD (opostos pelo vrtice) e o ngulo ABD
(alternos internos).
A

B
P

Como a razo
p entre suas reas 4/9, temos que a razo de semelhana entre esses
tringulos 4/9 = 2/3 . Logo,
PB
2
= .
DP
3
Por outro lado, os tringulos CP D e P CB tm a mesma altura em relao s
bases DP e P B, respectivamente. Portanto, a razo entre suas reas igual razo
entre suas bases,
PB
rea (P CB)
2
=
= .
rea (CP D)
DP
3
Como rea (CP D) = 9, segue que a rea do tringulo P CB mede 6 cm2 .
OBMEP 2010

237

Solues do Nvel 2
Observao: O mesmo argumento poderia ser usado para mostrar que tambm a rea
do tringulo ADP mede 6 cm2 . As medidas de 4 e 9 cm2 de rea dadas no enunciado
do problema no desempenham papel especial algum. O argumento exposto acima
prova que num trapzio qualquer os tringulos P CB e ADP tm reas iguais,
mesmo que o trapzio no seja equiltero.
40
182. Vista ruim Seja A o nmero total de alunos da sala. Sabemos que
A no
100
70
40
enxergam bem, portanto,

A usam culos. Assim,


100 100
70
21 100
40

A = 21 ou seja, A =
= 3 25 = 75 .
100 100
74
183. Idade mdia da populao de Campo Verde Se H indica o nmero de homens
e M o de mulheres, ento H/M = 2/3, de modo que M = (3H)/2 e, portanto, a
populao de Campo Verde dada por
H + M = H + 32 H = 52 H .
Se a idade mdia dos homens 37 anos, ento
37 = idade mdia dos H homens =

soma das idades de todos homens


,
H

de modo que 37 H a soma das idades de todos os homens. Da mesma forma, 42 M


a soma das idades de todas as mulheres. Segue que a soma das idades de toda a
populao dada por
37H + 42M = 37H + 42 32 H = 100H.
Assim, a idade mdia da populao de Campo Verde
37H + 42M
100H
100 2
= 5H =
= 40 anos.
H +M
5
2
184. rea de tringulo Os tringulos ABC e CBD tm bases AC e CD, respectivamente, e a mesma altura h em relao a essas bases.
B

.........
.............. .....
.......... .... .. ..
........... ..... .... .....
........... ..........
.
.
.
... ....
.
.
.
.
.
.
.
.
...
........
..
......
..........
...
.....
...........
...
.....
.....
...........
.
.
.
.
...
.
.
.
.
.
.
.
...
.
.
.
...
...
........
.
.
.
.
.
.
.
.
.
.
.
.
.
.
...
...
...
.......
.
.
.
.
.
.
.
.
.
...
.
.
.
.
.
...
......
.
.
.
...
.
.
.
.
.
.
.
.
.
.
.
.
.
.
...
...
...
........
.
.
.
.
.
.
.
.
.
.
.
.
.
...
...
.......
..
.
.
.
.
.
.
.
.
.
...
.
.
.
.
.
.
..
..
.......
.
...
.
.
.
.
.
.
.
.
.
.
.
.
.
.
..
...
...
........
.
.
.
.
.
.
.
.
.
.
.
.
.
..
.
.
.
.
....
.......................................................................................................................................................................................................................................................................................

Assim, temos
rea ABC =
238

AC h
2

e rea CBD =

OBMEP 2010

CD h
.
2

Solues do Nvel 2
Logo, a relao entre as reas dada por
rea ABC
=
rea CBD

ACh
2
CDh
2

AC
1,5
15
3
=
=
= .
CD
4 1,5
25
5

Lembrete: A rea de um tringulo a metade do produto de um dos seus lados pela


altura h relativa a este lado, como exemplificado nas duas figuras a seguir.
B

..
........
..... ....
...
.....
...
.....
.
.
.
.
...
.....
...
.....
.
...
.
.
.
...
....
.
.
.
.
...
...
.
.
.
...
.
....
...
.
.
.
.
...
....
.
.
.
.
...
...
.
.
...
.
.
...
....
.
.
.
...
...
.
.
.
.
...
...
.
...
.
.
.
..
..............................................................................................................................................................................

.
............
...............
........... .....
........... ........
.
.
.
.
.
.
.
.
.
....
...
...........
.....
..........
.....
..........
.....
...........
.....
..........
.
.
.
.
.
.
.
.
.
.
.
.
.
.
..
....
...........
.....
...........
.....
...........
.....
...........
.....
...........
.
.
.
.
.
.
.
.
.
.
.
.
.
....
...........
.....
..........
.....
...........
.....
...........
.................................................................................................................

CD h
rea do CBD =
2

C
rea do ABC =

AC h
2

185. Construindo quadrados perfeitos Sim, ser sempre um quadrado perfeito. De


fato, se n 1, n, n + 1 e n + 2 so quatro inteiros consecutivos, ento seu produto mais
1 um quadrado perfeito, como segue.
(n 1)n(n + 1)(n + 2) + 1 = n(n2 1)(n + 2) + 1
= n(n3 + 2n2 n 2) + 1
= n4 + 2n3 n2 2n + 1
= n4 + 2n3 + (n2 2n2 ) 2n + 1
= (n4 + 2n3 + n2 ) 2n2 2n + 1
= (n2 + n)2 2(n2 + n) + 1

2
= (n2 + n) 1 .
186. Feira de Cincias Sejam x e y o nmero de alunos do ensino fundamental e mdio,
respectivamente, presentes na feira. Sabemos que o nmero daqueles que compraram
um adesivo x/2 do ensino fundamental e y/4 do ensino mdio, portanto, o nmero
daqueles que no compraram um adesivo x/2 do ensino fundamental e 3y/4 do ensino
mdio. Dentre os alunos que no compraram adesivos, os do ensino mdio representam
o dobro dos do ensino fundamental. Logo,
x
3y
=2 ,
4
2

ou seja, x =

3y
4

x
3y
=
.
2
8

Sabendo que o total arrecadado foi de R$ 38,00, estabelecemos que


38 = 0,30

x
y
3y
y
1,90
+ 0,50 = 0,30
+ 0,50 =
y,
2
4
8
4
8

de modo que y = 160 e, como x = 3y/4, segue que x = 120.


OBMEP 2010

239

Solues do Nvel 2
187. Par perfeito Denotemos por n o nmero natural candidato a formar um par
perfeito com 122. Ento devemos ter 122 + n = A2 e 122 n = B 2 , onde A e B
so nmeros naturais. Como B 2 = 2 61 n, os fatores primos 2 e 61 de B 2 devem
aparecer um nmero par de vezes, o que garante que n tem os fatores primos 2 e 61,
ou seja, n = 2 61 m2 = 122 m2, para algum natural positivo m. Decorre disso que
A2 = 122 + 122 m2 = 122(1 + m2 ).
O menor valor de (1 + m2 ) que satisfaz essa igualdade 1 + m2 = 122, ou seja,
m2 = 121 e m = 11. Consequentemente, n = 122 121 e conclumos que
A2 = 122 + 122 121 = 1222 e B 2 = 122 122 121 = (122 11)2 . Assim, 122
e 122 121 formam um par perfeito.

Observao: Na verdade, 122 121 o menor natural que forma um par perfeito com
122. Ser que existem outros?
188. Um trapzio A resposta correta (d).
Seja P o ponto mdio do segmento DC e tracemos os
segmentos AP e BP . Os trs tringulos assim formados,
ADP, AP B e BP C, so equilteros (porqu?), de
b = 60 = P AB.
b Como o segmento AC
modo que D AP
b (porqu?), conclumos que
a bissetriz do ngulo P AB

b = 30 . Assim,
P AC

.................................................................................
..
......
... ....
... ...
...
...
... .....
...
..
..
...
.
...
..
.
.
.
.
.
.
...
...
..
..
.
.
...
.
.
.
...
.
...
.
...
.
.
.
...
...
.
..
.
.
.
...
...
..
..
...
.
.
.
... ..
...
..
.
.
... ..
...
..
.
.
.
....................................................................................................................................................................

b = D AP
b + P AC
b = 60 + 30 = 90 .
D AC

189. Mistrio das bolas Seja m o nmero de bolas pretas na primeira urna e n o de bolas
brancas na segunda urna. Inicialmente, Henrique retirou k bolas pretas da primeira
urna e as colocou na segunda urna. Nesse ponto, a situao a seguinte:
na 1a urna temos m
k} e
| {z
pretas

na 2a urna temos |{z}


n + |{z}
k .
brancas

pretas

Depois, ele retirou k bolas da segunda urna e as colocou na primeira urna. Agora, esse
grupo de k bolas pode ter bolas brancas e pretas. Denotemos por p o nmero de bolas
pretas e por b o de bolas brancas retiradas da 2a urna. Ento k = b + p e
na 1a urna temos m
k} + p + |{z}
b = m k + p + |{z}
b ,
| {z
|{z}
| {z }
pretas

pretas

brancas

pretas

brancas

na 2a urna temos |{z}


n + |{z}
k |{z}
b p =n
b +k p .
|{z} | {z } | {z }
brancas

pretas

brancas

pretas

brancas

pretas

Assim, ele ficou com b bolas brancas na primeira urna e k p bolas pretas na segunda
urna. No entanto, k = p+b, ou b = k p. Logo, o nmero de bolas brancas na primeira
urna igual ao nmero de bolas pretas na segunda urna.
240

OBMEP 2010

Solues do Nvel 2
190. Contando a palavra BRASIL Para ler a palavra BRASIL, devemos percorrer
um caminho que comece numa letra B e termine na letra L. Observemos que o caminho a ser percorrido composto sucessivamente de deslocamentos horizontais para a
direita e verticais para baixo. Representemos esses caminhos por sequncias de letras
H (significando deslocamento horizontal para a direita) e V (significando deslocamento
vertical para baixo). Vejamos dois exemplos.
(i) Comeamos em B na segunda linha (de cima para baixo) e seguimos o caminho
VHVVV.
(ii) Comeamos em B na quarta linha e seguimos o caminho HVVHH.
Para resolver o problema devemos contar quantos caminhos comeam com B e terminam com L. Para istso, vamos listar esses caminhos, escrevendo Cj para o nmero de
caminhos que comeam com o B da linha j, em que j varia de 1 a 6, como segue.
(1) Primeira linha: VVVVV C1 = 1;

(2) segunda: HVVVV, VHVVV, VVHVV, VVVHV, VVVVH C2 = 5;

(3) terceira: HHVVV, HVHVV, HVVHV, HVVVH, VHHVV, VHVHV,


VHVVH, VVHHV, VVHVH, VVVHH C3 = 10;
(4) quarta: HHHVV, HHVHV, HHVVH, HVHHV, HVHVH, HVVHH,
VHHHV, VHHVH, VHVHH, VVHHH C4 = 10;

(5) quinta: HHHHV, HHHVH, HHVHH, HVHHH, VHHHH C5 = 5;

(6) sexta: HHHHH C6 = 1.

Portanto, a palavra BRASIL aparece


C1 + C2 + C3 + C4 + C5 + C6 = 1 + 5 + 10 + 10 + 5 + 1 = 32
vezes na figura.
Observao: O que significa a simetria C1 = C6 , C2 = C5 e C3 = C4 ?
191. Quais so os nmeros? A equao pode ser escrita na forma x4 y 2 = 71 e,
fatorando x4 y 2 = (x2 y)(x2 + y), na forma
(x2 y)(x2 + y) = 71.
Como x e y so inteiros, cada um dos fatores x2 y e x2 + y tambm um nmero
inteiro, de modo que escrevemos 71 como o produto de dois nmeros inteiros. Como
71 um nmero primo, ele s pode ser escrito como produto de inteiros na forma
1 71 ou 71 1. Assim, temos somente dois casos a considerar, a saber, x2 y = 71
e x2 + y = 1, ou x2 y = 1 e x2 + y = 71. Como x, y so inteiros positivos, temos
x4 = y 2 + 71 72 > 16 = 24 , portanto, x > 2. Em particular, x2 + y = 1 impossvel,
pois implicaria 1 = x2 + y 9 + 1 = 10.
Assim, resta considerar o caso x2 y = 1 e x2 + y = 71. Somando essas duas equaes,
obtemos 2x2 = 72, o que fornece x = 6 e, portanto, y = (6)2 1 = 35. Como x, y
so inteiros positivos, conclumos que a nica soluo x = 6 e y = 35.
OBMEP 2010

241

Solues do Nvel 2
192. No jogo Seja T a quantidade total de dinheiro no jogo. Assim, no incio, os jogadores
possuam
7
Aldo:
T,
18
6
Bernardo:
T,
18
5
T
Carlos:
18
e, no final, eles possuam
Aldo:
Bernardo:
Carlos:

6
T,
15
5
T,
15
4
T.
15

Para comparar essas fraes, usamos o denominador comum de 18 e 15, a saber, 90.
Desse modo, no incio,
7
35
Aldo:
T =
T,
18
90
6
30
Bernardo:
T =
T,
18
90
5
25
Carlos:
T =
T
18
90
e, no final,
Aldo:
Bernardo:
Carlos:

36
6
T =
T,
15
90
30
5
T =
T,
15
90
4
24
T =
T.
15
90

1
T, que corresponde a 12 reais, de modo
Logo, foi Aldo quem ganhou um total de
90
1
que
T = 12, ou seja, o total T de dinheiro no incio o jogo foi
90
T = 90 12 = 1 080 reais.
Assim, no final da partida, os jogadores possuiam, em reais,
Aldo:
Bernardo:
Carlos:

242

36
de 1 080 = 432,
90
30
de 1 080 = 360,
90
24
de 1 080 = 288.
90

OBMEP 2010

Solues do Nvel 2
p
p

3
3
193. Um nmero inteiro Denotemos a =
5+2 e b=
5 2. Ento



a3 b3 =
5+2
52 = 4
e

3
3
a b = ( 5 + 2)( 5 2) = 3 5 4 = 1 = 1,

de modo que M = a b satisfaz M 3 = (a b)3 = a3 b3 3a b(a b) = 4 3 M.

Assim, M 3 + 3M 4 = 0, ou seja, o nmero M raiz do polinmio x3 + 3x 4. Como


o nmero 1 uma raiz desse polinmio, podemos fator-lo e escrever x3 + 3x 4 como
(x 1)(x2 + x + 4). O trinmio x2 + x + 4 tem discriminante negativo, de modo que a
nica raiz real de x3 + 3x 4 1 e, portanto, M = 1. Em particular, M um nmero
inteiro.
194. rea de tringulos
cC e AM
cD so ngulos opostos pelo vrtice, de
(a) Note que F M
c = AM
cD. Como MC = MD e os tringumodo que F MC
los AMD e F MC so retngulos, estabelecemos que eles
so congruentes. Assim, possuem a mesma rea, donde conclumos que a rea do tringulo ABF igual rea do quadrado
ABCD, que foi dada, medindo 300 cm2 .

(b) Como AD = F C (do item anterior) e MC = MD, segue que os tringulos


AMD, DMF e F MC tm a mesma rea. Por outro lado, a soma das reas
dos dois ltimos a metade da rea do quadrado. Portanto, a rea do tringulo
AF D a metade da rea do quadrado ABCD. Essa rea foi dada, medindo
300 cm2 , logo, a rea do tringulo AF D mede 150 cm2 .
195. Um quadriculado Sejam m e n, respectivamente, o nmero de segmentos ao longo
de dois lados consecutivos do retngulo desenhado por Rosa. Sabemos que o nmero
total de segmentos que so lados de quadrados na diviso de um retngulo em m n
quadrados m(n + 1) + n(m + 1) (prove isso). Assim, como Rosa usou 1 997 segmentos
em seu desenho, temos m(n + 1) + n(m + 1) = 1 997.
Alm disso, um dos lados considerados menor do que ou igual ao outro, digamos,
m n. Nesse caso, obtemos 1 997 = m(n + 1) + n(m + 1) 2m(n + 1).

Como 1 998 > 1 997, segue que 1 998 > 2m(m + 1), ou seja, 999 > m(m + 1),
do que podemos deduzir que 1 m 31. Por outro lado, multiplicando, obtemos
1 997 = mn + m + mn + n = m + n(2m + 1), de modo que n = (1 997 m)/(2m + 1)
e, portanto,
3 994 2m
3 995 (2m + 1)
3 995
=
=
1.
2n =
2m + 1
2m + 1
2m + 1
No entanto, n um inteiro positivo, portanto, 2m + 1 precisa ser um divisor de 3 995.
Como 3 995 = 5 17 47 e 1 m 31, as nicas trs opes so 2m + 1 = 5, 17 ou
47, que fornecem m = 2, m = 8 e m = 23 e os valores correspondentes de n = 399,
n = 117 e n = 42.
OBMEP 2010

243

Solues do Nvel 2
Portanto, Rosa poderia ter desenhado trs configuraes diferentes com os 1 997 segmentos, uma com 2 399 quadrados, outra com 8 117 quadrados e uma terceira,
com 23 42 quadrados. Entretanto, a folha de papel utilizada mede 21 por 29,7 cm e
os segmentos que formam os lados dos quadrados medem 0,5 cm. Assim, as duas primeiras configuraes no cabem no papel de Rosa e podemos afirmar que o retngulo
que Rosa desenhou consiste em 23 42 quadrados e que, portanto, constitudo de
966 quadrados.
196. Inteiros de quatro algarismos Temos 1 000 4a2 < 10 000, do que decorre
250 a2 < 2 500. Mas, 152 = 225, 162 = 256 e 502 = 2 500, portanto, como a um
4
nmero natural, obtemos 15 < a < 50. Tambm temos 1 000 a3 < 10 000, do
3
que decorre 750 a3 < 7 500. Mas, 93 = 729, 103 = 1 000, 193 = 6 859 e 203 = 8 000,
portanto, como a um natural, tambm temos 9 < a < 20. Desse modo, as nicas
opes so a = 16, 17, 18 ou 19.
4
Por outro lado, como a3 um nmero inteiro, conclumos que a = 18. De fato, isso
3
pode ser obtido substituindo os quatro possveis valores de a ou, ento, observando
que a3 deve ser um mltiplo de 3 e, consequentemente, que a um mltiplo de 3.
197. Pares positivos Como 501 3x = 3(167 x), a equao dada equivalente a
3
y = (167 x). Como y um inteiro positivo, 167 x deve ser algum mltiplo positivo
5
de 5, ou seja, 167 x = 5k, para algum inteiro positivo k e, portanto, x = 167 5k
ou, ainda, x = 5 33 + 2 5k = 5(33 k) + 2. Como x um inteiro positivo, devemos
ter 1 k 33. Consequentemente, podemos tomar qualquer k = 1, 2, . . . , 33, obtendo
trinta e trs pares de inteiros positivos (x, y) que so solues da equao 3x+57 = 501.
198. Diferena de quadrados A resposta correta (e).
Soluo 1: Observe que o quadrado de um nmero par par e o quadrado de um
nmero mpar mpar. Se os dois nmeros so consecutivos, ento um deles par
e o outro mpar. Portanto, elevando ao quadrado, um dos quadrados par e o
outro mpar. Mas, a diferena entre um nmero par e um nmero mpar sempre
um nmero mpar. Como 2.000 um nmero par, conclumos que no existem dois
nmeros consecutivos tais que a diferena de seus quadrados seja 2 000.
Soluo 2: Seja n um nmero inteiro. Ento (n + 1)2 n2 = n2 + 2n + 1 n2 = 2n + 1,
de modo que a diferena entre o quadrado dos nmeros consecutivos n e n+1 , sempre,
um nmero mpar. Como 2 000 um nmero par, conclumos que no existem dois
nmeros consecutivos tais que a diferena de seus quadrados seja 2 000.
199. Clculo de ngulos Na primeira figura, prolongamos o segmento BC at sua
interseo com o segmento ED, num ponto F. Como os segmentos AB e ED so
b e B FbD so alternos internos, portanto, possuem a mesma
paralelos, os ngulos ABF
b
medida, ou seja, C F D = 25o . Agora, observe que o ngulo x externo ao tringulo
CDF. Logo, x igual soma dos dois ngulos internos no adjacentes, ou seja,
x = 25o + 55o = 80o .
244

OBMEP 2010

Solues do Nvel 2

A...............................................................................................................................B

A................................................................................................................................B

..
....

..

....
160o .........
.

25o........

..
....
....
....
....
.
.
..
.......
.... ......
...
....
....
....
.
.
.
....
..
....
....
....
...
.
.
..
.
.
.
55o ......
.
..................................................................................................................................

....
....
....
....
....
.
.....
.... .......
....
...
.
.
.
....
...
.
.
....
.
....
....
....
...
.
.
....
o .....
..
150
............................................................................................................................................................................................................................................

Na segunda figura, novamente prolongamos o segmento BC at sua interseo com


o prolongamento do segmento ED, num ponto F. Como os segmentos AB e EF so
b e D FbB so colaterais internos, portanto, suplementares,
paralelos, os ngulos ABF
ou seja,
b = 180o 160o = 20o .
D FbC = D FbB = 180o ABF

b igual a 30o , por ser o suplemento do ngulo


Por outro lado, o ngulo C DF
b = 150o . Agora, observe que o ngulo x externo ao tringulo CDF. Logo, x
E DC
igual soma dos dois ngulos internos no adjacentes, ou seja, x = 20o + 30o = 50o .

200. Tabela Como a tabela tem seis colunas, em cada linha escrevemos seis
nmeros consecutivos. Dividindo 1 000
por 6, obtemos
1 000 = 6 166 + 4.

1a linha
2a linha
3a linha
..
.
a
167 linha

1
7
13
..
.
997

2
8
14
..
.
998

3
9
15
..
.
999

4
10
16
..
.
1 000

5
11
17
..
.

6
12
18
..
.

Desse modo, para escrever o nmero 1 000 na tabela so necessrias 166 linhas completas (terminando no nmero 6 166 = 996) e mais uma linha com os quatro nmeros
997, 998, 999 e 1 000. Logo, 1 000 est escrito na 167a linha e na quarta coluna.
201. Entre 1 e 2 Se uma frao positiva e menor do que 1, seu numerador deve ser
menor do que seu denominador. Assim, devemos ter
0 < a < 5 e 0 < b < 7.
Como

1
1
1
a + b = (7a + 5b), a condio dada equivale a
5
7
35
1<

7a + 5b
< 2,
35

ou seja, 35 < 7a + 5b < 70.

Desse modo, vemos que o problema consiste em obter todos os inteiros a e b tais que
0 < a < 5,

0 < b < 7 e 35 < 7a + 5b < 70.

Examinemos cada uma das quatro opes de a, de 1 a 4, com correspondentes possibilidades de b, dentre os inteiros 1, 2, 3, 4, 5 e 6.
a = 1. Ento 7a + 5b = 7 + 5b e, de 35 < 7 + 5b < 70, decorre que 28 < 5b < 63,
ou seja, 6 b 12. Como b {1, 2, 3, 4, 5, 6}, conclumos que a nica opo
b = 6.
OBMEP 2010

245

Solues do Nvel 2
a = 2. Ento 7a + 5b = 14 + 5b e, de 35 < 14 + 5b < 70, decorre que 21 < 5b < 56,
ou seja, 5 b 11. Como b {1, 2, 3, 4, 5, 6}, conclumos que as nicas opes
so b = 5 e b = 6.
a = 3. Ento 7a + 5b = 21 + 5b e, de 35 < 21 + 5b < 70, decorre que 14 < 5b < 49,
ou seja, 3 b 9. Como b {1, 2, 3, 4, 5, 6}, conclumos que as nicas opes
so b = 3, 4, 5 ou 6.
a = 4. Ento 7a + 5b = 28 + 5b e, de 35 < 28 + 5b < 70, decorre que 7 < 5b < 42,
ou seja, 2 b 8. Como b {1, 2, 3, 4, 5, 6}, conclumos que as nicas opes
so b = 2, 3, 4, 5 ou 6.
Conclumos exibindo as doze solues a, b na tabela seguinte.
a

2
3
4
5
6

1<
1
5
4
5
4
5
4
5
4
5
4
5

a
5

+
6
7
2
7
3
7
4
7
5
7
6
7

+
+
+
+
+
+

b
7

=
=
=
=
=
=

<2

37
35
38
35
43
35
48
35
53
35
58
35

5
6

3
4
5
6

1<
2
5
2
5
3
5
3
5
3
5
3
5

a
5

+
+
+
+
+
+

+
5
7
6
7
3
7
4
7
5
7
6
7

b
7

=
=
=
=
=
=

<2
39
35
44
35
36
35
41
35
46
35
51
35

202. Triatlon Seja x a velocidade com que Maria nada, em metros por minuto. Ento o
tempo que Maria gasta nadando 800 m dado por 800/x minutos. Sabemos que sua
velocidade na corrida 3 x e, na bicicleta, 2,5 3 x = 7,5 x metros por minuto. Assim,
o tempo total que Maria gasta nas trs etapas
800
4 800
20 000
4 000
800 7,5 + 20.000 + 4 000 2,5
+
=
=
+
x
7,5 x
3x
7,5 x
x
|{z}
| {z } | {z }
nadando

pedalando

correndo

minutos. Logo, para que ela vena as trs etapas em, no mximo, uma hora e vinte
minutos, ou seja, em 80 min, devemos ter, no mnimo, 80 = 4 800/x, ou seja, x =
4 800/80 = 60 metros por minuto. Segue que 3 x = 180 e 7,5 x = 450 metros por
minuto. Assim, para que Maria termine a prova em, no mximo, 1 hora e 20 minutos,
ela deve desenvolver as seguintes velocidades mnimas:
nadar a uma velocidade mnima de 60 m/min,

pedalar a uma velocidade mnima de 450 m/min e


correr a uma velocidade mnima de 180 m/min.
203. Foto de formatura Os diagramas a seguir representam a situao do problema,
onde os alunos que foram inicialmente retirados esto representados em preto e os
alunos retirados na segunda vez, em cinza.
. . .
. . . t
. . . t
. . .
. . . t
. . . t
.. .. ..
.. .. .. .. ..
.. .. ..
.. .. .. .. ..
. . .
. . . . .
. . .
. . . . .
.. .. .. .. .. ..
.. .. ..
. . . . . .
. . .
. . .
. . . t
. . . t
t t t. . . t
t t t. . . t
246

OBMEP 2010

Solues do Nvel 2
Sejam m e n o nmero de filas (linhas horizontais) e de colunas da formao inicial,
respectivamente. Com um aluno de cada uma das m filas formada uma nova fila,
incompleta: faltam quatro alunos para completar as atuais n 1 colunas, ou seja,
m + 4 = n 1 e, portanto, n = m + 5. Agora temos m filas de n 1 alunos, alm de
uma fila incompleta, em que faltam quatro alunos.
Tirando um aluno de cada uma das m filas completas, for . . .
mamos um retngulo com uma coluna a menos, portanto pre . . .
.. .. .. ..
.. .. ..
enchemos as atuais trs vagas da nova fila. Assim, m = 3 e,
. . . .
. . .
. . .
portanto, n = 8. O nmero total de alunos na foto dado por
t t t. . . t t t t
n m = 3 8 = 24.
204. Circunferncias tangentes
(a) Como as circunferncias de 1 e 3 cm de raio so concntricas, as novas circunferncias tangentes s originais tambm devem ter raio igual a 1 cm.
(b) Os centros das trs circunferncias de 1 cm de raio mostradas na figura formam um tringulo equiltero de 2
cm de lado. Logo, seus ngulos internos medem 60o .
Como 360/60 = 6, conclumos que podem ser dispostas,
no mximo, seis circunferncias sem sobreposio, nas
condies exigidas.

....................
................... . . ............................
........ ....
... ......
....
...... ....
...
....
.
.
.
. .............
.
.
.
......
..
... ..... ................ .................
....
...
.
.
....
.
..
.
.
.
.
.
.
.
.
.
....
.....
...... ... ...................
...
...
...
..................... .
...
......
....... ... ............. .........
.
.
.
.....
.
... ....
... .........
.
.
..
....
.
.
.
............ ................ ............. ....
...
...
.
.
...
.. .....
..
..
...
...
..
.
.
....
.
...
..
......
....
..
..
......................
...
...
...
..
...
...
...
...
....
...
.
....
.
.
.
.....
.....
......
.........
.......
..................................................

205. Festa na escola Representando o nmero de docinhos que cada um dos quatro
amigos levou pela inicial de seu nome, temos
(
A + P + M + F = 90,
A + 2 = P 2 = 2M = 12 F.

Segue da segunda equao que P = A + 4, M = 12 (A + 2) e F = 2(A + 2). Substituindo


esses valores na primeira equao, obtemos


90 = A + (A + 4) + 21 (A + 2) + 2(A + 2) = 12 9A + 9 = 9 21 A + 1 ,

de modo que

1
2

A + 1 = 10, ou seja, A = 18. Assim,

P = 18 + 4 = 22,

M=

18 + 2
= 10 e F = 2(18 + 2) = 40.
2

206. Inflao O preo antigo era inferior a 50 reais e sofreu um acrscimo de 20%, com
o que o preo novo ainda um nmero de dois algarismos, que representamos por a b,
onde a o algarismo das dezenas e b o algarismo das unidades, ou seja, a b = 10 a + b.
O preo novo o preo antigo b a com um acrscimo de 20%, ou seja,
10 a + b = a b = (1,2) b a = 1,2(10 b + a) = 12 b + 1,2a,
de modo que 10 a 1,2a = 12 b b, ou seja, 8,8 a = 11 b. Assim,
b=

8,8
4
a = a.
11
5

OBMEP 2010

247

Solues do Nvel 2
Como a e b so algarismos, s podemos ter a = 5 e b = 4 e decorre que o novo preo
R$ 54,00.
207. Gatos no condomnio Sejam x o nmero de famlias que possuem apenas um
ou exatamente cinco gatos e y o nmero de famlias que possuem exatamente trs
gatos. Segue que x + y + x = 29 e, portanto, 2x + y = 29. Como o nmero de gatos
x + 3y + 5x = 6x + 3y, obtemos
nmero de gatos = 6x + 3y = 3(2x + y) = 3 29 = 87.
208. Soma constante Sejam a, b, c, d, e
1 a
ou seja, consideremos a tabela b 9
d e
formar, a saber,
1
b

a
9

a
9

2
c

e f os nmeros que falta preencher na tabela,


2
c . Nas quatro subtabelas 2 2 que podemos
f
9
e

b
d

devemos ter

1+a+b+9=a+2+9+c
1+a+b+9=b+9+d+e

a+2+9+c=9+c+e+f

9
e

c
,
f

b1 =c
a+1=d+e
ou seja,

a+2=e+f

Subtraindo a segunda igualdade obtida da terceira, obtemos 1 = f 1, ou f = 1 + d.


A nossa tabela, ento, dada como segue.
1
b
d

2
b1
d+1

a
9
e

Para os nmeros a, b, c, d, e e f temos apenas as opes 3, 4, 5, 6, 7 e 8, sem repetio.


Se a = 3, 4 ou 5, resulta d + e = a + 1 = 4, 5 ou 6, o que impossvel para inteiros
distintos d e e maiores do que 2. Se a = 7, resulta d + e = a + 1 = 8 e poderamos
ter (d, e) = (3, 5) ou (5, 3). Se d = 3, ento e = 5, f = d + 1 = 4 e, necessariamente,
b = 6 ou 8: no entanto, isso impossvel, pois implicaria c = b 1 = 5 = e ou
c = b 1 = 7 = a.

Assim, a no pode ser 3, 4, 5 nem 6, restando, apenas, as alternativas a = 6 ou a = 8.


Usando as relaes a + 1 = d + e e c = b 1, obtemos as duas nicas opes de
preenchimento da tabela dada, como segue.
1
8
4

6
9
3

2
7
5

1
5
6

8
9
3

2
4
7

209. Qual o nmero? Note que 5 E um mltiplo de 5 e no caso, terminado em


A. Como A no pode ser 0, segue que A = 5 e E mpar. Observe que E no pode
ser 1, pois, nesse caso, 4D = 5, o que impossvel para algarismos. Logo, E = 3, 5, 7
ou 9. Analisemos cada uma dessas possibilidades.
248

OBMEP 2010

Solues do Nvel 2
Se E = 3, ento 4D + 1 termina em 5 e, portanto, D = 1 ou D = 6;
se E = 5, ento 4D + 2 par e termina em 5, o que impossvel;
se E = 7, ento 4D + 3 termina em 5 e, portanto, D = 3 ou D = 8;
Se E = 9, ento 4D + 4 par e termina em 5, o que impossvel.
Restam, ento, os quatro casos seguintes, de acordo com (D, E) ser dado por (1, 3),
(6, 3), (3, 7) ou (8, 7).
5BC 63
BC 63
C 63
63
3
............................................................................
55555

5BC 13
BC 13
C 13
13
3
............................................................................
55555

5BC 37
BC 37
C 37
37
7
............................................................................
55555

5BC 87
BC 87
C 87
87
7
............................................................................
55555

1o Caso: Se D = 1 e E = 3, ento 3C termina em 5 e, como C denota um algarismo,


a nica opo C = 5 = A, o que no pode ocorrer.
2o Caso: Se D = 6 e E = 3, ento 3C + 2 termina em 5, portanto, 3C termina em
3. Como C denota um algarismo, a nica opo C = 1, resultando 2B = 5, o que
impossvel para um algarismo.
3o Caso: Se D = 3 e E = 7, ento 3C + 1 termina em 5, portanto, 3C termina em 4.
Como C denota um algarismo, a nica opo C = 8, resultando 2B + 2 = 5, o que
impossvel.
4o Caso: Se D = 8 e E = 7, ento 3C + 3 termina em 5, portanto, 3C termina em 2.
Como C denota um algarismo, a nica opo C = 4, resultando 2B + 1 = 5, com o
que B = 2.
Assim, a nica soluo A B C D E = 52 487.
210. Proporo triangular Escolhamos o ponto H do segmento BC de tal modo que
o segmento F H seja paralelo ao segmento AE, como na figura dada. Decorre que os
tringulos AEC e F HC so semelhantes, pois tm lados paralelos.

A
F
G
B

Como F C = 2 AF, decorre, por semelhana, que tambm HC = 2 EH. Por outro lado,
os tringulos BHF e BEG tambm so semelhantes, pois tm lados paralelos.
Dessa semelhana e do fato de G ser o ponto mdio do segmento BF, conclumos que
E o ponto mdio do segmento BH. Assim, BE = EH e, portanto,
EC = EH + HC = EH + 2 EH = 3 EH = 3 BE.
Consequentemente, EC/BE = 3.
OBMEP 2010

249

Solues do Nvel 2
211. Nmeros primos entre si Temos 2000 = 16 125 = 24 53 ,
x y 
 x2 + y 2  16 125
N = 2000
+
= 16 125
=
(x2 + y 2 )
y x
xy
xy
um inteiro mpar e x < y so inteiros positivos primos entre si.

Soluo 1: Como x e y so primos entre si, possvel mostrar que xy e x2 + y 2 no


tm fatores primos em comum (prove isso), de modo que xy divide 2 000 e, portanto,
x e y dividem 2 000. Alm disso, 16 deve dividir xy, porque N mpar. Como x e y
so primos entre si, 16 divide x ou y.
1o Caso: Se 16 divide x, ento x = 16. De fato, se x > 16, ento x , no mnimo, 16 5 = 80, pois x divide 2 000; como tambm xy divide 2 000, resultaria que
y 25 < 80 = x, o que no permitido. Logo x = 16 e, como x e y so primos entre
si e y divide 2 000, necessariamente y = 25 ou 125.
2o Caso: Se 16 divide y, uma possibilidade y = 16, quando x s pode ser 1 ou 5,
pois x < y e xy divide 2 000. As outras opes para y so 16 5, 16 25 ou 2 000,
quando a nica opo para x , sempre, 1.
Assim, existem sete solues, a saber,
(16, 25), (16, 125), (1, 16), (5, 16), (1, 80), (1, 400) e (1, 2 000).
Soluo 2: Como N um inteiro mpar, resulta que (24 53 )/(xy) e x2 + y 2 so
inteiros mpares. As opes para xy so 24 , 24 5, 24 52 e 24 53 . Alm disso, x e y
devem ter paridades distintas, para garantir que x2 + y 2 seja mpar. Vamos determinar
x e y para cada uma dessas opes, lembrando que x < y.
xy
24
24 5
24 52
24 53

x
1
1
5
1
24
1
24

y
24
24 5
24
4
2 52
52
24 53
53

Assim, existem sete solues, a saber,


(1, 16), (1, 80), (5, 16), (1, 400), (16, 25), (1, 2 000) e (16, 125).
212. Fique atento Elevando ambos os membros da equao ao quadrado, obtemos x =
x2 4x + 4, que equivalente a x2 5x + 4 = 0. As razes dessa equaodo segundo
grau so
x = 1 e x = 4. Quando substitumos x = 4 na equao original x = x 2,
obtemos 4 = 2, que uma afirmao
verdadeira. Entretanto, quando substitumos

x = 1 naquela equao, obtemos 1 = 1, que falsa. Portanto, a equao dada


possui a nica soluo x = 4.
Observao: O aparecimento da soluo estranha x = 1 deve-se ao fato seguinte: a
afirmao
a2 = b2 = a = b
250

OBMEP 2010

Solues do Nvel 2
no verdadeira. O que correto a afirmao
a2 = b2

a = b.

Desse modo, quando elevamos os dois membros de uma equao ao quadrado, obtemos uma nova equao que pode, eventualmente, conter mais solues que a equao
original. Voc pode ver isso com clareza, por exemplo, nas equaes x = 1 e x2 = 12 .
213. Solues inteiras A equao original pressupe x 6= 0 e y 6= 0, portanto, podemos
considerar a equao equivalente
xy = 19(x + y),

com x 6= 0, y 6= 0.

Uma vez que estamos procurando solues inteiras e 19 um nmero primo, essa
igualdade implica que x ou y deve ser divisvel por 19. Como a equao simtrica em
relao s variveis x e y, podemos supor que x divisvel por 19, ou seja, x = 19 k,
para algum valor k 6= 0 inteiro. Assim, a equao original tambm equivalente a
k y = 19 k + y,

com k 6= 0, y 6= 0.

Dessa igualdade, obtemos que 19k + y divisvel por k. Uma vez que 19k j divisvel
por k, conclumos que y divisvel por k (prove isso), isto , y = km, para algum valor
m 6= 0 inteiro. Segue que
k k m = k y = 19 k + y = 19 k + k m = (19 + m)k,
ou seja, k m = 19 + m, que igual a 19 = (k 1) m. Desse modo estabelecemos que os
inteiros m e k1 so divisores do numero primo 19. Como k 6= 0, segue que k1 6= 1,
o que nos deixa com trs opes apenas, como segue.
Se m = 19 e k 1 = 1, ento x = 38 e y = 38;

Se m = 1 e k 1 = 19, ento x = 380 e y = 20;

Se m = 1 e k 1 = 19, ento x = 342 e y = 18.


Desse modo, por simetria, obtemos os nicos cinco pares de nmeros inteiros (x, y) que
so solues da equao dada:
(38, 38), (380, 20), (342, 18), (20, 380) e (18, 342).
214. No ponto de nibus Representemos por M o nmero de meninas e por H o
nmero de meninos que estavam no ponto antes de passar o primeiro nibus. Depois
do embarque das 15 meninas no primeiro nibus, ficaram M 15 meninas e H meninos
no ponto. Uma vez que, nesse momento, ficaram dois meninos para cada menina no
ponto, temos H = 2(M 15). No segundo nibus, embarcam 45 meninos e ficaram
M 15 meninas e H 45 meninos no ponto. Como, nesse momento, ficaram no ponto
cinco meninas para cada menino, temos M 15 = 5(H 45). Assim, obtemos o sistema
linear

H = 2(M 15)
M 15 = 5(H 45)

Substituindo a primeira equao na segunda, resulta M 15 = 5(2M 30 45) =


10 M 375.
Logo, 9 M = 360, de modo que M = 40 e H = 2(40 15) = 50.
OBMEP 2010

251

Solues do Nvel 2
215. Contorno circular Sejam A, B, C e D os centros dos quatro crculos e M, N, P e
Q os pontos de tangncia entre esses crculos, conforme figura.

Q
D

B
N

Observe que AD = DC = CB = BA = AC = 2a. Logo, os tringulos ABC e


ACD so equilteros e, por isso, seus ngulos internos so iguais a 60o . Portanto,
b = 60o = 1 360o e D AB
b = 120o = 1 360o , o que acarreta que os arcos dos
ABC
6
3
contornos internos a esses dois ngulos medem

NM =

1
1
2 a e MQ= 2 a
6
3

5
2 a e
e os contornos externos por B e A, de traado destacado, medem
6
2
2 a, respectivamente. Por simetria, segue que o contorno externo da figura dada
3
tem comprimento igual a

5
2
2 +2
2 a = 6 a.
6
3
216. Um quadriltero especial Como cada diagonal divide o quadriltero em duas
regies de mesma rea, temos
rea (ABD) = rea (BCD) e rea (ABC) = rea (ACD).
Denotemos as reas das quatro regies determinadas pelas diagonais por X, Y, Z e W,
conforme a figura, de modo que
rea (ABD)
rea (BCD)
rea (ABC)
rea (ADC)

=
=
=
=

qB

X + W,
Y + Z,
Z +W e
X + Y.

...
....................
................ ......... ....
...............
..... ....
...............
.
.
.
.
.
.
.
.
.
.
.
.
.
.
.
.
.
.
.....
....
..
...............
.....
..
...............
.....
..
...............
..
.....
................
..
..............
.....
.
.
.
.
.
.
.
.
.
.
.
.
.
.
.
.
.
.
.
.
.
...........
....
..
...............
.....
..............
.....
...
.. ..............
..
.....
........
..
..
.....
.
.
.
........
.
...
.
.
..
........
..
...
.....
........
..
..
.....
........
..
..
.....
........
..
........ .........
...
..
..........
...
.
.
.. ....
..
..... ..............
...
..
..
........
.....
..
........
..
.....
..
........
..
.....
.
.
.
.
.
...
.
.
........
.
........
..
.....
..
........ ...
....
..
..........
.....
..
...
.....
..
...............
.
.
....
.
.
.
.
..
.
.
.
.
.
.
.
.
.
.
.
.
.
.....
.
....
...............
..
....
...............
..
.....
...............
..
.....
...............
..
...............
.....
.
.
.
.
.
.
.
.
.
.
.
.
.
.
.
.
.
.
.
....
.
....
.
...............
.. ....
...............
...............
.. .....
.. ..........................
.................

Aq

Assim,

Z X = rea (ABC) rea (ABD)

= rea (ACD) rea (BCD) = X Z

252

OBMEP 2010

Solues do Nvel 2
e, portanto, Z = X. Consequentemente, tambm temos Y = W. Seja E o ponto de
corte das diagonais. Como as reas das regies opostas por E so iguais, resulta da
semelhana de tringulos que EA ED = EB EC e EA EB = EC ED.

Dividindo essas duas equaes, obtemos

ED
EB
=
,
EB
ED
portanto, ED = EB. Analogamente, podemos mostrar que EA = EC. Logo, as
diagonais se cortam no ponto mdio e, consequentemente, o quadriltero um paralelogramo. Como os lados medem 10 e 15 cm, o permetro do quadriltero mede
2 10 + 2 15 = 50 cm.
217. Nmero curioso Seja a b = 10 a + b um nmero de dois algarismos a e b que
divisvel pela soma a + b de seus algarismos. Note que, por ser de dois algarismos,
necessariamente a 6= 0 e que, por ser divisvel pela soma de seus algarismos, tambm
a diferena (10 a + b) (a + b) = 9a divisvel por a + b (prove isso). Assim, basta
atribuir os valores 1, 2, 3, 4, 5, 6, 7, 8 e 9 a a e calcular os valores de b para os quais
a + b divide 9 a. O resultado aparece na tabela.
a
1
2
3
4
5
6
7
8
9

9a
9
18
27
36
45
54
63
72
81

b
0, 2, 8
0, 1, 4, 7
0, 6
0, 2, 5, 8
0, 4
0, 3
0, 2
0, 1, 4
0

Assim, os nmeros que satisfazem a propriedade so


10, 12, 18, 20, 21, 24, 27, 30, 36, 40, 42, 45, 48, 50, 54, 60, 63, 70, 72, 80, 81, 84 e 90,
ou seja, existem 23 nmeros curiosos.
218. Nmero premiado
(a) O maior nmero premiado de seis algarismos distintos precisa comear com 98,
portanto, o nmero procurado da forma 98 c d e f. Por hiptese, temos 9+8+c =
d + e + f. Para que c seja mximo, precisamos que d + e + f seja mximo, e isso
acontece quando d = 7, e = 6 e f = 5. Nesse caso, c = 1 e, consequentemente,
o maior nmero premiado 981 765. Para determinar o menor nmero premiado
de seis algarismos distintos, tentamos um nmero da forma 10 c d e f. No difcil
verificar que 108.234 o menor nmero premiado.
(b) Dado qualquer nmero premiado a b c d e f de seis algarismos distintos, seu par
simtrico d e f a b c tambm premiado e tem seis algarismos distintos; a soma
OBMEP 2010

253

Solues do Nvel 2
desse par simtrico
a b c d e f + d e f a b c = (1000 a b c + d e f ) + (1000 d e f + a b c)
= 1001(a b c + d e f ) = 13 11 7 (a b c + d e f ),
que divisvel por 13. Assim, a soma de todos esses pares simtricos tambm
divisvel por 13. Como a soma de todos esses pares de nmeros premiados simtricos igual soma de todos os nmeros premiados de seis algarismos distintos,
resulta que essa soma divisvel por 13.
Observao: De fato, a soma de todos os nmeros premiados de seis algarismos distintos
tambm divisvel por 7 e por 11.
219. Altura versus lado Sejam ha e hc as alturas relativas aos lados BC = a e AB = c,
respectivamente. Por hiptese, temos que ha a e hc c. Como ha e hc so os
comprimentos das alturas, pelo Teorema de Pitgoras temos ha c e hc a. Um dos
lados considerados maior do que ou igual ao outro, digamos, a c. Das desigualdades
acima, obtemos ha a c ha e, portanto, a = c = ha . Assim, AB perpendicular
a BC e, portanto, o tringulo retngulo issceles. Conclumos que os ngulos do
tringulo medem 45 , 45 e 90 .
220. Fraes egpcias A equao original pressupe x 6= 0, y 6= 0 e pede solues inteiras,
positivas e distintas, portanto, podemos considerar a equao equivalente
2xy = 7(x + y),

com x > 0, y > 0 e x 6= y.

Como 2 e 7 so nmeros primos, segue que 7 divide x ou y. Como a equao simtrica


em x e y, podemos supor que 7 divide x. Ento, x = 7 k, para algum k > 0 inteiro e
decorre que 2 7 k y = 7(7 k + y), ou seja, simplificando, 2 k y = 7 k + y ou, ainda,
(2k 1) y = 7 k = x.
Se 7 dividisse y, teramos y = 7 m, para algum m > 0 inteiro. Nesse caso, teramos
49 2 k m = 2xy = 7(x + y) = 49(k + m), acarretando 2 k m = k + m. Mas, ento
2=

k+m
1
1
= +
1 + 1 = 2,
km
k m

o que significa que k = m = 1 e, portanto, x = 7 = y. Como queremos x 6= y,


conclumos que 7 no divide y, de modo que 7 divide 2k 1. Tomando k = 4, resulta
x = 28 e
7k
28
y=
=
= 4,
2k 1
7
fornecendo a soluo
1
1
2
+ = .
28 4
7
Observao: A soluo obtida nica. De fato, como 2k 1 , sempre, mpar e 7
divide 2k 1, o mltiplo de 7 que igual a 2k 1 deve ser mpar. Assim, existe algum
inteiro n > 0 tal que
7(2n 1) = 2k 1.
254

OBMEP 2010

Solues do Nvel 2
Isso acarreta que k = 7n 3 e, portanto,
y=

7n 3
n
7k
7(7n 3)
3(2n 1) + n
=
=
=
=3+
.
2k 1
7(2n 1)
2n 1
2n 1
2n 1

Como y deve ser inteiro, conclumos que 2n 1 divide n, de modo que 2n 1 n. No


entanto, n 1 e, portanto, 2n 1 n. A nica possibilidade 2n 1 = n e, portanto,
n = 1. Segue que k = 4 = b e a = 28 do a nica soluo.
221. Tabuleiro de xadrez Um tabuleiro de xadrez um quadrado reticulado de 64
quadradinhos, denominados casas, sendo 32 casas pretas e 32 brancas, posicionados
alternadamente. Uma das peas do xadrez recebe o nome de bispo, havendo um par
deles para cada jogador. Um dos dois bispos de um jogador s se movimenta pelas
casas pretas e o outro s pelas brancas.
Inicialmente, possvel colocar um dos dois bispos
em qualquer uma das 64 casas. Se o bispo estiver
numa casa branca, ento na fila em que ele est,
bem como na coluna, temos quatro casas pretas que
no podem ser ocupadas pelo segundo bispo, num
total de oito casas. Assim, o segundo bispo pode ser
colocado em qualquer uma das 32 8 = 24 casas
pretas restantes.
Conclumos, ento, que se um dos bispos ocupar uma
o outro ter 24 casas pretas disposio. Portanto, o
distintas que podem ser obtidas 32 24 = 768.

Nota: Aqui estamos entendendo que alternando a posio desses dois bispos no
muda a configurao no tabuleiro de xadrez. Mais precisamente, os bispos tm a mesma cor, isto , pertencem a um
mesmo jogador.

das 32 casas brancas, ento


nmero dessas configuraes

222. Quem menor? Observemos que:


3312 > 3212 = (25 )12 = 260 ;
6310 < 6410 = (26 )10 = 260 ;
1278 < 1288 = (27 )8 = 256 < 260 .
Logo, o maior dos nmeros 3312 . Por outro lado,
 127 2
63

e, portanto,

1
127
=2+
garante que
63
63


1
1 2
4
5
= 2+
=4+
+ 2 <4+
<5
63
63 63
63
 127 4

< 25 < 63.


63
Assim, 1274 < 635 , acarretando 1278 < 6310 . O menor dos trs nmeros dados 1278 .
223. Brincando com nmeros Como queremos encontrar o maior nmero que seja
divisvel pela soma de seus algarismos e tambm menor do que 900, podemos comear
nossa busca dentre os nmeros com o algarismo 8 na casa da centena, j que, no
mnimo, 800 divisvel pela soma 8 + 0 + 0 = 8 de seus algarismos e 899 no tem essa
propriedade. Assim, vamos examinar os nmeros entre 800 e 899.
Queremos, ento, encontrar algarismos b e c tais que 8 + b + c divida
8 b c = 800 + 10b + c. Lembrando que 8 + b + c divide 8 b c = 800 + 10b + c se, e
OBMEP 2010

255

Solues do Nvel 2
somente se, 8 + b + c divide 800 + 10b + c (8 + b + c) = 792 + 9b, basta procurar entre
os divisores de 792 + 9b. Para isso, atribumos valores para b em ordem decrescente, a
partir de 9, at encontrar o maior nmero procurado.
Se b = 9, ento 792 + 9 9 = 873 = 9 97 e esse nmero no possui divisor
8 + 9 + c entre 17 (c = 0) e 26 (c = 9).
Se b = 8, ento 792 + 9 8 = 864 = 25 33 . O maior divisor 8 + b + c desse
nmero entre 16 e 25 24, isto , c = 8.
Logo, o nmero procurado 888.
224. Cortando papis Se na primeira rodada Andr pega n1 pedaos de papel para
cortar cada um deles em sete pedaos, ao final dessa rodada ele ficar com 7 n1
pedaos sem cortar, mais 7n1 pedaos cortados, totalizando (7 n1 ) + 7n1 = 7 + 6n1
pedaos de papel. Analogamente, se na segunda rodada Andr pega n2 pedaos de
papel para cortar, ao final dessa rodada ele ficar com 7 + 6n1 n2 pedaos que no
foram cortados nessa rodada, mais 7n2 pedaos de papel provenientes dos cortes que
ele fez nessa rodada. Assim, ao final da segunda rodada, Andr ficar com
(7 + 6n1 n2 ) + 7n2 = 7 + 6(n1 + n2 ).
Continuando assim, conclui-se que, ao final de k rodadas, Andr fica com
7 + 6(n1 + n2 + + nk )
pedaos de papel. Ento, para ele ficar com 2 009 pedaos de papel ao final de alguma rodada, deveramos ter essa ltima expresso igual a 2 009 ou, equivalentemente,
subtraindo 7 de cada lado, 6(n1 + n2 + + nk ) = 2 002.

No entanto, 2 002 no um mtiplo de 6, de modo que essa equao no admite


soluo. Isso significa que Andr nunca poder ficar com 2 009 pedaos ao final de
alguma rodada de sua brincadeira.

225. Um trapzio especial Queremos provar que AE igual a BC. Para isso, suponhamos que AE seja maior do que BC e escolhamos o ponto A sobre AE tal que
EA = BC. Por construo, EA e BC so paralelos, de modo que A BCE um
paralelogramo e, em particular,
A B = CE.

Pela desigualdade triangular, temos


A A + AB > A B.

Logo,

EA + AB + BE = EA + A A + AB + BE
> EA + A B + BE = BC + CE + EB.
Disso decorre que o permetro do tringulo ABE maior do que o permetro do
tringulo BCE, contrrio aos dados do problema.
256

OBMEP 2010

Solues do Nvel 2
Por meio dessa contradio, estabelecemos que, diante das hipteses do problema,
AE no pode ser maior do que BC. Por um processo totalmente anlogo, tambm
podemos estabelecer que, reciprocamente, BC no pode ser maior do que AE, com o
que conclumos que BC = AE. O mesmo raciocnio pode ser utilizado para mostrar
que BC = ED. Assim,
BC = 12 (AE + ED) = 15 cm.
b = B EH.
b
226. Uma estrela Observe que J EI
No tringulo BEH temos
b = 180o ,
20o + 130o + B EH

20

portanto,

J
130

E
I

b = B EH
b = 30o .
J EI

227. Nmero palndromo Um nmero palndromo de quatro algarismos da forma


a b b a, onde a um algarismo entre 1 e 9 e b um algarismo entre 0 e 9. Como o
nmero divisvel por 9, ento a soma 2a + 2b = 2(a + b) de seus algarismos divisvel
por 9, ou seja, a + b divisvel por 9. Como 1 a + b 18, as nicas opes so
a + b = 9 ou 18. Se a + b = 18, necessariamente a = b = 9. Se a + b = 9, temos as nove
solues seguintes.
a=1 e b=8

a=2 e b=7

a=3 e b=6

a=4 e b=5

a=5 e b=4

a=6 e b=3

a=7 e b=2

a=8 e b=1

a=9 e b=0

Assim, existem dez nmeros palndromos de quatro algarismos divisveis por 9, a saber,
1 881, 2 772, 3 663, 4 554, 5 445, 6 336, 7 227, 8 118, 9 009 e 9 999.
228. Multiplicao com letras Como o produto de b por c termina em 1, ento b c
pode ser 21 ou 81 e, portanto, 37 ou 99. A nica possibilidade de escrever o produto
de dois nmeros distintos menores do que 10 21 = 3 7. Assim, temos somente dois
casos possveis.
1o Caso: Se b = 7 e c = 3, deveramos ter
a77
3
7371
7371
= 2457 tem quatro algarismos.
3
2o Caso: Se b = 3 e c = 7, temos

mas isso impossvel, pois,

a33
7
3731
OBMEP 2010

257

Solues do Nvel 2
3731
= 533, necessariamente a = 5.
7
Logo, a nica possibilidade a = 5, b = 3 e c = 7.

e, como

229. Nmeros sortudos


(a) A sequncia de oito nmeros consecutivos de 52 a 59 tem, exatamente, dois nmeros sortudos: 52 e 59. Outro exemplo qualquer sequncia de oito nmeros
que contenha 59 e 61, por exemplo, 55, 56, 57, 58, 59, 60, 61, 62.
(b) Dois exemplos so 994, . . . , 1 005 e 7 994, . . . , 8 005. Existem mais: encontre alguns.
(c) Digamos que uma dcada qualquer sequncia de dez nmeros consecutivos cujo
primeiro termo algum mltiplo de 10. Por exemplo,
10, 11, 12, 13, 14, 15, 16, 17, 18, 19
e
140, 141, 142, 143, 144, 145, 146, 147, 148, 149
so dcadas. Note que qualquer sequncia de sete nmeros consecutivos numa
dcada contm, pelo menos, um nmero sortudo, porque a soma de seus algarismos uma sequncia de sete nmeros consecutivos, um dos quais precisa ser
divisvel por 7. Finalmente, qualquer sequncia de treze nmeros consecutivos
contm pelo menos sete nmeros consecutivos de alguma dcada, que sempre
contm um nmero sortudo. (Examine alguns exemplos para melhor entender
essa justificativa.)
230. Uma sequncia especial Inicialmente escrevemos os primeiros termos dessa sequncia, como segue.
1, 3, 2, 1, 3, 2, 1, 3, 2, . . .
O stimo e o oitavo termos so, respectivamente, iguais ao primeiro e ao segundo. Isso
significa que a sequncia se repete de seis em seis termos. A soma dos seis primeiros
termos 1 + 3 + 2 1 3 2 = 0 e, portanto, a soma dos 96 primeiros termos tambm
0. Assim, a soma dos 100 primeiros termos dessa sequncia igual soma dos quatro
ltimos termos, ou seja, 1 + 3 + 2 1 = 5.

231. Tringulos e ngulos... No tringulo menor, dois ngulos medem 70 e


180 130 = 50 e o terceiro mede
Assim,

180 (50 + 70 ) = 60 .

130
50

= 180 60 = 120 .

70

Agora, no tringulo maior, temos

45 + + 50 = 180 ,
portanto,
= 180 95 = 85 .
258

OBMEP 2010

45

Solues do Nvel 3

Solues do Nvel 3
1. Usando velas A opo correta (d).
Com 43 velas a casa de Joo pode ser iluminada por 43 noites, sobrando 43 tocos de
vela. Como 43 = 4 10 + 3, com esses 43 tocos pode-se guardar 3 tocos e fazer 10
novas velas para iluminar 10 noites. Dessas 10 velas obtemos 10 tocos que, com os 3
que haviam sobrado, do 13 tocos. Como 13 = 4 3 + 1, com esses 13 tocos pode-se
guardar 1 toco e fazer 3 novas velas para iluminar 3 noites. Dessas 3 velas obtemos
3 tocos que, com o que havia sobrado, do 4 tocos, com os quais podemos fazer mais
uma vela. Assim, no total, a casa de Joo pode ser iluminada por 43 + 10 + 3 + 1 = 57
noites.
2. Rodas e bandeiras A opo correta (a).
Os dois discos giram em sentidos opostos; quando um gira no sentido horrio o outro
gira no sentido anti-horrio. Considerando que a engrenagem da esquerda girou um
certo ngulo em um sentido, a engrenagem da direita girou o mesmo ngulo no sentido
oposto, e portanto a bandeirinha ficou na posio mostrada na opo (a).
3. Nmero de latas A opo correta (a).
Em cada caixote de madeira de dimenses a b c cabem (a b c)/l3 cubos
de lado l, empilhados regularmente. No caso dos palmitos temos, em centmetros,
a = 60, b = 80, c = 120 e l = 20. Como 60, 80 e 120 so mltiplos de 20, podemos
preencher o caixote, sem deixar espaos, com (6080120)/203 = 72 caixas de papelo
de formato cbico com 20 cm de lado. Logo, em cada caixote cabem 72 8 = 576
latas de palmito.
4. Qual a menor frao? A opo correta (c).
n
Soluo 1: As fraes da forma
, com n inteiro positivo, so
n+1
1
2
3
4
5
,
,
,
,
,....
2 |{z}
3 |{z}
4 |{z}
5 |{z}
6
|{z}
n=1

n=2

n=3

n=4

n=5

2
3
4
1
<
<
<
< . . . , ou seja, essa sequncia de fraes
2
3
4
5
crescente. Para comparar cada uma dessas fraes com 7/9, precisamos igualar todos
1
9
14
7 2
6
7 3
27
28
7
os denominadores, obtendo
=
<
= ,
=
< ,
=
<
=
e
2
18
18
9 3
9
9 4
36
36
9
4
36
35
7
4
7
=
>
= . Logo, maior do que e, como a sequncia crescente, a partir
5
45
45
9
5
9
4
7
de , todas as fraes dessa sequncia so maiores do que . Assim, existem apenas
5
9
n
7
1 2 3
trs fraes da forma
que so menores do que , a saber, , e .
n+1
9
2 3 4
Soluo 2: Transformando tudo em nmeros decimais, temos 7/9 = 0,777. . . e 1/2
= 0,5, 2/3 = 0,666. . . , 3/4 = 0,75, 4/5 = 0,8, 5/6 = 0,8333. . . . Logo, a sequncia
crescente e apenas 1/2 = 0,5, 2/3 = 0,666. . . e 3/4 = 0,75 so menores do que 7/9 =
0,777. . . .
Observe que temos

OBMEP 2010

259

Solues do Nvel 3
5. Pistas de corrida A opo correta (c).
Soluo 1: Denotemos por x e y os comprimentos das pistas longa e curta, respectivamente. Numa certa semana, o atleta corre 6(x + 2y) e, na outra, 7(x + y). Como nas
duas semanas ele corre os mesmos 5 000 metros, obtemos 6(x + 2y) = 7(x + y). Logo,
6x + 12y = 7x + 7y e, portanto, 5y = x. Assim, a pista longa cinco vezes maior do
que a pista curta.
Soluo 2: Na semana em que o atleta treinou durante sete dias, ele correu uma pista
longa a mais e cinco pistas curtas a menos do que na semana em que ele treinou apenas
seis dias. Como a distncia corrida foi a mesma nas duas semanas, conclumos que o
comprimento da pista longa igual ao comprimento de cinco pistas curtas.
6. Brincos e brincos A opo correta (c).
Soluo 1: Sabemos que o nmero de mulheres que usam apenas um brinco
0,03 800 = 24. Restam 800 24 = 776 mulheres, das quais 388 usam dois brincos e 388 no usam brincos. Logo, o nmero total de brincos usados por todas as
mulheres 24 + 388 2 = 800.
Soluo 2: Se cada mulher com dois brincos emprestar um de seus brincos a uma das
mulheres que no usam brincos, todas as 800 mulheres estaro com um nico brinco.
Logo, o nmero de brincos igual ao de mulheres, ou seja, 800.
7. Perguntas e respostas A opo correta (e).
A partir da tabela obtemos o nmero de pontos de cada um dos trs participantes.
Ana: 5 12 + (3) 3 + (2) 5 = 60 9 10 = 41
Bento: 5 13 + (3) 7 + (2) 0 = 65 21 = 44

Lucas: 5 12 + (3) 4 + (2) 4 = 60 12 8 = 40

Logo, Bento foi o mais bem classificado, seguido de Ana e, depois de Lucas.
8. Qual a carga? A opo correta (b).
Como o peso de um saco de areia igual ao de oito tijolos e no caminho j h 32
sacos de areia, ele pode carregar ainda 18 sacos de areia, o que equivale a 18 8 = 144
tijolos.
9. Quanto mede a cerca? A opo correta (b).
Entre o terceiro e o sexto poste, temos trs espaos entre postes consecutivos. Logo,
a distncia entre dois postes consecutivos 13 3,3 m = 1,1 m e a distncia entre o
primeiro e o ltimo poste de 11 1,1 = 12,1 m.
10. Dzima peridica A opo correta (d).
Soluo 1: Como 1/3 = 0,333 . . . , segue que
0,1333 . . . = 0,333 . . . 0,2 =
260

1
2
1 1
2

= =
.
3 10
3 5
15

OBMEP 2010

Solues do Nvel 3
Soluo 2: Usando simplesmente a regra que fornece a geratriz de uma dzima peridica, tambm podemos obter
0,1333 . . . =

13 1
12
2
=
=
.
90
90
15

11. Valor absoluto A opo correta (e).


Temos: |5| = 5, |3 8| = | 5| = 5 e | 4| = 4. Logo, N = 5 + 5 4 = 6.
12. O peso das frutas A opo correta (b).
A partir das informaes fornecidas pelas trs figuras, podemos montar trs equaes
em que, informalmente, denotamos o peso de cada fruta pelo seu prprio nome.
mamo = banana + ma
banana + mamo = 200
banana + 200 = mamo + ma
Somando a primeira com a terceira obtemos, aps cancelamento, 2 ma = 200,
donde ma = 100. Substituindo esse valor na primeira equao, obtemos mamo =
banana + 100 e, substituindo na segunda equao, obtemos 2 banana + 100 = 200,
donde banana = 50. Esses valores fornecem, pela primeira equao, o valor mamo =
150. Assim, a soma dos pesos das frutas 100 + 50 + 150 = 300 gramas.
13. Maratona A opo correta (c).
O comprimento de uma circunferncia de raio r 2 r. Assim, em cada volta Andr
percorre 2 100 m = 200 m. Logo, o nmero de voltas que Andr precisa dar para
completar 42 km = 42.000 m
210
42 000
=
.
200

Agora podemos finalizar o problema de duas maneiras.


1
1
1
1a ) Como 3 < < 4, obtemos
<
< , portanto, multiplicando tudo por 210,
4

3
resulta
210
210
210
52,5 =
<
<
= 70
4

3
e conclumos que Andr deve dar entre 52 e 70 voltas para percorrer os 42 km.
2a ) A aproximao de at a segunda casa decimal 3,14. Da,
210
210

66,88

3,14
e conclumos que Andr deve dar entre 66 e 67 voltas para percorrer os 42 km.
14. Dobrando papel A opo correta (e).
Soluo 1: Denotemos por ABC o tringulo obtido aps dobrar o quadrado original
ao longo das duas diagonais e seja MN o corte pela base mdia nesse tringulo, paralelo
ao lado BC, que um dos lados do quadrado original. A rea do quadrado original
OBMEP 2010

261

Solues do Nvel 3
(BC)2 . Desdobrando-se a folha, vemos que o buraco um quadrado de lado MN e,
1
como MN = BC, sua rea
2
1
2 1
2
(MN) =
BC = (BC)2 .
2
4
Logo, o buraco tem um quarto da rea do quadrado original.

A
N

N
C

Soluo 2: O corte realizado pela base mdia do tringulo, retirando um pequeno


tringulo semelhante ao original, com razo de semelhana 1/2. Assim, a rea do
tringulo retirado um quarto da rea do tringulo original. Abrindo a folha, vemos
essa situao reproduzida quatro vezes, donde o buraco tem um quarto da rea do
quadrado original.
15. Encontre o nmero A opo correta (a).
N N N N N
e
sejam nmeros inteiros, N deve ser um mltiplo comum
Para que , , ,
3 4 5 6
7
de 3, 4, 5, 6 e 7. Como queremos o menor N possvel, ele deve ser o mnimo mltiplo
comum (MMC) de 3, 4, 5, 6 e 7, ou seja,
N = 3 4 5 7 = 420.
16. Equao quadrtica A opo correta (d).
1
Soluo 1: Como 3 e so razes da equao ax2 6x + c = 0, temos 9a 18 + c = 0
3
1
e a 2 + c = 0, ou seja, 9a + c = 18 e a + 9c = 18. Somando essas duas equaes,
9
resulta 10(a + c) = 10a + 10c = 36, ou seja, a + c = 36/10 = 18/5.
Soluo 2: Numa equao ax2 + bx + c = 0 do segundo grau, a soma das razes
1
1
b/a e o produto c/a. Como b = 6, obtemos 10/3 = 3 + = 6/a e 1 = 3 = c/a,
3
3
ou seja, a = c = 9/5. Assim, a + c = 18/5.
17. Cubo A opo correta (d).
Soluo 1: Desenhando o cubo e numerando seus vrtices
de acordo com o enunciado da questo, obtemos uma figura
em que podemos ver que o vrtice 5, por ser diametralmente
oposto, o mais distante do vrtice 6.

OBMEP 2010

3
7

262

5
2

Solues do Nvel 3
Soluo 2: O vrtice 6 est nas faces {1, 2, 6, 7}, {1, 4, 6, 8} e {3, 4, 6, 7}. Como
nessas faces s no aparece o 5, segue que este o vrtice diagonalmente oposto ao 6,
ou seja, o 5 o vrtice mais distante do 6.
18. Time de basquete A opo correta (a).
Basta ler o grfico para obter o nmero de pontos de cada aluno. A soma desses pontos
d um total de 7 + 8 + 2 + 11 + 6 + 12 + 1 + 7 = 54 pontos marcados pelo time.
19. O caminho da formiguinha A opo correta (e).
Para cada um dos trs caminhos para ir de A at B, existem trs opes para ir de
B a C. Logo, h um total de 3 3 = 9 possibilidades. Mais geralmente, se fossem m
os caminhos de A at B e n os de B at C, ento o nmero de caminhos que nossa
formiguinha poderia tomar de A at C seria mn; esta afirmativa um caso particular
do princpio multiplicativo.
20. Operao z A opo correta (a).
Fazendo a = 1 e b = 0 em a z b = a2 ab + b2 , obtemos 1 z 0 = 12 1 0 + 02 = 1.
21. Indo para a escola Os alunos da escola foram divididos em quatro grupos distintos,
de acordo com o tempo que gastam no trajeto de casa para a escola. Cada uma das
quatro barras do diagrama representa exatamente um desses quatro grupos e cada um
dos alunos dessa escola est em exatamente um desses quatro grupos.
(a) Os alunos que gastam menos de 20 minutos em seu trajeto de casa para a escola
esto representados pela primeira barra, a mais alta, que atinge a marca dos 90.
Logo, 90 alunos gastam menos do que 20 minutos para chegar escola.
(b) O total de alunos na escola a soma dos nmeros representados pelas quatro
barras, portanto, a escola tem um total de 90 + 60 + 10 + 20 = 180 alunos.
(c) Os alunos que gastam mais do que 40 minutos esto repartidos em dois grupos:
os que gastam de 41 a 60 minutos e os que gastam mais do que 60 minutos,
representados pela terceira e quarta barras, as duas mais baixas, uma atingindo
a marca dos 10 e a outra, a marca dos 20. Logo, o total de alunos que gastam
mais do que 40 minutos para chegar escola de 10 + 20 = 30 alunos.
(d) Os alunos que gastam entre 20 e 40 minutos em seu trajeto de casa para a escola
esto representados pela segunda barra, que atinge a marca dos 60. Junto com os
30 alunos que gastam mais do que 40 minutos (item precedente), temos um total
de 60 + 30 = 90 alunos que gastam mais do que 20 minutos para chegar escola.
No primeiro item vimos que 90 alunos gastam menos do que 20 minutos para
chegar escola, que o mesmo nmero dos que levam mais do que 20 minutos, ou
seja, a metade dos alunos da escola que leva mais do que 20 minutos. Conclumos
que no verdade que a maioria dos alunos gasta mais do que 20 minutos para
chegar escola.
22. Campeonato de futebol
(a) Cada uma das seis equipes disputou, com cada uma das outras cinco, exatamente
uma partida. Portanto, foram disputadas um total de 12 (6 5) = 15 partidas.
OBMEP 2010

263

Solues do Nvel 3
(b) Cada equipe disputou exatamente 5 partidas. Logo, de x + 1 + 0 = 5 decorre
x = 4. Da mesma forma, para a equipe D temos 1 + 1 + y = 5, portanto y = 3.
O nmero total de gols feitos num campeonato igual ao nmero total de gols
sofridos, ou seja, 6 + 6 + 2 + 3 + 1 + z = 2 + 6 + 6 + 6 + 5 + 3, ou 18 + z = 28,
portanto, z = 10.
23. Poste eltrico Nesta questo utilizamos o Teorema de
Pitgoras. Antes de rever o enunciado desse teorema, lembre
que um tringulo dito retngulo quando um de seus ngulos
reto, ou seja, mede 90 . O lado oposto ao ngulo reto a
hipotenusa e os outros dois lados so os catetos do tringulo
retngulo. Na figura, a hipotenusa a e os catetos so b e c.

Teorema de Pitgoras
C
b

c
A
B
Teorema de Pitgoras. Num tringulo retngulo de hipotenusa a e catetos b e c,
vale a relao a2 = b2 + c2 .

Agora resolvemos a questo.


Para que o poste fique perpendicular ao solo, o ngulo em A deve ser reto e, portanto,
o tringulo ABC deve ser retngulo (ver figura). Nesse caso, os dados do problema
do que a hipotenusa mede 2,5 m e os catetos 1,4 m e 2 m. Assim, pelo Teorema de
Pitgoras teramos (2,5)2 = (1,4)2 + 22 .

C
1,4

2,5
2

Entretanto, (1,4)2 + 22 = 1,96 + 4 = 5,96 e (2,5)2 = 6,25. Logo, essas medidas no


satisfazem o Teorema de Pitgoras e, portanto, o tringulo ABC no retngulo.
Assim, o ngulo em A no reto e, consequentemente, o poste no est perpendicular
ao solo. Conclumos que o professor est certo.
24. Equaes recprocas
1
. Usando as expanses do binmio e do trinmio, obtemos
x

1 2
1
1
1
y2 = x +
= x2 + 2x + 2 = x2 + 2 + 2
e
x
x x
x


1 3
1
1
1
1
1
y3 = x +
= x3 + 3x2 + 3x 2 + 3 = x3 + 3 x +
+ 3,
x
x
x
x
x
x

(a) Temos y = x +

portanto,

x2 +
264

1
= y2 2
x2

x3 +


1
1
3
=
y

3
x
+
= y 3 3y.
x3
x

OBMEP 2010

Solues do Nvel 3


1
1
(b) A equao dada equivalente a x2 + 2 5 x +
+ 8 = 0. Substituindo o
x
x
valor de y e utilizando a identidade do item anterior, obtemos
(y 2 2) 5y + 8 = 0,

ou seja, a equao de segundo grau y 2 5y + 6 = 0, cujas razes so y = 2 e


1
y = 3. Voltando para x, multiplicamos x + = y = 2 e y = 3 por x para obter as
x
equaes quadrticas (x 1)2 = x2 2x + 1 = 0 e x2 3x + 1 = 0, cujas razes so

1
x = 1 e x = 3 5 . Assim, obtivemos todas as trs razes da equao dada.
2
(c) Como x = 0 no raiz da equao dada, podemos dividir tudo por x2 . Desse
modo, encontramos exatamente a equao do item (b), cujas razes j obtivemos.
(d) Como x = 0 no raiz da equao dada, podemos dividir tudo por x3 . Desse
modo, reordenando os termos, obtemos a equao equivalente



1
1
1
+ 12 = 0.
x3 + 3 2 x2 + 2 5 x +
x
x
x
Substituindo o valor de y e utilizando as identidades do item (a), obtemos


y 3 3y 2 y 2 2 5y + 12 = 0,
equivalente equao cbica

y 3 2y 2 8y + 16 = 0.

A forma mais rpida de resolver essa equao ter um pouco de sorte e fatorar
por agrupamento, obtendo, por exemplo,

y 3 2y 2 8y + 16 = y 2 (y 2) 8(y 2) = y 2 8 (y 2),

de modo que as trs razes da equao cbica em y so y = 2 e y = 2 2.

1
Voltando para x, multiplicamos x + = y = 2 e y = 2 2 por x para obter
x

2
2
2
as equaes
quadrticas
(x

1)
=
x

2x
+
1
=
0,
x

2x + 1 = 0 e
2
x + 2 2x + 1 = 0, cujas razes so x = 1, x = 2 1 e x = 2 1. Assim,
obtivemos todas as cinco razes da equao dada.
25. Atirando flechas
(a) Os cinco pontos dados esto marcados na figura.
Ordenada
E
C

B
A

abcissa

OBMEP 2010

265

Solues do Nvel 3
(b) No crculo menor temos apenas o ponto A, portanto Manoel acertou apenas uma
vez neste crculo, o que lhe d 300 pontos.
(c) Para calcular o total de pontos, observe que pelo ponto B ele ganha 100 pontos,
por C ele ganha 50 pontos e, por D, 50 pontos. Entretanto, pelo ponto E, ele
no ganha pontos, porque est fora da zona de pontuao. Logo, o nmero total
de pontos que Manoel fez 300 + 100 + 50 + 50 = 500.
26. Festa de aniversrio Como podemos repartir o total de convidados em mesas de
6 ou 7, o nmero de convidados um mltiplo de 6 e de 7. Como o menor mltiplo
comum de 6 e 7 42, podemos ter 42, 84, 126, . . . convidados. Como so menos do
que 120 convidados, s podemos ter 42 ou 84 convidados. Por outro lado, como so
necessrias mais do que 10 mesas, temos mais do que 60 convidados. Logo, descartamos
o 42, e o nmero de convidados s pode ser 84.
27. Medida do cateto O segmento CF, cujo comprimento queremos calcular, um cateto do tringulo retngulo CDF. O Teorema de Pitgoras, aplicado a esse tringulo,
diz que (CD)2 = (CF )2 + (F D)2 = (CF )2 + 242 e, da, tiramos (CF )2 = (CD)2 242 .
Ou seja, para encontrar CF basta conhecer CD. Como os lados opostos de um retngulo (e, mais geralmente, de um paralelogramo) so iguais, temos CD = AB. Nosso
objetivo, ento, passa a ser o clculo de AB. Para isso, olhemos para o tringulo
ABE. Sua rea
 1
1
AE BE = (15 BE) = 150,
2
2
donde tiramos BE = 20. O Teorema de Pitgoras aplicado a esse tringulo nos d
(AB)2 = (AE)2 + (BE)2 = 152 + 202 = 625 = 252 , donde AB = 25. Logo, CD =
AB = 25 e, de acordo com nossa observao anterior, obtemos
(CF )2 = (CD)2 242 = 252 242 = (25 + 24)(25 24) = 49.
Assim, CF = 7.
Observe que a soluo independe da medida dos lados AD e BE.
28. Sequncia de Peri Agrupamos a sequncia em blocos numerados consecutivamente, cada bloco formado pelos termos iguais consecutivos, como mostrado a seguir.
1 , 2, 2 , 3, 3, 3, 4, 4, 4, 4, 5, 5, 5, 5, 5, 1, 1, 1, 1, 1, 1, 2, 2, 2, 2, 2, 2, 2,
|{z}
|{z} | {z } | {z } | {z } |
{z
} |
{z
}

bloco 1

bloco 2

bloco 3

bloco 4

bloco 5

bloco 6

bloco 7

3, 3, 3, 3, 3, 3, 3, 3, 4, 4, 4, 4, 4, 4, 4, 4, 4, 5, 5, 5, 5, 5, 5, 5, 5, 5, 5,
|
{z
} |
{z
} |
{z
}
bloco 8

bloco 9

bloco 10

1, 1, 1, 1, 1, 1, 1, 1, 1, 1, 1, . . . , k, k, k, k, k, k, k, k, k, k, k, k, k, . . . , k, . . .
|
{z
}
|
{z
}
bloco 11

bloco n, com k{1,2,3,4,5}

Observe que a numerao de cada bloco coincide com o nmero de termos que ele
contm: o bloco 1 tem um termo, o bloco 2 tem dois termos, o bloco 3 tem trs termos
e assim por diante, at o bloco n, que tem n termos. A posio na sequncia do ltimo
termo de cada bloco obtida somando todos os nmeros de 1 at o nmero atribudo
ao bloco. Por exemplo, como pode ser contado na enumerao acima,
266

OBMEP 2010

Solues do Nvel 3
o ltimo 3 do bloco 8 o 36 termo, pois 1 + 2 + 3 + 4 + 5 + 6 + 7 + 8 = 36.
o ltimo 1 do bloco 11 o 66 termo, pois 1 + 2 + 3 + + 10 + 11 = 66.

Em geral, o ltimo termo do ensimo bloco est na posio 1 + 2 + 3 + + n. Para


calcular o valor desta soma, lembramos que 1, 2, 3, . . . , n uma progresso aritmtica
de razo 1, termo inicial a1 = 1 e ensimo termo an = n. A soma de seus n primeiros
termos , ento,
1+2+3+4++n =

n(a1 + an )
n(n + 1)
=
.
2
2

Agora precisamos descobrir em qual bloco se encontra o centsimo termo da sequncia.


Supondo que ele esteja no ensimo bloco, sua posio ser, no mximo, a do ltimo
termo deste bloco. Como ele no estar no bloco n + 1, conclumos que n o menor
inteiro tal que 100 21 n(n + 1), ou seja, 200 n(n + 1).

Para determinar esse valor de n, devemos resolver essa inequao e escolher, dentre suas solues, o menor nmero inteiro. Como a expresso bastante simples,
mais fcil resolv-la por tentativa. Fazendo isso, vemos que n = 14. De fato,
13 (13 + 1) = 182 < 200 e 14 (14 + 1) = 210 > 200. Assim, o centsimo termo da
sequncia est no bloco 14. Os nmeros que aparecem nos blocos se repetem de cinco
em cinco, na ordem 1, 2, 3, 4 e 5. Como 14 = 5 2 + 4, o bloco 14 formado pelo
nmero 4. Assim, o centsimo termo da sequncia 4.
Observao: A resoluo acima apresentada da inequao 200 n(n + 1), apesar de
correta, no serviria se o problema pedisse, por exemplo, a determinao do 10 000o
termo da sequncia. Nesse caso, teramos que lidar com a inequao 20 000 n(n+1) e,
claro, achar sua menor soluo inteira por tentativa no parece promissor (a no ser
com muita, muita sorte!). Por isso, vamos resolver a inequao 200 n(n + 1) de uma
maneira que serve em geral.
Comeamos escrevendo 200 n(n + 1) como n2 + n 200 0.

Isso nos leva ao estudo do sinal da funo quadrtica f (x) = x2 +x200, cujo
grfico est ilustrado na figura. As razes de f (x) so

1 1 + 800
x1 =
e
2
1 + 1 + 800
x2 =
.
2

14

13

x2

x1

Observe que x1 negativa e x2 , aproximadamente, igual a 13,6. Como f (x) 0 para


x x1 e x x2 , segue que o n que estamos procurando o menor inteiro que maior
do que ou igual a x2 , ou seja, n = 14.
o
Agora, se quisssemos determinar o 10 000
termo
bastaria repetirmos o


da sequncia,
1
procedimento acima, encontrando x2 = 2 1+ 1 + 80 000 , que , aproximadamente,
igual a 140,9. Logo, n = 141 e o 10 000o termo da sequncia est no 141o bloco. Como
141 = 28 5 + 1, segue que o 10 000o termo 1.

OBMEP 2010

267

Solues do Nvel 3
29. rea em azulejo A figura dada pode ser decomposta
em quatro figuras congruentes figura dada. Para calcular a rea do tringulo sombreado nessa figura, escolhemos como base o lado BC.

E
D
Ento, a altura correspondente AE e, como os azulejos so quadrados com
10 cm de lado, segue que AE = BC = 10 cm. Logo, a rea do tringulo BCE
1
1
base altura = 10 10 = 50 cm2 . Assim, a rea da regio procurada
2
2
4 50 = 200 cm2 .

30. Os cartes de Capitu


Soluo 1: Capitu virou, em primeiro lugar, os 50 cartes pares. Depois disso, ficaram
na mesa os 50 cartes pares com a face amarela para cima e os 50 cartes mpares com
a face vermelha para cima. Ao virar, em seguida, os mltiplos de 3, ela virou apenas
os mltiplos de 3 mpares, que so 3, 9, 15, 21, 27, 33, 39, 45, 51, 57, 63, 69, 75, 81,
87, 93 e 99. Logo, temos 17 mltiplos de 3 que so mpares e Capitu virou para cima
a face amarela de 50 + 17 = 67 cartes. Assim, sobraram com a face vermelha para
cima 100 67 = 33 cartes.

Observao: Nessa soluo, para determinar a quantidade de mltiplos mpares de 3


menores do que 100 foi suficiente escrever esses mltiplos e contar quantos eram. No
entanto, se Capitu tivesse 1 000 cartes (ou mais) esse procedimento seria bastante
trabalhoso, mas, nesse caso, podemos proceder de modo mais geral. Notamos que
os mltiplos mpares de 3 desde 1 at 1 000 formam uma progresso aritmtica, com
primeiro termo a1 = 3, razo r = 6 e o ltimo termo an = 999. Para determinar n
usamos a frmula an = a1 + (n 1)r que, no caso presente, 999 = 3 + (n 1) 6.
Assim, n = 167, ou seja, temos 167 mltiplos mpares de 3 menores do que 1 000.
Soluo 2: Capitu virou, em primeiro lugar, os 50 cartes pares. Depois disso, ficaram
na mesa os 50 cartes pares com a face amarela para cima e os 50 cartes mpares com
a face vermelha para cima. Ao virar, em seguida, os mltiplos de 3, Capitu procedeu
como segue.
Entre os cartes pares ela virou os que eram tambm mltiplos de 3. Um nmero
que mltiplo de 2 e de 3 tambm mltiplo de 6.
Como
100 = 16 6 + 4, conclumos que Capitu virou 16 cartes entre os cartes pares.
Esses cartes voltaram a ficar com a face vermelha para cima, ficando os outros
34 com a face amarela para cima.
Entre os cartes mpares, como 100 = 33 3 + 1, segue que o nmero total
de cartes (pares e mpares) mltiplos de 3 33. Como vimos acima, entre
estes cartes, 16 so pares, logo 17 so mpares. Assim, Capitu virou 17 cartes
mpares, e esses cartes passaram a ter a face amarela para cima, enquanto que
os outros 33 continuaram com a face vermelha para cima.
31. Enchendo o tanque No que segue, todas as medidas de volume esto dadas em
cm3 .
O volume V do balde dado pela frmula habitual do volume de um cilindro, ou seja,
V = rea da base altura. A base do balde um crculo de 30 cm de dimetro; seu raio,
268

OBMEP 2010

Solues do Nvel 3
ento, mede r = 15 cm e sua rea r 2 = 225 cm2 . Logo, V = 48 225 = 10 800 .
A cada viagem, o volume de gua que o homem coloca no balde 4/5 de V e, desse
volume, ele perde 10%. Portanto, resta no balde 90% de 4/5 de V, ou seja,
9
4
18
V =
V = 0,72 V = 0,72 10 800 = 7 776 .
10 5
25
Essa quantidade B = 7 776 de gua a que ele efetivamente coloca no tanque em
cada viagem. O volume de 3/4 do tanque T = 34 300 36 50 = 405 000. Logo, o
nmero de baldes necessrios para atingir esse volume
405 000 405 000
625
=
=
.
B
7 776
12
625
625
Usando a aproximao 3,14 para o nmero , obtemos

16,587.
12
12 3,14
Assim, o homem necessitar de 16 baldes, mais 0,587 de um balde, e conclumos que
ele dever fazer 17 viagens.
Observao: Acima usamos uma aproximao para o valor de . importante entender o que isso significa. Como sabemos, um nmero irracional e sua expanso
decimal infinita e no peridica. O valor aproximado de , com 31 casas decimais,
3,1415926535897932384626433832795 (o smbolo quer dizer aproximadamente). Por que, ento, no usar 3,142 ou 3,1416 para resolver nosso
problema, em vez de 3,14? Para discutir isso, vamos a um exemplo.
Suponhamos que voc tenha um balde cilndrico com raio da base medindo 1 m e
altura 1 m, e uma caixa de gua de volume de, exatamente 3,141 m3 . O balde deve ser
enchido em uma fonte. Quantas viagens fonte sero necessrias para encher a caixa,
supondo que o volume de gua de cada balde seja integralmente transferido para a
caixa?

Usando a aproximao 3,14, obtemos 3,14 m3 para o volume do balde. Como


volume do tanque
3,141
maior do que 1 (e, claro, menor do que 2), conclumos que
volume do balde
3,14
sero necessrias duas viagens fonte para encher a caixa de gua.
Vamos, agora, usar a aproximao 3,1416. Aqui calculamos o volume do balde
do tanque
3,141
e obtemos 3,1415 m3 . Ento, volume
3,1416
menor do que 1, e conclumos,
volume do balde
assim, que basta uma viagem fonte para encher o balde, um resultado bem diferente
do anterior!
Deve ficar claro com esse exemplo que a escolha inicial de uma aproximao pode
influenciar fortemente o resultado final. Nesse caso, dizemos que as condies do
problema so sensveis aproximao. No nosso problema original de encher o tanque,
os dados iniciais no eram sensveis aproximao usada para , o que pode ser
verificado imediatamente repetindo a resoluo dada com 3,142 ou 3,1416.
Em ambos os casos, obtm-se o resultado de 17 viagens.
Em geral, os problemas desse tipo propostos em livros nos ensinos fundamental e mdio
so enunciados de modo pouco sensvel aproximao.
Isto justifica parcialmente o

uso de = 3,14 bem como o de, por exemplo, 2 = 1,41


(curiosidade: 2 1,4142135623730950488016887242097).
Observamos, tambm, que poucas casas decimais facilitam as contas, em particular
quando no se usam mquinas de calcular. Seria impossvel, na prtica, trabalhar
manualmente com a aproximao de 31 casas que demos para no incio desta conversa.
OBMEP 2010

269

Solues do Nvel 3
O tratamento de problemas de aproximao feito atravs de desigualdades. Infelizmente, tempo e espao no permitem que abordemos esse tpico com mais detalhes no
momento, mas esperamos ter despertado sua curiosidade para o assunto.
32. Fator primo A decomposio de 2 006 em fatores primos 2 006 = 2 17 59.
Assim, o maior fator primo de 2 006 59.
33. Altura de salrio A opo correta (d).
O enunciado diz que 1 real = 275 107 cruzados. O salrio de Joo 640 reais, o que
equivalente a 640 275 107 = 176 000 107 = 176 1010 cruzados. O nmero
de pilhas de cem notas que se pode fazer com essa quantidade de notas de 1 cruzado
176 1010 /102 = 176 108 . Como cada uma destas pilhas tem 1,5 cm de altura, a
altura de todas elas 1,5 176 108 = 264 108 cm.

Agora lembramos que 1 km = 1 000 m = 103 m e que 1 m = 100 cm = 102 cm, donde
1 km = 103 102 = 105 cm. Assim, a pilha de 264 108 cm tem 264 108 /105 =
264 103 = 264 000 km de altura.
34. S bala A opo correta (c).
A primeira bala pode ser de qualquer sabor. Para fixar as ideias, suponhamos que
seja de banana. Depois que essa bala retirada, sobram 1 002 + 1 001 balas na caixa
no nosso caso, 1 002 de ma e 1 001 de banana. A probabilidade q de que a segunda
bala seja diferente (no nosso exemplo, de ma) q = 1 002/2 003. A probabilidade p
de que a segunda bala seja igual (no nosso exemplo, de banana) p = 1 001/2 003. A
diferena q p , portanto,
qp=

1 002 1 001
1

=
.
2 003 2 003 2 003

35. Distncia ao centro A opo correta (e).


Os pontos que esto a 6 cm de distncia do ponto P
formam uma circunferncia de centro P e raio R = 6
cm. Se d denota a diagonal do quadrado, do Teorema
de Pitgoras temos

d = 102 + 102 = 2 102 = 10 2 .

L/2 R
D/2

A circunferncia de raio L/2 = 5 cm tangencia o quadrado em quatro pontos.


A circunferncia de raio D/2 toca o quadradoem quatro pontos, a saber, os vrtices
do quadrado. Temos L = 10, R = 6 e D = 10 2 , portanto

5 < |{z}
6 < 5|{z}2 .
|{z}
L/2

D/2

(Observe que 1,2 < 2 , 51,2 < 5 2 e, portanto, 6 < 5 2 ). Logo,


a circunferncia

de raio R = 6 est entre as duas circunferncias de raios 5 e 5 2 . Assim, ela corta


o quadrado em oito pontos.

270

OBMEP 2010

Solues do Nvel 3
36. Potncias e potncias A opo correta (e).
Soluo 1: Observamos que os termos do lado direito da equao dada podem ser
escritos como potncias de 2. De fato, 4x = (22 )x = 22x e 64 = 26 . Desse modo, a
equao se torna 2(2x ) = 22x +23 . Temos, ento, 2(22x )22x = 26 , donde 22x (21) = 26 ,
ou seja, 22x = 26 . Assim, 2x = 6 e segue que x = 3.

Soluo 2: 4x + 4x = 2(4x ) = 2 22x = 4x + 43 , portanto, 4x = 43 e segue que x = 3.

37. Um raio de luz A opo correta (b).

Vamos acompanhar o trajeto do raio de luz a partir do ponto S. Para isso, lembramos
a propriedade bsica da reflexo de um raio de luz num espelho: o ngulo de reflexo
igual ao ngulo de incidncia. Por exemplo, na figura dada, os ngulos a e b so iguais,
bem como os ngulos d e e. Observe que, na figura, as paralelas AS e BV so cortadas
pela transversal AB.
Assim,
a = 30 = b,

a + b + c = 180 ,
logo c = 120 e

a
c

c+d = 180 , logo


d = 60 = e.

f
e

30

Como a soma dos ngulos internos do tringulo BCV 180 , segue que f = 90 .
Isso quer dizer que o nosso raio de luz, ao atingir C, ser refletido sobre si mesmo e
far ento o caminho inverso, C B A S. Desse modo, o trajeto completo do
raio ser
S A B C B A S.
O comprimento desse trajeto do raio desde S at retornar a S duas vezes a soma dos
comprimentos dos segmentos AS, AB e BC. dado que AS = 1 m, portanto, resta
calcular AB e BC. Para isso, olhamos para o tringulo ABC. Ele um tringulo
retngulo com ngulos de 30 e 60 . Sabemos que num tal tringulo o cateto oposto
ao ngulo de 30 tem comprimento igual metade do comprimento da hipotenusa
1
(exerccio). No nosso caso, temos BC = AB.
2
Notamos, agora, que os tringulos ABC e CBF so congruentes, pois so tringulos retngulos (f = 90 ) com ngulos iguais (b = 30 ) e um cateto comum (BC), o que
1
1
1
nos mostra que AC = AV = m. Pondo AB = x, temos BC = x e o Teorema
2
2
2
1 2
1 2
2
de Pitgoras, aplicado ao tringulo ABC, nos d x =
+ x . Simplificando,
2
2

3 2 1
obtemos x = , donde x = 1/ 3 = 3 /3.
4
4
Desse modo, obtemos o comprimento do trajeto do raio de luz, como segue.

3
1 3
2(SA + AB + BC) = 2 1 +
+
= 2 + 3 m.
3
2 3


OBMEP 2010

271

Solues do Nvel 3
38. Diferena de quadrados Usando a fatorao x2 y 2 = (x y)(x + y) com
x = 666 666 666 e y = 333 333 333, vemos que x y = y e x + y = 999 999 999,
portanto,
666 666 6662 333 333 3332 =
=
=
=

333 333 333 999 999 999


333 333 333 (1 000 000 000 1)
333 333 333 000 000 000 333 333 333
333 333 332 666 666 667.

39. Escada de nmero A opo correta (e).


Usando a regra dada, preenchemos as casas vazias a
partir da segunda linha a contar de baixo e obtemos
a figura. Logo,

42
13+x 11+2x

(13 + x) + (11 + 2x) = 42


e, portanto, 24 + 3x = 42, ou seja, x = 6.

5+x x+6

8
3

40. Diferena de potncias A opo correta (c).


Soluo 1: O algarismo final de 9 867 3 o mesmo que o de 73 = 343, isto , 3. O
algarismo final de 9 867 2 o mesmo que o de 72 = 49, isto , 9. Se de um nmero
terminado em 3 subtramos outro terminado em 9, o algarismo final do resultado 4.
Observao: Observe que o algarismo das unidades da diferena 9 867 3 9 867 2 igual
ao algarismo das unidades de (73 72 ).
Soluo 2: n3 n2 = n2 (n 1), com n2 = 9 867 2 terminando em 9 e
n 1 = 9 866 em 6. Como 9 6 = 54, o algarismo final de n2 (n 1) 4.
41. Parbola girada A opo correta (e).

y
Uma rotao de 180 pode ser visualizada
B=(r,s)
r
como uma meia-volta. Aqui temos uma
A=(m,n)
meia-volta em torno da origem. A figura ilusn
tra o que uma meia-volta faz com as coordenadas dos pontos do plano. Por exemplo, o
-s
-m s
x
m
ponto A o resultado da meia-volta aplicada ao ponto A; em outras palavras, A
-n
A=(-m,-n)
onde o ponto A vai parar aps a meia-volta.
Do mesmo modo, B onde B vai parar aps
-r
B=(-r,-s)
a meia-volta.
fcil ver que na passagem de A para A as coordenadas trocam de sinal. Desse
modo, vemos que uma meia-volta em torno da origem leva um ponto qualquer (x, y)
no ponto (x, y). Assim, (a, b) pertence nova parbola se, e somente se, (a, b)
pertence parbola y = x2 5x + 9 original, ou seja, se b = a2 + 5a + 9 ou, ainda,
b = a2 5a 9. Logo, a equao da nova parbola y = x2 5x 9.
42. Logotipo Seja r o raio dos quatro crculos iguais.
272

OBMEP 2010

Solues do Nvel 3
Ligando os centros A e B de dois desses crculos ao
centro O dos crculos concntricos, obtemos o tringulo OAB, como na figura. Lembrando que a reta
que une os centros de dois crculos tangentes passa pelo
ponto de tangncia, vemos que OA = OB = 1 + r e
AB = 2r, tudo em cm. O tringulo OAB retngulo
em O e o Teorema de Pitgoras d

(2r)2 = (1 + r)2 + (1 + r)2 .


Logo, 4r 2 = 2(1 + r)2 , ou 2r 2 = (1 + r)2 , do que tiramos r 2 2r 1 = 0. Assim,


1
r = 2 8 = 1 2 . Como o raio r positivo, obtemos r = 1 + 2 . Segue que
2

o raio do crculo maior mede 1 + 2r = 3 + 2 2 cm.


43. Padeiro cansado A opo correta (d).
Seja x a quantidade de farinha, em quilos, de que o padeiro dispe. Trabalhando
sozinho, ele usaria x/6 quilos de farinha em uma hora. Trabalhando com seu ajudante,
eles usariam x/2 quilos de farinha em uma hora. Seja t o tempo, em horas, que o
padeiro trabalhou sozinho. Como a farinha acaba em 150 minutos, ou seja, em 2,5
horas (2 horas e 30 minutos), o tempo que ele trabalhou com seu ajudante foi 2,5 t
horas. Logo, a quantidade de farinha gasta durante o tempo que o padeiro trabalhou
sozinho foi de (x/6) t e a quantidade gasta durante o tempo que o padeiro trabalhou
com seu ajudante foi de (x/2) (2,5 t). Como
x

z
}|
{
quantidade total
de farinha

x t
6

x (2,5t)
2

}|
{
z
quantidade de farinha gasta
pelo padeiro
trabalhando sozinho

}|
{
z
quantidade de farinha gasta
pelo padeiro trabalhando
com o ajudante

temos x = 61 x t + 12 x (2,5 t) . A quantidade x de farinha que o padeiro tinha inicialmente era no nula. Logo, podemos dividir ambos os membros por x, encontrando
1 = 16 t + 21 (2,5 t) e, portanto, t = 0,75 horas, ou seja, o padeiro trabalhou sozinho
durante 45 minutos.

44. Muitas diagonais Num poliedro qualquer, dois vrtices distintos determinam uma
diagonal apenas no caso em que no estejam numa mesma face.
No caso do prisma hexagonal, vemos na figura que o vrtice v no forma uma diagonal
com os vrtices marcados com . Levando o prprio v em conta, vemos que v no forma
uma diagonal com exatamente nove vrtices. Como o prisma tem doze vrtices, segue
que v forma uma diagonal com exatamente 129 = 3 vrtices. O mesmo raciocnio vale
para qualquer vrtice, e conclumos que, de cada vrtice do prisma, partem exatamente
trs diagonais. Como a diagonal que parte de um vrtice v para o vrtice w a mesma
1
que parte de w para v, segue que o nmero de diagonais 12 3 = 18.
2
OBMEP 2010

273

Solues do Nvel 3

X
X
X

X
X

V
X

*
*

X
X

Seja V um vrtice do poliedro. Observando a figura, vemos que V no forma uma


diagonal com exatamente quatorze vrtices, os treze marcados com X e mais o prprio
V. Como o poliedro tem vinte e quatro vrtices no total, sobram 24 14 = 10 vrtices,
com os quais V forma uma diagonal. Logo, o nmero de diagonais desse poliedro
1
24 10 = 120.
2
45. Promoo de sabonete A opo correta (d).
Pela promoo, quem levar 2 unidades paga pelo preo de 1,5 unidade, logo quem levar
4 unidades paga pelo preo de 3 unidades, ou seja, leva quatro e paga trs.
46. Qual o ngulo? A opo correta (b).
Como ABC e DEF so tringulos equilteros, cada um de seus ngulos internos
mede 60 . No tringulo AGD temos
b = 180 75 60 = 45
GAD

b = 180 65 60 = 55 .
e GDA

b = 180 45 55 = 80 . Logo, no tringulo CGH, temos


Portanto, AGD
x + 80 + 60 = 180 , donde x = 40 .

47. Caixa de papelo A opo correta (b).


A figura mostra as dobras que sero feitas para montar a caixa, que ter as dimenses seguintes: 20 cm de largura,
15 cm de comprimento e 10 cm de altura. Logo, seu volume ser de
20 15 10 = 3 000 cm3 .
48. Soma de vizinhos A opo correta (b).
Seja x o primeiro termo da sequncia. Como o segundo termo 1 e, a partir do terceiro,
cada termo a soma dos dois anteriores, temos que
o terceiro termo 1 + x;
274

OBMEP 2010

Solues do Nvel 3
o quarto termo 1 + (1 + x) = 2 + x;
o quinto termo (1 + x) + (2 + x) = 3 + 2x;
o sexto termo (2 + x) + (3 + 2x) = 5 + 3x.
Como o quinto termo 2 005, temos 3 + 2x = 2 005, donde x = 1 001. Logo, o sexto
termo da sequncia 5 + 3 1 001 = 3 008.
49. Algarismos crescentes A opo correta (d).
Os nmeros em questo, com
dois algarismos, so 12, 23, 34, 45, 56, 67, 78 e 89 (8 nmeros);

trs algarismos, so 123, 234, 345, 456, 567, 678 e 789 (7 nmeros);
quatro algarismos, so 1 234, 2 345, 3 456, . . . , 6 789 (6 nmeros);
por fim, com cinco algarismos, somente 12 345,
num total de 8 + 7 + 6 + 1 = 22 nmeros.
50. Bloco girante A opo correta (b).
De acordo com a figura, podemos concluir que as
dimenses das faces X, Y e Z so 2, 3 e 6 cm2 ,
respectivamente. A seguir, indicaremos os movimentos feitos pelo bloco e as faces que entram em
contato com os quadradinhos em cada etapa. Lembre que giramos o bloco cinco vezes.
As figuras a seguir mostram os quadradinhos do tabuleiro que ficam em contato com
cada uma das trs faces do bloco, desde a posio inicial at a final, aps a ltima
rotao.

OBMEP 2010

275

Solues do Nvel 3
Alguns quadradinhos entram em contato com as faces mais de uma vez, conforme
figura a seguir, que mostra todos os quadradinhos que tiveram contato com as faces
do bloco desde a posio inicial at a ltima rotao.

Contando nesta ltima figura, vemos que o bloco esteve em contato com 19 quadradinhos do tabuleiro.
51. Iterando um ponto A opo correta (d).
1
4

x
f (x)

A partir da tabela

2
1

3
3

4
5

5
2

obtemos

f (4) = 5, f (f (4)) = f (5) = 2, f (f (f (4))) = f (f (5)) = f (2) = 1 e


|{z}
|{z}
|{z}
5

f (f (f (f (4)))) = f (f (f (f (4)))) = f (f (f (5))) = f (f (2)) = f (1) = 4.


|{z}
|{z}
|{z}
| {z }
4 vezes

Como 2 004 mltiplo de 4, segue que f (f (f (f (. . . f (4) . . .)))) = 4. O diagrama a


{z
}
|
2 004 vezes

seguir ilustra essa afirmao.


f

z}|{ z}|{ z}|{ z}|{ z}|{ z}|{ z}|{ z}|{ z}|{ z}|{ z}|{ z}|{
4| 5 {z
2 1 4} | 5 2{z 1 4} | 5 2{z 1 4}
4 vezes

8 vezes

12 vezes

z}|{ z}|{ z}|{ z}|{


. . . | 5 2{z 1 4}
2 004 vezes

52. Esmeralda e o 21 Primeiro vamos listar os nmeros que tm o agrupamento 21


no meio de sua representao decimal.
21, 121, 221, . . . , 921, num total de 10 nmeros.

210, 211, . . . , 219, num total de 10 nmeros.

Tambm devemos contar os agrupamentos 21 obtidos a partir de um par de nmeros


consecutivos tal que o primeiro termina com 2 e o segundo comea com 1, que so os
11 casos seguintes.
12 13, 120 103, 112 113, 122 123, 132 133, 142 143,
152 153, 162 163, 172 173, 182 183, 192 193.

Assim, temos um total de 20 + 11 = 31 agrupamentos 21 nesse nmero.


276

OBMEP 2010

Solues do Nvel 3
53. Muitos fatores A opo correta (d).
Cada um dos fatores uma diferena de quadrados, isto , a2 b2 , em que a = 1 e
b = 1/c2 = (1/c)2 . Usando a fatorao a2 b2 = (a b)(a + b), obtemos



1 
1
1 
1 
1
1
1
1
4
9
16
225


1 
1 
1
1 
= 1 2 1 2 1 2 1 2
2
3
4
15








1
1
1
1
1
1 
1
1
= 1
1+
1
1+
1
1+
1
1+
2
2
3
3
4
4
15
15
1 3 2 4 3 5
14 16
1 16
8
= ...
=
.
=
2 |2 {z 3} |3 {z 4} |4 {z }
15
15
2
15
15
| {z }
1

54. Falta um ngulo A opo correta (a).

Os ngulos internos do quadriltero dado so 50 , 180 30 = 150 , e


180 40 = 140. Como a soma dos ngulos internos de um quadriltero 360 ,
temos 50 + 150 + + 140 = 360 , donde = 20 .
55. Soma de distncias A opo correta (e).
Temos |z x| = 3,7 (1) = 4,7 e |w x| = 9,3 (1) = 10,3. Logo,
|z x| + |w x| = 4,7 + 10,3 = 15.
56. Espiral do Artur A opo correta (d).
A figura mostra que a espiral constituda por segmentos cujos comprimentos formam uma sequncia finita da forma 1, 1, 2, 2, 3, 3, 4, 4, . . . , n, n (se os dois ltimos segmentos da espiral tm o mesmo comprimento) ou, ento, da forma 1, 1, 2, 2, 3, 3, 4, 4, . . . ,
n, n, n + 1 (se os dois ltimos segmentos da espiral tm comprimentos diferentes).

A soma dos k primeiros nmeros naturais dada por


1+2+3++k =
OBMEP 2010

k(k + 1)
2
277

Solues do Nvel 3
e o comprimento total da espiral 4 m = 400 cm. Portanto,
2

n(n + 1)
= 1 + 1 + 2 + 2 + 3 + 3 + + n + n = 400
2

ou, ento,
2

n(n + 1)
+ n + 1 = 1 + 1 + 2 + 2 + + n + n + n + 1 = 400,
2

de modo que n(n + 1) = 400 ou (n + 1)2 = 400.


Entretanto, no existem dois nmeros naturais consecutivos cujo produto seja 400,
isto , a equao n(n + 1) = 400 no tem soluo. Assim, (n + 1)2 = 400, de modo
que n + 1 = 20. Portanto, o ltimo segmento da espiral tem 20 cm e o penltimo
19 cm. Os comprimentos dos segmentos da espiral formam a sequncia de nmeros
1, 1, 2, 2, 3, 3, 4, 4, . . . , 19, 19, 20. Assim, so 19 2 + 1 = 39 segmentos. Como sete j
foram traados, falta traar 32.
57. Quais so os ngulos? A opo correta (d).
b = D AC
b = y (ngulos alternos internos) e B DA
b = C BD
b = y (simetria).
Temos B CA
Seja O o ponto de interseo das duas diagonais.
Traando o segmento MN paralelo aos lados AD e BC
A....................................................................................................................................D
..
...... ..
b = B CA
b = y e N OD
b =
.... .............. y
....... ....
do retngulo, obtemos C ON
.......
...
.......
.......
...
.......
.......
...
...
.......
.
.
.......
.
.
.
.
.
.
.
..
.
b = y. Logo,
B DA
M..................................................................................................................................................................................... N
O

.......
...
..
......
.......
...
.......
...
.......
......
.
.......
..
.......
....... ....
... .............
.. .
............................................................................................................................................

b = C ON
b + N OD
b = 2y.
x = C OD

58. Raiz menor A equao


j foi dada
na forma fatorada a(x m)(x n) = 0, logo

suas razes so m = 3 5 e n = 5 3 . Devemos apenas decidir qual delas a maior.


Isso pode ser feito de duas maneiras, pelo menos.
2
2
Podemos elevar m e n ao quadrado,
obtendo
m
= 9 5 = 45

e n = 25 3 = 75.
Como 45 < 75, resulta que 3 5 = m = 45 < 75 < n = 5 3 .

Tambm podemos observar que 5 < 2,3 e 1,7 < 3 , portanto,

m < 3 2,3 = 6,9 < 8,5 = 5 1,7 < n.


3 5

6,9

8,5

5 3

59. Comparando reas A opo correta (c).


Os raios dos trs discos menores so 1, 2 e 2 e o do disco
maior 3. Denotemos por b a rea em branco. Ento
v = 1 +4 +4 = 9 b
e w = 9 b, ou seja, v = w.
278

OBMEP 2010

Solues do Nvel 3
60. Menor raiz A opo correta (b).
1o Caso: x 1. Nesse caso, x 1 0 e, portanto, |x 1| = x 1. A equao
dada toma a forma (x 1)/x2 = 6, ou 6x2 x + 1 = 0. Essa equao no tem
razes reais porque = (1)2 4 6 1 = 1 24 negativo. Logo, no temos
solues x maiores do que ou iguais a 1.
2o Caso: x < 1. Nesse caso, x 1 < 0 e, portanto, |x 1| = (x 1) = 1 x.
A equao dada toma a forma (1 x)/x2 = 6, ou 6x2 + x 1 = 0. Essa equao
tem as razes
p

1 1 4 6 (1)
1 25
1 5
x=
=
=
,
26
12
12
1
1
ou seja, x = e x = .
2
3
Com base nesses dois casos, conclumos que nossa equao tem apenas duas solues,
1
1 1
e . Logo, a menor soluo da equao .
2 3
2
61. Toalha redonda A opo correta (d).
Para que a toalha cubra inteiramente a mesa e que tenha o menor dimetro possvel, o quadrado deve estar inscrito no crculo. A diagonal
do quadrado o dimetro do crculo,
logo, pelo Teorema de Pitgoras,
temos d2 = 12 + 12 , ou seja, d = 2 .

1
d

62. Solues reais A opo correta (c).


Para que um produto de trs fatores seja negativo, devemos
ter dois fatores positivos e um fator negativo, ou os trs negativos. As possibilidades so as seguintes.
1) (x 1) (x 2) (x 3) .
| {z } | {z } | {z }
+

Isso equivale a x > 1, x > 2 e x < 3, ou seja,


2 < x < 3.

2) (x 1) (x 2) (x 3) .
| {z } | {z } | {z }
+

Isso equivale a x > 1, x < 2 e x > 3, o que no


possvel. Logo, no pode ocorrer esse caso.

3) (x 1) (x 2) (x 3) .
| {z } | {z } | {z }

Isso equivale a x < 1, x > 2 e x > 3, o que no


possvel. Logo, no pode ocorrer esse caso.

OBMEP 2010

279

Solues do Nvel 3
4) (x 1) (x 2) (x 3) .
| {z } | {z } | {z }

Isso equivale a x < 1, x < 2 e x < 3, ou seja,


x < 1.

Logo, os nicos reais x satisfazendo (x 1)(x 2)(x 3) < 0 so os reais x tais que
x < 1 ou 2 < x < 3. Assim, a unio de intervalos (0, 1) (2, 3) o conjunto formado
por todos os reais positivos de x tais que (x 1)(x 2)(x 3) < 0.
63. Cossenos crescentes De acordo com a definio de cosseno, temos
cos 25 = 1/(OM), cos 41 = 1/(ON) e cos 58 = 1/(OB). Na figura, vemos que
OM < ON < OB. Logo, cos 58 < cos 41 < cos 25 .
64. Central telefnica A opo correta (e).
Existem dois tipos de ramais que podem estar sendo usados. Temos os ramais com
os dois algarismos iguais (00, 11, 22, 33, 44, 55, 66, 77, 88 e 99), num total 10, e
os com
os dois algarismos distintos. Nesse caso, temos 10 9 = 90 nmeros, e metade
deles podem ser usados.
Logo, o maior nmero possvel de ramais em uso 10 + 45 = 55.
65. Horrio de avio Seja d a distncia entre as duas cidades e seja h o horrio de
partida comum do nibus, do trem e do avio. Como distncia = velocidade tempo,
temos d = 100(20h) e d = 300(14h). Logo, 100(20h) = 300(14h), donde
h = 11. Portanto, a distncia entre as duas cidades d = 100 (20 11) = 900 km.
Assim, o avio gasta 1 hora da cidade A cidade B e, portanto, o avio chega s 12
horas.
66. Discos de papelo A opo correta (c).
Lembre que a rea de um crculo de raio r r 2 . Se r o raio dos crculos da figura,
ento a rea no aproveitada a rea do quadrado, que dada por 10 10 = 100 cm2 ,
menos a soma das reas dos nove crculos, que 9 r 2 cm2 . Ocorre que o raio de
cada crculo r = 5/3 cm, j que os dimetros de trs desses crculos somam um lado
de 10 cm da folha de papelo. Assim, a rea no aproveitada dada por
100 9

 5 2
3

= 100 25 .

Usando a aproximao 3,14, resulta que a rea no aproveitada mede


100 25 100 25 3,14 = 21, 5 cm2 .
280

OBMEP 2010

Solues do Nvel 3
67. Afirmaes absolutas
(a) | 108| = 108 > 100, verdadeira.

(b) |2 9| = | 7| = 7 = 9 2, verdadeira.

(c) | 6a| = | 6| |a| = 6 |a|, verdadeira.

(d) |5 13| = | 8| = 8 6= 8 = 5 13 = |5| |13|, falsa.

(e) |a2 + 5| = a2 + 5, verdadeira, pois a2 + 5 > 0, para qualquer valor de a.

68. Frao radical A opo correta (e).

x
x
Elevando ao quadrado ambos os membros de = 5, obtemos = 25. Assim,
y
y
 1
x+y
1 x+y
1 x y  1 x
=
=
+
=
+ 1 = (25 + 1) = 13.
2y
2
x
2
y y
2
y
2

69. rea de tringulo A opo correta (d).


Os tringulos T KR e GRS so proporcionais por serem tringulos retngulos
RS
GS
com um ngulo agudo igual. Logo, temos
=
. Como GS = T K, segue que
T K T R
(T K)2 = RS T R = 2 6 = 12, ou seja, T K = 2 3. Tambm
KG = T R + RS = 6 + 2 = 8.
Assim, a rea do tringulo KGR mede


1
1
KG

T
K
=
8

2
3
=
8
3.
2 |{z} |{z} 2
base

altura

70. Pares de inteiros A opo correta (c).


Temos
13 =

a + 1b
=
1
+b
a

ab+1
b
1+ab
a

(ab + 1) a
a
= .
(1 + ab) b
b

Logo, a = 13 b. Como a + b 100, segue que 14 b 100 e, portanto, b 7,14. Como b


inteiro, devemos ter b 7. Logo, os pares so em nmero de sete, a saber,
(13, 1), (26, 2), (39, 3), (52, 4), (65, 5), (78, 6) e (91, 7).
71. Qual a soma? A opo correta (c).
1o Caso: Se x 0, ento |x| = x e, pela primeira equao, temos x + (x) + y = 5,
ou seja, y = 5. Substituindo esse valor na segunda equao, obtemos x = 6, o que no
possvel, pois estamos supondo x 0. Logo, no h soluo nesse caso x 0.
2o Caso: Se y 0, ento |y| = y e, pela segunda equao, temos
x + y y = 6,
OBMEP 2010

281

Solues do Nvel 3
ou seja, x = 6. Substituindo esse valor na primeira equao, obtemos y = 7, o que
no possvel, pois estamos supondo y 0.
3o Caso: Se x > 0 e y < 0, ento |x| = x e |y| = y. Pela primeira equao temos
2x + y = 5 e, pela segunda, x 2y = 6. Multiplicando 2x + y = 5 por 2 e somando com
x 2y = 6, obtemos 5x = 16, de modo que x = 16/5 e segue que y = 5 2x = 7/5.
Assim, x + y = 9/5.
72. Crculo intermedirio A opo correta (a).
A rea do maior crculo 132 = 169 e a do menor 52 = 25 , que tambm a
rea do maior anel. Seja r o raio do crculo intermedirio. Ento, a rea do maior anel
169 r 2 . Logo, 169 r 2 = 25 , ou seja, r 2 = 169 25 = 144 , donde
r 2 = 144 e r = 12 cm.
73. Fraes incompletas
(a) Observe que 21 6 = 126. Portanto, o numerador 21 foi multiplicado por 6 para
tambm
obter o numerador 126 do primeiro quociente. Logo, o denominador
foi multiplicado por 6 para dar o denominador 8 do primeiro quociente. O nico
nmero da forma 8 que divisvel por 6 84, e 84 6 = 18. Podemos, ento,
completar as fraes, obtendo
21 6
126
21
=
.
=
84
18 6
18
(b) Temos 4/5 = 0,8 e queremos ter
8 = 0,8 33 . Observe que 33 deve ser
mltiplo de 5, portanto, s pode ser 330 ou 335. Entretanto,
0,8 330 = 264 6=
Assim, 33 = 335, com 335 = 5 67, e
maneira de completar a frao,

8.

8 = 268 = 4 67. Logo, s existe uma

2 68
268 67
4
=
= .
335
335 67
5
74. Tringulos impossveis
Figura 1: No est correta, porque a soma dos ngulos internos do tringulo no
180 . De fato, 74 + 42 + 42 = 158 < 180.
Figura 2: No est correta, porque o comprimento dos lados no satisfaz o Teorema de Pitgoras: 182 = 324 6= 369 = 144 + 225 = 122 + 152 . Logo, o tringulo
no pode ser retngulo.
Figura 3: No est correta, porque um dos lados de um tringulo no pode ser
menor do que a soma dos outros dois: 15 > 6 + 8.

282

OBMEP 2010

Solues do Nvel 3
75. Razo de reas A opo correta (b).
Como o arco de 60 do crculo I tem o mesmo comprimento que o arco de 45 no crculo
II, conclumos que o raio do crculo I menor do que o do crculo II. Denotemos por r
e R os raios dos crculos I e II, respectivamente.
No crculo I, o comprimento do arco de 60 igual a 1/6
de seu comprimento, ou seja 2 r/6. Analogamente, no
crculo II, o comprimento do arco de 45 igual a 1/8
de seu comprimento, ou seja, 2 R/8. Logo, 2 r/6 =
2 R/8, ou seja, r/R = 6/8 = 3/4. Finalmente, temos
 r 2  3 2
rea do crculo I
9
r2
=
=
=
=
.
2
rea do crculo II
R
R
4
16
76. Inequao errada A opo correta (c).
Nessa questo usaremos as propriedades de desigualdades seguintes. Podemos somar
o mesmo nmero a ambos os membros de uma desigualdade sem alterar seu sentido.
Podemos multiplicar ambos os membros de uma desigualdade por um nmero positivo
sem alterar seu sentido. Assim,

x + z > y + z (somando z qualquer a ambos os lados)
x>y
xz > yz (muliplicando por z > 0 em ambos os lados)
Logo, (a) e (b) esto corretas, pois foi somado z e z a ambos os membros, bem
como (d) e (e), pois ambos os membros foram multiplicados por 1/z 2 e z 2 , ambos
positivos, j que z 6= 0. A opo (c) falsa, porque z pode ser negativo. Por exemplo,
se x = 5, y = 3 e z = 2, temos 5 > 3 e, no entanto,
5| {z
2} = 10 < 6 = 3 (2) .
| {z }
xz

77. Equaes geomtricas

yz

(a) |x 5| = 2 significa que a distncia de x a 5 2. Logo, as razes so 3 e 7.


2

(b) |x + 3| = 1 significa que a distncia de x a 3 1. Logo, as razes so 4 e 2.


1

(c) Denotando y = 3x, a equao toma a forma |y 7| = 9, o que equivale a dizer


que a distncia de y a 7 9. Logo, as razes so 2 e 16. Como y = 3x, temos
3x = 2 e 3x = 16, de modo que as razes da equao original so x = 23 e
x = 16
.
3
9

9
2

OBMEP 2010

16

283

Solues do Nvel 3
(d) As razes da equao |x + 2| = |x 5| so os nmeros equidistantes de 2 e de
5. No entanto, s pode haver um nico nmero equidistante de dois outros, e que
fica no meio do caminho entre os dois. Como a distncia de 5 a 2 7, o ponto
equidistante deve distar 3,5 de 2 e de 7. Logo, a soluo x = 1,5.
3,5

1,5

3,5

78. Pista circular A opo correta (c).


Vamos marcar os quatro pontos a partir de A. Como a pista
mede 20 km, o comprimento de cada um dos quatro quadrantes 5 km e podemos, ento, marcar os pontos. Como
367 = 18 20 + 7, o carro deu 18 voltas completas e percorreu
mais 7 km a partir de A. Logo, ele passa 2 km de B e para a
1 km de C. Portanto, C o ponto mais prximo.

79. Maior comprimento A opo correta (e).


Note que
AE a hipotenusa de um tringulo de catetos com 5 cm e 9 cm;
CF a hipotenusa de um tringulo de catetos com 2 cm e 4 cm;

AC a hipotenusa de um tringulo de catetos com 3 cm e 4 cm;


FD a hipotenusa de um tringulo de catetos com 2 cm e 9 cm;
CE a hipotenusa de um tringulo de catetos com 2 cm e 5 cm.
Usando o Teorema de Pitgoras calculamos essas hipotenusas.

AE = 52 + 92 = 106 10,3

CF = 22 + 42 = 20 4,47

CF = 32 + 42 = 25 = 5

FD = 22 + 92 = 85 9,22

CE = 22 + 52 = 29 5,39
Como CD = 5 cm, obtemos AE 10,3, CD + CF 5 + 4,47 = 9,47,
AC + CF 5 + 4,47 = 9,47, FD 9,22 e AC + CE 5 + 5,39 = 10,39. Logo,
o maior segmento AC + CE, que mede 10,39 cm.
80. Desigualdade entre inteiros A opo correta (d).
1
2
Se 3x2 < 14, ento 3x2 > 14, ou x2 > 14 = 4 . Como estamos olhando apenas
3
3
2
2
para valores inteiros de x, ento x tambm inteiro. Sendo x2 > 4 , conclumos que
3
x2 , no mnimo, 5. Dentre os nmeros 5, 4, 3, 2, 1, 0, 1, 2, 3 somente quatro,
a saber, 5, 4, 3 e 3 satisfazem x2 5.
284

OBMEP 2010

Solues do Nvel 3
81. Equao cbica A opo correta (d).


Observe que o polinmio cbico dado igual a x 2 007x2 + 2 006x + 2 005 , portanto,
x = 0 uma soluo da equao dada e a opo (a) fica descartada. Como a equao
cbica e x = 0 uma soluo, a opo (e) fica descartada. Agora, para ver se a equao
dada tem uma, duas ou trs solues, s precisamos ver se a equao de segundo grau
2 007x2 + 2 006x + 2 005 = 0 no tem soluo, ou tem uma ou tem duas solues. Mas
o discriminante dessa equao
= 2 0062 4 2 007 2 005 = 2 0062 4(2 006 + 1)(2 006 1)
= 2 0062 4(2 0062 1) = 3 2 0062 + 4 < 0,

de modo que essa equao no possui razes reais. Assim, a equao inicial tem uma
nica raiz real.
Observao: Uma outra maneira (e mais simples) de mostrar que < 0 observar
que 2 006 < 2 007 e 2 006 < 4 2 005, portanto,
2 006 2 006 < 4 2 005 2 007 e 2 0062 4 2 005 2 007 < 0.
82. O perfume de Rosa O volume de um cilindro o produto da rea da base pela
altura. Como o raio da base mede 7 cm, a rea da base 72 e, ento, o volume do
vidro
490
72 10 cm3 = 490 cm3 =
dm3 = 0,49 litros,
1 000
lembrando que 1 000 cm3 = 1 dm3 = 1 litro. Depois de duas semanas, restaram 0,45
litros de perfume, de modo que ela gastou (0,49 0,45) litros. Portanto, a frao que
representa o volume gasto
0,49 0, 45
volume gasto
49 45
=
=
.
volume total
0, 49
49
83. Igualdade com inteiros Como 2n = m2 1 = (m + 1)(m 1), estabelecemos que
m 1 e m + 1 so potncias de 2. Como a diferena de m + 1 e m 1 2, a nica
soluo possvel m 1 = 2 e m + 1 = 22 , donde m = 3. Assim, 2n + 1 = 32 = 9 e
obtemos n = 3. A resposta m = n = 3.

84. O caminho da pulga No primeiro pulo, a pulga percorre 10 21 m, no segundo
2
pulo, ela percorre 10 12 m, e assim por diante. Depois de 7 pulos, a pulga ter
percorrido
1
 1 2
 1 3
 1 4
 1 5
 1 6
 1 7
10
+ 10
+ 10
+ 10
+ 10
+ 10
+ 10
2
2
2
2
2
2
2
h 1  1 2  1 3  1 4  1 5  1 6  1 7 i
= 10
+
+
+
+
+
+
2
2
2
2
2
2
2
26 + 25 + 24 + 23 + 22 + 2 + 1
127
= 10
= 10
9,9.
27
128
Logo, em 7 dias, ela ter percorrido, aproximadamente 9,9 m. Em geral, depois de n
dias, a pulga ter percorrido
1
1
10
+ + n metros.
2
2
OBMEP 2010

285

Solues do Nvel 3
Para calcular a soma acima, note que 12 + + 21n a soma dos n termos de uma
progresso geomtrica cujo primeiro termo a1 = 1/2 e cuja razo q = 1/2. A
frmula para essa soma
1
a1 (1 q n )
1/2(1 1/2n )
=
= 1 n.
1q
1 1/2
2

Sn =
Assim,

1
1
10
++ n
2
2
Tomando n = 10, obtemos
10


1
10
= 10 1 n = 10 n .
2
2

10
10
1 023
= 10
= 10
9,99.
10
2
1 024
1 024

Portanto, ao final do dcimo dia, a pulga ter percorrido, aproximadamente, 9,99


metros.
A pulga estar a menos de 0,001 m do final do caminho quando ela j tiver percorrido,
pelo menos, 9,999 = 10 0,001 metros, ou seja, quando
10

10
10 0,0001,
2n

o que equivale a 0,001 10/2n , ou 2n 10/0,001 = 10 000.

Agora, 213 = 210 23 = 1 024 8 = 8 192 < 10 000 < 16 384 = 214 , de modo que
devemos tomar n = 14 e a pulga estar a menos do que 0,001 m do final do caminho
a partir do dcimo quarto dia.
85. Uma soma alternada A opo correta (d).
A expresso (1)n+1 na definio de Sn tem valor 1 se n for par e tem valor 1 se n
for mpar.
Soluo 1: Associando parcelas consecutivas duas a duas, obtemos uma soma de
vrias parcelas iguais a 1: (1 2) + (3 4) + (5 6) + . Logo,
S1 992 = (1 2) + (3 4) + + (1 991 1 992) = (1) 996 = 996
|
{z
}
1 9922=996 parcelas

S1 993 = (1 2) + (3 4) + + (1 991 1 992) + 1 993 = 996 + 1 993 = 997.


Assim, S1 992 + S1 193 = 996 + 997 = 1.
Soluo 2: Como


S2n = (1 2) + (3 4) + (5 6) + + 2n (2n + 1) ,
|
{z
}
n parcelas iguais a 1

obtemos S2n = n e S2n+1 = S2n +(2n+1) = n+2n+1 = n+1. Assim, S2n +S2n+1 =
1.
286

OBMEP 2010

Solues do Nvel 3
86. O raio da circunferncia A opo correta (c).
2
Soluo 1: Se o raio r, ento o comprimento de um arco de graus
r. Assim,
360
no problema dado, temos que
2 000 m =

2
5
300 r =
r,
360
3

portanto r = 2 000 (3/5) 382,17 m.

Soluo 2: Como a circunferncia tem 360, um arco de 300 representa 5/6 da circunferncia, portanto, seu comprimento de 2 km 5/6 do comprimento da circunferncia,
isto , (5/6) 2 r = 2 000 m, portanto
r=

1 200
2 000 6
=
382,17 m.
10

87. Quatro passageiros O passageiro que quer ficar na janela tem trs possveis lugares
para se sentar, o seguinte pode-se sentar em qualquer lugar livre, tendo, portanto, trs
possveis lugares; o seguinte tem dois possveis lugares e o ltimo no tem escolha.
Conclumos que o nmero dessas formas de se sentar 3 3 2 = 18.
88. Os cinco crculos Observemos que qualquer linha que passe pelo centro O do
quadrado ABCD, divide a rea formada pelos crculos C1 , C2 , C3 e C4 pela metade.
Por outro lado, qualquer linha reta que passe pelo centro F do crculo C5 , divide a rea
desse crculo pela metade. Assim, a reta procurada a reta F O.
C1 ...............................................................

C2

.....
.................. .........................
.......
.....
.
....
....
....
....
....
....
....
....
...
....
...
.
.
... ...
...
..
.
... ..
...
....
.
.....
..
...
.
..
...
..
...
...
..
....
..
..
...
..
.
...
. ...
....................
.
.
.
.
.
.
...
.
.
.. ..
...............
...
.. .....
............. .....
....
...
....
............
....
...
...
.............
....
.....
....
....
.............
.
.
.
.
.
.
.
.
.
.
.
.
.
.
.
.......
.
.
.
.
.
.
.
.
.
.
.....
....... ..............
.............
........
............
.
.............
............................................
.........................................
.......................................... ........................
............
.......
.........
......
........
.......
.....
...........
......
....
.....
............ .......
.....
....
.
.
.
....
....
....
.
.
.
.
.
.
.
.
.
.
.
.
.
.
.
....
....
.
.
.
.......
...
...
... ....
... .... ........................
...
.
..
.
.
.
.
.
.
.
.
...
.
.
.
.
.
... ..
................
...
.
.
.
.
....
.
.
.
.
....
........ .......
.
.
.
.
...
.
.
.
.
.
.
.
...
.
.
...
...
.......
.
.
.
...
.
.
.
.
.
.
.
.
.
.
...
.
.........
...
...
.
.
.
.
.
.
.
.
..
.
.
..
.
.
.
...
.. .....................
.....
..
.
.
.
..............
. ...
. ...
.
.
.
.
.
.
.
.
.
.
.
.
.. .
. ..
. ...
.
............. ....
... .....
... .....
...
...
...
...
...
....
....
....
...
...
.....
....
.....
.....
.....
....
.....
......
.
.
.
.
.
.
.
.
.
.
.
.
.
.
.
.
.
.
.
.
.
.
.
.
.
.........
.........
.
..
..........
.........................................
.......................................
....................................

Ds

Cs

Os

C5

C4

C3

89. O tringulo e o quadrado As diagonais do quadrado ABCD


dividem o quadrado em 4 tringulos iguais, portanto, a rea do
tringulo BCE mede uma quarta parte da rea do quadrado, ou
seja,
1 4 = 0,25 cm2 .

B............................................................................................................................ C
r

.. ....
. .
.....
... ....
............ ...... ....
.... ........................
...
...
..
......
....
...
...
....
...
.
.
...
.
...
....
.
.... ......
...
...
... ...
..
..
.....
...
...
.
.
.
...
...
.... .......
.
.
....
...
...
...
.
.
....
...
...
..
.
.
.
.
.
....
...
.
..
.
.
.
.
.... ....
... ....
.
.
.
... ...
.... ....
.
.
.
.......................................................................................

r E

Como o comprimento de BF a metade de BE e CE a altura comum s bases BF


e BE, conclumos que a rea do tringulo CBF necessariamente a metade da rea
do tringulo CBE. Assim, a rea do tringulo CBF 0,125 cm2 .
90. Uma refeio Se S corresponde ao nmero de sanduches e P ao nmero de pratos
de refeio, ento 5S + 7P = 90 e, portanto,
P =

90 5S
18 S
=5
.
7
7
OBMEP 2010

287

Solues do Nvel 3
Como queremos solues inteiras no-negativas P e Q, vemos que 7 deve dividir 18S.
Assim, S s pode ser 4, 11 ou 18 e, nesses casos, P igual a 10, 5 ou 0, respectivamente.
Portanto, temos somente trs formas de fazer a compra sem receber troco, a saber, 4
sanduches e 10 pratos, 11 sanduches e 5 pratos, ou 18 sanduches e nenhum prato.
91. Plano Cartesiano Somando 1 abscissa a do ponto
P = (a, b) transladamos esse ponto uma unidade para
a direita, trocando a por a refletimos esse ponto pelo
eixo y e dividindo a por 2, transladamos esse ponto
metade de sua distncia do eixo x. Analogamente, trocar a ordenada b de P por b 1, ou b 2, translada
P uma ou duas unidades para baixo, trocar b por b
reflete o ponto pelo eixo x e trocar b por b/2 translada P para o ponto metade de sua distncia do
eixo y. A figura mostra P junto com os quatro pontos
A = (a + 1, b/2), B = (a/2, b 1), C = (a, b) e
D = (1 a, b 2) no plano cartesiano.

B
r

Dr 1
2

Pr

A
r
2

Cr

2
3

92. Soma dos terminados em 9 A soma das k primeiras parcelas de uma progresso
aritmtica dada por Sk = 21 (a1 + ak ) k, em que a1 e ak = a1 + (k 1) r so o primeiro
e ltimo termos, respectivamente, e r a razo. Por exemplo, temos
1 + 2 + 3 + + (n 1) =


1
1
1 + (n 1) (n 1) = n(n 1).
2
2

A soma dada a de uma progresso aritmtica de n parcelas com primeiro termo


a1 = 9 e razo r = 10, de modo que temos an = 9 + (n 1) 10 e, portanto,
Sn =

i
1h
9 + 9 + (n 1) 10 n = 9n + (n 1) 5n = 5n2 + 4n.
2

Como queremos Sn 105 , precisamos encontrar o menor inteiro positivo n tal que
5n2 + 4n 105 ou, equivalentemente, 5n2 + 4n 105 0. Para isso, resolvemos a
equao de segundo grau 5x2 + 4x 105 = 0, obtendo as solues

4 16 + 20 105
x=
,
10



1
4 + 2 000 016 141,02. Como 5x2 + 4x 105 positivo
e a raiz positiva x1 = 10
fora das razes, por ter coeficiente dominante 5 > 0, resulta que n = 142 o menor
inteiro positivo n para o qual Sn maior do que 105 .
93. Trs cilindros O volume de um cilindro de raio R e altura h dado por R2 h.
(a) Os trs volumes so V1 = 103 = 1 000 , V2 = 52 10 = 250 e V3 =
52 20 = 500 , portanto, V1 > V3 > V2 .

(b) Como os cilindros V2 e V3 tm o mesmo raio, basta manter o raio do cilindro em


5 cm e a altura entre 10 e 20 cm; por exemplo, h = 15 cm. Nesse caso, o volume
V4 do novo cilindro 52 15 = 375 cm3 .
288

OBMEP 2010

Solues do Nvel 3
(c) Para construir um cilindro de volume V5 entre V1 e V3 , podemos tomar a menor
das duas alturas, que 10 cm, e diminuir o raio do cilindro de maior volume de
10 para 8 cm, obtendo um cilindro de volume V5 = 82 10 = 640 cm3 .
94. Porcentagem de mortalidade A opo correta (a).
15
A proporo de toda a populao que fica doente da enfermidade
e, entre os que
100
8
ficam doentes, a proporo dos que morrem
. Assim, a proporo da populao
100
15
8
que morre pela doena

, o que corresponde a
100 100
15 8
120
1,2
=
=
= 1,2%.
1002
10 000
100
95. Agenda de aulas Se a aula da manh segunda ou sexta (em qualquer um dos trs
horrios), ento o dia da aula de tarde pode ser escolhido de trs formas diferentes (em
qualquer um dos dois horrios), portanto, temos 2 3 3 2 = 36 formas diferentes
de escolher o horrio. No caso em que a aula de manh seja no sbado, o dia da aula
da tarde pode ser qualquer dia de segunda a quinta, portanto, temos 3 4 2 = 24
possveis formas de escolher o horrio. Por ltimo, se a aula da manh tera, quarta
ou quinta, ento a aula da tarde s pode ser escolhida de duas formas, portanto, temos
3 3 2 2 = 36 formas de escolher o horrio. Assim, Eliane pode escolher seu horrio
de 36 + 24 + 36 = 96 formas distintas.
96. Jogo de Cartas A estratgia abaixo permite realizar o jogo com 17 movimentos.
Em cada movimento, o primeiro nmero indica a pilha da qual a carta tomada e o
segundo a pilha em que a carta colocada. Por exemplo, o primeiro movimento (1)
e 4 sobre 2 significa pegar a carta superior da pilha 4 e colocar sobre a pilha 2.
(1) 4 sobre 2
(7) 3 sobre 4
(13) 2 sobre 1

(2) 4 sobre 3
(8) 1 sobre 3
(14) 2 sobre 1

(3) 4 sobre 2
(9) 1 sobre 4
(15) 4 sobre 2

(4) 3 sobre 4
(10) 2 sobre 1
(16) 4 sobre 2

(5) 3 sobre 4
(11) 2 sobre 4
(17) 4 sobre 2

(6) 1 sobre 4
(12) 2 sobre 3

O movimento 2 tambm poderia ser 4 sobre 1, o movimento 4 poderia ser 1 sobre


4, o movimento 5 poderia ser 1 sobre 4, o movimento 6 poderia ser 3 sobre 4. Os
movimentos 4, 5 e 6 poderiam ser permutados em qualquer ordem. Teramos, assim,
pelo menos, seis maneiras de realizar o jogo com 17 movimentos.
Esse jogo poderia ser realizado com um nmero menor de movimentos?
97. Fraes inteiras Como





2n2 + 4n + 18
2 (n2 + 2n + 1) + 8
1
16
=
=
2n + 2 +
,
3n + 3
3
n+1
3
n+1

segue que a expresso entre parnteses deve ser um mltiplo


de 3 e, em particular, n + 1 deve dividir 16. Assim, n pode ser
1, 3, 7 ou 15. Pela tabela ao lado, em cada um desses quatro
casos, ou seja, para n igual a 1, 3, 7 ou 15, o quociente dado
resulta ser um nmero inteiro.

OBMEP 2010

n
1
3
7
15

2n2 + 4n + 18
3n + 3
4
4
6
11
289

Solues do Nvel 3
98. Quatro prefeitos e um crculo O nmero de rodovias igual ao nmero de
pontos que podem ser o centro de um crculo (rodovia) que seja equidistante de quatro
pontos (cidades) dados. Como nenhum crculo passa pelos quatro pontos dados, se
algum crculo for equidistante dos quatro pontos, esse crculo no pode deixar todos os
quatro pontos do lado de dentro ou todos do lado de fora, de modo que deve dividir o
conjunto dos quatro pontos em dois, sem passar por algum deles. Assim, s podemos
ter trs tipos de configurao, de acordo com o nmero de pontos dentro e fora do
crculo. No primeiro, o crculo equidistante deixa trs pontos dentro e um fora; no
segundo, dois dentro e dois fora e, no terceiro, um dentro e dois fora.
Nas figuras abaixo esto ilustrados os dois primeiros tipos, em que o crculo contnuo
o equidistante.
.... .... .... .... ....
.... .
.... ....
...
....
...
...
.............
...
................ ........................
.
...
.
.
.
.
.
.
......
...
.....
...
.
.
.....
.
.
...
....
.. .... .... ............. ....
.
... .......
.
.
.
..
.. .... ... . ......
...
...
..
.
.
.
..
... ..
.. ...
.
...
.. ....
.
...
...
...
..
.
.
.
.
.
.
...
.. .... ...
.
..
.. .
.
.
...
.
.. .... ..
.. ....
...
..
..........
.
...
.
.
.
... ..
.
.
..
.
.
.
...
...
. .... ..
. .
... ....... ........
...
...
.... ..
.. .....
... .. ....
..
..
.. .
..
..
.
.
.
..
.. ..
... .. ... .
..
..
...
.
...
.. .
..
.
... .
..
..
... .... ..
..
..................... ....
.
.
.
.
.
.
.
.
.
.
.
.
.
.
.
.
.
.
.
.
.
.
.
.
... .. ..
.....
.
.
..
.. ......................................
.
..
... ..
....
... ......
...
.
...
... ..
....
....
....
...
..
.. ..
....
....
.... ..
. ....
.
..
.
.
.
.
.
.
.
.
.
.
.
.
.
.
...
.......
.
.......
...........
...
...
.............................................
....
...
....
...
.... ..
... .
..
.
.
... ..
.
.
.
.... ..
..
... ..
... ..

qs

.... .... .... .... ....


... .
.... ....
...
....
...
...
.........................................
.
.
.
.
.
.
...
.
.
.
...
.
........
.......
.
...
......
.
.
...
.
...
.
.
.
.
.
.
.
.
.
.
.
.
.
...
.
.
.
.
.
.
.
...
....
. ....
...
...
.
.
.
.
.
.
..
.
..
...
..
..
..
.
.
.
.
.
.
.
.
.
..
...
.
.
...
.
.
.
.
.
..
....................................................................................... ....
... ....
... ..
....
..
.. ...
..
.
..
...
...
.
..
..
..
..
...
..
..
.
.
.
..
..
.
...
.
...
...
.
..
.
.
..
..
.
..
.
...
..
..
.
.
.
.
.
.
.
...
..
...
.. .....
..
...
..
............ ..
.. ....
.
.
.
.
.
.
...
.
.... ..
.. ..... ....
...
.. .......
...
...
... ... ................. ..
..
....
... ..
. . .
....
.. ..
...
... ............... .......
.
.
...
.
.
.
.
.
..
..
.
.
.
.
.
......
...
..... .....
...
.......
...... ........
.............
...
...
..........................................
...
..
.
.
.
.
.
.
....
.
..
.... ..
..
... .
.........
.........
... ....
. ..
... ..
... ..

sq

Na primeira figura, o centro do crculo equidistante coincide com o centro do crculo


circunscrito ao tringulo formado pelos trs pontos internos. Essa mesma configurao
ocorre no terceiro tipo, em que o centro do crculo equidistante coincide com o centro
do crculo circunscrito ao tringulo formado pelos trs pontos externos. Assim, nesses
dois tipos, o nmero de crculos equidistantes igual ao nmero de tringulos que
podemos formar com trs dentre os quatro pontos, ou seja, quatro.
Na segunda figura, o centro do crculo equidistante est na mediatriz dos dois pontos
internos e, tambm, na mediatriz dos dois pontos externos. Assim, nesse tipo, o nmero
de crculos equidistantes igual ao nmero de maneiras de dividir o conjunto de quatro
pontos em dois conjuntos de dois pontos cada um, ou seja, trs.
Logo, o nmero possvel de projetos de rodovias circulares equidistantes das quatro
cidades 4 + 3 = 7.
99. Fatoriais Queremos a b c = a! + b! + c! com algarismos 0 a, b, c 9. Como
0! = 1! = 1, 2! = 2, 3! = 6 e 4! = 24, algum dos algarismos a, b ou c deve ser maior
do que 4, pois 0! + 1! + 2! + 3! + 4! = 34 s tem dois dgitos. Se algum dos algarismos
a, b ou c for maior do que ou igual a 6, teremos a b c = a! + b! + c! > 6! = 720,
o que acarreta que algum dos algarismos a, b ou c , pelo menos, igual a 7; mas
ento a b c = a! + b! + c! > 7! = 5 040 tem, pelo menos, quatro dgitos, o que uma
impossibilidade.
Assim, algum dentre a, b e c igual a 5 e os demais so menores do que 5. O menor
nmero possvel 5! + 1! + 0! = 120 + 1 + 1 = 122 e o maior nmero possvel
5! + 3! + 4! = 120 + 6 + 24 = 150. Logo, o algarismo a das centenas 1. Se o algarismo
b das dezenas for 5, ento c 4 e
1! + 5! + c! = 1 + 120 + c! = 121 + c! 121 + 4! = 121 + 24 = 145 6= 15 c.
290

OBMEP 2010

Solues do Nvel 3
Se o algarismo b das dezenas for 0, 2 ou 3, ento b! igual a 1, 2 ou 6 e, como
necessariamente c = 5, temos que 1! + b! + 5! = 1 + b! + 120 = 121 + b! igual a 122,
123 ou 127, todos diferentes de 1b 5. Resta apenas a opo b = 4 e c = 5. Nesse caso,
efetivamente 1! + 4! + 5! = 1 + 24 + 120 = 145, como queramos. Os trs nmeros
inteiros so a = 1, b = 4 e c = 5.
100. O Riquinho Os 1 000 reais de Riquinho foram repartidos em parcelas crescentes a
partir de 1, de modo que 1 + 2 + 3 + + n 1 000. Como 1 + 2 + 3 + + n a soma
dos n primeiros nmeros inteiros a partir de 1, temos 1 + 2 + 3 + + n = 21 (1 + n)n.
Assim, queremos encontrar o maior n tal que 21 (1 + n)n = 1 + 2 + 3 + + n 1 000,
ou seja, tal que n2 + n 2 000 0.
Como n2 + n 2 000 igual a 2 000 para n = 0 e o coeficiente dominante desse
polinmio 1 > 0, sabemos que os valores de n2 + n 2 000 so negativos para todo
n entre 0 e a raiz positiva do polinmio quadrtico x2 + x 2 000. Pela frmula de
Bhaskara, a raiz positiva dada por

1 + 1 + 8 000
44,22 ,
x=
2

portanto n2 + n > 2 000 para n 45. Assim, Riquinho distribuiu apenas 44 parcelas.
Como Bernardo recebeu a segunda parcela, a quinta parcela (5 = 2+3), a oitava parcela
(8 = 2 + 2 3), e assim por diante, tambm recebeu a ltima, j que 44 = 2 + 14 3,
num total de
1
2 + 5 + 8 + 11 + + 44 = (44 + 2) 15 = 23 15 = 345 reais.
2
Observao: Depois de distribuir as 44 parcelas, ainda sobram
1
1 000 (44 45) = 1 000 990 = 10
2
dos 1 000 reais de Riquinho.
101. Retngulo com dimenses inteiras Sejam a e b os comprimentos dos lados do
2
2
2
2
2
2
retngulo. Supondo
a b, temos b < a + b 2b , pois a 2 b .2As diagonais do
retngulo medem 1 993, portanto, pelo Teorema de Pitgoras, a + b = 1 993. Assim,
b2 < 1 993 2b2 , ou seja, 996,5 b2 < 1 993. Assim,

p
31 < 996,5 b < 1 993 < 45 .
Como b um nmero inteiro, 32 b 44. Alm disso, a b e, como a tambm um
nmero inteiro, 1 993 b2 = a2 deve ser um quadrado perfeito.
Para calcular os valores de a2 = 1 993 b2 , com b variando de 32 a 44, calculamos
primeiro 1 993 322 = 969 e observamos que
1 993 332 = 1 993 (32 + 1)2 = (1 993 322 ) (2 32 + 1)
= 969 65 = 904
e, em geral, 1 993 (n + 1)2 = (1 993 n2 ) (2n + 1). Desse modo, fcil obter a
tabela seguinte.
OBMEP 2010

291

Solues do Nvel 3
b
32
39

a2
969
472

b
33
40

a2
904
393

b
34
41

a2
837
312

b
35
42

a2
768
229

b
36
43

a2
697
144

b
37
44

a2
624
57

b
38

a2
549

Como 144 o nico quadrado perfeito da lista, a nica soluo um retngulo de lados
medindo 43 e 12 cm.
102. Mltiplos de 3 e quadrados perfeitos Escrevendo a para um nmero qualquer
da lista, sabemos que a um mltiplo de 3 e que a + 1 um quadrado perfeito, ou
seja, a + 1 = k 2 , para algum inteiro positivo k. Assim, a = k 2 1 = (k 1)(k + 1).

Como a divisvel por 3, ento ou k + 1 ou k 1 divisvel por 3. Logo, k s no


pode ser divisvel por 3 e, portanto, a cada trs inteiros k consecutivos (comeando em
2), dois deles fornecem um nmero da lista: de fato, k = 2 = 1 3 1 d o primeiro
nmero a = 22 1 = 3 da lista; k = 4 = 1 3 + 1 d a = 42 1 = 15, que o segundo;
k = 5 = 2 3 1 d a = 52 1 = 24, que o terceiro; k = 7 = 2 3 + 1 d o quarto
a = 72 1 = 48, e assim por diante. Em geral, uma posio par 2n da lista dada
por k = n 3 + 1, portanto a 2 006a posio par dada por k = 1 003 3 + 1 = 3 010.
Assim, o mltiplo de 3 na 2 006a posio da lista a = 3 0102 1 = 9 060 099.

103. Cinco cartas Simone no precisa virar a carta que tem o nmero 2 porque o outro
lado, vogal ou consoante, de qualquer forma cumpre a condio Se uma carta tem
uma vogal numa face, ento ela tem um nmero par na outra.
..................... ..................... ..................... ..................... .....................
. .
. .
...
. .
. .
.
... 2 ..... ..... 3 ..... ..... M ..... ..... A ..... ..... E .....
.. ..
.. ..
.. ..
..
. ..
...
................... ..................... ..................... ..................... .....................
Ela tambm no precisa virar a carta com a letra M, j que, do outro lado, o nmero
pode ser par ou mpar que a condio satisfeita. Entretanto, a carta que tem o
nmero 3 precisa ser virada para comprovar que do outro lado tem uma consoante,
bem como as cartas com as letras A e E, para comprovar que do outro lado o nmero
par. Assim, Simone precisa virar somente 3 cartas.
104. O lucro de uma companhia A opo correta (c).
Nos primeiros R$ 1 000,00, a companhia tem lucro de 1 000 6% = 60 reais e, para
os R$ 5 000,00 restantes, tem lucro de 5 000 5% = 250 reais. Logo, o lucro da
companhia nesse dia de R$ 310,00.
105. Sequncia triangular Observe que o 21o termo a soma de 21 nmeros consecutivos. Para descobrir quais so esses nmeros, basta encontrar a primeira parcela do 21o
termo que a soma de 21 nmeros consecutivos. Observamos que, a partir do segundo
termo, a primeira parcela do
2o termo 2
3o termo 4
4o termo 7
5o termo 11
6o termo 16

292

=
=
=
=
=

1 + 1, a do
2 + 1 + 1, a do
3 + 2 + 1 + 1, a do
4 + 3 + 2 + 1 + 1, a do
5 + 4 + 3 + 2 + 1 + 1,

OBMEP 2010

Solues do Nvel 3
e assim por diante. Portanto, podemos ver que a primeira parcela do 21o termo
20 + 19 + + 3 + 2 + 1 + 1 =

21 20
+ 1 = 211
2

e que, portanto, o 21o termo


1
211 + 212 + + 230 + 231 = (211 + 231) 21 = 221 21 = 4 641.
2
106. O jardim octogonal O comprimento total da cerca con...........................
siste na soma dos comprimentos dos contornos da roseira e do
.....
....
.
.
.....
..
.
jardim, ambos utilizando diagonais dos quadradinhos da fo.
.
....
..
.........
.
.
..
.
.... .......
..
lha quadriculada. O contorno da roseira formado por quatro
...
.
.
.
.
.
.
..
...
...... roseira
.
..
.
....
.
....
.
....
diagonais e o do jardim por oito diagonais e oito lados.
.... ......
...
.......
..
.
.....
.
.
..
Assim, o comprimento total da cerca de oito lados e doze
.....
....
.....
.
.
.........................
diagonais de quadradinhos. Para descobrir o comprimento
das diagonais, basta descobrir o comprimento dos lados dos
quadradinhos. Para isso, utilizamos a informao fornecida
relativamente rea do jardim.
Contando o nmero de quadradinhos do jardim, obtemos 24 quadradinhos inteiros
e oito metades, o que igual a 28 quadradinhos inteiros que representam 700 m2
de rea. Desse modo, cada quadradinho mede 700 28 = 25 m2 e, portanto, cada
lado de quadradinho representa
5 m. Pelo
Teorema de Pitgoras, a diagonal de cada

2
2
que o comprimento toquadradinho mede d = 5 + 5 = 5 2 m. Conclumos

tal da cerca de 8 5 + 12 5 2 = 40 + 60 2 m. Como o prefeito dispe de


R$ 650,00, cada metro dessa cerca pode custar, no mximo,
65
65
650
65
=

=
5,21 reais.
4 + 6 1,414
12,484
40 + 60 2
4+6 2
107. Nmero de caracteres Na 1a linha escrevemos os nmeros de 1 a 9, cada um
seguido de um espao, o que ocupa 18 espaos; sobram 82 espaos. Cada nmero de
dois algarismos mais um espao ocupa trs lugares na linha. Como 82 = 27 3 + 1,
completamos a 1a linha com 27 nmeros de dois algarismos, a partir do nmero 10.
Assim, o ltimo nmero da primeira linha o 36 e sobra um espao. Denotando cada
espao entre nmeros por um trao, podemos representar a 1a linha como segue.
1a linha : 1| 2 3 4 5{z 6 7 8 9} 10
36 }
| {z
18

81

Mas 100 = 33 3 + 1, portanto, podemos colocar 33 nmeros de dois algarismos na


segunda linha, cada um seguido de seu respectivo espao, e no final da linha sobra um
espao.
2a linha : 37
| 38 {z 69}
99

Na 3 linha, colocamos os nmeros de 70 a 99, ocupando 303 = 90 espaos. Ocupamos


os dez espaos restantes com os dois primeiros nmeros de trs algarismos, seguidos de
um espao; no final da linha, sobram dois espaos.
a

3a linha : 70
| 71 {z 99} 100
| {z101}
90

OBMEP 2010

293

Solues do Nvel 3
A partir da quarta linha, podemos colocar, em cada linha, 1004 = 25 nmeros de trs
algarismos com seus respectivos espaos. De 102 a 999, inclusive, temos 999102+1 =
898 nmeros. Como 898 = 25 35 + 23, ocupamos 35 linhas, desde a 4a at a 38a , com
esses nmeros de trs algarismos e ainda restam 23 desses nmeros, que colocamos na
39a linha, onde ocupam 23 4 = 92 espaos, restando oito espaos, que ocupamos com
1 000, restando trs.
39a linha : 977
999} 1| 000
| {z
{z }
92

A partir da 40 linha, podemos colocar, em cada linha, 1005 = 20 nmeros de quatro


algarismos com seus respectivos espaos. Assim, nas 61 linhas restantes, podemos
colocar exatamente 61 20 = 1 220 nmeros de quatro algarismos. Comeando com
1 001, terminamos a 100a linha com o nmero 2 220.
a

108. A rvore de Emlia Denotemos por gn o nmero de galhos


da rvore depois de n semanas. Como s depois de duas semanas
aparece um galho, g2 = 1. Na terceira semana esse galho produz um
novo galho, ou seja, g3 = 2. O nmero de galhos em uma semana
igual ao nmero de galhos que existiam na semana anterior, mais
os galhos novos.

q q q

..
...
. ...
.
..
..
...
.
...
..
.. ..
..
...
..
.. ....
....
..
.
.
.
.
.
.
.
.
..
...........
.
..
...
...........
...
...
..
...
..
...
...
..
.
.
.
.
...
.
.
..
...
....
..
.................... ....
..
..
..
..
....
.
.....
...
... ...................
...
..
...
.....
...
..

q
q
q

Entretanto, pela regra fornecida, os galhos novos nascem dos galhos que tm pelo
menos duas semanas de idade, isto , na semana n + 1 tem um galho novo para cada
galho que j existia na semana n 1. Assim, temos gn+1 = gn + gn1 , de modo que, a
partir de g2 = 1 e g3 = 2, obtemos
g4
g5
g6
g7
g8

=
=
=
=
=

2+1=3
3+2=5
5+3=8
8 + 5 = 13
13 + 8 = 21

No fim de oito semanas, a rvore de Emlia ter um total de 21 galhos.


109. Um teste vocacional
(a) De exatas temos 232 + 112 = 344 alunos e a probabilidade de escolher ao acaso
344
um de exatas
= 0,344.
1 000
(b) Do sexo masculino temos 232 + 116 + 207 = 555 alunos e a probabilidade de
116
escolher ao acaso um da rea de humanas
= 0,209.
555
(c) De biolgicas temos 207 + 180 = 387 alunos e a probabilidade de escolher ao acaso
180
um do sexo feminino
= 0,465.
387
110. Dois setores circulares Como 60 + 30 = 90 , segue que a rea destacada representa um quarto da rea total do crculo.
Como a rea do crculo mede
20 cm2 , a rea destacada mede 5 cm2 .
294

OBMEP 2010

Solues do Nvel 3
111. Compra de televisores Digamos que Maria tenha encomendado n televisores,
pagando R$ 1 994,00 por televisor. O total pago 1 994 n, e sabemos que nesse mltiplo
de 1 994 no aparecem os algarismos 0, 7, 8 e 9. Para limitar o valor de n, observamos
que
1 994n = 2 000n 6n.
Se 6n < 300, ento o nmero 2 000n 6n tem 7 ou 8 ou 9 no algarismo das centenas
(faa alguns exemplos, lembrando que 2 000n termina com trs zeros). Assim, devemos
ter 6n 300, ou seja, n 50.

Testando n igual a 50, 51, 52, 53, 54 e 55, obtemos 99 700, 101 694, 103 688, 105 682,
107 676 e 109 670, mas n = 56 d 111 664. Assim, o menor nmero de televisores que
Maria pode ter encomendado 56.

112. Distncia entre nmeros Observe que |x y| igual distncia entre os pontos
x e y e, em particular, |x| igual distncia entre x e a origem 0. Pela figura, temos
4 < a < 3 < 2 < b < 1 < c < 0 < 1 < d < 2,
de modo que (a), (b), (c), (d) e (e) so verdadeiras e (f) falsa, pois
|d| = d < 2 < 3 < |a|.
a
4

b
2

d
0

Tambm 1 < |a b| < 3, 1 < |c d| < 3, 0 < |b c| < 2 e 2 < |c a| < 4, o que
acarreta que (g), (j) e (l) so verdadeiras e (h), (i) e (k) so falsas.
113. Cartes premiados O carto de dgitos a b c d premiado se a + b = c + d. Consideremos dois casos. Se o nmero de dgitos c d de um carto premiado for igual ao
nmero de dgitos a b, ento a b c d = a b a b = a b 100 + a b = 101 a b e o nmero
desse carto divisvel por 101. Caso contrrio, c d 6= a b e, ento h um outro carto
premiado, a saber, c d a b. A soma dos nmeros desses dois cartes
a b c d + c d a b = (a b 100 + c d) + (c d 100 + a b) = 101(a b + c d),
que tambm divisvel por 101. Como todo carto premiado de um desses dois tipos,
a soma dos nmeros de todos os cartes premiados divisvel por 101.
114. O preo da gasolina A opo correta (d).
Soluo 1: O aumento do preo foi de 149,70 29,90 = 119,80 reais, o que corresponde
a
119,80
100% = 400,66%.
29,90
Soluo 2: O preo praticamente passou de 30 para 150, isto , foi multiplicado por 5,
o que equivale ao preo passar de 100 para 500, caracterizando um aumento de 400%.
OBMEP 2010

295

Solues do Nvel 3
115. O tringulo de moedas Supondo que o tringulo esteja formado por n linhas,
foram usadas 1 + 2 + 3 + + n moedas, ou seja,
1
n(n + 1) = 1 + 2 + + n = 480 15 = 465,
2
o que fornece n2 + n 930 = 0. Resolvendo essa equao, obtemos

1 1 + 4 930
1 61
n=
=
.
2
2
Como n = 30 a nica soluo positiva dessa equao, o tringulo tem 30 linhas.
116. Circunferncia e tringulo retngulo Seja r o raio da circunferncia inscrita no
tringulo. Como o tringulo retngulo, fcil verificar que os catetos do tringulo
medem 6+r e 7+r cm, conforme a figura. Pelo Teorema de Pitgoras, temos (6+7)2 =
(r + 6)2 + (r + 7)2 e, desenvolvendo, resulta 169 = 132 = 2(r 2 + 13r) + 85, de modo
que r 2 + 13r = 42.

6
6 ..............
.............
..............
......
.....
....

....
...
.
...

...
....
...
...
..
...
7
..
..
..
..
...

..
.
.
...
.
.

....
.
r .............. ................

..............................

Assim, a rea do tringulo mede


(r + 6)(r + 7)
r 2 + 13r + 42
42 + 42
=
=
2
2
2
= 42 cm2 .

1
1 1 1
1
Como Sn = + + + + n , segue que
2
2 4 8
2


1 1
1
1 1 1
1
1
1
Sn =
+ + + + n = + + + n+1 ,
2
2
2 4 8
2
4 8
2

117. Soma de razo

de modo que

1
1
1
1
Sn = Sn Sn = n+1 e, portanto,
2
2
2 2
Sn = 1

1
.
2n

Como queremos Sn > 0,99, isso equivale a


1

1
1
= Sn > 0,99 = 1 0,01 = 1
,
n
2
100

ou seja, a 2n > 100. Como 27 = 128 > 100 > 26 = 64, n = 7 o menor n tal que
2n > 100, ou Sn > 0,99.
Observao: Note que, no incio desta resoluo, deduzimos o valor da soma da progresso geomtrica de primeiro termo e razo ambos iguais a 1/2, sem usar a frmula
dessa soma, a saber,
1
1 1
n
1
Sn = 2 2 2 = 1 n .
1
2
1
2
296

OBMEP 2010

Solues do Nvel 3
118. Soma de razes quadradas
(a) Como

r 2 = ( 2 + 3)2 = ( 2)2 + 2( 2)( 3) + ( 3)2 = 2 + 2 6 + 3 = 5 + 2 6,

r2 5
segue que r 2 5 = 2 6, ou 6 =
.
2
(b) Pelo mesmo argumento,

s2 = ( 215 + 300)2 = 215 + 2 215 300 + 300

= 515 + 10 43 60 = 515 + 10 2 580

> 515 + 10 2 500 = 515 + 500 = 1 015.


119. Duas rodas Enquanto a roda A d 1 200 voltas, a roda B d 1 500 voltas ou,
equivalentemente, a roda A d 4 voltas a cada 5 voltas da roda B. Denotemos por R o
raio da roda A e por r o raio da roda B. O comprimento da roda A 2R e o da roda
B 2r, portanto, o comprimento de 4 voltas da roda A 4 (2R) e o comprimento
de 5 voltas da roda B 5 (2r). Como esses dois comprimentos so iguais, temos
que 4R = 5r. Por outro lado, na figura vemos que 2(r + R) = 9, de modo que
5  
9
5
r = r,
9 = 2(r + R) = 2r + 2 r = 2 +
4
2
2
e, assim, estabelecemos que r = 2 cm e R = 2,5 cm.

120. Dois divisores A opo correta (c).


O nmero N = 248 1 muito grande mas, mesmo assim, podemos descobrir vrios
de seus divisores. Para isso, utilizamos a igualdade

(x 1) xa1 + xa2 + + x + 1 = xa 1.

Notamos que os divisores de 48 so 1, 2, 3, 4, 6, 8, 12, 24 e 48. Tomando x = 22 e


a = 24 na igualdade acima, estabelecemos que x 1 = 22 1 = 3 um divisor de
(22 )24 1 = 248 1 = N. Analogamente, tomando x = 23 e a = 18, estabelecemos que
x 1 = 23 1 = 7 um outro divisor de (23 )18 1 = 248 1 = N. Procedendo dessa
maneira, verificamos que, para qualquer divisor d de 48, o nmero 2d 1 um divisor
de N. Dessa forma, conclumos que 22 1 = 3, 23 1 = 7, 24 1 = 17, 26 1 = 63,
28 1 = 251, e assim por diante, so divisores de N.

Alm desses, podemos ainda obter outros divisores de N considerando os divisores


pares d = 2n de 48 e usando o produto notvel


2
2d 1 = 2n 1 = 2n + 1 2n 1 .

Como 2d 1 um divisor de N, tambm 2n + 1 um divisor de N. Por exemplo,


d = 4 = 2 2 fornece o divisor 22 + 1 = 5 de N, d = 6 = 2 3 fornece o divisor
23 + 1 = 9 de N, d = 8 = 2 4 fornece o divisor 24 + 1 = 17 de N, d = 12 = 2 6
fornece o divisor 26 + 1 = 65 de N, e assim por diante.
Note que j obtivemos dois divisores de N = 248 1 entre 60 e 70, a saber, 63 e 65.
OBMEP 2010

297

Solues do Nvel 3
Observao: Com o auxlio de um computador, podemos ver que N , realmente, um
nmero muito grande, j que N = 248 1 = 281 474 976 710 655, e obter sua fatorao,
dada por

N = 248 1 = 32 5 7 13 17 97 241 257 673.


Empregando o argumento exposto na resoluo deste exerccio, possvel encontrar
vrios divisores de nmeros bastante grandes.

121. Rede de estaes O exemplo mostra que podemos conectar pelo menos sete estaes dentro das condies propostas. Comeamos com uma estao particular, e vamos
pensar nela como se fosse a base da rede. Ela pode ser conectada a uma, duas, ou
trs estaes, conforme mostra o primeiro dos dois diagramas a seguir. As estaes A,
B e C tm, ainda, duas linhas no utilizadas, portanto, podem ser conectadas a duas
outras estaes, como no segundo dos dois diagramas a seguir.

..................
...
..
...
... .....
............
...
...
...
...
....... .........
.....
..
.
...
.... .....
..... ..................
.
.
.
.
.....
...
.
.
.
.
.....
..
.....
.....
................
.....
....
..................
...
....
..
..
.....
.
...
.
.... .....
..... .....
..........
........

Base

..........
.................
... ......
...
...
.
....
....
..
...
...
........... .......
.
........................
.
.
....
.....
...
.
.
.
.... .......... ...
......... ........
...
....
..
...
................
..
...
..
...
..................
...
...
....
....... ........
..... .....
......
.....
.....
.....
.
.
.
.
.....
......
.....
.....
.
.................
.
.
.
.
.
.
.
.
.
.
.
.
.
.
.
.
.
.
.
.
.
.
...............
.
.
....
.
.
...
.. .....
...
.
..
.....
...
.
..
...
...
.
...
.
....
.
.
.
.
.
.
.
.
.
.
.
.
.
.
.
.
.
.
.
.
.
.
.
.
.
.
.
.
.
.
.
.
.
.
...
.
.
...
......................................... .....
.
.
.
....... ........
.
.
.
.
.
.
.
.
.
.
.
.
..........
.........
.........
..
...
...
...
...
....
....
..
..
...
...
.
.
.
.
.
.
.
.
.
.
.
.
.
.
... ......
.... ..........
.
.
....
..
..
...
..
.
.
...
.... ......
.................
............

Base

Agora, impossvel acrescentar mais estaes, pois qualquer outra a mais no poderia
ser conectada base satisfazendo as condies do problema. Isso mostra que no
podemos ter mais do que 10 estaes.
Vejamos, agora, se possvel montar uma rede com essas 10 estaes. Observe, no
diagrama acima, que apenas a base conectada a todas as outras estaes (atravs de
um cabo ou de uma conexo via uma estao). As estaes que esto nos extremos
ainda possuem duas linhas no utilizadas, e agora vamos us-las para fechar a rede.
Veja o diagrama a seguir.
298

OBMEP 2010

Solues do Nvel 3
..
....................
................
............
...........
...
..
...........
............
...
..
...........
............
..
...........
..
...........
.
.
.
.
.
...........
.
.
.
...
.
.
.
....
........... .................
...
...
.
.
..............
.
.
.
.
.
.
...
.
.
.
.
.
.
.
.
.
.
.
.
...........
.........
.
...
.
.
.
.
.
.
.
.
.
.
.
.
.
.
.
.
....
........... .......
..
.............
.
.
.
.
.
.
.
.
.
.
.
.
.
.
.
...
.
.
...
..
...
.
...
...
.
...
.
.
.
.
.
..
..
...
.
...
.
.
.
.
....
.
..... ...........
.. ....... ........
..
.
.
...
.
........ ....
.... ...
..
...
....
.
.
..
.
...
.
.
.
.... ............ ....
...
...
..
.
.
.
.
.
...
.
.
.....
.....
...
..
..
..
.
...
.
.
.
...
...
..
..
..
.
...
..
.
...
..... .....
..
.
..
.
.
.
.
.
.
.
.
.
.
..
..
...
.
.
.
.
.
...
.
..
...
..
..
..
.
.
.
...
..
...
..
.
.
....
.
...
..
..........
.
.
..
.
.
.
...
.
.
...
...
...
.
...
..
...
.
.
.
.
...
...
..
..
...
.
.
....
...
..... ....
..
...
...
.
.
.
.
.
.
.
...
... ...... .......
.
.
..
...
.
...
.
.
.....
...
....
..
.
..
.
.
.
.
...
.
.
.
.....
..
...
.
...
.
.
.
.
.
.
..
...
.
.
..... ...
.....
.
...
.
.
...
.
.
.
.
.
.
.
.
.
.
.
.
.
.
.
.
.
.
.
.
.
.
.
.
.
.
.
.
.
.
.
.
.
.
...
.
.
.
.
.
.
.
.. .....
.. .....
.. .....
.. .....
...
.
.
.
.
.
.
.
.
.
.
...
...
.
..
...
....
.......................................
..
....
.
.
.
.
.
.
.
.
.
.
.
.
.
.
.
.
.
.
.
.
.
.
.
.
.
.
.
.
.
.
.
.
.
...
.
.
.
.
..
.... ....
.... .....
..... .....
...... ......
.
...
.
.
.
.
.
.........
......... ..
.........
..
..
.. .......
.
...
.
.
.
.
...
...
...
....
....
....
..
...
.
.
.
.
.
.
.
.
.
.
.
...............
.
...
.......
.
.
.
.
.
.
.
.
.
.
.
.
.
.
.
............... .
..................
..
...
...............
.
.
.
.
.
.
.
.
.
.
..
......... ...
.
.
.
.
.
.
.. ......................
.
.
.
.
.
.
.
.
.
.
.
.
.
.
.
.
.
.
.
.
.
.
.
.
.
.
...
.. ...
...............
.........
... ......
.
.................... ......
...
..
................
...
....
.................
...................................................................
...
...
...
.... ......
..
...
.................
..........
..
...
...
...
...
..
...
..........................................................................................................................................................................................................................................................................

Base

122. Bolas brancas e pretas A opo correta (b).


Inicialmente observe que, depois de cada substituio, o nmero de bolas brancas, ou
permanece o mesmo, ou decresce de dois. Logo, o nmero de bolas brancas permanece
par. Por outro lado, cada grupo de bolas removidas que contm pelo menos uma bola
branca substitudo por outro grupo que tambm contm uma bola branca, portanto,
o nmero de bolas brancas nunca zero. Agora observe que a opo (b) a nica
que inclui pelo menos duas bolas brancas, logo ela a opo correta. Um modo de
obter esse resultado remover trs bolas brancas 49 vezes at obter 149 pretas e duas
brancas e, depois, remover uma preta e duas brancas 149 vezes.
123. O cubo Seja a a aresta do cubo que Alice quer construir. Como a rea lateral do
cubo mede 6a2 cm2 , devemos ter 6a2 25 60, isto , a2 250 e, portanto, a < 16.
Com a = 15 temos 4 = 60 15 quadrados de lado medindo 15 cm e sobra um retngulo
de 60 por 10 cm. Podemos cortar fora um retngulo de 60 por 2,5 cm e os pedaos
marcados com , de dimenses 15 por 7,5 cm. Assim, na figura, a linha pontilhada
indica dobradura e a linha contnua indica corte; com os dois pedaos de cartolina
marcados com formamos a tampa.

7, 5

124. Um quadrado e um tringulo


Soluo 1: Indiquemos por S1 , S2 e S3 as reas dos tringulos XAB, AY D e
BCZ e por Sq a rea do quadrado ABCD, conforme indicado na figura. Se S
denota a rea
 do tringulo XY Z, ento S = S1 + S2 + S3 + Sq e como, por hiptese,
Sq = 7/32 S, estabelecemos que
OBMEP 2010

299

Solues do Nvel 3

S1 + S2 + S3
Sq
S Sq
=
= 1
S
S
S
7
25
=1
=
.
32
32

...
........ ....
...
.........
.........
...
........
.
.
.
...
.
.
.
.
.
.
........
...........................................................................................
.
.
.
.
.
.
.
.
.
.
...
..... ....
.
.
.
.
.
.
... ....
.
.
......
.
.
.
.
.
.
.
.
.
... ....
..
........
....
... ....
........
.
.
.
.
.
.
...
.
.
...
.
.
......
.
.
...
.
.
.
.
.
.
...
.
.
......
...
.
.
.
.
.
.
.
.
...
.
.
...
....
.
.
.
.
.
.
.
.
.
.
.
.
...
.
.
......
.
.
.
.
.
.
.
.
.
.
.
...
.
.
......
.
.
.
.
.
.
.
...
.
.
.
.
.
.
....
.
.
.
...
.
.
.
.
.
.
.
.
.
.
.
......
...
.
.
.
.
.
.
.
.
.
.
.
...
.
.
......
.
.
.
.
.
.
.
.
.
.
.
..
.
.
....
.
.
.
.
.
.
.
.
.
.
.
.
...............................................................................................................................................................................................................................................................................................................

S1

Sq

S2

S3

Como XAB e XY Z so tringulos semelhantes, a razo entre suas reas igual


ao quadrado da razo de semelhana, isto ,
2

S1
XA
.
=
S
XY
Transladando horizontalmente o tringulo BCZ de modo a justap-lo ao tringulo
ADY, obtemos um tringulo semelhante a XY Z, mas de rea S2 + S3 . Assim,

2
S2 + S3
AY
=
.
S
XY
Somando esses dois quocientes obtidos e usando a proporo estabelecida acima, conclumos que
25
S1 + S2 + S3
(XA)2 + (AY )2
(XA)2 + (XY XA)2
=
=
=
32
S
(XY )2
(XY )2

2


(XY )2 + 2(XA)2 2(XY )(XA)
XA
XA
=
=1+2
2
,
(XY )2
XY
XY
ou seja, a razo entre XA e XY procurada satisfaz a equao de segundo grau

2


XA
XA
7
2
2
+
= 0.
XY
XY
32
Usando a frmula de Bhaskara, obtemos dois valores possveis para essa razo, a saber,
7 XA
1
XA
= e
= .
XY
8 XY
8
Soluo 2: Denotemos o comprimento do lado do quadrado ABCD por l, a altura
do tringulo XY Z por H, a altura do tringulo XAB por h e o comprimento do
lado Y Z por b. A rea do quadrado l2 e a rea do tringulo XY Z 21 bH. Como
os tringulos XY Z e XAB so semelhantes, temos
X

.
.............
........ ... ....
........
... ...
........
. ...
........
.
.
.
.
.
.
.
.
.
.
. ....
......
.
..
........
................................................................................................
........ ....
.
.
.
.
.
.
.
... ....
.
.
......
.
.
.
.
.
.
.
.
... ....
.
..
.
........
...
... ...
........
.
.
...
.
.
.
.
.
.
.
....
.
......
...
.
.
.
.
.
.
.
.
..
.
...
.
.....
.
.
.
.
.
.
.
.
...
.
...
.
......
.
.
.
.
.
.
.
.
.
.
.
...
.
.
.....
.
.
.
.
.
.
.
...
.
.
.
.
.
.
.....
.
.
.
.
.
.
...
.
.
.
.
.
.
.
.
......
...
.
.
.
.
.
.
.
.
.
.
.
.
.
...
......
.
.
.
.
.
.
.
.
.
.
.
...
.
.
....
.
.
.
.
.
.
.
.
.
.
.
.
.
...
.......................................................................................................................................................................................................................................................................................................................

l
h
XA
=
=
.
b
H
XY
Y

300

OBMEP 2010

Solues do Nvel 3
Hl
1
H2 l
(h + l)2 l
Portanto, b =
, a rea do tringulo XY Z bH =
=
e a razo
h
2
2h
2h
procurada dada por
XA
h
h
1
=
=
=
,
XY
H
h+l
1 + hl
de modo que resta calcular a razo l/h.
2

l
do tringulo XY Z e a rea l2 do quadrado
Como a razo entre a rea (h+l)
2h
32/7, obtemos 7(h + l)2 = 64hl. Expandindo e dividindo por h2 , obtemos a equao
quadrtica
l
 l 2
50
7
+ 7 = 0,
h
h
que tem solues

l
50 502 4 49
25 252 72
25 24
=
=
=
.
h
14
7
7

XA
tem dois possveis
Assim, l/h tem dois possveis valores, 1/7 e 7 e, portanto,
XY
7 XA
1
XA
= e
= .
valores,
XY
8 XY
8
125. A urna
Soluo 1: Os pares
{1, 2}, {1, 3}, {1, 4}, {1, 5}, {1, 6},
{2, 3}, {2, 4}, {2, 5}, {2, 6}, {3, 4},
{3, 5}, {3, 6}, {4, 5}, {4, 6}, {5, 6}
so os nicos pares de bolas diferentes que podem ser retirados da urna. Logo, podem
ser retirados da urna 5 + 4 + 3 + 2 + 1 = 15 diferentes pares de bolas. Dentre esses,
existem apenas 5 pares de bolas numeradas com nmeros que diferem por uma unidade,
a saber, {1, 2}, {2, 3}, {3, 4}, {4, 5} e {5, 6}. Assim, a probabilidade que um desses pares
5
1
seja retirado
= .
15
3
Soluo 2: Observemos que se extrairmos a primeira bola com um nmero entre
2 e 5, ento dentre as cinco bolas que ficam na urna, temos duas possveis bolas que
cumprem a condio do problema, portanto, nesse caso, a probabilidade que a segunda
bola cumpra a condio 52 e a probabilidade que a primeira bola tenha um nmero
entre 2 e 5 46 . Por outro lado, se a primeira bola extrada for 1 ou 6, s temos uma
bola na urna que cumpre a condio, portanto, nesse caso, a probabilidade para a
escolha da segunda bola 15 e a probabilidade da primeira bola ser 1 ou 6 62 . Assim,
a probabilidade das bolas serem consecutivas
4 2 2 1
5
1
+ =
= .
6 5 6 5
15
3
126. Soma das razes de uma equao Devemos considerar dois casos. Se x + 1 0,
ento |x + 1| = x + 1 e a equao x2 + 3x + 2 = x + 1, ou seja, x2 + 2x + 1 = 0,
que s possui a soluo x = 1. Se x + 1 < 0, ento |x + 1| = x 1 e a equao
OBMEP 2010

301

Solues do Nvel 3
x2 + 3x + 2 = x 1, ou seja, x2 + 4x + 3 = 0, que tem apenas a soluo x = 3 no
intervalo x < 1. Assim, as nicas solues distintas da equao dada so 1 e 3,
de modo que a soma de todas as razes distintas da equao 4.
127. Produto de trs nmeros Sejam a, b, c, . . . , i, j os nmeros nos 10 crculos, conforme indicado no diagrama.
.....................
...
..
.
....
.... ......
.............

................. ....................
....

................. .................... ....................


....
. .

.................. ..................... .................... .....................


....
.

. .
. .
... ..
... ..
...
. .
.
. .
.
.
.
.
.
.
.. ..
.. ..
..
a ..............b.................. .............c.................. .............d.................. ..............e................. .............f................. = .............g.................. .............h
... ... i ... ... j ...
........... ................. .................

Observe que a, c e f no podem ser zero, pois 0 x = 0, para qualquer x. entretanto,


o produto dos trs nmeros um nmero de quatro algarismos, portanto, a b d < 10 e
os nmeros que aparecem em a b d so 1,2 e 3 ou 1,2 e 4. Observemos que a segunda
opo impossvel de ocorrer, porque o mnimo produto que podemos obter nesse caso
1 23 456 = 10 488 , de modo que a b d = 6 e o produto maior do que 6 000.
Tampouco pode a ser 2 ou 3, porque nesse caso o mnimo valor que tem o produto
2 14 356 = 9 968 e os outros produtos ficam maiores do que 10 000. Assim, a = 1.
Continuando essa anlise, chegaremos soluo dada no diagrama.
.....................
...
...
...
...
..
....................

............
.............
.... .... .... ....

................. .................... ...................


.
....
. .

.................. ..................... ................... ....................


.
.
....
.

.
. .
. .
.
. .
. .
.
.
. .
.
.
. .
1 ..............2................................6................. ..............3.................. ..............4...............................5................. = ..............8................................9.................. ..............7...............................0..................

128. rea do tringulo Para determinar a rea de um tringulo, basta conhecer o comprimento de uma base e sua
respectiva altura. Tomando AC como base, a altura corta
AC no ponto H = (1, 0), j que o segmento AC vertical
e o segmento HB horizontal. Assim, a base AC mede 8
unidades e a altura BH relativa
 a essa base mede 7. Logo,
a rea do tringulo 12 7 8 2 = 28 unidades de rea.

A...........= (1, 2)

.. .......
.......
...
........
...
.......
...
.......
........
...
.......
...
.....
...
...
.....
..
.....
...
.
.
.
...
...
.
.
.
.
...
....
.....
...
.....
...
.....
... ........
.
... .....
...

Ht

B = (8,
0)
x

C = (1, 6)

129. Duas tabelas Vemos que na primeira tabela cada linha uma progresso aritmtica
de razo 3 e cada coluna uma progresso aritmtica de razo 7.
5
12
19
26
33

8
15
22
29
36

11
18
25
32
39

14
21
28
35
42

17
24
31
38
45

39
87
56

Digamos que na segunda tabela a razo das progresses aritmticas das linhas seja a
e a das colunas seja b. Assim, obtemos 39 + 2a + 2b = 87 e 39 2a + 3b = 56.
302

OBMEP 2010

Solues do Nvel 3
392a
392a + b
392a + 2b
56

39a

39

39+a

39+2a
39+2a + b
87

Somando essas duas equaes, resulta 78 + 5b = 143, donde b = 13. Subtraindo as duas
igualdades que anteriormente foram somadas, obtemos 4a b = 31. Segue que a = 11.
Assim, o nmero que estava na posio era 39 + a + 4b = 39 + 11 + 4 13 = 102.
130. A sequncia abc Sabendo que 30 = 2(10 + a), obtemos que a = 5. Assim, b =
2(30 + a) = 2(30 + 5) = 70 e, portanto, c = 2(b + 30) = 2(70 + 30) = 200.
131. Permetro e diagonal A opo correta (b).
Denotando por a e b os comprimentos dos lados do retngulo, temos 2a + 2b = 20, de
modo que a + b = 10. O quadrado do comprimento da diagonal, dado pelo Teorema
de Pitgoras, d2 = a2 + b2 . Mas,
(a + b)2 + (a b)2 = (a2 + 2ab + b2 ) + (a2 2ab + b2 ) = 2a2 + 2b2 = 2d2
e (a + b)2 = 100, portanto d2 = 50 12 (a + b)2 . Assim, vemos que o mnimo do

comprimento da diagonal ocorre quando a = b, caso em que d = 50.


132. As idades numa classe Denotemos por a a idade comum dos alunos e por n o
nmero de alunos dessa classe. Temos sete alunos com a 1 anos, dois com a + 2 anos
e os demais, ou seja, n 9 alunos, com a anos. Logo, a soma das idades de todos os
alunos, que 330, pode ser desdobrada em 330 = 7(a1)+2(a+2)+(n9) a = na3,
de modo que na = 330 + 3 = 333 = 9 37.
Como a classe tem mais do que 9 alunos, ento a = 9 e n = 37, ou seja, a classe tem
37 alunos.

133. A mesa redonda O permetro de mesa ampliada


140 + 40 6 140 3,14 + 240 = 679,60 cm.
Se cada convidado precisa de 60 cm de espao, podero sentar-se mesa, no mximo,
679,60
11,3,
60
ou seja, 11 convidados.
134. Brincadeira com sete nmeros
Soluo 1: Os sete nmeros podem ser escritos como
n 3, n 2, n 1, n, n + 1, n + 2, n + 3 .
|
{z
}
|
{z
}
3n6

3n+6

Observando que 3n 6 + 12 = 3n + 6, estabelecemos que n = 12. Logo, os nmeros


so 9 + 10 + 11 + 12 = 13 + 14 + 15.
OBMEP 2010

303

Solues do Nvel 3
Soluo 2: Sejam n + 1, n + 2, . . . , n + 7 os sete nmeros consecutivos tais que
(n + 1) + + (n + k) = (n + k + 1) + + (n + 7)
para algum k entre 1 e 6. Como todos os nmeros esquerda da igualdade so menores
do que os nmeros direita, existem mais parcelas esquerda, portanto, k = 6, 5 ou
4. Tomando k = 6, obtemos 6n + 1 + 2 + 3 + 4 + 5 + 6 = n + 7, ou seja, 5n + 14 = 0,
que no tem soluo inteira. Tambm k = 5 d 5n + 1 + 2 + 3 + 4 + 5 = n + 6 + n + 7,
ou seja, 3n + 2 = 0, que no tem soluo inteira. Finalmente, com k = 4 obtemos
4n + 1 + 2 + 3 + 4 = 3n + 5 + 6 + 7, portanto, n = 8 e 9 + 10 + 11 + 12 = 13 + 14 + 15
a nica soluo.
135. Um terreno compartilhado Como as reas dos tringulos ABM e ADN so iguais, temos
1
(BM
2

AB) = 21 (ND AD).

Como o terreno quadrado, temos AB = AD, de

B ...................................................................................................................... A

M
C

... . .
....
...... ... ..
...
...... .. ..
..
..... .... ....
.....
.
.
.
...
.
.
...
.
....
...
...
...
......
...
...
...
......
..
.....
.
.
.
.
...
.
.
....
.
....
.
.
.
...
.
..
.
.
..
.
.
.
... ..........
...
...
........
.
...
..
.
....
....
..
..
..
.
.
.
...
...
.
...
...
...
.
.
.
...
...
.
.
.........................................................................................................

modo que BM = DN e, portanto, a figura AMCN simtrica em relao diagonal


AC. Logo, a rea do tringulo ACN a metade da rea do tringulo ADN.
Agora, como esses tringulos tm a mesma altura, resulta DN = 2NC e, por simetria,
BM = 2MC. Conclumos que a distncia ao vrtice C do quadrado dos pontos M e
N deve ser 1/3 do comprimento do lado do quadrado.
136. As duas partculas Denotemos as partculas por A e B e seja v a velocidade da
partcula B. Supondo que A seja a mais rpida, temos que v + 2 a velocidade de A.
Assim, o tempo que B demora para dar uma volta 120/v e o tempo que A demora
120/(v+2). Como esse tempo trs segundos inferior ao de B, temos a equao bsica
120
120
3 =
.
v
v+2
Simplificando, isso equivale a v 2 + 2v 80 = 0, cuja raiz positiva
v=


1
2 + 4 + 320 = 1 + 81 = 8.
2

Portanto, a velocidade da partcula mais lenta 8 m/s e a da mais rpida 10 m/s.


137. Queda livre No primeiro segundo, o corpo percorre 4,9 m e, como a distncia
percorrida aumenta 9,8 m a cada segundo em relao ao segundo anterior, o corpo
percorre 4,9 + 9,8 m no segundo segundo, 4,9+29,8 m no terceiro segundo, 4,9+39,8
m no quarto segundo e assim por diante, at o dcimo primeiro segundo, em que o
corpo percorre 4,9 + 10 9,8 = 102,9 m. A distncia total percorrida pelo corpo at
o impacto
4,9 + (4,9 + 9,8) + (4,9 + 2 9,8) + + (4,9 + 10 9,8)
= 4,9 11 + 9,8(1 + 2 + + 10) = 53,9 + 9,8 55 = 592,9 m.
304

OBMEP 2010

Solues do Nvel 3
138. Um caminho triangular Se v representa a velocidade constante com que Janete
caminha, ento v = (1 992 m)/(24 min) = 83 m/min. Janete percorre o outro lado
BC e a hipotenusa CA com a mesma velocidade de v = 83 m/min e gasta 2 horas e
46 minutos, o que igual a 166 min. Assim, BC + AC = 83 166 = 13 778 m.
Mas, pelo Teorema de Pitgoras, a hipotenusa CA do tringulo satisfaz
(CA)2 = (1 922)2 + (BC)2 . Da decorre que
(1 992)2 = (CA)2 (BC)2 = (CA + BC)(CA BC)
= 13 778 (CA BC),

portanto CA BC = 1 9922/13 778 = 288 m. Assim, CA + BC = 13 778 e


CA BC = 288. Subtraindo, obtemos 2BC = 13 778 288 = 13 490 e, portanto,
BC = 12 13 490 = 6 745 m.
139. O preo do feijo A opo correta (a).
Se b denota o preo final e a o preo inicial de um bem, ento a variao b a e o
aumento percentual
ba
.
a
Observe que os valores intermedirios do bem no alteram a variao do aumento
percentual num certo perodo. Usando apenas os dados de janeiro e de abril da tabela
dada, obtemos os aumentos percentuais do
37,66
103,33 65,67
=
= 0,57 = 57%;
feijo A:
65,67
65,67
109,50 73,30
36,20
feijo B:
=
= 0,49 = 49%;
73,30
73,30
100,00 64,50
35,50
=
= 0,55 = 55%.
feijo C:
64,50
64,50
Portanto, o maior aumento percentual de preo foi o do feijo A e o menor foi o do
feijo B.
140. Interseo de tringulos Quando acrescentamos um novo tringulo a uma figura
constituda de tringulos, ele corta cada um dos lados dos tringulos que j existiam
em, no mximo, dois pontos. Inicialmente, comeando com um s tringulo, no
temos ponto de interseo algum. Acrescentando um segundo tringulo, introduzimos,
no mximo, 2 3 = 6 pontos de interseo. Do mesmo modo, introduzindo um terceiro
tringulo, introduzimos, no mximo, mais 2 6 = 12 pontos de interseo. Logo, trs
tringulos se intersectam em, no mximo, 6 + 12 = 18 pontos. A figura mostra que
esse caso de 18 pontos de interseo pode acontecer.
........
..... ..
...... ...
.
......
..... ...........................................
.
.
.
.
.
...
.
.. .
.......
..........................
.. ....
.. ..........................................................
.
.
.
.
....... ...... .....
. ...
............................
......
... ..
... ..
..........
......
..... ..........
.
.. ...
.
.
.
.
.
.....
..
.
..
......
.....
.......
....
.....
.. ..
... ... .....
......
.. ..
.. ........
..... .... ...
...............
.
.
.
.
.
.
.
.
.
.
.
.
.. .
..
..
... .............
.......
..
.....
.......
..
...
.......... ....
..
..
.. ......
..
..
...
... ..........
............
.
.
.
.
.
..
..
.....
.....
.
.
.
.
..
.
.
.
...
.
..........................................................................................
........................................................................................................
..... ..
....
..... ...
.
...
.......
..
.
.
.
.
.......
.
..
.
.
.. .......
.
.
...
.. ...... .....
.
..
..... ...
.
...
..
..
..
.........
...
..
.. ..........
...
.. ...
.....
.
.
.. ...
.....
.....
.. ..
..
.....
..... ...
........
.

s s s

s
s

s s
s

OBMEP 2010

305

Solues do Nvel 3
141. Comparar tringulos De acordo com os dados
do problema, temos
AB
AC
BC
2
=
=
= .
AC
AD
CD
3

Br

rC

.............................................
.....................
....
.....
....
.................
....
.
........
.....
.
.
.........
.....
.
.
.
...
.
.
.
.
.
.
.
.....
..
.......
.
.
.
.....
.
.
.
.
.
.
.
.....
..
......
.
.
.
.
.
.
.
.
.
.
.....
..
.....
.
.
.
.
.....
.
.
.
.
.
.
.....
..
......
.
.
.
.
.
.
.
.
.
.
.....
.. .............
.
.....
.
.
.....
..................
.
.
....
...........
.
.
.
.
.............................................................................................................................................................................................................

12

18

27

12

Segue que os tringulos ABC e ACD tm seus lados proporcionais e, portanto,


b = C AD.
b
so semelhantes. Em particular, obtemos B AC

142. Queima de velas A opo correta (c).

Como o verdadeiro comprimento das velas irrelevante, podemos estipular qualquer


tamanho para as velas, que a resposta ser, sempre, a mesma. O mais simples supor
que ambas velas tm comprimento igual a uma unidade. Assim, a que queima em 3
horas queima velocidade constante 1/3 (de vela por hora) e a que queima em 4 horas
queima velocidade 1/4 (de vela por hora). Logo, depois de algum tempo (em horas)
t, uma queima t/3 (de vela) e a outra t/4 (de vela), de modo que o que sobra de uma
vela depois de um tempo t 1 t/3 e da outra, 1 t/4. Queremos saber quanto tempo
decorre desde o instante t = 0 at o momento em que o comprimento da vela que
queima mais lentamente o dobro do comprimento da que queima mais rapidamente,
o que equivale a resolver a equao

t
2t
t
=2 .
1 = 2 1
4
3
3
1
5
2
12
2
, a nica soluo t =
= 2 horas, o que equivale a 2 horas
Como =
3
4
12
5
5
e 24 minutos. Assim, depois de 2 horas e 24 minutos, o comprimento de uma vela
o dobro do comprimento da outra. Como queremos que isso acontea s 16 horas, as
velas devem ser acesas s 13 horas e 36 minutos.

143. Uma distrao A opo correta (b).


Soluo 1: Seja x o nmero com que Jlia se distraiu. Ela deveria ter obtido 6x mas,
com sua distrao, obteve x/6. Logo, seu erro foi de 6 x x/6 = (35/6)x e, portanto,
(35/6)x
35
em termos percentuais, seu erro foi de
=
0,9722 = 97,22%.
6x
36
Soluo 2: Se N o valor que Jlia deveria ter obtido ento, com seu erro, ela
encontrou N/36, de modo que o erro absoluto cometido foi de N N/36 = (35/36)N
35
e, portanto, o erro relativo foi de
100% = 97,22%.
36
144. Problema de nota Seja c o nmero de problemas resolvidos corretamente e seja e a
soma do nmero de problemas resolvidos incorretamente e de problemas no resolvidos.
Logo, c+e = 80 e 5c3e o nmero de pontos do aluno na avaliao. No caso presente,

c + e = 80
5c 3e = 8
Resolvendo o sistema, encontramos c = 31 e e = 49. Logo, o aluno resolveu 31
problemas corretamente.
306

OBMEP 2010

Solues do Nvel 3
145. Quadrados e tringulos
(a) Os nicos quadrados que no tm nenhum de seus lados paralelos nem reta r,
nem reta s, so os do tipo 1 e os do tipo 2 (ver figuras).

Assim, h um total de seis quadrados, sendo quatro do tipo 1 e dois do tipo 2.


(b) Digamos que a distncia vertical ou horizontal entre dois pontos contguos do reticulado seja igual a uma unidade.
O total desses tringulos

dezesseis, cada um
deles com catetos iguais a 5 unidades e hipotenusa de 10 unidades. De fato,
cada um dos quadrados do tipo 2, como visto em (a), nos d quatro tringulos,
por diviso ao longo de cada uma das duas diagonais obtendo, assim, oito tringulos. Os oito tringulos restantes so obtidos atravs de uma nica translao
horizontal ou vertical de cada um dos anteriores. Na figura, exemplificamos a
nica translao possvel de um dos quatro tringulos obtidos de um quadrado
do tipo 2.

146. Clculo de reas


(a) A rea hachurada corresponde a um quarto da rea de um crculo de raio r,
1
portanto, a rea hachurada igual a r 2 .
4
(b) Observe que a rea da regio marcada com X, que no est hachurada na figura
(a), igual rea do quadrado todo, diminuda da rea da regio hachurada, ou
seja,
1
1
rea da regio marcada com X = r 2 r 2 = (4 )r 2 .
4
4
X

X
X

OBMEP 2010

307

Solues do Nvel 3
Voltando ao item (b), a rea da regio hachurada na figura (b) igual rea do
quadrado todo, menos duas vezes a rea da regio marcada com X, ou seja,
igual a
1
1
rea da regio hachurada = r 2 2 (4 )r 2 = r 2 r 2 .
4
2
147. Sequncia de algarismos Os nmeros com um algarismo formam os 9 primeiros
termos da sequncia. Os 90 nmeros de dois algarismos formam os 180 termos seguintes. Depois vm os 2 700 termos correspondentes aos nmeros de trs algarismos,
seguidos pelos 36 000 termos correspondentes aos nmeros de quatro algarismos e finalmente, os 450 000 termos que so os correspondentes aos nmeros de cinco algarismos.
Logo, enumerando os termos da sequncia, obtemos 488 889 termos.
a1 , . . . , a9 , a10 , . . . , a189 , a190 , . . . , a2 889 , a2 890 , . . . , a38 889 , a38 890 , . . . , a488 889
| {z } |
{z
} |
{z
} |
{z
} |
{z
}
1 alg

2 algs

3 algs

4 algs

5 algs

Para escrever todos os termos de 1, 2, 3 e 4 algarismos, chegamos 38 889a posio


da sequncia. Logo, o algarismo na 206 788a posio faz parte de um nmero de cinco
algarismos, ou seja, est no bloco
a38 890 , . . . , a488 889 .
|
{z
}
5 algs

Esse bloco da forma 10 000, 10 001, . . . 99 999. Para ver quantos nmeros de cinco
algarismos existem desde a posio 38 889 at a posio 206 788, dividimos essa diferena por 5. Assim, 206 788 38 889 = 167 899 e 167 899 = 5 33 579 + 4. Portanto,
precisamos de 33 579 nmeros de cinco algarismos, mais os quatro primeiros algarismos do 33 580o nmero de cinco algarismos, que 43 579, para chegar ao algarismo
na posio 206 788.
Como o quarto algarismo do nmero 43 579 7, temos que o algarismo procurado 7.
148. Soma constante Para resolver este problema, o mais fcil comear dispondo os
nmeros 1, 2, 3, 4, 5, 6, 7, 8 e 9 numa tabela 3 3 de modo que a soma de cada linha,
de cada coluna e de cada diagonal seja 15. Depois basta somar 662 a cada elemento
da tabela, obtendo, por exemplo, a soluo seguinte.
670
663
668

665
667
669

666
671
664

Assim como existem outras maneiras de dispor os nmeros 1, 2, 3, 4, 5, 6, 7, 8 e 9 na


tabela, tambm existem outras solues desse problema.
149. Contando os zeros Inicialmente, verificamos como terminam as potncias de 9,
ou seja, listamos os dois ltimos algarismos, os da dezena e da unidade, das potncias
9n , ordenadamente.
9n

308

Se n for
termina com

0
01

1
09

2
81

3
29

4
61

5
49

6
41

OBMEP 2010

7
69

8
21

9
89

10
01

11
09

12
81

Solues do Nvel 3
Assim, vemos que os dois ltimos algarismos de 910 , 911 e 912 so os mesmos de 90 , 91 e 92 .
A partir 910 , os dois ltimos algarismos das potncias comeam a se repetir, formando uma
sequncia peridica de perodo 10. Como 2 007 = 10 200 + 7 e os dois ltimos algarismos de
910200 so 01, segue que os dois ltimos algarismos de 92 007 so os dois ltimos algarismos
de 97 , ou seja, 69. Ento, os dois ltimos algarismos de 92 007 + 1 so iguais a 69 + 1 = 70.
Assim, existe um nico zero no final do nmero 92 007 + 1.
150. Crculos dentro do quadrado A resposta desse problema afirmativa: possvel colocar
um certo nmero de crculos sem superposio dentro de um quadrado de 1 centmetro de
lado, de tal forma que a soma dos raios desses crculos seja maior do que 2 008 centmetros.
Para exibir uma tal configurao, desenhamos linhas paralelas aos lados do quadrado, dividindoo em n2 quadradinhos menores; cada um desses quadradinhos tem lado igual a 1/n. Dentro
de cada um desses quadradinhos, desenhamos um crculo inscrito de raio igual a 1/2n. No
caso particular n = 4, essa construo dada na figura.
2
n = 42 = 16 crculos

lados dos quadradinhos = 4


1

raio dos crculos =

soma dos raios = 16 1 = 4 = 2


8
2

Desse modo, a soma dos raios dos n2 crculos igual a n2 1/2n = n/2. Como estamos
interessados no caso desta soma ser maior do que 2 008, devemos ter n/2 > 2 008, ou seja,
n > 4 016. Logo, dividindo o quadrado em 4 0172 quadradinhos (ou mais), a soma dos raios
dos crculos inscritos nos quadradinhos ser maior do que 2 008.
151. Construindo um nmero As condies dadas implicam que os nmeros devem satisfazer
todas as condies seguintes.
(i)

... 1

(ii)

... 2

(iii)

... 3

(iv)

... 4

1 ...
2 ...
3 ...
4 ...

Vamos estudar as possveis posies dos dois algarismos 4 em um nmero de oito algarismos.
De acordo com (iv), existem apenas trs possibilidades:
caso A:
4
4
caso B:
4
4
caso C:
4
4
Em cada um desses casos, existem duas possibilidades de colocar os algarismos 3:
caso A:

caso B:

3 4

caso C:

4
3

3
4
4

ou

ou

ou

3 4

4 3
3

4 3
3

Na tentativa de colocar os algarismos 1 e 2, percebemos que as duas possibilidade do caso B


so impossveis, bem como as primeiras possibilidades dos casos A e C. Os nicos casos que
levam a solues do problema so as segundas possibilidades dos casos A e C, que levam s
duas nicas solues
41 312 432 e 23 421 314.

OBMEP 2010

309

Solues do Nvel 3
152. Nmero na circunferncia Na figura a seguir representamos os nove algarismos escritos
ao redor da circunferncia.

a1
a9

a2

a8

a3

a4

a7
a5

a6

Lendo os algarismos escritos ao redor da circunferncia, de trs em trs, no sentido horrio,


obtemos os seguintes nmeros de trs algarismos cada:
a1 a2 a3 , a 2 a3 a4 , a 3 a4 a5 , a 4 a5 a6 , a 5 a6 a7 , a 6 a7 a8 , a 7 a8 a9 , a 8 a9 a1 e a9 a1 a2 .
Para somar esses nmeros usamos o algoritmo da adio, como indicado a seguir.

a1 a2 a3
a2 a3 a4
a3 a4 a5
a4 a5 a6
a5 a6 a7
a6 a7 a8
a7 a8 a9
a8 a9 a1
a9 a1 a2
???????

Analisando esses nove nmeros, notamos que todos tm os algarismos da unidade diferentes.
Logo,
a3 + a4 + a5 + a6 + a7 + a8 + a9 + a1 + a2 = 1 + 2 + 3 + 4 + 5 + 6 + 7 + 8 + 9 = 45 .
Do mesmo modo, esses nove nmeros tambm tm todos os algarismos das dezenas e todos
os algarismos das centenas diferentes. Logo, a soma dos algarismos das dezenas tambm
45 e o mesmo ocorre com os algarismos das centenas. Portanto, para somar basta calcular
45 + 45 10 + 45 100 = 4 995. Assim, a soma dos nove nmeros 4 995.
153. Cada pea em seu lugar A primeira informao certamente falsa, pois se fosse verdadeira, o ouro estaria no cofre 2 ou 3, mas deveria estar no prprio cofre 1, para que a
primeira informao fosse verdadeira. Essa contradio mostra que o ouro no est nem no
cofre 2 nem no cofre 3. Por ser falsa a informao na porta do cofre 1, conclumos que o outro
tambm no est nele. A segunda informao certamente falsa, pois se fosse verdadeira,
o ouro estaria no cofre 2, o que incorreto. Logo, a primeira e a segunda informaes so
falsas. Portanto, o ouro no est no cofre 1, nem no 2 nem no 3, e a prata no est no cofre
1.

310

OBMEP 2010

Solues do Nvel 3
As nicas possibilidades que restam para o ouro so os cofres 4 ou 5. Se o ouro estivesse no
cofre 4,
, |{z}
, |{z}
, ouro
,
|{z}
|{z} , |{z}
1

a informao 4 seria a correta e o nquel estaria na cofre 3. Ento a terceira informao deve
ser falsa e deveramos ter o bronze tambm no cofre 3, o que uma impossibilidade. Logo,
essa possibilidade fica descartada e o ouro deve estar no cofre 5.
, |{z}
, |{z}
, |{z}
, ouro
|{z}
|{z} .
1

De fato, com o ouro no cofre 5, a informao 5 a correta e a platina est no cofre cujo
nmero superior, em uma unidade, ao que contm o bronze. Pela afirmao do cofre 3,
que falsa, teramos o bronze no cofre 3. Logo, a platina est no cofre 4. Como a segunda
afirmao falsa, a prata no est no cofre 1, s podendo estar no cofre 2. Portanto, temos
a soluo seguinte.
nquel , prata , bronze , platina , ouro .
| {z } | {z } | {z } | {z } |{z}
1

154. Soma de quadrados Como a razo da progresso aritmtica 2, os trs nmeros podem
ser denotados por n 2, n e n + 2. A soma de seus quadrados um nmero de quatro
algarismos iguais, digamos, k k k k, em que k algum inteiro entre 1 e 9, ou seja,
k k k k = (n 2)2 + n2 + (n + 2)2 = 3n2 + 8 .
A partir deste ponto, apresentamos duas possibilidades de soluo.
Soluo 1: Como k k k k 8 = 3n2 um mltiplo de 3, o resto da diviso de k k k k por 3
igual ao resto da diviso de 8 por 3, que 2. Mas k k k k = k 1 111 e o resto da diviso
de 1 111 por 3 1, de modo que o resto da diviso de k por 3 2. Como 1 k 9, s
podemos ter k igual a 2, 5 ou 8. Como na Soluo 1, os casos k = 2 e k = 8 so impossveis e
k = 5 a nica opo, caso em que n = 43 e os trs nmeros procurados so 41, 43 e 45, que
constituem a nica soluo para o problema.
Soluo 2: Como k k k k = 3n2 + 8, obtemos
3n2 = k k k k 8 = (k k k 10 + k) (9 1) = (k k k 10 9) + (k + 1).
Mas kkk 10 9 = k k (k 1) 1 mltiplo de 3, portanto tambm k + 1 um mltiplo de
3. Como 1 k 9, s podemos ter k igual a 2, 5 ou 8. No caso k = 2, obtemos
n2 =

2 222 8
= 738 = 2 369,
3

o que impossvel, pois 738 no um quadrado perfeito. Analogamente, se k = 8, obtemos


n2 =

8 888 8
= 2 960 = 24 5 37,
3

o que, novamente, impossvel, j que esse ltimo nmero no um quadrado perfeito. Resta,
portanto, a ltima opo, k = 5. Nesse caso,
n2 =

5 555 8
= 1 849 = 432 ,
3

portanto, n = 43 e os trs nmeros em progresso aritmtica procurados so 41, 43 e 45, que


constituem a nica soluo para o problema.

OBMEP 2010

311

Solues do Nvel 3
155. Adivinhe o nmero
Soluo 1: Seja x o nmero procurado. Observe que x + 2 divisvel por 3, 4, 5 e 6. O
menor mltiplo comum desses nmeros 60. Logo, x + 2 = 60 e, ento, x = 58.
Soluo 2: Seja x o nmero procurado. O resto da diviso de x por 3 1. Portanto, x
da forma x = 3a + 1, com a 0 inteiro. A seguir, queremos determinar de que forma deve
ser a para que x = 3a + 1 deixe resto 2 na diviso por 4, isto , para que 3a deixe resto 1
na diviso por 4. Qual deve ser o resto da diviso de a por 4? Por um lado, se esse resto for
3, ento a da forma a = 4b + 3, de onde segue que 3a = 12b + 9 = 4 (3b + 2) + 1 deixa,
de fato, resto 1 na diviso por 4. Por outro lado, podemos verificar que qualquer outro resto
no funcionaria. Se, por exemplo, a deixasse resto 2 na diviso por 4, teramos a = 4b + 2 e
3a = 12b + 6 = 4 (3b + 1) + 2 deixaria resto 2 e no 1 na diviso por 4.
Substituindo a = 4b+3 em x = 3a+1, obtemos x = 12b+10. Usamos agora que x deixa resto
3 na diviso por 5. Como 10 mltiplo de 5, 12b tambm deixa resto 3 na diviso por 5. Mas
12b = 10b + 2b e 10b mltiplo de 5. Logo, 2b deixa resto 3 na diviso por 5. Ento, b deixa
resto 4 na diviso por 5. De fato, por um lado, se b = 5c+4, ento 2b = 10c+8 = 5(2c+1)+3
deixa realmente resto 3 na diviso por 5. Por outro lado, como acima, podemos verificar que
4 o nico resto que funciona. Ento, b = 5c + 4 e x = 12b + 10 = 12 (5c + 4) + 10 = 60c + 58.
Conclumos que as solues do problema so os nmeros x da forma x = 60n + 58, com n 0
inteiro. O menor deles, para n = 0, x = 58.

156. Um cdigo Observe que


AOBM EP = AOB 1 000 + M EP e M EP AOB = M EP 1 000 + AOB.
Denotemos AOB = m e M EP = n. Pelos dados do problema, temos
6 AOBM EP = 7 M EP AOB,
donde 6 (1 000 m + n) = 7 (1 000 n + m), ou 6 000 m 7 m = 7 000 n 6 n ou, ainda,
5 993 m = 6 994 n. Dividindo ambos os lados por 13, conclumos que 461 m = 538 n. A
fatorao de 538 em fatores primos 538 = 2 269 e 461 primo. Portanto, 538 e 461 so
primos entre si. Logo, 461 divide n e 538 divide m. Como AOB e M EP so nmeros de trs
algarismos, s podemos ter as solues n = 461, ou n = 822, e m = 538. fcil verificar que
6 538 461 = 3 230 766 = 7 461 538 e que 6 538 822 = 3 232 932 6= 5 757 766 = 7 822 538.
Portanto, n = 822 no serve, sendo AOB = 538 e M EP = 461 a nica soluo. Assim, os
algarismos so A = 5, O = 3, B = 8, M = 4, E = 6 e P = 1.
157. Calculando distncias
b De fato, o tringulo ABC equiltero,
Soluo 1: Observe que conhecido o ngulo ABD.

b
b
b = 150 . Assim, aplicando a Lei
portanto, ABC = 60 e, como C BD = 90 , obtemos ABD
dos Cossenos ao tringulo ABD, resulta

AD 2 = 32 + 42 2 3 4 cos 150 = 25 + 24 cos 30 = 25 + 12 3 .


p

Segue que AD = 25 + 12 3 cm.

Soluo 2: Seja E o ponto sobre a reta BD tal que o tringulo AEB seja retngulo no
vrtice E (veja figura). Nesse tringulo, temos

3
EB
1
AE
EB
AE
= cos 30 =
=
e
= sen 30 =
=
,
2
AB
3
2
AB
3

312

OBMEP 2010

Solues do Nvel 3
portanto,

3 3
EB =
2

3
e AE = ;
2

3 3
em particular, ED = 4 +
.
2

C
3
A

3
E

Aplicando o Teorema de Pitgoras no tringulo AED, obtemos


2

AD = AE + ED =
Segue que AD =

25 + 12 3 cm.

 3 2
2

 8 + 33 2
2

= 25 + 12 3.

158. Calculando lados de um tringulo Como o tringulo ABC equiltero, seus ngulos
so todos iguais a 60 . Sobre o lado CB desse tringulo, construmos um novo tringulo
b = P CB
b e ABP
b = C BP
b (girando o
CBP , congruente ao tringulo ABP, tal que P AB

tringulo ABP no sentido horrio por 60 em torno do ponto B).

C
3 cm

60

P
5 cm
4 cm

3 cm
60

4 cm
60

b congruente ao ngulo ABC,


b ou seja, mede 60 . Assim, se traarmos
Note que o ngulo P BP

o segmento P P , temos que o tringulo P BP , que j issceles, pois


P B = BP = 4 cm , de fato, equiltero e, em consequncia, temos que P P = 4 cm.
OBMEP 2010

313

Solues do Nvel 3
C
3 cm

60

5 cm
4 cm

4 cm

3 cm

4 cm

60

60

c C por a, aplicamos a Lei dos Cossenos ao tringulo CP P e obtemos


Denotando o ngulo P P
52 = 32 + 42 2 3 4 cos a,

ou seja, 25 = 25 24 cos a. Segue que cos a = 0 e, portanto, a = 90 . Dessa forma, estabelec B = a + 60 = 90 + 60 = 150 .
cemos C P
C

3 cm

60

P
150

5 cm

3 cm

4 cm

4 cm

60

60

Agora, denotando o lado do tringulo ABC por l, aplicamos a Lei dos Cossenos ao tringulo
CBP e obtemos

l2 = 32 + 42 2 3 4 cos 150 = 25 + 12 3.
p

Segue que 25 + 12 3 cm o comprimento dos lados do tringulo equiltero ABC.

159. Amigo oculto Primeiramente observemos que o nmero de formas de distribuir os presentes
sem nenhuma restrio 5! = 5 4 3 2 1 = 120. Da temos que tirar os casos ruins,
isto , os casos em pelo menos um amigo tirou o seu prprio presente. Esses casos a eliminar
podem ser listados pelo nmero de amigos que tiram seu prprio presente.
Os 5 amigos ficaram com seus prprios presentes. S h uma possibilidade de acontecer
isso.
Exatamente 4 amigos ficaram com seus prprios presentes. Isso no possvel.

Exatamente 3 amigos ficaram com seus prprios presentes. Nessa situao, os outros
dois amigos trocam os presentes. Assim, escolhemos 3 pessoas dentre as 5, isto ,
543
= 10 possibilidades.
32

314

OBMEP 2010

Solues do Nvel 3
Exatamente 2 amigos ficaram com seus prprios presentes. Nesse caso, escolhemos 2
54
= 10 possibilidades. Os outros 3 amigos trocam os
pessoas dentre as 5, isto ,
2
presentes entre si, dando um total de 10 2 = 20 possibilidades.

Por ltimo, exatamente um amigo ficou com seu prprio presente. Nesse caso, escolhemos uma pessoa dentre um total de 5, multiplicando pelo nmero de formas que os
outros amigos no fiquem com seu presente, que so 9 maneiras. Logo, nessa situao,
temos um total de 5 9 = 45 possibilidades.

Portanto, temos 120 45 20 10 1 = 44 maneiras de distribuir os presentes sem que


algum fique com seu prprio presente.
160. Contando solues A equao dada equivalente a xy = 144(x + y) = 144x + 144y,
144y
portanto, isolando x, obtemos x =
. Como x e y devem ser inteiros positivos, o
y 144
denominador y 144 deve ser um nmero inteiro positivo, digamos, y 144 = n. Substituindo
essa expresso no valor de x, obtemos
x=

1442
144(n + 144)
= 144 +
.
n
n

Como x deve ser um nmero inteiro, n deve ser um divisor de 1442 .


Sendo
1442 = 124 = 28 34 , seus divisores so os nmeros d da forma d = 2a 3b , com 0 a 8 e
0 b 4. Como h 9 valores possveis para a e 5 valores possveis para b, conclumos que
1442 tem 9 5 = 45 divisores.
Assim, para cada divisor n de 1442 , obtemos uma soluo



1442
(x, y) = 144 +
, n + 144
n

xy
= 144 dada. Portanto, essa equao possui 45 pares de nmeros inteiros
x+y
positivos (x, y) que a satisfazem.
da equao

161. Determinando uma sequncia Sejam a1 , a2 , . . . , a80 os nmeros dessa sequncia. Para
cada i 1, temos

ai+1 = ai ai+2 ,
ai+2 = ai+1 ai+3 .

Consequentemente, ai+1 = ai ai+1 ai+3 e, como ai+1 6= 0, j que o produto dos termos da
sequncia 8 6= 0, segue ai ai+3 = 1.

Quaisquer dois nmeros da sequncia, cujos ndices distem 3 um do outro, so tais que o
seu produto igual a 1. Portanto, o produto de seis nmeros consecutivos nessa sequncia ,
sempre, igual a 1. Sendo o produto dos 40 primeiros termos da sequncia igual a 8, conclumos
que o produto dos quatro primeiros termos tambm 8, pois os 36 termos restantes formam
seis grupos de 6 termos consecutivos da sequncia e, em cada grupo desses, o produto igual
a 1. Isto , a1 a2 a3 a4 = 8. Como ai ai+3 = 1, segue que a1 a4 = 1 e, da, a2 a3 = 8.
Temos, tambm, a hiptese de que os 80 termos da sequncia tm produto igual a 8, donde
podemos concluir que a1 a2 = 8, j que os 78 ltimos termos podem ser agrupados em 13
grupos de 6 termos consecutivos, cada um com produto igual a 1, como j vimos.
Ento, de a2 a3 = 8, a1 a2 = 8 e a2 = a1 a3 , segue que a1 a22 a3 = 64 e a32 = 64. Assim,
a1 = 2, a2 = 4 e a3 = 2.
OBMEP 2010

315

Solues do Nvel 3
Observe, ainda, que a sequncia inteira est, agora, determinada. De fato, temos
1 1 1
1 1 1
2, 4, 2, , , , 2, 4, 2, , , , . . . ,
2 4 2
2 4 2
em que os seis primeiros termos ficam se repetindo, sempre na mesma ordem.
162. Construindo uma cerca A soma dos comprimentos dos trs lados (os que no so de
pedra) 140 m.
(a) Se os dois lados vizinhos ao muro de pedra tm 40 m cada um, os dois juntos tm 80 m
e o terceiro lado ter 140 80 = 60 m.

(b) Se o maior dos lados a ser cercado tiver 85 m, ele no pode estar ser vizinho ao muro
de pedras, porque nesse caso esses dois lados mediriam 85 2 = 170 m, que maior do
que 140 m. Logo, ele deveria ser paralelo ao muro de pedra e, nesse caso, cada um dos
outros lados mediria 27,5 m, o que tambm no possvel, j que a cerca composta
de pedaos inteiros de 1 m cada um.

Os dois lados que encostam no muro de pedra podem ter 65 m cada um porque nesse
caso, o outro teria 140 2 65 = 10 m, o que no contraria as condies dadas.
163. Um quadriltero especial Como os tringulos ABC e ACD so retngulos e tm a
mesma hipotenusa AC, pelo Teorema de Pitgoras temos x2 + 112 = y 2 + 72 , onde AB = x
e DC = y. Ento,
(y x)(y + x) = y 2 x2 = 72 = 23 32
e, portanto, y x e y + x so divisores de 72. Para cada fatorao de 72, precisamos resolver
um sistema de duas equaes com duas incgnitas, como na tabela a seguir, e identificar os
casos em que existem solues inteiras.
Fator de 72
y+x yx
72
1
36
2
24
3
28
4
12
6
9
8

Medidas de
x
y
17
19
12
16
3
9
-

Observaes
No h soluo inteira
Possui soluo inteira
No h soluo inteira
Possui soluo inteira
Possui soluo inteira
No h soluo inteira

Assim, h trs solues inteiras para o comprimento dos lados x e y.


164. Trs quadrados Os tringulos retngulos AEB e EHF so congruentes, pois seus
b e F EH
b so iguais (lados respectivos perpendiculares) e as hipotenusas so
ngulos E BA
iguais (lados de um quadrado). Ento, AE = F H. Pelo Teorema de Pitgoras,
rea de BEF G = BE 2 = AB 2 + AE 2 = AB 2 + F H 2 = 30 + 20 = 50 cm2 .

165. Bolinha de gude


Soluo 1: Denotemos por x, y e z o nmero de bolinhas que cada um tinha no incio da
partida. Temos

316

OBMEP 2010

Solues do Nvel 3
Incio
1a rodada
2a rodada
3a rodada

1o
x
xyz
2(x y z)
4(x y z)

2o
y
2y
2y 2z (x y z)
2(3y x z)

3o
z
2z
4z
4z 2(x y z) (3y x z)

Como cada um terminou a partida com 64 bolinhas, segue que

4(x y z) = 64

2(3y x z) = 64

4z 2(x y z) (3y x z) = 64
donde

x y z = 16
x + 3y z = 32

x y + 7z = 64

Para resolver o sistema, somamos a primeira com a segunda equaes e a primeira com a
terceira, obtendo

y z = 24;
y + 3z = 40.

Da, obtemos z = 32 e y = 56, portanto, x = 16 + 56 + 32 = 104. Assim, o primeiro jogador


comeou a partida com 104 bolinhas, o segundo, com 56 e o terceiro, com 32.

Soluo 2: Preenchemos a tabela de baixo para cima, isto , do final para o incio do jogo.
Comeamos com 64 nas trs casas finais.

Incio
Aps a 1a rodada
Aps a 2a rodada
Aps a 3a rodada

1o

2o

3o

64

64

64

Como os dois primeiros jogadores dobraram a quantidade de bolinhas na terceira rodada,


cada um tinha 32 bolinhas e o terceiro jogador deu 32 a cada um deles. Deduzimos que ele
possua 64 + 32 + 32 = 128 bolinhas.

Incio
Aps a 1a rodada
Aps a 2a rodada
Aps a 3a rodada

1o

2o

3o

32
64

32
64

128
64

Quem perdeu a segunda rodada foi o segundo jogador. Logo, a tabela era

Incio
Aps a 1a rodada
Aps a 2a rodada
Aps a 3a rodada

1o

2o

3o

16
32
64

32+16+64=112
32
64

64
128
64

Finalmente,
OBMEP 2010

317

Solues do Nvel 3
Incio
Aps a 1a rodada
Aps a 2a rodada
Aps a 3a rodada

1o
16 + 56 + 32 = 104
16
32
64

2o
56
32 + 16 + 64 = 112
32
64

3o
32
64
128
64

Assim, o primeiro jogador comeou a partida com 104 bolinhas, o segundo, com 56 e o terceiro,
com 32.
166. Uma soma Inicialmente, observe que
1
1
=1 ,
12
2

Assim, temos

S =1
e, portanto, S = 1

1
1
1
=
. Logo,
k (k + 1)
k k+1

1
1 1
= ,
23
2 3

...,

1
1
1
=

.
2 007 2 008
2 007 2 008

1 1 1 1 1
1
1
1
1
+ + + ... +

2 2 3 3 4
2 006 2 007 2 007 2 008
2 007
1
=
.
2 008
2 008

167. Dobrando papel Sejam E e F os pontos de interseo, como na figura. Sejam AB = 2a e


BC = 2b. Ento AM = M B = DN = N C = a e M E = EN = b. Traamos AN e denotamos
por P o ponto de interseo dos segmentos AN e BD. Os segmentos AN e M C so paralelos
(pois AM = N C e AM k N C). Como M o ponto mdio de AB e M F k AP, temos que F
o ponto mdio do segmento P B. Analogamente, P o ponto mdio do segmento DF e segue
que DP = P F = F B. Por simetria, verificamos que P E = EF e, ento, EF/F B = 1/2. Por
outro lado, rea(M BE) = 41 rea(ABD) = 125 e, como M EF e M BE tm a mesma
altura relativamente ao vrtice M e a base do primeiro 1/3 da base do segundo, conclumos
que
1
rea(M EF ) = 125 cm2 .
3
168. Uma rea Os tringulos ABM e ABC tm a mesma altura d em relao s respectivas
bases AM e AC. Como M o ponto mdio de AC, obtemos
1
2
1
2

rea (ABM )
=
rea (ABC)
de modo que
rea (ABM ) =

AM d
AM
1
=
= ,
AC
2
AC d

1
1
rea (ABC) = 100 = 50 cm2 .
2
2
A
a

M
P

318

D
OBMEP 2010

Solues do Nvel 3
Teorema das Bissetrizes Internas

Analogamente,

rea (ABP )
BP
=
.
rea (ABM )
BM

a a

Pelo Teorema das Bissetrizes Internas,


BP
AB
10
2
=
=
= ,
PM
AM
15
3

D
AC =AB
8 BD
8
DC

3
portanto, P M = BP. Ento obtemos
2

rea (ABP )
BP
2
BP
BP
BP
= 5
= ,
=
=
=
3
rea (ABM )
BM
BP + P M
5
BP + 2 BP
2 BP
de modo que rea (ABP ) =

2
2
rea (ABM ) = 50 = 20 cm2 .
5
5

169. ltimos algarismos


Soluo 1: Como s queremos saber os dois
ltimos algarismos, basta conhecer as duas ltimas colunas dessa soma (a das dezenas e a das
unidades), ou seja,
8 + 88 2 007 = 8 + . . . 16.
Como 8 + 16 = 24, os dois ltimos algarismos do
nmero so 24.

.............

8 ........
8 8 ........
8 .8 8. ........
.. .. ......
2 008
.. .. ....... parcelas
. . ......
8 . . . . . . 8 8 .......
8 . . . . . . 8 8 ........
88 . . . . . . 8 8 .................
................................................................................................

8
8.
..
..
.
8
8
8

8.............
8 .......
8. ........
.. ......
2 007
.. ....... parcelas
. ......
8 ........
8 ........
8...............

............................................

........................................................................................................

Soluo 2: Observemos que os dois ltimos algarismos do nmero dado so iguais aos dois
ltimos algarismos do nmero
2 007

z
}|
{
8 + 88 + + 88 = 8 + 2 007 88,

que tambm coincidem com os dois ltimos algarismos de 8 + 7 88 = 624. Logo, os dois
ltimos algarismos do nmero procurado so 24.
170. Idades mltiplas Quando Isabel tem a anos, sua me tem 20 + a anos. Se a divisor
de 20 + a, ento (20 + a)/a = (20/a) + 1 um nmero inteiro e, consequentemente, 20/a
tambm inteiro. Ento, a um divisor de 20 e, portanto, a pode ser 1, 2, 4, 5, 10 ou 20.
Assim, temos um total de 6 vezes em que as idades das duas so mltiplos.
Isabel
Me

1
21

2
22

4
24

5
25

10
30

20
40

Observe que, depois dos 20 anos de Isabel, nunca mais a idade da me ser um mltiplo da
idade de Isabel.
171. Blocos diferentes O volume do cubo 10 10 10 = 1 000 cm3 . O volume V de um
bloco o produto de suas trs medidas, altura (= a), largura (= l) e comprimento (= c).
OBMEP 2010

319

Solues do Nvel 3
Para construir cada bloco, Ana deve usar todos os bloquinhos, portanto, o volume de cada
bloco
V = altura largura comprimento = a l c = 1 000 cm3 .
Assim, precisamos saber de quantas maneiras podemos escrever 1 000 como o produto de trs
nmeros inteiros positivos a, l e c. Para isso, fatoramos 1 000, obtendo a l c = 1 000 = 23 53 .

Soluo 1: Podemos encontrar todos esses nmeros listando as dimenses a, l e c dos blocos.
Sem perda de generalidade, podemos supor que a l c. Ento a3 alc 1 000 e, portanto,
a 10. Logo, a = 1, 2, 4, 5, 8 ou 10. Mas se a = 8, ento lc = 125 = 53 e, como 8 l c, no
h como obter medidas inteiras para l e c. Assim, a s pode ser 1, 2, 4, 5 ou 10. A tabela
mostra as 19 possibilidades para esses blocos.
a

l
1
2
4
5
8
10
20
25
5
10

c
1 000
500
250
200
125
100
50
40
50
25

10

l
2
4
5
10
20
5
8
10

c
250
125
100
50
25
40
25
20

10

10

Soluo 2: Podemos encontrar todos esses nmeros listando as potncias de 2 e 5, sem


esquecer que uma das medidas pode ser 1 (no caso de potncia 0). A tabela mostra as 19
possibilidades para esses blocos.

320

potncia de 2
3

potncia de 5
3

1, 2

1, 1, 1
3

3
1, 2

3
1, 2

1, 1, 1
1, 2

1, 2
1, 1, 1
1, 1, 1

1, 1, 1
1, 2
1, 1, 1

a
1
1
1
1
2
2
1
1
23
5
1
1
2
22
22
2
5
5
25

OBMEP 2010

l
1
23
2
22
22
2
3
2 5
23 52
5
5
25
2 52
5
25
2 52
22 5
25
25
25

c
53
53
22 53
2 53
53
2 53
52
5
52
23 5
22 52
22 5
22 52
52
5
52
22 5
2 52
25
23

Solues do Nvel 3
172. Quadro negro Inicialmente observe que, de 1 a 77, Joana apagou 11 mltiplos de 7 e 7
mltiplos de 11. Como 77 mltiplo de 7 e de 11, ento ela apagou 11 + 7 1 = 17 nmeros,
sobrando 77 17 = 60 nmeros. Agora, agrupando os 10 000 primeiros nmeros em grupos
de 77 nmeros consecutivos, esse raciocnio se aplica em cada uma das linhas abaixo, isto ,
em cada linha sobraram 60 nmeros.
1a linha: 1,
2, . . . , 77
a
2 linha: 78, 79, . . . , 154
3a linha: 155, 158, . . . , 231
..
..
..
..
.
.
.
.
Como, 2 008 = 33 60 + 28, sabemos que entre os primeiros 33 77 = 2 541 nmeros,
33 60 = 1 980 nmeros ficaram sem apagar.
..
..
..
..
.
.
.
.
a
33 linha: . . . , . . . , . . . , 2 541.
Ainda faltam contar 28 nmeros. Vamos, ento, examinar a 34a linha, que comea com 2 542.
Como os nmeros apagados esto nas colunas 7, 11, 14, 21, 22, 28, 33, 35, etc., e at a 35a
coluna foram apagados oito nmeros, restam 35 8 = 27 nmeros na 34a linha. Logo, depois
de apagados os mltiplos de 7 e de 11 nessa linha, o 28o nmero 2 577. Assim, o nmero na
2 008a posio o 2 577.
173. Conjunto sem mltiplos Inicialmente, observemos que o conjunto {51, 52, 53, . . . , 100}
tem 50 elementos e nenhum de seus elementos mltiplo de outro. Assim, o subconjunto
com o maior nmero de elementos e que satisfaz a propriedade exigida tem, no mnimo, 50
elementos. Para concluir que 50 o maior nmero possvel de elementos de um subconjunto
que satisfaa a propriedade exigida, basta mostrar que todo subconjunto com mais de 50
elementos possui dois nmeros mltiplos. Para isso, denotamos B = {1, 2, 3, . . . , 100} e
dividimos B em 50 subconjuntos disjuntos, considerando os diversos subconjuntos de B cujos
elementos so do tipo nmero mpar 2k , com k natural. Como existem apenas 50 nmeros
mpares entre 1 e 100, obtemos cinquenta subconjuntos dois a dois disjuntos construdos dessa
forma, como segue.
A1 = {1, 2, 4, 8, 16, 32, 64} = B {1 2k | para algum k = 0, 1, 2, 3, . . .};
A2 = {3, 6, 12, 24, 48, 96} = B {3 2k | para algum k = 0, 1, 2, 3, . . .};
A3 = {5, 10, 20, 40, 80} = B {5 2k | para algum k = 0, 1, 2, 3, . . .};
..
.

A50 = {99} = B {99 2k | para algum k = 0, 1, 2, 3, . . .}.


Observe que B a unio desses cinquenta subconjuntos, isto ,
B = {1, 2, . . . , 100} = A1 A2 . . . A50 ,
e que, se dois elementos de B estiverem num mesmo subconjunto Ai , ento um deles mltiplo
do outro. Assim, se um subconjunto A de B tiver mais do que 50 elementos, podemos afirmar
que existem pelo menos dois elementos de A num mesmo subconjunto Ai e, portanto, um
deles mltiplo do outro. Isso prova que 50 o nmero mximo de elementos de qualquer
subconjunto de B que no possua dois elementos tais que um deles seja mltiplo do outro.

OBMEP 2010

321

Solues do Nvel 3
174. Brincando com a calculadora O resultado o mesmo nmero inicial a b c de trs algarismos. De fato, se a b c um nmero de trs algarismos, ento o nmero de seis algarismos
a b c a b c da forma a b c a b c = 1 000 a b c + a b c = 1 001 a b c.
Como 1 001 = 7 11 13, dividindo, sucessivamente, a b c a b c por 7, 11 e 13, obtemos
abcabc
1 001 a b c
=
= a b c.
7 11 13
1 001
175. No galinheiro Sejam x e y, respectivamente, o nmero de galinhas e de pintinhos no
galinheiro.
(a) Temos 4x + 2y = 240, ou seja, 2x + y = 120. Como 8 kg = 8 000 g, temos
160x + 40y 8 000. Assim, 4x + y 200.
Em resumo, os nmeros x de galinhas e y de
pintinhos satisfazem

2x + y = 120
()
4x + y 200

200

r:y = 200- 4x

150
(0,120)
100
50
0

10

20

30

40

50

60

70

s:y = 120 - 2x

(b) A reta 2x + y = 120 corta o eixo Ox em x = 60 e o eixo Oy em y = 120. A reta


4x + y = 200 corta o eixo Ox em x = 50 e o eixo Oy em y = 200. Os grficos dessas
retas esto dadas na figura, em que a desigualdade 4x + y 200 representada pela
regio sombreada. Observe que, na figura, as condies () so representadas pelo
segmento que liga os pontos P e (0, 120). As coordenadas do ponto P so a soluo do
sistema

2x + y = 120
4x + y = 200,
ou seja, x = 40 e y = 40 e, portanto, P = (40, 40).
(c) Temos que 2 20 + 80 = 120 e 4 20 + 80 200. Logo, x = 20 e y = 80 satisfazem as
condies () e, por isso, o galinheiro comporta, sim, 20 galinhas e 80 pintinhos. Agora,
2 30 + 100 6= 120, logo, x = 30 e y = 100 no satisfazem as condies () e, por isso,
o galinheiro no comporta 30 galinhas e 100 pintinhos.
(d) O nmero mximo de galinhas 40 e, nesse caso, teremos tambm 40 pintinhos. O
nmero mximo de pintinhos 120 e, nesse caso, teremos 0 galinhas.
176. Um nmero perfeito Se 231 1 um nmero primo, seu nico divisor prprio o nmero
1. Ento, os divisores prprios de 230 (231 1) so
1, 2, 22 , . . . , 229 , 230 , (231 1), 2(231 1), 22 (231 1), . . . 229 (231 1).
A soma S desses divisores
S = [1 + 2 + 22 + + 229 + 230 ] + (231 1)[1 + 2 + 22 + + 229 ] .
Em cada um dos dois colchetes aparece a soma Sn de uma progresso geomtrica de n termos,
sendo o primeiro termo igual a 1 e a razo igual a 2: o primeiro colchete S31 , com 31 termos e

322

OBMEP 2010

Solues do Nvel 3
o segundo S30 , com trinta termos. Usando a frmula da soma dos termos de uma progresso
geomtrica, obtemos
S31 =

231 1
230 1
= 231 1 e S30 =
= 230 1.
21
21

Ento a soma dos divisores prprios de 230 (231 1)


S = (231 1) + (231 1)[230 1] = (231 1)(1 + 230 1) = 230 (231 1).
Logo, essa soma igual a 230 (231 1), como queramos provar.
177. Quinze minutos a mais Sabemos que a velocidade a razo da distncia percorrida
pelo tempo gasto.
Soluo 1: Denotando por t o tempo gasto, em horas, pelo carro mais lento, o que faz a
viagem a uma velocidade de 60 km/h, sabemos que o tempo gasto pelo outro carro de
t 1/4, j que 15 minutos um quarto de hora. Como ambos percorrem a mesma distncia,
segue que 60 t = 70 (t 1/4), portanto, t = 7/4 horas, ou 1 hora e trs quartos de hora.
Logo, a distncia entre as duas cidades 60 7/4 = 105 km.
Soluo 2: Vamos representar por d a distncia, em quilmetros, entre as cidades A e B e
por T o tempo gasto, em horas, pelo carro mais veloz. Como o outro carro gasta 15 minutos
a mais para fazer o mesmo percurso, temos que o tempo gasto pelo carro mais lento igual
a T + 0, 25 horas, pois 15 min = 0,25 h. Como o carro mais veloz anda a 70 km/h, temos
70 = d/T e, como o mais lento anda a 60 km/h, temos 60 = d/(T + 0, 25). Assim,
d = 70 T = 60(T + 0, 25),
ou seja, T = 1,5 h, e a distncia entre as cidades A e B igual a d = 70 1,5 = 105 km.
178. Outros caminhos Qualquer que seja o trajeto de Jlia da sua casa at a escola, se ela
deseja seguir um caminho mais curto, ela deve percorrer exatamente oito quarteires para a
direita e cinco quarteires para cima. Um caminho mais curto ligando a sua casa at a escola
, ento, uma sequncia de travessias de quarteires, sendo oito delas no sentido horizontal
(para a direita) e cinco no sentido vertical (para cima). Assim, para definir um caminho mais
curto, ela precisa apenas decidir em que ordem far essas treze travessias.
Para isso, imaginemos oito cartelas impressas com a letra D e cinco cartelas impressas com a letra C. Uma permutao qualquer dessas cartelas pode ser interpretada como um caminho mais
curto a ser percorrido por Jlia. Por exemplo, a sequncia de cartelas DDCDCCDDDDCDC
define o caminho indicado na figura dada.

OBMEP 2010

323

Solues do Nvel 3
Para determinar o nmero de maneiras pelas quais podem ser ordenadas essas cartelas, devemos contar de quantas maneiras diferentes se pode colocar cinco cartelas com a letra C
em uma fila com treze lugares vagos, sendo os demais oito lugares na fila ocupados com as
cartelas com a letra D.
Inicialmente, devemos escolher um dos treze lugares vagos para colocar uma letra C. Colocada
essa letra, sobram doze lugares vagos para a segunda letra C. Colocada essa letra, sobram onze
lugares vagos para a terceira letra, dez lugares para a quarta letra e, finalmente, nove lugares
para a quinta letra C. Agora, uma vez colocadas as cinco letras C, qualquer permutao dessas
letras entre si no altera a distribuio das letras na fila. Como a quantidade de permutaes
de cinco objetos 5! = 120, pelo princpio multiplicativo temos que o nmero de maneiras de
ordenar as treze cartelas dado por
13 12 11 10 9
= 1 287,
120
de modo que existem 1 287 caminhos mais curtos diferentes da casa de Jlia at a escola.
179. Escrevendo no tabuleiro Comeando com a letra A, ela pode ser escrita em qualquer
uma das nove casas do tabuleiro. Uma vez escrita a letra A, sobram seis casas nas quais pode
ser escrita a letra B. Uma vez escritas as letras A e B no tabuleiro, sobram trs casas para
a letra C ser escrita. Assim, pelo princpio multiplicativo, existem 9 6 3 = 162 maneiras
diferentes das letras A, B e C serem escritas no tabuleiro.
180. Frao e percentagem A opo correta (d).
Se um nmero x diminudo de 40%, ele passa a valer 60% de x, ou seja, 0,6x. Do mesmo
modo, quando um nmero y diminudo de 60%, ele passa a valer 0,4y. Portanto, a frao
x/y passa a ter o valor
0,6x
6x
x
=
= 1,5 ,
0,4y
4y
y
o que significa que a frao x/y aumentou 50% do seu valor.
181. Tringulos sobrepostos Os pontos A, B, C e D formam o retngulo ABCD.
7

C
4

Como as diagonais de um retngulo o dividem em quatro tringulos de mesma rea, a rea


sombreada igual
 a trs 2quartos da rea do retngulo ABCD. Assim, a rea sombreada
3
igual a 4 7 4 = 21 cm .

Vejamos, agora, o caso da outra figura. Sejam x = DE = CE, y = AE = BE e E o ponto


de interseo dos segmentos AC e BD.

C
x

x
E

A
324

B
OBMEP 2010

Solues do Nvel 3
A rea sombreada a soma das reas dos tringulos ADE e ABC, ou seja,
4x 47
+
= 2x + 14.
2
2
Logo, basta calcular x. Temos que x + y = 7 e, pelo Teorema de Pitgoras aplicado ao
tringulo AED, tambm y 2 = x2 + 42 . Substituindo y = 7 x nessa ltima equao, obtemos
(7 x)2 = x2 + 16, de modo que 49 14x + x2 = x2 + 16, ou seja,
x=

33
49 16
=
.
14
14

Finalmente, a rea sombreada dada por 2

33
33
131
+ 14 =
+ 14 =
cm2 .
14
7
7

182. Dois motoristas Sabemos que a velocidade a razo da distncia percorrida pelo tempo
gasto. Seja d a distncia entre as duas cidades A e B.
O primeiro motorista percorre a distncia de 2d velocidade constante de 80 km/h,
portanto, o tempo total gasto por esse motorista
t=

2d
d
=
horas.
80
40

O segundo motorista percorre a distncia d na ida uma velocidade constante de 90


km/h e, na volta, percorre a mesma distncia d velocidade constante de 70 km/h.
Logo, o tempo gasto na ida e volta
t=

d
d
16 d
8d
+
=
=
horas.
70 90
630
315

Como

d
8d
8d
=
<
,
40
320
315
verificamos que o motorista que viaja velocidade constante de 80 km/h o que gasta menos
tempo no percurso de ida e volta.
183. Soma e inverte Como 0 no o inverso de nmero algum, qualquer sequncia que comece
e termine em 0 deve ser dada por
+1

+1

0 1 1 0.
Uma sequncia dessas a seguinte.
1 +1 2 +1 5 +1 8 i
3 +1 5 +1 13 +1 21
+1
+1
+1
i
0 1 2 3

3
3
3
3
8
8
8
8
8 +1 13 i
21 +1
8 +1 5 i
13 +1 8 +1 3 +1 2


21
21
13
13
13
5
5
5
5
5 +1 3 +1 1 +1 1 i
i
+1
+1
2 1 0.
2
2
2
2
Uma outra sequncia, bem mais curta e simples , simplesmente,
i

+1

+1

0 1 1 0.
184. Carro flex
OBMEP 2010

325

Solues do Nvel 3
(a) Com cada litro de gasolina, que custa R$ 2,49, o carro roda 12,3 quilmetros. Logo, o
2,49
reais. Se o carro fizer y quilmetros por litro de
preo do quilmetro rodado de
12,3
1,59
lcool, o preo do quilmetro rodado com lcool de
reais. Para que a utilizao
y
do lcool seja mais vantajosa, financeiramente, necessrio que
1,59
2,49
<
,
y
12,3

ou seja, que y >

1,59 12,3
= 7,85.
2,49

Assim, o consumo desse carro com lcool deve ser maior do que 7,85 km/l.
(b) Supondo que o consumo do carro seja de x km/l de gasolina e de y km/l de lcool,
queremos saber quando o custo com gasolina maior do que o custo com lcool, isto ,
quando
1,59
2,49
>
,
x
y
o que acarreta 2,49 y > 1,59 x, ou seja, y > 0,64 x, j que x e y so valores positivos.
Um exemplo disso o carro como o do item (a), que consuma 12,3 km/l de gasolina e
8 km/l de lcool. Ou, ento, um carro que faa 10 km/l de gasolina e 7 km/l de lcool.
(c) Com cada litro de gasolina, que custa R$ 2,49, o carro roda x quilmetros. Logo, o
2,49
249
preo de 100 quilmetros rodados de g(x) = 100
=
com gasolina. Com cada
x
x
x
litro de lcool, que custa R$ 1,59, o carro roda + 1 quilmetros. Logo, o preo de 100
2
318
1,59
=
quilmetros rodados com lcool de a(x) = 100 x
.
x+2
2 +1
249
318
= g(x) = a(x) =
, ou seja,
x
x+2
249 (x + 2) = 318 x, cuja soluo x = 7,22 km/l, que deve ser o consumo com gasolina. Para que o custo seja o mesmo, o consumo do carro com lcool deve ser de
7,22
+ 1 = 4,61 km/l.
2
x
(e) Supondo que o consumo do carro seja de x km/l de gasolina e de + 1 km/l de lcool,
2
queremos saber quando o custo com lcool menor do que o custo com gasolina, isto
3,18
2,49
<
, o que acarreta 0,69 x < 4,98, ou seja, x < 7,22, j que x um
, quando
x+2
x
valor positivo. Assim, s financeiramente vantajoso abastecer com lcool se o consumo
com gasolina for menor do que 7,22 km/l. Um exemplo disso um carro que faa 6
km/l de gasolina e, portanto, 4 km/l com lcool: nesse caso, por exemplo, o custo de
100 quilmetros rodados com gasolina de g(6) = R$ 41,50 e com lcool de a(6) = R$
39,75. Observe que, a partir de um consumo de 7,22 km/l de gasolina, financeiramente
vantajoso abastecer s com gasolina; por exemplo, se o carro fizer 10 km/l de gasolina
e, portanto, 6 km/l de lcool, o custo de 100 quilmetros rodados com gasolina de
g(10) = R$ 24,90 e com lcool de a(10) = R$ 26,50.

(d) Para que o custo seja o mesmo, basta ter

Observao: Todos os valores utilizados nessas solues foram arredondados na segunda casa
decimal.
185. Contando tringulos Sejam A, B, . . . , K os onze pontos marcados, como na figura dada.

326

OBMEP 2010

Solues do Nvel 3
K
J
I
H
A

Dividiremos a contagem em trs casos.


(i) Um vrtice A. Nesse caso, um vrtice do tringulo deve estar no conjunto {H, I, J, K}
e o outro vrtice no conjunto {B, C, D, E, F, G}. Como existem quatro escolhas para um
vrtice e seis escolhas para o outro vrtice, a quantidade de tringulos com um vrtice
no ponto A 6 4 = 24.

(ii) Dois vrtices em {B, C, D, E, F, G}. O nmero de possveis escolhas de dois dentre esses
seis pontos
65
6!
C62 =
=
= 15.
4!2!
2
O outro vrtice do tringulo qualquer um dos quatro pontos H, I, J ou K. Da, a
quantidade desses tringulos 4 15 = 60.

(iii) Dois vrtices em {H, I, J, K}. O nmero de possveis escolhas de dois dentre esses quatro
pontos
43
4!
=
= 6.
C42 =
2!2!
2
Como o outro vrtice pode ser escolhido de seis maneiras diferentes no conjunto {B, C, D,
E, F, G}, resulta que a quantidade desses tringulos 6 6 = 36.

Logo, 24 + 60 + 36 = 120 a quantidade de tringulos cujos vrtices so tomados dentre os


onze pontos da figura.
186. Quadrado perfeito Seja x um nmero de oito algarismos, da forma x = 9999 .
Como o menor desses nmeros 99 990 000 e o maior 99 999 999, temos que
99 990 000 x 99 999 999.
Observemos que 10 0002 = 100 000 000 = 99 999 999 + 1 e que
2
9 9992 = 10 000 1 = 10 0002 20 000 + 1 = 99 980 001.

Isso mostra que 9 9992 < x < 10 0002 , ou seja, x est compreendido entre dois quadrados
perfeitos consecutivos. Assim, x no pode ser um quadrado perfeito, ou seja, no existe
algum quadrado perfeito da forma 9999 .

187. Diferena quase


nula

A
desigualdade
n

n 1 < 0,01 equivalente a

n < 0,01 + n 1. Como os dois lados da desigualdade so nmeros positivos, podemos


elevar ambos membros ao quadrado para obter a desigualdade equivalente

2
2
n < 0,01 + n 1 .

Mas isso equivale a n < 0,012 + 0,02 n 1 + n 1, donde obtemos

n1 >

1
1 100
1 0,012
1002 1
2
=
=
.
2
0,02
200
100

OBMEP 2010

327

Solues do Nvel 3
Elevando, novamente, ao quadrado os dois membros (no negativos) dessa desigualdade, obtemos
1004 2 1002 + 1
1
1
(1002 1)2
1002
=
=
+
n1 >
2
2
200
4 100
4
2 4 1002
ou seja,
1
1
n 1 > 2 500 +
2 40 000
e, finalmente,
1
1
n > 2 500 + +
.
2 40 000
Como 21 + 40 1000 < 1, temos que o menor nmero inteiro que satisfaz essa ltima desigualdade
2 501. Assim, estabelecemos que o menor nmero inteiro positivo que satisfaz a desigualdade
dada o nmero 2 501.
188. Conjunto de Cantor
(a) De acordo com a definio do conjunto de Cantor, temos o desenho seguinte.
C1
C2
C3

0
b

1/3
b

0 1/9 2/9 1/3

2/3
b

2/3 7/9 8/9 1

(b) 1/3 uma extremidade de C2 , portanto, pertence ao conjunto de Cantor.


3/81 = 1/27 e 1/27 uma extremidade de C4 , portanto, 3/81 pertence ao conjunto
de Cantor. 4/9 est entre 1/3 e 2/3, portanto, est no tero central de C1 , que foi
removido de C2 ; assim, 4/9 no pertence ao conjunto de Cantor. 4/81 est entre 1/27 e
2/27, portanto, est no tero central do primeiro segmento de C3 , que foi removido de
C4 ; assim, 4/81 no pertence ao conjunto de Cantor.
(c) Observe que C1 tem comprimento 1, C2 tem comprimento 2/3, C3 tem comprimento 4/9,
C4 tem comprimento 8/27 e C5 tem comprimento 16/81. Assim, os comprimentos de
C1 , C2 , C3 , . . . , Cn , . . . formam uma progresso geomtrica de razo q = 2/3 e primeiro
termo a1 = 1, como segue.
 2 n1
2  2 2  2 3  2 4
,
,
,... ,
,...
1, ,
3
3
3
3
3
Em particular, o comprimento de Cn (2/3)n1 .

189. Enchendo uma piscina Como as torneiras A e B despejam gua na piscina com vazo
constante, o volume de gua despejado na piscina de cada uma das torneiras proporcional
ao tempo em que ela fica aberta. Assim, se durante duas horas a torneira A enche 15% do
volume da piscina, ento em 4 horas ela encher 30% do volume da piscina.
Mas, quando as torneiras A e B ficam simultaneamente abertas durante quatro horas, elas
conseguem encher 50% do volume da piscina. Da, temos que a torneira B enche 50%30% =
20% do volume da piscina em quatro horas.
Para saber quanto tempo a torneira B deve ficar aberta para encher os 35% restantes do
volume da piscina, basta utilizar a regra de trs.
horas percentual
4

20%
x

35%

328

OBMEP 2010

Solues do Nvel 3
Logo, a torneira B gastar x =

35 4
= 7 horas para encher os 35% restantes.
20

190. Probabilidade de ser um nmero par


Soluo 1: Sejam a e b os nmeros escritos nas bolas retiradas por Jos e Maria, respectivamente. Existem, ento, nove possibilidades para a e oito possibilidades para b. Desse modo,
existem 9 8 = 72 possibilidades para o nmero a b. Para contar quantos desses nmeros a b
so pares, precisamos analisar separadamente dois casos, como segue.
Ambos nmeros a e b so pares.

O nmero a mpar e o nmero b par.


No primeiro caso, em que a e b so pares, existem quatro possibilidades para a e trs possibilidades para b. Desse modo, existem 4 3 = 12 possibilidades ao todo.

No segundo caso, em que a mpar e b par, existem cinco possibilidades para a e quatro
possibilidades para b. Desse modo, existem 5 4 = 20 possibilidades.
32
4
12 + 20
=
= .
Portanto, a probabilidade de o nmero a b ser par
72
72
9

Soluo 2: A paridade do nmero a ser formado depende da paridade do nmero escrito na


bola a ser retirada por Maria. Dentre os nmeros inteiros de 1 a 9, existem cinco mpares,
1, 3, 5, 7 e 9, e quatro pares, 2, 4, 6 e 8. Portanto, a probabilidade de que o nmero a ser
4
4
formado seja par
= .
5+4
9
191. Mltiplo de 7 N = (n + 6m)(2n + 5m)(3n + 4m) um mltiplo de 7.
Soluo 1: Inicialmente, observemos que, denotando k = n m, temos
N = (n + 6m)(2n + 5m)(3n + 4m)
= (n + 7m m)(2n + 7m 2m)(3n + 7m 3m)



= (n m + 7m) 2(n m) + 7m 3(n m) + 7m
= (k + 7m)(2k + 7m)(3k + 7m).

Como 7 primo e divide N, ento pelo menos um dos trs fatores k + 7m, 2k + 7m ou 3k + 7m
de N mltiplo de 7.
k + 7m
k
(i) Se k +7m mltiplo de 7, ento
= +m inteiro, logo k mltiplo de 7. Segue
7
7
que 2k e 3k tambm so mltiplos de 7 e, portanto, os trs fatores k + 7m, 2k + 7m e
3k + 7m de N so mltiplos de 7. Conclumos que N mltiplo de 73 .
2k + 7m
2k
(ii) Se 2k + 7m mltiplo de 7, ento
=
+ m inteiro, logo 2k mltiplo de 7.
7
7
Como 2 e 7 so primos entre si, segue que k mltiplo de 7, o que leva ao caso anterior
e N resulta ser mltiplo de 73 .
(iii) Se 3k + 7m mltiplo de 7, analogamente conclumos que k mltiplo de 7, o que leva
ao caso anterior e N mltiplo de 73 .
Assim, estabelecemos que N mltiplo de 73 .
Soluo 2: Consideremos os nmeros A = n + 6m, B = 2n + 5m e C = 3n + 4m. Como
o nmero primo 7 divide o produto N = A B C, ento 7 divide pelo menos um desses
fatores. Para concluir que 73 divide N, basta mostrar, portanto, que se 7 divide algum dos
nmeros A, B ou C ento 7 divide cada um deles.
OBMEP 2010

329

Solues do Nvel 3
Suponhamos que 7 divida A. Ento 7 divide 2A. Mas 2A = 2n + 12m = B + 7m. Como 7
tambm divide 7m, segue que 7 divide B. Da mesma forma, como 7 divide A, segue que 7
divide 3A. Mas 3A = 3n + 18m = C + 14m. Como 7 tambm divide 14m, conclumos que 7
divide C.
Suponhamos que 7 divida B. Ento 7 divide 4B. Mas 4B = 8n + 20m = A + 7(n + 2m). Como
7 tambm divide 7(n + 2m), segue que 7 divide A. Como j foi mostrado acima, dividindo A,
7 tambm divide C.
Suponhamos que 7 divida C. Ento 7 divide 5C. Mas 5C = 15n + 20m = A + 7(2n + 2m).
Como 7 tambm divide 7(2n + 2m), segue que 7 divide A. Como j foi mostrado acima,
dividindo A, 7 tambm divide B.
192. Os ngulos 15 e 75 Como DB a diagonal de um quadrado de
lado medindo 1 cm, o
2
1
2
Teorema de Pitgoras garante que DB = 1 + 1 = 2, ou seja, DB = 2 . Recordemos que

sen 60
=
3;
cos 60

sen 30
3
tg 30 =
=
.

cos 30
3

1
cos 60 = sen 30 = ;
2

sen 60 = cos 30 =
;
2

tg 60 =

(a) O tringulo BCE equiltero, logo seus ngulos internos medem 60 . A partir dessa
informao, obtemos os ngulos assinalados na figura.
1

30

M
60

F
60

30

1
CD
3

=
. Como sen 60 =
, segue que
=
No tringulo CDF temos
DF
DF
2

1
3
2
2 3
=
e, portanto, que DF = =
. Ainda no tringulo CDF temos
DF
2
3
3
CF
1
CF
1
CF
cos 60 =
. Mas cos 60 = , de onde se conclui que =
, ou seja,
=
DF
2
2
2
3/3
2
3/3

3
3
CF =
. Segue que BF = 1 CF = 1
. Temos, agora,
3
3

1
FN
FN
3 3

= sen 30 =
=
, de modo que F N =
2
BF
6
1 3/3
sen 60

3
BN
BN
,
= cos 30 =
=
2
BF
1 33

de modo que BN =

Assim, calculamos os trs lados do tringulo DBN, como segue.

DB = 2;

2 3 3 3
1+ 3
DN = DF + F N =
+
=
;
3
6
2

31
BN =
.
2

330

OBMEP 2010

31
.
2

Solues do Nvel 3
b
(b) No tringulo DBN temos D BN
= 45 + 30 = 75 , donde conclumos que

b = 15 . Assim, temos
B DN

31

BN
6 2
D

2
cos 75 =
=
=
DB
4
2
e
15

1+ 3

DN
6+ 2
cos 15 =
= 2
=
.
3 +1
2
DB
4
2
2
Resta observar que sen 75 = cos 15 , sen 15 = cos 75 e que

sen 75
6+ 2
sen 15
6 2

.
tg 75 =
=
=
e tg 15 =
cos 75
cos 15
6 2
6+ 2

75

3 -1
2

193. Crculos tangentes


(a) Na figura dada esto desenhadas dois crculos concntricos de
raios r e R e um crculo de raio x, simultaneamente tangente
aos dois crculos concntricos. Logo, r + 2x = R, donde
Rr
x=
.
2

R
r

(b) Na figura dada temos dois crculos tangentes de raio x que


tambm so tangentes aos dois crculos concntricos de raios
r e R. Os pontos A, B e C so os centros desses crculos. Para
traar doze crculos de raio x na regio entre os dois crculos

b = 360 = 30 .
concntricos, devemos ter ACB
12
Se T o ponto de tangncia dos crculos de raio x, ento T ponto
b = 15 . Nesse tringulo retngulo
mdio do segmento AB e ACT
temos
AT
x
C
sen 15 =
=
.
AC
r+x

R
r
C

B
x
x
A

T
x
15

r+x

Observe que
sen 15 = sen (45 30 ) = sen 45 cos 30 cos 45 sen 30

3 3
21
6 2
=

=
,
2 2
2 2
4
Rr
o que coincide com o valor obtido na questo precedente. Mas x =
, do que
2

Rr
6 2
conclumos que
=
. Dividindo por r o numerador e o denominador do
R+r
4
membro esquerdo dessa igualdade obtemos
R

1
6 2
r
.
=
R
4
+1
r
OBMEP 2010

331

Solues do Nvel 3
R



 R
Segue que 4
1 =
6 2
+ 1 e, finalmente,
r
r

4+ 6 2
R

.
=
r
4 6+ 2

Observao:
 p Uma outra maneira de obter o valor de sen 15 utilizar a frmula
sen /2 = (1 cos )/2 do ngulo metade. Para = 30 , obtemos
s
p
r

1 cos 30
1 3/2
2 3
30

=
=
=
.
sen 15 = sen
2
2
2
2

Repetindo o argumento apresentado acima com esse valor do seno, obtemos


p

R
2+ 2 3
p
=
.
r
2 2 3

bastante curioso e nada evidente, primeira vista, que essas duas expresses envolvendo
radicais sejam iguais:
p

4+ 6 2
2+ 2 3

=
p
.
4 6+ 2
2 2 3

194. Mudando a base Num tringulo issceles, a altura relativa base coincide com a mediana.
Traando essa altura no tringulo dado, de base 10, obtemos dois tringulos retngulos com
catetos medindo 5 e h e hipotenusa 13. Pelo
Teorema de Pitgoras, temos h2 + 52 = 132 ,

donde h2 = 132 52 = 144 e, portanto, h = 144 = 12. Logo, a rea do tringulo dado
10 12
bh
=
= 60 cm2 .
2
2
Agora colamos os dois tringulos retngulos ao longo do cateto medindo
5, obtendo um tringulo issceles com 12 + 12 = 24 cm de base, lados
de 13 cm e altura relativa base igual a 5 cm. Logo, esse novo tringulo
24 5
issceles tambm tem rea igual a
= 60 cm2 .
2
A=

195. Clube de Matemtica Sejam H e M os nmeros de homens e mulheres, respectivamente,


no clube. Temos duas possibilidades. Se eu sou um menino, temos M = H 1 e, quando
falta um menino, o nmero total de pessoas no clube
M + H 1 = H 1 + H 1 = 2H 2.

3
4 (2H

Logo, H 1 = M =
2), de modo que H = 1. Mas ento, M = 1 1 = 0, o que
no possvel. Assim, necessariamente, eu sou uma menina, e, portanto, M = H + 1 e temos
H + 1 = 34 (2H + 1 1), donde H = 2 e M = 3.
196. Uma calculadora diferente
Soluo 1: Para calcular (2 3) + (0 3) utilizamos as propriedades (i), (ii) e (iii), obtendo
(2 3) + (0 3)

(iii)

=
=

(iii)

(i) (ii)

=
=

332

(2 + 0) (3 + 3)
(6 + (4)) (6 + 0)
(6 6) + ((4) 0)
6 + (4) 2
6 8 = 2.

OBMEP 2010

Solues do Nvel 3
Para calcular 1 024 48, observe que 1 024 = 976 + 48. Assim,
1 024 48 = (976 + 48) (0 + 48)
= (976 0) + (48 48)
= 976 2 + 48

= 1 952 + 48 = 2 000.
Soluo 2: Pelas propriedades (i), (ii) e (iii),
a b = ((a b) + b) (0 + b)
= ((a b) 0) + (b b)
= (a b) 2 + b
= 2a 2b b

= 2a b,
para quaisquer inteiros a e b. Assim,

(2 3) + (0 3) = (2 2 3) + (2 0 3) = 1 3 = 2
e
1 024 48 = 2 1 024 48 = 2 048 48 = 2 000.
Observao: Existe uma nica operao sobre os inteiros com as propriedades (i), (ii) e
(iii) do enunciado, a saber, a b = 2a b, como mostramos na segunda soluo. No entanto,
mesmo restringindo o domnio de aos inteiros no negativos, possvel mostrar que uma
operao com as propriedades (i), (ii) e (iii) do enunciado existe, e nica, sendo dada por
a b = 2a b, s que, agora, precisamos nos restringir a nmeros inteiros no negativos a, b
tais que 2a b, para que o resultado da operao ainda seja um nmero inteiro no negativo.
Denotemos o conjunto dos inteiros no negativos por N .
claro que a deduo feita na segunda soluo se aplica somente se a b pois, nesse caso,
a b N . Para provar a existncia e unicidade da operao a b nos inteiros no negativos
tais que 2a b, precisamos de um argumento mais sutil, como segue.

Supondo que a operao a b esteja definida em N sempre que 2a b, dando um resultado em N e satisfazendo as propriedades (i), (ii) e (iii) do enunciado, afirmamos que
a b = 2a b vale sempre. De fato, dado c N , temos c (2c) = 0, j que podemos cancelar
2c de ambos lados da igualdade
2c = (2c) (2c) = (c + c) (2c + 0) = (c (2c)) + (c 0) = (c (2c)) + 2c.
Agora, dados a, b N , com 2a b e b > a, temos 2a b, b a N e


a b = (2a b) + (b a) (2a b) + (2b 2a)
= (2a b) (2a b) + (b a) (2(b a))

= (2a b) + 0 = 2a b.

Finalmente, para a, b N , com 2a b e a b, isso j foi mostrado na segunda soluo.


Assim, a b = 2a b vale para quaisquer a, b N tais que 2a b.
197. Cercando o globo terrestre Como o raio da Terra muito grande, e foi dado apenas
um acrscimo de 1 m ao comprimento do fio ao longo do Equador, parece que a folga entre o
fio e o Equador muito pequena. Mais ainda, se trocarmos a Terra por Jpiter ou por uma
OBMEP 2010

333

Solues do Nvel 3
bolinha de gude e realizarmos essa mesma experincia, parece que a altura da folga entre o
fio aumentado e o equador dessa esfera tambm muda, sendo que quanto maior a esfera
considerada, menor a folga entre o fio e o equador da esfera.
Mostremos que essa ideia intuitiva falsa e que a altura da folga, entre o fio e o Equador, sempre de aproximadamente 16 cm, independentemente do raio da esfera em que a experincia
for realizada.
Consideremos a circunferncia de comprimento 2 R de um crculo de raio R e tambm a
circunferncia de comprimento igual a 2 R + 1 de um outro crculo de mesmo centro, de
raio igual a R + h. Assim, h a altura da folga entre as duas circunferncias. Como a
circunferncia de um crculo de raio R + h tem comprimento igual a 2 (R + h), obtemos a
igualdade
h

2 R + 1 = 2 (R + h) = 2 R + 2 h
que, simplificada, fornece 1 = 2 h, ou seja,
h=

1
1

0,16.
2
6,28

Portanto, independentemente do valor de R, a altura da folga obtida com 1 m a mais de fio ,


sempre, de aproximadamente 16 cm. Em particular, somente uma formiga capaz de passar
por debaixo desse fio.
198. Comprimento de uma corda Sendo AB um dimetro, o tringulo ABC est inscrito numa semicircunferncia, implicando que esse tringulo retngulo no vrtice C. Pelo
Teorema de Pitgoras,
C

BC 2 = AB 2 AC 2 ,

12 cm

ou seja,
BC 2 = 202 122 = 256 = 162 .

20 cm

Assim, obtemos que BC = 16.


199. Dois irmos Sejam x e y

xy
z1

z + 20

as idades atuais dos dois irmos e z a idade do pai. Temos


= 3
= 2[(x 1) + (y 1)] = 2x + 2y 4
= (x + 20) + (y + 20) = x + y + 40

Uma maneira simples de encontrar z multiplicar a terceira equao por 2 e do resultado


subtrair a segunda equao, obtendo 2z + 40 (z 1) = 80 (4), o que implica z = 43.
Usando as duas primeiras equaes podemos calcular, agora, a idade dos dois filhos. Pela
primeira equao, x = y + 3 e, pela segunda, 43 1 = 2x + 2y 4, ou seja, x = 23 y.
Somando essas duas equaes obtidas, encontramos 2x = 26, donde x = 13 e, portanto,
y = 10.

200. Canelonis de ricota Colando os retngulos de massa ao longo do maior lado, Pedro
obtm um cilindro de base circular com 10 cm de comprimento e 16 cm de altura. O volume
que ele, ento, recheia com ricota o volume V = rea da base altura desse cilindro. A
rea da base dada por r 2 , onde r denota o raio da base. Vamos, ento, calcular o raio

334

OBMEP 2010

Solues do Nvel 3
sabendo que o permetro da base 10 cm. Temos 2 r = 10, ou seja, r = 5/. Assim, o
volume de ricota para cada caneloni dado, nesse caso, por
V =

400
16 25
52
16 =
=
cm3 .
2

Agora, colando os retngulos de massa ao longo do menor lado, Pedro obtm um cilindro de
base circular com 14 cm de permetro e 12 cm de altura.
O raio da base agora dado por r = 14/2 = 7/.
Assim, o volume de ricota para cada caneloni dado,
nesse caso, por
V =

72
588
12 =
cm3 .
2

Finalmente, para calcular o novo gasto com ricota, usamos uma regra de trs direta.
Volume (cm3 )
Ricota(g)
400

500

588

Segue que
500 588
= 735 g ,
400
de modo que agora Pedro gasta 235 g a mais de ricota por caneloni.
x=

201. Clculo de segmentos O tringulo ABP retngulo com catetos AB = 1 200 e BP =


150 + 350 = 500. Pelo Teorema de Pitgoras, temos
AP 2 = 1 2002 + 5002 = (144 + 25) 104 = 169 104 = (13 102 )2 ,
de modo que AP = 13 102 = 1 300 m. Analogamente, considerando o tringulo retngulo
P CD, temos
DP 2 = 3502 + 1 2002 = (72 + 122 22 )(52 102 ) = 252 502 ,
donde DP = 1 250 m. Os tringulos P CQ e P BA so retngulos com um ngulo em
comum, logo so semelhantes e segue que
PQ
PC
CQ
=
=
.
PA
PB
AB
Substituindo os valores conhecidos, obtemos
PQ
350
CQ
=
=
.
1 300
500
1 200
Assim,
PQ =

350 1 300
350 1 200
= 910 m e CQ =
= 840 m.
500
500

OBMEP 2010

335

Solues do Nvel 3
202. Pr chegar junto! Sabemos que a velocidade a razo da distncia percorrida pelo
tempo gasto. Como as velocidades de Luisa e Ada so constantes, a distncia percorrida por
cada uma proporcional ao tempo decorrido. Logo, se Ada percorre 3 000 120 = 2 880 m
no mesmo tempo em que Luisa percorre 3 000 m, ento Ada percorrer 3 000 m no mesmo
tempo em que Luisa percorrer d m, numa rega de trs direta.
Luisa Ada
3 000 2 880
d
3 000

3 0002
Assim, d =
= 3 125 e Luisa deve partir 125 m antes do ponto A para chegar junto com
2 880
Ada ao ponto B.
203. Um professor enfurecido Quem teve x como nota mensal vai ter um desconto de x%
sobre essa nota, ou seja vai perder
x% de x =

x2
x
x=
.
100
100

x2
Logo, uma nota inicial de x, depois do castigo, fica sendo x
. Consideremos essa funo
100
nota depois do castigo, dada por
x2
.
100
Como as notas mximas e mnimas so 0 e 100, podemos considerar essa funo apenas no
domnio [0, 100], ou seja, para 0 x 100. O grfico de f uma parbola com concavidade
para baixo. O valor mnimo dessa funo 0, que ocorre em x = 0 e x = 100 e o valor
mximo ocorre no vrtice, ou seja, no ponto x = 50, que a mdia aritmtica entre as duas
razes 0 e 100 de f.
f (x) = x

(a) A maior nota depois do castigo para os alunos que, antes do castigo, tiraram 50. Essa
nota
502
f (50) = 50
= 25 .
100
(b) A menor nota 0 e ocorre para os alunos que tiraram 0 ou, pasmem, 100 antes do
castigo. De fato, f (0) = f (100) = 0.
(c) Para cada nota maior do que 50 h uma outra nota, menor do que 50, que acaba sendo
igual depois do castigo. De fato, pela simetria da parbola, f (50 n) = f (50 + n), para
cada 0 n 50. Por exemplo, quem tirou 30 acaba com a mesma nota 21 de quem
tirou 70, pois f (30) = 21 = f (70). Assim, procede a reclamao dos alunos que tiraram
notas boas.
204. O percurso de um atleta O Polo Norte da Terra o ponto mais fcil de ser identificado
como soluo: saindo o atleta do Polo Norte, correndo 5 km para o Sul, depois 5 km para o
Leste e finalmente 5 km para o Norte, ele volta novamente para o Polo Norte.
Polo
Norte

A
B

C2
C1
Paralelo com 5 km
Polo
Sul

336

OBMEP 2010

de comprimento

Solues do Nvel 3
y
25

100 x

50

Vamos determinar outros ponto da Terra que satisfazem as hipteses do problema. Consideremos o paralelo (linha paralela ao Equador) de comprimento 5 km. Existem dois deles, um
prximo ao Polo Norte e outro prximo ao Polo Sul. Vamos denotar por C1 o que est mais
prximo do Polo Sul e por C2 o paralelo que est 5 km ao Norte de C1 , distncia essa medida
ao longo de um meridiano. Afirmamos que qualquer ponto A sobre o paralelo C2 satisfaz
as hipteses do problema. De fato, saindo de A e caminhando 5 km para o Sul, chega-se a
um ponto B do paralelo C1 . Como C1 mede 5 km, saindo de B e caminhando 5 km para o
Leste retorna-se novamente a B. Finalmente, saindo de B e caminhando 5 km para o norte,
retorna-se novamente ao ponto de partida A.
205. reas iguais Seja T a rea do tringulo ABC e denotemos por a e c as reas internas
aos semicrculos de dimetros AB e BC mas externas ao semicrculo de dimetro AC e por
b e d as reas compreendidas entre os catetos do tringulo e o semicrculo de dimetro AC.
Segue que a rea do semicrculo de dimetro AB dada por a + b, portanto,
1  AB 2
a+b=
= AB 2 ,
2
2
8
A
a rea do semicrculo de dimetro BC dada por c+d, portanto,
1  BC 2
c+d =
= BC 2
2
2
8

a b

e a rea do semicrculo de dimetro AC dada por


b + d + T, portanto,

C
d
c

1  AC 2
b+d+T =
= AC 2 .
2
2
8

AC 2 T. Alm disso, o Teorema de Pitgoras aplicado ao tringulo


8
retngulo ABC fornece AC 2 = AB 2 + BC 2 ou, ento, AB 2 + BC 2 AC 2 = 0. Assim,

e, em particular, b+d =

a + c = (a + b) + (c + d) (b + d) =

AB 2 + BC 2 AC 2 + T = T,
8

ou seja, a soma a + b das reas sombreadas igual rea T do tringulo retngulo ABC.
206. Funo definida por rea
(a) A reta r passa pelo ponto (0, 2), portanto, tem equao dada por y = mx + 2. Como essa
reta tambm passa pelo ponto (2, 0), temos 0 = 2m + 2, o que implica m = 1. Assim,
a equao de r y = x + 2. A reta s passa pelo ponto (0, 6), portanto, tem equao
dada por y = mx + 6 e, como tambm passa pelo ponto (3, 0), temos 0 = 3m + 6, o que
implica m = 2. Assim, a equao de s y = 2x + 6.
OBMEP 2010

337

Solues do Nvel 3
(b) Denotemos por A o ponto de encontro das retas r e s, B = (0, 2), O = (0, 0),
D = (3, 0) e por C o ponto de corte da reta s com a reta horizontal por B, como na
figura dada. Por definio, f (0) a soma das reas do tringulo ABC e do trapzio
BODC.
y
s

A
B

y
O

Para determinar as coordenadas de A, igualamos x + 2 = 2x + 6, obtendo


x = 4/3. Substituindo esse valor na equao de r ou s resulta y = 10/3, ou seja,
A = (4/3, 10/3). O ponto C pertence reta s e reta y = 2, portanto, 2x + 6 = 2, ou
seja, x = 2, e obtemos C = (2, 2). A altura do tringulo ABC em relao base BC
1 4
4
h = 10/3 2 = 4/3, portanto, a rea do tringulo ABC igual a 2 = .
2 3
3
3+2
J a rea do trapzio BODC 2
= 5, de modo que
2
f (0) =

19
4
+5=
.
3
3

(c) Observe que f (y) igual a f (0) menos a rea do trapzio de altura y e bases 3 e x,
sendo x a abscissa do ponto da reta s que tem ordenada y, ou seja, satisfaz y = 2x + 6.
Assim, x = (6 y)/2 = 3 21 y e a rea desse trapzio dada por
3 + 3 12 y
3+x
1
y=
y = 3y y 2
2
2
4
e obtemos, para 0 y < 2,
f (y) =

19
1
1
19
3y + y 2 = y 2 3y +
.
3
4
4
3

y2
19
(c) O grfico de f (y) =
3y +
uma parbola
4
3
cncava para cima. As coordenadas do vrtice V dessa
3
parbola so x =
=6e
2/4
19
19
8
62
y = f (6) =
36+
= 9 +
= ,
4
3
3
3

x
19
3

4
3
0

6
2

10

8
3


19
4
= , o grfico de f, com 0 y < 2,
ou seja, V = 6, 8/3 . Como f (2) = 1 6 +
3
3
o segmento de parbola em linha grossa na figura dada.

338

OBMEP 2010

Solues do Nvel 3
207. PA e PG Os quatro termos de uma progresso aritmtica de razo r podem ser escritos
como
x, x + r, x + 2r, x + 3r.
Assim, os trs nmeros em progresso geomtrica so x, x + 2r, x + 3r. Ento, pela prpria
definio de progresso geomtrica, x + 2r a mdia geomtrica de x e x + 3r, ou seja,

2
x x + 3r = x + 2r .

Segue da que x2 + 3xr = x2 + 4xr + 4r 2 e, portanto, xr = 4r 2 . O caso r = 0 no


interessante, pois daria origem a progresses constantes. Supondo r 6= 0, obtemos x = 4r.

Atribuindo valores no-nulos a x, obtemos solues do problema. Por exemplo, para x = 4,


obtemos (r = 1 e) a progresso aritmtica 4, 3, 2, 1 tal que os nmeros 4, 2, 1 formam uma
progresso geomtrica. Note que esse problema tem uma infinidade de solues, uma para
cada valor escolhido de x 6= 0.
208. Plano cartesiano Comecemos examinando alguns casos.
f (1) o nmero de pontos inteiros sobre o segmento que liga (0, 0) ao ponto (1, 4),
portanto, f (1) = 0.
f (2) o nmero de pontos inteiros sobre o segmento que liga (0, 0) ao ponto (2, 3),
portanto, f (2) = 0.
f (3) o nmero de pontos inteiros sobre o segmento que liga (0, 0) ao ponto (3, 6). Como
nesse segmento esto os dois pontos inteiros (1, 2) e (2, 4), segue que f (3) = 2.

.
..
..
..
...
.
..
..
..
...
..
..
..
..
...
..
..
..

..
...
...
....
..
...
...
..
.
...
...
...
..
.
...
...
..
..
.
.
..
...

4
2

.
...
...
...
..
.
...
...
...
..
.
...
...
...
..
.
...
...
...
..
.
...
...
..
..
.
.
..

Vejamos, agora, o caso geral. Note que se um ponto inteiro (x, y) est sobre o segmento que
une (0, 0) a (n, n + 3), sem ser uma das extremidades, ento 0 < x < n e 0 < y < n + 3.
Fixado n, temos dois casos: ou 3 divide n ou 3 no divide n.
1o Caso: 3 divide n. Mostremos que f (n) = 2. De fato, vamos supor que n = 3k, com k
inteiro. Queremos encontrar todos os pontos inteiros do segmento que une a origem (0, 0) ao
ponto (3k, 3k + 3). Seja (x, y) um desses pontos. Ento
x
3k
k
=
=
.
y
3k + 3
k+1
Como a ltima frao acima irredutvel, deduzimos que x um mltiplo de k e, como
0 < x < n = 3k, necessariamente x = k ou x = 2k. Os nicos pontos inteiros so, portanto,
(k, k + 1) e (2k, 2k + 2). Assim, f (n) = 2.
2o Caso: 3 no divide n. Mostremos que f (n) = 0. Vamos precisar do seguinte resultado.
Lema. Se 3 no divide n, ento n e n + 3 so primos entre si.
Demonstrao. Suponhamos que o MDC entre n e n + 3 seja d > 1. Ento d divide n e
n + 3, portanto d divide (n + 3) n = 3. Logo, como d > 1, temos d = 3, o que no possvel,
OBMEP 2010

339

Solues do Nvel 3
porque partimos da hiptese de que 3 no divide n. Assim, o MDC de n e n + 3 1, provando
o lema.
Mostremos, agora, que os nicos pontos inteiros sobre o segmento que une (0, 0) a (n, n + 3)
so as extremidades. De fato, suponhamos que esse segmento contenha algum ponto inteiro
(x, y) diferente das extremidades. Ento 0 < x < n e
n
x
=
.
y
n+3
n
irredutvel, portanto, x mltiplo de n, o que no pode
n+3
acontecer porque 0 < x < n. Assim, conclumos que f (n) = 0.

Mas, pelo lema, a frao

209. Trabalhando com um quadriltero Lembre que


qualquer lado de um tringulo maior do que a diferena e menor do que a soma dos outros dois lados. No
tringulo ADB temos AD AB < BD < AD + AB e no
tringulo CBD temos BC CD < BD < BC + CD.

...............
........................... .......
....
....
..
.
..
.
....
.
.
.
.
.
.
..
....
....
.......
....
...
.......
.
.
.
.
.
.
.
.
.
....
.....
.
.
....
.
.
.
.
....
.
.
.....
..
.
.
.
....
.
.
.
.
.
.
.. ...........
....
.
.
.
....
... ............
.
.
.
....
...............
.
..
.
........................................................................................................................................................................................................

A .................................................................................

Substituindo os valores conhecidos, obtemos 9 5 < BD < 5 + 9 e 17 5 < BD < 17 + 5,


ou seja,
4 < BD < 14 e 12 < BD < 22.
Dessas duas desigualdades conclumos que 12 < BD < 14 e, como BD inteiro, s podemos
ter BD = 13.
210. O tringulo de Reuleaux O tringulo de Reuleaux formado por 4 regies, um tringulo
equiltero e trs calotas. Cada calota um sexto de um crculo de raio 1, do qual foi retirado
um tringulo equiltero de lado 1. Pelo Teorema de Pitgoras, a altura desse tringulo
equiltero mede
r

1 2
3
1
=
,
2
2
portanto, a rea desse tringulo equiltero dada por

1 23
3
=
.
2
4
A rea de um setor circular que um sexto do crculo de raio 1 dada por
rea de cada calota dada pela diferena

.
6
4

1
6

, portanto, a

Assim, a rea do tringulo de Reuleaux mede




3
3

3
3

+
=
cm2 .
6
4
4
2
2
211. Interseo de crculos Seja G o baricentro do tringulo ABC, ou seja, o ponto de
encontro de suas medianas. Como a figura invariante por rotaes de 60 ao redor do ponto
G, vemos que o tringulo XY Z equiltero e que G tambm o seu baricentro.

Calculemos o comprimento L de seu lado em termos do lado a do tringulo ABC e do


raio r dos crculos. Seja CM a altura do tringulo ABC em relao base AB. Como a

340

OBMEP 2010

Solues do Nvel 3

altura de um tringulo equiltero de lado a mede 21 a 3 e o baricentro divide a altura em


dois segmentos, um com o dobro do comprimento do outro, obtemos

CM = 21 a 3 , GM = 13 CM = 61 a 3 e CG = 23 CM = 13 a 3 .
C
CG = 2 GM
G
M

Como AZ = BZ = r, vemos que o ponto Z est na reta mediatriz do segmento AB. Entretanto, essa mediatriz a reta suporte da altura CM do tringulo ABC. Desse modo,
vemos que os pontos C, G, M e Z esto alinhados e que o tringulo M ZB retngulo,
com hipotenusa BZ = r e cateto M B = 21 a.
Pelo Teorema de Pitgoras,
p
M Z = BZ 2 M B 2 =

C
1
2

GZ = GM + M Z =

1
6

3+

p
1
2

4r 2 a2

N
G

4r 2 a2 .

Z
Como GZ o baricentro do tringulo XY Z, resulta que a altura N Z desse tringulo satisfaz
N Z = 32 GZ. Por outro lado, a altura N Z desse tringulo equiltero de lado L dada por

1
1
3
2 L 3. Assim, 2 GZ = N Z = 2 L 3 e, portanto,
p

L = 3 GZ = 21 a + 12 12r 2 3a2 .

212. Valor mximo Estamos procurando o valor de k para o qual mximo o termo da
sequncia
12
22
32
k2
(k + 1)2
,
,
,
.
.
.
,
,
,...
1,001 1,0012 1,0013
1,001k 1,001k+1
Queremos comparar os tamanhos de dois termos quaisquer dessa sequncia. O mais simples
comparar um termo qualquer com o seguinte. Ocorre que mais fcil comparar duas fraes
com denominadores positivos iguais. Por exemplo, o terceiro termo maior do que o segundo
porque
22
22 1,001
32
=
<
,
1,0012
1,0013
1,0013
sendo evidente que 22 1,001 = 4,004 < 9 = 32 . Mais geralmente, como 1,001n > 0 para
qualquer n, para verificar se
k2
k 2 1,001
(k + 1)2
=
<
,
1,001k
1,001k+1
1,001k+1
basta verificar se k 2 1,001 < (k + 1)2 . Multiplicando tudo por 1 000, isso equivale a
k(k 2 000) < 1 000. Ora, para 1 k 2 000 temos k(k 2 000) 0 < 1 000 e, para
k 2 001, vale k(k 2 000) > 2 001 > 1 000. Em particular,
2 0002
2 0012
<
1,0012 000
1,0012 001

2 0012
2 0022
>
.
1,0012 001
1,0012 002

Logo, a sequncia dada cresce estritamente com 1 k 2 001 e da decresce estritamente


com k 2 001. Assim, o maior termo dessa sequncia atingido com k = 2 001.
OBMEP 2010

341

Solues do Nvel 3
213. Moedas falsas
(a) Aladim deve retirar de cada saco um nmero diferente de moedas, como segue. Primeiro
retira uma moeda do primeiro saco, depois duas do segundo, da trs do terceiro, e
assim, sucessivamente, at o ltimo saco, do qual retira as dez moedas. Ao todo, foram
retiradas 1 + 2 + 3 + 4 + 5 + 6 + 7 + 8 + 9 + 10 = 55 moedas, que so colocadas na
balana. Se todas essas moedas fossem verdadeiras, pesariam um total de 55 10 = 550
g. Mas, como algumas so falsas, o peso ser menor. Se faltar um grama porque h
somente uma moeda falsa e, portanto, o primeiro saco o procurado. Se faltarem dois
gramas porque h duas moedas falsas e, portanto, o segundo saco o procurado, e
assim sucessivamente.
(b) Vejamos que, em geral, uma tentativa de soluo como a anterior no permite a identificao dos sacos com moedas falsas. Suponhamos que Aladim tenha retirado uma moeda
do primeiro saco, duas moedas do segundo, e assim sucessivamente, at o ltimo saco,
de onde ele retirou dez moedas. Se existissem dois ou mais sacos com moedas falsas, o
procedimento de pesar essas 55 moedas pode ser inconclusivo. Por exemplo, digamos
que na pesagem das 55 moedas faltassem 7 g, ou seja, foram pesadas 7 moedas falsas.
Nesse caso, poderiam existir moedas falsas nos sacos 1 e 6, ou moedas falsas nos sacos
2 e 5, ou moedas falsas nos sacos 1, 2 e 4, etc. Ou seja, procedendo dessa maneira no
possvel identificar quais so os sacos de moedas falsas.
Para resolver esse problema, ele pode proceder de uma outra maneira, como segue.
Primeiro ele retira uma moeda do primeiro saco, depois duas moedas do segundo, da
quatro do terceiro, oito do quarto, dezesseis do quinto saco e assim, sucessivamente, at
o ltimo saco, sempre dobrando, a cada saco, o nmero de moedas retiradas do saco.
Dessa forma so retiradas, ao todo,
1 + 2 + 4 + 8 + 16 + 32 + 64 + 128 + 256 + 512 = 1 023
moedas que, juntas, pesariam 10 230 g, se todas as moedas retiradas fossem verdadeiras.
A diferena entre o peso real obtido na pesagem dessas 1.023 moedas e seu peso ideal de
10 230 g, indica a quantidade de moedas falsas pesadas e em quais dos sacos elas esto.
Vejamos isso atravs de um exemplo. Imaginemos que na pesagem tenham sido obtidos
10 125 g, ou seja, faltaram 10 230 10 125 = 105 g, que correspondem ao nmero
de moedas falsas. Subtraindo, sucessivamente, os nmeros correspondentes s moedas
retiradas de cada saco, comeando sempre do maior nmero menor do que 105, temos
105 64 = 41, 41 32 = 9, 9 8 = 1, ou seja, 105 = 1 + 8 + 32 + 64. Desse resultado,
Aladim pode concluir que foram retiradas 1, 8, 32 e 64 moedas falsas do primeiro, quarto,
sexto e stimo sacos.
Vamos, agora, justificar, de um modo mais formal, o raciocnio desenvolvido no exemplo
numrico. Seja p o peso obtido com a pesagem das 1 023 moedas retiradas. A diferena
10 230p o nmero de moedas falsas retiradas dos sacos. Efetuando divises sucessivas
por 2, pode-se provar que qualquer nmero inteiro positivo se escreve, de maneira
nica, como uma soma de potncias distintas de 2. De fato, isso que fornece a
justificativa terica para a expanso binria, ou seja, em base 2, dos nmeros naturais.
No nosso caso, isso implica que
10 230 p = 1 a0 + 2 a1 + 22 a2 + 23 a3 + + 29 a9 ,
em que cada um dos nmeros a0 , a1 , a2 , a3 , . . . , a9 0 ou 1.
De cada saco foram retiradas quantidades de moedas que so potncias de 2 e cada saco
ou contm somente moedas falsas ou contm somente moedas verdadeiras, isto , em
um mesmo saco no existem os dois tipos de moedas. Da, temos que se algum desses

342

OBMEP 2010

Solues do Nvel 3
nmeros, digamos aj , for 1, ento foram retiradas 2j moedas falsas do saco j + 1; por
outro lado, se o nmero aj for 0, ento foram retiradas 2j moedas verdadeiras do saco
j + 1.
214. Menor inteiro Como q = 2 005 p, queremos
5
p
7
<
< ,
8
2 005 p
8
de onde segue que 5(2 005 p) < 8p e 8p < 7(2 005 p). Logo,
5 2 005
7 2 005
<p<
,
13
15
de modo que 771,15 < p < 935,66. Assim, 772 o menor valor inteiro de p tal que p+q = 2 005
e 5/8 < p/q < 7/8.
215. Mais reas... Observe que a altura h relativa ao lado AB de todos os tringulos ABC
que tm o vrtice C na reta x + y = 7 sempre a mesma, pois a reta x + y = 7 paralela
reta x + y = 3 que passa por A e B. Logo, todos esses tringulos tm a mesma rea, a saber,
1
2

AB h).

Resta, portanto, determinar AB e h. Como AB a hipotenusa de um tringulo retngulo


que tem os dois catetos iguais a 3, segue do Teorema de Pitgoras que
p

AB = 33 + 32 = 18 = 3 2.

Por outro lado, h a distncia entre as retas paralelas x + y = 3 determinada pelos pontos
A e B e x + y = 7. A reta x = y perpendicular a essas retas paralelas e forma um ngulo
de 45 com o eixo Ox. Sejam M o p da perpendicular reta x + y = 7 traada a partir
de A e D = (7, 0) o ponto de corte da reta x + y = 7 com o eixo Ox. O tringulo AM D
assim formado retngulo issceles, com dois catetos iguais a h e hipotenusa
7 3 = 4. Pelo

Teorema de Pitgoras segue que 42 = h2 + h2 = 2h2 , ou seja, h = 8 = 2 2. Assim, a rea


procurada dada por


1
2 3 2 2 2 = 6.

216. Crculos tangentes Ligando os centros dos trs crculos obtemos o tringulo ABC de
lados AB = 3, AC = 4 e BC = 5. Como 32 + 42 = 52 , esse tringulo retngulo, com
hipotenusa BC.

B
2
1
A 1

B
2

2
3
3

1
A 1

D
2
3
3

Agora construmos o retngulo ADCB com uma cpia congruente ao tringulo ABC e
hipotenusa comum BC, conforme figura. Como DC = AB = 3 e o crculo de centro C
tambm tem raio 3, vemos que o ponto D est no crculo. Finalmente, ligamos o ponto D a
cada um dos vrtices do tringulo ABC e prolongamos esses segmentos at que cortem os
crculos centrados em A, B e C, obtendo os pontos P1 , P2 e P3 , conforme figura.
OBMEP 2010

343

Solues do Nvel 3

P2

P2

P1

P1

P3

P3

Observe que
DP2 = DB + BP2 = CA + BP2 = 4 + 2 = 6,.

DP1 = DA + AP1 = 5 + 1 = 6 e
DP3 = DC + CP3 = 3 + 3 = 6.

Assim, DP1 = DP2 = DP3 = 6 e, portanto, o crculo de centro D e raio 6 passa por P1 , P2
e P3 . Alm disso, como os pontos {D, A, P1 }, {D, B, P2 } e {D, C, P3 } esto alinhados, segue
que o crculo de centro D e raio 6 tangente exteriormente aos crculos dados de centros A, B
e C.
217. Soma
finita
que ospossveis
com 1 k 2 004 inteiro, so

Observe
produtos x2k1
x2k ,
( 2 1)( 2 1) = 3 2 2, ( 2 + 1)(
+ 1) = 1. Suponha
2 + 1) = 3 + 2 2 e ( 2 1)( 2
que a produtos sejam iguais a 3 2 2, b produtos sejam iguais a 3 + 2 2 e 1 002 a b
produtos sejam iguais a 1. A soma dada igual a

a(3 2 2) + b(3 + 2 2) + 1 002 a b = 1 002 + 2a + 2b + 2(b a) 2.


Assim, para que a soma seja inteira, devemos ter a = b. Logo, a soma igual a 1 002 + 4a.
Como a varia de 0 a 501, j que a + b = 2a no pode ser maior do que 1 002, a soma dada s
pode valer 502 valores inteiros distintos.
218. Mltiplos
Soluo 1: Observe que se um inteiro positivo m um mltiplo de um inteiro p, ento
2m p um mltiplo de p. De fato, se m = np, ento 2m p = 2np p = (2n 1)p. Em
particular, tomando m igual a a, a + 1, a + 2 e a + 3, estabelecemos que 2a 5 mltiplo de
5, 2(a + 1) 7 mltiplo de 7, 2(a + 2) 9 mltiplo de 9 e 2(a + 3) 11 mltiplo de 11.
No entanto, sucede que 2a 5 = 2(a + 1) 7 = 2(a + 2) 9 = 2(a + 3) 11, de modo que
2a 5 mltiplo de 5, 7, 9 e 11, donde mltiplo de 5 7 9 11 = 3 465. Assim, o menor
valor possvel de a satisfaz 2a 5 = 3 465, ou seja, a = 1 735.
Soluo 2: Como a mltiplo de 5, temos a = 5i, para algum i inteiro positivo. Mas
a + 1 = 1 + 5i deve ser mltiplo de 7. Por tentativas, verificamos que o menor valor de i para
o qual isso acontece i0 = 4. Para qualquer outro valor de i que torne 1 + 5i mltiplo de 7,
devemos ter (1+5i)(1+5i0 ) = 5(ii0 ) mltiplo de 7 e, portanto, ii0 mltiplo de 7. Logo,
i = i0 + 7j = 4 + 7j e a = 5i = 5(4 + 7j) = 20 + 35j, com j 0 inteiro. De maneira anloga,
a + 2 = 22 + 35j deve ser mltiplo de 9. O menor valor de j para o qual isso acontece j0 = 4,
pois 22 + 35 4 = 162 mltiplo de 9. Para qualquer outro valor de j que torne 22 + 35j
mltiplo de 9, devemos ter (22 + 35j) (22 + 35j0 ) = 35(j j0 ) mltiplo de 9 e, portanto,

344

OBMEP 2010

Solues do Nvel 3
j j0 mltiplo de 9. Logo, j = j0 + 9k = 4+ 9k e a = 20+ 35j = 20+ 35(4+ 9k) = 160+ 315k,
com k 0 inteiro. Finalmente, 11 deve dividir
a + 3 = 163 + 315k = 14 11 + 9 + (28 11 + 7)k
= (14 + 28k) 11 + (9 + 7k).

Temos, portanto, que 11 divide 9 + 7k. Novamente, por tentativas, simples verificar que o
menor menor valor de k para o qual isso acontece k0 = 5. Qualquer outro valor de k para o
qual isso acontea da forma k = 5 + 11l, com l 0 inteiro.

Conclumos que os inteiros positivos a para os quais 5 divide a, 7 divide a + 1, 9 divide a + 2


e 11 divide a + 3 so exatamente os inteiros da forma a = 160 + 315(5 + 11l) = 1 735 + 3 465l,
com l 0 inteiro. Em particular, o menor valor de a 1.735, obtido com l = 0.

219. Equao de duas variveis Observemos que


9xy x2 8y 2 = xy x2 + 8xy 8y 2 = x(y x) + 8y(x y)
= (x y)(8y x)

e que a fatorao de 2005 5 401. Como x e y so inteiros, temos somente quatro opes, a
saber, xy = 5 e 8y x = 401 ou xy = 401 e 8y x = 5. Se xy = 5 e 8y x = 401,
podemos somar essas equaes para obter 7y = 406, ou y = 58 e, portanto, x = 63. Da mesma
forma, se x y = 5 e 8y x = 401, obtemos 7y = 406 e, portanto, y = 58 e x = 63.
Analogamente, se x y = 401 e 8y x = 5, obtemos 7y = 406 e, portanto, y = 58 e
x = 459. As nicas solues so, portanto (63, 58), (63, 58), (459, 58) e (459, 58).
b = B DC
b = , como na figura dada. Ento temos
220. Trapzio retngulo Denotemos ABD
BD
CD =
e AD = BD sen , donde
cos
BD
1
CD
= cos =
AD
BD sen
sen cos
=

2
2.
sen 2

Assim, o menor valor da razo CD/AD 2, que ocorre quando sen 2 = 1, ou seja, quando
= 45 .
221. Jogos de futebol Para cada grupo de cinco alunos, existe um nico time formado que os
contm. Logo, contamos
12 11 10 9 8
5
C12
=
= 792
5!
times para cada cinco alunos escolhidos. Por outro lado, em cada time de seis jogadores,
temos C65 = 6 modos de escolhermos cinco jogadores, ou seja, existem seis grupos de cinco
jogadores que deram origem ao mesmo time na nossa primeira contagem. Assim, o total de
times formados 792/6 = 132.
222. A soma dos algarismos de um nmero
OBMEP 2010

345

Solues do Nvel 3
(a) imediato que se a um algarismos entre 1 e 9, ento s(a 10k ) = a, j que o nmero
a 10k formado pelo algarismo a seguido de k zeros. Da, temos
1} .
a 10k s(a 10k ) = a 10k a = a(10k 1) = a 9| {z
9} = a 9 1| {z
k noves

k uns

Como todo nmero pode ser decomposto em unidades, dezenas, centenas, etc, isto ,
todo nmero inteiro positivo n pode ser escrito, de maneira nica, na forma
n = a0 + a1 10 + a2 102 + + ak 10k ,
temos que
n s(n) = a1 9 + a2 99 + + ak 9| {z
9} .
k noves

Assim, a diferena n s(n) sempre divisvel por 9.

(b) Seguindo o mesmo raciocnio, temos que ambos s(n) s(s(n)) e s(s(n)) s(s(s(n))) so
divisveis por 9, portanto, n s(s(s(n))) divisvel por 9.
Em particular
2
009
2
009
2
009
2
s(s(s(2
))) divisvel por 9 ou, equivalentemente, 2
e s(s(s(22 009 ))) deixam o mesmo resto quando so divididos por 9.
Como 26 1 = 63 divisvel por 9, ento substituindo x = 26 e m = 334 na identidade

xm 1 = (x 1) xm1 + xm2 + + x2 + x + 1 ,
conclumos que (26 )334 1 = 22 004 1 divisvel por 9 e, portanto, 22 009 25 divisvel
por 9. Como 25 = 32 deixa resto 5 quando dividido por 9, temos que 22 009 deixa resto
5 quando dividido por 9. Por outro lado,
22 009 < (29 )224 < (103 )224 = 10672 .
Assim, 22 009 tem menos do que 672 algarismos e, portanto,
s(22 009 ) < 9 672 = 6.048,

s(s(22 009 )) 5 + 9 + 9 + 9 = 32e

s(s(s(22 009 ))) 2 + 9 = 13.

Como o nico nmero menor do que ou igual a 13 que deixa resto 5 ao ser dividido por
9 5, resulta que
s(s(s(22 009 ))) = 5.

346

OBMEP 2010

Solues dos Desafios

Solues dos Desafios


6 817

+221

51

13 259

z}|{ z}|{
z}|{
z}|{
1. Cadeia do menor nmero (N2/N3) 265 863 39 260 13 260 1.
2. Qual a metade? (N2/N3)

3. Cada um em seu estado (N1/N2/N3) Bruno ou Amlia (o desafio tem duas


solues).
4. Diviso (N1/N2) Resposta: 153.
5. Extra-terrestre (N1/N2) Seise.
6. Que famlia! (N1/N2) 3 meninas e 4 meninos.
7. Siga a pista (N1)

C
8. Cara ou Coroa (N2) Resposta: 12.
9. Contas do papagaio (N1) Resposta: 19.
10. As frias de Toms (N1/N2) Resposta: 16 dias.
11. Maratona de Matemtica (N3) Resposta: 25 e 63, respectivamente.
12. Fraes ignoradas (N1)

OBMEP 2010

347

Solues dos Desafios


13. Caminho de maior total (N2) ???
14. Produtos em linha (N1/N2/N3) casa B.
15. Cdigo Postal (N2/N3) Resposta: 47 679 e 47 779.
16. Anis olmpicos (N1/N2/N3)

17. Partidas de Denise (N2/N3) A primeira, a segunda, a terceira, a quarta e a


oitava.
18. Sete quadrados (N1/N2)

19. Ilha misteriosa (N1/N2/N3) 16 cinzas, 18 marrons e 11 vermelhos.


20. Universo hostil (N3) Resposta: 1 873.
21. O jogo das fichas
1
4
7

13
6
5

2
5
8

0
8
16

3
6
9

11
10
3

22. Um sistema Resposta: 23.


23. Constelaes floridas Pelo menos duas solues:

D = 25 169 = 4 225; R = 144 169 = 24 336


D = 49 289 = 14 161; R = 100 144 = 14 400.

348

OBMEP 2010

Solues dos Desafios


24. Dois meses iguais Setembro de 2006.
25. A faixa e o quadrado
..........................................................................................................................................................................................................................................................................
... .......
...
...
... .......
.......
.......
..
..
..
..
........
........
........
........
...
...
...
...
.......
.
.
.
.
.
.
...
.
...
.
.
.......
.......
.
..
........
........
...
...
.
...
.
........
........
.
.
...
...
.
.
........
........
.
..
........
...
...
.
...
........
........ ...
........ ..
...
..
........................................................................................................................................................................................................................................................................

26. Um nmero e seu sxtuplo Resposta: 746 a nica soluo.


27. Oito dentro de um retngulo Pelo menos duas solues:

....
...
..
...
....
..

....
...
.

...
......
....

...

....
...
......
....
...
....
..
......
....
....
...
.
.
.
.
.
.
.
.
.
..
.
.
.
....

....
..
...
.
.
.
.
.
.
.
.
.
.
.

.....................................................

........................................................

.......................................................

.....
.

..... ..
.....
.....
... .........
.
.....
.....
.....
.....
...
...
.....
.....
.....
.....
...
.
.....
..
.....
........................................................

.......................................................

....
...
.

.....
.

..... ..
.....
.....
... .........
.
.....
.....
.....
.....
...
...
.....
.....
.....
.....
...
.....
..
.
.....

....
...
..
...
....
..

...
......
....

...
....
...
......
....
...
....
..
......
....
....
...
.
.
.
.
.
.
.
.
.
..
.
.
.
.

....
....
..
...
.
.
.
.
.
.
.
.
.
.

.......................................................

........................................................

.....................................................

........................................................

.......................................................

28. Uma estratgia com um nmero muito grande Resposta: 99 999 585 960.
29. Um nmero surpreendente Resposta: 381 654 729.
30. Qual o erro? Cludia e Bruno.
31. Soma Trs solues:
231468
231468
+
5972

32. Bolinhas

264538
264538
+
9102

273548
273548
+
1602

.................................................................................................

.................................................................................................

.................................................................................................

468908

538178

548698

.
.........
.... .. ....
.... .... .......
....
...
...
.
.
.
....
..
....
....
....
....
....
...
...
.
.
.
.
............................................................................................
.
.... ..
....
.... .......
... ....
.
.
.
...
... .....
....
....
....
...
...
....
....
...
...
.... ... ......
.... .. ...
..
...
..................................................................................................

t
t

t
t

t
t

33. Um nmero que no divisvel por 5 Resposta: 2 004.


34. Quatro fraes e um inteiro Resposta: 1.
35. O Rei Arthur e o Drago das Trs Cabeas e Trs Caudas Resposta: 5.
OBMEP 2010

349

Solues dos Desafios


36. O passeio do cavalo
A
N
I
T
C

X
S
B
O
J

M
H
W
D
U

R
Y
F
K
P

G
L
Q
V
E

37. As faces do cubo Resposta: 24.


38. Data fatdica Resposta: 17.06.2345
39. Todos com o 2 A opo correta (e).
40. Tortas da vov A opo correta (d).
Vamos examinar cada uma das situaes propostas. Lembre que, no final, vov recebeu
7 + 6 + 3 2 = 14 docinhos.
(a) Impossvel, porque ela recebeu, no mnimo, 3 2 = 1 docinho de chocolate.

(b) Impossvel, porque ela recebeu, no mnimo, 6 2 = 4 docinhos de cco.


(c) Impossvel, porque 7 2 = 5 > 3.

(d) Possvel, porque Sofia pode ter comido um docinho de amora e um de chocolate,
restando 6 de amora, 6 de cco e 2 de chocolate para a vov.
(e) Impossvel, porque 7 no maior do que 6 + 3 2 = 7.
41. Famlia Stimo Os nascimentos ocorreram em seis dias 1o de abril, logo existem
irmos gmeos. Como nesse ano temos dois bolos a mais do que h dois anos, ento
h dois anos o mais jovem ainda no tinha nascido, o penltimo filho tinha acabado
de nascer e os gmeos j tinham nascido. Atualmente, o mais jovem tem um ano e os
gmeos tm x anos, com x 3. Como
1| + 2 + 3 + {z
4 + 5 + 6 + x} = 2
nmero de velas nesse ano

(1 + 2 + 3 + 4 + x 2) ,
{z
}
|

nmero de velas 2 anos atrs

temos x = 5. Logo, sero acesas 1 + 2 + 3 + 4 + 2 5 + 6 = 26 velinhas.


42. O salta-ficha

(a) A ficha 7 salta sobre as fichas 8 e 9 formando uma pilha com a ficha 10;
(b) a ficha 4 salta sobre as fichas 5 e 6 formando uma pilha com a ficha 8;
(c) a ficha 6 salta sobre as fichas 3 e 5 formando uma pilha com a ficha 2;
(d) a ficha 5 salta sobre a pilha (4, 8) formando uma pilha com a ficha 9 e
(e) a ficha 1 salta sobre a pilha (6, 2) formando uma pilha com a ficha 3.
O resultado segue.
350

OBMEP 2010

Solues dos Desafios


............ ............
............ ............ ...........
... ..... .... ..... ..... ...... ..... ..... .... .....
....
..
.
.
..
..
.... .... ...... ..... ...... ..... ...... .... ...... ....
..........
..........
..........
..........
..........
............... .................. .................. .................. ..................
.
.
... ..
.. ..
.. ...
... ...
...
.....
..
..
.
..... ..... ...... ...... ...... ..... ....... ..... ....... ......
.........
.........
........
........
........

9 10

43. O menor Como 52 = 32 + 42 , temos 52 002 = (32 + 42 )1 001 . Sabemos que, para a > 0
e b > 0,
(a + b)1 001 > a1 001 + b1 001 .
Assim, 52 002 > 32 002 + 42 002 .
44. O maior resultado Estamos procurando o maior valor de (10a + b)/(a + b), onde
a e b representam algarismos, sendo pelo menos um deles diferente de 0. Temos
9b
10a + b
10a + 10b 9b
10a + 10b
9b
=
=

= 10
10 .
a+b
a+b
a+b
a+b
a+b
Logo, se conseguirmos encontrar a e b tais que (10a + b)/(a + b) = 10, teremos o maior
resultado. Note que isso ocorre quando b = 0, ou seja,
10
20
30
40
50
60
70
80
90
=
=
=
=
=
=
=
=
= 10 .
1
2
3
4
5
6
7
8
9
Assim, a resposta 10.
45. Dois mil Observe que os nmeros 189, 8 307 e 99 tm todos peso 18 e que 99
o menor nmero que pesa 18. Para aumentar o peso de um nmero e minimizar o
nmero preciso que o nmero tenha o maior nmero possvel de algarismos 9. Por
outro lado, podemos dizer que o 0 est eliminado dos algarismos a ser considerados,
porque ele aumenta o nmero sem aumentar seu peso.
Temos que 2 000 = 9222+2. Logo, o nmero procurado deve ter 222 algarismos 9 e um
algarismo 2, ou dois algarismos 1. Eliminamos o caso dos nmeros com dois algarismos
1 porque eles tm 224 algarismos, sendo, portanto, maiores do que os nmeros que
possuem o algarismo 2 e tm 223 algarismos. Assim, o nmero procurado tem um 2
seguido de 222 algarismos 9: o nmero 299 . . . 999.
46. No cabeleireiro Seja x o montante inicial no caixa. Esse montante mais o que os
trs clientes pagaram nos dar o caixa zerado.
O primeiro cliente paga x 10 e, depois dele, h x + x 10 = 2x 10 reais no
caixa.
O segundo cliente paga (2x 10) 10 = 2x 20 e, depois dele, h
(2x 10) + (2x 20) = 4x 30 no caixa.
O terceiro cliente paga (4x 30) 10 = 4x 40 e depois dele h
(4x 30) + (4x 40) = 8x 70 no caixa.

Como o caixa est zerado depois do terceiro cliente, 8x 70 = 0, ou seja,


x = 70/8 = 8,75 reais.
OBMEP 2010

351

Solues dos Desafios


47. O macaco e a raposa 2 450 o produto dos nmeros primos 1, 2, 5, 5, 7 e 7.
As trs idades correspondem a uma combinao particular desses nmeros ou de seus
produtos. A raposa no pode descobrir as idades no incio porque pelo menos duas
dessas combinaes tm por soma o dobro de sua idade. De todas as combinaes
possveis, somente 5 + 10 + 49 e 7 + 7 + 50 tm a mesma soma 64.
1a concluso: a raposa tem 32 anos.
Depois da nova dica do macaco, a raposa descobriu as idades porque pode eliminar
uma combinao, a que contm dois nmeros iguais, uma vez que um deles o mais
jovem de todos.
2a concluso: as pessoas tm 5, 10 e 49 anos.
48. Nova sequncia Cada termo da sequncia a soma do termo precedente com os
quadrados de cada um de seus algarismos. De fato,
470 = 425 + 42 + 22 + 52 , 535 = 470 + 42 + 72 + 02 , . . .
Assim, depois de 802, os prximos termos sero 870 e 983.
49. Retngulo quase quadrado A rea um nmero da forma a a b b, onde a e b
representam algarismos, portanto
a a b b = 1100a + 11b = 11(100a + b).
Seja x a largura do terreno. Ento x(x + 1) = 11(100a + b) e deduzimos que x ou x + 1
um mltiplo de 11. Procurar mltiplos de 11 que satisfaam a condio obtida
bastante trabalhoso, por isso, para simplificar, vamos estabelecer quais os valores que
x pode ter, procurando seus valores mnimo e mximo.
Mnimo: a menor rea possvel 1 111, logo x(x + 1) = 1 111 e x > 32.

Mximo: a maior rea possvel 9 999, logo x(x + 1) = 9 999 e x < 100.
Agora testamos todos x entre 32 e 100 tais que x ou x + 1 seja mltiplo de 11 e que
x(x + 1) seja do tipo a a b b. Temos apenas doze opes, como segue.
33 34 = 1 122, 43 44 = 1 892, 44 45 = 1 980, 54 55 = 2 970,
55 56 = 2 970, 65 66 = 4 290, 66 67 = 4 422, 76 77 = 5 852,
77 78 = 6 006, 87 88 = 7 656, 88 89 = 7 832, 99 100 = 9 900.

Encontramos trs possibilidades para as dimenses do terreno, a saber, 33 34, 66 67


ou 99 100 metros.
50. Onde est o erro? Esse deixamos para os alunos!

352

OBMEP 2010

Contedo
. 05

09
11

27

45

Apresentao
Enunciados dos Problemas
Nvel 1
Aritmtica | 13 Geometria | 15 Diversos | 19 Desafios | 23

Nvel 2
Aritmtica e lgebra | 29 Geometria | 33 Combinatria | 37 Diversos | 41 Desafios | 43

Nvel 3
Aritmtica e lgebra | 47 Combinatria e Probabilidade | 51 Geometria | 53 Diversos | 57 Desafios | 59

Sugestes e Fatos que Ajudam

61

Solues

71

Nvel 1
Aritmtica | 73 Geometria | 81 Diversos | 89 Desafios | 95

Nvel 2
Aritmtica e lgebra | 103 Geometria | 109 Combinatria | 117 Diversos | 123 Desafios | 127

Nvel 3
Arit. e lgebra | 133 Comb. e Probabilidade | 141 Geometria | 147 Diversos | 155 Desafios | 159

163

Origem dos Problemas

167

+ Desafios

75

101

131

Apresentao

Desde da sua primeira edio em 2005, a OBMEP oferece a todas as escolas pblicas do pas um Banco
de Questes com problemas e desafios de matemtica para alunos e professores.
O Banco de Questes apresenta alguns problemas de matemtica originais e outros retirados de Olimpadas nacionais e internacionais passadas. Ele pretende despertar o prazer pela matemtica, estimular
o aluno interessado com perguntas instigantes e proporcionar um treinamento para as provas da OBMEP.
Esta nova edio, obra dos professores Paulo Rodrigues, Robrio Bacelar e Fbio Brochero, tem um
novo formato voltado para a segunda fase e prope 100 problemas e 20 desafios divididos por nvel e por
assunto. Ao final so propostos, sem resoluo, mais 30 desafios.
Percorrendo, ao final do livro, a origem dos problemas, o leitor poder constatar que esta edio traz
questes de um grau de dificuldade similar ao das olimpadas internacionais. Sugerimos portanto ao
aluno e ao professor comear com os problemas das edies anteriores do Banco de Questes, em regra
geral mais simples, e somente depois tentar resolver os problemas desta edio, sem nunca desanimar
se a soluo no vier imediatamente, lembrando que alguns problemas de matemtica famosos levaram
alguns sculos para serem resolvidos, e outros ainda no o foram at hoje.
Se voc, leitor(a), encontrar uma soluo para algum problema diferente da soluo apresentada ao
final do Banco de Questes, envie para bancodequestoes@obmep.org.br.

Boa diverso,
Claudio Landim
Coordenador Geral da OBMEP

Este banco dedicado a todos os professores


de matemtica, que no seu dia a dia, tm procurado superar cada desafio, mostrando aos
seus alunos que existem solues para cada
problema.

Enunciados dos Problemas

Nvel 1

1. Aritmtica
Nvel 1 Enunciados

1 | Mltiplo de 9 com Algarismos Pares


Encontre o menor mltiplo de 9 que no possui algarismos mpares.

(p. 75)

2 | Guardando Cubos
Uma caixa possui o formato de um bloco retangular de dimenses 102 cm, 255 cm e 170 cm. Queremos
guardar nessa caixa a menor quantidade possvel de pequenos cubos de aresta inteira, de forma a ocupar
toda a caixa.
(a) Qual a medida da aresta de cada bloco?
(b) Quantos blocos sero necessrios?
(p. 75)

3 | Calculadora Quebrada
Tio Man tem uma calculadora quebrada que no tem a tecla 0 e no visor nunca aparece 0 depois de
alguma operao. Assim, por exemplo, se ele multiplica 3 67, obtm como resposta 21, ao invs de 201.
Tio Man multiplicou dois nmeros de dois algarismos em sua calculadora e obteve no visor o nmero
11. Quais so os possveis nmeros que ele multiplicou?
(p. 76)

4 | Loja em Quixajuba
Uma loja em Quixajuba s vende artigos com preos de R$ 0, 99, R$ 1, 99, R$ 2, 99, e assim sucessivamente.
Tio Man realizou uma compra no valor total de R$ 125, 74. Quantos artigos ele pode ter comprado?(p. 76)

5 | Nmeros Sortudos
Dizemos que um nmero natural sortudo se todos os seus dgitos so iguais a 7. Por exemplo, 7 e 7777
so sortudos, mas 767 no . Joo escreveu num papel os vinte primeiros nmeros sortudos comeando
pelo 7, e depois somou-os. Qual o resto da diviso dessa soma por 1000?
(p. 77)

6 | Somando Idades
Cada pessoa de um grupo de dez pessoas calcula a soma das idades das outras nove integrantes do grupo.
As dez somas obtidas foram 82, 83, 84, 85, 87, 89, 90, 90, 91 e 92.
Determine a idade da pessoa mais jovem.
(p. 77)
13

14

Aritmtica | Nvel 1 | Enunciados

7 | Menor Soma Positiva


O produto de 50 nmeros inteiros consecutivos zero e a soma desses nmeros positiva. Qual o menor
valor que pode assumir essa soma?
(p. 77)

8 | Mdia dos Algarismos


Paulinho escreveu um nmero no quadro e depois inventou a seguinte brincadeira: escolhe dois algarismos do nmero que sejam ambos pares ou ambos mpares e troca cada um deles pela sua mdia
aritmtica. Ele repete este processo quantas vezes quiser, desde que o nmero disponha de dois algarismos com a mesma paridade. Por exemplo, ele escreveu o nmero 1368 e obteve a sequncia na qual
foram destacados os algarismos que sero trocados no passo seguinte.

1. 3 6

1. 3 7 7

4. 3 4 7

4. 5 4 5

(a) Com esta brincadeira, possvel obter o nmero 434434 a partir do nmero 324561?
(b) Paulinho escreveu o nmero 123456789 no quadro. Mostrar que com este processo, selecionando os
nmeros adequadamente, ele pode obter um nmero maior que 800000000.
(p. 78)

9 | Sequncia Numrica I
Todo termo de uma sequncia, a partir do segundo, igual soma do anterior com a soma de seus
algarismos. Os primeiros elementos da sequncia so

1, 2, 4, 8, 16, 23, 28, 38, 49, . . .


possvel que 793210041 pertena a essa sequncia?

(p. 78)

10 | Estrelas em Geometrix
Estrelix, um habitante de Geometrix, decidiu colocar os inteiros positivos seguindo a disposio indicada
na figura.

5
1.

4
3

16
8

7
6

12
11 14

10 13
9

27

15 19 23
18

22 25

17 21 24
20

26 30 34
29

33

28 32 35
31

Figura 10.1

Em quais estrelas aparece o nmero 2011? Posicione todos os nmeros que aparecem nas referidas
estrelas.
(p. 79)

www.obmep.org.br

OBMEP

2. Geometria
Nvel 1 Enunciados

11 | Bandeira do Tio Man


O Tio Man torcedor doente do Coco da Selva Futebol Clube e resolveu fazer uma bandeira para apoiar
seu time no jogo contra o Desportivo Quixajuba. Para isso, comprou um tecido branco retangular com
100 cm de largura e 60 cm de altura. Dividiu dois de seus lados em 5 partes iguais e os outros dois em 3
partes iguais, marcou o centro do retngulo e pintou o tecido da forma indicada na figura 11.1.

.
Figura 11.1

Qual a rea do tecido que Tio Man pintou?

(p. 81)

12 | Abelha na Flor
As flores de Geometrix tm formatos muito interessantes. Algumas delas possuem a forma mostrada
na figura 12.1, na qual h seis quadrados e doze tringulos equilteros.

Figura 12.1

Uma abelha pousou no ponto destacado e andou sobre a borda da flor no sentido horrio at voltar ao
ponto inicial. Sabendo que a regio cinza tem 24 cm2 de rea, qual a distncia percorrida pela abelha?
(p. 82)
15

16

Geometria | Nvel 1 | Enunciados

13 | ngulo da Asa Delta


Na figura 13.1, temos dois tringulos, ABC e ADC tais que AB = AD e CB = CD = CA. Sabendo que
= 25 , determine a medida do ngulo BCD
.
CBA

C
B

D
Figura 13.1

(p. 82)

14 | Azulejos de Pedro
Pedro um pedreiro. Ele tem um grande nmero de azulejos de trs tipos, como mostrado abaixo:

.
Figura 14.1

O menor lado de cada azulejo mede 10 cm. Ele quer ladrilhar completamente uma bancada de uma
cozinha sem cortar qualquer azulejo.
(a) Mostre como ele poder alcanar seu objetivo se a bancada for um retngulo 60 cm 50 cm.
(b) Mostre como ele poder alcanar seu objetivo se a bancada for um quadrado 60 cm 60 cm.
(p. 83)

15 | Retngulo 9 x 4
(a) Divida um retngulo 9 4 em trs peas e remonte-as de modo a formar um quadrado 6 6.
(b) Divida um retngulo 9 4 em duas peas e remonte-as de modo a formar um quadrado 6 6.
(p. 83)
www.obmep.org.br

OBMEP

Geometria | Nvel 1 | Enunciados

17

16 | Plantando Jasmins
O jardineiro Jacinto decidiu ajardinar um canteiro retangular com 10 m2 de rea. Dividiu o canteiro
traando uma diagonal e unindo cada um dos pontos mdios dos lados maiores com um vrtice do lado
oposto, como indicado na figura.

.
Figura 16.1

Na regio sombreada plantou jasmins. Qual a rea dessa regio?

(p. 84)

17 | Tangram
A figura 17.2 um retngulo cuja rea sombreada foi feita utilizando peas de um tangram que formam
um quadrado de 10 cm2 de rea, mostrado na figura 17.1.

.
Figura 17.1

Figura 17.2

Qual a rea do retngulo?

(p. 84)

18 | Tringulo Issceles I
= 30 e ABC
= 50 . A reta corta os lados AB, BC e o prolongamento
Seja ABC um tringulo com BAC
de AC em D, E e F, respectivamente.

F
C
E
50

30

Figura 18.1

Se o tringulo BDE issceles, quais so as trs possveis medidas para o ngulo C


FE?
www.obmep.org.br

(p. 85)
OBMEP

18

Geometria | Nvel 1 | Enunciados

19 | Formando um Retngulo
A partir de seis retngulos iguais e cinco quadrados iguais formado um retngulo de permetro 324 cm,
como mostrado na figura 19.1

.
Figura 19.1

Determine a rea do retngulo construdo.

(p. 86)

20 | Construindo uma Pipa


Para construir a pipa de papel representada na figura, Eduardo comeou por pintar um retngulo ABCD
numa folha de papel. Em seguida, prolongou cada um dos lados do retngulo triplicando o seu comprimento e obteve o quadriltero A B C D .

A
B .

C
Figura 20.1

Sabendo que a rea do retngulo ABCD 200 cm2 , qual a rea da pipa construda por Eduardo?(p. 87)

www.obmep.org.br

OBMEP

3. Diversos
Nvel 1 Enunciados

21 | Colorindo Mapas
No mapa da figura 21.1 a curva XY uma das fronteiras. Pases como I e II tm fronteira comum. O
ponto Y no considerado fronteira, ou seja, pases como I e V no tm fronteira comum. Voc deve
colorir o mapa fazendo pases de fronteira comum terem cores diferentes.
X

II

III

VI

IV
.

Figura 21.1

(a) Qual o nmero mnimo de cores para colorir o mapa? Mostre como colori-lo.
(b) Desenhe outro mapa de 6 pases, que precise de pelo menos 4 cores para ser pintado. Mostre como
colori-lo com cores A, B, C e D.
(p. 89)

22 | De Coco da Selva a Quixajuba


As cidades de Coco da Selva e Quixajuba esto ligadas por uma linha de nibus. De Coco da Selva saem
nibus para Quixajuba de hora em hora e o primeiro parte meia-noite em ponto. De Quixajuba saem
nibus para Coco da Selva de hora em hora e o primeiro parte meia-noite e meia em ponto. A viagem
de nibus feita em exatamente 5 horas.
Se um nibus sai de Coco da Selva ao meio-dia, quantos nibus vindo de Quixajuba ele encontra durante
o percurso?
(p. 89)

23 | O Baralho de Joo
Joo possui um baralho com 52 cartas numeradas de 1 at 52. Um conjunto de trs cartas chamado
sortudo se a soma dos algarismos em cada carta a mesma. Qual o nmero mnimo de cartas que
Joo tem de pegar do baralho, sem olhar, de tal forma que entre as cartas que ele pegou necessariamente
existam trs cartas que formam um conjunto de cartas sortudo?
(p. 90)
19

20

Diversos | Nvel 1 | Enunciados

24 | Moedas e Pesagens
Ana possui 48 moedas aparentemente iguais. Porm, exatamente uma das moedas falsa e tem peso
diferente do peso das outras. Ela possui uma balana eletrnica que mede o peso total de qualquer
quantidade de moedas. Mostre como ela pode determinar a moeda falsa realizando sete pesagens.(p. 90)

25 | Distribuindo Mas
Noventa e nove mas so distribudas entre alguns garotos de tal forma que todos recebem quantidades
diferentes de mas.
(a) Qual o nmero mximo de garotos que pode haver nesse grupo?
(b) Havendo dez garotos, qual o nmero mximo de mas que recebe o garoto que ganhou menos mas?
(p. 91)

26 | Maria e seus Convidados


Maria convidou nove garotos e oito garotas para sua festa de aniversrio. Ela preparou camisetas com
os nmeros de 1 a 18, ficou com a de nmero 1 e distribuiu as demais para seus convidados. Durante
uma dana, ela observou que a soma dos nmeros de cada casal era um quadrado perfeito. Quais pares
estavam danando?
(p. 91)

27 | Cartes de Apostas
Trs apostadores A, B e C preenchem individualmente um carto de apostas, dos possveis resultados
de cinco jogos de futebol (C = vitria do time da casa, E = empate, V = vitria do visitante). Os cartes
preenchidos foram:

1
2
3
4
5

Apostador A

C
1
2
3
4
5

Apostador B

1
2
3
4
5

Apostador C

Finalizadas as partidas, observou-se que A obteve trs acertos, B obteve trs acertos e C obteve dois
acertos. Construa um carto com cinco acertos.
(p. 92)

28 | Nmeros de 1 a 16
(a) Mostre que os nmeros de 1 a 16 podem ser escritos numa reta, de tal modo que a soma de quaisquer
dois nmeros vizinhos seja um quadrado perfeito.
(b) Mostre que os nmeros de 1 a 16 no podem ser escritos ao redor de uma circunferncia, de tal
modo que a soma de quaisquer dois nmeros vizinhos seja um quadrado perfeito.
(p. 92)
www.obmep.org.br

OBMEP

Diversos | Nvel 1 | Enunciados

21

29 | Calculando Somas
Considere um tabuleiro com 11 linhas e 11 colunas.

.
Figura 29.1

(a) Quantas casas formam este tabuleiro?


(b) A diagonal cujas casas esto sombreadas separa o tabuleiro em duas regies: uma acima e outra
abaixo. Quantas casas formam cada regio? possvel calcular esse nmero sem contar casa por
casa?
(c) Com a ajuda do tabuleiro, possvel calcular a soma 1 + 2 + + 10. Explique como.
(d) Com a ajuda de outro tabuleiro, com o raciocnio semelhante ao do item anterior, possvel calcular
a soma 1 + 2 + + 100. Qual deve ser a quantidade de linhas e colunas do tabuleiro? Qual o valor
da soma?
(p. 93)

30 | Herana para Cinco Filhos


Divida a figura 30.1 em cinco partes do mesmo formato e com reas iguais de tal modo que cada parte
contenha exatamente um quadrado cinza.

.
Figura 30.1

(p. 94)

www.obmep.org.br

OBMEP

4. Desafios
Nvel 1 Enunciados

31 | Vizinhos e Distantes
possvel escrever os nmeros naturais de 1 a 100 sobre uma reta de modo que a diferena entre quaisquer dois nmeros vizinhos seja maior ou igual a 50?
(p. 95)

32 | Truque com Cartas


Um mgico com os olhos vendados d 29 cartas numeradas de 1 a 29 para uma mulher da plateia. Ela
esconde duas cartas no bolso e devolve as restantes para a assistente do mgico.
A assistente escolhe duas cartas dentre as 27 e um homem da plateia l, na ordem que quiser, o nmero
destas cartas para o mgico. Aps isto, o mgico adivinha o nmero das cartas que foram escondidas
pela mulher.
Como o mgico e sua assistente podem combinar uma estratgia para realizarem esse truque? (p. 95)

33 | Campeonato de Quixajuba
A tabela mostra a classificao final do campeonato de futebol de Quixajuba. Neste campeonato cada
time jogou com cada um dos outros quatro vezes. Cada time ganha 3 pontos por vitria, 1 por empate e
no ganha pontos em caso de derrota.
Equipe
Bissetriz
Primo
Potncia
MDC

Pontos
22
19
14
12

(a) Quantas partidas foram disputadas no campeonato?


(b) Quantas partidas terminaram empatadas?
(p. 96)
23

24

Desafios | Nvel 1 | Enunciados

34 | Tabuleiro 6 x 6
Voc dispe de doze peas em formato de L, como a mostrada na figura 34.1. Cada figura formada por
trs quadrados de lado 1. Mostre como cobrir um quadrado 6 6 com essas peas, de modo que nenhum
retngulo 2 3 seja formado por exatamente duas de tais peas.

.
Figura 34.1

(p. 96)

35 | Somando Algarismos
Quantos nmeros naturais de trs algarismos so tais que a soma destes igual a 24?

(p. 97)

36 | Contando Quadrados
Doze pontos so marcados sobre uma grade de pontos, como mostrado na figura 36.1.

.
Figura 36.1

Quantos quadrados podem ser formados ligando quatro desses pontos?

(p. 97)

37 | A Moeda Falsa
Temos 25 moedas aparentemente iguais, mas sabemos que exatamente uma delas falsa e tem o peso
diferente do peso das outras.
No sabemos qual a moeda falsa. Todas as outras 24 moedas possuem o mesmo peso.
Queremos determinar, utilizando uma balana de pratos, se a moeda falsa mais leve ou mais pesada
que as outras.
Como podemos alcanar este objetivo realizando duas pesagens em uma balana de pratos?

No queremos encontrar a moeda falsa. Queremos saber se ela mais leve ou mais pesada que as outras.
Nesse tipo de balana podemos comparar os pesos colocados nos dois pratos, ou seja, a balana pode equilibrar
ou pender para o lado mais pesado.

(p. 98)
www.obmep.org.br

OBMEP

Desafios | Nvel 1 | Enunciados

25

38 | O Tabuleiro Mutilado
A figura abaixo mostra um tabuleiro 8 8 no qual duas casas foram retiradas (a do canto inferior direito
e a do canto superior esquerdo). possvel cobrir este tabuleiro com 31 domins 2 1? Cada domin
pode ser colocado na horizontal ou na vertical cobrindo exatamente duas casas.

Figura 38.1

(p. 98)

39 | Dividindo um Retngulo
(a) possvel dividir um retngulo 39 55 em retngulos 5 11?
(b) possvel dividir um retngulo 55 27 em retngulos 5 11?
(p. 99)

40 | Nmeros no Tabuleiro 4 x 4
Guilherme escreveu 0 ou 1 em cada casa de um tabuleiro 4 4. Ele colocou os nmeros de modo que a
soma dos nmeros das casas vizinhas de cada casa do tabuleiro fosse igual a 1.
Por exemplo, na figura 40.1, considerando a casa marcada com , a soma dos nmeros das casas sombreadas igual a 1.
.

.
Figura 40.1

Determine a soma de todos os 16 nmeros do tabuleiro.


(p. 100)

www.obmep.org.br

OBMEP

Nvel 2

.
5. Aritmtica
e lgebra

Nvel 2 Enunciados

41 | Mltiplo de 36
Determine o maior mltiplo de 36 que possui todos os algarismos pares e diferentes.

(p. 103)

42 | Quem maior?
Sejam

R = 3 9 + 4 10 + 5 11 + + 2003 2009.
e

S = 1 11 + 2 12 + 3 13 + + 2001 2011
(a) Qual o maior nmero: R ou S?
(b) Calcule a diferena entre o maior e o menor.
(p. 103)

43 | Resto da Diviso
Um nmero n de dois algarismos dividido pela soma de seus algarismos, obtendo resto r.
(a) Encontre um nmero n tal que r = 0.
(b) Mostre que r no pode ser maior que 15.
(c) Mostre que para qualquer r menor ou igual a 12, existe um n que deixa resto r ao dividi-lo pela soma
de seus algarismos.
(p. 104)

44 | Soma de Consecutivos
(a) A soma de quatro inteiros positivos consecutivos pode ser um nmero primo? Justifique sua resposta.
(b) A soma de trs inteiros positivos consecutivos pode ser um nmero primo? Justifique sua resposta.
(p. 104)
29

30

Aritmtica e lgebra | Nvel 2 | Enunciados

45 | Quadrado Perfeito
Observe que

12 + 22 + (1 2)2 = 32
22 + 32 + (2 3)2 = 72
32 + 42 + (3 4)2 = 132 .
Prove que se a e b so inteiros consecutivos ento o nmero

a2 + b2 + (ab)2
um quadrado perfeito.

(p. 105)

46 | Quantas Fraes!
Prove que

1
2+
4+

1
..

.+

1
1991

= 1.

1+

3+

1+

3+
4+

1
..

.+

1
1991

(p. 105)

47 | Primos No!
(a) Prove que o nmero 3999991 no primo.
(b) Prove que o nmero 1000343 no primo.
(p. 106)

48 | Trilegais
Um conjunto de nmeros chamado trilegal se pode ser dividido em subconjuntos com trs elementos de
tal modo que um dos elementos seja a soma dos outros dois. Por exemplo, o conjunto {1, 2, 3, . . . , 11, 12}
trilegal pois pode ser dividido em {1, 5, 6}, {2, 9, 11}, {3, 7, 10} e {4, 8, 12}.
(a) Mostre que {1, 2, . . . , 14, 15} trilegal.
(b) Mostre que {1, 2, . . . , 2010} no trilegal.
(p. 106)

49 | Diferena de Quadrados
(a) De quantas formas possvel escrever o nmero 105 como diferena de dois quadrados perfeitos?
(b) Mostre que no possvel escrever o nmero 106 como diferena de dois quadrados perfeitos.
(p. 107)
www.obmep.org.br

OBMEP

Aritmtica e lgebra | Nvel 2 | Enunciados

31

50 | Outra de Joozinho
Joozinho escreveu os nmeros de 1 at 100000 no quadro, depois foi trocando cada nmero pela soma
de seus algarismos e repetiu este processo at obter uma lista de 100000 nmeros de um algarismo. Por
exemplo, comeando pelo nmero 7234 obtemos 7 + 2 + 3 + 4 = 16 e 1 + 6 = 7.
(a) Que nmero ficou no lugar do nmero 98765?
(b) Quantas vezes aparece o nmero 8 na lista final?
(c) Qual o nmero que mais vezes se repete?
(p. 108)

www.obmep.org.br

OBMEP

6. Geometria

Nvel 2 Enunciados

51 | Colar de Ouro
Arquelogos encontraram um colar de ouro feito de placas no formato de pentgonos regulares. Cada
uma destas placas est conectada a outras duas placas, como ilustra a figura.
.

Figura 51.1

Quantas placas formam o colar?

(p. 109)

52 | AP x BN
ABCD um retngulo, AD = 5 e CD = 3.
A

D
N
P

C
Figura 52.1

Se BN perpendicular a AP, calcule AP BN.

(p. 109)

33

34

Geometria | Nvel 2 | Enunciados

53 | Dois Quadrados
Na figura, ABCD e CEFG so quadrados e o lado do quadrado CEFG mede 12 cm.

D
G

Figura 53.1

Quais so os possveis valores da rea do tringulo AEG?

(p. 110)

54 | O Tesouro do Pirata
Um pirata resolveu enterrar um tesouro em uma ilha. Para tal, ele caminhou da rvore A para a rocha R1 ,
e depois a mesma distncia e na mesma direo at o ponto X. Ele fez o mesmo em relao a entrada da
caverna C e em relao rocha R2 , alcanando os pontos Y e Z, respectivamente. Ele enterrou o tesouro
em T , ponto mdio de AZ.

Y
C
X
R2
R1 .

Figura 54.1

Ao voltar ilha para desenterrar o tesouro, o pirata encontrou as rochas e a caverna, mas no encontrou
a rvore. Como o pirata pode descobrir o tesouro?
(p. 111)

55 | Bissetrizes
Seja ABC um tringulo com AB = 13, BC = 15 e AC = 9. Seja r a reta paralela a BC traada por A. A
b corta a reta r em E e a bissetriz do ngulo ACB
b corta r em F. Calcular a medida
bissetriz do ngulo ABC
do segmento EF.
(p. 112)

56 | ngulos e ngulos!
= = ECA
,
No interior de um tringulo ABC, toma-se um ponto E tal que AE = BE e AB = EC. Se ABE

EAC = 2 e EBC = 5, determine .


(p. 112)
www.obmep.org.br

OBMEP

Geometria | Nvel 2 | Enunciados

35

57 | Quadrado, Pentgono e Icosgono


A figura mostra parte de um polgono regular de 20 lados (icosgono) ABCDEF..., um quadrado BCYZ
e um pentgono regular DEVWX.

C
D
X

F
W

Figura 57.1

.
(a) Determine a medida do ngulo Y DC
(b) Mostre que o vrtice X est sobre a reta DY .
(p. 113)

58 | Enegono Regular
A figura ilustra um polgono regular de 9 lados. A medida do lado do polgono a, a medida da menor
diagonal b e a medida da maior diagonal d.

A
B
a
b

d.

C
D

E
Figura 58.1

.
(a) Determine a medida do ngulo BAE
(b) Mostre que d = a + b.
(p. 114)

59 | Hexgono Equiangular
Todos os ngulos de um hexgono ABCDEF so iguais. Mostre que AB DE = EF BC = CD FA.
(p. 115)

60 | Pentgono Equiltero
Mostre que possvel construir um pentgono com todos os lados de mesma medida e cujos ngulos
internos meam 60 , 80 , 100 , 140 e 160 , em alguma ordem.
(p. 115)
www.obmep.org.br

OBMEP

7. Combinatria
Nvel 2 Enunciados

61 | Coloraes do Cubo
De quantas formas possvel colorir as 6 faces de um cubo de preto ou branco? Duas coloraes so
iguais se possvel obter uma a partir da outra por uma rotao.
(p. 117)

62 | Comparando Sequncias
Um professor e seus 30 alunos escreveram, cada um, os nmeros de 1 a 30 em uma ordem qualquer.
A seguir, o professor comparou as sequncias. Um aluno ganha um ponto cada vez que um nmero
aparece na mesma posio na sua sequncia e na do professor. Ao final, observou-se que todos os
alunos obtiveram quantidades diferentes de pontos. Mostre que a sequncia de um aluno coincidiu com
a sequncia do professor.
(p. 117)

63 | Segmentos e Tringulos
Dez pontos so marcados ao redor de uma circunferncia, como ilustra a figura.

Figura 63.1

(a) Quantas cordas podem ser formadas ligando dois quaisquer destes pontos? (Uma corda um segmento de reta ligando dois pontos sobre uma circunferncia.)
(b) Quantos tringulos podem ser formados ligando trs quaisquer destes pontos?
(p. 118)
37

38

Combinatria | Nvel 2 | Enunciados

64 | Esqueleto do Cubo
O esqueleto de um cubo 6 6 6, formado por cubinhos 1 1 1 mostrado na figura.

Figura 64.1

(a) Quantos cubinhos formam este esqueleto?


(b) dado um cubo 7 7 7 formado por cubinhos 1 1 1. Quantos cubinhos devemos retirar para
obter um esqueleto do cubo 7 7 7.
(p. 119)

65 | Placas das Bicicletas


Cada uma das placas das bicicletas de Quixajuba contm trs letras. A primeira letra escolhida dentre
os elementos do conjunto A = {G, H, L, P, R}, a segunda letra escolhida dentre os elementos do conjunto
B = {M, I, O} e a terceira letra escolhida dentre os elementos do conjunto C = {D, U, N, T }.
Devido ao aumento no nmero de bicicletas da cidade, teve-se que expandir a quantidade de possibilidades de placas. Ficou determinado acrescentar duas novas letras a apenas um dos conjuntos ou uma letra
nova a dois dos conjuntos.
Qual o maior nmero de novas placas que podem ser feitos, quando se acrescentam as duas novas letras?
(p. 119)

66 | Torneio de Tnis
Num torneio de tnis cada jogador passa para a rodada seguinte somente em caso de vitria. Se no for
possvel que sempre passe para a rodada seguinte um nmero par de jogadores, a organizao do torneio
decide quais rodadas determinados jogadores devem jogar. Por exemplo, um cabea de chave pode, a
critrio dos organizadores, entrar na segunda rodada, ou passar da primeira para a terceira, de modo que
o total de jogadores que participem de cada rodada seja par.
(a) Considere um torneio de tnis com 64 jogadores. Quantas partidas so disputadas?
(b) E em um torneio com 2011 jogadores?
(p. 120)

67 | Pesando Pedras
Possumos 32 pedras, todas com pesos diferentes. Descreva um processo para mostrar que podemos
encontrar as duas pedras mais pesadas com 35 pesagens em uma balana de pratos.
(p. 121)

www.obmep.org.br

OBMEP

Combinatria | Nvel 2 | Enunciados

39

68 | Produto 2000
Quantos nmeros naturais de cinco algarismos tm o produto de seus algarismos igual a 2000? (p. 121)

69 | Tabuleiro 123 x 123


Num tabuleiro 123 123, cada casa pintada de roxo ou azul de acordo com as seguintes condies:

Cada casa pintada de roxo que no est na borda do tabuleiro tem exatamente 5 casas azuis dentre
suas 8 vizinhas.
Cada casa pintada de azul que no est na borda do tabuleiro tem exatamente 4 casas roxas dentre
suas 8 vizinhas.
Nota: Duas casas so vizinhas se possuem um lado ou um vrtice em comum.
(a) Considere um tabuleiro 3 3 dentro do tabuleiro 123 123. Quantas casas de cada cor pode haver
neste tabuleiro 3 3?
(b) Calcule o nmero de casas pintadas de roxo no tabuleiro 123 123.
(p. 122)

www.obmep.org.br

OBMEP

8. Diversos
Nvel 2 Enunciados

70 | Nmeros no W
Em cada uma das casas do W da figura, escrevemos um nmero inteiro de 1 a 9 de modo que a soma dos
trs nmeros de cada uma das quatro linhas seja a mesma.
.

6
9
Figura 70.1

J esto escritos o 6 e o 9. Como devem ser posicionados os outros nmeros?

(p. 123)

71 | Montando Tabelas
Montar a tabela de um torneio em que todas as n equipes se enfrentam ao longo de n 1 rodadas (como,
por exemplo, em cada turno do Brasileiro) um problema matemtico bastante elaborado e que possui
vrios mtodos de soluo. Nesta questo, vamos conhecer uma dessas abordagens.
Vamos considerar um torneio com 6 equipes. Associaremos os nmeros 1, 2, 3, 4, 5 e (infinito) a cada
uma das equipes. A primeira rodada do torneio 1, 25, 34. Para montarmos a rodada i somamos
i 1 a cada nmero envolvido nas partidas da rodada inicial, considerando que

quando a soma ultrapassa 5, subtramos 5 do resultado;


adicionado a qualquer inteiro positivo . Por exemplo, a segunda rodada ser:
(1 + 1) ( + 1), isto , 2
(2 + 1) (5 + 1), isto , 3 1
(3 + 1) (4 + 1), isto , 4 5
(a) Determine as 3 rodadas restantes do torneio, seguindo o mtodo descrito acima.
(b) A partir do procedimento mostrado, exiba as 7 rodadas de um torneio com 8 equipes.
(p. 124)
41

42

Diversos | Nvel 2 | Enunciados

72 | Numerando os Vrtices
Distribumos nos vrtices de um bloco retangular oito nmeros dentre 1, 2, 3, 4, 5, 6, 7, 8, 9, 10 de tal
forma que a soma dos nmeros de uma face qualquer seja igual a 18.
(a) Quais os nmeros descartados na distribuio?
(b) Exiba uma possvel distribuio.
(p. 125)

73 | Corrida de So Paulo a Fortaleza


Numa corrida de So Paulo a Fortaleza participam quatro carros A, B, C, D que largaram na seguinte
ordem: primeiro A, segundo B, terceiro C e por ltimo D. Durante a corrida, A e B trocaram de posio
(ultrapassaram um ao outro) 9 vezes e B e C trocaram de posio 8 vezes.
Para saber em que ordem chegaram Fortaleza, s permitido fazer perguntas do tipo:
Quantas vezes trocaram de posio os carros X e Y ?
Antes de fazer uma pergunta se conhece a resposta da pergunta anterior. Formule trs perguntas que
permitam determinar a ordem em que os quatro terminaram a corrida.
(p. 125)

74 | Casas Pretas e Brancas


Considere um tabuleiro 6 6 com suas casas coloridas de branco ou preto. Duas casas so chamadas
vizinhas se possuem um lado comum. A colorao do tabuleiro vai mudando a cada segundo, respeitando
a seguinte condio: se num determinado segundo pelo menos duas casas vizinhas de uma determinada
casa esto coloridas de preto, ento no prximo segundo esta ltima casa ser colorida de preto.
(a) A figura abaixo mostra uma possvel colorao inicial. Como ficar o tabuleiro aps 12 segundos? E
aps 13 segundos?

.
(b) Exiba uma colorao inicial com 6 casas pretas de modo que, em algum momento, todas as casas
fiquem pretas.
(p. 125)

www.obmep.org.br

OBMEP

9. Desafios
Nvel 2 Enunciados

75 | Ora Bolas!
Cinco bolas iguais esto se movendo na mesma direo ao longo de uma reta fixa, mantendo uma certa
distncia de uma para outra. Na mesma direo, mas no sentido oposto, outras cinco bolas se movem de
encontro s primeiras. As velocidades de todas as bolas so iguais. Quando duas bolas colidem, voltam
na mesma velocidade de antes, ao longo da mesma direo. Quantas colises entre bolas vo ocorrer?
(p. 127)

76 | Distncia entre os Vilarejos


A estrada que liga dois vilarejos em uma montanha formada somente por trechos de subida ou descida.
Um nibus sempre viaja a 15 km/h em trechos de subida e a 30 km/h em trechos de descida. Encontre
a distncia entre os vilarejos se o nibus leva exatamente 4 horas para fazer a viagem completa de ida e
volta.
(p. 127)

77 | Amigos que voc pode Contar!


Considere um grupo de 15 pessoas. possvel que cada uma delas conhea exatamente:
(a) 4 pessoas do grupo?
(b) 3 pessoas do grupo?
(Admita que se A conhece B ento B conhece A.)

(p. 128)

78 | Trs Amigos e uma Bicicleta


A distncia entre Coco da Selva e Quixajuba 24 km. Dois amigos precisam ir de Quixajuba a Coco
da Selva e um terceiro amigo precisa ir de Coco da Selva a Quixajuba. Eles possuem uma bicicleta que
inicialmente est em Quixajuba. Cada um deles pode ir caminhando a velocidade de 6 km/h, ou de
bicicleta a velocidade de 18 km/h. Alm disso, podem deixar a bicicleta em qualquer ponto do trajeto.
Coco da Selva

Quixajuba

.
Mostre como eles podem proceder para chegarem a seus destinos em no mximo 2h 40min.

43

(p. 128)

44

Desafios | Nvel 2 | Enunciados

79 | Contando Polgonos
Em uma circunferncia foram marcados 15 pontos brancos e 1 ponto preto. Consideremos todos os
possveis polgonos (convexos) com seus vrtices nestes pontos.
Vamos separ-los em dois tipos:

Tipo 1: os que possuem somente vrtices brancos.


Tipo 2: os que possuem o ponto preto como um dos vrtices.
Existem mais polgonos do tipo 1 ou do tipo 2? Quantos existem a mais?

(p. 129)

80 | Desafiando os Amigos!
(a) Adriano escolheu secretamente cinco nmeros a, b, c, d e e e informou a Bruna os dez nmeros 24,
28, 30, 30, 32, 34, 36, 36, 40 e 42 obtidos pelo clculo de todas as somas de dois nmeros dentre os
cinco escolhidos.
O objetivo de Bruna descobrir a, b, c, d, e. Bruna pode alcanar seu objetivo?
(b) Adriano escolheu secretamente quatro nmeros m, n, p e q e informou a Carlos os seis nmeros 10,
20, 22, 24, 26 e 36 obtidos pelo clculo de todas as somas de dois nmeros dentre os quatro escolhidos.
O objetivo de Carlos descobrir m, n, p e q. Ele pode alcanar seu objetivo?
(p. 130)

www.obmep.org.br

OBMEP

Nvel 3

.
10. Aritmtica
e lgebra

Nvel 3 Enunciados

81 | Sequncia Numrica II
A sequncia de nmeros t1 , t2 , t3 , . . . est definida por

t1 = 2
tn+1 =

tn 1
tn + 1

para cada inteiro positivo n. Encontrar t2011 .

(p. 133)

82 | Progresso Geomtrica
A progresso geomtrica 121, 242, 484, 968, 1936,. . . possui trs termos inteiros entre 200 e 1200.
(a) Encontre uma progresso geomtrica crescente que possui quatro termos inteiros entre 200 e 1200.
(b) Encontre uma progresso geomtrica crescente que possui seis termos inteiros entre 200 e 1200.
(p. 134)

83 | Funciona?
Para um inteiro positivo n considere a funo

4n + 4n2 1

.
f(n) =
2n + 1 + 2n 1
Calcule o valor de

f(1) + f(2) + f(3) + + f(40).


(p. 134)

84 | Sistema de Trs Equaes


Sejam a e b nmeros reais tais que existam nmeros reais distintos m, n e p, satisfazendo as igualdades
abaixo:

m + am + b = 0
3
n + an + b = 0

3
p + ap + b = 0.

Mostre que m + n + p = 0.

(p. 135)

47

48

Aritmtica e lgebra | Nvel 3 | Enunciados

85 | Soma de Potncias
(a) Mostre que a identidade abaixo sempre verdadeira:

an+1 + bn+1 = (a + b)(an + bn ) ab(an1 + bn1 ).


(b) Sejam a e b nmeros reais tais que a + b = 1 e ab = 1. Mostre que o nmero a10 + b10 inteiro,
calculando seu valor.
(p. 135)

86 | Sistema com Potncias


(a) Verifique a identidade

(a + b + c)3 = a3 + b3 + c3 + 3(a + b)(b + c)(c + a).


(b) Resolva o sistema

x + y + z = 1
x2 + y2 + z2 = 1

3
x + y3 + z3 = 1.
(p. 136)

87 | Sistema com 7 Variveis


(a) Determine a, b e c tais que a igualdade

(n + 2)2 = a(n + 1)2 + bn2 + c(n 1)2


seja verdadeira qualquer que seja o nmero n.
(b) Suponha que x1 , x2 , . . . , x7 satisfazem o sistema

x1 + 4x2 + 9x3 + 16x4 + 25x5 + 36x6 + 49x7 = 1


4x1 + 9x2 + 16x3 + 25x4 + 36x5 + 49x6 + 64x7 = 12

9x1 + 16x2 + 25x3 + 36x4 + 49x5 + 64x6 + 81x7 = 123


Determine o valor de

16x1 + 25x2 + 36x3 + 49x4 + 64x5 + 81x6 + 100x7 .


(p. 137)

88 | Algarismo do Quadrado
O quadrado de 13 169, que tem como algarismo das dezenas o nmero 6. O quadrado de outro nmero tem como algarismo das dezenas o nmero 7. Quais so os possveis valores para o algarismo das
unidades desse quadrado?
(p. 138)

www.obmep.org.br

OBMEP

Aritmtica e lgebra | Nvel 3 | Enunciados

49

89 | Maior Divisor mpar


Seja n um nmero inteiro positivo. Para cada um dos inteiros n + 1, . . ., 2n considere o seu maior divisor
mpar. Prove que a soma de todos estes divisores igual a n2 .
(p. 138)

90 | Algarismos
Com os algarismos a, b e c construmos o nmero de trs algarismos abc e os nmeros de dois algarismos

ab, bc e ca. Ache todos os possveis valores de a, b e c tais que


(p. 139)

www.obmep.org.br

abc + a + b + c
seja um nmero inteiro.
ab + bc + ca

OBMEP

.
11. Combinatria
e Probabilidade

Nvel 3 Enunciados

91 | Produto Par
Tio Man tem duas caixas, uma com sete bolas distintas numeradas de 1 a 7 e outra com oito bolas
distintas numeradas com todos os nmeros primos menores que 20. Ele sorteia uma bola de cada caixa.
Qual a probabilidade de que o produto dos nmeros das bolas sorteadas seja par?
(p. 141)

92 | Subconjuntos com Soma Grande


Considere o conjunto A = {1, 2, 3, . . . , 2011}. Quantos subconjuntos de A existem de modo que a soma
de seus elementos seja 2023060?
(p. 141)

93 | Formiga Aleatria
Uma formiga se movimenta uma unidade por segundo sobre os pontos 0, 1 e 2 da figura a seguir, comeando do ponto 0.
.

Figura 93.1

(a) Quais so os possveis percursos da formiga at 3 segundos?


(b) Quantos possveis percursos pode fazer a formiga at 10 segundos?
(p. 142)

94 | Algarismos e Paridade
Tiago escreve todos os nmeros de quatro algarismos no nulos distintos que possuem a mesma paridade.
Qual a probabilidade de que, ao escolhermos um desses nmeros, ele seja par?
(p. 142)

95 | Bolas Pretas, Brancas e Azuis


Considere uma urna que contm uma bola preta, quatro bolas brancas e algumas bolas azuis. Uma bola
retirada ao acaso dessa urna, sua cor observada e a bola devolvida urna. Em seguida, retira-se
novamente, ao acaso, outra bola dessa urna. Para quais quantidades de bolas azuis, a probabilidade das
duas bolas retiradas terem mesma cor vale 1/2?
(p. 143)

51

52

Combinatria e Probabilidade | Nvel 3 | Enunciados

96 | Aparando um Poliedro
Considere um poliedro convexo com 100 arestas. Todos os vrtices foram aparados prximos a eles mesmos, usando uma faca plana afiada (isto foi feito de modo que os planos resultantes no se intersectassem
no interior ou na fronteira do poliedro). Calcule para o poliedro resultante:
(a) o nmero de vrtices.
(b) o nmero de arestas.
(p. 143)

97 | Bolas Azuis e Vermelhas


Existem bolas azuis e bolas vermelhas em uma caixa. A probabilidade de sortear duas bolas de cores
diferentes, ao retirar duas bolas ao acaso, 1/2. Prove que o nmero de bolas na caixa um quadrado
perfeito.
(p. 144)

98 | Dez Pontos no Plano


Dez pontos so dados no plano e no existem trs colineares. Quatro segmentos distintos ligando pares
destes pontos so escolhidos ao acaso, mas todos com a mesma probabilidade. Qual a probabilidade
de trs dos segmentos escolhidos formarem um tringulo?
(p. 144)

99 | Contando Diagonais no Poliedro


Um poliedro convexo P tem 26 vrtices, 60 arestas e 36 faces. 24 faces so triangulares e 12 so quadrilteros. Uma diagonal espacial um segmento de reta unindo dois vrtices no pertencentes a uma mesma
face. P possui quantas diagonais espaciais?
(p. 145)

100 | Grade de Pontos


Uma grade de pontos com 10 linhas e 10 colunas dada. Cada ponto colorido de vermelho ou de azul.
Sempre que dois pontos da mesma cor so vizinhos em uma mesma linha ou coluna, eles so ligados
por um segmento da mesma cor dos pontos. Se dois pontos so vizinhos mas de cores diferentes, so
ligados por um segmento verde. No total, existem 52 pontos vermelhos. Destes vermelhos, 2 esto nos
cantos e outros 16 esto no bordo da grade. Os outros pontos vermelhos esto no interior da grade.

.
Existem 98 segmentos verdes. Determine o nmero de segmentos azuis.

www.obmep.org.br

(p. 145)

OBMEP

12. Geometria
Nvel 3 Enunciados

101 | Tringulo 20 40 120


mede 20 e o ngulo ACB
mede 40 . Seja E um ponto sobre BC tal
Num tringulo ABC, o ngulo ABC
que BE = BA.
(a) Mostre que o tringulo CEA issceles.

2, determine BC AB.
(b) Sabendo que o comprimento da bissetriz do ngulo BAC
(p. 147)

102 | Um Problema Antigo!


Duas torres, uma com 30 passos e a outra com 40 passos de altura, esto distncia de 50 passos uma
da outra. Entre ambas se acha uma fonte, para a qual dois pssaros descem no mesmo momento do alto
das torres com a mesma velocidade e chegam ao mesmo tempo. Quais as distncias horizontais da fonte
s duas torres?(Leonardo de Pisa, Liber Abaci, 1202).
(p. 148)

103 | Circunferncias Tangentes


As circunferncias C1 e C2 so tangentes reta nos pontos A e B e tangentes entre si no ponto C. Prove
que o tringulo ABC retngulo.

O2
C

O1

Figura 103.1

(p. 148)

104 | Tringulo Issceles II


b = 30 . Seja D o ponto mdio da base BC. Sobre AD
Seja ABC um tringulo issceles com AB = AC e A
b .
e AB tome dois pontos P e Q, respectivamente, tais que PB = PQ. Determine a medida do ngulo P QC
(p. 149)

53

54

Geometria | Nvel 3 | Enunciados

105 | Circunferncia no Setor


Uma circunferncia de raio r est inscrita em um setor circular de raio R. O comprimento da corda AB
igual a 2a.

2a
R
A
Figura 105.1

Prove que

1
1
1
= + .
r
R a
(p. 149)

106 | Mais Circunferncias Tangentes


(a) Duas circunferncias de raios R e r so tangentes externamente (figura
106.1). Demonstre que o

segmento determinado pela tangente comum externa mede d = 2 Rr.

Figura 106.1

Figura 106.2

(b) Considere, como ilustrado na 106.2, as trs circunferncias de raios R, r e x, tangentes duas a duas e
tangentes reta . Mostre que

1
1
1
= + .
x
r
R
(p. 150)

107 | Reta Equilibrada


Seja ABC um tringulo tal que AB = 55, AC = 35 e BC = 72. Considere uma reta que corta o lado BC
em D e o lado AC em E e que divide o tringulo em duas figuras com permetros iguais e reas iguais.
Determine a medida do segmento CD.
(p. 151)
www.obmep.org.br

OBMEP

Geometria | Nvel 3 | Enunciados

55

108 | Alturas e Pontos Mdios


O tringulo acutngulo ABC de ortocentro H tal que AB = 48 e HC = 14. O ponto mdio do lado AB
M e o ponto mdio do segmento HC N.

reto.
(a) Mostre que o ngulo MEN
(b) Determine o comprimento do segmento MN.

A
E
M
F
H
N

C
Figura 108.1

(p. 152)

109 | Proibido usar Rgua!


(a) Sejam C uma circunferncia com centro O e raio r e X um ponto exterior a C . Construmos uma
circunferncia de centro em X passando por O, a qual intersecta C nos pontos P e Q. Com centro
em P construmos uma circunferncia passando por O e com centro em Q construmos uma outra
circunferncia passando por O. Estas duas circunferncias intersectam-se nos pontos O e Y .

C
O.

Q
Figura 109.1

Prove que OX OY = r2 .
(b) dado um segmento AB. Mostre como construir, usando somente compasso, um ponto C tal que B
seja o ponto mdio do segmento AC.
(c) dado um segmento AB. Mostre como construir, usando somente compasso, o ponto mdio do
segmento AB.
(p. 153)

110 | Ps das Perpendiculares


Seja ABC um tringulo acutngulo com alturas BD e CE. Os pontos F e G so os ps das perpendiculares
BF e CG a reta DE. Prove que EF = DG.
(p. 154)
www.obmep.org.br

OBMEP

13. Diversos
Nvel 3 Enunciados

111 | Jogo Triangulrio


Um jogo solitrio realizado em um tabuleiro no formato de tringulo equiltero, mostrado na figura
111.1. Sobre cada crculo coloca-se uma ficha. Cada ficha branca de um lado e preta do outro. Inicialmente, s a ficha que est situada em um vrtice tem a face preta para cima e as outras fichas tm a face
branca para cima. Em cada movimento, retira-se uma ficha preta do tabuleiro e cada uma das fichas que
ocupam um crculo vizinho ficha retirada so viradas. Crculos vizinhos so os que esto unidos por
um segmento.

.
Figura 111.1

Aps vrios movimentos, ser possvel tirar todas as fichas do tabuleiro?

(p. 155)

112 | Bolas nas Caixas


Duas caixas contm juntas 65 bolas de vrios tamanhos. Cada bola branca, preta, vermelha ou amarela.
Cada vez que pegamos cinco bolas da mesma cor, pelo menos duas so do mesmo tamanho.
(a) Qual o nmero mximo de tipos de bolas que existem nas caixas? Duas bolas so consideradas de
tipos distintos quando tm diferentes cores ou tamanhos.
(b) Mostrar que existem pelo menos trs bolas, que esto na mesma caixa, e que so do mesmo tipo.
(p. 155)

113 | Fraes Irredutveis


Duas fraes irredutveis tm seus denominadores iguais a 600 e 700. Encontrar o valor mnimo para o
denominador da soma das fraes.
(p. 156)
57

58

Diversos | Nvel 3 | Enunciados

114 | Soma das Quintas Potncias


Seja x1 , x2 , . . . , xn uma sequncia na qual cada termo 0, 1 ou 2. Se

{
x1 + x2 + + xn = 5
x21 + x22 + + x2n = 19
determine x51 + x52 + + x5n .

,
(p. 156)

115 | Comendo Pizzas


Um grupo de meninos e meninas se rene para comer pizzas que so cortadas em 12 pedaos. Cada
menino pode comer 6 ou 7 pedaos e cada menina pode comer 2 ou 3 pedaos. Sabemos que quatro
pizzas nunca so suficientes para alimentar o grupo e que com cinco pizzas sempre h sobra. Quantos
meninos e quantas meninas formam o grupo?
(p. 157)

www.obmep.org.br

OBMEP

14. Desafios
Nvel 3 Enunciados

116 | Quatro Cores no Tabuleiro


Considere o tabuleiro 9 9 mostrado abaixo. As linhas esto numeradas de 1 a 9.
Linha

9
8
7
6
5
4
3
2
1

.
Figura 116.1

Colorimos as casas das linhas mpares do tabuleiro com as cores azul e branco, alternadamente, comeando com azul e pintamos as casas das linhas pares do tabuleiro de cinza e vermelho, alternadamente,
comeando com a cor cinza.
(a) Quantas casas foram pintadas com cada cor?

(b) Qual o nmero mximo de peas da forma


nesse tabuleiro?

que podem ser colocadas, sem sobreposio,

(p. 159)

117 | Nmeros no Tabuleiro 8 x 8


Guilherme escreveu um nmero em cada casa de um tabuleiro 8 8 de modo que a soma dos nmeros
das casas vizinhas de cada casa do tabuleiro igual a 1. Calcule a soma de todos os nmeros escritos
por Guilherme.
Observao: duas casas so vizinhas se possuem um lado em comum.
(p. 160)

118 | Formigas Geomtricas!


Trs formigas esto paradas em trs dos quatro vrtices de um retngulo no plano. As formigas se movem
no plano uma por vez. A cada vez, a formiga que se move o faz segundo a reta paralela determinada
pelas posies das outras duas formigas. possvel que, aps alguns movimentos, as formigas se situem
nos pontos mdios de trs dos quatro lados do retngulo original?
(p. 160)

59

60

Desafios | Nvel 3 | Enunciados

119 | Ponto no Interior do Quadrado


P um ponto no interior do quadrado ABCD tal que PA = 1, PB = 2 e PC = 3. Qual a medida do
?
ngulo APB
A.

D
1
P
3
2

C
Figura 119.1

(p. 161)

120 | Pontos no Interior do Disco


(a) Mostre que no existem dois pontos
com coordenadas inteiras no plano cartesiano que esto igual
mente distanciados do ponto ( 2, 1/3).
(b) Mostre que existe um crculo no plano cartesiano que contm exatamente 2011 pontos com coordenadas inteiras em seu interior.
(p. 162)

www.obmep.org.br

OBMEP

Sugestes e Fatos que Ajudam


1. Mltiplo de 9 com Algarismos Pares. Sugesto: Determine o valor mnimo para a soma dos
algarismos do nmero.
Fatos que Ajudam: A soma dos algarismos de um mltiplo de 9 divisvel por 9.

2. Guardando Cubos. Sugesto: Note que a medida da aresta do cubo deve ser um divisor de cada
uma das trs medidas das dimenses da caixa.

3. Calculadora Quebrada. Sugesto: Determine os possveis valores para o produto e suas fatoraes.
Fatos que Ajudam: 101 primo.

4. Loja em Quixajuba. Sugesto: Mostre inicialmente que ele no pode ter comprado mais de 127
artigos.

5. Nmeros Sortudos. Sugesto: Observe que a partir do nmero 777, todos os nmeros deixam o
mesmo resto na diviso por 1000.
6. Somando Idades. Sugesto: Observe a quantidade de vezes que a idade de uma pessoa foi considerada nas dez somas.

7. Menor Soma Positiva. Sugesto: Se o produto dos nmeros igual a zero, ento um dos nmeros
deve ser igual a zero.

8. Mdia dos Algarismos. Sugesto: Observe o que ocorre com a soma dos algarismos do nmero
quando se faz a operao descrita no problema.
Fatos que Ajudam: A mdia aritmtica de dois nmeros a e b dada por

a+b
.
2
9. Sequncia Numrica I. Sugesto: Analise os restos dos nmeros da sequncia quando so divididos
por 3.
Fatos que Ajudam: Um nmero e a soma de seus algarismos deixam o mesmo resto quando divididos
por 3.

10. Estrelas em Geometrix. Sugesto: Separe as estrelas deixando os nmeros compartilhadas sempre
na estrela direita.

11. Bandeira do Tio Man. Sugesto: Trace as diagonais do retngulo e calcule a rea das quatro
partes determinadas.
Fatos que Ajudam: Tringulos com a mesma base e a mesma altura tm reas iguais.

12. Abelha na Flor. Sugesto: Determine a medida do lado do quadrado.


13. ngulo da Asa Delta. Sugesto: Mostre que os tringulos ABC e ADC so iguais.

62

Sugestes e Fatos que Ajudam

Fatos que Ajudam: A soma dos ngulos internos de um tringulo 180 .

14. Azulejos de Pedro. Sugesto: Perceba que deve haver uma pea em L cobrindo cada canto da
bancada. Alm disso, calcule quantas peas de cada tipo so necessrias para cobrir a rea de cada
bancada.

16. Plantando Jasmins. Sugesto: Trace um segmento de reta ligando os pontos mdios relatados no
problema.
Fatos que Ajudam: Traando uma diagonal de um retngulo, este fica dividido em dois tringulos de
mesma rea.

17. Tangram. Sugesto: Determine a que frao da rea do tangram corresponde cada uma das peas.
18. Tringulo Issceles I. Sugesto: Considere trs casos dependendo de quais dos lados do tringulo
BDE so iguais.
Fatos que Ajudam: A soma dos ngulos internos de um tringulo 180 . Tringulo issceles aquele
que tem dois lados iguais e, portanto, tambm tem dois ngulos internos iguais.

19. Formando um Retngulo. Sugesto: Divida o retngulo maior em quadrados.


20. Construindo uma Pipa. Sugesto: Mostre que a rea de cada um dos quatro tringulos igual ao
triplo da rea do retngulo ABCD.
Fatos que Ajudam: Construindo uma diagonal de um retngulo, este fica dividido em dois tringulos de
mesma rea.

24. Moedas e Pesagens. Sugesto: Divida as moedas em trs grupos de 16 moedas.


25. Distribuindo Mas. Sugesto: Para maximizar o nmero de garotos temos de minimizar o nmero
de mas que cada um recebe.

26. Maria e seus Convidados. Sugesto: Determine inicialmente o maior quadrado perfeito que a
soma de dois nmeros dentre os citados.

27. Cartes de Apostas. Sugesto: Comece comparando os cartes de A e de B.


28. Nmeros de 1 a 16. Sugesto: Encontre todos os possveis vizinhos do nmero 16.
29. Calculando Somas. Sugesto: Observe que as duas regies formadas so iguais. No item (c), conte
as casas de cada pea por linha.

31. Vizinhos e Distantes. Sugesto: Analise os possveis vizinhos do nmero 50 e do nmero 51.
33. Campeonato de Quixajuba. Sugesto: O nmero mximo de pontos no campeonato trs vezes
a quantidade de jogos. A cada empate, este nmero diminui em uma unidade.

35. Somando Algarismos. Sugesto: Observe que todos os algarismos no podem ser menores que 8.
36. Contando Quadrados. Sugesto: Verifique que existem quadrados inclinados, de dois tamanhos
diferentes.

38. O Tabuleiro Mutilado. Sugesto: Cada pea do domin sempre cobre uma casa preta e uma casa
branca.

39. Dividindo um Retngulo. Sugesto: Analise a possibilidade de se obter 39 e 27 como soma de


vrias parcelas 5 e 11.
40. Nmeros no Tabuleiro 4 x 4. Sugesto: Comece preenchendo o tabuleiro pelas casas vizinhas a
um canto.
www.obmep.org.br

OBMEP

63

Desafios | Nvel 2 | Enunciados

41. Mltiplo de 36. Fatos que Ajudam: A soma dos algarismos de um mltiplo de 9 divisvel por 9.
42. Quem maior?. Sugesto: Observe que cada parcela de S da forma

n (n + 10)
e cada parcela de R da forma

(n + 2) (n + 8).
Fatos que Ajudam:

(a + b) (c + d) =
ac + ad + bc + bd.
43. Resto da Diviso. Sugesto: No item (b), analise os nmeros que possuem a soma dos algarismos
maior ou igual a 17.
44. Soma de Consecutivos. Sugesto: Para quatro nmeros consecutivos use a notao x, x + 1, x + 2,
x + 3.
Fatos que Ajudam: (a) O nico nmero primo par 2. (b) O nico nmero primo mltiplo de 3 3.

45. Quadrado Perfeito. Sugesto: Mostre que a expresso considerada igual a

(ab + 1)2 .
Fatos que Ajudam:

(x + y)2 = x2 + 2xy + y2
46. Quantas Fraes!. Sugesto: Elimine as milhares de fraes, fazendo

A=

3+
4+

1
..

.+

1
1991

47. Primos No!. Sugesto: Tente fatorar os nmeros dados:


(a) Escrevendo o nmero dado como uma diferena de dois quadrados.
(b) Escrevendo o nmero dado como uma soma de dois cubos.
Fatos que Ajudam: Utilize as identidades:
(a) m2 n2 = (m n)(m + n)
(b) m3 + n3 = (m + n)(m2 mn + n2 )

48. Trilegais. Sugesto: Estude a quantidade de nmeros pares e mpares em um dos subconjuntos
com trs elementos.
Fatos que Ajudam: A soma de dois nmeros pares ou mpares resulta num nmero par. A soma de um
nmero par com um nmero mpar resulta num nmero mpar.

49. Diferena de Quadrados. Fatos que Ajudam: A diferena entre os quadrados de dois nmeros
igual ao produto da soma destes nmeros pela diferena dos mesmos nmeros. Algebricamente:

m2 n2 = (m + n)(m n).
www.obmep.org.br

OBMEP

64

Sugestes e Fatos que Ajudam

50. Outra de Joozinho. Sugesto: Verifique que a sequncia que fica no quadro depois de todo o
processo peridica.
Fatos que Ajudam: Um nmero e a soma de seus algarismos deixam o mesmo resto quando so divididos
por 9.

51. Colar de Ouro. Sugesto: Calcule o ngulo interno do polgono determinado pelo colar.
Fatos que Ajudam: A medida do ngulo interno de um polgono regular de n lados dada pela frmula
180 (n2)
.
n

52. AP x BN. Sugesto: Calcule a rea do tringulo APB de dois modos distintos.
Fatos que Ajudam: A rea de um tringulo igual a metade do produto da medida da base pela medida
da altura relativa essa base.

53. Dois Quadrados. Sugesto: Trace a diagonal AC.


Fatos que Ajudam: Tringulos com mesma base e mesma altura possuem reas iguais.

54. O Tesouro do Pirata. Sugesto: Mostre que a posio T do tesouro no depende do ponto inicial
A.
Fatos que Ajudam: Em todo quadriltero, os pontos mdios dos lados so vrtices de um paralelogramo.

55. Bissetrizes. Sugesto: Mostre que CAF e BAE so tringulos issceles.


Fatos que Ajudam: A bissetriz de um ngulo o divide em dois ngulos de mesma medida.

56. ngulos e ngulos!. Sugesto: Mostre que o tringulo BEC issceles.


Fatos que Ajudam: A soma das medidas dos ngulos internos de um tringulo igual a 180 .

57. Quadrado, Pentgono e Icosgono. Sugesto: Para o item (b), determine a medida do ngulo
.
CDX
Fatos que Ajudam: A medida do ngulo interno de um polgono regular de n lados dada pela frmula
180 (n2)
.
n

58. Enegono Regular. Sugesto: No item (b), prolongue os lados AB e ED, determinando o ponto de
interseo X.
Fatos que Ajudam: A soma das medidas dos ngulos de um polgono de n lados dada pela frmula

(n2)
180 (n2). A medida do ngulo interno de um polgono regular de n lados dada pela frmula 180 n
.

59. Hexgono Equiangular. Sugesto: Prolongue os lados do hexgono.


Fatos que Ajudam: A soma dos ngulos internos de um polgono com n lados igual a 180 (n 2).

60. Pentgono Equiltero. Sugesto: Suponha que o pentgono j foi construdo; comece investigando
pelo ngulo cuja medida 60 .
Fatos que Ajudam: Se um quadriltero possui os quatro lados de mesma medida, ento ele um losango.
Em um losango, os ngulos opostos possuem a mesma medida.

62. Comparando Sequncias. Sugesto: Selecione uma pessoa que no acertou todos os pontos e
determine o nmero mximo de pontos que ela pode ter acertado.

63. Segmentos e Tringulos. Sugesto: Para o item (a), conte o nmero de cordas que saem de um
determinado ponto.
www.obmep.org.br

OBMEP

65. Placas das Bicicletas. Sugesto: Calcule o nmero inicial de placas que podem ser feitas com
os elementos dos conjuntos A, B e C e depois refaa o clculo analisando as diversas possibilidades de
aumentar em 1 ou 2 os elementos dos conjuntos.
66. Torneio de Tnis. Sugesto: No item (b), considere os jogadores que so eliminados ao invs dos
que passam para as prximas rodadas.

67. Pesando Pedras. Sugesto: Divida as pedras em pares e realize as pesagens, eliminando as pedras
mais leves. Perceba que a segunda pedra mais pesada somente pode ser eliminada pela pedra mais
pesada.

68. Produto 2000. Sugesto: Decomponha 2000 em fatores primos.


69. Tabuleiro 123 x 123. Sugesto: (a) Divida em dois casos de acordo com a cor da casa central. (b)
Determine o nmero de tabuleiros 3 3 que podem ser colocados no tabuleiro 123 123.
70. Nmeros no W. Sugesto: Determine os possveis valores que podem ser colocados na casa vazia
comum s duas linhas.
.

6
9
Figura 70.1

Fatos que Ajudam: A soma dos 9 primeiros nmeros inteiros positivos

1 + 2 + + 9 = 45.
71. Montando Tabelas. Sugesto: Somar i 1 primeira rodada equivale a somar 1 rodada anterior.
72. Numerando os Vrtices. Sugesto: Calcule as somas dos nmeros de todas as faces do paraleleppedo e observe quantas vezes cada vrtice est sendo contado nessa soma.
Fatos que Ajudam:

1 + 2 + + 10 = 55.
73. Corrida de So Paulo a Fortaleza. Sugesto: Observe que se dois carros trocam de posio duas
vezes, a ordem entre eles continua a mesma.

77. Amigos que voc pode Contar!. Sugesto: Mostre que a situao do item (a) possvel e a do
item (b) no.

78. Trs Amigos e uma Bicicleta. Sugesto: Perceba que para chegarem em at 2 h 40 min, cada um
deve fazer pelo menos metade do percurso de bicicleta.

79. Contando Polgonos. Sugesto: Construa um polgono do tipo 2 a partir de um polgono do tipo
1.
80. Desafiando os Amigos!. Sugesto:
(a) Suponha

a b c d e.
O que podemos dizer sobre a + b? E sobre d + e? E sobre a + c?
(b) Carlos no conseguir alcanar seu objetivo porque existem dois conjuntos formados por quatro
nmeros que geram os nmeros 10, 20, 22, 24, 26 e 36.

Sugestes e Fatos que Ajudam

66

81. Sequncia Numrica II. Sugesto: Calcule os primeiros cinco termos da sequncia.
82. Progresso Geomtrica. Sugesto: A razo da progresso geomtrica tem que ser menor que 2.
83. Funciona?. Sugesto: Faa a =

2n + 1 e b = 2n 1.

Fatos que Ajudam: Utilize a identidade

(a2 + ab + b2 )(a b) = a3 b3 .
84. Sistema de Trs Equaes. Sugesto: Subtraia as equaes dadas e fatore o resultado. Depois,
faa o mesmo com a primeira e a terceira equaes.
Fatos que Ajudam: Diferena de dois cubos:

x3 y3 = (x y)(x2 + xy + y2 ).
A soma das razes da equao ax3 + bx2 + cx + d = 0 igual a b/a.

85. Soma de Potncias. Sugesto: Expanda

(a + b)(an + bn ).
87. Sistema com 7 Variveis. Sugesto: (a) Expanda os termos e os agrupe como o polinmio na
varivel n. (b) Utilize os valores encontrados em (a).
Fatos que Ajudam: Se um polinmio se anula para infinitos valores, ento todos os seus coeficientes so
nulos.

88. Algarismo do Quadrado. Sugesto: Escreva o nmero como 10a + b, sendo b um algarismo.
89. Maior Divisor mpar. Sugesto: Sendo Sn a soma de tais divisores, calcule a diferena Sn Sn1 .
Fatos que Ajudam: A soma dos n primeiros nmeros mpares

1 + 3 + + (2n 1) = n2 .
90. Algarismos. Sugesto: Mostre que o denominador sempre divisvel por 11 e que a + c = 11.
Fatos que Ajudam: abc = 100a + 10b + c, ab = 10a + b.

91. Produto Par. Sugesto: Calcule a probabilidade do produto ser mpar.


93. Formiga Aleatria. Sugesto: Observe que a formiga sempre est no 1 nos segundos mpares.
94. Algarismos e Paridade. Sugesto: Conte os nmeros pares e os nmeros mpares separadamente.
95. Bolas Pretas, Brancas e Azuis. Sugesto: Considere n o nmero de bolas azuis da urna e determine as probabilidades de as duas bolas retiradas serem ambas pretas, ambas brancas e ambas azuis.
Fatos que Ajudam: A probabilidade que acontea um dentre trs eventos independentes a soma das
probabilidades que cada um acontea.

96. Aparando um Poliedro. Sugesto: Determine a relao entre as arestas do antigo poliedro e os
vrtices do novo.

97. Bolas Azuis (e )Vermelhas. Fatos que Ajudam: O nmero de modos de escolher dois dentre n
n(n1)
. Veja Contando Subconjuntos na pgina 118.
objetos distintos n
2
2 =
www.obmep.org.br

OBMEP

67

Desafios | Nvel 3 | Enunciados

98. Dez Pontos no Plano. Fatos que Ajudam: O nmero de maneiras de escolher k objetos distintos
dentre n objetos distintos
( )
n
n(n 1) . . . (n k + 1)
.
=
k!
k
Veja o quadro na pgina 118.

99. Contando Diagonais no Poliedro. Sugesto: Conte o nmero total de segmentos determinados
pelos vrtices e retire os que no so diagonais espaciais.
Fatos que Ajudam: O nmero de modos de escolher dois objetos dentre n objetos distintos
n(n1)
.
2

(n)
2

Veja o quadro na pgina 118.

100. Grade de Pontos. Sugesto: Conte o nmero total de segmentos e conte o total de segmentos
que partem de pontos vermelhos.
Fatos que Ajudam: De pontos vermelhos no saem segmentos azuis.

101. Tringulo 20 40 120. Sugesto: Determine as medidas dos ngulos que aparecem na construo.

102. Um Problema Antigo!. Sugesto: Utilize o teorema de Pitgoras.


103. Circunferncias Tangentes. Sugesto: Trabalhe os ngulos dos tringulos issceles AO1 C e
BO2 C.
Fatos que Ajudam: Dadas duas circunferncias tangentes, o ponto de tangncia e os dois centros pertencem a uma mesma reta.

e ACP
somam 180 .
104. Tringulo Issceles II. Sugesto: Mostre que os ngulos AQP
Fatos que Ajudam: Um quadriltero inscritvel se a soma dos ngulos opostos 180 . ngulos inscritos
no mesmo arco so iguais.

105. Circunferncia no Setor. Sugesto: Ligue o centro da circunferncia inscrita no setor ao ponto
de tangncia desta com o raio do setor circular. Procure tringulos semelhantes.
Fatos que Ajudam: Se duas circunferncias so tangentes, ento o ponto de tangncia e os centros das
circunferncias so colineares.
Se uma reta tangente a uma circunferncia, ento o segmento que une o centro da circunferncia ao
ponto de tangncia perpendicular reta.

106. Mais Circunferncias Tangentes. Sugesto: (a) Trace uma reta pelo centro da menor circunferncia, paralela reta .
Fatos que Ajudam: Se duas circunferncias so tangentes, ento o ponto de tangncia e os centros das
circunferncias so colineares.
Se uma reta tangente a uma circunferncia, ento o segmento que une o centro da circunferncia ao
ponto de tangncia perpendicular reta.

107. Reta Equilibrada. Sugesto: Calcule a rea do CED, a qual metade da rea do ABC.
Fatos que Ajudam: A rea S de um tringulo que possui dois lados de medidas a e b e estes determinam
um ngulo pode ser calculada pela frmula

S=

ab sen
.
2

Demonstrao: A rea do tringulo da figura 107.1 ah/2, mas h = b sen .


www.obmep.org.br

OBMEP

Sugestes e Fatos que Ajudam

68

b
.

a
Figura 107.1

Ento,

ah
ab sen
=
.
2
2
108. Alturas e Pontos Mdios. Sugesto: Mostre que os tringulos BME e HEN so issceles.
Fatos que Ajudam: O ortocentro de um tringulo o ponto de interseco das alturas. Em um tringulo
retngulo, a mediana relativa a hipotenusa tem comprimento igual a metade da hipotenusa.

B
M

A
Figura 108.1

109. Proibido usar Rgua!. Sugesto: (a) Mostre que os tringulos XOP e PYO so semelhantes. (b)
Tente obter o ponto C construindo tringulos equilteros. (c) Utilize os itens (a) e (b).
Fatos que Ajudam: Dados dois pontos D e E, podemos construir um ponto F, utilizando somente compasso, tal que o DEF seja equiltero. O ponto F pode ser obtido como um dos dois pontos de interseo
da circunferncia de centro em D que contm E e da circunferncia de centro em E que contm D.

Figura 109.1

110. Ps das Perpendiculares. Sugesto: Mostre que os tringulos BEF e BCD so semelhantes.

= 90 .
Fatos que Ajudam: Sejam X, B e C pontos no plano tais que BXC
X
.

Figura 110.1

Ento o ponto X est sobre a circunferncia de dimetro BC.

X
B

Figura 110.2

www.obmep.org.br

OBMEP

69

Desafios | Nvel 3 | Enunciados

= CBY
, porque estes ngulos medem a metade do
Se Y outro ponto qualquer do arco XC, ento CXY
arco YC.
X
B

Y
.

Figura 110.3

111. Jogo Triangulrio. Sugesto: Observe que para uma ficha poder ser retirada ela teve que ser
virada um nmero mpar de vezes, e todos os crculos tm um nmero par de vizinhos.
112. Bolas nas Caixas. Sugesto: Existem no mximo 4 tamanhos distintos de bolas para cada cor.
113. Fraes Irredutveis. Sugesto: Sendo a/600 e b/700 as duas fraes, verifique quais fatores o
numerador e o denominador da soma podem ter em comum.
Fatos que Ajudam: Uma frao dita irredutvel se o numerador e o denominador no possuem fatores
primos em comum.

114. Soma das Quintas Potncias. Sugesto: Observe que os valores particulares de x1 , x2 , . . . , xn
no so importantes e sim a quantidade destes que so iguais a 1 e 2.
115. Comendo Pizzas. Sugesto: Analise a quantidade mnima e mxima de pedaos que o grupo
pode comer.

116. Quatro Cores no Tabuleiro. Sugesto: Para o item (b), verifique quantas casas de cada cor so
cobertas ao colocar uma pea no tabuleiro.
117. Nmeros no Tabuleiro 8 x 8. Sugesto: Veja o problema Nmeros no Tabuleiro 4 4, do nvel
1, na pgina 100.

118. Formigas Geomtricas!. Sugesto: Analise a rea do tringulo determinado pelas posies das
formigas.
Fatos que Ajudam: A rea de um tringulo no muda quando um dos vrtices se movimenta sobre uma
reta paralela reta formada pelos outros dois vrtices.

119. Ponto no Interior do Quadrado. Sugesto: Determine um ponto Q exterior ao quadrado, tal
que o tringulo APB seja congruente ao tringulo CQB.
Fatos que Ajudam: Se a, b e c so as medidas dos lados de um tringulo e a2 = b2 + c2 , ento o ngulo
oposto ao lado de medida a reto.

120. Pontos no Interior do Disco.Sugesto: Para o item (b), ordene os pontos de coordenadas inteiras
em ordem crescente de distncia a ( 2, 1/3).
Fatos que Ajudam: A distncia entre os pontos (x1 , y1 ) e (x2 , y2 ) dada pela expresso

(x1 x2 )2 + (y1 y2 )2 .
O produto de um nmero racional no nulo por um nmero irracional um nmero irracional.

www.obmep.org.br

OBMEP

Solues

Nvel 1

15. Aritmtica
Nvel 1 Solues

1 | Mltiplo de 9 com Algarismos Pares


Encontre o menor mltiplo de 9 que no possui algarismos mpares.

Soluo: Como o nmero divisvel por 9, a soma dos algarismos


divisvel por 9.
Por outro lado, como todos os algarismos so pares, a soma dos algarismos tambm par. Assim, a soma dos algarismos no mnimo
18. O menor mltiplo de 9 com a soma dos algarismos igual a 18
99, mas seus algarismos so mpares. Isto implica que o nmero deve
ter trs ou mais algarismos.

Sugesto: Determine o valor mnimo para a soma dos algarismos


do nmero.

Fatos que Ajudam: A soma dos algarismos de um mltiplo de 9 divisvel por 9.

Se queremos o menor nmero com 3 algarismos, o primeiro algarismo


deve ser no mnimo 2. Neste caso, a soma dos outros dois algarismos
igual a 16 e como so pares, a nica possibilidade 288.
Portanto, 288 = 9 32 o menor mltiplo de 9 com todos os algarismos pares.

2 | Guardando Cubos
Uma caixa possui o formato de um bloco retangular de dimenses
102 cm, 255 cm e 170 cm. Queremos guardar nessa caixa a menor
quantidade possvel de pequenos cubos de aresta inteira, de forma a
ocupar toda a caixa.
(a) Qual a medida da aresta de cada bloco?
(b) Quantos blocos sero necessrios?

Soluo:
(a) Como a quantidade de blocos a menor possvel, a aresta do
mesmo deve ser a maior possvel. A medida da aresta deve ser
um divisor de 102, 255 e 170. Como queremos a maior aresta possvel, a medida dela deve ser igual ao mdc(102, 255, 170) = 17.
Logo, a aresta do cubo mede 17 cm.
(b) O nmero de blocos

102 255 170


= 6 15 10 = 900.
17 17 17

75

Sugesto: Note que a medida da


aresta do cubo deve
. ser um divisor
de cada uma das trs medidas das
dimenses da caixa.

76

Aritmtica | Nvel 1 | Solues

3 | Calculadora Quebrada
Sugesto: Determine os possveis
valores para o produto e suas fa.
toraes.
Fatos que Ajudam: 101 primo.

Tio Man tem uma calculadora quebrada que no tem a tecla 0 e


no visor nunca aparece 0 depois de alguma operao. Assim, por
exemplo, se ele multiplica 3 67, obtm como resposta 21, ao invs
de 201.
Tio Man multiplicou dois nmeros de dois algarismos em sua calculadora e obteve no visor o nmero 11. Quais so os possveis nmeros
que ele multiplicou?

Soluo: Como a calculadora no possui a tecla O, o produto de


dois nmeros de dois algarismos nesta calculadora maior ou igual
a 1111 = 121 e menor que 100100 = 10000, as possveis respostas
para o produto so: 1001, 1010 e 1100. Para cada um dos casos temos:

1001 = 11 91 = 13 77, duas possveis solues;


1010 = 10110 e como 101 primo, no temos soluo neste caso;
1100 = 11 22 52 = 25 44 a nica soluo j que nenhum
dos dois fatores pode ser divisvel simultaneamente por 2 e 5.
Portanto, os possveis produtos efetuados por Tio Man so 11 91
ou 13 77 ou 25 44.

4 | Loja em Quixajuba
Sugesto: Mostre inicialmente que
.
ele no pode ter comprado
mais de
127 artigos.

Uma loja em Quixajuba s vende artigos com preos de R$ 0, 99,


R$ 1, 99, R$ 2, 99, e assim sucessivamente. Tio Man realizou uma
compra no valor total de R$ 125, 74. Quantos artigos ele pode ter
comprado?

Soluo: Inicialmente observe que 125,74


0,99 < 128, portanto Tio Man
comprou no mximo 127 artigos. Como a compra efetuada custa 26
centavos abaixo de um valor inteiro, ele comprou ou 26 artigos, ou 126
artigos, ou 226 artigos, etc. Porm, como s adquiriu no mximo 127
artigos, ento ele pode ter comprado 26 ou 126, que so quantidades
possveis de se comprar. Veja os exemplos:

26 artigos: 25 artigos de R$ 0, 99 e um no valor de R$ 100, 99.


126 artigos: 125 artigos de R$ 0, 99 e um no valor de R$ 1, 99.

www.obmep.org.br

OBMEP

Aritmtica | Nvel 1 | Solues

77

5 | Nmeros Sortudos
Dizemos que um nmero natural sortudo se todos os seus dgitos
so iguais a 7. Por exemplo, 7 e 7777 so sortudos, mas 767 no
. Joo escreveu num papel os vinte primeiros nmeros sortudos
comeando pelo 7, e depois somou-os. Qual o resto da diviso dessa
soma por 1000?

Sugesto: Observe que a partir


do nmero 777, todos
os nmeros
.
deixam o mesmo resto na diviso
por 1000.

Soluo: Observemos que se um nmero sortudo tem mais de 3


algarismos, o resto da diviso por 1000 777.
Assim, o resto que estamos procurando o mesmo resto da diviso
de

7 + 77 + 777
+ + 777}
| + 777{z
18 vezes

por 1000. Mas este nmero

84 + 18 777 = 84 + 13986 = 14070.


Assim, o resto 70.

6 | Somando Idades
Cada pessoa de um grupo de dez pessoas calcula a soma das idades
das outras nove integrantes do grupo. As dez somas obtidas foram
82, 83, 84, 85, 87, 89, 90, 90, 91 e 92.
Determine a idade da pessoa mais jovem.

Sugesto: Observe a quantidade


.
de vezes que a idade
de uma pessoa foi considerada nas dez somas.

Soluo: Observe que a idade de cada pessoa aparece como parcela


em 9 dos 10 nmeros. Assim, se somarmos os 10 nmeros obteremos
nove vezes a soma de todas as idades. Portanto, a soma das idades
das dez pessoas

82 + 83 + 84 + 85 + 87 + 89 + 90 + 90 + 91 + 92
873
=
= 97.
9
9
A pessoa mais jovem obteve a maior soma, que corresponde soma
das idades dos nove mais velhos, portanto sua idade 97 92 = 5
anos.

7 | Menor Soma Positiva


O produto de 50 nmeros inteiros consecutivos zero e a soma desses nmeros positiva. Qual o menor valor que pode assumir essa
soma?

Sugesto: Se o produto dos nmeros igual a zero, .ento um dos nmeros deve ser igual a zero.

Soluo: Como o produto igual a zero, um dos nmeros tem de ser


zero. Assim, para minimizar a soma devemos ter a maior quantidade
de nmeros negativos mas de forma que a soma ainda seja positiva.
Assim, a quantidade de nmeros negativos deve ser menor que a
quantidade de nmeros positivos. Logo, entre os 49 nmeros no
nulos 24 so negativos e 25 so positivos. Portanto, a soma mnima

24 + (23) + (22) + + (1) + 0 + 1 + + 25 =


25 + (24 + 24) + (23 + 23) + + (1 + 1) + 0 = 25.

www.obmep.org.br

OBMEP

78

Aritmtica | Nvel 1 | Solues

8 | Mdia dos Algarismos


Sugesto: Observe o que ocorre
com a soma dos algarismos do nmero quando se faz a operao
descrita no problema.

. A mdia aritFatos que Ajudam:


mtica de dois nmeros a e b
dada por

a+b
.
2

Paulinho escreveu um nmero no quadro e depois inventou a seguinte brincadeira: escolhe dois algarismos do nmero que sejam
ambos pares ou ambos mpares e troca cada um deles pela sua mdia aritmtica. Ele repete este processo quantas vezes quiser, desde
que o nmero disponha de dois algarismos com a mesma paridade.
Por exemplo, ele escreveu o nmero 1368 e obteve a sequncia na
qual foram destacados os algarismos que sero trocados no passo
seguinte.

1. 3 6

1. 3 7 7

4. 3 4 7

4. 5 4 5
(a) Com esta brincadeira, possvel obter o nmero 434434 a partir
do nmero 324561?
(b) Paulinho escreveu o nmero 123456789 no quadro. Mostrar que
com este processo, selecionando os nmeros adequadamente, ele
pode obter um nmero maior que 800000000.
1. 2 3 4 5 6 7 8 9

Soluo:

2. 2 2 4 5 6 7 8 9
3. 2 2 3 5 6 7 8 9
4. 2 2 3 4 6 7 8 9
5. 2 2 3 4 5 7 8 9
6. 2 2 3 4 5 6 8 9

(a) Observemos que com este processo a soma dos algarismos do


nmero no muda. Como a soma dos algarismos de 324561 21
e a soma dos algarismos de 434434 22, segue que impossvel
obter 434434 a partir de 324561.
(b) Apresentamos uma sequncia de passos que gera, a partir do nmero 123456789, um nmero maior que 800000000.

7. 2 2 3 4 5 6 7 9
8. 2 2 3 4 5 6 7 8

9 | Sequncia Numrica I
Sugesto: Analise os restos dos
nmeros da sequncia quando so
divididos por 3.

Fatos que Ajudam: Um nmero e


a soma de seus algarismos deixam
o mesmo resto quando divididos
por 3.

Todo termo de uma sequncia, a partir do segundo, igual soma


do anterior com a soma de seus algarismos. Os primeiros elementos
da sequncia so

1, 2, 4, 8, 16, 23, 28, 38, 49, . . .


possvel que 793210041 pertena a essa sequncia?

Soluo: Sabemos que um nmero e a soma de seus algarismos deixam o mesmo resto quando divididos por 3. Em cada caso, se o nmero deixa resto 1 na diviso por 3, ento o nmero mais a soma de
seus algarismos deixa resto 2 na diviso por 3, e se o nmero deixa
resto dois, ento a soma dele com a soma de seus algarismos deixa
resto 1 porque 2 + 2 = 4 deixa resto 1.
Calculando os restos da sequncia quando dividimos por 3, obtemos
uma nova sequncia

1, 2, 1, 2, 1, . . . ,
isto , uma sequncia peridica onde aparecem unicamente os restos
1 e 2. Como o nmero 793210041 divisvel por 3, ento ele no
pertence sequncia.

www.obmep.org.br

OBMEP

Aritmtica | Nvel 1 | Solues

79

10 | Estrelas em Geometrix
Estrelix, um habitante de Geometrix, decidiu colocar os inteiros positivos seguindo a disposio indicada na figura.

5
1.

4
3

16
8

7
6

12
11 14

10 13
9

27

15 19 23
18

Sugesto: Separe as estrelas dei. compartilhadas


xando os nmeros
sempre na estrela direita.

22 25

17 21 24
20

26 30 34
29

33

28 32 35
31

Figura 10.1

Em quais estrelas aparece o nmero 2011? Posicione todos os nmeros que aparecem nas referidas estrelas.

Soluo: Consideremos que cada estrela tem em sua composio


11 nmeros e outros dois nmeros, que sero contados na estrela
seguinte, conforme a figura 10.2. Dividindo 2011 por 11, obtemos
quociente 182 e resto 9. Assim, o nmero 2011 o nono nmero da
183a estrela, que est representada na figura 10.3.
.

...

...

nmeros compartilhados

Figura 10.2

2007

2003

2006
2009
.

2005
2004

2010

2008

2014

2013

2012

2015

2011
Figura 10.3

www.obmep.org.br

OBMEP

16. Geometria
Nvel 1 Solues

11 | Bandeira do Tio Man


O Tio Man torcedor doente do Coco da Selva Futebol Clube e resolveu fazer uma bandeira para apoiar seu time no jogo contra o Desportivo Quixajuba. Para isso, comprou um tecido branco retangular
com 100 cm de largura e 60 cm de altura. Dividiu dois de seus lados em 5 partes iguais e os outros dois em 3 partes iguais, marcou
o centro do retngulo e pintou o tecido da forma indicada na figura
11.1.

Sugesto: Trace as diagonais do


retngulo e calcule a rea das quatro partes determinadas.

Fatos que Ajudam: Tringulos


com a mesma base e a mesma altura tm reas iguais.

.
Figura 11.1

Qual a rea do tecido que Tio Man pintou?

Soluo: As diagonais da Bandeira dividem-na em 4 tringulos de


rea 60 100/4 = 1500 cm2 cada um.
Estas diagonais dividem a Bandeira em dois tipos de tringulo, como
mostrados nas figuras 11.3 e 11.4.
O tringulo do tipo 11.3 est dividido em 5 tringulos de mesma rea
porque possuem mesma base e altura. Assim, a rea pintada no tringulo da figura 11.3 (1500/5) 3 = 900 cm2 .

.
Figura 11.2

O tringulo da figura 11.4 est dividido em 3 tringulos de igual rea.


Logo, a rea pintada nesse tringulo (1500/3) 2 = 1000 cm2 .
Deste modo, a rea total pintada da bandeira

2 (900 + 1000) = 3800 cm2 .

.
Figura 11.3

.
Figura 11.4

81

82

Geometria | Nvel 1 | Solues

12 | Abelha na Flor
Sugesto: Determine
a medida do
.
lado do quadrado.

As flores de Geometrix tm formatos muito interessantes. Algumas delas possuem a forma mostrada na figura 12.1, na qual h seis
quadrados e doze tringulos equilteros.

Figura 12.1

Uma abelha pousou no ponto destacado e andou sobre a borda da flor


no sentido horrio at voltar ao ponto inicial. Sabendo que a regio
cinza tem 24 cm2 de rea, qual a distncia percorrida pela abelha?

Soluo: A rea destacada corresponde soma das reas de seis


quadrados. Portanto, cada quadrado possui 4 cm2 de rea e lado
2 cm.
Os lados dos quadrados e dos tringulos equilteros so todos iguais.
Uma volta completa da abelha em torno da flor corresponde a 24
vezes o lado do quadrado, ou seja, 48 cm.

13 | ngulo da Asa Delta


Sugesto: Mostre que os tringulos ABC e ADC so iguais.

Fatos que Ajudam: A soma dos ngulos internos de um tringulo


180 .

Na figura 13.1, temos dois tringulos, ABC e ADC tais que AB = AD


= 25 , determine a medida do
e CB = CD = CA. Sabendo que CBA

ngulo BCD.

C
B

D
Figura 13.1

Soluo: Observe que os tringulos ABC e ADC so iguais e issceles, pois os trs lados de cada tringulo possuem as mesmas medidas.
Por outro lado,

= BAC
= CAD
= ADC
= 25 .
CBA
Da,
Finalmente

www.obmep.org.br

= DCA
= 180 25 25 = 130 .
BCA
= 360 130 130 = 100 .
BCD

OBMEP

Geometria | Nvel 1 | Solues

83

14 | Azulejos de Pedro
Pedro um pedreiro. Ele tem um grande nmero de azulejos de trs
tipos, como mostrado abaixo:

Sugesto: Perceba que deve haver uma pea em L cobrindo cada


canto da bancada.. Alm disso, calcule quantas peas de cada tipo
so necessrias para cobrir a rea
de cada bancada.

.
Figura 14.1

O menor lado de cada azulejo mede 10 cm. Ele quer ladrilhar completamente uma bancada de uma cozinha sem cortar qualquer azulejo.
(a) Mostre como ele poder alcanar seu objetivo se a bancada for
um retngulo 60 cm 50 cm.
(b) Mostre como ele poder alcanar seu objetivo se a bancada for
um quadrado 60 cm 60 cm.
.
Figura 14.2

Soluo:
(a) A soluo exibida na figura 14.2.
(b) A soluo exibida na figura 14.3.

15 | Retngulo 9 x 4
(a) Divida um retngulo 9 4 em trs peas e remonte-as de modo a
formar um quadrado 6 6.

.
Figura 14.3

(b) Divida um retngulo 9 4 em duas peas e remonte-as de modo


a formar um quadrado 6 6.

Soluo:
3

(a) Dividimos o retngulo 9 4 em dois retngulos 2 3 e um retngulo 4 6 como mostra a figura 15.1 e os reagrupamos como
ilustra a figura 15.2, formando um quadrado 66. Veja as figuras
15.1 a 15.3.
(b) Dividimos o retngulo em duas figuras iguais e em forma de L e
as reagrupamos, como ilustram as figuras 15.3 e 15.4.

2
4
2

.
Figura 15.1

2
4

3
2

3
2

4
2

Figura 15.3

Figura 15.2

3
2

Figura 15.4

Comentrio: A soluo de (b) leva a infinitas solues para (a). Para tal,
basta dividir uma das duas peas de (b) em duas quaisquer, obtendo trs
peas.

www.obmep.org.br

OBMEP

84

Geometria | Nvel 1 | Solues

16 | Plantando Jasmins
Sugesto: Trace um segmento de
reta ligando os pontos mdios relatados no problema.

Fatos que Ajudam: Traando uma


diagonal de um retngulo, este
fica dividido em dois tringulos de
mesma rea.

O jardineiro Jacinto decidiu ajardinar um canteiro retangular com


10 m2 de rea. Dividiu o canteiro traando uma diagonal e unindo
cada um dos pontos mdios dos lados maiores com um vrtice do
lado oposto, como indicado na figura.

.
Figura 16.1

Na regio sombreada plantou jasmins. Qual a rea dessa regio?

B
Soluo: Sejam ABCD o canteiro e X e Y os pontos mdios de AB e
CD, respectivamente, como na figura 16.2. O ponto de interseo da
reta XY e da diagonal AC determina o centro O do retngulo.

Z
O

Como a figura simtrica em relao ao centro O, em particular temos que os tringulos XZO e YWO so iguais.

W
.

Conclumos que a rea do quadriltero XZWB igual rea do tringulo XYB que corresponde a 1/4 da rea do retngulo ABCD, isto
, 2, 5 m2 .

Figura 16.2

17 | Tangram
Sugesto: Determine a que frao
.
da rea do tangram
corresponde
cada uma das peas.

A figura 17.2 um retngulo cuja rea sombreada foi feita utilizando


peas de um tangram que formam um quadrado de 10 cm2 de rea,
mostrado na figura 17.1.

.
Figura 17.1

1
2

Figura 17.2

Qual a rea do retngulo?

.
Figura 17.3

Soluo:
2
2
1

2
1

Figura 17.4

2
2

No tangram temos: dois tringulos maiores de rea 1/4 do quadrado,


isto , 10/4 cm2 ; um tringulo, um quadrado e um paralelogramo de
rea 1/8 do quadrado, isto , 10/8 cm2 e dois tringulos de rea 1/16
do quadrado, isto , 10/16 cm2 .
Na decomposio mostrada na figura 17.4, o retngulo formado possui, alm das peas do tangram, quatro quadrados de rea 10/8 cm2
e seis tringulos de rea 10/16 cm2 , numa rea total de

10
35
10
+6
=
cm2 .
8
16
4

Finalmente, a rea do retngulo

10 +

www.obmep.org.br

75
35
=
= 18, 75 cm2 .
4
4
OBMEP

Geometria | Nvel 1 | Solues

85

18 | Tringulo Issceles I
= 30 e ABC
= 50 . A reta corta
Seja ABC um tringulo com BAC
os lados AB, BC e o prolongamento de AC em D, E e F, respectivamente.

Fatos que Ajudam: A soma dos n.


gulos internos de um tringulo

180 . Tringulo issceles aquele


que tem dois lados iguais e, portanto, tambm tem dois ngulos
internos iguais.

F
C
E
50

Sugesto: Considere trs casos dependendo de quais dos lados do


tringulo BDE so iguais.

30

Figura 18.1

Se o tringulo BDE issceles, quais so as trs possveis medidas


para o ngulo C
FE?

= 180 50 30 = 100 e ECF


= 80 .
Soluo: Sabemos que BCA

Assim, basta calcular a medida do ngulo CEF para depois calcular


a medida do ngulo C
FE. Temos trs possveis casos, dependendo
quais dos trs lados do tringulo BDE so iguais:
(a) Se BD = BE, temos que

= BED
= 180 50 = 65
BDE
2

C
FE = 180 80 65 = 35 .

(b) Se BD = DE, temos que

= DBE
= 50
BED
e

C
FE = 180 80 50 = 50 .

(c) Se DE = BE, temos que

= DBE
= 50 ,
BDE
= 180 50 50 = 80
BED
e

www.obmep.org.br

C
FE = 180 80 80 = 20 .

OBMEP

86

Geometria | Nvel 1 | Solues

19 | Formando um Retngulo
Sugesto:
Divida
o retngulo
.
maior em quadrados.

A partir de seis retngulos iguais e cinco quadrados iguais formado


um retngulo de permetro 324 cm, como mostrado na figura 19.1

.
Figura 19.1

Determine a rea do retngulo construdo.


.
Figura 19.2

Soluo: Do retngulo cinza destacado na figura 19.2, conclumos


que um dos lados do retngulo mede 4 vezes o lado do quadrado.
Assim, o outro lado do retngulo mede 3 vezes o lado do quadrado
(veja a figura 19.3). Segue que podemos dividir o retngulo em quadrados, como indicado na figura 19.4.

.
Figura 19.3

Desta forma, temos que o retngulo fica dividido em 11 7 = 77


quadrados. O permetro deste retngulo 11 + 11 + 7 + 7 = 36 vezes
o lado do quadrado. Portanto o lado do quadrado 324/36 = 9 cm e
a rea do retngulo 11 7 92 = 6237 cm2 .

.
Figura 19.4

www.obmep.org.br

OBMEP

Geometria | Nvel 1 | Solues

87

20 | Construindo uma Pipa


Para construir a pipa de papel representada na figura, Eduardo comeou por pintar um retngulo ABCD numa folha de papel. Em seguida, prolongou cada um dos lados do retngulo triplicando o seu
comprimento e obteve o quadriltero A B C D .

B .

Fatos que Ajudam: Construindo


uma diagonal de um retngulo,
este fica dividido em dois tringulos de mesma rea.

Sugesto: Mostre que a rea de


cada um dos quatro tringulos
igual ao triplo da rea do retngulo
ABCD.

C
Figura 20.1

Sabendo que a rea do retngulo ABCD 200 cm , qual a rea da


pipa construda por Eduardo?

Soluo: Observe que os tringulos AA D e CC B so iguais. De


igual forma os tringulos BB A e DD C so iguais.

Assim, se X e Y so pontos tais que A BB X e A AD Y so retngulos


(figura 20.2), a rea da pipa igual soma das reas destes retngulos
mais a rea do retngulo ABCD e cada um destes retngulos pode
ser dividido em 3 2 = 6 retngulos iguais a ABCD.
Conclumos que a pipa tem rea (6 + 6 + 1) 200 = 2600 cm2 .

www.obmep.org.br

A
B .

C
Figura 20.2

OBMEP

17. Diversos
Nvel 1 Solues

21 | Colorindo Mapas
No mapa da figura 21.1 a curva XY uma das fronteiras. Pases como
I e II tm fronteira comum. O ponto Y no considerado fronteira, ou
seja, pases como I e V no tm fronteira comum. Voc deve colorir
o mapa fazendo pases de fronteira comum terem cores diferentes.
X

II

III

VI

IV
.

Figura 21.1

(a) Qual o nmero mnimo de cores para colorir o mapa? Mostre


como colori-lo.
(b) Desenhe outro mapa de 6 pases, que precise de pelo menos 4 cores para ser pintado. Mostre como colori-lo com cores A, B, C e
D.

II

III

IV

VI

Soluo:
(a) No mnimo so necessrias duas cores, como mostrado na figura
21.2.

.
Figura 21.2

(b) As figuras 21.3 e 21.4 exibem dois mapas com seis pases que
precisam de no mnimo quatro cores para serem pintados.

C
A B A B

22 | De Coco da Selva a Quixajuba


As cidades de Coco da Selva e Quixajuba esto ligadas por uma linha
de nibus. De Coco da Selva saem nibus para Quixajuba de hora em
hora e o primeiro parte meia-noite em ponto. De Quixajuba saem
nibus para Coco da Selva de hora em hora e o primeiro parte meianoite e meia em ponto. A viagem de nibus feita em exatamente
5 horas.
Se um nibus sai de Coco da Selva ao meio-dia, quantos nibus vindo
de Quixajuba ele encontra durante o percurso?

Soluo: Observemos que o nibus que parte de Coco da Selva para


Quixajuba encontra os nibus que, no momento de sua sada, esto
89

Figura 21.3

B
A

D.
B

A
C

Figura 21.4

90

Diversos | Nvel 1 | Solues

no caminho de Quixajuba para Coco da Selva e mais os nibus que


partem nas cinco horas seguintes.
Os nibus que esto na estrada so aqueles que partiram at 5 horas
antes desse nibus, enquanto os nibus que ainda vo partir tm de
faz-lo at 5 horas depois. Assim o nibus se encontrar com todos
aqueles que partiram de Quixajuba entre 7h 30min e 16h 30 min, que
so 10.

23 | O Baralho de Joo
Joo possui um baralho com 52 cartas numeradas de 1 at 52. Um
conjunto de trs cartas chamado sortudo se a soma dos algarismos
em cada carta a mesma. Qual o nmero mnimo de cartas que Joo
tem de pegar do baralho, sem olhar, de tal forma que entre as cartas
que ele pegou necessariamente existam trs cartas que formam um
conjunto de cartas sortudo?

Soluo: Primeiro observemos que a soma dos algarismos das cartas


no mximo 4 + 9 = 13 o que somente acontece com a carta 49. J
para as somas que esto entre 1 e 12, h pelo menos duas cartas que
satisfaam cada soma, assim pegando a carta 49 mais duas cartas
para cada soma entre 1 e 12 , isto , 2 12 + 1 = 25 cartas, ainda no
temos trs cartas que formam um conjunto sortudo.
Agora, se pegamos 26 cartas, no mnimo 25 tm soma de seus algarismos entre 1 e 12. Logo, pelo menos, 3 cartas tm a mesma soma
dos algarismos.

24 | Moedas e Pesagens
Sugesto: Divida. as moedas em
trs grupos de 16 moedas.

Ana possui 48 moedas aparentemente iguais. Porm, exatamente


uma das moedas falsa e tem peso diferente do peso das outras.
Ela possui uma balana eletrnica que mede o peso total de qualquer
quantidade de moedas. Mostre como ela pode determinar a moeda
falsa realizando sete pesagens.

Soluo: Dividimos as 48 moedas em trs grupos de 16 moedas e


realizamos trs pesagens. A moeda falsa estar no grupo de peso
diferente.
Alm disso, j possvel determinar o peso da moeda falsa e das
moedas boas.
Pegamos o grupo de 16 moedas que contm a moeda falsa e dividimos
em dois grupos de 8. Escolhemos um grupo e o pesamos. Como
sabemos qual o peso que devemos obter se a moeda falsa ou boa,
podemos determinar se a moeda est nesse grupo ou no grupo que
no foi pesado.
Pegamos novamente o grupo que contm a moeda falsa, dividimos
em dois grupos com a mesma quantidade de moedas e pesamos um
dos grupos. Realizando mais quatro vezes este processo, at pesar
uma nica moeda, podemos determinar a moeda falsa.
Deste modo, precisamos de trs pesagens iniciais e mais quatro pesagens dividindo os grupos pela metade. Ao todo, precisamos de sete
pesagens.

www.obmep.org.br

OBMEP

Diversos | Nvel 1 | Solues

91

25 | Distribuindo Mas
Noventa e nove mas so distribudas entre alguns garotos de tal
forma que todos recebem quantidades diferentes de mas.
(a) Qual o nmero mximo de garotos que pode haver nesse grupo?

Sugesto: Para maximizar o nmero de garotos temos


de minimi.
zar o nmero de mas que cada
um recebe.

(b) Havendo dez garotos, qual o nmero mximo de mas que recebe o garoto que ganhou menos mas?

Soluo:
(a) Para maximizar o nmero de garotos temos de minimizar o nmero de mas que cada um pode receber. Neste caso, os primeiros nmeros naturais 1, 2, 3, 4, . . . , correspondem s quantidades
de mas que cada garoto dever receber, exceto o ltimo garoto.
Como

1 + 2 + 3 + + 12 + 13 = 91
e

1 + 2 + 3 + + 13 + 14 = 105,
o nmero mximo de garotos 13.
(b) Observemos que

1 + 2 + + 10 = 55
o nmero mnimo de mas que recebem os dez garotos. Para
cada ma que damos ao garoto com menor nmero de mas,
temos de dar uma ma a cada um dos outros para que todos
fiquem com quantidades distintas de mas.
Como 99 55 = 44 podemos dar 4 mas a mais para todos os
garotos. Portanto, o garoto com menos mas pode receber no
mximo 5 mas (Observe que 5+6+ +14 = 95 e 6+7+ +15 =
105).

26 | Maria e seus Convidados


Maria convidou nove garotos e oito garotas para sua festa de aniversrio. Ela preparou camisetas com os nmeros de 1 a 18, ficou com
a de nmero 1 e distribuiu as demais para seus convidados. Durante
uma dana, ela observou que a soma dos nmeros de cada casal era
um quadrado perfeito. Quais pares estavam danando?

Sugesto: Determine inicialmente


o maior quadrado. perfeito que a
soma de dois nmeros dentre os
citados.

Soluo: Observe inicialmente que a maior soma possvel para um


casal 18 + 17 = 35 < 62 , donde obtemos os pares {18, 7}, {17, 8} e
{16, 9}. Consideremos agora dois casos:

O par do 15 o 10.
Segue que o par do 6 o 3 e no h escolha para o par do 1.

O par do 15 o 1.
Segue que o par do 10 o 6, o par do 2 o 14, o par do 3 o 13, o
par do 12 o 4 e o par do 5 o 11. Portanto, existe somente uma
soluo:

{1, 15}, {2, 14}, {3, 13}, {4, 12}, {5, 11}, {6, 10}, {7, 18}, {8, 17}, {9, 16}.

www.obmep.org.br

OBMEP

92

Diversos | Nvel 1 | Solues

27 | Cartes de Apostas
Sugesto: Comece
. comparando os
cartes de A e de B.

Trs apostadores A, B e C preenchem individualmente um carto


de apostas, dos possveis resultados de cinco jogos de futebol (C =
vitria do time da casa, E = empate, V = vitria do visitante). Os
cartes preenchidos foram:

1
2
3
4
5

Apostador A

C
1
2
3
4
5

Apostador B

1
2
3
4
5

Apostador C

Finalizadas as partidas, observou-se que A obteve trs acertos, B obteve trs acertos e C obteve dois acertos. Construa um carto com
cinco acertos.

1
2
3
4
5

Soluo: A e B obtiveram juntos 6 acertos, mas s h 5 jogos, logo


houve pelo menos um jogo em que ambos acertaram. Comparando
seus cartes, apenas no jogo 4 houve respostas iguais. Logo, esse
jogo est certo e dos outros quatro jogos, A acertou 2 e B acertou
outros 2.
Comparando o carto do B com o carto do C, em todos os jogos
suas respostas foram diferentes, ento os 2 acertos de C tambm
so acertos de A. Mas os cartes de A e C unicamente coincidem nos
jogos 1 e 2, que devem ser os resultados corretos dos jogos. Portanto
os jogos 3 e 5 foram acertados por B, obtendo a tabela ao lado.

28 | Nmeros de 1 a 16
Sugesto: Encontre
. todos os possveis vizinhos do nmero 16.

(a) Mostre que os nmeros de 1 a 16 podem ser escritos numa reta,


de tal modo que a soma de quaisquer dois nmeros vizinhos
seja um quadrado perfeito.
(b) Mostre que os nmeros de 1 a 16 no podem ser escritos ao
redor de uma circunferncia, de tal modo que a soma de quaisquer dois nmeros vizinhos seja um quadrado perfeito.

Soluo: A observao-chave que ajuda a resolver (a) e resolve (b)


procurar os possveis vizinhos para o nmero 16.
Um vizinho de 16 um nmero que somado a 16 resulte em um quadrado perfeito. Um candidato o nmero 9, pois 16 + 9 = 52 .
No existem outros, pois o prximo quadrado perfeito aps o 25 o
36 e a maior soma que podemos obter dentre dois nmeros de 1 a 16
15 + 16 = 31.
(a) Como o 16 s tem um vizinho possvel, ele deve ficar numa
extremidade. Comeando com o 16 obtemos a soluo abaixo.

16 9 7 2 14 11 5 4 12 13 3 6 10 15 1 8
(b) Para que fosse possvel colocar todos os nmeros de 1 a 16
ao redor de uma circunferncia, todo nmero deveria ter dois
vizinhos. Mas o nico vizinho possvel para o 16 9, impossibilitando a construo circular.
www.obmep.org.br

OBMEP

Diversos | Nvel 1 | Solues

93

29 | Calculando Somas
Considere um tabuleiro com 11 linhas e 11 colunas.

Sugesto: Observe que as duas regies formadas so


. iguais. No item
(c), conte as casas de cada pea por
linha.

.
Figura 29.1

(a) Quantas casas formam este tabuleiro?


(b) A diagonal cujas casas esto sombreadas separa o tabuleiro em
duas regies: uma acima e outra abaixo. Quantas casas formam
cada regio? possvel calcular esse nmero sem contar casa por
casa?
(c) Com a ajuda do tabuleiro, possvel calcular a soma 1+2+ +10.
Explique como.
(d) Com a ajuda de outro tabuleiro, com o raciocnio semelhante ao
do item anterior, possvel calcular a soma 1 + 2 + + 100. Qual
deve ser a quantidade de linhas e colunas do tabuleiro? Qual o
valor da soma?

Soluo:
(a) Como h 11 casas em cada linha do tabuleiro e este possui 11
linhas, o total de casas 11 11 = 121.
(b) Como h uma casa da diagonal em cada linha do tabuleiro e este
possui 11 linhas, o total de casas da diagonal 11. Por outro
lado, a diagonal um eixo de simetria, separando duas regies
iguais. Existem 11 11 casas no tabuleiro e destas 11 esto na
diagonal. O nmero de casas que formam cada regio ento
(11 11 11)/2 = 55.
(c) Vamos contar o nmero de casas em cada pea por linha (veja
a figura 29.2). A primeira linha contm 1 casa, a segunda 2, a
terceira 3 e assim por diante, at a ltima linha, que contm 10
casas. Portanto, a soma 1 + 2 + + 10 o total de casas de cada
pea, as quais contm 55 casas:

1 + 2 + + 10 =

11 11 11
= 55.
2

.
Figura 29.2

(d) Vamos considerar um tabuleiro com 101 linhas e 101 colunas e


considerar a diagonal que o separa em duas regies iguais. A
diagonal contm 101 casas e cada regio contm (101 101
101)/2 = 5050 casas. Por outro lado, contando o nmero de casas
por linha, obtemos 1 + 2 + + 100. Portanto,

1 + 2 + + 100 = 5050.
www.obmep.org.br

OBMEP

94

Diversos | Nvel 1 | Solues

Problema Relacionado
Considere o tabuleiro com 10 linhas e 10 colunas, da figura 29.3. Ele est dividido em

10

dez peas no formato . coloridas alternadamente de branco e cinza. A primeira pea


formada somente por uma casa.

9
8
7

(a) Quantas casas formam a stima pea? E a dcima pea?

(b) possvel calcular a soma 1 + 3 + + 19 com a ajuda deste tabuleiro. Como?

5
4

(c) Com um raciocnio semelhante a este e com o auxlio de outro tabuleiro possvel
calcular a soma 1 + 3 + 5 + + 99. Quantas linhas e colunas deve ter o tabuleiro?
Qual o valor da soma?

3
2
1

.
Figura 29.3

30 | Herana para Cinco Filhos


Divida a figura 30.1 em cinco partes do mesmo formato e com reas
iguais de tal modo que cada parte contenha exatamente um quadrado
cinza.

.
Figura 30.1

Soluo: A figura 30.2 mostra a soluo do problema.

.
Figura 30.2

www.obmep.org.br

OBMEP

18. Desafios

Nvel 1 Solues

31 | Vizinhos e Distantes
possvel escrever os nmeros naturais de 1 a 100 sobre uma reta
de modo que a diferena entre quaisquer dois nmeros vizinhos seja
maior ou igual a 50?

Sugesto: Analise os possveis vi. 50 e do nmero


zinhos do nmero
51.

Soluo: Observe que o nico vizinho possvel para o 50 o nmero 100 e o nico vizinho possvel para o nmero 51 o nmero 1.
Portanto, 50 e 51 devem aparecer nas extremidades da configurao.
Comeando por 51, obtemos a configurao.

51 1 52 2 53 100 50.
possvel demonstrar que esta configurao e a que contm os nmeros na ordem inversa, so as nicas possveis.
De fato, os nicos possveis vizinhos de 52 so o 1 e o 2, logo os
vizinhos de 1 so 51 e 52. Como 1 no vizinho de 53, ento os
nicos possveis vizinhos de 53 so 2 e 3.

...

51 52 53

.
1

99 100

...
Figura 31.1

49 50

Do mesmo modo descobrimos que os nicos vizinhos possveis de


54 so o 3 e o 4 (pois o 2 e o 1 j tm vizinhos) e continuando esse
processo mostramos que esta a nica sequncia possvel.
Observe que a configurao formada intercalando os nmeros dos
conjuntos {51, 52, . . . , 100} e {1, 2, . . . 50}.

32 | Truque com Cartas


Um mgico com os olhos vendados d 29 cartas numeradas de 1 a
29 para uma mulher da plateia. Ela esconde duas cartas no bolso e
devolve as restantes para a assistente do mgico.
A assistente escolhe duas cartas dentre as 27 e um homem da plateia
l, na ordem que quiser, o nmero destas cartas para o mgico. Aps
isto, o mgico adivinha o nmero das cartas que foram escondidas
pela mulher.
Como o mgico e sua assistente podem combinar uma estratgia para
realizarem esse truque?

Soluo: Existem vrias estratgias possveis. Vamos apresentar


uma.
Separemos em dois casos:

Primeiro Caso: A mulher escolheu duas cartas no consecutivas


(estamos supondo que 29 e 1 so consecutivos). Nesse caso, a assistente escolhe as duas cartas posteriores s escolhidas pela mulher.
95

13

29

14

.
Figura 32.1

96

Desafios | Nvel 1 | Solues

Segundo Caso: A mulher escolheu duas cartas consecutivas. Nesse


caso, a assistente escolhe as duas cartas posteriores maior carta.
No caso em que a mulher escolhe as cartas 29 e 1, a assistente pega
as cartas 2 e 3.
Para realizar o truque, o mgico precisa somente dizer as duas cartas
anteriores em qualquer dos casos.

33 | Campeonato de Quixajuba
Sugesto: O nmero mximo de
pontos no campeonato trs vezes
.
a quantidade de jogos.
A cada empate, este nmero diminui em uma
unidade.

A tabela mostra a classificao final do campeonato de futebol de


Quixajuba. Neste campeonato cada time jogou com cada um dos
outros quatro vezes. Cada time ganha 3 pontos por vitria, 1 por
empate e no ganha pontos em caso de derrota.
Equipe
Bissetriz
Primo
Potncia
MDC

Pontos
22
19
14
12

(a) Quantas partidas foram disputadas no campeonato?


(b) Quantas partidas terminaram empatadas?

Soluo:
(a) Existem 6 possveis confrontos entre os quatro times (Bissetriz
Primo), (Bissetriz Potncia), (Bissetriz MDC), (Primo Potncia), (Primo MDC) e (Potncia MDC). Cada um destes confrontos aconteceu 4 vezes e logo o nmero de partidas igual a
4 6 = 24.
(b) O nmero mximo de pontos do campeonato igual a 3 vezes o
nmero de jogos, isto , 3 24 = 72. Cada vez que acontece um
empate este nmero diminui uma unidade. Como o nmero total
de pontos ao final do campeonato foi 22 + 19 + 14 + 12 = 67, o
nmero de partidas que terminaram empatadas 72 67 = 5.

34 | Tabuleiro 6 x 6
Voc dispe de doze peas em formato de L, como a mostrada na
figura 34.1. Cada figura formada por trs quadrados de lado 1.
Mostre como cobrir um quadrado 6 6 com essas peas, de modo
que nenhum retngulo 2 3 seja formado por exatamente duas de
tais peas.

.
Figura 34.1

.
Figura 34.2

www.obmep.org.br

Soluo: A figura 34.2 exibe uma possvel diviso.

OBMEP

Desafios | Nvel 1 | Solues

97

35 | Somando Algarismos
Quantos nmeros naturais de trs algarismos so tais que a soma
destes igual a 24?

Sugesto: Observe que todos os al.


garismos no podem
ser menores
que 8.

Soluo: Se todos os algarismos forem menores que 8, a soma ser


menor que 3 8 = 24.
Se um deles for igual a 8, a soma dos outros dois ser 16 e temos as
possibilidades: 16 = 8 + 8 = 7 + 9. Obtemos ento sete solues 888,
789, 798, 879, 897, 978 e 987.
Se um dos algarismos for igual a 9, a soma dos outros dois ser 15 e
temos as possibilidades: 15 = 7 + 8 = 6 + 9. A primeira igualdade
leva a solues j encontradas. A outra resulta nos nmeros 699, 969
e 996.
Existem ento dez naturais com a propriedade desejada: 888, 789,
798, 879, 897, 978, 987, 699, 969 e 996.

36 | Contando Quadrados
Doze pontos so marcados sobre uma grade de pontos, como mostrado na figura 36.1.

Sugesto: Verifique que existem


.
quadrados inclinados,
de dois tamanhos diferentes.

.
Figura 36.1

Quantos quadrados podem ser formados ligando quatro desses pontos?

Soluo: No total existem 11 quadrados, como indicado abaixo.

.
Figura 36.2

5 quadrados pequenos, como na figura 36.2.


4 quadrados maiores, como na figura 36.3.
E 2 quadrados maiores ainda, mostrados na figura 36.4.
.

Figura 36.3

.
Figura 36.4

www.obmep.org.br

OBMEP

98

Desafios | Nvel 1 | Solues

37 | A Moeda Falsa
Temos 25 moedas aparentemente iguais, mas sabemos que exatamente uma delas falsa e tem o peso diferente do peso das outras.
No sabemos qual a moeda falsa. Todas as outras 24 moedas possuem o mesmo peso.
Queremos determinar, utilizando uma balana de pratos, se a moeda
falsa mais leve ou mais pesada que as outras.
Como podemos alcanar este objetivo realizando duas pesagens em
uma balana de pratos?

No queremos encontrar a moeda falsa. Queremos saber se ela mais leve


ou mais pesada que as outras.

Nesse tipo de balana podemos comparar os pesos colocados nos dois pratos, ou seja, a balana pode equilibrar ou pender para o lado mais pesado.

Soluo: Separe uma das moedas e coloque as outras 24 na balana,


com 12 em cada prato. Temos duas possibilidades:
(1) A balana equilibra. Neste caso, conclumos que a moeda falsa
a que no est na balana e todas as que esto na balana so
verdadeiras. Basta realizar uma nova pesagem com a moeda falsa
e uma outra moeda qualquer.
(2) A balana no equilibra. Pegamos as 12 moedas do prato mais
leve e colocamos novamente na balana com 6 moedas em cada
prato. Temos novamente dois casos.
(a) Se a balana equilibrar, ento todas as 12 moedas so verdadeiras e podemos concluir que a moeda falsa era uma das
outras 12 do grupo mais pesado. Portanto, neste caso, a moeda falsa mais pesada.
(b) Se a balana no equilibrar, a moeda falsa uma destas 12
moedas e como este grupo mais leve que o outro, conclumos que a moeda falsa mais leve.

38 | O Tabuleiro Mutilado
Sugesto: Cada pea do domin
. casa preta e
sempre cobre uma
uma casa branca.

A figura abaixo mostra um tabuleiro 8 8 no qual duas casas foram


retiradas (a do canto inferior direito e a do canto superior esquerdo).
possvel cobrir este tabuleiro com 31 domins 2 1? Cada domin
pode ser colocado na horizontal ou na vertical cobrindo exatamente
duas casas.

Figura 38.1

www.obmep.org.br

OBMEP

Desafios | Nvel 1 | Solues

99

Soluo: Cada vez que colocamos uma pea de domin no tabuleiro,


cobrimos uma casa branca e uma casa preta. Deste modo, o nmero
de casas pretas cobertas igual ao nmero de casas brancas cobertas.
Como nosso tabuleiro tem 30 casas pretas e 32 casas brancas, no
possvel colocarmos 31 domins.

39 | Dividindo um Retngulo
(a) possvel dividir um retngulo 39 55 em retngulos 5 11?
(b) possvel dividir um retngulo 55 27 em retngulos 5 11?

Sugesto: Analise a possibilidade


. como soma de
de se obter 39 e 27
vrias parcelas 5 e 11.

Soluo:
(a) Suponha que seja possvel fazer tal diviso. O lado de medida
39 ser ento escrito como soma de mltiplos de 5 e 11. claro
que sero utilizadas no mximo 3 parcelas 11. Vamos analisar as
possibilidades:
(1) No possvel usar somente mltiplos de 5 porque 39 no
divisvel por 5.
(2) No possvel usar um 11 porque 39 11 = 28 no divisvel
por 5.
(3) No possvel usar duas parcelas 11 porque 39 2 11 = 17
no divisvel por 5.
(4) No possvel usar trs parcelas 11 porque 39 3 11 = 6
no divisvel por 5.
Logo, no possvel dividir um retngulo 39 55 em retngulos

5 11.

(b) J no caso do retngulo 55 27 podemos escrever

27 = 5 + 11 + 11.
Como o lado de medida 55 pode ser coberto tanto por 5 lados
de medida 11 quanto por 11 lados de medida 5, basta repetir a
posio dos retngulos usados na cobertura do lado de medida
27 at completar o retngulo, conforme a figura 39.1

www.obmep.org.br

11

11
5

.
5 11
Figura 39.1

OBMEP

100

Desafios | Nvel 1 | Solues

40 | Nmeros no Tabuleiro 4 x 4
Sugesto: Comece preenchendo o
.
tabuleiro pelas casas
vizinhas a
um canto.

Guilherme escreveu 0 ou 1 em cada casa de um tabuleiro 4 4. Ele


colocou os nmeros de modo que a soma dos nmeros das casas
vizinhas de cada casa do tabuleiro fosse igual a 1.
Por exemplo, na figura 40.1, considerando a casa marcada com , a
soma dos nmeros das casas sombreadas igual a 1.
.

.
Figura 40.1

Determine a soma de todos os 16 nmeros do tabuleiro.


.
Figura 40.2

1
0

Comeando do canto superior esquerdo, podemos supor sem perda


de generalidade que preenchemos o tabuleiro como na figura 40.2.

0
.
Figura 40.3

1
0

0
0

Portanto, a soma dos nmeros colocados no tabuleiro sempre igual


a 6.

0
0

Em cada passo, os nmeros preenchidos so nicos para respeitar as


condies do problema.

Figura 40.4

Nos passos seguintes, as casas preenchidas so as vizinhas da casa


marcada.

A soma dos nmeros nas casas preenchidas 3. Fazendo uma anlise


semelhante, comeando no canto inferior esquerdo ou no canto superior direito, conclumos que a soma dos nmeros das outras casas
tambm igual a 3.

Soluo: Cada casa s pode ter um vizinho com um nmero 1 e os


outros vizinhos devem ser zeros, j que a soma dos vizinhos 1.

.
Figura 40.5

1
0

0
0

.1
Figura 40.6

1
0

0
0

0
.1

1
0

Figura 40.7

www.obmep.org.br

OBMEP

Nvel 2

.
19. Aritmtica
e lgebra

Nvel 2 Solues

41 | Mltiplo de 36
Determine o maior mltiplo de 36 que possui todos os algarismos
pares e diferentes.

Fatos que Ajudam: A soma dos al.


garismos de um mltiplo
de 9 divisvel por 9.

Para um nmero ser divisvel por 36 = 4 9, deve ser


divisvel por 4 e por 9. Assim, a soma dos algarismos do nmero n
procurado deve ser divisvel por 9.

Soluo:

Por outro lado, como todos os algarismos so pares, a soma dos algarismos tambm par. Assim, a soma dos algarismos no mnimo
18. Como 0 + 2 + 4 + 6 + 8 = 20, o nmero n deve ser formado pelos
algarismos 0, 4, 6 e 8.
O maior nmero que podemos formar com esses algarismos, sem
repetir, 8640, o qual tambm divisvel por 4, assegurando que
este o nmero procurado.

42 | Quem maior?
Sejam

R = 3 9 + 4 10 + 5 11 + + 2003 2009.

Sugesto: Observe que cada parcela de S da forma

n (n + 10)

S = 1 11 + 2 12 + 3 13 + + 2001 2011

e cada parcela de R da forma

(n + 2) . (n + 8).

(a) Qual o maior nmero: R ou S?


(b) Calcule a diferena entre o maior e o menor.

Fatos que Ajudam:

(a + b) (c + d) =

Soluo:

ac + ad + bc + bd.

(a) Cada parcela de S da forma n (n + 10) = n + 10n e cada


parcela de R da forma (n + 2) (n + 8) = n2 + 10n + 16 com
n = {1, 2, . . . , 2001} em ambos os casos. Assim, para todo n, cada
parcela de R maior que a correspondente em S, o que torna
R > S.
2

(b) A diferena entre as parcelas correspondentes igual a

(n2 + 10n + 16) (n2 + 10n) = 16.


Como existem 2001 parcelas, a diferena entre R e S igual a
16 2001 = 32016.

103

104

Aritmtica e lgebra | Nvel 2 | Solues

43 | Resto da Diviso
Sugesto: No item (b), analise os
.
nmeros que possuem
a soma dos
algarismos maior ou igual a 17.

Um nmero n de dois algarismos dividido pela soma de seus algarismos, obtendo resto r.
(a) Encontre um nmero n tal que r = 0.
(b) Mostre que r no pode ser maior que 15.
(c) Mostre que para qualquer r menor ou igual a 12, existe um n que
deixa resto r ao dividi-lo pela soma de seus algarismos.

Soluo:
(a) Existem vrios exemplos onde o resto da diviso 0, sendo o menor deles n = 12.
(b) Denotemos por S a soma dos algarismos de n.
Observemos que S 18 e a igualdade somente acontece se n =
99, mas neste caso o resto da diviso 9.
Se S = 17, temos dois possveis valores de n = 89 e 98, que
quando divididos por 17 deixam respectivamente restos 4 e 13.
Nos nmeros restantes, a soma dos algarismos menor ou igual
a 16. Assim, o resto deve ser menor ou igual a 15.
O resto igual a 15 se n = 79. Verifique!
(c) Para terminar, basta mostrar um exemplo para cada resto entre
1 e 12. Se consideramos os nmeros 19, 28, 37, . . . , 91, em todos
a soma de seus algarismos 10 e os restos da diviso por 10 so
respectivamente 9, 8, . . . , 1. Para os restos 10, 11 e 12, basta considerar os nmeros 65, 76 e 87.

44 | Soma de Consecutivos
Sugesto: Para quatro nmeros
consecutivos use a notao x, x+1,
x + 2, x + 3.

(a) A soma de quatro inteiros positivos consecutivos pode ser um


nmero primo? Justifique sua resposta.

Fatos que Ajudam: (a) O nico nmero primo par 2. (b) O nico
nmero primo mltiplo de 3 3.

(b) A soma de trs inteiros positivos consecutivos pode ser um nmero primo? Justifique sua resposta.

Soluo:
(a) Seja x o menor dos nmeros. Ento, a soma em questo

x + (x + 1) + (x + 2) + (x + 3) = 4x + 6 = 2(x + 3).
Este nmero par maior que 2, portanto no pode ser um nmero
primo.
(b) Seja y o menor dos nmeros. Ento, a soma em questo

y + (y + 1) + (y + 2) = 3y + 3 = 3(y + 1).
Este nmero mltiplo de 3 e maior que 3, logo no pode ser um
nmero primo.

www.obmep.org.br

OBMEP

Aritmtica e lgebra | Nvel 2 | Solues

105

45 | Quadrado Perfeito
Observe que

Sugesto: Mostre que a expresso


considerada igual a

1 + 2 + (1 2) = 3
2

(ab + 1)2 .
.

22 + 32 + (2 3)2 = 72
32 + 42 + (3 4)2 = 132 .

Fatos que Ajudam:

Prove que se a e b so inteiros consecutivos ento o nmero

(x + y)2 = x2 + 2xy + y2

a2 + b2 + (ab)2
um quadrado perfeito.

Soluo: Suponha, sem perda de generalidade, que b > a, isto ,


b a = 1. Ento

(b a)2 = 12
2

Problema Relacionado

b 2ab + a = 1
2

Observe que

1 2 3 4 + 1 = 52

a + b = 2ab + 1.

2 3 4 5 + 1 = 112

Somando (ab)2 em cada lado da igualdade, temos

3 4 5 6 + 1 = 192 .

a2 +b2 +(ab)2 = (2ab+1)+(ab)2 = (ab)2 +2(ab)1+12 = (ab+1)2 .

Prove que o produto de quatro inteiros


positivos consecutivos, aumentado em
uma unidade, um quadrado perfeito.

46 | Quantas Fraes!
Prove que

1
2+
4+

..

.+

1
1991

3+

3+
4+

4+

1
..

A=

1+

= 1.

1+

3+

Sugesto: Elimine as milhares de


fraes, fazendo

.+

1
1
..

.+

1
1991

1
1991

Soluo: Faamos

A=

3+
4+

1
..

.+

1
1991

Assim, a soma em questo ser

1
+
2+A

www.obmep.org.br

1
1
1+
1+A

1
1+A
2+A
+
=
= 1.
2+A 2+A
2+A

OBMEP

106

Aritmtica e lgebra | Nvel 2 | Solues

47 | Primos No!
Sugesto: Tente fatorar os nmeros dados:

(a) Prove que o nmero 3999991 no primo.

(a) Escrevendo o nmero dado


como uma diferena de dois
quadrados.

(b) Prove que o nmero 1000343 no primo.

(b) Escrevendo o nmero dado


. de dois cubos.
como uma soma

Soluo:
(a) Observe que

Fatos que Ajudam: Utilize as identidades:

3999991 = 4000000 9
= 4 106 32

(a) m2 n2 = (m n)(m + n)

= (2 103 )2 32

(b) m3 + n3 = (m + n)(m2 mn +

= (2 103 3)(2 103 + 3) = 1997 2003,

n2 )

e portanto no um nmero primo.


(b) Observe que

1000343 = 106 + 73
= (102 )3 + 73 =
= (102 + 7)((102 )2 102 7 + 72 )
= 107 9349,
portanto no primo.

48 | Trilegais
Sugesto: Estude a quantidade de
nmeros pares e mpares em um
dos subconjuntos com trs elementos.

Fatos que Ajudam: A soma de dois


nmeros pares ou mpares resulta
num nmero par. A soma de um
nmero par com um nmero mpar resulta num nmero mpar.

Um conjunto de nmeros chamado trilegal se pode ser dividido em subconjuntos com trs elementos de tal modo que um
dos elementos seja a soma dos outros dois. Por exemplo, o conjunto {1, 2, 3, . . . , 11, 12} trilegal pois pode ser dividido em {1, 5, 6},
{2, 9, 11}, {3, 7, 10} e {4, 8, 12}.
(a) Mostre que {1, 2, . . . , 14, 15} trilegal.
(b) Mostre que {1, 2, . . . , 2010} no trilegal.

Soluo:
(a) Para a primeira parte basta encontrar uma distribuio em subconjuntos com trs elementos, por exemplo

{1, 6, 7}, {2, 12, 14}, {3, 8, 11}, {4, 9, 13}, {5, 10, 15}.
(b) Observemos que se um conjunto de trs elementos cumpre a condio de ser trilegal, ento ele tem de ser da forma

{par, par, par}


ou

{mpar, mpar, par}.


Suponhamos que podemos dividir o conjunto em subconjuntos
trilegais que tem A conjuntos do primeiro tipo e B conjuntos de
segundo tipo. Como a quantidade de nmeros mpares menores
que 2010 1005, devemos ter 2B = 1005, o que contraditrio.

www.obmep.org.br

OBMEP

Aritmtica e lgebra | Nvel 2 | Solues

107

49 | Diferena de Quadrados
(a) De quantas formas possvel escrever o nmero 105 como diferena de dois quadrados perfeitos?
(b) Mostre que no possvel escrever o nmero 106 como diferena
de dois quadrados perfeitos.

Fatos que Ajudam: A diferena entre os quadrados de dois nmeros


igual ao produto da soma destes
nmeros pela diferena dos mes.
mos nmeros. Algebricamente:

m2 n2 = (m + n)(m n).

Soluo:
(a) Sejam x e y dois inteiros positivos tais que a diferena entre seus
quadrados igual a 105, ou seja, x2 y2 = 105. Fatorando,
obtemos (x y)(x + y) = 105 e, portanto, x + y e x y devem ser divisores de 105, com x + y > x y. Observe que
1 105 = 3 35 = 5 21 = 7 15 so todas as maneiras de escrever
o nmero 105 como produto de dois inteiros positivos. Assim,
teremos quatro casos:

{
x + y = 105
xy=1
{
x + y = 35
xy=3
{
x + y = 21
xy=5
{
x + y = 15
xy=7

x = 53 e y = 52.
x = 19 e y = 16.
x = 13 e y = 8.
x = 11 e y = 4.

Portanto, possvel escrever 105 como diferena de dois quadrados de quatro formas, a saber: 532 522 , 192 162 , 132 82 e
112 42 .
(b) Observe que quaisquer que sejam os inteiros x e y, os nmeros
x + y e x y so ambos pares ou ambos mpares, pois a soma dos
dois nmeros igual a 2x, que par, logo no podemos ter um
par e o outro mpar.
Deste modo conclumos que o produto (x + y)(x y) = x2 y2
mltiplo de 4 (caso x + y e x y sejam pares) ou um nmero
mpar (caso x + y e x y sejam mpares).
Como 106 par mas no divisvel por 4, no pode ser escrito
como diferena de dois quadrados.

www.obmep.org.br

OBMEP

108

Aritmtica e lgebra | Nvel 2 | Solues

50 | Outra de Joozinho
Sugesto: Verifique que a sequncia que fica no quadro depois de
todo o processo peridica.

Fatos que Ajudam: Um nmero e a


soma de seus algarismos deixam o
mesmo resto quando so divididos
por 9.

Joozinho escreveu os nmeros de 1 at 100000 no quadro, depois foi


trocando cada nmero pela soma de seus algarismos e repetiu este
processo at obter uma lista de 100000 nmeros de um algarismo. Por
exemplo, comeando pelo nmero 7234 obtemos 7 + 2 + 3 + 4 = 16 e
1 + 6 = 7.
(a) Que nmero ficou no lugar do nmero 98765?
(b) Quantas vezes aparece o nmero 8 na lista final?
(c) Qual o nmero que mais vezes se repete?

Soluo:
(a) 98765 9 + 8 + 7 + 6 + 5 = 35 3 + 5 = 8.
(b) Observemos que um nmero e a soma de seus algarismos deixam
o mesmo resto quando divididos por 9. Assim, depois de terminar
todo o processo vamos obter uma lista da forma

1, 2, 3, 4, 5, 6, 7, 8, 9, 1, 2, 3, 4, . . . , 8, 9, 1.
Assim at 99999, cada um dos algarismos aparece 11111 vezes,
em particular o 8 aparece 11111 vezes.
(c) Do item anterior fica claro que o nmero que mais se repete o
1, pois aparece 11112 vezes na lista.

www.obmep.org.br

OBMEP

20. Geometria
Nvel 2 Solues

51 | Colar de Ouro
Arquelogos encontraram um colar de ouro feito de placas no formato de pentgonos regulares. Cada uma destas placas est conectada a outras duas placas, como ilustra a figura.
.

Sugesto: Calcule o ngulo interno


do polgono determinado pelo colar.

.
Fatos que Ajudam:
A medida do
ngulo interno de um polgono regular de n lados dada pela frmula

180 (n2)
.
n

.
144
Figura 51.1

108 108

Quantas placas formam o colar?


O ngulo interno de um pentgono regular mede 108 .
Assim, o ngulo interno do polgono determinado pelo colar mede
360 108 108 = 144 . Devemos ento encontrar n tal que

Soluo:

Figura 51.2

180 (n 2)
= 144 .
n
Resolvendo esta equao, obtemos n = 10. Portanto, dez placas formam o colar.

52 | AP x BN
ABCD um retngulo, AD = 5 e CD = 3.
A

Sugesto: Calcule a rea do tringulo APB de dois modos distintos.

D
N
P

C
Figura 52.1

Se BN perpendicular a AP, calcule AP BN.

Soluo: Vamos calcular a rea do tringulo APB de dois modos


diferentes.
109

. A rea de um
Fatos que Ajudam:
tringulo igual a metade do produto da medida da base pela medida da altura relativa essa base.

110

Geometria | Nvel 2 | Solues

D
N

Seja Q o p da altura relativa ao lado AB no tringulo APB. Ento a


rea do tringulo APB igual a
base altura

P
.

AB PQ
AB AD
35
15
=
=
=
.
2
2
2
2

Porm, podemos calcular a rea do tringulo APB escolhendo por


base o lado AP e, neste caso, BN a altura. Assim,

AP BN
15
=
,
2
2

Figura 52.2

donde AP BN = 15.

D
N
P

e APD
possuem a mesma meSegunda Soluo: Os ngulos BAN
. Ento os
dida, porque ambos so o complemento do ngulo DAP
tringulos ANB e PDA so semelhantes, pois possuem dois pares
de ngulos de mesma medida. Portanto,

BA
BN
=
,
AP
AD

Figura 52.3

e segue que AP BN = BA AD = 15.

53 | Dois Quadrados
Sugesto: Trace a diagonal AC.
Fatos que Ajudam:
Tringulos
.
com mesma base e mesma altura
possuem reas iguais.

Na figura, ABCD e CEFG so quadrados e o lado do quadrado CEFG


mede 12 cm.

D
G

D
G

Figura 53.1

Quais so os possveis valores da rea do tringulo AEG?

C
Figura 53.2

E
Soluo: Traamos a diagonal AC do quadrado ABCD. Como as retas AC e GE formam ngulo de 45o em relao reta BE, conclumos
que AC e GE so paralelas.
Seja X um ponto arbitrrio sobre AC. Os tringulos AGE e XGE
possuem a mesma rea, pois ambos tm a mesma base GE e a mesma
altura que corresponde distncia entre as retas paralelas AC e GE.
Tomando X = C, conclumos que a rea do tringulo AGE igual
rea de CGE, isto , 12 12/2 = 72 cm2 .

www.obmep.org.br

OBMEP

Geometria | Nvel 2 | Solues

111

54 | O Tesouro do Pirata
Um pirata resolveu enterrar um tesouro em uma ilha. Para tal, ele
caminhou da rvore A para a rocha R1 , e depois a mesma distncia
e na mesma direo at o ponto X. Ele fez o mesmo em relao a
entrada da caverna C e em relao rocha R2 , alcanando os pontos
Y e Z, respectivamente. Ele enterrou o tesouro em T , ponto mdio de
AZ.

Sugesto: Mostre que a posio T


do tesouro no depende do ponto
inicial A.

Fatos que Ajudam: Em todo quadriltero, os pontos mdios dos lados so vrtices de um paralelogramo.

Y
C
X
R2
R1 .

Figura 54.1

Ao voltar ilha para desenterrar o tesouro, o pirata encontrou as


rochas e a caverna, mas no encontrou a rvore. Como o pirata pode
descobrir o tesouro?

Soluo: A chave para o pirata encontrar o tesouro est no seguinte


fato geomtrico:

C
P

Afirmao: Em todo quadriltero, os pontos mdios dos lados so

vrtices de um paralelogramo.
Isto significa que a posio T do tesouro independe da posio da
rvore. No quadriltero AXYZ, R1 , C, R2 e T so os pontos mdios
dos lados. Portanto, R1 CR2 T um paralelogramo.
O pirata pode comear de um ponto qualquer e repetir os procedimentos, ou pode determinar T traando uma reta paralela a R1 C por
R2 e uma paralela a CR2 por R1 . O ponto de interseo das paralelas
o ponto T , localizao do tesouro.

N
Q
.

Figura 54.2

C
P

Demonstrao da Afirmao: Seja ABCD um quadriltero


convexo e M, N, P e Q os pontos mdios dos lados AB, BC, CD
e DA, respectivamente. Vamos provar que MNPQ um paralelogramo.
Considerando o tringulo ABC, o segmento MN a base mdia
relativa ao lado AC, sendo paralelo ao mesmo e medindo a metade
de AC.
Analogamente, olhando para o tringulo CDA, o segmento PQ
a base mdia relativa ao lado AC, e portanto paralelo a AC e
mede a metade de AC.
Segue que os segmentos MN e PQ so iguais e paralelos, mostrando que o quadriltero MNPQ um paralelogramo.

www.obmep.org.br

D
N
Q
.

Figura 54.3

OBMEP

112

Geometria | Nvel 2 | Solues

55 | Bissetrizes
Sugesto: Mostre que CAF e BAE
so tringulos issceles.

Fatos que Ajudam: A bissetriz de


um ngulo o divide em dois ngulos de mesma medida.

Seja ABC um tringulo com AB = 13, BC = 15 e AC = 9. Seja r a


b corta
reta paralela a BC traada por A. A bissetriz do ngulo ABC
b
a reta r em E e a bissetriz do ngulo ACB corta r em F. Calcular a
medida do segmento EF.

Soluo:

Como a reta EF paralela ao lado BC, os ngulos alternos internos


= CFA. Por outro
gerados pela transversal CF so iguais, isto , FCB
= FCA
e assim, FCA
= CFA,
lado, como CF bissetriz, temos FCB
donde o tringulo CAF issceles de base CF. Portanto, AF = AC =
9.
Analogamente, conclumos que o tringulo BAE issceles de base
BE e AE = AB = 13. Assim, EF = EA + AF = 22.

56 | ngulos e ngulos!
Sugesto: Mostre que o tringulo
BEC issceles.

Fatos que Ajudam: A soma das


medidas dos ngulos internos de
um tringulo igual a 180 .

No interior de um tringulo ABC, toma-se um ponto E tal que AE =


= = ECA
, EAC
= 2 e EBC
= 5,
BE e AB = EC. Se ABE
determine .

Soluo:
Sabendo que a soma dos ngulos internos de um tringulo 180
obtemos
{

= 180 ( + ) = 180 2
AEB
= 180 ( + 2) = 180 3.
AEC

Assim, temos que

= 360 (AEC
+ AEB)
=
CEB

= 360 (180 3 + 180 2) = 5.

tambm mede 5, segue que o tringulo BEC


Como o ngulo EBC
issceles. Assim, AB = CE = BC, isto , o tringulo ABC tambm
issceles.
= BAC
= 3 e BCA
+ CAB
+ ABC
= 180 , isto , 3 +
Logo, BCA

3 + 6 = 180 , o que resulta em = 15 .

www.obmep.org.br

OBMEP

Geometria | Nvel 2 | Solues

113

57 | Quadrado, Pentgono e Icosgono


A figura mostra parte de um polgono regular de 20 lados (icosgono)

ABCDEF..., um quadrado BCYZ e um pentgono regular DEVWX.


B

Fatos que Ajudam:


A medida do
.
ngulo interno de um polgono regular de n lados dada pela fr-

C
D

mula

X
Z

Sugesto: Para o item (b), deter .


mine a medida do ngulo CDX

180 (n2)
.
n

F
W

Figura 57.1

.
.
(a) Determine a medida do ngulo Y DC
(b) Mostre que o vrtice X est sobre a reta DY .

Soluo:

180 18
= 162 .
(a) O ngulo interno do icosgono regular mede
20
= 162 90 = 72 . Como YC = CD, o triSegue que Y CD
= DYC
=
ngulo YCD issceles de base YD. Assim, Y DC
180 72
= 54 .
2

C
D
E

Y
F

180 3
=
5

= 162 108 = 54 . Como as retas XD e YD


108 . Assim, CDX
formam o mesmo ngulo com a reta CD, segue que os pontos X,
Y e D pertencem a uma mesma reta.

Figura 57.2

(b) Cada ngulo interno de um pentgono regular mede

C
D

(c) Este problema no tem item (c), mas poderamos ter perguntado:
Qual a nica letra do alfabeto que ainda poderamos usar nesta
figura?

Resposta:

F
W

Figura 57.3

Como T a vigsima letra do alfabeto, o icosgono


ABCDEF . . . T . Como usamos tambm V , W , X, Y e Z, s faltou a letra U! Voc j tinha visto um problema de geometria com
tantas letras?
Problema Relacionado
Construmos dois tringulos equilteros: .ABE interno e BFC externo ao quadrado
ABCD. Prove que os pontos D, E e F se localizam na mesma reta.

www.obmep.org.br

OBMEP

114

Geometria | Nvel 2 | Solues

58 | Enegono Regular
Sugesto: No item (b), prolongue
os lados AB e ED, determinando o
ponto de interseo X.

A figura ilustra um polgono regular de 9 lados. A medida do lado


do polgono a, a medida da menor diagonal b e a medida da maior
diagonal d.

Fatos que Ajudam: A soma das


medidas dos ngulos
de um pol.
gono de n lados dada pela frmula 180 (n 2). A medida do ngulo interno de um polgono regular de n lados dada pela frmula
180 (n2)
.
n

A
B
a
b

d.

C
D

E
Figura 58.1

.
(a) Determine a medida do ngulo BAE
(b) Mostre que d = a + b.

Soluo:

A
B
C

(a) A medida do ngulo interno do enegono regular (9 lados) igual


a 180 7/9 = 140 .
Considere agora o pentgono ABCDE, como indicado na figura.
A soma de seus ngulos internos 180 (5 2) = 540 . Sabe = BCD
= CDE
= 140 e pela simetria da figura
mos que ABC
= AED
= . Portanto,
sabemos que EAB

E
Figura 58.2

2 + 3 140 = 540 ,
donde = 60 .

=
(b) Seja X o ponto de interseo das retas AB e DE. Como XAE
= 60 , o tringulo AXE equiltero. O tringulo BXD tamXEA
bm equiltero, pois a reta AE paralela reta BD.
Assim, temos AX = AE e BX = BD. Como AX = AB + BX, temos
AE = AB + BD, ou seja, d = a + b.

www.obmep.org.br

OBMEP

Geometria | Nvel 2 | Solues

115

59 | Hexgono Equiangular
Todos os ngulos de um hexgono ABCDEF so iguais. Mostre que

AB DE = EF BC = CD FA.

Sugesto: Prolongue os lados do


hexgono.

Soluo:

Fatos que Ajudam: A soma dos


ngulos internos de um polgono
com n lados igual a 180 (n 2).

Prolonguemos os segmentos AF, BC e DE determinando os pontos


de interseco X, Y , Z, como mostrado na figura.

X.

Como a soma dos ngulos internos de um hexgono convexo 180


(6 2) = 720 , cada ngulo interno deste hexgono mede 720 /6 =
= 180 120 = 60 e do mesmo modo XBA
= 60 .
120 . Assim, XAB

Segue que o ngulo AXB mede 60 e de igual forma os ngulos em Y


e Z medem 60 .
Portanto, os tringulos XAB, YCD, ZFE e XYZ so equilteros. Em
particular, XY = XZ. Mas

XY = XB + BC + CY = AB + BC + CD

B
C

F
Z E

Figura 59.1

XZ = XA + AF + FZ = AB + AF + EF.
Igualando obtemos BC + CD = AF + EF, donde obtemos EF BC =
CD FA. Pelo mesmo processo, de XY = YZ, obtemos AB DE =
EF BC.

60 | Pentgono Equiltero
Mostre que possvel construir um pentgono com todos os lados de
mesma medida e cujos ngulos internos meam 60 , 80 , 100 , 140
e 160 , em alguma ordem.

Soluo:
Suponhamos que j construmos o pentgono ABCDE e que o ngulo
em A mede 60 . Traando a reta BE, conclumos que o tringulo ABE
= 60 . Logo, BE = AB e, portanto,
equiltero, pois AB = AE e EAB
BCDE tem todos os seus lados com a mesma medida, isto , BCDE
um losango.

Sugesto: Suponha que o pentgono j foi construdo; comece investigando pelo ngulo cuja medida 60 .

. Se um quadriFatos que Ajudam:


ltero possui os quatro lados de
mesma medida, ento ele um losango. Em um losango, os ngulos opostos possuem a mesma medida.

Em particular, os ngulos opostos do losango so iguais. Isto implica


que, no pentgono, o ngulo em B igual ao ngulo em D mais 60 e
o ngulo em E igual ao ngulo em C mais 60 .

Como 160 = 100 + 80 e 140 = 80 + 60 , conclumos que os


ngulos em C e D devem assumir os valores 80 e 100 , no necessariamente nessa ordem, enquanto B e E assumem os respectivos
valores de D e C, adicionados de 60 .

60

E
Figura 60.1

Portanto, para construir tal pentgono basta construir um tringulo


equiltero ABE e um losango BCDE com ngulos de medidas 100 e
80 .

C
B

100
140

60

160

80

E
Figura 60.2

www.obmep.org.br

OBMEP

21. Combinatria
Nvel 2 Solues

61 | Coloraes do Cubo
De quantas formas possvel colorir as 6 faces de um cubo de preto
ou branco? Duas coloraes so iguais se possvel obter uma a
partir da outra por uma rotao.

Soluo: Observemos que basta contar quantas coloraes existem


que tm exatamente 0, 1, 2 e 3 faces pretas, porque os outros casos
so simtricos. Com uma ou nenhuma face preta existe uma nica
colorao para cada caso. Quando temos duas faces pretas temos
duas possveis coloraes que so: quando estas faces so opostas
e quando elas no so. Por ltimo, com trs faces pretas tambm
temos dois casos: quando duas dessas faces pretas so opostas e
quando no existem faces opostas de cor preta. Assim, no total temos
1 + 1 + 2 + 2 + 2 + 1 + 1 = 10 possveis coloraes.

Problema Relacionado
De quantas formas possvel colorir as 12 arestas de um cubo de branco ou de preto?
.
Duas coloraes so iguais quando possvel
obter uma a partir da outra por uma
rotao.

62 | Comparando Sequncias
Um professor e seus 30 alunos escreveram, cada um, os nmeros de
1 a 30 em uma ordem qualquer. A seguir, o professor comparou as
sequncias. Um aluno ganha um ponto cada vez que um nmero
aparece na mesma posio na sua sequncia e na do professor. Ao
final, observou-se que todos os alunos obtiveram quantidades diferentes de pontos. Mostre que a sequncia de um aluno coincidiu com
a sequncia do professor.

Soluo: O nmero de acertos um nmero entre 0 e 30 inclusive.


Mas, observe que 29 no pode ser obtido porque se 29 nmeros esto
em posio certa, s h uma maneira de colocar o 30 nmero, que
em posio certa tambm.
Como h 30 alunos e 30 possveis resultados, {0, 1, . . . , 28, 30}, ento
um aluno escreveu exatamente a sequncia do professor.

117

Sugesto: Selecione uma pessoa


que no acertou .todos os pontos
e determine o nmero mximo de
pontos que ela pode ter acertado.

118

Combinatria | Nvel 2 | Solues

63 | Segmentos e Tringulos
Sugesto: Para o item (a), conte o
nmero de cordas. que saem de um
determinado ponto.

Dez pontos so marcados ao redor de uma circunferncia, como ilustra a figura.

Figura 63.1

(a) Quantas cordas podem ser formadas ligando dois quaisquer destes pontos? (Uma corda um segmento de reta ligando dois pontos sobre uma circunferncia.)
(b) Quantos tringulos podem ser formados ligando trs quaisquer
destes pontos?
.

Soluo:

Figura 63.2

(a) De cada ponto saem 9 cordas e temos 10 pontos. Mas cada corda
contada duas vezes (uma corda AB contada por sair de A e
por sair de B), assim temos 9 10/2 = 45 cordas.
(b) Cada corda lado de 8 tringulos (basta escolher um ponto que
no seja extremidade da corda escolhida) mas cada tringulo
contado trs vezes (uma vez para cada corda). Como temos 45
cordas, ento temos 8 45/3 = 120 tringulos.

Contando Subconjuntos

Figura 63.3

Vamos resolver um problema mais geral em que temos n pontos distribudos na circunferncia. Como cada corda est determinada por dois
pontos, ento precisamos contar de quantas formas podemos escolher 2
pontos entre os n.
O primeiro ponto pode ser escolhido de n formas, j o segundo pode
ser escolhido de n1 formas, pois ele deve ser distinto do primeiro selecionado. Assim temos n(n 1) escolhas de pares ordenados, mas a ordem
em que foram selecionados no importa, porque eles geram o mesmo
subconjunto e assim o mesmo segmento. Portanto, o nmero de subconjuntos de dois pontos ou equivalentemente o nmero de segmentos
n(n 1)/2.
Seguindo este raciocnio, encontrar todos os tringulos equivale
a encontrar todos os subconjuntos de trs pontos dentre os n pontos. Assim, a escolha ordenada de trs pontos pode ser realizada de
n(n 1)(n 2) maneiras, mas como a ordem no importa, ento o subconjunto com trs elementos {a, b, c}, est sendo contado seis vezes:
abc, acb, bac, bca, cab, cba. Deste modo, o nmero de subconjuntos
com trs pontos, ou equivalentemente, o nmero de tringulos com vrtices nos n pontos n(n 1)(n 2)/6.
No caso geral, se queremos saber quantos polgonos convexos com k
vrtices existem (ou equivalentemente, quantos subconjuntos
de k pon(n)
tos temos entre os n pontos), a resposta dada por k (l-se n escolhe
k), que calculado como

( )
n(n 1)(n 2) . . . (n k + 1)
n
.
=
1 2 3 ... k
k

www.obmep.org.br

OBMEP

Combinatria | Nvel 2 | Solues

119

64 | Esqueleto do Cubo
O esqueleto de um cubo 6 6 6, formado por cubinhos 1 1 1
mostrado na figura.

Figura 64.1

(a) Quantos cubinhos formam este esqueleto?


(b) dado um cubo 777 formado por cubinhos 111. Quantos
cubinhos devemos retirar para obter um esqueleto do cubo 7
7 7.

Soluo:
(a) O esqueleto do cubo formado por uma camada superior e uma
inferior com 20 cubinhos cada e quatro colunas com 4 cubinhos
cada.
Assim, o total de cubinhos

2 20 + 4 4 = 56.
(b) Do cubo 7 7 7 foi retirado um cubo central 5 5 5 e em cada
uma das faces foram retirados 5 5 cubinhos.
Portanto, o total de cubinhos retirados foi

5 5 5 + 6 (5 5) = 125 + 150 = 275.

65 | Placas das Bicicletas


Cada uma das placas das bicicletas de Quixajuba contm trs letras. A primeira letra escolhida dentre os elementos do conjunto
A = {G, H, L, P, R}, a segunda letra escolhida dentre os elementos
do conjunto B = {M, I, O} e a terceira letra escolhida dentre os
elementos do conjunto C = {D, U, N, T }.
Devido ao aumento no nmero de bicicletas da cidade, teve-se que
expandir a quantidade de possibilidades de placas. Ficou determinado acrescentar duas novas letras a apenas um dos conjuntos ou
uma letra nova a dois dos conjuntos.
Qual o maior nmero de novas placas que podem ser feitos, quando
se acrescentam as duas novas letras?

Sugesto: Calcule o nmero inicial de placas que podem ser feitas com os elementos dos conjun.
tos A, B e C e depois
refaa o clculo analisando as diversas possibilidades de aumentar em 1 ou 2 os
elementos dos conjuntos.

Soluo: Inicialmente, possvel fazer o emplacamento de 534 =


60 bicicletas. Vamos analisar as duas situaes possveis:

Aumentamos duas letras num dos conjuntos. Com isso, podemos


ter
www.obmep.org.br

OBMEP

120

Combinatria | Nvel 2 | Solues

ABC
734
554
536

Nmero de Placas

84
100
90

Assim, com a modificao mostrada, o nmero de novas placas


no mximo 100 60 = 40.

Aumentar uma letra em dois dos conjuntos. Com isso, podemos


ter

ABC
644
635
545

Nmero de Placas

96
90
100

Neste caso, o nmero de placas novas tambm no mximo 40.

66 | Torneio de Tnis
Sugesto: No item (b), considere os
jogadores que so
. eliminados ao
invs dos que passam para as prximas rodadas.

Num torneio de tnis cada jogador passa para a rodada seguinte somente em caso de vitria. Se no for possvel que sempre passe para
a rodada seguinte um nmero par de jogadores, a organizao do
torneio decide quais rodadas determinados jogadores devem jogar.
Por exemplo, um cabea de chave pode, a critrio dos organizadores,
entrar na segunda rodada, ou passar da primeira para a terceira, de
modo que o total de jogadores que participem de cada rodada seja
par.
(a) Considere um torneio de tnis com 64 jogadores. Quantas partidas so disputadas?
(b) E em um torneio com 2011 jogadores?

Soluo:
(a) Na primeira rodada so realizadas 32 partidas, das quais 32 jogadores passam para a fase seguinte. Depois so realizadas 16
partidas, classificando 16 para a rodada seguinte e assim por diante. Assim, o nmero de partidas do torneio

32 + 16 + 8 + 4 + 2 + 1 = 63.
(b) Como em cada partida um jogador eliminado, ento o nmero
de partidas igual ao nmero de jogadores eliminados, isto ,
2011 1 = 2010.

Problema Relacionado
Um torneio de futebol com 57 times ser disputado com as seguintes regras:
(a) Nenhum jogo pode terminar empatado.

.
(b) O time que perder duas partidas ser eliminado.
(c) O torneio termina quando sobrar apenas um time, que ser o campeo.
Se o time campeo perder uma vez, quantas partidas sero disputadas no torneio?

www.obmep.org.br

OBMEP

Combinatria | Nvel 2 | Solues

121

67 | Pesando Pedras
Possumos 32 pedras, todas com pesos diferentes. Descreva um processo para mostrar que podemos encontrar as duas pedras mais pesadas com 35 pesagens em uma balana de pratos.

Soluo:

Dividimos as pedras em 16 pares, pesamos cada par e


pegamos as 16 mais pesadas. Repetimos o processo com as 16 pedras
obtendo 8 pedras com oito pesagens a mais, 4 pedras com quatro
pesagens, 2 pedras com 2 pesagens e a pedra mais pesada com a
ltima pesagem.

Sugesto: Divida as pedras em pares e realize as pesagens, eliminando as pedras .mais leves. Perceba que a segunda pedra mais pesada somente pode ser eliminada
pela pedra mais pesada.

At este momento foram usadas 16 + 8 + 4 + 2 + 1 = 31 pesagens


para encontrar a pedra mais pesada.
A segunda pedra mais pesada deve ser uma das pedras que foi comparada com a pedra mais pesada, que foram 5 pedras no total. claro
que para descobrir a segunda pedra mais pesada devem ser registradas as comparaes das pesagens anteriores para saber quais pedras
foram comparadas com a pedra mais pesada.
Para determinar a pedra mais pesada entre estas cinco pedras, precisamos de 4 pesagens porque cada vez que fazemos uma pesagem
eliminamos a pedra mais leve. Portanto, precisamos de 35 pesagens
para determinar as 2 pedras mais pesadas.
Temos 68 moedas com pesos diferentes. Fazendo 100 pesagens, encontre a moeda mais pesada e a mais leve.

68 | Produto 2000
Quantos nmeros naturais de cinco algarismos tm o produto de seus
algarismos igual a 2000?

Sugesto: Decomponha
2000 em
.
fatores primos.

Soluo: Inicialmente, observe que 2000 = 24 53 . Como os algarismos do nmero so menores que 10, cada fator 5 deve ser um
algarismo desse nmero. Alm disso, o produto dos outros algarismos deve ser 24 = 16. Assim, temos dois casos:

Os algarismos que faltam so 2 e 8. Nesse caso, existem cinco


possibilidades para posicionarmos o 2, quatro possibilidades para
posicionarmos o 8 e uma nica possibilidade para posicionarmos
cada 5 que resta. Portanto, podemos formar 5 4 = 20 nmeros.
Os algarismos que faltam so 4 e 4. Nesse( caso,
podemos esco)
lher dois lugares para os algarismos 4 de 52 = 10 modos (veja
Contando Subconjuntos na pgina 118) e uma maneira de posicionarmos cada 5 que resta. Portanto, podemos formar 10 nmeros.

Logo, podem ser formados 20 + 10 = 30 nmeros.

www.obmep.org.br

OBMEP

122

Combinatria | Nvel 2 | Solues

69 | Tabuleiro 123 x 123


Sugesto: (a) Divida em dois casos
de acordo com a cor da casa central. (b) Determine. o nmero de tabuleiros 3 3 que podem ser colocados no tabuleiro 123 123.

Num tabuleiro 123 123, cada casa pintada de roxo ou azul de


acordo com as seguintes condies:

Cada casa pintada de roxo que no est na borda do tabuleiro tem


exatamente 5 casas azuis dentre suas 8 vizinhas.
Cada casa pintada de azul que no est na borda do tabuleiro tem
exatamente 4 casas roxas dentre suas 8 vizinhas.
Nota: Duas casas so vizinhas se possuem um lado ou um vrtice em
comum.
(a) Considere um tabuleiro 33 dentro do tabuleiro 123123. Quantas casas de cada cor pode haver neste tabuleiro 3 3?
(b) Calcule o nmero de casas pintadas de roxo no tabuleiro 123123.

Soluo:
(a) Observando um tabuleiro 3 3, podemos claramente ver que seu
centro no est na borda do tabuleiro. A casa do centro pode:

Estar pintada de roxo. Nesse caso, temos dentre suas 8 vizinhas, 5 azuis e 3 roxas. No total, h 4 casas roxas e 5 casas
azuis nesse tabuleiro.

Estar pintada de azul. Nesse caso, temos dentre suas 8 vizinhas, 4 azuis e 4 roxas. No total, h 4 casas roxas e 5 casas
azuis nesse tabuleiro.
.

Figura 69.1

www.obmep.org.br

(b) Como em qualquer tabuleiro 3 3 dentro do tabuleiro 123 123 o


nmero de casas azuis 5 e o nmero de casas roxas 4, podemos
dividir o tabuleiro 123123 em tabuleiros menores 33 conforme
(
)2
a figura 69.1. Deste modo, o tabuleiro dividido em 123
=
3
412 = 1681 tabuleiros 3 3. Como cada tabuleiro 3 3 tem 4
casas roxas, ento h no total 1681 4 = 6724 casas roxas.

OBMEP

22. Diversos
Nvel 2 Solues

70 | Nmeros no W
Em cada uma das casas do W da figura, escrevemos um nmero inteiro de 1 a 9 de modo que a soma dos trs nmeros de cada uma das
quatro linhas seja a mesma.
.

Sugesto: Determine os possveis


valores que podem ser colocados
na casa vazia comum s duas linhas.

9
.
Figura 70.1

Figura 70.2

J esto escritos o 6 e o 9. Como devem ser posicionados os outros


nmeros?

Fatos que Ajudam: A soma dos 9


primeiros nmeros inteiros positivos

Soluo: Seja S a soma dos trs nmeros de cada linha e seja x o


nmero mostrado na figura 70.3. Como o 9, o 6 e x esto em duas

1 + 2 + + 9 = 45.

linhas, a soma de todas as somas das linhas

(1 + 2 + + 9) + (9 + 6 + x) = 45 + (15 + x) = 60 + x
que tambm igual a 4S. Assim,

x
4S = 60 + x S = 15 + .
4

x
Figura 70.3

Como a soma S um nmero inteiro, x deve ser divisvel por 4 e como


x um algarismo, temos que x = 4 ou x = 8, os quais correspondem
a valores de S iguais a 16 ou 17, respectivamente.
Se x = 4, o nmero que falta na linha que contm o 6 deve ser

16 6 4 = 6,
o que no possvel, pois no podemos repetir nmeros.
Logo, a nica possibilidade x = 8 e a soma dos elementos de cada
linha 17. Agora, basta combinar os demais nmeros nas linhas e
obter a distribuio mostrada na figura 70.4.

123

. 7

6
1

2
9

5
3

4
8

Figura 70.4

124

Diversos | Nvel 2 | Solues

71 | Montando Tabelas
Sugesto: Somar i 1 primeira
rodada equivale .a somar 1 rodada anterior.

Montar a tabela de um torneio em que todas as n equipes se enfrentam ao longo de n 1 rodadas (como, por exemplo, em cada turno
do Brasileiro) um problema matemtico bastante elaborado e que
possui vrios mtodos de soluo. Nesta questo, vamos conhecer
uma dessas abordagens.
Vamos considerar um torneio com 6 equipes. Associaremos os nmeros 1, 2, 3, 4, 5 e (infinito) a cada uma das equipes. A primeira
rodada do torneio 1 , 2 5, 3 4. Para montarmos a rodada
i somamos i 1 a cada nmero envolvido nas partidas da rodada
inicial, considerando que

quando a soma ultrapassa 5, subtramos 5 do resultado;


adicionado a qualquer inteiro positivo . Por exemplo, a segunda rodada ser:

(1 + 1) ( + 1), isto , 2
(2 + 1) (5 + 1), isto , 3 1
(3 + 1) (4 + 1), isto , 4 5
(a) Determine as 3 rodadas restantes do torneio, seguindo o mtodo
descrito acima.
(b) A partir do procedimento mostrado, exiba as 7 rodadas de um
torneio com 8 equipes.

Soluo:
(a)

1
2
3
253142
34
45
51

4
5
5314
12
23
(b)

1
2
3
4
27
31
42
53

36475162
45
56
67
71

5
6
7
64
75
16

731425
12
23
34

www.obmep.org.br

OBMEP

Diversos | Nvel 2 | Solues

125

72 | Numerando os Vrtices
Distribumos nos vrtices de um bloco retangular oito nmeros dentre 1, 2, 3, 4, 5, 6, 7, 8, 9, 10 de tal forma que a soma dos nmeros de
uma face qualquer seja igual a 18.
(a) Quais os nmeros descartados na distribuio?

Sugesto: Calcule as somas dos


nmeros de todas as faces do paraleleppedo e observe quantas vezes cada vrtice est sendo contado nessa soma.

Fatos que Ajudam:

(b) Exiba uma possvel distribuio.

1 + 2 + + 10 = 55.

Soluo:
(a) Como o bloco possui seis faces, a soma dos nmeros em todas
as faces 18 6 = 108, mas o nmero atribudo a cada vrtice
contado trs vezes nesta soma. Portanto, a soma dos nmeros
distribudos 108/3 = 36. Como a soma de todos os nmeros de
1 a 10 igual a 55, a soma dos dois nmeros descartados 19.
Conclumos que os nmeros descartados so 9 e 10.

7
1
.

8
(b) Uma possvel distribuio exibida na figura 72.1.

4
6

5
3

Figura 72.1

73 | Corrida de So Paulo a Fortaleza


Numa corrida de So Paulo a Fortaleza participam quatro carros A,
B, C, D que largaram na seguinte ordem: primeiro A, segundo B,
terceiro C e por ltimo D. Durante a corrida, A e B trocaram de
posio (ultrapassaram um ao outro) 9 vezes e B e C trocaram de
posio 8 vezes.
Para saber em que ordem chegaram Fortaleza, s permitido fazer
perguntas do tipo:

Sugesto: Observe que se dois carros trocam de posio


duas ve.
zes, a ordem entre eles continua a
mesma.

Quantas vezes trocaram de posio os carros X e Y ?


Antes de fazer uma pergunta se conhece a resposta da pergunta anterior. Formule trs perguntas que permitam determinar a ordem em
que os quatro terminaram a corrida.

Soluo: Inicialmente, observe que se dois carros trocaram de posio um nmero par de vezes, eles terminaram na mesma ordem em
que comearam e se trocaram de posio um nmero mpar de vezes, eles terminaram na ordem inversa. Isto nos leva a concluir que
B terminou a corrida na frente de A e de C.
Fazemos a primeira pergunta sobre os carros A e C. De acordo com
a resposta saberemos quem terminou na frente.
Suponhamos que A chegou na frente de C (o outro caso anlogo).
Falta determinar a posio de D, para a qual h quatro possibilidades
( frente de B, entre B e A, entre A e C e atrs de C). Fazemos a
segunda pergunta para A e D e dependendo de D chegar na frente
ou atrs de A, perguntamos para B e D ou C e D, respectivamente.
Com a ltima resposta descobriremos entre quais carros D chegou,
determinando a ordem de chegada.

www.obmep.org.br

OBMEP

126

Diversos | Nvel 2 | Solues

74 | Casas Pretas e Brancas


Considere um tabuleiro 6 6 com suas casas coloridas de branco
ou preto. Duas casas so chamadas vizinhas se possuem um lado
comum. A colorao do tabuleiro vai mudando a cada segundo, respeitando a seguinte condio: se num determinado segundo pelo menos duas casas vizinhas de uma determinada casa esto coloridas de
preto, ento no prximo segundo esta ltima casa ser colorida de
preto.
(a) A figura abaixo mostra uma possvel colorao inicial. Como ficar o tabuleiro aps 12 segundos? E aps 13 segundos?

.
(b) Exiba uma colorao inicial com 6 casas pretas de modo que, em
algum momento, todas as casas fiquem pretas.

Soluo:

(a) Seguem as coloraes do tabuleiro a cada segundo.


Observe que a partir de 12 segundos todos os tabuleiros so iguais.

10

11

12

(b) Colorimos inicialmente as casas de uma das diagonais. Aps 5


segundos, todas as casas estaro pretas.

www.obmep.org.br

OBMEP

23. Desafios

Nvel 2 Solues

75 | Ora Bolas!
Cinco bolas iguais esto se movendo na mesma direo ao longo de
uma reta fixa, mantendo uma certa distncia de uma para outra. Na
mesma direo, mas no sentido oposto, outras cinco bolas se movem de encontro s primeiras. As velocidades de todas as bolas so
iguais. Quando duas bolas colidem, voltam na mesma velocidade de
antes, ao longo da mesma direo. Quantas colises entre bolas vo
ocorrer?

Soluo:

0.

Uma soluo clara para o problema seria fazer todo o percurso das
bolas, mas adotaremos outra estratgia.
Imagine que quando h a coliso de duas bolas, ao invs de gerar a
volta das mesmas, uma bola se transforma na outra, como se no
houvesse a coliso. Chamaramos a esse processo de transmutao.
claro que cada coliso do problema inicial corresponde a uma transmutao na nossa interpretao.
Mas o nmero de transmutaes bem mais fcil de calcular, porque as bolas no mudam de direo. As cinco bolas esquerda encontraro as cinco bolas direita e o nmero procurado ser ento
5 5 = 25.

76 | Distncia entre os Vilarejos


A estrada que liga dois vilarejos em uma montanha formada somente por trechos de subida ou descida. Um nibus sempre viaja a
15 km/h em trechos de subida e a 30 km/h em trechos de descida.
Encontre a distncia entre os vilarejos se o nibus leva exatamente
4 horas para fazer a viagem completa de ida e volta.

Soluo: Observe que os trechos de subida no percurso de ida so


exatamente os trechos de descida para a volta e vice-versa. Assim,
em uma viagem de ida e volta a distncia percorrida nas subidas
igual a distncia percorrida nas descidas.
Chamemos de d a distncia entre os dois vilarejos. Como a distncia
total percorrida foi igual a 2d, ento o tempo gasto subindo foi d/15
127

128

Desafios | Nvel 2 | Solues

horas e o tempo gasto descendo foi d/30 horas. Como o tempo total
foi 4 horas, temos

d
d
+
= 4.
15 30
Resolvendo a equao, encontramos d = 40, ou seja, a distncia entre
os vilarejos igual a 40 km.

77 | Amigos que voc pode Contar!


Sugesto: Mostre que a situao
.
do item (a) possvel
e a do item
(b) no.

Considere um grupo de 15 pessoas. possvel que cada uma delas


conhea exatamente:
(a) 4 pessoas do grupo?

(b) 3 pessoas do grupo?

(Admita que se A conhece B ento B conhece A.)

7
2
8

Soluo:

9
15
10
11

14
12

13

Figura 77.1

(a) possvel. Representamos as 15 pessoas por pontos, conforme


o diagrama ao lado. Um arco entre dois pontos significa que as
duas pessoas representadas se conhecem. Como cada ponto est
ligado a dois pontos esquerda e a dois pontos direita, saem
quatro arcos de cada ponto, o que significa que possvel que
cada pessoa conhea exatamente 4 pessoas do grupo.
(b) No possvel! Vamos representar as pessoas por pontos. Ligamos dois pontos se as pessoas representadas se conhecem. Quantos arcos vamos precisar traar para representar todas as amizades? Cada ponto extremidade de 3 arcos, resultando num total
de 15 3 = 45 arcos que saem de todos os pontos. Porm, nesta
contagem, cada arco foi contado duas vezes, nas duas extremidades. Portanto, o nmero de segmentos deve ser 45/2, o que um
absurdo, pois este nmero no inteiro.

78 | Trs Amigos e uma Bicicleta


Sugesto: Perceba que para chegarem em at 2 h 40
. min, cada um
deve fazer pelo menos metade do
percurso de bicicleta.

A distncia entre Coco da Selva e Quixajuba 24 km. Dois amigos


precisam ir de Quixajuba a Coco da Selva e um terceiro amigo precisa
ir de Coco da Selva a Quixajuba. Eles possuem uma bicicleta que
inicialmente est em Quixajuba. Cada um deles pode ir caminhando
a velocidade de 6 km/h, ou de bicicleta a velocidade de 18 km/h. Alm
disso, podem deixar a bicicleta em qualquer ponto do trajeto.
Quixajuba

Coco da Selva

.
Mostre como eles podem proceder para chegarem a seus destinos em
no mximo 2h 40min.

Soluo: Chamaremos de A e B os amigos que esto em Quixajuba


e C o que est em Coco da Selva. Nossos personagens podem seguir
a seguinte estratgia:
www.obmep.org.br

OBMEP

Desafios | Nvel 2 | Solues

129

Na primeira hora, A vai de bicicleta enquanto B e C iro caminhando. Depois dessa hora, A e C se encontram no quilmetro
18 (medido desde Quixajuba) e B est no quilmetro 6.
A continua caminhando e chegar a seu destino depois de uma
hora. Enquanto isso, C continua de bicicleta e B fica parado esperando C chegar. Como a distncia entre C e B de 12 km, isso
acontecer depois de 12/18 = 2/3 h, isto , 40 minutos.
Nesse ponto, C passa a bicicleta para B e cada um continua seu
trajeto chegando a seus destinos em uma hora.
Assim o tempo total empregado por B e C foi de 2 h 40 min, enquanto

A gastou 2 h.

79 | Contando Polgonos
Em uma circunferncia foram marcados 15 pontos brancos e 1 ponto
preto. Consideremos todos os possveis polgonos (convexos) com
seus vrtices nestes pontos.
Vamos separ-los em dois tipos:

Sugesto: Construa um polgono


do tipo 2 a partir. de um polgono
do tipo 1.

Tipo 1: os que possuem somente vrtices brancos.


Tipo 2: os que possuem o ponto preto como um dos vrtices.
Existem mais polgonos do tipo 1 ou do tipo 2? Quantos existem a
mais?

Soluo:
Observe que para cada polgono do tipo 1 podemos construir um
polgono do tipo 2 adicionando o ponto preto.
Por outro lado, se temos um polgono do tipo 2 e retirarmos o ponto
preto, a nica forma de no gerar um polgono se sobrarem exatamente dois pontos brancos.
Portanto, existem mais polgonos do tipo 2 do que do tipo 1.
Para calcular a diferena, basta contar o nmero de pares de pontos
brancos. Para isso, observe que cada ponto branco pode formar um
par com cada um dos outros 14 pontos brancos. Assim, como existem 15 pontos brancos, teremos 15 14 pares ordenados. Segue que
temos 15 14/2 = 105 pares de pontos.

Figura 79.1

Observao: possvel determinar as quantidades de polgonos do


tipo 1 e do tipo 2. Veja a caixa Contando Subconjuntos, na pgina 118.

www.obmep.org.br

OBMEP

130

Desafios | Nvel 2 | Solues

80 | Desafiando os Amigos!
Sugesto:

(a) Suponha

a b c d e.

(a) Adriano escolheu secretamente cinco nmeros a, b, c, d e e e


informou a Bruna os dez nmeros 24, 28, 30, 30, 32, 34, 36, 36,
40 e 42 obtidos pelo clculo de todas as somas de dois nmeros
dentre os cinco escolhidos.

O que podemos dizer sobre a+


b? E sobre d+e
. ? E sobre a+c?

O objetivo de Bruna descobrir a, b, c, d, e. Bruna pode alcanar


seu objetivo?

(b) Carlos no conseguir alcanar seu objetivo porque existem dois conjuntos formados
por quatro nmeros que geram
os nmeros 10, 20, 22, 24, 26 e
36.

(b) Adriano escolheu secretamente quatro nmeros m, n, p e q e


informou a Carlos os seis nmeros 10, 20, 22, 24, 26 e 36 obtidos
pelo clculo de todas as somas de dois nmeros dentre os quatro
escolhidos.
O objetivo de Carlos descobrir m, n, p e q. Ele pode alcanar
seu objetivo?

Soluo:
(a) Suponha que a b c d e. Logo a menor soma a + b e a
maior soma d + e. A segunda menor a + c e a segunda maior
c + e. Assim, temos o sistema

a + b = 24

a + c = 28

c + e = 40

d + e = 42.
Por outro lado, cada nmero utilizado em quatro somas e ento

a+b+c+d+e=
24 + 28 + 30 + 30 + 32 + 34 + 36 + 36 + 40 + 42
= 83.
4
Assim,

c = (a + b + c + d + e) (a + b) (d + e) = 83 24 42 = 17.
Logo,

a = 28 c = 11
b = 24 a = 13
e = 40 c = 23
d = 42 e = 19.
(b) Observe que os nmeros 3, 7, 17 e 19 geram as somas 10, 20, 22,
24, 26 e 36 e o mesmo acontece com os nmeros 4, 6, 16 e 20.
Carlos no alcanar seu objetivo!

Problema Relacionado
Uma lista de seis inteiros positivos p, q, r, s, t, u satisfaz p < q < r < s < t < u.
Existem exatamente 15 pares de nmeros que podem ser formados escolhendo dois
nmeros diferentes desta lista. As somas destes
15 pares de nmeros so:
.

25, 30, 38, 41, 49, 52, 54, 63, 68, 76, 79, 90, 95, 103, 117.
Determine o valor da soma r + s.

www.obmep.org.br

OBMEP

Nvel 3

.
24. Aritmtica
e lgebra

Nvel 3 Solues

81 | Sequncia Numrica II
A sequncia de nmeros t1 , t2 , t3 , . . . est definida por

t1 = 2
tn+1 =

tn 1
tn + 1

para cada inteiro positivo n. Encontrar t2011 .

Soluo: Calculemos os primeiros termos da sequncia:

21
1
=
2+1
3
1/3 1
1
t3 =
=
1/3 + 1
2
1/2 1
t4 =
= 3
1/2 + 1
3 1
t5 =
= 2.
3 + 1
t2 =

Assim, os primeiros cinco termos da sequncia so 2, 1/3, 1/2, 3


e 2. Observamos que a sequncia se repete a cada 4 termos, isto ,

2 = t1 = t5 = t9 = t13 = = t2009 .
Assim, t2010 = 1/3 e t2011 = 1/2.

Problema Relacionado
A calculadora do Dod tem uma tecla especial com o smbolo . Se o visor mostra
um nmero x diferente de 2, ao apertar aparece o valor de 2x3
.
x2
(a) Se o Dod colocar 4 no visor e apertar , qual nmero vai aparecer?
(b) Dod colocou um nmero no visor e, ao
. apertar , apareceu o mesmo nmero.
Quais so os nmeros que ele pode ter colocado no visor?
(c) Dod percebeu que, colocando o 4 no visor e apertando duas vezes, aparece de
novo o 4; da mesma forma, colocando o 5 e apertando duas vezes, aparece de
novo o 5. O mesmo vai acontecer para qualquer nmero diferente de 2? Explique.

133

Sugesto: Calcule
. os primeiros
cinco termos da sequncia.

134

Aritmtica e lgebra | Nvel 3 | Solues

82 | Progresso Geomtrica
Sugesto: A razo da progresso
. ser menor que
geomtrica tem que
2.

A progresso geomtrica 121, 242, 484, 968, 1936,. . . possui trs termos inteiros entre 200 e 1200.
(a) Encontre uma progresso geomtrica crescente que possui quatro
termos inteiros entre 200 e 1200.
(b) Encontre uma progresso geomtrica crescente que possui seis
termos inteiros entre 200 e 1200.

Soluo: Observemos que para obter termos inteiros, a razo entre


p
os termos inteiros deve ser um racional q
e para obter mais de trs
termos a razo tem que ser menor do que 2, j que para uma progresso de razo maior ou igual a 2, com o primeiro termo maior ou
igual a 200, o quarto termo maior ou igual a 200 23 = 1600.
k

p
Como a progresso geomtrica da forma A q
k , para essa expresso
representar um nmero inteiro precisamos que q divida A muitas
vezes. Assim, possveis valores de A so as potncias de q.

Por exemplo, se A = 28 = 256, q = 2 e p = 3, obtemos a sequncia

256, 384, 576, 864, 1296, . . . ,


com quatro termos inteiros entre 200 e 1200.
Por outro lado, se A = 35 = 243, q = 3 e p = 4 obtemos a sequncia

243, 324, 432, 576, 768, 1024, . . . ,


que possui seis termos inteiros entre 200 e 1200.

83 | Funciona?
Sugesto:
Faa a =

2n 1.

2n + 1 e b =

Para um inteiro positivo n considere a funo

4n + 4n2 1

f(n) =
.
2n + 1 + 2n 1

Fatos que Ajudam: Utilize a iden.


tidade

(a2 + ab + b2 )(a b) = a3 b3 .

Calcule o valor de

f(1) + f(2) + f(3) + + f(40).

Soluo: Seja a = 2n + 1 e b = 2n 1. Ento ab = 4n2 1,


a2 + b2 = 4n e a2 b2 = 2. Portanto,
f(n) =

a2 + b2 + ab
.
a+b

Como a b = 0, podemos escrever

f(n) =

a2 + b2 + ab a b
a3 b 3
( 2n + 1)3 ( 2n 1)3

= 2
=
.
a+b
ab
a b2
2

Assim,

f(1) + f(2) + + f(40) =

3
3

( 5)3 ( 3)3
( 81)3 ( 79)3
( 3) ( 1)
+
+ +
=
=
2
2
2
3

( 81) ( 1)3
729 1
=
=
= 364.
2
2

www.obmep.org.br

OBMEP

Aritmtica e lgebra | Nvel 3 | Solues

135

84 | Sistema de Trs Equaes


Sejam a e b nmeros reais tais que existam nmeros reais distintos
m, n e p, satisfazendo as igualdades abaixo:

m + am + b = 0
3
n + an + b = 0

3
p + ap + b = 0.

Sugesto: Subtraia as equaes dadas e fatore o resultado. Depois,


faa o mesmo com a primeira e a
terceira equaes.
Fatos que Ajudam: Diferena de
.
dois cubos:

x3 y3 = (x y)(x2 + xy + y2 ).

Mostre que m + n + p = 0.

Soluo: Subtraindo a segunda equao da primeira, obtemos

A soma das razes da equao


ax3 + bx2 + cx + d = 0 igual a
b/a.

m3 n3 + am an = 0
(m n)(m2 + mn + n2 ) + a(m n) = 0
(m n)(m2 + mn + n2 + a) = 0
e como m n = 0, temos que m2 + mn + n2 + a = 0. Subtraindo a
terceira equao da primeira, obtemos de forma anloga m2 + mp +
p2 + a = 0.
Subtraindo estas duas ltimas relaes encontradas, temos

mn mp + n2 p2 = 0
m(n p) + (n + p)(n p) = 0
(n p)(m + n + p) = 0,
e como n p = 0, conclumos finalmente que m + n + p = 0.

Segunda Soluo: Considere o polinmio de terceiro grau P(x) =


x3 + 0x2 + ax + b. As relaes dadas no problema nos garantem
que m, n e p so as razes de P. Portanto, a soma das razes dessa
equao m + n + p = 0.

85 | Soma de Potncias
Sugesto: Expanda

(a) Mostre que a identidade abaixo sempre verdadeira:

n+1

n+1

+b

= (a + b)(a + b ) ab(a

n1

n1

+b

).

.
(a + b)(an + bn ).

(b) Sejam a e b nmeros reais tais que a+b = 1 e ab = 1. Mostre


que o nmero a10 + b10 inteiro, calculando seu valor.

Soluo:
(a) Observemos que

(a + b)(an + bn ) = an+1 + abn + ban + bn+1 =


= an+1 + bn+1 + ab(an1 + bn1 )
e a identidade segue.
(b) Chamemos de fn = an + bn . Observe que f1 = a + b = 1.
Calculemos f2 :

f2 = a2 + b2 = (a + b)2 2ab = 12 2 (1) = 3.


www.obmep.org.br

OBMEP

136

Aritmtica e lgebra | Nvel 3 | Solues

Pela identidade do item (a) temos que

an+1 + bn+1 = (a + b)(ab + bn ) ab(an1 + bn1 )


ou equivalentemente

fn+1 = (a + b)fn abfn1 = fn + fn1 .


Assim,

f3 = f2 + f1 = 4
f4 = f3 + f2 = 7
f5 = f4 + f3 = 11
f6 = f5 + f4 = 18
f7 = f6 + f5 = 29
f8 = f7 + f6 = 47
f9 = f8 + f7 = 76
f10 = f9 + f8 = 123.
Portanto, a10 + b10 = f10 = 123.

86 | Sistema com Potncias


(a) Verifique a identidade

(a + b + c)3 = a3 + b3 + c3 + 3(a + b)(b + c)(c + a).


(b) Resolva o sistema

x + y + z = 1
x2 + y2 + z2 = 1

3
x + y3 + z3 = 1.
Soluo:
(a) Vamos expandir (a + b + c)3 como [(a + b) + c]3 .

[(a + b) + c]3 = (a + b)3 + 3(a + b)c(a + b + c) + c3


= a3 + b3 + 3ab(a + b) + 3(a + b)c(a + b + c) + c3
= a3 + b3 + c3 + 3(a + b)[ab + c(a + b + c)]
= a3 + b3 + c3 + 3(a + b)(c2 + c(a + b) + ab]
= a3 + b3 + c3 + 3(a + b)(b + c)(c + a).
(b) Utilizando a identidade verificada no item (a), obtemos

(x + y + z)3 = x3 + y3 + z3 + 3(x + y)(y + z)(z + x).


Substituindo os valores de x + y + z e x3 + y3 + z3 chegamos a

13 = 1 + 3(x + y)(y + z)(z + x),


donde (x + y)(y + z)(z + x) = 0. Assim, x = y ou y = z ou
z = x. Como as solues so simtricas, vamos supor x = y.
Logo, de x + y + z = 1, obtemos z = 1 e de x2 + y2 + z2 = 1
obtemos 2x2 = 0, ou x = 0. Conclumos que as possveis solues
so (0, 0, 1), (0, 1, 0) e (1, 0, 0).

www.obmep.org.br

OBMEP

Aritmtica e lgebra | Nvel 3 | Solues

137

87 | Sistema com 7 Variveis


(a) Determine a, b e c tais que a igualdade

(n + 2)2 = a(n + 1)2 + bn2 + c(n 1)2


seja verdadeira qualquer que seja o nmero n.
(b) Suponha que x1 , x2 , . . . , x7 satisfazem o sistema

x1 + 4x2 + 9x3 + 16x4 + 25x5 + 36x6 + 49x7 = 1


4x1 + 9x2 + 16x3 + 25x4 + 36x5 + 49x6 + 64x7 = 12

9x1 + 16x2 + 25x3 + 36x4 + 49x5 + 64x6 + 81x7 = 123

Sugesto: (a) Expanda os termos e


os agrupe como o polinmio na varivel n. (b) Utilize os valores encontrados em (a).

Fatos que Ajudam: Se um polinmio se anula para infinitos valores, ento todos os seus coeficientes so nulos.

Determine o valor de

16x1 + 25x2 + 36x3 + 49x4 + 64x5 + 81x6 + 100x7 .


Soluo:
(a) Se um polinmio se anula para infinitos valores, ento todos os
seus coeficientes so nulos.
Expandindo a igualdade temos

n2 + 4n + 4 = a(n2 + 2n + 1) + bn2 + c(n2 2n + 1).


Assim,

(a + b + c 1)n2 + (2a 2c 4)n + (a + c 4) = 0,


qualquer que seja o nmero n. Logo,

a + b + c 1 = 0
2a 2c 4 = 0

a+c4=0

Resolvendo o sistema encontramos a = 3, c = 1 e b = 3.


(b) Sejam

S1 = x1 + 4x2 + 9x3 + 16x4 + 25x5 + 36x6 + 49x7 = 1


S2 = 4x1 + 9x2 + 16x3 + 25x4 + 36x5 + 49x6 + 64x7 = 12
S3 = 9x1 + 16x2 + 25x3 + 36x4 + 49x5 + 64x6 + 81x7 = 123.
Pela identidade da parte (a), temos que

16x1 + 25x2 + 36x3 + 49x4 + 64x5 + 81x6 + 100x7 =


= 3S3 3S2 + S1 = 3 123 3 12 + 1 = 334.

www.obmep.org.br

OBMEP

138

Aritmtica e lgebra | Nvel 3 | Solues

88 | Algarismo do Quadrado
Sugesto: Escreva. o nmero como
10a + b, sendo b um algarismo.

O quadrado de 13 169, que tem como algarismo das dezenas o nmero 6. O quadrado de outro nmero tem como algarismo das dezenas o nmero 7. Quais so os possveis valores para o algarismo das
unidades desse quadrado?

Soluo: Suponhamos que o nmero 10a+b, com b um algarismo.


Quando elevamos ao quadrado obtemos

(10a + b)2 = 100a2 + 20ab + b2 ,


que tem trs parcelas: 100a2 , 20ab e b2 .
A primeira parcela termina em 00, enquanto a segunda termina em
um nmero par seguido por zero. Assim para o algarismo das dezenas ser 7, isto , mpar, necessrio que o algarismo das dezenas de
b2 seja mpar, o que somente acontece quando b = 4 ou b = 6. Em
cada um dos casos, 42 = 16 e 62 = 36, o algarismo das unidades do
quadrado 6.

Problema Relacionado

. apenas por algarismos 0 e 6?


Existe um nmero quadrado perfeito formado

89 | Maior Divisor mpar


Sugesto: Sendo Sn a soma de tais
divisores, calcule a diferena Sn
Sn1 .
Fatos que Ajudam:
. A soma dos n
primeiros nmeros mpares

1 + 3 + + (2n 1) = n .
2

Seja n um nmero inteiro positivo. Para cada um dos inteiros n + 1,


. . ., 2n considere o seu maior divisor mpar. Prove que a soma de
todos estes divisores igual a n2 .

Soluo: Chamemos de Sn a soma dos maiores divisores mpares


dos nmeros n + 1, . . . , 2n. Por clculo direto temos que S1 = 1,
S2 = 3 + 1 = 4 = 22 , S3 = 1 + 5 + 3 = 9 = 32 e S4 = 5 + 3 + 7 + 1 =
16 = 42 .
Se queremos calcular Sn+1 , que a soma dos maiores divisores mpares dos nmeros

n + 2, n + 3, . . . , 2n, 2n + 1, 2(n + 1),


como n + 1 e 2(n + 1) tm os mesmos divisores mpares, isto equivalente a somar os maiores divisores mpares de

n + 2, n + 3, . . . , 2n + 1, n + 1
que igual a Sn + (2n + 1). Assim, Sn+1 = Sn + (2n + 1). Portanto,

S2 S1 = 3
S3 S2 = 5
..
.

Sn Sn1 = 2n 1.
Somando todas estas igualdades obtemos Sn S1 = 3 + 5 + +
(2n 1) e deste modo, Sn = 1 + 3 + + (2n 1) = n2 .

www.obmep.org.br

OBMEP

Aritmtica e lgebra | Nvel 3 | Solues

139

90 | Algarismos
Com os algarismos a, b e c construmos o nmero de trs algarismos
abc e os nmeros de dois algarismos ab, bc e ca. Ache todos os
possveis valores de a, b e c tais que
inteiro.

abc + a + b + c
seja um nmero
ab + bc + ca

Sugesto: Mostre que o denominador sempre divisvel por 11 e que


.
a + c = 11.
Fatos que Ajudam: abc = 100a +
10b + c, ab = 10a + b.

Soluo: Observemos que

ab + bc + ca = (10a + b) + (10b + c) + (10c + a) = 11(a + b + c),


de forma que o denominador da frao divisvel por 11. Como a
frao um inteiro, o numerador

abc + a + b + c = (100a + 10b + c) + a + b + c = 101a + 11b + 2c


tambm divisvel por 11. Como

101a + 11b + 2c = 11(9a + b) + 2(a + c),


segue que a + c divisvel por 11. Como a e c so algarismos e a = 0,
1 a + c 18, donde a + c = 11. Substituindo c na expresso dada
obtemos

abc + a + b + c
11(9a + b + 2)
=
ab + bc + ca
11(b + 11)
9a + b + 2
=
b + 11
b + 11 + 9a 9
=
b + 11
9(a 1)
=1+
,
b + 11
e 9(a1)/(b+11) um inteiro. O algarismo a no pode ser 1, porque
a + c = 11. Observamos que se b + 11 no divisvel por 3, teramos
que b + 11 divide a 1 que impossvel, j que b + 11 > a 1. Assim,
b + 11 igual a 12, 15 ou 18. Ento b = 1, 4 ou 7.

Se b = 1, como 9(a 1)/12 = 3(a 1)/4 inteiro temos que a = 5


ou 9 que gera os nmeros 516 e 912.
Se b = 4, 9(a 1)/15 = 3(a 1)/5 inteiro e assim, a = 6 que gera
o nmero 645.
Se b = 7, 9(a 1)/18 = (a 1)/2 e ento a = 3, a = 5, a = 7 ou
a = 9 gerando os nmeros 378, 576, 775 e 972.

www.obmep.org.br

OBMEP

.
25. Combinatria
e Probabilidade

Nvel 3 Solues

91 | Produto Par
Tio Man tem duas caixas, uma com sete bolas distintas numeradas
de 1 a 7 e outra com oito bolas distintas numeradas com todos os
nmeros primos menores que 20. Ele sorteia uma bola de cada caixa.
Qual a probabilidade de que o produto dos nmeros das bolas sorteadas seja par?

Soluo: O produto dos nmeros sorteados mpar somente se as


duas bolas sorteadas tm nmeros mpares.
A probabilidade de sortearmos da primeira caixa uma bola com nmero mpar 4/7 e a probabilidade de sortearmos uma bola mpar
da segunda caixa 7/8, porque esta contm bolas com os nmeros
{2, 3, 5, 7, 11, 13, 17, 19}.
Assim, a probabilidade do produto dos nmeros das caixas ser mpar

4 7
1
= .
7 8
2

Portanto, a probabilidade do produto ser par 1 1/2 = 1/2.

92 | Subconjuntos com Soma Grande


Considere o conjunto A = {1, 2, 3, . . . , 2011}. Quantos subconjuntos
de A existem de modo que a soma de seus elementos seja 2023060?

2011 2012
Soluo: Observe que a soma 1 + 2 + + 2011 =
=
2
2023066. Logo, para obtermos um subconjunto de A que tenha para
soma de seus elementos 2023060, basta retirarmos de A os elementos
cuja soma 6. Os possveis casos so:
Subconjuntos com um elemento : {6}.
Subconjuntos com dois elementos: {2, 4} e {1, 5}.
Subconjuntos com trs elementos: {1, 2, 3}.
Portanto, h quatro subconjuntos de A cuja soma de seus elementos
6 e por consequncia tambm h quatro subconjuntos de A cuja
soma dos elementos 2023060.

141

Sugesto: Calcule. a probabilidade


do produto ser mpar.

142

Combinatria e Probabilidade | Nvel 3 | Solues

93 | Formiga Aleatria
Sugesto: Observe que a formiga
. segundos msempre est no 1 nos
pares.

Uma formiga se movimenta uma unidade por segundo sobre os pontos 0, 1 e 2 da figura a seguir, comeando do ponto 0.
.

Figura 93.1

(a) Quais so os possveis percursos da formiga at 3 segundos?


(b) Quantos possveis percursos pode fazer a formiga at 10 segundos?

Soluo:
(a) At trs segundos temos dois possveis percursos: 0 1 0 1
ou 0 1 2 1.
(b) Observemos que quando a formiga est nos pontos 0 e 2 ela somente tem uma possibilidade para caminhar no segundo seguinte,
que ir para 1. Quando est em 1 ela tem duas possibilidades no
segundo seguinte, que ir para 0 ou 2. Assim, nos segundos mpares a formiga sempre est no 1, enquanto nos segundos pares
ela est no 0 ou no 2. Portanto, o nmero de caminhos possveis
depois de 10 segundos

1 2 1 2 1 2 1 2 1 2 = 32.

94 | Algarismos e Paridade
Sugesto: Conte os nmeros pares e os nmeros. mpares separadamente.

Tiago escreve todos os nmeros de quatro algarismos no nulos distintos que possuem a mesma paridade. Qual a probabilidade de que,
ao escolhermos um desses nmeros, ele seja par?

Soluo: Os quatro algarismos escolhidos fazem parte dos conjuntos A = {1, 3, 5, 7, 9} ou B = {2, 4, 6, 8}.
Com os elementos do conjunto A temos 5 possibilidades para o primeiro algarismo, 4 para o segundo, 3 para o terceiro e 2 para o quarto,
totalizando 5 4 3 2 = 120 nmeros com 4 algarismos distintos.
J com os elementos do conjunto B temos 4 possibilidades para o
primeiro algarismo, 3 para o segundo, 2 para o terceiro e 1 para o
quarto, totalizando 4321 = 24 nmeros com quatro algarismos
distintos.
Assim, possvel formar 120 + 24 = 144 nmeros. De todas as possibilidades calculadas, apenas as geradas pelo conjunto B so nmeros
pares.
Portanto, a probabilidade pedida 24/144 = 1/6.

www.obmep.org.br

OBMEP

Combinatria e Probabilidade | Nvel 3 | Solues

143

95 | Bolas Pretas, Brancas e Azuis


Considere uma urna que contm uma bola preta, quatro bolas brancas e algumas bolas azuis. Uma bola retirada ao acaso dessa urna,
sua cor observada e a bola devolvida urna. Em seguida, retira-se
novamente, ao acaso, outra bola dessa urna. Para quais quantidades de bolas azuis, a probabilidade das duas bolas retiradas terem
mesma cor vale 1/2?

Soluo: Chamemos de n o nmero de bolas azuis da caixa. Quando


retiramos as duas bolas, elas podem ser:

Sugesto: Considere n o nmero


de bolas azuis da urna e determine
as probabilidades de as duas bolas
retiradas serem ambas pretas, ambas brancas e ambas azuis.

Fatos que Ajudam: A probabilidade que acontea um dentre trs


eventos independentes a soma
das probabilidades que cada um
acontea.

Duas bolas pretas. A probabilidade


1
1

=
n+5 n+5

1
n+5

)2
;

Duas bolas brancas. A probabilidade


4
4

=
n+5 n+5

4
n+5

)2
;

Duas bolas azuis. A probabilidade


n
n

=
n+5 n+5

n
n+5

)2
.

Logo, a probabilidade das duas bolas serem da mesma cor a soma


das probabilidades individuais:

1
n+5

)2

(
+

4
n+5

)2

(
+

n
n+5

)2
=

1 + 16 + n2
1
= .
(n + 5)2
2

Simplificando a igualdade obtemos que n2 10n + 9 = 0, donde n


igual a 1 ou 9.

96 | Aparando um Poliedro
Considere um poliedro convexo com 100 arestas. Todos os vrtices
foram aparados prximos a eles mesmos, usando uma faca plana
afiada (isto foi feito de modo que os planos resultantes no se intersectassem no interior ou na fronteira do poliedro). Calcule para o
poliedro resultante:

Sugesto: Determine a relao entre as arestas do .antigo poliedro e


os vrtices do novo.

(a) o nmero de vrtices.


(b) o nmero de arestas.

Soluo:
(a) Quando realizamos os cortes, cada aresta antiga estar ligada a
dois vrtices novos, enquanto os vrtices antigos desaparecem.
Assim o novo poliedro tem 200 vrtices.
(b) Quando realizamos um corte, de cada novo vrtice surgem duas
arestas novas (correspondentes a duas arestas consecutivas na
nova face criada) e uma aresta antiga. Assim, de cada vrtice do
www.obmep.org.br

OBMEP

144

Combinatria e Probabilidade | Nvel 3 | Solues

novo poliedro saem exatamente 3 arestas. Deste modo, se somarmos a quantidade de arestas que partem de todos os vrtices, encontraremos 3 200 = 600. Este nmero corresponde ao
dobro do nmero de arestas, pois cada uma foi contada em dois
vrtices. Logo, o nmero de arestas 300.

Quantas faces tem este novo poliedro?

97 | Bolas Azuis e Vermelhas


Fatos que Ajudam: O nmero de
modos de escolher (dois
n
) dentre
n(n1)
n
objetos distintos . 2 =
.
2
Veja Contando Subconjuntos na
pgina 118.

Existem bolas azuis e bolas vermelhas em uma caixa. A probabilidade


de sortear duas bolas de cores diferentes, ao retirar duas bolas ao
acaso, 1/2. Prove que o nmero de bolas na caixa um quadrado
perfeito.

Soluo: Suponha que existam a bolas azuis e v bolas vermelhas na


caixa.
Problema Relacionado
Em um torneio de xadrez cada jogador disputou uma partida com cada um
dos demais participantes. A cada partida, havendo empate, cada jogador ganhou 1/2 ponto; caso contrrio, o vencedor ganhou 1 ponto
e o perdedor,
.
0 ponto. Participaram homens e mulheres e cada participante conquistou
o mesmo nmero de pontos contra homens que contra mulheres. Mostre que
o nmero total de participantes um
quadrado perfeito.

(1) O nmero de modos de escolher duas bolas de cores diferentes


av.
(2) O nmero de modos de escolher duas bolas quaisquer

(a+v)
2

(3) De (1) e (2), a probabilidade


de sortear duas bolas de cores di(
)
ferentes av/ a+v
.
2
Portanto,

av
1
(a + v)(a + v 1)
(a+v) =
2av =
,
2
2
2
donde

4av = (a + v)2 (a + v) a + v = (a v)2 .


Logo, a quantidade de bolas um quadrado perfeito.

98 | Dez Pontos no Plano


Fatos que Ajudam: O nmero de
maneiras de escolher k objetos distintos dentre n objetos distintos

( )
.
n(n 1) . . . (n k + 1)
n
=
.
k
k!
Veja o quadro na pgina 118.

Dez pontos so dados no plano e no existem trs colineares. Quatro


segmentos distintos ligando pares destes pontos so escolhidos ao
acaso, mas todos com a mesma probabilidade. Qual a probabilidade
de trs dos segmentos escolhidos formarem um tringulo?

Soluo:
O nmero de possveis segmentos entre os 10 pontos
(10)
=
45
e o nmero de formas de escolher 4 desses segmentos
2
(45)

J o nmero de formas de escolher 4 segmentos de tal modo que trs


deles formem um tringulo igual ao nmero de maneiras de escolher trs vrtices, que determinam os trs segmentos do tringulo,
multiplicado
pelo nmero de formas de escolher o outro segmento,
( )
isto 10
(453)
. Portanto, a probabilidade de que trs dos quatro
3
segmentos formem um tringulo

(10)

42
10 9 8 42 4!
16
=
=
.
(45)
3!

45

44

43

42
473
4

www.obmep.org.br

OBMEP

Combinatria e Probabilidade | Nvel 3 | Solues

145

99 | Contando Diagonais no Poliedro


Um poliedro convexo P tem 26 vrtices, 60 arestas e 36 faces. 24
faces so triangulares e 12 so quadrilteros. Uma diagonal espacial
um segmento de reta unindo dois vrtices no pertencentes a uma
mesma face. P possui quantas diagonais espaciais?

( )
Soluo: Os 26 vrtices determinam exatamente 26
2 = 26 25/2 =
325 segmentos. Destes segmentos, 60 so arestas e como cada quadriltero tem duas diagonais, ento temos 12 2 = 24 diagonais que

Sugesto: Conte o nmero total


de segmentos determinados pelos
vrtices e retire os que no so diagonais espaciais.

. O nmero de
Fatos que Ajudam:
modos de escolher dois objetos
(n)
dentre n objetos distintos 2 =
n(n1)
.
2

Veja o quadro na pgina

118.

no so espaciais.
Portanto, o nmero de diagonais espaciais 325 60 24 = 241.

100 | Grade de Pontos


Uma grade de pontos com 10 linhas e 10 colunas dada. Cada ponto
colorido de vermelho ou de azul. Sempre que dois pontos da mesma
cor so vizinhos em uma mesma linha ou coluna, eles so ligados
por um segmento da mesma cor dos pontos. Se dois pontos so vizinhos mas de cores diferentes, so ligados por um segmento verde.
No total, existem 52 pontos vermelhos. Destes vermelhos, 2 esto
nos cantos e outros 16 esto no bordo da grade. Os outros pontos
vermelhos esto no interior da grade.

Sugesto: Conte o nmero total de


segmentos e conte o total de segmentos que partem de pontos ver.
melhos.
Fatos que Ajudam: De pontos
vermelhos no saem segmentos
azuis.

.
Existem 98 segmentos verdes. Determine o nmero de segmentos
azuis.

Soluo: Inicialmente, observe que existem 9 segmentos em cada


linha e em cada coluna, de modo que existem 9 10 + 9 10 = 180
segmentos no total.
Seja A o nmero de segmentos azuis e V o nmero de segmentos
vermelhos. Ento A + V + 98 = 180, de modo que A + V = 82, j que
existem 98 segmentos verdes.
Observe que dos pontos vermelhos, s podem partir segmentos vermelhos ou verdes. Vamos contar o total de segmentos que partem
dos pontos vermelhos. Neste total os segmentos verdes so contados exatamente uma vez e os segmentos vermelhos duas vezes, pois
os segmentos vermelhos ligam dois pontos vermelhos.
.
.

Partindo de um canto, existem 2 segmentos:

.
De um ponto sobre o bordo partem 3 segmentos

.
De um ponto interior partem 4 segmentos
www.obmep.org.br

.
OBMEP

146

Combinatria e Probabilidade | Nvel 3 | Solues

Ento, o nmero total de segmentos que partem dos vrtices vermelhos

2 2 + 3 16 + 4 34 = 188,
mas como 98 segmentos que partem dos pontos vermelhos so os
segmentos verdes, os restantes 188 98 = 90 so vermelhos e foram
contados duas vezes, de modo que V = 45.
Portanto, A = 82 V = 37.

www.obmep.org.br

OBMEP

26. Geometria
Nvel 3 Solues

101 | Tringulo 20 40 120


mede 20 e o ngulo ACB
mede
Num tringulo ABC, o ngulo ABC

40 . Seja E um ponto sobre BC tal que BE = BA.

Sugesto: Determine as medidas


. aparecem na
dos ngulos que
construo.

(a) Mostre que o tringulo CEA issceles.

2, de(b) Sabendo que o comprimento da bissetriz do ngulo BAC


termine BC AB.
Soluo:

= 180 20 40 = 120 . Como o tringulo ABE


(a) Temos CAB
issceles, segue que

= EAB
= 180 20 = 80 .
AEB
2

= 120 80 = 40 e o tringulo ACE tem dois


Assim, CAE
ngulos de 40 , e, portanto, issceles com CE = EA.
. A bissetriz divide o
(b) Seja D o p da bissetriz do ngulo BAC

ngulo CAB em dois ngulos de 60 . Logo, o ngulo

= 180 40 60 = 80 .
CDA
tambm mede 80 , temos que o tringulo ADE issComo AEB
celes. Finalmente,

40

20

E D

Figura 101.1

BC AB = BC BE = CE = EA = AD = 2.

Problema Relacionado
= 48 . Os pontos D e E esto sobre
O tringulo ABC issceles de base BC e BAC
. DCA
= 9 e EBC
= 33 .
os lados AB e AC, respectivamente, tais que
.
Determine a medida do ngulo CDE

147

148

Geometria | Nvel 3 | Solues

102 | Um Problema Antigo!


Sugesto: Utilize .o teorema de Pitgoras.

Duas torres, uma com 30 passos e a outra com 40 passos de altura,


esto distncia de 50 passos uma da outra. Entre ambas se acha
uma fonte, para a qual dois pssaros descem no mesmo momento do
alto das torres com a mesma velocidade e chegam ao mesmo tempo.
Quais as distncias horizontais da fonte s duas torres?(Leonardo de
Pisa, Liber Abaci, 1202).

D
Soluo:
40
30

50 x

Figura 102.1

Na figura, AD e BC representam as duas torres e o ponto E representa a posio da fonte. Como os dois pssaros chegam ao mesmo
tempo, temos que DE = EC.
Denotemos por x a distncia de A a E e assim EB = 50 x. Usando
o teorema de Pitgoras nos tringulos DAE e EBC, temos que

{
DE2 = 302 + x2
EC2 = 402 + (50 x)2 .
Como DE = EC, temos:

900 + x2 = 1600 + 2500 100x + x2 x = 3200/100 = 32.


Portanto, as distncias horizontais da fonte s duas torres so AE =
x = 32 passos e EB = 50 x = 18 passos.

103 | Circunferncias Tangentes


Sugesto: Trabalhe os ngulos
dos tringulos issceles AO1 C e
BO2 C.

As circunferncias C1 e C2 so tangentes reta nos pontos A e B e


tangentes entre si no ponto C. Prove que o tringulo ABC retngulo.

Fatos que Ajudam: Dadas duas


circunferncias tangentes, o ponto
de tangncia e os dois centros pertencem a uma mesma reta.

O2
O1

C
.

Figura 103.1

O2
O1

C
.

Figura 103.2

www.obmep.org.br

Soluo: Como as circunferncias so tangentes, ento o ponto de


tangncia C e os centros O1 e O2 pertencem a uma mesma reta. Alm
disso, como as circunferncias so tangentes a , ento O1 A e O2 B
so perpendiculares a e, portanto, paralelas.
e a medida do ngulo O2 CB
.
Seja a medida do ngulo O1 CA

Como os tringulos AO1 C e BO2 C so issceles, segue que CAO1 =


2 = .
e CBO
1 C+BO
2 C = 180 ,
Como as retas O1 A e O2 B so paralelas, temos AO

donde 180 2 + 180 2 = 180 . Portanto, + = 90 .


= 180 ( + ) = 90 .
Assim, ACB

OBMEP

Geometria | Nvel 3 | Solues

149

104 | Tringulo Issceles II


b = 30 . Seja D
Seja ABC um tringulo issceles com AB = AC e A
o ponto mdio da base BC. Sobre AD e AB tome dois pontos P e Q,
respectivamente, tais que PB = PQ. Determine a medida do ngulo
b .
PQC
Soluo: Observemos que

Sugesto: Mostre que os ngulos


e ACP
somam 180 .
AQP

. Um quadrilFatos que Ajudam:


tero inscritvel se a soma dos ngulos opostos 180 . ngulos inscritos no mesmo arco so iguais.

= ACB
= 180 30 = 75 .
ABC
2
Como todos os pontos da altura AP esto mesma distncia de B e
de C, em particular, o tringulo BPC issceles com BP = PC. Pela
hiptese do problema, o tringulo BPQ tambm issceles. Denote , assim BCP
=e
mos por a medida do ngulo P BC

A.
30

= 180 BQP
= 180 QBP
= 180 (75 ) = 105
AQP
e

= 75 PCB
= 75 .
PCA

+ PCA
= 180 , portanto o quadriltero AQPC inscritAssim AQP
= PAC
= 15 .
vel, em particular P QC

105 | Circunferncia no Setor

D
Figura 104.1

Uma circunferncia de raio r est inscrita em um setor circular de


raio R. O comprimento da corda AB igual a 2a.

2a
R
A
Figura 105.1

Prove que

Sugesto: Ligue o centro da circunferncia inscrita no setor ao ponto


de tangncia desta com o raio do
setor circular. Procure tringulos
semelhantes.
Fatos que Ajudam: Se duas circunferncias so .tangentes, ento
o ponto de tangncia e os centros
das circunferncias so colineares.
Se uma reta tangente a uma circunferncia, ento o segmento que
une o centro da circunferncia ao
ponto de tangncia perpendicular reta.

1
1
1
= + .
r
R a

Soluo:

Denotemos por D o ponto de tangncia de AO com a


1 = 90 . Observe tambm que AC =
circunferncia. Ento ODO
AB/2 = a.

= 90 . Os tringulos ODO1 e OCA so semePor outro lado, OCA


lhantes pois possuem um ngulo comum e um ngulo reto. Portanto,
O1 D
OO1
=
,
OA
AC

O1 C

O.

D
Figura 105.2

isto ,

Rr
r
= ,
R
a
donde

www.obmep.org.br

1
1
1
+ = .
a R
r
OBMEP

150

Geometria | Nvel 3 | Solues

106 | Mais Circunferncias Tangentes


Sugesto: (a) Trace uma reta pelo
centro da menor circunferncia,
paralela reta .
Fatos que Ajudam: Se duas circunferncias so tangentes, ento
.
o ponto de tangncia
e os centros
das circunferncias so colineares.
Se uma reta tangente a uma circunferncia, ento o segmento que
une o centro da circunferncia ao
ponto de tangncia perpendicular reta.

(a) Duas circunferncias de raios R e r so tangentes externamente


(figura 106.1). Demonstre que o segmento
determinado pela tan
gente comum externa mede d = 2 Rr.

O2
r

1
1
1
= + .
x
r
R

Figura 106.2

(b) Considere, como ilustrado na 106.2, as trs circunferncias de


raios R, r e x, tangentes duas a duas e tangentes reta . Mostre
que

R+r
Rr

Figura 106.1

O1

B
Figura 106.3

Soluo: Sejam O1 e O2 os centros das circunferncias e A e B os


pontos de tangncia com a reta , conforme ilustrado na figura 106.3.
(a) Seja P o ponto sobre O1 A tal que PO2 paralelo a AB. Como
PO2 BA um retngulo, ento o tringulo O1 PO2 retngulo em
P. Assim, pelo teorema de Pitgoras temos que

O1
O2

AB2 = PO22

r
.

= (O1 O2 )2 (O1 P)2


= (R + r)2 (R r)2 = 4Rr.

Portanto, AB = 2 Rr.

Figura 106.4

(b) Seja C o ponto de tangncia da terceira circunferncia com a reta.


Pelo item (a), sabemos que

AC = 2 Rx, CB = 2 xr e AB = 2 Rr.

Segue que 2 Rr = 2 Rx+2 xr, que dividindo por 2 Rrx, obtmse

1
1
1
= + .
x
r
R

s
C3

C2

Problema Relacionado

C1

.
Figura 106.5

www.obmep.org.br

A figura 106.5 mostra duas retas paralelas r e s. A reta r tangente s circunferncias


C1 e C3 , a reta s tangente s circunferncias
C2 e C3 e as circunferncias tocam-se
.
como tambm mostra a figura. As circunferncias C1 e C2 tm raios a e b, respectivamente. Qual o raio da circunferncia C3 ?

OBMEP

Geometria | Nvel 3 | Solues

151

107 | Reta Equilibrada


Seja ABC um tringulo tal que AB = 55, AC = 35 e BC = 72. Considere uma reta que corta o lado BC em D e o lado AC em E e
que divide o tringulo em duas figuras com permetros iguais e reas
iguais. Determine a medida do segmento CD.

Soluo:

Sugesto:
Calcule a rea do
CED, a qual metade da rea
do ABC.
Fatos que Ajudam: A rea S de
um tringulo que possui dois lados
de medidas a e b e estes determinam um ngulo pode ser calculada pela frmula

S=

55

Demonstrao: A rea do tringulo da figura 107.1 ah/2, mas


h = b sen .
.

y
z
.

ab sen
.
2

Figura 107.2

b
.

Sejam CD = x, CE = y e DE = z.
(1) Como o tringulo CED tem o mesmo permetro do quadriltero
ABDE, temos

x + y + z = (35 y) + z + (72 x) + 55 y = 81 x.

Figura 107.1

Ento,

ah
ab sen
=
.
2
2

(2) Como eles tambm possuem a mesma rea, a rea do tringulo


DCE deve ser igual metade da rea do tringulo ABC. Deste
modo,

xy sen C
1 35 72 sen C
=
xy = 1260.
2
2
2
Utilizando as duas equaes encontradas obtemos x2 81x + 1260 =
0. Resolvendo esta equao, chegamos em x = 60 ou x = 21. No
primeiro caso obtemos y = 21 e no segundo y = 60. Como E est
sobre o lado AC, devemos ter y 35 e ento a soluo que nos
interessa x = 60 e y = 21. Portanto, CD = 60.

www.obmep.org.br

OBMEP

152

Geometria | Nvel 3 | Solues

108 | Alturas e Pontos Mdios


Sugesto: Mostre que os tringulos BME e HEN so issceles.
Fatos que Ajudam: O ortocentro
de um tringulo o ponto de interseco das alturas. Em um tringulo retngulo, a mediana relativa a hipotenusa tem comprimento igual a metade da hipotenusa.

O tringulo acutngulo ABC de ortocentro H tal que AB = 48


e HC = 14. O ponto mdio do lado AB M e o ponto mdio do
segmento HC N.

reto.
(a) Mostre que o ngulo MEN
(b) Determine o comprimento do segmento MN.

E
M

F
H

Figura 108.1

C
Figura 108.2

Soluo: Inicialmente observe que ME mediana relativa hipotenusa do tringulo AEB. Portanto, ME = AM = MB = 24. Desse
= MBE
= .
fato segue que o tringulo BME issceles. Ento MEB
Analogamente, como N o ponto mdio da hipotenusa do tringulo
HEC, temos EN = HN = NC = 7 e o tringulo HNE issceles.
= EHN
= .
Assim, HEN
+ HBF
= + = 90 .
O tringulo FHB retngulo em F e FHB
Assim, o tringulo MEN retngulo em E. Aplicando o teorema de

A
E
M
F
H

Pitgoras neste tringulo, obtemos

MN2 = ME2 + EN2

MN2 = 242 + 72 = 625,

Figura 108.3

donde MN = 25.

www.obmep.org.br

OBMEP

Geometria | Nvel 3 | Solues

153

109 | Proibido usar Rgua!


(a) Sejam C uma circunferncia com centro O e raio r e X um ponto
exterior a C . Construmos uma circunferncia de centro em X passando por O, a qual intersecta C nos pontos P e Q. Com centro
em P construmos uma circunferncia passando por O e com centro em Q construmos uma outra circunferncia passando por O.
Estas duas circunferncias intersectam-se nos pontos O e Y .

C
O.

Sugesto: (a) Mostre que os tringulos XOP e PYO so semelhantes. (b) Tente obter o ponto C construindo tringulos equilteros. (c)
Utilize os itens (a) e (b).
Fatos que Ajudam: Dados dois
pontos D e E, podemos construir
um ponto F, utilizando somente
compasso, tal que o DEF seja
equiltero. O ponto F pode ser
obtido como um dos dois pontos
.
de interseo da circunferncia de
centro em D que contm E e da
circunferncia de centro em E que
contm D.

F
Q
Figura 109.2

Prove que OX OY = r2 .

Figura 109.1

(b) dado um segmento AB. Mostre como construir, usando somente compasso, um ponto C tal que B seja o ponto mdio do
segmento AC.

(c) dado um segmento AB. Mostre como construir, usando somente compasso, o ponto mdio do segmento AB.

O.

Soluo:

Q
(a) Observe que os tringulos XOP e PYO so ambos issceles, de
bases OP e YO, respectivamente. Estes tringulos possuem n = Y OP

gulos da base de mesma medida, pois o ngulo P OX
comum aos dois tringulos. Deste modo, os tringulos XOP e
PYO so semelhantes e podemos escrever OX/OP = OP/OY , e,
como OP = r, conclumos que OX OY = r2 .

Figura 109.3

A
(b) Determinamos um ponto R tal que o tringulo ABR seja equiltero. Em seguida, determinamos um ponto S = A de modo que o
tringulo RBS seja equiltero e construmos C = R de forma que o
tringulo BSC tambm seja equiltero. Assim, BC = BS = BR =
= 60 + 60 + 60 = 180 ),
AB e A, B e C so colineares (ABC
logo B o ponto mdio de AC.
(c) Seja M o ponto mdio de AB. Construa a circunferncia com centro em A e raio r = AB. Como no item anterior, com o compasso
construmos um ponto C tal que B o ponto mdio de AC.

Figura 109.4

Q
Figura 109.5

Observe que AM AC = (r/2) 2r = r2 e, portanto, podemos construir o ponto M utilizando o processo de construo do
item (a): determinamos os pontos P e Q, pontos de interseo da
circunferncia de centro C que contm A e da circunferncia de
centro A que contm B. O ponto M obtido pela interseo das
circunferncias de centros P e Q que passam por A.
www.obmep.org.br

OBMEP

154

Geometria | Nvel 3 | Solues

Problema Relacionado

.
dada uma circunferncia C . Construir, usando
somente compasso, o centro de C .

110 | Ps das Perpendiculares


Sugesto: Mostre que os tringulos BEF e BCD so semelhantes.
Fatos que Ajudam: Sejam X, B e C
=
pontos no plano tais que BXC

90 .

Seja ABC um tringulo acutngulo com alturas BD e CE. Os pontos


F e G so os ps das perpendiculares BF e CG a reta DE. Prove que
EF = DG.

Soluo:

A
.

Figura 110.1

Ento o ponto X est sobre a circunferncia de dimetro BC.

X
.

Figura 110.4

e DEC
possuem a mesma medida, pois ambos so o
Os ngulos FBE
.
complemento do ngulo FEB

Figura 110.2

Se Y outro ponto qualquer do


= CBY
, porarco XC, ento CXY
que estes ngulos medem a metade do arco YC.

Observe que o quadriltero BCDE inscritvel. De fato, a circunfe = BDC


= 90 .
rncia de dimetro BC contm E e D, pois BEC

= DEC
= DBC
.
Segue que FBE
Portanto, BEF BCD e obtemos

Y
.

EF
BE
BE DC
=
= EF =
.
DC
BC
BC
Analogamente, o tringulo CDG semelhante ao tringulo CBE, donde
obtemos

Figura 110.3

DG =

DC BE
,
BC

e segue que EF = DG.

www.obmep.org.br

OBMEP

27. Diversos
Nvel 3 Solues

111 | Jogo Triangulrio


Um jogo solitrio realizado em um tabuleiro no formato de tringulo equiltero, mostrado na figura 111.1. Sobre cada crculo colocase uma ficha. Cada ficha branca de um lado e preta do outro. Inicialmente, s a ficha que est situada em um vrtice tem a face preta
para cima e as outras fichas tm a face branca para cima. Em cada
movimento, retira-se uma ficha preta do tabuleiro e cada uma das
fichas que ocupam um crculo vizinho ficha retirada so viradas.
Crculos vizinhos so os que esto unidos por um segmento.

Sugesto: Observe que para uma


ficha poder ser retirada ela teve
. nmero mpar
que ser virada um
de vezes, e todos os crculos tm
um nmero par de vizinhos.

.
Figura 111.1

Aps vrios movimentos, ser possvel tirar todas as fichas do tabuleiro?

Soluo: Suponha que seja possvel remover todas as fichas do tabuleiro e vejamos a ltima ficha removida. Ela deve ser preta para
que possamos remov-la, mas tambm preciso que todas as fichas
vizinhas tenham sido removidas. Como no tabuleiro, cada crculo
tem um nmero par de vizinhos, a ltima ficha trocou de cor um nmero par de vezes. Logo, ela era inicialmente preta. Mas no incio do
jogo, h somente uma ficha preta e o primeiro movimento do jogo
foi remov-la, o que absurdo.
Portanto, no possvel remover todas as fichas do tabuleiro.

112 | Bolas nas Caixas


Duas caixas contm juntas 65 bolas de vrios tamanhos. Cada bola
branca, preta, vermelha ou amarela. Cada vez que pegamos cinco
bolas da mesma cor, pelo menos duas so do mesmo tamanho.
(a) Qual o nmero mximo de tipos de bolas que existem nas caixas? Duas bolas so consideradas de tipos distintos quando tm
diferentes cores ou tamanhos.
(b) Mostrar que existem pelo menos trs bolas, que esto na mesma
caixa, e que so do mesmo tipo.
155

Sugesto: Existem no mximo 4


. de bolas para
tamanhos distintos
cada cor.

156

Diversos | Nvel 3 | Solues

Soluo:
(a) No podem existir cinco bolas da mesma cor e tamanhos diferentes porque cada vez que pegamos cinco bolas da mesma cor, duas
devem ser do mesmo tamanho. Assim, existem no mximo quatro tamanhos para cada cor. Logo, existem no mximo 4 4 = 16
tipos de bolas.
(b) As duas caixas possuem juntas, 65 bolas e uma delas deve conter
no mnimo 33 bolas. Por outro lado, existem no mximo 16 tipos
de bolas e como 2 16 = 32 < 33, conclumos que essa caixa
contm trs ou mais bolas do mesmo tipo.

113 | Fraes Irredutveis


Sugesto: Sendo a/600 e b/700 as
duas fraes, verifique quais fatores o numerador e o denominador
da soma podem ter em comum.

Fatos que Ajudam: Uma frao


dita irredutvel se o numerador e o
denominador no possuem fatores
primos em comum.

Duas fraes irredutveis tm seus denominadores iguais a 600 e 700.


Encontrar o valor mnimo para o denominador da soma das fraes.

Soluo: Suponhamos que as fraes so a/600 e b/700. Como so


irredutveis, ento a e 600 no tm fator comum maior que 1 e o
mesmo acontece com b e 700.
Somando as duas fraes obtemos

a
b
7a + 6b
7a + 6b
+
=
= 3
.
600 700
4200
2 3 52 7
Observe que o numerador no divisvel nem por 2 e nem por 3,
porque a no tem fator comum com 6. O numerador tambm no
divisvel por 7 porque b e 7 no tm fator comum.
Assim, o nico fator do denominador que possivelmente podemos
simplificar 52 = 25. Para isto basta pegar, por exemplo, a = 1 e
b = 3.

1
3
1
+
=
.
600 700
168

Portanto, o denominador mnimo da soma 168.

114 | Soma das Quintas Potncias


Sugesto: Observe que os valores
particulares de x1. , x2 , . . . , xn no
so importantes e sim a quantidade destes que so iguais a 1 e 2.

Seja x1 , x2 , . . . , xn uma sequncia na qual cada termo 0, 1 ou 2.


Se
{

x1 + x2 + + xn = 5
x21 + x22 + + x2n = 19

determine x51 + x52 + + x5n .

Soluo: Sejam a a quantidade de termos iguais a 1 e b a quantidade


de termos iguais a 2. Podemos escrever:

{
a 1 + b (2) = 5
a 12 + b (2)2 = 19

a 2b = 5
a + 4b = 19.

Resolvendo o sistema, obtemos a = 3 e b = 4. Logo,

x51 + x52 + + x5n = a 15 + b (2)5 = 3 4 32 = 125.

www.obmep.org.br

OBMEP

Diversos | Nvel 3 | Solues

157

115 | Comendo Pizzas


Um grupo de meninos e meninas se rene para comer pizzas que so
cortadas em 12 pedaos. Cada menino pode comer 6 ou 7 pedaos e
cada menina pode comer 2 ou 3 pedaos. Sabemos que quatro pizzas
nunca so suficientes para alimentar o grupo e que com cinco pizzas
sempre h sobra. Quantos meninos e quantas meninas formam o
grupo?

Sugesto: Analise a quantidade


mnima e mxima. de pedaos que
o grupo pode comer.

Soluo: Chamemos de x o nmero de meninos e de y o nmero de


meninas. Pelas condies do problema sabemos que se eles comem o
mnimo possvel, ainda assim quatro pizzas no so suficientes, isto
,

6x + 2y > 4 12 = 48.
Por outro lado, se eles comem o mximo possvel, com cinco pizzas
sobrar, isto ,

7x + 3y < 5 12 = 60.
Assim, precisamos encontrar dois nmeros naturais x e y que satisfaam simultaneamente

3x + y > 24
7x + 3y < 60.

Como 7x 7x + 3y < 60, x < 60/7 < 9, logo o nmero de meninos


menor ou igual a 8.

Por outro lado, como x e y so inteiros, ento 3x + y 25 > 24,


multiplicando por 3, obtemos 9x + 3y 75, e como 7x 3y > 60,
somando estas duas desigualdades (as duas tm o mesmo sentido),
encontramos que 2x > 75 60 = 15, ou x > 7, 5. Portanto, o nmero
de meninos 8.

6
5
4
3

Substituindo x = 8 nas desigualdades obtemos y > 0 e 3y < 4, que


tem como nica soluo y = 1. Assim, o grupo tem oito meninos e
uma menina.

Comentrio: O problema tambm pode ser resolvido geometrica.


mente. A soluo o nico ponto com coordenadas inteiras que 1est2
no interior da regio delimitada pelo eixo x e pelas retas 3x + y = 24
e 7x + 3y = 60. A figura 115.1 ilustra a situao.

www.obmep.org.br

2
1
3

Figura 115.1

OBMEP

28. Desafios

Nvel 3 Solues

116 | Quatro Cores no Tabuleiro


Considere o tabuleiro 9 9 mostrado abaixo. As linhas esto numeradas de 1 a 9.
Linha

Sugesto: Para o item (b), verifique


quantas casas de. cada cor so cobertas ao colocar uma pea no tabuleiro.

8
7
6
5
4
3
2
1

.
Figura 116.1

Colorimos as casas das linhas mpares do tabuleiro com as cores azul


e branco, alternadamente, comeando com azul e pintamos as casas
das linhas pares do tabuleiro de cinza e vermelho, alternadamente,
comeando com a cor cinza.
(a) Quantas casas foram pintadas com cada cor?

.
(b) Qual o nmero mximo de peas da forma
ser colocadas, sem sobreposio, nesse tabuleiro?

que podem
.
Figura 116.2

Soluo:
(a) Cada linha mpar contm 5 casas azuis e 4 casas brancas. Como o
tabuleiro tem 5 linhas mpares, o nmero de casas azuis 5 5 =
25 e o nmero de casas brancas 5 4 = 20.
Do mesmo modo, cada linha par tem 5 casas cinzas e 4 casas
vermelhas e o tabuleiro tem 4 linhas pares. Assim, o nmero de
casas cinzas 4 5 = 20 e o nmero de casas vermelhas 4 4 =
16.
.
(b) No importa como coloquemos a pea
, ela sempre vai
cobrir uma casa de cada cor no tabuleiro. Como o tabuleiro tem
apenas 16 casas vermelhas, o nmero de peas tem que ser menor
ou igual a 16.
Exibimos na figura 116.2 uma configurao com exatamente 16
peas.

159

Problema Relacionado
possvel dividir um
. tabuleiro 89 em
retngulos 1 6?

160

Desafios | Nvel 3 | Solues

117 | Nmeros no Tabuleiro 8 x 8


Sugesto: Veja o problema Nmeros no Tabuleiro.4 4, do nvel 1,
na pgina 100.

Guilherme escreveu um nmero em cada casa de um tabuleiro 8 8


de modo que a soma dos nmeros das casas vizinhas de cada casa
do tabuleiro igual a 1. Calcule a soma de todos os nmeros escritos
por Guilherme.
Observao: duas casas so vizinhas se possuem um lado em comum.

Soluo: Numere as casas do tabuleiro conforme mostrado na figura


117.1.
A soma dos nmeros das casas marcadas com um mesmo nmero
igual a 1, porque elas so as vizinhas a uma determinada casa.

9 10

6 11 10 9

6 11 12 11 10

5 18 17 12 11 12 13

5 18 17 18 17 12 13 14
19 20 18 17 16 15 14 13
. 20 19 20 16 15 16 15 14
Figura 117.1

Logo, a soma de todos os nmeros do tabuleiro igual a 20.

118 | Formigas Geomtricas!


Sugesto: Analise a rea do tringulo determinado pelas posies
das formigas.
Fatos que Ajudam:
. A rea de um
tringulo no muda quando um
dos vrtices se movimenta sobre
uma reta paralela reta formada
pelos outros dois vrtices.

Trs formigas esto paradas em trs dos quatro vrtices de um retngulo no plano. As formigas se movem no plano uma por vez. A
cada vez, a formiga que se move o faz segundo a reta paralela determinada pelas posies das outras duas formigas. possvel que,
aps alguns movimentos, as formigas se situem nos pontos mdios
de trs dos quatro lados do retngulo original?

Soluo: Observe que, se uma formiga A se movimenta sobre uma


reta paralela reta determinada pelas outras duas formigas B e C,
ento a rea do tringulo com vrtices sobre as trs formigas invariante, j que a base BC e a medida da altura do tringulo com relao
ao lado BC no mudam.
.
Figura 118.1

Inicialmente, a rea do tringulo ABC a metade da rea do retngulo. Porm, se as formigas conseguissem chegar aos pontos mdios,
a rea determinada por elas seria 1/4 da rea do retngulo.
Como a rea no a mesma, impossvel que as formigas se situem
nos pontos mdios dos lados do retngulo, a partir da configurao
inicial.

.
Figura 118.2

www.obmep.org.br

OBMEP

Desafios | Nvel 3 | Solues

161

119 | Ponto no Interior do Quadrado


P um ponto no interior do quadrado ABCD tal que PA = 1, PB = 2
?
e PC = 3. Qual a medida do ngulo APB
A.

Sugesto: Determine um ponto Q


exterior ao quadrado, tal que o tringulo APB seja congruente ao tringulo CQB.

Fatos que Ajudam: Se a, b e c so


as medidas dos lados de um tringulo e a2 = b2 +c2 , ento o ngulo
oposto ao lado de medida a reto.

1
P
3

A.

2
B

D
1

P
3

Figura 119.1

Soluo: Seja Q um ponto tal que os tringulos CQB e APB so


congruentes, como mostrado na figura. Isto equivalente a fazer
uma rotao do tringulo APB com centro em B e ngulo 90 no sen
2

tido horrio. Em particular, temos que


PBQ = 90 . Assim, PQ =
2
2
2
2
PB + BQ = 2 + 2 , donde PQ = 2 2.

Figura 119.2

Por outro lado,

PC2 = 9 = 8 + 1 = PQ2 + QC2


e segue que o tringulo PCQ retngulo com ngulo reto em Q.

= BQC
= BQP
+ PQC
= 45 + 90 = 135 .
Portanto, APB

Problema Relacionado
Seja P um ponto no interior do tringulo equiltero ABC tal que:

PA = 5,

PB = .7,

PC = 8.

Determine a medida do lado do tringulo ABC.

www.obmep.org.br

OBMEP

162

Desafios | Nvel 3 | Solues

120 | Pontos no Interior do Disco


Sugesto: Para o item (b), ordene
os pontos de coordenadas inteiras
em
ordem crescente de distncia a
( 2, 1/3).

(a) Mostre que no existem dois pontos com coordenadas inteiras


no
plano cartesiano que esto igualmente distanciados do ponto
( 2, 1/3).

Fatos que Ajudam: A distncia entre os pontos (x1., y1 ) e (x2 , y2 )


dada pela expresso

(b) Mostre que existe um crculo no plano cartesiano que contm exatamente 2011 pontos com coordenadas inteiras em seu interior.

(x1 x2 )2 + (y1 y2 )2 .
O produto de um nmero racional
no nulo por um nmero irracional
um nmero irracional.

Soluo:
(a) Suponhamos que os (a, b) e (c, d) so pontos com coordenadas

inteiras que esto igualmente distanciados do ponto ( 2, 1/3).


Assim,

(
)2

1
2
(a 2) + b
=
3

(
)2

1
2
(c 2) + d
.
3

Deste modo,

a2 + b2 c2 d2

2b 2d
+
= 2 2(a c).
3
3

Como a parte esquerda desta igualdade racional, devemos ter


a c = 0 e consequentemente

a2 + b2 c2 d2

2b 2d
+
= 0.
3
3

Portanto,

(
)
2b 2d
2
b d
+
= (b d) b + d
= 0,
3
3
3
2

P3

( 2, 1/3)

P5

P1

P4
Figura 120.1

www.obmep.org.br

P2

P6

e como b + d 2/3 = 0, segue que b d = 0, isto (a, b) e (c, d)


so o mesmo ponto.
(b) Pelo item (a), no existem
dois pontos de coordenadas inteiras

mesma distncia de ( 2, 1/3). Podemos ento ordenar estes


pontos em ordem estritamente crescente de distncias
a
(
2, 1/3).

Assim, sendo di a distncia


do
i
-simo
ponto
P
a
(
2,
1/3)
, a ciri

cunferncia de centro ( 2, 1/3) e raio r, com d2011 < r < d2012 ,


possui exatamente 2011 pontos de coordenadas inteiras em seu
interior.

OBMEP

Origem dos Problemas


1. Mltiplo de 9 com Algarismos Pares Olimpada de Matemtica do Reino Unido Junior 1989
3. Calculadora Quebrada Problems to Solve in Middle School Mathematics AMT.
4. Loja em Quixajuba Problemas Olimpada Matemtica Argentina volume 15. Red Olimpica. Buenos
Aires.

5. Nmeros Sortudos Olimpada Peruana de Matemtica 2007.


7. Menor Soma Positiva Olimpada Peruana de Matemtica 2007.
8. Mdia dos Algarismos Adaptado da Olimpada Ucraniana de Matemtica 2006.
10. Estrelas em Geometrix Olimpada Portuguesa de Matemtica 2011.
11. Bandeira do Tio Man Adaptado da XXII Olimpada Portuguesa de Matemtica 2003.
14. Azulejos de Pedro Adaptado dos Maths Challenge for Young Australians Junior, 1994.
15. Retngulo 9 x 4 Adaptado do Maths Challenge for Young Australians Junior, 1996.
17. Tangram Primary Mathematics World Contest 2008.
20. Construindo uma Pipa Olimpada Portuguesa de Matemtica 2004.
21. Colorindo Mapas Olimpada Paulista de Matemtica 1986.
22. De Coco da Selva a Quixajuba Primary Mathematics World Contest.
23. O Baralho de Joo Primary Mathematics World Contest 2006.
25. Distribuindo Mas Primary Mathematics World Contest.
26. Maria e seus Convidados Banco de Problemas da Olimpada de Matemtica do Cone Sul 1998.
27. Cartes de Apostas Olimpada Rioplatense de Matemtica
30. Herana para Cinco Filhos Olimpada de Matemtica de Moscou Fase Distrital 2001.
31. Vizinhos e Distantes Olimpada de Leningrado 1988.
32. Truque com Cartas Torneio Internacional das Cidades 2007.
33. Campeonato de Quixajuba Adaptado da Gauss Contest (Canad) 1999.
34. Tabuleiro 6 x 6 Olimpada de Moscou 2011.
36. Contando Quadrados Adaptado da Gauss Contest (Canad) 2000.
39. Dividindo um Retngulo Adaptado da Olimpada de Matemtica de Leningrado 1990.

40. Nmeros no Tabuleiro 4 x 4 Torneio Internacional das Cidades.


46. Quantas Fraes! Torneio Internacional das Cidades.
51. Colar de Ouro Olimpada Blgara de Matemtica.
52. AP x BN EduCabri Clase 7 Olimpada Matemtica Argentina.
55. Bissetrizes Olimpada Matemtica Argentina 2007.
56. ngulos e ngulos! Olimpada Peruana de Matemtica 2007.
57. Quadrado, Pentgono e Icosgono UK Junior Math Olympiad 2010.
58. Enegono Regular The Constest Problem Book IV The Mathematical Association of America. Adaptado do Problema 30 do Exame de 1977.

62. Comparando Sequncias Olimpada de Matemtica de Leningrado 1998.


64. Esqueleto do Cubo Problems to Solve in Middle School. AMT.
65. Placas das Bicicletas Olimpadas Colombianas de Matemtica 1999.
66. Torneio de Tnis O problema relacionado da OBMEP 2009, primeira fase.
68. Produto 2000 Olimpada Peruana de Matemtica 2007.
69. Tabuleiro 123 x 123 Olimpada Peruana de Matemtica 2005.
70. Nmeros no W Problemas Olimpada Matemtica Argentina volume 15
71. Montando Tabelas Olimpada Paulista de Matemtica 2010.
73. Corrida de So Paulo a Fortaleza Olimpada Matemtica Rioplatense
74. Casas Pretas e Brancas Olimpada Alagoana de Matemtica 2007.
75. Ora Bolas! Torneio Internacional das Cidades.
76. Distncia entre os Vilarejos Crculos Matemticos A Experincia Russa. IMPA 2010.
77. Amigos que voc pode Contar! Adaptado da Olimpada Rioplatense de Matemtica 1997.
79. Contando Polgonos Olimpada Rioplatense de Matemtica 1998.
80. Desafiando os Amigos! O problema relacionado da Gauss Contest (Canad) 2009.
81. Sequncia Numrica II Olimpada Peruana de Matemtica 2007. O problema relacionado da
OBMEP 2007, segunda fase, nvel 3.

87. Sistema com 7 Variveis A parte (b) da AIME 1989.


89. Maior Divisor mpar Torneio Internacional das Cidades.
92. Subconjuntos com Soma Grande Olimpada Peruana de Matemtica 2004.
96. Aparando um Poliedro Torneio Internacional das Cidades.
97. Bolas Azuis e Vermelhas O problema relacionado Torneio de Xadrez da Olimpada Brasileira de
Matemtica 1992.

98. Dez Pontos no Plano AIME

101. Tringulo 20 40 120 Adaptado do Canguru Sem Fronteiras 2009.


104. Tringulo Issceles II Problemas 19 Olimpada Matemtica Argentina.
106. Mais Circunferncias Tangentes O problema relacionado da Olimpada Brasileira de Matemtica
2003, primeira fase, nvel 3.

110. Ps das Perpendiculares Competio entre a ustria e a Polnia.


111. Jogo Triangulrio Olimpada Espanhola de Matemtica 1999.
113. Fraes Irredutveis Olimpada Russa de Matemtica 2009.
115. Comendo Pizzas Olimpada Espanhola de Matemtica 2000.
116. Quatro Cores no Tabuleiro Desafio da Real Sociedad Matemtica Espaola.
118. Formigas Geomtricas! EduCabri Clase 7 Olimpada Matemtica Argentina.
119. Ponto no Interior do Quadrado O problema relacionado da Olimpada Iberoamericana de Matemtica 1995.

165

+ Desafios
Nvel 1
121. (Soma 91) A soma de treze inteiros positivos distintos igual a 92. Determine estes nmeros.

122. (Formando um Quadrado) Mostre como formar um quadrado utilizando quatro figuras idnticas
mostrada na figura abaixo.

.
Figura 122.1

123. (Outro Tabuleiro 6 x 6) Pinte de preto seis casas de um tabuleiro branco 6 6, de tal modo que
no seja possvel cortar um retngulo branco 1 6 ou um quadrado branco 3 3.

124. (Moeda Falsa) Temos 9 moedas, uma das quais falsa (ela mais leve do que as outras). Encontre
a moeda falsa utilizando duas pesagens em uma balana de pratos.

125. (Castelos do Rei) O rei pretende construir seis castelos em seu reino e ligar dois quaisquer deles
por uma estrada. Faa um diagrama dos castelos e das estradas de modo que elas se cruzem ao todo trs
vezes e exatamente duas estradas passem em cada cruzamento.

126. (Quadrado Perfeito?) A soma dos algarismos de um nmero igual a 2010. Este nmero pode
ser um quadrado perfeito?

127. (Batalha Naval) O campo do jogo Batalha Naval um tabuleiro 10 10, o qual contm um navio
oculto no formato de um retngulo 1 3. sempre possvel acertar o navio com at 33 tentativas?

128. (Sequncia Numrica III) O primeiro termo de uma sequncia 439 e cada termo, a partir do
segundo, igual soma dos algarismos do termo anterior, multiplicada por 13. Qual o 100o termo
desta sequncia?

129. (Polgono Legal) Um polgono legal se seus vrtices esto sobre uma grade retangular de pontos
e cada um de seus lados horizontal ou vertical. A distncia entre dois pontos vizinhos da grade 1 cm.
Por exemplo, o polgono da figura seguinte legal.

Figura 129.1

.
(a) Existe um polgono legal com permetro igual a 22 cm e rea igual a 14 cm2 ? Em caso afirmativo,
mostre um exemplo e caso contrrio justifique.
(b) Existe um polgono legal com permetro igual a 21 cm e rea igual a 14 cm2 ? Em caso afirmativo,
mostre um exemplo e caso contrrio justifique.

130. (Soma dos Algarismos)


(a) Existem dois nmeros naturais consecutivos tais que as somas de seus algarismos so ambas divisveis por 7?
(b) Existem dois nmeros naturais consecutivos tais que as somas de seus algarismos so ambas divisveis por 9?
Em ambos os casos, se a resposta for afirmativa, d um exemplo. Se a resposta for negativa, justifique.

Nvel 2
131. (Dobrando uma folha) Cristiane dobrou uma folha retangular de papel de tal modo que um
vrtice coincidiu com o ponto mdio de um lado, como indicado na figura 131.1. Ela descobriu que os
tringulos I e II so iguais.

II

I
Figura 131.1

Determine a medida do maior lado da folha, sabendo que o lado mais curto mede 8 cm.
168

132. (Pedro e Paulo) Pedro nasceu no sculo 19, enquanto seu irmo Paulo nasceu no sculo 20. Certa
vez os irmos se encontraram em uma festa comemorando o aniversrio de ambos. Pedro disse, Minha
idade igual soma dos dgitos do meu ano de nascimento. A minha tambm, respondeu Paulo.
Quantos anos Paulo mais jovem que Pedro?

133. (Sequncia Numrica IV) Uma sequncia numrica formada de acordo com a seguinte regra:
o primeiro nmero 7 e cada nmero, a partir do segundo, igual a soma dos dgitos do quadrado do
nmero anterior, aumentada em uma unidade. Por exemplo, o segundo nmero 14, porque 72 = 49 e
4 + 9 + 1 = 14. O terceiro nmero 17 e assim por diante. Qual o milsimo nmero da sequncia?

134. (Nmeros na Estrela) Escreva um dos nmeros de 1 a 12 em cada um dos doze tringulos equilteros pequenos da figura de modo que, em cada tringulo equiltero formado por quatro tringulos
pequenos, a soma dos nmeros escritos seja igual a 20.

Figura 134.1

135. (Contando de 1 a 1000) Sete estudantes contam de 1 a 1000 como segue:

Andr diz todos os nmeros, com exceo do nmero do meio em cada grupo de trs nmeros consecutivos. Isto , Andr diz 1, 3, 4, 6, 7, 9, . . . , 997, 999, 1000.
Bruno diz todos os nmeros que Andr no disse, exceto que ele tambm salta o nmero do meio em
cada grupo de trs nmeros consecutivos.

Clara diz todos os nmeros que Andr e Bruno no disseram, exceto que ele tambm salta o nmero
do meio em cada grupo de trs nmeros consecutivos.

Daniel, Estevo e Fbio dizem todos os nmeros que nenhum dos estudantes com o primeiro nome
comeando antes do seu no alfabeto disseram, exceto que eles tambm saltam o nmero do meio em
cada grupos de trs nmeros consecutivos.

Finalmente, Gabriel diz o nico nmero que ningum disse.


Que nmero Gabriel disse?

136. (Equipe de Natao) O treinador da equipe de natao decidiu organizar uma srie de competies
entre os 7 integrantes da equipe. Em cada dia ser realizado uma nica prova com a participao de trs
nadadores. Cada nadador competir exatamente uma vez com cada um dos outros.
(a) Quantos dias durar esta srie de competies? Explique ou justifique por que no pode durar nem
mais dias, nem menos dias que o nmero afirmado.
(b) Mostre uma possvel distribuio indicando os trs nadadores que competem em cada dia.
169

137. (Repartindo o Tesouro) A lei pirata estabelece que para repartir as moedas de um tesouro o
capito deve escolher um grupo de piratas e repartir igualmente as moedas entre estes at que no
possua moedas suficientes para dar uma a mais a cada pirata. As moedas que sobram so a parte do
capito.
O capito Morgan deve repartir um tesouro que contm menos de 1000 moedas de ouro. Ele sabe que
se escolhe 99 piratas ficar com 51 moedas e se escolhe 77 piratas cabero a ele apenas 29 moedas.
Determinar quantos piratas deve escolher Morgan para ficar com a maior quantidade de moedas, e para
essa quantidade de piratas, quantas moedas ele ganhar. Observao: cada pirata escolhido deve receber
pelo menos uma moeda.

138. (Verificando Moedas) Voc possui 6 moedas de pesos 1, 2, 3, 4, 5 e 6 gramas que parecem
iguais, exceto por seus rtulos que indicam o respectivo peso de cada uma. Como determinar se todas
as indicaes dos rtulos esto corretas, usando uma balana de pratos somente duas vezes?

139. (Bissetriz no Tringulo Retngulo) O ponto K marcado sobre a hipotenusa AB do tringulo


retngulo ABC de modo que CK = BC. O segmento CK divide a bissetriz interna AL em dois segmentos
de mesma medida (L um ponto do lado BC). Determine as medidas dos ngulos do tringulo ABC.

140. (Somando ngulos) Em uma folha quadriculada marcamos os pontos A, B, C, D, M e N, como


mostra a figura 140.1.

A
B
C
D

N
Figura 140.1

, MBN
, MCN
e MDN
igual a 45 .
Prove que a soma dos ngulos MAN

Nvel 3
141. (Sistema em Trs Variveis) Encontre todas as ternas (x, y, z) de nmeros reais que satisfazem
o sistema

x(x + y + z) = 26
y(x + y + z) = 27

z(x + y + z) = 28.

170

142. (Equilibrando Quadrados) Seguem alguns exemplos nos quais a soma dos quadrados de k nmeros positivos consecutivos igual soma dos quadrados dos k 1 inteiros seguintes:

32 + 42 = 52 ,
362 + 372 + 382 + 392 + 402 = 412 + 422 + 432 + 442 ,
552 + 562 + 572 + 582 + 592 + 602 = 612 + 622 + 632 + 642 + 652 .
Encontre uma frmula geral para todos os casos.

143. (Tringulo 30 60 90) O tringulo retngulo ABC tem ngulo reto em C e o ngulo A mede 30 .
O centro da circunferncia inscrita no tringulo ponto I e D o ponto de interseo desta circunferncia
com o segmento BI. Prove que os segmentos AI e CD so perpendiculares.

144. (Poligonal no Quadrado) No quadrado ABCD, a linha poligonal KLAMN tal que os ngulos
, LAM
e AMN
medem 45 .
KLA

K
M
45
.

N D
Figura 144.1

Demonstre que KL2 + AM2 = AL2 + MN2 .

145. (Dividindo em reas Iguais) Considere os pontos M e N sobre os lados BC e CD do quadrado


mede 45 .
ABCD, tais que o ngulo MAN

N
45
.

D
Figura 145.1

Prove que a diagonal BD divide o tringulo AMN em duas partes de mesma rea.

146. (Cortando um Hexgono) Existe um hexgono que pode ser dividido em quatro tringulos congruentes por um nico corte reto?
171

147. (Truque com Cartas II) Duas pessoas realizam um truque. A primeira retira 5 cartas de um
baralho de 52 cartas (previamente embaralhado por um membro da plateia), olha-as, e coloca-as em uma
linha da esquerda para a direita: uma com a face para baixo (no necessariamente a primeira), e as outras
com a face para cima. A segunda pessoa deve adivinhar a carta que est com a face para baixo. Prove
que elas podem combinar um sistema que sempre torna isto possvel.

148. (Bissetrizes) No tringulo ABC, o ngulo B mede 60 . Traamos as bissetrizes AD e CE, sendo
D um ponto do lado BC e E um ponto do lado AB. As bissetrizes intersectam-se no ponto I. Prove que
ID = IE.

149. (Ministros) Um pas tem 12 ministros. Cada ministro amigo de 5 ministros e inimigo dos outros
6. Cada comit formado por 3 ministros. Um comit considerado legtimo se todos os seus membros
so amigos ou se todos so inimigos. Quantos comits legtimos podem ser formados?

150. (Voc sabe? Ento eu tambm sei!) Uma professora de matemtica pensou em um inteiro
positivo de dois algarismos. Ela deseja que seus dois inteligentes alunos Daniela e Adriano determinem
o valor exato do nmero pensado.
Para tal, informa reservadamente a Daniela a quantidade de divisores positivos do nmero e confidencia
a Adriano a soma dos algarismos do nmero.
Uma breve conversa entre Daniela e Adriano transcrita abaixo:

Adriano: Eu no posso determinar o nmero.


Daniela: Nem eu, mas posso dizer se ele par ou mpar.
Adriano: Agora eu sei qual o nmero.
Daniela: Voc sabe? Ento eu tambm sei.
Suponha que os estudantes so honestos e existe lgica perfeita em tudo o que falaram. Determine o
nmero pensado pela professora justificando sua resposta.

172

EUREKA!
Edio Especial, 2007

Sociedade Brasileira de Matemtica

Diretor: Csar Camacho


Sociedade Brasileira de Matemtica
Presidente: Joo Lucas Marques Barbosa
Apoio:
Conselho Nacional de Desenvolvimento Cientfico e Tecnolgico - CNPq
Instituto do Milnio - Avano Global e Integrado da Matemtica Brasileira
Academia Brasileira de Cincias
Ministrio da Cincia e Tecnologia - MCT
Ministrio da Educao - MEC
Comisso Nacional de Olimpadas de Matemtica
Estrada Dona Castorina, 110. Jardim Botnico, Rio de Janeiro RJ, CEP: 22460-320
Telefone : (21) 25295077
Fax: (21) 25295023
e-mail: obm@impa.br
Home-page: www.obm.org.br
Coordenador: Edmilson Luis Rodrigues Motta
Membros da Comisso: Antonio Caminha Muniz Neto, Carlos Gustavo Tamm de Araujo Moreira,
Carlos Yuzo Shine, Eduardo Wagner, lio Mega, Florncio F. Guimares Filho, Luciano Guimares Monteiro de Castro, Luzinalva Miranda de Amorim, Nicolau Coro Saldanha, Onofre Campos da Silva Farias, Paulo Cezar Pinto Carvalho, Pablo Rodrigo Ganassim, Ralph Costa Teixeira,
Ronaldo Alves Garcia, Yoshiharu Kohayakawa.
Secretria Executiva: Nelly Carvajal Flrez.
Secretria Assistente: Sonia de Souza Silva de Melo.
Comit Editorial da EUREKA!
Antonio Luiz Santos
Elon Lages Lima
Nicolau Coro Saldanha
Paulo Cezar Pinto Carvalho
Sergio Plaza Salinas
Editor Responsvel
Paulo Cezar Pinto Carvalho
Editorao Eletrnica
Marcos Machado

Desenho da Capa
Daniel Assuno Andrade
Carolina Fontenelle de Mello e Souza

Tiragem
4000 exemplares

Postagem
Primeiro Semestre de 2007

EUREKA! Edio Especial OBMEP, 2007

ISSN 1415-479X

Os artigos assinados so da responsabilidade dos autores. permitida a reproduo de artigos,


desde que seja citada a fonte.

EUREKA! Edio Especial, 2007

Sociedade Brasileira de Matemtica

NDICE
Nmeros mgicos e contas de dividir

05

Dois problemas sobre grafos

08

Paridade

15

Adedanha ou De como os deuses trouxeram paz ao mundo

22

Quadrilteros e tringulos

29

Contar duas vezes para generalizar o retorno

33

Os nmeros irracionais

38

XXVII Olimpada Brasileira de Matemtica


Problemas e Solues da Primeira Fase

49

XXVII Olimpada Brasileira de Matemtica


Problemas e Solues da Segunda Fase

62

Olimpada de Maio - Problemas

85

Olimpada de Maio - Solues

88

100

Coordenadores Regionais OBM

EUREKA! Edio Especial, 2007

Sociedade Brasileira de Matemtica

Caros Leitores
Este nmero da revista Eureka! foi especialmente preparado para uso no estgio
dos alunos contemplados com bolsas de Iniciao Cientfica na Olimpada Brasileira de Matemtica das Escolas Pblicas (OBMEP).
A Eureka! a revista da Olimpada Brasileira de Matemtica (OBM) e distribuda gratuitamente a todas as escolas participantes. J foram publicados 25
nmeros da revista, o primeiro em maio de 1998. Cada edio contm artigos
em diversos nveis de dificuldade e provas de competies nacionais e internacionais. Para este nmero especial, selecionamos sete artigos publicados em
nmeros anteriores da Eureka! e as provas da 1a e 2a fases da OBM-2005. Publicamos tambm as provas de 2006 da Olimpada de Maio, que uma competio internacional promovida pela Olimpada de Matemtica Argentina, aberta
a todas as escolas que dela queiram participar.
A OBM, realizada desde 1979, a mais tradicional das competies matemticas brasileiras. Vrios pesquisadores brasileiros de destaque na Matemtica ou
em reas afins tiveram na participao na OBM um importante ponto de partida
em suas carreiras. Os resultados obtidos na OBM so tambm fator fundamental
na escolha das equipes que representam o Brasil nas principais competies internacionais de Matemtica (a Olimpada do Cone Sul, a Olimpada IberoAmericana e, a mais importante delas, a Olimpada Internacional de Matemtica, na qual estudantes brasileiros j obtiveram 7 medalhas de ouro). A OBM
disputada em trs fases, sendo a ltima de nvel comparvel ao das olimpadas
internacionais.
Convidamos os alunos bolsistas da OBMEP a participarem tambm da OBM,
mesmo que sua escola no esteja inscrita para participar. Em carter excepcional, os bolsistas da OBMEP podem participar a partir da 2 fase, com as provas
sendo aplicadas pelas coordenaes de estgio.
Tambm convidamos todos a visitar a pgina de Internet da OBM:
http://www.obm.org.br . Nela, podem ser encontrados todos os nmeros da revista Eureka!, as provas dos anos anteriores da OBM e das diversas competies internacionais e, esperamos, ainda mais razes para apreciar a Matemtica.

Os Editores
EUREKA! Edio Especial, 2007

Sociedade Brasileira de Matemtica

NMEROS MGICOS E CONTAS DE DIVIDIR


Carlos Gustavo Tamm de Arajo Moreira

Nvel Iniciante.
Temas muito inocentes de aritmtica bsica, como contas de multiplicar,
podem gerar resultados bastante interessantes e surprendentes, como ao
multiplicar o nmero 142857 por 2, 3, 4, 5, 6 e 7:
142857 2
142857 3
142857 4
142857 5
142857 6

=
=
=
=
=

285714
428571
571428
714285
857142

Por que razo acontece essa repetio dos dgitos de 142857 ao multiplic-lo por 2, 3, 4, 5 e 6, sempre com a mesma ordem circular? Ser mera
coincidncia? Ser possvel obter outros exemplos desse tipo?
A resposta tem a ver com o resultado de 142857 7, que 999999. Isso
quer dizer que o perodo da representao decimal de 1/7 exatamente
142857. Vamos examinar com cuidado a conta de diviso de 1 por 7:
10
30
20
60
40
50
1

7
0,142857

repetindo o resto 1, o que quer dizer que todo o processo se repete e o


resultado da diviso 1/7 = 0,142857142857142857
Podemos reescrever o processo assim:
1
10
30
20

=07+1
=17+3
=47+2
=27+6

EUREKA! Edio Especial, 2007

Sociedade Brasileira de Matemtica

60
=87+4
40
=57+5
50
= 7 7 + 1. Da temos:
10 7 1 = 3, e portanto 100-7 10 = 30, e como 30 7 4 = 2 temos:
100 7 (10 + 4) = 2, e analogamente obtemos:
1000 7 (100 + 40 + 2) = 6
10000 7 (1000 + 400 + 20 +8) = 4
100000 7 (10000 + 4000 + 200 + 80 + 5) = 5
1000000 7 (100000 + 40000 + 2000 + 800 + 50 + 7 ) = 1
( A ltima igualdade diz que 142857 7 = 999999)
Desta forma, os restos sucessivos que aparecem na diviso de 1 por 7,
que so 3, 2, 6, 4, 5, 1 so, respectivamente, os restos na diviso por 7 de
10, 100, 1000, 10000, 100000 e 1000000. Estes restos assumem todos os
valores possveis entre 1 e 6 e isso equivale ao fato de o perodo de 1/7
ter 6 casas. Desta forma, temos:
2 0,142857142857142857 = 2/7 = 100/714 = 100 0, 14285714
2857142857 14 = 0,285714285714285714, e, portanto, temos 2
142857 = 285714
Da mesma maneira temos que 3/7 = 10/7 1 implica 3 142857 =
428571, e as outras igualdades seguem de modo anlogo.
Notemos agora que sempre que o perodo da representao decimal de
1/n tiver n 1 casas decimais (que o mximo possvel), o perodo (que
ser igual a (10n-1 1) / n ) ter as mesmas propiedades de 142857. O
primeiro valor de n maior que 7 para o qual isso acontece 17, e o perodo de 1/17 0588235294117647. Multiplique esse nmero por 2, 3, 4,
5, 6, 7, 8, 9, 10, 11, 12, 13, 14, 15, 16 e 17 para conferir.
Observe que, para que isso acontea, n deve ser um nmero primo, pois
se n = p b, com b maior que 1 e p um nmero primo diferente de 2 e
5, ento p nunca aparecer como resto na diviso de 1 por n, pois em geral um fator primo comum de n e de um resto que aparece na diviso de
1 por n s pode ser 2 ou 5 ( de fato, um resto que aparece na diviso de
EUREKA! Edio Especial, 2007

Sociedade Brasileira de Matemtica

1 por n resto da diviso de alguma potncia de 10 por n ). Por outro


lado, se os nicos fatores primos de n so 2 e 5, ento 1/n tem
representao decimal finita.
Concluso: Se o perodo de 1/n tiver n1 casas decimais, ele ter
propiedades anlogas s de 142857: os dgitos de seus produtos por 1, 2,
3, 4, , n1 sero sempre os mesmos, na mesma ordem circular. Para
que isso acontea, n deve ser primo e a menor potncia de 10 que deixa
resto 1 quando dividida por n deve ser 10n1. Dizemos que, nesse caso,
10 raiz primitiva mdulo n. No se sabe se existem infinitos primos n
com essa propriedade. Isso seguiria de uma famosa conjectura de teoria
dos nmeros, a conjectura de Artin (vide [V]).
Os nmeros primos n menores que 100 tais que o perodo de 1/n
na base 10 tem n 1 casas so 7, 17, 19, 23, 29, 47, 59, 61 e 97.
Por outro lado, para todo nmero primo n existem nmeros
naturais B entre 2 e n 1 tais que o perodo de 1/n na base B tem exatamente n 1 casas (nesses casos B raiz primitiva mdulo n). Se um
nmero B tem essa propriedade, todas as bases da forma kn + B com k
natural tambm tm. Nesses casos, o perodo de 1/n na base B ( ou seja,
o nmero (Bn-11)/n ), quando multiplicado por 1, 2, 3, , n 1 ter
representaes na base B que sero permutaes uma da outra com a
mesma ordem circular.
Por exemplo, com n = 5 e B = 8, temos que a representao de 1/5 na
base 8 0,146314631463 Na base 8 temos:
2 (1463)8 =(3146)8 , 3 (1463)8 = (4631)8 ,
4 (1463)8 = (6314)8 , 5 (1463)8 = (7777)8
Referncias:
[L] Lima, Elon L., Meu Professor de Matemtica e outras histrias, pp. 158-170
SBM, 1991.
[T] Tahan, Malba, O homen que calculava, Ed. Record.
[V] Voloch, Jos Felipe, Raizes Primitivas e a Conjectura de Artin, Revista Matemtica
Universitria N9/10, dezembro de 1989, pp. 153-158.

EUREKA! Edio Especial, 2007

Sociedade Brasileira de Matemtica

DOIS PROBLEMAS SOBRE GRAFOS


Paulo Cezar Pinto Carvalho
IMPA

Nvel Intermediario.
INTRODUO
A figura abaixo mostra um mapa rodovirio de um pas fictcio.
Neste artigo vamos examinar dois problemas relativos a este mapa:
1. Um funcionrio, encarregado de verificar, periodicamente, o estado
das estradas, deseja planejar a sua rota de inspeo. Idealmente, esta rota
deveria se iniciar na capital e percorrer cada estrada exatamente uma
vez, voltando, ento, ao ponto de partida. Existe tal rota?
2. Um representante de vendas de uma companhia deseja planejar uma
rota na qual ele visite cada cidade exatamente uma vez, voltando ao
ponto de partida. Existe tal rota?

Fig. 1 - Mapa rodovirio de um pas fictcio


H vrios pontos em comum entre os dois problemas. Por exemplo: em ambos se deseja verificar a existncia de um circuito (ou ciclo)
no grafo determinado pelo mapa (um grafo um par (V, A), em que V o
conjunto de vrtices do grafo, e A um conjunto de pares de vrtices
os arcos do grafo). No primeiro problema, este circuito deve incluir exatamente uma vez cada arco do grafo. No segundo problema, o circuito
deve incluir exatamente uma vez cada vrtice do grafo. Embora os dois
problemas sejam aparentemente semelhantes, h algumas diferenas fundamentais entre eles. Convidamos os leitores a refletir um pouco sobre
cada um deles antes de prosseguir.
EUREKA! Edio Especial, 2007

Sociedade Brasileira de Matemtica

CIRCUITOS EULERIANOS

O primeiro problema o do inspetor de estradas foi estudado


pela primeira vez por Euler (1707-1783). Por esta razo, um circuito que
percorre cada arco de um grafo exatamente uma vez chamado de
circuito euleriano e um grafo que possui um tal circuito chamado de
grafo euleriano. A situao estudada por Euler ficou imortalizada como
o Problema das Pontes de Knisberg, ilustrado na figura abaixo, e que
possivelmente j conhecido por muitos dos leitores. O objetivo
percorrer exatamente uma vez todas as sete pontes da cidade (hoje
Kaliningrado), que conectam as duas ilhas entre si e com as margens do
rio, voltando ao ponto de partida.

Fig. 2 O Problema das Pontes de Knisberg


Em linguagem de grafos, trata-se de encontrar um circuito
euleriano no grafo da figura acima, no qual os vrtices representam as
ilhas e as margens e os arcos so as pontes 1. Euler mostrou a
no-existncia de tal circuito atravs de um argumento extremamente
simples. Consideremos, por exemplo, a ilha da direita. Um circuito
qualquer deve chegar ilha e sair dela o mesmo nmero de vezes. Logo,
para que exista um circuito euleriano, deve haver um nmero par de
pontes com extremidade nesta ilha. Como existem trs pontes nessas
condies, conclumos que no possvel encontrar um circuito
euleriano. De modo mais geral, temos o seguinte:
Teorema: Existe um circuito euleriano em um grafo se e somente se o grafo
conexo (isto , existe um caminho ligando qualquer par de vrtices) e cada
vrtice tem grau par (ou seja, o nmero de arcos que nele incidem par).
1

A rigor, neste caso temos um multi-grafo, j que certos pares de vrtices so ligados
por mais de um arco.

EUREKA! Edio Especial, 2007

Sociedade Brasileira de Matemtica

O argumento acima mostra a necessidade de se ter grau em cada


vrtice para existir um circuito euleriano. tambm bvio que o grafo
precisa ser conexo. A prova de que essas duas condies implicam na
existncia de um circuito euleriano pode ser feita por induo finita no
nmero de arcos do grafo e deixada como um exerccio para o leitor.
[Sugesto: suponha a propriedade verdadeira para grafos com menos de n arcos
e considere um grafo com n arcos, satisfazendo s duas condies. Comeando
em um vrtice qualquer, percorra arcos do grafo, at voltar a um vrtice j
visitado (o caminho gerado possui, ento, um ciclo). Retirando do grafo os
arcos desse ciclo, obtm-se um ou mais grafos satisfazendo as duas condies e
com menor nmero de arcos (portanto, com circuitos eulerianos, de acordo com
a hiptese de induo). Basta explicar como costurar esses circuitos
eulerianos ao ciclo descrito acima].

Podemos aplicar este teorema ao nosso problema de inspeo de


estradas. Da mesma forma como no Problema das Pontes de Knisberg,
no existe qualquer circuito euleriano no grafo determinado pelo mapa
rodovirio, j que o vrtice correspondente capital tem grau 3. Assim,
se o nosso inspetor de estradas recebesse de seu chefe a incumbncia de
elaborar um trajeto nas condies do problema 1, ele poderia facilmente
convenc-lo da impossibilidade de faz-lo. Como veremos a seguir, a
situao do seu colega representante de vendas bem pior...
CIRCUITOS HAMILTONIANOS

Um circuito passando exatamente uma vez por cada vrtice de um


grafo chamado de circuito hamiltoniano, em homenagem ao
matemtico irlands William Rowan Hamilton (1805-1865), que estudou
este problema no grafo determinado pelas arestas de um dodecaedro
regular (existe ou no um circuito hamiltoniano neste caso?). Um grafo
que possui um circuito hamiltoniano chamado de grafo hamiltoniano.
A situao do problema de verificar se um grafo hamiltoniano
bem diferente da do problema anterior. Apesar de terem sido estudados
por vrios sculos, no h uma boa caracterizao dos grafos
hamiltonianos. H diversas famlias de grafos para os quais existe um
circuito hamiltoniano (um exemplo trivial um grafo completo, em que
EUREKA! Edio Especial, 2007

10

Sociedade Brasileira de Matemtica

cada vrtice ligado a todos os outros); tambm possvel estabelecer


certas condies que implicam na no-existncia de um circuito. Mas
uma caracterizao geral no foi encontrada e, luz de certos avanos em
teoria da computao das ltimas dcadas, parece improvvel que ela
seja encontrada algum dia.
O problema de decidir se um grafo hamiltoniano est na
companhia de diversos problemas ilustres, com as seguintes
caractersticas em comum:

O problema possui uma assimetria fundamental: muito fcil convencer


algum da existncia de um circuito hamiltoniano em um grafo: basta exibir
tal caminho. No entanto, difcil, em geral, convencer algum da
no-existncia de um tal circuito. Por exemplo, o grafo da figura abaixo (o
leitor capaz de reconhec-lo?) tem um circuito hamiltoniano, de cuja
existncia o leitor fica imediatamente convencido pela figura. J o grafo
dado no incio do artigo no tem circuito hamiltoniano, mas no existe um
argumento simples e geral para demonstrar esse fato (assim, nosso amigo
representante de vendas certamente ter mais trabalho para convencer seu
chefe da impossibilidade de elaborar uma rota nas condies do problema 2
do que seu colega inspetor de estradas).

Fig. 3 Um grafo hamiltoniano

No se conhece um algoritmo eficiente para verificar se um grafo


hamiltoniano (por eficiente, entendemos aqui um algoritmo em que o
nmero de passos seja limitado por um polinmio no nmero de
vrtices do grafo). Alm disso, parece improvvel que um tal
algoritmo possa algum dia ser encontrado, porque sua existncia
implicaria na existncia de algoritmos eficientes para um grande

EUREKA! Edio Especial, 2007

11

Sociedade Brasileira de Matemtica

nmero de outros problemas, para os quais tambm no se conhecem


algoritmos eficientes. Estes problemas (incluindo o de verificar a
existncia de circuito hamiltoniano) formam uma classe de problemas
chamados de NP-completos. Um outro problema famoso da classe
o de determinar o nmero mnimo de cores que podem ser usadas
para colorir os vrtices de um grafo de modo que vrtices de mesma
cor no sejam ligados por um arco.
O leitor poder estar pensando assim: mas ser que esta histria de
algoritmos eficientes tem relevncia, numa era de computadores cada vez
mais velozes? Afinal de contas, existe um algoritmo extremamente
simples para verificar se um grafo possui um circuito hamiltoniano. Se
existir um tal circuito, ele corresponder a uma permutao (circular) dos
vrtices com a propriedade de que vrtices consecutivos sejam ligados
por um arco do grafo. Ora, para verificar a existncia de circuito
hamiltoniano basta gerar todas as permutaes circulares dos vrtices e
testar se uma delas corresponde a um percurso no grafo.
claro que este algoritmo funciona para grafos de tamanho
moderado (ele poderia ser o recurso usado pelo nosso vendedor: como
so apenas 9 cidades, ele teria que testar apenas 8! = 40.320 caminhos,
o que seria feito com rapidez em um computador). Mas o que ocorre
com grafos maiores? Vejamos, por exemplo, uma situao em que o
nmero de cidades cresce para 50 (o que representaria um tamanho ainda
bastante razovel para uma situao real). Neste caso, o computador
deveria examinar 49! circuitos potenciais. Tentemos estimar a magnitude
deste nmero. A forma mais simples usar a frmula de Stirling, que
n

n
fornece a estimativa n! 2n . Mas, neste caso, podemos usar
e
estimativas mais elementares. Por exemplo, podemos usar apenas
potncias de 2. Temos:
49! = 1 2 3 4 5 6 7 8 ... 15 16 ... 31 32
49 > 1 2 2 4 4 4 4 8 ... 8 16 ... 16 32
32 = 22 x 44 x 88 x 1616 x 3218 = 22+8+64+90 = 2164.
Mas 210 = 1024 >103. Logo 49! > 16. 1048.
EUREKA! Edio Especial, 2007

12

Sociedade Brasileira de Matemtica

Ora, um computador moderno pode realizar cerca de 200 milhes


de operaes por segundo. Se em cada operao ele conseguir testar um
circuito, ele ainda assim precisar de mais de 16. 1048 / 2. 106 = 8 1042
segundos, o que corresponde a aproximadamente a 2 1035 anos. Assim,
trata-se claramente de uma misso impossvel para o algoritmo de fora
bruta baseado na anlise de cada permutao de vrtices.
PROBLEMAS DIFCEIS QUE TAMBM SO TEIS

O resultado da discusso acima pode parecer bastante desanimador:


no parece haver bons mtodos para verificar a existncia de um circuito
hamiltoniano e algoritmos de fora bruta s funcionam para problemas
com pequeno nmero de vrtices ( bom que se diga que existe um meio
termo: h estratgias que permitem resolver o problema acima para
valores razoveis de n, reduzindo substancialmente o nmero de
possibilidades a serem examinadas; mesmo estes algoritmos, no entanto,
tornam-se imprticos a partir de um certo ponto). O mesmo ocorre com
todos os chamados problemas NP-completos.
No entanto, ao invs de ficarmos deprimidos com esta
caracterstica desses problemas, podemos explor-la para uma importante
finalidade em criptografia, que a parte da Matemtica que estuda
mtodos para criar e decifrar cdigos. Para tal, tambm muito
importante a assimetria apontada acima (e que ocorre em todos os problemas NP-completos): difcil encontrar um circuito hamiltoniano (ou
mostrar que no existe um), mas fcil testar se uma seqncia de vrtices forma um circuito hamiltoniano.
Suponhamos que voc seja cliente de um banco. Para ter acesso aos
servios, voc usa o nmero de sua conta (que pblico) e uma senha,
que em princpio deve ser conhecida apenas por voc. O procedimento
mais simples seria ter a sua senha armazenada no sistema do banco. Mas
a voc correria o risco de que ela fosse descoberta, por exemplo, por um
funcionrio desonesto. Em lugar disto, o sistema do banco armazena
uma verso codificada da senha, que no precisa ficar em segredo. Esta
codificao deve ser feita de tal forma que seja simples verificar se sua
senha est correta (para que voc seja autorizado a retirar dinheiro do
EUREKA! Edio Especial, 2007

13

Sociedade Brasileira de Matemtica

caixa eletrnico), mas seja praticamente impossvel recuperar a senha a


partir da verso codificada.
Problemas NP-completos servem como uma luva para esta tarefa.
Se quisssemos usar o problema do circuito hamiltoniano, poderamos
agir mais ou menos da formadescrita a seguir. O cliente poderia escolher
uma permutao dos nmeros de 1 a 50, conhecida apenas por ele.
A partir dessa informao, seria gerado um grafo, contendo
necessariamente os arcos correspondentes ao circuito (os demais
poderiam, por exemplo, ser gerados por um mtodo aleatrio, em que
cada um dos possveis arcos teria uma certa probabilidade de sere
escolhido). Este grafo seria armazenado no sistema. A figura a seguir
mostra uma representao de uma permutao dos nmeros de 1 a 50 e
um grafo, gerado aleatoriamente, que possui um ciclo hamiltoniano dado
por esta permutao.

Fig. 4 Um ciclo hamiltoniano e um grafo gerado a partir dele


Quando o cliente fosse utilizar sua conta, o sistema simplesmente
verificaria se a permutao apresentada corresponde a um caminho no
grafo. Como improvvel que um tal ciclo pudesse ser encontrado para
um grafo deste tamanho, dificilmente um impostor conseguiria se fazer
passar pelo cliente, ainda que conhecesse o grafo-problema. Na prtica,
so utilizados outros problemas NP-completos para se fazer codificao
de senhas, mas a idia exatamente a mesma acima.
PALAVRAS FINAIS

Grafos so uma fonte inesgotvel de problemas com enunciado simples


mas que escondem, muitas vezes, uma sofisticada estrutura matemtica. Neste
artigo abordamos apenas alguns aspectos de dois desses problemas. Certamente
voltaremos a falar em grafos em outros artigos desta revista. Para o leitor que
deseja saber mais sobre o assunto, recomendamos os livros a seguir:
Jaime Luiz Szwarcfiter. Grafos e Algoritmos Computacionais. Editora Campus.
Oynstein Ore. Graphs and Their Uses. The Mathematical Association of America.
EUREKA! Edio Especial, 2007

14

Sociedade Brasileira de Matemtica

Nvel Iniciante.

PARIDADE
Eduardo Wagner

Todo nmero natural par ou mpar.


Elementar, no? A afirmao acima, que uma das mais simples
e bvias da Matemtica, tambm uma ferramenta de grande utilidade
na resoluo de muitos problemas envolvendo nmeros naturais. Vamos
comentar neste artigo alguns deles, em graus diferentes de dificuldade,
mas inicialmente precisamos recordar trs importantes propriedades:
a) a soma de dois nmeros pares par.
b) a soma de dois nmeros mpares par.
c) a soma de um nmero par com um nmero mpar mpar.
Dizemos que dois nmeros inteiros tm mesma paridade, quando
so ambos pares ou ambos mpares. Assim, podemos dizer que a soma de
dois nmeros inteiros par se, e somente se, eles tm mesma paridade.
Vamos aos problemas.
PROBLEMA 1

Em um quartel existem 100 soldados e, todas as noites, trs deles so


escolhidos para trabalhar de sentinela. possvel que aps certo
tempo um dos soldados tenha trabalhado com cada um dos outros
exatamente uma vez?
RESPOSTA : No.

Escolha um soldado. Em cada noite em que trabalha, ele est em


companhia de dois outros. Como 99 um nmero mpar, no
podemos formar pares de soldados sempre diferentes para trabalhar
com o escolhido.
PROBLEMA 2

Um jogo consiste de 9 botes luminosos (de cor verde ou vermelha)


dispostos da seguinte forma:

EUREKA! Edio Especial, 2007

15

Sociedade Brasileira de Matemtica

Apertando um boto do bordo do retngulo, trocam de cor ele e


seus vizinhos (do lado ou em diagonal). Apertando o boto do centro,
trocam de cor todos os seus 8 vizinhos porm ele no.
Exemplos:
Apertando 1, trocam de cor 1, 2, 4 e 5.
Apertando 2, trocam de cor 1, 2, 3, 4, 5 e 6.
Apertando 5, trocam de cor 1, 2, 3, 4, 6, 7, 8 e 9.
Inicialmente todos os botes esto verdes. possvel, apertando
sucessivamente alguns botes, torn-los todos vermelhos?
RESPOSTA : No possvel.

Observe que apertando um boto do vrtice do retngulo, trocam de cor 4


botes. Apertando um boto do meio de um lado, trocam de cor 6 botes
e apertando um boto do centro trocam de cor 8 botes. Assim, cada vez
que apertamos um boto trocam de cor um nmero par de botes. Como
existem 9 botes, no possvel que todos troquem de cor.
PROBLEMA 3

Escrevemos abaixo os nmeros naturais de 1 a 10.


1

10.

Antes de cada um deles, coloque sinais + ou de forma que a soma


de todos seja zero.
SOLUO: No possvel fazer isto.

Imaginando que fosse possvel, deveramos separar os nmeros dados em


dois grupos com a mesma soma. Ento colocaramos sinais negativos nos
nmeros de um dos grupos e sinais positivos nos nmeros do outro.
EUREKA! Edio Especial, 2007

16

Sociedade Brasileira de Matemtica

Teramos ento uma soma igual a zero. Acontece que a soma dos
nmeros naturais de 1 a 10 igual a 55. Como este nmero mpar, no
podemos separar os nmeros dados em dois grupos que tenham a mesma
soma.
Como o leitor deve estar percebendo, os argumentos utilizados
permitiram concluir que as respostas dos trs problemas propostos foram
iguais: no possvel fazer tal coisa. Na maioria das vezes, um
argumento de paridade serve exatamente para isto. Mostrar que um
determinado fato no pode ocorrer e isto no desanimador, muito pelo
contrrio. Serve para nos convencer que no adianta ficar gastando tempo
demais fazendo tentativas inteis. As experincias so valiosas
no sentido de nos abrir os olhos para a possibilidade do problema
no ter soluo e, a partir da, buscar um argumento que resolva
definitivamente a questo.
muito importante tambm explorar um problema, ou seja,
imaginar pequenas modificaes no enunciado e verificar o que ocorre
com sua resposta. Por exemplo, o problema 3 no tem soluo porque a
soma dos naturais de 1 at 10 55 (mpar). O que ocorreria se a soma
fosse par? Este um novo e atrativo problema. Vamos enunci-lo:
PROBLEMA 3A:

Escrevemos abaixo os nmeros naturais de 1 a 11.


1

10

11

Antes de cada um deles, coloque sinais + ou de forma que a soma


de todos seja zero.
SOLUO:

A soma dos nmeros naturais de 1 a 11 66. Como podemos separ-los


em dois grupos de soma 33? Comeando pelos maiores observe que 11 +
10 + 9 = 30. Logo, 11 + 10 + 9 + 3 = 33. O problema 3A tem como uma
soluo possvel:
+1 + 2 3 + 4 + 5 + 6 + 7 + 8 9 10 11 = 0
EUREKA! Edio Especial, 2007

17

Sociedade Brasileira de Matemtica

Fica ao encargo do leitor mostrar que sempre que a soma dos


naturais de 1 at n par ento podemos separ-los em dois grupos de
igual soma. Voc pode utilizar o caminho que utilizamos acima,
ou buscar uma outra forma.
Para saber mais e intrigar seus colegas

Voc pode propor aos seus amigos os problemas 3 ou 3A com uma lista
grande de nmeros naturais consecutivos. O problema ter ou no
soluo caso a soma desses nmeros seja par ou mpar, respectivamente.
Entretanto, possvel encontrar o resultado desta soma rapidamente, sem
precisar somar todas as parcelas. A soma de todos os naturais de 1 at n
igual a

(1 + n) n

. Por exemplo, a soma de todos os naturais de 1 at 10

2
(1 + 10)10 11 10
=
= 55 . Procure demonstrar este fato e, se no conseguir,
2
2

pergunte ao seu professor ou escreva para a EUREKA!

PROBLEMA 4

Mostre que se a, b e c so inteiros mpares, a equao ax 2 + bx + c = 0


no tem raiz racional.
Comentrios:
1) Um nmero raiz de uma equao dada se quando for substitudo no
lugar do x a igualdade ficar correta. Por exemplo, x =

2
3

raiz

(ou soluo) da equao 3 x 2 = 0 porque 3 2 = 0 . Ainda, x = 2


3

soluo da equao x x + x 10 = 0 porque 2 4 2 3 + 2 10 = 0 .


Freqentemente no sabemos como resolver uma equao mas, em geral,
podemos verificar se um certo valor de x ou no uma de suas razes.
4

EUREKA! Edio Especial, 2007

18

Sociedade Brasileira de Matemtica

2) Um nmero racional quando puder ser escrito como uma frao de


numerador e denominador inteiros. Por exemplo,

2
7

4
1

so exemplos de

nmeros racionais.
3) Quando desejamos demonstrar que certo fato impossvel utilizamos
freqentemente o mtodo da reduo ao absurdo. Este mtodo consiste
em imaginar o contrrio, ou seja, que tal fato seja possvel. A partir da
procuramos chegar a uma contradio, a um absurdo. Conseguindo isso,
teremos mostrado que nossa hiptese (a do contrrio) falsa e
conseqentemente, que a afirmao inicial verdadeira.
Vamos ver tudo isso na soluo do problema. No se preocupe se voc
ainda no sabe resolver uma equao do segundo grau. Isto no ser
necessrio. Tudo o que precisamos verificar se um nmero racional
pode ser uma raiz.

Soluo do problema 4

Imaginemos que o nmero racional

p
q

seja raiz da equao

ax 2 + bx + c = 0 onde a, b e c so inteiros mpares. Logo, fazendo a


substituio, devemos ter,
2

p
p
a + b + c = 0
q
q

p2
p
a 2 +b +c = 0
q
q
ap 2 + bpq + cq 2 = 0

EUREKA! Edio Especial, 2007

19

Sociedade Brasileira de Matemtica

Vamos acrescentar agora uma hiptese importante para facilitar nosso


trabalho. Vamos supor que a nossa frao

p
q

seja irredutvel, ou seja,

que ela j foi simplificada ao mximo. Por exemplo, no lugar de


estaremos considerando

2
3

4
6

o que a mesma coisa. Consideramos ento,

para a soluo do problema, que p e q no so ambos pares.


Observe agora a equao ap 2 + bpq + cq 2 = 0 nos seguintes casos:
a) p e q so mpares: neste caso, ap 2 mpar, bpq mpar e cq 2 mpar.
Como a soma de trs nmeros mpares mpar, o resultado no pode ser
zero.
b) p par e q mpar: neste caso, ap 2 par, bpq par e cq 2 mpar.
Como a soma de dois nmeros pares e um mpar mpar, o resultado no
pode ser zero.
c) p mpar e q par: vale o mesmo argumento do caso b).

Demonstramos ento que nenhuma frao de numerador e denominador


inteiros pode ser raiz da equao ax 2 + bx + c = 0 onde a, b e c so
inteiros mpares.
PROBLEMA 5

Um tabuleiro 6 6 est coberto com domins 2 1. Mostre que existe


uma reta que separa as peas do tabuleiro sem cortar nenhum domin.

EUREKA! Edio Especial, 2007

20

Sociedade Brasileira de Matemtica

SOLUO:

Cada domin formado por dois quadrados e portanto, se o tabuleiro est


inteiramente coberto, 18 domins foram utilizados. Imagine agora uma
reta (horizontal, por exemplo) que separe o tabuleiro em duas partes. Se
ela no corta nenhum domin, est resolvido o problema. Suponha ento
que ela corte ao meio um domin. Neste caso, acima desta reta teremos n
domins inteiros mais meio domin, ou seja, teremos acima desta reta
2n + 1 quadrados, que um nmero mpar. Mas isto impossvel porque
se o tabuleiro tem 6 unidades de largura, qualquer reta o dividir em
partes que contm nmeros pares de quadrados acima e abaixo dela.
Assim, se uma reta corta um domin, dever cortar um outro domin.
Para a diviso do tabuleiro, existem 10 retas possveis e, se cada uma
delas cortar dois domins, deveramos ter 20 domins no tabuleiro.
Como eles so apenas 18 ento existe uma reta (pelo menos) que no
corta nenhum domin.

Problemas para pesquisa


PROBLEMA 6

Os nmeros naturais de 1 at 1998 so escritos em um imenso quadro


negro. Em seguida, um aluno apaga dois quaisquer colocando no lugar
sua diferena (no negativa). Depois de muitas operaes, um nico
nmero ficar escrito no quadro. possvel que esse nmero seja zero?
PROBLEMA 7

Em uma ilha plana existem 11 cidades numeradas de 1 a 11. Estradas


retas ligam 1 a 2, 2 a 3, 3 a 4, ..., 10 a 11 e 11 a 1. possvel que uma
reta corte todas as estradas?

EUREKA! Edio Especial, 2007

21

Sociedade Brasileira de Matemtica

ADEDANHA OU DE COMO OS DEUSES MATEMTICOS


TROUXERAM A PAZ AO MUNDO
Pablo Emanuel IMPA

Nvel Iniciante
Diz a lenda que, h muitos milnios, o mundo vivia em guerra
constante, pois as pessoas no sabiam como resolver as suas
discordncias, a no ser pela fora bruta. Um dia, os deuses (que so
exmios matemticos), para resolver esta situao, enviaram um
mensageiro Terra, com a misso de ensinar os homens a resolverem as
suas disputas. O anjo se dirigiu ento aos homens, dizendo:
- Quando dois entre vs precisarem chegar a um acordo, que se
faa como vos digo: que um escolha par e o outro escolha mpar, ento
que ambos mostrem ao mesmo tempo a mo exibindo uma certa
quantidade de dedos. Sero ento somadas estas quantidades. Se a soma
for um nmero par declara-se vencedor o jogador que escolheu par e,
caso contrrio, declara-se vencedor aquele que escolheu mpar.
Os homens ficaram maravilhados com a sabedoria dos deuses e,
deste dia em diante, houve um grande perodo de paz, pois todas as
questes eram resolvidas com o jogo que eles haviam aprendido dos
deuses.
Um dia, porm, esta paz foi abalada. Trs reis disputavam um
pedao de terra, que ficava exatamente na divisa entre os trs pases. Eles
estavam prontos a utilizar o jogo divino do par-ou-mpar, mas o rei que
sabia mais matemtica entre os trs se levantou e disse:
- Caros colegas, ns todos sabemos que um nmero s pode ser
par ou mpar, no existindo uma terceira opo. Como somos trs, algum
de ns no vai ter opo alguma.
Este era realmente um problema muito srio. Para resolv-lo, foi
chamado o melhor matemtico da Terra na poca, chamado Zerinhoum.
Ele pensou durante vrias semanas em como resolver o problema dos
reis, e finalmente chegou a uma soluo:
EUREKA! Edio Especial, 2007

22

Sociedade Brasileira de Matemtica

- Majestades, encontrei a soluo para o vosso problema. Ao


mesmo tempo, vs estendereis vossas mos, mantendo-as ou com a
palma para cima ou com a palma para baixo. Aquele dentre vs que tiver
a mo em posio diferente dos demais ganha a disputa.
- E se todos ns tivermos as palmas das mos viradas para o
mesmo lado? -indagaram os reis.
- Neste caso, majestades, vs jogareis novamente, at que algum
entre vs vena.
Como a disputa era muito urgente, os reis aceitaram a sugesto do
eminente matemtico. Houve mais um perodo de paz, desta vez muito
mais curto. Em pouco tempo, as pessoas perceberam que o jogo de
Zerinhoum podia se alongar indefinidamente, e que era possvel se fazer
alianas para prejudicar adversrios polticos.
Ento as pessoas rezaram aos deuses, pedindo um novo jogo, que
trouxesse de novo a paz Terra. Os deuses ento enviaram novamente
um mensageiro. Quando ele chegou, os homens lhe cercaram dizendo:
- Mensageiro dos deuses, atendeste as nossas preces. Vivamos
em guerra, e os deuses nos enviaram o sagrado jogo do par-ou-mpar, que
nos trouxe a paz. Mas este jogo s podia ser jogado por dois jogadores, e
as trevas se abateram de novo sobre ns. Ento um grande homem nos
ensinou um novo jogo, que chamamos Zerinhoum em sua homenagem.
Mas este jogo tinha problemas, e a guerra voltou a nos assolar. Por favor,
grande sbio, que vem em nome dos deuses, ensina-nos um novo jogo,
que possa nos trazer de volta nossa paz.
E o anjo assim respondeu:
- Eu vos ensinarei um novo jogo. Zerinhoum era um grande
matemtico, mas no conhecia os segredos dos deuses. Eu vos revelarei
estes segredos.

EUREKA! Edio Especial, 2007

23

Sociedade Brasileira de Matemtica

Para isto, o melhor comear pelo antigo jogo do par-ou-mpar.


Como se decide se um nmero par ou mpar? Basta dividi-lo por 2. Se
o resto for igual a 0, o nmero ser par, se for igual a 1, o nmero ser
mpar. Estas so as nicas duas opes, porque o resto sempre menor
do que o dividendo (2). Reparai que se dividirmos o nmero por 3,
passam a existir 3 opes para o resto, pois ele pode ser 0, 1 ou 2.
Na diviso por 4, existem 4 restos possveis ( 0, 1, 2 e 3). Em geral,
quando dividimos um nmero por n , existem n restos possveis
( 0, 1, 2, , n 2 e n 1 ).
E o que isto tem a ver com o jogo? Tudo, eu vos digo. Se n
pessoas estiverem em uma disputa, vs fareis como eu vos digo:
As pessoas escolhero, cada uma, um nmero entre 0 e n 1 diferente.
Depois, ao mesmo tempo, elas mostraro as mos, exibindo uma
quantidade qualquer de dedos. As quantidades sero somadas,
e o nmero resultante ser dividido por n. A pessoa que escolheu o resto
desta diviso ser a vencedora.
Esta a forma que os deuses jogam. Mas vs da Terra sois muito
desorganizados para poder escolher tantos nmeros de forma tranqila.
Portanto, eu vos ensinarei uma forma alternativa de jogar este jogo. Vs
vos arrumareis em um crculo. Uma pessoa ser designada a contar.
Ento vs gritareis a palavra mgica Adedanha e todos mostraro as
mos. Os resultados sero somados, e aquele que havia sido designado
far o seguinte procedimento: Em primeiro lugar falar Um, e apontar
para o cu, para que nunca vos esqueais de que foram os deuses que vos
ensinaram este jogo. Ento apontar para si mesmo e falar Dois.
Depois apontar para o jogador sua esquerda e falar Trs, e depois
seguir apontando para o jogador esquerda deste e assim por diante,
sempre acrescentando um ao nmero que havia falado anteriormente, at
chegar soma que havia sido calculada. O jogador que estiver sendo
apontado neste momento ser o vencedor. Se a soma for 1, o jogador que
estiver direita do que estiver contando ser declarado vencedor. Se for
0, ser o que estiver direita deste.
Os homens entenderam as determinaes do mensageiro, mas
ainda no entendiam porque o segundo jogo era equivalente ao primeiro.
O anjo ento lhes explicou:
EUREKA! Edio Especial, 2007

24

Sociedade Brasileira de Matemtica

- A pessoa que est contando vai apontar para si mesma quando


estiver falando 2. Depois vai dar uma volta completa no crculo e vai
apontar para si mesma novamente quando estiver no 2 + n, e
novamente no 2 + 2n. Ou seja, ela vai estar apontando para si mesma
se e somente se estiver falando um nmero cujo resto na diviso por n
seja 2. Da mesma forma, vai estar apontando para o jogador sua
esquerda se e somente se estiver falando um nmero que deixa resto 3 ao
ser dividido por n. E assim por diante, de forma que cada jogador ter
associado a si um nmero entre 0 e n 1 tal que ele o vencedor se e
somente se o resultado da soma deixa aquele resto quando dividido por n.
Os homens estavam maravilhados com a explicao do
mensageiro, mas um sbio ancio levantou uma questo:
- , mensageiro divino, sem dvida s sbio e sagaz. No entanto,
uma dvida me corri o esprito. Tendo cada jogador 10 dedos, esta soma
pode atingir nmeros muito elevados, fazendo com que o responsvel
pela contagem passe um tempo enorme falando e apontando at que se
descubra o vencedor.
- Tens toda a razo, sbio homem. Mas em verdade vos digo que
tolice que um jogador exiba uma quantidade de dedos maior ou igual
quantidade de jogadores. Com efeito, suponde que um jogador coloque
um nmero maior ou igual a n. Os primeiros n dedos s vo ter o efeito
de fazer com que a contagem d uma volta completa no crculo, sem
alterar em nada quem ser o vencedor. Portanto, ele pode subtrair n da
sua quantidade sem que isto altere o resultado. Se o nmero persistir
maior ou igual a n, basta voltar a subtrair, at que o nmero fique entre
0 e n 1.
- Isto de fato diminui sobremaneira o esforo requerido- replicou
o ancio. Mas ainda assim o resultado pode chegar a n(n 1), que ainda
bastante grande.
- s de fato perspicaz, meu nobre homem. Mas no penseis que a
sabedoria dos deuses possui limite. O mesmo processo que foi aplicado a
cada nmero individualmente pode ser aplicado soma. Por exemplo,
EUREKA! Edio Especial, 2007

25

Sociedade Brasileira de Matemtica

considerai um jogo com 4 jogadores. Suponde que um dos jogadores


exibe 3 dedos e outro exibe 2 dedos. Por que considerar a sua soma como
sendo 5, se o efeito de somar 4 apenas fazer com que o responsvel
pela contagem d uma volta a mais? Em vez disto, muito mais sensato
considerar a sua soma como sendo 5 4 = 1. Mais geralmente, considere
um jogo com n jogadores. Em primeiro lugar diminui-se n dos valores
jogados por cada um, de forma que todos eles estejam entre 0 e n 1
(se todos os jogadores dessem ouvidos s palavras dos deuses, no
jogariam alm destes limites). Depois procede-se a soma, da seguinte
forma. Soma-se o primeiro valor com o segundo. Caso esta soma seja um
valor maior ou igual a n, subtrai-se n do resultado ( Sede espertos e
sabereis que fazendo isto sempre obtereis um nmero entre 0 e n 1 ).
Depois, a este resultado, soma-se o terceiro valor, tomando-se o cuidado
de se subtrair n caso a soma exceda n 1. Prossegue-se desta forma at
que todos os valores tenham sido somados. Se seguistes o meu raciocnio
at este ponto, no deveria ser-vos surpresa o fato que o resultado de uma
tal operao est sempre entre 0 e n 1, e portanto o jogador responsvel
pela contagem nunca precisar dar mais de uma volta.
E ento todos os habitantes se ajoelharam aos ps do anjo,
reconhecendo a sua suprema sabedoria, e o mundo conheceu enfim a paz.
At hoje os homens jogam os jogos de par-ou-mpar e adedanha da forma
como foram ensinados pelos deuses, embora, infelizmente, a maioria
tenha se esquecido da lio final e continue se extenuando em uma
interminvel contagem que d voltas e mais voltas.

E foi assim que a lenda me foi contada pela minha av, que ouviu
de sua av, que ouviu de sua prpria av, e assim por diante, at o
princpio dos tempos.

EUREKA! Edio Especial, 2007

26

Sociedade Brasileira de Matemtica

Voc deve estar achando meio esquisita a maneira de somar que


foi ensinada pelos deuses. No entanto, eles a usaram em vrias outras
coisas que nos so muito familiares. Se voc no acredita, responda
rpido a estas perguntas:
a) Se uma coisa comea em uma segunda-feira e dura 7 dias, em
que dia ela termina? E se durar 14 dias? E se durar 701 dias?
b) Se uma coisa comea s 8 horas da manh e dura 24 horas, a
que horas ela acaba? E se durar 48 horas? E se durar 4804 horas?
c) Se o ponteiro dos minutos de um relgio est apontando 23
minutos, para onde ele estar apontando daqui a 60 minutos? e
daqui a 120 minutos? e daqui a 66681 minutos?
Garanto que, se voc respondeu terceira pergunta dos 3 tens,
no contou de um em um (ou ento j estamos no terceiro milnio ).
Voc percebeu que os dias da semana se repetem de 7 em 7 dias, que as
horas do dia se repetem de 24 em 24 horas e que o ponteiro do relgio
volta a apontar para o mesmo ponto de 60 em 60 minutos. Garanto
tambm que voc, sem se dar conta, j pensou vrias vezes coisas como
5 horas depois das 21 horas so 2 horas da manh, ou seja, fez a conta
21 + 5 = 2 ! E, por incrvel que parea, esta conta est certa!!! Est certa,
porque voc est pensando a menos de mltiplos de 24 (ou, como
preferem os matemticos, mdulo 24) , ou seja:

21 + 5 = 2 ( + um mltiplo de 24 ) ,

ou, como preferem os matemticos,

21 + 5 = 2 (mod 24) .

EUREKA! Edio Especial, 2007

27

Sociedade Brasileira de Matemtica

Desta forma, a terceira pergunta do item c) pode ser reescrita


como Quanto 23 + 66681 (mod 60) . Se voc foi esperto(a) o
suficiente para responder quela pergunta, voc j deve ter percebido que
66681 = 21 (mod 60), e que 23 + 66681 = 23 + 21 (mod 60), ou seja,
23 + 66681 = 44 (mod 60), logo o ponteiro estar apontando para
o minuto 44. S para ver se voc entendeu at agora, preencha estas
lacunas:
2 + 2 = 1 (mod __ )
2 +__ = 0 (mod 17)
26 = 3 (mod __ )
No se esquea que a expresso mod n s uma forma
abreviada de + um mltiplo de n. Lembrando-se disto, veja quantas
coisas voc sabia, mas no sabia que sabia:
3 3 = 1 (mod 4)
1 = 1 (mod 2)
2 2 2 2 = 1 (mod 5)
3 3 3 3 = 1 (mod 5)
(esta talvez voc no saiba, mas n n n n = 1 (mod 5), sempre
que n no mltiplo de 5. Voc pode ver isto e muito mais no artigo do
professor Carlos Gustavo Moreira, na EUREKA! No. 2. Pergunta: se n
mltiplo de 5, quanto n n n n (mod 5)? )

Agora que voc j sabe o segredo dos deuses matemticos, j


pode jogar adedanha da forma original, como os deuses a conceberam, e
manter a paz no mundo sem fazer esforo.

EUREKA! Edio Especial, 2007

28

Sociedade Brasileira de Matemtica

QUADRILTEROS E TRINGULOS
Marcelo Mendes

Nvel Intermedirio
Apresentamos a seguir alguns resultados que servem de
ferramenta para resoluo de problemas de geometria elementar
envolvendo quadrilteros e tringulos, bastante freqentes em problemas
de olimpada.
QUADRILTEROS INSCRITVEIS

Os ngulos opostos de um quadriltero inscritvel so suplementares. Reciprocamente, se os ngulos opostos de um quadriltero so


suplementares, ento esse quadriltero inscritvel (cclico).
D
C
O

Alm disso, se ocorrer uma situao onde dois ngulos iguais


olham para um mesmo segmento, ento os extremos
desse segmento e os vrtices dos dois ngulos formam um
quadriltero inscritvel.
D

EUREKA! Edio Especial, 2007

29

Sociedade Brasileira de Matemtica

Exemplo: Seja AB o dimetro de um semicrculo. Um ponto M marcado


no semicrculo e um ponto K marcado sobre AB. Um crculo com o
centro P passa por A, M, K e um crculo com centro Q passa por M, K, B.
Prove que M, K, P, Q pertencem a um mesmo crculo.
Soluo: No crculo circunscrito de AMK, MPK = 2MAK; e no
crculo circunscrito de BMK, MQK = 2MBK. Como AB dimetro do
semicrculo, AMB = 90o e MAK+MBK = 90o. Da,
MPK+MQK = 180o e MPKQ inscritvel.
TEOREMA DE PTOLOMEU

Se ABCD um quadriltero inscritvel de diagonais AC


e BD, ento:
AB CD + AD BC = AC BD.
A

D
c

E
B

Prova: Seja x = BD e y = AC e a, b, c, d, os comprimentos dos lados.


Construa CDE = ABD, E AC. Da, CDE ADB e
ADE BCD, dando, respectivamente, ECx = ac e AEx = bd. Somando essas duas ltimas equaes, temos xy = ac + bd, como
queramos provar
H tambm uma extenso para esse teorema que vale para quadrilteros
no inscritveis: AB CD + AD BC > AC BD, isto , numa situao
geral vale AB CD + AD BC AC BD.

EUREKA! Edio Especial, 2007

30

Sociedade Brasileira de Matemtica

Exemplo: Prove que, se ABCDEFG um heptgono regular convexo, ento:


1
1
1
=
+
AB AC AD

Aplicando o Teorema de Ptolomeu no quadriltero inscritvel


ACDE, onde CD = DE = a = AB, AC = CE = b e AD = AE = c, temos bc
= ac + ab. Dividindo essa ltima equao por abc, segue o resultado.
A RELAO ENTRE A DISTNCIA DO ORTOCENTRO A UM VRTICE
E DO CIRCUNCENTRO AO LADO OPOSTO

Sejam H e O respectivamente o ortocentro e o circuncentro, do


ABC e M, o ponto mdio do lado BC. Ento AH = 2OM.
A
Y
H

O
M

N
C

Prova: Sejam AX e BY alturas e N, o ponto mdio de AC. Como MN base

mdia, MN // AB e MN = AB. Da, ABH OMN pois tm lados


paralelos entre si (e razo 2:1). Portanto, AH = 2OM
Exemplo: Prove que o ortocentro, o baricentro e o circuncentro de um

tringulo qualquer so colineares. (Reta de Euler)


Seja G a interseo de AM e HO (na figura acima). Ento,
AHG GOM na razo 2:1. Da, AG = 2GM. Portanto, G o
baricentro e pertence reta HO.

EUREKA! Edio Especial, 2007

31

Sociedade Brasileira de Matemtica

PROBLEMAS
1.

Seja P um ponto sobre o menor arco AC da circunferncia circunscrita a um


tringulo equiltero ABC. Calcule a medida do ngulo APC.

2.

Prove que um trapzio inscritvel se, e somente se, ele for issceles (lados
no paralelos iguais).

3.

Sejam AX e BY alturas de um tringulo issceles ABC (AC = BC) de ortocentro


H. Prove que 2HXXC = XYHC.

4.

Seja ABCD um losango inscritvel de lado 1 e P, um ponto sobre o menor arco


CD. Prove que PD2 + PCPA = 1.

5.

Seja P um ponto sobre o menor arco AC da circunferncia circunscrita a um


tringulo equiltero ABC. Prove que PB = PA + PC.

6.

Seja H o ortocentro de um tringulo ABC e P, o ponto diametralmente oposto a


B na circunferncia circunscrita a ABC. Prove que AHCP um paralelogramo.

7.

ABCD um paralelogramo. H o ortocentro do ABC e O, o circuncentro do


ACD.
Prove
que
H,
O,
D
so
colineares.

8.

Seja A1A2An um polgono regular de n lados. Se

1
1
1
=
+
,
A1 A2 A1 A3 A1 A4

calcule n.
9.

Sejam M, N, P os pontos mdios dos lados de um ABC acutngulo de


circuncentro O. Prolongue MO, NO, PO, a partir de O,
at
X,
Y,
Z,
respectivamente,
tais
que
MX = 2 OM , NY = 2 ON , PZ = 2 OP.
Prove
que
XYZ

semelhante ao ABC .

10.

Sejam M, N, P os pontos mdios dos lados de um ABC acutngulo


de circuncentro O. Prolongue MO, NO, PO, a partir de O, at
X, Y, Z, respectivamente, tais que MX, NY, PZ tenham
comprimentos respectivamente iguais s metades das alturas do
tringulo a partir dos vrtices A, B, C. Prove que XYZ semelhante
ao tringulo rtico de ABC (tringulo formado pelos ps das alturas do ABC).

EUREKA! Edio Especial, 2007

32

Sociedade Brasileira de Matemtica

CONTAR DUAS VEZES PARA GENERALIZAR


(O RETORNO)
Jos Paulo Q. Carneiro, Universidade Santa rsula

Nvel Intermedirio
1. A frmula que d diretamente a soma dos quadrados
S n( 2) = 12 + 22 + + n 2 dos n primeiros inteiros positivos pode ser
deduzida de vrias maneiras (por exemplo, [3]). Uma das mais comuns
3
partir da identidade: (k + 1) k 3 = 3k 2 + 3k + 1 , escrev-la para k
variando de 1 at n:
23 13 = 3 12 + 3 1 + 1
33 23 = 3 22 + 3 2 + 1
.........................................

(n + 1)3 n3 = 3n 2 + 3n + 1

e somar termo a termo estas n igualdades, obtendo:

(n + 1)3 13 = 3Sn( 2) + 3Sn(1) + n

n(n + 1)
, como bem conhecido (ver [1]).
2
Substituindo este valor e fazendo as contas, chega-se a :
n(n + 1)(2n + 1)
S n( 2) = 12 + 22 + + n 2 =
6
Esta deduo bastante eficiente e rpida, mas, quando apresentada pela
primeira vez a um estudante, costuma deixar aquela sensao de coelho
tirado da cartola, devido ao aparecimento sbito de uma identidade cuja
motivao no se sabe de onde veio. Este tipo de sensao desperta
admirao em uns, mas em outros inspira uma frustrao, proveniente da
reflexo: eu nunca vou conseguir bolar um artifcio destes!. Coloca-se,
portanto, a questo: h algum problema onde a soma dos quadrados
aparea naturalmente? E, para este problema, h alguma outra maneira de
resolv-lo, por meio da qual possamos deduzir a frmula da soma dos
quadrados?
onde S n(1) = 1 + 2 +

EUREKA! Edio Especial, 2007

+n =

33

Sociedade Brasileira de Matemtica

2. Tradicionalmente, em problemas de contagem, o smbolo Cnp


( combinao de n, p a p) representa o nmero de subconjuntos de p
elementos contidos em um conjunto de n elementos. Se, por exemplo,
fizermos p = 2 , ento Cn2 o nmero de pares (no ordenados) que se
pode extrair de um conjunto com n elementos. Exemplos: o nmero de
apertos de mo dados por n pessoas quando cada uma cumprimenta todas
as outras somente uma vez, ou ainda o nmero de partidas de futebol em
um campeonato com um s turno e n equipes. Em [1], um artigo com o
mesmo ttulo que o presente aproveitava justamente o ltimo exemplo
citado para mostrar como, resolvendo um mesmo problema de contagem
por dois
mtodos diferentes, era possvel deduzir que:
(n 1)n
Cn2 = 1 + 2 + + (n 1) =
.
2
3. Os pitagricos (sec.VI a.C.) chamavam os nmeros Cn2 de nmeros
triangulares.
nas figuras:

O motivo que eles podem ser vistos como tringulos

T1 = 1 T2 = 1 + 2 = 3

T3 = 1 + 2 + 3 = 6 T4 = 1 + 2 + 3 + 4 = 10

Deste modo: Cn2 = Tn 1 , para n > 1 .


Alm dos nmeros triangulares, os pitagricos consideravam tambm os
nmeros quadrados Q1 = 12 = 1 , Q2 = 22 = 4 , etc., que podem ser
visualizados como quadrados (da seu nome).
Estas figuras pitagricas sugerem tambm uma relao interessante entre os
nmeros triangulares e os nmeros quadrados. Se voc partir o quadrado
usando a diagonal sudoeste-nordeste, e incluindo esta diagonal na parte de
baixo, voc poder olhar cada nmero quadrado como a soma de dois
nmeros triangulares consecutivos; mais especificamente: Qn = Tn 1 + Tn .

EUREKA! Edio Especial, 2007

34

Sociedade Brasileira de Matemtica

22 = 1 + 3

32 = 3 + 6

42 = 6 + 10

Esta relao pode, claro, ser confirmada algebricamente, j que:


(n 1)n n(n + 1)
Tn 1 + Tn =
+
= n 2 = Qn .
2
2
4. A observao precedente pode ser usada para calcular a soma dos
quadrados dos n primeiros nmeros naturais. De fato:
Q1 =
T1
Q2 = T1 + T2
Q3 = T2 + T3
...................
Qn = Tn 1 + Tn

Somando
termo
a
termo,
temos:
( 2)
S n = Q1 + + Qn = 2(T1 + + Tn 1 ) + Tn . S resta agora calcular
T1 + + Tn 1 , isto , a soma dos n 1 primeiros nmeros triangulares.
Para isto, lembremos que esta soma o mesmo que C22 + C32 + Cn2 , a
qual vamos calcular pelo artifcio de resolver um mesmo problema por
duas contagens diferentes (ver [1]).
O nmero de subconjuntos de 3 elementos contidos em um conjunto A de
n + 1 elementos representado, como j se sabe, por C n3+1 . Vamos contar
estes subconjuntos.
Para formar um subconjunto de A com 3 elementos, primeiramente
escolhemos um elemento a A . Para isto, temos n + 1 escolhas. Uma
vez escolhido a, temos n escolhas possveis para tomar um segundo
EUREKA! Edio Especial, 2007

35

Sociedade Brasileira de Matemtica

elemento b; e para cada escolha de a e b, temos n 1 escolhas possveis


para selecionar o terceiro elemento c. Isto d ento um total de
(n + 1)n(n 1) escolhas. Mas claro que esta contagem inclui repeties.
Para cada a, b, c escolhidos, houve 6 repeties, correspondentes s 6
permutaes destes elementos, a saber: a, b, c ; a, c, b ; b, a, c ; b, c, a ;
c, a, b ; c, b, a . Portanto: C n3+1 =

(n + 1)n(n 1)
.
6

Por outro lado, se quisermos evitar desde o incio as repeties, podemos


contar do seguinte modo. Primeiramente, fixamos o elemento a;
o nmero de subconjuntos de A com 3 elementos e que possuem a o
mesmo que o de subconjuntos de A {a} com 2 elementos, isto : Cn2 .
Tomemos agora um segundo elemento b a . O nmero subconjuntos de
A com 3 elementos, que possuem b mas no a, o mesmo que o de
subconjuntos de A {a; b} com 2 elementos, isto : Cn21 . Analogamente,
o nmero subconjuntos de A com 3 elementos, que contm c, mas no
intersectam {a, b} , o mesmo que o de subconjuntos de A {a; b; c} com
2 elementos, isto : Cn2 2 . E assim por diante, at que cheguemos ao
antepenltimo elemento, quando j teremos contado todos os
subconjuntos A com 3 elementos. Logo: Cn3+1 = Cn2 + Cn21 + + C22 .
Deste modo, conclumos que:
(n + 1)n(n 1)
T1 + + Tn 1 = C22 + C32 + Cn2 = Cn3+1 =
. Conseguimos,
6
portanto, calcular a soma dos n 1 primeiros nmeros triangulares. Da
conclumos que:
(n + 1)n(n 1) (n + 1)n
S n( 2 ) = Q1 + + Qn = 2(T1 + + Tn 1 ) + Tn =
+
3
2
n(n + 1)(2n + 1)
=
.
6
Podemos generalizar as frmulas acima, calculando de duas maneiras
diferentes o nmero de subconjuntos de k + 1 elementos contidos em um
conjunto A de n + 1 elementos, que representado por C nk++11 .
A primeira expresso para C nk++11 clssica e pode ser provada do mesmo
modo que foi feito para k + 1 = 3: temos
EUREKA! Edio Especial, 2007

36

Sociedade Brasileira de Matemtica

C nk++11 =

(n + 1)n( n 1)(n 2)...(n k + 1)


( n + 1)!
=
(k + 1)!
(k + 1)!(n k )!

(lembremos que m! = 1 . 2 . ... . m).


Seja agora A = {a1 , a 2 ,..., a n +1 }. O nmero de subconjuntos de k + 1
elementos de A que contm a1 C nk (escolhemos os k elementos de
A diferentes de a1 ). O nmero de subconjuntos de k + 1 elementos de A
que contm a 2 mas no contm a1 C nk1 , e assim sucessivamente, o que
mostra a igualdade
C nk++11 = C nk + C nk1 + ... + C kk .
(n + k 1)(n + k 2)...(n + 1)n
o "polinmio triangular
Se Pk (n) = C nk+ k 1 =
k!
generalizado de dimenso k", temos que Pk (n) um polinmio em n de

grau k, e, pela frmula acima, temos


Pk (1) + Pk (2) + ... + Pk (m) = C kk + C kk+1 + ... + C mk + k 1 = C mk ++1k .

Podemos, como antes, escrever n k como uma combinao linear dos


polinmios Pj (n),0 j k , e usar a frmula acima para obter uma
frmula para S n( k ) = 1k + 2 k + ... + n k (essa frmula ser a combinao
correspondente dos termos C nj++1j , com 0 j k ).
Tal frmula tambm pode ser obtida recursivamente como no incio do
k

artigo, somando as identidades ( j + 1) k +1 j k +1 = C kr+1 j r , desde j = 1


at j = n, ficando o lado esquerdo igual a (n + 1)
k 1

(k + 1) S n( k ) + C kr +1 S n( r ) , o que d S n( k ) =
r =0

r =0
k +1

1 e o direito igual a

k 1

1
(n + 1) k +1 1 C kr +1 S n( r ) .
k +1
r =0

Referncias Bibliogrficas:
[1] Carneiro, J.P., Contar duas vezes para generalizar, Eureka!, n6, pp.15-17, 1999.
[2] Eves, H., Introduo Histria da Matemtica, Editora da UNICAMP, 1995
[3] Valadares, E.C., e Wagner, E., Usando geometria para somar, Revista do Professor
de Matemtica, n39, pp.1-8, 1999.

EUREKA! Edio Especial, 2007

37

Sociedade Brasileira de Matemtica

OS NMEROS IRRACIONAIS
Hermano Frid
Instituto de Matemtica Pura e Aplicada - IMPA

Nvel Intermedirio.
No texto a seguir fazemos uma breve introduo ao conceito de
nmero irracional. Na sua maior parte o texto ser acessvel a alunos
da ltima srie do primeiro grau. As duas ltimas sees talvez
requeiram um pouco mais de maturidade embora no exijam nenhum conhecimento prvio adicional. Para simplificar a exposio nos
restringiremos a nmeros positivos. A extenso dos fatos abordados ao
contexto geral de nmeros positivos, negativos e 0 no requer nenhuma
dificuldade adicional.
Pode-se imaginar que a idia de nmero inteiro positivo tenha surgido
num estgio primrio da civilizao, juntamente com a necessidade da
prtica da contagem. Por exemplo, era necessrio a um pastor saber
contar de algum modo o nmero de animais no seu rebanho. A maneira
de representar o resultado dessa contagem era no incio bastante diferente
da que usamos agora e provvel que no comeo cada pessoa tivesse sua
maneira prpria de faz-lo. Contar significa estabelecer um modo de
comparar quantidades de elementos de conjuntos distintos. Por exemplo,
a quantidade de pedrinhas em um saco com a quantidade de animais num
rebanho, ou a quantidade de alimentos conseguidos em uma caada ou
em colheita com a quantidade de membros da tribo. Tambm no difcil
imaginar que a ideia de frao tenha surgido na evoluo da civilizao
humana, primeiramente e de forma mais elementar, com a ocorrncia
usual da necessidade de um determinado grupo de pessoas partilhar um
ou mais bens de propriedade comum entre seus membros. E num estgio
mais avanado, dentre outras motivaes possveis, com a necessidade de
as pessoas trocarem entre si bens de tipos distintos. Por exemplo, um
pastor deseja trocar com um agricultor peles de carneiro por sacos de
milho numa razo de 3 peles de carneiro para cada grupo de 7 sacos
de milho. Por outro lado, a idia de um nmero que no seja nem
inteiro nem frao , em princpio, muito menos natural que a daqueles e
surge num estgio muito mais avanado da civilizao com a necessidade
da prtica da medio. Por exemplo, medir as dimenses ou a rea de um
terreno, comparar as distncias entre pares de pontos distintos, etc.
EUREKA! Edio Especial, 2007

38

Sociedade Brasileira de Matemtica

Procuraremos, a seguir, mostrar as propriedades bsicas destes nmeros


estranhos em contraste com as propriedades, na maior parte j bem conhecidas, daqueles mais intuitivos, os inteiros e as fraes.
1. BASE DECIMAL; DZIMAS

Os nmeros reais positivos podem ser representados no sistema decimal


por uma seqncia de algarismos elementos do conjunto {0, 1, 2, 3, 4,
5, 6, 7, 8, 9} Separados por uma vrgula. Assim, se
a N , a N 1 ,..., a 0 , a 1 , a 2 , a 3 ,..., so algarismos quaisquer, um nmero real
positivo representado no sistema decimal tem a forma
a N a N 1 a N 2 ...a1 a 0 , a 1 a 2 a 3 ...,
(1)
onde a N > 0. Nessa representao, esquerda da vrgula temos sempre
um nmero finito de algarismos, porm direita podemos ter uma
infinidade de algarismos. Por exemplo, 783,5231 representa o nmero
obtido como resultado da expresso
7 10 2 + 8 10 1 + 3 10 0 + 5 10 1 + 2 10 2 + 3 10 3 + 1 10 4 . (2)
Por outro lado, a frao

154
999

tem representao decimal 0, 1545454

com uma infinidade de algarismos direita. Essa representao se traduz


como resultado de uma expresso com infinitas parcelas
(3)
1 10 1 + 5 10 2 + 4 10 3 + 5 10 4 + 4 10 5 + 5 10 6 + ...
154
Essa expresso significa exatamente que se quisermos aproximar
no
999

sistema decimal com preciso de 8 casas decimais, por exemplo,


devemos tomar como aproximao o nmero 0,15454545 que resultado
da expresso
1 10 1 + 5 10 2 + 4 10 3 + 5 10 4 + 4 10 5 + 5 10 6
+ 4 10 7 + 5 10 8.
(4)
Claro, o nmero 0, 1545454 o que chamamos de uma dzima
peridica e por isso pode ser obtido como uma frao 154 .
999

EUREKA! Edio Especial, 2007

39

Sociedade Brasileira de Matemtica

O QUE ACONTECE NO CASO DE UMA DZIMA NO-PERIDICA?

Neste caso, assim como no peridico, temos uma infinidade de


algarismos direita da vrgula e assim s nos possvel escrever a
representao decimal at uma certa casa decimal, porm, diferentemente
do que acontece no caso peridico, no h repetio indefinidamente de
um determinado grupo de algarismos e, assim, o nmero em questo no
pode ser obtido como uma frao p com e e q diferente de 0. Os nmeq

ros que podem ser obtidos como fraes so chamados racionais; os que
no podem ser obtidos como fraes so chamados irracionais.
2. POR QUE PRECISAMOS DOS NMEROS IRRACIONAIS?

Responderemos esta pregunta atravs de um exemplo. Euclides provou


que o nmero positivo cujo quadrado 2, isto , o nmero positivo x que
satisfaz a equao x 2 = 2,
(5)
no racional. Euclides argumentou da seguinte forma:
Suponhamos que o nmero x satisfazendo (5) seja racional. Ento existem inteiros positivos p e q, primos entre si, tais que
ou seja p 2 = 2q 2 .

p2
= 2.
q2

(6)

Portanto p par e p tambm par; p pode ser escrito na forma p = 2k.


Assim, (2k ) 2 = 2q 2 2k 2 = q 2
(7)
Pela mesma razo que acabamos de expor, conclumos que q tambm
deve ser par. Mas isto nos leva a uma contradio pois p e q so primos
entre si por hiptese! Assim, a suposio de que x = p nos leva a uma
q

contradio e, portanto, deve ser descartada, considerada falsa.


Chegamos concluso que 2 , que como representamos o nmero
positivo cujo quadrado 2, um nmero irracional!!
3. COMO OBTER APROXIMAES RACIONAIS PARA

Podemos obter aproximaes cada vez melhores de 2 (o nmero x que


satisfaz (5)) atravs do seguinte procedimento que um caso particular
de um esquema inventado por Newton conhecido como mtodo de
Newton. (Com base nesse mtodo podemos programar as mquinas de
EUREKA! Edio Especial, 2007

40

Sociedade Brasileira de Matemtica

calcular para produzirem aproximaes de 2 to precisas quanto o


avano da eletrnica nos permitir). primeiro chutamos um nmero x0
como uma primeira aproximao de x que nos parea razovel; por
exemplo, x0 = 1. Em seguida observamos que
x 2 x02 = ( x + x0 )( x x 0 ) 2 x 0 ( x x0 ),
onde o smbolo significa aproximadamente igual a. Assim,
x 2 x02 2 x0 ( x x0 ),
e, portanto, dividindo a equao aproximada por 2x0 e arranjando os
x 2 x02
+ x0 .
termos, obtemos x
2 x0
substituindo

x2 = 2 e

(8)

x0 = 1 em

(8),

obtemos

3
2 1
+1= .
2
2

Assim temos uma segunda aproximao x1 = 3 . Encontramos tambm


x2 :

2
9
2
4 + 3 x 1 1 + 3 x 1 + 3 x 2 17 . Da
x2
2
2
3
2
4 3 2
12
2
12

mesma forma, podemos obter uma quarta aproximao x3 , fazendo


x 2 x 22
2 (17 / 12) 2 17
+ x2 =
+
2 x2
17 / 6
12
288 289 17 288 289 + 2 289 577
=
+
=
=
.
2 12 17 12
2 12 17
408
Assim, x3 = 577 seria a aproximao seguinte: Sua representao decimal
408
x3 =

dzima

peridica

x3 = 1,414215686274509803921568627...9....
perodo

Agora se voc pegar uma mquina de calcular e pedir (atravs dos


devidos comandos) que ela calcule 2 , voc obter, se sua mquina
puder exibir 33 dgitos (incluindo a vrgula ou ponto), a
expresso decimal
1,4142135623730950488016887242097.
EUREKA! Edio Especial, 2007

41

Sociedade Brasileira de Matemtica

Horrvel, no ? Voc obter uma expresso ainda maior se sua mquina


puder exibir mais dgitos. Repare como nossas aproximaes x1 , x 2 e x3
esto cada vez mais prximas desse nmero!
4. OS NMEROS RACIONAIS PODEM SER ENUMERADOS

Isto significa que podemos dispor os nmeros racionais numa sucesso


da forma r1 , r2 , r3 ,..., com uma infinidade de elementos. Podemos
interpretar este fato como significando que a quantidade de nmeros racionais, embora sendo infinita, de uma ordem de infinitude equivalente a dos nmeros naturais 1, 2, 3. O argumento para a demonstrao
desse fato devido a Georg Cantor.
Como todo racional tem uma representao nica como frao p com p
q

e q inteiros positivos primos entre si, basta que saibamos enumerar os


pares ordenados (p, q) de naturais primos entre si. A forma de obter essa
enumerao est descrita pela figura abaixo:

A enumerao obtida seguindo-se o caminho indicado pelas flechas,


iniciando a partir de (1,1), tendo o cuidado de descartar os pares de
naturais que no so primos entre si, como, por exemplo, (2,2), (4,2),
(3,3) etc.. Com isso, teramos
r1 =

3
1
1
1
2
= 1, r2 = , r3 = = 2, r4 = = 3, r5 = , etc.
2
1
1
3
1

EUREKA! Edio Especial, 2007

42

Sociedade Brasileira de Matemtica

5. REPRESENTAO DECIMAL DOS RACIONAIS

H pouco dissemos que no era possvel pr uma dzima no peridica


em forma de frao p com p e q naturais primos entre si. Vamos dar uma
q

explicao para este fato. Fixemos um natural q. Quando dividimos um


nmero qualquer N > q pelo nmero q. Obtemos como resto da diviso
um elemento do conjunto (finito) {0, 1, 2,, q 1}. Tomemos como
exemplo q = 7 e N = 17; nesse caso os restos possveis pertencem ao
conjunto {0, 1, 2, 3, 4, 5, 6}. Agora vamos recordar o algoritmo da
diviso com esse exemplo especfico:
17 7
1 4 2, 4 2 8 5 7 1 4 2 8 5 7 1
30
28
20
14
60
56
40
35
50
49
10
7
3 0

O que acontece que os restos possveis so elementos do conjunto finito


de q elementos {0, 1,, q 1}(no exemplo acima q = 7). Assim, em no
mximo q iteraes do algoritmo ou acabamos repetindo um elemento do
conjunto de restos possveis (no exemplo acima o primeiro a se repetir
foi o 3), ou o 0 ocorre como resto e o processo termina. No primeiro
caso, a partir da passamos a repetir os restos ocorridos anteriormente na
mesma ordem (3, 2, 6, 4, 5, 1, no exemplo acima). As casas decimais
no quociente por sua vez tambm se repetem o obtemos ento uma
dzima peridica. No segundo caso, obtemos simplesmente um nmero
finito de casas decimais.

EUREKA! Edio Especial, 2007

43

Sociedade Brasileira de Matemtica


6. REPRESENTAO DECIMAL DOS IRRACIONAIS

Todo nmero irracional positivo possui uma representao decimal nica


por meio de uma dzima no peridica. Para simplificar vamos nos
restringir aos nmeros entre 0 e 1. J sabemos que um nmero cuja
representao decimal possui uma quantidade finita de casas decimais
pertence ao conjunto dos racionais. Da mesma forma aprendemos que um
nmero cuja representao decimal uma dzima peridica tambm um
nmero racional. Por outro lado, vimos no item anterior que as
representaes decimais de um racional so necessariamente de um dos
dois tipos: ou possuem uma quantidade finita de casas decimais, ou terminam em uma dzima peridica. Logo, uma representao decimal para um nmero irracional tem necessariamente que ser uma dzima noperidica. Afirmamos que essa representao nica. Repare que isso
no ocorre em geral com os racionais. Por exemplo, 0, 21 e 0, 20999
representam o mesmo racional 21 . Suponhamos que um irracional x
100

entre 0 e 1 possua duas representaes decimais distintas:


x = 0, a 1 a 2 a 3 ...,
x = 0, b1b 2 b3 ...,

(10)
(11)

Se essas representaes so distintas certamente existe um p N tal que


a k = b k , para k = 0,..., p 1, e a p b p . Para fixar idias vamos
assumir ento que a p b p + 1 e por (10) e (11)
x 0, a 1 a 2 ...a p ,
x 0, a 1 a 2 ...b p 999... = 0, a 1 a 2 ...(b p + 1),

(12)
(13)

j que b k = a k se k = 0,..., p 1 e b k sempre menor ou igual a 9.


Mas (12) e (13) implicam que a p = b p + 1 e x = 0, a 1 a 2 ...a p .
Porm nesse caso x racional e chegamos a uma contradio!
Chegaramos a uma contradio semelhante tambm se tivssemos
assumido b p > a p , argumentando da mesma forma apenas trocando os
papis dos a k e b k . A contridio tem origem no fato de termos
suposto que havia duas representaes decimais distintas para o mesmo
EUREKA! Edio Especial, 2007

44

Sociedade Brasileira de Matemtica

irracional x. Logo essa possibilidade tem que ser descartada, considerada


falsa, e assim conclumos que todo irracional possui uma representao
decima nica como dzima no-peridica.
7. OS IRRACIONAIS NO PODEM SER ENUMERADOS

Isto significa que no podemos dispor os nmeros irracionais numa


sucesso s1 , s 2 , s 3 ,..., mesmo admitindo uma infinidade de elementos.
Quer dizer, diferentemente dos racionais, a ordem de infinitude da
quantidade dos nmeros irracionais maior que a dos nmeros naturais.
Conclumos da que existem muito mais nmeros irracionais do
que racionais!
Vamos tentar justificar nossa afirmao sobre a no-enumerabilidade dos
irracionais. O argumento uma adaptao de uma idia tambm devida a
G. Cantor. Suponhamos que fosse possvel dispor os irracionais numa
sucesso s1 , s 2 , s 3 ,..., . Basta considerarmos apenas os irracionais entre 0
e 1. Criamos um nmero irracional x, tambm entre 0 e 1, atravs de uma
representao decimal (portanto, uma dzima no peridica) da seguinte
forma. O nmero x tem representao decimal dada por
x = 0, x 1 x 2 x 3 ... onde x p escolhido dentro do conjunto {0, 1, , 9}
de modo que x p diferente de ( s p ) p onde este ltimo o algarismo que
aparece na casa decimal de ordem p do irracional s p (p-sima elemento
da sucesso s1 , s 2 ,...s p ,...). A escolha de cada x p tambm deve atender a
condio de no permitir que nenhum grupo de algarismos dentre os j
escolhidos x 1 , x 2 ,..., x ( p 1) possa se tornar o gerador de uma dzima
peridica. Desta forma obtemos uma dzima no peridica representando
um nico irracional que, no entanto, no pode constar na lista
s1 , s 2 , s 3 ,..., . De fato, se x = s r , para algum r N, ento como
x r ( s r ) r teramos um absurdo (uma contradio)!.
8. ESTUDO SUPLEMENTAR: O IRRACIONAL

O nmero definido como sendo a rea limitada por um crculo de raio


1. Ele certamente o irracional transcendente mais conhecido. A
expresso transcendente significa, neste contexto, um nmero irracional
que no raiz de nenhuma equao polinomial com coeficientes inteiros.
EUREKA! Edio Especial, 2007

45

Sociedade Brasileira de Matemtica

Por exemplo, os irracionais 2 ,1 + 3 no so transcendentes pois so


razes
das
equaes
polinomiais x 2 = 2, x 2 2 x 2 = 0,
respectivamente. Neste ltimo caso dizemos que os nmeros so
algbricos. A demonstrao de que um nmero irracional, apesar de
no ser trivial, pode ser feita usando-se apenas o clculo diferencial elementar que ensinado no primeiro perodo dos cursos de cincias exatas.
A primeira demonstrao de que irracional s foi obtida em 1766 por
J. H. Lambert, de forma no completamente rigorosa, tendo sido finalmente (re)obtida de modo rigoroso pelo famoso matemtico A. M. Legendre e publicada em 1855. A prova de que transcendente muito
mais complexa e s foi obtida em 1882 por F. Lindermann.
O fabuloso matemtica grego Arquimedes foi o primeiro a obter uma
aproximao razovel de por numeros racionais. Ele provou que
3+

10
1
< < 3+ ,
71
7

usando dois polgonos regulares de 96 lados, um inscrito e outro


circunscrito a um crculo de raio 1.
Podemos obter aproximaes cada vez melhores de , com o auxlio de
uma mquina de calcular bastante rudimentar, capaz apenas de fazer as
operaes bsicas (+, , ) e mais a operao de extrair raiz quadrada, da
seguinte forma. A idia aproximarmos o crculo de raio 1 por polgonos
regulares de 2 n lados inscritos neste crculo. Primeiramente, fcil
verificar que para a rea e o permetro do polgono regular de 2 n lados
inscritos num crculo de raio 1 temos
rea =

1
Permetro 4 l 2 ,
4

onde l o comprimento do lado do polgono. Como l se aproxima mais e


mais de 0 a medida que n cresce, vemos que para o crculo de
raio 1 devemos ter (fazendo l = 0 na frmula acima)
rea =

1
4

Permetro

Assim, podemos tambm definir como sendo a metade do permetro do


crculo de raio 1. Por outro lado, usando o teorema de Pitgoras que diz
que em um tringulo retngulo o quadrado da hipotenusa a soma dos
EUREKA! Edio Especial, 2007

46

Sociedade Brasileira de Matemtica

quadrados dos catetos, se l n denota o comprimento do lado do polgono


regular de 2 n lados, fcil mostrar que
l n +1 =

4 l n2 .

(14)
Para n = 2 temos o polgono regular de 4 lados, quadrado, inscrito no
crculo de raio 1, cujo lado, facilmente obtido usando-se o teorema
de Pitgoras,
l2 = 2.

Por meio de (14) obtemos sucessivamente


l3 = 2 2 ,
l4 = 2 2 + 2 ,
l5 = 2 2 + 2 + 2 ,
l6 = 2 2 + 2 + 2 + 2 ,
l7 =

l8 =

2+

2+

2+

2+

2 ,

2 2+ 2+ 2+ 2+ 2+ 2 ,

Para obter uma boa aproximao de calculemos, por exemplo, o valor


da metade do permetro do polgono de 2 8 = 256 lados, inscrito no
crculo de raio 1, cujo lado tem comprimento igual a l8 . Podemos obter
um valor aproximado para l8 executando a seguinte seqncia de
operaes numa calculadora
2 sqrt + 2 = sqrt + 2 = sqrt + 2 = sqrt
+ 2 = sqrt + 2 = sqrt + / + 2 = sqrt,
e obtemos
l8 = 0.0245430765714398521588165239020064.
EUREKA! Edio Especial, 2007

47

Sociedade Brasileira de Matemtica

Agora, multiplicaremos o resultado obtido para l8 por 256, que o


nmero de lados da polgono em questo, e em seguida dividimos por 2
o que nos d
3.14151380114430107632851505945682
~
o que fornece uma aproximao com erro menor que 0, 0001 j que
sabido que
3, 1415 < < 3, 1416.
Consideraes finais: Exceto pelas duas ltimas sees, o texto acima foi elaborado a
partir de um pedido de minha filha, Marina, atualmente na 8a. srie do primeiro grau,
urgida por um trabalho de casa em grupo passado por sua professora. O referido
trabalho, felizmente, resultou bastante diferente do que foi exposto acima, que acabou
servindo apenas como uma entre vrias referncias usadas pelo grupo. No entanto, as 7
primeiras sees foram bem compreendidas por ela e seu grupo; as duas ltimas foram
escritas depois que o prazo para a entrega do trabalho havia esgotado e, portanto, no
chegaram a ser testadas. Para concluir gostaria de deixar aqui meus agradecimentos
ao estimado professor e colega Elon Lages Lima pelas sugestes sobre uma verso
preliminar destas notas.

EXERCCIOS:

1) Usando o mesmo argumento de Euclides descrito em 2. prove que


3 , 5 e 7 so irracionais.
2) Usando o mtodo de Newton, descrito em 3, obtenha aproximaes
correspondentes ao x3 do texto para os irracionais 3 , 5 , 7 e
compare com o resultado fornecido pela mquina de calcular.
3) Pesquise sobre a vida e a obra dos grandes matemticos mencionados
no texto: Arquimedes, Pitgoras, Euclides, Isaac Newton e Georg
Cantor.
4) Prove a frmula (14).

EUREKA! Edio Especial, 2007

48

Sociedade Brasileira de Matemtica

XXVII OLIMPADA BRASILEIRA DE MATEMTICA


Problemas e Solues da Primeira Fase
PROBLEMAS NVEL 1

1. Sabendo-se que 9 174 532 13 = 119 268 916 , pode-se concluir que

divisvel por 13 o nmero:


A) 119 268 903 B) 119 268 907
D) 119 268 913 E) 119 268 923

C) 119 268 911

2. Numa caixa havia 3 meias vermelhas, 2 brancas e 1 preta. Professor

Piraldo retirou 3 meias da caixa. Sabendo-se que nenhuma delas era


preta, podemos afirmar sobre as 3 meias retiradas que:
A) so da mesma cor.
B) so vermelhas.
B) uma vermelha e duas so brancas.
D) uma branca e duas so vermelhas.
E) pelo menos uma vermelha.
3. Diamantino colocou em um recipiente trs litros de gua e um litro de

suco composto de 20% de polpa e 80% de gua. Depois de misturar


tudo, que porcentagem do volume final polpa?
A) 5%
B) 7%
C) 8%
D) 20
E) 60%
4. Perguntado, Arnaldo diz que 1 bilho o mesmo que um milho de

milhes. Professor Piraldo o corrigiu e disse que 1 bilho o mesmo


que mil milhes. Qual a diferena entre essas duas respostas?
A) 1 000
B) 999 000 C) 1 000 000
D) 999 000 000 E) 999 000 000 000
5. Numa seqncia, cada termo, a partir do terceiro, a soma dos dois

termos
anteriores mais prximos. O segundo termo igual a 1 e o quinto
termo vale 2005. Qual o sexto termo?
A) 3 002 B) 3 008
C) 3 010
D) 4 002
E) 5 004
EUREKA! Edio Especial, 2007

49

Sociedade Brasileira de Matemtica

6. Um galo de mel fornece energia suficiente para uma abelha

voar 7 milhes de quilmetros. Quantas abelhas iguais a ela


conseguiriam voar mil quilmetros se houvesse 10 gales de mel para
serem compartilhados entre elas?
A) 7 000
B) 70 000
C) 700 000
D) 7 000 000
E) 70 000 000
7. Trs anos atrs, a populao de Pirajussara era igual populao que

Tucupira tem hoje. De l para c, a populao de Pirajussara no


mudou mas a populao de Tucupira cresceu 50%. Atualmente, as
duas cidades somam 9000 habitantes. H trs anos, qual era a soma
das duas populaes?
A) 3 600
B) 4 500
C) 5 000
D) 6 000
E) 7 500
8. Um agricultor esperava receber cerca de 100 mil reais pela venda de

sua safra. Entretanto, a falta de chuva provocou uma perda da safra


avaliada entre 1 e 1 do total previsto. Qual dos valores a seguir pode
5

representar a perda do agricultor?


A) R$ 21.987,53
B) R$ 34.900,00
D) R$ 51.987,53
E) R$ 60.000,00

C) R$ 44.999,99

9. Devido a um defeito de impresso, um livro de 600 pginas apresenta em branco


todas as pginas cujos nmeros so mltiplos de 3 ou de 4. Quantas pginas esto
impressas?
A) 100
B) 150
C) 250
D)300 E) 430

10. Seis retngulos idnticos so reunidos pa-

ra formar um retngulo maior conforme


indicado na figura. Qual a rea deste retngulo maior?
A) 210 cm2
C) 430 cm2
E) 588 cm2

EUREKA! Edio Especial, 2007

B) 280 cm2
D) 504 cm2

50

21 cm

Sociedade Brasileira de Matemtica

11. O relgio do professor Piraldo, embora preciso, diferente, pois seus

ponteiros se movem no sentido anti-horrio. Se voc olhar no espelho


o relgio quando ele estiver marcando 2h23min, qual das seguintes
imagens voc ver?
E

A)

B)

E)

12. Uma

placa decorativa consiste num


quadrado de 4 metros de lado, pintada de
forma simtrica com algumas faixas, conforme indicaes no desenho ao lado. Qual
a frao da rea da placa que foi pintada?

A)

1
2

B)

1
3

C)

3
8

D)

E
D)

C)

6
13

E)

1m
1m
1m
1m
1m

7
11

1m

13. Pelculas de insulfilm so utilizadas em janelas de edifcios e vidros de veculos


para reduzir a radiao solar. As pelculas so classificadas de acordo com seu grau
de transparncia, ou seja, com o percentual da radiao solar que ela deixa passar.
Colocando-se uma pelcula de 70% de transparncia sobre um vidro com 90% de
transparncia, obtm-se uma reduo de radiao solar igual a :

A) 3%
63%

B) 37%
E) 160%

C) 40%

D)

14. Na figura, os dois tringulos so eqilteros. Qual o valor do

ngulo x?
x

75

A) 30o
EUREKA! Edio Especial, 2007

65

B) 40o

C) 50o
51

D) 60o

E) 70o

Sociedade Brasileira de Matemtica

15. Um serralheiro solda varetas de metal para produzir

10

peas iguais que sero juntadas para formar o painel


abaixo. O desenho ao lado apresenta as medidas, em
centmetros, de uma dessas peas. O serralheiro usa
exatamente 20 metros de vareta para fazer o seu
trabalho.

10
5

5
10

Qual dos desenhos abaixo representa o final do painel?

C)

B)

A)

D)

E)

16. Dentre os nmeros 1, 2, 3, 4, 5, 6, 7, 8, 9 e 10, escolha alguns e

coloque-os nos crculos brancos de tal forma que a soma dos nmeros
em dois crculos vizinhos seja sempre um quadrado perfeito.
Ateno: o 2 j foi colocado em um dos crculos e no permitido
colocar nmeros repetidos; alm disso, crculos separados pelo
retngulo preto no so vizinhos.
2

A soma dos nmeros colocados em todos os crculos brancos :


A) 36
B) 46
C) 47
D) 49

EUREKA! Edio Especial, 2007

52

E) 55

Sociedade Brasileira de Matemtica

17. Figuras com mesma forma representam objetos de mesma massa.

Quantos quadrados so necessrios para que a ltima balana fique


em equilbrio?

A) 7

B) 8

C) 9

D) 10

E) 12

18. As 10 cadeiras de uma mesa circular foram numeradas com nmeros

consecutivos de dois algarismos, entre os quais h dois que so quadrados perfeitos. Carlos sentou-se na cadeira com o maior nmero e
Janana, sua namorada, sentou-se na cadeira com o menor nmero.
Qual a soma dos nmeros dessas duas cadeiras?
A) 29
B) 36
C) 37
D) 41
E) 64
19. Em um ano, no mximo quantos meses tm cinco domingos?

A) 3

B) 4

C) 5

D) 6

E) 7

20. As nove casas de um tabuleiro 3 3 devem ser pintadas de foram que

cada coluna, cada linha e cada uma das duas diagonais no tenham
duas casas de mesma cor. Qual o menor nmero de cores necessrias para isso?
A) 3
B) 4
C) 5
D) 6
E) 7
PROBLEMAS NVEL 2

1. Uma loja de sabonetes realiza uma promoo com o anncio

"Compre um e leve outro pela metade do preo. Outra promoo


que a loja poderia fazer oferecendo o mesmo desconto percentual
A) "Leve dois e pague um
B) "Leve trs e pague um
C) "Leve trs e pague dois
D) "Leve quatro e pague trs
E) "Leve cinco e pague quatro
2. Veja o problema No. 13 do Nvel 1.
EUREKA! Edio Especial, 2007

53

Sociedade Brasileira de Matemtica

3. Veja o problema No. 10 do Nvel 1.


4. Veja o problema No. 4 do Nvel 1.
5. Veja o problema No. 9 do Nvel 1.
6. Platina um metal muito raro, mais raro at do que ouro. Sua

densidade 21,45 g/cm3. Suponha que a produo mundial de


platina foi de cerca de 110 toneladas em cada um dos ltimos 50
anos e desprezvel antes disso. Assinale a alternativa com o
objeto cujo volume mais prximo do volume de platina produzido
no mundo em toda a histria.
A) uma caixa de sapatos
B) uma piscina
C) um edifcio de dez andares
D) o monte Pascoal
E) a Lua

7. Veja o problema No. 5 do Nvel 1.


8. Veja o problema No. 17 do Nvel 1.
9. Entre treze reais no nulos h mais nmeros positivos do que

negativos. Dentre os 13 12 = 78 produtos


2

de

dois

dos

treze

nmeros, 22 so negativos. Quantos nmeros dentre os treze


nmeros dados so negativos?
A) 2
B) 7
C) 8
D) 9
E) 10
10. O desenho abaixo mostra um pedao de papelo que ser dobrado e

colado nas bordas para formar uma caixa retangular. Os ngulos nos
cantos do papelo so todos retos. Qual ser o volume da
caixa em cm3?

A) 1 500

B) 3 000

EUREKA! Edio Especial, 2007

20 cm

40 cm

15 cm

C) 4 500

54

D) 6 000

E) 12 000

Sociedade Brasileira de Matemtica

11. Sendo a, b e c nmeros reais, pela propriedade distributiva da

multiplicao em relao adio, verdade que a (b + c) = (a b)


+ (a c). A distributiva da adio em relao multiplicao a + (b
c) = (a + b) (a + c) no sempre verdadeira, mas ocorre se, e
somente se,
A) a = b = c = 1 ou a = 0

B) a = b = c

C) A igualdade nunca ocorre


E) a = b = c = 0

D) a + b + c = 1 ou a = 0

12. Em certa cidade, acontece um fato interessante. Dez por cento

dos Baianos dizem que so Paulistas e dez por cento dos


Paulistas dizem que so Baianos. Todos os outros Paulistas e
Baianos assumem a sua verdadeira origem. Dentre os Paulistas e
Baianos, 20% dizem que so Paulistas. Que percentual os realmente
Paulistas representam dentre os Paulistas e Baianos?
A) 12,5%

B) 18% C) 20% D) 22%

E) 22,5%

13. Veja o problema No. 14 do Nvel 1.


14. As letras O, B e M representam nmeros inteiros. Se O B M =

240, O B + M = 46 e O + B M = 64, quanto vale O + B + M?


A) 19

B) 20

C) 21

D) 24

E) 36

15. Veja o problema No. 15 do Nvel 1.


16. Veja o problema No. 19 do Nvel 1.
17. Quantos nmeros entre 10 e 13000, quando lidos da esquerda

para a direita, so formados por dgitos consecutivos e em ordem


crescente? Exemplificando, 456 um desses nmeros, mas 7890
no :
A) 10
B) 13
C) 18
D) 22
E) 25
EUREKA! Edio Especial, 2007

55

Sociedade Brasileira de Matemtica

18. Um piloto percorreu trs trechos de um rali, de extenses 240 km,

300 km e 400 km, respectivamente. As velocidades mdias nos trs


trechos foram 40 km/h, 75 km/h e 80 km/h, mas no necessariamente
nessa ordem. Podemos garantir que o tempo total em horas gasto pelo
piloto nos trs trechos :
A)
B)
C)
D)
E)

menor ou igual a 13 horas


maior ou igual a 13 horas e menor ou igual a 16 horas
maior ou igual a 14 horas e menor ou igual a 17 horas
maior ou igual a 15 horas e menor ou igual a 18 horas
maior ou igual a 18 horas

19. Na figura, todas as circunferncias menores tm o mesmo raio r e os

centros das circunferncias que tocam a circunferncia maior so


vrtices de um quadrado. Sejam a e b as reas cinzas indicadas na
figura. Ento a razo

a
igual a:
b

b
a

A) 1

B) 2

C) 1

D)

3
2

E) 2

20. Um professor de Ingls d aula particular para uma classe de 9

alunos, dos quais pelo menos um brasileiro. Se o professor escolher


4 alunos para fazer uma apresentao, ter no grupo pelo menos
dois alunos de mesma nacionalidade; se escolher 5 alunos, ter no
mximo trs alunos de mesma nacionalidade. Quantos brasileiros
existem na classe?
A) 1

B) 2

EUREKA! Edio Especial, 2007

C) 3

D) 4

E) 5

56

Sociedade Brasileira de Matemtica

21. Um

relgio, com ponteiros de horas, minutos e segundos,


faz plim toda vez que um ponteiro ultrapassa outro no
mostrador. O nmero de plins registrados em um certo dia, no
perodo entre as 12 horas e 1 segundo e as 23 horas, 59 minutos e 59
segundos :
A) 732
B) 1438
C) 1440
D) 1446
E) 1452

22. Na figura, a reta PQ toca em N o crculo que passa por L, M e N. A

reta LM corta a reta PQ em R. Se LM = LN e a medida do ngulo


PNL , < 60o, quanto mede o ngulo LRP?
L

A)3 180

B)180 2

C) 180

D) 90o /2 E)

23. Os inteiros positivos x e y satisfazem a equao

x + 12 y x 12 y = 1 .
Qual das alternativas apresenta um possvel valor de y?
A) 5
B) 6
C) 7
D) 8

24. Veja o problema No. 16 do Nvel 1.

EUREKA! Edio Especial, 2007

57

E) 9

Sociedade Brasileira de Matemtica

25. Um bloco de dimenses 1 2 3 colocado sobre um tabuleiro

8 8, como mostra a figura, com a face X, de dimenses 1 2, virada


para baixo. Giramos o bloco em torno de uma de suas arestas de
modo que a face Y fique virada para baixo. Em seguida, giramos
novamente o bloco, mas desta vez de modo que a face Z fique virada
para baixo. Giramos o bloco mais trs vezes, fazendo com que as
faces X, Y e Z fiquem viradas para baixo, nessa ordem.
Quantos quadradinhos diferentes do tabuleiro estiveram em
contato com o bloco?

Z
Y

A) 18

B) 19

C) 20

D) 21

E)22

PROBLEMAS NVEL 3

1. Veja o problema No. 17 do Nvel 2.


2. Os pontos L, M e N so pontos mdios de arestas do cubo, como mos-

tra a figura. Quanto mede o ngulo LMN?


N

L
o

A) 90

B) 105

C) 120o

3. Veja o problema No. 22 do Nvel 2.


EUREKA! Edio Especial, 2007

58

D) 135o

E) 150o

Sociedade Brasileira de Matemtica

4. Veja o problema No. 14 do Nvel 2.


5. Esmeralda digitou corretamente um mltiplo de 7 muito grande, com

4010 algarismos. Da esquerda para a direita, os seus algarismos so


2004 algarismos 1, um algarismo n e 2005 algarismos 2.
Qual o valor de n?
A) 3
B) 4
C) 5
D) 6
E) 7
6.
7.
8.
9.
10.

Veja o problema No. 23 do Nvel 2.


Veja o problema No. 25 do Nvel 2.
Veja o problema No. 1 do Nvel 2.
Veja o problema No. 6 do Nvel 2.
A figura mostra um cubo de aresta 1 no qual todas as doze
diagonais de face foram desenhadas. Com isso, criou-se uma rede
com 14 vrtices (os 8 vrtices do cubo e os 6 centros de faces) e 36
arestas (as 12 arestas do cubo e mais 4 sobre cada uma das 6 faces).
Qual o comprimento do menor caminho que formado por arestas
da rede e que passa por todos os 14 vrtices?

A) 1 + 6 2

B) 4 + 2 2

D) 8 + 6 2

C) 6

E) 12 + 12 2

11. Uma das faces de um poliedro um hexgono regular. Qual a quan-

tidade mnima de arestas que esse poliedro pode ter?


A) 7
B) 9
C) 12
D) 15

E) 18

12. Veja o problema No. 19 do Nvel 1.


13. O ponto D pertence ao lado BC do tringulo ABC. Sabendo que

AB = AD = 2, BD = 1 e os
ngulos BAD e CAD so congruentes,
ento a medida do segmento CD :
A)

3
2

EUREKA! Edio Especial, 2007

B)

4
3

C)
59

5
4

D)

6
5

E)

7
6

Sociedade Brasileira de Matemtica

14. Esmeralda adora os nmeros triangulares (ou seja, os nmeros

1, 3, 6, 10, 15, 21, 28), tanto que mudou de lugar os nmeros


1, 2, 3, , 11 do relgio de parede do seu quarto de modo que a soma de cada par de nmeros vizinhos um nmero triangular. Ela deixou o 12 no seu lugar original. Que nmero ocupa o lugar que era do
6 no relgio original?
A) 1
B) 4
C) 5
D) 10
E) 11
15. Os termos an de uma seqncia de inteiros positivos satisfazem a relao
an+3 = an+2(an+1 + an) para n = 1, 2, 3
Se a5 = 35, quanto a4?
A) 1
B) 3
C) 5
D) 7
E) 9
16. Veja o problema No. 11 do Nvel 2.
17. Veja o problema No. 19 do Nvel 2.
18. Entre treze reais no nulos h mais nmeros positivos do que
13 12
= 78 produtos de dois dos treze nmenegativos. Dentre os
2

ros, 22 so negativos. Quantos nmeros dentre os treze nmeros dados so negativos?


A) 2
B) 7
C) 8
D) 9
E) 10
19. Traando as quatro retas perpendiculares aos lados de um para-

lelogramo no retngulo pelos seus pontos mdios, obtm-se uma regio do plano limitada por essas quatro retas. Podemos afirmar que a
rea dessa regio igual rea do paralelogramo se um dos ngulos
do paralelogramo for igual a:
A) 30o
B) 45o
C) 60o
D) 75o
E) 90o
20. O nmero (2 + 2)3 (3 2) 4 + (2 2)3 (3 + 2) 4 :

A) inteiro mpar
C) racional no inteiro
E) irracional negativo

EUREKA! Edio Especial, 2007

B) inteiro par
D) irracional positivo

60

Sociedade Brasileira de Matemtica


2

21. Sejam A = 10(log10 2005) , B = 20053 e C = 2

A) A < B < C
C) B < A < C

. Ento:

2005

B) A < C < B
D) B < C < A

E) C < A < B

22. Veja o problema No. 18 do Nvel 2.


23. Dois nmeros inteiros so chamados

de primanos quando
pertencem a uma progresso aritmtica de nmeros primos com pelo
menos trs termos. Por exemplo, os nmeros 41 e 59 so primanos
pois pertencem progresso aritmtica (41; 47; 53; 59) que contm
somente nmeros primos. Assinale a alternativa com dois nmeros
que no so primanos.
A) 7 e 11 B) 13 e 53 C) 41 e 131 D) 31 e 43
E) 23 e 41

24. Um relgio, com ponteiros de horas, minutos e segundos, faz plim

toda vez que um ponteiro ultrapassa outro no mostrador. O nmero


de plins registrados em um certo dia no perodo entre as
12 horas e 1 segundo e as 23 horas, 59 minutos e 59 segundos :
A) 732
B) 1438
C) 1440
D) 1446
E) 1452
25. Veja o problema No. 20 do Nvel 2.
GABARITO
NVEL 1 (5a. e 6a. sries)
1) A
6) B
2) E
7) E
3) A
8) A
4) E
9) D
5) B
10) E
NVEL 2 (7a. e 8a. sries)
1) D
6) B
2) B
7) B
3) E
8) D
4) E
9) A
5) D
10) B
NVEL 3 (Ensino Mdio)
1) D
6) C
2) C
7) B
3) Anulada
8) D
4) B
9) B
5) B
10) A

EUREKA! Edio Especial, 2007

11) A
12) C
13) B
14) B
15) B

16) B
17) D
18) D
19) C
20) C

11) D
12) A
13) B
14) B
15) B

16) C
17) D
18) Anulada
19) C
20) C

21) Anulada
22) Anulada
23) C
24) B
25) B

11) C
12) C
13) B
14) C
15) D

16) D
17) C
18) A
19) B
20) B

21) C
22) Anulada
23) B
24) Anulada
25) C

61

Sociedade Brasileira de Matemtica

XXVII OLIMPADA BRASILEIRA DE MATEMTICA


Problemas e Solues da Segunda Fase
PROBLEMAS Nvel 1 PARTE A
(Cada problema vale 5 pontos)

01. O tanque do carro de Esmeralda, com capacidade de 60 litros, contm


uma mistura de 20% de lcool e 80% de gasolina ocupando metade de
sua capacidade. Esmeralda pediu para colocar lcool no tanque at que a
mistura ficasse com quantidades iguais de lcool e gasolina. Quantos litros de lcool devem ser colocados?
02. Na seqncia de nmeros 1, a, 2, b, c, d, ... dizemos que o primeiro
termo 1, o segundo termo a, o terceiro termo 2, o quarto termo b, e
assim por diante.
Sabe-se que esta seqncia tem 2005 termos e que cada termo, a partir do
terceiro, a mdia aritmtica de todos os termos anteriores. Qual o ltimo termo dessa seqncia?
03. Natasha supersticiosa e, ao numerar as 200 pginas de seu dirio,
comeou do 1 mas pulou todos os nmeros nos quais os algarismos 1 e 3
aparecem juntos, em qualquer ordem. Por exemplo, os nmeros 31 e 137
no aparecem no dirio, porm 103 aparece.
Qual foi o nmero que Natasha escreveu na ltima pgina do seu dirio?
04. Juliana foi escrevendo os nmeros inteiros positivos em quadrados de
papelo, colados lado a lado por fitas adesivas representadas pelos retngulos escuros no desenho abaixo. Note que cada fila de quadrados tem
um quadrado a mais que a fila de cima. Ela escreveu at o nmero 105 e
parou. Quantos pedaos de fita adesiva ela usou?
1

EUREKA! Edio Especial, 2007

62

10

Sociedade Brasileira de Matemtica

05. Lara tem cubos iguais e quer pint-los de maneiras diferentes, utilizando as cores laranja ou azul para colorir cada uma de suas faces.
Para que dois cubos no se confundam, no deve ser possvel girar um
deles de forma que fique idntico ao outro. Por exemplo, h uma nica
maneira de pintar o cubo com uma face laranja e cinco azuis.
Quantos cubos pintados de modos diferentes ela consegue obter?
06. Um carpinteiro fabrica caixas de madeira abertas na parte de cima,
pregando duas placas retangulares de 600 cm2 cada uma, duas placas retangulares de 1200 cm2 cada uma e uma placa retangular de 800 cm2,
conforme representado no desenho.
Qual o volume, em litros, da caixa? Note que l litro = 1000 cm3.

PROBLEMAS Nvel 1 PARTE B


(Cada problema vale 10 pontos)

PROBLEMA 1

Quatro peas iguais, em forma de tringulo retngulo, foram dispostas de


dois modos diferentes, como mostram as figuras.
H
J

I
D
A

N
G

C
B

P
F

Os quadrados ABCD e EFGH tm lados respectivamente iguais a 3 cm e


9 cm. Calcule as reas dos quadrados IJKL e MNOP.
EUREKA! Edio Especial, 2007

63

Sociedade Brasileira de Matemtica

PROBLEMA 2

Considere trs nmeros inteiros positivos consecutivos de trs algarismos


tais que o menor mltiplo de 7, o seguinte mltiplo de 9 e o maior
mltiplo de 11. Escreva todas as seqncias de nmeros que satisfazem
essas propriedades.
PROBLEMA 3

Cada pea de um jogo de domin possui duas casas numeradas.


Considere as 6 peas formadas apenas pelos nmeros 1, 2 e 3.
(a) De quantos modos possvel colocar todas estas peas alinhadas em
seqncia, de modo que o nmero da casa da direita de cada pea seja
igual ao nmero da casa da esquerda da pea imediatamente direita?
A seguir, mostramos dois exemplos:

(b) Explique por que no possvel fazer o mesmo com todas as


10 peas formadas apenas pelos nmeros 1, 2, 3 e 4.
PROBLEMAS Nvel 2 PARTE A
(Cada problema vale 4 pontos)

01. Veja o problema No. 3 do Nvel 1 Parte A.


02. Quatro peas iguais, em forma de tringulo retngulo, foram
dispostas de dois modos diferentes, como mostram as figuras abaixo.
H
J

I
D
A

N
G

C
B

P
F

EUREKA! Edio Especial, 2007

64

Sociedade Brasileira de Matemtica

Os quadrados ABCD e EFGH tm lados respectivamente iguais a 3 cm e


9 cm. Determine a medida do lado do quadrado IJKL.
03. Veja o problema No. 4 do Nvel 1 parte A.
04. Um terreno quadrangular foi dividido em quatro lotes menores por
duas cercas retas unindo os pontos mdios dos lados do terreno. As reas
de trs dos lotes esto indicadas em metros quadrados no mapa a seguir.

250

200

210

Qual a rea do quarto lote, representado pela regio escura no mapa?


05. Seja a um nmero inteiro positivo tal que a mltiplo de 5, a + 1
mltiplo de 7, a + 2 mltiplo de 9 e a + 3 mltiplo de 11. Determine o
menor valor que a pode assumir.

PROBLEMAS Nvel 2 PARTE B


(Cada problema vale 10 pontos)

PROBLEMA 1

Gabriel resolveu uma prova de matemtica com questes de lgebra,


geometria e lgica. Aps checar o resultado da prova Gabriel observou
que respondeu corretamente 50% das questes de lgebra, 70% das
questes de geometria e 80% das questes de lgica. Gabriel observou,
tambm, que respondeu corretamente 62% das questes de lgebra e
lgica e 74% das questes de geometria e lgica. Qual a porcentagem de
questes corretas da prova de Gabriel?
PROBLEMA 2

O canto de um quadrado de cartolina foi cortado com uma tesoura. A


soma dos comprimentos dos catetos do tringulo recortado igual ao
EUREKA! Edio Especial, 2007

65

Sociedade Brasileira de Matemtica

comprimento do lado do quadrado. Qual o valor da soma dos ngulos e


marcados na figura abaixo?
27

PROBLEMA 3

(a) Fatore a expresso x 2 9 xy + 8 y 2 .


(b) Determine todos os pares de
9 xy x 2 8 y 2 = 2005 .

inteiros

(x;

y)

tais

que

PROBLEMA 4

Veja o problema No. 3 do Nvel 1 Parte B.


PROBLEMAS Nvel 3 PARTE A
(Cada problema vale 4 pontos)

01. Na figura, ABCDE um pentgono regular e AEF um tringulo


eqiltero. Seja P um ponto sobre o segmento BF , no interior de
ABCDE, e tal que o ngulo PE A mede 12, como mostra a figura abaixo.
F
A
P
B

Calcule a medida, em graus, do ngulo PC.


EUREKA! Edio Especial, 2007

66

Sociedade Brasileira de Matemtica

02. Seja a um nmero inteiro positivo tal que a mltiplo de 5, a + 1


mltiplo de 7, a + 2 mltiplo de 9 e a + 3 mltiplo de 11. Determine o
menor valor que a pode assumir.
03. Veja o problema No. 4 do Nvel 2 parte A.
04. A funo f : satisfaz f ( x + f ( y )) = x + f ( f ( y )) para todos os
nmeros reais x e y. Sabendo que f ( 2) = 8 , calcule f(2005).
05. Voc tem que determinar o polinmio p(x) de coeficientes inteiros
positivos fazendo perguntas da forma Qual o valor numrico de
p(k)?, sendo k um inteiro positivo sua escolha.

Qual o menor nmero de perguntas suficiente para garantir que se


descubra o polinmio?
PROBLEMAS Nvel 3 PARTE B
(Cada problema vale 10 pontos)

PROBLEMA 1

Determine todos os pares de inteiros (x; y) tais que 9 xy x 2 8 y 2 = 2005 .


PROBLEMA 2

Um prisma reto e tem como base um tringulo equiltero. Um plano


corta o prisma mas no corta nenhuma de suas bases, determinando uma
seco triangular de lados a, b e c. Calcule o lado da base do prisma em
funo de a, b e c.
PROBLEMA 3

No campeonato tumboliano de futebol, cada vitria vale trs pontos,


cada empate vale um ponto e cada derrota vale zero ponto. Um resultado
uma vitria, empate ou derrota. Sabe-se que o Flameiras no sofreu
nenhuma derrota e tem 20 pontos, mas no se sabe quantas partidas
esse time jogou. Quantas seqncias ordenadas de resultados o
Flameiras pode ter obtido? Representando vitria por V, empate por E e
derrota por D, duas possibilidades, por exemplo, so (V, E, E, V, E, V, V,
V, E, E) e (E, V, V, V, V, V, E, V).
EUREKA! Edio Especial, 2007

67

Sociedade Brasileira de Matemtica

PROBLEMA 4

Determine o menor valor possvel do maior termo de uma progresso


aritmtica com todos os seus sete termos a1, a2, a3, a4, a5, a6, a7 primos
positivos distintos.
Curiosidade: No ano passado, os ex-olmpicos Terence Tao (Austrlia, ouro

na IMO 1988) e Ben Green (Reino Unido, prata na IMO 1994) provaram
que existem progresses aritmticas arbitrariamente grandes com todos
os termos primos positivos. Tal questo remonta ao sculo XVIII,
aparecendo nas pesquisas de Lagrange e Waring.
Solues Nvel 1 Segunda Fase Parte A
Problema
01
02
03
04

05

06

Resposta

10

24

18

214

182

01. O tanque contm uma mistura de 30 litros, sendo 0, 2 30 = 6 litros


de lcool e 30 6 = 24 litros de gasolina. Portanto, para que as
quantidades de gasolina e lcool fiquem iguais, devem ser colocados
no tanque 24 6 = 18 litros de lcool.

1+ a
=2,
2
1+ 2 + 3
= 2;
b=
3

02. Como 2 a mdia aritmtica de 1 e a, podemos escrever

logo

1+ a = 4 a = 3;

portanto,

1+ 3 + 2 + 2
1+ 3 + 2 + 2 + 2
= 2 . Esses exemplos
= 2; d =
4
5
sugerem que todos os termos, a partir do terceiro, so iguais a 2. De
fato, quando introduzimos em uma seqncia um termo igual mdia
de todos os termos da seqncia, a mdia da nova seqncia
a mesma que a da seqncia anterior. Assim, o ltimo
termo da seqncia dada 2.
c=

03. Natasha pulou os nmeros 13, 31, 113, 130,131, 132, ..., 139, num
total de 13 nmeros. Portanto, na ltima pgina do seu dirio
escreveu o nmero 200 + 13 +1 = 214.
EUREKA! Edio Especial, 2007

68

Sociedade Brasileira de Matemtica

04. Olhando para o ltimo nmero da fila n, vemos que ele a soma de
todos os nmeros de 1 a n: por exemplo, na fila 4, o ltimo nmero
da fila 1 + 2 + 3 + 4 = 10. Note que para obter a quantidade de
nmeros at uma certa fila, basta somar o nmero da fila ao total de
nmeros que havia antes dessa fila. Assim, temos, fila 5 : 15, fila 6:
21, fila 7: 28, fila 8: 36, fila 9: 45, fila 10: 55, fila 11: 66, fila 12: 78,
fila 13: 91, fila 14: 105
O nmero de fitas adesivas horizontais entre uma fila n 1 e uma fila
n igual a n 1 e o nmero de fitas adesivas verticais numa fila n
igual n 1. Portanto, at a fila nmero 14, o nmero de fitas
13 14
= 182.
(1 + 2 + + 13) + (1 + 2 + + 13) = 2
2
05. Todas as faces azuis: uma maneira.
Cinco faces azuis e uma amarela: uma maneira.
Quatro faces azuis e duas amarelas: duas maneiras (duas faces
amarelas opostas ou duas faces amarelas adjacentes).
Trs faces azuis e trs faces amarelas: duas maneiras (trs azuis com
um vrtice comum uma maneira ou trs azuis com uma aresta
comum duas a duas uma maneira)
Duas faces azuis e quatro amarelas: duas maneiras
Uma face azul e cinco amarelas: uma maneira.
Todas as faces amarelas: uma maneira.
Portanto, o nmero de maneiras diferentes de pintar o cubo 10.
06. Sejam a, b e c as medidas da caixa, conforme indicado no desenho ao
lado.
Segundo o enunciado, podemos escrever ab = 600, ac = 1200 e
bc = 800. Sabemos que o volume da caixa abc. Utilizando as
propriedades das igualdades e de potncias, podemos escrever

( ab) ( ac) ( bc) = 600 1200 800 a2 b2 c2 = 2 3102 22 3102 23 102


2
( abc) = 26 32 106 abc = 26 32 106 abc = 23 3103 = 24 1000 cm3
Como 1 litro igual a 1000 cm3, conclumos que o volume da
caixa de 24 litros.
EUREKA! Edio Especial, 2007

69

Sociedade Brasileira de Matemtica

Solues Nvel 1 Segunda Fase Parte B


SOLUO DO PROBLEMA 1:
1 maneira: O quadrado IJKL e o quadrado MNOP tm como lados as
hipotenusas dos tringulos retngulos dados, logo tm a mesma rea s.
Fazendo os dois quadrados coincidirem, conclumos que o dobro da soma
t das reas dos quatro tringulos retngulos a diferena entre as reas
dos quadrados IJKL e EFGH, ou seja, 2t = 92 32 2t = 72 t = 36 .
Assim, s = 9 + 36 = 81 36 = 45 cm2.
2 maneira: No quadrado IJKL, seja JC = x. Ento

IC = ID + DC = JC + DC = x + 3. Ento, no quadrado EFGH, temos


HN + NG = x + 3 + x = 9 2 x = 6 x = 3 . Portanto, a rea do
quadrado IJKL, igual soma das reas dos quatro tringulos retngulos
3 (3 + 3)
+ 32 = 36 + 9 = 45 e a
com a rea do quadrado ABCD, vale 4
2
rea do quadrado MNOP, igual diferena entre a rea do quadrado
EFGH e a soma das reas dos quatro tringulos retngulos, vale
3 (3 + 3)
92 4
= 81 36 = 45 cm2.
2
SOLUO DO PROBLEMA 2:

Seja n = abc mltiplo de 11; ento n 1 deve ser mltiplo de 9 e n 2


deve ser mltiplo de 7.
Seja c 0 :
Como
abc

mltiplo
de
11,
podemos
ter
a b + c = 0 ou a b + c = 11 . Como abc 1 mltiplo de 9,
podemos ter a + b + c 1 = 9 ou a + b + c 1 = 18 . No caso de
a + b + c 1 = 0 , teramos
n 1 = 99 n = 100 , que no
mltiplo de 11. Assim, simultaneamente, somente podemos
a + b + c = 10
2b = 10
b=5

ou
ter (i )
a+c=b
a+c=b
a+c=5

(ii )

a + b + c = 19
2b + 11 = 19
b=4

a + c = b + 11
a + c = b + 11
a + c = 15

EUREKA! Edio Especial, 2007

70

Sociedade Brasileira de Matemtica

No caso (i) existem as seguintes possibilidades para n: 154, 253, 352,


451, que so mltiplos de 11; para n 1 temos os nmeros 153, 252,
351, 450 e 549 so mltiplos de 9. Para os nmeros n 2 temos 152, 251,
350, 449 e 548, dos quais apenas 350 mltiplo de 7.
No caso (ii) existem as seguintes possibilidades para n: 649, 748, 847 e
946, que so mltiplos de 11; para n 1 temos os nmeros 648, 747,
846 e 945 so mltiplos de 9. Para os nmeros n 2 temos 647, 746, 845
e 944, dos quais nenhum mltiplo de 7.
Seja c = 0:
Neste caso, n 1 tem os algarismos a, b 1 e 9. Assim,
a + b 1 + 9 = 9 ou a + b 1 + 9 = 18 ou seja, a + b = 1 ou a + b = 10 .
Como a b + c = a b = 0 ou a b + c = a b = 11 , conclumos que
a = b. Assim, a = b = 5, o que fornece os nmeros n = 550, n 1 = 549 e
n 2 = 548, que no divisvel por 7.
Portanto, a nica seqncia de trs nmeros inteiros consecutivos nas
condies dadas 350, 351 e 352.
SOLUO DO PROBLEMA 3:
1a maneira:

a) Podemos representar uma seqncia vlida como uma seqncia de


pares
ordenados.
O
primeiro
exemplo

a
seqncia
[(1,1),(1,2),(2,2),(2,3),(3,3),(3,1)] e, a partir dela, podemos criar outras
seqncias vlidas movendo o par da esquerda para a direita (ou da
direita para a esquerda). Assim, so vlidas as seqncias
[(1,2),(2,2),(2,3),(3,3),(3,1),(1,1)], [(2,2),(2,3),(3,3),(3,1),(1,1), (1,2)],etc.
num total de 6 seqncias diferentes. Mudando a posio dos nmeros
dos pares ordenados, podemos criar outras 6 seqncias: [(2,1), (1,1),
(1,3), (3,3),(3,2),(2,2)], [ (1,1), (1,3), (3,3),(3,2),(2,2), (2,1)], etc.
Portanto, de acordo com as regras dadas h 12 modos de colocar as peas
em seqncia.

EUREKA! Edio Especial, 2007

71

Sociedade Brasileira de Matemtica

2a maneira:

As pontas devem ter o mesmo nmero, pois eles aparecem um


nmero par de vezes (se aparecer um nmero numa ponta e outro na
outra, ento h pelo menos dois nmeros que aparecem um nmero
mpar de vezes, o que no ocorre). Alguma pea com dois nmeros
iguais deve aparecer em uma das pontas, pois do contrrio teramos trs
das quatro peas centrais com duas iguais, vizinhas, o que impossvel).
Sendo
assim,
a
seqncia
pode
ser
representada
por
XX-XY-YY-YZ-ZZ-ZX, onde para X temos trs possibilidades, para
Y temos duas possibilidade e para Z, uma possibilidade, num total de
3.2.1 = 6 possibilidades para a seqncia que comea com uma dupla. Se
a seqncia terminar com uma dupla, teremos novamente
6 possibilidades. Portanto, h 12 modos de colocar as seis
peas em seqncia.
a) Para cada nmero, existem 4
peas. Por exemplo, as peas com
o nmero 1 esto desenhadas ao
lado. O nmero de vezes em que
aparece o nmero 1 mpar, logo a
seqncia deveria comear com 1
e terminar com outro nmero ou
comear com outro nmero e
terminar com 1. Neste caso, os
outros dois nmeros deveriam
aparecer um nmero par de vezes,
pois no estariam na ponta, mas
isso no ocorre: todos os quatro
nmeros aparecem um nmero
mpar de vezes.

Solues Nvel 2 Segunda Fase Parte A

Problema
Resposta

01
214

EUREKA! Edio Especial, 2007

02
-------

03
182

04
240
72

05
1735

Sociedade Brasileira de Matemtica

01. Natasha pulou os nmeros 13, 31, 113, 130,131, 132, ..., 139, num
total de 13 nmeros. Portanto, na ltima pgina do seu dirio
escreveu o nmero 200 + 13 +1 = 214.
02. Sejam x e y o maior e o menor catetos, respectivamente, do tringulo
retngulo. Como o lado do quadrado ABCD mede 3 cm, temos x y =
3. Por outro lado, como o lado de EFGH mede 9 cm, temos x + y = 9.
Resolvendo o sistema, encontramos x = 6 e y = 3. Logo, o lado do
quadrado IJKL, que a hipotenusa do tringulo retngulo, mede

6 2 + 3 2 = 45 = 3 5 cm.
OUTRA SOLUO: O quadrado IJKL e o quadrado MNOP tm como

lados as hipotenusas dos tringulos retngulos dados, logo tm a


mesma rea s. Fazendo os dois quadrados coincidirem, conclumos
que o dobro da soma t das reas dos quatro tringulos retngulos a
diferena entre as reas dos quadrados IJKL e EFGH, ou seja, 2t = 92
32 , o que fornece t = 36.. Assim, s = 9 + 36 = 81 36 = 45 cm2 e o
lado do quadrado IJKL 45 = 3 5 cm.
03. Olhando para o ltimo nmero da fila n, vemos que ele a soma de
todos os nmeros de 1 a n: por exemplo, na fila 4, o ltimo nmero
da fila 1 + 2 + 3 + 4 = 10. Note que para obter a quantidade de
nmeros at uma certa fila, basta somar o nmero da fila ao total de
nmeros que havia antes dessa fila. Assim, temos, fila 5 : 15, fila 6:
21, fila 7: 28, fila 8: 36, fila 9: 45, fila 10: 55, fila 11: 66, fila 12: 78,
fila 13: 91, fila 14: 105
O nmero de fitas adesivas horizontais entre uma fila n 1 e uma fila
n igual a n 1 e o nmero de fitas adesivas verticais numa fila n
igual n 1. Portanto, at a fila nmero 14, o nmero de fitas
13 14
= 182.
(1 + 2 + + 13) + (1 + 2 + + 13) = 2
2
04. Primeira Soluo: Unindo os pontos mdios de lados consecutivos do
quadriltero, obtemos segmentos paralelos s suas diagonais e iguais
metade delas. Portanto, o quadriltero assim obtido um
EUREKA! Edio Especial, 2007

73

Sociedade Brasileira de Matemtica

paralelogramo. Os segmentos traados dividem cada um dos quatro


lotes em duas partes. Todas as partes internas tm a mesma rea s,
igual a 1/4 da rea do paralelogramo. Cada uma das partes
externas tem rea igual a 1/4 do tringulo determinado pela
diagonal correspondente. Assim, a + c igual metade da
rea do quadriltero, o mesmo ocorrendo com b + c. Da,
a + s + c + s = b + s + d + s. Portanto, a rea S desconhecida satisfaz
S + 210 = 200 + 250, ou seja, S = 240.

b
a

s
d

Segunda Soluo: Ligando o ponto de interseo das retas que representam


as duas cercas aos vrtices, obtemos:
B

Observemos que, como AQ = QD e as alturas de OAQ e OQD que


passam por O so iguais, as reas de OAQ e OQD so iguais.
Analogamente, as reas de OAM e OMB; OBN e ONC; OCP e OPD so
iguais. Logo rea OAQ + rea OAM + rea OCP + rea ONC = rea OQD
+ rea OMB + rea OPD + rea OBN rea AMOQ + rea CNOP =
rea DPOQ + rea BMON rea AMOQ = 200 + 250 210 = 240.
EUREKA! Edio Especial, 2007

74

Sociedade Brasileira de Matemtica

05. Como a + 3 mltiplo de 11, a + 3 = 11b, b Z. Sendo a mltiplo de 5,


tambm , de modo que b 3 =
a 10b = b 3
5c b = 5c + 3 a = 11(5c + 3) 3 = 55c + 30, c +2 O nmero a + 2 mltiplo
de 9, assim como a + 2 54c 36 = c 4. Portanto
c 4 = 9d c = 9d + 4 a = 55(9d + 4) + 30 = 495d + 250, d . Por fim,
sendo a + 1 mltiplo de 7, ento a + 1 497d 245 = a + 1 7
(71d + 35) = 2d + 6 = 2(d 3) tambm , ou seja,
d 3 = 7k d = 7 k + 3, k e a = 495(7 k + 3) + 250 = 3465t + 1735 Logo o
menor valor de a 1735.
Solues Nvel 2 Segunda Fase Parte B
SOLUO DO PROBLEMA 1:

Vamos representar por A, G e L a quantidade de questes de lgebra,


Geometria e Lgica da Prova e por a, g e l as questes respondidas
acertadamente em cada uma destas reas. As condies do problema
fornecem as seguintes equaes:
a
g
l
a+l
g +l
= 0,5;
= 0,7;
= 0,8;
= 0,62;
= 0,74
A
G
L
A+ L
G+L

Substituindo as relaes expressas pelas trs primeiras equaes nas


outras duas, obtemos:
0,5 A + 0,8 L
3L
= 0,62 0,12 A = 0,18 L A =
A+ L
2
0,7G + 0,8 L
3L
= 0,74 0,04G = 0,06 L G =
G+L
2

A porcentagem de questes acertadas :


a + g + l 0,5 A + 0,7G + 0,8L
=
=
A+G + L
A+G + L

EUREKA! Edio Especial, 2007

3
3
0,5. L + 0,7. L + 0,8L
2,6
2
2
=
= 0,65 = 65%
3
3
4
L+ L+L
2
2

75

Sociedade Brasileira de Matemtica

SOLUO DO PROBLEMA 2:

Vamos denotar por A, B, C e D os vrtices do quadrado e por MN o corte


efetuado. Como CM + CN = BC = CD, resulta que BM = CN e DN = MC.
Em conseqncia, os tringulos ADN e DCM so congruentes, o mesmo
ocorrendo com ABM e BCN (em cada caso, os tringulos so retngulos e
possuem catetos iguais). Logo, DN = CDM = e BM = CBN = . Assim, + + 27o = 90o e + = 63o.
A

B
27o

SOLUO DO PROBLEMA 3:

a) x2 9xy + 8y2 = x2 xy 8xy + 8y2 = x(x y) 8y (x y) =


(x 8y)(x y).
Alternativamente, as razes da equao do 2o grau x2 9xy + 8y2, de incgnita x, so y e 8y. Logo, x2 9xy + 8y2 fatora em (x 8y)(x y).
b) A equao a ser resolvida (x y)(8y x) = 2005 (*)
Observemos que a fatorao em primos de 2005 5 401.
Alm disso, a soma dos fatores x y e 8y x 7y, que mltiplo de 7.
A soma dos fatores 406, sendo que somente 406 mltiplo de 7.
Assim,
x y = 5 e 8 y x = 401
ou
x y = 401 e 8 y x = 5
(*)
ou
x y = 5 e 8 y x = 401
ou
x y = 401 e 8 y x = 5

x = 63 e y = 58
ou
x = 459 e y = 58
ou

x = 63 e y = 58
ou
x = 459 e y = 58

As solues so, portanto, (63; 58), (459;58), (63; 58) e (459; 58).

EUREKA! Edio Especial, 2007

76

Sociedade Brasileira de Matemtica

OUTRA SOLUO:

Observando a equao dada como uma equao do segundo grau em x,


obtemos
x2 9yx + 8y2 + 2005 = 0 (*),
cujo discriminante
= (9y)2 4(8y2 + 2005) = 49y2 8020
Para que (*) admita solues inteiras, seu discriminante deve ser um
quadrado perfeito; portanto
49y2 8020 = m2 (7y m)(7y + m) = 8020 = 22 5 401 (**)
Podemos supor, sem perda de generalidade, que m 0, pois se (m; y)
soluo de (**), ento ( m; y) tambm . Observando tambm que
7y m e 7y + m tm a mesma paridade e y m 7y + m, ento podemos
dividir o problema em 4 casos:

7y m = 2 e 7y + m = 4010 m = 2004 e y = 2006/7, impossvel;


7y m = 10 e 7y + m = 802 m = 396 e y = 58;
7y m = 802 e 7y + m = 10 m = 396 e y = 58;
7y m = 4010 e 7y + m = 2 m = 2004 e y = 2006/7,
impossvel.
9 y + m 9 58 + 396
Se y = 58, as solues em x de (*) so
=
= 459 e
2
2
9 y m 9 58 396
=
= 63 .
2
2
9 y + m 9 (58) + 396
=
= 63
Se y = 58, as solues em x de (*) so
2
2
9 y m 9 (58) 396
=
= 459 .
e
2
2

Logo as solues so (63 ; 58), (459 ; 58), ( 63 ; 58) e ( 459 ; 58).


SOLUO DO PROBLEMA 4:

Veja a soluo do problema No. 3 do Nvel 1 parte B

Solues Nvel 3 Segunda Fase Parte A

Problema
Resposta

01
12

EUREKA! Edio Especial, 2007

02
03
1735 240

04
05
2011 2
77

Sociedade Brasileira de Matemtica

01 . Primeiro observamos que os ngulos internos de um pentgono

regular medem

(5 2) 180
= 108 .
5

Como AF = AE = AB, o tringulo ABF issceles com


m( ABF ) = m( AFB) =

.
No

180 m( B AF ) 180 m( B AE ) m( E AF ) 180 108 60


=
=
= 6
2
2
2

tringulo

PEF,

m( E FP ) = m( AFE ) m( AFB ) = 60 6 = 54 e

m( E PF ) = 180 m( PEF ) m( E FP) = 180 60 12 54 = 54 , ou seja, o

tringulo PEF issceles com PE = EF. Assim, como EF = AE,


o tringulo PEA tambm issceles com
180 m( PEA) 180 12
=
= 84.
2
2
180 m( ABC ) 180 108
Alm
disso,
m(C AB) =
=
= 36
2
2
m(C AE ) = m( B AE ) m(C AB ) = 108 36 = 72.
m( P AE ) = m( E PA) =

Logo, m( P AC ) = m( P AE ) m(C AE ) = 84 72 = 12.


02. PRIMEIRA SOLUO:

Como a + 3 mltiplo de 11, a + 3 = 11b, b . Sendo a mltiplo de 5,


a 10b = b 3
tambm , de modo que b 3 =
5c b = 5c + 3 a = 11(5c + 3) 3 = 55c + 30, c
O nmero a + 2 mltiplo de 9, assim como a + 2 54c 36 = c 4.
Portanto c 4 = 9d c = 9d + 4 a = 55(9d + 4) + 30 = 495d + 250, d .
Por fim, sendo a + 1 mltiplo de 7, ento a + 1 497d 245 = a + 1 7
(71d + 35) = 2d + 6 =
2(d 3) tambm , ou seja, d 3 = 7k d = 7 k + 3, k e
a = 495(7 k + 3) + 250 = 3465t + 1735. Logo o menor valor de a 1735.
SEGUNDA SOLUO:

As

condies

do

problema

equivalem

dizer

que

2a 5 = 2(a + 1) 7 = 2(a + 2) 9 = 2(a + 3) 11 mltiplo de 5, 7, 9 e 11,


donde mltiplo de 5 7 9 11 = 3465. Assim, o menor valor de a tal que
2a 5 = 3465 , ou seja, a = 1735.
EUREKA! Edio Especial, 2007

78

Sociedade Brasileira de Matemtica

03. Ligando o ponto de interseo das retas que representam as duas


cercas aos vrtices, obtemos:
B

Observemos que, como AQ = QD e as alturas de OAQ e OQD que passam por O so iguais, as reas de OAQ e OQD so iguais.
Analogamente, as reas de OAM e OMB; OBN e ONC; OCP e OPD so
iguais. Logo rea OAQ + rea OAM + rea OCP + rea ONC = rea OQD
+ rea OMB + rea OPD + rea OBN rea AMOQ + rea CNOP =
rea DPOQ + rea BMON rea AMOQ = 200 + 250 210 = 240.
04. Substituindo y por 2 e x por a f(2) = a 8, obtemos f(a f(2) + f(2))
= a 8 + f ( f (2)) f(a) = a 8 + f(8).
Substituindo a por 2 na ltima equao, obtemos f(2) = 2 8 + f(8) 8
= 2 8 + f(8) f(8) = 14. Assim f(a) = a 8 + 14 = a + 6 e f(2005) =
2005 + 6 = 2011.
05. A idia da soluo perguntar o valor numrico de p(k) para k
suficientemente grande. Suponha que o polinmio seja:
p(x) = an xn + an1 xn 1 + ... + a0, com an, an 1, ..., a0 inteiros positivos.
Se k um inteiro, tal que: k > M = mx {an, an-1, ..., a0}, ento p(k) um
inteiro, cujos dgitos na representao em base k so exatamente os
coeficientes do polinmio p(x). Podemos ento tomar k igual a
uma potncia de 10 suficientemente grande.
Logo para resolver o problema, basta perguntarmos o valor de p(1),
assim obtemos uma cota superior para M, e ento perguntamos o valor de
p(x) para x igual a uma potncia de 10 maior do que p(1). Portanto, o
nmero mnimo de perguntas que devemos fazer, para garantir que
o polinmio p(x) seja determinado sem sombra de dvidas, 2.
EUREKA! Edio Especial, 2007

79

Sociedade Brasileira de Matemtica

Por exemplo: Se p(1) = 29, perguntamos p(100), digamos que


p(100) = 100613. Ento o nosso polinmio p(x) = 10x2 + 6x + 13.
Solues Nvel 3 Segunda Fase Parte B
SOLUO DO PROBLEMA 1:

Temos 9 xy x 2 8 y 2 = 2005 xy x 2 + 8 xy 8 y 2 = 2005


x( y x) + 8 y ( x y ) = 2005 ( x y )(8 y x) = 2005(*)

Observemos que a fatorao em primos de 2005 5 401.


Alm disso, a soma dos fatores x y e 8y x 7y, que mltiplo de 7.
Devemos ento escrever 2005 como produto de dois fatores, cuja soma
um mltiplo de 7. Para isso, os fatores devem ser 5 e 401. A soma
dos fatores 406.
x y = 5 e 8 y x = 401
ou
x y = 401 e 8 y x = 5
(*)
ou
x y = 5 e 8 y x = 401
ou
x y = 401 e 8 y x = 5

x = 63 e y = 58
ou
x = 459 e y = 58

ou
x = 63 e y = 58
ou
x = 459 e y = 58

As solues so, portanto, (63; 58), (459;58), (63; 58) e (459; 58).
OUTRA SOLUO:

Observando a equao dada como uma equao do segundo grau em x,


obtemos
x2 9yx + 8y2 + 2005 = 0 (*),
cujo discriminante
= (9y)2 4(8y2 + 2005) = 49y2 8020
Para que (*) admita solues inteiras, seu discriminante deve ser um
quadrado perfeito; portanto
49y2 8020 = m2 (7y m)(7y + m) = 8020 = 22 5 401 (**)
EUREKA! Edio Especial, 2007

80

Sociedade Brasileira de Matemtica

Podemos supor, sem perda de generalidade, que m 0, pois se (m; y)


soluo de (**), ento
( m; y) tambm . Observando tambm que 7y m e 7y + m tm a
mesma paridade e
7y m 7y + m, podemos dividir o problema em 4 casos:

7y m = 2 e 7y + m = 4010 m = 2004 e y = 2006/7, impossvel;


7y m = 10 e 7y + m = 802 m = 396 e y = 58;
7y m = 802 e 7y + m = 10 m = 396 e y = 58;
7y m = 4010 e 7y + m = 2 m = 2004 e y = 2006/7, impossvel.

Se y = 58, as solues em x de (*) so

9 y + m 9 58 + 396
=
= 459
2
2

9 y m 9 58 396
=
= 63 .
2
2

Se y = 58, as solues em x de (*) so


e

9 y + m 9 (58) + 396
=
= 63
2
2

9 y m 9 (58) 396
=
= 459 .
2
2

Logo as solues so (63 ; 58), (459 ; 58), ( 63 ; 58) e ( 459 ; 58).

SOLUO DO PROBLEMA 2:

c
b2

a2

Podemos supor, sem perda de generalidade, a configurao acima e,


portanto, pelo teorema de Pitgoras:
EUREKA! Edio Especial, 2007

81

Sociedade Brasileira de Matemtica

b2

a2

4( b2a2 b2 2 a2 2 +
4

2 ( a2 + b2 + c2 )

)=
2

)
4

= c2 2

(b

)( a

)=a

+ b2 c2

+ a4 + b4 + c4 2a2 2 2b2 2 + 2c2 2 + 2a2b2 2a2c2 2b2c2

(a 4 + b 4 + c 4 2a 2 b 2 2a 2 c 2 2b 2 c 2 ) = 0

O discriminante da equao do segundo grau acima, em

= 2( a 2 + b 2 + c 2 ) + 4 3 ( a 4 + b 4 + c 4 2a 2b 2 2a 2 c 2 2b 2 c 2 ) =
2

16(a 4 + b 4 + c 4 a 2b 2 a 2 c 2 b 2 c 2 ).

Logo
2

2(a 2 + b 2 + c 2 ) 16( a 4 + b 4 + c 4 a 2b 2 a 2 c 2 b 2 c 2 )

23

(a 2 + b 2 + c 2 ) 2 a 4 + b 4 + c 4 a 2b 2 a 2 c 2 b2 c 2
3

De fato, observando que


2

menor ou igual a min {a, b, c}, temos

a +b +c
. Portanto
3
2

(a 2 + b 2 + c 2 ) 2 a 4 + b 4 + c 4 a 2b 2 a 2 c 2 b 2 c 2
.
3

Observao: Outra maneira de obter as equaes trabalhar em R ,

supondo, sem perda de generalidade, que C = (0, 0, 0), A = ( ,0, h) e

3
B = ,
, z , com h, z 0 . Obteramos, ento, as equaes
2 2

+ h2 = a 2 ,

+ z 2 = b2 e

+ ( z h) 2 = c 2 , que nos leva mesma equao

da soluo acima.
EUREKA! Edio Especial, 2007

82

Sociedade Brasileira de Matemtica

Curiosidade: Para o tringulo 3, 4, 5 a medida do lado da projeo que um

tringulo equiltero aproximadamente e. O erro de apenas 0,1%.


SOLUO DO PROBLEMA 3:
Primeira Soluo:

Seja an o nmero de ordenadas de resultados (sem derrotas), cujo total de


pontos seja n. A pergunta do problema : quanto vale a20?
Para responder a tal pergunta, iremos determinar uma relao recursiva
entre os termos dessa seqncia. Pensando no ltimo resultado de uma
ordenada de resultados totalizando n pontos, ele pode ser E ou V. Se for
E, ento retirando o ltimo termo da ordenada, ela passa a totalizar n 1
pontos. Se for V, ento ao retiramos o ltimo resultado, a ordenada passa
a totalizar n 3 pontos. Disto, conclumos que:
an = an 1 + an 3.
Calculando os valores da seqncia, temos: a1 = 1, a2 = 1, a3 = 2, a4 = 3,
a5 = 4, a6 = 6, a7 = 9,
a8 = 13, a9 = 19, a10 = 28, a11 = 41, a12 = 60, a13 = 88, a14 = 129, a15 =
189, a16 = 277, a17 = 406, a18 = 595, a19 = 872 e a20 = 1278.
Logo existem 1278 possveis seqncias ordenadas de resultados que o
Flameiras pode ter obtido.
Segunda Soluo:

Sejam x e y o nmero de vitrias e empates do Flameiras,


respectivamente. Temos que: x 0,
y 0 e 3x + y = 20. Dividindo em 7 possveis casos:
1 caso: x = 0 e y = 20: Temos exatamente uma seqncia ordenada de
resultados.
2 caso: x = 1 e y = 17: Uma seqncia ordenada dever conter
exatamente um V e 17 E, portanto o nmero de seqncias
ordenadas exatamente o nmero de anagramas da palavra:
VEEEEEEEEEEEEEEEEE, que : (17 + 1)! / (17! 1!) = 18.
3 caso: x = 2 e y = 14: Analogamente ao 2 caso, o nmero de
seqncias ordenadas igual ao nmero de anagramas da palavra
VVEEEEEEEEEEEEEE, que : (14 + 2)! / (14! 2!) = 120.
4 caso: x = 3 e y = 11: (11 + 3)! / (11! 3!) = 364 seqncias ordenadas.
5 caso: x = 4 e y = 8: (8 + 4)! / (8! 4!) = 495 seqncias ordenadas.
EUREKA! Edio Especial, 2007

83

Sociedade Brasileira de Matemtica

6 caso: x = 5 e y = 5: (5 + 5)! / (5! 5!) = 252 seqncias ordenadas.


7 caso: x = 6 e y = 2: (2 + 6)! / (2! 6!) = 28 seqncias ordenadas.
Temos um total de 1 + 18 + 120 + 364 + 495 + 252 + 28 = 1278
seqncias ordenadas de resultados possveis.
SOLUO DO PROBLEMA 4:

Seja p, p + d, p + 2d, p + 3d, p + 4d, p + 5d, p + 6d a progresso


aritmtica, que podemos supor crescente sem perda de generalidade.
Ento:
1) p 2.
De fato, se p = 2, p + 2d par e maior do que 2 e, portanto, no primo.
2) d mltiplo de 2.
Caso contrrio, como p mpar, p + d seria par e maior do que 2.
3) p 3
Seno, teramos p + 3d mltiplo de 3, maior do que 3.
4) d mltiplo de 3
Caso contrrio, p + d ou p + 2d seria mltiplo de 3 e maior do que 3.
5) p 5
Seno teramos p + 5d mltiplo de 5, maior do que 5.
6) d mltiplo de 5.
Caso contrrio, p + d , p + 2d, p + 3d ou p + 4d seria mltiplo de 5, maior
do que 5.
De 1), 2), 3), 4), 5) e 6), p 7 e d mltiplo de 30.
Se p = 7, observando que 187 = 11 17, ento d 120.
Para d = 120, a seqncia 7, 127, 247, 367, 487, 607, 727 a qual no
serve, pois 247 = 13 19.
Para d = 150, a seqncia 7, 157, 307, 457, 607, 757, 907 e satisfaz as
condies do problema.
Finalmente, se p 7, ento d mltiplo de 210 e o menor ltimo termo
possvel para tais seqncias 11 + 6 210 = 1271.
Portanto a resposta 907.

EUREKA! Edio Especial, 2007

84

Sociedade Brasileira de Matemtica

OLIMPADA DE MAYO
PROBLEMAS

Primeiro Nvel
Problema 1

Um calendrio digital exibe a data: dia, ms e ano, com 2 dgitos para o


dia, 2 dgitos para o ms e 2 dgitos para o ano. Por exemplo, 01-01-01
o primeiro dia de janeiro de 2001 e 25-05-23 o dia 25 de maio de 2023.
Em frente ao calendrio h um espelho. Os dgitos do calendrio so os
mesmos da figura.

Se 0, 1, 2, 5 e 8 refletem-se, respectivamente, em 0, 1, 5, 2 e 8, e os
demais dgitos perdem o sentido ao se refletirem, determinar quantos dias
do sculo, ao se refletirem no espelho, tambm correspondem
a uma data.
Problema 2

Um retngulo de papel de 3 cm por 9 cm dobrado ao longo de uma reta,


fazendo coincidir dois vrtices opostos. Desse modo, forma-se um
pentgono. Calcular a sua rea.
Problema 3

H 20 pontos alinhados, separados por uma mesma distncia:

Miguel tem que pintar de vermelho trs ou mais desses pontos, de


maneira tal que os pontos vermelhos estejam separados pela mesma
distncia e seja impossvel pintar de vermelho exatamente um ponto a
mais sem violar a condio anterior. Determinar de quantas maneiras
Miguel pode fazer sua tarefa.
EUREKA! Edio Especial, 2007

85

Sociedade Brasileira de Matemtica

Problema 4

Com 150 cubinhos brancos de 11 1 monta-se um prisma de


6 5 5 , pintam-se suas seis faces de azul e, a seguir,
desmonta-se o prisma. Lucrecia deve montar um novo prisma,
sem buracos, usando exclusivamente cubinhos que tenham
pelo menos uma face azul e de modo que as faces do prisma
de Lucrecia sejam todas completamente azuis. Dar as dimenses
do prisma de maior volume que Lucrecia pode montar.
Problema 5

Em algumas casas de um tabuleiro 10 10 coloca-se uma


ficha de maneira que valha a seguinte propriedade: para cada casa que
tenha uma ficha, a quantidade de fichas colocadas na
sua mesma linha deve ser maior ou igual quantidade
de
fichas
colocadas
na
sua
mesma
coluna.
Quantas
fichas pode haver no tabuleiro?
Dar todas as possibilidades.

Segundo Nvel
Problema 1

Determinar todos os pares de nmeros naturais a e b tais que


b +1
so nmeros naturais.
a

a +1
e
b

Problema 2

No quadro negro esto escritos vrios nmeros primos


(alguns repetidos). Mauro somou os nmeros do quadro negro
e Fernando multiplicou os nmeros do quadro negro.
EUREKA! Edio Especial, 2007

86

Sociedade Brasileira de Matemtica

O resultado que Fernando obteve igual a 40 vezes o resultado


que Mauro obteve. Determinar quais podem ser os nmeros
do quadro negro.
Dar todas as possibilidades.
Problema 3

Escrever um nmero inteiro positivo em cada


casa, de modo que:

os seis nmeros sejam distintos;


a soma dos seis nmeros seja 100;
se multiplicarmos cada nmero por seu vizinho
(no sentido dos ponteiros do relgio) e somarmos os seis resultados das seis multiplicaes,
obtemos
o
menor
valor
possvel.

Explicar porque no se pode obter um valor menor.


Problema 4

Seja ABCD um trapzio de bases AB e CD. Seja O ponto de interseo


de suas diagonais AC e BD. Se a rea do tringulo ABC for 150 e a rea
do tringulo ACD for 120, calcular a rea do tringulo BCO.
Problema 5

Com 28 pontos forma-se uma grade

triangular de lados iguais, como mostra a

gura. Uma operao consiste em escolher

trs pontos que sejam os vrtices de um

tringulo eqiltero e retirar esses trs


pontos da grade. Se aps realizar vrias

dessas operaes resta somente um ponto, e

que posies este ponto pode ficar?

Dar todas as possibilidades e indicar,


em cada caso, as operaes realizadas.

EUREKA! Edio Especial, 2007

87

Sociedade Brasileira de Matemtica

OLIMPADA DE MAYO
SOLUES

Primeiro Nvel
Soluo do Problema 1

Qualquer combinao de dgitos representa um ano, portanto se um dia


tem sentido, ao refletir-se corresponde a um ano. Os dias que se refletem
em anos so 14:
01, 02, 05, 08, 10, 11, 12, 15, 18, 20, 21, 22, 25, 28.
Por outro lado, tambm so 14 os anos que se refletem em dias
(os simtricos da lista anterior):
10, 50, 20, 80, 01, 11, 51, 21, 81, 05, 15, 55, 25, 85.
Os meses que se refletem em meses so 3: 01, 10 e 11.
No total h 14 3 14 = 588 datas que se refletem em datas.
Soluo do Problema 2

Seja ABCDE o pentgono obtido ao dobrar o papel. A dobra CD


perpendicular diagonal AP do retngulo e o ponto de interseo de
ambas o centro do retngulo.
Se BC = a e AC = b , ento
DQ = BC = a e AD = PC =
AC =b, logo a + b = BP = 9
e, como o tringulo ABC
retngulo em B, temos
32 + a 2 = b 2 , ento
2
9 + a2 = b2 , 9 + a 2 = ( 9 a ) ;
9 = 81 18a e temos
a = 4 e b = 5.

3 4
=6.
2
Como BC paralelo a AD, a altura do tringulo ACD traada de C igual
a AB, logo
AD AB 5 3 15
rea ( ACD ) =
=
= .
2
2
2
Logo, rea ( ABC ) = rea ( ADE ) =

EUREKA! Edio Especial, 2007

88

Sociedade Brasileira de Matemtica

Finalmente,
rea ( ABCDE ) = rea ( ABC ) + rea ( ACD ) + rea ( ADE ) = 2 6 +

15 39
.
=
2
2

Soluo do Problema 3
Soluo I

Numeramos os pontos da esquerda para a direita, de 1 a 20. Se d for a


distncia entre os pontos vermelhos consecutivos, o primeiro ponto
vermelho menor ou igual a d (para que no se possa agregar um ponto
vermelho antes). Por outro lado, se o primeiro ponto vermelho for x,
tambm sero vermelhos x + d e x + 2d, de modo que 1 + 2d 20, e
teremos 1 d 9. Alm disso, para as distncias d menores ou iguais a 6
h d maneiras de escolher os pontos, o primeiro ponto tem que ser
menor ou igual a 20 2d, para que possamos marcar ao menos trs pontos; e h 20 2d escolhas para cada d entre 7 e 9. No total so:
1 + 2 + 3 + 4 + 5 + 6 + (20 14) + (20 16) + (20 18) = 33 possibilidades.
Soluo II

Com separao 1 h s 1 distribuio: pintar os 20 pontos de vermelho.


Com separao 2 h 2 distribuies: pintar os mpares (1, 3, 5, ..., 19) ou
pintar os pares (2, 4, 6, ..., 20).
Com separao 3 h 3 distribuies: 1, 4, 7, 10, 13, 16, 19; 2, 5, 8, 11,
14, 17, 20 e 3, 6, 9, 12, 15, 18.
Com separao 4 h 4 distribuies: comeando com 1, com 2, com 3 ou
com 4.
Com separao 5 h 5 distribuies: comeando com 1, com 2, com 3,
com 4 ou com 5.
Com separao 6 h 6 distribuies: comeando com 1, com 2, com 3,
com 4, com 5 ou com 6.
Com separao 7 h 6 distribuies: 1, 8, 15; 2, 9, 16; 3, 10, 17; 4, 11,
18; 5, 12, 19 e 6, 13, 20.
Com separao 8 h 4 distribuies: 1, 9, 17; 2, 10, 18; 3, 11, 19 e 4,
12, 20.
Com separao 9 h 2 distribuies: 1, 10, 19 e 2, 11, 20.
No total so 1 + 2 + 3 + 4 + 5 + 6 + 6 + 4 + 2 = 33 escolhas possveis
para os pontos vermelhos.
EUREKA! Edio Especial, 2007

89

Sociedade Brasileira de Matemtica

Soluo do Problema 4

O prisma de 6 5 5 pintado de azul contm 4 3 3 = 36 cubinhos


completamente brancos, que so os que formam um prisma de 4 3 3
no centro do prisma inicial. Os restantes 150 36 = 114 cubinhos tm
pelo menos uma face azul. Os 8 cubinhos dos vrtices tm exatamente 3
faces azuis (concorrentes); os cubinhos das arestas, exceto os 8 que
contm os vrtices, tm exatamente duas faces azuis (adjacentes). Desses
cubinhos, h 4 em cada aresta de comprimento 6 e 3 em cada aresta de
comprimento 5, de modo que o prisma inicial tem 4 4 + 8 3 = 40
cubinhos com 2 faces azuis.
Os restantes 114 8 40 = 66 cubinhos tm exatamente 1 face azul.
No possvel que uma dimenso seja 1, porque para isso seriam
necessrios cubinhos com 2 faces opostas azuis, o que no ocorre com
nenhum dos 114 disponveis.
Vejamos que Lucrecia no pode usar em seu prisma os 114 cubinhos
disponveis. Com efeito, como 114 = 2 3 19 , qualquer prisma que se
forme com todos os 114 cubinhos ter uma de suas dimenses igual a 19
ou um mltiplo de 19. Como conseqncia, haver 4 arestas de
comprimento maior ou igual a 19. Essas 4 arestas requerem, cada uma,
pelo menos 17 cubinhos com dois lados adjacentes azuis, o que faz um
total de 17 4 = 68 desses cubinhos, nmero que excede os 40
disponveis. Portanto, no possvel usar 114 cubinhos.
Se descartarmos somente um cubinho, restam 113, e como 113 um
nmero primo, o nico prisma que se forma com 113 cubinhos de
1 1 113 , que no possvel. Logo, Lucrecia deve descartar pelo menos
2 cubinhos.
Vemos que com 112 = 24 7 cubinhos possvel montar um prisma azul
de 4 4 7 . Nos vrtices temos os 8 cubinhos de 3 faces azuis
concorrentes. Nas 8 arestas de comprimento 4, colocamos 16 cubinhos
de 2 faces adjacentes azuis e, nas 4 arestas de comprimento 7, colocamos
20 desses cubinhos. Usam-se no total 36 cubinhos de duas faces azuis
(temos 40). Para completar o prisma s precisamos cuidar que todos os
cubinhos das faces tenham uma face azul vista.
(Esse o nico prisma azul que se pode montar com 112 cubinhos).

EUREKA! Edio Especial, 2007

90

Sociedade Brasileira de Matemtica

Soluo do Problema 5

Seja N a quantidade de fichas do tabuleiro. Demonstraremos que N pode


ser qualquer valor de 0 at 10 9 = 90 , alm de 102 .
Em primeiro lugar, veremos que N pode ser qualquer valor inteiro nos
intervalos da forma [10k, 10 (k + 1)], 0 k 8.
Se N = 10k, colocamos fichas em todas as casas de k linhas.
Se N = 10k + p, 1 p 9 k, tambm podemos escrever N como 9k + (k
+ p). Logo, podemos distribuir as fichas em um retngulo k 9 completo
e (k + p) fichas em outra linha, da seguinte maneira:
... ...
... ...
... ...
... ...
...
...
...

...

Se N = 10k + p , ( 9 k ) < p 9 , tambm podemos escrever como


N = 10 ( p + k 9 ) + 9 (10 p ) .

Logo, podemos distribuir as fichas em dois retngulos, um


( p + k 9 ) 10 e outro (10 p ) 9 .
... ...
...
...
...
...
...

...
...
...
... ...
...

...
...
Em todos os casos, cada ficha tem na sua linha igual ou maior quantidade
de fichas do que na sua coluna.
EUREKA! Edio Especial, 2007

91

Sociedade Brasileira de Matemtica

A unio de todos os intervalos descritos anteriormente permite obter


todos os valores de N de 1 at 10 9 = 90 . Ademais, trivialmente, vale a
propriedade quando o tabuleiro est totalmente vazio ou totalmente
preenchido.
Por outro lado, se 10 9 < N < 102 , h menos de 10 casas vazias. Logo,
existe pelo menos uma coluna com fichas em todas as suas casas. Ento
uma ficha, situada na interseo dessa coluna e alguma linha com alguma
casa vazia, no tem a propriedade requerida.
Finalmente, N s pode tomar os valores: 0, 1, 2,..., 88, 89, 90, 100.
Segundo Nvel

Soluo do Problema 1

a +1 b +1
e
so nmeros naturais, devemos ter b a + 1 e
b
a
a b + 1 . Ento a b + 1 a + 2 .
Analisaremos as trs possibilidades
b +1 = a , b +1 = a +1, b +1 = a + 2 .
Se b + 1 = a ,
a +1 b + 2
2
=
= 1+ .
b
b
b
Esse nmero inteiro se, e somente se, b = 1 ou b = 2. Os valores
correspondentes de a so, respectivamente, a = 2 ou a = 3.
Se b + 1 = a + 1 ,
a +1 a +1
1
=
= 1+ .
b
a
a
Esse nmero inteiro se, e somente se, a = 1, e ento b = 1.
Se b + 1 = a + 2 ,
b +1 a + 2
2
=
= 1+ .
a
a
a
Esse nmero inteiro se, somente se, a = 1 ou a = 2, e os valores
correspondentes de b so, respectivamente, b = 2 e b = 3.
Os nicos pares possveis so:
1 e 1, 1 e 2, 2 e 1, 2 e 3, 3 e 2.
Como

EUREKA! Edio Especial, 2007

92

Sociedade Brasileira de Matemtica

Soluo do Problema 2
Soluo I

Como o produto dos primos mltiplo de 40, entre os primos h pelo


menos trs 2 e um 5 (pois 40 = 23 5 ). Alm disso, esses no so os
nicos primos que h, pois 40 ( 2 + 2 + 2 + 5 ) > 2 2 2 5 . Seja q o maior
dos primos restantes; denotemos, por S a soma e P o produto dos primos
restantes, sem contar q nem os trs 2 nem o 5. Ento,
40 ( 2 + 2 + 2 + 5 + q + S ) = 2 2 2 5 q P , ou seja,

11 + q + S = q P
(1)
Como o produto de um ou mais nmeros maiores do que 1 maior ou
igual sua soma, temos que P S , conseqentemente,
11 + q + S q S .
Essa igualdade equivale a
12 q S q S + 1 = ( q 1)( S 1) .
Portanto, q 1 12, e os valores possveis de q so 13, 11, 7, 5, 3, 2.
Se q = 13, ento a nica possibilidade para S S = 2. Nesse caso, os
novos primos so 2 e 13, logo q = 13 e P = 2 e, efetivamente,
11 + 13 + 2 = 13 2 .
Se q = 11, novamente, a nica possibilidade para S S = 2. Nesse caso,
os novos primos so 2 e 11, logo q = 11 e P = 2, porm
11 + 11 + 2 11 2 .
Se q = 7, as possibilidades para S so S = 2 e S = 3. No primeiro caso, os
novos primos so 2 e 7, q = 7 e P = 2, logo 11 + 7 + 2 7 2 e no
soluo. No segundo caso, os novos primos so 3 e 7, q = 7 e P = 3 e,
efetivamente, 11 + 7 + 3 = 7 3 .
Se q = 5, as possibilidades para S so S = 2, S = 3, S = 4. No primeiro
caso, os novos primos so 2 e 5, q = 5 e P = 2; temos 11 + 5 + 2 5 2 e
no soluo. No segundo caso, os novos primos so 3 e 5, logo q = 5 e
P = 3. Como 11 + 5 + 3 5 3 , no soluo. No terceiro caso, os novos
primos so 2, 2 e 5, q = 5 e P = 4, e verificamos que 11 + 5 + 4 = 5 4 .
EUREKA! Edio Especial, 2007

93

Sociedade Brasileira de Matemtica

Se q = 3, as possibilidades para S so 2, 3, 4, 5, 6 e 7, e os novos primos


s podem ser 2 e 3, pois so menores ou iguais a 3. A equao (1)
11 + 3 + S = 14 + S = 3 P . Logo, 14 + S deve ser mltiplo de 3. Dos
possveis valores de 14 + S , os nicos mltiplos de 3 so 14 + 4 = 18 e
14 + 7 = 21. Se S = 4, ento os novos primos so 2, 2 e 3, q = 3 e P = 4.
Como 14 + 4 3 4 , no soluo; se S = 7, os novos primos so 2, 2,
3 e 3, q = 3 e P = 12. Como 14 + 7 3 12 , to pouco soluo. Portanto, descarta-se a possibilidade q = 3.
Se q = 2, os novos primos s podem ser iguais a 2. Ento, S par,
11 + q + S = 11 + 2 + S mpar, enquanto que q P = 2 P par. Portanto,
nesse caso no h soluo.
No total h trs possibilidades para os nmeros do quadro negro:
2, 2, 2, 2, 5, 13; 2, 2, 2, 3, 5, 7 e 2, 2, 2, 2, 2, 5, 5.
Soluo II

Como o produto dos primos mltiplo de 40, entre os primos h pelo


menos trs 2 e um 5 (pois 40 = 23 5 ). Alm disso, esses no so os
nicos primos que h, pois 40 ( 2 + 2 + 2 + 5 ) > 2 2 2 5 .
Se alm de 2, 2, 2 e 5 h um primo a mais, digamos p,
ento 40(2 + 2 + 2 + 5 + p) = 2 2 2 5 p , ou seja, 11 + p = p , o que
impossvel.
Se alm de 2, 2, 2 e 5 h dois primos a mais, digamos p e q, p q,
ento 40 ( 2 + 2 + 2 + 5 + p + q ) = 2 2 2 5 p q , ou seja, 11 + p + q = p q .
Essa equao equivalente a
12 = p q p q + 1 = ( p 1)( q 1) .
Como p e q so primos e p q, as nicas possibilidades so
p = 2 e q = 13; p = 3 e q = 7.
Se alm de 2, 2, 2 e 5 h trs primos a mais, p q r, ento
40 ( 2 + 2 + 2 + 5 + p + q + r ) = 2 2 2 5 p q r , ou seja,

11 + p + q + r = p q r .
Se p = 2, temos que p q r = 2 q r par, ento 11 + p + q + r = 11 +
2 + q + r par e, conseqentemente, q = 2 ( par) e r 3 ( mpar).
Logo, a equao inicial 4r = 15 + r , donde r = 5. Obtemos p = 2, q = 2
e r = 5.
EUREKA! Edio Especial, 2007

94

Sociedade Brasileira de Matemtica

Se p 3, ento p mpar. Logo, p, q e r so mpares, de modo que o


produto p q r mpar. Como 11 + p + q + r par (soma de 4 mpares),
no h solues.
Se alm de 2, 2, 2 e 5 h quatro primos a mais, p q r s, ento
40 ( 2 + 2 + 2 + 5 + p + q + r + s ) = 2 2 2 5 p q r s , ou seja,
11 + p + q + r + s = p q r s .
Se p = 2, temos que 2 q r s = 11 + 2 + q + r + s = 13 + q + r + s . Como
esse nmero par, q, r e s so mpares, de modo que q 3 e,
conseqentemente,
2 q r s = q r s + q r s 27 + q r s .
Como o produto de um ou mais nmeros maiores do que 1 maior ou
igual sua soma, temos que q r s q + r + s e, conseqentemente,
2 q r s > 13 + q + r + s , de modo que no h solues.
Se p 3 , ento,
pqrs 3qrs = qrs + 2qrs qrs + q + r + s + q + r + s > 11 + p + q + r + s ,
pois qrs 27 e q + r + s + q + r + s > p + q + r + s.
Por ltimo, impossvel que alm de 2, 2, 2 e 5 haja cinco ou mais
primos.
Com efeito, nesse caso a equao
40 ( 2 + 2 + 2 + 5 + p1 + + pn ) = 2 2 2 5 p1 pn , com p1 pn
primos, n 5. Simplificando, temos 11 + p1 + + pn = p1 pn .
Dividimos ambos os membros pelo maior dos primos, pn , ento
p
11 p1
+
+ + n = p1 pn 1
pn pn
pn
O lado esquerdo no mximo 5+n, enquanto que o lado direito maior
ou igual a 2n 1 .
De modo que, para que existam esses n primos devemos ter 2n 1 5 + n ,
que impossvel se n 5.

EUREKA! Edio Especial, 2007

95

Sociedade Brasileira de Matemtica

Soluo do Problema 3

A resposta a dada pela figura ao lado ou suas


simetrias e rotaes. O resultado
85 + 2 85 + 4 2 + 3 4 + 5 3 + 5 = 295
Vamos justificar que este o menor valor possvel. Sejam a > b > c > d > e > f, os seis nmeros em ordem decrescente.

1
5

85

2
4

Se a soma mnima, os vizinhos de a so e e f (os vizinhos do maior


so os menores).
Com efeito, se f no vizinho de a, percorremos os nmeros, de um ao
seu vizinho, at encontrar f, e logo fazemos uma espcie de reflexo:
substitumos a seqncia x, , f por sua oposta, f, ,x. Aqui x o
vizinho de a e os pontos anteriores so os nmeros percorridos at
encontrar f. Se refazemos a conta de multiplicar cada nmero por seu
vizinho e somamos os seis produtos, os nicos produtos que mudam so
ax e fy (y o vizinho de f), que se substituem por af e xy. Como ax + fy >
af + xy, pois equivalente a a ( x f ) > y ( x f ) , que verdadeira, em
virtude da ordem dos nmeros a, , f, ento a soma diminui.
Analogamente, o outro vizinho de a deve ser e.
a
Repetindo esse argumento, chega-se que uma
e
vez escolhidos os seis nmeros, a ordem relativa
que produz a menor soma a da figura ao lado.
Agora vejamos quais so os nmeros. Como so c
distintos e somam 100, o maior valor possvel de
d
a 85 = 100 (1 + 2 + 3 + 4 + 5).

Verifiquemos que 85, 5, 4, 3, 2, 1 so os nmeros com os quais se obtm


o mnimo. De fato, se a = 85 t , ento b = 5 + i , c = 4 + j , d = 3 + h ,
e = 2 + k , f = 1 + com i , j , h , k , inteiros no-negativos, j que 1
f, 2 e , 3 d, 4 c, 5 b, pois so nmeros distintos, e
i+ j+h+k + =t .
A nova soma :
(85 t )(1+ ) + (85t )( 2 + k ) + ( 5+ i)(1+ ) + ( 5+ i)( 3+ h) + ( 4 + j )( 2+ k ) + ( 3+ h)( 4+ j) =

85+ 285+5+5 3+ 4 2 + 3 4 +85 t t +85k 2t tk + 5 + i + i + 5h + 3i + ih + 4k + 2 j +


EUREKA! Edio Especial, 2007

96

Sociedade Brasileira de Matemtica

jk + 3 j + 4h + jh = 295 + 87 + i + 86k + 2 j + 6h + i + ih + jk + jh t tk
(usamos que 3t = 3 ( i + j + h + k + ) ).
Como 87 t e 86k tk, a expresso maior do que 295.
Soluo do Problema 4
Soluo I

Os tringulos ACD e BCD tm rea igual, pois os vrtices A e B


pertencem a uma reta paralela ao lado comum CD. Conseqentemente,
os tringulos ADO e BCO tm rea igual e a rea do tringulo OCD
comum.
Chamemos S1 a rea do tringulo AOB, S2 a rea dos tringulos BOC e
ADO, e S3 a rea do tringulo OCD.
Se h1 for a altura dos tringulos ADO e COD, traada de D, temos:
1
AO h1
S2 2
AO
=
=
.
S3 1 CO h CO
1
2
Analogamente,
1
AO h2
S1 2
AO
=
=
.
1
S2
CO h2 CO
2
Portanto, S 22 = S1S3 .

(1)

Como S1 + S2 = 150 e S 2 + S3 = 120 , temos S1 = 150 S2 e S3 = 120 S2 .


Substituindo esses valores em (1):
S 22 = (150 S 2 )(120 S 2 ) = 150 120 150 S 2 120 S 2 + S 22 ,
Obtemos S 2 =

150 120 200


.
=
270
3

Soluo II

Os tringulos ACD e BCD tm rea igual, pois os vrtices A e B


pertencem a uma reta paralela ao lado comum CD. Ento, rea(BCD) =
120.
EUREKA! Edio Especial, 2007

97

Sociedade Brasileira de Matemtica

Sejam h1 e h2 as alturas dos tringulos ACD e ABC correspondentes


base AC. Ento,
AC h1
AC h2
rea(ACD) =
e rea(ABC) =
.
2
2
h 150 5
= .
Conseqentemente, 2 =
h1 120 4

Os tringulos BOC e COD tm a mesma base OC e as alturas


correspondentes a essa base so, respectivamente, h2 e h1 , logo
OC h2
OC h1
.
e rea(COD) =
2
2
h 5
rea BOC
Ento, rea ((COD)) = 2 = . Como a unio dos tringulos BOC e COD o
h1 4
tringulo BCD, da relao anterior, obtemos que:
5
5
200
rea(BOC) = rea(BCD)= 120 =
.
9
9
3

rea(BOC) =

Soluo do Problema 5

0
1
2
0
1
2
0

0
1

2
0

2
1
2
0

1
2

0
1

2
0

2
0

1
2

Etiquetamos cada ponto com


um 0, um 1 ou um 2, como
na figura.
Afirmamos o seguinte: se
tomamos dois pontos da grade com o mesmo nmero,
o ponto que forma com eles
um tringulo eqiltero deve
ter o mesmo nmero.

Isso se pode verificar, por exemplo, esgotando todos os casos possveis (a


simetria da figura reduz o nmero de casos).
Da afirmao anterior, deduz-se que os ternos de nmeros com trs pontos da grade que so vrtices de um tringulo eqiltero devem ser um
dos seguintes:
0 0 0 (soma 0); 1 1 1 (soma 3);
2 2 2 (soma 6); 0 1 2 (soma 3).
EUREKA! Edio Especial, 2007

98

Sociedade Brasileira de Matemtica

Logo, vemos que a soma das etiquetas desses vrtices sempre um mltiplo de 3.
Como a soma no incio 10 0 + 9 1 + 9 2 = 27 e, cada vez que se retiram pontos que formam um tringulo eqiltero, se retiram pontos cuja
soma mltipla de 3, se resta no final um ponto, este deve ter um nmero
mltiplo de 3; quer dizer, somente pode restar o 0. Da, as possveis posies para o ltimo ponto so os lugares etiquetados por 0.
Agora s nos falta dar um exemplo para cada uma das posies do 0. Pela simetria da figura, somente precisamos dar exemplos para 4 posies.
Colocaremos letras iguais para os pontos de um mesmo tringulo eqiltero retirado e o smbolo * para o ltimo ponto.
1) Quando o ltimo ponto est
no centro

2) Quando o ltimo ponto est no vrtice do


tringulo eqiltero maior

B
B
E
E
D
A
A

F
E

D
D

*
B
F
F

*
H

A
B
G
G

I
H

B
G

I
I

D
C

E
C

3) Quando o ltimo ponto est no


ponto mdio de um lado do tringulo
retngulo maior

A
B

F
E

E
G

G
D

A
B
A

K
E

H
I

H
I

C
F

4) Quando o ltimo ponto o ponto mdio


do segmento formado pelo centro e um vrtice

C
C

E
F

H
H

H
I

EUREKA! Edio Especial, 2007

F
C

D
C

F
D

E
C

99

G
F

D
E

K
K
G
G

D
D

H
B
B
C

B
*

A
A

Sociedade Brasileira de Matemtica

COORDENADORES REGIONAIS DA OBM


Alberto Hassen Raad
Amrico Lpez Glvez
Amarsio da Silva Arajo
Andreia Goldani
Antonio Carlos Nogueira
Ali Tahzibi
Benedito Tadeu Vasconcelos Freire
Carlos Alexandre Ribeiro Martins
Carmen Vieira Mathias
Claus Haetinger
Cleonor Crescncio das Neves
Cludio de Lima Vidal
Edson Roberto Abe
lio Mega
Eudes Antonio da Costa
Fbio Brochero Martnez
Florncio Ferreira Guimares Filho
Genildo Alves Marinho
Ivanilde Fernandes Saad
Jacqueline Rojas Arancibia
Janice T. Reichert
Joo Bencio de Melo Neto
Joo Francisco Melo Libonati
Jos Cloves Saraiva
Jos Luiz Rosas Pinho
Jos Vieira Alves
Jos William Costa
Krerley Oliveira
Licio Hernandes Bezerra
Luzinalva Miranda de Amorim
Mrio Rocha Retamoso
Marcelo Rufino de Oliveira
Marcelo Mendes
Newman Simes
Nivaldo Costa Muniz
Ral Cintra de Negreiros Ribeiro
Ronaldo Alves Garcia
Rogrio da Silva Igncio
Reginaldo de Lima Pereira
Reinaldo Gen Ichiro Arakaki
Ricardo Amorim
Srgio Cludio Ramos
Seme Gebara Neto
Tadeu Ferreira Gomes
Toms Menndez Rodrigues
Valdenberg Arajo da Silva
Valdeni Soliani Franco
Vnia Cristina Silva Rodrigues
Wagner Pereira Lopes
William Beline

EUREKA! Edio Especial, 2007

(UFJF)
(USP)
(UFV)
FACOS
(UFU)
(USP)
(UFRN)
(Univ. Tec. Fed. de Paran)
(UNIFRA)
(UNIVATES)
(UTAM)
(UNESP)
(Colgio Objetivo de Campinas)
(Colgio Etapa)
(Univ. Federal do Tocantins)
(UFMG)
(UFES)
(Centro Educacional Leonardo Da Vinci)
(UC. Dom Bosco)
(UFPB))
(UNOCHAPEC)
(UFPI)
(Grupo Educacional Ideal)
(UFMA)
(UFSC)
(UFPB)
(Instituto Pueri Domus)
(UFAL)
(UFSC)
(UFBA)
(UFRG)
(Grupo Educacional Ideal)
(Colgio Farias Brito, Pr-vestibular)
(Cursinho CLQ Objetivo)
(UFMA)
(Colgio Anglo)
(UFGO)
(Col. Aplic. da UFPE)
(Escola Tcnica Federal de Roraima)
(UNIFESP)
(Centro Educacional Logos)
(IM-UFRGS)
(UFMG)
(UEBA)
(U. Federal de Rondnia)
(U. Federal de Sergipe)
(U. Estadual de Maring)
(U. Metodista de SP)
(CEFET GO)
(UNESPAR/FECILCAM)

100

Juiz de Fora MG
Ribeiro Preto SP
Viosa MG
Osrio RS
Uberlndia MG
So Carlos SP
Natal RN
Pato Branco - PR
Santa Mara RS
Lajeado RS
Manaus AM
S.J. do Rio Preto SP
Campinas SP
So Paulo SP
Arraias TO
Belo Horizonte MG
Vitria ES
Taguatingua DF
Campo Grande MS
Joo Pessoa PB
Chapec SC
Teresina PI
Belm PA
So Luis MA
Florianpolis SC
Campina Grande PB
Santo Andr SP
Macei AL
Florianpolis SC
Salvador BA
Rio Grande RS
Belm PA
Fortaleza CE
Piracicaba SP
So Luis MA
Atibaia SP
Goinia GO
Recife PE
Boa Vista RR
SJ dos Campos SP
Nova Iguau RJ
Porto Alegre RS
Belo Horizonte MG
Juazeiro BA
Porto Velho RO
So Cristovo SE
Maring PR
S.B. do Campo SP
Jata GO
Campo Mouro PR

manual 2010/7/12 9:12 page i #1

Programa de Iniciao Cientfica Jnior PIC

Manual

manual 2010/7/12 9:12 page ii #2

Olimpada Brasileira de Matemtica das Escolas Pblicas OBMEP


Direo Acadmica: Csar Camacho (IMPA), Joo Lucas Barbosa (UFC), Suely Druck (UFF).
Programa de Iniciao Cientfica PIC
Coordenadora Acadmica: Claudia Vasconcelos.
Coordenadores do Frum: Ana Lcia da Silva (UEL), Francisco Dutenhefner (UFMG) e Florncio Guimares Filho (UFES).

Coordenadores Regionais de Iniciao Cientfica: AC: Jerssiney de Oliveira Escola Joaquim Falco Macedo, AL: Sinvaldo da Gama UFAL, AM: Danilo Benars UFAM, AP: Marcio
Bahia UNIFAP, BA: Luzinalva Morim UFBA, CE: Onofre Farias C7S, DF: Kellcio Arajo
UNB, ES: Luzia Casati UFES, GO: Jos Hilrio UFG, MA: Joo de Deus da Silva UFMA,
MG01: Gilcione Costa UFMG, MG02: Luiz Cludio Pereira UFV, MG03: Francinildo Ferreira UFSJ, MG04: Luiz Alberto Salomo UFU, MG05: Marise Silveira Unimontes, MG06:
Joo Batista Zuliani Cefet MG, MS: Sonia di Giacomo UFMS, MT: Martinho Arajo UFMT,

PA: Augusto Csar Costa UFPA, PB: Jos de Arimatia UFCG, PE: Maite Kulesza UFRPE,
PI: Gilvan de Oliveira UFPI, PR01: Ana Lucia da Silva UEL, PR02: Florinda Miyaoka UFPR,
RJ01: Jair Salvador UFRJ, RJ02: Jones Colombo UFF, RN: Jos Querginaldo UFRN, RO:
Carlos Vincius Ramos UNIR, RR: Raimundo Nonato Arajo UFRR, RS: Elizabeth aa Costa
UFRGS, SC: Lcio Bezerra UFSC, SE: Valdenberg da Silva UFS, SP01: Jos Carlos Rodrigues
FTC - UNESP, SP02: Aparecida da Silva IBILCE/UNESP, SP03: Raul Ribeiro Anglo Atibaia,
SP04: Pablo Ganassim Etapa, SP05: Vnia Rodrigues Universidade Metodista de So Paulo,
TO: Dirlei Ruscheinsky UFT.

manual 2010/7/12 9:12 page iii #3

0 Sumrio

Quase tudo que voc deve saber sobre o PIC


1.1 O que a Iniciao Cientca . . . . . . . . . .
1.2 A equipe . . . . . . . . . . . . . . . . . . . . .
1.3 Como funcionar o PIC-2009 . . . . . . . . . .
1.4 O bolsista do PIC . . . . . . . . . . . . . . . . .
1.5 As obrigaes dos alunos que participam do PIC
1.6 Como aproveitar bem os encontros presenciais .
1.7 Como participar do frum . . . . . . . . . . . .
1.8 Algumas boas ideias . . . . . . . . . . . . . . .
1.9 Regras de participao no frum . . . . . . . .
1.10 Certicado . . . . . . . . . . . . . . . . . . . .
1.11 Encontro dos Medalhistas da OBMEP (EMO) .

.
.
.
.
.
.
.
.
.
.
.

.
.
.
.
.
.
.
.
.
.
.

.
.
.
.
.
.
.
.
.
.
.

.
.
.
.
.
.
.
.
.
.
.

.
.
.
.
.
.
.
.
.
.
.

.
.
.
.
.
.
.
.
.
.
.

.
.
.
.
.
.
.
.
.
.
.

.
.
.
.
.
.
.
.
.
.
.

1
1
2
2
3
4
5
5
6
6
7
8

Contedos
2.1 Nvel 1 Multiplicidades 1 e 2 . . . . . . . . . . . . .
2.2 Nvel 2 Multiplicidade 1 . . . . . . . . . . . . . . .
2.3 Nvel 2 Multiplicidade 2 . . . . . . . . . . . . . . .
2.4 Nvel 2 Multiplicidade 3 e Nvel 3 Multiplicidade 1
2.5 Nvel 2 Multiplicidade 4 e Nvel 3 Multiplicidade 2
2.6 Nvel 3 Multiplicidades 3, 4 e 5 . . . . . . . . . . .

.
.
.
.
.
.

.
.
.
.
.
.

.
.
.
.
.
.

.
.
.
.
.
.

.
.
.
.
.
.

.
.
.
.
.
.

.
.
.
.
.
.

9
9
10
11
11
12
13

O que TEX?
3.1 Knuth inventou o TEX... . . . . . . . . . . . . . . . . . . . . . . .
3.2 ... e Lamport criou o LATEX . . . . . . . . . . . . . . . . . . . . . .

15
15
16

iii

.
.
.
.
.
.
.
.
.
.
.

.
.
.
.
.
.
.
.
.
.
.

manual 2010/7/12 9:12 page iv #4

iv

Sumrio

Escrevendo e desenhando no frum


4.1 Exemplos Bsicos . . . . . . . .
4.2 Letras Gregas . . . . . . . . . .
4.3 Aritmtica . . . . . . . . . . .
4.4 Geometria . . . . . . . . . . .
4.5 Setas . . . . . . . . . . . . . .
4.6 Smbolos Diversos . . . . . . .
4.7 Conjuntos . . . . . . . . . . . .
4.8 Matrizes e Determinantes . . .
4.9 Somatrios e Produtrios . . .
4.10 Diversos . . . . . . . . . . . .
4.11 Construindo guras com LATEX

.
.
.
.
.
.
.
.
.
.
.

.
.
.
.
.
.
.
.
.
.
.

.
.
.
.
.
.
.
.
.
.
.

.
.
.
.
.
.
.
.
.
.
.

.
.
.
.
.
.
.
.
.
.
.

.
.
.
.
.
.
.
.
.
.
.

.
.
.
.
.
.
.
.
.
.
.

.
.
.
.
.
.
.
.
.
.
.

.
.
.
.
.
.
.
.
.
.
.

.
.
.
.
.
.
.
.
.
.
.

.
.
.
.
.
.
.
.
.
.
.

.
.
.
.
.
.
.
.
.
.
.

.
.
.
.
.
.
.
.
.
.
.

.
.
.
.
.
.
.
.
.
.
.

.
.
.
.
.
.
.
.
.
.
.

.
.
.
.
.
.
.
.
.
.
.

.
.
.
.
.
.
.
.
.
.
.

.
.
.
.
.
.
.
.
.
.
.

.
.
.
.
.
.
.
.
.
.
.

19
19
20
20
21
21
22
22
22
23
24
24

Calendrio dos Encontros

27

Coordenadores Regionais de Iniciao Cientca CRICS

49

Desaos
[]

55

manual 2010/7/12 9:12 page 1 #5

1 Quase tudo que voc deve


saber sobre o PIC
Como premiado da OBMEP-2009, voc foi convidado a participar do Programa de
Iniciao Cientca da OBMEP (PIC). Antes de tudo, entenda que participar desse
Programa um privilgio porque voc vai aprender Matemtica muitssimo interessante e tecnologias modernas de comunicao, e ainda conhecer muitos colegas
que tambm tm interesse pela Matemtica.
muito importante que voc conhea o que esse Programa, quais so as suas
obrigaes como aluno ou como bolsista e que se prepare para aproveit-lo da melhor maneira possvel. Vamos l!

1.1
O que a Iniciao Cientfica
A Iniciao Cientca em Matemtica serve para transmitir aos alunos cultura matemtica bsica e trein-los no rigor da leitura e da escrita de resultados, nas tcnicas
e mtodos, na independncia do raciocnio analtico, entre outros. Serve tambm
para estimular a criatividade atravs do confronto com os problemas interessantes
da Matemtica.

manual 2010/7/12 9:12 page 2 #6

Quase tudo que voc deve saber sobre o PIC

1.2
A equipe
Como todas as atividades da OBMEP, o PIC de responsabilidade da Direo Acadmica da OBMEP, que conta com a seguinte equipe para sua execuo:
Coordenao Acadmica
Coordenadores Regionais de Iniciao Cientca (CRIC)
Administradores do Frum
Coordenadores do Frum (CF)
Moderadores do Frum (MF)
Professores Orientadores (PO)
Monitores
Os Coordenadores Regionais de Iniciao Cientca (CRICs), em sua maioria professores universitrios, so os responsveis regionais pelo PIC. Consulte-os
sempre que achar necessrio. Os dados para contato podem ser encontrados na
pgina 49 ou no site www.obmep.org.br.
O Administrador do Frum e Coordenadores do Frum so responsveis pela
coordenao dos trabalhos virtuais.
Os Professores Orientadores e os Monitores trabalham com os alunos em encontros presenciais.
A Coordenao Acadmica, sediada no IMPA, responsvel pela parte administrativa do PIC.

1.3
Como funcionar o PIC-2009
O PIC consta das seguintes atividades:
Dez encontros presenciais.
Discusses virtuais no frum.

manual 2010/7/12 9:12 page 3 #7

O bolsista do PIC

Tarefas para serem executadas em casa e no frum.


Estudo em grupo ou individual em casa.
Os encontros presenciais so dirigidos por Professores Orientadores e neles os
alunos recebem o material de estudo, orientao e o cronograma de Estudos sobre
os temas a serem estudados entre dois encontros. Esse material ser discutido no
frum, entre os alunos, sob orientao dos Moderadores do Frum. Os Professores
Orientadores e os Moderadores em sua maioria so professores universitrios.
Alm dos encontros presenciais, os alunos devem dedicar 4 a 6 horas semanais, no mnimo, para cumprir suas atividades no PIC (estudar contedos, resolver
problemas, participar do frum, etc.).

1.4
O bolsista do PIC
Ser bolsista do CNPq um diferencial importante na vida dos alunos pela excelncia
que caracteriza essa bolsa e, com certeza, uma valorizao especial do currculo
escolar. Espera-se de um bolsista uma grande dedicao ao Programa, e que sua
participao seja uma experincia enriquecedora pessoal e acadmica, a marc-lo
por toda a sua vida.
S podem ter bolsa do CNPq os alunos que durante a vigncia do PIC estiverem regularmente matriculados em escolas pblicas. Por isso, no incio do
programa e em janeiro de 2011, voc dever enviar o comprovante de matrcula
em escola pblica.
Ateno: voc receber uma mensagem do CNPq no seu e-mail,
com o Termo de Aceitao da sua bolsa de estudos. Para receber
a sua bolsa, voc precisar fazer todos os procedimentos que
constaro na mensagem.
Fique atento aos comunicados da Coordenao Acadmica
para fazer o passo a passo do processo de aceite da sua bolsa
de estudos!

manual 2010/7/12 9:12 page 4 #8

Quase tudo que voc deve saber sobre o PIC

1.5
As obrigaes dos alunos que participam do PIC
Bom desempenho nas atividades de estudos: ateno, dedicao, cumprimento
das tarefas;
Assiduidade nas aulas presenciais de no mnimo 80%;
Participao regular nas atividades do frum, no se ausentando por mais de
15 dias. A ausncia do frum por um perodo superior a 45 dias acarretar no
desligamento automtico do programa;
Cumprimento do Calendrio do PIC;
Enviar os documentos solicitados;
Manter seus dados atualizados;
Disciplina de acordo com as normas estabelecidas.
E ainda: os alunos tero uma avaliao a cada encontro presencial com o Professor Orientador e tarefas quinzenais no frum. Os alunos do nvel 3 com multiplicidades 3, 4 e 5 tero uma avaliao mensal no frum (esta avaliao substitui
a avaliao presencial dos outros grupos). Essas tarefas e avaliaes recebero um
conceito A, B, C, D ou E, de acordo com o critrio:
A = timo
B = bom
C = deve melhorar
D = precisa melhorar muito
E = insuciente ou no postou no frum
O no cumprimento de uma dessas obrigaes pode levar ao
desligamento do aluno do Programa e, no caso dos bolsistas,
ao cancelamento da bolsa.
O aluno que receber trs conceitos E na avaliao presencial ou
na tarefa quinzenal no frum ou avaliao mensal no frum
(N3 com multiplicidades 3, 4 e 5) ser desligado do Programa
e, se bolsista, ter a bolsa cancelada.

manual 2010/7/12 9:12 page 5 #9

Como aproveitar bem os encontros presenciais

1.6
Como aproveitar bem os encontros presenciais
No espere aulas expositivas como as da escola, elas sero raras.
Discuta com seu Professor Orientador e colegas os assuntos e problemas tratados
no frum.
Avalie com o Professor Orientador o cumprimento do seu cronograma de estudos.
Apresente pontualmente as tarefas que lhe foram solicitadas.
Estude com antecedncia o material a ser tratado no encontro presencial e aproveite esses encontros para esclarecer as dvidas.

1.7
Como participar do frum
O endereo do frum www.obmep.org.br/forum.
O seu acesso ao frum ser mediante o login e a senha.
Acesse o frum com frequncia para estar em dia com os assuntos em discusso.
Participe das discusses com perguntas, sugestes, dvidas e respostas.
Tente resolver os problemas e postar solues ou ideias de soluo.
Tire suas dvidas sobre os textos estudados.
Sinta-se vontade para propor problemas.
Lembre-se de que existem vrias maneiras de resolver um mesmo problema. Assim, se um determinado problema no frum j foi resolvido, leia a soluo do seu
colega com ateno. Caso tenha alguma dvida, pea esclarecimentos. Escreva
tambm a sua soluo no frum, mesmo que o problema j tenha sido resolvido.
Obedea s regras de participao no frum.

manual 2010/7/12 9:12 page 6 #10

Quase tudo que voc deve saber sobre o PIC

1.8
Algumas boas ideias
Mantenha-se em dia com o seu Cronograma de Estudos no se atrase!
Tente ler tudo de primeira mo, antes de qualquer explicao. Caso no entenda, no se desespere, isso absolutamente normal e um excelente exerccio
para o seu crebro. Leia outra vez e muitas outras vezes mais.
No desista nunca, alguns assuntos e problemas so mesmo duros de vencer e
exigem tempo, pacincia e garra.
Estude com antecedncia o material a ser tratado no encontro presencial ou no
frum.
Aproveite para se tornar um autodidata e usufruir disso pelo resto da vida!

1.9
Regras de participao no frum
1. As regras de convivncia e boas maneiras usuais em qualquer sala de aula so
essenciais no frum.
2. Aqui utilizamos o portugus formal. No use linguagem de internet, como
vc no lugar de voc, Aki no lugar de aqui, etc.
3. No escreva o texto todo em maisculas. Utilize maisculas apenas quando
necessrio.
4. Evite usar fontes grandes demais ou pequenas demais.
5. Evite abusar de cores nas mensagens. Elas s devem ser usadas para destacar
parte da mensagem.
6. No esquea que este frum um ambiente de ensino-aprendizagem. No
escreva ou coloque imagens que fujam desse interesse.
7. Mensagens que fogem ao contedo abordado em determinado subfrum podem ser movidas para outros subfruns e as que fugirem ao contedo do
frum podem ser apagadas.

manual 2010/7/12 9:12 page 7 #11

Certificado

8. Antes de criar um novo tpico, verique se um tpico com o mesmo problema ou assunto j foi criado, evitando a duplicao de tpicos.
9. No coloque um novo problema em um tpico j criado. Se voc quer propor um novo problema, crie um novo tpico.
10. Links para sites externos so permitidos somente quando tratar de material
relacionado ao contedo estudado no frum.
11. Para usar imagens, faa upload diretamente no frum. No utilize sistema de
compartilhamento de terceiros porque nesses sistemas as imagens so apagadas aps determinado tempo.
12. Procure escrever mensagens com enunciados de problemas completos e solues claras. Use a ferramenta LATEX para contedos com equaes e smbolos
matemticos.
13. Procure colocar no ttulo das mensagens uma descrio do problema proposto, como Altura do tringulo a partir dos lados. Jamais utilize ttulos
do tipo: Mais um problema, Resolvam essa, Problema Fcil.
14. O frum poder ser fechado em dias especcos para manuteno tcnica ou
em dias de provas importantes, como a primeira fase da OBMEP.
O principal objetivo do frum a troca de conhecimento entre todos os seus usurios. No se preocupe em postar apenas as solues para os problemas propostos.
Escreva solues ainda que incompletas, leia as solues de seus colegas de turma,
esclarea suas dvidas e as de seus colegas, seja participativo. Alm da discusso de
problemas, o frum deve ser usado tambm para que o aluno esclarea dvidas sobre
a teoria estudada.

1.10
Certificado
Os alunos que cumprirem o programa, com bom aproveitamento e assiduidade,
recebero um certicado de participao.

manual 2010/7/12 9:12 page 8 #12

Quase tudo que voc deve saber sobre o PIC

1.11
Encontro dos Medalhistas da OBMEP (EMO)
Fique atento! At 2010, o EMO era dirigido apenas aos medalhistas de ouro, mas
o EMO-2011 ser dirigido a 300 alunos do PIC selecionados de acordo com o seu
desempenho a partir de junho de 2010.
Esse encontro a ser realizado em janeiro de 2011 ter a durao de 6 dias e
constar de minicursos, gincanas e atividades sociais, culturais e esportivas.

manual 2010/7/12 9:12 page 9 #13

2 Contedos
Os contedos estudados no PIC variam de acordo com o nvel (1, 2 ou 3) e com o
nmero de participaes no programa, o qual chamamos de multiplicidade. Assim,
um aluno que participa pela segunda vez do programa tem mutiplicidade 2.

2.1
Nvel 1 Multiplicidades 1 e 2
(1) Aritmtica: Paridade, Sistema Decimal e Operaes, Bases Numricas.
(2) Resoluo de Problemas, Algoritmo da Diviso, Mltiplos e Divisores, Critrios
de Divisibilidade, Decomposio em Fatores Primos.
(3) Resoluo de Problemas sobre Divisibilidade e Restos, MMC e MDC.
(4) Combinatria: Resoluo de Problemas envolvendo o Princpio Multiplicativo.
(5) Princpios bsicos de Probabilidade. Resoluo de Problemas sobre Probabilidade.
(6) Atividades de Contagem a partir da Criptograa.
(7) Geometria: Resoluo de Problemas Bsicos de Geometria. Frmula de
Pick.

manual 2010/7/12 9:12 page 10 #14

10

Contedos

(8) Teorema de Pitgoras e reas.


(9) Desigualdade Triangular.
(10) Dobraduras. Resoluo de Problemas estudados em todos os mdulos anteriores.

2.2
Nvel 2 Multiplicidade 1
(1) Aritmtica: Nmeros naturais (sistema decimal, ordem, operaes, mltiplos e divisores, critrios de divisibilidade (2, 3, 5, 9, 10), potenciao).
(2) Nmeros naturais (nmeros primos, crivo de Eratstenes, Teorema Fundamental da Aritmtica).
(3) Nmeros inteiros (paridade, mltiplos e divisores, algoritmo da diviso, mmc,
MDC, congruncias e somas, congruncias e produtos).
(4) Combinatria: Mtodos de contagem e probabilidade.
(5) Permutaes e Combinaes.
(6) Geometria: Semelhana de Tringulos, Teorema de Pitgoras.
(7) Construes Geomtricas.
(8) Trigonometria do Tringulo Retngulo.
(9) Equaes e Inequaes: Equaes de primeiro e segundo grau. Inequaes
do primeiro grau.
(10) Equaes e inequaes do segundo grau, mximos e mnimos.

manual 2010/7/12 9:12 page 11 #15

Nvel 2 Multiplicidade 2

11

2.3
Nvel 2 Multiplicidade 2
(1) Aritmtica: Nmeros naturais (sistema decimal, ordem, operaes, mltiplos e divisores, critrios de divisibilidade (2, 3, 5, 9, 10), potenciao, nmeros
primos, crivo de Eratstenes, Teorema Fundamental da Aritmtica).
(2) Nmeros inteiros (mltiplos, divisores, paridade, mmc, MDC, algoritmo de
Euclides, relao de Bezout, congruncias).
(3) Aritmtica dos restos, equaes diofantinas e critrios de divisibilidade.
(4) Combinatria: Mtodos de contagem e probabilidade.
(5) Permutaes, Combinaes e o Tringulo de Pascal.
(6) Geometria: Semelhana de Tringulos, Teorema de Pitgoras.
(7) reas
(8) Trigonometria do Tringulo Retngulo.
(9) Equaes, Inequaes e Desigualdades: Equaes e inequaes de primeiro e segundo graus.
(10) Inequaes do segundo grau. Mximos e mnimos. Desigualdades (mdia aritmtica e mdia geomtrica).

2.4
Nvel 2 Multiplicidade 3 e Nvel 3 Multiplicidade 1
(1) Aritmtica e Criptografia: Os Nmeros Naturais. Representao dos Naturais. Os Nmeros Inteiros e o Algoritmo da Diviso.
(2) Algoritmo do mdc de Euclides. Aplicaes da Relao de Bzout. Equaes
Diofantinas Lineares. Congruncias.

manual 2010/7/12 9:12 page 12 #16

12

Contedos

(3) Teorema Chins do Resto. Criptograa RSA.


(4) Combinatria: Mtodos de Contagem. Probabilidade.
(5) Mais Permutaes e Combinaes. Probabilidade Condicional.
(6) Geometria: Teorema de Pitgoras. reas.
(7) Construes Elementares. Lugares Geomtricos. Expresses Algbricas.
(8) Induo: Induo Matemtica
(9) Trigonometria.
(10) Nmeros Complexos.

2.5
Nvel 2 Multiplicidade 4 e Nvel 3 Multiplicidade 2
(1) Aritmtica e Criptografia: Divisibilidade e restos. Congruncia e equaes diofantinas.
(2) Aritmtica Modular. Teorema de Euler (via teoria de nmeros, sem usar grupos).
(3) Sistemas de Congruncias. Teorema Chins do Resto. Criptograa RSA.
(4) Combinatria: Princpio Multiplicativo.
(5) Geometria analtica plana: coordenadas, distncias, equaes da reta,
ngulo entre retas, rea de um tringulo, circunferncia.
(6) Geometria Plana: Desigualdades. Movimentos rgidos e congruncia. ngulos.

manual 2010/7/12 9:12 page 13 #17

Nvel 3 Multiplicidades 3, 4 e 5

13

(7) rea. Problemas diversos.


(8) Induo: Induo Matemtica.
(9) Complexos e Equaes Algbricas: Forma algbrica e forma trigonomtrica de um nmero complexo. Razes da unidade.
(10) Polinmios complexos. Diviso de polinmios. Reduo do grau de uma equao algbrica. Teorema Fundamental da lgebra. Relao entre coecientes e
razes.

2.6
Nvel 3 Multiplicidades 3, 4 e 5
Para estes alunos ser desenvolvido um curso de Matemtica
Discreta, inteiramente no frum. Os alunos faro atividades
nos encontros presenciais denidas pelo CRIC responsvel.
(1) Contagem
(2) Ferramentas combinatrias.
(3) Coecientes binomiais e Tringulo de Pascal.
(4) Grafos.
(5) rvores.
(6) Optimizao.
(7) Emparelhamento em grafos.
(8) Combinatria em Geometria.
(9) Frmula de Euler.
(10) Colorindo mapas e grafos.

manual 2010/7/12 9:12 page 14 #18

manual 2010/7/12 9:12 page 15 #19

3 O que TEX?
3.1
Knuth inventou o TEX...
O problema de escrever matemtica
no computador surgiu na dcada de 70.
Um dos primeiros grandes matemticos
a pesquisar em Cincia da Computao,
Donald Knuth da Universidade de Stanford, encontrou uma soluo que continua atual mais de trinta anos depois.
Por volta de 1976, Knuth tinha escrito os
dois primeiros volumes da coleo e
Art of Computer Programming e estava
totalmente insatisfeito com o resultado
impresso. Ele no queria apenas que
o livro fosse impresso, mas queria algo
belo.

Knuth partiu para a busca de uma soluo. Um dos primeiros passos foi a interrupo de sua pesquisa por um ano para, acompanhado por sua esposa, assistir
aulas de design com o professor de arte de Stanford, Matthew Kahn. A ideia era
tentar capturar a essncia do design, no apenas seu visual. Por exemplo, como um
processador deveria quebrar as linhas em um pargrafo? Esteticamente, o ideal

15

manual 2010/7/12 9:12 page 16 #20

16

O que TEX?

que no existam espaos excessivos entre as palavras e que no existam muitos hifens. Knuth transformou esse problema em combinatria e fez um algoritmo que
calcula a maneira tima de quebrar as linhas em um pargrafo.
Como resultado do seu trabalho, surgiram o processador de textos TEX e o sistema de descrio de fontes METAFONT, ambos colocados em domnio pblico.
O TEX foi projetado com dois objetivos principais em mente: permitir que qualquer pessoa possa produzir livros de alta qualidade com um esforo razovel e dar
exatamente o mesmo resultado em todos os computadores, agora e no futuro.

3.2
... e Lamport criou o LATEX
No incio da dcada de 80, o matemtico Leslie Lamport planejava escrever o livro
Great American Concurrency Book e digitar utilizando TEX. Ele escreveu um conjunto de macros que facilitaram bastante o trabalho. Essas macros foram posteriormente colocadas em domnio pblico. Era o incio do LATEX. Lamport at hoje no
escreveu o livro pretendido, mas em 1986 lanou o livro LATEX: A Document Preparation System, que ajudou a popularizar o LATEX. De l para c, vrios conjuntos de
macros para TEX surgiram, como ConTeXt e JadeTeX, mas sem dvida o LATEX o
mais utilizado.
Hoje o TEX popular em todo o mundo, principalmente na rea acadmica,
notadamente em matemtica, fsica, cincia da computao e engenharias.
Uma das vantagens do LATEX a sua modularizao. Qualquer um pode escrever
um conjunto de macros que automatizam determinados procedimentos e facilitam
a vida de todos. No caso do LATEX, estas macros so chamadas de pacotes e existem
milhares de pacotes escritos por centenas de usurios ao redor do mundo.
A principal desvantagem inicial do TEX que no um editor WYSIWYG (acrnimo da expresso em ingls What You See Is What You Get O que voc v
o que voc recebe). Isto signica que digitamos o texto usando uma linguagem
especca, compilamos e depois vemos o resultado. Isto pode parecer muito estranho para quem est acostumado a editores WYSIWYG, como o Word, mas uma
barreira que pode ser facilmente superada.

manual 2010/7/12 9:12 page 17 #21

... e Lamport criou o LATEX

17

Curiosidades
Knuth at hoje no terminou a coleo e Art of Computer Programming. Dos sete volumes previstos inicialmente, somente trs volumes completos foram lanados, alm de
quatro fascculos do volume 4.
Ele recebeu inmeros prmios como pesquisador em Cincia da Computao e em agosto
de 1999 seu nome foi dado a um pequeno planeta descoberto por P. Pravec and P. Kusnirk.
Desde 2001, Lamport pesquisador da Microso.
Existem verses de TEX para praticamente todos os sistemas operacionais, incluindo Windows, Mac OS X e Linux.
O cdigo fonte do TEX foi colocado em domnio pblico, e Knuth recomenda modicaes ou experincias com esse cdigo fonte, mas, para garantir a mesma sada em todas
as verses do TEX, ele deseja que qualquer novo programa obtido tenha outro nome. Para
garantir isso, a American Mathematical Society registrou a marca TEX e qualquer implementao do sistema deve passar por um teste antes de ser chamada de TEX.
O nome TEX deve ser pronunciado como tekh. O X representa a letra grega (chi). TEX
uma abreviao de (techn), que tambm a origem da palavra tcnico.
Knuth escreveu cinco livros sobre TEX: e TEX book, TEX: e Program, e METAFONT book, METAFONT: e Program e Computer Modern Typefaces, todos lanados
pela Addison-Wesley.
As verses de TEX so numeradas como aproximaes do nmero . A verso atual a
3.141592. Knuth deseja que, aps a sua morte o TEX no seja alterado, com exceo da
verso, que dever ser a . Analogamente, as verses de METAFONT so numeradas
como aproximaes do nmero e, base dos logaritmos naturais.
Atualmente, os grupos de usurios TEX (www.tug.org) de diversos pases so responsveis
pela distribuio, manuteno e atualizaes nas macros para TEX.

manual 2010/7/12 9:12 page 18 #22

manual 2010/7/12 9:12 page 19 #23

4 Escrevendo e desenhando
no frum
O frum do Programa de Iniciao Cientca est associado a uma instalao do
programa LATEX, utilizado para digitar matemtica.
Para tal, voc deve digitar [tex] comandos [/tex].
Por exemplo, digitando
[ tex ]\ frac { 3 } { 8 } [ / tex ]
o sistema converter seu cdigo para uma imagem contendo 38 . A imagem s exibida na mensagem a ser visualizada clicando no boto Prever ou no boto Enviar.
Sugerimos que voc aprenda inicialmente a escrever os exemplos bsicos abaixo, os
quais representam mais de 90% da utilizao do LATEX no frum.

4.1
Exemplos Bsicos
3
8
20\div3 .......... 20 3
\ne ..................... =
\pm .....................
10\% ................. 10%
3
\frac{3}{8} ......... 8
a_1 .................... a1
3/8 ................... 3/8
b_{23} ............... b23
3+5
7-2 ................. 7 2
\times .................
3\times2 .......... 3 2
3\cdot2 .............. 3 2
3+5 .................

\dfrac{3}{8}

19

........

x11

\sqrt{2} .............
2

3
\sqrt[3]{2} ........
2
< ........................ <
2<3 ................. 2 < 3
\le .....................
a\le b ............ a b
x^{11} ...............

manual 2010/7/12 9:12 page 20 #24

20

Escrevendo e desenhando no frum

ab
.............. (1, 2)

[1, 2]
{
........................ }

\ge .....................

[1,2] ...............

\{1,2\} ............

a\ge b ............

\{ ........................

\mid-3\mid .......

(1,2)

\}

{1, 2}
| 3 |

Ateno! A melhor maneira de aprender a digitar em LATEX


praticando. Voc no precisa decorar todos os comandos.
Consulte-os neste manual, quando precisar. Aps alguma prtica, voc j conhecer os principais comandos e com certeza
ter prazer em escrever usando o LATEX.

4.2
Letras Gregas

\alpha ..................
\beta ...................
\gamma ..................
\Gamma ..................
\pi ......................
\Pi ......................

\epsilon ...............
\phi ....................
\lambda .................
\mu ......................

..................
..................
.................
.................

\Delta .................

\rho .....................

\delta ..................

\sigma ..................
\Sigma
\theta
\Omega
\omega

4.3
Aritmtica

(mod n)
a\equiv b\pmod{n} ..................................................... a b (mod n)
\phi(n) ................................................................................ (n)
\lfloor x\rfloor .................................................................... x
\lceil x\rceil ....................................................................... x
\equiv .....................................................................................
\pmod{n} .........................................................................

manual 2010/7/12 9:12 page 21 #25

Geometria

21

4.4
Geometria
ABC
ABC

.......................... ABC

\angle ABC .......................

\vec{v} ..................................

\measuredangle ABC ............

\arco{AB}.............................. AB1

A\hat{B}C

[
ABC
rs
r\perp s ........................... r s
\circ .....................................
90^{\circ} ........................... 90
\overline{AB} ....................... AB

\triangle ABC ..................

ABC

\widehat{ABC} .....................

\cong ....................................

r\parallel s .......................

\triangle ABC \cong \triangle XYZ

XYZ
ABC =

\sim ......................................

\triangle ABC \sim \triangle XYZ

ABC XYZ

4.5
Setas

\Leftarrow ...........................
\Longrightarrow .................. =
\longrightarrow ..................
\mapsto ............................... 7
\rightarrow .........................
\iff

...............................

\Rightarrow

1 Comando

.........................

personalizado para o frum

\swarrow ..............................
\leftarrow ...........................
\nwarrow ..............................
\uparrow ...............................
\nearrow ..............................

\searrow

..............................

\downarrow

............................

manual 2010/7/12 9:12 page 22 #26

22

Escrevendo e desenhando no frum

4.6
Smbolos Diversos
\$

\ldots.................... . . \bullet ...............


. \diamond ..............
.
\vdots......................
\Diamond ............. 3
\cdots...................
\Box ................... 2
..
. \heartsuit ..........
\ddots.................
\ell........................ \spadesuit ..........
\infty ................... \diamondsuit ........
.......................

\dots.................... . . .

\therefore

...........

\approx ................

\bigstar ...............
\square ............... 
\blacksquare ........ 
\clubsuit

............

\star.....................

\TeX

.................. TEX
.............. LATEX

\LaTeX
\S

........................

\P

......................

4.7
Conjuntos
\in

...................

\not\in

...............

\subset.................
\not\subset ..........
\cap ....................

\cup ....................
\emptyset ..............
\mathbb{N} ............
\mathbb{Z} ............
\mathbb{Q} ............

N
Z
Q

R
C
P(X)

\mathbb{R} ............
\mathbb{C} ............
\mathcal{P}(X) ...

4.8
Matrizes e Determinantes
\begin{matrix}1 & 2\\ 3 & 4\end{matrix}

\begin{pmatrix}1 & 2\\ 3 & 4\end{pmatrix}

\begin{bmatrix}1 & 2\\ 3 & 4\end{bmatrix}

1 2
3 4
(
)
1 2
............................
3 4
[
]
1 2
.............................
3 4

...................................

manual 2010/7/12 9:12 page 23 #27

Somatrios e Produtrios

23

}
1 2
3 4


1 2


& 4\end{vmatrix} ...............................
3 4


1 2


& 4\end{Vmatrix} .............................
3 4

1
3\end{pmatrix} ................................... 2
3

\begin{Bmatrix}1 & 2\\ 3 & 4\end{Bmatrix}

\begin{vmatrix}1 & 2\\ 3

\begin{Vmatrix}1 & 2\\ 3

\begin{pmatrix}1 \\ 2 \\

............................

det A

\det{A} ................................................................................

4.9
Somatrios e Produtrios
\sum_{i=1}^{n} i^2..............................................................

i=1 i
n

i2

\displaystyle\sum_{i=1}^{n} i^2................................................

\sum_{i=1}^{100} i(i+1)................................................

100
i=1
100

\displaystyle\sum_{i=1}^{100} i(i+1)..................................

i=1

i(i + 1)
i(i + 1)

i=1

\prod_{i=1}^{10} \frac{i}{i+1} ...........................................

10

i
i=1 i+1

10

\displaystyle\prod_{i=1}^{10} \frac{i}{i+1}...........................
i=1

\displaystyle\sum_{i\ge 1} \frac{1}{i^2} = \frac{\pi^2}{6}..

i
i+1

1
2
=
i2
6
i1

manual 2010/7/12 9:12 page 24 #28

24

Escrevendo e desenhando no frum

4.10
Diversos
\begin{cases}x+y=10\\ x-y=4\end{cases} ..........................

{
x + y = 10
xy=4
40

1\overbrace{22\dots2}^{40}5 ..............................................

z }| {
1 22 . . . 2 5

\underbrace{11\dots1}_{100} ..................................................

11
. . . 1}
| {z
100

ax + b
(\dfrac{ax+b}{cx+d}).......................................................... (
)
cx + d
(
)
ax + b
\left(\dfrac{ax+b}{cx+d}\right).........................................
cx + d

Voc poder praticar e aprender mais sobre LATEX no frum de


Suporte. Acesse o tpico Escrevendo Matemtica com comandos
LATEX.
No mesmo frum, voc aprender a inserir imagens e a criar
guras com o LATEX.

4.11
Construindo figuras com LATEX
possvel fazer desenhos dos mais variados tipos usando LATEX. Existem centenas de pacotes para fazer guras. No frum foram instalados os pacotes pstricks e
pst-eucl. Os comandos para desenhar guras devem estar entre as tags [teximg] e
[/teximg].
Apresentamos a seguir alguns exemplos bsicos de guras feitas com pstricks.
Em todos os casos, a imagem mostrada direita contm, para facilitar a compreenso, uma grade de pontos que no consta no cdigo exibido.

manual 2010/7/12 9:12 page 25 #29

Construindo figuras com LATEX

25

4.11.1
Traando segmentos de reta
Os comandos abaixo denem uma caixa com extremidades (0,0) e (4,4) e dois
segmentos de reta que tm por extremidades os pontos designados. O parmetro
opcional no segundo segmento ([linestyle=dashed]) muda o estilo da linha para
tracejado.
4

\begin{pspicture}(0,0)(4,4)
\psline(0,1)(3,4)

\psline[linestyle=dashed](2,0)(2,4)
\psline{->}(1,0)(4,3)
\end{pspicture}

0
0

4.11.2
Circunferncias
Para construir uma circunferncia, devemos informar o centro e o raio com o comando \pscircle(x,y){r}, sendo (x,y) as coordenadas do centro da circunferncia e r o raio.
Na segunda circunferncia, aumentamos a espessura da linha com o parmetro
[linewidth=2pt].
4

\begin{pspicture}(0,0)(4,4)
\pscircle(2,2){2}

\pscircle[linewidth=2pt](3,3){1}
\end{pspicture}
2

0
0

manual 2010/7/12 9:12 page 26 #30

26

Escrevendo e desenhando no frum

4.11.3
Rotulando Pontos
O pacote pst-euclides nos permite dar nome aos pontos e depois utilizar os nomes dados para fazer outras construes.
O comando
\pstGeonode[PosAngle=](x,y){Nome} marca um ponto de coordenadas (x,y)
com o rtulo Nome. O ngulo do rtulo em relao ao ponto dado pelo parmetro
opcional PosAngle.
B
4

\begin{pspicture}(0,0)(4,4)
\pstGeonode(4,3){A}

3
b

\pstGeonode[PosAngle=90](1,4){B}

\pstGeonode[PosAngle=180](0,1){C}
2

\pstGeonode[PosAngle=-90](3,0){D}
\psline(A)(B)(C)(D)(A)
\end{pspicture}

1 b

0
b

4.11.4
Utilizando coordenadas polares

possvel utilizar coordenadas polares para denir os pontos. Neste caso, devemos
indicar a distncia r do ponto origem e o ngulo formado entre o segmento de
reta que une esse ponto origem e o eixo x.
As coordenadas polares devem ser separadas por ponto e vrgula: (r;).
2

\begin{pspicture}(-2,-2)(2,2)
\psline(2;0)(2;72)

\psline(2;72)(2;144)
\psline(2;144)(2;216)
\psline(2;216)(2;288)

\psline(2;288)(2;0)
\psline[linestyle=dashed](2;0)(2;144)

-1

\pscircle(0,0){2}
\end{pspicture}
-2
-2

-1

manual 2010/7/12 9:12 page 27 #31

5 Calendrio dos Encontros


A parte presencial do Programa de Iniciao Cientca ser realizada em dez encontros presenciais, cada um com 8 horas/aula.
Os Coordenadores Regionais de Iniciao Cientca devem indicar as datas dos
encontros nos polos, respeitando o seguinte calendrio.

Encontro
1
2
3
4
5
6
7
8
9
10

Datas Provveis
19 ou 26 de junho
31 de julho ou 7 de agosto
21 ou 28 de agosto
25 de setembro ou 2 de outubro
23 ou 30 outubro
20 ou 27 de novembro
11 ou 18 de dezembro
12 ou 19 de fevereiro de 2011
19 ou 26 de maro de 2011
09 ou 16 de abril de 2011

Nas pginas a seguir, voc encontrar espao para anotar a agenda dos encontros presenciais.
Nossa sugesto que voc faa a anotao no nal da aula, juntamente com o
seu Professor Orientador.

27

manual 2010/7/12 9:12 page 28 #32

Encontro

Data:

Local:

Nota da Avaliao:

Contedos Trabalhados:

Contedos a serem estudados em Casa: (indicar material e pginas)

Exerccios Resolvidos:

28

manual 2010/7/12 9:12 page 29 #33

29

Exerccios a serem resolvidos em Casa:

Anotaes:

Comunicaes entre a Famlia e o Professor Orientador:

manual 2010/7/12 9:12 page 30 #34

Encontro

Data:

Local:

Nota da Avaliao:

Contedos Trabalhados:

Contedos a serem estudados em Casa: (indicar material e pginas)

Exerccios Resolvidos:

30

manual 2010/7/12 9:12 page 31 #35

31

Exerccios a serem resolvidos em Casa:

Anotaes:

Comunicaes entre a Famlia e o Professor Orientador:

manual 2010/7/12 9:12 page 32 #36

Encontro

Data:

Local:

Nota da Avaliao:

Contedos Trabalhados:

Contedos a serem estudados em Casa: (indicar material e pginas)

Exerccios Resolvidos:

32

manual 2010/7/12 9:12 page 33 #37

33

Exerccios a serem resolvidos em Casa:

Anotaes:

Comunicaes entre a Famlia e o Professor Orientador:

manual 2010/7/12 9:12 page 34 #38

Encontro

Data:

Local:

Nota da Avaliao:

Contedos Trabalhados:

Contedos a serem estudados em Casa: (indicar material e pginas)

Exerccios Resolvidos:

34

manual 2010/7/12 9:12 page 35 #39

35

Exerccios a serem resolvidos em Casa:

Anotaes:

Comunicaes entre a Famlia e o Professor Orientador:

manual 2010/7/12 9:12 page 36 #40

Encontro

Data:

Local:

Nota da Avaliao:

Contedos Trabalhados:

Contedos a serem estudados em Casa: (indicar material e pginas)

Exerccios Resolvidos:

36

manual 2010/7/12 9:12 page 37 #41

37

Exerccios a serem resolvidos em Casa:

Anotaes:

Comunicaes entre a Famlia e o Professor Orientador:

manual 2010/7/12 9:12 page 38 #42

Encontro

Data:

Local:

Nota da Avaliao:

Contedos Trabalhados:

Contedos a serem estudados em Casa: (indicar material e pginas)

Exerccios Resolvidos:

38

manual 2010/7/12 9:12 page 39 #43

39

Exerccios a serem resolvidos em Casa:

Anotaes:

Comunicaes entre a Famlia e o Professor Orientador:

manual 2010/7/12 9:12 page 40 #44

Encontro

Data:

Local:

Nota da Avaliao:

Contedos Trabalhados:

Contedos a serem estudados em Casa: (indicar material e pginas)

Exerccios Resolvidos:

40

manual 2010/7/12 9:12 page 41 #45

41

Exerccios a serem resolvidos em Casa:

Anotaes:

Comunicaes entre a Famlia e o Professor Orientador:

manual 2010/7/12 9:12 page 42 #46

Encontro

Data:

Local:

Nota da Avaliao:

Contedos Trabalhados:

Contedos a serem estudados em Casa: (indicar material e pginas)

Exerccios Resolvidos:

42

manual 2010/7/12 9:12 page 43 #47

43

Exerccios a serem resolvidos em Casa:

Anotaes:

Comunicaes entre a Famlia e o Professor Orientador:

manual 2010/7/12 9:12 page 44 #48

Encontro

Data:

Local:

Nota da Avaliao:

Contedos Trabalhados:

Contedos a serem estudados em Casa: (indicar material e pginas)

Exerccios Resolvidos:

44

manual 2010/7/12 9:12 page 45 #49

45

Exerccios a serem resolvidos em Casa:

Anotaes:

Comunicaes entre a Famlia e o Professor Orientador:

manual 2010/7/12 9:12 page 46 #50

Encontro

Data:

10

Local:

Nota da Avaliao:

Contedos Trabalhados:

Contedos a serem estudados em Casa: (indicar material e pginas)

Exerccios Resolvidos:

46

manual 2010/7/12 9:12 page 47 #51

47

Exerccios a serem resolvidos em Casa:

Anotaes:

Comunicaes entre a Famlia e o Professor Orientador:

manual 2010/7/12 9:12 page 48 #52

manual 2010/7/12 9:12 page 49 #53

6 Coordenadores Regionais
de Iniciao Cientfica CRICS
O Coordenador Regional de Iniciao Cientca (CRIC) o responsvel em sua
Regio/UF pelo Programa de Iniciao Cientca (PIC), bem como pelo acompanhamento dos bolsistas da OBMEP. Abaixo, encontra-se a relao de todos os Coordenadores Regionais de Iniciao Cientca. Alguns Estados esto divididos em
sub-regies.
AC Jerssiney Correa de Oliveira
Endereo Eletrnico: cric_ac@obmep.org.br
Telefone: (68) 9962-1643
AL Sinvaldo Silva da Gama
Endereo Eletrnico: cric_al@obmep.org.br
Telefone: (82) 3214-1407
AM Danilo Benarrs (CR)
Endereo Eletrnico: cric_am@obmep.org.br
Telefone: (92) 3647-4400
AP Marcio Aldo Lobato Bahia
Endereo Eletrnico: cric_ap@obmep.org.br
Telefone: (96) 3312-1700
BA Luzinalva Miranda de Amorim (CR)
Endereo Eletrnico: cric_ba@obmep.org.br
Telefone: (71) 3283-6287

49

manual 2010/7/12 9:12 page 50 #54

50

Coordenadores Regionais de Iniciao Cientfica CRICS

CE Onofre Campos da Silva Farias


Endereo Eletrnico: cric_ce@obmep.org.br
Telefone: (85) 9673-0309
DF Kellcio Oliveira Araujo
Endereo Eletrnico: cric_df@obmep.org.br
Telefone: (61) 3107-6488
ES Luzia Maria Casati
Endereo Eletrnico: cric_es@obmep.org.br
Telefone: (27) 4009-7665
GO Jos Hilrio da Cruz
Endereo Eletrnico: cric_go@obmep.org.br
Telefone: (62) 3521-1288
MA Joo de Deus Mendes da Silva
Endereo Eletrnico: cric_ma@obmep.org.br
Telefone: (98) 8121-9137
MG01 Gilcione Nonato Costa
Endereo Eletrnico: cric_mg01@obmep.org.br
Telefone: (31) 3409-5790
MG02 Luiz Cludio Pereira
Endereo Eletrnico: cric_mg02@obmep.org.br
Telefone: (31) 3899-2393
MG03 Francinildo Nobre Ferreira
Endereo Eletrnico: cric_mg03@obmep.org.br
Telefone: (32) 3379-2549
MG04 Luiz Alberto Duran Salomo
Endereo Eletrnico: cric_mg04@obmep.org.br
Telefone: (34) 3239-4156
MG05 Marise Fagundes Silveira
Endereo Eletrnico: cric_mg05@obmep.org.br
Telefone: (38) 3229-8274/75

manual 2010/7/12 9:12 page 51 #55

51

MG06 Joo Batista Queiroz Zuliani


Endereo Eletrnico: cric_mg06@obmep.org.br
Telefone: (31) 3845-4600
MS Sonia Regina Di Giacomo
Endereo Eletrnico: cric_ms@obmep.org.br
Telefone: (67) 3345-7043
MT Martinho da Costa Araujo (CR)
Endereo Eletrnico: cric_mt@obmep.org.br
Telefone: (65) 3615-8917
PA Augusto Cesar dos Reis Costa
Endereo Eletrnico: cric_pa@obmep.org.br
Telefone: (91) 9124-1227
PB Jos de Arimatia Fernandes (CR)
Endereo Eletrnico: cric_pb@obmep.org.br
Telefone: (83) 3310-1161
PE Mait Kulesza
Endereo Eletrnico: cric_pe@obmep.org.br
Telefone: (81) 8888-3960
PI Gilvan Lima de Oliveira
Endereo Eletrnico: cric_pi@obmep.org.br
Telefone: (86) 3215-5835
PR01 Ana Lucia da Silva
Endereo Eletrnico: cric_pr01@obmep.org.br
Telefone: (43) 3371-5868
PR02 Florinda Katsume Miyaoka
Endereo Eletrnico: cric_pr02@obmep.org.br
Telefone: (41) 3361-3660
RJ01 Jair Salvador
Endereo Eletrnico: cric_rj01@obmep.org.br
Telefone: (21) 2562-7505 r. 214

manual 2010/7/12 9:12 page 52 #56

52

Coordenadores Regionais de Iniciao Cientfica CRICS

RJ02 Jones Colombo


Endereo Eletrnico: cric_rj02@obmep.org.br
Telefone: (21) 2620-5809
RN Jose Querginaldo Bezerra
Endereo Eletrnico: cric_rn@obmep.org.br
Telefone: (84) 3215-3780
RO Carlos Vincius da Costa Ramos
Endereo Eletrnico: cric_ro@obmep.org.br
Telefone: (69) 9983-1514
RR Raimundo Nonato Araujo Pedro
Endereo Eletrnico: cric_rr@obmep.org.br
Telefone: (95) 3621-3141
RS Elizabeth Quintana Ferreira da Costa
Endereo Eletrnico: cric_rs@obmep.org.br
Telefone: (51) 3308-6217
SC Licio Hernanes Bezerra (CR)
Endereo Eletrnico: cric_sc@obmep.org.br
Telefone: (48) 3721-9558 r.4205
SE Valdenberg Araujo da Silva (CR)
Endereo Eletrnico: cric_se@obmep.org.br
Telefone: (79) 3212-6836
SP01 Jos Carlos Rodrigues (CR)
Endereo Eletrnico: cric_sp01@obmep.org.br
Telefone: (18) 3229-5385/5359
SP02 Aparecida Francisco da Silva (CR)
Endereo Eletrnico: cric_sp02@obmep.org.br
Telefone: (17) 3221-2335
SP03 Raul Cintra de Negreiros Ribeiro
Endereo Eletrnico: cric_sp03@obmep.org.br
Telefone: (11) 4412-8163

manual 2010/7/12 9:12 page 53 #57

53

SP04 Pablo Rodrigo Ganassim


Endereo Eletrnico: cric_sp04@obmep.org.br
Telefone: (11) 5084-7508
SP05 Vania Cristina da Silva Rodrigues (CR)
Endereo Eletrnico: cric_sp05@obmep.org.br
Telefone: (11) 4366-5886
TO Dirlei Ruscheinsky
Endereo Eletrnico: cric_to@obmep.org.br
Telefone: (63) 3653-1531

manual 2010/7/12 9:12 page 54 #58

manual 2010/7/12 9:12 page 55 #59

7 Desafios
1.

Soma dos Algarismos

(a) Existem dois nmeros naturais consecutivos tais que a soma dos algarismos de cada um
deles divisvel por 9?
(b) Existem dois nmeros naturais consecutivos tais que a soma dos algarismos de cada um
deles divisvel por 19?

2.

Cartas azuis e brancas So dadas 17 cartas azuis, numeradas de 1 a 17, e 17 cartas


brancas, numeradas de 1 a 17. Forme 17 pares com 1 carta azul e 1 carta branca de tal modo
que as somas dos 17 pares sejam 17 nmeros consecutivos.
3.

Um truque com cartas Um mgico com os olhos vendados d 29 cartas numeradas de 1 a 29 para uma mulher da plateia. Ela esconde duas cartas no bolso e devolve as
restantes para a assistente do mgico.
A assistente escolhe duas cartas dentre as 27 e um homem da platia l, na ordem que
quiser, o nmero dessas cartas para o mgico. Aps isso, o mgico adivinha o nmero das
cartas que foram escondidas pela mulher.
Como o mgico e sua assistente podem combinar uma estratgia para realizarem esse
truque?
4. Formigas Geomtricas Trs formigas esto paradas em trs dos vrtices de um
retngulo no plano. As formigas se movem uma por vez. A cada vez, a formiga que se move o
faz segundo a reta paralela determinada pelas posies das outras duas formigas. possvel
que, aps alguns movimentos, as formigas se situem nos pontos mdios de trs dos lados do
retngulo original?

55

manual 2010/7/12 9:12 page 56 #60

56

5.

Desafios

e
Quadriltero Curioso No quadriltero convexo ABCD, BAC
= CBD

= BDA.
Mostre que
ACD
AC2 = BC2 + AD2 .

6.

Semideserto de nmeros primos Existe um bloco de 1000 inteiros positivos


consecutivos que no contm nenhum nmero primo. Um exemplo 1001! + 2, 1001! + 3,
. . . , 1001! + 1001, porque o primeiro nmero mltiplo de 2, o segundo mltiplo de 3 e
assim por diante.
Existe uma sequncia de 1000 inteiros positivos consecutivos que contm exatamente
5 nmeros primos?

7.

Outro truque Um mgico e sua assistente realizam um truque com um baralho de


52 cartas. Inicialmente o mgico est com os olhos vendados e uma pessoa da plateia retira
5 cartas e entrega para a assistente. Ela olha as cartas e coloca-as lado a lado: uma com a face
para baixo (no necessariamente a primeira), e as outras quatro com a face para cima. Em
seguida o mgico retira a venda e adivinha a carta que est com a face voltada para baixo.
Como o mgico e a sua assistente podem combinar um sistema que torne esse truque possvel?
Observao: Um baralho comum contm 52 cartas divididas em 4 conjuntos chamados naipes
( ). Cada naipe contm um conjunto de 13 cartas: A, 2, 3, . . . , 10, J, Q e K.

8.

Circunferncias Tangentes Sejam ABCD um retngulo e M o ponto mdio do


lado AB. A semicircunferncia de dimetro CD tangente semicircunferncia de dimetro
AM no ponto P.
B

(b) Prove que o ponto P est sobre a semicircunferncia de dimetro AD.

(a) Calcule a razo AB/BC.


b

b
b
b

(c) Prove que o ponto P est sobre a circunferncia com centro em B e raio igual
medida do lado BC.
b

P
b

manual 2010/7/12 9:12 page 57 #61

57

9. Circunferncias Tangentes O tringulo ABC est inscrito em uma circunferncia e AB um dimetro dessa circunferncia. Uma segunda circunferncia de centro O1
tangente internamente primeira e aos lados AC e CB do tringulo, como indica a gura.
Os lados do tringulo medem BC = a, CA = b e AB = c.
C
bb

bb

O1

Mostre que a medida do raio r1 da segunda circunferncia pode ser calculada como
r1 = a + b c.

10.

Alunos Inteligentes... Uma professora de matemtica pensou em um inteiro


positivo de dois algarismos. Ela deseja que seus dois inteligentes alunos Daniela e Adriano
determinem o valor exato do nmero pensado.
Para tal, informa reservadamente a Daniela a quantidade de divisores positivos do
nmero e condencia a Adriano a soma dos algarismos do nmero.
Uma breve conversa entre Daniela e Adriano transcrita abaixo:
Adriano: Eu no posso determinar o nmero.
Daniela: Nem eu, mas posso dizer se ele par ou mpar.
Adriano: Agora eu sei qual o nmero.
Daniela: Voc sabe? Ento eu tambm sei.
Suponha que os estudantes so honestos e existe lgica perfeita em tudo o que falaram. Determine o nmero pensado pela professora, justicando sua resposta.

Apresentao

Caros professores orientadores e alunos do Programa de


Iniciao Cientfica OBMEP 2007
Esta a 2a edio do nmero especial da Revista do
Professor de Matemtica RPM, que foi inicialmente
elaborado para utilizao no Estgio da 2a edio da OBMEP,
finalizado em maio/junho de 2008.
Esta 2a edio ser utilizada no Programa de Iniciao
Cientfica OBMEP 2007, com incio em junho de 2008.
A RPM, como seu nome diz, uma revista dedicada aos
professores de Matemtica da educao bsica, a alunos e
professores de cursos de licenciatura em Matemtica e a todos
aqueles que se interessam pela Matemtica do nvel mdio. O
tratamento dado aos temas abordados procura ser acessvel e
agradvel, sem sacrificar o rigor. A revista uma publicao da
Sociedade Brasileira de Matemtica SBM e tem sido editada e
distribuda sem interrupes desde 1982.
A revista publica crnicas, artigos e sees, como
Problemas, O leitor pergunta, Livros, Olhando mais de cima,
etc. Nos artigos, temas interessantes de nvel elementar ou
avanado so apresentados de modo acessvel ao professor e ao
aluno do ensino mdio ou de cursos de Licenciatura em
Matemtica. Uma experincia interessante em sala de aula, um
problema que suscita uma questo pouco conhecida, uma histria
que merea ser contada ou at uma nova abordagem de um assunto
conhecido podem compor um artigo da revista. Nas sees, a
revista conversa com os leitores, publicando problemas e/ou
i

solues propostas por eles, cartas, resenhas de livros, erros


encontrados em textos didticos, etc., sempre visando ao
aperfeioamento do trabalho do professor na sua sala de aula.
Para este exemplar especial, o Comit Editorial da RPM
escolheu artigos que pretendem ampliar o conhecimento dos alunos
em diferentes tpicos, bem como temas que motivem discusses
ou satisfaam a curiosidade terica e histrica de alunos
interessados em Matemtica. Por exemplo, as cnicas so tratadas
de modo prtico no texto Sorrisos, sussurros, antenas e
telescpios; a intuio desafiada em diferentes situaes no
texto Quando a intuio falha; a anlise combinatria utilizada
para discutir a funcionalidade da brincadeira Amigo oculto (ou
secreto), etc.
Apresentamos tambm uma seleo de 30 problemas,
cuidadosamente escolhidos entre os publicados na seo
Problemas, que abrangem a maioria dos tpicos do ensino mdio.
As solues dos problemas propostos esto no fim da revista.
Para o ensino fundamental, e tambm para o ensino mdio,
selecionamos 30 ...probleminhas, parte integrante da seo
Problemas dos nmeros usuais da revista. Os probleminhas so
caracterizados por exigir muito pouco conhecimento de contedo
especfico, apenas raciocnio lgico-dedutivo e domnio de
operaes elementares. a parte ldica, permitindo que
professores e alunos se divirtam, resolvendo problemas
desafiadores, e se sintam realizados ao obter as solues. As
respostas dos probleminhas tambm esto no final da revista.
Os artigos aqui apresentados no esto com as referncias
bibliogrficas: elas podem ser encontradas nos exemplares
originais da RPM.
Comit Editorial da RPM

ii

Contedo
Como escolher namorada pelo horrio dos trens

................ 1

Quando a intuio falha . . . . . . . . . . . . . . . . . . . . . . . . . . . . . . . . . . .

Eleies preferncia transitiva? . . . . . . . . . . . . . . . . . . . . . . . . . 9


A divisibilidade e o dgito verificador . . . . . . . . . . . . . . . . . . . . . . . . 11
O tamanho da Terra . . . . . . . . . . . . . . . . . . . . . . . . . . . . . . . . . . .

16

Problema das idades . . . . . . . . . . . . . . . . . . . . . . . . . . . . . . . . . . . .

21

A ilha dos sapatos gratuitos . . . . . . . . . . . . . . . . . . . . . . . . . . . . . . . . . 23


Fraes egpcias . . . . . . . . . . . . . . . . . . . . . . . . . . . . . . . . . . . . . .

27

As dzimas peridicas e a calculadora

.......................

30

...................................

34

Usando reas . . . . . . . . . . . . . . . . . . . . . . . . . . . . . . . . . . . . . . . . .

40

Trigonometria e um antigo problema de otimizao

45

Mania de Pitgoras

............

Vale para 1, para 2, para 3, ... Vale sempre? . . . . . . . . . . . . . . . . . . . . 48


Semelhanas, pizzas e chopes . . . . . . . . . . . . . . . . . . . . . . . . . . . . .

54

Sorrisos, sussurros, antenas e telescpios . . . . . . . . . . . . . . . . . . . . . . . 58


A Matemtica do GPS . . . . . . . . . . . . . . . . . . . . . . . . . . . . . . . . . . .

65

O problema do amigo oculto . . . . . . . . . . . . . . . . . . . . . . . . . . . . . . . . . 72


O princpio da casa dos pombos . . . . . . . . . . . . . . . . . . . . . . . . . . . . . . . 77
Probabilidade geomtrica:
os problemas dos ladrilhos, do encontro e do macarro . . . . . . . .

83

Alguns problemas clssicos sobre grafos . . . . . . . . . . . . . . . . . . . . . .

87

Srie harmnica . . . . . . . . . . . . . . . . . . . . . . . . . . . . . . . . . . . . . . . . . .

95

O que tem mais: racionais ou naturais? . . . . . . . . . . . . . . . . . . . . . . . . . 100


Problemas . . . . . . . . . . . . . . . . . . . . . . . . . . . . . . . . . . . . . . . . . . . .

104

...probleminhas . . . . . . . . . . . . . . . . . . . . . . . . . . . . . . . . . . . . . . . . . . 108
Solues dos problemas . . . . . . . . . . . . . . . . . . . . . . . . . . . . . . . . . . . 113
Respostas dos ...probleminhas . . . . . . . . . . . . . . . . . . . . . . . . . . . . . . . 131
iii

iv

Como escolher namorada


pelo horrio dos trens

Joo amava Lcia, que amava Joo. S que Joo, alm


de amar Lcia, tambm amava Letcia e tentava namorar
as duas ao mesmo tempo. Durante a semana, at que
dava, mas quando chegava o sbado noite era terrvel.
As duas queriam Joo e este no possua o dom da
presena ao mesmo tempo em dois lugares. Assim,
alternadamente, ou Lcia ou Letcia ficava sem sair com
Joo, nos embalos de sbado noite. Honesto, Joo decidiu
informar Lcia sobre a existncia de Letcia e Letcia
sobre Lcia. Com choros e lamrias de todos os lados,
Joo continuou dividido, sem saber quem escolher.
Joo usava como meio de transporte os trens
metropolitanos. Para visitar Lcia, Joo pegava trens que
iam no sentido da direita e para visitar Letcia pegava trens
que iam para a esquerda. Quanto a horrios no havia
dvidas: trens para cada lado de meia em meia hora. Mas
como escolher entre Lcia e Letcia?
Letcia, que era professora de Matemtica, props a
Joo um critrio justo, equnime, salomnico para escolher
entre as duas namoradas. A proposta foi: Joo iria para a
estao de trens sem nenhuma deciso. Ao chegar pegaria
o primeiro trem que passasse, fosse para a direita, fosse
para a esquerda. Proposta aceita, Joo comeou a usar
esse critrio aparentemente justo e aleatrio. Depois de
usar o critrio por cerca de trs meses, descobriu que
visitara Letcia muito mais que Lcia, e, se a sorte quis
1

assim, ficou com Letcia e com ela se casou sem nunca haver entendido
por que a sorte a privilegiara tanto. S nas bodas de prata do seu casamento
que Letcia contou a Joo a razo de o trem a ter escolhido muito mais
vezes que a concorrente. Letcia estudara os horrios dos trens e verificara
que os horrios eram:
Trens para a esquerda (Letcia): 8h00; 8h30; 9h00; 9h30; ...
Trens para a direita (Lcia):

8h05; 8h35; 9h05; 9h35; ...

Ou seja, considerando, por exemplo, o intervalo de tempo, 8h00 8h30,


o horrio H de chegada na estao, que faria Joo tomar o trem para a
direita, deveria ser tal que 8h00 < H < 8h05. Se 8h05 < H < 8h30, Joo
pegaria o trem para a esquerda. A situao se repete em qualquer outro
intervalo de 30 minutos: 25 minutos so favorveis ao trem da esquerda
e 5 minutos ao da direita.
Na guerra como no amor tudo vale..., at usar Matemtica.

Baseado no artigo
Como escolher namorada pelos
horrios do trem do subrbio
Manoel Henrique Campos Botelho, RPM 14

Quando a intuio falha

Problema 1
Suponhamos que seja possvel colocar uma corda
circundando a Terra, ajustando-a ao equador. Em seguida,
retiramos essa corda, aumentamos 1 m no seu
comprimento e a recolocamos em volta da Terra, formando
uma circunferncia concntrica com o equador. Sabendo
que o raio da Terra aproximadamente igual a
6 355 000 m, teramos substitudo uma corda de
aproximadamente 2 x 3,14 x 6 355 000 m = 39 909 400 m
por uma de 39 909 401 m. Assim, teremos um vo entre
o equador e a corda, ou melhor, uma diferena d entre os
raios das duas circunferncias.
Ento, perguntamos: usando-se somente a intuio, qual
o valor aproximado de d? Ou seja, qual a largura
aproximada desse vo entre o equador e a corda? Cremos
que o leitor dir: no existe vo algum... desprezvel
essa diferena... Como a Terra to grande e s se
aumentou um metro na corda, claro que o vo muito
pequeno e, por conseguinte, desprezvel... Ser?
Soluo
Vamos calcular o valor de d:
2R 2RT = 1 ou
d = R RT = 1/2 0,16 m = 16 cm!
Notamos que d independente do raio, independente,
portanto, do comprimento da circunferncia. Que tal fazer
algumas experincias?
3

Problema 2
Passemos, agora, ao segundo exemplo: consideremos um crculo com
raio igual ao raio da Terra. Suponhamos ser possvel cobrir toda a superfcie
desse crculo por uma outra superfcie, modelvel, ajustada a ele. Retiramos,
em seguida, essa segunda superfcie, aumentamos sua rea de 1 m2 e a
remodelamos, at se transformar novamente num crculo, com rea 1 m2
maior. Em seguida, justapomos os dois discos de modo a obter dois crculos
concntricos. Assim, haver uma diferena D entre os raios dos dois
crculos. Perguntamos novamente: usando-se apenas a intuio, qual o
valor aproximado de D?
Cremos que o leitor, dessa vez, alertado pelo problema anterior, teria
maior cautela para emitir um juzo, baseado apenas em sua intuio. Deixamos
o clculo de D para o leitor que deve concluir que, agora, D depende do
raio e que decresce na medida em que o raio cresce.
Problema 3
Tome uma corda esticada, medindo 400 km, unindo dois pontos, A e
B, um em SP e outro no RJ. Tome outra corda com 1 m a mais do que a
anterior e fixe suas extremidades nos mesmos pontos A e B. Como ela
fica bamba, coloque uma estaca de modo a mant-la esticada. Considere a
estaca no
a) ponto mdio da corda.
b) ponto A correspondente a SP.
Qual a altura, h, dessa estaca? maior ou menor que 1 m?
a)
A
SP

b)
h

A
SP
4

200 000,0

200 000

B
RJ

400 001 - h
400 000

B
RJ

Soluo
a) No tringulo retngulo de hipotenusa medindo 400 001/2 m e cateto
maior medindo 400 000/2, temos, por Pitgoras:
(200 000,05)2 200 0002 = h2, logo,
h2 = (200 000,05 200 000)( 200 000,05 + 200 000), ou h 447 m. Ou
seja, a estaca da altura de um prdio de aproximadamente 127 andares!
b) Neste caso, o tringulo retngulo tem cateto maior medindo 400 000 m,
e a soma dos comprimentos da hipotenusa e do cateto menor, h, igual
a 400 001 m. Por Pitgoras:
400 0002 + h2 = (400 001 h)2 ou h = 0,999 m 1m!
Perplexos com os resultados?
Problema 4
Quantos quadrados so necessrios para cobrir o Brasil, supondo
o processo indicado na figura em que
o quadradinho inicial tem 1 cm de lado
e o quadrado externo tem lado igual a
4.500 km?
Antes de resolver, faa estimativas
do resultado e compare com os
palpites de seus colegas.
Soluo
1o quadrado: 1 cm de lado
3o quadrado: 2 cm de lado
5o quadrado: 4 cm de lado
...
...
(2n + 1)o quadrado: 2n cm de lado.
Por tentativas, verifica-se que 229 = 536 870 912 a primeira potncia
maior que 450 000 000 (4 500 km = 450 000 000 cm). Portanto, o
(2 x 29 +1)o = 59o quadrado j cobre o Brasil.
Podemos resolver o problema de modo mais formal, usando que os
lados de todos os quadrados:
5

1,

2 , 2, 2 2 , 4, 4

2 , etc.

formam uma progresso geomtrica de razo 2 logo, queremos


determinar o menor inteiro n tal que n 1 > x, sendo x tal que

( 2)
ou,

x = log

450 000 000

= 450 000 000

2(log 4, 5 + 8)
57, 5 e n = 59.
log 2

Portanto, o 58o quadrado no cobre o Brasil, mas o 59o, sim.


Este mesmo problema pode ser resolvido com hexgonos e pentgonos.
Que tal tentar?
Vejamos agora o que diz nossa intuio na lenda:
O jogo de xadrez
Segundo uma lenda antiga, o jogo de xadrez foi inventado na ndia, para
agradar a um soberano, como passatempo que o ajudasse a esquecer os
aborrecimentos que tivera com uma desastrada batalha. Encantado com o
invento, o soberano, rei Shirham, quis recompensar seu sdito Sissa Ben
Dahir, o inventor do xadrez. Shirham disse a Sissa que lhe fizesse um pedido,
que ele, rei Shirham, o atenderia prontamente. Sissa disse, simplesmente:
Bondoso rei, d-me ento um gro de trigo pela primeira casa do
tabuleiro, dois pela segunda casa, quatro (= 22) pela terceira, oito (= 23)
pela quarta, e assim por diante, at 263 gros de trigo pela ltima casa do
tabuleiro, isto , a 64a casa.
O rei achou esse pedido demasiado modesto e, sem dissimular seu
desgosto, disse a Sissa:
Meu amigo, tu me pedes to pouco, apenas um punhado de gros de
trigo. Eu desejava cumular-te de muitas riquezas: palcios, servos e tesouros
de ouro e prata.
Como Sissa insistisse em seu pedido original, o rei ordenou a seus
auxiliares e criados que tratassem de satisfaz-lo. O administrador do palcio
real mandou que um dos servos buscasse um balde de trigo e fizesse logo a
contagem. Um balde com cerca de 5 kg de trigo contm aproximadamente
6

115 000 gros (como o leitor pode verificar, fazendo, ele mesmo, a
contagem...); foi o suficiente para chegar 16a casa do tabuleiro, mas no
alm, pois
1 + 2 + 22 + 23 + . . . + 215 = 216 1 = 65 535*,
enquanto, para chegar 17a casa, seriam necessrios
1 + 2 + 22 + 23 + . . . + 216 = 217 1 = 131 071
gros de trigo. (Um fato interessante a observar: o nmero de gros de
trigo a colocar numa casa igual a todos os gros j colocados nas casas
precedentes mais 1. De fato, pelo penltimo clculo v-se que todos os
gros colocados at a 16a casa mais 1 216, que o nmero de gros
correspondentes 17a casa.)
Traga logo um saco inteiro (60 kg, aproximadamente 1 380 000 gros)
ordenou o administrador a um dos servos , depois voc leva de volta o
que sobrar. Ao mesmo tempo providenciou a vinda de mais uma dezena de
contadores de trigo para ajudar na tarefa, que se tornava mais e mais
trabalhosa.
O administrador, os servos e os contadores j haviam terminado com 10
sacos de trigo (= 10 x 60 x 23 000 = 13 800 000 de gros) e mal haviam
passado da 23a casa do tabuleiro, visto que
1 + 2 + 22 + 23 + ... + 222 = 223 1 = 8 388607
1 + 2 + 22 + 23 + ... + 223 = 224 1 = 16 777215.
A essa altura o rei foi notificado do que estava acontecendo e alertado
de que as reservas do celeiro real estavam sob sria ameaa. Insistindo,
porm, em atender ao pedido de seu sdito, ordenou que o trabalho
continuasse.

Estamos usando o seguinte resultado: dado um nmero q 1 e n um inteiro positivo


arbitrrio, seja S = 1 + q + q2 + q3 + . . . + qn, logo qS = q + q2 + q3 + q4 + . . . + qn+1.
Subtraindo a primeira igualdade da segunda, obtemos
qS S = q n +1 1, ou S =

q n +1 1
, que a frmula da soma usada, neste texto, para
q 1

q = 2 (frmula da soma dos termos de uma progresso geomtrica).


7

Mandou convocar mais servos e mais contadores; ao mesmo tempo,


mandou chamar os melhores calculistas do reino para uma avaliao do
problema. Esses vieram e, cientificados do que se passava, debruaram-se
nos clculos. Em menos de uma hora de trabalho, puderam esclarecer o rei
de que no havia trigo suficiente em seu reino para atender ao pedido de
Sissa. Mais do que isso, em todo o mundo conhecido na poca no havia
trigo suficiente para atender quele pedido!
No tempo em que isso aconteceu, pensava-se que o mundo fora criado
havia menos de 5 000 anos. Assim, os calculistas do rei puderam dizer-lhe
que nem mesmo toda a produo mundial de trigo, desde a criao do
mundo, seria suficiente para atender ao pedido de Sissa.
Vamos ver por qu.
O nmero de gros pedidos por Sissa:
1 + 2 + 22 + 23 + ... + 263 = 264 1 = 18 446 744 073 709 551 615,
valor obtido usando uma calculadora cientfica.
Como verificamos no incio, um balde de 5 kg de trigo contm 115 000
gros, logo 1 tonelada de trigo (200 baldes) contm 23 x 106 gros. A
produo mundial de trigo da ordem de 590 milhes de toneladas (Internet),
ou seja, 23 x 590 x 1012 gros. Ora, 264 1 dividido por esse nmero de
gros resulta aproximadamente 1360, isto , seriam necessrios 1360 anos
de produo mundial de trigo no nvel de hoje para atender ao pedido de
Sissa.
Incrvel, no ?!
Baseado nos artigos
Logaritmos um curso alternativo
Renato Fraenkel, RPM 4
Quando a intuio falha
Joel Faria de Abreu, RPM 8
De So Paulo ao Rio de Janeiro
com uma corda ideal
Geraldo G. Duarte Jr., RPM 22
Nmeros muito grandes
Geraldo vila, RPM 25

Eleies - preferncia transitiva?

Antes de qualquer eleio nacional importante, sempre


so feitas pesquisas, que a populao acompanha com
interesse, em inmeros setores da sociedade: empresas,
clubes, escolas, etc. Vou falar aqui de uma pesquisa feita
em uma escola, antes do primeiro turno de uma eleio
para presidente da Repblica.
A histria comeou quando ouvi um colega, professor
de Histria, conversando com os alunos de uma turma da
3a srie do ensino mdio. Todos eleitores, naturalmente.
Perguntava esse meu colega em quem eles votariam no
segundo turno, considerando as hipteses, que ele iria
apresentar, em relao aos trs cadidatos principais, que
chamarei aqui de A, B e C. Esse meu colega perguntou
ento para a turma em quem eles votariam se A e B
fossem para o segundo turno. E a maioria da turma votaria
em A. Em seguida ele perguntou em quem votariam se B
e C fossem para o segundo turno. E agora a maioria da
turma votaria em B. Dando-se por satisfeito, o professor
resolveu comear a aula, mas foi interpelado por um aluno,
que lhe perguntou se ele no iria propor a hiptese de A
e C irem para o segundo turno. Esse colega respondeu
que no havia necessidade dessa pergunta porque
naturalmente A ganharia de barbada.
A aula comeou e eu me retirei para pensar no caso
que agora relato. Na realidade, por incrvel que parea, o
professor estava errado. Ele no poderia concluir que a
maioria da turma preferiria A a C. Para mostrar que
esse raciocnio falso, imaginemos que num grupo de
pessoas a disputa entre A, B e C seja equilibrada da
9

seguinte forma: 1/3 das pessoas desse grupo


tem preferncia por A, B e C nessa ordem;
1/3 das pessoas tem preferncia por B, C e A
nessa ordem, e o restante por C, A e B nessa
ordem.

1/3
1/3
1/3

1
A
B
C

2
B
C
A

3
C
A
B

Se esse grupo for submetido s perguntas feitas pelo meu caro colega,
veremos que, na deciso entre A e B, 2/3 preferiro A; tendo que optar
entre B e C, 2/3 preferiro B; mas, surpreendentemente, se a deciso
for entre A e C, 2/3 preferiro C! O aluno estava, portanto, certo e a
terceira pergunta deveria ter sido feita.
Temos aqui um exemplo de uma relao que intuitivamente esperamos
ser transitiva, mas que, na realidade, no . Divagando um pouco, essa notransitividade da relao preferir pode ter espantado algum dia um
cozinheiro de restaurante que s sabia fazer trs pratos: um peixe, uma
galinha e uma carne, mas, como nunca tinha tempo de fazer os trs, sempre
oferecia dois deles. perfeitamente possvel que, quando havia peixe e
galinha, a maioria dos fregueses preferisse peixe. No dia em que havia
galinha e carne, a maioria preferisse galinha e que no dia em que havia
peixe e carne a maioria preferisse carne! Isso pode ocorrer mesmo que os
fregueses sejam sempre os mesmos. natural.
Para dar um outro exemplo (as mulheres agora me perdoem), diria que
o espanto do cozinheiro pode ser comparado ao da moa que recebeu pedido
de casamento de trs pessoas A, B e C. Essa moa, que desejava fazer
o melhor casamento possvel (na opinio dela, naturalmente), dava
importncia igualmente a trs coisas que os candidatos deveriam ter: cultura,
beleza e situao financeira.
Para melhor avaliar os pretendentes, ela
resolveu dar notas a esses quesitos para
cada um deles. Nota 3 significando bom;
nota 2 significando mdio e nota 1 para
ruim. Os resultados esto no quadro:

A
B
C

cultura beleza finanas


2
3
1
3
2
1
3
1
2

Veja ento que, apesar de haver um empate tcnico, se os candidatos


fossem comparados aos pares, ela iria preferir A a B porque A vence em
dois dos trs quesitos; iria preferir B a C pela mesma razo e ainda iria
preferir C a A. Incrvel, no?
Baseado no artigo Eleies
Eduardo Wagner, RPM 16
10

A divisibilidade e o dgito verificador

Introduo
Recentemente fui obrigado a solicitar uma segunda via
do meu documento de identidade e, para minha surpresa,
acrescentaram um dgito ao final do meu antigo nmero
de registro geral (RG). Na ocasio, fiquei curioso: quais
as razes desse dgito adicional? Esclarecimento que s
recentemente obtive e que compartilho com o leitor neste
artigo.
Sistemas de informao e a segurana
na transmisso de dados
Por mais cuidadoso que seja o digitador, erros podem
ocorrer e suas conseqncias podem ser muito srias.
preciso, ento, criar mecanismos para detectar o maior
nmero possvel de tais erros.
Pesquisas recentes sobre a natureza dos erros de
digitao revelam um fato curioso. Cerca de 79% dos
erros ocorrem com a digitao equivocada de um nico
dgito (ou algarismo), como, por exemplo, digitar 1 573,
quando o correto seria 1 673.
Esse tipo de erro recebe o nome de erro singular.
Outros 11% dos erros, chamados de erros de
transposio, referem-se troca de dois dgitos (ou
algarismos), como, por exemplo, escrever
MTAEMTICA, quando o correto seria MATEMTICA.
Esses so chamados de erros de transposio. Os demais
11

10% dos erros esto distribudos em diversas categorias, nenhuma delas


representando mais de 1% do total.
bom que fique claro que existem particularidades em cada sistema de
cdigos, ou at mesmo em cada idioma, que podem mudar significativamente
essa distribuio de probabilidades. Apenas para citar um exemplo, na Sucia
os nmeros de identificao de cada cidado so constitudos por 6
algarismos para a data de nascimento (ano/ms/dia), seguidos de 3
algarismos para dar conta de duplicaes de datas coincidentes. Muitas
pessoas, no entanto, ao digitar, permutam os algarismos do ano com os do
dia, criando um erro muito freqente, que no singular nem de transposio
(trata-se aqui de um erro de trocas duplas).
Sabendo-se que nos dias de hoje cada vez mais usamos os computadores
para armazenar e processar as informaes digitadas, seria possvel criar
um sistema que pudesse identificar com 100% de segurana um erro de
digitao do tipo singular ou de transposio? Um tal sistema daria conta de
evitar cerca de 90% dos erros mais freqentes de digitao.
A divisibilidade e uma soluo do problema
O sistema ISBN (International Standard Book Number), criado em 1969
para a identificao numrica de livros, CD-Roms e publicaes em braille,
talvez seja um dos pioneiros na utilizao de um dgito de verificao ao
final de cada cdigo, capaz de resolver o problema dos erros singulares e
de transposio. Por exemplo, o cdigo ISBN 97-26-62792-3 refere-se ao
livro O mistrio do bilhete de identidade e outras histrias (Editora
Gradiva, Lisboa, 2001). Com exceo do ltimo dgito da direita, que o
dgito verificador (DV) (ou dgito de controle, como conhecido em
Portugal), os demais 9 dgitos so responsveis por identificar o pas de
origem da obra, a editora e o livro propriamente dito.
Os equipamentos que recebem a digitao de um cdigo ISBN, x1 x2 x3
x4 x5 x6 x7 x8 x9 e seu dgito de verificao x10, esto programados para
verificar se o resultado, S, da expresso
10 x1 + 9 x2 + 8 x3 + ... + 2 x9 + 1 x10
divisvel por 11 ou no: o algarismo de verificao x10 escolhido de tal
forma que o resultado dessa conta tenha sempre resto zero na diviso por
11. Veja, no exemplo do livro acima, que
12

10 9 + 9 7 + 8 2 + 7 6 + 6 6 + 5 2 + 4 7 + 3 9 + 2 2 + 1 3
igual a 319, que divisvel por 11.
Podemos demonstrar um importante resultado com relao a esse
sistema:
Resultado
Se ocorrer na leitura de um cdigo ISBN um, e apenas um, dos dois
erros (singular ou de transposio), ento a soma S no ser um
mltiplo de 11.
Demonstrao
Caso 1: Quando ocorre um erro singular.
Seja x1 ... xi ... xj ... x10 um cdigo ISBN com dgito de verificao x10
*
e x1 K xi K x10 o resultado da ocorrncia de um erro singular na i-sima
posio. Chamemos de S e S* as somas correta e errada, respectivamente.
Temos, evidentemente, que S divisvel por 11 e

S * S = (11 i )( xi* xi ) 0.

Se admitirmos por hiptese que S* seja mltiplo de 11, ento, como 11


primo, conclumos que 11 divide 11 i ou divide xi* xi , o que um
absurdo, pois 11 i e xi* xi so nmeros inteiros no nulos entre 10
e 10. Logo, S* no mltiplo de 11, o que acusa o erro cometido.
Caso 2: Quando ocorre um erro de transposio.
Seja x1 ... xi ... xj ... x10 um cdigo ISBN, x10 o dgito de verificao e
x1 ... xi ... xj ... x10 o resultado da ocorrncia de uma transposio dos
algarismos xi e xj nas posies i e j (i j). Nesse caso, a diferena
S* S igual a
(11 i)xj + (11 j)xi (11 i)xi (11 j)xj = (j i)(xj xi) 0.
A hiptese de S* ser mltiplo de 11 mais uma vez absurda porque
nos conduziria concluso de que um dos nmeros j i ou xj xi, que
so nmeros inteiros no nulos entre 10 e 10, mltiplo de 11. Segue
que S* no pode ser mltiplo de 11.
Se agora admitirmos que na digitao de um cdigo ISBN s ocorrem
erros singulares ou de transposio, no mais do que um erro em cada
13

nmero, ento no ocorrem erros na digitao de um cdigo ISBN se e


somente se a soma S for um mltiplo de 11. bom lembrar que, ao
digitarmos um cdigo ISBN cometendo um erro singular ou de transposio,
o equipamento que recebe os dados ser capaz apenas de acusar a existncia
de um erro devido ao fato de S no ser divisvel por 11, mas no ser
capaz de encontr-lo; o que implica dizer que o digitador tem ainda como
tarefa procurar o erro cometido.
O dgito de verificao do RG
Para o Estado de So Paulo e muitos outros Estados brasileiros, o dgito
de verificao do RG calculado da seguinte maneira:
Seja x1x2x3x4x5x6x7x8x9 o RG de um indivduo. O dgito de verificao,
x , calculado de modo que a soma
10

100 x10 + 9 x9 + 8 x8 + 7 x7 + ... + 2 x2 + 1 x1


seja divisvel por 11. Como normalmente se reserva apenas um algarismo
para o dgito de verificao, que, neste caso, um inteiro entre 0 e 10 (os
restos possveis na diviso de um inteiro por 11), normalmente se usa a
letra X para representar o dgito de verificao 10. Por exemplo, no RG
nmero 25 135 622 X, verifique que
100 10 + 9 2 + 8 2 + 7 6 + 6 5 + 5 3 + 4 1 + 3 5 + 2 2 + 1 0
divisvel por 11.
Observa-se que os raciocnios utilizados na demonstrao do Resultado
anterior, aplicam-se quase totalmente nova expresso aqui utilizada. Com
efeito, na ocorrncia de um erro singular no dgito xi na digitao de um tal
RG, tem-se S* S = i (xi* xi) para i = 1, 2, ..., 9 e se i = 10, S* S =
100 (x10* x10), que no podem ser mltiplos de 11 para xi* xi 0, i =
1, 2, ..., 10. Na ocorrncia de um erro de transposio entre xi e xj, com
1 i < j 9, tem-se S* S = (j i)(xi xj), que no divisvel por 11, se
xi xj 0. No caso, entretanto, em que a transposio se d entre xi e x10,
S* S = (100 i) (xi x10), que um mltiplo de 11 se i = 1, mesmo que
x1 x10 no seja nulo. Isso no tem efeito prtico negativo, pois erros de
transposio de alta probabilidade so aqueles entre dgitos consecutivos. A
troca, portanto, entre o primeiro e ltimo dgitos no nada comum.
J em Portugal, onde o algoritmo de verificao dos documentos de
identificao igual ao nosso, com a diferena de que l se utiliza peso 10
14

no dgito de verificao em vez de peso 100, esse problema no se d. Os


responsveis pela execuo do sistema decidiram, porm, no utilizar a
letra X para o dgito de verificao 10, optando pelo uso do zero para
represent-lo. curioso notar, no caso portugus, onde um dgito de
verificao 0 pode significar o nmero zero ou o nmero dez, que a
concepo do sistema de deteco de erros singulares e de transposio
est comprometida para os portadores de documentos de identificao com
dgito de verificao igual a 0 ou 10.
Ficaria a questo: para que o dgito verificador utilize uma s posio,
por que no usar a divisibilidade por 10 (cujos restos possveis so s
0, 1, ... , 9), em vez de 11? O argumento na prova da proposio mostra
que foi essencial que 11 fosse primo e maior que 10.
bom notar ainda que o sistema brasileiro tambm no uniforme.
Recentemente descobri que o dgito de verificao do RG, emitido no Rio
Grande do Sul, de um amigo gacho, no segue o mesmo algoritmo vlido
para So Paulo e muitos outros Estados.
Baseado no artigo
Aritmtica modular e sistemas de identificao
Jos Luiz Pastore Mello, RPM 48

15

O tamanho da Terra

O raio da Terra aproximadamente 6 400 km..., mas


como que se mede o raio da Terra?
Um grande sbio da Antiguidade, Eratstenes, calculou
o raio da Terra h mais de 2 200 anos! Mais do que isso,
os sbios daquela poca calcularam tambm as distncias
da Terra Lua e da Terra ao Sol, e os tamanhos desses
astros; e para isso utilizaram noes bsicas de
semelhana e proporcionalidade.
Eratstenes viveu no terceiro
sculo a.C., na cidade de Alexandria,
que fica no extremo oeste do delta
do rio Nilo. Mais ao sul, onde hoje se
localiza a grande represa de Assu,
ficava a cidade de Siena, como ilustra
o mapa. Naquela poca deveria
haver um trfego regular de
caravanas entre as duas cidades; e,
talvez por causa desse trfego, sabiase que a distncia entre Alexandria e Siena era de
aproximadamente 5000 estdios, ou seja, 800 km (tomando
o estdio como igual a 160 metros).
Decerto os viajantes experientes j haviam feito uma
boa estimativa dessa distncia. Quem viaja com freqncia
por anos a fio sabe calcular as distncias percorridas, muito
provavelmente pelo nmero de dias gastos na viagem e
16

pelo que se consegue percorrer numa jornada. E, uma vez conhecida a


distncia ao longo das estradas, seria possvel fazer uma estimativa da
distncia em linha reta.
Outra coisa que se sabia que as duas cidades estavam mais ou menos
no mesmo meridiano, ou seja, tinham a mesma longitude. Isso intrigante,
pois, enquanto seja relativamente fcil fazer uma estimativa da latitude de
um lugar, a comparao das longitudes de dois lugares diferentes um
problema bem mais complicado. Decerto eles achavam que as duas cidades
estavam no mesmo meridiano porque para ir de Alexandria a Siena viajavase diretamente na direo sul.
O que fez Eratstenes
Alm desses dois fatos a distncia de 800 km entre as duas cidades e
elas estarem no mesmo meridiano1 , dois outros fatos foram cruciais no
raciocnio de Eratstenes: devido grande distncia que o Sol se encontra
da Terra, os raios solares que chegam ao nosso planeta so praticamente
paralelos; e quando os raios solares caam verticalmente ao meio-dia em
Siena (o que era comprovado vendo que as cisternas ficavam totalmente
iluminadas ao meio-dia e o disco solar podia ser visto refletido no fundo
dessas cisternas),2 em Alexandria eles formavam, com a vertical do lugar,
um ngulo igual a 1/50 da circunferncia completa. Com a medida em graus,
isso equivale a dizer que esse ngulo era de 7,2.
Plo Norte
A

7,2

raios solares

7,2

Isso s verdade aproximadamente, tanto no que se refere distncia entre as duas


cidades, quanto igualdade das longitudes. Veja Alexandria e Assu num bom mapa do
Egito: Assu, a antiga Siena, fica s margens do lago Nasser, pouco mais de 3o a leste de
Alexandria.
17

Veja:

7, 2
72
2
1
=
=
= .
360 36 100 100 50
Nesse ponto entra o raciocnio de Eratstenes: se a 1/50 de ngulo
correspondem 800 km de arco, ao ngulo de 360 corresponder 50 x 800 =
40 000 km.
Que Matemtica foi usada?
Vamos rever o raciocnio de Eratstenes para identificar os fatos
matemticos usados. Ele entendeu que o ngulo de 7,2 em Alexandria
(A na figura anterior) igual ao ngulo central em O, o que pressupe que
os raios solares que chegam Terra so paralelos, devido grande distncia
do Sol3. Portanto, a igualdade dos ngulos em O e A devida ao fato de
eles serem ngulos correspondentes em duas paralelas (AB e OS) cortadas
pela transversal OA. O outro fato matemtico utilizado o da
proporcionalidade entre arcos e ngulos: os ngulos centrais so proporcionais
aos arcos que subentendem; assim, o ngulo de 7,2 est para o arco AS,
assim como 360 est para a circunferncia completa.
Ser que foi isso mesmo?
Sim, ser que Eratstenes mediu mesmo o ngulo de incidncia dos
raios solares? Para isso ele teria de se valer de algum aparelho, e teria de
realizar uma operao meio sofisticada, difcil de ser feita com preciso.
Parece que ele procedeu de maneira muito mais simples. Em Alexandria
certamente havia um relgio solar, com uma coluna construda bem na
vertical, cujas sombras projetadas serviam para marcar a hora do dia. Ele
decerto esperou o dia do ano em que se sabia que os raios solares incidiam
verticalmente em Siena ao meio-dia; e, nesse instante, mediu o comprimento
da sombra projetada pela coluna do relgio solar em Alexandria.

Isso tambm s verdade aproximadamente; hoje sabemos que a antiga Siena ficava uns
60 km ao norte do Trpico de Cncer, que o paralelo de maior afastamento norte do Sol
em relao ao equador.

No tempo de Eratstenes j era sabido que o Sol se encontrava a uma imensa distncia da
Terra.

18

C
7,2

7,2

De posse do comprimento dessa sombra (AB na figura) e da altura BC


da coluna, ele teria desenhado um tringulo retngulo ABC (numa folha
de papiro, com certeza), com lados AB e BC proporcionais aos lados
AB e BC, respectivamente, do tringulo ABC, que tambm retngulo
em B (veja as figuras). Seria agora relativamente fcil medir o ngulo de
incidncia, ou seja, o ngulo ACB do tringulo ABC da figura.
Eratstenes teria verificado que esse ngulo era de 1/50 da circunferncia
completa, ou seja, 7,2.
A igualdade do ngulo de incidncia em A com o ngulo ACB decorre
de esses ngulos serem alternos internos; e a igualdade dos ngulos ACB
e ACB devida semelhana dos tringulos ACB e ACB.
O raio da Terra
Da circunferncia terrestre podemos passar ao raio da Terra sem
necessidade de novas medies.
No caso da Terra, como C = 400 000 km e lembrando que C = 2r,
calcula-se r = C 2 6370 km, usando para a aproximao 3,14.
Eratstenes, Ptolomeu e Cristvo Colombo
J dissemos que Eratstenes viveu no sculo terceiro a.C.,
provavelmente entre 276 e 196 a.C., dizem os historiadores mais abalizados.
Portanto, era pouco mais jovem que Arquimedes (287-212 a.C.). Ele no
foi o primeiro a se preocupar com a medida do tamanho da Terra. Aristteles
(384-322) e Arquimedes fazem referncias a outras estimativas e citam
valores do tamanho da Terra. Mas eles no explicam de onde provm suas
informaes, por isso mesmo esses eventuais clculos anteriores a
Eratstenes no so levados em conta.
19

O clculo do tamanho da Terra aparece num livro de Ptolomeu sobre


Geografia, livro esse que foi muito usado no tempo das grandes navegaes.
Por razes no bem esclarecidas at hoje, ou Ptolomeu valeu-se de um
clculo do raio terrestre diferente do que fez Eratstenes, ou registrou um
estdio de outro comprimento que o do tempo de Eratstenes4.
Seja como for, em sua Geografia, Ptolomeu utiliza um valor do raio da
Terra que est abaixo do valor fornecido por Eratstenes. E apresenta um
mapa do mundo ento conhecido, o qual contm mais dois erros importantes:
a largura leste-oeste do mar Mediterrneo est exageradamente alta, bem
como a largura leste-oeste da sia. Em conseqncia desses trs erros, a
distncia do Oeste europeu (Espanha, Portugal) ao Leste asitico (Japo,
Coria), para quem navegasse pelo oceano Atlntico em direo oeste,
seria bem mais curta do que realmente . Cristvo Colombo valeu-se disso
para convencer os reis de Espanha de que sua viagem s ndias seria vivel5.
Sua sorte foi estar errado em pensar que no havia terra em seu caminho,
pois, fosse isso verdade, ele teria perecido.
Baseado no artigo Se eu fosse professor de Matemtica
Geraldo vila, RPM 54

Cabe notar tambm que no h acordo sobre o valor exato do estdio em metros.

interessante notar que razes de ordem tcnica ao menos em parte levaram Portugal
a no aprovar a proposta de Colombo. Com efeito, os especialistas encarregados de julgar
essa proposta constataram corretamente que a distncia a ser percorrida na viagem seria
muito mais longa do que Colombo previa, sendo impossvel levar vveres e gua em
quantidades suficientes para toda a viagem.

20

Problema das idades

Tenho o triplo da idade que tu tinhas quando eu tinha a


idade que tu tens. Quando tu tiveres a idade que eu tenho,
teremos juntos 56 anos. Qual a minha idade?
Esse problema, com enunciado em estilo de uma
charada, est hoje meio fora de moda, mas foi clebre
numa poca em que havia uma preocupao de resolver
esse e outros tipos de problemas por Aritmtica e no
por lgebra.
Vamos abordar o problema geometricamente. Se
representarmos graficamente, num sistema de
coordenadas cartesianas, a evoluo da idade de um
indivduo atravs do tempo, obteremos sempre uma reta
paralela bissetriz do primeiro quadrante.

Na realidade, obteremos a prpria bissetriz se tomarmos


o ano zero como sendo o ano de seu nascimento, pois
no ano 1 ele ter 1 ano, e assim sucessivamente (isso
um fato do qual a experincia j mostrou que podemos
convencer mesmo uma pessoa que
jamais estudou Geometria Analtica).
J a idade de uma pessoa d anos mais
velha ter como grfico uma reta
paralela, j que a diferena entre as
idades dos dois permanecer constante
e igual a d.

t
21

Voltemos ento ao nosso problema. H dois indivduos em causa, um


que fala, chamamo-lo de E, e um que escuta, T. Evidentemente E mais
velho que T (... quando eu tinha a idade que tu tens...), digamos, d anos,
de modo que seus grficos de idades se assemelham aos da figura da pgina
anterior.
H trs pocas mencionadas no
problema, que chamaremos P (passada), A
(atual) e F (futura). A maneira como se
relacionam A e P (... quando eu tinha a idade
que tu tens...) e a maneira como se
relacionam A e F (... quando tu tiveres a
idade que eu tenho...) mostram que elas se
situam no grfico como nos casos da figura
ao lado.

X
Y
Z
0

E
T

P A F

A inclinao de 45o das retas desenhadas acarreta que todos os


segmentos verticais compreendidos entre elas tm comprimento d.
O dado de que a idade que E tem na
poca A (isto , OX) o triplo da idade
que T tinha na poca P (isto , OZ),
juntamente com o fato de XY = YZ = d,
obriga a que OZ seja tambm d (estou
evitando escrever a equao 2d + OZ =
3OZ, j que isso pode ser visto na figura).
Mas, ento, a reta grfica da idade de E
tem que passar por Z e a figura correta
a que est ao lado.

E
T

X
d

P A F

Agora ento claro que, na poca F, a idade de T 3d enquanto a


de E 3d + d. Logo os dois juntos tm 7d, que deve ser 56. Logo, d
tem que ser 8 e a idade que eu tenho 3 x 8 = 24, que a resposta.

Baseado no artigo
Uma soluo geomtrica para
o problema das idades
Jos Paulo Q. Carneiro, RPM 16

22

A ilha dos sapatos gratuitos

Cena no 1 O problema
Um dia, estava eu na faculdade tranqilamente
pensando na vida quando chegou um colega e me fez uma
proposta inusitada:
Voc quer comprar de graa (?!) um sapato?
claro que eu topei de cara comprar de graa (?!) um
sapato, embora desconfiasse que houvesse algum rolo.
As condies eram:
1. Comprar um selinho desse meu amigo. Preo R$ 3,00;
2. Juntar mais R$ 27,00 e o selinho e levar a uma
determinada loja. Eu receberia um par de sapatos com
valor de mercado de R$ 30,00 e mais dez selinhos no
valor de R$ 3,00;
3. Vender os dez selinhos que eu seria restitudo dos
R$ 3,00 iniciais de compra do selinho do meu amigo e
dos R$ 27,00 que anexei para retirar o sapato da loja.
Dei R$ 3,00 ao meu colega pelo selo, fui loja, retirei
um par de sapatos por R$ 27,00 e ganhei os dez selinhos
que me iriam restituir tudo o que investira.
Vendi os dez selinhos com alguma facilidade. Fiz ento
um balano: eu tinha at ento gasto R$ 30,00, recebido
R$ 30,00 e mais um par de sapatos. Um par de sapatos de
graa, portanto. Como isso seria possvel? No estaria
essa promoo violando a Lei de Lavoisier ou a Segunda
Lei da Termodinmica? Fiquei estarrecido com o problema.
Como interpret-lo?
23

Cena no 2 As explicaes convencionais


Aturdido com o problema que aparentemente violava leis naturais nunca
dantes questionadas, sa a conversar com meus colegas de faculdade. O
primeiro a tentar responder foi Altarimando. Ele se entusiasmou.
No se preocupe se essa promoo fere ou no as leis da natureza. O
importante que funciona. Assim como voc conseguiu comprar sapatos
de graa, vamos expandir o negcio para comprar arroz de graa, roupa de
graa, etc. Talvez esse seja o perdido caminho para a humanidade alcanar
o Nirvana, o to desejado Shangril. No se esquea de que as Leis de
Mercado so superiores Lei de Lavoisier.
Desconfiei que ele estava mais para poeta transcendental que crtico de
Matemtica e Fsica e fui procurar o Souzinha, um crtico de tudo. Logo
deu seu parecer, claro e taxativo, incisivo e demolidor, caracterstico de
todo jovem de menos de quarenta anos:
Estamos diante da chamada Bola-de-neve, Conto da venda sucessiva
ou ainda da Corrente da felicidade. um estratagema que favorece
barbaramente os compradores iniciais e altamente desvantajoso para os
finais. O universo possvel de compradores um nmero finito e os
compradores dos selinhos so: 1 na primeira etapa, 10 na segunda, 100 na
terceira, etc. Ou seja, os envolvidos na corrente so em nmero de
100 + 101 + 102 + 103+ ...
Quando o somatrio excede o nmero de possveis compradores, a
corrente pra e os ltimos no tero para quem vender os selos, sendo
prejudicados.
Logo, essa artimanha to simplesmente uma falcia. Continuam vlidos,
portanto, a Lei de Lavoisier e o Segundo Princpio da Termodinmica.
Fiquei feliz, confesso, por essa explicao do Souzinha.
As pessoas como ns, que estudam Matemtica, com a mente criada e
disciplinada por critrios lgico-formais cartesianos tm verdadeiro horror
a situaes que fujam desse modo e, o que pior, funcionem. Se isso pudesse
ocorrer, ficaramos inseguros, e toda uma vida ficaria questionada.
Cena no 3 A explicao diferente
Quando eu j estava disposto a encerrar o assunto, encontrei um velho
amigo, Ado, estudante de Economia na Getlio Vargas.
24

Apesar de jovem, Ado crtico ponderado e profundo em seus


conhecimentos.
S como curiosidade, expus a ele o problema e as duas respostas que eu
tinha ouvido at ento.
Ado, filosoficamente, comeou a raciocinar socraticamente.
Quanto mesmo que a loja recebe por par de sapatos vendido?
Ora, Ado, respondi, o enunciado claro. Ela recebe R$ 30,00 por par
de sapatos.
Acho que a temos uma pista, no esse o valor, ponderou Ado. E
continuou:
Admitamos uma ilha com 1 111 pessoas potencialmente clientes dos
sapatos e mais uma pessoa, que o dono da loja, totalizando 1 112 pessoas.
O dono da loja prope o negcio a um primeiro cliente. Compre um selo por
R$ 3,00, adicione R$ 27,00 e deflagre o processo. Esse primeiro cliente
vende dez selos. Dez compradores vendem depois para 100 outros
compradores. J so 111 compradores. Os cem compradores vendem agora
para 1 000 compradores. Esses ltimos 1000 compradores, que j gastaram,
cada um, R$ 3,00 pelo selo, no tm mais para quem vender. Uma de suas
opes perder esse selo. Outra (mais razovel) acrescentar R$ 27,00 e
ir buscar o seu par de sapatos, que, como sabemos, vale no mercado
R$ 30,00. Logo, esses ltimos compradores no sero prejudicados
financeiramente (s no tero os seus sonhos de sapatos grtis).
Agora faamos um raciocnio. Quanto recebeu a loja de sapatos e quantos
pares de sapatos foram entregues? Curiosamente voc ver que a loja no
recebe R$ 30,00 por par de sapatos vendido.
A loja recebeu em dinheiro:
do 1o comprador:
de 10 compradores:
de 100 compradores:
de 1000 compradores:
Total

3,00 + 27,00
10 x 27,00
100 x 27,00
1000 x 27,00
R$ 30 000,00

=
30,00
=
270,00
= 2 700,00
= 27 000,00

Total de pares de sapatos vendidos = 1111


Receita mdia da loja por par de sapatos: R$ 30 000,00/1111 R$ 27,03
25

Concluso
A loja vende cada par de sapatos a R$ 30,00 e recebe na prtica R$ 27,00
e no R$ 30,00, como supostamente se poderia pensar. V-se, portanto,
que cada pessoa para ganhar um par de sapatos precisa entregar o sinal
(entrada) e ter o trabalho de vender dez outros sapatos. O caso em estudo
um processo que traz embutido um trabalho de venda como custo. Custo
esse que pago pela loja (30,00 27,03) = 2,97 por par de sapatos. uma
comisso de venda. Tudo claro, Botelho?
Fiquei a pensar. Como as coisas ainda estavam algo confusas dentro de
mim, pedi apoio Revista do Professor de Matemtica.
A resposta da RPM
1. Se a histria se passasse no instante em que nosso amigo Botelho acabou
de vender seus dez selinhos, o que estaria acontecendo que dez pessoas
(os compradores dos selinhos) teriam se cotizado para comprar um par
de sapatos para ele.
2. Na histria, nada obriga que cada comprador se limite a adquirir um par
de sapatos apenas. Para citar um caso extremo, podemos supor que o
primeiro comprador, em vez de vender os 10 selinhos que recebeu da
loja, fica com eles e com isso compra mais dez pares de sapatos a
R$ 27,00 cada, recebe 100 selinhos, etc., at acabar com o estoque da
loja. Depois, revende todos os sapatos ao preo oficial de R$ 30,00. Em
vez de um par de sapatos de graa, ganha muito mais.
3. Do ponto de vista da loja, o que ela fez corresponde simplesmente a
vender cada par de sapatos a R$ 27,00, exceto o primeiro, vendido por
R$ 30,00. Os selinhos so apenas um truque de marketing. A loja vende
por R$ 27,00, mas, como o preo usual R$ 30,00, a diferena dividida
entre alguns felizardos, ou espertos. O exemplo do economista Ado,
em que cada habitante da ilha compra apenas um par de sapatos, o
extremo oposto do caso 2 acima. Na prtica ocorrem, em geral, situaes
intermedirias em que algumas pessoas formam estoque para revenda
(podendo em seguida organizar cartis para manipular os preos, mas
isso j seria outra histria).
Baseado no artigo Na ilha dos sapatos gratuitos
Manoel Henrique C. Botelho, RPM 7

26

Fraes egpcias

Quando se menciona Fibonacci, ou seja, Leonardo


Fibonacci (1170, 1240?), tambm conhecido como
Leonardo Pisano ou Leonardo de Pisa, pensa-se logo no
clebre problema dos coelhos, apresentado e resolvido no
seu Liber Abaci, conduzindo clebre seqncia 1, 1,
2, 3, 5, 13, ..., que at hoje leva seu nome. Mas o livro
contm muito mais: entre os problemas nele tratados, a
maioria sem grande interesse para ns, leitores de hoje,
pois tratam de Aritmtica usando os algarismos indoarbicos ou de Matemtica Comercial, encontramos
verdadeiras jias matemticas, como um relacionado com
a maneira egpcia de lidar com fraes.
Como sabemos, os egpcios s trabalhavam com
fraes unitrias, isto , da forma 1/n, sendo n um
nmero natural [ exceo de 2/3 e, s vezes, das fraes
da forma n/(n + 1)]. Obviamente, em seus problemas
matemticos apareciam fraes da forma m/n, que
deviam ento ser escritas usando-se somente fraes
unitrias distintas. Ou seja, era necessrio escrever
m 1 1
1
= +
+L +
n n1 n2
nk , com n1, n2, ..., nk naturais

distintos.
No discutiremos aqui as interpretaes apresentadas
pelos eruditos para essa insistncia egpcia em trabalhar
com fraes unitrias. Esse hbito, embora pesado e
inconveniente, sobreviveu at a Idade Mdia. Em verdade,
os egpcios, por meio de tabelas apropriadas e mtodos
27

engenhosos, conseguiam lidar muito bem com as fraes unitrias. O leitor


mais curioso poder consultar o livro Mathematics in the Time of the
Pharaohs de autoria de R. J. Gillings, Dover, 1982, ou, para uma leitura
leve, a RPM 15, p. 21.
No bvio que qualquer nmero racional m/n, com m < n, possa ser
escrito como soma de fraes unitrias. Uma prova da acuidade matemtica
de Fibonacci ter percebido a necessidade de mostrar isso. Ele no apresenta
uma demonstrao formal, como o faramos hoje, mas d um mtodo
inteiramente geral que resolve o problema.
A regra ... que voc divide o nmero maior pelo menor; e
quando a diviso no exata, verifique entre que dois naturais
a diviso est. Tome a maior parte, subtraia-a, e conserve o
resto ...
Em linguagem de hoje, a regra seria:
Subtraia da frao dada a maior frao unitria que no
maior do que ela. Repita o processo at obter 0.
Por exemplo, escrevamos a frao 4/13 como soma de fraes unitrias
distintas:
3 < 13/4 < 4 1/3 > 4/13 > 1/4
Portanto, 4/13 1/4 = 3/52.
Mas, ento, 17< 52/3 < 18 1/17 > 3/52 > 1/18.
Logo, 3/52 1/18 = 2/936 = 1/468. Aqui, a diviso de 936 por 2 exata,
e o processo termina.
Assim, 4/13 = 1/4 + 1/18 + 1/468.
No difcil demonstrar que o processo descrito por Fibonacci sempre
funciona. Para mostrar que o mtodo funciona, demonstraremos que os
numeradores das diferenas sucessivas (mesmo antes de simplificar)
decrescem estritamente (no exemplo acima, as diferenas so 3/52 e
2/936). Ento, como toda sucesso estritamente decrescente de nmeros
naturais no negativos finita (veja O princpio da descida infinita de
Fermat, RPM 32), o processo obrigatoriamente tem fim.
a
com a < b.
Com efeito, consideremos a frao
b
28

a 1
Suponha que b = qa + r, 0 r < a. Se r = 0, ento, b = q e a
demonstrao est terminada. Podemos, portanto, supor que r 0.
Ento,
Assim,

b
r
=q+
a
a

implicando q <

b
< q + 1, ou
a

1 a
1
> >
.
q b q +1

a
1
r + a
.

=
b q + 1 b(q + 1)

Mas, como a r < a, os numeradores das diferenas sucessivas so


estritamente decrescentes quando r 0, o que queramos demonstrar.
Baseado no artigo
Um problema de Fibonacci
Joo Pitombeira de Carvalho, RPM 17

29

As dzimas peridicas
e a calculadora

Em uma prova de concurso, destinado principalmente


a professores de Matemtica, figurava a seguinte questo:
Os nmeros racionais a e b so representados, no
sistema decimal, pelas dzimas peridicas

a = 3, 0181818... = 3, 018 e b = 1,148148... = 1,148


Encontre, justificando, uma representao decimal
de a b.
Como a e b so racionais, tambm o a b; e,
portanto, sua representao decimal peridica. Na prova,
era permitido o uso de calculadora. Mas por meio da
calculadora jamais se descobrir o perodo, pelo menos
com a certeza exigida pelo justifique. Alm disso, a
calculadora no conseguir nem mesmo dar uma idia do
perodo, se ele for muito longo. De fato, o perodo pode
ter um comprimento maior do que o nmero de dgitos
que a calculadora exibe no visor.
Um primeiro expediente que poderia ocorrer seria fazer
a subtrao por meio do esquema usado habitualmente
para decimais finitos. Isso funcionaria bem em casos mais
simples. Por exemplo:
0, 444...
0, 333...
0,111...
o que estaria correto, pois
30

4 3 1
= .
9 9 9

Mas, no caso em questo, o desencontro entre os perodos das duas


dzimas apresentadas dificulta o emprego dessa estratgia (a qual, alis,
precisaria ser discutida em termos conceituais). Vejamos:
Como a subtrao usual feita da direita para a esquerda, no se sabe
bem por onde comear, antes de descobrir o perodo.
Por conseguinte, o caminho natural calcular as geratrizes de a e b,
subtrair as fraes correspondentes, e ento encontrar uma representao
decimal para essa frao.
Utilizando esse procedimento, encontra-se:

a b = 3+

18 148
1292 2777
1 +
=
.
=1+
990 999
1485 1485

Neste ponto, o mtodo mais usado por todo mundo dividir 2777 por
1485 (ou 1292 por 1485, ganhando uma etapa) pelo algoritmo tradicional,
e aguardar o primeiro resto que repete. Deste modo, obtm-se:
1 2 9 2 0
1 0 4 0 0
5 0 0 0
5 4 5 0
9 9 5 0
1 0 4 0

1485
0, 8700336

Como se repetiu o resto 1040, a partir da, os algarismos 7, 0, 0, 3, 3, 6


iro se repetir. Logo, a b = 1, 8700336 .
Vamos agora fazer alguns comentrios:
1. Algumas pessoas envolvidas no processo de aprendizagem da Matemtica
(alunos, professores, pais, etc.) expressam s vezes a crena de que,
com o advento da calculadora, nunca mais haver ocasio de usar o
algoritmo tradicional da diviso. Alguns at usam isso como um argumento
para proibir o uso da calculadora em certas fases iniciais da aprendizagem:
necessrio primeiro que o aluno aprenda o algoritmo tradicional, e s
depois lhe ser permitido usar a calculadora; seno, ele no ter
motivao para aprender tal algoritmo.

31

Na realidade, o exemplo aqui tratado mostra que ns, professores, temos


que exercer nossa criatividade para criar problemas desafiadores, que
coloquem em xeque at mesmo a calculadora, deixando claras as suas
limitaes, em vez de proibir o uso da calculadora, que uma atitude
antiptica, repressora, e totalmente contrria ao que um aluno espera de
um professor de Matemtica. De fato, para um leigo ou iniciante em
Matemtica, nada mais matemtico do que uma calculadora, e ele
espera que um professor v inici-lo ou ajud-lo com essa ferramenta, e
no proibi-lo de us-la.
Note-se tambm que, mesmo usando o algoritmo tradicional da diviso,
como fizemos, a calculadora permanece til para efetuar as
multiplicaes e subtraes envolvidas no processo, minorando as
possibilidades de erro e poupando trabalhos repetitivos e inteis.
2. O trabalho de diviso ficaria simplificado, se tivssemos observado que
o divisor 1485 tem o fator comum 5 com a base do sistema decimal
(um detalhe nem sempre lembrado). Desse modo:

1292 1292
1 2584 1
208
=
=
= 8 +
=
1485 5 297 10 297 10
297
1 208
= 0, 8 + 0, 070336 = 0, 8700336, pois
0, 8 +
10 297
2 0 8 0
1 0 0 0
1 0 9 0
1 9 9 0
2 0 8

297
0, 70336

Os nmeros envolvidos no algoritmo da diviso ficam menores.


3. Existiria um outro mtodo para encontrar uma representao decimal de

208
1292
, mas j vimos que basta o primeiro), que no fosse
(ou de
297
1485
o algoritmo tradicional da diviso? A resposta sim.
Basta tomar as sucessivas potncias de 10, a saber: 10, 100, etc., at
que encontremos uma que deixe resto l, quando dividida por 297.
32

No difcil fazer isso, experimentando com a calculadora:


103 = 3 x 297 + 109; 104 = 33 x 297 + 199;
106 = 3367 x 297 + 1.
A partir da, obtm-se:

105 = 336 x 297 + 208;

1
1
= 3367 6 , e portanto:
297
10 1

208
1
1 106
= 208 3367 6
= 700336
=
297
10 1
1 1 106
700336
1
1

1 + 6 + 12 + K = 0, 700336700336700336K = 0, 700336,
6
10 10 10

onde a ltima passagem vem da propriedade das progresses geomtricas


infinitas: 1 + q + q 2 + K =

1
, quando 1 < q < 1.
1 q

Observe que o perodo da dzima tem comprimento 6, que justamente


o expoente da menor potncia de 10 que deixa resto 1, quando dividida
por 297.
4. Pode-se ter certeza de que, ao testar as potncias de 10, vamos acabar
encontrando sempre uma que deixe resto 1, quando dividida por 297 ?
A resposta positiva, sempre que o denominador (no caso, o 297) for
primo com 10 ( por isso que devemos antes deixar de fora os fatores
2 e 5), e pode ser encontrada nos livros de Teoria dos Nmeros.

Baseado no artigo
As dzimas peridicas e a calculadora
Jos Paulo Q. Carneiro, RPM 52

33

Mania de Pitgoras

Elisha Scott Loomis, professor de Matemtica em


Cleveland, Ohio (Estados Unidos), era realmente um
apaixonado pelo Teorema de Pitgoras. Durante 20 anos,
de 1907 a 1927, colecionou demonstraes desse teorema,
agrupou-as e as organizou num livro, ao qual chamou The
Pythagorean Proposition (A Proposio de Pitgoras).
A primeira edio, em 1927, continha 230 demonstraes.
Na segunda edio, publicada em 1940, esse nmero foi
aumentado para 370 demonstraes. Depois do falecimento
do autor, o livro foi reimpresso, em 1968 e 1972, pelo
National Council of Teachers of Mathematics daquele
pas.
O Professor Loomis classifica as demonstraes do
teorema de Pitgoras em basicamente dois tipos: provas
algbricas (baseadas nas relaes mtricas nos
tringulos retngulos) e provas geomtricas (baseadas
em comparaes de reas). Ele se d ao trabalho de
observar que no possvel provar o teorema de Pitgoras
com argumentos trigonomtricos porque a igualdade
fundamental da Trigonometria, cos2 x + sen2 x = 1, j um
caso particular daquele teorema.
Como sabemos, o enunciado do teorema de Pitgoras
o seguinte: A rea do quadrado cujo lado a hipotenusa
de um tringulo retngulo igual soma das reas dos
quadrados que tm como lados cada um dos catetos. Se
a, b so as medidas dos catetos e c a medida
da hipotenusa, o enunciado equivale a afirmar
que a2 + b2 = c2.
34

Documentos histricos mostram que os egpcios e os babilnios muito


antes dos gregos conheciam casos particulares desse teorema, expressos
em relaes como
3
1
32 + 42 = 52 e 12 + ( ) 2 = (1 ) 2 .
4
4

O fato de que o tringulo de lados 3, 4 e 5 retngulo era (e ainda ) til


aos agrimensores. H tambm um manuscrito chins, datado de mais de
mil anos antes de Cristo, onde se encontra a seguinte afirmao: Tome o
quadrado do primeiro lado e o quadrado do segundo e os some; a raiz
quadrada dessa soma a hipotenusa. Outros documentos antigos mostram
que na ndia, bem antes da era Crist, sabia-se que os tringulos de lados
3, 4, 5, ou 5, 12, 13, ou 12, 35, 37 so retngulos.
O que parece certo, todavia, que nenhum desses povos sabia demonstrar
o teorema. Tudo indica que Pitgoras foi o primeiro a prov-lo. (Ou algum
da sua Escola o fez, o que d no mesmo, pois o conhecimento cientfico
naquele grupo era propriedade comum.)
A mais bela prova
Qual foi a demonstrao dada por Pitgoras? No se sabe ao certo, pois
ele no deixou trabalhos escritos. A maioria dos historiadores acredita que
foi uma demonstrao do tipo geomtrico, isto , baseada na comparao
de reas. No foi a que se encontra nos Elementos de Euclides, e que
ainda hoje muito encontrada nos livros de Geometria, pois tal demonstrao
parece ter sido concebida pelo prprio Euclides. A demonstrao de Pitgoras
pode muito bem ter sido a que decorre das figuras abaixo:
a
c

b
a

c
b

b c

a
b

Do quadrado que tem a + b como lado, retiremos 4 tringulos iguais ao


dado. Se fizermos isso como na figura esquerda, obteremos um quadrado
de lado c. Mas se a mesma operao for feita como na figura direita,
restaro dois quadrados, de lados a e b respectivamente. Logo, a rea do
35

quadrado de lado c a soma das reas dos quadrados cujos lados medem a
e b.
Essa , provavelmente, a mais bela demonstrao do teorema de
Pitgoras. Entretanto, no livro de Loomis ela aparece sem maior destaque,
como variante de uma das provas dadas, no sendo sequer contada entre
as 370 numeradas.
Apresentamos a seguir algumas demonstraes do teorema de Pitgoras
que tm algum interesse especial, por um motivo ou por outro. As 4 primeiras
constam da lista do Professor Loomis.
A prova mais curta
tambm a mais conhecida. Baseiase na seguinte conseqncia da
semelhana de tringulos retngulos:
Num tringulo retngulo, cada cateto
a mdia geomtrica entre a hipotenusa
e sua projeo sobre ela. Assim, se m e
n so respectivamente as projees dos
catetos a e b sobre a hipotenusa c, temos
a2 = mc, b2 = nc, enquanto m + n = c.
Somando, vem a2 + b2 = c2.

a
m

A demonstrao do presidente
James Abram Garfield, presidente dos Estados Unidos durante apenas
4 meses (pois foi assassinado em 1881), era tambm general e gostava de
Matemtica. Ele deu uma prova do teorema de Pitgoras baseada na
seguinte figura.

b
a

36

a
b

A rea do trapzio com bases a, b e


altura a + b igual semi-soma das
bases vezes a altura. Por outro lado, a
mesma rea tambm igual soma das
reas de 3 tringulos:
a+b
a2
b2
c2
+ ab +
= ab + ,
( a + b) =
2
2
2
2
2
2
2
implicando a + b = c .

A demonstrao de Leonardo da Vinci


O grande gnio tambm concebeu uma
demonstrao do teorema de Pitgoras, que
se baseia na figura ao lado.
Os quadrilteros ABCD, DEFA, GFHI
e GEJI so congruentes. Logo, os
hexgonos ABCDEF e GEJIHF tm a
mesma rea. Da resulta que a rea do
quadrado FEJH a soma das reas dos
quadrados ABGF e CDEG.

C
B

A
F

J
I

A demonstrao de Papus
Na realidade, no se trata apenas de uma nova demonstrao, mas de
uma generalizao bastante interessante do teorema de Pitgoras. Em vez
de um tringulo retngulo, toma-se um tringulo arbitrrio ABC; em vez de
quadrados sobre os lados, tomam-se paralelogramos, sendo dois deles
quaisquer, exigindo-se que o terceiro cumpra a condio de CD ser paralelo
a HA, e com o mesmo comprimento.
G H
I
L
K
F
J
A
B
E

M
N

O teorema de Papus afirma que a rea do paralelogramo BCDE a


soma das reas de ABFG e AIJC. A demonstrao se baseia na simples
observao de que dois paralelogramos com bases e alturas de mesmo
comprimento tm a mesma rea.
Assim, por um lado, AHKB tem a mesma rea que ABFG e, por outro
lado, a mesma rea que BMNE. Segue-se que as reas de BMNE e
ABFG so iguais. Analogamente, so iguais as reas de CDNM e CAIJ.
Portanto, a rea de BCDE a soma das reas de ABFG e CAIJ.
O teorema de Pitgoras caso particular do de Papus. Basta tomar o
tringulo ABC retngulo e trs quadrados em lugar dos trs paralelogramos.
37

O argumento de Polya
No meu entender, entretanto, a demonstrao mais inteligente do teorema
de Pitgoras no est includa entre as 370 colecionadas pelo Professor
Loomis. Ela se acha no livro Induction and Analogy in Mathematics, de
autoria do matemtico hngaro George Polya.
O raciocnio de Polya se baseia na conhecida proposio, segundo a
qual as reas de duas figuras semelhantes esto entre si como o quadrado
da razo de semelhana.
Lembremos que duas figuras F e F dizem-se semelhantes quando a
cada ponto A da figura F corresponde um ponto A em F, chamado o seu
homlogo, de tal maneira que se, A, B so pontos quaisquer de F e
A, B so seus homlogos em F ento a razo AB/AB uma constante
k, chamada a razo de semelhana de F para F. Por exemplo, dois
tringulos so semelhantes se, e somente se, os ngulos de um deles so
congruentes aos ngulos do outro. Por outro lado, dois quadrados quaisquer,
um de lado l e outro de lado l, so semelhantes e a razo de semelhana
do primeiro para o segundo k = l/l.
Em vez do teorema de Pitgoras, Polya procura provar a seguinte
proposio mais geral (que, diga-se de passagem, j se acha nos Elementos
de Euclides):
F

Se F, F e F so figuras semelhantes, construdas respectivamente


sobre a hipotenusa c e sobre os catetos a, b de um tringulo
retngulo, ento a rea de F igual soma das reas de F e F.
O enunciado acima implica que a razo de semelhana de F para F
b/a, de F para F c/a e de F para F c/b.
Por simplicidade, escrevamos F em vez de rea de F, G em vez de
rea de G, etc.
38

Se G, G, G so outras figuras semelhantes construdas sobre a


hipotenusa e os catetos, respectivamente, em virtude da proposio acima
enunciada, teremos:

G F
G b 2 F
, logo
=
.
= 2 =
G" a
F"
G" F "
De modo anlogo teremos

G G
= .
F F

Portanto, G/F = G/F = G/F = , digamos. Escrevendo de outro


modo: G = F, G = F e G = F.
Que significam essas 3 ltimas igualdades? Elas querem dizer que, se
conseguirmos achar 3 figuras semelhante especiais F, F e F, construdas
sobre a hipotenusa e os catetos do nosso tringulo, de tal maneira que se
tenha F = F + F, ento teremos tambm G = G + G sejam quais
forem as figuras semelhantes G, G e G construdas do mesmo modo.
Com efeito, teremos G = F, G = F e G = F, logo G + G =
F + F = (F + F) = F = G.
Agora s procurar as figuras especiais. Mas elas esto facilmente ao
nosso alcance. Dado o tringulo retngulo ABC, tracemos a altura CD,
baixada do vrtice do ngulo reto C sobre a hipotenusa AB.

A figura F ser o prprio tringulo ABC. Para F escolheremos ADC e


faremos F = BCD. Evidentemente, F, F e F so figuras semelhantes.
Mais evidentemente ainda, temos F = F + F.
Baseado no artigo
Mania de Pitgoras
Euclides Rosa, RPM 02

39

Usando reas

Neste artigo, procuraremos mostrar que diversas


demonstraes em Geometria e Trigonometria tornam-se
fceis e elegantes quando usamos o conceito de rea.
Como primeiro exemplo, comparemos duas solues de
um conhecido problema.
Provar que a soma das distncias de um ponto
qualquer interior a um tringulo equiltero aos lados
constante.
1a soluo
Consideremos o tringulo equiltero ABC da figura, um
ponto P interior e as perpendiculares PX, PY e PZ aos
seus lados. Tracemos por P, BC paralelo a BC,
nformando o tringulo equiltero ABC. Tracemos ainda
as alturas AE e CF desse tringulo e a perpendicular
PQ a CF.
A
F

Z
B
B

X D

Y
C
C

Pela congruncia dos tringulos PQC e PYC,


conclumos que PY = CQ e, como PQFZ um retngulo,
temos que PZ = QF. Logo,
40

PY + PZ = CQ + QF = CF . (Para simplificar a notao, usaremos o


mesmo smbolo para representar um segmento ou a sua medida.)
Ora, as alturas AE e CF do tringulo equiltero ABC so iguais e,
portanto,
PY + PZ = AE.
(1)
Prolongando AE at a base BC do tringulo, obtemos ED = PX.
Finalmente, na igualdade (1), somamos PX do lado esquerdo e ED do
lado direito para obter
PX + PY + PZ = AE + ED = AD, altura de ABC.
2a soluo

Consideremos agora o tringulo


equiltero ABC com lado a e altura
h, como na figura ao lado. Traando
os segmentos PA, PB e PC, temos
que a soma das reas dos tringulos
PBC, PCA e PAB igual rea de
ABC. Logo,

aPX aPY aPZ ah


+
+
=
2
2
2
2

P
B

e o problema est resolvido. Repare que na primeira soluo usamos apenas


o conceito de congruncia de tringulos, mas a construo das linhas
auxiliares pode ser considerada um pouco artificial. Na segunda soluo,
quando o conceito de rea foi utilizado, o resultado apareceu de forma bem
mais natural.
Vejamos, ento, alguns teoremas que podem ser demonstrados com o
auxlio das reas.
1) O teorema da bissetriz
A bissetriz de um ngulo de um tringulo divide o lado oposto em
segmentos proporcionais aos lados adjacentes.
Esse enunciado quer dizer que se, AD for bissetriz do ngulo A do
tringulo ABC, ento

DB AB
=
.
DC AC
41

Para demonstrar, preciso lembrar que, se dois tringulos possuem


mesma altura, a razo entre suas reas igual razo entre suas bases.
Portanto, na figura, a razo entre as reas dos tringulos ADB e ADC
igual a BD/DC. Por outro lado, qualquer ponto da bissetriz de um ngulo
eqidista de seus lados e, portanto, as perpendiculares DE e DF aos
lados AB e AC so iguais. Logo,
A

1
AB.DE
BD A ( ABD) 2
AB
=
=
=
1
DC A ( ADC )
AC.DF AC
2

E
F
B

e o teorema est demonstrado.


2) O teorema de Tales
Sejam B e C pontos dos lados AB e AC, respectivamente, do
tringulo ABC. Se BC for paralelo a BC, ento

AB AC
=
.
AB AC

Demonstrao
Se BC paralelo a BC, ento os tringulos
BCB e BCC tm mesma rea porque
possuem mesma base BC e alturas relativas
a essa base tambm iguais. Acrescentando a
esses tringulos o tringulo ABC, conclumos
que os tringulos ABC e ABC tambm
possuem mesma rea. Se dois tringulos
possuem mesma altura, ento a razo entre suas
reas igual razo entre suas bases, logo,

B
B

C
C

AB A (ABC ) A (ABC ) AC
=
=
=
AB A (ABC ) A (ABC ) AC

o que prova o teorema.


O teorema de Tales e sua recproca so importantssimos em Geometria
porque a partir deles podemos obter os teoremas relativos semelhana de
tringulos e as propriedades da homotetia. A vantagem da demonstrao
42

que aqui apresentamos est no fato que nela no importa se os segmentos


AB e AB so comensurveis ou no. A demonstrao tradicional, que usa
o feixe de paralelas, s fica completa com a incmoda passagem ao limite.
3) As frmulas trigonomtricas
As funes trigonomtricas aparecem pela primeira vez na escola
secundria, definidas para ngulos agudos, como razes entre lados de um
tringulo retngulo. Usando figuras particulares, conseguimos calcular os
valores das funes trigonomtricas para 30, 45, 60 e 18 e podemos
antecipar diversas frmulas que, mais tarde, sero deduzidas em contexto
mais geral. Para ilustrar, vamos mostrar a frmula do seno do arco duplo.
Se 0 < a < 45, ento sen2a = 2 sen a cos a
Para demonstrar, consideremos a figura
formada por dois tringulos retngulos
congruentes OCA e OCB, em que fizemos
OA = OB = 1.
Temos, ento, que CA = CB = sen a,
OC = cosa e, traando AD perpendicular a
OB, AD = sen 2a. Ora, o dobro da rea do
tringulo OAB igual a OB . AD e tambm
igual a AB . OC.

A
1

a
a

C
D

Logo, 1 sen 2a = 2 sen a cos a, ficando demonstrada a frmula.


4) A lei dos senos
Os lados de um tringulo so proporcionais aos senos dos ngulos
opostos.
Para uma demonstrao alternativa da lei dos senos, podemos partir do
fato de que a rea, A , de um tringulo igual metade do produto de dois
lados pelo seno do ngulo formado por eles, ou seja,
1
1
1
A = bc sen A = ac sen B = ba sen C.
2
2
2

Ora, considerando a primeira igualdade e multiplicando por a ambos os


membros, obtemos
1
a
abc
aA = abc sen A ou
=
.
2
sen A 2A
43

Como o mesmo pode ser feito para as outras igualdades, conclumos

a
b
c
=
=
.
sen A sen B sen C
Muitos problemas possuem tambm solues bonitas e elegantes usando
reas e este artigo termina convidando o leitor a incluir a idia em sua
caixa de ferramentas de soluo de problemas.
Baseado no artigo
Usando reas
Eduardo Wagner, RPM 21

44

Trigonometria e um
antigo problema de otimizao

Regiomontanus
A cidade de Kningsberg, na Prssia (atual
Kalimingrado, na Rssia), conhecida na Matemtica
devido ao famoso problema das pontes (ver artigo neste
livro), resolvido pelo matemtico suo Leonhard Euler
(1707-1783). Outro acontecimento importante que marca
a vida da cidade, cujo nome significa Montanha do Rei,
o fato de nela ter nascido Johann Mller (1436-1476), um
dos maiores matemticos do sculo XV, mais conhecido
como Regiomontanus, uma latinizao do nome de sua
cidade natal.
Regiomontanus realizou diversos estudos nas reas de
Astronomia, Geometria e Trigonometria. Em seu livro mais
famoso, De Triangulus Omnimodes, escrito em 1464 e
impresso apenas em 1533, Regiomontanus apresenta uma
viso moderna da Trigonometria com dados tabelados de
vrias funes trigonomtricas. curioso notar que,
mesmo tendo sido escrito antes do conceito de notao
decimal, as tabelas trigonomtricas contidas no livro no
apresentam fraes devido utilizao de um crculo de
raio 100 000 000 de unidades, o que produzia apenas
valores inteiros para as aproximaes utilizadas.
A importncia dos conhecimentos em Astronomia de
Regiomontanus fez com que ele fosse convidado pelo
Papa Sisto IV para trabalhar na confeco de um
calendrio mais acurado do que o que vinha sendo usado
pela Igreja. Aps a realizao do trabalho, a gratido do
Papa foi tal que rapidamente o astrnomo se tornou seu
principal conselheiro. Depois de um ano em Roma,
Regiomontanus faleceu, tendo sido anunciado, como causa

45

de sua morte, o flagelo de uma peste. Existem especulaes de que ele


tenha sido envenenado por alguma pessoa descontente com a alta influncia
de um no italiano sobre o Papa e a Igreja romana. Alguns historiadores
especulam ainda que, se no tivesse falecido to cedo, talvez tivesse
condies de realizar uma moderna compreenso do sistema solar, como a
feita por Coprnico, 100 anos depois.
Entre os interessantes problemas propostos por Regiomontanus,
destacamos um de 1471 como o primeiro problema de extremos encontrado
na Histria da Matemtica desde a antiguidade. O problema o seguinte:
Suponha uma esttua de altura h sobre
um pedestal de altura p. Um homem de
altura m (m < p) enxerga do p ao topo
da esttua sob um ngulo , que varia de
acordo com a distncia d entre o homem e
a base do pedestal. Determinar d para que
o ngulo de viso seja o maior possvel.
Uma soluo engenhosa para o problema
Apesar de o problema poder ser resolvido com tcnicas do Clculo,
apresentamos uma soluo que, embora engenhosa, dispensa essas tcnicas.
Inicialmente marcamos na figura os pontos
A, B e C representando respectivamente o
A
topo da esttua, o p da esttua e os olhos do
l
observador. Em seguida, traamos a reta r
que passa por C e paralela linha do cho.
Traamos ento a nica circunferncia , com
B
centro na mediatriz do segmento AB, que
a
C
r
passa pelos pontos A e B e tangencia a reta
Ct
r. Chamamos de Ct o ponto de tangncia da
circunferncia com a reta r. Se C percorrer
livremente a reta r, qualquer possibilidade para o ngulo de viso, , ser
dada por uma localizao de C em r.
Provaremos que assume o maior valor possvel quando C coincide
com Ct . Para isso, mostraremos que, se a medida do ngulo ACtB,
ento > para qualquer posio de C diferente de Ct.

46

Se D o ponto de encontro da
reta AC com a circunferncia ,
temos que tambm a
medida do ngulo ADB e,
denotando por a medida do
ngulo CBD, tem-se, no tringulo
BCD,

B
r

+ + 180 = 180. Logo,


= + implicando > .

Ct

Uma vez verificado que ACtB o ngulo de mximo campo visual,


determinaremos agora a distncia d, entre o observador e a base do
pedestal, para que esse ngulo seja atingido.
Se Q o ponto de interseco da reta AB com r, sendo as retas r e
AB, respectivamente, tangente e secante a aplicando potncia no ponto
Q encontraremos a distncia d procurada:
(QCt)2 = QB.QA ou d2 = (p m)(p m + h)
Uma aplicao
Em outubro de 1931, aps cinco
anos de construo, foi inaugurado no
alto do morro do Corcovado o carto
de visitas do Rio de Janeiro, a esttua
do Cristo Redentor. A altura total da
esttua de 30 m, seu pedestal mede
8 m, e admitiremos um observador
com 1,70 m de altura.
A que distncia esse observador deve ficar da base do pedestal do Cristo
Redentor para que o seu ngulo de viso seja o maior possvel?
Usando a frmula d2 = (p m)(p m + h) para p = 8 m, m = 1,70 m
e h = 30 m, obtemos uma distncia de aproximadamente 15 m. Seria preciso,
porm, que o terreno em volta do Cristo fosse aproximadamente plano dentro
desse raio.
Baseado no artigo
Trigonometria e um antigo
problema de otimizao
Jos Luiz Pastore Mello, RPM 52
47

Vale para 1, para 2, para 3, ...


Vale sempre?

As afirmaes abaixo, sobre nmeros naturais, so


verdadeiras para os nmeros 1, 2, 3 e muitos outros.
Perguntamos: elas so verdadeiras sempre?
Verdadeiro ou falso?
1. n N, n < 100.
2. n N, n2 + n + 41 um nmero primo.
3. n N*, 991n2 + 1 no um quadrado perfeito.
4. n N*, a soma dos n primeiros nmeros mpares
n2.
5. n N*, 2n + 2 a soma de dois nmeros primos.
Vejamos:
1. n < 100 uma sentena verdadeira para n = 1,
n = 2, n = 3 e outros, mas torna-se falsa para qualquer
nmero natural maior do que 99. Portanto,
n N, n < 100 uma sentena falsa.
2. n2 + n + 41 um nmero primo uma sentena
verdadeira para n = 1, n = 2, n = 3 e outros. De fato,
ela verdadeira para todos os nmeros naturais
menores do que 40 (o que foi verificado por Euler em
1772). Porm, o nmero
402 + 40 + 41 = 40(40 + 1) + 41 = 41 x 41 no
primo, mostrando que a sentena
n N, n2 + n + 41 um nmero primo uma
sentena falsa.
48

3. 991n2 + 1 no um quadrado perfeito uma sentena verdadeira para


n = 1, n = 2, n = 3 e, mesmo aps muitas e muitas tentativas, no se
acha um nmero que a torne falsa.
Pudera! O menor nmero natural n para o qual 991n2 + 1 um
quadrado perfeito
12 055 735 790 331 359 447 442 538 767 e, portanto, a sentena
n N*, 991n2 + 1 no um quadrado perfeito falsa.
4. A soma dos n primeiros nmeros mpares n2 uma sentena
verdadeira para n = 1, n = 2, n = 3 e, como no caso anterior, aps
muitas e muitas tentativas, no se acha um nmero natural que a torne
falsa. Neste caso, tal nmero no existe, pois, como veremos adiante,
essa sentena verdadeira sempre.
5. 2n + 2 a soma de dois nmeros primos uma sentena verdadeira
para n = 1, n = 2, n = 3 e, como nos dois exemplos anteriores, aps
muitas e muitas tentativas, no se encontra um nmero natural que a
torne falsa. Mas agora temos uma situao nova: ningum, at hoje,
encontrou um nmero que tornasse a sentena falsa e ningum, at
hoje, sabe demonstrar que a sentena verdadeira sempre.
A sentena a famosa conjetura de Goldbach feita em 1742, em uma
carta dirigida a Euler:
Todo inteiro par, maior do que 2, a soma de dois nmeros primos.
No se sabe, at hoje, se essa sentena verdadeira ou falsa.
Em suma, dada uma afirmao sobre nmeros naturais, se encontrarmos
um contra-exemplo, saberemos que a afirmao no sempre verdadeira.
E se no acharmos um contra-exemplo? Nesse caso, suspeitando que a
afirmao seja verdadeira sempre, uma possibilidade tentar demonstr-la
recorrendo ao princpio da induo.
Princpio da induo finita
Seja S um conjunto de nmeros naturais, com as seguintes propriedades:
1. 0 S
2. k N, se k S, ento k + 1 S.
Nessas condies, S = N.

49

Vamos ver como esse princpio nos permite demonstrar que verdadeira
a sentena 4: n N*, a soma dos n primeiros nmeros mpares n2.
Demonstrao
Seja S o conjunto dos nmeros naturais n para os quais a soma dos n
primeiros nmeros mpares n2.
1. 1 S, pois a soma dos 1 primeiros nmeros mpares 1 = 12.
2. Vamos supor que k S, isto , que a soma dos k primeiros nmeros
mpares seja k2.
Vamos provar que k + 1 S, isto , que a soma dos k + 1 primeiros
nmeros mpares (k + 1)2.
Estamos supondo que 1 + 3 + 5 + ... + 2k 1 = k2 e queremos provar
que
1 + 3 + 5 + ... + 2k + 1 = (k + 1)2. Basta observar que
1 + 3 + 5 + ... + (2k 1) + (2k + 1) = k2 + (2k + 1) = (k + 1)2.
O princpio da induo nos garante, agora, que S = IN*, ou seja, a
afirmao a soma dos n primeiros mpares n2 verdadeira para todos
os nmeros naturais maiores do que zero.
Uma lenda
Aps a criao do mundo, em um mosteiro escondido na ndia, o Grande
Criador colocou uma placa de bronze e nela fixou trs bastes cobertos de
diamantes. Em um dos bastes, em ordem decrescente de tamanho, colocou
64 discos de ouro. E assim disse aos monges: Transfiram esta pilha de
discos para outro basto, movendo, ininterruptamente, um disco de cada
vez e nunca permitindo que um disco fique acima de um menor. Quando
terminarem esta tarefa e os 64 discos estiverem em outro basto, este
templo se reduzir a p e com um estrondo de troves o mundo acabar.
Dizem os sbios que o mundo foi criado h 4 bilhes de anos
aproximadamente e os monges, desde a criao, esto movendo os discos
na razo de um disco por segundo. Ser que veremos o mundo acabar?
Como muito difcil imaginar os
movimentos feitos com uma pilha de
64 discos, imaginemos uma pilha com
Um disco: a transferncia se d com
apenas 1 movimento: m1 = 1.
50

Dois discos

Para 2 discos, a transferncia requer 3 movimentos: m2 = 3.


Trs discos: m3 = 7.

Quatro discos: m4 = 15.

J podemos deduzir como deslocar n discos com um menor nmero


possvel de movimentos. Para tal, observe que o deslocamento do maior
disco, do basto em que se encontra inicialmente para um outro, requer que
esse segundo basto esteja vazio, pois o maior disco no pode ficar sobre
um menor. Como, para se mover o maior disco, nenhum outro pode estar
sobre ele, todos os outros discos tero que estar no terceiro basto. Assim,
a estratgia com menor nmero de movimentos ser: movem-se n 1
discos para o basto de trs, com mn-1 movimentos; em seguida, move-se
o n-simo disco para o outro basto da frente, com 1 movimento; finalmente
movem-se os n 1 discos do basto de trs para o da frente, com mn1
movimentos. Tem-se:
mn = mn1 + 1 + mn1 = 2mn1 + 1
Faamos uma tabela com o nmero de discos e o nmero de movimentos
mnimo para mud-los de um basto para outro:

51

...

mn

15

31

63

...

Precisamos descobrir o valor de m64 porque, m64 segundos aps a


criao do mundo, ele acabar e j se passaram 4 bilhes de anos!
Observando a segunda linha da tabela, vemos que os seus nmeros so,
a menos de 1: 2, 4, 8, 16, 32, 64, ou seja, 21, 22, 23, 24, 25, 26, o que
nos leva a fazer a seguinte conjetura:
mn = 2n 1
Essa sentena verdadeira para n = 1, 2, 3, 4, 5, 6, mas ser
verdadeira sempre?
Tentemos demonstr-la por induo.
Seja S o conjunto dos nmeros naturais n tais que n discos so
movidos com 2n 1 movimentos.
1. 1 S, pois para 1 disco necessitamos de 1 = 21 1 movimentos.
2. Vamos supor que k S, isto , k discos so removidos com 2k 1
movimentos.
Vamos provar que k + 1 S, isto , que mk +1 = 2k +1 1.
J vimos que mk + 1 = 2mk + 1.
mk + 1 = 2k 1 + 1 + 2k 1 = 2 . 2k 1 = 2k + 1 1,
e isso mostra que k + 1 S.
O princpio da induo nos garante que n discos podem sempre ser
removidos com 2n 1 movimentos e, em particular, m64 = 264 1.
E assim ficamos sabendo que, 264 1 segundos aps a criao do
mundo, ele terminar. Com um pouco mais de Matemtica ficaremos sabendo
se isso ocorrer logo.
Faamos alguns clculos.
Quantos segundos tem um ano?
Resposta:

1
60 60 24 365 = 31 557 600 < 225 = 1024 1024 32 = 33 554 432.
4

52

Exagerando, vamos supor que os monges faam 225 movimentos por


ano (na verdade fazem uns milhes a menos). Com isso, o mundo acabar
em

264
= 239 anos.
225
239 = 210 x 210 x 29 = 1 024 x 1 024 x 1 024 x 512 > 512 x 109

Passaram-se at hoje 4 bilhes de anos, ou seja, 4 x 109 anos.


Podemos ficar tranqilos faltam mais do que 508 bilhes de anos para
os monges terminarem sua tarefa isso, supondo que eles no errem no
caminho.
Baseado no artigo
Vale para 1, para 2, para 3, ...
Vale sempre?
Renate Watanabe, RPM 09

53

Semelhanas, pizzas e chopes

As histrias que vamos contar envolvem dois amigos


que gostam de freqentar bares e restaurantes, alm de
discutir problemas de Matemtica. Em pelo menos duas
situaes, surgiram interessantes problemas cujas solues,
alm de elegantes, so bastante educativas.
Primeira histria
Augusto e Joo foram a um restaurante para comer
pizza. O primeiro pediu uma grande e o segundo, uma
mdia e uma pequena, todas do mesmo sabor.
Curiosamente, o preo da pizza grande era exatamente
igual soma dos preos das pizzas mdia e pequena. Logo
aps os pedidos, surgiu naturalmente o problema de saber
quem vai comer mais. O fato de os preos a pagar serem
iguais no quer dizer nada, porque, nos restaurantes, o
preo no costuma ser proporcional quantidade de
comida servida. Augusto argumenta que, se tivesse uma
rgua, poderia medir os dimetros, calcular as reas e
verificar se a rea da pizza grande maior, igual ou menor
do que a soma das reas das outras duas. Porm, no
havia rgua disponvel. Pensando um pouco, Joo, bom
gemetra, declarou ter resolvido o problema, dizendo que
assim que as pizzas chegassem diria quem comeria mais,
e para isso usaria apenas objetos que estavam em cima
da mesa. Augusto, estupefato, duvidou. Como possvel?
No temos instrumento de medida algum. Em cima da
mesa s h talheres, copos, guardanapos e o cardpio,
responsvel por nossa incrvel discusso! A espera no
54

foi longa e as pizzas chegaram. Rapidamente, ento, Joo cortou ao meio


cada uma delas, obtendo as reas A, B e C.

A
B

Sobre a mesa (de mrmore) juntou os dimetros para formar um


tringulo.
Utilizando o canto do cardpio como um modelo para o ngulo reto,
Joo verificou que o ngulo, , oposto ao dimetro da maior metade era
menor do que 90 e declarou eu como mais. E Augusto, aps pensar
alguns momentos, concordou.
Qual a explicao?
A explicao depende de dois resultados importantes. O primeiro bastante
conhecido e o segundo no muito.
1. A razo entre as reas de figuras semelhantes igual ao quadrado
da razo de semelhana.
2. Se figuras semelhantes so construdas sobre a hipotenusa e sobre
os catetos de um tringulo retngulo, ento a rea da figura maior
igual soma das reas das outras duas.
A demonstrao desse segundo resultado pode ser vista no artigo Mania
de Pitgoras, publicado neste mesmo exemplar.
Para concluir que no nosso problema Joo estava certo, observe que, se
o ngulo oposto ao lado a do tringulo de lados a, b e c, temos:
< 90 a2 < b2 + c2 A < B + C,
> 90 a2 > b2 + c2 A > B + C,
Portanto, se na nossa histria Joo constatou que o ngulo era menor
que 90, ento a rea da semipizza grande era menor que a soma das reas
das outras duas metades.
55

Segunda histria
Dias depois, Augusto, afobado com o calor, senta em um bar e pede um
chope (na verdade, o primeiro de muitos). Nesse lugar, o chope servido
em tulipas, que so copos com a forma de um cone invertido. O garom
chega com a bebida ao mesmo tempo que Joo encontra seu amigo. Como
vai, Joo? Sente e tome rpido a metade deste copo. Eu tomo a outra
metade. A fisionomia de Joo mostra alguma tristeza. Como determinar a
altura do nvel da bebida quando um copo cnico contm a metade do seu
contedo? Augusto ento alivia a situao. Meu caro amigo, para esse
problema, seus artifcios so insuficientes. Eu hoje vim prevenido e trouxe
uma rgua e uma calculadora. Desculpe a brincadeira e vamos juntos resolver
o nosso problema.
Augusto ento saca de sua rgua, calculadora, caneta e sobre um
guardanapo mostra a soluo sob o olhar de um estupefato garom.
Observe, Joo, que o copo tem 20 cm de altura. Desejamos obter a
altura da superfcie do lquido que corresponda metade do volume do
copo. Para isso, precisamos recordar dois outros fatos:
3. Toda seo paralela base de um cone forma um outro cone
semelhante ao primeiro.
4 . A razo entre o volume de slidos semelhantes igual ao cubo da
razo de semelhana.
Augusto continua sua explicao.
Se voc tiver tomado uma parte do contedo deste copo, teremos aqui,
por 2, dois objetos semelhantes: o cone formado pelo lquido e o prprio
copo. A razo de semelhana entre esses dois cones a razo entre suas
alturas, ou seja, h/20. Como desejamos que o lquido tenha a metade do
volume do copo, por 3, podemos escrever:
3
h
1
1 h
=3 .
= , ou seja,
20
2
2 20

20

Assim, a altura que corresponde


3

metade do volume do copo h = 10 4 cm.

56

Joo concorda com a perfeita explicao, mas repara que a resposta


no resolve ainda o problema porque ele no tem a menor idia de quanto
10 3 4 . E ento Augusto, com a sua calculadora e seu sorriso irnico, diz:
Ah! bom saber que esse valor d aproximadamente 16 cm.

Bem. O problema foi resolvido e o chope, j meio quente, foi


adequadamente dividido. Falta apenas o final da histria.
Nessa altura, as pessoas das outras mesas ouviam atentamente nossos
personagens com um misto de admirao e espanto. Nisso, Joo faz uma
descoberta, que anuncia em alto e bom som: Esse problema me revela
que quando somos servidos em tulipas com 4 cm de colarinho estamos
tomando apenas metade do contedo do copo. Assim, se eu digo que tomei
10 chopes, na verdade tomei 5, mas paguei 10!.
E foram expulsos do bar.
Baseado no artigo
Semelhanas, pizzas e chopes
Eduardo Wagner, RPM 25

57

Sorrisos, sussurros,
antenas e telescpios

Elipses, sorrisos e sussurros


Para cuidar do sorriso dos pacientes, os dentistas
utilizam uma luminria com espelho elptico.
De maneira diferente dos holofotes comuns, como os
faris de carro, que refletem os raios luminosos em uma
mesma direo (valendo-se, para isso, de um espelho
parablico), os holofotes dentrios se valem de espelhos
elpticos para concentrar os raios luminosos emitidos pela
lmpada em um determinado ponto: o dente a ser tratado.
Isso possvel devido ao fato de que, como veremos
adiante, todo raio emitido em um dos focos se dirigir,
aps a reflexo no espelho elptico, exatamente para o
outro foco (estamos pensando na elipse plana com focos
na lmpada e no dente sendo tratado e parcialmente
contida no espelho elptico). Isso tambm explica o
funcionamento de diversos aparelhos de emisso de raios
usados em tratamentos mdicos, como, por exemplo, o de
radioterapia, cujos raios devem destruir os tecidos doentes
sem afetar os tecidos sadios que se encontram ao redor.
J as salas de sussurros so construes de forma oval
onde esto marcados dois pontos no cho. Duas pessoas
em p, uma em cada um desses pontos, podem se
comunicar em voz sussurrada, inaudvel no restante da
sala. A forma da sala de fundamental importncia.
Ao projet-la, fixam-se dois pontos P e Q, que ficam
58

na altura da cabea das pessoas que vo se


comunicar. A seguir, toma-se uma elipse E que
admita P e Q como focos, e a sala construda
de tal maneira que qualquer plano que passe
por esses pontos intercepte a sala segundo uma
elipse congruente com a escolhida.

A elipse de focos P e Q por definio o conjunto dos pontos de um


plano por P e Q tais que a soma das distncias do ponto aos focos
constante. Assim, todas as ondas sonoras emitidas em um dos focos, ao se
refletirem nas paredes da sala, chegaro ao segundo foco tendo percorrido
a mesma distncia, ou seja, ao mesmo tempo, o que, sem dvida, proporciona
uma amplificao natural do som, explicando o funcionamento das salas de
sussurros. Vejamos ento uma propriedade da elipse da qual decorre a
propriedade de reflexo mencionada.
Propriedade
Seja uma elipse E com focos P e
Q e seja um ponto X E. Nesse caso
a reta r, tangente a E em X, forma
ngulos iguais com PX e QX.

Para no quebrar a continuidade do estudo das propriedades de reflexo


das outras cnicas, colocamos a demonstrao da propriedade da elipse no
final do artigo.
Para concluir que a propriedade da elipse garante os fenmenos
anteriormente citados, lembremos duas leis fsicas sobre a reflexo. A
primeira diz que o ngulo de incidncia e o ngulo de reflexo em um plano
so iguais. A outra lei diz que a reflexo em cada ponto de uma superfcie
se comporta como se fosse no plano tangente superfcie, no respectivo
ponto. Logo, a propriedade garante os fenmenos de reflexo mencionados.
Por que as antenas so parablicas?
A palavra parbola est, para os estudantes do ensino mdio, associada
ao grfico da funo do segundo grau. Entretanto, quase todos conhecem
as antenas parablicas, mas nem todos fazem ligao entre uma coisa e
outra. Os espelhos dos telescpios e dos faris dos automveis tambm so
parablicos. Por qu?
59

Consideremos uma reta d e um ponto F.


A parbola de foco F e diretriz d , por
definio, o conjunto de todos os pontos do
plano definido por F e d cuja distncia
reta d igual distncia ao ponto F. O
segmento EF chama-se parmetro da
parbola e o ponto mdio V, mdio de EF,
o vrtice da parbola.

D
V F

E
d

A equao da parbola
Em um sistema de coordenadas, no difcil encontrar a equao da
parbola, dados o foco e a diretriz. Tomemos F = (0, p) como foco e
y = p como diretriz.
x 2 + ( y p ) 2 = y + p.

Se P = (x, y) tal que PF = PD, temos:

Elevando ao quadrado e cancelando os termos iguais dos dois lados,


obtemos: x 2 = 4 py ou y =

1 2
x , o que mostra que a equao de uma
4p

parbola da forma y = ax2 (uma funo polinomial de grau 2).


y

F = (0, p)

P = (x, y)
x

y = -p
D

Reciprocamente, dada uma funo da forma y = ax2, fcil provar


que qualquer um de seus pontos possui distncia ao ponto (0,

1
) igual
4a

1
, o que mostra que o grfico de y = ax2 uma
4a
parbola. Com um pouco mais de trabalho, o leitor poder demonstrar que
o grfico de y = ax2 + bx + c (com a 0) tambm uma parbola,
distncia reta y =

60

exatamente igual ao grfico de y = ax2, mas agora com vrtice no ponto


(

b
b 2 4ac
,
).
2a
4a

Antenas e espelhos
Vamos voltar agora s nossas perguntas iniciais. Por que as antenas que
captam sinais do espao so parablicas? Por que os espelhos dos faris
dos carros so parablicos? Nas antenas os sinais que recebemos (ondas
de rdio ou luz) so muito fracos. Por isso, necessrio capt-los em uma
rea relativamente grande e concentr-los em um nico ponto para que
sejam naturalmente amplificados. Portanto, a superfcie da antena (ou do
espelho) deve ser tal que todos os sinais recebidos de uma mesma direo
sejam direcionados para um nico ponto aps a reflexo. Nos faris dos
carros usa-se a propriedade em direo contrria: os raios de luz emitidos
pela lmpada se refletem no espelho parablico e saem paralelos iluminando
uma regio maior.

antena

farol

Esses fenmenos so garantidos pela propriedade enunciada a seguir.


No demonstraremos aqui essa propriedade, uma vez que sua demonstrao
anloga correspondente para a elipse (consultar a RPM 33, p. 14).
Propriedade
Consideremos agora um ponto P
qualquer da parbola de foco F e
diretriz d. Se PD perpendicular
reta d, ento a reta tangente
parbola em P forma ngulos
iguais com PF e PD.

aa

Q
d

D
61

A hiprbole e os telescpios
Dados dois pontos F e F, a hiprbole de focos nesses pontos o
conjunto dos pontos de um plano por F e F cuja diferena das distncia
a F e F uma constante.
De modo anlogo elipse e parbola, a hiprbole tambm tem uma
propriedade de reflexo que conseqncia da enunciada a seguir (os
interessados podero encontrar uma demonstrao na RPM 34, p. 27).
Propriedade
Seja uma hiprbole com focos F e F e seja um ponto P da
hiprbole. Nesse caso, a reta t, tangente hiprbole em P, forma
ngulos iguais com PF e PF.
A
A propriedade garante que um raio de
luz proveniente de um ponto A, de forma que
P
F
F
a reta AP passe pelo foco F, que incide
num espelho hiperblico em P, seja refletido
de modo a passar pelo outro foco F.
O telescpio refletor nada mais do que
um espelho parablico no fundo de um tubo.
Os raios provenientes de um corpo celeste
distante (estrela, galxia, planeta, etc.)
formam um feixe praticamente paralelo, que
se reflete no espelho e vai formar a imagem
do objeto no foco F.

O problema agora que, para observar essa imagem, o observador teria


de estar com seu olho no foco da parbola, mas isso impossvel na prtica.
Isaac Newton (1642-1727) resolveu
esse problema em seu telescpio refletor,
colocando um espelho plano E entre o
espelho parablico e o foco F. Com isso,
os raios que iriam formar a imagem em F
so novamente refletidos e vo formar essa
imagem num ponto F fora do tubo do
telescpio, onde se posiciona o observador.

62

E
F
F

Em 1672 o astrnomo francs


Cassegrain props a utilizao de um
espelho hiperblico E, em lugar do espelho
plano de Newton. Um dos focos da
hiprbole coincide com o foco F da
parbola.

Agora os raios que iriam formar a imagem no foco F so refletidos pelo


espelho E e formaro essa imagem no outro foco F da hiprbole.
Para compreender a vantagem desse espelho hiperblico de Cassegrain
sobre o espelho plano de Newton, devemos observar que o espelho plano
no pode ficar muito prximo do foco F, sob pena de o ponto F ficar
dentro do telescpio; em conseqncia, o espelho plano precisa ser de
razovel tamanho, o que resulta num bloqueio significativo da luz incidente
no espelho parablico que forma a parte principal do telescpio.
O espelho de Cassegrain, pelo contrrio, pode ser construdo mais prximo
ou mais afastado do foco F, mantendo-se fixa a distncia FF entre os
focos da hiprbole; em conseqncia, o tamanho desse espelho pode ser
maior ou menor. A distncia entre os focos F e F tambm pode ser
alterada sem mudar a posio do foco F. A combinao desses fatores
permite grande flexibilidade na montagem do refletor hiperblico E,
adequando-a, assim, s exigncias das observaes.
Demonstrao da propriedade da elipse
Lembramos que, tal como na circunferncia, uma reta r tangente a
uma elipse E no ponto X se, e somente se, r E = {X}.
Denotando a distncia entre dois pontos R e S por d(R, S) e
caracterizando a elipse E como o lugar geomtrico dos pontos X que
satisfazem a propriedade mtrica,
d(X, P) + d(X, Q) = k (constante),
segue-se que um ponto A no estar na elipse se e somente se
d(A, P) + d(A, Q) k.
Logo, uma reta r ser tangente elipse E em um ponto X se e
somente se intersectar E em X e qualquer que seja o ponto A em r,
A X, se tenha:
63

d(A, P) + d(A, Q) d(X, P) + d(X, Q)


Seja, agora, um ponto X na elipse E e tomemos uma reta r (bissetriz
de um dos ngulos formados pelas retas PX e QX) passando por X de
tal forma que o ngulo entre PX e r seja igual ao ngulo entre QX e r.
Se mostrarmos que r tangente a E em X, teremos mostrado a
propriedade, devido unicidade da tangente elipse por um de seus pontos.
Seja X um ponto de E, logo d(X, P) + d(X, Q) = k. Tomemos sobre r
um ponto A X e consideremos o ponto P, simtrico de P em relao
a r.
A reta r ento mediatriz de PP.
Logo, d(X, P) = d(X, P) e tambm
d(A, P) = d(A, P). Por construo, a
reta r faz ngulos iguais com XP e
XQ e, pela simetria, os ngulos AXP e
AXP so tambm iguais. Da, os
segmentos XQ e XP fazem ngulos
iguais com r e, portanto, os pontos P,
X e Q so colineares.

X
r

Segue-se ento:
k = d(X, P) + d(X, Q) = d(X, P) + d(X, Q) = d(P, Q) < d(A, P) + d(A, Q)
= d(A, P) + d(A, Q), ou d(A, P) + d(A, Q) > k, o que mostra que A E.
Conclumos que X o nico ponto de r que pertence elipse, o que
mostra que essa reta tangente em X a essa elipse.
Baseado nos artigos
Por que as antenas so parablicas?
Eduardo Wagner, RPM 33
A hiprbole e os telescpios
Geraldo vila, RPM 34
Elipses, sorrisos e sussurros
Renato J. C. Valladares, RPM 36

64

A Matemtica do GPS

O que o GPS e como funciona?


O estudo da esfera e seus elementos fica naturalmente
contextualizado quando exploramos sua associao com
o globo terrestre. Conceitos geogrficos como paralelos,
meridianos, latitudes, longitudes e fusos horrios esto
baseados em importantes idias geomtricas, e o
estabelecimento das relaes entre eles conduz a
problemas geomtricos relevantes. Aqui veremos a
fundamentao matemtica necessria para o
entendimento de um moderno sistema de navegao por
satlites, o GPS.
A sigla GPS a abreviatura para Global Positioning
System (sistema de posicionamento global). O sistema
NAVSTAR, nome oficial dado pelo Departamento de
Defesa dos Estados Unidos ao GPS, consiste em um
segmento espacial, formado por 24 satlites, um segmento
de controle, formado pelas estaes terrestres de
gerenciamento, e um segmento do usurio. O projeto foi
iniciado em 1973 pelo Departamento de Defesa dos
Estados Unidos com propsitos militares, e logo estendido
tambm para uso civil. Hoje em dia, com auxlio do piloto
automtico e do GPS, uma aeronave civil capaz de
percorrer distncias transatlnticas e pousar sem a
interferncia do piloto com erro de alguns centmetros com
o eixo da pista. O GPS tem se mostrado til em diversas
situaes, tais como: roteiro de viagens; monitoramento

65

de abalos ssmicos; meteorologia; localizao para resgate; monitoramento


de caminhes de carga.
Os satlites orbitam em torno da Terra a uma altura aproximada de
20.200 km acima do nvel do mar, em seis rbitas estveis e predeterminadas,
com quatro satlites em cada rbita. Percorrem uma rbita completa a
cada 12 horas e cada satlite tem 28 de visualizao sobre a Terra. Isso
assegura que todo ponto da superfcie terrestre, em qualquer instante, esteja
visualizado por pelo menos quatro satlites. Vrias reas da Terra so, por
alguns momentos, visualizados por at dez satlites.
Todos os satlites so controlados pelas estaes terrestres de
gerenciamento. Uma estao master, com o auxlio de cinco estaes de
gerenciamento espalhadas pelo planeta, monitora o desempenho total do
sistema, corrigindo as posies dos satlites e reprogramando o sistema,
quando necessrio. Aps o processamento de todos esses dados, as correes
e sinais de controle so transferidos de volta para os satlites.
Cada um dos satlites do GPS transmite por rdio um padro fixado,
que recebido por um receptor na Terra (segmento do usurio), funcionando
como um cronmetro extremamente acurado. O receptor mede a diferena
entre o tempo que o padro recebido e o tempo que foi emitido. Essa
diferena, no mais do que um dcimo de segundo, permite que o receptor
calcule a distncia ao satlite emissor, multiplicando a velocidade do sinal
(aproximadamente 2,99792458.108 m/s a velocidade da luz ) pelo tempo
que o sinal de rdio levou do satlite ao receptor. Essa informao localiza
uma pessoa sobre uma imaginria superfcie esfrica com centro no satlite
e raio igual distncia acima calculada.
Cada satlite programado para emitir o que se chama efemride, que
informa a sua posio exata, naquele instante, em relao a um fixado
sistema ortogonal de coordenadas. Tal posio permanentemente
rastreada, conferida e processada pelas estaes terrestres. Com a posio
do satlite e a distncia acima calculada, obtm-se a equao da imaginria
superfcie esfrica.
Coletando-se sinais emitidos por quatro satlites, o receptor determina a
posio do usurio calculando-a como interseco das quatro superfcies
esfricas obtidas. A localizao dada, no em coordenadas cartesianas,
mas por meio das coordenadas geogrficas (latitude, longitude e a elevao).
66

A preciso do tempo essencial na operao do GPS. Um erro de um


microssegundo (106 segundos) no registro do lapso de tempo desde a
transmisso at a sua recepo resulta num erro de 300 metros. Unidades
receptoras do GPS extremamente precisas (e caras!) podem determinar
sua posio a menos de um metro.
A superfcie esfrica em coordenadas cartesianas
Em um sistema ortogonal de coordenadas cartesianas em trs dimenses,
a distncia entre os pontos P = (x, y, z) e C = (u, v, w) dada pela frmula
d ( P, C ) = ( x u ) 2 + ( y v) 2 + ( z w) 2 . Portanto, sendo r um nmero
real positivo e C = (u, v, w) um ponto fixado, a superfcie esfrica S de
centro C e raio r, que o conjunto dos pontos do espao cuja distncia a C
igual a r, tem equao (denominada equao reduzida de S):

(x u)2 + (y v)2 + (z w)2 = r2.

(1)

Desenvolvendo os quadrados em (1), obtemos (a chamada equao geral


de S)
x2 + y2 + z2 2xu 2yv 2zw + u2 + v2 + w2 r2 = 0

(2)

que uma equao da forma


x2 + y2 + z2 + ax + by + cz + d = 0

(3)

onde a, b, c, d so nmeros reais.


A interseo de duas superfcies esfricas de centros distintos vazia, ou
um ponto ou uma circunferncia, conforme a distncia entre os seus centros
maior que, igual a ou menor que a soma dos raios. O teorema a seguir
desempenha um papel importante na fundamentao matemtica do
funcionamento do GPS:
Se quatro superfcies esfricas se intersectam e seus centros so no
coplanares, ento essa interseco consiste em um nico ponto.
Demonstrao
Sejam S1, S2, S3 e S4 superfcies esfricas de centros C1, C2, C3 e C4,
satisfazendo as hipteses.
Sendo x2 + y2 + z2 + ajx + bjy + cjz + dj = 0 as equaes gerais de Sj, onde
j = 1, 2, 3, 4, ao subtrairmos essas equaes duas a duas, obtemos equaes
lineares em x, y e z, uma vez que os termos x2, y2 e z2 so eliminados.
67

Tal equao linear determina um plano que contm a correspondente


interseco. Por exemplo, subtraindo as equaes de S1 e S2, obtm-se a
equao de um plano que contm S1 S2. Considerando-se os planos que
contm S1 S2, S1 S3 e S1 S4, temos que, se P = (x, y, z) est em
S1 S2 S3 S4, ento (x, y, z) soluo do sistema linear
(a1 a2)x + (b1 b2)y + (c1 c2)z + (d1 d2) = 0
(*)

(a1 a3)x + (b1 b3)y + (c1 c3)z + (d1 d3) = 0


(a1 a4)x + (b1 b4)y + (c1 c4)z + (d1 d4) = 0

A demonstrao do teorema estar terminada se mostrarmos que o sistema


(*) tem uma nica soluo, pois a existncia de dois pontos distintos em
S1 S2 S3 S4 acarretaria duas solues distintas do sistema linear (*).
Sendo Cj = (uj, vj, wj) o centro de Sj, j = 1, 2, 3, 4, comparando as equaes
(2) e (3), temos aj = 2uj, bj = 2vj, cj = 2wj de modo que
a1 a2
a1 a3
a1 a4

b1 b2
b1 b3
b1 b4

c1 c2
u2 u1 v2 v1
c1 c3 = 8 u3 u1 v3 v1
c1 c4
u4 u1 v4 v1

w2 w1
w3 w1
w4 w1

Como C1, C2, C3, C4 so no coplanares, segue que o determinante direita


no nulo e, portanto, (*) um sistema linear com determinante no nulo,
tendo assim uma nica soluo.
Note que o simples fato de o sistema linear (*) ter uma nica soluo, o que
equivale a dizer que os centros so no coplanares, no acarreta
necessariamente que a interseco das quatro superfcies esfricas consiste
em um nico ponto P. A hiptese S1 S2 S3 S4 essencial para a
validade do teorema. interessante observar que, na situao real do GPS,
essa hiptese comprovada pela existncia do prprio usurio!
As coordenadas geogrficas de um ponto do espao
Fixemos um sistema ortogonal de coordenadas cartesianas com origem
O no centro da Terra, o eixo Oz positivo apontando na direo do Plo
Norte, o plano Oxy sendo o plano do equador com o eixo Ox positivo cortando
o meridiano de Greenwich e o eixo Oy positivo cortando o meridiano de
longitude 90E.

68

Dado um ponto P = (x, y, z) do espao, sejam e as medidas dos


ngulos assinalados na figura a seguir.

(0, 0, z) = B
P = (x, y, z)

q = m(AOP)
j = m(COA)

0
j

(x, 0, 0) = C

A = (x, y, 0)

Quando P est sobre a superfcie terrestre, os valores e acima


indicados correspondem exatamente habitual latitude e longitude do ponto
P e, por isso, manteremos a mesma nomenclatura para e .
A diferena entre OP = d (O, P ) = x 2 + y 2 + z 2 e o raio da Terra
chamada elevao (ou altitude) de P = (x, y, z).
A latitude, a longitude e a elevao so chamadas coordenadas
geogrficas do ponto P. Vejamos como relacion-las com as coordenadas
cartesianas de P.
No tringulo retngulo OPB da figura acima, temos:
cos(90 ) = sen =

OB
=
OP

z
x2 + y 2 + z 2

Essa expresso atribui a um nico valor entre 0 e 90 quando z > 0


e um nico valor entre 90 e 0 quando z < 0. No primeiro caso, dizemos
que a latitude de P N (norte), enquanto no segundo a latitude de P
() S (sul). Por outro lado, no tringulo retngulo OAC temos
sen =

AC
=
OA

y
2

x +y

e cos =

OC
=
OA

x
2

x + y2

Essas expresses definem um nico entre 0 e 180 quando y > 0 e


dizemos que a longitude de P E (leste). Quando y < 0, assume um
nico valor entre 180 e 0 e, nesse caso, a longitude de P () W
(oeste).
69

Como exemplo, vamos determinar as coordenadas geogrficas do ponto


P cujas coordenadas cartesianas so dadas, em metros, por

P = (3 3.106 , 3.106 , 6 3.106 ) .


Temos x 2 + y 2 + z 2 = 27.10 12 + 9.10 12 + 108.10 12 = 144.10 12 e
x + y2 = 27.1012 + 9.1012 = 36.1012.
2

Logo, sen =

6 3.106
12.10

3
; portanto, = 60.
2

1
3 3.106
3
e
, obtemos
cos

=
=
6
6
2
2
6.10
6.10
= 30. Assim, as coordenadas geogrficas de P so = 60 N e
= 30 W. Supondo o raio da Terra igual a 6,4.106 metros, temos que a
elevao de P mede 12.106 6,4.106 = 5,6.106 metros.

Como sen =

3.106

Uma situao real


O exemplo abaixo retrata uma situao real em que um usurio do GPS
detectado por quatro satlites. A tabela indica as efemrides (em metros)
de cada satlite tomadas em relao ao nosso fixado sistema ortogonal de
coordenadas cartesianas.

Satlite 1
Satlite 2
Satlite 3
Satlite 4

1,877191188.106
1,098145713.107
2,459587359.107
3,855818937.106

1,064608026.107
1,308719098.107
4,336916128.106
7,251740720.106

z
2,428036099.107
2,036005484.107
9,090267461.106
2,527733606.107

O receptor GPS registra os seguintes lapsos de tempo (em segundos)


entre a transmisso e a recepo do sinal de cada satlite.
Satlite 1

Satlite 2

Satlite 3

Satlite 4

0,08251731391

0,07718558331

0,06890629029

0,07815826940

Note que as informaes transmitidas no sistema GPS envolvem, por


uma questo de preciso, dez ou mais dgitos, tornando imprescindvel a
utilizao de calculadoras ou softwares com capacidade de resolver sistemas
70

lineares com coeficientes dessa ordem. Outra alternativa, abrindo mo da


preciso, trabalhar com um nmero menor de dgitos e utilizar a notao
cientfica.
Multiplicando cada lapso de tempo pela velocidade da luz,
2,99792458.108 m/s, obtemos a distncia entre o receptor e cada satlite.
Isso permite escrever as equaes reduzidas das superfcies esfricas
centradas em cada satlite e raios iguais s distncias calculadas:
S1 : (x 1,8.106 )2 + (y + 10,6.106 )2 + (z 24,2.106 )2 = 611,9.1012
S2 : (x 10,9.106 )2 + (y + 13.106 )2 + (z 20,3.106 )2 = 535,4.1012
S3 : (x 24,5.106 )2 + (y + 4,3.106 )2 + (z 9.106 )2 = 426,7.1012
S4 : (x 3,8.106 )2 + (y 7,2.106 )2 + (z 25,2.106 )2 = 549.1012
Desenvolvendo os quadrados, obtemos as respectivas equaes gerais,
e o sistema linear (*) dado por
18,2x 4,88y 7,84z 76,52.106 = 0
45,43x + 12,61y 30,38z 185,23.106 = 0
3,95x + 35,79y + 1,99z 62,95.106 = 0
cuja nica soluo x = 0,5660.107, y = 0,0978.107 e z = 0,2775.107.
O ponto P com essas coordenadas cartesianas pertence simultaneamente
s quatro imaginrias superfcies esfricas e suas coordenadas geogrficas,
calculadas como no pargrafo anterior (considerando o raio da Terra medindo
6,378164.106 metros), so
Latitude: = 26 N; Longitude: = 10 E; Elevao: 919,71 metros.
Consultando um atlas geogrfico ou um globo terrestre, identificamos a
posio desse usurio do GPS como sendo a cidade de Djanet, localizada
nos Montes Tssili, na fronteira entre Arglia e Lbia.
Baseado no artigo
A Matemtica do GPS
Srgio Alves, RPM 59

71

O problema do amigo oculto

Por ocasio das festas de fim de ano, um grupo de 9


pessoas resolveu planejar a clebre brincadeira do amigo
oculto (ou secreto). Foi escrito o nome de cada pessoa
em um papelzinho, e procedeu-se ao sorteio, para
determinar quem iria dar presente a quem. Feito o sorteio,
logo apareceu algum que tirou a si mesmo. Sendo contra
as regras da brincadeira que algum presenteie a si mesmo,
e para preservar o sigilo, foi necessrio proceder a outro
sorteio. No segundo sorteio, o mesmo fenmeno ocorreu,
dessa vez com outra pessoa. Uma das pessoas presentes
levantou a questo: Isso vai ficar acontecendo a vida
toda? Qual a probabilidade de isso acontecer?.
Na realidade, essa uma ocorrncia de um clebre
problema de Anlise Combinatria, o das chamadas
permutaes caticas.
Cada sorteio define uma funo f do conjunto das 9
pessoas em si mesmo. f(x) = y significa que x deve
presentear y. Como duas pessoas diferentes no podem
tirar o mesmo amigo oculto (o sorteio feito sem
reposio), e todas as 9 pessoas sero presenteadas, f
uma bijeo do conjunto A das 9 pessoas sobre si
mesmo, ou seja, uma permutao desse conjunto. Algum
ser amigo oculto de si mesmo quando existir em A um
certo x tal que f(x) = x. Na nomenclatura usual de
funes, um tal x chamado ponto fixo de f. O problema
agora consiste em determinar, dentre o total das 9! =
362.880 permutaes dos elementos de A, quantas so
72

as que tm ponto fixo correspondentes aos sorteios fracassados e


quantas no tm ponto fixo correspondentes aos sorteios que deram
certo. Pode parecer estranho que justamente os casos que aqui do certo
que so chamados, na nomenclatura clssica, de permutaes caticas.
O motivo que essa nomenclatura se prende interpretao de permutaes
como arrumaes dos elementos 1, . . .,9 nos lugares de 1 a 9; uma
permutao catica ento uma permutao em que todo o mundo est
fora de seu lugar natural.
Antes de resolver o problema, vamos introduzir uma forma de representar
permutaes. Adotando o clssico smbolo a b para designar que
f(a) = b, e numerando as pessoas de 1 a 9, uma possvel permutao ,
por exemplo:
1 8 2 1 3 3 4 9 5 7 6 6 7 4 8 2 9 5
Observe que podemos colocar essas informaes na seguinte ordem:
l82l

33

49574

66

Note que as pessoas 1; 8; 2; 1 formam, nessa ordem, um ciclo (de


tamanho 3): 1 presenteia 8, que presenteia 2, que presenteia 1.
Representaremos esse ciclo por (182). O mesmo ciclo poderia ser
representado tambm por (821) ou (218) (certo?), mas no por (128),
que significaria: 1 2 8 1, que diferente. Situao anloga ocorre
com os elementos 4; 9; 5; 7, que formam o ciclo (4957). Os pontos fixos
3 e 6 podem ser considerados como ciclos de tamanho 1. Desse modo, essa
permutao pode ser representada por: (182) (3) (4957) (6). Repare
que, se trocarmos os ciclos de lugar, nada muda nas informaes, de modo
que a mesma permutao poderia ser representada, por exemplo, por (4957)
(6) (3) (182). J trocar a ordem das pessoas dentro dos ciclos pode alterar
ou no a permutao, como vimos.
Fica claro agora que, quando procuramos as permutaes que no
possuem pontos fixos, estamos procurando quais as permutaes que no
apresentam ciclos de tamanho 1.
Para adquirir uma familiaridade com o problema, comecemos por
examinar como seria o problema com nmeros menores. Chamando de n
o nmero de pessoas, e de Kn o nmero de permutaes do conjunto
dessas pessoas, que no tm elementos fixos, ento a probabilidade de que
o sorteio d certo ser: pn = Kn/n!.
73

Para n = 1, a nica permutao que existe : 1 1, ou, na nossa


notao: (1), a qual tem ponto fixo. claro ento que K1 = 0 e p1 = 0.
Para n = 2, as duas permutaes so: (1) (2) e (12). S a segunda
catica; portanto: K2 = 1 e p2 = 1/2.
Para n = 3, existem 6 permutaes: (1)(2)(3), (1)(23), (2) (13),
(3) (12), (123) e (132). Dessas, s as duas ltimas no tm ciclos de
tamanho 1, isto , no tm pontos fixos. Logo, K3 = 2 e p3 = 1/3.
claro que no podemos contar dessa maneira para o caso n = 9, com
um total de mais de 300 mil permutaes. Vamos ento fazer um raciocnio
mais sutil, para esse caso. Imaginemos todas as permutaes caticas das
9 pessoas. Fixemos a ateno na pessoa de nmero 9. Em qualquer das 9!
permutaes, essa pessoa tem que estar em algum ciclo de tamanho maior
que 1 (lembre-se de que no h ponto fixo numa permutao catica!).
Chamemos ento de D9 o nmero de permutaes caticas (das 9 pessoas)
em que a pessoa 9 est num ciclo de tamanho 2, e de B9 o nmero de
permutaes caticas (das 9 pessoas) em que a pessoa 9 est num ciclo
de tamanho maior que 2. claro que K9 = B9 + D9.
Se tomarmos uma permutao catica em que 9 esteja num ciclo de
tamanho maior que 2 (por exemplo, (15) (3246) (798)) e suprimirmos
o 9, obteremos uma permutao catica das 8 pessoas restantes (no
exemplo anterior, obteramos: (15) (3246) (78)); por outro lado, o caminho
inverso ou seja, inserir o 9 nessa permutao catica das 8 primeiras
pessoas, para obter uma permutao catica das 9 originais pode ser
feito de 8 maneiras diferentes, como vemos no exemplo dado:
(195)(3246)(78), ou (159)(3246)(78), ou (15)(39246)(78), ou
(15)(32946)(78), ou (15)(32496)(78), ou (15)(32469)(78), ou
(15)(3246)(798), ou (15)(3246)(789)). Na realidade, o processo descrito
nesse caminho inverso consiste em substituir cada flecha a b por
a 9 b. No exemplo, fizemos isso, sucessivamente, com as flechas
1 5, 5 1, 3 2, 2 4, 4 6, 6 3, 7 8, 8 7, que so as
oito flechas da permutao. Portanto, a concluso que cada permutao
catica de 8 pessoas gera, por esse processo, 8 permutaes caticas de
9 pessoas nas quais a pessoa 9 est num ciclo de tamanho maior que 2,
ou seja: B9 = 8K8.
Se tomarmos agora uma permutao catica em que 9 esteja num
74

ciclo de tamanho igual a 2 (por exemplo, (178) (3426) (59)) e suprimirmos


o 9, obteremos no uma permutao catica das 8 pessoas restantes, e
sim uma permutao das 8 pessoas com um nico ponto fixo (no exemplo
anterior, obteramos: (178) (3426) (5)). Essa pode ser olhada como um
ponto fixo (no caso, o 5) justaposto a uma permutao catica das outras 7
pessoas. Como existem 8 candidatos a serem o ponto fixo, conclui-se que
cada permutao catica de 7 pessoas gerar, pelo processo de
acrescentar o 9 ao ponto fixo, 8 permutaes caticas de 9 pessoas
nas quais 9 est num ciclo de tamanho 2, ou seja: B9 = 8K7.
Como K9 = B9 + D9, segue que: K9 = 8K8 + 8K7.
O leitor pode agora repetir o mesmo raciocnio para n em vez de 9,
para concluir que:
Kn = (n 1)Kn1 + (n 1)Kn2.
Dividindo por n! e simplificando, passa-se s probabilidades que nos
interessam, obtendo:

1
1
pn = (1 ) pn 1 + pn 2 .
n
n

(*)

Essa uma frmula de recorrncia, que permite calcular pn, uma vez
que j saibamos as probabilidades anteriores pn1 e pn2 . Por exemplo:

1
1
2 1 1
1
p3 = (1 ) p2 + p1 = + 0 = .
3
3
3 2 3
3
1
1
3 1 1 1 3
p4 = (1 ) p3 + p2 = + = .
4
4
4 3 4 2 8

Continuando, encontram-se:

pn

0 = 0,00000

1/2 = 0,50000

1/3 = 0,33333

3/8 = 0,37500

11/30 = 0,36667

53/144 = 0,36806

e assim por diante.


75

Para obter uma frmula geral, observemos que a frmula (*) pode ser
escrita como:
pn pn-1 = (1/n)(pn-1 pn-2), ou ainda, chamando pn pn1 de dn , como:
dn = (l/n)dn1.
Observando ainda que d2 = p2 p1 = 1/2 0 = 1/2, tem-se, sucessivamente:

d2 =

1
1
1 1
1
1
1 1 1
; d3 = d 2 = = ; d 4 = d3 = = ; etc.
2!
3!
3! 2!
3!
4
4 3! 4!

De um modo geral: d n = (1) n

1
.
n!

Por fim, a relao pn pn1 = dn acarreta:


pn = ( p2 p1 ) + ( p3 p2 ) + K + ( pn pn 1 ) =

1 1 1
1
+ K + (1) n
2! 3! 4!
n!

que a frmula geral que resolve o problema.


Observando a tabela de valores de pn, o leitor vai reparar que esses
valores crescem (cada vez menos) quando n passa de mpar para par, e
diminuem (cada vez menos) quando n passa de par para mpar, sugerindo
que pn deva tender a se aproximar de um certo valor (entre 0,36667 e
0,36806), ora por excesso, ora por falta. Isso de fato verdade. Esse valor
l/e, em que e 2,71828 a clebre base dos logaritmos naturais. Se o
leitor tiver acesso a uma calculadora com a tecla ex, poder verificar que
l/e = e1 0,36788. A justificativa desse fato pode ser feita atravs da
frmula de Taylor para a funo exponencial, estudada em Clculo
Diferencial, segundo a qual: ex = 1 + (x/l!) + (x2/2!) + ... .
Em suma, pode-se dizer que a probabilidade de que o sorteio do amigo
oculto d certo oscila em torno de aproximadamente 37%
(conseqentemente 63% de no dar certo), estando j bem perto desse
valor a partir de 5, pessoas.
Baseado no artigo
O problema do amigo oculto
Jos Paulo Carneiro, RPM 28
Sobre o mesmo assunto, veja tambm
Amigo oculto, por C. G. Tamm Moreira, RPM 15 e
Uma pequena prola de Euler, por Geraldo Garbi, RPM 50
76

O princpio da casa dos pombos

A Anlise Combinatria, que poderia ser chamada


de arte de contar, inspira, freqentemente, temor ou
desagrado aos alunos do ensino mdio, s voltas com
problemas mecnicos envolvendo combinaes,
permutaes, arranjos, etc.
No entanto, trata-se de uma parte fascinante da
Matemtica que contm problemas de enunciado
extremamente simples, mas que exigem, por vezes, para
sua soluo, raciocnios penetrantes e engenhosos.
Grandes matemticos, como Euler, atacaram
problemas de Combinatria. Hoje, com o rpido
desenvolvimento da chamada Matemtica Finita,
principalmente devido ao uso dos computadores, a
Combinatria cresce rapidamente, atraindo a ateno de
muitos matemticos jovens e promissores.
Um dos princpios bsicos da Combinatria o
chamado princpio da casa dos pombos, ou ainda
princpio das gavetas de Dirichlet, que diz simplesmente:
Se forem dados n objetos, n > 2, a serem
colocados em, no mximo, (n 1) gavetas,
ento uma delas conter pelo menos dois
objetos.
Certamente poucos duvidaro da veracidade do
princpio. Para os mais cticos pode-se argumentar por
reduo ao absurdo. Se cada uma das gavetas contiver,
77

no mximo, um objeto, o nmero total de objetos colocados nelas ser, no


mximo, (n 1), o que uma contradio.
Uma aplicao trivial do princpio :

Exemplo 1
Dado um conjunto de 13 pessoas, pelo menos duas delas tero
aniversrios no mesmo ms.
No entanto, o princpio da casa dos pombos se presta a aplicaes mais
interessantes e significativas do que essa; de outra maneira, no valeria a
pena apresent-lo.

Exemplo 2
Escolha 101 nmeros quaisquer dentre os inteiros 1, 2,...,200. Mostre
que entre os nmeros escolhidos h dois nmeros tais que um deles divisvel
pelo outro.
Soluo
Em primeiro lugar, observe que qualquer inteiro n se escreve sob a
forma n = 2k b, sendo k um inteiro no negativo, e b um inteiro mpar.
Por exemplo, 36 = 22 9; 25 = 20 25; 16 = 24 1.
Assim, se n pertence ao conjunto {1, 2,..., 200}, n = 2k b e b um dos
nmeros mpares 1, 3, 5,..., 199. Ora, h 100 nmeros mpares no conjunto
{1, 2,..., 200}. Logo, quando escolhemos 101 nmeros desse conjunto,
dois deles tero suas partes mpares iguais, pelo princpio da casa dos
pombos; sejam n1 e n2 esses nmeros. Ento,
n1 = 2rb e n2 = 2sb.
Se r < s, ento n1 divide n2, pois

n2 2 s b
=
= 2 s r . Se s < r, ento
n1 2 r b

n2 dividir n1, o que conclui a demonstrao.

Exemplo 3
Mostre que em um conjunto de n (n > 2) pessoas h duas pessoas que
conhecem exatamente o mesmo nmero de pessoas do conjunto (obs.: se
A conhece B, B conhece A, ou seja, conhecer uma relao simtrica).
Soluo
Observe, em primeiro lugar, que qualquer das pessoas do conjunto
conhece no mnimo zero e no mximo (n 1) das outras pessoas.
78

Seja P = {A1, A2, ..., An} o conjunto das n pessoas. Dividiremos a


demonstrao em dois casos.
1o caso
Todas as pessoas conhecem pelo menos uma outra pessoa do conjunto.
Nesse caso, podemos colocar as pessoas em n 1 gavetas como
segue:
1a gaveta: pessoas de P que conhecem exatamente uma outra pessoa do
conjunto P.
2a gaveta: pessoas de P que conhecem exatamente duas outras pessoas
do conjunto P.

M
(n 1) gaveta: pessoas de P que conhecem exatamente outras (n 1)
pessoas do conjunto P.
a

Temos ento n pessoas a serem distribudas por (n 1) gavetas, e o


problema est resolvido, pois, pelo princpio da casa dos pombos, duas das
pessoas ocuparo a mesma gaveta.
2o caso
Uma das pessoas, que chamaremos de A1, conhece zero pessoa (ou
seja, no conhece ningum do conjunto).
Nesse caso, nenhuma pessoa de P conhece A1 . Portanto, ningum
conhece mais do que outras n 2 pessoas e novamente podemos colocar
as n pessoas em (n 1) gavetas como segue:
1a gaveta: pessoas de P que conhecem zero pessoa do conjunto P.
2a gaveta: pessoas de P que conhecem exatamente uma outra pessoa do
conjunto P.

M
(n 1) gaveta: pessoas de P que conhecem exatamente outras (n 2)
pessoas do conjunto P.
a

Novamente, pelo princpio da casa dos pombos, duas das pessoas

ocuparo a mesma gaveta.


O princpio da casa dos pombos pode ser reformulado da seguinte forma.
79

Teorema
Se m pombos ocupam n casas, ento pelo menos uma casa contm

m 1
n + 1 pombos ([x] o maior inteiro menor do que ou igual a x).

m 1
Demonstrao: Se cada casa contiver, no mximo,
pombos, ento
n
m 1 n(m 1)
m 1 < m , uma
o nmero mximo de pombos ser n

n
n
contradio, j que temos m pombos.
Ainda outra formulao possvel para o princpio da casa dos pombos
a seguinte:

Teorema
Sejam n gavetas e r um inteiro positivo dado. Coloquemos a1 objetos
na primeira gaveta, a2 objetos na segunda, e assim sucessivamente, at an
objetos na n-sima gaveta. Ento, se a mdia (a1 + a2 + ... + an )/n for
maior do que r 1, uma das n gavetas conter pelo menos r objetos.
Demonstrao: A demonstrao bem simples. Se todos os ai forem
menores do que r, ento
a1 < r 1; a2 < r 1; ...; an < r 1. Logo,
a1 + a2 + ... + an < nr n = n(r 1), que implica
(a1 + a2 + ... + an )/n < r 1, o que uma contradio.
Observao
O teorema anterior pode ser apresentado sem nenhuma referncia a objetos
e gavetas, mas to-somente como uma propriedade simples da mdia: se a
mdia dos nmeros naturais a1, a2, ..., an for maior do que r 1, ento
um deles dever ser maior do que ou igual a r. O princpio da casa dos
pombos pode ser deduzido desse ltimo teorema. Com efeito, se tivermos
n objetos para distribuir entre (n 1) gavetas, ento a mdia n /(n 1)
certamente ser maior do que 1. Logo, fazendo r = 2, teremos que uma
das gavetas deve conter pelo menos 2 objetos.

80

O teorema ainda pode ser usado para demonstrar o seguinte resultado,


que pode parecer surpreendente primeira vista.
Exemplo 4
So dados dois discos, A e B, cada um deles dividido em 200 setores
iguais. Os setores dos discos so pintados de branco ou de preto. Sabe-se
que no disco A h 100 setores brancos e 100 pretos, em ordem
desconhecida. O nmero de setores brancos de B arbitrrio e desconhecido.
Coloquemos o disco A sobre o disco B de modo que cada setor de A
fique exatamente sobre um setor de B (sempre que dissermos que o disco
A foi colocado sobre o disco B, fica convencionado que h essa coincidncia
de setores).
Mostre que possvel escolher a posio de A de maneira que existam
pelo menos 100 setores de A que tenham a mesma cor que os
correspondentes setores de B.
Soluo
Coloque A sobre B. Seja n1 o nmero de setores sobrepostos com
cores coincidentes.
Mantendo B fixo, gire A de um ngulo igual a um setor no sentido dos
ponteiros do relgio. Seja ento n2 o nmero de setores sobrepostos
coincidentes.
Continue com o processo, girando A sempre de um setor no sentido dos
ponteiros dos relgios e obtendo n3, n4, ..., n200.
ento verdade que o nmero total de coincidncias
n1 + n2 +... + n200 = (200 100) = 2 (l00)2.
Com efeito, fixado um setor do disco B (preto, por exemplo), como o
disco A tem exatamente 100 setores pretos, haver 100 posies em que
esse setor de B ter a mesma cor que o setor correspondente de A.
Assim, o nmero total de coincidncias ser o nmero de setores de B
(200) vezes 100 (o nmero de setores vezes o nmero de coincidncias por
setor).
Ento, pelo teorema, temos
(n1 + n2 +... + n200 )/ 200 = 100 > 100 1 (neste caso, r = 100).
81

Logo, pelo menos um dos ni deve ser maior ou igual a 100, ou seja,
para uma das posies o nmero de coincidncias de pelo menos 100.
Esperamos que os exemplos apresentados tenham dado uma idia de
como aplicar o princpio da casa dos pombos. Como Matemtica s se
aprende fazendo, propomos a seguir alguns exerccios sobre o assunto. Se
possvel, tente generalizar os enunciados e demonstrar suas generalizaes.
Exerccios
1. Mostre que, se do conjunto {1, 2,..., 2n} retirarmos (n + 1) nmeros ao
acaso, ento:
a) um deles dividir um outro.
b) dois dos nmeros sero primos entre si.
2. Escolha 5 pontos ao acaso sobre a superfcie de um quadrado de lado 2.
Mostre que pelo menos um dos segmentos que eles determinam tem
comprimento menor do que ou igual a 2 .
3. Em uma gaveta, h 12 meias brancas e 12 meias pretas. Quantas
meias devemos retirar, ao acaso, para termos certeza de obtermos um
par de meias da mesma cor?
4. Chame um ponto B = (x, y, z) de R3 de bom se todas as suas trs
coordenadas forem inteiras. Considere nove pontos bons de R3. Mostre
que o ponto mdio de algum dos segmentos que ligam esses pontos
bom.
5. Seja x um nmero real e n um inteiro positivo. Mostre que, entre os
nmeros x, 2x, 3x, ..., (n 1)x, existe um cuja distncia a algum inteiro
, no mximo, 1/n.
Baseado no artigo
Princpio da casa dos pombos
Joo Bosco Pitombeira, RPM 08

82

Probabilidade geomtrica:
os problemas dos ladrilhos,
do encontro e do macarro

Conde de Buffon, os ladrilhos e as agulhas


Georges Louis Leclerc, Conde de Buffon, nasceu em
7 de setembro de 1707, em Montbard, na Frana, e morreu
em 16 de abril de 1788, em Paris.
Nascido na aristocracia, estudou Medicina e Direito.
Mostrou interesse pela Matemtica, tendo descoberto
sozinho a Frmula do Binmio e mantido correspondncia
com Cramer sobre Mecnica, Geometria, Probabilidade,
Teoria dos Nmeros e Clculo Diferencial e Integral. Mas
era a Natureza a sua paixo. Dedicou-se principalmente
Histria Natural, tendo sido o maior responsvel pelo
crescimento do interesse pela Histria Natural na Europa,
no sculo XVIII.
No 4o volume do seu Suplemento Histria Natural,
publicado em 1777, tem 3 de suas 35 sees dedicadas ao
Clculo de Probabilidades. Uma delas Sur le jeu de
franc-carreau (Sobre o jogo do ladrilho), na qual Buffon
discute o jogo do ladrilho e apresenta o Problema da
Agulha, que no discutiremos aqui, uma vez que sua
soluo exige tcnicas de integrao (pode ser encontrado
na RPM 20). Foi o primeiro escrito sobre o que hoje se
conhece por Probabilidade Geomtrica: problemas de
probabilidades que tm espaos amostrais equivalentes a
pontos representados por figuras geomtricas. A
probabilidade de um determinado evento pode ser calculada
pela razo entre medidas geomtricas como comprimento,
rea ou volume.
83

O jogo do ladrilho
Era bastante jogado pelas crianas francesas no sculo XVIII. Uma
pequena moeda de raio R lanada ao acaso em um cho coberto por
ladrilhos quadrados de lado l (l > 2r). As crianas apostavam que a moeda
cairia inteiramente dentro de um ladrilho.
Buffon notou que a probabilidade de a moeda cair inteiramente dentro
de um ladrilho era a probabilidade de o centro da moeda cair dentro de um
quadrado de lado l 2r.
l-2r

r
favorvel

no favorvel

Essa probabilidade a razo entre as reas do quadrado e do ladrilho,


pois a probabilidade de o centro da moeda cair em uma regio proporcional
rea dessa regio. Portanto, a probabilidade de a moeda cair inteiramente
dentro de um ladrilho

( l 2r ) 2
l2

Um exemplo atual: considerando um piso formado por quadrados de


Paviflex de 30 cm de lado e um disco de raio 6 cm, a probabilidade de o
disco cair inteiramente dentro de um dos ladrilhos igual a

(30 12) 2
30

324
= 0, 36 ou 36%.
900

Poderamos tambm perguntar, nessa situao, qual o dimetro d do


disco que daria 60% de chances de vitria ao jogador:
que implica d = 6,77 cm.

84

(30 d ) 2
302

= 0, 60 , o

O problema do encontro
Duas pessoas decidiram se encontrar em um determinado local entre 11
e 12 horas. Combinou-se previamente que a primeira pessoa a chegar
esperar no mximo 15 minutos pela outra. Ache a probabilidade P de o
encontro acontecer, admitindo que cada uma das pessoas pode chegar, de
modo eqiprovvel, em qualquer instante entre 11 e 12 horas.
Podemos associar os instantes de chegada das duas pessoas, no intervalo
de 60 min, entre 11 e 12 horas, a um par (x,y) de [0, 60] x [0, 60]
representados por pontos em eixos ortogonais x e y em R2. Cada ponto
teria coordenadas x, y numericamente iguais quantidade de minutos dos
respectivos instantes de chegada, 11h e x min, 11h e y min , das duas pessoas.
De acordo com o enunciado, o encontro somente ter lugar se
y x 15, ou seja, y x + 15 e y x 15.
Essas duas inequaes definem a regio em cinza da figura.
y

Logo, se A a rea da
regio cinza, temos P = 602/A.

y = x + 15

60

y = x - 15

A = 602 2(45 x 45)/2 =


602 452 = 105 x 15 = 1 575
P = 1 575/3600 = 0,4375 ou
43,75%.

15
15

60

O problema do macarro
Em uma sala de aula distribuiu-se um espaguete para cada aluno, pedindo
a cada um que partisse o espaguete, ao acaso, em trs pedaos. Em seguida,
pediu-se que cada um verificasse se era possvel formar um tringulo com
os seus trs pedaos.
Colocou-se a pergunta: supondo que todas as possveis divises ocorram
de forma eqiprovvel, qual a probabilidade de se obter um tringulo?

85

O problema pode ser enunciado do seguinte modo:


Dividindo-se aleatoriamente um segmento em trs partes, qual a
probabilidade de que esses novos segmentos formem um tringulo?
Tomemos um segmento de reta AB de comprimento 1. Vamos dividi-lo
em trs partes: uma de comprimento x, outra de comprimento y e a
terceira, naturalmente, de comprimento 1 x y.
A

1-x-y

Cada forma de dividir o segmento unitrio fica


ento associada ao par ordenado (x, y) com x > 0,
y > 0 e x + y < 1. Isso corresponde no plano
cartesiano regio triangular da figura. Portanto,
cada forma de dividir um segmento em trs partes
est agora representada por um ponto interior ao
tringulo da figura.

Entretanto, no so todas as divises que formam tringulos. Um tringulo


existe se, e somente se, cada lado for menor que a soma dos outros dois.
Isso equivalente a dizer que, em um tringulo, cada lado menor que o
seu semipermetro, que no nosso caso igual a 1/2. Temos, portanto,
x < 1/2, y < 1/2 e 1 x y < 1/2 ou x + y > 1/2.
Logo, a regio favorvel o interior do tringulo
formado pelos pontos mdios dos lados do tringulo
inicial, que tem rea igual a 1/4 da rea do tringulo
grande, o que nos leva a concluir que a probabilidade
de que os trs segmentos formem um tringulo
0,25 ou 25%.

Baseado nos artigos


Determinao de probabilidades por mtodos geomtricos
Nelson Tunala, RPM 20
Probabilidade Geomtrica
Eduardo Wagner, RPM 34
O problema do jogo dos discos
Roberto R. Paterlini, RPM 48

86

Alguns problemas clssicos


sobre grafos

O conceito de grafo simples, porm frtil em


aplicaes e problemas atraentes. Ele j foi abordado,
nesta Revista, em pelo menos trs ocasies: no nmero 3,
quando o Prof. G. de La Penha descreveu o problema das
pontes de Knigsberg, no nmero 10 (implicitamente),
quando o Prof. J. B. Pitombeira tratou da questo de
determinar o nmero de regies em que n retas em
posio geral decompem o plano e, no nmero 11, quando
este mesmo autor estudou o problema de ligar gua, luz e
telefone em trs casas.
Creio que nossos leitores apreciaro uma anlise do
problema das pontes. E, para aproveitar o embalo,
ofereceremos solues diferentes para os outros dois
problemas acima mencionados. sempre instrutivo ter
diversas alternativas para resolver questes interessantes.
As setes pontes de Knigsberg
Imaginemos um rio, com duas margens A e B. No
rio, duas ilhas C e D. A ilha C est ligada a cada uma
das margens por duas pontes. Em cada margem, h
tambm uma ponte para a ilha D. A stima ponte liga as
ilhas entre si.
O problema das sete pontes de Knigsberg consiste
em achar um caminho, ao longo do qual um pedestre,
partindo de uma das margens ou de qualquer das ilhas,
percorra todas as pontes, sem passar mais de uma vez
por qualquer uma delas.
87

A
D

B
figura 1

Este problema foi resolvido, em 1735, pelo matemtico suo Leonhard


Euler. Ele fez a observao fundamental de que, para efeito da questo
proposta, as margens e as ilhas so como
A
se fossem pontos A, B, C, D. As pontes
so como arcos que tm esses pontos como
1
extremidades. Tudo se resume a analisar o
2
3
diagrama ao lado, onde os arcos ligam os
1 C
D
pontos, de acordo com a disposio das
4
pontes dada no enunciado do problema.
5
1

O desenho da figura 2, provavelmente,


B
o primeiro exemplo de um grafo a ocorrer
figura 2
como modelo matemtico para resolver um
problema, que agora se exprime assim:
partindo de um dos vrtices A, B, C, ou D, achar um caminho que
percorra todo o grafo sem passar mais de uma vez pelo mesmo arco.
De um modo geral, um grafo isto: um conjunto finito de pontos,
chamados os vrtices do grafo, e um conjunto finito de arcos, chamados as
arestas do grafo. As extremidades de cada aresta devem ser vrtices.
Alm disso, duas arestas quaisquer do grafo no podem ter pontos interiores
em comum: ou so disjuntas ou se tocam apenas numa ou em duas das
extremidades.
Euler chamou ateno para uma noo muito simples, porm crucial,
que a ordem de um vrtice do grafo. A ordem de um vrtice o nmero
de arcos que emanam dele. Por exemplo, no grafo das pontes de Knigsberg,
o vrtice C tem ordem 5, enquanto os demais vrtices A, B e D tm
todos ordem 3.

88

Um caminho num grafo uma seqncia finita de vrtices


= (A0, A1, ..., Ap) tal que, para cada i = 1, ..., p, Ai1 e Ai so
extremidades de uma aresta (juntamente com a escolha da aresta ligando
Ai1 e Ai, j que pode haver mais de uma aresta). Diz-se que o caminho
parte do vrtice A 0, percorre as arestas escolhidas e termina no
vrtice Ap.
Um caminho chama-se unicursal quando no percorre a mesma aresta
mais de uma vez. Um grafo G chama-se unicursal quando existe um
caminho unicursal que percorre todas as arestas de G. Observe-se que um
caminho unicursal pode passar vrias vezes pelo mesmo vrtice.
Toda vez que um caminho unicursal chegar a um vrtice, deve sair dele
por um arco diferente daquele por onde chegou. (A menos que esse vrtice
seja o fim do caminho.) Portanto, se um caminho unicursal percorrer todas
as arestas do grafo, os vrtices desse grafo, com exceo do incio e do fim
do caminho, devem ter todos um nmero par de arestas emanando deles,
isto , devem ter ordem par. O vrtice que serviu de incio e o que serviu de
fim para o caminho tm ordem mpar. Se o incio e o fim do caminho
coincidirem (isto , se o caminho for fechado), ento todos os vrtices do
grafo, sem exceo, tm ordem par.
Conclumos ento que se um grafo unicursal, ou todos os seus
vrtices tm ordem par ou exatamente dois vrtices tm ordem mpar.
No primeiro caso, todo caminho unicursal fechado. No segundo caso,
um caminho unicursal deve comear num dos vrtices de ordem mpar
e terminar no outro.
Segue-se, da, que o grafo da figura 2 no unicursal, pois seus quatro
vrtices tm todos ordem mpar. Fica, ento, resolvido o problemas das
sete pontes: impossvel percorr-las todas, sem passar duas vezes
por alguma ponte.
Observao: A cidade de Knigsberg ficava na Prssia, regio do leste da Alemanha.
Hoje, ela se chama Kaliningrado, pertence Rssia e j possvel percorrer todas
as suas pontes sem passar mais de uma vez por cada uma delas. que foi construda
uma nova ponte. A bem da verdade, devemos esclarecer tambm que, de fato, Euler
no menciona a ilha D. No seu mapa h uma pennsula D, a partir da qual o rio
Pregel se bifurca, depois de passar pela ilha C (que se chama Kneiphof). Mas
claro que o problema fica bem mais fcil de enunciar se substituirmos a pennsula
por uma ilha, o que no faz diferena alguma do nosso ponto de vista.
89

Voltando s antigas pontes de Knigsberg, podemos trocar o ponto de


vista terrestre pelo aqutico e indagar: seria possvel a um barqueiro (ou
nadador) no rio passar por baixo das sete pontes sem passar mais de
uma vez sob nenhuma delas? Esta questo, ao que parece, nunca foi
considerada por Euler. O leitor interessado pode, entretanto, tomar seu lpis
e papel. Se tiver um pouco de pacincia vai conseguir uma rota adequada
para o barqueiro, como por exemplo a da figura 3.

A
D

B
figura 3

Tendo sido bem-sucedido em sua tentativa, o leitor pode indagar se foi


apenas uma questo de sorte ou se existe uma razo matemtica que permita
ao barqueiro cruzar as pontes, do mesmo modo que probe o pedestre de
percorr-las. Seria possvel reformular este segundo problema em termos
de grafos, como fizemos com o primeiro?
Existe sim, a razo matemtica. , sim, possvel enquadrar o barqueiro
no contexto dos grafos. Vejamos como.
Um grafo no plano divide esse plano em regies. Por exemplo, o grafo
da figura 2 determina cinco regies. A regio exterior, que naquela figura
indicamos com o algarismo 1, e mais quatro regies limitadas, as quais
indicamos com os algarismos 2, 3, 4 e 5 na figura.
Usando essas regies, pode-se, a partir de um grafo G, construir um
novo grafo G*, chamado o dual de G. Os vrtices de G* so tantos
quantas so as regies de G. Dois vrtices do novo grafo G* estaro
ligados por tantas arestas quantas forem as arestas adjacentes s regies
correspondentes.
Por exemplo, seja G o grafo da figura 2. Para formar o grafo dual G*
tomamos cinco vrtices, correspondentes s cinco regies 1, 2, 3, 4 e 5. A
regio 1, no grafo G, adjacente a todas as outras. Logo devemos traar
90

arestas em G* ligando o vrtice 1 a todos os outros quatro. As regies, 2


e 3, 3 e 4, 4 e 5 so adjacentes. Ento devem existir arestas em G*
ligando os vrtices com esses nmeros. Por outro lado, no h outros pares
de regies adjacentes. Logo no h outras arestas em G*. O grafo G*,
dual daquele na figura 2, est desenhado na figura 4.
4

figura 4

Note-se que, no grafo G*, apenas dois vrtices (3 e 4) tm ordem


mpar (ambos tm ordem 3). Os vrtices 2 e 5 tm ordem 2 e o vrtice
1 tem ordem 4. Portanto G* cumpre a condio necessria para ser
unicursal. Pode-se demonstrar que essa condio tambm suficiente para
um grafo conexo. Mais ainda, um caminho unicursal no grafo G* deve
comear num dos vrtices 3 ou 4 e terminar no outro. Isso justifica
matematicamente por que um barqueiro pode passar por baixo das sete
pontes de Knigsberg sem repetir nenhuma delas, mas um pedestre no
pode fazer seu passeio unicursal ao longo dessas pontes. que o grafo G
no unicursal, enquanto seu dual G* . Alm disso, o percurso do
barqueiro deve comear ao lado da ponte que liga as duas ilhas e terminar
do outro lado dessa mesma ponte.
De um modo geral, juntamente com o problema de percorrer todas as
arestas de um grafo plano, pode-se sempre considerar o problema dual de,
partindo de uma das regies por ele determinadas, descrever um caminho
que corte todas as arestas uma nica vez. Isto corresponde a indagar se o
grafo dual unicursal.
O leitor convidado a desenhar diferentes grafos e examinar, para cada
um deles, a possibilidade de traar um caminho unicursal, no grafo ou no
seu dual.

91

Em quantas regies n retas dividem o plano?


A pergunta formulada acima no admite uma resposta nica. Com 3
retas distintas, por exemplo, podemos dividir o plano em 4, 6 ou 7 regies,
conforme se v na figura 5.

4 regies

1
6

2
5

6 regies

1
5

2
1

2
3
4

6 regies

7
5

3
4

7 regies

figura 5

A formulao correta do problema, para que ele tenha uma resposta


nica, a seguinte: qual o nmero mximo de regies em que n retas
dividem o plano? Evidentemente, o nmero mximo de regies ocorre
quando essas retas esto situadas de modo a terem o nmero mximo possvel
de pontos de interseco. Esse nmero mximo acontece quando:
1o) Entre as retas dadas no h paralelas;
2o) Nenhum ponto a interseco de mais de duas retas dadas.
Neste caso, diz-se que as n retas dadas esto em posio geral.
Dadas n retas em posio geral, para determinar o nmero R de regies
em que elas dividem o plano, procederemos da seguinte maneira. Em primeiro
lugar, traamos um crculo to grande que contenha em seu interior todos
os pontos de interseco das n retas. Os requisitos 1o) e 2o) acima
asseguram que, para cada duas das n retas dadas, h um ponto de
interseco e vice-versa. Logo, o nmero dos pontos de interseco, todos

n
situados no interior do nosso crculo, = n(n 1) / 2.
2
Na figura 6, temos quatro retas em posio geral. Seus 6 pontos de
interseco esto no interior do crculo ali traado.
Agora consideremos o grafo plano G, obtido quando desprezamos as
partes das retas que ficam no exterior do crculo que traamos.
92

figura 6

Os vrtices de G so as interseces das n retas duas a duas e mais os


2n pontos em que essas n retas intersectam a circunferncia: ao todo,
temos V = 2n + n(n l)/2 vrtices no grafo G.
As arestas de G so os 2n arcos de crculo correspondentes e mais os
segmentos de reta interiores ao crculo. Sobre cada uma das n retas h
n + 1 vrtices, a saber: os n 1 pontos de interseco dessa reta com as
n 1 outras e os 2 pontos em que ela corta a circunferncia. Logo, temos
n segmentos, isto , n arestas do grafo G, sobre cada uma das n retas
dadas. Ao todo, so n2 arestas de G interiores ao crculo, com o total de
A = n2 + 2n arestas em G.
O nmero R de regies em que as n retas dadas dividem o plano
igual ao nmero de regies determinadas pelo grafo G menos uma, que
a regio exterior ao crculo. A frmula de Euler diz que se um grafo com V
vrtices e A arestas decompe o plano em F regies, tem-se
V A + F = 2.
Pela frmula de Euler temos, portanto, V A + R = 1, ou seja,
2n + n(n 1)/2 n 2 2n + R = 1, donde R = 1 + n(n +1)/2.
Equivalentemente: R = 1 + n +

n(n 1) n n n
= + + .
2
0 1 2

gua, luz e telefone


Um problema muito popular, desde meus tempos de ginsio, consiste em
propor que se ligue, em trs casas, gua, luz e telefone, a partir de 3 centrais
diferentes. Casas, centrais, e ligaes esto no mesmo plano. No se permite
que as ligaes se cruzem.

93

No possvel fazer isto. Uma demonstrao dessa impossibilidade foi


apresentada no nmero 11 da RPM, usando a frmula de Euler.
No que se segue, daremos uma demonstrao diferente do mesmo
resultado, sem fazer uso daquela frmula.
Representaremos as centrais de gua, luz e telefone pelas letras A, L,
T e as trs casas por pontos X, Y e Z. Comecemos ligando gua e luz s
casas X e Y. Obteremos um quadriltero XAYL, cujos lados podem
ser curvilneos.
A central telefnica T pode estar dentro ou fora deste quadriltero. Isto
no far diferena alguma mas, para fixar as idias, suponhamos que esteja
fora, como na figura 7.
T
X

3
A

Y
3

figura 7

Liguemos o telefone nas casas X e Y. Ficamos com dois quadrilteros


adjacentes XAYL e XLYT, os quais decompem o plano em trs regies,
que designamos por 1, 2 e 3. (As regies 1 e 2 so interiores aos
quadrilteros, enquanto a regio 3 exterior.)
A terceira casa, Z, dever estar numa dessas trs regies. Examinemos
cada uma das possibilidades. Se Z estiver na regio 1, poderemos ligarlhe gua e luz porm no telefone. Se estiver na regio 2, ficar com luz e
telefone, mas sem gua. Finalmente, se Z estiver na regio 3, poder ter
gua e telefone, mas no ter luz. Portanto, as nove ligaes no podem
ser todas feitas sem que se cruzem, e o problema est resolvido.
Baseado no artigo Alguns problemas clssicos sobre grafos
Elon Lages Lima, RPM 12

94

Srie harmnica

Introduo
O objetivo deste artigo o de fazer uma apresentao
simples da chamada srie harmnica, que possui
propriedades muito interessantes.
Um pouco de Histria
As sries infinitas so conhecidas desde a antiguidade,
e a primeira a ocorrer na Histria da Matemtica uma
srie geomtrica de razo 1/4, que intervm no clculo da
rea da parbola feito por Arquimedes.
Depois da ocorrncia de uma srie geomtrica num
trabalho de Arquimedes, as sries infinitas s voltaram a
aparecer na Matemtica cerca de 1500 anos mais tarde,
no sculo XIV. Nessa poca havia um grupo de
matemticos na Universidade de Oxford que estudava a
cinemtica, ou fenmeno do movimento; e, ao que parece,
foi esse estudo que levou reconsiderao das sries
infinitas.
Ao lado dos pesquisadores de Oxford, havia tambm
pesquisadores em outros centros. Na Universidade de
Paris, em particular, havia um professor chamado Nicole
Oresme (1325-1382), um destacado intelectual em vrios
ramos do conhecimento, como Filosofia, Matemtica,
Astronomia, Cincias Fsicas e Naturais. Alm de professor
universitrio, Oresme era conselheiro do rei,
principalmente na rea de finanas pblicas; e nessa
funo revelou-se um homem de larga viso,
95

recomendando medidas monetrias que tiveram grande sucesso na prtica.


Ao lado de tudo isso, Oresme foi tambm bispo de Lisieux.
Um dos trabalhos mais notveis de Oresme sobre as sries infinitas est
ligado srie harmnica.
Antes, porm, de falar da srie harmnica, temos de explicar o que
significa dizer que uma srie convergente ou divergente.
A idia de srie infinita aparece na Matemtica quando imaginamos a
operao de somar parcelas sucessivamente sem que essa operao termine
aps um nmero finito de parcelas somadas. Deixando de lado qualquer
preocupao com a rigorizao desse conceito, vamos examinar algumas
sries infinitas simples. Por exemplo,
1+

1 1 1 1
1
1
+ + + +
+
+K
2 4 8 16 32 64

Trata-se de uma progresso geomtrica infinita de razo


de seus termos dada por S =

1
e a soma
2

= 2.
1
2
Sries que tm soma finita so chamadas de sries convergentes. Mas
fcil imaginar sries que no sejam convergentes. Por exemplo, claro
que as sries

1 + 2 + 3 + 4 + 5 + ... ,

2 + 4 + 6 + 8 + . . .,

1 + 2 + 1 + 2 + 1 + 2...,

no so convergentes; elas so ditas divergentes. Um exemplo menos

1 2 3 4
n
+ + + +K +
+ K . Para
2 3 4 5
n +1
ver que essa srie diverge, basta notar que todos os seus termos, a partir do
segundo, so maiores do que 1/2.
trivial de srie divergente dado por

A srie harmnica
A srie harmnica uma srie muito simples, dada por

n =1 + 2 + 3 + 4 + 5 + K
n =1

96

Como se v, os termos da srie harmnica esto decrescendo para zero.


Mas ser que, quando o termo geral de uma srie tende a zero, ela converge?
Se for assim e primeira vista parece que , ento a srie harmnica
deve ser convergente.
Vamos investigar. Aps a soma de um grande nmero de termos da
srie harmnica, quando chegarmos a n = 1020, n= 1030, n = 10100, etc.,
estaremos somando to pouco que teremos a impresso de que a soma de
todos os termos da srie infinita realmente um nmero finito. Alis, hoje
em dia, com a ajuda do computador, podemos at fazer clculos
experimentais interessantes.
Vamos supor que fssemos capazes de somar cada termo da srie em
um segundo de tempo. Como um ano tem aproximadamente
365,25 x 24 x 60 x 60 = 31 557 600 segundos,
nesse perodo de tempo seramos capazes de somar a srie at
n = 31 557 600, obtendo para a soma um valor pouco superior a 17; em 10
anos a soma chegaria a pouco mais de 20; em 100 anos, a pouco mais de
22. Como se v, somas parciais de termos da srie harmnica jamais nos
levariam a suspeitar que ela diverge. Pelo contrrio, essas somas s nos
levam a pensar que a srie seja convergente.
Isso, todavia, falso! Embora surpreendente, esse resultado pode ser
facilmente demonstrado. Para isso agrupamos os termos da srie assim:
1 1 1 1
+ + + +K
2 3 4 5
1 1 1
1 1 1 1
1 1
1
=1+ + ( + ) + ( + + + ) + ( + +K ) +K
2 3 4
5 6 7 8
9 10
16
1+

Observe agora que a soma dentro de cada parntese sempre maior do


que 1/2. Veja:
1 1 1 1 1
+ > + = ;
3 4 4 4 2
1 1 1 1 1 1 1 1 1
+ + + > + + + = ;
5 6 7 8 8 8 8 8 2
1 1
1 1
1 1 1
+ +K + > + +K + = ,
16 2
9 10
16 16 16

e assim por diante.


97

Ento,

n =1 + 2 + 3 + 4 + 5 + K > 1 + 2 + 2 + 2 + K = ,

o que

n =1

prova que a srie mesmo divergente.


A demonstrao de que a srie harmnica diverge, feita pela primeira
vez por Oresme, mostra como decisivo o papel do raciocnio lgico para
estabelecer uma verdade que jamais seria descoberta de outra maneira.
De fato, como vimos acima, mesmo somando os termos da srie durante
um sculo (se isso fosse possvel), no chegaramos a um resultado que nos
desse qualquer indcio de que a srie seria divergente...
Para terminar, vamos fazer mais um exerccio de imaginao. Hoje em
dia temos computadores muito rpidos, e a tecnologia est produzindo
mquinas cada vez mais rpidas. Mas isso tem um limite, pois, como
sabemos, nenhum sinal fsico pode ser transmitido com velocidade superior
da luz. Portanto, nenhum computador poder efetuar uma soma em tempo
inferior a 10-23 segundos, que o tempo gasto pela luz para percorrer uma
distncia igual ao dimetro de um eltron. Pois bem, com tal computador,
em um ano, mil anos e um bilho de anos, respectivamente, o nmero de
termos que poderamos somar seria
315576 x 1025,

315576 x 1028

315576 x 1034.

Veja os resultados aproximados que obteramos para a soma da srie


harmnica, em cada um desses casos, respectivamente:
70,804; 77,718 e 91,5273 .
Imagine, finalmente, que esse computador estivesse ligado desde a origem
do universo, h 16 bilhes de anos. Ele estaria hoje obtendo o valor
aproximado de 94,2999 para a soma da srie harmnica, um nmero ainda
muito pequeno... O leitor tem toda razo em perguntar:
Como se chega ao nmero 94,2999, se o (idealizado) computador
mais rpido que se possa construir deveria ficar ligado durante 16 bilhes
de anos?
Sim, no h como fazer essa soma diretamente, mas existem mtodos
que permitem substituir a soma Sn dos n primeiros termos da srie por
uma expresso matemtica que aproxima Sn e que pode ser calculada
numericamente, o que, no entanto, requer conhecimentos de Clculo Integral.

98

Alergia pelo nmero 7


Imaginem um matemtico alrgico ao nmero 7 que decidisse eliminar
da srie harmnica todas as fraes que contivessem o algarismo 7. A
nova srie ficaria assim:

1 1 1 1 1 1 1 1 1
1 1 1 1
+ + + + + + + + +K + + + + +K
1 2 3 4 5 6 8 9 10
15 16 18 19
Como todos os demais algarismos podero ser usados, salvo, to-somente,
o 7, era de se esperar que a nova srie tambm divergisse. Mas, vejam s,
a srie acima converge e a sua soma no chega a oitenta! S provando
para acreditar:
* Cada uma das 8 primeiras fraes, de 1 at 1 , menor ou igual a 1. A
1
9
soma dessas fraes menor do que 8.

1
, menor ou igual
* Cada uma das 8 x 9 fraes seguintes, de 1 at
99
10
1
a 1 . A soma dessas fraes menor do que 8 9 .
10
10
* Cada uma das 8 x 9 x 9 fraes seguintes, de
ou igual a

1
1
at
, menor
100
999

1
1
. A soma dessas fraes menor do que 8 92 2 .
100
10

E assim, sucessivamente, a soma dos termos da srie ser menor do


que 8 +

8 9 8 92 8 93
8
+
+
+K +
= 80.
2
3
9
10
10
10
1
10

No incrvel?
Baseado nos artigos
As sries infinitas
Geraldo vila, RPM 30
Alergia pelo nmero 7
Renate Watanabe, RPM 31
99

O que tem mais:


racionais ou naturais?

Cantor e a Teoria dos Conjuntos


A reforma do ensino da Matemtica de 50 anos atrs
introduziu a utilizao de conjuntos no ensino bsico, mas
apenas a parte referente notao e linguagem de
conjuntos, nada de substancial sobre a verdadeira Teoria
dos Conjuntos. Em conseqncia, no apenas os alunos,
mas tambm muitos professores so pouco informados
sobre a importncia desse ramo de estudos, da a razo
de tratarmos aqui de alguns poucos aspectos interessantes
dessa disciplina.
O criador da Teoria dos Conjuntos foi o matemtico
alemo Georg Cantor (1845-1918), que foi professor na
Universidade de Halle, onde iniciou uma srie de pesquisas
sobre as chamadas sries trigonomtricas. Essas sries
ocuparam a ateno dos mais eminentes matemticos
durante todo o sculo XIX; e seu estudo, pelos muitos
desdobramentos e ramificaes que teve, foi, em verdade,
o impulso mais significativo para o progresso da Anlise
Matemtica durante a maior parte do sculo. Atravs de
suas investigaes nesse domnio, Cantor foi levado a
estudar os conjuntos de pontos de descontinuidade das
funes que considerava, logo chegando a estudar
conjuntos infinitos de pontos de descontinuidade. Da ele
passou naturalmente a estudar conjuntos em si, sem
referncia a funes. Assim nascia a Teoria dos
Conjuntos.

100

Conjuntos enumerveis
Um dos primeiros fatos surpreendentes que surgem na considerao de
conjuntos infinitos diz respeito possibilidade de haver uma equivalncia
entre um conjunto e um seu subconjunto prprio. Isso pode ser visto
facilmente atravs da seguinte correspondncia (restrita a nmeros positivos,
por simplicidade):
1 2 3 4 5 6 7 8 9 10 K
b b b b b b b b b b
2 4 6 8 10 12 14 16 18 20 K

Existe aqui uma correspondncia biunvoca entre elementos dos dois


conjuntos (n 2n) de tal sorte que a cada elemento de cada conjunto
corresponde um nico elemento do outro. Segundo Cantor, dois conjuntos
so equivalentes, ou tm a mesma cardinalidade, quando possvel
estabelecer entre eles uma tal correspondncia. No caso de conjuntos finitos,
serem equivalentes corresponde a terem o mesmo nmero de elementos,
de sorte que o conceito de equivalncia ou cardinalidade uma extenso, a
conjuntos infinitos, da noo de nmero de elementos de um conjunto.
Cantor passou a chamar de enumervel a todo conjunto que tem a
mesma cardinalidade do conjunto dos nmeros naturais 1, 2, 3, 4, ...Vamos
mostrar que os nmeros racionais tambm formam um conjunto enumervel.
Por simplicidade restringimo-nos aos racionais positivos, que distribumos
em vrios grupos, cada grupo contendo as fraes cujos numerador e

1 2 3 4
, , ,
o grupo de
4 3 4 1
todas as fraes cujos termos tm soma 5. Vamos fazer uma lista de todos
esses grupos, comeando com aquele cuja soma dos termos das fraes
2 (e que s contm a frao 1/1); depois o grupo das fraes 1/2 e 2/1,
cuja soma dos termos 3; e assim por diante, sucessivamente. Ao mesmo
tempo, riscamos as fraes que representam o mesmo nmero j
representado por fraes que apareceram antes. Eis a lista:

denominador tm a mesma soma; por exemplo,

1
;
1
1 2 3 4
, , , ;
4 3 2 1

1 2
, ;
2 1

1 2 3
, ,
3 2 1
1 2 3 4 5
, , , , ;
5 4 3 2 1
101

1 2 3 4 5 6
, , , , , ;
6 5 4 3 2 1
1 2 3 4 5 6 7
, , , , , , ;
7 6 5 4 3 2 1
1 2 3 4 5 6 7 8
, , , , , , , ;
8 7 6 5 4 3 2 1
claro que esse procedimento resulta numa lista de todos os nmeros
racionais. Basta agora enumer-los na ordem em que aparecem, isto ,

r1 = 1,
r5 = 3,
r9 = 4,

r2 = 1 / 2, r3 = 2,
r6 = 1 / 4, r7 = 2 / 3,
r10 = 1 / 5, r11 = 5,

r4 = 1 / 3,
r8 = 3 / 2,
r12 = 1 / 6, etc.

Dessa maneira obtemos uma correspondncia biunvoca entre o conjunto


dos nmeros racionais (positivos) e dos nmeros naturais, que tambm
podemos expressar assim:
1 2 3 4 5 6
7
8 9 10 K
b b b b b b
b
b b b
2 1/ 2 2 1/ 3 3 1/ 4 2 / 3 3 / 2 4 1/ 5 K

Isso mostra que os nmeros racionais formam, de fato, um conjunto


enumervel.
Conjuntos no-enumerveis
Se todos os conjuntos infinitos fossem enumerveis, tendo, pois, a mesma
cardinalidade, esse conceito no teria utilidade. O primeiro grande mrito
de Cantor foi a descoberta de que os nmeros reais no so enumerveis.
O leitor interessado encontra a demonstrao desse fato, por exemplo, na
RPM 4. Com essa descoberta, Cantor estabeleceu um fato muito
surpreendente, qual seja, o de que existem pelo menos dois tipos diferentes
de infinito: o do conjunto dos nmeros naturais e o do conjunto dos nmeros
reais. Cantor provou outro fato no menos perturbador: o de que, dado um
conjunto qualquer, sempre possvel construir outro conjunto maior ainda,
isto , cuja cardinalidade maior que a do conjunto dado. Ele obteve
assim um modo de construir toda uma infinidade de conjuntos infinitos com
cardinalidades diferentes, e ordenou os conjuntos infinitos de acordo com
102

sua cardinalidade, do mesmo modo que se ordenam os conjuntos finitos de


acordo com o nmero de seus elementos, ou seja, sua cardinalidade. Surgia
assim a teoria dos nmeros transfinitos.
Conseqncias do trabalho de Cantor
As descobertas de Cantor tiveram grande impacto no mundo matemtico
de fins do sculo passado e comeo do presente sculo. Para bem apreciar
o que ento acontecia, bom lembrar que desde o incio do sculo XIX era
crescente a preocupao com o rigor, primeiro na Anlise Matemtica,
porm mais tarde tambm na Geometria, depois das descobertas das
geometrias no euclidianas. A partir de 1870, quando Cantor iniciava sua
vida profissional, as atividades de pesquisa na rea de axiomatizao e
fundamentos intensificavam-se rapidamente. E a Teoria dos Conjuntos, que
ento se desenvolvia, revelou-se muito adequada para ser o fundamento de
toda a Matemtica.
H uma outra razo por que a Teoria dos Conjuntos importante em
Matemtica, fora da rea dos fundamentos propriamente dita. que desde
os tempos de Cantor muitas disciplinas matemticas novas surgiram e se
desenvolveram extensamente, como a Topologia, a lgebra Abstrata, a
Teoria da Medida e Integrao, a Teoria da Probabilidade, a Anlise
Funcional e outras mais. E em todas essas disciplinas que, ao contrrio de
estanques e separadas, no mais das vezes se entrelaam atravs de fronteiras
indistinguveis a linguagem, a notao e os resultados da Teoria dos
Conjuntos se revelaram instrumento natural de trabalho, a ponto de ser
impossvel conceber o desenvolvimento de toda essa Matemtica sem a
Teoria dos Conjuntos. Tentando uma analogia, diramos que a Teoria dos
Conjuntos aqui to necessria e indispensvel como a notao literal
necessria e indispensvel lgebra Elementar.
Para concluir queremos deixar claro que a Teoria dos Conjuntos uma
disciplina cuja importncia difcil exagerar, no s para a Matemtica,
mas para o conhecimento humano de um modo geral. Ela no importante,
isto sim, para o ensino bsico da Matemtica, onde somente um pouco de
notao e linguagem de conjuntos suficiente.
Baseado no artigo
A Teoria dos Conjuntos e o ensino de Matemtica
Geraldo vila, RPM 4
103

Problemas
1. Uma calculadora cientfica com diversos circuitos danificados s est
fazendo adies, subtraes, multiplicaes, divises e calculando as
funes trigonomtricas seno e cosseno e suas inversas. Como podemos
obter a raiz quadrada de um nmero positivo com essa calculadora usando
um nmero finito de operaes?
2. (O teste da diagonal) Dois retngulos ABCD e ABCD so sobrepostos
como na figura. Prove que os retngulos so semelhantes se e s se as
diagonais AC e AC esto na mesma reta. Logo, para verificar se dois
retngulos so semelhantes, basta coloc-los como na figura e verificar
se as diagonais AC e AC esto na mesma reta.
C
D
D
A=A

C
C
B

3. Sabe-se que o nmero de 7 algarismos 21358ab, em que a o dgito das


dezenas e b o das unidades, divisvel por 99. Determine a e b.
4. As medidas dos lados de um tringulo retngulo (numa mesma unidade)
podem ser nmeros primos?
5. Um trem atravessa uma ponte de 171 m em 27 segundos. Determine a
velocidade e o comprimento do comboio se o tempo de passar um
pedestre, que anda em sentido contrrio, com a velocidade de 1 m/s,
de 9 segundos.
6. Dados dois espelhos planos paralelos, considere dois pontos A e B entre
eles. Determine a trajetria que deve percorrer um raio de luz partindo
de A para atingir o ponto B aps ter sido refletido 3 vezes num espelho e
2 vezes no outro. Admite-se que o ngulo de incidncia seja igual ao de
reflexo.

104

7. Prove que todo tringulo com duas bissetrizes iguais issceles.


8. Prove que vale a seguinte desigualdade
log56 + log67 + log78 + log85 > 4.

9. Sabe-se que cada uma dentre as pessoas A, B e C diz a verdade em


qualquer situao, com probabilidade 1/3. Suponha que A faa uma
afirmao e que C diz que B diz que A falou a verdade. Qual a
probabilidade de que A realmente tenha falado a verdade?
10. Prove que
cos

2
8
2
14
8
14
3
cos + cos cos
+ cos cos
= .
9
9
9
9
9
9
4

11. a) Dada uma equao do segundo grau, com coeficientes inteiros, mostre
que o seu discriminante no pode ser igual a 23.
b) Para quantos valores reais do nmero a equao x2 +ax +6a = 0
possui somente razes inteiras?
12. Prove que, se sen(2x + y) = 5 sen y, ento tg ( x + y ) =

3
tg x.
2

13. Pelo ponto mdio M do lado BC de um quadriltero ABCD, traar a


reta que divide esse quadriltero em duas partes de reas iguais.
14. Uma urna contm n bolas numeradas de 1 a n. Bolas so retiradas
dessa urna sucessivamente, sem reposio, at que pela primeira vez
aparea um nmero maior que todos os anteriores. Caso isso no
acontea, o processo prossegue at que se esgotem as bolas da urna.
Para k = 2, ..., n, determine a probabilidade de que o processo pare na
k-sima retirada.
15. Numa circunferncia de raio R fixe dois pontos B e C. Mostre que o
lugar geomtrico dos baricentros dos tringulos ABC, onde A um
ponto qualquer dessa circunferncia, uma outra circunferncia de
raio R/3 que corta BC em trs segmentos congruentes.
16. Determine as solues inteiras e positivas da equao.
x3 y3 = 602
(Sugesto: fatore x3 y3 e 602).

105

17. Se dois tringulos tm dois ngulos respectivamente iguais e dois ngulos

respectivamente suplementares, mostre que os lados opostos aos


ngulos iguais so proporcionais aos lados opostos aos ngulos
suplementares.
18. Considere o conjunto A de todas as combinaes simples de 10 elementos
em grupos de 5. Duas combinaes distintas so escolhidas ao acaso
no conjunto A. Determine as probabilidades de que elas:
a) no tenham nenhum elemento em comum;
b) tenham exatamente 4 elementos em comum.
19. Num icosaedro regular de aresta a, cada vrtice est ligado a 5 outros
vrtices formando uma pirmide pentagonal. Calcule a altura dessa
pirmide.
20. Dados dois pontos A e B do plano e uma constante
o lugar geomtrico dos pontos P do plano tais que

m
> 0, determinar
n
PA m
= .
PB n

21. Encontre todos os nmeros naturais de dois dgitos tais que sua soma
com o nmero formado pelos mesmos dgitos em ordem contrria resulta
um quadrado perfeito.
22. Considere em um plano as retas paralelas a, b, c distintas duas a
duas. Mostre que existem tringulos equilteros cujos vrtices A, B,
C so pontos das retas a, b, c respectivamente.
23. Use um argumento combinatrio para determinar o valor de
2

n
n n
+ +L +
0 1
n
sendo n um inteiro maior ou igual a 1.

24. O produto de 3 nmeros pares e consecutivos 88 XXXXX 2, onde


cada X representa um algarismo que falta. Determine esses 5
algarismos.
25. Mostre que, quaisquer que sejam os nmeros inteiros a, b, c, d, e, a
equao
106

x7 + 2x6 + 3x5 + ax4 +bx3 + cx2 + dx + e = 0


no pode ter todas as razes reais.
26. Supondo que o polinmio
P(x) = x100 600x99 + a98x98 + ... + a1x + a0
tenha 100 razes reais e que P(7) > 1, mostre que existe pelo menos
uma raiz maior do que 7.
27. Prove que um pentgono com os cinco lados congruentes e trs ngulos
congruentes regular.
28. Mostre que, se a, b, c so nmeros inteiros mpares, ento a equao
ax2 + bx + c = 0 no tem razes racionais.
29. Suponha que cada ponto de um plano seja pintado de uma cor escolhida
entre trs cores dadas. Prove que existem dois pontos de mesma cor
cuja distncia k, sendo k > 0 um nmero real dado.
30. Considere, num plano, uma infinidade de pontos. Sabendo-se que a
distncia entre dois quaisquer desses pontos um nmero inteiro, mostre
que eles so colineares.

107

...probleminhas
1. Um homem entra numa livraria, compra Pequenos Golpes, que custa
20 reais, e paga com uma nota de 100 reais. Sem troco, o livreiro vai at
a banca de jornais e troca a nota de 100 por 10 notas de 10 reais. O
comprador leva o livro e 8 notas de 10 reais. Em seguida entra o jornaleiro
dizendo que a nota de 100 reais falsa. O livreiro troca a nota falsa por
outra de 100, verdadeira. Sem o dinheiro do troco, sem o livro e sem a
nota que deu ao jornaleiro, qual foi, afinal, o prejuzo do livreiro?
2. Uma pessoa, escrevendo a sucesso dos nmeros naturais (comeando
pelo zero), interrompeu seu trabalho em um certo nmero. Qual esse
nmero se, at parar, a pessoa escreveu 7350 algarismos?

=
3. Escreva nas casas vazias algarismos de 1 a 8
de modo que as igualdades se verifiquem, no
sentido das flechas.

=
4. Qual o maior nmero que se pode escrever usando nica e
exclusivamente quatro vezes o algarismo 2?
5. Construa trs cercas quadradas de modo que
todas as nove ovelhas fiquem presas e
separadas.

6. Preencha os quadrados com nmeros distintos


de 0 a 9 de modo que os nmeros que se lem
nas linhas, colunas e diagonais sejam mltiplos
de 11.

108

7. Uma pessoa ctica quanto s boas intenes da humanidade afirma que


70% dos homens so desonestos, 70% so intolerantes e 70% so
violentos. Se ela estiver certa, numa amostra perfeita de 100 homens,
qual o nmero mnimo de pessoas simultaneamente desonestas,
intolerantes e violentas?
2
8. Complete a estrela mgica com os nmeros
1, 3, 4, 5, 8, 9, 10, 12 de modo que a soma de
cada linha seja igual a 26.

7
11

9. Uma loja est fazendo uma promoo na venda de balas: Compre x


balas e ganhe x% de desconto. A promoo vlida para compras de no
mximo 60 balas. Carlos e Daniel compraram 30 e 45 balas,
respectivamente. Qual deles poderia ter comprado mais balas e gasto a
mesma quantia, se empregasse melhor seus conhecimentos de
Matemtica?
10. Quatro vacas negras e trs marrons do tanto leite em cinco dias
quanto trs vacas negras e cinco marrons em quatro dias. Qual raa de
vacas melhor leiteira, as negras ou as marrons?
11. Em um povoado vivem 700 mulheres. 4% delas usam um pendente
cada uma, metade das restantes usa dois pendentes cada uma e o
restante no usa adornos. Quantos pendentes usa o total das mulheres?
12. 95% da massa de uma melancia de 10 quilos constituda de gua. A
fruta submetida a um processo de desidratao (que elimina apenas a
gua) at que a participao da gua na massa de melancia se reduz a
90%. Qual a massa da melancia aps o processo de desidratao?
13. O nmero 15873 interessante, pois, se o multiplicarmos por um nmero
de um algarismo e depois por 7, o resultado ser um nmero formado
apenas pelo algarismo escolhido. Por exemplo: 15 873 x 5 = 79 365 e
79 365 x 7 = 555 555. Por qu?
14. Se gato e meio come rato e meio em minuto e meio, em quanto tempo
um gato come dois ratos?

109

15. Coloque parnteses para que a expresso 5 2 x 1 + 4 6 = 5 se torne


verdadeira.
16. Num concurso de televiso trs concorrentes procuram acertar o nmero
de caramelos contidos numa taa de cristal. Jos diz que h 260, Maria
cr que h 274 e Carlota prope que sejam 234. Sabe-se que um deles
se enganou em 9 caramelos, outro em 17 e outro em 31. Pode-se deduzir
qual o nmero de caramelos na taa?
17. Um nmero formado por 7 algarismos escolhidos entre os algarismos
1, 2, 3, 4, 5, 6 e 7. Se a soma de cada par de algarismos sucessivos
igual soma do primeiro par e a soma de todos os algarismos 15, qual
o nmero?
18. Um ciclista saiu para treinar levando consigo uma terceira roda de
reposio. Durante o percurso de 60 km foi alternando as rodas de
maneira que cada um rodasse uma distncia igual das outras. Quantos
quilmetros rodou cada roda?
19. O Bernardo e o seu irmo Artur receberam no Natal um quebra-cabea
com 2005 peas. Nesse mesmo dia, decidiram comear a constru-lo.
O Bernardo desafiou o seu irmo: Vamos fazer um jogo. Voc comea
por colocar uma, duas, trs ou quatro peas do quebra-cabea. Em
seguida, coloco eu uma, duas, trs ou quatro peas, e assim
sucessivamente. Quem colocar a ltima pea perde. Entusiasmados,
preparavam-se para comear a jogar, quando, de repente, um deles
exclamou: Jogue voc como jogar, eu vou conseguir ganhar!. Sabendo
que ele tinha razo, qual deles disse isso e que estratgia planejou?
20. Sobre uma mesa h 137 fichas iguais, cada uma com um lado vermelho
e outro azul, sendo que 10 esto com o lado vermelho para cima e as
outras com o lado azul. Voc est de olhos vendados e deve separar as
fichas em dois grupos, cada um com a mesma quantidade de fichas
vermelhas. Voc pode virar as fichas, se necessrio. Como fazer?
21. Um destacamento de soldados precisa atravessar um rio muito profundo
e sem pontes. Eles pedem ajuda a dois meninos que esto passando
pelo rio num barco. Porm, o barco to pequeno que nele s cabem
os dois meninos ou um soldado de cada vez. Como eles fizeram para
todos os soldados atravessarem o rio?

110

22. Num reino distante quaisquer dois cavaleiros ou so amigos ou inimigos


e cada cavaleiro tem exatamente trs inimigos. Nesse reino vigora a
seguinte lei entre os cavaleiros: Um inimigo do meu amigo meu
inimigo. Quantas possibilidades h para o nmero de cavaleiros desse
reino?
23. Andr escreveu um nmero inteiro em cada crculo e depois, em cada
quadrado, escreveu o resultado da multiplicao dos nmeros que
estavam nos dois crculos vizinhos. Coloque na figura os nmeros que
foram apagados.
24. Num hotel para ces e gatos, 10% dos ces julgam que so gatos e
10% dos gatos julgam que so ces. Aps cuidadosas observaes,
conclui-se que 20% de todos os hspedes pensam que so gatos e que
os restantes pensam que so ces. Se no hotel esto hospedados 10
gatos, quantos so os ces hospedados?
25. Csar e Sergio so amigos e gostam de fazer caminhadas. Enquanto
Csar d 4 passos, Sergio d 5 passos, contudo, 2 passos de Csar
equivalem a 3 passos de Sergio. Certo dia eles resolveram caminhar
juntos, porm o Csar chegou atrasado e o Sergio j havia dado 20
passos. Quantos passos Csar teve que dar para alcanar seu amigo,
que no alterou o seu ritmo at o momento do encontro?
26. Ordene os cartes 1, 2, 3 e 4 de cor cinza e 5, 6, 7 e 8 de cor
branca, de modo que todas as frases resultem verdadeiras.
1

Os dois seguintes
so brancos.

O anterior da
mesma cor que
o seguinte.

Os dois seguintes
so de cores
distintas.

O anterior
cinza.

O anterior da
mesma cor que
o seguinte.

Os dois seguintes
so da mesma cor.

H tantos brancos
antes como depois.

O anterior
branco.

27. Um casal tem filhos e filhas. Cada filho tem um nmero de irmos igual
ao nmero de irms. Cada filha tem um nmero de irmos igual ao
dobro do nmero de irms. Quantos filhos e filhas tem o casal?

111

28. Dois trens esto a uma distncia de 200 km e se aproximam um do


outro com uma velocidade de 50 km/h cada um. Uma mosca voa
constantemente entre as locomotivas dos 2 trens, de um pra-choque
ao outro, com uma velocidade de 75 km/h at o instante em que os
trens se chocam e a mosca morre esmagada. Qual foi a distncia total
percorrida pela mosca?
29. Mostre que em qualquer ano existe pelo menos uma sexta-feira 13.

30. O que 100% pior do que cair um raio


sobre a sua cabea?

112

Solues dos problemas

1.
Soluo 1
Dado x > 1, seja N = x e seja 0 < < /2 tal que
1
2
=
2
cos cos 2 + 1
2
cos 2 + 1 =
x
2 x
2 x
1
cos 2 =
e = arc cos
x
x
2
1
N=
1
2 x

cos arc cos


x
2
x=

Dado x tal que 0 < x < 1, use o mtodo acima para 1/x e inverta o
resultado.
Soluo 2

Se x > 0 ento x = tg2 para


algum [0, /2[. Ento:

x = tg 2
x + 1 = sec 2
1
= cos 2
x +1
1
1 x
2
1 =
= cos 2
x +1
1+ x
1 x
arc cos
= 2
1+ x
1
1 x
arc cos
=
2
1+ x
1 x
1
sen arc cos
1 + x
2
= tg = x .
1 x
1
cos arc cos
1 + x
2
113

2. Suponhamos inicialmente que as


diagonais estejam na mesma reta.
Nesse caso, os tringulos ABC e
ABC so semelhantes e, portanto,
A'B' B'C'
=
. Analogamente, v-se
AB
BC
que os tringulos ACD e ACD so
semelhantes.

D
D

A=A

A'D' D'C' A'B' B'C'


=
=
=
. Como os ngulos correspondentes dos
AD
DC
AB
BC
retngulos so iguais e os lados correspondentes so proporcionais segue a
semelhana dos retngulos ABCD e ABCD.
Vamos agora provar a recproca. Suponhamos que os retngulos sejam

Logo,

A'B' B'C'
e, como os ngulos em B e B
=
AB
BC
so retos, os tringulos ABC e ABC so semelhantes. Assim, os ngulos
semelhantes. Temos ento

C AB e C' A' B' so congruentes e, portanto, as diagonais esto na mesma


reta.
3. Como 21358ab divisvel por 99, temos que 21358ab = 99q, q
natural e 0 < a, b < 9.
Logo,
2135800 < 99q < 2135899 ou 21573,7373... < q < 21574,7373... . Sendo
q um natural, ento, q = 21574.
Assim, 21358ab = 99. 21574 = 2135826, o que implica a = 2 e b = 6.
4. A resposta no. Do teorema de Pitgoras temos a igualdade
a2 = b2 + c2. Sendo a, b e c primos, no podem ser todos mpares e,
como a > b e a > c, devemos ter b = 2 ou c = 2. Digamos c = 2.
Teremos ento:
a2 b2 = 4
(a + b)(a b) = 4
e analisando os possveis valores de a + b e a b, que so 1, 2 ou 4,
conclumos que a situao impossvel.
5. Seja x o comprimento do trem e v a sua velocidade. Assim:
114

v=

171 + x
x9
e v=
.
27
9

Resolvendo x = 99 m e v = 10 m/s.
6.

A5

A3

2l

2l

E2
E1
A1 a a la l+a A2

A
B
5

2l

A4

B4

B3
B1

B2


B
l

Construmos A1 simtrico de A com respeito ao espelho E1,


A2 simtrico de A1 com respeito ao espelho E2,
A3 simtrico de A2 com respeito ao espelho E1,
A4 simtrico de A3 com respeito ao espelho E2,
A5 simtrico de A4 com respeito ao espelho E1.
A trajetria do raio de luz a poligonal AB5B4B3B2B1B onde
B1 = BA5 E1, B2 = B1A4 E2, B3 = B2A3 E1,
B4 = B3A2 E2, B5 = B4A1 E1.
d
, o que permite achar o ngulo de
4l + a + b
incidncia, conhecendo-se a, b, l e d.

Temos tambm

tg =

7.
Lema 1

AB < AC C < B
B

115

A
A

Lema 2

AC < A'C' B < B'

C B

Na figura a seguir voc v o ABC e as bissetrizes BD e CE dos


ngulos B e C. Seja BD = CE.
Construindo o paralelogramo BDFE, temos que EF = BD = EC e
portanto
+ = +
(1)
Imagine que os ngulos B e C sejam desiguais, B > C , por exemplo.
Ento teramos:
A
B > C
F
>
>
(paralelograma BDFE)


<
(por (1))
D
E
DC < DF (Lema 1)

DC < BE (paralelograma BDFE)


<
(Lema 2)


C
B
B < C (Contradio!)
Como chegaramos tambm a uma contradio supondo inicialmente que

B < C conclumos que B = C .


8.
Soluo 1
Aplicamos a identidade logab . logbc = logac e a desigualdade clssica
entre a mdia aritmtica e geomtrica. imediato que

1
(log 5 6 + log 6 7 + log 7 8 + log8 5) 4 log 5 6 log 6 7 log 7 8 log8 5 = 1,
4
pois os logaritmos envolvidos so positivos. A igualdade vale se e somente
se todos os termos forem iguais, mas isto no nosso caso no acontece, pois
os trs primeiros logaritmos so maiores que 1 e o quarto menor que 1.
Logo vale a desigualdade estrita.
116

Soluo 2
fcil provar que se a > b > 1 e x > 1, logax < logbx. Transformando os
termos do lado esquerdo da desigualdade pelo modelo

log 5 6 = 1 + log 5

6
5

basta provar que

6
7
8
5
+ log 6 + log 7 + log8 > 0 .
5
6
7
8
Mas isso verdade, pois em base 8,
log 5

log 5

6
7
8
5
6
7
8
5
6.7.8.5
+ log 6 + log 7 + log8 > log8 + log8 + log8 + log8 = log8
=0
5
6
7
8
5
6
7
8
5.6.7.8

9. Cada uma das 3 pessoas, A, B ou C, ao fazer uma afirmao, poder


estar mentindo (M) ou falando a verdade (V). Como o problema envolve
afirmaes das 3 pessoas, o conjunto das possibilidades ser formado por
ternos ordenados dos smbolos M ou V. Assim, por exemplo, (M, M, V)
representaria o caso no qual A fala a verdade e B e C mentem. Nessas
condies, o conjunto das possibilidades (ou, como dizem os probabilistas, o
espao amostral do experimento) seria formado pelos pontos:
(M, M, M); (V, M, M); (M, V, M); (M, M, V); (M, V, V); (V, M, V);
(V, V, M) e (V, V, V).
Nesse espao, vamos considerar os eventos:
E A fala a verdade
F C diz que B diz que A falou a verdade
O que o problema pede a probabilidade condicional, P(E/F), do evento E
dado que ocorreu o evento F. Por definio, essa probabilidade dada por:
P( E / F ) =

P( E F )
P( F ) .

claro que o evento E formado pelos pontos (V, V, V), (M, V, V),
(V, M, V) e (M, M, V). Vamos, agora, identificar quais os pontos que
pertencem ao evento F. Para maior clareza vamos considerar
separadamente dois casos:
1o caso: A fala a verdade
Nesse caso, para que F ocorra, necessrio que o nmero de mentiras
ditas por B e C seja par, pois s assim elas iro se anular, permitindo que
117

C diga que B disse que A falou a verdade. Segue-se, portanto, que, nesse
caso, os pontos de F so (V, V, V) e (M, M, V).
2o caso: A mente
Um raciocnio anlogo mostra que F s ir ocorrer se uma e apenas uma
das pessoas B ou C mentir. Logo os pontos de F, nesse caso, so
(M, V, M) e (V, M, M).
Segue-se, portanto, que o evento F formado pelos quatro pontos,
(V, V, V), (M, M, V), (V, M, M) e (M, V, M).
Admitindo-se, agora, a independncia entre as afirmaes das 3 pessoas,
teremos:

1 1 1 2 2 1 1 2 2 2 1 2 13
P( F ) = + + + =
3 3 3 3 3 3 3 3 3 3 3 3 27
Por outro lado, fcil ver que o evento E F formado pelos pontos
(V, V, V) e (M, M, V) e portanto: P(E F) = 5/27. Segue-se finalmente
que: P ( E / F ) =

5 / 27
5
= .
13 / 27 13

10. Observe inicialmente que cos


cos

2
1
e que:
=
3
2

2
14
8
6
8
+ cos
= 2 cos .cos
= cos
9
9
9
9
9

2
8
14
+ cos + cos
= 0 . Desenvolvendo o quadrado
9
9
9
dessa expresso vemos que aquilo que queremos calcular igual a:

Segue-se que cos

1
2
8
14
cos 2
+ cos 2
+ cos 2
2
9
9
9

Usando a relao cos 2a = 2 cos2a 1, essa expresso se transforma em:


1
4
16
28

cos
+ cos
+ cos
+ 3
4
9
9
9

Por outro lado,

cos
118

4
28
16
12
16
16

2 2
+ cos
= 2 cos
= 2 cos
1 = cos
cos
2 cos
9
9
9
9
9
3
9

de onde se segue que:


1
4
16
28
3

cos
+ cos
+ cos
+ 3 = .
4
9
9
9
4

11.
a) Seja ax2 + bx + c = 0, com a, b e c inteiros e a 0. Suponhamos
b2 4ac = 23. Segue-se que b2 4ac = 23 mpar e, portanto, b
mpar. Se b mpar, b 1 e b + 1 so pares, e, portanto,
b2 1 = (b + 1)(b 1) mltiplo de 4. Mas b2 1 = 4ac + 22 e, como
22 no mltiplo de 4, segue-se que b2 4ac no pode ser igual
a 23.
b)
1) claro que a deve ser inteiro, uma vez que a soma das razes
a.
2) a2 24a deve ser o quadrado de um nmero inteiro.
Suponha a2 24a = n2, com n inteiro. Como a2 24a =
(a 12)2 144, temos (a 12)2 = 122 + n2. Essa equao admite
a soluo trivial n = 0 e, nesse caso, a = 0 ou a = 24. Se n
diferente de zero, n e 12 podem ser pensados como os catetos
de um tringulo retngulo cuja hipotenusa (a 12). O leitor deve
verificar que existem 4 tringulos pitagricos com um cateto igual
a 12 (veja, por exemplo, RPM 47, p. 49).
So eles:
(5, 12, 13), (9, 12, 15), (12, 16, 20), (12, 35, 37)
para
para
para
para

n = 5,
n = 9,
n = 16,
n = 35,

a = 1
a = 3
a = 8
a = 25

ou
ou
ou
ou

a = 25
a = 27
a = 32
a = 49

Se acrescentarmos os valores correspondentes a n = 0, teremos


exatamente dez valores de a que satisfazem as condies do
problema.

119

12. Decorre da hiptese que cos x = 0 se, e somente se, cos (x + y) = 0


(verifique!). Ou seja, se um dos membros da igualdade que queremos mostrar
no estiver definido, o outro tambm no estar. Suponhamos, ento, que
cos (x + y) 0 e cos x 0.
Ora, de 5 sen y = sen 2x cos y + sen y cos 2x e como
1
2 tg x
1 tg 2 x
cos y 0 (pois, sen y ), sen 2 x =
e
cos
2
x
=
temos
5
1 + tg 2 x
1 + tg 2 x
5tg y =

2 tg x
1 tg 2 x
+
tg
y
o que implica
1 + tg 2 x
1 + tg 2 x

5tg y + 5tg y tg 2 x = 2 tg x + tg y tg y tg 2 x .
E, como de cos(x + y) 0 tem-se 1 tg x tg y 0, podemos deduzir
tg x + tg y 6
3
= tg x , donde, finalmente tg ( x + y ) = tg x.
1 tg x tg y 4
2

13. Traar AE // BC. Pelo ponto mdio F de AE, traar FG // MD.


(Quando BC // AD, F e G coincidem.) Afirmamos que MG a
soluo. De fato, pela construo, rea MBADF = rea MFDC .
B

E
F
A

H
G

Por meio de MG, estamos tirando da rea direita o tringulo MFH e


acrescentando o tringulo HGD para obter MGDC. Porm,
rea MFH = rea HGD, pois FG // MD e MHD comum aos tringulos
MGD e MFD. Logo, rea MGDC = rea MFDC. Do mesmo modo,
rea MBAG = rea MBADF.
14. Se imaginarmos que todas as bolas sero retiradas da urna, existiro ao
todo n! configuraes numricas possveis para esse experimento. O nosso
problema contar em quantas dessas configuraes teremos, na k-sima
retirada (2 < k < n), pela primeira vez, uma bola cujo nmero seja maior do
que o de todas as anteriores.
120

Em primeiro lugar, vamos observar que o evento no qual estamos


interessados no depende dos particulares nmeros das bolas que sairo
nos k primeiros lugares, mas apenas da ordem em que eles sairam.

n
Existem escolhas possveis para esses nmeros. Uma vez fixados
k
esses k nmeros, para que o evento ocorra, duas condies precisam ser
satisfeitas:
1) A bola com o maior nmero (entre os k escolhidos) deve sair na
k-sima retirada.
2) A bola com o 2o maior nmero deve sair na primeira retirada.
Observe que a primeira condio garante que o processo pra na k-sima
retirada, enquanto que a segunda garante que ele no pra antes da
k-sima retirada. As outras k 2 bolas podem ocupar qualquer posio,
o que nos d um total de (k 2)! configuraes possveis. De maneira
anloga, as n k bolas que sairo aps a k-sima retirada podero
aparecer em qualquer ordem, o que nos d (n k)! possibilidades.
Segue-se que o nmero total de configuraes nas quais, pela primeira vez,
na k-sima retirada, aparece uma bola cujo nmero maior do que todas as
n
anteriores, vale (k 2)!(n k )! e, portanto, a probabilidade desse
k

evento

n
(k 2)!(n k )!
1
k
=
para 2 k < n.
n!
k (k 1)

O caso k = n tem que ser analisado em separado, pois o processo para no


instante n no apenas quando as condies 1 e 2 esto satisfeitas para
k = n (a probabilidade de que isso ocorra 1/n(n 1)), mas tambm quando
elas no so satisfeitas em nenhum instante. Esta segunda hiptese ocorre
se, e somente se, a bola com o nmero n sair na primeira retirada. (A
probabilidade de isso ocorrer , obviamente, l/n. ) Para k = n, a probabilidade
desejada vale

1
1
1
+ =
.
n(n 1) n n 1

121

15. Sejam O o centro da circunferncia dada de raio R, M o ponto


mdio de BC e P OM tal que OM = 3PM.
Para cada A na circunferncia dada considere G o baricentro do ABC.
A

AM

GM = 3
, ento, GP // OA e
Como
PM = OM

O G
P

OA R
= . Assim, G pertence
3
3
circunferncia de centro P e raio R/3.
GP =

Observe que nos casos degenerados onde A = B (A = C) consideramos G

1
1
no segmento BC com BG = BC (GC = BC ).
3
3
Logo os baricentros dos tringulos ABC pertencem circunferncia de
centro P e raio R/3.
Reciprocamente, cada ponto dessa circunferncia baricentro de algum
ABC com A na circunferncia dada. (Por qu?)
16. Como x3 y3 = (x y)(x2 +xy + y2) e 602 = 2.7.43, devemos resolver
o sistema
xy=A
x2 + xy + y2 = B.
Como 0 < y < x < x2 < B, basta experimentar os pares (A, B) com A < B
tais que A.B = 602: (1,602); (2, 301); (14, 43) e (7, 86). Somente o par (2,
301) fornece solues inteiras, de onde temos que as solues positivas
so 11 e 9.
17. Os tringulos do enunciado podem ser considerados justapostos como
ABD e AFG da figura.
A

BD AB
=
.
FG AG
Aplicado o teorema da bissetriz interna

Queremos mostrar que

ao tringulo ABC, temos

BD CD
=
.
AB AC
E

122

D
F

Como r // s, temos

CD FG
=
. Logo, BD = FG .
AC AG
AB AG

18. Vamos observar inicialmente que o conjunto A tem


10
= 252 elementos.
5

a) Sorteada a primeira combinao nos 251 elementos restantes, existe


apenas uma combinao que no tem elementos em comum com a
combinao sorteada. Segue-se que a probabilidade pedida vale 1/251.
5
b) Efetuando o 1o sorteio, existem = 5 grupos de 4 elementos da
4
combinao sorteada que podem ser combinados com qualquer um dos
5 elementos que no pertencem a ela para formar uma combinao
que tenha 4 elementos em comum com a sorteada. Segue-se que a
probabilidade pedida vale 25/25l.

19. Sejam h = altura da pirmide, R = raio da circunferncia circunscrita


a um pentgono regular convexo de lado a.

R
4a 2
10 2 5 ou
= R2 .
2
10 2 5
Temos que a, h e R formam um tringulo retngulo e assim

Ento a =

h2 = a 2 R 2 = a 2

Logo, h = a

4a 2
a 2 (50 10 5 )
=
100
10 2 5

( 50 10 5 )
.
10

20.
Soluo 1
Quando m/n = 1, o lugar geomtrico dos pontos P do plano tais que

PA m
=
a mediatriz de AB, pois
PB n

PA
= 1 P mediatriz de AB).
PB
123

Quando m/n 1, consideraremos, sem perda de generalidade, o caso


m > n > 0.
a) A construo ao lado mostra que
existem dois pontos P1, P2 da reta
AB tais que
m
Pi A m
n
= , i = 1, 2.
PB
n
i
A
P2
P1 n B
A verificao se faz atravs de
semelhana de tringulos.
b) Os pontos P1, P2 obtidos em a) so os nicos da reta AB tais que
Pi A m
= . (Por qu?)
PB
n
i

c) Se P um ponto fora da reta AB tal que PA = m , mostraremos que


PB n
P pertence circunferncia C que passa por P1 e P2 e tem dimetros
P1 P2. De fato:
Sendo PC bissetriz interna
P
do ngulo APB , segue do
teorema da bissetriz interna:
AC PA m
=
= .
BC PB n

Sendo PD bissetriz externa do ngulo APB , segue do teorema da


bissetriz externa:

AD PA m
=
= .
BD PB n

AC AD m
=
= , ento
BC BD n
por b), {C, D} = {P1, P2} (A existncia do ponto D garantida por
termos PA PB).

Como C e D so pontos da reta AB tais que

Logo, P pertence circunferncia C, j que as bissetrizes interna e


externa so perpendiculares.

124

d) Reciprocamente, dado um ponto P na circunferncia C, mostremos


que PA = m e ento concluiremos que o lugar geomtrico dos pontos
PB n
P a circunferncia C.
Tracemos por B a reta r paralela a AP. Temos:

AP AP m
=
=
(de APP2 BB'P2 )
BB' BP2 n
AP AP1 m
=
=
(de APP1 BB"P1 ).
BB" BP1 n
Logo, BB = BB, isto ,
PB mediana do tringulo
retngulo PBB. Isso
implica (verifique!)
PB = BB = BB e,
portanto, PA = m .
PB n

B
P1
A

P2

Observao: O raio da circunferncia C

ABmn
, obtido de:
m2 n2

n
ABm
AP1 =
,
m
m+n
m
ABn
.
BP2 = AP2 AB = BP2 AB BP2 =
mn
n
AP1 = AB P1 B = AB AP1

Soluo 2
O problema clssico em Geometria Analtica, com a soluo
x2 + y 2
( x b) 2 + y 2

m
,
n de onde

n2(x2 + y2) = m2(x2 + y2 2bx + b2) ou


x (n2 m2) + y2(n2 m2) + 2m2bx m2b2 = 0.
2

A = (0, 0)

B = (b, 0) x

Se n = m, obtemos a mediatriz; se n m, obtemos a circunferncia.

125

21. Sejam a e b, respectivamente, os algarismos das dezenas e das


unidades do nmero procurado. Como a 10 + b + b 10 + a = 11(a + b)
um quadrado perfeito, ento 11 um divisor de a + b. Observando que
1 < a + b < 18, resulta a + b = 11. Verificando as possibilidades para a e
b, encontramos os seguintes nmeros: 29, 38, 47, 56, 65, 74, 83 e 92.
22. Considere a, b, c retas paralelas coplanares.
A seguinte construo pode ser feita usando apenas rgua e compasso.
Fixamos A a e fazemos uma rotao de 60o, no sentido anti-horrio, da
reta b em torno de A, obtendo uma reta b que corta a reta c no
ponto C.
b

90 90
$%&


60

H
b

Fazemos a rotao, no sentido horrio, do ponto C em torno de A, obtendo


na interseo com b o ponto B b.
Os tringulos retngulos AHC e AHB so congruentes, uma vez que
AH = AH e AB = AC.
Logo, ACH ' = ABH = , que implica B AC = 90 + ( 30 ) = 60 , de
modo que o ABC eqiltero.
23. Um grupo de 2n pessoas formado por n homens e n mulheres.
2n
Existem maneiras de escolhermos um conjunto de n pessoas desse
n
grupo. Vamos determinar em quantos desses conjuntos existem exatamente
k homens. Para isso vamos observar que os k homens podem ser escolhidos
n
n
de maneiras e as n k mulheres de
maneiras. Segue-se
k
n k

126

que, para k = 0, 1, ..., n o nmero de escolhas de n pessoas que contm


2

n n n
exatamente k homens ser dado por
= .
k n k k
2

n n
n 2n
Conclui-se portanto que 0 + 1 + + n = n .


24. Considere o produto dos 3 nmeros pares consecutivos:
88.106 < (x 2) x (x + 2) = x3 4x < x3
Temos:
85184000 = 4403 < 88.106 < 4503 = 91125000
Trs nmeros pares consecutivos podem terminar em:
0, 2, 4
2, 4, 6
4, 6, 8
6, 8, 0
8, 0, 2.
O nico produto dessas triplas que termina em 2 4.6.8 = 192, logo os
nmeros so 444, 446, 448 cujo produto 88714752. Portanto, os
algarismos procurados so 7, 1, 4, 7, 5.
25. Sejam r1, r2, ..., r7 as 7 razes da equao. Temos ento:
r1 + r2 + ...+ r7 = 2
r1r2 + r1r3 + ... + r6r7 = 3
Segue-se 4 = (r1 + r2 + ... + r7 )2 = r12 + r22 + ... + r72 + 6 e, portanto,
7

= 2 , o que mostra que nem todas as razes podem ser reais.

26. Sejam xi R, i = 1;..., 100 tais que P(xi) = 0 para todo i entre 1
e 100.
Suponhamos, por contradio, que xi < 7, i = 1, ..., 100.
Ento como P(x) = (x x1)(x x2)...(x x100), temos que:
1 < P(7) = (7 x1)(7 x2)...(7 x100).
Logo,
127

1 = 100 1 < 100 (7 x1 )(7 x2 ) (7 x100 )


100

7 x1 + 7 x2 + + 7 x100
1<
100

700 xi
i =1

100

.
98

Mas como

P ( x) = ( x x1 ) ( x x100 ) = x100 600 x99 + a j x j ento


j =0

100

i =1

= 600. Logo, 1 <

700 600
= 1 (absurdo!).
100

27. Como o pentgono tem todos os lados iguais, basta mostrar que ele
inscritvel.
1 o caso: os ngulos congruentes so consecutivos A B E . Os
quadrilteros BAED e EABC so trapzios issceles, logo inscritveis e a
circunferncia que passa pelos pontos B, A, E e D tambm passa pelos
pontos E, A, B e C.
A

A
B

2o caso: os ngulos congruentes no so consecutivos A B D . O AED


issceles, logo os seus ngulos da base AD so congruentes
e, portanto, o quadriltero ABCD um trapzio issceles.
Portanto, B C , recaindo no 1o caso.
28. Se existe uma raiz racional, temos b2 > 4ac e tambm temos que
b2 4ac um quadrado perfeito m2. Sendo b mpar, b2 mpar e, como
4ac par, temos b2 4ac mpar, implicando m2 mpar, que, por sua vez,
implica m mpar. Como b2 m2 = 4ac e a diferena dos quadrados de dois
nmeros mpares sempre um mltiplo de 8 (verifique!), conclui-se que
4ac mltiplo de 8. Mas, sendo a e c mpares, 4ac no um mltiplo de
8; logo, a equao ax2 + bx + c = 0 no tem razes racionais.

128

29. Suponha que a afirmao seja falsa, isto , os pontos do plano foram
pintados usando-se trs cores A, B e C e todos os segmentos de
comprimento k possuem extremidades de cores diferentes.
Seja O um ponto do plano e, sem perda de
X
generalidade, suponhamos que ele seja da cor A.
k
k
N
M
Sejam 1 e 2 as circunferncias de centro O e
raios respectivamente k e k 3 .

O
Todos os pontos de 1 tero sido pintados de cor
B ou C, pois, caso contrrio, haveria um raio
(segmento) 1 de cujas extremidades seriam ambas
da cor A.
Tome um ponto X em 2 e pontos M e N em 1 satisfazendo:
MN = k = MX = NX .

O valor do raio de 2, k 3 , garante a existncia do losango OMXN .


Assim, M e N possuem cores diferentes (B e C) e X deve ter a cor A.
Como todos os pontos de 2 podem ser obtidos dessa forma, provamos
que todos eles esto pintados com a cor A, o que uma contradio, pois
sobre 2 existem cordas de comprimento k.
30. Seja S o conjunto dado. Suponhamos que seus pontos no so colineares.
Sejam A e B dois deles. Dado P S, temos PA PB AB com
AB = n N, logo P pertence a uma das n + 1 hiprboles de focos A e
B dadas por PA PB = k com k N, 0 < k < n.
Observamos que, para k = 0 e para k = n (casos nos quais o ponto P
pertence mediatriz de AB ou reta AB), temos hiprboles degeneradas.
Considere C S tal que A, B, C no sejam colineares. Todo ponto
P S pertence a um outro feixe de m + 1 hiprboles de focos A e C,
onde AC = m N.
S est contido na interseco dos dois feixes (finitos) de hiprboles. A
interseco de duas hiprboles sempre finita, se elas forem distintas. Como
os dois feixes no tem nenhuma hiprbole em comum (convena-se disso),
seguiria que S finito. Contradio!

129

130

Respostas dos ...probleminhas


1. 100
2. 2114
3. Comeando no canto superior
esquerdo e girando no sentido antihorrio: 6, 3, 2, 4, 8, 7, 1, 5.
4. 22

22

5.

6.

20. Separe um grupo de 10 fichas


quaisquer e vire-as. Este grupo e o
grupo das fichas restantes ficam
com o mesmo nmero de fichas
vermelhas.

19. Como 2005 = 4 + 400 5 + 1, o


primeiro a jogar, Artur, pode colocar
4 peas e, em seguida, coloque o
Bernardo o que colocar, Artur, pode
jogar de modo que, na sua jogada e
na anterior, o total de peas
colocadas seja 5. Com essa estratgia, Artur vence e foi ele quem
disse a frase.

10. As vacas marrons.

21. O menino A fica na margem oposta


margem na qual esto os soldados
e o menino B leva o barco at os
soldados. O primeiro soldado
atravessa o rio e o menino A traz o
barco de volta. Os dois meninos
atravessam o rio, o menino A fica e
o menino B leva novamente o barco
at os soldados. O segundo
soldado atravessa o rio e...

11. 700

22. 4 ou 6

12. 5 kg

23. No sentido horrio, a partir do 85: 5,


45, 135, 15, 8 e 17.

7. 10
8.

2
10

11

12

9. Daniel

13. Porque 15873 7 = 111 111.


14. Em 3 minutos.

24. 70

15. (5 2) (1 + 4 6) = 5

25. 80 passos

16. 243

26. 2, 5, 3, 7, 4, 1, 6 e 8

17. 3131313

27. 4 filhos e 3 filhas

18. 40 km

28. 150 km
30. Cair um dimetro.
131

RPM
Sociedade Brasileira de Matemtica
Presidente: Joo Lucas Marques Barbosa
Vice-Presidente: Hilrio Alencar
Secretrio-Geral: Marco Antnio Teixeira
Tesoureiro: Walcy Santos
Comit Editorial da RPM
Alcila Augusto editora responsvel
Ana Catarina P. Hellmeister editora executiva
Alberto Carvalho P. de Azevedo
Antonio Luiz Pereira
Eduardo Wagner
Elon Lages Lima
Geraldo vila
Jos Paulo Q. Carneiro
Paulo Cezar Pinto Carvalho
Renate G. Watanabe

RPM Revista do Professor de Matemtica


Caixa Postal 66281 CEP 05311-970
So Paulo, SP
www.rpm.org.br
rpm@ime.usp.br
telefone/fax: (11) 3091 6124

132

Apresentao

Caros professores orientadores e alunos do Programa


de Iniciao Cientfica PIC da OBMEP
Este um nmero especial da Revista do Professor de Matemtica
RPM, elaborado para utilizao no PIC da OBMEP a ser
realizado a partir do primeiro semestre de 2009.

A revista publica crnicas, artigos e sees, como Problemas, O


leitor pergunta, Livros, Olhando mais de cima, etc. Nos artigos,
temas interessantes de nvel elementar ou avanado so
apresentados de modo acessvel ao professor e ao aluno do ensino
bsico ou de cursos de Licenciatura em Matemtica. Uma
experincia interessante em sala de aula, um problema que suscita
uma questo pouco conhecida, uma histria que merea ser
contada ou at uma nova abordagem de um assunto conhecido.
Nas sees, a revista conversa com o leitor, publicando
problemas e/ou solues propostas por eles, cartas, resenhas de
livros, erros encontrados em textos didticos, etc.

RPM OBMEP

A RPM, como seu nome diz, uma revista dedicada aos


professores de Matemtica da educao bsica, a alunos e
professores de cursos de licenciatura em Matemtica e a todos
aqueles que se interessam pela Matemtica do nvel mdio. O
tratamento dado aos temas abordados procura ser acessvel e
agradvel, sem sacrificar o rigor. A revista uma publicao da
Sociedade Brasileira de Matemtica SBM e tem sido editada e
distribuda sem interrupes desde 1982.

Para este exemplar especial, o Comit Editorial da RPM escolheu


artigos que pretendem ampliar o conhecimento dos alunos em diferentes
tpicos, bem como temas que motivem discusses ou satisfaam
curiosidade terica e histrica de alunos interessados em Matemtica.
Alm disso, publicamos vrias propostas de atividades que podem ser
aplicadas nas salas de alunos de ensino fundamental e mdio. As
atividades tentam despertar a curiosidade dos alunos para tpicos
importantes da Matemtica que so explicitados nas justificativas dos
procedimentos propostos.
Apresentamos tambm uma seo Problemas com:
Problemas I: problemas interessantes com nmeros primos.
Problemas II: uma seleo de problemas extrados do PISA
(Programme for International Student Assessment Programa
Internacional de Avaliao de Alunos).
Problemas III: vinte problemas selecionados entre os publicados na
seo Problemas da RPM, que abrangem a maioria dos tpicos da
educao bsica.
As solues dos problemas propostos esto no final deste fascculo.
Os artigos aqui publicados no apresentam as referncias bibliogrficas, citaes ou agradecimentos que constam nos artigos originais da
RPM.

RPM OBMEP

Comit Editorial da RPM

Contedo
Atividades em sala de aula . . . . . . . . . . . . . . . . . . . . . . . . . . . . . . . 05
Um jogo aritmtico . . . . . . . . . . . . . . . . . . . . . . . . . . . . . . . . . . . . . . . 12
Seis problemas no triviais equivalentes . . . . . . . . . . . . . . . . . . . . . . . 16
O menino . . . . . . . . . . . . . . . . . . . . . . . . . . . . . . . . . . . . . . . . . . . . . 22
O problema dos cinco discos: sorte ou sabedoria? . . . . . . . . . . . . . . . 26
Calculadora padro: um problema interessante . . . . . . . . . . . . . . . . . . 31
Uma equao interessante . . . . . . . . . . . . . . . . . . . . . . . . . . . . . . . . . 37
Painis
41
43
45
47
49
51
52
54
56
58

Funes interessantes . . . . . . . . . . . . . . . . . . . . . . . . . . . . . . . . . . . 60
A formiga inteligente . . . . . . . . . . . . . . . . . . . . . . . . . . . . . . . . . . . 66
A demonstrao feita por Heron . . . . . . . . . . . . . . . . . . . . . . . . . . . . . 72

RPM OBMEP

Painel I
O nmero 12 . . . . . . . . . . . . . . . . . . . . . . . . . . . . . . . . . . . . . . . . . . . . . .
Painel II
Sexta-feira 13 . . . . . . . . . . . . . . . . . . . . . . . . . . . . . . . . . . . . . . . . . . . . . .
Painel III
O jogo de bilhar . . . . . . . . . . . . . . . . . . . . . . . . . . . . . . . . . . . . . . . . . . . . .
Painel IV
Codificando e decifrando mensagens . . . . . . . . . . . . . . . . . . . . . . . . . . . .
Painel V
Qual a relao entre os nmeros 102.564 e 410.256? . . . . . . . . . . . . . . .
Painel VI
Uma demonstrao visual para a frmula do sen(A + B) . . . . . . . . . . . . .
Painel VII
Valores irracionais de funes trigonomtricas . . . . . . . . . . . . . . . . . . . .
Painel VIII
Mgica com nmeros . . . . . . . . . . . . . . . . . . . . . . . . . . . . . . . . . . . . . . . . .
Painel IX
Destreza ou esperteza? . . . . . . . . . . . . . . . . . . . . . . . . . . . . . . . . . . . . . . .
Painel X
Determinante para fatorar . . . . . . . . . . . . . . . . . . . . . . . . . . . . . . . . . . . . .

A Matemtica da folha de papel A4 . . . . . . . . . . . . . . . . . . . . . . . . . .

75

Retngulo ureo, diviso urea e sequncia de Fibonacci . . . . . . . . . 82


Usando Geometria para somar . . . . . . . . . . . . . . . . . . . . . . . . . . . . . 95
Mdias . . . . . . . . . . . . . . . . . . . . . . . . . . . . . . . . . . . . . . . . . . . . . . . 100
Problemas diversos resolvidos com Geometria Analtica . . . . . . . . 112
A sombra do meu abajur . . . . . . . . . . . . . . . . . . . . . . . . . . . . . . . . 119
Ilha do tesouro. Dois problemas e duas solues . . . . . . . . . . . . . . . 125
Qual mesmo a definio de polgono convexo? . . . . . . . . . . . . . . . 129
A soluo de Tartaglia para a equao do 3o grau
e a emergncia dos nmeros complexos . . . . . . . . . . . . . . . . . . . . . . . 135
Grandezas incomensurveis e nmeros irracionais . . . . . . . . . . . . . 153
A outra face da moeda honesta . . . . . . . . . . . . . . . . . . . . . . . . . . . . . . 162
Nmero de regies: um problema de contagem . . . . . . . . . . . . . . . . . 165
Intuio e Probabilidade . . . . . . . . . . . . . . . . . . . . . . . . . . . . . . . . . . .

170

Problemas I . . . . . . . . . . . . . . . . . . . . . . . . . . . . . . . . . . . . . . . . . . . . 172
Problemas II . . . . . . . . . . . . . . . . . . . . . . . . . . . . . . . . . . . . . . . . . . . . 175
Problemas III . . . . . . . . . . . . . . . . . . . . . . . . . . . . . . . . . . . . . . . . . . . . 181

RPM OBMEP

Soluo dos Problemas . . . . . . . . . . . . . . . . . . . . . . . . . . . . . . . . . . . . 185

Atividades em sala de aula

Aprende-se Matemtica fazendo Matemtica.


Apresentamos aqui dois grupos de atividades que
permitem a alunos do ensino fundamental fazer
Matemtica. As folhas contendo as atividades foram
copiadas de nmeros da revista The Mathematics
Teacher* (O professor de Matemtica).
No incio da primeira atividade, alguns exemplos podem
ser feitos coletivamente. Ao fim de cada atividade
interessante comparar os resultados obtidos pelos alunos
reforando o fato de que um mesmo problema pode ter
vrias solues.

Na pgina 15 deste exemplar est uma soluo de cada


problema proposto no Primeiro grupo.
*

Publicao do National Council of Teachers of Mathematics


NCTM, Reston, Virginia, USA.

RPM OBMEP

O primeiro grupo de atividades trabalha as operaes


aritmticas com nmeros inteiros. O segundo grupo de
atividades trabalha com visualizao de figuras no espao,
permitindo aos alunos descobrir padres e fazer
conjecturas. As primeiras partes podem ser aplicadas em
salas do ensino fundamental e a parte final no ensino
mdio, explorando generalizaes e suas representaes
algbricas.

Primeiro grupo: atividades I, II e III.


I. Instrues
Em cada linha h 5 nmeros e um sexto nmero, chamado total.
Coloque os sinais +, , , e parntesis, colchetes, chaves, de

modo que o resultado das contas indicadas seja o total.


Os 5 nmeros devem ser usados, cada um deles uma s vez, em

qualquer ordem.
Exemplo: 7, 8, 1, 9, 9 total: 16.
Uma soluo: (9 9) (7 + 8 + 1) = 16.
1. 1, 5, 3, 6, 10

total: 5

2. 8, 11, 9, 1, 8

total: 2

3. 11, 10, 15, 20, 3 total: 6


4. 12, 18, 3, 11, 12 total: 8
5. 4, 16, 10, 24, 25 total: 1

RPM OBMEP

6. 17, 14, 7, 17, 13 total: 7

7. 2, 9, 5, 9, 4

total: 22

8. 3, 6, 10, 5, 7

total: 2

9. 8, 6, 11, 5, 21

total: 7

10. 6, 1, 2, 2, 17

total: 8

II. Instrues
Trabalhe com os nmeros 11, 14, 3, 19 e 9.
Coloque os sinais +, , , e parntesis, colchetes, chaves, para
obter todos os nmeros de 1 at 11.
Os nmeros 11, 14, 3, 19 e 9 devem ser usados, cada um deles uma

s vez, em qualquer ordem.


1. (11 + 14 19 + 3) 9 = 1
2.
3.
4.
5.
6. 11 [(19 + 9) 14 + 3] = 6
7.
8.
9.

11. [9 (19 14) 3] 11 = 11

RPM OBMEP

10.

III. Instrues
Trabalhe com os nmeros 2, 3, 5, 7 e 11. (Observe que so os cinco

primeiros nmeros primos.)


Coloque os sinais +, , , e parntesis, colchetes, chaves, para

obter os nmeros pedidos nos itens de 1 a 10 abaixo.


Os nmeros 2, 3, 5, 7 e 11 devem ser usados, cada um deles uma s

vez, em qualquer ordem.


1. Escreva (seguindo as instrues) o menor primo mpar.
3 = [(2 5) + (7 3)] 11
2. Escreva o menor nmero natural mpar.
3. Escreva o menor nmero natural primo.
4. Escreva o menor nmero natural composto.
5. Qual o maior nmero natural composto que voc consegue escrever?
6. Qual o maior nmero natural mpar que voc consegue escrever?
7. Escreva o menor nmero natural que voc consegue achar, usando
uma s vez cada uma das operaes.
8. Determine e escreva o maior nmero natural par possvel, usando
uma s vez cada uma das operaes.

RPM OBMEP

9. Escreva um nmero natural usando apenas subtraes.

10. Determine e escreva o maior nmero primo possvel obedecendo s


instrues.

Segundo grupo Descobertas com cubos: atividades I, II, III e IV


Descrio
Um cubo grande, decomposto em cubos pequenos, mergulhado
numa lata com tinta. Pergunta-se quantas faces dos cubos pequenos
ficaro pintadas.
Objetivos
Estudantes visualizaro figuras no espao, construiro uma tabela,
descobriro padres na tabela e, usando esses padres, faro conjeturas.
Diretrizes
Distribuir para cada aluno folhas com as atividades ou coloc-las
no quadro-negro. Sugere-se dividir a classe em grupos de dois alunos,
deixando-os trabalhar juntos.
Aps completar a atividade I, os estudantes devem registrar seus
resultados na tabela (atividade III). Certifique-se de que todos os
estudantes tm os valores corretos, pois conjeturas sero feitas a partir
dos dados da tabela. Poucos estudantes conseguiro completar a tabela
para um cubo 10 10 10, a menos que algum padro tenha sido
identificado (atividade IV). Pergunte: Existem constantes em uma
coluna? Existem mltiplos?. Sugerir aos alunos que procurem fatores
comuns vai ajud-los a reconhecer padres. Por exemplo, 0, 6, 24, 54 e
96 so as 5 primeiras entradas em uma das colunas. Um padro torna-se
mais visvel se esses nmeros forem escritos como 0, 6 1, 6 4,
6 9 e 6 16.
I. Responda s perguntas a seguir para cada um dos cubos das figuras 1,
2, 3 e 4.
a) Quantos cubos pequenos h no cubo grande?
Se esse cubo maior for jogado numa lata de tinta e totalmente
submerso:
c) Quantos cubos pequenos tero 2 faces pintadas?
d) Quantos cubos pequenos tero 1 face pintada?
e) Quantos cubos pequenos tero 0 face pintada?

RPM OBMEP

b) Quantos cubos pequenos tero 3 faces pintadas?

f) Qual a soma de suas respostas em b), c), d) e e)?

II. Complete a figura 5, desenhando um cubo 6 6 6. Responda


novamente s perguntas a), b), c), d), e), e f).
III. Agora registre as informaes na tabela abaixo. Considerando, em
cada caso, o lado dos cubos pequenos como unidade.
Comprimento do
lado do cubo
maior

RPM OBMEP

2
3
4
5
6

10

nmero de cubos pequenos


com faces pintadas

nmero total
de cubos
pequenos

IV. Voc observa padres na tabela? Em caso afirmativo, complete a


tabela para um cubo 7 7 7. Em caso negativo, desenhe o cubo e
ento complete a tabela.
Voc realmente pegou o jeito? Se voc acha que sim, complete a
tabela para um cubo 10 10 10.
Eis uma questo que pode ser usada para culminar essa atividade:
Seja n o comprimento de um lado do cubo. Quando voc completar
na tabela a linha correspondente a n, a soma dos valores dessa
linha ser n3?
Nota do tradutor
Para completar a tabela para um cubo de lado n, pode-se considerar
o cubo grande como sendo formado por n camadas horizontais. Cada
camada um quadrado n x n . Nos esboos abaixo, o nmero em cada
cubo pequeno indica quantas de suas faces ficam pintadas aps a imerso
do cubo na lata de tinta:

camadas sup. e inf.

2(n 2)2

8(n 2)

camadas intermedirias

(n 2)3

4(n 2)2

4(n 2)

total

(n 2)3

6(n 2)2

12(n 2)

3
8

Adaptado do artigo
Atividades em sala de aula
Renate Watanabe, RPM 61

RPM OBMEP

faces pintadas

11

Um jogo aritmtico

Introduo
Como seria bom se pudssemos fazer da Matemtica uma
fonte de prazer ainda maior do que ela j . Isso possvel
se tivermos como aliado um poderoso recurso ldico: o
jogo. Proponho aqui um jogo aritmtico, que muito fcil
de aprender, e pode ser jogado por duas ou mais pessoas.
A idia sortear um nmero que, em seguida, deve ser
obtido de outros, atravs das quatro operaes. Para
representar os inteiros usamos as cartas de um baralho
comum, com exceo dos coringas. O s (A), o valete
(J), a dama (Q) e o rei (K) representam os nmeros 1, 11,
12 e 13 respectivamente.

RPM OBMEP

Formando nmeros

12

Estamos acostumados ao clculo de expresses aritmticas, isto , dada uma expresso envolvendo nmeros e
operaes matemticas, encontrar o nmero que lhe
corresponde. Aqui se pede a soluo do problema recproco: dado um nmero, encontrar uma expresso aritmtica que corresponde a esse nmero. No jogo s permitido o uso das 4 operaes aritmticas bsicas (adio,
subtrao, multiplicao e diviso) e de parnteses. Por
exemplo, com os nmeros 2, 5, 7, 8 e 11, alguns dos
nmeros que podemos formar so:

19 = 11 + 8
33 = (5 2) 11
64 = (8 2) (5 + 11)
81 = 2 5 7 + 11
80 = ((5 2) 7 11) 8
100 = (7 + 2 + 11) 5
Note que:
1. No necessrio usar todos os inteiros disponveis;
2. O uso de parnteses no tem restries. Podemos tambm usar
parnteses encaixados como na expresso do nmero 80;
3. S podemos usar cada inteiro disponvel uma nica vez;
4. No se pode formar nmeros por justaposio, isto , com o 5 e o 2
no podemos formar nem o 25 nem o 52.
Na prtica, no precisamos escrever a expresso usando parnteses.
Para formar o 80, declaramos: 5 menos 2 3; 3 vezes 7 21; 21 menos
11 10; 10 vezes 8 80. Para formar o 64, declaramos: 8 dividido por 2
4; 5 mais 11 16; 4 vezes 16 64.
O que necessrio
1. Um baralho (descartam-se os coringas);
2. Cada jogador pode, se julgar necessrio, ter caneta ou lpis e uma ou
mais folhas de papel.
Incio do jogo
Colocamos o baralho na mesa, com as cartas voltadas para baixo,
num monte, de modo que no se possa ver que nmeros representam.
Escolhe-se de comum acordo um participante para iniciar a rodada. Ento
os itens 1, 2 e 3 a seguir devem ser repetidos at que haja um vencedor.
1. Escolhemos a carta de cima do monte e multiplicamos seu valor por
13: em seguida, somamos o resultado do produto ao valor de uma
segunda carta retirada de cima do monte. Obteremos um nmero
entre 14 a 182 (13 1 + 1 = 14 e 13 13 + 13 = 182). Esse o
nmero que deve ser formado na rodada. As duas cartas tiradas vo
para baixo do monte;

RPM OBMEP

O jogo

13

2. O jogador da vez retira uma carta de cima do monte e a pe com o


nmero para cima, no centro da mesa, ou ao lado da ltima carta
retirada;
3. Ele ento faz suas anotaes e clculos, e ter duas opes:
a) Formar o nmero sorteado ganhando a rodada (1 ponto). Nesse
caso, o jogador da vez passa a ser aquele que est sua esquerda
e colocam-se as cartas retiradas debaixo do monte. A partir daqui,
precisa-se sortear um novo nmero, portanto retorna-se ao item 1
para o incio de outra rodada;
b) Passar a vez ao jogador da sua esquerda. Em seguida d-se
prosseguimento rodada retornando-se ao item 2.
Quem vencer um total de 3 rodadas primeiro vence o jogo. Enquanto
isso no ocorrer, repetem-se os itens 1, 2 e 3 sucessivamente.
Um exemplo
O primeiro jogador, A, tira a carta de cima do monte, digamos, 5 e a
carta seguinte, uma dama. Ento, o nmero a ser formado na rodada
ser 13 5 + 12 = 77. As duas cartas tiradas vo para baixo do monte.
O segundo jogador, B, tira a carta de cima do monte, digamos 8 e a
coloca aberta na mesa. No d para formar 77 com o nmero 8. Ele
passa a vez para A (se o jogo s tiver dois jogadores), que tira, digamos,
6. Com 8 e 6 e as quatro operaes ainda no d para obter 77. O 6 fica
aberto na mesa e A passa a vez para B que tira, digamos, um valete. Com
6, 8 e 11 no d para obter 77. a vez de A que tira, digamos, 3.

RPM OBMEP

A dois casos podem ocorrer:

14

(1) A percebe que 6 11 + 8 + 3 = 77. Ento a rodada termina, A ganha


1 ponto, as cartas vo para baixo do monte e tudo comea de novo
com B tirando as duas cartas de cima do monte para obter um novo
nmero.
(2) A no percebeu que podia obter 77 com as cartas da mesa e passa a
vez para B. Se B obtiver o 77, ele que ganha um ponto e uma nova
rodada se inicia. Se B no obtiver o 77, ele tira mais uma carta do
monte e assim, sucessivamente, at que um dos jogadores conseguir
formar o 77 com as cartas que esto abertas na mesa.

Concluso
Tenho jogado com amigos j h algum tempo. Estou convencido de
que esse um jogo intelectualmente estimulante e muito agradvel.
claro que existem muitos jogos com essas qualidades, mas esse tem a
vantagem de ser matematicamente educativo. Alm disso, uma forma
de viver a Matemtica, interagir com ela, senti-la, toc-la. Tambm estou
certo de que podemos criar jogos matemticos que trabalhem a
compreenso de teoremas e suas demonstraes, bem como suas
aplicaes na resoluo de problemas..., mas esse j um outro assunto...
Adaptado do artigo
Um jogo aritmtico
Eric Campos Bastos Guedes, RPM 55.

____________________________________

II

III

1. 10 (6 + 3 + 1) + 5
2. 11 + 1 9 (8 8)
3. 11 [(20 15) 3 10]
4. (11 + 3) 12 + 18 12
5. (4 + 16) 10 (25 24)
6. 17 17 + (14 13) 7
7. (9 9) + ( 5 4) + 2
8. (7 6) + [3 (10 5)]
9. 5 [(11 + 21) 8] + 6
10. (17 1) [(6 2) 2]

2. 9 {14 [(19 + 3) 11]}


3. {11 [(19 + 9) 14]} 3
4. 19 9 + 11 14 3
5. 9 {14 [(11 + 19) 3]}
7. 14 {[(3 + 19) 9] 11}
8. 9 [(11 3) (14 + 19)]
9. 11 3 (14 + 19) + 9
10. 19 9 + 11 14 + 3

2. 1 = 5 [( 11 + 3) 7 + 2]
3. 2 = [(5 + 3) (11 7)] 2
4. 4 = [(5 + 3) (11 7)] 2
5. 2 3 5 7 11 = 2310
6. 11 7 5 (3 + 2) = 1925
7. 0 = [(11 + 3) 2 7] 5
8. [(11 3) (5 + 7)] 2 = 48
9. 5 {3 [(11 2) 7]} = 4
10. 11 7 5 3 2 = 1153
RPM OBMEP

Respostas das Atividades Primeiro grupo

15

Seis problemas
no triviais equivalentes

O poder da Matemtica de relacionar o que


aparentemente no tem relao.

Neste artigo dois problemas sero chamados equivalentes


se sua resoluo fizer uso do mesmo tipo de Matemtica.
Problemas equivalentes evidenciam talvez a qualidade
mais importante da Matemtica: a possibilidade de um
conceito terico ser usado como modelo para muitas
idias diferentes. fcil produzir exemplos.

RPM OBMEP

Se o conceito terico for combinaes, como em


Probabilidade, essa ideia tambm pode ser usada para
determinar as leis de Mendel em Biologia, para calcular
coeficientes binomiais, para calcular certas probabilidades em jogos de baralho, para achar o nmero de
polgonos de vrios tipos que tenham pontos arbitrrios,
como vrtices, e assim por diante, quase que indefinidamente.

16

Mas difcil produzir bons exemplos quando se desejam


problemas equivalentes em uma escala muito menor, onde
mesmo tipo de Matemtica no significa Matemtica de
um mesmo campo, ou de um mesmo tpico dentro de
um campo ou assunto que usem as mesmas ideias.
Especificamente tentei encontrar problemas satisfazendo
as seguintes condies:
1) Os problemas deveriam ser matematicamente
idnticos at nos nmeros usados na sua resoluo.

2) At que uma resoluo fosse examinada, nada no problema deveria


indicar que o mesmo tipo de Matemtica pudesse ser usado. Assim
os problemas deveriam, na medida do possvel, vir de tpicos
totalmente desvinculados dentro da Matemtica ou dentro de
aplicaes da Matemtica.
3) Os problemas deveriam estar no mbito da Matemtica do ensino
fundamental ou ensino mdio, quanto mais simples, melhor.
Problemas
1. Expresse 1 como soma de duas fraes de numerador 1 (fraes do
2
1
, n um inteiro positivo).
tipo
n
2. Ache todos os retngulos cujos lados tenham por medida nmeros
inteiros e que tenham rea e permetro numericamente iguais.
3. Quais pares de inteiros positivos tm mdia harmnica igual a 4?
4. Ache os possveis pares de inteiros cujo produto seja positivo e igual
ao dobro de sua soma.
5. Dado um ponto P, ache todos os n tais que o espao em torno de P
possa ser coberto, sem superposio, por polgonos regulares,
congruentes, de n lados.
6. Para quais inteiros positivos n > 2, o nmero 2n divisvel por
n 2?
Para mostrar a equivalncia, verificaremos que os seis problemas se
reduzem resoluo de uma equao que a caracterizao do primeiro.
Reduo dos problemas a uma equao

1 1 1
Se 1 for a soma de duas fraes de numerador 1 ento
= + ,
2 p q
2
onde p e q so inteiros positivos. (A equao ser resolvida mais
adiante.)

RPM OBMEP

Problema 1

17

Problema 2
Sejam a e b o comprimento e a largura do retngulo procurado. Como
a rea e o permetro so numericamente iguais, temos:
2a+ 2b = ab
2(a + b) = ab
a+b 1
=
ab
2
1 1 1
+ = .
a b 2

Como a e b devem ser inteiros e positivos essa ltima equao tem a


mesma forma que a equao do Problema 1.
Problema 3
A mdia harmnica de dois nmeros x e y

2xy
.
x+ y

Sejam x e y inteiros positivos. Das condies dadas:


2 xy
=4
x+ y
xy
=2
x+ y
x+ y 1
= .
xy
2

A ltima equao tem a mesma forma que a equao na 3a linha do


Problema 2 e assim se reduz equao do Problema 1.
Problema 4

RPM OBMEP

Sejam x e y dois inteiros, z o seu produto, z > 0. Os nmeros x e y


devem ser positivos pois a sua soma e produto so positivos. Das

18

condies dadas obtm-se xy = z e x + y =


implicam: x + y =

xy
2

x+ y 1
= .
xy
2

z
. As condies juntas
2

Essa ltima equao idntica equao da ltima linha do Problema 3


e assim reduz-se equao do Problema 1.
Problema 5
Este o problema mais difcil de caracterizar. Seja k o nmero de
polgonos com vrtice em P. Se os polgonos no se sobrepuserem,
forem regulares e congruentes, utilizando a notao da figura abaixo,
obter-se-: 1 = 2 = ... = k =

360
em graus.
k

Mas os ai so medidas de ngulos de polgonos regulares de n lados,


portanto

i =

(n 2)180
n

1 < i < k.

Temos ento:

360 (n 2)180
=
k
n
2 (n 2)
=
k
n
2n = (n 2)k
2n + 2k = nk.

Problema 6
Se 2n divisvel por n 2 ento 2n = (n 2)k, onde k um numero
inteiro. Essa equao idntica a uma das equaes do Problema 5 e
portanto se reduz do Problema 1.

RPM OBMEP

Das condies do problema segue-se que n e k devem ser inteiros


positivos e portanto essa equao tem a mesma forma que a da primeira
linha do Problema 2.

19

Equao diofantina
Assim, os seis problemas podem ser resolvidos considerando-se a
equao do Problema 1. Devido s condies, essa equao uma
equao diofantina e sua soluo interessante.
1. Seja

1 1 1
onde p e q so inteiros positivos.
= +
2 p q

2. impossvel termos

1 1
1 1
e
(pois a soma no chegaria a
>
>
4 p
4 q

1
1
ser 1 ) e assim pelo menos uma das fraes
ou
deve ser maior
p
q
2

do que ou igual a

1 1
1
. Suponham os
.
p 4
4

3. Ento p = 1, 2, 3 ou 4.
4. p = 1

1
1
= 1 + q = 2 o que no possvel pois q posi2
q

tivo; p = 2

1 1 1
1
= + = 0 , que no tem soluo;
2 2 q
q
p = 3 q = 6;
p = 4 q = 4.

5. Por causa da simetria de p e q na equao original, obtemos resultados


correspondentes se

1
1
.
q
4

6. Portanto temos 3 solues:


(p, q) = (3, 6); (p, q) = (4, 4); (p, q) = (6, 3).
RPM OBMEP

Solues

20

Todos os problemas esto agora resolvidos.


Problema 1 A resposta

1 1 1 1 1 1 1
= + = + = + .
2 3 6 4 4 6 3

Problema 2 Existem dois retngulos satisfazendo as condies dadas:


um 4 4 e o outro, 3 6.
Problema 3 Duas respostas: 4 e 4 ou 3 e 6 so pares de inteiros cuja
mdia harmnica 4.
Problema 4 Os pares so idnticos ao do problema 3.
Problema 5 Os nicos polgonos regulares congruentes que, sem
superposio, cobrem o espao em torno de P (e assim cobrem o plano)
so os polgonos de 3 lados (seis tringulos eqilteros em torno de P),
os de 4 lados (quatro quadrados em torno de P) e os de 6 lados (trs
hexgonos regulares em torno de P), como se v na figura.

Seis tringulos, quatro quadrados, trs hexgonos.

Problema 6 A resposta : n 2 um divisor de 2n quando n = 3,


n = 4 ou n = 6. (A condio n > 2 no problema original garante ser
n 2 positivo. Sem essa condio existiriam as solues n = 1, n = 0
ou n = 2).
Resumo

Adaptado do artigo
Seis problemas no triviais equivalentes
Zalman Usiskin, RPM 04.

RPM OBMEP

Os seis problemas formam um grupo de problemas, no triviais


equivalentes que podem ser usados em classes de ensino fundamental e
mdio. fcil desenvolver outros grupos de problemas mais apropriados
para o uso em lgebra Elementar ou Geometria. Tais grupos de
problemas podem ser usados para demonstrar o poder de um pouco de
Matemtica abstrata na resoluo de exerccios que, primeira vista,
pareciam no relacionados.

21

O menino

RPM OBMEP

No havia sada. Teria que esperar por trs horas o


prximo vo para Salvador. Arquiteto por formao e
profisso, tinha que apresentar um projeto na manh
seguinte, numa cidade prxima capital da Bahia.
Assentei-me como pude. Teria que olhar para aquele
relgio pendurado no teto por trs horas. Como se no
bastasse, o relgio registrava os segundos. Relgios que
registram segundos demoram mais que os que no o
fazem.

22

Alguns apelam para palavras cruzadas, outros giram os


polegares e eu, como o vcio do cachimbo entorta a boca,
trao em folhas de papel as formas que se me apresentam
no ambiente que alcanado pelas retinas. Lpis e papel
na mo, registrava dois lances de escada e uma escada
rolante que surgiram a minha frente. Mal traara as
primeiras linhas, deparei-me com uma questo que me
intrigou: quantos degraus deveria desenhar na escada
rolante? Em vo, tentei contar os degraus visveis. Se a
escada parasse, poderia cont-los. Tive mpetos de apertar
o boto vermelho prximo ao corrimo, onde se lia
PARAR. Meu censurador no permitiu que o fizesse.
Fiquei ali, inerte, com o cachimbo na mo e sem poder
fumar.

Um menino sentou-se ao meu lado, brincando com uma bola. Sem


tirar os olhos da bola, ela disse em voz clara e pausada:
Pepino no parece inreal?
Olhei-o, ligeiramente, com o canto dos olhos e, sem nada dizer,
retornei ao meu cachimbo apagado. Alguns instantes depois, senti minha
camisa ser puxada e escutei novamente:
Pepino no parece inreal?
Dessa vez, com uma mo segurando a bola e com a outra puxando a
minha camisa, ele me olhava firmemente.
No inreal, irreal.
Pois , no parece?
Aquela insistncia irritou-me. Eu, diante do mais intrincado problema
da existncia humana quantos degraus ficam visveis quando a escada
rolante pra e aquele menino me questionando sobre a realidade de
um pepino! Tentando dissuadi-lo, resolvi apresentar-lhe a complexidade
do problema que me afligia.
Olha, menino, estou tentando desenhar aquelas escadas e no sei
como acabar o desenho da escada rolante. Quantos degraus devo
desenhar? Meu desenho est parado e a escada est subindo. Se a escada
parasse de repente, quantos degraus ficariam visveis?
Sem nada dizer, colocou a bola sobre a cadeira, subiu e desceu a
escada (que sobe). Apontando para o relgio, disse:

E repetiu sua viagem ao vo da escada, mostrando-me que, no mesmo


tempo em que dava um passo para subir, dava dois para descer.
Novamente sem nada dizer, comeou a subir a escada rolante, contando
os passos: um, dois, trs, ..., num total de vinte passos. Do alto da escada,
olhou-me como quem estivesse fazendo a mais bvia das coisas, e
comeou a descer a mesma escada rolante, contando os passos: um,
dois, trs, ..., num total de trinta e cinco passos.
Em seguida tomou o lpis e o papel de minhas mos e completou,
com traos infantis, o meu desenho.

RPM OBMEP

Eu deso a escada duas vezes mais rpido do que subo.

23

Nenhum censurador poderia me conter. Levantei-me bruscamente e


apertei o boto vermelho. Ansioso, comecei a contar os degraus. Para
meu espanto, correspondia ao desenho do menino. Com a maior seriedade
que j tive em minha vida voltei-me para o menino e perguntei-lhe:
Por que o pepino parece inreal?
Quantos degraus o menino desenhou?
Vamos resposta:
Vamos tomar como unidade de tempo o tempo no qual o menino d
um passo subindo a escada. Seja n o nmero de degraus da escada rolante
que desaparecem (ou surgem) na unidade de tempo. Como o menino
deu 20 passos para chegar ao topo da escada, ele demorou 20 unidades
de tempo.
Isso significa que desapareceram 20n degraus. Chamando de N o
nmero de degraus visveis, temos:
N = 20 + 20n ou n =

N 20
.
20

(1)

O menino deu 35 passos para descer a escada rolante (que sobe).


Lembremos que a frequncia de seus passos duas vezes maior na
descida que na subida. Ou seja, o tempo de dar dois passos descendo
igual ao de um passo subindo. Cada passo na descida demora

1
da
2

unidade de tempo.
Ele demorou

35
unidades de tempo para descer a escada. Isso
2

significa que surgiram 35n degraus novos. Assim,


2

RPM OBMEP

N = 35

24

35n
2

ou n =

70 2 N
.
35

Igualando (1) e (2):

N 20 70 2 N
=
20
35

(2)

35N 700 = 1400 40N


ou
75N = 2100, de onde

2100
= 28
75
O menino desenhou 28 degraus.
N=

Adaptado do artigo
O menino
Ledo Vaccaro Machado, RPM 42.

Desafio

Resposta na pgina 171.

RPM OBMEP

Distribuir os nmeros de 1 a 9 dentro dos pontos brancos (de interseco),


sem repetir, de forma que a soma dos nmeros pertencentes circunferncia externa seja exatamente igual soma dos nmeros pertencentes
a cada uma das circunferncias internas.

25

O problema dos cinco discos:


sorte ou sabedoria?

Neste artigo queremos mostrar uma curiosidade sobre o


antigo problema dos cinco discos. A mais bela
apresentao desse problema encontra-se em O homem
que calculava (Tahan, Malba 32a edio. Record, Rio
de Janeiro, 1986). Nele contada uma lenda onde trs
prncipes muito sbios e conhecedores da Matemtica
que pretendiam casar-se com a princesa Dahiz, filha do
rei Cassim.

RPM OBMEP

A prova dos cinco discos foi proposta por um grande


sbio da corte para decidir qual dos trs pretendentes era
o mais inteligente.

26

Foram mostrados aos prncipes cinco discos, sendo dois


pretos e trs brancos, todos de mesmo peso e tamanho.
Em seguida vendaram-lhe os olhos e, ao acaso, foi
pendurado um desses discos s costas de cada um dos
trs. Disse o rei: Cada um de vs ser interrogado
particularmente e aquele que descobrir a cor do disco
que lhe coube por sorte, ser declarado o vencedor. O
primeiro a ser interrogado poder ver os discos dos
outros dois, ao segundo ser permitido ver o disco do
terceiro, e o terceiro ter que formular a resposta sem
ver nada. Aquele que der a resposta certa ter que
justific-la.

Aconteceu ento que o prncipe Camoz quis ser o primeiro. Viu os


dois discos dos seus adversrios e errou. Em seguida, sabendo que
Camoz havia errado, o prncipe Benefir se prontificou em ser o segundo,
mas tambm errou. Aradim, o terceiro prncipe, acertou com absoluta
segurana. Qual foi a resposta do prncipe Aradim e como ele descobriu?
Esse o problema dos cinco discos. Malba Tahan d uma inteligente
soluo desse problema, onde conclui tambm que Aradim foi
considerado o mais inteligente entre os trs prncipes.
Eis a soluo de Malba Tahan: o prncipe Aradim afirmou que o seu
disco era branco e justificou da seguinte maneira: Se Camoz (o primeiro
a falar) tivesse visto dois discos pretos, ele obviamente teria acertado.
Como ele errou, conclui-se que viu dois discos brancos, ou um preto e
um branco. Na hiptese de Benefir ter visto em minhas costas um disco
preto, ele (usando o mesmo raciocnio que fiz com relao a Camoz)
teria acertado. Logo, ele s pode ter visto um disco branco e, portanto, o
meu disco branco.
A curiosidade que pretendemos apresentar que, sob o ponto de
vista matemtico e levando em conta somente o acerto da cor do disco,
a chance de erro dos dois anteriores era bem pequena, o que torna
discutvel a concluso de que Aradim fosse mais inteligente que Camoz
ou Benefir.
Com efeito, vamos calcular as probabilidades de acerto da cor do
disco de cada um dos trs prncipes, levando em conta que todos eles
so sbios.
As possveis distribuies dos discos

Ento a ordem em que os prncipes se apresentaram para serem


interrogados pode ser representada por uma terna ordenada (C, B, A).
A ttulo de exemplo, perguntamos quantas maneiras diferentes podem
C possuir disco branco, B possuir disco preto e A possuir disco preto?
Isto , de ocorrer (b, p, p).

RPM OBMEP

Sejam b = (disco branco) e p = (disco preto). Por simplicidade


escrevemos
A = Aradim, B =Benefir e C = Camoz.

27

Sabemos que existem trs discos brancos b1, b2 e b3 e dois discos


pretos p1 e p2. Por uma simples contagem, obtemos seis maneiras
diferentes de ocorrer (b, p, p), a saber:
(b1, p1, p2), (b1, p2, p1), (b2, p1, p2), (b2, p2, p1),
(b3, p1, p2) e (b3, p2, p1).
claro que o nmero total de maneiras em que podem ser distribudos
os discos aos prncipes A5, 3 = 60. Descrevendo esses casos, obtemos:
Eventos

Frequncia

E1 = (b, b, b)

E2 = (p, b, b)

12

E3 = (b, p, b)

12

E4 = (b, b, p)

12

E5 = (p, p, b)

E6 = (p, b, p)

E7 = (b, p, p)

Lembretes
a) Se os conjuntos unitrios de um espao amostral finito U tm todos
a mesma probabilidade, ento a probabilidade de um evento A
qualquer de U ser dada por:

P ( A) =

n( A)
n(U )

onde n(A) o nmero de elementos do evento A e n(U) o


nmero total de elementos do espao amostral U.
b) Nas mesmas condies de a), se A1, A2, ..., An so eventos disjuntos
entre si,

RPM OBMEP

P ( A1 A2 ... An ) =

28

n( A1 ) + n( A2 ) + ... + n( An )
.
n(U )

Como o problema afirma que a escolha dos discos feita ao acaso,


segue-se que o espao amostral associado ao problema satisfaz as
condies necessrias para a validade de a) e b). No difcil verificar
tambm que o acerto da cor do disco admite uma estratgia que maximiza

a probabilidade de vitria de cada concorrente e garante, com


probabilidade 1, a existncia de um vencedor, que certamente ser nico
uma vez que o processo terminaria no momento em que um dos
concorrentes acertasse a cor do seu disco. Como os concorrentes
supostamente so sbios, razovel admitir que eles seguiro a melhor
estratgia em cada situao e portanto teremos
P(C) + P(B) + P(A) = 1
onde P(C), P(B) e P(A) so, respectivamente, as probabilidades de
vitria de C, B e A.
A estratgia tima e a correspondente probabilidade de vitria de C.
Se C vir dois discos pretos nos seus adversrios, saber que restam
trs discos brancos. Responder ento com absoluta segurana que possui
um disco branco. Assim o evento E7 lhe favorvel. Caso C veja dois
discos brancos, saber que restam dois discos pretos e um disco branco.
Logo responder possuir disco preto, contando com a probabilidade
2/3 de acertar. Consequentemente, o evento E2 lhe favorvel e o
evento E1 lhe desfavorvel. Suponhamos agora que C tenha visto
um disco branco e um disco preto em seus concorrentes. Concluir que
restam dois discos brancos e um disco preto. Logo, dever responder
que possui um disco branco, contando com a probabilidade 2/3 de
acertar. Segue que os eventos E3 e E4 lhe so favorveis e o evento E6
lhe desfavorvel.
Em resumo, usando essa estratgia, C ir acertar na hiptese de ter
ocorrido qualquer um dos eventos disjuntos E2, E3, E4 ou E7 e ir
errar se houver E1, E5 ou E6. Segue-se, ento, que:

n( E2 ) + n( E3 ) + n( E4 ) + n( E7 ) 6 + 12 + 12 + 12 7 .
=
=
n(U )
60
10

Isso mostra que a probabilidade vitria do prncipe Camoz, o


primeiro candidato, de 70%, contando com a sua sabedoria, restando
assim apenas 30% de probabilidade para que os outros dois prncipes
tivessem chance de serem apenas interrogados.

RPM OBMEP

P (C ) =

29

Considerando ainda que Aradim s seria interrogado caso Benefir (o


segundo interrogado) tambm errasse, pode-se mostrar que ele o que
teria a menor chance de ser escolhido como noivo de Dahiz.
No entanto, Aradim possuidor de muita sorte, pois os dois primeiros
concorrentes erraram.
Para completar, a probabilidade de Benefir acertar de 20% e a
probabilidade do prncipe Aradim acertar de apenas 10%.
A reabilitao de Aradim
Esse clculo, entretanto, diz respeito s ao fato de acertar, ou no, a
cor do seu disco. Acontece que o rei dissera que os prncipes deveriam,
tambm, justificar a resposta correta.
Fica a pergunta do que o rei entendia por justificar.
Seria aceitvel, em caso de dvida, uma adivinhao educada, isto ,
uma opo pela alternativa mais provvel? Ou seria necessria uma
explicao lgica de como se chegou nica alternativa correta possvel?
Neste caso, quais seriam as probabilidades de vitria de cada um dos
trs concorrentes?
Adaptado do artigo

RPM OBMEP

O problema dos cinco discos: Sorte ou Sabedoria?


Ma-To Fu e Roberto Elias, RPM 11.

30

Calculadora padro:
um problema interessante

Suponhamos que voc tem uma calculdadora de bolso,


padro, de 8 dgitos, que efetua as quatro operaes,
+, , , e extrai razes quadradas. Ser possvel, usando
essa calculadora, extrair a raiz n-sima de um nmero
qualquer?
Na verdade, dado um nmero real x, no negativo, usando
, +, , , possvel achar xp/q,
onde p e q so nmeros naturais. Vamos explicar como
isso possvel mostrando alguns exemplos.
somente as teclas

Exemplo 1: Calcule
Seja x =

5.

5 . Ento x3 = 5. Multiplicando por x os dois

lados da igualdade, obtemos x4 = 5x ou x = 4 5 x .


Inicialmente criamos uma sequncia de nmeros reais
sendo x1 uma aproximao de

5 (por exemplo, x1 = 1)

e xn +1 = 4 5 xn , n = 1, 2, 3,". Vamos mostrar os valores

sequncia converge para

5.

Tomemos x1 = 1. Ento, x2 = 4 5 , x3 = 4 5 4 5 , e assim


por diante.

RPM OBMEP

de alguns termos da sequncia obtidos na calculadora e,


em seguida vamos dar uma justificativa do por que a

31

Vejamos os valores calculados sendo que a notao [*] significa


que apertamos a tecla *:
[5] [
][
] 1.4953487
Com o valor anterior mantido na tela, fazemos
[] [5] [=] [

] 1.653591

][

Repetindo sempre os comandos [] [5] [=] [


1.6957019
1.7063962
1.7090802
1.7097519
1.7099199
1.7099619
1.7099724
1.7099750
1.7099757
1.7099758
1.7099759
1.7099759,
o que indica que uma aproximao para
1,7099759.

][

] obtemos

5 com erro menor que 107

Uma idia do por que funciona: consideremos a expresso


4 4 4 4
5 5 5 5"
4 4 4 4
5 5 5 5"

com infinitos radicais. Podemos escrever


1

1 1

= 5 4 516 5 64 5 256 " = 5 4 +16 + 64 + 256 " .


1

1 1
1
1
1
4
Sabemos que 4 + 16 + 64 + 256 + " = 1 1 = 3 , pela frmula da
4

RPM OBMEP

soma dos termos de uma PG infinita, logo, temos

32

4 4 4 4
5 5 5 5"

= 51/3 =

5 = x. Ou seja, admitindo que existe o limite

de xn quando n tende ao infinito, ento, esse limite ser

5.

Um outro modo de verificar o valor x


(admitindo que esse limite exista) :

do limite da sequncia

x 4 = 54 54 54 5" = 5 x. Como x

x = 4 54 54 54 5" , logo,

diferente de zero, a igualdade implica x =

5.

Exemplo 2: Calcule 5 5 .
Seja x = 5 5 . Ento, x5 = 5 e vemos que x8 = x5x3 = 5x3 que implica
8
x = 5 x3 . Vamos construir uma sequncia x1, x2, x3, ..., xn, ..., com

x1=1 e os outros termos como na tabela, usando a calculadora para obter


os nmeros aps as flechas.
1

x2 = 8 5 x13 = 8 5 = 58 =

5 1.2228445
1+ 3

8 8
x3 = 8 5 x23 = 5 53 = 58 64 =

x4 = 8 5 x33 ==
x5 = 5
x6 = 5

125

59

125 1.3186680
1+ 3 + 9

= 58 64 864 1.3565069

1 + 3 + 9 + 27
8 82 83 84
1 + 3 + 9 + 27 + 81
8 82 83 84 85

#
Aqui, a cada passo, utilizamos apenas as teclas [], [5], [=] e [

].

Novamente aceitando que a sequncia escolhida converge para um


limite x diferente de zero, podemos fazer:

x=5

1 3 9 27
+ + + +"
8 82 83 84

1
= 55

= 5 5.

RPM OBMEP

1 3
9 27
1
+
+ +
+ " = 8 = , logo,
2
3
4
3
8 8
1 8 5
8
8

33

Ou, como no exemplo anterior, elevando x a oitava potncia, obtemos


x = 5x3, e como x diferente de zero vem que x = 5 5 .
8

A argumentao dos exemplos apresentados foi baseada no fato de


que a sucesso x1 , x2 , x3 , " , x n , x n +1 , " converge. Isso realmente
acontece? Vamos responder a essa pergunta no caso do ltimo exemplo.
Os grficos das funes y = x8 e y = 5x3 esto mostrados na figura
abaixo e como o nmero procurado, x = 5 5 , satisfaz x8 = 5x3, vemos
que x ser a bscissa no nula do ponto de interseco dos dois grficos.
y=x 8





1

2

y=5x 3

5

Partindo, por exemplo, de x1=1, examinemos a sucesso de pontos,


representados no grfico:
8

P1 = (1, 0); P2 = (1, 5); P3 = ( 8 5 , 5); P4 = ( 8 5 , 5 53 );


8 8

8 8

P5 = ( 5 53 , 5 53 ); P6 = ( 5 53 , 5 53 59 ); "
Observe que os pontos P2, P4, P6, ... pertencem ao grfico de
y = 5x3 e que os pontos P3, P5, P7, ... pertencem ao grfico de y = x8.
A sucesso de todos os pontos converge para o ponto de interseco

RPM OBMEP

dos dois grficos que o ponto P = ( 5 5 , 5 58 ) , o que mostra que a

34

sucesso das abscissas x1, x2, x3, ..., xn, ..., converge para x = 5 5 .
Isso mostrado rigorosamente usando-se tcnicas de Anlise Real.
Quanto a escolha x1=1, ela foi feita simplesmente para facilitar os

clculos e tambm por sabermos que 5 5 est prximo de 1. Se


tivssemos escolhido qualquer outro valor, por exemplo, x1=100, o limite
da sucesso x1, x2, x3, ..., xn, ... continuaria satisfazendo x8 = 5x3.
y = x 8 e y = 5x 3 esto mostrados na figura a seguir e como o

nmero procurado, x = 5 5 , satisfaz x8 = 5 x 3 , vemos que x ser


exatamente a abscissa, com x 0 , do ponto de interseco dos grficos
de y = x 8 e y = 5x 3 .
Ora, o fato algbrico de que a raiz x 8 = 5x 3 existe, equivalente ao
fato de que os grficos se cortam. Para 0 x 5 5 1,379, o grfico de
y = x 8 estar acima do grfico de y = 5x 3 .

Partindo de um valor arbitrrio, por exemplo, x1 = 1 , examinemos a


seguinte sucesso de pontos do plano cartesiano, representados no grfico
8 3
acima: P1 = (1, 0); P2 = (1, 5); P3 = (8 5 , 5); P4 = (8 5 , 5 5 );

P5 = ( 8 5 8 5 3 , 5 8 5 3 ); P6 = ( 8 5 8 5 3 , 5 8 5 3 8 5 9 ); "
Observe que os pontos P2 , P4 , P6 , " esto sobre o grfico de y = 5x 3
8
e que os pontos P3 , P5 , P7 ," esto sobre o grfico de y = x .

Os pontos P1 , P2 , P3 ,", Pn ," convergem para o ponto P = (51 5 , 58 5 ) ,


que exatamente o ponto de interseco dos dois grficos. ento bvio
que a sucesso x1 , x2 , x3 , " , x n , x n +1 , " converge exatamente para 5 5 .
Pode-se mostrar, rigorosamente, usando as tcnicas de Anlise Real, que
isso realmente acontece.
Quanto a escolha, x1 = 1, ela foi feita simplesmente para facilitar os
escolhido qualquer outro valor para x1 , por exemplo, x1 = 100, o limite da
sucesso, que estamos supondo existir, continuaria satisfazendo x8 = 5x3 .

RPM OBMEP

clculos e tambm por sabermos que 5 5 est prximo de 1. Se tivssemos

35

Voc certamente j percebeu o procedimento geral: se quisermos


calcular x = n a ou seja, achar x tal que xn = a, devemos transformar
essa igualdade de modo a obter, do lado esquerdo, um expoente que seja
uma potncia de 2. Isso feito multiplicando-se os dois membros por
uma potncia conveniente de x. Assim, por exemplo, se x = 11 a ento
x11 = a, que implica x16 = x11x5 = a x5 ou x = 16 ax 5 e utilizaremos a
5
16
sucesso x1, x2, x3, ..., xn, ..., na qual xn +1 = axn .

Adaptado dos artigos


Vamos usar a calculadora?
Hideo Kamayama e Eduardo Wagner, RPM 26.

RPM OBMEP

Vamos continuar usando a calculadora


Joo Bosco Pitombeira de Carvalho, RPM 51.

36

Uma equao interessante

H algum tempo, um amigo professor mostrou-me a


equao
3

2x 1 + 3 x 1 = 1

e fez a seguinte observao: apesar de, no decorrer da


resoluo, elevarmos as equaes somente a potncias
mpares (duas elevaes ao cubo), ainda assim,
surpreendentemente, aparece uma raiz falsa. Por qu?
Antes de mostrar como o professor resolveu a equao,
vejamos o porqu da sua surpresa.
Sabemos que x = y xn = yn, x, y R, n N,
mas a recproca desta afirmao s verdadeira se n for
mpar. Isto ,
xn = yn x = y, x, y R, se n for mpar.
fcil ver que a propriedade xn = yn x = y no vale se
n for par basta observar que 52=(5)2 e 5 5. O
que vale :
As falsas solues aparecem nitidamente quando,
resolvemos equaes irracionais. Vejamos um exemplo:
Quais so as solues da equao

2x 3 = x 3 ?

RPM OBMEP

xn = yn | x | = | y |, x, y R, n N.

37

2 x 3 = x 3 ( 2 x 3 ) 2 = ( x 3) 2 2 x 3 =
3

= x 2 6 x + 9 x 2 8 x + 12 = 0 x = 6 ou x = 2.

As passagens 2, 3 e 4 so equivalncias, mas a recproca da implicao


1 no verdadeira. por isso que, aps resolvermos a equao,
testamos as razes encontradas, para ver se elas, de fato, satisfazem a
equao inicial. No exemplo, 6 raiz de (2), mas 2 no .
Portanto, estamos acostumados com o aparecimento de falsas razes
na resoluo de equaes irracionais.
Mas, no exemplo que o professor apresentou, o fato de aparecer uma
raiz falsa era estranho, pois a resoluo da equao exigia apenas que
seus membros fossem elevados ao cubo e sabemos que, em R,
x3 = y3 x = y.
Vejamos como o professor resolveu a equao:
3

2x 1 + 3 x 1 = 1 .

(1)

Elevando ao cubo, obtemos

2 x 1 + 3( 3 2 x 1) 2 .3 x 1 + 3 3 2 x 1.( 3 x 1) 2 + x 1 = 1 (2)
3 x 2 + 3 3 2 x 1.3 x 1( 3 2 x 1 + 3 x 1) = 1

(3)

o termo entre parnteses vale 1 ( a prpria equao (1)!)

3 x 2 + 3 3 (2 x 1)( x 1) = 1

(4)

3 x + 3 3 (2 x 1)( x 1) = 3

(5)

2 x 2 3x + 1 = 1 x

(6)

2x2 3x + 1 = (1 x)3
2x 3x + 1 = 1 3x + 3x x
RPM OBMEP

38

x x = 0.
3

(7)
3

(8)
(9)

E, portanto, x = 0 ou x = 1.
Verifica-se, por substituio em (1), que 1 soluo, mas 0 no .

Onde e por que apareceu essa falsa raiz?


Sugiro que o leitor tente responder a essa pergunta antes de prosseguir.
Observe que x = 0 no soluo das equaes (1), (2) e (3), mas
soluo das equaes a partir de (4). Na verdade, (1), (2) e (3) so
equivalentes entre si (possuem o mesmo conjunto soluo), e as equaes
de (4) a (9) tambm so equivalentes entre si, mas (3) e (4) no so
equivalentes. Foi nessa passagem que fizemos algo ilcito.
O que fizemos para passar de (3) a (4)? Ora, usamos novamente a
equao (1) substituindo 3 2 x 1 + 3 x 1 por 1, e esse procedimento
no gera uma equao equivalente anterior. Tendo duas equaes
equivalentes, (1) e (3), se substituirmos uma na outra, obtemos uma
nova equao que consequncia das anteriores, mas no ,
necessariamente, equivalente a elas. Assim (3) (4), mas no vale a
recproca.
Vejamos um exemplo onde esse fato mais evidente:
x = 2 (o conjunto soluo {2}),
2 = x ( equivalente a de cima).
Substituindo uma na outra, obtemos
x = x, cujo conjunto soluo R!
Assim, o aparecimento de uma raiz falsa no est ligado ao fato de a
equao ser irracional nem s potncias que tomamos, e sim, ao
procedimento da resoluo.

Vamos ilustrar o aparecimento de falsas razes atravs de mais dois


exemplos:
x = 1 x (e, portanto, x = 1/2).
Se elevarmos ambos os membros ao cubo, teremos:

RPM OBMEP

Mais uma palavra sobre esse fato: o truque utilizado na passagem


de (3) para (4) til, pois limpou a equao, mas no uma
equivalncia no podemos perder de vista a equao original. Situaes
como essa so comuns, por exemplo, na trigonometria quando usamos,
numa equao, a identidade sen2 x + cos2 x = 1.

39

x = l x x3 = (l x)3 x3 = 1 3x + 3x2 x3
(substituindo x por 1 x)
x3 = 1 3(1 x) + 3x2 x3 2x3 3x2 3x + 2 = 0
x = 1/2; x = 1; x = 2.
Outro exemplo:
x = 1.
x = 1 (x l)2 = 0 x2 2x + 1 = 0
(substituindo x por 1)
x2 2.1 + 1 = 0 x2 = 1 x = l ou x = l.
Adaptado do artigo

RPM OBMEP

Uma equao interessante


Cludio Possani, RPM 19.

40

PAINIS

Painel I
O nmero 12
Ao nmero 12 so atribudos muitos significados, sobretudo
de ndole religiosa ou espiritual, cuja influncia provocou
alguns efeitos na organizao de nosso cotidiano.
Na historiografia judaico-crist, temos os 12 filhos de Jacob,
filho de Isaac e neto de Abrao, dos quais derivaram as 12
tribos de Israel. Refere-se ainda que Jacob usava um peitoral
sobre o qual haviam sido incrustadas 12 pedras preciosas
que so a revelao de 12 poderes csmicos. Tambm a
coroa usada na sagrao da monarquia inglesa tem 12 pedras
preciosas.
So 12 os deuses principais da mitologia grega, que vivem
no Monte Olimpo. O ano tem 12 meses. O zodaco divide a
esfera celeste em 12 casas. O relgio est dividido em 12
horas.

No consigo acreditar que os nmeros tenham algum


significado que os transcenda, porm creio que a natureza
abstrata dos nmeros propicia a sua utilizao como
representantes de significados que os transcendem.
Atribuies e interpretaes do significado do nmero 12,
sobretudo relacionadas com questes religiosas e sociais,
decorrem do fato, do domnio aritmtico, de 12 ser o produto
de 3 por 4 e, alm disso, parece-me que o fato de 12 ter
muitos divisores, 1, 2, 3, 4, 6 e 12, pode ter ajudado na sua

RPM OBMEP

A bandeira da Unio Europia tem 12 estrelas douradas,


que, segundo a Comisso Europia, representam a
solidariedade e harmonia entre os povos da Europa, porque
o nmero 12 tradicionalmente um smbolo de perfeio,
de plenitude e de unio.

41

projeo. que, entre as quatro operaes aritmticas elementares


efetuadas no conjunto dos nmeros inteiros, a diviso a nica que nem
sempre d resultados inteiros, sendo uma minoria os casos em que isso
acontece. Conjugando esse fato com a circunstncia de que a maioria
dos seres humanos se mostra mais disponvel para o clculo com inteiros
do que com outros tipos de nmeros, compreende-se a importncia dada
aos nmeros inteiros que se evidenciam por terem muitos divisores como
12, 24 e 60, por exemplo.
Mas o objetivo deste artigo mostar uma interveno do nmero 12
numa relao entre os domnios algbrico e geomtrico. Consideremos
uma funo quadrtica f(x) = ax2 + bx + c com a > 0 e = b2 4ac >
0, condio essa, como bem sabemos, que implica a existncia de duas
razes reais distintas da equao ax2 + bx + c = 0. Essas razes so as
abcissas dos pontos em que a parbola, grfico de f, intersecta o eixo x.
Consideramos o tringulo formado
pelos pontos A, B e pelo ponto V, vrtice
da parbola, como na figura. Esse tringulo
issceles, j que AV = VB. Vamos
verificar que o muito falado nmero 12
relaciona a funo quadrtica com a
possibilidade de o tringulo AVB ser
equiltero.
Como conhecido, as coordenadas dos pontos A, V e B so dadas
em funo dos coeficientes da funo quadrtica:
A=(

b
; 0),
2a

B=(

b +
; 0)
2a

V =(

b
; ).
2a 4a

Podemos usar o teorema de Pitgoras para calcular AV:


2
b (b
AV =

+
2a
4a

RPM OBMEP

42

Por outro lado, AB =

ou

AV =

b + b

=
.
2a
2a
a

( + 4)
.
4a

Para que AVB seja equiltero, devemos ter AB = AV, o que


equivalente a 12 = 2 ou, como > 0, equivalente a = 12.
Adaptado do artigo
O nmero 12
Carlos Grosso, RPM 67.

Painel II
Sexta-feira 13
As pessoas criaram um mito sobre as sextas-feiras 13, dizendo que
essas datas so propcias para ocorrer coisas macabras, horrveis... Parece
que nas ltimas dcadas esse mito foi bastante reforado e divulgado
pela srie de filmes Sexta-feira 13 que o cinema exibiu. Mas no so
apenas coisas ruins que esto ligadas sexta-feira 13, muito pelo
contrrio; temos um belo problema de Matemtica: todo ano h pelo
menos uma sexta-feira 13.

1o de janeiro dia 1
2 de janeiro dia 2
3 de janeiro dia 3
...
13 de janeiro dia 13
...
13 de fevereiro dia 44

13 de maro dia 72
...
13 de abril dia 103
...
13 de maio dia 133
...
13 de junho dia 164

RPM OBMEP

Para verificarmos o prometido, vamos inicialmente enumerar os dias


13 de um determinado ano. Para isso imaginemos um ano de 365 dias
(se o ano tiver 366 dias, o mesmo mtodo funciona!). Lembre que, num
ano de 365 dias, os meses de janeiro, maro, maio, julho, agosto, outubro
e dezembro tm 31 dias, enquanto abril, junho, setembro e novembro
tm 30 dias e fevereiro tem 28 dias. Assim, temos que:

43

...
13 de julho dia 194
...
13 de agosto dia 225
...
13 de setembro dia 256
...
13 de outubro dia 286
...
13 de novembro dia 317
...
13 de dezembro dia 347

Assim, temos que os dias 13 de um determinado ano de 365 dias so


13, 44, 72, 103, 133, 164, 194, 225, 256, 286, 317 e 347, que, quando
divididos por 7 (uma semana tem sete dias), deixam restos 6, 2, 2, 5, 0, 3,
5, 1, 4, 6, 2 e 4, respectivamente. Perceba que todos os restos possveis
de uma diviso por 7 apareceram, isto , 0, 1, 2, 3, 4, 5 e 6. Assim,
perceba que:

RPM OBMEP

se a primeira sexta-feira do ano for dia x (x no mximo


7), as sextas-feiras seguintes sero os dias x + 7, x + 14, x
+ 21, ... Se, por exemplo, x for 7, ento todas as sextasfeiras do ano cairo nos dias 7, 14, 21, 28, 35, 42, ... do
referido ano. Como entre os dias 13 h um que mltiplo
de 7 (o dia 133), segue que esse dia ser uma sexta-feira 13
(isso ocorreu em 2005; veja que 13 de maio de 2005 foi
uma sexta-feira 13). Seguindo o mesmo raciocnio, se a
primeira sexta-feira do ano fosse dia 6 de janeiro, ento as
sextas-feiras seriam os dias 6, 13, 20, 27, 34, 41, ... Como
entre os dias 13 h um cujo resto da diviso por 7 6 (o
dia 13), segue que, nesse ano, 13 de janeiro seria uma sextafeira 13.

44

Esse raciocnio mostra que em qualquer ano existe pelo menos uma
sexta-feira 13. Perceba que pode haver mais de uma sexta-feira 13. Se,
por exemplo, o dia 2 de janeiro for uma sexta-feira, ento as demais
sextas-feiras desse ano sero os dias 2, 9, 16, 23, 30, 37,44, ..., ou seja,
os dias que deixam resto 2 quando divididos por 7.
Assim, em um ano de 365 dias em que 2 de janeiro uma sexta-feira,

os dias 44, 72 e 317 (que divididos por 7 deixam resto 2) seriam sextasfeiras 13. Noutras palavras, 13 de fevereiro, 13 de maro e 13 de novembro
seriam sextas-feiras 13 (que ano azarado, hein??? Prepare-se, 2009 ser
assim!).
Adaptado do artigo
Sexta-feira 13

Carlos A. Gomes, RPM 59.

Painel III
O jogo de bilhar
Estava numa pousada, no salo de jogos, observando uma partida de
bilhar. Em dado momento, apresentou-se a situao ilustrada na figura,
sendo que o jogador precisava acertar a bola cinza, mas no podia bater
na bola preta.
Para ajudar, um amigo do jogador
adotou a estratgia:

mediu, com um outro taco,


colocado apoiado na direo
perpendicular borda da mesa,
como na figura, a distncia d da
bola cinza at o ponto B, na borda
da mesa.

marcou nesse taco o ponto A tal


que a distncia BA vale d.

Ao bater no ponto C, na borda da


mesa, a bola branca, no movimento
refletido, acertou a bola cinza.
A pergunta que me ocorreu foi: Por
que deu certo? A resposta fundamentase na lei fsica que afirma que, na

RPM OBMEP

disse ao jogador para mirar no ponto


A e bater na bola branca.

45

situao descrita, a medida do ngulo


de incidncia da bola, ao bater na mesa,
igual ao ngulo de reflexo. O ngulo BCA
e o ngulo de incidncia so opostos pelo
vrtice, logo tm a mesma medida ,
mostrando que a reta r a bissetriz do
ngulo DCA, sendo D um ponto da
trajetria de reflexo. Em consequncia, o simtrico de A, em relao a
r, que o ponto no qual est a bola cinza, pertence reta CD. Logo, a
trajetria de reflexo da bola branca passa pela bola cinza.
Alm disso, a estratgia adotada fornece a trajetria de menor percurso
para a bola branca atingir a bola cinza nas condies do problema. Isso
garantido pelo teorema a seguir, atribudo a Heron, matemtico de
Alexandria que viveu no primeiro sculo depois de Cristo.
Teorema de Heron
Dada uma reta r e dois pontos P e Q, no mesmo lado da reta r, o
ponto R sobre a reta r tal que a distncia PR + RQ a menor possvel
aquele em que os ngulos que os segmentos PR e RQ fazem com a
reta r so iguais.
Demonstrao do teorema
Seja Q o simtrico de Q em relao reta r. Por hiptese, a reta
r bissecciona o ngulo QRQ. Segue, por congruncia de tringulos, a
igualdade QR = QR. Seja R qualquer ponto sobre a reta r, diferente
de R.
Ento,
QR + RP = QR + RP = QP.
Mas
QP < QR + RP= QR + RP.
RPM OBMEP

Logo, QR + RP < QR + RP.

46

Adaptado do artigo
O jogo de bilhar
Jos Carlos Magossi, RPM 69.

Painel IV
Codificando e decifrando mensagens
Operaes de servios disponveis na Internet, movimentaes bancrias
e outras transaes eletrnicas necessitam da criptografia para
comunicao confidencial de dados.
A palavra criptografia tem origem grega (kripto = escondido, oculto;
grapho = grafia) e define a arte ou cincia de escrever mensagens em
cdigos, de forma que somente pessoas autorizadas possam decifr-las.
A criptografia to antiga quanto a prpria escrita; j estava presente no
sistema de escrita hieroglfica dos egpcios e os romanos utilizavam
cdigos secretos para comunicar planos de batalha. Contudo, desde
aquele tempo, seu princpio bsico continua o mesmo: encontrar uma
transformao (funo) injetiva f entre um conjunto de mensagens
escritas em um determinado alfabeto (de letras, nmeros ou outros
smbolos) para um conjunto de mensagens codificadas. O fato de f ser
inversvel a garantia de o processo ser reversvel e as mensagens
poderem ser reveladas pelos receptores. O grande desafio de um processo
criptogrfico, portanto, est em ocultar eficientemente os mecanismos
(chaves) para a inverso de f, de modo que estranhos no possam fazlo.

Descreveremos aqui dois exemplos elementares de processos criptogrficos, sendo o primeiro acessvel inclusive para alunos do ensino
fundamental.

Portanto, cifrar uma mensagem recai no problema de permutar


nmeros por meio de uma regra f. Pode-se fazer isso, de forma muito

RPM OBMEP

Inicialmente, relacionamos nmeros ao alfabeto (o smbolo #


representa um espao em branco) que vamos utilizar nos modelos. Assim:

47

prtica, por exemplo, atravs das funes afins f(x) = ax + b com a, b


inteiros, a 0, definidas no conjunto {0, 1, ..., 26}. Suponhamos que
Ana e Ivo desejem trocar mensagens sigilosas utilizando o alfabeto
escolhido. O primeiro passo a tomarem definirem a funo cifradora,
digamos f(x) = 2x 3. Assim, por exemplo,
mensagem
R E V I S T A
R P M
Ana associa a sequncia numrica 18 5 22 9 19 20 1 0 18 16 13

mas transmite a Ivo a seqncia numrica obtida pelas imagens de f,


isto ,
33 7 41 15 35 37 1 3 33 29 23.

x+3
nessa
2
sequncia e utilizando a correspondncia alfabeto-numrica, obtm a
mensagem original.
1
Ao receb-la, Ivo, calculando a imagem de f ( x) =

Depois de os alunos dominarem o processo, seria oportuno que o


professor propusesse situaes em que um intruso tente decifrar
mensagens apoderando-se das sequncias numricas codificadas. Como
estamos utilizando funes afins, para tanto suficiente apenas duas
associaes corretas entre nmeros das sequncias original e codificada.
Admitindo conhecidas essas associaes, um exerccio interessante
para os alunos determinarem f.
O segundo mtodo criptogrfico que apresentaremos utiliza matrizes
invertveis como chaves, o que dificulta um pouco mais sua violao.

RPM OBMEP

Suponhamos que Ana e Ivo combinem previamente utilizar a matriz

48

2
3 2
1 1
A=
e sua inversa A =
como chaves. Para trans1 1
1 3
mitir a mesma mensagem acima, Ana inicialmente monta uma matriz
mensagem M dispondo a sequncia numrica associada em coluna e
completa a posio restante com 0, ou seja, obtm
18 22 19 1 18 13
M =
.
5 9 20 0 16 0
Em seguida, codifica-a calculando,

3 2 18 22 19 1 18 13 64 84 97 3 86 39 .
AM =

4 13
1 1 5 9 20 0 16 0 23 31 39 1 34

e transmite a seqncia 64 23 84 31 97 39 3 1 86 34 39 13. Para ler a


mensagem recebida, Ivo, da mesma forma, restaura a forma matricial
AM, e em seguida, com sua chave A1, pode recuperar M atravs da
identidade matricial, M = A1(AM).
Os mtodos tratados neste trabalho tem apenas carter instrutivo. Na
prtica atual so pouco utilizados pela inconvenincia de exigirem trocas
prvias de chaves entre os usurios. So, portanto, inviveis na descrio
de transaes eletrnicas nas quais um nico receptor recebe dados de
milhares de emissores, como ocorre em vendas pela Internet, transaes
bancrias e outras. Mesmo nesses casos mais complexos, a Matemtica
resolveu a trama, e desta vez, quem diria, o ramo da Teoria dos Nmeros.
O leitor interessado neste envolvente tema poder consultar a apostila
Criptografia, IC-OBMEP 2007.
Adaptado do artigo
Codificando e decifrando mensagens
Antonio Carlos Tamarozzi, RPM 45.

Painel V
Qual a relao entre os
nmeros 102.564 e 410.256?

Admitamos agora a questo sendo apresentada sob o seguinte aspecto:


determinar um nmero inteiro positivo N, formado de n algarismos e
terminando pelo algarismo 4, tal que ao trasladarmos esse 4 (algarismo
das unidades) para a primeira posio, obtemos outro nmero que o
qudruplo desse nmero N.

RPM OBMEP

Facilmente observamos que o segundo nmero do ttulo, 410 256,


o qudruplo do primeiro, 102 564. O que nos chama a ateno que os
algarismos desses nmeros so os mesmos, tendo sido o bastante
trasladarmos o algarismo 4 das unidades do primeiro nmero para a
esquerda, a fim de obtermos o segundo nmero.

49

Resoluo
Seja N = a1a2a3...an1 4 um nmero de n algarismos, n natural no
nulo. Retirando o algarismo 4 desse nmero, obtemos:

N 4
= a1a2 a3 ...an 1 . [No exemplo: 10256 = (102 564 4)/10.]
10
Colocando o algarismo 4 esquerda do primeiro algarismo de N,

N' =

obtemos

N " = 4a1a2 a3 ...an 1 = 4 10n 1 + a1a2 a3 ...an 1 = 4 10n 1 +

N 4
.
10

Para que N = 4N precisamos ter


4 N = 4.10n 1 +

N 4
4(10n 1)
40 N = 4.10n + N 4 N =
.
10
39

Para N ser inteiro, devemos ter 39 como divisor de 10n1. O menor


valor de n que satisfaz essa condio n = 6:
N=

4(106 1) 4(999999)
=
= 4 25641 = 102564. N = 410256 = 4N.
39
39

Podemos mostrar que, fazendo n = 6k, k = 1, 2, 3, ..., k N*,


obtemos todos os nmeros N terminados em 4 e que satisfazem a
condio procurada; logo, N =

4(106 k 1)
, k = 1,2.3,.........., k N*.
39

Exemplos
Para k = 2, obtemos N =

4(1012 1)
, donde N = 4 25641025641 =
39

102564102564.
4(1018 1)
, donde
39
N = 4 2564102564125641= 102564102564102564.

RPM OBMEP

Para k = 3, obtemos N =

50

Podemos propor problemas semelhantes ao anterior, como, por


exemplo, obter um nmero inteiro positivo N, formado por n algarismos
e terminando com o algarismo a, tal que ao trasladarmos esse a
(algarismo das unidades) para a primeira posio, temos como resultado

outro nmero que igual a aN. Procure obter N fazendo a = 1, 2, 3,


..., 9, verificando os resultados curiosos que sero obtidos, podendo,
em alguns casos, no haver soluo.
Adaptado do artigo
Qual a relao que existe entre os nmeros 102.564 e 410.256?
Augusto Manoel de Albuquerque Barros, RPM 63.

Painel VI
Uma demonstrao visual para
a frmula do sen(A + B)
Cada um tem a sua demonstrao favorita das importantes frmulas
de sen(A B) e cos (A B). De qualquer forma, sabido que, deduzida
uma delas, as outras podem ser obtidas por complemento, suplemento,
etc. Uma das mais simples e rpidas uma demonstrao visual, que
se baseia, na igualdade
a = b cosC + c cosB
onde a, b, c, A, B, C so os lados e
ngulos respectivos de um tringulo. A
igualdade pode ser obtida facilmente e diz
apenas que o lado a a soma (ou a
diferena, se B ou C for obtuso) das
projees ortogonais dos lados b e c sobre
o prprio a, como se v na figura ao lado.
Por outro lado, tambm bastante conhecida a lei dos senos em um
tringulo, segundo a qual:

onde R o raio do crculo circunscrito. Isso decorre imediatamente da


figura da pgina seguinte.
Ento num crculo de dimetro 1, tem-se: a = senA, b = senB e
c = senC.

RPM OBMEP

a
b
c
=
=
= 2R
senA senB senC

51

Para um tringulo inscrito nesse crculo, a


igualdade inicial fica:
senA = senB cosC + senC cosB .
E como, finalmente, o ngulo A o
suplemento de B + C, ou seja, tm o mesmo
seno, obtm-se a clebre frmula:
sen(B + C) = senB cosC + senC cosB .
Essa deduo vlida para B + C < 180o,
o que suficiente para deduzir o caso geral.

senA =

a 2
a

= 2R
R
senA

A demonstrao anterior baseia-se numa


ideia de S.H. Kung, encontrada na revista Mathematics Magazine, vol.
64, no 2, abril de 1991.
Adaptado do artigo
Demonstraes visuais
Jos Paulo Q. Carneiro, RPM 27.

Painel VII
Valores irracionais de funes
trigonomtricas
RPM: O que segue uma transcrio adaptada de alguns resultados
encontrados no livro Nmeros: racionais e irracionais, de I. Niven, SBM,

RJ, 1984, que decidimos publicar por julgar do interesse de nossos


leitores.
So conhecidas as identidades trigonomtricas
cos2 = cos2 sen2, sen2 = 2sencos,
sen(a + b) = sena cosb + senb cosa e
RPM OBMEP

cos(a + b) = cosa cosb + sena senb,

52

as quais, juntamente com a relao fundamental, cos2 + sen2 = 1,


implicam
cos3 = 4cos3 3cos.
Fazendo = 20 na ltima igualdade, obtemos:

1
= cos 60D = 4 cos3 20D 3 cos 20D .
2
Se escrevemos x no lugar de cos20, obtemos a equao
8x3 6x 1 = 0,
que por construo tem cos20 como raiz.
Aplicando a essa equao o conhecido resultado sobre razes racionais
de equaes polinomiais:
Se p/q, frao irredutvel, raiz de uma equao com
coeficientes inteiros anxn + an1xn1 + ... + a1x + a0 = 0,
ento p divisor de a0 e q divisor de an,
temos que as nicas possveis razes racionais da equao so

1
1
1
1, ,
e . Mas, substituindo-se na equao, um clculo
2
4
8
simples mostra que nenhum desses nmeros raiz; logo, a equao no
tem razes racionais e, portanto, cos20 um nmero irracional.
Tambm temos cos20 = cos210o sen210o = 1 2sen210o.
Logo, se sen10 fosse racional, ento 1 2sen210o seria racional, o
que implicaria cos20 racional, o que uma contradio.
Portanto, sen10 irracional.
Usando cos20 = cos210o sen210o = 2cos210o 1, conclui-se, de
modo anlogo, que cos10 tambm irracional.
Generalizando, temos o resultado:
Se for um ngulo tal que cos2 irracional, ento cos, sen
e tg so tambm irracionais.
A verificao de que cos e sen so irracionais se faz de modo
anlogo ao utilizado para = 10, usando as igualdades
Finalmente, se tg fosse racional, ento tg2 seria racional e de
1 + tg 2 = sec 2 =

1
cos 2

RPM OBMEP

cos2 = cos2 sen2 = 1 2sen2 = 2cos2 1.

53

teramos cos racional e, novamente, concluiramos que cos2


racional, uma contradio. Portanto, tg irracional.
Com repetidas aplicaes do resultado anterior mostra-se que cos,
sen e tg so irracionais, para, por exemplo, os valores de :
5; 2 30; 1 15; 3730", etc.
Adaptado do artigo
Valores irracionais de funes trigonomtricas
Paulo A. da Mata Machado e Aldo Trajano Lourdo, RPM 46.

Painel VIII
Mgica com nmeros
Truques de adivinhaes aritmticas tm sido apresentados a pessoas
e alunos de vrios nveis de escolaridade e sempre causam surpresa e
fazem muito sucesso. Vamos apresentar o truque da adivinhao egpcia
com a subseqente explorao das propriedades aritmticas subjacentes
a ele.
Nesse truque o apresentador pede a um espectador que pense em um
nmero de 10 a 100. O apresentador segue ento os seguintes passos:
1. Pergunta ao espectador se o nmero par ou mpar. Ouvida a resposta,
se for par, pede ao espectador que divida o nmero por 2. Se for
mpar, pede a ele que subtraia 1 e que ento divida o resultado por
dois.

RPM OBMEP

2. Pergunta se o resultado obtido par ou mpar e, ouvida a resposta,


pede ao espectador para repetir o procedimento descrito no item 1.

54

3. O procedimento continua com cada novo resultado at o resultado


(quociente de uma diviso por 2) tornar-se igual a 1, quando ento os
clculos do espectador terminaro.
Quando o apresentador informado de que o resultado igual a 1,
ele revela imediatamente ao espectador o nmero pensado por ele.

Como funciona o truque da adivinhao egpcia


Suponhamos que o nmero pensado pelo espectador seja 52. Nas
sucessivas etapas, ele efetuar as contas da coluna abaixo esquerda,
enquanto simultaneamente o apresentador ir fazendo, secretamente, as
anotaes da coluna direita.
Aluno
52 (nmero pensado)
26
13
6
3
1

Professor
1
2
4 9
8
16 9
32 9

Para cada nmero mpar informado pelo espectador, o apresentador


anota 9. Nos sucessivos estgios da brincadeira, o apresentador marca
as potncias de 2, iniciando em 20 = 1. Em seguida, o apresentador
soma as potncias de 2 correspondentes s marcas 9,
4 + 16 + 32 = 52,
e resgata o nmero que foi pensado pelo espectador!

Lendo da direita para a


esquerda os 0s e 1s, que
so o ltimo quociente e os
restos das divises, obtemos a representao do
nmero 52 (aqui representado no sistema decimal) no
sistema de numerao de
base 2:

RPM OBMEP

O truque foi concebido observando o mtodo das divises sucessivas


por 2, usado para representar um inteiro positivo no sistema binrio,
isto , como soma de potncias (distintas) de 2, a partir de sua
representao no sistema decimal. Nesse mtodo, tomando como
exemplo o nmero 52, fazemos a seguinte escada de divises
sucessivas por 2, at atingirmos quociente igual a 1, quando o algoritmo
termina.

55

52 = (110100)2 = 1 25 + 1 24 + 0 23 + 1 22 + 0 21 + 0 20
= 22 + 24 + 25.
Na seqncia das divises, um resto ser 0 quando o dividendo for
par, e 1 quando o dividendo for mpar, da a importncia de tomar nota
apenas das potncias de 2 correspondentes aos restos mpares.
O ttulo adivinhao egpcia inspirado nos algoritmos de
multiplicao dos antigos egpcios, baseados na decomposio de inteiros
positivos como somas de potncias distintas de 2.
Adaptado do artigo
Mgicas com nmeros
Joo C. V. Sampaio, RPM 60.

Painel IX
Destreza ou esperteza?
Certa vez, quando eu tinha 15 anos, um amigo da minha famlia
afirmou que sabia fazer contas mentalmente e com muita rapidez. Para
provar isso, props a seguinte brincadeira:
Vou escrever um nmero com sete algarismos. Em seguida, voc
escreve, abaixo do meu nmero, outro nmero com sete algarismos.
Repetimos isso mais uma vez, eu escrevo meu terceiro nmero e, ento,
eu direi a voc, sem fazer clculos, qual o valor da soma dos cinco
nmeros.

RPM OBMEP

Eu, um tanto desconfiado, aceitei a proposta, ocorrendo o seguinte:

56

1o nmero escrito por ele:


1o nmero escrito por mim:
2o nmero escrito por ele:
2o nmero escrito por mim:
3o nmero escrito por ele:
Soma fornecida por ele:

3 574 186
1 247 064
8 752 935
4 955 231
5 044 768
23 574 184

Conferi a soma manualmente e constatei que estava correta. Fiquei


atnito observando aqueles nmeros por alguns instantes, mas nada
consegui concluir. Ele props outra conta e novamente acertou o resultado
em poucos segundos. Claro que eu sabia (ou desconfiava) que existia
algum truque por trs daquilo, mas fiquei por alguns anos sem saber
qual era. Vamos agora mostrar que, na realidade, tudo no passa de um
pouquinho de lgebra: observe que o segundo e o terceiro nmeros
escritos por ele so construdos a partir do anterior, de modo que a soma
com o anterior seja igual a 9 999 999. Veja:
1o nmero escrito por mim + 2o nmero escrito por ele
1 247 064 + 8 752 935 = 9 999 999
2o nmero escrito por mim + 3o nmero escrito por ele
4 955 231 + 5 044 768 = 9 999 999
Observe agora que, como 9 999 999 = 10 000 000 1, a soma total
igual a: primeiro nmero somado + 2 (10 000 000 1) = 20 000 000
2, ou seja, (3 574 186 + 20 000 000) 2 . Para efetuar a soma entre
parnteses, observando que o nmero de zeros em 20 000 000 igual
ao nmero de dgitos do nmero inicial, basta acrescentar o dgito 2 na
frente do nmero original, o que resulta em 23 574 186. Subtraindo 2,
obtemos a soma.

Observe que, no caso do desafio proposto pelo amigo de minha


famlia, o nmero inicial 3 574 186. Colocando 2 no incio, obtemos
23 574 186. Subtraindo 2 do algarismo das unidades, obtemos
23 574 184, que a soma procurada.

RPM OBMEP

Note que, para realizar a ltima operao, no caso em que o algarismo


das unidades do primeiro nmero maior do que ou igual a 2, basta
subtrair 2 do algarismo das unidades, mantendo os outros dgitos
inalterados. Se ele for 0 ou 1, ento a subtrao um pouco mais
complicada, sendo necessrio emprestar 1 do algarismo das dezenas
para depois subtrair 2. Como 10 2 = 8, isso equivalente a subtrair
1 do algarismo das dezenas e somar 8 ao algarismo das unidades, se
esse no for nulo. Se o algarismo das dezenas for nulo, ento preciso
emprestar 1 do algarismo das centenas e assim por diante.

57

Se algum o desafiar, voc pode tentar dificultar o trabalho para o


desafiante dizendo: Quero ver se voc acerta o resultado no caso do
primeiro nmero escrito ter o algarismo das unidades menor que 2, ou
seja, igual a 0 ou 1, e o das dezenas nulo. Isso testar se ele entendeu
realmente como funciona o truque, que pode ser adaptado facilmente
para o caso de mais dgitos ou para um nmero maior de somandos.
Deixamos para o leitor esse trabalho.
Adaptado do artigo
Destreza ou esperteza?
Vanderlei Nemitz, RPM 64.

Painel X
Determinante para fatorar
H alguns anos, quando ainda existia a Unio Sovitica, submeteuse aos participantes de uma olimpada juvenil de Matemtica a seguinte
questo, aparentemente simples:
Fatorar a expresso a3 + b3 + c3 3abc
Mesmo bons professores de Matemtica, se no conhecerem algum
truque, tero dificuldade em resolver esse problema pelo mtodo direto.
Quem duvidar, que o tente.
Entretanto, a teoria dos determinantes d uma soluo fulminante ao
problema. Vejamos: seja o determinante
a b c
c a b = a3 + b3 + c3 abc abc abc = a3 + b3 + c3 3abc ,
b c a
RPM OBMEP

exatamente a expresso que desejamos fatorar.

58

O determinante no se altera se substitumos, por exemplo, a primeira


linha da matriz por sua soma com as duas outras, ou seja

a b c
(a + b + c) (a + b + c) (a + b + c)
c a b =
c
a
b
=
b c a
b
c
a
1 1 1
(a + b + c) c a b = (a + b + c)(a 2 + b 2 + c 2 ab ac bc) .
b c a
e o problema foi resolvido.
Muitos realmente so os caminhos da Matemtica e precisamos ter a
mente aberta e desbloqueada para encontr-los.

RPM OBMEP

Adaptado do artigo
Usando determinantes para fatorar
Gilberto Garbi, RPM 41.

59

Funes interessantes

A aplicao de situaes do cotidiano na motivao,


estudo e ensino de tpicos de contedos programticos
aumenta, na maioria da vezes, o interesse e compreenso
dos alunos da educao bsica, alm de evidenciar que a
Matemtica faz realmente parte da vida de todos ns. No
ensino de funes, que pode ser iniciado j no nvel
fundamental, as aplicaes so muito indicadas para fugir
do formalismo terico. Nessa direo, vou apresentar e
estudar alguns aspectos de funes bastante simples que
modelam situaes reais e comuns.
I. Em uma capital brasileira, os preos das corridas de
txi tiveram o seguinte aumento:
bandeirada: passou de R$ 3,20 para R$ 3,50, tendo,
portanto, um aumento de aproximadamente 9,3%;

RPM OBMEP

quilmetro rodado: passou de R$ 1,80 para R$2,20,


tendo, portanto, um aumento de aproximadamente
22,2%.

60

A determinao da funo que fornece o preo de uma


corrida j suscita uma discusso interessante. Vrios textos
didticos apresentam funes que modelam situaes
desse tipo como polinomiais de primeiro grau, cujo grfico
uma reta. No nosso caso seria:
P1(x) = 3,20 + x 1,80

P2(x) = 3,50 + x 2,20,

onde P1(x) e P2(x) denotam o preo da corrida de x km antes e depois


do aumento, respectivamente.
Essa interpretao pressupe uma variao contnua no preo da
corrida em funo dos quilmetros rodados. Mas a realidade no assim.
O taxmetro varia em fraes no valor de R$ 0,30, ou seja, supondo que
o carro no pare durante a corrida:
os valores de P1(x) variam 1,80/0,30 = 6 vezes durante cada km

rodado, o que significa a cada intervalo de rodagem de


aproximadamente 166,66 m;
os valores de P2(x) variam 2,20/0,30 = 7, 333... vezes durante cada

km rodado, o que significa a cada intervalo de rodagem de


aproximadamente 136,36 m.
Conversas com taxistas nos fizeram concluir que eles no tm em
mente o valor exato do comprimento do trecho percorrido antes de cada
mudana no preo, apenas deduzem valores aproximados (recebemos
respostas de 200 m, 150 m, etc.); dizem que quem determina o valor
exato o INMETRO ao ajustar os aparelhos dos txis.
Aqui cabe uma observao interessante: no caso da P1, o taxmetro
muda um nmero inteiro de vezes, 6, em cada km rodado, o que no
acontece na P2, uma vez que 2,20 no mltiplo de 0,30. Nesse caso,
para que o taxmetro mude um nmero inteiro de vezes, necessrio que

2, 20
x seja inteiro, isto , que x seja mltiplo de 0,30, sendo x o
0, 30

Voltemos ento s funes, P1 e P2 reais, que mudam de valor


aos saltos, a cada intervalo de 166,66 m ou de 136,36 m. Seus grficos
tm a forma de escada, um exemplo no usual de funo . Esboamos,
tambm, os grficos das P1 e P2 afins.

RPM OBMEP

nmero de km rodados. Isso significa que a expresso afim da funo


P1 ou P2 fornece o preo exato de uma corrida de x km, se x ,
respectivamente, inteiro ou inteiro mltiplo de 3.

61

Perguntas
1. Quais os preos, antigo e depois do aumento, de uma corrida de
3,5 km = 3500 m?
Como 3500/166,6 um valor entre 21 e 22, vemos que o preo
antigo dado pelo 22o degrau do grfico da funo P1(x), ento
P1(3500) = 3,20 + 21 0,30 = 9,50, ou seja, o preo R$ 9,50. Um
clculo anlogo mostra que o preo novo dessa corrida seria R$ 11,00.
Vamos responder s perguntas a seguir, considerando as aproximaes
de P1(x) e P2(x) pelas funes afins anteriormente consideradas.
Isso permite estabelecer expresses algbricas simples para as funes
envolvidas, alm do que os grficos acima mostram que a funo
afim uma aproximao razoavelmente boa.
2. Qual ser o aumento percentual no preo de uma corrida de 10 km?

RPM OBMEP

Considerando P1(10) = 3,20 + 10 1,80 = 21,20 e


P2(10) = 3,50 + 10 2,20 = 25,50, vemos que o aumento percentual
de 20,28%.

62

3. Qual a funo que fornece o aumento percentual numa corrida de


x km?
Considerando as funes afins, queremos, em funo de x, o valor de
p tal que P2 ( x) = P1 ( x) +

p
P1 ( x) , sendo P1(x) = 3,20 + x 1,80 e
100

P2(x) = 3,50 + x 2,20. Substituindo os valores e fazendo os clculos,

30 + 40 x
. Como x > 0, temos que o domnio
3, 20 + 1, 80 x
dessa funo o intervalo [0, +]. interessante observar que, para
x = 0, o aumento igual a 30/3,20, que aproximadamente 9,3%,
aumento da bandeirada. Para valores de x muito grandes, observando
obtemos p ( x) =

30
+ 40
, vemos que p tende para p = 40/1,80 =
que p ( x) = x
3, 20
+ 1, 80
x
22,22222... que o aumento percentual do km rodado, isto , para
corridas muito grandes, o aumento da bandeirada no conta, valendo
apenas o aumento do km rodado. O grfico da funo p(x), a seguir,
ilustra esse resultado e tambm evidencia uma peculiaridade dos
taxistas: eles no tm como receber aumentos de um percentual fixo.

0,06 R$1700,00 = R$ 102,00 e a atual


0,11 (R$ 1700,00 R$ 1200,00) = R$ 55,00.
Provavelmente os alunos no tero dificuldades em determinar as
funes que fornecem o valor das contribuies em funo do valor x

RPM OBMEP

II. O governo de um Estado brasileiro mudou a contribuio


previdenciria de seus contribuintes: de 6% sobre qualquer salrio
passou para 11% sobre o que excede R$ 1200,00 nos salrios. Por
exemplo, sobre um salrio de R$ 1700,00, a contribuio anterior
era

63

do salrio. Sendo C1(x) a contribuio anterior e C2(x) a atual, temos:

C1 ( x) =

6
x = 0, 06 x
100

0 se 0 x < 1200

.
C2 ( x) = 11
( x 1200) se x 1200

100
Os grficos dessas funes esto esboados a seguir e uma anlise
deles permite tirar vrias concluses, por exemplo:
1. Para um salrio de, aproximadamente, R$ 2700,00, o valor da
contribuio permanece o mesmo, por volta de R$ 160,00. Para obter
o valor exato do salrio que mantm a contribuio, basta resolver a
equao 0,06x = 0,11(x 1200), chegando a x = 2640 e C1(2640) =
C2(2640) = 158,40.

RPM OBMEP

2. Para salrios abaixo de R$ 2640,00, a contribuio previdenciria


diminuiu, pois nesse caso temos, para um mesmo x, C2(x) menor do
que C1(x). Fica interessante fazer simulaes com salrios e populao
para calcular os valores das arrecadaes antes e depois da mudana
da lei, verificando que em determinadas situaes, bastante provveis,
a arrecadao estadual diminui consideravelmente.

64

3. C2(x) maior que C1(x) para salrios maiores que R$ 2640,00, logo a
nova lei aumenta a contribuio dos salrios maiores que esse valor.
4. A inclinao da reta do grfico de C2(x), x > 1200, maior que a da
reta de C1(x); logo, a contribuio, com a nova lei, aumenta mais
rapidamente do que antes, medida que o salrio aumenta.
Adaptado do artigo

RPM OBMEP

Funes interessantes
Ana Catarina P. Hellmeister, RPM 63.

65

A formiga inteligente

Um problema

RPM OBMEP

Imagine dois postes verticais AA e BB de tamanhos


diferentes no plano horizontal . Para que posies uma
formiga P, no plano, v os dois postes do mesmo
tamanho?

66

Em primeiro lugar, devemos pensar o que ocorre quando


vemos dois objetos com o mesmo tamanho. Por exemplo,
uma moedinha de 1 centavo segura entre os dedos com o
brao esticado tem, aparentemente, o mesmo tamanho
da lua cheia. A concluso a seguinte: dois objetos
aparentam ter o mesmo tamanho para certo observador,
quando os ngulos de visada so iguais. Portanto,
observando a figura acima, a formiga v os postes AA e
BB do mesmo tamanho se os ngulos de visada APA
e BPB forem iguais.
Mesmo sem pensar ainda como resolver o problema,
a formiga inteligente pode verificar que existem dois

lugares onde isso acontece, ambos na reta AB. Andando na reta AB,
de A para B, certamente encontraremos um ponto interior ao segmento
AB, onde APA = BPB, como mostra a figura.

Se o poste A for maior que o poste B, esse ponto P estar obviamente


mais prximo de B do que de A e, para obter exatamente a posio de
P, basta ligar o ponto A ao simtrico de B em relao reta AB.
Por outro lado, uma segunda posio para P a interseo da reta
AB com a reta AB. A figura a seguir mostra uma outra posio de P,
onde APA = BPB.

Alm dessas duas posies determinadas intuitivamente, existe


certamente uma infinidade de outras, no plano, mas fora da reta AB. J
sabemos que os ngulos de visada APA e BPB so iguais para que a
formiga veja os dois postes do mesmo tamanho. Porm, como os postes
so verticais, isso significa que os tringulos APA e BPB so

PA AA
que constante, pois a razo entre
=
PB BB
os comprimentos dos dois postes (veja novamente a primeira figura).
Temos ento dois pontos fixos A e B no plano e buscamos o lugar
geomtrico dos pontos cuja razo das distncias a esses pontos constante
e igual razo entre os comprimentos dos postes.

RPM OBMEP

semelhantes. Portanto,

67

Antes de resolver o problema, interessante recordar o teorema das


bissetrizes:
Uma bissetriz de um ngulo de um tringulo divide o lado
oposto na mesma razo dos lados adjacentes.
Veja a seguir uma demonstrao desse importante teorema.
a) Na figura abaixo, AD bissetriz interna do ngulo A do ABC.
Traando por D as perpendiculares
DM e DN aos lados AB e AC, temos
que DM = DN, pois o ponto D est na
bissetriz do ngulo A. Por outro lado,
como os tringulos ADB e ADC tm
mesma altura a partir de A, ento a razo
entre suas reas, A, igual razo entre
suas bases, ou seja,
DB A( ADB ) ( AB DM ) / 2 AB
=
=
=
.
DC A( ADC ) ( AC DN ) / 2 AC

RPM OBMEP

b) Na figura a seguir, AE bissetriz externa do ngulo A do ABC.

68

Traando por E as perpendiculares EP e EQ s retas AB e AC,


temos que EP = EQ, pois o ponto D est na bissetriz do ngulo
externo A. Por outro lado, como os tringulos AEB e AEC tm mesma
altura a partir de A, ento a razo entre suas reas, A, igual razo
entre suas bases, ou seja,

EB A( AEB ) ( AB EP ) / 2 AB
=
=
=
.
EC A( AEC ) ( AC EQ) / 2 AC

Demonstrado o teorema das bissetrizes, importante lembrar que


vale a sua recproca, ou seja, se D um ponto da base BC do ABC
e

DB AB
, ento AD bissetriz interna do ngulo A e, se E um
=
DC AC

ponto do prolongamento de BC e

EB AB
=
, ento AE bissetriz
EC AC

externa.
A circunferncia de Apolnio
Passamos agora a analisar o problema seguinte:
Dados dois pontos A e B no plano e um nmero k > 0, determinar
o lugar geomtrico do ponto P tal que

PA
= k.
PB

Em primeiro lugar, se k = 1, temos PA = PB e o lugar geomtrico de


P naturalmente a mediatriz de AB.
Em seguida, vamos imaginar k > 1 (o caso 0 < k < 1 inteiramente
anlogo). Como j vimos antes, podemos encontrar com alguma
facilidade dois pontos da reta AB que possuem a propriedade desejada.
Sejam portanto M e N pontos da reta AB tais que

MA NA
=
= k.
MB NB

Sabemos que, dado um segmento unitrio, para cada real positivo k


existe um segmento de comprimento k e a semelhana de tringulos

RPM OBMEP

Observe ento a figura a seguir em que as retas r e s so paralelas.

69

MA NA
=
= k . Ateno:
MB NB
a figura acima mostra que os pontos M e N existem, mas a construo
com rgua e compasso s possvel se k construtvel (racionais, por
exemplo, so construtveis). Na situao acima, dizemos que os pontos
M e N dividem harmonicamente o segmento AB.

permite concluir que, na figura acima, temos

Vamos agora considerar um ponto P fora da reta AB tal que PA = k


PB
e investigar seu lugar geomtrico.
Observe a figura a seguir.

Como MA = PA , ento PM bissetriz interna do ngulo APB e


MB PB

NA PA
, ento PN bissetriz externa. Mas essas bissetrizes
=
NB PB
so perpendiculares (verifique!) e, como M e N so fixos, o lugar
geomtrico de P a circunferncia de dimetro MN.

RPM OBMEP

como

70

Essa circunferncia chama-se circunferncia de Apolnio do segmento


AB na razo k. Ela o lugar geomtrico dos pontos cuja razo das
distncias a dois pontos fixos igual a uma constante dada.

O problema da formiga est resolvido. O lugar geomtrico dos pontos


de onde a formiga v os postes de mesmo tamanho a circunferncia de
Apolnio do segmento AB na razo AA/BB.

Nota
Apolnio de Perga viveu no sculo 3 a.C. Foi clebre gemetra e
astrnomo, mas a maior parte de sua vasta obra desapareceu. Felizmente,
a sua obra-prima As Cnicas foi quase toda preservada. Entretanto,
so conhecidos os ttulos e contedos dos muitos tratados que escreveu
devido a relatos de matemticos posteriores. Sabemos por isso que
Apolnio escreveu um livro chamado Lugares Planos dedicado anlise
de diversos lugares geomtricos e que um deles era justamente o lugar
geomtrico dos pontos cuja razo das distncias a dois pontos fixos
constante. Esse lugar geomtrico ficou conhecido at hoje como
Circunferncia de Apolnio, um tanto injustamente, pois Aristteles j
o tinha descoberto anos antes.
Adaptado do artigo

RPM OBMEP

A formiga inteligente
Eduardo Wagner, RPM 61.

71

A demonstrao feita
por Heron

Quando pequeno, li sobre Heron de Alexandria em uma


enciclopdia biogrfica que havia em casa. Fiquei
sabendo que ele viveu no sculo II d.C. na cidade de
Alexandria (obviamente), que foi engenheiro e
matemtico. No me lembro que outras coisas mais havia
sobre Heron, mas ficou gravada em minha memria a
frmula que l estava para calcular a rea de um tringulo:
A=

p ( p a )( p b)( p c) ,

sendo p a metade do permetro do tringulo.


O que me encantou nessa frmula? No sei. Talvez por
ter uma raiz quadrada, que naqueles dias escolares lhe
dava um ar de Matemtica superior; ou pelo fato de s
usar os lados do tringulo, e no a altura, como na
formulinha usada na escola.

RPM OBMEP

Anos mais tarde, aps ter encontrado vrias vezes a


frmula e at depois de ter visto sua demonstrao como
mero corolrio de um clculo de medianas, continuava
intrigado: como Heron a havia demonstrado?

72

Este ano comprei o livro Introduo Histria da


Matemtica, de Howard Eves, e qual no foi minha
surpresa ao encontrar no livro a meno de que, a
demonstrao feita por Heron (que est em seu livro A

mtrica) estava esquematizada num dos exerccios do livro. Com algumas


pequenas modificaes aqui vai ela:

BC = a
AC = b
AB = c

p=

a+b+c
2

1. rea ABC = rea ABI + rea IBC + rea AIC =


r
( AB + BC + CA) = rp , sendo r o raio da circunferncia inscrita.
2

2. Como ADI AIF, DBI IBE e FIC IEC, temos


AD = AF, DB = BE e CE = CF.
3. Seja J o ponto da semi-reta AB tal que BJ = CE.

AJ =

AD + AF BD + BE CE + CF AB + BC + CA
+
+
=
= p.
2
2
2
2

Ento
p c = AJ AB = BJ; p b = AJ AC = DB e
4.
i) Seja K o ponto construdo como indicado na figura. O quadriltero
AKBI inscritvel numa circunferncia de dimetro AK; logo
AIB + AKB = 180 o e, como + + = 180, temos
AIB + CIE = 180o, de onde AKB = CIE = .

RPM OBMEP

p a = AJ BC = AD.

73

Ento temos CIE AKB, o que implica AB = CE = BJ .


BK
r
r
ii) No tringulo retngulo ALI temos r2 = DL.AD e de DLI
BLK (verifique) temos

BK
r
.
=
LB DL

iii) De i) e ii) temos AB = LB , o que implica AB + BJ = LB + DL


BJ DL
BJ
DL
AJ AJ DB AD , que juntamente com r2 = DL.AD leva a
.
=
.
BJ AJ DL AD
AJ2.r2 = BJ.AJ.BD.AD.

ou

Usando-se as igualdades apresentadas em 3, obtemos


p2r2 = (p c)p(p b)(p a),
que, pela igualdade exibida em 1, demonstra a frmula.
Adaptado do artigo

RPM OBMEP

A demonstrao feita por Heron


Mrio Dalcin, RPM 36.

74

A Matemtica da folha
de papel A4

Introduo
O formato do papel que usamos rotineiramente nos
servios de impresso ou fotocpia possui uma histria
fascinante e repleta de Matemtica. Neste artigo,
compartilho com o leitor algumas idias que esto por
trs dessa histria.
A intrigante folha de papel A4
O formato de papel mais usado para impresses e
fotocpias, que recebe a denominao A4, tem
210 milmetros de altura por 297 milmetros de largura.
Diferentemente do que se possa imaginar, a razo
297/210 no a razo urea (ver artigo Retngulo ureo,
diviso urea e sequnci de Fibonacci). Analisaremos, a
seguir, de onde vem essas estranhas medidas.

Imagine-se tendo que resolver o seguinte problema: qual


deve ser a largura e a altura de uma folha retangular de
modo que, quando ela for dividida ao meio, os dois novos
retngulos obtidos mantenham a razo entre altura e
largura da folha original?

RPM OBMEP

Inicialmente padronizaremos neste artigo que as palavras


largura e altura sempre sero usadas como referncia
ao maior e ao menor lado de um retngulo, respectivamente.

75

O problema de soluo simples, como se v a seguir:

L A
= L2 = 2 A2 L = 2 A
A L
2
Portanto, a folha retangular com razo L/A igual a 2 a nica
que, quando dividida ao meio, conforme processo descrito, resultar em
retngulos semelhantes ao da folha original. Lembramos que de forma
diferente dos tringulos, onde bastam ngulos congruentes para que sejam
figuras semelhantes, no caso dos quadrilteros a semelhana s se garante
se os ngulos forem congruentes e se a razo entre os lados das figuras
for preservada. No caso das medidas de uma folha A4, note que
297/210 uma tima aproximao racional para
pequeno, da ordem de centsimo de milsimo.

2 , com erro muito

A classificao de papis da qual A4 faz parte chama-se srie A,


que comea com o A0 e vai at o A10. Essas folhas tm em comum a
razo 2 entre largura e altura. A srie comea com uma folha retangular
de rea 1 m, definida como A0. A partir dela obtemos a folha do
formato seguinte, A1, dividindo-se A0 ao meio. As dimenses da
folha A0, em metros, podem ser obtidas a partir da soluo do seguinte
sistema de equaes:
1

L = 2 A
8
e L = 4 2 ou A = 2 4 e L = 2 4 .
A

2
L A = 1

RPM OBMEP

Passando essas medidas para milmetros, e aproximando para o


milmetro mais prximo, encontramos as dimenses da folha A0, que
so 841 mm de altura por 1189 mm de largura.

76

Faamos agora os clculos da folha A1, que obtida a partir da


diviso ao meio da folha A0:

L = 2A
3
1

4
e L=2 4.

1 A=2
L
A
=

Adota-se, nesse caso, a aproximao 594 mm por 841 mm.


Dividindo-se A1 ao meio, obtemos A2, que dividida ao meio
resultar A3, e assim por diante at A10. Pode-se verificar de maneira
simples que a altura e a largura de uma folha A(k), em metros, sero

1+ 2 k

1 2 k

dadas, respectivamente, por 2 4 e 2 4 . Para o caso da folha


A4, aplicando k = 4 na frmula, obtemos os misteriosos valores
padronizados do formato, que so 210 mm por 297 mm.
Qual a vantagem da proporo 1: 2 ?
A literatura sobre artes grficas cita dois aspectos importantes sobre
a convenincia do uso de uma folha retangular de razo 1 : 2 . As pginas de um livro so impressas em uma folha de mquina de grande
formato. Nela so feitas dobras e cortes e, a partir disso, so montados
os cadernos que, juntos, compem o livro. Normalmente as dobras so
feitas ao meio, fazendo com que o nmero de pginas seja uma potncia
de 2. Se o papel for dobrado ao meio por uma dobra, resultar em 2
folhas (chamado in-flio) que, quando impressas frente e verso,
constituiro 4 pginas do livro. Se essa ltima folha for novamente
dobrada ao meio, agora com dobras cruzadas, resultar em 4 folhas
(in-quarto), ou seja, 8 pginas de livro. Com uma nova dobra teremos
o in-oitavo: 3 dobras, 8 folhas e 16 pginas de livro; e assim
sucessivamente.
dobra sobre o maior lado do retngulo, a razo inicial 1 : 2 sempre
ser mantida em todas as pginas do livro, seja qual for o nmero de
dobras feitas na composio. Outros formatos no permitiriam isso como,
por exemplo, um retngulo de razo 3:4 (tambm usado na confeco
de livros) que obedece a um padro de alternncia no decorrer das
sucessivas dobras. A primeira dobra gera retngulos de razo 2:3; a

RPM OBMEP

Uma vez que cada formato deriva do seu precedente fazendo uma

77

segunda gera retngulos 3:4, a terceira retngulos 2:3, e assim sucessivamente. Deixo por conta do leitor a demonstrao do resultado: dada
y
x , ento as razes se alternam entre x:y e y:2x no
2
decorrer das sucessivas dobras que dividem o lado maior do retngulo
ao meio (obs.: o nico caso em que no h alternncia ser quando

a razo x:y, se

x
y
=
, que justamente o caso em que temos a razo 1 : 2 ).
y 2x
Vale citar que nem todos os estudiosos de composio em artes
grficas esto de acordo sobre a relevncia da vantagem que acabamos
de descrever da razo 1 : 2 sobre outras razes. Para um bom acabamento final das dobras de um livro recomenda-se que as dobras sejam
feitas paralelamente s fibras do papel. Com isso, folhas de papel que,
em virtude da direo das fibras, so adequadas ao in-quarto no
poderiam ser usadas para livros in-oitavo porque a fibra correria em
direo errada. Portanto, a vantagem da razo preservada em 1 : 2
aps as dobras fica comprometida quando levamos em considerao a
direo das fibras [1].
Outra vantagem que os papis de razo 1 : 2 da srie A apresentam
e essa aceita por todos os especialistas a de que evitam o desperdcio
de papel nos trabalhos de fotocpias.

RPM OBMEP

Imagine que voc queira copiar duas folhas quadradas, juntas, em


uma nova folha quadrada. Essa tarefa no pode ser realizada sem o
desperdcio de papel. Se os quadrados tm lado 10 cm, lado a lado
formaro um retngulo de 10 por 20 cm, o que exigir uma folha
quadrada de 20 por 20 cm para que o servio seja feito. Nesse caso,
haver desperdcio de metade da folha. O mesmo no ocorre, por
exemplo, com duas folhas A4 lado a lado, que podem ser copiadas,
sem desperdcio de papel, em uma folha A3.

78

Se voc observar com ateno, as fotocopiadoras que fazem


ampliao e reduo a partir das folhas da srie A possuem alguns
comandos pr-definidos, como, por exemplo, os de reduo de 71%,
50%, 35%, 25%, 18% e 12,5%.

Voc j se perguntou de onde vm essas estranhas porcentagens?


Responderemos essa pergunta calculando qual deve ser o fator de reduo
usado na altura e na largura de uma folha A(k) para que ela seja reduzida
a uma folha A(k +1):

2
0, 71 , uma reduo de 71% far o servio desejado. As
2
demais redues indicadas referem-se, respectivamente, s redues de
A(k) para A(k + 2), A(k + 3), A(k + 4), A(k + 5) e A(k + 6).

Como

Outros formatos de papel: as sries B e C


H registros do uso da razo 1 : 2 durante a Alta Idade Mdia,
quando muitos livros eram escritos em duas colunas. Gutenberg
(1398-1468), porm, preferia para suas pginas a razo 2:3, e, durante a
Renascena, raramente se produziu livro na razo 1 : 2 .

O padro internacional para o tamanho de papis o ISO 216


(International Organization for Standartization, norma 216), que
adotado por todos os pases industrializados do mundo, exceto EUA,
Canad e partes do Mxico. Essa norma regulamenta o formato de
algumas sries bsicas de papel, como as sries A, B e C. As sries B

RPM OBMEP

A idia de se padronizar um formato de papel surge no sculo XX, e


tem a ver com aspectos relacionados praticidade e economia. Com o
uso generalizado de um formato padro de papel o que se reflete
diretamente na padronizao dos formatos de livros, revistas, jornais,
envelopes , as bibliotecas podem planejar de forma mais eficiente as
alturas de suas prateleiras, as grficas podem trabalhar com ajustes de
mquina pr-definidos, as fotocopiadoras e impressoras podem
padronizar programas para reduo e ampliao, etc.

79

e C destinam-se, entre outras aplicaes, aos formatos de envelopes


que podem ser usados para conter folhas da srie A. O formato de uma
folha B(k) definido como a mdia geomtrica entre A(k) e A(k 1),
e o da folha C(k) como a mdia geomtrica entre A(k) e B(k). Usando
a frmula que vimos anteriormente para altura de uma folha A(k), as
frmulas de clculo da altura das folhas B(k) e C(k) sero:

B (k ) = A(k ) A(k 1) = 2

C (k ) = A(k ) B (k ) = 2

1+ 2 k 1+ 2 ( k 1)

4
4

1+ 2 k
4

k
2

= 2

= 2
1+ 4 k
4

=2

k
22

1+ 4 k
8

Ficam a cargo do leitor a formulao de B(k) e C(k) para a largura


das folhas dessas duas sries, bem como a demonstrao de que tambm
nas sries B e C a razo 1 : 2 se preserva.
Seja qual for o nmero k da srie, sempre teremos, tanto para a
altura quanto para a largura, a relao A(k) < C(k) < B(k). Verificaremos
tal fato para a altura, cujos dados j foram calculados anteriormente:
2

1+ 2 k
4

<2

1+ 4 k
8

<2

k
2

2 + 4k
1 + 4k
4k
<
<
2 < 1 < 0 ,
8
8
8

para qualquer k.

RPM OBMEP

Demonstrao anloga pode ser feita entre as larguras das trs sries.

80

Os formatos das sries B e C so maiores que os da srie A e, por


esse motivo, so usados nos envelopes que devero conter folhas da
srie A. Como A(k) < C(k) < B(k), se queremos enviar pelo correio um
documento com poucas folhas A4, devemos usar um envelope C4, porm,
se a quantidade de folhas for muito grande, provvel que elas fiquem
melhor acomodadas em um envelope B4. Se voc quiser enviar uma
folha A4 dobrada uma nica vez, recomenda-se um envelope C5. Para
uma folha A4 com duas dobras cruzadas, o envelope ideal o C6 e, se as
duas dobras forem paralelas, o envelope ideal o DL (ilustrado na figura
a seguir).

Adaptado do artigo

RPM OBMEP

A Matemtica da folha de papel A4


Jos Luiz Pastore Mello, RPM 66.

81

Retngulo ureo, diviso urea


e sequncia de Fibonacci

O retngulo ureo
Chama-se retngulo ureo qualquer retngulo ABCD
(figura 1) com a seguinte propriedade: se dele
suprimirmos um quadrado, como ABFE, o retngulo
restante, CDEF, ser semelhante ao retngulo original.
a
F b C
B
Se a + b e a so os comprimentos dos lados do retngulo
a
original a definio acima se
traduz na relao

a
b
= . (1)
a+b a

E
figura 1

RPM OBMEP

Como veremos logo adiante, esse tipo de retngulo tem


muitas propriedades interessantes que justificam o
qualificativo ureo. Ele tem sido considerado por

82

figura 2

arquitetos e artistas como o retngulo mais bem


proporcionado e de grande valor esttico. A figura 2

reproduz a foto de uma residncia suburbana de Paris, projetada pelo


famoso arquiteto Le Corbusier, na qual ele utiliza o retngulo ureo. H
a dois retngulos ureos, um deles representado pelo corpo inteiro da
casa e o outro, disposto verticalmente, representado pela parte da casa
esquerda da escada.

figura 3

O Partenon (figura 3), ou templo da deusa Atena, uma das mais


admiradas obras da arquitetura universal, revela, em seu frontispcio
(figura 4) um quase exato retngulo ureo. Todavia no h evidencia
histrica de que, ao construir o templo no 5o sculo a.C., os arquiteto de
Pricles tenham conscientemente usado o retngulo ureo.

figura 4

Voltemos relao (1). Dela decorre, por uma propriedade bem


conhecida das propores, que:
ou seja

b a b
=
.
a
b
a

Isto significa que se o retngulo de


lados a + b e a ureo, ento tambm
o o retngulo de lados a e b.

2b  a

ab
figura 5

RPM OBMEP

a
b
a b
= =
a + b a ( a + b) a

83

Evidentemente o mesmo raciocnio se aplica para mostrar que tambm


so ureos os retngulos de lados b e a b, a b e 2b a, etc.
(figura 5). Em outras palavras, dados os nmeros positivos a e b,
satisfazendo a relao (1), formemos a seqncia a + b, a, b, a b,
2b a, 2a 3b, 5b 3a, 5a 8b, 13b 8a, ... (2)
Sendo, a partir do terceiro, an = an2 an1.
O raciocnio anterior estabelece que quaisquer dois elementos
consecutivos dessa sequncia so os lados de um retngulo ureo.
Portanto, o processo anterior de retirar quadrados de retngulos ureos
conduz a uma sequncia infinita de retngulos ureos, com dimenses
cada vez menores.
A figura tambm sugere que a seqncia (2) tende a zero, e isso
verdade porque as dimenses de cada retngulo da seqncia podem ser
obtidas multiplicando as dimenses correspondentes do retngulo
anterior por b/a, que menor que 1; ora, o termo geral de uma progresso
geomtrica de razo menor do que 1 tende a zero quando o nmero de
termos tende a infinito. Deve ser notado que o smbolo da Sociedade
Brasileira de Matemtica utiliza a mencionada sucesso de retngulos
ureos, unidas por quadrantes de circunferncias.

Os lados de um retngulo ureo so grandezas incomensurveis. (Veja


a definio desse e de outros conceitos correlatos no artigo Grandezas
Incomensurveis, desta apostila.) De fato, se fossem comensurveis,
teriam um submltiplo comum s, e, com referencia figura 1,

RPM OBMEP

AD = (a + b)s e AB = as,

84

onde a e b seriam ento nmeros inteiros. Em consequncia, todos os


nmeros da sequncia (2) seriam inteiros e positivos. Isso um absurdo,
pois no existe sequncia infinita e decrescente de nmeros inteiros
positivos (Princpio da Descida Infinita de Fermat). Conclumos, ento,
que os lados de um retngulo ureo so incomensurveis.

A diviso urea
O retngulo ureo est intimamente ligado com a chamada diviso
urea de um segmento, ou diviso em mdia e extrema razo, que
introduziremos a seguir.
Diz-se que um ponto C de um segmento AB (figura 6) divide esse

AC CB
.
=
AB AC

segmento em mdia e extrema razo se

(3)

figura 6

A relao (3) precisamente a relao (1) se pusermos AC = a e


CB = b, de sorte que os segmentos AC e CB da diviso urea (ou
AB = a + b e AC = a) so os lados de um retngulo ureo.
interessante notar que se C1 divide AB em mdia e extrema
razo, e se marcarmos no segmento AB os pontos C2, C3, C4,... de tal
maneira que AC2 = C1B, AC3 = C2C1, AC4 = C3C2, ... (figura 7), ento
Cn divide ACn1 em mdia e extrema razo n = 2, 3, 4,... . Esse
resultado segue facilmente do que j provamos antes sobre a sequncia
infinita de retngulos ureos, donde segue tambm que os segmentos
AC1 e C1B da diviso urea de AB so incomensurveis. Sugerimos
que o leitor faa uma demonstrao completa destes resultados.
A

C4

C3

C2

C1

figura 7

Como j observamos h pouco, as relaes (1) e (3) so idnticas


quando pomos AC = a e CB = b. Delas segue-se que
(4)

O nmero m = b/a conhecido como a razo urea. Dividindo a


equao anterior por a2 obtemos: m2 + m = 1.
(5)
A raiz positiva dessa equao do segundo grau :

M=

5 1
0, 618 . (6)
2

RPM OBMEP

b2 + ab = a2.

85

A razo urea, como foi definida, a razo entre o menor e o maior


lados de um retngulo ureo. Deve ser observado que muitos autores
usam como razo urea (ou nmero ureo) o seu inverso u, que a
razo entre o maior e o menor lados de um retngulo ureo:
u=

1
5 +1
=
1, 618 .
m
2

O nmero u a raiz positiva da equao u2 = u + 1.


Entre estes dois nmeros, existem as relaes: u = 1 + m e mu = 1.
Construes geomtricas
Vamos construir um retngulo ureo a partir de seu menor lado
AE = a (figura 8). Para isso construmos EF = AE perpendicularmente
a AE. Com centro em G, ponto mdio do segmento AE, traamos o

a
A

a/2

a/2

figura 8

A
b

H
b

ab

D
b

D
figura 9

p , com D na reta AE e E interno ao segmento AD. Como


arco FD
GF = GD = b + a/2, o teorema de Pitgoras aplicado ao triangulo retngulo
GEF nos d:
a
a
(b + ) 2 = a 2 + ( ) 2 .
2
2

RPM OBMEP

Simplificando, obtemos daqui a relao (4) que, como vimos, equivale


relao (1). Logo ABCD um retngulo ureo.

86

Se o problema fosse dividir o segmento AE = EF em mdia e extrema


razo, bastaria completar a construo anterior marcando, no segmento
AE, o ponto H tal que AH = b (figura 9).

A sequncia de Fibonacci e a razo urea


surpreendente que a razo urea esteja intimamente relacionada
com a chamada sequncia de Fibonacci, como veremos a seguir, pois
aparentemente uma coisa nada tem a ver com a outra.
Leonardo de Pisa, muito conhecido como Fibonacci (filho de
Bonaccio), viveu no perodo de aproximadamente 1170 a 1250. Ele foi
educado na frica e viajou muito pela Europa e sia Menor. Tornou-se
famoso por conhecer muito bem toda a Matemtica ento acumulada.
Em 1202 ele publicou o Liber Abaci, ou Livro do Clculo, que teve
importncia decisiva na tarefa de tornar conhecida na Europa a
Matemtica dos rabes e hindus. Foi esse livro que popularizou no
Ocidente o uso dos algarismos arbicos e os mtodos hindus de clculo
com nmeros inteiros, fraes e razes.
A seqncia de Fibonacci aparece num dos problemas tratados no
Liber Abaci e que consiste no seguinte:
Um casal de coelhos torna-se produtivo aps dois meses de vida e, a
partir de ento, produz um novo casal a cada ms. Comeando com um
nico casal de coelhos recm-nascidos, quantos casais existiro ao final
de um ano?

f0 = 1, f1 = 1, f2 = f0 + f1 = 2,
f3 = f1 + f2 = 3, f4 = f2 + f3 = 5,
f5 = f3 + f4 = 8, f6 = f4 + f5 = 13, ...
ou seja,
1, 1, 2, 3, 5, 8, 13, 21, 34, 55, 89, 144,...

RPM OBMEP

Vamos designar com fn o nmero de casais de coelhos existentes


aps n meses. Evidentemente, f0 = f1 = 1. Por outro lado, o nmero de
casais existentes no n-simo ms, fn, igual ao numero existente um
ms antes, fn1, mais o numero de nascimentos novos. Ora, esse nmero
precisamente o nmero de casais existentes h dois meses, fn2, que
tm pelo menos dois meses de vida, portanto em condies de reproduzir.
Ento, cada elemento da sequncia de Fibonacci a soma dos dois
precedentes. Como j sabemos que f0 = f1 = 1, podemos construir toda
a sequncia:

87

Que relao pode existir entre esta sequncia e a razo urea?


Aparentemente nenhuma. No entanto, vamos demonstrar agora para
surpresa de todos ns! ... que a razo urea m o limite, com n tende
a infinito, da razo fn1/fn, isto ,
m=

f
5 1
= lim n 1 .
n f
2
n

(7)

De fato, pode-se observar (e demonstra-se por induo) que os


coeficientes de a e tambm os de b na sequncia (2), a partir do quarto
termo, a menos de sinal, so os nmeros de Fibonacci, isto , se
a2 = a b, a3 = a + 2b, a4 = 2a 3b, ... ento
an = (1)n(fn2a fn1b).

(8)

Lembrando que a sequncia (2) tende a zero quando n , tem-se:


fn2a fn1b 0.
Dividindo por afn1, v-se que

f
b
= lim n 2 .
n

a
f n 1

A ttulo de curiosidade, vamos calcular fn1/fn, com trs casas


decimais, para n = 1, 2, ..., 8:

1
1
2
3
5
= 1, = 0, 5, 0, 666, = 0, 6, = 0, 625,
1
2
3
5
8
8
13
21
0, 615,
0, 619,
0, 618.
13
21
34
Para n > 8, a razo fn1/fn sempre 0,618, com arredondamento na
3 casa decimal.
a

RPM OBMEP

O pentagrama e observaes finais

88

A diviso urea conhecida desde os pitagricos de cinco sculos


a.C. Ao que tudo indica, essa diviso foi descoberta no pentgono regular,
que exibe uma surpreendente profuso de segmentos na razo urea.
Talvez este tenha sido o motivo que levou os pitagricos a adotarem o
pentagrama (pentgono regular estrelado) como smbolo de sua seita
(figura 10).

Na figura, os vrtices da estrela dividem o


crculo em cinco partes iguais. Portanto, cada
arco mede 72. Os tringulos NBE e ABE so
semelhantes porque seus ngulos internos
medem 36, 72 e 72. Logo,
NB BE
=
.
BE AB

D
M

E
figura 10

n = NEA
n = 36D ) e NE = BE (porque
Mas AN = NE (porque NAE
n = NBE
n = 72D ). Temos portanto,
BNE

NB BE
=
.
AN AB

e consequentemente essas razes so ureas. Pela semelhana dos


tringulos DMN e AND, prova-se, da mesma forma, que MN/AM
tambm urea.
muito improvvel que Pitgoras ou seus primeiros discpulos
soubessem que os segmentos da diviso urea fossem incomensurveis,
embora haja fundadas razes para se acreditar que a descoberta dos
incomensurveis tenha ocorrido com o pentgono regular no fim do 5o
sculo A.C. Certamente, Pitgoras e seus discpulos sabiam como
construir geometricamente a soluo (6) da equao (5). As construes
correspondentes s figuras 8 e 9 acima se encontram nos Elementos
de Euclides, de cerca de 300 anos A.C.

A Geometria possui dois grandes tesouros: um o Teorema de


Pitgoras; o outro, a diviso de um segmento em mdia e extrema
razo. Podemos comparar o primeiro a uma poro de ouro e o
segundo a uma jia preciosa.
Tanto a razo urea, como os nmeros de Fibonacci, aparecem numa
variedade enorme de situaes inesperadas. Hoje a literatura sobre os
nmeros de Fibonacci enorme. Existe at uma revista The Fibonacci

RPM OBMEP

Na antiguidade, a diviso de um segmento em mdia e extrema razo


tornou-se to familiar que era conhecida simplesmente como a seo,
em qualquer qualificativo. O nome diviso urea lhe foi dado por
Kepler (1571-1630), que escreveu:

89

Quartely fundada em 1963, dedicada pesquisa em torno desses


nmeros!
O Teorema de Lam
Vamos mostrar agora uma aplicao surpreendente dos nmeros de
Fibonacci.
Para encontrar o o mximo divisor comum de dois inteiros, bastante
conhecido o processo que exemplificamos no caso do mdc(243, 37):
6
243 37
21 16

1
21
5

1
16
1

3
5
0

5
1

As operaes efetuadas acima foram as seguintes:


243 = 6 37 + 21
37 = 21 1 + 16
21 = l 16 + 5
16 = 5 3+1
5 = 1 5.
O algoritmo termina quando o resto da diviso nulo; o mximo
divisor comum o ltimo divisor obtido.
O algoritmo do processo apresentado acima o clebre algoritmo de
Euclides, conhecido desde a antiguidade. Ele encontra-se exposto, para
nmeros, na Proposio 1 do livro VII dos Elementos de Euclides,
escritos em torno de 300 a.C.

RPM OBMEP

Os historiadores da Matemtica acreditam que esse algoritmo era


conhecido j em 400 a.C. Ele de importncia fundamental em teoria
dos nmeros.

90

O que fizemos no exemplo acima pode ser generalizado: Sejam a e


b inteiros positivos, com a > b. Usando sucessivamente o algoritmo da
diviso, escreva
a = bq1 + b1, 0 < b1 < b,
b = b1q2 + b2, 0 < b2 < b1,
b1 = b2q3 + b3, 0 < b3 < b2,
...

bn2 = bnl qn + bn, 0 < bn< bn1,


bnl = bnqn+l.
Ento mdc(a, b) = bn.
Com efeito, em primeiro lugar, o processo acima realmente chega
ao fim. De fato, como 0 < bn < bn1 < ... < b1 < b, vemos que esse
processo no pode repetir-se indefinidamente, pois temos uma seqncia
estritamente decrescente de inteiros positivos e h um nmero finito de
inteiros entre 0 e b.
Alm disso, usando estas equaes de baixo para cima, constatamos
que bn divide bn1, bn2, etc, at concluir que bn um divisor comum
de a e b. Em seguida, usando estas mesmas equaes de cima para
baixo, constatamos que todo divisor comum d de a e b tambm um
divisor comum de b1 e b2, de b2 e b3, etc, at concluir que d
tambm um divisor de bn, ou seja, sendo bn um mltiplo de d, bn
maior que ou igual a d, o que mostra que bn o mximo divisor
comum de a e b.
Usando o algoritmo de Euclides, so necessrias n + 1 divises
para vermos que mdc(a, b) = bn, pois s chegamos a uma concluso
quando verificarmos que bn1 = bnqn+1 + bn+1 = bnqn+1 + 0 = bnqn+1.
Chamaremos de comprimento do algoritmo de Euclides o nmero
de divises necessrias para calcular o mdc(a, b). Usando a notao do
teorema, o comprimento do algoritmo de Euclides n + 1.
O algoritmo de Euclides bem eficiente. Por exemplo, se
quisermos verificar que mdc(97, 24) = 1, sero necessrios apenas dois
passos:
97 = 4 24 + 1
24 = 24 1.
21479 = 894 24 + 23,
24 = 1 23 + 1,
23 = 1 23.
Ou seja, em 3 passos vemos que mdc(21479, 24) = 1. Por fim, como
ltimo exemplo, para calcular mdc(49745692, 24), temos

RPM OBMEP

Agora, se queremos calcular mdc(21479, 24), temos

91

49745692 = 2072737 24 + 4,
24 = 6 4;
isto , em apenas 2 passos chegamos ao resultado desejado.
Dados dois nmeros inteiros e positivos a e b, uma pergunta natural
: qual o comprimento do algoritmo de Euclides aplicado a eles? Em
outras palavras, quantas divises so necessrias para calcular o mximo
divisor comum de a e b.
imediato verificar que, se mantivermos b fixo, mesmo que a seja
muito grande em relao a b, o nmero de divises no algoritmo de
Euclides no pode crescer. Em verdade, esse nmero depende apenas de
b. Com efeito, usando mais uma vez a notao acima, sabemos que, no
algoritmo, mdc(a, b) = bn e que 0 < bn < bn1 < ... < b1 < b. Como h no
mximo b 1 inteiros distintos no negativos entre 0 e b, vemos que
n < b 1, donde n + 1 < b. Ora, como j vimos, so necessrias n + 1
divises para determinar o mximo divisor comum. Assim, so
necessrias no mximo b divises para achar mdc(a, b).
No entanto, esse resultado no muito bom. Por exemplo, se b =
99, devemos ter que n + 1 < 99 e chegamos concluso de que talvez
tenhamos que efetuar 99 divises para calcular o mximo divisor
comum!
O Teorema de Lam melhora muito essa situao:
Teorema (Lam). Sejam a e b inteiros positivos. Ento, o comprimento
do algoritmo de Euclides aplicado aos nmeros a e b menor que ou
igual a cinco vezes o nmero de dgitos na representao decimal de b.

RPM OBMEP

Segundo o teorema, se b igual a 99, que tem dois algarismos,


ento o nmero de divises no algoritmo de Euclides no mximo 10,
no sendo influenciado por a. Isso representa um progresso notvel em
relao estimativa anterior.

92

Esse teorema devido a Gabriel Lam (1795-1870) engenheiro e


matemtico francs, conhecido por seus trabalhos sobre a equao do
calor e criador das coordenadas curvilneas. Embora no tenha se
dedicado sistematicamente teoria dos nmeros, ele deixou algumas
jias sobre o assunto, uma das quais o teorema anterior.

A demonstrao do Teorema de Lam um exemplo de utilizao


inteligente dos nmeros de Fibonacci. Em verdade, essa foi, em 1844, a
primeira aplicao significativa desses nmeros.
Para efetuarmos a demonstrao, voltemos ao algoritmo de Euclides.
Em primeiro lugar, b n > 1, pois bn um nmero inteiro. De
bn1 = bnqn+1, vemos que bn1 > 2, pois bn1 > bn. Assim, bn > f1 e
bn1 > f2. Ento,
bn2 = bn1qn + bn > f2 + f1 = f3, pois qn > 1.
Analogamente,
bn3 = bn2 qn1 + bn1 > f3 + f2 = f4 , pois qn1 > 1.
Continuando dessa maneira, vemos, de maneira geral, que
bn-k > fk+1 para k = 0, 1, 2, ..., n 1,
e, enfim,
b = b1q2 + b2 > fn + fn1 = fn+1,
ou seja, fazendo b0 = b, temos:
bnk > fk+1, para k = 0, 1, 2, ..., n.
Ilustrando:

...

bn

bn 1

bn 

>

>

>

...

f1

f2

f3

...

b1

>

>

fn

fn + 1

Podemos ver que esse resultado o melhor possvel achando o


mximo divisor comum entre dois nmeros de Fibonacci consecutivos.
Calculemos, por exemplo, mdc(21,13) = mdc(f7, f6):
21 = 13 + 8
13 = 8 + 5
8=5+3
5=3+2
3=2+1
2 = 1 2 + 0.
Nesse exemplo, f7 e f6 no desempenham nenhum papel essencial,

RPM OBMEP

Esse resultado nos mostra que o comprimento do algoritmo de


Euclides menor ou igual ao nmero de ordem do maior nmero de
Fibonacci menor ou igual a b.

93

pois o mesmo acontece no caso geral, para achar mdc(fn1, fn).


5 +1
 1, 618... .
2
Temos, como visto anteriormente, u2 = 1 + u. Logo,

Consideremos agora o nmero ureo u =

u2 = u + 1 < 2 +1 < f2 + f1 = f3.


u3 = u2 + u < f3 + 2 < f3 + f2 = f4,
u4 = u3 + u2 < f4 + f3 = f5,
e assim sucessivamente, chegando enfim a uj < fj+1, j = 2, 3, 4, ... .
Em particular, un < fn +1 < b.

Como a funo log10x estritamente crescente, temos que


nlog10u < log10b, ou, equivalente, n <

log10 b
.
log10 u

Ora, calcula-se facilmente, usando uma tbua de logaritmos ou uma


mquina de calcular, que

log10 u = log10
Assim, n <

1
1+ 5
1
< 5.
= 0, 20898 > 0, 20 = , ou seja,
log10 u
2
5

log10 b
< 5 log10 b
log10 u

Se o nmero de algarismos na representao decimal de b s,


ento
b = ts110s1 + ts210s2 + ... + t110 + t0,
e, portanto, b < 10s, donde log10b < s, e vemos que n < 5s. Como n
um inteiro estritamente menor do que 5s, temos que n + 1 < 5s, o
resultado procurado.
Adaptado dos artigos

RPM OBMEP

Retngulo ureo, diviso urea e seqncia de Fibonacci.


Geraldo vila, RPM 06.

94

O smbolo da SBM
Eduardo Wagner, RPM 20.
Euclides, Fibonacci e Lam
Joo Bosco Pitombeira de Carvalho, RPM 24.

Usando Geometria
para somar

Introduo
Um dos maiores prazeres da Matemtica o da descoberta
(ou da redescoberta) de resultados matemticos mesmo
que j conhecidos. Iniciamos este artigo com o problema
clssico de calcular a soma dos n primeiros nmeros
naturais pelo mtodo que Gauss teria utilizado aos dez
anos para somar de 1 a 100 de cabea, para surpresa do
seu professor. Mostraremos ainda como calcular a soma
dos quadrados dos n primeiros naturais e outros
resultados interessantes.
Seja S = 1 + 2 + 3 + ... a soma dos n primeiros nmeros
naturais. O mtodo utilizado pelo jovem Gauss para
calcular essa soma bastante simples, embora engenhoso.
Ele escreveu a soma pedida e, embaixo, escreveu a mesma
soma ao contrrio
S = 1 + 2 + (n 1) + n
S = n + (n 1) + ... + 2 + 1

n(n + 1)
.
2
A demonstrao a seguir usa essencialmente a mesma
ideia.
com n parcelas, ou seja, S =

RPM OBMEP

Somando, obtemos 2S = (n + 1) + (n + 1) + ... + (n + 1)

95

A soma dos n primeiros nmeros naturais


I. A soma dos n primeiros nmeros naturais pode ser visualizada
geometricamente atravs da figura abaixo. Nela v-se um retngulo
formado por bolinhas. A base do retngulo possui n + 1 bolinhas e a
altura tem n bolinhas. No total, temos ento n(n + 1) bolinhas. Observe
agora que elas esto divididas em duas partes iguais pela linha poligonal
e em cada uma delas aparece a soma S = 1 + 2 + 3 + ... + n. Obtm-se,
ento, a frmula da soma dos n primeiros nmeros naturais.

1
1 + 2 + ... + n = n(n + 1)
2

Nota da RPM: A figura acima apareceu na coluna de Martin Gardner


Mathematical Games, da Scientific American de outubro de 1973, junto
com vrias outras chamadas look-see diagrams (diagramas olhe-veja) e
o autor cita que ela j era conhecida pelos gregos antigos. Posteriormente,
figuras demonstrando resultados matemticos conhecidos apareceram
em vrias revistas. Em 1993, as melhores demonstraes dessas revistas
foram reunidas por Roger Nelsen e publicadas no livro Proofs without
words Exercices in visual thinking pela MAA (Mathematical
Association of America). Esse livro ser a nossa principal referncia
neste artigo, doravante designado por [Pww] (Pww - Proofs without
words Demonstraes sem palavras)

RPM OBMEP

II. Uma outra forma de obter a soma dos n primeiros nmeros naturais
utiliza a figura a seguir e o conceito de rea. Observe que a soma
1 + 2 + 3 + ... + n igual rea do tringulo grande (metade de um
Quadrado de lado n) mais a metade de n quadrados.

96

1 + 2 + ... + n =

n 2 n n(n + 1)
+ =
2 2
2

A soma dos nmeros mpares


A soma dos n primeiros nmeros mpares pode ser visualizada atravs
da figura a seguir. O fato de que essa soma igual a n2 j era do
conhecimento dos antigos pitagricos, mas a figura da autoria de
Nicmaco de Gerasa (um pitagrico tardio), que viveu em torno do ano
100 d.C. [Pww].

1 + 3 + 5 + ... + (2n 1) = n 2

A soma dos quadrados dos n primeiros nmeros naturais

figura A

figura B
figuras de [Pww]

RPM OBMEP

I. Considere inicialmente trs castelos iguais como os da figura A.


Cada um deles formado por 1 + 4 + 9 + ... + n2 cubos unitrios. Na
figura B os trs castelos foram reunidos e nota-se que o ltimo andar
possui apenas a metade dos cubos necessrios para completar um
paraleleppedo.

97

A figura C mostra os cubos do ltimo andar cortados horizontalmente


pela metade, sendo a parte de cima (mais escura) utilizada para completar
o paraleleppedo. Pela figura D vemos que a soma 1 + 22 + ...+ n2, que
o volume dos castelos iniciais, um tero do volume de um
paraleleppedo de base n por n + 1 e altura n +

1
. Portanto,
2

1
1
n(n + 1)(2n + 1)
1 + 22 + ... + n 2 = n(n + 1)(n + ) =
.
3
2
6

figura C

figura D
figuras de [Pww]

A soma de uma srie geomtrica


Sendo r um nmero positivo menor que 1, quanto vale a soma
infinita 1 + r + r2 + r3 + ...? A resposta pode ser obtida atravs de um
desenho bastante engenhoso.
Na figura seguinte, ASPQ um quadrado de lado 1 e AR = r. A reta
PR forma o tringulo PTS no qual ST = 1 + r + r2 + r3 + ...
(Essa ltima igualdade pode ser verificada construindo-se um novo
quadrado de lado r obtendo o segmento de medida r2 uma vez que a
1 r
. Continuando o
=
r r2
processo, construindo-se quadrados de lados r2, r3, ... obtemos a medida
indicada para o segmento ST.)

RPM OBMEP

razo entre as medidas dos segmentos

98

Da semelhana entre os tringulos PTS e RPQ temos

ST QP
,
=
SP QR

ou seja, 1 + r + r 2 + r 3 + ... = 1 .
1 r
Para voc pensar: O que cada umas das figuras seguintes pode mostrar?

Sugesto: figura da esquerda: 4(1 + 3 + ...)


figura da direita: 1 + 2.22 + 3.32 + 4.42 + 5.52 = ...

Adaptado do artigo
Usando Geometria para somar
Eduardo de Campos Valadares e Eduardo Wagner, RPM 39.

RPM OBMEP

Observao
certo que, as demonstraes visuais podem parecer, em um primeiro
momento, ter algo de magia. importante perceber que a mgica
apenas aparente: a descoberta de um resultado matemtico fruto de
experimentao, dedicao e compreenso dos conceitos envolvidos.
Deve ficar clara tambm a necessidade, no caso de nmeros naturais,
por exemplo, do axioma da induo, para provar definitivamente certos
resultados.

99

Mdias

As mdias mais conhecidas pelos estudantes e professores


de Matemtica so a mdia aritmtica, a mdia
geomtrica e a mdia harmnica.
Para dois nmeros a e b, estas mdias so,
respectivamente:

A=

a+b
2

G = ab

H=

2ab
.
a+b

Para calcular a mdia geomtrica G, costuma-se exigir


que a e b sejam positivos e, para calcular a mdia
harmnica H, exige-se que a e b sejam no nulos. De
agora por diante, consideraremos apenas nmeros
positivos.
A mdia geomtrica tambm pode ser escrita como:
G2 = ab ou a = G . A mdia harmnica tambm pode
G b
ser vista como o inverso da mdia aritmtica dos inversos,

RPM OBMEP

isto : 1 = 1 a + 1 b .
H
2

100

Decorre imediatamente das definies que AH = G2, o


que mostra que a mdia geomtrica de dois nmeros
tambm a mdia geomtrica entre a mdia aritmtica e a
mdia harmnica destes nmeros.

Para ter uma viso unificada destas trs mdias, considere as relaes
seguintes envolvendo os nmeros reais a, b e c, positivos e distintos:
ac a
=
cb a

(1)

ac a
=
cb b

(2)

ac a
=
cb c

(3)

Estas equaes diferem apenas nos segundos membros: na equao


(1) o denominador do quociente a, na (2) b, e na (3) c.
Isolando c na equao (1), obtemos c = (a + b)/2, ou seja, c a
mdia aritmtica de a e b; isolando c em (2), obtemos
c = 2ab/(a + b), ou seja, c a mdia harmnica de a e b; isolando c
em (3), obtemos c = ab , ou seja, c a mdia geomtrica de a e b.
Por exemplo, para os nmeros 2 e 18, temos:

A=

2 + 18
= 10
2

G = 2.18 = 6

H=

2.2.18
= 3, 6 .
2 + 18

Neste exemplo, observa-se que qualquer destas mdias est entre o


menor e o maior dos nmeros. Mais ainda, no exemplo:
2 < 3,6 < 6 < 10, ou seja a < H < G < A < b.
Vamos mostrar agora que este fato geral, isto :
Dados os nmeros positivos a e b, com a < b,
tem-se: a < H < G < A < b.
Alm disso, se a < b, ento: a < H < G < A < b.
De fato, se a = b, ento A =

a+a
2aa
= a; G = a.a = a; H =
= a,
2
a+a

Por outro lado, se a < b, temos, sucessivamente:


a + b < b + b = 2b; a(a + b) < 2ab; a <

2ab
, isto : a < H.
a+b

RPM OBMEP

isto , a = H = G = A = b.

101

2
(a b)2 = a2 2ab + b2 > 0; a2 + 2ab + b2 > 4ab; (a + b) >

ab >

4a 2 b 2
( a + b)

ab >

4a 2 b 2
;
ab

2ab
, ou seja: H < G.
a+b

0 < ( a b )2 = a + b 2 a b ;

a + b < b + b; a + b < 2b;

ab <

a+b
, ou seja: G < A.
2

a+b
< b , isto : A < b.
2

possvel tambm visualizar geometricamente essas desigualdades.


Para isso, como na figura 1, colocamos consecutivamente numa mesma
reta os segmentos PQ = a e QS = b, com Q entre P e S. Com centro
no ponto mdio M de PS, construmos uma semicircunferncia K, e
os segmentos MT e QD, perpendiculares a PS, com T e D em K.
T
D
K
A
H
E
G
P

figura 1

Q
b

Construimos tambm o segmento DM e o ponto E, projeo


ortogonal de Q sobre DM. Como a + b dimetro e MT o raio de K,
ento MT a mdia aritmtica de a e b, isto , MT = A.

RPM OBMEP

Alm disso, o tringulo PDS retngulo em D, por estar inscrito


em uma semicircunferencia de dimetro PS; logo, a altura QD a
mdia geomtrica de PQ = a e QS = b, ou seja: QD = G.

102

Finalmente, no tringulo retngulo DQM, DE a projeo ortogonal


do cateto DQ sobre a hipotenusa DM = A como DE; logo: DQ2 =
DM.DE, isto : DE = DQ2/DM = G2/A = H. Logo: DE = H a mdia
harmnica de a e b.

Na figura 1, pode-se verificar que H < G < A. De fato, H < G


porque H cateto e G hipotenusa no tringulo DEQ, enquanto
G < A porque G cateto e A hipotenusa no tringulo DQM.
Note que quando a = b, o ponto Q coincide com M. Neste caso,
H = G = A, como era de esperar.
Outra maneira de visualizar as desigualdades entre as mdias
considerar um trapzio com bases a e b.
a

b
figura 2

Observamos que conforme um segmento paralelo s bases a e b,


com extremidades nos dois lados transversos, caminha se afastando
de a e se aproximando de b, sua medida assume todos os valores entre
a e b, e, consequentemente, todas as mdias consideradas.
Veremos a seguir que os valores A, G e H aparecem conforme
o segmento paralelo s bases assume alguma posio notvel no trapzio.
1) Suponhamos o segmento de medida m equidistando das bases.
a
m

h
h

A soma das reas dos dois trapzios menores a rea do trapzio


inicial:

(a + m)h (m + b)h (a + b)2h o que implica m = a + b = A .


+
=
2
2
2
2

RPM OBMEP

b
figura 3

103

2) Consideremos agora que o segmento de medida m divida o trapzio


inicial em dois trapzios semelhantes.
a
h1

m
A

h2

figura 4
Temos

a m
= , implicando m = ab = G .
m b

Note que, sendo

h
a
= 1 , pois os trapzios so semelhantes,
ab h2

temos h1 < h2, pois a < ab , e, por isso, h1 < h. Logo, m = G < A.
3) Finalmente, consideremos o segmento de medida m passando pelo
encontro das diagonais.
a

h3
h4

b
figura 5

Por semelhana de tringulos temos:

RPM OBMEP

h3
h4
h4
x
x
y
=
=
;
; e =
,
a h4 + h3 b h4 + h3
a h4 + h3

104

implicando x = y e

x y
+ = 1.
a b

Substituindo x = y na ltima igualdade, encontramos x =


portanto, m = 2 x =

2ab
=H .
a+b

Ainda, usando semelhana de tringulos, temos

ab
e,
a+b

a h3
= . Mas
b h4

h
a
a
a
h
h
= 1 e
, ou seja, 3 < 1 e como h3 + h4 = h1 + h2
<
b
ab h2
h4 h2
ab

tem-se h3 < h1 e, portanto, H < G.


Caso permitssemos que as bases do trapzio se igualassem, ou seja,
ter a = b, o trapzio se transformaria num paralelogramo e assim, obviamente, as trs mdias se igualariam a a e b.
Onde aparece a mdia harmnica
So inevitveis as perguntas pragmticas que alunos e professores
costumam fazer: Para que serve o estudo da mdia harmnica? Onde se
aplica a mdia harmnica?
Sem a pretenso de responder cabalmente a essas perguntas, vou
apenas salientar a importncia da mdia harmnica, assinalando a sua
presena em alguns problemas da vida prtica.
O problema das velocidades

A resposta mais imediata que surge em nosso crebro que a


velocidade mdia no percurso todo a mdia aritmtica das velocidades
na ida e na volta, o que daria 90 km/h. Essa resposta, embora intuitiva,
est errada! Temos que estar sempre alertas, maneira dos escoteiros,
para no deixar a razo matemtica ser desgovernada por falsas
intuies.

RPM OBMEP

O sr. Mrio, um imprudente vendedor de filtros de gua, costuma


acordar cedo e viajar de carro, da cidade A at a cidade B, com a
velocidade mdia de 120 km/h. Depois de visitar seus clientes e tomar
com eles algumas garrafas de cerveja, ele volta de B para A, com a
velocidade mdia de 60 km/h. Qual a velocidade mdia que o sr.
Mrio desenvolve no percurso todo?

105

A resoluo correta do problema a seguinte. Sejam:


d: a distncia entre as cidades A e B
v1: a velocidade mdia na ida

t1: o tempo de viagem na ida

v2: a velocidade mdia na volta

t2: o tempo de viagem na volta

Temos ento que d = v1t1 = v2t2. Se v a velocidade mdia no percurso todo, temos:
2d = v(t1 + t2). Logo, 2d = v(d/v1 + d/v2).
Simplificando: v = 2v1v2/(v1 + v2).
Substituindo os valores v1 = 120 km/h e v2 = 60 km/h, obtemos
v = 80 km/h.
Moral da histria: a velocidade mdia no percurso todo a mdia
harmnica das velocidades na ida e na volta.
A mdia harmnica geralmente aparece em problemas que envolvem
velocidades, vazes, frequncias e taxas. O exemplo seguinte uma
verso simples de um problema de vazo bastante conhecido.
O problema das torneiras
Se uma torneira enche um tanque em 60 minutos e uma outra torneira
enche o mesmo tanque em 30 minutos, em quanto tempo as duas
torneiras juntas enchem o tanque?
Os leitores esto convidados a resolver mais esse problema, e para
isso damos uma pequena dica: a resposta no a mdia harmnica de
60 min e 30 min, mas est relacionada a ela.

RPM OBMEP

Problemas de torneiras so antiqussimos. Uma de suas verses


aparece por exemplo na Antologia grega organizada por Metrodoro, um
matemtico grego que vivia por volta do ano 500 depois de Cristo. A
traduo para o portugus seria mais ou menos a seguinte:

106

Eu sou um leo de bronze; de meus olhos, boca e p direito jorra


gua. Meu olho direito enche uma jarra em dois dias, meu olho
esquerdo em trs dias, e meu p direito em quatro dias. Minha
boca capaz de ench-la em seis horas, diga-me quanto tempo
os quatro juntos levaro para ench-la?
Para finalizar esta seo, mais um problema.

O problema do usque
Durante 4 meses consecutivos, o sr. Mrio comprou usque para o
bar de sua casa aos preos, respectivamente, de 16, 18, 21 e 25 reais
por garrafa. Qual foi o custo mdio do usque para o sr. Mrio nesse
perodo todo?
Esse um daqueles problemas que nos deixam frustrados, pois s
depois de muita batalha notamos que faltam dados; temos
necessariamente que introduzir alguma hiptese para poder resolver o
problema.
(i) Uma hiptese plausvel que, talvez por ser um bebedor regular, o
sr. Mrio tenha comprado a mesma quantidade x de usque a cada
ms.
Logo, ele despendeu 16x + 18x + 21x + 25x = 80x reais para comprar
usque no perodo. Da, o custo mdio no perodo de 4 meses foi de
80x/4x = 20 reais por garrafa. Portanto, caso essa hiptese seja verdadeira,
o custo mdio no perodo a mdia aritmtica dos custos mensais.
(ii) Uma outra hiptese plausvel que, talvez por no ter tido aumento
de salrio nesse perodo, o sr. Mrio tenha gasto a mesma quantia y
de reais a cada ms.
Logo, ele consumiu y/16 + y/18 + y/21 + y/25 garrafas no perodo.
Assim, o custo mdio nesse perodo foi, aproximadamente:
4y/(y/16 + y/18 + y/21 + y/25) = 19,5 reais por garrafa.
Portanto, neste caso, o custo mdio no perodo a mdia harmnica
dos custos mensais.
Mdias para mais de dois nmeros

Dados n nmeros positivos x1, x2, ... , xn, definimos.


A=

x1 + x2 + ... + xn e
G = n x1 x2 ... xn .
n

RPM OBMEP

As mdias que vimos para dois nmeros podem ser generalizadas


para mais nmeros. Vamos fazer isto aqui somente para as mdias
aritmtica e geomtrica.

107

Deixamos para o leitor verificar que, se todos os nmeros forem iguais


a um valor v, ento A = G = v. Porm, se eles no forem todos iguais,
a mdia geomtrica sempre menor que a mdia aritmtica, mas a
demonstrao, nesse caso, no to fcil como no caso n = 2. Existem
demonstraes de vrios tipos, de diversos graus de sofisticao e
baseadas em diferentes teorias. A mais conhecida a demonstrao de
Cauchy (1789-1857), que pode ser encontrada no livro Meu Professor
de Matemtica e outras histrias de Elon Lages Lima, p. 153, publicado
pela SBM. Vamos dar aqui a demonstrao concebida por Polya, que se
baseia na desigualdade ex > 1 + x, justificada a seguir:
Observemos que se pode definir o logaritmo (natural) de um nmero
positivo a como sendo a rea limitada pelo
eixo das abscissas, pela curva y = l/x e pelas
y = 1x
retas verticais x = 1 e x = a. Como essa
regio est contida no retngulo de altura 1 e
base a 1, temos, claro, ln a < a 1. 1
Fazendo a = 1 + x, obtemos ln(1+ x) < x ou
lna
ex > 1 + x, valendo a igualdade apenas se
a
1
a = 1, ou seja, x = 0.
Resolvida essa parte, podemos ento demonstrar a desigualdade das
mdias para n nmeros.
Na desigualdade ex > 1 + x vamos substituir x por

xi
1 , com
A

i = 1, 2, ..., n, obtendo as relaes


x1
1

x1
A

eA

x2
A

xn
1
eA

xn
A

eA

RPM OBMEP

x2
1

108

Multiplicando, obtemos

x1 + x2 +...+ xn
n
A
e

x1 x2 ... xn
An

Mas, x1 + x2 + ... + xn = nA, logo 1

Gn

ou A > G.
An
como queramos demonstrar. claro que a igualdade vale se, e somente
se,

xi
1 = 0 ou seja, xi = A para todo i = 1, 2, ..., n.
A

Aplicaes das desigualdades das mdias

1
2.
x
Soluo: Aplicando a desigualdade das mdias aos nmeros x e l/x,
Exemplo 1: Mostre que se x > 0, ento: x +

1
x x. 1 = 1 ou seja, x + 1 2 , ocorrendo a igualdade
x
2
x

x+
obtemos

se e somente se, x = 1.
Exemplo 2
Para todos os valores das variveis x, y, z, w, reais positivas, qual o
menor valor da expresso
E=

x y z w
+ + +
y z w x ?

Soluo:

x y z w
+ + +
x y z w
y z w x
4 . . . =1
4
y z w x
Logo, E > 4, ocorrendo a igualdade se x = y = z = w.
Exemplo 3

Soluo: Escrevemos y = x 2 +

1
?
x

1
1
. Desta forma temos
+
2x 2x

RPM OBMEP

2
Para x > 0, qual o valor mnimo de y = x +

109

x2 +

Portanto, y

x=

1
.
2

1
1
+
2 x 2 x 3 x2 . 1 . 1 = 3 1 .
3
2x 2x
4

3
1
, ocorrendo a igualdade quando x 2 =
, ou seja,
2x
4

Exemplo 4
Se x, y e z so positivos, qual o valor mnimo de

1 1 1
( x + y + z )( + + ) ?
x y z
Este exemplo ser deixado como exerccio para o leitor. A resposta 9.
A desigualdade entre as mdias aritmtica e geomtrica tem como
consequncia as seguintes afirmaes:
I) Se a soma de n nmeros positivos for constante, ento o produto
ser mximo quando todos os nmeros forem iguais.
II) Se o produto de n nmeros positivos for constante, ento a soma
ser mnima quando todos os nmeros forem iguais.
Daremos mais dois exemplos para mostrar como funciona a afirmao I).
Exemplo 5
Sendo x e y nmeros reais positivos, determinar o mximo de
E = xy(1 x y).

RPM OBMEP

Soluo: Consideremos apenas os valores de x e y tais que x + y < 1


(se x + y > 1, teremos 1 x y < 0 e o mximo que estamos procurando
obviamente positivo).

110

Ento, os nmeros x, y e 1 x y so positivos e possuem soma igual


a 1. Logo, o produto ser mximo quando todos forem iguais ou seja,

1 1 1 1
.
Emax = . . =
3 3 3 27

Exemplo 6
Provar que, de todos os tringulos de mesmo permetro, o equiltero
possui a maior rea.
Soluo: Consideremos um tringulo de lados a, b e c com
a + b + c = 2p. A rea S desse tringulo dada pela frmula de Heron,
S=

p.( p a )( p b)( p c)

Para poder aplicar a afirmativa I, devemos escrever


S=

p . ( p a )( p b)( p c)

Ora, o semipermetro p constante. Ento S ser mximo quando


(p a)(p b)(p c) for mximo. Mas
p a + p b + p c = 3p 2p = p (constante)
logo o produto ser mximo quando

p a = p b = p c =

2p
como queramos demonstrar.
3
Adaptado dos artigos
Duas mdias
Eduardo Wagner, RPM 18.

Mdia harmnica
Seiji Hariki, RPM 32.

Uma aula sobre mdias


Chico Nery, RPM 68.

RPM OBMEP

a=b=c=

p
, ou seja
3

111

Problemas diversos resolvidos


com Geometria Analtica

A Geometria Analtica, ou melhor o mtodo das


coordenadas, uma ferramente til para resolver
problemas diversos, mesmo aqueles que no contm
equaes ou coordenadas. interessante observar
problemas que permitem a introduo de um sistema
adequado de coordenadas e conseguir assim, solues
simples e convincentes.
A seguir, mostraremos alguns problemas desse tipo. Para
cada um deles outras formas de resoluo so possveis,
mas o mtodo das coordenadas uma boa opo.
Problema 1

RPM OBMEP

Um pesado caminho parte ao meio dia da cidade A para


a cidade B viajando com velocidade constante de
40 km/h, e s 6 horas da tarde chega cidade B. Um
automvel parte da cidade B s 2 horas da tarde desse
dia e, viajando com velocidade constante pela mesma
estrada, chega cidade A tambm s 6 da tarde. Perguntase em que momento o caminho e o automvel se
cruzaram na estrada.

112

Soluo
A distncia entre as cidades A e B, ao longo da estrada
de 6 40 = 240 km. Vamos introduzir o seguinte sistema
de coordenadas: para um objeto qualquer que se mova

ao longo da estrada, seja x o tempo (em horas) decorrido aps o meio


dia e seja y (em quilmetros) a sua distncia cidade A. Os grficos
correspondentes aos movimentos do caminho e do automvel so retas,
uma vez que eles viajam com velocidades constantes. De acordo com os
dados do problema, o grfico que mostra o movimento do caminho
um segmento de reta cujos extremos so os pontos (0, 0) e (6, 240), e
o grfico que mostra o movimento do automvel um segmento de reta
cujos extremos so os pontos (2, 240) e (6, 0). As equaes das retas
do grfico abaixo so y = 40x e y = 60x + 360.
y

Resolvendo o sistema encontramos x = 3,6 e y = 144. Conclu- 240


mos ento que o encontro se deu
3,6 horas aps o meio dia, ou
seja s 3 horas e 36 minutos da
tarde e, nesse momento, ambos
estavam a 144 km da cidade A.

Esse problema nada tem de original, mas serve para ilustrar que
diversos problemas de cinemtica escalar podem ser resolvidos com o
mtodo analtico.
Problema 2
Considere todos os nmeros reais x e y tais que x + 2y = 10. Para que
valores de x e y a expresso E = x2 + y2 assume menor valor?
Soluo
Este um problema de lgebra. Sua soluo, uma vez que o enunciado
esteja bem entendido, no difcil. Entretanto, a soluo analtica
interessante.
y
s

P
r

RPM OBMEP

Estabelecendo um sistema de
coordenadas, todos os pontos
P(x, y) tais que x + 2y = 10
pertencem a uma reta r e o valor
de E o quadrado da distncia
de P origem do sistema de
coordenadas.

113

Precisamos ento encontrar o ponto de r cuja distncia ao ponto (0, 0)


mnima. A reta s, perpendicular a r e passando pela origem tem equao
2x y = 0 e a interseo dessas retas o ponto que procuramos.
Resolvendo o sistema formado pelas duas equaes encontramos x = 2
e y = 4 que a soluo do problema. Conclumos ainda que o valor
mnimo de E 22 + 42 = 20.
Problema 3
Na molcula do metano (CH4) o tomo de carbono ocupa o centro de
um tetraedro regular em cujos vrtices esto os tomos de hidrognio.
Determine o ngulo entre duas das valncias do carbono.
Soluo
O resultado deste problema est presente em todos os cursos de qumica
orgnica. O estranho nmero fornecido aceito por todos, mas, em geral,
no se tem a menor idia de como esse resultado foi obtido. Para calcular
esse ngulo, a Geometria Analtica um mtodo imbatvel, aliada
claro, com alguma inventividade.
Vamos utilizar um sistema de coordenadas no espao e usar a frmula
G
G
que fornece o cosseno do ngulo entre dois vetores u e v :
G G
u v
cos = G G . Consideremos inicialmente um cubo de aresta 2 (para
| u || v |
facilitar) com um vrtice na origem, outro no eixo X, outro no eixo Y
e outro no eixo Z. No difcil escolher quatro vrtices deste cubo que
formem um tetraedro regular.

RPM OBMEP

Os pontos A = (0, 0, 0), B = (2, 2, 0), C = (0, 2, 2) e D = (2, 0, 2)


formam um tetraedro regular (uma vez que as distncias entre dois
quaisquer deles so diagonais de faces do cubo) e so ocupados pelos
hidrognios.

114

O ponto P = (1, 1, 1), centro do cubo e tambm centro do tetraedro,


est ocupado pelo carbono.
O resto fcil. Para calcular, por exemplo o ngulo APB,
G JJJG
G JJJG
consideremos os vetores u = PA = (1, 1, 1) e v = PB = (1,1, 1) .

O cosseno do ngulo entre eles :


cos =

1 1 + 1
1
= .
3
3 3

D
P

Com uma calculadora, determinamos um


valor muito aproximado para esse ngulo:
= 1092816,395".

Vamos, nos prximos exemplos, resolver vrios problemas de


Geometria Plana, usando Geometria Analtica. Naturalmente, todos os
problemas apresentados podem ser resolvidos utilizando-se geometria
sinttica. Sugerimos que os leitores tentem obter essas solues.
Problema 4
O quadrado ABCD tem lado 10. Sendo M o ponto mdio de BC, trace
DP perpendicular a AM. Qual o comprimento do segmento DP?
Soluo
A soluo pela geometria sinttica passa pela descoberta que os tringulos
ABM e DPA so semelhantes. Com isso, e mais uma aplicao do
teorema de Pitgoras se resolve o problema. Entretanto, com os recursos
da Geometria Analtica, a soluo no depende da descoberta dessa
semelhana. Podemos escolher um sistema de coordenadas com o eixo
X passando por AB e com o eixo Y passando por AD.
Na figura ao lado temos:

d=
Temos, ento, DP =

| ax0 + by0 + c | .

| 0 2.10 |
12 + (2) 2

a 2 + b2

20
=4 5.
5

M
P
A

B
RPM OBMEP

A = (0, 0), B = (10, 0), C = (10, 10),


D = (0, 10) e M = (10, 5). A equao da
reta AM x 2y = 0 e o comprimento do
segmento DP a distncia do ponto D
reta AM. A distncia do ponto (x0, y0)
reta ax + by + c = 0 dada por:

115

Problema 5
Na figura, o quadrado ABCD tem lado 9 e os
pontos P e Q dividem o lado CD em trs
segmentos congruentes. Calcule a distncia do
vrtice A ao baricentro G do tringulo BPQ.

G
D

Soluo

Encaixemos o quadrado ABCD no primeiro quadrante do plano


cartesiano, com o vrtice D coincidindo com a origem. Sendo
A= (0, 9), B = (9, 9), P = (3, 0) e Q = (6, 0), conhecido que as coordenadas do baricentro G so:
y

xB + xP + xQ

9+3+ 6
= 6,
3
3
yB + yP + yQ 9 + 0 + 0
yG =
=
= 3,
3
3
portanto, G = (6, 3).
xG =

G
D

A distncia procurada :

d AG = ( xG x A ) 2 + ( yG y A ) 2 = (6 0) 2 + (3 9) 2 = 72 = 6 2 .

Problema 6

As medianas AM e BN de um
tringulo ABC so perpendiculares
e medem, respectivamente, 9 cm e
12 cm. Calcule o comprimento da
terceira mediana desse tringulo.

G
B

RPM OBMEP

Soluo

116

O encontro das medianas o


baricentro G do tringulo ABC.
Usando o fato de que as medianas
AM e BN se cortam perpendicularmente em G, coloquemos
esse tringulo no plano cartesiano

M
y

6 A

8


N
M

com origem em G. Usando a conhecida proporo em que G divide as


medianas, temos:
AM = 9 AG = 6 e GM = 3
, ou seja, A = (0, 6), M = (0, 3),

BN = 12 BG = 8 e GN = 4

B = (8, 0) e N = (4, 0). As equaes segmentrias das retas AN e BM


so, respectivamente,
x y
+ =1 e
4 6

x
y
+
= 1.
8 3

Resolvendo o sistema anterior, encontramos o ponto C = (8, 6). O


comprimento da terceira mediana 3/2 da distncia entre C e G:

3
3
( xC xG ) 2 + ( yC yG ) 2 =
(8 0) 2 + (6 0) 2 = 15 cm .
2
2
Problema 7

Calcule a rea do tringulo ADE,


retngulo em E, inscrito num trapzio
retngulo ABCD, com AB = 10 cm,
AD = 30 cm e CD = 20 cm (figura).
Encaixemos o trapzio ABCD no
primeiro quadrante do plano cartesiano,
fazendo os lados AD e AB ficarem
contidos, respectivamente, nos eixos

x e y. Como a reta BC tem coeficiente

10

C
E

coeficiente linear 10, sua equao reduzida

20

M
15

D x

1
y = x + 10 . A
3

circunferncia de dimetro AD, com centro M = (15, 0), passa pelo


ponto E e tem equao:
(x 15)2 + y2 = 152.

RPM OBMEP

yC yB 20 10 1
=
= e
xC xB
30 0 3

Soluo

angular m =

117

y = x + 10
O ponto E dado pela soluo do sistema:
,
3
( x 15) 2 + y 2 = 225

ou seja, E = (6, 12) ou E = (15, 15). Portanto, a rea do tringulo ADE

: S ADE

0 0 1
0 0 1
1
1
2
= 30 0 1 = 180 cm ou
30 0 1 = 225 cm 2 .
2
2
6 12 1
15 15 1

Adaptado dos artigos


Sobre o ensino de Geometria Analtica
Eduardo Wagner, RPM 41

RPM OBMEP

A Geometria Analtica no ensino mdio


Chico Nery, RPM 67

118

A sombra do meu abajur

Introduo
A fotografia abaixo reproduz o abajur do meu quarto e a
sombra que ele projeta na parede. Que curvas so essas?

figura 1

RPM OBMEP

A cpula do abajur um tronco de cone, com altura h e


raios das bases R e r. A lmpada centralizada, de modo
a ficar no eixo do tronco de cone. Para simplificar,
podemos imaginar a lmpada concentrada em um ponto,
situado a uma distncia b da base inferior e a uma
distncia c da base superior do tronco, sendo b + c = h
(ver figura 1).

119

A funo da cpula barrar uma parte dos raios de luz, evitando que
a luz atinja diretamente a vista. Os raios de luz que escapam dessa
barragem formam um par de cones, ambos com vrtice na lmpada (ver
figura 2). Devemos imaginar esses cones prolongados para alm das
bases da cpula, um para cima e outro para baixo.

figura 2

Equacionamento e resoluo do problema


Para descobrir a natureza da sombra do abajur, vamos cuidar primeiro
da parte superior da sombra, que a interseo da parede com o cone de
luz superior. Esse cone fica caracterizado pelo ngulo da figura 3, que
define sua abertura e tal que: m = tg = r/c.

RPM OBMEP

figura 3

120

Criemos um sistema de coordenadas em trs dimenses OXYZ, de


modo que a origem O do sistema esteja sobre a lmpada (concebida
como um ponto) e o eixo do cone coincida com o semi-eixo positivo
OZ, como na figura 4.

figura 4

Nessa figura, vemos que um ponto genrico P = (x, y, z) pertence


superfcie do cone se e s se, enquanto ele distar z = AP do plano XOY,
sua distncia PB ao eixo permanecer igual a OA = x 2 + y 2 . Porm,
a figura 5, vista no plano AOB, mostra que: PB/PA = tg = m, ou seja:

figura 5

A parede um plano paralelo ao eixo do cone. Podemos ajustar os


eixos OX e OY de modo que o plano XOZ fique paralelo parede. Nesse

RPM OBMEP

PB = mPA = mz. Logo, a equao do cone : mz = x 2 + y 2 .

121

caso, a parede tem equao y = d, onde d a distncia da lmpada


parede.
Finalmente, a curva que procuramos a interseo do cone de equao
mz = x 2 + y 2 com o plano de equao y = d, isto , o conjunto dos
2
2

pontos (x, y, z) que so solues do sistema: mz = x + y .


y = d
2
2

Esse sistema obviamente equivalente ao sistema: mz = x + d ,


y = d

e, se olharmos esses pontos no plano y = d, poderemos ficar somente


com a equao mz = x 2 + d 2 , que a equao dessa curva plana
nesse plano.
Lembrando que m = r/c, essa equao fica: z =

c 2
x + d2 .
r

Para a parte inferior da sombra, um raciocnio inteiramente anlogo


concluiria que a equao dessa parte da sombra : z =

b
x2 + d 2 .
R

Para o abajur do meu quarto, essas dimenses so, aproximadamente:


r =10 cm, R = 25 cm, b = 10 cm, c = 20 cm e
(quando o abajur est no seu lugar mais usual) d = 40 cm.
Nesse caso, as equaes da
sombra so, em centmetros:

z = 2 x 2 + 1600

RPM OBMEP

e z = 0, 4 x 2 + 1600 .

122

A figura 6 mostra esses


grficos obtidos por um programa
de computador.
figura 6

Identificao das curvas


Essas curvas que obtivemos so uma novidade ou ser que j as vimos
por a no ensino mdio? Na verdade, elas so velhas conhecidas. A sombra
superior tem equao z =

c 2
x + d 2 . Elevando essa equao ao
r

quadrado e manipulando, obtemos:

z2

( )
cd
r

x2
d2

= 1 , que a equao de

uma hiprbole com centro na origem do plano XZ, eixo transverso sobre
o eixo Z, de comprimento

2cd
, e eixo no transverso sobre o eixo X, de
r

comprimento 2d.
Como a equao z =

c 2
x + d 2 equivalente ao sistema,
r

z2
x2

=1

2
d2
cdr
, v-se que a sombra superior o ramo positivo dessa

z > 0

( )

hiprbole. Analogamente, a sombra inferior o ramo negativo da


hiprbole de equao:

z2
( bd
)2
R

x2
d2

= 1.

interessante observar que, quando a lmpada se situa exatamente

c b
=
r R
(verifique!), de modo que a sombra superior e a sombra inferior so os
dois ramos de uma mesma hiprbole.
O leitor pode verificar tambm que, se o abajur for cilndrico, as
duas partes da sombra sero tambm os dois ramos de uma mesma
hiprbole. Mais ainda: se o cilindro for eqiltero (altura igual ao
dimetro da base) e a lmpada estiver centralizada, a resultante hiprbole

RPM OBMEP

no encontro das diagonais do trapzio da figura 1, ento

123

ser equiltera (eixos transverso e no transverso de mesmo comprimento).


Comentrios
1. No deve ser novidade para muitos leitores que a interseo de um
cone e um plano possa ser, em certos casos, uma hiprbole. Na
verdade, o nome cnicas vem justamente do fato de que a seo de
um cone de duas folhas por um plano , em geral, uma elipse, uma
parbola ou uma hiprbole, conforme o plano seccionador forme com
o eixo do cone um ngulo maior, igual ou menor do que o ngulo que
a geratriz do cone forma com o seu eixo. No caso em questo, o
plano da parede paralelo geratriz do cone, formando portanto um
ngulo nulo, menor do que o ngulo que a geratriz do cone forma
com o seu eixo.
Esse teorema j era conhecido por Apolnio de Perga (sc. III a.C.) e
j foi algumas vezes citado na RPM.
2. Um tema-chave no ensino mdio o ensino de funes (reais de uma
varivel real) e seus grficos. Dentre essas, ningum pode negar a
grande importncia das funes polinomiais de 1o e 2o graus, das
funes logaritmo e exponencial e das funes trigonomtricas.
Muitas vezes, porm, queremos sair um pouco da rotina e apresentar
outras funes que tenham uma definio simples e estejam ligadas
a aplicaes prticas. Est a um interessante exemplo: as funes da
forma f ( x) = k x 2 + d 2 , cujo grfico um ramo de hiprbole. Alis,
uma boa ocasio para pensar tambm nos grficos de funes da
forma f ( x) = k x 2 d 2 ou f ( x) = k d 2 x 2 , cujos grficos so
tambm partes de cnicas (desafio: quais?).

RPM OBMEP

Adaptado do artigo

124

A sombra do meu abajur


Jos Paulo Q. Carneiro, RPM 59.

A ilha do tesouro
Dois problemas e duas solues

Problema 1
O problema a seguir foi inspirado numa histria do livro
Um, dois, trs, ..., infinito de George Gamow.
Era uma vez dois irmos aventureiros que encontraram,
no ba das lembranas de seu bisav, o mapa de um
tesouro, juntamente com as instrues para localiz-lo.

Encontrariam o tesouro enterrado exatamente no ponto


de interseco de AM com T1T2.
Os jovens viajaram muito contentes at a ilha, levando
cordas e outras ferramentas necessrias. L estavam a
formosa plancie, a grande clareira circular e a comprida
fila de belas palmeiras. Mas todas as palmeiras

RPM OBMEP

O tesouro estava numa ilha, cuja localizao estava


descrita de forma clara; encontrada a ilha, deveriam
procurar um campo aberto com um grande espao
arenoso, perfeitamente circular. No exterior do dito
crculo encontrariam numerosas palmeiras alinhadas ao
longo de uma reta. Deveriam, ento, procurar a palmeira
com um desenho geomtrico no seu tronco e, partindo
de sua base, traar as tangentes pista circular, chamando
de T1 e T2 os pontos de tangncia. A seguir, deveriam
traar tambm o dimetro, AM, da circunferncia fronteira da clareira, perpendicular reta das palmeiras.

125

apresentavam figuras geomtricas nos seus grossos troncos!


Esse inesperado fato derrubou todos os planos. No sabiam qual era
o ponto inicial e, sem ele, imaginaram que o trabalho seria gigantesco
ou impossvel. Dessa forma tiveram de voltar com as mos vazias
Entretanto, se aqueles aventureiros soubessem um pouco de
Geometria, teriam escolhido uma palmeira qualquer da fila, como ponto
inicial, e teriam encontrado o tesouro. Vejamos por qu.

Na figura:
O e r so, respectivamente, o centro e o raio da circunferncia fronteira
da clareira circular; H o ponto de interseco da reta determinada por
AM com a reta das palmeiras; P o ponto que representa a palmeira
escolhida, eleita para iniciar a procura do tesouro; B o ponto de
interseco de OP com T1T2; T interseco de T1T2 com AM, ponto
onde deveriam cavar para encontrar o tesouro.
Temos ento:
Os tringulos retngulos OBT1 e OT1P so semelhantes.

RPM OBMEP

Logo,

126

OT1 OP
, ou seja, r 2 = OB.OP .
=
OB OT1

Analogamente, os tringulos retngulos OBT e OHP so


semelhantes, o que implica:

OT OB
=
, ou seja, OT .OH = OB.OP .
OP OH

r2
, o que mostra que a posio
OH
do ponto T independe do ponto P, ou seja, independe da palmeira
escolhida inicialmente.

Assim, OT .OH = r 2 ou OT =

Problema 2
O problema a seguir foi inspirado em um exerccio do livro
Polynomials, de E. J. Barbeau, e foi apresentado a professores do ensino
mdio, alunos de um curso, de formao continuada, sobre nmeros
complexos.
Dois piratas decidem enterrar um tesouro em uma ilha. Escolhem,
como pontos de referncia, uma rvore e duas pedras. Comeando na
rvore, medem o nmero de passos at a primeira pedra. Em seguida,
dobram, segundo um ngulo reto, direita e caminham o mesmo nmero
de passos at alcanar um ponto, onde fazem uma marca. Voltam rvore,
medem o nmero de passos desde a rvore at a segunda pedra, dobram
esquerda, segundo um ngulo reto, e caminham o mesmo nmero de
passos at alcanar um ponto, onde fazem outra marca. Finalmente,
enterram o tesouro exatamente no ponto mdio entre as duas marcas.
Anos mais tarde, os dois piratas voltam ilha e decidem desenterrar
o tesouro, mas, para sua decepo, constatam que a rvore no existe
mais (o vento, a chuva e os depredadores a haviam arrancado). Ento
um dos piratas decide arriscar. Escolhe ao acaso um ponto da ilha e diz:
Vamos imaginar que a rvore estivesse aqui. Repete ento os mesmos
procedimentos de quando havia enterrado o tesouro: conta os passos at
a primeira pedra, dobra direita, etc., e encontra o tesouro.

Mesmo tendo sido apresentado em um curso sobre nmeros


complexos, e para alunos que tinham bastante experincia eram
professores de Matemtica , o problema da ilha do tesouro causou uma
comoo. Na verdade, todos admitiram que, se o curso no fosse sobre
nmeros complexos, a nenhum dos presentes teria ocorrido a ideia de
resolver esse problema usando a lgebra dos nmeros complexos. E,
mesmo depois da sugesto para faz-lo, quase ningum conseguiu.

RPM OBMEP

A pergunta : esse pirata era sortudo ou um matemtico?

127

Qual a relao entre o problema e os nmeros complexos? Bem,


tudo se baseia em dois fatos fundamentais:
1) no plano complexo, a diferena entre dois complexos traduz o vetor
com origem no primeiro ponto e extremidade no segundo; o que se
JJJG
costuma formular por: AB = B A ;
2) multiplicar um complexo pelo nmero i (a unidade imaginria)
equivale a gir-lo de um ngulo reto positivo.
A figura ilustra a situao do
problema. Sendo A a rvore, e P e
Q as pedras, o tesouro est no ponto
T mdio dos pontos P e Q.
Considerando os pontos pertencentes
ao plano complexo, no importando
onde esteja a origem, tem-se:

T=

P + Q P i ( P A) + Q + i (Q A) P + Q Q P
.
=
=
+i
2
2
2
2

Observando que P + Q o ponto mdio


2
JJJG
de PQ e que Q P = PQ , esse resultado
no s demonstra que a localizao do tesouro
independe da posio da rvore (o pirata era
um matemtico...), como tambm permite
localiz-lo como o terceiro vrtice de um dos
tringulos retngulos issceles com
hipotenusa PQ.
Adaptado do artigo

RPM OBMEP

A ilha do tesouro. Dois problemas e duas solues

128

Jess A. P. Snchez e Jos Paulo Q. Carneiro, RPM 47.

Qual o mesmo a definio


de polgono convexo?

Quando pensamos num polgono convexo, imaginamos


seus vrtices todos apontando para fora, ou seja, que ele
no possui vrtices reentrantes. Como os dois polgonos
da esquerda na figura 1.

figura 1: Dois polgonos convexos e dois no convexos.

Essa idia intuitiva necessita, entretanto, uma formulao


mais precisa, para poder ser usada com segurana e
generalidade. Alm disso, h outras maneiras de pensar
num polgono convexo. Conforme o contexto, uma dessas
definies pode ser mais adequada do que as outras. Por
isso conveniente conhecer as principais alternativas e
saber mostrar que elas so equivalentes.

Chamamos polgono a uma linha poligonal fechada sem


auto-intersees, isto , cada lado tem apenas um ponto
comum com o lado anterior e com o seguinte, mas no
com os demais.
s vezes, a palavra polgono tambm designa a
regio do plano limitada por essa linha poligonal fechada

RPM OBMEP

A seguir, daremos trs definies diferentes de polgono


convexo e provaremos a equivalncia entre elas.

129

sem auto-intersees. Por exemplo, quando falamos da rea de um


polgono, claro que nos referimos regio poligonal, no linha que
a limita.
Um subconjunto F do plano chama-se uma figura plana convexa
quando, para quaisquer dois pontos X e Y em F, o segmento de reta
XY est inteiramente contido em F.

figura 2: Duas figuras planas convexas e duas no convexas.

Primeira definio
Um polgono diz-se convexo quando a regio por
ele limitada uma figura plana convexa.
Segue-se desta definio que toda diagonal de um polgono convexo
est inteiramente contida na regio por ele limitada.
Para a segunda definio, lembremos que toda reta r decompe o
plano em duas regies que tm r como fronteira comum. Chamaremos
essas regies as margens de r.
As margens de uma reta so figuras planas convexas. Se os pontos X
e Y esto em margens opostas da reta r, o segmento de reta XY
corta r.

RPM OBMEP

Diz-se que r uma reta de apoio do polgono P quando P tem


pelo menos um ponto em comum com r e situa-se inteiramente numa
das margens de r.

130

figura 3: r a reta de apoio dos polgonos P1 e P2 mas no de P3 e P4.

Segunda definio
Um polgono chama-se convexo quando a reta que
contm qualquer dos seus lados uma reta de apoio.
Por exemplo, dos polgonos na figura 3, apenas P4 convexo.
Para formular a terceira definio de polgono convexo, definimos
um ziguezague ABCD como uma poligonal com trs lados, AB, BC e
CD, dispostos de modo que AB e CD se situem em margens opostas
da reta (que contm o segmento) BC.

figura 4: A poligonal ABCD um ziguezague mas ABCD no .

Terceira definio
Um polgono diz-se convexo quando no contm ziguezagues.
Notemos que se ABCD um ziguezague contido no polgono P,
ento um dos vrtices B, C saliente e o outro reentrante.
figura 5: No ziguezague ABCD, o vrtice C
saliente para o polgono P e reentrante
para Q. O contrrio ocorre com o vrtice B.

Para demonstrar a equivalncia entre estas trs definies de polgono


convexo, usaremos a noo de ponta de um polgono.

figura 6: Os vrtices B e D (mas no A e


C) so pontas de ABCD.

RPM OBMEP

Sejam A, B, C vrtices consecutivos do polgono P. Diz-se que B


uma ponta de P quando o segmento AC uma diagonal interna desse
polgono.

131

Lema
Todo polgono tem pelo menos uma ponta.
Demonstrao
Um polgono P, de n lados, decompe-se, mediante diagonais
internas, em n 2 tringulos justapostos (RPM 18, p. 36). Cada um
dos n lados de P pertence a pelo menos um desses n 2 tringulos.
Pelo princpio da casa dos pombos (RPM 8, p. 21) h 2 lados de P no
mesmo tringulo. O vrtice comum a esses dois lados uma ponta de P.
O teorema seguinte estabelece a equivalncia entre as trs definies
de polgono convexo dadas acima.
Teorema 1
Cada uma das seguintes afirmaes a respeito de um polgono P
implica, as demais:
1) A regio limitada por P uma figura plana convexa;
2) A reta que contm qualquer lado de P uma reta de apoio;
3) P no possui ziguezagues.
Demonstrao
Provaremos as implicaes 1) 2) 3) 1).
1) 2). Admitindo 1), suponhamos,
por absurdo, que 2) seja falsa, isto , que
exista um lado AB do polgono P e pontos
X, Y da regio F limitada por P situados
em margens opostas da reta AB, como na
figura 7.

RPM OBMEP

Sendo F convexa, todos os pontos do segmento XY, e da todos os


pontos do tringulo AXY, obtidos ligando A aos pontos de XY, esto
contidos em F. Ento AB no lado de P. Contradio.

132

2) 3). Se ABCD um ziguezague, AB e CD esto em margens


opostas da reta BC. Portanto, um polgono onde a reta que contm
qualquer dos seus lados de apoio no pode conter ziguezagues.
3) 1). Para provar esta ltima implicao suponhamos, por
absurdo, que exista um polgono P, com n lados, que no contm

ziguezagues mas a regio F, por ela limitada, no uma figura plana


convexa. Tomemos P de modo que n seja o menor possvel. Ento
3) 1) para polgonos com menos de n lados. Pelo lema, existem
vrtices consecutivos L, A, B, C, D de P tais que B uma ponta. A
diagonal AC decompe P em dois polgonos justapostos: o tringulo
ABC e um polgono Q, de n 1 lados, que no contm ziguezagues,
logo limita uma figura plana convexa G.

figura 8: O polgono P decomposto no tringulo ABC e


no polgono Q, de n 1 lados.

Assim, para provar que F uma figura plana convexa, basta tomar
um ponto X no tringulo ABC, um ponto Y na regio G e mostrar que
o segmento de reta XY est contido em F. Como j vimos que 1) 2),
sabemos que AC uma reta de apoio para Q, logo X e Y esto em
margens opostas de AC. Alm disso, como LABC e ABCD no so
ziguezagues, X e Y esto na mesma margem em relao s retas AB e
BC. Tudo isto significa que o segmento XY corta a reta AC mas no as
retas AB ou BC. Noutras palavras, o segmento XY sai do tringulo
ABC por um ponto Z do segmento AC. Ento XZ est contido no
tringulo ABC e ZY est contido na regio G, logo XY est contido
na regio F, como queramos demonstrar.

1. A primeira definio a que melhor se adapta aos padres atuais da


Matemtica, tanto Pura, como Aplicada. Ela se aplica literalmente a
figuras slidas com um nmero qualquer de dimenses. Dela resulta
facilmente que a interseo de duas ou mais figuras convexas uma
figura convexa. Por isso simples deduzir dela que um polgono
convexo se, e somente se, tem exatamente dois pontos em comum
com qualquer reta que passa pelo seu interior. (Isto seria uma quarta
definio de polgono convexo.)

RPM OBMEP

Para finalizar, breves observaes sobre as definies acima propostas:

133

2. A segunda definio tambm se estende a poliedros em espaos com


um nmero qualquer de dimenses. Ela permite caracterizar um
polgono convexo como o conjunto das solues (x, y) de um sistema
de desigualdades lineares do tipo ax + by < c. Por isso desempenha
papel fundamental em Programao Linear.
3. As duas primeiras definies tm carter global enquanto a terceira
nitidamente local. Para verificar se um dado polgono convexo no
sentido das duas primeiras definies necessrio examinar (vrias
vezes) todos os seus lados ao mesmo tempo. J na terceira definio,
para cada lado, olha-se apenas para o lado sua esquerda e para o
lado sua direita. Do ponto de vista computacional, isto bem mais
simples. Por outro lado, a no existncia de ziguezagues s faz sentido
no plano. Alm disso, trata-se de uma hiptese da qual, em que pese
seu grande apelo geomtrico, difcil deduzir conseqncias.
(Compare 3) 1) com as outras implicaes.)
Adaptado do artigo

RPM OBMEP

Qual mesmo a definio de polgono convexo?


Elon Lages Lima, RPM 21.

134

A soluo de Tartaglia para a


equao do 3o grau e a emergncia
dos nmeros complexos

Introduo

Uma das personagens dessa histria Niccol Fontana


(1500-1557 aprox.). Em 1512 os franceses saquearam
Brescia, sua cidade natal. Sua me buscou refgio para o
filho na igreja, mas os soldados tambm invadiram o
santurio, e a criana foi ferida no rosto. O ferimento lhe
causou uma gagueira permanente, que lhe valeu o apelido
de Tartaglia (gago, em italiano), pelo qual se tornou
conhecido. Ele no foi o primeiro a obter o mtodo de
resoluo dessas equaes; Scipione del Ferro (14651562 aprox.), que foi professor na Universidade de
Bolonha e cuja biografia pouco conhecida, foi o
verdadeiro descobridor. Antes de morrer, del Ferro
ensinou seu mtodo a dois discpulos, Annibale della
Nave - seu futuro genro e sucessor na ctedra em Bolonha
- e Antnio Maria Fior (ou Floridus, em latim).
Em 1535 houve uma disputa matemtica entre Fior e
Tartaglia. Tais confrontos intelectuais no eram infrequentes na poca e, muitas vezes, a permanncia de um
matemtico numa ctedra dependia de seu bom

RPM OBMEP

A histria da resoluo da equao de terceiro grau


muito pitoresca, plena de lances dramticos, paixes e
disputas pela fama e a fortuna que seu achado poderia
trazer a seus autores.

135

desempenho nesses encontros. Cada um dos adversrios props ao outro


trinta problemas e foi combinado que o perdedor deveria pagar trinta
banquetes ao ganhador. Tartaglia preparou questes variadas, mas todos
os problemas propostos por Fior implicavam equaes do tipo X3 + aX
= b. Precisamente na noite de 12 para 13 de fevereiro, Tartaglia conseguiu
descobrir o mtodo de resoluo de tais equaes e, na hora do confronto,
verificou-se que Tartaglia tinha resolvido todas as questes propostas
por Fior, enquanto este no tinha conseguido resolver a maioria das
questes submetidas por Tartaglia. Declarado vencedor, Tartaglia
voluntariamente renunciou aos trinta banquetes.
A notcia do triunfo de Tartaglia logo se espalhou e chegou aos
ouvidos de Girolamo Cardano (1501-1576), que, na poca, ocupava
uma cadeira de medicina na Universidade de Pavia e era membro do
Colgio Mdico de Milo. De todos os participantes da nossa histria,
talvez seja Cardano o mais enigmtico, aquele cuja vida mais pitoresca
e, certamente, que teve uma formao mais universal.
Para termos uma idia de quo extenso e profundo era seu
conhecimento, citamos a seguir os comentrios de Gabriel Naud (16001653), que publicou a autobiografia de Cardano pela primeira vez em
1643:

RPM OBMEP

No somente era ele inquestionavelmente um mdico notvel,


como foi tambm provavelmente o primeiro e nico homem a se
distinguir em todas as cincias ao mesmo tempo. uma das
ilustraes da Natureza daquilo que um homem capaz de atingir.
Nada de significativo lhe era desconhecido em filosofia, medicina,
astronomia, matemtica, histria, metafsica ou as cincias
sociais, ou em outras reas mais remotas do conhecimento. Ele
tambm errava, claro, isto apenas humano; maravilhoso,
porm, quo raramente ele errava.

136

Na poca da descoberta de Tartaglia, Cardano gozava de boa posio


em Milo e o convidou a sua casa. Uma vez l, com muita insistncia
Cardano conseguiu que lhe fosse revelado o segredo da resoluo das
equaes do terceiro grau.
Tartaglia consentiu em lhe ensinar a regra de resoluo (embora no
lhe ensinasse a demonstrao da mesma), sob forma de versos, em troca

do juramento solene de que Cardano jamais publicaria esse segredo.


Conhecendo um mtodo de resoluo, Cardano procurou e achou
uma demonstrao que o justificasse. De posse da soluo, Cardano
deve ter se sentido fortemente tentado a public-las. Em 1544 fez uma
viagem a Florena e, no caminho, visitou Annibale delia Nave, em
Bologna, que lhe mostrou um manuscrito de del Ferro que continha a
famosa regra de Tartaglia, manuscrito este que ainda se conserva.
Aparentemente, ao saber que a frmula de Tartaglia existia j desde
trinta anos antes, Cardano se sentiu desobrigado de cumprir seu juramento
e publicou, em 1545, em Nuremberg, uma obra intitulada Ars Magna,
que o tornou verdadeiramente famoso em todo o continente. Nas palavras
de C. Boyer, ele provavelmente era o matemtico mais competente da
Europa. Nessa obra aparecem, pela primeira vez, as regras de resoluo
das equaes do terceiro e quarto graus. A seu favor, podemos dizer que
Cardano no esquece de fazer as devidas atribuies de mrito aos
respectivos descobridores.
A seguir, faremos uma anlise do mtodo que Tartaglia confiou a
Cardano.
Os Versos de Tartaglia
Como dissemos acima, Tartaglia comunicou a Cardano o segredo da
sua descoberta por meio de versos. Tal idia no to estranha quanto
pode parecer a princpio; devemos lembrar que, na poca, os autores
no dispunham ainda de uma notao adequada para tratar as equaes
em sua generalidade e no podiam, portanto, expressar seus mtodos
resumidamente mediante frmulas, como fazemos hoje em dia.

1. Quando chel cubo con le cose appreso


Se aggaglia a qualque nmero discreto
Trovati due altri differenti in esso
2. Depoi terrai questo por consueto
Chel lor produtto sempre sia eguale
Al terzo cubo delle cose neto

RPM OBMEP

A seguir, reproduzimos os versos na sua verso original, tal como


transcritos na edio de 1554 dos Quesiti et inventione diverse de
Tartaglia.

137

3. El resduo poi suo generale


Delli lor lati cubi ben sostratti
Verra la tua cosa principale
4. In el secondo de coiesti aiti
Quando chel cubo restasse lui solo
Tu osserverai questaltri contratti
5. Del nmero farai due, tal parta volo
Cha luno e laltro si produca schietto
El terzo delle cose in stelo
6. Delle qual poi, per commun precetto
Torrai li lati cubi incieme gionti
Et cotal somma sera il tuo concetto
7. El terzo poi de questi nostri conti
Se solve con secondo, se ben guardi
Che ser natura son quasi congiontri
8. Questi trovai, et non con passi tardi
nel mille cinquecento quatro et trinta
Nella citt dal mare intorno centa.
Uma traduo para o portugus ficaria, mais ou menos, assim:
1. Quando o cubo com a coisa em apreo
Se igualam a qualquer nmero discreto
Acha dois outros diferentes nisso

RPM OBMEP

2. Depois ters isto por consenso


Que seu produto seja sempre igual
Ao cubo do tero da coisa certo

138

3. Depois, o resduo geral


Das razes cbicas subtradas
Ser tua coisa principal
4. Na segunda destas operaes,
Quando o cubo estiver sozinho
Observars estas outras redues

5. Do nmero fars dois, de tal forma


Que um e outro produzam exatamente
O cubo da tera parte da coisa
6. Depois, por um preceito comum
Toma o lado dos cubos juntos
E tal soma ser teu conceito
7. Depois, a terceira destas nossas contas
Se resolve como a segunda, se observas bem
Que suas naturezas so quase idnticas
8. Isto eu achei, e no com passo tardo
No mil quinhentos e trinta e quatro
Com fundamentos bem firmes e rigorosos
Na cidade cingida pelo mar
Analisaremos, a seguir, esses versos numa linguagem acessvel ao
leitor contemporneo. Antes de tudo, conveniente lembrar que Tartaglia
(assim como depois faria tambm Cardano) no utiliza coeficientes
negativos em suas equaes. Ento, em vez de uma equao geral do
terceiro grau, ele deve considerar trs casos possveis:
x3 + ax = b
x3 = ax + b
x3 + b = ax .
Tartaglia chama cada um desses casos de operaes e afirma que ir
considerar, de incio, equaes do primeiro tipo: cubo e coisa igual a
nmero. No quarto verso comea a considerar o segundo tipo quando
o cubo estiver sozinho e, no stimo, faz referncia ao terceiro caso.

O nmero se refere ao termo independente, que ns denotamos aqui


por b. Quando diz acha dois outros diferentes nisso, est sugerindo
tomar duas novas variveis cuja diferena seja precisamente b, i.e.,
escolher U e V tais que:
U V = b.

RPM OBMEP

Vejamos agora como se prope a resolver o primeiro caso, nos trs


versos iniciais, para depois justificar seu mtodo, de uma forma simples.

139

A frase ... que seu produto seja sempre igual a cubo da tera parte
da coisa significa que U e V devem verificar:
a
UV = ( )3 .
3
Finalmente, o resduo geral das razes cbicas subtradas ser tua
coisa principal significa que a soluo estar dada por

x = 3U 3V .
Os outros dois casos carecem de interesse para o leitor moderno,
uma vez que podemos reduzi-los ao primeiro, mudando termos de um
membro a outro da equao.
A frase final ... a cidade cingida pelo mar uma referncia a Veneza,
onde realizou suas descobertas.
A Resoluo da Equao do Terceiro Grau
Nesta seo veremos como justificar a frmula de Tartaglia para
resolver equaes do terceiro grau. Naturalmente, utilizaremos mtodos
e notaes modernos, o que nos permitir dar uma exposio
relativamente simples.
Vamos considerar uma equao do terceiro grau escrita na forma:
x3 + ax = b.
para compar-la com a primeira destas operaes . . . cubo e coisa igual
a nmero, discutida nos trs primeiros versos de Tartaglia. Na verdade,
h um caminho muito simples para ach-la. Comecemos por lembrar a
frmula do cubo de um binmio:
(u v)3 = u3 3u2v + 3uv2 v3.
Pondo em evidncia o produto uv, temos:
(u v)3 = 3uv(u v) + (u3 v3),
isto ,
RPM OBMEP

(u v)3 + 3uv(u v) = u3 v3 .

140

Se podemos escolher, de alguma forma, u e v de modo que verifiquem:


uv = a/3
u v3 = b,
3

a relao acima se transformar em:


(u v)3 + a(u v) = b
o que significa que x = u v ser uma soluo da equao dada.
Em outras palavras, se conseguirmos achar u e v que sejam solues
do sistema acima, tomando x = u v obter-se- uma soluo da equao
proposta. Resta-nos ento o problema de resolver o sistema. Para isso,
observemos que, elevando ao cubo a primeira equao, ela se transforma
em:
u3v3 = (a/3)3
u3 v3 = b.
Finalmente, fazendo u3 = U e v3 = V, temos:
UV = (a/3)3
U V = b.
Isso muito fcil de resolver; U e V so as razes da equao:
X2 bX + (a/3)3 = 0
que so dadas por:

X=

b b 2 4(
2

a 3
)
b
b
a
3
= ( ) 2 + ( )3 .
2
2
3

Podemos tomar uma dessas razes como sendo U e a outra como


V, logo temos u = 3 U e v = 3 V . Portanto, obtemos precisamente a
soluo enunciada por Tartaglia:

x = 3U 3V .

x=3

b
b
a
b
b
a
+ ( ) 2 + ( )3 + 3 ( ) 2 + ( )3 .
2
2
3
2
2
3

Uma observao final: a equao geral do terceiro grau, que podemos


escrever na forma:

RPM OBMEP

Mais explicitamente, substituindo U e V pelos seus respectivos


valores, resulta a conhecida frmula que, nos textos, chamada de
frmula de Cardano ou de Tartaglia:

141

x3 + a1x2 + a2x + a3 = 0,
pode-se reduzir ao caso acima, mediante a mudana de varivel
x = y (a1/3). Alis, essa reduo era conhecida por Tartaglia, mas no
por Fior, e foi justamente esse fato que determinou a vitria do primeiro.
Isso significa que, na verdade, Tartaglia conhecia um mtodo geral para
resolver qualquer equao do terceiro grau.
A emergncia dos nmeros complexos
Os nmeros complexos desempenham um papel sumamente
importante nos mais diversos ramos da Matemtica e, atravs destes,
em muitas das aplicaes a outras reas do conhecimento.
Em geral, o estudante se depara com eles, pela primeira vez, ainda
no curso secundrio e sua introduo justificada pela necessidade de
resolver equaes de segundo grau com discriminante negativo. Isso
cria uma falsa impresso, j que, historicamente, no foram as equaes
de segundo grau que levaram introduo dos nmeros complexos.
Neste texto analisaremos essa questo e alguns outros aspectos ligados
ao desenvolvimento do assunto.
O fato de que um nmero negativo no tem raiz quadrada parece ter
sido sempre claro para os matemticos que se depararam com a questo.

RPM OBMEP

As equaes de segundo grau apareceram na Matemtica j nas


tabuletas de argila da Sumria, aproximadamente 1700 anos antes de
Cristo e, ocasionalmente, levaram a radicais de nmeros negativos;
porm, no foram elas, em momento algum, que sugeriram o uso de
nmeros complexos.

142

Em rigor, uma equao era vista como a formulao matemtica de


um problema concreto; assim, se no processo de resoluo aparecia uma
raiz quadrada de um nmero negativo, isso era interpretado apenas como
uma indicao de que o problema originalmente proposto no tinha
soluo. Como veremos adiante, foram s as equaes de terceiro grau
que impuseram a necessidade de trabalhar com esses nmeros.
Vejamos inicialmente alguns antecedentes. Na Arithmetica, de
Diophanto, aproximadamente no ano de 275 d.C. ele considera o seguinte
problema:

Um tringulo retngulo tem rea igual a 7 e seu permetro de 12


unidades. Encontre o comprimento dos seus lados.
Chamando de x e y o comprimento dos catetos desse tringulo,
temos, na nossa notao atual:

1
xy = 7; x 2 + y 2 = (12 x y ) 2 .
2
Substituindo y em funo de x, obtemos a equao
24x2 172x + 336 = 0.
Nesse ponto Diophanto observa que s poderia haver soluo se
172 2
(
) 24 336 . Nesse contexto, claro que no h necessidade
2

alguma de introduzir um sentido para a expresso


o discriminante da equao.

167 , sendo 167

Na verdade, o primeiro registro de um radical de um nmero negativo


um pouco anterior: ele aparece na Estereometria de Heron, matemtico
grego do perodo Alexandrino, publicada aproximadamente em 75 d.C.
Num clculo sobre o desenho de uma pirmide surge a necessidade de

81 144 . A questo parece no causar nenhum problema


avaliar
simplesmente porque logo em seguida os nmeros apresentam-se
trocados:

144 81 , resultando

damente igual a 7

63 , que calculado como aproxima-

15
.
16

Encontram-se novas referncias questo na Matemtica indiana.


Aproximadamente no ano de 850 d.C, o matemtico indiano Mahavira
afirma:

J no sculo XII, o famoso matemtico Bhaskara (1114-1185 aprox.)


escreve:
O quadrado de um afirmativo afirmativo; e a raiz quadrada de um
afirmativo dupla: positiva e negativa. No h raiz quadrada de um
negativo; pois ele no um quadrado.

RPM OBMEP

... como na natureza das coisas um negativo no um quadrado, ele


no tem, portanto, raiz quadrada.

143

Tambm na Matemtica europia aparecem observaes dessa


natureza; Luca Paccioli, na sua Summa de arithmetica, geomtrica,
proportioni et proportionalita, publicada em 1494, escreve que a equao

1 2
b c , e o matemtico francs
4
Nicolas Chuquet (1445-1500 aproximadamente) faz observaes
semelhantes sobre solues impossveis num manuscrito, no
publicado, de 1484.

x2 + c = bx solvel somente se

O prprio Cardano se deparou com esse tipo de questes e, embora


mantivesse a atitude dos seus contemporneos, no sentido de entender
que razes de nmeros negativos indicavam apenas a no-existncia de
solues de um determinado problema, pelo menos em um caso ele deu
um passo a mais. No Captulo 37 do Ars Magna, ele considera o problema
de dividir um segmento de comprimento 10 em duas partes cujo produto
seja 40.

Se chamamos de x o comprimento de uma das partes, a outra ter


comprimento 10 x, e a condio do problema se traduz na equao:
x(10 x) = 40.
Isso leva equao x2 l0x + 40 = 0, cujas solues so 5 15 .
Cardano reconhece que o problema dado no tem soluo mas, talvez a
ttulo de curiosidade, observa que, trabalhando com essas expresses
como se fossem nmeros, deixando de lado as torturas mentais
envolvidas e multiplicando
25 (15), que igual a 40.

5 + 15 por 5 15 , obtm-se

RPM OBMEP

Em consequncia, ele chama essas expresses de razes sofsticas da


equao e diz, a respeito delas, que so to sutis quanto inteis.

144

A necessidade dos nmeros complexos


Raphael Bombelli (1526-1573) era um admirador da Ars Magna de
Cardano, mas achava que seu estilo de exposio no era claro (ou, em
suas prprias palavras, ma nel dire f oscuro). Decidiu, ento, escrever

um livro expondo os mesmos assuntos, mas de forma tal que um


principiante pudesse estud-los sem necessidade de nenhuma outra
referncia. Publicou lAlgebra, em trs volumes, em 1572, em Veneza,
obra que viria a se tornar muito influente. No captulo II dessa obra, ele
estuda a resoluo de equaes de grau no superior a quatro. Em
particular na pgina 294 e nas seguintes, ele considera a equao
x3 = 15x+ 4. Ao aplicar a frmula de Cardano para o clculo de uma
raiz, ele obtm:
x = 3 2 + 121 + 3 2 121 .

Seguindo Cardano, ele tambm chama essa expresso de sofstica,


mas, por outro lado, ele percebe que x = 4 , de fato, uma raiz da
equao proposta.
Assim, pela primeira vez, nos deparamos com uma situao em que,
apesar de termos radicais de nmeros negativos, existe verdadeiramente
uma soluo da equao proposta. necessrio, ento, compreender o
que est acontecendo.
Bombelli concebe ento a possibilidade de que exista uma expresso
da forma a + b

que possa ser considerada como raiz cbica de

2 + 121 i.e., que verifique (a + b )3 = 2 + 121 . A forma em


que ele calcula essa raiz um tanto peculiar; ele assume que a raiz cbica
de 2 121 seja da forma a b . Como ele sabe que 4 deve ser
raiz da equao, necessariamente a + b + a b = 4 . Nesse ponto,
felizmente, as quantidades no existentes se cancelam e obtemos
a = 2. Com esse resultado, muito fcil voltar equao
(a + b )3 = 2 + 121 e deduzir que b = 1. Assim, ele obtm que

2 + 121 = 2 + 1 e que:
x = 2 + 1 + 2 1 = 4

uma soluo da equao dada.


Bombelli percebeu claramente a importncia desse achado. Ele diz:
Eu achei uma espcie de raiz cbica muito diferente das outras, que
aparece no captulo sobre o cubo igual a uma quantidade e um nmero.

RPM OBMEP

145

... A princpio, a coisa toda me pareceu mais baseada em sofismas que


na verdade, mas eu procurei at que achei uma prova... .
Isto pode parecer muito sofisticado mas, na realidade, eu tinha essa
opinio, e no pude achar a demonstrao por meio de linhas [i.e.
geometricamente], assim, tratarei da multiplicao dando as regras
para mais e menos.

Ele utiliza a expresso pi di meno para se referir ao que ns


denotaramos como +i e meno di meno para i. Ele enuncia ento o
que chama de regras do produto, que citamos abaixo junto com sua
traduo na nossa simbologia:
Pi via pi di meno fa pi di meno,

+.(+i) = +i

Meno via pi di meno fa meno di meno,

.(+i) = i

Pi via meno di meno fa meno di meno,

+.(i) = i

Meno via meno di meno fa pi di meno,

.(i) = +i

Pi di meno via pi di meno fa meno,

(+i).(+i) =

Meno di meno via pi di meno fa pi,

(i).(+i) = +

Meno di meno via meno di meno fa meno.

(i).(i) =

E interessante notar que Bombelli se deparava com a dificuldade


adicional de no dispor de uma boa notao. Ele utilizava p (plus) para
indicar a soma; m (minus) para a subtrao; R (radix) para raiz quadrada
e R3 para a raiz cbica. Tambm no dispunha de parnteses; nos seus
manuscritos sublinhava expresses para indicar quais os termos afetados
por um radical. Assim, por exemplo, a expresso

2 + 121

era

escrita na forma R3 | 2 pR| 0 121 | | .


Note que, como no escrevia diretamente nmeros negativos, ele
escreveu 121 como 0 121. Dessa forma, a soluo da equao

RPM OBMEP

discutida acima aparecia como: R3 | 2 pR| 0 121 | | pR3 | 2mR| 0 121 | | .

146

Progressos ulteriores
Faremos aqui um pequeno resumo da evoluo dos nmeros
complexos, para que o leitor tenha uma viso global da histria do

assunto. Comearemos listando alguns progressos na notao para depois


nos ocuparmos da evoluo dos conhecimentos.
O smbolo

1 foi introduzido em 1629 por Albert Girard.

O smbolo i foi usado pela primeira vez para representar 1 por


Leonhard Euler em 1777, apareceu impresso pela primeira vez em
1794 e se tornou amplamente aceito aps seu uso por Gauss em 1801.
Os termos real e imaginrio foram empregados pela primeira vez por
Ren Descartes em 1637.
O expresso nmero complexo foi introduzida por Carl Friederich
Gauss em 1832.
Como observamos na seo anterior, a partir do trabalho de Bombelli,
os nmeros complexos comearam a ser utilizados devido a sua bvia
utilidade para resolver equaes de terceiro grau mas, ao mesmo tempo,
era claro que tais nmeros no poderiam existir. A primeira tentativa de
legitimao, via uma interpretao geomtrica, devida a John Wallis
(1616-1703), contemporneo de Newton e professor na Universidade
de Oxford. Em 1673 ele publicou um tratado intitulado lgebra, em
cujo captulo LXVI discute a impossibilidade da existncia de
quantidades imaginrias e compara essa questo com a da existncia
de quantidades negativas.
Essas quantidades imaginrias (como so freqentemente chamadas)
surgem das supostas razes de um quadrado negativo (quando aparecem)
e se considera que implicam que o caso proposto impossvel.

Mas tambm impossvel que qualquer quantidade (embora no um


suposto quadrado) possa ser negativa. Pois no possvel que qualquer
magnitude possa ser menos que nada, ou qualquer nmero menor que
nada.
Porm, no esta suposio (das quantidades negativas) nem intil
nem absurda, quando corretamente compreendida. E, embora para a
simples notao algbrica representa uma quantidade menor do que

RPM OBMEP

E assim , de fato, no sentido estrito do que foi proposto. Pois no


possvel que qualquer nmero (negativo ou afirmativo), multiplicado
por si mesmo, possa produzir (por exemplo) 4. Pois sinais iguais
(tanto + quanto ) produziro +; e portanto no 4.

147

nada, quando se trata de uma aplicao fsica, denota uma quantidade


to real como se o sinal fosse +; mas interpretada no sentido contrrio.

Depois de considerar diversos exemplos de nmeros negativos


interpretados em termos de segmentos sobre uma reta orientada, ele tenta
uma interpretao para as quantidades imaginrias:
Suponhamos que num local ganhamos do mar 30 acres, mas perdemos
em outro local 20 acres: se agora formos perguntados quantos acres
ganhamos ao todo a resposta 10 acres, ou +10 (pois 30 20 = 10).
... Mas se num terceiro local perdemos mais 20 acres, a resposta deve
ser 10 (pois 30 20 20 = 10) ... . Mas agora, supondo que esta
plancie negativa de 1600 square perches [20 acres correspondem a
1600 square perches, uma outra medida inglesa da poca] tem a forma
de um quadrado, no devemos supor que este quadrado tem um lado?
E, assim, qual ser esse lado?
No podemos dizer que 40, nem 40 ... Mas sim que

1600 (a

suposta raiz de um quadrado negativo) ou 10 16 ou 20 4 ou


40 1 .

Como era de se esperar, essa interpretao no teve uma grande


acolhida entre seus contemporneos e nenhuma repercusso posterior.

RPM OBMEP

Notemos que, at aqui, nada garante que razes cbicas - ou, em


geral, razes n-simas de complexos sejam, de fato, complexos. Tal
como assinala M. Kline, no comeo do sculo XVIII, a maioria dos
matemticos ainda acreditava que razes de diferente ordem de nmeros
complexos levariam introduo de diferentes tipos de complexos.

148

Jean Le Rond dAlembert (1717-1783), aps estudar Direito e


Medicina, decidiu dedicar sua vida Matemtica. Trabalhou em lgebra,
clculo e suas aplicaes, equaes diferenciais ordinrias e parciais,
funes de varivel complexa, mecnica e dinmica. Em 1747 publicou
Reflxions sur Ia cause gnrale des vents, em que afirmou que toda
expresso construda algbricamente a partir de um nmero complexo
(onde inclua tambm a extrao de razes) da forma a + b 1 . No
formulou uma prova satisfatria no caso de expresses da forma
(a + bi)c+di, tarefa que seria completada por Euler.

DAlembert foi amigo de Voltaire e colaborou com diversos artigos


para a Enciclopdie, mas manteve nessa um discreto silncio sobre os
nmeros complexos.
Roger Cotes (1682-1716) foi um jovem professor no famoso Trinity
College de Cambridge e, aps sua prematura morte, dele disse Newton:
Se Cotes tivesse vivido, teramos aprendido alguma coisa. Em 1714 ele
obteve um importante resultado, relacionado com a obteno de razes
n-simas da unidade que, na notao moderna, poderamos explicitar
como:
loge(cos + isen) = i.
Isso poderia ter levado famosa relao de Euler:
cos + isen = ei.
que, por sua vez, implica a frmula de De Moivre:
(cos + isen)n = cos(n) + isen(n)
o que resolveria o problema de achar razes.

Essa tarefa coube a Leonhard Euler (1707-1783), considerado o


mais prolfico matemtico de todos os tempos. Numa carta endereada
a Jean Bernoulli, datada de 18 de outubro de 1740, ele afirma que
y = 2 cos e y = eix + eix eram ambas solues da mesma equao
diferencial (o que reconheceu atravs do desenvolvimento em srie das
solues) e que, portanto, deviam ser iguais. Publicou esse resultado em
1743; explicitamente:
cos =

ei + e i
ei e i
e sen =
.
2
2i

RPM OBMEP

Porm, o caminho foi outro. Abraham De Moivre (1667-1754)


nasceu na Frana, mas viveu na Inglaterra a partir dos dezoito anos.
Estudou Matemtica sozinho, aps ler os Principia de Newton, chegando
a se tornar membro da Royal Society e das academias de Paris e Berlim.
Seu trabalho versou fundamentalmente sobre trigonometria,
probabilidade e clculo de anuidades. Em 1722, utilizando fatos que j
havia publicado em 1707, ele obteve um resultado que implicou a frmula
que leva seu nome, embora tenha se limitado a casos particulares e nunca
tenha chegado a enunciar ou demonstrar a frmula no caso geral.

149

Em 1748 ele redescobriu o resultado de Cotes, demonstrou a frmula


de De Moivre e estendeu sua validade para todo exponente n real. Com
isso, a existncia de razes no campo complexo ficou definitivamente
estabelecida.
Obviamente, Euler compreendia e utilizava muito bem os nmeros
complexos. O fato de ele prprio ter grandes dvidas quanto a sua
legitimidade ilustra claramente o status desse corpo numrico na poca.
Em Vollstndige Anleitung zur Algebra, publicada primeiro em russo,
em 1768-69, e depois em alemo, em 1770, que se tornou uma referncia
clssica nessa rea nos dois sculos seguintes, Euler escreve:
Uma vez que todos os nmeros concebveis so maiores do que 0, ou
menores do que 0 ou iguais a 0, claro que a raiz quadrada de um
nmero negativo no pode ser includa entre os nmeros possveis.
Consequentemente, devemos dizer que esses so nmeros impossveis.
E essa circunstncia nos conduz a tais nmeros, que por sua natureza
so impossveis, e que so chamados costumeiramente de imaginrios,
pois eles s existem na imaginao.

A representao grfica
A representao geomtrica dos nmeros complexos mediante pontos
do plano foi decisiva para sua aceitao. A possibilidade dessa representao era clara para vrios autores, como Cotes, De Moivre, Euler e
Vandermonde; todos eles tentaram resolver a equao xn 1 = 0 pensando
em suas solues como vrtices de um polgono regular de n lados.
Essa ideia era ainda incompleta, pois nenhum desses autores achou
tambm uma interpretao geomtrica para as operaes com complexos.

RPM OBMEP

O primeiro a formular uma tal interpretao foi um agrimensor


noruegus chamado Caspar Wessel (1745-1818), um autodidata. Ele
autor de um artigo intitulado Sobre a representao analtica da direo:
uma tentativa, que foi publicado em 1799 nas memrias da Real
Academia da Dinamarca. Ali, escreveu:

150

Vamos designar por +1 a unidade retilnea positiva, por + outra


perpendicular primeira, com a mesma origem; ento o ngulo de
direo de +1 ser 0, o de 1 ser 180, o de ser 90 e o de
ser 90 ou 270.

Tal como fazemos hoje em dia, ele representa o complexo a + bi


pelo vetor do plano com origem O a origem do sistema de eixos coordenados e com extremo no ponto P de coordenadas (a, b). Depois d
uma representao geomtrica da soma de dois complexos a + bi e
c + di, representando-os pelos vetores OP e OQ, respectivamente, e
observando que a soma estar respresentada pela diagonal do
paralelogramo construdo sobre OP e OQ.
De forma anloga, o produto desses complexos estar representado
por um vetor OR tal que o comprimento de OR o produto dos
comprimentos de OP e OQ, e o ngulo que OR forma com o eixo Ox
igual soma dos ngulos formados por OP e OQ com esse eixo.
Uma representao semelhante foi dada por Jean-Robert Argand
(1768-1822), um bibliotecrio suo, tambm autodidata, que em 1806
publicou um pequeno livro intitulado Essai sur la manire de reprsenter
les quantits imaginaires dans les constructions gomtriques. Ele
observa que se multiplicamos +1 por i obtemos i e se multiplicamos
esse resultado novamente por i obtemos 1. Ele pensa, ento, em
representar i por uma operao que aja de modo anlogo. Assim, podemos
representar i por uma rotao de 90 em sentido anti-horrio.
A partir daqui, tal como Wessel, ele d interpretaes para nmeros
da forma a + bi e para as operaes com complexos, aplicando seus
resultados demonstrao de teoremas de lgebra, geometria e
trigonometria.

A julgar pelas suas demonstraes do teorema fundamental da lgebra,


ele j conhecia a interpretao grfica dos complexos em torno de 1815,
embora escrevesse, numa carta de 1825, que a verdadeira metafsica de
1 elusiva. Finalmente, em 1831, ele escreveu um artigo muito
explcito sobre a questo. Diz na introduo:

RPM OBMEP

Esses trabalhos tiveram pouco ou nenhum efeito sobre os matemticos


da poca; a memria de Wessel s foi notada quando publicada em
traduo francesa em 1897, e o livro de Argand, embora causasse uma
certa controvrsia, teve pouco impacto, talvez por ser a nica contribuio
de seu autor Matemtica. Quem verdadeiramente tornou a interpretao
geomtrica amplamente aceita foi Carl Friederich Gauss (1777-1855).

151

O autor tem considerado h vrios anos essa parte importante da


Matemtica sob um ponto de vista diferente, que permite conferir s
quantidades imaginrias, como as negativas, uma existncia objetiva.
O significado intuitivo dos nmeros complexos fica completamente
estabelecido e no se precisa mais para admitir estas quantidades no
domnio da aritmtica.

Ele observa tambm que se as unidades 1, 1, 1 no fossem


chamadas de positiva, negativa e imaginria, mas direta, inversa e lateral,
as pessoas no teriam tido a impresso de que h algo de misterioso
nesses nmeros.
A observao de Gauss a respeito da existncia objetiva dos nmeros
complexos ilustra a viso da Matemtica na poca. Parece que o fato de
esses nmeros poderem ser representados geometricamente lhes d essa
existncia. Em outras palavras, parece que, para os matemticos daquele
perodo, os entes geomtricos tinham um tipo de realidade que faltava
aos objetos da aritmtica.
Finalmente, a formalizao completa dos nmeros complexos como
pares ordenados de nmeros reais ser desenvolvida por William Rowan
Hamilton (1805-1865) em 1833, e ainda Agustin Cauchy (1789-1857)
daria outro tipo de formalizao em 1847.
Adaptado dos artigos
A soluo de Tartaglia para a equao do terceiro grau
Csar Polcino Milies, RPM 25.

RPM OBMEP

A emergncia dos nmeros complexos


Csar Polcino Milies, RPM 24.

152

Grandezas incomensurveis
e nmeros irracionais

Existem, em Matemtica, conceitos que parecem muito


simples a uma viso superficial, mas que, submetidos a
uma anlise mais cuidadosa, revelam aspectos verdadeiramente surpreendentes.

Uma questo com que lidavam os matemticos gregos


daquela poca era a de comparar grandezas da mesma
espcie, como dois segmentos de reta, duas reas ou dois
volumes. No caso de dois segmentos retilneos AB e
CD, dizer que a razo AB/CD o nmero racional
m/n, significava para eles (e ainda significa para ns)
que existia um terceiro segmento EF tal que AB fosse
m vezes EF e CD n vezes esse mesmo segmento EF.
Na figura 1 ilustramos essa situao com m = 8 e n = 5.
A
B
figura 1

E F

AB 8
=
CD 5

RPM OBMEP

Vamos tratar aqui da reta na sua representao numrica


em termos das abscissas de seus pontos para mostrar que
esses conceitos de reta e de nmero no tm uma simplicidade to inocente como parecem revelar a uma viso
menos profunda. Exploraremos alguns fatos notveis e
inesperados, que esto ligados primeira grande crise
do desenvolvimento da Matemtica, ocorrida no final do
5o sculo a.C.

153

No tempo de Pitgoras (580 500 a.C. aproximadamente) e mesmo


durante boa parte do 5o sculo a.C. pensava-se que os nmeros racionais
fossem suficientes para comparar segmentos de reta; isto , dados dois
segmentos AB e CD, seria sempre possvel encontrar um terceiro
segmento EF contido um nmero inteiro de vezes em AB e outro
nmero inteiro de vezes em CD, situao esta que descrevemos dizendo
que EF um submltiplo comum de AB e CD. Uma simples reflexo
revela que essa uma idia muito razovel. Afinal, se EF no serve,
podemos imaginar um segmento menor, outro menor ainda, e assim por
diante. Nossa intuio geomtrica parece dizer-nos que h de existir um
certo segmento EF, talvez muito pequeno, mas satisfazendo aos
propsitos desejados. Na figura 2 ilustramos uma situao com segmento
EF bem menor que o da figura 1. O leitor deve ir muito alm, imaginando
um segmento EF to pequeno que nem se possa mais desenhar, para se
convencer, pela sua intuio geomtrica, da possibilidade de sempre
encontrar um submltiplo comum de AB e CD.
B
A

DE F

AB 29
=
CD 26

figura 2

RPM OBMEP

Dois segmentos nessas condies so ditos comensurveis, justamente


por ser possvel medi-los ao mesmo tempo, com a mesma unidade EF.
Entretanto, no verdade que dois segmentos quaisquer sejam sempre
comensurveis. Em outras palavras, existem segmentos AB e CD sem
unidade comum EF, os chamados segmentos incomensurveis. Esse
um fato que contraria nossa intuio geomtrica, e por isso mesmo a
descoberta de grandezas incomensurveis na Antiguidade representou
um momento de crise no desenvolvimento da Matemtica.

154

Foram os prprios pitagricos que descobriram grandezas incomensurveis, provavelmente entre 450 e 400 a.C.; e, ao que tudo indica, isto
se fez atravs de um argumento geomtrico, como o que apresentaremos
a seguir, demonstrando que o lado e a diagonal de um quadrado so
segmentos incomensurveis.

C
E
F

D
B

figura 3

Na figura 3 representamos um quadrado com diagonal = AB e


lado = AC. Suponhamos que e sejam comensurveis. Ento
existir um terceiro segmento que seja submltiplo comum de e .
p com centro
Fazemos agora a seguinte construo: traamos o arco CD
em A e o segmento ED tangente a esse arco em D. Ento, nos
tringulos retngulos ACE e ADE, os catetos AC e AD so iguais e
a hipotenusa AE comum, logo so tambm iguais os catetos CE e
DE (= BD).

Portanto,
= AB = AD + BD = + BD
= BC = BE + EC = BE + BD
= + BD

(1)

= BE + BD.

(2)

Se um segmento submltiplo comum de e , conclumos, por


(1), que tambm submltiplo de BD. Daqui e de (2) segue-se que
tambm submltiplo de BE. Provamos assim que se houver um
segmento que seja submltiplo comum de = AB e = AC, ento
o mesmo segmento ser submltiplo comum de BE e BD, segmentos
esses que so a diagonal e o lado do quadrado BDEF. Ora, a mesma
construo geomtrica que nos permitiu passar do quadrado original ao
quadrado BDEF pode ser repetida com este ltimo para chegarmos a
um quadrado menor ainda; e assim por diante, indefinidamente; e esses
quadrados vo se tornando arbitrariamente pequenos, pois, como fcil
ver, as dimenses de cada quadrado diminuem em mais da metade quando
passamos de um deles a seu sucessor. Dessa maneira, provamos que o

RPM OBMEP

ou seja,

155

segmento dever ser submltiplo comum do lado e da diagonal de um


quadrado to pequeno quanto desejemos. Evidentemente, isso um
absurdo! Somos, pois, levados a rejeitar a suposio inicial de que o
lado AC e a diagonal AB do quadrado original sejam comensurveis.
Conclumos, pois, que o lado e a diagonal de qualquer quadrado so
grandezas incomensurveis C.Q.D.
A descoberta dos incomensurveis representou, no 5o sculo a.C.,
uma derrota para os pitagricos. De fato, para eles o nmero era a essncia
de tudo. Eles acreditavam na possibilidade de explicar todos os
fenmenos do mundo sensvel em termos dos nmeros e de suas relaes,
tanto na Geometria como na Msica, na Astronomia ou na Fsica, enfim,
o nmero seria a essncia ltima do ser e de todos os fenmenos. Mas
por nmero eles entendiam apenas o que chamamos hoje de nmeros
naturais, ou inteiros positivos: 1, 2, 3, 4, .... Nem as fraes eram
nmeros, j que elas apareciam como relaes entre grandezas da mesma
espcie. Agora que haviam sido descobertas grandezas incomensurveis,
estava claro que os nmeros (naturais) eram insuficientes at mesmo
para definir a razo entre duas grandezas, o que se constitua num srio
entrave Filosofia Pitagrica.
Ao mesmo tempo em que essas coisas aconteciam, outros argumentos
propostos pelos filsofos da poca dentre os quais os de Zeno so os
mais famosos tambm apontavam dificuldades na suposta harmonia
entre a Geometria e os nmeros. Tudo isso culminou numa crise no
desenvolvimento da Matemtica, crise essa que s foi definitivamente
superada com a criao da teoria dos nmeros reais (racionais e
irracionais) no sculo passado, devido, sobretudo aos trabalhos do
matemtico alemo Richard Dedekind (1831-1916).

RPM OBMEP

Uma consequncia da existncia de grandezas incomensurveis a


existncia de pontos na reta sem abscissas racionais.
A

156

U
figura 4

De fato, com referncia figura 4, basta tomar OP = AO, onde AO


a diagonal de um quadrado de lado unitrio OU. Como OP e OU
so incomensurveis, no possvel expressar a razo OP/OU como
um nmero racional.
Que nmero seria a abscissa de P? Pelo teorema de Pitgoras,
OA2 = OU2 + UA2.
Como AO = OP e UA = OU = 1, obtemos
OP2 = 2OU2 = 2
ou seja, OP = 2 .
essa a abscissa de P, tomando OU como unidade de comprimento.
interessante analisar essas questes do ponto de vista moderno dos
nmeros como abscissas dos pontos de uma reta. Para maior simplicidade,
vamos restringir-nos apenas a uma semirreta OU, tomando o segmento
OU como unidade de comprimento (figura abaixo).
O

P
x

figura 5

Ento, todo ponto P da semirreta, que no seja a origem O, tem


abscissa positiva x, que a razo OP/OU.

n ,
10
com n variando de 1 a 9. Isso porque dividimos o intervalo AB em 10
subintervalos de comprimento 1/10 cada um (figura 6). Mas podemos
7+

RPM OBMEP

Evidentemente, se todos os pares de segmentos OU e OP fossem


comensurveis, bastariam os nmeros racionais no-negativos para
caracterizar os pontos da semirreta, isto , os nmeros da forma m/n,
com m e n inteiros, m > 0 e n > 0. E bom observar que isso condiz
muito bem com nossa intuio geomtrica: afinal, esses nmeros ficam
densamente distribudos ao longo da semirreta, de tal forma que entre
dois deles h sempre uma infinidade de nmeros do mesmo tipo. Assim,
entre os pontos A e B de abscissas 7 e 8 existem 9 nmeros do tipo

157

dividir esse intervalo em 100 subintervalos, cada um de comprimento


0,01; ou 1000 subintervalos, cada um de comprimento 0,001;
A

B
7,5 7,630589

7 7,09

figura 6

e assim por diante. Se, digamos, adotarmos a diviso em 1.000.000


subintervalos iguais, encontraremos entre A e B, 999.999 pontos com
abscissas racionais do tipo
7+

n
,
1.000.000

com n variando de 1 at 999 999. Na figura 6 ilustramos um desses


pontos, aquele que tem abscissa 7,630598.

RPM OBMEP

Pois bem, vamos confiar ainda que provisoriamente na suposio


de que todos os pontos da semirreta tenham abscissas racionais e ver
onde isso nos leva. Uma primeira consequncia que os pontos da semireta formam um conjunto enumervel, pois o conjunto dos nmeros
racionais enumervel (ver textos de Anlise Real). Se r1, r2, r3, ...
uma enumerao dos racionais, faremos uma cobertura da semirreta por
meio de segmentos, da seguinte maneira: cobrimos o ponto r1 com um
segmento de comprimento c/2, centrado em r1; cobrimos r2 com um
segmento de comprimento c/22, centrado em r2; fazemos o mesmo
com r3, utilizando agora um segmento de comprimento c/23; com r4
utilizamos um segmento de comprimento c/24; e assim por diante.
Dessa maneira a semirreta ficar toda coberta com uma famlia infinita
de segmentos no necessariamente disjuntos.

158

Vamos agora somar os comprimentos dos segmentos dessa famlia.


Por simplicidade e para enfatizar a visualizao geomtrica colocamos
os segmentos em fila, um em seguida ao outro e na ordem em que
aparecem, como ilustra a figura 7. Isso o bastante para nos convencer
de que a soma de todos os seus comprimentos exatamente igual a c,
pois comeamos com um segmento de comprimento c/2, adicionamos
sua metade, depois a metade deste ltimo e assim por diante.

c
c/2

c/22

c/23

figura 7

O que acabamos de demonstrar uma impossibilidade! Certamente


no possvel cobrir a semirreta com um a famlia de segmentos cuja
soma total dos comprimentos seja um nmero finito c! (E o nmero c
arbitrrio!) Afinal, a semirreta tem comprimento infinito! Para sairmos
dessa contradio temos de voltar atrs em nossa hiptese inicial de que
os pontos da reta numrica tm todos eles abscissas racionais. Em outras
palavras, os nmeros racionais so insuficientes para marcar todos os
pontos de uma reta; ou ainda, em termos mais inteligveis aos gregos da
Antiguidade, existem segmentos AB e CD para os quais impossvel
encontrar um segmento EF que seja submltiplo comum de AB e CD.
Como se v, acabamos de estabelecer a existncia de segmentos
incomensurveis com um raciocnio tpico da Anlise moderna! Ele
certamente causaria, na Antiguidade, tanta controvrsia quanto causaram
os famosos argumentos de Zeno. Talvez mais ainda, pois os argumentos
de Zeno foram rebatidos por Aristteles que, atravs de seus escritos,
f-los chegar at ns. Mas como rebater o argumento que demos acima
sobre a cobertura dos pontos de abscissas racionais? Seria necessrio
admitir a existncia de uma infinidade muito maior (uma infinidade no
enumervel) de pontos sem abscissas racionais! claro que isso seria
totalmente inaceitvel para quem j tinha srias objees ao infinito
enumervel. Mesmo hoje muito surpreendente que se possa cobrir
todos os pontos de abscissas racionais numa reta com uma famlia de
segmentos cuja soma total dos comprimentos seja to pequena quanto
desejemos!
nmeros irracionais (desde que esses nmeros sejam criados!) e 2
um deles, como decorre do argumento que demos antes referente
figura 4. No entanto, para completar este artigo, vamos reproduzir aqui
a demonstrao desse fato com um argumento puramente numrico e
bem conhecido.

RPM OBMEP

Esses pontos da reta sem abscissas racionais tm por abscissas

159

Comeamos supondo que existisse uma frao irredutvel m/n tal


que

2 = m / n . Ento

2=

m 2 m2 = 2n2.
n2

Daqui segue-se que m2 um nmero par, portanto o mesmo verdade


para m, isto , m = 2r, sendo r outro nmero inteiro. Substituindo
m = 2r em m2 = 2n2 obtemos
4r2 = 2n2

n2 = 2r2.

Mas essa ltima relao nos diz que n2 nmero par, logo n tambm
par. Chegamos a um absurdo, pois m/n frao irredutvel, no sendo
possvel que m e n sejam ambos pares. Somos, assim, forados a
rejeitar a suposio inicial de que

2 seja um nmero racional m/n.

A demonstrao que acabamos de dar est baseada num argumento


que, segundo Aristteles, teria sido usado na descoberta de grandezas
incomensurveis. um argumento que encerra um alto grau de abstrao,
razo pela qual muitos historiadores da Cincia acreditam que a
descoberta dos incomensurveis tenha ocorrido com um raciocnio mais
concreto, como o argumento geomtrico da figura 3.
Demonstraes como as que apresentamos, da incomensurabilidade

RPM OBMEP

do lado e da diagonal do quadrado, ou da irracionalidade de 2 , foram


as primeiras demonstraes por reduo ao absurdo que se fizeram na
Antiguidade. notvel que por volta de 400 a.C. a Matemtica j tivesse
alcanado to avanado grau de sofisticao. O mesmo no aconteceu
com outras cincias, como a Fsica, que somente no sculo XVII, com
os trabalhos de vrios cientistas, notadamente Galileu e Newton,
alcanaria desenvolvimento comparvel ao da Matemtica de dois
milnios antes.

160

Finalmente, um ltimo comentrio sobre a crise desencadeada com a


descoberta dos incomensurveis. De imediato isso tornou impossvel
falar em razo entre duas grandezas quando essas fossem incomensurveis.

Havia a necessidade de se inventarem os nmeros irracionais, o que


s ocorreu nos tempos modernos. Mas os gregos souberam contornar
esse problema, logo na primeira metade do 4o sculo a.C., e com muita
genialidade! Foi Eudoxo (408? 355? a.C.), da escola de Plato, quem
desenvolveu, de maneira brilhante, uma teoria das propores, com a
qual foi possvel superar a dificuldade dos incomensurveis, usando
apenas os nmeros inteiros positivos. Mas isso uma outra histria...
Adaptado do artigo

RPM OBMEP

Grandezas incomensurveis e nmeros irracionais


Geraldo vila, RPM 05.

161

A outra face da moeda honesta

No outro dia, brincava com a minha filha Ana Letcia o


seguinte jogo: cada uma tinha um peo avanando por
uma linha de um tabuleiro de xadrez. A regra para avanar
baseava-se no lanamento de uma moeda honesta: se
sasse cara, o peo avanava uma casa, se fosse coroa,
avanava duas casas. Quem chegasse primeiro ao fim (ou
passasse) da linha, ganhava.
O jogo muito rpido, de modo que depois de algumas
revanches, a Ana me disse: u, no ltimo lanamento s
sai coroa?.

RPM OBMEP

De fato, comecei a perceber que, no ltimo lanamento,


o nmero de coroas era muito maior que o de caras. No
entanto, ao observar todos os lanamentos, o nmero de
caras e o de coroas eram praticamente iguais, o que indica
a honestidade da moeda.

162

A Ana com seus sete anos foi brincar de outra coisa e eu


fiquei pensando no assunto: o lanamento final poder
ser cara somente se o peo estiver na penltima casa, j
que com cara o peo avana apenas uma casa. Enquanto
que poder ser coroa, tanto se o peo estiver na penltima
quanto na antepenltima casa.
Portanto, a face coroa tem realmente mais chance de
aparecer no ltimo lanamento que a face cara!!!

Mais formalmente, denotemos por N o nmero de lanamentos


necessrios para chegar oitava casa (ou passar), por SN1 a posio do
peo antes do lanamento final, e por XN a face nele observada. Se
SN1 = 7, ento o lanamento seguinte pode ser cara ou coroa, com
mesma chance. Por outro lado, SN1 = 6 implica que o lanamento
seguinte deva ser coroa, seno este no seria o ltimo.
Assim, pela lei da probabilidade total,
1
2
1
= coroa ) = P ( S N 1 = 7) + P ( S N 1 = 6) 1
2
> P ( X N = cara ).

P ( X N = cara ) = P ( S N 1 = 7)
P( X N

(1)

A ltima desigualdade encerra o assunto, mas eu ainda queria


conhecer a probabilidade de obter coroa no ltimo lanamento.
O evento [SN1 = 6] ocorre se e somente se forem observadas as
seguintes seqncias nos lanamentos:
{(2, 2, 2, 2), (1, 1, 2, 2, 2), (1, 1, 1, 1, 2, 2), (1, 1, 1, 1, 1, 1, 2)}
ou suas possveis permutaes (lembrando que o ltimo lanamento deve
ser coroa). Portanto,
P ( S N 1 = 6) =

4 1 5 1
1
+
5 + 6 + 7 ,
4
2
1
2
2 2 2
1

que aproximadamente 1/3. Como a penltima posio s pode ser 6


ou 7, ento P(SN1 = 7) aproximadamente 2/3.
Substituindo esse resultado em (1), temos que a probabilidade de
obter coroa no lanamento final aproximadamente 2/3.

Comentrios
O problema abordado neste texto uma verso discreta do conhecido
Paradoxo do tempo esperado, definido originalmente no contexto de
processos de renovao a tempo contnuo.

RPM OBMEP

A primeira concluso disso tudo que minha filha algo exagerada


em seus comentrios. A segunda que moeda honesta quando chega ao
fim da linha...

163

A novidade nesta apresentao que inspira a construo fsica de


um mecanismo que permite visualizar esse resultado: por exemplo,
mediante uma trilha na qual se avana de acordo com o lanamento de
um dado equilibrado, ou a prpria corrida de oito passos no tabuleiro de
xadrez.
A reao usual das pessoas de espanto ao constatar o desequilbrio
da distribuio obtida, ou de dvida a respeito da honestidade da moeda.
Mesmo sendo contra-intuitivo, o resultado natural se levarmos em
considerao que h outra varivel aleatria envolvida no processo, que
o nmero de lanamentos necessrios para se alcanar o fim da
linha, N.
Se bem verdade que o resultado de qualquer lanamento tpico,
XN, uniforme no conjunto {cara, coroa} e independente dos demais
lanamentos, o mesmo no ocorre com o lanamento final, XN. A varivel
aleatria XN no independente dos lanamentos anteriores, depende
da penltima posio.
Uma outra forma de entender o resultado que mais fcil cobrir a
ltima casa com um passo grande (de duas casas) que com um passo
pequeno (de uma casa). Assim, a diferena entre a frequncia de caras e
coroas fica ainda mais evidente se avanarmos uma casa ao obter cara e
trs ao obter coroa, por exemplo. No caso extremo de avanarmos uma
casa com cara e oito casas com coroa, a chance de obter cara no ltimo
lanamento 1/256, menos que 4 em 1000!!!
Adaptado do artigo

RPM OBMEP

A outra face da moeda honesta


Laura L. R. Rifo, RPM 64.

164

Nmeros de regies:
um problema de contagem

Muitos problemas em Matemtica envolvem processos


adequados de contagem que, frequentemente,
conduzem a frmulas gerais extremamente teis; por
exemplo, para contar de quantas maneiras podemos
combinar n objetos em grupos de r desses objetos, usamos
a conhecida frmula que d o nmero de combinaes
de n objetos tomados r a r, a saber:

n
n!
.
C ( n, r ) = =
r r !(n r )!
Vamos analisar um problema de contagem do nmero de
regies no plano que pode ser resolvido de maneira direta,
simples e interessante. Trata-se do seguinte:

Inicialmente, tentamos resolver o problema com um


nmero menor de pontos. Examinando os casos 2, 3, 4
e 5 pontos, temos:

RPM OBMEP

Considere 100 pontos distribudos sobre uma


circunferncia, de tal modo que o segmento ligando dois
quaisquer desses pontos no passe pelo ponto de
interseco de outros dois segmentos. Calcular o nmero
R de regies obtidas no crculo quando todos os 100
pontos estiverem ligados.

165

R2 = 2

R3 = 4

R4 = 8

R5 = 16

figura 1

Observamos que:
2
3
4
5

pontos:
pontos:
pontos:
pontos:

21
22
23
24

regies;
regies;
regies;
regies.

Os resultados levam a acreditar que 6 pontos fornerceriam 25 = 32


regies, logo 100 pontos forneceriam 299 regies, e, por analogia
(incorreta, como veremos) n pontos determinariam 2n1 regies! Mas,
ao verificar diretamente o que acontece com 6 pontos, vemos que ficam
determinadas 31 regies, e no 32.
Logo, a generalizao pretendida no verdadeira.
1

A
B

RPM OBMEP

figura 2

166

Como determinar uma frmula que fornea o nmero de regies


obtidas com 100 (ou um outro nmero qualquer) de pontos?
Soluo 1
Os segmentos ligando dois a dois os 100 pontos sero chamados
diagonais; como para cada dois pontos temos uma diagonal, o nmero

100
delas C(100, 2) =
, e o nmero de pontos de interseco das
2
100
diagonais C(100, 4) =
, visto que cada 4 pontos determinam
4
duas diagonais, as quais tm um ponto em comum.

Vamos descrever um processo que nos permite obter o nmero de


regies pela eliminao sucessiva de diagonais.
Ao retirarmos uma das diagonais, o nmero de regies vai diminuir,
visto que duas regies que tm em comum um segmento da diagonal
retirada fundem-se em uma nica regio.
Por exemplo, na figura 2, a retirada da diagonal D12, que liga os
pontos 1 e 2, faz com que as regies A e B se transformem em uma
nica regio; a retirada da diagonal D35 transforma em quatro as oito
regies que tm partes dessa diagonal como arestas.

Notemos que, no processo de retirada sucessiva das diagonais,


considera-se o nmero de pontos de interseco de cada diagonal com
aquelas que ainda no foram retiradas; no final do processo, ao serem
retiradas, sucessivamente, todas as diagonais, tal nmero igual ao
nmero total de pontos de interseco de todas as diagonais, ou seja
100
C(100, 4) =
; ao mesmo tempo, o nmero de regies decresce at
4
reduzir-se a uma nica regio, quando todas as diagonais tiverem sido

RPM OBMEP

Podemos observar que, ao retirarmos uma diagonal, o nmero de


regies decresce conforme o nmero de pontos de interseco dessa
diagonal com aquelas que ainda no foram removidas, mais um. Com
efeito, esse o nmero de segmentos nos quais os referidos pontos de
interseco dividem a diagonal, e a remoo de cada um desses segmentos
transforma duas regies em uma. Assim, a remoo da diagonal D12,
que no tem ponto de interseco com as demais, produz um decrscimo
de apenas um no nmero total de regies; j a retirada da diagonal D35,
que tem 3 pontos de interseco com as demais diagonais, produz um
decrscimo de 4 regies.

167

eliminadas. Podemos ento concluir que o nmero de regies eliminadas


no processo de retirada sucessiva de todas as diagonais dado pelo
nmero total de pontos de interseco de todas as diagonais, ou seja
100
C(100, 4) =
, acrescido de tantas parcelas iguais a 1 quantas so
4
100
as diagonais, ento, C(100, 2) =
. Portanto, o nmero inicial de
2
regies, que igual ao nmero de regies eliminadas mais uma, a que
restou no final do processo, dado por

100 100 100


R100 =
+
+
= 3.926.176.
0 2 4
Observe que, para n pontos, temos a mesma expresso, apenas
trocando o 100 por n. E, para 6 pontos, a frmula obtida fornece
6 6 6
R6 = + + = 31, como havamos verificado!
0 2 4

Soluo 2 (No volume anterior (2007) do nmero especial da RPM para a


OBMEP esta soluo, com menos detalhes, aparece na pgina 93.)

RPM OBMEP

Em Geometria, uma das frmulas mais notveis a chamada frmula


de Euler, que estabelece uma relao entre o nmero de vrtices, arestas
e faces de um poliedro: V A + F = 2.

168

Mostraremos, em seguida, como a frmula que fornece o nmero de


regies determinadas por n pontos, distribudos em uma circunferncia,
pode ser obtida a partir da frmula de Euler, o que era de se esperar, pois
a demonstrao mais conhecida da frmula de Euler, devida a Cauchy,
comea removendo uma face do poliedro e deformando a parte restante
em uma regio plana que um polgono subdividido pelas arestas do
poliedro. Para poliedros planos, como o da figura 2, obtidos pela
interligao de n pontos na circunferncia, a frmula de Euler se reduz
a V A + F = 1.
(1)
Vamos calcular, separadamente, V, A e F em funo de n e
substitu-los na frmula (1) para obter Rn .

Clculo do nmero de vrtices


Para cada 4 vrtices na circunferncia existem dois, e apenas dois,
segmentos que se cruzam, e portanto determinam um vrtice chamado

n
interno, de modo que o nmero desses vrtices C (n, 4) = , ou
4
n
seja: V = n + .
4

(2)

Clculo do nmero de arestas


Cada vrtice externo contribui com (n 1) arestas, cada vrtice
interno com 4 arestas e como cada aresta contm 2 vrtices:

n
2 A = n(n 1) + 4 e, portanto,
4
A=

n
n n
n(n 1)
+ 2 = + 2 .
2
4 2
4

(3)

Clculo do nmero de regies


O nmero Rn obtido acrescentando-se a F o nmero n de
regies compreendidas entre o poliedro plano e a circunferncia, de
modo que
F = Rn n.
(4)
Basta agora substituir (2), (3) e (4) na frmula (1) para se obter o
valor de Rn , na mesma expresso da soluo 1.
Observamos que o argumento usado na soluo 1 serve tambm para
demonstarr a frmula de Euler.

Nmero de regies: um problema de contagem


Antnio C. Patrocnio, RPM 12.

RPM OBMEP

Adaptado do artigo

169

Intuio e Probabilidade

Em muitas situaes do cotidiano estamos interessados


no clculo de determinadas probabilidades. Ocorre que,
em certos casos, especialmente aqueles envolvendo o
conceito de probabilidade condicional, esses clculos
levam a concluses que podem contrariar a intuio.
Apresentamos, neste texto, um exemplo dessa situao.
Num pas, 10% da populao portadora de um vrus.
Um teste para detectar a presena do vrus d 90% de
acertos quando aplicado a portadores e 80% de acertos
quando aplicado a no portadores.
Qual o percentual de pessoas realmente portadoras do
vrus, dentre aquelas que o teste classificou como
portadoras?
Vejamos uma soluo que no cita teoremas de
Probabilidade ou Estatstica.

RPM OBMEP

Considere que o teste foi aplicado aos I habitantes do


pas. O nmero de testes que indicou a presena do vrus
foi:

170

0,9 0,1 I + 0,2 0,9 I = 0,09 I + 0,18 I = 0,27 I,


sendo que a primeira parcela representa os 90% que so
realmente portadores e a segunda parcela representa os
20% que no so portadores. Logo, do total, 0,27 I, so
portadoras 0,09 I.

Assim, so realmente portadoras do vrus 0,09I / 0,27 I = 1/3


33,3% das pessoas que o teste classificou como portadoras.
Esse nmero no mnimo curioso e mostra que uma pessoa que fez
o teste e foi classificada como portadora tem grande possibilidade de
ser um falso-positivo (normalmente, quando uma pessoa faz um teste
desse tipo e o resultado positivo, os mdicos recomendam um novo
teste).
Por exemplo, o nmero de testes que indicaram a ausncia do vrus
foi 0,73 I e, dentre esses, 0,72 I no so portadores, o que d
0,72 I / 0,73I 98,6% de no portadores dentre os classificados
como no portadores.
Algumas variaes nos dados tambm originam resultados
interessantes. Por exemplo:
Se 0,5% da populao portadora e o teste acerta em 98% dos
casos, ento somente 20% das pessoas que o teste classificou como
portadoras so realmente portadoras.
Adaptado do artigo
Intuio e Probabilidade
Raul F. W. Agostino, RPM 27.

Resposta do Desafio
Como cada um dos pontos marcados na figura pertence a duas
circunferncias, a soma total dos nmeros colocados nas quatro
circunferncias (contando cada circunferncia em separado) igual a
Para que a soma S dos nmeros pertencentes circunferncia externa
seja exatamente igual soma (S) dos nmeros pertencentes a cada uma
das circunferncias internas, deveramos ter 90 = 4S, o que impossvel,
uma vez que 90 no divisvel por 4.
O problema no tem soluo.

RPM OBMEP

2(1 + 2 + 3 + 4 + 5 + 6 + 7 + 8 + 9) = 90.

171

Problemas I: problemas interessantes


com nmeros primos

P1. Escreva o nmero 91 como soma de dois nmeros primos.


P2. Eu e meu irmo caula temos idades entre 10 e 20 anos e hoje nossas
idades so expressas ambas por nmeros primos, fato que se repetir
pela prxima vez daqui h 18 anos. Determine minha idade sabendo
que a idade de nosso irmo mais velho, que, hoje, tambm um nmero
primo, uma unidade maior do que a soma das nossas idades.
P3. Uma equao do 2o grau, cujos coeficientes so todos nmeros
primos, pode apresentar duas razes iguais?
P4. Os nmeros a, b e logba podem ser todos primos?
A resposta aos dois problemas acima no, e eles no devem apresentar
maiores dificuldades ao leitor.
P5. Quantos pontos da reta y = x + 51 so tais que as suas duas coordenadas so nmeros primos?
Observe-se que, trocando o nmero 51 por outro valor, o problema
pode tornar-se muito mais difcil. Para a reta y = x + 2 somos conduzidos
ao conceito de primos gmeos (diferem por 2 unidades). At hoje
um problema em aberto saber se existem ou no infinitos pares de
primos gmeos.

RPM OBMEP

Se tomssemos a reta y + x = 40 obteramos seis solues: (3, 37),


(37, 3), (11, 29), (29, 11), (17, 23) e (23, 17), todas no primeiro quadrante
e que podem ser obtidas por inspeo direta.

172

Neste instante natural lembrar que a famosa conjectura de Goldbach


todo nmero natural par pode ser escrito como soma de dois nmeros
primos ainda no foi provada e nem se encontrou um contra-exemplo.
P6. As medidas dos lados de um tringulo retngulo (numa mesma
unidade) podem ser nmeros primos?

P7. Para quantos pontos da circunferncia x2 + y2 = 361 as duas coordenadas so nmeros primos?
P8. Para quantos pontos da circunferncia x2 + y2 = 461 as duas coordenadas so nmeros inteiros?
Esse problema se assemelha ao anterior, embora seja mais difcil que
ele. Para resolv-lo sugerimos a leitura de um artigo de Gilberto Garbi,
Outro belo teorema de Fermat, publicado na RPM 38.
P9. Determine as medidas, em graus, dos ngulos internos de um
tringulo acutngulo, sabendo que elas so expressas por nmeros
primos.
A mesma pergunta sem a hiptese de ser acutngulo exige um pouco
mais de trabalho.
P10. Quantos divisores positivos possui o nmero 2 420?
Esse exerccio uma aplicao clssica do Teorema Fundamental da
Aritmtica e do Princpio Fundamental da Contagem.
P11. Verifique que todos os n 1 nmeros da sequncia n! + 2, n! + 3,
..., n! + n so nmeros compostos so nmeros compostos, isto , nenhum
deles um nmero primo.
P12. Quantos so os nmeros naturais, de 1 a 100, que podem ser escritos
como um produto de dois nmeros naturais distintos entre si e diferentes
de 1?
P13. Apresente algum nmero natural n para o qual o valor numrico
p(n) do polinmio p(n) = x2 + x + 41 no seja um nmero primo.

P15. H dois anos, ano em que finalmente conclu meu Doutorado em


Matemtica, nasceu meu segundo filho e ocorreu uma notvel
coincidncia: eu e meus dois filhos passamos a fazer aniversrio no
mesmo dia do ano. A partir da outras coincidncias aconteceram. No

RPM OBMEP

P14. Quantos polgonos regulares, com nmero par de lados, podem ter
todas as diagonais expressas (numa mesma unidade) por nmeros
primos?

173

ano passado nossas trs idades foram representadas por quadrados


perfeitos e hoje, dia em que estamos comemorando mais um aniversrio,
percebo que nossas idades so representadas por trs nmeros primos.
Supondo que vivamos cem anos cada um, pergunto: qual minha idade
hoje? Nos prximos anos, quantas vezes todas as nossas idades voltaro
a ser representadas por nmeros primos?
Adaptado do artigo

RPM OBMEP

Os primos esquecidos
Chico Nery e Cludio Possani, RPM.

174

Problemas II: problemas do PISA*


*Programme for International Student Assessment
Programa Internacional de Avaliao de Alunos 2000/2003
http://www.inep.gov.br/internacional/pisa/novo (julho de 2009)

Questes
Estantes
Para construir uma estante completa, um marceneiro precisa do seguinte
material:
4 pranchas grandes de madeira,
6 pranchas pequenas de madeira,
12 braadeiras pequenas,
2 braadeiras grandes e
14 parafusos.
O marceneiro possui em estoque 26 pranchas grandes de madeira, 33
pranchas pequenas de madeira, 200 braadeiras pequenas, 20 braadeiras
grandes e 510 parafusos. Quantas estantes completas o marceneiro poder
fazer?

Carpinteiro
Um carpinteiro tem 32 metros de tbua para cercar um canteiro em uma
horta. Ele est pensando em utilizar um dos seguintes modelos para o
canteiro.

RPM OBMEP

Bombons coloridos
A me de Roberto permite que ele pegue
um bombom de um saco. Ele no consegue
ver os bombons. O grfico ao lado mostra
o nmero de bombons de cada cor contidos
no saco.
Qual a probabilidade de Roberto pegar
um bombom vermelho?
A) 10% B) 20% C) 25%
D) 50%

175

Na tabela abaixo, faa um crculo em Sim ou No para cada modelo,


indicando se ele pode ou no ser feito com 32 metros de tbuas.
Modelo de canteiro
modelo A
modelo B
modelo C
modelo D

Usando esse modelo, o canteiro pode ser construdo


com 32 metros de tbua?
Sim/No
Sim/No
Sim/No
Sim/No

RPM OBMEP

Dados
O desenho da direita representa dois dados.
Os dados so cubos com faces numeradas de acordo com
a seguinte regra: em um mesmo dado, o nmero total de
pontos de duas faces opostas sempre sete.

176

Questo 1
direita, vemos trs dados empilhados um sobre
o outro. O dado 1 possui quatro pontos na face
superior.
Quantos pontos h, no total, nas cinco faces
horizontais que no podemos ver (face inferior
do dado 1, faces superiores e inferiores dos dados
2 e 3)?
Questo 2
Voc pode fazer um dado, cortando, dobrando e colando uma cartolina.
Isso pode ser feito de diversas maneiras. Na figura a seguir, h quatro

opes que podem ser utilizadas para fazer dados.


Quais dos formatos podem ser dobrados para formar um dado que obedea
regra na qual a soma das faces opostas 7? Para cada formato, faa um
crculo em Sim ou No na tabela abaixo.

Formato

Obedece regra na qual a soma dos lados opostos 7?

Sim/No

II

Sim/No

III
IV

Sim/No
Sim/No

Sequncia em escada
Roberto constri uma sequncia com o formato de uma escada, utilizando
quadrados. Ele segue as seguintes etapas:

Como voc pode ver, ele utiliza um quadrado na Etapa 1, trs quadrados
na Etapa 2 e seis na Etapa 3.
Quantos quadrados ele vai utilizar na Etapa 4?

Foi divulgado um documentrio sobre terremotos e a frequncia com


que eles ocorrem. Essa reportagem inclui uma discusso sobre a
previsibilidade dos terremotos.
Um gelogo declarou: Nos prximos vinte anos, a probabilidade de
que ocorra um terremoto em Zedpolis de dois sobre trs.
Qual das opes a seguir exprime melhor o significado da declarao do
gelogo?

RPM OBMEP

Terremoto

177

A) 2/3 20 = 13,3, portanto no perodo de 13 a 14 anos, a partir de hoje,


haver um terremoto em Zedpolis.
B) 2/3 maior que 1/2, portanto podemos ter certeza de que haver um
terremoto em Zedpolis nos prximos 20 anos.
C) A probabilidade de haver um terremoto em Zedpolis nos prximos
20 anos maior do que a probabilidade de no haver um terremoto.
D) No se pode afirmar o que acontecer porque ningum pode ter
certeza de quando ocorrer um terremoto.
Feira de Vero
Uma barraca de uma feira de
vero prope um jogo no qual se
utiliza primeiro uma roleta. Em
seguida, se a roleta parar em um
nmero par, o jogador poder
pegar uma bolinha de gude de
dentro de um saco. A roleta e as bolinhas de gude contidas no saco esto
representadas na ilustrao.
Os prmios so distribudos s pessoas que pegam uma bolinha de gude
preta. Qual a probabilidade de Sueli ganhar um prmio?
A) Impossvel.
B) Pouco provvel.
C) Cerca de 50% de probabilidade.
D) Muito provvel.
E) Certeza.
Bate-papo pela Internet

RPM OBMEP

Mark (de Sydney, na Austrlia) e Hans (de Berlim, na Alemanha)


comunicam-se com frequncia por meio de uma sala de bate-papo da
Internet. Para isso, eles precisam conectar-se Internet ao mesmo tempo.
Para determinar um horrio apropriado para bater papo, Mark consultou
uma tabela de fusos horrios do mundo e encontrou o seguinte:

178

Questo 1
Que horas so em Berlim quando so 19 horas em Sydney?
Questo 2
Mark e Hans no podem bater papo das 9h s 16h30 de seus horrios
locais respectivos, porque eles devem ir para a escola. Alm disso, no
podero bater papo entre 23h e 7h porque estaro dormindo.
Qual seria um bom horrio para Mark
e Hans baterem papo? Escreva os
horrios locais na tabela ao lado:

Local

Horrio

Sydney
Berlim

Assaltos
Um reprter de TV apresentou o grfico
ao lado e disse:
O grfico mostra que, de 1998 para
1999, houve um grande aumento no
nmero de assaltos.
Voc considera que a afirmao do
reprter uma interpretao razovel do
grfico? D uma explicao que justifique
a sua resposta.
Colnia de frias
Os Servios Comunitrios de Zedpolis esto organizando um acampamento para um perodo de cinco dias. Inscreveram-se 46 crianas (26
meninas e 20 meninos) e 8 adultos (4 homens e 4 mulheres) apresentaramse como voluntrios para acompanh-las e organizar o acampamento.

Sra. Marlia
Sra. Carolina
Sra. Graa
Srta. Ktia
Sr. Slvio
Sr. Nelson
Sr. William
Sr. Pedro

Tabela 2: Dormitrios
Nome
Nmero de camas
Vermelho
Azul
Verde
Roxo
Laranja
Amarelo
Branco

12
8
8
8
8
6
6

RPM OBMEP

Tabela 1: Adultos

179

Regra dos dormitrios


1. Os meninos e as meninas devem dormir em dormitrios separados.
2. Em cada dormitrio deve dormir, pelo menos, um adulto.
3. O(s) adulto(s) que ficar(em) nos dormitrios deve(m) ser do mesmo
sexo que as crianas.
Preencha a tabela abaixo, distribuindo as 46 crianas e os 8 adultos
nos dormitrios, de maneira que todas as regras sejam obedecidas.
Nome

Nmero de meninos

Nmero de meninas

Nome(s) do(s) adulto(s)

Vermelho
Azul
Verde
Roxo
Laranja
Amarelo
Branco

O leitor deve ter observado que, em linhas gerais, as questes so


diferentes das que normalmente so apresentadas aos nossos alunos em
sala de aula. Elas exigem pouco contedo, pouca memria, mas, nas
palavras dos idealizadores do PISA, examinam a capacidade dos alunos
de analisar, raciocinar e refletir ativamente sobre seus conhecimentos e
experincias, enfocando competncias que sero relevantes para suas
vidas futuras.

RPM OBMEP

No site indicado, alm das questes de 2003, esto algumas questes


de 2000, e tambm uma descrio dos mecanismos utilizados para a
seleo dos estudantes que participaro do teste, normas de avaliao,
algumas explicaes sobre o mau desempenho dos brasileiros e outros
temas correlatos.

180

PROBLEMAS III

A numerao entre parnteses a original dos exemplares da RPM


1. (184) Os nmeros reais a, b e c so tais que a + b + c = 3,
a2 + b2 + c2 = 13 e a3 + b3 + c3 = 27. Determine a4 + b4 + c4.
2. (205) Trs aranhas caminham pelos lados de um tringulo ABC e
movimentam-se de modo que em qualquer instante formam um
tringulo e o baricentro de todos os tringulos formados sempre o
mesmo ponto (fixo) P. Sabendo-se que uma das aranhas percorre
todo o tringulo ABC, mostrar que P tambm o baricentro do
tringulo ABC.
3. (207) Um rapaz esqueceu o ltimo algarismo do telefone da namorada
e resolveu tentar falar com ela escolhendo ao acaso o ltimo dgito.
Se ele est num telefone pblico e s tem duas fichas, qual a
probabilidade de que ele consiga conversar com a namorada?
4. (219) Dados x e y nmeros inteiros positivos, mostre que, se
x2 + y2 + xy divisvel por 10, ento divisvel por 100.
5. (220) Considere duas retas paralelas que
distam a entre si e um quadrado ABCD, de
lado a, situado no plano das paralelas numa
posio tal que os vrtices A e C estejam em
lados opostos do plano dividido pela faixa
das paralelas. Calcule a soma dos permetros
dos tringulos sombreados.
n2n1 + 1 = m2.
7. (265) Seja ABC um tringulo tal que = 60o. Seja H o seu
ortocentro e J AC tal que AJ = 2JC e JC = JH. Mostre que o
tringulo ABC equiltero.

RPM OBMEP

6. (236) Achar todos os nmeros m e n naturais que resolvam

181

8. (266) Seja p(x) o polinmio de grau 2007, com coeficientes reais,

2
2 2007
+ cos
)
.
2007
2007
Determine o resto da diviso de p(x) por x2 + 1.
p ( x) = ( x sen

9. (267) Numa folha quadrada de papel desenhe ou dobre um ngulo ,


marque a metade da folha e a metade da metade. Dobre a folha de
modo que A caia em um ponto A pertencente a r e B em um
ponto B pertencente a s (ver figura). Marque os pontos A, B e
C, o correspondente de C na dobra. Prove que AB, AA e AC
trisseccionam o ngulo .

10. (268) Seja f: N N uma funo tal que


f(f(x)) = x para todo x N.
a) Mostre que f bijetora.
b) Exiba uma funo f com a propriedade acima e tal que
f(x) x para todo x N.
11. (270) Dispe-se de 2007 moedas viciadas M1, M2, ..., M2007.
Sabe-se que, em um lanamento, a probabilidade de se obter cara
na moeda Mi, i = 1, 2, ... 2007,

1
. Se as 2007 moedas
(2i + 1)

so lanadas simultaneamente, qual a probabilidade de que o


nmero de caras obtidas seja mpar?
RPM OBMEP

182

12. (271) Na figura, AD = 2BD.


Determine .

D
B

45

15

13. (272) Encontre as razes reais da equao

x + 2 x 1 + x 2 x 1 = 2.
14. (274) Prove que:
a) num tringulo retngulo a medida da mediana relativa
hipotenusa igual metade da medida da hipotenusa.
b) todo tringulo pode ser decomposto em n tringulos issceles,
para todo n > 4.
15. (276) Determine todas as funes f: R R satisfazendo
2f(x) + f(1 x) = x2008 para todo x R.
16. (277) Qual a maior potncia de 2 que divide 32008 1?
(280) (Jogo de Kontsevich) Consideremos
o tabuleiro infinito, ilustrado na figura, que
ocupa o primeiro quadrante. Inicialmente,
h 6 peas nas casas sombreadas no canto
inferior esquerdo. Um movimento consiste
em escolher uma pea sem vizinhos nas
casas imediatamente acima e direita e
substituir essa pea por duas, colocandoas nas casas vizinhas vagas:

movimento

O objetivo do jogo , a partir da configurao inicial, realizar uma


sequncia de movimentos de modo a deixar as 6 casas sombreadas
sem peas.
a) Atribua o peso 2ij para a casa na posio (i, j), como
mostra a figura (a casa inicial, inferior esquerda, situa-se na
posio (0,0)). Mostre que a soma dos pesos das casas ocupadas
por peas, depois de se realizarem movimentos, constante,
isto , no se altera aps qualquer sequncia de movimentos.

RPM OBMEP

17.

183

1 8 1 16 1 32 1 64
14

1 8 1 16 1 32

12

14

1 8 1 16

12

14

18

...
...
...
...
...

b) Determine a soma de todos os pesos do tabuleiro.


c) Mostre que impossvel realizar o objetivo do jogo: no existe
nenhuma sequncia de movimentos que deixa as 6 casas
iniciais, sombreadas, livres de peas!
18. (281) Simplifique:

n3 1

2 n 2008 n

+1

23 1 33 1 43 1
20083 1
.
.
.

.
.
23 + 1 33 + 1 43 + 1
20083 + 1

19. (282) Prove que, se x e y so inteiros tais que


N = (x + 6y)(2x + 5y)(3x + 4y)
mltiplo de 7, ento N mltiplo de 343.
20. (284) Dado um ngulo qualquer AOB com AO = BO, dividimos
AB em 3 partes iguais: AC = CD = BD. Pergunta-se: existe algum

RPM OBMEP

AOB para o qual os ngulos AOC ,


congruentes?

184

COD e BOD

so

Solues dos Problemas I


P1.
Os alunos no devero ter dificuldade em perceber que, como a soma de
dois mpares par e como 2 o nico primo par, os nmeros so 2 e 89.
Alis, esse pode ser um bom momento para recordar com os alunos os
testes de primalidade, para verificar que 89, efetivamente, primo.
P2.
As duplas de primos entre 10 e 20 so:
11 e 13, 11 e 17, 11 e 19, 13 e 17, 13 e 19 e 17 e 19.
Como a soma dos nmeros adicionada de 1 deve resultar um primo,
descarto as duplas 11 e 13 e 13 e 19. Como daqui a 18 anos as idades
voltam a ser representadas por nmeros primos, descarto as duplas que
incluem o 17. Resta apenas uma possibilidade: minha idade 19 anos e
a do meu irmo 11 anos.
P3.
Para que a equao ax2 + bx + c = 0 (com a, b e c primos) admita duas
razes iguais, devemos ter b2 4ac = 0 ou b2 = 4ac, o que implica b2
par. Logo, b tambm par e, como primo, b = 2. De b2 = 4ac, com
b = 2, temos ac = 1, o que absurdo para a e c primos.
Portanto, nas condies impostas, a equao no pode admitir duas razes
iguais.
P4.
Seja x = logba, portanto bx = a. Se b e x so nmeros primos, ento
bx no primo; logo, a no primo.
Se x = 2 , temos y = x + 51 = 53, que primo. Se x for qualquer outro
primo, ser um nmero mpar, implicando y par maior que 2, logo,
no primo. Assim, existe um nico par, (2, 53), da reta de equao
y = x + 51 que tem ambas as coordenadas dadas por nmeros mpares.

RPM OBMEP

P5.

185

P6.
Soluo: A resposta no. Do teorema de Pitgoras temos a igualdade
a2 = b2 + c2. Sendo a, b e c primos, no podem ser todos mpares e,
como a > b e a > c, devemos ter b = 2 ou c = 2. Digamos c = 2.
Teremos ento:
a2 b2 = 4
(a + b)(a b) = 4
e analisando os possveis valores de a + b e a b, que so 1, 2 ou 4,
conclumos que a situao impossvel.
P7.
Se x e y satisfazem a equao x2 + y2 = 361, sendo 361 mpar,
devemos ter x par e y mpar ou x mpar e y par. Se x par e primo,
ento, x = 2; logo, y2 = 357 e y no , ento, um nmero inteiro. Do
mesmo modo verificamos ser impossvel ter y par e x mpar; logo,
nenhum ponto da circunferncia de equao x2 + y2 = 361 tem ambas
as coordenadas dadas por nmeros primos.
P8.
Observamos, inicialmente, que 461 = 100 + 361 = 102 + 192, logo os
seguintes oito pontos, de coordenadas inteiras, pertencem circunferncia
de equao x2 + y2 = 461: (10, 19), (10, 19), (10, 19), (10, 19),
(19, 10), (19, 10), (19, 10) e (19, 10).
Alm disso, sendo 461 um nmero primo que dividido por 4 deixa
resto 1, o resultado de Fermat, todo nmero primo que dividido por
4 deixa resto 1 pode ser escrito como soma dos quadrados de dois
nmeros inteiros, de modo nico, a menos da ordem, prova que esses
oito so os nicos pontos de coordenadas inteiras pertencentes
circunferncia.

RPM OBMEP

P9.

186

Se a + b + c = 180, com a, b e c primos, no possvel ter a, b e


c mpares; logo, pelo menos um deles, digamos o a, deve ser igual a 2,
o que implica b + c = 178. Podemos ter b = c = 89, que primo e, por
verificao direta, mostra-se que no h outra possibilidade, j que o
tringulo, sendo acutngulo, implica b < 90 e c < 90.

Sem a hiptese de o tringulo ser acutngulo, obtemos, por tentativa, as


possibilidades: 5 e 173, 11 e 167, 29 e 149, 47 e 131 e 71 e 107.
P10.
Decompondo 2420 em fatores primos encontramos: 2420 = 22 5 112.
Os divisores positivos de 2420 so todos do tipo 2 5 11 com
valendo 0, 1 ou 2, valendo 0 ou 1 e valendo 0, 1 ou 2.
Logo, pelo Princpio Fundamental da Contagem, a quantidade de
divisores positivos de 2420 : 3 2 3 = 18.
P11.
Observemos que: n! + 2 divisvel por 2, n! + 3 divisvel por 3, ...,
n! + n divisvel por n, e assim sendo, nenhum deles primo.
Para valores grandes de n essas sequncias de nmeros naturais
consecutivos so chamadas desertos de nmeros primos.
P12.
De 1 a 100 temos 100 nmeros. Para obtermos a resposta nossa
pergunta, subtramos de 100 o nmero de primos entre 1 e 100, que
25; o nmero de quadrados de nmeros primos, que 4, e o nmero 1.
A resposta 70.
P13.
Para x = 40,
x2 + x + 41 = 402 + 40 + 41 = 40(40 + 1) + 41 = 40.41 + 41 = 41(40 + 1)
= 412 que no primo. Tambm para x = 41, x2 + x + 41 = 412 + 41 + 41
= 41(41 + 1 + 1) = 41.43, que no primo.
Prova-se que para qualquer valor inteiro de x, 40 < x < 39, tem-se
x2 + x + 41 igual a um nmero primo (ver RPM 09, p. 33).

ou qualquer outro valor primo. O lado do quadrado medindo p


faz a diagonal medir p, para qualquer p primo.

2
2

RPM OBMEP

P14.
1. Como podemos construir um quadrado com qualquer tamanho,
podemos constru-lo com suas diagonais medindo: 2 ou 3 ou 5, ...,

187

2. J no caso do hexgono regular, se traarmos


duas diagonais, uma passando pelo centro e
a outra no, vejamos o que acontece.
F
Sendo ABCDEF um hexgono regular, no
tringulo ABD temos

D
C
o

60

BD
3
A
B
, implicando que as me= sen60D =
AD
2
didas das diagonais BD e AD no podem ser simultaneamente
expressas por nmeros inteiros, logo no podem ser ambas nmeros
primos.
3. Para qualquer outro polgono regular com nmero
P
par de lados (octgono, decgono, ... etc), se
considerarmos dois vrtices P e Q diametralmente
opostos e um vrtice M no consecutivo de P nem
M
de Q, eles determinaro o tringulo PQM, retngulo O
em M, cujos lados so trs das diagonais desse
polgono e, como j foi provado no P6, as medidas
Q
desses trs lados no podem ser simultaneamente
expressas por nmeros primos.
Concluso, apenas o quadrado pode ter todas as suas diagonais com
medidas expressas por nmeros primos.

P15.
No ano passado meu filho caula certamente tinha 1 ano de idade. Meu
outro filho tinha 4 ou 16 anos e eu, o pai, 36 anos.
Portanto, hoje, minha idade 37 anos.

RPM OBMEP

Quando a minha idade mpar, a do meu caula par e vice-versa;


portanto, nunca mais nossas idades voltaro a ser todas simultaneamente
representadas por nmeros primos.

188

Solues dos Problemas II


Estantes
5
Bombons coloridos
B) 20%
Carpinteiro
modelo A sim
modelo B no
modelo C sim
modelo D sim
Dados
Questo 1
17 = 21 4
Questo 2
No, Sim, Sim, No (nesta ordem).
Sequncia em escada
10
Terremoto
C) A probabilidade de haver um terremoto em Zedpolis nos prximos
20 anos maior do que a probabilidade de no haver um terremoto.

Bate-papo pela Internet


Questo 1
10 da manh ou 10h.
Questo 2
Qualquer horrio ou intervalo de tempo que satisfaa a diferena de 9
horas e que esteja compreendido entre um dos seguintes intervalos:
Sydney: 16:30 h 18:00 h; Berlim: 7:30 h 9:00 h.
ou
Sydney: 7:00 h 8:00 h; Berlim: 22:00 h 23:00 h.

RPM OBMEP

Feira de Vero
B) Pouco provvel.

189

Assaltos
No, um crescimento de aproximadamente 10 no muito grande
comparado a um total de aproximadamente 500.
Colnia de frias: uma soluo
Nome
Vermelho
Azul
Verde
Roxo
Laranja
Amarelo
Branco
Total

meninos
10

meninas
7
7

6
7
4
20

5
26

RPM OBMEP

H = homem
M = mulher

190

adultos
HH
M
M
H
M
H
M

Solues dos Problemas III


1.
Vamos observar inicialmente que (a + b + c)3 = a3 + b3 + c3.
Segue-se, ento, que: a2(b + c) + b2(a + c) + c2(a + b) + 2abc = 0 ou
equivalentemente, (a + b)(a + c)(b + c) = 0.
Para que isso ocorra, pelo menos um dos fatores deve ser nulo. Supondo
a + b = 0, a primeira equao nos d c = 3. Substituindo na segunda,
obtemos a = 2 e b = 2 . Segue-se que a4 + b4 + c4 = 89.
2.
Suponhamos que seja a aranha A1 que percorre todo o ABC e
consideremos o instante em que ela est em A. Seja M o ponto mdio
do lado A2A3 do A1A2A3 formado pelas trs aranhas nesse instante.

Traamos r e s retas paralelas a BC e que dividem a altura AH em


trs partes iguais. Prolongamos AM at encontrar BC em M. Como

RPM OBMEP

r e s dividem AM em trs partes iguais (Tales), ento o baricentro


do A1A2A3 que o ponto que dista AM/3 de M, est na regio
pintada.

191

Repetindo o argumento quando a aranha A1 est em B e quando est


em C, conclumos que o baricentro P comum dos tringulos formados
pelas trs aranhas est em trs regies que se cortam exatamente no
baricentro do ABC, o que demonstra o resultado pedido. Para justificar
esta ltima afirmao, observamos que a reta r e as outras duas
construdas analogamente, quando a aranha A1 est em B ou C, encontram-se no baricentro do ABC.
3.
a) A probabilidade de que o rapaz acerte na primeira tentativa igual a
1/10, uma vez que ele escolhe ao acaso um dos dez dgitos possveis.
b) Para que ocorra a segunda tentativa necessrio que ele tenha errado
na primeira, e a probabilidade de isso acontecer igual a 9/10. Dado
que errou na primeira tentativa, a probabilidade (condicional) de que
ele acerte na segunda igual a 1/9, uma vez que, agora, o nmero de
dgitos possveis igual a 9. Logo, a probabilidade de que ele acerte
na segunda tentativa (9/10)(1/9) = 1/10.
Segue que a probabilidade de que ele consiga conversar com a namorada
igual a (1/10) + (1/10) = 1/5.
4.
Se 10 divide x2 + y2 + xy, ento 2 divide x2 + y2 + xy; logo,
x2 + y2 + xy par, implicando x e y pares, o que, por sua vez, implica
x2 + y2 + xy mltiplo de 4. Se 10 divide x2 + y2 + xy, ento 5 divide
x2 + y2 + xy.
Se mostrarmos que isso implica x e y mltiplos de 5, teremos que 25
dividir x2 + y2 + xy, que mltiplo de 4, logo 100 dividir
x2 + y2 + xy.
Prova de que x e y so mltiplos de 5

RPM OBMEP

Escrevendo x = 5a + b, y = 5c + d, com a, b, c e d inteiros no


negativos e 0 < b < 4, 0 < c < 4, obtemos

192

x2 + y2 + xy = (5a + b)2 + (5c + d)2 + (5a + b)(5c + d) =


5(5a2 + 2ab + 5c2 + 2cd + 5ac + ad + bc) + b2 + d2 + db.
Como 5 divide x2 + y2 + xy, temos que 5 divide b2 + d2 + bd.

Se b = d 0 temos b2 + d2 + bd = 3b2, que no mltiplo de 5, j que


b = 1, 2, 3 ou 4.
Se b d, fazendo todas as possveis substituies para b e d em
b2 + d2 + bd, obtemos:
12 + 22 + 1 2 = 7, 12 + 32 + 1 3 = 13, 12 + 42 + 1 4 = 21,
22 + 32 + 2 3 = 19, 22 + 42 + 2 4 = 28, 32 + 42 + 3 4 = 37.
Como nenhum dos resultados divisvel por 5, conclumos que
b = d = 0 e, portanto, x e y so mltiplos de 5.
5.
Consideremos o quadrado MNPQ, com lados paralelos s retas paralelas
dadas e contendo os vrtices ABCD, como na figura. Se o ngulo
indicado, ento DQ = asen e MD = acos.
Logo, a medida dos lados do quadrado MNPQ igual a asen + acos.
Indicando por z e z as hipotenusas dos tringulos sombreados e por h
e h as alturas desses tringulos, temos:
Q

x h y
z

B
z

h + h + a = asen + acos ou h + h = a(sen + cos 1). Por outro


h + h'
. Sendo
lado, zsencos = h e zsencos = h ou z + z' =
sen cos

x=

h
h'
h + h'
e x' =
, o que implica x + x' =
;
sen
sen
sen

RPM OBMEP

x, y, x e y os catetos indicados na figura, temos

193

y=

h
cos

e y' =

h'
h + h'
, o que implica y + y' =
.
cos
cos

A soma, s, dos permetros dos tringulos sombreados ser:

s = x + x + y + y + z + z
s=

(h + h' )(sen + cos + 1)


h + h' h + h'
h + h'
+
+
=
sen cos sen cos
sen cos

a (sen + cos 1)(sen + cos + 1) a[(sen + cos ) 2 1]


=
sen cos
sen cos
a 2sen cos
s=
s = 2a.
sen cos
s=

6.
Dois naturais m e n resolvem n2n1 + 1 = m2 se e s se m = 2k + 1 e
n2n3 = k(k + 1) (*) para algum natural k.
fcil verificar diretamente que, para 0 < n < 6, as nicas solues do
problema so (n, m) = (0, 1) e (n, m) = (5, 9).
Mostremos que, para n > 7, no existe soluo.
De fato, observe em (*) que 2n3 divide k ou k + 1.
Se 2n3 divide k + 1, ento k divide n e
n k +1
=
n > k e k + 1 > 2n3 n + 1 > 2n3.
k 2 n 3
Se 2n3 divide k, ento k + 1 divide n e
n
k
= n 3 n > k +1 e k > 2n3 n > 2n3 + 1.
k +1 2
Porm, fcil ver, por induo, que n + 1 < 2n3 para n > 7.

RPM OBMEP

7.

194

Seja x = JC, de modo que AC = 3x. No tringulo retngulo ACC,


temos que a medida do ngulo CAC igual a 60o e, portanto, a
medida do ngulo ACC igual a 30o. Como JH = JC, temos o ngulo
JHC igual a 30o; logo, o ngulo HJB mede 60o.

x
Portanto, B'J = HJ cos 60D = ,
2
implicando CB' = x +

x 3 x AC
=
=
, isto
2 2
2

, B ponto mdio de AC.


Logo, BB' simultaneamente altura e
mediana do ABC, que , portanto,
l=l
issceles com AB = BC. Portanto, C
A = 60D , o que mostra que ABC
de fato equiltero.
8.
Como x2 + 1 tem grau 2, temos que o resto da diviso de p(x) por
x2 + 1 tem a forma ax + b com a, b . Temos, portanto,
p(x) = (x2 + 1)q(x) + ax + b.
Fazendo x = i e x = i, obtemos
p (i ) p (i )

a =
p (i ) = ai + b
2i

.
p
(

i
)
=

ai
+
b
+
p
(
i
)
p (i )

b =

2
Pela frmula de Moivre, temos

p (i ) = (cos
p (i ) = (cos

2
2 2007
+ i sen
)
= cos 2 + i sen 2 = 1 e
2007
2007

2
2 2007
2
2 2007
i sen
)
= [cos(
) + i sen(
)]
= 1.
2007
2007
2007
2007

9.
Na figura a seguir, seja M o ponto de interseco de XY e CA' .
Como MC perpendicular a AB , temos que MC' perpendicular
a A'B' . Vamos mostrar primeiramente que A, M e C so colineares,

RPM OBMEP

Substituindo nas expresses de a e b, temos a = 0 e b = 1, isto , o


resto da diviso 1.

195

e que, portanto, AC' perpendicular a A'B' . Para isso, basta mostrar

m ) = m( A' MC
m ' ) . Como X l
que m( AMC
AC
' ' reto, temos
m( A'm
MC' ) = 90D m( M l
AC
' ' ) = m( M l
A' X ) .
Agora, como MA = MA e XA = XA, temos que AMA e AXA so
issceles e assim m( M l
A' X ) = m( M l
AX ) .

m ) , pois A'C paralelo a AZ , o que mostra


Mas m( M l
AX ) = m( AMC
m ) = m( A' MC
m ') .
que m( AMC
Assim, os tringulos ACB e ACA so
congruentes pelo caso de congruncia LAL, j
que BC = BC = AC = AC e o ngulo C reto.
Logo, = , pois so ngulos correspondentes.
Os tringulos ACA e AZA so congruentes
pelo caso especial cateto-hipotenusa, pois AC
= AZ. Logo, = e, portanto, = = , ou
seja, o ngulo foi dividido em 3 partes iguais.
10.
Sendo f: N N tal que f(f(x)) = x, x N, temos:
a) f injetora, pois, se f(x1) = f(x2), ento f(f(x1)) = f(f(x2)), isto ,
x1 = x2. Temos tambm f sobrejetora, pois, dado y N, seja
x = f(y) N e ento f(x) = f(f(y)) = y.

RPM OBMEP

n + 1, se n par
b) A funo f: N N dada por f (n) =
n 1, se n mpar
obedece condio f(f(n)) = n, nN, no caso em que
N = {0, 1, 2, 3, ..., n, ...}. E no caso que N = {1, 2, 3, ..., n, ...}, basta
tomar f(n) = n 1, se n par e f(n) = n + 1, se n mpar.

196

Observao
Existem diversas outras solues. Por exemplo, sejam
A = { a1, a2, a3, ...} e B = {b1, b2, b3, ...} conjuntos de uma partio qual-

quer de N (isto , A B = N e A B = ). Basta ento definir


f(ai) = bi e f(bi) = ai, i natural. A soluo apresentada corresponde
partio de N nos conjuntos A e B dos nmeros pares e mpares,
respectivamente.
11.
Sejam Mi, i = 1, 2, ..., n, cada uma das moedas,
1
, i = 1, 2, ..., n , a probabilidade de se obter cara jogando a
2i + 1
moeda Mi e PTi a probabilidade de haver um nmero mpar de caras
jogando-se as moedas M1, M2, ..., Mi.
Pi =

Queremos achar PT2007.

1
Para i = 1, temos que PT1 = P1 = , pois a probabilidade de haver 1
3
cara no lanamento de M1.
Para i = 2, temos que PT2 a probabilidade de M1 ser cara e M2 ser
coroa ou M1 ser coroa e M2 ser cara: PT2 = P1(1 P2) + (1 P1)P2.
1
1
1 1 4 2
6 2
PT2 = .(1 ) + (1 ). = + = = .
3
5
3 5 15 15 15 5
n
1
2
e PT2 = , parece que PTn =
. Vamos tentar
2
n
+1
3
5
provar essa igualdade pelo princpio da induo finita.

Sendo PT1 =

Tese: PTn =
PTn1 =

Hiptese: PTn1 =

(n 1)
.
2(n 1) + 1

(n 1)
n 1
n 1
. Porm, PT n a
=
=
2(n 1) + 1 2n 2 + 1 2n 1

probabilidade de haver um nmero mpar de caras em M1, M2, ..., Mn1


e Mn ser coroa ou haver um nmero par de caras em M1, M2, ..., Mn1
e Mn ser cara. Portanto, PTn = PTn1(1 Pn) + (1 PTn1)Pn ou
PTn = (

n
n 1
1
n 1
1
.
)(1
) + [1 (
)](
)=
2n + 1
2n 1
2n + 1
2n 1 2n + 1

RPM OBMEP

Temos

n
.
2n + 1

197

Portanto, PT2007 =

2007
2007
=
.
2(2007) + 1 4015

12.
Traamos a perpendicular reta CD, por A, que corta o segmento CD
l agudo (mede 60o).
no ponto E, pois ADC
O BDE issceles, pois
DE = 2acos 60o = a = BD. Logo,
l
o BEC issceles, pois E BC
mede 15o e, portanto, BE = EC.
Alm disso, BE = AE, pois BEA
l e Bl
issceles, j que ABE
AE
medem 30o. Logo, AE = EC e o
AEC issceles com E l
AC meo
dindo 45 . Assim, = 75o.
Observaes
1. No site www.gogeometry.com possvel encontrar outros problemas desse
tipo.
2. Este problema tambm pode ser resolvido utilizando-se a lei dos senos ou
dos cossenos. Que tal tentar?

13.
Para que a equao

x + 2 x 1 + x 2 x 1 = 2 tenha soluo

real necessrio que 2x 1 > 0, isto , x

1
. Nesse caso, o primeiro
2

radicando sempre positivo e para ver que x 2 x 1 basta observar


que: ( x 1) 2 0 x 2 2 x 1 x 2 2 x 1 x = x 2 x 1.

RPM OBMEP

Para x > 1/2,

198

tem-se: x soluo da equao se e somente se

( x + 2 x 1 + x 2 x 1 ) 2 = 2 ( x 1) 2 = 1 x x 1 = 1 x

1 x = 1 x 1 x 0 x 1.
Logo, o conjunto soluo da equao o intervalo S = [1/2, 1].

14.
a) No ABC, retngulo em B, M o ponto
mdio de AC e MH perpendicular a BC.
Como BH = HC, o BMH e o CMH
so congruentes e, portanto, BM = MC =
AM. Assim, todo tringulo retngulo pode
ser decomposto em dois tringulos issceles
ABM e BMC.

A
M
B

b) Dado um PQR, qualquer, traamos uma altura interna que o


decompe em dois tringulos retngulos. Usando a), vemos que o
PQR pode ser decomposto em n = 4 tringulos issceles. Repetindo
o procedimento em um, dois, trs ou quatro desses tringulos,
decompe-se o PQR em sete, dez, treze ou dezesseis tringulos
issceles (que passaremos a chamar de partes). E assim por diante,
PQR pode ser decomposto em n = 4 + 3k partes, k .
Para decompor em n = 5 partes, consideremos dois casos:
i) PQR no equiltero.
Decompomos o tringulo em dois
tringulos, um deles issceles, e
aplicamos a) no outro (dividido
em dois tringulos retngulos),
obtendo 5 partes.

Repetindo o procedimento de a) em uma, duas, trs, quatro ou


cinco partes, decompe-se o PQR em oito, onze, catorze,
dezessete ou vinte partes. E assim, em n = 5 + 3k partes, k .
P

M
Q

C
R

RPM OBMEP

ii) PQR equiltero.


Uma altura separa o PQR em dois tringulos retngulos. Num deles aplicamos a),
obtendo duas partes, e decompomos o
outro num tringulo issceles e outro
retngulo com vrtice comum no
circuncentro C do PQR. Aplicando a)
nesse tringulo retngulo, decompe-se o
PQR em n = 5 partes. Como em i),

199

mostra-se que possvel decompor o PQR em n = 5 + 3k


partes, k .
Um PQR no equiltero pode ser
decomposto em trs tringulos,
sendo dois issceles, e aplicando a)
no terceiro tringulo (dividido em
dois tringulos retngulos), decompomos o PQR em n = 6 partes.

R
P

Se PQR equiltero, utilizando o


circuncentro C, decompomos em trs partes,
e aplicando a) numa delas obtemos a
decomposio em 6 partes. Como nos casos
anteriores, o PQR pode ser decomposto
em n = 6 + 3k partes, k .

C
Q

15.
A funo f deve satisfazer a igualdade 2f(x) + f(1 x) = x2008 para todo
x R. Escrevendo a igualdade para (1 x), obtemos
2f(1 x) + f( 1 (1 x)) = (1 x)2008. Ento,
2 f (1 x) + f ( x) = (1 x) 2008

2008
2 f ( x) + f (1 x) = x
ou, isolando f(1 x) na segunda equao e substituindo na primeira,
2[ x 2008 2 f ( x)] + f ( x) = (1 x) 2008 , o que leva a

2 x 2008 (1 x) 2008
.
3
fcil verificar que a ltima funo satisfaz a equao funcional dada,
logo a nica soluo.
RPM OBMEP

f ( x) =

200

16.
Podemos fatorar 32008 1 da forma seguinte:
32008 1 = (38)251 1 = (38 1)(38250 + 38249 + 38248 + ... + 381 + 1).
Observe que a soma 38250 + 38249 + 38248 + ... + 381 + 1 tem 251

parcelas, todas mpares; logo, um nmero mpar, no sendo, portanto,


divisvel por 2. Por outro lado,
38 1 = (34 1)(34 + 1) = 80 82 = 25 5 41.
Ento, a maior potncia de 2 que divide 32008 1 25.
17.
a) Como cada pea na posio (i, j) substituda por duas, ocupando
as posies (i + 1, j) e (i, j + 1), temos que a soma dos pesos no
varia a cada jogada, pois 2ij = 2(i +1)j + 2i(j +1).
b) Para determinar a soma de todos os pesos do tabuleiro, observamos
que a soma da primeira coluna

1
= 2 (PG infinita de razo
11 2

1/2 e termo inicial 1). Por outro lado, a soma da segunda coluna
metade da primeira, a da terceira, 1/4 da primeira, e assim por diante,
logo a soma de todos os pesos do tabuleiro

2 +1+

1 1 1
2
+ + + ... =
= 4.
2 4 8
11 2

c) A soma dos pesos das seis casas iniciais sombreadas

1+

1 1 1 1 1
3
+ + + + = 2 + , enquanto a soma dos pesos das casas
2 2 4 4 4
4

3
1
no hachuradas 4 (2 + ) = 1 + .
4
4
Assim, impossvel mover as peas das casas hachuradas inicialmente para fora dessa regio, uma vez que a soma dos pesos das
3
ser mantida, em cada jogada, e maior do que
4

a soma dos pesos de todas as casas no hachuradas, 1 +

1
.
4

Observao: o problema ainda no teria soluo caso as peas


iniciais ocupassem apenas as casas de pesos 1, 1/2, 1/2, j que para
atingir o peso 2 da regio no hachurada necessrio utilizar infinitas
casas. Logo, o jogo no terminaria aps um nmero finito de
movimentos.

RPM OBMEP

casas iniciais 2 +

201

18.
Temos p =

n3 1

2 n 2008 n

+1

(n 1)(n 2 + n + 1)

2 n 2008 ( n + 1)( n

n + 1)

n 1
n2 + n + 1
.
.

2
2 n 2008 n + 1 2 n 2008 n n + 1

Por um lado, temos


(1)

n 1 1 2 3 4
2006 2007
1.2
.
= ...

=
n
+
1
3
4
5
6
2008
2009
2008
2
009
.2
2 n 2008

Por outro lado, se g(n) = n2 n + 1, temos que


g(n + 1) = (n + 1)2 (n + 1) + 1 = n2 + n + 1 e assim
(2)

n2 + n + 1

g (n + 1) g (3) g (4)
g (2009)
=

...
g
(
n
)
g
(
2
)
g
(
3
)
g (2008)
2 n 2008 n n + 1 2 n 2008

g (2009) 20092 2009 + 1 .


=
g (2)
22 2 + 1

De (1) e (2), conclumos que

p=

1.2
20092 2009 + 1 2 2009.2008 + 1

=
2008.2009
3 2008.2009
22 2 + 1

2
1
(1 +
).
3
2008.2009

RPM OBMEP

19.
Observe inicialmente que, sendo 7 um nmero primo que divide N,
ento necessariamente ele deve dividir algum dos fatores inteiros
(x + 6y), (2x + 5y) ou (3x + 4y). Mas ento observe as relaes abaixo:

202

3(2 x + 5 y ) + ( x + 6 y ) = 7( x + 3 y )

2(3 x + 4 y ) + ( x + 6 y ) = 7( x + 2 y ) .
2(2 x + 5 y ) + (3 x + 4 y ) = 7( x + 2 y )

Se algum dos fatores for mltiplo de 7, elas implicam que os outros


tambm so, portanto N divisvel por 73 = 343.

20.

RPM OBMEP

l tal que AO = BO,


Suponhamos que exista um ngulo AOB
l , COD
l e BOD
l congruentes. No
AC = CD = BD com os ngulos AOC
AOD, a ceviana OC mediana e bissetriz; logo, tambm a altura e
l reto. No COB, a ceviana OD mediana e bissetriz;
o ngulo ACO
l reto.
logo, tambm a altura e o ngulo ODC
Assim, o DOC tem dois ngulos retos, o que um absurdo.

203

RPM
Sociedade Brasileira de Matemtica
Presidente: Joo Lucas Marques Barbosa
Vice-Presidente: Hilrio Alencar
Secretrio-Geral: Marco Antnio Teixeira
Tesoureiro: Walcy Santos
Comit Editorial da RPM
Alcila Augusto editora responsvel
Ana Catarina P. Hellmeister editora executiva
Alberto Carvalho P. de Azevedo
Antonio Luiz Pereira
Eduardo Wagner
Elon Lages Lima
Geraldo vila
Jos Paulo Q. Carneiro
Paulo Cezar Pinto Carvalho
Renate G. Watanabe
Severino Toscano do Rego Melo

Seleo e Organizao: Comit Editorial da RPM


Diagramao e Ilustrao: Mrcio Alexandre de Castro
Silvana Cunha de Vasconcelos

RPM Revista do Professor de Matemtica


Rua do Mato, 1010 bloco B sala 105
CEP 05508-090 So Paulo, SP
www.rpm.org.br
rpm@ime.usp.br
telefone/fax: (11) 3091-6124

Das könnte Ihnen auch gefallen